Sie sind auf Seite 1von 1017

Surgery: EvidenceBased Practice

PMPH_FM.indd i

5/22/2012 6:23:42 PM

PMPH_FM.indd ii

5/22/2012 6:23:43 PM

Surgery: EvidenceBased Practice

Stephen M. Cohn, MD, FACS


Witten B. Russ Professor of Surgery
Department of Surgery
University of Texas Health Science Center
San Antonio, Texas

Steven T. Brower, MD, FACS


Vice Chairman, Department of Surgery
Chief, Division of Surgical Oncology
Beth Israel Medical Center, NY
Director of Strategic Planning and
Extramural Affairs
Continuum Cancer Centers of New York
New York, New York

ERRNVPHGLFRVRUJ
2012

PEOPLES MEDICAL PUBLISHING HOUSEUSA


SHELTON, CONNECTICUT

PMPH_FM.indd iii

5/22/2012 6:23:43 PM

Peoples Medical Publishing House-USA


2 Enterprise Drive, Suite 509
Shelton, CT 06484
Tel: 203-402-0646
Fax: 203-402-0854
E-mail: info@pmph-usa.com
2012 PMPH-USA, Ltd.
All rights reserved. Without limiting the rights under copyright reserved above, no part of this publication may be reproduced, stored in or introduced into a retrieval system, or transmitted,
in any form or by any means (electronic, mechanical, photocopying, recording, or otherwise), without the prior written permission of the publisher.
12 13 14 15 16/PMPH/9 8 7 6 5 4 3 2 1
ISBN-13: 978-1-60795-109-4
ISBN-10: 1-60795-109-6
eISBN-13: 978-1-60795-242-8
Printed in China by Peoples Medical Publishing House
Editor: Linda Mehta; Copyeditor/Typesetter: Newgen; Cover designer: Marguerite Bunyan
Library of Congress Cataloging-in-Publication Data
CIP data on file

Sales and Distribution


Canada
McGraw-Hill Ryerson Education
Customer Care
300 Water St
Whitby, Ontario L1N 9B6
Canada
Tel: 1-800-565-5758
Fax: 1-800-463-5885
www.mcgrawhill.ca

United Kingdom, Europe, Middle East, Africa


McGraw Hill Education
Shoppenhangers Road
Maidenhead
Berkshire, SL6 2QL
ngland
Tel: 44-0-1628-502500
Fax: 44-0-1628-635895
www.mcgraw-hill.co.uk

Foreign Rights
John Scott & Company
International Publishers Agency
P.O. Box 878
Kimberton, PA 19442
USA
Tel: 610827-1640
Fax: 610827-1671

Singapore, Thailand, Philippines, Indonesia,


Vietnam, Pacific Rim, Korea
McGraw-Hill Education
60 Tuas Basin Link
Singapore 638775
Tel: 65-6863-1580
Fax: 65-6862-3354
www.mcgraw-hill.com.sg

Japan
United Publishers Services Limited
1-32-5 Higashi-Shinagawa
Shinagawa-ku, Tokyo 140-0002
Japan
Tel: 03-5479-7251
Fax: 03-5479-7307
Email: kakimoto@ups.co.jp

Australia, New Zealand


Elsevier Australia
Locked Bag 7500
Chatswood DC NSW 2067
Australia
Tel: 161 (2) 9422-8500
Fax: 161 (2) 9422-8562
www.elsevier.com.au

Brazil
SuperPedidoTecmedd
Beatriz Alves, Foreign Trade Department
R. SansaoAlves dos Santos, 102 | 7th floor
Brooklin Novo
Sao Paolo 04571-090
Brazil
Tel: 55-16-3512-5539
www.superpedidotecmedd.com.br
India, Bangladesh, Pakistan, Sri Lanka, Malaysia
CBS Publishers
4819/X1 Prahlad Street 24
Ansari Road, Darya Ganj, New Delhi-110002
India
Tel: 91-11-23266861/67
Fax: 91-11-23266818
Email:cbspubs@vsnl.com
Peoples Republic of China
Peoples Medical Publishing House
International Trade Department
No. 19, Pan Jia Yuan Nan Li
Chaoyang District
Beijing 100021
P.R. China
Tel: 8610-67653342
Fax: 8610-67691034
www.pmph.com/en/

Notice: The authors and publisher have made every effort to ensure that the patient care recommended herein, including choice of drugs and drug dosages, is in accord with the accepted
standard and practice at the time of publication. However, since research and regulation constantly change clinical standards, the reader is urged to check the product information sheet
included in the package of each drug, which includes recommended doses, warnings, and contraindications. Th is is particularly important with new or infrequently used drugs. Any treatment regimen, particularly one involving medication, involves inherent risk that must be weighed on a case-by-case basis against the benefits anticipated. The reader is cautioned that the
purpose of this book is to inform and enlighten; the information contained herein is not intended as, and should not be employed as, a substitute for individual diagnosis and treatment.

PMPH_FM.indd iv

5/22/2012 6:23:43 PM

To my parents, Lee and Iris Cohn, and my family, Sam and Elizabeth Cohn, for their support through the years.
Stephen M. Cohn
To my familyJoshua, Evan, and Marianne Browerfor their unwavering support .
Steven T. Brower

PMPH_FM.indd v

5/22/2012 6:23:43 PM

PMPH_FM.indd vi

5/22/2012 6:23:43 PM

CONTENTS
List of Contributors
Foreword
Basil A. Pruitt, Jr.
Preface
Stephen M. Cohn & Steven T. Brower
1. Understanding the Elements that Comprise
Evidence-Based Medicine
Joel E. Michalek, John E. Cornell & Brad H. Pollock
2. Patient Safety in Surgical Care
Kenneth Stahl & Susan E. Brien

xiii
xxix

Commentary
Frederick A. Moore

xxxi

1
14

PART I. THE ESOPHAGUS

118

14. Zollinger-Ellison Syndrome


Geoff rey W. Krampitz & Jeff rey A. Norton

119

15. Evidence-Based Bariatric Surgery


Amir H. Sharif & Robert L. Bell

132

16. Gastric Adenocarcinoma


Antonio I. Picon & Martin S. Karpeh
Commentary
Scott A. Hundahl

146

17. Management of Upper GI Bleeding


Bruce A. Crookes
Commentary
Gregory J. Jurkovich

154

152

160

3. Esophageal Perforation
Jonathan B. Lundy & G. Travis Clifton
Commentaries
Scott B. Johnson, Jeremy S. Juern & John A Weigelt

25

4. Achalasia and Esophageal Spasms


Kalyana C. Nandipati & Edward Lin

37

5. Esophageal Diverticula
G. Travis Clifton & Jonathan B. Lundy

45

6. Gastroesophageal Reflux Disease


Alejandro F. Sanz & Blair A. Jobe

59

7. Inguinal Hernias
George Kasotakis & Marc A. de Moya

64

8. Esophageal Caustic Injury


Yoram Klein

70

9. Esophageal Tumors
Daniel S. Oh & Steven R. DeMeester

74

20. Small Bowel Tumors and Diverticular


Disease of SB
John D. Cunningham & Akintunde Akinleye

83

21. Enterocutaneous Fistula


Peter A. Learn

192

90

22. Short Bowel Syndrome


Andrea MacNeill & S. Morad Hameed
Commentary
James Davis

196

10. The Use of Esophageal Stents


Yaron Perry & Robert Jones
Commentary
Scott B. Johnson
11. Neoadjuvant and Adjuvant Treatment of
Esophageal Cancer
J. Camilo Barreto & Mitchell C. Posner
12. Esophageal Atresia and
Tracheoesophageal Fistula
Miller C. Hamrick, David E. Carney &
William C. Boswell
Commentary
Michael Hirsch

34

PART III. SMALL BOWEL

91

165

19. Crohns Disease of the Small Bowel


Thomas Donkar, Andrea Bafford &
Randolf M. Steinhagen
Commentary
Tomas M. Heimann

174

173

184

186

203

PART IV. LARGE BOWEL


97

104

PART II. THE STOMACH


13. Peptic Ulcer Disease
Wayne H. Schwesinger

18. Small Bowel Surgery


James H. Lee, John J. Hong, Dale Dangleben &
Michael M. Badellino
Commentary
Ronald M. Stewart

109

23. Diverticular Disease of the Colon


Brent Izu & A. Peter Ekeh
Commentary
Richard J. Mullins

207

24. Crohns Colitis and Ulcerative Colitis


Kervin Arroyo & Barry Salky

215

25. Large Bowel Obstruction


Ryan A. Lawless, Dale A. Dangleben &
Michael M. Badellino

221

214

vii

PMPH_FM.indd vii

5/22/2012 6:23:43 PM

viii

Contents

Commentary
Martin A. Schreiber

227

26. Radiation Injury to the Small and Large Bowel 228


Ali Y. Mejaddam & David R. King
Commentary
231
Daniel L. Dent
27. Ischemic Colitis
Thomas D. Conlee, Daniel J. Bonville &
Jonathan J. Canete
Commentary
Fred A. Luchette

232

28. Pseudomembranous Colitis


Burke Thompson
Commentary
Pamela A. Lipsett

239

237

243

40. Hepatic Infections: Pyogenic Abscess,


Amebic Abscess, and Hydatid Cyst
David M. Levi & Andreas G. Tzakis
41. Malignant Liver Tumors
Susanne Carpenter & Yuman Fong

334

PART VI. PORTAL HYPERTENSION


42. Management of Portal Hypertension:
A Surgical Perspective
Sukru Emre & Manuel I. Rodriguez-Davalos

345

43. Management of Intractable Ascites:


The Evidence
Mark I. E. Cockburn & Adora Fou-Cockburn

353

44. Hepatic Encephalopathy


Terence OKeefe & Tun Jie
Commentary
Jamal J. Hoballah

29. Colon and Rectal Cancer Including Adjuvant


Robert P. Sticca, Erik G. Fetner & Jay M. MacGregor
Commentary
Nicholas J. Petrelli

245

30. Tumors of the Anal Region


Marjun P. Duldulao & Julio Garcia-Aguilar

253

31. Inherited Colorectal Cancer Syndromes


Vitaliy Y. Poylin, Kristin B. Niendorf &
Robert D. Madoff

260

45. Elective Non-hepatic Surgery in


Cirrhotic Patients
Jared C. Brandenberger & Vafa Ghaemmaghami
Commentary
Bruce Gelb & H. Leon Pachter

32. Preoperative Bowel Preparation


John K. Bini
Commentary
Donald E. Fry

267

PART VII. THE GALLBLADDER


AND BILE DUCTS

251

329

357
365

366
371

33. Appendicitis
Damon Kalcich & Peter P. Lopez

275

46. Silent Gallstones


Abdul Saied & James C. Doherty
Commentary
David H. Livingston

287

47. Acute Cholecystitis


John S. Oh

380

34. Hemorrhoids
Clarence E. Clark III
Commentary
Stanley M. Goldberg

48. Common Bile Duct Stones


Adrian W. Ong & Charles F. Cobb

386

49. Benign Biliary Strictures


Demetrius Pertsemlidis & David S. Pertsemlidis

394

50. Gallstone Ileus


David W. Smith & Ara J. Feinstein

399

51. Bile Duct and Gallbladder Tumors


T. Peter Kingham & Michael DAngelica

403

35. Anorectal Fissure, Stricture,


Abscess, and Fistula
W. Brian Perry & Joshua A. Tyler
Commentary
Stanley M. Goldberg
36. Fecal Incontinence and Surgical Management
of Constipation
Sarah Pesek & Neil Hyman

273

293

294
298

299

37. Rectovaginal Fistula


Joshua D. Schulte, Kelly Ming, Michelle M. Olsen &
Philip F. Caushaj
Commentary
Patricia L. Roberts

305

52. Obstructive Jaundice: Transhepatic


and Endoscopic Interventions
Brian J. Dunkin

312

PART VIII. THE PANCREAS

38. Lower Gastrointestinal Bleeding


Kerry G. Bennett & Steven Schwaitzberg

314

53. Acute Pancreatitis


Stephen W. Behrman
Commentary
Wayne H. Schwesinger

PART V. THE LIVER


39. Newer Techniques in Liver Surgery
Anil S. Paramesh, Robert Cannon & Joseph F. Buell

PMPH_FM.indd viii

323

375
379

410

421
429

54. Infected Pancreatic Collections


Nader N. Massarweh & Karen D. Horvath

430

55. Pancreatic Pseudocysts


Olga N. Tucker & Raul J. Rosenthal

437

5/22/2012 6:23:43 PM

Contents

56. Chronic Pancreatitis


Katherine A. Morgan & David B. Adams

449

57. Pancreatic Adenocarcinoma


Jamii St. Julien, Alexander A. Parikh &
Nipun G. Merchant
Commentary
Matthew H. G. Katz, Jason B. Fleming,
Jeff rey E. Lee & Peter W. T. Pisters

458

58. Unusual Pancreatic Tumors


David W. Rittenhouse, Charles J. Yeo, &
Samuel D. Gross

467

465

PART IX. THE SPLEEN

Commentary
Richelle Williams & David P. Winchester

ix

585

73. Breast Cancer: Surgical Therapy


Alyssa Gillego, Manjeet Chadha, Beth Freedman &
Susan K. Boolbol

587

74. Breast Cancer: Lymphatic Mapping


and Sentinel Lymph Node Biopsy
Abigail S. Caudle, Elizabeth A. Mittendorf &
Henry M. Kuerer

597

75. Systemic Treatment Strategies


for Early-Stage Breast Cancers
Jennifer K. Litton & Kelly K. Hunt

604

76. In Situ Carcinoma of the Breast:


Ductal and Lobular Carcinoma
Jane E. Mndez

612

59. Hematologic Indications for Splenectomy


Mark T. Muir

481

60. Tumors, Cysts, and Abscesses of the Spleen


Robert Benjamin

488

77. Male Breast Cancer


Jessica Keto & Paul Ian Tartter

617

61. Splenic Salvage


Dror Soffer & Daniel Abraham
Commentary
Sherry Sixta, John B. Holcolm & Jack H. Mayfield

490

78. Breast Reconstruction Following Mastectomy


David M. Adelman & Steven J. Kronowitz

622

62. Postsplenectomy Sepsis


Regan J. Berg & Kenji Inaba

498

496

PART X. HERNIA
63. Inguinal Hernias
George Kasotakis & Marc A. de Moya

513

64. Recurrent Inguinal Hernia


H. R. Nanda Kumar & Kent R. Van Sickle
Commentary
Thomas E. Knuth

518
523

65. Epigastric and Umbilical Hernia


Rachel Beard & Steven D. Schwaitzberg

525

66. Incisional and Ventral Hernias


Rachel Beard & Steven D. Schwaitzberg

530

PART XIII. CHEST WALL, MEDIASTINUM,


TRACHEA
79. Lung Cancer Staging
Joe B. Putnam, Jr.

631

80. Primary Chest Wall Tumors


Adam H. Lackey, Joseph B. Levin & Harvey I. Pass

636

81. Tracheostomy: Timing and Techniques


Matthew O. Dolich
Commentary
Matthew E. Lissauer & Thomas M. Scalea

642

82. Pneumothorax and Hemothorax


Joseph J. DuBose
Commentary
J. D. Richardson

648
650
657

PART XIV. VASCULAR SYSTEM


PART XI. ENDOCRINE GLANDS
67. Adrenocortical Tumors and Incidentalomas
Paul Karanicolas & Murray Brennan

537

68. Pheochromocytoma
Raymon H. Grogan & Quan-Yang Duh

544

69. Thyroid Nodules


Gerard M. Doherty

552

70. Hyperthyroidism, Thyroiditis,


and Nontoxic Goiter
Prashant Khullar & Geeta Lal
71. Hyperparathyroidism
Jason D. Prescott & Robert Udelsman

559
568

PART XII. THE BREAST


72. Screening, Breast Biopsy, Benign Disease
Adora Fou-Cockburn & Mark I. Cockburn

PMPH_FM.indd ix

577

83. Abdominal Aortic Aneurysm


Boulos Toursarkissian
Commentary
Gregorio Sicard
84. Aortic Dissection
Benjamin J. Pearce
Commentary
Karthikeshwar Kasirajan
85. Arterial Pseudoaneurysms and
Arteriovenous Fistulae
J. Leigh Eidson III & Marvin D. Atkins

661
667
669
676

677

86. Carotid Occlusive and Aneurysmal Disease


Matthew J. Sideman & Lori L. Pounds

684

87. Aortoiliac Occlusive Disease


Jeff rey S. Horn & William D. Jordan, Jr.
Commentary
Alan B. Lumsden

691
697

5/22/2012 6:23:43 PM

Contents

88. Femoropopliteal and Tibioperoneal


Occlusive and Aneurysmal Disease
Luke X. Zhan & Joseph L. Mills
Commentary
Frank J. Veith

102. Soft Tissue Sarcoma


Keila E. Torres & Raphael E. Pollock

807

706

103. Solitary Neck Mass


Luc G. T. Morris & Ashok R. Shaha

814

89. Thoracic Outlet Syndromes


Robert W. Thompson
Commentary
Julie Ann Freischlag

707

104. Peripheral Nerve Injury Repair


Marisa H. Amaral, Pranay Parikh &
Joseph H. Shin
Commentary
Howard T. Wang

818

90. Peripheral Arterial Embolus


Shane OKeeffe & Eric Endean
Commentary
Jamal J. Hoballah

722

699

721

728

91. Vascular Syndromes


John E. Campbell & Ali F. Aburahma

730

92. Management of Graft Occlusion


David Vogel & Anthony J. Comerota
Commentary
Daniel B. Walsh

735

93. Vascular Access for Hemodialysis


Yazan Duwayri & Brian G. Rubin

742

94. Atherosclerotic Renovascular Disease


Eric Hager & Luke Marone
Commentary
Daniel G. Clair

749

105. Necrotizing Soft Tissue Infections


Including Gas Gangrene
Kevin M. Schuster & Erik S. Barquist
106. Necrotizing Soft Tissue Infections
Mark D. Sawyer
Commentary
A. Patchen Dellinger

754

107. Electrolytes and Fluids


Daun J. Milligan, Megan B. Steigelman,
Tristan T. Lai & Aaron M. Fields
Commentary
David B. Hoyt
108. Surgical Nutrition
Jayson D. Aydelotte

760

96. The Diabetic Foot


Alexandra A. MacLean & Maria Codreanu
Commentary
Jodi Walters & David G. Armstrong

762

109. Preoperative Risk Factor Assessment


of the Surgical Patient
Edgar Joseph Pierre, Shawn Michael Cantie &
Faisal Huda
Commentary
Elliott Bennett-Guerrero

97. Varicose Veins and Venous Insufficiency


M. K. Sheehan
Commentary
Seshadri Raju

769

100. Management of Lymphedema


Magdiel Trinidad-Hernandez & Peter Gloviczki

767

772
775

783
785
791
792

PART XV. SKIN AND SOFT TISSUE


101. Malignant Melanoma
Robert E. Roses & Daniel F. Roses

PMPH_FM.indd x

833
838

PART XVI. PREOPERATIVE AND


POSTOPERATIVE CARE

756

99. Pulmonary Embolism


George C. Velmahos
Commentary
Kenneth L. Mattox

826

741

95. Acute and Chronic Mesenteric Ischemia


Luke P. Brewster & Elliot L. Chaikof
Commentary
Michael J. Sise

98. Deep Venous Thrombosis


Mark L. Ryan, Chad M. Thorson, Thai Vu,
Christian A. Otero & Enrique Ginzburg
Commentary
M. M. Knudson

824

799

110. Perioperative Cardiac Monitoring


Steven G. Venticinque
111. Bacteremia: An Evidence-Based Review of
Recommendations for Elective
General Surgery
Katherine Hetz, Kevin K. Chung &
Christopher E. White
Commentary
John C. Marshall
112. Prevention of Central Venous
Catheter Infections
Antonio Aponte-Feliciano, J. Matthias Walz &
Stephen O. Heard
Commentary
Nicole J. Krumrei & Donald H. Jenkins
113. Surgical Site Infection (SSI) and
Prophylactic Antibiotics
Robert Krell & Lena M. Napolitano
Commentary
Timothy C. Fabian

841

853
854

859

867
869

876

882

884

890

891
899

5/22/2012 6:23:43 PM

Contents

114. Management of Acute Myocardial Infarction


and Cardiogenic Shock
Antonio Hernandez
Commentary
Kenneth Waxman
115. Abnormal Surgical and
Postoperative Bleeding
Andrew L. Tang, Nicholas Tarmey,
Joseph Dubose & Peter Rhee
116. Management of Sedation
and Delirium in the ICU
Robert Chen & Sangeeta Mehta
117. Postoperative Respiratory Failure
Mollie M. James & Gregory J. Beilman

PMPH_FM.indd xi

901

Commentary
Suresh Agarwal

xi

934

907

118. Postoperative Hepatic Failure


Joseph Love & Raphael Diaz-Flores

936
942

909

119. Acute Renal Dysfunction


Matthew ORourke & Jonathan Barasch
Commentary
Rao R. Ivatury
120. Perioperative Endocrine Dysfunction
Catherine A. Madorin & Kaare J. Weber

949

Index

955

917
926

947

5/22/2012 6:23:43 PM

PMPH_FM.indd xii

5/22/2012 6:23:43 PM

LIST OF CONTRIBUTORS
Daniel Abraham, MD [61]
Director, Yitzhak Rabin Trauma Division
Assistant Professor of Surgery
Division of Surgery
Tel Aviv Sourasky Medical Center
Tel Aviv, Israel

University of Arizona College of Medicine


Tucson, Arizona
Kervin Arroyo, MD [24]
Fellow Minimally Invasive Surgery
Division of Laparoscopic Surgery
The Mount Sinai Hospital
New York, New York

Ali F. Aburahma, MD [91]


Professor of Surgery
West Virginia University
Director, Vascular Surgery Fellowship Program
Co-Director, Vascular Center of Excellence
Chief of Vascular and Endovascular Surgery
Charleston Area Medical Center
Charleston, West Virginia

Marvin D. Atkins, MD [85]


Texas A&M University Health Science Center
Scott and White Hospital & Clinic
Division of Vascular Surgery
Temple, Texas

David B. Adams, MD, FACS [56]


Professor of Surgery
Head, Section of General and Gastrointestinal Surgery
Medical University South Carolina
Charleston, South Carolina

Jayson D. Aydelotte, MD [108]


Associate Professor/Clinical
Program Director, Critical Care Fellowship
Division of Trauma
University of Texas Health Science Center
San Antonio, Texas

David M. Adelman, MD, PhD [78]


Assistant Professor
Department of Plastic Surgery, Division of Surgery
The University of Texas MD Anderson Cancer Center
Houston, Texas

Michael M. Badellino, MD, FACS [18, 25]


Division of Trauma/Surgical Critical Care
Program Director, General Surgery Residency
Lehigh Valley Health Network
Allentown, Pennsylvania

Suresh Agarwal, MD [117C]


Chief, Surgical Critical Care
Boston Medical Center
Associate Professor of Surgery
Boston University School of Medicine
Boston, Masachussets

Andrea Chao Bafford, MD [19]


Assistant Professor of Surgery
Digestive Health Center
University of Maryland School of Medicine
Baltimore, Maryland

Akinleye Akintunde, MD
Internal Medicine Resident
Overlook Hospital
Summit, New Jersey

Jonathan Barasch, MD, PhD [119]


Associate Professor of Medicine and Cell Biology
Department of Medicine
Columbia University Medical Center
New York, New York

[20]

Erik S. Barquist, MD, FACS, FCCM [105]


Chief of Surgery
Jackson South Community Hospital
Miami, Florida

Marisa H. Amaral, MD [104]


Resident, General Surgery
Tufts University School of Medicine
Baystate Medical Center
Springfield, Massachusetts

J. Camilo Barreto, MD [11]


Section of General Surgery and Surgical Oncology
Department of Surgery
University of Chicago Pritzker School of Medicine
Chicago, Illinois

Antonio Aponte-Feliciano, MD [112]


Assistant Professor of Anesthesiology
University of Massachusetts Medical School
Department of Anesthesiology and Critical Care
UMASS Memorial Medical Center
Worcester, Massachusetts

Rachel Beard, MD [65, 66]


Department of Surgery
Beth Israel Deaconess Medical Center Department of Surgery
Cambridge Health Alliance

David G. Armstrong, DPM, MD, PhD [96C]


Southern Arizona Limb Salvage Alliance (SALSA)
xiii

PMPH_FM.indd xiii

5/22/2012 6:23:43 PM

xiv

List of Contributors

Harvard Medical School


Boston, Massachusetts
Stephen W. Behrman, MD, FACS [53]
Associate Professor of Surgery
University of Tennessee, Memphis Health Science Center
Germantown, Tennessee
Greg Beilman, MD [117]
Professor and Vice Chair of Perioperative Services and Quality
Improvement
Chief of General Surgery
Division Director, Critical Care and Acute Care Surgery
University of Minnesota
Minneapolis, Minnesota
Robert L. Bell, MD [15]
Associate Professor of Surgery
Section of Gastrointestinal Surgery
Department of Surgery
Yale University School of Medicine
New Haven, Connecticut
Kerry G. Bennett, MD, MPH
Department of Surgery
St Lukes Hospital
New Bedford, Massachusetts

[38]

Elliott Bennett-Guerrero, MD [109C]


Professor and Director of Perioperative Clinical Research,
Duke Clinical Research Institute
Duke University
Durham, North Carolina
Regan J. Berg, MD [62]
Department of Surgery
University of Southern California
Los Angeles, California
John K. Bini, MD [32]
Clinical Instructor
Division of Trauma
University of Texas Health Science Center
San Antonio, Texas
Daniel J. Bonville, DO, FACS
Department of Surgery
Albany Medical College
Albany, New York

[27]

Susan K. Boolbol, MD [73]


Department of Surgery
Chief, Appel-Venet Comprehensive Breast Service
Beth Israel Medical Center
Assistant Professor of Surgery
Albert Einstein College of Medicine
New York, New York
William C. Boswell, MD [12]
Pediatric Surgery
Memorial University Medical Center
Savannah, Georgia

PMPH_FM.indd xiv

Jared C. Brandenberger, MD [45]


Kidney Care and Transplantation Services
University of Washington Medical Center
Seattle, Washington
Murray F. Brennan, MD [67]
Benno C. Schmidt Chair of Clinical Oncology
Department of Surgery
Memorial Sloan-Kettering Cancer Center
New York, New York
Luke P. Brewster, MD, PhD [95]
Assistant Professor of Surgery
Division of Vascular Surgery, Department of Surgery
Emory University School of Medicine
Atlanta, Georgia
Stephen T. Brower, MD [EDITOR]
Vice Chairman, Department of Surgery
Chief, Division of Surgical Oncology
Beth Israel Medical Center
Director of Strategic Planning and Extramural Affairs
Continuum Cancer Centers of New York
New York, New York
Susan Brien, MD Med, CSPQ, FRCSC, CPE
Registrar & Associate Director Professional Affairs
Royal College of Physicians & Surgeons of Canada
Chief of Trauma and Staff Neurosurgeon
Gatineau, Quebec
Adjunct Professor, Department of Surgery
University of Ottawa
Ottawa, Ontario Canada

[2]

Joseph F. Buell, MD [39]


Professor of Surgery
Director of the Tulane Transplant Abdominal Institute
Tulane University
New Orleans, Louisiana
John E. Campbell, MD [91]
Assistant Professor of Surgery and Medicine
West Virginia University
Division of Vascular and Endovascular Surgery
Charleston Area Medical Center
Charleston, West Virginia
Jonathan Canete, MD [27]
Assistant Professor of Surgery
Albany Medical Center
Albany, New York
Shawn Michael Cantie, MD [109]
Resident in Anesthesiology
University of Miami School of Medicine
Miami, Florida
David E. Carney, MD [12]
Pediatric Surgery
Memorial University Medical Center
Savannah, Georgia

5/22/2012 6:23:43 PM

List of Contributors

Susanne Carpenter [41]


Department of Surgery
Memorial Sloan Kettering Cancer Center
New York, New York

G. Travis Clifton, MD [3, 5]


Trauma/Burn/Critical Care Surgeon
United States Army Institute of Surgical Research
Fort Sam Houston, Texas

Abigail S. Caudle [74]


Assistant Professor
Department of Surgical Oncology, Division of Surgery
The University of Texas MD Anderson Cancer
Center
Houston, Texas

Charles F. Cobb, MD [48]


Associate Professor of Surgery
Drexel University College of Medicine
Department of Surgery
Allegheny General Hospital
Pittsburgh, Pennsylvania

Philip F. Caushaj, MD, PhD, FACS, FASCRS


Chief of Colon and Rectal Surgery
Director of Minimally Invasive Surgery
Kern Medical Center
Bakersfield, California

[37]

Manjeet Chadha, MD [73]


Associate Professor of Radiation Oncology
Albert Einstein College of Medicine
Associate Chairman, Radiation Oncology
Beth Israel Medical Center
Director of Breast and Gynecologic Cancer Programs in
Radiation Oncology
Continuum Cancer Centers of New York
New York, New York
Elliot L. Chaikof, M.D., PhD [95]
Chairman of the Roberta and Stephen R. Weiner
Department of Surgery
Surgeon-in-Chief
Beth Israel Deaconess Medical Center
Boston, Massachusetts
Robert Chen, MD [116]
Attending Anesthetist and Intensivist
Assistant Professor
St. Michaels Hospital
Department of Anaesthesia
University of Toronto
Toronto, Ontario, Canada
Kevin K. Chung, MD [111]
US Army Institute of Surgical Research
Brooke Army Medical Center
San Antonio, Texas
Daniel Clair, MD
Department Chair
Vascular Surgery
Cleveland Clinic
Cleveland, Ohio

[94C]

Clarence E. Clark III, MD [34]


Assistant Professor of Surgery
Texas A&M Health Science Center
Division of Surgical Oncology, Section of Colon and
Rectal Surgery
Scott and White Memorial Hospital
Temple, Texas

PMPH_FM.indd xv

xv

Mark Ian Cockburn, MD [43, 72]


General and Critical Care Surgery
Stamford Hospital
Stamford, Connecticut
Adora Ann Fou-Cockburn, MD, FACS [43, 72]
WestMed Medical Group
Rye, New York
Maria Codreanu, MD [96]
General Surgery Resident
New York Hospital-Queens
Flushing, New York
Stephen M. Cohn, MD, FACS [EDITOR]
Witten B. Russ Professor of Surgery
Division of Trauma and General Surgery
UT Health Science Center, San Antonio
San Antonio, Texas
Anthony J. Comerota, MD, FACS [92]
Director, Jobst Vascular Institute
Toledo, Ohio
Adjunct Professor of Surgery
University of Michigan
Section of Vascular Surgery
Ann Arbor, Michigan
Thomas D. Conlee, MD [27]
Resident in Surgery
Albany Medical Center
Albany, New York
John E. Cornell, PhD [1]
Professor, Department of Epidemiology and Biostatistics
University of Texas Health Science Center
San Antonio, Texas
Bruce Crookes, MD [17]
Associate Professor of Surgery
Division of Trauma & Acute Care Surgery
Medical University of South Carolina
Charleston, South Carolina
John David Cunningham, MD, FACS [20]
Assistant Professor of Surgery
Columbia University
New York, New York
Attending Surgeon

5/22/2012 6:23:43 PM

xvi

List of Contributors

Overlook Hospital
Summit, New Jersey

University of Illinois College of Medicine


Chicago, Illinois

Michael DAngelica, MD [51]


Associate Attending
Division of Hepatopancreatobiliary Surgery
Department of Surgery
Memorial Sloan-Kettering Cancer Center
New York, New York

Gerard M. Doherty, MD [69]


Chair, Department of Surgery
Boston University School of Medicine
Chief of Surgery
Boston Medical Center
Boston, Massachusetts

Dale A. Dangleben, MD [18, 25]


Associate Program Director, General Surgery Residency
Program
Department of Surgery
Division of Trauma-Surgical Critical Care/General Surgery
Lehigh Valley Health Network
Allentown, Pennsylvania

Matthew O. Dolich, MD, FACS [81]


Professor and Residency Program Director
Department of Surgery
University of California, Irvine
Orange, California

James W. Davis, MD [22C]


Professor
Chief of Surgery and Program Director
University of California
San Francisco, California
E. Patchen Dellinger, MD [106C]
Professor and Vice Chairman
Department of Surgery
Chief, Division of General Surgery
University of Washington
Seattle, Washington
Steven R. DeMeester, MD [9]
Associate Professor of Surgery
Department of Surgery
University of Southern California, Keck School of Medicine
Los Angeles, California
Marc A de Moya, MD [7, 63]
Division of Trauma, Emergency Surgery & Surgical Critical Care
Massachusetts General Hospital
Boston, Massachusetts
Daniel L. Dent, MD [26C]
Distinguished Teaching Professor
General Surgery Residency Program Director
Professor of Surgery
Division of Trauma
University of Texas Health Science Center
San Antonio, Texas

Thomas Donkar, DO, MPH


Resident in Surgery
Maimonides Medical Center
Brooklyn, New York

[19]

Major Joseph J. DuBose, MD, FACS


University of Maryland Medical System
R Adams Cowley Shock Trauma Center
Air Force/C-STARS
Baltimore, Maryland

[82, 115]

Quan-Yang Duh, MD [68]


Professor, Department of Surgery
University of California, San Francisco
San Francisco, California
Marjun P. Duldulao, MD [30]
Department of Oncologic Surgery
City of Hope Comprehensive Cancer Center
Duarte, California
Brian J. Dunkin, MD, FACS [52]
Professor of Clinical Surgery
Weill Cornell Medical College
Head, Section Endoscopic Surgery
The Methodist Hospital
Houston, Texas
Yazan Duwayri, MD [93]
Assistant Professor of Surgery
Division of Vascular Surgery and Endovascular Therapy
Department of Surgery
Emory University School of Medicine
Atlanta, Georgia

Rafael F. Diaz Flores, MD, MPH [118]


General Surgery Resident
Department of Surgery
University of Texas Health Science Center
San Antonio, Texas

J. Leigh Eidson, MD [85]


Texas A&M University Health Science Center
Scott and White Hospital & Clinic
Division of Vascular Surgery
Temple, Texas

James C. Doherty MD, MPH [46]


Director of Trauma Surgery and Critical Care Programs
Advocate Christ Medical Center
Oak Lawn, Illinois
Clinical Assistant Professor of Surgery

Akpofure Peter Ekeh, MD [23]


Associate Professor, Surgery
Boonshoft School of Medicine
Wright State University
Director, Injury Prevention Center

PMPH_FM.indd xvi

5/22/2012 6:23:43 PM

List of Contributors

Chief, Division of Trauma, Critical Care, and Emergency


General Surgery
Miami Valley Hospital
Dayton, Ohio
Sukru Emre, MD, FACS [42]
Professor of Surgery and Pediatrics
Yale University School of Medicine
Chief, Section of Transplantation and Immunology
New Haven, Connecticut
Eric D. Endean, MD [90]
Professor, Department of Surgery
University of Kentucky College of Medicine
Lexington, Kentucky

Beth Israel Medical Center


New York, New York
Julio Garcia-Aguilar, MD, PhD [30]
Professor, General Oncologic Surgery
Department of Surgery
City of Hope
Duarte, California
Bruce E. Gelb, MD [45C]
Assistant Professor
Department of Surgery (Transplant)
NYU School of Medicine
New York, New York

Timothy C. Fabian, MD, FACS [113C]


Harwell Wilson Professor and Chairman
Department of Surgery
University of Tennessee Health Science Center
Memphis, Tennessee

Vafa Ghaemmaghami, MD, FACS [45]


Interim Program Director
Phoenix Integrated Residency Program
Interim Director, Trauma/Surgical Intensive Care Unit
Banner Good Samaritan Medical Center
Phoenix, Arizona

Ara J. Feinstein, MD [50]


Assistant Professor
University of Arizona, Phoenix
Trauma, Critical Care and Acute Care Surgery Division
Banner Good Samaritan Medical Center
Phoenix, Arizona

Alyssa Gillego, MD [73]


Attending Surgeon
Department of Surgery
Beth Israel Medical Center
New York, New York

Eric G. Fetner, MD [29]


Clinical Assistant Professor of Surgery
University of North Dakota School of Medicine
and Health Sciences
Grand Forks, North Dakota

Enrique Ginzburg, MD, FACS [98]


Professor of Clinical Surgery
DeWitt Daughtry Family Department of Surgery
Ryder Trauma Center
Miami, Florida

Major Aaron M. Fields, MD [107]


Staff, Anesthesiology and Critical Care Medicine
US Air Force, 13th Air Force
Hickam AFB
Honolulu, Hawaii

Peter Gloviczki, MD
Professor of Surgery
Chair, Vascular Surgery
Mayo Clinic
Rochester, Minnesota

Jason B. Fleming, MD [57C]


Associate Professor
Department of Surgical Oncology
The University of Texas MD Anderson Cancer Center
Houston, Texas

Stanley M. Goldberg, MD [34C, 35C]


Colon and Rectal Surgery AssociatesMinneapolis
Minneapolis, Minnesota

Yuman Fong, MD [41]


Murray F. Brennan Chair in Surgery
Memorial Sloan-Kettering Cancer Center
Professor of Surgery
Weill Cornell Medical College
New York, New York
Julie Ann Freischlag, MD [89C]
The William Stewart Halsted Professor
Chair, Department of Surgery
Surgeon-in-Chief, Johns Hopkins Hospital
Baltimore, Maryland
Beth Friedman, MD
Department of Surgery

PMPH_FM.indd xvii

[73]

xvii

[100]

Raymon H. Grogan, MD [68]


Section of Endocrine Surgery
University of California, San Francisco
San Francisco, California
Eric Hager, MD [94]
University of Pittsburgh Medical Center
Division of Vascular and Endovascular Surgery
Pittsburgh, Pennsylvania
S. Morad Hameed, MD, MPH [22]
Assistant Professor of Surgery and Critical Care Medicine
Director, General Surgery Residency Program
The University of British Columbia
Trauma Services
Vancouver, British Columbia Canada

5/22/2012 6:23:43 PM

xviii

List of Contributors

Miller C. Hamrick, MD [12]


Department of Surgery
Memorial University Medical Center
Savannah, Georgia
Stephen O. Heard, MD [112]
Chair and Professor
Department of Anesthesiology
University of Massachusetts Medical School
Worcester, Massachusetts
Tomas M. Heimann, MD, FACS [19C]
Professor of Surgery
Mount Sinai School of Medicine
New York, New York
Chief, General Surgical Services
James J. Peters VA Medical Center
Bronx, New York
Antonio Hernandez, MD [114]
Associate Professor
Department of Anesthesiology
University of Texas Health Science Center
San Antonio, Texas
Katherine Hetz, MD [111]
Department of General Surgery
Brooke Army Medical Center
San Antonio, Texas
Michael P. Hirsh, MD, FACS, FAAP [12C]
Surgeon-in-Chief
Chief, Division of Pediatric Surgery and Trauma
University of Massachusetts Memorial Childrens Medical Center
Professor of Surgery and Pediatrics
University of Massachusetts Medical School
Worcester, Massachusetts
Jamal J. Hoballah, MD, MBA, FACS [32C, 44C, 90C]
Professor & Chairman
Department of Surgery
American University of Beirut Medical Center
Professor of Surgery
Vascular Surgery Division
The University of Iowa Hospitals and Clinics
Iowa City, Iowa
John B. Holcomb, MD, FACS [61C]
Vice Chair and Professor of Surgery
Chief, Division of Acute Care Surgery
University of Texas Health Science Center
Houston, Texas
John J. Hong MD [18]
Vice Chair for Research
Department of Surgery
Lehigh Valley Health Network
Allentown, Pennsylvania
Jeffrey S. Horn, MD [87]
Section of Vascular Surgery and Endovascular Therapy

PMPH_FM.indd xviii

University of Alabama Birmingham


Birmingham, AL
Karen D. Horvath, MD, FACS [54]
Professor of Surgery
Residency Program Director
University of Washington
Seattle, Washington
David B. Hoyt, MD, FACS [107C]
Executive Director
American College of Surgeons
Professor Emeritus, University of California, Irvine
Chicago, Illinois
Faisal Huda, MD [109]
Resident in Anesthesiology
University of Miami School of Medicine
Miami, Florida
Scott A. Hundahl, MD [16C]
Professor of Clinical Surgery
University of California, Davis
Chief of Surgery for VA Northern California
UC Davis Cancer Center
Sacramento, California
Chief of Surgery
VA Northern CA Health Care System
Mather, California
Kelly K. Hunt, MD, FACS [75]
Professor of Surgical Oncology
The University of Texas MD Anderson Cancer Center
Houston, Texas
Neil Hyman, MD [36]
Samuel B. and Michelle D. Labow Professor of Surgery
Co-director, Digestive Disease Center
University of Vermont College of Medicine
Burlington, Vermont
Kenji Inaba, MD, FACS, FRCSC [62]
Assistant Professor of Surgery
Medical Director, Surgical ICU
Program Director, Critical Care Fellowship
University of Southern California
Los Angeles, California
Rao R. Ivatury, MD, FACS, FCCM [121C]
Professor of Surgery
Virginia Commonwealth University
Chair, Division of Trauma, Critical Care & Emergency Surgery
VCU Medical Center
Richmond, Virginia
Brent Izu, MD [23]
Resident, Department of Surgery
Wright State University
Boonshoft School of Medicine
Dayton, Ohio

5/22/2012 6:23:43 PM

List of Contributors

xix

Donald H. Jenkins, MD, FACS [112C]


Division of Trauma, Critical Care and Emergency General Surgery
Mayo Clinic
Rochester, Minnesota

Director of Clinical Research


Division of Vascular Surgery
Emory University School of Medicine
Atlanta, Georgia

Tun Jie, MD, MS [44]


Assistant Professor of Surgery
General Surgery/Abdominal Transplantation
Department of Surgery
University of Arizona College of Medicine
Tucson, Arizona

George Kasotakis, MD [7, 63]


Department of Surgery
Division of Trauma, Emergency Surgery & Surgical Critical Care
Harvard Medical School
Massachusetts General Hospital
Boston, Massachusetts

Blair A. Jobe, MD, FACS [6]


University of Pittsburgh Physicians
Department of Cardiothoracic Surgery
Division of Thoracic Surgery
University of Pittsburgh Medical Center
Pittsburgh, Pennsylvania

Matthew H.G. Katz, MD [57C]


Assistant Professor
Department of Surgical Oncology
Division of Surgery
The University of Texas MD Anderson Cancer Center
Houston, Texas

Scott B. Johnson, MD [3C, 10C]


Associate Professor
Head of the Section of General Thoracic Surgery
University of Texas Health Science Center
San Antonio, Texas

Jessica Keto, MD [77]


Breast Surgeon
Associate Medical Director
Saint Marys Comprehensive Breast Center
Grand Rapids, Michigan

William D. Jordan, Jr., MD [87]


Holt A. McDowell Professor of Surgery
Chief, Section of Vascular Surgery and Endovascular Therapy
University of Alabama Birmingham
The Kirklin Clinic
Birmingham, Alabama

Prashant Khullar, MD [70]


Resident
Department of Surgery
University of Iowa Health Center
Iowa City, Iowa

Jeremy Juern, MD [3C]


Division of Trauma and Critical Care
Department of Surgery
Medical College of Wisconsin
Milwaukee, Wisconsin

David R. King, MD, FACS, MAJ, MC, USAR [26]


Assistant Professor
Division of Trauma, Emergency Surgery, and Surgical Critical Care
Harvard Medical School
Boston, Massachusetts

Gregory J Jurkovich, MD
Professor of Surgery
University of Washington
Chief of Trauma
Harborview Medical Center
Seattle, Washington

T. Peter Kingham, MD [51]


Assistant Attending
Division of Hepatopancreatobiliary Surgery
Department of Surgery
Memorial Sloan-Kettering Cancer Center
New York, New York

[17C]

Damon Kalcich, DO [33]


General Surgery
Sinai Grace Hospital
Detroit, Michigan

Yoram Klein, MD [8]


Chief, Department of Trauma and Acute Care Surgery
Kaplan Medical Center
Rehovot, Israel

Paul J. Karanicolas, MD, PhD [67]


Fellow, Department of Surgery
Memorial Sloan-Kettering Cancer Center
New York, New York

M. Margaret Knudson, MD, FACS [98C]


Professor and Interim Chief of Surgery
San Francisco General Hospital and Trauma Center
UCSF Division of General Surgery
University of California, San Francisco
San Francisco, California

Martin S. Karpeh, MD [16]


Chairman, Department of Surgery
Beth Israel Medical Center
New York, New York
Karthikeshwar Kasirajan, MD, FACS [84C]
Associate Professor

PMPH_FM.indd xix

Thomas E. Knuth, MD, MPH, FACS


Senior Surgeon
Division of Acute Care Surgery
Henry Ford Hospital
Detroit, Michigan

[64C]

5/22/2012 6:23:43 PM

xx

List of Contributors

Geoffrey W. Krampitz, MD [14]


Department of Surgery
Stanford University School of Medicine
Stanford, California
Robert Krell, MD [113]
Department of Surgery
Division of Acute Care Surgery
University of Michigan Health Care System
Ann Arbor, Michigan
Steven J. Kronowitz, MD [78]
Professor, Department of Plastic Surgery
University of Texas MD Anderson Cancer Center
Houston, Texas
Nicole J. Krumrei, MD [112C]
Division of Trauma, Critical Care and Emergency
General Surgery
Mayo Clinic
Rochester, Minnesota
Henry M. Kuerer, MD, PhD, FACS [74]
Professor with Tenure
Department of Surgical Oncology
University of Texas MD Anderson Cancer Center
Houston, Texas
Hanuma Reddy Nanda Kumar, MD [64]
General Surgery Resident
Postdoctoral Fellow Trauma
University of Texas Health Science Center
San Antonio, Texas
Adam H. Lackey, MD [80]
Fellow, Department of Cardiothoracic Surgery
New York University School of Medicine
New York, New York
Tristan T. Lai, MD [107]
Department of Anesthesiology
US Air Force
MAJ Wilford Hall Medical Center
Lackland Air Force Base, Texas
Geeta Lal, MD, MSc, FACS [70]
Assistant Professor
Division of Surgical Oncology and Endocrine Surgery
Department of Surgery
University of Iowa
Iowa City, Iowa

James H. Lee, MD [18]


Chief Surgical Resident
Department of Surgery
Lehigh Valley Health Network
Allentown, Pennsylvania
Jeffrey E. Lee, MD [57C]
Department Chair and Professor
Department of Surgical Oncology
Division of Surgery
Co-Director, Melanoma and Skin Cancer Research Program
The University of Texas MD Anderson Cancer Center
Houston, Texas
David M. Levi, MD, FACS [40]
Professor of Clinical Surgery
Miami Transplant Institute
University of Miami Miller School of Medicine
Miami, Florida
Joseph B. Levin, PA [80]
Research Assistant
Department of Cardiothoracic Surgery
New York University School of Medicine
New York, New York
Edward Lin, DO, FACS [4]
Associate Professor of Surgery
Director, Emory Endosurgery Unit and Gastroesophageal
Treatment Center
Surgical Director, Emory Bariatrics
Department of Surgery
Atlanta, Georgia
Pamela A. Lipsett, MD, FACS [28C]
Professor, Surgery, ACCM and Nursing
Surgical Critical Care Fellowship Director
Johns Hopkins Health System
Baltimore, Maryland
Matthew E. Lissauer, MD [81C]
Assistant Professor of Surgery
Shock Trauma Center
University of Maryland Medical Center
Baltimore, Maryland
Jennifer K. Litton, MD [75]
Assistant Professor
Department of Breast Medical Oncology
Division of Cancer Medicine
The University of Texas MD Anderson Cancer Center
Houston, Texas

Ryan A. Lawless, MD [25]


Department of Surgery
Lehigh Valley Health Network
Allentown, Pennsylvania

David H. Livingston, MD [46C]


Chief, Division of Trauma
Department of Surgery
UMDNJ-New Jersey Medical School
Newark, New Jersey

Peter A. Learn, MD [21]


Department of Surgery
San Antonio Military Medical Center
Ft. Sam Houston, Texas

Peter P. Lopez, MD FACS [33]


Clinical Assistant Professor of Surgery
Wayne State University School of Medicine
Detroit Medical Center

PMPH_FM.indd xx

5/22/2012 6:23:43 PM

List of Contributors

Sinai-Grace Hospital
Detroit, Michigan
Joseph Love, DO, FACS [118]
Assistant Professor of Surgery
Division of Acute Care Surgery
The University of Texas Medical School at Houston
Houston, Texas
Fred A. Luchette, MD [27C]
Professor of Surgery
Division of General Surgery
Loyola University Chicago
Stritch School of Medicine
Chicago, Illinois

Luke Marone, MD, FACS [94]


University of Pittsburgh Medical Center
Division of Vascular and Endovascular Surgery
Pittsburgh, Pennsylvania
John C. Marshall MD, FRCSC, FACS [111C]
Chair, Canadian Critical Care Trials Group
Critical Care, Trauma and General Surgery
St Michaels Hospital
Toronto, Ontario Canada
Nader N. Massarweh, MD, MPH [54]
Surgical Resident
Department of Surgery
University of Washington
Seattle, Washington

Alan B Lumsden, MD [87C]


Chair, Department of Cardiovascular Surgery
Medical Director, Methodist DeBakey Heart and Vascular Center
The Methodist Hospital
Professor of Cardiothoracic Surgery
Houston, Texas

Kenneth L. Mattox, MD [99C]


Professor and Vice Chairman of Surgery
General Surgery Division of the Michael E. DeBakey
Department of Surgery
Baylor College of Medicine
Houston, Texas

Jonathan B. Lundy, MD [3, 5]


Trauma/Burn/Critical Care Surgeon
United States Army Institute of Surgical Research
Fort Sam Houston, Texas

Sangeeta Mehta, MD, FRCP(C) [116]


Assistant Professor of Medicine
Division of Critical Care Medicine
University of Toronto
Toronto, Ontario Canada

Jay M. MacGregor, MD [29]


Chief Resident in Surgery
University of North Dakota School of Medicine and Health Sciences
Grand Forks, North Dakota
Alexandra A. MacLean, MD
Medical Director
Medical/Clinical Affairs
Energy-Based Devices
Covidien
Boulder, Colorado

Jane E. Mndez, MD
Professor of Surgery
Boston Medical Center
Boston, Massachusetts

Robert Madoff, MD [31]


Professor, Department of Surgery
University of Minnesota Medical School
Stanley M. Goldberg, MD Endowed Chair in Colon and
Rectal Surgery
Minneapolis, Minnesota

PMPH_FM.indd xxi

Ali Y. Mejaddam, MD [26]


Research Fellow in Surgery
Massachusetts General Hospital
Boston, Massachusetts

[96]

Andrea MacNeill, MD [22]


Resident, Division of General Surgery
University of British Columbia
Vancouver, Canada
MSc Candidate
University of Oxford
Oxford, United Kingdom

Catherine A. Madorin, MD
House Staff
Department of Surgery
Mount Sinai School of Medicine
The Mount Sinai Hospital
New York, New York

xxi

[76]

Nipun Merchant, MD [57]


Associate Professor of Surgery
Vanderbilt University Medical Center
Nashville, Tennessee
Joel E. Michalek, PhD [1]
Professor and Vice Chairman
Department of Epidemiology & Biostatistics
University of Texas Health Science Center
San Antonio, Texas
Daun J. Milligan, MD [107]
Critical Care Fellow
Department of Surgery
University of Texas Health Science Center
San Antonio, Texas

[120]
Joseph L. Mills, MD [88]
Chief, Division of Vascular and Endovascular Surgery
Director, Vascular Fellowship and Residency Programs
Professor of Surgery
Tucson, Arizona

5/22/2012 6:23:44 PM

xxii

List of Contributors

Kelly Ming, MD [37]


Senior Surgical Resident
Kalamazoo Center for Medical Studies
Kalamazoo, Michigan
Elizabeth A. Mittendorf, MD [74]
Assistant Professor
Department of Surgical Oncology
Division of Surgery
The University of Texas MD Anderson Cancer Center
Houston, Texas
Frederick A. Moore, MD, FACS, FCCM [13C]
Professor and Chief, Acute Care Surgery
Department of Surgery
University of Florida College of Medicine
Gainesville, Florida
Katherine A. Morgan, MD [56]
Associate Professor of Surgery
Medical University of South Carolina
Charleston, South Carolina
Luc G.T. Morris, MD [103]
Head and Neck Service
Department of Surgery
Memorial Sloan-Kettering Cancer Center
New York, New York
Mark T. Muir, MD [59]
General Surgery Resident
Department of Surgery
University of Texas Health Science Center
San Antonio, Texas
Richard J. Mullins, MD, FACS [23C]
Professor of Surgery
Section of Trauma/Critical Care
Oregon Health & Science University
Portland, Oregon
Kalyana Nandipati, MD [4]
Assistant Professor of Surgery
Department of Surgery
Creighton University School of Medicine
Omaha, Nebraska
Lena M. Napolitano MD [113]
Professor, Department of Surgery
Division Chief, Acute Care Surgery (Trauma, Burn, Critical Care,
Emergency Surgery)
Associate Chair of Surgery for Critical Care
Director, Surgical Critical Care
University of Michigan
Ann Arbor, Michigan
Kristin B. Niendorf, MS [31]
Genetic Counselor, Coordinator of the Bernstein Registry
Masonic Cancer Center

PMPH_FM.indd xxii

University of Minnesota
Minneapolis, Minnesota
Jeffrey A. Norton, MD [14]
The Robert L. and Mary Ellenburg Professor in Surgery
Surgical Oncology and General Surgery
Stanford University School of Medicine
Stanford, California
Daniel S. Oh, MD [9]
Assistant Professor of Surgery
USC Norris Cancer Hospital
Keck School of Medicine
Los Angeles, California
John S. Oh, MD [47]
Chief of Trauma
Department of Surgery
Landstuhl Regional Medical Center
US Army
Landstuhl, Germany
Shane D. OKeeffe, MD [90]
Assistant Professor
Department of Surgery
University of Kentucky College of Medicine
Lexington, Kentucky
Terence OKeeffe, MD, ChB, MSPH [44]
Assistant Professor
Trauma Division
Department of Surgery
University of Arizona College of Medicine
Tucson, Arizona
Michelle M. Olson, MD, FACS, FASCRS
Assistant Professor of Surgery
Temple University School of Medicine
Philadelpia, Pennsylvania

[37]

Matthew ORourke, MD [119]


Resident, Department of Pediatrics
Columbia University Medical Center
New York, New York
Adrian Ong, MD [48]
Assistant Professor of Surgery
Drexel University School of Medicine
Allegheny General Hospital
Pittsburgh, Pennsylvania
Christian A. Otero, MD [98]
Division of Trauma and Surgical Critical Care Services
University of Miami Miller School of Medicine
Jackson Memorial Hospital / Ryder Trauma Center
Miami, Florida
H. Leon Pachter, MD [45C]
George David Stewart Professor of Surgery
Chair & G.D. Stewart Professor Surgery
Department of Surgery
NYU Langone Medical Center
New York, New York

5/22/2012 6:23:44 PM

List of Contributors

Alexander A. Parikh, MD [57]


Assistant Professor of Surgery
Division of Surgical Oncology & Endocrine Surgery
Vanderbilt University Medical Center
Nashville, Tennessee

Nicholas J. Petrelli, M.D. FACS [29C]


Professor of Surgery
Bank of America Endowed Medical Director, Helen F.
Graham Cancer Center at Christiana Care
Thomas Jefferson University
Newark, Delaware

Pranay Parikh, MD [104]


Assistant Professor of Surgery
Division of Plastic Surgery
Tufts University School of Medicine
Baystate Medical Center
Springfield, Massachusetts

Antonio I. Picon, MD [16]


Assistant Professor of Surgery
Beth Israel Medical Center
New York, New York

Michael D. Pasquale, MD [25]


Chair, Department of Surgery
Division of Trauma-Surgical Critical Care/General Surgery/Burn
Lehigh Valley Health Network
Allentown, Pennsylvania

Edgar Joseph Pierre, MD [109]


Associate Professor of Anesthesia and Surgery
University of Miami School of Medicine
Anesthesiologist
Jackson Memorial Hospital
Miami, Florida

Harvey I. Pass, MD [80]


Professor, Department of Cardiothoracic Surgery
Director, Division of Thoracic Surgery and Thoracic Oncology
New York University School of Medicine
NYU Langone Medical Center & Cancer Center
New York, New York

Peter W.T. Pister, MD, FACS [57C]


Professor, Department of Surgical Oncology
Division of Surgery
The University of Texas MD Anderson Cancer Center
Houston, Texas

Benjamin J. Pearce, MD [84]


Assistant Professor, Vascular Surgery
University of Texas Health Science Center
San Antonio, Texas
W. Brian Perry, M.D., FACS, FASCRS
Department of General Surgery
San Antonio Military Medical Center
San Antonio, Texas
Yaron Perry, MD [10]
Assistant Professor of Surgery
Mercer University School of Medicine
Memorial University Medical Center
Director, Minimally Invasive Thoracic Surgery
Curtis & Elizabeth Anderson Cancer Institute
Savannah, Georgia
David S. Pertsemlidis, MD
Assistant Clinical Professor
Department of Surgery
The Mount Sinai Hospital
New York, New York

[49]

Demetrius Pertsemlidis, MD
Clinical Professor of Surgery
Department of Surgery
The Mount Sinai Hospital
New York, New York

Brad H. Pollock, MPH, PhD [1]


Dielmann Distinguished University Endowed Professor and
Chairman
Department of Epidemiology & Biostatistics
University of Texas Health Sciences Center
San Antonio, Texas
Raphael E. Pollock, MD, PhD [102]
Professor of Molecular and Cellular Oncology
Professor of Surgery
The University of Texas MD Anderson Cancer Center
Houston, Texas
Mitchell C. Posner MD [11]
Thomas D. Jones Professor and Vice-Chairman
Chief, Section of General Surgery and Surgical Oncology
University of Chicago
Chicago Illinois
Lori Pounds, MD [86]
Assistant Professor/Clinical
Division of Vascular Surgery, Department of Surgery
University of Texas Health Sciences Center
San Antonio, Texas

[49]

Sarah Pesek, MD [36]


Clinical Instructor of Surgery
University of Vermont College of Medicine
Fletcher Allen Health Care
Burlington, Vermont

PMPH_FM.indd xxiii

[35]

xxiii

Vitaliy Y. Poylin, MD [31]


Professor of Surgery
Harvard Medical School
Department of Colon & Rectal Surgery
Beth Israel Deaconess Medical Center
Boston, Massachusetts
Jason D. Prescott, MD
Clinical Fellow in Surgery
Department of Surgery

[71]

5/22/2012 6:23:44 PM

xxiv

List of Contributors

Massachusetts General Hospital


Boston, Massachusetts
Basil A. Pruitt Jr., MD [Foreword]
Professor, Department of Surgery
Trauma Division
University of Texas Health Sciences Center
San Antonio, Texas
Joe B. Putnam, Jr., MD [79]
Professor of Surgery and Chairman
Department of Thoracic Surgery
Ingram Professor of Cancer Research
Professor of Biomedical Informatics
Vanderbilt University Medical Center
Nashville, Tennessee
Seshadri Raju, MD, FACS
Vascular Surgeon
Flowood, Mississippi

Robert E. Roses, MD [101]


Fellow in Surgical Oncology
The University of Texas MD Anderson Cancer Center
Houston, Texas
Brian G. Rubin, MD [93]
Professor, Surgery, Division of General Surgery
Vascular Surgery Section
Washington University School of Medicine
St. Louis, Missouri

[97C]

Peter Rhee, MD, MPH [115]


Professor of Surgery
Chief, Section of Trauma, Critical Care and Emergency
Surgery
Department of Surgery
Arizona Health Sciences Center, University of
Arizona
Tucson, Arizona
J. David Richardson, MD [82C]
Professor and Vice Chairman of Surgery
Division of General Surgery
University of Louisville School of Medicine
Department of Surgery
Louisville, Kentucky
David W. Rittenhouse, MD [58]
Senior Surgical Resident
Department of Surgery
Thomas Jefferson University Hospital
Philadelphia, Pennsylvania
Patricia L. Roberts MD [37C]
Chair, Department of Colon and Rectal Surgery
Lahey Clinic
Burlington, Massachusetts
Professor of Surgery
Tufts University School of Medicine
Boston, Massachusetts
Manuel Rodriguez-Davalos, MD, FACS [42]
Assistant Professor of Surgery (Transplant) and Pediatrics
Yale University School of Medicine
New Haven, Connecticut
Raul J. Rosenthal, MD, FACS, FASMBS [55]
Department Chair
Bariatric and Metabolic Institute (BMI)
Weston, Florida

PMPH_FM.indd xxiv

Daniel F. Roses, MD [101]


Jules Leonard Whitehill Professor of Surgery and Oncology
New York University School of Medicine
Director of Surgical Oncology
NYU Langone Medical Center
New York, New York

Mark L. Ryan, MD [98]


Division of Trauma and Surgical Critical Care Services
University of Miami Miller School of Medicine
Jackson Memorial Hospital / Ryder Trauma Center
Miami, Florida
Abdul Saied, MD [46]
General Surgery Resident
University of Illinois College of Medicine
Chicago, Illinois
Barry Salky, MD FACS [24]
Chief, Division of Laparoscopic Surgery (Emeritus)
Professor of Surgery
The Mount Sinai Hospital
New York, New York
Alejandro F. Sanz, MD [6]
Fellow, Department of Cardiothoracic Surgery
University of Pittsburgh Medical Center
Pittsburgh, Pennsylvania
Mark D. Sawyer, MD [106]
Assistant Professor
Trauma, Critical Care and General Surgery
Mayo Clinic
Rochester, Minnesota
Thomas M. Scalea, MD, FACS, FCCM [81C]
Physician-in-Chief, Shock Trauma Center
Francis X. Kelly/Professor of Trauma Surgery
Director, Program in Trauma
University of Maryland School of Medicine
Baltimore, Maryland
Martin A. Schreiber, MD, FACS [25C]
Professor of Surgery
Section of Trauma/Critical Care
Oregon Health & Science University
Portland, Oregon
Joshua Schulte, MD
Chief Surgical Resident

[37]

5/22/2012 6:23:44 PM

List of Contributors

Department of General Surgery


Michigan State University
Kalamazoo Center for Medical Studies
Kalamazoo, Michigan
Kevin Schuster, MD, FACS [105]
Assistant Professor of Surgery (Trauma)
Yale School of Medicine
Trauma, Surgical Critical Care and Surgical Emergencies
Yale Medical Group
New Haven, Connecticut
Steven Schwaitzberg, MD
Chief of Surgery
Cambridge Health Alliance
Associate Professor Surgery
Harvard Medical School
Department of Surgery
Cambridge, Massachusetts

[38, 65, 66]

Wayne H. Schwesinger, MD [13, 53C]


Professor and Interim Chief
Division of General and Laparoendoscopic Surgery
Director and Chief, General Surgery B and Surgical Endoscopy
University of Texas Health Science Center
San Antonio, Texas
Ashok R. Shaha, MD [103]
Jatin P. Shah Chair in Head and Neck Surgery and Oncology
Memorial Sloan-Kettering Cancer Center
New York, New York
Amir Sharif, MD [15]
Fellow, Minimally Invasive Surgery
Section of Gastrointestinal Surgery
Department of Surgery
Yale University School of Medicine
New Haven, Connecticut

University of Texas Health Science Center


San Antonio, Texas
Michael J. Sise, MD, FACS [95C]
Clinical Professor of Surgery
University of California, San Diego School of Medicine
Trauma Medical Director
Scripps Mercy Hospital
San Diego, California
Sherry Sixta, MD [61C]
Acute Care Surgery/Surgical Critical Care Fellow
University of Texas Health Science Center
Houston, Texas
David W. Smith, MD [50]
Surgical Group
Banner Estrella Medical Center
Phoenix, Arizona
Dror Soffer, MD [61]
Director, Yitzhak Rabin Trauma Division
Tel Aviv Sourasky Medical Center
Department of Surgery
Tel Aviv, Israel
Jamii St. Julien, MD, MPH [57]
Post Doctoral Fellow
TREAT Lung Cancer Program
Vanderbilt University School of Medicine
Nashville, Tennessee
Kenneth Stahl, MD, FACS [2]
Associate Professor of Surgery
Director of Patient Safety
DeWitt Daughtry Family Department of Surgery
William Lehman Injury Research Center Medical Director
Ryder Trauma Patient Safety Organization
The University of Miami Leonard M. Miller School of Medicine
Miami, Florida

Maureen K. Sheehan, MD [97]


Assistant Professor of Surgery
Division of Vascular/Endovascular Surgery
University of Texas Health Science Center
San Antonio, Texas

Megan Steigelman, MD [107]


US Air Force General Surgery
Travis AFB
Vacaville, California

Joseph H. Shin, MD, FACS [104]


Chief, Division of Plastic Surgery
Tufts University School of Medicine
Baystate Medical Center
Springfield, Massachusetts

Randolph Steinhagen, MD [19]


Professor of Surgery
Chief, Division of Colon and Rectal Surgery
The Mount Sinai School of Medicine
New York, New York

Gregorio Sicard, MD, FACS [83C]


Eugene M. Bricker Professor of Surgery
Executive Vice Chairman, Department of Surgery
Washington University School of Medicine
St. Louis, Missouri

Ronald M. Stewart, MD [18C]


Professor and Chair
Department of Surgery
Jocelyn and Joe Straus Endowed Chair in Trauma Research
University of Texas Health Science Center
San Antonio, Texas

Matthew J. Sideman, MD [86]


Associate Professor of Surgery
Division of Vascular/Endovascular Surgery

PMPH_FM.indd xxv

xxv

Robert P. Sticca, MD, FACS


Professor and Chairman

[29]

5/22/2012 6:23:44 PM

xxvi

List of Contributors

Department of Surgery
University of North Dakota School of Medicine and Health
Sciences
Grand Forks, North Dakota
Andrew Tang, MD [115]
Assistant Professor
Department of Trauma, Critical Care and Emergency Surgery
Arizona Health Sciences Center, University of Arizona
Tucson, Arizona
Nicholas Tarmey, MBChB, FRCA, RAMC [115]
Divsion of Trauma Anesthesiology
R. Adams Cowley Shock Trauma Center Baltimore, Baltimore,
Maryland

[28]

Robert W. Thompson, MD [89]


Professor of Surgery
Center for Thoracic Outlet Syndrome
Section of Vascular Surgery, Department of Surgery
Washington University School of Medicine
St. Louis, Missouri
Chad M. Thorson, MD [98]
University of Miami Miller School of Medicine
Division of Trauma and Surgical Critical Care Services
Jackson Memorial Hospital / Ryder Trauma Center
Miami, Florida
Keila E. Torres, MD, PhD [102]
Assistant Professor
Department of Surgical Oncology
The University of Texas MD Anderson Cancer Center
Houston, Texas
Boulos Toursarkissian, MD [83]
Professor and Chief
Division of Vascular Surgery
University of Texas Health Science Center
San Antonio, Texas
Magdiel Trinidad-Hernandez, MD
Assistant Professor of Surgery
Department of Surgery
University of Arizona
Tucson, Arizona

[100]

Olga Tucker, MD, FRCSI, FRCS [55]


Senior Lecturer/Consultant Surgeon

PMPH_FM.indd xxvi

Joshua A. Tyler, MD [35]


Department of General Surgery
San Antonio Military Medical Center
San Antonio, Texas
Andreas G. Tzakis, MD, PhD, FACS [40]
Professor of Surgery
Miami Transplant Institute
University of Miami Miller School of Medicine
Miami, Florida
Robert Udelsman, MD, MBA [71]
Chairman, Department of Surgery
Carmalt Professor of Surgery and Oncology
Yale University School of Medicine
Surgeon-in-Chief
Yale New Haven Hospital
New Haven, Connecticut

Paul Ian Tartter, MD, FACS [77]


Associate Professor
Division of Breast Surgery
Department of Surgery
St. Lukes-Roosevelt Hospital Center
New York, New York
Burke Thompson, MD, MPH
Central Carolina Surgery
Greensboro, North Carolina

The Academic Department of Surgery


Queen Elizabeth Medical Centre
Edgbaston Birmingham, West Midlands, United Kingdom

Kent R. Van Sickle, MD [64]


Associate Professor of Surgery
Director, UTHSCSA Johnson Center for Surgical Innovation
Division of General and Laparoendoscopic Surgery
University of Texas Health Science Center
San Antonio, Texas
Frank J. Veith, MD [88C]
Professor, Department of Surgery
New York University School of Medicine
NYU Langone Medical Center
New York, New York
George C. Velmahos, MD, PhD, MSEd [99]
John F. Burke Professor of Surgery
Harvard Medical School
Chief, Division of Trauma, Emergency Surgery, and Surgical
Critical Care
Massachusetts General Hospital
Boston, Massachusetts
Steven G. Venticinque, MD [110]
J. Jeff rey Andrews, MD and R. Brian Smith Endowed
Professor and Chair
University of Texas Health Science Center
San Antonio, Texas
David Vogel, MD [92]
Senior Vascular Surgery Fellow
Jobst Vascular Institute
The Toledo Hospital
Toledo, Ohio
Thai Vu, MD [98]
Division of Trauma and Surgical Critical Care Services
University of Miami Miller School of Medicine
Jackson Memorial Hospital / Ryder Trauma Center
Miami, Florida

5/22/2012 6:23:44 PM

List of Contributors

Daniel B. Walsh, MD [92C]


Medical Director, Referring Physician Services
Vascular Surgery
Dartmouth-Hitchcock Medical Center
Lebanon, New Hampshire
Jodi Walters, MD [96C]
Southern Arizona Limb Salvage Alliance (SALSA)
University of Arizona College of Medicine
Tucson, Arizona
J. Matthias Walz, MD [112]
Chief, Vascular Anesthesiology
UMass Memorial Medical Center
Associate Professor of Anesthesiology
University of Massachusetts Medical School
Worcester, Massachusetts
Howard T. Wang, MD [104C]
Associate Professor and Chief
Plastic and Reconstructive Surgery
Program Director, Plastic Surgery Residency Program
Division of Plastic and Reconstructive Surgery
University of Texas Health Science Center
San Antonio, Texas
Kenneth Waxman, MD [114C]
Medical Director
Cottage Hospital Trauma Center
Cottage Childrens Hospital
Santa Barbara, California
Kaare J. Weber, MD [120]
Assistant Professor of Surgery
Department of Surgery
Mount Sinai School of Medicine
The Mount Sinai Hospital
New York, New York

PMPH_FM.indd xxvii

xxvii

John A. Weigelt, MD [3C]


Professor
Department of Surgery
Division of Trauma/Critical Care
Medical College of Wisconsin
Milwaukee, Wisconsin
Christopher E. White, MD, MSc, FACS
Department of General Surgery
Brooke Army Medical Center
San Antonio, Texas

[111]

Richelle Williams, MD [72C]


American College of Surgeons
Department of Surgery
University of Chicago Pritzker School of Medicine
Cancer Programs
Chicago, Illinois
David P. Winchester, MD, FACS [72C]
American College of Surgeons
Department of Surgery
University of Chicago Pritzker School of Medicine
Cancer Programs
Chicago, Illinois
Charles J. Yeo, MD, FACS [58]
Chair and Samuel D. Gross Professor
Department of Surgery
Thomas Jefferson University Hospitals
Philadelphia, Pennsylvania
Luke X. Zhan, MD [88]
Division of Vascular and Endovascular Surgery
University of Arizona Health Science Center
Southern Arizona Limb Salvage Alliance (SALSA)
Tucson, Arizona

5/22/2012 6:23:44 PM

PMPH_FM.indd xxviii

5/22/2012 6:23:44 PM

FOREWORD
The practice of surgery has always been evidence-based, with
early evidence being the opinion of authorities of the day such
as Hippocrates, Ambroise Par, John Hunter, and Samuel Gross.
The importance of adding data to support authoritarian opinion
is well illustrated by the largely ignored 17th century observations
of John Woodall on scurvy and the response, even though forty
years delayed, to James Linds 18th century study of scurvy. Linds
1754 study, considered to be the first controlled clinical trial,
provided the evidence to mandate, in 1795, the use of lemons to
prevent scurvy in British seamen. The importance of evidence is
further exemplified by Florence Nightingale, who has been credited with being one of the first to use statistics in her studies of
hospital infection in the Crimean War (18541856). Her statistics
generated evidence to support improvements in care.
In the past 150 years, authoritative opinion has been more
and more often supported by randomized, controlled clinical trials or correlative laboratory studies that have become progressively more complex and scientifically rigorous as technology has
advanced and sophisticated statistical assays have been utilized.
Today, evidence is instantly available in the publications of the
Cochrane Database of Systemic Reviews and the quality of evidence can be readily graded according to criteria promulgated by
the Oxford Centre for Evidence-Based Medicine.
The textbook has occupied a central role as the transmitter of evidence-based practice to the surgical community at
large, extending from medical students to practicing surgeons.
It too has evolved from a compendium of personal experience
to the present volume for which the editors have selected clinicians with broad understanding of evidence-based medicine to
author each chapter. The opening chapter presents a concise yet

thorough review of the development of evidence-based medicine,


describes the process of data evaluation, gives consideration to
statistical principles and pitfalls, and provides an example of a
recent systematic review, that is, it sets the stage for the following chapters. Subsequent chapters bring Socrates to the surgical
textbook by posing questions about the what, when, and why
related to important, typically controversial or uncertain, aspects
of surgical care of specific diseases and conditions. The authors
of those chapters have provided what they consider to be the best
available recent evidence to answer those questions and guide the
readers in making management decisions in their practice of elective surgery.
Recognizing that evidence-based surgery does not provide a
hard and fast solution to every surgical problem, broadly experienced more senior surgical authorities provide commentary
for selected chapters. These commentaries illustrate the importance of integrating the published evidence with surgeon-specific
expertise and individual patient needs in developing a clinically
feasible, patient-centered approach to a surgical problem. An
understanding of that methodology will extend the influence of
this text and enable the readers throughout their surgical careers
to incorporate new findings and study results into their evidence
data base and broaden the applicability of the refi ned evidence.
The Socratic process used by the editors to address controversial
aspects of elective surgical management will enhance and expand
the readers use of evidence-based surgery to achieve optimum
patient outcomes and document, as did Socrates pupil and successor Plato, their evidence-based competency.
Basil A. Pruitt, Jr., MD

xxix

PMPH_FM.indd xxix

5/22/2012 6:23:44 PM

PMPH_FM.indd xxx

5/22/2012 6:23:44 PM

PREFACE
Attention to evidence-based surgical medicine began for the two
editors in 1983 during our surgical residency at Boston University and at morning report at Boston City Hospital Surgical Services. Our Chairman of Surgery, Lester.F Williams,James F. Utley
Professor of Surgery, conducted Rounds in a Socratic manner, and
he answered every question with the inquiry Show me the data.
He invariably had the data organized within an encyclopedic
brain and instilled within all his residents s a thirst for EvidenceBased Medicine (EBM).
The basic principle guiding the preparation of this text was
that it should be a dynamic, comprehensive reference focused on
the important surgical questions about each disease. The answers
to each clinical question should be authoritative and reflect evidenced-based medicine recommendations for level of evidence
and grade of recommendation.
Evidenced-based medicine connotes a commitment on the
part of the author and a confidence level among the readership
that the data presented is obtained from the highest levels of
surgical scientific research and observation. We believe that our

distinguished chapter authors have fulfi lled this obligation and


that our readers can be confident in the answers provided to the
questions posed.
The book is organized with a fi nite number of surgical questions for each chapter subject for general, vascular, and pediatric content. Each question is answered with concise EBM
data,including a table demonstrating the level of evidence and
grade of recommendation. Subsequent editions to the textbook
will easily incorporate new studies and relevant surgical science
questions and answers. We have attempted to combine the best
of EBM for surgery with the most up-to-date, prospective, randomized controlled trials, large meta-analyses, and cohort data
scrutinized for validity and recommendations. We hope that
this data can guide the experienced surgeon as well as the novice learner through the management and treatment of surgical
diseases.
Steven T. Brower
Stephen M. Cohn

xxxi

PMPH_FM.indd xxxi

5/22/2012 6:23:44 PM

PMPH_FM.indd xxxii

5/22/2012 6:23:44 PM

CHAPTER 1

Understanding the Elements that


Comprise Evidence-Based Medicine
Joel E. Michalek, John E. Cornell, and Brad H. Pollock

INTRODUCTION

through an objective, rigorous, and critical systematic review


of the available evidence. Guyatt et al. posited two fundamental
principles of evidence-based medicine8: (1) a hierarchy of evidence
to guide clinical decision making; and (2) evidence alone is never
sufficient to make a clinical decision. Decision makers must always
consider tradeoffs between benefits and risks, inconvenience, and
costs associated with alternative management strategies and, in
doing so, consider their patients values and preferences.

Background
The idea of using evidence in medicine has been traced to ancient
Greece1 and ancient China,1,2 and testing medical interventions for
safety has existed since Avicennas The Canon of Medicine in the 11th
century.3,4 It was only in the 20th century that this effort evolved to
affect nearly all fields of health care. Professor Archie Cochranes
book Effectiveness and Efficacy: Random Reflections on Health
Services,5 and his subsequent advocacy for a scientific foundation for
clinical practice, led to increased acceptance of the concepts behind
evidence-based medicine. Cochranes work is recognized through
the naming of centers of evidence-based medical research, the
Cochrane Centers, and an international organization, the Cochrane
Collaboration (www.cochrane.org), after him. The goal of this international collaboration is to collect, evaluate, and summarize all the
best available scientific evidence regarding the benefits and harms
of a medical intervention. Explicit methodologies that are used
to determine best evidence were developed by a research group
at McMaster University led by David Sackett and Gordon Guyatt.
Guyatt later used the term evidence-based in 1990,6 and the term
evidence-based medicine first appeared in the medical literature
in 1992 in a paper by The Evidence-Based Medicine Working Group.7
A recent summary of evidence-based medicine, Users Guides to the
Medical Literature. A Manual for Evidence-Based Clinical Practice,
has been published.8 Relevant journals include the British Medical
Journals Clinical Evidence, the Journal of Evidence-Based Health
Care, and Evidence-Based Health, Policy, all co-founded by Anna
Donald, an Australian pioneer in the discipline.9
Cochrane defines evidence-based medicine as the conscientious, explicit, and judicious use of current best evidence in making decisions about the care of individual patients. The practice
of evidence-based medicine means integrating individual clinical expertise with the best available scientific evidence compiled

WHAT IT IS AND WHAT IT ISNT


In an editorial, David Sackett defined evidence-based medicine
as the explicit and judicious use of current best evidence in making clinical decisions about individual patients.1 Evidence regarding the relative benefits and harms of a medical intervention
comes from clinically relevant scientific research, especially from
patient-oriented clinical research into the accuracy and precision
of diagnostic tests (including the clinical examination), the power
of prognostic markers, and the efficacy and safety of therapeutic,
rehabilitative, and preventive regimens. Evidence-based medicine
is not cookbook medicine. It requires a bottom-up approach
that integrates best external evidence with individual expertise
and patients choice, emphasizing that external evidence can
never replace individual expertise and patient preference.1
In this chapter, we focus on the first principle, systems for rating evidence, assessing the strength of evidence, and an example
of a systematic review, considered by some to be one of the highest
forms of evidence. Throughout the chapter we attempt to address
aspects of this topic as it pertains to surgery.

SYSTEMS FOR RATING EVIDENCE


The quality and strength of evidence provided by scientific clinical
research is largely determined by study design and the degree to
1

PMPH_CH01.indd 1

5/21/2012 7:41:09 PM

Surgery: Evidence-Based Practice

which investigators minimize the risk of bias in the conduct and


reporting of their findings. Consideration of these factors has led
to the development of hierarchies of evidence and various scoring
systems to help clinicians and researchers evaluate the quality and
strength of the available evidence.
Hierarchies of Evidence: At least three hierarchies of evidence
have been published. These hierarchies are largely based on study
design. Evidence derived from a randomized controlled clinical trial
(RCT) or a quantitative summary of treatment effects from multiple
RCTs investigating a common intervention provides the greatest
level of scientific rigor and internal validity. Traditional clinical case
studies and case-series designs represent the weakest level of evidence. Some of the hierarchies incorporate general assessments of
how well the study was designed, conducted, and reported.
US Preventive Services Task Force,65 for example, developed a
system that considers well-designed RCTs as providing the highest
level of evidence and opinions of respected authorities at the lowest level of evidence:
Level I: Evidence obtained from at least one properly designed
and randomized controlled trial
Level II-1: Evidence obtained from well-designed controlled trials
without randomization
Level II-2: Evidence obtained from well-designed cohort or casecontrol analytic studies, preferably from more than one center
or research group
Level II-3: Evidence obtained from multiple time series with or
without intervention. Dramatic results in uncontrolled trials
might also be regarded as this type of evidence
Level III: Opinions of respected authorities, based on clinical
experience, descriptive studies, or reports of expert committees
The UK National Health Service developed a similar system, with
categories labeled A, B, C, and D. This system was developed by the
Oxford Centre for Evidence-Based Medicine (http:/www.cebm.net/
index.aspx?o=1025), and it emphasizes consistency of the evidence
across a variety of study designs, as well as type of study design:
Level A: Consistent RCT, cohort study, all or none, clinical
decision rule validated in different populations
Level B: Consistent retrospective cohort, exploratory cohort,
ecological study, outcomes research, case-control study, or
extrapolation from level A studies
Level C: Case-series study or extrapolation from level B studies
Level D: Expert opinion without explicit critical appraisal, or
based on physiology, bench research, or first principles
The McMaster University group developed a system based largely on
a study design that gives more weight to randomized N of 1 studies
that directly assess the efficacy and safety of a medical intervention at
the individual patient level. In Users Guides to the Medical Literature.
A Manual for Evidence-Based Clinical Practice, Guyatt et al.8 emphasized the importance of the N of 1 studies to address the individual
patient and systematic reviews to guide clinical decision making:

N of 1 randomized trial
Systematic reviews of randomized trials
Single randomized trial
Systematic review of observational studies addressing patientimportant outcomes
Single observational study addressing patient-important outcomes
Physiologic studies (studies of blood pressure, cardiac output,
exercise capacity, bone density, and so forth)
Unsystematic clinical observations.

PMPH_CH01.indd 2

The N of 1 randomized trial is a study of one patient with repeated


blocks of two or more treatments given in a random order in each
block. The N of 1 study is preferably double-blind and placebo or
otherwise controlled and is intended to address the efficacy and
safety of a single therapy in an individual patient. While this
design has little or no direct application in surgical intervention
research, systematic reviews of randomized and nonrandomized
observational studies can provide substantial evidence about the
efficacy and safety of a surgical intervention.
A systematic review applies the fundamental principles of the
scientific method to the process of reviewing and evaluating the
benefits and harms associated with a medical or surgical intervention. Systematic reviews follow a prespecified protocol with
explicit criteria used to find, select, critique, and synthesize evidence relevant to a well-defined clinical question about diagnostic accuracy, clinical prognosis, or the safety and efficacy of an
intervention. Critical to this process is a careful assessment of the
quality and consistency of the available evidence.
The Grading of Recommendations Assessment, Development, and Evaluation (GRADE) Working Group, an international
collaboration of clinical scientists, developed a system for grading the quality of evidence and the strength of recommendations
provided by a systematic review (http://www.gradeworkinggroup.
org/intro.htm). It combines information about study design, consistency of evidence, and risk of bias into a unified system to assess
the quality and strength of the evidence available to assess the efficacy and safety of a medical or surgical intervention.
GRADE Criteria: The GRADE working group was established
in 2008 to develop a new system of rating quality of evidence and
providing medical recommendations that depends less on hierarchy and more on the types of evidence and specific strengths and
weaknesses.10 Its primary focus was to develop a set of criteria for
organizations and groups involved in the development of clinical
guidelines to objectively evaluate the degree to which a particular body of evidence can support a proposed guideline for clinical
practice. Evidence is categorized as being high, low, or very low
grade, and recommendations are given as strong and weak. The
GRADE criteria for assigning grade of evidence are summarized
in Table 1.1.
The strength of a GRADE recommendation (strong or weak)
is based on quality of evidence, uncertainty about the balance
between desirable and undesirable effects, uncertainty or variability in values and preferences, and uncertainty about whether
the intervention represents a wise use of resources.
The GRADE system provides a detailed and explicit set of criteria for ratings of quality and grading of strength evidence that
makes judgments supporting clinical guidelines and recommendations more transparent than simple reliance on a hierarchy of evidence.11 While study design provides a starting point for evaluating
the quality and strength of the evidence provided by a single study
or a collection of studies, the GRADE approach acknowledges that
poorly designed and/or executed RCTs can produce biased and
misleading results; well-designed and executed observational studies can produce quite robust and valid assessments of the comparative effects of a medical or surgical intervention. Collectively, these
methodological concerns address the internal validity or risk of bias
associated with a particular study or collection of studies.
Randomized trials that lack allocation concealment or blinding tend to produce biased results that tend to favor an intervention. Patient attrition and failure to capture primary outcomes
compromise randomization and threaten the integrity of the

5/21/2012 7:41:09 PM

Evidence-Based Medicine

Table 1.1 GRADE Criteria for Rating the Quality and Strength of Evidence Regarding a Medical or
Surgical Intervention
Type of evidence

Randomized trial = high


Observational study = low
Any other evidence = very low

Decrease* grade if

Increase grade if

Range

Serious or very serious limitation to study quality


Important inconsistency
Some or major uncertainty about directness
Imprecise or sparse data
High probability of reporting bias
Strong evidence of associationsignificant relative risk >2 (<0.5) based on consistent evidence from
two or more observational studies, with no plausible confounders (+1)
Very strong evidence of associationsignificant relative risk >5 (<0.2) based on direct evidence with
no major threats to validity (+2)
Evidence of a dose response gradient (+1)
All plausible confounders would have reduced the effect (+1)
High-quality evidence
Moderate-quality evidence
Low-quality evidence
Very-low-quality evidence

* Each quality criterion can reduce the quality by one or, if very serious, two levels.

data unless an adequate intent-to-treat analysis is used. Selective reporting of outcomes tends to overestimate the benefits of
an intervention. Randomized trials that fail to provide a proper
accounting of allocation concealment, blinding, patient attrition,
and missing data, and all prespecified outcomes are penalized
(downgraded) in the GRADE system.
Observational studies that develop and apply appropriate eligibility criteria in selection of exposed and unexposed subjects in
cohort studies or provide adequate matching of cases and controls in case-control studies can provide robust estimates of the
comparative effectiveness of a medical or surgical intervention.
The quality and strength of the evidence from these observational
studies is further upgraded or downgraded based on how completely exposures and outcomes were ascertained, how adequately
and completely the study controlled for sources of confounding,
and how well the study maintained follow-up on all participants.
The evolution of the GRADE system into a mature comprehensive approach for assessing study quality and strength of evidence
is documented in a new series of articles published in the Journal of
Clinical Epidemiology.11 The value of the GRADE system is widely
recognized and used by a number of government organizations
and professional medical societies to assist in guideline development, including the World Health Organization, the American
College of Physicians, The American Thoracic Society, UpToDate
(www.uptodate.com), and at least 20 other organizations. Here we
summarize methods to assess the strength of evidence in terms of
bias, study type, statistical precision, and the choice of endpoint.

ASSESSING THE QUALITY AND


STRENGTH OF EVIDENCE
Four attributes define the strength of evidence. The first is the
level of the evidencedictated by the type of study design that
was used. The second is the quality of evidencedirectly related
to risk of bias. The third is statistical precisionthe degree to

PMPH_CH01.indd 3

which true effects can be distinguished from spurious effects due


to random chance. The fourth is the choice of a study endpoint
to measure an effectan endpoints appropriateness to truly represent a clinically meaningful effectand the magnitude of the
observed effect. For practical reasons, the selection of study subjects for a particular study is almost always a compromise. The
degree to which a chosen study population represents an intended
target population must also be considered; selection bias can
compromise a studys weight of evidence.

STUDY DESIGN
The first step in evaluation of the quality and strength of evidence is
understanding the scope and limitations inherent in the type(s) of
study design(s) used to collect the data. These factors form the basis
for the various hierarchies of evidence described in the preceding
section. Here we describe the scope and limitations of many of the
popular study designs used to collect and report evidence regarding the efficacy and safety of clinical and surgical interventions.
Several different types of studies are used in clinical research.
Researchers have developed and evaluated various ontologies
for study design classifications and developed electronic tools to
assist in classifying published studies according to their design
features.12,13 The primary distinction made in the GRADE system
is between randomized and nonrandomized studies. The latter is
collectively referred to as observational studies.
The simplest observational designs start with case reports and
case series that can be used to document the effects of an intervention or clinical course. However, these sources are subject to
selection bias, often use unblinded subjective rather than blinded
objective outcome assessment, are imprecise due to measurement
errors, and typically include only small number of patients. Case
reports and case series usually have no control groups for comparisons. Case-control studies sample patients based on the presence/
absence of a medical condition. Patients with the outcome of interest

5/21/2012 7:41:09 PM

Surgery: Evidence-Based Practice

are referred to as the cases, and a group of unaffected subjects


with similar sociodemographic and medical profi les are selected
as controls. Case-control studies usually have little problem
accruing case subjects, are free from the burden of patient recruitment, can be performed in a shorter period of time, and are often
much less expensive to conduct than cohort or other prospective studies. However, a temporal relationship between cause and
effect can only be inferred and not directly measured because of
the retrospective nature of case-control studies. These studies are
also subject to biased recall of antecedent exposures and selection
bias, especially in the selection of controls. Case-control studies are most often used for rare outcomes or when there is a long
induction period between an exposure and the outcome.
Prospective cohort studies recruit subjects free of the outcome
of interest and are composed of two or more groups, including
an exposed or index group and a control group. Subjects are then
dynamically followed over time for the occurrence of the outcome
of interest. Recruitment may be selective and based on enrolling an
equal number of subjects from preselected exposures categories;
matching on other factors may reduce confounding and improve
the precision of comparisons across exposure groups. Alternatively, recruitment to cohort studies need not be based on predetermined categories of exposure; these are commonly studies with
several exposures of interest. An alternative design is the historical
cohort study. These studies use preexisting information, often in
a comprehensive database, to historically classify exposure status.
The database is then gleaned for information about subsequent
outcome events. Prospective cohort studies can be more expensive
than other designs. An exposure of interest, such as a new surgical procedure versus a conventional procedure, may be associated
with confounders, known or unknown, measurable or unmeasurable. Because cohort studies are not randomized, the distribution
of confounders may not be balanced between the treatment groups,
thus leading to confounding. Prospective studies are more time
consuming than case-control studies. A major advantage of cohort
designs is that they provide a clear picture of the temporal relationship between a cause and an effect. Matching can efficiently reduce
confounding due to a subset of known and measurable confounders. In addition, multiple outcomes can be evaluated in contrast to
case-control studies with a single outcome of interest.
In all but the Guyatt et al.8 hierarchy, RCTs provide the greatest weight of evidence. In these studies, the allocation of subjects
to an exposure of interest or control is done solely for the purpose
of obtaining an unbiased estimate of the treatment effect. The key
advantage of RCTs is the low likelihood of bias due to confounding.
Although controlling for known confounders can be performed
using techniques such as restriction, stratification, or statistical
adjustment, randomization tends to balance the distribution of all
confounders, known or unknown, measurable or unmeasurable,
between treatment groups. Some RCTs can be blinded. The disadvantages of RCTs are possibly high costs and recruitment barriers
(particularly for subjects who do not give consent).
Other study designs are used less frequently in medical
research. Cross-sectional studies collect both exposure and outcome information simultaneously, may be more applicable for prevalent rather than for acute conditions, and do not address cause and
effect temporal relationships. In crossover studies, subjects serve as
their own controls. In a two treatment, two period crossover, half
the study population receives the primary treatment first and then
crosses over to receive the second treatment; the other half receives
the treatments in reverse order. An assumption of crossover studies

PMPH_CH01.indd 4

is that the residual effects of a treatment disappear by the time the


groups are crossed over, and will be violated, for example, with
surgical interventions where a subjects condition is permanently
altered by the therapy (e.g., limb amputation). Crossover trials,
when possible, are more efficient than parallel arm clinical trials, in
that they require fewer patients than the parallel group design and
have the advantage that every patient will receive all treatments.

BIAS
The strength of scientific evidence provided by an individual
study is dependent on a number of key factors that are associated
with how well a study design is implemented and reported. These
key factors address the internal (risk of bias) and external validity (generalizability) of a study, and careful consideration of these
factors are essential before attempting to make clinical inferences from a study. Failure to minimize risk of bias undermines
the value of a randomized study, whereas a well-designed cohort
or case-control study that minimizes risk of bias enhances the
strength of its contribution to the body of evidence. Ideally, studies are both internally and externally valid. Compromised validity
lowers a studys contribution to the body of evidence.
Bias is a systematic error that affects inferences derived from
the results of a study. Internal validity refers to a studys lack of bias.
External validity refers to the generalizability of a study, addressing whether the results can be extrapolated to another population of
interest. Internal validity should be the primary consideration when
reviewing a study. If a study is not internally valid, one need not
consider whether it is externally valid; that is, biased study results
should never be extrapolated to another population. For intervention studies, internal validity addresses whether observed effects can
be attributed to the treatment effect or whether they are attributed to
alternative explanations such as bias or lack of statistical precision.
The validity of all patient-oriented research studies is strongly
related to the risk of bias inherent in the design and execution
of a study. The higher the risk of bias in an individual study, the
more compromised are its findings. Higher risk of bias tends to
produce overly optimistic estimates of the benefits of a medical or
surgical intervention. The internal validity of a particular study
is affected by observer bias, measurement bias, confounding and
statistical precision. Confounding is the mixing up of sources of
bias with the true effects of an intervention so that the primary
effect under study cannot be separated from the influence of
extraneous factors. For example, failing to account for preoperative disease severity in a randomized trial evaluating two surgical
approaches might lead to confounding if the severity distribution
differed between groups. These potential problems can manifest
themselves in different ways for different study designs.
Measurement bias is inaccuracy related to the method of measuring outcomes for a study. Examples include poorly calibrated
blood pressure readings, inaccurate height measurements, flawed
laboratory methods that give erroneous values or that fails to accurately reflect clinically meaningful categories. Observer bias is inaccuracy related to measuring a study outcome where the observer
knows the intervention group assignment. A technique known
as blinding is used in randomized and nonrandomized studies
to minimize observer bias. Study participants, health professionals involved in patient management, and study personnel responsible for collecting data are denied access to information regarding
treatment assignment or the primary risk factors and outcomes of

5/21/2012 7:41:09 PM

Evidence-Based Medicine

interest. Observer bias is more likely to occur in unblinded studies


when the chosen outcome measure is subjective, such as the occurrence of symptoms or toxicities, patient self-report measures, and
interpretations of physical examination findings. Double blinding
is a technique where neither the observer nor the patient knows
the treatment assignment. For many surgical interventions such
as total limb versus partial limb amputation, or for regimens with
very idiosyncratic symptom or toxicity profiles, double blinding
may be impractical or impossible. Surgical studies are more closely
aligned with epidemiological studies in this respect. Blinding of
study personnel responsible for collecting and recording pre-, peri-,
and postoperative information and outcomes is often possible and
an effective way to minimize observer bias in surgical studies.

STATISTICAL PRECISION
It is all too easy to equate evidence with a summary statistic like
median survival time or the relative risk ratio. The evidence from
a single study or even a collection of studies is always captured
with some degree of uncertainty. Proper weighing of the evidence
requires that we pay close attention to the margin of error in our
estimates of the relative efficacy and safety of a medical or surgical intervention. Statistical precision captures and quantifies
the uncertainty surrounding any given estimate. It enables us to
distinguish real effects from those due to random chance, that is,
chance associations. Statistical precision is primarily driven by
sample size. For example, with just 10 subjects (5 in each group) a
randomized clinical trial comparing a new postsurgical antibiotic
regimen to a conventional regimen for sepsis prophylaxis is likely
to result in an extreme finding that can be attributed to random
chance, not the true biological drug effect. This phenomenon is
referred to as a small study effect in systematic reviews and clinical guideline documents, and is associated with the concept of
publication bias. Small studies with statistically significant results
have a greater likelihood of being published than studies, large
or small, that report nonstatistically significant findings. Chance
errors are less likely to occur with larger sample sizes. Trials are
always planned to limit the likelihood of chance errors; acceptable
levels of error (for Type 1 and Type 2 statistical errors) are selected
in advance and the target minimum detectable effect size is chosen. Formal sample size and power calculations are performed
during the studys design to ensure adequate statistical precision.

EXTERNAL VALIDITY
External validity is a function of whether a studys results can be
generalized to other clinical settings or populations. The question
is, Does the study population possess unique characteristics which
might modify the effect of an intervention in a way which would
render it ineffective in some other group? Subjects enrolled in a
trial may not represent the population to which the intervention
is intended to be applied. Surgical and nonsurgical intervention
studies sometimes enroll subjects at large academic institutions
and the characteristics for these enrolled patients may not represent patients seen at smaller nonacademic centers. Even within a
center, patients who consent to participate may not represent the
institutions entire clinical population.
Selection bias can occur with the self-selection of individuals who consent to participate in a research study. Both researchers and participants may bring a multitude of characteristics to a

PMPH_CH01.indd 5

clinical study, some inherent and some acquired. These can include
factors such as gender, race/ethnicity, hair, eye, and skin color,
personality, mental capability, physical status, and psychological
attitudes like motivation or willingness to participate. Differences
in the distribution of these factors between a source population
and a protocol-enrolled study population may introduce selection
bias. For example, some investigators may preferentially select
more athletic-looking subjects for an elective orthopedic surgery
clinical trial. Multicenter trials may improve the generalizability
of a study, but such studies may still suffer from selection bias.

WEIGHT OF EVIDENCE
Study design, lack of bias, statistical precision, and external validity affect a studys weight of evidence. Each of these factors must
be considered when evaluating a study. For practical reasons, the
investigator who is designing a new study is always confronted
with tradeoffs between these factors and cost. For example, having highly restrictive eligibility criteria reduces the possibility of
confounding but lowers the generalizability of a study. The choice
of an objective endpoint for an antibiotic trial (e.g., death versus
sepsis) decreases observer bias at the cost of decreased statistical precisionfewer deaths compared to the number of incident
sepsis cases. Investigators are faced with many challenges when
designing intervention studies. Because resources are almost
always limited, design compromises are made that ultimately
affect the overall weight of evidence provided by a study.

COMBINING AND SYNTHESIZING


THE EVIDENCE
Literature reviews are the primary vehicle for collecting, collating, critiquing, and combining evidence in medical and surgical
research. Ideally, the goals of a literature review are to (1) provide a
comprehensive critical review of the available evidence that identifies
the boundaries of our knowledge, (2) provide insight into sources of
inconsistency in the evidence, and (3) provide a perspective on the
generalizability and applicability of the evidence to mainstream medical and surgical practice.14,15 Historically, literature reviews appear
in various forms. Reviews can be done of single studies. Single studies may be used as the basis for making treatment decisions. A large
randomized clinical trial that appropriately evaluated a single clinical endpoint with high validity may be sufficient for medical decision
making. Alternatively, narrative reviews or systematic reviews accumulate and evaluate the evidence regarding the efficacy and safety of
a medical or surgical intervention from multiple publications.
Narrative reviews often address a broad set of clinical questions and are thus less focused on a specific question; they appear
more often in the literature and are more qualitative and less
quantitative than other reviews. In contrast, systematic reviews
are usually focused on a specific clinical issue, incorporate objective criteria for selection of studies, include an evaluation of quality and worthiness, and often use a quantitative summary to
synthesize combined results.

NARRATIVE REVIEWS
Narrative reviews are often one of the first academic endeavors that
young physicians complete during their training. The methods used

5/21/2012 7:41:09 PM

Surgery: Evidence-Based Practice

to identify and select studies for inclusion in a narrative review are


driven by subjective judgments that often lack sufficient rigor and
transparency to enable an unbiased assessment of the available evidence. The subjective nature of narrative reviews increases the likelihood that inferences are affected by imprecision and bias. Often, a
count of included studies supporting or refuting a particular issue
is determined and a winner is declared. For narrative reviews, little
consideration may be given to issues of study design, sample size,
statistical power, or study validity.

SYSTEMATIC REVIEWS
Systematic reviews are a staple of evidence-based medicine.16
These reviews provide the best means to combine evidence from
multiple studies by following a defined protocol to identify, critically appraise, summarize, and combine information on a welldefined clinical problem. They use explicit inclusion/exclusion
criteria that may restrict the inclusion of studies to specific study
designs such as RCTs, or they may include a broader set of designs.
Systematic reviews are labor intensive and costly, and they may
search for and use information from unpublished studies that
meet their prespecified inclusion/exclusion criteria. Reviewers
who undertake a systematic review face a number of challenges,
including critically appraising and combining evidence from
studies that use different designs, or different endpoints, or that
vary by other methodological characteristics.
A protocol for a systematic review follows a strict set of guidelines for selecting and amalgamating information from the literature that ensures transparency and reproducibility of its findings.
Cochrane Collaboration (http://www.cochrane.org/) guidelines
for developing a systematic review protocol requires a background
section explaining the context and rationale for the review; a statement of the objectives; a clear definition of the inclusion and exclusion criteria for studies (including study designs, study populations,
types of interventions, and outcome measures); the search strategy
for identification of studies; and the methodological approach to
the review process, including the selection of trials, assignment of
methodological quality, data handling procedures, and data synthesis. Data synthesis includes statistical considerations such as
choice of summary effect measures, assessment of heterogeneity
of effect across studies, subgroup analyses, use of random or fi xedeffect statistical models, and assessment of publication bias.

similar enough with respect to patient populations and outcome


measures to permit combining the individual point-estimates to
estimate the average effect across all of the studies. The decision
to pool or to not pool studies is made by an assessment of the
clinical equivalence of the studies and statistical heterogeneity of
effect across studies. Large variations in the clinical characteristics of the patients, treatment protocols, timing and management
of the intervention, and operational definitions for the primary
and secondary outcomes undermine the clinical equivalence of
the included studies. These sources of variation are often referred
to as sources of clinical heterogeneity, and are the most critical
elements in determining whether it is reasonable or even valid to
combine the individual study estimates into a single global estimate of the relative effectiveness of an intervention. Statistical
heterogeneity suggests that the true underlying treatment effects
in the trials are not identical and that the observed treatment
effects have greater study-to-study differences than one should
expect due to random error alone.
Uncovering, describing, and explaining clinical and statistical heterogeneity across multiple studies is a primary goal of a
meta-analysis, and it is used to elucidate previously unrecognized
differences between studies. Only in the absence of significant
clinical heterogeneity can study results be combined and a single
summary measure of effect calculated.
The calculation of a summary measure of a treatments relative effectiveness relies on the computation of a weighted mean
that gives more weight to results from studies that provide more
precise estimates of the treatment effect. These are normally the
larger and higher quality studies. This simple weighted average is
referred to as a fi xed-effect estimator. Significant statistical heterogeneity requires the use of statistical methods that incorporate
the additional uncertainty induced by study-to-study variation in
estimation of a pooled overall measure of a treatments effectiveness. These are referred to as random-effect estimators.
Often, data for all included studies are plotted on a graph
known as a forest plot, which includes a graphical representation of the magnitude of effect for each study and its degree of
uncertainty (plotted as confidence intervals). Meta-regression and
subgroup analyses are often used to assess the influence that study
characteristics have on the magnitude of a treatment effect. These
analyses are prespecified in the protocol and are used to evaluate
the effect of potential confounders and other sources of clinical
and statistical heterogeneity on the treatment effect.

META-ANALYSIS

PUBLICATION BIAS

Systematic reviews often but not always include a meta-analysis.


The goals of meta-analysis are to provide a precise estimate of
the effect, and to determine if the effect is robust across a range
of populations.17 Meta-analyses may also be performed with the
primary intent of identifying differences between studies. Metaanalyses summarize and describe the pattern of results reported
in the included studies and, when appropriate, calculate pooled
estimate of the effect of an intervention.
Data are first extracted from each individual study and then
used to calculate a point estimate for a treatment effect and a
measure of uncertainty, such as the 95% confidence interval. Th is
is repeated for each of the studies included in the meta-analysis,
and a decision is rendered as to whether the included studies are

A meta-analysis or a systematic review that includes a meta-analysis


needs to address the issue of publication bias. Publication bias refers
to the tendency on the part of researchers, reviewers, and editors
to submit, accept, and publish studies that report statistically significant results that are consistent with theoretical or previously
established empirical expectations.15 The factors shown to influence
publication include reporting significant results, type of research
design, rating of scientific importance, and external funding.18,19 In
the cases where studies with negative results are published, there
appears to be a substantial delay in publication.19
Publication bias is typically reflected in the distribution of
effects reported by smaller studies. Smaller studies with statistically significant results are more likely to be published and

PMPH_CH01.indd 6

5/21/2012 7:41:09 PM

Evidence-Based Medicine

published earlier than larger studies that fail to fi nd significant


effects for an intervention. Th is phenomenon enhances the trend
toward potentially spurious positive results in a meta-analysis,
since it is always possible that well-designed unpublished studies exist that fail to support the efficacy of a medical or surgical
intervention. Most methods that are used to detect publication
bias are based on measures of asymmetry in a funnel plot.
A funnel plot is a graph of study-effect estimates against a
measure of its precision. When publication bias is absent, the distribution of effects conform to an inverted funnel shape with estimates from the most precise studies clustered tightly at the apex.
As precision decreases, the effect estimates at each level of precision
show greater and greater variation around the average estimate.
The smallest studies show the greatest degree of variation. When
publication bias is absent, the funnel plot takes on a distinctive
asymmetric appearance with less precise studies showing a greater
tendency toward large positive effects. Statistical tests are often used
to assess and quantify the degree of asymmetry visualized in the
funnel plot.20,21

SUMMARY
Evidence-based medicine is not limited to the evaluation of RCTs
and meta-analyses. A broad range of external evidence can be
brought to bear on addressing clinical questions. Practice guidelines developed using evidence-based medicine can have a positive
effect on patient outcomes. Evidence-based medicine supplements, but does not replace, physicians judgments regarding an
individual patient. Surgical practice can benefit from evidencebased medicine and should be incorporated into the standard of
care. Evidence-based medicine guidelines have reduced mortality
from myocardial infarctions and also improved care for persons
with diabetes and other common medical problems. An example
of evidence-based medicine in surgery is given by a systematic
review of mechanical preparation for elective colorectal surgery,
summarized in the next section.

EXAMPLE
A recent systematic review of mechanical bowel preparation for
elective colorectal surgery published by the Cochrane Collaboration22 and described next provides an example of this important
component of evidence-based medicine.

MECHANICAL BOWEL PREPARATION FOR


ELECTIVE COLORECTAL SURGERY: AN
EXAMPLE OF A SYSTEMATIC REVIEW
Background
For more than a century, surgeons have believed, dogmatically,
that efficient mechanical bowel preparation is an important factor
in preventing infectious complications and anastomotic dehiscence after colorectal surgery.23,24 Clinical experience and observational studies have shown that mechanical removal of gross
feces from the colon has been associated with decreased morbidity and mortality in patients undergoing operations of the colon.25

PMPH_CH01.indd 7

Authors were categorical that the most important factor to affect


the outcome of a colonic operation that is within the control of a
surgeon is the degree of emptiness of the bowels.26,27
An early randomized clinical trial questioned this dogma, and
concluded that vigorous mechanical bowel preparation was not necessary, stating omission of enemas and bowel washes from the preoperative procedures will be welcomed by both patients and nursing
staff.28 The authors of another meta-analysis concurred with this
point of view,29 whereas in another trial30 the authors argued that
preoperative bowel preparation is time consuming and expensive,
unpleasant to the patientseven dangerous on occasions (increased
risk for inflammatory processes)31and completely unnecessary.
Different methods of mechanical bowel preparation have been
tested and approved. The potential danger of having feces in contact with newly performed anastomosis has led to the construction
of a defunctioning stoma when the colon was not prepared.32,33
Both experimental studies34-36 and clinical trials in emergency
surgery37,38 have been published to support this theory.
In a review of the literature in 1998, Platell and Hall concluded that there was limited evidence in the literature to support
the use of mechanical bowel preparation in patients undergoing
colorectal surgery.39 Platell conducted a randomized trial comparing single phosphate enema with polyethylene glycol (PEG) before
colorectal surgery, and found the results favored the group that
received PEG for bowel preparation.40
In 2002, Fa-Si-Oen was coauthor in a study that prospectively
evaluated a consecutive series of patients who underwent resection
and primary anastomosis and he concluded that mechanical bowel
preparation is not a sina qua non for safe colorectal surgery.41
The authors of a trial that analyzed the bowel contents suggested that participants receiving mechanical bowel preparation had a tendency toward a higher incidence of spillage of
bowel contents compared with participants who did not receive
it, but without statistical significance. Spillage of bowel contents
into the peritoneal cavity may increase the rate of postoperative
complications.42
A recent case-control study that analyzed patients with rectal
cancer who submitted to elective resection with mesorectal excision concluded that elective rectal surgery for cancer without
mechanical bowel preparation may be associated with reduced
postoperative morbidity.43
Randomized trials from several countries have concluded
that the role of bowel preparation in colorectal surgery requires
reevaluation. When analyzed, the individual trial results did not
show any significant difference, and this fact motivated this systematic review of the literature. In the first version of this review
there were only five included trials.44-48 The first update included
four more studies,31,49-51 whereas the second update included five
more trials.52-56
The objectives of this review were to determine the effectiveness and risk of prophylactic mechanical bowel preparation for
morbidity and mortality rates in colorectal surgery, with anastomotic leakage as the primary outcome.
As the incidence of anastomotic leakage is increasing, the
closer to the anus the anastomosis is applied,57 bowel preparation
might have different effects in colon and rectum. Therefore, analyses were stratified for colonic and rectal surgery as well as for colorectal surgery as a whole.
The aim of this update was to determine whether mechanical bowel preparation before elective colorectal surgery is really
essential for patients.

5/21/2012 7:41:09 PM

Surgery: Evidence-Based Practice

METHODS

(8) Noninfectious extra-abdominal complication (e.g., deep


venous thrombosis, cardiac complications, wound rupture).

Types of Studies
The inclusion criteria were
(1) Randomized clinical trials comparing preoperative
mechanical bowel preparation versus no preparation (or
placebo) in
(2) Participants undergoing elective colorectal surgery and in
which
(3) The primary outcome (anastomotic leakage) was clearly stated
in both treatment arms.
All three criteria must have been met for inclusion of a trial.
The exclusion criteria were
(1) Studies evaluating two or more different cleansing methods.
(2) Studies including participants undergoing emergency
surgery.

Types of Participants
The types of participants included patients undergoing elective
colorectal surgery. Both adults and children were eligible for
inclusion.

Types of Interventions
The types of interventions included any strategy in mechanical bowel preparation compared with no mechanical bowel
preparation.

Types of Outcome Measures


Primary Outcome Measures
(1) Anastomotic leakage (defined as a discharge of feces from
the anastomosis site that externalized through the drainage
opening or the wound excision; or could be characterized
on the basis of an abscess adjacent to the site) confirmed by
clinical or radiological investigation.
The types of surgery and sites of anastomosis were
stratified into
(a) Low anterior resection with extraperitoneal anastomosis
(rectum considered to be extraperitoneal); and
(b) Colonic surgery with intraperitoneal anastomosis.
(2) Overall anastomotic leakage: total number of anastomotic
dehiscences in trials (in all of colon and rectum).

Secondary Outcome Measures


(3) Mortality: number of postoperative deaths related to the
surgery.
(4) Peritonitis: presence of postoperative infections in the
abdominal cavity, localized (abscess) or not.
(5) Reoperation: surgical reintervention for anastomotic
complication or peritonitis.
(6) Wound infection: defined as a discharge of pus from the
abdominal wound.
(7) Infection extra-abdominal complication: postoperative
infectious complication at extra-abdominal site.

PMPH_CH01.indd 8

Sensitivity and Subgroup Analyses


(9) Anastomotic leakage and wound infection in studies with
adequate randomization.
(10) Anastomotic leakage and wound infection in studies with
bowel continuity restored.

Search Methods for Identification of Studies


See: Collaborative Colorectal Cancer Review Group search strategy.58 The studies were identified from the following sources:
Cochrane Colorectal Cancer Group Specialized Register,
Cochrane Central Register of Controlled Trials. MEDLINE,
EMBASE, CINAHL, LILACS, SCISEARCH, and the Controlled
Clinical Trials Database.
Additional trials were sought by checking the reference lists
of relevant papers, writing to experts in the field, hand searching journals, and by contacting the authors of relevant papers.
Conference proceedings from major gastrointestinal conferences
(World Congress of Gastroenterology, annual meetings of the
American Society of Colon and Rectal Surgery, annual meetings
of the Association of Coloproctology of Great Britain and Ireland,
annual meetings of the European Association of Coloproctology,
and the Tripartites meetings) were scrutinized back to 1994 (the
last possible retrieval of abstract material). No language restrictions or date restrictions were applied in the searches. Searches
were performed on March 13, 2008. One unpublished study was
identified by personal contact with the authors.53

DATA COLLECTION AND ANALYSIS


Locating and Selecting Studies
The reviewers selected the trials to be included in this review independently. Disagreements about selection were resolved by consensus (via email correspondence). One unpublished study was
identified through personal contact with the authors.53

Critical Appraisal of Studies


The methodological quality of each trial was assessed by at least
two of the authors. Details of the randomization method, blinding,
whether an intention-to-treat analysis was done, and the number
of participants lost to follow-up was recorded, thus enabling the
risk of bias in individual studies to be evaluated.59 The external
validity of the studies was assessed by analysis of the characteristics of participants and the interventions as listed below. Disagreements were resolved by consensus.

Participants
Category of disease (colorectal cancer, inflammatory disease,
magacolon, polyposis, diverticular disease), sex of participants,
age, topography of the affected area, operative procedure, antibiotic therapy, and surgeon experience were evaluated for each
study. Prestudy calculation of the sample size and whether the

5/21/2012 7:41:09 PM

Evidence-Based Medicine

sample was representative were registered. Duplicate publications


of the same set of data were identified and each sample of participants was only included once.

Collecting Data
Studies where the allocation of concealment was deemed to be
adequate were included. Where the allocation procedure was
unclear, studies were included if the primary authors stated that
it was a randomized study, but these studies were excluded from a
sensitivity analysis. Data were independently extracted by at least
two of the reviewers and cross-checked. The results of each trial
were summarized in 2 2 tables for each outcome. Disagreements
were solved by consensus.

Interventions
The interventions included any type of mechanical bowel preparation (anterograde [oral] or retrograde [enemas]) versus no
mechanical preparation. Data were entered into Review Manager
4.2 by single data-entry (KFG);22 all data entries were controlled
by a second author (PWJ).58

ANALYZING AND PRESENTING RESULTS


Where appropriate, the studies were stratified for different metaanalyses according to the analysis of the defined outcomes. The
meta-analyses were performed using various techniques (Review
Manager 4.2): for dichotomous outcome measures, the combined
logarithm of the Peto odds ratio (fixed-effect model) was used as the
default. A test for statistical heterogeneity was performed in each
case. Where heterogeneity was encountered, results were reported
as odds ratio (OR) using random-effects modeling. For the analysis, only participants submitted for elective colorectal surgery were
reviewed according to type of intervention and type of participant, to
assess whether there were important differences between them. All
inclusion criteria had to be met. One unpublished study53 described
rectal surgery only. The data were included separately in the stratified analyses. In all other outcomes, the data were compiled with
those from the large Swedish study,54 in which these participants
originally had been included. The included studies were assessed
to determine whether clinical heterogeneity was present. Potential
publication bias in the results of the meta-analysis was assessed
both by inspection of graphical presentations (by means of a funnel
plot: plotting the study weight or sample size [on the Y-axis] against
the OR [on the X-axis]) and by calculating a test of heterogeneity
(standard chi-squared test on N degrees of freedom, where N equals
the number of trials contributing data minus one). The funnel plot
was possible only for an outcome described in more than five trials. Three possible reasons for heterogeneity were prespecified: (1)
responses differ according to difference in the quality of the trial; (2)
responses differ according to sample size; and (3) responses differ
according to clinical heterogeneity. When heterogeneity was discovered, sensitivity-analyses were performed in subgroups.

Sensitivity Analysis
Sensitivity analysis was performed for anastomotic leakage and
wound infection in studies with adequate randomization and in

PMPH_CH01.indd 9

studies in which bowel continuity was restored. Since the randomeffects model provides a more conservative estimate of an overall
effect, it was used as the default in these sensitivity analyses.

RESULTS
Description of Studies
Seven new studies were accessed in this update, but two of them
were excluded.40,43 This brought the total number of excluded trials to eight. Three were excluded because of the absence of a control group30,37,38; one because of an elemental diet in the control
group60; one due to lack of description of the primary outcome
and insufficient description of the secondary outcomes28; one
because it was a retrospective study61; one because the control
group received a single sodium phosphate enema40; and the last
was a case-control study.43
A total of 14 studies were included in the review. Five new
trials were included in this update. 52-56 In addition, two abstracts
that were newly included at the fi rst update have now been published as full articles. 31,49 Most of the studies were published in
English, though one was published in Portuguese,62 and another
in Spanish50; both of these were found on the LILACS database.
One study was published as abstract only,44 but data from this
study were retrieved from another publication.29 One of the
authors on a primary study gave supplementary data regarding
rectal cancer to this update of the review.54 These data are currently unpublished, but have been handled as a separate publication. Two trial authors supplied supplementary data directly to
two trial authors (PWJ or KFG).48,51 Six of the included trials were
multicentered.31,49,51-54 Only two studies mentioned the educational status of the surgeon: two of them described the operations
performed by a consultant surgeon or under supervision.45,47; two
trials described operations as having been performed chiefly, or
exclusively, by residents or senior residents.48,56 In one study, 55 all
surgery was performed by one surgeon only. Jung et al.54 mentioned that the surgeons involved in the trial specialized in colorectal surgery. The other trials did not describe the experience of
the surgeons. The Zmora study was published three times: fi rst as
an abstract,63 then as a full article, 51 and fi nally as an article analyzing only those participants who underwent left-sided colonic
anastomosis.64

Types of Participants
The inclusion criteria were the same for all studies; that is, participants admitted for elective colorectal surgery. One trial
included children.48 Some of them included participants without
anastomosis46,48; one study excluded participants with anastomosis
in only one outcomeanastomosis leakage44; two studies excluded
participants in whom bowel continuity was not restored.45,47 Seven
trials described the two allocation groups as being equal according to gender, age, types of operation, and diagnosis.31,45-49,51 One
trial did not give details.44 One called attention to the statistical
difference between the two groups with regard to age, hemoglobin
level, and serum albumin.50 Most trials specified exclusion criteria, but these varied tremendously between trials. One trial did
not provide details of any exclusion criteria,44 whereas in another
trial no participants were excluded.46

5/21/2012 7:41:09 PM

10

Surgery: Evidence-Based Practice

Types of Interventions
All studies compared no mechanical bowel preparation with a
method of mechanical preparation: PEG solution; laxatives (mineral oil, agar, and phenolphthalein); mannitol; enemas (900 mL
water containing 100 mL glycerin); sodium picosulfate 10 mg;
Bisacodyl (10 mg) and enemas; and diets: low and nonresidue.
The duration of follow-up varied between trials, and ranged from
7 days after surgery50 to 3 months.49 The methods used to cleanse
the bowel were PEG31,49,51,55 or sodium phosphate solution56;
although three multicentered trials used both.52-54 Two trials
reported an enema of saline solution before surgery for participants scheduled for an anterior resection of the rectum.31,51

Types of Outcome Measurements


Primary Outcomes
Anastomotic Leakage
Two studies stratified the anastomosis between rectal and
colonic.45,47 Stratification data were obtained through personal
contact with three trial authors.48,51,54 The other trials did not refer
to the site of the anastomosis.44,46,50,52,56 Two trials excluded anastomosis below the peritoneal line.49,55
Overall Anastomotic Leakage
All of the included trials described this outcome.

Secondary Outcomes
Secondary outcomes included mortality, peritonitis, reoperation,
wound infection, infectious extra-abdominal complications, and
noninfectious extra-abdominal complications.

RISK OF BIAS IN INCLUDED STUDIES


Selection Bias
Selection bias consists of systematic differences in the comparison
groups. In two trials the allocation procedure used randomized
cards.47,48 Fillmann,64 the author who answered our correspondence, described using a random number table for randomization.
In one study a computer generated list was used.51 In these studies
the allocation process was considered sufficient. In others the allocation process was not clearly specified,44,45,50 and thus considered
unclear, which led to the planned sensitivity analysis. In general,
allocation concealment was not sufficiently described in the earlier
trials, but in this update, four of the trials52-55 had adequate allocation concealment (graded A), and the process of randomization
was well defined. In one of the newer trials,56 however, the allocation concealment was considered inadequate (grade C) because the
authors used the identification number of the participants (even
or odd) to determine randomization. Two of the conference proceedings (now published as full articles) explained their methods
of randomization and these were considered adequate.31,49

Performance Bias
Performance bias consists of systematic differences in care provided, apart from the intervention being evaluated. All trials used

PMPH_CH01.indd 10

prophylactic antibiotics, and all but one described the two allocation groups as being equal according to gender, age, types of operation, and diagnosis.44 One trial indicated a difference between the
allocation groups by age, hemoglobin level, and serum albumin.50
The new studies included in this update reported that the groups
were well matched with regard to age, sex, and diagnosis54-56;
though, according to the randomization, in one trial there were
more participants who smoked and had inflammatory bowel disease in the mechanical bowel preparation group.52 This trial also
included participants with preoperative radiation therapy, whereas
two trials excluded participants who had this treatment49,55; the
others did not mention this point.

Blinding
One trial that was described as a double-blind study used orange
juice as a placebo.46 Another study was described as being singleblinded, because the surgeons were aware whether bowel preparation had taken place or not.45 The trial of Pena-Soria55 was
described as single-blinded because the participants were followed by an independent observer. Two publications by Jung
et al.53,54 reported that the participating surgeons did not know the
size of the blocks of permutations, and the hospital charts of participants were reviewed by a statistician and a surgeon who were
not involved in conducting the study. One of the trials reported
that observers were not blinded, but this fact probably did not
cause bias.52 Investigators assessing the endpoints were blinded to
the use of mechanical bowel preparation in one of the studies.31
Information about blinding was not provided in the other trials.

Attrition Bias
Attrition bias occurs when there are systematic differences in
withdrawals from a trial. In the original review, the Brownson44
and Tabusso50 trials did not describe withdrawals or dropouts.
The Burke trial45 had a 9.1% withdrawal rate (17/186 participants),
but no dropouts; the Santos trial48 had a 5% withdrawal rate (8/157
participants), but no dropouts; whereas the Zmora trial51 had a
8.6% withdrawal rate (35/415 participants). In two of the trials all
participants completed the study (Fillmann, personal correspondence, 1995).47 In three of the new trials included in this update,
all participants finished the study as per protocol31,49,56; however,
77 participants were excluded from the Contant trial52; and, in the
Jung study,54 21 participants were excluded before randomization,
128 did not receive the intervention, 13 were lost to follow-up, and
for 13 the data were not submitted. The Pena-Soria study55 had 2
exclusions preoperatively, and 11 at the point of surgery.

Detection Bias
Detection bias occurs when there are systematic differences in
assessment of outcomes. In the first half of the Burke study,45 the
incidence of anastomotic leaks was established by means of watersoluble contrast enemas in all participants, but in the second half
of the study enemas were used only on clinical suspicion of leakage. This was because two of the six leaks identified on the seventh day after surgery occurred immediately after administration
of the routine water-soluble contrast enema. Contrast radiography
was used to confirm clinical suspicion of anastomotic leaks in
seven trials.31,45-48,52,56 In one trial anastomotic leakage was divided

5/21/2012 7:41:09 PM

Evidence-Based Medicine

into major leakagethat is, clinically significant and leading to a


relaparotomyand minor leakagethat is, subclinical, which was
verified by radiographic examination and treated conservatively.49
Pena-Soria et al.55 used CT scan with contrast or laparotomy
major dehiscenceto confirm the clinical suspicion of leakage.

11

Of the 4776 participants in the 14 included RCTs, 2398 were allocated for mechanical bowel preparation (group A), and 2378 for
no preparation (group B), before elective colorectal surgery.

identified all known trials that compared any kind of mechanical


bowel preparation with no preparation in patients receiving elective colorectal surgery. Five new trials were included in this second update of the review, bringing the total number of included
trials to 14 (4821 participants). Analysis of these 14 trials showed
no statistically significant differences in how well the two groups
of patients (mechanical bowel preparation group and the no preparation group) did after surgery in terms of leakage at the surgical
joint of the bowel, mortality rates, peritonitis, need for reoperation, wound infection, and other nonabdominal complications.
Consequently, there was no evidence that mechanical bowel preparation improves the outcome for patients. Further research on
mechanical bowel preparation versus no preparation in patients
submitted for elective colorectal surgery is warranted.

Primary Outcomes

CONCLUSION

EFFECTS OF INTERVENTIONS
Preparation versus Nonpreparation

Anastomotic Leakage (Stratified)


(1) Low Anterior Resection
Ten percent of participants in group A (14 out of 139) who had low
anterior resections suffered anastomotic leakage, compared with
6.6% in group B (9 out of 136); the Peto OR was 1.73 (95% confidence interval [CI]: 0.734.10 [nonsignificant]), and there was no
statistical heterogeneity.45,47,48,51,53
(2) Colonic Surgery
In group A, 2.9% of participants (32 out of 1226) who had colonic
surgery suffered anastomotic leakage, compared with 2.5% (31 out
of 1228) in group B; the Peto OR was 1.13 (95% CI: 0.691.85 [nonsignificant]), and there was no statistical heterogeneity.45,47-49,51,54,55

Overall Anastomotic Leakage


Overall anastomotic leakage was 4.2% in group A (102 out of 2398
participants) compared with 3.4% (82 of 2378 participants) in group
B; the Peto OR was 1.26 (95% CI: 0.9411.69 [nonsignificant]). The
chi-square test for heterogeneity gave the following results: chisquare = 16.94, df = 12 (P value = 0.15), I2 = 29.2%.31,44-56

Secondary Outcomes
No significant differences between group A and group B were
found with regard to mortality, peritonitis, reoperation, wound
infection, infectious extra-abdominal complications, and noninfectious extra-abdominal complications.

Cochrane defined evidence-based medicine as the conscientious,


explicit, and judicious use of current best evidence in making decisions about the care of individual patients. The practice of evidencebased medicine means integrating individual clinical expertise with
the best available external clinical evidence from systemic research.
Since 1990, evidence-based medicine has developed as a topic
of courses, and text books. Relevant to the conduct of evidencebased medicine, resources including search engines, electronic
journals, and systematic reviews are now widely available. Readers should review the Cochrane Handbook for Systematic Reviews
(http://cochrane-handbook.org/), especially Part 2, Chapter 8,
for a detailed summary of sources of bias and bias assessments. A
sophisticated site for current perspectives on evidence-based medicine is the British National Institute for Health and Clinical Excellence (NICE); the NICE website (http://www.nice.org.uk/) has links
to many resources, guidance, and documents.
In conclusion, we endorse the two basic principles stated by
Guyatt et al.8: (1) evidence-based medicine posits a hierarchy of
evidence as a guide to clinical decision making; and (2) evidence
alone is never sufficient to make a clinical decision. In contrast
to the traditional paradigm of medical practice, evidence-based
medicine places a low value on unsystematic clinical experience
and pathophysiologic rationale, and suggests that interpreting results of clinical research requires a formal set of rules and
places a low value on authority than the traditional paradigm. We
encourage the reader to access the texts and resources mentioned
and to explore this topic more fully.

REFERENCES
SUMMARY
Until recently it was thought that vigorous preoperative mechanical cleansing of the bowel (mechanical bowel preparation),
together with the use of oral antibiotics, reduced the risk of septic complications after nonemergency (elective) colorectal operations. Mechanical bowel preparation was performed routinely
prior to colorectal surgery until 1972, when this procedure started
to be questioned. Well-designed clinical trials were published,
and their results caused colorectal surgeons to doubt this traditional belief. Preoperative bowel preparation is time consuming
and expensive, unpleasant to the patients, and even dangerous on
occasion (increased risk for inflammatory processes). This review

PMPH_CH01.indd 11

1. Sackett DL, Rosenberg WMC, Gray JAM, Haynes RB, Richardson WS. Evidence based medicine: what it is and what it isnt.
BMJ. 1996;312:71-72.
2. Woolf SH, George JN. Evidence based medicine. Interpreting studies and setting policy. Hematol Oncol Clin North Am.
2000;14(4):761-784.
3. Brater DC, Daly WJ. Clinical pharmacology in the Middle Ages:
principles that presage the 21st century. Clin Pharmacol Ther.
2000;67(5):447-450.
4. Daly WJ, Brater DC. Medieval contributions to the search for
truth in clinical medicine. Perspect Biol Med. 2000;43(4):530-540.
5. Cochrane AL. Effectiveness and Efficacy: Random Reflections on
Health Services. London: Royal Society Medical Press, 1972.

5/21/2012 7:41:10 PM

12

Surgery: Evidence-Based Practice

6. Guyatt G. Gordan Guyatt. McMaster University Faculty Profi les


1990.
7. Evidence-Based Medicine Working Group (November 1992).
Evidence based medicine. A new approach to teaching the practice of medicine. JAMA. 1992;268(17):2420-2425.
8. Guyatt G, Rennie D, Meade, MO, Cook DJ. Users Guides to the
Medical Literature. A Manual for Evidence-Based Clinical Practice. 2nd ed. New York: McGraw Hill Medical, 2008b.
9. Smith R, Gray M. Obituary Anna Donald. BMJ. 2009;338:b436.
10. Guyatt G, Oxman AD, Vist GE, et al. GRADE: an emerging consensus on rating quality of evidence and strength of recommendations. BMJ. 2008a;26:924-926.
11. Guyatt G, Oxman AD, Schnemann HJ, Tugwell P, Knotterus A.
GRADE guidelines: A new series of articles in the J Clin Epidemiol. 2010;64:383-394.
12. Sim I, Carini S, Tu S, et al. The Human Studies Database Project: Federating Human Studies Design Data Using the Ontology
of Clinical Research. 2010 AMIA Clinical Research Informatics
Summit.
13. Carini S, Pollock BH, Lehmann HP, et al. Development and Evaluation of a Study Design Typology for Human Research. Proceedings of the 2009 AMIA Annual Symposium, 2009.
14. Cook DJ, Sackett DL, Spitzer WO. Methodologic guidelines for
systematic reviews od randomized control trials in health care
from the Potsam Consultation on Meta-Analysis. J Clin Epidemiol. 1995;48:167-171.
15. Cornell J, Mulrow C. Meta-analysis. In: Adr HJ and Mellenbergh GJ, eds. Research Methodology in the Social, Behavioral,
and Life Sciences. London: SAGE Publications; 1999.
16. Egger M, Smith GD, Altman DG, eds. Systematic Reviews in
Health Care. Meta-analysis in Context. 2nd ed. London: BMJ
Publishing Group; 2001.
17. Borenstein M, Hedges LV, Higgins JPT, Rothstein HR. Introduction to Meta-Analysis. West Sussex: Wiley; 2009.
18. Dickersin K, Min Y-I, Meinert CL. Factors influencing publication of research results. JAMA. 1992;267:374-378.
19. Stern JM, Simes RJ. Publication bias: evidence of delayed publication in a cohort study of clinical research projects. BMJ.
1997;315:640-645.
20. Egger M, Smith GD, Schneider M, Minder C. Bias in metaanalysis detected by a simple, graphical test. BMJ. 1997;315:
629-634.
21. Begg CB, Mazumdar M. Operating characteristics of a rank
correlation test for publication bias. Biometrics. 1994;50:
1088-1101.
22. Guenaga KFG, Matos D, Wille-Jorgensen P. Mechanical Bowel
Preparation for Elective Colorectal Surgery (Review). The
Cochrane Collaboration. New York: Wiley; 2009.
23. Halsted WS. Circular suture of the intestine: an experimental
study. Am J Med Sci.1887;94:436-461.
24. Thornton FJ, Barbul A. Anastomotic healing in gastrointestinal
surgery. Surg Clin North Am. 1997;3:549-573.
25. Nichols RL, Condon RE. Preoperative preparation of the colon.
Surg Gynecol Obstet. 1971;2:323-337.
26. Chung RS, Gurll NJ, Berglund EM. A controlled trial of whole
gut lavage as a method of bowel preparation for colonic operations. Am J Surg. 1979;137:75-81.
27. Clarke JS, Condon RE, Bartlett JG, Gorbach SL, Nichols RL,
Ochi S. Preoperative oral antibiotics reduce septic complications
of colon operations: results of prospective, randomized, doubleblind clinical study. Ann Surg. 1977;186(3):251-259.
28. Hughes ESR. Asepsis in large-bowel surgery. Ann R Coll Surg
Eng. 1972;51:347-356.

PMPH_CH01.indd 12

29. Slim K, Vicaut E, Panis Y, Chipponi J. Meta-analysis of randomized clinical trials of colorectal surgery with or without mechanical bowel preparation. Br J Surg. 2004;91:1125-1130.
30. Irving AD, Scrimgeour D. Mechanical bowel preparation for
colonic resection and anastomosis. Br J Surg. 1987;74:580-581.
31. Bucher P, Gervaz P, Soravia C, Mermiollod B, Ern M, Morel P.
Randomized clinical trial of mechanical bowel preparation versus no preparation before elective left-sided colorectal surgery.
Br J Surg. 2005;92:409-414.
32. Grabham JA, Moran BJ, Lane RHS. Defunctioning colostomy for low anterior resection: a selective approach. Br J Surg.
1995;82:1331-1332.
33. Mealy K, Burke P, Hyland J. Anterior resection without a defunctioning colostomy: questions of safety. Br J Surg. 1992;79:305-307.
34. ODwyer PJ, Conway W, McDermott EWM, OHiggins NJ. Effect
of mechanical bowel preparation on anastomotic integrity following low anterior resection in dogs. Br J Surg. 1989;76:756-758.
35. Schein M, Assalia A, Eldar S, Wittmann DH. Is mechanical
bowel preparation necessary before primary colonic anastomosis? Dis Colon Rectum. 1995;38:749-754.
36. Smith SRG, Connolly JC, Gilmore OJA. The effect of faecal loading on colonic anastomotic healing. Br J Surg. 1983;70:49-50.
37. Dorudi S, Wilson NM, Heddle RM. [Primary restorative colectomy in malignant left-sided large bowel obstruction]. Ann R
Coll Surg Eng. 1990;72:393-395.
38. Duthie GS, Foster ME, Price-Thomas JM, Leaper DJ. Bowel preparation or not for elective colorectal surgery. J R Coll Surg Edinb.
1990;35:169-171.
39. Platell C, Hall J. The role of mechanical bowel preparation in
patients undergoing colorectal surgery?. Dis Colon Rectum.
1998;41:875-883.
40. Platell C, Barwood N, Makin G. Randomized clinical trial of bowel
preparation with a single phosphate enema or polyethylene glycol
before elective colorectal surgery. Br J Surg. 2006;93:427-433.
41. van Geldere D, Fa-Si-Oen P, Noach LA, Rietra PJ, Peterse JL, Boom
RP. Complications after colorectal surgery without mechanical
bowel preparation. J Am Coll Surg. 2002;194(1):40-47.
42. Mahajna A, Krausz M, Rosin D, et al. Bowel preparation is associated with spillage of bowel contents in colorectal surgery. Dis
Colon Rectum. 2005;48:1626-1631.
43. Bretagnol F, Alves A, Ricci A, Valleur P, Panis Y. Rectal cancer surgery without mechanical bowel preparation. Br J Surg.
2007;94:1266-1271.
44. Brownson P, Jenkins AS, Nott D, et al. Mechanical bowel preparation before colorectal surgery: results of a prospective randomized trial. Br J Surg. 1992;79:461-462.
45. Burke P, Mealy K, Gillen P, Joyce W, Traynor O, Hyland J.
[Requirement for bowel preparation in colorectal surgery]. Br J
Surg. 1994;81:907-910.
46. Fillmann EEP, Fillmann HS, Fillmann LS. Elective colorectal
surgery without preparation. Revista Brasileira de Coloproctologia. Cidade Editora Cientfica Ltda. 1995;15(2):70-71.
47. Miettinen RPJ, Laitinen ST, Makela JT, Paakkonen ME. Bowel
preparation with oral polyethylene glycol electrolyte solution
vs. no preparation in elective open colorectal surgery. Dis Colon
Rectum. 2000;43(5):669-677.
48. Santos JCM Jr, Batista J, Sirimarco MT, Guimares AS, Levy
CE. Prospective randomized trial of mechanical bowel preparation in patients undergoing elective colorectal surgery. Br J Surg.
1994;81:1673-1676.
49. Fa-Si-Oen P, Roumen R, Buitenweg JA, et al. Mechanical bowel
preparation or not? Outcome of a multicenter, randomized trial in
elective open colon surgery. Dis Colon Rectum. 2005;48:1509-1516.

5/21/2012 7:41:10 PM

Evidence-Based Medicine

50. Tabusso FY, Zapata JC, Espinoza FB, Meza EP, Figueroa ER.
Mechanical preparation in elective colorectal surgery, a useful
practice or need? [Preparacin mcanica et cirga electiva colorectal, costumbre o necesidad]. Revista de Gastroenterologia del
Peru. 2002;22(2):152-158.
51. Zmora O, Mahajna A, Bar-Zakai B, et al. Colon and rectal surgery without mechanical bowel preparation. A randomized prospective trial. Ann Surg. 2003;237:363-367.
52. Contant CME, Hop WCJ, Vant Sant HP, et al. Mechanical bowel
preparation for elective colorectal surgery: a multicentre randomised trial. Lancet 2007;370:2112-2117.
53. Jung, B. Mechanical bowel preparation for rectal surgery. Personal communication December 2006.
54. Jung B, Pahlman L, Nystrm PO, Nilsson E for the Mechanical
Bowel Preparation Study Group. Multicentre randomized clinical trial of mechanical bowel preparation in elective colonic surgery. Br J Surg. 2007;94:689-695.
55. Pena-Soria MJ, Mayol JM, Anula-Fernandez R, Arbeo-Escolar A,
Ferrnandez-Represa JA. Mechanical bowel preparation for elective
colorectal surgery with primary intraperitoneal anastomosis by a
single surgeon: interim analysis of a prospective single-blinded
randomized trial. J Gastrointestin Surg. 2007;11:562-567.
56. Ram E, Sherman Y, Weil R, Vishne T, Kravarusic D, Dreznik Z.
Is mechanical bowel preparation mandatory for elective colon
surgery. Arch Surg. 2005;140:285-288.
57. Goligher JC, Graham NG, De Dombal FT. Anastomotic dehiscence after anterior resection of rectum and sigmoid. Br J Surg.
1970;57(2):109-118.

PMPH_CH01.indd 13

13

58. Wille-Jrgensen P, Kronborg O, Simon N, Munro A, McLeod R,


Nelson R, eds. Colorectal Cancer Groups Module of the Cochrane
Database of Systematic Reviews. The Cochrane Library [database
on disk and CD-ROM]. Oxford: Update Soft ware, 1999, Issue 4.
59. Higgins JPT, Green S, eds. Cochrane Handbook for Systematic
Reviews of Interventions Version 5.0.0 [updated February 2008].
Available from www.cochrane-handbook.org. Chichester: Wiley
& Blackwell Publishers, 2008.
60. Matheson DM, Arabi Y, Baxter-Smith D, Alexander-Williams
J, Keighley MRB. Randomized multicentre trial of oral bowel
preparation and microbials for elective colorectal operations. Br
J Surg. 1978;65(9):597-600.
61. Memon MA, Devine J, Freeney J, From SG. Is mechanical bowel
preparation really necessary for elective left sided colon and rectal surgery?. Int J Colorectal Dis. 1997;12:298-302.
62. Fillmann HS, Fillmann LS. Ciurugia colorectal eletiva sem preparo [Elective Colorectal Surgery Without Preparation]. So Paulo,
1995.
63. Zmora O, Mahajna A, Bar-Zakai B, et al. Left-sided anastomosis
without mechanical bowel preparation: a randomized, prospective trial. Dis Colon Rectum 2002;45(4):A7-A8.
64. Zmora O, Mahajna A, Bar-Zakai B, et al. Is mechanical bowel
preparation mandatory for leftsided colonic anastomosis? Results
of a prospective randomized trial. Techniques in Coloproctology
2006;10:131-135.
65. US Preventive Services Task Force. Guide to Clinical Preventive
Services: Report of the US Preventive Services Task Force. Virginia: International Medical Publishing; 1989.

5/21/2012 7:41:10 PM

CHAPTER 2

Patient Safety in Surgical Care


Kenneth Stahl and Susan E. Brien

No person is so perfect in knowledge and experiences that


error in opinion or action is impossible. In the art of surgery
error is more likely to occur than in almost any other line of
human endeavor, and it is in this field that it should be most
carefully guarded against, since lack of surgical safeguards
may result in a serious handicap for the rest of the life of the
patient or may even result in the sacrifice of that life.

1. Where and how do errors occur during the care of surgical


patients?
The reasons for these errors and adverse events are numerous. The
larger incidence of errors in surgery is explained in part by the
higher number of treatment events surgical patients undergo during an operation compared to hospitalized medical patients and the
increased complexity of the tasks surgeons perform. The complexity of the surgical procedure has been shown to offer more opportunities for human error to occur in surgical patients than single
event care in nonsurgical medical management.10 The incidence of
errors increases with the complexity of the surgical procedure as
documented in a study of errors in neurosurgical procedures. Out
of 1108 elective neurosurgical procedures reported over a 6-year
period, 87% of patients incurred at least one error, 23% of the errors
were major, and 79% were deemed preventable.11
In a broad sense, errors in the care of surgical patients occur
on both a system level and an individual level. One of the particular
system issues affecting whether or not a surgical patient is at risk
of an adverse event is the type of facility where care is delivered.
Although teaching-affiliated hospitals have been shown to have
lower mortality rates and shorter length of stays, this is not the case
for surgical morbidity. Supported by risk adjustment data, two studies in the Veterans Health Administration care systems found that
university affiliation hospitals were associated with higher surgical
morbidity.12 The reasons for this are variable and include frequent
turnover of key personnel in the care of surgical patients that introduce errors discussed in subsequent sections of this chapter.
A useful construct of the individual human components of
error in surgery is based on the Rasmussen Skill, Knowledge & Rule
Error Model.13 The skill level refers to actions that are automatic
and are carried out based on stored patterns of preprogrammed
sets of actions. These are the skills taught to surgical residents and
can be acquired with repetitive training and practice. Failure to
execute these skills correctly leads to procedural complications.
The rule level references tasks that are completed using stored
sets of conventions. These rules consist of familiar, rehearsed algorithms such as steps of a surgical procedure or advanced trauma

Max Thorek introduced his 1932 book Surgical Errors and


Safeguards1 with these sentences, and the words are as true today
as they were when written over three quarters of a century ago.
The unique risks faced by patients who undergo surgery is supported by published data from the United States and Canada that
document that approximately half of all errors leading to adverse
outcomes in hospitalized patients are associated with surgical
procedures.2,3 This is further supported by medical legal statistics,
which shows that nearly 50% of successful malpractice claims
were against surgeons in the United States in 20014 and, over the
course of that year, led to a staggering $5.5 billion in payouts.5
The exact number of patients who suffer adverse outcomes
as a result of individual or system errors is difficult to determine.
Most studies of errors document an incidence of between 3.5%
and 10% of hospitalized patients.6,7 Healthcare Grades8 in their
2008 review of hospital error cites a 5.5% incidence of errors in
surgical patients of which 29.1% resulted in fatalities. These statistics indicate that there is an approximate fatality rate of 1% in the
group of all hospitalized surgical patients due to avoidable events.
Another important category of error is shown in the most recent
statistics from the Joint Commission9 that indicates that 13.4% of
sentinel events were related to wrong site surgery, which is a preventable adverse outcome.
Assuring the safe and error-free care of patients undergoing surgery adds to the already significant responsibilities of surgeons. There is a need for surgeons to understand the mechanisms
of error and the risks that patients face while undergoing surgical
procedures, and as Dr. Thorek penned so long ago, most carefully
guard against and avoid them in daily practice.
14

PMPH_CH02.indd 14

5/21/2012 7:46:12 PM

Patient Safety in Surgical Care

life support (ATLS)14 management of injured patients. Failure to


execute these rule patterns correctly leads to both procedural and
cognitive errors. Tasks that use rule-based cognitive mechanisms
require a greater degree of thought than skill-based tasks, as the
rules that need to be applied to complete the clinical task must be
evaluated for appropriateness and then correctly selected. Failure
to carry out any of these steps introduces errors in this process.
On a knowledge level, unfamiliar tasks are performed with a
high degree of conscious thought. In this case, the surgeon attempts
to devise a novel solution or find a workable method from a similar
scenario that could be used in a situation that has not previously
been encountered. Failures of knowledge base impact care of surgical patients on all levels and a specifically critical transition period
in the operating room has been identified. The pause phenomena
during surgery is a time when an expert surgeon transitions from
an automatic mode when all is going well, to a more effortful attentive mode when preparing to manage a unique surgical issue. This
has been well studied and is a critical phase of surgical care since
error-free surgical outcomes depend on this transition. Surgeons
need to recognize this as a period of heightened risk and surgical
teams need to perceive these circumstances accurately as extraneous conversations and potential distractions must be eliminated to
achieve successful management of the potential crisis.15
The wealth of todays technological and medical advances,
added to an increasing acuity of patient illnesses make surgical
care a high-risk and labor intensive profession with a significant
degree of complexity. Because of this complexity, adverse events
and errors are intricate matters involving many levels of the system
all of which are intertwined in a complicated organization. System
errors are best viewed with the now famous Reason Swiss Cheese
analogy.16 This model postulates that there are multiple safety
shields against error as well as opportunities for multiple levels
of possible error within the system. This ranges from organization,
supervisory, and preconditions for error that represent either failures of the broad system to catch potential error or actual fostering
of error by setting up conditions where error is likely to occur.
In this model, the individual practitioner occupies a critical position in the error chain not only as the last opportunity to
catch and prevent a system-based error that has bypassed previous
shields but also as a possible source of individual- or skill-based
errors. Because of this unique dual role, the individual final barrier
represents two critical issues in patient care errors. By definition,
any error that passes through holes in the system to the final safety
shield of the individual represents a failure of the system to have
blocked these potential mistakes. Any error that passes past the
individual as the last safety shield is a failure of that individual to
have recognized and stopped the error. Secondarily, any event that
originates within the final safety shield results from procedural,
cognitive, or decision-making factors by the individual. It is very
important to understand that there are no further barriers beyond
which it is possible to catch these individual errors. As we will talk
about later in this chapter, teamwork, communication, and an
awareness of sources of error can strengthen this final safety shield
and support and enhance individual performance in surgery.
This detailed model tells us that safe and error-free delivery of
surgical care is, to a large extent, dependent on detailed internal systems working smoothly and efficiently together. Systems with this
level of complexity require sophisticated design elements to prevent
the multiplication of human error or generation of errors within
themselves.17 A breakdown in system function at any of these levels
at best fails to catch errors and at worst is the cause of these errors.

PMPH_CH02.indd 15

15

Surgery is a dynamic specialty with a multitude of possible mishaps awaiting patients. Error-free outcomes and surgical competency are a combination of knowledge of anatomy and
pathophysiology, technical skills, team performance, communication, and decision making.18 The subsequent chapters of this book
deal with management of specific surgical diseases, and the focus of
this section is not to duplicate that information but to break down
the various phases of delivery of this surgical care and reinforce
the principle that the opportunity for error is introduced at each
of these steps. Overall management of surgical patients involves
the following:
1.
2.
3.
4.
5.
6.
7.

Initial assessment of the surgical patient


Diagnosis of surgical disease and patient selection
Timing of surgical intervention
Immediate presurgical care
Surgical procedures
Postsurgical care
Discharge

A recent review of 2015 adverse events by Krizek and coworkers19


identified major categories of surgical errors that included 164
diagnostic errors (7.5%) of which 24 (5.2%) were judged to be serious. Another 230 (10.5%) errors occurred during surgery (17.9%
serious), 693 (29.3%) during monitoring and daily care of which
19 (17.1%) were judged to be serious. Each of these broad phases of
surgical care will be examined in the following sections to understand sources and risks of error.

PHASE 1: INITIAL ASSESSMENT


OF THE SURGICAL PATIENT
This is one of the most critical aspects of the care surgeons render to patients since formulating the correct assessment of surgical
disease impacts all further management. It is also one of the most
complicated as occasionally surgeons are faced with a variety of
symptoms from an illness that can appear to be similar to other
surgical or medical diseases. Understanding how surgeons process
complex clinical information and arrive at critical decisions, especially under conditions of time constraint, fatigue, and stress, are
crucial to making the right decisions when there is no room for
error.20 Faulty decision making is a common source of error on the
basis of the aviation accident data that indicate that 47% to 80% of
accidents are caused by these types of errors.21 Decision-making
skills are among the most important nontechnical skills surgical
trainees need to learn and surgeons need to master. It is not excessive to suggest that decision making is nearly synonymous with
thinking.22
Training methods to sharpen decision-making skills are
included along with the basic sciences of disease that make up
the curriculum taught to medical students and surgical residents. Teaching decision-making skills with simulations, repetitive task rehearsals, and didactic lessons have been shown to
enhance outcomes in the emergency room 23 and anesthetic care
in the operating room.24 Misjudging the acuity of a patients
illness, misinterpreting laboratory or radiographic studies,
or simply imprecise communication of these data can lead to
errors during the initial assessment of surgical patients. Despite
a sophisticated diagnostic armamentarium, the initial assessment remains one of clinical skill, acumen, and experience of
the surgeon.

5/21/2012 7:46:12 PM

16

Surgery: Evidence-Based Practice

PHASE 2: DIAGNOSIS OF SURGICAL


DISEASE AND PATIENT SELECTION
In a meta-analysis review of 53 literature citations of autopsy data,
Shojania and colleagues25 found a major error rate from 8.4% to
24.4% of class I errors (likely cause of death) that occurred in
4.1% to 6.7% of patients. When surgical patients are reviewed as
an independent group, the diagnostic error rate ranges from 26%
to 47.2% and class I error rate from 5.5% to 11%. Extending this
review to deaths within 30 days of surgery, the incorrect assessment or misdiagnosis rate was between 42.9% and 56.6% with
20.7% judged by autopsy to be the cause of the patients death.
Errors in diagnosis and triage for surgical care can occur from
lack of knowledge or unfamiliarity with the clinical presentation
of the disease, errors in communication of data, conflicting information, or misinterpretation of existing information.
Recent cognitive psychology literature details a dual model
that is useful to understand some of the thought processes that
surgeons depend on to formulate diagnostic decisions. Th is dual
process model describes two modes of decision making and judgment that depend on pattern recognition. When the initial visual
pattern of a combination of signs and symptoms are recognized,
the first of the two processes is engaged. This system is intuitive
and characterized by reflexive, fast, and effortless reasoning. The
experienced clinician requires few resources and uses pattern
recognition for diagnostic decision making most frequently. If
the clinical pattern is not recognized, an alternative decisionmaking skill is required that is analytical and characterized by
more effort, deliberation, and slower reasoning.26 This second reasoning process is dependent on a rich knowledge base to arrive at
error-free diagnostic decisions. Diagnostic errors are introduced
when the first mode of decision making is used if there is a failure to quickly recognize and construct an accurate diagnoses
based on rehearsed algorithms. If a failure in the second mode of
thought process occurs and the surgeon does not possess or seek
adequate knowledge to make the correct diagnosis, a diagnostic
error is likely to occur.
Another source of error in diagnosis can occur when the surgeon overrides his or her knowledge base with an incorrect intuition or impulsively rushes to a conclusion. This can occur even
when the surgeon has the appropriate knowledge but fails to apply
it correctly. Clinical decision rules and diagnostic algorithms will
usefully aid in the surgeons judgment but this can be defeated as
there is frequently pushback on the part of some clinicians to be
guided by standardized sets of rules. To avoid these types of diagnostic errors, surgeons and surgical residents should be encouraged
to supplement their reliance on memory with decision-support
tools, checklists, and diagnostic algorithms to help focus on the
diagnostic process.27

PHASE 3 AND PHASE 4: TIMING OF


SURGERY AND IMMEDIATE
PRESURGICAL CARE
The next phase of surgical care involves selection criteria for surgical or nonsurgical management and timing the procedure to gain
maximum beneficial outcomes. This involves exchanging and
processing information from many sources. Failure to accurately
communicate this information and weigh it thoughtfully and
logically can result in suboptimal timing and patient selection for

PMPH_CH02.indd 16

surgery with poor outcomes. The judgment to bring a patient to


the operating room, and when that is best done, involves all the
clinical acumen and experience of the surgeon. There is often a
fine balance between patients who are too sick to tolerate surgery
and those who are too sick not to bring to the operating room.
These decisions are based on a careful deliberative process similar to that outlined above that includes a complete but sometimes
subtle assessment of the overall condition of the patient. Faulty
risk perception is a factor that can contribute to adverse surgical
outcomes.28 The surgeon must guard against a failure to weigh the
risks a patient faces from their underlying illness and concurrent
diseases such as cardiac or renal dysfunction in the operative plan.
This can be compounded by a rarity of occurrence of certain surgical diseases that require specialized care. Studies of emergency and
trauma surgery show that these types of errors are frequent causes
of death in patients with otherwise survivable injuries.29
Preparation of the patient by the surgeon for the surgical procedure is an essential part of patient care that is required to achieve
error-free outcomes. This includes preparation of the surgical team,
which is dealt in detail later in this chapter. The surgeon must be
aware preoperatively that each intervention introduces another
opportunity for error and plan for these contingencies. The potential sources of error include failing to recognize or correct metabolic derangements and lab abnormalities, assuring preoperative
antibiotic and DVT prophylaxis use and appropriate site and side
markings, as well as failing to carry out a preoperative time out.30
Various models from industry and high-reliability organizations
have been applied to this complex process with significant success.
Timely and appropriate use of preoperative antibiotics, assuring
adequate blood products are available, and preprocedure briefing
with the operative team are all elements of this process that have
been shown to enhance surgical outcomes.31

PHASE 5: SURGICAL PROCEDURES


Excellent surgical outcomes depend on excellent surgical procedures with respect to both cognitive and technical aspects of the
operation. The surgical procedure itself has been the subject of
numerous safety analyses and publications over the last decade.
The nature of this entire text is designed around principles to
avoid errors in the surgical procedure and beyond the scope of
any one chapter.
The system of delivery of surgical care is an area of prime
importance to the surgeon who must assure system safety before
the patient is even brought to the pre-op holding area. This is
another surgical responsibility but the surgeon has direct control
over only a limited number of factors that impact outcomes. The
surgeon as well as surgical team, which includes the intraoperative
personnel and support staffs all the way up to the administrators,
are jointly responsible for creating a culture of safety.32 The environment of the operating room is a critical component in ensuring
that surgical care is delivered safely. It is particularly important to
minimize distractions and promote accurate communication of
critical patient information.33 Teamwork in the operating room is
an essential element of these patient safety efforts.34 On an individual level, meticulous attention to detail, a quiet and interruption
free operating room environment, and training and preparation
for every procedure are necessary for excellent postoperative
outcomes. Planning ahead and use of good situational awareness
and crew resource management skills are equally important and
is discussed subsequently in this chapter.35

5/21/2012 7:46:13 PM

Patient Safety in Surgical Care

PHASE 6: POSTSURGICAL CARE


Optimum patient outcomes depend on attention to detail during
the postsurgical phase of surgical care. Numerous publications29
have analyzed the postoperative period of surgical care and identified that approximately 50% of errors occur in this phase of care of
surgical patients.36 In a recent study of 1047 postoperative patients
who were admitted to three study units, one or more adverse
events occurred in 480 patients (45.8%). A total of 2183 errors were
identified, of which 462 (21.2%) were considered serious (potentially life- or limb-threatening event). One hundred seventy-five
patients (17.7%) had at least one serious event.20 These publications
have also outlined that error patterns are not uniquely different
from preoperative patterns of error with similar contributing factors and similar requirements to minimize these adverse events.
Several potential sources of errors in the immediate postoperative period have been identified and some involve handoff of care
from the operating room team to the postsurgical care team. Studies have found that the odds of adverse events could decrease from
OR 5.2 to OR 1.5 with standardization of communication and handoffs.37 Other postsurgical errors can occur if there is a failure to
renew preoperative medications or failure to provide antibiotic and
DVT prophylaxis. Care of patients who are transferred to the regular
nursing floor or the intensive care unit (ICU) also requires special
attention. Many other postsurgical complications are avoidable and
involve medication errors, falls, respiratory decompensation, and
decubitus ulcers. Surgeons must be aware of these potential issues
and be on guard to prevent those problems that can be avoided.38

PHASE 7: DISCHARGE
Timely and thoughtful discharge of postoperative patients is
another aspect of surgical care that requires careful planning to
avoid potential error. Specific surgical readmission rates have been
quoted to be 11.1% to 20.3% of discharges.39 Readmission rates
after discharge have been used by the Joint Commission and other
accrediting agencies as a metric for assessing the quality of patient
care. Readmission is associated with indicators of standards of care
during the index hospitalization, such as poor resolution of the
main problem, unstable therapy at discharge, and inadequate postdischarge planning. It has been reported that from 9% to 48% of
all readmissions have been judged to be preventable. Randomized
prospective trials have shown that 12% to 75% of all readmissions
can be prevented by patient education, predischarge assessment,
and domiciliary home care.40 Minimizing errors in the surgical
patient depends on timely and correct discharge procedures.
Answer: Errors can occur in all seven phases of surgical management. These types of errors include decision errors, knowledge
errors, and procedural errors.
2. Are there unique patterns of errors that occur during surgical care?
Adverse events and errors in surgical care follow patterns that can
be understood by a careful analysis of the multiple factors and
conditions that contribute to these incidents. For this reason, to
minimize the risk of error it is necessary to study where and how
errors may occur and anticipate these conditions. This makes it
possible to construct individual and system approaches to avoid
them. Overall, a system-oriented approach that emphasizes proactive and preemptive error management is in accordance with the

PMPH_CH02.indd 17

17

concepts of preventive medicine and patient safety that lie at the


core of enhancing outcomes for surgical patients.41 However, it is
necessary for the individual surgeon to carefully review individual
as well as team member performances for every operation because
a common reason for adverse outcomes is still human error.42
Another important aspect to prevent errors is to be able to predict the conditions that lead to errors.43 There are known sets of circumstances that allow surgeons to make those predictions.28 These
sets of circumstances are known as Error Producing Conditions,
and they link traditional systems approaches to error with advanced
human factors analysis of individual performance. The top error
producing conditions that have been described in aviation accident
investigation that are also very important in surgical care are
1.
2.
3.
4.
5.

Fatigue/physiological degradation
High-risk/low-frequency event
Time pressure
Inadequate standardization
Poor information transfer

The day-time mortality rate of a cohort of 20,547 children admitted


to 15 pediatric ICUs compared to night-time admission revealed
that night admission carries an incremental risk of death from
1.1 to 4.5.44 The associated physiological degradation of skills and
judgment that accompanies fatigue requires no further emphasis
than to extrapolate Dawsons often cited laboratory study45 to the
fatigued surgeon who performs with the same aptitude as a person who is legally drunk. Fatigue acts as a force multiplier on both
individuals and teams, exaggerating small missteps and making it
much more difficult to catch misperceptions and fostering errors
in communications and information exchange.46
The negative impact of fatigue has been well documented in a
retrospective cohort study of general and vascular surgical procedures. Operation start time was the independent variable of interest. Operative cases starting at night demonstrated a strong effect
on morbidity.47 Numerous other studies are found in the literature specific to physician performance during periods of fatigue.48
Demands for surgery respect no time of the day or night, therefore
surgeons must be aware of the impact of fatigue on performance of
complicated tasks and guard against errors during these periods.
Another source of errors is known as plan continuation bias,
which represents the unconscious human bias to pursue a course of
action, a treatment plan, or procedure in spite of changing conditions. Surgeons are goal-directed and oriented toward completing a
course of action and the inability to recognize when modifications
and/or changes in direction are needed leads to adverse events.
This type of task fi xation can be exacerbated by stress, fatigue, and
time compression.49
For surgeons to avoid errors, it is important to understand
these various sets of conditions that make error more likely and
with this information understand the nature and extent of error.
The next step is using this information to account for the conditions that induce error, determine behaviors that prevent or mitigate error, and train personnel in use of error avoidance tools.
Answer: Errors in management of surgical patients follow
known patterns of adverse outcomes in other organizations and
impact all aspects of surgical care.
3. Is there efficacy for reducing errors in surgical patients by using
models known to reduce error in other high-risk endeavors?
Organizations that repeatedly carry out potentially dangerous
procedures with minimal error are known as high-reliability

5/21/2012 7:46:13 PM

18

Surgery: Evidence-Based Practice

organizations (HROs) and safety models from these systems can


be used to enhance surgical outcomes. HROs are characterized as
high-risk, error-intolerant systems that are capable of repetitively
carrying out potentially dangerous tasks with virtually no occurrence of adverse outcomes. Commercial aviation serves as the primary example of the HRO safety model as evidenced by a decade
of annual Federal Aviation Authority statistics indicating that the
risk of a major commercial carrier accident ranges from 0.00 to
0.218 per 1,000,000 flight hours.50 An individual must fly 24 hours/
day every day for 570 years before standing a 1% chance of being
involved in a fatal commercial aviation accident. This safety record
is based on a thorough comprehension of the mechanisms of errors
gleaned from collecting incident and error reports. In aviation, as
in other HROs, reporting and avoiding error has become a compulsion and a cultural norm.
Detailed analysis of pertinent adverse events and near miss
case studies have generated a body of knowledge known as highreliability theory that defines a number of organizational features
likely to reduce the risk of organizational accidents and other
hazards.51 In order to make these principles apply to patient safety
in surgical care, it is necessary to identify specific practices to
adopt from complex, high-risk industries that manage to operate at consistently high levels of reliability. The primary tasks in
designing a system to prevent errors in surgical care are embracing principles that make small and potentially preventable errors
visible when they do occur so that they may be intercepted, and
formulate methods for minimizing the impact of adverse events
when they cannot be intercepted.52
HROs manage risks by understanding how errors occur and
anticipating all possible chances for errors. An important principle of risk management is trapping small missteps before major
adverse events have a chance to take place. This points out an
important difference between safety designs in health care and
HROs in that HRO systems are engineered with the expectation
that individuals can and will make mistakes and that the system
itself must be engineered to catch these mistakes. Physicians and
nurses have been granted a loftier status. We are expected not to
make mistakes despite operating under stressful conditions in a
system that is not attuned to catching small missteps. The system as it exists in health care is not programmed to trap small
errors. For this reason it is important to sustain a mindset that
makes awareness of the potential for errors part of daily surgical care. Safe outcomes depend on managing those risks well and
continually detecting and intervening in small errors before they
are allowed to produce adverse outcomes.
Another HRO safety model is to continually rehearse familiar scenarios of error and strive to imagine novel ones. Instead
of isolating failures, HROs generalize them, and instead of making local repairs, HROs look for system reforms. This is an ideal
model to reduce errors in the care of surgical patients before, during, and after surgical interventions.
One of the keys to understanding error threats is derived
from HRO system-wide use of critical incident reporting (CIR)
programs that collect and objectively analyze errors, incidents,
and near misses. The largest critical incident data base is maintained by NASA for the FAA known as the Aviation Safety Reporting System (ASRS), and it receives over 30,000 reports annually
containing almost 800,000 voluntary, anonymous near misses,
incidents, safety violations, and aviation safety risks.53 Incident
reporting allows for the identification of risks of adverse events
and patterns of risky actions before they propagate through the

PMPH_CH02.indd 18

system and become realized tragedies. Although it is impossible to


know what has not occurred because proactive steps are routinely
taken based on incident reporting to avoid adverse events, the best
gauge of almost events is near miss reporting. One method that
has improved reporting is by broadening the targets of incident
reporting to include no harm events and near misses and easing
the disincentive to reporting actual adverse outcomes. This has
been shown to increase reporting by 3 to 300 fold.54 The value of
critical incident reporting and near miss reporting in lowering
adverse outcomes has been demonstrated in every HRO system.55
In order to successfully collect critical incident information,
the data must be used to generate light not heat requiring the
system to be totally anonymous and nonpunitive.56 Other important characteristics of suitable reporting systems are that they
provide an incentive to report and ensure objective fair reporting
without assigning blame.43 Health care lags significantly behind
in the collecting and analyzing of critical incident data as health
care incident reporting systems collect only 1.5% to 10% of actual
adverse events. The use of CIR systems is gaining momentum in
several specialties such as anesthesia,57 emergency medicine,58 and
critical care.59 Further progress has been made by the National
Patient Safety Foundation that has commissioned the development and implementation of an Internet-based CIR system for
use in critical care environments.60 The ultimate aim of incident
reporting in health care and surgery is several-fold. It can be used
to guide focused enhancements in training, organization, and
management, as well as examining past practices to understand
how things might be improved and done differently.
HROs have accomplished their safety goals through multiple
avenues of training in both simulation and didactic training in
nontechnical skills (NTSs). Nontechnical skills are defined as the
cognitive, social, and personal resource skills that complement
and enhance technical skills and as such contribute to safe and
efficient task performance. This is especially applicable to operating room performance. NTSs focus as much on individual interpersonal skills as on team dynamics since it is the individual that
is the basic building block from which teams and larger organizational groupings are formed.61 Some of these nontechnical skills
that have become the foundation of HRO safety are
1.
2.
3.
4.
5.

Situational awareness (SA)


Crew resource management (CRM)
Communication skills
Leadership and supervision
Human factors (HFs)

These skills are interrelated and revolve on the central axis of


teamwork and communication. Th is is such a crucial aspect of
surgical patient safety and will be dealt with in the fi nal section of this chapter. Other HRO safety skills that can be used to
enhance surgical safety require an acknowledgment that surgical team composition is rarely fi xed due to shift and rotations
patterns and other organizational constraints. The flux of team
composition mandates that each team member carries with him
or her skills that apply regardless of the team make-up at any
given period.
Situational awareness (SA) is simply the big picture; it is the
accurate perception of what is going on with you, your patient, your
team members and the surrounding environment 5 minutes ago,
NOW and 5 minutes from now.62 Building SA requires developing
and maintaining an overall dynamic and temporal awareness of

5/21/2012 7:46:13 PM

Patient Safety in Surgical Care

the clinical entirety based on perceiving all elements in the environment, understanding their interrelationship and implications of
each, and using this understanding to think ahead, to predict, and
to anticipate the most likely eventualities. Forming the composite picture of SA also requires the nontechnical skill of perception,
derived from the clinical experience that comes with repetition, and
develops into astute clinical judgment.63 Prioritizing information
and actions are important aspects of situational awareness in the
care of surgical patients. This fosters quality and timely decisions
and projects the current situation into the future to make educated
guesses as to what lies ahead so that changes in the clinical big
picture do not come as unmanageable surprises. Although building and maintaining SA is largely an individual skill, it requires
team participation in that team members must combine all of
their perceptions and experiences in order to form a correct big
clinical picture. This must be shared through accurate and timely
communication with the team allowing all team members to modify and reassess their clinical impressions as moment to moment
situations change. HROs teach and diligently drill SA skills with
simulation, repetition, team role playing, and supervision and have
demonstrated effectiveness of this training.64
Crew resource management (CRM) is an educational program
that has been developed and based on over three decades of HRO
safety studies. It has evolved from a program focused on individual
attitude and awareness to a broad curriculum of behavioral skills and
teamwork attitudes integrated with technical competencies. CRM
programs evolved steadily, in part because they were informed by
data that validated the importance of human factors. CRM training results in positive reactions to teamwork concepts, increased
knowledge of teamwork principles, and improved teamwork
performance.65 CRM encompasses skills such as clearly defining
team roles and duties, managing distractions, prioritizing tasks,
and avoiding task overload all of which are integral components of
safe and effective operating room teams. HRO and aviation-based
CRM is defined as maximizing procedural effectiveness by using
all available resources, including hardware, software, people,
information, and environment. This is closely related to team communication that is dealt with separately in the final section.
Effective leadership and team supervision are crucial requirements to reduce adverse outcomes and have been repeatedly emphasized in HRO and aviation safety.66 Surgeons provide leadership
in three key areas: strategic direction, monitoring team performance, and teaching team members by providing instructionall
tasks that match those that researchers identified in the functional team leadership literature.67 The characteristics of surgical
leadership in trauma teams have been studied by Yun et al. in a
Level I trauma center.68 They stress the importance of leadership
adaptability since surgeons often work in an uncertain and timeconstrained environment. The ability to get the best performance
from all team members and encourage each person on the team
to share information and knowledge are traits of good leaders and
supervisors that have been emphasized in both the HRO literature
as well as in reviews by the American College of Surgeons.69
Human factors (HF) is the science of understanding and analyzing human physiology and how these factors impact performance.
The most important of these factors is fatigue and awareness of the
impact of fatigue and sleep debt on performance of critical tasks.
Sleep and nap physiology, work attitude, caffeine use, interpersonal
relationships, focus, and work environment are all involved in the
study and management of HF. These factors have been emphasized
in both the HRO safety literature and medical literature.70

PMPH_CH02.indd 19

19

The central usable tenet of high-reliability theory in surgical care is a constant preoccupation with the possibility of error,
and then to use these documented skill sets, as outlined above, to
minimize the occurrence of error or trap small errors before they
cause major patient harm.
Answer: There are useful skills from HROs that have been
shown to reduce adverse outcomes in health care.
4. Can improving teamwork and communication skills of surgical teams mitigate and reduce errors in the care of surgical
patients?
The Joint Commission (2003) statistics have identified 67% of the
root causes of sentinel events are the result of errors of communication between team members.71 Both medical and surgical team communications have been studied in detail.72 Poor teamwork function
and communication lapses among members of the surgical team
have been shown to be key factors in the occurrence of errors. The
integration of improved communication and teamwork skills must
involve all phases of surgical care. Teamwork and communication
are key competencies to improve surgical patient outcomes.
Use of CRM driven communication skills, as outlined previously, by surgeons and surgical teams lead to individual and team
situational awareness, judgment, safety, resource preservation, and
timely contingency planning. This important set of skills has been
emphasized in the medical literature to optimize and manage workload and task assignments, clinical task planning, and review and
critique strategies.73 Skills such as preprocedure briefings, timeouts, and postprocedure debriefings are essential safety skills to
plan procedures and capture lessons learned during the operation.
CRM skills are an indispensible component of communication in
the operating room, and Level II data support the conclusion that
these skills enhance the performance of the operating team and
patient outcomes.74,75
Additionally, information sharing tools such as simple preoperative and surgical checklists have been shown to improve communication between team members, mitigate potential critical events,
and improve patient outcomes.76,77 One of the challenges in the
implementation of tools that could assist surgical teams in mitigating errors is the lag in acquiring evidence that an intervention translates into improved patient outcomes. The use of the World Health
Organizations surgical check list was reviewed in over 3000 surgical
procedures. This revealed that both the surgical mortality rate and
complication rates were reduced. The checklist includes a presurgical briefing that is an important part of the CRM philosophy and
serves as a basic plan of the event prior to its commencement. This
briefing communicates key information about the procedure and
scripts possible ways to deal with unexpected events. Following the
briefing, team members are asked to verify their understanding of
the planned procedure, ensuring a shared mental model.78 The mortality rate prior to the implementation of the checklist was 1.5% and
it was reduced to 0.8% following the utilization of the checklist and
the rate of complications decreased from 11.0% to 7.0%.
Failure to communicate critical information in the operating
room occurs in approximately 30% of team exchanges.73 Bollomo79
has documented impressive reductions in mortality, morbidity, and length of stay in patients after major operations after the
implementation of a formal plan to improve team communication
skills. This prospective cohort study shows a reduction in relative
risk of 57.8% (P < .0001) for major complication, reduction in relative risk of postoperative death by 36.6% (P < .0178), and reduction

5/21/2012 7:46:13 PM

20

Surgery: Evidence-Based Practice

of postoperative length of stay by 4 days (P < .0092). Observational


studies in the operating room have consistently demonstrated that
training clinicians in interpersonal proficiency, teamwork, and
communication skills provides important safety advantages.80
Practicing teamwork and team communication skills with
simulation has also been effective. Simulation training promotes
expert medical competencies, high performance communications,
and collaboration competencies as well. Review of the current literature shows that simulation training can promote expert medical
competencies and communication and collaboration competencies
as well.81 Models of health professional training that do not expose
patients to the risk of learners error include simulation-based training methods.82 Such methods have now been incorporated into the
training milestones of general surgery in the United States.83 Many
studies have demonstrated that teamwork simulation training can
improve patient outcomes in high-risk areas of hospital systems.84
In labor and delivery, team training that includes cross-training,
simulation, and practice of technical skills has shown reduction
in errors in the operating room.85 Evidence-based principles that
lead to effective team training include identification of teamwork
as a key competency, emphasis of teamwork rather than task work,
value of feedback, and reinforcement of team behaviors.86 It is necessary for all team members to be included in team training with
simulation. This is important in surgical safety training since qualitative studies have shown that surgeons perceive a higher degree
of organizational culture, communication, and teamwork within
the operating rooms than anesthetist and nursing staff.87
With increasing frequency, changes in the surgical staff that
care for patients occur now more than ever; therefore, there is a
need to augment system redundancy to improve the teamwork and
communication strategies to keep patients safe. Ineffective handoffs
of patients between surgical teams are increasingly recognized as
sources of adverse events.88,89 The care of surgical patients is distributed over time and location requiring effective handoff practices to
ensure appropriate coordination and continuity of care, especially

at care transition points and during shift changes.90 Handoffs of care


of the surgical patient involve precise communication and require
the exchange of important patient data between individuals and
teams members. This is necessary to facilitate uninterrupted patient
care and maintain professional responsibility and accountability.
Formal handoff skills are being integrated in residency training91
and have been a major safety focus of the Agency for Healthcare
Research and Quality.92 Computer and web based tools have been
shown to enhance the accuracy of patient handoffs.93
Adopting these methods of patient safety in surgery is an
important principle that is being influenced by public pressure,
current legislative initiatives, and an unlimited drive on the part of
surgeons to improve professional performance. The sustainability
of patient safety initiatives that reduce errors in health care has
been affected by both organizational culture and the paucity of evidence that these initiatives are effective. Patient safety leaders agree
that organizational culture and the lack of a just culture in medical
education have hindered progress in diminishing adverse events.94
This is perceived as a hidden curriculum that presents a major cultural barrier. The hidden curriculum is the physical and workforce
organizational infrastructure that influences the learning process
and the socialization to professional norms and rituals.95 The use
of high-reliability safety principles and improved teamwork and
communication strategies can only improve patient care if the
environment in which the skills are acquired and practice allows
such behaviors. Informal, everyday learning experiences outside
of the structured learning events not only contribute to the learning culture but the safety culture as well. It is the responsibility of
individual surgeons to assist in the system-wide adoption of these
principles and to find areas within each individual practice that
this high-reliability mindset can be integrated to most carefully
guard against adverse patient outcomes.
Answer: Improving teamwork with information sharing and
team communication of critical patient information contributes
to reducing errors in surgical care.

Clinical Question Summary


Question

Answer

1 When and how do errors occur during the


care of surgical patients?

Errors can occur in all seven phases of surgical management. These


consist of decision errors, knowledge errors, and procedural errors.

2 Are there unique patterns of errors in


surgical care?

Errors in management of surgical patients follow known patterns of


adverse outcomes in other organizations and all as pects of surgical
care.

3 Do safety models from other high-risk


endeavors have efficacy for reducing
errors in surgical patients?

Safety models based on High-Reliability Organizational theory are


efficacious to minimize the risks of errors in surgical care.

4 Can improving teamwork and communication


skills to surgical teams mitigate and reduce
errors in the care of surgical patients?

Improving teamwork and team communication of critical patient


information contributes to reducing errors in surgical care.

REFERENCES
1. Thorek M. Surgical Errors and Safeguards. JB Lippincott
Company; 1932:vii.
2. Brennan TA, Leape LL, Laird NM, et al. Incidence of adverse
events and negligence in hospitalized patients. Results of
the Harvard Medical Practice Study. NEJM. 1991;324(6):
370-376.

PMPH_CH02.indd 20

Grade

3. Baker GR, Norton PG, Flintoft V, et al. The Canadian Adverse


Events Study: the incidence of adverse events among hospital
patients in Canada. CMAJ. 2004;170(11):1678-1686.
4. Bureau of Justice Statistics. http://bjs.ojp.usdoj.gov. Accessed
11/24/10.
5. Medical Malpractice and Surgical errors and complications. http://
www.wrongdiagnosis.com/s/surgical_errors_complications/
malpractice.htm. Accessed 11/16/10.

5/21/2012 7:46:13 PM

Patient Safety in Surgical Care

6. Kohn LT, Corrigan J, Donaldson MS. To Err Is Human: Building a Safer Health System. Washington: National Academy Press;
2000.
7. Aron D, Headrick L. Educating physicians prepared to improve
care and safety is no accident: it requires a systematic approach.
Qual Saf Health Care. 2002;11:168-173.
8. HealthGrades Quality Study: Fift h Annual Patient Safety in
American Hospitals Study. Golden, CO. Healthgrades, Inc; April
2008.
9. Joint Commission (2010). Sentinel events statistics. March 31,
2010. Oakbrook Terrace Il. http://www.jointcommission.org.
Accessed 11/29/10.
10. Mendelsohn D, Bernstein M. Patient safety in surgery. Israeli
Journal of Emergency Medicine. 2009;9(2):15-21.
11. Stone S, Bernstein M. Prospective error recording in surgery:
an analysis of 1108 elective neurosurgical cases. Neurosurgery.
2007;60(6):1075-1080.
12. Schiff ner TL, Grunwald G, Henderson WG, et al. Relationship
of process and structures of care in general surgery to postoperation outcomes: a hierarchical analysis. J Am Coll Surg.
2007;204:1166-1177.
13. Rasmussen J. Skills, rules, knowledge: signals, signs and symbols and other distinctions in human performance models. IEEE
Trans Syst, Man, Cybern. 1983;SMC-13:257e67.
14. American College of Surgeons, Committee on Trauma, Revised
November 9, 2009.
15. Moulton CA, Regehr G, Lingard L. Slowing down to stay out of
trouble in the operating room: remaining attentive in automaticity. Acad Med. 2010;85:1571-1577.
16. Reason J. Human error: modes and management. BMJ. 2000;
320:768e70.
17. Merry MD, Brown JP. From a culture of safety to a culture of
excellence. J Innov Manage. 2002;7:29-46.
18. Sarker SK. Courses, counsellors & cadavers: reducing errors in
the operating theatre. BMJ. 2003;327:s109.
19. Krizek TJ. Surgical error ethical issues of adverse events. Arch
Surg. 2000;135:1359-1366.
20. Croskerry P. The cognitive imperative: thinking about how we
think. Acad Emerg Med. 2000;7:1223-1231.
21. Shappell S, Detwiler C, Holcomb K, et al. Human error and
commercial aviation accidents: an analysis using the human factors analysis and classification system. Hum Factors. 2007;49:
227-242.
22. Patel VL, Kaufman DR, Arocha JF. Emerging paradigms
of cognition in medical decision-making. J Biomed Inform.
2002;35:52-75.
23. Small SD, Wuerz RC, Simon R, et al. Demonstration of highfidelity simulation team training for emergency medicine. Acad
Emerg Med. 1999;6:312-323.
24. Gaba DM, Howard SK, Fish KJ, et al. Simulation-based training
in anesthesia crisis resource management (ACRM): a decade of
experience. Simulat Gaming. 2001;32(2):175-193.
25. Shojania KG, Burton EC, McDonald KM, et al. Changes in rates
of autopsy-detected diagnostic errors over time: a systematic
review. JAMA. 2003;289(21):2849-2856.
26. Croskerry P. Clinical cognition and diagnostic error: applications of a dual process model of reasoning. Adv Health Sci Educ
Theory Pract. 2009;14(S1):27-35.
27. Croskerry P. A universal model of diagnostic reasoning. Acad
Med. 2009;84(8):1022-1028.
28. Williams JC. A data based method for assessing and reducing
human error to improve operational performance. Proceedings
of IEEE Fourth Conference on Human Factors in Power Plants;
June 5-9, 1988; Monterey, CA. pp. 436-450.

PMPH_CH02.indd 21

21

29. Stahl K, Brien S. Reducing patient errors in trauma care.


In: Stephen Cohn, ed. Acute Care Surgery: Evidenced-Based
Practice. New York: M. D. Informa Healthcare USA Inc.; 2009:
276-287.
30. Catania R. The pre-op guide http://www.preopguide.com. Accessed December 3, 2010.
31. Culig MH, Kunkle RF, Frndak DC, et al. Improving patient care
in cardiac surgery using Toyota production system based methodology. Ann Thorac Surg. 2011;91:394-400.
32. Rose S, Thomas C, Tersigni A, et al. A leadership framework
for culture change in health care. Jt Comm J Qual Patient Saf.
2006;32(8):433-442.
33. McCarthy D, Blumenthal D. Committed to Safety: Ten case
studies on reducing harm to patients. The Commonwealth Fund,
April 2006.
34. Pronovost P. Freischlag JA. Improving teamwork to reduce surgical mortality. JAMA. 2010;304(15):1721-1722.
35. Hurlbert S, Garrett J. Improving operating room safety. Patient
Saf Surg. 2009;3:25
36. Gruen RL, Jurkovich GJ, McIntyre LK, et al. Patterns of errors
contributing to trauma mortality: lessons learned from 2594
deaths. Ann Surg. 2006;244:371-380.
37. Petersen LA, Orav FA, Teich JM, et al. Using a computerized
sign-out program to improve continuity of inpatient care
and prevent adverse events. J Comm J Qual Improv. 1998;24:
77-87.
38. Lee RC, Cooke DL, Richards M. A system analysis of a suboptimal surgical experience. Patient Saf Surg. 2009;3(1):1.
39. Andersen J, Hjort-Jakobsen P, Christiansen D, et al. Readmission rates after a planned hospital stay of 2 versus 3 days in fasttrack colonic surgery. Br J Surg. 2007;94:890-893.
40. Benbassat J, Taragin M. Hospital readmissions as a measure of
quality of health care advantages and limitations. Arch Intern
Med. 2000;160:1074-1081.
41. Stockwell DC, Slonim AD. Quality and safety in the intensive
care unit. J Intensive Care Med. 2006;21:199-210.
42. Gaba DM, Fish KJ, Howard SK. Crisis Management in Anesthesiology. New York: Churchill Livingstone; 1994.
43. Hollnagel E. Barriers and Accident Prevention. Hampshire UK:
Ashgate Publishing Limited; 2004.
44. Arias Y, Taylor DS, Marcin JP. Association between evening
admissions and higher mortality rates in the pediatric intensive
care unit. Pediatrics. 2004;113(6):530-534.
45. Dawson D, Reid K. Fatigue, alcohol and performance impairment. Nature. 1997;388:235.
46. Helmreich RL, Merritt AC. Culture at Work: National, Organizational and Professional Influences. Aldershot: Ashgate; 1998.
47. Kelz R, Timothy T, Hosokawa P, et al. Time-of-day effects on
surgical outcomes in the private sector: a retrospective cohort
study. J Am Coll Surg. 2009;209:434-445.
48. Weinger M, Ancoli-Israel S. Sleep deprivation and clinical performance. JAMA. 2002;287(8):955-957.
49. Durden R. Going below minimums. Aviation Safety. 2008(28):4-7.
50. http://www.ntsb.gov/aviation/Stats.htm. Accessed Dec. 17, 2010.
51. Shojania KG, Duncan BW, McDonald KM, et al., eds. Making
Health Care Safer: A Critical Analysis of Patient Safety Practices.
Evidence Report/Technology Assessment No. 43 (Prepared by
the University of California at San FranciscoStanford Evidencebased Practice Center under Contract No. 290-97-0013), AHRQ
Publication No. 01-E058, Rockville, MD: Agency for Healthcare
Research and Quality. July 2001.
52. Nolan TW. System changes to improve patient safety. BMJ.
2000;320:771-773.
53. http://asrs.arc.nasa.gov. Accessed November 19, 2010.

5/21/2012 7:46:13 PM

22

Surgery: Evidence-Based Practice

54. Battles J, Kaplan H, Van der Schaaf T, et al. The attributes of


medical event reporting systems. Arch Pathol Lab Med. 1998;122:
231-238.
55. Barach P, Small S. Reporting and preventing medial mishaps:
lessons from non-medical near miss reporting systems. BMJ.
2000;320:759-763.
56. Geiduschek JM. Registry offers insight on preventing cardiac
arrests in children. ASA Newsletter. 1998;62(6):16-18.
57. Choy CY. Critical incident monitoring in anaesthesia. Curr Opin
Anaesthesiol. 2008;21(2):183-186.
58. Kram R. Critical incident reporting system in emergency medicine. Curr Opin Anaesthesiol. 2008;21(2):240-244.
59. Pronovost PJ, Thompson DA, Holzmueller CG, et al. Toward
learning from patient safety reporting systems. J Crit Care.
2006;21(4):305-315.
60. Wu AW, Holzmueller CG, Lubomski LH, et al. Development of
the ICU safety reporting system. J Patient Saf. 2005;1:23-32.
61. Flin R, OConnor P, Crichton M. Safety at the Sharp End: A Guide
to Non-technical Skills. Ashgate Publishing Ltd; 2008.
62. Kern T. Redefining Airmanship. New York: McGraw-Hill Corp.;
1997.
63. Endsley M. Expertise and situation awareness. In: Ericsson KA,
et al. eds. Cambridge Handbook of Expertise and Expert Performance. New York: Cambridge University Press; 2006.
64. Banbury S, Dudefield H, Hormann J, et al. FASA: Development
and validation of a novel measure to assess the effectiveness of
commercial airline pilot situation awareness training. Int J Aviat
Psych. 2007;17:131-152.
65. Salas E, Burke SC, Bowers CA, et al. Team training in the skies:
does crew resource management (CRM) training work? Hum
Factors. 2001;43:641-674.
66. Day DV, Halpin SM. Leadership development; a review of industry best practices. 2001; Technical Report 111, Alexandria VA. US
Army Research Institute for the Behavioral and Social Sciences.
67. Klein KJ. Teamwork in a shock trauma unit: new lessons in leadership: 2006; Knowledge@Wharton http://knowledge.wharton.
upenn.edu/article.cfm?articleid=1048. Accessed November 22,
2010.
68. Yun S, Faraj S, Sims HP. Contingent leadership and effectiveness of
trauma resuscitation teams. J Appl Psychol. 2005;90:1288-1296.
69. Healy GB, Barker J, Madonna G. The surgeon as a leader. Bul Am
Col Surg. 2006;91:26-29.
70. Jha AK, Duncan BW, Bates DW. Fatigue, Sleepiness and Medical Errors. in Making Health Care Safer: A Critical Analysis of
Patient Safety Practices. Evidence Report/Technology Assessment. AHRQ Publication 01-E058 July 20, 2001. Chap 46.
71. http://www.jcaho.org/accreditation+organizations.htm. Accessed December 20, 2010.
72. Salas E, Cannon-Bowers J, Weaver J. Command and control
teams; principles for training and assessment. In: Flin R, Arbuthnot K, eds. Incident Command: Tales from the Hot Seat. Aldershot: Ashgate; 2002.
73. Risser DT, Rice MM, Salisbury ML, et al. The potential for
improved teamwork to reduce medical errors in the emergency
department. The MedTeams Research Consortium. Ann Emerg
Med. 1999;34:373-383.

PMPH_CH02.indd 22

74. Awad SS, Fagan SP, Bellows C, et al. Bridging the communication gap in the operating room with medical team training.
Am J Surg. 2005;190(5):770-774.
75. Lingard L, Epsin S, Whyte S, et al. Communication failures in
the operating room: an observational classification of recurrent
types and effects. Qual Saf Health Care. 2004;13:330-334.
76. Haynes AB, Weiser TG, Berry WR, et al. A surgical safety checklist to reduce morbidity and mortality in a global population.
NEJM. 2009;360:491-499.
77. Stahl K, Palileo A, Schulman C, et al. Enhancing patient safety in
the trauma/surgical intensive care unit. J Trauma. 2009;67:430-435.
78. Musson DM, Helmreich RL. Team training and resource management in health care: current issues and future directions.
Harvard Health Policy Rev. 2004;5(1):25-35.
79. Bellomo R, Goldsmith S, Uchino J, et al. Prospective controlled
trial of effect of medical emergency team on postoperative morbidity and mortality rates. Critical Care Med. 2004;32(4);916-921.
80. Healey AN, Undre S, Vincent CA. Defining the technical skills
of teamwork in surgery. Qual Saf Health Care. 2006;15:231-234.
81. Aggaral R, Myttas OT, Dertreau M. Training and simulation for
patient safety. Qual Saf Health Care. 2010;19(Supp2):i34-i43.
82. Reznick RK, MacRae H. Teaching surgical skills-changes in the
wind. NEJM. 2006;355:2664-2669.
83. Swanstrom LL, Fried GM, Hoffmna KI, et al. Beta test results of a
new system for assessment of competence in laproscopic surgery.
J Am Coll Surg. 2006;202:62-69 .
84. Reade TW, Flin R, Mearns K, et al. Developing a team performance framework for the intensive care unit. Crit Care Med.
2009;37:1787-1793.
85. Birnbach D, Salas E. Can medical simulation and team training
reduce errors in labour and delivery. Anesth Clin. 2008;26:159-169.
86. Salas E, Diazbrenadrs D, Weaver SJ, et al. Does team training
work? Acad Emerg Med. 2008;15:1002-1009.
87. Mills P, Neily, J, Dunn El. Teamwork and communication for
surgical teams: implications for patient safety. J Am Coll Surg.
2008;206:107-112.
88. Mistry NJ, Toulany A, Edmonds JF, et al. Optimizing physician
handover through the creation of a comprehensive Minimum
Data Set. Healthcare Quart. 2010;13:102-109.
89. Beach C. Lost in transition. AHRQ morbidity & mortality rounds
on the web. http://www.webmm.ahrq.gov/case.aspx?caseID116.
Accessed February 1, 2011.
90. Manser T, Foster S, Gisin S, et al. Assessing the quality of patient
handoffs at care transitions. Qual Saf Health Care. 2010;19:e44.
91. Telem DA, Buch KE, Ellis S, et al. Integration of a formalized handoff system into the surgical curriculum. Arch
Surg. 2011;146(1):89-93.
92. http://psnet.ahrq.gov/primer.aspx?primerID=9. Accessed February
9, 2011.
93. Motamedi SM, Posadas-Calleja J, Straus S, et al. The efficacy of
computer-enabled discharge communication interventions: a
systematic review. Qual Saf Health Care. 2011;24.
94. Leape LL, Berwick DM. Five years after To Err Is Human: what
have we learned? JAMA. 2005;293:2384-2390.
95. Haffety FW. Beyond curriculum reform: confronting medicines
hidden curriculum. Acad Med. 1998;73:403-407.

5/21/2012 7:46:13 PM

PART 1

THE ESOPHAGUS

PMPH_CH03.indd 23

5/21/2012 7:48:24 PM

PMPH_CH03.indd 24

5/21/2012 7:48:25 PM

CHAPTER 3

Esophageal Perforation
Jonathan B. Lundy and G. Travis Clifton

In the byways of surgery there can be few conditions more


dramatic in their presentation and more terrible in their
symptoms than spontaneous perforation of the oesophagus.
Norman Rupert Barrett, 1946

with gastric and oral secretions. The mediastinal pleura may initially remain intact; however, after a short period of soilage, the
layer is prone to breakdown and ipsilateral pleural contamination. Shock from sepsis with large volumes of fluid sequestration
into these contaminated cavities is often part of the clinical scenario especially when the diagnosis is delayed. Delay in diagnosis
of Boerhaaves syndrome is unfortunately the norm. Described
as a diagnostic masquerader, it may mimic other disease processes causing severe chest pain such as myocardial infarction,
aortic dissection, pancreatitis, pulmonary embolism, peptic
ulcer disease, cholecystitis, and pneumonia.9 The true incidence
of spontaneous esophageal perforation has not been clarified in
the surgical literature.
The initial phase of care of patients with esophageal disruption consists of resuscitation to include the use of crystalloid
infusion, vasoactive medications in the setting of hypotension
not responsive to volume repletion, broad spectrum antibiotics
for the coverage of flora potentially contaminating the mediastinum, and limiting continued contamination by stopping oral
intake. Some experts recommend gentle placement of a nasogastric tube for decompression of the stomach.10 A perforation
severity score was described by Abbas and colleagues in their
review of 119 patients with perforation from multiple causes and
included hemodynamic, inflammatory, age-related, and respiratory variables.11 Although higher total severity scores correlated
with longer duration of stay and increased morbidity and mortality, the scoring system has not yet been validated. Outcome
following esophageal perforation varies based on etiology, but
classically it is much worse following Boerhaaves syndrome.
Some reports document mortality between 30% and 40%, with
lower mortality if the diagnosis is made within 24 hours.1,12-14
However, if diagnosis is delayed, the rate of death increases to
greater than 50%.15 Through improved diagnostic and intensive
care capabilities, and aggressive treatment (either operative or
conservative), some recent series document mortality rates less
than 10% even in the setting of diagnostic delay greater than
24 hours.16 The goal of this chapter is to describe the evidence
available to assist in the early diagnosis and in the optimal

BACKGROUND/OVERVIEW
Spontaneous esophageal rupture has gone by many names including atraumatic panmural rupture, barogenic rupture, effort rupture, primary pressure rupture, and emetogenic rupture.1,2 The
first description of spontaneous thoracic esophageal perforation
is credited to Dr Hermann Boerhaave, who in 1724 reported postmortem findings of the Grand Admiral of the Dutch fleet who
died after developing postemetic, severe chest pain.3 The eponymous entity, now termed Boerhaaves syndrome, is believed to be
caused by high intraluminal pressure in the esophagus. Intragastric pressures measured during emesis can reach 120 mm Hg.
In the esophagus, this pressure rise may occur in the setting of
a constricted upper esophageal sphincter and result in transmural rupture of the esophagus in the distal portion, typically
the left posterolateral wall (as many as 90%) 2 to 3 cm proximal
to the lower esophageal sphincter.4,5 Spontaneous rupture has
been described involving the mid-thoracic and cervical portion
of the esophagus as well.2 Brinsters series of esophageal injury
patients outlines the frequency of various etiologies causing
esophageal disruption.6 Iatrogenic perforation, postinstrumentation accounts for 60% of cases, Boerhaaves syndrome occurs
in 15%, foreign body ingestion in another 12%, trauma in 9%,
operative injury (bariatric/gastroesophageal reflux disease procedures) in 2%, tumor related causes in 1%, and other causes in
2%. Although Boerhaaves syndrome is classically described after
emesis, vomiting is not a prerequisite. Esophageal rupture has
been described after strenuous defecation, parturition, asthma,
seizure, and blunt trauma.7 The typical male to female ratio of
spontaneous rupture is 5:1 with the development classically
described between the fourth and sixth decade of life.8 The tear
is usually longitudinal and leads to mediastinal contamination
25

PMPH_CH03.indd 25

5/21/2012 7:48:25 PM

26

Surgery: Evidence-Based Practice

management strategy for patients suffering esophageal perforation. Although the focus is on spontaneous perforation, significant overlap exists in the diagnostic and treatment strategies
applied to all causes of esophageal perforation. The body of lit-

erature this chapter is based on consists almost entirely of singlecenter, retrospective case series. Uniformly, the evidence is Level
IV with Grade C or D recommendations made based on published findings. (See Table 3.1)

Table 3.1 Evidence-Based Table Based on Type of Intervention for Esophageal Perforation
Year, Author,
Reference #

Number
of
Patients

Study Design

Findings/Recommendations

Level of
Evidence

2010, Keeling
et al. #16

97

Retrospective, singlecenter case series

Primary repair should be considered the first-line


treatment even if treatment is carried out beyond
24 hours.

2010, Neel
et al. #64

31

Retrospective, singlecenter case series

Operative intervention with repair recommended even if


performed in delayed fashion.

2010, Shaker
et al. #36

27

Retrospective, singlecenter case series

Recommended education of primary providers about


early diagnosis and referral of patients with esophageal
perforation.

2010,
Vallbohmer
et al. #59

44

Retrospective, singlecenter case series

Varied treatments (suture repair, esophagectomy, stent


placement), outcomes greatly improved if treatment
occurred within 24 hours.

119

Retrospective, singlecenter case series

Base approach on injury severity and amount of


contamination of chest. Nonoperative management
likely successful with favorable clinical and radiographic
findings.

2009, Freeman
et al. #91

19

Retrospective, singlecenter case series

Stent placement viable option for esophageal perforation.

2009, Sutcliffe
et al. #53

21

Retrospective, singlecenter case series

Operative intervention varied based on timing of


treatment, state that surgical intervention is superior
to nonoperative treatment.

2009, Wang
et al. #65

18

Retrospective, singlecenter case series

Primary repair appropriate despite delay in treatment.

2008, Cho et al.


#67

10

Retrospective, singlecenter case series

Primary repair acceptable even if delay in treatment


beyond 24 hours. Leak more common if treatment is
delayed.

2007, Erdogan
et al. #78

23

Retrospective, singlecenter case series

Primary repair reinforced with fibrin tissue patch is ideal


intervention.

2006, Fischer
et al. #90

15

Retrospective, singlecenter case series

Self-expanding metal stents may be utilized for esophageal


injury even in setting of delayed presentation.

2005, Chao
et al. #68

28

Retrospective, singlecenter case series

Primary repair even if delay in treatment has occurred.

2005, Vogel
et al. #54

47

Retrospective, singlecenter case series

Advocate aggressive, conservative management with


control of sepsis and drainage of esophageal leak. No
deaths occurred in 34 patients treated nonoperatively.

2004, Jourgon
et al. #69

25

Retrospective, singlecenter case series

Long interval between symptom development and


treatment does not preclude primary repair.

2004, Gupta
et al. #87

57

Retrospective, singlecenter case series

Recommend esophagectomy for perforation in setting of


preexisting esophageal disease.

2003, Kollmar
et al. #5

17

Meta-analysis with
retrospective, singlecenter case series

Although reported outcomes as a meta-analysis, no


statistical results were given and a very limited number
of sources were utilized in table form. Recommended
primary suture repair only if treated within 12 hours of
symptoms. All other patients should be evaluated for
esophageal resection and two-stage reconstruction.

2009, Abbas
et al. #11

(Continued)

PMPH_CH03.indd 26

5/21/2012 7:48:25 PM

Esophageal Perforation

27

Table 3.1 (Continued)


Year, Author,
Reference #

Number
of
Patients

Study Design

Findings/Recommendations

Level of
Evidence

1999, Lawrence
et al. #57

21

Retrospective, singlecenter case series

Recommend primary repair with mediastinal drainage and


decompressive gastrostomy even if treated after
24 hours of symptoms.

1998, Altorjay
et al. #26

27

Retrospective, singlecenter case series

Advocate for single-stage resection and reconstruction in


setting of perforation.

1997, Kotsis
et al. #82

36

Retrospective, singlecenter case series

No deaths or leaks with primary repair with


reinforcement with vascularized tissue.

1996, Wang
et al. #66

27

Retrospective, singlecenter case series

Primary repair appropriate no matter the time from


symptom development.

1995, Wright
et al. #81

28

Retrospective, singlecenter case series

Recommend tissue buttressed repair unless patient has


nondilatable stricture or malignancy.

1995, Whyte
et al. #58

22

Retrospective, singlecenter case series

Recommend primary repair even with delay unless patient


has a distal obstructing lesion or malignancy.

1994,
Richardson
et al. #79

14

Retrospective, singlecenter case series

Recommend use of autologous muscle tissue to reinforce


repair. In addition, described cohort of patients with
difficult defects that obtained definitive closure with
use of only diaphragmatic muscle flap.

1993, Salo et al.


#86

90

Retrospective, singlecenter case series

Mortality of primary repair 68%, esophagectomy 13%.


Recommend staged esophagectomy/reconstruction in
setting of delayed perforation with sepsis.

1989, Pate et al.


#24

34

Retrospective, singlecenter case series

Recommend primary repair with use of pleura or


intercostals muscle for reinforcement.

1989, Gouge
et al. #80

18

Retrospective, singlecenter case series

Recommend pleural flap reinforcement of primary repair.


Resection appropriate for extensive necrosis/damage.

1987, Nesbitt
et al. #29

115

Retrospective, singlecenter case series

Treatment of choice is primary repair with mediastinal/


pleural drainage despite interval from symptoms.

1984, Ajalat
et al. #26

21

Retrospective, singlecenter case series

Primary repair with chest drainage is procedure of choice.

1981, Michel
et al. #22

85

Retrospective, singlecenter case series

Advocate treatment of thoracic perforation with primary


suture repair with pleural wrap reinforcement.

1978, Symbas
et al. #71

Retrospective, singlecenter case series

Primary repair reasonable even if patients present late.


Recommend gastric decompression and the use of
fundoplication if perforation diagnosed late.

1973, Hardy
et al. #61

36

Retrospective, singlecenter case series

Recommend direct attempt at suture repair when


diagnosis is made early. Mediastinal drainage and
gastrostomy only when diagnosed late.

1972, Keighley
et al. #7

12

Retrospective, singlecenter case series

Recommend increasing awareness of diagnosis, identifying


full extent of mucosal injury, and direct suture repair of
disruption.

1972, Keighley
et al. #70

33

Retrospective, singlecenter case series

Low, 8% mortality for suture repair for spontaneous


perforation.

1965, Foster
et al. #60

42

Retrospective, singlecenter case series

Recommend primary closure and drainage of


mediastinum/pleura.

1. What are the most reliable symptoms and physical findings


for diagnosing esophageal perforation?
The classically described Macklers triad seen with spontaneous
esophageal perforation consists of antecedent emesis, chest pain,
and subcutaneous emphysema and is frequently absent at initial

PMPH_CH03.indd 27

presentation leading to a delay in diagnosis.17 Early diagnosis is


difficult as a result of the nonspecific symptoms, physical findings,
and imaging results that are seen after esophageal perforation;
many overlapping with findings of other acute processes. The correct diagnosis after spontaneous esophageal perforation was made
within 12 hours in only 21% of cases in Abbotts classic series

5/21/2012 7:48:25 PM

28

Surgery: Evidence-Based Practice

reported in 1970.1 Along with being difficult to diagnose, the high


mortality of spontaneous esophageal perforation has led some to
consider esophageal perforation to be the most lethal perforation
of the gastrointestinal tract.18 Although reported before the availability of current standards of imaging, antibiotics, nutrition, and
intensive care capabilities, Derbes and Mitchell described in their
series of 157 cases of Boerhaaves syndrome a mortality of 25% by
12 hours in a subgroup of 71 untreated patients.19 Many case series
in fact include postmortem examples of Boerhaaves syndrome.20-23
As previously stated, a history of vomiting is not mandatory
for the diagnosis of Boerhaaves syndrome to exist. Antecedent
emesis occurs in up to 75% of patients in series that report this
history.20,24 Uniformly, chest pain is the most common presenting
symptom in patients suffering Boerhaaves syndrome.7,20,24-29 Fever
follows as the second most common presenting symptom. Dyspnea and subcutaneous emphysema occur less frequently but are
reported in most series as the next most frequent signs on examination. Subcutaneous emphysema detected by palpation has been
reported in as few as 30% of patients in recent series on the subject
of esophageal perforation and is considered a finding of delayed
presentation/diagnosis.24,30,31
Summary: The most common symptoms associated with
spontaneous esophageal perforation include chest pain, fevers,
and dyspnea. Emesis is common but not mandatory for the development of Boerhaaves syndrome. Subcutaneous emphysema, and
Macklers triad (cervical subcutaneous emphysema, emesis, and
chest pain) are typically findings of late presentation/diagnosis.
(Grade C recommendations.)
2. What are the most reliable diagnostic studies to evaluate for
esophageal perforation?
The classic radiographic finding of Naclerios V sign was first
reported in 1957 by Dr Emil A. Naclerio.32 The finding represents
localized emphysema outside of the ipsilateral mediastinal and
diaphragmatic pleura without a violation of the pleura itself.20
Naclerio suggested that this finding correlated with early diagnosis, and prompt surgery in this setting had a high likelihood
of success. Han and McElvein suggested that at least an hour
must transpire since perforation before the finding of mediastinal
emphysema can develop.33 Chest x-ray is commonly abnormal in
reports of patients with Boerhaaves syndrome. Bradley reported
100% of chest radiographs were abnormal in his 1981 series.25
Rates of abnormal plain chest roentgenograms range from 85%
to 97% in modern series, meaning that a substantial portion of
patients will have unremarkable plain fi lm imaging.24,27,33,34 The
high rate of normal chest radiographs, make this study less than
ideal for definitively excluding esophageal perforation. The list of
abnormalities that can be found on chest radiograph after spontaneous esophageal perforation includes ipsilateral pleural eff usion,
atelecatasis, infi ltrate, hydropneumothorax, tension pneumothorax, subcutaneous emphysema, and normal appearance of the
chest. Panzini reported that pneumomediastinum was the most
common chest x-ray finding in their retrospective case series.35
Most importantly, when caring for a patient with a history of
emesis or the spontaneous development of severe chest pain, who
is found to have an abnormality on chest radiograph listed above,
a high index of suspicion for the diagnosis discussed in this chapter is vital and may be life saving. The fact that the initial evaluation of a patient with chest pain will likely not be carried out by a

PMPH_CH03.indd 28

practitioner with extensive experience with the care of spontaneous esophageal perforation makes early diagnosis based on subtle
imaging findings less likely. Shaker et al. have suggested that a
program for educating primary care providers about this uncommon disease is appropriate to ensure more rapid diagnosis and
referral for intervention.36
Because chest radiographs are not often a definitive diagnostic
study, other imaging modalities are typically required. The tools
available include contrast esophagography, computed tomography,
and esophagoscopy using fiberoptic technology. The use of contrast
esophagography provides the theoretic benefit of coating the entire
esophageal mucosa to identify defects. Using fluoroscopy, the study
can be repeated for areas of concern. Historically, it has been recommended that when evaluating for esophageal extravasation, oral
Gastrograffin be administered first, followed by barium esophagogram if the prior procedure is negative, due to reports of barium
inducing a desmoplastic reaction when extravasation occurs, leading to mediastinitis or peritonitis. It is important to note, however,
that Gastrograffin extravasates in only 50% of cervical perforations
and 80% of thoracic esophageal perforations in some reports.37 In
addition, water soluble agents such as Gastrograffin have been associated with pulmonary edema when aspiration occurs. This risk is
most commonly seen when using iodinated agents such as Hypaque
or Renografin.38-40 Barium has a higher density and better mucosal
adherence improving detection of cervical perforation to 60% and
thoracic esophageal perforation to 90%.37 Ultimately, between 25%
and 50% of esophageal perforations not seen with water soluble contrast material will be demonstrated with barium.41,42 Rubesin and
Levine published a review paper on the subject of gastrointestinal
perforations and included their institutional experience with the
radiographic diagnosis of these emergencies.43 During their total
of 20 years experience, no cases of barium induced mediastinitis/
peritonitis or pulmonary edema due to aspiration of water soluble
contrast material had occurred. Most experts continue to recommend the initial use of water soluble contrast esophagography followed by barium for the detection of esophageal perforation. In
review of available retrospective trials of esophageal perforation
that include the imaging modality used, esophagogram is diagnostic in 73% to 100% of cases.24,29,34,44,45 The disadvantages to relying on
contrast esophagography for diagnosing of esophageal perforation
include upto a 12% false-negative rate and the fact that it requires
in-house technicians to perform (only 11 of 14 patients in Ghanems
radiologic diagnosis study of Boerhaaves syndrome patients were
able to have esophagogram performed).46
Direct visualization of an esophageal defect with fiberoptic
endoscopy is another technique to diagnose rupture. A potential
benefit to esophagogastroduodenoscopy (EGD) over other modalities, especially with modern endoscopic technology, is the potential for endoluminal therapy. More recent series have reported the
use of esophagoscopy to diagnose perforation with a sensitivity
as high as 100% but a specificity of only 60% to 83%.47 Mizutani
reported 26 total esophageal defects (11 were due to spontaneous
rupture) with 9 of 15 (60%) having a positive EGD.34 Although
some authors suggest that EGD has a higher sensitivity and specificity, it is dependent on the skill of the endoscopist and there is a
theoretic risk of conversion of a submucosal or small full-thickness
disruption into a more complex process.48
All spontaneous esophageal perforation patients who underwent computed tomography (CT) (n = 14) in the retrospective series reported by Ghanem and colleagues had evidence of

5/21/2012 7:48:25 PM

Esophageal Perforation

periesophageal air tracks.46 This finding along with periaortic and


perispinal air were also noted to be helpful diagnostic fi ndings on
CT imaging. Ghanems report is consistent with other small series
describing CT findings with esophageal perforation.49,50 Less frequent and less specific findings in the setting of esophageal perforation include esophageal wall thickening, intramural esophageal
hematoma, and mediastinal fluid.50,51 When one combines the
facts that CT imaging can be used quickly without additional personnel, its technology is continually improving and evolving, and
it has high specificity (as noted earlier), it is likely that CT will
eclipse esophagography as the imaging procedure of choice for the
diagnosis of spontaneous esophageal perforation.
Summary: For the diagnosis of esophageal perforation, no recommendation can be given for any one specific diagnostic study
over another as various imaging methods appear to be equivalent
in accuracy. (Grade C recommendation.)
3. Is there a role for nonoperative management of patients with
esophageal perforation?
The first series to describe extensive success with nonoperative management of esophageal perforation was the report by
Mengoli and Klassen published in 1965.52 A total of 21 patients
were included, 18 managed conservatively with a remarkable 6%
mortality. Lyons and colleagues compared a group of 18 patients
who underwent esophageal repair with 11 patients treated conservatively.13 Mortality was 38% in patients who underwent thoracotomy and 9% in patients managed nonoperatively. In 1979,
John Cameron and colleagues first described a set of criteria that
support nonoperative management of a select group of patients
after esophageal perforation.15 The criteria include rupture containment evidenced by reflux of extraluminal contrast back into
the esophagus, no pleural contamination, and no systemic signs
of uncontrolled sepsis. Abbas and colleagues reported successful nonoperative management of Boerhaaves syndrome in 10 of
44 patients, with no oral intake for 24 to 72 hours. Despite being
relegated to nonoperative management, this subgroup of patients
typically underwent some form of invasive intervention to include
endoscopy, intraluminal stent placement, thoracostomy tube
placement for pleural or mediastinal drainage, gastrostomy for
decompression, and possible feeding jejunostomy.11 Sutcliffe et al.,
however, reported a case series of 21 patients, and advocated that
operative intervention is superior to nonoperative management if
patients are diagnosed/referred after 24 hours.53 Vogel and colleagues have provided the most recent series of patients that contribute support for nonoperative management after esophageal
perforation.54 Their report describes a total of 47 perforations, with
34 of these being managed nonoperatively. Thirteen of 14 patients
with Boerhaaves were managed conservatively with no deaths
in the nonoperative group. This approach must include frequent
radiologic support to diagnose and treat undrained, intrathoracic
fluid collections and document healing of the esophageal disruption. The authors report that aggressive, conservative management eliminates the source for morbidity and mortality, avoids
major surgical procedures, and allows esophageal healing.
Summary: Patients without signs of systemic sepsis and a
contained esophageal perforation may be managed nonoperatively. These patients typically will require some form of invasive
therapy such as tube thoracostomy or feeding access. (Grade C
recommendation.)

PMPH_CH03.indd 29

29

4. Is there a role for primary repair of intrathoracic esophageal


perforation?
Descriptions of the first two cases of successful repair after spontaneous esophageal perforation were reported by Barrett and
Clagett in 1947.55,56 The most frequently utilized technique for
management is simple, direct suture repair in one or multiple layers. The details of the procedure are as follows: after posterolateral
thoracotomy, all nonviable tissue is debrided and the perforation
is exposed. The mucosal defect is typically larger than the muscular injury, thereby, myotomy proximal and distal to the injury is
required to define these mucosal edges.7,8 After debridement of the
edges, interrupted sutures are used to close the mucosal defect.57
Several series also advocate an interrupted closure of the muscular defect. Intraesophageal dilators of 40 F to 46 F may be useful to
ensure that an adequate lumen remains after repair.58 Insufflation
of the repair site to ensure a watertight closure is also described.
Wide drainage of the mediastinum and pleura is prudent. Consideration for enteral feeding access can be made and options
include gastrostomy and jejunostomy; however, these may require
entrance into the peritoneal cavity.
One of the most controversial subjects regarding the management of Boerhaaves syndrome is the appropriate intervention
for a patient diagnosed in a delayed fashion. The cutoff time for
increased mortality has typically been set at 24 hours after symptom development. Several series have reported a dramatic increase
in mortality when diagnosis or treatment is delayed beyond
24 hours.26,53,59 Eloquently stated by Foster in his series of 42 patients
with esophageal injury, however, is that the cutoff of 24 hours
is arbitrary and any delay in intervention in an ill patient is
hazardous.60 Wilsons series in 1971 contained the recommendation that delay in diagnosis should prompt the surgeon to perform
mediastinal and pleural drainage, and suture repair should be
relegated to patients presenting early.20 Simple debridement and
drainage has been advocated by others if the esophagus adjacent
to the perforation is found to be necrotic.26,61 The debate about
the optimal operative repair after a delay in diagnosis is ongoing.
Advocates for intervention other than suture repair alone contend
that the most common cause of morbidity and mortality is dehiscence of the suture with esophageal leak and ongoing mediastinal
sepsis. These authors recommend the use of autologous material to
buttress the repair site, an exclusion and diversion procedure, or
esophagectomy.4,62,63 Despite the diversity of opinions, the majority of case series describe and advocate for direct suture repair
of spontaneous esophageal perforation no matter the length of
time between occurrence and treatment.7,8,11,16,57,60,64-71 Although
the classic approach for treatment of the perforated esophagus
has been the use of the posterolateral thoracotomy, reports of
the management of thoracic perforation via laparotomy exist.72
In addition, minimally invasive techniques such as laparoscopy
and thoracoscopy have been utilized successfully in patients with
Boerhaaves syndrome.11,73,74
The use of vascularized tissue to reinforce suture repair has
been described as an adjunct for management of esophageal perforation. The autologous tissue used to buttress repair include
pleura, pericardial fat, intercostals muscle, diaphragm, fundus
of stomach, aortic adventitia, omentum, rhomboid muscle, and
latissimus muscle.75,76 Fibrin has also been described as an allogenic material that can be used to reinforce the esophageal suture
repair.77,78 Richardson and colleagues have also described the

5/21/2012 7:48:25 PM

30

Surgery: Evidence-Based Practice

use of a diaphragmatic muscle flap as definitive closure of difficult esophageal defects that were not amenable to primary
suture repair.79 Proponents for the use of a vascularized tissue
flap to reinforce the primary repair report a potential benefit in
decreased esophageal fistula rate as well as an improvement in
mortality.22,71,79-82 However, even with autologous tissue reinforcement, delay in diagnosis and repair can result in esophagocutaneous fistula rates as high as 83%.66
Summary: Primary repair of esophageal perforation is appropriate even if the intervention is carried out more than 24 hours
after symptoms develop. Consideration may be made for buttressing of the suture line with vascularized tissue but is not mandatory. (Grade C recommendation.)
5. Is there a role for an exclusion and diverting procedure after
esophageal perforation?
Drainage or diverting procedures for esophageal perforation first
appeared in the literature in the latter half of the 20th century.
The use of a T-tube inserted into the esophageal defect with passage of a nasogastric tube through the lumen of the drainage tube
was reported by Abbott and colleagues in 1970.1 Shor-Pinsker and
colleagues used division of the proximal stomach with double gastrostomy (one for drainage of the proximal segment, one for enteral
feeding) and reported the technique in 1970.83 Menguy described the
use of the loop cervical esophagostomy for diversion of oral secretions 1 year later.84 Urschel reported his technique of side-cervical
esophagostomy (in continuity) with pleural drainage of the perforated esophagus, closure of the distal esophagus via an esophageal
band, and feeding gastrostomy in 1974.63 Proponents of diversion/
exclusion procedures site the elimination of continued contamination of the mediastinum as well as preservation of the esophagus
in situ allowing for later reconstruction as benefits of this management technique.1,63 Exclusion/diversion procedures have fallen out
of favor in most recent series except in patients too unstable for a
more definitive repair.11 The review of the management of esophageal injury from all causes by Wu and Mattox includes an algorithm with the role of T-tube drainage or an exclusion/diversion
procedure relegated to unstable patients who would not tolerate a
more aggressive approach.75 In the series by Attar and colleagues,
of the five patients managed with exclusion and diversion, only one
lived.85 In summary, if primary esophageal repair or resection is
impossible, consideration can be given to wide mediastinal drainage with exclusion and diversion (cervical esophagostomy, decompressive gastrostomy, feeding jejunostomy, planned reconstruction
in a delayed fashion) or the creation of a controlled esophagocutaneous fistula using the T-tube technique described by Abbott.
Summary: There may be a limited role for a diversion/
exclusion procedure in the setting of significant esophageal
necrosis in a patient with severe physiologic derangements. (Grade
C recommendation.)
6. Is there a role for esophagectomy for esophageal perforation?
Esophagectomy has been advocated in the setting of perforation
when the native esophagus is either involved with malignancy
or nonfunctional. Sarr and colleagues reported their experience
with perforation after instrumentation of the esophagus with a
50% mortality in patients treated with esophagectomy.31 Most case
series support the use of esophagectomy in the setting of malignant perforation, perforation in the setting of severe intrinsic
esophageal disease such as achalasia or stricture, and then only if

PMPH_CH03.indd 30

the patients current physiologic state will tolerate such an extensive procedure.26 A more compelling argument for extirpation of
the esophagus can be made in the setting of delayed treatment
of the rupture.5,86 Salo and colleagues reported a 22-year experience with 90 patients, focusing on the 34 with diagnosis made
over 24 hours after perforation.86 The series included 18 patients
with spontaneous perforation. Nineteen patients had primary
repair with only six survivors (68% mortality) whereas only two
of the fifteen patients undergoing esophagectomy died (mortality of 13%). The mortality in the primary repair subgroup was
attributed to the high rate of suture dehiscence and subsequent
esophagopleural fistula (the two deaths in the resection group
were due to myocardial infarction). The mortality difference was
statistically significant between the two treatment arms (P = .001).
Gupta and Kaman reported their experience with 11 malignant
esophageal perforations after these patients underwent transhiatal resection with only one perioperative mortality.87 In summary, esophagectomy can be performed with favorable outcomes
in the appropriate patient and should be considered in the setting
of marked mediastinal/esophageal necrosis, malignancy, or diffuse/end-stage esophageal disease.86,88
Summary: Esophagectomy is appropriate for the management of esophageal perforation in the setting of malignancy or
end-stage intrinsic esophageal disease. There is no evidence to
support esophagectomy over any other intervention. (Grade C
recommendations.)
7. What is the role of endoluminal therapy for esophageal
perforation?
Endoluminal therapies for esophageal perforation have emerged
for use primarily in patients suffering perforation in the setting
of esophageal malignancy. These approaches evolved as a palliative procedure for perforation of an esophageal malignancy in
the setting of unresectable disease. The most common technique
employed has been the use of the self-expanding metallic stent
(SEMS). Ferri et al. described two cases of spontaneous malignant
esophageal perforation with successful coverage and healing using
SEMS.89 Fischer described a series of 15 patients with esophageal
perforation treated with SEMS and reported excellent outcomes
even in the setting of delayed presentation.90 Most recently, Freeman et al. reported 19 patients with spontaneous esophageal perforation treated with stent placement.91 Only two patients required
operative intervention for a persistent leak. Endoscopic clip application is another minimally invasive technique that has been used
for the management of esophageal perforation. Qadeer and colleagues reported a single case along with a pooled analysis of cases
treated with endoscopic clip application.92 The report included
17 additional subjects with various etiologies of perforation.
Although the size of perforation ranged from 3 to 25 mm, a total of
only six adjunct procedures were required to achieve healing with
only three patients requiring open operative intervention. This
series included two patients with Boerhaaves syndrome, one of
whom required exploration and primary closure. As endoluminal
therapies continue to evolve, these modalities may allow for more
rapid control of mediastinal contamination without the physiologic insult of operative intervention, thereby allowing quicker
recovery and potentially better outcomes.
Summary: Not enough evidence currently exists to support
the use of endoluminal techniques for the management of patients
with esophageal perforation. (Grade D recommendation.)

5/21/2012 7:48:25 PM

Esophageal Perforation

31

Clinical Question Summary


Question

Answer

1 What are the most reliable


symptoms and physical findings
for the diagnosis of esophageal
perforation?

Chest pain, fever, a prior history of emesis,


and dyspnea. Subcutaneous emphysema
may be considered a finding of delay in
presentation/diagnosis.

20, 24-31

2 What are the appropriate


diagnostic studies to evaluate
for esophageal perforation?

Contrast esophagogram,
esophagogastroduodenoscopy, or computed
tomography.

24, 29, 34, 37, 41, 42,


44, 45, 46, 47, 49,
50, 51

3 Is there a role for nonoperative


management of esophageal
perforation?

Yes, in the setting of contained perforation


with no significant systemic signs of sepsis.

11, 13, 15, 53, 54

4 Is there a role for primary repair


of intrathoracic esophageal
perforation?

Yes, including after a delay in presentation or


diagnosis. Consideration can be made for
vascularized tissue buttress of suture site
but is not mandatory.

7, 8, 11, 16, 57, 60,


64-71, 75-82

5 Is there a role for an exclusion/


diversion procedure after
esophageal perforation?

There may be a limited role for this type of


procedure in the setting of a moribund
patient with a large defect/extensive
esophageal necrosis not amenable to repair
who would not tolerate esophagectomy.

1, 11, 63, 75

6 Is there a role for


esophagectomy for
management of esophageal
perforation?

Yes, in the setting of malignancy or end-stage


intrinsic esophageal disease. Not enough
evidence exists to support esophagectomy
over other operative interventions.

5, 26, 31, 86-88

7 What is the role of endoluminal


therapy for esophageal
perforation?

There is no conclusive evidence to recommend


endoluminal techniques over other
strategies at this time.

89-92

REFERENCES
1. Abbott OA, Mansour KA, Logan WD Jr, Hatcher CR Jr, Symbas
PN. Atraumatic so-called spontaneous rupture of the esophagus.
A review of 47 personal cases with comments on a new method of
surgical therapy. J Thorac Cardiovasc Surg. 1970;59(1):67-83.
2. Curci JJ, Horman MJ. Boerhaaves syndrome: the importance of
early diagnosis and treatment. Ann Surg. 1976;183(4):401-408.
3. Derbes VJ, Mitchell RE Jr. Hermann Boerhaaves Atrocis, nec
descripti prius, morbbi historia, the first translation of the classic
case report of rupture of the esophagus, with annotations. Bull
Med Libr Assoc. 1955;43:217-240.
4. Wu JT, Mattox KL, Wall MJ Jr. Esophageal perforations: new perspectives and treatment paradigms. J Trauma. 2007;63:1173-1184.
5. Kollmar O, Lindemann W, Richter S, Steffen I, Pistorius G,
Schilling MK. Boerhaaves syndrome: primary repair vs.
esophageal resection-case reports and meta-analysis of the literature. J Gastrointest Surg. 2003;77:726-734.
6. Brinster CJ, Singhal S, Lee L, Marshall MB, Kaiser LR, Kucharczuk
JC. Evolving options in the management of esophageal perforation.
Ann Thorac Surg. 2004;77:1475-1483.
7. Keighley MRB, Girdwood RW, Ionescu MI, Wooler GH. Spontaneous rupture of the oesophagus. Avoidance of postoperative
morbidity. Br J Surg. 1972;59:649-652.
8. Bolooki H, Anderson I, Garcia-Rivera C, Jude JR. Spontaneous rupture of the esophagus: Boerhaaves syndrome. Ann Surg.
1971;174:319-324.

PMPH_CH03.indd 31

Grade of
Recommendation

References

9. Henderson JA, Peloquin AJ. Boerhaave revisited: spontaneous


esophageal perforation as a diagnostic masquerader. Am J Med.
1989;86:559-567.
10. Vial CM, Whyte RI. Boerhaaves syndrome: diagnosis and treatment. Surg Clin N Am. 2005;85:515-524.
11. Abbas G, Schuchert MJ, Pettiford BL, et al. Contemporaneous management of esophageal perforation. Surgery. 2009;146:
749-756.
12. Lawson RA, Butchart EG, Soriano A, et al. Spontaneous ruptures
of the esophagus. J R Coll Surg Edinb. 1974;19:363-369.
13. Lyons WS, Seremetis MG, deGuzman VC, et al. Ruptures and
perforations of the esophagus: the case for conservative supportive management. Ann Thorac Surg. 1978;25:346-350.
14. Sawyers JL, Lance CE, Foster JH, et al. Esophageal perforation:
an increasing challenge. Ann Thorac Surg. 1975;19:233-238.
15. Cameron JL, Kieffer RF, Hendrix TR, et al. Selective nonoperative management of contained intrathoracic esophageal disruptions. Ann Thorac Surg. 1979;27:404-408.
16. Keeling WB, Miller DL, Lam GT, et al. Low mortality after treatment for esophageal perforation: a single-center experience. Ann
Thorac Surg. 2010;90:1669-1673.
17. Mackler SA. Spontaneous rupture of the esophagus. Surg Gynecol Obstet. 1952;95:344-356.
18. Sealy WC. Rupture of the esophagus. Am J Surg. 1963;105:
505-510.
19. Derbes VJ, Mitchell RE Jr. Rupture of the esophagus. Surgery.
1956;39:688-709.

5/21/2012 7:48:25 PM

32

Surgery: Evidence-Based Practice

20. Wilson RF, Sarver EJ, Arbulu A, Sukhnandan R. Spontaneous


perforation of the esophagus. Ann Thorac Surg. 1971;12:291-296.
21. Rosoff L Sr, White EJ. Perforation of the esophagus. Am J Surg.
1974;128:207-218.
22. Michel L, Grillo HC, Malt RA. Operative and nonoperative management of esophageal perforations. Ann Surg. 1981;194:57-63.
23. Youngs J, Nicoloff D. Management of esophageal perforation.
Surgery. 1969;65:264-268.
24. Pate JW, Walker WA, Cole FH Jr, et al. Spontaneous rupture of
the esophagus: a 30-year experience. Ann Thorac Surg. 1989;47:
689-692.
25. Bradley SL, Pairolero PC, Payne WS, Gracey DR. Spontaneous
rupture of the esophagus. Arch Surg. 1981;116:755-758.
26. Ajalat GM, Mulder DG. Esophageal perforations. The need for
an individualized approach. Arch Surg. 1984;119:1318-1320.
27. Wu CS, Chang KY, Kuo YC, et al. Clinical challengean experience of spontaneous transmural rupture of the oesophagus in
Taiwan. Br J Clin Pract. 1996;50:298-301.
28. Flynn AE, Verrier ED, Way LW, et al. Esophageal perforation.
Arch Surg. 1989;124:1211-1215.
29. Nesbitt JC, Sawyers JL. Surgical management of esophageal perforation. Am Surg. 1987;53:183-191.
30. Griffiths EA, Yap N, Poulter J, et al. Thirty-four cases of esophageal perforation: the experience of a district general hospital in
the UK. Dis Esophagus. 2009;22:616-625.
31. Sarr MG, Pemberton JH, Payne WS. Management of instrumental perforations of the esophagus. J Thorac Cardiovasc Surg.
1982;84:211-218.
32. Naclerio EA. The V sign in the diagnosis of spontaneous rupture
of the esophagus (an early roentgen clue). Am J Surg. 1957;93:
291-298.
33. Han SY, McElvein RB, Aldrete JS, Tishler JM. Perforation of the
esophagus: correlation of site and cause with plain fi lm findings.
AJR Am J Roentgenol. 1985;145:537-540.
34. Mizutani K, Makuuchi H, Tajima T, Mitomi T. The diagnosis
and treatment of esophageal perforations resulting from nonmalignant causes. Surg Today. 1997;27:793-800.
35. Panzini L, Burrell MI, Traube M. Instrumental esophageal perforation: chest film findings. Am J Gastroenterol. 1994;89:367-370.
36. Shaker H, Elsayed H, Whittle I, et al. The influence of the golden
24-h rule on the prognosis of oesophageal perforation in the
modern era. Eur J Cardiothorac Surg. 2010;38:216-222.
37. Foley MJ, Ghahremani GG, Rogers LF. Reappraisal of contrast
media used to detect upper gastrointestinal perforations: comparison of ionic water-soluble media with barium sulfate. Radiology. 1982;144:231-237.
38. Chiu CL, Gambach RR. Hypaque pulmonary edema: a case
report. Radiology. 1974;111:91-92.
39. Reich SB. Production of pulmonary edema by aspiration of
water-soluble nonabsorbable contrast media. Radiology. 1969;92:
367-370.
40. Dunbar JS, Skinner GB, Wortzman G, et al. An investigation
of the effects of opaque media on the lungs with comparison of
barium, Lipiodol and Dionosil. AJR Am J Roentgenol. 1959;82:
902-926.
41. Dodds WJ, Stewart ET, Vlymen WJ. Appropriate contrast media
for evaluation of esophageal disruption. Radiology. 1982;144:
439-441.
42. Buecker A, Wein BB, Neuerburg JM, et al. Esophageal perforation: comparison of use of aqueous and barium-containing contrast media. Radiology. 1997;202:683-686.
43. Rubesin SE, Levine MS. Radiologic diagnosis of gastrointestinal
perforation. Radiol Clin N Am. 2003;41:1095-1115.

PMPH_CH03.indd 32

44. Wilde PH, Mullany CJ. Oesophageal perforationa review of 37


cases. Aust N Z J Surg. 1987;57:743-747.
45. Larsen K, Skov Jensen B, Axelsen F. Perforation and rupture of
the esophagus. Scand J Thorac Cardiovasc Surg. 1983;17:311-316.
46. Ghanem N, Altehoefer C, Springer O, et al. Radiological findings
in Boerhaaves syndrome. Emerg Radiol. 2003;10:8-13.
47. Horowitz B, Krevsky B, Buckman RF Jr, et al. Endoscopic evaluation of penetrating esophageal injuries. Am J Gastroenterol.
1993;88:1249-1253.
48. Pasricha PJ, Fleischer DE, Kalloo AN. Endoscopic perforation of
the upper digestive tract: a review of their pathogenesis, prevention, and management. Gastroenterology. 1994;106:787-802.
49. Di Maggio EM, Preda L, La Fianza A, et al. Spontaneous rupture
of the esophagus (Boerhaave syndrome): computerized diagnosis in atypical clinical presentation. Radiol Med. 1997;94:52-57.
50. White CS, Templeton PA, Attar S. Esophageal perforation: CT
findings. AJR Am J Roentgenol. 1993;160:767-770.
51. Jaworski A, Fischer R, Lippmann M. Boerhaaves syndrome:
computed tomographic findings and diagnostic considerations.
Arch Intern Med. 1988;148:223-234.
52. Mengoli LR, Klassen KP. Conservative management of esophageal perforation. AMA Arch Surg. 91;1965:238-240.
53. Sutcliffe RP, Forshaw MJ, Datta G, et al. Surgical management
of Boerhaaves syndrome in a tertiary oesophagogastric centre.
Ann R Coll Surg Engl. 2009;91:374-380.
54. Vogel SB, Rout WR, Martin TD, Abbitt PL. Esophageal perforation in adults: aggressive, conservative treatment lowers morbidity and mortality. Ann Surg. 2005;241:1016-1023.
55. Barrett NR. Report of a case of spontaneous perforation of the
oesophagus successfully treated by operation. Br J Surg. 1947;35:
216-218.
56. Olsen AM, Clagett OT. Spontaneous rupture of the esophagus:
report of a case with immediate diagnosis and successful surgical repair. Postgrad Med J. 1947;2:417-421.
57. Lawrence DR, Ohri SK, Moxon RE, et al. Primary esophageal
repair for Boerhaaves syndrome. Ann Thorac Surg. 1999;67:
818-820.
58. Whyte RI, Iannettoni MD, Orringer MB. Intrathoracic esophageal perforation. The merit of primary repair. J Thorac Cardiovasc Surg. 1995;109:140-146.
59. Vallbohmer D, Holscher AH, Holscher M, et al. Options in the
management of esophageal perforation: analysis over a 12-year
period. Dis Esophagus. 2010;23:185-190.
60. Foster JH, Jolly PC, Sawyers JL, Daniel RA. Esophageal perforation: diagnosis and treatment. Ann Surg. 1965;161:701-709.
61. Hardy JD, Tompkins WC Jr, Ching EC, Chavez CM. Esophageal
perforations and fistulas: review of 36 cases with operative closure of four chronic fistulas. Ann Surg. 1973;177:788-797.
62. Grillo HC, Wilkins EW Jr. Esophageal repair following late
diagnosis of intrathoracic perforation. Ann Thorac Surg. 1975;20:
387-399.
63. Urschel HC Jr, Razzuk MA, Wood RE, et al. Improved management of esophageal perforation: exclusion and diversion in continuity. Ann Surg. 1974;179:587-591.
64. Neel D, Davis EG, Farmer R, Richardson JD. Aggressive operative treatment for emetogenic rupture yields superior results. Am
Surg. 2010;76:865-868.
65. Wang Y, Zhang R, Zhou Y, et al. Our experience on management
of Boerhaaves syndrome with late presentation. Dis Esophagus.
2009;22:62-67.
66. Wang N, Razzouk AJ, Safavi A, et al. Delayed primary repair of
intrathoracic esophageal perforation: is it safe? J Thorac Cardiovasc Surg. 1996;111:114-121.

5/21/2012 7:48:25 PM

Esophageal Perforation

67. Cho S, Jheon S, Ryu KM, Lee EB. Primary esophageal repair in
Boerhaaves syndrome. Dis Esophagus. 2008;21:660-663.
68. Chao YK, Liu YH, Ko PJ, et al. Treatment of esophageal perforation in a referral center in Taiwan. Surg Today. 2005;35:
828-832.
69. Jougon J, Mc Bride T, Delcambre F, et al. Primary esophageal
repair for Boerhaaves syndrome whatever the free interval
between perforation and treatment. Eur J Cardiothorac Surg.
2004;25:475-479.
70. Keighley MR, Girdwood RW, Wooler GH, Ionescu MI. Morbidity and mortality of oesophageal perforation. Thorax. 1972;27:
353-358.
71. Symbas PN, Hatcher CR Jr, Harlaftis N. Spontaneous rupture of
the esophagus. Ann Surg. 1978;187:634-640.
72. Berne CJ, Shader AE, Doty DB. Treatment of effort rupture of
the esophagus by epigastric celiotomy. Surg Gynecol Obstet.
1969;129:277-280.
73. Landen S, El Nakadi I. Minimally invasive approach to Boerhaaves syndrome: a pilot study of three cases. Surg Endosc.
2002;16:1354-1357.
74. Toelen C, Hendrickx L, Van Hee R. Laparoscopic treatment of
Boerhaaves syndrome: a case report and review of the literature.
Acta Chir Belg. 2007;107:402-404.
75. Wu JT, Mattox KL, Wall MJ. Esophageal perforations: new perspectives and treatment. J Trauma. 2007;63:1173-1184.
76. Goldstein LA, Thompson WR. Esophageal perforations: a 15
year experience. Am J Surg. 1982;143:495-503.
77. Bardaxoglou E, Manganas D, Meunier B, et al. New approach
to surgical management of early esophageal thoracic perforation: primary suture repair reinforced with absorbable mesh and
fibrin glue. World J Surg. 1997;21:618-621.
78. Erdogan A, Gurses G, Keskin H, Demircan A. The sealing effect
of a fibrin tissue patch on the esophageal perforation area in primary repair. World J Surg. 2007;31:2199-2203.
79. Richardson JD, Tobin GR. Closure of esophageal defects with
muscle flaps. Arch Surg. 1994;129:541-548.

PMPH_CH03.indd 33

33

80. Gouge TH, Depan HJ, Spencer FC. Experience with the Grillo
pleural wrap procedure in 18 patients with perforation of the
thoracic esophagus. Ann Surg. 1989;209:612-617.
81. Wright CD, Mathisen DJ, Wain JC, et al. Reinforced primary
repair of thoracic esophageal perforation. Ann Thorac Surg. 1995;
60:245-248.
82. Kotsis L, Kostic S, Zubovits K. Multimodality treatment of
esophageal disruptions. Chest. 1997;112:1304-1309.
83. Shor-Pinsky E, Silva-Cuevas A, Franco-Vasquez R, et al. Gastrotomy with double gastrostomy in the perforation of the esophagus. Arch Surg. 1970;101:433-435.
84. Menguy R. Near-total esophageal exclusion by cervical esophagostomy and tube gastrostomy in the management of massive
esophageal perforation. Ann Surg. 1971;173:613-616.
85. Attar S, Hankins JR, Suter CM, et al. Esophageal perforation: a
therapeutic challenge. Ann Thorac Surg. 1990;50:45-49.
86. Salo JA, Isolauri JO, Heikkila LJ, et al. Management of delayed
esophageal perforation with mediastinal sepsis. Esophagectomy
or primary repair? J Thorac Cardiovasc Surg. 1993;106:1088-1091.
87. Gupta NM, Kaman L. Personal management of 57 consecutive
patients with esophageal perforation. Am J Surg. 2004;187:58-63.
88. Altorjay A, Kiss J Voros A, Sziranyi E. The role of esophagectomy
in the management of esophageal perforations. Ann Thorac Surg.
1998;65:1433-1436.
89. Ferri L, Lee JK, Law S, et al. Management of spontaneous perforation of esophageal cancer with covered self expanding metallic
stents. Dis Esophagus. 2005;18:67-69.
90. Fischer A, Thomusch O, Benz S, et al. Nonoperative treatment of
15 benign esophageal perforations with self-expandable covered
metal stents. Ann Thorac Surg. 2008;81:467-472.
91. Freeman RK, Van Woerkom JM, Vyverberg A, Ascioti AJ.
Esophageal stent placement for the treatment of spontaneous
esophageal perforations. Ann Thorac Surg. 2009;88:194-198.
92. Qadeer MA, Dumot JA, Vargo JJ, et al. Endoscopic clips for closing esophageal perforations: case report and pooled analysis.
Gastrointest Endosc. 2007;66:605-611.

5/21/2012 7:48:25 PM

Commentaries on
Esophageal Perforation
Scott B. Johnson

The chapter entitled Esophageal Perforation by Drs Lundy and


Clofton provides an excellent overview and review of the diagnostic modalities and treatment strategies for esophageal perforation
as well as the level of evidence supporting their use. They specifically address symptoms and physical findings, diagnostic studies,
nonoperative as well as operative treatment strategies to include
primary repair versus exclusion/diversion versus esophagectomy,
as well as stent placement as treatment modalities.
Esophageal perforation continues to have a high morbidity
and mortality regardless of etiology. The development of uniquely
individual patient treatment strategies are necessary to affect
overall success, taking into account underlying disease process,
mode and severity of injury, degree of illness, and overall prognosis. Even though the level of evidence supporting recommendations regarding the optimal diagnostic workup and treatment
of esophageal perforation is generally inadequate to permit firm
conclusions regarding definite recommendations, the authors
have done an excellent job in reviewing the available literature
regarding the preferred strategies in evaluating and treating these
often difficult patients. Despite the lack of solid evidence-based
treatment strategies, several principles of evaluation and management have evolved that include the following:

7.

8.

9.

10.
11.

12.

1. The illness severity of the patient is important to consider,


since a patient that is in extremis and in septic shock
must be approached differently than one without systemic
symptoms.
2. Perforated malignancies dont tend to heal and therefore
resection should be strongly considered.
3. Loop cervical esophagostomy, commonly referred to in many
textbooks, is actually technically difficult (if not impossible)
to perform unless the patient has a long, thin neck.
4. Thin barium should be considered as the initial contrast
medium of choice since it has never been shown to cause
mediastinitis and is a more sensitive contrast agent than its
water soluble counterpart.
5. The decision to perform a primary repair of an esophageal
injury should be based on the quality of the tissues found at
surgery rather than on the length of time from onset of injury
to repair.
6. Esophageal reconstruction is an elective procedure, and can be
done as a delayed procedure following resection if the patient
is septic and in shock. In this case, placement of feeding tubes,

13.
14.
15.

16.

17.

esophagectomy, and end-cervical esophagostomy can be done


as a bail-out procedure.
When resecting the esophagus from the chest and planning
delayed reconstruction, staple off and resect the esophagus
as far distally near the hiatus as can be safely done but save
as much proximal esophagus as possible to bring out over
the clavicle as an end, cervical esophagostomythe longer
the proximal length the better the long-term function when
performing a delayed, substernal reconstruction.
A gastrostomy tube does NOT necessarily negate the stomachs
use for later reconstruction, so long as the gastroepiploic
artery is not injured.
Esophagoscopy and contrast esophagography are complementary, and one should not be necessarily used to the
exclusion of the other.
CT imaging can be helpful in identifying accompanying
abscesses and undrained fluid collections.
On table endoscopy performed by the operative surgeon can
be very helpful in determining surgical approach (i.e., collar
incision vs. left neck incision vs. right thoracotomy vs. left
thoracotomy vs. celiotomy).
Never prep yourself out of an operation (i.e., will an end-cervical
esophagostomy be necessary? a transhiatal esophagectomy?
feeding tubes? etc.).
The native esophagus is always preferred as the conduit of
choice . . . . save it if you can.
Exclusionary and diverting procedures (i.e., T-tubes) can be
done, but should be done only as a last resort.
If doing a primary repair, make sure the mucosal extent of the
injury is well delineated . . . . often times the muscular defect is
much smaller and the injury underestimated until the entire
mucosal injury is exposed and repaired.
The type of suture used (e.g., absorbable vs. nonabsorbable)
and the type of stitch used (e.g., running vs. interrupted) is
NOT as important as delineating and repairing the mucosal
defect and overlying muscular defect.
Tissue flaps are not necessary to use and may cause
structuring if wrapped around the circumference of the
esophagus. If used, the Latissimus Dorsi muscle flap can be
a robust, well-vascularized flap that can be brought through
a separate, small thoracotomy incision (but would need to
be spared during performance of the thoracotomy if planned
to use).

34

PMPH_CH03.indd 34

5/21/2012 7:48:25 PM

Esophageal Perforation

18. A left thoracotomy is best for approaching a distal thoracic


esophageal perforation. In addition, it can allow access to the
belly through a radial diaphragm incision if necessary, as well
as allow exposure of the stomach through the hiatus.
19. A right thoracotomy is best for approaching a proximal or
mid-thoracic esophageal perforation.
20. Esophageal stenting is a viable treatment option to seal
actively leaking esophageal perforations in poor-risk patients.
Drainage of any fluid collections or abscess is an important
adjunct.

35

In summary, the authors provide a firm knowledge base to


guide the clinician in the diagnosis and treatment of these often
difficult-to-treat patients. Although evidence-based decision
making generally lacks in guiding surgical treatment, several
Pearls of Wisdom have emerged as a result of a collective experience in treating patients with esophageal perforations. It should
be emphasized that optimal critical care of these often very sick
patients is equally important in affecting their successful outcome
as deciding their initial surgical management, but is beyond the
scope of this discussion.

Jeremy S. Juern and John A. Weigelt

Esophageal Perforation covers the topic but focuses more on Boerhaaves syndrome than necessary. Iatrogenic esophageal perforation
is the most likely etiology accounting for more than 50% of cases.
Boerhaaves perforation is the etiology in less than 10% in most series
and no more than 15%. We suggest that a general surgeon is much
more likely to be called for an iatrogenic injury than for a Boerhaaves
syndrome. This knowledge is important since the suspected etiology
of the perforation will alter the workup and management.
Although always a dramatic condition, emetogenic esophageal perforation can be subtle if the patient presents shortly after
the event. This reinforces the importance of a careful history and
physical examination in a patient presenting with chest pain. In
the case of iatrogenic injury, no physical signs may be present at
the time of the procedure. Subcutaneous emphysema will be a late
finding. Therefore, appropriate imaging tests are needed with a
high sensitivity. We agree that CT scanning is the imaging test of
choice since it can give crucial information such as the presence
of mediastinal air, mediastinal mass, small pleural eff usion, and
extravasation of oral contrast.
The Cameron paper from 1979 is from the pre-CT era and
was the first to describe criteria for nonoperative management.
Caution should be used when interpreting this paper since there
were only 8 patients, 5 of which were leaks after an esophageal
operation. In the contemporary literature on esophageal perforation, post-op leaks are usually not included. We agree with the
authors criteria for nonoperative management. Patients in whom
the esophagram shows flow of contrast out of a perforation and
then back into the esophagus without there being any pleural
contamination can be observed. A perforation above the upper
esophageal sphincter will have a higher likelihood of successful
nonoperative management since it is not under pressure and any
saliva that accumulates is swallowed down.

PMPH_CH03.indd 35

The use of endoscopic ultrasound (EUS) is increasing, both for


diagnostic and therapeutic purposes. These echoendoscopes are a
few millimeters wider than normal gastroscopes and the viewing
field is oblique. Thus there is an increased risk of perforation because
the insertion is a partially blind maneuver in addition to the added
bulk. A prospective study of 4894 patients undergoing EUS by a single endoscopist had 3 patients with cervical esophageal perforations,
a rate of 0.06%.1 All three patients were octogenarian women without cervical osteophytes, kyphosis, or dysphagia. Symptomatology
in one was chest pain, and in the others was excessive salivation
and sore throat. One had crepitus on exam. In all three, water soluble esophagram showed a leak, which was followed by CT scan. All
three underwent surgical repair via a neck incision with no mortality. It is important to note that all three perforations happened after
this endoscopists first 2500 procedures were performed.
Similar to the rise in the use of EUS is an increase in the use of
transesophageal echocardiography (TEE). Incidence of esophageal
perforation in this group of patients is 0.01%$ to 0.04%.2 Patients
undergoing TEE may be in the operating room or intensive care
unit and hence unable to make their symptoms known. The result
is a late presentation of esophageal perforation. Therefore a high
index of suspicion is needed in patients who have undergone TEE
and subsequently develop postprocedure pneumothorax, pleural
eff usion, or sepsis.
As with most surgical conditions and especially associated
with a gastrointestinal perforation, initial care begins with fluid
resuscitation. Two other principles include nothing by mouth
and broad spectrum antibiotics. Antibiotic choice should cover
the common mouth and upper GI tract organisms and take into
account time from perforation and data obtained from imaging
studies. Initial treatment with clindamycin or an aminopenicillin with -lactamase inhibitor is appropriate. Depending on the

5/21/2012 7:48:25 PM

36

Surgery: Evidence-Based Practice

course of treatment, a broader regimen or antifungal coverage


may be needed. If it is a late presentation and an abscess is present, antimicrobials are based on culture results. The microbiologic data from a recent study of late (1 week) presentations of
esophageal perforations after cervical spine surgery showed the
expected microbes: Staphylococcus (including MRSA), Streptococcus, fungi (Candida is common), and gram negatives.3 Further
therapy is determined based on location of perforation, information from CT scan, patient symptoms, patient comorbidities, and
a valid assessment of clinician and institution capabilities.
The appropriate therapy for the majority of patients whose
injury is recognized early is primary repair. This is a great option
if our goal of sewing healthy tissue to healthy tissue can be
achieved. This repair should still be covered by a vascularized flap.
Unfortunately, prospective trials documenting this best are just
not available. When healthy tissue is not present, common sense
dictates exclusion and diversion as we do for most gastrointestinal
perforations. Esophagectomy is also a possibility, but reserved for
circumstances where the esophagus is so diseased that any type
of salvage would be unwise. This could occur in cases with late
recognition, advanced malignancy, or achalasia.
Endoluminal therapies with stents and clips finishes the
authors management options. There is a recent series from a
specialized European center of 33 patients with benign perforation treated with esophageal stents.4 Three patients (9%) went on
to esophageal resection. Stent migration happened in 11 patients
(33%), and 12 patients (36%) required a total of 17 additional stents
because of leakage or stent migration. Stent extraction was also a
problem. Of the 10 stent extractions that took place at greater than
6 weeks after insertion, 50% had a complication such as bleeding,
stent fracture, and stent impaction. One patient required gastrotomy to remove the stent. The 90-day mortality was 15%. Bear in

PMPH_CH03.indd 36

mind that stenting cannot be used for the cervical esophagus. We


have been unimpressed with clips and stents and they may end up
only being an option for highly specialized centers and in patients
who otherwise would not be candidates for an operation. Endoluminal therapy is an interesting concept, but we must remember it
is a Grade D recommendation.
Recognition and diagnosis of esophageal perforation has some
common sense approaches using clinical judgment and existing
diagnostic studies. As usual for surgical conditions, our evidence
for what should be done is weak. All grades of recommendation
on this topic are C or D and the evidence level for the types of
intervention is Grade 4. Better answers may never be available.
However, the authors outline is good and allows the reader to follow a logical path for dealing with a patient who has suffered an
esophageal perforation.

REFERENCES
1. Eloubeidi MA, Tamhane A, Lopes TL, Morgan DE, Cerfolio RJ.
Cervical esophageal perforations at the time of endoscopic ultrasound: a prospective evaluation of frequency, outcomes, and
patient management. Am J Gastroenterol. 2009;104(1):53-56.
2. Hilberath JN, Oakes DA, Shernan SK, Bulwer BE, DAmbra MN,
Eltzschig HK. Safety of transesophageal echocardiography. J Am
Soc Echocardiogr. 2010;23(11):1115-1127.
3. Rueth N, Shaw D, Groth S, et al. Management of cervical esophageal injury after spinal surgery. Ann Thoracic Surg. 2010;90(4):
1128-1133.
4. van Heel NC, Haringsma J, Spaander MC, Bruno MJ, Kuipers EJ.
Short-term esophageal stenting in the management of benign
perforations. Am J Gastroenterol. 2010;105(7):1515-1520.

5/21/2012 7:48:25 PM

CHAPTER 4

Achalasia and Esophageal Spasms


Kalyana C. Nandipati and Edward Lin

Achalasia and diff use esophageal spasm (DES) are uncommon


esophageal motility disorders in the general population. The incidence of achalasia is 1 in 100,000 in the United States and the
prevalence of diff use esophageal spasm is 0.6% to 2.8% in patients
referred with chest pain.1,2

2. How can we diagnose both achalasia and DES. Is esophageal


function testing mandatory?
The commonly used investigations in the diagnosis are barium
swallow, endoscopy, esophageal manometry, and 24- or 48-hour
pH monitoring. Barium swallow is a good initial test for patients
with dysphagia. Absence of peristalsis or repetitive nonperistaltic
contractions that fail to propel the barium to the distal esophagus
and dilated esophagus are some of the common radiographic features of achalasia. The classic birds beak (dilated esophagus with
sharp tapering at the Lower Esophageal sphincter [LES]) appearance is seen late in the disease process. Sigmoid esophagus or
dilated and tortuous esophagus is also seen in advanced cases of
achalasia. The upper endoscopy is useful to differentiate between
true and pseudoachalasia. The main role of endoscopy is to exclude
malignant stricture, and biopsy can be obtained from the segment
of esophagus with any mucosal abnormality. In the diagnosis of
achalasia, endoscopy is sensitive in only up to 40% and esophagogram in up to 64% to 66%.5 Most of these features on barium swallow and endoscopy appear late in the disease process. Most of the
patients will be suffering from symptoms with subtle or no changes
in the prior investigations. Whenever available, esophageal function tests (EFTs) can help substantiate the diagnosis and initiate
management. The main advantages of manometry over radiography are the ability to perform prolonged studies and quantify subtle
motor abnormalities in various regions of the esophagus. The classic manometric findings in patients with achalasia are absence of
peristalsis in the lower segment of esophagus and failed or incomplete relaxation of LES. Several manometric variants can exist in
achalasia. One such variation is vigorous achalasia, which is defined
by the presence of normal to high-amplitude esophageal body contractions in the presence of a nonrelaxing LES. This entity is often
considered an early form of achalasia where some ganglion cells
are still present in the myenteric plexus of the esophagus. These
patients may initially respond well to botulinum injection until the
disease progresses to the more classic form. Other variants include
intact peristalsis with diminutive LES relaxation with deglutition. The introduction of high-resolution manometry (HRM) has
led to more detailed characterization of achalasia, with distinct
manometric features that are beyond the scope of this discussion.6

1. Can a diagnosis be made on clinical presentation of achalasia and diff use esophageal spasm?
Achalasia is known to involve any age group but most commonly
presents in patients between the ages of 25 and 60 years. It can be
primary or secondary. Primary achalasia is due to an idiopathic
disorder leading to degeneration of the myenteric plexus. Secondary achalasia can be caused by infection with the trypanosomal
disease known as Chagas disease. The diagnosis is often delayed
because of its insidious clinical presentation, which can be mistaken with other conditions such as reflux for years. A healthy
index of suspicion and a contrast esophagogram can be the initial
steps in the diagnosis of achalasia.
Dysphagia to solids and liquids (>90%) and regurgitation
(60%) are the two most common symptoms associated with
achalasia. A small subgroup of patients denies the presence of
dysphagia despite having radiographic and manometric features
consistent with achalasia. This may be attributed to impaired visceral sensation, the absence of primary and secondary peristalsis,
and the adaptation to chronic esophageal dilation.3,4 Other symptoms include chest pain (20%60%), cough or choking, and halitosis.4 Secondary symptoms like weight loss, fever, and respiratory
symptoms from aspiration can also coexist. Unlike achalasia,
chest pain is the most common presenting symptom in patients
with DES. DES can also be associated with dysphagia in some but
it is not as common as in achalasia, and it does not affect the general condition of the patient like achalasia.
Even with careful history and physical examination, it can
be challenging to reach a certain diagnosis (Level 2 evidence with
Class B recommendation). This is because symptoms like chest pain
and discomfort should appropriately initiate workup of more severe
conditions such as cardiovascular disease. However, the result is
frequently a delay in the diagnosis of DES that can affect the quality
of life in these patients, even after a cardiac condition is treated.
37

PMPH_CH04.indd 37

5/21/2012 8:44:41 PM

38

Surgery: Evidence-Based Practice

It is unclear whether these unique manometric features and subtypes of achalasia will eventually be useful to predict response to
surgical management. It is safe to conclude that achalasia can manifest at any point in the disease spectrum.
Key features that differentiate DES from achalasia are the
presence of some peristaltic waveforms, normal LES resting
pressure and LES relaxation on swallowing. A criterion of 30%
or more peristaltic waveforms out of 10 wet swallows has been
used to differentiate DES from vigorous achalasia. The primary
diagnostic feature of DES is the presence of simultaneous contractions of the distal esophageal smooth muscle, manifested as
synchronous pressure waves (>8 cm/s propagation) with a minimum amplitude of 30 mmHg.7,8 This should be associated with
normal relaxation of LES at the same time, which is different from
achalasia patients with high LES pressures or nonrelaxing LES.
However, the manometric characteristics of DES can often mimic
or overlap features similar to other conditions. In the diagnosis
of achalasia and DES, it is more prudent to rely on a constellation
of symptoms and tests rather than any one single test. If there is
doubt, repeating studies at a later time may identify a trend and
allow the condition to declare itself. Manometry is essential and
key in the diagnosis of both achalasia and DES (Level 1 evidence
with Class A recommendation).
3. Who should undergo surgery for achalasia and DES?
The goals of treatment for achalasia include relief from dysphagia
and improving emptying of the esophagus by disrupting the LES
muscles as well as averting long-term complications like megaesophagus. Young patients respond best to early primary surgical
management of achalasia. However, with the availability of minimally invasive surgery, laparoscopic myotomy has become the
primary treatment option for achalasia.
In contrast, medical management has been the primary treatment for DES. The myotomy is beneficial in patients with hypertensive LES and persistent symptoms despite medical management.
Patti et al. reported that dysphagia was relieved in 80% of DES
patients after thoracoscopic myotomy and in 86% of patients after
laparoscopic myotomy. Chest pain was relieved in 75% and 80%
of these patients, respectively. Regurgitation and heartburn scores
were also significantly improved after operation.9 Subsequent longterm follow-up studies reported symptomatic improvement in up to
75% of patients after long myotomy for DES.10 Although myotomy
is not the initial treatment of choice for DES, the surgery provides
the best symptomatic improvement in patients with associated
chest pain (Level 2 to 3 evidence with Class B recommendation).
4. What nonsurgical treatment options are there for those with
achalasia and DES?
Medical and endoscopic treatments are commonly used for
nonsurgical treatment of achalasia. Endoscopic options include
pneumatic dilatation (PD) and botox injection. Recent experimental work has been described for endoscopic myotomy as a
treatment for achalasia.

MEDICAL MANAGEMENT
Medical treatment is currently used in patients who are not suitable
for either surgical or endoscopic treatments. Nitrites, nifedipine
(calcium channel blocker), and sildenafi l (phosphodiesterase

PMPH_CH04.indd 38

inhibitor) are the commonly used drugs for medical treatment of


achalasia and DES. Pharmacological agents induce smooth muscle relaxation intended to decrease LES pressure. Nitrites were the
first to be used more than six decades ago, but systemic vasodilatory effects and intolerable headaches limited their utility. Nifedipine has a better side-effect profi le when compared to nitrates.
Nifedipine is reported to decrease the LES pressure by 30% to
40% in the literature. However, placebo controlled trials reported
little clinical benefit.11 Sildenafil is a phosphodiesterase inhibitor
that is reported to decrease the LES pressure. The desired effect of
sildenafi l was short-lived, and no long-term outcome studies have
been performed to date.12 Medical treatment is associated with
certain side effects and only modest clinical benefit, so their utility is limited to patients who are not suitable candidates for either
endoscopic or surgical intervention.

ENDOSCOPIC MANAGEMENT
Botulinum Toxin
Botulinum toxin (BT, Botox) is a potent inhibitor of acetylcholine
release from the nerve endings. It acts by inhibiting unopposed acetylcholine stimulation at the LES to lower baseline sphincter pressure. A placebo controlled trial reported symptomatic improvement
in 82% of patients who received BT injection compared with 10% of
those who received placebo.13,14 On long-term follow-up, the authors
reported that age greater than 50 years and the presence of vigorous achalasia (esophageal body contractile amplitudes in excess of
40 mm Hg) are the two positive predictors of response to BT injection.15 Since the initial reports, several studies have been published
in the literature with variable response. The average response rate
with BT at 1 month was 78% (range, 63% to 90%), which later drops
to 58% (range, 25% to 78%) at 6 months and 49% (range, 15% to
64%) by 12 months.16-19 Randomized controlled trials comparing
BT with Heller myotomy reported significantly higher success rate
(34% vs. 87.5%) with myotomy after a 2-year follow-up.20 Studies also
reported that most of the patients required repeated injection and
the response to repeated injections decreases with each treatment.
Repeat injections have been associated with increased incidence of
fibrosis at the gastroesophageal (GE) junction. Surgical intervention after repeated injections is associated with higher incidence of
mucosal injury.21 The adverse effects of BT include mild transient
chest pain and heartburn in up to 5% to 10% of patients.
The current role of BT is reserved for patients who are not
suitable for more invasive treatments with pneumatic dilation or
surgical myotomy. It can also be useful in patients with recurrence
after myotomy or dilatation.

Pneumatic Dilatation
Pneumatic dilatation (PD) is the most commonly used endoscopic
treatment option with reported success rates of 53% to 100%.
Randomized controlled trials comparing PD to botox reported
a significant better response rates with PD that range from 60%
to 95% compared to that of botox (26%45%).22,23 Dilation can be
performed with endoscopy or under fluoroscopic guidance. The
Rigiflex pneumatic dilator (Boston Scientific, Boston, MA) is the
most widely used system for achalasia. The balloon is placed at
the GE junction and inflated to 8 to 15 pounds per square inch (psi)
and held in place for 15 to 60 s. These balloon dilators expand to

5/21/2012 8:44:42 PM

Achalasia and Esophageal Spasms

a 30- or 40-mm diameter. Some perform repeated dilatation over


a period of several weeks to achieve a final resting manometric
pressure below 10 to 15 mm Hg. Studies reported that more than a
third of patients do require repeated dilatations for symptomatic
recurrence over a period of 5 years. Young patients (<40 years),
male sex, and posttreatment LES pressure > 10 to 15 mmHg portend higher incidence of recurrence after PD. This patient population responds better with surgical management.
PD is associated with significant complications like perforation and reflux. The incidence of perforation is reported from 1%
to 8% with a mean of 2.8%. The incidence of perforation increases
in patients with previous botulinum injections, prior myotomy,
and in patients with vigorous achalasia. GE reflux is another significant sequelae of pneumatic dilation. Prospective studies using
pH monitoring have diagnosed significant acid reflux in 25% to
35% of patients following dilitation.
PD might be a reasonable option for older patients and
patients with recurrence after myotomy. It may be an alternative treatment option to myotomy in selected patient population
(Level 1 evidence with Class A recommendation).
5. What is the best surgical approach and technique for Heller
cardiomyotomy?
In 1913, Heller first performed the myotomy through an open
abdominal approach. Over the next several decades, after several
modifications in the technique, Hellers myotomy has become
the treatment of choice for achalasia. In his initial reports, he
performed an anterior and posterior myotomy through a laparotomy. Ellis et al. popularized the thoracic approach, in which
distal esophagomyotomy was performed through a left thoracotomy with good success rate (6094%).24,25 However, the inability to extend the myotomy sufficiently onto the stomach, leaving
subhiatal LES intact has been associated with higher recurrence
rates. Therefore, a properly performed thoracic cardiomyotomy
requires delivering the gastric cardia into the chest to complete
the distal extent of the myotomy. The open thoracic approach is
also associated with higher postoperative morbidity. These limitations make the open thoracotomy a less favorable primary surgical
approach. However, it is still useful to have this approach in the
armamentarium of esophageal surgeons, especially for patients
with extensive abdominal surgical history or recurrence after an
abdominal approach. In early 1990s, introduction of the laparoscopic approach has revolutionized the treatment of achalasia, and
it rapidly became the treatment of choice. The major advantages of
the laparoscopic approach are elimination of a thoracotomy, faster
recovery, and ability to extend the myotomy adequately onto the
stomach. Long-term studies with the follow-up of 10 years or more
reported success rate ranging from 87% to 93%.26 Studies comparing myotomy with standard endoscopic treatment options report
significantly better long-term success rate with myotomy. Randomized controlled trails reported a significant higher response
rate ranging from 85% to 95% with myotomy compared to the
PD.27,28 (Level 1 evidence with Class A recommendation).

SURGICAL TECHNIQUE
Preoperative Preparation
Presurgical preparations are standard for all patients undergoing foregut surgery. One area that is often overlooked is the need

PMPH_CH04.indd 39

39

for nutritional optimization in patients who are debilitated or


malnourished. For example, patients with vitamin K depletion
may have more intraoperative bleeding that may hinder visualization during the myotomy. The patient is positioned supine on
the table either with a split leg or straight table. During induction of general anesthesia, care is taken to prevent aspiration.
Whether a Foley catheter is inserted is the surgeons preference.
An atraumatic liver retractor for the left lobe and the reverse
Trendlenburg position is often useful to gain exposure under the
diaphragm. A second generation cephalosporin is given before
the procedure. In addition to standard laparoscopic equipment,
atraumatic gaspers and a mechanical camera holder are helpful
adjuncts to surgery.

Trocar Placement
Trocar placement is variable from surgeon to surgeon. However,
trocar placement should maximize visualization and permit
adequate extension of the myotomy both cephalad and caudally.
We place the camera trocar in the mid epigastrium, with the two
operating trocars on each side of the subcostal margin in order to
form a baseball diamondthe home base being the GE junction
and second base being the camera trocar. An assistant trocar is
placed laterally along the left subcostal margin.

Mobilization of Stomach and Esophagus


The short gastric vessels are divided toward the angle of His
beginning at upper third or half of the greater curvature. Th is
can be accomplished most expeditiously with ultrasonic shears or
bipolar cautery. Once the angle of His is mobilized completely,
we divide the gastrohepatic ligament. The pillar of the right crus
is exposed and a tunnel is made posterior to the GE junction in
order to encircle the entire GE junction with a vessel loop or penrose drain. The penrose drain is used to manipulate the esophagus
without directly handling the esophagus with graspers. We mobilize phrenoesophageal membrane with blunt dissection. Following the avascular mediastinal plane, the esophagus is mobilized
circumferentially. The anterior and posterior vagus are identified
and preserved at this point.

Myotomy
The myotomy itself can be made with blunt dissection, low energy
cautery attached to scissors, a hook cautery, ultrasonic shears, or
bipolar device. There is no need to employ multiple devices for a
myotomy and each surgeon uses the instrument that portends
the best outcomes in his or her hands. We use cautery to superficially mark our myotomy path and blunt dissection to perform
the myotomy from the GE junction in a cephalad direction to
extend at least 6 to 9 cm above the squamocolumnar junction
(Z-line). The myotomy starts immediately above the GE junction on the esophagus because the mucosal layer is easiest to
identify at this level. To disrupt the muscle fibers, graspers are
used to gently elevate the esophageal muscle fibers off the bulging mucosa beneath. Once the cephalad dissection is completed,
we start our dissection toward squamocolumnar junction and
extend it approximately 3 cm onto the stomach. Oelschlager
and colleagues reported that extending 3 cm onto the stomach
instead of 1.5 cm reduces the lower esophageal sphincter pressure

5/21/2012 8:44:42 PM

40

Surgery: Evidence-Based Practice

and symptom recurrence without any increase in the incidence of


reflux.29 If an energy source is used to perform the myotomy, caution should be taken to avoid any energy contact with the mucosal aspect of the anterior esophagus or allowing heat to transmit
from a blood vessel down to the mucosal surface. Burn necrosis
may not be apparent at the time of surgery and can manifest as a
perforation 24 to 48 hours later.
Intraoperative endoscopy and manometry are the two different techniques reported to be useful to assess the adequacy of a
myotomy. The endoscopic view can identify the squamocolumnar
junction precisely and is useful to assess the proximal and distal
extent of the myotomy. During insufflations, the mucosa is distended and any remaining circular fibers appearing as a waist
formation should be divided. We simply require the lower esophageal sphincter to open easily with endoscopic air insufflations to
be satisfied with the myotomy. If the GE junction fails to open
with air insufflations, the myotomy should be further extended
onto the stomach.30,31
Intraoperative manometry theoretically offers more functional assessment of the GE junction and was initially used to
assess the adequacy of a fundoplication after myotomy. The
manometry probe is positioned before the operation and the position is confirmed by doing intraoperative endoscopy. At the end
of the procedure, residual high-pressure zones are identified and
divided further.32-34 Initial studies were encouraging and showed
that up to 44% to 50% of patients required further myotomy on
the basis of manometry findings. However, there was increased
risk of mucosal injury and one series reported perforation in
three of five patients.35 Chapman et al. suggested that intraoperative manometry is more useful in the initial part of the surgeons
experience.36 Studies so far reported that the incidence of postoperative dysphagia is comparable to the general literature without

manometry, raising questions regarding its usefulness. Although


intraoperative manometry is useful for research, we believe this
modality is cumbersome to perform, requires an additional setup
of delicate instruments in the operating room, and is time consuming compared to endoscopy that has the added advantage of
identifying leaks.
When a rent is identified at the site of the myotomy, repair is
performed with a fine absorbable suture in figure-of-eight fashion. Biologic glues have been used to further buttress a repair but
the efficacies of adhesives are unknown.
With endoscopy, the GE junction is immersed under saline
irrigation and checked for air leaks with insufflation. Endoscopically, we look for bleeding or mucosal abnormalities that may suggest mucosal injury. We close the hiatal opening posteriorly with
0-silk sutures and bioabsorbable pledgets, but other nonabsorbable sutures can also be used. The hiatus is intentionally left wider
than for antireflux surgery to minimize encroachment onto the
dilated esophagus above.

Fundoplication
The literature has divergent views regarding the need for fundoplication after myotomy as well as the type of fundoplication. The
only randomized controlled trial reported that pathologic reflux
disease occurred in 10 of 21 patients (47.6%) after laparoscopic
Heller myotomy alone, but only 2 of 22 patients (9.1%) after Heller with a Dor fundoplication (P = .005).37 The initial experience
from our institution also revealed that regurgitation is relieved in
95% of the patients with a posterior partial fundoplication (i.e.,
the Toupet 270 fundoplication).38 These studies underscore the
importance of fundoplication after myotomy. The thoracic literature affirms the need for a fundoplication with the addition of

Table 4.1 Showing Results of Fundoplication after Myotomy


Author/Year

Type of Study

F/U
(Mo)

Type of
Fundoplication

Results/ Comments

Richards/200437

RCT

43

None (21) vs. Dor (22)

GERD47% (10/21) vs. 9.1%


(2/22) (P < .05)

Anselmino/199739

Prospective case series

38

12

Dor

5.7% (2/38) had abnormal


24-h pH study

Mattioli/2010 40

Database review

60

48

Dor (laparoscopic)

3.3% (2/60) had evidence of


esophagitis

Rebechi/200841

RCT

138

125

Dor (n = 71) vs. Nissen


(n = 67)

GERD2.8% vs. 0% (P > .05)


Dysphagia2.8 % vs. 15%
(P < .001)

Falkenback/200342

RCT

20

40

None (n = 10) vs Nissen


(n = 10)

GERD13.1 % (4/10) vs .15%


Nissen group1 dysphagia

Oelschlager/200329

Database review

110

Standard myotomy +
Dor (n = 52) vs.
Extended myotomy +
Toupet (n = 58)

Dysphagia17 % (9/52) vs. 3%


(2/58) (P < .05)

Perrone/2004 43

Retrospective

100

26

Toupet

GERD1/100
Dysphagia4/100

Ortiz/2008 44

Retrospective

33

10 years

Toupet

75% response rate


GERD24%

RCTrandomized controlled trials; GERDgastroesophageal reflux disease.

PMPH_CH04.indd 40

5/21/2012 8:44:42 PM

Achalasia and Esophageal Spasms

41

consists of liquids advancing to soft diet for prescribed periods.


How fast diet is advanced is also variable from surgeon to surgeon, which can span from 1 to 4 weeks. We generally recommend
an upper endoscopy in 6 to 12 months to examine the esophageal mucosa, which is often difficult to do before surgery when the
lumen is filled with debris.
7. When is esophagectomy indicated for achalasia?

Figure 4.1 Myotomy with Toupet fundoplication.


the Belsey procedure after a cardiomyotomy. Currently, two types
of fundoplications are performed following a Heller myotomy:
anterior Dor fundoplication or posterior Toupet fundoplication.
The Nissen fundoplication is far less commonly performed after a
myotomy to avoid dysphagia (Table 4.1).
In a Dor fundoplication, the fundus was brought over the
myotomy and anchored to the left muscular edge of the myotomy
with 2-0 silk sutures. Then fundus is rotated over the myotomy
toward the right pillar. Two to three sutures are placed on the
right crural pillar in interrupted fashion. The uppermost suture
includes gastric fundus and the right crus. The rest of the sutures
are placed between stomach and edge of myotomy. As an additional buttress against mucosal disruptions, the Dor fundoplication is also preferable if a mucosal rent is repaired.
In a Toupet fundoplication, the fundus is delivered posteriorly around the space between the gastric cardia and the
aorta. Interrupted 2-0 silk sutures are placed from the stomach
to the muscular edge of the myotomy on the right and left side
(Figure 4.1). The anterior aspect of the myotomy is exposed.
After the fundoplication is completed, we repeat another
endoscopy to ensure the wrap is in good position and that no injuries are created during the suturing. We routinely use a pedicle of
omental flap to cover the exposed belly of the myotomy to minimize any adhesions to the liver surface and serve as an additional
buttress. However, there is no data that an omental patch leads to
better outcomes than leaving the myotomy completely exposed.
6. What is the postsurgical management for patients who have
had cardiomyotomy?

Postoperative Care
Patients typically recover in standard hospital rooms. Pain control is treated with intravenous narcotic medications as needed.
A gastrograffin swallow study can be performed on postoperative
day 1 at the surgeons discretion. The aim of the esophagogram
is primarily to ascertain contrast flow. We find that a contrast
esophagogram may not necessarily identify leaks, and reliance on
signs of early sepsis gives a better prediction of leaks. Clear liquid
diet is started on postoperative day 1. Patients are discharged on
postoperative day 1 if they tolerate clear liquids. Postoperative diet

PMPH_CH04.indd 41

Esophagectomy has been a treatment for long-term achalasia with


severely dilated esophagus also known as sigmoid esophagus.
The University of Michigan reported that 63% of patients with
failed myotomy underwent esophagectomy with good functional
results.45 However, the reported mortality after esophagectomy for
achalasia ranges from 2% to 4%.45,46 Subsequent reports showed
that myotomy relieved dysphagia in a significant portion of
patients with sigmoid esophagus, avoiding an esophagectomy.47-49
For end-stage achalasia, defined as a massively dilated and tortuous esophagus, an esophagectomy may be the only option (Level
2-4 evidence and Class B recommendation). Options for reconstruction may include a gastric pull-up into the chest, or a colonic
interposition.
8. Treatment options for patients who have failed cardiomyotomy?
Recurrent dysphagia after cardiomyotomy occurs in 10% to 20%
depending on the length of follow-up. Recurrence after myotomy is a difficult problem if there is no response to additional
dilation therapy. The causes of recurrence includes an incomplete myotomy at the time of primary surgery, adhesions, a tight
fundoplication, an esophageal diverticulum, or reflux-induced
strictures.50,51 Failure to extend the myotomy adequately on to
the stomach is the most common cause of recurrent dysphagia.
Mattioli et al. reported that at least 1.5-cm extension onto the
stomach is essential for complete relaxation of the LES.52 Later
reports from Oelschlager et al. recommended that a 3-cm extension is associated with better symptomatic outcome and lower
recurrence rate.29 Extension on the stomach is difficult due to
the fact that the plane between the submucosa and the muscle
layer is less evident and bleeding is more likely. Patients with
sigmoid esophagus or terminally dilated esophagus preoperatively also have higher recurrence of dysphagia after myotomy.
However, this should not deter the surgeons from performing a
myotomy because it is still less invasive a treatment compared to
an esophagectomy. In patients with persistent or recurrent symptoms, barium swallow, endoscopy, manometry, and pH monitoring should be repeated.
Treatment options for failed myotomy include endoscopic
dilatation, BT injection, repeat myotomy, and esophagectomy.
The decision to perform a reoperation is influenced by the etiology of recurrence, the patients general condition, and time of
recurrence. Early recurrences are better treated with endoscopic
interventions like PD or botox injection.51 PD is reported to have
success rates up to 80% in patients with recurrent dysphagia. PD
for early recurrence should be done with great caution as it has a
higher chance of perforation. Use of low-pressure balloons for initial dilatation and gradually increasing the pressure in subsequent
dilatations are the key to avoiding complication perforation. Several studies show that recurrent achalasia can be treated surgically
with redo myotomy in suitable patients with success rates ranging
from 70% to 85%.53,54

5/21/2012 8:44:42 PM

42

Surgery: Evidence-Based Practice

In patients who have failed myotomy, a final attempt at promoting esophageal emptying is to perform a longitudinal esophagogastrostomy where a linear cutting stapler is deployed intraluminally
at the angle of His, creating a wider aperture at the lower esophageal
sphincter. This is best accomplished laparoscopically by creating an
anterior gastrotomy and passing a laparoscopic cutting stapler into
the GE lumen. This is a nonphysiologic operation and will certainly
worsen reflux. Although this procedure has been described by

Heyrovsky and Barrett several decades earlier, it should be viewed as


an option of last resort. In these patients, who are frequently debilitated, a feeding tube and supportive care may be the best options.
PD is the primary and first-line treatment option for patients
with failed myotomy. Other alternative option is botox injection. Redo myotomy was also reported to have good outcome in
selected patients in experienced hands. (Level 3 to 4 evidence with
Class C recommendation.)

Clinical Question Summary


Question

Answer

1 Can diagnosis made on the


clinical presentation of achalasia
and diffuse esophageal spasm?

Clinical features are often nonspecific and


investigations are necessary for the
diagnosis.

2-3

1-5

2 How can we diagnose


both achalasia and DES. Is
esophageal function testing
mandatory?

Esophagogram, endoscopy, and


manometry are useful in the diagnosis
of achalasia. Manometry is not only
useful for the diagnosis but also guide
the treatment and follow-up.

1-3

6-8

3 Who should undergo surgery


for achalasia and DES?

Surgery has been the primary


treatment of choice for achalasia.
All symptomatic patients should be
offered surgical treatment option for
long-term benefit.

1-3

9-10

4 What nonsurgical treatment


options are there for those
with achalasia and DES?

Medical and endoscopic treatments


(botox and dilatation) are two
nonsurgical treatment options for
achalasia. Dilatation reported longterm symptomatic relief compared
to the botox injections. However,
repeated dilatations are necessary for
long-term relief.

1-2

11-23

5 What is the best surgical


approach and technique for
Heller cardiomyotomy?

Hellers myotomy has been the treatment


of choice for achalasia. Fundoplication
is reported to decrease the incidence
of reflux.

1-3

24-38

6 What is the postsurgical


management for patients who
have had cardiomyotomy?

Surveillance endoscopy to check for


mucosal abnormality.

39-44

7 When is esophagectomy
indicated for achalasia?

Esophagectomy is considered as an
option in severely dilated and endstage achalasia.

2-3

45-49

8 What are the treatment options


for patients who have failed
cardiomyotomy?

Endoscopic treatment options like


dilatation or botox injections are
main treatment options for patients
who failed or had recurrence after
myotomy. Redo myotomy is also
reported to be successful in selected
series in experienced hands.

3-4

50-54

PMPH_CH04.indd 42

Levels of
Evidence

Grade of
References
Recommendation

5/21/2012 8:44:43 PM

Achalasia and Esophageal Spasms

REFERENCES
1. Mayberry, JF. Epidemiology and demographics of achalasia.
Gastrointest Endosc Clin N Am. 2001;11(2):235-248, v.
2. Hewson EG, Sinclair JW, Dalton CB, Richter JE. Twenty-fourhour esophageal pH monitoring: the most useful test for evaluating noncardiac chest pain. Am J Med. 1991;90(5):576-583.
3. Blam ME, Delfyett W, Levine MS, Metz DC, Katzka DA. Achalasia: a disease of varied and subtle symptoms that do not correlate with radiographic findings. Am J Gastroenterol. 2002;97(8):
1916-1923.
4. Eckardt VF, Stauf B, Bernhard G. Chest pain in achalasia:
patient characteristics and clinical course. Gastroenterology.
1999;116(6):1300-1304.
5. Howard PJ, Maher L, Pryde A, Cameron EW, Heading RC. Five
year prospective study of the incidence, clinical features, and
diagnosis of achalasia in Edinburgh. Gut. 1992;33(8):1011-1015.
6. Pandolfino JE, Kwiatek MA, Nealis T, Bulsiewicz W, Post J,
Kahrilas PJ. Achalasia: a new clinically relevant classification
by high-resolution manometry. Gastroenterology. 2008;135(5):
1526-1533.
7. Grbel C, Borovicka J, Schwizer W, Fox M, Hebbard G. Diff use
esophageal spasm. Am J Gastroenterol. 2008;103(2):450-457.
8. Spechler SJ, Castell DO. Classification of oesophageal motility
abnormalities. Gut. 2001;49(1):145-151.
9. Patti MG, Gorodner MV, Galvani C, Tedesco P, Fisichella PM,
Ostroff JW, et al. Spectrum of esophageal motility disorders:
implications for diagnosis and treatment. Arch Surg. 2005;140(5):
442-448; discussion 448-449.
10. Leconte M, Douard R, Gaudric M, Dumontier I, Chaussade S,
Dousset B. Functional results after extended myotomy for diffuse oesophageal spasm. Br J Surg. 2007;94(9):1113-1118.
11. Wen ZH, Gardener E, Wang YP. Nitrates for achalasia. Cochrane
Database Syst Rev. 2002;(4):CD002299.
12. Bortolotti M, Mari C, Lopilato C, Porrazzo G, Miglioli M. Effects
of sildenafi l on esophageal motility of patients with idiopathic
achalasia. Gastroenterology. 2000;118(2):253-257.
13. Annese V, Basciani M, Perri F, Lombardi G, Frusciante V,
Simone P, et al. Controlled trial of botulinum toxin injection versus placebo and pneumatic dilation in achalasia. Gastroenterology. 1996;111(6):1418-1424.
14. Pasricha PJ, Ravich WJ, Hendrix TR, Sostre S, Jones B, Kalloo AN. Intrasphincteric botulinum toxin for the treatment of
achalasia. N Engl J Med. 1995;332(12):774-778.
15. Pasricha PJ, Rai R, Ravich WJ, Hendrix TR, Kalloo AN. Botulinum toxin for achalasia: long-term outcome and predictors of
response. Gastroenterology. 1996;110(5):1410-1415.
16. Hoogerwerf WA, Pasricha PJ. Pharmacologic therapy in treating achalasia. Gastrointest Endosc Clin N Am. 2001;11(2):
311-324, vii.
17. Hoogerwerf WA, Pasricha PJ. Achalasia: treatment options revisited. Can J Gastroenterol. 2000;14(5):406-409.
18. Mikaeli J, Fazel A, Montazeri G, Yaghoobi M, Malekzadeh R.
Randomized controlled trial comparing botulinum toxin injection to pneumatic dilatation for the treatment of achalasia. Aliment Pharmacol Ther. 2001;15(9):1389-1396.
19. Walzer N, Hirano I. Achalasia. Gastroenterol Clin North Am.
2008;37(4):807-825, viii.
20. Zaninotto G, Annese V, Costantini M, Del Genio A, Costantino
M, Epifani M, et al. Randomized controlled trial of botulinum
toxin versus laparoscopic heller myotomy for esophageal achalasia. Ann Surg. 2004;239(3):364-370.

PMPH_CH04.indd 43

43

21. Patti MG, Feo CV, Arcerito M, De Pinto M, Tamburini A, Diener


U, et al. Effects of previous treatment on results of laparoscopic
Heller myotomy for achalasia. Dig Dis Sci. 1999;44(11):2270-2276.
22. Vaezi MF, Richter JE, Wilcox CM, Schroeder PL, Birgisson S,
Slaughter RL, et al. Botulinum toxin versus pneumatic dilatation in the treatment of achalasia: a randomised trial. Gut. 1999;
44(2):231-239.
23. Leyden JE, Moss AC, MacMathuna P. Endoscopic pneumatic dilation versus botulinum toxin injection in the management of primary achalasia. Cochrane Database Syst Rev.
2006;(4):CD005046.
24. Ellis FH. Esophagomyotomy by the thoracic approach for esophageal achalasia. Hepatogastroenterology. 1991;38(6):498-501.
25. Ellis FH, Jr. Oesophagomyotomy for achalasia: a 22-year experience. Br J Surg. 1993;80(7):882-885.
26. Jeansonne LO, White BC, Pilger KE, Shane MD, Zagorski S, Davis
SS, et al. Ten-year follow-up of laparoscopic Heller myotomy for
achalasia shows durability. Surg Endosc. 2007;21(9):1498-1502.
27. Wang L, Li YM, Li L. Meta-analysis of randomized and controlled treatment trials for achalasia. Dig Dis Sci. 2009.
28. Csendes JA, Braghetto I, Henrquez, Corts C. Late results of
a prospective randomised study comparing forceful dilatation and oesophagomyotomy in patients with achalasia. Gut.
1989;30(3):299-304.
29. Oelschlager BK, Chang L, Pellegrini CA. Improved outcome
after extended gastric myotomy for achalasia. Arch Surg.
2003;138(5):490-495; discussion 495-497.
30. Donahue PE, Teresi M, Patel S, Schlesinger PK. Laparoscopic
myotomy in achalasia: intraoperative evidence for myotomy of
the gastric cardia. Dis Esophagus. 1999;12(1):30-36.
31. Decanini TC, Varela GG, Galicia JA. Laparoscopic esophagomyotomy and antireflux procedure with intraoperative manometry.
Surg Laparosc Endosc. 1996;6(5):398-402.
32. Hill JW, DeLuca SA. Achalasia. Am Fam Physician. 1988;37(3):
201-203.
33. Hill LD. Intraoperative measurement of lower esophageal
spincter pressure. J Thorac Cardiovasc Surg. 1978;75(3):378-382.
34. Hill LD, Asplund CM, Roberts PN. Intraoperative manometry:
adjunct to surgery for esophageal motility disorders. Am J Surg.
1984;147(1):171-174.
35. Tatum RP, Kahrilas PJ, Manka M, Joehl RJ. Operative manometry and endoscopy during laparoscopic Heller myotomy. An
initial experience. Surg Endosc. 1999;13(10):1015-1020.
36. Chapman JR, Joehl RJ, Murayama KM, Tatum RP, Shi G,
Hirano I, et al. Achalasia treatment: improved outcome of laparoscopic myotomy with operative manometry. Arch Surg. 2004;139(5):
508-513; discussion 513.
37. Richards WO, Torquati A, Holzman MD, Khaitan L, Byrne D,
Lutfi R, Sharp KW. Heller myotomy versus Heller myotomy
with Dor fundoplication for achalasia: a prospective randomized double-blind clinical trial. Ann Surg. 2004;240(3):405-412;
discussion 412-415.
38. Hunter JG, Trus TL, Branum GD, Waring JP. Laparoscopic
Heller myotomy and fundoplication for achalasia. Ann Surg.
1997;225(6):655-664; discussion 664-665.
39. Anselmino M, Zaninotto G, Costantini M, Rossi M, Boccu C,
Molena D, et al. One-year follow-up after laparoscopic Heller-Dor
operation for esophageal achalasia. Surg Endosc. 1997;11(1):3-7.
40. Mattioli S, Ruffato A, Lugaresi M, Pilotti V, Aramini B,
DOvidio F. Long-term results of the Heller-Dor operation with
intraoperative manometry for the treatment of esophageal achalasia. J Thorac Cardiovasc Surg. 2010;140(5):962-969.

5/21/2012 8:44:43 PM

44

Surgery: Evidence-Based Practice

41. Rebecchi F, Giaccone C, Farinella E, Campaci R, Morino M.


Randomized controlled trial of laparoscopic Heller myotomy
plus Dor fundoplication versus Nissen fundoplication for achalasia: long-term results. Ann Surg. 2008;248(6):1023-1030.
42. Falkenback D, Johansson J, Oberg S, Kjellin A, Wenner J,
Zilling T, et al. Hellers esophagomyotomy with or without a 360
degrees floppy Nissen fundoplication for achalasia. Long-term
results from a prospective randomized study. Dis Esophagus.
2003;16(4):284-290.
43. Perrone JM, Frisella MM, Desai KM, Soper NJ. Results of laparoscopic Heller-Toupet operation for achalasia. Surg Endosc.
2004;18(11):1565-1571.
44. Ortiz A, de Haro LF, Parrilla P, Lage A, Perez D, Munitiz V,
et al. Very long-term objective evaluation of heller myotomy plus
posterior partial fundoplication in patients with achalasia of the
cardia. Ann Surg. 2008;247(2):258-264.
45. Devaney EJ, Iannettoni MD, Orringer MB, Marshall B. Esophagectomy for achalasia: patient selection and clinical experience. Ann
Thorac Surg. 2001;72(3):854-858.
46. Ellis FH, Jr. Esophagectomy for achalasia: who, when, and how
much? Ann Thorac Surg. 1989;47(3):334-335.
47. Patti MG, Feo CV, Diener U, Tamburini A, Arcerito M, Safadi
B, et al. Laparoscopic Heller myotomy relieves dysphagia in
achalasia when the esophagus is dilated. Surg Endosc. 1999;13(9):
843-847.

PMPH_CH04.indd 44

48. Mineo TC, Ambrogi V. Long-term results and quality of life after
surgery for oesophageal achalasia: one surgeons experience. Eur
J Cardiothorac Surg. 2004;25(6):1089-1096.
49. Eldaif SM, Mutrie CJ, Rutledge WC, Lin E, Force SD, Miller JI,
Jr, et al. The risk of esophageal resection after esophagomyotomy
for achalasia. Ann Thorac Surg. 2009;87(5):1558-1562; discussion
1562-1563.
50. Ellis FH, Jr. Failure after esophagomyotomy for esophageal motor
disorders. Causes, prevention, and management. Chest Surg Clin
N Am. 1997;7(3):477-487; discussion 488.
51. Zaninotto G, Costantini M, Portale, G, ZBattaglia G, Molena D,
Carta A, et al. Etiology, diagnosis, and treatment of failures
after laparoscopic Heller myotomy for achalasia. Ann Surg.
2002;235(2):186-192.
52. Mattioli S, Pilotti V, Felice V, Di Simone MP, DOvidio F, Gozzetti G. Intraoperative study on the relationship between the
lower esophageal sphincter pressure and the muscular components of the gastro-esophageal junction in achalasic patients.
Ann Surg. 1993;218(5):635-639.
53. Wang L, Li YM, Li L, Yu CH. A systematic review and metaanalysis of the Chinese literature for the treatment of achalasia.
World J Gastroenterol. 2008;14(38):5900-5906.
54. Iqbal A, Tierney B, Haider M, Salinas VK, Karu A, Turaqa KK,
et al. Laparoscopic re-operation for failed Heller myotomy. Dis
Esophagus. 2006;19(3):193-199.

5/21/2012 8:44:43 PM

CHAPTER 5

Esophageal Diverticula
G. Travis Clifton and Jonathan B. Lundy

women) of men to women in the trials summarized in Table 5.1.5-8


Killian-Jamieson diverticula are rare lateral pharyngoesophageal
diverticula that are distinct from the more common Zenkers diverticula (ZD). The incidence is not clear, but is much less common
than ZD with only a few case reports in the literature.9-11
Pharyngoesophageal diverticula most commonly refer to ZD.
A ZD, first described in 1769 by Ludow and further characterized by Zenker in 1778, is a pseudodiverticular herniation of the
esophageal mucosa through Killeans triangle, which is a potential
area of weakness in the posterior hypopharynx.12-14 Killeans triangle is composed of the cricopharyngeus muscle inferiorly and
the inferior pharyngeal constrictor muscles laterally and superiorly.
Although there is some disagreement, the pathogenesis seems to
center on dysfunction of the cricopharyngeus, the key component
of the upper esophageal sphincter (UES), causing high intrabolus
food pressures that result in posterior herniation and progressive
enlargement of the diverticula. This is supported by findings of high
intrabolus pressures in patients with ZD; relief of symptoms and
drop in these pressures after appropriate treatment; and recurrence
or persistence of the diverticulum with persistence of high intrabolus UES pressures due to incomplete cricopharyngeal myotomy.
The cause of UES dysfunction is the subject of some controversy
with some reporting mistimed relaxation and contraction of the
UES, cricopharyngeal spasm, or cricopharyngeal hypertrophy.
There are conflicting reports of manometric findings, with some
reporting abnormalities, whereas others have found normal manometric findings; however, mistimed contraction of the cricopharyngeus can be demonstrated with videofluoroscopic studies.20-27
The assertion that the UES abnormalities are caused by refluxing
low pH gastric contents, as suggested by Hunt, has not been demonstrated in subsequent studies.1,8,28 In addition, some people may
have a genetic or anatomic predisposition to developing a ZD, as an
autopsy study demonstrated an anatomic Killeans triangle present
in 60% (9/15) of men and 34% (11/32) of women, which may explain
the asymmetric increased prevalence in men.14
Killean-Jamieson diverticula occur through an anatomic
weakness inferior to the cricopharyngeus muscle and superiorlateral to the longitudinal muscle of the esophagus. The etiology of

BACKGROUND/OVERVIEW
Esophageal diverticula are outpouchings of the esophagus that
tend to occur in predictable patterns. Esophageal diverticula are
most frequently divided into three subgroups: pharyngoesophageal diverticula, esophageal body pulsion diverticula, and traction
diverticula. Although they are relatively uncommon, these diverticula can cause severe and even life-threatening complications.
Like much of surgery, more recent developments in the treatment
of esophageal diverticula have focused on addressing diverticula
through endoscopic or minimally invasive techniques to lessen
the morbidity (and mortality) associated with treatment in this
often elderly patient population. Controversy remains about what
the optimal surgical techniques are to address pharyngoesophageal diverticula and pulsion diverticula. Unfortunately, because
of the low incidence of these conditions, there is no randomized
data and very little controlled data for the treatment of esophageal diverticula. Fortunately, in spite of controversies, the existing literature shows that symptomatic esophageal diverticula can
be treated with a high rate of symptomatic improvement and low
morbidity and mortality.

PHARYNGOESOPHOGEAL DIVERTICULA
Overview
Pharyngoesophageal diverticula are relatively rare conditions. The
reported prevalence of Zenkers diverticula, the most common
pharyngoesophageal diverticula, is between 0.01% and 0.11% in
the general population with an incidence of 2/100,000 people per
year in the United Kingdom.1-4 The ethnic and geographic differences in pharyngoesophageal diverticula prevalence are not clear.
In all likelihood, the prevalence is increasing as life expectancies
increase and populations age, as the incidence clearly increases with
age. A typical presentation is in the seventh or eighth decades of
life.1 Zenkers pharyngoesophageal diverticula are more common
in men than women. There is a more than 2:1 ratio (416 men, 153
45

PMPH_CH05.indd 45

5/21/2012 8:28:49 PM

46

PMPH_CH05.indd 46

5/21/2012 8:28:49 PM

51
77

Stapled

19

Open

Open

28

116

Open

Stapled

181

Stapled

80

31

141

94

70

19

106

3.9% (2/51)

14% (4/28)

4.4% (8/181)

13% (1/23)

10.4%

5.8%

5.3%

3.6%

1.6%

1.1%

7.4%

10.0%

10.4%

0.0%

0.0%

0.0%

1.7%

0.0%

7.4%

5.50%

21.50%

1.70%

4.40%

3.70%

5%

4%**

6.50%**

6%

8%

7.40%

10%

Excluded diverticula <2 to 3 cm from the endoscopic arm of treatment.


Recurrence defined as persistent or recurrence of symptoms or radiographic recurrence of the diverticulum.
* Results reported as mean dysphagia score, pre- and postoperatively. Scores improved significantly in both endoscopic and open groups after surgery
without a significant difference between the groups.
** Reflects long-term dissatisfaction after recurrent or persistent symptoms had been addressed with additional procedures as indicated.
*** Recurrence rate significantly higher in after endoscopic treatment for diverticula 3 cm (36%, 9/25) compared to diverticula >3 cm (8%, 2/26), P < .05.

Rizetto, 2008 (6)***

Morse, 2006 (7),*

Bonivina, 2007 (8)

27

Open

32

40

17

48

27

41.5

n/a

n/a

1.3

1.3

2.4

2.1

n/a

n/a

12.3

5.5

5.0%

20

Wirth, 2006 (5)

Stapled

Leak Recurrence Unsatisfactory Follow-Up Time to


Hospital
Rate Rate
Results
(Months)
Resumption Stay
of PO (Days) (Days)

Author, Year (Ref) Method Number of Operative Conversion Complication


Time
to Open
Rate
Patients

Table 5.1 Single Institution Retrospective Case-Control Series Comparing Open Surgical Management and Endoscopic Stapled Diverticulotomy

Esophageal Diverticula

this diverticulum is not clear and may be congenital or an acquired


pulsion diverticulum.10,11
1. What are the symptoms associated with ZD?
The symptom most commonly experienced by patients with ZD
is dysphagia. In the series listed in Table 5.1 that reported symptom frequencies, virtually all patients experienced some degree
of dysphagia when carefully questioned.5-8 The dysphagia from
Zenkers diverticulum is thought to be due to dysfunction of the
UES, extrinsic compression of the esophagus from the diverticulum, or both. Additional symptoms associated with ZD include
regurgitation of undigested food, a feeling of a globus in the neck
(particularly with swallowing), cough, aspiration pneumonia,
and weight loss.5-8,18,20,29-34 There is suggestion in some studies that
symptoms other than dysphagia are more likely with larger diverticula.35 Often patients have coped with symptoms for years or
even decades prior to having the diagnosis established.36 Various
degrees of weight loss and cachexia, which can be substantial and
even life threatening, are often present.36 Less commonly, ZD can
present as an acute upper gastrointestinal (GI) bleed due to ulceration or esophageal varices.37 In addition, there are case reports of
squamous cell carcinoma arising in ZD, so-called Zenkers carcinoma, which is postulated to be due to chronic inflammation
from impacted food within the diverticulum.38 Killiean-Jamieson
diverticula present with symptoms similar to ZD.11
The symptoms associated with Zenkers diverticulum are
dysphagia, regurgitation of undigested food, globus sensation in
the neck (particularly with swallowing), cough, aspiration, pneumonia, and weight loss (Grade C).
2. What workup is necessary for a known or suspected ZD?
When there is concern or suspicion of a ZD based on symptoms,
the workup should include a barium swallow with or without
videofluorographic images that include lateral views. This study
both confirms the presence of the diverticulum and defines the
anatomy that may influence the surgical approach.1 Some experts
recommend upper endoscopy of the diverticulum and esophagus
due to the occasional finding of varices or carcinoma within the
diverticulum and relative frequency of other pathology within
the esophagus.1,5 Endoscopic evaluations should be performed
by an experienced endoscopist given the difficulty intubating the
diverticulum and risk of perforation.38 Upper esophageal manometry is not necessary in the evaluation of a first Zenkers diverticulum, particularly given that a variety of findings have been
demonstrated.20-27 The possible exception to this is evaluating
recurrent or persistent dysphagia after open or endoscopic cricopharyngeal myotomy that may reveal persistently elevated UES
tone or high intrabolus pressures with an incomplete myotomy.6
Zenkers diverticulum is best evaluated by videofluorographic
contrast swallow study and endoscopy (Grade C). Upper esophageal manometry may be beneficial, although there are divergent
opinions on its importance (Grade D).
3. What are the treatment options for ZD?
The natural history of asymptomatic pharyngoesophageal diverticula is unclear. ZD typically present at an advanced age in
patients who often have multiple comorbidities. It would seem
reasonable to observe a patient without symptoms after a detailed
history and physical examination but this has not been adequately
studied due to the relative infrequency of incidentally discovered

PMPH_CH05.indd 47

47

ZD.1 There are multiple procedures described for treating ZD.


These procedures include open cricopharyngeal myotomy alone;
open cricopharyngeal myotomy with diverticulopexy or diverticulectomy; rigid endoscopic stapled diverticulotomy; and flexible
endoscopic diverticulotomy though various techniques.
Open surgical management of a ZD through a cervical incision is safe and effective at relieving dysphagia associated with ZD,
according to available case series and case-control studies.5-8,35,39
The evidence suggests that performing a cricopharyngeal myotomy is a necessary portion of the operation as a diverticulectomy
without myotomy, as had been historically performed, leads to an
unacceptably high rate of recurrence of the diverticulum and or
dysphagia or persistence of dysphagia symptoms.6,35,40-43 Cricopharyngeal mytotomy alone may be sufficient in patients with smaller
diverticula (<23 cm) in relieving dysphagia.35 For larger diverticula, a diverticulopexy or diverticulectomy is necessary to eliminate accumulation of food within the diverticulum remnant.35,39
A diverticulopexy is performed by suturing the diverticulum to
the prevertebral fascia so that food does not collect into the diverticulum. Diverticulectomy can be accomplished with a stapler or
a sutured closure in layers.44
Rigid endoscopic management of ZD involves transoral visualization via a rigid endoscope (Weerda diverticuloscope) and
division of the common wall of the esophagus and diverticulum
(posterior wall of the esophagus and anterior wall of the diverticulum; diverticulotomy). In diverticula of sufficient length, division
of this wall also divides the cricopharyngeus muscle allowing for
simultaneous cricopharyngeal myotomy and creation of a common
channel to prevent accumulation of food within the diverticulum.
This technique of dividing the common wall was first described by
Mosher in 1917, although he later abandoned the technique reportedly after a patient died of mediastinitis which, before the era of
modern antibiotics, was a devastating complication.36,45 Since then,
the technique has been refined with the use of linear cutting staplers
introduced by Collard in 1993.46 Endoscopic linear cutting staplers
have the advantage of reliably sealing the divided esophageal and
diverticular walls together to decrease the risk of leakage. Others
have reported performing the endoscopic diverticulotomy with a
CO2 laser, and, more recently, harmonic scalpel.30,36,47
More recently, there have been several case series reporting flexible endoscopic diverticulotomy for the treatment of ZD.
The procedure, in principle, is similar to rigid endoscopic diverticulotomy. Using a flexible endoscope, the common wall of the
posterior esophagus and diverticulum are divided using needleknife electrocautery or argon plasma coagulation.48-51 Tang et al.
have reported their experience using endoscopic clips to seal the
divided walls.50
Treatment of Killean-Jamieson diverticula typically consists
of open surgical diverticulectomy with and without esophageal
myotomy, although one case of endoscopic distal diverticulotomy
has been described.9-11 Whichever technique is used, care must be
taken to avoid injury to the recurrent laryngeal nerve that enters
the pharynx adjacent to the base of the diverticulum.
Treatment options for ZD include open surgical management
(cricopharyngeal myotomy, with or without diverticulectomy or
diverticulopexy and rigid or flexible endoscopic diverticulotomy)
(Grade B).
4. What is the optimal intervention for ZD?
Unfortunately, there have not been randomized trials comparing the treatment of ZD using the various techniques described.

5/21/2012 8:28:49 PM

48

Surgery: Evidence-Based Practice

It is evident from the relatively robust experience reported in


the literature that treatment of ZD, despite being in an elderly
population with significant comorbidities, can be accomplished
safely with generally excellent results; for example, all series in
Table 5.1 report complete resolution of dysphagia or significant
improvement in >80%, and most >90%, of patients. In addition,
serious complication rates and recurrence rates generally remain
low, regardless of the technique used.5-8,18,20,29-36,39,40,46-50
Comparing the risks and benefits of the various treatment
methods for ZD is problematic given the current literature. First,
there is a diverse set of techniques used to treat ZD, which are being
performed at centers with various levels of experience with most
of the experience reported as case series. Next, in published casecontrol series, when two procedures are directly compared, there
is naturally some selection bias in these nonrandomized trials, and
often the selection criteria for the different procedures are not clear.
There is also discrepancy among authors regarding what constitutes a complication, as well as the definition of recurrence (recurrent dysphagia vs. radiographic recurrence of a diverticulum). In
addition, there is wide variability in the postoperative management
based on institutional or individual experience. Specifically, there is
variability in the need for a postoperative swallow study, the timing
of resumption of oral feeds, and the length of inpatient observation,
if any, needed after treatment.5-8,18,20,29-34,39-40,46-50
Regardless of the technique used, patients have a risk of leakage of enteral contents outside of the esophagus, persistence or
recurrence of symptoms, and complications of comorbidities
common in this patient population. Leakage occurs after endoscopic management from missed perforation of the diverticulum
by instruments or leakage through the sides of the diverticulotomy incision. The risk of leakage may be increased in techniques
that do not as reliably seal the sides of the diverticulotomy incision, such as needle-knife palitome or CO2 laser.36,48,52-55 The use of
a harmonic scalpel has promising initial results but needs further
validation.30,47 In open surgery, leakage typically occurs through
the staple or suture line and can lead to a salivary fistula.
Persistence of dysphagia or recurrence of the diverticulum is
often due to incomplete cricopharyngeal myotomy.6,56 This is of
particular concern in patients being considered for endoscopic
management with small diverticula. Many authors consider
a diverticulum of less than 2 to 3 cm a formal contraindication
to endoscopic management because dividing the short common
esophageal-diverticular wall may lead to an inadequate cricopharyngeal myotomy.5,8,57 This high recurrence/persistence rate is
demonstrated in the retrospective case-control series by Rizzetto
et al. and Bonavina et al. (Table 5.1), comparing open and endoscopic management and has been seen in other case series as
well.6,58 Rizzettos results demonstrated that in patients managed
with rigid endoscopic stapled diverticulotomy, there were persistent symptoms of dysphagia in 36% (9/25) patients with diverticula
<3 cm, compared to 8% (2/26) with diverticula >3 cm.6 Bonavinas
results demonstrated that 50% (4/8) of patients with diverticula
<3 cm treated with endoscopic stapled diverticulotomy were symptomatic at 5-year follow-up, compared to 6.4% (3/47) with diverticula >3 cm.8 In the other two series in Table 5.1, a diverticulum
of <2 to 3 cm was a contraindication to endoscopic management.5,7
There are reports of using stay sutures to retract the common wall
cephalad, which, in addition to making it easier to seat the stapler
in place, may make the cricopharygeal myotomy more effective in
small diverticula.7,59
Endoscopic techniques have the advantages of no incision and not requiring a neck dissection. Th is decreases the

PMPH_CH05.indd 48

risk of postoperative wound complications, neck hematoma,


salivary fistula, and laryngeal nerve injury. Th is may be particularly important in patients with a recurrent ZD after open
surgery or in patients with prior neck surgery or radiation, who
are at higher risk of these complications, particularly recurrent laryngeal nerve injury. However, surgeons performing
rigid endoscopic diverticulectomy should be proficient at open
operative management as conversion to open is not uncommon
(4%13%). 5-8,18,29,31,36,58,59,60 The reasons for converting an endoscopic case to open are most frequently inability to adequately
extend the neck, inability to adequately visualize the diverticulum or common wall between the diverticulum and the esophagus, perforation of the diverticulum by the laryngoscope or
stapler, and inability to adequately introduce the stapler into
small diverticula. 5-8,18,29,31,36,58,59,60 In addition, rigid endoscopic
techniques have a risk of dental trauma secondary to passing
instruments through the mouth.8,36,21
One of the more frequently cited benefits of endoscopic management is decreased operative time with all studies that reported
this data in Table 5.1 taking between 20 and 94 min to complete
the operation on average compared to 70 to 141 min in the open
cases (Table 5.1).5-8 In addition, most authors performing endoscopic management reported quicker resumption of oral feeds
and decreased hospital stay (and therefore decreased hospital
costs), with some performing the procedure on an outpatient
basis. However, as previously stated, there is wide variability in the
postoperative management of patients. With endoscopic diverticulotomy, most authors resume a liquid or soft diet the day of
surgery or postoperative day 1 with or without a contrasted swallow study.5,7,8,21,29,32,60,61 Most surgeons performing open cricopharyngeal myotomy with diverticulectomy wait longer (25 days)
to allow healing of the diverticulectomy suture or staple line.5,8,61
To our knowledge, there is no data to show the postoperative nilper-os period, if any, that is appropriate after diverticulectomy.
However, after open cricopharyngeal myotomy alone or with
diverticulopexy, where there is no mucosal violation, it is likely
safe to resume per-os intake shortly after surgery. In Lerut et al.s
retrospective series of 289 patients treated primarily with open
CP myotomy with diverticulopexy, patients are routinely fed after
a contrasted swallow study on postoperative day 1, and excellent
results were reported.39
Flexible endoscopic techniques have the advantages of allowing endoscopic management in patients who have inadequate
mouth opening or neck extension. In addition, they can be performed under conscious sedation.34,62 Currently, the described
techniques use eletrocautery or plasma argon to seal the divided
walls of the diverticular common wall that may have a higher
rate of leakage.36,48,52-55 There may be a role for flexible endoscopic
therapy in select patients who have a hostile neck and anatomy not
suitable for rigid endoscopic management or patients who are felt
to be unsuitable candidates for general anesthesia.62
For the treatment of ZD, cricopharyngeal myotomy (whether
open or endoscopic) decreases persistence or recurrence of
symptoms compared to open diverticulectomy alone (Grade C).
Open surgical management (cricopharyngeal myotomy with or
without diverticulectomy or diverticulopexy) and rigid endoscopic stapling both produce acceptable outcomes (Grade C).
Rigid endoscopic management may allow shorter operative
times compared to open surgical management (Grade B), and
it may allow for quicker resumption of oral feeds and shorter
length of stay compared to open diverticulectomy (Grade D).
Endoscopic diverticulectomy of small diverticula (<3 cm) may

5/21/2012 8:28:49 PM

Esophageal Diverticula

49

have higher persistence or recurrence of symptoms compared to


open surgical management (Grade C). Flexible endoscopic (nonstapled) diverticulostomy can be performed in patients who are
not candidates for rigid endoscopic diverticulostomy but may
have a higher leak rate (Grade C).

esophagopulmonary fistula.96-99 In addition, there are case reports of


squamous cell carcinoma arising within the diverticulum.100-102
Esophageal pulsion diverticula may be asymptomatic (up to
two thirds of the time). Symptoms associated with the diverticula
include chest pain, dysphagia, heartburn, regurgitation of food,
cough, aspiration, weight loss, bleeding, and obstruction (Grade C).

EPIPHRENIC DIVERTICU1A

6. What are the diagnostic tests necessary for pulsion esophageal diverticula?

Overview
Thoracic pulsion diverticula, often called epiphrenic diverticula,
are rare conditions. The true prevalence of these diverticula is
unclear as they are often asymptomatic and discovered incidentally on imaging studies. The estimated prevalence varies widely
and has been estimated between 0.015% and 2% of the population depending on the population studied.63-65 They are seen in
less than 1% of EGDs but in as many as 3% of EGDs performed for
dysphagia.66 The diverticula have peak incidence in the sixth and
seventh decade and may be more common in men.67
Thoracic pulsion diverticula typically occur within 10 cm of
the gastroesophageal junction and, thus, are commonly referred to
as epiphrenic diverticula.68,69 They can occur more proximally in
the esophagus, although this is rare.70,71 There are multiple diverticula in up to 10% to 15% of cases.72-76 It is generally believed that
an underlying esophageal motility disorder is present in the vast
majority, if not all patients, with an intrathoracic pulsion diverticulum that is not due to a prior esophageal myotomy injury, or distal
of obstruction.75,77-79 The higher intraesophageal pressures experienced proximal to an anatomic or functional (in motility disorders)
obstruction cause the wall of the esophagus to give way creating the
diverticulum that can progressively enlarge over time. Manometry
studies performed on patients with esophageal pulsion diverticula
demonstrate a motility disorder 50% to 100% of the time.65,73,75-77,80-88
More extensive studies, including 24-hour ambulatory manometry,
have been used to characterize motility disorders that are not evident on a traditional manometric swallow study.78 The motility
disorders associated with esophageal pulsion diverticula include
nonspecific disorders, diffuse esophageal spasm, hypertensive lower
esophageal sphincter (LES), and achalasia.65,75-77,80-86,88 The motility
disorders are typically associated with increased amplitude of contractions, which may explain why diverticula are seen more often in
patients with diffuse esophageal spasm (up to 42%) compared to
patients with achalasia (less than 5%).64,89-91 Causes of mechanical obstruction that have been associated with esophageal pulsion
diverticula include leiomyomas and stricture.73,84,92
5. What are the symptoms associated with esophageal pulsion
diverticula?
Esophageal diverticula are discovered incidentally in patients who
are asymptomatic or with minimal symptoms in up to two thirds of
cases.73 When diverticula are symptomatic, patients complaints are
often likely due to an underlying esophageal motility disorder and
include chest pain, dysphagia, and heartburn. Symptoms that may
be due to the diverticulum itself include regurgitation of undigested
food, particularly when lying flat, and aspiration.73,78,91,93 In symptomatic patients, various degrees of weight loss are common and
symptoms have often been present for months to years.78,91,94 Pulmonary complaints may be the sole symptom in up to 25% of patients.95
Less common findings include esophageal obstruction secondary to compression from the diverticulum, upper GI bleeding, and

PMPH_CH05.indd 49

Esophageal diverticula can be diagnosed on plain chest x-ray as a


mediastinal density with or without an air-fluid level.69 If it is diagnosed by other means, imaging of the chest is indicated to evaluate
for pulmonary disease or changes from aspiration.69,73,103 Contrasted esophagogram is the single best study to evaluate for an
esophageal diverticulum in that it reliably diagnoses the presence
of the diverticulum as well as defines the number, size, and shape
of the diverticulum and other potential esophageal pathology
including motility disorders.67,77,104 Esophageal endoscopy is recommended to further evaluate for additional esophageal pathology
and should include attempts to visualize the diverticular mucosa
when possible because of the risk of cancer.100-102,105 There are divergent opinions on the need for esophageal manometry in patients
with esophageal pulsion diverticula. Although most patients will
have abnormal findings, some consider this of academic value only
because they believe that esophageal myotomy above the level of
the diverticulum is indicated even if findings are normal.105 Some,
however, advocate performing a selected esophageal myotomy only
in patients with a demonstrated motility disorder on esophageal
manometry.71 In addition, some surgeons use pH probe findings of
reflux to determine whether an antireflux procedure is necessary,
whereas others perform antireflux procedures routinely.106,107 CT of
the chest has also been used to evaluate esophageal diverticula and
may be of value in defining the surrounding anatomy, particularly in
mid-esophageal diverticula if it is unclear if the diverticula is due
to traction or pulsion physiology.108
Contrasted esophagogram is the best study to evaluate for the
presence of, and characteristics of, esophageal diverticula. Chest
x-ray and/or CT of the chest are indicated to evaluate for signs of
aspiration or pulmonary complication. Esophageal endoscopy is
useful to assess for cancer, dysplasia, and other esophageal disease
(Grade C). Esophageal manometry is useful for defining esophageal motility disorders, although this does not alter the surgical
plan for some surgeons (Grade D). In addition, some advocate for
a 24-hour esophageal pH study to determine the presence of preoperative gastroesophageal reflux disease, which influences the
type of antireflux procedure used (Grade D).
7. When is intervention indicated in an esophageal pulsion
diverticula?
Intervention is indicated with esophageal pulsion diverticula in
patients who present with symptoms such as weight loss, pulmonary
complications, significant dysphagia, or pain. In patients with
minimal or no symptoms, most authors favor observation, as the
surgical treatment caries significant risk of morbidity and mortality. Studies following asymptomatic or minimally symptomatic
patients suggest a relatively low rate (~10%) of development of
symptoms over time although the data is not uniform.67,70,73,109 In
contrast, patients with a history of aspiration or pulmonary complaints are at high risk of aspiration and death without a timely
operation.77

5/21/2012 8:28:49 PM

50

Surgery: Evidence-Based Practice

Asymptomatic or minimally symptomatic diverticula can


be safely observed. Intervention is indicated for symptomatic
diverticula particularly in patients with pulmonary complaints
(Grade C).
8. What treatment is recommended for esophageal pulsion
diverticula?
Endoscopic treatment of patients underlying esophageal motility
disorder has been proposed in symptomatic patients who are not
operative candidates, particularly with small epiphrenic diverticula; however, there is limited outcome data to support this.110-112
Proposed treatments include endoscopic dilation and injections
of botulinum toxin (or both), which is largely extrapolated for
experience in treating esophageal motility disorders.70 In motility disorders, both of these treatments provide temporary relief
in patients with motility disorders affecting the lower esophageal
sphincter; however, the symptoms tend to recur over time. In addition, dilation carries a 4% to 6% risk of esophageal perforation,
and botulinum toxin causes localized inflammation and scarring
that increase the risk of esophageal mucosal violation should a
myotomy be performed later.95,112
Operative management generally consists of esophageal
mytotomy with or without diverticulectomy although other
approaches have been described. Myotomy is generally accepted
as a necessary component of the operation.105 Although there
is no randomized controlled data to prove this point, the published collective experience in case series suggests higher rates
of esophageal leak and recurrence of the diverticulum with
diverticulectomy without myotomy.67,82 Most authors advocate
esophageal myotomy even when no esophageal motility disorder can be demonstrated, although this is not uniform.71,113 The
proximal and distal extent of the myotomy are the source of
some controversy. Some feel the distal myotomy should extend
distally 1 to 2 cm onto the gastric cardia to ensure complete
division of the lower esophageal sphincter and, thus, preventing a functional distal high-pressure zone (whether revealed
by manometry or not).76,77,82,114,115 Others believe that leaving a
manometrically normal LES intact is safe and eliminates the
need for a fundoplication.83,86 The esophageal myotomy should
extend proximally at least to the level of the diverticular neck
to ensure adequate relief of the distal functional obstruction.
Failure to perform adequate proximal myotomy has been associated with early recurrences in case reports.116,117 Some advocate a
more extensive proximal myotomy to the level of the aortic arch
in all patients or to the level of demonstrated manometric abnormalities in patients with symptoms attributed to an esophageal
motility disorder.70,71
Diverticulectomy is likely unnecessary in patients with small,
wide-necked diverticula as they tend to resolve intraoperatively
after an esophageal myotomy is performed. Some surgeons advocate imbrication or pexis of diverticula in selected cases.76,77,79,87,115
By not performing a diverticulectomy, the risk of esophageal leak
is decreased as the esophageal mucosa is not violated. However, if
the diverticulum is large, particularly with a dependent portion, or
has dysplastic changes, a diverticulectomy is necessary.27,75,76,82,85
Most authors advocate the addition of an antireflux procedure with a partial wrap as part of the operative management when an LES myotomy is performed.67,70,71,76,91,94,105-107,118
Although the experience with wraps in patients with esophageal

PMPH_CH05.indd 50

pulsion diverticula is limited to case series, there are randomized


data in patients undergoing a Heller lower esophageal sphincter
myotomy for achalasia that demonstrate significant reduction
in postoperative reflux with the addition of a partial fundoplication (43% vs. 9%, P = .005).119 A complete (Nissen) fundoplication
may not be advisable as it has been associated with high rates of
dysphagia and progressive esophageal dialation in patients with
achalasia who have undergone esophageal myotomy.120 In addition, in patients with esophageal diverticula, there is concern
that the higher intraesophageal pressure associated with a complete wrap could cause an esophageal suture line leak or recurrence of the diverticulum.95,121 However, others advocate using
manometric data or the preoperative presence of gastroesophageal reflux disease (GERD) to determine the type of wrap to be
performed, and report good results in performing complete
(Nissen) fundoplications in patients with normal esophageal
motility or GERD.70,71,107,121
Classically, esophageal pulsion diverticula are addressed
though a left thoracotomy. There is a growing body of experience within the literature for minimal invasive management
of esophageal pulsion diverticula.70,71,91,94,107,109,118,122 This can be
accomplished through a laparoscopic transhiatal approach or
thoracoscopic approach. The laparoscopic transhiatal approach
has the advantage of excellent visualization of the hiatus facilitating performance of the esophageal myotomy and fundoplication. In addition, the transhiatal approach allows passage of an
endoscopic stapler in-line with the esophagus to perform the
diverticulectomy.91,118,122,123 However, it may be difficult to perform a more proximal esophageal myotomy in patients of the
proximal esophagus or to address mid-thoracic or large diverticula, although all have been successfully accomplished in case
reports.71,124,125 Alternatively, the thoracoscopic approach allows
for greater access to the proximal esophagus but it may be more
technically challenging to perform the myotomy and diverticulectomy given the angle of approach.118,123,126,127
The obvious advantage of the minimally invasive approaches
is avoidance of a thoracotomy in this often elderly patient population. This may decrease the morbidity and length of stay associated with repair compared to open operation. In the reported case
series of minimally invasive treatment, most patients required
inpatient admission of less than 1 week.118 The reported rates of
symptomatic improvement (83%100%), leak, and recurrence
(0%20%) appear to be similar to results in published series of
open operative management.107,118,122 However, given the technical difficulty of the operation and potential pit-falls, this should
only be attempted by surgeons with extensive laparoscopic
experience.106
Endoscopic dilation or botulinum toxin cannot be recommended for symptomatic esophageal pulsion diverticula based
on the existing literature (Grade D). Esophageal myotomy (proximally to at least above the level of the diverticulum) with or without diverticulectomy is the best operative treatment (Grade C).
A long esophageal myotomy may benefit in patients with diff use
esophageal motility disorders (Grade C). There may be a benefit to extension of the myotomy distally onto the gastric cardia
(Grade D). A partial or complete fundoplication may decrease
postoperative gastroesophageal reflux associated with a lower
esophageal myotomy (Grade C). Minimally invasive approaches
can achieve similar results as open operative techniques without
the morbidity of a laparotomy or thoracotomy (Grade C).

5/21/2012 8:28:50 PM

Esophageal Diverticula

51

Table 5.2 Evidence-Based Table Based on Intervention for Zenkers Diverticula


Year, Author,
Reference #

Number of Study Design


Patients

Findings/Recommendations

Level of
Evidence

2010, Nicholas
et al. #59

Retrospective, single-center
case series

Using an endostitch to retract the diverticulum


common wall assists with endoscopic stapled
diverticulotomy in patients with difficult anatomy.

2010, Al-Kadi
et al. #48

18

Retrospective, single-center
case series

Flexible endoscopic diverticulotomy with a needleknife papillotome is an effective approach in highly


morbid patients.

2010, Brace
et al. #60

19

Retrospective, single-center
case-control series

Endoscopic stapled diverticulotomy achieves


comparable results to open surgery with shorter
operative times and hospital stay. Methodological
flaws in patient selection.

2010, Repici
et al. #34

58

Retrospective, single-center
case-control series

Similar results between rigid endoscopic stapled


diverticulotomy and flexible needle-knife
diverticulotomy.

2010, Case
et al. #62

22

Retrospective, single-center
case series

Flexible endoscopic needle-knife diverticulotomy


is effective, often without requiring general
anesthesia. Perforation is a common complication.

2009, Sharp
et al. #30

48

Retrospective, single-center
case-control series

Endoscopic stapled and endoscopic harmonic scalpel


diverticulotomy produce similar results with a
higher complication rate for diverticula <2 cm.

2009, Fama
et al. #47

25

Retrospective, single-center
case series

Endoscopic diverticulotomy with a harmonic scalpel is


safe and effective.

2009,
Wasserzug
et al. #31

57

Retrospective, single-center
case series

Endoscopic stapled diverticulectomy is safe and


effective.

2009, Harris
et al. #29

31

Retrospective, single-center
case series

Endoscopic stapled diverticulectomy is safe and


effective. Conversion to open is required in some
patients.

128

Retrospective, single-center
case-control series

Endoscopic stapled diverticulotomy is safe and


effective in diverticula >3 cm. Open surgery has
better long-term results and is recommended in
younger, healthy patients.

Retrospective, single-center
case series

Flexible endoscopic clip-assisted diverticulotomy is


feasible, safe, and effective.

297

Retrospective, single-center
case series

Endoscopic stapled diverticulectomy results in similar


good results as open surgical management except
in patients with diverticula <3 cm.

2007, Palmer
et al. #57

35

Retrospective, single-center
case series

Endoscopic stapled diverticulectomy is safe and


effective. Repeat surgery for persistent or
recurrent dysphagia can be safely performed
endoscopically.

2006, Morse
et al. #7

47

Retrospective, single-center
case-control series

Endoscopic stapled diverticulectomy results in similar


results as open surgery with shorter operative
times.

2006, Lang
et al. #21

63

Prospective, single-center
case-control series

Endoscopic stapled diverticulotomy is safer with


shorter operative times and hospital stays than
open surgical management. Methodological flaws in
patient selection/comparison.

2006, Wirth
et al. #5

47

Retrospective, single-center
case-control series

Both endoscopic stapled diverticulotomy and open


cricopharyngeal myotomy showed good results in
long-term follow-up.

2008, Rizzetto
et al. #6

2008, Tang
et al. #50
2007, Bonavina
et al. #8

(Continued)

PMPH_CH05.indd 51

5/21/2012 8:28:50 PM

52

Surgery: Evidence-Based Practice

Table 5.2 (Continued)


Year, Author,
Reference #

Number of Study Design


Patients

Findings/Recommendations

Level of
Evidence

2006, Miller
et al. #36

40

Retrospective, single-center
case-control series

Endoscopic stapled diverticulectomy has improved


efficacy and safety compared to flexible endoscopic
CO2 laser diverticulotomy.

2006, Tsikoudas
et al. #58

21

Retrospective, single-center
case series

It is possible to predict surgical outcome based on the


radiographic characteristics of the diverticulum.

2002, Smith
et al. #61

16

Retrospective, single-center
case-control series

Endoscopic stapled diverticulectomy results in


decreased hospital charges secondary to shorter
operative time and hospital stay.

2001, Lerut
et al. #39

325

Retrospective, single-center
case series

Open cricopharyngeal myotomy and diverticulopexy


achieves excellent results with short hospital stay.

2001, Sakai
et al. #49

10

Retrospective, single-center
case series

An oblique hood on the end of a flexible


endoscope simplifies performing a needle-knife
diverticulotomy.

1999, Narne
et al. #18

102

Retrospective, single-center
case series

Endoscopic stapled diverticulectomy is safe and


effective.

1999, Osmote
et al. #33

22

Retrospective, single-center
case series

Endoscopic stapled diverticulectomy is effective with


low risk of complications and a short hospital stay.

1996, Koay
et al. #32

14

Retrospective, single-center
case series

Endoscopic stapled diverticulectomy is fast, safe, and


effective.

1990, Barthlen
et al. #35

43

Retrospective, single-center
case series

Recommends cricopharygeal myotomy based on


manometry findings and treatment results with
open surgical therapy.

Table 5.3 Evidence-Based Table Based on Intervention for Esophageal Pulsion Diverticula
Year, Author,
Reference #

Number of
Patients

Study Design

Findings/Recommendations

2009, Castrucci,
et al. #83

51

2009, Katsinelos
et al. #110

Level of
Evidence

Retrospective, singlecenter case series

Recommend tailored approach to treatment of esophageal


diverticula based on the patients underlying esophageal
motility disorder.

Retrospective, case
report

Effectively treated dysphagia in a patient with a large


epiphrenic diverticulum who was not an operative
candidate with repeat combined dilation and botulinum
toxin injection.

2009, Katsinelos
et al. #111

Retrospective, singlecenter case series

Effectively treated dysphagia in two patients with a large


epiphrenic diverticulum who was not an operative
candidate with repeat combined botulinum toxin
injections.

2008, Palanivelu
et al. #71

Retrospective, singlecenter case series

Laparoscopic transhiatal is the approach of choice.

2008, Zaninotto
et al. #109

41

Retrospective, singlecenter case series

19 patients with small, mildly symptomatic diverticula were


safely followed without surgery. Patients who were
operated on improved symptomatically.

2006, Rosati
et al. #107

11

Retrospective,
single-center casecontrol series

Laparoscopy offers good access to the distal esophagus


and inferior mediastinum. Recommend diverticulectomy
with myotomy and partial fundoplication.

2005, Fernando
et al. #106

20

Retrospective,
single-center casecontrol series

Procedures performed laparoscopically and/or with VATS.


Potential for morbidity is significant. Recommend open
surgery except in centers experienced in minimally
invasive esophageal surgery.

(Continued)

PMPH_CH05.indd 52

5/21/2012 8:28:50 PM

Esophageal Diverticula

53

Table 5.3 (Continued)


Year, Author,
Reference #

Number of
Patients

Study Design

Findings/Recommendations

Level of
Evidence

2005, Tedesco
et al. #91

21

Retrospective, singlecenter case series

A primary esophageal motility disorder is the cause of most


epiphrenic diverticula; laparoscopic treatment should be
the method of choice.

2004, del Genio


et al. #122

13

Retrospective, singlecenter case series

Laparoscopic management of epiphrenic diverticula is safe


and effective.

2003, Klaus
et al. #70

17

Retrospective, singlecenter case series

Asymptomatic patients may not require therapy.


Laparoscopic approach is the treatment of choice.
A long esophageal myotomy and fundoplication should
be performed.

Retrospective, singlecenter case series

Laparoscopic or thoracoscopic approaches are feasible,


safe, and effective at alleviating dysphagia in patients with
epiphrenic esophageal diverticula.

2002, Nehra
et al. #78

21

Retrospective, singlecenter case series

There is a high prevalence of esophageal motility disorders


with esophageal diverticula. Resection of the diverticula
and myotomy of manometrically defined abnormal
segment should be performed.

2002, Nastos
et al. #114

16

Retrospective,
single-center casecontrol series

Patients with esophageal diverticula had dysfunction


primarily of the distal esophagus and responded better
to esophageal myotomy and fundoplication than patients
with esophageal spastic disorders.

1999, Altorki
et al. #77

102

Retrospective, singlecenter case series

All patients with thoracic esophageal diverticula should


undergo operative intervention, regardless of symptoms,
because of the risk of aspiration. This assertion is based
on the high rate of pulmonary complications observed.

1999, Jordan
et al. #79

25

Retrospective, singlecenter case series

Asymptomatic epiphrenic diverticula do not require


surgery. Resection or imbrications of the diverticula
are effective. LES myotomy is contraindicated when
gastroesophageal reflux is present or LES pressure is
below normal.

1993, Benacci
et al. #73

112

Retrospective, singlecenter case series

Operation for esophageal diverticula has significant risks


and is not warranted in patients with minimal symptoms;
however, it is warranted in patients with incapacitating
symptoms.

1993, Hudspeth
et al. #87

18

Retrospective, singlecenter case series

Diverticulectomy combined with selective myotomy


permits excellent operative results.

1992, Fekete
et al. #82

33

Retrospective,
single-center casecontrol series

Recommend esophageal myotomy distally onto the gastric


cardia and addition of antireflux procedure.

1992, DUgo
et al. #85

19

Retrospective, singlecenter case series

Recommend treatment of the underlying esophageal


motility disorder in patients with esophageal pulsion
diverticula with diverticulectomy in selected cases.

1992, Streitz
et al. #86

16

Retrospective, singlecenter case series

Recommend selected esophageal myotomy of areas only


with demonstrated manometric abnormalities. Advocate
leaving manometrically normal LES intact.

1986, Evander
et al. #76

Retrospective, singlecenter case series

Good results with esophageal myotomy though the


LES with or without diverticulectomy and with
fundoplication.

46

Retrospective, singlecenter case series

Epiphrenic diverticulum is associated with esophageal


motility disorders in the majority of patients and
mechanical obstruction in the minority. Surgery should
include diverticulectomy and myotomy or resection of
mechanical obstruction.

2003, Matthews
et al. #94

1980, Debas
et al. #84

PMPH_CH05.indd 53

5/21/2012 8:28:50 PM

54

Surgery: Evidence-Based Practice

TRACTION DIVERTICULA
Traction diverticula are true diverticula caused by inflammation
of mediastinal lymph nodes. As the inflammatory process causes
scarring and retraction, it pulls the adjacent esophageal wall, forming a diverticulum. They tend to occur in the mid-thoracic esophagus and are often small and asymptomatic. They are uncommon,
particularly in Western countries. The conditions associated with

traction diverticula include granulomatous diseases such as histoplasmosis, tuberculosis, sarcoidosis although other causes have
been described.128-134 Treatment typically involves medical management of the causative inflammatory process. Surgical management
is indicated for complications associated with the diverticulum such
as esophagobronchial fistula or bleeding.44,131 Surgical treatment
typically consists of resection of the inflammatory process with
repair of the esophageal wall or resection of the diverticulum.44,131

Clinical Question Summary


Grade of
Recommendation

References

Question

Answer

1 What are the symptoms


associated with
Zenkers diverticulum?

Dysphagia, regurgitation of undigested food, a feeling of


a globus in the neck (particularly with swallowing),
cough, aspiration pneumonia, and weight loss.

5-8, 18, 20, 29-34

2 What workup is
necessary for a known
or suspected Zenkers
diverticulum?

Videofluorographic contrast swallow study; endoscopy.


upper esophageal manometry

1, 5, 38

6, 20-27

3 What are the treatment


options for Zenkers
diverticulum?

Open surgical management (cricopharyngeal myotomy,


with or without diverticulectomy or diverticulopexy.
Rigid or flexible endoscopic diverticulotomy.

5-8, 18, 20, 21, 29-36,


39-43, 52-55, 57,
60, 61

4 Which treatment is
best for Zenkers
diverticulum?

Cricopharyngeal myotomy (whether open or


endoscopic) decreases persistence or recurrence of
symptoms compared to open diverticulectomy alone.
Open (cricopharyngeal myotomy with or without
diverticulectomy or diverticulopexy) and rigid
endoscopic stapling both produce acceptable
outcomes.
Rigid endoscopic management may allow shorter
operative times compared to open surgical
management.
Rigid endoscopic management may allow for quicker
resumption of oral feeds, and shorter length of stay
compared to open diverticulectomy.
Endoscopic diverticulectomy of diverticula <3 cm may
have higher persistence or recurrence of symptoms.
Flexible endoscopic (nonstapled) diverticulotomy can
be performed in patients who are not candidates
for rigid endoscopic diverticulotomy but may have a
higher leak rate.

6, 35, 40-43

5-8, 18, 20, 29-36,


39, 40

5-8

5, 7, 8, 21, 29, 32, 60,


61

5, 8, 57

36, 48, 52-55

5 What are the symptoms


associated with
esophageal pulsion
diverticula?

May be asymptomatic (up to two thirds of the time).


Chest pain, dysphagia, heartburn, regurgitation of
food, cough, aspiration, weight loss. Less commonly,
bleeding or obstruction.

73, 78, 91, 93-99

6 What are the diagnostic


tests necessary for
pulsion esophageal
diverticula?

Contrasted esophagogram. Chest x-ray and/or CT of


the chest. Esophageal endoscopy.
Esophageal manometry.
24-hour esophageal pH study.

C
D
D

67, 69, 73, 77, 100102, 104, 105


70, 71, 83, 86, 105
106, 107

7 When is intervention
indicated in an
esophageal pulsion
diverticula?

Asymptomatic diverticula can be safely watched with


follow-up.
Intervention is indicated for symptomatic diverticula.
Patients with pulmonary complaints have a history
of aspiration.

67, 70, 73, 77

77

8 What treatment is best


for esophageal pulsion
diverticula?

Endoscopic dilation or botulinum toxin cannot be


recommended for symptomatic esophageal pulsion
diverticula based on the existing literature.

110-112

(Continued)

PMPH_CH05.indd 54

5/21/2012 8:28:50 PM

Esophageal Diverticula

55

(Continued)
Question

Answer

Grade of
Recommendation

Esophageal myotomy (proximally to at least above


the level of the diverticulum) with or without
diverticulectomy is the best operative treatment.
A long esophageal myotomy may benefit those with
diffuse involvement of esophageal motility disorders.
The myotomy should extend distally onto the gastric
cardia.
A partial or complete fundoplication may decrease
postoperative gastroesophageal reflux associated
with a lower esophageal myotomy.
Minimally invasive approaches can achieve similar
results as open operative techniques without the
morbidity of a laparotomy or thoracotomy.

REFERENCES
1. Watermberg S, Landau O, Avrahami R. Zenkers diverticulum:
reappraisal. Am J Gastroenterol. 1996;91(8):1494-1498.
2. Wheeler D. Diverticula of the foregut. Radiology 1947;49:
476-482.
3. Siewert JR, Blum AL. Divertikel. In: Siewert JR, ed. Chirurg
Gastroenterologie, vol 53. Berlin: Springer-Verlag; 1990:331-337.
4. Siddiq MA, Sood S, Strachan D. Pharyngeal pouch (Zenkers
diverticulum). Postgrad Med J. 2001;77:506-511.
5. Wirth D, Kern B, Guenin MO, Montali I, Peterli R, Ackermann
C, von Flue M. Outcome and quality of life after open surgery
versus endoscopic stapler-assisted esophagodiverticulostomy
for Zenkers diverticulum. Dis Esophagus. 2006;19(4):294-298.
6. Rizzetto C, Zaninotto G, Costantini M, et al. Zenkers diverticula:
feasibility of a tailored approach based on diverticulum size. J
Gastrointest Surg. 2008;12(12):2057-2064.
7. Morse CR, Fernando HC, Ferson PF, Landreneau RJ, Luketich
JD. Preliminary experience by a thoracic service with endoscopic
transoral stapling of cervical (Zenkers) diverticulum. J
Gastrointest Surg. 2007;11(9):1091-1094.
8. Bonavina L, Bona D, Abraham M, Saino G, Abate E. Long-term
results of endosurgical and open surgical approach for Zenker
diverticulum. World J Gastroenterol. 2007;13(18):2586-2589.
9. Rodgers PJ, Armstrong WB, Dana E. Killian-Jamieson
diverticulum: a case report and a review of the literature. Ann
Otol Rhinol Laryngol. 2000;109:1087-1091.
10. Tang SJ, Tang L, Chen E, Myers LL. Flexible endoscopic KillianJamieson diverticulotomy and literature review (with video).
Gastrointest Endosc. 2008;68(4):790-793.
11. Boisvert RD, Bethune DC, Acton D, Klassen DR. Bilateral
Killian-Jamieson diverticula: a case report and literature review.
Can J Gastroenterol. 2010;24(3):173-174.
12. Ludlow A. A case of obstructed deglutition, from a prenatural
dilatation of, and bag formed in the pharynx. Med Observ Inq.
1769;3:85.
13. Zenker TA, vonZiemssen H. Krankheiten des oesophagus. In:
von Ziemssen H. ed. Handbuch der Speciellen Pathologie und
Therapie. Vol. 7. Leipzig: FCW Vogel; 1877:1-87.
14. Anagiotos A, Preuss SF, Koebke J. Morphometric and anthropometric analysis of Killians triangle. Laryngoscope. 2010;
120(6):1082-1088.

PMPH_CH05.indd 55

References

27, 67-70, 72-79,


82-96 104-109,
113-118, 122-126

70, 71

76, 77, 82, 115, 116

67, 70, 71, 91, 94,


105-107, 114, 121
70, 71, 91, 94, 107,
109, 114, 122-126

15. Muoz AA, Shapiro J, Cuddy LD, Misono S, Bhattacharyya


N. Videofluoroscopic findings in dysphagic patients
with cricopharyngeal dysfunction: before and after open
cricopharyngeal myotomy. Ann Otol Rhinol Laryngol.
2007;116(1):49-56.
16. Cook IJ, Gabb M, Panagopoulos V, et al. Pharyngeal (Zenkers)
diverticulum is a disorder of upper esophageal sphincter
opening. Gastroenterology. 1992;103:1229-1235.
17. Cook I J, Blumbergs P, Cash K, Jamieson G G, Shearman D
J. Structural abnormalities of the cricopharyngeus muscle
in patients with pharyngeal (Zenkers) diverticulum. J
Gastroenterol Hepatol. 1992;7:556-562.
18. Narne S, Cutrone C, Bonavina L, Chella B, Peracchia A.
Endoscopic diverticulotomy for the treatment of Zenkers
diverticulum: results in 102 patients with staple-assisted
endoscopy. Ann Otol Rhinol Laryngol. 1999;108(8):810-815.
19. Peracchia A, Bonavina L, Narne S, Segalin A, Antoniazzi L,
Marotta G. Minimally invasive surgery for Zenker diverticulum:
analysis of results in 95 consecutive patients. Arch Surg.
1998;133(7):695-700.
20. Fulp S R, Castell D O. Manometric aspects of Zenkers
diverticulum. Hepatogastroenterology. 1992;39:123-126.
21. Lang RA, Spelsberg FW, Winter H, Jauch KW, Httl TP. Transoral
diverticulostomy with a modified Endo-Gia stapler: results after
4 years of experience. Surg Endosc. 2007;21(4):532-536.
22. Frieling T, Berges W, Lbke HJ, Enck P, Wienbeck M. Upper
esophageal sphincter function in patients with Zenkers
diverticulum. Dysphagia. 1988;3(2):90-92.
23. Hurwitz AL, Nelson JA, Haddad JK. Oropharyngeal dysphagia:
manometric and cine esophagographic findings. Am J Dig Dis.
1975;20:313-324.
24. Frasson P, Anconna E, Tremolda C, et al. The importance of
extramucosal sphincter myotomy in the treatment of upper and
lower esophageal diverticula. Surg Int. 1976;6:139-147.
25. Lichter I. Motor disorder in pharyngoesophageal pouch.
J Thorac Cardiovasc Surg. 1978;76:272-275.
26. Borrie J, Wilson RLK. Oesophageal diverticula: principles of
management and appraisal of classification. Thorax. 1980;35:
759-767.
27. Nilsson ME, Isberg A, Schiratzki H. The hypopharyngeal
diverticulum, a simultaneous cineradiographic and manometric
examination. Acta Otolaryngol. 1988;106:314-320.

5/21/2012 8:28:50 PM

56

Surgery: Evidence-Based Practice

28. Hunt PS, Connel AM, Smiley TB. The cricopharyngeal sphincter
in gastric reflux. Gut. 1970;11:303-306.
29. Harris RP, Weller MD, Porter MJ. A follow up audit of
pharyngeal pouch surgery using endoscopic stapling. Eur Arch
Otorhinolaryngol. 2010;267(6):939-943.
30. Sharp DB, Newman JR, Magnuson JS. Endoscopic management
of Zenkers diverticulum: stapler assisted versus Harmonic Ace.
Laryngoscope. 2009;119(10):1906-1912.
31. Wasserzug O, Zikk D, Raziel A, Cavel O, Fleece D, Szold
A. Endoscopically stapled diverticulostomy for Zenkers
diverticulum: results of a multidisciplinary team approach.
Surg Endosc. 2010;24(3):637-641.
32. Koay CB, Bates GJ. Endoscopic stapling diverticulotomy for
pharyngeal pouch. Clin Otolaryngol Allied Sci. 1996;21(4):
371-376.
33. Osmote K, Feussner H, Stein HJ, Ungeheuer A, Siewert JR.
Endoscopic stapling diverticulostomy for Zenkers diverticulum.
Surg Endosc. 1999;13(5):535-538.
34. Repici A, Pagano N, Fumagalli U, et al. Transoral treatment
of Zenker diverticulum: flexible endoscopy versus endoscopic
stapling. A retrospective comparison of outcomes. Dis
Esophagus. 2010 Dec 10. [Epub ahead of print]
35. Barthlen W, Feussner H, Hannig C, Hlscher AH, Siewert JR.
Surgical therapy of Zenkers diverticulum: low risk and high
efficiency. Dysphagia. 1990;5(1):13-19.
36. Miller FR, Bartley J, Otto RA. The endoscopic management of
Zenker diverticulum: CO2 laser versus endoscopic stapling.
Laryngoscope. 2006;116(9):1608-1611.
37. Haas I, Gutman M, Paran HJ. Massive upper GI bleeding: a
rare complication of Zenkers diverticulum. Postgrad Med.
2008;54(3):209-210.
38. Brcher BL, Sarbia M, Oestreicher E, et al. Squamous cell
carcinoma and Zenker diverticulum. Dis Esophagus. 2007;20(1):
75-78.
39. Lerut A, Coosemans A, Decker G, De Leyn P, Nafteux P,
Raemdonck D. Pathophysiology and treatment of Zenker
diverticulum. In: Fischer J, Bland K. eds. Mastery of Surgery., 4th
ed., Philadelphia: Wolters Kluwer Health/Lippincott Williams
& Wilkins; 2001:863-866.
40. Skinner DB, Altorki N, Ferguson M, Little G. Zenkers
diverticulum: clinical features and surgical management. Dis
Esoph. 1988;1(1):19-22.
41. Payne WS, King RM. Pharyngoesophageal (Zenkers)
diverticulum. Surg Clhz North Am. 1983;63:815-824.
42. Escher F. Zur Therapie des Zenkerschen Divertikels. Schweiz
Med Wochenschr. 1984;114:1428-1433.
43. Franke KD, Weiss R. Die Myotomie (pars fundiformis m.
cricopharyngei) bei der Behandlung Zcnkerscher Divertikel.
HNO. 1984;32:32-34.
44. Cassivi SD, Deschamps C, Nichols FC 3rd, Allen MS, Pairolero
PC. Diverticula of the esophagus. Surg Clin North Am. 2005;
85(3):495-503, ix.
45. Mosher HP. Webs and pouches of the oesophagus, their
diagnosis and treatment. Surg Gynecol Obstet. 1917;25:
175-187.
46. Collard JM, Otte JB, Kestens PJ. Endoscopic stapling technique
of esophagodiverticulostomy for Zenkers diverticulum. Ann
Thorac Surg. 1993;56(3):573-576.
47. Fama AF, Moore EJ, Kasperbauer JL. Harmonic scalpel in the
treatment of Zenkers diverticulum. Laryngoscope. 2009;119(7):
1265-1269.
48. Al-Kadi AS, Maghrabi AA, Thomson D, Gillman LM, Dhalla
S. Endoscopic treatment of Zenker diverticulum: results of a
7-year experience. J Am Coll Surg. 2010;211(2):239-243.

PMPH_CH05.indd 56

49. Sakai P, Ishioka S, Maluf-Filho F, Chaves D, Moura EG.


Endoscopic treatment of Zenkers diverticulum with an
oblique-end hood attached to the endoscope. Gastrointest
Endosc. 2001;54(6):760-763.
50. Tang SJ, Jazrawi SF, Chen E, Tang L, Myers LL. Flexible
endoscopic clip-assisted Zenkers diverticulotomy: the first case
series (with videos). Laryngoscope. 2008;118(7):1199-1205.
51. Rabenstein T, May A, Michel J, Manner H, Pech O, Gossner
L, Ell C. Argon plasma coagulation for flexible endoscopic
Zenkers diverticulotomy. Endoscopy. 2007;39(2):141-145.
52. Dohlman G, Mattson O. The endoscopic operation for
hypopharyngeal diverticula. Arch Otolaryngol. 1960;71:744-752.
53. Costamagna G, Mutignani M, Tringali A, Perri V. Treatment of
Zenkers diverticulum with the help of a plastic hood attached
to the endoscope. Gastrointest Endosc. 2002;56(4):611-612.
54. Vogelsang A, Preiss C, Neuhaus H, Schumacher B. Endotherapy
of Zenkers diverticulum using the needle-knife technique:
long-term follow-up. Endoscopy. 2007;39(2):131-136.
55. Helmstaedter V, Engel A, Huttenbrink KB, Guntinas-Lichius O.
Carbon dioxide laser endoscopic diverticulotomy for Zenkers
diverticulum: results and complications in a consecutive series
of 40 patients. ORL J Otorhinolaryngol Relat Spec. 2009;71(1):
40-44.
56. Gutschow CA, Hamoir M, Rombaux P, Otte JB, Goncette L,
Collard JM. Management of pharyngoesophageal (Zenkers)
diverticulum: which technique? Ann Thorac Surg. 2002;74(5):
1677-1682.
57. Palmer AD, Herrington HC, Rad IC, Cohen JI. Dysphagia after
endoscopic repair of Zenkers diverticulum. Laryngoscope.
2007;117(4):617-622.
58. Tsikoudas A, Eason D, Kara N, Brunton JN, Mountain RE.
Correlation of radiologic findings and clinical outcome
in pharyngeal pouch stapling. Ann Otol Rhinol Laryngol.
2006;115(10):721-726.
59. Nicholas BD, Devitt S, Rosen D, Spiegel J, Boon M. Endostitchassisted endoscopic Zenkers diverticulostomy: a tried approach
for difficult cases. Dis Esophagus. 2010;23(4):296-299.
60. Brace M, Taylor SM, Trites JR, Bethune D, Attia E, Hart RD.
Endoscopic stapling versus external transcervical approach for
the treatment of Zenker diverticulum. J Otolaryngol Head Neck
Surg. 2010;39(1):102-106.
61. Smith SR, Genden EM, Urken ML. Endoscopic stapling
technique for the treatment of Zenker diverticulum vs standard
open-neck technique: a direct comparison and charge analysis.
Arch Otolaryngol Head Neck Surg. 2002;128(2):141-144.
62. Case DJ, Baron TH. Flexible endoscopic management of Zenker
diverticulum: the Mayo Clinic experience. Mayo Clin Proc.
2010;85(8):719-722.
63. Wheeler D. Diverticula of the foregut. Radiology. 1947;49:
476-481.
64. Punto L. Functional disorders of the oesophagus,
Roentegenological, cineradiographic, and manometric study.
Ann Clin Res. 1974;6(supp 14):1-72.
65. Schima W, Schober E, Stacher G, et al. Association of
midoesophageal diverticula with oesophageal motor disorders.
Videofluoroscopy and manometry. Acta Radiol. 1997;38:
108-114.
66. Baker ME, Zuccaro G Jr, Achkar E, Rice TW. Esophageal
diverticula: patient assessment. Semin Thorac Cardiovasc Surg.
1999;11:326-336.
67. Thomas ML, Anthony AA, Fosh BG, Finch JG, Maddern GJ.
Oesophageal diverticula. Br J Surg. 2001;88(5):629-642.
68. Harrington SW. The surgical treatment of pulsion diverticula of
the thoracic esophagus. Ann Surg. 1949;129:606-618.

5/21/2012 8:28:50 PM

Esophageal Diverticula

69. Beal JM. Epiphrenic diverticulum of the esophagus. IMJ Ill


Med J. 1970;137:348-351.
70. Klaus A, Hinder RA, Swain J, Achem SR. Management of
epiphrenic diverticula. J Gastrointest Surg. 2003;7:906-911.
71. Palanivelu C, Rangarajan M, John SJ, Parthasarathi R,
Senthilkumar R. Laparoscopic transhiatal approach for benign
supradiaphragmatic lesions of the esophagus: a replacement for
thoracoscopy? Dis Esophagus. 2008;21:176-180.
72. Vinson PP. Diverticula of the thoracic portion of the esophagus:
report of 42 cases. Arch Otolaryngol. 1934;19:508-513.
73. Benacci JC, Deschamps C, Trastek VF, et al. Epiphrenic
diverticulum: results of surgical treatment. Ann Thorac Surg.
1993;55:1109-1113.
74. Bruggeman LL, Seaman WB. Epiphrenic diverticula. An
analysis of 80 cases. Am J Roentgenol Radium Ther Nucl Med.
1973;119:266-276.
75. Fegiz G, Paolini A, De Marchi C, Tosato F. Surgical management
of esophageal diverticula. W J Surg. 1984;8:757-765.
76. Evander A, Little AG, Ferguson MK, Skinner DB. Diverticula
of the mid- and lower esophagus: pathogenesis and surgical
management. World J Surg. 1986;10:820-828.
77. Altorki, NK, Sunagawa M, Skinner DB. Thoracic esophageal
diverticula. Why is operation necessary? J Thorac Cardiovasc
Surg. 1993;105:260-264.
78. Nehra D, Lord RV, DeMeester TR, et al. Physiologic basis for
the treatment of epiphrenic diverticulum. Ann Surg. 2002;235:
346-354.
79. Jordan PH Jr, Kinner BM. New look at epiphrenic diverticula.
World J Surg. 1999;23:147-152.
80. Kaye MD. Oesophageal motor dysfunction in patients with
diverticula of the mid-thoracic esophagus. Thorax. 1974;29:
666-672.
81. Rivkin L, Bremner CG, Bremner CH. Pathophysiology of midoesophageal and epiphrenic diverticula of the oesophagus. S
Afr Med J. 1984;66:127-129.
82. Fekete F, Vonns C. Surgical management of esophageal thoracic
diverticula. Hepatogastroenterology. 1992;39:97-99.
83. Castrucci G, Porziella V, Granone PL, Picciocchi A. Tailored
surgery of esophageal body diverticula. Eur J Cardiothorac
Surg. 1998;14:380-387.
84. Debas HT, Payne WS, Cameron AJ, Carlson HC. Physiopathology
of lower esophageal diverticulum and its implications for
treatment. Surg Gynecol Obstet. 1980;151:593-600.
85. DUgo D, Cardillo G, Granone P, Coppola R, Margaritora S,
Picciocchi A. Esophageal diverticula. Physiopathological basis
for surgical management. Eur J Cardiothorac Surg. 1992;6:
330-334.
86. Streitz JM Jr, Glick ME, Ellis FH Jr. Selective use of myotomy for
treatment of epiphrenic diverticula. Manometric and clinical
analysis. Arch Surg. 1992;127:585-587.
87. Hudspeth DA, Thorne MT, Conroy R, Pennell TC. Management
of epiphrenic esophageal diverticula. A fifteen year experience.
Am J Sug. 1993;59:40-42.
88. Peracchia A, Bonavina L, Rosati R, Bona S. Thoracoscopic
resection of epiphrenic esophageal diverticula. In: Peters JH,
DeMeester TR, eds. Minimally Invasive Surgery of the Foregut.
St. Louis, MO: Quality Medical Publishing; 1994:110-116.
89. Ott DJ, Hodge RG, Chen MY, Wu WC, Gelfand DW. Achalasia
associated with esophageal diverticula. Prevalence and potential
implications. J Clin Gastroenterol. 1994;18:343-346.
90. Ellis FH Jr. Surgical Management of esophageal motility
disturbances. Am J Surg. 1980;139:752-759.
91. Tedesco P, Fisichella PM, Way LW, Patti MG. Cause and treatment
of epiphrenic diverticula. Am J Surg. 2005;190(6):891-894.

PMPH_CH05.indd 57

57

92. Hamilton S. Esophageal leiomyoma arising in an epiphrenic


diverticulum of the esophagus. Indian J Gastroenterol. 1991;10:
150-151.
93. Fasano NC, Levine MS, Rubesin SE, Redfern RO, Laufer I.
Epiphrenic diverticulum: clinical and radiographic findings in
27 patients. Dysphagia. 2003;18(1):9-15.
94. Matthews BD, Nelms CD, Lohr CE, Harold KL, Kercher KW,
Heniford BT. Minimally invasive management of epiphrenic
esophageal diverticula. Am Surg. 2003;69(6):465-470.
95. Michael H, Fisher RS. Treatment of epiphrenic and mid-esophageal
diverticula. Curr Treat Options Gastroenterol. 2004;7(1):41-52.
96. Widmar M, Nguyen SQ, Newell P, Patel P, Divino CM. Large
epiphrenic diverticulum. J Gastrointest Surg. 2010;14(6):
1062-1064.
97. Chen L, Walser EM, Schnadig V. Fatal hemorrhage secondary
to ulcerated epiphrenic pseudodiverticulum. Arch Pathol Lab
Med. 2006;130(6):867-870.
98. Sam AD Jr, Chaer RA, Cintron J, Teresi M, Massad MG. Upper
gastrointestinal bleeding caused by a hypophrenic diverticulum
of the distal esophagus. Am Surg. 2005;71(4):333-335.
99. Herbella FA, Del Grande JC. Benign esophagopulmonary
fistula through an epiphrenic diverticulum and asymptomatic
achalasia. Dig Dis Sci. 2010;55(4):1177-1178.
100. Lai ST, Hsu CP. Carcinoma arising from an epiphrenic
diverticulum: a frequently misdiagnosed disease. Ann Thorac
Cardiovasc Surg. 2007;13(2):110-113.
101. Hung JJ, Hsieh CC, Lin SC, Wang LS. Squamous cell carcinoma
in a large epiphrenic esophageal diverticulum. Dig Dis Sci.
2009;54(6):1365-1368. Epub 2009 Jan 30.
102. Honda H, Kume K, Tashiro M, et al. Early stage esophageal
carcinoma in an epiphrenic diverticulum. Gastrointest Endosc.
2003;57(7):980-982.
103. Mann NS, Borkar BB, Mann SK. Phlegmonous esophagitis
associated with epiphrenic diverticulum. Am J Gastroenterol.
1978;70(5):510-513.
104. Tobin RW. Esophageal rings, webs, and diverticula. J Clin
Gastroenterol. 1998;27:285-295.
105. Soares R, Herbella FA, Prachand VN, Ferguson MK, Patti MG.
Epiphrenic diverticulum of the esophagus. From pathophysiology
to treatment. J Gastrointest Surg. 2010;14(12):2009-2015.
106. Fernando HC, Luketich JD, Samphire J, et al. Minimally
invasive operation for esophageal diverticula. Ann Thorac Surg.
2005;80:2076-2080.
107. Rosati R, Fumagalli U, Bona S, et al. Laparoscopic treatment
of epiphrenic diverticula. J Laparoendosc Adv Surg Tech A.
2001;11:371-375.
108. do Nascimento FA, Lemme EM, Costa MM. Esophageal
diverticula: pathogenesis, clinical aspects, and natural history.
Dysphagia. 2006;21(3):198-205.
109. Zaninotto G, Portale G, Costantini M, et al. Long-term outcome
of operated and unoperated epiphrenic diverticula. J Gastrointest
Surg. 2008;12(9):1485-1490.
110. Katsinelos P, Chatzimavroudis G, Fasoulas K, et al. Severe
dysphagia due to a huge epiphrenic diverticulum: long-term
treatment with balloon dilation and botulinum toxin injection:
a case report. Cases J. 2009;2:7418.
111. Katsinelos P, Chatzimavroudis G, Zavos C, Pilpilidis I, Paroutoglou
G, Kountouras J. Long-term botulinum toxin treatment for
dysphagia due to large epiphrenic diverticulum in elderly patients:
a report of two cases. Dysphagia. 2009;24(1):109-113.
112. Zhao X, Pasricha PJ. Botulinum toxin for spastic GI disorders:
a systematic review. Gastrointest Endosc. 2003, 57:219-235.
113. Ipek T, Eyuboglu E. Laparoscopic resection of an esophageal
epiphrenic diverticulum. Acta Chir Belgium. 2002;102:270-273.

5/21/2012 8:28:50 PM

58

Surgery: Evidence-Based Practice

114. Nastos D, Chen L, Ferraro P, Taillefer R, Duranceau A. Long


myotomy with antireflux repair for esophageal spastic disorders.
J Gastrointest Surg. 2002;6:713-722.
115. Belsey R. Functional disease of the esophagus. J Thorac
Cardiovasc Surg. 1966;52(2):164-188.
116. Valentini M, Pera M, Vidal O, Lacima G, Belda J, de Lacy AM.
Incomplete esophageal myotomy and early recurrence of an
epiphrenic diverticulum. Dis Esophagus. 2005;18(1):64-66.
117. Habein HC Jr, Kirklin JW, Clagett OT, Moersch HJ. Surgical
treatment of lower esophageal pulsion diverticula. AMA Arch
Surg. 1956;72(6):1018-1024.
118. Kilic A, Schuchert MJ, Awais O, Luketich JD, Landreneau RJ.
Surgical management of epiphrenic diverticula in the minimally
invasive era. JSLS. 2009;13(2):160-164.
119. Richards WO, Torquati A, Holzman MD, et al. Heller myotomy
versus Heller myotomy with Dor fundoplication for achalasia:
a prospective randomized double-blind clinical trial. Ann Surg.
2004;240(3):405-412.
120. Topart P, Deschamps C, Taillefer R, et al. Long-term effect of
total fundoplication on the myotomized esophagus. Ann Thorac
Surg. 1992;54:1046-1051.
121. Falkenback D, Johansson J, Oberg S, et al. Hellers esophagomyotomy with or without a 360 degrees floppy Nissen
fundoplication for achalasia. Long-term results from a prospective
randomized study. Dis Esophagus. 2003;16(4):284-290.
122. Del Genio A, Rossetti G, Maffetton V, et al. Laparoscopic
approach in the treatment of epiphrenic diverticula: long-term
results. Surg Endosc. 2004;18(5):741-745.
123. Silecchia G, Casella G, Recchia CL, Bianchi E, Lomartire
N. Laparoscopic transhiatal treatment of large epiphrenic
esophageal diverticulum. JSLS. 2008;12(1):104-108.

PMPH_CH05.indd 58

124. Palanivelu C, Rangarajan M, Senthilkumar R, Velusamy M.


Combined thoracoscopic and endoscopic management of midesophageal benign lesions: use of the prone patient position:
Thoracoscopic surgery for mid-esophageal benign tumors and
diverticula. Surg Endosc. 2008;22(1):250-254.
125. Granderath FA, Pointner R. Laparoscopic transhiatal resection
of giant epiphrenic esophageal diverticulum. Dis Esophagus.
2007;20(4):353-357.
126. Dado G, Bresadola V, Terrosu G, Bresadola F. Diverticulum
of the midthoracic esophagus: pathogenesis and surgical
treatment. Surg Endosc. 2002;16(5):871.
127. Steiner SJ, Cox EG, Gupta SK, Kleiman MB, Fitzgerald JF.
Esophageal diverticulum: a complication of histoplasmosis in
children. J Pediatr. 2005;146(3):426-428.
128. Raziel A, Landau O, Fintsi Y, Fass R, Charuzi I. Sarcoidosis
and giant midesophageal diverticulum. Dis Esophagus. 2000;
13(4):317-319.
129. Bke E, Sarigl A, Sungur A, Uzunalimoglu B. Benign
mesenchymoma of the esophagus. Eur J Cardiothorac Surg.
1997;11(1):196-198.
130. Lpez A, Rodrguez P, Santana N, Freixinet J. Esophagobronchial
fistula caused by traction esophageal diverticulum. Eur J
Cardiothorac Surg. 2003;23(1):128-130.
131. Kutty CP, Carstens SA, Funahashi A.Traction diverticula of
the esophagus in the middle lobe syndrome. Can Med Assoc J.
1981;124(10):1320-1322.
132. Kutty CP, Funahashi A. Thickening of the posterior tracheal
stripe in esophageal traction diverticula. Chest. 1980;77(1):
126-127.
133. Ikeda Y. Traction mid-esophageal diverticulum associated with
Potts spinal caries. Dig Endosc. 2010;22(2):158-159.

5/21/2012 8:28:50 PM

CHAPTER 6

Gastroesophageal Reflux Disease


Alejandro F. Sanz and Blair A. Jobe

INTRODUCTION

GERD are clearly related, both from a prevalence and causality association. GERD symptoms increase in severity when a person gains
weight. A meta-analysis confirmed a positive association between
the presence of GERD and increasing body mass index (BMI).7
Another meta-analysis showed a statistically significant increase
in the risk for GERD symptoms, erosive esophagitis, esophageal
adenocarcinoma, with progressively increasing weight.8
Behavioral factors: The most commonly investigated behavioral factors potentially affecting GERD episodes are cigarette
smoking, alcohol consumption, and coffee consumption. Three
cross-sectional studies demonstrated a significant relationship
between GERD symptoms and smoking.3,5,6 On the other hand,
there is not enough data to support or rule out a relationship
between coffee or alcohol consumption and GERD.6
Comorbid factors: In a study using the UK General Practice
Research database, the risk of GERD was significantly increased
in individuals who had visited a general practitioner three times
or more in the preceding year, who had been referred to a specialist or hospitalized, or been diagnosed with irritable bowel syndrome, ischemic heart disease, peptic ulcer disease, or one of a
range of painful conditions in the year before the index date (the
date they were diagnosed with GERD). The risk of GERD was also
increased among overweight individuals and ex-smokers.6,9
Answer: A number of potential risk factors for GERD have
been identified. However, all of the positive associations have
rather small odds ratios, leaving their clinical implications for
preventive or therapeutic strategies in doubt. Grade of recommendation: B.

Gastroesophageal reflux disease (GERD) is one of the most common disorders in medical practice, affecting not only Americans
but also becoming a worldwide health problem. Detailed data are
still lacking regarding the incidence and prevalence of GERD in
North America. The fi rst data on GERD incidence came from
a study done in San Diego, California.1 Using a questionnaire,
the authors evaluated the incidence and precipitating factors of
GERD in 446 hospitalized and 558 nonhospitalized subjects.
The authors concluded that 36% of the individuals experienced
heartburn at least once a month and 14% experienced heartburn daily. Another interesting study on GERD prevalence in
a North American population was done in Olmsted County,
Minnesota. 2 In this study, the authors surveyed a large, random sample of residents aged 25 to 74 years to determine the
prevalence and clinical spectrum of GERD in that community.
The prevalence of heartburn was 42%. Acid regurgitation was
reported by an additional 28% of the individuals. Remarkably,
this population-based study also addressed the occurrence of
other symptoms associated with reflux. For example, dysphagia
was reported by 13.5% of the Olmsted county residents and globus was reported by 7%.

RISK FACTORS
1. Who is at risk for GERD?
The risk factors of GERD can be divided into four categories:
genetic, demographic, behavioral, and comorbid associations.
Genetic factors: A genetic contribution to the etiology of
GERD has been identified in studies of twins.3,4 Genetic factors
account for 30% to 40% of the liability to GERD.
Demographic factors: Despite the well-known association
between GERD and pregnancy, there is no significant relationship
between sex and GERD.2,5 The effect of increasing age and GERD is
still unclear, with two European studies reporting a slight but significant association of GERD with increasing age.3,5,6 Obesity and

DIAGNOSIS OF GERD
2. What is the role and priority of endoscopy in diagnosis of
GERD?
Endoscopy allows direct visualization of the esophageal mucosa.
Endoscopic findings in patients with GERD include esophagitis, erosions and ulcers, strictures, and Barretts esophagus. If
moderate-to-severe symptoms of GERD and endoscopic injury
59

PMPH_CH06.indd 59

5/21/2012 8:29:59 PM

60

Surgery: Evidence-Based Practice

are both present, the diagnosis of GERD can be made without


further studies (Level of evidence 4), as this combination has 97%
specificity and 64% sensitivity for accurately diagnosing GERD.10
However, patients with GERD symptoms can have normal endoscopic findings, for example, nonerosive reflux disease.
Some data also suggest a role of upper endoscopy in the
evaluation of heartburn in patients in whom once-daily proton
pump inhibitor (PPI) treatment failed compared with those not
receiving any treatment.11 Endoscopy is the first diagnostic test to
consider when medical therapy fails. Endoscopy can also demonstrate Barretts metaplasia, stricture, or an alternative upper gastrointestinal diagnosis.
There are a few alarm features that can accompany suspected
GERD, which can lead to an alternative diagnosis. Clinically relevant differential diagnoses in the context of GERD include coronary artery disease; gallbladder disease; gastric or esophageal
malignancy; peptic ulcer disease; and eosinophilic, infectious, or
caustic esophagitis. The evidence supporting the utility of alarm
features as a diagnostic tool is very limited. However, some of
these features are very characteristic and should lead to suspicion
of a gastric or esophageal malignancy, for example, weight loss,
dysphagia, and epigastric mass on examination. In this clinical
setting, endoscopy is the gold standard for evaluation.
Answer: Any patient with a suspected GERD syndrome, who
has not responded to an empirical trial of twice-daily PPI therapy,
should undergo an endoscopic evaluation with biopsies targeted to
suspected areas of metaplasia, dysplasia, or malignancy.12 (Grade
B recommendation.)
In addition, there is fair evidence that recommends endoscopy with biopsy for patients with a GERD syndrome with troublesome dysphagia (Level 3 of evidence). Biopsies should target
any areas of suspected metaplasia, dysplasia, or in the absence of
visual abnormalities, normal mucosa. At least five samples should
be collected to evaluate for eosinophilic esophagitis.
3. What is the role of 24-hour esophageal pH monitoring in the
diagnosis of GERD?
Ambulatory 24-hour esophageal pH monitoring is currently
considered the gold standard for diagnosing GERD, with a sensitivity of 79% to 96% and specificity of 85% to 100%.13-16 In addition, ambulatory 24-hour esophageal pH monitoring is the most
sensitive diagnostic test for identifying acid reflux as the likely
cause for chest pain of noncardiac origin17 and helps to confirm
GERD in patients with persistent symptoms (both typical and
atypical) without evidence of mucosal damage, especially when a
trial of acid suppression has failed. Ambulatory 24-hour esophageal pH monitoring may also be used to monitor the control of
reflux in patients with continued symptoms while on therapy.18
There have been two recent advances in the way we can study
and diagnose GERD. One is combined impedance and acid testing. With this new technology, acid and nonacid reflux can be
recorded. The other development is a wireless method of acid
monitoring. The new wireless device allows a radiotelemetry
capsule to be attached to the esophageal mucosa and monitored
without the discomfort of a nasoesophageal tube. Th is decreases
patient discomfort and allows longer periods of monitoring (48
hours), which may improve accuracy in the measurement of
reflux episodes. In preliminary pH studies performed over periods > 24 hours, abnormal pH exposure was detected only on the
second day of monitoring in up to one third of subjects. Therefore, pH monitoring for 48 hours may yield more information

PMPH_CH06.indd 60

about pathological acid reflux in patients being investigated for


GERD (Level 3 of evidence).18
Answer: Ambulatory impedance pH, catheter pH, or wireless
pH monitoring (with the PPI therapy withheld for a period of
7 days) is indicated in patients with a suspected GERD syndrome
who have not responded to a trial of PPI therapy, who have normal
findings on endoscopy, and who have no major abnormalities on
manometry. Wireless pH monitoring has a superior sensitivity to
catheter pH monitoring for detecting pathologic esophageal acid
exposure because of the extended period of recording (48 hours)
and also has superior recording accuracy compared with some
catheter designs. (Grade B recommendation.) There is insufficient
evidence to recommend any type of esophageal impedance or pH
testing (wireless or catheter) while taking PPIs.12

TREATMENT OF GERD
4. What is the mainstay of therapy for GERD?
In the original guidelines for the diagnosis and treatment for GERD,
published by DeVault et al. in 1995, the authors included a review
of 33 randomized trials including over 3000 patients with erosive
esophagitis. They founded symptomatic relief in 27% of placebotreated patients, 60% of patients treated with histamine 2 receptor antagonists (H2RAs), and 83% of patients treated with PPIs.
The esophagitis healed in 24% of placebo-treated patients, 50% of
H2RA-treated patients, and 78% of PPI-treated patients. As a result
of this review, the updated guidelines from the American College
of Gastroenterology state that both higher doses and more frequent
doses of H2RAs produce results inferior to those of PPIs. Moreover, the advantages of PPI therapy increase with the severity of the
disease. In addition, randomized and double-blinded studies have
shown the efficacy of PPIs (omeprazole, single and double dose)
over placebo in providing relief of reflux symptoms in patients with
heartburn and improving patients general well-being.19
Answer: Acid suppression is the mainstay of therapy for
GERD. PPIs provide the most rapid symptomatic relief and
heal esophagitis in the highest percentage of patients (Level 1 of
evidence).18 There is no statistical difference in the response to
treatment with PPI taken as two daily doses, when compared with
a single daily dose.20,21 (Grade A recommendation.)
5. How does medical treatment of GERD compare with surgical
treatment of GERD?
There is a lot of controversy in the literature defining the best
treatment for GERD, medical or surgical. In the very first articles
that compared both modalities, surgery was much more efficient
than medical treatment. However, those articles usually compare
surgery with medical therapies that are known to be ineffective
today, for example, H2 receptor blockade. With the development
of PPIs, the gap in efficiency between medical and surgical treatment is getting smaller and investigators are following up to evaluate long-term outcomes. For example, Lundell et al.22 compared
antireflux surgery with omeprazole therapy for the treatment of
GERD and followed the patients for 5 years. They compared 310
patients with erosive esophagitis, randomized into two groups of
155 patients each and found antireflux surgery to be more effective than omeprazole. In the study of Lundell and colleagues,
patients were treated with either 20 mg of omeprazole, usually for
4 to 8 weeks, with dose increments to 40 mg in cases of incomplete

5/21/2012 8:29:59 PM

Gastroesophageal Reflux Disease

response in controlling gastroesophageal reflux disease. However,


if the dose of omeprazole was adjusted in cases of relapse, the two
therapeutic strategies displayed levels of efficacy that were not statistically different.
In an open, parallel-group multicenter, randomized and
controlled trial conducted in 11 European countries called the
LOTUS trial, published by Lundell et al.,23 the authors compared
laparoscopic surgery with medical treatment (esomeprazole 20
mg once daily), and reported initial results for a follow-up period
of 3 years. A total of 554 patients were randomized in the LOTUS
trial. Two hundred eighty-eight underwent laparoscopic surgery
and 260 were treated with PPIs. They concluded that both laparoscopic total fundoplication and continuous esomeprazole
treatment were similarly effective and well-tolerated therapeutic
strategies for providing effective control of GERD. However, postfundoplication complaints (e.g., dysphagia) remain a problem
after laparoscopic surgery. The same European group published
a 12-year follow-up study,24 comparing the medical treatment
with omeprazole and antireflux surgery. They found that both
treatment modalities were similarly effective in the treatment of
GERD and well tolerated by the patients. Antireflux surgery was
found to be superior to omeprazole in controlling overall disease
manifestations, but post-fundoplication complaints continue after
surgery.
Mehta et al.25 performed a prospective, randomized study with
a 7-year follow-up to obtain long-term follow-up data. A total of
183 patients were randomized into two groups, 91 to surgery and
92 to optimized PPI therapy (20 mg/day). After 1 year, patients in
the PPI group were offered the opportunity to have surgery. Mehta
and colleague concluded that both optimal PPI therapy and laparoscopic Nissen fundoplication are effective and durable treatments for GERD. Patients who had surgery at the beginning of
the trial and patients who remained on PPIs for the entire course
of the study both had significantly improved symptoms. However, there was a substantial difference in those patients who had a
moderate response to the medical treatment during the first year
and then decided to undergo surgery. These patients experienced
a significant reduction in symptoms after the laparoscopic Nissen fundoplication. Furthermore, of the patients who remained
on PPIs for the entire duration of the study, only 59% were very
satisfied with symptom control, compared with 80% of patients
who had undergone surgery and were very satisfied.
Answer: Antireflux surgery, performed by an experienced
surgeon, is a maintenance option for the patient with welldocumented GERD.18 (Level 2 of evidence.) There is a decrease in

61

therapeutic failure of 12% when we compare surgical and medical treatment for GERD, and these numbers dont change after an
observational period of 7 years.26 (Grade A recommendation.)
If a patient can benefit from both forms of treatments for
GERD (medical or surgical) with a similar efficacy, PPI therapy
should be recommended first because it is safer and less invasive.
If a patient is diagnosed with a GERD syndrome, but is intolerant
to acid suppression therapy, antireflux surgery should be recommended as an alternative. (Grade A recommendation.)
6. Which is the best fundoplication technique for treatment of
GERD: Total (Nissen) or partial (Toupet)?
Laparoscopic Nissen fundoplication is the most frequently performed operation for GERD in the United States. Laparoscopic
Toupet fundoplication has been proposed as an alternative operation, due to some adverse events experienced by patients after Nissen fundoplication. Adverse events after laparoscopic Nissen include
dysphagia in 8% to 12%27-30 of patients and gas-related symptoms in
19%.31 In a prospective clinical trial, Lundell et al.32 randomized
137 patients with GERD to either laparoscopic Nissen (65 patients)
or laparoscopic Toupet (72 patients) fundoplication. Both surgical
techniques were equally effective in the resolution of GERD. Dysphagia was more common in the early postoperative period after a
360-degree wrap (Nissen), but this difference disappeared with time.
In addition, gas-related symptoms, such as flatulence, were more
frequently seen after laparoscopic Nissen fundoplication (P < .05
at 2 years and P < .01 at 3 years). Even though laparoscopic Toupet
fundoplication appears to be as effective as laparoscopic Nissen fundoplication, with fewer side effects, it is not a widely used technique,
probably because there are some studies that report less effective
reflux control using the Toupet, partial-wrap technique.33,34
In two consecutive, prospective, randomized studies performed by the same group, Strate et al.35 and Zornig et al.36 analyzed patient satisfaction after antireflux surgery during a 2- or
3-year follow-up period. Regardless of the surgical technique
employed (Nissen or Toupet), 85% of patients were satisfied with
the operative result. However, similar to the findings of Lundell
and colleagues, dysphagia was more frequent after Nissen fundoplication than after Toupet fundoplication (19 vs. 8, P < .05).
Answer: There is no difference in therapeutic response
between Toupet and Nissen fundoplications performed laparoscopically. However, laparoscopic Nissen fundoplication can
cause dysphagia, which is not correlated with esophageal motility.
(Level 1C of evidence; Grade A recommendation.)

Clinical Question Summary


Level of
Evidence

Grade of
Recommendation

Potential risk factors for GERD, including


genetic, behavioral, and demographic
factors, have been identified. However, all
of the positive associations have rather
small odds ratios.

2-8

Endoscopy with biopsy is recommended in


patients with GERD and dysphagia or in
patients who have not responded to an
empirical trial of PPI therapy.

12

Question

Answer

1 Who is at risk for GERD?

2 What is the role and


priority of endoscopy in
GERD?

References

(Continued)

PMPH_CH06.indd 61

5/21/2012 8:29:59 PM

62

Surgery: Evidence-Based Practice

(Continued)
Question

Answer

Level of
Evidence

Grade of
Recommendation

3 What is the role of 24-hour


esophageal pH monitoring
in the diagnosis of GERD?

24-hour esophageal pH monitoring is the


gold standard for diagnosing GERD, with a
sensitivity of 79% to 96% and specificity of
85% to 100%.

12-16

4 What is the mainstay of


therapy for GERD?

PPIs provide the most rapid symptomatic


relief and heal esophagitis in the highest
percentage of patients.

18, 20, 21

5 How does medical


treatment of GERD
compare with surgical
treatment of GERD?

Antireflux surgery, performed by an


experienced surgeon, is a maintenance
option for the patient with GERD. In many
patients, medical therapy and surgery
therapy lead to equivalent results.

18, 25

6 Which is the best


fundoplication technique
for treatment of GERD:
total (Nissen) or partial
(Toupet)?

There is no difference in therapeutic


response between the two options.
However, laparoscopic Nissen
fundoplication can cause dysphagia. The
durability of partial fundoplication is
thought to be less than that of Nissen
fundoplication.

1C

26-36

REFERENCES
1. Nebel OT, Fornes MF, Castell DO. Symptomatic gastroesophageal reflux: incidence and precipitating factors. Am J Dig Dis.
1976;21(11):953-956.
2. Locke GR 3rd, Talley NJ, Fett SL, Zinsmeister AR, Melton LJ 3rd.
Prevalence and clinical spectrum of gastroesophageal reflux: a
population-based study in Olmsted County, Minnesota. Gastroenterology. 1997;112(5):1448-1456.
3. Mohammed I, Cherkas LF, Riley SA, et al. Genetic influences
in gastrooesophageal reflux disease: a twin study. Gut. 2003;52:
1085-1089.
4. Cameron AJ, Lagergren J, Henriksson C, et al. Gastroesophageal
reflux disease in monozygotic and dizygotic twins. Gastroenterology. 2001;122:55-59.
5. Isolauri J, Laippala P. Prevalence of symptoms suggestive of gastroesophageal reflux disease in an adult population. Ann Med.
1995;27:67-70.
6. J Dent, H B El-Serag, M-A Wallander, S Johansson. Epidemiology of gastro-oesophageal reflux disease: a systematic review.
Gut. 2005;54:710-717.
7. Corley DA, Kubo A. Body mass index and gastroesophageal
reflux disease: a systematic review and meta-analysis. Am J Gastroenterol. 2006;101(11):2619-2628.
8. Hampel H, Abraham NS, El-Serag HB. Meta-analysis: obesity
and the risk for gastroesophageal reflux disease and its complications. Ann Intern Med. 2005;143(3):199-211.
9. Ruigomez A, Wallander MA, Johansson S, et al. Natural history of gastroesophageal reflux disease diagnosed in UK general
practice. Aliment Pharmacol Ther. 2004;20:751-760.
10. Tefera L, Fein M, Ritter MP, et al. Can the combination of symptoms and endoscopy confirm the presence of gastroesophageal
reflux disease? Am Surg. 1997;63(10):933-936.
11. Poh CH, Gasiorowska A, Navarro-Rodriguez T, et al. Upper GI
tract findings in patients with heartburn in whom proton pump

PMPH_CH06.indd 62

12.

13.

14.

15.
16.

17.

18.

19.

20.

21.

References

inhibitor treatment failed versus those not receiving antireflux


treatment. Gastrointest Endosc. 2010;71(1):28-34.
Kahrilas PJ, Shaheen NJ, Vaezi MF, et al. American Gastroenterological Association Medical Position Statement on the management of gastroesophageal reflux disease. Gastroenterology.
2008;135(4):1383-1391, 1391.e1-5.
Rosen SN, Pope CE II. Extended esophageal pH monitoring. An
analysis of the literature and assessment of its role in the diagnosis and management of gastroesophageal reflux disease. J Clin
Gastroenterol. 1989;11:260-270.
Euler AR, Byrne WJ. Twenty-four hour esophageal intraluminal pH probe testing: a comparative analysis. Gastroenterology.
1981;80:957-961.
Richter JE, Castell DO. Gastroesophageal reflux: pathogenesis,
diagnosis and therapy. Ann Intern Med. 1982;97:93-103.
Jamieson JR, Sten HJ, DeMeester TR, et al. Ambulatory 24-hour
esophageal pH monitoring: normal values, optimal thresholds,
specificity, sensitivity and reproducibility. Am J Gastroenterol.
1992;87:1102-1111.
Hewson EG, Sinclair JW, Dalton CB, et al. Twenty-four hour
esophageal pH monitoring. The most useful test for evaluating
noncardiac chest pain. Am J Med. 1991;90:576-583.
DeVault KR, Castell DO; American College of Gastroenterology.
Updated guidelines for the diagnosis and treatment of gastroesophageal reflux disease. Am J Gastroenterol. 2005;100(1):190-200.
Havelund T, Lind T, Biklund I, et al. Quality of life in patients
with heartburn but without esophagitis: effects of treatment with
omeprazole. Am J Gastroenterol. 1999;94:1782-1789.
Johnsson F, Hatlebakk JG, Klintenberg AC, Romn J. Symptom
relieving effect of esomeprazole 40 mg daily in patients with
heartburn. Scand J Gastroenterol. 2003;38:347-353.
Johnsson F, Hatlebakk JG, Klintenberg AC, et al. One-week
omeprazole treatment: an effective confirmatory test in patients
with suspected gastroesophageal reflux disease. Scand J Gastroenterol. 2003;38:354-359.

5/21/2012 8:29:59 PM

Gastroesophageal Reflux Disease

22. Lundell L, Dalenvack J, Hattlevakk J, et al. Continued (5-year)


followup of a randomized clinical study comparing antireflux
surgery and omeprazole in GERD. J Am Coll Surg. 2001;192:
172-179.
23. Lundell L, Attwood S, Ell C, et al. Comparing laparoscopic antireflux surgery with esomeprazole in the management of patients
with chronic gastro-oesophageal reflux disease: a 3- year interim
analysis of the LOTUS trial. Gut. 2008;57(9):1207-1213.
24. Lundell L, Miettinen P, Myrvold HE, et al. Comparison of outcomes twelve years after antireflux surgery or omeprazole maintenance therapy for reflux esophagitis. Clin Gastroenterol Hepatol.
2009;7(12):1264-1265.
25. Mehta S, Bennett J, Mahon D, Rhodes M. Prospective trial of laparoscopic Nissen fundoplication versus proton pump inhibitor
therapy for gastroesophageal reflux disease: seven-year follow-up.
J Gastrointest Surg. 2006;10:1312-1316.
26. Moraes-Filho JP, Navarro-Rodriguez T, Barbuti R, Eisig J, Chinzon D, Bernardo W. Guidelines for the diagnosis and management
of gastroesophageal reflux disease: an evidence-based consensus.
Arq Gastroenterol. 2010;47(1):99-115.
27. McKenzie D, Grayson T, Polk HC Jr. The impact of omeprazole
and laparoscopy upon hiatal hernia and reflux esophagitis. J Am
Coll Surg. 1996;183:413-418.
28. Klaus A, Hinder RA, DeVault KR, Achem SR. Bowel dysfunction
after Laparoscopic antireflux surgery: incidence, severity, and
clinical course. Am J Med. 2003;114:6-9.

PMPH_CH06.indd 63

63

29. Spechler SJ. Comparison of medical and surgical therapy for


complicated gastroesophageal reflux disease in veterans. The
Department of Veterans Affairs Gastroesophageal Reflux Disease Study Group. N Engl J Med. 1992;326:786-792.
30. Bais JE, Bartelsman JF, Bonjer HJ, et al. Laparoscopic or conventional Nissen fundoplication for gastro-oesophageal reflux
disease: randomised clinical trial. The Netherlands Antireflux
Surgery Study Group. Lancet. 2000; 355:170-174.
31. Catarci M, Gentileschi P, Papi C, et al. Evidence-based appraisal
of antireflux fundoplication. Ann Surg. 2004;239:325-337.
32. Lundell L, Abrahamsson H, Ruth M, et al. Long-term results
of a prospective randomized comparison of total fundic wrap
(NissenRossetti) or semifundoplication (Toupet) for gastrooesophageal reflux. Br J Surg. 1996;83:830-835.
33. Fernando HC, Lucketich JD, Christie NA, Ikramuddin S, Schauer
PR. Outcomes of laparoscopic Toupet compared to laparoscopic
Nissen fundoplication. Surg Endosc. 2002;16:905-908.
34. Horvath KD, Jobe BA, Herron DM, Swanstrom LL. Laparoscopic
Toupet fundoplication is an inadequate procedure for patients
with severe reflux disease. J Gastrointest Surg. 1999;3:583-591.
35. Strate U, Emmermann A, Fibbe C, Layer P, Zornig C. Laparoscopic fundoplication: Nissen versus Toupet two-year outcome
of a prospective randomized study of 200 patients regarding preoperative esophageal motility. Surg Endosc. 2008;22(1):21-30.
36. Zornig C, Strate U, Fibbe C, Emmermann A, Layer P. Nissen vs Toupet laparoscopic fundoplication. Surg Endosc. 2002;16(5):758-766.

5/21/2012 8:29:59 PM

CHAPTER 7

Inguinal Hernias
George Kasotakis and Marc A. de Moya

INTRODUCTION

repair at diagnosis of an asymptomatic or minimally symptomatic hernia in men.4,5


Fitzgibbons and colleagues followed prospectively 720 men
with reducible, minimally symptomatic or asymptomatic inguinal hernias after randomizing them to watchful waiting versus
a Lichtenstein tension-free repair with a prosthetic mesh.4 Primary outcome measures included worsening pain that interfered with daily activities at 2 years and a change in quality of
life metrics. Both groups developed pain interfering with activities in equal proportions (5.1% for the watchful waiting group
vs. 2.2% for the surgically treated, P = .52) in the intention-totreat analysis. However, 23% of watchful waiting patients had
crossed over to the intervention group by 2 years, citing worsening symptomatology, while 17% of the men assigned to hernia
repair crossed over to watchful waiting. Acute hernia accidents
(bowel obstruction without strangulation) were very rare at a
cumulative accident rate of 0.0018 events per patient-year, and
the patients that presented with those were managed successfully with urgent or semielective repairs without significant
complications.
Contrary to popular belief, there appears to be no penalty
for delaying operation in the men with minimally symptomatic
or asymptomatic hernias. A follow-up study on the same cohort
of patients assessed a range of objective measures after grouping patients in an immediate (<6 months) or delayed (>6
months) repair group.6 Operative time (64 vs. 67 min, P = .382),
complication (17% vs. 21.5%, P = .375), and recurrence rates
(1% vs. 3.1%), as well as patient satisfaction scores were similar between the two groups. Watchful waiting also appears to
be a cost-effective approach in managing minimally symptomatic patients with hernias7 that does not overburden patients
caregivers.8
The second trial took place in the United Kingdom and
included 160 males over the age of 55 years with asymptomatic
inguinal hernias.5 The primary outcome was pain at 1 year and
was similar between the two groups. Twenty-three of the 80
observation patients crossed over to repair due to pain or increase
in size. Hernia-related adverse events occurred only in three

Inguinal hernia repair represents the most commonly performed procedure by General Surgeons in the United States with
more than 770,000 repairs performed annually according to the
National Center for Health Statistics.1 These figures have significant socioeconomic ramifications, as both the condition and the
operation are associated with significant costs, morbidity, and
affl ict caregivers with a nontrivial burden. Until recently, a belief
commonly held among surgical training programs worldwide
purported that all inguinal hernias be repaired at diagnosis. The
reasoning behind this principle was twofold: early intervention
helped prevent complicating events with unacceptably high morbidity and mortality and allowed a less technically challenging
operation later on. However, a growing body of evidence suggests that the incidence of long-term complications after herniorrhaphy might be higher than previously thought, whereas
little is known about the natural history of hernias in men who
elect to not have an operation. Other controversial issues surrounding inguinal hernias include the routine use of mesh and
neurectomies to prevent recurrences and postoperative groin
pain respectively; the role of laparoscopy in unilateral, bilateral,
and recurrent hernias; optimal anesthesia selection for elective
herniorrhaphies; as well as factors predisposing recurrence are
among the topics discussed in this chapter.
1. Should asymptomatic hernias be repaired?
Although the question of whether to intervene in a patient with a
symptomatic hernia is easily answered, defining whether asymptomatic or minimally symptomatic patients warrant herniorrhaphy is much more difficult to tackle. The difficulty in this
undertaking lies in estimating the incidence of potentially lifethreatening hernia accidents, which appear to be lower than initially thought.2,3
Two prospective randomized controlled clinical trials have
been published in the last few years testing the hypothesis that a
strategy of watchful waiting is an acceptable alternative to routine
64

PMPH_CH07.indd 64

5/21/2012 8:45:32 PM

Inguinal Hernias

of those, and those were successfully managed with either an


urgent or elective herniorrhaphy after reduction. When the same
cohort of watchful waiting patients were followed for a period of
8 years, more than half (46 out of 80, 57.5%) elected to have their
hernias repaired, with most citing pain or increase in size as an
indication.9 Over the course of the 8 years, only two patients presented with acute hernia accidents, but neither required bowel
resection.
Given the aforementioned data, it appears that watchful waiting can be safely offered as an option in asymptomatic or minimally symptomatic men with inguinal hernias. (This is a Grade A
recommendation.)
2. Are mesh repairs better than native tissue repairs?
Since the late 19th century, when Bassini introduced the musculoaponeurotic approximation for inguinal hernias,10 the
primary tissue repair dominated the general surgical circles
with multiple variations (Shouldice, McVay) for over a century.
However, it was the unacceptably high recurrence rates of 10%
to 15% that prompted surgeons to look for new approaches.
The concept of tension-free repair that was first introduced
by Lichtenstein in the 1980s11 attracted attention early on, as
the addition of a mesh allowed reconstruction without the
need to pull layers of tissue under tension. This technically
easier approach continued to gain ground and multiple prospective randomized trials demonstrated its advantages over
the more traditional native tissue approaches, including a significant reduction in recurrence, the ability to perform under
local anesthesia, less pain, and more rapid return to work and
routine physical activities.
In 2001, the Cochrane Collaboration identified 20 prospective randomized or quasi-randomized clinical trials comparing
open mesh with nonmesh repairs.12 Of those, 17 used a flat mesh,
2 used plug-and-mesh, and 1 used a mesh placed preperitoneally.
The control groups in all studies included a variety of primary
tissue repairs. Despite the marked heterogeneity of the studies
included, it appeared that tension-free approaches required on
average 7 to 10 min less to perform than Shouldice repairs, but
1 to 4 min longer than the Bassini or McVay approach. There was
no statistically significant difference in minor postoperative complications, such as hematoma and seroma formation, or wound
infections. Serious complications, including femoral neurovascular bundle, spermatic cord, and visceral injuries were rare in
both groups. Hospital stay was slightly shorter in the mesh-based
repairs (OR 0.28; 95% CI 0.350.22), but significant heterogeneity was noted, likely reflecting variability in the local discharge
practices. Return to usual activities/work was also shorter in
the mesh-repair group (OR 0.81; 95% CI 0.730.91), and persisting pain or numbness in the genitofemoral area also favored the
tension-free approach (OR 0.68; 95% CI 0.470.98 and OR 0.7;
95% CI 0.291.72, respectively). Most importantly, recurrence
after mesh repair was consistently less frequently reported and
overall was reduced by between 50% and 75% (OR 0.37; 95% CI
0.260.51).
Subsequent prospective trials reported similar findings,
favoring mesh-based repairs over primary herniorrhaphies.13-15
Given the aforementioned data, we conclude that the use of mesh
during open inguinal hernia repair is associated with a significant
reduction in the recurrence risk and may act favorably in reducing postoperative groin pain and numbness and allowing earlier

PMPH_CH07.indd 65

65

return to preoperative functioning status. We therefore recommend routine application of mesh-based tension-free inguinal
herniorrhaphy. (This is a Grade A recommendation.)
3. Open or laparoscopic repair for inguinal hernias?
With the improved outcomes and subsequent popularization of
the Lichtenstein herniorrhaphy in the 1980s, and the explosion
of laparoscopic surgery in the 1990s, minimally invasive attempts
at tension-free repairs yielded encouraging results. The dominating techniques for laparoscopic inguinal hernia repair include the
transabdominal preperitoneal (TAPP) and the totally extraperitoneal (TEP) approaches.
In an attempt to answer what patient population might benefit
the most from a laparoscopic approach, multiple prospective clinical trials have been conducted. The EU Hernia Trialists Collaboration meta-analysis16 included 41 prospective randomized trials,
comparing laparoscopic to open tension-free herniorrhaphies.
This meta-analysis, that included a total of 7294 patients, demonstrated no significant difference in recurrence rates between the
laparoscopic and open approach (2.2% vs. 1.7%; OR 1.26; 95% CI
0.762.08), but showed a benefit in chronic groin pain in the former (OR 0.64; 95% CI, 0.520.78; P < .001).
Another meta-analysis17 that included 29 prospective randomized trials comprising a total of 5588 patients concluded that
short-term hernia recurrence was higher in the laparoscopically
managed by about 50%, although this result was not statistically
significant (OR 1.51; 95% CI 0.812.79). Postoperative complications were fewer in the laparoscopic group (OR 0.62; 95% CI
0.460.84), discharge from the hospital occurred earlier (3.43
hours, 95% CI 0.356.5 hours) and patients returned sooner to
normal activity by about 5 days (95% CI 3.515.96). The downside
was a slightly longer operative time (15.2 min longer; 95% CI 7.78
22.63 min).
The Veterans Affairs cooperative trial,18 one of the most
commonly cited trials in the United States, included 1983
patients who were randomized to a tension-free open versus a
laparoscopic repair and had some contradictory fi ndings: Recurrence at 2 years was lower in the open group (10.1% vs. 4.9%;
OR 2.2; 95% CI 1.53.2), but when the experience of the surgeon
was taken into account (>250 laparoscopic hernia repairs) the
recurrence was low and below 5% with either approach. Complication rate was slightly higher in the laparoscopic group (39%
vs. 33.4%; OR 1.3; 95% CI 1.11.3). The laparoscopically treated
patients reported less groin pain at 2 weeks postoperatively and
were able to return to their usual activity 1 day earlier than the
open group.
A more recent meta-analysis from Dedemadi and associates,19
which comprised 1542 patients undergoing laparoscopic versus
Lichtenstein repair, reported similar postoperative complications
and recurrence risk, but noted more recurrences when the TAPP
group was compared with the TEP (RR 3.25; 95% CI 1.327.9;
P = .01).
With regard to the optimal approach for managing recurrent
hernias, the Danish Hernia Database analysis,20 which included a
total of 67,306 prospectively recorded herniorrhaphies, demonstrated
a reduced reoperation rate if a laparoscopic approach was used for
the first recurrence (1.3%; 95% CI 0.43.0) compared to Lichtenstein
(11.3%; 95% CI 8.215.2). Another randomized trial comparing laparoscopic with open repairs for recurrent inguinal hernias21 demonstrated no difference in operative time or recurrence rates at 5 years

5/21/2012 8:45:32 PM

66

Surgery: Evidence-Based Practice

(18% for TAPP vs. 19% for Lichtenstein), but postoperative pain and
time to return to work were less with the former.
In summary, laparoscopic repairs offer an equivalent recurrence risk and a slightly earlier return to normal activity compared to open repairs, at the expense of longer operative times,
a greater equipment cost, and the need for general anesthesia.
Laparoscopic hernia repairs may offer an advantage for managing
recurrent hernias. (This is a Grade A recommendation.)
4. What are the risk factors for hernia recurrence?
Inguinal hernia recurrence is estimated at <5% with either a
Lichtenstein or laparoscopic repair when performed by experienced surgeons. Using a mesh to reapproximate the tissues in a
tension-free fashion is one of the best known methods employed
to significantly reduce recurrence, however, other risk factors for
hernia recurrence are not clearly delineated.
Mayagoitia et al.22 looked at 551 open hernia repairs performed with either a flat mesh (Lichtenstein), a Prolene Hernia
System or Mesh-Plug and concluded that recurrence was greater
with the latter (2.5%; RR 4.35; 95% CI 0.8522.23), yet the difference was not statistically significant. Previous herniorrhaphy, an
internal ring >4.5 cm and postoperative complications were also
found to be predictive of future recurrence.
Data from the Veterans Affairs trial23 demonstrated that
independent predictors of recurrence in the open repair group
were recurrent hernia, lack of a caregiver, and operating time <72
min. Among the patients treated laparoscopically, low surgeon
volume, active lifestyle, and a body mass index (BMI) <25 were
independent predictors.
Although the aforementioned data provide an idea for what
might contribute to a hernia recurrence, larger studies designed to
detect risk factors for recurrence should be performed. (This is a
Grade C recommendation.)
5. Should neurectomy be done routinely for prevention of postoperative groin pain?
Inguinodynia or chronic groin pain is one of the most dreaded complications following inguinal hernia repair and one that affects quality of life significantly.24 It is usually attributed to intraoperative nerve
damage or postoperative mesh-related fibrosis. Although traditional
surgical teaching holds that the nerves (ilioinguinal, iliohypogastric,
and genitofemoral) should be identified and preserved during repair,
recent cohort studies demonstrate that routine ilioinguinal nerve
sacrifice is associated with less chronic groin pain, while subjective
paresthesia is usually only temporary.25-27 In addition, ilioinguinal
neurectomy appears to be effective treatment for chronic groin pain
relief after open herniorrhaphy.28,29 However, results of prospective randomized clinical trials comparing the preservation versus
routine ilioinguinal neurectomy during open tension-free herniorrhaphies are conflicting.30-33 In light of the contradictory evidence
and while a Cochrane review is underway, preservation or routine
resection of the inguinal nerves should be left to the discretion of the
treating surgeon. (This is a Grade D recommendation.)
6. What is the optimal approach for bilateral hernias: open or
laparoscopic?
Concurrent repair of bilateral hernias may best be accomplished
laparoscopically. Long-term data demonstrate no difference
in recurrence between bilateral open compared with bilateral

PMPH_CH07.indd 66

laparoscopic repairs,34 and it appears that overall recurrence


(0.63% vs. 0.42%), postoperative complications (1.9% vs. 1.4%),
need for reoperation (0.5% vs. 0.43%), and time to return to previous activity (14 days vs. 14 days) are the same for unilateral versus
bilateral laparoscopic hernia repair.35
Postoperative complications, operative time, length of stay,
and groin pain appear to favor the laparoscopic approach compared to a Lichtenstein repair in three prospective randomized
trials.36-38 With these data in mind, and given the easier concurrent
access to both groins with the laparoscopic approach, we recommend the TAPP or the TEP for bilateral inguinal hernias. (This is a
Grade B recommendation.)
7. Is a plug necessary for mesh repairs?
Owing to the rising popularity of tension-free herniorrhaphies
over the last few decades, plug-and-patch prostheses, which consist of a plug covering the abdominal wall defect and a flat mesh
reinforcing the inguinal canal floor, were developed. The unique
feature of these repairs is that they require minimal dissection,
theoretically allowing for shorter operating times, reducing postoperative pain and affording earlier recovery. However, these
claims were not confirmed in rigorous clinical trials.
Dalenbck et al.39 randomized 472 men undergoing tensionfree herniorrhaphy to a Lichtenstein, Prolene Hernia System, or
plug-and-patch repair and followed them for 3 years. Although
operative time was slightly shorter in the latter two groups (40.4
1 min vs. 37.4 1 min and 35.5 1 min, respectively), the difference was not clinically significant. Postoperative complications, groin pain, return to full functional ability, and incidence
of recurrence did not differ between the groups. Similarly, Nienhuijs et al.40 randomized 334 patients to the same procedures, and
assessed quality of life and pain with the SF-36 and the visual
analogue scale at 2, 12, and 60 weeks after surgery. There were no
differences noted in either.
Plug-and-patch repair was compared to the Lichentein repair
by Frey et al. In 595 patients undergoing 700 primary or recurrent herniorrhaphies there was no differences found in recurrence
rates or postoperative complications.41 Similarly, postoperative
pain and time to recovery did not differ in 141 individuals studied
by Kingsnorth and colleagues.42 Operative time was significantly
shorter in the plug-and-patch group (32 vs. 37.6 min, P = .01), but
the difference was not clinically significant.
On the basis of the above findings, mesh-type selection should be
left to the surgeons discretion. (This is a Grade B recommendation.)
8. Local versus general anesthesia: does one confer a better
outcome than the other?
There have been several case series that have described the feasibility and safety of performing inguinal hernia repairs using local
anesthesia.43,44 Other larger database retrospective series have also
suggested that local anesthesia is underutilized as a method for
inguinal hernia repair.45 This has been extended to include laparoscopic hernia repairs,46 as well as open. We focus our question
on the use of local anesthesia versus general anesthesia in open
hernia repairs.
In 2001, Gonullu et al.47 performed a randomized clinical trial
directly comparing the use of local anesthesia to the use of general
anesthesia with a primary outcome of pulmonary effects, postoperative pain and fatigue, morbidity, and patient satisfaction. They
demonstrated a significant difference in pain relief but only at one

5/21/2012 8:45:32 PM

Inguinal Hernias

time point (8 hours post-op), but significantly improved CO2 clearance and oxygenation in the local anesthesia group. There was no
significant difference in patient satisfaction. They concluded that
local anesthesia provided slightly better pain control and improved
pulmonary function.
In 2003, ODwyer et al. performed a randomized trial comparing local and general anesthesia on 279 patients with ultimately 138
in each group. They found that intraoperative pain led to patient
dissatisfaction but postoperative pain was better at 6 hours than
the general anesthesia group. In addition, they noted that open
repair using general anesthesia was 4% more in cost than local
anesthesia. They concluded that there were no major differences in
patient recovery after local or general anesthesia and patients could
be presented with both options.48
This was followed by a randomized clinical trial comparing local, regional, and general anesthesia in Sweden. There were

67

approximately 200 patients in each arm of this multi-institutional


trial. The investigators seemed to standardize methods across the
institutions with favorable outcomes for those undergoing local
anesthesia. There was a significant decrease in admission duration, less immediate postoperative pain, and fewer problems with
urinary retention. However, there was no difference in pain after
the first day and no difference in time to normal activity. This
study suggested that local anesthesia should be used more frequently for open inguinal hernia repairs.49
There is currently a meta-analysis being performed by the
Cochrane Collaboration to evaluate the randomized trials concerning this question. However, after review of the evidence, it appears
as though the use of local anesthesia is safe and effective and in some
patients may be a better alternative to general anesthesia. However,
there is no compelling data that suggest that local anesthesia is superior to general anesthesia. (This is a Grade B recommendation.)

Clinical Question Summary


Question

Answer

1 Should asymptomatic hernias


be repaired?

Watchful waiting can be safely offered as an option


in asymptomatic or minimally asymptomatic men
with inguinal hernias.

4-9

2 Are mesh repairs better than


native tissue repairs?

Tension-free, mesh-based repairs are associated with


decreased recurrences and groin pain, and allow
for earlier return to full activity.

12-15

3 Does laparoscopy confer


an advantage over open
inguinal herniorrhaphies?

Laparoscopic repairs offer an equivalent recurrence


risk and slightly earlier return to normal activity
compared to open herniorrhaphies, at the expense
of longer operating times and the requirement for
general anesthesia.

16-21

4 What are the risk factors for


hernia recurrence?

Native tissue repairs, previous herniorrhaphy, an


internal ring >4.5 cm, postoperative complications,
lack of caregiver, and operating time <72 min were
independent predictors for recurrence after open
hernia repair. Low surgeon volume, active lifestyle,
and BMI <25 were predictors of recurrence in
those treated laparoscopically.

12-15, 22, 23

5 Should neurectomy for


prevention of postoperative
groin pain be done
routinely?

Preservation or routine resection of the inguinal


nerves should be left to the discretion of the
treating surgeon.

25-33

6 What is the optimal


approach for bilateral
hernias: open or
laparoscopic?

Postoperative complications, operative time, length


of stay, inguinal pain, and easier concurrent access
to both groins favor the laparoscopic approach for
bilateral hernia repairs.

34-38

7 Is a plug necessary for mesh


repairs?

The plug-and-patch repair does not offer a clinically


significant advantage over flat-mesh repairs in
terms of postoperative complications, chronic
groin pain, return to normal activity, and incidence
of recurrence. Mesh-type selection should be left
to the surgeons discretion.

39-42

8 Local versus general


anesthesia: Does one
confer a better outcome
than the other?

There are no long-term differences between the two.


In the short term, there seems to be less pain in
those who undergo local anesthesia.

46-49

PMPH_CH07.indd 67

Grade

References

5/21/2012 8:45:32 PM

68

Surgery: Evidence-Based Practice

REFERENCES
1. Rutkow IM, Robbins AW. Demographic, classificatory, and
socioeconomic aspects of hernia repair in the United States. Surg
Clin North Am. 1993;73:413-426.
2. Hair A, Paterson C, Wright D, Baxter JN, ODwyer PJ. What
effect does the duration of an inguinal hernia have on patient
symptoms? J Am Coll Surg. 2001;193:125-129.
3. Gallegos NC, Dawson J, Jarvis M, Hobsley M. Risk of strangulation in groin hernias. Br J Surg. 1991;78:1171-1173.
4. Fitzgibbons RJ, Jr., Giobbie-Hurder A, Gibbs JO, et al. Watchful
waiting vs repair of inguinal hernia in minimally symptomatic
men: a randomized clinical trial. JAMA. 2006;295:285-292.
5. ODwyer PJ, Norrie J, Alani A, Walker A, Duff y F, Horgan P.
Observation or operation for patients with an asymptomatic inguinal hernia: a randomized clinical trial. Ann Surg.
2006;244:167-173.
6. Thompson JS, Gibbs JO, Reda DJ, et al. Does delaying repair of an
asymptomatic hernia have a penalty? Am J Surg. 2008;195:89-93.
7. Stroupe KT, Manheim LM, Luo P, et al. Tension-free repair versus watchful waiting for men with asymptomatic or minimally
symptomatic inguinal hernias: a cost-effectiveness analysis. J
Am Coll Surg. 2006;203:458-468.
8. Gibbs JO, Giobbie-Hurder A, Edelman P, McCarthy M, Jr.,
Fitzgibbons RJ, Jr. Does delay of hernia repair in minimally
symptomatic men burden the patients family? J Am Coll Surg.
2007;205:409-412.
9. Chung L, Norrie J, ODwyer PJ. Long-term follow-up of patients
with a painless inguinal hernia from a randomized clinical trial.
Br J Surg. 2011;98(4):596-599.
10. Bassini E. Nuovo metodo sulla cura radicale dell ernia inguinale. Arch Soc Ital Chir. 1887;4:380.
11. Lichtenstein IL, Shulman AG, Amid PK, Montllor MM. The
tension-free hernioplasty. Am J Surg. 1989;157:188-193.
12. Scott NW, McCormack K, Graham P, Go PM, Ross SJ, Grant AM.
Open mesh versus non-mesh for repair of femoral and inguinal
hernia. Cochrane Database Syst Rev. 2002;(4):CD002197.
13. Miedema BW, Ibrahim SM, Davis BD, Koivunen DG. A prospective trial of primary inguinal hernia repair by surgical trainees.
Hernia. 2004;8:28-32.
14. Koninger J, Redecke J, Butters M. Chronic pain after hernia
repair: a randomized trial comparing Shouldice, Lichtenstein
and TAPP. Langenbecks Arch Surg. 2004;389:361-365.
15. Nordin P, Bartelmess P, Jansson C, Svensson C, Edlund G. Randomized trial of Lichtenstein versus Shouldice hernia repair in
general surgical practice. Br J Surg. 2002;89:45-49.
16. McCormack K, Scott NW, Go PM, Ross S, Grant AM. Laparoscopic techniques versus open techniques for inguinal hernia
repair. Cochrane Database Syst Rev. 2003:CD001785.
17. Memon MA, Cooper NJ, Memon B, Memon MI, Abrams KR.
Meta-analysis of randomized clinical trials comparing open
and laparoscopic inguinal hernia repair. Br J Surg. 2003;90:14791492.
18. Neumayer L, Giobbie-Hurder A, Jonasson O, et al. Open mesh
versus laparoscopic mesh repair of inguinal hernia. N Engl J
Med. 2004;350:1819-1827.
19. Dedemadi G, Sgourakis G, Radtke A, et al. Laparoscopic versus
open mesh repair for recurrent inguinal hernia: a meta-analysis
of outcomes. Am J Surg. 2010;200:291-297.
20. Bisgaard T, Bay-Nielsen M, Kehlet H. Re-recurrence after operation for recurrent inguinal hernia. A nationwide 8-year follow-up
study on the role of type of repair. Ann Surg. 2008;247:707-711.

PMPH_CH07.indd 68

21. Eklund A, Rudberg C, Leijonmarck CE, et al. Recurrent inguinal


hernia: randomized multicenter trial comparing laparoscopic
and Lichtenstein repair. Surg Endosc. 2007;21:634-640.
22. Mayagoitia JC, Prieto-Diaz Chavez E, Suarez D, Cisneros HA,
Tene CE. Predictive factors comparison of complications and
recurrences in three tension-free herniorraphy techniques. Hernia. 2006;10:147-151.
23. Matthews RD, Anthony T, Kim LT, et al. Factors associated with
postoperative complications and hernia recurrence for patients
undergoing inguinal hernia repair: a report from the VA Cooperative Hernia Study Group. Am J Surg. 2007;194:611-617.
24. van Hanswijck de Jonge P, Lloyd A, Horsfall L, Tan R, ODwyer
PJ. The measurement of chronic pain and health-related quality
of life following inguinal hernia repair: a review of the literature.
Hernia. 2008;12:561-569.
25. Zieren J, Tsigris C, Menenakos C. Open tension-free hernia
repair in soccer players: preservation or primary neurectomy of
the ilioinguinal nerve? Clin J Sport Med. 2007;17:398-400.
26. Dittrick GW, Ridl K, Kuhn JA, McCarty TM. Routine ilioinguinal nerve excision in inguinal hernia repairs. Am J Surg.
2004;188:736-740.
27. Tsakayannis DE, Kiriakopoulos AC, Linos DA. Elective neurectomy during open, tension free inguinal hernia repair. Hernia.
2004;8:67-69.
28. Loos MJ, Scheltinga MR, Roumen RM. Tailored neurectomy
for treatment of postherniorrhaphy inguinal neuralgia. Surgery.
2010;147:275-281.
29. Aasvang EK, Kehlet H. The effect of mesh removal and selective neurectomy on persistent postherniotomy pain. Ann Surg.
2009;249:327-334.
30. Malekpour F, Mirhashemi SH, Hajinasrolah E, Salehi N, Khoshkar A, Kolahi AA. Ilioinguinal nerve excision in open mesh
repair of inguinal herniaresults of a randomized clinical trial:
simple solution for a difficult problem? Am J Surg. 2008;195:735740.
31. Mui WL, Ng CS, Fung TM, et al. Prophylactic ilioinguinal
neurectomy in open inguinal hernia repair: a double-blind randomized controlled trial. Ann Surg. 2006;244:27-33.
32. Picchio M, Palimento D, Attanasio U, Matarazzo PF, Bambini
C, Caliendo A. Randomized controlled trial of preservation or
elective division of ilioinguinal nerve on open inguinal hernia
repair with polypropylene mesh. Arch Surg. 2004;139:755-758;
discussion 9.
33. Ravichandran D, Kalambe BG, Pain JA. Pilot randomized controlled study of preservation or division of ilioinguinal nerve in
open mesh repair of inguinal hernia. Br J Surg. 2000;87:11661167.
34. Kald A, Fridsten S, Nordin P, Nilsson E. Outcome of repair of
bilateral groin hernias: a prospective evaluation of 1,487 patients.
Eur J Surg. 2002;168:150-153.
35. Wauschkuhn CA, Schwarz J, Boekeler U, Bittner R. Laparoscopic
inguinal hernia repair: gold standard in bilateral hernia repair?
Results of more than 2800 patients in comparison to literature.
Surg Endosc. 2010;24:3026-3030.
36. Feliu X, Claveria R, Besora P, et al. Bilateral inguinal hernia
repair: laparoscopic or open approach? Hernia. 2011;15:15-18.
37. Sarli L, Iusco DR, Sansebastiano G, Costi R. Simultaneous repair
of bilateral inguinal hernias: a prospective, randomized study of
open, tension-free versus laparoscopic approach. Surg Laparosc
Endosc Percutan Tech. 2001;11:262-267.
38. Mahon D, Decadt B, Rhodes M. Prospective randomized trial
of laparoscopic (transabdominal preperitoneal) vs open (mesh)

5/21/2012 8:45:32 PM

Inguinal Hernias

39.

40.

41.

42.

PMPH_CH07.indd 69

repair for bilateral and recurrent inguinal hernia. Surg Endosc.


2003;17:1386-1390.
Dalenbck J, Andersson C, Anesten B, et al. Prolene Hernia System, Lichtenstein mesh and plug-and-patch for primary inguinal
hernia repair: 3-year outcome of a prospective randomised controlled trial. The BOOP study: bi-layer and connector, on-lay, and
on-lay with plug for inguinal hernia repair. Hernia. 2009;13:121129; discussion 231.
Nienhuijs S, van Oort I, Keeners-Gels M, Strobbe L, Rosman C.
Randomized clinical trial comparing the Prolene Hernia System,
mesh plug repair and Lichtenstein method for open inguinal
hernia repair. Br J Surg. 2005;92:33-38.
Frey DM, Wildisen A, Hamel CT, Zuber M, Oertli D, Metzger
J. Randomized clinical trial of Lichtensteins operation
versus mesh plug for inguinal hernia repair. Br J Surg. 2007;
94:36-41.
Kingsnorth AN, Porter CS, Bennett DH, Walker AJ, Hyland
ME, Sodergren S. Lichtenstein patch or Perfi x plug-and-patch
in inguinal hernia: a prospective double-blind randomized controlled trial of short-term outcome. Surgery. 2000;127:276-283.

69

43. Callesen T, Bech K, and Kehlet H. Feasibility of local infi ltration anaesthesia for recurrent groin hernia repair. Eur J Surg.
2001;167:851-854.
44. Makuria T, Alexander-Williams J, Keighley MRB. Comparison
between general and local anaesthesia for repair of groin hernias.
Ann R Coll Surg Engl. 1979;61:291-294.
45. Kehlet H, Nielsen M. Anaesthetic practice for groin hernia
repaira nation-wide study in Denmark 1998-2003. Acta Anaesthesiol Scand. 2005;49:143-146.
46. Frezza E, Ferzli G. Local and general anesthesia in the laparoscopic preperitoneal hernia repair. JSLS. 2000;4:221-224.
47. Gonullu NN, Cubukcu A, Alponat A. Comparison of local and
general anesthesia in tension-free (Lichtenstein) hernioplasty: a
prospective randomized trial. Hernia. 2002;6:29-32.
48. ODwyer PJ, Serpell MG, Millar K, Paterson C, et al. Local or
general anesthesia for open hernia repair: a randomized trial.
Ann Surg. 2003;237:574-579.
49. Nordin P, Zetterstrom H, Gunnarsson U, Nilsson E. Local,
regional, or general anaesthesia in groin hernia repair: multicentre randomised trial. Lancet. 2003;362:853-858.

5/21/2012 8:45:32 PM

CHAPTER 8

Esophageal Caustic Injury


Yoram Klein

INTRODUCTION

the damage is determined by the type, quantity, and concentration


of the offensive material together with the time of contact.4 Acid was
believed to cause less severe damage due to the quick eschar formation that prevents further deep penetration, and to its foul taste
and odor that limit incidental ingestion. Nevertheless, studies that
compared the damage potential of acid and alkali found that strong
acid ingestion is more harmful and causes more systemic and local
complications than ingestion of strong alkali substances.5

A caustic substance is defined as a chemical agent that causes tissue injury on contact. Most incidents are due to domestic accidents, affecting mainly children, or suicide attempts among adults.
Approximately 200,000 cases of caustic exposure are reported annually in the United States, with ingestion being responsible for 5000
of them. This form of exposure carries the highest mortality rate.1
The resulted chemical damage to the upper gastrointestinal tract is
a common problem in acute care surgery that might cause severe
acute and chronic complications. There are three clinical phases of
caustic ingestion: acute, latent, and chronic. In the first few days, the
efforts are focused on stabilizing the patient, establishing the need
for urgent surgical intervention, and an attempt to prevent complications. In severe cases, those who survived the acute phase will go
into the latent phase that might last for up to 4 weeks. In this phase,
the treatment is focused on maintaining adequate nutritional status while dealing with possible systemic complications. The chronic
phase is characterized by the formation of upper gastrointestinal
fibrosis with subsequent strictures. The increased occurrence of
esophageal malignancy among patients who sustained caustic ingestion is also considered a late complication of the chronic phase.
Each phase mandates different diagnostic approach and treatment modalities.2

DIAGNOSIS
1. Can early signs and symptoms predict the severity of the
esophageal or gastric damage?
In most cases anamnesis will make the diagnosis of caustic ingestion straight forward, whether it was an accidental or suicidal
event. The extent and location of tissue damage must be evaluated
in a timely fashion in order to plan the treatment and to anticipate
the prognosis. The patient may complain of burning sensation in
the mouth and throat, chest or epigastric pain, nausea and vomiting, odynophagia, or drooling. Respiratory symptoms might suggest exposure by inhalation or aspiration. Most published series
showed that early symptoms may not correlate with the severity of
the tissue damage.6,7 The absence of oropharyngeal signs of injury
upon direct visualization does not rule out significant gastrointestinal tract injury.8,9 It is estimated that around third of the patients
with clinically important esophageal or gastric mucosal injury
will not have signs of oropharengeal burns.
Answer: Although based on low level of evidence (mainly 2b),
the lack of signs and symptoms do not rule out significant esophageal or gastric mucosal injury (Grade B recommendation).
In extreme, rare cases, early laboratory results might show
acute renal failure, acute hepatic insufficiency or disseminated
intravascular coagulation.5 Nevertheless, usually, the laboratory
tests do not contribute to the diagnosis and dont have an important prognostic value.10 Early chest and abdominal plain films are
indicated to exclude pneumoperitoneum or pneumomediastinum. Some authors refer to mediastinal or peritoneal free air in

PATHOPHYSIOLOGY
Substances with pH of less than 2 or greater than 12 are considered
to be highly corrosive. Alkali ingestion causes liquefaction necrosis of the esophageal wall and lesser damage to the stomach due to
neutralization effect of the gastric acid. In severe cases, the injury
can result in perforation that will cause mediastinitis or peritonitis.
Over the next few days mucosal ulceration will take place. These
processes will lead to intense fibroblastic activity, collagen deposition, and finally to chronic stricture formation that might appear
after several weeks.3 Acid ingestion causes coagulation necrosis and
tends to affect the stomach more than the esophagus. The severity of
70

PMPH_CH08.indd 70

5/21/2012 8:46:17 PM

Esophageal Caustic Injury

Table 8.1 Zargars Grading Classification of Mucosal


Injury Caused by Ingestion of Caustic Substances
0

Normal examination

Edema and hypermia of the mucosa

2a

Superficial ulceration, erosions, friability, blisters,


exudates, hemorrhages, whitish membranes

2b

Grade 2a plus deep discrete or circumferential ulcerations

3a

Small scattered areas of multiple ulceration and areas of


necrosis with brown-black or grayish discoloration

3b

Extensive necrosis

the plain fi lms as the most important indication for emergency


operation in the acute phase.11 On the other hand, plain fi lms were
shown to have low diagnostic value.12 In cases where perforation
was not excluded by plain films, an oral contrast enhanced computerized tomography (CT) of the chest and abdomen is indicated.13
Evidence for severe damage in early CT scan, can also predict the
development of late complications in the chronic phase.14 The CT
has replaced the contrast swallow fluoroscopy, which are seldom
used today in the early management of caustic ingestion.
There is a consensus that the most valuable diagnostic tool
in the initial management of caustic ingestion is upper gastrointestinal endoscopy. As mentioned before, both lack of symptoms
and the absence of findings in the physical oropharyngeal examination do not exclude important gastrointestinal mucosal burn.
This makes the endoscopy an obligatory diagnostic study in every
case as stated by most authors. Still, few series reported selective
endoscopy, only for symptomatic patients, as a safe approach especially in the pediatric population.15,16 The upper endoscopy is used
to diagnose the presence of mucosal damage and to evaluate its
extent. The most commonly used endoscopic grading system for
early caustic gastrointestinal burns was established by Zargar in
1992 (Table 8.1). Grades I and IIA burns will heal most of the time
without any sequelae.3 However, strictures will develop in 70%
to 100% of patients with Grade IIB injury, with circumferential
ulceration, and Grade III injury, with necrosis. High grade injuries will also increase the likelihood of systemic complications in
the acute phase.17 Mortality that was directly related to the caustic
injury was reported mostly for patients with Grade 3 burns.
2. What is the proper timing for endoscopy in the acute phase
of caustic injury?
If perforation is not suspected, endoscopy should be done within
the first 24 hours as stated by most authors.17-19 Few others used
96 hours as the target for early endoscopy.20 No study that specifically examined the correct timing for endoscopy was found till
date (Level of evidence 3).
Answer: Endoscopy is mandatory and safe after the initial
stabilization of the patient. It should be done within the fi rst
24 hours after admission to the emergency department. (Grade of
recommendation C.)

MEDICAL TREATMENT
If no signs of injury were found in endoscopy, oral feeding can
be resumed as soon as the patient can swallow saliva, and the
patient can be safely discharged from the hospital. In more

PMPH_CH08.indd 71

71

severe burns the fi rst priority is to maintain a patent airway.


Any sign of oropharyngeal, glotic edema or upper airway burn
should be addressed as a pending airway obstruction and prompt
tracheal intubation is indicated. If the patient is showing signs
of hemodynamic compromise, rapid fluid resuscitation should
be started. Early consultation with the regional toxicology consulting service is also warranted. An attempt to clear the stomach from the offending corrosive agent by inducing vomiting is
strictly contraindicated to avoid the risk of repeated esophageal
exposure. Despite being practiced in several centers, irrigation
of the stomach with neutralizing agents was found to be ineffective.21 As mentioned before, any sign of perforation mandates immediate surgical exploration. Patients in whom such a
catastrophe was ruled-out are candidates for medical supportive
care. The patient should be on a nothing per os order. Insertion
of nasogastric tube (NGT) should be placed. Nevertheless, no
conclusive data to support this doctrine can be found in the literature. Except for one retrospective study from the 1980s, no
other studies have challenged the need for NGT. Th is study actually attributed the low occurrence of late stricture formation to
early insertion of NGT.22 In the acute and latent phase of patients
with severe esophageal burn (Grade 2B and 3), the NGT can be
an important channel for enteral feeding. In cases where gastric injury prevents using the NGT for nutrition support, either
intravenous total parenteral nutrition or enteral nutrition via
surgically inserted feeding jejeunostomy is indicated.
3. Should prophylactic antibiotic therapy be used?
Prophylactic antibiotic is used in several series in order to reduce
occurrence of systemic infection. Like in other clinical scenarios,
prophylactic antibiotic was never found to have any benefit.23,24
(Level of evidence 2B.)
Answer: according to the current evidence, the use of prophylactic antibiotic therapy is not recommended (Grade B).
4. Should corticosteroids be used?
The most common late sequel of caustic ingestion is the formation
of esophageal stricture. Up to third of the patients will develop
this complication between 2 weeks and several years after the
exposure.3 The use of corticosteroids in an attempt to attenuate
the formation of stricture was suggested. Prospective controlled
study of this intervention in the pediatric population failed to
show any benefit.25 In a pooled review of 572 patients, the use
of corticosteroids in Grade 2 and 3 esophageal burns was found
to be ineffective. Th is review also found evidence for increased
morbidity in patients who received corticosteroids therapy.26
Repeated pooled analysis on a larger database showed the same
results.27 (Level of evidence 2C.) There are also scattered reports
on the use of local injection of corticosteroids directly into the
stenotic lesion as an adjuvant to endoscopic dilation. One study
showed the potential of this technically challenging procedure
to reduce the severity of future stenosis.28 Anyhow this intervention never gained popularity and it is rarely used in the clinical
setting.
Answer: The use of corticosteroids for prevention of late
esophageal stricture is not recommended (Grade of recommendation B).
The topical application of the anti fibroblastic agent mitomycin C was investigated in the past several years with inconclusive
results, and continued future investigation is needed.29

5/21/2012 8:46:17 PM

72

Surgery: Evidence-Based Practice

Endoscopic interventions offer the most effective treatment for esophageal stricture. This can be achieved with
bougienage, balloon dilatation of stent placement. Endoscopic
procedure is considered to be dangerous in days 7 to 21 due
to the increased fragility of the damaged esophageal wall and
the increased risk of perforation. 30 Despite this, there were few
reports on earlier endoscopic dilatation as a safe and effective
preventive measure. 31,32 In a later stage, when fibrotic strictures
are already formed in the esophagus, repeated dilatation is
the treatment of choice. Successful and safe repeated selfbougienage by the patients in their home was also described. 33,34
The use of endoluminal stents to prevent stricture formation
was described anecdotally in the literature since the late seventies, with various successes. 35,36 No recommendations can be
made in this stage regarding this intervention due to the paucity of data.

attempts. Lately a defi nition for failed endoscopic dilatation


was introduced. Resistant stricture was defi ned as inability to
reach a minimal luminal diameter of 14 mm during 2 weeks
of endoscopy attempts. Recurrent stricture was defi ned as failure to maintain this diameter for more than 4 weeks after it
was achieved. 37 It is estimated that approximately half of the
patients with established esophageal stricture will need an
esophagectomy due to failed endoscopic dilatations. 38 (Level of
evidence 3)
Answer: Failed endoscopic dilatation to reach or maintain a
minimal esophageal luminal diameter of 14 mm, together with
clinical incapacitating dysphagia, is an indication for surgery for
esophageal replacement (Grade C).

SURGERY

Animal studies and physiology research have shown that collagen deposition and fibrotic changes are concluded at least 6
months after the injury. 39 Th is fact suggests that defi nitive reconstruction should be attempted at least 6 months after the primary insult, in order to prevent restenosis after the operation.40
Th is is our policy as well, despite reports of earlier successful
esophageal replacement after as early as 2 months postinjury.41
(Level of evidence 3.)
Answer: Definitive surgical procedure for chronic esophageal
stricture after a caustic injury should be attempted after at least 6
months after the incident (Grade C).
Another possible indication for late surgical intervention is
iatrogenic esophageal perforation that can complicate endoscopic
dilatation in up to 0.8% in this setting. Lastly, surgery might be
indicated for the development of esophageal malignancy, which
affected postcaustic ingestion patients 1000 times more frequently
than the general population.42

In the acute and latent phases, signs of perforation or intractable


severe sepsis are indications for immediate surgical intervention.
The main objective is source control of the sepsis, which is achieved
in most instances by esophagectomy with wide debridement and
drainage of necrotic tissue in the mediastinum and/or peritoneal
cavity. Usually, the patients systemic and local condition precludes
definitive reconstruction of the upper gastrointestinal tract, and
cervical esophagostomy with feeding jejeunostomy are indicated.
According to the authors experience, the transhiatal esophagectomy
is the most appropriate and safe procedure in this circumstances.
5. What is the indication for surgery in the chronic phase?
In the chronic stage the most common indication for surgery
are strictures that are resistant to repeated endoscopic dilatation

6. What is the proper timing for definitive surgical intervention in the chronic phase?

Clinical Question Summary


Question

Answer

1 Can early signs and symptoms predict the


severity of caustic injury?

The lack of signs and symptoms do not


rule out significant esophageal or
gastric mucosal injury.

6, 7, 8, 9

2 What is the proper timing for endoscopy in


the acute phase of caustic injury?

Endoscopy should be done within the first


24 hours after caustic ingestion.

17-20

3 Should prophylactic antibiotic therapy be


used?

The use of prophylactic antibiotic therapy


is not recommended.

23, 24

4 Should corticosteroids be used?

The use of corticosteroids for prevention


of late esophageal stricture is not
recommended.

25-27

5 What is the indication for surgery in the


chronic phase?

Failed endoscopic dilatation is an


indication for surgery for esophageal
replacement.

37

6 What is the proper timing for definitive


surgical intervention in the chronic phase?

Definitive surgical procedure should be


attempted at least 6 months after the
incident.

39, 40

PMPH_CH08.indd 72

Grade of
Recommendation

References

5/21/2012 8:46:17 PM

Esophageal Caustic Injury

REFERENCES
1. Watson WA, Litovitz TL, Rodgers GC Jr, et al. 2004 Annual report
of the American Association of Poison Control Centers Toxic Exposure Surveillance System. Am J Emerg Med. 2005;23(5):589-666.
2. Katzka DA. Caustic injury to the esophagus. Curr Treat Options
Gastroenterol. 2001;4(1):59-66.
3. Zargar SA, Kochlar R, Nagi B, et al. Ingestion of corrosive alkalis. Spectrum of injury to upper gastrointestinal tract and natural history. Am J Gastroenterol. 1992;87:337-341.
4. Mamede RC, de Mello Filho FV. Ingestion of caustic substances
and its complications. Sao Paulo Med J. 2001;119(1):10-15.
5. Poley JW, Steyerberg EW, Kuipers EJ, et al. Ingestion of acid and
alkaline agents: outcome and prognostic value of early upper
endoscopy. Gastrointest Endosc. 2004;60(3):372-377.
6. Sarfati E, Gossot D, Assens P, Celerier M. Management of caustic
ingestion in adults. Br J Surg. 1987;74(2):146-148.
7. Gorman RL, Khin-Maung-Gyi MT, Klein-Schwartz W, et al.
Initial symptoms as predictors of esophageal injury in alkaline
corrosive ingestions. Am J Emerg Med. 1992;10(3):189-194.
8. Previtera C, Giusti F, Guglielmi M. Predictive value of visible
lesions (cheeks, lips, oropharynx) in suspected caustic ingestion:
may endoscopy reasonably be omitted in completely negative
pediatric patients? Pediatr Emerg Care. 1990;6(3):176-178.
9. Gaudreault P, Parent M, McGuigan MA, Chicoine L, Lovejoy
FH Jr. Predictability of esophageal injury from signs and symptoms: a study of caustic ingestion in 378 children. Pediatrics.
1983;71(5):767-770.
10. Chen TY, Ko SF, Chuang JH, et al. Predictors of esophageal stricture in children with unintentional ingestion of caustic agents.
Chang Gung Med J. 2003;26(4):233-239.
11. Chou SH, Chang YT, Li HP, Huang MF, Lee CH, Lee KW. Factors predicting the hospital mortality of patients with corrosive
gastrointestinal injuries receiving esophagogastrectomy in the
acute stage. World J Surg. 2010;34(10):2383-2388.
12. Chiu HM, Lin JT, Huang SP et al. Prediction of bleeding and
stricture formation after corrosive ingestion by EUS concurrent
with upper endoscopy. Gastrointest Endosc. 2004;60:827-833.
13. Lee M. Caustic ingestion and upper digestive tract injury. Dig
Dis Sci. 2010;55:1547-1549.
14. Ryu HH, Jeung KW, Lee BK, et al. Caustic injury: can CT grading system enable prediction of esophageal stricture? Clin Toxicol (Phila). 2010;48(2):137-142.
15. Crain EF, Gershel JC, Mezey AP. Caustic ingestions. Symptoms
as predictors of esophageal injury. Am J Dis Child. 1984;138(9):
863-865.
16. Gupta SK, Croffie JM, Fitzgerald JF. Is esophagogastroduodenoscopy necessary in all caustic ingestions? J Pediatr Gastroenterol
Nutr. 2001;32(1):50-53.
17. Cheng HT, Cheng CL, Lin CH, et al. Caustic ingestion in adults:
the role of endoscopic classification in predicting outcome. BMC
Gastroenterol. 2008;8:31.
18. Poley JW, Steyerberg EW, Kuipers EJ, et al. Ingestion of acid and
alkaline agents: outcome and prognostic value of early upper
endoscopy. Gastrointest Endosc. 2004;60(3):372-377.
19. Tohda G, Sugawa C, Gayer C, Chino A, McGuire TW, Lucas
CE. Clinical evaluation and management of caustic injury in
the upper gastrointestinal tract in 95 adult patients in an urban
medical center. Surg Endosc. 2008;22(4):1119-1125. Epub 2007
Oct 27.
20. Zargar SA, Kochhar R, Mehta S, Mehta SK. The role of fiberoptic endoscopy in the management of corrosive ingestion and
modified endoscopic classification of burns. Gastrointest Endosc.
1991;37(2):165-169.

PMPH_CH08.indd 73

73

21. Kirsh MM, Peterson A, Brown JW, et al. Treatment of caustic injuries of the esophagus. A ten-year experience. Ann Surg.
1978;188:675-678.
22. Wijburg FA, Beukers MM, Heymans HS, Bartelsman JF, den Hartog Jager FC. Nasogastric intubation as sole treatment of caustic
esophageal lesions. Ann Otol Rhinol Laryngol. 1985;94:337-341.
23. Karnak I, Tanyel FC, Bykpamuku N, Hisnmez A. Combined use of steroid, antibiotics and early bougienage against
stricture formation following caustic esophageal burns. J Cardiovasc Surg (Torino). 1999;40(2):307-310.
24. Rao RB, Hoff man RS. Caustics and batteries. In: Goldfrank LR,
ed. Goldfranks Toxicologic Emergencies. Norwalk, CT: Appleton
& Lange; 1998:1399-1428.
25. Pelclov D, Navrtil T. Do corticosteroids prevent oesophageal
stricture after corrosive ingestion? Toxicol Rev. 2005;24:125-129.
26. Anderson KD, Rouse TM, Randolph JG. A controlled trial of
corticosteroids in children with corrosive injury of the esophagus. N Engl J Med. 1990;323:637-640.
27. Fulton JA, Hoff man RS. Steroids in second degree caustic burns
of the esophagus: a systematic pooled analysis of fift y years of
human data: 19562006. Clin Toxicol. 2007;45:402-408.
28. Kochhar R, Ray JD, Sriram PV, et al. Intralesional steroids augment the effects of endoscopic dilation in corrosive esophageal
strictures. Gastrointest Endosc. 1999;49:509-513.
29. Pace F, Antinori S, Repici A. What is new in esophageal injury
(infection, drug-induced, caustic, stricture, perforation)? Curr
Opin Gastroenterol. 2009;25(4):372-379.
30. Katzka DA. Caustic injury to the esophagus. Curr Treat Options
Gastroenterol. 2001;4(1):59-66.
31. Tiryaki T, Livanelioglu Z, Atayurt H. Early bougienage for relief
of stricture formation following caustic esophageal burns. Pediatr Surg Int. 2005;21(2):78-80.
32. Kochhar R, Poornachandra KS, Dutta U, Agrawal A, Singh K.
Early endoscopic balloon dilation in caustic-induced gastric
injury. Gastrointest Endosc. 2010;71(4):737-744.
33. Bapat RD, Bakhshi GD, Kantharia CV, et al. Self-bougienage:
long-term relief of corrosive esophageal strictures. Indian J Gastroenterol 2001;20(5):180-182.
34. Lee HJ, Lee JH, Seo JM, Lee SK, Choe YH. A single center experience
of self-bougienage on stricture recurrence after surgery for corrosive
esophageal strictures in children. Yonsei Med J. 2010;51(2):202-205.
35. Mills LJ, Estrera AS, Platt MR. Avoidance of esophageal stricture following severe caustic burns by the use of an intraluminal
stent. Ann Thorac Surg. 1979;28:60-65.
36. Wang RW, Zhou JH, Jiang YG, et al. Prevention of stricture with
intraluminal stenting through laparotomy after corrosive esophageal burns. Eur J Cardiothorac Surg. 2006;30(2):207-211.
37. Kochman ML, McClave SA, Boyce HW. The refractory and the
recurrent esophageal stricture: a definition. Gastrointest Endosc.
2005;62:474-475.
38. Adegboye VO, Brimmo A, Adebo OA. Transhiatal esophagectomy in children with corrosive esophageal stricture. Afr J Med
Med Sci. 2000;29:223-226.
39. Demirbilek S, Aydin G, Yucesan S, Vural H, Bitiren M. Polyunsaturated phosphatidylcholine lowers collagen deposition in a rat model
of corrosive esophageal burn. Eur J Pediatr Surg. 2002;12:8-12.
40. Han Y, Cheng QS, Li XF, Wang XP. Surgical management of
esophageal strictures after caustic burns: a 30 years of experience. World J Gastroenterol. 2004;10(19):2846-2849.
41. Munoz-Bongrand N, Gornet JM, Sarfati E. Diagnostic and therapeutic management of digestive caustic burns. J Chir. 2002;139:72-76.
42. Zwischenberger JB, Savage C, Bidani A. Surgical aspects of
esophageal disease: perforation and caustic injury. Am J Respir
Crit Care Med. 2002;165(8):1037-1040.

5/21/2012 8:46:17 PM

CHAPTER 9

Esophageal Tumors
Daniel S. Oh and Steven R. DeMeester

BACKGROUND

1. How should esophageal cancer be clinically staged?

Cancer of the esophagus represents 1.1% of cancers in the


United States, with 16,640 new cases of esophageal cancer in
this country in 2010.1 Despite being a relatively uncommon
cancer in the population at large, it is highly lethal, and has
the worst 5-year survival of all malignancies after pancreatic
cancer and lung cancer, respectively.1 Moreover, the incidence
of esophageal cancer in the Western world is growing faster
than all other solid organ malignancies combined, with a 600%
rise in incidence between 1975 and 2001. 2 This rapid rise in
incidence has been accompanied by a shift in the histologic
type of esophageal cancers, with squamous cell carcinoma on
the decline and adenocarcinoma on the rise such that in the
United States adenocarcinoma has represented the majority of
esophageal tumors since 1990. 3 It is well established that the
primary risk factor for esophageal adenocarcinoma is chronic
gastroesophageal reflux disease, a condition that has reached
epidemic proportions in the United States. Caucasian men are
the demographic group at the highest risk for this disease, and
esophageal cancer is the fifth leading cause of cancer deaths
in American men aged 40 to 79.1 In the setting of severe gastroesophageal reflux disease, a well-characterized sequence of
reflux induces mucosal injury such that the damaged squamous
mucosa of the distal esophagus is replaced with columnar
mucosa that can eventually result in intestinal metaplasia or
Barretts esophagus. With further progression of disease, Barretts esophagus can become dysplastic, and ultimately progress to invasive adenocarcinoma.
As with all gastrointestinal malignancies, complete surgical
extirpation represents the cornerstone of therapy, but this can
only have a benefit when the disease is locoregional. Unfortunately, only 23% of newly diagnosed esophageal cancer patients
present with localized disease, eliminating the possibility of curative resection in the majority of patients.1 Patients with distant
metastases have a poor prognosis, with a 5-year survival of 3%.1
Thus, in the evaluation of the newly diagnosed patient with esophageal cancer, staging becomes the first priority.

Clinical staging is accomplished through imaging modalities,


and it allows for the rational selection of appropriate therapy
while minimizing the risk of performing unnecessary surgery in
patients with systemic disease. It should be noted that in 2010, the
7th edition of the American Joint Committee on Cancer (AJCC)
TNM staging system was published with significant changes, particularly with regard to N staging.4 Rather than being a binary
staging system of nodal metastases (yes versus no), the new staging system stresses the number of involved nodes as being important for outcome, similar to gastric cancer. Further, celiac nodal
involvement is no longer considered synonymous with systemic
metastases or M1 disease, and the location of the involved nodes
is not considered important in the new system.
Over the past decade, two imaging modalities have emerged as
the mainstays of clinical staging of esophageal cancer, endoscopic
ultrasound (EUS) and integrated PET/CT scans. Combined, these
complementary modalities have been the most reliable and informative ways to assess the depth of primary tumor invasion (T),
the extent of nodal metastases (N), and systemic metastases (M).
For assessment of the T stage of the esophageal lesion, EUS has
been shown to be the most accurate imaging modality currently
available. The inherent difficulty of assessing the T stage is primarily due to the thin nature of the esophagus, where the difference
between T1 and T3 is a matter of a few millimeters. Endoscopic
ultrasound is the only modality with the resolution that allows
such a fine distinction to be made, as the resolution of CT scans
and MRI cannot distinguish the different layers of the esophagus.
At the typical 7.5-Mhz frequency, the modality gives a depth of
field of approximately 5 cm. At this time, the best available data
examining the performance of EUS in tumor staging comes from a
recent meta-analysis of 43 published studies.5 The pooled sensitivity of EUS for T stage ranged from 81.4% to 92.4% and the pooled
specificity ranged from 94.4% to 99.4%, depending on the T stage
examined (Table 9.1). EUS is still somewhat limited in sensitivity for earlier stage tumors, especially distinguishing between T1
and T2 tumors, with equal chances of under- or over-staging intramural disease. Further, with the possibility of using nonsurgical
74

PMPH_CH09.indd 74

5/21/2012 8:47:46 PM

Esophageal Tumors

75

Table 9.1 Summary of Imaging Modalities Used in the Clinical Staging of Esophageal Cancer
Stage

Modality

Number of Studies

Pooled Sensitivity
(95% CI)

T1

EUS

43

0.82 (0.780.85)

0.99 (0.991.0)

T2

EUS

43

0.81 (0.780.85)

0.96 (0.950.97)

T3

EUS

43

0.91 (0.890.93)

0.94 (0.930.95)

T4

EUS

43

0.92 (0.890.95)

0.97 (0.970.98)

EUS

44

0.85 (0.830.86)

0.85 (0.830.86)

EUS-FNA

0.97 (0.920.99)

0.96 (0.910.98)

CT

17

0.5 (0.410.60)

0.83 (0.770.89)

PET

10

0.57 (0.430.70)

0.85 (0.760.95)

CT

0.52 (0.330.71)

0.91 (0.860.96)

PET

0.71 (0.620.79)

0.93 (0.890.97)

options for intramucosal or T1a disease, it is critical to note that


EUS does not have reliable results in accurately assessing these
early tumors despite using higher frequencies of up to 20 MHz.6
As an alternative, an endoscopic mucosal resection with resection
of both the mucosa and submucosa is the most accurate method of
assessing the depth of invasion of a superficial lesion.7 In reviewing
the data on EUS, it should also be noted that like any ultrasound
based modality, the results are highly operator dependent. Finally,
an additional limitation of EUS is that in up to one third of patients
with newly diagnosed esophageal cancer, the tumor is too bulky to
allow passage of the EUS endoscope, although in these scenarios
dilatation to allow accommodation of the EUS instrument may be
considered.
EUS is also the best imaging modality available for assessment of the N stage (Table 9.1). Since the majority of potential
lymph node metastases are located in the region of the esophagus, EUS provides a clear image of the area directly surrounding
the esophagus to assess lymph nodes in the mediastinum and the
abdomen. The best available data on the performance of EUS on
N staging comes from a recent meta-analysis of 44 publications,
which showed a pooled sensitivity of 84.7% and a specificity of
84.6%.5 The inferior performance of EUS in assessing N status
compared to its performance in assessing T status is primarily
due to reliance on the visible characteristics of the nodes. Reliance on visible characteristics such as size greater than 1 cm or
a homogeneous appearing lymph node are not highly reliable in
predicting the pathologic presence of microscopic tumor cells in
a lymph node. More information is achieved when EUS is combined with fine needle aspiration (FNA) to biopsy the node of
interest under real-time ultrasound guidance. On the basis of
a meta-analysis of four publications specifically examining the
performance of EUS-FNA, the pooled sensitivity and specificity
of EUS-FNA to detect metastatic nodes rose to 96.7% and 95.5%,
respectively.5 A major limitation for FNA, however, is that the
needle may potentially have to traverse the tumor to access the
lymph node of interest, which can produce a false positive result,
and therefore FNA is not possible in these circumstances. The use
of a protected needle may be considered to minimize this risk of
contamination.
Perhaps the most important aspect of clinical staging for the
surgeon is to detect systemic metastases or determine the M stage,
since the presence of such disease will preclude curative resection.

PMPH_CH09.indd 75

Pooled Specificity
(95% CI)

Reference

The modality that has shown to be best suited for this purpose is
the FDG-PET scan. As evident from a recent meta-analysis of nine
publications, PET scans have improved the sensitivity of detecting
systemic metastases compared to CT scans, although they have comparable specificity (Table 9.1).8 More recently, integrated PET-CT
scans have become commonplace due to the convenience of having
the patient undergo both scans in the same setting. Further, this
allows accurate coregistration of the PET and CT data for improved
interpretation of the images. Due to this convenience, integrated
PET-CT has gained popularity as the preferred modality of assessing for distant metastases. Two retrospective single-institutional
studies have demonstrated improved sensitivity and specificity of
integrated PET-CT compared to separate PET and CT scans.9,10
2. Is transthoracic esophagectomy superior to transhiatal
esophagectomy and does extended lymphadenectomy improve
survival?
There is possibly no other area of alimentary tract surgery with
more debate than the subject of the esophagectomy. There are
several different techniques of esophagectomy that are performed
by esophageal surgeons around the world, and the fundamental
argument has centered on whether there is any oncologic benefit
of the more extensive lymphadenectomy that can be achieved in
a transthoracic approach compared to a transhiatal approach.
Proponents of the transthoracic approach argue that it is inherently safer to dissect the esophagus under direct visualization in
the chest and allows for an extended lymph node dissection in
the mediastinum that is beneficial in patients with locoregional
esophageal cancer. Opponents to the transthoracic approach
believe the benefits are outweighed by the additional morbidity
introduced by a thoracotomy, with its attendant pain and pulmonary compromise. In addition, the concept of extended lymphadenectomy to improve survival remains controversial.
To date, there have been four prospective, randomized trials
published from 1993 to 2007 comparing the outcomes of transhiatal
and transthoracic resections.11-15 Three of these studies are of
suboptimal quality due to the inclusion of patients who received
adjuvant therapy and a mixture of squamous cell and adenocarcinoma pathology. Although no difference in overall survival was
demonstrated in the three studies by Goldminc et al., Chu et al.,
and Jacobi et al., there is a concern for a type II error given the

5/21/2012 8:47:47 PM

76

Surgery: Evidence-Based Practice

small sample sizes, with a mean of only 23 patients in each arm


in these trials. The best data on this issue comes from Hulscher
and colleagues in the Netherlands.14,15 This prospective, randomized control trial was conducted in patients with esophageal
adenocarcinoma and were treated by primary surgical resection
alone without neoadjuvant or adjuvant chemotherapy or radiation. A total of 220 patients were randomized to either transhiatal
or transthoracic esophagectomy with a minimum follow-up of
5 years. All patients had adenocarcinoma of the distal esophagus
or the gastroesophageal junction. The transhiatal resection
removed a mean of 16 nodes compared to 31 nodes with a transthoracic en bloc resection. After a minimum of 5 years of
follow-up, it was observed that while survival was similar between
the two approaches overall (34% vs. 36%, P = .71), when outcome
was assessed in patients with locoregional disease, defined as having 1 to 8 metastatic nodes in the specimen, there was a significant
improvement in disease-free and overall survival with the transthoracic approach and its more extensive lymphadenectomy. These
patients, who represented the majority of patients in the trial,
benefited from an absolute improvement in survival of 20% at
5 years (19% vs. 39% for transhiatal and transthoracic resections,
respectively). To place this in context, there have been few innovations in the treatment of esophageal cancer that have resulted in a
20% absolute improvement in 5-year survival in the current era,
either by the introduction of induction chemotherapy or the addition of adjuvant therapy. Importantly, there was no difference in
in-hospital mortality between the two groups, although the complication rate was higher with the transthoracic approach.
Of interest is the observation that no survival advantage was
observed in the remaining patients based on the transthoracic
approach, in the patients without any nodal involvement, or in the
patients who had more than eight metastatic nodes present. The
finding that transthoracic esophagectomy with extended lymphadenectomy benefited only patients with one to eight metastatic
nodes is likely due to the risk of systemic disease that is reflected
by the extent of nodal metastases. When more than eight nodes are
present, systemic disease is nearly always present, and the type of
resection does not matter since locoregional control with an extensive operation is outweighed by the burden of systemic disease. At
the other extreme, when the tumor is early such that there are no
nodal metastases, there is little advantage in removing more nodes
because the disease is confined to the esophagus. This observation has been noted in the surgical experience with intramucosal
tumors, in which lymphadenectomy does not provide any benefit
to reducing risk or recurrence or survival.16,17 Thus, the survival
benefit of the transthoracic approach in patients with locoregional
disease, which is reflected by the presence of one to eight involved
nodes, is due to the superior locoregional control with the more
extensive lymphadenectomy. In this trial, locoregional recurrence
was 25% in those who underwent transthoracic resection compared to 42% in those who had a transhiatal resection.
Underlying the improved survival associated with the transthoracic approach to esophagectomy is the principle of extended
lymphadenectomy. Although transthoracic esophagectomy has been
shown to be superior to transhiatal resection, not all transthoracic
operations are equivalent. There is convincing data from around
the world indicating that the more lymph nodes removed allows for
both accurate staging as well as a therapeutic benefit. An analysis of
SEER data from the United States between 1973 and 2003 included
3568 patients who underwent surgical resection for esophageal cancer.18 In this epidemiologic study, the number of nodes removed as a

PMPH_CH09.indd 76

continuous variable was an independent predictor of survival,


with the greater number of nodes removed being associated with
improved survival. The optimal number of nodes to be removed was
observed to be 30 lymph nodes. This benefit on overall survival
was observed in both N0 patients as well as in N1 patients with metastatic lymph nodes (6th AJCC Staging System). The survival advantage gained even among N1 patients is evidence that the improved
survival is not a function of stage cleansing or stage migration.
Two recent publications have examined the optimal number
of nodes to remove during esophagectomy using large multiinstitutional, international databases.19,20 Combined, these two publications encompass nearly 7000 patients with esophageal cancer
who were treated with surgery alone at 21 high-volume academic
centers from North America, Europe, and Asia. The exclusion of
patients who received neoadjuvant and adjuvant therapy allows
for a reliable estimate of the effect of surgery alone on the natural
history of the disease, and the effect of various degrees of lymphadenectomy on outcomes. Similar to the SEER data from the United
States, these international studies showed that the number of nodes
removed was indeed an independent predictor of outcome, with
improved survival following more extensive lymphadenectomy. In
one study, the optimal cutoff was found to be 23 to 29 nodes and
in the other study the cutoff was 30 nodes. Each study used different statistical modeling in different patient populations, but both
came to a similar conclusion, that a minimum of between 23 and 30
nodes are required to maximize the benefit of surgery, and this is in
line with the conclusions from the SEER data. Interestingly, there
continued to be a benefit with increasing number of nodes removed
above the threshold, and there was no number at which there was
no additional benefit on survival.19 Notably, only the transthoracic
en bloc or modified McKeown esophagectomy allowed for the reliable removal of this minimum threshold compared to a simple Ivor
Lewis esophagectomy or transhiatal approach.
3. Does a pyloric drainage procedure improve outcomes of
esophagectomy?
Another issue related to esophagectomy that generates heated
debate is whether or not to perform a pyloroplasty or pyloromyotomy at the time of operation. Since a truncal vagotomy is unavoidable for most types of esophageal resection, it was extrapolated
from the historical experience of treating peptic ulcer disease
that a significant proportion of patients would have delayed gastric emptying. However, debate persists as to whether esophagectomy patients really benefit to the same extent as do peptic ulcer
patients from a pyloric drainage procedure. Proponents of routine
pyloric drainage argue that the two primary reasons to perform
such a procedure is to prevent gastric stasis with its attendant risk
of aspiration and to allow for improved gastrointestinal function
and return to regular eating. Those who omit pyloric drainage
argue that the concerns of performing a vagotomy in this setting
are overestimated (particularly with the construction of a narrow
gastric tube), that the procedure introduces unnecessary perioperative complications, and that it increases bile reflux and dumping. Moreover, delayed gastric emptying in the current era may be
addressed postoperatively through the use of endoscopic balloon
dilatation or botulinum toxin injection of the pylorus.
In the published literature there have been nine randomized
control trials investigating the effect of a pyloric drainage procedure at the time of esophagectomy and reconstruction. These trials
were compiled in a recent meta-analysis by Urschel and colleagues

5/21/2012 8:47:47 PM

Esophageal Tumors

with outcomes combined from 553 patients.21 Over 90% of the


pyloric drainage procedures in the studies were pyloroplasties, but
notably there was a heterogeneous mix of methods of esophagectomy, routes of reconstruction, and technical specifics to the construction of the gastric conduit.21 In particular, it is difficult to
discern the effect of gastric tube width on this issue, since a thin
gastric tube without pyloroplasty may actually function better than
a large remnant stomach with a pyloromyotomy. In these nine studies, half used a whole stomach, whereas the other half used a gastric
tube. In addition, there was a mix of posterior and anterior (substernal) position of the gastric conduit. Given these differences and the
heterogeneous nature of these surgical methodologies even within
the same trial, the data must be interpreted with caution.
Three of the nine trials reported early postoperative outcomes with and without a pyloroplasty.22-24 In the meta-analysis,
the only statistically significant difference was a lower rate of
early gastric outlet obstruction in the postoperative period with
a pyloroplasty compared to those who did not have a pyloroplasty
(3.5% vs. 18.3%, respectively; relative risk 0.18; 95% CI 0.030.97;
P = .046).21 There was no difference in operative mortality, anastomotic leak, or pulmonary morbidity. However, five patients or 3%
who did not have a pyloroplasty suffered a fatal aspiration event
compared to none of the patients who did have a pyloroplasty.
Although this did not reach statistical significance (P = .14), it is
possibly due to the underpowered nature of the study. To place a
3% fatal aspiration rate in perspective, the overall perioperative
mortality of esophagectomy of all causes in high-volume centers
is approximately 3% to 5%. In fact, fatal aspiration events in one
of these trials accounted for all of the postoperative mortalities.22
Complications directly related to the pyloroplasty were uncommon, occurring in 1.7% of the sum of patients from three trials
reporting this data.21 There were no instances of injury to the gastroepiploic pedicle or issues reported with conduit shortening.
With regards to the long-term outcomes of pyloric drainage
beyond the immediate postoperative period, the meta-analysis of
the nine randomized control studies showed that while there was
a trend for shorter gastric emptying time, increased intake of food,
and less obstructive symptoms in patients who underwent pyloric
drainage, this did not reach statistical significance.21 Moreover,
there was no difference in dumping or diarrhea between the two
groups. However, there was also a trend for more bile reflux in
the pyloric drainage patients, but again this did not reach statistical significance. On the basis of the data from these trials, it was
observed that all of these differences between patients with and
without pyloroplasty became less evident as time passed. Thus,
the long-term outcomes between the patients who did and did not
have a pyloric drainage procedure appear to be similar, and the
most beneficial effect of the procedure is protecting against immediate postoperative complications.

DOES CHEMOTHERAPY OR RADIATION


IMPROVE OUTCOME IN SURGICALLY
RESECTABLE PATIENTS?
4. What is the outcome of surgery alone?
Complete surgical resection is the cornerstone of therapy for
locoregional esophageal cancer. However, esophagectomy has
historically been associated with a high perioperative mortality
rate of 10% to 20% compounded by poor survival, especially in

PMPH_CH09.indd 77

77

the era when the majority of patients were high-risk individuals


with squamous cell carcinoma who had significant smoking and
drinking backgrounds, and presented for surgery severely malnourished. As a consequence, chemotherapy and radiation were
widely adopted as adjuvant therapies to improve outcome. When
interpreting the published data on this issue, however, it is useful
to understand what can realistically be accomplished with surgery
alone in the current era, given the changes in patient demographics, refinements in surgical technique, and improvement in the
postoperative care of patients. Such data on the baseline results
of surgery alone can place in context the results from trials evaluating the effect of adding chemotherapy and/or radiation to the
treatment of surgical patients.
There have been several retrospective single institution cohort
series reporting outcomes after esophagectomy. At the authors
own institution, a review of 100 consecutive esophageal adenocarcinoma patients treated with transthoracic en bloc esophagectomy provides unique insights into the effect of radical surgical
resection, since none of these patients received neoadjuvant or
adjuvant therapy.25 This series reviewed the experience from 1982
to 2000, when the perioperative mortality rate was 6% overall,
although the rate dropped to 2.5% in the latter time period of the
study. All patients had an R0 resection and 5-year overall survival
was 52.2%, with 1% developing an anastomotic local recurrence,
9% developing regional recurrence, and 31% developing systemic
disease. Thus, in patients who are well staged and resected with
a transthoracic esophagectomy with a removal of a median of 48
lymph nodes, surgery alone can result in much better outcomes
than what was assumed based on historical experience.
The only prospective data on the outcome of surgery alone
comes from the randomized control trial of Hulscher and colleagues, in which patients with esophageal adenocarcinoma had
either transthoracic or transhiatal esophagectomy between 1994
and 2000 without any chemotherapy or radiation.14,15 In this series
of 220 patients representing stage I-III of disease, in-hospital mortality for the whole group was 3.2%, and esophagectomy alone
resulted in an overall 5-year survival of approximately 35% (transthoracic 36% vs. transhiatal 34%). It is important to recognize
that nearly one third of the patients had residual tumor left behind
with either an R1 or R2 resection in both surgical arms, and this
undoubtedly effected the overall survival in a negative manner.
Nevertheless, this prospective trial demonstrates the natural
history of the disease treated with surgery alone at high-volume
esophageal centers in the contemporary era.
5. What is the outcome of adjuvant therapy?
Adjuvant chemotherapy for esophageal adenocarcinoma patients
who have undergone esophagectomy appears to be associated with
slightly improved survival, but the data is sparse and applicability
to esophageal adenocarcinoma must be extrapolated from more
robust gastric cancer data. There has been only one prospective randomized control trial evaluating the role of adjuvant chemotherapy
in patients with esophageal cancer who have undergone resection
with curative intent, reported by Ando and colleagues for the Japanese Clinical Oncology Group.26 All 242 patients had squamous cell
carcinoma, and were randomized into surgery alone versus surgery
followed by two cycles of cisplatin and fluorouracil. Despite 24% of
patients in the adjuvant arm not completing the intended protocol, there was an improvement in disease-free 5-year survival in the
adjuvant chemotherapy arm, from 45% to 55% (P = .037).

5/21/2012 8:47:47 PM

78

Surgery: Evidence-Based Practice

Regarding patients with adenocarcinoma, the best data on


adjuvant chemotherapy in these patients is from a phase II trial
from the Eastern Cooperative Oncology Group (E8296), reported
by Armanios and colleagues.27 This study comprised 59 patients
with tumors of the distal esophagus (16%), gastroesophageal junction (62%), and gastric cardia (22%). Following surgery with R0
resection at 20 different centers, the protocol consisted of four
cycles of paclitaxel and cisplatin. Eighty-four percent of patients
completed the protocol, and 3-year disease-free survival was 42%.
Since this was not a phase III prospective randomized trial, the
authors used a historical control arm to conclude that this protocol
improved survival compared to surgery alone. This specific historical surgical control arm survival data was extrapolated from
another Intergroup trial28 that had an aberrantly low 2-year survival rate of 38%. These poor surgical outcomes are not representative of what can be accomplished with surgery alone in the current
era with an R0 resection, where 5-year survival can be 50%.25
The addition of radiotherapy in an adjuvant setting for
patients with esophageal adenocarcinoma has not been studied in
a prospective fashion. However, data can be extrapolated from the
Southwest Oncology Group (SWOG-9008) trial reported by Macdonald and colleagues.29 This trial was a prospective, randomized
control study investigating the effect of adjuvant fluorouracil and
leukovorin with 45 Gy of radiation after gastrectomy compared
to surgery alone. Approximately 20% of the patients had adenocarcinoma of the gastric cardia, which is commonly assumed
to be applicable for distal esophageal adenocarcinoma. Notably,
heterogeneity of treatment effect was not performed in this study
to determine if the outcome differed based on tumor location.

There was improved relapse-free 3-year survival in the adjuvant


chemoradiation arm (48% vs. 31%), and the hazard ratio for death
with surgery alone was 1.35 (95% CI 1.091.66; P = 0.05). The
applicability of this trial to distal esophageal adenocarcinoma is
speculative and must be interpreted with caution.
6. What is the outcome of neoadjuvant therapy?
In contrast to adjuvant therapy administered in the postoperative
setting, neoadjuvant or induction therapy has several theoretical advantages. It is assumed to have better tolerability since the
patient does not need to recover from a major operation and subsequently could increase the chances of successful completion of the
intended therapy. Further, it is possible that the ability to obtain an
R0 resection is increased due to shrinkage of the tumor. Finally,
the response of the tumor to the therapy can give a good indicator
of the biology of the disease, and its response to the given therapy
may be informative in future management. Neoadjuvant therapy
with chemotherapy alone or in combination with radiation has
been studied for esophageal adenocarcinoma through several prospective trials, in contrast to the investigation of adjuvant therapy.
Neoadjuvant chemotherapy alone followed by surgical resection for esophageal cancer has been investigated by four phase III
prospective randomized control trials, although one remains in
abstract form (Table 9.2). Three studies concluded that neoadjuvant
chemotherapy improved survival, whereas one study concluded that
there was no difference. The largest published trial is the Medical Research Council (MRC) OEO2 trial, which randomized 802
patients with esophageal and cardia cancer, of whom two thirds had

Table 9.2 Phase III Randomized Trials in Neoadjuvant Chemotherapy for Esophageal Adenocarcinoma
Study

Tumor
Location

MRC OEO2 Esophagus and


GE junction

Intergroup
113

MAGIC

FFCD 9703

Esophagus and
GE junction

Distal
esophagus,
GE junction,
stomach

Distal
esophagus,
GE junction,
stomach

Adenocarcinoma
66%

54%

100%

100%

Protocol

Patients

Curative
Resection

Survival

HR
(95% CI)

Pre-op

Post-Op

802 Total

R0

Overall
5-year

HR death

Cis + 5-FU x2
then surgery

400

60%

23.00%

0.84
(0.72-0.98,
P = 0.03)

Surgery alone

402

54%

17.00%

Pre-op

Post-op

440 Total

R0

Overall
3-year

HR death

Cis + 5-FU x3
then surgery

213

62%

23%

1.07
(0.891.32)

Surgery alone

227

59%

26%

Pre-op

Post-op

503 Total

R0

Overall
5-year

HR death

ECF x3 then
surgery

ECF x3

250

69.30%

36.30%

0.75
(0.600.93,
P = .009)

Surgery alone

253

66.40%

23.00%

Pre-op

Post-op

224 Total

R0

Overall
5-year

HR death

Cis + 5-FU x2 or Cis + 5-FU 113


3 then surgery
in 47.8%

84%

38%

0.69
(0.500.95,
P = .02)

Surgery alone

73%

111

GE junction = gastroesophageal junction; Cis = cisplatin; 5-FU = 5-fluorouracil; ECF = epirubicin, cisplatin, fluorouracil; HR = hazard ratio.

PMPH_CH09.indd 78

5/21/2012 8:47:47 PM

Esophageal Tumors

adenocarcinoma.30 The neoadjuvant arm consisted of two cycles of


cisplatin and fluorouracil, which was completed by 90% of participants. After a median follow-up of 6 years, the neoadjuvant chemotherapy arm had improved survival compared to the surgery alone
arm with a hazard ratio of 0.84 (95% CI 0.720.98; P = .03). Absolute 5-year survival was 23% in the chemotherapy arm compared to
17.1% in the surgery alone arm. However, this trial has been criticized for numerous issues, including a 10% perioperative mortality
rate, a significantly lower complete resection (R0) rate in the surgery
alone arm, and the inclusion of 9% of patients who received concurrent radiation. These issues appear to place the surgical arm at a
disadvantage despite randomization, and could have influenced the
outcome of this trial. In contrast to this study, the Intergroup 8911
trial from the United States, using three cycles of neoadjuvant cisplatin and fluorouracil, did not show any survival advantage compared
to surgery alone in a total study population of 443 patients.28
More recently, a prospective, randomized perioperative chemotherapy trial was reported by Cunningham and colleagues
(the MRC Adjuvant Gastric Infusional Chemotherapy or MAGIC
trial).31 This trial is unique in that it consisted of both a perioperative as well as postoperative chemotherapy regimen, consisting of
surgical resection sandwiched between three cycles of epirubicin,
cisplatin, and fluorouracil (ECF) before and after surgery for a
total of six cycles. This study comprised 503 patients who were

79

randomized to surgery alone or the perioperative chemotherapy


and surgery protocol. All patients had adenocarcinoma, although
the majority had gastric cancer (74%), with the remainder consisting of gastroesophageal and distal esophageal tumors. Whereas
86% of patients assigned to the chemotherapy and surgery arm
completed the preoperative three cycles of ECF, only 41.6% completed all six pre- and postoperative cycles. At 4 years of median
follow-up, the group who had perioperative chemotherapy had
a significantly higher likelihood of disease-free survival with a
hazard ratio of 0.66 (95% CI 0.530.81; P < .001). Both the local
and distant recurrence rates were lower in the perioperative chemotherapy group. Overall 5-year survival was 36.3% with chemotherapy compared to 23.0% with surgery alone. There was no
evidence of heterogeneity of treatment effect when tested according to the site of tumor, indicating reliable applicability to patients
with lower esophageal and gastroesophageal adenocarcinomas.
The addition of radiation therapy to chemotherapy in the
neoadjuvant setting followed by surgical resection has been investigated by five phase III prospective randomized control trials in
esophageal adenocarcinoma (Table 9.3).32-36 The theoretical advantages of adding radiation are to induce greater complete response
rates, improve complete resection rates, and to maintain local
control during systemic treatment with chemotherapy while awaiting surgery. Of the published trials, only the Walsh trial was limited

Table 9.3 Phase III Randomized Trials in Neoadjuvant Chemoradiation for Esophageal Adenocarcinoma
Study

Tumor
Location

Adenocarcinoma

Australia

Esophagus

63%

Michigan

Esophagus

76%

Pre-op
Cis + 5-FU x1 + 35
Gy radiation then
surgery
Surgery alone
Pre-op

100%

Cis + 5-FU +
vinblastine + 45
Gy radiation then
surgery
Surgery alone
Pre-op

75%

Cis + 5-FU x2 + 40
Gy radiation then
surgery
Surgery alone
Pre-op

75.20%

Cis + 5-FU x2 +
50.4 Gy radiation
then surgery
Surgery alone
Pre-Op

Ireland

CALGB
9781

CROSS

Esophagus,
cardia

Esophagus,
cardia

Esophagus, GE
junction

Protocol

Paclitaxel +
carboplatin x5 +
41.4 Gy radiation
then surgery
Surgery alone

PMPH_CH09.indd 79

Patients

Curative
Resection

Survival

HR
(95% CI)

Post-Op
-

256 Total
128

R0
80%

Overall
NA

HR Death
0.89
(0.67-1.19)

Post-Op

128
100 Total

59%
R0

HR Death

50

45%

NA
Overall
3-year
30%

Post-Op

50
113 Total

45%
R0

58

NA

Post-Op

55
56 Total

NA
R0

30

NA

Post-Op

26
363 Total

NA
R0

175

188

0.73
(0.48-1.12)

16%
Overall
3-year
32%
(P = 0.01)

HR Death

6%
Overall
5-year
39%

HR Death

HR Death

92.30%

16%
Overall
3-Year
59%

64.90%

48%

NA

NA
(1.46-5.69)

0.67
(0.50-0.92)

5/21/2012 8:47:47 PM

80

Surgery: Evidence-Based Practice

to adenocarcinoma, with the remaining studies having a mixture


of squamous cell carcinoma and adenocarcinoma. It is apparent
that these trials are significantly smaller than trials examining the
impact of induction chemotherapy alone. Two of these published
trials have shown improved survival with induction chemoradiation: the Irish trial reported by Walsh et al. and the CALGB 9781
trial reported by Tepper et al.32,37 Both of these studies have been
severely criticized and require cautious analysis of the data.
The Irish trial comprised 113 patients with esophageal adenocarcinoma who were randomized to either surgery alone or two cycles
of fluorouracil and cisplatin with 40 Gy of radiation followed by surgical resection.32 Complete pathologic response occurred in 25% of
the chemoradiation arm. Median follow-up was only 10 months, and
survival was significantly improved with induction chemoradiation.
The 3-year survival rate was 32% in those who received chemoradiation compared to 6% in those had surgery alone (P = .01). Criticisms
of this study include an unacceptable survival rate for the surgical
arm in the context of what is accomplished with surgery alone in the
current era; the suboptimal preoperative staging that is performed
resulting in incomplete resections; and the short follow-up.
The CALGB 9781 trial also showed a significant improvement
in survival with induction chemoradiation that comprised two
cycles of cisplatin and fluorouracil with 50.4 Gy of radiation.37 This
trial was closed prematurely after 3 years due to poor accrual, with
only 56 total patients enrolled from 18 centers. The majority (75%)
of these patients had adenocarcinoma, and after a median follow-up
of 6 years, improved survival was noted in the neoadjuvant therapy
arm compared to the surgery alone arm. Five-year overall survival was 39% for 30 patients in the neoadjuvant arm and 16% for
26 patients in the surgery arm. The most significant criticism of
this trial is its lack of power, as it was originally designed to include
475 patients. Such a trial is subject to publication bias given its positive result and must be interpreted with caution.
The largest neoadjuvant chemoradiation trial to date for
esophageal cancer has been completed in the Netherlands and has

been reported in abstract form.36 This study included 363 patients


who were randomized to five cycles of preoperative paclitaxel and
carboplatin with 41.4 Gy of radiation followed by surgery or surgery alone. Seventy-five percent of patients had adenocarcinoma
and the complete pathologic response was 32.6%. There were significantly more R0 resections with neoadjuvant therapy than in
the surgery alone group. Median follow-up was 32 months and
there was improved survival with neoadjuvant therapy compared
to surgery alone, with a hazard ratio of 0.67 (95% CI 0.500.92;
P = .011). Three-year survival was 59% in the neoadjuvant arm
compared to 48% in the surgery only arm.

SUMMARY
The surgical management of esophageal cancer is complex, but
there is Level 1 evidence to guide the decision-making process
in the care of patients with this disease (see the following table).
Initial clinical staging of esophageal cancer should be performed
with EUS to determine the T and N stage, and integrated PET/
CT for the M stage (Level 1a). If locoregional disease is present,
there appears to be improved survival with neoadjuvant therapy
followed by surgical resection in many patients (Level 1b). There is
significant variability in the precise protocol of chemotherapy and
radiation used in published trials, and the selection of patients for
neoadjuvant therapy as well as the specific protocol can only be
guided by individual clinician and institutional preference at this
time. If surgical resection is performed, a transthoracic esophagectomy with radical lymphadenectomy results in superior survival
(Level 1b), and a minimum of 23 to 30 lymph nodes should be
removed for both accurate staging and maximal curative effect
(Level 1b). A pyloric drainage procedure is recommended to prevent gastric outlet obstruction that may lead to fatal aspiration in
the early postoperative period, but long-term functional outcomes
do not appear to be different (Level 1a).

Clinical Question Summary


Question

Answer

Level of Evidence

Grade of
Recommendation

1 How should esophageal cancer be


clinically staged?

EUS is used for T and N staging, and PET/CT


is used for M staging.

Level 1a

2 Is transthoracic esophagectomy
superior to transhiatal
esophagectomy?

Transthoracic esophagectomy results in


improved survival compared to transhiatal
esophagectomy when locoregional disease
(1 to 8 involved nodes) is present.

Level 1b

3 Does extended lymphadenectomy


improve survival?

Extended lymphadenectomy improves


survival when a minimum of 23 to 30
nodes is removed.

Level 1b

4 Does a pyloric drainage


procedure improve outcomes of
esophagectomy?

Routine pyloric drainage procedure reduces


aspiration events in the postoperative
setting.

Level 1a

5 Does chemotherapy or radiation


improve outcome in surgically
resectable esophageal cancer?

Neoadjuvant therapy improves survival


in selected patients. Patient selection,
specific chemotherapy agents, and the
addition of radiation have not been
standardized.

Level 1b

PMPH_CH09.indd 80

5/21/2012 8:47:47 PM

Esophageal Tumors

REFERENCES
1. Jemal A, Siegel R, Xu J, Ward E. Cancer Statistics, 2010. CA Cancer J Clin. 2010;60:277-300.
2. Pohl H, Welch HG. The role of overdiagnosis and reclassification
in the marked increase of esophageal adenocarcinoma incidence.
J Natl Cancer Inst. 2005;97:142-146.
3. Devesa SS, Blot WJ, Fraumeni JF, Jr. Changing patterns in the
incidence of esophageal and gastric carcinoma in the United
States. Cancer. 1998;83:2049-2053.
4. Rice TW, Blackstone EH, Rusch VW. 7th Edition of the AJCC
Cancer Staging Manual: esophagus and esophagogastric junction. Ann Surg Oncol. 2010;17:1721-1724.
5. Puli S-R, Reddy J-B, Bechtold M-L, Antillon D, Ibdah J-A, Antillon M-R. Staging accuracy of esophageal cancer by endoscopic
ultrasound: a meta-analysis and systematic review. World J Gastroenterol. 2008;14:1479-1490.
6. Thomas T, Gilbert D, Kaye PV, Penman I, Aithal GP, Ragunath
K. High-resolution endoscopy and endoscopic ultrasound for
evaluation of early neoplasia in Barretts esophagus. Surg. Endoscopy. 2010;24:1110-1116.
7. Maish MS, DeMeester SR. Endoscopic mucosal resection as a
staging technique to determine the depth of invasion of esophageal adenocarcinoma. Ann Thoracic Surg. 2004;78:1777-1782.
8. van Vliet EPM, Heijenbrok-Kal MH, Hunink MGM, Kuipers EJ,
Siersema PD. Staging investigations for oesophageal cancer: a
meta-analysis. Br J Cancer. 2008;98:547-557.
9. Bar-Shalom R, Guralnik L, Tsalic M, et al. The additional value
of PET/CT over PET in FDG imaging of oesophageal cancer. Eur
J Nucl Med Mol. Imaging 2005;32:918-924.
10. Kato H, Kimura H, Nakajima M, et al. The additional value of
integrated PET/CT over PET in initial lymph node staging of
esophageal cancer. Oncol Rep. 2008;20:857-862.
11. Goldminc M, Maddern G, Le Prise E, Meunier B, Campion JP,
Launois B. Oesophagectomy by a transhiatal approach or thoracotomy: a prospective randomized trial. Br J Surg. 1993;80:367-370.
12. Chu KM, Law SY, Fok M, Wong J. A prospective randomized
comparison of transhiatal and transthoracic resection for lowerthird esophageal carcinoma. Am J Surg. 1997;174:320-324.
13. Jacobi CA, Zieren HU, Muller JM, Pichlmaier H. Surgical therapy of esophageal carcinoma: the influence of surgical approach
and esophageal resection on cardiopulmonary function. Eur J
Cardiothorac Surg. 1997;11:32-37.
14. Hulscher JBF, van Sandick JW, de Boer AGEM, et al. Extended
transthoracic resection compared with limited transhiatal resection for adenocarcinoma of the esophagus [see comment]. N Engl
J Med. 2002;347:1662-1669.
15. Omloo JMTMD, Lagarde SMMD, Hulscher JBFMD, et al.
Extended transthoracic resection compared with limited transhiatal resection for adenocarcinoma of the mid/distal esophagus: five-year survival of a randomized clinical trial [article].
Ann Surg. 2007;246:992-1001.
16. Oh DS, Hagen JA, Chandrasoma PT, et al. Clinical biology and
surgical therapy of intramucosal adenocarcinoma of the esophagus. J Am Coll Surg. 2006;203:152-161.
17. Peyre CG, DeMeester SR, Rizzetto C, et al. Vagal-sparing
esophagectomy: the ideal operation for intramucosal adenocarcinoma and barrett with high-grade dysplasia. Ann Surg.
2007;246:665-671; discussion 71-74.
18. Schwarz RE, Smith DD. Clinical impact of lymphadenectomy
extent in resectable esophageal cancer. J Gastrointestin Surg.
2007;11:1384-1393; discussion 93-94.

PMPH_CH09.indd 81

81

19. Peyre CG, Hagen JA, DeMeester SR, et al. The number of lymph
nodes removed predicts survival in esophageal cancer: an international study on the impact of extent of surgical resection.
Trans Meet Am Surg Assoc. 2008;126:190-197.
20. Rizk NP, Ishwaran H, Rice TW, et al. Optimum lymphadenectomy for esophageal cancer. Ann Surg. 2010;251:46-50.
21. Urschel JD, Blewett CJ, Young JEM, Miller JD, Bennett WF.
Pyloric drainage (pyloroplasty) or no drainage in gastric reconstruction after esophagectomy: a meta-analysis of randomized
controlled trials. Dig Surg. 2002;19:160-164.
22. Mannell A, McKnight A, Esser JD. Role of pyloroplasty in the
retrosternal stomach: results of a prospective, randomized, controlled trial. Br J Surg 1990;77:57-59.
23. Fok M, Cheng SWK, Wong J. Pyloroplasty versus no drainage in
gastric replacement of the esophagus. Am J Surg. 1991;162:447-452.
24. Zieren HU, Muller JM, Jacobi CA, Pichlmaier H. Should a
pyloroplasty be carried out in stomach transposition after subtotal esophagectomy with esophago-gastric anastomosis at the
neck? A prospective randomized study [in German]. Chirug.
1995;66:319-325.
25. Hagen JA, DeMeester SR, Peters JH, Chandrasoma P, DeMeester
TR. Curative resection for esophageal adenocarcinoma: analysis of 100 en bloc esophagectomies. Ann Surg. 2001;234:520-530;
discussion 30-31.
26. Ando N, Iizuka T, Ide H, et al. Surgery plus chemotherapy compared with surgery alone for localized squamous cell carcinoma
of the thoracic esophagus: a Japan Clinical Oncology Group
StudyJCOG9204. J Clin Oncol. 2003;21:4592-4596.
27. Armanios M, Xu R, Forastiere AA, et al. Adjuvant chemotherapy
for resected adenocarcinoma of the esophagus, gastro-esophageal
junction, and cardia: phase II trial (E8296) of the Eastern Cooperative Oncology Group [erratum appears in J Clin Oncol.
2008;26(22):3819]. J Clin Oncol. 2004;22:4495-4499.
28. Kelsen DP, Winter KA, Gunderson LL, et al. Long-term results
of RTOG trial 8911 (USA Intergroup 113): a random assignment
trial comparison of chemotherapy followed by surgery compared with surgery alone for esophageal cancer. J Clin Oncol.
2007;25:3719-3725.
29. Macdonald JS, Smalley SR, Benedetti J, et al. Chemoradiotherapy
after surgery compared with surgery alone for adenocarcinoma
of the stomach or gastroesophageal junction. N Engl J Med.
2001;345:725-730.
30. Allum WH, Stenning SP, Bancewicz J, Clark PI, Langley RE.
Long-term results of a randomized trial of surgery with or without preoperative chemotherapy in esophageal cancer. J Clin
Oncol. 2009;27:5062-5067.
31. Cunningham D, Allum WH, Stenning SP, et al. Perioperative
chemotherapy versus surgery alone for resectable gastroesophageal cancer. N Engl J Med. 2006;355:11-20.
32. Walsh TN, Noonan N, Hollywood D, Kelly A, Keeling N,
Hennessy TP. A comparison of multimodal therapy and surgery for esophageal adenocarcinoma [see comment][erratum appears in N Engl J Med 1999;341(5):384]. N Engl J Med.
1996;335:462-467.
33. Urba SG, Orringer MB, Turrisi A, Iannettoni M, Forastiere A,
Strawderman M. Randomized trial of preoperative chemoradiation versus surgery alone in patients with locoregional esophageal carcinoma [see comment]. J Clin Oncol. 2001;19:305-313.
34. Burmeister BH, Smithers BM, Gebski V, et al. Surgery alone versus chemoradiotherapy followed by surgery for resectable cancer
of the oesophagus: a randomised controlled phase III trial. Lancet Oncol. 2005;6:659-668.

5/21/2012 8:47:47 PM

82

Surgery: Evidence-Based Practice

35. Boige V, Pignon J, Saint-Aubert B, et al. Final results of a randomized trial comparing preoperative 5-fluorouracil/cisplatin to surgery alone in adenocarcinoma of stomach and lower
esophagus: FNLCC ACCORD07-FFCD 9703 trial. J Clin Oncol.
2007;25:Abstr 4510.
36. Gaast AV, Hagen Pv, Hulshof M, et al. Effect of preoperative
concurrent chemoradiotherapy on survival of patients with

PMPH_CH09.indd 82

resectable esophageal or esophagogastric junction cancer: results


from a multicenter randomized phase III study. J Clin Oncol.
2010;28:Abstr 4004.
37. Tepper J, Krasna MJ, Niedzwiecki D, et al. Phase III trial of trimodality therapy with cisplatin, fluorouracil, radiotherapy, and
surgery compared with surgery alone for esophageal cancer:
CALGB 9781. J Clin Oncol. 2008;26:1086-1092.

5/21/2012 8:47:47 PM

CHAPTER 10

The Use of Esophageal Stents


Yaron Perry and Robert Jones

Esophageal cancer is the eighth most common cancer and is the


sixth leading cause of cancer-related death worldwide.1 In the
USA, in 2010, esophageal cancer was diagnosed in 16,640 new
patients and 14,500 esophageal cancer death were recorded2 with
an overall annual incidence of approximately 5.4 cases per 100,000
population.3 The advancement in diagnostic technology, surgical
equipment, and intensive care did not change significantly the
grim overall prognosis.
Most patients (>60%) with esophageal cancer are presenting
with unresectable or inoperable esophageal cancer due to local
invasion, distant metastases, or medical comorbidities. Qua lity of
life takes precedence over long-term prognosis. Several options
exist for esophageal cancer palliation, including best supportive
care, chemotherapy and/or radiation therapy, esophagectomy,
retrosternal bypass, as well as endoluminal therapy, neodymium:
yttrium-aluminum garnet (Nd:YAG) laser, photodynamic therapy, brachytherapy, and stents.4
According to the National Comprehensive Cancer Network
(NCCN) Guidelines for esophageal cancer, the primary objectives
for best supportive care are centered on the restoration and maintenance of the ability to swallow, as well as control of pain and bleeding. The patients nutritional status is optimized, and the patients
sense of well-being and quality of life are preserved. Endoscopic
stent placement is a well-accepted and effective treatment option
for dysphagia, but it is associated with recurrent dysphagia due
to tumor and tissue in-growth, stent migration, food impaction,
bleeding, perforation, and aspiration pneumonia.
The first successful insertion of an esophageal tube was
reported by Sir Charles Symonds in 1887 and was followed by the
development of plastic esophageal tubes. In 1983, Frimbbeger initially used expanding spiral metallic stents for the treatment of
malignant esophageal stricture and dysphagia, and their use has
now been well established for palliation of malignant dysphagia.5

studies. In 1993, Knyrim et al. published a controlled prospective randomized study in the New England Journal of Medicine
comparing metal with plastic stents, and observed 95% to 100%
technical success in using metal stents. In the metal stents group,
they had not reported any complications such as perforation, aspiration or migration, and stent mortality.6
The advancement in the stents design and delivery systems
led to their use for other indications in addition to intra-luminal
esophageal cancer and dysphagia. Oral and enteric nutrition are
key to the support of esophageal cancer patients. Stenting the
esophagus early in the stage of concentric narrowing will lead
to better nutrition options such as oral intake of semisolids and
puree diet, and increase the calorie support and quality of life.7
Palliative treatment of malignant esophagopulmonary fistula
has been explored in two major studies discussed below.
Kim et al. reported a prospective analysis of 14 patients with
esophagopulmonary fistula caused by esophageal and bronchogenic carcinoma. Palliative treatment was done successfully in 12
patients who had complete sealing of the fistula resulting in resolution of aspiration symptoms. A mean survival of 100.9 days was
reported.8
Shin et al. reported the long-term outcomes of 61 patients
with esophagorespiratory fistula after palliative treatment with
covered expandable metallic stents. The clinical success was
80% with mean survival of 93.8 days. In patients with malignant
esophagorespiratory fistula, the option of covered stents prevents
ongoing aspiration and it is used in patients with dismal prognosis with an average of 90 to 100 days from stent placement. The
surgical alternative for palliative resection and repair of the fistula
has perioperative mortality of 29% to 47%.9
The initial concern about migration and the older generation design made palliative treatment underutilized for the relief
of malignant dysphagia due to extrinsic compression. Recently,
Van Heel et al. conducted a prospective single-center study of
50 consecutive patients with extrinsic compression, mostly by
obstructive lung cancer, mediastinal metastases, or extrinsic
local recurrence after esophagectomy. They demonstrated technical insertion success in all patients. They used self-expandable

1. What are the indications for esophageal stents: Is it only for


palliation?
The safety and efficacy of the self-expending metal stents for the
palliation of malignant dysphagia have been shown in multiple
83

PMPH_CH10.indd 83

5/21/2012 8:46:59 PM

84

Surgery: Evidence-Based Practice

metallic stents with 10% complication rate, which included hemorrhage (6%) and perforation due to prestent dilation (4%). The
median survival was 44 days with median patency of 46 days.10
Until the past few years, it was generally believed that dysphagia
due to extrinsic compression should be treated with an uncovered stent to prevent stent migration.11-13 This will obviate the side
effects of the permanent uncovered stent such as tissue in-growth,
reflux, and risk of aspiration. In the study by Peter D. Siersema on
the use of covered stents, dysphagia improvement and the occurrence of complications and recurrent dysphagia were no different in patients with extrinsic compression of the esophagus and
in patients with primary esophageal cancer. Only one (3%) stent
migrated to the stomach. These positive results need to be balanced
with the poor overall prognosis of this patient group, with onethird dying from progressive disease within 30 days after treatment.14 The associated complications that were described with the
early generations of esophageal stents, such as tumor and
tissue in-growth, migration, bleeding, perforation, aspiration and
gastroesophageal reflux, prevented their use in benign esophageal
stricture. The new-generation covered stents and the option of
simple retrieval led to a new interest of treating benign strictures
with esophageal stents (Figure 10.1).
Treatment of achalasia with self-expending metal stents
was fi rst described by De Palma in 1998 for patients who failed
medical therapy or pneumatic dilation, or who were poor surgical candidates.15 Only a few reports followed this study.16-19
A recent prospective study by Cheng et al. was published in
World Journal of Gastroenterology, which includes 90 achalasia patients who were treated with a temporary self-expanding
metallic stents. Stent retrieval was performed by gastroscopy
4 to 5 days after placement. Treatment success was achieved in
all patients with a patency of the lower esophageal sphincter at
1 month after stent removal, and the dysphagia scores significantly improved for all patients. Clinical remission rate in the
patient who received 3-cm diameter stent declined slowly from
100% at 6 months to 83.3% at the 10 years assessment. 20 The
results are encouraging, but still not comparable with the current long-term success rates of laparoscopic Heller myotomy,
which is a valid option for inoperable patients or patient with
multiple comorbidities.
Apart from palliation of malignant dysphagia, stents are
proven to benefit benign stricture and obstruction in the settings
of achalasia, anastomotic stricture, and esophagopulmonary fistula (Level 2b evidence; Grade C recommendation).
2. What are the stents options: Metal versus plastic? Covered or
uncovered? Removable or Permanent?
More than 130 patient series have reported on stent placement for
the palliation of malignant dysphagia caused by mid- or distalesophageal cancer. These studies have mainly dealt with one
stent type only and cannot be used to compare the effectiveness
of different stent types. A few retrospective studies have compared different stent types.21-23 In these studies uncovered stents
were used, and it has been convincingly shown that partially or
fully covered stents give better long-term palliation of malignant
dysphagia than uncovered stents.24 There have been few prospective studies that have directly compared stent types for the
palliation of malignant esophageal strictures. In one study 100
patients with inoperable mid- and distal-esophageal or gastric

PMPH_CH10.indd 84

Figure 10.1 The variety of stents described and stent positioning. ANitinol fully covered stent.** BNitinol covered stent
with traction loop.*** CPlastic covered stents.** DThe Ella
stent is composed of the biodegradable polymer polydioxanone.*
EIntroduction systems for the self-expandable stents.**/***
FFluoroscopic Guidance insertion of esophageal stents; Barium swallow delineate the stricture. GFluoroscopic positioning and patency proven with oral contrast. HFinal position.
*doi:10.1016/j.gie.2010.07.031. How to Cite or Link Using DOI
Copyright 2010 American Society for Gastrointestinal Endoscopy Published by Mosby, Inc. **With permission from Boston
Scientific Corporation. ***With permission from Merit EndoTek,
Merit Medical Systems.
cardia cancer were randomly allocated to have one of three commonly used stent types placed (see Figure 10.1): the UltraflexTM
stent (UltraflexTM Esophageal NG Stent System, Boston Scientific,
Natick, MA), the Flamingo Wallstent (Boston Scientific, MA),
or the Z-stent (Wilson-Cook Medical, Winston-Salem, NC). 25
Both the UltraflexTM stent and Flamingo Wallstent are made of
nitinol and covered at their midsections with a polyester cover.
The Z-stent is made of stainless steel and covered with polyethylene over its entire length. No statistically significant differences
were found between the stents for dysphagia improvement,
the occurrence of complications, such as perforation or hemorrhage, and the occurrence of recurrent dysphagia, as determined by stent migration or tissue overgrowth and in-growth.

5/21/2012 8:47:00 PM

The Use of Esophageal Stents

A trend toward more complications with Z-stents was, however,


observed; this difference would have been statistically significant if 150 patients instead of 100 patients had been randomized.
Similar fi ndings with regard to dysphagia improvement and
complications were observed in another study, which was a randomized trial performed in the UK, in which UltraflexTM stents
(n = 31) and Flamingo Wallstents (n = 22) were compared in
patients with malignant dysphagia caused by distal-esophageal
cancer.26 An ongoing issue with stents is the occurrence of recurrent dysphagia because of stent migration, tumoral or nontumoral tissue growth and food obstruction. Recurrent dysphagia
occurs in 30% to 40% of patients after a mean follow-up of
2 to 3 months.27 Reintervention for stent-related recurrent dysphagia is effective in more than 90% of patients.28 According to
Homs group experience, 28 the most effective treatment strategy
for tissue overgrowth or in-growth and stent migration is the
placement of a second stent, or, in some cases of migration, stent
repositioning. For cases of food obstruction, endoscopic stent
clearance is an easy and effective strategy to employ. After treatment for recurrent dysphagia, the median survival of patients
has been shown to be longer than 2 months.28 Reintervention is
expensive because in many cases endoscopy, a new stent, or even
admissions to the hospital is required. To reduce the need for
reinterventions, two new stent designs, the Polyflex stent (Boston Scientific) and the Niti-S double stent (Taewoong Medical,
Seoul, Korea), have been developed. Polyflex stents are made
of silicone and polyester, fully covered, and were designed to
reduce nontumoral tissue overgrowth and in-growth, which is
mostly seen after the placement of partially covered stents made
of nitinol because of hyperplastic tissue growth. By contrast, the
Niti-S double stent was developed to reduce stent migration. It
combines a flare at both ends and a double layer configuration,
with an inner cover and an outer uncovered nitinol wire tube
to allow the mesh of the stent to embed itself in the esophageal
wall. The Polyflex stent, the Niti-S double stent and the UltraflexTM stent (the most commonly used stent type worldwide) were
recently compared in a randomized trial including 125 patients
with esophageal or gastric cardia cancer.29 The improvement of
dysphagia and complications was similar for the three stents,
but, overall, it was found that recurrent dysphagia occurred more
frequently with UltraflexTM stents (n = 22 [52%]) than with Polyflex stents (n = 15 [37%]) or Niti-S stents (n = 13 [31%]; P = 0.03).
Of the main causes of recurrent dysphagia, stent migration was
most commonly seen with Polyflex stents (n = 12 [29%] vs.
UltraflexTM stents n = 7 [17%] and Niti-S stents n = 5 [12%]). Tissue in-growth and overgrowth were more frequent with UltraflexTM stents (n = 13 [31%] and, to a lesser degree, Niti-S stents
n = 10 [24%] vs. Polyflex stents n = 4 [10%]). Food obstruction occurred frequently with UltraflexTM stents (n = 10 [24%])
vs. Polyflex stents n = 2 [5%] and Niti-S stents n = 1 [2%]). In
conclusion, Flamingo Wallstents and Niti-S stents are both
appropriate for the palliation of dysphagia from esophageal cancer. The same is probably true for UltraflexTM stents, although
recurrent dysphagia caused by tissue in-growth and overgrowth
or food obstruction occurs more frequently with this stent
design. By contrast, Z-stents and Polyflex stents seem less preferable in these patients, because, compared with the other stents,
Z-stents are likely to be associated with a higher risk of complications, and Polyflex stents are more prone to migrate. Also
placement of Polyflex is technically demanding.

PMPH_CH10.indd 85

85

Randomized controlled studies have proven the advantage of


metal expandable covered stents from patency rate, migration, and
reintervention rate (Level 2b evidence; Grade B recommendation).
3. What is the optimal insertion technique?
The surgeon must become well acquainted with the stents characteristics, as these devices should not only be able to palliate dysphagia from primary esophageal cancer, but also from extrinsic
malignant compression. In this regard, it is important to realize
that various stents differ in characteristics such as radial force,
buckling characteristics, shortening or no shortening on deployment, and releasing proximally or distally. The selection of stent
type is also influenced by stricture characteristics, such as being
straight or angulated, fully or semi-circumferential, intrinsic or
extrinsic, and location in the esophagus. To predict which types
of strictures are most likely to recur, it is important to differentiate
between esophageal strictures that are simple (i.e., focal, straight
strictures with a diameter that allows endoscope passage) and
those that are more complex (i.e., long [>2 cm], tortuous strictures with a narrow diameter). Complex strictures are considered
refractory when they cannot be dilated to an adequate diameter.
Upper endoscopy is the diagnostic procedure of choice for the
detection of an esophageal stricture and its underlying cause. It
is mandatory that biopsy samples are taken to confi rm whether
the stricture is benign or malignant in nature, particularly if the
suspicion of malignancy is high. We perform all our stent placement procedures in the operating room or in the endoscopy suite
with anesthesia support. The procedures are performed with the
patients under sedation with midazolam, propofol, or general
anesthesia in case we expect prolonged procedure, technical difficulty and a need for intubation for airway protection. Flouroscopy is advisable because most of our stent placements are done
under fluoroscopic guidance, before insertion, the proximal
margin of the stricture is marked by radiopaque markers (paper
clip attached to adhesive band can be used as well). Savary dilation over a guide-wire under fluoroscopic guidance is performed
to facilitate the introduction of the stent applicator, in cases of
smaller lumen than the pediatric scope diameter (18F). Subsequently the applicator with the uploaded stent was advanced over
the guide-wire and positioned with the proximal radiopaque
marker approximately 2 cm proximal to the marker site. One has
to measure the total length of the stricture and has to select the
correct stent length accordingly. The stent is released under fluoroscopic control by withdrawing the transparent external sleeve
while holding the internal pusher stationary. The introducing
systems are unique to each stents brand, although these principals are universal. Endoscopy is performed immediately after
stent release to confirm the correct positioning and stent expansion across the stricture.
In case of malpositioning or a need to readjust the position of
the stent, most of the current commercial expanded covered stents
have a mechanism. Loop, starts, or side wires facilitate grasping
and pull/push reposition under direct endoscopic vision. We use
the side channel of the adult endoscope to insert biopsy forceps or
endograsper to hold on the stent and reposition it.
Most of the current commercial stents can be repositioned
especially after the initial deployment. Coming back a couple of
days later may result in more technical difficulties, due to full
expansion of the stent, scarring, tissue in-growth, and edema.

5/21/2012 8:47:01 PM

86

Surgery: Evidence-Based Practice

Occasionally, removal of the stent or repositioning of it in a significant distance from its original faulty position has to be done
using an endo-loop or a snare, which grasp the stent circumferentially and can facilitate a distal migration issues. Ingestion
of liquids is usually allowed immediately after stent insertion.
Patients are instructed to progressively resume a puree diet
without any specific restriction. Routine follow-up is 1, 2, 3,
and 6 months. Patients are questioned about relief of dysphagia,
pain, heartburn, or any other symptoms potentially related to
the stent. In any case of dysphagia or significant symptoms, contrast radiography is performed, occasionally followed by upper
endoscopy according to presenting symptoms and radiographic
fi ndings.

Proximal Esophageal Cancer


Esophageal cancer that is located close to the upper esophageal
sphincter (i.e., 7% to 10% of all esophageal cancers) has traditionally been regarded as too difficult to manage with stents. The
reason for this belief is that placing stents at this location was
thought to be associated with a high risk of complications, such
as perforation, aspiration pneumonia, proximal migration, and
patient intolerance caused by pain and foreign body sensation.
Over the last few years, this view has begun to change. Two studies that included 22 and 104 patients, respectively, reported the
results of stent placement in the proximal esophagus.30,31 In the
latter study, 44 patients had a malignant stricture within 4 cm of
the upper esophageal sphincter. 30 Endoscopic visualization and/
or fluoroscopic monitoring were used to control for precise positioning of the proximal stent end just below the upper esophageal sphincter. Dysphagia improved in most patients, and the
occurrence of complications and recurrent dysphagia was comparable with that in patients who underwent stent placement in
the mid and distal esophagus. In total, 5% to 15% of patients
had foreign body sensation; however, in none of the patients
was stent removal indicated. A recommended technique of stent
removal is to grasp the nylon loop that is attached to the proximal end of most stents. Th is decreases the stent diameter and
facilitates pulling the stent out. Yoon et al. 31 designed a special
hook to grasp this loop and to remove the stent under fluoroscopic control. Although they reported that this technique was
successful in the removal of 127 of 130 stents (98%), the caveat of
this technique is that these loops quite often break during stent
removal. Grasping the proximal end of the stent with a snare or
a rat-toothed forceps can be easily done in the same technique
of repositioning.
Fluoroscopic guidance and endoscopic grasping mechanism (loop, string, etc.) have been proven to be safe and reproducible way to position the esophageal stent (Level 3a evidence;
Grade C recommendation).
4. What is the role of stents in benign esophageal stricture and
anastomotic leak?
The most common causes of benign esophageal strictures include
peptic injury, Schatzkis ring, esophageal web, radiation injury,
caustic injury and anastomotic strictures. Strictures caused by
radiation or caustic injury and anastomotic strictures are the
most resistant to endoscopic dilation, which is the customary treatment modality. 32 Although dilation usually results in

PMPH_CH10.indd 86

symptomatic relief, recurrent strictures do occur. To predict


which types of strictures are most likely to recur, it is important to differentiate between esophageal strictures that are simple
and those that are more complex.32 In most patients with simple esophageal strictures, 1 to 3 dilations are required to relieve
symptoms, with an additional 25% to 35% of patients requiring
repeat dilations.33
Strictures that are long (>2 cm), tortuous, or associated with
a diameter that precludes passage of a normal diameter endoscope are defined as complex esophageal strictures.31 The most
common causes include radiation injury, anastomotic stricture, 34
caustic ingestion, 35 photodynamic therapy-related stricture, and
severe peptic injury. Severe peptic injury can occur in patients
in the intensive care unit or in mentally disabled people. Complex esophageal strictures are more difficult to treat than simple
esophageal strictures, require at least three dilation sessions to
relieve symptoms, and are associated with high recurrence rates.
If complex strictures cannot be dilated to an adequate diameter
allowing passage of solid food, it will recur within a time interval
of 2 to 4 weeks or require ongoing (more than 7 to 10) dilation
sessions.31 Novel treatment modalities for refractory strictures
include temporary stent placement and incisional therapy. To
prevent recurrence due to in-growth of granulation tissue, the
completely covered Polyflex stent made of silicone and polyester was evaluated in three retrospective series.36-37 In the first
two series, 36,37 the experience was favorable. In one series, relief
of dysphagia occurred in 17 of 21 patients (81%) after a median
follow-up of 21 months, especially in those with caustic and
hyperplastic (due to partially covered stent placement) strictures.36 In the other series, relief occurred in 12 of 15 patients
(80%) with caustic, post-radiation, anastomotic, or peptic strictures after a median follow-up of 22.7 months.37 No complications were mentioned in the fi rst series, 36 but in the other study
recurrent dysphagia was seen in 33% of patients, which was
due to mucosal hyperproliferation in four patients and stent
migration in one patient.37 Less optimistic results on Polyflex
stents have been reported since. Holm et al.38 placed 84 Polyflex
stents in 20 patients, most with benign or anastomotic strictures.
Migration was the most frequent complication, noted in 18 of 29
patients (62%) and for 53 of 83 stent placements (64%). Hyperplastic tissue growth and stricture formation around the stent
were seen in 5 patients (17%) after 15 procedures (18%). Remarkably, only 5 of 83 procedures (6%) resulted in long-term symptom
relief after stent removal, which is in line with the experience at
my institution. The management of patients who have refractory
hypopharyngeal strictures after chemoradiation and/or surgery
can be unsatisfactory as normal diameter stents placed in this
location can cause a foreign body sensation, severe pain, fistula
formation, or perforation. Patients with refractory hypopharyngeal strictures who have undergone stent placement sometimes
need a feeding tube, but experience difficulties in saliva control.
To prevent the need for a feeding tube and to allow patients to
eat, a cervical Niti-S stent was developed that has a body diameter of 10, 12, or 14 mm. Th is stent is available with or without a
flare that is 2 mm wider than the body diameter and is covered or
uncovered. Th is stent effectively improved dysphagia in a small
series of seven patients.39 As six of the seven patients developed
stent migration and/or granulation tissue in-growth or overgrowth, additional new stents were placed a median of 3 months
after the previous stent placement. Hypopharyngeal strictures

5/21/2012 8:47:01 PM

The Use of Esophageal Stents

have a high recurrence rate, and prolonged stent placement with


periodic stent exchanges at intervals of 6 weeks to 3 months is,
therefore, indicated.
In conclusion, Polyflex stents (plastic stents) have originally
been advocated to be a promising stent type for the treatment of
complex benign strictures. However, recent experience has questioned this optimistic view and further studies are required to
select the most optimal patient population. In addition to stent
migration, another disadvantage of using Polyflex stents is that
the stent applicator is large and stiff compared with the applicators used for metallic stents. Dilation before stent placement is
often required. For these reasons, metal stents, particularly partially covered UltraflexTM stents, are recommended to treat benign
esophageal strictures and it is advisable to retrieve the stents 4 to 6
weeks after placement. UltraflexTM stents have the advantage that
they are less likely to migrate than Polyflex stents. For patients
with complex hypopharyngeal strictures, the use of the modified Niti-S stents may be beneficial.39 In this study, the best results
obtained with the 10- or 12-mm cervical Niti-S stent types that
are flared and fully covered. The use of covered self-expanding
stents have additionally been used for management of spontaneous perforations,40-42 iatrogenic perforations,43 and anastomotic
leaks.44
Freeman et al.40 reported on the use of silicone-coated stents
in 21 patients with spontaneous esophageal perforation all were
placed endoscopically using general anesthesia and fluoroscopy.
Adequate drainage of infected areas was achieved, Leak occlusion occurred in 17 patients (89%). Fifteen patients (79%) were
able to initiate oral nutrition within 72 h of stent placement.
Two patients (10%) with a perforation extending across the gastroesophageal junction experienced a continued leak after stent
placement and underwent operative repair. Leak occlusion was
confirmed by esophagram. Stent migration was documented in
four patients (21%) requiring repositioning or replacement. Stents
were removed at a mean of 20 15 days after placement. Hospital
length of stay was 9 12 days.
A recent prospective study,44 one from Germany, included
30 patients with postoperative anastomotic leaks, all were treated
by endoscopic insertion of self-expanding covered plastic stents.
Complete leak healing was obtained in 27 of 30 patients (90%).
The mean healing time was 30 days. In-hospital mortality after
treatment of esophageal leaks with stents was 3% (1 patient). The
second study by Blackmon et al.45 from the department of surgery in the Methodist hospital in Huston Texas reported a mean
follow-up of 15 months, 23 of the 25 patients with esophageal
or gastric leaks during a 15-month period were managed with
endoscopic stenting as primary treatment. Healing occurred in
patients who were stented for anastomotic leakage after gastric
bypass or sleeve gastrectomy (n = 10). One patient with three
esophageal iatrogenic perforations healed with stenting. Eight
patients successfully avoided esophageal diversion and healed
with stenting and adjunctive therapy. Two of the four patients
with tracheoesophageal fistulas sealed, one patient with an upper
esophageal perforation treated successfully with stent placement. These studies imply that stenting is a viable option for the
treatment of esophageal anastomotic leaks and perforations.

PMPH_CH10.indd 87

87

This treatment modality includes adequate drainage and stents


placement in selected patients. When done selectively it resulted
in rapid leak occlusion, provided the opportunity for early oral
nutrition, while future studies may give more information on
hospital length of stay and comparing its outcomes and morbidity to operative repair.
Multiple reports and prospective nonrandomized study support the use of removable covered stents in anastomotic leak and
benign strictures (Level 3b evidence; Grade C recommendation).

WHAT IS THE FUTURE?


Future developments in stent design include the development of
biodegradable stents for benign stenoses,46 but another possible
application could be the treatment of malignant dysphagia in
patients undergoing palliative chemotherapy.
Biodegradable stents theoretically represent an ideal treatment approach for refractory benign esophageal stricture patients
because it may provide prolonged temporary patency in association with stricture remodeling. The potential efficacy of biodegradable stents relies on the simple concept that a stricture that
needs repeat dilation will respond to prolonged dilation with an
endoprosthesis left in situ for several weeks or months. This temporary remodeling of the fibrotic stenosis is obtained with a stent
made of highly biocompatible material that should not induce
mechanical or irritative damage with growth of granulation tissue, new stricture formation, or fistula development. Until now,
few studies have reported the use of biodegradable stents for the
treatment of benign esophageal conditions that could potentially
decrease the need for reinterventions for stent removal.47 In a single case series from Japan, 13 patients (2 with caustic strictures,
4 with anastomotic strictures, 7 with esophageal cancer after
endoscopic mucosal dissection) were treated with a biodegradable stent constructed of poly-l-lactic acid monofilaments. Stent
migrations were seen in 10 patients (77%) within 10 to 21 days of
placement, whereas the stent remained in position in 3 patients
(23%). No symptoms of restenosis were observed and further
endoscopic therapies were not required. The same investigators
reported encouraging results in two other patients with postendoscopic submucosal dissection, stent placement with the intent
to reduce the risk of esophageal stricture.48
Other developments for malignant esophageal strictures
include the incorporation of Beta-emitting agents49 and cytotoxic agents in esophageal stents, which may prevent recurrent
tumor overgrowth at both ends of the stent. An exciting option
for refractory benign esophageal strictures is the possibility of
resecting these strictures with a circular stapling device through
a minimally invasive endoscopic and transgastric approach.50
Finally, it can be anticipated that new devices for endoscopic surgery will be developed, which might provide additional tools for
use in patients with refractory benign strictures.51
Thoracic and esophageal surgeons must be familiar with the
stents options to treat esophageal pathology. They should master
endoscopic skills and have the stenting procedure available as part
of the growing field of esophageal surgery.

5/21/2012 8:47:01 PM

88

Surgery: Evidence-Based Practice

Clinical Question Summary


Question

Answer

1 What are the


indications for
esophageal stents: Is
it only for palliation?

Esophageal stents are very useful in providing


immediate relief of dysphagia and
represent the treatment of choice in cases
involving a fistula between the esophagus
and the airway.

2b

8-14

2 What are the stents


options: Metal versus
plastic? Covered
or uncovered?
Removable or
permanent?

Most of the recent study support the


advantage of metal covered stents over
the over products, in patency, less
reintervention, and better outcome. Most
of these type of stents can be removed in
the initial couple of weeks.

2b

23-30

3 What is the optimal


insertion technique?

Fluroscopic control placement with


endoscopic assistance for direct
visualization has been described in most of
the studies to give optimal results.

3a

31-33

4 What is the role of


stents in benign
esophageal stricture
and anastomotic
leak?

Endoscopic insertion of self-expanding


covered plastic stents in benign stricture
and in anastomotic leak have been
described in couple prospective studies
with limited recommendation of treatment
in highly selective patient.

3b

41-45

REFERENCES
1. Jemal A, Siegel R, Ward E, et al. Cancer statistics, CA Cancer J
Clin. 2008;58(2):71-96.
2. National cancer institute online data baseNCI Office of Communications and Education Bethesda, MD 20892-8322 www.
cancer.gov.
3. El-Serag HB. The epidemic of esophageal adenocarcinoma. Gastroenterol Clin N Am. 2002;31:421-40.
4. Kamangar F, Dores GM, Anderson WF. Patterns of cancer incidence, mortality, and prevalence across five continents: Defining priorities to reduce cancer disparities in different geographic
regions of the world. J Clin Oncol. May 10, 2006;24(14):21372150.
5. Javle M, Ailawadhi S, Yang GY, et al. Palliation of malignant dysphagia in esophageal cancer: A literature-based review. J Support
Oncol. 2006;4(8):365-373.
6. Knyrim K, Wagner HJ, Bethge N, Keymling M, Vakil N. A controlled trial of an expansile metal stent for palliation of esophageal obstruction due to inoperable cancer, New Eng J Med.
October 1993;329(18):1302-1307.
7. Weigel TL, Frumiento C, Gaumintz E. Endoluminal palliation
for dysphagia secondary to esophageal carcinoma. Surg Clin
North Am. 2002;82(4):747-761.
8. Kyung Rae Kim, Ji Hoon Shin, Ho-Young Song, et al. Palliative
treatment of malignant esophagopulmonary fistulas with covered expandable metallic stent. AJR. October 2009;193:w278w282.
9. Spivak H, Katariya K, Lo AY, Harvey JC. Malignant tracheoesophageal fistula: Use of esophageal endoprosthesis. J Surg Oncol.
September 1996;63(1):65-70.
10. Van Heel NC, Haringsma J, Spaander MC, et al. Esophageal
stents for the relief of malignant dysphagia due to extrinsic compression. Endoscopy. July 2010;42:536-540.

PMPH_CH10.indd 88

Levels of
Evidence

Grade of
Recommendation

References

11. Bethge N, et al. Palliation of malignant esophageal obstruction


due to intrinsic and extrinsic lesions with expandable metal
stents. Am J Gastroenterol. 1998;93:1829-1832.
12. Gupta NK, et al. Self-expanding oesophageal metal stents for the
palliation of dysphagia due to extrinsic compression. Eur Radiol.
1999;9:1893-1897.
13. De Palma GD, Catanzano C. Removable self expending metal
stents: A pilot study for treatment of achalasia of the esophagus.
Endoscopy. 1998;30:S95-S96.
14. Siersema PD, Treatment options for esophageal strictures. Nat
Clin Prac Gastroenter Hepatol. 2008;5:142-152.
15. De Palma GD, lovino P, Masone S, Persico M, Persico G. Selfexpanding metal stents for endoscopic treatment of esophageal
achalasia unresponsive to conventional treatments. Long-term
results in eight patients. Endoscopy. 2001;33:1027-1030.
16. Lee JG, Hsu R, Leung JW. Are self-expanding metal mesh stents
useful in the treatment of benign esophageal stenoses and fistulas? An experience of four cases. Am J Gastroenterol. 2000;
95:1920-1925.
17. Cheng YS, Li MH, Chen WX, Zhuang QX, Chen NW, Shang KZ.
Follow-up evaluation for benign stricture of upper gastrointestinal tract with stent insertion. World J Gastroenterol. 2003;9:
2609-2611.
18. Cheng YS, Li MH, Chen WX, Chen NW, Zhuang QX, Shang KZ.
Selection and evaluation of three interventional procedures for
achalasia based on long-term follow-up. World J Gastroenterol.
2003;9:2370-2373.
19. Mukherjee S, Kaplan DS, Parasher G, Sipple MS. Expandable
metal stents in achalasiais there a role? Am J Gastroenterol.
2000;95:2185-2188.
20. Ying-Sheng Cheng, Fang Ma, Yong-Dong Li, Ni-Wei Chen, WeiXiong Chen, Jun-Gong Zhao, Chun-Gen Wu. Temporary selfexpanding metallic stents for achalasia: A prospective study with
a long-term follow-up. World J Gastroenterol. 2010;16:5111-5117.

5/21/2012 8:47:01 PM

The Use of Esophageal Stents

21. May A, et al. Self-expanding metal stents for palliation of malignant obstruction in the upper gastrointestinal tract. Comparative
assessment of three stent types implemented in 96 implantations.
J Clin Gastroenterol. 1996;22:261-266.
22. Dorta G, et al. Comparison between esophageal Wallstent and
Ultraflex stents in the treatment of malignant stenoses of the
esophagus and cardia. Endoscopy. 1997;29:149-154.
23. Schmassmann A, et al. Self-expanding metal stents in malignant
esophageal obstruction: A comparison between two stent types.
Am J Gastroenterol. 1997;92:400-406.
24. Vakil N, et al. A prospective, randomized, controlled trial of
covered expandable metal stents in the palliation of malignant
esophageal obstruction at the gastroesophageal junction. Am J
Gastroenterol. 2001;96:1791-1796.
25. Siersema PD, et al. A comparison of 3 types of covered metal
stents for the palliation of patients with dysphagia caused by
esophagogastric carcinoma: A prospective, randomized study.
Gastrointest Endosc. 2001;54:145-153.
26. Sabharwal T, et al. A randomized prospective comparison of
the Flamingo Wallstent and Ultraflex stent for palliation of dysphagia associated with lower third oesophageal carcinoma. Gut.
2003;52:922-926.
27. Homs MY, et al. Causes and treatment for recurrent dysphagia
after self-expanding metal stent placement for palliation of
esophageal carcinoma. Endoscopy. 2004;36:880-886.
28. Verschuur EM, et al. New design esophageal stents for the palliation of dysphagia from esophageal or gastric cardia cancer:
A randomized trial. Am J Gastroenterol [doi: 10.1111/j.15720241.2007.01542.x
29. Dumonceau JM, et al. Esophageal fistula sealing: choice of stent,
practical management, and cost. Gastrointest Endosc 1999;49:
70-78.
30. Verschuur EM, et al. Esophageal stents for malignant strictures
close to the upper esophageal sphincter. Gastrointest Endosc.
2007;66:1082-1090.
31. Yoon CJ, et al. Removal of retrievable esophageal and gastrointestinal stents: experience in 113 patients. AJR Am J Roentgenol.
2004;183:1437-1444.
32. Lew RJ, et al. A review of endoscopic methods of esophageal
dilation. J Clin Gastroenterol. 2002;35:117-126.
33. Pereira-Lima JC, et al. Endoscopic dilation of benign esophageal strictures: Report on 1043 procedures. Am J Gastroenterol.
1999;94:1497-1501.
34. Honkoop P, et al. Benign anastomotic strictures after transhiatal
esophagectomy and cervical esophagogastrostomy: Risk factors
and management. J Thorac Cardiovasc Surg. 1996;111:1141-1148.
35. Poley JW, et al. Ingestion of acid and alkaline agents: outcome
and prognostic value of early endoscopy. Gastrointest Endosc.
2004;60:372-377.

PMPH_CH10.indd 89

89

36. Evrard S, et al. Self-expanding plastic stents for benign esophageal lesions. Gastrointest Endosc. 2004;60:894-900.
37. Repici A, et al. Temporary placement of an expandable polyester
silicone-covered stent for treatment of refractory benign esophageal strictures. Gastrointest Endosc. 2004;60:513-519.
38. Holm AN, et al. Self-expanding plastic stents in treatment of
benign esophageal conditions. Gastrointest Endosc. [doi: 10.1016/
j.gie.2007.04.031]
39. Conio M, et al. A modified self-expanding Niti-S stent for the
management of benign hypopharyngeal strictures. Gastrointest
Endosc 2007;65:714-720.
40. Freeman RK, Van Woerkom JM, Ascioti AJ. Esophageal stent
placement for the treatment of iatrogenic intrathoracic esophageal perforation. Ann Thorac Surg. 2007;83:2003-2007.
41. Amrani L, Menard C, Berdah S, et al. From iatrogenic digestive perforation to complete anastomotic disunion: endoscopic
stenting as a new concept of stent-guided regeneration and reepithelialization. Gastrointest Endosc. 2009;69:1282-1287.
42. McLoughlin MT, Byrne MF. Endoscopic stenting: Where are we
now and where can we go? World J Gastroenterol. 2008;14:37983803.
43. Salminen P, Gullichsen R, Laine S. Use of self-expandable metal
stents for the treatment of esophageal perforations and anastomotic leaks. Surg Endosc. 2009;23:1526-1530.
44. Sandha GS, et al. Expandable metal stents for benign esophageal
obstruction. Gastrointest Endosc Clin N Am. 1999;9:437-446.
45. Blackmon SH, Santora R, Schwarz P, et al. Utility of removable
esophageal covered self-expanding metal stents for leak and fistula management. Ann Thorac Surg, 2010;89(3): 931-937.
46. Fry SW, Fleischer DE. Management of a refractory benign
esophageal stricture with a new biodegradable stent. Gastrointest Endosc. 1997;45:179-182.
47. Repici A, Vleggaar FP, Hassan C, van Boeckel PG, et al. Efficacy
and safety of biodegradable stents for refractory benign esophageal strictures: The BEST (Biodegradable Esophageal Stent)
study. Gastrointest Endosc. 2010;72(5):927-934.
48. Saito Y, Tanaka T, Andoh A, et al. Novel biodegradable stents for
benign esophageal strictures following endoscopic submucosal
dissection. Dig Dis Sci. 2008;53:330-333.
49. Won JH, et al. Self-expandable covered metallic esophageal
stent impregnated with beta-emitting radionuclide: an experimental study in canine esophagus. Int J Radiat Oncol Biol Phys.
2002;53:1005-1013.
50. Lucktong TA, et al. Resection of benign esophageal stricture
through a minimally invasive endoscopic and transgastric approach. Am Surg. 2002;68:720-723.
51. Willingham FF, et al. Taking NOTES: Translumenal flexible
endoscopy and endoscopic surgery. Curr Opin Gastroenterol.
2007;23:550-555.

5/21/2012 8:47:01 PM

Commentary on
The Use of Esophageal Stents
Scott B. Johnson

The chapter entitled The Use of Esophageal Stents by Drs. Perry


and Jones provides an excellent literature review on the history, use,
and types of esophageal stents. It also provides an excellent summary of their use in: (1) malignant obstructions, including both
intrinsic strictures and extrinsic compression; (2) esophagorespiratory fistulas; (3) achalasia; (4) benign strictures; and (5) esophageal leaks, both from anastomotic disruptions and perforations. It
also provides what I consider to be a detailed description of the different types of stents available, their indications, the pros and cons
concerning their use, and the future of esophageal stenting.
Esophageal stenting has emerged as an important adjunctive tool in treatment of esophageal diseases. As is common with
many new and emerging technologies, esophageal stenting was
initially met with some skepticism, especially when considered
as treatment for historically difficult-to-treat problems such as
tracheoesophageal fistulas and anastomotic leaks, which had
up until relatively recently either been treated nonoperatively
providing little to no palliationor alternatively with large-scale
operations associated with significant morbidity and mortality.
However, as the use of esophageal stents for treating such difficult
problems have been studied and experience gained, indications
for their use has generally increased in popularity, and skepticism
for their use lessened. Although esophageal stenting has been
around a relatively long time for the treatment and palliation of
malignant strictures, their use was always tempered by tissue and
tumor in-growth, as well as their inability (or at best, difficulty)
to remove due to the fact that they were uncovered, exposing the
metal interstices as a lattice to the in-growth of tissue and tumor.
However, with the advent of covered stents a whole new range of
esophageal pathology is now able to be palliated and definitively
treated with esophageal stenting. In addition, their placement
and removal can be done with relative ease even in the endoscopy suite under moderate sedation as an outpatient procedure.
As their use increases for palliation of terminal disease however,
an inevitable increase in the emergence of stent-related complications will occur, altering terminal events and time frames.
One example of this is stenting of a tracheoesophageal fistula that
results after treatment of a bulky, inoperable cervical esophageal
cancer treated with combined chemoradiotherapy. Such a fistula
is doomed to heal, yet placement of a stent can provide additional
survival benefit preventing both fatal aspiration as well as what
would otherwise be nutritionally depleting dysphagia. Although
placement of a stent in this situation can actually prolong life, it

does so modestly and the ultimate outcome remains poor. However, instead of the terminal event being aspiration or malnutrition, the immediate cause of death may shift to aortic erosion and
bleeding, or, alternatively, mediastinal sepsis. Often patients get
such good palliation that they forget the severity of their disease.
One such example is a patient that had an inoperable esophageal
cancer causing both obstruction and airway fistulization that was
treated with a total of three stents in his esophagus, placed one
inside the others. His esophagus essentially ended up being a lead
pipe, although he continued to swallow without significant signs
or symptoms of aspiration or dysphagia for a prolonged period of
time. He kept returning expecting additional palliation and success until no further stents could be placed, at which time a difficult discussion was had with the patient and his family. However,
it is without question that without the stents the patient would
have died several months earlier although from a different terminal event.
Covered esophageal stents are relatively easy to deploy, easy
to remove, and usually only cause relatively minor, if any complications when initially inserted. Most complications are secondary
to stent migration or obstruction and only cause minor problems
such as abdominal pain or dysphagia, usually easily treated with
stent removal and when necessary, replacement. Serious complications are relatively rare but do occur, and include erosion into
the adjacent aorta, as well as fistulous erosion into the trachea
possibly secondary to pressure necrosis, although the underlying
disease process and its primary treatment continue to be contributory factors.
Even though the level of evidence supporting recommendations regarding using esophageal stenting is generally inadequate
to permit firm conclusions regarding definite indications and
contraindications for their use, the authors have done an excellent job in reviewing the available literature regarding the clinical
applications of esophageal stents. Despite their potential drawbacks, esophageal stents can successfully treat esophageal leaks,
anastomotic dehiscence, airway fistulas, undilatable strictures,
and iatrogenic injuries that would have otherwise been devastating complications to either palliate or definitively treat, requiring
either long-term hospitalizations at best, or extensive, commando-type operations to correct when more conservative treatment
has failed. As such, they have become an invaluable and an indispensible tool that every esophageal surgeon should be comfortable and facile to use and deploy.
90

PMPH_CH10.indd 90

5/21/2012 8:47:01 PM

CHAPTER 11

Neoadjuvant and Adjuvant


Treatment of Esophageal Cancer
J. Camilo Barreto and Mitchell C. Posner

INTRODUCTION

stage influences therapeutic decisions, the next step to define


further treatment consists of the imaging workup to adequately
characterize the tumor, nodal and metastatic (TNM) extent of the
disease. For this purpose, imaging studies can be broadly categorized according to whether they are used for local, regional, or
systemic staging.
Endoscopic ultrasound (EUS) is most commonly used for
local tumor staging and for regional lymph node assessment. Retrospective studies have shown higher accuracy of EUS (76%89%)
compared with computed tomography (CT) scan (50%) for primary tumor staging.4 For lymph node staging, the accuracy of EUS
ranges from 70% to 80% compared with 50% for CT scan.4 Based
on morphology alone, EUS may not be capable to discriminate
between metastatic and enlarged benign lymph nodes. However,
EUS-guided fine needle biopsy (FNA) can improve the accuracy
for nodal staging up to 90%.5 EUS has also demonstrated to be
superior to magnetic resonance imaging (MRI) and 18fluorodeoxyglucose (FDG) positron emission tomography (PET) in local and
regional staging.6,7 An analysis by Lowe et al.7 showed an accuracy
of 71% for EUS in local tumor staging, compared with 42% for CT
or PET. However, the sensitivity and specificity between the three
modalities was not different for lymph node staging. In addition
to these reports, a meta-analysis by Van Vliet et al.6 showed that
EUS has higher sensitivity (80%) than CT (50%) and FDG-PET
(57%) for detection of regional lymph node metastases, although
lower specificity (70%, 80%, and 83%, respectively). The same
study also demonstrated the superiority of EUS to detect celiac
lymph node involvement.
CT scan of the chest and abdomen is currently used routinely
in most centers to assess local tumor extent but most importantly
to rule out disseminated disease. However, PET has been added
recently to the staging armamentarium of esophageal cancer as
it can change the treatment plan in up to 20% of patients.8-10 In
the aforementioned meta-analysis by Van Vliet et al.,6 FDG-PET
showed an increased sensitivity for distant metastases (71%),
when compared with CT scan (52%), and both imaging modalities

Esophageal cancer is a highly lethal tumor. Despite continued


attempts to improve outcomes with adjuvant or neoadjuvant therapy, long-term survival remains poor. There has been significant
progress in reducing perioperative morbidity and mortality in
the past decades as a result of improved surgical techniques and
perioperative management. However, the oncologic outcomes of
surgical resection alone are unacceptably low, yielding 5-year survival rates in the range of 10% to 30%.1-3 The majority of patients
succumb to metastatic disease, underscoring the need for systemic treatment. The fact that most patients are diagnosed at a
locally advanced stage has stimulated the use of combined modality therapy, especially in the neoadjuvant setting, in an effort to
improve outcome. Numerous regimens and combinations have
been evaluated in clinical trials in the past decades, including chemotherapy and radiation therapy (RT), alone or in combination in
the perioperative setting. Other studies have suggested a role for
definitive chemoradiotherapy (CRT) without surgical resection.
Despite multiple published trials and some progress achieved in
treatment outcomes, there is still significant room for improvement and many questions remain unanswered regarding the best
approach to treat esophageal cancer. The pathology and treatment
of cervical esophageal cancer (usually with squamous cell histology) is more similar to that of other head and neck cancers. For
this reason, this chapter will be more focused on adjuvant and
neoadjuvant treatment of carcinoma of the thoracic esophagus
and GE junction.
1. What are the proper diagnostic and imaging studies prior to
and during treatment of esophageal cancer?
ANSWER: The initial diagnosis of esophageal cancer is achieved
with upper endoscopy and tissue biopsies to obtain histologic
confirmation. In addition, histologic diagnosis is a prerequisite
if either RT and/or chemotherapy is being planned. Since tumor
91

PMPH_CH11.indd 91

5/21/2012 8:49:05 PM

92

Surgery: Evidence-Based Practice

demonstrated high specificity (>90%). In a prospective study of


74 patients, Flamen et al.8 showed that PET had a higher accuracy to diagnose stage IV disease compared with CT scan and
EUS combined (82% vs. 64%). Although it was less sensitive than
EUS for nodal staging, its specificity was higher (98%) than CT
and EUS combined (90%). Another retrospective analysis by van
Westreenen et al.9 found that the number of unnecessary explorations due to metastatic disease could be reduced from 44% when
patients were staged with CT scan alone to 21% if patients had
a preoperative FDG-PET scan. The ACOSOG trial Z0060 was
designed to define the role of FDG-PET in the detection of metastatic disease after patients had completed conventional staging
with CT scan. FDG-PET identified unsuspected metastatic disease
that was subsequently confirmed by biopsy or further work up in
4.8% of patients, with an additional 3.7% histologically unconfirmed cases.10 More recently, FDG-PET has been combined with
CT images (PET-CT) to complement the functional and anatomic
information provided by both modalities.
PET scan has shown another potential role in assessing tumor
response during systemic treatment for esophageal cancer. An
objective metabolic response, manifested by a decrease in the FDG
standard uptake value (SUV) in the primary tumor, correlates with
improved histopathologic response and survival compared with
patients who do not respond,11,12 and can help to tailor treatment
depending on response.11 In addition, the proportional decrease in
SUV seems to correlate with the survival outcome after treatment,
and higher preoperative SUV has been correlated with decreased
survival.13 It should be noted that in these reports, a PET response
was not an accurate predictor of pathologic complete response.
2. Should adenocarcinoma and squamous cell carcinoma (SCC)
of the esophagus be treated differently?
ANSWER: Adenocarcinoma and SCC of the esophagus are distinct histologic entities that should be studied and managed as
such. Although some risk factors are common for both tumor
types, such as smoking and previous RT to the mediastinum,
other factors are more histologic-type specific (i.e., chronic irritation conditions and alcohol abuse predispose to SCC, whereas
gastroesophageal reflux disease, Barretts metaplasia, and obesity are risk factors for adenocarcinoma). In addition, while SCC
tends to be more common in endemic areas of the world, in the
USA adenocarcinoma has become more prevalent in the last three
decades. This is a germane consideration when examining the
published literature on therapeutic approaches, since older studies usually included a higher proportion of patients with SCC, and
still a significant proportion of recent studies have included mixed
populations with both histologic types. However, a more careful
analysis of the current evidence can provide some clues to the
value of a tailored therapeutic approach to each tumor type.
Due to the significant morbidity and mortality associated
with esophagectomy, and level 1 evidence of improved outcomes
with CRT compared with RT alone without surgery, some investigators have suggested that patients with esophageal cancer may
derive an equivalent benefit from definitive CRT alone. Two trials
have directly addressed this question in patients with SCC of the
esophagus. The German Esophageal Cancer Study Group14 randomized 172 patients with locally advanced (T3 and T4) SCC of
upper- and mid-third esophagus to CRT followed by surgery versus CRT alone. Sixty-six percent of the patients in the surgery arm
group ultimately underwent resection. In the intention-to-treat

PMPH_CH11.indd 92

analysis, the trial demonstrated comparable 2-year overall survival (OS) in both treatment groups (39.9% vs. 35.4% respectively),
although the group that underwent surgery had better 2-year local
progression-free survival (64.3% vs. 40.7%). Notably, patients in
the surgical group also had a lower rate of cancer-related mortality, but this was offset by an increase in treatment-related mortality (12.8% in the surgical group vs. 3.5% in the definitive CRT
group). The FFCD 9102 trial,15 in which 90% of the patients had
a diagnosis of SCC, included 259 patients with T3 tumors. The
main difference with the German trial was that only patients who
responded to initial CRT were then randomized to definitive CRT
or resection. In a similar fashion to the German trial, there was
no significant difference in 2-year OS (40% vs. 34%, respectively),
although there was a higher frequency of locoregional recurrence
in the group without surgery. The 3-month mortality was 9.3%
in the surgical group, compared with 0.8% in the definitive CRT
group. From these two trials it can be concluded that in patients
with SCC tumor, response correlates with a better prognosis, and
that in patients who respond, definitive CRT is an acceptable
alternative to resection, with lower treatment-related mortality
although with a higher risk of local recurrence. Non-responders
have a worse prognosis, but salvage surgery may play a role if an
R0 resection is accomplished.14 These trials provide high-level evidence favoring definitive CRT for patients with SCC histology, but
suggest that if perioperative mortality is minimized, that oncologic outcome may be enhanced with the addition of resection.
In patients with adenocarcinoma histology, the available
literature has focused more on the value of combining chemotherapy and/or RT with surgery, rather than assessing the role of
surgery. Many randomized trials have been conducted to compare
multimodality therapy including surgery with surgery alone.16-19
Despite the fact that most centers routinely use combined therapy
as the standard of care, the available literature has shown confl icting results, and based on evidence, its role in improving the outcomes of surgical treatment remains under investigation. For this
reason, surgery remains the cornerstone of therapy in patients
with adenocarcinoma and is the preferred option, generally in the
context of multimodality therapy. The trials comparing multimodality therapy with surgery alone will be addressed in more detail
in the following sections.
3. What is the role of adjuvant or neoadjuvant RT alone in
esophageal cancer?
ANSWER: RT has the theoretical advantage of increasing local
control rates either by decreasing the tumor size before resection
thereby maximizing the chance for an R0 resection or by treating residual microscopic disease when administered after surgery.
However, it is also associated with a higher risk of complications
due to fibrosis and impaired healing. There have been five randomized trials evaluating RT alone in the adjuvant setting and
another five trials for neoadjuvant RT with varying patient numbers. None has demonstrated a benefit in OS. The two largest trials were conducted by French and Chinese investigators. Tnire
et al.20 randomized over 200 patients with SCC of the esophagus
after curative resection to 45 to 55 Gy versus observation. Postoperative irradiation did not improve survival, although there
were fewer recurrences in the radiation group. In the only study
to show more favorable results, Xiao et al.21 randomized almost
500 patients to doses of 50 to 60 Gy postoperatively. In this study,
there was no OS benefit with the addition of RT (32% vs. 41%;

5/21/2012 8:49:05 PM

Neoadjuvant and Adjuvant Treatment of Esophageal Cancer

P = 0.44). In the subgroup of patients with lymph node metastases


and stage III, they found a survival benefit. Other trials22-24 have
failed to show a survival advantage, and although they were generally associated with better local control rates, there were also
increased complication rates in the group treated with radiation.
Five randomized trials have assessed the value of neoadjuvant RT
and none have demonstrated a significant benefit with the use of
RT alone;25-29 a meta-analysis from the Cochrane collaboration
did not find clear evidence of a benefit from neoadjuvant RT.30
Although not a trial of adjuvant or neoadjuvant therapy, the
RTOG 850131 was a landmark trial to establish the superiority of
CRT over RT alone for definitive treatment of esophageal cancer.
One hundred and twenty-three patients, most with SCC, were randomized to either CRT or RT, without surgery, and even though
the CRT group received a lower dose of radiation, there was a benefit with combined therapy with regard to OS, local recurrence
and distant metastases rates.
Given these results and considering that generally better
outcome has been reported with combined modality therapy, RT
alone is not usually indicated in the adjuvant or neoadjuvant setting. Its main role is for palliation of symptoms in patients who
are deemed not candidates for surgery.
4. Is preoperative CRT better than preoperative chemotherapy
alone for esophageal cancer?
ANSWER: In North America, most cancer centers use neoadjuvant CRT as the standard of care as opposed to neoadjuvant
chemotherapy alone for patients with esophageal cancer. Current
evidence suggests that the advantages of CRT over chemotherapy
include higher rates of R0 resection, complete pathologic response,
and local tumor control. However, no consistent improvements in
OS have been demonstrated.
The trials examining preoperative chemotherapy versus
surgery alone have shown conflicting results. Older studies
included, in the main, patients with SCC who were underpowered
and did not demonstrate a significant benefit with preoperative
chemotherapy.29,32,33
More recent and larger trials, including patients with both
histologies have reported inconsistent results.1,34,35 The Intergroup
trial 113,1 which included 452 patients, did not demonstrate a benefit from preoperative chemotherapy when compared with surgery
alone, although most patients did not complete the intended regimen. In contrast, a similar trial from the British Medical Research
Council34 enrolled 802 patients, most of who had adenocarcinoma,
and showed a significant improvement in OS for both tumor histologies (43% in the neoadjuvant chemotherapy group vs. 34% in
the surgery alone group at 2 years). The Medical Research Council
(MRC) also conducted the MAGIC trial35 in patients with gastric
and gastroesophageal junction adenocarcinoma, in which onefourth of the patients had adenocarcinoma of the distal esophagus.
It compared pre- and postoperative chemotherapy with resection
alone. There was a significant benefit in 5-year survival favoring
the perioperative chemotherapy group (36% vs. 23%), although it
was not specifically powered for patients with esophageal cancer.
There have been several trials conducted evaluating concurrent preoperative CRT versus surgery alone. Five of these trials
have included 5-fluorouracil (5-FU)- and cisplatin-based CRT regimens. Walsh et al.17 reported improved OS with CRT (32% vs. 6%
at 3 years) in patients with adenocarcinoma, as well as increased
median survival (16 months vs. 11 months). The much lower than

PMPH_CH11.indd 93

93

expected survival in the surgery-only arm (6% 5-year survival)


raises a cautionary note in this trial. Following this report, three
other randomized trials2,16,18 failed to demonstrate a benefit in OS
with the use of preoperative CRT. The EORTC trial,2 designed
for patients with SCC, showed improved DFS in the combination
group, but an OS benefit may have been potentially compromised
by a high postoperative mortality rate. The trials from the University of Michigan16 and the Australasian Group18 did not show
an improvement in OS. The former was underpowered to demonstrate an OS benefit while the latter delivered lower than standard
doses of both chemotherapy and RT. Finally, although the Cancer
and Leukemia Group B (CALGB) 9871 trial19 demonstrated an
impressive improvement in OS in the combined treatment group
(39%) versus the surgery-only group (16%), this trial was closed
early after accruing only 56 of 500 planned patients with adenocarcinoma of the esophagus and therefore the results are suspect.
Only one trial compared directly preoperative chemotherapy
with preoperative CRT.36 One hundred and twenty-six patients
with locally advanced adenocarcinomas of the GE junction were
randomly assigned to either treatment, followed by surgery. Again,
accrual goals were not achieved, although a trend toward improved
3-year survival (47.4% vs. 17.7%) and higher rates of pathologic
complete response were observed in the trimodality group.
A recent report provides the best evidence to date of the
potential value of preoperative CRT. The phase III CROSS study
compared a more modern regimen of carboplatin, paclitaxel, and
concurrent RT with surgery alone in resectable esophageal or
EG junction cancers.37 The combined treatment group had significantly improved OS (HR 0.67) and R0 resection rates (92% vs.
65%). Importantly operative mortality was low in both groups suggesting that if excellent surgical results are achieved and consistent
with what is reported in high-volume centers, neoadjuvant therapy
may prove to be of value.
Due to the inconsistent results reported in clinical trials of
neoadjuvant CRT, there have been several meta-analyses that have
attempted to address the shortcomings of single small, heterogeneous trials that were designed to test the worth of preoperative
CRT.38-40 These meta-analyses have reported results in favor of CRT
and a significant proportion of patients were downstaged prior to
surgery. The most recent meta-analysis40 identified a 19% decrease in
the HR for death, which was significant for both tumor histologies. It
should be noted that in this meta-analysis, they separately analyzed
the role of neoadjuvant chemotherapy without radiation and a survival benefit was also identified, although of a lesser degree.
In the USA, preoperative CRT followed by surgery is the most
common approach for patients with resectable disease, although
it remains investigational. In summary, although there are conflicting results, the available data suggests that neoadjuvant CRT
improves local control and can modestly improve survival, compared with surgery alone.
5. What is the role of surgery in the combined modality therapy
of esophageal cancer?
ANSWER: As described above, there is evidence to support a regimen of definitive CRT in patients with SCC, in whom the addition of
surgery may not improve survival. Briefly, the German Esophageal
Cancer Study Group14 comparing patients with locally advanced
SCC treated with a regimen of CRT plus surgery versus definitive
CRT, found similar 2-year OS between the two groups, even though
there was better local control and cancer-related mortality in the

5/21/2012 8:49:05 PM

94

Surgery: Evidence-Based Practice

surgical group. These findings suggest that any potential benefit in


cancer outcome from preoperative CRT combined with surgery
were potentially abrogated due to the high treatment-related mortality associated with surgery. The FFCD 9102 trial,15 which randomized responders to initial CRT to definitive CRT versus surgery
also did not find an OS benefit with preoperative CRT, but did demonstrate a lower rate of disease- specific deaths. Again preoperative
CRT was associated with significant treatment mortality. Patients
with SCC that do not respond to CRT may derive a benefit from
salvage surgery. Based on these trials, either definitive CRT without surgery or trimodality treatment including resection is a viable
treatment option for patients with esophageal SCC.
No such trials have been conducted to compare combined
treatment with and without surgery exclusively in patients with
adenocarcinoma of the esophagus. In the FFCD 9102 trial,15 only
11% of 259 patients had this histology, and although no difference
between SCC and adenocarcinoma outcomes was observed in a
multivariate analysis, the authors acknowledged that their results
were applicable mainly for patients with SCC. As detailed earlier,
CRT trials have often shown conflicting results.2,16-19 Thus, surgical
resection has been the standard of care for patients with adenocarcinoma, and the trials that have assessed combined modality
therapy in this subset of patients have always included surgery in
both treatment arms.
Even in patients with SCC, based in the aforementioned
trials,14,15 surgery appears to enhance locoregional control. Patients
who obtain a complete pathological response to CRT derive no
benefit from resection, whereas those with residual local disease

following CRT would likely fail without surgery. Unfortunately,


current imaging methods are not accurate enough to discriminate between these two groups of patients. One can reasonably
conclude that esophagectomy is a critical component of multimodality therapy, but only contributes to improved outcome if the
mortality associated with resection is minimized.
6. What is the optimal neoadjuvant regimen for esophageal
cancer?
ANSWER: The optimal treatment strategy for esophageal cancer is still under investigation. From the trials described above,
and despite many of them failing to demonstrate a clear improvement in survival, in most cancer centers combined neoadjuvant
chemotherapy and RT is considered the standard approach for
patients with resectable disease. The recommended regimens used
in trials that have shown a survival benefit include cisplatin plus
5-FU.17,19,40 Based on the most recent multicenter phase III CROSS
trial,37 paclitaxel- and carboplatin-based CRT can prolong median
survival from 26 to 49 months with a HR for death of 0.67 in the
CRT group and should be now considered as an acceptable standard regimen. In the specific subset of patients with distal third
or GE junction adenocarcinoma, the chemotherapy regimen of
epirubicin, cisplatin, and 5-FU before and after surgery, based on
the results of the MAGIC trial,35 which showed improved 5-year
survival from 23% to 36% and a HR for death of 0.75, is a viable
option if radiation is not a component of the combined modality
approach. Current trials are under investigation to examine the
role of targeted therapy for esophageal cancer.

Clinical Question Summary


Levels of
Evidence

Grade of
References
Recommendation

Question

Answer

1 What are the proper diagnostic


and imaging studies prior
to and during treatment of
esophageal cancer?

Upper endoscopy and biopsy for


diagnosis. EUS for local and regional
staging. CT and PET for systemic
staging.

1b

5-13

2 Should adenocarcinoma and


squamous cell carcinoma
(SCC) of the esophagus be
treated differently?

Surgery with combined therapy is


preferred for adenocarcinoma.
Definitive chemoradiation can be
considered for SCC.

1b

14, 15

3 What is the role of adjuvant


or neoadjuvant RT alone in
esophageal cancer?

RT alone has not demonstrated a survival


benefit either in the adjuvant or in the
neoadjuvant setting.

1a

20-31

4 Is preoperative CRT better than


preoperative chemotherapy
alone for esophageal cancer?

Most centers prefer CRT over


chemotherapy alone. CRT can improve
rates of pCR, R0 resection and local
recurrence. Improved OS was shown
in a meta-analysis.

1a

2, 17, 19, 34,


36-40

5 What is the role of surgery in


the combined modality therapy
of esophageal cancer?

Surgery improves local control and diseasespecific mortality, although with higher
treatment-related mortality. Selected
patients with SCC do not obtain a
survival benefit from it after CRT.

1b

2, 14-19

6 What is the optimal neoadjuvant


regimen for esophageal cancer?

Cisplatin- and 5-FU-based CRT or


paclitaxel- and 5-FU-based CRT. In
distal or GE junction adenocarcinoma,
ECF pre- and postoperatively.

1b

17, 19, 37, 40

ECF: epirubicin, cisplatin, 5-FU; pCR: pathologic complete response.

PMPH_CH11.indd 94

5/21/2012 8:49:05 PM

Neoadjuvant and Adjuvant Treatment of Esophageal Cancer

REFERENCES
1. Kelsen DP, Ginsberg R, Pajak TF, et al. Chemotherapy followed
by surgery compared with surgery alone for localized esophageal
cancer. N Engl J Med. 1998;339:1979-1984.
2. Bosset JF, Gignoux M, Triboulet JP, et al. Chemoradiotherapy
followed by surgery compared with surgery alone in squamouscell cancer of the esophagus. N Engl J Med. 1997;337:161-167.
3. Orringer MB, Marshall B, Iannettoni MD. Transhiatal
esophagectomy: Clinical experience and refinements. Ann Surg.
1999;230:392-400.
4. Van Dam J. Endosonographic evaluation of the patient with
esophageal cancer. Chest. 1997;112:184S-190S.
5. Vazquez-Sequeiros E, Norton ID, Clain JE, et al. Impact of
EUS-guided fi ne-needle aspiration on lymph node staging
in patients with esophageal carcinoma. Gastrointest Endosc.
2001;53:751-757.
6. Van Vliet EP, Heijenbrok-Kal MH, Hunink MG, Kuipers EJ,
Siersema PD. Staging investigations for oesophageal cancer: a
meta-analysis. Br J Cancer. 2008;98:547-557.
7. Lowe VJ, Booya F, Fletcher JG, et al. Comparison of positron
emission tomography, computed tomography, and endoscopic
ultrasound in the initial staging of patients with esophageal cancer. Mol Imaging Biol. 2005;7:422-430.
8. Flamen P, Lerut A, Van Cutsem E, et al. Utility of positron emission tomography for the staging of patients with potentially operable esophageal carcinoma. J Clin Oncol. 2000;18:3202-3210.
9. Van Westreenen HL, Heeren PA, van Dullemen HM, et al. Positron emission tomography with F-18-fluorodeoxyglucose in a
combined staging strategy of esophageal cancer prevents unnecessary surgical explorations. J Gastrointest Surg. 2005;9:54-61.
10. Meyers BF, Downey RJ, Decker PA, et al. The utility of positron
emission tomography in staging of potentially operable carcinoma of the thoracic esophagus: Results of the American College
of Surgeons Oncology Group Z0060 trial. J Thorac Cardiovasc
Surg. 2007;133:738-745.
11. Lordick F, Ott K, Krause BJ, et al. PET to assess early metabolic response and to guide treatment of adenocarcinoma of the
oesophagogastric junction: The MUNICON phase II trial. Lancet Oncol. 2007;8:797-805.
12. Kato H, Nakajima M, Sohda M, et al. The clinical application
of (18)F-fluorodeoxyglucose positron emission tomography to
predict survival in patients with operable esophageal cancer.
Cancer. 2009;115:3196-3203.
13. Javeri H, Xiao L, Rohren E, et al. The higher the decrease in
the standardized uptake value of positron emission tomography after chemoradiation, the better the survival of patients
with gastroesophageal adenocarcinoma. Cancer. 2009;115:
5184-5192.
14. Stahl M, Stuschke M, Lehmann N, et al. Chemoradiation with
and without surgery in patients with locally advanced squamous
cell carcinoma of the esophagus. J Clin Oncol. 2005;23:2310-2317.
15. Bedenne L, Michel P, Bouche O, et al. Chemoradiation followed
by surgery compared with chemoradiation alone in squamous
cancer of the esophagus: FFCD 9102. J Clin Oncol. 2007;25:
1160-1168.
16. Urba SG, Orringer MB, Turrisi A, et al. Randomized trial of
preoperative chemoradiation versus surgery alone in patients
with locoregional esophageal carcinoma. J Clin Oncol. 2001;
19:305-313.
17. Walsh TN, Noonan N, Hollywood D, et al. A comparison of multimodal therapy and surgery for esophageal adenocarcinoma.
N Engl J Med. 1996;335:462-467.

PMPH_CH11.indd 95

95

18. Burmeister BH, Smithers BM, Gebski V, et al. Surgery alone versus chemoradiotherapy followed by surgery for resectable cancer
of the oesophagus: A randomised controlled phase III trial. Lancet Oncol. 2005;6:659-668.
19. Tepper J, Krasna MJ, Niedzwiecki D, et al. Phase III trial of trimodality therapy with cisplatin, fluorouracil, radiotherapy, and
surgery compared with surgery alone for esophageal cancer:
CALGB 9781. J Clin Oncol. 2008;26:1086-1092.
20. Teniere P, Hay JM, Fingerhut A, Fagniez PL. Postoperative radiation therapy does not increase survival after curative resection
for squamous cell carcinoma of the middle and lower esophagus as shown by a multicenter controlled trial. French University Association for Surgical Research. Surg Gynecol Obstet.
1991;173:123-130.
21. Xiao ZF, Yang ZY, Liang J, et al. Value of radiotherapy after radical surgery for esophageal carcinoma: a report of 495 patients.
Ann Thorac Surg. 2003;75:331-336.
22. Kunath U, Fischer P. [Radical nature and life expectancy in the
surgical treatment of esophageal and cardial carcinoma] Dtsch
Med Wochenschr 1984;109:450-453.
23. Fok M, Sham JS, Choy D, Cheng SW, Wong J. Postoperative
radiotherapy for carcinoma of the esophagus: a prospective, randomized controlled study. Surgery. 1993;113:138-147.
24. Zieren HU, Mller JM, Jacobi CA, et al. Adjuvant postoperative
radiation therapy after curative resection of squamous cell carcinoma of the thoracic esophagus: A prospective randomized
study. World J Surg. 1995;19:444-449.
25. Launois B, Delarue D, Campion JP, Kerbaol M. Preoperative
radiotherapy for carcinoma of the esophagus. Surg Gynecol
Obstet. 1981;153:690-692.
26. Gignoux M, Roussel A, Paillot B, et al. The value of preoperative radiotherapy in esophageal cancer: Results of a study of the
EORTC. World J Surg. 1987;11:426-432.
27. Wang M, Gu XZ, Yin WB, et al. Randomized clinical trial on
the combination of preoperative irradiation and surgery in the
treatment of esophageal carcinoma: Report on 206 patients. Int J
Radiat Oncol Biol Phys 1989;16:325-327.
28. Arnott SJ, Duncan W, Kerr GR, et al. Low dose preoperative
radiotherapy for carcinoma of the oesophagus: results of a randomized clinical trial. Radiother Oncol. 1992;24:108-113.
29. Nygaard K, Hagen S, Hansen HS, et al. Pre-operative radiotherapy prolongs survival in operable esophageal carcinoma: A
randomized, multicenter study of pre-operative radiotherapy
and chemotherapy. The second Scandinavian trial in esophageal
cancer. World J Surg. 1992;16:1104-1109.
30. Arnott SJ, Duncan W, Gignoux M, et al. Oesophageal Cancer
Collaborative Group. Preoperative radiotherapy for esophageal carcinoma. Cochrane Database of Systematic Reviews
2005, Issue 4. Art. No.: CD001799. DOI: 10.1002/14651858.
CD001799.pub2
31. Al-Sarraf M, Martz K, Herskovic A, et al. Progress report of combined chemoradiotherapy versus radiotherapy alone in patients
with esophageal cancer: An Intergroup study. J Clin Oncol.
1997;15:277-284.
32. Roth JA, Pass HI, Flanagan MM, et al. Randomized clinical trial
of preoperative and postoperative adjuvant chemotherapy with
cisplatin, vindesine, and bleomycin for carcinoma of the esophagus. J Thorac Cardiovasc Surg. 1988;96:242-248.
33. Schlag PM. Randomized trial of preoperative chemotherapy
for squamous cell cancer of the esophagus. The Chirurgische
Arbeitsgemeinschaft Fuer Onkologie der Deutschen Gesellschaft Fuer Chirurgie Study Group. Arch Surg. 1992;127:
1446-1450.

5/21/2012 8:49:05 PM

96

Surgery: Evidence-Based Practice

34. Medical Research Council Oesophageal Cancer Working Group.


Surgical resection with and without chemotherapy in oesophageal cancer. Lancet. 2002;359:1727-1733.
35. Cunningham D, Allum WH, Stenning SP, et al. Perioperative
chemotherapy versus surgery alone for resectable gastroesophageal cancer. N Engl J Med. 2006;355:11-20.
36. Stahl M, Walz MK, Stuschke M, et al. Phase III comparison of
preoperative chemotherapy compared with chemoradiotherapy
in patients with locally advanced adenocarcinoma of the esophagogastric junction. J Clin Oncol. 2009;27:851-856.
37. Gaast AV, van Hagen P, Hulshof M, et al. Effect of preoperative concurrent chemoradiotherapy on survival of patients
with resectable esophageal or esophagogastric junction cancer:

PMPH_CH11.indd 96

Results from a multicenter randomized phase III study. J Clin


Oncol. 2010;28(suppl; abstr 4004):15s.
38. Fiorica F, Di Bona D, Schepis F, et al. Preoperative chemoradiotherapy for oesophageal cancer: A systematic review and metaanalysis. Gut. 2004;53:925-930.
39. Urschel JD, Vasan H. A meta-analysis of randomized controlled
trials that compared neoadjuvant chemoradiation and surgery
to surgery alone for resectable esophageal cancer. Am J Surg
2003;185:538-543.
40. Gebski V, Burmeister B, Smithers BM, et al. Survival benefits from neoadjuvant chemoradiotherapy or chemotherapy
in oesophageal carcinoma: A meta-analysis. Lancet Oncol.
2007;8:226-234.

5/21/2012 8:49:05 PM

CHAPTER 12

Esophageal Atresia and


Tracheoesophageal Fistula
Miller C. Hamrick, David E. Carney, and William C. Boswell

INTRODUCTION

associated defects. It is generally accepted that 50% to 70% of


patients with EA/TEF have associated anomalies, with 35% having cardiac defects and another 25% having genitourinary or gastrointestinal anomalies.6-8 Leonard et al. reported that the 1-year
survival rate of infants with congenital heart disease and EA/TEF
was 67% versus 95% for those with isolated EA/TEF.8 Of the congenital heart defects, the most common single defects were ventricular septal defect (VSD) in 19%, atrial septal defect (ASD) in 20%,
patent ductus artery (PDA) in 13%, and tetralogy of fallot in 5%.8

Esophageal atresia and tracheoesophageal fistula (EA/TEF) incorporate a range of congenital anomalies wherein maldevelopment of
the foregut presents clinically with discontinuity of the esophagus
and/or a fistula between the esophagus and trachea. EA/TEF is an
uncommon anomaly, occurring in 1 out of every 3000 to 5000 live
births. This congenital anomaly is identified with a slightly greater
incidence in Caucasian and male patients. As compared to the general population, there is a slight increased risk of development in
twins (2.5%) and siblings of a previously affected child (1%).
Esophageal atresia with tracheoesophageal fistula has been
described with great interest throughout the surgical literature
since its original description in 1670 by Dr William Durston,
who identified a blind-ending upper esophagus in one of a set of
conjoined twins.1 Although attempts at operative repair began as
early as the late 1800s, mortality remained lethal. Ultimately, primary repair was abandoned and management included a cervical
esophagostomy and gastrostomy tube. The first successful primary repair of esophageal atresia with tracheoesophageal fistula
was performed in 1941 by Dr Cameron Haight.2
Numerous varieties of this anomaly have been described,
with five subtypes commonly illustrated. The three most frequent
anomalies incorporate 98% of all cases (Fig. 12.1). The most common anomaly is Type C that presents with esophageal atresia and a
fistula between the distal trachea and the lower esophageal pouch
(86% of cases). Isolated esophageal atresia without a fistula (Type
A) occurs in 8% of cases, whereas an H-type tracheoesophageal
fistula without esophageal atresia (Type E) occurs in 4%.
In nearly one half of all cases of esophageal atresia, there
are malformations of other organ systems, most commonly the
cardiovascular system.3,4 The most well-defined cluster of malformations is the VACTERL association, which comprises vertebral, anorectal, cardiac, tracheoesophageal, renal, and limb
anomalies.5 Typically, for a patient to be classified as having
the VACTERL association, he or she must have two or more

Type A (8%)

Type B (1%)

Type D (1%)

Type C (86%)

Type E (4%)

Figure 12.1 Types of esophageal atresia and tracheoesophageal fistula.


97

PMPH_CH12.indd 97

5/21/2012 8:50:05 PM

98

Surgery: Evidence-Based Practice

Pertinent when considering the operative approach, 4% of patients


were found to have a right-sided aortic arch.
Additional anomalies are more common in the setting of Type
A atresia (proximal atresia with no fistula), and least common in
Type E (esophageal continuity with a cervical H-Type fistula).6-8
Dr de Jong and colleagues reported that 107 of the 463 patients
(23.1%) with EA or TEF qualified to have VACTERL anomaly.
Thirty percent of that cohort had only VACTERL-type defects
whereas 70% had VACTERL defects in association with other
structural defects. The additional structural defects included single
umbilical artery (20%), genital anomalies (23%), duodenal atresia
(8.9%), and cleft lip/palate (4.4%).9 Recently a VACTERL-H association has been described that includes congenital hydrocephalus.10
In addition, multiple chromosomal anomalies, genetic disorders,
and somatic syndromes are associated with EA and TEF.11-20
The initial workup of patients carrying a diagnosis of EA
or TEF must include evaluation for other anomalies. A complete
physical exam must be followed by a chest and pelvic radiograph,
echocardiogram, renal ultrasound, and spinal ultrasound. As
previously described, complex congenital heart defects are responsible for the majority of deaths in this patient population.8
Unfortunately, the rarity of congenital anomalies and the
limited number of clinical cases at any given institution makes
prospective treatment studies difficult. Unless otherwise noted,
the data referenced below are almost exclusively opinions of
respected authorities (Oxford Centre Level 3-5). Most categorical
recommendations are Grade C or D, based on the results of case
series from varied, international institutions that demonstrate
similar and reproducible outcomes.

RISK FACTORS
1. What are environmental and/or genetic risk factors that
predispose to the development of esophageal atresia and
tracheoesophageal fistula?
For years, lack of clarity has surrounded the mechanisms responsible
for normal embryogenesis of the trachea and esophagus. A rudimentary consensus is that the esophagus and trachea develop as a primitive tube that ultimately becomes a separate trachea and esophagus.
The mechanism by which this process occurs is still unclear though
theories include propagation of the respiratory system,22,23 septal
formation,24-27 or elongation and proliferation of the foregut.28,29
In part, the abnormal morphogenesis associated with EA
and TEF remains poorly understood due to limited availability
of human embryos for investigation, and the absence of satisfactory in vitro or computer-generated models. Attempts to recreate
the congenital malformation led to manipulation of environmental factors, yet consistent reproduction of this lesion was not
observed. Finally in the late 1970s, Thompson and coworkers generated malformations in rat and rabbit fetuses in response to the
chemotherapeutic agent methimazole (Adriamycin).30 Though the
anomalies produced by Adriamycin in rat models cause tracheoesophageal defects similar to those seen in humans, true investigation remains limited since the Adriamycin-induced anomalies
are predicated on interfering with DNA and RNA synthesis.23,31,32
Likewise, other environmental factors and genetic abnormalities
have been implicated in the development of EA/TEF.3,9,23,30-39
Answer: Although associated with many genetic disorders
and syndromes, methimazole is the only known causative factor

PMPH_CH12.indd 98

to have a reproducible association with tracheoesophageal anomalies. Level 2c evidence.

DIAGNOSIS
2. What is the safest and most accurate method to confirm the
diagnosis of EA with TEF?
With expanded prenatal screening, antenatal diagnosis of EA/TEF
is possible; however, the positive predictive value of a prenatal ultrasound is only 20% to 40%.40 In these cases, diagnosis relies on a small
or absent stomach bubble in the setting of maternal polyhydramnios. After birth, infants typically manifest symptoms with the first
feeding demonstrated by gagging, choking, and excessive drooling.
The diagnosis is initially suspected based on these symptoms and an
inability to pass an orogastric tube. Often an air-filled, dilated, proximal esophagus is visualized on plain radiographic imaging. Air in the
stomach and lower intestine suggests the presence of a distal fistula.
As with most esophageal lesions confirmation is obtained
through direct visualization, radiographic imaging, or both.
When the diagnosis of EA is unclear or if there is concern for a
proximal cervical fistula, a small amount of dilute, nonionic contrast material may be used for an upper pouch study under fluoroscopic guidance. Ideally, this should be performed at a center that
has both experience with pediatric care and has full resuscitative
capability in the event of aspiration. It remains imperative to use
isosomotic contrast as hyperosmolar contrast such as Gastrograffin, if aspirated, may result in severe pulmonary edema, respiratory
distress, and chemical pneumonitis.41 The benefits of bronchoscopy
over contrast esophagram include a higher rate of diagnosis, the
ability to evaluate for associated laryngotracheal and esophageal
anomalies, and safety.42 For the H-type fistula, the two methods
may prove complementary and repeated attempts at endoscopy
and imaging are often necessary to confirm the diagnosis.43
Answer: The diagnosis of a TEF requires a high index of
suspicion. Bronchoscopy/esophagoscopy is considered the best
method for safe, accurate diagnosis of common lesions (Level 3b
evidenceGrade D recommendation).

TREATMENT
The surgical treatment of EA/TEF will center on management as
it relates to short-gap and long-gap atresia. The most common presentation in the current era is a presumptive diagnosis of short-gap
atresia based on the clinical scenario and initial imaging. Rarely
is an attempt at primary repair hindered by insufficient length.
The alternative scenario is a presumptive diagnosis of long-gap
atresia at birth, wherein the surgeon needs to decide when and
how to reconstruct the esophagus. Once a patient has been appropriately screened for associated anomalies, there are three specific
concerns to consider. First the surgeon must be aware of options
to manage a high-output fistula in the unstable patient. Next it
is paramount to understand safe options to obtain sufficient
length in order to perform primary repair in the management of
short-gap esophageal atresia. Finally the management of long-gap,
Type A, esophageal atresia requires the broad understanding of
alternative conduits when esophageal replacement is mandated.
The inherent risk of fistulas communicating the distal trachea
and gastrointestinal tract include respiratory distress as a result

5/21/2012 8:50:05 PM

Esophageal Atresia and Tracheoesophageal Fistula

of ineffective ventilation and/or reflux with subsequent aspiration


and chemical pneumonitis. Elevated airway pressure of the newborn allows passive ventilation into the stomach leading to abdominal distention, elevated diaphragms, hypercarbia, and even gastric
perforation.44 Premature infants are at greatest risk of this complication and it is this scenario that requires emergent intervention.
3. What is the best method to stabilize the patient with a highoutput fistula?
Emergent treatment of a patient suffering respiratory or hemodynamic compromise secondary to a high-output tracheoesophageal
fistula can be managed in several ways. Passing a fogerty catheter
through or adjacent to the endotracheal tube, through the fistula,
with subsequent balloon occlusion of the distal esophagus has
been described in the emergent setting.45 Similarly, the creation
of a gastrostomy and retrograde passage of a balloon may effectively occlude the fistula.44 High-frequency jet ventilation has also
been offered as a method to stabilize a high-output fistula.46 Additional management options including gastrostomy waterseal,47
gastric division,48 and banding of the gastroesophageal junction49 have been described in the literature but are not commonly
recommended secondary to excess morbidity. Finally, a rapid
transpleural thorocotomy and ligation of the distal fistula along
with subsequent gastrostomy is a reliable temporizing measure.50
Answer: Of the numerous techniques described, the most
reliable technique employed by the general surgeon would be
transpleural ligation of the fistula combined with placement of a
gastrostomy tube (Level 5 evidenceGrade D recommendation).
Primary repair of the esophageal atresia in this setting is not routinely recommended, particularly in the preterm infant (Level 3b
evidenceGrade D recommendation).
4. What is the best method to obtain length for primary repair
in the setting of short-gap esophageal atresia?
Type C lesions (proximal atresia with distal TEF) are by far the most
common esophageal malformation and rarely require emergent
intervention as described above. Surgery may be delayed for several
days while a full assessment of the infant is completed. Formal repair
involves a right posterolateral thoracotomy employing an extrapleural dissection, exposing the posterior mediastinum. The tracheal fistula should be controlled early in the operation with suture ligation
close to the trachea, carefully avoiding compromise of the tracheal
lumen. The proximal esophagus should be extensively mobilized
into the thoracic inlet, which increases length of the proximal pouch
and allows assessment for a fistula from the proximal esophageal
pouch to the trachea. After patency is assured, the distal esophagus
is mobilized in a limited manner sufficient to facilitate the anastomosis, yet avoiding disruption of the tenuous blood supply. Most often
the proximal and distal ends may be mobilized to allow for primary
repair under mild to moderate tension. At times, additional length is
required to minimize tension on the esophageal anastomosis.
Numerous techniques have been described when it is anticipated that extensive mobilization will not yield sufficient length to
allow for primary repair.51-64 Most of the reported procedures are of
historical significance only. Preservation of the native esophagus is
always the best option in the setting of short-gap atresia and efforts
should be made to perform a primary repair even in the setting of
moderate tension.65 This notion is best supported when comparing
rates of stricture and anastomotic leak in patients with short- and

PMPH_CH12.indd 99

99

long-gap atresia who had primary repair without additional lengthening procedures. Bagolan et al. report 71 consecutive cases repaired
primarily without the need for esophageal replacement with no significant difference in regards to leak rate, stenosis, fistula formation,
and postoperative reflux for long- versus short-gap atresia.66
Answer: The best method to allow primary repair in shortgap esophageal atresia remains extensive mobilization with
primary closure under tension. (Level 3b evidenceGrade C
recommendation)
Treatment of long-gap, Type A, esophageal atresia presents a
major challenge for the general and pediatric surgeon. Typically,
this is defined as a gap of 2 or more centimeters or, two or more vertebral bodies. Historically, attempts at primary repair when the gap
exceeds 3 cm are associated with more frequent complications.67-69
5. What is the best timing for surgical repair of long-gap
atresia?
As techniques to deal with this formidable problem have evolved,
controversy still surrounds the timing of intervention and the
selection of an esophageal substitute. The timing of the surgical
intervention has been debated; many experts suggest that immediate repair in the first 72 hours after birth can be accomplished,66
whereas others suggest that a better outcome can be gained by
delaying repair for weeks to months.70 It is widely believed that
in long-gap atresia, the lower esophagus and stomach are hypoplastic, and delayed repair is favored when large bolus gastrostomy tube feeds can be used to create reflux and distention of the
blind-ending lower pouch allowing for spontaneous growth.66,71
Some groups are strong proponents of waiting at least 3 months
to repair all cases of long-gap atresia citing greater longitudinal
growth of the esophagus as compared to the thoracic vertebral
column, facilitating anastomosis.65 This longitudinal growth is
hypothesized to evolve secondary to the swallowing reflex in the
upper pouch, and reflux into the lower pouch.72 Obviously, this
requires periodic suctioning of the upper pouch, which may be
accomplished in the outpatient setting.73
Answer: Despite the theoretical advantages of time, there is
no firm data to support delaying repair past the neonatal period
in near-term infants devoid of additional anomalies (Level 3
evidenceGrade D recommendation) Preterm, low-birth-weight
infants clearly benefit from staged or delayed repair of esophageal
atresia (Level 2c evidenceGrade B recommendation).
6. What is the best conduit for creation of a neoesophagus?
The discussion regarding esophageal conduits is broad but often
centers on the type of conduit and best anatomic location for
placement of the neoesophagus. Esophageal conduits are used for
many reasons including esophageal atresia, esophageal injury due
to caustic ingestion, trauma, infectious conditions, and congenital
absence of the stomach. Although it is generally well accepted that a
childs native esophagus should be preserved, this is not often possible, especially when the defect extends multiple vertebral bodies
in length. There are many described methods for creating a neoesophagus, including the use of stomach, colon, or small intestine.
Preoperative considerations are paramount when anticipating that
a child may require an esophageal conduit. Many of these infants
have other associated anomalies and careful attention to detail
increases a surgeons options at the time of esophageal reconstruction. For example, a child that initially required a gastrostomy

5/21/2012 8:50:05 PM

100

Surgery: Evidence-Based Practice

Table 12.1 Acute Morbidity and Mortality from Case Series Employing Various Conduits for the Treatment of
Esophageal Atresia
Year

Author

Conduit

Patients (n)

Mortality (n)

Stricture n (%)

Anastomotic
Leak n (%)

2009

Spitz

Gastric transposition

192

34 (19.6)

21 (12)

2002

Hirschl

Gastric transposition

41

20 (49)

15 (36)

1998

Ein

Gastric tube

11

8 (72)

9 (81)

1985

Goon

Gastric tube

46

27 (59)

35 (76)

2003

Hamza

Colonic interposition

475

25 (5)

47 (10)

2000

Erdogan

Colonic interposition

18

3 (17)

11 (61)

1993

Cusick

Jejunal interposition

2 (33)

1 (17)

1988

Saeki

Jejunal interposition

19

2 (10)

3 (15)

1994

Bax

Jejunal free graft

1 (25)

tube must have the location carefully considered should a gastric


conduit be considered for reconstruction. Likewise, a colostomy
created for an associated anorectal malformation can damage the
blood supply and limit options for a future colonic transposition
if great care is not exercised. Obviously, any congenital anatomic
abnormality may limit the availability of certain conduits.
The technical aspects regarding harvesting and placement
of the neoesophageal conduits are beyond the scope of this discussion. The inherent advantages and disadvantages are well
documented elsewhere.75 A comparison of acute morbidity and
mortality across varied case series yields no discernable difference
in complications (Table 12.1).
Answer: There is no data to support the use of any specific
conduit for the creation of a neoesophagus. Initial rates of stricture and leak are similar as is the incidence of late dysphagia. Gastric conduits appear to have fewer late complications that require
operative correction (Level 3 evidence). The best conduit is dictated
by the comorbidity and anatomic limitations of the patient along
with the clinical experience of the operating surgeon (Grade D
recommendation).

POSTOPERATIVE MANAGEMENT
The most common acute complications following repair of EA/
TEF are anastomotic leak and stricture. Long-term complications
are related to gastroesophageal reflux and esophageal dysmotility. Anastomotic leak following repair of EA occurs in 15% of all
cases. Most leaks can be managed with drainage and nutritional
support. The rate of spontaneous resolution of an anastomotic leak
approaches 95% when a retropleural dissection plane was used at
the time of the original repair.75 In the minority of cases (35%),
the leaks are uncontrolled with drainage and antibiotics alone,
leading to clinical deterioration and sepsis. These major leaks are
often identified in the initial 48 h after surgery and reoperation
is critical to control drainage and attempt repair. In this setting,
the repair should be buttressed with a pleural or pericardial flap
to assist in vascularization and healing. Most surgeons advocate
7 days without feeding following index repair of the esophagus.
An upper gastrointestinal swallow study to evaluate for leak is
performed on day 7. A leak identified by this study, in the absence
of clinical deterioration, can be managed conservatively. The

PMPH_CH12.indd 100

consensus opinion advocates a repeat study the following week to


confirm closure prior to initiating oral feeding.
Esophageal stricture is a common complication following
repair of EA. Quoted incidence ranges from 30% to 80% of all
patients. Many factors have been implicated in stricture formation
including ischemia, tension, anastomotic leak, and poor operative
technique. Once a clinically symptomatic stricture is diagnosed
on esophagram, bougienage is typically performed over a wire.
Some studies quote 90% success with operative.
7. Do all children with EA/TEF require antireflux therapy with
medication and/or surgical procedures?
With improved survival and a marked decrease in acute complications following repair of esophageal atresia, late complications
including tracheomalacia, stricture, and reflux have required
increased attention.78 In children who have undergone repair of
esophageal atresia, 40% to 50% suffer from clinically relevant reflux
with an even greater percentage noted in patients with a history
of long-gap atresia.79,80 Many patients develop symptoms months
to years after the original repair.81 Greater than of 30% of patients
have severe reflux, refractory to acid suppression therapy, and
require surgery to relieve symptoms.81,82 These findings result from
inherent esophageal dysmotility and anatomic distortion where
the cardia anchors to the diaphragmatic crura and pleuroperitoneal membrane. Manometric study reveals that it is the characteristic dysmotility of the disrupted esophagus and not nerve injury
that contributes to the development of advanced reflux.83 Despite
structural and functional abnormalities, half of all neonates will
not manifest clinically significant reflux following repair.
Short- and long-term morbidity from reflux can be very
significant in this patient population and most surgeons would
provide empiric, medical treatment for reflux following repair of
esophageal atresia.84 Absolute indications for operative intervention include recurrent stricture following dilation, severe dysmotility, presence of a paraesophageal or sliding hiatal hernia, or
endoscopic findings of severe esophagitis. As children grow and
develop clinical symptoms of reflux, they should be evaluated and
managed similar to any other child. When an antireflux procedure is warranted, authors are divided, advocating either a Nissen
fundoplication or Thal procedure in patients with previous repair
of esophageal atresia. Those proponents of the Thal procedure feel

5/21/2012 8:50:05 PM

Esophageal Atresia and Tracheoesophageal Fistula

a Nissen may exacerbate the inherent esophageal dysmotility and


increase the risk of dysphagia and aspiration.
Operative treatment for reflux in patients with a history of
esophageal atresia should take into account the potential for significant scarring at the hiatus secondary to the previous dissection and
ongoing esophagitis. Patients with a history of repair for esophageal
atresia have an increase in wound complications and postoperative
pneumonia following an antireflux operation, as compared to those
without previous surgery for atresia.85 Recurrent or persistent reflux
following antireflux surgery approaches 15% for patients with previous repair of esophageal atresia compared to 6.5% for all other

101

patients.86 In an attempt to limit recurrent symptoms in this group


of patients, some authors advocate long-term use of proton pump
inhibitors for recurrent reflux following antireflux surgery. As with
any procedure for reflux, the best method to prevent recurrence is
proper surgical technique at the index operation.87
Answer: All patients should be treated with acid suppression
therapy following repair of esophageal atresia until it is clear there
are no symptoms of stricture or pathologic reflux. Recurrent stricture following dilation and sudden death spells remain absolute
indications for antireflux surgery (Level 3 evidenceGrade C
recommendation).

Clinical Question Summary


Level of
Evidence

Grade of
Recommendation

Methimazole is the only known causative


factor to have a reproducible
association with tracheoesophageal
anomalies.

2c

N/A

2 What is the safest and


most accurate method to
confirm the diagnosis of
esophageal atresia with
tracheoesophageal fistula?

Bronchoscopy/esophagoscopy is
considered the best method for safe,
accurate diagnosis of common lesions.

3b

42, 43

3 What is the best method to


stabilize the patient with a
high-output fistula?

The most reliable method is transpleural


ligation of the fistula combined with
gastrostomy tube.

44-50

4 What is the best method to


obtain length for primary
repair in the setting of
short-gap esophageal
atresia?

The best method for primary repair


in short-gap esophageal atresia is
extensive mobilization with primary
closure under tension.

3b

65, 66

5 What is the best timing for


surgical repair of long-gap
atresia?

There is no data to support delaying


repair past the neonatal period in
near-term infants devoid of additional
anomalies. / Preterm, low-birth-weight
infants clearly benefit from staged or
delayed repair of esophageal atresia.

3/2c

D/B

66, 70

6 What is the best conduit


for creation of a
neoesophagus?

The best conduit is dictated by the


comorbidity and anatomic limitations
of the patient along with the clinical
experience of the operating surgeon.

71, 72, 74, 75

7 Do all children with


esophageal atresia and
tracheoesophageal fistula
require antireflux therapy
with medication and/or
surgical procedures?

All patients should be treated with acid


suppression therapy following repair of
esophageal atresia. Recurrent stricture
following dilation and sudden death
spells are absolute indications for
antireflux surgery.

79-82, 86, 87

Question

Answer

1 What are environmental


and/or genetic risk
factors that predispose
to the development of
esophageal atresia and
tracheoesophageal fistula?

REFERENCES
1. Durston W. A narrative of a monstrous birth in Plymouth. PhilTrans R Soc. 1670.
2. Haight C, Townsley H. Congenital atresia of the esophagus with
tracheoesophageal fistula: extrapleural ligation of fistula and

PMPH_CH12.indd 101

References
30

end-to-end anastomosis of esophageal segments. Surg Gynecol


Obstet. 1943;76:672.
3. Chittmittrapap S, Spitz L, Kiely EM, et al. Oesophageal atresia and
associated anomalies. Arch Dis Child. 1989;64:364-368.
4. Spitz L. Esophageal atresia: past, present, and future. J Pediatr
Surg. 1996;31:19-25.

5/21/2012 8:50:05 PM

102

Surgery: Evidence-Based Practice

5. Brown AK, Roddam AW, Spitz L, et al. Oesophageal atresia,


related malformations, and medical problems: a family study.
Am J Med Genet. 1999;85:31-37.
6. Deurloo JA, Ekkelkamp S, Schoorl M, et al. Esophageal atresia:
historical evolution of management and results in 371 patients.
Ann Thorac Surg. 2002;73:267.
7. Driver CP, Shankar KR, Jones MO, et al. Phenotypic presentation and outcome of esophageal atresia in the era of the Spitz
Classification. J Pediatr Surg. 2001;36:1419.
8. Leonard H, Barrett AM, Scott JE, et al. The influence of congenital heart disease on survival of infants with oesophageal atresia.
Arch Dis Child Fetal Neonat Ed. 2001;85:F204.
9. de Jong EM, Felix JF, Deurloo JA, et al. Non-VACTERL-type
anomalies are frequent in patients with esophageal atresia/
tracheo-esophageal fistula and full or partial VACTERL association. Birth Defects Res. (Part A) 2008;82:92-97.
10. Herman TE, Seigel MJ. VACTERL-H syndrome. J Perinatol.
2002;22(6):496-498.
11. Depaepe A, Dolk H, Lechat MF. The epidemiology of tracheooesophageal fistula and oesophageal atresia in Europe. EUROCAT Working Group. Arch Dis Child. 1993;68:743-748.
12. Walsh LE, Vance GH, Weaver DD. Distal 13q Deletion Syndrome
and the VACTERL association: case report, literature review, and
possible implications. Am J Med Genet. 2001;98:137-144.
13. Marsh AJ, Wellesley D, Burge D, et al. Interstitial deletion of
chromosome 17(del(17)(q22q23.3)) confirms a link with oesophageal atresia. J Med Genet. 2000;37:701-704.
14. Digilio MC, Marino B, Bagolan P, et al. Microdeletion 22q11 and
oesophageal atresia. J Med Genet. 1999;36:137-139.
15. Bowen AD, Parry WH. Bronchopulmonary-foregut malformation in the Goldenhar anomalad. AJR. 1980;134:186-188.
16. Vissers LE, van Ravenswaaij CM, Admiraal R, et al. Mutations in a new member of the chromodomain gene family cause
CHARGE syndrome. Nat Genet. 2004;36:955-957.
17. Johnston JJ, Olivos-Glander I, Killoran C, et al. Molecular and
clinical analyses of Greig cephalopolysyndactyly and PallisterHall syndromes: robust phenotype prediction from the type and
position of GLI3 mutations. Am J Hum Genet. 2005;76:609-622.
18. Williamson KA, Hever AM, Rainger J, et al. Mutations in SOX2
cause anophthalmia-esophageal-genital (AEG) syndrome. Hum
Mol Genet. 2006;15:1413-1422.
19. Van Bokhoven H, Celli J, van Reeuwijk J, et al. MYCN haploinsufficiency is associated with reduced brain size and intestinal
atresias in Feingold syndrome. Nat Genet. 2005;37:465-467.
20. Cox PM, Gibson RA, Morgan N, et al. VACTERL with hydrocephalus in twins due to Fanconi anemia (FA): mutation in the
FAC gene. Am J Med Genet. 1997;68:86-90.
21. Czeizel AE. Schisis association. Am J Med Genet. 1981;10:25-35.
22. Ten Have-Opbroek AAW. Lung development in the mouse
embryo. Exp Lung Res. 1991;17:111-130.
23. Sasaki T, Kusafuka T, Okada A. Analysis of the development of
normal foregut and tracheoesophageal fistula in an adriamycin
rat model using three-dimensional image reconstruction. Surg
Today. 2001;31:133-139.
24. Arey LB. Developmental Anatomy. 7th ed. Philadelphia, PA:
Saunders; 1966:263-265.
25. Gray SW, Scandalakis JE. Embryology for Surgeons. Philadelphia,
PA: Saunders; 1972:63-100.
26. Zaw-Tun HA. The tracheo-esophageal septum fact or fantasy?
Origin and development of the respiratory primordium and
esophagus. Acta Anat. 1982;114:1-21.
27. ORahilly R, Muller F. Chevalier Jackson lecture. Respiratory and
alimentary relations in staged human embryos. New embryological

PMPH_CH12.indd 102

28.

29.

30.

31.

32.

33.

34.

35.

36.

37.
38.

39.

40.

41.

42.
43.

44.

45.

46.

data and congenital anomalies. Ann Otol Rhinol Laryngol.


1984;93:421-429.
Qi BQ, Beasley SW. Stages of normal tracheo-bronchial development in rat embryos: resolution of a controversy. Dev Growth
Diff. 2000;42:145-153.
Sakiyama J, Yamagishi A, Kuroiwa A. Tbx4-Fgf10 system controls lung bud formation during chicken embryonic development. Development. 2003;130:1225-1234.
Thompson DJ, Molello JA, Strebing RJ, et al. Teratogenicity of
adriamycin and daunomycin in the rat and rabbit. Teratology.
1978;17:151-157.
Ioannides AS, Chaudhry B, Henderson DJ, et al. Dorso-ventral
patterning in oesophageal atresia with tracheo-oesophageal
fistula: evidence from a new mouse model. J Pediatr Surg.
2002;37:185-191.
Dawrant MJ, Giles S, Bannigan J, et al. Adriamycin mouse
model: a variable but reproducible model of tracheo-oesophageal
malformations. Pediatr Surg Int. 2007;23:469-472.
Nora JJ, Nora AH, Perinchief AG, et al. Congenital abnormalities
and first-trimester exposure to progestagen/oestrogen [letter].
Lancet. 1976;1:313-314.
Wong-Gibbons DL, Romitti PA, Sun L, et al. Maternal periconceptional exposure to cigarette smoking and alcohol and esophageal
atresia +/tracheo-esophageal fistula. Birth Defects Res A Clin
Mol Teratol. 2008;82:776-784.
Kyyronen P, Hemminki K. Gastro-intestinal atresias in Finland
in 197079, indicating time-place clustering. J Epidemiol Community Health. 1988;42:257-265.
Felix JF, Steegers-Theunissen RP, de Walle HE, et al. Esophageal atresia and tracheoesophageal fistula in children of women
exposed to diethylstilbestrol in utero. Am J Obstet Gynecol.
2007;197:38 e31-e35.
Loane M, Dolk H, Morris JK. Maternal age-specific risk of nonchromosomal anomalies. Bjog. 2009;116:1111-1119.
Wilson JG, Roth CB, Warkany J. An analysis of the syndrome
of malformations induced by maternal vitamin A deficiency.
Effects of restoration of vitamin A at various times during gestation. Am J Anat. 1953;92:189-217.
Holden ST, Cox JJ, Kesterton I, et al. Fanconi anaemia complementation group B presenting as X linked VACTERL with
hydrocephalus syndrome. J Med Genet. 2006;43:750-754.
Sparey C, Jawaheer G, Barrett AM, et al. Esophageal atresia in the
northern region Congenital Anomaly Survey, 1985-1997: prenatal diagnosis and outcome. Am J Obstet Gynecol. 2000;182:427.
McDuffie, LA, Wakeman D, Warner, B. Diagnosis of esophageal
atresia with tracheoesophageal fistula: Is there a need for gastrointestinal contrast? J Pediatr. 2010;156:852.
Benjamin B, Pham T. Diagnosis of H-type tracheoesophageal fistula. J Pediatr Surg. 1991;26(6):667-671.
Brookes JT, Smith MC, Smith RJ, et al. H-Type tracheoesophageal fistula: University of Iowa experience 1985 to 2005. Ann Otol
Rhinol Laryngol. 2007;116(5):363-368.
Jones TB, Kirchner SG, Lee FA, et al. Stomach rupture associated
with esophageal atresia, tracheoesophageal fistula and ventilatory assistance. AJR. 1980;134:675-677.
Filston HC, Chitwood WR Jr, Schkolne B, et al. The Fogarty balloon catheter as an aid to management of the infant with esophageal atresia and tracheoesophageal fistula complicated by severe
RDS or pneumonia. J Pediatr Surg. 1982;17:149-151.
Donn SM, Zak LK, Bozynski ME, et al. Use of high-frequency jet
ventilation in the management of congenital tracheoesophageal
fistula associated with respiratory distress syndrome. J Pediatr
Surg. 1990;25(12):1219-1221.

5/21/2012 8:50:05 PM

Esophageal Atresia and Tracheoesophageal Fistula

47. Fann JI, Hartman GE, Shochat SJ. Waterseal gastrostomy in the
management of premature infants with tracheoesophageal fistula and pulmonary insufficiency. J Pediatr Surg. 1988;23:29-31.
48. . andolph JG, Tunell WP, Lilly JR. Gastric division in the critically
ill infant with esophageal atresia and tracheoesophageal fistula.
Surgery. 1968;63:496.
49. Leininger BJ. Silastic banding of esophagus with subsequent
repair of esophageal atresia and tracheoesophageal fistula.
J Pediatr Surg. 1972;7:404.
50. Templeton JM, Templeton JJ, Schnaufer L, et al. Management
of esophageal atresia and tracheoesophageal fistula in the neonate with severe respiratory distress syndrome. J Pediatr Surg.
1985;20:394-397.
51. Howard R, Myers NA. Esophageal atresia: a technique for elongating the upper pouch. Surgery. 1965;58:725-727.
52. Hays DM, Woolley MM, Snyder WH Jr. Changing techniques
in the management of esophageal atresia. Arch Surg. 1966;92(4):
611-616.
53. Hendren WH, Hale JR. Esophageal atresia treated by electromagnetic bougienage and subsequent repair. J Pediatr Surg.
1976;11:713-722.
54. Puri P, Blake N, ODonnell B, et al. Delayed primary anastomosis following spontaneous growth of esophageal segments in
esophageal atresia. J Pediatr Surg. 1981;16:180-183.
55. Rehbein F, Schweder N. Reconstruction of the esophagus without colon transplantation in cases of atresia. J Pediatr Surg.
1971;6(6):746-752.
56. Shafer AD, David TE. Suture fistula as a means of connecting
upper and lower segments in esophageal atresia. J Pediatr Surg.
1974;9:669-673.
57. Schullinger JN, Vinocur CD, Santulli TV. The suture fistula
technique in the repair of selected cases of esophageal atresia.
J Pediatr Surg. 1982;17:234-236.
58. Lavaditis A. End-to-end anastomosis in esophageal atresia. A
clinical and experimental study. Scand J Thorac Cardiovasc Surg
Suppl. 1969;2:7-20.
59. Vizas D, Ein SH, Simpson JS. The value of circular myotomy for
esophageal atresia. J Pediatr Surg. 1978;13(4):357-359.
60. Gough MH. Esophageal atresia-use of an anterior flap in the difficult anastomosis. J Pediatr Surg. 1980;15(3):310-311.
61. Bar-Maor JA, Shoshany G, Sweed Y. Wide gap esophageal atresia: a new method to elongate the upper pouch. J Pediatr Surg.
1989;24(9):882-883.
62. Davenport M, Bianchi A. Early experience with oesophageal flap
oesophagoplasty for repair of oesophageal atresia. Pediatr Surg
Int. 1990;5:332-335.
63. Foker JE, Linden BC, Boyle EM, et al. Development of a true primary repair for the full spectrum of oesophageal atresia. Ann
Surg. 1997;226:533-543.
64. Kimura K, Nishijima E, Tsugawa C, et al. Multistaged extrathoracic oesophageal elongation procedure for long gap oesophageal atresia: Experience with 12 patients. J Pediatr Surg. 2001;36:
1725-1727.
65. Myers NA. Oesophageal atresia: the epitome of modern surgery.
Ann R Coll Surg Engl. 1974;54:277-287.
66. Bagolan P, Iacobelli BD, De Angelis P, et al. Long gap esophageal
atresia and esophageal replacement: moving toward a separation? J Pediatr Surg. 2004;7:1084-1090.
67. Ein Sh, Shandling B, Heiss K. Pure esophageal atresia: outlook in
the 1990s. J Pediatr Surg. 1993;28:1147-1150.
68. Brown AK, Tam PKH. Measurement of gap length in oesophageal atresia: a simple predictor of outcome. J Am Coll Surg.
1996;182:41-45.

PMPH_CH12.indd 103

103

69. Rescorla FR, West KW, Scherer LR III, et al. The complex nature of
type A (long-gap) oesophageal atresia. Surgery. 1994;116:658-664.
70. Puri P, Khurana S. Delayed primary esophageal anastomosis for
pure esophageal atresia. Semin Pediatr Surg. 1998;7:126-129.
71. Myers NA, Beasly SW, Auldist AW, et al. Oesophageal atresia
without fistula-anastomosis or replacement. Pediatr Surg Int.
1987;2:216-222.
72. Puri P, Ninan GK, Blake NS, et al. Delayed primary anastomosis
for esophageal atresia: 18 months to 11 years follow-up. J Pediatr
Surg. 1992;27:1127-1130.
73. Aziz D, Schiller D, Gerstle JT, et al. Can long-gap esophageal
atresia be safely managed at home while awaiting anastomosis?
J Pediatr Surg. 2003;38:705-708.
74. Spitz L., Hitchcock R. Oesophageal atresia and trachea-esophageal
fistula. In: Om Freeman, NV et al., eds. Surgery of the Newborn.
New York: Churchill Livingstone; 1994.
75. Manning PB, Morgan RA, Coran AG, et al. Fift y years experience with esophageal atresia and tracheoesophageal fistula.
Beginning with Cameron Haights first operation in 1935. Ann
Surg. 1986;204:446.
76. Serhal L, Gottrand F, Sfeir R, et al. Anastomotic stricture after
surgical repair of esophageal atresia: frequency, risk factors,
and efficacy of esophageal bougie dilatations. J Pediatr Surg.
2010;45:1459-1462.
77. Said M, Mekki M, Golli M, et al. Balloon dilation of anastomotic
strictures secondary to surgical repair of oesophageal atreasia.
Br J Radiol. 2003;76:26.
78. Okada A, Usui N, Inoue M, et al. Esophageal atresia in Osaka: a
review of 39 years experience. J Pediatr Surg. 1997;32:1570-1574.
79. Guo W, Fonkalsrud E, Swaniker F, et al. Relationship of esophageal anastomotic tension to the development of gastroesophageal
reflux. J Pediatr Surg. 1997;32:1337-1340.
80. Jolley SG, Johnson DG, Roberts CC, et al. Patterns of gastroesophageal reflux in children following repair of esophageal
atresia and distal tracheoesophageal fistula. J Pediatr Surg.
1980;15(6):857-862.
81. Koivusalo A, Pakarinen MP, Rintala RJ. The cumulative incidence of significant gastrooesophageal reflux in patients with
oesophageal atresia with a distal fistulaa systematic clinical, pH-metric, and endoscopic follow-up study. J Pediatr Surg.
2007;42:370-374.
82. Little DC, Rescorla FJ, Grosfeld JL, et al. Long-term analysis of
children with esophageal atresia and tracheoesophageal fistula.
J Pediatr Surg. 2003;38:852-856.
83. Kawahara H, Kubota A, Hasegawa T, et al. Lack of distal esophageal contractions is a key determinant of gastroesophageal
reflux disease after repair of esophageal atresia. J Pediatr Surg.
2007;42:2017-2021.
84. Burjonrappa SC, Youssef S, St-Vil D. What is the incidence of
Barretts and gastric metaplasia in esophageal atresia/tracheoesophageal fistula (EA/TEF) patients? Eur J Pediatr Surg.
2011;21(1):25-29.
85. Holschneider P, Dubbers M, Engelskirchen, et al. Results of the
operative treatment of gastroesophageal reflux in childhood
with particular focus on patients with esophageal atresia. Eur J
Pediatr Surg. 2007;17(3):16.
86. Holschneider P, Dubbers M, Engelskirchen R, et al. Results of
the operative treatment of gastroesophageal reflux in childhood
with particular focus on patients with esophageal atresia. Eur J
Pediatr Surg. 2007;17:163-175.
87. Pashankar D, Blair GK, Israel DM. Omeprazole maintenance
therapy for gastroesophageal reflux disease after failure of fundoplication. J Pediatr Gastroenterol Nutr. 2001;32:145-149.

5/21/2012 8:50:05 PM

Commentary on
Esophageal Atresia and
Tracheoesophageal Fistula
Michael Hirsch

The chapter entitled Esophageal Atresia and Tracheoesophageal


Fistula by Miller C. Hamrick, David E. Carney, and William C.
Boswell does an excellent job of summarizing the surgical history
of the approach to this difficult congenital anomaly. Prior to Cameron Haights successful repair of a tracheoesophageal fistula (TEF)
in a newborn in 1939, this uniformly lethal anomaly seemed destined to be a postmortem finding only. Over the ensuing 70 years,
huge strides in the perinatal, neonatal, and pediatric surgical care
of these infants have led to a survival of approximately 95%. The
chapter addresses seven major questions about the pathophysiology and surgical management of infants with esophageal atresia/
tracheoesophageal fistula (EA/TEF). I will provide commentary
on all seven questions but in general must state at the outset that
the difficulty in pediatric surgical evidence-based treatises is
that the relatively infrequent nature of any one anomaly deprives
authors of the critical numbers needed to give the recommendations/conclusions that offer the necessary weight. Nonetheless,
multiple small studies from reputable pediatric surgical centers
unfortunately have to substitute for randomized, prospective,
double-blinded studies that other medical conundrums organize
to promote consensus for best practice guidelines.
Question 1 addressed the environmental/genetic factors
associated with TEF. Adriamycin has been the only drug identified in an animal model to cause similar birth anomalies in
rats. The RNA/DNA damage that this drug causes has led to the
conclusion that there is likely chromosomal damage that leads to
this anomaly. The fact that there is an overall 50% to 70% associated anomaly rate would imply that in utero organogenesis is
commonly adversely affected in multiple systems when EA/TEF
babies are born. It should be noted that the table showing the five
common varieties of TEF has been changed in many textbooks
to provide new classifications based on frequency rather that the
older labels used by the authors.Thus, Type C that is seen in 86% of
TEF cases is now called Type 1, Type A seen in 7% of the anomalies is called Type 2, Type E seen in 5% of the cases is called Type
3, with Type B (approx. 1%) being Type 4 and Type D (<1%) being
Type 5 in the new nomenclature.
Question 2 tried to address the question of how best to classify the TEF/EA anomaly facing the pediatric surgeon. Proximal
fistulae (Type B and D) and H-type fistula are notoriously difficult to diagnose. Though we would agree with the authors that
bronchoscopy is the most likely test to ascertain with certainty
the presence of these fistulae, pediatric surgeons have long argued

that the type of mobilization of the proximal pouch that needs


to be done to achieve tension-free anastamosis will reveal the
presence of the proximal fistulae in the overwhelming majority
of cases. Only in the H-type (E) fistulae cases is bronchoscopy
imperative; it can also be used to cannulate the fistula with small
tubing that can greatly facilitate the intraoperative repair.
Question 3: The high output fistula described in section
is fortunately a relatively rare occurrence. More commonly, the
problem with instability in a pre-repaired TEF patient is the gaseous dilatation of the GI tract that cannot be decompressed. This
rarely becomes so critical that ligation of the TEF is necessary
as a stabilizing technique, but ligation is clearly the most definitive management modality. The other less invasive temporizing
methods described by the authors, including tube placement with
water seal drainage and balloon occlusion of the fistula either retrograde through the gastrostomy or antegrade next to the endotracheal tube, are more commonly used.
Question 4 suggests that OR mobilization of the two esophageal segments and intraoperative repair of the short gap TEF/EA
anomaly is usually sufficient to solve the problem. The authors
are correct in emphasizing that most of this mobilization must
be derived from the proximal segment of esophagus, as the distal segments segmental blood supply will be compromised if it is
extensively dissected.
Question 5 addresses the thorny dilemma of how to handle
the long-gap EA. The authors conclude that there is no advantage in delaying the repair during the newborn period, but instead
preparing for esophageal replacement if necessary, if the native
esophagus cannot be used. Despite their citations, we believe
there have been many descriptions over the last few years, of techniques to stimulate the linear growth of the native esophageal segments (the Foker technique or serial dilatations) that can promote
delayed primary repair.1 With the very discouraging data pointed
to in the Question 6 discussion of the optimal mode of esophageal
replacement, with high leak, stricture rates in all modalities, we
believe that native esophageal preservation must be a tantamount
goal. In addition, we feel that it is important to add the increasing use of minimally invasive approach to esophageal atresia
repair.2,3
The subject of Question 7 is the need for antireflux surgery
in repaired TEF/EA patients with gastroesophageal reflux disease
(GERD). We agree with the authors conclusions that this surgery
should be reserved for children with near sudden infant death

104

PMPH_CH12.indd 104

5/21/2012 8:50:06 PM

Esophageal Atresia and Tracheoesophageal Fistula

syndrome (SIDS) experiences from reflux or from clear peptic


stricture complications of GERD. It must also be recognized that
the gold-standard antireflux procedure, the Nissen fundoplication,
has been implicated in causing the dysmotile repaired esophagus
or esophageal replacements to develop an achalasia-like emptying
problem. This points to the potential advantage of a less than 360degree circumferential fundoplication such as the Toupet, Thal, or
Boix-Ochoa approaches.
Overall, we congratulate the authors for their thorough yet
concise description of this very challenging anomaly complex and
its surgical management.

PMPH_CH12.indd 105

105

REFERENCES
1. Foker JE, Linden BC, Boyle EM, et al. Development of a true primary repair for the full spectrum of esophageal atresia. Ann Surg.
1997;226(4):533-541; discussion 541-543.
2. Szavay PO, Zundel S, Blumenstock G, Kirschner HJ, Luithle T,
Girisch M, et al. Perioperative outcome of patients with esophageal atresia and tracheo-esophageal fistula undergoing open
versus thoracoscopic surgery. J Laparoendosc Adv Surg Tech A.
3. MacKinlay GA. Esophageal atresia surgery in the 21st century.
Seminars in Pediatric Surgery, 2009;18(1):20-22.

5/21/2012 8:50:06 PM

PMPH_CH12.indd 106

5/21/2012 8:50:06 PM

PART 1I

THE STOMACH

PMPH_CH13.indd 107

5/21/2012 8:50:52 PM

PMPH_CH13.indd 108

5/21/2012 8:50:53 PM

CHAPTER 13

Peptic Ulcer Disease


Wayne H. Schwesinger

They are also more likely to require an emergency operation.12


(Evidence Grade B)

1. How has the surgery of peptic ulcer disease (PUD) changed


over time?
Few diseases in western society have been so dramatically transformed over time as PUD. Although rarely described in the early
medical literature, its incidence reached epidemic proportions by
the mid-1900s then slowly began to decline, a trend that continues
to this day.1,2 From the beginning, operative therapy served as an
important cornerstone in the management of PUD since the available medical measures were often ineffective. Landmark investigations by Beaumont, Pavlov, Dragsted, and Edkins among others
served as the pathophysiologic foundation on which many new
surgical strategies for the management of PUD were developed.3
While these operations appeared to be very effective at controlling
both intractable and complicated PUD, they could be associated
with significant short term and long-term consequences.4
Simultaneously, major progress was also being made with two
different nonoperative approaches to the management of PUD:
pharmacotherapy and flexible endoscopy. Antisecretory drugs,
introduced in 1977, largely replaced both the Sippy diet and antacid
therapy and a decade later they were superseded by the introduction
of the first proton pump inhibitor. Moreover, a specific infectious etiology for PUD was suggested by Warren and Marshall in 1983; within
a decade, more than 1500 scientific articles were being published on
the topic annually.5 Soon, specific antimicrobial drug regimens were
validated as a primary method of treating and curing PUD.
As a result of these pharamacologic advances and evolving
endoscopic techniques, the operative approach to PUD has dramatically changed. Since 1960, multiple investigators have noted a
significant reduction in total surgical volume.6-8
A personal survey of a 20-year experience in two major Texas
teaching hospitals demonstrated an 80% decrease in the overall
number of operations for PUD.9 Currently, the most common
indications for surgery are perforation and bleeding while intractability has become nearly obsolete.
Patient demographics are also continuing to change. Individuals hospitalized with PUD are now older, more frequently
female, and more likely to have major comorbid conditions.10,11

2. What are the major risk factors for PUD?


By the mid-20th century, it was generally agreed that gastric
hyperacidity was the primary cause of PUD. Thus, the oft quoted
aphorism: no acid, no ulcer.13 Early speculations on the specific
pathogenetic factors causing hyperacidity focused on the relative
contributions of stress, smoking, familial predisposition, hormonal changes, aspirin intake, and dietary indiscretions.
A paradigm shift occurred when it was recognized that either
Helicobacter pylori infection or NSAID use could be implicated
in most cases of peptic ulceration albeit through entirely different mechanisms.14-16 With H. pylori infection, a complex interaction occurs between bacterial virulence factors (cagA, cagPAI,
vacA), host factors (interleukins, TNF-, chemokines), and environmental factors (smoking, high salt intake).17,18 The result is a
persistent chronic gastritis. The location of the infection in the
stomach helps to determine the clinical course. Antral-dominant
infections result in reduced somatostatin levels, hypergastrinemia
and gastric acid hypersecretion and can produce duodenal ulcers.
Gastric body-dominant infections are associated with mucosal
atrophy and hypochlorhydria; this pattern may result in either
benign or malignant gastric ulcers.
In contrast, the gastric mucosal injury caused by NSAIDs
(including aspirin) results mostly from inhibition of the constitutive enzyme cyclo-oxygenase-1 (COX-1) a major product of
arachidonic acid metabolism in the gastric mucosa. COX-1 is
responsible for the release of prostacyclin a potent cytoprotectant.
When release is inhibited, gastric acid and other irritants can
more easily damage the gastric mucosa. Another isoform, COX-2,
is induced by inflammatory stimuli and has significant antiinflammatory activity, but fewer gastric side effects.19
Importantly, a synergism can develop between the two major
risk factors. In a meta-analysis of 25 related studies, the presence
of H. pylori infection was found to increase the risk of PUD in
NSAID users 3.5-fold compared with non-infected patients.20
109

PMPH_CH13.indd 109

5/21/2012 8:50:53 PM

110

Surgery: Evidence-Based Practice

The special category of idiopathic PUD (non-NSAID and


non-H. pylori) can be accounted for by severely altered gastric
physiology, inaccurate H. pylori testing, or covert NSAID use.
Overall, non-NSAID and non-H. pylori ulcers tend to occur in
older and sicker patients and are associated with a higher recurrence rate.21 Rare causes of ulceration include ZollingerEllison
syndrome, G-cell hyperplasia, Crohns disease, cocaine abuse,
and systemic mastocytosis.22 (Evidence Grade A)
3. What is the appropriate therapy for H. pylori positive PUD?
In patients with PUD who are H. pylori positive, eradication of the
organism is crucial. In a Cochrane analysis of over 3900 patients in
34 trials, the ulcer healing rate after therapy was 75% to 85% and
the recurrence rate was 12% to 14%.23 First-line therapy as recommended in the Maastricht Consensus Report 2-2000 combines a
PPI with clarithromycin and amoxicillin or metronidazole twice
daily for 1 to 2 weeks.24 Eradication rates with initial treatment of
75% to 90% have been reported, but are declining in most countries
because of antibiotic resistance.25 The Helicobacter Antimicrobial
Resistance Monitoring Program (HARP) studied 347 clinical isolates and found that the highest rate of resistant strains occurred
with metronidazole (25.1%) and clarithromycin (12.9%), while
amoxicillin resistance was uncommon (0.9%).26
Second-line rescue treatment with quadruple therapy typically combines a PPI with bismuth, metronidazole, and tetracycline. Eradication should always be confirmed with either a urea
breath test or a stool antigen. Alternative therapeutic approaches
are desperately needed. Studies are ongoing with new antibiotics
(e.g., levofloxacin) and with a sequential regimen involving a PPI
plus amoxicillin for 5 days followed by a PPI plus clarithromycin
and tinidazole for another 5 days.27 (Evidence Grade A)
4. How can the risk of PUD be minimized in patients requiring
regular NSAID therapy?
Several options are available for reducing the risk of peptic ulcers
in patients receiving NSAIDs. In NSAID-nave patients who are
infected with H. pylori, eradication of the organism before beginning NSAID therapy is associated with a significant reduction in
the risk of ulcer.28 Even in patients taking low-dose aspirin (80 mg),
eradication of H. pylori can provide significant protection from
PUD and its complications. Proton pump inhibitors, the prostaglandin analogue Misoprostol, and high doses of histamine-2-receptor
antagonists are also effective in reducing the prevalence of NSAIDinduced ulcers.29,30 Misoprostol usage is limited, however, by its frequent association with dose-related abdominal pain and diarrhea.
(Evidence Grade A)
5. In a patient suspected of having a bleeding peptic ulcer, what
should the initial approach be?
Optimal management depends on a timely and accurate diagnosis and an adequate resuscitation, processes which should proceed
concurrently. The initial clinical approach should be determined,
in large part, by the patients presentation. A history of red blood
or dark coffee ground emesis nearly always indicates an upper
gastrointestinal (UGI) source. The rectal passage of black, digested
blood (melanic stool) is also indicative of a lesion proximal to the
ligament of Trietz.31 The passage of bright red blood per rectum
usually suggests a primary colorectal source, but can also occur in
patients with upper tract lesions if the rate of bleeding is torrential
and the transit time is brief.

PMPH_CH13.indd 110

A rapid assessment of the patients hemodynamic and physical status should guide early therapy and appropriate triage. The
presence of hemorrhagic shock, either compensated or decompensated, mandates aggressive fluid management with blood products
and/or crystalloid. In addition, preexisting comorbities such as
cardiac disease or hepatic or renal dysfunction must be addressed
since most deaths are related to nonbleeding causes.32 Specific
coagulation abnormalities must also be rapidly corrected.
End-points for resuscitation include (1) normalization of
blood pressure, (2) restoration of hemoglobin concentration, (3)
correction of coagulopathies, and (4) correction of end-organ
dysfunction.33 Such an aggressive and multifaceted approach is
supported in the controlled study of Baradarian et al., in which
intensive monitoring and early hemodynamic stabilization were
provided by a specialized resuscitation group.34 This resulted in a
significant reduction in the associated mortality when compared
with routine floor management.
During resuscitation, nasogastric tube placement is used
to sample the contents of the stomach. In patients who present
with a history of hematemesis, nasogastric aspiration of fresh, red
blood indicates the presence of ongoing bleeding and is an independent predictor of poor clinical outcome when compared with
aspiration of either clear or coffee ground material.35 In bleeding patients who present with melena but without hematemesis, a
bloody nasogastric aspirate provides strong evidence for a lesion
in the UGI tract.36 The finding of a negative aspirate is less reliable
as it may fail to detect duodenal lesions.
Nasogastric tubes can also used for gastric lavage prior to
endoscopy, but success is limited by the size of the tube and the presence of clots. Alternatively, prokinetic agents are able to effectively
clear the stomach. Several randomized, controlled clinical trials
have documented that a preendoscopic bolus or infusion of erythromycin improves the quality of the subsequent endoscopic examination and reduces the need for repeat endoscopic procedures.37,38
This approach appears to be cost-effective.39 (Evidence Grade A)
6. In patients with bleeding ulcers, what is the current role of
endoscopy?
Endoscopy is the definitive diagnostic study for UGI bleeding. In
a multicenter study of 11,160 patients with nonvariceal UGI bleeding, endoscopy identified the bleeding site in 83% of cases. The
most common finding was peptic ulcers (32%) with gastric ulcers
more common than duodenal ulcers (54% vs. 37%).40
Risk stratification is another important function of endoscopy
as it guides early management.41 Patients with a clean-based ulcer
or a nonprotruding pigmented spot are at low risk of rebleeding
and generally require no further endoscopic interventions. Conversely, high-risk stigmata such as an actively bleeding ulcer or
an ulcer with a visible vessel forecast a poor outcome and indicate the need for aggressive endoscopic therapy. Another subset of
patients who have an ulcer with an adherent clot also appears to
be at high risk for rebleeding and benefits from removal of the clot
and directed endoscopic therapy.42
The optimal timing for endoscopy continues to be debated.
It is generally agreed that patients who are actively bleeding or
who are unstable require urgent endoscopy to prevent further
deterioration. In the remaining patients, the recommended timing ranges from urgent to elective. Recent data appear to support
the use of early endoscopy to identify low risk patients who will
require only brief or no hospitalization.43 It has been argued that

5/21/2012 8:50:53 PM

Peptic Ulcer Disease

such an approach improves resource utilization. However, two


randomized trials addressing this issue differ in their conclusions
with only one showing significant cost-savings.44,45 Presumably,
this disparity results from differences in practice patterns.
Ulcer bleeding stops spontaneously in the majority of patients
(8085%).46 In the remainder, endoscopic therapy has proven
efficacious and cost-effective.43,47 By 1992, a meta-analysis of 30
randomized clinical trials was reported, in which a variety of endoscopic hemostatic techniques were compared with medical therapy
alone.48 The endoscopically treated group demonstrated significant
reductions in further bleeding (odds ratio (OR), 0.38; 95% confidence interval (CI), 0.320.45), need for surgery (OR, 0.36; 95% CI,
0.280.45), and mortality (OR, 0.55; 95% CI, 0.400.76). Serious
complications directly related to endoscopy were infrequent and
included induced rebleeding (0.4%) and perforation (00.9%).
Numerous studies have been conducted comparing the wide
variety of available hemostatic techniques: injection, thermal, and
mechanical. Overall, initial hemostasis rates range from 85% to
100% for all methods.49 However, injection therapy alone, whether
with dilute epinephrine, thrombin, polidocanol, or cyanoacrylate, is
associated with a higher rebleeding rate and a more frequent need
for surgery than when combined with any other therapy.50 A recent
Cockrane systematic review of 17 randomized studies has confirmed
that dual therapy with epinephrine injection and any other technique reduces the rebleeding rate from 18.8% to 10.4%, emergency
surgeries from 10.8% to 7.1%, and mortality from 5% to 2.5% without
increasing the complication rate.51 Interestingly, evidence is emerging that monotherapy with thermal or mechanical methods may
be as effective as epinephrine-based combination therapy except in
those patients with profound bleeding.52,53 (Evidence Grade A)
7. What is the role of pharmacotherapy in the management of
a bleeding peptic ulcer?
Pharmacotherapy in patients with bleeding PUD significantly
impacts outcome. Proton pump inhibitor (PPI) infusion, when
used as an adjunct to endoscopic therapy, reduces the risk of further bleeding (OR, 0.49; 95% CI, 0.370.65) and decreases the need
for surgery (OR, 0.61; 95% CI, 0.480.78).54 Such therapy should be
initiated as early as possible. In a recent randomized study, preendoscopic PPI was found to facilitate clot formation at the bleeding site and to reduce the need for endoscopic therapy.55 A recent
international consensus recommendation proscribed the use of an
intravenous bolus of PPI followed by a continuous PPI infusion to
reduce rebleeding and mortality in patients with high-risk endoscopic stigmata.56 The optimal dosage of PPI is still debated. A contemporary meta-analysis has suggested that high-dose regimens
do not further reduce the rate of rebleeding or the need for surgical
intervention when compared with nonhigh dose regimens.57
All patients who are found to be infected with H. pylori should
receive oral eradication therapy as soon as practicable because the
continued long-term presence of H. pylori predicts rebleeding.58
Thus, in a meta-analysis of seven studies, successful eradication
therapy was found to reduce rebleeding rates to 2.9% compared with
20% in the noneradicated group (OR, 0.17; 95% CI, 0.100.32).59
In the absence of H. pylori infection, NSAID usage is often
found to be a major contributing cause of ulcer bleeding and may
actually have more severe consequences. A single casecontrol
study of this issue has demonstrated that NSAID-related bleeding
when compared with matched H. pylori positive cases is associated
with an increased risk of rebleeding (32.4% vs. 13.3%, p = 0.001), an

PMPH_CH13.indd 111

111

increased need for surgical therapy (15.2% vs. 4.8%, p = 0.01), and
an increased mortality (15.2% vs. 3.8%, p = 0.005).60 All NSAIDs,
including low-dose aspirin, should be immediately stopped in
bleeding patients. (Evidence Grade A)
8. Under what circumstances is an operation indicated for a
bleeding peptic ulcer? What surgical techniques are associated
with the lowest rate of UGI rebleeding?
Surgical therapy is indicated in patients whose bleeding is not controlled by nonoperative measures.61,62 The presence of exsanguinating hemorrhage or the lack of endoscopic support are self-evident
indications for emergency operation. Recurrent bleeding after
endoscopy is a more common; albeit, less precise indication. In 5%
to 20% of patients who undergo endoscopic hemostasis, bleeding
continues or recurs, a finding that is associated with an increase in
mortality of 5- to 16-fold over controls.63 Risk factors identified by
two separate logistic regression analyses as independent predictors
of rebleeding or mortality include advanced age, shock, comorbidities, size of ulcer, and presence of major stigmata of hemorrhage.64,65
Most patients with rebleeding can benefit from a secondlook endoscopy performed to provide additional hemostatic
therapy if necessary. In a randomized trial comparing endoscopic
retreatment with surgery, control of bleeding was achieved endoscopically in 35 of 48 patients (72.9%) with fewer complications
experienced than in the surgery alone group (7 vs. 16).66 Another
randomized trial compared a scheduled second therapeutic
endoscopy within 16 to 24 h after the initial endoscopy with a
control group without a routine second endoscopy. The rate of
recurrent bleeding was significantly lower in the second-look
group (5% vs. 13.8%, p = 0.03) and a trend toward fewer operations for rebleeding was identified.67 Further controlled studies
are clearly necessary, but both selective and routine second-look
endoscopy appear capable of favorably influencing the outcome
of peptic ulcer bleeding.
Unsuccessful endoscopic retreatment manifest as persistent or
recurrent bleeding should be addressed by endovascular embolization or operation. The choice of a specific procedure depends on the
availability of resources and expertise and, to some degree, usually
reflects specific local preferences. Embolization can achieve high
clinical success rates with a low morbidity rate when performed in
specialized centers. However, no controlled studies are available
that directly compare the two strategies, angiography and surgery,
in bleeding patients.68,69
In the case of surgery, the choice of the most appropriate operation remains problematic because relevant, high-grade surgical evidence is rare. To this point, only two controlled, randomized trials
comparing various operative approaches have been published since
1990; one from France and a second from England (Table 13.1).70,71
A small, retrospective, multicenter study from Scotland and a large
audit from the Department of Veterans Affairs National Surgical
Quality Improvement Program (NSQIP) database have also been
reported.72,73 The former studies by Poxon et al. and Kubba et al.
compare conservative operations (ulcer oversewing or excision)
with more advanced procedures (vagotomy plus pyloroplasty or
resection). The latter studies by Millat et al. and de la Fuente et al.
compare two different advanced therapies (vagotomy, pyloroplasty
and ulcer oversewing vs. vagotomy and resection).
Notably, all operative techniques described in these studies
resulted in a similar, relatively high mortality rate. In two of the
four studies, the least aggressive forms of therapy were associated

5/21/2012 8:50:53 PM

112

Surgery: Evidence-Based Practice

Table 13.1 Selected Trials of Surgical Management of Bleeding Peptic Ulcer


Author (Ref)

Study Type

Operation

Poxon (71)

CRT

62

Milatt (70)

Kubba (72)

de la Fuente (73)

CRT

NRT

NSQIP audit

Rebleeding (%)

Mortality (%)

Oversewing/excision

7 (11.3)*

16 (26)

67

TV/P or TV/A

4 (5.0)*

13 (19)

58

Oversewing/TV/P

60

10 (17)

13 (22.4)

Resection

2 (3)

14 (23.3)

31

Oversewing

7 (23)*

7 (23)

36

TV/P or TV/A

1 (2.7)*

5 (14)

518

TV and drainage

57 (11.0)

93 (18.0)

389

TV and resection

46 (11.8)

70 (17.22)

CRT: controlled randomized trial; NRT: nonrandomized trial; TV: truncal vagotomy; P: pyloroplasty; A: antrectomy; *p < 0.05.

with a significantly higher rate of rebleeding. Taken together, such


sparse data provides little evidence-based guidance for the surgeon
faced with treating a patient who presents with hemorrhage refractory to nonoperative therapies. Based on the available literature
and personal experience, the author prefers to oversew the bleeding vessel in duodenal ulcers with the addition of pyloroplasty and/
or a truncal vagotomy. Bleeding gastric ulcers are managed with
partial gastric resection but without vagotomy. In each situation,
the patient should be tested for H. pylori infection and treated, if
positive. As noted before, eradication of the organism should be
documented. (Evidence Grade A)
9. What approach is preferred for the management of perforated
PUD?
Perforation is a potentially catastrophic complication of PUD that
is usually heralded by the abrupt and dramatic onset of severe,
midepigastric or generalized abdominal pain. Since the large
majority of perforations occur on the anterior aspect of the stomach or duodenum, pneumoperitoneum is present in 80% to 90%
of cases and is readily detectable on plain abdominal fi lms or CT
scan.74 Posterior perforations are rare (2%) and have a more insidious presentation and a worse outcome.
The therapy of ulcer perforation should address three separate but related issues: the perforation itself, its underlying cause,
and the resultant peritonitis and sepsis. In regards to the last issue,
initial management commonly includes rapid fluid resuscitation,
nasogastric tube drainage, and systemic antibiotic administration. There is less certainty about the respective roles of operative
and nonoperative therapies for the actual perforation and its associated pathogenetic factors.
Peptic ulcer perforation remains a highly morbid condition
with a reported mortality of 8% to 30%.75,76 Based on a number of
multifactorial analyses, several predictors of postoperative complications and death have been consistently identified, including treatment delay, circulatory shock, and major concurrent
illness.77,78 Moreover, it has been suggested that these predictors
can be used to stratify patients and can help to plan initial therapy. Using such an approach, Rahman et al. identified 84 highrisk patients in a cohort of 626 patients with perforated ulcer and
managed them nonoperatively with peritoneal tube drainage.79
They found a significant decrease in overall mortality compared
with historical controls who had undergone conventional operative treatment (9.5% vs. 3.9%; p < 0.0001).

PMPH_CH13.indd 112

The use of nonoperative therapy for ulcer perforation remains


controversial, but appears to be gaining wider acceptance. As
early as 1961, contrast studies documented that more than 40%
of perforations were sealed by the time of clinical presentation.80
Subsequently, multiple nonrandomized studies indicated that
nonoperative treatment of sealed perforations can result in a
lower morbidity and mortality than conventional surgical therapy (Table 13.2).81-84 However, nonoperative treatment fails in 16%
to 32% of perforated patients necessitating emergency operation.
In the only controlled, randomized trial published to date, Crofts
et al. compared initial nonoperative therapy with early operation in 83 patients with perforation.85 No difference was noted
in mortality (4.7% vs. 5.0%), but the hospital stay was 35% longer
in the nonoperative group. The mixed results with nonoperative therapy suggest that such an approach cannot be universally
applied; however, its selective use especially in high-risk patients
may be appropriate if a strict protocol and close follow-up can be
assured.86
Specific operative strategies for the management of ulcer
perforation have continued to evolve. Prior to recognition of the
pathogenic roles of H. pylori and NSAIDs, numerous studies comparing simple closure with definitive operations (vagotomy with
or without resection) appeared to demonstrate a lower recurrence
rate following the more aggressive approach (Table 13.3).87-89 Subsequently, excellent long-term results were observed in 107 carefully selected patients followed for 2 to 20 years after patch closure
and parietal cell vagotomy.90 The authors reported an operative
mortality of only 0.9% and a recurrence rate of 7.4%. However, a
more recent controlled randomized study comparing simple closure with vagotomy and pyloroplasty in over 200 patients failed to
demonstrate an advantage in mortality or recurrence compared
with the more definitive procedure.91
In the current Helicobacter era, pharmacotherapy has further confused the role of definitive surgery in the management
of perforation. Worldwide, the prevalence of H. pylori infection
in patients with perforated PUD is reported to range from 47%
to 100%.92 Persistence of the infection after perforation predicts
recurrence of the ulcer, while successful eradication of the organism results in a significant reduction in recurrence rates.93,94 In a
related study of patients followed for 18 months, ulcer recurrence
was noted in 70% of patients with persistent infection but in only
19% of those in whom H. pylori was eradicated.95 In a controlled,
randomized trial comparing PPI therapy with anti-Helicobacter
therapy following simple closure of the ulcer, the relapse rate was

5/21/2012 8:50:53 PM

Peptic Ulcer Disease

113

Table 13.2 Selected Trials of Nonoperative Management of Perforated Peptic Ulcer


Author (Ref)

Study Type

Treatment

Recurrent, %

Mortality, %

Marshall (84)

NRT

49

Nonoperative

16.3

6.1

21

Operative

14.3

40

Nonoperative

27.5

5.0

43

Operative

4.7

35

Nonoperative

3.0

294

Operative

NA

6.2

Recurrent, %

Mortality, %

Crofts (85)

Berne (82)

CRT

NRT

CRT: controlled randomized trial; NRT: nonrandomized trial; *p < 0.05.

Table 13.3 Selected Trials of Surgical Management of Perforated Peptic Ulcer


Author (Ref)

Study Type

Treatment

Boey (87)

CRT

41

Closure

36.6*

37

Closure/PCV

10.6

15

Closure

63.6*

26.7*

24

Closure/TV

38.1

12.5

306

Closure

35.9

10.4

208

Antrectomy/TV

24.4

2.5

117

Closure

7.1

4.3

V/P

4.4

4.4

Tsugawa (88)

Jordan (89)

Gutierrez de La Pena (91)

NRT

NRT

CRT

90

CRT: controlled randomized trial; NRT: nonrandomized trial; TV: truncal vagotomy; PCV: parietal cell vagotomy; *p < 0.05.

found to be significantly reduced in the anti-Helicobacter group


after 1 year (4.8% vs. 38.1%; p < 0.001).96
These data suggest that the majority of patients with perforated peptic ulcers can be treated with simple closure of the ulcer
when the procedure is combined with appropriate medical measures such as anti-Helicobacter therapy, PPI administration, or
NSAID modulation. More definitive surgical approaches may be
reserved for patients with recurrent ulcer disease or for perforations that are associated with hemorrhage or obstruction.
Successful closure of perforations can be achieved with either
open or laparoscopic techniques. In addition, a hybrid approach
has recently been described, in which intraoperative endoscopy is

used with laparoscopy to plug the perforation with omentum.97 A


meta-analysis of 1113 patients from 15 selected studies found that
the purely laparoscopic approach required longer operating times,
but was associated with less postoperative analgesic use, a shorter
hospital stay, and fewer wound infections.98 To date, three randomized studies have confirmed that laparoscopic and open repairs are
equally safe and effective and have confirmed that postoperative pain
is improved in the laparoscopic group.99-101 However, not all patients
are candidates for laparoscopy. In particular, age over 70 years and
the persistence of symptoms longer than 24 h are associated with
increased morbidity and mortality and are considered relative contraindications to the laparoscopic approach.102 (Evidence Grade B)

Clinical Question Summary


Question

Answer

Grade

References

1 How has the surgery of peptic ulcer


disease (PUD) changed over time?

The number of operations performed for PUD has declined


by >80% over the past two decades. The most common
indications are perforation and bleeding. Operations for
intractability are rare. Patients are generally older, sicker,
and more often female.

7-10

2 What are the major risk factors for


PUD?

In western countries, PUD is primarily caused by H. pylori


infection and NSAID use. Idiopathic causes include ZES,
G-cell hyperplasia, Crohns disease, cocaine abuse, and
systemic mastocytosis.

18-21

(Continued)

PMPH_CH13.indd 113

5/21/2012 8:50:53 PM

114

Surgery: Evidence-Based Practice

(Continued)
Question

Answer

Grade

References

3 What is the appropriate therapy for


H. pylori positive PUD?

First-line therapy combines PPI + clarithromycin +


amoxicillin or metrinidazole and has a successful
eradication rate of 7890%. Rescue therapies are
available for nonresponders.

23-25

4 How can the risk of PUD be minimized


in patients requiring regular NSAID
therapy?

If found, H. pylori should be eradicated before NSAID


therapy is initiated. Misoprostol, PPI therapy, and
double-dose H2RA therapy are all effective in reducing
the risk of NSAID-induced gastrointestinal complications.

27-30

5 In a patient suspected of having a


bleeding peptic ulcer, what should the
initial approach be?

A rapid and accurate diagnosis and aggressive resuscitation


should proceed simultaneously in patients thought to be
bleeding from an UGI source.

33-34

6 In patients with bleeding ulcers, what is


the current role of endoscopy?

Endoscopy is the definitive diagnostic and prognostic


study. It is also an effective therapeutic tool with initial
hemostasis rates of 85100%.

43-53

7 What is the role of pharmacotherapy in


the management of a bleeding peptic
ulcer?

When used as an adjunct to endoscopic therapy, PPIs reduce


the risk of further bleeding and the need for surgery.

54, 55

8a Under what circumstances is an


operation indicated for a bleeding
peptic ulcer?

Surgery is indicated for peptic ulcer hemorrhage that is not


controlled by endoscopic therapy or recurs following
apparently successful endoscopic therapy (520%).

61-63

8b What surgical techniques are associated


with the lowest rate of UGI rebleeding?

When used for peptic ulcer bleeding, combined partial


gastric resection and vagotomy is associated with the
lowest recurrence rate. However, this approach also has
a higher rate of short term and long-term postoperative
complications. Vagotomy with oversewing of the ulcer is
effective when combined with anti-H. pylori therapy in
H. pylori + patients.

83, 85, 86

9 What approach is preferred for the


management of perforated PUD?

Nonoperative therapy can be used in selected patients


who are found to have a sealed perforation on contrast
study. Patch closure is indicated in most patients.
H. pylori should be eradicated when infection is present.
The laparoscopic approach is being used with increased
frequency.

92-99

REFERENCES
1. Baron J, Sonnenberg A. Hospital admission for peptic ulcer and
indigestion in London and New York in the 19th and early 20th
centuries. Gut. 2002;50:568-570.
2. Feinstein L, Holman R, Christensen K, et al. Trends in hospitalizations for peptic ulcer disease, United States, 19982005. Emerg
Infect Dis. 2010;16:1410-1418.
3. Modlin I. From Prout to the proton pump. A History of the science of gastric acid secretion and the surgery of peptic ulcer. Surg
Gynecol Obstet. 1990;170:81-95.
4. Grob G. The rise of peptic ulcer 19001950. Perspecti Biol Med.
2003;46:550-566.
5. Marshall B. Unidentified curved bacilli on gastric epithelium in
active chronic gastritis (letter). Lancet. 1983;1:1273-1275.
6. Gustavsson S, Kelly K, Melton L III, Zinsmeister A. Trends
in peptic ulcer surgery. A population-based study in
Rochester, Minnesota. 19561985. Gastroenterology. 1988; 94:
688-694.
7. Larson G, Davidson P. The decline in surgery for peptic ulcer disease. J Ky Med Assoc. 1986;84:233-236.

PMPH_CH13.indd 114

8. Smith B, Stabile B. Emerging trends in peptic ulcer disease


and damage control surgery in the H. pylori era. Am Surg.
2005;71:797-801.
9. Schwesinger W, Page C, Sirinek K, et al. Operations for peptic
ulcer disease: Paradigm lost. J Gastrointest Surg. 2001;5:438-443.
10. Ohmann C, Imhof M, Ruppert C, et al. Time-trends in the
epidemiology of peptic ulcer bleeding. Scand J Gastroenterol.
2005;40:914-920.
11. Rockall T, Logan R, Devlin H, et al. Incidence of and mortality
from acute upper gastrointestinal haemorrhage in the United
Kingdom. BMJ. 1995;311:222-226.
12. Fowler S, Khoubian J, Mathiasen R, et al. Peptic ulcers in the
elderly is a surgical disease. Am J Surg. 2001;182:733-737.
13. Schwarz K. Uber penetricrende magen und jejunalgeschwur.
Beitr Klin Surg. 1919;67:96.
14. Schwesinger W. Is Helicobacter pylori a myth or the missing
link? Am J Surg. 1996;172:411-417.
15. Garcia Rodriguez L, Barreales Tolosa L. Risk of upper
gastrointestinal complications among users of traditional
NSAIDs and COXIBs in the general population. Gastroenterology
2007;132:498-506.

5/21/2012 8:50:53 PM

Peptic Ulcer Disease

16. Bliss D, Stabile B. The impact of ulcerogenic drugs on surgery


for the treatment of peptic ulcer disease. Arch Surg. 1991;126:
609-612.
17. Dixon M. Pathophysiology of Helicobacter pylori infection.
Scand J Gastroenterol. 1994;29:7-10.
18. Das J, Paul N. Epidemiology and pathophysiology of Helicobacter
pylori infection in children. Indian J Ped. 2007;74:287-290.
19. Vane J, Botting R. Mechanism of action of nonsteroidal antiinflammatory drugs. Am J Med. 1998;104:2S-8S.
20. Huang JQ, Sridhar S, Hunt R, et al. Role of Helicobacter pylori
infection and non-steroidal anti-inflammatory drugs in pepticulcer disease: A meta-analysis. Lancet. 2002;359:14-22.
21. Peura D. The problem of Helicobacter pylori-negative
idiopathic ulcer disease. Baillires Clin Gastroenterol 2000;14:
109-117.
22. Yuan Y, Hunt R. Treatment of non-NSAID and non-H. Pylori
gastroduodenal ulcers and hypersecretory states. In: Wolfe M,
ed. Therapy of Digestive Disorders. 2nd ed. Philadelphia, PA:
Saunders Elsevier; 2006:315-336.
23. Ford A, Delaney B, Forman D, et al. Eradication therapy for
peptic ulcer disease in Helicobacter pylori positive patients.
Cochrane Database of Systematic Reviews 2006, Issue 1. Art
No.:CD003840. DOI:10.1002/14651858.CD003840.pub 4.
24. Malfertheiner P, Mgraud F, OMorain C, et al. Current
concepts in the management of Helicobacter pylori infectionThe Maastricht 2-2000 Consensus Report. Aliment Pharmacol
Ther. 2002;16:167-180.
25. Laine L, Suchower L, Frantz J, et al. Twice-daily, 10-day triple
therapy with omeprazole, amoxicillin, and clarithromycin
for Helicobacter pylori eradication in duodenal ulcer disease:
Result of three multicenter, double-blind, United States trials.
Am J Gastroenterol. 1998;93:2106-2112.
26. Duck W, Sobel J, Pruckler J, et al. Antimicrobial resistance
among Helicobacter pylori-infected persons, United States.
Emerging Infect Dis. 2004;10:1088-1094.
27. Gisbert J, Pajares J. Treatment of Helicobacter pylori infection:
The past and the future. Eur J Int Med. 2010;21:357-359.
28. Chan F, Graham D. Prevention of non-steroidal anti-inflammatory
drug gastrointestinal complications review and recommendations
based on risk assessment. Aliment Pharmacol Ther 2004;19:10511061.
29. Rostom A, Dube C, Wells G, et al. Prevention of NSAID-induced
gastroduodenal ulcers. Cochrane Database of Systematic
Reviews 2002, Issue 4. Art. No.:CD002296. DOI:10.1002/
14651858.CD002296.
30. Vonkeman H, Fernandes R, van der Palen J, et al. Proton-pump
inhibitors are associated with a reduced risk for bleeding and
perforated gastroduodenal ulcers attributable to non-steroidal
anti-inflammatory drugs: A nested case-control study. Arth Res
Ther. 2007;9:R52 (doi:10.1186/ar2207).
31. Blecker E, Heller, J, Schmitz V, et al, Diagnosis and management
of upper gastrointestinal bleeding. Dtsch Arztebl Int. 2008;105:
85-94.
32. Sung J, Tsoi K, Ma T, et al. Causes of mortality in patients with
peptic ulcer bleeding: A prospective cohort study of 10, 428
cases. Am J Gastroenterol. 2010;105:84-89.
33. Matlock J, Freeman M. Non-variceal upper GI hemorrhage:
Doorway to diagnosis. Gastrointest Endoscc. 2005;7:112-117.
34. Baradarian R, Ramdhaney S, Chapalamadugu R, et al. Early
intensive resuscitation of patients with upper gastrointestinal
bleeding decreases mortality. Am J Gastroenterol. 2004;99:
619-622.

PMPH_CH13.indd 115

115

35. Aljebreen A, Fallone C, Barkun A. Nasogastric aspirate predicts


high-risk endoscopic lesions in patients with acute upper-GI
bleeding. Gastrointest Endosc. 2004;59:172-178.
36. Witting M, Magder L, Heins A, et al. Usefulness and validity
of diagnostic nasogastric aspiration in patients without
hematemesis. Ann Emerg Med. 2004;43:525-532.
37. Frossard J, Spahr L, Queneau P, et al. Erythromycin intravenous
bolus infusion in acute upper gastrointestinal bleeding: A
randomized, controlled, double-blind trial. Gastroenterology.
2002;123:17-23.
38. Carbonnell N, Pauwels A, Serfaty L, et al. Erythromycin
infusion prior to endoscopy for acute upper gastrointestinal
bleeding: A randomized, controlled, double-blind trial. Am J
Gastroenterol. 2006;101:1211-1215.
39. Winstead N, Wilcox C. Erythromycin prior to endoscopy for
acute upper gastrointestinal haemorrhage: A cost-effectiveness
analysis. Aliment Pharmacol Ther. 2007;26:1371-1377.
40. Enestvedt B, Gralnek I, Mattek N, et al. An evaluation of
endoscopic indications and findings related to nonvariceal
upper-GI hemorrhage in a large multicenter consortium.
Gastrointest Endosc. 2008;67:422-429.
41. Laine L, Peterson W. Bleeding peptic ulcer. N Eng J Med.
1994;331:717-727.
42. Kahi C, Jensen D, Sung J, et al. Endoscopic therapy versus
medical therapy for bleeding peptic ulcer with adherent clot:
A meta-analysis. Gastroenterology. 2005;129:855-862.
43. Gralnek I, Jensen D, Kovacs T, et al. An economic analysis of
patients with active arterial peptic ulcer hemorrhage treated
with endoscopic heater probe, injection sclerosis, or surgery in
a prospective, randomized trial. Gastrointest Endosc. 1997;46:
105-112.
44. Lee J, Turnipseed S, Romano S, et al. Endoscopy-based triage
significantly reduces hospitalization rates and costs of treating
upper GI bleeding: a randomized controlled trial. Gastrointest
Endosc. 1999;50:755-761.
45. Bjorkman DJ, Zaman A, Fennerty B, et al. Urgent vs. elective
endoscopy for acute non-variceal upper-GI bleeding: An
effectiveness study. Gastrointest Endosc. 2004;60:1-8.
46. Sung J. Current management of peptic ulcer bleeding. Nat Clin
Pract Gastroenterol Hepatol. 2006;3:24-32.
47. Sacks H, Chalmers T, Blum A, et al. Endoscopic hemostasis
an effective therapy for bleeding peptic ulcers. JAMA.
1990;264:494-499.
48. Cook D, Guyatt G, Salens B, et al. Endoscopic therapy for
acute nonvariceal upper gastrointestinal hemorrhage: A metaanalysis. Gastroenterology. 1992;102:139-148.
49. Park C, Lee S, Park J, et al. Optimal injection volume of
epinephrine for endoscopic prevention of recurrent peptic ulcer
bleeding. Gastrointest Endosc. 2004;60:875-880.
50. Marmo R, Rotondano G, Piscopo R, et al. Dual therapy versus
monotherapy in the endoscopic treatment of high-risk bleeding
ulcers: A meta-analysis of controlled trials. Am J Gastroenterol.
2007;102:279-289.
51. Vergara M, Calvet X, Gisbert J. Epinephrine injection versus
epinephrine injection and a second endoscopic method in high
risk bleeding ulcers. Cochrane Database of Systematic Reviews
2007. Issue 2. Art. No. CD005584. DOI:10.1002/14651858.
CD005584.pub2.
52. Sung J, Tsoi K, Lai L, et al. Endoscopic clipping versus infection
and thermo-coagulation in the treatment of non-variceal
upper gastrointestinal bleeding: A meta-analysis. Gut. 2007;56:
1364-1372.

5/21/2012 8:50:53 PM

116

Surgery: Evidence-Based Practice

53. Saltzman J, Strate L, Di Sena V, et al. Prospective trial of


endoscopic clips versus combination therapy in upper GI bleeding
(Protect-UGI Bleeding). Am J Gastroenterol. 2005;100:1-6.
54. Leontiadis G, Sharma V, Howden C. Proton pump inhibitor
therapy for peptic ulcer bleeding: Cochrane Collaboration
Meta-analysis of Randomized Controlled Trials. Mayo Clin
Proc. 2007;82:286-296.
55. Lau J, Sung J, Lee K, et al. Effect of intravenous omeprazole
on recurrent bleeding after endoscopic treatment of bleeding
peptic ulcers. N Eng J Med. 2000;343:310-316.
56. Barkun A, Bardou M, Kuipers E, et al. International consensus
recommendations on the management of patients with
nonvariceal upper gastrointestinal bleeding. Ann Int Med.
2010;152:101-113.
57. Wang C-H, Ma M, Chou H-C, et al. High-dose vs. non-high-dose
proton pump inhibitors after endoscopic treatment in patients
with bleeding ulcer. A systematic review and meta analysis of
randomized controlled trials. Arch Int Med. 2010;170:751-758.
58. Lai K, Hui W, Wong W, et al. Treatment of Helicobacter
pylori in patients with duodenal ulcer hemorrhageA longterm randomized, controlled study. Am J Gastroenterol.
2000;95:2225-2232.
59. Gisbert J, Pajares J. Helicobacter pylori Infection and
Perforated Peptic Ulcer Prevalence of the Infection and Role of
Antimicrobial Treatment. Helicobacter. 2003;8:159-167.
60. Adamopoulos A, Efstathiou S, Tsioulos D, et al. Bleeding
duodenal ulcer: Comparison between Helicobacter pylori
positive and Helicobacter pylori negative bleeders. Digestive and
Liver Disease. 2004;36:13-20.
61. Lau J, Chung S. Surgery in the acute management of bleeding
peptic ulcer. Baillieres Clin Gastroenterol. 2000;14:505-518.
62. Cowles R, Mulholland M. Surgical management of peptic ulcer
disease the Helicobacter era-management of bleeding peptic
ulcer. Surg Laparosc Endosc Percutan Tech. 2001;11:2-8.
63. Rockall T, Logan R, Devlin H, et al. Incidence of and mortality
from acute upper gastrointestinal haemorrhage in the United
Kingdom. BMJ. 1995;311:222-226.
64. Rockall T, Logan R, Devlin H, et al. Risk Assessment after acute
upper gastrointestinal haemorrhage. Gut. 1996;38:316-321.
65. Branicki F, Coleman S, Fok P, et al. Bleeding peptic ulcer:
A prospective evaluation of risk factors for rebleeding and
mortality. World J Surg. 1990;14:262-270.
66. Lau J, Sung J, Lam Y, et al. Endoscopic retreatment compared
with surgery in patients with recurrent bleeding after initial
endoscopic control of bleeding ulcers. N Eng J Med. 1999;340:
751-756.
67. Chiu P, Lam C, Lee S, et al. Effect of scheduled second
therapeutic endoscopy on peptic ulcer rebleeding: A prospective
randomized trial. Gut. 2003;52:1403-1407.
68. Burris J, Lin P, Johnston W, et al. Emergent embolization of the
gastroduodenal artery in the treatment of upper gastrointestinal
bleeding. The experience from a surgeon-initiated intervention
program. Am J Surg. 2009;108:59-63.
69. Loff roy R, Gulu B, DAthis P, et al. Arterial embolotherapy
for endoscopically unmanageable acute gastroduodenal
hemorrhage: Predictors of early rebleeding. Clin Gastroenterol
Hepatol. 2009;7:515-523.
70. Millat B, Hay J, Valleur P, et al. Emergency Surgical Treatment
for Bleeding Duodenal Ulcer: Oversewing plus vagotomy versus
gastric resection, a controlled randomized trial. World J Surg.
1993;17:568-574.
71. Poxon V, Keighley M, Dykes PW, et al. Comparison of minimal
and conventional surgery in patients with bleeding peptic ulcer:
A multicentre trial. Br J Surg. 1991;78:1344-1345.

PMPH_CH13.indd 116

72. Kubba A, Choudari C, Rajgopal C, et al. The outcome of urgent


surgery for major peptic ulcer haemorrhage following failed
endoscopic therapy. Euro J Gastroenterol Hepatol. 1996;8:1175-1178.
73. De la Fuente S, Khuri S, Schifft ner T, et al. Comparative analysis
of vagotomy and drainage versus vagotomy and resection
procedures for bleeding peptic ulcer disease: Results of 907
patients from the Department of Veterans Affairs National
Surgical Quality Improvement Program Database. J Am Coll
Surg. 2006;202:78-86.
74. Chen C, Huang H, Yang C, Yeh Y. The features of perforated
peptic ulcers in conventional computed tomography.
Hepatogastroenterology. 2001;48:1393-1396.
75. Svanes C, Salvesen H, Stangeland L, et al. Perforated peptic
ulcer over 56 years. Time trends in patients and disease
characteristics. Gut. 1993;34:1666-1671.
76. Bucher P, Oulbaci W, Morel P, et al. Results of conservative
treatment for perforated gastroduodenal ulcer in patients not
eligible for surgical repair. Swiss Med Wkly. 2007;137:337-340.
77. Testini M, Portincasa P, Piccinni G, et al. Significant factors
associated with fatal outcome in emergency open surgery for
perforated peptic ulcer. World J Gastroenterol. 2003;9:2338-2340.
78. Sharma SS, Mamtani MR, Sharma MS, et al. A prospective
cohort study of postoperative complications in the management
of perforated peptic ulcer. BMC Surg. 2006;6:8.
79. Rahman M, Islam M, Flora S, et al. Mortality in perforated
peptic ulcer patients after selective management of stratified
poor risk cases. World J Surg. 2007;31:2341-2344.
80. Jacobson G, Berne C, Meyers H, et al. The examination of
patients with suspected perforated ulcer using a water-soluble
contrast medium. Am J Roentgenol Radium Therap Nucl Med.
1961;86:37-49.
81. Donovan A, Vinson T, Maulsby G, et al. Selective treatment of
duodenal ulcer with perforation. Ann Surg. 1979;189:627-636.
82. Berne T, Donovan A. Nonoperative treatment of perforated
duodenal ulcer. Arch Surg. 1989;124:830-832.
83. Cocks J, Kernutt R, Singlair W, et al. Perforated peptic ulcer: A
deliberative approach. Aust N.Z. J Surg. 1989;59:379-385.
84. Marshall C, Ramaswamy P, Bergin F, et al. Evaluation of a
protocol for the non-operative management of perforated
peptic ulcer. Br J Surg. 1999;86:131-134.
85. Crofts T, Park K, Steele R, et al. A randomized trial of
nonoperative treatment for perforated peptic ulcer. N Eng J
Med. 1989;320:970-973.
86. Donovan AJ, Berne V, Donovan JA. Perforated duodenal ulcer:
An alternative therapeutic plan. Arch Surg. 1998;133:1166-1171.
87. Boey J, Branicki F, Alagaratnam T, et al, Proximal gastric
vagotomy. The preferred operation for perforations in acute
duodenal ulcer. Ann Surg. 1988;208:169-174.
88. Tsugawa K, Koyanagi N, Hashizume M, et al. The therapeutic
strategies in performing emergency surgery for gastroduodenal
ulcer perforation in 130 patients over 70 years of age. HepatolGastroenterol. 2001;48:156-162.
89. Jordan G, DeBakey M, Duncan J, et al. Surgical Management of
Perforated Peptic Ulcer. Ann Surg. 1974 179:628-633.
90. Jordan PH, Thornby J. Perforated Pyloroduodenal Ulcers LongTerm Results with Omental Patch Closure and Parietal Cell
Vagotomy. Ann of Surg. 1995;221:479-488.
91. Gutierrez de la Pena C, Marquez, Fakih F. Simple closure or
vagotomy and pyloroplasty for the treatment of a perforated
duodenal ulcer: Comparison of results. Dig Surg. 2000;17:
225-228.
92. Gisbert J, Pajares J. Helicobacter pylori infection and perforated
peptic ulcer. Prevalence of the infection and role of antimicrobial
treatment. Helicobacter. 2003;8:159-167.

5/21/2012 8:50:53 PM

Peptic Ulcer Disease

93. Sebastian M, Prem Chandran V, Elashaal Y, et al. Helicobacter


pylori infection in perforated peptic ulcer disease. Br J Surg.
1995;82:360-362.
94. Kate V, Anathakrishnan N, Badrinath S, et al. Effect of Helicobacter
pylori eradication on the ulcer recurrence rate after simple closure
of perforated duodenal ulcer: Retrospective and prospective
randomized controlled studies. Br J Surg. 2001;88:1054-1058.
95. Bose AC, Kate V, Ananthakrishnan N, et al. Helicobacter pylori
eradication prevents recurrence after simple closure of perforated
duodenal ulcer. J Gastroenterol Hepatol. 2007;22:345-348.
96. Ng E, Lam Y, Sung J, et al. Eradication of Helicobacter pylori
prevents recurrence of ulcer after simple closure of duodenal
ulcer perforation. Ann Surg. 2000;231:153-158.
97. Lunevicius R, Morkevicius M. Comparison of laparoscopic
vs open repair for perforated duodenal ulcers. Surg Endosc.
2005;19:1565-1571.

PMPH_CH13.indd 117

117

98. Siu W, Leong H, Law B, et al. Laparoscopic repair for perforated


peptic ulcer: A randomized controlled trial. Ann Surg. 2002;
235:313-319.
99. Lau WY, Leung KL, Kwong KH, et al. A Randomized Study
Comparing Laparoscopic Versus Open Repair of Perforated
Peptic Ulcer Using Suture or Sutureless Technique. Ann Surg.
1996;224:131-138.
100. Malkov I, Zaynutdinov A, Veliyev N, et al. Laparoscopic and
endoscopic management of perforated duodenal ulcer. J Am
Coll Surg. 2004;198:352-355.
101. Bertleff M, Halm J, Bemelman W, et al. Randomized clinical
trial of laparoscopic versus open repair of the perforated peptic
ulcer: The LAMA trial. World J Surg. 2009;33:1368-1373.
102. Bertleff M, Lange J. Laparoscopic correction of perforated
peptic ulcer: First choice. A review of the literature. Surg Endosc.
2010;24:1231-1239.

5/21/2012 8:50:53 PM

Commentary on
Peptic Ulcer Disease
Frederick A. Moore

with clarithromycin (500 mg bid) and amoxicillin (1 gm bid)


or metronidazole (500 mg bid) for 10 to 14 days. Second-line
rescue therapy treatment includes quadruple therapy which
combines a PPI with bismuth subsalicycle (2 tablets daily),
metronidazole (250 mg qid) and tetracycline (500 mg qid) for
14 days. Eradication should always be confirmed with either a
urea breath test or a stool antigen. Eradication rates with either
regimen range from 75% to 90%.
2. Management of Upper Gastrointestinal Bleeding: This has
shifted to more aggressive endoscopy for risk stratification
and therapeutic interventions. Resuscitation and correction of
coagulopathy are key early interventions prior to endoscopy.
PPI infusions should be started as early as possible. Stop
NSAIDs and aspirin. Do not forget the H. pylori story.
If rebleeding occurs after endoscopic intervention, repeat
endoscopy (not surgery) is indicated. If bleeding cannot be
controlled, interventional radiology embolization is an option
in specialized centers. If you have to operate, what procedure
should be performed is debatable. I agree with the authorbe a
minimalist: (1) wedge resection of gastric ulcers (if feasible) with
no truncal vagotomy and (2) oversew the bleeding vessel within
the duodenal ulcers with pyloroplasty and a truncal vagotomy.
Again do not forget the H. pylori story.
3. Treatment of Perforated Duodenal Ulcer: In this era of H. pylori,
the treatment of a perforated PUD has been simplified. After
volume resuscitation and antibiotic administration, go to the OR
and perform open or laparoscopic repair with peritoneal washout.
Usually, the perforation is on the anterior surface of the postpyloric
duodenum and is <5 mm in diameter. This is best closed with an
omental patch. Attempts to close the hole directly can result in a
bigger hole, when the sutures pull through the inflamed tissue. Do
not forget the H. pylori story. For patients who have clearly failed
medical management or those who cannot afford, tolerate or
comply with medical management, a highly selective vagotomy is
reasonably definitive ulcer operation. Nonoperative management
of perforated PUD in selected patients is an option supported by
the literature, but I rarely pursue this.
4. Management of a Perforated Gastric Ulcer: These are
commonly associated with NSAID use. In a stable patient, with
a perforated ulcer on the greater curvature or in the body of the
stomach are frequently amenable to wedge resection and this is
a reasonable option. Perforated ulcers on the lesser curvature of
the stomach, distal gastrectomy including the ulcer is usually
required. Billroth I reconstruction is preferred and a vagotomy
is not required. In the unstable patient, biopsy and omental
patch closure can be life-saving. Do not forget the H. pylori
story.

Management of peptic ulcer disease (PUD) has changed drastically over the course of my career. For a young surgeon at Denver
General Hospital in the late 1980s, elective operations for intractable PUD and gastric outlet obstruction as well as emergency operations for bleeding and perforation were surprisingly common. We
debated the optimal role of various procedures (including subtotal
resection, vagotomy and antrectomy, vagotomy with pyloroplasty
or gastroenterostomy and highly selective vagotomy) based the
indication and patient stability. We fretted over how to close the
difficult duodenal stump and when to use a lateral duodenostomy tube. On teaching rounds we discussed (1) different types of
pyloroplasty (e.g., Heineke-Mikulicz, Finney, and Jaboulay), (2)
different ways to reconstruct after gastric resection (e.g., Billroth
I, Billroth II, Hofmeister, Roux-en-y, and Polya) and their relative
advantages/disadvantages, (3) giant duodenal ulcer, (e) classic presentation of a blown duodenal stump, (f) the ZollingerEllison
syndrome, and (6) different types postgastrectomy syndromes
(e.g., dumping, bile gastritis, afferent loop, and efferent loop) and
how they would be managed surgically. We were diligent in these
discussions because we were certain that there would be questions
related to PUD on the in-service exams as well as the written and
oral board exams. These operations and the associated discussions
are now largely irrelevant in my practice as an Acute Care Surgeon. I occasionally operate for perforation, rarely for bleeding or
obstruction and never for intractability. Occasionally, I am called
to assist my junior partners, because they have done so few of these
operations. The manuscript nicely outlines the reasons for these
changes including (1) the wide spread use of new pharmacotherapy, (2) the expanded role of interventional endoscopy, and (3) the
changing epidemiology of PUD where pathologic hyperacidity has
been replaced by the Helicobacter pylori infection and nonsteroidal
anti-inflammatory drug (NSAID) use as prime inciting events.

SO WHAT DOES A SURGEON REALLY


NEED TO KNOW ABOUT PUD?
1. The H. pylori story: Most patients (above 90%) with PUD
have an H. pylori infection and/or recent use of NSAIDs. The
location of the H. pylori infection helps determine the clinical
presentation. Antral-dominant infection causes hyperacidity
with duodenal ulcer while body-dominant infection causes
mucosal atrophy with hypochlorhydria and gastric ulcers.
NSAIDs can aggravate this pathophysiology by inhibiting
constitutive COX-1, which decreases the local production
cytoprotective prostacyline. The treatment of H. pylori
includes a combination of a proton pump inhibitor (PPI)
118

PMPH_CH13.indd 118

5/21/2012 8:50:53 PM

CHAPTER 14

ZollingerEllison Syndrome
Geoffrey W. Krampitz and Jeffrey A. Norton

INTRODUCTION

Stomach distension, vagal


stimulation, GRP, intraluminal
peptides, and hypercalcemia

ZollingerEllison Syndrome (ZES) is a syndrome of severe peptic ulcer disease caused by gastrin hypersecretion by a functional
neuroendocrine tumor called gastrinoma. In 1955, Zollinger and
Ellison reported cases of islet cell tumors of the pancreas and gastric acid hypersecretionin association with unusual occurrences of
jejunal peptic ulcer disease. These recurrent ulcers were refractory
to conventional acid-reduction surgery and ultimately required
total gastrectomy for symptomatic control.1 Zollinger and Ellison theorized that the associated pancreatic tumors were the cause
of the severe peptic ulcer disease in these patients. We now know
that these tumors first described by Zollinger and Ellison are in fact
gastrinomas that produce unregulated amounts of the hormone
gastrin that in turn stimulates excessive gastric acid secretion, leading to intractable peptic ulcer disease.

Stomach lumen

Stomach pH below 3,
somatostatin, secretin, GIP,
VIP, glucagon and calcitonin

G-cell

Gastrin
Gastrin-R

CCK2R
Pl
ECL cell

Ca2+

H-K-ATPase

PK
Histamine

H+

Histamine-R
AC

cAMP
Parietal cell

PATHOPHYSIOLOGY

Figure 14.1 Effect of gastrin on oxyntic parietal cells. AC:


adenylatecyclase, cAMP: cyclic adenosine monophosphate, Ca2+:
calcium ion, CCK2R: cholecystokinin-2 receptor, ECL: enterochromaffin-like, gastrin-R: gastrin receptor, H+: hydrogen ion,
histamine-R: histamine receptor, H-K-ATPase: hydrogen potassium adenosine triphosphatase, PI: phosphatidyl inositol, and PK:
protein kinase.

Parietal cells of the oxyntic mucosa of the stomach secrete hydrochloric acid into the stomach lumen in response to histamine from
mast cells and enterochromaffin-like (ECL) cells, acetylcholine
from vagal innervations, and the linear peptide hormone gastrin
from G cells of the duodenum, antro-pyloric mucosa, and pancreas.
Gastrin is central to the pathophysiology of ZES. Gastrin release
is stimulated by stomach distension, vagal stimulation mediated
by gastrin-releasing peptide, intraluminal peptides, and hypercalcemia. It is inhibited by stomach acidity (pH < 3) via a negative
feedback mechanism mediated by the release of somatostatin by
delta cells, secretin, gastroinhibitory peptide, vasoactive intestinal peptide, glucagon, and calcitonin. Gastrin exerts its effects on
the oxyntic mucosa of the stomach via two independent pathways,
directly on parietal cells and indirectly via ECL cells (Fig. 14.1).
In the indirect pathway, gastrin binds the cholecystokinin-2
receptor on ECL cells, potentiating the release of histamine, that
in turn interacts with histamine-2 receptors on parietal cells to
activate adenylatecyclase to increase intracellular cyclic AMP

(cAMP).2 In the direct pathway, gastrin binds receptors on the


parietal cells that increase intracellular calcium levels via the
phosphatidyl inositol pathway. Both cAMP and calcium act via
protein kinases to activate the hydrogen potassium ATPase (H-KATPase) on the apical membrane of parietal cells, thereby increasing gastric acid production.3
In summary, gastric acid production is stimulated by three
mechanisms mediated by gastrin, histamine, and acetylcholine.
Gastrin is the principal effector in the gastrin pathway and plays
a major role in the histamine pathway. Although conventional
acid-reduction surgery eliminates the cholinergic stimulus for
119

PMPH_CH14.indd 119

5/21/2012 8:51:35 PM

120

Surgery: Evidence-Based Practice

gastric acid production, it does not address the effects of gastrin.


Thus, it becomes clear why the sentinel cases of peptic ulcer disease in the setting of gastrinoma that so intrigued Zollinger and
Ellison were refractory to conventional acid-reduction surgery.

EPIDEMIOLOGY
Incidence
Gastrinoma is the second most common neuroendocrine tumor
with a yearly incidence of approximately 0.1 to 3 cases per million
people. ZES is the underlying cause in approximately 0.1% to 1% of
patients with peptic ulcer disease.4 Because of an increased awareness of ZES and the widespread availability of accurate immunoassays to measure serum concentrations of gastrin, gastrinoma
is increasingly diagnosed and treated at an early stage of disease.
However, the mean time from symptoms to diagnosis is 8 years in
many studies, so that improvements in detection are needed.

Figure 14.2 Distribution of gastrinomas within the duodenum and pancreas. Within the duodenum, 57% of gastrinomas
are found in the first portion, 33% in the second portion, 8.5%
in the third portion, and only 3% in the fourth portion. Within
the pancreas, 48% of gastrinomas are found in the tail, 30% in the
head, and 22% in the body.

ASSOCIATION WITH MULTIPLE


ENDOCRINE NEOPLASIA TYPE 1
ZES occurs in both sporadic and familial forms. In 80% of
cases, ZES occurs sporadically. However, approximately 20%
of patients with ZES have the familial form associated with multiple endocrine neoplasia type 1 (MEN1). Fift y percent of patients
with MEN1 have ZES making gastrinoma the most common
functional neuroendocrine in MEN1. Thus, during the workup
for ZES, MEN1 must always be excluded.

Figure 14.3 Duodenal gastrinoma. (A) Endoscopic view of duodenal gastrinoma. (B) Pathology specimen of a solitary duodenal
gastrinoma.

CHARACTERISTICS
176 Patients
with ZES

Size and Location


Approximately 80% of gastrinomas are found within the gastrinoma triangle, the apices of which are bounded by the junction
of the cystic and common bile ducts superiorly, the junction of
the second and third portions of the duodenum laterally, and
the neck of the pancreas medially. Primary gastrinomas have
been reported in a number of ectopic anatomical sites including the jejunum, stomach, liver, spleen, mesentery, ovary, heart,
and lymph nodes.5 Gastrinomas are three times more likely to
occur in the duodenum than in the pancreas, with the highest
proportion in the fi rst portion of the duodenum and becoming
progressively less common in the distal duodenum (Fig. 14.2).6
Duodenal gastrinomas (Fig. 14.3) are often smaller compared
with pancreatic tumors (0.95 vs. 2.1 cm) that are larger and usually solitary.7 In MEN1, both pancreatic and duodenal gastrinomas are multiple (Fig. 14.4). Gastrinomas have been found in
extrapancreatic, extraintestinal lymph nodes with no identifiable primary pancreatic or duodenal tumor.8-10 Whether these
represent lymph node primary tumors or metastases from occult
pancreatic or intestinal primary tumors is controversial. In one
series of 138 patients who underwent exploration for gastrinoma,
10% of patients with sporadic ZES achieved long-term cure after
resection of a lymph node only. These patients were followed
for a mean of 10 years, suggesting that these tumors represent
true lymph node primary gastrinoma (Fig. 14.5). 8 The discov-

PMPH_CH14.indd 120

82 (47%)
Duodenal tumor

45 (26%)
Lymph node
only

19 (11%)
Not disease-free

36 (20%)
Pancreatic
tumor

26 (15%)
Disease-free

11 (6%)
11 (6%)
Primary in other No tumor found
location

Possible
LN Primary

F/U 10.4 1.2 yrs


18 (10%)
Remained
disease-free
LN Primary

8 (5%)
Relapsed
3 = Duod. tumor
4/8 Reop

1 = Unknown primary

Figure 14.4 Distribution of patients with ZES dependent on


surgical results. Of the 176 patients with ZES undergoing exploratory laparotomy, 45 had an LN only removed, and 26 of the 45
patients (15%) were disease-free postresection and had a possible LN primary. By permission.8
ery that neuroendocrine cells may be found within abdominal
lymph nodes offers a possible explanation for the origin of these
tumors.11,12 The characteristics of gastrinomas are summarized
in Table 14.1.

5/21/2012 8:51:36 PM

ZollingerEllison Syndrome

121

Table 14.1 Characteristics of Gastrinoma


Tumor

Gastrinoma

Incidence
(people/
million/year)
0.13

Hormone
secreted

Signs or
symptoms

Duodenum
Gastrin

Epigastric pain,
diarrhea,
esophagitis

Figure 14.5 Specimens from Patient 7. Panel A shows the


bisected duodenum (D), common bile duct (C), and head of
the pancreas (P). A small gastrinoma is present in the duodenal
mucosa (arrow). Panel B shows a histologic section of a duodenal
gastrinoma (solid arrow). The size of the tumor barely exceeds
the size of a normal mucosal fold (open arrow). (Hematoxylin
and eosin, x10.) By permission. 52

METASTATIC POTENTIAL AND SURVIVAL


Gastrinomas are slow growing, but approximately 60% to 90% are
malignant, with patients having lymph node, liver, or distant metastatic disease at the time of diagnosis. In 25% of cases, the tumor may
pursue a particularly aggressive course. Gastrinoma is associated
with lymph node involvement in 50% to 80% of patients and, unlike
many other types of cancers, lymph node involvement alone without
hepatic or distant metastases does not decrease survival.13-15 Pancreatic gastrinomas appear to have a higher incidence of liver metastases
compared with duodenal tumors (50% vs. 10%), whereas duodenal
tumors have a higher incidence of lymph node metastases (4070%).
In addition, tumors in the distal pancreas are more likely to metastasize to the liver than are tumors arising within the gastrinoma triangle.16 Overall, however, gastrinomas of the duodenum and pancreas
appear to have a similar incidence of metastases. Because liver metastases have a direct affect on survival, pancreatic gastrinomas have
a decreased long-term survival compared with duodenal primary
tumors. Patients without any liver metastases had a 95% 20-year
survival, whereas patients with diffuse bilobar liver metastases had
a 10-year survival of only 15%. Patients who had a solitary liver
metastasis or fewer than five discrete metastases in both liver lobes
had an intermediate survival (60% at 15 years).17 The extent of liver
involvementneuroendocrine tumor is therefore the most important predictor of survival in neuroendocrine tumors (Fig. 14.6).

CLINICAL PRESENTATION
Patients with ZES most often present with symptoms of peptic ulcer disease. Gastrinomas secrete excessive amounts of the

PMPH_CH14.indd 121

Location (%)

60

Pancreas
20

Malignant (%)

MEN1
(%)

Lymph Node
10

6090

20

Figure 14.6 Multiple duodenal gastrinomas in MEN1 with ZES.


hormone gastrin, and thus the gastric parietal cells are under constant stimulation to produce acid, which causes peptic ulceration
and epigastric abdominal pain in 80% of patients with ZES.
Diarrhea, caused by gastrin-induced acid hypersecretion and
increased bowel motility, is the second most common symptom
and may be the only manifestation of ZES in 20% of patients.
Esophagitis with or without stricture occurs with more severe
forms of the syndrome.
As mentioned previously, approximately 20% of patients with
ZES will have it as part of MEN1, and this syndrome must always
be excluded. A significant family history of ulcers and peptic ulceration occurring at a young age are clues to familial gastrinoma. In
addition, peptic ulcers in association with hyperparathyroidism
and/or nephrolithiasis, pituitary tumors or prolactinomas, benign
thyroid tumors, benign and malignant adrenocortical tumors,
lipomas, and cutaneous angiofibromas all may be indicative of
MEN1.
Patients with ZES usually have a solitary ulcer in the proximal duodenum much like patients with peptic ulcer disease
unrelated to gastrinoma. However, atypical presentations consisting of peptic ulceration in multiple locations or in unusual
locations such as distal duodenum or jejunum should raise
concern for ZES. In addition, recurrent ulceration refractory
to appropriate medical treatment, or after acid-reducing surgical procedures, should raise suspicion of ZES. All patients with
peptic ulcer disease severe enough to require surgery should be
screened preoperatively for gastrinoma. Furthermore, patients
with peptic ulcer disease in the presence of persistent diarrhea
or the absence of Helicobacter pylori infection should be investigated. However, not all patients with ZES have peptic ulcer disease, and 20% of patients with ZES have no evidence of peptic
ulceration at the time of presentation.
Because ZES is rare and few clinicians have seen many cases,
there may be a failure to include ZES in the differential diagnosis.
ZES should be considered in all patients who present with one or

5/21/2012 8:51:37 PM

122

Surgery: Evidence-Based Practice

more symptoms referable to the upper gastrointestinal tract or in


those with peptic ulcer disease and primary hyperparathyroidism
and nephrolithiasis or family history suspicious for MEN1.

DIFFERENTIAL DIAGNOSIS
Hypergastrinemia may occur as a manifestation of many diseases or conditions apart from ZES. Ulcerogenic conditions with
excessive gastric acid secretion include gastric outlet obstruction, retained gastric antrum after Bilroth II reconstruction, and
G-cell hyperplasia. Nonulcerogenic conditions without excessive
gastric acid secretion include postvagotomy, postgastric bypass,
pernicious anemia, atrophic gastritis, short gut syndrome after
significant intestinal resection, and renal failure, Helicobacter
pylori infection, VIPoma, and stomach irradiation. Many of
these conditions are associated with achlorohydria, in which
stomach acid production is absent, resulting in hypergastrinemia
and mimicking ZES.

DIAGNOSIS

A BAO greater than 15 mEq/h (>5 mEq/h in patients who have


undergone previous acid-reducing operations) is abnormal and
occurs in 98% of patients with ZES. Measurement of gastric pH
is a simpler, but less accurate, indicator of gastric acid hypersecretion. A gastric pH > 3 essentially excludes ZES, whereas a pH 2
is consistent with ZES.

Postsecretin Challenge
An increased fasting serum gastrin concentration (>100 pg/mL)
and abnormally elevated BAO (>15 mEq/L) establish the diagnosis
of ZES. Many patients with ZES have gastric acid hypersecretion
and minimally increased fasting serum gastrin concentrations
(1001000 pg/mL). For these patients, the secretin stimulation
test is the provocative test of choice. After an overnight fast, secretin is administered intravenously (2 U/kg), and blood samples
are collected immediately before and at 2, 5, 10, and 15 min after
giving the secretin. Secretin normally inhibits gastrin production; however, in the setting of a gastrinoma, secretin produces a
paradoxical increase in gastrin secretion. An increase in gastrin
concentration of 200 pg/mL above baseline is diagnostic of ZES.
The test sensitivity is not 100%, and approximately 15% of patients
with gastrinoma may have a negative secretin test.

Laboratory Studies
Table 14.2 summarizes the laboratory values for the diagnosis of
ZES.

Fasting Serum Gastrin


The evaluation of a patient in whom ZES is suspected begins by
obtaining a fasting serum concentration of gastrin. Hypergastrinemia occurs in almost all patients with ZES and is defined as
a serum gastrin concentration >100 pg/mL. Therefore, a normal
fasting serum gastrin concentration effectively excludes ZES.
Antacid medications like histamine receptor antagonists or proton pump inhibitors may cause a false-positive increase in serum
gastrin concentration, and those medicines should be withheld
for at least 1 week before measurement of the serum gastrin
concentration.

Basal Acid Output

Serum Calcium and Parathyroid Hormone


All patients undergoing an initial workup for ZES should be
screened for hyperparathyroidism associated with MEN1. ZES
was the initial clinical manifestation in 40% of patients with
MEN1, and the onset of ZES symptoms preceded the diagnosis
of hyperparathyroidism in 45% of patients.18 The initial screen
should include a serum calcium measurement, which exploits the
high penetrance of hyperparathyroidism in MEN1. If hypercalcemia is detected, parathyroid hormone levels should be measured
in order to confirm the diagnosis of hyperparathyroidism. Hypercalcemia resulting from hyperparathyroidism can significantly
exacerbate the symptoms of ZES and further elevate serum gastrin levels due to the underlying gastrinoma. As such, the diagnosis of hyperparathyroidism coexistent with ZES may alter the
surgical management of the associated conditions.

LOCALIZATION

Achlorhydria is a common cause of hypergastrinemia, and gastric acid secretion is measured to exclude this condition. A normal adult basal acid output (BAO) is approximately 2 to 5 mEq/h.

The diagnosis of ZES can be achieved by clinical symptoms and


biochemical analysis. However, locating the primary tumor and

Table 14.2 Diagnosis of Gastrinoma


Blood Measurement

Fasting Normal Range

Result with Gastrinoma

Gastrin

<100 pg/mL

>100 pg/mL

99

BAO

2 to 5 mEq/hr

>15 mEq/h
(>5 mEq/h in patients who have
undergone previous acidreducing operations)

98

Fasting gastric acid concentration


Secretin stimulation test

PMPH_CH14.indd 122

Test Sensitivity (%)

pH 2
<200 pg/mL increase of gastrin
concentration above baseline

200 pg/mL increase of gastrin


concentration above baseline

85

5/21/2012 8:51:39 PM

ZollingerEllison Syndrome

123

Table 14.3 Sensitivities of Tumor Localization Modalities


Study

% of Tumors Localized
Overall

Pancreas

Duodenum

Liver Metastases

PREOPERATIVE
Noninvasive
Transabdominal ultrasonography

2030

Abdominal computed tomography

50

Abdominal magnetic resonance imaging

25

Somatostatin receptor scintigraphy

14
80

35

50
83

7190

50

Invasive
Endoscopic ultrasonography

85

Selective arterial secretin injection test

90

75100

2857

INTRAOPERATIVE
Palpation

65

91

60

Intraoperative ultrasonography

83

95

58

Duodenotomy

100

identifying any evidence of metastases is critical to developing


appropriate treatment strategies. Localization of gastrinomas
should begin with noninvasive imaging to assess the extent of
tumor spread and exclude unresectable disease. Invasive modalities can then be used to localize the primary tumor prior to surgery because accurate localization of gastrinomas offers the best
chance at curative resection. The sensitivities of various tumor
localization modalities are summarized in Table 14.3.

Ultrasound
Transabdominal ultrasonography is often the initial imaging
study obtained during the workup of abdominal symptoms. It
is noninvasive, relatively inexpensive, and readily available. On
ultrasound, gastrinomas appear as well-defined, homogeneous
hypo- or iso-echogenic mass lesion. However, transabdominal
ultrasound is of limited use in identifying gastrinomas, as its sensitivity is less than 30%.19
Endoscopic ultrasound (EUS) is an invasive procedure that
combines endoscopy and ultrasound to produce high quality,
detailed, cost-effective, images of the walls of the hollow gastrointestinal tract and adjacent organs during surveillance for
gastrinoma (Fig. 14.7). EUS has a sensitivity of 85% for detecting pancreatic gastrinomas, but only 43% for detecting duodenal
gastrinomas.20 There was great variability in the sensitivities for
EUS localization of pancreatic (75100%) and duodenal (2857%)
tumors among a number of studies reviewed. Whether the differences in sensitivities are due to operator-dependence, patient
populations, or instrumentation is unclear. Nevertheless, the low
sensitivity of EUS for duodenal tumor given the frequency of
small duodenal gastrinomas is a significant limitation.21
Intraoperative ultrasound (IOUS) is very useful for localizing
intrapancreatic gastrinomas. IOUS can localize pancreatic tumors

PMPH_CH14.indd 123

Figure 14.7 Computed tomography scan of gastrinoma.


(A) Duodenal gastrinoma. (B, C) Pancreatic head gastrinoma.
(D) Hepatic metastases.
as small as 5 mm and is therefore a powerful adjunct for detecting
tumors not identified during preoperative imaging.22,23 However,
IOUS is poor at detecting duodenal gastrinomas, 24 and thus is no
substitute for duodenotomy with direct palpation.

Duodenotomy
Duodenotomy is necessary to allow direct inspection and exploration of the duodenal mucosa. A recent prospective study of
patients with sporadic ZES who underwent surgical exploration

5/21/2012 8:51:39 PM

124

Surgery: Evidence-Based Practice

revealed a significantly higher cure rate following duodenotomy,


both immediately and long term.25 Duodenotomy was particularly
important in the detection of small duodenal tumors, allowing
localization of 90% of tumors <1 cm versus only 50% discovered
on preoperative imaging. Duodenotomy is the intraoperative
procedure of choice for detection of gastrinoma and should be
performed in all surgical explorations for gastrinoma. Duodenal
wall gastrinomas occur in greatest density more proximally in
the duodenum. Regional lymph nodes should be systematically
sampled, as lymph node metastases may be inapparent at exploration and will be found in 55% of patients with duodenal tumours.
Gastrinomas can ultimately be found by an experienced surgeon
in nearly all patients.26-28

Computed Tomography
Due to their relative hypervascularity, gastrinomas appear
as hyperattenuating lesions in the arterial phase on contrastenhanced computed tomography (CT) (Fig. 14.8). Overall, abdominal CT detects approximately 50% of gastrinomas. However, the
sensitivity of CT depends greatly on tumor size, tumor location,
and the presence of metastases.29 CT reliably detects gastrinomas
larger than 3 cm in diameter, whereas tumors smaller than 1 cm
in diameter are rarely detected. Intermediate tumors between 1
and 3 cm are identified in 30% of cases. Primary gastrinomas that
arise within the pancreas are identified much more reliably than
those in extrapancreatic, extrahepatic locations (80% vs. 35%). In
addition, CT scanning identifies only 50% of liver metastases.

Magnetic Resonance Imaging


Gastrinomas are relatively hypervascular when compared with
normal intraabdominal tissues, and therefore gastrinomas have
lower signal intensity on T1 imaging and higher signal intensity
on T2 imaging compared with surrounding organs. Although,

Figure 14.8 SPECT/SRS of duodenal gastrinoma.

PMPH_CH14.indd 124

abdominal magnetic resonance imaging (MRI) has a low sensitivity (25%) in localizing primary gastrinomas, it is particularly
useful in detecting hepatic metastases (sensitivity of 83%). Gastrinoma metastases in the liver appear bright with distinct peripheral enhancement on dynamic T2-weighted images. In addition,
MRI is especially useful to differentiate gastrinoma metastases
within the liver from hemangiomas.

Somatostatin Receptor Scintigraphy


Somatostatin receptor scintigraphy (SRS), also called octreoscan,
is the study of choice for localizing both primary and metastatic
gastrinoma (Fig. 14.9). During SRS, patients suspected of harboring gastrinoma are administered 6 mCi of 111In-labeled octreotide
by intravenous injection. The compound identifies gastrinomas
because the vast majority of these tumors avidly express type 2
somatostatin receptors to which octreotide has a high affi nity.
Gamma recorders estimate tracer uptake at particular time points
(usually 4 hours, and if necessary 24 and 48 h) after administration of the radioactive compound.
One prospective study using SRS in 146 patients found a sensitivity of 71%, specificity of 86%, positive predictive value of 85%
and a negative predictive value of 52%.30 With a high pretest probability, as in the setting of ZES, SRS has an overall sensitivity of
approximately 90%, specificity approaching 100%, and positive
predictive value near 100%.30,31 The sensitivity of SRS for gastrinoma exceeds all other imaging modalities combined (angiography, MRI, CT, ultrasonography).
In the study of gastrinoma, SRS altered clinical management
in almost 50% of patients, reflecting the ability of SRS not only
to identify the primary tumor location, but also clarify equivocal
results generated by conventional imaging studies.32 The accuracy
of SRS can be further improved by the application of anatomical imaging by single photon emission computed tomography
(SPECT).33,34 SRS/SPECT enables convenient whole body imaging
to detect the presence of gastrinomas throughout the body.
Like CT, however, the sensitivity of SRS correlates directly
with the size of tumors. SRS was able to detect 96% of gastrinomas
larger than 2.2 cm, but only 30% of tumors smaller than 1.1 cm.

Figure 14.9 Endoscopic ultrasound of extraduodenal gastrinoma.

5/21/2012 8:51:40 PM

ZollingerEllison Syndrome

Probability of survival (percent)

90
80

p = 0.028

Developed liver
metastases
(n = 13)

70
60

Single liver lobe


metastases
(n = 14)

50

125

outside the vascular distribution of the infused vessels. Further,


in patients with nonimaged tumors, SASI always seems to localize the tumor to the gastrinoma triangle limiting its utility.

No liver metastases (n = 158)

100

TREATMENT
Medical Management

p = 0.0004

40

Proton Pump Inhibitors

30
20
Diffuse liver metastases (n = 27)

10
0
0

10

15

20

25

Years since diagnosis

Figure 14.10 Survival of patients with liver metastases from


gastrinoma. By permission.20
SRS had a sensitivity of 64% for intermediate tumors sized
between 1.1 and 2.2 cm. Because duodenal gastrinomas are usually subcentimeter in size (Fig. 14.10), SRS fails to detect about
50% of these tumors.7,20,22
A positive SRS study strongly predicts the presence of tumor,
but the inconsistent negative predictive value (33100%) cautions against excluding a tumor on the basis of a negative study.
SRS has a false-positive localization rate of about 12%. Extraabdominal false-positives localization scans were more common
than intraabdominal false-positive scans and were attributed
to thyroid, breast or granulomatous lung disease. 35 The most
common cause of false-positive intraabdominal SRS scans was
accessory spleens, localization to prior operative sites and renal
parapelvic cysts. Only 2.7% of these false-positive studies actually altered management, suggesting the importance of a high
awareness of other potential causes for a positive SRS scan in the
clinical setting.

The introduction of proton pump inhibitors (PPIs) in the 1980s


has made medical management of gastric acid hypersecretion
possible in patients with ZES. PPIs are the most potent inhibitors of gastric acid secretion and function by targeting the
H-K-ATPase on the apical membrane of parietal cells. PPIs are weak
protonatable pyridines that undergo acid catalyzed conversion to
thiophilic sulfonamides. These permanent cations then form disulfide bonds with cysteine residues within the alpha-subunit of
the H-K-ATPase.37 By irreversibly and covalently altering chemical structures critical to proton transport, PPIs provide potent,
specific, and durable inhibition of gastric acid production.38
Oral omeprazole39 and intravenous pantoprazole must be
dose adjusted in patients with ZES in order to normalize BAO levels to less than 15 mEq/h (less than 5 mEq/h in patients who have
reflux esophagitis or who have had prior operations to reduce acid
secretion, such as subtotal gastrectomy). Measuring BAO after
initiating drug therapy is necessary because relief of symptoms
alone is not a reliable indicator of effective acid control.40,41 Conversely, if acid hypersecretion is controlled, epigastric discomfort
resolves and ulcers heal in virtually all patients.42,43 Because of the
recent advances in the effective medical treatment of ZES, total
gastrectomy is no longer indicated in patients with gastrinoma.

Surgical Management
MEN1 and ZES

Selective Arterial Secret Injection Test

Table 14.4 shows outcomes of surgical management of gastrinomas.

Selective arterial secretin injection test (SASI) evolved from earlier invasive localization techniques, namely selective angiography
and portal venous sampling, with increasing degrees of overall
sensitivity (60% and 80%, respectively). SAIS can demonstrate
the artery supplying a gastrinoma by exploiting the paradoxical
stimulating effect of secretin on gastrinomas to produce gastrin.36
By using catheter directed injection, 30 U of secretin is infused
sequentially into the tributaries of the splenic, gastroduodenal,
and the superior mesenteric arteries. A blood-sampling catheter
is placed in the right hepatic vein to collect blood samples for
gastrin measurement following each injection. At different time
points, the hepatic venous serum immunoreactive gastrin level is
measured. An increase in gastrin greater than 80 pg/mL and more
than 20% above the basal levels is considered a positive result.
By identifying the vessel that supplies the gastrinoma, SASI
localizes the tumor to the vascular distribution attributed to that
particular vessel. SASI can localize gastrinomas smaller than
5 mm in diameter with 90% sensitivity.22 However, SASI is an
invasive procedure, does not directly image the lesion, requires
specialized expertise, and cannot locate occult gastrinomas

Neck Exploration
Hypercalcemia resulting from hyperparathyroidism can significantly exacerbate the symptoms of ZES and further elevate
serum gastrin levels due to the underlying gastrinoma. Thus, in
ZES patients with coexistent hyperparathyroidism, neck exploration for resection of parathyroid hyperplasia should be performed
prior to embarking on removal of gastrinomas. In these patients,
subtotal parathyroidectomy (3 and glands with the cervical
thymus) can significantly decrease end-organ effects of hypergastrinemia allowing for better medical control of ZES symptoms.

PMPH_CH14.indd 125

Gastrinoma Resection
Gastrinomas associated with MEN1 tend to be multiple, small,
usually originate in the duodenum, and frequently develop
lymph node metastases; however, these tumors may be more
indolent than sporadic tumors and may have a more favorable
long-term prognosis.43 In patients with MEN1 and ZES, surgical resection is seldom curative (010%), but may be effective
to prevent or decrease the development of liver metastases.6
Tumor size correlates with progression to liver metastasis, which

5/21/2012 8:51:43 PM

PMPH_CH14.indd 126

151 (151
surgery)

III

Norton et al.
(1999)
(27)

19

21 vs. 26

195 (160
surgery, 35
no surgery)

III

Norton et al.
(2006)
(45)

12

40 (40 surgery)

III

Thompson
(1998)
(58)

100

100

6.9

81 (48 surgerya,
33 no
surgeryb)

II

Norton et al.
(2001)
(53)

100

With
MEN1 (%)

15

18

12 (12 surgery)

III

Mortellaro
et al.
(2009)
(57)

Median
Follow Up
(Years)

Level of
Evidence

8 (7 sporadic
vs. 14
MEN1)

54

43

5 vs. 24

With
Malignancy
(%)

5 vs. 29

NR

93

6 vs. 24

NR

Metastases
(%)

94

NR

NR

92

Tumor
Resected
(%)

51 vs. 16 (45)
immediately
(sporadic vs.
MEN1)
40 vs. 4 at 5 yrs
(sporadic vs.
MEN1)
34 vs. 0 at
10 years
(sporadic vs.
MEN1)

51 immediately,
41 at last
follow-up

68

19 immediately,
0 at 5 yrs

8 at 3 yrs,
0 at last
follow-up

Disease-free
Survival (%)

97 at 5 yrs
94 at 10 yrs
94 at 15 yrs
21 vs. 54

14

1 vs. 23

0 at 5 yrs
(sporadic &
MEN1)
5 vs. 14 at
10 yrs
(sporadic
vs. MEN1)

0 vs. 3 at
10 yrs

33

2 vs. 50

Overall
Mortality
(%)

Diseaserelated
Mortality (%)

Series

Table 14.4 Outcome of Surgical Management of Gastrinoma

126
Surgery: Evidence-Based Practice

5/21/2012 8:51:43 PM

PMPH_CH14.indd 127

17

25

II

II

Jaskowiak
et al.
(1996)
(55)

Kisker et al.
(1998)
(14)

5.2

2.3

16

NR

13

14

100

48

94

44

60

96

100

41

70

NR

NR

NR

NR

44

35

14

NR

0 w/o liver
mets at 5
years vs.
72 w liver
mets

NR

NR

Group 2A (n = 17; single PET 2.56 cm) and group 2B (n = 31; two or more lesions, 2.5 cm in diameter or larger, or one lesion larger than 6 cm) underwent laparotomy.
Group 1 (n = 17) (all PETs smaller than 2.5 cm) and group 3 (n = 8) (diffuse liver metastases) did not undergo surgery.

22

II

McArthur
et al.
(1996)
(54)

10

II

MacFarlane
et al.
(1995)
(56)

NR

12

19 at 10 yrs

ZollingerEllison Syndrome

127

5/21/2012 8:51:43 PM

128

Surgery: Evidence-Based Practice


One or more symptoms related to upper GI tract (diarrhea, epigastric abdominal
pain), PUD (refractory, recurrent, atypical, requiring surgery, without H. pylori)
+/ hyperparathyroidism/nephrolithiasis/family history of MEN1

ZES excluded,
reevaluate 3-6
months

No

Serum gastrin >


100 ng/dl?

No

Yes

Secretin stim test


with serum gastrin >
200 ng/gl?

BAO > 15 mEq/hr


(5 if prior surgery) or
gastric pH < 2?

No

Yes
Yes

Sporatic ZES

Diagnosis of ZES

Hypercalcemia,
hyperparathyroid
?

No

Yes

Familial ZES/MEN1

Treatment with PPI


(titrate to BAO < 10
mEq/hr)

Palliative medical
management

Resection with
adequate margins
and regional lymph
node resection

SRS/SPECT, CT, MRI

No

No
Cytoreductive surgery,
TACE/RFA

Extent of disease
(90% tumor may be
safely removed)?

Yes

Yes

Whipple
pancreaticoduodenectomy

Metastases?

Yes

No

Subtotal
parathyroidectomy

Yes

Familial ZES/MEN1?

Ampullary
involvement?

Yes
No

Localized to
duodenum?

Exploratory laparotomy
and duodenotomy

No
Reimage in 36
months

No

Gastrinoma > 2 cm?

Mobilization of
pancreas with
palpation/IOS

Yes

Yes

Yes
Whipple
pancreaticoduodenectomy

Involving major
ductal/vascular
structures or bulky?

No
Tumor enucleation
with lymph node
resection

Yes

Localized to
pancreatic
head/neck?
No
Distal pancreatectomy
with lymph node
resection

Figure 14.11 Summary algorithm.


in turn is the strongest predictor of survival. Consequently, in
patients who have MEN1, surgery is recommended only if there
is an identifiable tumor greater than 2 cm in size. The operation should include resection of body and tail pancreatic NETs,
enucleation of palpable pancreatic head tumors, duodenotomy
with excision of duodenal tumors, and peripancreatic lymph
node sampling.45 The goal of such an operation is to prevent
liver metastases and thus decrease tumor-related mortality, not
to cure ZES.6 However, some studies have documented cure of
ZES in MEN1 patients by performing Whipple pancreaticoduodenectomy. We do not favor that approach because the longterm survival is excellent with the surgical approach described
above and the morbidity is less.

Sporadic Gastrinomas
All patients with sporadic gastrinoma who do not have unresectable metastatic disease should undergo exploratory laparotomy
for potential cure of ZES.46 A recent study followed 160 ZES
patients for a mean of 12 years and demonstrated that routine surgical removal of gastrinoma increased survival (54% vs. 21%) by
increasing disease-related survival (23% vs. 1%) and decreasing the
development of distant disease (rate of metastasis 29% vs. 5%).47
Surgical exploration and duodenotomy should be performed even
in patients without an identifiable tumor on preoperative imaging,
but with clear biochemical evidence of sporadic ZES because of
the high probability of an occult duodenal gastrinoma.45
Gastrinomas localized to the duodenum may be locally
resected with adequate margins.48 Resection should allow for

PMPH_CH14.indd 128

nonconstricting closure of the remaining duodenum and special


attention is paid to avoid injury to the ampulla of Vater. Regional
lymph nodes should be systematically sampled and excised, as lymph
node metastases may be unapparent at exploration and will be found
in 55% of patients with duodenal tumors. Resection of a single duodenal gastrinoma and regional lymph node dissection resulted in a
cure in 60% of patients with sporadic gastrinoma.22 Multiple duodenal gastrinomas localized to either the upper or the lower aspects
of the duodenum may undergo partial resection of the duodenum.
For patients with large tumors or ampullary involvement, a pyloruspreserving pancreaticoduodenectomy may be indicated.22
Patients with pancreatic disease should undergo mobilization
of the pancreas and palpation with intraoperative ultrasound. For
disease in the body and tail of the pancreas, the patient should
undergo a distal pancreatectomy with lymph node excision. Pancreatic head and neck tumors not involving major ductal or vascular structures are enucleated. For bulky large tumors localized to
the pancreatic head, a pylorus-preserving pancreaticoduodenectomy may be necessary.49

Advanced Disease
Because 60% to 90% of gastrinomas are malignant, management
of advanced disease is a significant problem. At the time of diagnosis of ZES, 25% to 33% of patients have liver metastases5 with
5% to 15% limited to one lobe of the liver.20,50 In these patients,
cytoreductive surgery should be considered if more than 90% of
the visible tumor can be safely removed.5 Other cytoreductive
strategies, such as transarterial chemo-embolization (TACE) and
radiofrequency ablation (RFA), may be performed preoperatively
or in lieu of an operation in the case of liver metastases. Surgery
remains the primary option for patients, as alternative therapies,
including chemotherapy, radiofrequency ablation, transarterial
chemoembolization, biotherapy, polypeptide radionuclide receptor therapy, antiangiogenic therapy, and selective internal radiotherapy, have failed to demonstrate a long-term survival benefit.51

SUMMARY AND RECOMMENDATIONS


In summary, ZES is a syndrome caused by gastrinoma usually
located within the gastrinoma triangle and associated with symptoms of peptic ulcer disease and diarrhea. The diagnosis of ZES is
achieved by measuring fasting levels of serum gastrin, BAO, and
postsecretin challenge testing. Due to the high association of ZES
with MEN1, hyperparathyroidism must be excluded by obtaining a
serum calcium and parathyroid hormone level. Treatment of ZES
consists of medical control of symptoms with PPIs and evaluation
for potentially curative surgical intervention. Noninvasive imaging
studies including SRS, CT, and MRI should be performed initially
to evaluate for metastases and identify resectable disease. Invasive
imaging modalities, such as EUS may be performed to further
evaluate primary tumors. IOUS, palpation, and duodenotomy are
used for intraoperative localization of gastrinomas. In patients with
MEN1, surgical resection should be pursued only if there is an identifiable tumor larger than 2 cm. All patients with resectable sporadic gastrinoma should undergo surgical exploration. In patients
with liver metastases, cytoreductive surgery should be performed if
more than 90% of the visible tumor can be safely removed. Figure
14.11 summarizes the workup, medical management, and surgical
approach to ZES.

5/21/2012 8:51:43 PM

ZollingerEllison Syndrome

129

Clinical Question Summary


Question

Answer

Levels of Grade of
References
Evidence Recommendation

What percent of patients


with MEN1 are
surgically cured?

Surgical cure rate in patients with MEN1 is low


(010%) without pancreaticoduodenectomy.
Some have reported cures, but they did not
demonstrate negative postoperative secretin
stimulation test and normal fasting gastrin
and the follow-up was short. The role of
pancreaticoduodenectomy remains controversial.

II-III

27, 28, 59,


60

What percent of patients


without MEN1 are
surgically cured?

Cure rates are variable due to studies with small


number of patients and short or incomplete
follow-up. The largest analysis involving 123
patients with a mean follow-up of 8 years showed
a postoperative cure rate of 60%, 40% at 5 years,
and 34% at 10 years.

III

27, 61, 62

What is the biological


behavior of duodenal
and pancreatic
gastrinomas?

Both locations are equally malignant. Pancreatic


gastrinomas tend to be larger and have a higher
incidence of liver metastases. Duodenal tumors
are smaller and tend to metastasize to the lymph
nodes. As a result, pancreatic lesions tend to
have a worse prognosis. Gastrinomas pursue a
particularly aggressive course in 24% of cases
that is more common in women and patients
without MEN1; it has shorter disease duration,
higher serum gastrin levels, large pancreatic
tumors, liver metastasis, and a long-term
survival rate of 30% compared with 95% for the
nonaggressive form.

III

16, 27, 50,


63-67

What is the best


noninvasive imaging
modality to localize
tumors?

SRS is more sensitive than all other imaging


modalities combined, the accuracy of which
can be further improved by the application of
anatomical imaging such as CT. CT is usually
the first study performed, but is limited by its
inability to resolve small tumors. MRI has a poor
sensitivity for primary tumors but is useful in
evaluating hepatic lesions.

II

31, 33, 34,


68-70

Can primary gastrinoma


tumors arise from
lymph nodes?

In one series of 138 patients who underwent


exploration for gastrinoma with a mean
follow-up of 10 years, 10% of patients with
sporadic ZES achieved long-term cure after
resection of a lymph node only. The notion
of lymph node primary gastrinoma is further
supported by reports of neuroendocrine cells
within abdominal lymph nodes.

IV

8, 11, 12

What are the most


important predictors of
survival?

The presence and extent of liver metastases are


the most important determinants of survival in
patients with gastrinoma.

17, 50, 63,


71, 72

Should surgical
exploration be
undertaken in patients
without MEN1
with biochemical
evidence of ZES but
without definitive
tumor localization by
preoperative imaging?

All patients with sporadic tumors and biochemical


evidence of gastrinoma should undergo surgical
exploration, even without image localization.
Because small duodenal tumors are the most
common and most difficult to localize on
preoperative imaging, duodenotomy and lymph
node sampling should be performed in all surgical
explorations for gastrinoma. Using this approach,
gastrinomas can ultimately be found by an
experienced surgeon in nearly all patients.

III

26-28, 46,
47

(Continued)

PMPH_CH14.indd 129

5/21/2012 8:51:44 PM

130

Surgery: Evidence-Based Practice

(Continued)
Question

Answer

When should surgery be


performed in patients
with ZES and MEN1?

In patients with MEN1, surgery is recommended


only if there is an identifiable tumor greater
than 2 cm in size, as even with advanced disease,
long-term survival is common. The goal of
such an operation is not to affect a cure, since
surgery is seldom curative in these patients, but
rather to ameliorate symptoms and prevent liver
metastases.

II-III

27, 45, 73

Should surgical
interventions be
attempted in patients
with liver metastases?

Cytoreductive surgery should be considered if


the tumor could be safely removed. Other
cytoreductive strategies, such as transarterial
chemo-embolization (TACE) and radiofrequency
ablation (RFA), may be performed preoperatively.
TACE, RFA, polypeptide radionuclide receptor
therapy, antiangiogenic therapy, and selective
internal radiotherapy may be performed in cases
with unresectable disease in order to relieve
symptoms, but they have failed to consistently
demonstrate a long-term survival benefit. Liver
transplantation may be considered in selected
patients.

III-IV

5, 51, 71,
72, 74

REFERENCES
1. Zollinger RM, Ellison EH. Primary peptic ulceration of the jejunum associated with islet cell tumors of the pancreas. Ann Surg.
1955;142:708-728.
2. Watson SA, Grabowska AM, El-Zaatari M, Takhar A. Gastrin
active participant. Nat Rev Cancer. 2006;6(12):936-946. Review.
3. Yao X, Forte JG. Cell biology of acid secretion by the parietal cell.
Annual Rev Physiol. 2003;65:103-131. Review.
4. Eriksson B, Oberg K, Skogseid B. Neuro-endocrine pancreatic
tumors: Clinical findings in a prospective study of 84 patients.
Acta Oncol. 1989;28:373-377.
5. Gibril F, Jensen RT. Advances in evaluation and management of
gastrinoma in patients with ZollingerEllison syndrome. Curr
Gastroenterol Rep. 2005;7(2):114-121.
6. Cisco RM, Norton JA. Surgery for gastrinoma. Adv Surg.
2007;41:165-176.
7. Bh M, Gotthardt M, Behr TM. Imaging of gastrinomas by
nuclear medicine methods. Wien Klin Wochenschr. 2007;119(1920):593-596. Review.
8. Norton JA, Alexander HR, Fraker DL, Venzon DJ, Gibril F, Jensen
RT. Possible primary lymph node gastrinoma: Occurrence, natural history, and predictive factors: A prospective study. Ann Surg.
2003;237(5):650-657; discussion 657-659.
9. Norton, JA. Advances in the management of ZollingerEllison
syndrome. Adv Surg. 1994;27:129-159.
10. Wolfe MM, Alexander RW, McGuigan JE. Extrapancreatic,
extraintestinal gastrinoma: Effective treatment by surgery. N
Engl J Med. 1982;306:1533-1536.
11. Perrier ND, Batts KP, Thompson GB, et al. An immunohistochemical survey for neuro-endocrine cells in regional pancreatic lymph nodes a plausible explanation for primary nodal
gastrinomas. Surgery. 1995;118:957-965.

PMPH_CH14.indd 130

Levels of Grade of
References
Evidence Recommendation

12. Herrmann ME, Ciesla MC, Chejfec G, DeJong SA, Yong SL. Primary
nodal gastrinomas. Arch Pathol Lab Med. 2000;124(6):832-835.
13. Peplinski GR, Norton JA. Gastrointestinal endocrine cancers
and nodal metastases: biological significance and therapeutic
implications. Surg Oncol Clin North Am. 1996;5:159-171.
14. Kisker O, Bastian D, Bartsch D, et al. Localization, malignant
potential, and surgical management of gastrinomas. World J
Surg. 1998;22:651-658.
15. Ellison, EC. Forty year appraisal of gastrinoma: back to the
future. Ann Surg. 1995;222:11-21.
16. Sutliff VE, Doppman JL, Gibril F, et al. Growth of newly diagnosed, untreated metastatic gastrinomas and predictors of
growth patterns. J Clin Oncol. 1997;15:2420-2431.
17. Norton, JA. Endocrine tumours of the gastrointestinal tract.
Surgical treatment of neuroendocrine metastases. Best Pract Res
Clin Gastroenterol. 2005;19(4):577-583.
18. Gibril F, Schumann M, Pace A, Jensen RT. Multiple endocrine
neoplasia type 1. Medicine. 2004;83(1):43-83. Review.
19. Frucht H, Doppman JL, Norton JA, et al. Gastrinomas: Comparison of MR imaging with CT, angiography, and US. Radiology.
1989;171(3):713-717.
20. Norton JA, Jensen RT. Resolved and unresolved controversies in
the surgical management of patients with ZollingerEllison syndrome. Ann Surg. 2004;240(5):757-773. Review.
21. Rsch T, Lightdale CJ, Botet JF, et al. Localization of pancreatic
endocrine tumors by endoscopic ultrasonography. N Engl J Med.
1992;326(26):1721-1726.
22. Imamura M, Komoto I, Ota S. Changing treatment strategy for
gastrinoma in patients with ZollingerEllison syndrome. World
J Surg. 2006;30(1):1-11. Review.
23. Norton JA, Cromack DT, Shawker TH, et al. Intraoperative
ultrasonographic localization of islet cell tumors. A prospective
comparison to palpation. Ann Surg. 1988;207(2):160-168.

5/21/2012 8:51:44 PM

ZollingerEllison Syndrome

24. Norton, JA. Intra-operative procedures to localize endocrine


tumours of the pancreas and duodenum. Ital J Gastroenterol
Hepatol. 1999;31(Suppl 2):S195-S197.
25. von Eyben FE, Grodum E, Gjessing HJ, et al. Metabolic remission with octreotide in patients with insulinoma. J Intern Med.
1994;235:245-248.
26. Thompson NW, Vinik AI, Eckhauser FE. Micro-gastrinomas of
the duodenum. Ann Surg. 1989;209:396-404.
27. Norton JA, Fraker DL, Alexander HR, et al. Surgery to cure the
ZollingerEllison syndrome. N Engl J Med. 1999;341:635-644.
28. Howard TJ, Zinner MJ, Stabile BE, et al. Gastrinoma excision for
cure. Ann Surg. 1990;211:9-14.
29. Wank SA, Doppman HL, Miller DL, et al. Prospective study of
the ability of computerized axial tomography to localize gastrinomas in patients with ZollingerEllison syndrome. Gastroenterology. 1987;92:905-912.
30. Alexander HR, Fraker DL, Norton JA, et al. Prospective study
of somatostatin receptor scintigraphy and its effect on operative
outcome in patients with ZollingerEllison syndrome. Ann Surg.
1998;228(2):228-238.
31. Gibril F, Reynolds JC, Doppman JL, et al. Somatostatin receptor
scintigraphy: Its sensitivity compared with that of other imaging
methods in detecting primary and metastatic gastrinomas. A prospective study [see comments]. Ann Int Med. 1996;125(1):26-34.
32. Modlin IM, Tang LH. Approaches to the diagnosis of gut neuroendocrine tumors: The last word (today). Gastroenterology.
1997;112(2):583-590.
33. Schillaci O, Corleto VD, Annibale B, Scopinaro F, Delle Fave
G. Single photon emission computed tomography procedure
improves accuracy of somatostatin receptor scintigraphy in
gastro-entero pancreatic tumours. Ital J Gastroenterol Hepatol.
1999;31(Suppl 2):S186-S189.
34. Krausz Y, Keidar Z, Kogan I, et al. SPECT/CT hybrid imaging with 111In-pentetreotide in assessment of neuroendocrine
tumours. Clin Endocrinol (Oxf). 2003;59(5):565-573.
35. Meko JB, Doherty GM, Siegel BA, et al. Evaluation of somatostatinreceptor scintigraphy for detecting neuroendocrine tumors. Surgery. 1996;120:975-984.
36. Imamura M, Takahashi K, Adachi H, et al. Usefulness of selective arterial secretin injection test for localization of gastrinoma
in ZollingerEllison syndrome. Ann Surg. 1987;205:230-239.
37. Shin JM, Cho YM, Sachs G. Chemistry of covalent inhibition of
the gastric (H+, K+)-ATPase by proton pump inhibitors. J Am
Chem Soc. 2004;126(25):7800-7811.
38. Wolfe, MM, Sachs, G. Acid suppression: Optimizing therapy for
gastroduodenal ulcer healing, gastroesophageal reflux disease, and
stress-related erosive syndrome. Gastroenterology. 2000;118:S9.
39. Maton PN, Vinayek R, Frucht R, et al. Long-term efficacy and
safety of omeprazole in patients with ZollingerEllison syndrome:
A prospective study. Gastroenterology. 1989;97(4)827-836.
40. Norton, JA. Gastrinoma: Advances in localization and treatments. Surg Oncol Clin N Am. 1998;7(4):845-861.
41. Maton PN, Frucht H, Vinayek R, et al. Medical management
of patients with ZollingerEllison syndrome. Gastroenterology.
1988;94:294-299.

PMPH_CH14.indd 131

131

42. Fox PS, Hofmann JW, DeCosse JJ, et al. The influence of total
gastrectomy on survival in malignant ZollingerEllison tumors.
Ann Surg. 1974;180:558-566.
43. Zollinger RM, Ellison EC, ODorsio TM, et al. Thirty years
experience with gastrinoma. World J Surg. 1984;8:427-435.
44. Li ML, Norton JA. Gastrinoma. Curr Treat Options Oncol.
2001;2(4):337-346.
45. Norton JA, Fang TD, Jensen RT. Surgery for gastrinoma and
insulinoma in multiple endocrine neoplasia type 1. J Natl Compr
Canc Netw. 2006;4(2):148-153.
46. Meko JB, Norton JA. Management of patients with Zollinger
Ellison syndrome. Annual Rev Med. 1995;46:395-411.
47. Norton JA, Fraker DL, Alexander HR, et al. Surgery increases
survival in patients with gastrinoma. Ann Surg. 2006;244:3:
410-419.
48. Frucht H, Norton JA, London JF, et al. Detection of duodenal
gastrinomas by operative endoscopic transillumination, a prospective study. Gastroenterology. 1999;90:1622-1627.
49. Brandi ML, Gagel RF, Angeli A, et al. Consensus guidelines for
diagnosis and therapy of MEN type 1 and type 2. J Clin Endocrinol Metab. 2001;86:5658-5671.
50. Yu F, Venzon DJ, Serrano J, et al. Prospective study of the clinical course, prognostic factors, causes of death, and survival in
patients with long-standing ZollingerEllison syndrome. J Clin
Oncol. 1999;17(2):615-630.
51. Ehehalt F, Saeger HD, Schmidt CM, Grtzmann R. Neuroendocrine tumors of the pancreas. Oncologist. 2009;14(5):456-467.
52. Pipeleers-Marichal M, Somers G, Willems G, et al. Gastrinomas
in the duodenums of patients with multiple endocrine neoplasia
type 1. N Engl J Med. 1990;322(11):723-727.
53. Mortellaro VE, Hochwald SN, McGuigan JE, Copeland EM,
Vogel SB, Grobmyer SR. Long-term results of a selective surgical approach to management of ZollingerEllison syndrome in
patients with MEN1. Am Surg. 2009;75(8):730-733.
54. Norton JA, Alexander HR, Fraker DL, Venzon DJ, Gibril F, Jensen
RT. Comparison of surgical results in patients with advanced
and limited disease with multiple endocrine neoplasia type 1 and
ZollingerEllison syndrome. Ann Surg. 2001;234(4):495-505.
55. Thompson NW. Current concepts in the surgical management
of multiple endocrine neoplasia type 1 pancreatic-duodenal
disease. Results in the treatment of 40 patients with Zollinger
Ellison syndrome, hypoglycaemia or both. J Intern Med. 1998;
243(6):495-500.
56. MacFarlane MP, Fraker DL, Alexander HR, Norton JA, Lubensky
I, Jensen RT. Prospective study of surgical resection of duodenal
and pancreatic gastrinomas in multiple endocrine neoplasia type
1. Surgery. 1995;118(6):973-979; discussion 979-980.
57. McArthur KE, Richardson CT, Barnett CC, et al. Laparotomy
and proximal gastric vagotomy in ZollingerEllison syndrome:
results of a 16-year prospective study. Am J Gastroenterol. 1996;
91(6):1104-1111.
58. Jaskowiak NT, Fraker DL, Alexander HR, Norton JA, Doppman
JL, Jensen RT. Is reoperation for gastrinoma excision indicated
in ZollingerEllison syndrome? Surgery. 1996;120(6):1055-1062;
discussion 1062-1063.

5/21/2012 8:51:44 PM

CHAPTER 15

Evidence-Based Bariatric Surgery


Amir H. Shariff and Robert L. Bell

controls.11 Decreased mortality in the surgical groups was primarily due to fewer deaths from cardiovascular disease (especially
myocardial infarction [MI]) and cancer. A study of 7925 gastric
bypass patients in Utah12 similarly showed significant reductions
in mortality linked to fewer deaths from coronary artery disease,
diabetes, and cancer.
As WLS continues to increase, patients are getting both older
and heavier; however, at the same time, the hospital length of
stay is becoming shorter and the laparoscopic approach is almost
exclusively being used.13,14 Since the early 2000s, WLS has been
integrated into accredited general surgical training programs in
the United States; this has helped shorten the learning curve for
laparoscopic procedures.15

1. What is the magnitude and impact of the global obesity


epidemic?
Obesity is an increasingly serious health problem globally and in
the Western world in particular. The World Health Organization
(WHO) estimated that globally in 2008 approximately 1.5 billion
adults (age 20 years) were overweight; of these, more than
200 million men and nearly 300 million women were obese. WHO
further projects that by 2015, approximately 2.3 billion adults will
be overweight and more than 700 million will be obese. Nearly
43 million children under the age of 5 were overweight globally in
2010. Once considered a problem only in high-income countries,
overweight and obesity are now dramatically on the rise in low- and
middle-income countries, particularly in urban settings.1
The definition and classification of obesity is based primarily on the body mass index (BMI), calculated as weight divided by
the square of height with kilograms per square meter as the unit
of measurement. WHO classifies a BMI of greater than 25 kg/m2
as overweight with obesity being a BMI of greater than 30 kg/m2.
Obesity is more specifically defined as a BMI of 30 to 35 kg/m2 falling in the category of class I obesity, a BMI of 35 to 40 kg/m2 as class
II and a BMI of over 40 kg/m2 as class III obesity.2
Obesity is associated with premature mortality and other
adverse health consequences. It has been estimated that obesityrelated illnesses are responsible for an estimated 3% to 6% of total
health care costs.3,4 Most of these related illnesses can be improved
or reversed with weight loss.5
It is these increases in the prevalence of severe obesity that
have continued to fuel demand for weight loss surgery (WLS).
Accordingly, the number of bariatric operations being performed
is increasing every year, with a 22-fold increase from 1996 to
2008.6 Controlled trials and observational studies demonstrate
that WLS produces significant and sustained weight loss compared with alternative forms of treatment.7,8 The mortality rates
of severely obese patients who underwent WLS has been found
to be lower than those of severely obese patients who had not.9,10
Landmark findings from the Swedish Obese Subjects study show
an estimated 28% reduction in the adjusted overall mortality
rate in the surgical groups compared with conventionally treated

2. What are the indications for WLS?


Traditionally, obesity surgery is considered appropriate for adult
patients with a BMI greater than 40 kg/m2, with or without comorbidites, or a BMI between 35 and 40 kg/m2 with obesity-related
comorbidity. These selection criteria were suggested in March
1991 by the National Institutes of Health Consensus Development
Panel16-18 and have been subsequently adopted by all major surgical
and nonsurgical societies. In the near future, the selection criteria
are likely to expand to include patients with a BMI between 25 and
35 kg/m2, with obesity-related comorbidities, though this remains
to be validated. In a prospective randomized trial by Dixon
et al.,19 an operative method, gastric banding, was compared with
a conservative therapy (encouragement to lifestyle changes plus,
if accepted, hypocaloric diet) in diabetic patients with a BMI of
30 to 40 kg/m2. The excessive weight loss was 62.5% in 2 years after
gastric banding compared with 4.3% in the conservative control,
and the remission rate of diabetes was 73% after gastric banding
compared with 13% with conservative therapy.
A complex issue in the National Institutes of Health (NIH)
selection criteria is the proper definition of comorbidities, which
warrant WLS due to their seriousness and potential alleviation
through weight loss. Comorbidities may be divided into medical,
physical, and psychological categories. In this respect, medical conditions such as sleep apnea and other hypoventilation
132

PMPH_CH15.indd 132

5/21/2012 8:52:38 PM

Evidence-Based Bariatric Surgery

syndromes,20 type II diabetes mellitus,21 obesity-related cardiomyopathy and hypertension,22 hyperlipidemia,23 asthma,23 pseudotumor cerebri,24 knee osteoarthritis,25 low-back pain,26 female
urinary incontinence,25 and infertility27 are well-documented
indications for WLS.
Additionally, physical, social, and psychological problems are
important factors in the quality of life of obese persons and play a
leading role in deciding on conservative or surgical treatment of
obesity. Studies have shown improvements in quality of life indicators following WLS compared with nonsurgical controls.28,29
Thus, deliberation on WLS options must incorporate an assessment of the patients current physical, social, and psychological
status as well as the expected effects of therapy on these indices.
Prior to considering WLS, it is advisable that all patients have
tried alternative methods of weight loss. The following indicate
components of conservative methods for weight loss:30

Nutrition: Energy-reduced diet in addition to another


nutritional intervention (such as a liquid diet or other energyreduced form of a mixed diet).
Physical exercise: Implementation of a cardiovascular and/or
strength endurance sport with at least 2 h per week, if no barriers exist (such as osteoarthritis of the knee for walking).
Psychotherapy: Implementation of an inpatient or outpatient
psychotherapy (behavioral therapy or depth psychology) in
cases of an eating disorder (binge-eating, night-eating) or psychopathology (e.g., depression, anxiety).
Group therapy: Modification of life style should be conducted
and supervised within a group, if possible.

Surgical therapy should be considered not only in the setting of


the failure of conservative multimodal therapy, but also in the
case of futility of efforts. This can occur when the chances of success of conservative therapy appear remote due to the nature and/
or severity of the disease or due to psychosocial factors. Primary
surgery may also be indicated if radical weight loss should not be
postponed for medical reasons.
Special consideration is given to women of reproductive age
and obese adolescents. Women who wish to have children after
WLS should not be denied an operation, because the course of
pregnancy and the health of the baby are usually unaffected by
previous obesity surgery.27,31-33 Furthermore, the risks of pregnancy
after WLS are far less than the risks of pregnancy in the setting
of uncontrolled obesity.34 Patients are generally advised to avoid
getting pregnant within 1 year after WLS, and once pregnancy
is confirmed, special consideration must be given to nutritional
counseling. In addition to primary obstetrical care, WLS patients
should have continued follow-up with the bariatric surgical team
during the pregnancy.
As previously stated, the prevalence of childhood obesity is
steadily rising. Careful consideration should be given before offering WLS to adolescents who have yet to reach skeletal maturity.
However, recent studies on adolescents (1219 years old) suggest
that surgery in this age group is as effective as in adults.35-37
Various contraindications must also be taken into account.
As patients noncompliance with follow-up schedules can lead
to potentially life-threatening complications, all candidates for
obesity surgery must hold a realistic view of the operation and
the necessity for lifelong aftercare. Severe mental or cognitive
retardation and malignant hyperphagia are therefore generally
considered absolute contraindications, because such patients will

PMPH_CH15.indd 133

133

be unable to eat or exercise as required postoperatively.38 That


being stated, minor and major mental and personality disorders
are highly prevalent in the severely obese patients.39 Surgery is
contraindicated only in the cases of severe mental disease not
responding to treatment.
3. What factors must be considered prior to WLS?
Some patients may pursue WLS without fully understanding the
procedures or their implications. Preoperative teaching by a multidisciplinary team improves patient selection, and enables patients
to choose the surgical procedure most appropriate for them.
However, Orth et al. have shown that preoperative knowledge, as
assessed by a test, did not predict success after laparoscopic gastric
bypass surgery.40 Therefore, patients who do not, at first, have full
knowledge of bariatric surgery should not be discriminated against
undergoing surgery if they are eventually properly educated.
Gupta et al.41 used the American College of Surgeons National
Surgical Quality Improvement Program (NSQIP) dataset to study
patients undergoing WLS. In 11,023 patients, they found the
30-day morbidity and mortality to be 4.2% and 0.19%, respectively.
This mortality rate compares very well with commonly performed
operations, such as aortic aneurysms (mortality 3.9%), coronary
artery bypass graft (3.5%), craniotomy (10.7%), and pancreatectomy
(8.3%).42 The incidence of specific complications including superficial incisional infection (1.8%), sepsis (0.7%), pneumonia (0.6%),
reintubation (0.5%), failure to wean from ventilator (0.4%), renal
insufficiency (0.2%), and MI (<0.1%) was very low given the degree
of illness that accompanies these patients. Risk factors associated
with increased risk of postoperative morbidity included recent MI/
angina, dependent functional status, stroke, bleeding disorder,
hypertension, BMI, and type of bariatric surgery. Interestingly,
patients with a BMI of 35 to <45 and >60 kg/m2 had significantly
higher risks compared with patients with BMI of 45 to 60 kg/m2.
Although at first these findings seem counterintuitive, they tend to
match the patients who have the most weight-related comorbidities.
Patients with the lower BMI cannot have the surgery unless they
have a life-threatening comorbidity such as diabetes, hypertension,
cardiac failure, or sleep apnea while patients with the higher BMI
(>60 kg/m2) more likely to have weight-related comorbidities.
In view of these data, there must be an individual assessment
of risk factors relative to potential reduction in long-term disease risk and burden. For instance, though coronary artery disease remains a risk factor for morbidity and mortality after WLS,
this risk factor reduces significantly after weight reduction from
WLS.43,44 Patients with a history of coronary artery disease should
undergo preoperative assessment of their cardiovascular status
and undergo medical optimization and stabilization as indicated.
Smoking has been shown in studies to be a risk factor for postoperative complications, for example, thromboembolic and pulmonary complications.45 Smoking also significantly increases the risk
of postoperative marginal ulceration.46,47 Therefore, weight loss
surgeons should strongly encourage their patients to quit smoking
prior to surgical intervention.
Patients referred for bariatric surgery are more likely than
the overall population to have psychopathology such as somatization, social phobia, obsessivecompulsive disorder, substance
abuse/dependency, binge-eating disorder, posttraumatic stress
disorder, generalized anxiety disorder, and depression.48 Patients
with poorly controlled psychiatric disorders may have a suboptimal outcome after bariatric surgery. However, no consensus

5/21/2012 8:52:38 PM

134

Surgery: Evidence-Based Practice

recommendations exist regarding preoperative psychological


evaluation.49 Approximately 88% of US bariatric programs use
some form of preoperative psychological evaluation, with half
requiring a formal standardized assessment.49 Many insurance
companies require such psychological evaluations before granting precertification for a bariatric procedure. Nevertheless, the
bulk of evidence shows no relationship between preexisting axis
1 psychiatric diagnoses or axis 2 personality disorders and postoperative weight loss.49,50
The nutrition professional is an integral part of multidisciplinary bariatric care.51 They are charged with the preoperative
nutritional assessment and weight loss efforts as well as postoperative diet education and eating behaviors.52 Despite the wide use of
preoperative nutritional education and the requirement by many
insurance companies for dietary counseling, data still are needed
to prove an association with postoperative weight loss or dietary
compliance.53
The management of cholelithiasis at the time of WLS remains
controversial with some advocating routine cholecystectomy during WLS and others advocating cholecystectomy only if the patient
is symptomatic.54 Performing a cholecystectomy at the time of WLS
has not been shown to significantly increase the operative times,
morbidity or length of stay.55 In postoperative patients with no
prior history of cholelithiasis, the overall rate of symptomatic gallstone formation was found to be approximately 8% and the mean
time for its development was 10.2 (range 237) months. Logistic
regression analysis showed that only postoperative weight loss of
more than 25% of original weight was associated with symptomatic gallstones formation.56
Immediate preoperative considerations include the use of
antibiotic prophylaxis. This was first studied by Pories et al.57 who
observed a reduction in wound infections in gastric bypass patients
who received cefazolin compared with those who received a placebo, thus making antibiotic administration routine. Prophylaxis
of thromboembolic complications is also an essential part of WLS.
The use of low molecular weight heparin and intermittent pneumatic compression stockings at the time of surgery has reduced
the incidence of thromboembolic events to near 0%.58
4. What are the most common bariatric surgery procedures?
There are several competing aims in WLS, including weight loss,
adjustability, reversibility, and safety. WLS reduces caloric intake
by modifying the anatomy of the gastrointestinal tract via restriction, malabsorption or a combination of the two techniques. Ensuing changes in the gutbrain axis alter peptides that may regulate
appetite and satiety (e.g., ghrelin, glucagon-like peptide-1, and
peptide YY3-36).59 For all types of surgery, there is overwhelming
evidence on safety, efficacy, and effectiveness in terms of weight
loss. Therefore, the decision about the specific type of WLS must
be taken with the patients individual situation and the surgeons
expertise in mind. Figure 15.1 demonstrates the most commonly
performed WLS procedures.

JEJUNOILEAL BYPASS
The jejunoileal bypass (JIB) arose as the forerunner to modern
bariatric surgical techniques in the late 1950s.60 At that time, it
was the most effective surgical intervention for achieving and
maintaining weight loss. Several variations existed, but typically,

PMPH_CH15.indd 134

(a)

(b)

(c)

(d)

Figure 15.1 Weight loss surgical procedures: (a) Roux-en-Y


gastric bypass; (b) adjustable gastric band; (c) sleeve gastrectomy;
and (d) biliopancreatic diversion.
35 centimeters of proximal jejunum was anastomosed, end-toside or end-to-end, to the terminal 10 cm of ileum. The weight loss
would be accomplished through malabsorption.61
However, JIB was associated with severe complications such
as renal failure (37%), diarrhea (29%) and consequent electrolyte
imbalances, calcium oxalate nephrolithiasis (29%), liver disease
(10%), acute fulminant liver failure (7%), fat-soluble vitamin deficiencies, malnutrition, and death.62 It is now well established that
JIB is devastating because of its dramatic complications, and is no
longer used for the management of morbid obesity. Most patients
who underwent this procedure have either died or have undergone
a conversion to a different bariatric procedure.

LAPAROSCOPIC ROUX-Y
GASTRIC BYPASS
The Roux-Y gastric bypass (RYGB) was first described by Mason
and Ito63,64 in 1967 and laparoscopic gastric bypass (LRYGB) is currently the most commonly performed WLS in the United States.
LRYGB involves the creation of a small (2030 mL) gastric pouch,
which is anastomosed to a 60 to 150 cm Roux limb (Figure 15.1a).
In this way, a portion of the alimentary tract is rerouted to bypass
the distal stomach and the proximal small bowel. Typically, the
small bowel is transected just distal to the ligament of Treitz and
the Roux limb is elevated and anastomosed to the gastric pouch
in an antecolic, antegastric fashion using a linear or circular stapler.65 The proximal bowel segment, also called the biliopancreatic
limb, usually is connected to the alimentary limb 60 to 150 cm
distal to the gastrojejunostomy and this jejuno-jejunostomy is
constructed using a linear stapler. The ensuing mesenteric defect
at the jejuno-jejunostomy and the Petersens defect (between the
transverse mesocolon and the Roux limb) are closed to prevent
internal herniation.66,67
Modifications to the LRYGB can be performed, the so-called
long limb or distal gastric bypass variations, where the length
of the Roux limb is increased to enhance malabsorption. The Roux
limb can be lengthened up to 300 cm, creating a common channel
of only about 100 cm. In addition, the gastric pouch can be made
smaller. A recent study compared patients who had undergone
LRYGB with a 150-cm Roux limb to patients who underwent a
distal gastric bypass. Although the distal gastric bypass patients
had a significantly greater percentage of excess weight loss after
5 years, they also had significantly lower albumin, hemoglobin,
iron, and calcium levels.68 Thus, although these variations afford
superior long-term weight loss, they do cause protein-calorie

5/21/2012 8:52:38 PM

Evidence-Based Bariatric Surgery

malnutrition requiring frequent revision, and should not be the


primary operation for obese or super-obese patients.
Close, long-term follow-up is recommended after LRYGB.
A typical follow-up schedule after LRYGB would be evaluation at
1 to 3 weeks, followed by quarterly visits during the 1st year and
annually thereafter to assess weight loss, resolution of comorbidities, long-term complications, and need for continuing education
and support. Patients are counseled to eat small, frequent meals
of high-protein and low-carbohydrate content. They should take
long-term vitamin supplements (multivitamins, vitamin B12, and
calcium, with some patients requiring iron supplementation) and
undergo periodic blood testing to identify and treat deficiencies
accordingly. Patients should be encouraged to develop regular
exercise practices.69
The mortality rate after LRYGB ranges from 0.3% to 1%.7 The
most frequently reported perioperative complications associated
with laparoscopic RGB are wound infection (2.98%), anastomotic
leak (2.05%), gastrointestinal tract hemorrhage (1.93%), bowel
obstruction (1.73%), and pulmonary embolus (0.41%), whereas the
most frequently reported late complications are stomal stenosis
(4.73%), bowel obstruction (3.15%), and incisional hernia (0.47%).70
Anastomotic leak is probably the most feared of all the early
postoperative complications. An anastomotic leak occurs in 1%
to 2% of patients after RYGB and usually becomes evident in
the first 2 weeks postoperatively.71 The leak can occur at the site
of the gastrojejunostomy, the staple line of the bypassed gastric
remnant, the staple line of the gastric pouch itself or the jejunojejunostomy. Tachycardia, dyspnea, abdominal pain, peritonitis,
or unexplained oliguria can be the predominant symptoms of an
anastomotic leak. These patients will most likely require surgical
intervention.
The possibility of intestinal obstruction is the second, major
postoperative concern. Patients can present with an obstruction
of any of the three limbs of the LRYGB.72,73 Potential origins for
obstruction include internal hernias, external hernias (abdominal wall) associated with the incision or trocar site, intestinointestinal intussusception (albeit rare), acute gastric distension or
iatrogenic problems related to surgical error in the formation of
the Roux limbs or anastomoses.
The LRYGB has established long-term effectiveness for sustained weight loss, reduction of comorbidities, and a low risk for
long-term nutritional sequelae in comparison with other options
for WLS.74,75

LAPAROSCOPIC ADJUSTABLE
GASTRIC BANDING
Laparoscopic adjustable gastric banding (LAGB) is the most common restrictive WLS performed globally69 (Figure 15.1b). The
original technique was described by Belachew76 and by Favretti.77
It involves no bowel transaction, is reversible, and has lower operative mortality and morbidity compared with combination weight
loss procedures.78 In systematic reviews, mortality has occurred in
1 in 2000 to 1 in 3000 patients.7,79
Placement of the gastric band via the pars flaccida approach
has become the consensus choice for better handling of instruments and band, low complexity in dissection maneuvers, and low
complication rate.80 Dissection starts near the angle of His above
the greater curvature of the stomach. Then the lesser omentum

PMPH_CH15.indd 135

135

is opened through the pars flaccida component to provide exposure of the right crus of the diaphragm. A point along the anterior
border of this muscle, at its lowermost aspect, is selected, and the
peritoneum is opened. The band placer is then passed along this
path until it appears on the greater curvature of the stomach at the
site of prior dissection at the angle of His. Using the band placer,
the band is drawn along this pathway and then locked around the
stomach. A subcutaneous reservoir, or port, is connected to the
band tubing, which allows adjustability of the band circumference. The port is usually secured to the anterior rectus fascia.
All patients undergo follow-up following LAGB to optimize
weight loss, detect postoperative complications, and perform band
volume adjustment. In the early days of LAGB, band fills were
performed almost always under radiologic guidance.81 This was
necessary as the exact volume requirements were unknown and
the mechanisms of band restriction less understood.82 However,
as experience has increased with both the duration of follow-up
and the number of patients, decisions on the quantity of band fi lls
can be safely made clinically.
Long-term complications after LAGB include band slippage,
gastric pouch dilatation, and gastric erosion. Rates of slippage in
the literature have ranged from less than 1% to over 20%. Posterior slippage of the band is almost exclusively seen in those
patients whose gastric band was placed using the perigastric
technique. Opening of the lesser sac during this technique is
thought to be the predominant factor predisposing to posterior
slip. This complication has become far less common following the
introduction of the pars flaccida technique.80 Anterior slippage of
the band is more controversial. It has been postulated that inadequate anterior fi xation of the band is a major etiological factor.83
The technique employed for anterior fi xation is highly variable,
with the majority of proponents for fi xation using between two
and five interrupted gastrogastric imbrication sutures to create an anterior wrap, or gastrogastric tunnel around the band.
Variations on this include the use of a continuous running suture,
mesh pledgets to reinforce suture fi xations, and the use of fundocrural suture fi xation.84-86 Despite these measures, there are
reports of similar slippage rates with and without the use of imbrication sutures.87 Other factors potentially associated with slippage
include size of the gastric pouch, premature band inflation, hiatus hernia, and recurrent vomiting.88,89 Conservative treatment
such as band deflation is certain to fail in a definitive slip. Surgical options for slippage include reduction of the gastric prolapse
with refastening of the primary band, band repositioning, band
removal and replacement with a new band either immediately or
at a subsequent operation, and band removal with conversion to
an alternative bariatric procedure.90
Gastric pouch dilation is often confused with slippage, but
can be differentiated radiologically. If the band has not slipped,
the band remains in its normal oblique lie. Gastric pouch dilation
is thought to be a result of the high pressures generated within the
proximal gastric pouch from excessive or inappropriate eating.91
Patients often report a lack of restriction, and present seeking further fi lls when paradoxically, band deflation is indicated.92 Once
pouch dilatation has resolved, the band can then be reinflated in
the future. If pouch dilation persists, band repositioning or conversion to other procedures may be necessary.
Intragastric erosion of the gastric band is reported with a
frequency of 0.5% to 3.8%.93 Different hypotheses have been suggested to explain this complication: (1) damage of the gastric wall

5/21/2012 8:52:38 PM

136

Surgery: Evidence-Based Practice

during band implantation;94 (2) infection of the band-site;95 (3)


overfi lling the band;96 and (4) abnormal reaction of the periprosthesic tissue to the presence of the band.96 The gastric band must
then be removed, typically via laparoscopy. However, an endoscopic approach has been used, especially when the band has
nearly completely migrated into the stomach.93,97

LAPAROSCOPIC SLEEVE GASTRECTOMY


Laparoscopic sleeve gastrectomy (LSG) originally proved to be a
beneficial procedure for interval weight loss as the first stage of a
two-staged bypass procedure98 (Figure 15.1c). More recently, LSG
is showing promise as a primary bariatric procedure for appropriate candidates.99,100 Currently, LSG comprises only about 2% of all
bariatric operations in the United States.99
The technique of LSG follows the transoral placement of 34
to 60 Fr bougie, which is advanced into the gastric antrum and
pylorus. Starting approximately 4 cm proximal to the pylorus,
the stomach is transected, initially transversely, and subsequently
longitudinally. The stomach is then divided proximally using the
sequential firings of 4.8 and 3.5 mm linear staplers. This results
in a narrow, tubular, stomach. The greater curvature vessels and
short gastrics can be divided using endoscopic staplers, thermal
ligature devices, or ultrascission.101,102 The resected portion of
stomach is then removed.
LSG appears to be a safe and effective procedure with low
morbidity and mortality. It is important to remember that the staple line is long, with potential for leakage and bleeding. The incidence of staple-line leakage after LSG ranges from 0% to 5.5%102,103
and with overall complication rates ranging from 0% to 24%.103 It
has been suggested that staple-line reinforcement can decrease the
incidence of leaks and different techniques have be implemented to
reinforce the staple line, including buttressing and oversewing.103,104
The efficacy of these techniques has been contested. A recent report
of 529 LSG performed without the use of staple-line reinforcement
demonstrated a 0% incidence of gastric leak and 0.19% mortality
rate.102 The authors contend that there are two main tenets to adopt
to minimize leaks. First, and of utmost importance, is to avoid creating a physiologic stricture at the incisura angularis and second,
to avoid stapling too close to the esophagus in the area of the cardia. A recent meta-analysis concluded that despite several drawbacks in the study, staple-line reinforcement did not seem to have
any clear benefit, at least concerning leak rate.105
Although classified as a restrictive procedure, LSG appears to
be more than just a gastric restrictive operation. With the removal
of the gastric corpus and fundus, the number of orexigenic cells
that produce the hormone ghrelin is significantly reduced.106 Thus,
LSG is not only a multipurpose operation, but also a multifactorial one, with a restrictive aspect and a complex neurohormonal
aspect, not yet fully elucidated.107,108

BILIOPANCREATIC DIVERSION WITH


DUODENAL SWITCH
Biliopancreatic diversion (BPD) was first described by
Scopinaro109,110 (Figure 15.1d). This was later modified with
the additions of a duodenal switch and a sleeve gastrectomy

PMPH_CH15.indd 136

to create the biliopancreatic diversion with duodenal switch


(BPD-DS).111,112
The standard BPD-DS technique involves dividing the small
bowel 250 cm above the ileocecal valve with a stapler, and then
forming a biliopancreatic limb by connecting the bowel proximal
to the transection to a point 100 cm above the ileocecal valve. The
bowel distal to the transection is elevated as an alimentary limb
to the upper abdomen. Gastric sleeve resection creates a tubularized stomach of approximately 100 cm3. The duodenum is divided
3 cm distal to the pylorus, and duodenoileostomy establishes continuity of the alimentary limb. Limb lengths determine weight
loss and complications. A common limb that is too long provides
inadequate weight loss, whereas one that is too short causes debilitating diarrhea and nutritional deficiencies. The gastric remnant
size should provide some restriction, but not prevent initiation of
protein digestion.
Close follow-up evaluation in the postoperative period is
recommended. Typical postoperative follow-up includes visits at
2 and 6 weeks, then quarterly for the 1st year, biannually for the
2nd year, and annually thereafter.113,114 Assessments are made by
both the surgeon and the nutritionist, and biochemical surveillance by complete blood count, chemical metabolic profi le, and
parathyroid hormone level is performed at regularly scheduled
intervals. An exercise program is helpful, as are multivitamin,
iron, vitamin D, and calcium supplements.
The 30-day mortality of early laparoscopic BPD series ranges
from 2.6% to 7.6%.115,116 Major complications, which occur in up to
25% of cases, may include early occurrence of anastomotic leak,
duodenal stump leak, intraabdominal infection, hemorrhage,
and venous thromboembolism.116,117 Late complications include
stomal ulceration, bowel obstruction, incarceration, or stricture.118
Diarrhea is a frequent chronic complication of BPD. Iron deficiency is common, with 6% of patients experiencing serious iron
deficiency anemia (hemoglobin <10 mg/dL).119 Surveillance of
biochemical and hematologic markers of iron deficiency should
drive replacement. Calcium and vitamin D malabsorption also
are common, manifesting as secondary hyperparathyroidism.120
The high incidence of stomal ulceration, severe protein-calorie
malnutrition, and diarrhea have limited its broad acceptance.121

VERTICAL BANDED GASTOPLASTY


In 1982, Mason introduced the vertical banded gastroplasty
(VBG).122 A transgastric window is created using a circular stapler.
A gastric pouch of approximately 20 mL is fashioned after a vertical gastric division is performed using a linear stapler through
the transgastric window. By using a mesh band (polypropylene
or polytetrafluoroethylene) or, less commonly, a silastic ring, the
gastric pouch outlet can be calibrated and reinforced.123
VBG was a very popular restrictive procedure in the United
States in the 1980s, but with time, it became clear that gastroesophageal reflux, complications associated with the band
(penetration, infection, bleeding, and obstruction) and unsatisfactory weight loss124 were too frequent and the procedure has since
been almost completely abandoned. As many as 25% to 54% of
VBG patients ultimately seek revisional surgery125,126 with the goal
being to reverse the complications of VBG and to induce weight
loss. Options include conversion to a RYGB123 or VBG reversal via
gastrogastrostomy.

5/21/2012 8:52:38 PM

Evidence-Based Bariatric Surgery

REVISIONAL BARIATRIC SURGERY


When patients are unsatisfied with their weight loss or develop
complications after their initial procedure, they frequently seek
revisional bariatric surgery (RBS). Approximately 5% of bariatric surgery patients have been estimated to require RBS. The
RBS population could be quite heterogeneous, depending on
their initial procedure, the indications for surgical revision, and
the revisional procedure performed.127 For instance, as previously discussed, patients might seek RBS after VBG because of
dysphagia due to esophageal dysmotility, intractable vomiting,
gastroesophageal reflux caused by stricture or gastroparesis of
the distal stomach, symptomatic erosion of the band into the gastric lumen, or, most commonly, weight regain due to staple line
or band disruption.125,128 Complications after LRYGB that might
require surgical revision include intractable stricture, nonhealing marginal ulcers, and metabolic complications.129 Failure of
weight loss after RYGB is commonly attributed to eating behaviors, staple line dehiscence with gastrogastric fistulas, and gastric pouch and gastrojejunal anastomosis dilation, resulting in a
loss of restriction. Regardless of the initial procedure, RBS is
commonly associated with an increased incidence of complications quoted as high as 22% to50%, with a reported mortality rate
of 1% to 2%.129,130 It is vital that the surgeon make every effort,
using medical record review as well as preoperative radiographic
and/or endoscopic assessment, to define the prior procedure or
procedures performed and understand the anatomy.
Patients who never lose weight may have had a technical
complication such as incomplete stapling131,132 or an inappropriate
operation. Those who regain weight after years may have experienced staple line recanalization or behavioral failure.130 Reoperation on a previous gastroplasty usually involves creating a Roux-Y
to a newly stapled proximal stomach pouch cephalad to the prior
gastric alteration.128 Likewise, most authors advocate LRYGB for
revision of LAGB because of complications or insufficient weight
loss.133 Finally, in cases of failed BPD-DS, some have advocated
the use of a pouch reduction procedure,134 and in cases of failed
LRYGB, either LAGB to improve the restrictive component135
or lengthening the Roux limb to improve the malabsorptive
component.136
Revisional bariatric operations may be performed
laparoscopically137,138 or via open techniques.139,140 Surgeons may
prefer an open approach to address severe adhesions or to permit tactile localization of prior partitions in the stomach to avoid
creation of undrained or ischemic segments during restapling.141
5. How do the different WLS procedures compare?
LRYGB, LAGB, LSG, and BPD-DS remain the popular WLS
options. Meta-analyses of studies comparing the different
surgical procedures have several limitations, such as lack of
randomization, heterogeneity of study populations, lack of
consistent control populations, variations in patient follow-up
and the reporting of comorbidities, and patients that are lost to
follow-up.105
Garb et al. performed a meta-analysis of studies comparing
LRYGB and LAGB between 2003 and 2007, encompassing 7383
patients.142 They found a composite percentage excess weight loss
(%EWL) of 49.4% for LAGB versus 62.6% for LRYGB and found
%EWL outcomes for LRYGB significantly superior to those for
LAGB at the three time points examined (1, 2, and >3 years). The

PMPH_CH15.indd 137

137

improvement in %EWL for LRYGB over LAGB was approximately


19% at 1 year and 16% at >3 years.
Three large meta-analyses were performed by Buchwald et
al. Mean weight loss was 47.5% after LAGB, 61.6% after LRYGB,
and 70.1% after BPD (with or without duodenal switch).143 The
postoperative 30-day mortality was 0.07% for all laparoscopic
restrictive procedures, 0.16% for LRYGB and 1.11% for the laparoscopic malabsorptive procedures.144 When focusing on diabetes, 79.3% of diabetic patients had resolution of their clinical and
laboratory manifestations of diabetes and 98.9% had resolution
or improvement after WLS. They observed greater resolution for
BPD-DS followed by LRYGB and then LAGB.145
Since LSG is a fairly new WLS technique, there are fewer
studies comparing its efficacy. Benedix et al. found that patients
who had undergone LSG experienced a greater %EWL (41.5%)
than those who had undergone LAGB (28.9%), but similar to the
%EWL in those patients that underwent a LRYGB (45%).146 Similarly, in a randomized study from Belgium, LSG achieved a higher
weight loss than LAGB.147 However, further trials are needed to
document the benefits of LSG over other forms of WLS.
6. Are there endoscopic techniques to effect weight loss?
Greater focus has turned toward minimally invasive endoscopic
therapies in the management of obesity. Most of these therapies,
however, are cumbersome and unlikely to be practical in an outpatient endoscopy setting. Several transoral endoluminal procedures
that avoid permanent surgical modification of the gastrointestinal
tract are under investigation in the United States.148
The primary endoscopic treatment modalities for obesity are
restrictive interventions, including intragastric balloons, transoral gastroplasty, and endoluminal vertical gastroplasty. The
duodenojejunal bypass sleeve is the only malabsorptive endoluminal device that has been studied in humans. Electrical stimulation to delay gastric emptying is also under investigation, with
some early experience in humans.149
By far, the most widely studied of the minimally invasive
endoscopic therapies for obesity is the gastric balloon. In 1985,
the first widely used intragastric balloon, the GarrenEdwards
gastric bubble (GEGB), was approved for use in the United States.
Approved as an adjunctive modality to a multifaceted approach
to obesity, the GEGB was a polyurethane cylindrical device with
a self-sealing valve through which a removable air-insufflation
catheter was inserted. The bubble was insufflated with 220 mL of
air and detached. The bubble was then left to float freely in the
stomach and could be removed endoscopically after being punctured with a forceps. Several studies were published showing that
diet and behavior modification were equally as efficacious as the
GEGB in producing weight loss.150,151 Complications of the GEGB
were significant and included gastric erosions, gastric ulcers,
small bowel obstruction, MalloryWeiss tears, and esophageal
lacerations due to the use of a cumbersome overtube during balloon placement.152,153 As a result, the GEGB is no longer used or
available in the United States. Research continues in developing
the ideal gastric balloon.154
7. What are the important aspects of postoperative care
following WLS?
Obesity is a chronic disorder that requires a continuous care
model of treatment.155 Bariatric surgery is not a guarantee of

5/21/2012 8:52:38 PM

138

Surgery: Evidence-Based Practice

success, and patients require ongoing postoperative care. To


reduce the likelihood of weight regain and to ensure that comorbid
conditions are adequately managed, all patients should receive
careful medical follow-up postoperatively. To guide patients
through the transition to life after bariatric surgery, a multidisciplinary team that includes an experienced primary care physician,
endocrinologist, or gastroenterologist is needed. Patients should
consider enrolling postoperatively in a comprehensive program
for nutrition and lifestyle management. Such support can ease the
transition to life after bariatric surgery.156

EXERCISE AND PHYSICAL ACTIVITY


After WLS, patients experience a rapid loss of body weight, 14.1%
to 48.5% of which may include lean body mass.157 Postoperative patients that exercise were shown to lose 28% more fat and
retained 8% more lean body mass compared with those who did
not exercise.158 The American College of Sports Medicine recommendations on healthy exercise levels are a minimum of 150 min
(2.5 h) of moderate intensity exercise per week, stating that there
may be advantages to increasing exercise to as much as 3.5 h a
week for maintaining long-term weight loss.159 The activity type,
duration, and intensity of exercise in post-WLS patients are quite
distinct from the exercise habits of young, physically fit, controls.
Successful WLS patients typically exercise longer, but at lowerlevel intensity.160 Aerobic physical activity positively influences
weight loss, body composition, and comorbidity resolution after
LRYGB for obesity.160

NUTRITIONAL MANAGEMENT
Protein malnutrition, defined by hypoalbuminemia (albumin <
3.5 mg/dL), remains the most severe macronutrient complication
associated with malabsorptive surgical procedures. Protein malnutrition causes an annual hospitalization rate of 1% per year after
malabsorptive procedures and leads to significant morbidity.161,162
When it occurs, protein malnutrition is generally observed at 3 to
6 months after surgery and is largely attributed to the development
of food intolerance to protein-rich foods.163 Prevention of protein
malnutrition requires regular assessment of protein intake and
counseling regarding ingestion of protein from protein-rich foods
and modular protein supplements. To maintain lean body mass
during weight loss, roughly 60 g of protein should be consumed
per day.164
The anatomic changes imposed by malabsorptive surgery
increase the risk for various vitamin and mineral deficiencies.161
After LRYGB, screening and supplementation of deficiencies
with a multivitamin-mineral, iron, vitamin B12, or calcium with
vitamin D is routinely conducted, and prophylactic supplementation should be considered in all patients.165 A daily multivitamin
and calcium supplementation with added vitamin D is recommended for all WLS patients.166 Recommended doses of elemental calcium after bariatric surgery range from 1200 to 2000 mg
daily, and these usually contain vitamin D as well.161 Patients
must be instructed to take calcium carbonate preparations with
meals to enhance intestinal absorption. Calcium citrate preparations are preferred because this salt is better absorbed in the
absence of gastric acid production.167 A rise in serum parathyroid

PMPH_CH15.indd 138

hormone indicates negative calcium balance and/or a vitamin D


deficiency and this can serve as a screening tool for metabolic
bone disease.168
Vitamin B12 deficiencies can occur after bariatric surgery
procedures that bypass the lower stomach. In the absence of vitamin B12 supplementation in post-LRYGB patients, the incidence
of vitamin B12 deficiency is 33% to 40% at the first postoperative
year169 and 8 to 37% by 2 to 4 years161,170. Anemias as a result of vitamin B12 deficiency have been reported to occur in more than 30%
of patients 1 to 9 years after LRYGB.165 The initiation of vitamin
B12 supplementation within 6 months postoperatively is recommended for all LRYGB, LSG, and BPD-DS patients.
Steatorrhea induced by malabsorptive surgical procedures can
lead to deficiencies in fat-soluble vitamins.165 Vitamin A deficiency
after bariatric surgery results from poor nutritional intake, maldigestion, malabsorption, and impaired hepatic release of vitamin
A. The incidence of vitamin A deficiency was 61% to 69% at 2 to
4 years after BPD (with or without duodenal switch).113,171 Vitamin
K levels have also been found to be low in 50% to 60% of patients
who underwent BPD (with or without duodenal switch).167,172
The extent of metabolic and nutritional evaluations necessary
after bariatric surgery should be guided by the surgical procedure
performed. Purely gastric restrictive procedures are not associated
with alterations in intestinal continuity and do not alter normal
digestive physiology. As a result, selective nutritional deficiencies
are uncommon. Nutritional intake assessments and laboratory
surveillance of vitamin and mineral levels are key to ensuring the
adequacy of nutrition after WLS. Both are recommended after all
bariatric surgeries, even if patients tolerate their diet well, with no
vomiting or diarrhea, to detect subclinical nutritional deficiencies
and prevent development of frank deficiencies.173-175 Malabsorptive procedures can be associated with micronutrient and macronutrient deficiencies and require lifelong supplementation and
monitoring of laboratory data by a team familiar with possible
deficiencies.173,174

TYPE 2 DIABETES MELLITUS


Type 2 diabetes mellitus (T2DM) is a common morbidity associated with obesity and can improve to the point that little or
no medication is necessary in patients after WLS.21 After combination or malabsorptive procedures, insulin-treated patients
experience a significant decrease in insulin requirements; the
majority of patients can discontinue insulin therapy by 6 weeks
after surgery,176 and some may even be able to discontinue insulin
before hospital discharge. The long-term effects of these bypass
operations on T2DM appear to include both fat-loss-dependent
and hormonal mechanisms.177 By contrast, purely restrictive
operations appear to improve T2DM as a result of fat loss alone.19
Though not fully elucidated, improvements in insulin resistance
are observed almost immediately after LRYGB and are, in part,
due to increased levels of GLP-1 seen after LRYGB.21,178
Due to the rapid resolution of T2DM after LRYGB, oral hypoglycemic agents and long-acting insulins should be avoided in the
immediate postoperative period as changes in GLP-1 and other
incretins increase insulin sensitivity. Consequently, administration of these medications in the postoperative period increases the
risk of hypoglycemia. These agents should be reintroduced later
only if clinically indicated.

5/21/2012 8:52:39 PM

Evidence-Based Bariatric Surgery

LIPID DISORDERS
Triglyceride and low-density lipoprotein (LDL)-cholesterol decrease
and high-density lipoprotein-cholesterol increases after WLS.179-181
The improvement in dyslipidemia appears to be related not only to
the percentage of excess weight loss, and more specifically adipocyte
mass loss, but also to the decrease in insulin resistance.179 Given the
improvement in cardiovascular mortality after bariatric surgery,
these changes have likely led to a decreased risk of cardiovascular
disease.12,182 Lipid-lowering therapy for LDL-cholesterol and triglyceride values that remain above desired goals after surgery should be
continued. Due to the dramatic reductions in lipid levels, the doses
of lipid-lowering drugs should be periodically reevaluated.

DUMPING SYNDROME
Abdominal pain and cramping, nausea, diarrhea, lightheadedness,
flushing, tachycardia, and syncope are all indicative of dumping.
These symptoms are reported frequently after the intake of simple
sugars and may serve to discourage the intake of energy-dense
foods and beverages.183 Gastric dumping occurs initially in 70% to
76% of patients who have had a RYGB.184,185 Historically, dumping
was thought to be the result of the hyperosmolarity of intestinal
contents, which resulted in an influx of fluid into the intestinal
lumen with subsequent intestinal distention, fluid sequestration in
the intestinal lumen, decreased intravascular volume, and hypotension. More recent data suggest that food bypassing the stomach and
entering the small intestine leads to the release of gut peptides that
are responsible for dumping symptoms.186 Dumping symptoms tend
to become less prominent with time183 and can usually be controlled
with certain nutritional changes, such as (1) eating small meals; (2)
avoiding ingestion of liquids within 30 min of a solid-food meal; (3)
avoiding simple sugars and increasing intake of fiber and complex
carbohydrates; and (4) increasing protein intake.187
8. Does accreditation of WLS centers improve outcomes?
It is generally accepted that surgical outcomes are related to the
volume of procedures performed per year by a surgeon or at a
hospital.188,189 Consequently, several policy-generating bodies have
recommended limiting the conduct of several technically complex
operations to high-volume centers. This philosophy was adopted
by the accrediting bodies for bariatric surgery centers of excellence. The American College of Surgeons Bariatric Surgery Center Network Program (ACS/BSCNP)190 and the American Society
of Metabolic and Bariatric Surgerys Surgical Review Corporation (ASMBS/SRC)191 have published guidelines for requirements
for hospitals seeking designation as accredited WLS centers or

139

Centers of Excellence (COE). Both of these accrediting bodies


require that at least 125 bariatric operations be performed annually to achieve this designation. In 2006, Medicare required that
hospitals providing WLS services for its beneficiaries must be
accredited by one of these organizations.192
Multidisciplinary care is generally considered essential in the
management of clinically complex and high-risk WLS patients.
Such care involves the creation of a WLS team with a dedicated and
appropriately credentialed director. It also includes appropriately
trained surgical, medical, psychiatric, nutrition, and nursing personnel. Such staffing is now required by both the ACS/BSCNP190 as
well as the ASMBS/SRC.191 It is necessary not only in the immediate postoperative period, but also in the long run as patients adjust
medically and psychosocially to the results of WLS.
To achieve accreditation, the onus has been put on facilities to
adequately outfit their hospitals in ways that accommodate WLS
patients at all points of care. Extended-size gowns, pants, and robes
must be routinely available, as well as respiratory support with
Continuous Positive Airway Pressure and Bilevel Positive Airway
Pressure therapy. Pneumatic, size-appropriate lower extremity
compression sleeves should also be available. The center of excellence accrediting bodies recommend that CT or MRI scanners
have a weight capacity 400 lbs or greater. The American Institute
of Architects has established guidelines for planning and designing WLS healthcare facilities.193 These specify a barrier-free environment that avoids stairs and provides ramp access when possible.
They set minimal clearances for rooms that will accommodate WLS
patients, including operating rooms, and address appropriate-sized
inpatient rooms, and postanesthesia care units. The guidelines set
clearance for a floor-mounted toilet at 5 ft, and weight ratings for
toilets and sinks at 700 lb and 300 lb, respectively.
Challenges have been made to the concept of COEs. Livingston194 contends that the major difference between COEs and
non-COEs was the higher procedure volume in the COEs. Despite
the higher procedure volume requirements and the rigorous standards imposed on COEs, their outcomes were equivalent to nonCOEs. When analyzing data from 102,069 bariatric operations
and adjusting for comorbidities, Kohn et al. found that greater
bariatric case volume was associated with improvements in the
incidence of total complications.195 Their findings supported the
concept of volume-dependent COE programs, though they demonstrated no differences on the basis of actual accreditation status
per ACS/BSCN or ASMBS/SRC.
Certification systems need to evolve and should continue to
emphasize the value of higher-volume centers, especially for higherrisk patients, but should not restrict provision of care at lowervolume centers to the point that patients cannot achieve access
and therefore suffer disproportionately from untreated obesity.196

Clinical Question Summary


Questions

Answers

Grade

References

1 What is the magnitude and impact of


the global obesity epidemic?

Obesity is a burgeoning health problem with


significant associated comorbidities. Weight
reduction can result in improved survival.

3, 8-12

2 What are the indications for WLS?

A BMI of more than 40 kg/m2 or a BMI of more than


35 kg/m2 with obesity-related comorbidity meets
current NIH criteria for WLS.

16, 17

(Continued)

PMPH_CH15.indd 139

5/21/2012 8:52:39 PM

140

Surgery: Evidence-Based Practice

(Continued)
Questions

Answers

3 What factors must be considered


prior to WLS?

Factors associated with increased postoperative


morbidity and mortality include recent MI,
hypertension, stroke, dependent functional
status, smoking, and poorly controlled
psychiatric disorders. Management of preexisting
cholelithiasis remains controversial. Prophylaxis
against thromboembolic complications is
indicated.

41, 54, 55, 58

4 What are the common bariatric


surgery procedures?

These are the laparoscopic Roux-en-Y gastric


bypass, laparoscopic adjustable gastric banding,
LSG, and BPD. JIB remains of historical
significance. Revisonal bariatric surgery for
unsatisfactory weight loss is associated with
increased morbidity and mortality.

69, 74, 75, 99, 100, 118

5 How do the different WLS


procedures compare?

Large meta-analyses comparing the WLS procedures


have suffered from many limitations. LRYGB
remains the current gold standard for bariatric
surgery.

142-146

6 Are there endoscopic techniques to


effect weight loss?

There does exist an endoscopically deployed


intragastric balloon though is currently not used
secondary to its complication profile. Research
continues in the search for endoscopic therapies.

150

7 What are the important aspects of


postoperative care following WLS?

WLS necessitates undergoing a lifestyle change with


focus on exercise and nutritional management to
monitor caloric intake and prevent micronutrient
and vitamin deficiencies. Diabetes mellitus and
dyslipidemias show dramatic improvement, thus
medication adjustments are necessary.

17, 158, 161, 173, 177,


178

8 Does accreditation of WLS centers


improve outcomes?

It is recommended that all bariatric surgeries


be performed at accredited centers, though
outcomes may be equivalent.

194-196

REFERENCES
1. Health topics: Obesity. (Accessed 2/23/2011, at http://www.who.
int/topics/obesity/en/.)
2. Global database on body mass index. (Accessed 2/23/2011, at
http://apps.who.int/bmi/.)
3. Must A, Spadano J, Coakley EH, Field AE, Colditz G, Dietz WH.
The disease burden associated with overweight and obesity. JAMA.
1999;282:1523-1529.
4. Allison DB, Zannolli R, Narayan KM. The direct health care costs
of obesity in the United States. Am J Public Health. 1999;89:11941199.
5. U.S. Department of Health and Human Services. The practical
guide - identification, evaluation, and treatment of overweight and
obesity in adults. In: National Institutes of Health, ed. Bethesda,
MD; 2000.
6. Buchwald H, Oien DM. Metabolic/bariatric surgery Worldwide
2008. Obes Surg. 2009;19:1605-1611.
7. Maggard MA, Shugarman LR, Suttorp M, et al. Meta-analysis: Surgical treatment of obesity. Ann Intern Med. 2005;142:
547-559.
8. Li Z, Maglione M, Tu W, et al. Meta-analysis: Pharmacologic treatment of obesity. Ann Intern Med. 2005;142:532-546.

PMPH_CH15.indd 140

Grade

References

9. Flum DR, Dellinger EP. Impact of gastric bypass operation


on survival: A population-based analysis. J Am Coll Surg.
2004;199:543-551.
10. Christou NV, Sampalis JS, Liberman M, et al. Surgery decreases
long-term mortality, morbidity, and health care use in morbidly obese patients. Ann Surg. 2004;240:416-423; discussion
23-24.
11. Sjostrom L, Narbro K, Sjostrom CD, et al. Effects of bariatric
surgery on mortality in Swedish obese subjects. N Engl J Med.
2007;357:741-752.
12. Adams TD, Gress RE, Smith SC, et al. Long-term mortality after
gastric bypass surgery. N Engl J Med. 2007;357:753-761.
13. Samuel I, Mason EE, Renquist KE, Huang YH, Zimmerman
MB, Jamal M. Bariatric surgery trends: An 18-year report
from the International Bariatric Surgery Registry. Am J Surg.
2006;192:657-662.
14. Sturm R. Increases in morbid obesity in the USA: 20002005.
Pub Health. 2007;121:492-496.
15. Buchwald H, Williams SE. Bariatric surgery training in the United
States. Surg Obes Relat Dis. 2006;2:52-55; discussion 5-6.
16. NIH conference. Gastrointestinal surgery for severe obesity.
Consensus Development Conference Panel. Ann Intern Med.
1991;115:956-961.

5/21/2012 8:52:39 PM

Evidence-Based Bariatric Surgery

17. Clinical Guidelines on the Identification, Evaluation, and


Treatment of Overweight and Obesity in AdultsThe Evidence
Report. National Institutes of Health. Obes Res. 1998;6(2):
51S-209S.
18. Gastrointestinal surgery for severe obesity: National Institutes
of Health Consensus Development Conference Statement. Am J
Clin Nutr. 1992;55:615S-619S.
19. Dixon JB, OBrien PE, Playfair J, et al. Adjustable gastric banding
and conventional therapy for type 2 diabetes: A randomized
controlled trial. JAMA. 2008;299:316-323.
20. Charuzi I, Lavie P, Peiser J, Peled R. Bariatric surgery in morbidly
obese sleep-apnea patients: Short- and long-term follow-up. Am
J Clin Nutr. 1992;55:594S-596S.
21. Pournaras DJ, Osborne A, Hawkins SC, et al. Remission of type
2 diabetes after gastric bypass and banding: Mechanisms and
2 year outcomes. Ann Surg. 2010;252:966-971.
22. Carson JL, Ruddy ME, Duff AE, Holmes NJ, Cody RP, Brolin
RE. The effect of gastric bypass surgery on hypertension in
morbidly obese patients. Arch Intern Med. 1994;154:193-200.
23. OBrien PE, Dixon JB, Brown W, et al. The laparoscopic
adjustable gastric band (Lap-Band): A prospective study of
medium-term effects on weight, health and quality of life. Obes
Surg. 2002;12:652-660.
24. Michaelides EM, Sismanis A, Sugerman HJ, Felton WL, 3rd.
Pulsatile tinnitus in patients with morbid obesity: The effectiveness
of weight reduction surgery. Am J Otol. 2000;21:682-685.
25. Frigg A, Peterli R, Peters T, Ackermann C, Tondelli P. Reduction
in co-morbidities 4 years after laparoscopic adjustable gastric
banding. Obes Surg. 2004;14:216-223.
26. Melissas J, Volakakis E, Hadjipavlou A. Low-back pain in
morbidly obese patients and the effect of weight loss following
surgery. Obes Surg. 2003;13:389-393.
27. Wittgrove AC, Jester L, Wittgrove P, Clark GW. Pregnancy
following gastric bypass for morbid obesity. Obes Surg.
1998;8:461-464; discussion 5-6.
28. Buddeberg-Fischer B, Klaghofer R, Sigrist S, Buddeberg C.
Impact of psychosocial stress and symptoms on indication for
bariatric surgery and outcome in morbidly obese patients. Obes
Surg. 2004;14:361-369.
29. Arcila D, Velazquez D, Gamino R, et al. Quality of life in
bariatric surgery. Obes Surg. 2002;12:661-665.
30. Runkel N, Colombo-Benkmann M, Huttl TP, et al. Evidencebased German guidelines for surgery for obesity. Int J Colorectal
Dis. 2011;26(4):397-404.
31. Dixon JB, Dixon ME, OBrien PE. Pregnancy after Lap-Band
surgery: Management of the band to achieve healthy weight
outcomes. Obes Surg. 2001;11:59-65.
32. Friedman D, Cuneo S, Valenzano M, et al. Pregnancies in an
18-Year Follow-up after Biliopancreatic Diversion. Obes Surg.
1995;5:308-313.
33. Kominiarek MA. Pregnancy after bariatric surgery. Obstet
Gynecol Clin North Am. 2010;37:305-320.
34. Beard JH, Bell RL, Duff y AJ. Reproductive considerations
and pregnancy after bariatric surgery: Current evidence and
recommendations. Obes Surg. 2008;18:1023-1027.
35. Capella JF, Capella RF. Bariatric surgery in adolescence. Is this
the best age to operate? Obes Surg. 2003;13:826-832.
36. Dolan K, Fielding G. A comparison of laparoscopic adjustable
gastric banding in adolescents and adults. Surg Endosc.
2004;18:45-47.
37. Inge TH, Garcia V, Daniels S, et al. A multidisciplinary
approach to the adolescent bariatric surgical patient. J Pediatr
Surg. 2004;39:442-447; discussion 6-7.

PMPH_CH15.indd 141

141

38. Gertler R, Ramsey-Stewart G. Pre-operative psychiatric


assessment of patients presenting for gastric bariatric surgery
(surgical control of morbid obesity). Aust N Z J Surg. 1986;56:
157-161.
39. Black DW, Goldstein RB, Mason EE. Psychiatric diagnosis and
weight loss following gastric surgery for obesity. Obes Surg.
2003;13:746-751.
40. Orth WS, Madan AK, Ternovits CA, Tichansky DS. Effect of
preoperative knowledge on weight loss after laparoscopic gastric
bypass. Obes Surg. 2008;18:768-771.
41. Gupta PK, Franck C, Miller WJ, Gupta H, Forse RA. Development
and Validation of a Bariatric Surgery Morbidity Risk Calculator
Using the Prospective, Multicenter NSQIP Dataset. J Am Coll
Surg. 2011; in press.
42. Dimick JB, Welch HG, Birkmeyer JD. Surgical mortality as an
indicator of hospital quality: The problem with small sample
size. Jama. 2004;292:847-851.
43. Sampalis JS, Sampalis F, Christou N. Impact of bariatric surgery
on cardiovascular and musculoskeletal morbidity. Surg Obes
Relat Dis. 2006;2:587-591.
44. Vogel JA, Franklin BA, Zalesin KC, et al. Reduction in predicted
coronary heart disease risk after substantial weight reduction
after bariatric surgery. Am J Cardiol. 2007;99:222-226.
45. Gonzalez R, Haines K, Nelson LG, Gallagher SF, Murr MM.
Predictive factors of thromboembolic events in patients
undergoing Roux-en-Y gastric bypass. Surg Obes Relat Dis.
2006;2:30-35; discussion 5-6.
46. Felix EL, Kettelle J, Mobley E, Swartz D. Perforated marginal
ulcers after laparoscopic gastric bypass. Surg Endosc. 2008;22:
2128-2132.
47. Patel RA, Brolin RE, Gandhi A. Revisional operations for
marginal ulcer after Roux-en-Y gastric bypass. Surg Obes Relat
Dis. 2009;5:317-322.
48. Kinzl JF, Schrattenecker M, Traweger C, Mattesich M, Fiala M,
Biebl W. Psychosocial predictors of weight loss after bariatric
surgery. Obes Surg. 2006;16:1609-1614.
49. Bauchowitz AU, Gonder-Frederick LA, Olbrisch ME, et al.
Psychosocial evaluation of bariatric surgery candidates:
A survey of present practices. Psychosom Med. 2005;67:
825-832.
50. Herpertz S, Kielmann R, Wolf AM, Hebebrand J, Senf W. Do
psychosocial variables predict weight loss or mental health after
obesity surgery? A systematic review. Obes Res. 2004;12:15541569.
51. Coughlin K, Bell RM, Bivins BA, Wrobel S, Griffen WO, Jr.
Preoperative and postoperative assessment of nutrient intakes
in patients who have undergone gastric bypass surgery. Arch
Surg. 1983;118:813-816.
52. Still CD, Benotti P, Wood GC, et al. Outcomes of preoperative
weight loss in high-risk patients undergoing gastric bypass
surgery. Arch Surg. 2007;142:994-998; discussion 9.
53. Jamal MK, DeMaria EJ, Johnson JM, et al. Insurance-mandated
preoperative dietary counseling does not improve outcome
and increases dropout rates in patients considering gastric
bypass surgery for morbid obesity. Surg Obes Relat Dis. 2006;2:
122-127.
54. Quesada BM, Kohan G, Roff HE, Canullan CM, Chiappetta
Porras LT. Management of gallstones and gallbladder disease
in patients undergoing gastric bypass. World J Gastroenterol.
2010;16:2075-2079.
55. Kim JJ, Schirmer B. Safety and efficacy of simultaneous
cholecystectomy at Roux-en-Y gastric bypass. Surg Obes Relat
Dis. 2009;5:48-53.

5/21/2012 8:52:39 PM

142

Surgery: Evidence-Based Practice

56. Li VK, Pulido N, Fajnwaks P, Szomstein S, Rosenthal R, MartinezDuartez P. Predictors of gallstone formation after bariatric
surgery: A multivariate analysis of risk factors comparing gastric
bypass, gastric banding, and sleeve gastrectomy. Surg Endosc.
2009;23:1640-1644.
57. Pories WJ, van Rij AM, Burlingham BT, Fulghum RS, Meelheim
D. Prophylactic cefazolin in gastric bypass surgery. Surgery.
1981;90:426-432.
58. Magee CJ, Barry J, Javed S, Macadam R, Kerrigan D. Extended
thromboprophylaxis reduces incidence of postoperative venous
thromboembolism in laparoscopic bariatric surgery. Surg Obes
Relat Dis. 2010;6:322-325.
59. Ochner CN, Gibson C, Shanik M, Goel V, Geliebter A. Changes
in neurohormonal gut peptides following bariatric surgery. Int
J Obes (Lond). 2011;35:153-166.
60. Griffen WO, Jr., Bivins BA, Bell RM. The decline and fall of the
jejunoileal bypass. Surg Gynecol Obstet. 1983;157:301-308.
61. Griffen WO, Jr., Young VL, Stevenson CC. A prospective
comparison of gastric and jejunoileal bypass procedures for
morbid obesity. Ann Surg. 1977;186:500-509.
62. Requarth JA, Burchard KW, Colacchio TA, et al. Long-term
morbidity following jejunoileal bypass. The continuing potential
need for surgical reversal. Arch Surg. 1995;130:318-325.
63. Mason EE, Ito C. Gastric bypass. Ann Surg. 1969;170:329-339.
64. Mason EE, Ito C. Gastric bypass in obesity. Surg Clin North Am.
1967;47:1345-1351.
65. Bohdjalian A, Langer FB, Kranner A, Shakeri-Leidenmuhler
S, Zacherl J, Prager G. Circular- vs. linear-stapled gastrojejunostomy in laparoscopic Roux-en-Y gastric bypass. Obes
Surg. 2010;20:440-446.
66. Cruz-Munoz ND, Cabrera JC, Cuesta M, Hartnett S, Rojas R.
Closure of mesenteric defect can lead to decrease in internal
hernias after Roux-en-Y gastric bypass. Surg Obes Relat Dis. 2011;
7(2):176-180.
67. Bauman RW, Pirrello JR. Internal hernia at Petersens space after
laparoscopic Roux-en-Y gastric bypass: 6.2% incidence without
closure--a single surgeon series of 1047 cases. Surg Obes Relat
Dis. 2009;5:565-570.
68. Kellum JM, Chikunguwo SM, Maher JW, Wolfe LG, Sugerman
HJ. Long-term results of malabsorptive distal Roux-en-Y
gastric bypass in superobese patients. Surg Obes Relat Dis. 2011;
7(2):189-193.
69. Harper J, Madan AK, Ternovits CA, Tichansky DS. What
happens to patients who do not follow-up after bariatric surgery?
Am Surg. 2007;73:181-184.
70. Podnos YD, Jimenez JC, Wilson SE, Stevens CM, Nguyen NT.
Complications after laparoscopic gastric bypass: A review of
3464 cases. Arch Surg. 2003;138:957-961.
71. Gonzalez R, Sarr MG, Smith CD, et al. Diagnosis and
contemporary management of anastomotic leaks after gastric
bypass for obesity. J Am Coll Surg. 2007;204:47-55.
72. Cho M, Carrodeguas L, Pinto D, et al. Diagnosis and management
of partial small bowel obstruction after laparoscopic antecolic
antegastric Roux-en-Y gastric bypass for morbid obesity. J Am
Coll Surg. 2006;202:262-268.
73. Sunnapwar A, Sandrasegaran K, Menias CO, Lockhart M,
Chintapalli KN, Prasad SR. Taxonomy and imaging spectrum
of small bowel obstruction after Roux-en-Y gastric bypass
surgery. AJR Am J Roentgenol. 2010;194:120-128.
74. Campos GM, Rabl C, Roll GR, et al. Better Weight Loss,
Resolution of Diabetes, and Quality of Life for Laparoscopic
Gastric Bypass vs Banding: Results of a 2-Cohort Pair-Matched
Study. Arch Surg. 2011;146:149-155.

PMPH_CH15.indd 142

75. Laurenius A, Taha O, Maleckas A, Lonroth H, Olbers T.


Laparoscopic biliopancreatic diversion/duodenal switch or
laparoscopic Roux-en-Y gastric bypass for super-obesity-weight
loss versus side effects. Surg Obes Relat Dis. 2010;6:408-414.
76. Belachew M, Legrand M, Vincenti VV, et al. Laparoscopic
Placement of Adjustable Silicone Gastric Band in the Treatment
of Morbid Obesity: How to Do It. Obes Surg. 1995;5:66-70.
77. Favretti F, Cadiere GB, Segato G, et al. Laparoscopic Adjustable
Silicone Gastric Banding: Technique and Results. Obes Surg.
1995;5:364-371.
78. DeMaria EJ, Jamal MK. Laparoscopic adjustable gastric
banding: Evolving clinical experience. Surg Clin North Am.
2005;85:773-787, vii.
79. Chapman AE, Kiroff G, Game P, et al. Laparoscopic adjustable
gastric banding in the treatment of obesity: A systematic
literature review. Surgery. 2004;135:326-351.
80. Di Lorenzo N, Furbetta F, Favretti F, et al. Laparoscopic
adjustable gastric banding via pars flaccida versus perigastric
positioning: Technique, complications, and results in 2,549
patients. Surg Endosc. 2010;24:1519-1523.
81. Pomerri F, Liberati L, Curtolo S, Muzzio PC. Adjustable silicone
gastric banding for obesity. Gastrointest Radiol. 1992;17:
207-210.
82. Poulos A, Peat K, Lorman JG, Hatfield DR, Griffen WO, Jr.
Gastric operation for the morbidly obese. AJR Am J Roentgenol.
1981;136:867-870.
83. Sherwinter DA, Powers CJ, Geiss AC, Howard M, Warman J.
Posterior prolapse: An important entity even in the modern age
of the pars flaccida approach to lap-band placement. Obes Surg.
2006;16:1312-1317.
84. Berrevoet F, Pattyn P, Cardon A, de Ryck F, Hesse UJ, de
Hemptinne B. Retrospective analysis of laparoscopic gastric
banding technique: Short-term and mid-term follow-up. Obes
Surg. 1999;9:272-275.
85. Parikh MS, Fielding GA, Ren CJ. U.S. experience with 749
laparoscopic adjustable gastric bands: Intermediate outcomes.
Surg Endosc. 2005;19:1631-1635.
86. Thornton CM, Rozen WM, So D, Kaplan ED, Wilkinson S.
Reducing band slippage in laparoscopic adjustable gastric
banding: The mesh plication pars flaccida technique. Obes Surg.
2009;19:1702-1706.
87. Fried M, Dolezalova K, Sramkova P. Adjustable gastric
banding outcomes with and without gastrogastric imbrication
sutures: A randomized controlled trial. Surg Obes Relat Dis.
2011;7:23-31.
88. Spivak H, Favretti F. Avoiding postoperative complications
with the LAP-BAND system. Am J Surg. 2002;184:31S-37S.
89. Gulkarov I, Wetterau M, Ren CJ, Fielding GA. Hiatal hernia
repair at the initial laparoscopic adjustable gastric band
operation reduces the need for reoperation. Surg Endosc.
2008;22:1035-1041.
90. Schouten R, van Dielen FM, Greve JW. Re-operation after
laparoscopic adjustable gastric banding leads to a further
decrease in BMI and obesity-related co-morbidities: Results in
33 patients. Obes Surg. 2006;16:821-828.
91. Brown WA, Burton PR, Anderson M, et al. Symmetrical
pouch dilatation after laparoscopic adjustable gastric banding:
Incidence and management. Obes Surg. 2008;18:1104-1108.
92. Moser F, Gorodner MV, Galvani CA, Baptista M, Chretien C,
Horgan S. Pouch enlargement and band slippage: Two different
entities. Surg Endosc. 2006;20:1021-1029.
93. Lattuada E, Zappa MA, Mozzi E, et al. Band erosion following
gastric banding: How to treat it. Obes Surg. 2007;17:329-333.

5/21/2012 8:52:39 PM

Evidence-Based Bariatric Surgery

94. Abu-Abeid S, Keidar A, Gavert N, Blanc A, Szold A. The clinical


spectrum of band erosion following laparoscopic adjustable
silicone gastric banding for morbid obesity. Surg Endosc.
2003;17:861-863.
95. Biagini J. Intragastric band erosion. Obes Surg. 2001;11:100.
96. Niville E, Dams A, Vlasselaers J. Lap-Band erosion: Incidence
and treatment. Obes Surg. 2001;11:744-747.
97. Campos JM, Evangelista LF, Galvao Neto MP, et al. Small
erosion of adjustable gastric band: Endoscopic removal through
incision in gastric wall. Surg Laparosc Endosc Percutan Tech.
2010;20:e215-e217.
98. Regan JP, Inabnet WB, Gagner M, Pomp A. Early experience
with two-stage laparoscopic Roux-en-Y gastric bypass as
an alternative in the super-super obese patient. Obes Surg.
2003;13:861-864.
99. Deitel M, Crosby RD, Gagner M. The First International
Consensus Summit for Sleeve Gastrectomy (SG), New York
City, October 25-27, 2007. Obes Surg. 2008;18:487-496.
100. Basso N, Casella G, Rizzello M, et al. Laparoscopic sleeve
gastrectomy as first stage or definitive intent in 300 consecutive
cases. Surg Endosc. 2011;25:444-449.
101. Daskalakis M, Berdan Y, Theodoridou S, Weigand G, Weiner
RA. Impact of surgeon experience and buttress material
on postoperative complications after laparoscopic sleeve
gastrectomy. Surg Endosc. 2011;25:88-97.
102. Bellanger DE, Greenway FL. Laparoscopic sleeve gastrectomy,
529 cases without a leak: Short-term results and technical
considerations. Obes Surg. 2011;21:146-150.
103. Aggarwal S, Kini SU, Herron DM. Laparoscopic sleeve
gastrectomy for morbid obesity: A review. Surg Obes Relat Dis.
2007;3:189-194.
104. Baker RS, Foote J, Kemmeter P, Brady R, Vroegop T, Serveld
M. The science of stapling and leaks. Obes Surg. 2004;14:
1290-1298.
105. Giannopoulos GA, Tzanakis NE, Rallis GE, Efstathiou SP,
Tsigris C, Nikiteas NI. Staple line reinforcement in laparoscopic
bariatric surgery: Does it actually make a difference? A systematic
review and meta-analysis. Surg Endosc. 2010;24:2782-2788.
106. Karamanakos SN, Vagenas K, Kalfarentzos F, Alexandrides
TK. Weight loss, appetite suppression, and changes in fasting
and postprandial ghrelin and peptide-YY levels after Roux-en-Y
gastric bypass and sleeve gastrectomy: A prospective, double
blind study. Ann Surg. 2008;247:401-407.
107. Melissas J, Daskalakis M, Koukouraki S, et al. Sleeve
gastrectomy-a food limiting operation. Obes Surg. 2008;18:
1251-1256.
108. Langer FB, Bohdjalian A, Shakeri-Manesch S, et al. Eating
behavior in laparoscopic sleeve gastrectomy: Correlation between
plasma ghrelin levels and hunger. Eur Surg. 2008;40:120-124.
109. Scopinaro N, Gianetta E, Civalleri D, Bonalumi U, Bachi V.
Bilio-pancreatic bypass for obesity: 1. An experimental study in
dogs. Br J Surg. 1979;66:613-617.
110. Scopinaro N, Gianetta E, Civalleri D, Bonalumi U, Bachi V.
Bilio-pancreatic bypass for obesity: II. Initial experience in
man. Br J Surg. 1979;66:618-620.
111. Marceau P, Biron S, Bourque RA, Potvin M, Hould FS, Simard
S. Biliopancreatic Diversion with a New Type of Gastrectomy.
Obes Surg. 1993;3:29-35.
112. Marceau P, Hould FS, Simard S, et al. Biliopancreatic diversion
with duodenal switch. World J Surg. 1998;22:947-954.
113. Dolan K, Hatzifotis M, Newbury L, Lowe N, Fielding G. A
clinical and nutritional comparison of biliopancreatic diversion
with and without duodenal switch. Ann Surg. 2004;240:51-56.

PMPH_CH15.indd 143

143

114. Resa JJ, Solano J, Fatas JA, et al. Laparoscopic biliopancreatic


diversion: Technical aspects and results of our protocol. Obes
Surg. 2004;14:329-333; discussion 33.
115. Paiva D, Bernardes L, Suretti L. Laparoscopic biliopancreatic
diversion: Technique and initial results. Obes Surg. 2002;12:
358-361.
116. Ren CJ, Patterson E, Gagner M. Early results of laparoscopic
biliopancreatic diversion with duodenal switch: A case series of
40 consecutive patients. Obes Surg. 2000;10:514-523; discussion
24.
117. Hess DS, Hess DW, Oakley RS. The biliopancreatic diversion
with the duodenal switch: Results beyond 10 years. Obes Surg.
2005;15:408-416.
118. Parikh MS, Laker S, Weiner M, Hajiseyedjavadi O, Ren
CJ. Objective comparison of complications resulting from
laparoscopic bariatric procedures. J Am Coll Surg. 2006;202:
252-261.
119. Marceau P, Hould FS, Lebel S, Marceau S, Biron S. Malabsorptive
obesity surgery. Surg Clin North Am. 2001;81:1113-1127.
120. Chapin BL, LeMar HJ, Jr., Knodel DH, Carter PL. Secondary
hyperparathyroidism following biliopancreatic diversion. Arch
Surg. 1996;131:1048-1052; discussion 53.
121. Kendrick ML, Dakin GF. Surgical approaches to obesity. Mayo
Clin Proc. 2006;81:S18-S24.
122. Mason EE. Vertical banded gastroplasty for obesity. Arch Sur.
1982;117:701-706.
123. Waaddegaard P, Clemmesen T, Jess P. Vertical gastric banding
for morbid obesity: A long-term follow-up study. Eur J Surg.
2002;168:220-222.
124. Balsiger BM, Poggio JL, Mai J, Kelly KA, Sarr MG. Ten and more
years after vertical banded gastroplasty as primary operation
for morbid obesity. J Gastrointest Surg. 2000;4:598-605.
125. van Gemert WG, van Wersch MM, Greve JW, Soeters PB.
Revisional surgery after failed vertical banded gastroplasty:
Restoration of vertical banded gastroplasty or conversion to
gastric bypass. Obes Surg. 1998;8:21-28.
126. Tevis S, Garren MJ, Gould JC. Revisional Surgery for Failed
Vertical-Banded Gastroplasty. Obes Surg. 2011;21(8):1220-1224.
127. Gumbs AA, Pomp A, Gagner M. Revisional bariatric surgery
for inadequate weight loss. Obes Surg. 2007;17:1137-1145.
128. Cariani S, Nottola D, Grani S, Vittimberga G, Lucchi A, Amenta E.
Complications after gastroplasty and gastric bypass as a primary
operation and as a reoperation. Obes Surg. 2001;11:487-490.
129. Schwartz RW, Strodel WE, Simpson WS, Griffen WO, Jr. Gastric
bypass revision: Lessons learned from 920 cases. Surgery.
1988;104:806-812.
130. Behrns KE, Smith CD, Kelly KA, Sarr MG. Reoperative
bariatric surgery. Lessons learned to improve patient selection
and results. Ann Surg. 1993;218:646-653.
131. Carrodeguas L, Szomstein S, Soto F, et al. Management of
gastrogastric fistulas after divided Roux-en-Y gastric bypass
surgery for morbid obesity: Analysis of 1,292 consecutive
patients and review of literature. Surg Obes Relat Dis. 2005;1:
467-474.
132. Filho AJ, Kondo W, Nassif LS, Garcia MJ, Tirapelle Rde A, Dotti
CM. Gastrogastric fistula: A possible complication of Rouxen-Y gastric bypass. Jsls. 2006;10:326-331.
133. van Wageningen B, Berends FJ, Van Ramshorst B, Janssen IF.
Revision of failed laparoscopic adjustable gastric banding to
Roux-en-Y gastric bypass. Obes Surg. 2006;16:137-141.
134. Gagner M, Rogula T. Laparoscopic reoperative sleeve
gastrectomy for poor weight loss after biliopancreatic diversion
with duodenal switch. Obes Surg. 2003;13:649-654.

5/21/2012 8:52:39 PM

144

Surgery: Evidence-Based Practice

135. Assalia A, Ueda K, Matteotti R, Cuenca-Abente F, Rogula T,


Gagner M. Staple-line reinforcement with bovine pericardium
in laparoscopic sleeve gastrectomy: Experimental comparative
study in pigs. Obes Surg. 2007;17:222-228.
136. Brolin RE, Cody RP. Adding malabsorption for weight loss
failure after gastric bypass. Surg Endosc. 2007;21:1924-1926.
137. Goergen M, Arapis K, Limgba A, Schiltz M, Lens V, Azagra
JS. Laparoscopic Roux-en-Y gastric bypass versus laparoscopic
vertical banded gastroplasty: Results of a 2-year follow-up
study. Surg Endosc. 2007;21:659-664.
138. Muller MK, Wildi S, Scholz T, Clavien PA, Weber M.
Laparoscopic pouch resizing and redo of gastro-jejunal
anastomosis for pouch dilatation following gastric bypass. Obes
Surg. 2005;15:1089-1095.
139. Khaitan L, Van Sickle K, Gonzalez R, Lin E, Ramshaw B, Smith
CD. Laparoscopic revision of bariatric procedures: Is it feasible?
Am Surg. 2005;71:6-10; discussion 2.
140. Nesset EM, Kendrick ML, Houghton SG, et al. A two-decade
spectrum of revisional bariatric surgery at a tertiary referral
center. Surg Obes Relat Dis. 2007;3:25-30; discussion.
141. Gonzalez R, Gallagher SF, Haines K, Murr MM. Operative
technique for converting a failed vertical banded gastroplasty
to Roux-en-Y gastric bypass. J Am Coll Surg. 2005;201:366-374.
142. Garb J, Welch G, Zagarins S, Kuhn J, Romanelli J. Bariatric
surgery for the treatment of morbid obesity: A meta-analysis of
weight loss outcomes for laparoscopic adjustable gastric banding
and laparoscopic gastric bypass. Obes Surg. 2009;19:1447-1455.
143. Buchwald H, Avidor Y, Braunwald E, et al. Bariatric surgery: A
systematic review and meta-analysis. Jama. 2004;292:1724-1737.
144. Buchwald H, Estok R, Fahrbach K, Banel D, Sledge I. Trends in
mortality in bariatric surgery: A systematic review and metaanalysis. Surgery. 2007;142:621-632; discussion 32-35.
145. Buchwald H, Estok R, Fahrbach K, et al. Weight and type 2
diabetes after bariatric surgery: Systematic review and metaanalysis. Am J Med. 2009;122:248-256, e5.
146. Benedix F, Westphal S, Patschke R, et al. Weight Loss and
Changes in Salivary Ghrelin and Adiponectin: Comparison
Between Sleeve Gastrectomy and Roux-en-Y Gastric Bypass
and Gastric Banding. Obes Surg. 2011;21(5):616-624.
147. Himpens J, Dapri G, Cadiere GB. A prospective randomized
study between laparoscopic gastric banding and laparoscopic
isolated sleeve gastrectomy: Results after 1 and 3 years. Obes
Surg. 2006;16:1450-1456.
148. Cote GA, Edmundowicz SA. Emerging technology: Endoluminal
treatment of obesity. Gastrointest Endosc 2009;70:991-999.
149. Yao S, Ke M, Wang Z, Xu D, Zhang Y, Chen JD. Retrograde
gastric pacing reduces food intake and delays gastric emptying
in humans: A potential therapy for obesity? Dig Dis Sci.
2005;50:1569-1575.
150. Hogan RB, Johnston JH, Long BW, et al. A double-blind,
randomized, sham-controlled trial of the gastric bubble for
obesity. Gastrointest Endosc. 1989;35:381-385.
151. Kirby DF, Wade JB, Mills PR, et al. A prospective assessment
of the Garren-Edwards Gastric Bubble and bariatric surgery
in the treatment of morbid obesity. Am Surg. 1990;56:
575-580.
152. Ulicny KS, Jr., Goldberg SJ, Harper WJ, Korelitz JL, Podore PC,
Fegelman RH. Surgical complications of the Garren-Edwards
Gastric Bubble. Surg Gynecol Obstet. 1988;166:535-540.
153. Benjamin SB. Small bowel obstruction and the Garren-Edwards
gastric bubble: An iatrogenic bezoar. Gastrointest Endosc.
1988;34:463-467.

PMPH_CH15.indd 144

154. Evans JT, DeLegge MH. Intragastric balloon therapy in the


management of obesity: Why the bad wrap? JPEN J Parenter
Enteral Nutr. 2011;35:25-31.
155. Goodrick GK, Poston WS, 2nd, Foreyt JP. Methods for voluntary
weight loss and control: Update 1996. Nutrition. 1996;12:
672-676.
156. Heber D, Greenway FL, Kaplan LM, Livingston E, Salvador J,
Still C. Endocrine and nutritional management of the postbariatric surgery patient: An Endocrine Society Clinical Practice
Guideline. J Clin Endocrinol Metab. 2010;95:4823-4843.
157. Chaston TB, Dixon JB, OBrien PE. Changes in fat-free mass
during significant weight loss: A systematic review. Int J Obes
(Lond). 2007;31:743-750.
158. Metcalf B, Rabkin RA, Rabkin JM, Metcalf LJ, Lehman-Becker
LB. Weight loss composition: The effects of exercise following
obesity surgery as measured by bioelectrical impedance
analysis. Obes Surg. 2005;15:183-186.
159. Jakicic JM, Clark K, Coleman E, et al. American College of
Sports Medicine position stand. Appropriate intervention
strategies for weight loss and prevention of weight regain for
adults. Med Sci Sports Exerc. 2001;33:2145-2156.
160. Akkary E, Cramer T, Chaar O, et al. Survey of the effective
exercise habits of the formerly obese. Jsls. 2010;14:106-114.
161. Skroubis G, Sakellaropoulos G, Pouggouras K, Mead N,
Nikiforidis G, Kalfarentzos F. Comparison of nutritional
deficiencies after Roux-en-Y gastric bypass and after
biliopancreatic diversion with Roux-en-Y gastric bypass. Obes
Surg. 2002;12:551-558.
162. Faintuch J, Matsuda M, Cruz ME, et al. Severe protein-calorie
malnutrition after bariatric procedures. Obes Surg. 2004;14:
175-181.
163. Bock MA. Roux-en-Y gastric bypass: The dietitians and patients
perspectives. Nutr Clin Pract. 2003;18:141-144.
164. Layman DK, Walker DA. Potential importance of leucine
in treatment of obesity and the metabolic syndrome. J Nutr.
2006;136:319S-323S.
165. Stocker DJ. Management of the bariatric surgery patient.
Endocrinol Metab Clin North Am. 2003;32:437-457.
166. Shikora SA, Kim JJ, Tarnoff ME. Nutrition and gastrointestinal
complications of bariatric surgery. Nutr Clin Pract. 2007;
22:29-40.
167. Goode LR, Brolin RE, Chowdhury HA, Shapses SA. Bone and
gastric bypass surgery: Effects of dietary calcium and vitamin
D. Obes Res. 2004;12:40-47.
168. Ott MT, Fanti P, Malluche HH, et al. Biochemical Evidence of
Metabolic Bone Disease in Women Following Roux-Y Gastric
Bypass for Morbid Obesity. Obes Surg. 1992;2:341-348.
169. Brolin RE, LaMarca LB, Kenler HA, Cody RP. Malabsorptive
gastric bypass in patients with superobesity. J Gastrointest Surg.
2002;6:195-203; discussion 4-5.
170. Marcuard SP, Sinar DR, Swanson MS, Silverman JF, Levine JS.
Absence of luminal intrinsic factor after gastric bypass surgery
for morbid obesity. Dig Dis Sci. 1989;34:1238-1242.
171. Slater GH, Ren CJ, Siegel N, et al. Serum fat-soluble vitamin
deficiency and abnormal calcium metabolism after malabsorptive
bariatric surgery. J Gastrointest Surg. 2004;8:48-55; discussion
4-5.
172. Davies DJ, Baxter JM, Baxter JN. Nutritional deficiencies after
bariatric surgery. Obes Surg. 2007;17:1150-1158.
173. Ernst B, Thurnheer M, Schmid SM, Schultes B. Evidence for the
necessity to systematically assess micronutrient status prior to
bariatric surgery. Obes Surg. 2009;19:66-73.

5/21/2012 8:52:39 PM

Evidence-Based Bariatric Surgery

174. Shah M, Simha V, Garg A. Review: Long-term impact of bariatric


surgery on body weight, comorbidities, and nutritional status. J
Clin Endocrinol Metab. 2006;91:4223-4231.
175. Xanthakos SA, Inge TH. Nutritional consequences of bariatric
surgery. Curr Opin Clin Nutr Metab Care. 2006;9:489-496.
176. Rubino F, Gagner M, Gentileschi P, et al. The early effect of
the Roux-en-Y gastric bypass on hormones involved in body
weight regulation and glucose metabolism. Ann Surg. 2004;240:
236-242.
177. Patriti A, Facchiano E, Sanna A, Gulla N, Donini A. The
enteroinsular axis and the recovery from type 2 diabetes after
bariatric surgery. Obes Surg. 2004;14:840-848.
178. Bose M, Teixeira J, Olivan B, et al. Weight loss and incretin
responsiveness improve glucose control independently after
gastric bypass surgery. J Diabetes. 2010;2:47-55.
179. Dixon JB, OBrien PE. Lipid profile in the severely obese: Changes
with weight loss after lap-band surgery. Obes Res. 2002;10:903-910.
180. Busetto L, Pisent C, Rinaldi D, et al. Variation in lipid levels
in morbidly obese patients operated with the LAP-BAND
adjustable gastric banding system: Effects of different levels of
weight loss. Obes Surg. 2000;10:569-577.
181. Zlabek JA, Grimm MS, Larson CJ, Mathiason MA, Lambert PJ,
Kothari SN. The effect of laparoscopic gastric bypass surgery
on dyslipidemia in severely obese patients. Surg Obes Relat Dis.
2005;1:537-542.
182. Williams DB, Hagedorn JC, Lawson EH, et al. Gastric bypass
reduces biochemical cardiac risk factors. Surg Obes Relat Dis.
2007;3:8-13.
183. Mallory GN, Macgregor AM, Rand CS. The Influence of
Dumping on Weight Loss After Gastric Restrictive Surgery for
Morbid Obesity. Obes Surg. 1996;6:474-478.
184. Pories WJ, Swanson MS, MacDonald KG, et al. Who would have
thought it? An operation proves to be the most effective therapy
for adult-onset diabetes mellitus. Ann Surg. 1995;222:339-350;
discussion 50-52.

PMPH_CH15.indd 145

145

185. Hsu LK, Mulliken B, McDonagh B, et al. Binge eating disorder


in extreme obesity. Int J Obes Relat Metab Disord. 2002;26:
1398-1403.
186. Ukleja A. Dumping syndrome: Pathophysiology and treatment.
Nutr Clin Pract. 2005;20:517-525.
187. Carvajal SH, Mulvihill SJ. Postgastrectomy syndromes: Dumping
and diarrhea. Gastroenterol Clin North Am. 1994;23:261-279.
188. Birkmeyer JD, Siewers AE, Finlayson EV, et al. Hospital volume
and surgical mortality in the United States. N Engl J Med.
2002;346:1128-1137.
189. Birkmeyer JD, Stukel TA, Siewers AE, Goodney PP, Wennberg
DE, Lucas FL. Surgeon volume and operative mortality in the
United States. N Engl J Med. 2003;349:2117-2127.
190. American College of Surgeons Bariatric Surgery Center Network
Program. (Accessed 02/23/2011, at http://www.acsbscn.org/
Public/AboutBSCN.jsp.)
191. Surgical Review Corporation. (Accessed 02/23/2011, at http://
www.surgicalreview.org/.)
192. CMS bariatric surgery national coverage determination.
(Accessed 2/23/2011, at https://www.cms.gov/manuals/
downloads/ncd103c1_Part2.pdf.)
193. Andrade SD. Planning and design guidelines for bariatric
healthcare facilities. AIA J. 2006; accessed 2/23/11, at http://www.
aia.org/journal_aah.cfm?pagename=aah_jrnl_20061018_
award_winner&dspl=1&article=article.
194. Livingston EH. Bariatric surgery outcomes at designated
centers of excellence vs nondesignated programs. Arch Surg.
2009;144:319-325; discussion 25.
195. Kohn GP, Galanko JA, Overby DW, Farrell TM. High case
volumes and surgical fellowships are associated with improved
outcomes for bariatric surgery patients: A justification of
current credentialing initiatives for practice and training. J Am
Coll Surg. 2010;210:909-918.
196. Livingston EH. High case volumes and bariatric surgery
outcomes. J Am Coll Surg. 2010;211:687-688; author reply 8-9.

5/21/2012 8:52:39 PM

CHAPTER 16

Gastric Adenocarcinoma
Antonio I. Picon and Martin S. Karpeh

INTRODUCTION

poorly differentiated, diffuse gastric cancer (DGC) and display


signet-ring cell features. The estimated cumulative risk for gastric
cancer by the age of 80 in HDGC families is 67% for men and 83%
for women.9 Prophylactic gastrectomy should be considered in this
high-risk group of young and asymptomatic carriers with germline
truncating CDH1 mutations, after appropriate genetic counseling
is obtained.10,11 The majority of gastric tumors are adenocarcinoma,
and histopathologically divided into two major subtypes.12 The
intestinal-type or well-differentiated tumor that originates from
gastric mucosa. This histotype is often associated with cancers of
the distal stomach, older-age patients, H. pylori infection, and a
propensity for liver metastases. The diffuse type or poorly differentiated is believed to originate from the lamina propria of the stomach, it grows along the submucosa and it has more of an infiltrative
pattern. The diffuse type is seen in younger patients, associated with
familial cancers, early metastases, and lymphatic spread.

Gastric adenocarcinoma constitutes a major health problem in


many countries around the world. According to World Health
Organizations (WHO) report, gastric cancer is the second most
common cause of cancer mortality. It is estimated that in United
States, 27,000 new gastric cancer cases were diagnosed in 2010
and 10,570 deaths from this disease.1 There has been a progressive decrease in the incidence of gastric cancer around the world
largely due to eradication of Helicobacter pylori and other risk factors that are not completely understood.2,3 There has been a dramatic increase of proximal gastric and gastroesophageal junction
(GEJ) cancers along with changes in histology, with the diff usetype adenocarcinoma increasing in prevalence.4 Across the globe
and particularly in Asia, gastric cancer remains the most common cancer among men and the most common cause of cancerrelated deaths in countries such as Japan. In Japan and other
parts of the Asia, noncardiac tumors continue to be more common compared with the West. In the United States and in many
other western countries, gastric cancer is typically diagnosed at
an advanced stage.5,6 In countries with a high incidence of gastric cancer such as Japan and Korea, there is widespread awareness of the disease, and screening programs are more widely
available, consequently, early gastric cancer (EGC) is diagnosed
more frequently in these countries.7 EGC is defined as a cancer
confined to the mucosa and submucosa independently of the
nodal status. In many cases, patients are asymptomatic and diagnosed incidentally by experienced endoscopist. The incidence of
EGC in countries where there is mass screening for gastric cancer is around 53% and 5-year survival rates are above 90% in
well-differentiated, mucosal lesions without nodal involvement.8
Environmental risk factors include H. pylori infection, high salt
intake, smoking, and other dietary factors. Patients with a family history of nonhereditary gastric cancer have a higher risk of
developing gastric cancer. E-cadherin mutations occur in approximately 25% of families with the autosomal dominant form of
hereditary diff use gastric cancer (HDGC). Pathologically, all
the gastric cancers with CDH1 mutations have shown invasive,

DIAGNOSIS AND STAGING


1. What is the ideal diagnostic and staging work-up for gastric
adenocarcinoma?
Gastric adenocarcinoma is usually associated with vague and
nonspecific gastrointestinal symptoms. In countries with high
incidence of gastric cancer and massive screening after the age of
40, the prevalence of early and asymptomatic lesion is high. In the
United States the majority of patients present with locoregional or
systemic disease that may preclude curative treatment. The diagnostic modality of choice is upper gastrointestinal endoscopy. The
identification of early lesions will depend on the experience of the
endoscopist and systematic evaluation of the stomach to detect
subtle changes in color, vascularity, or texture.8 Endoscopic ultrasound, as an adjunct to endoscopy, is used selectively.13 In institutions where endoscopist are experienced in the techniques of
endoscopic mucosal resection (EMR) or endoscopic submucosal
dissection (ESD), EUS is helpful in selecting tumors for endoscopic
removal. EUS plays an important role in determining the depth
of invasion and nodal status of the tumor that will determine the
146

PMPH_CH16.indd 146

5/21/2012 8:54:03 PM

Gastric Adenocarcinoma

subsequent treatment.14,15 It can aide in identifying locally advanced


disease that would be appropriate for neoadjuvant therapy protocols and can guide the surgeon to determine the extent of resection
and lymphadenectomy.16 It also plays a role in the surveillance of
patients after EMR or ESD.
Computed tomography (CT) of the chest and abdomen with
contrast has a long track record in the detection of metastatic disease. The sensitivity for determining nodal status has typically been
a weakness, but recently multislice CT was shown to be accurate in
detecting paraaortic nodal metastases.17 Attempts have been made
to compare CT with magnetic resonance imaging (MRI) of the
stomach for staging, no significant advantage was noted for either
modality.18 Positron emission tomography-computed tomography
(PET-CT) should be used selectively when there is high suspicious
of metastatic disease that is not evident with other modalities, and
in proximal and GEJ tumors.19 There is no role of PET-CT in the
work-up of T1 lesions where the sensitivity is just 10%.20 Mucinous
and signet-ring cell tumors may have low standardized uptake
value (SUV) in the presence of metastatic disease. Tumor markers (CEA and CA-19-9) are elevated in approximately one-third
of patients. They are useful to assess response to adjuvant therapy
and assess recurrence in those patients with elevated levels before
treatment.21,22 Tumor markers should not be used to change the
surgical treatment. Laparoscopy should be used for staging in
selective group of patients. Many have shown that laparoscopy can
change patient management in 20% to 30%.23-25 The yield from laparoscopy increases with increasing stages of disease and hence it
should be considered in patients with advanced (deep T2, T3, and
T4) tumors more than 8 cm in size, and locally advanced tumors
with or without locoregional disease on preoperative imaging.26
Laparoscopic peritoneal lavage cytology is an important prognostic factor. Patients with positive cytology behave like stage IV
gastric cancer with dismal survival in 5 years.21,27,28 The finding of
positive cytology or carcinomatosis will identify patients with a
very poor prognosis and will change the treatment algorithm of
those patients.25
ANSWER: The ideal work-up should include esophagoduodenoscopy with biopsy. EUS is used selectively when considering
endoscopic resection for selected T1 cancers or when the T stage is
needed to help choosing treatment. Chest imaging and multislice
CT of the abdomen and pelvis have a long track record for ruling
out metastatic disease and is performed in all cases. PET-CT can
be informative in cases of advanced disease and especially in GE
junction and proximal tumors, but is not helpful in staging T1
cancers. Tumor markers (CEA and CA-19-9) are of value during
surveillance, if elevated at presentation. Staging laparoscopy with
peritoneal cytology for locally advanced cancers plays an important role in detecting clinical and radiologic occult disease.

147

TREATMENT OF EGC
2. What are the treatment options in the treatment of EGC?
EGC is defined as tumors confined to the mucosa or submucosa
independently of the nodal status. Radical surgery had always been
considered the standard of care for EGC. In Japan, Korea, and other
countries with high incidence of EGCs, EMR and ESD are well
accepted techniques in the hands of experienced endoscopist, and
they have been recognized as definitive therapy in selected group
of patients.29 The selection criteria came from the paper published
by the National Cancer Center in Tokyo, Japan.30,31 The results of
this large retrospective analysis of prospectively collected data
revealed that none of the well-differentiated intramucosal cancers of less than 30 mm in diameter, regardless of the presence of
ulceration; intramucosal cancers without ulceration, regardless of
size, and well-differentiated adenocarcinomas of less than 30 mm,
without lymphovascular invasion were associated with lymph node
metastases. Of note, tumors invading the submucosa, but less than
0.5 cm in size were also all free of lymph node metastases. The
selection criteria for EMR or ESD are listed in Table 16.1.
ESD is considered curative for patients that meet the following criteria: well-to-moderately differentiated histopathology;
negative vertical margin (intramucosal lesion or extension into the
submucosa for less than 500 m); negative lateral margin; and no
lymphatic or vascular invasion. If these guidelines are followed,
published data suggests that patients will be at minimal or no risk
of nodal metastases and results will be comparable to radical surgery. If after EMR or ESD the above criteria are not met, patient
should undergo radical surgery. The exception to the rule is a positive lateral margin that may undergo repeat ESD, if the endoscopist
and pathologist are comfortable repeating the procedure. The most
common complication after EMRESD is delayed bleeding with an
incidence up to 7% to 8% and perforation in about 4% of cases.31
Follow-up after ESD consists of repeating EGD at 3 months and
annually thereafter. CT scan should be obtained at 1 year for distant metastases and EUS in 3 to 6 months for nodal disease in the
selected group of patients who refused radical surgery. For patients
who have EGC, but refuse or do not meet the criteria for endoscopic
resections, subtotal gastrectomy with a D1 dissection or modified
D1 lymphadenectomy,32 pylorus preserving gastrectomy,33 or proximal gastrectomy with or without jejunal interposition is an option
supported by Level II-3 evidence.34
ANSWER: EMR and ESD are comparable techniques to radical surgery in the treatment of EGC that meet the strict criteria
for endoscopic treatments based on large Level II-3 data sources.
There are no randomized trials comparing EMR and gastrectomy. Radical surgery with limited lymphadenectomy (modified

Table 16.1 EGC: Criteria for Curative Endoscopic Resection


Mucosal Cancer
No ulceration

Submucosal Cancer
Ulcerated

SM1 (<500 m)

SM2 (>500 m)

Size (mm)

20

>20

30

>30

30

Any size

Intestinal

EMR

ESD

ESD

Gastrectomy + LN
dissection

ESD

Gastrectomy + LN
dissection

Diffuse

Consider
Surgery

Gastrectomy + LN dissection

PMPH_CH16.indd 147

5/21/2012 8:54:04 PM

148

Surgery: Evidence-Based Practice

gastrectomy type A [MGA] or type B [MGB]) for EGC that do not


meet the criteria for endoscopic procedures should be offered as
the standard of care.

TREATMENT OF ADVANCE
GASTRIC CANCER
3. When should neoadjuvant therapy be considered?
The rational for using neoadjuvant therapy is based on the assumption that occult systemic disease will be best treated early before
resecting the primary tumor thus avoiding the immunosuppressive and debilitating effects of surgery and increasing treatment
efficacy. The primary goal of this therapy is to improve progression-free survival (PFS) and overall survival (OS). Potential benefits of neoadjuvant therapy are that it is better tolerated prior to
surgery, patients tend to get more of the intended treatment, the
tumors can be downstaged, response to therapy can be assessed,
and surgery can be avoided in patient that progress.16
The British Medical Research Council provided the first
level I evidence for a conclusive survival benefit for R0 resection
following the use of preoperative chemotherapy in patients
with resectable gastric, gastroesophageal, and distal esophageal
adenocarcinomas over surgical resection alone.35 The MAGIC
trial used preoperative and postoperative epirubicin, cisplatin,
and 5-fluorouracil (ECF) combination chemotherapy. Criticisms
of this trial included the lack of optimal preoperative staging and
subgroup analysis by stage, but the selection criteria were consistent with established treatment the study was powered to show a
survival benefit for the entire treatment cohort rather than a select
subgroup. There was no difference in the postoperative complications or operative mortality between the groups. The response
rate as assessed by tumor size was significantly improved in the
group randomized to chemotherapy. The study demonstrated
improved resectability, PFS, and OS. Assessment of response
to neoadjuvant therapy by PET-CT is not only used as a guide
in the treatment of these patients, but also metabolic as well as
pathologic response has been correlated with better outcome.36,37
The role of neoadjuvant chemoradiation therapy in patients with
potentially resectable esophageal and cardia adenocarcinomas
was supported in a sentinel study by Walsh et al.38 This study was
criticized for the lack of rigid preoperative staging measures and
for the poor survival results of the surgery alone group, but the
randomization was sound and the selection criteria reflected the
standards used at the time. The multimodality treated group had
a 25% complete response rate and a significant improvement in
3-year survival over surgery alone. Other phase II and III trials
have shown improved pathologic response and survival advantages when preoperative chemoradiation therapy was used.39-42 In
a phase III trial of 126 patients with locally advanced adenocarcinoma of the lower esophagus or gastric cardia, patients were
randomized to chemotherapy followed by surgery or chemoradiation therapy followed by surgery. In patients receiving chemoradiation therapy, there was a significant probability of showing
a complete response or tumor-free lymph nodes at resection.
Operative mortality was not significantly increased in the chemoradiation group. Preoperative chemoradiation therapy improved
3-year survival rate, but was not statistically significant.39 There
is no large prospective randomized trial assessing the role of preoperative chemoradiation in gastric cancer. Its value has yet to be

PMPH_CH16.indd 148

determined as most studies have been limited to proximal gastric


and GEJ tumors.
ANSWER: Neoadjuvant chemotherapy should be recommended for patients at risk of systemic failure. Tumors with radiologic and endoscopic evidence of locally advanced disease (>T3
or N positive) in patients without symptoms requiring palliative
therapy, and without evidence of distant disease or positive peritoneal cytology should be considered for neoadjuvant treatment.
The goal is to treat micrometastatic disease, improve the R0 resection rate, and prolong survival.
4. What is the ideal extent of resection and lymphadenectomy?
The type and extent of resection and the lymphadenectomy to
be performed will depend on the location and the clinical stage
of the tumor. Treatment should be tailored according to the biology of the tumor to avoid under- or overtreatment of the disease.
The controversy regarding the extent of resection, total versus
subtotal gastrectomy, was answered in three prospective randomized trial comparing both techniques.43-45 There was no difference
in survival after R0 resection. An analysis of a large prospective
database found proximal subtotal gastrectomy to have similar
survival results compared with total gastrectomy in the treatment
of early proximal gastric cancers.46 Proximal gastrectomy is an
option in the treatment of early proximal gastric cancers, but it
has been associated with acid reflux and esophagitis.47 Proximal
gastrectomy has been associated with few long-term complications
when performed with jejunal interposition and with excellent
outcome.34,48 Gastric adenocarcinoma, especially the diffuse type,
tends to spread submucosally several centimeters from the primary
tumor, the duodenal margin should be at least 3 cm and proximal
margin should be at least 6 cm, if possible, to minimize the risk of
leaving a positive margin.49 The management of R1 resection (positive macroscopic margin) should be addressed taking into account
the surgical risk to the patient and the biology of the tumor. In one
retrospective study, patients with five or more positive nodes had
the same poor survival regardless of margin status.50 The goal of
surgery is to achieve an R0 resection with minimal morbidity and
mortality to maximize the probability of cure. When faced with a
positive margin, the presence of positive cytology and/or N2-N3
disease, reresection may not make a difference in survival.51
The extent of lymphadenectomy should be tailored according
to the clinical stage of the disease.52 D0 lymphadenectomy is done
when no attempt is made to harvest perigastric lymph nodes and is
done when resection is performed only when palliation is the goal.
D1 lymphadenectomy is done when the perigastric lymph nodes
(stations 16 in the Japanese nomenclature), and a D2 lymphadenectomy is done when D1 nodes are included with lymph nodes along
the left gastric artery, splenic artery, and the celiac axis. A modified
lymphadenectomy has been advocated for treating EGC patients.
MGA includes 2/3 subtotal gastrectomy and a D1 lymphadenectomy
plus the left gastric artery nodes (station 7) and it should include
celiac axis nodes (station 8a) for lower third cancers. MGA is indicated for tumors confined to the mucosa, but not eligible for endoscopic removal. MGB includes 2/3 subtotal gastrectomy, D1 plus
station 7, 8a, and 9 (common hepatic artery nodes). MGB is indicated for deeper T1 tumors confined to the submucosa. Extended
D3 and D4 lymphadenectomies have been shown not to increase
survival, therefore they are not currently recommended.53,54
The ideal extent of the lymphadenectomy has been debated
for decades. There are no Japanese randomized control trials

5/21/2012 8:54:04 PM

Gastric Adenocarcinoma

(RCT) comparing D1 and D2 dissections. The advantage of D2


compared with D1 was reported by the Japanese in large retrospective analysis.55,56 Two well-designed prosective randomized
western trials did not show a survival advantage for routine use
of the D2 lymphadenectomy versus the D1 and demonstrated an
increase in morbidity and mortality.57,58 Much has been written
extolling the positives and negatives of these trials, the morbidity
and mortality was higher than expected, patients had less extensive disease and better OS than expected raising questions about
the power of these trials to show a difference.59
Routine distal pancreatectomy and splenectomy in the D2
lymphadectomy group was associated with increased morbidity
and mortality in both the MRC and Dutch trials. Distal pancreatectomy and/or splenectomy should be performed selectively
when there is direct organ involvement.58,60,61 It has been shown
that the number of positive lymph nodes, and not the location, is a
more reliable measure of prognosis. The current recommendation
is to harvest at least 15 lymph nodes to optimize staging.62
ANSWER: The extent of gastrectomy and lymphadenectomy
should be tailored according to the stage of the tumor. A subtotal gastrectomy should be sufficient provided that an R0 resection
can be achieved. An MGA or MGB is recommended for EGC.
For advanced gastric cancers expert opinion would favor radical
gastectomy with a D2 lymphadenectomy, but the best evidence
currently supports gastrectomy with D1 lymphadenectomy in
western patients. Extended lymphadenectomy (D3-D4) and routine pancreatectomy and/or splenectomy are not supported by
the data. A minimum of 15 lymph nodes should be harvested for
adequate staging of advanced gastric cancers.
5. When should adjuvant therapy be considered?
A number of meta-analyses of RCT have examined the impact
of postoperative chemotherapy in the treatment of gastric cancer
compared with surgery alone. A recent study looked at individual
patient data from 17 trials (3838 patients) representing 60% of the
targeted data with more than 7 years of median follow-up. Adjuvant
5-fluorouracil-based chemotherapy was associated with a significant OS benefit (hazard ratio [HR], 0.82; 95% confidence interval
[CI], 0.760.90; P < 0.001) and disease-free survival (HR, 0.82; 95%
CI, 0.750.90; P < 0.001).63 A large phase III Japanese trial randomized patients with stages II and III gastric cancer who underwent
gastrectomy with D2 or more extensive lymphadenectomy, with R0
resection and no hepatic, peritoneal or distant metastasis. Patients
were randomly assigned to undergo surgery followed by S-1 chemotherapy or surgery alone. S-1 chemotherapy showed superiority
to surgery alone with improved 3-year OS rate.64
The results of the intergroup 0116 prospective randomized trial
comparing 5-fluorouracil and leucovorin and external beam radiation therapy with observation set the standard for adjuvant chemoradiation therapy after surgery in the United States. This trial has
been criticized for poor quality control of surgical therapy.65 The
trial contained a large percentage of T3 and node-positive tumors
and reanalysis of the data suggested that the degree of residual disease post resection was not insignificant.66,67 There were no T1N0,
and just 35 stage-IB patients were randomized leaving open questions regarding suitability of this regimen for early-stage tumors.
ANSWER: There is insufficient data to recommended adjuvant therapy for node negative T1 and T2 gastric cancers. In nodepositive and advanced gastric cancers, adjuvant chemoradiation
therapy should be offered due to the high locoregional and systemic

PMPH_CH16.indd 149

149

recurrence rate.63,68 An acceptable minimal alternative after adequate R0 resection of advanced gastric cancers would be 5-fluorouracil or S-1 based chemotherapy. The high incidence of recurrence
after R0 resection of advanced gastric cancer makes it imperative to
recommend adjuvant therapy to optimize survival probability.
6. What is the ideal surgical management of stage IV disease?
Treatment options in the management of stage IV gastric cancer
are and have been controversial in part due to historic data supporting the limited option of surgery alone for palliation. There is a
distinct lack of prospective randomized and adequate prospective
studies evaluating surgical treatment in these patients. With current refinements, chemotherapy patients with metastatic disease
have been shown to improve their quality of life compared with
best supportive care. The patients should be offered chemotherapy
and there is no standard regimen currently recommended. In a
recent Cochrane review of chemotherapy versus best supportive
care, both single agent and combination chemotherapy regimen
resulted in superior palliation and improved survival.69 There is
a trend to use combination of drugs and oral fluoropyrimidines
in combination with other drugs have shown to be effective.70-72
In HER2-positive disease addition of trastuzumab to chemotherapy improves survival for patients with locally advanced disease,
recurrent or metastatic gastric cancer.73
In a recent retrospective review of stage IV gastric cancer
diagnoses using the SEER database, the authors analyzed three
subgroups divided on the basis on whether cancer-directed therapy was recommended but not performed, recommended and
performed, or not recommended. Patients who had recommended
cancer-directed surgery had significantly improved survival. They
concluded that highly selected group of stage IV gastric cancer
patients who undergo surgery have significantly greater survival
than unresected patients, including those patients that surgery was
recommended but not performed.74 This retrospective review suggests that surgeons were able to select those of stage IV patients
that could tolerate and benefit from surgical resection. It is of interest that Asian race and age less than 60 years were associated with
prolonged survival. The authors concluded that surgery should be
considered in highly selected stage IV patients with acceptable surgical risk.75 Palliative gastrectomy for incurable disease has been
associated with significant morbidity and mortality.76 The majority
of patients with stage IV cancer can have their symptoms palliated
without surgery. Despite these facts, a poorly defined group of stage
IV patients undergoing surgery had longer survival compared with
patients undergoing other types of intervention.77-79 Prospective
randomized trials of defined stage IV patients are needed to clearly
identify which patients will benefit most from resection.
In a multicenter randomized trial comparing surgical gastrojejunostomy (GJJ) and endoscopic stent placement for the palliation of malignant gastric outlet obstruction in unresectable or
metastatic, GJJ was associated with better long-term results and
is the treatment of choice in patients with a life expectancy of
2 months or longer. For those patients with a life expectancy of
less than 2 months, they favored stent placement.80
ANSWER: Stage IV gastric patients should be offered chemotherapy as first-line therapy to prolong survival and palliation of
symptoms. In highly selected group of patients, with good performance status surgery can be considered. In the presence of gastric
outlet obstruction, GJJ should be offered in patients with a life
expectancy of 2 months or longer.

5/21/2012 8:54:04 PM

150

Surgery: Evidence-Based Practice

REFERENCES
1. Jemal A, et al. Cancer statistics, 2010. CA Cancer J Clin. 2010;
60(5):277-300.
2. Hirayama T. Epidemiology of cancer of the stomach with special
reference to its recent decrease in Japan. Cancer Res. 1975;35(11
Pt. 2):3460-3463.
3. Kuipers EJ, Pena AS, Meuwissen SG. H. pylori and gastric cancer: Limitations of retrospective studies. Gastroenterology.
1994;106(5):1398-1400.
4. Henson DE, et al. Differential trends in the intestinal and diffuse types of gastric carcinoma in the United States, 1973-2000:
Increase in the signet ring cell type. Arch Pathol Lab Med.
2004;128(7):765-770.
5. Kampschoer GH, Nakajima T, van de Velde CJ. Changing patterns in gastric adenocarcinoma. Br J Surg. 1989;76(9):914-916.
6. Powell J, McConkey CC. Increasing incidence of adenocarcinoma of the gastric cardia and adjacent sites. Br J Cancer. 1990;
62(3):440-443.
7. Talley NJ, Fock KM, Moayyedi P. Gastric Cancer Consensus
conference recommends Helicobacter pylori screening and
treatment in asymptomatic persons from high-risk populations to prevent gastric cancer. Am J Gastroenterol. 2008;103(3):
510-514.
8. Ang TL, Khor CJ, Gotoda T. Diagnosis and endoscopic resection
of early gastric cancer. Singapore Med J. 2010;51(2):93-100.
9. Pharoah PD, Guilford P, Caldas C. Incidence of gastric cancer
and breast cancer in CDH1 (E-cadherin) mutation carriers from
hereditary diff use gastric cancer families. Gastroenterology.
2001;121(6):1348-1353.
10. Medina-Franco H, Barreto-Zuniga R, Garcia-Alvarez MN. Preemptive total gastrectomy for hereditary gastric cancer. J Gastrointest Surg. 2007;11(3):314-317.
11. Lynch HT, Lynch JF. Hereditary diff use gastric cancer: Lifesaving total gastrectomy for CDH1 mutation carriers. J Med Genet.
2010;47(7):433-435.
12. Lauren P. The two histological main types of gastric carcinoma:
Diff use and so-called intestinal-type carcinoma. An attempt at
a histo-clinical classification. Acta Pathol Microbiol Scand. 1965;
64:31-49.
13. Ang TL, et al. Accuracy of endoscopic ultrasound staging of gastric cancer in routine clinical practice in Singapore. Chin J Dig
Dis. 2006;7(4):191-196.
14. Puli SR, et al. How good is endoscopic ultrasound for TNM staging of gastric cancers? A meta-analysis and systematic review.
World J Gastroenterol. 2008;14(25):4011-4019.
15. Bentrem D, et al. Clinical correlation of endoscopic ultrasonography with pathologic stage and outcome in patients undergoing curative resection for gastric cancer. Ann Surg Oncol. 2007;
14(6):1853-1859.
16. Mezhir JJ, Tang LH, Coit DG. Neoadjuvant therapy of locally
advanced gastric cancer. J Surg Oncol. 2010;101(4):305-314.
17. Marrelli D, et al. High Accuracy of Multislices Computed
Tomography (MSCT) for Para-Aortic Lymph Node Metastases
from Gastric Cancer: A Prospective Single-Center Study. Ann
Surg Oncol. 2011;10:365-373.
18. Maccioni F, et al. Preoperative T and N staging of gastric cancer: Magnetic Resonance Imaging (MRI) versus Multi Detector
Computed Tomography (MDCT). Clin Ter. 2010;161(2):e57-e62.
19. Flamen P. Positron emission tomography in gastric and esophageal cancer. Curr Opin Oncol. 2004;16(4):359-363.
20. Shoda H, et al. Evaluation of 18F-2-deoxy-2-fluoro-glucose positron
emission tomography for gastric cancer screening in asymptomatic

PMPH_CH16.indd 150

21.

22.

23.

24.
25.

26.
27.

28.
29.

30.

31.
32.
33.

34.

35.

36.

37.

38.

39.

40.

41.

individuals undergoing endoscopy. Br J Cancer. 2007;97(11):


1493-1498.
Kochi M, et al. Evaluation of serum CEA and CA19-9 levels as
prognostic factors in patients with gastric cancer. Gastric Cancer.
2000;3(4):177-186.
Marrelli D, et al. Prognostic significance of CEA, CA 19-9 and
CA 72-4 preoperative serum levels in gastric carcinoma. Oncology. 1999;57(1):55-62.
Conlon KC, Karpeh MS, Jr. Laparoscopy and laparoscopic
ultrasound in the staging of gastric cancer. Semin Oncol.
1996;23(3):347-351.
Burke EC, et al. Laparoscopy in the management of gastric adenocarcinoma. Ann Surg. 1997;225(3):262-267.
Lehnert T, et al. Impact of diagnostic laparoscopy on the management of gastric cancer: prospective study of 120 consecutive patients with primary gastric adenocarcinoma. Br J Surg.
2002;89(4):471-475.
Shimizu H, et al. Usefulness of staging laparoscopy for advanced
gastric cancer. Surg Today. 2010;40(2):119-124.
Bentrem D, et al. The value of peritoneal cytology as a preoperative predictor in patients with gastric carcinoma undergoing a
curative resection. Ann Surg Oncol. 2005;12(5):347-353.
Fukagawa T, et al. Significance of lavage cytology in advanced
gastric cancer patients. World J Surg. 2010;34(3):563-568.
Gotoda T, et al. Endoscopic resection of early gastric cancer
treated by guideline and expanded National Cancer Centre criteria. Br J Surg. 2010;97(6):868-871.
Sano T, Kobori O, Muto T. Lymph node metastasis from early
gastric cancer: Endoscopic resection of tumour. Br J Surg.
1992;79(3):241-244.
Gotoda T. Endoscopic resection of early gastric cancer. Gastric
Cancer. 2007;10(1):1-11.
Shimada Y. JGCA (The Japan Gastric Cancer Association). Gastric cancer treatment guidelines. Jpn J Clin Oncol. 2004;34(1):58.
Shibata C, et al. Outcomes after pylorus-preserving gastrectomy
for early gastric cancer: A prospective multicenter trial. World J
Surg. 2004;28(9):857-861.
Katai H, et al. Long-term outcome after proximal gastrectomy
with jejunal interposition for suspected early cancer in the upper
third of the stomach. Br J Surg. 2010;97(4):558-562.
Cunningham D, et al. Perioperative chemotherapy versus surgery alone for resectable gastroesophageal cancer. N Engl J Med.
2006;355(1):11-20.
Weber WA, et al. Prediction of response to preoperative chemotherapy in adenocarcinomas of the esophagogastric junction by
metabolic imaging. J Clin Oncol. 2001;19(12):3058-3065.
Ott K, et al. Prediction of response to preoperative chemotherapy
in gastric carcinoma by metabolic imaging: Results of a prospective trial. J Clin Oncol. 2003;21(24):4604-4610.
Walsh TN, et al. A comparison of multimodal therapy and surgery for esophageal adenocarcinoma. The New England journal
of medicine. 1996;335(7):462-467.
Stahl M, et al. Phase III comparison of preoperative chemotherapy compared with chemoradiotherapy in patients with locally
advanced adenocarcinoma of the esophagogastric junction. J
Clin Oncol. 2009;27(6):8516.
Ajani JA, et al. Paclitaxel-based chemoradiotherapy in localized gastric carcinoma: Degree of pathologic response and not
clinical parameters dictated patient outcome. J Clin Oncol.
2005;23(6):1237-1244.
Ajani JA, et al. Multi-institutional trial of preoperative chemoradiotherapy in patients with potentially resectable gastric carcinoma. J Clin Oncol. 2004;22(14):2774-2780.

5/21/2012 8:54:04 PM

Gastric Adenocarcinoma

42. Ajani JA, et al. Phase II trial of preoperative chemoradiation in


patients with localized gastric adenocarcinoma (RTOG 9904):
Quality of combined modality therapy and pathologic response.
J Clin Oncol. 2006;24(24):3953-3958.
43. Bozzetti F, et al. Subtotal versus total gastrectomy for gastric cancer: Five-year survival rates in a multicenter randomized Italian trial. Italian Gastrointestinal Tumor Study Group. Ann Surg.
1999;230(2):170-178.
44. Robertson CS, et al. A prospective randomized trial comparing
R1 subtotal gastrectomy with R3 total gastrectomy for antral
cancer. Ann Surg. 1994;220(2):176-182.
45. Gouzi JL, et al. Total versus subtotal gastrectomy for adenocarcinoma of the gastric antrum. A French prospective controlled
study. Ann Surg. 1989;209(2):162-166.
46. Harrison LE, Karpeh MS, Brennan MF. Total gastrectomy is not
necessary for proximal gastric cancer. Surgery. 1998;123(2):127-130.
47. Hsu CP, et al. Esophageal reflux after total or proximal gastrectomy in patients with adenocarcinoma of the gastric cardia. Am
J Gastroenterol. 1997;92(8):1347-1350.
48. Katai H, et al. Prospective study of proximal gastrectomy for
early gastric cancer in the upper third of the stomach. Br J Surg.
2003;90(7):8503.
49. Bozzetti F, et al. Adequacy of margins of resection in gastrectomy for cancer. Ann Surg. 1982;196(6):685-690.
50. Kim SH, et al. Effect of microscopic resection line disease on gastric cancer survival. J Gastrointest Surg. 1999;3(1):24-33.
51. Sun Z, Xu HM. Stage and microscopic positive margins in
the treatment of patients with gastric cancer. Ann Surg Oncol.
2010;17(3):943-945.
52. Nakajima T. Gastric cancer treatment guidelines in Japan. Gastric Cancer. 2002;5(1):1-5.
53. Yonemura Y, et al. Randomized clinical trial of D2 and extended
paraaortic lymphadenectomy in patients with gastric cancer. Int
J Clin Oncol. 2008;13(2):132-137.
54. Sasako M, et al. D2 lymphadenectomy alone or with para-aortic
nodal dissection for gastric cancer. N Engl J Med. 2008;359(5):
453-462.
55. Sano T, et al. Recurrence of early gastric cancer. Follow-up of
1475 patients and review of the Japanese literature. Cancer.
1993;72(11):3174-3178.
56. Maruyama K, et al. Lymph node metastases of gastric cancer.
General pattern in 1931 patients. Ann Surg. 1989;210(5):596-602.
57. Bonenkamp JJ, et al. Extended lymph-node dissection for gastric
cancer. N Engl J Med. 1999.340(12):908-914.
58. Cuschieri A, et al. Patient survival after D1 and D2 resections
for gastric cancer: long-term results of the MRC randomized surgical trial. Surgical Co-operative Group. Br J Cancer.
1999;79(910):1522-1530.
59. Fayers PM, et al. Sample size calculation for clinical trials: the
impact of clinician beliefs. Br J Cancer. 2000;82(1):213-219.
60. Otsuji E, et al. Total gastrectomy with simultaneous pancreaticosplenectomy or splenectomy in patients with advanced gastric
carcinoma. Br J Cancer. 1999;79(1112):1789-1793.
61. Takeuchi K., et al. Total gastrectomy with distal pancreatectomy and splenectomy for advanced gastric cancer. J Surg Res.
2001;101(2):196-201.

PMPH_CH16.indd 151

151

62. Karpeh MS, et al. Lymph node staging in gastric cancer: Is location more important than Number? An analysis of 1,038 patients.
Ann Surg. 2000;232(3):362-371.
63. Paoletti X, et al. Benefit of adjuvant chemotherapy for resectable
gastric cancer: a meta-analysis. JAMA. 2010;303(17):1729-1737.
64. Sakuramoto S, et al. Adjuvant chemotherapy for gastric
cancer with S-1, an oral fluoropyrimidine. N Engl J Med.
2007;357(18):1810-1820.
65. Macdonald JS, et al. Chemoradiotherapy after surgery compared
with surgery alone for adenocarcinoma of the stomach or gastroesophageal junction. N Engl J Med. 2001;345(10):725-730.
66. Hundahl SA, et al. Improved regional control and survival with low Maruyama Index surgery in gastric cancer:
Autopsy findings from the Dutch D1-D2 Trial. Gastric Cancer.
2007;10(2):84-86.
67. Douglass HO, Jr., et al. Gastric cancer: D2 dissection or low Maruyama Index-based surgerya debate. Surg Oncol Clin N Am.
2007;16(1):133-155.
68. Panzini I, et al. Adjuvant chemotherapy in gastric cancer: A
meta-analysis of randomized trials and a comparison with previous meta-analyses. Tumori. 2002;88(1):21-27.
69. Wagner AD, et al. Chemotherapy for advanced gastric cancer.
Cochrane Database Syst Rev. 2010;(3):CD004064.
70. Cunningham D, et al. Capecitabine and oxaliplatin for advanced
esophagogastric cancer. N Engl J Med. 2008;358(1):36-46.
71. Kang YK, et al. Capecitabine/cisplatin versus 5-fluorouracil/
cisplatin as first-line therapy in patients with advanced gastric
cancer: A randomised phase III noninferiority trial. Ann Oncol.
2009;20(4):666-673.
72. Ajani JA, et al. Multicenter phase III comparison of cisplatin/S-1
with cisplatin/infusional fluorouracil in advanced gastric or gastroesophageal adenocarcinoma study: The FLAGS trial. J Clin
Oncol. 2010;28(9):1547-1553.
73 . Roukos DH. Targeting gastric cancer with trastuzumab: New
clinical practice and innovative developments to overcome resistance. Ann Surg Oncol. 2010;17(1):14-17.
74. Smith JK, et al. Potential benefit of resection for stage IV gastric
cancer: A national survey. J Gastrointest Surg. 2010;14(11):16601668.
75. Lim S, et al. Results following resection for stage IV gastric cancer; Are better outcomes observed in selected patient subgroups?
J Surg Oncol. 2007;95(2):118-122.
76. Wanebo HJ, et al. Cancer of the stomach. A patient care study
by the American College of Surgeons. Ann Surg. 1993;218(5):
583-592.
77. Doglietto GB, et al. Surgery: Independent prognostic factor in curable and far advanced gastric cancer. World J Surg.
2000;24(4):459-463; discussion 464.
78. Hanazaki K, et al. Palliative gastrectomy for advanced gastric
cancer. Hepatogastroenterology. 2001;48(37):285-289.
79. Meijer S, De Bakker OJ, Hoitsma HF. Palliative resection in gastric cancer. J Surg Oncol. 1983;23(2):77-80.
80. Jeurnink SM, et al. Surgical gastrojejunostomy or endoscopic
stent placement for the palliation of malignant gastric outlet
obstruction (SUSTENT study): a multicenter randomized trial.
Gastrointest Endosc. 2010;71(3):490-499.

5/21/2012 8:54:04 PM

Commentary on Gastric
Adenocarcinoma
Scott A. Hundahl

ment can be delivered with no increase in surgical mortality. And


because neoadjuvant protocols allow surgeons to deselect those
with rapid progression/metastases, results for those completing
surgical treatment generally appear enhanced.
The section on surgical resection and lymphadenectomy succinctly covers a truly vast surgical controversy. Some additional
trials and reports deserve mention. They include the following:

With a slightly different lumper view of epidemiology, one


can discern three broad categories of nonhereditary gastric carcinoma, with the first two closely linked to Heliobacter pylori
infection1: (1) Intestinal-type tumors arising in long-standing
(H. pylorirelated) intestinal metaplasia. These often arise near
the antrum-corpus junction along the lesser curvature. This type
of disease typifies the pattern seen in epidemic areas, especially
in Asia. (2) Diff use-type tumors arising in the body, typically
associated with intense, H. pylorirelated gastritis, but with negligible intestinal metaplasia. (3) Gastroesophageal junction tumors
whose incidence continues to increase in the United States and
in most Western countries. These tumors are associated with
obesity and reflux, but not with H. pylori infection.
With respect to staging, readers should be aware that current
seventh edition AJCC/UICC staging, to be used for all cases after
January 1, 2010, represents a major departure from previous versions. For example, tumors arising from the proximal 5 cm of
the stomach and extending to the gastroesophageal junction, as
well as all EG junction tumors, are now classified, for staging purposes, as esophageal cancer (i.e., not stomach).2 This, combined
with other changes, has created substantial migration of site and
stage from previous editions. The authors have, perhaps diplomatically, skipped over the potentially impact of these changes on
treatment (e.g., teams, paradigms).
A second staging issue should be highlighted. Japanese
Gastric Cancer Association (JGCA) staging (a.k.a. the General
Rules) represents an internationally popular alternative staging
system used throughout Asia. This system has also undergone
considerable revision over years.3 For example, the N4 designation was eliminated, and, with it, the term D4 lymphadenectomy. And given the redefinition of N2 nodes, a modern D2
operation approximates the D3 of former years. The reshuffling
of nodal designations has created confusion among non-Japanese
surgeons. To clarify current definitions, an updated third English version has recently been published.4
The authors appropriately highlight the revolution in early
gastric cancer (EGC) management represented by endoscopic
resection techniques. The selection criteria in Table 1 are largely
based on nodal analysis from a 5,265-patient cohort of surgically
treated EGC patients treated at the two major cancer centers in
Tokyo.5 This classic 2000 paper highlighted subgroups with zero
nodal involvement and merits close study.
The section on neoadjuvant treatment emphasizes a key finding, but one which still generates controversy: neoadjuvant treat-

1. A Taiwanese prospective randomized single-institution trail


by Wu et al.6 In a setting of 0% postoperative mortality, the
investigators observed a 5.9% survival advantage to D3
lymphadenectomy over D1 lymphadenectomy (59.5% vs.
53.6%, p = .04). A related 10.3% difference in recurrence
at 5 years was reported, but this did not reach significance
(p = .197). Because of changes in JGCA definitions, what
was, at the time of trials initiation, a D3 operation actually
approximates a modern D2 lymphadenectomy.
2. Dutch D1-D2 Trial updates with 10- and 15-year follow-up and
are available.7,8 Songun et al. reported that at 15 years, death
from gastric cancer was 48% for D1 versus 37% for D2 (p = .01),
but, as noted, the death from toxicity of treatment was 4% for
the D1 cohort and 10% for the D2 cohort. Overall survival was
21% for D1 and 29% for D2 (p = n.s.).
3. Proof that modern pancreas-preserving D2 surgery, as
advocated by Maruyama,9 can be safely performed in a
European population is provided by a well-done 191-patient
Italian study by Deguili et al. in which in-hospital mortality
was only 3.1%.10,11 Thus, many gastric cancer experts continue
to favor pancreas-preserving D2 surgery for patients with localregional T2-or-greater disease.
An alternative method of customizing a lymphadenectomy
involves use of the Maruyama Program12,13 to guide surgeons to
an operation which does not leave regional nodal disease behind.
The concept of low Maruyama Index gastric cancer surgery was
developed by the author (S.H.) for the Intergroup 0116 Macdonald chemoradiation trial to prospectively analyze the survival
impact of variation in surgical lymphadenectomy. The Maruyama
Index is simply the sum of Maruyama Program predictions of disease in regional node stations #1 through #12, which the surgeon
leaves in the patient. In the Macdonald chemoradiation study,
with T and N as covariates, Maruyama Index proved to be an
independent predictor of survival overall and for both treatment
groups.14 A retrospective blinded analysis of Maruyama Index in
the Dutch D1 versus D2 trial was subsequently performed. Again,
152

PMPH_CH16.indd 152

5/21/2012 8:54:04 PM

Gastric Adenocarcinoma

Maruyama Index proved to be an independent predictor of survival, and compelling doseresponse was noted.15 Autopsy analysis among Dutch Trial patients expiring within 10 years indicated
that low Maruyama Index surgery was associated with lower
local-regional failure.16
The authors summarize the importance of properly selecting
gastric cancer patients who may benefit from surgical palliation
in the face of unresectable or disseminated disease. Selection of
appropriate candidates might also be guided by results of a nonrandomized analysis of 285 ineligible Dutch Trial cases with good
performance status who had Stage IV disease.17 Overall survival
time was greater if a resection was performed (8.1 vs. 5.4 months;
p < .001). For patients aged above 70 years, a survival advantage of
about 3 months with resection was observed, but morbidity and
perioperative mortality in such patients was very high at 50% and
20%, respectively. Patients with only one metastatic site benefitted
form resection (survival 10.5 vs. 6.7 months; p = .034). For patients
with two or more metastatic sites, however, resection carried no
significant survival advantage (5.7 vs. 4.6 months; p = .084). Hartgrink et al. concluded that patients with good performance status
aged below 70 years with one metastatic site appear reasonable
candidates for palliative resection.

REFERENCES
1. Stemmermann G, Fenoglio-Preiser C. Gastric cancer: epidemiology. In: Kelsen D, Daly J, Kern S, Levin B, Tepper J, eds. Gastrointestinal Oncology: Principles and Practice. Philadelphia: Lippincott
Williams & Wilkins; 2002:p. 311-324.
2. Edge SBB, DR, Compton CC, Fritz AG, Greene FL, Trotti A. AJCC
Cancer Staging Manual. 7th ed. New York: Springer; 2009.
3. Sayegh ME, Sano T, Dexter S, Katai H, Fukagawa T, Sasako M.
TNM and Japanese staging systems for gastric cancer: How do
they coexist? Gastric Cancer. 2004;7(3):140-148.
4. Sano T KYe. Japanese classification of gastric carcinoma: 3rd English edition. Gastric Cancer. 2011;14(2):101-112.
5. Gotoda T, Yanagisawa A, Sasako M, Ono H, Nakanishi Y, Shimoda T, et al. Incidence of lymph node metastasis from early gastric cancer: estimation with a large number of cases at two large
centers. Gastric Cancer. 2000;3(4):219-225.

PMPH_CH16.indd 153

153

6. Wu CW, Hsiung CA, Lo SS, Hsieh MC, Chen JH, Li AF, et al.
Nodal dissection for patients with gastric cancer: A randomised
controlled trial. Lancet Oncol. 2006;7(4):309-315.
7. Hartgrink HH, van de Velde CJ. Status of extended lymph node
dissection: Locoregional control is the only way to survive gastric cancer. J Surg Oncol. 2005;90(3):153-165.
8. Songun I, Putter H, Kranenbarg EM, Sasako M, van de Velde CJ.
Surgical treatment of gastric cancer: 15-year follow-up results
of the randomised nationwide Dutch D1D2 trial. Lancet Oncol.
2010;11(5):439-449.
9. Maruyama K, Sasako M, Kinoshita T, Sano T, Katai H, Okajima
K. Pancreas-preserving total gastrectomy for proximal gastric
cancer. World J Surg. 1995;19(4):532-536.
10. Degiuli M, Sasako M, Ponti A, Calvo F. Survival results of a multicentre phase II study to evaluate D2 gastrectomy for gastric
cancer. Br J Cancer 2004;90(9):1727-1732.
11. Degiuli M, Sasako M, Ponti A, Soldati T, Danese F, Calvo F.
Morbidity and mortality after D2 gastrectomy for gastric cancer:
Results of the Italian Gastric Cancer Study Group prospective
multicenter surgical study. J Clin Oncol. 1998;16(4):1490-1493.
12. Kampschoer GH, Maruyama K, van de Velde CJ, Sasako M,
Kinoshita T, Okabayashi K. Computer analysis in making preoperative decisions: a rational approach to lymph node dissection in gastric cancer patients. Br J Surg. 1989;76(9):905-908.
13. Siewert JR, Kelsen D, Maruyama K, Feussner H, Omote K, Etter
M, et al. Gastric Cancer Diagnosis and Treatment An Interactive Training Program. 1 ed. Berlin, Germany: Spinger Electronic
Media; 2000.
14. Hundahl SA, Macdonald JS, Benedetti J, Fitzsimmons T.
Surgical treatment variation in a prospective, randomized trial
of chemoradiotherapy in gastric cancer: The effect of undertreatment. Ann Surg Oncol. 2002;9(3):278-286.
15. Peeters KCMJ, Hundahl SA, Kranenbarg EK, Hartgrink H,
van de Velde CJH. Low-Maruyama-Index surgery for gastric
cancer A blinded re-analysis of the Dutch D1-D2 Trial. World
J Surg. 2005;29:1576-1584.
16. Hundahl SA, Peeters KC, Kranenbarg EK, Hartgrink H, van
de Velde CJ. Improved regional control and survival with low
Maruyama Index surgery in gastric cancer: Autopsy findings
from the Dutch D1-D2 Trial. Gastric Cancer. 2007;10(2):84-86.
17. Hartgrink HH, Putter H, Klein Kranenbarg E, Bonenkamp JJ,
van de Velde CJ. Value of palliative resection in gastric cancer.
Br J Surg. 2002;89(11):1438-1443.

5/21/2012 8:54:04 PM

CHAPTER 17

Management of Upper GI Bleeding


Bruce A. Crookes

INTRODUCTION

Patients who are critically ill have a number of causes for


ulcer formation including decreased mucous secretion, altered
GI motility, and mucosal ischemia.8 These factors are especially
prevalent in patients with large burns, head injury, coagulopathy,
or in patients who require mechanical ventilation. Traditionally,
antacids, sucralfate, or histamine-2 receptor antagonists (H2RA)
have been used,8 which were all able to reduce bleeding episodes, but none was clearly superior. More recently, proton pump
inhibitors (PPIs) have been studied for stress ulcer prophylaxis.
These agents are able to keep gastric pH > 4 by suppressing acid
secretion.8
Cook et al. found that ranitidine had lower bleeding rates
than sucralfate.9 However, several other studies showed decreased
mortality and pneumonia rates with sucralfate.8,10 Conrad et al.,11
in a randomized, double-blind study, found omeprazole to be
more effective than cimetidine in preventing GI bleeding in
critically ill patients. Omeprazole was able to reduce the rate of
bleeding from 6.8% to 4.5%, but neither pneumonia nor mortality rates had improved. Most recently, Somberg et al.12 examined
the concept of tolerance to pantoprazole, and found that iv pantoprazol controlled gastric pH well, whereas the administration
of cimetidine seemed to show decreases in gastric pH the longer it was administered. There was no significant increase in the
incidence of UGI bleeding or pneumonia, however. The lack of
difference between PPIs and H2 receptor agonists with respect
to ICU mortality, UGI bleeding prophylaxis, and pneumonia is
consistent across multiple studies, a finding that was confirmed
in a meta-analysis by Lin et al.13 Note that the preventative effects
of stress ulcer prophylaxis with omeprazole has not been shown
to hold in non-ICU patients, probably because the incidence of
UGI bleeding is so low.14
Similarly, pharmacotherapy is beneficial for preventing bleeding related to NSAIDs used for pain relief or cardiovascular disease. Some have suggested changing from aspirin to clopidogrel
for cardio- or cerebrovascular disease. Chan et al.15 found in a
randomized placebo-controlled study that patients with a history
of bleeding ulcers have less frequent bleeding when esomeprazole
was added to aspirin as opposed to a change to clopidogrel, by a

Upper gastrointestinal (UGI) bleeding is a common cause for


admission to the intensive care unit (ICU) and accounts for over
300,000 admissions in the United States.1,2 Optimal outcomes
depend on rapid identification of the etiology of the hemorrhage,
and the implementation of appropriate pharmacologic and procedural therapies.
Ulcer disease accounts for the majority of these cases. Other
etiologies include varices, Mallory-Weiss syndrome, vascular
lesions, and inflammatory states of the UGI tract. Despite advances
in pharmacology and endoscopic therapies over the last several
decades, all causes of mortality have remained constant, ranging
from 4.5% to 10%,1-4 and up to 50% for variceal bleeding.5 Medical comorbidities and the use of anticoagulants complicate treatment.4 Fortunately, over 80% of UGI bleeds stop spontaneously,
and only 2% require surgical intervention.6 Prompt and decisive
management is required, particularly in patients in whom arterial
bleeding cannot be controlled by endoscopy,7 when bleeding continues despite endoscopic therapy, or for high-risk patients. Best
practices should be used to prevent further episodes.
Initial guidelines for the management of UGI bleeding
were published almost 20 years ago. Over this period significant
advancements in treatment have developed. The purpose of this
chapter is to review the current evidence, including recent practice
guidelines, for the prevention and management of UGI bleeding.
1. What is the role of medical therapy in the prevention of UGI
bleeds and how successful is it?
The use of medical prophylaxis is highly dependent on the potential etiology of the bleeding. In some cases this is primary prevention, whereas in others secondary prevention is the goal. Because
ulcer disease is the primary cause of many UGI bleeds, providers must identify the reason for ulceration. There are three principal causes of ulcer disease: (1) stress-related mucosal damage
(SRMD), (2) nonsteroidal anti-inflammatory drug (NSAID) use,
and (3) Helicobacter pylori infection.
154

PMPH_CH17.indd 154

5/21/2012 8:55:00 PM

Management of Upper GI Bleeding

rate of 0.7% as compared with 8.6%. In a large, prospective, randomized controlled trial of 3873 patients taking clopidogrel with
omeprazole or placebo, the event rate of UGI bleeding was significantly less with omeprazole, with no difference in the rate of
cardiovascular events.16 Alternative drugs, such as COX-2 inhibitors, also can be used when NSAIDs are used for pain control in
arthritis. Another study by Chan et al.17 found that in patients who
were H. pylori negative and taking non-aspirin NSAIDs there was
additional reduction in UGI bleeding from 8.8% to 0% with the
addition of esomeprazole after they were changed to celecoxib. In
another study by Chan et al. that focused on patients on who were
H. pylori negative and taking a non-aspirin NSAID (celecoxib) the
incidence of UGI bleeding was reduced from 8.8% to 0% when
esomeprazole was administered. Lai et al.18 studied patients who
were taking aspirin and who were H. pylori positive. After eradication therapy, patients were randomized to lansoprazole or placebo
while continuing aspirin. The PPI group had an ulcer complication
rate of 1.6% compared with 14.8% with placebo. Udd et al. 19 found
that regular- and high-dose omeprazole are equally effective for
preventing peptic ulcer bleeding.
Whereas acid suppression is the hallmark of prevention for
ulcer-related bleeding, reduction of portal venous pressure is most
effective for preventing esophageal bleeding. -Blockers are the
main class of drugs that are used to accomplish this goal. They
were first used in this role in the 1980s after introduction by Lebrec20 and others. Lebrec et al. found that patients with large varices
were significantly less likely to bleed when nadolol was administered instead of a placebo.21 Kiire similarly found that propranolol
significantly reduced bleeding from varices as compared to placebo.22 Other drugs, such as isosorbide mononitrate (IM), have
also been investigated to prevent variceal bleeding. Angelico et
al.23 found that propranolol and IM provided similar protection
against variceal bleeding. However, long-term use of nitrates has
been linked to increased mortality. A recent review by Talwalkar
and Kamath showed that -blockers provide a 9% absolute risk
reduction for primary prophylaxis and a 21% reduction for secondary prevention. They also note that no individual trial has linked
-blocker prophylaxis to improved survival, but this has been
demonstrated in meta-analysis.24 Some authors have investigated
a combination of -blockers and nitrates. Merkel et al.25 demonstrated a decreased bleeding risk from 29% to 12% with a combination treatment. However, some studies have shown an increased
rate of adverse events with these combination therapies.24 Other
authors have investigated the use of -blockers to prevent the formation of growth of varices. Merkel and colleagues26 found that
the risk of variceal growth decreased from 21% to 7% and 51% to
20%, at the 1- and 5-year follow-up, respectively. However, Groszmann et al. 27 studied patients with cirrhosis and portal hypertension and were unable to show that -blockers prevented variceal
formation. In addition, recent studies have compared the use of
-blockers and endoscopic ligation for primary prophylaxis.

RECOMMENDATIONS
1. PPIs or H2RAs should be used as stress ulcer prophylaxis in
critically ill patients to prevent GI bleeding.
2. Risk of ulcer formation for patients taking NSAIDs is
significantly reduced when a PPI or H2RA prophylaxis is
utilized.

PMPH_CH17.indd 155

155

3. In patients taking clopidogrel in the setting of coronary


artery disease, UGI bleeding risk is reduced in patients taking
omeprazole.
4. -lockers can be used safely for primary prophylaxis from
variceal bleeding and may slow the growth rate of small varices.
Strength of Recommendations: 1. A, 2. A, 3. A, 4. B.
2. What is the role of medical therapy in treating UGI bleeds
and how effective is it?
As mentioned in the introduction, most UGI bleeds stop spontaneously. However, clinicians can optimize patient outcomes
through both pharmacologic and procedural interventions. UGI
bleeds caused by ulcer disease are frequently treated with acid
suppression. Initially H2RAs were used, and a review by Collins
and Langman28 in the mid-1980s found that these drugs decreased
rates of surgery and death in certain populations. Over the next
decade, however, PPIs were introduced, opening new avenues of
therapy. Lanas et al.29 found that omeprazole was superior to ranitidine in decreasing rebleeding episodes, a finding that has since
be reaffirmed.30 No differences were found, however, in mortality
or units of blood transfused. Khuroo and colleagues31 found that
PPIs reduced ongoing bleeding from 36.4% to 10.9% and reduced
the need for surgery as compared to placebo. In nonvariceal
bleeding, pre-endoscopy proton pump therapy has been shown to
downstage UGI bleeding lesions, and iv PPI administration after
successful endoscopy may decrease both mortality and rebleeding events, although the data is conflicting.32,33 For example,
Lau et al.34 found that PPI treatment was superior in preventing
rebleeding after endoscopic treatment of ulcer bleeding. Daneshmend et al.,35 however, did not find omeprazole to reduce mortality, rebleeding, or transfusion requirements, but that it was only
able to demonstrate a decrease in the endoscopic signs of bleeding
with PPI treatment. Regional differences in patient populations
may account for these differences as one study was conducted
in Europe and the other in Asia. Lau et al.36 similarly found a
decrease in the signs of recent bleeding with PPI treatment, and
also demonstrated a decreased need for endoscopic therapy.
In addition to acid suppression, treating ulcer etiology is
imperative. This includes managing critical illness, limiting NSAID
use, and treating H. pylori when appropriate. In H. pylori positive
patients, Riemann et al.37 demonstrated that curative triple therapy
with PPI was superior to maintenance therapy with H2RAs.
Sung et al.38 showed that medical therapy should not stand
alone: their study found that patients treated with both endoscopy
and PPI were much less likely to rebleed than patients treated with
PPI alone (1.1% compared with 11.6%). Intravenous administration
of prokinetic agents (i.e., erythromycin or metoclopramide) prior
to esophagogastroduodenoscopy (EGD) in the setting of acute
UGI bleeding also decreases the need for repeat endoscopy.39
Medical treatment of variceal bleeding differs from ulcer
bleeding in that therapeutic agents are different than those used for
prophylaxis. The mainstays of active variceal bleeding pharmacologic treatment are vasoconstrictive and vasoactive drugs. Vasopressin and terlipressin are vasoconstrictive agents that have been
shown to decrease active variceal bleeding. However, these drugs
can have significant side effects including headache, pulmonary
edema, and coronary vasoconstriction.40 Octreotide is the main
vasoactive drug used to treat variceal bleeding. It is a hormone analogue of somatostatin that alters GI hormone signaling, decreases

5/21/2012 8:55:01 PM

156

Surgery: Evidence-Based Practice

gastric and pancreatic secretions, and alters splanchnic blood flow.


Multiple studies have demonstrated the superior efficacy of octreotide over vasopressin40-42 for stopping active bleeding and preventing rebleeds. It appears, however, that terlipressin is not inferior
to octreotide in the control of variceal hemorrhage.43 Despite this,
no mortality benefit is gained. The recent meta-analysis by Gross
et al.44 found that vasoconstrictive therapy was only 68.7% successful as compared to vasoactive therapy, which was 75.9% successful.
Banding ligation, however, is the most effective therapy and should
be the primary intervention for stopping variceal bleeding.44
The role of transfusion does deserve special mention.
Although restrictive transfusion practices have been shown to
decrease mortality in ICU patients,45 transfusion in the setting of
UGI bleeding has been less well studied. A recent Cochrane review
of red cell transfusion in the setting of UGI bleeding showed that
more deaths and rebleeding occurred in patients receiving transfusions, although the paucity of patients in the included trials did
not allow for the development of any useful conclusions.46

RECOMMENDATIONS
1. PPIs should be preferentially used over H2RAs to reduce
rebleeding episodes after successful endoscopic therapy.
2. Octreotide should be used to slow the rate of variceal bleeding,
until definitive endoscopic therapy can be implemented.
3. Pre-endoscopy PPI therapy should be utilized to downstage the
grade of the lesion in nonvariceal bleeds
4. Intravenous PPI therapy decreases mortality and rebleeding in
nonvariceal UGI bleeds after successful endoscopic therapy.
Strength of Recommendations: 1. A, 2. B, 3. A, 4. A.
3. What is the role of endoscopy in treating, or for prophylaxis
of, UGI bleeds and how successful is it?
Endoscopy is beneficial in UGI bleeds because it can be simultaneously diagnostic and therapeutic, particularly in patients with
no prior history of bleeding. Ulcer bleeding can be stopped or
reduced with medical treatment as previously discussed. Multiple
studies, however, have shown that endoscopy confers further prevention of rebleeding.38,47 Endoscopic findings of active bleeding or
a visible vessel require treatment due to their high rates of rebleeding. Ulcers with adherent clots are more controversial: Bini and
Cohen47 directly compared endoscopy with medical treatment in
patients with adherent clots. They found that recurrent bleeding
episodes mean hospital stay, and transfusion requirements were
significantly reduced with endoscopy.
Several methods are available to achieve endoscopic hemostasis, including adrenaline injection, laser, and heater probes. No
significant differences have been found among individual injection or thermal coagulation therapies.2 Chung et al.48 found that
initial hemostasis was achieved equally by injection and heater
probe. Injection monotherapy, however, has been associated
with higher rates of repeat endoscopy.49 For ulcers with spurting
vessels, combination treatment with injection and heater probe
reduced the rate of surgery from 29.6% to 6.5%. There continue
to be patients who fail endoscopic treatments. Lau et al.50 studied
patients who had undergone successful initial endoscopic treatment and randomized them to surgery or repeat endoscopy if
they rebled. Over one-quarter of patients who were randomized
to have repeat endoscopy still required salvage surgery.50

PMPH_CH17.indd 156

While endoscopy is used solely for the treatment of ulcer disease, this intervention can be used for both treatment of active
bleeding and prophylaxis for patients with varices. Options for
endoscopic management of varices include injection sclerotherapy and banding ligation. Both techniques have been used for
the control of acute hemorrhage, but multiple studies have found
that ligation is superior to sclerotherapy.5,51,52 Banding has a lower
rebleeding rate and reduced complications. Stiegmann et al.53 also
showed a higher mortality rate in patients who used sclerotherapy
for the control of hemorrhage. In addition, the recent meta-analysis
by Gross et al.44 demonstrated the superiority of endoscopic banding ligation over medical therapy in the treatment of acute variceal
bleeding. The combination of banding and sclerotherapy has been
evaluated as well. Neither Laine54 nor Saeed55 was able to demonstrate additional benefit to combination therapy, with Saeeds
study showing an increased complication rate with dual treatment.
Although endoscopic banding is superior for the treatment
of acute variceal bleeding, the role of endoscopy and the optimal
type of treatment for prophylaxis of variceal bleeding remains
controversial. Van Buuren et al.56 found that there was no difference in the number of episodes of bleeding when sclerotherapy was
compared with cases where no treatment was given. Villanueva
et al.,57 however, found that combination medical therapy was
more successful in preventing variceal bleeding. In addition, other
trials have shown increased mortality rates with sclerotherapy,
and this practice is not recommended.5 Endoscopic banding has
been widely studied for prophylaxis of variceal bleeding. Th is
technique is often compared with medical prophylaxis with
-blockers alone or in combination with IM. A recent study by
Wang et al.58 found that combined medical (-blocker plus IM)
and procedural therapies were equally effective for primary prophylaxis. Conversely, Sarin et al.59 showed that banding reduced
the initial bleeding risk from 43% to 15%, as compared to
-blockers alone. Villanueva et al.60 showed that combined medical therapy was superior for secondary prophylaxis without an
all cause mortality benefit. Lo et al.61 recently found that banding was better for secondary prevention, but that combined medical therapy improved overall survival. A meta-analysis by Gluud
et al.62 showed that banding ligation reduced bleeding episodes
as compared to -blockers without any difference in morality.

RECOMMENDATIONS
1. Endoscopic treatment should be used to stop active hemorrhage
from ulcer disease, as it confers additional prevention of
rebleeding episodes. Injection monotherapy should be
avoided.
2. Endoscopic banding ligation is the treatment of choice for
acute variceal hemorrhage and should be undertaken as soon
as possible.
3. Banding ligation is an effective means of preventing variceal
bleeding and can be used when medical prophylaxis cannot be
tolerated.
Strength of Evidence: 1. A, 2. A, 3. B.
4. What is the role of interventional radiology in treating UGI
bleeds?
Angiography has been established as the primary therapy for
many lower GI bleeds. Its role in UGI bleeding, however, is not

5/21/2012 8:55:01 PM

Management of Upper GI Bleeding

as well defined. Angiography has been used since the 1970s for
control of GI hemorrhage for both diagnosis and therapy.63
Defreyne et al.64 published a study about a series of patients with
GI bleeding treated with angio-embolization, which showed that
patients with an upper GI source had higher rates of rebleeding
and lower success rate when compared to patients with lower GI
sources. Carreira,65 however, showed that embolization was successful 90% of the time in a study with predominately UGI bleeds.
Other studies have found similar success rates.66,67 Poultsides
et al.68 recently published an article on a series of patients with
gastroduodenal hemorrhage that underwent embolization with a
94% technical and 51% clinical success rate. Most of these studies
indicate that embolization should be used in patients with massive
ongoing hemorrhage who cannot tolerate surgery due to medical
comorbidities.69

157

Eriksson et al. recently published a retrospective study of


patients undergoing transcatheter arterial embolization (TAE) or
surgery for patients with recurrent UGI bleeding after endoscopy.
The authors found that despite a higher-risk profi le in the patients
undergoing TAE, the 30-day mortality was lower in the TAE
group (3% vs. 14%), although the results did not reach statistical
significance.70

RECOMMENDATION
Angiography should be used in patients with massive hemorrhage
who are too ill to undergo an operation.
Strength of Recommendation: C.

Clinical Question Summary


Question

Answer

1 What is the role of medical therapy


in prevention of UGI bleeds?

(1) PPI or H2RA should be used as stress ulcer prophylaxis


in critically ill patients.
(2) Risk of ulcer formation is significantly reduced with
maintenance PPI or H2RA for patients who take
NSAIDs regularly.
(3) In patients taking clopidogrel in the setting of coronary
artery disease, UGI bleeding risk is reduced in patients
taking omeprazole.
(4) -Blockers can be used safely for primary prophylaxis
of variceal bleeding.

11, 13

15, 17, 18

16

21, 22, 24, 26

(1) PPI should be preferentially used over H2RAs to reduce


bleeding episodes after successful endoscopy.
(2) Octreotide should be used to slow the rate of variceal
bleeding until definitive endoscopy is performed.

29, 31-34, 36

40-42, 44

(1) Endoscopic treatment should be used to stop


active hemorrhage from ulcer disease, as it confers
additional prevention of rebleeding episodes. Injection
monotherapy should be avoided.
(2) Endoscopic banding ligation is the treatment of choice
for acute variceal bleeding and should be undertaken as
soon as possible.
(3) Banding ligation is effective for preventing variceal
bleeding and should be used when medical prophylaxis
cannot be tolerated.

38, 47, 49

5, 44, 51, 52

58-61

(1) Angiography is safe and should be used in patients with


massive UGI bleeding who are too ill to undergo an
operation.

64, 66, 67

2 What is the role of medical therapy


in treating active UGI bleeds?

3 What is the role of endoscopy for


treating or preventing UGI bleeds?

4 What is the role for interventional


radiology in treating UGI bleeds?

REFERENCES
1. Conrad SA. Acute upper gastrointestinal bleeding in critically
ill patients: causes and treatment modalities. Crit Care Med.
2002;30:S365-S368.
2. Barkun A, Bardou M, Marshall JK. Consensus recommendations
for managing patients with nonvariceal upper gastrointestinal
bleeding. Ann Intern Med. 2003;139:843-857.
3. Eisen GM, Dominitz JA, Faigel DO, et al. An annotated algorithmic approach to upper gastrointestinal bleeding. Gastrointest
Endosc. 2001;53:853-858.

PMPH_CH17.indd 157

Grade

References

4. Marmo R, Koch M, Cipolletta L, et al. Predictive factors of mortality from nonvariceal upper gastrointestinal hemorrhage: a multicenter study. Am J Gastroenterol. 2008;103:1639-1647; quiz 48.
5. Qureshi W, Adler DG, Davila R, et al. ASGE Guideline: the role of
endoscopy in the management of variceal hemorrhage, updated
July 2005. Gastrointest Endosc. 2005;62:651-655.
6. Clarke MG, Bunting D, Smart NJ, Lowes J, Mitchell SJ. The surgical management of acute upper gastrointestinal bleeding: a 12-year
experience. Int J Surg. 2010;8:377-380.
7. Cheung FK, Lau JY. Management of massive peptic ulcer bleeding.
Gastroenterol Clin North Am. 2009;38:231-243.

5/21/2012 8:55:01 PM

158

Surgery: Evidence-Based Practice

8. Jung R, MacLaren R. Proton-pump inhibitors for stress ulcer


prophylaxis in critically ill patients. Ann Pharmacother.
2002;36:1929-1937.
9. Cook D, Guyatt G, Marshall J, et al. A comparison of sucralfate and ranitidine for the prevention of upper gastrointestinal
bleeding in patients requiring mechanical ventilation. Canadian
Critical Care Trials Group. N Engl J Med. 1998;338:791-797.
10. Cook DJ, Reeve BK, Guyatt GH, et al. Stress ulcer prophylaxis
in critically ill patients. Resolving discordant meta-analyses.
JAMA. 1996;275:308-314.
11. Conrad SA, Gabrielli A, Margolis B, et al. Randomized, doubleblind comparison of immediate-release omeprazole oral suspension versus intravenous cimetidine for the prevention of upper
gastrointestinal bleeding in critically ill patients. Crit Care Med.
2005;33:760-765.
12. Somberg L, Morris J, Jr., Fantus R, et al. Intermittent intravenous pantoprazole and continuous cimetidine infusion: effect on
gastric pH control in critically ill patients at risk of developing
stress-related mucosal disease. J Trauma. 2008;64:1202-1210.
13. Lin PC, Chang CH, Hsu PI, Tseng PL, Huang YB. The efficacy
and safety of proton pump inhibitors vs histamine-2 receptor
antagonists for stress ulcer bleeding prophylaxis among critical care patients: a meta-analysis. Crit Care Med. 2010;38:11971205.
14. Amaral MC, Favas C, Alves JD, Riso N, Riscado MV. Stressrelated mucosal disease: incidence of bleeding and the role
of omeprazole in its prophylaxis. Eur J Intern Med. 2010;21:
386-388.
15. Chan FK, Ching JY, Hung LC, et al. Clopidogrel versus aspirin
and esomeprazole to prevent recurrent ulcer bleeding. N Engl J
Med. 2005;352:238-244.
16. Bhatt DL, Cryer BL, Contant CF, et al. Clopidogrel with or
without omeprazole in coronary artery disease. N Engl J Med.
2010;363:1909-1917.
17. Chan FK, Wong VW, Suen BY, et al. Combination of a cyclooxygenase-2 inhibitor and a proton-pump inhibitor for prevention of recurrent ulcer bleeding in patients at very high risk: a
double-blind, randomised trial. Lancet. 2007;369:1621-1626.
18. Lai KC, Lam SK, Chu KM, et al. Lansoprazole for the prevention
of recurrences of ulcer complications from long-term low-dose
aspirin use. N Engl J Med. 2002;346:2033-2038.
19. Udd M, Miettinen P, Palmu A, et al. Regular-dose versus high-dose
omeprazole in peptic ulcer bleeding: a prospective randomized
double-blind study. Scand J Gastroenterol. 2001;36:1332-1338.
20. Lebrec D, Nouel O, Bernuau J, Bouygues M, Rueff B, Benhamou
JP. Propranolol in prevention of recurrent gastrointestinal bleeding in cirrhotic patients. Lancet. 1981;1:920-921.
21. Lebrec D, Poynard T, Capron JP, et al. Nadolol for prophylaxis of
gastrointestinal bleeding in patients with cirrhosis. A randomized trial. J Hepatol. 1988;7:118-125.
22. Kiire CF. Controlled trial of propranolol to prevent recurrent
variceal bleeding in patients with non-cirrhotic portal fibrosis.
BMJ. 1989;298:1363-1365.
23. Angelico M, Carli L, Piat C, et al. Isosorbide-5-mononitrate versus propranolol in the prevention of first bleeding in cirrhosis.
Gastroenterology. 1993;104:1460-1465.
24. Talwalkar JA, Kamath PS. An evidence-based medicine approach
to beta-blocker therapy in patients with cirrhosis. Am J Med.
2004;116:759-766.
25. Merkel C, Marin R, Sacerdoti D, et al. Long-term results of a
clinical trial of nadolol with or without isosorbide mononitrate
for primary prophylaxis of variceal bleeding in cirrhosis. Hepatology. 2000;31:324-329.

PMPH_CH17.indd 158

26. Merkel C, Marin R, Angeli P, et al. A placebo-controlled clinical


trial of nadolol in the prophylaxis of growth of small esophageal
varices in cirrhosis. Gastroenterology. 2004;127:476-484.
27. Groszmann RJ, Garcia-Tsao G, Bosch J, et al. Beta-blockers
to prevent gastroesophageal varices in patients with cirrhosis.
N Engl J Med. 2005;353:2254-2261.
28. Collins R, Langman M. Treatment with histamine H2 antagonists in acute upper gastrointestinal hemorrhage. Implications
of randomized trials. N Engl J Med. 1985;313:660-666.
29. Lanas A, Artal A, Blas JM, Arroyo MT, Lopez-Zaborras J, Sainz
R. Effect of parenteral omeprazole and ranitidine on gastric pH
and the outcome of bleeding peptic ulcer. J Clin Gastroenterol.
1995;21:103-106.
30. Kellici I, Kraja B, Mone I, Prift i S. Role of intravenous omeprazole
on non-variceal upper gastrointestinal bleeding after endoscopic
treatment: a comparative study. Med Arh. 2010;64:324-327.
31. Khuroo MS, Yattoo GN, Javid G, et al. A comparison of omeprazole and placebo for bleeding peptic ulcer. N Engl J Med. 1997;
336:1054-1058.
32. Barkun AN, Bardou M, Kuipers EJ, et al. International consensus recommendations on the management of patients with
nonvariceal upper gastrointestinal bleeding. Ann Intern Med.
2010;152:101-113.
33. Sreedharan A, Martin J, Leontiadis GI, et al. Proton pump
inhibitor treatment initiated prior to endoscopic diagnosis in
upper gastrointestinal bleeding. Cochrane Database Syst Rev.
2010:CD005415.
34. Lau JY, Sung JJ, Lee KK, et al. Effect of intravenous omeprazole
on recurrent bleeding after endoscopic treatment of bleeding
peptic ulcers. N Engl J Med. 2000;343:310-316.
35. Daneshmend TK, Hawkey CJ, Langman MJ, Logan RF, Long
RG, Walt RP. Omeprazole versus placebo for acute upper gastrointestinal bleeding: randomised double blind controlled trial.
BMJ. 1992;304:143-147.
36. Lau JY, Leung WK, Wu JC, et al. Omeprazole before endoscopy
in patients with gastrointestinal bleeding. N Engl J Med. 2007;
356:1631-1640.
37. Riemann JF, Schilling D, Schauwecker P, et al. Cure with
omeprazole plus amoxicillin versus long-term ranitidine therapy
in Helicobacter pylori-associated peptic ulcer bleeding. Gastrointest Endosc. 1997;46:299-304.
38. Sung JJ, Chan FK, Lau JY, et al. The effect of endoscopic therapy
in patients receiving omeprazole for bleeding ulcers with nonbleeding visible vessels or adherent clots: a randomized comparison. Ann Intern Med. 2003;139:237-243.
39. Barkun AN, Bardou M, Martel M, Gralnek IM, Sung JJ. Prokinetics in acute upper GI bleeding: a meta-analysis. Gastrointest
Endosc. 2010;72:1138-1145.
40. Jenkins SA, Baxter JN, Corbett W, Devitt P, Ware J, Shields R.
A prospective randomised controlled clinical trial comparing
somatostatin and vasopressin in controlling acute variceal haemorrhage. Br Med J (Clin Res Ed). 1985;290:275-278.
41. Hwang SJ, Lin HC, Chang CF, et al. A randomized controlled trial comparing octreotide and vasopressin in the control of acute esophageal variceal bleeding. J Hepatol. 1992;16:
320-325.
42. Corley DA, Cello JP, Adkisson W, Ko WF, Kerlikowske K. Octreotide for acute esophageal variceal bleeding: a meta-analysis.
Gastroenterology. 2001;120:946-954.
43. Abid S, Jafri W, Hamid S, et al. Terlipressin vs. octreotide in
bleeding esophageal varices as an adjuvant therapy with endoscopic band ligation: a randomized double-blind placebocontrolled trial. Am J Gastroenterol. 2009;104:617-623.

5/21/2012 8:55:01 PM

Management of Upper GI Bleeding

44. Gross M, Schiemann U, Muhlhofer A, Zoller WG. Meta-analysis:


efficacy of therapeutic regimens in ongoing variceal bleeding.
Endoscopy. 2001;33:737-746.
45. Hebert PC, Wells G, Blajchman MA, et al. A multicenter, randomized, controlled clinical trial of transfusion requirements in
critical care. Transfusion Requirements in Critical Care Investigators, Canadian Critical Care Trials Group. N Engl J Med.
1999;340:409-417.
46. Jairath V, Hearnshaw S, Brunskill SJ, et al. Red cell transfusion
for the management of upper gastrointestinal haemorrhage.
Cochrane Database Syst Rev. 2010:CD006613.
47. Bini EJ, Cohen J. Endoscopic treatment compared with medical therapy for the prevention of recurrent ulcer hemorrhage in
patients with adherent clots. Gastrointest Endosc. 2003;58:707-714.
48. Chung SS, Lau JY, Sung JJ, et al. Randomised comparison between
adrenaline injection alone and adrenaline injection plus heat
probe treatment for actively bleeding ulcers. BMJ. 1997;314:13071311.
49. Enestvedt BK, Gralnek IM, Mattek N, Lieberman DA, Eisen
GM. Endoscopic therapy for peptic ulcer hemorrhage: practice variations in a multi-center U.S. consortium. Dig Dis Sci.
2010;55:2568-2576.
50. Lau JY, Sung JJ, Lam YH, et al. Endoscopic retreatment compared with surgery in patients with recurrent bleeding after
initial endoscopic control of bleeding ulcers. N Engl J Med.
1999;340:751-756.
51. Laine L, el-Newihi HM, Migikovsky B, Sloane R, Garcia F. Endoscopic ligation compared with sclerotherapy for the treatment of
bleeding esophageal varices. Ann Intern Med. 1993;119:1-7.
52. Gimson AE, Ramage JK, Panos MZ, et al. Randomised trial
of variceal banding ligation versus injection sclerotherapy for
bleeding oesophageal varices. Lancet. 1993;342:391-394.
53. Stiegmann GV, Goff JS, Michaletz-Onody PA, et al. Endoscopic
sclerotherapy as compared with endoscopic ligation for bleeding
esophageal varices. N Engl J Med 1992;326:1527-1532.
54. Laine L, Stein C, Sharma V. Randomized comparison of ligation versus ligation plus sclerotherapy in patients with bleeding
esophageal varices. Gastroenterology. 1996;110:529-533.
55. Saeed ZA, Stiegmann GV, Ramirez FC, et al. Endoscopic variceal
ligation is superior to combined ligation and sclerotherapy for
esophageal varices: a multicenter prospective randomized trial.
Hepatology. 1997;25:71-74.
56. van Buuren HR, Rasch MC, Batenburg PL, et al. Endoscopic sclerotherapy compared with no specific treatment for the primary
prevention of bleeding from esophageal varices. A randomized
controlled multicentre trial [ISRCTN03215899]. BMC Gastroenterol. 2003;3:22.
57. Villanueva C, Balanzo J, Novella MT, et al. Nadolol plus
isosorbide mononitrate compared with sclerotherapy for the

PMPH_CH17.indd 159

58.

59.

60.

61.

62.

63.

64.

65.

66.

67.

68.

69.

70.

159

prevention of variceal rebleeding. N Engl J Med. 1996;334:


1624-169.
Wang HM, Lo GH, Chen WC, et al. Comparison of endoscopic
variceal ligation and nadolol plus isosorbide-5-mononitrate in
the prevention of first variceal bleeding in cirrhotic patients.
J Chin Med Assoc. 2006;69:453-460.
Sarin SK, Lamba GS, Kumar M, Misra A, Murthy NS. Comparison of endoscopic ligation and propranolol for the primary prevention of variceal bleeding. N Engl J Med. 1999;340:988-993.
Villanueva C, Minana J, Ortiz J, et al. Endoscopic ligation compared with combined treatment with nadolol and isosorbide
mononitrate to prevent recurrent variceal bleeding. N Engl J
Med. 2001;345:647-655.
Lo GH, Chen WC, Lin CK, et al. Improved survival in patients
receiving medical therapy as compared with banding ligation
for the prevention of esophageal variceal rebleeding. Hepatology.
2008;48:580-587.
Gluud LL, Klingenberg S, Nikolova D, Gluud C. Banding ligation versus beta-blockers as primary prophylaxis in esophageal
varices: systematic review of randomized trials. Am J Gastroenterol. 2007;102:2842-2848; quiz 1, 9.
Rahn NH, 3rd, Tishler JM, Han SY, Russinovich NA. Diagnostic
and interventional angiography in acute gastrointestinal hemorrhage. Radiology. 1982;143:361-6.
Defreyne L, Vanlangenhove P, De Vos M, et al. Embolization as
a first approach with endoscopically unmanageable acute nonvariceal gastrointestinal hemorrhage. Radiology. 2001;218:739-748.
Carreira JM, Reyes R, Pulido-Duque JM, et al. Diagnosis and
percutaneous treatment of gastrointestinal hemorrhage. Longterm experience. Rev Esp Enferm Dig. 1999;91:684-692.
Toyoda H, Nakano S, Takeda I, et al. Transcatheter arterial
embolization for massive bleeding from duodenal ulcers not
controlled by endoscopic hemostasis. Endoscopy. 1995;27:304307.
Park MH, Park GS, Park SW, et al. [Clinical effectiveness of
transcatheter arterial embolization for acute upper and lower
non-variceal gastrointestinal bleeding]. Korean J Gastroenterol.
2005;46:262-268.
Poultsides GA, Kim CJ, Orlando R, 3rd, Peros G, Hallisey MJ,
Vignati PV. Angiographic embolization for gastroduodenal
hemorrhage: safety, efficacy, and predictors of outcome. Arch
Surg. 2008;143:457-461.
Loff roy R, Guiu B. Role of transcatheter arterial embolization for
massive bleeding from gastroduodenal ulcers. World J Gastroenterol. 2009;15:5889-5897.
Eriksson LG, Ljungdahl M, Sundbom M, Nyman R. Transcatheter arterial embolization versus surgery in the treatment of
upper gastrointestinal bleeding after therapeutic endoscopy failure. J Vasc Interv Radiol. 2008;19:1413-1418.

5/21/2012 8:55:01 PM

Commentary on Management of
Upper GI Bleeding
Gregory J. Jurkovich

This chapter addresses four important questions in the management of upper gastrointestinal (UGI) bleeding. For long a challenge to surgeons and gastroenterologists alike, the management
of bleeding from the UGI tract begins with determining the
source. This is often a challenge in itself, since patients can present
with hypotension and no obvious source, blood from the rectum,
melena, or the most dramatic hemetemesis (vomiting of blood).
Interrogating the stomach via a nasogastric tube and looking
for blood is usually the first step, concomitant with intravenous
access and resuscitation. When blood is found, UGI endoscopy
is mandatory, as there is no alternative method of distinguishing
variceal, gastric ulcer, duodenal ulcer, gastritits/duodenitis, or the
rarer hemobilia as the source.
Once the source is determined, the following four questions
illustrate the management dilemmas facing the clinician:

calling the surgeon is seen as a sign of defeat, rather than an


acknowledgment that a stitch in a bleeding vessel might be a
wise option.
With less experience in the surgical management of acute
GI bleeding, and the undeniable frequent success noted in this
chapter with medical management and endoscopic control of
bleeding, many surgeons are inexperienced or unskilled in the
surgical approach to GI bleeding. Th is can lead to further delays
in surgical management, more blood loss and transfusions, and
a self-fulfi lling worse outcome for surgical therapy. The key
question for surgeons is when to intervene. When medications,
endoscopic coagulation, or angio-embolization fail to stop the
bleeding, what is the surgical approach? When is an operation
indicated? What is the operation of choice for the variety of
causes of GI bleeding? These questions go well beyond the content
of this chapter, as there are perhaps as many surgical technical
options for the management of GI bleeding as there are alternative therapies. Nonetheless, the single defi ning concept should be
clear: stop the bleeding. That is the fundamental key to the management of GI bleeding. Resuscitation strategies should focus on
an earlier and more balanced use of red blood cells, plasma, and
platelet transfusions and a relatively more restricted use of crystalloids. Although this approach has been considered beneficial
primarily in the context of traumatic hemorrhage requiring massive resuscitation, it seems applicable to patients with GI bleeding who also require multiple units of packed red blood cells. It
is particularly important to correct measured coagulopathy and
thrombocytopenia.2,3
Although surgical therapy is generally reserved for patients
with life-threatening hemorrhage who have failed the management options highlighted in this chapter, perhaps the greatest challenge is the timely, early identification of those patients
who require surgical therapy. Recognizing this select group of
patients prior to inflicting delays will avoid unnecessary blood
transfusion, prolonged hemodynamic instability, and the subsequent risk of complications. The historical surgical recommendations (shock, >6 units of blood transfused) reflected a concern
and respect for the severity and temporal sequence of shock, but
they are also currently supported by a concern for the deleterious immunosuppressive effects of blood transfusions. Balancing
this long-held surgical adage is the new recognition that bleeding from peptic ulcer disease can be effectively and efficiently
controlled by endoscopic techniques, including retreatment

1. What is the role of medical therapy in prevention of


UGI bleeds?
2. What is the role of medical therapy in treating active UGI
bleeds?
3. What is the role of endoscopy for treating or preventing UGI
bleeds?
4. What is the role for interventional radiology in treating
UGI bleeds?
What is missing from this chapter is the question What is
the role of surgery in the management of UGI bleeds? As this
chapter illustrates, surgery has been relegated to secondary, tertiary, or even quaternary management strategy for UGI bleeding.
Medical management, endoscopic interventions, and even angioembolization are currently all thought to have advantages over
surgical interventions in the management of UGI bleeding. Th is
chapter highlights the literature supporting this radical change
in management strategy. Surgical textbooks from the end of
the 20th century routinely advised surgical approach to GI
bleeding if greater than 6 units of blood transfusion were
required.While this chapter succinctly outlines new therapies
to control hemorrhage (drugs, endoscopy, angio-embolization),
the adage that mortality increases with increasing requirement
of blood transfusion still holds, as evidenced by the Cochrane
review.1 Th is is common sense. However, at what point in an
extended trial of repeated endoscopies or medical management
is the surgeon called? The answer to this question, in the early
part of the 21st century, is often too late. It often appears that
160

PMPH_CH17.indd 160

5/21/2012 8:55:01 PM

Management of Upper GI Bleeding

after initial control of bleeding, which has shown to decrease


the need for surgery and results in fewer complications than
surgical management, even in the presence of large volumes of
blood transfusion.4
Surgical management retains a role for the giant duodenal
or gastric ulcer, the posterior penetrating duodenal ulcer into
the gastroduodenal artery, the failed recurrent bleeding peptic
ulcer of any location and size, and, rarely, diff use stress or druginduced gastritis. Not included in this discussion is the role of
surgery for GI tract tumors and malignancy. The management of
bleeding esophageal varices has almost entirely been relegated to
endoscopic control and/or endovascular TIPS (transjugular intrahepatic portosystemic shunt), although the rare patientand
perhaps even rarer surgeonscould be identified for acute portal
decompression via surgical methods.5
In all cases, multiple factors including the source of hemorrhage, the appearance of the bleeding site at the time of endoscopy,
and patient comorbidities must be considered. The decision to
proceed with medications, endoscopy, angio-embolization,
or surgery ultimately depends on clinical judgment, but surgical

PMPH_CH17.indd 161

161

options remain important tools in the management of GI


bleeding.

REFERENCES
1. Jairath V, Hearnshaw S, Brunskill SJ, et al. Red cell transfusion for
the management of upper gastrointestinal haemorrhage. Cochrane
Database Syst Rev. 2010:CD006613.
2. Kwok A, Faigel DO. Management of anticoagulation before
and after gastrointestinal endoscopy. Am J Gastroenterol. 2009;
104(12):3085-3097; quiz 3098.
3. Anderson MA, Ben-Menachem T, Gan SI, et al. Management of
antithrombotic agents for endoscopic procedures. Gastrointest
Endosc. 2009;70(6):1060-1070.
4. Lau JY, Sung JJ, Lam YH, et al. Endoscopic retreatment compared
with surgery in patients with recurrent bleeding after initial endoscopic control of bleeding ulcers. N Engl J Med. 1999;340:751-756.
5. Henderson JM. Surgery versus transjugular intrahepatic portal
systemic shunt in the treatment of severe variceal bleeding. Clin
Liver Dis. 2006;10(3):599-612.

5/21/2012 8:55:01 PM

PMPH_CH17.indd 162

5/21/2012 8:55:01 PM

PART 111

SMALL BOWEL

PMPH_CH18.indd 163

5/21/2012 8:55:43 PM

PMPH_CH18.indd 164

5/21/2012 8:55:43 PM

CHAPTER 18
CHAPTER
Small1Bowel

Surgery

James H. Lee, John J. Hong, Dale Dangleben, and


Michael M. Badellino

POSTOPERATIVE ILEUS

can be altered by surgery and anesthetic agents, including opioid


medications, to effect bowel motility.6
The diagnosis of POI cannot be defi nitively excluded or confi rmed by any diagnostic test. Abdominal radiographs may demonstrate nonspecific dilated loops of small and large bowels. The
utilization of upper GI series and abdominal computerized axial
tomography (CAT) scans may be required to differentiate an
ileus from a bowel obstruction. The recognition of POI is based
on the usual signs and symptoms in the proper postoperative
setting.
Typically the resolution of POI is marked with the passage
of flatus or defecation. The return of bowel sounds, decreasing
amounts of bile in the nasogastric tube (NGT) drainage, decrease
in NGT output, and the tolerance of oral intake are other measures that can demonstrate the resolution of POI.

The historical definition of ileus is the functional inhibition of


propulsive bowel activity irrespective of cause. Postoperative
ileus (POI) is an uncomplicated ileus which occurs following surgery, and generally resolves spontaneously in approximately 2 to
3 days. A paralytic POI is any POI lasting longer than 3 days.1
POI increases hospital length of stay (LOS) by hindering patient
mobility, delaying enteral feeding, and contributing to postoperative patient discomfort.2
The extent and duration of POI in patients undergoing elective
abdominal surgery was examined by Artinyan et al.3 The median
duration of POI was 5 days, with the duration of POI 10 days or
less in 96.6% of the 88 patients in the study. Variables such as age,
body mass index, anesthesia time, surgery time, estimated blood
loss (EBL), and total opioid dose were analyzed to determine the
presence of a correlation between duration of POI and any of the
aforementioned factors. The only statistically significant factors
that were independently associated with duration of POI were EBL
and total opioid dose. The initiation of unrestricted clear liquids
took a mean number of 1.6 days, with 22.7% of patients tolerating
a solid diet by the 6th postoperative day.3
POI features the impairment of gastrointestinal motility after
abdominal, or other surgical procedures, leading to the accumulation of gas and fluids in the bowel, with the resultant delay in
defecation and passage of flatus.4 Patients can present in variable
ways; some patients are essentially asymptomatic, whereas others
present with absent bowel sounds, diet intolerance, nausea and
vomiting, and abdominal pain and distention. All portions of
the gastrointestinal tract are affected by POI, with the recovery
of each occurring at different rates. Typically, return of function
of the small intestine is seen within 4 to 24 h after surgery, followed by the stomach in 24 to 48 h, and ends with the return of
large intestine function within 48 to 72 h.5
Several physiologic mechanisms control gastrointestinal
motility. These mechanisms include the autonomic nervous system,
inflammation, GI hormones, acid/base status, electrolyte imbalances, and various other metabolic disturbances. These factors

1. Are there other agents and/or techniques that can be used to


improve the duration of POI?
A variety of management approaches can be utilized to reduce the
prevalence of POI. Minimally invasive surgical techniques, such
as laparoscopy, have been shown to have advantages over traditional open procedures. These include improved cosmesis, faster
recovery, fewer surgery-related complications such as adhesions
or hernias, diminished pain, shorter LOS, and faster resolution
of POI. A reduced duration of POI and an associated reduction in
LOS has been documented in a number of controlled studies. Lacyrandomized patients with colon cancer to either laparoscopicassisted colectomy or open colectomy. Patients randomized to
the laparoscopic group had a quicker time to oral intake, lower
requirement for NGT insertion, and a quicker mean time to recovery of POI (36 h) compared with the open group (55 h). The laparoscopic group also had a shorter mean LOS (5.2 days) than the
open group (7.9 days).7 Salimath et al. performed a retrospective
review of all consecutive patients who underwent either laparoscopic-assisted colectomy or open colectomy. Out of 247 patients,
the laparoscopic group had an earlier first passage of flatus
(2.9 days), an earlier first bowel movement (3.7 days), and a shorter
165

PMPH_CH18.indd 165

5/21/2012 8:55:43 PM

166

Surgery: Evidence-Based Practice

LOS (4.4 days) compared with the open group (first passage of
flatus 3.6 days; first bowel movement 4.4 days; LOS 8 days).8
The profound impact that opioid analgesia has on GI motility
and length of POI is well established.9 The gastrointestinal effects
of opioids are observed with systemic opioid administration with
intravenous patient controlled analgesia, intramuscular opioid
injection, or epidural opioid administration.10 An improved affect
of epidural local anesthetics on the duration of POI was demonstrated by a literature review encompassing multiple studies.11
Epidural local anesthetics reduced the duration of POI by 36 h
when compared with systemic opioids and by 24 h when compared with epidural opioids in a recent review.12 No statistical
difference exists when comparing epidural local anesthetic with
epidural combination local and opioid.
Numerous methods have been implemented to decrease opioid
usage in providing analgesia, based on the information that opioids inhibit GI motility and prolong POI. The use of nonsteroidal
anti-inflammatory (NSAID) medications is the most-established
technique of opioid-sparing analgesia. The administration of
NSAIDs results in a shortening of POI, improved GI motility, and
less postoperative nausea and vomiting in experimental and clinical studies.13,14
Despite popular belief, use of the NGT may potentially exacerbate POI.15 Complications such as pharyngitis, maxillary sinusitis,
and rhinitis may result from prolonged NGT usage. The current
recommendation is routine NGT removal after surgery to help
avoid the complications associated with prolonged NGT usage.16
Postoperative NGT use was shown to be a major risk factor for
pulmonary complications, including atelectasis, pneumonia, or
respiratory failure requiring mechanical ventilation in a study
of 1055 patients undergoing nonthoracic surgery by McAlister
et al.17 An odds ratio of 7.7 in causing pulmonary complications is
seen with the perioperative use of an NGT. Nelson et al. showed
that the use of an NGT delayed the return of bowel function in
abdominal operations of any type in a recent meta-analysis.18 This
review encompassed 28 studies and fulfi lled the eligibility criteria
of patients having abdominal operations of any type, emergency or
elective. The patients were randomized before completion of the
operation for selective NGT use with early removal or to receive
an NGT and have it remain in place until intestinal function had
returned. The data showed an earlier return of bowel function in
postsurgical patients not having an NGT routinely inserted.
Early resumption of enteral feedings have now been shown
to be safe and beneficial for the patient, allowing earlier tolerance
of a solid diet and return of bowel function.19 Han-Guerts et al.
conducted a randomized prospective study using 128 patients
undergoing open abdominal colorectal or vascular procedures.
Patients were assigned to either a conventional return to regular
diet group or a group that resumed a diet as soon as tolerated. A
shortened LOS was observed in the as tolerated group because
a normal diet was tolerated after a median of 2 days compared
with 5 days in the conventional group. An earlier passage of flatus, a bowel movement of approximately 1 day sooner, the ability
to tolerate a regular diet 3 days earlier, and a hospital discharge 2
days before controls was seen in patients started on clear liquids
4 h after elective colorectal surgery in a prospective randomized
study by Stewart et al.20
Recommendation: Earlier resolution of POI and decreased
hospital LOS is associated with minimally invasive surgical techniques, the use of local epidural anesthetic anesthesia, avoidance

PMPH_CH18.indd 166

of NGT usage and early enteral feedings. When these strategies


are utilized in patients after surgery, they lead to a faster recovery
time from POI and as a result a clinically significant shortened
hospital LOS. (Grade A recommendation)
2. Does chewing gum shorten the duration of POI?
Asao et al., based on the evidence that early enteral feedings
lessened the extent of POI, examined an alternative approach
of using gum chewing to stimulate bowel function in the postoperative period. It is postulated that gum increases vagal tone
and increases the release of GI hormones associated with bowel
motility. In this small series of 19 patients who underwent laparoscopic colon resection for cancer, an earlier return of bowel
function was seen in patients prospectively randomized to the
gum-chewing group compared with controls. The gum-chewing
group experienced passage of flatus about 24 h sooner and first
defecation approximately 2.7 days earlier than controls.21 A metaanalysis and systematic review of the literature conducted by
Chan showed that when combined with usual postoperative care,
surgical patients randomized to gum chewing passed flatus 24.3%
earlier, had bowel movements 32.7% sooner, and were discharged
from the hospital 17.6% faster than those with standard postoperative care alone.22
Recommendation: Gum chewing shortens the duration of
POI and may be utilized in postoperative patients, according to
prospective studies in the colorectal literature. Although statistically significant, the reduction in time to return of bowel function is not necessarily clinically meaningful. Conversely, the use
of chewing gum does not appear to have any associated morbidity.
(Grade C recommendation)
3. Does the use of selective opiate receptor inhibitors decrease
duration of POI?
A novel approach for the management of POI recently has been
the development of a selective opioid receptor antagonist. The
, , and opioid receptor subtypes are involved in the regulation of GI tract function.23 The most important subtype involved
in GI motility, transit time, and central pain management is the
receptor.24 Alvimopan (Entereg) is a synthetic, peripherally acting opioid antagonist with limited GI absorption that does not
cross the bloodbrain barrier.25 The use of alvimopan to accelerate GI recovery in the management of POI has been supported by
clinical trials. Alvimopan 6 mg shortened the length of POI as
measured by the median time to first flatus which decreased from
70 to 49 h, median time to fi rst bowel movement which decreased
from 111 to 70 h, and median time until readiness to hospital discharge which decreased from 91 to 68 h compared with placebo in
the first published study of 78 patients undergoing partial colectomy or total abdominal hysterectomy.28
Wolff et al. prospectively randomized 500 patients undergoing bowel resection or radical hysterectomy to receive either alvimopan or placebo. They found an improved time to GI recovery by
15 to 20 h following alvimopan 6 mg and by 22 to 28 h following
alvimopan 12 mg. The mean time to hospital discharge was 13 h
sooner for the alvimopan 6 mg group and 20 h sooner for the alvimopan 12 mg group.27 Delaney also observed an earlier return
of bowel function in patients given postoperative alvimopan in
a study of approximately 400 patients undergoing bowel resection, simple hysterectomy, or radical hysterectomy. The mean

5/21/2012 8:55:43 PM

Small Bowel Surgery

return of GI function for all patients was improved by 14.1 h


after doses of Alvimopan 6 mg and by 7.5 hours after Alvimopan 12 mg. Patients given Alvimopan 6 mg were discharged from
the hospital a mean of 14 h earlier when compared with patients
given placebo.28 Viscusi found GI recovery, as measured by tolerance of solid diet, passage of flatus, and defecation, accelerated by
7.5 h for patients given 6 mg alvimopan versus 9.9 h for patients
given alvimopan 12 mg dose in approximately 600 patients who
underwent open laparotomy for bowel resection, simple hysterectomy, or radical hysterectomy. The mean time to discharge order
was reduced by 14.2 h for the 6 mg dose and by 15.2 h for the
12 mg dose.29 No differences between placebo and alvimopan
were observed in regards to postoperative opioid consumption
and POI related morbidity as shown by a higher postoperative
NGT insertion, increased LOS, or readmission rate in these trials. Using pooled data from alvimopan trials, Wolff et al. showed
that overall POI morbidity is lower in groups treated with alvimopan (6.611.2%) versus placebo (14.119.7%).30 Two percent of
patients in the alvimopan groups complications of POI resulting
in increased LOS compared with approximately 7% of patients in
the placebo groups. Furthermore, patients treated with alvimopan
are less likely to undergo readmission (4.9% alvimopan, 8.3% placebo) for POI complications at 10 days, whereas those readmitted
at 7 days were comparable between groups.30
Recommendation: Although sponsored by the manufacturer,
prospective randomized studies demonstrate improvement in the
duration of POI and hospital LOS with the use of alvimopan, a
selective opiate receptor inhibitor. These differences, while statistically significant, do not consistently translate into clinical relevance. Data on cost effectiveness of alvimopan are needed prior
to adopting its use. (Grade C recommendation)

INTRAABDOMINAL ADHESIONS
The most frequent complication of abdominal surgery is intraabdominal adhesions which develop in over 93% of patients undergoing laparotomy. 31 In patients operated on once for adhesive small
bowel obstruction (ASBO), the cumulative recurrence rate and
need for hospital admission for ASBO is 18% and 29% at 10 years
and 30 years, respectively.32
The peritoneal cavity is a closed sac in males or an open sac
through the gynecological tract in females that lies in the space
between the visceral and parietal peritoneum. The peritoneum
consists of a connective tissue layer covered by a mesothelium.
The peritoneal cavity contains approximately 10 mL of serous fluid
under normal conditions. This fluid circulates within the abdominal cavity through well-defined routes and joins with the vascular
system via the lymphatics.
Peritoneal trauma and inflammation leads to in a decrease
of blood flow and local angiogenesis which then results in the
formation of intraabdominal adhesions.33 Vascular permeability
increases and inflammatory cells are released. A fibrin gel then
forms from actived fibrinogen at the site of peritoneal injury
which acts to connect the two damaged layers of peritoneum. A
fibrinolytic process is then initiated by plasmin, an active protease
formed by the action of plasminogen activators on its precursor,
plasminogen, that attempts to control fibrin formation by hydrolyzing fibrin to fibrin split products. The theory of adhesion formation is a pathologic alteration in fibrinolysis resulting from a

PMPH_CH18.indd 167

167

decrease in the plasminogen activator activity, largely mediated


through tissue plasminogen activator.34 The regeneration process leads to the organization of non degraded fibrin matrix and
causes the apposition of two damaged peritoneal surfaces resulting in adhesion development.35 Inflammatory cells such as macrophages, mast cells, fibroblasts, eosinophils, red blood cells, and
tissue debris are contained in adhesions. Adhesions mature into
fibrous collagen bands covered by mesothelium with decreasing
amounts of cells.
Small bowel obstruction (SBO) is the most frequently encountered clinical manifestation of abdominal adhesions, with 74%
of cases being related to previous surgery.36 A mortality rate of
almost 10% has been identified in reviews of hospital admissions
for adhesional SBO,37 which increases to approximately 15% in
patients undergoing small bowel resection.38 There is a 33% risk of
inadvertent enterotomy when surgery is required for adhesional
SBO and a 19% risk of inadvertent enterotomy with the presence of adhesions during a reoperative laparotomy.39 Adhesions
from previous surgery significantly increased operative time by a
median time of 18 min, in a study that examined effects of previous surgery on operative times.40 The mortality rate ranges from
20% to 50% in patients who have an undetected bowel injury after
undergoing operation for adhesional SBO.41 Although adhesions
reform in approximately 85% of patients, adhesiolysis remains the
treatment for adhesions.42
4. Are there any techniques/agents that have been shown to
decrease intraabdominal adhesion formation following laparotomy?
A variety of technical methods have been employed in an attempt
to minimize postoperative abdominal adhesion formation. Compared with other means of transection, sharp mechanical transection of tissue is followed by the least amount of tissue reaction
and necrosis in one study.43 Peritoneal inflammation results from
the presence of foreign material or debris that may arise from
gauze, sponges, starch powder, suture, surgical drapes, gowns,
masks, and many other items that are found in postoperative
adhesions, inferring a causal relationship between the formation of adhesions and the presence of foreign material.44 In a
prospective study by Tulandi et al. analyzing patients undergoing laparotomy for gynecological procedures, peritoneal closure
offered no benefit in reducing postoperative adhesion formation. A comparison between the incidence of adhesion formation
between patients with separate closure of the peritoneum and
fascia to patients with closure of the fascia only was made. No
statistical difference was seen in the incidence of adhesion formation in patients with peritoneal closure (22.2%) and those without
peritoneal closure (15.8%).45 Another prospective study randomized patients to a separate layer closure of the peritoneum versus a single layer closure of the fascia. There was no difference in
wound complications including infection, incisional hernia, or
dehiscence between the two groups.46 Nonclosure of the peritoneum reduces operative time and is safer, allowing the underlying
viscera to remain under direct visualization during closure.
A lower rate of postoperative adhesions is observed in studies
evaluating laparoscopic to open surgery. Polymeneas et al. found
that 100% of all open cholecystectomy patients had thick and extensive adhesions to the operative site compared with a rate of 44%
of loose, easily separable adhesions between the gallbladder liver

5/21/2012 8:55:43 PM

168

Surgery: Evidence-Based Practice

bed and omentum or duodenum after laparoscopic cholecystectomy.47 One hundred and twenty-five patients with different prior
laparoscopic procedures were compared with 131 patients with
previous horizontal suprapubic laparotomy and 89 patients
with previous midline laparotomy in a study conducted by Audebert et al. The rates of umbilical adhesions were highest in those
with a midline laparotomy (51.7%), were intermediate in those with
horizontal suprapubic laparotomy (19.8%), and lowest in those
after laparoscopy (1.6%).48 Studies show that fewer adhesions form
between the incision and the operative site with a laparoscopic
approach. Research has also demonstrated a lower rate of reformation of adhesions after laparoscopic adhesiolysis.49 Milingos
et al. assessed adhesion reformation as a secondary endpoint in
his clinical study regarding pregnancy rates after open microsurgical and laparoscopic adhesiolysis for periadnexal adhesions.
Initial adhesion scores were calculated and compared with adhesion scores obtained on the same patients after open adhesiolysis
through laparotomy or laparoscopic adhesiolysis. Although scores
were similar before adhesiolysis, there was a greater reduction in
the adhesion scores in the laparoscopic group on second-look laparoscopy performed 3 to 6 months after the operation compared
with the group that underwent laparotomy.50
NSAIDs and corticosteroids have shown an ability to reduce
postoperative adhesions in animal models and a correlation has
been sought with human subjects.51-55 Other studies have examined agents that may interfere with the pathways of fibrin degradation and deposition. Anticoagulants such as heparin and low
molecular weight heparins (LMWH) have also demonstrated a
decrease in adhesion formation in animal studies.56,57 Recombinant tissue plasminogen activator, a fibrinolytic agent, has also
showed promise in animal models in reduction of postoperative
adhesions.58-60 Unfortunately, similar results in human investigations is lacking and the majority of studies that have reported
success in using these various agents to prevent postoperative
adhesions are limited to animals.
The premise of barrier devices is to provide protection from
adhesion formation by separating the layers of the peritoneum.
An ideal barrier device should provide unrestricted coverage of
the affected peritoneum, be easily applied by both laparoscopic
and open surgical methods, and remain effective throughout the
healing process.61 Various forms of barriers have been developed,
which include solid membranes and polymer solutions of polysaccharides such as cellulose, dextran, hyaluronic acid, and chitosan.
Viscous gels may form when these solutions are applied at the
end of the procedure. Membranes are placed directly on potential sites of adhesions. FDA-approved barriers currently include
hyaluronic acid-carboxymethylcellulose (Seprafi lm), polylactide membrane (Surgiwrap), regenerated cellulose (Interceed),
expanded polytetrafluoroethylene (Preclude), and icodextrin
solution (Adept). Seprafilm was designed as a nontoxic, nonimmunogenic, biocompatible material that reduces postoperative
abdominal adhesion formation by forming a hydrophilic gel that
provides a protective coating around traumatized tissues for up to
7 days during remesothelialization approximately 24 h after placement. A recent meta-analysis of eight randomized controlled trials, studied the efficacy of Seprafi lm in 4203 patients. Adhesions
were classified as grade 0: no adhesions; grade 1: least severe and
filmy, avascular, and translucent; grade 2: moderately severe and
medium thickness and limited vascularity; and grade 3: very
severe and dense and highly vascularized. There was a statistically

PMPH_CH18.indd 168

higher incidence of grade 0 adhesions in patients treated with


Seprafi lm compared with controls. The severity of grade 2 and 3
adhesions among Seprafi lm-treated patients was also significantly less than observed for the control group. The incidence
of grade 1 adhesions was statistically insignificant between Seprafi lm and control groups. According to this meta-analysis,
Seprafi lm decreases abdominal adhesions following surgery.62
However, there are studies that demonstrate no difference in the
overall incidence of postoperative SBO between patients treated
with Seprafim and controls.63 Although not statistically significant, there is a slightly higher frequency of abdominal abscesses
and a more frequent incidence of fistulas, sepsis, and peritonitis
with the use of Seprafim compared with controls (2% vs. <1%)
and a higher frequency of anastomotic leaks (4% vs. 2%) when
wrapped around a fresh anastomosis.64
Recommendation: A variety of methods have been utilized
to decrease the formation of postoperative abdominal adhesions. These include sharp dissection and minimization of tissue
trauma, the reduction of foreign body contamination within the
surgical field, and the application of minimally invasive surgical
techniques when indicated. Although the use of barriers between
the peritoneal layers has shown a decreased incidence and severity
of postoperative abdominal adhesions, there has been no impact
on the rate of postoperative SBO. (Grade A recommendation)

ADHESIONAL SBO
With clinical evidence of strangulation, patients with adhesional
SBO may require immediate operative intervention to minimize
the risk of necrotic bowel and perforation.65 A trial of nonoperative management is acceptable for patients without this clinical
picture.66 The conventional approach allows 48 h of nonoperative
management as the majority of adhesional SBO resolve during
this time period.67
5. Is the early use of water-soluble contrast indicated in the
diagnosis/management of SBO?
Recently, the role of water-soluble contrast medium in predicting the need for surgery after the failure of conservative management in the setting of adhesive small bowel obstruction has
been evaluated. The most commonly used water-soluble contrast
agent is meglumine amidotrizoate (Gastrografin) is which has an
osmolarity of 1900 mosm/L and is a mixture of sodium diatrizoate and meglumine diatrizoate. Bowel wall edema contributes
to proximal bowel distention and increases the pressure gradient
across an obstructing region. Gastrografin causes water to enter
the bowel lumen thereby decreasing bowel wall edema through
the process of osmosis.68
The use of Gastrografin has been studied for its possible therapeutic function in the resolution of SBO and its potential ability to
predict successful nonoperative management of SBO. Abbas et al.
conducted a recent meta-analysis to investigate the diagnostic
use of Gastrografin in determining the successful conservative
management of SBO. Patients with SBO without indications for
immediate surgery were evaluated with 100 mL of Gastrografin,
orally or by NGT, and followed by abdominal imaging in 4 to
24 h. A partial SBO was indicated by the presence of contrast in
the colon which indicated a higher probability of resolution with

5/21/2012 8:55:43 PM

Small Bowel Surgery

conservative management. On the other hand, a complete SBO is


signified when contrast does not reach the colon and is unlikely to
resolve without surgical intervention. There were six prospective
randomized studies that addressed whether Gastrografin has a
therapeutic role in the treatment of SBO. In patients who would
require surgery, there was no observed difference in resolution of
SBO in patients treated with Gastrografin versus placebo. Based
on the meta-analysis, Gastrografin is useful in the prediction of
successful conservative management of SBO but has no therapeutic benefit in patients with SBO.
Recommendation: Water-soluble contrast agents such as Gastrografin have been able to predict successful conservative management in patients with SBO without clinical indications for
operation. However treatment with Gastrografin has not been
shown to reduce the need for surgical intervention in patients
with postoperative SBO. (Grade A recommendation)
6. Can computed tomography (CT) predict the need for operation in patients with incomplete SBO?
The diagnosis of SBO can be made on plain abdominal radiographs in only 67 to 80% of patients.70 Patients with complete,
closed-loop obstruction or strangulation associated with obstruction may demonstrate abdominal radiographs that are entirely
normal.71 CT scan is well established in the diagnosis of small
bowel obstruction since the first large published series showing its utility and efficacy.72 Findings indicative of a SBO on CT
include air-fluid levels, collapsed loops distal to distended bowel
loops, and a possible transition point. CT has a sensitivity of
94% to 100% and an accuracy of 90% to 95% in confirming the
diagnosis and revealing the cause of small bowel obstruction.73
Reviews have also shown CT to be highly accurate for diagnosing ischemic bowel with a sensitivity of 83%, specificity of 92%,
positive predictive value of 79%, and a negative predictive value of
93%.74 Potential signs of reversible or early small bowel strangulation on CT include mesenteric edema, engorgement of the mesenteric vasculature, slight thickening of the bowel wall, and the
target sign. The CT findings of high attenuation of the bowel
wall, pneumatosis, hemorrhagic changes in the mesentery, gas in
the portal vein, and poor or no enhancement portend bowel wall
bowel infarction or gangrene. 85% of patients with strangulated
SBO were correctly identified when investigators used of a combination of five highly specific findings on CT, which included a
large amount of ascites, diff use engorgement of the mesenteric
vasculature or mesenteric haziness, an unusual course of the
mesenteric vasculature, poor enhancement of the bowel wall, and
a serrated beak.75
Recommendation: Specific CT scan findings have the ability
to identify SBO and in conjunction with other specific findings
may diagnose bowel ischemia and allow for surgical intervention. If clinically indicated, patients presenting with SBO should
be evaluated with CT scan of the abdomen following obstruction
series. (Grade A recommendation)
7. Is there any difference between stapled or handsewn techniques for bowel anastomosis?
Handsewn and stapled anastomosis in colon and rectal surgery
were compared in a meta-analysis combining data from 13 randomized controlled trials. No significant differences in wound
infection rate, cancer recurrence rate, mortality, or total, clinical,

PMPH_CH18.indd 169

169

and radiological leak rates between handsewn and stapled techniques. However, there was a statistically higher rate of strictures
and intraoperative technical problems with a stapled anastomosis.76 The increased stricture rate with stapled anastomoses may
result from higher collagen levels and an overactive inflammatory
response.77
Recommendation: Prospective randomized trials have shown
no differences between handsewn and stapled anastomoses in
regards to wound infection, leak rate, mortality, and cancer
recurrence. However, the rate of postoperative stricture formation may be higher with stapled anastamoses. The type of anastomosis that is performed, whether handsewn or stapled, should
be based on the surgeons comfortability and preference. (Grade A
recommendation)
8. Are there any techniques to predict outcome of a small bowel
anastomosis (i.e. flourescein, doppler, surface oximetry)?
There has been interest in investigating the usefulness of flourescein, doppler, and even surface oximetry measurements for the
intraoperative determination of small bowel viability dating
back to 1980. Bulkley et al. conducted a prospective, controlled
trial comparing Doppler and fluoroscein techniques with standard clinical judgement in the intraoperative determination of
small intestinal viability following acute intestinal ischemic disease.78 Seventy-one ischemic bowel segments were independently
assessed with doppler, fluoroscein, and clinical judgement 15 min
after surgical correction of the underlying lesion in 28 consecutive
patients operated on for acute intestinal ischemia. The resected
segments were evaluated by a pathologist, blinded with respect
to specific viability assessment technique, in areas with the greatest concern for viability as indicated by any of the three assessment
techniques. A sensitivity of 100%, specificity of 100%, predictive
value of 100%, and overall accuracy of 100% was observed in the
assessment of viability of determinant segments with the fluorescein method. The differences, favoring the fluorescein method,
in specificity, predictive value, and overall accuracy were all statistically significant when compared with the Doppler method.
Although the overall accuracy of standard clinical judgment was
relatively high at 89%, the predictive value was relatively low at
64%. Although not statistically significant, the Doppler method
was less reliable than clinical judgment in the assessment of small
intestinal viability.
Locke et al. conducted a study on 11 mongrel dogs to evaluate the utility of surface oximetry to assess bowel viability.79 A
baseline surface oxygen tension tension (Pso2) was determined
through the placement of miniaturized oxygen electrodes placed
on stomach serosa and several sites along the antimesenteric border of small intestine. The small intestine was then sequentially
devascularized to a specific Pso2. Both high and low Pso2 anastomoses were performed on each dog with reexploration 48 h after
the first operation. The researchers found that all anastomoses
healed when Pso2 exceeded 50% of the initial normal value, that
1/3 of anastomoses leaked when Pso2 ranged from 30% to 50%
of the initial value, and that anastomoses necrosed when created
with Pso2 below 30% predevascularization. However, in a recent
similar investigation in rats by Posma et al. blood flow was significantly decreased to even less that 10% of baseline as measured by
near-infrared spectroscopy but had no significant effect on anastomotic wound strength.78

5/21/2012 8:55:44 PM

170

Surgery: Evidence-Based Practice

Although there are numerous techniques that have been


implemented for the intraoperative determination of small bowel
viability, there have been no reported studies to date using these
techniques to predict small bowel anastomotic failure in humans.
Recommendation: There is currently a lack of literature
regarding the use of techniques such as fluorescein, Doppler, and
surface oxygen tension to predict the probability of anastomotic
failure in human populations. Fluorescein has been shown to be a
useful adjunct in the determination of intestinal viability, whereas
Doppler flowmeter appears to add little to clinical judgement of
intestinal viability. (Grade C recommendation)
9. Are there any predictors of ischemia in patients with partial
SBO?
There are a multitude of studies regarding the use of CT scan
to predict the need for an operation to prevent strangulation in
patients with PSBO. There are no published prospective studies
to date that identify clinical features or CT scan findings that predict small bowel ischemia alone in patients with PSBO. Although
indicators such as leukocytosis, fever, tachycardia, focal tenderness, and elevated lactate levels suggest the intestinal ischemia,

they are not very specific.80 However, it is useful to identify the


factors associated with strangulating obstruction. Zielinski et al.
performed a recent retrospective review in 100 consecutive SBO
patients treated both operatively and nonoperatively to identify preoperative risk factors associated with strangulating SBO.81 The
clinical findings of vomiting, absence of small bowel feces sign,
mesenteric edema, and free intraperitoneal fluid were significantly associated with the need for operative intervention on multivariate analysis. Mesenteric edema and free intraperitoneal fluid
increased the risk of the need for operative intervention by 3.6and 3.8-fold, respectively. Patients with small bowel feces sign
were 5-fold less likely to require operation whereas patients with
vomiting were 4.7 times more likely to require operation. When
patients presented with all of these clinical features they were 16
times more likely to require operation.
Recommendation: There are currently no reliable clinical or
radiographic indicators that predict ischemia in patients with PSBO.
The creation of a multivariate model incorporating laboratory studies, CT findings, and features of the clinical scenario may predict
which patients may need operative intervention to prevent delayed
recognition of infarcted bowel. (Grade C recommendation)

Clinical Question Summary


Question

Answer

1 Are there other agents and/or


techniques that can be used to
improve the duration of POI?

Minimally invasive surgical techniques, local epidural


anesthetics, avoidance of NGT, and early enteric
feedings.

7-20

2 Does chewing gum shorten the


duration of POI?

Yes, chewing gum has been shown to decrease length of


POI and LOS, but not to a clinically meaningful degree.

21, 22

3 Does the use of selective opiate


receptor inhibitors decrease duration
of POI?

Yes, it has an impact of both the duration of POI,


tolerance of solid diet, and LOS, but not to a clinically
meaningful degree. The cost/benefit ratio is unclear.

23-30

4 Are there any techniques/agents


that have been shown to decrease
intraabdominal adhesion formation
following laparotomy?

Sharp dissection, minimizing tissue trauma, decreasing


foreign bodies in surgical field, barrier devices.

43-64

5 Is the early use of water-soluble


contrast indicated in the diagnosis/
management of SBO?

The use of water-soluble contrast has shown to predict


the success of conservative management, but has not
shown to decrease the need for operation.

69

6 Can computer tomography (CT)


predict the need for operation in
patients with incomplete SBO?

CT scan can diagnose SBO and SBO with associated


ischemia requiring surgical intervention.

70-75

7 Is there any difference between stapled


or handsewn techniques for bowel
anstamosis?

There has been no difference shown between the two,


however, stapled anastomoses have a higher rate of
stricture.

76, 77

8 Are there any techniques to predict


outcome of a small bowel anastomosis
(i.e. flourescein, doppler, surface
oximetry)?

There is currently a lack of literature regarding the


use of techniques such as fluorescein, Doppler, and
surface oxygen tension to predict the probability of
anastomotic failure in human populations.

78, 79

9 Are there any predictors of ischemia in


patients with partial SBO?

There are currently no reliable clinical or radiographic


indicators that predict ischemia in patients with PSBO.

80, 81

PMPH_CH18.indd 170

Grade

References

5/21/2012 8:55:44 PM

Small Bowel Surgery

REFERENCES
1. Livingston EH, Passaro EP. Postoperative ileus. Dig Dis Sci.
1990;35:121-131.
2. Kehlet H, Holte K. Review of postoperative ileus. Am J Surg.
2001;182(suppl):3-10.
3. Artinyan A, Nunoo-Mensah JW, Balasubramaniam S, et al. Prolonged postoperative ileus-definition, risk factors, and predictors after surgery. World J Surg. 2008;32:1495-1500.
4. Holte K, Kehlet H. Postoperative ileus:a preventable event. Br J
Surg. 2000;87:1480-1493.
5. Clevers GJ, Smout AJ. The natural course of postoperative ileus
following abdominal surgery. Neth J Surg. 1989;41:97-99.
6. Mattei P, Rombeau JL. Review of the pathophysiology and management of postoperative ileus. World J Surg. 2006;30:1382-1391.
7. Lacy AM, Garcia-Valdecasas JC, Delgado S, et al. Laparoscopyassisted colectomy versus open colectomy for treatment of nonmetastatic colon cancer:a randomized trial. Lancet. 2002;359:
2224-2229.
8. Salimath J, Jones MW, Hunt DL, et al. Comparison of return of
bowel function and length of stay in patients undergoing laparoscopic versus open colectomy. JSLS. 2007;11:72-75.
9. DeWinter BY, Boeckxstaens GE, DeMan JG, et al. Effects of muand kappa-opioid receptors on postoperative ileus in rats. Eur J
Pharmacol. 1997;339:63-67.
10. Petros JG, Realica R, Ahmad S, et al. Patient-controlled analgesia
and prolonged ileus after uncomplicated colectomy. Am J Surg.
1995;170:371-374.
11. Fotiadis RJ, Badvie S, Weston MD, et al. Epidural analgesia in
gastrointestinal surgery. Br J Surg. 2004;91:828-841.
12. Jorgensen H, Wetterslev J, Moiniche S, et al. Epidural local anesthetics versus opioid-based analgesic regimens on postoperative gastrointestinal paralysis, PONV, and pain after abdominal
surgery. Cochrane Database Syst Rev. 2001;Issue 1:CD001893.
DOI:10.1002/14651858.CD001893.
13. Kelley MC, Hocking MP, Marchand SD, et al. Ketorolac prevents postoperative small intestine ileus in rats. Am J Surg.
1993;171:85-88.
14. Ferraz AA, Cowles VE, Condon RE, et al. Nonopioid analgesics shorten the duration of postoperative ileus. Am Surg.
1995;61:1079-1083.
15. Cheatham ML, Chapman WC, Key SP, et al. A meta-analysis of
selective versus routine nasogastric decompression after elective
laparotomy. Ann Surg. 1995;221:469-476.
16. Desmond P, Raman R, Idikula J. Effect of nasogastric tubes on
the nose and maxillary sinus. Crit Care Med. 1991;19:509-511.
17. McAlister FA, Bertsch K, Man J, et al. Incidence of and risk factors for pulmonary complications after nonthoracic surgery. Am
J Respir Crit Care Med. 2005;171:514-517.
18. Nelson R, Tse B, Edwards S. Systematic review of prophylactic
nasogastric decompression after abdominal operations. Br J
Surg. 2005;92:673-680.
19. Han-Geurts IJ, Hop WC, Kok NF, et al. Randomized clinical
trial of the impact of early enteral feeding on postoperative ileus
and recovery. Br J Surg. 2007;94:555-561.
20. Stewart BT, Woods RJ, Collopy BT, et al. Early feeding after elective open colorectal resections:a prospective randomized trial.
Aust N Z J Surg. 1998;68:125-128.
21. Asao T, Kuwano H, Nakamura J, et al. Gum chewing enhances
early recovery from postoperative ileus after laparoscopic colectomy. J Am Coll Surg. 2002;195:30-32.

PMPH_CH18.indd 171

171

22. Chan MK, Law WL. Use of chewing gum in reducing postoperative ileus after elective colorectal resection:a systematic review.
Dis Colon Rectum. 2007;50:2149-2157.
23. Sternini C, Patierno S, Selmer IS, et al. The opioid system in the gastrointestinal tract. Neurogastroenterol Motil. 2004;16(suppl):3-16.
24. Schmidt WK. Alvimopan (ADL 8-2698) is a novel peripheral
opioid antagonist. Am J Surg. 2001;182(suppl):27-38.
25. Leslie JB. Alvimopan for the management of postoperative ileus.
Ann Pharmacother. 2005;39:1502-1510.
26. Taguchi AT, Sharma N, Saleem RM, et al. Selective postoperative inhibition of gastrointestinal opioid receptors. N Engl J Med.
2001;345:935-940.
27. Wolff BG, Michelassi F, Gerkin TM, et al. Alvimopan, a novel,
peripherally acting opioid antagonist. Results of a multicenter,
randomized, double-blind, placebo-controlled, phase III trial
or major abdominal surgery and postoperative ileus. Ann Surg.
2004;240:728-735.
28. Delaney CP, Weese JL, Hyman NH, et al. Phase III trial of alvimopan, a novel, peripherally acting, mu opioid antagonist, for
postoperative ileus after major abdominal surgery. Dis Colon
Rectum. 2005;48:1114-1125.
29. Viscusi ER, Goldstein S, Witkowski T, et al. Alvimopan, a peripherally acting mu-opioid receptor antagonist, compared with placebo in postoperative ileus after major abdominal surgery. Surg
Endosc. 2006;20:64-70.
30. Wolff BG, Weese JL, Ludwig KA, et al. Postoperative ileusrelated morbidity profi le in patients treated with alvimopan
after bowel resection. J Am Coll Surg. 2007;204:609-616.
31. Menzies D, Ellis H. Intestinal obstruction from adhesions-how
big is the problem? Ann R Coll Surg Engl. 1990;72:60-63.
32. Svanes Fevang BT, Fevang J, Lie SA, et al. Long-term prognosis
after operation for adhesive small bowel bbstruction. Ann Surg.
2004;240:193-201.
33. Cheong YC, Laird SM, Li TC, et al. Peritoneal healing and adhesion formation/reformation. Hum Reprod Update. 2001;7:556-566.
34. Holmdahl L. The role of fibrinolysis in adhesion formation. Eur J
Surg. 1997;577(suppl):24-31.
35. Holmdahl L, Eriksson E, Eriksson BI, et al. Depression of peritoneal fibrinolysis during operation is a local response to trauma.
Surgery. 1998;123:539-544.
36. Menzies D. Postoperative adhesion: their treatment and relevance in clinical practice. Ann R Coll Surg Engl. 1993;75:147-153.
37. Menzies D, Parker M, Hoare R, et al. Small bowel obstruction
due to postoperative adhesions: treatment patterns and associated costs in 110 hospital admissions. Ann R Coll Surg Engl.
2001;83:40-46.
38. Wysocki A, Pozniczek M, Kulawik J, et al. Peritoneal adhesions
as a cause of small bowel obstruction. Przegl Lek. 2003;60:32-35.
39. Van der Krabben AA, Dijkstra FR, Nieuwenhuijzen M, et al.
Morbidity and mortality of inadvertent enterotomy during
adhesiotomy. Br J Surg. 2000;87:467-471.
40. Coleman MG, McLain AD, Moran BJ. Impact of previous surgery on time taken for incision and division of adhesions during
laparotomy. Dis Colon Rectum. 2000;43:1297-1299.
41. Van Goor H. Consequences and complications of peritoneal
adhesions. Colorectal Dis. 2007;9(suppl):25-34.
42. Diamond MP, Freeman ML. Clinical implications of postsurgical adhesions. Hum Reprod Update. 2001;7:567-576.
43. Elkins TE, Stovall TG, Warren J, et al. A histologic evaluation of
peritoneal injury and repair: implications for adhesion formation. Obstet Gynecol. 1987;70:225-228.

5/21/2012 8:55:44 PM

172

Surgery: Evidence-Based Practice

44. Saxen L, Myllarniemi H. Foreign material and postoperative


adhesions. N Engl J Med. 1968;279:200-202.
45. Tulandi T, Hum HS, Gelfand MM. Closure of laparotomy incisions with or without peritoneal suturing and second-look laparoscopy. Am J Obstet Gynecol. 1988;158:536-537.
46. Hugh TB, Nankivell C, Meagher AP, et al. Is closure of the peritoneal layer necessary in the repair of midline surgical abdominal wounds? World J Surg. 1990;14:231-234.
47. Polymeneas G, Theodosopoulos T, Stamatiadis A, et al. A comparative study of postoperative adhesion formation after laparoscopic vs open cholecystectomy. Surg Endosc. 2001;15:41-43.
48. Audebert AJ, Gomel V. Role of microlaparoscopy in the diagnosis of peritoneal and visceral adhesions and in the prevention of
bowel injury associated with blind trocar insertion. Fertil Steril.
2000;73:631-635.
49. Gutt CN, Oniu T, Schemmer P, et al. Fewer adhesions induced by
laparoscopic surgery? Surg Endosc. 2004;18:898-906.
50. Milingos S, Kallipolitis G, Loutradis D, et al. Adhesions: laparoscopic surgery versus laparotomy. Ann NY Acad Sci. 2000;900:
272-285.
51. De Leon FD, Toledo AA, Sanfi lippo JS, et al. The prevention of
adhesion formation by nonsteroidal antiinflammatory drugs:
an animal study comparing ibuprofen and indomethacin. Fertil
Steril. 1984;41:639-642.
52. Guvenal T, Cetin A, Ozdemir H, et al. Prevention of postoperative adhesion formation in rat uterine horn model by nimesulide:
a selective COX-2 inhibitor. Hum Reprod. 2001;16:1932-1735.
53. Nishimura K, Nakamura RM, DiZerega GS. Ibuprofen inhibition
of postsurgical adhesion formation: a time and dose response
biochemical evaluation in rabbits. J Surg Res. 1984;36:115-124.
54. Maurer JH. Bonaventura LM. The effect of aqueous progesterone
on operative adhesion formation. Fertil Steril. 1983;39:485-489.
55. Hckel M, Ott S, Siemann U, et al. Prevention of peritoneal
adhesions in the rat with sustained intraperitoneal dexamethasone delivered by a novel therapeutic system. Ann Chir Gynaecol.
1987;76:306-313.
56. Kutlay J, Ozer Y, Isk B, et al. Comparative effectiveness of several agents for preventing postoperative adhesions. World J Surg.
2004;28:662-665.
57. Bahadir I, Oncel M, Kement M, et al. Intra-abdominal use of
taurolidine or heparin as alternative products to an antiadhesive barrier (Seprafi lm) in adhesion prevention:an experimental
study on mice. Dis Colon Rectum. 2007;50:2209-2214.
58. Doody KJ, Dunn RC, Buttram VC. Recombinant tissue plasminogen activator reduces adhesion formation in a rabbit uterine
horn model. Fertil Steril. 1989;51:509-512.
59. Orita H, Fukasawa M, Girgis W, et al. Inhibition of postsurgical adhesions in a standardized rabbit model: intraperitoneal
treatment with tissue plasminogen activator. Int J Fertil. 1991;36:
172-177.
60. Menzies D, Ellis H. The role of plasminogen activator in adhesion prevention. Surg Gynecol Obstet. 1991;172:362-366.
61. Yeo Y, Kohane DS. Polymers in the prevention of peritoneal
adhesions. Eur J Pharm Biopharm. 2008;68:57-66.
62. Zeng Q, Yu Z, You J, et al. Efficacy and safety of Seprafi lm for
preventing postoperative abdominal adhesions:systematic review
and meta-analysis. World J Surg. 2007;31:2125-2131.

PMPH_CH18.indd 172

63. Fazio VW, Cohen Z, Fleshman JW, et al. Reduction in adhesive


small-bowel obstruction by Seprafi lm adhesion barrier after
intestinal resection. Dis Colon Rectum. 2005;49:1-11.
64. Beck DE, Cohen Z, Fleshman JW, et al. A prospective, randomized, multicenter, controlled study of the safety of Seprafi lm
adhesion barrier in abdominopelvic surgery of the intestine. Dis
Colon Rectum. 2003;46:1310-1319.
65. Playforth RH, Holloway JB, Griffin WO. Mechanical small bowel
obstruction: a plea for early surgical intervention. Ann Surg.
1970;171:783-788.
66. Seror D, Feigin E, Szold A, et al. How conservatively can postoperative small bowel obstruction be treated? Am J Surg.
1993;165:121-125.
67. Cox MR, Gunn IF, Eastman MC, et al. The safety and duration of
non-operative treatment for adhesive small bowel obstruction.
Aust N Z J Surg. 1993;63:367-371.
68. Laerum F, Stordahl A, Aase S. Water-soluble contrast media
compared with barium in enteric follow-through. Local effects
and radiographic efficacy in rats with simple obstruction of the
small bowel. Acta Radiol. 1988;29:603-610.
69. Abbas SM, Bissett IP, Parry BR. Meta-analysis of oral watersoluble contrast agent in the management of adhesive small
bowel obstruction. Br J Surg. 2007;94:404-411.
70. Maglinte DD, Balthazar EJ, Kelvin FM, et al. The role of radiology in the diagnosis of small bowel obstruction. AJR. 1997;168:
1171-1180.
71. Gough IR. Strangulating adhesive small bowel obstruction with
normal radiographs. Br J Surg. 1978;65:431-434.
72. Megibow AJ, Balthazar EJ, Cho KC, et al. Bowel obstruction:
evaluation with CT. Radiology. 1991;180:313-318.
73. Maglinte DD, Gage SN, Harmon BH, et al. Obstruction of the
small intestine:accuracy and role of CT in diagnosis. Radiology.
1993;188:61-64.
74. Mallo RD, Salem L, Lalani T, et al. Computed tomography of
ischemia and complete obstruction in small bowel obstruction:a
systematic review. J Gastrointest Surg. 2005;9:690-694.
75. Ha HK, Kim JS, Lee MS, et al. Differentiation of simple and
strangulated small-bowel obstructions: usefulness of known CT
criteria. Radiology. 1997;204:507-512.
76. MacRae HM, McLeod RS. Handsewn vs. stapled anastomoses
in colon and rectal surgery:a meta-analysis. Dis Colon Rectum.
1998;41:180-189.
77. Dziki AJ, Duncan MD, Harmon JW, et al. Advantages of
handsewn over stapled bowel anastomosis. Dis Colon Rectum.
1991;34:625-627.
78. Bulkley GB, Zuidema GD, Hamilton SR, et al. Intraoperative
determination of two adjuvant methods (Doppler and fluorescein) compared with standard clinical judgement. Ann Surg.
1981;193:628-635.
79. Locke R, Hauser CJ, Shoemaker WC. The use of surface oximetry
to assess bowel viability. Arch Surg. 1984;119:1252-1256.
80. Landercasper J, Cogbill TH, Merry WH, et al. Long-term outcome after hospitalization for small-bowel obstruction. Arch
Surg. 1993;128:765-770.
81. Zielinski MD, Eiken PW, Bannon MP, et al. Small bowel obstruction-who needs and operation? A multivariate prediction model.
World J Surg. 2010;24:910-919.

5/21/2012 8:55:44 PM

Commentary on
Small Bowel Surgery
Ronald M. Stewart

Lee, Hong, Dangleben, and Badellino have written a lucid summary of the evidence concerning the management of a number of
clinically relevant issues concerning surgery of the small bowel. On
the whole, I agree with their assessment and would commend this
chapter to the practicing surgeon. The authors and their editors
seem to have imposed a measure on the grading scale that is not
listed in the Oxford Centre criteria: criteria of clinical relevance
and cost utility, as they downgrade some evidence based on these
criteria.

4. Regarding the risk of adhesion formation the authors conclude


sharp dissection, minimizing tissue trauma, decreasing foreign
bodies in surgical field and the use of barrier devices lead to fewer
adhesions, although the extrapolation to reduced risk of small
bowel obstruction is largely lacking. This seems sensible. Although
the authors do not explicitly state this it seems to me that the use
of barriers (hyaluronic acid-carboxymethylcellulose [Seprafilm],
polylactide membrane [Surgiwrap], regenerated cellulose
[Interceed], expanded polytetrafluoroethylene [Preclude], and
icodextrin solution [Adept]) should be selectively used based on
the clinical situation.
5. Concerning the use of water-soluble contrast and computed
tomography scan the authors conclude that these options are
helpful in the diagnosis of small bowel obstruction. This fits
with my clinical experience. These two options can be combined
and seem to have significant utility in situations where it is not
clear that there is a complete bowel obstruction.
6. Concerning whether there are any differences between hand
sewn and stapled anastomoses the authors conclude that there is
no difference; however, stapled anastomoses have a higher rate
of stricture. I do not believe that there is a difference between
the two techniques including stricture formation, except the
circular EEA anastomoses, which would hardly ever be used
on the small bowel. I cannot recall ever seeing a stricture from
the side-side stapled anastomoses (functional end-to-end) with
the GIA type staplers. The weight of evidence supports there is
no difference, including stricture formation.
7. There are no reliable techniques to determine the outcome of
a small bowel anastomosis and the authors conclude this in
their review; however, it is known that patients who have shock,
immunocompromise, a systemic inflammatory state or exposed
small bowel have a greater risk of anastomotic failure.
8. The authors conclude there are no reliable predictors of small
bowel ischemia in patients with small bowel obstruction; however,
they cite papers that at least add improved predictive ability for
complete bowel obstruction. I believe traditional clinical findings,
combined with CT scan criteria are helpful at predicting those at
risk for dead bowel or ultimate need for operation.

1. The authors conclude minimally invasive surgical techniques,


local epidural anesthetics, avoidance of nasogastric tubes, and
early enteric feedings result in a shortened duration of adynamic
ileus. They believe the evidence supporting this deserves a grade
of A. Although this is a broad sweeping number of techniques,
their conclusions are sensible and seem well supported by
modern evidence. These techniques and approaches also are
sensible and plausible; therefore, it seems prudent that we should
adopt these measures to shorten the recovery time following
abdominal procedures.
2. Concerning the question of does chewing gum reduce the
duration of postoperative ileus, the authors conclude that it
does, but not to a clinically significant degree. They give this
evidence a grade of C. Chewing gum is an extremely low-cost
measure that has been shown in at least two randomized trials
to reduce the duration of ileus, so I believe Lee et al. should
have graded the evidence for its use higher. Given the fact that
it is very low cost, very low risk, may reduce the risk of parotitis
and is probably more comfortable for the patient, I would
recommend its use in patients with normal mental status.
3. The authors conclude that postoperative use of selective opiate
antagonists do reduce the duration of ileus and length of stay,
but not to a clinically significant degree. I believe the level
of evidence is higher than a C grade, but share the authors
skepticism and their concern regarding costbenefit concerns.
These assessments are broader than the Oxford criteria. It seems
that there is the need for a large scale, nonindustry sponsored
clinical trial for these agents.

173

PMPH_CH18.indd 173

5/21/2012 8:55:44 PM

CHAPTER 19

Crohns Disease of the


Small Bowel
Thomas Donkar, Andrea Bafford, and Randolph M. Steinhagen

INTRODUCTION

usually for complications related to either fibrostenosis (obstruction) of perforation (abscess/fistula), which occur when the disease
becomes refractory to medical therapy. Other intractable symptoms or complications that may lead to surgery include major hemorrhage, failure to thrive, cancer, and extraintestinal symptoms.5,6
Typically surgery for small bowel CD involves some combination of bowel resection, strictureplasty, or drainage of abscess.

Inflammatory bowel disease (IBD) is comprised of two major


groups of disorders: Crohns disease (CD) and ulcerative colitis
(UC). Both are chronic, relapsing inflammatory disorders of the
gastrointestinal (GI) system, with unclear etiology. CD can affect
the entire GI tract from mouth to anus, but most commonly affects
the ileum and the colon.1 It is not uncommon for CD to affect multiple sites in the intestinal tract simultaneously and in discontinuous fashion, that is skip lesions. Its incidence and prevalence
within the United States is 5/100,000 and 100/100,000, respectively.
CD is most commonly diagnosed in adolescence and early adulthood, but there is also a second peak in age of onset during the
fifth and sixth decades of life. CD is more prevalent in certain ethnic groups, including those with Jewish backgrounds from Europe
and the United States and also in those from Scandanavia, when
compared with those from Asia and Africa.2
Clinically, patients with CD present with symptoms of abdominal pain, diarrhea, weight loss, fever, or rectal bleeding, depending on the region of the GI tract involved. Patients with CD of the
small bowel typically follow one of the two distinct forms: fibrostenotic and perforating disease. The fibrostenotic form results from
chronic inflammation and produces fibrosis in the bowel wall and
narrowing of the lumen, which can lead to bowel obstruction. The
perforating form results from transmural inflammation which can
lead to abscess formation and subsequent fistulazation, or rarely,
free perforation. As there is no definitive cure, treatment of CD is
directed toward symptomatic relief and improving overall quality of life. While medical management is important for induction
and maintenance of remission and amelioration of symptoms,
surgery, when indicated, has been shown to alleviate symptoms
and improve overall quality of life.3,4

INTESTINAL OBSTRUCTION
Intestinal obstruction is a common complication which is the result
of inflammation, fibrosis, thickening of the wall, and narrowing of
the lumen with resultant stricture formation. Patients present with
abdominal pain, cramping, nausea, vomiting, and abdominal distention. While it is possible for the patient to present with acute
obstruction, in most cases obstructive symptoms are chronic and
the patients learn to restrict their diet, lose weight, and often develop
significant abdominal distension. Acute obstruction can usually be
managed nonoperatively with bowel rest and medications designed
to reduce the inflammation and edema. Surgery is indicated only if
there is a failure to respond or if the clinical picture worsens.5,6
Chronic obstruction results from the presence of fibrostenotic
strictures, which restrict passage of intestinal contents but do
not stop it completely. Crohns patients with chronic obstruction
report an impaired quality of life, are afraid to eat, and frequently
lose weight. Once the fibrosis becomes established, there is little
that medical management can do to reverse it and without surgical intervention these patients can become severely malnourished
and debilitated.5-7

INTRAABDOMINAL ABSCESSES

1. What are the indications for surgery in small bowel CD?

Approximately 10% to 30% of patients affected with CD are expected


to develop an intraabdominal abscess during the course of their lifetime.8 Patients present clinically with fever, generalized malaise, and
abdominal pain. Primary treatment for intraabdominal abscess is

CD typically produces a clinical course characterized by remission


and exacerbation. Most patients do not require surgery during the
first 8 to 10 years of their disease.5 When surgery is indicated, it is
174

PMPH_CH19.indd 174

5/21/2012 8:56:32 PM

Crohns Disease of the Small Bowel

drainage, either percutaneously or operatively, which generally will


result in control of systemic sepsis and allow time for the patient
to be repleated prior to resection of the diseased bowel. Percutaneous drainage may serve as a viable alternative to surgical intervention, with success dependent upon abscess complexity and patient
clinical characteristics.9 Gutierrez found no significant difference
between time to abscess resolution when comparing percutaneous
and surgical drainage, and noted that approximately 1/3 of percutaneously drained abscesses needed definitive surgery within
1 year following drainage.10 Garcia reported that percutaneous
drainage was successful about 50% of the time, but concluded that
surgical drainage was superior to both medical management and a
percutaneous approach, with regards to abscess recurrence.11

175

the involved small bowel is necessary. Closure rates with surgical


intervention are approximately 60% to 80%.13
Medical management has been shown to frequently be effective, particular in those patients with EC fistula secondary to
recurrent disease. In a randomized clinical trial, Present showed
a closure rate of 55% with use of a 3-dose regiment of infliximab,
an anti-TNF monoclonal antibody. It should be noted however,
that the best overall success was seen in patients with fistulizing
perianal disease.14 Other nonsurgical approaches include the use
of immunomodulators such as 6-mercaptopurine, which also has
been shown to aid in spontaneous closure; however, recurrence is
common after the medication is discontinued.13,14

FREE PERFORATION
FISTULA
Fistula formation in CD is common, occurring in approximately
30% of patients. The most common enteric fistula observed is
enteroenteric, between two portions of small intestine. Th is may be
asymptomatic; however, if the communication is very distal in the
intestinal tract, this may result in the bypass of a large segment of
bowel leading to diarrhea and malabsorption, thus requiring surgical intervention. Symptomatic fistulas require resection of the
fistula source, usually with reanastomosis; if the secondary loop
is normal and is just an innocent bystander it may be treated by
simple wedge resection and closure. If however, it is also involved
with CD, it also will need to be resected.5,6
Enterocolic fistulas are typically found between ileum and
sigmoid colon. This communication can allow for rapid transit of
enteric contents and thus can cause metabolic abnormalities and a
hypovolemic state, if there is a large amount of diarrhea. Repair of
ileo-sigmoid fistulas involves resection of the involved ileum, and
primary repair or resection of the sigmoid colon.5,6 If CD is evident
in the sigmoid colon, or if the location of the opening precludes safe
primary closure, then a sigmoid resection should be undertaken.12
An enterovesical fistula is a connection between the small
bowel and bladder that typically presents with pneumaturia,
fecaluria, or most characteristically, a urinary tract infection containing multiple organisms. This is treated with surgical resection
of the involved small bowel and closure of the hole in the bladder, if it can be identified. Often there is so much inflammation
involving the bladder wall that the opening cannot be accurately
seen. Since the bladder is intrinsically normal, it is important not
to debride the fibrotic, inflamed tissue as reduced bladder capacity may result. Whether the hole is closed or not, an indwelling
urinary catheter should be left in place for 7 to 10 days, with the
expectation that the bladder will heal. To be on the safe side,
cystography may be done prior to removal of the catheter.
Enterocutaneous (EC) fistulas are communications between
the small intestine and the abdominal wall, usually communicating through the skin at the site of a previous incision. This type of
fistula is most often seen from postoperative anastomotic breakdown or from bowel with recurrent disease. The etiology of the EC
fistula dictates the most effective management strategy. Patients
with EC fistulas secondary to anastomotic breakdown are usually
managed initially nonoperatively. If the fistula fails to heal with
a regimen of bowel rest, total parenteral nutrition and adequate
control of sepsis in a period of about 6 weeks, then resection of

PMPH_CH19.indd 175

Free perforation is a rare complication of CD and has been reported


at 1% to 3% lifetime risk.6 However, Werbin reported rates as high
as 8%, and concluded that a more conservative approach to therapy
and delayed surgery increases this risk.15 Patients who clinically
present with signs of an acute abdomen, indicative of free perforation, should be taken emergently for surgery. Resection of the
perforated segment results in a 10-fold reduction in mortality, as
compared with simple closure of the perforated site.16

MALIGNANCY
The annual incidence of small bowel carcinoma in the general
population is small, approximately 2%. Although uncommon in
the Crohns population as well, a meta-analysis by Jess estimated
that patients with CD have a 27-fold increase in overall risk for
developing small bowel cancer.17 A literature review by Dossett
showed that patients with CD who develop small bowel adenocarcinoma are younger, and more likely male, when compared
with patients without Crohns.18 CD patients were also found to
have a poorer prognosis, as the cancer was usually found at a more
advanced stage and was more likely to be poorly differentiated.
Small bowel adenocarcinoma in CD occurs predominantly in the
ileum. The clinical presentation is likely to be obstruction, hemorrhage, fistula, or perforation. Unfortunately, the diagnosis is most
often made during postoperative histological examination of the
resected specimen.

FAILED MEDICAL MANAGEMENT


(INTRACTABILITY)
The initial management of CD is medical. Failure of medical management is defined as uncontrolled symptoms on maximal doses of
medication; disease progression while on maximum medical therapy; significant side effects of the medication; or noncompliance.
2. What is the etiology of CD?
Although no single factor has been identified as the precipitating insult leading to the development of CD, research has concentrated on four main areas: environmental, immunologic, genetic,
and microbiologic.19

5/21/2012 8:56:32 PM

176

Surgery: Evidence-Based Practice

ENVIRONMENTAL
As documented by Ouyang, the number of cases of CD and UC
continues to rise in once low-incidence areas of Southern Europe
and Asia.19 The new influx of reported cases in these areas has
given rise to the hypothesis of an association between IBD and
social and economic progress. The hygiene hypothesis proposes
that the increase in incidence of IBD in these regions is due to
a dramatic change from a dirty lifestyle, with high microbial
exposure, to a clean lifestyle, with low microbial exposure and
overall improvement in hygiene and sanitation.20 This lifestyle
change is a result of safer food and water, and better access to
antibiotics, vaccines, contributing to fewer infections. Thus, with
deceased microbial exposure so early in life, the bodys immune
system has faced fewer challenges and is less prepared, therefore
setting the stage for a surge in allergic, autoimmune, and chronic
inflammatory diseases.20,21
A meta-analysis by Mahid confirmed that smoking remains
the strongest environmental risk factor for IBD, specifically CD.22
A multicenter Japanese study showed that dietary factors, specifically an increased consumption of sugars and sweeteners, fats,
fish, and shellfish, have a positive association with an increased
risk of CD.23 Studies of environmental triggers such as adverse live
events, that is, stress-inducing events, have been inconclusive in
showing a direct relationship between stress and IBD.24

MICROBIOLOGIC
Recent studies have determined that the role of infectious agents
in the etiology of IBD is less likely. Newer research has discredited
the proposal that the measles vaccine was a factor in the etiology of
CD. Mycobacterium paratuberculosis has also been shown to be a
noncontributor to CD, as studies of patients who received a 2-year
course of a combination of clarithromycin, rifabutin, and clofazimine antituberculous therapy showed no clinical improvement.25
However, adherent-invasive Escherichia coli has been shown to
colonize the ileal mucosa of CD patients, and has the ability to
adhere to and invade the intestinal epithelial cells.26 Although it is
unclear as to whether this bacteria directly causes ileal CD or if it
is a secondary invader of the previously inflamed mucosa.
The loss of immunologic tolerance against intestinal flora
remains an active theory of the etiology of the development of
IBD.27 Animal studies involving mice show that intestinal flora
exacerbate the colitis rather than directly causing the disease.28
However, studies comparing the intestinal flora of IBD and those
of healthy patients, in an attempt to establish a connection, have
been inconclusive.20

GENETIC
Genetic information pertaining to IBD etiology includes the identification of the IBD1 region on chromosome 16, and the discovery
of the NOD2/CARD15 gene, regarded as the first susceptibility
gene in CD.29-32 A mutation in the NOD2 gene leads to the activation of nuclear factor (NF) B,33 a factor in the inflammatory
response, and as shown by Hampe et al., substantially increases a
patients susceptibility to developing CD.34 Other potential genetic
associations include the DLG5 gene and the IL23R gene, which
encode for a scaffolding protein involved in maintaining epithe-

PMPH_CH19.indd 176

lial integrity, and a subunit of the receptor for the cytokine IL-23,
respectively.35-37
There is a vast amount of information that has been developed
in recent years that demonstrates the important role of genetic
factors in the etiology of IBD. Only a tiny fraction of this material
is touched upon in this discussion. Regardless, this area of study
presents a wealth of untapped knowledge concerning the susceptibility to and the underlying mechanisms of these diseases. It is
an area that warrants continued study and one that will heavily
influence the future of clinical management.

IMMUNOLOGIC
The role of immunologic factors in the etiology of CD has been
studied extensively, with the major focus on adaptive immunity. Studies have attempted to distinguish CD from UC based
on immunologic characteristics of adaptive immunity and T-cell
lineage, with CD being associated with Th-1 and production of
IFN-, and UC with Th-2 and production of IL-13.38-40 Recent
studies have reported a third addition to the Th lineage, Th-17,
which produces mainly IL-17 as well as IL-6 and TNF-.40 Like
Th-1, Th-17 plays a role in mediation of immunity and inflammation in autoimmune and inflammatory conditions.41 Furthermore, IL-17+ cells have been detected by immunohistochemical
staining, along with Th-17 cells in the inflamed mucosa of CD
patients, suggesting that Th-17 may have a possible immunologic
role in the etiology of CD.42,43
As immunologic therapy continues to be a promising treatment for Crohns patients, a better understanding of the immunologic and genetic factors in the pathogenesis of CD are important
to help tailor future clinical therapies.
3. What is the role of strictureplasty in the surgical treatment
of CD?
Approximately 30% to 50% of patients who require surgical intervention for CD will develop recurrent disease that requires additional surgery in the future.44,45 Patients that undergo multiple
surgical resections are at significant risk for the development of
short bowel syndrome. Thus, a surgical policy of bowel preservation is warranted to reduce this risk. This topic was addressed
in 1982 by Lee and Papaioannou, who published results on the
use of strictureplasty to relieve obstructive symptoms in Crohns
patients.46 A bowel conservation approach, via strictureplasty, has
become the accepted means of treating obstructive CD, especially
in those patients with multiple strictures who would otherwise
require multiple resections or resection of a very significant length
of intestine to extirpate the disease. Patients treated with repeated
strictureplasty operations are less likely to develop short bowel
syndrome than those treated with multiple resections.47
A meta-analysis by Yamamoto et al. looked at 1112 patients
in 23 studies. They found that patients treated by strictureplasty
most commonly presented with obstruction secondary to fibrotic
stenosis. In this review of more than 3000 strictureplasties, the
most common areas of small bowel involvement were jejunum
and/or ileum;48 a finding further substantiated by a meta-analysis
by Tichanskly et al.49 Both studies confirmed that the majority of
patients who undergo strictureplasty, do so by one of two techniques, HeinekeMikulicz or Finney. The length of affected bowel
usually determines the technique used, as the HeinekeMikulicz
strictureplasty is used with strictures less than 10 cm in length

5/21/2012 8:56:32 PM

Crohns Disease of the Small Bowel

and the Finney strictureplasty is used for strictures between 10


and 20 cm in length. The Finney technique involves making a
U-shaped incision over the stricture site and then closing the opposing surfaces of bowel. When strictured bowel segments are greater
than 20 cm in length, a side-to-side isoperistaltic strictureplasty, as
reported by Michelassi may be used as Finney or other techniques
may not be technically feasible in such long segments.50
Yamamoto found that the overall recurrence rates following strictureplasty at 5- and 10-year follow-up were 41% and 51%,
respectively. Several other studies in this analysis also reported no
difference in recurrence rates when comparing the two most common strictureplasty techniques, HeinekeMikulicz and Finney.48
A prospective study by Tonelli51 comparing recurrence rates following ileocolic strictureplasty with those following resection concluded that there was difference between the two groups. However,
this study included only 28 patients, and therefore larger studies
are needed to confirm the results. Comparison of recurrence rates
between strictureplasty and bowel resection have proved difficult
and currently there are no other randomized trials on which to base
any conclusions. However, it is clear that strictureplasty remains
an important option for bowel preservation in CD when surgery is
indicated for the treatment of obstructive complications.
4. Does the type of anastomosis affect outcome or recurrence?
While medical management is the mainstay of initial therapy
for CD, it has been estimated that 70% of patients will undergo
surgery by 10 to 15 years following diagnosis.44 However, surgery
is not curative and recurrence of disease is common. A Cochran
review by Doherty found endoscopic recurrence rates of approximately 70% to 80%, and reoperative rates of nearly 25% to 30%,
5 years after surgical intervention.45 Several studies have attempted
to determine whether surgical technique can impact these high
rates of recurrence.
In a multicenter, randomized control trial, McLeod looked at
the effect of anastomotic construction on recurrence of CD following ileocolic resection.52 The types of anastomosis evaluated
included end-to-end hand-sewn anastomosis and side-to-side
stapled anastomosis. After a mean follow-up of 11.9 months, an
endoscopic recurrence rate was 42.5% in the former compared
with 37.9% in the latter (p = 0.55), thus showing no statistical difference. Symptomatic recurrence was also found to not be significantly different at 21.9% and 22.7% (p = 0.92), respectively. This
study also reported no difference in complication rates between
the anastomotic types and showed rates that were similar to other
studies. Another meta-analysis performed by Simillis, which
included a total of 661 patients in eight studies, also reported
similar results with regards to the effect of anastomotic technique
on Crohns recurrence rates, finding no significant difference
between end-to-end and side-to-side anastomoses.53
5. How is CD differentiated from UC?
Distinguishing CD from UC is an important clinical decision when the disease is limited to the colon. It is a process that
requires reviewing findings across multiple diagnostic modalities. It is combination of medical history, clinical evaluation, and
endoscopic and histological findings that help make the determination. Additionally recent studies have shown a possible use of
serologic markers, pANCA and ASCA, as a way of differentiating
between CD and UC.54-56 A definitive diagnosis is not always clear

PMPH_CH19.indd 177

177

resulting in a diagnosis of indeterminate colitis (IC) in approximately 10% of patients.57,58


Patients afflicted with either of the IBDs may present with
abdominal pain, diarrhea, weight loss, and fever. A large majority of UC patients will also present with blood in their stool and
at times, may have symptomatic anemia. Although bloody stool
is present in almost all patients with UC, patients with CD may
also experience bleeding, making a clear distinction based on
bleeding alone, difficult. Since CD can involve any portion of the
intestinal tract, while UC affects only the large intestine, involvement of any portion of the intestine other than the large intestine
will favor the diagnosis of CD. Other features that point to the
diagnosis of CD are rectal sparing, the presence of strictures or
fistulas, skip lesions, perianal disease, transmural inflammation,
and granulomas, which are features that can be seen on histologic
examination. In particular, perianal suppuration with abscesses
and fistulae, is a hallmark of CD. It must be emphasized, however,
that none of these distinctions can make either diagnosis with
absolute certainty (except perhaps granulomas, which are found
in a relatively small percentage of patients with CD) and in some
cases clear and certain determination is not possible.
Colonoscopic evaluation with biopsies can be very useful in
differentiating CD from UC.59 As CD often involves the terminal
ileum, while UC generally does not, a full colonoscopy with evaluation of this region is essential. Grossly, CD has been described as
inflammation with discrete ulcers, known as aphthous ulcers, that
can appear throughout the colon or small bowel in noncontiguous fashion, referred to as skip lesions.60,61 These aphthous ulcers
sometimes combine to form a larger network of longitudinal ulceration, creating an appearance known as cobblestoning.61 While
colonoscopy is useful for examining the TI and colon, additional
modalities such as upper GI X-rays with small bowel follow through,
esophagogastroduodenoscopy, and capsule endoscopy may be useful for diagnosing and evaluating CD proximal to the terminal
ileum. In comparison, UC is limited to the large intestine (with the
exception of occasional backwash ileitis which can produce ileal
inflammation), always involves the rectum and extends proximally
for a variable distance, but always in a contiguous fashion.60,61 No
skip lesions are present and during active disease superficial ulcers,
pseudopolyps, and mucosa that is erythematous and friable can be
observed.60,61 Specimens from Crohns patients show transmural
inflammation and lymphocytic aggregates evident in all layers of
bowel wall. In addition, noncaseating granulomas are sometimes
found. In comparison, UC specimens show inflammation limited
to the mucosa and submucosa. In areas of active inflammation,
superficial erosions and or ulcerations can also be found, as well as
a decrease in mucin producing goblet cells.61
Serological markers, such as pANCA and ASCA, have recently
been described as having utility for distinguishing CD from UC.
pANCA has been found in about 60% to 80% of UC patients,54,55 and
also in a subgroup of CD patients described by Vasiliauskas as having UC-like Crohns that is, those CD patients who present with
left-sided colitis, abdominal pain, urgency, and hematochezia.56
A prospective study performed by Joosens found that patients diagnosed with IC who had a positive serology were more likely, over
time to eventually be given a definitive diagnosis then those with
negative serology; specifically ASCA+/pANCA was predictive of
CD and ASCA/pANCA+ was predictive of UC or UC-like CD.62
While biological markers have not yet been able to make a clear
distinction between CD and UC in all cases, they may provide a
window into future diagnosis and treatment.

5/21/2012 8:56:32 PM

178

Surgery: Evidence-Based Practice

6. When is fecal diversion necessary in the surgical treatment


of CD?
Fecal diversion has been shown to decrease the incidence of and
increase the time interval to CD recurrence.63-65 However, restoration of GI tract continuity is preferred by both patients and
surgeons and therefore permanent stoma is avoided whenever
possible. Nevertheless, certain conditions, including severe fistulizing perianal CD and uncontrolled colitis or proctitis, are best
managed with stoma creation, possibly on a temporary basis.
The frequency of perianal involvement in patients with CD
has been reported to range from 13% to 38%.66 Manifestations of
perianal CD include anal skin tags, perianal abscesses and fistulae, anal fissures, anorectal strictures and rectovaginal fistulae.67
Local treatment of these conditions is frequently unsuccessful.68 Studies have demonstrated both effective symptom control
and improved healing rates when stomas are used in the treatment of perianal CD.69-71
Rehg et al. compared outcomes of patients with perianal CD
who were treated with local surgical therapy and fecal diversion
to those who underwent local surgical therapy alone. Eightyfive percent of patients treated with fecal diversion had complete
resolution of their fistulae compared with only 19% of patients
treated with local surgery alone. Intestinal continuity was reestablished in 46% of diverted patients, of which 50% remained disease
free.70 In another study, fecal diversion led to temporary symptom improvement in 29% of patients, initial improvement with
later plateau in 33% of patients, and healing in 19% of patients.
The remaining 19% of patients had no effect.71 Makowiec et al.
prospectively evaluated 90 Crohns patients with perianal fistulae and found that fecal diversion significantly increased healing
rates and decreased recurrence rates.72 Other studies have similarly demonstrated improved healing with the creation of diverting stomas.73,74
Fecal diversion has also been shown to induce remission in
patients with Crohns coloproctitis. In an early study by Oberhelman et al., diverting ileostomy performed in 13 patients with CD
of the colon led to immediate and persistent symptom relief in
all patients.75 Similarly, Edwards et al. found that defunctioning
stomas led to acute remission in 48 of 55 (87%) of patients with
refractory Crohns colitis.76
Finally, as with other disease processes, diverting stomas
are used in Crohns surgery to protect high-risk anastomoses or
delay anastomosis construction until risk factors are minimized.
Known risk factors for anastomotic complication include prior
radiation, poor nutrition, bowel obstruction, sepsis, immunosuppressive medications, and low rectal anastomosis.77
A 12% to 49% rate of permanent stoma is reported in patients
with perianal CD.78-83 Predictors of permanent stoma include
anorectal strictures, complex perianal fistulae, rectovaginal fistulae, colonic disease, rectal disease, subtotal, left-sided, or rectal resection, history of temporary fecal diversion, and fecal
incontinence.78,82-84
7. How much small bowel should be resected at the time of
surgery (i.e., do macroscopically or microscopically involved
margins make a difference)?
In the early days of surgical therapy for CD surgical bypass or
exclusion procedures were advocated. However, the frequency of
persistent symptoms and need for reoperation following bypass
procedures led to concern over their efficacy.85,86 Further, studies

PMPH_CH19.indd 178

demonstrated an incidence of carcinoma arising within bypassed


segments.86,87 For these reasons and because of equivalent safety
profi les, resection has come to replace bypass as the surgical technique of choice for the treatment of intestinal CD complications.
Nevertheless, the rate of recurrence and reoperation after surgery
for CD remains high, with one study demonstrating a 31% chance
of needing a second operation within 10 years and a 28% likelihood of needing a third procedure within another 10 years following initial surgery.88 When CD does recur, it most frequently
occurs at the site of previous anastomosis.89-92 When deciding upon
length of resection, surgeons must therefore balance the need to
relieve symptoms, prevent anastomotic complications, and avoid
recurrence with the need to preserve bowel length.
Clearance of all macroscopic disease during intestinal resection for CD is necessary to prevent unacceptably high recurrence
rates and possibly also an increased risk of anastomotic complications.93 The need to obtain microscopically negative margins,
however, is more controversial. In a retrospective study published
in 1977, Bergman and Krause found that patients with CD who
underwent intestinal resections with a 10 cm or greater margin
of microscopically normal bowel had significantly lower rates
of disease recurrence than those who did not; 29% versus 84%,
respectively.94 Additional reports have also favored wide surgical
margins.95-100
In contrast, other studies have demonstrated no association
between disease recurrence and presence of histologic abnormalities at resection margins. Pennington et al. compared rates of
clinical recurrence, suture line recurrence, and need for reoperation between patients with positive and negative histologic surgical margins and found no difference.91 Further, the incidences of
postoperative leak, fistula, abscess, and obstruction were identical
(6%) in both groups. Additional retrospective studies have also
demonstrated no relationship between rate of clinical recurrence
and presence of microscopic disease at resection margins.64,101-106
Kotanagi et al. categorized pathologic margin appearance as histologically normal, showing nonspecific changes, showing changes
suggestive of CD, and showing changes diagnostic of CD. These
authors found no association between histologic changes at
the resection margin and anastomotic recurrence in 100 patients
with CD.107
The potential relationship between histologic changes at the
resection margin and disease recurrence has led authors to evaluate the role of intraoperative frozen section. Hamilton et al. compared patients who had resection margins determined by frozen
section with those with margins chosen on the basis of visual
inspection alone.108 Frozen section was found to be a poor predictor of actual margin involvement. Further, the incidences of
postoperative anastomotic leakage, clinical recurrence, and reoperation were similar whether or not intraoperative frozen section
was utilized. Two earlier studies also demonstrated the low reliability of frozen section in predicting disease recurrence.109,110
Only one study has addressed the effect of histologic disease
at surgical margins in a prospective manner. Fazio et al. randomly assigned 152 patients undergoing ileocolic resection for
CD to either 2 cm or 12 cm macroscopically negative margins.111
Follow-up data was available in 131 patients. The authors found
no difference in disease recurrence between the two groups. In
addition, recurrence was unrelated to the presence or absence of
microscopic disease at the resection margin. Interestingly, in this
study, extended resection did not predict achievement of microscopically disease-free margins.

5/21/2012 8:56:32 PM

Crohns Disease of the Small Bowel

In summary, although early studies seemed to favor wide


resection of bowel involved with CD, more contemporary reports
and the only randomized controlled trial to address this subject
matter have shown no association between residual microscopic
disease and disease recurrence. Given this data, conservative
resection margins should be utilized in Crohns intestinal surgery
to preserve bowel length.
8. What is the role of laparoscopy in small bowel CD?
Laparoscopy in colon and rectal surgery has been shown to
decrease postoperative pain and narcotic usage,112 duration of
hospital stay,113 and time to first bowel function.113 Because of the
technical challenge posed by acute bowel wall and mesenteric
inflammation as well as fistulizing and phlegmonous processes,
CD was initially considered a contraindication to laparoscopic
surgery. Further, the ability to assess for diff use disease and skip
lesions via laparoscopy has been questioned given the diminished
tactile feedback and potentially, decreased exposure associated
with this surgical technique. Multiple studies, however, have demonstrated the safety and efficacy of laparoscopy in CD.114-119
Two large meta-analyses address the topic of laparoscopy in
CD. Tan et al. pooled the results of 14 studies, published between
1990 and 2006, which compared laparoscopic with open bowel
resection for CD.114 The majority of these reports focused on ileocolic resections. Two prospective, randomized, controlled trials
and two case-matched studies were included. The remaining

179

10 studies were retrospective, single institution reviews. An 11.2%


rate of conversion from laparoscopy to open surgery was found.
Laparoscopy was associated with significantly shorter time to first
bowel function and oral intake, decreased duration of hospital
stay, decreased cost, and decreased operative morbidity. Operative time was longer for laparoscopic cases. There was no difference in rate of disease recurrence, whether surgery was performed
laparoscopically or in open fashion.
Lesperance et al. utilized the 20002004 Nationwide Inpatient Sample to identify 49,609 bowel resections performed for
CD. Laparoscopic surgery was associated with improved outcomes compared with open surgery with respect to cost, length of
hospital stay, discharge disposition, postoperative GI, pulmonary
and cardiovascular complication, and mortality. There was no difference between the two groups in intraoperative, wound or infectious complications.115
Other studies have similarly demonstrated decreased narcotic usage, time to first bowel function and length of hospital stay
with laparoscopy compared with open surgery for CD.116-118,120,121
Operative time, however, is typically longer with laparoscopy.120,121
Mortality and recurrence have consistently been found to be
similar regardless of surgical technique.116-120 In conclusion, laparoscopy has led to improved surgical outcomes in CD without
negatively impacting mortality or rate of recurrence. Although
operative time is increased with laparoscopy, overall cost appears
to be reduced.

Clinical Question Summary


Question

Answer

Grade of
Recommendation

References

1 What are the indications for surgery in


small bowel CD?

Acute bowel obstruction that fails to resolve;


perforation leading to abscess, fistula or
peritonitis; malignancy; failed medical therapy;
and intolerance to the medications.

A
B

14, 16
8-12, 15, 17

2 What is the etiology of CD?

Some combination of environmental,


immunologic, genetic, and microbiologic
factors.

A
B
B

21, 24
22, 25, 28-30
34-37, 41

3 What is the role of strictureplasty in


the surgical treatment of CD?

Strictureplasty is a safe alternative to bowel


resection for the treatment of fibrostenotic
obstructing CD; it is especially useful in
recurrent disease, when long and/or multiple
segments of small intestine are involved,
and any situation in which preservation of
intestinal length is of paramount concern.

A
B
C

44, 48, 49
52
47, 50

4 Does the type of anastomosis affect


outcome or recurrence?

The type of anastomosis used for restoration


of intestinal continuity following resection
for CD does not affect the outcome or
recurrence rates.

A
B

44-46
43

5 How is CD differentiated from UC?

A combination of factors including clinical


presentation, endoscopic and radiographic
appearance, histologic examination, and
serologic markers can be used to try to
differentiate UC from Crohns colitis; in some
cases the distinction is not possible.

54-57, 59, 62

(Continued)

PMPH_CH19.indd 179

5/21/2012 8:56:32 PM

180

Surgery: Evidence-Based Practice

(Continued)
Question

Answer

6 When is fecal diversion necessary in


CD?

Fecal diversion is used primarily in patients


with fulminant proctocolitis; in patients with
high operative risk; in patients with severe
fistulizing perianal disease; and in the presence
of high-risk anastomoses.

69-76

7 How much small bowel should be


resected at the time of surgery (i.e.,
do macroscopically or microscopically
involved margins make a difference)?

The incidence of surgical complications and


subsequent recurrence rates are lower
when the surgical resection margins are
macroscopically free of disease; whether or
not the margins are microscopically involved
does not affect the surgical outcome or the
long-term recurrence rate.

64, 91, 93-111

8 What is the role of laparoscopy


in CD?

Laparoscopy in CD is associated with decreased


postoperative pain and narcotic use, shorter
time to resumption of bowel function, shorter
hospital lengths of stay, and decreased
cost when compared with open surgery.
Laparoscopy does not appear to affect
operative mortality or disease recurrence.

114-120

REFERENCES
1. Lichtenstein GR, Hanauer SB, Sandborn WJ. Management of
Crohns disease in adults. Am J Gastroenterol. 2009;104:465-483.
2. Loftus EV. Clinical epidemiology of inflammatory bowel
disease: Incidence, prevalence, and environmental influences.
Gastroenterology. 2004;126:1504-1517.
3. Scarpa M, Ruffolo C, Bassi D, et al. Intestinal surgery for
Crohns disease: Predictors of recovery, quality of life, and costs.
J Gastrointest Surg. 2009;13:2128-2135.
4. Delaney CP, Kiran RP, Senagore J, et al. Quality of life improves
within 30 days of surgery for Crohns disease. J Am Coll Surg.
2003;196:714-721.
5. Sachar DB. Indications for surgery in Crohns disease. Am J
Gastroenterol. 2007;102:S76-S78.
6. Gardiner KR, Dasari BVM. Operative management of small
bowel Crohns disease. Surg Clin N Am. 2007;87:587-610.
7. Spinelli A, Correale C, Szabo H, et al. Intestinal fibrosis in
Crohns disease: Medical treatment or surgery? Curr Drug
Target. 2010;11:242-248.
8. Cellini C, Safar B, Fleshman J. Surgical management of
pyogenic complications of Crohns disease. Inflamm Bowel Dis.
2010;16:512-517.
9. Jawhari A, Kamm MA, Ong C, et al. Intra-abdominal and pelvic
abscess in Crohns disease: results of non-invasive and surgical
management. Br J Surg. 1998;85:367-371.
10. Gutierrez A, Lee H, Sands B. Outcome of surgical versus
percutaneous drainage of abdominal and pelvic abscesses in
Crohns disease. J Gastroenterol. 2006;101:2283-2289.
11. Garcia JC, Persky SE, Bonis PA, et al. Abscesses in Crohns
disease: Outcome of medical versus surgical treatment. J Clin
Gastroenterol. 2001;32(5):409-412.
12. Michelassi F, Stella M, Balestracci T, et al. Incidence, diagnosis,
and treatment of enteric and colorectal fistulae in patients with
Crohns disease. Ann Surg. 1993;218(5):660-666.

PMPH_CH19.indd 180

Grade of
Recommendation

References

13. Portiz LS, Gagliano GA, McLeod RS, et al. Surgical management
of entero and colocutaneous fistulae in Crohns disease: 17 years
experience. Int J Colorectal Dis. 2004;19:481-485.
14. Present DH, Rutgeerts P, Targan S, et al. Infliximab for the
treatment of fistulas in patients with Crohns disease. N Engl J
Med. 1999;340:1398-405.
15. Werbin N, Haddad R, Greenberg R, et al. Free perforation in
Crohns disease. IMAJ. 2003;5:175-177.
16. Greenstein AJ, Sachar DB, Mann D, et al. Spontaneous free
perforation and perforated abscess in 30 patients with Crohns
disease. Ann Surg. 1987;205:72-76.
17. Jess T, Gamborg M, Matzen P, et al. Increased risk of intestinal
cancer in Crohns disease: A meta-analysis of population-based
cohort studies. Am J Gastoenterol. 2005;100:2724-2729.
18. Dossett LA, White LM, Welch DC, et al. Small bowel
adenocarcinoma complicating Crohns disease: Case series and
review of the literature. Am Surg. 2007;73(11):1181-1187.
19. Ouyang Q, Tandon R, Goh K-L, Ooi CJ, Ogata H, Fiocchi C. The
emergence of inflammatory bowel disease in the Asian Pacific
region. Curr Opin Gastroenterol. 2005;21:408-413.
20. Scaldaferri F, Fiocchi C. Inflammatory bowel disease: Progress
and current concepts of etiopathogenesis. J Digest Dis.
2007;8;171-178.
21. Yazdanbakhsh M, Kremsner PG, van Ree R. Allergy, parasites,
and the hygiene hypothesis. Science. 2002;296:490-494.
22. Mahid SS, Minor KS, Soto RE, Hornung CA, Galandiuk S.
Smoking and inflammatory bowel disease: A meta-analysis.
Mayo Clinic Proc. 2006;81:1462-1471.
23. Sakamoto N, Kono S, Wakai K, Fukuda Y, Satomi M, Shimoyama
T, Inaba Y, Miyake Y, Sasaki S, Okamoto K, Kobashi G, Washio
M, Yokoyama T, Date C, Tanaka H; Epidemiology Group of
the Research Committee on Inflammatory Bowel Disease in
Japan. Dietary risk factors for inflammatory bowel disease:
A multicenter case-control study in Japan. Inflamm Bowel Dis.
2005;11:154-163.

5/21/2012 8:56:32 PM

Crohns Disease of the Small Bowel

24. Mawdsley JE, Rampton DS. The role of psychological stress in


inflammatory bowel disease. Neuroimmunomodulation. 2006;
13:327-336.
25. Selby W, Pavli P, Crotty B, et al. Two-year combination antibiotic
therapy with clarithromycin, rifabutin, and clofazimine for
Crohns disease. Gastroenterology. 2007;32:2313-2319.
26. Barnich N, Carvalho FA, Glasser A-L, et al. CEACAM6 acts as
a receptor for adherent-invasive E. coli, supporting ileal mucosa
colonization in Crohns disease. J Clin Invest. 2007;117:15661574.
27. Elson CO, McCracken VJ, Dimmit RA, Lorenz RG, Weaver
CT. Experimental models of inflammatory bowel disease reveal
innate, adaptive and regulatory mechanisms of host dialogue
with the microbiota. Immunol Rev. 2005;206:260-276.
28. Bamias G, Okazawa A, Rivera-Nieves J, et al. Commensal
bacteria exacerbate intestinal inflammation but are not essential
for the development of murine colitis. J Immunol. 2007;178:
1809-1818.
29. Duerr RH. Genome-wide association studies herald a new era
of rapid discoveries in inflammatory bowel disease research.
Gastroenterology. 2007;132:2045-2062.
30. Curran ME, Lau KF, Hampe J, et al. Genetic analysis of
inflammatory bowel disease in a large European cohort
supports linkage to chromosomes 12 and 16. Gastroenterology.
1998;115:1066-1071.
31. Brant SR, Fu Y, Fields CT, et al. American families with Crohns
disease have strong evidence for linkage to chromosome 16 but
not chromosome 12. Gastroenterology. 1998;115:1056-1061.
32. Hugot J-P, Chamaiilard M, Zouali H, et al. Association of NOD2
leucine-rich repeat variants with susceptibility to Crohns
disease. Nature. 2001;411:599-603.
33. Inohara N, Ogura Y, Fontalba A, et al. Host recognition of
bacterial muramyl dipeptide mediated through NOD2. J Biol
Chem. 2003;278:5509-5512.
34. Hampe J, Cuthbert A, Croucher PJ, et al. Association between
insertion mutation in NOD2gene and Crohns disease in
German and British populations. Lancet. 2001;357:1925-1928.
35. Stoll M, Corneliussen B, Costello CM, et al. Genetic variation
in DLG5 is associated with inflammatory bowel disease. Nat
Genet. 2004;36:476-480.
36. Tremelling M, Waller S, Bredin F, et al. Genetic variants in TNFalpha but not DLG5 are associated with inflammatory bowel
disease in a large United Kingdom cohort. Inflamm Bowel Dis.
2006;12:178-184.
37. Duerr RH, Taylor KD, Brant SR, et al. A genome-wide association
study identifies IL23R as an inflammatory bowel disease gene.
Science. 2006;314:1461-1463.
38. Cobrin GM, Abreu MT. Defects of mucosal immunity leading
to Crohns disease. Immunol Rev. 2005;206:277-295.
39. Targan SR, Karp LC. Defects of mucosal immunity leading to
ulcerative colitis. Immunol Rev. 2005;206:296-305.
40. Weaver CT, Harrington LE, Mangan PR, Gavrieli M, Murphy
KM. Th17: An effector CD4 T cell lineage with regulatory T cell
ties. Immunity. 2006;24:677-688.
41. Bibiloni R, Mangold M, Madsen KL, Fedorak RN, Tannock
Steinman L. A brief history of Th17, the first major revision in
the Th1/Th2 hypothesis of T cell-mediated tissue damage. Nat
Med. 2007;13:139-145.
42. Fujino S, Andoh A, Bamba S, et al. Increased expression of
interleukin 17 in inflammatory bowel disease. Gut. 2003;52:65-70.
43. Annunziato F, Cosmi L, Santarlasci V, et al. Phenotypic
and functional features of human Th17 cells. J Exp Med.
2007;204:1849-1861.

PMPH_CH19.indd 181

181

44. Bernell O, Lapidus A, Hellers G. Risk factors for surgery and


postoperative recurrence in Crohns disease. Ann Surg. 2000;
231:38-45.
45. Doherty G, Bennett G, Patil S, et al. Interventions for prevention
of post-operative recurrence of Crohns disease (Review). The
Cochran Library 2009, Issue 4.
46. Lee EC, Papaioannau N. Minimal surgery for chronic
obstruction in patients with extensive or universal Crohns
disease. Ann R Coll Surg Engl. 1982;64:229-233.
47. Dietz DW, Fazio VW, Laureti S, et al. Strictureplasty in diff use
Crohns jejunoileitis: Safe and durable. Dis Colon Rectum. 2002;
45:764-770.
48. Yamamoto T, Fazio VW, Tekkis PP. Safety and efficacy of
strictureplasty for Crohns disease: A systematic review and
meta-analysis. Dis Colon Rectum. 2007;50:1968-1986.
49. Tichansky D, Cagir B, Yoo E, et al. Strictureplasty for Crohns
disease: Meta-analysis. Dis Colon Rectum. 2000;43:911-919.
50. Michelassi F. Side-to-side isoperistaltic strictureplasty for
multiple Crohns strictures. Dis Colon Rectum. 1996;39:345-349.
51. Tonelli F, Fazi M, Di Martino C. Ileocecal strictureplasy for
Crohns disease: Long-term results and comparison with
ileocecal resection. World J Surg. 2010;34(12):2860-2866.
52. McLeod RS, Wolff BG, Ross S, et al. Recurrence of Crohns
disease after ileocolic resection is not affected by anastomotic
type: results of a multicenter, randomized, controlled trial. Dis
Colon Rectum. 2009;52:919-927.
53. Simillis C, Purkayastha S, Tamamoto T, et al. A meta-analysis
comparing conventional end-to-end anastomosis vs other
anastomotic configurations after resection of Crohns disease.
Dis Colon Rectum. 2007;50:1674-1687.
54. Cambridge G, Rampton DS, Stevens JRT, et al. Anti-neutrophil
antibodies in inflammatory bowel disease: prevalence and
diagnostic role. Gut. 1992;33:668-674.
55. Duerr RH, Targan SR, Landers C J, et al. Anti-neutrophil
cytoplasmic antibodies in ulcerative colitis. Comparison with
other colitides/diarrheal illnesses. Gastroenterology. 1991;100:
1590-1596.
56. Vasiliauskas EA, Plevy SE, Landers C J, et al. Perinuclear antineutrophil cytoplasmic antibodies in patients with Crohns
disease define a clinical subgroup. Gastroenterology. 1996;110:
1810-1819.
57. Price AB. Overlap in the spectrum of non-specific inflammatory
bowel disease: Colitis indeterminate. J Clin Pathol. 1978;31:
567-577.
58. Wells AD, McMillan I, Price AB, et al. Natural history of
indeterminate colitis. Br J Surg. 1991;78:179-181.
59. Dejaco C, Oesterreicher C, Angelberger S. Diagnosing Colitis:
A prospective study on essential parameters for reaching a
diagnosis. Endoscopy. 2003;25(12):1004-1008.
60. Lennard-Jones JE. Classification of inflammatory bowel disease.
Scand J Gastroenterol. 1989;24(Suppl):2-6.
61. Sanders DS. The differential diagnosis of Crohns disease and
ulcerative colitis. Baillieres Clin Gastroenterol. 1998;12(1):
19-33.
62. Joosens S, Reinisch W, Vermeire S, et al. The value of serologic
markers in indeterminate colitis: A prospective follow-up study.
Gastroenterology. 2002;122:1242-1247.
63. Rutgeerts P, Goboes K, Peeters M, Hiele M, Penninckx F,
Aerts R, Kerremans R, Vantrappen G. Effect of faecal stream
diversion on recurrence of Crohns disease in the neoterminal
ileum. Lancet. 1991;338:771-774.
64. Heimann TM, Greenstein AJ, Lewis B, Kaufman D, Heimann
DM, Aufses AH, Jr. Prediction of early symptomatic recurrence

5/21/2012 8:56:32 PM

182

65.

66.

67.

68.
69.
70.

71.

72.
73.
74.

75.

76.

77.

78.

79.
80.

81.

82.

83.

84.

85.

Surgery: Evidence-Based Practice

after intestinal resection in Crohns disease. Ann Surg.


1993;218:294-298; discussion 298-299.
Sachar DB, Wolfson DM, Greenstein AJ, Goldberg J, Styczynski
R, Janowitz HD. Risk factors for postoperative recurrence of
Crohns disease. Gastroenterology. 1983;85:917-921.
American Gastroenterological Association Clinical Practice
Committee. American Gastroenterological Association medical
position statement: Perianal Crohns disease. Gastroenterology.
2003;125:1503-1507.
Sandborn WJ, Fazio VW, Feagan BG, Hanauer SB. AGA
technical review on perianal Crohns disease. Gastroenterology.
2003;125:1508-1530.
Frizelle FA, Santoro GA, Pemberton JH. The management of
perianal Crohns disease. Int J Colorectal Dis. 1996;11:227-237.
Grant DR, Cohen Z, McLeod RS. Loop ileostomy for anorectal
Crohns disease. Can J Surg. 1986;29:32-35.
Rehg KL, Sanchez JE, Krieger BR, Marcet JE. Fecal diversion
in perirectal fistulizing Crohns disease is an underutilized and
potentially temporary means of successful treatment. Am Surg.
2009;75:715-718.
Hong MK, Craig Lynch A, Bell S, Woods RJ, Keck JO, Johnston
MJ, Heriot AG. Faecal diversion in the management of perianal
Crohns disease. Colorectal Dis. 2011;13:171-176.
Makowiec F, Jehle EC, Starlinger M. Clinical course of perianal
fistulas in Crohns disease. Gut. 1995;37:696-701.
Harper PH, Kettlewell MG, Lee EC. The effect of split ileostomy
on perianal Crohns disease. Br J Surg. 1982;69:608-610.
Sher ME, Bauer JJ, Gorfi ne S, Gelernt I. Low Hartmanns
procedure for severe anorectal Crohns disease. Dis Colon
Rectum. 1992;35:975-980.
Oberhelman HA, Jr., Kohatsu S, Taylor KB, Kivel RM. Diverting
ileostomy in the surgical management of Crohns disease of the
colon. Am J Surg. 1968;115:231-240.
Edwards CM, George BD, Jewell DP, Warren BF, Mortensen NJ,
Kettlewell MG. Role of a defunctioning stoma in the management
of large bowel Crohns disease. Br J Surg. 2000;87:1063-1066.
Kingham TP, Pachter HL. Colonic anastomotic leak: Risk
factors, diagnosis, and treatment. J Am Coll Surg. 2009;208:269278.
Galandiuk S, Kimberling J, Al-Mishlab TG, Stromberg AJ.
Perianal Crohn disease: predictors of need for permanent
diversion. Ann Surg. 2005;241:796-801; discussion 801-802.
van Dongen LM, Lubbers EJ. Perianal fistulas in patients with
Crohns disease. Arch Surg. 1986;121:1187-1190.
Fry RD, Shemesh EI, Kodner IJ, Timmcke A. Techniques and
results in the management of anal and perianal Crohns disease.
Surg Gynecol Obstet. 1989;168:42-48.
Bell SJ, Williams AB, Wiesel P, Wilkinson K, Cohen RC, Kamm
MA. The clinical course of fistulating Crohns disease. Aliment
Pharmacol Ther. 2003;17:1145-1151.
Mueller MH, Geis M, Glatzle J, Kasparek M, Meile T, Jehle
EC, Kreis ME, Zittel TT. Risk of fecal diversion in complicated
perianal Crohns disease. J Gastrointest Surg. 2007;11:529-537.
Post S, Herfarth C, Schumacher H, Golling M, Schrmann G,
Timmermanns G. Experience with ileostomy and colostomy in
Crohns disease. Br J Surg. 1995;82:1629-1633.
Hurst RD, Molinari M, Chung TP, Rubin M, Michelassi F.
Prospective study of the features, indications, and surgical
treatment in 513 consecutive patients affected by Crohns
disease. Surgery. 1997;122:661-667; discussion 667-668.
Alexander-Williams J, Fielding JF, Cooke WT. A comparison
of results of excision and bypass for ileal Crohns disease. Gut.
1972;13:973-975.

PMPH_CH19.indd 182

86. Homan WP, Dineen P. Comparison of the results of resection,


bypass, and bypass with exclusion for ileocecal Crohns disease.
Ann Surg. 1978;187:530-535.
87. Greenstein AJ, Sachar D, Pucillo A, Kreel I, Geller S, Janowitz
HD, Aufses A, Jr. Cancer in Crohns disease after diversionary
surgery. A report of seven carcinomas occurring in excluded
bowel. Am J Surg. 1978;135:86-90.
88. Peyrin-Biroulet L, Loftus EV, Jr., Colombel JF, Sandborn WJ.
The natural history of adult Crohns disease in populationbased cohorts. Am J Gastroenterol. 2010;105:289-297.
89. De Dombal FT, Burton I, Goligher JC. Recurrence of Crohns
disease after primary excisional surgery. Gut. 1971;12:519-527.
90. Rutgeerts P, Geboes K, Vantrappen G, Kerremans R,
Coenegrachts JL, Coremans G. Natural history of recurrent
Crohns disease at the ileocolonic anastomosis after curative
surgery. Gut. 1984;25:665-672.
91. Pennington L, Hamilton SR, Bayless TM, Cameron JL. Surgical
management of Crohns disease. Influence of disease at margin
of resection. Ann Surg. 1980;192:311-318.
92. Cameron JL, Hamilton SR, Coleman J, Sitzmann JV, Bayless TM.
Patterns of ileal recurrence in Crohns disease. A prospective
randomized study. Ann Surg. 1992;215:546-551; discussion
551-552.
93. Burman JH, Cooke WT, Williams JA. The fate of ileorectal
anastomosis in Crohns disease. Gut. 1971;12:432-436.
94. Bergman L, Krause U. Crohns disease. A long-term study
of the clinical course in 186 patients. Scand J Gastroenterol.
1977;12:937-944.
95. Nygaard K, Fausa O. Crohns disease. Recurrence after surgical
treatment. Scand J Gastroenterol. 1977;12:577-584.
96. Ihsz M, Mester E, Rfi M. Experience in the surgical
management of Crohns disease. Am J Proctol. 1975;26:47-62.
97. Kresen R, Serch-Hanssen A, Thoresen BO, Hertzberg J.
Crohns disease: Long-term results of surgical treatment. Scand
J Gastroenterol. 1981;16:57-64.
98. Lindhagen T, Ekelund G, Leandoer L, Hildell J, Lindstrm C,
Wenckert A. Recurrence rate after surgical treatment of Crohns
disease. Scand J Gastroenterol. 1983;18:1037-1044.
99. Wolff BG, Beart RW, Jr., Frydenberg HB, Weiland LH,
Agrez MV, Ilstrup DM. The importance of disease-free margins
in resections for Crohns disease. Dis Colon Rectum. 1983;26:
239-243.
100. Fasth S, Hellberg R, Hultn L, Ahrn C. Site of recurrence,
extent of ileal disease and magnitude of resection in primary
and recurrent Crohns disease. Acta Chir Scand. 1981;147:
569-576.
101. Heuman R, Boeryd B, Bolin T, Sjdahl R. The influence of
disease at the margin of resection on the outcome of Crohns
disease. Br J Surg. 1983;70:519-521.
102. Adloff M, Arnaud JP, Ollier JC. Does the histologic appearance
at the margin of resection affect the postoperative recurrence
rate in Crohns disease? Am Surg. 1987;53:543-546.
103. Cooper JC, Williams NS. The influence of microscopic disease at
the margin of resection on recurrence rates in Crohns disease.
Ann R Coll Surg Engl. 1986;68:23-26.
104. Wettergren A, Christiansen J. Risk of recurrence and
reoperation after resection for ileocolic Crohns disease. Scand J
Gastroenterol. 1991;26:1319-1322.
105. Funayama Y, Sasaki I, Naito H, Kamiyama Y, Takahashi
M, Fukushima K, Segami H, Matsuno S. Surgical results in
Crohns diseasean analysis in view of cumulative risk or
recurrence and reoperation. Nippon Shokakibyo Gakkai Zasshi.
1991;88:33-39.

5/21/2012 8:56:32 PM

Crohns Disease of the Small Bowel

106. Chardavoyne R, Flint GW, Pollack S, Wise L. Factors affecting


recurrence following resection for Crohns disease. Dis Colon
Rectum. 1986;29:495-502.
107. Kotanagi H, Kramer K, Fazio VW, Petras RE. Do microscopic
abnormalities at resection margins correlate with increased
anastomotic recurrence in Crohns disease? Retrospective
analysis of 100 cases. Dis Colon Rectum. 1991;34:909-916.
108. Hamilton SR, Reese J, Pennington L, Boitnott JK, Bayless TM,
Cameron JL. The role of resection margin frozen section in the
surgical management of Crohns disease. Surg Gynecol Obstet.
1985;160:57-62.
109. Krause U, Bergman L, Norln BJ. Crohns disease. A clinical study
based on 186 patients. Scand J Gastroenterol. 1971;6:97-108.
110. Holmes SJ, Richter HM, 3rd, Garberoglio CA, Block GE, Moossa
AR. The surgical treatment of Crohns disease. Br J Clin Pract.
1981;35:5-12.
111. Fazio VW, Marchetti F, Church M, Goldblum JR, Lavery C,
Hull TL, Milsom JW, Strong SA, Oakley JR, Secic M. Effect
of resection margins on the recurrence of Crohns disease in
the small bowel. A randomized controlled trial. Ann Surg.
1996;224:563-571; discussion 571-573.
112. Weeks JC, Nelson H, Gelber S, Sargent D, Schroeder G; Clinical
Outcomes of Surgical Therapy (COST) Study Group. Shortterm quality-of-life outcomes following laparoscopic-assisted
colectomy vs open colectomy for colon cancer: A randomized
trial. JAMA. 2002;287:321-328.
113. Milsom JW, Bhm B, Hammerhofer KA, Fazio V, Steiger E,
Elson P. A prospective, randomized trial comparing laparoscopic versus conventional techniques in colorectal cancer
surgery: a preliminary report. J Am Coll Surg. 1998;187:46-54;
discussion 54-55.

PMPH_CH19.indd 183

183

114. Tan JJ, Tjandra JJ. Laparoscopic surgery for Crohns disease:
A meta-analysis. Dis Colon Rectum. 2007;50:576-585.
115. Lesperance K, Martin MJ, Lehmann R, Brounts L, Steele SR.
National trends and outcomes for the surgical therapy of
ileocolonic Crohns disease: A population-based analysis of
laparoscopic vs. open approaches. J Gastrointest Surg. 2009;
13:1251-1259.
116. Fichera A, Peng SL, Elisseou NM, Rubin MA, Hurst RD.
Laparoscopy or conventional open surgery for patients with
ileocolonic Crohns disease? A prospective study. Surgery.
2007;142:566-571; discussion 571.e1.
117. Stocchi L, Milsom JW, Fazio VW. Long-term outcomes of
laparoscopic versus open ileocolic resection for Crohns
disease: follow-up of a prospective randomized trial. Surgery.
2008;144:622-627; discussion 627-628.
118. Umanskiy K, Malhotra G, Chase A, Rubin MA, Hurst RD,
Fichera A. Laparoscopic colectomy for Crohns colitis. A large
prospective comparative study. J Gastrointest Surg. 2010;14:658663.
119. Lowney JK, Dietz DW, Birnbaum EH, Kodner IJ, Mutch MG,
Fleshman JW. Is there any difference in recurrence rates in
laparoscopic ileocolic resection for Crohns disease compared
with conventional surgery? A long-term, follow-up study. Dis
Colon Rectum. 2006;49:58-63.
120. Alessandroni L, Bertolini R, Campanelli A, Di Castro A, Natuzzi
G, Saraco E, Scotti A, Tersigni R. Video-assisted versus open
ileocolic resection in primary Crohns disease: A comparative
case-matched study. Updates Surg. 2010;62:35-40.
121. da Luz Moreira A, Stocchi L, Remzi FH, Geisler D, Hammel J,
Fazio VW. Laparoscopic surgery for patients with Crohns colitis:
A case-matched study. J Gastrointest Surg. 2007;11:1529-1533.

5/21/2012 8:56:32 PM

Commentary on Crohns Disease


of the Small Bowel
Tomas M. Heimann

This is an excellent review of the current state of our knowledge


of Crohns disease. Since its first description by Crohn, Oppenheimer and Ginsburg at Mount Sinai in 1932, we have made great
strides in the manipulation of the immune system and control of
symptoms, but we still have only a rudimentary understanding of
the etiology of this disease. With the recent advent of biological
drugs that allow immune modulation, the surgical treatment of
many of our patients has changed. In the last 10 years, the need for
proctectomy in patients with severe perianal disease has decreased
drastically and most can now be managed with local surgical
treatment of the fistulas using setons, and maintenance medical
treatment with biologic agents that are able to keep the perianal
disease under control. This treatment is effective for both patients
with Crohns disease and patients with indeterminate colitis that
have undergone restorative proctocolectomy and subsequently
develop severe pouchitis with perianal fistulas and behave like
Crohns disease. This combination treatment has been very effective and has decreased the excision of J-pouches to a minimum
in a group of patients that were very difficult to treat in the past.
In patients that still require abdominoperineal resection, healing
problems in the perineal wound seem to respond to the addition
of topical growth factors if the perineal wound remains unhealed
after 3 months, thereby decreasing the need for surgical wound
revisions and addition of gracilis or rectus muscle flaps.1
Surgical treatment of Crohns disease can be classified in
different ways. One method that has been previously described
is to divide the patients into obstructing and fistulizing disease.
Patients with obstruction tend to be easier to treat both medically
and surgically while those with fistulizing disease tend to have
more complications and need surgery more frequently. Another
classification which I find useful from a surgical point of view is to
classify Crohns disease patients requiring surgical treatment by
degree of difficulty.2 Patients with localized short segment disease
in the terminal ileum undergoing a limited resection can often
be done laparoscopically with a single resection and anastomosis. I prefer a side-to-side ileocolic anastomosis performed with a
long linear stapler instead of an end-side since anastomosis which
has a smaller surface area and may develop a symptomatic stricture sooner and therefore may shorten the symptom free interval.
Reoperations for recurrent disease and more complex resections
with fistulas and abscesses are more difficult to perform laparoscopically and often open surgery becomes necessary. These cases
range from relatively straight forward resections if there are few
adhesions, to very difficult procedures if the recurrent disease
involves multiple sites, severe adhesions and internal fistulas and

abscesses. I feel very strongly that these cases should be done by


an experienced gastrointestinal (GI) surgeon, able to perform difficult GI surgery to minimize the risk of complications, anastomotic leaks, and ureteral injury. In the final category, are patients
with short length residual small bowel, with multiple sites of
recurrent disease, often with internal or enterocutaneous fistulas
and abdominal wall issues which result from incisional hernias
caused by previous surgeries. These cases can be very challenging.
Reopening the abdominal cavity and sorting out the various loops
of bowel can take several hours and avoidance of enterotomies
is crucial to preserve bowel length and reduce the risk of creating new fistulas and abscesses, which may produce catastrophic
results. Bowel preservation is of utmost importance and closure
of the incision can be challenging if the abdominal wall is thinned
out from previous wound infections and the presence of large ventral hernias where the bowel may be covered only with skin. In
these situations, reinforcing the closure with acellular dermis may
be the best way to obtain a reliable closure especially in the face
of significant wound contamination. Although great progress has
been made in the area of small bowel transplant, it is always better
to be able to preserve sufficient small bowel to allow the patient to
rely on his own intestines for nutrition, preferably with minimal
or no need for long-term intravenous nutrition. Septic patients
with multiple fistulas and abscesses and an empty abdominal cavity with only a few remaining loops of small bowel are not the best
candidates for good outcomes after intestinal transplant as it is
difficult to eradicate the infection which is necessary before the
transplant may be performed.
The distinction between Crohns disease and ulcerative colitis is of critical importance in determining surgical treatment in
patients with colitis. Performance of restorative proctocolectomy,
in a patient who is subsequently is found to have Crohns disease,
can lead to failure of the operation and permanent ileostomy. In
most instances, colonoscopy and multiple biopsies can differentiate Crohns disease from ulcerative colitis. When necessary, small
bowel study may also be helpful to rule out the presence of small
bowel disease. If considerable doubt exists, a subtotal colectomy
may be performed with and end ileostomy and rectal preservation. The final decision about the next operative procedure can be
made after the colon is examined by pathology and a diagnosis is
made. If the colon shows ulcerative colitis then a restorative proctocolectomy can be performed. If the diagnosis is Crohns disease
and the rectal segment is spared, an ileorectal anastomosis is performed instead. Some patients may opt to keep the ileostomy and
undergo completion proctectomy at a later date. A major problem
184

PMPH_CH19.indd 184

5/21/2012 8:56:32 PM

Crohns Disease of the Small Bowel

may occur when the colitis is diagnosed as ulcerative colitis and


the patient undergoes restorative proctocolectomy and years later
he/she develops severe pouchitis with intractable stricture of the
inflow tract, recurrent pelvic abscesses or severe perianal disease
with multiple fistulas and the diagnosis of Crohns disease becomes
obvious, yet a review of the colonic pathology still shows ulcerative
colitis. A significant percentage of these patients will require pouch
excision and permanent ileostomy.
The need for permanent diversion versus temporary diversion
in Crohns disease differs depending on the extent and location of
the disease and the difficulty of the operative procedure being performed. Patients with intractable perianal disease, rectal inflammation with strictures and rectal cancer will need proctectomy
with a permanent stoma, most likely an ileostomy. Temporary
stomas are used when the surgeon feels that multiple anastomosis
need protection or when multiple enterotomies in a difficult case
require proximal diversion to avoid extensive bowel resection.
These stomas are usually loop ileostomies although occasionally
may be proximal jejunostomies and then the patient will require
supplementation with parenteral nutrition.
Regarding the question about the amount of bowel resection,
there are several factors that must be considered. The amount of
residual small bowel and colon after surgical resection is of major
importance. If the colon is normal, about 4-5 of normal small
intestine constitutes the minimum needed for adequate nutrition.
If the patient has an ileostomy, a longer length, usually over 8 is
necessary. Strictureplasty may be used to preserve bowel length, but
it cannot be used in fistulizing disease where a resection is necessary. As far as resection margins and anastomosis are concerned,
grossly visible disease should be resected if a resection is planned,
because anastomosis between grossly diseased bowel may result in
a leak with life threatening consequences. In general, it is better to
take a slightly longer segment of bowel and perform a safe anastomosis than to compromise the surgical outcome to save a few inches
of diseased small bowel. The more complicated the operation, with
multiple sites of recurrent disease, the more likely it is that there will
be microscopic inflammation in the resected margins.
Recurrence after surgical resection for Crohns disease remains
a major problem.3 Postoperative prophylaxis is rapidly changing
from antiinflammatory drugs such as 6-mercaptopurine to biological immunomodulators such as infliximab, especially in high-risk
patients with extensive disease, residual disease after surgery, or
previous early postoperative recurrence. The postoperative period
offers an ideal opportunity to control recurrence of the disease
because in the majority of patients all visible disease is removed and
this may be the only opportunity to treat normal appearing bowel
with immunomodulators that may be able to delay the reactivation
of the immune process that causes recurrent disease.

PMPH_CH19.indd 185

185

The problem of carcinoma in Crohns disease occurs with some


frequency and must always be kept in mind when treating these
patients. Small bowel tumors usually present as a site of obstruction, especially in patients with longstanding disease.4 These tumors
may be difficult to distinguish from benign strictures and often
only biopsy or resection reveals the true nature of the obstructing
lesion. Another group at high-risk to develop anorectal cancer is
patients with perianal disease that require subtotal colectomy and
ileostomy. As the perianal disease becomes quiescent after diversion, many patients delay the completion proctectomy. This is inadvisable because development of an anal squamous cell carcinoma or
rectal adenocarcinoma often does not produce any new symptoms
until the lesion is advanced and then the outcome is poor. Except
in rare instances, we always recommend completion proctectomy
within a year of colectomy if restoration of intestinal continuity is
not contemplated.
In summary, recent advances in medical treatment have had
some beneficial effects mainly in decreasing the rate of proctectomy for patients with perianal disease. Patients with significant
strictures causing obstruction and patients with abscesses and
enteric fistulas still require surgical resection or strictureplasty
as the biological drugs are unable to control severe advanced
disease. Hopefully, as we continue to learn to understand the
mechanism of immunomodulation better and perhaps learn
the etiology of Crohns disease, we will be able to treat this
disease earlier and diminish the need for repeated surgical
intervention.

REFERENCES
1. Kurtz MP, Svensson E, Heimann TM. Use of platelet-derived
growth factor for delayed perineal wound healing in patients with
inflammatory bowel disease: A case series. Ostomy Wound Manag.
2011;57:24-31.
2. Heimann TM, Greenstein AJ, Lewis B, Kaufman D, Heimann DM,
Aufses AH, Jr. Comparison of primary and reoperative surgery in
patients with Crohns disease. Ann Surg. 1998;227:492-495.
3. Heimann TM, Greenstein AJ, Lewis B, Kaufman D, Heimann
DM, Aufses AH, Jr. Prediction of early symptomatic recurrence
after intestinal resection in Crohns disease. Ann Surg. 1993;218:
294-298; discussion 298-299.
4. Ribeiro MB, Greenstein AJ, Heimann TM, Aufses AH, Jr. Adenocarcinoma of the small intestine in Crohns disease. Surg Gynecol
Obstet. 1991;173:343-349.
5. Ribeiro MB, Greenstein AJ, Sachar DB, Barth J, Balasubramanian S,
Harpaz N, Heimann TM, Aufses AH, Jr. Colorectal adenocarcinoma in Crohns disease. Ann Surg. 1996;223:186-193.

5/21/2012 8:56:32 PM

CHAPTER 20

Small Bowel Tumors and


Diverticular Disease of SB
John D. Cunningham and Akintunde Akinleye

INTRODUCTION

classified as gastrointestinal stromal tumors (GIST). It had previously been believed that mesenchymal gastrointestinal tumors
were of smooth muscle origin but with the advent of immunohistochemical staining and electron microscopy that was shown not to
be the case. Most evidence now suggests that most sarcomas actually originate from the interstitial cells of Cajal and small bowel
sarcomas are now referred to as GIST. The use of the term GIST
became widespread in the late 1990s and is currently the appropriate nomenclature for mesenchymal tumors of the small bowel.

There are approximately 7000 small bowel malignant tumors


diagnosed each year in the United States.1 The small number of
cases per year allows for little information related to their presentation, natural history, surgical treatment, adjuvant therapy, and
outcomes. There are currently no randomized studies concerning surgical management of small bowel tumors because of their
vague presentation, difficult diagnostic workup, and due to the
fact that diagnosis is often made at the time of laparotomy. There
are some trials regarding the use of adjuvant therapy for small
bowel tumors but these trials contain few patients, often are not
randomized, and contain patients with similar tumor histology in
other gastrointestinal sites. This chapter has been written based
on retrospective reviews from major medical centers and from
national cancer databases. This is the best data that is currently
available. The likelihood of there being randomized trials regarding treatment of small bowel tumors that have enough patients
to reach statistical significance is unlikely. The evidence-based
medicine for this chapter is C level at best.
Although the small bowel mucosa accounts for over 70% of
the length and over 90% of the surface area of the intestine, only
1% to 3% of all gastrointestinal tumors arise there.2,3 A number
of reasons have been given for this finding. Small bowel contents
have a rapid transit time through the small bowel and the mucosa
is not exposed to possible carcinogens for long periods of time
and the liquid nature of the contents that may be less irritating to
the small bowel. Other reasons that prevent the transformation of
small bowel mucosa to malignancy include the decreased bacterial count in the small bowel compared to the colon, increased
amount of lymphoid tissue, alkaline pH, and the presence of the
enzyme benzyprene hydoxylase that helps to detoxify potential
carcinogens. The true explanation most likely is a result of many
of these factors.

ADENOCARCINOMA
Adenocarcinomas of the small bowel are most commonly found
in males in their sixth decade of life.2-6 Over half of the lesions are
found in the duodenum while a quarter is found in the jejunum
and the rest in the ileum.2-7 The most common presenting symptoms are pain, nausea and vomiting, and bleeding.2-5

CARCINOID
Carcinoids arise from the Kulchitsky cells in the crypts of Lieberkhn. Men are slightly more likely to get carcinoids than
women.8,9 The median age of onset is 66 years.8 The most common
site for carcinoid tumors is in the ileum.2,3,7-9 The most common
presenting symptom is abdominal pain while nausea, vomiting,
and bleeding are less common complaints.3,7,8 The carcinoid syndrome is found in patients whose disease has spread to the liver.

LYMPHOMA
The gastrointestinal tract is the most common site of extranodal lymphoma. Men are more likely to have lymphoma than
women.2,10 Patients present at a median age of 62 years.10 The jejunum and ileum account for the vast majority of lymphomas.2,3,7,10
The most common presenting symptoms in order are pain, fever,
weight loss, and anemia.3,10

1. What are the most common types of small bowel cancer?


The most common types of small bowel cancer are carcinoid,
adenocarcinoma, lymphoma, and sarcoma. Sarcomas are now
186

PMPH_CH20.indd 186

5/21/2012 8:57:15 PM

Small Bowel Tumors and Diverticular Disease of SB

SARCOMA/GIST
Men are more likely to be diagnosed with sarcomas/GIST than
women.11-13 The majority of these tumors are found in the jejunum
and ileum and the remainder in the duodenum.2,3,7,11,13 Abdominal pain, an abdominal mass, gastrointestinal bleeding, fever, and
weight loss are the most common presenting symptoms.3,13
2. Has the incidence, histology, or location of small bowel
tumors changed over the last 20 years?
A recent review of the National Cancer Data Base (NCDB) from
1985 to 2005, the SEER database from 1973 to 2004, and the Connecticut Tumor Registry from 1980 to 2000 has shown that there has
been changes in the incidence of small bowel tumors over the last 30
years.2,7 The incidence has risen from 11.8 to 22.7 cases per million
persons from 1973 to 2004.2 The Connecticut tumor registry showed
an increase from 10.5 to 14.9 cases/100,000 over 20 years.7 There has
also been a significant change in the location of tumors in the small
bowel over the last 20 years. Duodenal tumors have increased dramatically while the number of jejunal and ileal tumors has decreased.2

ADENOCARCINOMA
The incidence of adenocarcinomas increased from 5.7 to 7.3 cases per
million from 1973 to 2004.2 This is an annual increase of 1.3% and
an overall increase of 26%.2 The proportion of small bowel tumors
that are adenocarcinomas decreased from 42.1% to 32.6% from 1985
to 2005.1 There were no changes in patient or tumor characteristics
during this time period.2 Adenocarcinomas were more frequently
found in the duodenum over this same time period while there was
a concomitant decrease in jejunal and ileal adenocarcinomas.1,2

CARCINOID
Carcinoid tumors increased in incidence dramatically from 2.1 to
9.3 cases per million over the same time period.2 The annual incidence increased by 3.6% per year while the overall percent change
was 341% from 1973 to 2004.2 This increased incidence accounts
for carcinoids becoming the most common type of small bowel
tumor with an increase in proportion from 27.5% to 44.3%.2 There
was an increase in the number of carcinoids found in the duodenum also. From 1973 to 2005, the rate of duodenal carcinoids
increased from 10.9% to 22.3%.2

LYMPHOMA
The rate of increase of lymphomas averaged 2.3% per year and
totaled 94.3%.2 This calculates to a rate of 4.4 cases per million up
from 2.2 cases per million.2 The proportion of small bowel cancers
being lymphomas did not change over 30 years of study.2 Lymphomas also showed a change in location with more tumors being
found in the duodenum and less in the jejunum and ileum.2

SARCOMA/GIST
The incidence and proportion of small bowel tumors that are
sarcomas/GIST did not change over the last 30 years.2 However,

PMPH_CH20.indd 187

187

there was a slight increase in incidence of duodenal sarcoma/GIST


tumors over the same time period.2
3. What is the most sensitive test for diagnosing small bowel
tumors?
Small bowel tumors usually present with vague, nonspecific symptoms that could be attributed to a large number of other possible
diagnoses. This accounts for a delay in the diagnosis of most tumors
and the late stage seen with small bowel tumors at the time of diagnosis. The nonspecific complaints lead to the ordering of multiple
tests to try to make the diagnosis. Most tests are ordered based on the
presenting symptoms. The most common symptoms that patients
present with are abdominal pain, nausea and vomiting, bleeding,
anemia, obstruction, and an acute abdomen. Imaging of the small
intestine is the most difficult of all the areas of the gastrointestinal
tract to evaluate. As one might have guessed the use of diagnostic
imaging tests have changed dramatically over the last 30 years. The
last 20 years has seen a significant increase in the use of computed
tomography (CT) and endoscopy. Prior to this, an upper gastrointestinal study with a small bowel follow through (UGISBFT) was
the most common test to evaluate the nonspecific symptoms associated with small bowel tumors.3,14

UGISBFT
This test requires the patient to swallow barium and the barium
is followed through the small bowel with serial abdominal X-rays.
The sensitivity of the test is limited because the images are not
obtained continuously and overlapping bowel loops can obscure
small mucosal defects. This test found direct evidence of small
bowel tumors in less than 30% of cases.14 This test is used infrequently today because of its poor accuracy and the advent of upper
endoscopy and CT imaging.

UPPER ENDOSCOPY
The passage of an endoscope through the stomach into the duodenum is a very accurate test for finding lesions in the duodenum up
to the ligament of Treitz. The advantage of this technique is that it
visualizes the lesion and allows one to biopsy the lesion to make
the diagnosis. Many centers today use endoscopic ultrasound to
evaluate tumor depth of penetration and to evaluate nodal disease
for patients with duodenal, pancreatic, and gastric lesions.
Colonoscopy can be used in some instances to visualize the distal
10 to 60 cm of the ileum. This is often patient and user dependent.

ENTEROCLYSIS
If one suspects a small bowel tumor in the jejunum or ileum, enteroclysis can be performed; and this has replaced UGISBFT.14 Enteroclysis is a double contrast study of the small bowel that requires the
placement of a tube into the proximal small bowel. The patient is
then injected with methylcellulose and contrast to image the small
bowel. This test is useful to delineate mucosal defects associated
with jejunal and ileal tumors. Although this test is more accurate
than a UGISBFT, it requires a skilled radiologist to perform, there
is significant patient discomfort, it requires sedation, there is a

5/21/2012 8:57:15 PM

188

Surgery: Evidence-Based Practice

significant amount of radiation, and it is expensive and very time


consuming.14 Therefore, it is used infrequently.

COMPUTED TOMOGRAPHY (CT)


CT imaging for delineating small intraluminal lesions in the small
bowel is not very accurate. CT imaging for small bowel lesions is
more likely to identify lymph node metastases, mesenteric stranding, abdominal masses, and distant disease. However, due to the
vague presentation and nonspecific symptoms seen with small
bowel tumors, most patients will have a CT done during their
diagnostic evaluation. CT imaging may be performed following
the presumed diagnosis of a small bowel tumor by another diagnostic test to determine the extent of disease.

small bowel cancer as it has been for colonic cancer. The recommendations for resection include the primary tumor with grossly
clear margins and a wedge of mesentery to include lymph nodes
for evaluation. The exception to this rule is for sarcomas/GIST
that can be resected without a lymphadenectomy because these
tumors rarely spread to lymph nodes.

ADENOCARCINOMA
Surgical resection for cure for adenocarcinoma is possible in up to
67% of patients.2-5 This may reach as high as 80% for nonmetastatic
disease.2 Patients who present with duodenal lesions will need to
undergo pancreaticoduodenectomy. Jejunal and ileal resections
can be performed with wedge resections.

CAPSULE ENDOSCOPY
Capsule endoscopy has become a much more common diagnostic
test for patients with suspected small bowel pathology. The test
involves the ingestion of a small video camera in a capsule. It is
most commonly used for patients who have occult gastrointestinal
bleeding with repeatedly normal endoscopy and colonoscopy. It
can be used to visualize mucosal disease, angiodysplasias, ulcers,
polyps, and cancer. Its main advantage is that it can visualize the
entire small bowel. As it is usually performed for occult bleeding,
only few small bowel cancers are found with this test.14
Since the preoperative diagnosis of a small bowel tumor may
be made in up to 50% of cases, surgery appears to be the best diagnostic test for small bowel tumors.2-4,7 Surgery allows for evaluation
of the entire small bowel and it allows the surgeon to evaluate any
abnormal findings found on preoperative imaging. Although most
studies were done before the widespread use of diagnostic laparoscopy, this would seem to be the perfect test to evaluate patients
for a presumed small bowel pathology not found on preoperative
imaging. Surgery also allows the surgeon to therapeutically treat
the tumor. In a series of 217 small bowel adenocarcinomas, surgery
made the diagnosis in 30% of patients, CT 14%, UGISBFT 26%,
upper endoscopy 24%, physical exam 4%, and ultrasound 2%.4
4. What are the risk factors for developing small bowel cancer?
It is difficult to determine patient risks for small bowel tumors
due to the small number of cases. There appears to be a slight
sexual predilection with males being more likely to develop a
small bowel tumor than women.2,4,6,7,9-13 Age appears to be a factor
related to small bowel tumors with the median age of diagnosis
being between 55 and 65 years.2-13
Adenocarcinomas of the small bowel are associated with
Crohns disease and are usually found in the distal ileum.3 Familial polyposis has been associated with an increased risk of adenocarcinoma of the periampullary area and this area needs to be
followed closely in patients who have had their colon removed for
disease. Lymphoma has been associated with celiac sprue disease,
parasitic infections, and in the immuno-compromised patient.
Meckels diverticulum has been found to be a risk factor for the
development of carcinoid tumors in the ileum.15
5. What is the principal treatment for small bowel cancer?
Surgery is the mainstay of treatment for small bowel cancer. The
optimal surgical resection has not been as well delineated for

PMPH_CH20.indd 188

CARCINOID
The majority of carcinoids are located in the ileum. The surgical
procedure of choice is a wedge resection. One must evaluate the
rest of the gastrointestinal tract as carcinoids can present with synchronous lesions. Carcinoids can also have a desmoplastic reaction
in the mesentery that may make them difficult to resect. Resection
for cure can be accomplished in up to 84% of patients.2,9,10

LYMPHOMA
The most common site of small bowel lymphomas are in the
jejunum and the ileum and therefore the optimal treatment is a
wedge resection.2,3,10 Resection for cure can be achieved in over
two-thirds of patients.10 Surgical treatment of this disease has not
changed in the last 30 years.2

SARCOMA/GIST
Sarcomas/GIST are most commonly found in the duodenum and
jejunum and rarely spread to lymph nodes. Surgical resection
should include the primary tumor, but an extensive lymphadenectomy need not be performed. Avoidance of tumor rupture during
surgery is imperative to prevent tumor spread. The goal of surgery
is macroscopic negative margins with an intact pseudocapsule. The
primary tumor can be resected in up to 88% of cases.2,11,12 Although
a large number of patients can be resected, surgery is usually not
curative in the majority of patients.11-13,16
6. What is the role of adjuvant therapy in small bowel cancer?
A number of different chemotherapy regimens have been used
to treat small bowel cancer. For the most part, none have been
shown to be effective with the recent exception of the use of
imatinib for GIST. There is very little evidence-based medicine
regarding these drugs. There is no role for radiotherapy in the
treatment of small bowel cancer as the entire abdominal cavity
would need to be treated because of the mobility of the small
bowel. In addition, the small bowel is the most sensitive organ to
radiotherapy in the abdomen and complications would be very
common.

5/21/2012 8:57:15 PM

Small Bowel Tumors and Diverticular Disease of SB

ADENOCARCINOMA
Two-thirds of adenocarcinomas will present with nodal or distant
metastases.2-6 Patients with full thickness bowel involvement and
nodal disease are candidates for adjuvant therapy. If the distant
disease is resected, stage IV patients can also receive chemotherapy in an adjuvant setting. There has been an increase in the use
of adjuvant therapy for patients with adenocarcinoma from 8% to
22% over the last 20 years.2 Despite the use of adjuvant therapy, no
survival advantage has ever been demonstrated.2-6 The use of chemotherapy in a palliative setting for small bowel adenocarcinomas has resulted in a median survival of 8 months and no overall
improvement in survival.5

CARCINOID
Carcinoids tend to be diagnosed late with nodal involvement or
distant disease found in the majority of patients at the time of
presentation.2,3,9 The risk of nodal and distant disease increases
with the size of the primary tumor and multicentric disease is a
common finding. The use of chemotherapy in the adjuvant setting
has been shown to be of no benefit.2,3,9 There is no role for the use
of palliative chemotherapy if surgical resection can be performed
for metastatic disease.2,3,9 Octreotide has been used in the palliative setting to control symptoms of the carcinoid syndrome. There
has been some data to suggest it may keep disease from progressing for short periods of time.

LYMPHOMA
Lymphoma is a very chemo-sensitive type of tumor. Unfortunately, lymphoma of the small bowel is usually diagnosed at the
time of laparotomy and the final pathology is not available for
several days precluding the use of chemotherapy as the primary
treatment. If the patients are rendered free of disease from surgery, most patients are not given chemotherapy.2,3 There has been
no change in the adjuvant treatment pattern of small bowel lymphoma over the past 20 years.2 Patients with metastatic disease are
all treated with chemotherapy.

SARCOMA/GIST
With the discovery that most sarcomas of the small bowel actually
arise from the interstitial cells of Cajal and not smooth muscle
cells, the role of adjuvant chemotherapy has changed dramatically.
The use of adjuvant therapy for sarcomas did not alter survival.2,3,11
However, the discovery that GIST has a mutation in the c-KIT
oncogene resulting in the activation of the KIT receptor tyrosine
kinase has made them susceptible to imatinib. Imatinib exerts its
influence in GIST by blocking the adenosine triphosphate binding site of KIT to the transmembrane receptor. Thereby, imatinib
inhibits the proliferation and promotes apoptosis of GIST cells by
interrupting the tyrosine kinase mediated intracellular signaling.
The use of chemotherapy for GIST has increased dramatically
from 2001 to 2005 because of this finding.2
The use of imatinib started in patients with metastatic disease
and the majority of patients were shown to have rapid and sustained

PMPH_CH20.indd 189

189

responses. These findings coupled with the high rate of recurrence


after curative resection has led to the use of imatinib in an adjuvant setting for patients with GIST. The ACOSOG Z9001 trial used
imatinib for patients with resected tumors larger than 3 cm for
1 year versus placebo.17 The study was stopped after 1 year due to
the recurrence free survival advantage seen in the imatinib arm.17
There was no difference in overall survival and it was thought that
the use of imatinib in this setting only delayed the time to recurrence.17 One has to ask the question, would it be better to wait until
the recurrence occurs and then treat the patient?
GIST often presents with advanced disease and the use of
imatinib in a neoadjuvant setting has been studied. The RTOG
0312 trial treated patients with unresectable disease for 8 to 12
weeks and found that over 90% had an objective response and
most of them went on to be resected for cure.18
Patients with GIST that have metastatic disease to the liver or
peritoneum are treated with imatinib until progression of disease.
There is now a second line drug, sunitinib, for use in patients with
refractory GIST to imatinib.
The low toxicity profile of imatinib along with its oral dosing
and significant efficacy has altered the natural history of this disease. Currently, there is not enough data to accurately address the
correct dosing, timing of treatment, and length of treatment for
patients receiving imatinib to define the subset(s) of patients who
will most likely benefit from treatment.
7. What are the prognostic factors for survival for small bowel
tumors?
The two most important prognostic factors for survival in small
bowel cancer are tumor stage at the time of presentation and the
ability to completely resect the tumor.2-6,9-13

ADENOCARCINOMA
The median survival for patients with small bowel adenocarcinoma is less than 20 months.2-5 The 5-year survival for all comers is about 30%.2-5 Improvements in survival have been seen in
patients with early stage disease, complete surgical resections, and
duodenal lesions.2-5 Poor prognostic findings include age greater
than 55 years, ileal or jejunal location, Afro-American descent,
poorly differentiated tumors, and microscopic positive margins.2-5
There has been no change in survival over the past 20 years for
patients with adenocarcinoma of the small bowel.2

CARCINOID
Carcinoid tumors have the longest survival of all small bowel
tumors. The median survival for all patients with carcinoid
tumors is greater than 10 years.2,3,8,9 Stage was the most important
predictor of survival in this group.2,3,8,9 Other favorable prognostic
indicators included tumors less than 1 cm, incidentally found
lesions, duodenal location, age less than 50 years, and female
sex.8,9 There had been no change in survival for patients with carcinoid over the past 20 years.2 The majority of patients with carcinoid tumors eventually die from progression of disease. This fact
coupled with the increasing incidence of carcinoid tumors warrant the need for the development of adjuvant trials to treat this
disease.

5/21/2012 8:57:15 PM

190

Surgery: Evidence-Based Practice

LYMPHOMA
The 5-year survival for patients with small bowel lymphoma is
approximately 50%.2 There has been no change in survival over
the past 20 years.2 Survival is based on stage of disease and resection for cure.2,3,10 Poor prognostic findings include male sex, age
greater than 75 years, and non-Caucasian ethnicity.2,3,10

SARCOMA/GIST
The median survival for patients with sarcomas prior to the new
nomenclature and treatment with imatinib was up to 34 months
with a 5-year survival of 39%.2,3,11 Survival was improved for
patients with smaller tumors, less mitotic activity, complete resections, and age less than 75 years.2,3,11 Despite the increased use of
imatinib for GIST, the median survival is only 40% at 5 years.19
The long-term effects of imatinib on the survival of GIST patients
may not be seen for several more years as the use of the drug has
only recently become more widespread.
Tumor size less than 10 cm, gross residual disease after resection, high mitotic activity, and male sex have also been implicated
as poor prognostic findings for survival in GIST patients.11,12,16
8. What is the optimal treatment for small bowel tumor
recurrence?
The most common location for recurrence of small bowel cancer
is the abdominal cavity followed by the liver and omentum. The
majority of patients are not resectable at the time of recurrence
and palliative surgery, chemotherapy, or pain management are the
main treatment options.2-6,9-13,19

ADENOCARCINOMA
The most common site of recurrence is in the peritoneum or omentum. Most patients will develop liver metastases also. If the recurrence is isolated which it rarely is, surgical resection would be the
treatment of choice. There is no surgical approach for widely metastatic adenocarcinoma and these patients are treated with palliative
intent. The survival following recurrence is less than 6 months.5

CARCINOID
The most common site of recurrence for carcinoid tumors is the
liver. Intraabdominal spread is also common. There is a role for
debulking disease for carcinoid tumors because there is no effective chemotherapy, and the tumors often become symptomatic
before they are life threatening.19 Liver metastases that cause carcinoid syndrome can be treated with surgery or octreotide. Both
modalities have been shown to reduce symptoms, but disease
progression is the rule. Complete surgical resection of disease can
improve long-term outcomes and surgery is also very important
in relieving symptoms from recurrent disease.19

LYMPHOMA
Recurrence following surgical resection for lymphoma occurs in
up to one-third of patients.3 Treatment is chemotherapy and may

PMPH_CH20.indd 190

include radiation therapy in certain instances. Gastrointestinal


complications during treatment of intestinal lymphomas are well
documented and must be watched for.

SARCOMA/GIST
Recurrence following curative resection is the rule rather than the
exception for sarcomas/GIST.2,3,11-13 The most common locations
for recurrence are the peritoneum, omentum, and the liver.2,11-13
If the patient has resectable disease, the patient should undergo
resection.11-13 Survival following complete surgical excision of a
recurrence can be as good as for an initial resection for cure.11-13,16
Five-year survivals are up to 40% following a second complete
resection.12 Patients with unresectable disease should be treated
with imatinib and patients with rapid and sustained responses may
become candidates for exploration.12,16

SMALL BOWEL DIVERTICULAR DISEASE


There are two main types of small bowel diverticular disease. There
are true diverticula of the small bowel and the most common of
these is a Meckels diverticulum. False diverticula of the small
bowel can happen throughout the small intestine and they resemble
their colonic counterparts. The mean age at the time of diagnosis is
68 years and women are more likely to be diagnosed than males.20
The incidence of small bowel diverticulosis is estimated to be less
than 1% with the most common location being in the duodenum.20
This most likely represents an increased incidence due to the widespread use of upper endoscopy. The most common explanation for
the development of small bowel false diverticula is an underlying
abnormality in intestinal peristalsis.20
The vast majority of diverticula are asymptomatic.20 Complications from diverticula include pain, rupture, abscess formation,
and peritonitis; and occur in 10% to 20% of patients.20 Surgical
treatment is usually required for patients who present with symptomatic disease.20
The majority of small bowel diverticula are diagnosed during upper endoscopy or laparotomy for another indication.20 Over
three-quarters of the diverticula are found in the duodenum while
the rest are dispersed between the jejunum and the ileum.20 There
is no role for prophylactic surgical resection of the false diverticula because they are often multiple and they are found throughout
the length of the small bowel.20
9. Is there a role for surgery to remove a Meckels diverticulum
found incidentally at the time of surgery for another reason?
Meckels diverticula are usually remembered by the rule of twos.
They are found within 2 ft of the ileocecal valve, they are 2-in.
long, they have two possible types of mucosa, and they are too bad
if you have one. Symptoms can include inflammation that mimics
appendicitis, peritonitis from rupture, bleeding, obstruction, and
abscess formation.15 These complications are usually corrected
surgically because the diagnosis is rarely made preoperatively.15
Meckels diverticula are usually asymptomatic and are found
most commonly at the time of laparotomy for another indication.
A recent study15 recommends that incidental Meckels diverticula
should be removed to prevent possible complications and to decrease
the risk of developing cancer in the Meckels diverticula.15

5/21/2012 8:57:15 PM

Small Bowel Tumors and Diverticular Disease of SB

191

Summary Table
Adenocarcinoma

Carcinoid

Lymphoma

Sarcoma/GIST

Mean age (years)

65

51

62

59

Symptoms

Pain, bleeding,
nausea, vomiting,
obstruction

Pain, bleeding, nausea,


vomiting

Pain, fever, weight loss,

Bleeding, pain

Location

Duodenum

Distal ileum

Jej-ileal

Jejunal

Diagnostic Test

Upper endoscopy

Laparotomy, CT

Laparotomy

CT, capsule endoscopy

Med survival (months)

13

182

13

22

Overall survival
(months)

23

Not reached

15

66

Curative resection (%)

65

73

67

85

Poor prognosis

Advanced stage, age


> 55 years, ileal
location

Size, liver metastases,


age < 50 years,
jejunal location

Age > 75 years,


nonwhites

Tumor size, positive


margins

Adjuvant therapy

No benefit

No benefit

No benefit

Beneficial, imatinib

Time to recurrence
(months)

23

25

14

25

Sites of recurrence

Peritoneum, liver,
omentum

Mesentery, liver

Peritoneum, bone
marrow

Abdominal, omentum,
liver

Surgery

REFERENCES
1. Jemel A, Siegel R, Xu J, Ward E. Cancer Statistics 2010. CA Cancer J Clin. 2010;60:277-300.
2. Bilimora KY, Bentrem DJ, Wayne JD, et al. Small bowel cancer in
the United States: Changes in epidemiology, treatment and survival over the last 20 years. Ann Surg. 2009;249:63-71.
3. Cunningham JD, Aleali R, Brower ST, et al. Malignant small
bowel neoplasms: Histopathologic determinants of recurrence
and survival. Ann Surg. 1997;225:300-306.
4. Dabaja BS, Suki D, Pro B, et al. Adenocarcinoma of the small
bowel: Presentation, prognostic factors, and outcome in 217
patients. Cancer. 2004;101:518-526.
5. Hong SH, Koh YH, Rho SY, et al. Primary adenocarcinoma of
the small intestine: presentation prognostic factors and clinical
outcome. Jpn J Clin Oncol. 2008;39:54-61.
6. Nicholl MB, Ahuja V, Conway C, et al. Small bowel adenocarcinoma: Under staged and undertreated? Ann Surg Oncol.
2010;17:2728-2732.
7. Hatzarras I, Plaesty JA, Abir F, et al. Small-bowel tumors: epidemiologic and clinical characteristics of 1260 cases from the Connecticut tumor registry. Arch Surg. 2007;142:229-235.
8. Landry CS, Brock G, Scoggins CR, et al. A proposed system
for small bowel carcinoid tumors based on an analysis of 6380
patients. Amer J Surg. 2008;196:896-903.
9. Shebani KO, Souba WW, Finkelstein DM, et al. Prognosis and
survival in patients with gastrointestinal tract carcinoid tumors.
Ann Surg. 1999;229:815-823.
10. Fischbach W, Kestel W, Kirchner T, et al. Malignant lymphomas of the upper gastrointestinal tract. Cancer 1992;70:10751080.

PMPH_CH20.indd 191

11. Howe JR, Karnell LH, Scott-Conner C. Small bowel sarcoma:


Analysis of survival from the National Cancer Data Base. Ann
Surg Onco. 2001;8:496-508.
12. Dematteo RP, Lewis JJ, Leung D, et al. Two hundred gastrointestinal stromal tumors: Recurrence patterns and prognostic factors for survival. Ann Surg. 2000;231:51-58.
13. Crosby JA, Catton CN, Davis A, et al. Malignant gastrointestinal
stromal tumors of the small intestine: A review of 50 cases from
a prospective database. Ann Surg Oncol. 2001;8:50-59.
14. Hara AK, Leighton JA, Sharma VK, et al. Imaging of small bowel
disease: Comparison of capsule endoscopy, standard endoscopy,
barium examination and CT. Radiographics. 2005;25:697-711.
15. Thirunavukarasu P, Sathaiah M, Sukumar S, et al. Meckels
diverticulum: A high risk region for malignancy in the ileum.
Ann Surg. 2011;253:223-230.
16. Eisenberg BL, Judson I. Surgery and imatinib in the management of GIST: Emerging approaches to adjuvant and neoadjuvant therapy. Ann Surg Oncol. 2004;11:465-475.
17. DeMatteo R, Owzar K, Maki R, et al. Adjuvant imatinib mesylate increases recurrence free survival (RFS) in patients with
completely localized primary gastrointestinal stromal tumor
(GIST): North American Intergroup Phase III trial ACOSOG
Z9001. Proc Am Soc Clin Oncol. 2007:Abstract 10079.
18. DeMatteo RP, Maki RG, Singer S, et al. Results of tyrosine kinase
inhibitor therapy followed by surgical resection for metastatic
gastrointestinal stromal tumor. Ann Surg. 2007;245:347-352.
19. Bourdeaux JP, Putty B, Frey DJ, et al. Surgical treatment of
advanced-stage carcinoid tumors. Lessons learned. Ann Surg.
2005;241:839-846.
20. Akhrass R, Yaffe MB, Fischer C, et al. Small bowel diverticulosis:
perceptions and reality. J Amer Coll Surg. 1996;184:383-388.

5/21/2012 8:57:15 PM

CHAPTER 21

Enterocutaneous Fistula
Peter A. Learn

Enterocutaneous fistula (ECF) remains one of the most challenging surgical problems in alimentary tract surgery with continued
high rates of associated morbidity and mortality. The difficulty
of fistula management can be compounded by the interplay of
the underlying pathology, ongoing sepsis, and the physiologic
derangements caused by the fistula itself. The heterogeneity of
the etiology, location, and behavior of ECF further complicates
the study of interventions for ECF. The treatment of patients with
ECF remains in large part guided by fundamental principles of
resuscitation, control of sepsis, aggressive wound care and control of effluent, nutritional support, and appropriately timed
surgical intervention. Although many of these principles of management still rely heavily on historical experience and expert
opinion, adequate data exists on some aspects of care to provide
an evidence-based framework for ECF management. This chapter will specifically address the nonacute management of external intestinal fistulas. Studies limited to pancreatic, anorectal, or
inflammatory bowel disease (IBD) fistulas are excluded.

malnutrition, intestinal failure resulting from fistula diversion,


fistula-related sepsis, and persisting underlying pathology. Ensuring adequate nutritional support is a fundamental principal in
fistula management, and both enteral and parenteral approaches
have been evaluated to varying degrees.
Total parenteral nutrition (TPN) reached widespread availability in the 1970s and was quickly applied to the nutritional support of patients with ECFs. The best data available supporting its
use are obtained from prospective cohort studies performed in
that decade using historical controls (Level 4 evidence). In these
early studies, patients receiving TPN had approximately twice the
ECF closure rate and at least half to one-quarter the mortality.2,3
While significant methodological flaws make the exact degree of
improvement attributable strictly to TPN difficult to ascertain, the
clear benefits of TPN in modern management preclude the ability
to subject the intervention to placebo-controlled study.
Enteral nutrition has also been advocated as a viable means
of supporting patients with ECF, and published case series reports
have demonstrated the ability to meet nutritional requirements
with various forms of enteral nutrition in select populations (Level
4 evidence).4,5 Additionally recognized benefits of enteral nutrition
are lower costs compared to TPN. However, a trial comparing enteral
versus parenteral nutrition strategies has not been performed.
Answer: TPN can be used to provide complete nutritional support to patients with ECF and potentially increase rates of fistula
closure (Level 4 evidence, Grade C recommendation). Enteral nutrition is attractive for its potential benefits over long-term TPN and
deserves further evaluation against TPN in controlled studies. As
enteral nutrition has been observed to meet the nutrition needs of
some patients, it can be considered for use on a selective basis, in the
setting of a clinical study, or as an adjunct to TPN; it remains unclear
if rates of fistula closure or time to closure are positively affected relative to TPN alone (Level 4 evidence, Grade C recommendation).

1. What factors predict mortality in ECF?


While many studies on ECF report mortality and associated factors,
almost none have rigorously evaluated models to predict mortality.
One exception is a study by Altomare et al. in which a predictive
model was derived from logistic regression analysis on a retrospective cohort of 70 consecutive patients from 1981 to 1986.1 The model,
incorporating only the APACHE II score and serum albumin at
time of diagnosis was then prospectively evaluated in a subsequent
17 patients and demonstrated an accuracy of 94% in predicting mortality (Level 1b evidence).
Answer: A reasonably accurate estimation of mortality can be
calculated using a model incorporating APACHE II score and serum
albumin, although the model would benefit from further validation
in other cohorts (Level 1b evidence, Grade B recommendation).
2. What is the optimal nutritional strategy in ECF?

3. What is the role of somatostatin analogs in the management


of ECF?

The significant nutritional demands of ECF are well-recognized


and result from the interplay of several factors including premorbid

Early case series reporting the use of somatostatin and its analogs
in the treatment of ECF fostered optimism about the potential for
192

PMPH_CH21.indd 192

5/21/2012 8:58:35 PM

Enterocutaneous Fistula

this therapy and ultimately led to the reporting of several clinical


trials, making this question the most rigorously studied aspect of
ECF management. The ability to combine the results of the trials,
however, is limited by their usage of different analogs and doses
for different lengths of time in heterogeneous populations.
The most recently published of these trials evaluated the use
of the long-acting somatostatin analog, lanreotide PR (prolonged
release), in both pancreatic and ECF.6 In this multicenter, randomized, double-blind, placebo-controlled trial, patients were
included based on clear definitions for pancreatic and EC fistulae.
The primary endpoint was reduction in fistula output by 50% at
72 h after the first injection. Nonresponders at this first assessment
were unblinded and permitted to cross over. Secondary analyses
included time to fistula closure. The study met its power requirements after enrolling 111 patients, among whom there were 18
duodenal and 18 nonduodenal small bowel ECFs. Overall, 64.8%
of patients receiving lanreotide met the primary endpoint compared to only 37.7% of patients on placebo (OR 3.07, p = 0.006),
a trend that was preserved in the subset of patients with ECF. A
shorter time to fistula closure was suggested in the analysis of
patients who remained on lanreotide in the blinded group, but
the study was not powered to evaluate this outcome. Although
the study makes a strong argument for the ability of lanreotide to
decrease fistula output (Level 1b evidence), conclusions regarding
time to closure are limited by the trial design to educated speculation and extrapolation.
A smaller but well-designed study by Sancho et al. reported
the results of a randomized, double-blind, placebo-controlled
trial of octreotide therapy in the early treatment of postoperative
ECF (Level 1b evidence).7 The study population included patients
with gastric, intestinal, and a small number of pancreatic fistulae
diagnosed within 8 days of enrollment, stratified into two groups
by daily volume of output and regardless of presence of ongoing
sepsis. Patients received standardized support therapy, including TPN and the trial treatment: either subcutaneous octreotide
100 mcg three times a day (14 patients) or saline placebo administered identically (17 patients). The trial treatment was administered either for 20 days of continuous treatment or until fistula
closure. Randomization scheme and postrandomization exclusion events were reported, and the trial was powered to detect a
decrease of fistula output of 50% or greater. In this trial, administration of octreotide failed to reduce fistula output better than
placebo. Spontaneous closure events occurred in each arm, but
neither the incidence nor the mean time to closure reached statistical significance, although the study was not powered to study
these endpoints.
These results contrasted somewhat with the same groups
prior results published in 1987.8 In this randomized, placebocontrolled, double-blind, crossover study (Level 1b evidence),
14 patients with postoperative small bowel ECF underwent
standardized treatment with TPN for 7 days. The patients then
received either octreotide (225300 mcg daily divided into three
doses) or saline placebo for 2 days. After 2 days, the groups were
crossed over to the other treatment. After the 4 days of the crossover portion of the trial, all patients were treated indefinitely with
octreotide to the point of fistula closure or surgical intervention.
While a power analysis and other important information were not
reported as in the later trial, octreotide in this trial clearly reduced
fistula output volume by at least half with a rebound in output
following interruption of octreotide therapy by placebo. The differences in the results of these two studies by the same group are

PMPH_CH21.indd 193

193

attributed by study authors to a possible effect of the timing of


octreotide administration after stabilization of the ECF, noting
that control of sepsis was not required in the later study.
Two further randomized-controlled trials have been published
on this specific topic. In 1992, Torres et al. published the results
of a multicenter, randomized (scheme unreported), single-blind,
placebo-controlled trial of continuous intravenous somatostatin
infusion (Level 2b evidence).9 The study results demonstrated no
difference in the frequency of fistula closure, although it appeared
that fistula output and time to closure was decreased in the somatostatin group. However, absence of information on the age of the
fistulas, crossover of three patients from the placebo arm to the
somatostatin arm, and intergroup differences in pretreatment fistula output complicates the interpretation of the results. In addition, the evaluation of time to closure would have benefited from
a more sophisticated event analysis technique (i.e., KaplanMeier
analysis). In 1993, Scott et al. published results of their randomized, double-blind, placebo-controlled trial of octreotide (Level 2b
evidence).10 As they did not observe significant reductions in fistula
losses or increased rates of spontaneous closure, a high dropout
rate and small sample size impair the interpretation of the results.
Answer: No evidence from controlled trials definitively supports the use of somatostatin analogs to increase the frequency of
spontaneous ECF closure (Level 1b evidence, Grade A recommendation). Somatostatin may decrease fistula output and time to closure but would benefit from further well-designed confirmatory
studies (Level 2b evidence, Grade B recommendation). Octreotide
has not been consistently demonstrated to decrease fistula output or time to closure and support for its use must be considered
limited (Conflicting results from Level 1b and 2b studies, Grade
D recommendation). Prolonged release lanreotide can be used to
reduce fistula output volume, but its effect on time to fistula closure or frequency of closure has not been definitively answered
(Level 1b evidence, Grade A recommendation).
4. When is the optimal timing for elective surgical intervention for ECF?
The subject of optimal timing for elective surgical intervention
has not been prospectively evaluated. In retrospective studies,
the data is mixed. In a cohort of 205 patients undergoing surgical
treatment of ECF, among whom more than half had IBD, Lynch
et al. noted higher rates of recurrence in patients undergoing surgery 2 to 12 weeks after their last operations (28% vs. 20% within
2 weeks and 15% greater than 12 weeks), although this finding
did not achieve statistical significance (Level 4 evidence).11 Somewhat contradictory results were observed by Brenner et al. in their
cohort of 135 patients undergoing surgery for ECF repair, among
whom a quarter had IBD. In this study, ECF recurrence was 12%
when operation was performed within 36 weeks of fistula formation compared to 36% for those undergoing operation after 36
weeks.12 Although the method by which this interval was identified is only partially addressed in the paper, this dichotomized
factor was independently associated with recurrence on multivariate logistic regression (Level 4 evidence).
When discussing the timing of surgery, it is also instructive to
consider the findings of the Gayrals lanreotide study: even in the
placebo treated arm, spontaneous fistula closure occurred within
17 days in half the patients (Level 1b evidence). Thus, planning for
surgical repair of ECF should account for the natural history of
the disease which favors spontaneous closure.

5/21/2012 8:58:36 PM

194

Surgery: Evidence-Based Practice

Answer: The optimal timing for fistula repair remains


unknown. As about half of fistulae will spontaneously close within
3 weeks, this seems the minimum reasonable period for observation (conflicting Level 4 evidence, Grade D recommendation).
5. Is surgical closure of the fistula best accomplished by
resectional or nonresectional approaches?
The choice between resectional (i.e., small bowel resection) and
nonresectional (i.e., oversewing or wedge repair) techniques
to address the originating bowel segment has not been studied prospectively. In the previously mentioned study by Lynch
et al., fistula recurrence occurred in 32.7% of patients treated
with nonresectional techniques compared with 18.4% of patients

undergoing segmental resection of the fistulous origin.11 The


association of nonresectional techniques with recurrence of the
fistula was significant on multivariate analysis (Level 4 evidence).
Similarly, the multivariate analysis by Brenner et al. for factors
associated with recurrence after surgery appeared to favor resection with a hand-sewn anastomosis over resection with a stapled
anastomosis or nonresectional repair (Level 4 evidence).12 While
these results have been interpreted to suggest a superiority of
resectional approaches, it is very easy to postulate on factors that
could have confounded recurrence and the inability to resect a
fistula.
Answer: Resection of the segment of bowel from which the
fistula originates might be preferable to nonresectional techniques
when feasible (Level 4 evidence, Grade C recommendation).

Clinical Question Summary


Question

Answer

1 What factors predict


mortality in ECF?

A model incorporating APACHE II


scoring and serum albumin is highly
accurate in predicting mortality.

1b

2 What is the optimal


nutritional strategy in
ECF?

TPN can not only provide complete


nutritional support to patients, but
may also increase rates of closure.

2, 3

Enteral nutrition should be considered


on a selective basis.

4, 5

Somatostatin analogs do not


significantly increase the frequency
of spontaneous ECF closure.

1b, 2b

6-10

Somatostatin may decrease both


fistula output and time to
spontaneous closure.

2b

Octreotide is inconsistently
demonstrated to decrease fistula
output.

1b, 2b

Prolonged release lanreotide can be


used to decrease fistula output.

1b

4 What is the optimal timing


for elective surgical
intervention for ECF?

Inadequate data exists to endorse a


strong recommendation on timing.

11, 12

5 Is surgical closure of the


fistula best accomplished
by resectional or
nonresectional
approaches?

Resection of the originating segment


of bowel may decrease likelihood of
recurrence.

11, 12

3 What is the role of


somatostatin analogs in
the management of ECF?

REFERENCES
1. Altomare DF, Serio G, Pannarale OC, Lupo L, Palasciano N. Prediction of mortality by logistic regression analysis in patients with
postoperative enterocutaneous fistulae. Br J Surg. 1990;77(4):450.
2. Deitel M. Nutritional management of external gastrointestinal fistulas. Can J Surg. 1976;19(6):505-509.

PMPH_CH21.indd 194

Grade of
Recommendation

Level of
Evidence

References

7, 8, 10

3. Thomas RJ. The response of patients with fistulas of the gastrointestinal tract to parenteral nutrition. Surg Gynecol Obstet.
1981;153(1):77-80.
4. Levy E, Frileux P, Cugnenc PH, Honiger J, Ollivier JM. High-output external fistulae of the small bowel: management with continuous enteral nutrition. Br J Surg. 1989;76(7):676.
5. Teubner A, Morrison K, Ravishankar HR, Anderson ID, Scott
NA, Carlson GL. Fistuloclysis can successfully replace parenteral

5/21/2012 8:58:36 PM

Enterocutaneous Fistula

feeding in the nutritional support of patients with enterocutaneous fistula. Br J Surg. 2004;91(5):625-631.
6. Gayral F, Campion JP, Regimbeau JM, Blumberg J, Maisonobe P,
Topart P, et al. Randomized, placebo-controlled, double-blind study
of the efficacy of lanreotide 30 mg PR in the treatment of pancreatic
and enterocutaneous fistulae. Ann Surg. 2009;250(6):872-877.
7. Sancho JJ, di Costanzo J, Nubiola P, Larrad A, Beguiristain A.
Randomized double-blind placebo-controlled trial of early octreotide in patients with postoperative enterocutaneous fistula.
Br J Surg. 1995;82(5):638.
8. Nubiola C, Nubiola-Calonge P, Badia JM, Sancho J, Gil MJ, Segura M.
Blind evaluation of the effect of octreotide (SMS 201-995), a somatostatin analogue, on small-bowel fistula output. Lancet. 1987;2(8560):672.

PMPH_CH21.indd 195

195

9. Torres AJ, Landa JI, Moreno-Azcoita M, Arguello JM, Silecchia


G. Somatostatin in the management of gastrointestinal fistulas.
A multicenter trial. Arch Surg. 1992;127(1):97.
10. Scott NA, Finnegan S, Irving MH. Octreotide and postoperative
enterocutaneous fistulae: A controlled prospective study. Acta
Gastroenterologica Belgica. 1993;56(3-4):266.
11. Lynch AC, Delaney CP, Senagore AJ, Connor JT, Remzi FH,
Fazio VW. Clinical outcome and factors predictive of recurrence
after enterocutaneous fistula surgery. Ann Surg. 2004;240(5):
825-831.
12. Brenner M, Clayton JL, Tillou A, Hiatt JR, Cryer HG. Risk factors for recurrence after repair of enterocutaneous fistula. Arch
Surg. 2009;144(6):500-505.

5/21/2012 8:58:36 PM

CHAPTER 22

Short Bowel Syndrome


Andrea MacNeill and S. Morad Hameed

INTRODUCTION

ever, patients with less than 100 cm of jejunum and no colon (or
less than 50 to 60 cm of jejunum with an intact colon) and those
with pathology in the residual small intestine (Crohns disease,
radiation enteritis, pseudo-obstruction) are at high risk of IF and
long-term PN dependence.1,2 Preservation of the ileocecal valve
can protect the small intestinal remnant by preventing reflux
of colonic material and resultant bacterial overgrowth. These
thresholds are important to consider intraoperatively, and may
influence decisions regarding resection versus stricturoplasty in
Crohns disease, or resection versus temporary closure, resuscitation and reevaluation in cases of borderline viability in the context of obstruction or ischemia.
Postoperatively, plasma levels of citrulline (a non-protein
amino acid produced in the intestinal mucosa, and a marker of
viable intestinal mass) have some prognostic value: levels less than
20 mmol/L have been shown to be highly predictive of permanent
IF.5 Interestingly, the postresection phase is far from static: intestinal adaptation, which has been the subject of intense scientific
investigation, may ultimately make the difference between lifelong dependence on PN and enteral autonomy.

The decision to undertake a massive intestinal resection is among


the most difficult and dreaded in surgery. Unfortunately, conditions such as Crohns disease, radiation enteritis, neoplasms (such
as desmoid tumors), and abdominal catastrophes (including mesenteric ischemia, closed loop obstruction or intestinal volvulus)
sometimes leave no other options.1 Massive resection can lead to
short bowel syndrome (SBS), a state of nutrient malabsorption,
or ultimately to intestinal failure (IF), a situation where adequate
nutrition and hydration cannot be maintained without nutritional
supplementation.2 These conditions, once considered uniformly
fatal, have tested the limits of medical and surgical innovation
and care, and have prompted exciting advances and promising
new strategies.
The emergence of parenteral nutrition (PN) in the 1960s, a
landmark event in surgical history,3 changed the course of IF, suddenly making it survivable in the short-term.3 Recently, intestinal rehabilitation efforts have sought to capitalize on improved
understanding of intestinal adaptation to achieve enteral autonomy. Advances in surgical reconstruction have expanded the
nontransplant armamentarium against SBS at the same time as
survival rates from intestinal transplantation (IT) are improving.

2. What is the optimal nutrition support strategy in SBS?

1. What is a massive resection and who is at risk of Intestinal


Failure?

INITIAL CONSIDERATIONS

Enteral nutrition (EN) is dependent on the highly integrated function of the stomach, pancreas, liver, small intestine, and colon.
Resection or dysfunction of any combination of these organs confers a unique IF risk. The small intestine is typically 800 cm long
and sees about 8 to 9 L of fluid per day.4 Macronutrients (proteins,
carbohydrates) are preferentially absorbed in the jejunum, while
the ileum is primarily responsible for the absorption of fluid, bile
salts, and vitamin B12. The colon absorbs residual water, and generates and absorbs short chain fatty acids. Together, the ileum and
colon act to slow intestinal transit through neurohumoral mechanisms (ileocolic brake), coordinated peristalsis, and the action of
the ileocecal valve.
In general, patients with 200 cm of healthy small intestine
and an intact ileocecal valve and colon have a high likelihood of
achieving nutritional independence after massive resection. How-

An aggressive approach to rehydration, correction of electrolyte


abnormalities, and provision of macro- and micronutrients must
be taken in intestinal resection patients considered to be at high
risk of IF. Fluid balance and serum electrolyte levels should initially be monitored every few hours, and replaced proactively. Diarrhea, which is often due to a combination of low-absorptive surface
area, gastric hypersecretion, and the osmotic effects of bile salts in
patients with ileal resections, can be addressed initially by limiting
oral intake and administering antidiarrheals such as loperamide
or codeine, proton pump inhibitors, or cholestyramine to bind bile
salts. Low-dose broad-spectrum antibiotics can be administered if
there is suspicion that small bowel bacterial overgrowth may be a
contributing factor, but this is more common later in the disease
process. Interestingly, in patients with less than 60 cm of small intestine, the initiation of enteral feeds can have a net secretory effect on
196

PMPH_CH22.indd 196

5/21/2012 8:59:13 PM

Short Bowel Syndrome

the gut and actually lead to increased fluid losses. The effects of oral
intake should be carefully monitored in these patients.

PN
For patients with less than 100 cm of jejunum remaining, the early
initiation of PN has been lifesaving. PN should be started within
a few days postoperatively and continued until EN can be ramped
up to 80% of goal. Priorities for PN administration include adequate provision of macronutrients (protein, carbohydrate, lipid),
replacement of electrolytes including magnesium (which is particularly prone to depletion), and replacement of micronutrients
such as zinc, which is often depleted. Patients with ileal resections
are prone to deficiencies of vitamin B12 and fat-soluble vitamins
(A, D, E, K). Levels should be monitored and vitamins should
be replaced accordingly. Transition to EN should be gradually
attempted in these patients as adaptation proceeds.
Patients with less than 60 cm of small intestine remaining
will require long-term or home PN. These patients are at high risk
of complications such as line sepsis, venous thrombosis, steatosis,
cholestasis, and hepatic cirrhosis. Among these complications,
end-stage liver disease is among the most feared and is a leading cause of PN-related mortality. Cholestasis may benefit from
reductions of PN dose or PN cycling. Although the mechanisms of
PN-associated liver disease (PNALD) are still unclear, vegetableor soy-based lipid emulsions may promote cholestasis through the
generation of inflammatory mediators. Recent studies have provided early evidence to suggest that omega 3 fatty acid preparations (omegaven) may reduce the effect of PNALD in children.6,7

TRANSITION TO EN
In animal studies, EN, has been shown to limit mucosal atrophy and
preserve surface area, promote endocrine function, and enhance
mucosal immunity.8 Besides these advantages, EN is more physiologic, safe, and cost-effective than PN. For these reasons, numerous experts have promoted a stepwise transition to EN over days
and weeks.2 Based on small trials, various authors have argued that
carbohydrate-to-fat ratios may determine diarrhea volumes: highcarbohydrate diet may favor diarrhea in patients with jejunostomy,
while reducing output in patients with intact colons.9,10
ANSWER: Early efforts in nutrition support in IF should focus
on parenteral administration of macronutrients and maintenance
of euvolemia and electrolyte balance. Pharmacologic interventions
may be needed to reduce GI losses. For patients with favorable prognosis for nutritional independence, a graded escalation of enteral
feeds with weaning from PN should be initiated. For patients with
less than 50 cm of small intestine, preparations should be made for
long-term PN. All patients should be monitored closely for vitamin
and micronutrient deficiencies, with special emphasis on vulnerabilities due to specific gastrointestinal anatomy (e.g., vitamin B12
deficiencies with ileal resection) (Grade D recommendation).
3. What pharmacologic strategies can promote intestinal
adaptation?
In the immediate postoperative phase, the intestinal remnant undergoes both structural and functional adaptation to compensate for
the loss of absorptive and digestive capacity. Whether or not these

PMPH_CH22.indd 197

197

measures will be sufficient to achieve enteral autonomy is dependent upon the length, location, and quality of remaining bowel, with
the presence of a colon of particular importance. The majority of
the adaptive response occurs within the first few months following
resection, but the process may continue beyond 2 years.
Following ileal resection, the jejunum undergoes morphologic
changes in villus height and crypt depth, along with increased
enzymatic activity. The ileum has relatively greater adaptive capacity, responding to jejunal loss by increasing in overall length and
diameter, in addition to histologic alterations, to maximize absorptive surface area.

EN
Intestinal adaptation is contingent upon enteral stimulation, and
will not occur in the fasted state. In particular, enteral fat stimulates the release of trophic hormones important to the adaptive
process. Continuous enteral feeding has been shown to be superior to bolus feeds in inducing adaptation, as mucosal receptors
and transporters remain maximally saturated.11

GROWTH HORMONE AND GLUTAMINE


Recent years have witnessed a surge of interest in pharmacologic
manipulation of the trophic hormones involved in intestinal adaptation, with some promising results. Growth hormone (GH) is
known to induce structural and functional adaptation in animal
models12,13 and has been studied in combination with the amino acid
glutamine, which is believed to be the primary fuel of enterocytes.14
Early case series of GH + glutamine in conjunction with a highcarbohydrate diet generated considerable optimism of improved
intestinal absorption,15,16 but the results of larger, more rigorous
trials have been mixed. A 2010 Cochrane Collaboration systematic
review analyzed five double-blind, placebo-controlled, randomized
trials from 1997 to 2005(Level 1a evidence).17 These studies consistently demonstrated that treatment with GH resulted in significant
increases in body weight and lean body mass (with or without glutamine), as well as the absorptive capacities of energy, nitrogen, and
fat, although these latter findings displayed considerable heterogeneity. Although PN requirements declined for some patients during
the treatment period, the benefits of GH + glutamine were not sustained after cessation of therapy. Concern regarding the neoplastic
risk of ongoing treatment with growth factors calls into question
the utility of this therapy. These trials provided little evidence of
synergy with glutamine, and are insufficient to recommend the
routine use of GH in SBS patients at present.
It has been suggested that the observed increases in body
weight may simply reflect extracellular fluid retention, and that
altered fluid balance may be secondary to renal-modulating effects
of GH, rather than actions on the gastrointestinal tract.18 The overwhelming incidence of peripheral edema documented in the trials
described above (77% overall) would support the notion that GHinduced weight gain may not reflect enhanced nutrient absorption.
In addition to peripheral edema, GH is associated with a variety of
adverse effects including carpal tunnel syndrome (32%), arthralgias (10%), and gynecomastia, which are thought to result from the
nonspecific nature of GH.17

5/21/2012 8:59:13 PM

198

Surgery: Evidence-Based Practice

Future studies investigating the effect of exogenous GH


administration during the early postoperative phase, rather than
after a period of homeostatic IF, might help elucidate the therapeutic potential of this intervention.

in PN requirements. Both hormonal therapies are currently only


recommended for use within the context of clinical trials (Grade B
recommendation).
4. What is the role of autologous intestinal reconstruction surgery in adults?

GLUCAGON-LIKE PEPTIDE 2
Glucagon-like peptide 2 (GLP-2) is another growth factor whose
potential role in the treatment of SBS has been extensively investigated. In addition to being intestinotrophic, GLP-2 slows gastric
emptying, inhibits gastric acid secretion, enhances mesenteric
blood flow, and may exert a protective effect against inflammatory
states. In vivo, GLP-2 is quickly inactivated by dipeptidyl peptidase
IV, prompting the development of a degradation-resistant synthetic
analog, teduglutide.
GLP-2 is synthesized in the distal ileum and proximal colon.
Patients with end jejunostomies therefore display a markedly reduced
postprandial rise in GLP-2, and benefit most from supplementation.
The first human study of GLP-2 involved eight SBS patients with end
jejunostomies, and demonstrated significant improvement in nutrient absorption, with histologic evidence of adaptation, and increased
body weight.19 Phase II trials of teduglutide20 yielded similar results,
with increased absorption, decreased fecal weight and increased
urine output. In a Phase III RCT of 83 PN-dependent patients, lowdose teduglutide resulted in >20% reduction in PN requirements,
with three patients able to discontinue PN altogether.21 A separate
trial examined the effects of continued GLP-2 administration over a
2-year period, and found a significant reduction in fecal wet weight
(from 3.0 to 2.0 kg/day) with improved renal function.22 While
jejunostomy patients derive maximal benefit from GLP-2 supplementation, patients with intact colons exhibit similar responses, suggesting that GLP-2 remains advantageous even at supraphysiologic
levels. In contrast to GH, GLP-2 is generally well tolerated (Level 1b
evidence).

FUTURE DIRECTIONS
A number of other hormonal agents are under investigation for
possible intestinotrophic roles in SBS. Thyroid hormone supplementation has been shown to enhance intestinal adaptation during the phase of transient hypothyroidism following massive small
bowel resection.23 EGF, IGF-1, and leptin have all demonstrated
promise in animal studies, but have yet to be trialed in humans.

A ROLE FOR OCTREOTIDE?


Care must be taken to avoid substances which may interfere with
the adaptation process. Octreotide is a potent antisecretory agent
which may be beneficial in the setting of high volume intestinal
losses, but it is also antitrophic. Therefore, current recommendations are to avoid administration of octreotide in the early adaptive phase, and to employ it judiciously thereafter if high intestinal
outputs render fluid and electrolyte balance problematic.
ANSWER: Enteral feeding is a necessary prerequisite for
intestinal adaptation. Administration of GH, with or without glutamine, can stimulate a short-term increase in body weight. GLP-2
increases fluid and nutrient absorption, with significant reductions

PMPH_CH22.indd 198

The goals of surgical intervention for SBS are three-fold: to slow


intestinal transit, improve peristaltic function, and enhance
mucosal absorption.24 In general, intestinal continuity should
be restored whenever possible, but other techniques to decrease
transit time (artificial valves, antiperistaltic segments, colonic
interposition) have fallen out of favor. Peristaltic function can be
impaired by strictured segments or excessive bowel dilatation.
Stricturoplasty and serosal patching are preferred over recurrent
resection to preserve absorptive surface area. Historically, dilated
bowel at risk of stasis and bacterial overgrowth was rectified using
tapering enteroplasty or intestinal plication techniques. These have
been largely supplanted by two intestinal lengthening procedures,
which also serve to enhance mucosal absorption and have emerged
as the mainstays of autologous intestinal reconstructive surgery.
Bianchis eponymous longitudinal intestinal lengthening
and tailoring (LILT) procedure was first described in 1980. This
technique involves separation of the two leaves of mesentery, with
transection of the bowel along its long axis and subsequent anastomosis in continuity. To date there are more than 130 reported
cases in the literature, with an overall survival rate of 80%, a mean
increase in intestinal length of 40% to 53% and enteral autonomy
achieved in 54%.25-27
Serial transverse enteroplasty (STEP) was first performed in
2003, and there are now more than 60 reported cases, the vast
majority in children.25,28-32
By sequentially partially transecting the small bowel using
alternate fi rings of the linear stapler along the mesenteric and
antimesenteric borders, this technique is eclipsing the Bianchi
procedure as a fi rst-line intervention due to its relative technical simplicity. Survival and enteral autonomy are comparable
to the Bianchi procedure to date (92% and 49%, respectively),
but STEP offers the added benefit of being able to more than
double initial remnant length, and it can be repeated if necessary in the event of bowel redilatation. Andres and colleagues33
report the largest single-center experience with repeat STEP
(reSTEP) involving 16 procedures in 14 patients (including two
adults) after previous Bianchi or STEP. At median follow-up of
14.5 months, survival was 100% with 43% weaned from PN;
36% were referred for IT, with good outcomes and no additional technical difficulty encountered as a result of the previous lengthening procedures.
Sudan et al.25 report the single largest experience with intestinal lengthening, comparing results with both Bianchi and
STEP procedures over a 24-year period, taking into account the
belated introduction of STEP. They performed 77 procedures (43
Bianchi vs 34 STEP) in 64 patients, including 14 adults. At median
follow-up of 3.8 years, there was no difference in survival (91%
overall) and comparable rates of enteral autonomy (58% overall).
There was, however, a nonsignificant trend toward longer time
to complete discontinuation of PN after the Bianchi procedure
(8.4 vs. 4.8 months after STEP, p = 0.07). STEP achieved marginally superior lengthening (52% vs. 48% with Bianchi, p = 0.01) and
complication rates were approximately 10% in both groups. Early
reported complications with both procedures include anastomotic

5/21/2012 8:59:13 PM

Short Bowel Syndrome

and staple line leaks, mechanical obstruction, and vascular injury


resulting in small bowel necrosis. Late complications documented
after Bianchi procedure include interloop fistulae and anastomotic stricture.
The efficacy of these procedures has led to many SBS patients
being liberated from the complications of PN-dependence. Although
data in adults are lacking, autologous intestinal reconstruction has
been postulated to be an effective bridge to transplantation.
ANSWER: The STEP and Bianchi procedures are two intestinal lengthening procedures whose safety and efficacy have been
demonstrated in the cumulative international experience in children. The literature in adults with SBS is meager, but there are case
reports of progression to enteral autonomy. Autologous intestinal
reconstruction may be sufficient to enable long-term survival after
SBS, or may be employed as a bridge to transplantation (Grade D
recommendation).
5. What is the role of IT in the management of SBS?
Despite ongoing refinements in technical aspects of IT and immunosuppression, including the use of tacrolimus-based regimens, as
well as steady improvements in clinical outcomes and postoperative
quality of life, the decision to pursue transplantation as a therapeutic
option must be made with careful consideration of its risks and benefits. The procedure is risky, with up to 50% of patients experiencing
complications such as mesenteric ischemia from vascular occlusion,
anastamotic leaks, intestinal perforation, stricture, line sepsis and
other nosocomial infections, opportunistic infections, and lymphoproliferative disease. The small intestine is highly immunogenic
and prone to rejection. During episodes of rejection, its mucosal
defenses may weaken, subjecting the recipient to sepsis risk at a time
when escalation of immunosuppression is often required. This combination of immunosuppression and bacterial colonization of the
graft leaves IT recipients vulnerable to rapid deterioration.34
Indications for transplantation were considered in a prospective study of 389 patients on home PN, which compared outcomes in patients with IF who remained on PN against those who
received IT. The study confirmed higher survival rates among PN
patients than IT patients, and supported it as a primary treatment
for IF. IT was still considered to be indicated for patients with desmoid tumors and PN-related liver failure, where hazard ratios for
death were high, and possibly for patients with recurrent catheterrelated complications and ultra short bowel.35
Because of the potential for morbidity and mortality, ITs have
mainly been performed in situations where no other therapeutic
options are available.36Therefore, no randomized trials and very
few cohort studies have been done to evaluate their efficacy and
safety in comparison to nontransplant alternatives. Most of our
insights about the role of IT are derived from national and international registries. These databases suggest that IF patients who
progress to end-stage liver disease (ESLD) due to PN-associated
cholestasis have the highest mortality among all patients on solid
organ waiting lists. The standard approach to these patients is combined hepatic and IT, as hepatic transplantation alone often leads
to recurrent liver failure if the IF is not addressed. Patients undergoing IT alone (no ESLD) have better prognosis than those with
multivisceral transplants, likely because of more stable physiology
at the time of the operation. Overall patient and graft survival at 1
year is about 80% and 65%, respectively for IT, and 50% and 49%
for combined intestinal and hepatic transplants. Survival figures
have improved over time.36 In recent years, living-related donor

PMPH_CH22.indd 199

199

liver and IT has been advanced to address high waiting list mortality among patients with combined organ failures. Outcomes
have been similar to those in conventional transplantation.39
ANSWER: Outcomes after IT are steadily improving. Combined intestinal and liver transplantation is indicated for IF patients
with ESLD. IT may also be indicated for those with high predicted
mortality due to conditions such as recurrent catheter-related
complications and ultra short bowel. The treatment of uncomplicated IF with PN and multidisciplinary care is associated with
high survival and favorable quality of life. PN therefore remains
the primary treatment for IF (Grade C recommendation).
6. What is the evidence for specialized centers of intestinal
rehabilitation?
The complexity of SBS and the rapid evolution of both medical
and surgical treatment strategies have prompted regional concentration of SBS management to specialized centers. Over the
past decade, intestinal rehabilitation programs (IRPs) have been
established at tertiary-care institutions offering comprehensive,
multidisciplinary therapy to promote intestinal adaptation with
the goal of eventual enteral autonomy. They provide aggressive
nutritional management with precise metabolic control, optimization of pharmacologic therapies with access to investigational
agents, prompt recognition and treatment of complications,
effective use of surgical rehabilitation techniques, and early referral for transplantation when indicated. The localization of IRPs
to centers with transplant programs facilitates and expedites the
referral process and may contribute to the high survival rates
after transplantation in these centers.
IRPs have demonstrated excellent outcomes in terms of survival
and enteral independence. To date, they have been predominantly
localized within pediatric medical centers, and the majority of the
literature regarding outcomes of multidisciplinary care reflects the
pediatric experience. The Center for Advanced Intestinal Rehabilitation at Childrens Hospital Boston compared the first 6 years of
their experience with historical controls, and found significantly
improved survival with comprehensive, multidisciplinary care
(89% cf 70%) and significant gains in enteral tolerance.38 Likewise,
Seattle Childrens Hospital documented an 88% survival rate over
the first four years of its Intestinal Failure Program, with 45%
enteral autonomy and a decrease in PN caloric requirements from
100% to 41% (p < 0.01).39
A common theme, and important feature of these referral
programs, is the successful rehabilitation of patients with preexisting liver disease. Torres et al.40 report the 4-year experience
of the University of Nebraska Medical Center, which includes an
adult SBS population. The majority of patients referred to their
program had hyperbilirubinemia and evidence of chronic liver
disease on pretreatment biopsy. Of these patients, 82% experienced normalization of liver enzymes with multidisciplinary
treatment. Another regional IRP reported a dramatic reversal of
IF-associated liver disease with aggressive rehabilitation. Of these
patients, 75% with evidence of liver disease treated over a 6-year
period experienced complete resolution, prompting the authors to
conclude that hepatic dysfunction within this population is often
reversible and may be rescued by referral to a specialized center.41
The preliminary results of a Canadian multidisciplinary IF unit
corroborate this finding. Although overall survival rates remain
unchanged compared to historical controls, mortality from liver
failure has declined significantly (from 90% to 46%).42

5/21/2012 8:59:13 PM

200

Surgery: Evidence-Based Practice

The success of these programs has been attributed to the expertise of dedicated practitioners delivered via a multimodal approach
with meticulous attention to detail. In addition, improved communication between care providers and family members allows for
expeditious decision-making. Close collaboration with transplant
teams enables early assessment and ensures continuity of care
throughout the entirety of the rehabilitation process.
The concentration of SBS care to dedicated centers has given
rise to the Pediatric Intestinal Failure Consortiuma multidisciplinary collaboration of pediatric hospitals in the U.S. and Canada
that plans to review current practices to better define the optimal
management of SBS in this population. Although much can be
extrapolated to the adult SBS patient from this body of knowledge,
a similar collaborative approach to the management of adult SBS
would undoubtedly be of great benefit.
ANSWER: Dedicated IRPs offering comprehensive, multidisciplinary treatment of SBS may offer some advantage in terms
of survival, enteral autonomy, and reversal of liver disease over

the care of patients in nonspecialized institutions (Grade C


recommendation).

CONCLUSIONS
Prompt surgical care for intestinal catastrophes, followed by judicious resection of small bowel and aggressive perioperative nutritional support are turning the tide in the management of SBS.
PN has permitted survival with high quality of life and low PNassociate mortality (0% to 22%). A new emphasis on intestinal
adaptation, selective use of autologous surgical reconstruction,
selective intestinal or multivisceral transplantation, and meticulous perioperative care at high volume centers has resulted in
4-year survival as high as 90%40 for a condition that, until recently,
was considered to be uniformly fatal. Dedicated clinical care and
more research are needed to continue to make advances in this
complex and inspiring area.43

Clinical Question Summary


Question

Recommendation

1 What is the role of PN in SBS?

Early PN should focus closely on


administration of macronutrients, careful
control of fluid and electrolyte balance,
and correction of micronutrient and
vitamin deficiencies. Patients should
be carefully transitioned to EN at the
earliest opportunity.

2 What is the evidence for GH in the


treatment of SBS?

GH can lead to short-term weight gain


with some evidence of increased
nutrient absorption, but the literature
does not support its routine use in the
management of SBS.

17

3 What is the evidence for


teduglutide in the treatment of
SBS?

The safety and efficacy of GLP-2 have been


demonstrated in clinical trials, which are
ongoing at present. GLP-2 administration
increases intestinal absorption,
decreases fecal output, and improves
renal function. Its use remains restricted
to an experimental context.

20-22

4 What is the role of autologous


intestinal reconstruction surgery
in adults with SBS?

Both STEP and Bianchi procedures are


effective in increasing intestinal length
and decreasing PN requirements in
children. There is a paucity of experience
in the adult SBS population, with case
reports of improved enteral tolerance
and autonomy.

25, 27, 32, 33


44

5 What is the current role for IT in


SBS?

IT is a good strategy in situations where


no other therapeutic options are viable
(e.g., PNALD with ESLD, catheterrelated complications).

36

6 Do IRPs offer any benefit in the


treatment of SBS?

Multidisciplinary IRPs have demonstrated


excellent outcomes in terms of survival,
increased enteral tolerance, and reversal
of hepatic dysfunction. They may offer
some advantage over nonspecialized
centers in the treatment of SBS.

38-42

PMPH_CH22.indd 200

Level of Evidence

References

5/21/2012 8:59:14 PM

Short Bowel Syndrome

REFERENCES
1. Donohoe CL, Reynolds von J. Short bowel syndrome. Surgeon.
2010;8(5):270-279.
2. Jeejeebhoy K. Management of short bowel syndrome: Avoidance
of total parenteral nutrition. Gastroenterol. 2006;130(2):S60-S66.
3. Dudrick SJ, Wilmore DW, Vars HM, Rhoads JE. Long-term total
parenteral nutrition with growth, development, and positive
nitrogen balance. Surgery. 1968;64(1):134-142.
4. Bines JE. Intestinal failure: A new era in clinical management.
J Gastroenterol Hepatol. 2009;24:S86-S92.
5. Crenn P, Coudray-Lucas C, Thuillier F, Cynober L, Messing B.
Postabsorptive plasma citrulline concentration is a marker of
absorptive enterocyte mass and intestinal failure in humans. Gastroenterol. 2000;119(6):1496-1505.
6. de Meijer VE, Gura KM, Meisel JA, Le HD, Puder M. Parenteral fish oil monotherapy in the management of patients
with parenteral nutrition-associated liver disease. Arch Surg.
2010;145(6):547-551.
7. Fallon EM, Le HD, Puder M. Prevention of parenteral nutritionassociated liver disease: Role of omega-3 fish oil. Curr Opin Org
Transplant. 2010;15(3):334-340.
8. Sigalet DL, Mackenzie SL, Hameed SM. Enteral nutrition and
mucosal immunity: implications for feeding strategies in surgery
and trauma. Can J Surg. 2004;47(2):109-116.
9. Nordgaard I, Hansen BS, Mortensen PB. Colon as a digestive organ
in patients with short bowel. Lancet. 1994;343(8894):373-376.
10. Matarese LE, OKeefe SJ, Kandil HM, et al. Short bowel syndrome: Clinical guidelines for nutrition management. Nutr Clin
Prac. 2005;20(5):493-502.
11. Joly F, Mayeur C, Messing B, et al. Morphological adaptation
with preserved proliferation/transporter content in the colon of
patients with short bowel syndrome. Am J Physiol Gastrointest
Liver Physiol. 2009;297(1):G116-G123.
12. Shulman DI, Hu CS, Duckett G, Lavallee-Grey M. Effects of shortterm growth hormone therapy in rats undergoing 75% small
intestinal resection. J Pediatr Gastroenterol Nutr. 1992;14(1):3-11.
13. Inoue Y, Copeland EM, Souba WW. Growth hormone enhances
amino acid uptake by the human small intestine. Ann Surg.
1994;219(6):715-722; discussion 722-724.
14. van der Hulst RR, van Kreel BK, Meyenfeldt von MF, et al.
Glutamine and the preservation of gut integrity. Lancet.
1993;341(8857):1363-1365.
15. Byrne TA, Persinger RL, Young LS, Ziegler TR, Wilmore DW. A new
treatment for patients with short-bowel syndrome. Growth hormone, glutamine, and a modified diet. Ann Surg. 1995;222(3):243254; discussion 254-255.
16. Byrne TA, Morrissey TB, Nattakom TV, Ziegler TR, Wilmore
DW. Growth hormone, glutamine, and a modified diet enhance
nutrient absorption in patients with severe short bowel syndrome. JPEN J Parenter Enteral Nutr. 1995;19(4):296-302.
17. Wales PW, Nasr A, de Silva N, Yamada J. Human growth hormone and glutamine for patients with short bowel syndrome.
Cochrane Database Syst Rev. 2010;16(6):CD006321.
18. Jeppesen P. Growth factors in short-bowel syndrome patients.
Gastroenterol Clin N Am. 2007;36(1):109-121.
19. Jeppesen P, Hartmann B, Thulesen J, et al. Glucagon-like peptide 2
improves nutrient absorption and nutritional status in short-bowel
patients with no colon4. Gastroenterol. 2001;120(4):806-815.
20. Jeppesen PB. Teduglutide (ALX-0600), a dipeptidyl peptidase
IV resistant glucagon-like peptide 2 analogue, improves intestinal function in short bowel syndrome patients. Gut. 2005;54(9):
1224-1231.

PMPH_CH22.indd 201

201

21. Jeppesen PB, Gilroy R, Pertkiewicz M, et al. Randomised placebocontrolled trial of teduglutide in reducing parenteral nutrition and/
or intravenous fluid requirements in patients with short bowel syndrome. Gut. 2011.
22. Jeppesen PB, Lund P, Gottschalck IB, et al. Short bowel patients
treated for two years with glucagon-like Peptide 2: effects on
intestinal morphology and absorption, renal function, bone and
body composition, and muscle function. Gastroenterol Res Prac.
2009;2009:Article ID 616054.
23. Yuksel O, Tatlicioglu E, Poyraz A, et al. Effects of thyroid hormone on the adaptation in short bowel syndrome. J Surg Res.
2009;155(1):116-124.
24. Petty JK, Ziegler MM. Operative strategies for necrotizing enterocolitis: The prevention and treatment of short-bowel syndrome.
Semin Pediatr Surg. 2005;14(3):191-198.
25. Sudan D, Thompson J, Botha J, et al. Comparison of intestinal
lengthening procedures for patients with short bowel syndrome.
Ann Surg. 2007;246(4):593-601; discussion 601-604.
26. Hosie S, Loff S, Wirth H, et al. Experience of 49 longitudinal
intestinal lengthening procedures for short bowel syndrome.
Eur J Pediatr Surg: official journal of Austrian Association of
Pediatric Surgery ... [et al] = Zeitschrift fr Kinderchirurgie. 2006;
16(3):171-175.
27. Reinshagen K, Kabs C, Wirth H, et al. Long-term outcome in patients
with short bowel syndrome after longitudinal intestinal lengthening and tailoring. J Pediatr Gastroenterol Nutr. 2008;47(5):573-578.
28. Kim HB, Lee PW, Garza J, et al. Serial transverse enteroplasty
for short bowel syndrome: a case report. J Pediatr Surg. 2003;
38(6):881-885.
29. Ismail A, Alkadhi A, Alnagaar O, Khirate A. Serial transverse
enteroplasty in intestinal atresia management. J Pediatr Surg.
2005;40(2):E5-E6.
30. Javid PJ, Kim HB, Duggan CP, Jaksic T. Serial transverse enteroplasty is associated with successful short-term outcomes in infants
with short bowel syndrome. J Pediatr Surg. 2005;40(6):1019-1023;
discussion 1023-1024.
31. Wales PW, Dutta S. Serial transverse enteroplasty as primary
therapy for neonates with proximal jejunal atresia. J Pediatr Surg.
2005;40(3):E31-E34.
32. Modi B, Javid P, Jaksic T, et al. First Report of the International
Serial Transverse Enteroplasty Data Registry: Indications, Efficacy, and Complications. J Am Coll Surg. 2007;204(3):365-371.
33. Andres AM, Thompson J, Grant W, et al. Repeat surgical
bowel lengthening with the STEP procedure. Transplantation.
2008;85(9):1294-1299.
34. Gilroy R, Sudan D. Liver and small bowel transplantation: Therapeutic alternatives for the treatment of liver disease and intestinal failure. Semin Liver Dis. 2000;20(4):437-450.
35. Pironi L, Joly F, Forbes A, et al. Long-term follow-up of patients
on home parenteral nutrition in Europe: Implications for intestinal transplantation. Gut. 2010;60(1):17-25.
36. American Gastroenterological Association. American Gastroenterological Association medical position statement: Short
bowel syndrome and intestinal transplantation. Gastroenterology. 2003;124(4):1105-1110.
37. Tzvetanov IG, Oberholzer J, Benedetti E. Current status of living
donor small bowel transplantation. Curr Opin Organ Transplant.
2010;15(3):346-348.
38. Modi BP, Langer M, Ching YA, et al. Improved survival in a
multidisciplinary short bowel syndrome program. J Pediatr Surg.
2008;43(1):20-24.
39. Javid PJ, Malone FR, Reyes J, Healey PJ, Horslen SP. The experience of a regional pediatric intestinal failure program: Successful

5/21/2012 8:59:14 PM

202

Surgery: Evidence-Based Practice

outcomes from intestinal rehabilitation. Am J Surg. 2010;


199(5):676-679.
40. Torres C, Sudan D, Vanderhoof J, et al. Role of an intestinal
rehabilitation program in the treatment of advanced intestinal
failure. J Pediatr Gastroenterol Nutr. 2007;45(2):204-212.
41. Cowles RA, Ventura KA, Martinez M, et al. Reversal of intestinal
failure-associated liver disease in infants and children on parenteral nutrition: experience with 93 patients at a referral center for
intestinal rehabilitation. J Pediatr Surg. 2010;45(1):84-87; discussion 87-88.

PMPH_CH22.indd 202

42. Diamond IR, de Silva N, Pencharz PB, et al. Neonatal short bowel
syndrome outcomes after the establishment of the first Canadian
multidisciplinary intestinal rehabilitation program: preliminary
experience. J Pediatr Surg. 2007;42(5):806-811.
43. Nightingale J. Guidelines for management of patients with a
short bowel. Gut. 2006;55(Suppl 4):iv1-iv12.
44. Wales PW, Brindle M, Sauer CJ, Patel S, de Silva N, Chait P.
Percutaneous cholangiography for the treatment of parenteral
nutrition-associated cholestasis in surgical neonates: preliminary experience. J Pediatr Surg. 2007;42(11):1913-1918.

5/21/2012 8:59:14 PM

Commentary on Short
Bowel Syndrome
James Davis

The chapter Short Bowel Syndrome and Intestinal Failure provides insight and recommendations based on level of evidence
supporting decision-making in five areas associated with management of patients with short bowel syndrome (SBS). The questions
addressed are (1) What is the optimal nutrition support strategy
in SBS? (2) What pharmacologic strategies can promote intestinal
adaptation? (3) What is the role of autologous intestinal reconstruction surgery in adults? (4) What is the role of IT (intestinal
transplantation) in the management of SBS? (5) What is the evidence of specialized centers of intestinal rehabilitation?
With regards to the nutrition support strategy, optimizing
nutritional support for these patients is crucial, starting with
parenteral nutrition in the early postoperative stages and moving toward enteral nutrition (EN). There has been a general trend
toward early feeding of patients that have undergone gastrointestinal surgery. The advantages of EN over parenteral nutrition include maintenance of intestinal barriers and better stress
hormone responses. EN has also been reported to preserve gut
structure and function as well as enhance gut mediated immunity.1 More recently, a meta-analysis on early feeding versus late
feeding in patients with gastrointestinal surgery showed that
there was no difference in complications between the two groups
and in fact a decrease in mortality in the early feeding group.2
With regards to what pharmacologic strategies can promote
intestinal adaptation, this area appears to have immense potential
for research. The level of evidence used to support the recommendations is mostly based on expert opinion or case studies, which
makes it difficult to obtain a firm conclusion. The exception to this
is seen in question (2) on glutamine and growth hormone, as well
as glucagon-like peptide (GL-2) which provided level 1a and 1b
evidence. Glutamine has been shown to be crucial in maintaining
intestinal health. Not only is it the primary energy source for the
small intestine, it also helps in maintaining the gut mucosal barrier, as well as plays a major role in the maintenance of metabolism and function.3,4 However, growth hormone has not shown a
consistent beneficial effect. Currently, GL-2, as well as epidermal
growth factor (EGF), has shown promise in promoting intestinal
growth and enhancing absorption. GL-2 is produced in the distal
intestine and colon and has a role in stimulating mucosal growth
and absorption. EGF functions to increase absorption as well as

stimulates intestinal adaptation. Both are currently in clinic trials


and show real potential for the future.
With regards to the role of autologous intestinal reconstruction in adults, experience has been in the pediatric population. As
stated by the author, literature in the adult population is scarce at
best. There is a recent study of 20 adult patients, 6 who underwent
the Biachi procedure and 14 who underwent the STEP procedure,
looking at outcome of intestinal lengthening.5 Their results of a
90% survival rate with mean follow-up of 4.1 years and 59% enteral
autonomy were comparable to the published data in the pediatric
population. Given the paucity of data it is difficult to ascertain
the safety and efficacy in the adult population. It appears to be an
option in adults who have no further progression in enteral tolerance and are dependent on parenteral nutrition.
Patients who develop life-threatening complications from
parenteral nutrition are candidates for IT. There have been approximately 2000 cases reported in the United States with majority of
patients below 18 years old. Looking at IT in the United States
between 1999 and 2008, there has been an improvement in time
to transplant as well as waiting list mortality.6 Short-term results
have improved although the difficulty still lies in preventing rejection in this population. As more experience is gained in this field
and long-term data are gathered, a better understanding of the
role for transplant may be seen.
There is a lack of evidence supporting specialized center in
intestinal rehabilitation. However, there is extensive experience
and evidence of the importance of specialized centers in the cardiac population7 as well as the trauma population.8 In both areas,
it has been shown that regionalization led to improved outcomes
as well as cost-effectiveness. Extrapolating this evidence to the
short bowel population, we could make an argument for developing specialized centers for intestinal rehabilitation in order to
improve outcome as well as providing a comprehensive data base
for further research.
These questions, as important as they are, are difficult to
answer due to the rarity of this disease. Most of our understanding is extrapolated from the neonatal population. Given the difficulty of this subject and the level of evidence available, the author
has done an excellent review of the literature to obtain recommendation to these difficult questions.

203

PMPH_CH22.indd 203

5/21/2012 8:59:14 PM

204

Surgery: Evidence-Based Practice

REFERENCES
1. Winter TA, OKeefe SJ, Callanan M, Marks T. Effect of severe
undernutrition and subsequent refeeding on gut mucosal protein
fractional synthesis in human subject. Nutrition. 2007;23:29-35.
2. Lewis SJ, Andersen HK, Thomas S. Early enteral nutrition within
24 h of intestinal surgery versus later commencement of feeding: a systematic review and meta-analysis. J Gastrointest Surg.
2009;13(3):569-575.
3. van der Hulst RR, van Kreel BK, von Meyenfeldt MF, Brummer RJ,
Arends JW, Deutz NE, et al. Glutamine and the preservation of
gut integrity. Lancet. 1993;341:1363-1365.

PMPH_CH22.indd 204

4. Souba WW, Klimberg VS, Plumley DA, Salloum RM, Flynn TC,
Bland KI, et al. The role of glutamine in maintaining a healthy gut
and supporting the metabolic response to injury and infection.
J Surg Res. 1990;48(4):383-391.
5. Yannam GR, et al. Intestinal lengthening in adult patients with
short bowel syndrome. J Gastrointest Surg. 2010;14(12):1931-1936.
6. Mazariegos GV, et al. Intestinal transplantation in the United
States 19992008. Am J Transplant. 2010;10:1020-1034.
7. Kereiakes DJ. Specialized center sand systems for heart attack
care. Am Heart Hosp J 2008;6(1):14-20.
8. MacKenzie EJ et al. The value of trauma center care. J Trauma.
2010;69(1):1-10.

5/21/2012 8:59:14 PM

PART IV

LARGE BOWEL

PMPH_CH23.indd 205

5/21/2012 9:01:17 PM

PMPH_CH23.indd 206

5/21/2012 9:01:18 PM

CHAPTER
CHAPTER 23
1

Diverticular Disease of the Colon


Brent Izu and A. Peter Ekeh

the Hinchey Stages IIV, with worse clinical features and mortality with each successive stage. Stage IV disease involves feculent
peritonitis and was described to be accompanied with a high
mortality.8 This staging system has been widely adopted to provide a standard for comparison of the severity of acute diverticulitis. The Hinchey classification has also been used as a basis for
decision-making with regards to specific surgical therapy in the
acute setting.
The operative management of complicated acute diverticulitis has evolved over time. Fortunately, surgical management is
necessary in less than 10% of patients admitted to the hospital
with an attack of diverticulitis.9 Historically, staged operations
were commonly performed involving the initial closure of the
perforation with a proximal diversion (ileostomy or transverse
colostomy) and then a subsequent delayed resection of the diseased portion and anastomosis. This approach has been largely
supplanted by the Hartmann operationresection of the acutely
inflamed bowel including the perforated portion of colon and a
proximal end colostomy.
In one of the very few randomized prospective trials that
have been performed relating to management of diverticulitis,
Zeitoun et al. compared primary resection with suture drainage with proximal colostomy followed by secondary resection. In
this study, most of the patients in the primary resection group
had Hartmann operations with a small minority (5%) undergoing
primary anastomoses. They found a lower incidence of postoperative peritonitis; fewer reoperations and a shorter hospital stay in
the primary resection group.10 (Level 1 evidence) This study only
included patients with Hinchley Stages III and IV. There was no
difference in mortality between the two groups. This confirmed
prior retrospective cohort studies which had arrived at similar
conclusions. Finlay et al. compared outcomes in patients undergoing primary resection with proximal drainage and diversion.
Morbidity was significantly higher in patients undergoing diversion without resection, including a higher incidence of fistula formation in the patients with staged operations.11
More recently, other authors have published data on the use
of primary resection and anastomosis in patients with perforated
sigmoid diverticulitis. This has been described in some instances
with extra measures like proximal protective ileostomies and
intraoperative on-table colonic lavage. The reports involving

Diverticular disease of the colon is considered as a disease of modern times, prevalent principally in Western Societies. The appearance of colonic diverticulosis in postmortem specimens paralleled
both industrialization and the widespread use of refined wheat
products.1 The incidence of colonic diverticular disease, particularly its acute manifestations, has continued to increase.2,3 Along
with its ensuing complications, diverticular disease is estimated to
result in at least USD 2.4 billion in direct health-care costs annually in the United States.4
Acute diverticulitis is the most frequent complication arising from the presence of colonic diverticula. The sigmoid colon
is affected in 95% of cases. Patients may present initially with
complications such as perforation requiring immediate operative
intervention. Most patients, however, recover following conservative medical management with appropriate antibiotic therapy.
Up to 85% of cases of acute diverticulitis can be managed nonoperatively.5 Improvements in antibiotic therapy, better imaging
diagnostic modalities, and the use of appropriate percutaneous
drainage techniques have aided the prevalence of nonoperative
therapy in the acute setting.6,7
The methods of management of diverticulitis in the acute
setting and following discharge from the hospital have evolved
over the past few decades. Several of the longstanding commonly
accepted dogmas and guidelines have been based on a small collection of studies generally with lower-level evidence. Many of
these standards have been challenged in recent years.
Pertinent areas of interest and controversy in the management of patients with colonic diverticular disease include the
indications and timing for operative management, the relevance
of age in operative decision-making, the appropriate methods
and techniques of operative management, the place of laparoscopy, and strategies for preventing recurrent attacks. The relevant
recent literature pertaining to these areas will be discussed, examined, and reviewed below.
1. What is the optimal operation for patients requiring surgery
for complicated acute diverticulitis? Is performing a primary
anastomosis an option?
Hinchey in his 1978 paper on the treatment of diverticulitis
divided acute diverticulitis into four stages, now referred to as
207

PMPH_CH23.indd 207

5/21/2012 9:01:18 PM

208

Surgery: Evidence-Based Practice

primary anastomosis in the acute setting are all retrospective.


Stumpf et al. found 19 of 36 patients considered low risk (Hinchey
Stages I and II) had no complications following primary resection
and anastomosis. They concluded that this method of management be considered in low risk patients with acute diverticulitis.
(Level 2b evidence)12 Similarly, a small retrospective series from
Australia comparing 33 patients who had a primary anastomosis
with 64 who had Hartmann procedures revealed no marked difference in morbidity, and no increased mortality. They also concluded that primary anastomosis has an acceptable morbidity and
mortality. (Level 2 evidence) The patients in the primary anastomosis group were in earlier Hinchey stages.13 These two studies mentioned are representative of several other reports in the
literature with similar findings and similar levels of evidence.14-17
Protective ileostomies and colonic lavage were utilized in some
of these studies, but no recommendation can be made on these
practices based on their irregular and random use.
Overall, the literature regarding primary anastomosis of
acute complicated diverticulitis involves heterogeneous populations, differing methods (e.g., the use of protective ileostomy and
intraoperative colonic lavage), and possess problems of selection
bias. Two recent systemic reviews of the retrospective literature on
this topic highlighted this observation, recognizing that patients
selected for primary resection and anastomosis have a lower mortality than those treated by Hartmanns procedure. These factors
limit clinically sound conclusions and demonstrate the need for
prospective randomized studies in this area.18,19
In recent years, a novel approach in the operative management of acute diverticular perforations with the use of laparoscopy
has been reported. This involves laparoscopy with intraperitoneal
irrigation and drain placement in lieu of the standard operative
procedures. The goal in this approach is to avoid a Hartmann procedure and delay the resection of the diseased colon to an elective
setting. A number of case series have described there experiences
with this technique, generally with favorable results (Level 4
evidence).20-23 A small retrospective study comparing laparoscopic
peritoneal lavage with primary anastomosis with a defunctioning stoma for Hinchey Stage III complicated diverticulitis demonstrated a shorter hospital stay in the laparoscopic cohort. Most of
the patients in the laparoscopic group eventually underwent elective surgical resection laparoscopically.24 (Level 2 evidence)
Summary: Primary resection of the inflamed colon (with or
without primary anastomosis) is the optimal method of treating
complicated sigmoid diverticulitis. (Grade A recommendation)
Primary anastomosis of the colon at the initial operation can be
considered in Hinchey Stage I and II patients. (Grade B recommendation) Laparoscopic intraperitoneal lavage may be considered in
perforated diverticulitis but more studies addressing this technique
are needed to validate this practice. (Grade B recommendation)
2. What are appropriate indications for elective sigmoid resection
after uncomplicated diverticulitis?
There is little controversy with regard to the utility of operative
intervention for complications arising from diverticular disease in
the acute period or in the long term. The appropriate indications
for surgical intervention after uncomplicated acute diverticulitis,
that is, disease in the absence of the complications of fistulas, strictures, abscess or free perforation, have been subject to much debate.
The primary purpose of surgical intervention after acute attacks of
diverticulitis is the prevention of recurrence and the accompanying

PMPH_CH23.indd 208

morbidity. Consequently, recommendations are generally based on


the risk of recurrence and the severity of subsequent attacks. Most
studies assessing the risk of repeat attacks are retrospective studies
and have very wide variability of follow-up, making it difficult to
establish an accurate recurrence rate.
Clinical practice patterns and recommended guidelines have
proposed elective colon resection after two or more acute attacks
of diverticulitis successfully treated medically, or after a single
attack requiring in a patient below 40 years of age. (Grade C recommendation) Although these recommendations are currently
considered to be dated, their origins are of importance to gain
a balanced historical perspective and understanding of the management of this disease process.5
The recommendation for surgery following two attacks in older
patients is based primarily on case series data indicating significant recurrence rates after medically managed acute diverticulitis.
(Level 4 evidence) Parks, in 1969, published a review of 455 patients
admitted with acute diverticulitis. Of the patients treated medically,
24.6% subsequently had a second attack and 3.8% a third. Furthermore, the paper suggested that medical management was less effective for symptom control with subsequent bouts.25 Makela et al.26
similarly showed recurrences of 22% of patients with diverticulitis
managed medically and complications seen in 50% of patients who
presented with a second attack (Level 4 evidence).
Recent series have challenged the true incidence of the recurrence of acute diverticulitis as well as the severity of subsequent
attacks stated in the older literature. Addressing the incidence
of recurrence, a large retrospective cohort series involving over
3000 patients admitted for episodes of acute diverticulitis showed
that 9.4% of patients had a single recurrence and 3.9% had a second episode.27 This is lower than has reported in the older studies
and has a longer follow-up period than any of the prior reports
(8.9 years). (Level 2 evidence) Another retrospective review showed
that only 2.7% of patients who presented emergently with acute
diverticulitis and required surgery had a prior history of medical management. The majority of the cases were initial presentations.28 (Level 4 evidence) Other contemporary series on the other
hand have identified high recurrence rates after a single episode
of diverticulitis, although the recurrences were not necessarily
complicated presentations and could be typically treated medically. A study from the Lahey Clinic in Massachusetts involving
over 600 patients demonstrated a recurrence rate of 36% after
an initial attack although only 3.9% of these presentations were
complicated.29 (Level 2 evidence) Similarly, a series from the
Netherlands demonstrated a 48% recurrence rate in noncomplicated diverticulitis treated medically.30
Regarding the severity of further subsequent attacks of diverticulitis, a retrospective study by Chapman et al. found that multiple episodes of diverticulitis are not associated with increased risk
of mortality or poor outcomes from complicated diverticulitis.31
(Level 2 evidence) Salem and others used a state-wide database to
construct a Markov model to evaluate lifetime risks of death and
colostomy, care costs, and quality of life associated with elective
colectomy after subsequent episodes of diverticulitis using hypothetical cohorts of 35- and 50-year-old patients who recovered
from a nonsurgically treated diverticulitis episode. They found
that performing a colectomy after the fourth rather than the second episode in patients older than 50 years resulted in 0.5% fewer
deaths, 0.7% fewer colostomies, and saved USD 1035 per patient.
They concluded that expectant medical management after uncomplicated diverticulitis was associated with lower rates of death and

5/21/2012 9:01:18 PM

Diverticular Disease of the Colon

colostomy, and was cost saving compared with a strategy of elective prophylactic colectomy. (Level 2 evidence)32
To date, there are no randomized controlled trials pitting
observation against elective sigmoid resection following recurrent episodes of uncomplicated diverticular disease of the colon.
A multicenter randomized clinical trial is currently underway in
Europe, which intends to compare elective colon resection with
conservative therapy in patients with recurrent diverticulitis or
continuing symptoms after an attack.33
Summary: Though elective sigmoid resection has been traditionally recommended after two attacks of uncomplicated diverticulitis, a case-by-case determination made to establish the need for
operative management is necessary. (Grade C recommendation)
Waiting until at least the fourth episode attack of diverticulitis before elective surgery can result in lower colostomy and lower
death rates. (Grade B recommendation)
Given the strength of these recommendations, there is no
strong basis for the conventional decision to routinely proceed to
a colectomy after two episodes of uncomplicated diverticulitis.
3. Should younger patients (<4050 years) undergo elective sigmoid
colon resection after a single attack of acute diverticulitis?
Traditional clinical practice and expert guidelines have advocated
elective sigmoid colectomy after the first attack of uncomplicated
diverticulitis in patients below 40 years of age. Th is recommendation is based on multiple case series and retrospective studies
from the 1960s and 70s demonstrating more virulent presentation and more recurrences in patients below 40 years of age.
Studies confirming this notion and others challenging this are
continuously presented in the literatureall similarly retrospective. There are no prospective studies comparing observation to
elective surgery in patients below 40 years of age.
Multiple retrospective cohort studies have compared rates
of complication and operation in younger patients with those of
older patients. Some reports show that younger patients develop
more subsequent complications, have more recurrences, and that
the disease displays a more aggressive course compared with older
patients (Level 2b evidence).34-37 Other studies of comparable
quality draw different conclusions disputing the claim of a more
virulent disease process in younger patients.38-40
A recent retrospective review comparing patients older than
and younger than 50 years of age demonstrated more sever complications in the older group, although there were more recurrences in the group younger than 50 years of age.41
The decision tree analysis model described above by Salem
et al. specifically addressing younger patients concluded that performing colectomy after the fourth episode compared with the
first episode resulted in 0.1% fewer deaths, 2% fewer colostomies,
and saved US$5429 per patient (Level 2 evidence).32
Summary: Given the conflicting conclusions based on evidence of similar quality (Level 2) and studies of varying quality,
no definite evidence-based recommendations can be made with
regard to the indication for surgery after an uncomplicated attack
of acute diverticulitis in younger patients. Individualized decisions based on patients circumstance will need to be made prior
to proceeding with surgery. (Grade C recommendation)
However, as in older patients, there is a reduction in the number of colostomies and death rate associated with holding elective surgery until after the fourth attackit ever occurs. (Grade B
recommendation)

PMPH_CH23.indd 209

209

4. Are there any evidence-based dietary recommendations to


prevent the recurrence of acute uncomplicated diverticulitis? Is
the practice of prohibiting the intake of seeds, popcorn, etc.
after an acute episode valid?
In conjunction with standard antibiotic therapy for uncomplicated
diverticulitis, patients are often given dietary recommendations.
Fiber intake is the most significant dietary factor in preventing
and reducing recurrence of diverticulitis. The progression of
colonic diverticular disease has paralleled the drop in dietary
fiber consumption in the United States, Europe, and Asia.1
A number of studies have highlighted the benefits of fiber intake
in the prevention and recurrence of diverticular disease. Burkitt,
based on his experiences in rural Africa, published the original
observations of the effect of fiber, over 30 years ago. He compared
colonic transit times and stool weight in three populations with low,
mixed, and high residue diets. Colonic transit time was decreased
and stool weight was increased in patients with high residue diets.
He further obtained epidemiological data from various countries,
noting the very low prevalence of diverticular disease in populations
with high residue diets compared with those with low and mixed residue diets.1 A cohort study from Greece demonstrated that patients
with radiologically confirmed diverticulitis were demonstrated to
have a lower intake of fiber and higher intake of red meat.42
A prospective questionnaire-based study with a 4-year
follow-up by Aldoori et al. evaluated the effect of various diets on
the incidence of diverticular disease in a prospective longitudinal
cohort study involving over 40,000 men. The participants reporting
diets high in fruit and vegetable fiber had a significantly lower incidence of symptomatic diverticulitis. Diets high in fat and red meat
were also noted to augment the risk.43 (Level 2b evidence) The same
group of investigators recently updated this study, 18 years after its
commencement, focusing specifically on the relationship between
the incidence of diverticular disease and nut, corn, and popcorn
consumption. They noted no associations between the intake of
these foods and the incidence of diverticulitis or diverticular bleeding. There was in fact an inverse association between the consumption of these foods and diverticulitis.44 (Level 2b evidence)
There is some good evidence that supports the clinical recommendation given to patients to increase the fiber intake after acute
attacks of diverticulitis as a means to lower recurrence rates. Brodribb demonstrated in a randomized controlled trial with fiber
versus placebo that fiber improved symptoms of dyspepsia, bowel
dysfunction, and pain in patients with symptomatic diverticular
disease.45 (Level 1 evidence) This is in line with another randomized
controlled crossover trial by Taylor who performed in the 1970s,
comparing bran tablets (18 g/day) with a high-roughage diet and a
laxative. The bran group was found to have better results in improving symptom score, stool weight, transit time, and motility.46
The advice given against the consumption of popcorn, seeds,
and nuts in an attempt to prevent obstruction of diverticula and
subsequent inflammation has no basis in the medical literature.
Summary: Dietary fiber can play a role in both the prevention
of initial and recurrent attacks of diverticulitis. Patients should be
advised to increase their fiber content in their diet after a bout of
uncomplicated diverticulitis. (Grade A recommendation)
5. Is laparoscopic colectomy equivalent or superior to open
colectomy for diverticular disease? Is the overall cost different?
A multi-institutional randomized prospective trial published in the
New England Journal of Medicine in 2004 demonstrated a clear

5/21/2012 9:01:18 PM

210

Surgery: Evidence-Based Practice

role for laparoscopy in colon surgery. This study, however, focused


specifically on colon cancer and demonstrated noninferiority in
recurrence rates, a shorter length of stay, and a decreased use of parenteral narcotics in the laparoscopic group. A recently concluded
prospective multicenter, randomized controlled study specifically
compared laparoscopic sigmoid colon resection with the open procedure in patients with diverticular disease. This European trial,
which ran over a 4-year period and included 104 patients, showed
a reduction in complications in the laparoscopic cohort of patients
compared with the open group (9.6% vs. 25.0%; p = .038). The laparoscopic group also had statistically significant decreases in postoperative pain, less systemic analgesic needs, and shorter hospital
stays. The operative time was longer in the laparoscopic group.47
A 6-month follow-up of this study further showed a reduction of
major complications in the laparoscopic group.48 (Level 1 evidence)
There are a number of other retrospective cohort and case controlled studies offering Level 2 and 3 evidence on a range of outcome
measures comparing laparoscopic sigmoid colectomy for diverticular disease with the traditional open procedure.
Comparing laparoscopic colon resection with the open procedure, a number of these studies found no difference in complication rates or mortality.49 A quicker return to diet, shorter time to

first bowel movement, reduced length of hospital stay, and reduced


estimated blood loss have, however, been consistently reported to
be associated with laparoscopy.50-54 (Level 2b evidence)
Laparoscopic sigmoidectomy in most series had longer operative time but typically a shorter length of stay and increased hospital charges.51,52 (Level 2 evidence)
A single study from France comparing laparoscopic with open
approaches in elderly patients concluded based on their findings that
laparoscopic colectomy could be safely applied to older patients with
a reduction in complications, less pain, and a shorter hospital stay.55
(Level 2 evidence)
Two studies examining the cost per case report an overall
reduction with laparoscopic sigmoid colectomies possibly related
to the reduced length of stay.53,55,56
Summary: Laparoscopic colon resection is a safe and effective
approach for the elective treatment of patients with diverticular
disease and is associated with fewer complications, less operative
pain, and a shorter postoperative hospital stay. (Grade A recommendation) Laparoscopic colon resection is furthermore associated with a quicker resumption of bowel function and reduced
intraoperative blood loss. It is appropriate for elderly patients.
(Grade B recommendation)

Clinical Question Summary


Question

Answer

1 Is laparoscopic colectomy
equivalent or superior to open
colectomy for diverticular
disease? Is the overall cost
different?

Laparoscopic colon resection is a


safe and effective approach for the
elective treatment of patients with
diverticular disease. Laparoscopic
colon resection is associated with
a lower anastomotic leak rate and
morbidity rate.

BA

47-56

2 What are appropriate


indications for elective sigmoid
resection after uncomplicated
diverticulitis?

A case-by-case determination of the


need for operative management is
necessary.

27, 31, 32

3 Should younger patients


(<4050 years) undergo
elective sigmoid colon
resection after a single attack
of acute diverticulitis?

Individualized decisions based on


patients circumstance will need to
be made prior to proceeding with
surgery.

32, 34-38

4 Are there any evidence-based


dietary recommendations
to prevent the recurrence
of acute uncomplicated
diverticulitis? Is the practice
of prohibiting the intake of
seeds, popcorn, etc. after an
acute episode valid?

Dietary fiber can play a role in both


the prevention of initial and
recurrent attacks of uncomplicated
diverticulitis.

44-46

5 What is the optimal operation


for patients requiring surgery
for complicated acute
diverticulitis? Is performing
a primary anastomosis an
option?

Primary resection of the inflamed


colon (with or without primary
anastomosis). Primary anastomosis of
the colon at the initial operation can
be considered in Hinchey Stage I and
II patients.

AB

10-17

PMPH_CH23.indd 210

Grade of
Recommendation

References

5/21/2012 9:01:18 PM

Diverticular Disease of the Colon

211

Summary Table
Subject

Year

References

Level of
Evidence

Strength of
Recommendation

Indication for elective


colectomy after
uncomplicated
diverticulitis.

2004

32

Expectant medical management


after uncomplicated diverticulitis
with lower cost, fewer deaths,
and fewer colostomies.

Elective colectomy in
younger patients
following a single
episode

2004

32

Colectomy after the fourth episode


is associated with lower rates of
death and colostomy.

Dietary fiber intake and


risk of diverticulitis

1994

43

Diets high in fruit and vegetable


fiber had a significantly lower
incidence of symptomatic
diverticulitis.

Primary anastomosis for


sigmoid diverticulitis

2007

12

Primary resection and anastomosis


can be considered in low risk
patients (Hinchey Stages I and II)
with acute diverticulitis.

Laparoscopy for
diverticular disease

2005 2009
2010

50 47 48

2I

BA

Laparoscopy associated with quicker


return to diet, shorter time, and
reduced length of hospital stay.
Laparoscopic sigmoid colectomy
is associated with a lower
anastomotic leak rate and a
lower postoperative morbidity.

REFERENCES
1. Burkitt DP, Walker AR, Painter NS. Effect of dietary fibre on stools
and the transit-times, and its role in the causation of disease. Lancet. 1972;7792:1408-1412.
2. Masoomi H, Buchberg BS, Magno C, Mills SD, Stamos MJ. Trends
in diverticulitis management in the United States from 2002 to
2007. Arch Surg. December 20, 2010. [Epub ahead of print.]
3. Jeyarajah S, Faiz O, Bottle A, Aylin P, Bjarnason I, Tekkis PP,
Papagrigriadis S. Diverticular disease hospital admissions are
increasing, with poor outcomes in the elderly and emergency
admissions. Aliment Pharmacol Ther. 2009;30:1171-1182.
4. Sandler RS, Everhart JE, Donowitz M, Adams E, Cronin K, Goodman C, Gemmen E, Shah S, Avdic A, Rubin R. The burden of
selected digestive diseases in the United States. Gastroenterology.
2002;122:1500-1511.
5. Rafferty J, Shellito P, Hyman NH, Buie WD. Standards Committee of American Society of Colon and Rectal Surgeons: Practice parameters for sigmoid diverticulitis. Dis Colon Rectum.
2006;49:939-44.
6. Siewert B, Tye G, Kruskal J, et al. Impact of CT-guided drainage in
the treatment of diverticular abscesses: size matters. Am J Roentgenol. 2006;186:680-686.
7. Kumar RR, Kim JT, Haukoos JS, et al. Factors affecting the successful management of intra-abdominal abscesses with antibiotics and the need for percutaneous drainage. Dis Colon Rectum.
2006;49:183-189.
8. Hinchey EF, Schaal PG, Richards GK. Treatment of perforated
diverticular disease. Adv Surg. 1978;12:85-109.

PMPH_CH23.indd 211

Findings

9. Stollman NH, Raskin JB. Diverticular disease of the colon. J Clin


Gastroenterol. 1999;29:241-252.
10. Zeitoun G, Laurent A, Rouffet F, Hay J, Fingerhut A, Paquet J,
Peillon C, Research TF. Multicentre, randomized clinical trial
of primary versus secondary sigmoid resection in generalized peritonitis complicating sigmoid diverticulitis. Br J Surg.
2000;87:1366-1374.
11. Finlay IG, Carter DC. A comparison of emergency resection and
staged management in perforated divertciular disease. Dis Colon
Rectum. 1987;30:929-933.
12. Stumpf MJ, Vinces FY, Edwards J. Is primary anastomosis safe
in the surgical management of complications of acute diverticulitis? Am Surg. 2007:73:787-790.
13. Blair NP, Germann E. Surgical management of acute sigmoid
diverticulitis. Am J Surg. 2002;183:525-528.
14. Schilling MK, Maurer CA, Kollmar O, Bchler MW. Primary
vs. secondary anastomosis after sigmoid colon resection for perforated diverticulitis (Hinchey Stage III and IV): A prospective
outcome and cost analysis. Dis Colon Rectum. 2001;44:699-703.
15. Richter S, Lindemann W, Kollmar O, Pistorius GA, Maurer CA,
Schilling MK. One-stage sigmoid colon resection for perforated
sigmoid diverticulitis (Hinchey stages III and IV). World J Surg.
2006;30:1027-1032.
16. Regenet N, Pessaux P, Hennekinne S, Lermite E, Tuech JJ, Brehant O, Arnaud JP. Primary anastomosis after intraoperative
colonic lavage vs. Hartmanns procedure in generalized peritonitis complicating diverticular disease of the colon. Int J Colorectal
Dis. 2003;18:503-507.
17. Aydin HN, Tekkis PP, Remzi FH, Constantinides V, Fazio
VW. Evaluation of the risk of a nonrestorative resection for the

5/21/2012 9:01:18 PM

212

18.

19.

20.

21.

22.

23.

24.

25.
26.

27.

28.

29.

30.

31.

32.

33.

34.
35.

Surgery: Evidence-Based Practice

treatment of diverticular disease: the Cleveland Clinic diverticular disease propensity score. Dis Colon Rectum. 2006;49:629-639.
Salem L. Primary anastomosis or Hartmanns procedure for
patients with diverticular peritonitis? A systematic review. Dis
Colon Rectum. 2004;47:1953-1964.
Constantinides VA, Tekkis PP, Athanasiou T, Aziz O, Purkayastha S, Remzi FH, Fazio VW, Aydin N, Darzi A, Senapati A.
Primary resection with anastomosis vs. Hartmanns procedure
in nonelective surgery for acute colonic diverticulitis: A systematic review. Dis Colon Rectum. 2006;49:966-981.
Franklin ME, Jr., Portillo G, Trevio JM, Gonzalez JJ, Glass JL.
Long-term experience with the laparoscopic approach to perforated diverticulitis plus generalized peritonitis. World J Surg.
2008;32:1507-1511.
White SI, Frenkiel B, Martin PJ.: A ten-year audit of perforated
sigmoid diverticulitis: Highlighting the outcomes of laparoscopic lavage. Dis Colon Rectum. 2010;53:1537-1541.
Myers E, Hurley M, OSullivan GC, Kavanagh D, Wilson I, Winter DC. Laparoscopic peritoneal lavage for generalized peritonitis due to perforated diverticulitis. Br J Surg. 2008;95:97-101.
Alamili M, Ggenur I, Rosenberg J. Acute complicated diverticulitis managed by laparoscopic lavage. Dis Colon Rectum.
2009;52:1345-1349.
Karoui M, Champault A, Pautrat K, Valleur P, Cherqui D, Champault G. Laparoscopic peritoneal lavage or primary anastomosis
with defunctioning stoma for Hinchey 3 complicated diverticulitis:
Results of a comparative study. Dis Colon Rectum. 2009;52:609-615.
Parks TG. Natural history of diverticular disease of the colon.
A review of 521 cases. Br Med J. 1969;4:639-645.
Makela J, Vuolio S, Kiviniemi H, Laitinen S. Natural History
of diverticular disease: When to operate? Dis Colon Rectum.
1998;41:1523-1528.
Broderick-Villa G, Bruchette RJ, Collins JC, et al. Hospitalization for acute diverticulitis does not mandate routine elective
colectomy. Arch Surg. 2005;140:576-581.
Somasekar K, Foster ME, Haray PN. The natural history of diverticular disease: Is there a role for elective colectomy? J R Coll Surg
Edinb. 2002;47:481-484.
Hall JF, Roberts PL, Ricciardi R, Read T, Scheirey C, Wald C,
Marcello PW, Schoetz DJ. Long-term follow-up after an initial
episode of diverticulitis: What are the predictors of recurrence?
Dis Colon Rectum. 2011;54:283-288.
Klarenbeek BR, Samuels M, van der Wal MA, van der Peet DL,
Meijerink WJ, Cuesta MA. Indications for elective sigmoid resection in diverticular disease. Ann Surg. 2010;251:670-674.
Chapman JR, Dozois EJ, Wolff BG, et al. Diverticulitis: A progressive disease? Do multiple recurrences predict less favorable
outcomes? Ann Surg. 2006;243:876-883.
Salem L, Veenstra DL, Sullivan SD, et al. The timing of elective
colectomy in diverticulitis: A decision analysis. J Am Coll Surg.
2004;199:904-912.
van de Wall BJ, Draaisma WA, Consten EC, van der Graaf Y,
Otten MH, de Wit GA, van Stel HF, Gerhards MF, Wiezer MJ,
Cense HA, Stockmann HB, Leijtens JW, et al. Dutch Diverticular
Disease (3D) Collaborative Study Group : DIRECT trial. Diverticulitis recurrences or continuing symptoms: Operative versus
conservative treatment. A multicenter randomised clinical trial.
BMC Surg. 2010;10:25.
Minardi AJ, Jr., Johnson LW, Sehon JK, Zibari GB, McDonald JC.
Diverticulitis in the young patient. Am Surg. 2001;67:458-461.
Lahat A, Menachem Y, Avidan B, Yanai H, Sakhnini E, Bardan
E, Bar-Meir S. Diverticulitis in the young patientis it different?
World J Gastroenterol. 2006;14:2932-2935.

PMPH_CH23.indd 212

36. Pautrat K, Bretagnol F, Huten N, de Calan L. Acute diverticulitis in very young patients: A frequent surgical management. Dis
Colon Rectum. 2006;50:472-477.
37. Anaya DA, Flum DR. Risk of emergency colectomy and colostomy in patients with diverticular disease. Arch Surg. 2005;140:
681-685.
38. Biondo S, Pars D, Mart Ragu J, Kreisler E, Fraccalvieri D, Jaurrieta E. Acute colonic diverticulitis in patients under 50 years of
age. Br J Surg. 2002;89:1137-1141.
39. Guzzo J, Hyman N. Diverticulitis in young patients: Is resection after a single attack always warranted? Dis Colon Rectum.
2004;47:1187-1190.
40. Nelson RS, Velasco A, Mukesh BN. Management of diverticulitis
in younger patients. Dis Colon Rectum. 2006;49:1341-1345.
41. Faria GR, Almeida AB, Moreira H, Pinto-de-Sousa J, Correia-daSilva P, Pimenta AP. Acute diverticulitis in younger patients:
Any rationale for a different approach? World J Gastroenterol.
2011;17:207-212.
42. Manousos O, Day NE, Tzonou A, Papadimitriou C, Kapetanakis A, Polychronopoulou-Trichopoulou A, and Trichopoulos D:
Diet and other factors in the aetiology of diverticulosis: An epidemiological study in Greece. Gut. 1985;26:544-549.
43. Aldoori WH, Giovannucci EL, Rimm EB, Wing AL, Trichopoulos DV, and Willet WC. A prospective study of diet and the
risk of symptomatic diverticular disease in men. Am J Clin Nutr.
1994;60:757-764.
44. Strate LL, Liu YL, Syngal S, Aldoori WH, Giovannucci EL. Nut,
corn, and popcorn consumption and the incidence of diverticular disease. JAMA. 2008;300:907-914.
45. Brodribb AJ. Treatment of symptomatic diverticular disease
with a high-fibre diet. Lancet. 1977;1:664-666.
46. Taylor I, Duthie HL. Bran tablets and diverticular disease. Br
Med J. 1976; 24:988-990.
47. Klarenbeek BR, Veenhof AA, Bergamaschi R, van der Peet DL,
van den Broek WT, de Lange ES, Bemelman WA, Heres P, Lacy
AM, Engel AF, Cuesta MA. Laparoscopic sigmoid resection for
diverticulitis decreases major morbidity rates: A randomized
control trial: short-term results of the Sigma Trial. Ann Surg.
2009;249:39-44.
48. Klarenbeek BR, Bergamaschi R, Veenhof AA, van der Peet DL,
van den Broek WT, de Lange ES, Bemelman WA, Heres P, Lacy
AM, Cuesta MA. Laparoscopic versus open sigmoid resection
for diverticular disease: Follow-up assessment of the randomized control Sigma trial. Surg Endosc. September 25, 2010. [Epub
ahead of print.]
49. Gonzalez R, Smith CD, Mattar SG, Venkatesh KR, Mason E,
Duncan T, Wilson R, Miller J, Ramshaw BJ. Laparoscopic vs
open resection for the treatment of diverticular disease. Surg
Endosc. 2004;18:276-280.
50. Alves A, Panis Y, Slim K, Heyd B, Kwiatkowski F, Mantion G.
Association Franais de Chirurgie: French multicentre prospective observational study of laparoscopic versus open colectomy
for sigmoid diverticular disease. Br J Surg. 2005;92:1520-1525.
51. Lawrence DM, Pasquale MD, Wasser TE. Laparoscopic versus
open sigmoid colectomy for diverticulitis. Am Surg. 2003;69:
499-503.
52. Dwivedi A, Chahin F, Agrawal S, Chau WY, Tootla A, Tootla F,
Silva YJ. Laparoscopic colectomy vs. open colectomy for sigmoid
diverticular disease. Dis Colon Rectum. 2002;45:1309-1314.
53. Senagore AJ, Duepree HJ, Delaney CP, Dissanaike S, Brady KM,
Fazio VW. Cost structure of laparoscopic and open sigmoid
colectomy for diverticular disease: Similarities and differences.
Dis Colon Rectum. 2002;45:485-490.

5/21/2012 9:01:18 PM

Diverticular Disease of the Colon

54. Gonzalez R, Smith CD, Mattar SG, Venkatesh KR, Mason E, Duncan T,
Wilson R, Miller J, Ramshaw BJ. Laparoscopic vs. open resection for
the treatment of diverticular disease. Surg Endosc. 2004;18:276-280.
55. Tuech JJ, Pessaux P, Rouge C, Regenet N, Bergamaschi R, Arnaud
JP. Laparoscopic vs open colectomy for sigmoid diverticulitis:

PMPH_CH23.indd 213

213

a prospective comparative study in the elderly. Surg Endosc.


2000;14:1031-1033.
56. Liberman MA, Phillips EH, Carroll BJ, Fallas M, Rosenthal R.
Laparoscopic colectomy vs traditional colectomy for diverticulitis. Outcome and costs. Surg Endosc. 1996;10:15-18.

5/21/2012 9:01:18 PM

Commentary on
Diverticular Disease of the Colon
Richard J. Mullins

Surgeons who manage patients with colonic diverticulitis encounter


a spectrum of clinical problems and they must select a therapy from
an array of therapies, which they conclude is optimal for an individual patient. Drs Izu and Ekeh provide an informative review of the
options available to surgeons who manage patients with diverticular disease of the colon. The authors emphasize several key points,
which they characterize in terms of available supporting evidence.
The majority of surgeons use an abdominal computed tomography (CT) scan to evaluate a patient with suspected diverticulitis.
Surgeons categorize the patient into four categories of severity, primarily based on the CT images. Drs Izu and Ekeh endorse the value
of the Hinchey scoring paradigm. Patients with minimal diverticulitis, Hinchey Stage I or II, are usually successfully treated with
antibiotics and procedural interventions are not required as the
majority of patients make an uneventful and permanent recovery.
Patients with a Hinchey Stage III or IV usually require an intervention to assist them in their recovery, and the surgeon should expect
a more complex problem and a prolonged recovery.
Drs Izu and Ekeh identify a study published in 2000 by
Zeitoun et al. as providing high-quality evidence. The Zeitoun et
al. paper concludes that outcome is superior in Hinchey Stage III
and IV diverticulitis if a Hartmann procedure is performed, presumably because with resection of the infected colon, the source
of sepsis is eliminated. However the authors point out that there is
a substantial body of lower-level evidence that supports the therapeutic value of percutaneous drainage of abscess. This procedure
has the obvious benefit of being performed by an interventional
radiologist using local anesthesia. Drs Izu and Ekeh reference several studies which report that series of patients acutely ill with
diverticulitis have been cured of their sepsis syndrome without
emergency surgery on their colon.

Drs Izu and Ekeh highlight that a surgeon planning emergency


surgery on a patient can select from several options. While surgeons
have used a formal laparotomy for decades as the means of controlling complicated acute diverticulitis, the authors of this chapter point
to a growing body of evidence that skilled laparoscopic surgeons
can manage the patient with Henchey Stage III or even IV diverticulitis with minimally invasive methods. Another treatment trend
supported by observational series of patients in high-volume colon
surgery centers is resection and primary anastamosis of disease-free
ends of the colon in patients having an emergency operation.
There is wide spread agreement among surgeons that antibiotics are usually an effective treatment for acute uncomplicated
colonic diverticulitis. Successful treatment leads to the question
should the patient have an elective colon resection, performed by
laparotomy or laparoscopy, combined with a primary colonic anastomosis. Drs Izu and Ekeh report favorable results that patients
who have one episode of acute uncomplicated diverticulitis can be
followed through two, three, or even four additional bouts before
elective surgical resection is needed. This recommendation is based
on the clinical experience published in manuscripts by many colon
surgeons that only a small minority of patients with diverticulosis
of the colon have recurrent bouts of infection. The authors of the
chapter prudently point out there is room for considerable clinical
judgment when deciding when to recommend surgical resection.
Operations on patients with diverticulitis can be a daunting task.
Surgeons planning to attempt resection of complicated diverticulitis
of the colon, or surgeons planning to take down a Hartmann procedure colostomy and perform an anastamosis should expect that they
may encounter a densely inflamed, scarred surgical field, where conventional dissection planes do not exist, and shortened mesenteries
severely restrict the ability to achieve a tension-free anastamosis.

214

PMPH_CH23.indd 214

5/21/2012 9:01:18 PM

CHAPTER
CHAPTER 24
1

Crohns Colitis and


Ulcerative Colitis
Kervin Arroyo and Barry Salky

INTRODUCTION

been estimated in population-based cohort studies that 20% to


40% of individuals with CD will develop fistulas over the lifetime
of their disease.4 Fistulas occur in 17% to 43% of the patients with
CD.5 According to a study in Minnesota, the risk of developing a
fistula is 33% at 10 years and 50% at 20 years.6
The clinical presentation of CD can be varied depending on
the pathophysiology of the disease. Diarrhea secondary to inflammation is the most common presentation followed by obstruction
secondary to fibro-stenotic disease. Fistulas can present in any
form, but internal fistulas are most common. Perianal disease is
common, and it is not seen in UC.
Mucosal ulceration, rectal bleeding, diarrhea, and abdominal pain characterize UC. Patients frequently have symptoms for
many months prior to the diagnosis. Obstruction and fistulas are
not part of the clinical presentation of UC. Unless fulminant colitis is present, fever is not a common presentation of UC.

Inflammatory bowel disease (IBD) includes Crohns disease (CD)


and ulcerative colitis (UC). Although there is a small percentage
of overlap in the clinical and pathologic manifestations of these
diseases, they tend to be clinically separate and distinct. The etiology of both CD and UC is unknown. UC involves the colon and
rectum, whereas CD can involve any portion of the gastrointestinal tract from the mouth to the anus. UC tends to involve the
rectum and spread proximally in the vast majority of the cases,
although relative rectal sparing does occur. It tends to be a continuous disease without skip areas. UC is a mucosal disease unless
it presents in a fulminant manner, at which time all layers of the
bowel wall can be involved. CD is a transmural inflammatory
disease that begins in the mucosa, but rapidly involves all layers
of the bowel wall. This explains the clinical manifestations of the
pathophysiologic process in which fistulas, abscesses, and fibrostenotic disease are commonly seen. To complicate matters even
more, in about 6% of the pathologic specimens (post op), features
of both CD and UC can be found. This has been termed as indeterminate colitis, and it has implications in terms of clinical management later.

2. What is the optimal medical management of patients


with CD?
Since the presentation of CD can be so varied, it is important to
compare treatments with similar manifestations of the disease.
Therefore, the Crohns Disease Activity Index (CDAI) was developed. The CDAI is based on the symptoms and is divided into four
different categories.7,8 A CDAI of <150 corresponds to patients who
are asymptomatic. A CDAI between 150 and 220 (mild to moderate
CD) corresponds to patients that ambulate and are able to tolerate an oral diet without dehydration. A CDAI between 220 and 450
(moderated to severe) corresponds to patients who have failed treatment or with symptoms such as fever, weight loss, abdominal pain,
and nausea or vomiting. The last category is the severe or fulminant
form (CDAI > 450). These patients have symptoms despite conventional treatment. They can present with high fevers, persistent vomiting, intestinal obstruction, or evidence of an abscess.8 Selected
trials of medical management of IBD are presented in Table 24.1.
Corticosteroids are effective for the induction of remission
in CD, but approximately 20% of patients are refractory to steroid therapy,9 particularly when used for more than 15 weeks.10

1. How do patients with IBD colitis present?


Patients with CD can manifest with one or more different symptoms such as abdominal pain, bleeding, abscess, and fistulas.
Patients can have symptoms sometimes for many years prior to
the diagnosis.1-3 This was illustrated in a series that included 66
patients with IBD of whom 45 had CD and 21 had UC.2 Nonspecific digestive symptoms resembling irritable bowel syndrome
were present for an average of 7.7 years prior to the diagnosis of
CD, which was significantly longer than for UC (average 1.2 years).
Symptoms were present for the longest amount of time in patients
with disease limited to the colon compared with the small bowel
(average 11.4 vs. 4.9 years).2
Physical examination will frequently demonstrate an inflammatory mass most commonly in the right lower quadrant. It has
215

PMPH_CH24.indd 215

5/21/2012 9:09:06 PM

216

Surgery: Evidence-Based Practice

However, other studies have shown that corticosteroids are not


effective for maintenance of remission in CD.11
Sulfasalazine is commonly used in CD but the efficacy is
not well supported by randomized controlled trials (RCTs).12,13 A
meta-analysis of three double-blind RTCs of mesalamine (Pentasa
4 g daily) in active CD found that there was an overall mean
reduction in the CDAI of 67 points, about 25 points better than
with placebo. However, this decrease in the CDAI represents only
a modest clinical benefit and is mainly seen after 4 weeks of treatment.13 A meta-analysis published by Hanauer et al. failed to demonstrate a significant benefit of these drugs.13 Also, mesalamine
reduces the risk of clinical recurrence of CD.46
Azathioprine is an immunosuppressant that inhibits T lymphocytes. Several RCTs have demonstrated the efficacy of azathioprine and mercaptopurine as induction treatment of CD.
This Cochrane review reported an overall response rate of 54%
for active CD compared with a placebo response rate of approximately 33%.14 Azathioprine reduces the risk of clinical recurrence
of CD.46
Methotrexate is a dihydrofolate reductase inhibitor that suppresses the bodys natural immune responses.17 A review performed by Cochrane collaboration in 2009 included three RCTs.16
Of the three studies, one study demonstrated that intramuscular methotrexate at doses of 15 mg/week was more effective than
placebo for the maintenance of remission of CD. Another one
demonstrated no difference between methotrexate and 6-MP for
maintenance of remission in CD.16 A randomized, double-blind
study by Feagan et al.17 showed that relapse was more frequent
in the placebo group versus the methotrexate group, 61% versus 35%, respectively. Mat and Jimnez published a study of 28
patients with CD who achieved remission in about 80% of the
patients taking methotrexate.16 Feagan et al. published a series of
CD patients in 2000. In 35 of 40 patients, the remission was maintained at 40 weeks of treatment.16,17 He concluded that in Crohns
patients who had been induced into remission with methotrexate,
more patients remained in remission while taking intramuscular
methotrexate compared with placebo.17 The MatJimnez et al.
and Oren et al. studies suggest that methotrexate is safe, but failed
to show a benefit for lower doses given orally.16,18 Methotrexate at a
dose of 25 mg intramuscularly every weekly was linked to induce
remission and complete withdrawal from steroids in patients with
refractory CD. There is evidence that methotrexate reduces the
need for steroid treatment (steroid sparing effect).15
The chimeric monoclonal antibody (cA2) was supported in
this study of 108 patients with CD resistant to treatment.1 A single
infusion of cA2 was an effective short-term treatment in many
patients with moderate to severe, treatment-resistant CD.1
Infliximab is a chimeric IgG1 monoclonal antibody that acts
against tumor necrosis factor alpha (TNF-). This medication is
effective and has played a significant role in the treatment of CD.4
This RCT evaluated the effects of maintenance with infliximab
and compared the reduction in hospitalizations, surgeries, and
procedures. They randomized a total of 282 patients at week 14
as responders. The 5 mg/kg infliximab group had a reduction of
>50% in hospitalizations when compared with the placebo group.
In terms of surgeries, the 5 mg/kg infliximab group had a reduction of >50% in surgeries when compared with the placebo group,
60 versus 118 patients, respectively.4
An analysis of patients from the ACCENT phase III clinical trial demonstrated that when patients entered in remission
with infliximab, they had fewer hospitalizations, surgeries, and

PMPH_CH24.indd 216

improved quality of life.19 Patients with CD who respond to an initial dose of infliximab are more likely to be in remission at weeks
30 and 54 if infliximab treatment is maintained every 8 weeks.19
In another study, Sands et al. performed an RCT of 306 patients
with CD and draining fistulas. Of the 306 patients, 195 responded
to the drug and 87 patients did not. All the patients were randomized to receive placebo or infliximab every 8 weeks and followed
up at week 54. The time to loss response was longer for patients
who received infliximab maintenance therapy than the placebo
group. Nineteen percent of the patients in the placebo group and
36% in the treatment group had a complete absence of draining
fistulas.20
In another study by Present et al., 58% to 68% of patients
achieved the reduction of 50% or more from base line in the number of draining fistulas observed at two or more consecutive study
visits. During the same period of time, this was achieved in 26%
of the patients in the placebo group. Closure of the fistulas was
achieved in 55% and 38% of the patients who received infliximab
5 and 10 mg/kg, respectively. These fistulas remained closed for
a mean time of 3 months.21 According to all these reviews, there
is extensive evidence supporting the utility of infliximab as an
induction agent for CD.
Adalimumab is a subcutaneously administered immunoglobulin G1 monoclonal antibody that binds with high affinity to
human TNF and modulates its biologic drug. Adalimumab was
significantly more effective than placebo in maintaining remission
in moderate to severe CD through 56 weeks.2 The fistula closure
was higher and the exacerbation of CD was lower.2 Adalimumab
was superior to placebo for induction of remission in patients with
moderate to severe CD naive to anti-TNF therapy. The optimal
dose regimen for adalimumab in this study was 160 mg.3
Natalizumab is an IgG4 monoclonal antibody and a selective adhesion molecule (SAM) inhibitor.22 The data of this review
showed that natalizumab was effective for induction of clinical
response and remission in patients with moderately to severely
active CD.22 The dose of natalizumab used in this study was 300
mg or 3 to 4 mg/kg. These studies also support the results that
natalizumab may be an effective treatment for active CD.23,25
Certolizumab pegol is a monoclonal antibody that has a binding affinity for TNF-, which does not induce apoptosis of T cells
or monocytes. A RCT study26 of 668 patients at 147 centers, the
induction therapy of 400 mg of certolizumab pegol was administered at 0, 2, and 4 weeks. Sixty-four percent of the patients
responded to induction therapy, and the response was maintained
through week 26 in 62% of the patients. At the end of the study,
the patients that responded to induction therapy were more likely
to maintain response and remission at week 26.26 Another RCT
performed by Sandborn et al.6 included 662 patients with at least 3
months with moderate to severe CD in 171 centers. The treatment
with certolizumab was associated with a modest benefit in the
response at weeks 6 and 26, but the rate of remission was not.6
3. When is surgical intervention required in the management
of CD?
Surgery is necessary to treat some complications of the disease.
Selected trials of surgical management of IBD are presented in
Table 24.2. A prospective study evaluating the natural history of CD
showed that 33% of patients with chronically or intermittently active
disease developed complications requiring hospitalization and surgery in the first year after diagnosis, 13% in the second year, and 3%

5/21/2012 9:09:07 PM

Crohns Colitis and Ulcerative Colitis

in each subsequent year.27 Surgical treatment is required in approximately 70% of patients.28 The need for operation is determined by the
interaction among the gastroenterologist, surgeon, and the patient.
Patients have better outcome when they are well-prepared medically,
psychologically, and had surgery in a timely fashion.
4. What are the advantages of laparoscopic approach in CD
An early case comparison feasibility study in 1999 demonstrated a
significant decrease in the length of stay (LOS) with less complications in the laparoscopic group compared with the open group.31
There is a clear advantage to decreased adhesions and smaller
incisions in these patients with laparoscopic surgery.29,30 This RCT
followed up 60 patients after elective ileocolic resection for refractory, noncomplicated CD. They concluded that laparoscopic techniques offered a faster recovery of pulmonary function, fewer
complications, and shorter LOS compared with conventional
surgery in these patients. These were all statistically significant.32

217

A 10-year follow-up study confirmed that laparoscopic ileocolic


equals open ileocolic resection with the same incidence of recurrent disease. There were significantly less incisional hernias in the
laparoscopic group.33 This meta-analysis study published in 2008
showed statistically significant benefit in return of bowel function,
oral intake, LOS, morbidity, and equal recurrence in the laparoscopic groups.35 This Cochrane review demonstrates that the laparoscopic approach reduces the operative blood loss, allows quicker
return of bowel function, and shorter LOS.47
We need to do a better job at training surgeons to use laparoscopic surgery for CD. In a population-based analysis of laparoscopic versus open approaches to CD, there were 396,911 admissions
for CD and 49,609 surgeries from 2000 to 2004. Only 2826 surgeries (6%) were performed laparoscopically even though there was a
statistically significant lower complication rate, shorter LOS, lower
charges, and lower mortality.36 Yet, the vast majority of surgeries for
CD are being performed in an open manner.

TABLE 24.1 Selected trials of medical management of IBD


Author (ref)

Study
Type

Treatment

Clinical Response

Comments

Colombel (2)

RCT

261
260
257

Placebo adalimumab 40 mg,


eowadalimumab 40 mg
weekly

28 (16.5%)
71 (41.3%)
75 (47.8%)

Clinical response;
decrease CDAI Score
100 at week 56

Hanauer (3)

RCT

74
75
76

Placebo adalimumab 80 mg,


adalimumab 160 mg

9/74 (12%) remission


18/75 (24%) remission
27/76 (36%) remission

Clinical response;
remission rates

Lichtenstein (4)

RCT

143
139

Placebo infliximab 5mg/kg

45/143 (31%)
19/139 (14%)

Number of
hospitalizations

Hanauer (19)

RCT

110
113
112

Placebo infliximab 5 mg/kg,


Infliximab 10 mg/kg

23/110 (21%)
44/113 (39%)
50/112 (45%)

Clinical response;
remission at 2 weeks

Sands (20)

RCT

143
139

Placebo infliximab 5 mg/kg

27/143 (19%)
50/139 (36%)

Clinical response;
complete absent of
draining fistulas

Ghosh (23)

NRCT

63
68
66
51

Placebo infusion
natalizumab 3 mg/kg2
infusions natalizumab
3 mg/kg2 infusions
natalizumab 6 mg/kg

Remission rate 27%


response 43%
Remission rate 28%
response 50%
Remission rate 42%
response 61%
Remission rate 39%
response 65%

Clinical remission; score


of <150 on the CDAI

Sandborn (25)

NRCT

18
17
24

Placebo 300 mg of
natalizumab

Remission rate 26%


response 28%
Remission rate 44%
response 61%

Response; decrease
(CDAI) score of at
least 70 points

Rutgeerts (27)

RCT

Placebo infliximab
5mg/kg Infliximab
10 mg/kg Placebo
infliximab 5mg/kg
Infliximab 10 mg/kg

37% clinical response


UC69% clinical
response UC61%
clinical response
UC29% clinical
response UC64%
clinical response
UC69% clinical
response UC

Clinical Response;
decrease in the
Mayo score of at
least 3 points and
at least 30%

ACT
1121121122
ACT
2123121120

NRCT = no randomized control trial.

PMPH_CH24.indd 217

5/21/2012 9:09:07 PM

218

Surgery: Evidence-Based Practice

TABLE 24.2 Selected Trials of Surgical Management of IBD


Author (ref)

Study

Treatment

Clinical Response

Ali (28)

NRCT

253354

Laparoscopic IPAA Open IPAA

Lap Group had a longer OR time with


higher cosmesis scores.

Milsom (32)

NRCT

29 31

Open ileocolic for CD Laparoscopic


ileocolic for CD

Lap group had a faster recovery


of pulmonary function, fewer
complications, and shorter LOS.

Stocchi (33)

NRCT

2927

Open ileocolic for CD Laparoscopic


ileocolic for Crohns

Lap ileocolic is at least comparable to


OC in the treatment of ileocolic
CD.

Wu (34)

MA

16 Trials 396527

Laparoscopic surgery for UC Open


surgery for UC

Lap surgery for UC had a better


postoperative fasting time,
postoperative hospital stay, and
overall complication rate.

TAN (35)

MA

286595

Laparoscopic Surgery for CD Open


Surgery for CD

Laparoscopic surgery for CD takes


longer to perform and lower
morbidity.

Maartense (42)

NRCT

3030

Lap hand-assisted Restorative


Proctocolectomy with IPAA Open
Restorative Proctocolectomy With
IPAA

Quality of life questionnaires were


comparable for lap hand-assisted
laparoscopic vs open restorative
proctocolectomy with IPAA.

Polle (43)

NRCT

3030

Lap hand-assisted proctocolectomy


Open restorative proctocolectomy

Open proctocolectomy has a negative


impact on body image and cosmesis.
Lap group had longer operating
times and higher costs.

NRCT = no randomized control trial, MA = meta-analysis.

5. What is the optimal medical management of patients with UC?


The medical management of UC has been mainly 5-aminosalicylates,
corticosteroids, and immunosuppressants. High-dose intravenous steroid therapy forms the basis of pharmacologic treatment
for acute UC.37 If this fails, immunosuppressive medications can
be used. The second line of treatment consists of cyclosporine A,
with a switch to azathioprine for maintenance of therapy.37
Infliximab is a chimeric IgG1 monoclonal antibody that acts
against TNF-.. Two RCTs in the treatment of UC were presented
in this paper.27 There were 364 patients in each group with moderate to severe active UC. They divided the patients into two groups,
placebo and infliximab. The infliximab group was subdivided into
two groups based on 5 and 10 mg/kg of body weight dose. They
administered the medication at 0, 2, 6, and 8 weeks. In the first
trial, 69% of the patients who received 5 mg of infliximab and
61% of those who received 10 mg had a clinical response at week 8
compared with 37% of those who received placebo.27 In the second
RCT, 64% of the patients who received 5 mg of infliximab and
69% of those who received 10 mg had a clinical response at week 8
compared with 29% of those who received placebo.27 The response
of treatment at week 54 was 45% for the infliximab group versus
20% for the placebo group.
This multicenter RCT of 45 patients demonstrated that infliximab reduced the need for colectomy compared with placebo
(29% vs. 67%).38 The ACT Trials published that infliximab was
associated with improved health-related quality of life scores.38
Does infliximab affect the surgical outcome after operation? This
case-matched study compared only 13 patients; the study showed

PMPH_CH24.indd 218

no adverse impact from the previous infliximab therapy on the


laparoscopic ileal pouch anal anastomosis (IPAA) postoperative
course; that is, mean operative time (353 vs. 355 min), complication rate (23% vs. 38%), and mean hospital stay (22 vs. 25 days).37
6. What are the advantages of laparoscopic approach in UC?
The open approach has been the standard of care for the surgical
treatment of the UC. All newer laparoscopic approaches have to
judge against the open approach. The transformation to laparoscopic surgery for UC has been slow. However, with the development
of newer energy devices and the extensive experience being gained
with the laparoscopic approach to UC, more and more patients are
being offered this type of surgery.39 Elective laparoscopic proctocolectomy for UC is safe and provides many short-term benefits in
both the acute and chronic setting.39-41 The reasons why it is not
being used more frequently are multiple, and they include technical
expertise, complicated intestinal surgery in sick patients, multiple quadrant surgery, and time constraints. High-dose steroids and
other immunosuppressant medications make for technically challenging surgery as well. While short-term advantage has been demonstrated (decreased LOS and cosmetics), long-term postoperative
quality of life has been comparable to open cohorts.42
In terms of urgent cases, there are only a small number of reports
that have studied the laparoscopic approach in the setting of severe
colitis. This study demonstrated that in the urgent setting, the laparoscopic approach is associated with the short-term benefits of faster
return of bowel function, decreased inpatient narcotic requirement,
and shorter LOS compared with the open approach.39 The patients

5/21/2012 9:09:07 PM

Crohns Colitis and Ulcerative Colitis

that underwent a laparoscopic colectomy progress to the second and


third operative stages of restorative proctocolectomy faster than those
patients who underwent open colectomy.39 The short- and long-term
results of the laparoscopic approach were compared in this study of
73 patients. They concluded that laparoscopic IPAA offers significant
advantages over the open conventional procedure in terms of body
image and cosmesis.43,44 The postoperative return of bowel function
and analgesic requirements after laparoscopic IPAA has been less
concordant. They observed a faster return of bowel function; that is,
passing flatus and tolerating liquid diet, in the laparoscopic group.45
In this meta-analysis, they evaluated 16 controlled trials that
include 923 patients. They found that laparoscopic surgery compared with open surgery (39.3% vs. 54.8%) had a longer operative

219

time (weighted mean difference 69.29 min). As to recovery of bowel


function, peritoneal abscess, anastomotic leakage, postoperative
bowel obstruction, wound infection, blood loss, and mortality, laparoscopic surgery did not show any superiority over open surgery.
They concluded that further evaluation with more well-designed
and long-term follow-up studies is required.34
This Cochrane study evaluated the effects of laparoscopic versus open ileal pouch anal anastomosis for patients with UC and
familial adenomatous polyposis. They included 11 trials for a total
of 607 patients. No significant differences were found in mortality,
complications, reoperation, and readmission. The operative time
was significantly longer in the laparoscopic group. The cosmesis
scores were higher in the laparoscopic group.28

Clinical Question Summary


Question

Answer

1 How do patients with IBD


colitis present?

Symptoms such as abdominal pain, bleeding, abscess, and


fistulas in CD. Mucosal ulceration, rectal bleeding, diarrhea,
and abdominal pain characterize UC.

1-3

2 What is the optimal medical


management of patients
with CD?

Corticosteroids can induct remission, but 20% of patients


exhibit refractoriness to therapy. Infliximab reduced by
>50% the need for surgery and improved QOL. Patients
achieved the reduction of 50% or more from base line in
the number of draining fistulas. Natalizumab is effective for
induction of clinical response and remission in patients with
moderately to severely active CD. Adalimumab is effective
in maintaining remission in moderate to severe CD. The
optimal dose regimen for adalimumab was 160 mg.

2-4, 10, 11,


19-23, 25

3 When is surgical intervention


required in the management
of CD?

Surgery is necessary to treat some complications in patients


with chronically or intermittently active disease.

28

4 What are the advantages of


laparoscopic approach in CD?

With the laparoscopic approach, the patients have a faster


recovery of pulmonary function, faster return of bowel
function, shorter LOS, less adhesions, and less incisional
hernias.

32, 33, 35

5 What is the optimal medical


management of patients
with UC?

Infliximab is an effective treatment for UC. About 69% of the


patients who are treated had a clinical response. Reduced
the need for colectomy compared with placebo. It was
associated with improved health-related quality of life
scores.

27, 38

6 What are the advantages of


laparoscopic approach in UC?

Short-term advantage has been demonstrated to have a faster


return of bowel function, decreased inpatient narcotic
requirement, and better body image.

28, 34, 42, 43

REFERENCES
1.

2.

3.

PMPH_CH24.indd 219

Targan SR, Hanauer SB, van Deventer SJ, et al. A short-term study
of chimeric monoclonal antibody cA2 to tumor necrosis factor
alpha for Crohns disease. Crohns Disease cA2 Study Group.
N Engl J Med. 1997;337:1029-1035.
Colombel JF, Sandborn WJ, Rutgeerts P, et al. Adalimumab for maintenance of clinical response and remission in patients with Crohns
disease: The CHARM trial. Gastroenterology. 2007;132:52-65.
Hanauer SB, Sandborn WJ, Rutgeerts P, et al. Human anti-tumor
necrosis factor monoclonal antibody (adalimumab) in Crohns
disease: The CLASSIC-I trial. Gastroenterology. 2006;130:323-333.

Grade

4.

5.

6.
7.
8.

References

Lichtenstein GR, Yan S, Bala M, et al. Infl iximab maintenance


treatment reduces hospitalizations, surgeries, and procedures in
fistulizing Crohns disease. Gastroenterology. 2005;128:862-869.
Schwartz DA, Pemberton JH, Sandborn WJ. Diagnosis and
treatment of perianal fistulas in Crohns disease. Ann Intern Med
2001;135:906-918.
Sandborn WJ, Feagan BG, Stoinov S, et al. Certolizumab pegol for
the treatment of Crohns disease. N Engl J Med. 2007;357:228-238.
Harvey RF, Bradshaw JM. A simple index of Crohns-disease
activity. Lancet. 1980;1(8167):514.
Lichtenstein GR, Hanauer SB, Sandborn WJ. Management of
Crohns disease in adults. Am J Gastroenterol. 2009;104:465.

5/21/2012 9:09:07 PM

220

Surgery: Evidence-Based Practice

9. McDonald JW, Feagan BG, Jewell D, et al. Cyclosporine for


induction of remission in Crohns disease. Cochrane Database of
Systematic Reviews. 2005:CD000297.
10. Benchimol E, Seow Ch, Steinhart A, Griffiths, A. Traditional
corticosteroids for induction of remission in Crohns disease.
Cochrane Database of Systematic Reviews. 2008.
11. Steinhart AH, Ewe K, Griffiths AM, et al. Corticosteroids for
maintenance of remission in Crohns disease. Cochrane Database of Systematic Reviews. 2003.
12. Sandborn WJ, Feagan BG. Review article: Mild to moderate
Crohns disease--defining the basis for a new treatment algorithm. Aliment Pharmacol Ther. 2003 Aug 1;18(3):263-277.
13. Hanauer SB, Stromberg U. Oral Pentasa in the treatment of
active Crohns disease: A meta-analysis of double-blind, placebocontrolled trials. Clin Gastroenterol Hepatol. 2004;2:379.
14. Prefontaine E, Macdonald JK, Sutherland LR. Azathioprine or
6-mercaptopurine for induction of remission in Crohns disease.
Cochrane Database of Systematic Reviews. June 16, 2010;6.
15. Alfadhli AA, McDonald JW, Feagan BG. Methotrexate for
induction of remission in refractory Crohns disease. Cochrane
Database of Systematic Reviews. 2004.
16. Patel V, Macdonald JK, McDonald JW, et al. Methotrexate for
maintenance of remission in Crohns disease. Cochrane Database of Systematic Reviews. 2009.
17. Feagan BG, Rochon J, Fedorak RN, et al.Methotrexate for the
treatment of Crohns disease. The North American Crohns
Study Group Investigators. N Engl J Med. 1995;332(5):292-297.
18. Oren R, Moshkowitz M, Odes S, et al. Methotrexate in chronic
active Crohns disease: A double-blind, randomized, Israeli multicenter trial. Am J Gastroenterol. 1997;92:2203-2209.
19. Hanauer SB, Feagan BG, Lichtenstein GR, et al. Maintenance
infliximab for Crohns disease: The ACCENT I randomised trial.
Lancet. 2002;359(9317):1541-1549.
20. Sands BE, Anderson FH, Bernstein CN, et al. Infl iximab maintenance therapy for fistulizing Crohns disease. N Engl J Med.
2004;350:876-885.
21. Present DH, Rutgeerts P, Targan S, et al. Infliximab for the treatment of fistulas in patients with Crohns disease. N Engl J Med.
1999;340:1398-1405.
22. MacDonald JK, McDonald JW. Natalizumab for induction of
remission in Crohns disease. Cochrane Database of Systematic
Reviews. 2007:CD006097.
23. Ghosh S, Goldin E, Gordon FH, et al. Natalizumab for active
Crohns disease. N Engl J Med. 2003;348(1):24-32.
24. Gordon FH, Lai CW, Hamilton MI, et al. A randomized placebocontrolled trial of humanized monoclonal antibody to
alpha4 integrin in active Crohns disease. Gastroenterology.
2001;121(2):268-274.
25. Sandborn WJ, Colombel JF, Enns R, et al. Natalizumab induction and maintenance therapy for Crohns disease. N Engl J Med.
2005;353(18):1912-1925.
26. Schreiber S, Khaliq-Kareemi M, Lawrance IC, et al. Maintenance
therapy with certolizumab pegol for Crohns disease. N Engl J
Med. 2007;357:239-250.
27. Rutgeerts P, Sandborn WJ, Feagan BG, et al. Infliximab for
induction and maintenance therapy for ulcerative colitis. N Engl
J Med. 2005;353:2462-2476.
28. Ahmed Ali U, Keus F, Heikens JT, et al. Open versus laparoscopic
(assisted) ileo pouch anal anastomosis for ulcerative colitis and
familial adenomatous polyposis (Review). Cochrane Database of
Systematic Review. January 21, 2009;1:CD006267.
29. Naidu MN, Trang AC, Salky BA. Laparoscopy in Crohns disease. Clin Colon Rectal Surg. 2007;20(4):329-335.

PMPH_CH24.indd 220

30. Indar AA, Efron JE, Young-Fadok TM. Laparoscopic ileal pouchanal anastomosis reduces abdominal and pelvic adhesions. Surg
Endosc. 2009;23(1):174-177.
31. Canin-Endres J, Salky B, Gattorno F, Edye M. Laparoscopically
assisted intestinal resection in 88 patients with Crohns disease.
Surg Endosc. 1999;13(6):595-599.
32. Milsom JW, Hammerhofer KA, Bhm B, et al. Prospective,
randomized trial comparing laparoscopic vs. conventional surgery for refractory ileocolic Crohns disease. Dis Colon Rectum.
2001;44(1):1-8.
33. Stocchi L, Milsom JW, Fazio VW. Long-term outcomes of
laparoscopic versus open ileocolic resection for Crohns disease: Follow-up of a prospective randomized trial. Surgery.
2008;144(4):622-627; discussion 627-628.
34. Wu XJ, He XS, Zhou XY, et al. The role of laparoscopic surgery
for ulcerative colitis: Systematic review with meta-analysis. Int J
Colorectal Dis. 2010;25:949-957.
35. Tan JJ, Tjandra JJ. Laparoscopic surgery for Crohns disease:
A meta-analysis. Dis Colon Rectum. 2007;;50(5):576-585.
36. Lesperance K, Martin MJ, Lehmann R, et al. National trends and
outcomes for the surgical therapy of ileocolonic Crohns disease:
A population-based analysis of laparoscopic vs. open approaches.
J Gastrointest Surg. 2009;13(7):1251-1259.
37. Coquet-Reinier B, Berdah SV, Grimaud JC, et al. Preoperative
infliximab treatment and postoperative complications after laparoscopic restorative proctocolectomy with ileal pouch-anal anastomosis: A case-matched study. Surg Endosc. 2010;24(8):1866-1871.
38. Wilhelm SM,. McKenney KA, Rivait KN, et al. A Review of Infliximab Use in Ulcerative Colitis. Clin Ther. 2008;30:223-223.
39. Chung TP, Fleshman JW, Birnbaum EH, et al. Laparoscopic vs.
open total abdominal colectomy for severe colitis: Impact on
recovery and subsequent completion restorative proctectomy.
Dis Colon Rectum. 2009;52(1):4-10.
40. Larson DW, Cima RR, Dozois EJ, et al. Safety, feasibility, and
short-term outcomes of laparoscopic ileal pouch-anal anastomosis: A single institutional case-matched experience. Ann Surg.
2006;243:667-670.
41. Marcello PW, Milsom JW, Wong SK, et al. Laparoscopic restorative
proctocolectomy: Case-matched comparative study with open
restorative proctocolectomy. Dis Colon Rectum. 2000;43:604-608.
42. Maartense S, Dunker MS, Slors JF, et al. Hand-assisted laparoscopic versus open restorative proctocolectomy with ileal pouch
anal anastomosis: A randomized trial. Ann Surg. 2004;240:
984-991.
43. Polle SW, Dunker MS, Slors JF, et al. Body image, cosmesis, quality of life, and functional outcome of hand-assisted laparoscopic
versus open restorative proctocolectomy: Long-term results of a
randomized trial. Surg Endosc. 2007;21(8):1301-1307.
44. Dunker MS, Bemelman WA, Slors JF, van Duijvendijk P, Gouma
DJ. Functional outcome, quality of life, body image, and cosmesis in patients after laparoscopic-assisted and conventional
restorative proctocolectomy: A comparative study. Dis Colon
Rectum. 2001;44(12):1800-1807.
45. Fichera A, Silvestri MT, Hurst RD, et al. Laparoscopic restorative
proctocolectomy with ileal pouch anal anastomosis: A comparative observational study on long-term functional results. J Gastrointest Surg. 2009;13(3):526-532.
46. Doherty G, Bennett G, Patil S, et al. Interventions for prevention
of post-operative recurrence of Crohns disease. Cochrane Database of Systematic Reviews. October 7, 2009;4.
47. Umanskiy K, Malhotra G, Chase A, et al. Laparoscopic colectomy for Crohns colitis. A large prospective comparative study.
J Gastrointest Surg. 2010;14(4):658-663.

5/21/2012 9:09:07 PM

CHAPTER
CHAPTER 25
1

Large Bowel Obstruction


Ryan A. Lawless, Dale A. Dangleben, and Michael M. Badellino

INTRODUCTION

the passage of flatus (90%) and/or feces (80.6%) and abdominal distention (65.3%). The most common etiologies noted for both small
and large bowel obstructions were adhesions, incarcerated hernias,
and large bowel malignancy (64.8%, 14.8%, and 13.4%, respectively).
The most common causes for large bowel obstruction were large
bowel malignancy, adhesions, retroperitoneal tumors, and hernias
in 47.4%, 36.3%, 5.5%, and 2.7%, respectively.1

Large bowel obstructions are a diagnostic as well as management


challenge for even the most experienced surgeon. This is due to
the multitude of possible diagnoses and physiologic derangements
with which these patients present. Maneuvering through the
anatomy, diagnosis, management, and operative technique with
care and due diligence will allow the surgeon to adequately management this most challenging entity. There remains a paucity of
Level 1 evidence in the literature to assist with surgical decisionmaking even today. Recent articles were reviewed to support the
recommendations made in the discussion to follow.

COLON CANCER

What is the preferred operative approach?

In approximately 10% of patients, colon cancer presents as a large


bowel obstruction.2 Obstruction is most likely to present in the sigmoid colon (75%), followed by ascending colon (10%), transverse
colon (5%), and descending colon (5%). Of patients who presented
with obstruction, 20% had ischemia, 15% had necrosis, and 10% had
perforation. In this subset of patients with mechanical large bowel
obstruction, 16.6% were intraoperatively diagnosed with ischemia,
16.6% with necrosis, and 11.1% with perforation. The incidence of
necrosis and perforation was significantly greater in patients with
large bowel obstruction than with small bowel obstruction.1

What is the role of laparoscopy in the treatment of large bowel


obstruction?

HERNIAS

Clinical Questions
How do patients with large bowel obstruction present?
What are the most common causes of large bowel obstruction?
What is the proper diagnostic evaluation?

What is the role of colonic stenting?


Hernias cause <3% of large bowel obstructions. Although infrequent, large bowel obstruction caused by hernia is more likely to
be associated with ischemia, necrosis, and perforation.1 The presence of skin erythema or laboratory data to suggest bowel ischemia, including leukocytosis, lactic acidosis, or metabolic acidosis
suggest bowel compromise and should prompt early operative
intervention.

Clinical Presentation
1. How do patients with large bowel obstruction present?
The presentation of large bowel obstruction depends on the degree of
intestinal luminal narrowing and the duration of obstruction. There
is very little evidence in the reviewed literature to characterize the
presenting symptoms of acute large bowel obstruction. The underlying etiology of the obstruction often dictates the presentation. In a
prospective observational study of 150 patients admitted with acute
mechanical bowel obstruction, it was reported that 36 patients
(24%) had a large bowel obstruction. Presenting symptoms and
physical examination findings most commonly included absence of

COLONIC VOLVULUS
Colonic volvulus is responsible for 10% to 15% of large bowel
obstructions. In a recent review, it was noted that 76% of cases
occurred in the sigmoid colon and in older patients. Risk
221

PMPH_CH25.indd 221

5/22/2012 6:30:52 PM

222

Surgery: Evidence-Based Practice

factors for sigmoid volvulus are high fiber diet, African American
heritage, institutionalization, laxative abuse, previous abdominal
surgery, pregnancy, Chagas disease, Hirschsprungs disease, and
Parkinsonism.2
Cecal volvulus accounts for approximately 22% of cases.
Overall, cecal volvulus tends to occur in younger patients that
may suggest a congenital and abnormal attachment of the right
colon to the abdominal wall.2

INTRINSIC TO THE BOWEL WALL


Neoplasms within the colonic wall account for around 47% of cases.1 Other intrinsic causes include strictures secondary to Crohns
colitis, ulcerative colitis, chronic ischemia, diverticular disease,
or external radiation. Endometriosis has also been implicated as
being an intrinsic cause of large bowel obstruction.9

INTRALUMINAL LESIONS
ACUTE COLONIC
PSEUDO-OBSTRUCTION
(OGILVIES SYNDROME)
Acute colonic pseudo-obstruction (ACPO) is secondary to a
number of metabolic and physiologic causes. It is characterized
by abdominal distention, pain (80%), nausea with or without
vomiting (60%), and obstipation (40%) in the absence of mechanical obstruction. Synonyms include adynamic ileus, acute colonic
ileus, and Ogilvies Syndrome.3-6 The pathophysiologic basis of
ACPO is multifactoral.

INFREQUENT CAUSES
Other, less frequent, causes of large bowel obstruction include
stool impaction, strictures from inflammatory bowel disease, or
chronic diverticulitis, and acute diverticulitis with extracolonic
abscesses.
In summary, the most common clinical symptoms associated
with large bowel obstruction are the absence of flatus and/or feces,
and abdominal distention. A significant number of patients with
large bowel obstruction will also have colonic ischemia, necrosis,
and/or perforation.1 (Grade C recommendation)
Differential Diagnosis
2. What are the most common causes of large bowel obstruction?
There was no Level 1 evidence identified to establish succinct diagnostic or treatment algorithms. There are a wide range of etiologies
of large bowel obstruction. Thus, evaluation and treatment algorithms can be complex.
In 2000, Jenkins et al. retrospectively reviewed 73 patients
with secondary causes of small and large bowel obstruction. The
etiology of obstruction was metastatic neoplastic process (19%),
colonic volvulus (17%), Crohns disease (14%), hernia (11%), and
diverticular disease (7%).7 A comprehensive outline classification
scheme is illustrated in Current Therapy in Colon and Rectal Surgery.8 This simple outline divides the causes into extrinsic to the
bowel wall, intrinsic to the bowel wall, and intraluminal lesions.

EXTRINSIC TO THE BOWEL WALL


Lesions extrinsic to the bowel wall lead to external compression
of the colonic lumen. Possible extrinsic lesions include neoplastic
growth from an adjacent organ or, more frequently, a retroperitoneal tumor.1 As with small bowel obstruction, the two other main
extrinsic causes are hernias and adhesions.

PMPH_CH25.indd 222

Intraluminal lesions come in the form of iatrogenic causes and


physiologic derangements. Iatrogenic lesions include foreign bodies introduced by several routes that become misplaced and lead
to colonic obstruction. Biliary stents or colonic stents may migrate
from their intended position and become lodged in the colon
leading to obstruction.10,11 Small et al.12 evaluated 23 patients who
underwent colonic stenting for left colon obstruction secondary
to benign obstruction. Thirty-eight percent of the patients have a
major morbidity including stent migration (n = 2) and reobstruction (n = 4). Seymour et al.13 reviewed another series of 18 patients
and observed that only 1 of the 18 patients had stent migration
when placed for left-sided colon cancer. The stent migration in
this patient did not lead to obstruction. Other intraluminal causes
of colonic obstruction include phytobezoars, large gallstones, and
rectal foreign bodies.14
Diagnostic Evaluation
3. What is the proper diagnostic modality?
Options for confirming a radiographic diagnosis of large bowel
obstruction are limited and frequently require a multidisciplinary
approach. Often, large bowel obstruction is diagnosed by clinical
presentation, plain abdominal radiograph, and specialized radiologic tests. Plain abdominal radiographs have 84% sensitivity and
72% specificity for diagnosing large bowel obstruction.2
A water-soluble contrast enema may be used in many cases to
establish the diagnosis with 96% sensitivity and 98% specificity.2
Computed tomography (CT) scanners are now readily available
in most hospital settings. In a 7-year, single-institutional review, it
was noted that multidetector CT imaging was more accurate than
contrast enema in diagnosing large bowel obstruction and allowed
for the evaluation of other disease process.15 In a recent literature
review by Finan et al., CT imaging of the abdomen and pelvis with
contrast was recommended in the evaluation of patients with suspected large bowel obstruction as Level 3 evidence. CT scans allow
for the evaluation of the primary process as well as possible metastatic disease.16 It is the opinion of the authors that although CT
imaging and water-soluble enema have comparable diagnostic
capability, CT is preferable due to its ability to further evaluate the
abdomen and pelvis for other disease. Also, the ready availability of
CT imaging machines makes this imaging modality preferable.
As stated earlier, a full laboratory evaluation is needed in the
diagnosis of large bowel obstruction. It is useful in the determination of the patients overall clinical status and may indicate the
presence of intestinal ischemia, necrosis, and perforation. No Level
1 evidence could be found to support obtaining specific laboratory
tests. However, it is known that patients with a large bowel obstruction can present with multiple metabolic derangements that must
be corrected prior to operative intervention. Initial laboratory

5/22/2012 6:30:53 PM

Large Bowel Obstruction

evaluation should include a complete metabolic count to evaluate


for leukocytosis and anemia. Also, a complete metabolic profi le
to include liver function tests as well as a lactate level, coagulation profile, and arterial blood gas analysis should be obtained. As
previously noted, the most common cause of colonic obstruction is
colon cancer. Given this observation, a baseline carcinoembrionic
antigen (CEA) level should be obtained.
In addition to the routine hematologic testing discussed
above, it is the option of the authors that CT imaging of the abdomen and pelvis with intravenous and oral contrast is preferable
for diagnosing a large bowel obstruction. CT imaging lends itself
to being readily available, quickly interpreted, and allows for evaluation of the extent of the primary process and possible metastatic
disease. If necessary, water-soluble contrast can be given rectally
for further evaluation.15 (Grade C recommendation)
Surgical Treatment
4. What is the preferred operative approach?
Operative therapy of acute large bowel obstruction begins with the
correction of fluid status, electrolytes, and acid/base disturbances.
Insertion of a Foley catheter, nasogastric tube, and possibly central venous pressure monitoring catheter should be considered.
Careful attention to antibiotic and prophylactic (venous thromboembolism and gastrointestinal) regimens is necessary.2 Finan
et al. in 2007 published recommendations regarding placement
of self-expanding metallic stents (SEMS) for acute large bowel
obstruction in the absence of signs of peritonitis/perforation. The
authors also provided Level 3 evidence for the efficacy of SEMS for
conversion of an emergent/urgent situation into an elective one.16
If there is concern over ischemia or perforation, and the patient
has not clinically improved, or cecal diameter is increasing, laparotomy should be performed.17 Because most cases of colonic obstruction are due to colon cancer, it is incumbent on the operating surgeon
to thoroughly evaluate the remaining colon for synchronous lesions.

OPERATIVE MANAGEMENT OF
OBSTRUCTING COLON CANCER
The debate over management in obstructing colorectal cancer centers on three main options: complete nonoperative management
using stents, performing resection with primary anastamosis versus colostomy/ileostomy, and initial nonoperative management
with stent decompression followed by interval laparoscopic resection and anastamosis.
In right-sided colon cancer, a right hemicolectomy should
be performed.2 The distal resection margin may include the right
branch of the middle colic artery, especially if the lesion is located
at the hepatic flexure.8 In patients who are hemodynamically stable, this can be accomplished with a primary anastamosis at the
time of resection. The lack of bowel preparation in these patients
is not a contraindication to primary anastomosis.16 However, in
patients who are hemodynamically compromised, with perforated peritonitis, or with distended, edematous bowel, a resection
with end ileostomy should be performed.2 In a 1998 review of 232
cases of obstructing colon cancer requiring urgent surgical intervention, in 160 patients, lesions were located in the colon, and the
remainder being in the rectum. In this group, there was a 25%
mortality rate. Further analysis of the group who died revealed a

PMPH_CH25.indd 223

223

slightly higher mortality rate associated with primary anastamosis in the setting of urgent operative intervention.18
In 2002, de Aguilar-Nascimento et al. reviewed 23 patients with
obstructing lesions of the left colon. The patients underwent two
different surgical treatments: (1) 14 underwent one-stage colonic
resection with colonic lavage (n = 10) or a subtotal colectomy (n = 4)
with primary anastamosis and (2) nine patients underwent staged
resection with either Hartmanns or loop colostomy. There was one
case of anastomotic dehiscence in the resection and primary anastamosis group and two cases in the staged resection group. These
authors concluded that one-stage resection and primary anastamosis are safe and may be indicated for the management of the
majority of cases.19 Finan et al. found that mortality of one-stage
procedures was similar to that for a staged approach, and hospital
length of stay was longer for a staged resection. They concluded
that primary resection and anastomosis are the preferred option
for uncomplicated malignant left-sided large bowel obstruction.16
For those patients who present with disseminated disease, a palliative resection should be performed. For recurrent disease, a bypass
procedure or proximal stoma is the preferred approach.2
Subtotal colectomy and segmental resection have been determined to be equally safe procedures. The choice between the two procedures should be based on certain characteristics of the individual
case. Cecal ischemia, perforation, or serosal tearing favors subtotal
colectomy. Patients with synchronous lesions should also undergo
subtotal colectomy. A segmental resection is favored if a rectal anastomosis is to be performed. Known preoperative continence disturbance favors segmental resection.16 (Grade C recommendation)
Role of Laparoscopy
5. What is the role of laparoscopy in the treatment of large bowel
obstruction?
A Cochrane review, updated in 2008, reported that laparoscopic
colon resection for colon cancer is a safe procedure. Survival rates
are equivalent to open resection. Although there was a trend toward
a lower number of lymph nodes harvested with laparoscopic resection, this did not translate to an increased development of distant
metastasis, tumor recurrence, or any difference in 5-year diseasefree survival compared with the open technique.20,21

OPERATIVE MANAGEMENT OF
COLONIC VOLVULUS
Sigmoid Volvulus
In hemodynamically unstable patients or those with peritonitis,
early operative intervention is indicated. In stable patients, endoscopic
decompression should be attempted. If endoscopic reduction is unsuccessful, or gangrenous mucosa is encountered during endoscopy,
emergent surgery should be performed. If the volvulus is reduced
endoscopically, elective sigmoid resection is strongly advised as there
is a 60% recurrence rate. Sigmoid resection with end-colostomy
(Hartmanns procedure) should be performed in the setting of gangrene, hemodynamic compromise, or free perforation. In cases amenable to resection and primary anastomosis, a subtotal colectomy
with ileorectal anastomosis may be needed if there is a large size discrepancy between the proximal and distal resection margins.2
Two retrospective reviews, one in patients undergoing emergency resection and primary anastomosis with or without colon

5/22/2012 6:30:53 PM

224

Surgery: Evidence-Based Practice

lavage, and a second comparing one- and two-stage operations,


concluded that one-stage resection without on-table lavage or
cecostomy is a safe and acceptable treatment option. Despite the
higher rate of shock in patients with gangrenous bowel, overall
anastomotic leak and mortality rates did not differ significantly.22
Single-stage operations resulted in shorter hospital stays compared with two-stage operations.23

Cecal Vovulus
Nonoperative approaches to cecal volvulus are rarely successful.
Although cecopexy and cecostomy are acceptable treatment alternatives in high-risk patients and have a lower operative mortality,
the long-term morbidity and mortality are higher. Patients with
viable bowel should undergo right hemicolectomy with primary
anastomosis; those with nonviable bowel require end ileostomy
and proximal colostomy.2

OPERATIVE MANAGEMENT OF
OTHER CAUSES
The treatment of benign stricture is segmental resection. In
Crohns disease, stricturoplasty may be more desirable. Preoperative screening colonoscopy is warranted to rule out primary malignancy. Strong consideration must be given to diverting colostomy
in the presence of radiation-induced stricture. However, radiation
does not preclude resection with primary anastamosis.2
Nonoperative management
6. What is the role of colonic stenting?
The role of colonic stenting as a primary treatment modality has
recently been a topic of debate in the literature with respect to
efficacy and safety. Endoluminal stenting has made a significant
impact on the nonoperative treatment of large bowel obstruction.
Given that patients with large bowel obstruction have a significant
morbidity and mortality from diverting colostomy (16% and 5%,
respectively), SEMS have become an acceptable treatment option
for those patients with inoperable disease and for those who are
poor surgical candidates.9
The minimally invasive nature of colonic stents offers an acceptable alternative therapy for the treatment of large bowel obstruction
in patients who are poor surgical candidates or those with inoperable malignant lesions.4,24 Benign strictures caused by diverticulitis,
inflammatory bowel disease, radiation, or ischemic colitis can also
be treated minimally invasively.4 Digital dilation; balloon, or rigid
dilation; steroid injection; electroincision; tube decompression; and
SEMS are all potential minimally invasive treatments.
In 2010, the American Association for Gastrointestinal Endoscopy (ASGE) published a review of the current literature and recommendations for the use of endoscopy in the management of colonic
obstruction. SEMS were successfully placed in 91% of patients with
malignant colonic obstruction. When placed successfully, 90% of
patients had relief of obstructive symptoms.4 SEMS can be used in
both palliative and preoperative settings. Poor surgical candidates
with malignant colonic obstruction can receive relief with palliative
colonic stenting. The ASGE group supported favorable outcomes
in this group with a clinical success rate of 90% to 93%. SEMS can
also be used as a bridge to surgery. Colonic decompression prior

PMPH_CH25.indd 224

to surgical resection has resulted in fewer complications, shorter


hospital stays, and decreased need for colostomy with a higher
rate of primary anastomosis when compared with urgent surgery.4
Complications associated with placement of colonic stents include
intestinal perforation, stent occlusion, and migration.24
Colonic stenting is the most widely accepted method of
nonoperative treatment. Its use is limited by the small number
of adequately trained physicians and centers performing the procedure. Small et al.24 reported that endoscopists experienced in
pancreaticobiliary endoscopy had a lower complication rate compared with endoscopists without the same experience. The majority of literature regarding SEMS involves the treatment of rectal,
sigmoid, left, and distal transverse colon lesions. Placement of
SEMS allows for the relief of obstruction, full evaluation of the
primary process by diminishing the urgency of the situation, and
creating the possibility of a one-staged procedure.
Other options available for nonoperative management
include photodynamic therapy, electrocoagulation, laser coagulation, and balloon dilation. A recent Cochrane review demonstrated that these other modalities have little role in relieving
acute obstruction but may be of value in subsequent treatment.5
(Grade C recommendation)

ACUTE COLONIC PSEUDO-OBSTRUCTION


The prevalence of ACPO is not known; however, it has been postulated
to be responsible for 20% of cases of large bowel obstruction, occurring in 1% of hospitalized patients undergoing orthopedic procedures
and 0.3% of patients with major burns. Predisposing factors include
myocardial infarction, neurologic disease, severe infection, electrolyte
imbalance, surgery, and trauma. The most common causes found in
large retrospective evaluations included operative (23%) and nonoperative (11%) trauma, cardiac disease (1018%), and infection (10%).3
The most significant complications of ACPO are intestinal ischemia and colonic perforation. The rate of perforation in
ACPO has been reported between 3% and 15%. Cecal diameter
determines the rate of intestinal ischemia with rapid increasing
rates at diameters >012 cm for more than 6 days.
Initial management of ACPO is evaluation of potential correctable causes such as electrolyte imbalance and narcotic or anticholinergic medication use. Conservative management includes
nothing by mouth, gastric decompression as necessary, body positioning, and placement of a rectal tube. This conservative approach
is a reasonable approach for up to 48 h with serial abdominal
examination, laboratory testing, and abdominal radiographs. The
reported success rate is between 20% and 92%.4
A variety of pharmacologic agents have been investigated for the
treatment of ACPO. Neostigmine, an anticholinesterase parasympathomimetic agent, has been shown in multiple studies in a review
by ASGE to have consistently positive results. There are many potential anticholinergic side effects with neostigmine administration.4,6,26
A patient that is to be treated with neostigmine must be under a
cardiac monitor with atropine readily available.4 A randomized
controlled trial published in the New England Journal of Medicine
in 1999 showed that 10 of 11 patients who failed 24 h of conservative therapy given neostigmine had prompt colonic decompression.
The one patient who failed initial therapy had decompression following a second dose. Two patients relapsed requiring colonoscopic
decompression with one eventually requiring subtotal colectomy.26
None of the patients in the control group decompressed with

5/22/2012 6:30:53 PM

Large Bowel Obstruction

administration of a placebo. Furthermore, seven patients in the placebo group received open-label treatment with neostigmine and had
prompt colonic decompression. The use of neostigmine is supported
by Level 2 data and has changed primary surgical management for
colonic pseudo-obstruction. Contraindications include mechanical
urinary and intestinal obstruction.
Endoscopic decompression of ACPO has a reported success
rate of 80%, with approximately 20% of patients requiring further colonoscopic decompression secondary to recurrence. Two
nonrandomized studies have shown that there is a decreased
recurrence rate when a tube is placed during colonoscopy. Other
procedures described for the treatment of ACPO include percutaneous endoscopic colostomy in either the cecum or the left colon
and CT-guided transperitoneal percutaneous cecostomy.3
Surgical intervention is indicated for patients who have failed
conservative management or who have progressed to colonic perforation. Mortality rates from 30% to 60% have been reported
for patients requiring surgical intervention for ACPO. Surgical
options include a venting stoma, cecostomy, colostomy, or colonic
tube placement.3 There are a paucity of clinical trials comparing
these different surgical interventions. It is our opinion that surgical intervention should be reserved for the subset of patients with
either a complication secondary to ACPO or those who fail conservative management.

225

CONCLUSION
The outcomes for patients presenting with large bowel obstruction vary depending on the cause of obstruction and the presence of compromised bowel at the time of operation. Mortality
rates for patients with large bowel obstruction from colon
cancer range from 5% to 25%. The mortality rate increases
with findings of necrosis or perforation.1,27 A study by Zorcolo
et al. 28 retrospectively reviewed records of 323 patients who
presented with obstruction from left-sided colorectal cancer
and diverticular disease and underwent urgent surgery. Primary anastomosis was performed in 176 (55.7%) patients with
a 30-day mortality of 5.7%. Nine patients (5.1%) had anastomotic breakdown. Hartmanns resection was associated with
a higher incidence of systemic and surgical morbidity (39.5%
and 24.3%, respectively). Mortality from primary anastomosis
(5.7%) compared favorably with those undergoing Hartmanns
resections (20.4%).
As mentioned, colonic stenting has the ability to convert an
emergent/urgent procedure to an elective procedure allowing for
adequate colonic preparation and preoperative evaluation/optimization of the patient. The reviewed studies reveal a lower morbidity and mortality when colonic stents were used as a bridge to
a one-stage procedure.

Clinical Question Summary


Question

Answer

1 How do patients with large bowel


obstruction present?

Absence of passage of flatus (905) and/or feces


(80.6%) and abdominal distension (65.3%) were
the most common symptoms and physical finding.

2 What are the most common causes


of large bowel obstruction?

Large bowel cancer, adhesions, retroperitoneal


tumors, and hernias were the most common
causes of large bowel obstruction. Hernias,
adhesions, strictures, endometriosis, ingested
foreign bodies, phytobezoars, gallstones, and
rectal foreign bodies have all been found to cause
large bowel obstruction.

4, 5, 7-11

3 What is the proper diagnostic


modality?

CT imaging is more accurate for the diagnosis of


large bowel obstruction than contrast enema. CT
imaging in conjunction with contrast enema may
yield superior results.

2, 12

4 What is the preferred operative


approach?

Stomas are preferred for patients with recurrent


disease or palliative resection. A primary
anastomosis can be safely performed for
obstructing colon lesions.

2, 4, 14, 15

5 What is the role of laparoscopy


in the treatment of large bowel
obstruction?

Laparoscopic colon resection is a safe and feasible


treatment option. Colonic stenting is a safe and
effective bridge to subsequent minimally invasive
left colectomy.

20, 21

6 What is the role of colonic


stenting?

Colonic stenting can be used as a bridge to a


one-stage procedure or as a palliative option.
The success rate for relieving obstruction is
approximately 90%. Three-quarters of patients
in which colonic stents are placed as a bridge to
surgery undergo elective resection.

4, 5, 9, 24

PMPH_CH25.indd 225

Grade of
Recommendation

References

5/22/2012 6:30:53 PM

226

Surgery: Evidence-Based Practice

REFERENCES
1. Markogiannakis H, Messaris E, Dardamanis D, et al. Acute mechanical bowel obstruction: Clinical presentation, etiology, management
and outcome. World J Gastroenterol. 2007;13:432-437.
2. Cappell MS, Batke M. Mechanical obstruction of the small bowel
and colon. Med Clin North Am. 2008;92:575-597.
3. De Giorgio R, Knowles CH. Acute colonic pseudo-obstruction.
Br J Surg. 2009;96:229-239.
4. American Society for Gastrointestinal Surgery. The role of
endoscopy in the management of patients with known and suspected colonic obstruction and pseudo-obstruction. GI Endosc.
2010;71(4):669-679.
5. Ramirez R, Zuckerman MJ, Hejazi RA, Chokhavatia S. Treatment of acute colonic pseudo-obstruction with tegaserod. Am J
Med Sciences. 2010;339(6):575-576.
6. Saunders MD, Kimmey MD. Systematic review: Acute colonic
pseudo-obstruction. Aliment Pharmacol Ther. 2005;22:917-925.
7. Jenkins JT, Taylor AJ, Behrns KE. Secondary causes of intestinal
obstruction: Rigorous preoperative evaluation is required. Am
Surg. 2000;66:662-666.
8. Fazio V, Church J, Delaney C. Current Therapy in Colon and Rectal Surgery. 2nd ed. St. Louis, MO: Mosby; 2004.
9. Varras M, Kostopanagiotou E, Katis K, et al. Endometriosis causing extensive intestinal obstruction simulating carcinoma of the
sigmoid colon: A case report and review of the literature. Eur J
Gynaecol Oncol. 2002;23:353-357.
10. Diller R, Senninger N, Kautz G, Tbergen D. Stent migration
necessitating surgical intervention. Surg Endosc. 2003;17:18031807. [Epub September 29, 2003.]
11. Lopera JE, Ferral H, Wholey M, et al. Treatment of colonic
obstructions with metallic stents: Indications, technique, and
complications. Am J Roentgenol. 1997;169:1285-1290.
12. Small AJ, Young-Fadok TM, Baron TH. Expandable metal stent
placement for benign colorectal obstruction: Outcomes for 23
cases. Surg Endosc. 2008;22:454-462.
13. Seymour K, Johnson R, Marsh R, Corson J. Palliative stenting of
malignant large bowel obstruction. Colorectal Dis. 2002;4:240-245.
14. Efrati Y, Freud E, Serour F, Klin B. Phytobezoar-induced ileal
and colonic obstruction in childhood. J Pediatr Gastroenterol
Nutr. 1997;25:214-216.
15. Jacob SE, Lee SH, Hill J. The demise of the instant/unprepared
contrast enema in large bowel obstruction. Colorectal Dis. 2007;
November 12. [Epub ahead of print.]

PMPH_CH25.indd 226

16. Finan PJ, Campbell S, Verma R, MacFie J, Gatt M, Parker MC,


Bhardwaj R, Hall NR. The management of malignant large
bowel obstruction: ACPGBI position statement. Colorectal Dise.
2007;9(4):1-17.
17. Lopez-Kostner F, Hool GR, Lavery IC. Management and causes of
acute large-bowel obstruction. Surg Clin North Am. 1997;77:12651290.
18. Stoyanov H, Julianov A, Valtchev D, et al. Results of the treatment of colorectal cancer complicated by obstruction. Wien Klin
Wochenschr. 1998;110:262-265.
19. de Aguilar-Nascimento JE, Caporossi C, Nascimento M. Comparison between resection and primary anastomosis and staged
resection in obstructing adenocarcinoma of the left colon. Arq
Gastroenterol. 2002;39:240-245.
20. Kuhry E, Schwenk W, Gaupser R, Romild U, Bonjer HJ. Longterm results of laparoscopic colorectal cancer resection. Cochrane
Database of Systematic Review. 2008;2:CD003432.
21. Chueng HYS, Chung CC, Chieng WW, Wong JCH, Yau KKK,
Li MKW. Endolaparoscopic approach vs conventional open surgery in the treatment of obstructing left-sided colon cancer. Arch
Surg. 2009;144(12):1127-1132.
22. Raveenthiran V. Restorative resection of unprepared left-colon
in gangrenous vs. viable sigmoid volvulus. Int J Colorectal Dis.
2004;19:258-263.
23. Akcan A, Akyildiz H, Artis T, et al. Feasibility of singlestage resection and primary anastomosis in patients with
acute noncomplicated sigmoid volvulus. Am J Surg. 2007;193:
421-426.
24. Small AJ, Coelho-Prabhu N, Baron TH. Endoscopic placement
of self-expanding metal stents for malignant colonic obstruction: Long-term outcomes and complication factors. GI Endosc.
2010;71(3):560-572.
25. Baraza W, Lee F, Brown S, Hurlstone DP. Combination endoradiological colorectal stenting: A prospective 5-year clinical
evaluation. Ann Surg Oncol. 2002;9:574-579.
26. Ponec RJ, Saunders MD, Kimmey MB. Neostigmine for the
treatment of acute colonic pseudo-obstruction. N Engl J Med.
1999;341(3):137-141.
27. Hennekine-Mucci S, Tuech JJ, Brehant O, et al. Management
of obstructed left colon carcinoma. Hepatogastroenterology.
2007;54:1098-1101.
28. Zorcolo L, Covotta L, Carlomagno N, et al. Safety of primary
anastomosis in emergency colo-rectal surgery. Colorectal Dis.
2003;5:262-269.

5/22/2012 6:30:53 PM

Commentary on
Large Bowel Obstruction
Martin A. Schreiber

The authors of the chapter Large Bowel Obstruction, Ogilvies


and Volvulus have performed an evidenced-based review summarizing the diagnosis, management and outcomes of patients
with large bowel obstruction. The review highlights the fact that
the literature does not contain Level 1 evidence or support Grade A
recommendations in this field. The authors provide a nice description of the differential diagnosis of large bowel obstruction and
they conclude based on the literature that computed tomography
is the best modality available to establish a diagnosis. The authors
outline the indications for operative management of large bowel
obstruction and options for therapy that include open exploration,
laparoscopic exploration, primary anastomosis, damage control
techniques, and the creation of diverting ostomies. The indications for each of the modalities are well-described. The authors
focus on the fact that colonic stenting is a viable option to either
establish colonic continuity prior to definitive therapy or palliate
patients who are not candidates for resection.
In general, the review is well-balanced and it touches on the
critical elements of the topic. However, the review only superficially addresses diverticulitis as a cause of large bowel obstruction
classifying it as a less frequent cause of large bowel obstruction.
In fact, diverticulitis is frequently cited as the third most common
cause of large bowel obstruction following neoplasm and volvulus.1 Complete large bowel obstruction secondary to diverticulitis
has traditionally been treated with sigmoid resection and proximal colostomy. Resection with on-table preparation followed by
primary anastomosis with or without proximal diversion has also
been reported.2,3 Although the use of stents for benign disease
remains controversial, stent placement followed by semi-elective
one-stage resection has been reported.2
The authors correctly identify colonic pseudo-obstruction or
Ogilvies Syndrome as an important element in the differential
diagnosis of large bowel obstruction. They briefly mention that

mechanical obstruction is a contraindication to the use of neostigmine. This important point requires additional discussion.
The use of neostigmine in patients with mechanical obstruction
may result in life threatening perforation. Mechanical obstruction
should be definitively excluded prior to prescribing neostigmine
in these patients. Appropriate imaging modalities include computed tomography or water-soluble contrast enema.4,5 Colonoscopy can also be used to rule out distal obstruction and, as the
authors point out, it has the potential to be a therapeutic intervention. Side effects of the administration of neostigmine include
bronchospasm, bradycardia, and hypotension, potentially leading
to syncope. The incidence of these complications can be reduced
by giving the drug as a continuous infusion as opposed to bolus
administration and reducing the dose from 2 to 1 mg.6

REFERENCES
1. Webb AL, Fink AS. Large Bowel Obstruction; Current Surgical
Therapy. 10th ed. Philadelphia, PA: Elsevier; 2011: 154-157.
2. McCafferty MH, Roth L, Jorden J. Current management of diverticulitis. Am Surg. 2008;74:1041-1049.
3. Edward CL, Murray JJ, Coller JA, et al. Intraoperative colonic
lavage in nonelective surgery for diverticulitis. Dis Colon Rect.
1997;40:669-674.
4. Chapmann AH, McNamara M, Porter G. The acute contrast
enema in suspected large bowel obstruction: Value and technique.
Clin Radiol. 1992;46:273-278.
5. Beattie GC, Peters RT, Guy S, Mendelson RM. Computed tomography in the assessment of suspected large bowel obstruction.
ANZ J Surg. 2007;77:160-165.
6. Delgado-Aros S, Camilleri M. Pseudo-obsruction in the critically
ill. Best Prac Res Clin Gastroenterol. 2003;17:427-444.

227

PMPH_CH25.indd 227

5/22/2012 6:30:53 PM

CHAPTER 26

Radiation Injury to the


Small and Large Bowel
Ali Y. Mejaddam and David R. King

Acute radiation injury usually occurs 2 to 3 weeks after medical


radiation exposure and is histologically characterized by epithelial denudation and mucosal ulceration.2 Symptoms of acute
radiation enteritis include abdominal pain, diarrhea, and frequent bloody stools. These symptoms often subside within 2 to 6
weeks after completion of radiotherapy and rarely require surgical
intervention.10
Chronic radiation enteritis typically develops 6 months to 6
years after radiotherapy, even though toxicity after several decades
has been reported.3,11 The pathology of chronic radiation enteritis is believed to result from an obliterative endarteritis resulting in chronic microvascular ischemia of the intestines.2 Clinical
features include malabsorption and chronic diarrhea, as well
as obstructions, perforations, and fistula formation; it has been
reported that one-third of these patients will require surgery.7
Answer: Radiation enteritis is distinguished into an acute
and a chronic phase. The acute phase is often transient and rarely
requires surgical intervention. Some patients develop chronic
radiation enteritis due to ischemia caused by obliterative endarteritis, and about one-third of these patients may present with
symptoms that require an operation. (Grade C recommendation)

The first report of radiation-induced injury to the gastrointestinal tract occurred 2 years after the discovery of x-rays in 1897, in
a scientist who developed colic and diarrhea after daily experimental exposure of his stomach to x-rays.1 Radiotherapy has since
become an essential part of the multimodal treatment of various
neoplasms. Expectedly, the increasing number of patients treated
with radiation is followed by a comparable increase in the number suffering from its complications. Radiation-induced injury to
the small and large bowel, known clinically as radiation enteritis, is a well-recognized complication following radiotherapy for
abdominopelvic malignancies.2,3 The literature on the medical
and surgical management of patients with radiation enteritis is
extremely limited and consists predominantly of case series and
small cohorts.2,3 In fact, the only Level 1 study is a double-blinded
randomized-controlled trial showing that loperamide is superior
to placebo in the management of diarrhea in patients with chronic
radiation enteritis.4
The diagnosis of chronic radiation-induced injury to the bowel
is established in patients with prior abdominopelvic irradiation
whose clinical symptoms are compatible with suggestive radiologic findings, such as bowel thickening or distension2,5,6 without
an alternative etiology (ischemia or infection). For those patients
who go on to require an operation, most commonly for obstruction or perforation, it is important to note that irradiated intestines are vulnerable due to poor wound-healing, and dissection
and manipulation of the bowel in this region may be technically
challenging.3,6,7 The optimal surgical management is supported by
limited data and, as such, remains under much debate.

2. Is there a critical radiation dose in which serious bowel


injury occurs?
There are several risk factors associated with the development
of chronic radiation enteritis including radiation dose, volume
of irradiated small bowel, and concomitant use of chemotherapy.2 In a literature review by Emami et al., the total dose at which
5% of patients are expected to experience serious bowel toxicity
(defined as requiring an intervention) at 5 years (TD5/5) is 50 Gy
if a third of the volume of the small bowel was irradiated.12 For
the same small bowel volume, the dose at which 50% of patients
would experience serious toxicity at 5 years (TD50/5) is 60 Gy. The
TD5/5 and TD50/5 for a third of the volume of the colon is slightly
higher at 55 and 65 Gy, respectively.12 These results are corroborated by subsequent studies of patients with chronic radiation
enteritis who needed surgical therapy, showing a total radiation
dose in the range of 47 to 62 Gy.7,11

1. How does radiation affect the bowel?


Radiation therapy causes damage to cellular DNA which subsequently leads to cell death.8 This effect is more pronounced in
mitotically active cells such as tumor cells; however, other dividing cells in normal tissue may be affected as well. The small and
large bowel may be particularly sensitive to radiation due to its
rapid cellular turnover rate,9 particularly at the mucosal level.
The toxicity of radiation to intestines is often distinguished into
an acute phase and a chronic (sometimes progressive) phase.2,10
228

PMPH_CH26.indd 228

5/21/2012 9:09:21 PM

Radiation Injury to the Small and Large Bowel

Answer: TD5/5 for the small and large bowel is 50 and 55 Gy,
respectively. (Grade C recommendation)
3. Is IMRT less harmful to the bowel than conventional
radiotherapy?
Traditionally, external-beam radiation was delivered using a twobeam or four-beam approach in which a constant dose rate is
administered to a defined field. Newer techniques, such as threedimensionalconformal radiation therapy (3D-CRT) and intensitymodulated radiation therapy (IMRT) use advanced imaging to
improved precision of irradiation to target tissues.13 In particular,
IMRT permits variable control of the intensity of the beam within
the same field, which limits normal tissue exposure.13 Portelance
et al. showed a significant reduction in mean percentage volume
of small bowel receiving >45 Gy by use of IMRT (11%) compared
with conventional four-beam radiation (34%), in patients with cervical cancer.13 Other authors have corroborated these results,14,15
but whether IMRT actually reduces the incidence of chronic radiation enteritis remains unclear. In a casecontrol study by Mundt
et al., a significant 40% reduction in chronic gastrointestinal toxicity was noted in patients with gynecologic malignancy treated
with IMRT compared with historical controls treated with conventional radiotherapy.16 The median follow-up time in this study
was 19 months, suggesting that long-term data are warranted to
support its conclusions.
Answer: IMRT reduces the volume of healthy small bowel
irradiated and lowers the rate of gastrointestinal toxicity at
19 months. (Grade B recommendation)
4. Surgical management: bowel resection or bypass?
Symptoms of acute radiation enteritis are normally transient and
usually resolve within 2 weeks of completion of radiotherapy.11
For chronic radiation enteritis, most studies suggest that surgical therapy should be avoided as long as possible because of difficulties operating irradiated bowel.5-7,11 Fibrosis and adhesions
may make resection technically challenging (risking iatrogenic
enterotomy), and anastomotic leak rates are higher due to poor
tissue healing.3 Also, short bowel syndrome (SBS) can develop if
there is extensive resection of bowel (or even subextensive resection if absorptive capacity has been affected).7 Despite attempts
at conservative management, approximately one-third of patients
with chronic radiation enteritis will undergo surgery.2,7 The most
common indications for surgery are persistent obstruction due
to strictures or adhesions, perforation, and fistulization.3,5-7 The
data on surgical management of patients with radiation enteritis
is limited and consists primarily of case series (Level 4 evidence).
The choice between resection with anastomosis or bypass as the
most appropriate surgical approach has been the topic of most
studies.
In an influential review of historical series published in 1976,
Swan et al. noted that patients who had intestinal bypass rather
than resection had a lower operative mortality (21% vs. 10%) and
a lower rate of anastomotic dehiscence (36% vs. 6%), suggesting
bypass is the procedure of choice.5 A decade later, Galland et al.
showed that extensive resection allowing anastomosis of healthy
bowel ends in 12 patients resulted in only one anastomotic leak.6
In a more recent literature analysis, Meissner evaluated data of all
patients reported in 41 publications (search criteria not defi ned)
on the surgical management of chronic radiation enteritis.3 The

PMPH_CH26.indd 229

229

author noted that of 801 patients, 71% of patients presented with


obstruction and the most common sites of radiation injury were
the lower ileum, caecum, and sigmoid colon, while low rates of
injury were noted in the ascending and transverse colon. Bowel
resection with ileotransversecolonic anastomosis was associated
with a decreased rate of anastomotic insufficiency and postoperative mortality compared with ileoileal anastomosis (4% vs.
25.5% anastomotic insufficiency; 3% vs. 24.7% mortality), suggesting that frequently compromised areas of the bowel are
prone to anastomotic dehiscence. Ileotransversecolonic bypass
was also associated with low rates of anastomotic leak (1.6%),
but the author noted a higher rate of progressive radiation injury
for bypass procedures (38%) when compared with resection with
ileotransversecolonic anastomosis (7.2%), possibly reflecting the
fact that irradiated bowel is left behind. Meissner concludes that,
when feasible, extensive resection with anastomosis of healthy
bowel segments is safe for complications of chronic radiation
enteritis.3
In addition, there is concern about leaving irradiated bowel
in the abdomen following bypass procedures due to the risk of
malignant transformation,7 even though there is no study to date
that examines if rates of malignant bowel disease are lower after
surgical resection for radiation enteritis. Subsequent authors have
continued to favor bowel resection as the procedure of choice,
noting an anastomotic dehiscence rate in the range of 0% to
9%.7,17 In a study of 109 patients with chronic radiation enteritis
who underwent intestinal resection or conservative management,
Regimbeau et al. note that 5% of patients died postoperatively,
while 5-year survival for patients without recurrence of primary
malignancy was 71% after resection compared with 51% after conservative management.7
Answer: There is no convincing evidence suggesting the
optimal surgical procedure for complications of chronic radiation enteritis. The available data suggest that extensive resection
with anastomosis of healthy bowel segments is associated with a
lower rate of morbidity and mortality than bowel bypass surgery.
(Grade C recommendation)
5. What is the risk of SBS after surgery?
Complications following surgery for chronic radiation enteritis
are intimately related to the condition of the irradiated bowel.3
Onodera et al. and Regimbeau et al. present similar operative morbidity rates of 22% and 29%, respectively, including anastomotic
leak, wound infection, and small bowel obstruction.7,17 Regimbeau
et al. also noted that reoperation with further intestinal resection
was performed in 34% of patients, raising the concern for SBS. In
fact, 20% of all cases of SBS are caused by radiation enteritis,18,19
but the risk of developing SBS after surgery for radiogenic bowel
injury remains unreported.20
The prognosis for these patients is poor, as shown in a study
of 48 adults with postresection SBS secondary to radiotherapy,
in which 1- and 5-year survival were 83% and 68%, respectively.20
Vantini et al. reported similar 1- and 5-year survival (85% and
65%, respectively), noting that this patient group had more complications and lower survival than patients with other causes
of SBS.19
Answer: The incidence of SBS after surgery for radiogenic
bowel injury remains unknown. Notably, patients with SBS secondary to radiotherapy seem to have a worse prognosis than
patients with other causes of SBS. (Grade C recommendation)

5/21/2012 9:09:21 PM

230

Surgery: Evidence-Based Practice

Clinical Question Summary


Level of
Evidence

Grade of
Recommendation

Chronic radiation enteritis is caused by


radiation-induced obliterative endarteritis,
and about one-third of these patients will
require surgery for their symptoms.

2, 7-11

2 Is there a critical radiation


dose in which serious
bowel injury occurs?

Five percent of patients experience serious


bowel toxicity at 5 years (requiring
intervention) if the bowel is irradiated with
a total dose of 50 Gy.

12

3 Is IMRT less harmful to the


bowel than conventional
radiotherapy?

IMRT reduces the rate of chronic bowel


toxicity at 19 months, compared with
conventional radiotherapy.

3b

16

4 Surgical management: bowel


resection or bypass?

Extensive resection with anastomosis of


healthy bowel segments is associated with
lower morbidity and mortality than bowel
bypass surgery.

3, 5-7

5 What is the risk of SBS


after surgery?

Incidence of SBS after surgery for radiogenic


bowel injury remains unknown.

20

Question

Answer

1 How does radiation affect


the bowel?

REFERENCES
1. Walsh D. Deep tissue traumatism from roentgen ray exposure.
Br Med J. 1897;2(1909):272-273.
2. Theis VS, Sripadam R, Ramani V, et al. Chronic radiation enteritis. Clin Oncol (R Coll Radiol). 2010;22(1):70-83.
3. Meissner K. Late radiogenic small bowel damage: Guidelines for
the general surgeon. Dig Surg. 1999;16(3):169-174.
4. Yeoh EK, Horowitz M, Russo A, et al. Gastrointestinal function
in chronic radiation enteritiseffects of loperamide-N-oxide.
Gut. 1993;34(4):476-482.
5. Swan RW, Fowler WC, Jr., Boronow RC. Surgical management
of radiation injury to the small intestine. Surg Gynecol Obstet.
1976;142(3):325-327.
6. Galland RB, Spencer J. Natural history and surgical management of radiation enteritis. Br J Surg. 1987;74(8):742-747.
7. Regimbeau JM, Panis Y, Gouzi JL, et al. Operative and long term
results after surgery for chronic radiation enteritis. Am J Surg.
2001;182(3):237-242.
8. Bismar MM, Sinicrope FA. Radiation enteritis. Curr Gastroenterol Rep. 2002;4(5):361-365.
9. MacNaughton WK. Review article: New insights into the pathogenesis of radiation-induced intestinal dysfunction. Aliment
Pharmacol Ther. 2000;14(5):523-528.
10. Hauer-Jensen M. Late radiation injury of the small intestine.
Clinical, pathophysiologic and radiobiologic aspects. A review.
Acta Oncol. 1990;29(4):401-415.
11. Turina M, Mulhall AM, Mahid SS, et al. Frequency and surgical
management of chronic complications related to pelvic radiation. Arch Surg. 2008;143(1):46-52; discussion 52.

PMPH_CH26.indd 230

References

12. Emami B, Lyman J, Brown A, et al. Tolerance of normal tissue to


therapeutic irradiation. Int J Radiat Oncol Biol Phys. 1991;21(1):
109-122.
13. Portelance L, Chao KS, Grigsby PW, et al. Intensity-modulated
radiation therapy (IMRT) reduces small bowel, rectum, and
bladder doses in patients with cervical cancer receiving pelvic and para-aortic irradiation. Int J Radiat Oncol Biol Phys.
2001;51(1):261-266.
14. Nutting CM, Convery DJ, Cosgrove VP, et al. Reduction of small
and large bowel irradiation using an optimized intensity-modulated
pelvic radiotherapy technique in patients with prostate cancer. Int J
Radiat Oncol Biol Phys. 2000;48(3):649-656.
15. Guerrero Urbano MT, Henrys AJ, et al. Intensity-modulated
radiotherapy in patients with locally advanced rectal cancer
reduces volume of bowel treated to high dose levels. Int J Radiat
Oncol Biol Phys. 2006;65(3):907-916.
16. Mundt AJ, Lujan AE, Rotmensch J, et al. Intensity-modulated
whole pelvic radiotherapy in women with gynecologic malignancies. Int J Radiat Oncol Biol Phys. 2002;52(5):1330-1337.
17. Onodera H, Nagayama S, Mori A, et al. Reappraisal of surgical treatment for radiation enteritis. World J Surg. 2005;29(4):
459-463.
18. Thompson JS. Inflammatory disease and outcome of short bowel
syndrome. Am J Surg. 2000;180(6):551-554; discussion 554-555.
19. Vantini I, Benini L, Bonfante F, et al. Survival rate and prognostic factors in patients with intestinal failure. Dig Liver Dis.
2004;36(1):46-55.
20. Boland E, Thompson J, Rochling F, et al. A 25-year experience
with postresection short-bowel syndrome secondary to radiation therapy. Am J Surg. 2010;200(6):690-693; discussion 693.

5/21/2012 9:09:21 PM

Commentary to Radiation Injury to


the Small and Large Bowel
Daniel L. Dent

Mejaddam and King are to be commended for their review and


analysis of the literature on the topic of radiation enteritis. This is
an extremely timely topic given the recent devastating events in
Japan. This is also a very challenging topic due to the lack of highgrade literature and as a result it is a particularly difficult topic for
young surgeons to grasp. However, I can honestly state that this
summary of the literature represents the single most clear summary on radiation enteritis that I have ever read. This is due to a
combination of the clarity of writing by the authors and the format of the chapter that gets at key clinical questions that trouble
the clinician when managing the patient with the disease.
The only caveat in the authors recommended approach is
that the reader must be careful to recognize that while surgical therapy should be avoided as long as possible, radiation
enteritis is a progressive disease and many patients do ultimately

require surgical intervention. It is imperative not to delay surgery when it is indicated as this can negatively impact the
patients outcome.
These operations are as challenging as any in the realm of
general surgery. As the authors indicated, efforts should be made
to resect so as to remove the diseased bowel. At times, this may
require removing the most damaged portion of the GI tract in
a piecemeal fashion, and this is unsettling to the inexperienced
surgeon. However, the guiding principle during this dissection
relates to damage to the surrounding structures. If the surrounding structures can be preserved, the irradiated bowel is resectable
regardless of the damage that will be created to the diseased intestine that is being removed.
Again, the authors are to be commended for their concise and
extremely clear summary of a very difficult disease.

231

PMPH_CH26.indd 231

5/21/2012 9:09:21 PM

CHAPTER 27

Ischemic Colitis
Thomas D. Conlee, Daniel J. Bonville, and Jonathan J. Canete

INTRODUCTION

RISK FACTORS

Ischemic colitis (IC), first described as such by Marston et al. in


1966,1 is considered to be an illness of the elderly. It is most often a
reversible condition where the colonic mucosa is starved of adequate
blood supply. Additionally, ischemia of the colon is the most common form of intestinal ischemia. While it still affects elderly patients
with multiple medical comorbidities most often, it can be seen in
younger patients with hypercoagulopathies and woman using oral
contraceptives.
The incidence of IC is thought to be grossly underestimated
due to its often subclinical nature, but has been responsible for
1 in every 2000 hospital admissions with the average patient age
of 70 years.2 IC manifests as varying degrees of disease depending on its cause and is most often transient without the need for
surgical intervention. Marston also described three forms of the
disease: transient, stricture, and gangrenous. Early diagnosis with
CT scan and colonoscopy, the gold standard in evaluating colonic
ischemia, and intervention are important to control it as a source
of septic shock or even death.
Although its etiology is most often due to low-flow states and/
or small-vessel occlusion, a thoughtful history would elucidate
recent aortic intervention that would explain an acute change in
the blood supply to the colon. Watershed areas of the colon, represented by the splenic flexure and distal sigmoid colon, are often
affected when the natural blood supply has been altered.
IC ranges from mild episodes manifested as transient abdominal pain and diarrhea to septic shock as seen in the most severe
disease progression following colonic perforation. Additional signs
and symptoms observed depending on the degree of disease are
abdominal tenderness, bloody diarrhea, nausea, vomiting, fever,
leukocytosis, and bandemia. Approximately 15% of patients with
IC progressing to gangrene.3 Even with timely surgical intervention, this group of patients has a mortality range of 40% to 75%.4,5
The degree of disease is the determining factor for intervention.
Mild disease is often treated with intravenous antibiotics and bowel
rest, while severe disease is managed with segmental or complete
colonic resection, most often as a surgical emergent procedure.

1. What are the risk factors for IC?


The evidence elucidating risk factors for IC is inconsistent, and
there are no prospective studies to evaluate. However, there are
numerous retrospective studies that depict a number of risk factors that are frequently identified.
Many medical conditions have been cited that put patients at
risk of IC. Fernndez et al.6 studied risk factors of IC in 161 patients
admitted to a single institution and applied control patients in a
1:2 ratio to develop their conclusions. Of these subjects, diabetes
mellitus, dyslipidemia, heart failure, peripheral arteriopathy, the
use of aspirin and digoxin were found to be significant, independent risk factors. Similarly, Chung et al.,7 retrospectively analyzed
153 patients and determined hypertension, cardiovascular disease, diabetes mellitus, and malignancy to be significant risk factors in the prognosis of IC.
Other retrospective studies analyzed risk factors other than
those directly affecting the underlying disease of the cardiovascular system. Chang et al., 8 in a large-scale, pharmaceutical
funded retrospective study sought to confirm irritable bowel
syndrome (IBS) and constipation as significant risk factors for
IC. In addition to confirming this, other risk factors with odds
ratios >2 were identified as shock, dysentery, bloating, colon
carcinoma, cardiovascular disease, dyspepsia, abdominal, aortic or cardiovascular surgery, 12-month laxative, H2 receptor
blocker, and oral contraceptive use.
In a single institution study, Preventza et al.9 retrospectively evaluated young women (<50 years) diagnosed with IC via
colonoscopy and found 52% of the 25 patients were taking oral
contraception. This finding only strengthens the previously sited
study regarding oral contraceptive pill (OCP).
Answer: Risk factors for IC appear to be shock, cardiovascular
disease, diabetes mellitus, peripheral arterial disease, hypertension,
hyperlipidemia, IBS, constipation, malignancy, aortic and cardiovascular surgery, dyspepsia, prolonged laxative use, and OCP in
young women. (Level 2 evidence, Grade B recommendation)
232

PMPH_CH27.indd 232

5/21/2012 9:10:44 PM

Ischemic Colitis

2. Is there an association of IC with thrombotic or thromboembolic conditions?


A number of small studies evaluate the prevalence of coagulation
abnormalities in patients with IC. The references below describe
the larger, more robust studies. Arterial thromboembolic disease
affecting the mesenteric vasculature often spares the blood supply to the colon. The embolus, which traditionally stems from a
cardiac source, often travels embolizing to a point distal to the
middle colic artery take-off ,and therefore more often occludes
small bowel blood supply.
Midian-Singh et al.,10 in an attempt to elucidate the potential
cause of otherwise idiopathic IC, evaluated 18 patients and found
five to have coagulation abnormalities including factor V Leiden
and activated protein C resistance, protein S deficiency, prothrombin 20210G/A mutation, and anticardiolipin antibody.
Another study by Koutroubakis et al.,11 in a prospective,
casecontrol design looked for the prevalence of hypercoaguable
states in ambulatory patients diagnosed with mild to moderate
IC. The study showed one or several prothrombotic disorders in
26 of the 36 patients (72%) with IC. An elevated antiphospholipid
antibody was found most common among acquired thrombotic
diseases while factor V Leiden disease was most common among
congenital diseases. Although this study is limited by its size, the
prevalence of these disorders in ambulatory patients may justify a
thrombotic workup when other risk factors are not apparent.
Hourmand-Ollivier et al.12 found patients with nongangrenous, IC were more likely to have a potential cardiac source of a
thromboembolism when compared with an appropriate control
group. Therefore, they recommended that these patients undergo
a cardiac work-up to rule out the source of an embolism. In
another study, by Collet et al.,13 33% of ambulatory patients with
nongangrenous IC were found to have a potential cardiac source
of thromboembolism. The work-up included taking a focused history, obtaining an electrocardiogram, analysis of a Holter monitor, and a transthoracic echocardiogram.
Answer: Limited literature shows that there is an association between coagulation abnormalities and IC. The degree of
this association is difficult to define further, hence when IC is
diagnosed without a clear explanation of its cause, a coagulation
work-up appears to be worthwhile. Thromboembolic disease may
affect the colon and a search for a cardiac source may be warranted in at least ambulatory patients with IC. (Level 3 evidence,
Grade C recommendation)

DIAGNOSIS

233

In a series by Champagne et al. out of Albany, New York,15


patients who survived emergency surgery for a ruptured abdominal aortic aneurysm (AAA) were offered a colonoscopy to evaluate
the integrity of the bowel wall. Overall, 62 of 72 patients underwent endoscopic evaluation and 39% were found to have some
degree of colonic ischemia. Eighteen patients (Grades 1 and 2)
were initially managed nonoperatively, while those with Grade 3
ischemia underwent an exploratory laparotomy for bowel resection. One patient initially found to have a negative colonoscopy
was later found to have transmural ischemia during cholecystectomy. Also, two patients progressed from Grade 2 to 3 ischemia
during repeat colonoscopy.
Chung et al.7 described ulcerations found endoscopically
to be an independent risk factor of severe IC with an odds ratio
of 9.9 and a 95% CI of 2.0 to 48.8. Houe et al.16 published a review
of prior literature regarding the usefulness of endoscopy following aortic repair. Without objective data, it was concluded based
on seven prospective studies that only an exploratory lapartomy
can conclusively document transmural bowel ischemia, while
endoscopy can merely raise the suspicion.
Answer: Reliable data has shown that colonoscopy can generally be used to determine the likelihood of being full-thickness
disease warranting surgical resection. The limitation is its ability to only visualize the mucosa. (Level 2 evidence, Grade B
recommendation)

PREVENTION
4. Are there any appropriate screening tests?
There is very little literature that describes screening tests to
evaluate IC due to its nature of disease. One may view the use of
colonoscopy in patients at risk (i.e., patients after AAA repair) as
a screening tool. See Question 3 in this chapter for further details
on the utility of colonoscopy in patients with IC.
One study describes a serum marker as a screening tool following aortic repair. Nagata et al.17 studied serum procalcitonin
(PCT) level to assess its value as a screening marker for colonic
ischemia following elective aortic repair in Japan. This prospective study analyzed data from 93 patients where the PCT levels
were determined on post-operative day #2. With a cut-off level of
2.0 ng/ml, its sensitivity and specificity to diagnose IC was 100%
and 83.9% respectively, with a false-negative rate of 0%, and falsepositive rate of 16.1%.
Answer: There is a little evidence in the literature showing
any reliable screening tests are helpful to predict which patients
will develop IC. (Level 3 evidence, Grade C recommendation)

3. Do colonoscopic findings predict disease progression?


Although IC is a clinical diagnosis most frequently aided by visualizing the colonic mucosa via colonoscopy or sigmoidoscopy, it is
important to understand the clinical significance of its findings.
Grades 13 of IC during endoscopic evaluation have been
defined to guide treatment. Mild to moderate disease has been
described as Grades 1 and 2, respectively. Severe disease
described as Grade 3, which shows evidence of full-thickness bowel
wall ischemiais managed with resection of at least that segment
of bowel to control the gangrenous source of illness. Forde et al.14
has documented a reliable relationship between endoscopic and
the histological findings of IC.

PMPH_CH27.indd 233

TREATMENT
5. Do antibiotics make a difference in the management of IC?
Antibiotic use is routinely recommended for the management
of moderate and severe colonic ischemia.18 However, the basis of
such recommendations stems from experimental studies from the
early 20th century and its shared use in other forms of colonic
inflammation.
In 1945, Sarnoff et al.19 demonstrated a decreased incidence
of ischemic necrosis and a 40% improved survival with the use

5/21/2012 9:10:44 PM

234

Surgery: Evidence-Based Practice

of antibiotics in a canine model of mesenteric venous ischemia.19


Poth and McClure20 saw a four-fold improved survival with
antibiotic-treated dogs, using their canine model of mesenteric
arterial ischemia. Animal models have also demonstrated the
utility of antibiotics in minimizing ischemia-associated bacterial
translocation.21 A systematic review by Diaz-Nieto22 reported on
several retrospective case series which included antibiotics in the
conservative management of patients with IC.
The initial management of IC can parallel other forms of colitis. The symptoms and signs of IC (bloody diarrhea, fever, tachycardia and leukocytosis) are also seen in colitis from other causes
(inflammatory bowel disease and infectious). In these settings,
antibiotics are empirically added as the diagnostic work-up is
initiated. In the case of inflammatory bowel disease, progression
to fulminant colitis and toxic megacolon are routine indications
for treatment. A 3- to 5-day course is routinely recommended, at
which time surgery may be necessary if no improvement is seen.
Answer: There are a limited number of retrospective case
series that include antibiotics as part of the treatment of IC. There
is no comparative human data that antibiotic use confers an
advantage. Experimental animal studies suggest decreased bowel
damage, decreased bacterial translocation, and prolonged survival. Because antibiotics are useful in the management of other
severe forms of colitis of which ischemia is part of the differential diagnosis, the recommendation for their routine use remains
sound. (Level 4 evidence, Grade C recommendation)
6. Can primary anastomosis be safely performed during surgical management of IC?
Historically, an emergent colectomy was managed over two or even
three separate procedures. As literature has dictated that a primary
anastomosis can safely be performed following a colonic injury
under certain circumstances, the above question must be asked.
However, due to the vascular nature of IC and the uncertainty of
the colons future blood supply, there is significant risk great risk in
not treating this disease most conservatively.
Biondo et al.23 out of Spain studied the outcomes of 211 patients
who underwent emergent left colon resection and all had primary
anastomoses. Three of these patients were diagnosed with colonic
ischemia as the source of the problem. Of these three patients who
underwent a resection and primary anastomosis, one patient died,

but none were documented as having a dehiscence of the anastomosis. Paterno et al.24 reported performing primary anastomosis
in 17 of 48 patients who had surgery for IC with good results; 13
of these patients had a right hemicolectomy. However, they did
not report whether any of these patients had developed an anastomotic leak or whether any had protective loop ileostomy.
Antolovic et al.25 reviewed 85 prospectively collected cases
in which 20% had resection and primary anastomosis only, 36%
had resection, anastomosis, and diverting ostomy, and 42% had
resection and ostomy. The patients with no stoma formation had a
lower mortality, but this likely is due to a selection bias in treating
the most critical patients with an ostomy.
Answer: There are reports of primary anastomosis during resection for IC both with and without a diverting ostomy.
Patients who are hemodynamically stable with isolated right
colon ischemia may be better candidates when considering this
strategy. However, careful consideration in each case is warranted
when considering primary anastomosis following resection for
IC. (Level 4 evidence, Grade C recommendation)
7. Does reimplantation of the Inferior Mesenteric Artery (IMA)
improve colonic viability following aortic reconstruction?
Senekowitsch et al.26 in a randomized trial out of Austria compares IMA reimplantation with controls after both symptomatic and asymptomatic infrarenal AAA repair. No significant
difference was found when comparing situations where IC was
found.
Of the 128 comparable patients by means of IMA patency
who were randomized to ligation or reimplant, 16 patients had a
histologically proven IC. The IMA was replanted in 6 and ligated
in 10 patients. Ligation of the IMA did not lead to a significantly
elevated risk for developing IC (p = .203).
However, even in endovascular aneurysm repair (EVAR),
where the IMA is universally sacrificed by exclusion, the incidence of clinically symptomatic IC is 1.3% to 2.9%.27 In another
report, Perry et al.28 performed a large outcomes study showed the
incidence of IC after EVAR to be 0.5% compared with 1.9% IC for
open elective repair, and 8.9% for repair of a ruptured AAA.
Answer: Reimplantation of the IMA does not improve colonic
viability following aortic reconstruction. (Level 2 evidence, Grade
B recommendation)

Clinical Question Summary


Question

Answer

Level of
Evidence

Grade of
Recommendation

1 What are the risk factors for


IC?

2 Is there an association of
IC with thrombotic or
thromboembolic conditions?

References

Cardiovascular disease, diabetes mellitus,


shock, peripheral arterial disease,
hypertension, hyperlipidemia, IBS,
constipation, malignancy, aortic and
cardiovascular surgery, dyspepsia,
prolonged laxative use, OCP in young
women.

5-9

Thromboembolic disease may affect the colon


and a search for a cardiac source may be
warranted in at least ambulatory patients
with IC which do not present with an
obvious cause (i.e., low-flow state).

10-13

(Continued)

PMPH_CH27.indd 234

5/21/2012 9:10:44 PM

Ischemic Colitis

235

(Continued)
Level of
Evidence

Grade of
Recommendation

In most cases, yes. Colonoscopy should


continue to be performed for diagnosis
of IC. While it can predict the degree
of disease, it cannot confirm fullthickness colonic injury requiring surgical
intervention.

7, 14-16

4 Are there any appropriate


screening tests?

No. In a rare diseases like IC, tests such


as colonoscopy or blood tests in
asymptomatic patients at risk are not cost
worthy.

17

5 Do Antibiotics make a
difference in the management
of IC?

There are a limited number of retrospective


case series that include antibiotics as part
of the treatment of IC. However, there is
no comparative human data that antibiotic
use confers an advantage. There are animal
data that support antibiotic use in IC.

18-22

6 Can primary anastomosis


be safely performed during
surgical management of IC?

There are reports of primary anastomosis


during resection for IC both with and
without a diverting ostomy. However,
careful consideration in each case is
warranted when considering primary
anastomosis following resection for IC.

23-25

7 Does Reimplantation of
the IMA improve colonic
viability following aortic
reconstruction??

No. No significant difference was found in


a randomized study. EVAR is now being
performed in greater volume without an
increase in ischemic bowel.

26-28

Question

Answer

3 Do colonoscopic findings
predict disease progression?

REFERENCES
1. Marston A, Pheils MT, Thomas ML, Morson BC. Ischemic colitis. Gut. 1966;7:1-15.
2. Brandt L, Boley S, Goldberg L, Mitsudo S, Berman A. Colitis in
the elderly. A reappraisal. A J Gastroenterol. 1981;76:239-245.
3. Greenwald DA, Brandt LJ. Colonic ischemia. J Clin Gastroenterol. 1998;27:122.
4. Guivarch M, Roullet-Audy JC, Mosnier H, Boche O. Ischemic
colitis. [A surgical case series of 88 patients]. J Chir (Paris).
1997;134:103.
5. Longo WE, Ward D, Vernava AM, Kaminski DL. Outcome of patients
with total colonic ischemia. Dis Colon Rectum. 1997;40:1448.
6. Fernandez JC, Calvo LN, Vazquez EG, et al. Risk factors associated with the development of ischemic colits. W J Gastroenterol.
2010;16(36):4564-4569.
7. Chung JW, Cheon JH, Park JJ, Jung ES, Choi EH, Kim H. Development and validation of a novel prognostic scoring model for
ischemic colitis. Dis Colon Rect. 2010;53:1287-1294.
8. Chang L, Kahler KH, Sarawate C, Quimbo R, Kralstein J. Assessment of potential risk factors associated with ischemic colitis.
Neurogastroenterol Motil. 2008;20:36-42.
9. Preventza OA, Lazarides K, Sawyer MA. Ischemic colitis in
young adults: A single-institution experience. J Gastroenterol
Surg. 2001;5:388-392.
10. Midian-Singh R, Polen A, Durishin C, et al. Ischemic colitis
revisited: A prospective study identifying hypercoagulability as
a risk factor. South Med J. 2004;97(2):120-123.

PMPH_CH27.indd 235

References

11. Koutroubakis IE, Sfiridaki A, Theodoropoulou A, Kouroumalis EA.


Role of acquired and hereditary thrombotic risk factors in colonic
ischemia of ambulatory patients. Gastroenterol. 2001;121:56-65.
12. Hourmand-Ollivier I, Bouin M, Saloux E, et al. Cardiac sources
of embolism should be routinely screened in ischemic colitis. Am
J Gastroenterol. 2003;98:1573-1577.
13. Collet T, Even C, Bouin M, et al. Prevalence of electrocardiographic and echocardiographic abnormalities in ambulatory
ischemic colitis digestive diseases and sciences. Dig Dis Sci. 2000:
45(1):23-25.
14. Ford KA. The endoscopy corner: Reversible ischemic colitis
correlation of colonoscopic and pathologic changes. Am J
Gastroenterol. 1979;72(2):182-185.
15. Champagne BJ, Darling RC, Daneshmand M, et al. Outcomes
of aggressive surveillance colonoscopy in ruptured abdominal
aortic aneurysms. J Vasc Surg. 2004;39:792-796.
16. Houe T, Thorboll JE, Sigild U, Liisberg-Larsen O, Schroeder TV.
Can colonoscopy diagnose transmural ischaemic colitis after
abdominal aortic surgery? An evidence-based approach. Eur J
Vasc Endovasc Surg. 2000;19:304-307.
17. Nagata J, Kobayashi M, Nishikimi N, Komori K. Serum procalcitonin (PCT) as a negative screening test for colonic ischemia
after open abdominal aortic surgery. Eur J Vasc Endovasc Surg.
2008;35:694-697.
18. Brandt LJ, Boley SJ. AGA technical review on intestinal ischemia.
Gastroenterology. 2000;118:954-968.
19. Sarnoff SJ, Fine J. Effect of chemotherapy on ileum subjected to
vascular injury. Ann Surg. 1945;121:74-82.

5/21/2012 9:10:44 PM

236

Surgery: Evidence-Based Practice

20. Poth EJ, McClure JN. Intestinal obstruction: The protective


action of sulfasuxidine and sulfathalidine to ileum following
vascular damage. Ann Surg. 1950;131:159-170.
21. Redan JA, Rush BF, Lysz TW, Smith S, Machiedo GW. Organ
distribution of gut-derived bacteria caused by bowel manipulation or ischemia. Am J Surg. 1990;159:85-90.
22. Diaz-Nieto R, Varcada M, Ogunbiyi O, et al. Systematic review
on the treatment of ischaemic colitis. Colorectal Dis. March 30,
2010;(Epub).
23. Biondo S, Pares D, Kreisler E, et al. Anastomotic dehiscence after
resection and primary anastomosis in left-sided colonic emergencies. Dis Colon Rectum. 2005;48:2272-2280.
24. Paterno F, McGollicuddy EA, Schuster KM, Longo WE. Ischemic
Colitis: Risk factors for eventual surgery. Am J Surg. 2010;200:
646-650.

PMPH_CH27.indd 236

25. Antolovic D, Koch M, Hinz U, et al. Ischemic colitis analysis of risk factors for postoperative mortality. Langen Arch Surg.
2008;393:507-512.
26. Senekowitsch C, Assadian A, Assadian O, Hartleb H, Ptakovsky H,
Hagmuller GW. Replanting the inferior mesentery artery during
infrarenal aortic aneurysm repair: Influence on postoperative
colon ischemia. J Vasc Surg. 2006;43:689-694.
27. Mehta M, Roddy S, Darling C, III, et al. Infrarenal
abdominal aortic aneurysm repair via endovascular versus
open retroperitoneal approach. Ann Vasc Surg. 2005;19(3):
374-378.
28. Perry RJ, Martin MJ, Eckert MJ, et al. Colonic Ischemia complicating open vs endovascular abdominal aortic aneurysm repair.
J Vasc Surg. 2008;48:272-277.

5/21/2012 9:10:44 PM

Commentary on Ischemic Colitis


Fred A. Luchette

colon affected by ischemia, and should be the first test utilized for
assessing the extent of the disease. It is the mainstay of diagnosis.

Ischemic colitis (IC), first described by Boley in 1963, remains a diagnostic dilemma despite the advances in radiographic imaging and
laboratory testing.1 This is largely due to nonspecific nature of the
signs and symptoms at the time a patient seeks medical attention. This
evidence-based review classifies the existing literature using seven
practical clinically based questions. Most of the literature investigating these areas consists of Class 2 and 3 studies. However, the authors
have done a good job summarizing the studies of IC from the past
10 years and make appropriate recommendations. My comments will
be focused on the authors approach to those seven questions.

4. Are there any appropriate screening tests?


Numerous other studies are worth mentioning. Various laboratory
markers of ischemia are known to lack sensitivity and specificity
to reliably make the diagnosis of IC, including L-lactate, D-lactate,
lactate dehydrogenase, d-dimer, creatine phosphokinase, amylase, leukocytes, alkaline phosphatase, inorganic phosphate,
acid-binding protein, and -glutathione S-transferase. However,
a recent study used intestinal fatty acid binding protein (i-FABP)
as an early marker of intestinal ischemia. This enzyme is present
in mature enterocytes of the small intestine, with the highest concentration in the villi.3
Additional screening tests that may be useful in evaluating
the vasculature include duplex ultrasound of the splanchnic vessels, contrast-enhanced magnetic resonance angiography, computed tomography (CT) angiography, and digital subtraction
angiography, the gold standard for assessing the vasculature of the
intestine. Recent studies have demonstrated that state-of-the-art
multislice scanners, which offer slice thicknesses of no more than
2 mm, have several advantages over conventional angiography.4,5

1. What are the risk factors for IC?


The authors succinctly identify the various comorbidities, physiologic states, and some medications associated with IC. Additional
classes of medications that predispose to colon ischemia are also
worthy of mentioning, including antibiotics, appetite suppressants
(phentermine), chemotherapeutic agents (vinca alkaloids and taxanes), constipation-inducing agents, decongestants (pseudoephedrine), cardiac glucosides, diuretics, ergot alkaloids, statins, illicit
drugs, immunosuppressive agents, nonsteroidal anti-inflammatory
drugs, psychotropic medications, serotonin agonists/antagonists,
and vasopressors. Other iatrogenic causes for IC, particularly
right-sided ischemia, are in the domain of diagnostic and interventional angiography due to plaque embolization.

5. Do antibiotics make a difference in the management of IC?


Despite the lack of Class 1 studies, our intuition tells us that broadspectrum antibiotics make common physiologic sense as the standard of care for IC to cover both aerobic and anaerobic bacteria. In
the presence of a compromised colonic mucosal barrier, the endogenous microflora of the colon will quickly translocate into the portal
venous circulation, resulting in bacteremia and an increased risk for
hepatic abscess. Antibiotics should be considered adjunctive therapy for ischemic colitis to reduce morbidity and mortality rates. It
is inconceivable to think that any institutional review board would
approve any type of prospective study to address this question.

2. Is there an association of IC with thrombotic or thromboembolic conditions?


Another etiology for the hypercoagulable state is deficiency of
protein Z, an important regulator of coagulation. Plasma concentrations were demonstrated to be significantly lower in patients
who also had factor V Leiden deficiency or antiphospholipid antibodies and a definitive diagnosis of IC.2 Cardiac surgery requiring
cardiopulmonary bypass may be a source of atheromatous plaque
emboli leading to IC.
3. Do colonoscopic findings predict disease progression?

6. Can primary anastomosis be safely performed during surgical management of IC?

The recommendation is that colonoscopy can generally be used to


determine the degree of ischemia on the basis of standard grading criteria and thus identify patients requiring surgical resection
for full-thickness gangrene. However, this recommendation does
not answer the posed question. Rather, colonoscopy allows direct
assessment of the colonic mucosa, the most sensitive area of the

About 20% of patients with acute IC will require an emergent operation with an associated mortality rate of up to 60%, depending on the
overall septic state of the patient.6 Only rarely would a patient who
requires emergent colectomy not also have a perforation with associated
fecal peritonitis, or at least bacterial peritonitis. This clinical situation is
237

PMPH_CH27.indd 237

5/21/2012 9:10:44 PM

238

Surgery: Evidence-Based Practice

completely different from a stab wound to the left colon, which is diagnosed only a few hours after the injury was created and is associated
with minimal contamination in the absence of established peritonitis.
Thus, with IC limited to the right colon without perforation and viable
margins, primary anastomosis is appropriate. However, in the setting of
an emergent operation and a patient with disease that involves the left
colon, the procedure must be directed at correcting the life-threatening
disease with minimal anesthesia time. When the IC is related to aortic
reconstructive surgery, anastomosis is contraindicated because of the
risk of anastomotic leak and subsequent contamination of the aortic
prosthesis. Thus, until there is an appropriate prospective randomized
trial to evaluate the outcome of primary anastomosis versus ostomy for
left colon IC, resection and ostomy remain the appropriate procedures.
7. Does reimplantation of the IMA improve colonic viability
following aortic reconstruction?
One approach to reducing the risk of IC during aortic reconstruction is assessment of the stump pressure index when considering
reimplantation of patent inferior mesenteric arteries.7 This practice
has not been widely embraced. As noted by the authors, routine
reimplantation does not prevent postoperative colonic ischemia
but may be advantageous in patients in whom colonic perfusion
is borderline. Most reports on this topic include small numbers of
patients, which limits the power of the study.

PMPH_CH27.indd 238

REFERENCES
1. Boley SJ, Schwartz S, Lash J, Sternhill V. Reversible vascular occlusion of the colon. Surg Gynecol Obstet. 1963;116:53-60.
2. Koutroubakis IE, Theodoropoulus A, Sfiridaki A, Kouroumalis
EA. Low plasma Z levels in patients with ischemic colitis. Dig Dis
Sci. 2003;48:1673-1676.
3. Lieberman JM, Sacchettini J, Marks C, Marks WH. Human intestinal fatty acid binding protein: Report of an assay with studies
in normal volunteers and intestinal ischemia. Surgery. 1997;121:
335-342.
4. Savastano S, Teso S, Corra S, Fantozzi O, Miotto D. Multislice
CT angiography of the celiac and superior mesenteric arteries:
Comparison with arteriographic fi ndings. Radiol Med. 2002;103:
456-463.
5. Stuercke CA, Haegele KF, Jendreck M, et al. Multislice computed tomography angiography of the abdominal arteries:
comparison between computed tomography angiography and
digital subtraction angiography fi ndings in 52 cases. Australas
Radiol. 2004;48:142-147.
6. Theodoropoulou A, Koutroubakis IE. Ischemic colitis: Clinical practice in diagnosis and treatment. World J Gastroenterol.
2008;14(48):7302-7308.
7. Van Damme H, Creemers E, Limet R. Ischaemic colitis following
aortoiliac surgery. Acta Chir Belg. 2000;100:21-27.

5/21/2012 9:10:44 PM

CHAPTER 28

Pseudomembranous Colitis
Burke Thompson

INTRODUCTION

data have shown organisms producing toxin B alone can cause


pseudomembranous colitis, but these strains only account for 7%
to 10% of cases.10 A binary toxin called C. difficile transferase is
found in more virulent strains.

Pseudomembranous colitis, also known as antibiotic-associated


colitis is an inflammatory process of the colon that is almost
always associated with an overgrowth of the bacterium Clostridium difficile. C. difficile is an anaerobic, gram-positive bacterium
that is highly transmissible. It was first described in 1935 as a colonizing organism in children.1 Later, it became recognized as a
widespread environmental organism located in soil and mud. It
was also found in dung of many large mammals.2 Pseudomembranous colitis was initially described by Finney in 18933 but was
not linked to any bacterial involvement until 1978.4 This discovery
followed a flurry of study in the late 1970s.
Pseudomembranous colitis is characterized by several symptoms including diarrhea with mucous, crampy abdominal pain,
nausea, fever, and dehydration. There is often an associated ileus.
Colonoscopy shows characteristic pseudomembranes on the
colonic mucosa. The disease can progress, even with treatment,
and complications such as toxic megacolon and sepsis with mulitorgan failure can be lethal. It is the leading cause of health-care
associated diarrhea and was estimated to total USD1.3 billion
in U.S. health-care costs in 2002. This amounted to USD12,825
per hospital admission.5 Further study in 2009 included hospitalizations for recurrent disease and totaled USD3.4 billion.6 The
incidence and associated costs are increasing. C. difficile colitis
is responsible for nearly 350,000 admissions annually.7 This is a
focus of American hospital systems, as Medicare reimbursement
is now limited for many hospital-acquired complications.

2. How do you diagnose C. difficile colitis?


In the clinical setting of diarrhea associated with current hospitalization or recent antibiotic therapy, diagnosis of C. difficile colitis
is critical to guide treatment. Patients have watery diarrhea often
with mucous of blood. They may have as many as 10 stools a day or
more. C. difficile colitis diarrhea has a characteristic odor, familiar
to nurses caring for these patients. On examination, the abdomen
may be distended and tender. Leukocytosis can be significant.
Colonoscopy or sigmoidoscopy can show pseudomembranes on
the colonic mucosa. Biopsies can be taken to confirm the diagnosis. Histologic findings include a volcano lesion representing
an eruption of fibrinopurulent exudates and debris in an area of
denuded mucosa.11 The current mainstay of diagnosis starts with
the demonstration of C. difficile toxins in a diarrhea sample.12 This
alone is not always definitive. Full diagnosis includes stool culture,
detection of toxin in stool, and detection of C. difficile antigen.
Culture detection of C. difficile in the stool may not confi rm active disease. Toxin detection in the stool is required to be
certain. C. difficile toxin A and toxin B are now detected using
enzyme immunoassays (ELISA). The sensitivity of this test for
the diagnosis of C. difficile colitis ranges from 63% to 94% with a
specificity between 75% and 100%.7 The positive predictive value
varies according to the incidence of infection in the population
so many hospital systems require repeated testing (two or three
consecutive samples) to confi rm diagnosis. Antigen testing by
ELISA has a higher sensitivity but a lower specificity than toxin
ELISA. Some facilities use the antigen test as a screen. If it is
positive, however, a confi rmatory toxin ELISA must be done.
Recently, a toxin B polymerase chain reaction (PCR) test has
been tested;7 it is still in development and is not yet commercially available. Th is test has shown promise with high sensitivity and specificity.

1. What is the pathogenesis of pseudomembranous colitis?


C. difficile produces toxins that lead to its pathogenic effects; the
two most important are toxin A and toxin B.8,9 These have been
linked to genes that are absent on nonpathogenic strains. Toxin A
and toxin B are cytotoxins causing glycosylation of proteins
in the cytoskeleton. This loosens the tight junctions between
colonic cells causing profound secretory diarrhea.7 The majority
of pathogenic strains produce both toxin A and toxin B. Recent
239

PMPH_CH28.indd 239

5/21/2012 9:11:18 PM

240

Surgery: Evidence-Based Practice

3. What patients are at risk for developing pseudomembranous


colitis?
Intrinsic risk factors for pseudomembranous colitis include
advanced age and number of comorbidities. The rate of pseudomembranous colitis in people over 65 years is 20 times higher than in
younger people.13 Increased number of comorbidities increases a
patients exposure to health-care facilities and therefore exposure
to C. difficile.
Extrinsic risk factors are more familiar. The exposure to
broad-spectrum antibiotics is the most well-recognized risk
factor.14-16 One study, a retrospective review of 1384 patients
with pseudomembranous colitis, showed that two-thirds had
received antibiotics within the 2-month period prior to diagnosis.7 Another study showed that 94% of hospitalized patients
with pseudomembranous colitis had exposure to antibiotics
before or during hospitalization.17 The first antibiotic linked to
pseudomembranous colitis is clindamycin. Even though other
antibiotics, including metronidazole, have also been associated
with pseudomembranous colitis, clindamycin is one of the main
agents used in treatment. The usual antibiotic responsible for
C. difficile colitis parallels the common, in vogue, antibiotics used.
Initially these were clindamycin, cephalosporins, and amoxicillin.
More recently, fluoroquinolones have been linked. Pepin et al.18
demonstrated the current hazard risk associated with using fluoroquinolones was significantly higher than for clindamycin, macrolides, or cephalosporins.
Residence at acute care hospitals and long-term care facilities
is also a strong risk factor for C. difficile colitis. The colonization in
such populations is 20% to 40%, which is much higher than in the
community.7 Patient-to-patient transfer is suspected. Many facilities use isolation procedures to try to prevent spread. Length of
stay in such facilities has also been associated with increased risk.
Debast et al.19 showed a 4% increased risk of C. difficile colitis with
each day of hospitalization. Dial et al. showed that hospitalization
longer than a week increases the risk by fivefold.
Immunosuppression also increases the risk of developing
pseudomembranous colitis.16,20 Disruption of the normal colonic
barrier and malnutrition in these patients may exacerbate this
added risk. Similarly, patients with Crohns disease or ulcerative
colitis have some increased risk of C. difficile colitis and may have
higher associated morbidity and mortality.21
Gastric acid suppression has also been implicated in increasing the risk of C. difficile colitis. While there is no Level 1 data, a
casecontrol study by Dial et al.22 reviewed 1672 cases in the U.K.
matching each case to 10 healthy controls. Multivariate analysis
showed that current use of proton pump inhibitors lead to a threefold increased risk of C. difficile colitis. Given the other benefits of
proton pump inhibitor therapy, it was not recommended to stop
the therapy based on this finding. Other comorbidities may be a
source of confounding in the studied population.

MILD TO MODERATE DISEASE


These patients are clinically stable. The antibiotic of concern should
be stopped and the patients should be fluid resuscitated. Supportive care with attention to electrolytes and nutrition is important.
Strong consideration should be given to starting metronidazole
500 mg orally three times a day for 10 to 14 days. Antiperistaltic
agents should be avoided as these may lead to toxic megacolon.24,25

SEVERE DISEASE

4. How do you treat pseudomembranous colitis?

There is a lack of Level 1 data and consensus on defining severe


disease in pseudomembranous colitis. An attempt to define severe
pseudomembranous colitis was made by the UK Department of
Health Steering Group on Healthcare Associated Infection.26 Signs
of severe disease include white blood cell count over 15,000, rising
creatinine, fever, and signs on physical examination and radiography of colitis. Metronidazole has been shown to have a significant treatment failure rate. Oral vancomycin 125 mg orally four
times a day for 10 to 14 days should be used in these patients.27,28
Zar et al.29 showed that the rate of cure for pseudomembranous
colitis treated with vancomycin was 97% compared to that of metronidazole which was 76%. The reason for this is thought to be
that oral vancomycin is not as readily absorbed by the gut so it
acts intraluminally in the colon.
Severe cases that do not respond to oral vancomycin can be
given higher doses, up to 500 mg four times a day via nasogastric
tube. Vancomycin enemas may also be given. Intravenous metronidazole 500 mg four times a day is also recommended.30,31 Oral
rifampicin 300 mg twice daily and intravenous immunoglobulin
400 mg/kg may also be given. The data to support these therapies are far from robust, but the poor prognosis may justify trying these modalities. Patients not responding to medical therapy
should be seen by a surgeon.
Alternate therapies have been investigated with limited success. Agents include fusidic acid, teicoplanin, nitazoxanide, and
rifaximin.32 Probiotics have failed to demonstrate efficacy.31
Anion-binding resins including oral cholestyramine do not work
as primary agents but may have some benefit as an adjunct, especially in recurrent cases.33 These agents bind the C. difficile toxin
in the gut and have the benefit of not altering gut flora. A new
toxin-binding resin tolevamer has been developed to specifically
treat C. difficile colitis.33,34 Trials are ongoing, and this agent is not
yet commercially available. Fecal bacteriotherapy has been shown
in a limited study by Bakken35 to have a success rate of nearly 90%
in patients with recurrent disease. In this procedure, donor stool is
given as an enema with varying volumes investigated. Administration via a nasoduodenal tube is also described in this study.
It is hoped that commercial artificial stool products might be
developed in the future. Vaccines have been successful in animal
studies, and early human data are encouraging.36

Once diagnosed, it is helpful to stratify patients to guide


therapy.23

EMERGENCE OF A VIRULENT STRAIN

ASYMPTOMATIC PATIENTS
No therapy is required. These patients must be closely followed for
changes in clinical status that may warrant therapy.

PMPH_CH28.indd 240

A hypervirulent strain has been identified in several outbreaks


of C. difficile-associated diarrhea.37,38 This strain was identified as
ribotype 27, or B1/NAP1/027, by pulse-field electrophoresis and
restriction endonuclease analysis. A study from Quebec identified
this new strain in 80% of patients in a severe outbreak. It had been

5/21/2012 9:11:19 PM

Pseudomembranous Colitis

previously isolated in 1.8% of cases at that facility.39 The virulent


strain leads to twice the normal mortality compared with the
other strain. This virulent strain has been reported in the United
States and also in other parts of the world.40 Some studies refute
the reported increased morbidity and mortality attributed to this
strain.41,42 This virulent strain is responsible for some nosocomial
epidemics and will play an important ongoing role.
5. When do you operate on pseudomembranous colitis?
There is a lack of Level 1 data to direct the timing of surgical intervention for pseudomembranous colitis. Large-scale randomizedcontrolled studies have not been done. That being said, there are
some published opinions, guidelines, and recommendations. The
procedure of choice is subtotal colectomy with end ileostomy. At
operation, parts of the colon may appear normally externally, but
it is important to remove all of the colon down to the distal rectum
because the disease begins on the mucosa first and progresses to a
full thickness process. A small retrospective study by Koss et al.43
found indications for surgery to include peritonitis, systemic

241

toxicity, toxic megacolon, and perforation. Synnott et al.44 did a


similar study determining the indications for surgery to include
systemic toxicity with pyrexia, marked leukocytosis, and progressive organ failure despite appropriate medical management.
The patients with fulminant pseudomembranous colitis that
require operation have a mortality of 33% to 57%.45 Mortality
is reduced if surgery is done before a long period of pressor therapy is required. A multidisciplinary panel of experts convened
to clarify the surgeons role in C. difficile colitis. These guidelines were published in Infection Control and Hospital Epidemiology.46 They state colectomy should be considered for severely ill
patients. Serum lactate levels and white blood cell count were
found to be useful guides. Lactate level of 5 mmol/L and white
blood cell count of 50,000 cells/ml were found to be associated
with increased perioperative mortality. Subtotal colectomy with
ileostomy, as described above, is the recommended operation.
The timing of surgery remains unclear. Certainly, once sepsis
and organ failure set in, it is indicated. Optimal surgical timing
would be just prior to this, which is obviously often difficult to
predict.

Clinical Question Summary


Question

Answer

Grade of
Recommendation

1 How do you diagnose Clostridium


dificile colitis?

Use ELISA to detect toxin A and toxin B

References
7, 12

Consider PCR once available.

2 How do you treat asymptomatic


pseudomembranous colitis?

No treatment

24, 25

3 How do you treat mild to moderate


pseudomembranous colitis?

Stop antibiotic of concern.

24, 25

Resuscitate with fluids and correct electrolytes.

24, 25

Metronidazole 500mg orally three times a day.

24, 25

Oral vancomycin 125mg four times a day for 10 to


14 days.

29

Give IV metronidazole 500mg four times a day.

26, 30

In refractory cases, give vancomycin enemas four


times a day.

26, 30

In recurrent cases, consider fecal bacteriotherapy.

35

Perform subtotal colectomy and ileostomy once


there is evidence of perforation, severe sepsis,
or organ failure.

43, 44, 45, 46

4 How do you treat severe


pseudomembranous colitis?

5 When do you operate on


pseudomembranous colitis?

REFERENCES
1. Hall JC, OToole E. Intestinal flora in new-born infants with
a description of a new pathogenic anaerobe, Bacillus difficilis.
Am J Dis Child. 1935;49:390-402.
2. Hafiz S. Clostridium difficile and its toxins [PhD Thesis]. Leeds:
Department of Microbiology, University of Leeds; 1974.
3. Larson HE, Parry JV, Price AB, Davies DR, Dolby J, Tyrrell DA.
Undescribed toxin in pseudomembranous colitis. Br Med J. 1977;
1:1246-1248.
4. Finney JMT. Gastroenterostomy for cicatrizing ulcer of the pylorus. Johns Hopkins Med J. 1893;4:53-55.

PMPH_CH28.indd 241

5. Kyne L, Hamel MB, Polavaram R, Kelly CP. Healthcare costs associated with nosocomial diarrhea due to Clostridium difficile. Clin
Infect Dis. 2002;34:346-353.
6. Dubberke ER, Wertheimer AI. Review of current literature on the
economic burden of Clostridium difficile infection. Infect Control
Hosp Epid. 2009;30:57-66.
7. Ananthakrishnan AN. Clostridium difficile infection: Epidemiology, risk factors and management. Nat Rev Gastroenterol Hepatol.
2011;8:17-26.
8. Rupnik M, Wilcox, MH, Gerding DN. Clostridium difficile infection: New developments in epidemiology and pathogenesis. Nat
Rev Micro. 2009;7:526-536.

5/21/2012 9:11:19 PM

242

Surgery: Evidence-Based Practice

9. Voth DE, Ballard JD. Clostridium difficile toxins: Mechanism of


action and role in disease. Clin Micro Rev. 2005;18:247-263.
10. Drudy D, Harnedy N, Fanning S, et al. Isolation and characterization of toxin A-negative, toxin B-positive Clostridium difficile
in Dublin, Ireland. Clin Micro Infect. 2007;13:298-304.
11. Fekety RN, Shah AB. Diagnosis and treatment of Clostridium
difficile colitis. JAMA. 1993;269:71-75.
12. Wren, M. Clostridium difficile isolation and culture techniques.
Clostridium difficile, Methods in Molecular Biology, Mullany P
and Roberts AP, eds. Springer Science+Business Media; 2010.
13. Bartlett JG, Gerding DN. Clinical recognition and diagnosis of
Clostridium difficile infection. Clin Infect Dis. 2008;46(Suppl 1):
S12-S18.
14. Gerding DN. Clostridium difficile 30 years on: What has or has not
changed and why? Int J Antimicrob Agents. 2009;33(Suppl 1):S2-S8.
15. Gerding DN, Johnson S, Mulligan ME, Silva J, Jr. Clostridium
difficile associated diarrhea and colitis. Infect Contr Hosp Epidemiol. 1995;16:459-477.
16. Diggs NG, Surawicz CM. Evolving concepts in Clostridium difficile colitis. Curr Gastroent Rep. 2009;11:400-405.
17. Owens RC, et al. Antimicrobial associated risk factors for Clostridium difficile colitis. Clin Infect Dis. 2008;46(Suppl 1):S19-S31.
18. Pepin J, et al. Emergence of fluoroquinolones as the predominant risk factor for Clostridium difficile-associated diarrhea:
a cohort study during an epidemic in Quebec. Clin Infect Dis.
2005;41:1254-1260.
19. Debast SB, et al. Successful combat of an outbreak due to
Clostridium difficile ribotype 027 and recognition of specific risk
factors. Clin Micro Infect. 2009;15:427-434.
20. Cohen SH, et al. Clinical practice guidelines for Clostridium difficile infection in adults. 2010 update by the Society of Healthcare
Epidemiology of America and the Infectious Diseases Society of
America. Infect Contr Hosp Epidemiol. 2010;31:431-455.
21. Issa M, et al. Impact of Clostridium difficile on inflammatory
bowel disease. Clin Gastroenterol Hepatol 2007;5:345-351.
22. Dial S, Alrasadi K, Manoukian C, et al. Risk of Clostridium difficile diarrhea among hospital inpatients prescribed proton pump
inhibitors: Cohort and case control studies. Can Med Assoc J.
2004;171:33-38.
23. Faris B, Blackmore A, Haboubi N. Review of medical and surgical management of Clostridium difficile infection. Tech Coloproctol. 2010;14:97-105.
24. Novak E, Lee JG, Seckman CE, et al. Unfavorable effect of atropinediphenoxylate (Lomotil) therapy in lincomycin caused
diarrhea. JAMA. 1976;235:1451-1454.
25. Poutanen SM, Simor AE. Clostridium difficile associated diarrhea in adults. Can Med Assoc J. 2004;171:51-58.
26. Department of Health. Clostridium difficile infection: How to
deal with the problem, a board to ward approach. Department of
Health Steering Group on Healthcare Associated Infection. 2008
Department of Health; Gateway reference 9833.
27. Louie T, Gerson M, Grimard D, et al. Results of a phase three
trial comparing tolevamer, vancomycin, and metronidazole
in patients with Clostridium difficile associated diarrhea. 47th
Interscience Conference on Antimicrobial Agents and Chemotherapy, Chicago; 2007.
28. Wilcox MH, Howe R. Diarrhoea caused by Clostridium difficile:
response time for treatment with metronidazole and vancomycin. J Antimicrob Chemother. 1995;36:673-679.

PMPH_CH28.indd 242

29. Zar FA, Bakkanagari SR, Moorthi KM, Davis MB. A comparison of vancomycin and metronidazole for the treatment on
Clostridium difficile-associated diarrhea, stratified by disease
severity. Clin Infect Dis. 2007;45:302-307.
30. Bartlett JG. Clinical practice: Antibiotic-associated diarrhea.
N Eng J Med. 2002;346:334-339.
31. Pillai A, Nelson R. Probiotics for treatment of Clostridium difficile associated colitis in adults. Cochrane Database Systematic
Review. 2008;CD004611.
32. Nelson R. Antibiotic treatment for Clostridium difficile associated diarrhea in adults. Cochrane Database Systematic Review.
2007;CD004610.
33. Tedesco FJ. Treatment of recurrent antibiotic associated
pseudomembranous colitis. Am J Gastroenterol. 1982;77:
220-221.
34. Louie TJ, Peppe J, Watt CK, et al. Tolevamer, a novel non-antibiotic
polymer, compared with vancomycin in the treatment of mild
to moderately severe Clostridium difficile associated diarrhea.
Clin Infect Dis. 2006;43(4):411-420.
35. Bakken JS. Fecal bacteriotherapy for recurrent Clostridium difficile infection. Clin Micro. 2009;15:285-289.
36. McMaster-Baxter NL, Musher DM. Clostridium difficile: Recent
epidemiologic findings and advances in therapy. Pharmacotherapy. 2007;27:1029-1039.
37. Loo VG, Poirier L, Miller MA, et al. A predominately clonal
multi-institutional outbreak of Clostridium difficile associated
diarrhea with high morbidity and mortality. N Engl J Med.
2005;353(23):2442-2449.
38. McDonald LC, Killgore GE, Thompson A, et al. An epidemic,
toxin-gene variant strain of Clostridium difficile. N Engl J Med.
2005;353(23):2433-2441.
39. Labbe AC, Poirier L, Maccannell D, et al. Clostridium difficile
infections in a Canadian tertiary care hospital before and during a regional epidemic associated with the B1/NAP1/027 strain.
Antimicrob Agents Chemother. 2008;52(9):3180-3187.
40. Khanna S, Pardi DS. The growing incidence and severity of
Clostridium difficile infection in inpatient and outpatient settings. Expert Rev Gastroenterol Hepatol. 2010;4(4):409-416.
41. Cloud J, Noddin L, Pressman A, Hu M, Kelly C. Clostridium
difficile strain NAP1 is not associated with severe disease in a
non-epidemic setting. Clin Gastroenterol Hepatol. 2009;7(8):
868-873.
42. Morgan OW, Rodrigues B, Elston T, et al. Clinical severity of
Clostridium difficile PCR ribotype 027: A case-case study. PloS
One. 2008;3(3):1812.
43. Koss K, Clark MA, Sanders DSA, Morton D, Keighley MRB, Goh
J. The outcome of surgery in fulminant Clostridium difficile colitis. Col Dis. 2005;8:149-154.
44. Synnott K, Mealy K, Merry C, Kyne L, Keane C, Quill R. Timing
of surgery for fulminating pseudomembranous colitis. Br J Surg.
1998;85:229-231.
45. Dallal RM, Harbrecht BG, Boujoukas AJ, et al. Fulminant
Clostridium difficile: An underappreciated and increasing cause
of death and complications. Ann Surg. 2002;235(3):363-372.
46. McDonald LC, Coignard B, Dubberke E, Song X, Horan T, Kutty
PK; Ad Hoc Clostridium difficile Surveillance Working Group.
Recommendations for surveillance of Clostridium difficileassociated disease. Infect Control Hosp Epidemiol. 2007;28(2):
140-145.

5/21/2012 9:11:19 PM

Commentary on
Pseudomembranous Colitis
Pamela A. Lipsett

This chapter addresses five important questions regarding the recognition, identification, diagnosis of patients with pseudomembranous colitis, and the medical and surgical therapy. Clostridium
difficile-associated diarrhea (CDAD) is a major cause of nosocomial and antibiotic-associated diarrhea (AAD) (1030%), and it
is the commonest recognized cause of pseudomembranous colitis
(96100%). The overall attack rate of AAD in hospitals is 3.2% to
29%, with an incidence among hospitalized and long-term care
patients of 25 to 60 cases per 100,000 bed-days. The identification
of patients with CDAD is important whether or not the patients
are symptomatic, as C. difficile is easily transmissible, with spores
long-lasting in the environment. Further, hand hygiene with commonly used gels is not effective at killing these spores.
Rapid identification of those patients with CDAD as opposed
to another cause of systemic illness is essential. Identification of
CDAD as a definitive cause of infectious signs and symptoms is
especially important in critically ill patients and postoperative
patients who have many alternative potential causes of systemic
signs of inflammation, and even potentially organ failure. Has
this chapter been fully clear in educating us about the most current thoughts about diagnostic strategies in this rapidly evolving
field? Perhaps some additional information is needed.
Methods currently in use for the detection of C. difficile toxin
include toxigenic culture, cytotoxicity assay, initial screening
with glutamate dehydrogenase (GDH) antigen tests with positive
screens followed by subsequent assays to detect toxins A and B, and
most recently molecular assays to detect the tcdB gene.1-4 Because
the sensitivity and specificity of these diagnostic tests vary greatly,
it is essential for the clinician to know which test their laboratory uses and to what extent the test will reject the possibility of

Diagnostic test*

C. difficile infection, or confirm the presence of the pathogen and


toxin.1-4 This chapter does not describe the previously commonly
used nonmolecular two-step diagnostic method, and only briefly
mentions the possible use of PCR for diagnosis. A two-step protocol has been recommended: testing for an abundant C. difficile
antigen, glutamate dehydrogenase (GDH), by a rapid and sensitive
enzyme-linked immunosorbent serologic assay (ELISA), followed
by cytotoxin testing of GDH-positive samples to confirm toxin
production in vivo. This method achieves relatively high sensitivity and specificity and can rapidly report results for most samples
that are negative for C. difficile but can still take up to 48 h to report
low-level cytotoxin positivity. The first commercially available realtime PCR assay (the BD GeneOhm Cdiff assay; BD Diagnostics,
San Diego, CA) was approved in 2008 to directly detect the toxin
B (tcdB) gene in stool to aid in the diagnosis of CDAD. There are
now at several commercially available molecular methods to diagnose C. difficile toxin.3 The estimated sensitivity and specificity
of these tests are shown in the table below. Generally speaking,
when compared with nonmolecular methods, molecular methods detect 30% to 35% more positive specimens. Further the
methods compare very favorably with the gold standard of the
toxigenic culture and they are available much more quickly for
clinical decision making.4
Some patients with C. difficile will not have diarrhea and
will simply have a distended abdomen, leuckocytosis, and signs
of systemic illness. As part of the search for an etiology, patients
may undergo computerized tomographic (CT) scans, which may
demonstrate unexpected colonic disease. The scan may show, in
the case of fulminant colitis, edematous and thick-walled colon
with thumbprinting. Other findings could include the presence

Sensitivity

Specificity

Predictive Value
Positive

Negative

GeneOhm Cdiff assay (PCR)

9296

94100

100

9798

Xpert C. difficile test (PCR)

9496

9697

9293

9699

ProGastro Cd assay (PCR)

7792

9598

6994

9599

Quick Chek Complete

6284

96100

52100

8599

GDH/EIA

4290

90100

99100

7994

*Compilation of Refs [1-4].

243

PMPH_CH28.indd 243

5/21/2012 9:11:19 PM

244

Surgery: Evidence-Based Practice

of pericolic stranding, ascites, pancolitis, and megacolon. These


findings should prompt consideration of C. difficile as a potential
etiology.
The most recent Cochrane analysis addressing antibiotic treatment for CDAD suggests there is uncertainty about whether mild
CDAD requires treatment.5 Further most studies exclude patients
with severe disease and have a high risk of bias. This evidencebased review suggests that vancomycin may not be better than the
other proposed treatments. However, current expert opinion does
clearly prefer vancomycin for severe disease. We should further
emphasize the importance of not treating asymptomatic patients.
There is growing concern about resistance to metronidazole and
expert opinion suggests this should not be used for first line treatment in severely ill patients.
Several novel therapies have been studied in randomizedcontrolled trials, mostly for prevention of recurrence of CDAD.
For example, the use of fidaxomicin versus vancomycin for the
treatment of recurrent C. difficile has shown that the rates of clinical cure were not inferior to those with vancomycin (88.2% with
fidaxomicin and 85.8% with vancomycin).6 However, there were
fewer patients with recurrence in patients who received fidaxomicin (15.4%) versus vancomycin (25.3%). The role of fidaxomicin in
primary CDAD is currently uncertain. Further, two monoclonal
antibodies against toxin A and toxin B were tested against placebo
in a randomized control trial with an endpoint of recurrence at
84 days.7 Patients who were treated with antibodies had a recurrence rate of 7% versus 25% (p < .0001).
Perhaps the most difficult decision for a surgeon to make is
the one to proceed to surgical intervention in the setting of severe
CDAD disease. While the indication for operation is obvious in
the presence of perforation or peritonitis (which are rare), the
decision to operate and the timing of such have not emanated
from clinical trials, but rather from reports of clinical experience. Using lactate and white blood cell elevations as prompters
for invasive action comes from reports that have used statistical
methods in case series where favorable and unfavorable outcomes
were used to determine important predictive measures. Expert
opinions considered these recommendations to aide providers as
to when an operation may be indicated. However, these few clinical features simplify a complex and integrated decision process.
In a severely ill patient in whom CDAD is considered likely or is
confirmed, medical therapy (vancomycin and intravenous metronidazole) and resuscitation should be initiated immediately. If
the trajectory of illness is not abated or is worsening within 24 h,
operative intervention should be considered. As correctly noted
by Thompson, the colon is often normal appearing externally
and one may be tempted to reconsider the operative plan. While
the operative procedure of choice has not been studied prospectively, the series with the best outcomes over the past 10 years
have performed a subtotal colectomy. Often the right colon is
quite severely diseased and a more selective left-sided colectomy
has been associated with a high mortality in some series. Having said this, a novel approach has been recently reported. Th is
approach utilized laparoscopic ileostomy and on-table irrigation

PMPH_CH28.indd 244

with 8 L of PEG3350/electrolyte colonic irrigation, followed by


postoperative antegrade infusion of colonic vancomycin. 8 Using
a historical controls who underwent open surgery and colon
resection, mortality was substantially lower (19% vs. 50%), and
patients were successfully managed without resection (93%)
with this less-invasive approach. Th is new procedure was performed in patients who were quite ill and appeared to have many
of the risk factors present and in whom resection was previously
advised. Thus, while no high-level-evidencebased recommendations can be made, this new approach seems to be reasonable
in skilled hands.
Finally, the prudent use of antibiotics for defi ned infections with shorter duration and using antibiotics for accepted
prophylaxis may well help prevent a significant number of
C. difficile infections. Further, attention to appropriate infection control measures to limit the spread of disease may well
address the rising rates of C. difficile.

REFERENCES
1. Chapin KC, Dickenson RA, Wu F, Andrea SB. Comparison of
five assays for detection of Clostridium difficile toxin. J Mol Diag.
2011;13(4):395-400. Epub April 29, 2011.
2. Naaber P, Stsepetova J, Smidt I, Rtsep M, Kljalg S, Livukene K,
Jaanime L, Lhr IH, Nats OB, Truusalu K, Sepp E. Quantification of Clostridium difficile in antibiotic-associated diarrhea patients. J Clin Microbiol. August 24, 2011. [Epub ahead
of print.]
3. Tenover FC, Baron EJ, Peterson LR, Persing DH. Laboratory diagnosis of clostridium difficile infection can molecular amplification
methods move us out of uncertainty? J Mol Diag. August 17, 2011.
[Epub ahead of print.]
4. Kvach EJ, Ferguson D, Riska PF, Landry ML. Comparison of BD
GeneOhm Cdiff real-time PCR assay with a two-step algorithm
and a toxin A/B enzyme-linked immunosorbent assay for diagnosis of toxigenic Clostridium difficile infection. J Clin Microbiol.
2010;48(1):109-114. Epub October 28, 2009.
5. Nelson RL, Kelsey P, Leeman H, Meardon N, Patel H, Paul K, Rees R,
Taylor B, Wood E, Malakun R. Antibiotic treatment for Clostridium difficile-associated diarrhea in adults. Cochrane Database Systematic Review. September 7, 2011;9:CD004610.
6. Louie TJ, Miller MA, Mullane KM, Weiss K, Lentnek A, Golan Y,
Gorbach S, Sears P, Shue YK; OPT-80-003 Clinical Study Group.
Fidaxomicin versus vancomycin for Clostridium difficile infection.
N Engl J Med. 2011;364(5):422-431.
7. Lowy I, Molrine DC, Leav BA, Blair BM, Baxter R, Gerding DN,
Nichol G, Thomas WD, Jr., Leney M, Sloan S, Hay CA, Ambrosino
DM. Treatment with monoclonal antibodies against Clostridium
difficile toxins. N Engl J Med. 2010;362(3):197-205.
8. Neal MD, Alverdy JC, Hall DE, Simmons RL, Zuckerbraun
BS. Diverting loop ileostomy and colonic lavage: An alternative to total abdominal colectomy for the treatment of severe,
complicated Clostridium difficile associated disease. Ann Surg.
2011;254(3):423-429.

5/21/2012 9:11:19 PM

CHAPTER 29

Colon and Rectal Cancer


Including Adjuvant
Robert P. Sticca, Erik G. Fetner, and Jay M. MacGregor

INTRODUCTION

INITIAL EVALUATION AND DIAGNOSIS

Colorectal cancer is both a national and worldwide health problem. In 2008, it was estimated to affect 1,235,208 people worldwide1 and in 2010 it was estimated to affect 142,570 people in
the United States.2 Although in recent years, US statistics have
demonstrated improvements in survival after initial diagnosis
of colorectal cancer, it still remains the second leading cause of
cancer-related death, accounting for an estimated 51,370 deaths
in 2010.1 Worldwide there was an estimated 609,051 deaths2 from
colorectal cancer in 2008. Risk factors that have been cited for
colorectal cancer include age >50 years, cigarette smoking, excessive red meat consumption, lack of dietary fiber, obesity, sedentary lifestyle, excessive alcohol consumption, inflammatory bowel
disease, radiation exposure, and genetic predisposition. The exact
etiology of sporadic colorectal cancer is unknown and is probably
multifactorial. Approximately 20% of cancers of the colon and rectum are felt to be attributable to hereditary syndromes including
familial adenomatous polyposis (FAP), attenuated FAP (AFAP),
hereditary nonpolyposis colorectal cancer (HNPCC or Lynch syndrome), and familial colorectal cancer. Intensive research over the
past two decades coupled with the discovery of specific mutations
in the hereditary colorectal cancer syndromes have elucidated the
steps in carcinogenesis of colorectal cancer.
Once established, colorectal cancer can spread through four
different pathways including direct extension to other organs, lymphatic metastasis to regional lymph nodes, intraperitoneal metastasis, and hematogenous metastasis to distant organs. The principle
mode of therapy for colorectal cancer is surgical excision, which
is designed to remove the primary tumor and regional lymph
nodes. The anatomy of the blood supply and lymphatic drainage
of the colon and rectum dictate the type and extent of resection for
colorectal cancers. Adenocarcinomas of the rectum have a worse
prognosis than their colonic counterparts when comparing similar
stages, due to the lack of a serosal covering of the rectum and the
enclosed spaces of the pelvis. Several questions in the management
of colorectal carcinoma have engendered controversy, some due to
new technology and others due to conflicting or inadequate data.

1. What is the best preoperative staging method for locoregional disease in rectal cancer?
Advances in imaging have greatly improved the preoperative
evaluation of patients diagnosed with colorectal cancer. The accuracy, applicability and cost-effectiveness of these technologies to
the management of rectal cancer have been studied extensively.
Accurate staging of rectal cancer is necessary for prognosis, management, and the evaluation of therapeutic response. Endorectal
ultrasound (ERUS) is the most accurate method for assessment of
invasion of rectal carcinoma into the rectal wall (T-stage), which
remains the most important factor in treating patients with rectal
cancer.3 The accuracy of ERUS for T-staging is 80% to 95% versus
65% to 75% for computed tomography (CT) and 75% to 85% for
magnetic resonance imaging (MRI).4 Advantages of ERUS include
cost-effectiveness, well tolerated by patients, able to perform without anesthesia, ability to accurately measure size, circumference,
and distance of tumor from anatomic landmarks, and the ability
to examine anal sphincters.5
The sensitivity and specificity of ERUS vary by T-stage. Recent
meta-analysis reports an ERUS sensitivity and specificity of 88%
and 98% for T1, 81% and 96% for T2, 96% and 91% for T3, and 95%
and 98% for T4.6 T2 lesions are subject to overstaging by ERUS,
which may be related to classification of peritumoral fibrosis as
tumor, hematomas from associated preoperative biopsies, and
fear of understaging.
The accuracy of ERUS is directly correlated to the skill of the
operator. Garcia-Aguilar report a 10% to 15% difference in T-staging
between three board-certified colorectal surgeons.7 With training, ERUS accuracy has been shown to increase from 50% to over
90%.8 Interestingly, more recent data and larger sample sizes generally show a decrease in the accuracy of ERUS. A 2002 study of
more than 400 patients found ERUS to be less than 70% when less
experienced practitioners were performing the ultrasonography.8
A 2005 meta-analysis of ERUS showed accuracy of 85% with a
trend toward decreased accuracy in more recent studies.9 These
data suggest accuracy of ERUS may be highly correlated with
245

PMPH_CH29.indd 245

5/21/2012 9:11:55 PM

246

Surgery: Evidence-Based Practice

operator experience. There appears to be a learning curve of 50


cases for interpretation of tumor depth and greater than 75 cases
for node assessment when using ERUS.8
A meta-analysis by Puli et al.10 found the nodal sensitivity and
specificity of ERUS to be 73.2% and 75.8%, respectively. There is a
wide variance in the accuracy of metastatic nodal disease by imaging modality: 62% to 87% for ERUS, 22% to 73% for CT, and 39% to
95% for MRI.11 Part of the difficulty in accurately evaluating metastatic nodal disease relates to the fact that size is a poor predictor
of lymphatic metastasis in rectal cancer. Half of metastatic lymph
nodes are smaller than 5 mm and 8% are smaller than 2 mm.12
Although more than half of metastatic nodes are located
within 3 cm of the primary tumor, an inherent limitation of ERUS
is its inability to evaluate nodes outside the focal range of the
transducer. MRI is able to evaluate the entire mesorectum and
iliac anatomy. The best reports of MRI evaluation of nodal disease
in rectal cancer were obtained when value was placed on irregular
contour of nodes and heterogenous signal, rather than node size,
resulting in a sensitivity of 85% and a specificity of 97%.13 MRI
is preferable over CT for evaluation of infi ltration of neighboring organs due to its greater contrast resolution.14 CT scan and
positron emission tomography (PET) scan are both useful in the
staging of distant metastasis.
Answer: ERUS is the most accurate method for staging of
depth of invasion, while MRI is the most accurate method for
staging lymph node metastasis.

MANAGEMENT
2. Is local excision an appropriate method of treatment for
rectal cancer?
Having traditionally been a modality used in poor-risk surgical
patients unable to withstand a major resection, local surgery for
rectal cancer began to be investigated and reported in the past
three decades as a possible curative alternative to radical surgery
in early stage rectal cancers. Early reports in the 1980s by Grigg
et al.15 and Hager et al.16 showed encouraging results and helped
foster an interest in transanal excision (TAE). Follow-up studies
in the early-1990s by Gall and Hermanek17 and Willet et al.18 had
low recurrence rates for T1 and T2 tumors with favorable histology. Beginning in 19982000 and throughout the next decade
evidence was mounting that the early promise of TAE as an
equivalent oncologic option was not holding up. Multiple studies
showed that TAE was associated with a three- to five-fold higher
risk of tumor recurrence and a low rate of salvage once recurrence
was detected.19-21 Taylor et al.22 reported the Vancouver experience
of a 30% local recurrence rate for T1 and T2 tumors treated by
local excision alone. The University of Minnesota reported a comparison of 108 patients with T1 and T2 treated by TAE versus 153
with T1N0 and T2N0 rectal adenocarcinomas treated by radical
resection. The overall recurrence for T1 treated locally was 21%
compared with 9% in radical resections. Differences in survival
rate between local and radical resection were significantly worse
in T2 tumors.23 Madbouly et al.24 reported the Cleveland Clinic
experience on locally resected T1 tumors showing an estimated
5-year recurrence rate of 29%. Fourteen of 15 patients with a recurrence underwent salvage treatment with 56.2% 5-year survival.
Memorial Sloan-Kettering published their experience revealing
both a higher risk of local recurrence and significantly poorer

PMPH_CH29.indd 246

long-term survival among patients treated with TAE as compared


with radical resection. There will be circumstances in which
patients have significant comorbidities prohibiting major surgical resection or who have made the informed decision to choose
the functional benefits of TAE over rectal resection, but, as Nash25
concludes, they must be informed that TAE offers significantly
inferior oncologic results, including a greater risk of cancerrelated death.
Answer: Local excision has been shown to have higher local
recurrence and poorer long-term survival in retrospective studies;
therefore, it is not recommended unless significant comorbidities
prohibit radical surgical resection.
3. Is minimally invasive surgery equivalent to open surgery for
rectal cancer?
The accepted standard for rectal cancer surgery is by open technique with total mesorectal excision (TME) championed by Heald
et al.26 of the Basingstoke Group, United Kingdom. TME is of singular importance in improving local recurrence rates and survival after rectal cancer surgery. The ability to answer the above
question is ultimately dependent on oncologic outcome provided
by laparoscopic surgery, but perioperative issues and complication rates are also a component to be considered. Extended longterm oncologic results from randomized-controlled trials (RCTs)
are still pending, but there are indicators that suggest that laparoscopic surgery for rectal cancer will prove to provide equivalent
long-term results and the short-term benefits that are associated
with minimally invasive surgery for patients with rectal cancer.
There are reports showing that local recurrence, lymph node
harvest, and oncologic clearance are not compromised and may
be equivalent to open surgery.27 The short-term outcomes of open
versus laparoscopic-assisted low anterior resection (LAR) in
patients with mid to low rectal cancers who had been treated with
preoperative chemoradiation were examined in a randomized
trial of 340 patients reported by Kang et al.28 They reported that
the laparoscopic group had significantly better time to return of
bowel function (38 vs. 60 h), time to normal oral diet (85 vs. 93 h),
and time to defecation (96 vs. 123 h). There were no significant
differences in respect to circumferential resection margin, macroscopic quality of TME specimen, number of nodes harvested, or
perioperative morbidity.
Prospective randomized trials with long-term results by
Lujan et al.,29 Braga et al.,30 and Ng et al.31 have shown similar
local recurrence rates and disease-free survival of laparoscopically treated patients compared with their open surgery counterparts. The CLASICC trial for colon cancer also included rectal
cancer and showed a higher rate of positive margins in the 3-year
follow-up.32 These negative findings have been suggested to be due
to the learning curve in the use of laparoscopic technique. Other
studies have shown equivalent outcomes in laparoscopic versus
open surgery.33
As with any tool, its efficacy is related to the skill of the user
(learning curve) and the situation in which its use is required
(patient selection). Laparoscopic surgery certainly has a learning
curve and appropriate patient selection is necessary in any surgery.34 In the hands of an experienced surgeon who employs TME,
laparoscopic surgery for rectal cancer seems to provide comparable benefit and improved short-term advantage to the carefully
selected patient.35 Future results on long-term outcomes will continue to define this option for surgeons and patients.

5/21/2012 9:11:55 PM

Colon and Rectal Cancer Including Adjuvant

Answer: RCTs have shown that laparoscopic surgery is equivalent to open surgery for oncologic outcomes, with the benefits of
shorter recovery and quicker return of bowel function.
4. Should diverting ostomy be performed with LAR in rectal
cancer?
Resection of low-lying rectal cancers as low as 4 cm superior to
the anal verge has been made possible by improvements in surgical
techniques and equipment as well as improved neoadjuvant therapies over the past three decades. Circular staplers and better understanding of pelvic anatomy has allowed for LAR of rectal tumors,
defined by many as located below the peritoneal reflection. These
methods have allowed for lower anastamosis, preservation of anal
sphincters, and avoidance of permanent colostomy. Leaks from
these anastamosis can be disastrous, resulting in pelvic abscess,
sepsis, reoperation, prolonged hospital stay, subsequent poor function, and increased mortality. Mortality rates as high as 26%36 have
been reported while the risk of local recurrence is increased after
anastomotic leak from LAR. The role of a diverting ostomy proximal to the anastamosis has been controversial. Recent meta-analysis and RCTs have provided better insight into this question.
There have been six RCTs37-42 comparing the use of a defunctioning ostomy with no ostomy in patients who have undergone
LAR for rectal cancer. In addition, there have been three recent
meta-analyses43,44 including both the RCTs and many retrospective reports. The RCTs ranged in sample size from 34 to 256
patients, with a total of 648 patients included in all six trials. In
these trials patients were excluded if there was evidence of leak
intraoperatively or if the surgeon made the decision to perform a
defunctioning stoma. A definition of anastomotic leak has been
proposed by Peel45 in 1991 as the leak of luminal contents from
a surgical joining between two hollow viscera. This definition
can include both clinical and nonclinical (radiographic) leaks.
In these RCTs, nonclinical leaks were excluded from the study as
the evaluation for them was variable between studies, and they
were not of clinical significance. Recently the International Study
Group of Rectal Cancer has proposed a classification of anastamotic leaks after LAR (Table 29.1) and a more encompassing definition of an anastomotic leak as A defect of the intestinal wall
integrity at the colorectal or colo-anal anastomotic site (including
suture and staple lines of neorectal reservoirs) leading to a communication between the intra- and extraluminal compartments. A
pelvic abscess close to the anastomosis is also considered as anastomotic leakage.46 The six RCTs were conducted prior to this definition of anastamotic leak. Therefore in these RCTS the definition of
clinical leaks varied, but all included clinical findings of localized
or generalized peritonitis, feculent drainage from drains, or need
for urgent reoperation. The distance of the anastamosis from the
anal verge varied in the studies, ranging from a maximum of 4
cm to anywhere below the peritoneal reflection. All RCTs individually found the clinical leak rate to be significantly higher in

247

the patients who did not have a diverting ostomy with LAR while
combined results of all RCTs demonstrated clinical leak rates of
19.6% in the no ostomy group, and 6.3% in the diverting ostomy
group (risk ratio 0.33, 95% confidence interval 0.210.53).
In these RCTs, the need for urgent reoperation was four times
greater in the patients who did not have a diverting ostomy with
4% of the diverting ostomy group requiring reoperation and 16%
of the no ostomy patients requiring reoperation (risk ratio 0.28,
95% confidence interval 0.170.48). There was no significant
difference in mortality between the patients who had diverting
ostomy or no ostomy in the RCTs, as there were very few deaths
in either arm of these studies. The ostomy group mortality was
0.6% while the no ostomy group was 1.2%, with two deaths in the
ostomy group and four deaths in the no ostomy group (risk ratio
0.58, 95% confidence interval 0.142.33). Recognition and timely
treatment of anastomotic leaks and better critical care capabilities may be responsible for the low mortality rates. Quality of life
evaluations were not performed in these studies, with the quality
of life factors of urgent reoperation or drainage of pelvic abscess
in the no ostomy patients competing with the quality of life issues
of an ostomy and subsequent reoperation for ostomy takedown
in the ostomy group. In addition, there is recognized morbidity
and mortality (albeit low) from the ostomy takedown procedure
should be recognized when considering this question.
There have been over 21 nonrandomized studies, including
11,429 patients between 1984 and 2008, that have analyzed this
question. A meta-analysis of these studies demonstrated similar
results to the RCTs with a lower clinical anastomotic leak rate
(relative risk 0.74, 95% confidence interval 0.670.83, p < .001) and
lower reoperation rate (relative risk 0.28, 95% confidence interval
0.230.35, p < .001) in the patients who had a diverting ostomy
performed at the time of LAR. In the nonrandomized studies
the mortality between the cohorts was significantly lower with
a mortality of 0.7% (relative risk 0.42, 95% confidence interval
0.280.61, p < .001) in the ostomy group versus 2% in the no
ostomy group.36
Answer: The use of a diverting ostomy after LAR decreases
the clinical leak rate and need for urgent reoperation; therefore, it
is a recommended component of this procedure for rectal cancer.
5. Does total mesorectal excision (TME) decrease local recurrence and improve long-term survival in rectal cancer?
Local recurrence remains a significant problem in the treatment of
rectal cancer. Heald et al.47 first described the complete excision of
visceral mesorectal tissue, now known as TME, in 1982 showing
a decrease in local recurrence from nearly 40% to less than 10%.
More recent data have shown overall local recurrence rates of 3%
to 13% after TME.48-50 The addition of preoperative radiation to
TME further decreases local recurrence rates without an increase
in overall survival.51 Local recurrence rates after 5 and 10 years are
reported to be less than 5% in curative cases.52 TME involves sharp

Table 29.1 Classification of anastomotic leaks (proposed by the International Study Group of Rectal Cancer)46
Grade

Type of Leak

Anastomotic leakage requiring no active therapeutic intervention.

Anastomotic leakage requiring active therapeutic intervention but manageable without re-laparotomy.

Anastomotic leakage requiring re-laparotomy.

PMPH_CH29.indd 247

5/21/2012 9:11:55 PM

248

Surgery: Evidence-Based Practice

dissection to keep the visceral layer intact while recognizing the


importance of the circumferential margins predictive value in local
recurrence.53 Meticulous dissection is required to avoid breach of
the mesorectum which, when perforated, is associated with local
recurrence.54 TME is associated with increased sphincter preservation and decreased frequency of abdominoperineal resection.55,56
The 5-year tumor-free survival rate after TME is approximately 80%.52 Heald and Ryall55 report 5-year disease-free survival
by stage at 94%, 87%, and 58% for stages 1, 2, and 3, respectively.
TME is associated with an absolute overall and cancer-specific
survival benefit of 30%.47,55,56 Thanks in part to TME, some countries report rectal cancer survival having overtaken that of colon
cancer.57,58 Cases involving TME are associated with a mortality

of 3% to 7%, and a morbidity of 53%.59 Anastomotic leakage is a


major complication of TME; the incidence of leakage following
TME in low rectal tumors is 7% to 9%.60 Heald et al.61 reported
a leak rate of 12% when performing TME while Carlsen et al.62
reported a leak rate of 16%. Anastomotic leakage is the most significant cause of postoperative death in patients undergoing TME
and LAR. Preoperative radiotherapy is known to further increase
the risk for leakage.63 Additional drawbacks to TME include an
increased operative time and a higher rate of gastrointestinal, urinary, and sexual dysfunction although nerve sparing techniques
appear to mitigate the risk of genitourinary complications.64
Answer: TME decreases local recurrence and improves longterm survival in rectal cancer.

Clinical Question Summary


Question

Answer

Level of
Evidence

Grade of
Recommendation

1 What is the best preoperative


staging method for locoregional
disease in rectal cancer?

ERUS for depth of invasion. MRI for


lymph nodes.

2b

3-14

2 Is local excision an appropriate


method of treatment for rectal
cancer?

No, unless unfit for radical surgery.

2b

15-25

3 Is minimally invasive surgery


equivalent to open surgery for
rectal cancer?

Initial RCTs show equivalent oncologic


outcomes with shorter recovery.
Long-term oncologic results are
pending.

26-35

4 Should diverting ostomy be


performed with LAR in rectal
cancer?

Yes, it decreases the clinical leak rate


and need for reoperation.

1a

36-46

5 Does TME decrease local


recurrence and improve long-term
survival in rectal cancer?

TME decreases local recurrence and


improves long-term survival in
rectal cancer.

2b

47-64

REFERENCES
1. Ferlay J, Shin HR, Bray F, Forman D, Mathers C and Parkin
DM. GLOBOCAN 2008, Cancer Incidence and Mortality Worldwide: IARC CancerBase No. 10 [Internet]. Lyon, France: International Agency for Research on Cancer; 2010. Available from:
http://globocan.iarc.fr
2. Jemal A, Siegel R, Xu J,Ward E. Cancer Statistics, 2010. CA Cancer
J Clin. 2010;60:277-300.
3. Karantanas AH, Yarmenitis S, Papanikolaou N, Gourtsoyiannis N. Preoperative imaging staging of rectal cancer. Dig Dis.
2007;25:20-32.
4. Gleesan FC, Clain JE, Papachristou GI, Rajan E, Topazian MD,
Wang KK, Levy MJ. Prospective assessment of EUS criteria for
lymphadenopathy associated with rectal cancer. Gastrointest
Endosc. 2009;69:896-903.
5. Rieger N, Tjandra J, Solomon M. Endoanal and endorectal ultrasound: Applications in colorectal surgery. ANZ J Surg. 2004;74:
671-675.
6. Puli SR, Bechtold ML, Reddy JB, Choudhary A, Antillon MR,
Brugge WR. How good is endoscopic ultrasound in differentiating various T stages of rectal cancer? Meta-analysis and systemic
review. Ann Surg Oncol. 2009;16:254-265.

PMPH_CH29.indd 248

References

7. Garcia-Aguilar J, Pollack J, Lee SH, et al. Accuracy of endorectal ultrasonography in preoperative staging of rectal tumors. Dis
Colon Rectum. 2002;45:10-15.
8. Marusch F, Koch A, Schmidt U, Zippel R, Kuhn R, Wolff S, Pross
M, Wierth A, Gastinger I, Lippert H. Routine use of transrectal
ultrasound in rectal carcinoma: Results of a prospective multicenter study. Endoscopy. 2002;34:385-390.
9. Harewood GC. Assessment of publication bias in the reporting
of EUS performance in staging rectal cancer. Am J Gastroenterol.
2005;100:808-816.
10. Puli SR, Reddy JB, Bechtold ML, Choudhary A, Antillon MR,
Brugge WR. Accuracy of endoscopic ultrasound to diagnose
nodal invasion by rectal cancers: A meta-analysis and systemic
review. Ann Surg Oncol. 2009;16:1255-1265.
11. Maor Y, Nadler M, Barshack I, Zmora O, Koller M, Kundel Y,
Fidder H, Bar-Meir S, Avidan B. Endoscopic ultrasound staging
of rectal cancer: Diagnostic value before and following chemoradiation. J Gastroenterol Hepatol. 2006;21:454-458.
12. Edelman BR, Weiser MR. Endorectal ultrasound: Its role in the
diagnosis and treatment of rectal cancer. Clin Colon Rectal Surg.
2008;21(3):167-177.
13. Brown G, Richards CJ, Bourne MW, Newcombe RG, Radcliffe AG,
Dallimore NS, et al. Morphologic predictors of lymph node status in

5/21/2012 9:11:55 PM

Colon and Rectal Cancer Including Adjuvant

14.

15.

16.
17.
18.

19.

20.
21.

22.
23.

24.

25.
26.
27.

28.

29.

30.

31.

32.

33.
34.
35.

PMPH_CH29.indd 249

rectal cancer with use of high-spatial-resolution MR imaging with


histopathologic comparison. Radiology. 2003;227:371-377.
Blomqvist L, Holm T, Nyren S, Svanstrom R, Ulvskog Y, Iselius
L. MR imaging and computed tomography in patients with rectal tumours clinically judged as locally advanced. Clin Radiol.
2002;57:211-218.
Grigg M, McDermott FT, Phil EA, Hughes ES. Curative local
excision in the treatment of carcinoma of the rectum. Dis Colon
Rectum. 1984;27:81-83.
Hager T, Gall FP, Hermanek P. Local excision of the cancer of the
rectum. Dis Colon Rectum. 1983;26:149-151.
Gall FP, Hermanek P. Update of the German experience with local
excision of rectal cancer. Surg Oncol Clin N Am. 1992;1:99-109.
Willet CG, Compton CC, Shellito PC, Efird JT. Selection factors
for local excision or abdominoperineal resection of early stage
rectal cancer. Cancer. 1994;73:2716-2720.
Bentrem DJ, Okabe S, Wong WD, et al. T1 adenocarcinoma of the
rectum: transanal excision or radical surgery? Ann Surg. 2005;
242:472-479.
Paty PB, Nash GM, Baron P, et al. Long-term results of local excision for rectal cancer. Ann Surg. 2002;236:522-530.
Weiser MR, Landmann RG, Wong WD, et al. Surgical salvage
of recurrent rectal cancer after transanal excision. Dis Colon
Rectum. 2005;48:1169-1175.
Taylor RH, Hay JH, Larsson SN. Transanal local excision of
selected low rectal cancers. Am J Surg. 1998;175:360-363.
Mellgren A, Sirivongs P, Rothenberger DA, Madoff RD, GarciaAguilar J. Is local excision adequate therapy for early rectal cancer?
Dis Colon Rectum. 2000;43:1064-1074.
Madbouly K, Remzi F, et al. Recurrence after transanal excision
of T1 rectal cancer: Should we be concerned? Dis Colon Rectum.
2005;48:711-721.
Nash G, Weiser M, et al. Long-term survival after transanal excision of T1 rectal cancer. Dis Colon Rectum. 2009;52:577-582.
Macfarlane JK, Ryall RD, Heald RJ. Mesorectal excision for rectal cancer. Lancet. 1993;341:457-460.
Anderson C, Umam G, Piqazzi A. Oncologic outcomes of laparoscopic surgery for rectal cancer: A systemic review and meta-analysis
of the literature. Eur J Surg Oncol. 2008;34:1135-1342.
Kang SB, Park JW,Jeong SY, et al. Open versus laparoscopic surgery for mid or low rectal cancer after neoadjuvant chemoradiotherapy (COREAN trial): Short-term outcomes of an open-label
randomized controlled trial. Lancet Oncol. 2010;11(7):637-645.
Lujan J, Valero G, Hernandez Q, et al. Randomized clinical trial
comparing laparoscopic and open surgery in patients with rectal
cancer. Br J Surg. 2009;96:982-989.
Braga M, Frasson M, Vignali A, et al. Laparoscopic resection in
rectal cancer patients: Outcome and cost-benefit analysis. Dis
Colon Rectum. 2007;50:464-471.
Ng SS, Leung KL, Lee JF, et al. Laparoscopic-assisted versus open
abdominoperineal resection for low rectal cancer: A prospective
randomized trial. Ann Surg Oncol. 2008;15:2418-2425.
Guillou PJ, Quirke P, Thorpe H, et al. Short-term endpoints of
conventional versus laparoscopic-assisted surgery in patients
with colorectal cancer (MRC CLASICC trial): Multicentre, randomized controlled trial. Lancet. 2005;365(9472):1718-1726.
Laurent C, Leblanc F, et al. Laparoscopic versus open surgery for rectal cancer (long term oncologic results). Ann Surg. 2009;250:54-61.
Chand M, Heald RJ. Laparoscopic rectal cancer surgery.
Br J Surg. 2011;98:166-167.
Angst E, Hiatt JR, Gloor B, et al. Laparoscopic surgery for cancer: A systematic review and a way forward. J Am Coll Surg.
2010;211:412-423.

249

36. Tan WS, Tang CL, Shi L, Eu KW. Meta-analysis of defunctioning stomas in low anterior resection for rectal cancer. Br J Surg.
2009;96(5):462-472.
37. Matthiessen P, Hallbook O, Rutegard J, Simert G, Sjodahl R.
Defunctioning stoma reduces symptomatic anastomotic leakage
after low anterior resection of the rectum for cancer: A randomized multicenter trial. Ann Surg. 2007;246:207-214.
38. Pakkastie TE, Ovaska JT, Pekkala ES, Luukkonen PE, Jrvinen
HJ. A randomised study of colostomies in low colorectal anastomoses. Eur J Surg. 1997;163:929-933.
39. Graff ner H, Fredlund P, Olsson SA, Oscarson J, Petersson BG.
Protective colostomy in low anterior resection of the rectum
using the EEA stapling instrument. A randomized study. Dis
Colon Rectum. 1983;26:87-90.
40. Pimentel JM, Duarte A, Patricio J. The role of a protecting stoma
in low anterior resection with TME and colonic J-pouch for rectal cancer: Results of a prospective randomized trial. Colorectal
Dis. 2003;5(Suppl 2):83(Abstract).
41. Chude GG, Rayate NV, Patris V, Koshariya M, Jagad R, Kawamoto J, Lygidakis NJ. Defunctioning loop ileostomy with low
anterior resection for distal rectal cancer: Should we make an
ileostomy as a routine procedure? A prospective randomized
study. Hepatogastroenterology. 2008;55(86-87):1562-1567.
42. Ulrich AB, Seiler C, Rahbari N, Weitz J, Buchler MW. Diverting
stoma after low anterior resection: More arguments in favor. Dis
Colon Rectum. 2009;52(3):412-418.
43. Hser N, Michalski CW, ErkanM, Schuster T, Rosenberg R,
Kleeff J, Friess H. Systematic review and meta-analysis of the role
of defunctioning stoma in low rectal cancer surgery. Ann Surg.
2008;248(1):52-60.
44. Montadori A, Cirocchi R, Farinella E, Sciannameo F, Abraha I.
Covering ileo- or colostomy in anterior resection for rectal carcinoma. Cochrane Database of Systematic Reviews. 2010;5: CD006878.
DOI:10.1002/14651858.CD006878.pub2.
45. Peel AL, Taylor EW. Proposed definitions for the audit of postoperative infection: A discussion paper. Surgical Infection Study
Group. Ann Roy Coll Surg Engl. 1991;73(6):385-388.
46. Rahbari NN, Weitz J, Hohenberger W, Heald RJ, Moran B, Ulrich
A, et al. Definition and grading of anastomotic leakage following
anterior resection of the rectum: A proposal by the International
Study Group of Rectal Cancer. Surgery. 2010;147(3):339-351.
47. Heald RJ, Husband EM, Ryall RD. The mesorectum in rectal cancer
surgery-the clue to pelvic recurrence? Br J Surg. 1982;69:613-616.
48. Enker WE, Thaler H, Cranor M, et al. Total mesorectal excision
in the operative treatment of carcinoma of the rectum. J Am Coll
Surg. 1995;181:335-346.
49. Arenas RB, Fischera H, Mhoon D, et al. Total mesenteric excision
in the surgical treatment of rectal cancer. Arch Surg. 1998;133:
608-612.
50. Aitken RJ. Mesorectal excision for rectal cancer. Br J Surg.
1996;83:214-226.
51. Peeters KC, Marijnen CA, Nagtegaal ID, et al. The TME trial
after a medium follow-up of 6 years: Increased local control but
no survival benefit in irradiated patients with resectable rectal
carcinoma. Ann Surg. 2007;246:693-701.
52. MacFarlane JK, Ryall RDH, Heald RJ. Mesorectal excision for
rectal cancer. Lancet. 1993;341:457-460.
53. Quirke P, Dixon MF. The prediction of local recurrence
in rectal adenocarcinoma by histopathological examination.
Int J Colorectal Dis. 1988;3:127-131.
54. Law WL, Chu KW. Anterior resection for rectal cancer with
mesorectal excision: A prospective evaluation of 622 patients.
Ann Surg. 2004;240:260-268.

5/21/2012 9:11:55 PM

250

Surgery: Evidence-Based Practice

55. Heald RJ, Ryall RDH. Recurrence and survival after total
mesorectal excision for rectal cancer. Lancet. 1986;1:1479-1482.
56. Kapiteijn E, Putter H, van de Velde CJ. Impact of the introduction and training of total mesorectal excision on recurrence and
survival in rectal cancer in The Netherlands. Br J Surg. 2002;89:
1142-1149.
57. Iversen LH, Norgaard M, Jepsen P, et al. Trends in colorectal
cancer survival in northern Denmark: 1985-2004. Colorectal
Dis. 2007;9:210-217.
58. Birgisson H, Talback M, Gunnarsson U, et al. Improved survival
in cancer of the colon and rectum in Sweden. Eur J Surg Oncol.
2005;31:845-853.
59. Carlsen E, Schlichting E, Guldvog I, et al. Effect of the introduction of total mesorectal excision for the treatment of rectal
cancer. Br J Surg. 1998;85:526-529.

PMPH_CH29.indd 250

60. Vignali A, Fazio VW, Lavery IC, et al. Factors associated with
the occurrence of leaks in stapled rectal anastomoses: Review of
1,014 patients. J Am Coll Surg. 997;185:105-113.
61. Heald RJ, Smedh RK, Kald A, Sexton R, Moran BJ. Abdominoperineal excision of the rectuman endangered operation. Norman Nigro Lectureship. Dis Colon Rectum. 1997;40:747-751.
62. Heald RJ, Moran BJ, Ryall RDH, et al. The Basingstoke experience of total mesorectal excision, 1978-1997. Arch Surg. 1998;133:
894-899.
63. Matthiessen P, Hallbook O, Andersson M, Rutegard J, Sjodahl R.
Risk factors for anastomotic leakage after anterior resection of
the rectum. Colorectal Dis. 2004;6:462-469.
64. Enker WE, Havenga K, Polyak T, et al. Abdominoperineal resection via total mesorectal excision and autonomic nerve preservation for low rectal cancer. World J Surg. 1997;21:715-720.

5/21/2012 9:11:55 PM

Commentary on Colon and Rectal


Cancer Including Adjuvant
Nicholas J. Petrelli

The chapter entitled Colon and Rectal Cancer including Adjuvant goes into detail with an excellent review of the literature of
five provocative questions in the diagnosis and management of
colon and rectal cancer. The questions that are reviewed are the
following:

here is that there are more than likely patients that will benefit
from local excision of rectal cancer, but we do not have the means
to identify those individuals today. The morphology and pathology of the tumor does not necessarily help in separating out those
patients who will benefit from local excision versus those who will
fail. Again, as in the first question, this is where genomic profiling
may eventually help. The first prospective clinical trial on local
excision for rectal cancer was done by Steele et al.2 of the Cancer
and Leukemia Group B (CALGB) NCI Cooperative Group (Protocol #8984). In this trial of 110 eligible patients for local excision at a
median follow-up of 48 months, there were 4 of 59 T1 (7%) failures
and 10 of 51 T2 (20%) failures with a 6-year failure-free survival
of 78%. Just as importantly, this trial also demonstrated that the
technical aspects of local excision are not as easy as some surgical
atlases seem to make. For example, 51 post-surgery patients were
ineligible either because of staging, positive margins, or size criteria of the trial. I agree with the authors that unless there are significant comorbidities which prohibit radical surgical resection, or if
the patient refuses radical surgical resection, then local excision
is an option. It is important to remember that local recurrence in
rectal cancer can have severe consequences on the patients quality of life. Hence, performing the correct surgical procedure the
first time is critical to avoid a local recurrence and its subsequent
consequences.3,4
The third question deals with minimally invasive surgery versus open surgery for rectal cancer. For this question the authors
document the randomized control trials that have shown that laparoscopic colon surgery is equivalent to open surgery for oncologic outcomes with benefits of shorter recovery and quicker
return of bowel function. This question demonstrates the importance of prospective randomized Phase III trials in the oncology
arena. The Phase III prospective randomized trial is the gold
standard in oncology. The first prospective Phase III randomized
trial comparing laparoscopically assisted colectomy versus open
colectomy for colon cancer in the United States was performed
by the Clinical Outcomes of Surgical Therapy Study Group. This
multi-institutional study demonstrated that the rates of recurrent
cancer were similar after laparoscopically assisted colectomy and
open colectomy suggesting that the laparoscopic approach is an
acceptable alternative to open surgery for colon cancer.5
Question four involved a diverting ostomy performed with low
anterior resection in rectal cancer. The authors answer is that the
use of a diverting colostomy after low anterior resection decreases
the anastomotic leak rate and need for urgent reoperation; therefore,

1. What is the best preoperative staging method for locoregional


disease in rectal cancer?
2. Is local excision an appropriate method of treatment for rectal
cancer?
3. Is minimally invasive surgery equivalent to open surgery for
rectal cancer?
4. Should diverting ostomy be performed with LAR [low anterior
resection] in rectal cancer?
5. Does TME [total mesorectal excision] decrease local recurrence
and improve long-term survival in rectal cancer?
I will take each of these questions and comment on the answers
by Sticca et al.
The first question deals with the best preoperative staging
technology for local regional disease in rectal cancer. The authors
state that endorectal ultrasound is the most accurate method for
staging the depth of invasion of a cancer while magnetic resonance
imaging (MRI) is the most accurate method for staging lymph
node metastases. I agree with this conclusion. However, in the era
of modern technology, we sometimes forget the importance of a
physical examination. Hence, a rectal examination by an educated
finger can establish fi xation of the tumor. As the authors state, the
accuracy of endorectal ultrasound is directly related to the skill of
the operator. However, this is also true of the radiologist reading
the MRI. Interestingly, the literature has documented that a 10%
to 15% difference in T-staging for endorectal ultrasound occurred
between three board-certified colorectal surgeons.1 The staging of
lymph node metastases in rectal cancer by MRI is still not optimal. However, at present, it is the best technology that is available
to surgeons. Within the decade, there is no question that genomic
profi ling will be a tremendous advantage in the staging of lymph
node metastases.
Regarding the second question as to whether local excision is
an appropriate method of treatment for rectal cancer, the authors
conclude that local excision has been shown to have higher local
recurrence and poorer long-term survival in retrospective studies
and, therefore, is not recommended unless significant comorbidities prohibit radical surgical resection. I believe that the majority of
surgeons would agree with the authors conclusion. The real issue
251

PMPH_CH29.indd 251

5/21/2012 9:11:56 PM

252

Surgery: Evidence-Based Practice

it is a recommended component of this procedure for rectal cancer.


There is no question that an anastomotic leak can lead to severe
morbidity and potential mortality. Mortality rates as high as 26%
have been reported.6 It is important to note that several of the studies referenced by the authors were prior to the era of preoperative
radiation for rectal cancer. In general, preoperative radiation is
considered in patients with T3 and/or N positive lymph nodes on
endoscopic rectal ultrasound and MRI as described in question
one. I agree with the authors that the use of a diverting ostomy
after low anterior resection decreases the clinical leak rate and the
need for reoperation and is a recommended component for this
procedure. In essence, it is the safest approach for the patient.
The last question approached by Sticca et al. concerns total
mesorectal excision; whether it decreases local recurrence and
improves long-term survival in patients with rectal cancer. The
authors answer this question in the affirmative. My only comment to this question, which I would have liked the authors to
have added to their answer, is the following: surgeons who are
not trained to performed total mesorectal excision should not be
allowed to treat patients with rectal cancer.

PMPH_CH29.indd 252

REFERENCES
1. Garcia-Aguilar J, Pollack J, Lee SH, et al. Accuracy of endorectal ultrasonography in preoperative staging of rectal tumors. Dis
Colon Rectum. 2002;45:10-15.
2. Steele GD, Herndon JE, Bleday R, Russell A, Benson AB, Hussain
M, Burgess AM, Tepper JE, Mayer RJ. Sphincter sparing treatment for distal rectal adenocarcinoma. Ann Surg Oncol. 1999;6(5):
443-441.
3. Sticca R, Rodriguez-Bigas M, Penetrante R, Petrelli N. Curative
resection of Stage I rectal cancer: Natural history, prognostic factors and recurrence patterns. Cancer Invest. 1996;14(5):491-497.
4. Chorost M, Petrelli N, McKenna M, Kraybill W, Rodriguez-Bigas
M. Local excision of rectal cancer. Am Surg. 2011;67(8):774-779.
5. Nelson H, Sargent D, Wieand S, Fleshman J, Anvari M, Stryker S,
Beart R, Hellinger M, Flanagan R, Peters W, Ota D. A comparison
of laparoscopically assisted and open colectomy for colon cancer.
N Engl J Med. 2004;350(20):2050-2059.
6. Tan WS, Tang CL, Shi L, Eu KW. Meta-analysis of defunctioning stomas in low anterior resection for rectal cancer. Br J Surg.
2009;96(5):462-472.

5/21/2012 9:11:56 PM

CHAPTER 30

Tumors of the Anal Region


Marjun P. Duldulao and Julio Garcia-Aguilar

BACKGROUND

subclassified according to their histologic features. In this chapter,


we present an overview of the different cancers that affect each
anatomic region and their histologic features, detailing risk factors, prognosis, treatment modalities, and outcomes for each.

Anal cancer is a rare malignancy accounting for approximately 2%


of all gastrointestinal cancers.1 In the United States, approximately
5200 new cases are diagnosed annually, resulting in an estimated 700
deaths, and the incidence of anal carcinoma continues to increase.2

TUMORS OF THE ANAL CANAL

ANAL REGION: ANATOMY AND


HISTOLOGY

Squamous Cell Carcinoma


Squamous cell carcinoma (SCC) is the most common cancer of
the anal canal, representing 80% of all anal cancers. Numerous
risk factors for SCC have been described, including human papilloma virus (HPV),3 smoking, receptive anal intercourse, human
immunodeficiency virus (HIV),4 immunosuppression, sexually
transmitted diseases, and cervical, vulvar, or vaginal cancer.5

The anal region is divided into two main areas: the anal canal and
the anal margin. The division that demarcates these two regions
coincides with the intersphincteric groove or anal verge. The proximal portion of the anal canal begins at the palpable superior edge
of the anorectal ring, which corresponds with the point where the
distal rectum enters the puborectalis sling, and extends to the anal
verge at the palpable outer edge of the internal sphincter muscle or
intersphincteric groove. The anal margin corresponds to the perianal skin extending 5 cm from the anal verge. The dentate line is
an important anatomic landmark of the anal canal that represents
the fusion of the hindgut and the proctodeum during embryologic
life. Below the dentate line the anal canal is covered by squamous
epithelium. Immediately above the dentate line there is a 1- to
2-cm region of transitional or cloacogenic epithelium that contains squamous, transitional, and cuboidal cells. Proximally, the
cloacogenic epithelium progressively transforms into the normal
columnar epithelium of the lower rectum. The anal glands open to
the anal crypts located al the level of the dentate line. The crypts
are separated from one another by the columns of Morgani. The
proximal anal canal above the dentate line typically drains to the
internal iliac and pelvic lymph nodes, and the distal anal canal
and anal margin drain to the inguinal nodal basin.

Diagnosis and Evaluation


The most common presenting symptoms in patients with anal
canal SCC are bleeding, anal pain, or the sensation of a mass.
However, up to 20% of patients may be asymptomatic.6 Diagnosis includes digital rectal examination (DRE) combined with
anoscopy or proctoscopy, and lesions are biopsied to confirm
diagnosis. Careful palpation of the inguinal nodal basin with fineneedle aspiration or core-needle biopsy of enlarged lymph nodes
is important to determine the extent of disease. Imaging modalities such as computed tomography (CT) are also routinely used
to identify distant metastatic disease (in the lungs or liver) and
enlarged lymph nodes in regions not amenable to direct physical
examination (such as the paraaortic and iliac nodes). The overall sensitivity of CT in detecting regional lymph node disease in
pelvic malignancies is approximately 50%.7 Positron emission
tomography (PET) is another imaging option that may facilitate
the detection of regional lymph node disease (Level 3b evidence),
and the combination of PET and CT has been shown to improve
the sensitivity for detection of nodal regional disease compared
with conventional CT (89% vs. 62%).8 Other studies substantiate
the usefulness of PET imaging for detecting regional disease;9,10

CANCERS OF THE ANAL REGION


Tumors of the anal region are defined based on their location in
the anal canal or anal margin. Tumors of the anal canal are further
253

PMPH_CH30.indd 253

5/21/2012 9:12:30 PM

254

Surgery: Evidence-Based Practice

but as many of these patients never undergo surgery; the true sensitivity of this imaging modality cannot be confirmed without
histologic confirmation of the nodal staging. Furthermore, current imaging modalities are not able to detect microscopic disease
and are only useful to identify patients with relatively large nodal
disease. Endoanal ultrasound (EAUS) is also used in the evaluation and staging of anal cancer because it can determine the depth
of invasion into the sphincter complex and in assessing regional
lymph nodes; however, its role in staging is not yet clear.
The extent of the disease is determined by the size of the primary tumor (T), the location of the regional nodal metastasis (N),
and the presence of distant metastasis (M), according to the American Joint Committee on Cancer (AJCC) staging guidelines.11

Prognosis and Treatment, and Follow-up


Prognosis for SCC in the anal canal is directly related to staging.
This has been highlighted in recent studies that have examined
tumor staging and oncologic outcomes.12-14 Bilimoria et al.13 used
the National Cancer Database (19852000) to evaluate 19,199
patients with SCC in the anal canal and determined survival rates
based on tumor stage. They found that the highest survival was
observed in patients with lower T-stage and lower overall disease stage (Stages III), and that the 5-year overall survival rates
decreased with increasing disease stage from 70% (Stage I) to 19%
(Stage IV). Lymph node metastasis also influenced prognosis, with
5-year overall survival rates decreasing from 63% to 38% in lymph
node negative and positive patients, respectively.14 Ajani et al.12
prospectively studied 644 patients and determined that patients
with tumors >5 cm in diameter and positive lymph nodes had significantly worse disease-free and overall survival compared with
patients with smaller tumors and negative lymph nodes. Other
factors associated with prognosis include gender, age, socioeconomic status, and ethnicity; however, studies have yet to substantiate the reasons underlying these disparities.
SCC of the anal canal has historically been treated by abdominoperineal resection (APR) and permanent colostomy. Patients
undergoing APR as primary treatment have a 27% to 47% local
failure rate and a 5-year survival rate of 40% to 70%.6,15 However,
over three decades ago Nigro et al.16 demonstrated that a complete
clinical response (cCR) could be achieved in patients using a combination of chemotherapy and radiation (CRT). This observation
led to numerous large studies designed to investigate the efficacy
of chemotherapy and/or radiation as a treatment for anal-canal
SCC.17-19 The United Kingdom Coordinating Committee on Cancer Research (UKCCCR) Anal Cancer Working Group launched
one of the first randomized Phase III trials comparing the efficacies of radiation (RT) alone with CRT for treating anal-canal SCC
cancer.17 Five hundred and eighty-five patients were randomized
into two groups; each group received 45 Gy of RT with or without
concurrent 5-fluorouracil (5-FU) and mitomycin C (MMC). The
3-year local failure rate, defined as persistent disease 6 weeks after
initial therapy, colostomy for recurrent disease or colostomy after
treatment morbidity was significantly lower in patients treated
with CRT compared with those treated with RT alone (39% vs. 61%;
p < .0001). A similar trial conducted by the European Organization
for Research and Treatment of Cancer (EORTC 22861) randomized 110 patients to receive RT alone or concurrently with 5-FU
and MMC.18 The 5-year local failure rate was lower in the CRT
group compared with RT alone (32% vs. 50%; p = .02). Patients
in the CRT group also showed an increase in cCR compared

PMPH_CH30.indd 254

with RT (80% vs. 54%), a lower incidence of local recurrence, and


lower colostomy rate at 5-year follow-up. The RTOG 87-04 trial
conducted in the United States evaluated the efficacy of MMC in
the treatment regimen for anal-canal SCC.19 Their trial randomized 291 patients to receive either 5-FU and RT or 5-FU/MMC
and RT. Local failure was found to be lower in the 5-FU/MMC-RT
group compared with the 5-FU-RT treatment arm (16% vs. 34%;
p = .0008). These patients also experienced a lower incidence of
colostomy at 4-year follow-up (9% vs. 22%; p = .002). No difference was observed in overall survival between the two treatment
arms; however, disease-free survival at 4 years was higher in the
5-FU/MMC-RT group (73% vs. 51%; p = .0003). These studies
established the combined modality treatment of 5-FU, MMC, and
RT as the primary treatment for SCC of the anal canal.
Cisplatin has been utilized for the treatment of SCC of the
head and neck, cervix, and the esophagus with beneficial effects.
The RTOG 98-11 trial compared the efficacy of a cisplatin-based
regimen with 5-FU/MMC chemotherapy for the treatment of
SCC of anal canal.20 Six hundred and forty-four patients were
randomized to receive either standard 5-FU/MMC and RT or
cisplatin/5-FU and RT. The 5-year disease-free and overall survival was comparable between the groups. The most significant difference was observed in the colostomy rate, which was
higher in patients who received cisplatin compared with MMC
(19% vs. 10%; p = .02).
Newer chemotherapeutic regimens are also being evaluated
to improve treatment outcomes. In the EORTC 22011-40014
Phase II trial, Matzinger et al.21 demonstrated improved complete
response rates using a combination of MMC and cisplatin compared with MMC and continuous infusion 5-FU. However, the
cisplatin-containing regimen was associated with higher toxicity.
Other biologic agents such as cetuximab, a monoclonal antibody
against epidermal growth factor receptor (EGFR), have also been
identified as a feasible treatment option for the treatment of anal
canal SCC.22-24 The efficacy of these newer regimens has yet to be
confirmed in Phase III clinical trials.
Patients are usually evaluated 8 to 12 weeks after initial CRT
by physical examination with DRE. Biopsies are only performed
if there is persistence of disease or suspicion of progression. Indications for biopsy include persistence or development of a new
ulceration, enlarging mass, or increasing anal pain.25 Patients
with complete eradication of disease should have continued
follow-up every 3 to 6 months for 5 years with physical examination and anoscopy and possible radiologic evaluation of involved
nodal basins at least annually for the first 3 years. If no continued regression of disease is observed after serial examination or if
there is disease progression, more aggressive therapy may be warranted. Evidence of progression on physical examination should
be confirmed with biopsy and patients should be restaged with
radiologic imaging.
Despite the efficacy of CRT for anal-canal SCC, approximately
30% of patients experience treatment failure. For these patients,
APR may be the only option. However, APR in this patient population requires complex reconstructive myocutaneous flap procedures
and is associated with high morbidity due to surgical complications related to the perianal wound.26-29 As such, the option of less
invasive surgical therapies, such as local excision (LE), or further
CRT have been studied as possible treatment options for selected
patients with who fail primary treatment with CRT. However, use
of LE for the management of recurrent or persistent disease after
CRT is very limited.30 No studies have reported the effectiveness

5/21/2012 9:12:30 PM

Tumors of the Anal Region

of LE after failed CRT. Thus, APR remains the treatment option


of choice for salvage therapy in patients with loco-regional failure
after CRT and several studies report a 5-year survival rate ranging
from 39% to 64% after salvage surgery.30-32 Following salvage surgery, patients should be followed every 36 months for 5 years.
LE has also been considered as a treatment option for early and
small (<1-cm diameter) anal cancers. In a retrospective study performed across 17 institutions in France, Ortholan et al.33 reviewed
69 patients with early stage tumors (T1 or Tis; <1 cm diameter).
Of the 69 patients, 26 underwent LE and 43 were treated with RT
alone. Twenty-three of the 26 patients who underwent LE were
treated with concurrent RT; the remaining three patients had LE
alone. Recurrence was low in patients who received RT either alone
(9%) or in conjunction with LE (12%) compared with patients
treated with LE alone (33%).33 No prospective clinical trials have
examined the efficacy of CRT and LE for small anal-canal SCC,
but the studies which have investigated LE alone for anal-canal
SCC report inconsistent results,34,35 though most report a high
rate of local recurrence.36,37 These studies suggest that LE alone
may not be sufficient in controlling the disease.

Anal Intraepithelial Neoplasia


Anal intraepithelial neoplasia (AIN) is highly associated with HPV
infection and the precise overall incidence is currently unknown.
AIN can be classified into three grades: AIN I is classified as lowgrade dysplasia, while AIN II and III are equivalent to high-grade
dysplasia and has the highest malignant potential. Recently, the
term squamous intraepithelial lesion (SIL) has replaced AIN. Now,
SIL is subdivided into high-grade (HSIL), which is equivalent to
AIN II and III, and low-grade (LSIL) which is equivalent to AIN I.38

Diagnosis and Evaluation


Anal cytology similar to Pap smear has been utilized to screen
high-risk patients for the presence of SIL. High-resolution anoscopy with the application of 3% acetic acid and appropriate magnification is the best way to diagnose these lesions in patients with
a positive anal cytology. The malignant potential of LSIL and HSIL
is not entirely known but, similar to cervical cancer, these lesions
are thought to progress to invasive anal cancer. In patients with
HIV, the disease appears to be more aggressive and this population may be at highest risk for developing invasive cancer.39

Prognosis and Treatment, and Follow-up


Very few studies have examined the management of SIL, especially in high-risk patients. LSIL is thought to have low malignant
potential and is mainly managed with close follow-up at 6-month
intervals. Topical applications of trichloroacetic acid, imiquimod,
podophyllotoxin, and cryotherapy have been utilized to treat
LSIL.39 However, the management of HSIL remains highly debatable. Devaraj et al.40 followed 40 HSIL patients with HIV with
close observation and found that patients who progressed to invasive carcinoma could be managed successfully with CRT. HIV
status did not influence treatment outcomes in these patients.40-42
Surgical options including wide LE or ablation with electrocautery, photodynamic therapy, and infrared coagulation of all visible lesions have been described for the treatment of HSIL, but
studies are limited.43,44 Currently, frequent surveillance of patients
with HSIL at 6-month intervals is recommended.42

PMPH_CH30.indd 255

255

Adenocarcinoma
Adenocarcinoma of the anal canal represents less than 20% of
anal cancers.45 These tumors arise from the anal glands within the
cloacogenic zone of the anal canal or from the columnar elements
within the transitional epithelium of the upper anal canal. The
diagnosis of these tumors is controversial because they are histologically indistinguishable from rectal adenocarcinoma and in
most patients it is impossible to differentiate between a low rectal
adenocarcinoma extending to the anal canal and an anal adenocarcinoma extending to the distal rectum.

Diagnosis and Evaluation


Common presenting symptoms of adenocarcinoma include a
lump, pruritus, or bleeding. There is often a delay in diagnosis
and patients may present with metastatic disease upon initial
diagnosis.46

Prognosis and Treatment, and Follow-up


The long-term survival is poor with a 5-year survival of 4% to 5%.
In a survey of colorectal surgeons the most appropriate procedure
for the surgical management of anal adenocarcinoma was shown
to be APR.47 Combined modality treatments including CRT and
surgery is also an option for adenocarcinoma of the anal canal.
Patients undergo a resection after CRT, similar to patients with
rectal adenocarcinoma. Chang et al.46 has recently reviewed the
outcomes of 34 patients diagnosed with adenocarcinoma of the
anal canal in a single institution over a 20-year span and found
that patients treated with combined modality treatment and
radical resection had improved survival compared with patients
treated with combined modality treatment and LE.46 Other retrospective reviews also support the use of multi-modality treatment
approaches, although the optimal CRT regimen for adenocarcinoma has yet to be determined.48 Nonetheless, these studies suggest that neoadjuvant CRT followed by radical surgical resection
may provide the best local control and survival,48,49 and thus treatment of anal adenocarcinoma mirrors the recommendations for
treatment of rectal cancer.50

Anorectal Melanoma
Anorectal melanoma accounts for less than 3% of all malignancies of the anorectum and accounts for approximately 1% of all
melanomas.51

Diagnosis and Evaluation


Symptoms are indistinguishable from other benign conditions,
and most lesions are diagnosed at a late stage. The presence of
a pigmented lesion in the anus may be indicative of melanoma;
however 10% to 29% of lesions may be amelanotic, and thus
histologic diagnosis by immunohistochemical staining with
melanoma antigen HMB-45 and S-100 protein or Fontana staining for melanin are utilized.52

Prognosis and Treatment, and Follow-up


The prognosis of patients with anorectal melanoma is poor and
up to 40% of patients have distant metastasis at the time of diagnosis. The role of CRT remains unclear, and optimal surgical

5/21/2012 9:12:30 PM

256

Surgery: Evidence-Based Practice

management continues to be debated.52-55 A recent study by Kiran


et al.53 evaluated survival in 109 patients from the SEER database
undergoing either LE or APR for anal melanoma. They found that
patients who underwent LE had similar 3- and 5-year survival
rates compared with patients who were treated with APR. Additional studies also support these findings.52-55
In nonmetastatic disease, APR may be associated with improved local control;56 however, studies suggest that despite this
aggressive treatment there is no survival benefit compared with
LE.57 Given the overall poor prognosis of anorectal melanoma, LE
is advised for localized tumors, reserving APR for palliation.

TUMORS OF THE ANAL MARGIN


Squamous Cell Carcinoma
SCC of the anal margin is approximately five to tenfold less common than SCC of the anal canal.58

Diagnosis and Evaluation


These tumors resemble SCC occurring elsewhere in the skin
and typically have rolled everted edges with central ulcerations.
Patients present with complaints of a painful mass, pruritus,
bleeding, tenesmus, or incontinence, and small lesions are often
misdiagnosed. Staging SCC of the anal margin is similar to SCC
of the skin and is based on overall size and lymph node involvement. Physical examination with DRE and palpation of inguinal
nodal basins is imperative. Although slow growing, chest radiograph and CT may be warranted to determine distant metastasis.
The incidence of lymph node metastasis increases with increasing
tumor size; 0% in tumors <2 cm, 23% in tumors 2 to 5 cm, and
67% in tumors >5 cm.58

Prognosis and Treatment, and Follow-up


Management of small localized lesions (<2 cm) may be adequately
treated by wide LE with 1 cm margins with a favorable 5-year survival rate of approximately 88%.59 For larger, deeper tumors (T24
or N1), recurrent tumors after repeated LE or tumors that involve
the anal sphincter, APR may be required.60 In patients with synchronous inguinal lymph node metastasis, a groin dissection is
recommended.61 Radiation with or without chemotherapy may be
utilized as primary treatment or for salvage treatment after failure
of LE. Mendenhall et al.61 reported on a small cohort of 16 patients
treated with RT alone or CRT for T2 and T3 anal-margin SCC,
and all were disease-free on 2-year follow-up. However, Papillon
et al.58 described a series of 57 patients treated with brachytherapy
and external beam irradiation (EBRT) who had a local recurrence
rate of 12% and a 16% inguinal metastasis rate. In patients treated
with EBRT alone, 22% died from primary disease. Thus, the utilization of RT alone for the treatment of SCC of the anal margin
should be used with caution.

Diagnosis and Evaluation


BCC of the anal margin presents with similar symptoms to
benign disease and is often misdiagnosed as hemorrhoids or anal
fissures.

Prognosis and Treatment, and Follow-up


Wide LE is the treatment of choice for primary surgical management. In one study, the recurrence rate following this approach
was shown to be 24%.62 However, the local recurrences were adequately treated with repeated wide LE and subsequently associated with favorable outcomes.62,63 APR is recommended only in
patients with advanced disease that extends deep into the anal
canal and invades surrounding tissues.60

Bowens Disease
Bowens disease is another rare lesion of the anal margin and is
often described as a slow-growing intraepidermal SCC. Similar to
HSIL of the anal canal, Bowens disease may serve as a precursor
to SCC of the anal margin. These lesions most often occur in the
fift h or sixth decades of life.

Diagnosis and Evaluation


Bowens disease is characterized by discrete erythematous and
scaly or crusting plaques. Intralesional foci of ulcerations may
indicate invasive SCC which occurs in approximately 6% of these
patients.64 Patients often present with burning or itching sensation in the anal region, or bleeding.

Prognosis and Treatment, and Follow-up


Surgical management of Bowens disease consists of wide LE with
biopsies taken 1 cm from the edge of lesion and microscopically
clear margins; this is the current treatment of choice.64 Recurrence
is often treated with a wider LE and reported 5-year survival rate
in patients with recurrent disease is approximately 75%.65

Pagets Disease
Pagets disease of the perianal skin is rare. These lesions may occur
within the sixth or seventh decades of life.

Diagnosis and Evaluation


These lesions represent a very slow-progressing intraepithelial
adenocarcinoma characterized histologically by the presence of
Paget cells which stain positive with Periodic acidSchiff base.66
Pagets disease is often associated with synchronous visceral
malignancies. These occur with a reported occurrence ranging
from 32% to 86%.67,68 A complete colonoscopy is therefore warranted to determine the presence of other cancers.

Prognosis and Treatment, and Follow-up


Basal Cell Carcinoma
The incidence of basal cell carcinoma (BCC) of the anal margin is
exceedingly low, comprising 0.2% of all anorectal cancers.62 Similar
to BCC of the skin, BCC of the anal margin rarely metastasizes.

PMPH_CH30.indd 256

Surgical management of Pagets disease consists of wide LE with


microscopically clear margins. Pagets disease often extends
beyond gross margins and has a high recurrence rate. Large
lesions often necessitate a skin-graft or local flap for closure of

5/21/2012 9:12:30 PM

Tumors of the Anal Region

257

the remaining defect. APR is only recommended for perianal


Pagets disease with invasive adenocarcinoma or other associated
malignancies.60

invade into the sphincter muscles may need APR.69 Only a few
case series describe the surgical management of these patients and
thus the optimal treatment remains under debate.60

Verrucous Carcinoma

CONCLUSION

Verrucous carcinoma is often referred to as giant condyloma


acuminatum or Buschke-Lowenstein tumor.

Diagnosis and Evaluation


Tumors often present as a large, exophytic, warty, cauliflowerlike mass. These lesions range in size from 1 to 30 cm. Larger
lesions may harbor invasive SCC and invade into the surrounding
tissue and pelvic cavity. The extent of disease is best assessed by
CT scan.

Prognosis and Treatment, and Follow-up


CT scan is necessary to determine the extent of resection.60 Superficial lesions may be managed by wide LE, however lesions that

Tumors of the anal region are rare. SCC of the anal canal is often
associated with HPV, and prognosis of patients depends on initial T and N stage. Combined CRT is the primary treatment of
SCC of the anal canal with surgery reserved for salvage therapy.
Follow-up is recommended with frequent physical examination, routine imaging, and selective biopsy of suspicious lesions.
In general, lesions of the anal margin can be managed surgically
with either wide LE or APR depending on tumor invasiveness.

ACKNOWLEDGMENTS
The authors thank Nicola Solomon, PhD, for assistance in writing
and editing the manuscript.

Clinical Question Summary


Question

Answer

Risks for anal-canal SCC

HPV infection, history of cervical cancer


in women, immunosuppression, HIV
infection.

3-5

Best imaging modality for


initial evaluation anal-canal
adenocarcinoma

Diagnostic CT. The efficacy of PET


imaging as an adjunct or replacement of
diagnostic CT is still debatable, however,
recent reports have found clinical useful
in evaluating lymph nodes not enlarged
on CT scans.

6-10

Factors associated with OS in


anal-canal adenocarcinoma

Disease stage as established by AJCC


staging criteria.

12-14

Most appropriate treatment


for anal-canal SCC

5-FU/MMC plus RT.

16-24

Using LE for the management


of anal-canal SCC

Local excision is not recommended for


anal-canal SCC due to increased
likelihood of local recurrence. APR
provides optimal control in the event of
failed CRT and necessary salvage therapy
after failed CRT.

30-37

Management for AIN II/III


or HSIL

Frequent surveillance every 6 months,


especially in high-risk populations such
as HIV infected patients.

38-42

REFERENCES
1. Jemal A, Siegel R, Xu J, et al. Cancer statistics, 2010. CA Cancer
J Clin. 2010;60:277-300.
2. Johnson LG, Madeleine MM, Newcomer LM, et al. Anal cancer
incidence and survival: The surveillance, epidemiology, and end
results experience, 1973-2000. Cancer. 2004;101:281-288.

PMPH_CH30.indd 257

Levels of
Evidence

Grade of
Recommendation

References

3. Daling JR, Madeleine MM, Johnson LG, et al. Human papillomavirus, smoking, and sexual practices in the etiology of anal
cancer. Cancer. 2004;101:270-280.
4. De Vuyst H, Clifford GM, Nascimento MC, et al. Prevalence and
type distribution of human papillomavirus in carcinoma and
intraepithelial neoplasia of the vulva, vagina and anus: A metaanalysis. Int J Cancer. 2009;124:1626-1636.

5/21/2012 9:12:30 PM

258

Surgery: Evidence-Based Practice

5. Smith JS, Lindsay L, Hoots BE, et al. Human papillomavirus type


distribution in invasive cervical cancer and high-grade cervical
lesions: A meta-analysis update. Int J Cancer. 2007;121:621-632.
6. Ryan DP, Compton CC, Mayer RJ. Carcinoma of the anal canal.
N Engl J Med. 2000;342:792-800.
7. Sato H, Koh PK, Bartolo DC. Management of anal canal cancer.
Dis Colon Rectum. 2005;48:1301-1315.
8. De Winton E, Heriot AG, Ng M, et al. The impact of 18fluorodeoxyglucose positron emission tomography on the staging, management and outcome of anal cancer. Br J Cancer. 2009;
100:693-700.
9. Cotter SE, Grigsby PW, Siegel BA, et al. FDG-PET/CT in the
evaluation of anal carcinoma. Int J Radiat Oncol Biol Phys.
2006;65:720-725.
10. Pepek JM, Willett CG, Czito BG. Radiation therapy advances for
treatment of anal cancer. J Natl Compr Canc Netw. 2010;8:123-129.
11. Edge SB, Byrd DR, Compton CC, et al. American Joint Committee on Cancer (AJCC) Cancer Staging Manual. 7th ed. New York,
NY: Springer; 2010.
12. Ajani JA, Winter KA, Gunderson LL, et al. Prognostic factors derived
from a prospective database dictate clinical biology of anal cancer:
The intergroup trial (RTOG 98-11). Cancer. 2010;116:4007-4013.
13. Bilimoria KY, Bentrem DJ, Rock CE, et al. Outcomes and prognostic factors for squamous-cell carcinoma of the anal canal:
Analysis of patients from the National Cancer Data Base.
Dis Colon Rectum. 2009;52:624-631.
14. Gerard JP, Chapet O, Samiei F, et al. Management of inguinal
lymph node metastases in patients with carcinoma of the anal
canal: Experience in a series of 270 patients treated in Lyon and
review of the literature. Cancer. 2001;92:77-84.
15. Schraut WH, Wang CH, Dawson PJ, et al. Depth of invasion,
location, and size of cancer of the anus dictate operative treatment. Cancer. 1983;51:1291-1296.
16. Nigro ND, Vaitkevicius VK, Considine B, Jr. Combined therapy
for cancer of the anal canal: A preliminary report. Dis Colon
Rectum. 1974;17:354-356.
17. Epidermoid anal cancer: Results from the UKCCCR randomised
trial of radiotherapy alone versus radiotherapy, 5-fluorouracil,
and mitomycin. UKCCCR Anal Cancer Trial Working Party. UK
Co-ordinating Committee on Cancer Research. Lancet. 1996;
348:1049-1054.
18. Bartelink H, Roelofsen F, Eschwege F, et al. Concomitant radiotherapy and chemotherapy is superior to radiotherapy alone in
the treatment of locally advanced anal cancer: Results of a phase
III randomized trial of the European Organization for Research
and Treatment of Cancer Radiotherapy and Gastrointestinal
Cooperative Groups. J Clin Oncol. 1998;15:2040-2049.
19. Flam M, John M, Pajak TF, et al. Role of mitomycin in combination with fluorouracil and radiotherapy, and of salvage chemoradiation in the definitive nonsurgical treatment of epidermoid
carcinoma of the anal canal: Results of a phase III randomized
intergroup study. J Clin Oncol. 1996;14:2527-2539.
20. Ajani JA, Winter KA, Gunderson LL, et al. Fluorouracil, mitomycin, and radiotherapy vs fluorouracil, cislplatin, and radiotherapy
for carcinoma of the anal canal: A randomized controlled trial.
JAMA. 2008;299:1914-1921.
21. Matzinger O, Roelofsen F, Mineur L, et al. Mitomycin C with
continuous fluorouracil or with cisplatin in combination with
radiotherapy for locally advanced anal cancer (European Organisation for Research and Treatment of Cancer phase II study
22011-40014). Eur J Cancer. 2009;45:2782-2791.
22. Czito BG, Willett CG. Current management of anal canal cancer.
Curr Oncol Rep. 2009;11:186-192.

PMPH_CH30.indd 258

23. Jiang Y, Mackley H, Cheng H, et al. Anal carcinoma therapy:


Can we improve on 5-fluorouracil/mitomycin/radiotherapy?
J Natl Compr Canc Netw. 2010;8:135-144.
24. Olivatto LO, Meton F, Bezerra M, et al. Phase I study of cetuximab (CET) in combination with 5-fluorouracil (5-FU), cisplatin
(CP), and radiotherapy (RT) in patients with locally advanced
squamous cell anal carcinoma (LAAC) [abstract]. J Clin Oncol.
2008;27 (Suppl 1):Abstract 4116.
25. Cummings BJ, Keane TJ, OSullivan B, et al. Epidermoid anal
cancer: Treatment by radiation alone or by radiation and
5-fluorouracil with and without mitomycin C. Int J Radiat Oncol
Biol Phys. 1991;21:1115-1125.
26. Chessin DB, Hartley J, Cohen AM, et al. Rectus flap reconstruction decreases perineal wound complications after pelvic
chemoradiation and surgery: A cohort study. Ann Surg Oncol.
2005;12:104-110.
27. Ellenhorn JD, Enker WE, Quan SH. Salvage abdominoperineal
resection following combined chemotherapy and radiotherapy for
epidermoid carcinoma of the anus. Ann Surg Oncol. 1994;1:105-110.
28. Ferenschild FT, Vermaas M, Hofer SO, et al. Salvage abdominoperineal resection and perineal wound healing in local recurrent
or persistent anal cancer. World J Surg. 2005;29:1452-1457.
29. Nilsson PJ, Svensson C, Goldman S, et al. Salvage abdominoperineal resection in anal epidermoid cancer. Br J Surg.
2002;89:425-429.
30. Schiller DE, Cummings BJ, Rai S, et al. Outcomes of salvage surgery for squamous cell carcinoma of the anal canal. Ann Surg
Oncol. 2007;14:2780-2789.
31. Mariani P, Ghanneme A, De la Rochefordiere A, et al. Abdominoperineal resection for anal cancer. Dis Colon Rectum.
2008;51:1495-1501.
32. Mullen JT, Rodriguez-Bigas MA, Chang GJ, et al. Results of surgical salvage after failed chemoradiation therapy for epidermoid
carcinoma of the anal canal. Ann Surg Oncol. 2007;14:478-483.
33. Orthalan C, Ramaioli A, Peiffert D, et al. Anal canal carcinoma:
Early-stage tumors <10 mm (T1 or Tis): Therapeutic options and
original pattern of local failure after radiotherapy. Int J Radiat
Oncol Biol Phys. 2005;62:479-485.
34. Al-Jurf AS, Turnbull RP, Fazio VW. Local treatment of squamous
cell carcinoma of the anus. Surg Gynecol Obstet. 1979;148:576-578.
35. Boman BM, Moertel CG, OConnell MJ, et al. Carcinoma of the
anal canal. A clinical and pathologic study of 188 cases. Cancer.
1984;54:114-125.
36. Clark J, Petrelli N, Herrara L, et al. Epidermoid carcinoma of the
anal canal. Cancer. 1986;57:400-406.
37. Singh R, Nime F, Mittelman A. Malignant epithelial tumors of
the anal canal. Cancer. 1981;48:411-415.
38. Palefsky JM, Holly EA, Ralston ML, et al. Anal squamous intraepithelial lesions in HIV-positive and HIV-negative homosexual
and bisexual men: Prevalence and risk factors. J Acquir Immune
Defic Syndr Hum Retrovirol. 1998;17:320-326.
39. Palefsky JM, Rubin M. The epidemiology of anal human papillomavirus and related neoplasia. Obstet Gynecol Clin North Am.
2009;36:187-200.
40. Devaraj B, Cosman BC. Expectant management of anal squamous
dysplasia in patients with HIV. Dis Colon Rectum. 2006;49:
36-40.
41. Chiao EY, Giordano TP, Richardson P, et al. Human immunodeficiency virus-associated squamous cell cancer of the anus:
Epidemiology and outcomes in the highly active antiretroviral
therapy era. J Clin Oncol. 2008;26:474-479.
42. Fleshner PR, Chalasani S, Chang GJ, et al. Practice paramenters
for anal squamous neoplsms. Dis Colon Rectum. 2008;51:2-9.

5/21/2012 9:12:30 PM

Tumors of the Anal Region

43. Goldstone SE, Kawalek AZ, Huyett JW. Infrared coagulator: A


useful tool for treating anal squamous intraepithelial lesions.
Dis Colon Rectum. 2005;48:1042-1054.
44. Runfola MA, Weber TK, Rodriguez-Bigas MA, et al. Photodynamic therapy for residual neoplasms of the perianal skin.
Dis Colon Rectum. 2000;43:499-502.
45. Joon DL, Chao MW, Ngan SY, et al. Primary adenocarcinoma of
the anus: A retrospective analysis. Int J Radiat Oncol Biol Phys.
1999;45:1199-1205.
46. Chang GJ, Gonzalez RJ, Skibber JM, et al. A twenty-year experience with adenocarcinoma of the anal canal. Dis Colon Rectum.
2009;52:1375-1380.
47. Abel ME, Chiu YS, Russell TR, et al. Adenocarcinoma of the anal
glands. Results of a survey. Dis Colon Rectum. 1993;36:383-387.
48. Belkacemi Y, Berger C, Poortmans P, et al. Management of primary anal canal adenocarcinoma: A large retrospective study
from the Rare Cancer Network. Int J Radiat Oncol Biol Phys.
2003;56:1274-1283.
49. Kounalakis N, Artinyan A, Smith D, et al. Abdominal perineal
resection improves survival for nonmetastatic adenocarcinoma
of the anal canal. Ann Surg Oncol. 2009;16:1310-1315.
50. Sauer R, Becker H, Hohenberger W, et al. Preoperative versus postoperative chemoradiotherapy for rectal cancer. N Engl
J Med. 2004;351:1731-1740.
51. Longo WE, Vernava, AM, 3rd, Wade TP, Coplin MA, et al. Rare
anal cancers in the U.S. veteran: Patterns of disease and results
of treatment. Am Surg. 1995;61:495-500.
52. Ishizone S, Koide N, Karasawa F, et al. Surgical treatment for
anorectal malignant melanoma: Report of five cases and review
of 79 Japanese cases. Int J Colorectal Dis. 2008;23:1257-1262.
53. Kiran RP, Rottoli M, Pokala N, et al. Long-term outcomes after
local excision and radical surgery for anal melanoma: Data from
a population database. Dis Colon Rectum. 2010;53:402-408.
54. Yeh JJ, Shia J, Hwu WJ, et al. The role of abdominoperineal
resection as surgical therapy for anorectal melanoma. Ann Surg.
2006;244:1012-1017.
55. Yeh JJ, Weiser MR, Shia J, et al. Response of stage IV anal mucosal
melanoma to chemotherapy. Lancet Oncol. 2005;6:438-439.

PMPH_CH30.indd 259

259

56. Brady MS, Kavolius JP, Quan SH. Anorectal melanoma. A


64-year experience at Memorial Sloan-Kettering Cancer Center.
Dis Colon Rectum. 1995;38:146-151.
57. Iddings DM, Fleisig AJ, Chen SL, et al. Practice patterns and
outcomes for anorectal melanoma in the USA, reviewing three
decades of treatment: Is more extensive surgical resection beneficial in all patients? Ann Surg Oncol. 2010;17:40-44.
58. Papillon J, Chassard JL. Respective roles of radiotherapy and
surgery in the management of epidermoid carcinoma of the
anal margin. Series of 57 patients. Dis Colon Rectum. 1992;35:
422-429.
59. Jensen SL, Hagen K, Harling H, et al. Long-term prognosis after
radical treatment for squamous-cell carcinoma of the anal canal
and anal margin. Dis Colon Rectum. 1988;31:273-278.
60. Skibber J, Rodriguez-Bigas MA, Gordon PH. Surgical considerations in anal cancer. Surg Oncol Clin N Am. 2004;13:321-338.
61. Mendenhall WM, Zlotecki RA, Vauthey JN, 3rd EMC. Squamous
cell carcinoma of the anal margin treated with radiotherapy. Surg
Oncol. 1996;5:29-35.
62. Nielsen OV, Jensen SL. Basal cell carcinoma of the anus - A clinical study of 34 cases. Br J Surg. 1981;68:856-857.
63. Paterson CA, Young-Fadok TM, Dozois RR. Basal cell carcinoma
of the perianal region: 20-year experience. Dis Colon Rectum.
1999;42:1200-1202.
64. Beck DE, Fazio VW, Jagelman DG, et al. Perianal Bowens disease. Dis Colon Rectum. 1988;31:419-422.
65. Sarmiento JM, Wolff BG, Burgart LJ, et al. Perianal Bowens disease: Associated tumors, human papillomavirus, surgery, and
other controversies. Dis Colon Rectum. 1997;40:912-918.
66. Armitage NC, Jass JR, Richman PI, et al. Pagets disease of the
anus: A clinicopathological study. Br J Surg. 1989;76:60-63.
67. Beck DE, Fazio VW. Perianal Pagets disease. Dis Colon Rectum.
1987;30:263-266.
68. McCarter MD, Quan SH, Busam K, et al. Long-term outcome of
perianal Pagets disease. Dis Colon Rectum. 2003;46:612-616.
69. Elliot MS, Werner ID, Immelman EJ, et al. Giant condyloma
(Buschke-Loewenstein tumor) of the anorectum. Dis Colon
Rectum. 1979;22:497-500.

5/21/2012 9:12:30 PM

CHAPTER 31

Inherited Colorectal
Cancer Syndromes
Vitaliy Y. Poylin, Kristin B. Niendorf, and Robert D. Madoff

INTRODUCTION

Answer: There is significant variability in the estimated prevalence of inherited colorectal cancers. Based on existing studies,
approximately 5% to 7% of colon cancers have an identifiable
inherited syndrome; however, this number may only be true for
patients of European descent. (Grade B recommendation)

As early as 1913, a pathologist Aldred Warthin used a simple tool,


family history collection, to describe the first hereditary cancer
syndrome, now considered to be the Lynch syndrome (hereditary
colorectal cancer syndrome or HNPCC). Between 5% and 10% of
all colorectal cancers arise in the setting of hereditary cancer syndromes. With the advent of the molecular age, the genetic basis of
these diseases has been clarified, and effective cancer screening
and prevention strategies exist. A century later, it has become even
clearer that family history collection is the first step in identifying
those families at risk. A precise understanding of inherited colorectal cancer genetics is important for identifying at-risk individuals, improving cancer surveillance and prevention, as well as
developing better diagnostic and therapeutic approaches.

2. What are the major inherited colorectal cancer syndromes?


The classic hereditary cancer syndromes include familial adenomatous polyposis (FAP) syndrome and Lynch syndrome (HNPCC).
More recently, several other familial associations with colorectal
cancer and/or polyposis have been described including MYHassociated polyposis (MAP), hyperplastic polyposis (HPP)/
serrated pathway syndrome, familial colorectal cancer X, Peutz
Jeghers syndrome (PJS), PTEN hamartoma tumor syndrome
(PHTS), and juvenile polyposis syndrome (JPS).

1. How prevalent are inherited colorectal cancer syndromes?

ADENOMATOSIS POLYPOSIS SYNDROMES

Identifying the true prevalence of inherited colorectal cancer has


proven complex. Approximately 25% to 40% of cases have been
estimated to have a familial component (at least one first-degree
relative affected by colorectal cancer).1-5 Between 5% and 10% of
patients with new diagnosis of colon cancer thought to have recognized inherited cancer syndromes.1-3,5 Most of the available data
come from a combination of population studies and cancer registries. These sources can help explain the variability in the prevalence of reported hereditary colon cancers. Epidemiologic studies
of North American and Western European populations suggest
that the rate is between 5% and 7%.3-5 A Colorectal Registry from
Spain places the rate at 5% or below.6 The California Cancer Survey suggests that the number is even lower.7 However, a recent twin
study of the Swedish and Finnish populations suggested that the
rates were higher, possibly up to 15%. The significant variability of
prevalence may be explained by differences in populations studied,
selection bias of colorectal registries as well as variable methodology
and difficulties defining hereditary cancer. In addition, the majority of studies listed above concentrate mostly on patients of western
European descent and thus it may not be possible to generalize the
observations to the rest of the world population.

FAP syndrome and related syndromes account for about 1% of


all colorectal cancers. Classic FAP is characterized by hundreds
of adenomas developing, typically, by the second decade of life.
Nearly all patients with classic FAP will develop colorectal carcinoma (average age at diagnosis of 39 years) if untreated via colectomy.3 Extracolonic manifestations include polyps of the gastric
fundus and duodenum; osteomas; dental anomalies; congenital
hypertrophy of the retinal pigment epithelium (CHRPE); soft
tissue tumors; desmoid tumors; and other cancers (small bowel,
pancreas, thyroid [papillary], CNS [medulloblastoma], liver
[hepatoblastoma in children], and bile duct). Three subtypes of
FAP exist: attenuated FAP (AFAP), Gardner syndrome, and Turcot syndrome. AFAP is characterized by later onset disease with
fewer than 100 polyps; usually localized to the right side of the
colon.2,8,9 Turcot syndrome involves colonic polyposis and CNS
tumors, most notably medulloblastomas. Turcot syndrome can be
due to mutations in the same gene, APC, as classic FAP but has
also been described in individuals with mismatch repair genes
typically associated with Lynch syndrome. Gardner syndrome
260

PMPH_CH31.indd 260

5/22/2012 5:23:14 PM

Inherited Colorectal Cancer Syndromes

carries adenomatous colonic polyposis with additional features


that include benign osteomas and soft tissue tumors.
MAP syndrome frequently presents similarly to FAP/AFAP
with multiple adenomas or polyposis coli and early-onset colorectal cancers. However, MAP is inherited in an autosomal-recessive
pattern without clear dominant transmission, as is typically seen
in classic FAP. MAP can also present with few or no polyps and, in
some cases, duodenal polyposis and/or duodenal adenocarcinoma. Reported extraintestinal features are similar to FAP and can
be variably expressed: cystic fundic gland polyps/gastric cancer,
CHRPEs, osteomas, dental anomalies, skin lesions, and sebaceous
gland hyperplasia and carcinoma.8,10
Familial colorectal cancer type X (FCCTX) was coined in
2005 to describe the accumulated evidence of families with colorectal cancer, but without clearly demonstrated FAP or HNPCC.4
The average age of colon cancer diagnosis is almost 10 years older,
higher adenoma to carcinoma ratio, with more adenomas. At
this time, it was proposed that the term Lynch syndrome be used
for families with known hereditary mismatch-repair defects and
FCCTX for those without identifiable defects.11

HYPERPLASTIC/MIXED POLYPOSIS
SYNDROMES
HPP syndrome has been proposed for those families with a predilection for hyperplastic or serrated polyposis. Distinction of
syndromes has proven difficult and various other terminologies
include serrated pathway syndrome (SPS), dependent on the histologic features of the polyps.11 Increased risk for colorectal cancers has been reported in several families with HPP.12 The similar
hereditary mixed polyposis syndrome (HMPS) presents with
a mixture of juvenile, adenomatous, hyperplastic, and mixed
polyps.

HAMARTOMATOUS POLYPOSIS
SYNDROMES
Juvenile polyposis (JPS) is characterized by large numbers of hamartomatous polyps of the gastrointestinal tract. When five or more
of these polyps (or within the context of a family history JPS) are
recognizable as juvenile ( i.e., similarity to inflammatory solitary
colorectal hamartomas that occur in children, but JPS can occur in
adults) this meets the criteria for diagnosis of JPS.7 There is an
increased risk for malignancies with colon cancer being the most
frequent (~40% lifetime risk).7 JPS can also concurrently occur
with hereditary hemorrhagic telangiectasia (HHT), a syndrome
associated with arteriovenous malformations and telangiectasias
of the skin and mucosa.
PJS typically presents with multiple gastrointestinal hamartomatous polyps and mucocutaneous pigmentation. Cancer risks
include colorectal malignancies as well as cancers of the ovary, testis
and other organs. PTEN hamartoma tumor syndrome (including
Cowden syndrome and BannayanRileyRuvalcaba syndromes)
has polyposis, typically hamartomatous, but also includes tumors
of the breast and uterus. Noncancerous features include thyroid
anomalies, macrocephaly, and several other features. Hamartomatous polyp syndromes, PJS, JPS, and Cowden disease when combined, cause less than 1% of colorectal cancers.2,6

PMPH_CH31.indd 261

261

NONPOLYPOSIS SYNDROMES
Nonpolyposis syndromes, most significantly including Lynch
syndrome (HNPCC), accounts for 4% to 6% of all colon
cancers.2,13,14 Individuals with Lynch syndrome have risks for the
following cancers (life time rate): colorectal (80%), endometrium
(2060%), stomach (1119%), ovary (912%), hepatobiliary tract
(27%), urinary tract (45%), small bowel (14%), and central
nervous system (13%).15 Two-thirds of colon cancers occur in the
proximal colon and, pathologically, the tumors frequently show
a poorly differentiated mucinous appearance, lymphocytic infi ltrates, histologic heterogeneity, and signet-cell features. Muir
Torre syndrome is a subtype of Lynch syndrome that includes
sebaceous gland adenomas and keratoacanthomas in addition to
visceral cancers.
Answer: FAP, MAP, and Lynch syndrome (HNPCC) account
for more than 90% of currently identified inherited colon cancer
syndromes. (Grade B recommendation)
3. What are the molecular mechanisms of the most common
inherited colorectal cancer syndromes?
FAP is caused by a germ-line mutation in APC, a tumor suppressor
gene. The inheritance pattern is autosomal-dominant. Individuals
with FAP are born with one mutated copy of APC in all of their cells;
the somatic inactivation of the second copy leads to polyp development. Approximately 80% of patients with FAP have a familial history of the disease. In the other 20%, mutations are presumed to
develop in somatic cells de novo. The APC gene is located on chromosome 5 and mutations in various parts of the APC gene tend to
lead to different phenotypes. For example, AFAP most commonly
occurs when mutations are at extreme 5 and 3 ends of the APC
gene; the presence of desmoids tumors is associated with mutations
between codons 1403 and 1578.9 MAP is inherited in an autosomalrecessive fashion via biallelic mutations in the MYH (MUTYH)
gene. Lynch syndrome (HNPCC) is caused by a germ-line mutation
in any of several genes that participate in the DNA mismatch repair
(MMR) system. These germ-line defects in the MMR system result
in microsatellite instability in the tumor. Genes responsible for
this condition include MLH1, MSH2, MSH6, and PMS2, with the
first two listed genes responsible for most Lynch syndrome cases.
Lynch syndrome is inherited in an autosomal-dominant pattern.
Biallelic mutations in MMR genes have been reported and appear
to result in a syndrome, which includes neurofibromatosis-like
features (e.g., caf-au-lait macules) as well as childhood hematologic malignancies, brain tumors, and colorectal cancers. Of note,
recent reports suggest that deletions in the EPCAM (TACSTD1)
gene impact the MMR pathway and lead to Lynch syndrome with
similarly increased colorectal cancer and other risks.10,16 All of the
hamartomatous polyposis syndromes are inherited in an autosomal-dominant fashion. Specific mutations include the STK11 (LKB1)
gene for PJS; MADH4 (SMAD4), BMPR1A, or ENG for JPS; PTEN
for Cowden syndrome. Causative genes for HPP/serrated pathway
syndrome are unknown at this point. A locus for hereditary mixed
polyposis syndrome has been mapped to 15q although there is suggested linkage to a JPS-related gene as well.
Answer: Major inherited cancer syndromes identified at this
point are caused by mutations in a distinct gene or genes, most
often inherited in an autosomal-dominant fashion. FAP is caused
by a mutation in the APC gene, MAP is caused by mutations in
the MYH gene, and Lynch syndrome (HNPCC) is caused by a

5/22/2012 5:23:14 PM

262

Surgery: Evidence-Based Practice

mutation in one of the following: MLH1, MSH2, MSH6, PMS2,


and EPCAM (TACSTD1) (Grade A recommendation).
4. How should an individual with a suspected inherited colorectal cancer syndrome be evaluated?
Hereditary colorectal cancer syndromes are identified via careful
medical and family history evaluation. Families with hereditary
cancer syndromes tend to carry the following hallmark features:
multiple individuals with the same or related cancers (e.g., colorectal cancers or endometrial plus colorectal cancers); early age of
onset for given cancer; multiple generations affected by the cancer
(except in the case of MAP, which is recessive); and unusual features
(e.g., sebaceous neoplasms in MuirTorre syndrome). FAP and
MAP are typically identified via the presentation of adenomatous
polyposis, frequently in the context of a family history of early-onset colorectal cancer. Given the less obvious clinical presentation
of Lynch syndrome (HNPCC), several criteria sets based on family
history have been proposed: Amsterdam (classic), Amsterdam II,
Modified Amsterdam, Bethesda Guidelines, and Revised Bethesda
Guidelines (outlined in Table 31.1). The sensitivity for some of
these criteria range, in at least one study, from approximately 61%
to 94% when applied to families carrying germ-line mutations in
MLH1 and MSH2.14 The National Comprehensive Cancer Network (NCCN) publishes guidelines for detection, prevention, and
risk reduction with a subsection on colorectal cancer and highrisk syndromes. These comprehensive guidelines outline the complex identification process for patients likely to have hereditary

cancer syndromes and propose testing, treatment, and management protocols (www.nccn.org). The NCCN recommends referral
for genetic counseling for any of the following:
1. Individuals meeting Revised Bethesda or Amsterdam Criteria
(Table 31.1).
2. Individuals with >10 adenomas.
3. Individuals with multiple gastrointestinal hamartomatous or
HPP.
4. Individuals with a known family history of a hereditary cancer
syndrome.
In addition to family and medical history cues, colorectal
tumors in young patients (under the age of 60) exhibiting pathologic evidence of microsatellite instability including tumorinfi ltrating lymphocytes, Crohns-like lymphocytic reaction,
mucinous/signet ring differentiation, medullary growth pattern
should be further evaluated for possible Lynch syndrome. Tumor
studies begin with immunohistochemistry for the genes products implicated in Lynch syndrome (MLH1, MSH2, MSH6, and
PMS2). With IHC, absence of staining for a given gene product
suggests a germ-line mutation in that gene but approximately 10%
of individuals with Lynch syndrome will have normal staining
for MMR proteins.17 Microsatellite instability testing (MSI) may
be performed at certain centers. MSI-High (MSI-H) tumors are
suggestive, but not diagnostic for Lynch syndrome. Clarification
of MSI-H can be done by testing for hypermethylation of the MLH1
promoter or due to somatic mutation of the BRAF gene, both of

Table 31.1 Lynch Syndrome Criteria (as Described in National Comprehensive Cancer Network Colorectal
Cancer Guidelines)
Amsterdam Criteria24
At least three relatives with colorectal cancer; all of the following should be present:
1. One should be a first-degree relatives of the other two
2. At least two successive generations must be affected
3. At least one of the relatives with colorectal cancer must have received a diagnosis before the age of 50 years
4. Familial adenomatous polyposis (FAP) should be excluded
5. Tumors should verified by pathological examination

Amsterdam II
At least three relatives must have a cancer associated with hereditary nonpolyposis colorectal cancer (colorectal, endometrial, small
bowel, ureter, or renal-pelvis); all of the following should be present:
1. One must be a first-degree relative of the other two
2. At least two successive generations must be affected
3. At least one of the relatives with cancer associated with hereditary nonpolyposis colorectal cancer should be diagnosed before the
age of 50 years
4. Familial adenomatous polyposis (FAP) should be excluded
5. Tumors should be verified whenever possible

Revised Bethesda Criteria36


Tumors from individuals should be tested for MSI in the following situations:
1. Colorectal cancer diagnosed in a patient who is less than 50 years of age
2. Presence of synchronous or metachronous Lynch-syndrome-associated tumors, regardless of age (CRC, endometrial, gastric,
ovarian, pancreas, ureter and renal pevis, biliary tract, brain [usually glioblastoma], small intestinal cancers as well as sebaceous
gland adenomas and keratoacanthomas [Muir-Torre syndrome])
3. Colorectal cancer with the MSI-H histology (tumor-infiltrating lymphocytes, Crohns-like lymphocytic reaction, mucinous/signet
ring differentiation, medullary growth pattern) diagnosed in a patient who is less than 60 years of age
4. Colorectal cancer diagnosed in a patient with one or more first-degree relatives with a Lynch syndrome related cancer, with one of
the cancers being diagnosed under the age of 50 years
5. Colorectal cancer diagnosed in a patient with two or more first- or second-degree relatives with Lynch syndromerelated cancers
(see above), regardless of age

PMPH_CH31.indd 262

5/22/2012 5:23:14 PM

Inherited Colorectal Cancer Syndromes

which cause a MSI-H effect in tumors and make Lynch unlikely


if present. The NCCN Guidelines provides suggestions for genetic
testing protocols; however, genetic counseling is recommended
prior to germ-line genetic testing to ensure appropriate evaluation.
Answer: Patients with significant family history or significant
clinical features should be suspected of having hereditary colorectal
cancer syndrome and evaluated with molecular genetic testing and
should receive genetic counseling. (Grade B recommendation)
5. What is the appropriate surgery for a patient with an inherited colorectal cancer syndrome?
Surgery is indicated for almost all patients with classic FAP. Colon
cancer under the age of 10 has not been reported and cancer
between ages 11 and 15 years is relatively uncommon. Although
the chance of developing cancer increases as patients get older, age
itself should not be the sole determinate in the timing of surgery.
The severity of the disease (based on family history and the number
and distribution of colonic polyps) should dictate the timing and
type of surgery. The two main types of surgery available for patients
with FAP are total abdominal colectomy with an ileorectal anastomosis (IRA) and a total proctocolectomy (TPC) with either an end
ileostomy or an ileal pouch anal anastomosis (IPAA). Patients with
<20 rectal polyps and no rectal malignancy qualify for an IRA.
IRA is a simple, straightforward operation that has lower complication rates, allows for better function, and is much less likely to
affect fertility rates than IPAA; important considerations in this
are generally younger population of patients. IRA is particularly
well suited to patients with AFAP. However, if this option is chosen,
a strict surveillance schedule must be followed and many patients
will require an interval completion proctectomy. For patients with
severe rectal polyposis or rectal cancer, proctocolectomy should be
performed with either end ileostomy or IPAA.
Another factor that can affect the timing and type of surgery
is the relative success of chemoprevention. The use of nonsteroidal anti-inflammatory drugs such as sulindac has been shown to
reduce the number and size of colonic adenomas. This effect can
be maintained for a number of years, but does not completely protect the individual from developing colon cancer. For individuals
with mild disease, chemoprevention in combination with surveillance can be used to delay surgery, and similarly, for patients with
minimal rectal disease, chemoprevention can be used after IRA to
delay timing of a proctectomy.18
Duodenectomy or pancreatoduodenectomy is sometimes
required if patients have large duodenal adenomas that are not
amendable to endoscopic resection or if dysplasia is identified.
These are very complex procedures, but the chance of recurrent
polyp formation and cancer is significantly lower when compared
with the much less invasive local polyp resection.19
Desmoid tumors are an increasing cause of morbidity and
mortality in FAP. They tend to be slow growing, but often involve
areas, such as the root of the mesentery, that can make resection
very difficult, hazardous, and sometimes impossible. Their growth
is sometimes triggered by surgery itself. Surgical treatment for
desmoid tumors remains controversial. In general, minimal surgery with attempts at a clear margin should be performed and
only for symptomatic patients.20,16
Patients with Lynch syndrome have a high chance of developing multiple colorectal cancers (synchronous and metachronous)
as well as extracolonic malignancies (especially endometrial and
ovarian). Emphasis should be placed on colorectal surveillance

PMPH_CH31.indd 263

263

and removal of any polyps. Partial colectomy followed by continued surveillance are the mainstay of surgical treatment for patients
with Lynch syndrome once cancer develops. The rate of metachronous large-bowel malignancy is at least 45%, high enough that
prophylactic TAC with IRA should at least be discussed.21 Rectal
cancer is not as common in HNPCC, so proctectomy should not
be suggested during an initial consultation for colon cancer.
The chance of gynecologic malignancies is significantly increased in patients with HNPCC and prophylactic hysterectomy
should be discussed as an alternative to surveillance, particularly
if the patient is undergoing a colon resection.22 The risk of gynecologic malignancy increases with age. Prophylactic oophorectomy
and hysterectomy should be discussed when the patient completes
childbearing (even without the presence of colonic malignancy),
since these procedures are risk reducing and there is no evidence
that surveillance lowers the risk of endometrial or ovarian cancer
in patients with HNPCC.
Patients with hamartomatous polyposis syndromes have a
higher chance of developing colonic and extracolonic malignancies when compared with the general population, but the risk is
lower when compared with adenomatous polyposis syndromes.
While strict surveillance is required, a segmental resection can be
considered if a cancer is diagnosed. If a metachronous cancer or
more polyps develop, a prophylactic colectomy can be considered,
but currently there is no firm evidence to support this approach.
For patients with HPP, prophylactic surgery should be discussed,
but no conclusive recommendations can be made at this time.
Answer: For most patients with FAP, total proctocolectomy
with or without an ileoanal pouch should be performed. Total
abdominal colectomy with an ileorectal anastomosis can be considered for patients with AFAP. Prophylactic surgery is not
indicated for patients with HNPCC, but close colonoscopic surveillance is required. Should a colon cancer be diagnosed in a
HNPCC patient, total abdominal colectomy with an ileorectal
anastomosis is generally recommended, but segmental colectomy
can be considered. In addition, a discussion about prophylactic
surgery for gynecologic malignancy should be had, especially in
postmenopausal women. (Grade B recommendation)
6. What is appropriate surveillance for patients with an inherited colorectal cancer syndrome?
For patients with known FAP, a surveillance schedule depends on
the severity of the disease at the time of presentation as well as
the family history. If no adenomas are identified during the initial
colonoscopy, then a sigmoidoscopy or colonoscopy every 1 to 2
years is recommended or until adenomas start to develop. When
adenomas appear, a colonoscopy and polypectomy should be performed every 6 to 12 months (depending on polyp burden) until
a colectomy is planned. If an IRA is chosen as a surgical option,
then a sigmoidoscopy should be performed every 6 to 12 months
(depending on polyp burden) after surgery. There have been some
reports of polyps developing in the pouch after an IPAA, and there
is the possibility of retained rectal mucosa, which predisposes
these patients to the subsequent development of malignancy; the
optimal schedule of surveillance after an IPAA is not clear at this
point.23 For family members of affected individuals with classic FAP
(APC mutation positive), surveillance schedule will depend on the
APC status. Individuals with detected APC mutations should be
treated as classic FAP patients: flexible sigmoidoscopy or colonoscopy every 6 to 12 months starting between 10 and 15 years of age.

5/22/2012 5:23:14 PM

264

Surgery: Evidence-Based Practice

If no APC mutation is detected, these individuals are considered


to be at average risk and are screened accordingly. If the individual is not tested, more frequent surveillance schedule should be
suggestedflexible sigmoidoscopy or colonoscopy annually starting at 10 to 15 years until the age of 24, then every 2 years until
the age of 34 if no polyps were detected or genetic testing done. If
no polyps were detected, endoscopy should be performed every
3 years until the age of 44 and 3 to 5 years thereafter.9,24,25
If APC mutation is not detected, MYH testing should be
considered, and family members are entered into the surveillance program similar to FAP patients who were not tested for
APC (see above).
For patients with AFAP, surveillance should start before the
age of 21 with colonoscopy every 1 to 2 years as long as polyp
burden is small. Between the ages of 21 and 40, if polyp burden
remains small, colonoscopy should be performed after every 1 to 2
years. After the age of 40, or if polyp burden increases, colectomy
with ileorectal anastomosis should be strongly considered with
continuous surveillance of the rectum every 6 to 12 months after
surgery.24 Duodenal adenomas and carcinomas tend to develop
later in life. Surveillance frequency will depend on polyp burden
detected at upper endoscopy. For patients without polyps, endoscopy can be repeated every 4 to 5 years. If 1 to 4 small adenomas are
detected, the surveillance interval is decreased to 2 to 3 years. For
5 to 19 and larger polyps, upper endoscopy is recommended every
1 to 2 years. For moderate polyposis of >20 polyps, EGD should be
repeated every 6 to 12 months. For severe polyposis of with signs
of dysplasia, surgical resection should be recommended.23,24
When genetic testing does not reveal biallelic MYH mutation
in patients with suspected MAP, they should be surveyed as FAP
individuals. For patients who are biallelic-MYH-mutation positive, colonoscopy is recommended every 2 to 3 years as long as
polyp burden is small, and EGD for every 3 to 5 years starting at
the age of 35. If polyp burden is large or is increasing overtime, the
surveillance period is decreased to every 6 to 12 months and
surgery should be recommended. For family members of MYHmutation positive patients, if the MYH status is unknown or if
they are biallelic MYH positive, colonoscopy starting between the
age of 25 and 30 for every 3 to 5 years, and EGD for every 3 to 5
years starting at the age of 30 to 35 are recommended.
For HNPCC, periodic and frequent surveillance has been
shown to decrease the chance of colorectal cancer and allows
for the detection of cancer at an earlier stage. There are no studies that directly compare the various surveillance periods. However, new cancer has been detected as early as 2 to 3 years after a
colonoscopy in these patients.13 Colonoscopy screening should be
started at 20 to 25 years of age and continued every 1 to 2 years
until polyps are detected. At that point colonoscopy should done
every 6 to 12 months and surgical options discussed. There is
no clear evidence for routine screening for endometrial cancer,
but yearly gynecologic examination is recommended and prophylactic hysterectomy and bilateral salpingo-oopherectomy after
childbearing is done.14,22,24 After a partial colectomy is performed,
surveillance should continue on a similar schedule since the chance
of metachronous cancer remains high. Although rectal cancer is less
common in HNPCC than colon cancer, it is still significantly higher
than in the general population. If an IRA is performed, surveillance
of the rectum should be continued, but the optimal surveillance
schedule has not been established. Sigmoidoscopy should be performed at least every 2 to 3 years,26 and as the procedure is simple,
many authorities recommend annual examinations. To screen for

PMPH_CH31.indd 264

gastric and small bowel cancers, EGD should be considered starting


between 30 and 35 years of age, repeated every 2 to 3 years. Capsule
endoscopy every 2 to 3 years has been suggested, but there are no
compelling evidence supporting this strategy at this point.24
Evidence for surveillance in hamartomatous polyposis syndromes is very limited due to the rarity of these conditions. Periodic
surveillance of the colon and small bowel should be performed, but
the interval between these studies is not clear and should probably
be referred to centers with significant experience with these conditions. For PJS, colonoscopy and EGD should be started in late teens
and continued every 2 to 3 years. Other extracolonic malignancies
will require radiologic surveillance with computed tomography
(CT) scans (small bowel and pancreas) as well as magnetic resonance imaging (MRI) for screening of breast cancer.
Answer: For FAP, a sigmoidoscopy or colonoscopy should be
performed every 6 to 12 months starting at the age of 12 or until
polyps start to develop; once polyps develop, a full colonoscopy
should be performed until surgery is planned. Even after surgery,
especially after an IRA, surveillance should be continued regularly
because the chance of cancer is not totally eradicated. For patients
with AFAP, colonoscopy surveillance should start before the age of
21 and continued every 1 to 2 years. Colonoscopy frequency should
be decreased after the age of 40 or if polyp burden is increasing. A
periodic upper endoscopy should be performed in both conditions
to screen for duodenal adenomas. For HNPCC, a colonoscopy
every 1 to 2 years should be performed starting between the ages
of 20 and 25 and continued even after a partial colectomy or an
IRA is performed. Regular screening for gynecologic malignancies
should be performed. (Grade B recommendation)
7. What is the role of genetic counseling and testing in inherited colorectal cancer?
To date, over 100 hereditary cancer syndromes have been reported
and each syndrome carries a degree of complexity. Currently in
Lynch syndrome alone at least five clinical criteria exist, more
than five models are available for calculating mutation probabilities, and three (or more) steps are available for tumor analysis, and
germ-line testing for the multiple associated genes may include
sequencing, Southern blotting, multiplex ligation-dependent
probe amplification, and cDNA sequencing/RT-PCR expression
analysesall of which contribute to calculation of risk assessment for the disease, but may not ultimately prove or disprove
the existence of a syndrome. Genetic counselorshealthcare
professionals with graduate degrees in medical genetics/genetic
counselinguse medical histories and pedigree analyses to determine the probability of a hereditary cancer syndrome and to offer
the appropriate options of genetic tests. Patients are informed of
the risks and benefits of genetic testing including the potential
impact on the patients health and family members health. Discussions also address patient concerns regarding genetic discrimination and sharing of information with family members. The NCCN
recommends genetic counseling referral for any patient likely to
carry a hereditary cancer syndrome. Genetic counseling services
are available in person or via telephone or Internet (www.nsgc.
org) and are frequently covered benefits in many health plans.3
Answer: Individuals with suspected inherited colorectal cancer syndrome should be referred to an experience genetic counselor. Counseling will help with identification, risk stratification,
and long-term management of patients and families with these
conditions. (Grade B recommendation)

5/22/2012 5:23:14 PM

Inherited Colorectal Cancer Syndromes

265

Clinical Question Summary


Question

Answer

1 How prevalent are


inherited colorectal
cancer syndromes?

There is significant variability in the estimated prevalence of


inherited colorectal cancers. Based on existing studies,
approximately 5% to 7% of colon cancers have an identifiable
inherited syndrome; however, this number may only be true
for patients of European descent

1, 3, 4, 6, 8

2 What are the major


inherited colorectal
cancer syndromes?

Familial adenomatous polyposis (FAP), MYH associated


polyposis (MAP), and Lynch syndrome (HNPCC) account for
more than 90 percent of currently identified inherited colon
cancer syndromes

2-5, 7-10,
12-14, 36

3 What are the molecular


mechanisms of
the most common
inherited colorectal
cancer syndromes?

Major inherited cancer syndromes identified at this point are


caused by mutations in a distinct gene or genes, most often
inherited in an autosomal dominant fashion. FAP is caused
by a mutation in APC gene, MAP is caused by mutations in
the MYH gene, and Lynch syndrome is caused by a mutation
in one of the following: MLH1, MSH2, MSH6, PMS2 and
EPCAM (TACSTD1)

2, 9, 27

4 How should an
individual with a
suspected inherited
colorectal cancer
syndrome be
evaluated?

Patients with significant family history or significant clinical


features should be suspected of having hereditary colorectal
cancer syndrome and evaluated with molecular genetic
testing and should receive genetic counseling

3-4, 10, 12, 33

5 What is the appropriate


surgery for patient
with inherited
colorectal cancer
syndrome?

For most patients with FAP, total proctocolectomy with or


without an ileoanal pouch should be performed. Total
abdominal colectomy with an ileorectal anastomosis can be
considered for patients with attenuated FAP. Prophylactic
surgery is not indicated for patients with Lynch syndrome,
but close colonoscopic surveillance is required. Should a
colon cancer be diagnosed in a Lynch syndrome patient,
total abdominal colectomy with an ileorectal anastomosis
is generally recommended, but segmental colectomy can
be considered. In addition, a discussion about prophylactic
surgery for gynecological malignancy should be had,
especially in postmenopausal women

19, 22-29

6 What is appropriate
surveillance for patient
with an inherited
colorectal cancer
syndrome?

For FAP, a sigmoidoscopy or colonoscopy should be performed


every 6 to 12 months starting at age 12 years or until polyps
start to develop; once polyps develop, a full colonoscopy
should be performed until surgery is planned. Even after
surgery, especially an IRA, surveillance should be continued
regularly since the chance of cancer is not totally eradicated.
For patients with attenuated FAP, colonoscopy surveillance
should start before age of 21 years and continued every 1
or 2 years. Colonoscopy frequency should be decreased
after age of 40 years if polyp burden is increasing. A periodic
upper endoscopy should be performed in both conditions
to screen for duodenal adenomas. For Lynch syndrome,
a colonoscopy every 1 or 2 years should be performed
starting between the ages of 20 and 25 years and continued
even after a partial colectomy or an IRA is performed.
Regular screening for gynecological malignancies should be
performed

9, 10, 29-31,
34, 35, 37

7 What is the role of


genetic counseling and
testing in inherited
colorectal cancer?

Individuals with suspected inherited colorectal cancer


syndrome should be referred to an experienced genetic
counselor. Counseling will help with identification, risk
stratification, and long term management of patients and
families with these conditions

4, 13, 15, 32,


33

PMPH_CH31.indd 265

Grade of
References
Recommendation

5/22/2012 5:23:14 PM

266

Surgery: Evidence-Based Practice

REFERENCES
1. Lichtenstein P, Holm NV, Varkasalo PK, Iliadou A, Kaprio J,
Koskenvuo M, et al. Environmental and heritable factors in the
causation of cancer. N Eng J Med. 2000;343:2.
2. Power DG, Gloglowski E, Lipkin SM. Clinical genetics of hereditary colorectal cancer. Hematol Oncol Clin N Am. 2010;24:837-859.
3. Trepanier A, McKinnon MA, Peters J, Stopfer J, Grumet SC,
Manley S, et al.; National Society of Genetic Counselors. Genetic
cancer risk assessment and counseling: Recommendations
of the National Society of Genetic Counselors. J Genet Couns.
2004;13(2):83-114.
4. Lindor NM, Rabe K, Petersen GM, Haile R, Casey G, Baron J,
et al. Lower cancer incidence in Amsterdam-I criteria families
without mismatch repair deficiency: Familial colorectal cancer
type X. JAMA. 2005;293(16):1979-1985.
5. Maren TS, Timothy SM, Andrew NF. Population prevalence of
familial cancer and common hereditary cancer syndromes. The
2005 California Health Interview Survey. Genet Med. 2010;12(11):
726-735.
6. Ponz de Leon M, Sassatelli R, Sacchetti C, Zanghieri G, Scalmati
A, Roncucci L. Familial aggregation of tumors in the three-year
experience of a population-based colorectal cancer registry, Cancer Res. 1989;49:4344-4348.
7. Jass JR, Williams CB, Bussey HJ, Morson BC. Juvenile polyposis
a precancerous condition. Histopathology. 1998;13(6):619-630.
8. Lindor NM. Hereditary colorectal cancer: MYH-associated polyposis and other newly identified disorders. Best Pract Res Clin
Gastroenterol. 2009;23(1):75-87.
9. Vasen HF, Mslein G, Alonso A, Aretz S, Bernstein I, Bertario L,
et al. Guidelines for the clinical management of familial adenomatous polyposis (FAP).Gut. 2008;57:704-713.
10. Kempers MJ, Kuiper RP, Ockeloen CW, Chappuis PO, Hutter
P, Rahner N, et al. Risk of colorectal and endometrial cancers
in EPCAM deletion-positive Lynch syndrome: A cohort study.
Lancet Oncol. 2011;12(1):49-55.
11. Young J, Jass JR. The case for a genetic predisposition to serrated
neoplasia in the colorectum: Hypothesis and review of the literature. Cancer Epidemiol Biomarkers Prev. 2006;15(10):1778-1784.
12. Hyman NH, Anderson P, Blasyk H. Hyperplastic polyposis and the
risk of colorectal cancer. Dis Colon Rectum. 2004;47(12):2101-2104.
13. Jrvinen HJ, Aarnio M, Mustonen H, Aktan-Collan K, Aaltonen
LA, Peltomki P, et al. Controlled 15-year trial on screening for
colorectal cancer in families with hereditary nonpolyposis colorectal cancer. Gastroenterology. 2000;118:829-834.
14. Vasen HF, Mslein G, Alonso A, Bernstein I, Bertario L, Blanco I,
et al. Guidelines for the clinical management of Lynch syndrome
(hereditary non-polyposis cancer). J Med Genet. 2007;44:353-362.
15. Lynch HT, Lynch JF, Lynch PM, Attard T. Hereditary colorectal cancer syndromes: Molecular genetics, genetic counseling,
diagnosis and management. Familial Cancer. (2008);7:27-39.
16. Kempers MJ, Kuiper RP, Ockeloen CW, Chappuis PO, Hutter P,
Rahner N, et al. Risk of colorectal and endometrial cancers in
EPCAM deletion-positive Lynch syndrome: A cohort study.
Lancet Oncol. 2011;12(1):49-55.
17. Mller A, Giuffre G, Edmonston TB, Mathiak M, Roggendorf B,
Heinmller E, et al.; German HNPCC Consortium German
Cancer Aid (Deutsche Krebshilfe). Challenges and pitfalls in
HNPCC screening by microsatellite analysis and immunohistochemistry. J Mol Diag. 2004;6(4):308-315.
18. Giardiello FM, Hamilton SR, Krush AJ, Piantadosi S, Hylind
LM, Celano P, et al. Treatment of colonic and rectal adenomas
with sulindac in familial adenomatous polyposis. N Engl J Med.
1993;328:1313-1316.

PMPH_CH31.indd 266

19. Brosens LA, Keller JJ, Offerhaus GJ, Goggins M, Giardiello FM.
Prevention and management of duodenal polyps in familial
adenomatous polyposis. Gut. 2005;54:1034-1043.
20. Clark SK, Neale KF, Landgrebe JC, Phillips RK. Desmoids in
familial adenomatous polyposis. Br J Surg. 1996;83:1494-1504.
21. de Vos tot Nederveen Cappel WH, Buskens E, van Duijvendijk P,
Cats A, Menko FH, Griffioen G, et al. Decision analysis in the
surgical treatment of colorectal cancer due to a mismatch repair
gene defect. Gut. 2003;52:1752-1755.
22. Brown GJ, St John DJ, Macrae FA, Aittomki K. Cancer risk in
young women at risk of hereditary no polyposis colorectal cancer: Implications for gynecological surveillance. Gynecol Oncol.
2001;80:346-349.
23. Saurin JC, Gutknecht C, Napoleon B, Chavaillon A, Ecochard R,
Scoazec JY, et al. Surveillance of duodenal adenomas in familial
adenomatous polyposis reveals high cumulative risk of advanced
disease. J Clin Oncol. 2004;22:493-498.
24. Vasen HF. Clinical diagnosis and management of hereditary
colorectal cancer syndromes. J Clin Oncol. 2000;18(21 Suppl):
81S-92S.
25. Guillem JG, Wood WC, Moley JF, Berchuck A, Karlan BY, Mutch
DG, et al; ASCO; SSO. ASCO/SSO review of current role of riskreducing surgery in common hereditary cancer syndromes.
J Clin Oncol. 2006;24(28):4642-4660.
26. Rodrguez-Bigas MA, Vasen HF, Pekka-Mecklin J, Myrhj T,
Rozen P, Bertario L, et al. Rectal cancer risk in hereditary nonpolyposis colorectal cancer after abdominal colectomy. International
Collaborative Group on HNPCC. Ann Surg. 1997;225:202-207.
27. Nielsen M, Hes FJ, Nagengast FM, Weiss MM, Mathus-Vliegen
EM, Morreau H, et al. Germline mutations in APC and MUTYH
are responsible for the majority of families with attenuated
familial adenomatous polyposis. Clin Genet. 2007;71(5):427-433.
28. Lindor NM, Petersen GM, Hadley DW, Kinney AY, Miesfeldt S,
Lu KH, et al. Recommendations for the care of individuals with
an inherited predisposition to Lynch syndrome: A systematic
review. JAMA. 2006;296(12):1507-1517..
29. Stupart DA, Goldberg PA, Baigrie RJ, Algar U, Ramesar R. Surgery for colonic cancer in HNPCC: Total versus segmental colectomy. Colorectal Dis. 2011;13(12):1395-1399.
30. Vitellaro M, Bonfanti G, Sala P, Poiasina E, Barisella M, Signoroni S, et al. Laparoscopic colectomy and restorative proctocolectomy for familial adenomatous polyposis. Surg Endosc.
2011;25(6):1866-1875.
31. Aarnio M, Mecklin JP, Aaltonen LA, Nystrm-Lahti M, Jrvinen HJ.
Life-time risk of different cancers in hereditary non-polyposis
colorectal cancer (HNPCC) syndrome. Int J Cancer. 1995;64(6):
430-433.
32. Arvanitis ML, Jagelman DG, Fazio VW, Lavery IC, McGannon E.
Mortality in patients with familial adenomatous polyposis. Dis
Colon Rectum. 1990;33:639-642.
33. Howe JR, Mitros FA, Summers RW. The risk of gastrointestinal carcinoma in familial juvenile polyposis. Ann Surg Oncol.
1998;5(8):751-756.
34. Aziz O, Athanasiou T, Fazio VW, Nicholls RJ, Darzi AW, Church J.
Meta-analysis of observational studies of ileorectal versus ileal
pouchanal anastomosis for familial adenomatous polyposis. Br
J Surg. 2006;93:407-417.
35. Olsen K, Juul S, Blow S, Jrvinen HJ, Bakka A, Bjrk J,
et al. Female fecundity before and after operation for familial
adenomatous polyposis. Br J Surg. 2003;90:227-231.
36. Umar A, Boland CR, Terdiman JP, Syngal S, de la Chapelle A,
Rschoff J, et al. Revised Bethesda Guidelines for hereditary
nonpolyposis colorectal cancer (Lynch syndrome) and microsatellite instability. J Natl Cancer Inst. 2004;96(4):261-268.

5/22/2012 5:23:14 PM

CHAPTER 32

Preoperative Bowel Preparation


John K. Bini

INTRODUCTION

and 51 in the nonprepped group, Hughes looked at three outcome


measures: wound infection, peritonitis, and death. He found that
vigorous MBP conveyed no advantage in any of his outcome measures.4 Since Hughes study in 1972, multiple other studies have
been conducted refuting the efficacy of MBP.5-15 There have also
been several meta-analysis that have failed to show any positive
advantage regarding anastomotic or infectious complications
with the routine application of preoperative MBP.16-24 Of the prospective trials refuting the routine use of MBP, all looked at anastomotic and infectious complications.5-15
Four studies in particular standout. Bucher published a study
in the British Journal of Surgery in 2005.11 In this study, which
compared patients prepared with 3 L of polyethylene glycol as
an MBP and no preparation, postoperative complications were
recorded prospectively. They considered abdominal infectious
complications to be anastomotic leak, intra-abdominal abscess,
peritonitis, and wound infection. They found that the group who
underwent bowel preparation had an overall abdominal infectious complication rate of 22%, compared with 8% of those who
did not receive a mechanical preparation. Anastomotic leak rate
in the treated group was 6% compared with 1% in the nontreated
group although this difference did not reach statistical significance.11 Th is study also found that patients undergoing bowel
preparation had statistically significant longer lengths of hospital stay as well as a significantly higher rate of extra-abdominal
morbidity.11
A randomized clinical trial published by Pena-Soria et al.
in 2008 looked at patients undergoing elective colon or proximal rectal resection with primary anastomosis performed by a
single surgeon who was blinded to study arm.15 They analyzed
129 patients, 65 who received polyethylene glycol preparation
(Group A) and 64 who did not (Group B).15 Thirty patients (23.2%)
developed wound infection (Group A = 24.6% and Group B = 17.2%;
NS). There were three cases of intra-abdominal sepsis (Group A =
4.6%).15 The anastomotic failure rate was 5.4% (n = 7), four patients
in Group A (6.2%) versus three patients in Group B (4.7%) (NS).15
When SSI and anastomotic failure were combined, the complication rate in Group A was 35.4% versus 21.9% for Group B.15

In many areas in medicine and surgery, decisions are often based


on tradition, dogma, and the teachings of those who have come
before us based on their experience. Not that there is anything
inherently wrong with practicing experience-based medicine
when that experience is the best available evidence upon which to
base our clinical decisions. The problem lies in the all too present
case when higher-quality evidence emerges and we find ourselves
as clinicians and surgeons tied to the ingrained dogma of our past
practice. Like many areas in this text, the topic of bowel preparation is one where significant clinical equipoise, at least from a
practice standpoint, exists. This chapter attempts to address the
issue of whether or not the best current evidence supports the
routine use of mechanical bowel preparation (MBP) for colon surgery. It will also touch upon the emerging questions of perioperative tissue oxygenation and fluid management when it comes to
outcomes during colon surgery.

MECHANICAL BOWEL PREPARATION


1. Does MBP prior to colon surgery affect the incidence of surgical site infection (SSI) or anastomotic complications when
compared with no bowel preparation?
MBP before elective colon surgery has been the standard surgical practice for over a century.1 Many of us have grown up and
matured as surgeons in an era where MBP was the standard of
practice. Historically, for the most part, the controversy with this
topic was regarding the particular protocol used to prepare the
bowel for surgery. The theory behind MBP with or without an oral
antibiotic regimen was based on the theory that by reducing the
intraluminal contents and bacterial load, postoperative complications of anastomotic failure and infections would be reduced.1-3
The utility of bowel preparation was questioned by Hughes in
a prospective randomized trial presented in 1972. Albeit a small,
single center trial with 46 patients in the bowel preparation group
267

PMPH_CH32.indd 267

5/22/2012 5:23:54 PM

268

Surgery: Evidence-Based Practice

Pena-Soria concluded that a single surgeon will not have a higher


rate of either SSI or anastomotic failure if he/she routinely omits
preoperative MBP.15
Contant et al. completed a multicenter noninferiority trial
that was published in The Lancet in 2007, which compared bowel
preparation with polyethylene glycol or sodium phosphate to no
bowel preparation.13 The study was performed at 13 hospitals and
they randomly assigned 1431 patients who were going to have
elective colorectal surgery to either receive MBP or not.13 Patients
who did not have MBP had a normal meal on the day before the
operation.13 Those who did were given a fluid diet, and MBP with
either polyethylene glycol or sodium phosphate.13 The primary
endpoint was anastomotic leakage, and the study was designed
to test the hypothesis that patients who are given MBP before colorectal surgery do not have a lower risk of anastomotic leakage
than those who are not.13 Seventy-seven patients were excluded:
46 who did not have a bowel resection; 21 because of missing
outcome data; and 10 who withdrew, cancelled, or were excluded
for other reasons.13 The rate of anastomotic leakage did not differ
between groups: 32/670 (4.8%) patients who had MBP and 37/684
(5.4%) in those who did not (difference 0.6%; 95% confidence
interval (CI), 1.7% to 2.9%; p = .69).13 Patients who had MBP had
fewer abscesses after anastomotic leakage than those who did not
(2/670 [0.3%] vs. 17/684 [2.5%], p = .001).13 Other septic complications, fascia dehiscence, and mortality did not differ between
groups and based on their findings, they advised that MBP before
elective colorectal surgery can safely be abandoned.13
Jung et al. published the results of their multicenter randomized clinical trial comparing MBP with no MBP in patients
undergoing elective colon surgery.14 Their primary endpoints
were cardiovascular, general infectious, and surgical-site complications within 30 days, and secondary endpoints were death
and reoperations within 30 days.14 A total of 1343 patients were
evaluated, 686 randomized to MBP and 657 to no MBP.14 There
were no significant differences in overall complications between
the two groups: cardiovascular complications occurred in 5.1% and
4.6%, respectively, general infectious complications in 7.9% and
6.8%, and surgical-site complications in 15.1% and 16.1%.14 At
least one complication was recorded in 24.5% of patients who
had MBP and 23.7% who did not.14 The authors of this study concluded that MBP does not lower the complication rate and can be
omitted before elective colonic resection.14
Multiple meta-analyses and reviews exist in the current
literature that address the subject MBP.1,16-24 Four of the metaanalyses reported statistical differences in leak rate.19-21,24 Three
meta-analyses found no significant difference between the MBP
and the no MBP groups.18,22,23
A meta-analysis published by Guenaga et al. in 2005 as a
Cochrane systematic review was an update of a previously published review from 2003.19,21 This review included nine trials with
a total of 1592 patients.19 The primary outcome measure was anastomotic leakage. Other outcomes looked at included mortality,
peritonitis, the need for reoperation, and superficial surgical site
infection. The authors of this review found a statistically higher
anastomotic leak rate in the group that received a preoperative
MBP (odds ratio [OR], 2.03; 95% CI, 1.2763.26; p = .003).19 They
concluded that routine MBP should be eliminated as it has no
proven benefit and it may also increase anastomotic failure.19
Slim et al. published a recent meta-analysis that included 4859
patients (2452 in the MBP group and 2407 in the no MBP group)

PMPH_CH32.indd 268

and 14 clinical trials.23 Their primary outcome was anastomotic


leakage; secondary outcomes were other septic complications.
The authors of this review found no statistical difference between
the groups for anastomotic leakage (OR, 1.12; 95% CI, 0.821.53;
p = .46), pelvic or abdominal abscess (p = .75), and wound sepsis
(p = .11). When all SSIs were considered, the meta-analysis favored
no MBP (OR, 1.40; 95% CI, 1.051.87; p = .02). Slim et al. also
found that the particular regimen of MBP used did not impact
the outcome measures reviewed.23 They concluded that although
it did not confirm the harmful effect of MBP as suggested by previous meta-analyses. Their meta-analysis including almost 5000
patients demonstrated with a high level of evidence that any kind
of MBP should be omitted before colonic surgery.23
The current literature shows no clear evidence that MBP significantly affects the incidence of SSI either positively or negatively.
However, it is clear that MBP does not increase the risk of SSI after
elective colon surgery. Concerning anastomotic complications,
again, no clear evidence exists that MBP significantly affects the
incidence of anastomotic complications. Based on available trials
and reviews, these are Grade A recommendations.
2. Are there major risks associated with preoperative MBP?
Although it is accepted that the practice of MBP is generally safe
in and of itself, there are many potential complications associated
with it especially in patients with preexisting cardiac and renal
disease. The majority of these complications, which exist in multiple case reports, although rare, are primarily due to the sequel
of severe electrolyte derangements. The electrolyte derangements
include renal failure requiring dialysis, ischemic colitis, dehydration, seizures, cardiac arrythmias, cardiac arrest, and death.25-37
There are significant and potentially lethal outcomes associated with bowel preparation in and of itself. Although the practice
of clearing the colon of contents is necessary for accurate endoscopic visualization, in light of no clear benefit regarding its use
prior to elective surgery, the rare but significant complications
associated with the process must be considered. The current available data support a Grade B recommendation that YES, there are
significant risks associated with preoperative MBP.
3. Should MBP be performed prior to colorectal surgery?
Despite the strong evidence that mechanical bowel preparation
does not reduce complications in elective colon surgery, many
surgeons continue to routinely prepare their patients. It is because
of this that Eskicioglu et al. embarked upon a comprehensive
review which was published in the Canadian Journal of Surgery
as a clinical practice guideline endorsed by the Canadian Society
of Colon and Rectal Surgeons in December 2010.1 These authors
reviewed 14 randomized clinical trials and 8 meta-analyses.1
Based on their findings, these authors made six clinical recommendation regarding various aspects of perioperative management of elective colorectal surgery patients.1 Importantly, they
concluded that there was good evidence to support the routine
omission of MBP for patients undergoing routine elective rightand left-sided colon resections.1 Regarding low-anterior resection
and rectal resections with and without protective ileostomy, they
found insufficient evidence to support or refute the routine use of
bowel preparation.1
Regarding the routine use of MBP prior to elective colon surgery, the majority of the evidence supports omitting the practice.

5/22/2012 5:23:55 PM

Preoperative Bowel Preparation

The major body of the evidence is presented in the setting of


open surgical procedures involving the colon. Specific evidence
addressing laparoscopic surgery and rectal surgery does not support recommendations either for or against MBP in these settings. There are some randomized trials and meta-analyses that
report higher infectious and anastomotic complication rates in
patients undergoing MBP prior to elective surgery. However, most
evidence merely support the assertion that no advantage to MBP
exists. The procedure is uncomfortable for patients and may be
associated with rare but serious medical complications in patients
with preexisting renal and cardiac disease.1,25-37
Based on the availability of high-quality medical evidence,
the answer to the question of whether MBP should be routinely
performed prior to colorectal surgery is NO. It should also be
noted that data are mixed regarding low-rectal surgery requiring
anastomosis as one study did show an increase in pelvic abscess
with bowel preparation when a protective ostomy was not used.13
It is therefore recommended that in cases of elective colon surgery
that the practice of MBP be eliminated. In low-rectal surgery, if
diversion is not planned, MBP is probably appropriate. The literature supports this as a Grade A recommendation.

PERIOPERATIVE FLUID MANAGEMENT


4. Does intraoperative fluid volume affect the incidences of
postoperative complications and anastomotic complications in
colorectal surgery?
The effect of perioperative fluid management on outcomes in
elective colon surgery is another area that warrants critical inspection. There are a limited number of clinical studies in the last
decade that compared the effect of restrictive with liberal perioperative fluid volume regimens on postoperative recovery time and
complications.38-41 Patients treated with restrictive fluid management show significantly lower increase in body weight and pass
flatus and feces earlier.38 However, data of the impact of volume
restriction on postoperative hospital stay are inconsistent.38,41
A recent animal study by Marjanovic et al. sought to determine the effect of perioperative volume management on the integrity of intestinal anastomosis.38 The authors randomized 21 rats
to three experimental groups (n = 7 rats/group): control group
CO (9 mL/kg/h crystalloid infusion), volume restriction group
(3 mL/kg/h), and animals with volume overload (36 mL/kg/h). An
end-to-end small bowel anastomosis was performed with eight
nonabsorbable interrupted inverting sutures.38 At reoperation on
the 4th postoperative day, the anastomotic segment was dissected
and the bursting pressure was measured. As a second parameter for the quality of anastomotic healing, hydroxyproline concentration was examined with a spectrophotometric method.38
Histologically, structural changes of the anastomotic segments
were assessed by two pathologists. The group found that bursting pressure was lowest in volume overload and significantly
lower in volume restricted group. 38 Additionally, hydroxyproline concentration in volume overload group was significantly
lower compared with volume restricted group, and in all animals
with volume overload a marked submucosal edema was found.38
Marjanovic et al. concluded that the quantity of crystalloid infusion, applied intraoperatively, has a significant impact on functional (bursting pressure) and structural (hydroxyproline) stability

PMPH_CH32.indd 269

269

of intestinal anastomoses in the early postoperative period and


that volume overload may have deleterious effects on anastomotic
healing and postoperative complications in digestive surgery, possibly because of a marked bowel wall edema.38
Brandstrup et al. conducted a randomized clinical trial that
investigated the effect of a restrictive fluid management strategy
versus a standard regimen on complications after colorectal surgery.40 They randomized 172 patients to either a restricted or a standard intraoperative and postoperative intravenous fluid regimen.
The restricted regimen aimed at maintaining preoperative body
weight and the standard regimen resembled everyday practice.
The primary outcome measures were complications and the secondary measures were death and adverse effects.40 The restrictive
fluid regimen significantly reduced postoperative complications
both by intention-to-treat (33% vs. 51%, p = .013) and per-protocol
(30% vs. 56%, p = .003) analyses.40 The numbers of both cardiopulmonary (7% vs. 24%, p = .007) and tissue-healing complications
(16% vs. 31%, p = .04) were significantly reduced.40 There were no
deaths in the restricted group and four deaths in the standard
group. They showed that restricted fluid management in elective
abdominal surgery significantly reduces postoperative mortality and morbidity.40 This was expressed in lower cardiopulmonary (i.e., myocardial infarction, pneumonia) and tissue healing
(i.e., anastomotic failure) complications.40
Khoo et al. conducted a prospective randomized controlled
trial (RCT) of multimodal perioperative management protocol in
patients undergoing elective colorectal resection for cancer.39 Participants were stratified by sex and requirement for a total mesorectal excision and were centrally randomized.39 Protocol patients
received intravenous fluid restriction, unrestricted oral intake with
prokinetic agents, early ambulation, and fixed regimen epidural
analgesia.39 Control patients received intravenous fluids to prevent
oliguria, restricted oral intake until return of bowel motility, and
weaning regimen epidural analgesia criteria.39 The primary endpoint was postoperative stay. Secondary endpoints were postoperative complications, readmission rates, and mortality.39 Of the 70
patients recruited, the median stay was significantly reduced in the
protocol group (5 vs. 7 days; p = .001).39 Patients in the control arm
were 2.5 times as likely to require a postoperative stay of more than
5 days.39 Patients in the group that was fluid restricted had fewer
cardiorespiratory and anastomotic complications and two deaths,
both occurred in controls.39 These authors concluded that a multimodal management protocol can significantly reduce postoperative stay following colorectal cancer surgery and that morbidity and
mortality are not increased.39 Although limited in numbers of participants, these studies indicate a higher risk for anastomotic insufficiency for patients with liberal fluid management.38-40
The available evidence in pertaining to fluid volume during colon surgery support a Grade A recommendation that YES,
fluid volume affects the incidence of operative and anastomotic
complications in colorectal surgery. Regarding a restrictive fluid
management strategy during colorectal surgery, there are clinical and laboratory data to support the practice. Although clinical
trials are few and relatively small when considering the numbers
of patients, they do favor a restrictive vice liberal perioperative
fluid management strategy.38-41 It is further recommended that
attempts should be made in hemodynamically stable patients
undergoing elective colon surgery requiring an anastomosis to
follow a restrictive fluid management strategy to minimize the
risk of anastomotic complications.

5/22/2012 5:23:55 PM

270

Surgery: Evidence-Based Practice

PERIOPERATIVE OXYGEN
ADMINISTRATION
5. Does perioperative fraction of inspired oxygen (FiO2) affect
the incidences of SSI and anastomotic complications in colorectal surgery?
The perioperative administration of supplemental oxygen has
been variably reported with respect to SSIs.42-44 In 2007, Brasel and
the members of the Evidence Based Reviews in Surgery Group
determined that the risk of SSI was reduced with administration
of perioperative oxygen.42 They based their assertion largely on
the findings of a 2005 RCT by Belda et al.42,43
Belda conducted a double-blind, RCT of 300 patients aged
between 18 and 80 years who underwent elective colorectal surgery in 14 Spanish hospitals.43 Patients were randomly assigned
to either 30% or 80% FIO2 intraoperatively and for 6 h after surgery. Anesthetic treatment and antibiotic administration were
standardized.43 The primary outcome measure was any SSI. Secondary outcomes included the return of bowel function and the
ability to tolerate solid food, ambulation, suture removal, and
duration of hospitalization.43 SSI occurred in 35 patients (24.4%)
administered 30% FIO2 and in 22 patients (14.9%) administered
80% FIO2 (p = .04).43 The risk of SSI was 39% lower in the 80%
FIO2 group (relative risk [RR], 0.61; 95% CI, 0.380.98) than in
the 30% FIO2 group.43 None of the secondary outcomes varied
significantly between the two treatment groups.43 These findings
led the authors to conclude that patients receiving supplemental
inspired oxygen had a significant reduction in the risk of wound
infection, which appears to be an effective intervention to reduce
SSI in patients undergoing colon or rectal surgery.43
Most recently, Meyhoff and the PROXI Trial group conducted
a trial in 14 hospitals with 1386 patients that looked at SSI infection in patients undergoing acute or elective laparotomy.44 This
group set out to assess whether the use of 80% oxygen reduces
the frequency of SSI without increasing the frequency of pulmonary complications in patients undergoing abdominal surgery.44
The PROXI trial was a patient- and observer-blinded randomized clinical trial.44 Patients were randomly assigned to receive
either 80% or 30% oxygen during and 2 h after surgery.44 The
primary outcome was SSI within 14 days. Secondary outcomes

included atelectasis, pneumonia, respiratory failure, and mortality.44 The authors found that SSI occurred in 131 of 685 patients
(19.1%) assigned to receive 80% oxygen versus 141 of 701 (20.1%)
assigned to receive 30% oxygen (OR, 0.94; 95% CI, 0.721.22;
p = .64).44 Atelectasis occurred in 54 of 685 patients (7.9%) assigned
to receive 80% oxygen versus 50 of 701 (7.1%) assigned to receive
30% oxygen (OR, 1.11; 95% CI, 0.751.66; p = .60), pneumonia in
41 (6.0%) versus 44 (6.3%) (OR, 0.95; 95% CI, 0.611.48; p = .82),
respiratory failure in 38 (5.5%) versus 31 (4.4%) (OR, 1.27; 95%
CI, 0.782.07; p = .34), and mortality within 30 days in 30 (4.4%)
versus 20 (2.9%) (OR, 1.56; 95% CI, 0.882.77; p = .13).44 This large
study concluded that the administration of 80% oxygen compared
with 30% oxygen did not result in a difference in risk of SSI after
abdominal surgery.44
Results of various studies and reviews provide conflicting findings regarding the use of supranormal levels of oxygen to reduce SSI. However, it does not appear that elevated
supplemental oxygen levels are deleterious. Certain types of
gastrointestinal surgery may benefit from higher oxygen levels.
However, no clear data exist to support the general practice of
using higher levels of oxygen than what are needed to support
systemic oxygenation.
On the subject of increased perioperative oxygen levels to
reduce SSI, results of various studies and reviews are conflict
ing. Although certain types of gastrointestinal surgery may benefit from higher oxygen levels, no clear data exist to support the
general practice of using higher levels of oxygen than what are
needed to support systemic oxygenation.42-44 However, it should
be noted that there is little cost or risk associated with the use
of high oxygen levels. Also, there appears to be a mechanism for
its benefit via higher measured oxygen tension in the tissues,
improved leukocyte activity and lower bacterial counts.45 Therefore, one may consider a higher FiO2, particularly when operating
on obese patients as they have higher SSI rate and more poorly
perfused subcutaneous tissues. Because of the conflicting data, we
can only make a Grade D recommendation regarding the relationship between perioperative FiO2 and complications following
colorectal surgery, and can only conclude that significant clinical
equipoise still exists regarding the appropriate level of oxygen to
be administered during colorectal surgery.

Clinical Question Summary


Question

Answer

1 Does mechanical bowel


preparation prior to
colon surgery affect the
incidence of surgical site
infection or anastomotic
complications when
compared to no bowel
preparation?

No clear evidence exists that mechanical bowel preparation


significantly affects the incidence of SSI. There is some
data that suggest that SSI may be increased in patients who
undergo mechanical bowel preparation prior to elective
colon surgery. It is clear however that mechanical bowel
preparation does not increase the risk of SSI after elective
colon surgery.

Grade of
References
Reccomendation
A

1, 3, 4, 5, 6,
7, 8, 9, 10,
11, 12, 13,
14, 15, 16,
17, 18, 19,
20, 21, 22,
23, 24

(Continued)

PMPH_CH32.indd 270

5/22/2012 5:23:55 PM

Preoperative Bowel Preparation

271

(Continued)
Question

Answer

Grade of
References
Reccomendation

No clear evidence exists that mechanical bowel preparation


significantly affects the incidence of anastomotic
complications. There is some data that suggest that
anastomotic complications may be increased in patients
who undergo mechanical bowel preparation prior to
elective colon surgery. It is clear however that mechanical
bowel preparation does not increase the risk of
anastomotic complications after elective colon surgery.
2 Are there majors risks
associated with preoperative mechanical
bowel preparation?

Yes. While it is accepted that the practice of mechanical bowel


preparation is generally safe in and of itself, there are many
potential risks associated with it especially in patients with
pre-existing cardiac and renal disease. They include renal
failure requiring dialysis, ischemic colitis, dehydration,
seizures, cardiac arrythmias, cardiac arrest and death.

25, 26, 27,


28, 29,
30, 31,
32, 33,
34, 35,
36, 37

3 Should mechanical bowel


preparation be performed
prior to colorectal
surgery?

No. Routine mechanical bowel preparation should not be


performed in patients undergoing elective colon surgery
where there is no plan to perform intra-operative
endoscopy.

1, 3, 4, 5, 6,
7, 8, 9,
10, 11, 12,
13, 14, 15,
16, 17, 18,
19, 20, 21,
22, 23, 24

4 Does intra-operative
fluid volume affect the
incidences of postoperative complications
and anastomotic
complications in colorectal
surgery?

Yes. Although limited in numbers of participants, clinical


studies indicate a higher risk for anastomotic insufficiency
and medical complications for patients with liberal
perioperative fluid management compared to those who
have a restrictive fluid management regimen.

38, 39, 40,


41

5 Does peri-operative FiO2


affect the incidences of
surgical site infection and
anastomotic complications
in colorectal surgery?

Results of various studies and reviews provide conflicting


findings regarding the use of supranormal levels of oxygen
in order to reduce SSI. It does not however appear that
elevated supplemental oxygen levels are deleterious. Certain
types of gastrointestinal surgery may benefit from higher
oxygen levels, however, no clear data exist to support the
general practice of using higher levels of oxygen than what is
needed to support systemic oxygenation.

42, 43, 44,


45

REFERENCES
1. Eskicioglu C, Forbes SS, Fenech DS, McLeod RS. Preoperative
bowel preparation for patients undergoing elective colorectal surgery: A clinical practice guideline endorsed by the Canadian Society of Colon and Rectal Surgeons. Canadian J Surgery. 2010;53(6):
385-395.
2. Bartlett JG, Condon RE, Gorbach SL, Clarke JS, Nichols RL,
Ochi S.Veterans administration cooperative study on bowel
preparation for elective colorectal operations: Impact of oral
antibiotic regimen on colonic flora, wound irrigation cultures and bacteriology of septic complications. Ann Surg.
1978;188(2):249-254.
3. Chung RS, Gurll NJ, Berglund EM. A controlled trial of whole
gut lavage as a method of bowel preparation for colonic operations. Am J Surg. 1979;137:75-81.

PMPH_CH32.indd 271

4. Hughes ES. Asepsis in large bowel surgery. Ann R Coll Surg Engl.
1972;137:347-356.
5. Zmora O, Mahajna A, Bar-Zakai B, Rosin D, Hershko D, Shabtai
M, Krausz MM, Ayalon A. Colon and rectal surgery without
mechanical bowel preparation a randomized prospective trial.
Ann Surg. 237(3):363-367.
6. Burke P, Mealy K, Gillen P, et al. Requirement for bowel preparation in colorectal surgery. Br J Surg. 1994;81:907-910.
7. Santos JC, Jr., Batista J, Sirimarco MT, Guimares AS, Levy CE.
Prospective randomized trial of mechanical bowel preparation
in patients undergoing elective colorectal surgery. Br J Surg.
1994;81:1673-1676.
8. Miettien RP, Laitinen ST, Makela JT, Paakkonen ME. Bowel
preparation with oral polyethylene glycol electrolyte solution vs.
no preparation in elective colorectal surgery:Prospective randomized study. Dis Colon Rectum. 2000;43(5):669-675.

5/22/2012 5:23:55 PM

272

Surgery: Evidence-Based Practice

9. Fa-Si-Oen P, Roumen R, Buitenweg J, van de Velde C, van Geldere


D, Putter H, Verwaest C, Verhoef L, de Waard JW, Swank D,
DHoore A, Croiset van Uchelen F. Mechanical bowel preparation or not? Outcome of a multicenter, randomized trial in elective open colon surgery. Dis Colon Rectum. 2005;48(8):1509-1516.
10. Ram E, Sherman Y, Weil R, Vishne T, Kravarusic D, Dreznik Z.
Is mechanical bowel preparation mandatory for elective colon
surgery? A prospective randomized study. Arch Surg. 2005;140:
285-288.
11. Bucher P, Gervaz P, Soravia C, et al. Randomized clinical trial
of mechanical bowel preparation versus no preparation before
elective left-sided colorectal surgery. Br J Surg. 2005;92:409-414.
12. Platell C, Barwood N, Makin G. Randomized clinical trial of
bowel preparation with a single phosphate enema or polyethylene glycol before elective colorectal surgery. Br J Surg. 2006;93(4):
427-433.
13. Contant CME, Hop WCJ, van t Sant HP, Oostvogel HJM, Smeets
HJ, Stassen LPS, Neijenhuis PA, Idenburg FJ, Dijkhuis CM, Heres
P, van Tets WF, Gerritsen JJGM, Weidema WF. Mechanical
bowel preparation for elective colorectal surgery: A multicentre
randomised trial. Lancet. 2007;370:2112-2117.
14. Jung B, Pahlman L, Nystrom PO, Nilsson E. Multicentre randomized clinical trial of bowel preparation in elective colonic
resection. Br J Surg. 2007;94(6):689-695.
15. Pena-Soria MJ, Mayol JM, Anula R, Arbeo-Escolar A, Fernandez-Represa JA. Single-blinded randomized trial of mechanical
bowel preparation for colon surgery with primary intraperitoneal anastomosis. J Gastrointest Surg. 2008;12(12):2103-2108.
16. Slim K, Vicaut E, Panis Y, et al. Meta-analysis of randomized
clinical trials of colorectal surgery with or without mechanical
bowel preparation. Br J Surg. 2004;91:1125-1130.
17. Wille-Jorgensen P, Guenaga KF, Matos D, et al. Preoperative
mechanical bowel cleansing or not? An updated meta-analysis.
Colorectal Dis. 2005;7:304-310.
18. Pineda CE, Shelton AA, Hernandez-Boussard T, et al. Mechanical bowel preparation in intestinal surgery: A meta-analysis and
review of the literature. J Gastrointest Surg. 2008;12:2037-2044.
19. Guenaga KF, Matos D, Castro AA, et al. Mechanical bowel
preparation for elective colorectal surgery. Cochrane Database
Systematic Reviews. 2005;(1):CD001544.
20. Bucher P, Mermillod B, Gervaz P, et al. Mechanical bowel preparation for elective colorectal surgery. A meta-analysis. Arch Surg.
2004;139:1359-1364.
21. Wille-Jorgensen P, Guenaga KF, Castro AA, et al. Clinical value
of preoperative mechanical bowel cleansing in elective colorectal surgery: A systematic review. Dis Colon Rectum. 2003;46:
1013-1020.
22. Platell C, Hall J. What is the role of mechanical bowel preparation in patients undergoing colorectal surgery. Dis Colon Rectum.
1998;41:875-883.
23. Slim K, Vicaut E, Launay-Savary M, et al. Updated systematic
review and meta-analysis of randomized clinical trials on the
role of mechanical bowel preparation before colorectal surgery.
Ann Surg. 2009;249:203-209.
24. Muller-Stich BP, Choudhry A, Vetter G, et al. Preoperative
bowel preparation: Surgical standard or past? Dig Surg. 2006;23:
375-380.
25. Gray M, Colwell JC. Mechanical bowel preparation before elective colorectal surgery. J Wound Ostomy Continence Nurs. 2005;
32:360-364.
26. Frizelle FA, Colls BM. Hyponatremia and seizures after
bowel preparation: Report of three cases. Dis Colon Rectum.
2005;48:393-396.

PMPH_CH32.indd 272

27. Ayus JC, Levine R, Arieff AI. Fatal dysnatraemia caused by elective colonoscopy. Br Med J. 2003;326:382-384.
28. Mackey AC, Shaffer D, Prizant R. Seizure associated with the use
of visicol for colonoscopy. N Engl J Med. 2002;346:2095.
29. Hookey LC, Depew WT, Vanner S. The safety profile of oral
sodium phosphate for colonic cleansing before colonoscopy in
adults. Gastrointest Endosc. 2002;56:895-902.
30. Tan HL, Liew QY, Loo S, et al. Severe hyperphosphatemia and
associated electrolyte and metabolic derangement following
the administration of sodium phosphate for bowel preparation.
Anaesthesia. 2002;57:478-483.
31. Ullah N, Yeh R, Ehrinpreis M. Fatal hyperphosphatemia from a
phosphasoda bowel preparation. J Clin Gastroenterol. 2002;34:
457-458.
32. Adverse Drug Reactions Advisory Committee. Electrolyte disturbances with sodium picosulfate bowel cleansing products.
Aust Advers Drug React Bull. 2002;21:1.
33. Franga DL, Harris JA. Polyethylene glycol-induced pancreatitis.
Gastrointest Endosc. 2000;52:789-791.
34. Boivin MA, Kahn SR. Symptomatic hypocalcemia from oral
sodium phosphate: A report of two cases. Am J Gastroenterol.
1998;93:2577-2579.
35. Oh JK, Meiselman M, Lataif LE, Jr. Ischemic colitis caused by
oral hyperosmotic saline laxatives. Gastrointest Endosc. 1997;45:
319-322.
36. Vukasin P, Weston LA, Beart RW. Oral fleet phospho-soda
laxative-induced hyperphosphatemia and hypocalcemic tetany
in an adult: Report of a case. Dis Colon Rectum. 1997;40:497-499.
37. Adverse Drug Reactions Advisory Committee. Electrolyte disturbances with oral phosphate bowel preparations. Aust Advers
Drug React Bull. 1997;16:2.
38. Marjanovic G, Villain C, Juettner E, zur Hausen A, Hoeppner J,
Hopt UT, Drognitz O, Obermaier R. Impact of different crystalloid volume regimes on intestinal anastomotic stability. Ann
Surg. 2009;249:181-185.
39. Khoo CK, Vickery CJ, Forsyth N, et al. A prospective randomized controlled trial of multimodal perioperative management
protocol in patients undergoing elective colorectal resection for
cancer. Ann Surg. 2007;245:867-872.
40. Brandstrup B, Tonnesen H, Beier-Holgersen R, et al. Effects of
intravenous fluid restriction on postoperative complications:
Comparison of two perioperative fluid regimens: A randomized
assessor-blinded multicenter trial. Ann Surg. 2003;238:641-648.
41. MacKay G, Fearon K, McConnachie A, et al. Randomized clinical trial of the effect of postoperative intravenous fluid restriction
on recovery after elective colorectal surgery. Br J Surg. 2006;93:
1469-1474.
42. Brasel K, McRitchie D, Dellinger P. Canadian Association of General Surgeons and American College of Surgeons Evidence Based
Reviews in Surgery. 21. The risk of surgical site infection is reduced
with perioperative oxygen. Can J Surg. 2007;50(3):214-216.
43. Meyhoff CS, Wetterslev J, Jorgensen LN, Henneberg SW, Hgdall
C, Lundvall L, et al. Effect of high perioperative oxygen fraction
on surgical site infection and pulmonary complications after
abdominal surgery: The proxi randomized clinical trial. JAMA.
2009;302(14):1543-1550.
44. Belda FJ, Aguilera L, Garca de la Asuncin J, Alberti J, Vicente R,
Ferrndiz L, et al. Supplemental perioperative oxygen and the
risk of surgical wound infection: A randomized controlled trial.
JAMA. 2005;294(16):2035-2042.
45. Greif R, Akca O, Horn EP, Kurz A, Sessler DI. Supplemental
perioperative oxygen to reduce the incidence of surgical-wound
infection. N Engl J Med. 2000;342(3):161-167.

5/22/2012 5:23:55 PM

Commentary on Preoperative
Bowel Preparation
To Prep or Not to Prep: What is the Question?
Donald E. Fry

The chapter by Bini entitled Preoperative Bowel Preparation


correctly summarizes the clinical trials over the past decade that
have demonstrated no benefit in the prevention of surgical site
infection (SSI) by using mechanical bowel preparation (MBP)
alone in elective colon surgery. This chapter and other referenced
articles have concluded that old dogma has been overturned and
that the new evidence favors the omission of MBP in the future.
To reach this conclusion requires that the history and evolution of
colon surgery be ignored. MBP remains a critical part of the constellation of several interventions that are necessary for all elective colon resections. To ignore its appropriate context is to put
patients at-risk.

In parallel developments of the late-1960s and 1970s, systemic


antibiotics prevented SSIs in elective colon surgery. This was first
demonstrated by Polk and Lopez-Mayor.8 In 1983, Baum et al.9
showed that additional trials of systemic antibiotics in elective
colon surgery were not necessary. Song and Glenny10 presented
evidence that perioperative systemic antibiotics need not be continued postoperatively. Through this development of systemic
antibiotics, MBP alone persisted as a practice.
Thus, in the 1970s and 1980s systemic preventive antibiotics
and the oral antibiotic bowel preparation evolved. By the 1990s,
surgeons in the United States used both methods together.11,12 The
logic was that oral antibiotics reduced luminal bacterial concentrations, and systemic antibiotic provided a safety net to cover
contamination of the soft tissues. Lewis provided a convincing
clinical trial and a meta-analysis of an additional 12 clinical trials which demonstrated reduction (p < .0001) in SSI rates using
the oral antibiotic bowel preparation and systemic antibiotics
together, as opposed to systemic antibiotics alone.13
However, in the United States the 1990s also brought the era
of managed care. The preoperative admission day for MBP and
the oral antibiotic preparation were eliminated. MBP was to be
performed by the patient and their family at home prior to sameday admission and operation. Compliance with MBP was poor
and negated potential benefits of orally administered antibiotics.
Patients complained about the discomfort of MBP and the oral
antibiotics (e.g., erythromycin) at home. Surgeons correctly challenged whether suboptimal MBP actually served any purpose,
and disillusionment emerged about any preparation for elective
colon surgery. This led to clinical trials which have been consistent and correct in their conclusions. As was known 70 years ago,
the answer remained that MBP alone did not reduce SSIs.

HISTORY OF COLON PREPARATION


In the late-1930s, sulfa compounds were developed with activity
against pathogenic bacteria. One of the earliest efforts to use sulfa
derivatives was for oral colonic antimicrobial bowel preparation.
Systemic antibiotics for the prevention of SSI had not been developed. Elective colon surgery in the 1930s was noted to have a mortality rate of 10% to 12% and SSI rates greater than 80%. The reason
for the staggering rate of infection in elective colon surgery,1 and
the reason for the vigorous pursuit of oral intestinal antisepsis was
because MBP alone was ineffective in the prevention of SSI.
MBP alone has been recognized since the 1930s not to reduce
microbial concentration on the mucosal surface. Garlock and
Seley2 knew that. Firor and Poth knew it.3 The distal human colorectum has 1012 bacteria per gram on the mucosal surface.4 The
oral antibiotic bowel preparation was pursued through the 1950s
and 1960s, because MBP alone did not reduce SSI rates.
Finally, in the 1970s the oral antibiotic bowel preparation in
conjunction with MBP was shown to reduce SSIs when compared
to placebo controls. Washington et al.5 successfully used oral
neomycin and tetracycline, while Clarke et al.6 demonstrated the
effectiveness of oral neomycin and erythromycin base that had
been popularized by Nichols and Condon.7 MBP was necessary
to evacuate the massive microbe-laden stool bulk. MBP permitted
oral antibiotics to access and reduce the microbial concentration
at the mucosal surface.

WHAT IS THE QUESTION?


The real question should be How can we improve MBP, oral antibiotic administration, and preoperative systemic antibiotics into
a refined strategy that gives the best outcomes? Preoperative systemic antibiotics alone have left us with an SSI rate that exceeds
20% in most studies of colon resection with 30 days of postoperative
273

PMPH_CH32.indd 273

5/22/2012 5:23:55 PM

274

Surgery: Evidence-Based Practice

follow-up.14-16 The study by Lewis13 and my own meta-analysis17


indicate that SSIs can be further reduced with MBP and oral antibiotic bowel preparation. The real questions are as follows:
How can we improve mechanical preparation? It must be thorough
and complete. Patients and surgeons must understand its
importance. MBP may need to extend over 2 to 3 days to be
tolerated and complete. Elderly patients may need supervised
MBP in ambulatory facilities or even an additional preoperative
day to insure completeness. Intravenous volume support may
be necessary in the elderly. Ineffective preparation is a patient
liability. Bucher et al.18 used a polyethylene glycol preparation
only 12 to 16 h before the operation. Residual polyethylene
glycol solution laden with billions of microbes will increase
SSIs if MBP is ineffective. We need better tolerated and more
effective MBP methods.
What is the best timing of oral antibiotic administration? The
oral antibiotics cannot be given simultaneously with MBP as
antibiotics will pass undissolved from the colon. MBP must be
complete before the administration of the oral antibiotics. The
best timing appears to be 18 to 24 h before the skin incision, but
that has not been tested with controlled trials.
What is the best oral antibiotic choice? Neomycin-erythromycin
has been effective as has neomycin-metronidazole. Lewis19 has
challenged whether neomycin is needed at all. There are many
drugs that are poorly absorbed that could have utility in this
clinical role. Better choices require alternative evaluation.
Is Clostridium difficile infection a risk of oral antibiotics with
MBP? A report identifies a retrospective association.20 This risk
may underscore the need for a probiotic strategy to recolonize
the patient after operation. More studies are necessary.

CONCLUSIONS
MBP alone does not reduce SSI. No further studies are needed.
The real question is whether evidence-based efforts can be generated to refine and improve oral antibiotic bowel preparation,
which must be performed in conjunction with effective MBP.

REFERENCES
1. Poth EJ. Historical development of intestinal antisepsis. World
J Surg. 1982;6:153-159.
2. Garlock JH, Seley GP. The use of sulfanilamide in surgery of the
colon and rectum. Preliminary report. Surgery. 1939;5:787.
3. Firor WM, Poth EJ. Intestinal antisepsis with special reference to
sulfanilylguanidine. Ann Surg. 1941;114:663-671.
4. Ahmed S, Macfarlane GT, Fite A, et al. Mucosa-associated bacterial density in relation to human terminal ileum and colonic
biopsy samples. Appl Environ Microbiol. 2007;73:7435-442.

PMPH_CH32.indd 274

5. Washington JA, II, Dearing WH, Judd ES, Elveback LR. Effect
of preoperative antibiotic regimen on development of infection
after intestinal surgery: Prospective, randomized, double-blind
study. Ann Surg. 1974;180:567-571.
6. Clarke JS, Condon RE, Bartlett JG, et al. Preoperative oral antibiotics reduce septic complications of colon operations: Results of
prospective, randomized, double-blind clinical study. Ann Surg.
1977;186:251-259.
7. Nichols RL, Briodo P, Condon RE, et al. Effect of preoperative
neomycin-erythromycin intestinal preparation on the incidence
of infectious complications following colon surgery. Ann Surg.
1973;178:453-459.
8. Polk HC, Jr., Lopez-Mayor JF. Postoperative wound infection: A
prospective study of determinant factors and prevention. Surgery. 1969;66:97-103.
9. Baum ML, Anish DS, Chalmers TC, et al. A survey of clinical trials of antibiotic prophylaxis in colon surgery: Evidence against
further use of no-treatment controls. N Engl J Med. 1981;305:
795-799.
10. Song F, Glenny AM: Antimicrobial prophylaxis in colorectal
surgery: a systematic review of randomized controlled trials.
Br J Surg. 1998;85:1232-1244.
11. Solla JA, Rothenberger DA. Preoperative bowel preparation.
A survey of colon and rectal surgeons. Dis Colon Rectum.
1990;33:154-159.
12. Nichols RL, Smith JW, Garcia RY, et al. Current practices of preoperative bowel preparation among North American colorectal
surgeons. Clin Infect Dis. 1997;24:609-619.
13. Lewis RT. Oral versus systemic antibiotic prophylaxis in elective
colon surgery: A randomized study and meta-analysis send a
message from the 1990s. Can J Surg. 2002;45:173-180.
14. Milsom JW, Smith DL, Corman ML, et al. Double-blind comparison of single-dose alatrofloxacin and cefotetan as prophylaxis of infection following elective colorectal surgery. Am
J Surg. 1998;176(Suppl 6A):46S-52S.
15. Itani KMF, Wilson SE, Awad SS, et al. Ertapenem versus cefotetan prophylaxis in elective colorectal surgery. N Engl J Med.
2006;355:2640-2651.
16. Smith RL, Bohl JK, McElearney ST, et al. Wound infection after
elective colorectal resection. Ann Surg. 2004;239:599-607.
17. Fry DE. Colon preparation and surgical site infection. Am J Surg.
2011 (in press).
18. Bucher P, Gervaz P, Soravia C, et al. Randomized clinical
trial of mechanical bowel preparation versus no preparation
before elective left-sided colorectal surgery. Br J Surg. 2005;92:
409-414.
19. Lewis RT, Goodall RG, Marien M, et al. Is neomycin necessary
for bowel preparation in surgery of the colon? Oral neomycin
plus erythromycin versus erythromycin-metronidazole. Can
J Surg. 1989;32:265-278.
20. Wren SM, Ahmed N, Jamal A, Safadi BY. Preoperative oral antibiotics in colorectal surgery increase the rate of Clostridium difficile colitis. Arch Surg. 2005;140:752-756.

5/22/2012 5:23:55 PM

CHAPTER 33

Appendicitis
Damon Kalcich and Peter P. Lopez

INTRODUCTION

of acute appendicitis. Important questions to consider in the care


of a patient with appendicitis include how to make an accurate
diagnosis, whether to treat it medically or surgically, and whether
to proceed surgically with an open or laparoscopic approach. The
answers to these questions are based on an evidence-based review
of the literature.

Acute appendicitis remains the most common intra-abdominal


surgical emergency requiring operation. The lifetime risk of developing appendicitis is around 7% to 12%; however, the lifetime rate
of an appendectomy is 12% for men and 25% for women. Appendicitis can occur at any age but is most frequently seen in patients
in their second and fourth decade of life, with a mean age of 31.3
and a median age of 22 years.1
Early reports describe a potentially lethal inflammatory disease process of the right lower quadrant (RLQ), then known as
perityphlitis. In 1886, Reginald Fitz2 first described this inflammatory disease process of the RLQ as appendicitis, including
the clinical sequelae of abscess formation and perforation. Even
today, the diagnosis of acute appendicitis remains a challenging
clinical entity. This condition is more difficult to diagnose at the
extremes of age: in the very young and elderly because of a lack of
history, late presentation and often less than impressive physical
examination. The diagnoses can also be challenging in women of
childbearing age who have a wider list of differential diagnoses.
The timely and accurate recognition of patients requiring
urgent surgical and nonsurgical management continues to be the
overriding principle in the workup and treatment of patients with
suspected appendicitis. Delays in the diagnosis and treatment of
appendicitis can result in an increased morbidity and mortality.
In addition, untimely presentation and a lack of access to emergency healthcare present surgeons with complicated patients
and associated morbidities. Krajewski et al. recently published
a retrospective review that shows a significant, inverse relationship between economic status and perforated appendicitis. This
study compared Canadian data of health systems with the U.S.
data, revealing that rates of perforated appendicitis in American
patients increased as income level decreases.3 As patients continue
to present, the body of evidence grows; as budgets are scrutinized
and resources strained, it seems desirable that evidence-based
guidelines ought to be developed.
In this chapter, we try to answer a few common issues that
clinicians face when dealing with the diagnosis and management

1. What clinical signs and symptoms are most reliable to rule


in or out appendicitis?
The history and physical examination remains the most reliable
predictor for the diagnosis of appendicitis.4 By performing a thorough history and physical examination, an experienced clinician can accurately diagnose acute appendicitis in the majority
of cases.5 The presentation of acute appendicitis can vary widely.
A typical patient will present with vague abdominal pain (usually epigastric region) followed by anorexia, nausea, with or
without vomiting. The pain then shifts to the RLQ as the inflammation of the appendix progresses to involve the overlying peritoneum. Common symptoms of appendicitis include periumbilical
abdominal pain and anorexia in nearly 100%, of cases, nausea in
90%, and migration of pain from the periumbilical area to the
RLQ around 50% of the time.6 Lee et al.7 reported that the most
reliable symptom in making the diagnosis of appendicitis is the
classic pattern of migratory abdominal pain from periumbilical to
the RLQ. Occasionally, patients will complain of dysuria, hematuria, urgency and frequent urination, and diarrhea or constipation
from inflammation adjacent to the ureter, bladder, colon, and rectum. As all clinicians have found, these clinical features are not
entirely reliable. However, the history and physical examination
continues to remain the most reliable indicator for appendicitis.
Most patients with appendicitis except the very young, very old,
and those who are neurologically impaired will have some degree
of tenderness on palpation of the abdomen. In more than 95% of
patients with acute appendicitis, the sequence of symptoms was
anorexia, followed by abdominal pain, and then vomiting.8 In
1996, a meta-analysis performed by Wagner et al.9 reported the
sensitivity, specificity, and positive likelihood ratio with a 95%
confidence interval (CI) for fi ndings on the clinical examination
275

PMPH_CH33.indd 275

5/22/2012 5:24:34 PM

276

Surgery: Evidence-Based Practice

characteristic of appendicitis. They reported the sensitivity, specificity, and positive likelihood ratio for RLQ pain (0.81, 0.53, 7.31
8.46); fever (0.67, 0.79, 1.94); and anorexia (0.68, 0.36, 1.27).
Physical examination findings are determined by the anatomic position of the inflamed appendix and whether it has
ruptured. A retrocecal appendix can give rise to tenderness in
the right flank or in the right upper quadrant, whereas a pelvic
appendix can give rise to little abdominal tenderness but pain on
rectal examination. A patient who presents with uncomplicated
appendicitis may present with a slight elevation in temperature
(by 1C or 1.8F) and a slight elevation in heart rate, otherwise
vital signs are normal. Patients with peritonitis will prefer to lie
still, as any motion will tend to worsen their pain. If the appendix
lies in the classic anterior position, abdominal pain will be maximal at McBurneys point, with rebound tenderness elucidated in
the RLQ.10 Palpation of the left lower quadrant (LLQ) may cause
RLQ pain, also known as Rovsings sign.
Deviations from these commonly associated physical findings
usually are related to the anatomic position of the inflamed appendix. The common anatomic locations of the appendix include
paracolic (the appendix lies in the right paracolic gutter lateral to
the cecum), retrocecal (the appendix lies posterior to the cecum
and may be partially or totally extraperitoneal), preileal (the
appendix is anterior to the terminal ileum), postileal (the appendix is posterior to the ileum), promontoric ( the tip of the appendix
lies in the vicinity of the sacral promontory), pelvic (the tip of the
appendix lies in or toward the pelvis), and subcecal (the appendix
lies inferior to the cecum).11 Wakeley12 performed a postmortem
analysis of 10,000 cases and described the frequency of the location of the appendix as follows: retrocecal, 65.3%; pelvic, 31%; subcecal, 2.3%; preileal, 1%; and right paracolic and postileal, 0.4%.
When the appendix occupies an unusual location, the diagnosis
of appendicitis can be more difficult and may contribute to delays
in presentation, diagnosis, and treatment.
Answer: Abdominal pain often localized to the epigastrium
or periumbilical associated with anorexia and nausea is the most
reliable diagnostic finding on history and physical examination.
(Grade B recommendations)
2. What is the best laboratory test to help make the diagnosis
of appendicitis?
The use of laboratory values in diagnosing appendicitis has been
disappointing as no one test has been found to be highly sensitive and specific. The white blood cell count (WBC) was found
to be of limited value for making the diagnoses of appendicitis
in one study.13 On the other hand, Anderson et al. concluded
that a leukocytosis was actually more diagnostic of advanced
or complicated appendicitis than noncomplicated acute cases.14
The sensitivity of an elevated WBC above 10,000 cells/L for acute
appendicitis is 7090%, but the specificity is very low.15 A value
greater than 18,000 cells/L suggests complicated appendicitis with either gangrene or perforation. The diagnostic value of
C-reactive protein (CRP) and erythrocyte sedimentation rate in
diagnosing appendicitis has been both controversial and disappointing.16 A recent paper by Yang et al.17 found that the use of
WBC and CRP individually or together had a high sensitivity to
differentiate patients with appendicitis but a very low specificity.
In another study in adults, the finding of a normal WBC count
and a normal CRP level was highly predictive of no appendicitis.18 Other studies have shown that inflammatory cytokines and

PMPH_CH33.indd 276

acute-phase reaction proteins such as interleukin-6 (IL-6), tumor


necrosis factor (TNF-alpha), lipopolysaccharide-binding protein,
alpha1-glycoprotein (alpha1GP), and endotoxin are also elevated
in this patient population. The result of many of these studies
is that these inflammatory markers are elevated in appendicitis
(high sensitivity) but that many are not specific enough to reliably
diagnose the disease.19-21 In a study, Lycopoulou et al.22 reported
the sensitivity and specificity of the use of WBC >10, (75% and
76%), CRP >10 mg/L, (62% and 94%), and serum amyloid protein
(SSA) >45 mg/L, (86% and 83%), in diagnosing acute appendicitis
in children. Procalcitonin was found to be increased in rare cases
of severe inflammation after appendiceal perforation and gangrenous appendicitis but because of its low sensitivity, it cannot be
recommended for the diagnosis of acute appendicitis.23
Answer: Overall, laboratory markers of acute inflammation in acute appendicitis remain highly sensitive but relatively
nonspecific when it comes to making the diagnosis of acute
appendicitis. No one test has been found to be both highly sensitive and specific for reliable diagnosis of appendicitis. (Grade B
recommendations)
3. Does giving a patient with suspected appendicitis pain medicine decrease the ability to make the diagnosis of appendicitis?
It has been taught that patients with abdominal pain should not
receive narcotics for fear of masking a surgical condition, such as
appendicitis in patients who present with acute abdominal pain.
In a retrospective study by Aydelotte et al., 24 charts were reviewed
for 75 patients diagnosed with acute appendicitis and were confirmed intra-operatively. A total of 10 men and 14 women received
narcotics prior to surgical evaluation, and 28 men and 14 women
were not given narcotics prior to surgical evaluation. In this study,
there was no statistically significant difference between the two
groups of patients in regard to the length of hospital stay, the
time to operation, complication rate, perforation rate, or negative
appendectomy rate. The authors concluded that the administration of narcotics before evaluation of the patient by a surgeon
for acute appendicitis had no effect on patient outcomes. Attard
et al. assessed the safety of early pain relief in patients with acute
abdominal pain and found that the use of analgesia did not affect
the diagnostic workup of these patients.25 A matched casecontrol
study performed in an emergency room (ER) with patients presenting with suspected appendicitis did the early use of analgesia
delay treatment of these patients acute appendicitis.26 The authors
found that there was no delay in treatment with opiates but there
appears to be a delay in treatment with the use of nonsteroidal
anti-inflammatory analgesia. A prospective randomized doubleblind study of parenteral tramadol analgesic use versus placebo in
68 emergency department (ED) patients with RLQ pain resulted
in significant levels of pain control without concurrent normalization of abdominal pain.27 In another prospective, doubleblind study in ED patients with undifferentiated abdominal pain,
patients were randomized to receive placebo or morphine sulfate
(MS).28 Diagnostic accuracy, however, did not differ between MS
and control groups (64.2% vs. 66.7%). These results support the
practice of early provision of analgesia to patients with undifferentiated abdominal pain. A prospective double-blind crossover
study by Wolfe et al.29 also evaluated administration of morphine
to the suspected appendicitis patient and its impact on their physical examination. The authors concluded that patients with signs of
appendicitis who received morphine had significant improvement

5/22/2012 5:24:34 PM

Appendicitis

in their pain but were without changes in their physical examination. In the pediatric population, the findings were consistent as
well. In another prospective randomized study performed in children with a presumptive diagnosis of appendicitis, the children
were given either parenteral MS or placebo.30 The authors found
no difference in the time to surgical decision and no decrease in
pain after 30 min between morphine at a dose of 0.1 mg/kg and
placebo. A separate randomized study in children with acute
abdominal pain concluded that morphine was found to effectively
reduce the intensity of pain and did not seem to impede the diagnosis of appendicitis.31
Answer: Giving pain medicine to adults and children suspected of acute appendicitis does not adversely affect the ability to
diagnosis appendicitis. Analgesia should not be withheld pending
clinical investigation in patients with suspected acute appendicitis.
(Grade B recommendation)
4. What is the best diagnostic imaging modality to diagnose
acute appendicitis?
Many different radiologic modalities have been used to diagnose acute appendicitis. The optimal radiologic technique used
to diagnose acute appendicitis should be accurate, quick, safe,
readily available, cost efficient, and should provide little risk or
discomfort to the patient. The use of abdominal ultrasound (US)
and computed tomography (CT) has proven extremely useful in
diagnosing this disease. However, routine use of these modalities
in all patients with suspected appendicitis is not well established.32
Despite the recent increase in their use, these tests have not consistently increased the diagnostic accuracy of making the diagnosis
of acute appendicitis in all patient populations.
The use of plain radiography for diagnosing acute gastrointestinal diseases has been around since the early 1900s. The appearance of an opaque fecalith in the RLQ is often quoted as being the
hallmark radiographic finding in acute appendicitis, but less than
5% to 8% of patients present with this finding.33 Other common
but nonspecific findings on plain fi lms include localized paralytic
ileus, loss of the cecal shadow, blurring of the right psoas muscle,
and rightward scoliosis of the lumbar spine.34 In a recent study
of 821 consecutive patients hospitalized for suspected appendicitis, no individual radiographic finding was highly sensitive
or specific in ultimately making the diagnosis of appendicitis.35
Plain abdominal radiographs may be indicated when other acute
abdominal conditions such as gastric or duodenal perforation,
intestinal obstruction, or ureteral calculus are part of the differential as the cause of the RLQ abdominal pain.36 Overall, plain
abdominal radiographs are not cost effective and lack both sensitivity and specificity in the diagnosis of appendicitis.
Deutsch and Leopold37 first visualized the inflamed appendix
using US in 1981. US has become a more frequently used radiologic test to rule out appendicitis in children and pregnant women
because of concerns toward exposure to ionizing radiation from
CT scans. Its accuracy in diagnosing appendicitis has been hampered by the interference of the US image by overlying bowel gas, the
slow development of a transducer with enough spatial resolution
to pick up small structures such as the appendix as well as the
highly variable operator-dependent interpretation and technical expertise at individual hospitals.38 With the advancement in
US technology and the use of the graded compression technique
when scanning the RLQ, the ability to visualize the appendix has
improved. The graded compression technique involves applying

PMPH_CH33.indd 277

277

steady, gradual pressure to the RLQ in an effort to collapse the


normal bowel and eliminate bowel gas in the area in order to visualize the appendix. The inflamed appendix when seen by US commonly includes the following findings: an appendix of 7 mm or
more in an anteroposterior diameter, an immobile, thick-walled,
noncompressible luminal structure seen in cross-section referred
to as a target lesion, or the presence of an appendicolith, a blindending structure consisting of anechoic lumen surrounded by
mucosa, and a hypoechoic thickened wall adjacent to the cecum.39
Despite these well-described US findings, there is no evidencebased standard of findings by which an individual radiologist can
use to make the diagnosis of appendicitis. In a prospective study
by Rettenbacher et al.40 using US to diagnose appendicitis, the
authors used six different US findings. Unfortunately, they did not
state the number of these findings necessary to make their diagnosis of appendicitis. In a recent systemic review of the use of US to
diagnose acute appendicitis in adults and adolescent patients, the
accuracy of using graded compression US was reported to have an
overall sensitivity of 0.86 (CI, 0.830.88) and a specificity of 0.81
(CI, 0.780.84) with a positive likelihood ratio 5.8 (CI, 3.59.5).41
US is most reliable in centers with considerable experience
using this modality to diagnose acute appendicitis. In a metaanalysis looking at the sensitivity and specificity of US in diagnosing appendicitis in adults and children, the results are as
follows: for U/S in children 88% sensitivity and 94% specificity,
and for U/S in adults 83% sensitivity and 93% for specificity.42 In
another study reporting on the sensitivity and specificity of U/S
in making the diagnosis of appendicitis in adults and children,
it was reported an overall 83% sensitivity and 98% sensitivity.43
The sensitivity (range 66100%) and specificity (range 9596%)
of US in diagnosing acute appendicitis in pregnant patients are
reported.44 However, it is recommended that if an U/S is negative
or inconclusive in a pregnant patient with a suspected diagnosis
of appendicitis, another imaging study such as CT or magnetic
resonance imaging (MRI) should be performed.45
CT has been used as a diagnostic modality for acute abdominal pain since it became available in the late 1970s. Helical CT
scans have excellent resolution, are widely available, are operator independent, and are easy to interpret making them often
the preferred diagnostic test to rule out appendicitis. Findings strongly suggestive of acute appendicitis on a standard
abdominal CT scan include (1) a thick wall (>2 mm), often with
targeting (concentric thickening of the inflamed appendix wall);
(2) increased diameter of the appendix (>7 mm); (3) an appendicolith; (4) a phlegmon or abscess; or (5) free fluid.46 Stranding
of the adjacent fatty tissues in the RLQ is also commonly associated. The top four CT findings suggestive of appendicitis are
an enlarged appendix, appendiceal wall thickening, appendiceal
wall enhancement, and periappendiceal fat stranding.47,48 If air
is seen in the appendix or if the appendiceal lumen is fi lled with
contrast and there are no other abnormalities seen on a CT,
these fi ndings virtually eliminate appendicitis as the diagnosis.
It has been thought that appendicitis could not be excluded if
the appendix was not visualized on a CT scan. A more recent
report, however, concluded that nonvisualization of the appendix on a CT scan was negative for appendicitis in 98% of cases.49
CT is also useful in diagnosing an appendiceal abscess and can
be used to guide percutaneous drainage. CT can also be helpful
in diagnosing other causes of acute abdominal pain in patients
suspected of acute appendicitis.

5/22/2012 5:24:34 PM

278

Surgery: Evidence-Based Practice

The performance of CT scans to evaluate the RLQ pain has


increased considerably since Rao et al.50 reported an accuracy rate
of 98% with administration of rectal contrast in diagnosing acute
appendicitis. Rao also reported that the use of CT at his institution decreased the rate of removal of a normal appendix from
20% before the introduction of CT scanning to 7% after.51 Other
authors have not found CT to be as accurate. Perez et al.52 found
the accuracy of CT in diagnosing appendicitis to be 80%. Morris
et al.53 reported a diagnostic accuracy of 90% at their institution.
In a study performed by Holloway et al.,54 using a well-defined CT
imaging protocol as an adjunct to the clinical diagnosis of acute
appendicitis, they found the accuracy of CT to be 97.8% with a negative appendectomy rate of 3%. These same authors also reported
on 104 patients who underwent appendectomy without their CT
protocol who had a negative appendectomy rate of 12.5%. In a retrospective review of CT use in the pediatric population suspected
of appendicitis, a normal appendix was removed in 7% of children who underwent CT prior to appendectomy, 11% with the use
of US prior to appendectomy, and an 8% negative appendectomy
rate when no preoperative radiologic study was performed.55 In a
recent retrospective study, the use of preoperative CT scans only
decreased the negative appendectomy rate for women of childbearing age (women 45 years and younger).56 Livingston et al.57
found that the rate of nonperforated appendicitis increased since
1995 with the liberal use of CT scans but the rate of perforated
appendicitis also increased during the same period. Krajewski
et al.58 showed that the incidence of perforated appendicitis was
higher for lower socioeconomic patients than that of the patients
in the higher socioeconomic group in the United States from 2001
to 2005 but those same authors found no difference in perforated
appendicitis rates between the lower and higher socioeconomic
patients in Canada.
Although many studies have found CT to be accurate in
diagnosing acute appendicitis, there is still controversy regarding
the optimal technique. Three common techniques used include a
focused appendiceal CT using rectally administered contrast, the
unenhanced or the use of oral and/or intravenous contrasted CT
of the abdomen and pelvis. Every institution has their own preference to which version they prefer to use to diagnose appendicitis,
all of which seem to have the same reported accuracy.59,60
In a systemic review performed by van Redan et al.61 comparing graded compression US with CT in the diagnosis of appendicitis, the authors found that the respective mean sensitivities for
CT and graded compression US were 91% (95% CI, 84%, 95%) and
78% (95% CI, 67%, 86%) (p < 0.017) and the respective mean specificities for CT and graded compression US were 90% (95% CI, 85%,
94%) and 83% (95% CI, 76%, 88%) (p < 0.037). Calculated positive OR or Odds Ratio for CT and graded compression US were
9.29 (95% CI, 6.86, 12.58) and 4.5 (95% CI, 3.03, 6.68), respectively
(p = 0.011) The authors from their meta-analysis of head-to-head
comparison studies in patient populations with a high prevalence
of appendicitis concluded that CT was found to have a better test
performance than the graded compression US in making the
diagnosis. The authors recommend the use of CT in patients suspected of acute appendicitis.
Should CT be used routinely in the diagnostic evaluation
of patients suspected of appendicitis? Because of the increasing
reports of excellent accuracy rates of CT diagnosing appendicitis,
some have called for the routine use of CT for all patients with
possible appendicitis.52,62-64 Others have questioned the need for

PMPH_CH33.indd 278

the routine use of CT for all patients especially those with classic clinical presentations. McCay and Shepherd65 recommended
only ordering CT on patients presenting to the ER suspected of
having appendicitis if their Alvarado score66 is between 4 and 6.
For a score of less than 3, no CT or U/S was recommended as
appendicitis was doubtful. The authors do, however, recommend
a surgical consult for an Alvarado score of 7 or more. In a prospective randomized study of patients presenting to the ER for
possible appendicitis comparing clinical assessment versus CT,
the reported diagnostic accuracy was 90% for clinical assessment
and 92% for CT.67 The authors concluded that clinical assessment
unaided by CT reliably identifies patients with acute appendicitis who need an operation. They do not advocate the routine use
of CT for diagnosis of suspected appendicitis. In our prospective
randomized study performed in women of childbearing age who
presented to the ER with the suspected diagnosis of appendicitis,
patients were randomized to the clinical assessment only arm or
the CT arm.68 In this study, the reported accuracy for the diagnosis of appendicitis was 93% for both clinical assessment and CT.
The authors concluded that a CT scan is as good as clinical assessment alone and reliably identifies women of childbearing age
who need an appendectomy. In a recent retrospective study, the
negative appendectomy rate for patients who had a CT scan prior
to appendectomy was 6%. The negative appendectomy rate was
unchanged for patients who underwent an appendectomy based
on clinical examination alone.69 The study also found that preoperative CT scans increased the appendectomy rate only in patients
with a low clinical suspicion of appendicitis. In a retrospective
study in children reported by Martin et al.,70 the liberal use of CT
scans did not decrease the negative appendectomy rate. In conclusion, the selective use of CT scans seems more appropriate in
diagnosing suspected appendicitis. This study should be reserved
as an adjunct in clinical settings in which other sources of pathology other than appendicitis may cause pain or the clinical history
alone is not helpful in making the diagnosis.
MRI for the evaluation of acute appendicitis has been performed more frequently recently in order to avoid the risks associated with ionizing radiation. MRI has become a frequently
performed test in pregnant women and children with symptoms
of appendicitis and a nondiagnostic US.71 MRI has good resolution and has been shown to be accurate in diagnosing acute
appendicitis.72 MRI is considered positive for acute appendicitis
when the appendix is enlarged (>7 mm); the appendiceal wall
is thicker than 2 mm, or there are signs of inflammatory changes
surrounding the appendix, such as fat stranding, phlegmon, or
abscess formation.73 MRI has been shown to be safe and reliable
in diagnosing acute appendicitis in pregnant patients.74,75 No IV
contrast should be given to pregnant patients because gadolinium
is a category C drug and potentially teratogenic.
In a recent multicenter diagnostic study of MRI in patients
with suspected appendicitis, the authors suggest that if MRI
is found to be sufficiently accurate in the general population of
patients with suspected appendicitis, MRI could replace CT in
some or all patients. This could limit or obviate the ionizing radiation exposure and decrease the risk of contrast medium-induced
nephropathy with CT.76 Limitations to the use of MRI are that it is
a more expensive test, MRI is not always widely available, images
can be degraded by motion, and a specialist needs to interpret the
MRI images. Until these limitations can be overcome, MRI should
not be a first-line test to rule out appendicitis.

5/22/2012 5:24:34 PM

Appendicitis

Answer: The most accurate imaging modality for making the


diagnosis of appendicitis is CT. The routine use of performing CT
on all patients suspected of appendicitis cannot be recommended
at this time. (Grade B recommendation)
5. Does giving antibiotics to patients with appendicitis who
undergo appendectomy decrease postoperative infectious
complications?
Appendicitis once diagnosed is usually followed by an appendectomy. Antibiotics should be given as soon as the diagnosis is
suspected. The bacteria that populate the appendix are similar to
the bacterial flora of the colon. The antibiotics chosen for patients
with appendicitis should provide coverage for gram-negative and
gram-positive aerobic and anaerobic bacteria, along with anaerobes. Bacteriodes fragilis and Escherichia coli are the two most
common organisms grown from peritoneal cultures after acute
appendicitis.
Acute appendicitis is a polymicrobial infection. In 1938,
William Altemeier isolated at least four different organisms per
specimen in patients with perforated appendicitis.77 More recent
reports demonstrate on average up to 12 organisms per specimen
from patients with gangrenous or perforated appendicitis.78 Few
bacteria are cultured from the peritoneal fluid of patients with
acute appendicitis only; however, bacteria are recovered from
peritoneal fluid in over 80% patients with a gangrenous or perforated appendix. Two common postoperative complications following appendectomy are wound infections and intra-abdominal
abscesses. Prior to the use of antibiotics, there was a 10% to 40%
rate of wound infections and intra-abdominal abscesses after
appendectomy.79,80
All patients undergoing appendectomy for acute appendicitis
should receive antibiotics preoperatively.81 The use of antibiotics to
reduce postoperative morbidity following appendectomy has been
studied. Gorbach in his review of antimicrobial prophylaxis for
appendectomy reported a reduction in the rate of postoperative
infectious complication in all operations for acute appendicitis and
especially in patients with perforated and/or gangrenous appendicitis.82 In another analysis of clinical studies by Pottecher et al.,83
they reported that a single preoperative dose of systemic antibiotic
reduced the postoperative sepsis rates after appendectomy. They
also reported that if the appendix was perforated, then antibiotic
therapy should last longer than one dose and should not be considered prophylaxis but treatment. In a study by Mui et al., a single
dose of preoperative antibiotics was found to be adequate for the
prevention of postoperative infective complications in patients
with nonperforated appendicitis.84 Another study also supported
the fact that only one preoperative dose of antibiotic is needed to
prevent postoperative infectious complications in patients with
nonperforated appendicitis and the use of any further postoperative antibiotics does not decrease the rate of surgical site infections
(SSI).85 Patients with perforated and complicated appendicitis can
be treated with antibiotics for 57 days.86
Anderson et al. performed a meta-analysis of randomized or
controlled clinical trails investigating the use of antibiotics versus placebo for patients with suspected appendicitis who underwent an appendectomy.87 The authors evaluated 45 studies with
9576 patients. Their outcome measures were wound infection,
intra-abdominal abscess, hospital length of stay, and mortality.
They concluded that the use of antibiotics is superior to placebo

PMPH_CH33.indd 279

279

in preventing wound infection and intra-abdominal abscesses


in patients with acute, gangrenous, and perforated appendicitis.
They were unable to determine the optimal duration of antibiotic
treatment for complicated cases from their analysis. The authors
found that a single dose of antibiotics may have the same impact
as multiple doses, although it is best to administer the first dose of
antibiotics preoperatively. The choice of antibiotic selection should
be based on the bacteriology of appendix and provide a coverage
for gram-negative, gram-positive, and anaerobic organisms.
A recent prospective study by Fraser et al. of pediatric patients
with perforated appendicitis examined early transition to oral
antibiotics in comparison with a traditional 5-day intravenous
antibiotic course. Patients were transitioned to oral antibiotics as soon as they tolerated a diet and discharged, to complete
a total 7-day regimen of IV/PO antibiotics. This group had no
increased morbidity compared with the group receiving 5 days of
IV antibiotics.88
Answer: Antibiotic prophylaxis is effective in preventing
postoperative wound infections and intra-abdominal abscesses.
For nonperforated appendicitis, the one-time preoperative dose of
antibiotic seems to be sufficient to decrease infectious complications. The optimal duration of administration of antibiotic needs
to be further evaluated in patients with complicated appendicitis.
(Grade B recommendation)
6. What operation is better for treating acute appendicitis:
laparoscopic (LA) or open appendectomy (OA)?
The treatment for acute appendicitis has been to perform an
appendectomy through a RLQ incision since its introduction by
McBurney89 in 1894. The first LA was performed by Semm90 in
1983. This new surgical technique was slow to be accepted because
the standard open technique provided excellent therapeutic efficacy combined with its low morbidity and mortality rates. The
use of LA varies considerably. It seems that the most important
determinate of whether a patient will have an open or an LA is
the preference or the experience of the treating surgeon, which
may vary significantly even within an institution.91 During the
traditional OA technique performed through a muscle splitting incision in the RLQ, the appendix is usually ligated with an
absorbable suture. Inversion of the appendiceal stump has been
advocated to prevent leakage and fistulization, but studies have
shown no difference in complication rates between inversion and
simple ligation of the appendiceal stump.92 The peritoneal cavity
is typically irrigated after an appendectomy. The skin incision is
normally closed without complications, although if the wound is
grossly contaminated, one may consider delayed primary closure
or simply allow the wound to heal by secondary intention.93 Leaving an intraperitoneal drain has not been shown to be useful even
in cases of a perforated appendix.94
Is LA better than OA? The answer to this question depends
on the outcomes being measured. Over the last 20 years, various
studies have looked at the duration of operation, the cost of operation, the cost of hospitalization, the length of hospital stay, the
time to return to work, and postoperative pain often with conflicting results.95-98 Although many of the randomized control trails
(RCTs) comparing LA and OA are plagued by several biases, they
represent the best evidence available.
Two early meta-analyses of LA versus OA for acute appendicitis have confirmed the benefit of the laparoscopic approach

5/22/2012 5:24:34 PM

280

Surgery: Evidence-Based Practice

in relation to less pain, a faster recovery, and a lower incidence


of wound infections compared with OA.99,100 As surgeons become
more skilled in minimally invasive surgical techniques, the incidence of LA has become more common.101 Reported complications
after LA include injury to bowel, bladder, and ureter, bleeding
from epigastric vessels, iliac vessels, and mesentery, appendiceal
stump leak, wound infection, and intra-abdominal abscess. One
of the reported complications of LA is leaving a long stump and
risking recurrent symptoms.102
The Cochrane Library published a systemic review of randomized clinical trails comparing open with LA in 2010.103 This
review included randomized clinical trials comparing LA versus
OA in adults and children. The authors included 67 studies, of
which the majority of (56) studies compared laparoscopic (with
or without diagnostic laparoscopy) versus OA in adults. The
authors reported that wound infections were less likely after LA
than after OA (OR 0.43; CI, 0.340.54), but the incidence of intraabdominal abscess was increased after LA (OR 1.87; CI, 1.192.93).
The duration of LA was 10 min (CI, 615) longer to perform than
of OA. Pain on post-op day 1 was reduced by 8 mm (CI, 511 mm)
on a 100-mm visual analog scale after LA compared with OA. Hospital stay was shortened by 1.1 day (CI, 0.71.5) after LA. Return to
normal activity, work, and sports occurred earlier after LA than
after OA. Although the operation costs of LA were significantly
higher, the costs outside the hospital were reduced. Diagnostic
laparoscopy reduced the risk of a negative appendectomy, but
this effect was stronger in fertile women (RR 0.2; CI, 0.110.34)
as compared with unselected adults (RR 0.37; CI, 0.131.01). The
authors concluded that in clinical settings where surgical expertise and equipment are available and affordable, LA seems to hold
various advantages over OA. They recommend that LA be done for
patients with suspected appendicitis especially in young patients,
female patients, obese patients, and employed patients.
Answer: The data supports the use of performing LA for
patients with acute appendicitis if the surgical expertise and
equipment are available. (Grade B recommendation)
7. Is interval appendectomy necessary?
Patients presenting with a periappendiceal mass or abscess diagnosed preoperatively by physical examination or imaging studies
can be treated with antibiotics with the potential of having their
periappendiceal abscess drained by image-guided percutaneous
catheter.104 With the increased use of CT in the workup of acute
appendicitis, the ability to identify complicated appendicitis preoperatively has allowed for the utilization of initial nonoperative
therapy.105 Generally, antibiotics for 7 to 14 days with or without
catheter drainage have been necessary to treat those patients. An
interval appendectomy has been advocated after the abscess and
surrounding inflammation have resolved, usually 6 to 8 weeks
after initial nonoperative treatment to prevent recurrent appendicitis and to treat other tumor pathology of the cecum and appendix.106 Alternative treatment options of complicated appendicitis
have included early aggressive resection,107 or initial conservative
treatment with interval appendectomy only if symptoms recur.108,109
Immediate appendectomy may be technically demanding because
of the distorted anatomy and the challenges faced when closing an
inflamed/necrotic appendiceal stump. Many times, the immediate exploration ends up with an ileocecal resection or a right-sided
hemicolectomy due to inflammation distorting the tissue planes or

PMPH_CH33.indd 280

a suspicion of malignancy. Following successful nonsurgical treatment of a periappendiceal mass, the need for interval appentectomy
(IA) has recently been questioned as the risk of recurrence is relatively small (0.27%).8,110,111
In two other recent retrospective studies, it was found that children presenting with complicated appendicitis could be successfully
treated with conservative treatment followed by appendectomy.112,113
Roach et al.114 concluded from their data that children who presented
with prolonged symptoms and a discrete appendiceal abscess or
phlegmon, drainage, and performance of a delayed appendectomy
should be the treatment of choice. In another study, children with
complicated appendicitis were initially treated nonoperatively and
then had a laparoscopic interval appendectomy; the conclusion
was that the surgery could be safely performed, and was associated
with a shorter hospital stay, with minimal morbidity, analgesia, and
scarring. These authors recommended that interval LA be routinely
performed because it eliminates the risk of recurrent appendicitis
and serves to excise undiagnosed carcinoid tumors.115 Another
group compared initial LA with initial nonoperative management
and interval appendectomy for complicated appendicitis in children in a randomized prospective study.116 These authors found that
the initial laparoscopic surgery took longer but that the overall days
in the hospital, infection rates, and total costs did not differ between
the two treatment strategies.
In a large retrospective study performed by Kaminski et al.,117
32,938 patients were hospitalized with acute appendicitis. Emergency appendectomy was performed in 31,926 (97%) patients.
Nonoperative treatment was used initially in 1012 patients (3%). Of
these, 148 (15%) had an IA and the remaining 864 (85%) did not. In
their study, only 39 patients (5%) had a recurrence of appendicitis
after a median follow-up of 4 years. Males were more likely to have
a recurrence of their symptoms than females. Median length of
hospital stay was 4 days for the admission for recurrent appendicitis
compared with 6 days for the IA admission. The authors concluded
that they cannot justify the practice of routine interval append
ectomy after initial successful nonoperative treatment of appendicitis based on the observation that most patients undergo appendectomy initially, and those who are treated nonoperatively have
a low recurrence rate of appendicitis. In a similar retrospective
study in children reported by Paupong et al.,118 there were 6439
patients, of which 6367 (99%) underwent initial appendectomy for
acute appendicitis. Seventy-two (1%) patients were initially managed nonoperatively and 11 patients had IA. Of the remaining 61
patients without IA, 5 (8%) developed recurrent appendicitis. The
authors concluded that because recurrent appendicitis is rare in
children after successful nonoperative treatment of perforated
appendicitis, performance of routine interval appendectomy is
not necessarily indicated.
Adult patients who present with an appendiceal mass in the
RLQ are commonly managed nonoperatively and then scheduled
for an interval appendectomy following resolution of the inflammatory appendiceal mass. This mass could represent a perforated
appendix, complicated Crohns disease, or a perforated colon cancer. Tekin et al.119 reported their experience with not performing
routine interval appendectomy after successful treatment of an
appendiceal mass. Four patients (4%) in their series had another
diagnosis found for their appendiceal mass (two cecal cancers, one
cecal diverticulitis, and one Crohns disease). The recurrence rate
of appendicitis in their series was 14.6% with most recurrences
happening in the first 6 months after initial presentation. Patients

5/22/2012 5:24:34 PM

Appendicitis

who present with recurrent symptoms should undergo interval


appendectomy. They concluded that routine interval appendectomy after initial successful conservative treatment is not justified but they recommend that a protocol should be developed for
the management of patients presenting with an appendiceal mass.
Similar recommendations were reported by Lai et al.120 In their
study, five patients were found to have colon cancer, and the rate
of recurrent appendicitis was 25.5% with 83% of patients present
with recurrent symptoms within 6 months of their initial presentation. They recommend that adult patients who recover from
conservative treatment of an appendiceal mass should undergo
colonoscopy to detect any underlying disease and interval appendectomy should only be offered to patients who present with
recurrent symptoms.
Stevens and de Vries121 reported on their experience of performing an interval appendectomy only after symptoms developed
rather than routinely offering it to their patients with complicated
appendicitis. They concluded that the rate of appendectomies
performed dropped by 63% and the total length of hospital stay
also decreased by 4 days. A group from China reported that by
performing an interval appendectomy only after symptoms developed was more cost effective than performing routine interval
appendectomy.122 In their study, the authors showed that performance of routine interval appendectomy would increase the cost
per patient by 38% compared with follow-up and appendectomy
after recurrence of symptoms.
In a systemic review of the nonsurgical treatment of appendiceal abscess or phlegmon, the need for an interval appendectomy
was evaluated.123 Findings from the meta-analysis: nonsurgical
treatment fails in 7.2% of cases (CI, 4.010.5), the risk of recurrent
symptoms is 7.4% (CI, 3.711.1), the risk of finding malignant disease
is 1.2% (CI, 0.61.7), and the risk of finding an important benign
disease is 0.7% (CI, 0.211.9) during follow-up. From their metaanalysis (mainly from retrospective studies), the authors support
the practice of nonsurgical treatment without interval appendectomy in patients with appendiceal abscess or phlegmon. Another
recent systemic review has confirmed that nonoperative management of complicated appendicitis will be successful in the majority
of cases with a low incidence of recurrent symptoms. As a result, the
routine use of interval appendectomy is no longer justified.124
Answer: The routine performance of interval appendectomy
after nonoperative treatment of complicated acute appendicitis
is not supported. Interval appendectomy should be performed
when patients present with recurrent symptoms. Patients presenting with an appendiceal mass managed conservatively should
undergo further workup to rule out other pathologies for their
mass. (Grade B recommendation)
8. Should antibiotic treatment replace appendectomy for acute
appendicitis?
Nonoperative treatment of acute appendicitis with antibiotics
alone has been reported to be successful.125,126 Andersson writes that
an increasing amount of circumstantial evidence suggests that
not all patients with appendicitis will progress to perforation and
that resolution may be a common event.127,128 Other evidences
of resolving appendicitis are reports of a history of recurrence,
obviously a consequence of spontaneous resolution, which can be
found in up to 6.5% of patients not operated on for appendicitis.129 In the past, appendectomy has been associated with higher

PMPH_CH33.indd 281

281

morbidity and mortality especially in older patients, those with


perforation and sepsis, and those who have a normal appendix
at the time of appendectomy. To date, unfavorably high rates of
recurrent symptoms of appendicitis (up to 70% at 1 year) have
been reported in patients who have received antibiotic treatment
alone for their acute appendicitis.130,131
In 1995, Eriksson and Granstrom132 reported an RCT of
appendectomy versus antibiotics alone in 40 patients suspected
to have appendicitis, who presented with abdominal pain for less
than 72 h. Twenty patients underwent surgery and 20 patients
received intravenous antibiotics for 2 days, followed by an 8-day
course of oral antibiotics. The authors concluded that antibiotic
treatment in patients with acute appendicitis was as effective as
surgery. However, they reported a 15% negative appendectomy
rate for the surgery group and a 40% recurrence rate of appendicitis that led to appendectomy within 1 year of treatment in
the nonoperative group. A recent multicenter trial randomly
allocated 252 male patients (age 1850) to either antibiotic treatment (intravenous cefotaxime and tinidazole for 2 days followed
by oral ofloxacin and tinidazole for 10 days) or appendectomy
for acute, uncomplicated appendicitis. The trial concluded that
antibiotic treatment could serve as an alternative to appendectomy.133 The complication rate among the surgery group was 14%
(17/124), mainly wound infections. Of the 128 patients enrolled
in the antibiotic group, 15 patients (12%) were operated on within
the first 24 h due to the lack of improvement in symptoms and
apparent local peritonitis. The operation showed that seven of
these patients (5%) had a perforation of their appendix. The rate
of recurrence of appendicitis in the antibiotic group was 14%. In
another meta-analysis of antibiotic therapy versus appendectomy
for acute appendicitis, the authors concluded that even though
antibiotics may be used as primary treatment for selected patients
with uncomplicated appendicitis, they do not feel that this treatment should supersede appendectomy at the present time.134 They
found that selection bias and crossover to surgery in the RCTs
suggest that appendectomy is still the gold standard therapy for
acute appendicitis.
A total of 113 patients were successfully treated with antibiotics
and were sent home for oral antibiotic therapy for 10 days. The recurrence rate within 1 year was 15% (16 patients) in the group treated
with antibiotics. Overall, the success rate of conservative management of acute appendicitis with antibiotics is ~70% at best for male
patients with unequivocal clinical and laboratory signs of uncomplicated appendicitis. In another recent randomized clinical trail
of antibiotic therapy versus appendectomy for acute appendicitis in
unselected patients, the authors concluded that antibiotic treatment
appears to be a safe first-line therapy in unselected patients with
appendicitis.135 In this study, only 52% of patients randomized to
antibiotics were followed up through. In both of these studies, the
conclusions have, however, been made on the basis of only 1 month
to 1 year of follow-up data. The continued lifetime risk for, and the
associated morbidity and mortality of, nonoperative treatment with
antibiotics only for acute appendicitis remains unknown and needs
to be investigated. The recommendation of antibiotic treatment as
an alternative to the surgical treatment of acute appendicitis cannot
be recommended at this time.136
Answer: Although antibiotics may be used as the primary
treatment for selected patients with uncomplicated appendicitis,
surgery continues to remain the primary treatment option for the
treatment of acute appendicitis. (Grade B recommendation)

5/22/2012 5:24:34 PM

282

Surgery: Evidence-Based Practice

Clinical Question Summary


Question

Answer

1 What clinical signs


and symptoms
are most reliable
to rule in or out
appendicitis?

Abdominal pain often localized to the


epigastrium or periumbilical region and
associated with anorexia and nausea is
the most reliable diagnostic findings on
history and physical examination.

4-12

2 What is the best


laboratory test
to help make
the diagnosis of
appendicitis?

Overall laboratory markers of acute


inflammation in acute appendicitis remain
highly sensitive but relatively nonspecific
when it comes to making the diagnosis
of acute appendicitis. No one test has
been found to be both highly sensitive and
specific for acute appendicitis.

2b

13-23

3 Does giving a patient


with suspected
appendicitis pain
medicine decrease
the ability to make
the diagnosis of
appendicitis?

Giving pain medicine to adults and children


suspected of acute appendicitis does not
adversely affect the ability to diagnose
appendicitis. Analgesia should not be
withheld pending clinical investigation
with suspected acute appendicitis.

3b

24-31

4 What is the best


diagnostic imaging
modality to diagnose
acute appendicitis?

The most accurate imaging modality for


making the diagnosis of appendicitis is CT.
The routine use of performing CT on all
patients suspected of appendicitis cannot
be supported at this time.

2b

32-74

5 Does giving antibiotics


to patients with
appendicitis
who undergo
appendectomy
decrease
postoperative
complication rates?

Antibiotic prophylaxis is effective in


preventing postoperative wound
infections and intra-abdominal abscesses.
For nonperforated appendicitis, the
one-time preoperative dose seems
to be sufficient to decrease infection
complications. The optimal duration of
administration of antibiotics needs to
be further evaluated in patients with
complicated appendicitis.

2b

75-86

6 What operation is
better for treating
acute appendicitis:
LA or OA?

The data support the use of performing LA


for patients with acute appendicitis if the
surgical expertise and equipment are
available.

2b

87-101

7 Is interval
appendectomy
necessary?

The routine performance of interval


appendectomy after nonoperative
treatment of complicated acute
appendicitis is not supported. Interval
appendectomy should be performed
when patients present with recurrent
symptoms. Patients presenting with an
appendiceal mass managed conservatively
should undergo further workup to rule
out other pathologies for their mass.

3b

102-122

8 Should antibiotic
treatment replace
appendectomy for
acute appendicitis?

Although antibiotics may be used as the


primary treatment for selected patients
with uncomplicated appendicitis,
surgery continues to remain the primary
treatment option for the treatment of
acute appendicitis.

3b

123-134

PMPH_CH33.indd 282

Level of
evidence

Grade of
Recommendation

References

5/22/2012 5:24:34 PM

Appendicitis

REFERENCES
1. Addiss DG, Schaffer N, Fowler BS, et al. The epidemiology of
appendicitis and appendectomy in the United States. Am J Epidemiol. 1990;132:910-925.
2. Fitz RH. Perforating inflammation of the vermiform appendix:
With special reference to its early diagnosis and treatment. Trans
Assoc Am Physicians. 1886;1:107.
3. Krajewski SA, Hameed SM, Smink DS, Rogers SO Jr. Access
to emergency operative care: A comparative study between
the Canadian and American health care systems. Surgery.
2009;146(2):300-307.
4. Bergeron E, Richer B, Gharib R, Giard A. Appendicitis a place for
clinical judgement. Am J Surg. 1999;177:460-462.
5. Silen W. Copes Early Diagnosis of the Acute Abdomen. 19th ed.
New York: Oxford University Press; 1996.
6. Shelton T, McKinlay R, Schwartz RW. Acute appendicitis: Current diagnosis and treatment. Curr Surg. 2003;60(5):502-505.
7. Lee SL, Ho HS. Acute Appendicitis: Is there a difference between
children and adults? Am Surg. 2006;72:409-413.
8. Jaffe BM, Berger DH. The Appendix. In Schwartzs Principles of
Surgery. 9th ed. New York: McGraw-Hill; 2010.
9. Wagner J, McKinney WP, Carpenter JL. Does this patient have
appendicitis? JAMA. 1996;276:1589-1594.
10. McBurney C. Experience with early operative interference in
cases of disease of the vermiform appendix. NY State Med J.
1889;50:676.
11. Prystowsky JP, Pugh CM, Nagle AP. Appendicitis. Curr Probl
Surg. 2005;42(10):685-742.
12. Wakeley CP. The position of the vermiform appendix as ascertained by an analysis of 10,000 cases. J Anat. 1933;67:277-283.
13. Vermeulen B, Morabia A, Unger PF. Influence of white blood
cell count on surgical decision making in patients with abdominal pain in the right lower quadrant. Eur J Surg. 1995;161:
483-486.
14. Andersson RE, Hugander AP, Ghazi SH, Ravn H, Offenbartl SK,
Nystrom PO, Olaison GP. Diagnostic value of disease history,
clinical presentation, and inflammatory parameters of appendicitis. World J Surg. 1999;23:133-140.
15. Hoff mann J, Rausmussen O. Aids in the diagnosis of acute
appendicitis. Br J Surg. 1989;76:774-777.
16. Jaye DL, Waites KB. Clinical applications of C-reactive protein
in pediatrics. J Pediatr Infect Dis J. 1997;16:735-746.
17. Yang HR, Wang YC, Chung PK, et al. Laboratory tests in patients
with acute appendicitis. ANZ J Surg. 2006;76(1-2):71-74.
18. Sengupta A, Bax G, Paterson-Brown S. White blood cell count
and C-reactive protein measurement in patients with possible
appendicitis. Ann R Coll Surg Engl. 2009;91(2):113-115.
19. Sack U, Biereder B, Elouahidi T, et al. Diagnostic value of blood
inflammatory markers for detection of acute appendicitis in
children. BMC Surg. 2006;6:15.
20. Paajanen H, Mansikka A, Laato M, et al. Novel serum inflammatory markers in acute appendicitis. Scand J Clin Lab Invest.
2002;62(8):579-584.
21. Yildirim O, Solak C, Kocer B, et al. The role of serum inflammatory markers in acute appendicitis and their success in preventing negative laparotomy. J Invest Surg. 2006;19(6):345-352.
22. Lycopoulou L, Mamoulakis C, Hantzi E, et al. Serum amyloid A
protein levels as a possible aid in the diagnosis of acute appendicitis in children. Clin Chem Lab Med. 2005;43(1):49-53.
23. Sand M, Trullen XV, Bechara FG, et al. A prospective bicenter
study investigating the diagnostic value of procalciton in patients
with acute appendicitis. Eur Surg Res. 2009;43(3):291-297.

PMPH_CH33.indd 283

283

24. Aydelotte JD, Collen JF, Martin R. Analgesic administration


prior to surgical evaluation for acute appendicitis. Curr Surg.
2004;61(4):373-375.
25. Attard AR, Corlett MJ, Kinder NJ, et al. Safety of early pain relief
for acute abdominal pain. Br Med J. 1992;30:554-556.
26. Frei SP, Bond WF, Bazuro RK, et al. Is early analgesia associated with delayed treatment of appendicitis? Am J Emerg Med.
2008;26(2):176-180.
27. Mahadevan M, Graff L. Prospective randomized double blind
study of analgesic use for ED patients with right lower quadrant
abdominal pain. Am J Emerg Med. 2000;18:753-756.
28. Thomas SH, Silen W, Cheema F, Reisner A, Aman S, Goldstein JN, Kumar AM, Stair TO. Effects of morphine analgesia
on diagnostic accuracy in emergency department patients with
abdominal pain: A prospective, randomized trail. J Am Coll Surg.
2003;196:18-31.
29. Wolfe JM, Smithline HA, Phipen S, Montano G, Grab JL,
Fiallo V. Does morphine change the physical examination in
patients with acute appendicitis? Am J Emerg Med. 2004;22:
280-285.
30. Bailey B, Bergeron S, Gravel J, Bussieres JF, Bensoussan A. Efficacy and impact of intravenous morphine before surgical consultation in children with right lower quadrant pain suggestive
of appendicitis: A randomized controlled trail. Ann Emerg Med.
2007;50:371-378.
31. Green R, Bulloch B, Kabani A, Hancock BJ, Tenenbein M. Early
analgesia for children with acute abdominal pain. Pediatrics.
2005;116:978-983.
32. Old JL, Dusing RW, Yap W, Dirks J. Imaging for suspected
appendicitis. Am Fam Physician. 2005;71:71-78.
33. Berry J Jr., Malt RA. Appendicitis near its centenary. Ann Surg.
1984;200:567-575.
34. Graffeo CS, Counselman FL. Appendicitis. Emerg Med Clin
North Am. 1996;14:653-671.
35. Rao PM, Rhea JT, Rao JA, Conn AK. Plain abdominal radiography in clinically suspected appendicitis: Diagnostic yield,
resource use, and comparison with CT. Am J Emerg Med. 1999;17:
325-328.
36. Boleslawski E, Panis Y, Benoist S, Denet C, Mariani P, Valleur P.
Plain abdominal radiography as a routine procedure for acute
abdominal pain of the right lower quadrant: Prospective evaluation. World J Surg. 1999;23:262-264.
37. Deutsch A, Leopold GR. Ultrasonic demonstration of the
inflamed appendix: Case report. Radiology. 1981;140:163-164.
38. Prystowsky JB, Pugh CM, Nagle AP. Current problems in surgery: Appendicitis. Curr Probl Surg. 2005;42(10):688-742.
39. Adams DH, Fine C, Brooks DC. High-resolution real-time ultrasonography. A new tool in the diagnosis of acute appendicitis.
Am J Surg. 1988;155:93-97.
40. Rettenbacher T, Hollerweger A, Macheiner P, Rettenbacher
L, Frass R, Schneider B, et al. Presence or absence of gas in
the appendix: Additional criteria to rule out or confirm
acute appendicitis-evaluation with US. Radiology. 2000;214:
183-187.
41. Terasawa T, Blackmore CC, Bent S. Systemic review: Computed
tomography and ultrasonography to detect acute appendicitis in
adults and adolescents. Ann Inter Med. 2004;141(7):537.
42. Doria AS, Moineddin R, Kellenberger CJ, et al. US or CT for
diagnosis in children and adults? A meta-analysis. Radiology.
2006;241(1):83-94.
43. Johansson EP, Rydh A, Rilund KA. Ultrasound, computed
tomography, and laboratory findings in the diagnosis of appendicitis. Acta Radiol. 2007;48(3):267-273.

5/22/2012 5:24:34 PM

284

Surgery: Evidence-Based Practice

44. Patel SJ, Reede DL, Katz DS, et al. Imaging the pregnant patient
for nonobstretric conditions: Algorithms and radiation dose
considerations. Radiographics. 2007;27(6):1705-1722.
45. Parks NA, Schroeppel TJ. Update on imaging for acute appendicitis. Surg Clin N Am. 2011;91:141-154.
46. Melton GB, Duncan MD. Acute appendicitis. In: Cameron JL,
ed. Current Surgical Therapy. 9th ed. St. Louis, MO: Mosby;
2008:257-261.
47. Choi D, Park H, Lee YR, Kook SH, Kim SK, Kwag HJ, Chung
EC. The most useful findings for diagnosing acute appendicitis in contrast enhanced helical CT. Acta Radiologica. 2003;44:
574-582.
48. Hansen AJ, Young SW, De Petris G, et al. Histologic severity of
appendicitis can be predicted by computed tomography. Arch
Surg. 2004;139:1304-1308.
49. Ganguli S, Raptopoulos V, Komlos F, Siewert B, Kruskal J. Right
lower quadrant pain: Value of the nonvisualized appendix in
patients at multidetector CT. Radiology. 2006;241(1):175-180.
50. Rao PM, Rhea JT, Novelline RA, et al. Effect of computed tomography of the appendix on treatment of patients and use of hospital resources. N Engl J Med. 1998;338:141-146.
51. Rao PM, Rhea JT, Rattner DW, et al. Introduction of appendiceal
CT: Impact on negative appendectomy and appendiceal perforation rates. Ann Surg. 1999;229:344-339.
52. Perez J, Barone JE, Wilbanks TO, Jorgensson D, Corvo PR. Liberal use of computed tomography scanning does not improve
the diagnostic accuracy in appendicitis. Am J Surg. 2003;185:
194-197.
53. Morris KT, Kavanagh M, Hansen P, Whiteford MH, Deveney K,
Standage B. The rational use of computed tomography scans in
the diagnosis of appendicitis. Am J Surg. 2002;183:547-550.
54. Holloway JA, Westerbuhr LM, Chain J, Forney GA, White TW,
Hughes RJ, Blankenship JD. Is appendiceal computed tomography
in a community hospital helpful? Am J Surg. 2003;186:682-684.
55. Patrick DA, Janik JE, Janik JS Bensard DD, Karrer FM. Increased
CT scan utilization does not improve the diagnostic accuracy of
appendicitis in children. J Pediatr Surg. 2003;38:659-662.
56. Coursey CA, Nelson RC, Patel MB, et al. Making the diagnosis of acute appendicitis: Do more preoperative scans mean
fewer negative appendectomies? A 10-year study. Radiology.
2010;254(2):460-468.
57. Livingston EH, Woodward WA, Sarosi GA, Haley RW. Disconnect between incidence of nonperforated and perforated appendicitis: Implications for pathophysiology and management. Ann
Surg. 2007;245(6):888-892.
58. Krajewski SA, Hameed SM, Smink DS, Rodgers SO Jr. Access to
emergency care: A comparative study between the Canadian and
American health care systems. Surgery. 2009;186(2):300-307.
59. Guiliano V, Guiliano C, Pinto F, et al. CT method for visualization of the appendix using fi xed oral dosage of diatrizoate clinical experience in 525 cases. Emerg Radiol. 2005;190:1300-1306.
60. Weltman DI, Yu J, et al. Diagnosis of acute appendicitis: Comparison of 5- and 10- mm CT sections in the same patient. Radiology. 2000;216:172-177.
61. Van Randen A, Bipat S, Zwindermann AH, Ubbink DT, Stoker J,
Boermeester MA. Acute appendicitis: Meta-analysis of diagnostic
performance of CT and graded compression US related to prevalence of disease. Radiology. 5 August, 2008. [Epub ahead of print.]
62. Peck J, et al. The clinical role of noncontrast helical computed tomography in the diagnosis of appendicitis. Am J Surg.
2000;180:133-136.
63. Rhea JT, Rao PM, Novelline RA, McCabe CJ. A focused appendiceal CT technique to reduce the cost of caring for patients

PMPH_CH33.indd 284

64.

65.

66.
67.

68.

69.

70.

71.

72.

73.
74.

75.

76.

77.

78.

79.
80.

81.

82.
83.

with clinically suspected appendicitis. AJR Am J Roentgenol.


1997;169:113-118.
Pena BM, Taylor GA, Lund DP, Mandl KD. Effect of computed
tomography on patient management and costs in children with
suspected appendicitis. Pediatrics. 2001;107:1231.
McKay R, Shepherd J. The use of the clinical scoring system by
Alvardo in the decision to perform computed tomography for
acute appendicitis in the ED. Am J Emerg Med. 2007;25:489-493.
Alvarado A. A practical score for early diagnosis of acute appendicitis. Ann Emerg Med. 1986;15:557-565.
Hong JJ, Cohn SM, Ekeh AP, Newman M, et al. A prospective randomized study of clinical assessment versus computed
tomography for the diagnosis of acute appendicitis. Surg Infect
(Larchmt). 2003;3:231-239.
Lopez PP, Cohn SM, Popkin CA, et al. The use of computed
tomography scan to rule out appendicitis in women of childbearing age is as accurate as clinical exam: A prospective randomized
trail. Am Surg. 2007;73:1232-1236.
Petrosyan M, Estrada J, Chan S, Somers S, et al. CT scan in
patients with suspected appendicitis: Clinical implications for
the acute care surgeon. Eur Surg Res. 2008;40:211-219.
Martin AE, Vollman D, Adler B, Caniano DA. CT scans may
not reduce the negative appendectomy rate in children. J Pediatr
Surg. 2004;39:886-890.
Singh A, Danrad R, Hahn PF, et al. MR imaging of the acute
abdomen and pelvis: Acute appendicitis and beyond. Radiographics. 2007;27:1419-1431.
Cobben L, Groot I, Kingma L, et al. A simple MRI protocol in
patients with clinically suspected appendicitis: Results in 138
patients and effect on outcome of appendectomy. Eur Radiol.
2009;19:1175-1183.
Tkacz JN, Anderson SA, Soto J. MR imaging in gastrointestinal
emergencies. Radiographics. 2009;29:1767-1780.
Tannus JF, Dagoglu G, Oto A. Magnetic resonance imaging of
maternal diseases of the abdomen and pelvis in the pregnant
patient. Am J Perinatol. 2008;25:605-610.
Beddy P, Keogan MT, Sala E, Griffi n N. Magnetic resonance
imaging for the evaluation of acute abdominal pain in pregnancy. Semin Ultrasound CT MR. 2010;31:433-441.
Leeuwenburgh MM, Lameris W, van Randen A, et al. Optimizing imaging in suspected appendicitis (OPTIMAP-Study): A
multicenter diagnostic accuracy study of MRI in patients with
suspected acute appendicitis. Study protocol. BMC Emerg Med.
2010;20:10-19.
Altmeier WA. The bacterial flora of acute perforated appendicitis
with peritonitis. A bacteriological study based upon a hundred
cases. Ann Surg. 1938;107:517-528.
Bennion RS, Thompson JE Jr., Baron EJ, Finegold SM. Gangrenous and perforated appendicitis with peritonitis: Treatment
and bacteriology. Clin Ther. 1990;12 suppl C:31-44.
Fischer AC. Acute appendicitis. In: Cameron JL, ed. Current Surgical Therapy. 7th ed. St. Louis, MO: Mosby; 2001:267-272.
Almqvist P, Leandoer L, Tornqvist A. Timing of Antibiotic treatment in non-perforated gangrenous appendicitis. Eur J Surg.
1995;161:431-433.
Bauer T, Vennitis B, Holm B, et al. Antibiotic prophylaxis in
acute non-perforated appendicitis. The Danish Multicenter
Study GroupIII. Ann Surg. 1989;209:307-311.
Gorbach SL. Antimicrobial prophylaxis for appendectomy and
colorectal surgery. Rev Infect Dis. 1991;13 Suppl 10:15-20.
Pottecher T, Gogny E, Pain L. Antibiotic prophylaxis and
appendectomy. Ann Fr Anesth Reanim. 1994;13(5 Suppl):
S154-157.

5/22/2012 5:24:34 PM

Appendicitis

84. Mui LM, Ng CS, Wong SK, et al. Optimun duration of prophylactic antibiotics in acute non-perforated appendicitis. ANZ
J Surg. 2005;75(6):425-428.
85. Le D, Rusin W, Hill B, Langell J. Post-operative antibiotic use in
non-perforated appendicitis. Am J Surg. 2009;198(6):748-752.
86. Solomkin JS, Mazuski JE, Bradley JS, et al. Diagnosis and
management of complicated intra-abdominal infection in
adults and children: Guidelines by the Surgical Infection
Society and the Infectious Diseases Society of America. Clin Inf
Dis. 2010;50:133-164.
87. Andersen BR, Kallehave FL, Andersen HK. Antibiotics versus
placebo for prevention of postoperative infection after
appendectomy. Cochrane Database Syst Rev. 2005;(3):
CD001439.
88. Fraser JD, Aguayo P, Leys CM, et al. A complete course of
intravenous antibiotics vs a combination of intravenous and oral
antibiotics for perforated appendicitis in children: A prospective,
randomized trial. J Ped Surg. 2010;45(6):1198-1202.
89. McBurney C. The incision made in the abdominal wall in cases
of appendicitis, with description of a new method of operating.
Ann Surg. 1894;20:38.
90. Semm K. Endoscopic appendectomy. Endoscopy. 1983;15:
59-64.
91. Cervini P, Smith LC, Urbach DR. The surgeon on call is a
strong factor determining the use of a laparoscopic approach
for appendectomy. Surg Endosc. 2002;16:1774-1777.
92. Street D, Bodai BI, Owens LJ, et al. Simple ligation vs. stump
ligation in appendectomy. Arch Surg. 1988;123:689.
93. Cohn SM, Giannotti G, Ong AW, et al. Prospective randomized
trail of two wound management strategies for dirty abdominal
wounds. Ann Surg. 2001;233:409-413.
94. Greenall MJ, Evans M, Pollack AV. Should you drain a perforated
appendix? Br J Surg. 1978;65:880.
95. Kum CK, Ngoi SS, Goh PMY, et al. Randomized controlled trail
comparing laparoscopic and open appendectomy. Br J Surg.
1993;80:1599-1600.
96. Martin LC, Puente I, Sosa JL, et al. Open versus laparoscopic
appendectomy. Ann Surg. 1995:222:256-262.
97. Minne L, Burnell A, Ratzer E, et al. Laparoscopic vs open
appendectomy. Arch Surg. 1997;132:708-712.
98. Hellberg A, Rudberg C, Kullman E, et al. Prospective randomized multicenter study of laparoscopic versus open appendectomy. Br J Surg. 1999;86:48-53.
99. Sauerland S, Lefering R, Holthausen U, et al. A meta-analysis
of studies comparing laparoscopic with conventional
appendectomy. In: Krahenbuhl L, Frei E, Klaiber CH, et al.
eds. Acute Appendicitis: Standard Treatment or Laparoscopic
Surgery? Basel: Krager; 1998:109-114.
100. Golub R, Siddiqui F, Pohl D. Laparaoscopic versus open
appendectomy: A meta-analysis. J AM Coll Surg. 1998;186:
545-553.
101. Paterson HM, Qadan M, de Luca SM, et al. Changing trends in
surgery for acute appendicitis. Br J Surg. 2008;95:363-368.
102. Milne AA, Bradbury AW. Residual appendicitis following
incomplete laparoscopic appendicectomy. Br J Surg. 1996;
83:217.
103. Sauerland S, Jaschinski T, Neugebauer EAM. Laparoscopic
versus open laparoscopic surgery for suspected appendicitis.
Cochrane Database Syst. Rev. 2010;6(10):CD001546. DOI:
10.1002/14651858. CD001546.pub3.
104. Von Sonnenberg E, Wittich GR, Casola G, Neff CC, Hoyt DB,
et al. Periappendiceal abscesses: Percutaneous drainage. Radiology.
1987;163:23-26.

PMPH_CH33.indd 285

285

105. Frei SP, Bond WF, Bazuro RK, et al. Appendicitis outcomes
with increased computed tomographic scanning. Am J Emerg
Med. 2008;26:39.
106. Nitecki S, Assalia A, Schein M. Contemporary management of
appendiceal mass. Br J Surg. 1993;30:18.
107. Thompson JE Jr., Bennion RS, Schmit PJ, et al. Cecectomy for
complicated appendicitis. J Am Coll Surg. 1994;179:135.
108. Adalla SA. Appendiceal mass: Interval appendectomy should
not be the rule. Br J Clin Pract. 1996;50:168.
109. Ein SH, Shandling B. Is interval appendectomy necessary after
rupture of an appendiceal mass? J Pediatr Surg. 1996;31:849.
110. Nguyen DB, Silen W, Hodin RA. Interval appendectomy in the
laparoscopic era. 1999;3:189-193.
111. Andersson RE, Petzhold MG. Nonsurgical treatment of
appendiceal abscess or phlegmon: A systematic review and
meta-analysis. Ann Surg. 2007;246:741-748.
112. Roach JP, Patrick DA, Bruny JL, Allshouse MJ, Karrer FM,
Ziegler MM. Complicated appendicitis in children: A clear role
for drainage and delayed appendectomy. Am J Surg. 2007;194:
769-772.
113. Bufo AJ, Shah RS, Li MH, Cyr NA, Hollabaugh RS, Hixson
SD, Schropp KP, Lasater OE, Joyner RE, Lobe TE. Interval
appendectomy for perforated appendicitis in children. J
Laparoendosc Adv Surg Tech A. 1998;8(4):209-214.
114. Roach JP, Patrick DA, Bruny JL, Allshouse MJ, Karrer FM,
Ziegler MM. Complicated appendicitis in children: A clear
role for drainage and delayed appendectomy. Am J Surg.
2007;194:769-772.
115. Owen A, Moore O, Marven S, Roberts J. Interval laparoscopic
appendectomy in children. J Laparoendosc Adv Surg Tech A.
2006;16:308-311.
116. St Peter SD, Aguayo P, Fraser JD, et al. Initial laparoscopic
appendectomy versus initial nonoperative management and
interval appendectomy for perforated appendicitis with abscess: A
prospective, randomized trail. J Pediatr Surg. 2010;45(1):236-240.
117. Kaminski A, Liu IL, Appelbaum H, Lee SL, Haigh PI. Routine
interval appendectomy is not justified after intial nonoperative
treatment of acute appendicitis. Arch Surg. 2005;140(9):897-901.
118. Paupong D, Lee SL, Haigh PI, Kaminski A, Lui IL, Applebaum
H. Routine interval appendectomy in children is not indicated.
J Pediatr Surg. 2007;42(9):1500-1503.
119. Tekin A, Kurtoglu HC, Can I, Oztan S. Routine interval
appendectomy is unnecessary after conservative treatment of
appendiceal mass. Colorectal Dis. 2008;10(5):465-468.
120. Lai HW, Loong CC, Chiu JH, et al. Interval appendectomy after
conservative treatment of an appendiceal mass. World J Surg.
2006;30(3):352-357.
121. Stevens CT, de Vries JE. Interval appendectomy as indicated
rather than as routine therapy: Fewer operations and shorter
hospital stays. Ned Tijdschr Geneeskd. 2007;151(13):759-763.
122. Lai HW, Loong CC, Wu CW, Lui WY. Watchful waiting
versus interval appendectomy for patients who recovered from
acute appendicitis with tumor formation: A cost-effectiveness
analysis. J Chin Med Assoc. 2005;68(9):431-434.
123. Andersson RE, Petzold MG. Nonsurgical treatment of
appendiceal abscess or phlegmon: A systemic review and metaanalysis. Ann Surg. 2007;246:741-748.
124. Deakin DE, Ahmed I. Interval appendectomy after resolution
of adult inflammatory appendix massis it necessary? Surgeon.
2007;5(1):45-50.
125. Groetsch SM, Shaughnessy JM. Medical management of acute
appendicitis: A case report. J AM Board Fam Prac. 2001;14(3):
225-226.

5/22/2012 5:24:35 PM

286

Surgery: Evidence-Based Practice

126. Salim AS, Ahmed TM. Antibiotic treatment of acute


appendicitisinitial observations. Saudi Med J. 2001;22;
643-644.
127. Migraine S, Atri M, Bret PM, Lough JO, Hinchey JE. Spontaneously resolving acute appendicitis: Clinical and sonographic
documentation. Radiology. 1997;205:55-58.
128. Andersson RE. The natural history and traditional management
of appendicitis revisited: Spontaneous resolution and
predominance of prehospital perforations imply that a correct
diagnosis is more important than an early diagnosis. World J
Surg. 2007;31:86-92.
129. Barber MD, McLaren J, Rainey JB. Recurrent appendicitis. Br J
Surg. 1997;84:110-112.
130. Humes DJ and Simpson J. Acute appendicitis. BMJ. 2006;333:
530-534.
131. Cobben LP, de Van Otterloo AM, Puylaert JB. Spontaneously
resolving appendicitis: Frequency and natural history in 60
patients. Radiology. 2000;215:349-352.

PMPH_CH33.indd 286

132. Eriksson S, Granstrom L. Randomized controlled trail of


appendectomy versus antibiotic therapy for acute appendicitis.
Br J Surg. 1995;82:166-169.
133. Styrud J, Eriksson S, Nilsson I, Ahlberg G, Haapaniemi S,
Neovius G, Rex L, Badume I, Granstrm L. Appendectomy
versus antibiotic treatment in acute appendicitis: A prospective
multicenter randomized controlled trial. World J Surg.
2006;30:1033-1037.
134. Varadhan KK, Humes DJ, Neal KR, Lobo DN. Antibiotic
therapy versus appendectomy for acute appendicitis: A metaanalysis. World J Surg. 2010;34(2):199-209.
135. Hansson J, Komer U, Khorram-Manesh A, et al. Randomized
clinical trail of antibiotic therapy versus appendectomy as
primary treatment of acute appendicitis in unselected patients.
Br J Surg. 2009 May;(5):473-481.
136. Soreide K. Should antibiotic treatment replace appendectomy
for acute appendicitis? Nat Clin Pract Gastroenterol Hepatol.
2007;4:584-585.

5/22/2012 5:24:35 PM

CHAPTER 34

Hemorrhoids
Clarence E. Clark III

INTRODUCTION

patients with prolapse, pain, or itching. The majority of the patients


in these studies had Grade III hemorrhoids. No study addressed the
effects of fiber on Grade IIIIV hemorrhoids alone in an RCT.
Answer: Dietary fiber improves symptoms including bleeding in patients with Grade III hemorrhoids with a potential benefit in higher grade hemorrhoids. (Grade A recommendation)

There are one million new cases of hemorrhoids per year in


the United States.1 Sixteen million people in total are affected
annually2,3 and 168,000 people are hospitalized for complications
related to this disease.4 Nearly two million ambulatory care visits
per year are reported for hemorrhoids in the United States5 making this disease a significant healthcare issue.
Hemorrhoids are classified as internal, external, or mixed.
Internal hemorrhoids (IHs) are vascular cushions found above the
dentate line and external hemorrhoids (EHs) are found below
the dentate line.1 IHs are further classified based on their symptoms: first degree hemorrhoids are those that cause bleeding but
do not prolapse; second degree hemorrhoids prolapse out of the
anal canal during defecation and spontaneously return to their
anatomical position; third degree hemorrhoids prolapse and
require digital replacement; and fourth-degree hemorrhoids are
permanently prolapsed and cannot be reduced.
Evaluation starts with history and physical examination paying close attention to complaints of anal bleeding, itching, discharge, discomfort, pain, or prolapse. One should also document
the type of bowel function the patient is having including consistency of stool, the presence of straining, and any incontinence if
present. Endoscopic evaluation of the rectum should be utilized to
exclude the presence of any additional pathology prior to considering a management strategy.

2. Is observation with no procedural intervention a viable option?


The potential impact of doing nothing for symptomatic hemorrhoids
should be discussed with the patient along with described minor
procedures and operative treatment options. An examination of the
natural history of a first episode symptomatic Grade II hemorrhoids
showed that treatment with rubber band ligation (RBL) had a better
prognosis over observation alone over a 48-month follow-up, including the need to treat (hemorrhoidectomy for recurrent symptoms
performed in 29.6% vs. 40.2%) and relief of symptoms after treatment
(48% vs. 19.8%).7 However, the authors note that 25% of the observation group did remit within their 48-month follow-up period.
Answer: Early intervention with RBL is superior to the observation of IHs. This study shows that the observation significantly increases the risk of developing symptomatic hemorrhoids requiring surgery.
Intervention should be considered early in these patients in light of
the clear benefit of symptom relief. (Grade A recommendation)

MANAGEMENT IHS
Anal dilation, injection sclerotherapy (IS), cryotherapy, infrared
coagulation (IRC), laser therapy, diathermy coagulation, and RBL
have been described as outpatient options for treating hemorrhoids. Here we will discuss the evidence-based data of these
treatment modalities.

Nonoperative Management
1. Does fiber reduce the symptoms of hemorrhoids?
After ones workup confirms the presence of symptomatic hemorrhoids, the management starts with diet modification. In a metaanalysis of seven randomized control trials (RCTs), the fiber had a
beneficial effect in the treatment of symptomatic hemorrhoids.6 The
risk of persistent symptoms was reduced by 47% in patients treated
with the fiber (risk reduction [RR], 0.47; 95% CI, 0.320.68). The effect
on arm bleeding showed a significant difference in favor of the fiber
(RR, 0.50; 95% CI, 0.280.89). No significant difference was seen in

3. Is there a clear advantage of one minor procedure strategy


over another?
A randomized prospective study in Europe with a 17-year follow-up
compared anal dilatation with surgical hemorrhoidectomy for
Grade IIIII hemorrhoids.8 Three groups were examined: Group A
287

PMPH_CH34.indd 287

5/22/2012 5:25:10 PM

288

Surgery: Evidence-Based Practice

underwent Milligan9 hemorrhoidectomy (41 patients) with no


retractor, Group B had the original Lords six finger dilation with
a dilator (46 patients), and Group C underwent anal dilation as
described previously without the dilator (51 patients). More patients
were symptom-free in Group A (52%) versus Group B (23%) and
Group C (27%) after treatment. Recurrence of hemorrhoids was
lower for the hemorrhoidectomy group. Fecal incontinence was
the major complication found during follow-up for Groups B and
C (52% of the total patients).
A meta-analysis by MacRae et al.10 compared several of the
minor procedures and surgical hemorrhoidectomy with no treatment. Overall, patients undergoing hemorrhoidectomy had a significantly better response to treatment than did patients treated with
RBL (p = .001), although this was at a cost of a significantly greater
risk of complications (p = .02) and pain (p < .0001). For Grade III
hemorrhoids alone, no difference was shown. RBL was shown to be
significantly better than IS in resolution of symptoms (p = .005).
This difference was shown for both Grades I and II hemorrhoids
(p = .007) and Grade III hemorrhoids (p = .042), with no significant difference in the complication rate. Patients treated with RBL
were less likely to require further therapy than those treated with
either sclerotherapy (p = .031) or IRC (p = .0014), although the pain
was significantly more likely to occur following RBL (IS, p = .03;
IRC, p < .0001). No difference was found between sclerotherapy
and infrared photocoagulation for any of the outcome measures.
Recommendations from this article reveal that RBL should
be the first-line treatment for Grade III prolapsing hemorrhoids,
reserving hemorrhoidectomy for patients whose symptoms are
not relieved after treatment, and RBL appears to be the therapy of
choice for Grades III hemorrhoids.
A more recent meta-analysis of RCT of RBL versus excisional
hemorrhoidectomy (closed [CH] or open [OH])11 found RBL to be
as effective for Grade II hemorrhoids. For Grade III hemorrhoids,
the recurrence rate was improved with hemorrhoidectomy. Symptoms (incontinence, anal stenosis, sepsis, and significant bleeding),
the time from intervention to return to work, and complications
were higher for excisional hemorrhoidectomy.
Answer: RBL is preferred over anal dilation, sclerotherapy, and
infrared photocoagulation. RBL is the therapy of choice for Grades I
and II IHs. RBL should be the first-line treatment for Grade III prolapsing hemorrhoids, reserving hemorrhoidectomy for patients whose
symptoms are not relieved after treatment. Anal dilation should be
abandoned due to significant morbidity. (Grade A recommendation)
4. Has one operative strategy proven to be superior to others?
If nonoperative management fails, surgery may be required. Various operative approaches to hemorrhoids have been prospectively
analyzed, including OH versus CH, stapled hemorrhoidectomy or
hemorrhoidopexy (SH), and hemorrhoidectomy with bipolar scissors hemeroidectomy (BSH) and harmonic scalpel (HSH).

OH versus CH
Many RCTs have compared OH versus CH with no clear advantage
of one technique over another. Recent RCTs have shown that CH
offers faster healing time. Arbman et al. found that at 3 weeks,
86% of patients in the Ferguson group (CH, n = 38) had completely healed wounds compared with 18% in the MMH (OH,
n = 39) group (p < .001).12 Arroyo et al. also found that healing
during the first postoperative month was faster in the CH group

PMPH_CH34.indd 288

(n = 100) compared with OH group (n = 100) (90% vs. 40%;


p < .05).13 Another RCT of 80 patients (40 OH, 40 CH) showed that
the mean operating time in the OH group (35 7 min) was significantly shorter than in the CH group (45 8 min; p < .001).14 No significant differences were observed in the duration of hospital stay or
in the mean duration of the inability to work. They also found that
the mean healing time was significantly shorter in the CH group
(2.8 0.5 weeks) than in the OH group (3.5 0.6 weeks; p < .001).
The data for CH versus OH do not favor one procedure over
another. No clear difference is seen with regard to postoperative
pain. CH appears to offer faster wound healing but OH offers shorter
operative time and possibly improved morbidity.13 It is important to
note that most of these studies did include internal and EHs.

Harmonic Scalpel, Bipolar Diathermy


The original description of MilliganMorgan hemorrhoidectomy
(MMH) used scissors for excision.9 HSH and BSH are alternative
modalities for hemorrhoid excision. Recently, a prospective doubleblind randomized trial of 86 patients with prolapsing hemorrhoids looked at MMH versus BSH versus HSH.15 There was no
significant difference in the complication rates among the three
groups. There was no case of failure of hemostasis in the BSH
and HSH groups. HSH and BSH were found to be associated with
less operative blood loss when compared with MMH (p = .036
and .028, respectively), though this has little clinical relevance.
Cheung et al. noted that HSH is as safe and effective, with similar complication and recurrence rates, as diathermy or scissor
excisionligation hemorrhoidectomy. They also noted that HSH
has less postoperative pain.

Stapled Hemorrhoidopexy
A more recent, novel approach to hemorrhoidectomy is SH also
known as Procedure for Prolapse and Hemorrhoids (PPH) and
stapled hemorrhoidectomy. Longos hemorrhoidopexy, as described in 1998, does not involve removing mucosa or hemorrhoidal tissue.16,17 The purpose of the procedure is to remove the
feeding vessels to the hemorrhoids to treat symptomatic hemorrhoids. Jayaraman et al.18 looked at the outcomes of this technique
by performing a meta-analysis of 12 RCTs of stapled circular hemorrhoidopexy versus conventional OH or CH for the treatment of
Grade III and IV hemorrhoids. Follow-up periods ranged from 6
to 39 months with a median follow-up period of 714 months.
A trend to more complaints of hemorrhoidal bleeding in
patients with SH was seen (9 trials, 699 patients, OR 1.33, CI, 0.84
2.08) as well as a significantly higher proportion of patients with
complaints of prolapse after SH (8 studies, 798 patients, OR 2.96, CI,
1.336.58, p = .008). Patients with SH were less likely to complain
of pruritus ani at the final follow-up than those with CH (4 studies,
273 patients, OR 0.66, CI, 0.291.50).
A nonsignificant trend showed that patients with SH were more
likely to complain of difficulties with soiling, hygiene, or incontinence. Trends showing a higher proportion of patients with perianal skin tags were seen in the SH group as compared with CH at
all time points. A nonsignificant trend demonstrated that patients
with SH were more likely to require repeat operations of any nature
in the long-term follow-up for their hemorrhoids. Patients with SH
were significantly more likely to have recurrent hemorrhoids in the
long-term follow-up at all time points than those with CH (7 trials,
537 patients, OR 3.85, CI, 1.4710.07, p = .006).

5/22/2012 5:25:11 PM

Hemorrhoids

Chung et al. analyzed 88 patients with Grade III hemorrhoids


(HSH = 45, PPH = 43) and a median follow-up period of 15 months
(range, 630).19 Comparing the two groups, the authors found no
significant observable difference in operation time, blood loss, or
time to first bowel movement. They state that PPH derives greater
short-term benefits, with reduced pain, shorter length of hospital
stay, and earlier return to work. Recommendations cannot be drawn
from this data due to the short follow-up and small sample size.
Computer-guided BSH (LigaSureTM) has also been compared with
PPH20 showing that postoperative pain scores, patient satisfaction,
and self-assessment of activity were almost identical in both treatment groups. They also note that 9 of 25 PPH patients had skin tags
and prolapsed hemorrhoids removed with conventional diathermy.
The authors revealed that the inability of PPH to address these additional findings is a disadvantage of this treatment modality. Kraemer
et al. used LigaSureTM whenever there is a need for more extensive
excision or anodermal reconstruction, such as in fourth-degree
piles. The design of the RCT prevents one to draw definitive recommendations regarding the use of PPH. The data do confirm findings
that PPH is not a good choice for Grade IV hemorrhoids.
Answer: Conventional excisional hemorrhoidectomy is preferred over PPH. Conventional excisional surgery is the gold
standard in the surgical treatment of Grade III and IV IHs. The
data for CH versus OH do not favor one procedure over the other.
PPH is not a good choice for Grade IV IHs. No clear recommendations can be made based on the current data favoring PPH over
BSH or HSH for the treatment of symptomatic hemorrhoids.
Recommendations could not be made favoring BSH or HSH over
traditional MMH. Trends of intraoperative bleeding, however,
do favor the use of newer technologies over scissors for excision.
(Grade B recommendations)

PERIOPERATIVE CONSIDERATIONS
5. What is the best strategy for controlling postoperative pain?
Various perioperative analgesia strategies have been studied
including the use of viscous lidocaine, epidural anesthesia, locally
injected bupivacaine/ropivacaine, botulinum toxin injection,
posterior perineal block, ischiorectal fossa block, transcutaneous
electrical nerve stimulation, transdermal fentanyl patch, ketorolac
tromethamine injection, and spinal block.9,15,21-38
In one RCT, local injection of bupivacaine after hemorrhoidectomy provided initial pain relief, but patients did not obtain an
overall analgesic benefit.22 Botulinum injection (20 U) into the internal sphincter showed pain relief at days 6 and 7 with no significant
difference in pain control in the early postoperative period.9
RCT that did show significant pain relief in the early postoperative period and reduced opioid consumption utilized posterior
perineal block, perianal ropivacaine injection, ischiorectal fossa
block, lidocaine injection with topical anesthetic prior to injection, and pudendal nerve block.23,24,26,34,35 One study compared
local with spinal anesthesia and found that the local perianal
nerve block for hemorrhoidectomy is superior to the spinal block
due to a lower incidence of post-op urinary retention and less
requirement of parenteral analgesics after surgery.32
Answer: Injection of local anesthetic reduces postoperative
pain after hemorrhoidectomy. Perianal, posterior perineal, and
pudendal nerve injections are all effective in pain control. Botox
injection improves pain control in the late recovery period. (Grade
A recommendation)

PMPH_CH34.indd 289

289

6. Does low volume resuscitation reduce postoperative urinary


retention?
Toyonaga et al. reviewed series of over 2000 anorectal surgeries
and found that hemorrhoidectomy, female sex, the presence of preoperative urinary symptoms, diabetes mellitus, the need for postoperative analgesics, and more than three hemorrhoids resected
were independent risk factors for urinary retention.39 They also
showed that perioperative fluid restriction, including limiting the
administration of intravenous fluids, significantly decreased the
incidence of urinary retention (7.9 vs. 16.7%, p < .0001). Other
studies support that this finding40-42 including the administration
of at least 1000 mL of intravenous fluid perioperatively produced
a significant increase in postoperative urinary retention.42 Zaheer
et al. in contrary reported that perioperative fluids were not associated with urinary retention in hemorrhoid surgery.28
Answer: Data support the use of perioperative fluid restriction in anorectal surgery. Hemorrhoidectomy has been associated
with urinary retention and limiting fluid administration may prevent this morbidity. (Grade B recommendation)

MANAGEMENT OF THROMBOSED EHS


7. What is the best management strategy for symptomatic EHs?
The most common findings with EHs are pain and/or ulceration
of a thrombus through the skin.1 Conservative measures are often
utilized which include a combination of localize hygiene, tub
baths, dietary changes, stool softeners, and oral and topical analgesics. There are very few quality studies looking at the management of EHs exclusively.
Greenspon et al.43 retrospectively reviewed that outcomes of
231 patients with thrombosed EHs. 119 (51.5%) were initially
treated conservatively and 112 (48.5%) were treated surgically with
a mean follow-up of 7.6 months (up to 7 years); 97.3% of the surgical patients had excision of their EHs and 2.7% incision. Time
to symptom resolution was 24 days for conservatively managed
patients versus 3.9 days for surgical patients (p < .0001). The frequency of recurrence was significantly higher for the conservative
group (25.4%) than for the surgical group (6.3%; p < .0001). These
data favor excision of thrombosed EHs over conservative therapy.
Jongen et al. focused on the clinical outcomes of the 340
patients who underwent outpatient office excision of symptomatic
EHs.44 All wounds were left open and follow-up was achieved in
70% of the patients. Postoperative complications are rare (1.37.7%).
Anal stenosis, urinary retention, and fecal retention were not seen
in this series. Recurrence rate was 9.2%. Based on their analysis,
the authors recommend excision under local anesthesia in the
office for thrombosed EHs.
A prospective randomized trial examined conservative therapy versus surgery for the treatment of thrombosed EHs.45 Three
arms each had 50 patients: the first group was treated conservatively with 0.2% glyceryl trinitrate (GTN) ointment; the second
group by incision and the third by excision of the thrombosed EH.
At 4 days, there was a significantly less pain in patients treated
by excision as compared with those treated with GTN or incision
(p < .001). At 1 year, all clinical outcomes significantly favored
excision of thrombosed hemorrhoids. Based on their data, the
authors recommend excision of perianal thrombosis under local
anesthesia as the method of choice because it prevents recurrence
of perianal thrombosis and development of anal skin tags.

5/22/2012 5:25:11 PM

290

Surgery: Evidence-Based Practice

Answer: Excision of symptomatic EHs is preferred over


observation with topical agents or incision of hemorrhoid. Excision of EHs under local anesthesia is the method of choice for

symptomatic EHs due to improved symptom relief. Practitioners


can safely perform this procedure in the outpatient office setting.
(Grade B recommendation)

Clinical Question Summary


Question

Answer

Grade of
Recommendation

References

1 Does fiber reduce the symptoms of


hemorrhoids?

Yes. Fiber has a clear benefit for Grade III


hemorrhoids with a likely benefit for
Grade IIIIV as well.

2 Is observation with no procedural


intervention a viable option?

No. Clear benefit with intervention for Grade II


and greater haemorrhoids.

3 Is there a clear advantage of one


minor procedure over the others?

Yes. RBL is superior to anal dilation,


sclerotherapy and infrared photocoagulation.

8, 10, 11

4 Has one operative strategy proven


to be superior to the others?

Yes. Conventional hemorrhoidectomy is superior


to PPH but no significant difference between
MMH and FH.

12-14, 18-20, 22

Clinical Question Summary


Question

Answer

Grade of
Recommendation

References

1 Does low volume resuscitation


reduce postoperative urinary
retention?

Yes. Keeping fluid administration less than 1 L


perioperatively will reduce the occurrence of
urinary retention.

28, 39-42

2 What is the best strategy for


controlling postoperative pain?

Injection of local anesthetic perianal, posterior


perineal, or pudendal nerve improves pain
control and reduces opioid use.

22-25, 34, 35

3 What is the best management


strategy for symptomatic EHs?

Excision over topical agents and incision.

9, 20, 21

Levels of Evidence
Subject

Year

References

Level of
Evidence

Strength of
Recommendation

Fiber diet

2006

1a

Fiber improves and reduces


symptoms from hemorrhoids.

First-line treatment of
IHs

2005

10

1b

RBL is the first-line therapy


followed by hemorrhoidectomy if
symptoms persist or Grade IV.

Open or closed
technique

2002

14

2b

Both are acceptable operative


strategies with no significant
difference in outcomes.

Conventional
hemorrhoidectomy
or PPH

2006

18

1a

Conventional hemorrhoidectomy is
superior to PPH.

Fluid restriction

2006

39

2b

Fluid restriction perioperatively


reduces postoperative urinary
retention.

Management of
symptomatic EHs

2004

43

2b

Excision is superior to topical agents


and incision of EHs.

PMPH_CH34.indd 290

Findings

5/22/2012 5:25:11 PM

Hemorrhoids

REFERENCES
1. Kaidar-Person O, Person B, Wexner S. Hemorrhoidal disease: A
comprehensive review. Journal of the American College of Surgeons. 2007;204(1):102-117.
2. Adams P, Hendershot G, Marano M. Current estimates from
the National Health Interview Survey, 1996. National Center for
Health Statistics. Vital and Health Stat. 1999;10(200).
3. Johanson J. Evidence based approach to the treatment of hemorrhoidal disease. Evidence Based Gastroenterology. 2002;3:26-31.
4. Kozak L, Owings M, Hall M. National Hospital Discharge Survey: 2002 annual summary with detailed diagnosis and procedure data. National Center for Health Statistics. Vital Health
Stat. 2005;13(158).
5. Burt C, Schappert S. Ambulatory care visits to physician offices,
hospital outpatient departments, and emergency departments:
United States, 19992000. National Center for Health Statistics.
Vital and Health Stat. 2004;13(157).
6. Alonso-Coello P, Mills E, Heels-Ansdell D, Lpez-Yarto M,
Zhou Q, Johanson JF, Guyatt G. Fiber for the treatment of hemorrhoids complications: A systematic review and meta-analysis.
Am J Gastroenterol. 2006;101(1):181-188.
7. Jensen S, Harling H, Arseth-hansen P, et al. The natural history of
symptomatic hemorrhoids. Int J Colorectal Dis. 1989;4(1):41-44.
8. Konsten J, Baeten C. Hemorrhoidectomy vs. Lords method:
17-year follow-up of a prospective, randomized trial. Dis Colon
Rectum. 2000;43(4):503-506.
9. Davies J, Duff y D, Boyt N, Aghahoseini A, Alexander D, Leveson
S. Botulinum toxin (botox) reduces pain after hemorrhoidectomy: Results of a double-blind, randomized study. Dis Colon
Rectum. 2003;46(8):1097-1102.
10. MacRae H, McLeod R. Comparison of hemorrhoidal treatment
modalities: A meta-analysis. Dis Colon Rectum. 1995;38(7):
687-694.
11. Shanmugam V, Thaha M, Rabindranath K, et al. Rubber band
ligation versus excisional haemorrhoidectomy for haemorrhoids.
Cochrane Database Syst Rev. 2005;(1):CD005034.
12. Arbman G, Krook H, Haapaniemi S. Closed vs. open
hemorrhoidectomyis there any difference? Dis Colon Rectum.
2000;43(1):31-34.
13. Arroyo A, Perez F, Miranda E, et al. Open versus closed daycase haemorrhoidectomy: Is there any difference? Results of a
prospective randomized study. Int J Colorectal Dis. 2004;19(4):
370-373.
14. Gencosmanoglu R, Sad O, Koc D, et al. Hemorrhoidectomy:
Open or closed technique? A prospective, randomized clinical
trial. Dis Colon Rectum. 2002;45(1):70-75.
15. Kuo RJ. Epidural morphine for post-hemorrhoidectomy analgesia. Dis Colon Rectum. 1984;27(8):529-530.
16. Longo, A. Treatment of hemorrhoids disease by reduction of
mucosa and hemorrhoidal prolapse with a circular suturing
device: A new procedure. Proceedings of the 6th World Congress
of Endoscopic Surgery. 1998.
17. Corman M, Gravi T, Hager M, et al. Longo Stapled haemorrhoidopexy: A consensus position paper by an international
working party-indications, contra-indications and technique.
Colorectal Dis. 5(4):304-310.
18. Jayaraman S, Colquhoun P, Malthaner R. Stapled versus conventional surgery for hemorrhoids. Cochrane Database Syst Rev.
2006;(4):CD005393.
19. Chung C, Cheung H, Chan E, et al. Stapled hemorrhoidopexy vs.
harmonic scalpel hemorrhoidectomy: A randomized trial. Dis
Colon Rectum. 2005;48(6):1213-1219.

PMPH_CH34.indd 291

291

20. Kraemer M, Parulava T, Roblick M, et al. Prospective, randomized study: Proximate PPH stapler vs. LigaSure for hemorrhoidal
surgery. Dis Colon Rectum. 2005;48(8):1517-1522.
21. Smith SL, Simon R. Viscous lidocaine as a posthemorrhoidectomy analgesic. Dis Colon Rectum. 1979;22(1):40-41.
22. Chester JF, Stanford BJ, Gazet JC. Analgesic benefit of locally
injected bupivacaine after hemorrhoidectomy. Dis Colon Rectum. 1990;33(6):487-489.
23. Brunat G, Pouzeratte Y, Mann C, et al. Posterior perineal block
with ropivacaine 0.75% for pain control during and after hemorrhoidectomy. Reg Anesth Pain Med. 2003;28(3):228-232.
24. Vinson-Bonnet B, Coltat JC, Fingerhut A, et al. Local infi ltration
with ropivacaine improves immediate postoperative pain control after hemorrhoidal surgery. Dis Colon Rectum. 2002;45(1):
104-108.
25. Gabrielli F, Cioffi U, Chiarelli M, et al. Hemorrhoidectomy with
posterior perineal block: Experience with 400 cases. Dis Colon
Rectum. 2000;43(6):809-812.
26. Luck AJ, Hewett PJ. Ischiorectal fossa block decreases posthemorrhoidectomy pain: Randomized, prospective, double-blind
clinical trial. Dis Colon Rectum. 2000;43(2):142-145.
27. Chiu JH, Chen WS, Chen CH, et al. Effect of transcutaneous
electrical nerve stimulation for pain relief on patients undergoing hemorrhoidectomy: Prospective, randomized, controlled
trial. Dis Colon Rectum. 1999;42(2):180-185.
28. Zaheer S, Reilly WT, Pemberton JH, et al. Urinary retention
after operations for benign anorectal diseases. Dis Colon Rectum.
1998;41(6):696-704.
29. Kilbride M, Morse M, Senagore A. Transdermal fentanyl improves management of postoperative hemorrhoidectomy pain. Dis
Colon Rectum. 1994;37(11):1070-1072.
30. ODonovan S, Ferrara A, Larach S, et al. Intraoperative use of
Toradol facilitates outpatient hemorrhoidectomy. Dis Colon Rectum. 1994;37(8):793-799.
31. Chen KP, Chan HC, Chen FS, et al. Lumbar extradural morphine
and caudal extradural morphine for postoperative analgesia and
their adverse effects. Ma Zui Xue Za Zhi. 1993;31(1):25-30.
32. Anannamcharoen S, Cheeranont P, Boonya-usadon C. Local
perianal nerve block versus spinal block for closed hemorrhoidectomy: A randomized controlled trial. J Med Assoc Thai.
2008;91(12):1862-1866.
33. Baptista JF, Paulo DN, Paulo IC, et al. Epidural anesthesia using
a 0,75% ropivacaine and subarachnoid anesthesia with a 0,5%
bupivacaine associated or not with clonidine in hemorrhoidectomies. Acta Cir Bras. 2008;23(6):536-542.
34. Shiau JM, Hung KC, Chen HH, et al. Combination of topical
EMLA with local injection of lidocaine: Superior pain relief after
Ferguson hemorrhoidectomy. Clin J Pain. 2007;23(7):586-590.
35. Imbelloni LE, Vieira EM, Gouveia MA, et al. Pudendal block
with bupivacaine for postoperative pain relief. Dis Colon Rectum. 2007;50(10):1656-1661.
36. Naja Z, El-Rajab M, Al-Tannir M, et al. Nerve stimulator guided
pudendal nerve block versus general anesthesia for hemorrhoidectomy. Can J Anaesth. 2006;53(6):579-585.
37. Imbelloni LE, Beato L, Beato C, et al. Bilateral pudendal nerves
block for postoperative analgesia with 0.25% S75:R25 bupivacaine. Pilot study on outpatient hemorrhoidectomy. Rev Bras
Anestesiol. 2005;55(6):614-621.
38. Jirasiritham S, Tantivitayatan K, Jirasiritham S. Perianal blockage with 0.5% bupivacaine for postoperative pain relief in hemorrhoidectomy. J Med Assoc Thai. 2004;87(6):660-664.
39. Toyonaga T, Matsushima M, Sogawa N, et al. Postoperative urinary
retention after surgery for benign anorectal disease: Potential

5/22/2012 5:25:11 PM

292

40.

41.

42.

43.

Surgery: Evidence-Based Practice

risk factors and strategy for prevention. Int J Colorectal Dis.


2006;21:676-682.
Bailey HR, Ferguson JA. Prevention of urinary retention by fluid
restriction following anorectal operations. Dis Colon Rectum.
1976;19:250-252.
Kozol RA, Mason K, McGee K. Post-herniorrhaphy urinary
retention: A randomized prospective study. J Surg Res. 1992;52:
111-112.
Petros JG, Bradley TM. Factors influencing postoperative urinary retention in patients undergoing surgery for benign anorectal disease. Am J Surg. 1990;159:374-376.
Greenspon J, Williams S, Young H, et al. Thrombosed external
hemorrhoids: Outcome after conservative or surgical management. Diseases of the colon and rectum. 2004;47(9):1493-1498.

PMPH_CH34.indd 292

44. Jongen J, Bach S, Stubinger S, et al. Excision of thrombosed external hemorrhoid under local anesthesia: A retrospective evaluation of 340 patients. Dis Colon Rectum. 2003;46(9):1226-1231.
45. Cavcic J, Turcic J, Martinac P, et al. Comparison of topically
applied 0.2% glyceryl trinitrate ointment, incision and excision
in the treatment of perianal thrombosis. Dig Liver Dis. 2001;33:
335-340.
46. Ortiz H, Marzo J, Armendariz P, et al. Stapled hemorrhoidopexy
vs. diathermy excision for fourth-degree hemorrhoids: A randomized, clinical trial and review of the literature. Dis Colon
Rectum. 2005;48(4):809-815.
47. Peng B, Jayne D, Ho Y. Randomized trial of rubber band ligation vs. stapled hemorrhoidectomy for prolapsed piles. Dis Colon
Rectum. 2003;46(3):291-297;discussion 296-297.

5/22/2012 5:25:11 PM

Commentary on Hemorrhoids
Stanley M. Goldberg

SOURCES IN THE LITERATURE

Clarence E. Clark deserves a lot of credit for a first-class review of


the literature on the management of hemorrhoids.
I was pleased to see the recommendation of the use of
rubber band ligation, but unfortunately some of the literature
is based on using poor equipment in performing this simple
clinic procedure. It is critical to band the tissue high in the anal
canal and the Hinkel-James retractor is a great asset in this
operation.
Regarding anal dilatation, I was pleased to see that this technique has no p lace in the armamentarium of the modern colon
and rectal surgeon.
Clark has utilized the best studies to date regarding rubber band ligation for Grade II and some Grade III hemorrhoids.
Excisional hemorrhoidectomy is indicated if the symptoms do
not respond to rubber band ligation or the major symptoms are
related to the external component.
I was so pleased to see that Clark found that several large
studies suggested that the strategy of excision not incision is the
proper management of a thrombosed external hemorrhoid with
multiple blood clots as an outpatient procedure. It is rare for us
to take the patient with a thrombosed external hemorrhoid to the
operating room. Local anesthesia in the prone jack-knife position
works beautifully.
Regarding open versus closed hemorrhoidectomy, the only
benefit of the closed technique is that the surgeon is always thinking about closing the wound and therefore he/she does not remove
too much tissue and create a stricture.
The Harmonic Scalpel is an expensive tool to use while
performing hemorrhoid surgery. Regarding the Procedure for
Prolapse and Hemorrhoids (PPH), I wish the authors would
have elaborated more about the serious complications associated with the PPH procedure. Conventional excisional surgery is the gold standard in the surgical treatment of Grade
III (not responding to rubber band ligation) and Grade IV
hemorrhoids.

1. Brusciano L, Ayabaca SM, Pescatori M, et al. Reinterventions


after complicated and failed stapled haemorrhoidopexy. Dis
Colon Rectum. 2004;47:1846-1851.
2. Angelone G, Giardiello C, Prota C. Stapled hemorrhoidopexy.
Complications and 2-year postoperative follow-up. Ann Surg.
2006;242:29-35.
3. Cheetham MJ, Mortensen NJ, Nystrom PO, Kamm MA, Phillips
RK. Persistent pain and fecal urgency after stapled haemorrhoidectomy. Lancet. 2000;356:730-733.
4. McDonald PJ, Bona R, Cohen CR. Rectovaginal fistula after stapled haemorrhoidopexy. Colorectal Dis. 2004;6:64-65 (letter).
5. Cipriani S, Pescatori M. Acute rectal obstruction after PPH stapled haemorrhoidectomy. Colorectal Dis. 2002;4:367-370.
6. Giordano P, Bradley B, Peiris L. Stapled closure of the rectal
lumen following stapled haemorrhoidopexy: Case report. Dis
Colon Rectum. 2008;(in press).
7. Brown S, Baraza W, Shorthouse A. Total rectal lumen obliteration after stapled haemorrhoidopexy: A cautionary tale. Tech
Coloproctol. 2008;11:357-358.
8. McCloud JM, Jameson JS, Scott AN. Life-threatening sepsis following treatment for haemorrhoids: A systematic review. Colorectal Dis. 2006;8:748-755.
9. Cirocco WC. Life threatening sepsis and mortality following
stapled hemorrhoidopexy. Surgery. 2008;143:824-829.
10. Herold A, Kirsch JJ. Pain after stapled haemorrhoidectomy.
Lancet. 2000;356:2187-2190 (letter).
11. Pessaux P, Lermite E, Tuech JJ, et al. Pelvic sepsis after stapled
hemorrhoidectomy. J Am Coll Surg. 2004;199:824-825.
12. Maw A, Eu KW, Seow-Choen F. Retroperitoneal sepsis complicating stapled hemorrhoidectomy: Report of a case and review of
the literature. Dis Colon Rectum. 2002;45:826-828.
13. Jayaraman S, Colquhoun PH, Malthaner R. Staples versus conventional surgery for hemorrhoids (Review). The Cochrane
Collaboration 2008, Issue 4.

293

PMPH_CH34.indd 293

5/22/2012 5:25:11 PM

CHAPTER 35

Anorectal Fissure, Stricture,


Abscess, and Fistula
W. Brian Perry and Joshua A. Tyler

INTRODUCTION

consensus exists regarding dosing or optimal injection site(s), but


all studies show an advantage over placebo. Recurrence rates of
40% to 50% at 1 year have been reported.2
Summary: Nonoperative therapies that relax the internal anal
sphincter are safe and are superior to placebo in healing anal fissures. Recurrence rates after cessation of therapy are significant.
(Level 1a evidence, Grade A recommendation)

Complaints involving the anorectal region are common in clinical


practice, but are frequently poorly understood and misdiagnosed.
Often, patients are referred for hemorrhoids, which may actually
be a number of other conditions. Correct treatment depends on
timely and accurate diagnosis. Once this occurs, evidence-based
interventions may be applied for anal fissure, stricture, abscess,
and fistula. Hemorrhoids and pilonidal disease are covered in
other chapters.

2. What are the results of lateral internal sphincterotomy (LIS)


for fissures?
Multiple studies and a recent Cochrane review have clearly established LIS as the surgical treatment of choice for chronic anal
fissures. Healing rates of >90% and minor incontinence rates of
<10% are the norm. No difference has been shown between open
or closed techniques.1,3 A tailored or calibrated sphincterotomy,
which aims to limit the amount of sphincter divided, has been
shown to have equivalent or slightly worse healing rates, but less
incontinence.4 Posterior sphincterotomy with fissurectomy is
inferior to LIS in both healing and incontinence rates.5 Small
studies on surgical options such as advancement flaps that do not
divide muscle show promising results, but more work is needed
before these techniques can be universally recommended.6,7 Multiple studies and both Cochrane Reviews confirm the superiority
of LIS to any medical therapy in terms of durable fissure healing
with low rates of troubling incontinence.
Summary: LIS is the surgical treatment of choice for anal fissures and is far superior to any medical therapy. (Level 1a evidence,
Grade A recommendation)

1. What nonoperative therapies are effective in the treatment


of anal fissures?
Many patients with acute anal fissures experience complete healing with little direct intervention. Another sizable portion will
improve with simple means such as increasing dietary fiber, sitz
baths, and nonsteroidal anti-inflammatory drugs. Should these
methods fail, medical therapies directed at lowering anal sphincter pressures have been shown to be effective.1
Topically applied nitric oxide donors such as glyceryl trinitrate
(GTN) reliably relax the internal anal sphincter. Multiple controlled
randomized trials and a recent Cochrane Review2 have shown that
GTN is statistically better than placebo at healing anal fissure
(about 5035%). Headache is the most commonly reported adverse
event associated with GTN use and results in therapy interruption in as much as a quarter of patients. Recurrence of anal fissures
approaches 50% in patients who report initial healing with GTN.
Topical calcium channel blockers have been shown to facilitate healing in 65% to 95% of patients with anal fissures, also
statistically better than placebo. Headache is again the primary,
therapy limiting side effect, but is less frequent than is seen with
GTN. Oral calcium channel blockers are more effective than placebo, but less effective than topical preparations.1
Botulinum toxin injection directly into the internal anal
sphincter causes a temporary chemical sphincterotomy which
allows for healing of 60% to 80% of anal fissures. Minor incontinence to flatus (up to 18%) and stool (<5%) is the most common
side effect and typically improves as the toxin wears off. Little

3. How are anal strictures best treated?


Stricture or stenosis of the anal canal is typically caused by hemorrhoidectomy or other anorectal procedures. This rare condition
causes difficulty with evacuation of the rectum, pain, and bleeding. Patients with mild to moderate degrees of narrowing are
often helped by the addition of fiber and fluids to their diet, digital
dilation, or LIS, although those with chronic tight strictures
require surgical management.
294

PMPH_CH35.indd 294

5/22/2012 5:25:46 PM

Anorectal Fissure, Stricture, Abscess, and Fistula

Several local skin advancement flap techniques have been


described; each aims to bring healthy tissue into the anal canal to
increase its diameter while minimizing donor site morbidity. The
house advancement flap brings a broad-based flap into the anal
canal and allows primary closure of the donor site and can be done
bilaterally if necessary.8 Results have generally been good, with significant improvement seen in up to 90%.9 A YV sliding flap may
be technically less challenging, but necrosis of the tip limits its use
to strictures in the distal anal canal. Rhomboid rotational flaps
are more difficult to perform and have success rates between the
other two. Because of the heterogeneity of these patients and the
relative rarity of this condition, randomized trials are difficult; most reports are case series of less than 50 patients using a
single technique. Farid et al. randomized 60 patients with anal
stenosis to receive a house flap, rhomboid flap, or YV anoplasty.
The house flap performed the best, with durable quality of life
improvement at 1 year.10
Summary: Flap procedures are effective in treating anal strictures. Choice of technique depends on the patient and surgeon
factors. (Level 2b3 evidence, Grade C recommendation)
4. What is the role of antibiotics in the treatment of perirectal
abscesses?
Incision and drainage is the definitive treatment of perirectal
abscess. Antibiotics have not been shown to have any effect on
the time to complete healing nor recurrence rates in patients with
uncomplicated perirectal abscesses. Of the available studies, immunocompromised patients were excluded, including those with diabetes, immunosuppression, or cellulitis of the wound. Although no
studies have been done comparing antibiotics in addition to drainage versus drainage alone in the above populations, the decision to
use antibiotics must be made on a case-by-case basis.11 In addition,
according to the American Heart Associations 2007 revision of the
guidelines for the prevention of infective endocarditis, antibiotic
prophylaxis for patients with gastrointestinal-associated infection,
or infection of the skin or soft tissue is indicated only for patients
with a prosthetic valve, heart transplant, history of infectious endocarditis, and congenital heart defects.12
Summary: Antibiotics are generally not indicated for perirectal abscess following adequate incision and drainage. Exceptions may include immunocompromised patients and those at
high risk for infectious endocarditis. (Level 2c evidence, Grade B
recommendation)
5. What are the best operative strategies for drainage of perirectal abscess?
No randomized controlled trials comparing drainage techniques
for perirectal abscess exist in the current literature. Uncomplicated perirectal abscesses are drained through the perianal skin
at the point of maximal induration, with care taken to avoid the
sphincter complex.11 The cavity should be thoroughly examined
to assess for extensions such as a horseshoe abscess which may
require a counter-incision.13 If a fistula tract is found, a draining
seton may be placed to promote abscess drainage, allowing defi nitive treatment to be delayed until a later date if necessary.
Summary: Incision and drainage is the mainstay of treatment
for perirectal abscesses. If a fistula-in-ano or unexpected abscess
extensions are found, additional techniques such as seton placement or counter incisions may be necessary. (Level 2c evidence,
Grade B recommendation)

PMPH_CH35.indd 295

295

6. Following drainage of a perirectal abscess, can the development of a fistula-in-ano be predicted or prevented?
Little literature exists on the risk of subsequent fistula development following initial perirectal abscess drainage. In a retrospective observational study, Hamadani et al. observed 148 patients
after perirectal abscess drainage without evidence of fistula at the
time of initial operation. Mean follow-up was 38 months. Predictors of subsequent fistula-in-ano development or recurrence of
abscess included individuals younger than age 40 at the time
of presentation (p < .01), as well as patients that were nondiabetic.
Nondiabetics were greater than twice as likely to develop fistula or
recurrence than diabetics (HR 2.69, p = .04). Gender, smoking status, perioperative antibiotics, and human immunodeficiency virus
(HIV) status were not found to be significant. (Level 2s evidence,
Grade B recommendation)
7. What are the best operative approaches to simple fistulain-ano?
A simple fistula-in-ano is generally accepted to primarily involve
the internal sphincter muscle (intersphincteric fistulae). More
complex fistulas tend to branch, and are more likely to involve
the external sphincter muscle. Simple fistulae may be treated by
fistulotomy with or without marsupialization of wound edges,
and fistulectomy. Numerous studies of simple fistulae show fistulotomy to be superior to fistulectomy, with shorter healing times,11
and lower risks of recurrence and incontinence (less than 10% for
each), provided that the internal opening has been properly identified and the fistula tract did not involve the external sphincter.14
These results were replicated in a recent study of a prospectively
collected regional outcomes registry for fistula treatment, which
showed a high success rate in the treatment of simple fistula with
fistulotomy compared with other treatment modalities.15 Fistulotomy and fistulectomy are generally contraindicated in complex
fistulae as well as in patients with Crohns disease due to longer
healing times and an increased risk of incontinence.
Fistulotomy is appropriate for simple fistula-in-ano with high
rates of healing and low rates of incontinence. (Level 2b evidence,
Grade B recommendation)
8. How are setons used in the treatment of anal fistulas?
Setons are used in the treatment of anal fistula to control perianal or perirectal sepsis. Setons are used either in a draining or in
a cutting capacity. Draining setons are used most commonly in
complex fistula tracts as well as in patients with Crohns disease.
These hold the fistula tract open and allow purulence to drain,
thus controlling the abscess. These allow a decrease in perirectal
inflammation, both controlling acute abscesses and allowing time
for perioperative planning when sphincter-sparing techniques
may be employed. Cutting setons have utility in complex fistulae
that involve the external sphincter. Made of either silk suture or
vessel loops, they are serially tightened, allowing a slow division
of the external sphincter muscle with subsequent fibrosis and scar
formation to limit the anatomic defect. The use of cutting setons
results in a low recurrence rate; however, this leads to an incontinence rates of 10% to 60%, particularly in patients with high fistula openings.16,17
Summary: Setons have long been used in the treatment of
complex fistula-in-ano. Draining setons may be used to control
perianal sepsis although awaiting optimal timing for definitive

5/22/2012 5:25:46 PM

296

Surgery: Evidence-Based Practice

surgical management, as well as in Crohns disease. Cutting setons may be used in complex fistula management with low recurrence rates, but may lead to higher rates of incontinence. (Level
2c3b evidence, Grade C recommendation)
9. What are the results of various sphincter-sparing techniques
in the treatment of fistula-in-ano?
Sphincter-sparing options for the treatment of complex fistula-inano include fibrin glue, fistula plug, seton placement, endorectal
advancement flap (ERAF), and ligation of transsphincteric fistula tract. Fistulotomy is contraindicated in fistulae which would
require the division of a significant portion of the external sphincter muscle due to high rates of incontinence. No single study has
prospectively compared all of these modalities, but several small
series have evaluated each individually, or in comparison with
one another.
The use of fibrin glue, an attractive option because no sphincter is divided, showed promise in early studies, with initial closure rates of 60% to 70%.11 Subsequent studies, however, have
been unable to achieve similar results with many showing
closure rates of closer to 40%. Singer randomized patients to
fibrin sealant plus closure of internal opening, fibrin sealant
plus antibiotics, or fibrin sealant plus both, with failures being
offered retreatment. Closure rates at 1-year follow-up were only
44%, 25%, and 35%, respectively, with no significant difference
between groups.18
Bioabsorbable xenograft fistula plugs, one made by Cook Surgical (Surgisis, Cook Surgical, Bloomington, IN) and the other
made by Gore (Bio-A Fistula Plug, W.L. Gore and Associates,
Newark, Delaware), represent another method of tract obliteration. Similar to the results with fibrin glue, initial studies were
promising; Champagne et al. showed an 83% success rate at
1-year average follow-up.10 Later studies have been unable to replicate this, with success rates ranging from 40% to 60% depending
on both the complexity of the fistula and the length of the tract.
Simple fistulas tend to fare better, as do those with tracts longer
than 4 cm; one study has noted that smokers and diabetics tend
to not heal with plug placement.19,20 Both fibrin glue and fistula
plugs have been shown to enable the closure of the fistula tract in

some patients with a minimal side-effect profi le and virtually no


reports of effects on continence.
ERAF also remains a viable option in the closure of complex
anal fistulae. This procedure entails the mobilization of either a
partial or a full-thickness flap of the rectum that covers the internal opening of an anal fistula. It is generally used in complex
fistulae and those with high openings, as well as those that have
failed prior plug treatment. Several trials have compared ERAF
with fistula plug. One small prospective randomized study was
halted early due to high recurrence rates with the fistula plug
compared with ERAF (relative risk 6.40 (95% confidence interval, 1.7023.97); p < .001).21 This study was small, however, with
only 16 patients per treatment group. Another retrospective case
control study also compared ERAF with plug, and found similar
results, with 63% success rate in the ERAF group compared with
32% in the plug group (p = .008).22
Another relatively new procedure that has gained recent
attention is the ligation of intersphincteric fistula tract (LIFT)
procedure. This procedure involves exposing the intersphincteric groove, identifying the fistula tract, and ligating of the tract.
Success rates using the LIFT procedure ranged from 57% to 82%
in two recent prospective observational studies, and most of the
patients undergoing the procedure had complex fistulae that had
failed prior attempts at repair.23,24 Another study incorporated a
reinforcing bioprosthetic material in conjunction with the LIFT
procedure and reported 94% success rates with minimal to no
effects on continence.25
Many options exist in the treatment of complex fistula-inano, and there are limited well-done randomized, prospective
trials comparing the options discussed above. No definitive algorithm exists, and there is great variability in complex fistula types
(cryptoglandular origin vs. Crohns disease) and anatomic locations (transsphincteric vs. suprasphincteric). More studies are
needed to better delineate optimal treatment regimens in this
patient population.
Complex fistula-in-ano may be successfully treated with fibrin
glue, fistula plug, ERAF, or LIFT procedure. Success rates vary
widely, but incontinence rates are infrequent with all of the above
procedures. (Level 2a3b evidence, Grade C recommendation)

Summary Table
Question

Year

References

Level of
Evidence

Grade of
Recommendation

Findings

What nonoperative therapies are


effective in the treatment of anal
fissures?

2005

1, 2

1a

Better than placebo;


recurrence common.

What are the results of LIS for


fissures?

2005

1, 3

1a

LIS is treatment of choice.

How are anal strictures best treated?

2010

10

2c

House flap seems to work


best.

What is the role of antibiotics in the


treatment of perirectal abscesses?

2008

1, 2

2c

Not indicated for majority


of patients.

What are the best operative


strategies for drainage of
perirectal abscess?

2006

1, 3

2c

Incision and drainage


mainstay of treatment.
(Continued)

PMPH_CH35.indd 296

5/22/2012 5:25:46 PM

Anorectal Fissure, Stricture, Abscess, and Fistula

297

(Continued)
Question

Year

References

Following drainage of a perirectal


abscess, can the development of
a fistula-in-ano be predicted or
prevented?

2009

2a

Nondiabetics and patients


<40 higher risk.

What are the best operative


approaches to simple fistula-in-ano?

2010

1, 5

2b

Fistulotomy appropriate
for simple fissures.

How are setons used in the treatment


of anal fistulas?

2009

7, 8

2c3b

Setons are appropriate


for some patients.

What are the results of various


sphincter-sparing techniques in the
treatment of fistula-in-ano?

2010

1, 916, 26

2a3b

Success rates vary,


incontinence rare.

REFERENCES
1. Perry WB, Dykes SL, Buie WD, et al. Practice parameters for the
management of anal fissures (3rd revision). Dis Colon Rectum.
2010;53(8):1110-1115.
2. Nelson R. Non surgical therapy for anal fissure. Cochrane Database Syst Rev. 2006;(4):CD003431. Review.
3. Nelson R. Operative procedures for fissure in ano. Cochrane
Database Syst Rev 2005;(2):CD002199.
4. Elsebae MM. A study of fecal incontinence in patients with
chronic anal fissure: Prospective, randomized, controlled trial
of the extent of internal anal sphincter division during lateral
sphincterotomy. World J Surg. 2007;31:2052-2057.
5. Abcarian H. Surgical correction of chronic anal fissure: Results
of lateral internal sphincterotomy vs. fissurectomy-midline
sphincterotomy. Dis Colon Rectum. 1980;23:31-36.
6. Leong AF, Seow-Choen F. Lateral sphincterotomy compared
with anal advancement flap for chronic anal fissure. Dis Colon
Rectum. 1995;38:69-71.
7. Singh M, Sharma A, Gardiner A, et al. Early results of a rotational flap to treat chronic anal fissures. Int J Colorectal Dis.
2005;20:339-342.
8. Christensen MA, Pitsch RM, Jr., Cali RL, et al. House advancement pedicle flap for anal stenosis. Dis Colon Rectum. 1992;35:
201-203.
9. Sentovich SM, Falk PM, Christensen MA, et al. Operative results
of House advancement anoplasty. Br J Surg. 1996;83:1242-1244.
10. Farid M, Youssef M, El Nakeeb A, et al. Comparative study of
the house advancement flap, rhomboid flap, and Y-V anoplasty
in treatment of anal stenosis: A prospective randomized study.
Dis Colon Rectum. 2010;53:790-797.
11. Whiteford MH, Kilkenny J, 3rd, Hyman N, et al. Practice parameters for the treatment of perianal abscess and fistula-in-ano
(revised). Dis Colon Rectum. 2005;48(7):1337-1342.
12. Wilson W, Taubert KA, Gewitz M, et al. Prevention of infective
endocarditis: Guidelines from the American Heart Association: A
guideline from the American Heart Association Rheumatic Fever,
Endocarditis and Kawasaki Disease Committee, Council on Cardiovascular Disease in the Young, and the Council on Clinical
Cardiology, Council on Cardiovascular Surgery and Anesthesia,
and the Quality of Care and Outcomes Research Interdisciplinary
Working Group. J Am Dent Assoc. 2008;139 Suppl:3S-24S. Review.
13. Rosen SA, Colquhoun P, Efron J, et al. Horseshoe abscesses and
fistulas: How are we doing? Surg Innov. 2006;13(1):17-21.

PMPH_CH35.indd 297

Level of
Evidence

Grade of
Recommendation

Findings

14. Stremitzer S, Strobl S, Kure V, et al. Treatment of perianal sepsis


and long-term outcome of recurrence and continence. Colorectal
Dis. 2010. [Epub ahead of print]
15. Hyman N, OBrien S, Osler T. Outcomes after fistulotomy:
Results of a prospective, multicenter regional study. Dis Colon
Rectum. 2009;52(12):2022-2027.
16. Ritchie RD, Sackier JM, Hodde JP. Incontinence rates after cutting seton treatment for anal fistula. Colorectal Dis. 2009;11(6):
564-571.
17. Malik AI, Nelson RL. Surgical management of anal fistulae: A
systematic review. Colorectal Dis. 2008;10(5):420-430.
18. Singer M, Cintron J, Nelson R, et al. Treatment of fistulas-in-ano
with fibrin sealant in combination with intra-adhesive antibiotics and/or surgical closure of the internal fistula opening. Dis
Colon Rectum. 2005;48(4):799-808.
19. McGee MF, Champagne BJ, Stulberg JJ, et al. Tract length predicts successful closure with anal fistula plug in cryptoglandular
fistulas. Dis Colon Rectum. 2010;53(8):1116-1120.
20. Schwandner T, Roblick MH, Kierer W, et al. Surgical treatment
of complex anal fistulas with the anal fistula plug: A prospective,
multicenter study. Dis Colon Rectum. 2009;52(9):1578-1583.
21. Ortiz H, Marzo J, Ciga MA, et al. Randomized clinical trial of anal
fistula plug versus endorectal advancement flap for the treatment of
high cryptoglandular fistula in ano. Br J Surg. 2009;96(6):608-612.
22. Christoforidis D, Pieh MC, Madoff RD, et al. Treatment of transsphincteric anal fistulas by endorectal advancement flap or
collagen fistula plug: A comparative study. Dis Colon Rectum.
2009;52(1):18-22.
23. Bleier JI, Moloo H, Goldberg SM. Ligation of the intersphincteric
fistula tract: An effective new technique for complex fistulas. Dis
Colon Rectum. 2010;53(1):43-46.
24. Shanwani A, Nor AM, Amri N. Ligation of the intersphincteric
fistula tract (LIFT): A sphincter-saving technique for fistula-inano. Dis Colon Rectum. 2010;53(1):39-42.
25. Ellis CN. Outcomes with the use of bioprosthetic grafts to reinforce the ligation of the intersphincteric fistula tract (BioLIFT
procedure) for the management of complex anal fistulas. Dis
Colon Rectum. 2010;53(10):1361-1364.
26. Hamadani A, Haigh PI, Liu IL, et al. Who is at risk for developing chronic anal fistula or recurrent anal sepsis after initial
perianal abscess? Dis Colon Rectum. 2009;52(2):217-221.
27. Champagne BJ, OConnor LM, Ferguson M, et al. Efficacy of
anal fistula plug in closure of cryptoglandular fistulas: Longterm follow-up. Dis Colon Rectum. 2006;49(12):1817-1821.

5/22/2012 5:25:46 PM

Commentary on Anorectal Fissure,


Stricture, Abscess, and Fistula
Stanley M. Goldberg

W. Brian Perry and Joshua A. Tyler had a large topic to summarize


and they have reached some excellent conclusions. Their advice
on the management of anal fissure is spot-on. I have read all this
literature and have tried all the new therapies, but a properly performed lateral internal sphincterotomy with minimal division of
the sphincter muscle is the treatment of choice for a chronic anal
fissure. The surgeons should never incise muscle cephalad to the
proximal portion of the fissure.
Fortunately, anal strictures are rare in the United States
because most colon and rectal surgeons perform a closed hemorrhoidectomy and are very cautious regarding the removal of anoderm. Because of the paucity of this problem, good randomized
studies are not available.
I was especially pleased to see their discussions regarding the
use of antibiotic in the management of perirectal abscesses. I only
wish all the family practitioners and emergency physicians could
read this chapter.

The discussion of setons was excellent and this technique


should be utilized whenever a surgeon is in doubt as to how much
muscle is involved with the fistula or if there is any indication that
a patient may have Crohns disease.
The authors were correct in their assessment of the current
method to management complex fistulas. More prospective trials
are indicated, but I think LIFT (Lateral Intersphincteric Fistula
Transection) will win out in the end because no foreign material
is needed and no sphincter muscle is divided.1

REFERENCE
1. Bleier J, Moloo H, Goldberg SM. Ligation of the intersphincteric
fistulas tract: An effective new technique for complex fistulas. Dis
Colon Rectum. 2010;53(1):43-46.

298

PMPH_CH35.indd 298

5/22/2012 5:25:46 PM

CHAPTER 36

Fecal Incontinence and Surgical


Management of Constipation
Sarah Pesek and Neil Hyman

INTRODUCTION

Factors that contribute to fecal incontinence can be noted


while taking a past medical history. Obstetrical trauma is the
most common contributing factor in adult females, though incontinence may not manifest until years afterward. There is increased
risk with instrumental delivery, large birth weight, and prolonged
second stage of labor. Up to 35% of primiparas have sphincter
injury though some remain asymptomatic.6 Trauma also may
lead to incontinence in the form of pelvic fracture leading to nerve
damage or by direct sphincter injury. Furthermore, spinal injuries
may also contribute. Neurologic damage from diabetes and stroke,
and neurologic disorders such as multiple sclerosis and dementia
and mental disorders may also lead to incontinence.7
Several scoring systems have been developed to assess the
severity of fecal incontinence. Grading systems can be used to
objectify and measure improvement or worsening of fecal incontinence after treatment or intervention.5 Two primary domains are
considered: the frequency and type of stool loss and the impact of
fecal incontinence on daily life.8 The most commonly used scale is
the Cleveland Clinic Florida (Wexner) fecal incontinence score.9
Initial treatment begins with medical management. Dietary
and lifestyle modifications are effective for patients with mild symptoms. Foods that cause urgency or diarrhea should be avoided.7
Fecal incontinence commonly is made worse by liquid stools or
diarrhea. Fiber supplementation has been shown to decrease the
proportion of incontinent stools in patients with loose or liquid
stools by more than 50%.10
Loperamide (Imodium299) reduces stool weight, small bowel
motility, urgency, stool volume, and frequency of bowel movements.11 In two studies in patients with chronic diarrhea, loperamide use results in improved incontinence symptoms when
compared with placebo. More patients on diphenoxylate for chronic
diarrhea achieved full continence when compared with placebo.
However, loperamide may be superior to diphenoxylate.12
Biofeedback is another method used to improve bowel control.
Three main modalities have been described: rectal sensitivity training, strength training, and co-ordination training. One small trial
showed significant improvement in incontinent episodes per week
and overall incontinence status in patients receiving biofeedback

Constipation is a symptom-based disorder of unsatisfactory defecation. It involves infrequent stools, difficult stool passage, or
a combination of the two. More than 2 million clinic visits per
year can be attributed to constipation. Prevalence has been estimated between 1.9% and 27.2% in North America with an average
of 14.2%.1 Constipation can be a symptom of other disease processes, a side effect of medications, or inadequate dietary habits,
or have no identifiable cause. It can be classified as normal transit,
obstructed defecation, or slow transit.2
Fecal incontinence is the loss of voluntary control of feces,
either liquid or solid, from the bowel. It is a condition that is
socially embarrassing and isolating and those affected often need
to plan their lives around the disorder. The true prevalence of
the disorder is unknown and estimates vary greatly because of
the influence of social stigma, varying the defi nition of incontinence, and varying frequencies of occurrence. In a survey of
the general population, 12.1% of women had an episode of fecal
incontinence in the past year with 2.5% of women having one or
more episodes per week.3 In general, fecal incontinence is more
common in women and institutionalized patients and its prevalence increases with age.4

FECAL INCONTINENCE
1. What is the initial evaluation and management of patients
with fecal incontinence?
The evaluation of fecal incontinence begins with a careful history and physical examination. The type of incontinence, whether
it is to gas, liquid stool, or solid stool, should be ascertained as
well as the frequency at which it occurs. In addition, the use of
coping mechanisms, such as wearing a pad or a diaper, and lifestyle changes should be assessed. Physical examination includes
anal inspection and digital rectal examination. Both are poor for
detecting sphincter defects.5
299

PMPH_CH36.indd 299

5/22/2012 5:26:21 PM

300

Surgery: Evidence-Based Practice

compared with controls.13 However, a 2006 Cochrane Review


found no evidence that biofeedback offers any advantage over
other conservative managements.14
Answer: The initial evaluation of fecal incontinence begins
with a history and physical examination. Initial management is
medical, including fiber and loperamide. Biofeedback may
improve symptoms. (Grade B recommendation)
2. What is the workup for patients being considered for surgery to treat fecal incontinence?
Endoanal ultrasound can be used to diagnose sphincter defects in
patient with suspected injury. In identifying internal and external sphincter defects, it has sensitivity and specificity approaching 100% when performed by an experienced clinician.5 However,
the presence of a sphincter defect does not always correlate with
symptoms of incontinence. In a study of 335 incontinent patients,
115 continents and 18 asymptomatic volunteers, defects were
found in 65%, 43%, and 22%, respectively.15
Anal manometry can be used to define the functional weakness of the internal or external anal sphincters and the length of
the high pressure zone of the anal canal. Patients with incontinence have been shown to have low resting and low squeeze
pressures, but there are significant variations even within asymptomatic subjects dependent on age, gender, and parity.5,7 Studies
have shown that preoperative manometric readings do not correlate with outcomes following anal sphincter repair and peripheral
nerve evaluation, but can predict the likelihood of restoration of
continence following repair of rectal prolapse.16-19
The pudendal nerve terminal motor latency (PNTML) measures neuromuscular integrity between the terminal portion of
the pudendal nerve and the anal sphincter. Bilateral, not unilateral, prolonged PNTML is associated with poorer function in the
incontinent patient with the intact sphincter, but the majority
of incontinent patients have normal PNTML.20 The presence of
pudendal neuropathy has been proposed to be a factor predicting
the success following anal sphincter repair.21 Laurberg et al. analyzed a group of 19 patients and saw an 80% success rate in those
without neuropathy, compared with only 11% with neuropathy
(p < .05).22 This finding has been confirmed by other studies. However, pudendal neuropathy is not universally accepted as a predictor of failure following repair.21
Answer: Endoanal ultrasound is useful for diagnosing anal
sphincter defects. Anal manometry readings may be used to define
functional weakness. PNTML studies may predict the outcome of
anal sphincter repair. (Grade B recommendation)
3. What are the indications and outcomes for anal sphincter
repair?
Anal sphincter repair can be done for any type of injury of the
external anal sphincter. Anterior reconstruction is most commonly reported, because obstetrical injury is the most frequent
indication and most outcome data pertain to this type of patient.
Good-to-excellent results have been suggested in 31% to 83% of
patients.5 However, good results tend to deteriorate over time. In
one study, less than 10% were completely continent to liquid and
solid stools for 5 years.23 In another, only 14% were fully continent at a median of 69 months.24 In a third study, no patients were
found to be fully continent after 10 years.25 For patients undergoing repeat anal sphincter repair, the short-term and long-term
results are similar to those without a previous repair.26

PMPH_CH36.indd 300

When overlapping sphincter repair was compared with direct


end-to-end repair in a randomized clinical trial, there was no benefit
noted with an overlapping sphincteroplasty.27 Similarly, in another
nonrandomized study, outcomes were similar between end-toend, overlapping, and plication techniques.28 Levatorplasty can be
performed at the same time as anal sphincter repair. In one study
when performed simultaneously, 63% of patients reported good-toexcellent results.29 In another, 77% report subjective improvement.30
These results are comparable with anal sphincter repair alone, but
no studies exist directly comparing the two methods.
Answer: Anal sphincter repair is indicated for properly
selected patients with external anal sphincter defects. Short-term
results are reasonable but deteriorate over time. The role of levatorplasty is unclear. (Grade A recommendation)
4. Is there a role for an artificial sphincter or a sacral nerve
stimulator (SNS)?
Artificial sphincters were initially developed to treat urinary
incontinence and were modified and adapted for treatment of fecal
incontinence.31 The devise is placed around the existing sphincter complex after perineal dissection. For patients that retain the
device, artificial bowel sphincter provides good continence of
solid and liquid stools. Complete continence has been achieved
in up to 63% with 79% of patients continent to both solid and liquid stools.32 However, the complication rate is high and adverse
events are seen in virtually all patients.33 Revisional surgery was
required in 46% of patients and the explantation rate is 20% to
30%.5,32 Absolute contraindications to this procedure include
active perineal sepsis, Crohns disease, radiation proctitis, severe
scarring in the perineum, or anoreceptive intercourse.5
Candidates for SNS typically have an intact external sphincter
(with or without previous repair), symptoms of fecal incontinence
(1+ episodes per week), failed conservative therapy, and be able to
follow up reliably.34 SNS is a two-stage procedure with a diagnostic
peripheral nerve evaluation stage followed by a permanent therapeutic implantation stage. Patients receive the permanent implant
only if the diagnostic stage produces clinical improvement, defined
as a reduction in the frequency of fecal incontinence by at least
50% over a 2-week period.35
For patients that achieve chronic implantation, 83% achieve
50% reduction in episodes of incontinence at 1 year. At 3 years,
87% meet this goal. Approximately 40% of patients have perfect
continence at 1 year.36 Interventions to optimize function are common and include reprogramming of the device and repositioning of
leads.37 The incidence of complications range from 5% to 26% in various studies.5 The most common adverse effect is pain at the implantation site.36,37 The screening test has a low complication rate.36
Answer: Artificial bowel sphincters are effective in treating
fecal incontinence in patients who retain the device; however,
there is a high complication rate. SNS is a promising intervention
for fecal incontinence. Both modalities remain largely experimental at the present time. (Grade B recommendation)

CONSTIPATION
5. What treatments are effective in the medical management
of constipation?
Management of symptomatic constipation typically begins with
dietary modification, which includes a high-fiber diet and fluid

5/22/2012 5:26:21 PM

Fecal Incontinence and Surgical Management of Constipation

supplementation. Physical activity may also be helpful.38 A graded


increase of up to 25 g per day of fiber has been suggested.39 This
dose has been shown to increase stool frequency. Increasing fluid
intake to 1.5 to 2 L per day has also been shown to increase stool
frequency and decreases the need for laxatives in those already
consuming a high-fiber diet.40 However, for patients with slowtransit constipation or pelvic floor dysfunction (PFD), fewer than
30% improve.41
When dietary management fails, laxatives and other drugs
may be used.38 Polyethylene glycol (PEG) can be used to promote
bowel function. A meta-analysis reviewing studies comparing
PEG with placebo showed a significant increase in stools per
week and a good tolerability to the drug in general.42 A Cochrane
review comparing PEG and lactulose found PEG to be better in
terms of stools per week, improvement in abdominal pain and
the need for additional products.43 Tegaserod has been shown to
increase spontaneous bowel movements, but has been withdrawn
from general use due to its increasing cardiovascular risk.39
Lubiprostone has been shown to have an effect in both acute and
chronic treatments. When used in a 4-week study, it was found
to increase the number of spontaneous bowel movement when
compared with placebo at each weekly time point.44 Similarly,
a meta-analysis looking at studies lubiprostone showed benefit
when compared with placebo for treatment of constipation.45
However, adverse effects are frequent with nausea occurring in
up to 31% of patients.44
Other common agents, such as milk of magnesia, senna,
bisacodyl, and stool softeners, may be used for chronic constipation. However, inconsistent results are reported in the literature.46
Additionally, long-term laxative use can result in the patient developing a cathartic colon.
Answer: Initial management of constipation begins with
dietary management. If this fails, PEG and lubiprostone have been
shown to increase stool frequency. (Grade A recommendation)
6. What role do anorectal physiology and colonic transit time
studies have in assessment of constipation?
A colonic transit study documents gastrointestinal motility
objectively. The pattern of radio-opaque markers can distinguish
between colonic inertia and PFD. The test is simple, inexpensive,
and repeatable in assessing slow-transit constipation.38 When slow
colonic transit has been adequately documented before colectomy
for refractory constipation, an improved rate of good outcomes
has been reported (90% vs. 67%).47
Defecography provides a dynamic evaluation of the entire
defecation process. It can identify abnormalities such as rectocele, sigmoidocele, abnormal perineal descent, nonrelaxation of the puborectalis, intussusception, occult prolapse, and
incomplete emptying of the rectum.48 However, the relevance
of defocography findings is controversial as many normal individuals show abnormal findings.49 Anorectal manometry may
show paradoxical sphincter contraction in patients with anismus or with obstructed defecation, but this may also be seen
in normal patients. 50 Hirschsprungs disease may be suggested
by an absent rectoanal inhibitory reflex. 38 Electromyography
may be used to evaluate puborectalis relaxation and is used to
identify patients with paradoxical puborectalis contraction or
nonrelaxation.48
A balloon expulsion test quickly estimates the overall pelvic floor function and can be used to exclude PFD. A prospective

PMPH_CH36.indd 301

301

study showed it to have a specificity and negative predictive value for excluding PFD of 89% and 97%, respectively.
A nonpathologic test may avoid the use of other pelvic f loor
investigations. 51
Answer: Anorectal physiology and colonic transit time
studies may serve to differentiate between slow transit and outlet obstruction and identify the underlying pathophysiology in
patients who fail to improve with dietary management. (Grade B
recommendation)
7. What are the selection criteria and outcomes for colectomy
in slow-transit constipation?
Colectomy is only considered for patients after diet and medical interventions have failed and following a thorough workup.
Candidates for total abdominal colectomy have slow-transit constipation without concomitant PFD. Patients with documented
slow colonic transit before colectomy reported an improved
rate of good outcomes. In general, patients who undergo complete physiologic evaluation with manometry, defecography,
and colonic transit studies showed improved median satisfaction rate when compared with incomplete evaluation, 89%
vs. 80%.47
Total abdominal colectomy with ileorectal anastomosis yields
a clinical improvement in 50% to 100% of patients with slow-transit
constipation.52 Segmental colonic resection has disappointing
results with up to a 100% failure rate.47 Factors that predict failure
include psychiatric factors, PFD, and abnormal small bowel transit.
Patients with generalized intestinal dysmotility have a diminished
long-term success rate of 13%.53
Complications occur in approximately one-third of patients.2
They include small bowel obstruction, persisting abdominal pain,
frequent bowel movements, fecal incontinence, and persistent or
recurring constipation. Fecal incontinence was found in 6% of
patients following total abdominal colectomy.52
Answer: Total abdominal colectomy with ileorectal anastomosis should be cautiously considered for patients with slowtransit constipation who have failed nonsurgical management. A
careful physiologic evaluation should be performed prior to surgery. (Grade B recommendation)
8. What is the management of PFD?
Patients with nonrelaxing or paradoxical puborectalis are unable
to relax the pelvic floor appropriately at straining or Valsalva. Pelvic floor exercises and biofeedback can be used for these patients.
Success rates of biofeedback have been reported that range
between 30% and 90% with no technique having a superior success rate.54,55 A randomized clinical trial of patients with severe
PFD who failed initial medical management compared the outcome of five weekly biofeedback sessions with that of PEG)and
five weekly sessions of constipation prevention. Eighty percent in
the biofeedback group reported major improvement which was
significantly greater than the PEG group. Results were sustained
at 12 and 24 months.56 A retrospective study showed that the outcome of biofeedback was not influenced by age, gender, duration
of symptoms, or rectal pain. Outcomes improved if the patient
completed five sessions and were related to the patients willingness to complete treatment.57
Answer: Biofeedback is an appropriate treatment for PFD.
(Grade B recommendation)

5/22/2012 5:26:21 PM

302

Surgery: Evidence-Based Practice

9. What is the role of surgery for rectocele with an associated


defecation disorder?
A rectocele results from the breakdown of endopelvic fascial support in the rectovaginal septum. Repair is generally indicated in
patients where a symptomatic rectocele is large, some say >4 cm, if
it fails to empty on defocography and if manual support of the vaginal wall or rectum is necessary for satisfactory defecation.38,58
Approaches for rectocele repair include transvaginal, transperineal, and transanal repair. The transvaginal repair is traditionally performed by gynecologists. It has been shown to correct
the anatomic defect in 76% to 96% of patients.59 Improvement in
defecation symptoms has been reported in up to 90% of patients.2
However, there is a failure to relieve evacuatory difficulty or lower
rectal symptoms in 33%. Fecal incontinence is found in 36%, and
25% suffer dyspareunia postoperatively. Some authors recommend the use of defect-specific repair to reduce the risk of dyspareunia, which has been shown to reduce constipation in more
than 80% of patients.38
The transanal repair is traditionally used by colorectal surgeons. Outcomes of this method include decreased constipation
in 48% to 71% and correction of need for digital assistance of defecation in 54% to 100%.38 A Cochrane review of two small trials
comparing the transanal and transvaginal approaches showed the

transvaginal approach to have lower subjective and objective failure rates and lower rate of postoperative enterocele.60 No difference in rates of incontinence or dyspareunia was observed.
The transperineal approach can be performed in combination with anal sphincter repair and/or levatorplasty for symptomatic rectocele and sphincter defect. An improvement in evacuation
and continence has been seen in 75% of patients.61 Mesh can be
inserted via a transvaginal or a transperineal approval to reduce
the risk of recurrence and dyspareunia. However, this has not
been shown to be more effective than traditional repair.60
The stapled transanal rectal resection (STARR) procedure has
been advocated for patients with obstructive defecation syndrome
(ODS) with internal rectal prolapsed with or without rectocele.62
In an analysis of two large STARR registries in Europe, a significant improvement in ODS symptoms at 6 months that was maintained at 12 months was found. Complications occurred in 21%
to 36% of patients and included staple line complications, major
bleeding, and postsurgical stenosis. Defecatory urgency and newonset incontinence were also observed.63,64
Answer: Surgical repair of rectocele can be performed via
a transvaginal, transanal, or transperineal repair. Encouraging
results have been seen in patients who underwent a STARR procedure for ODS. (Grade B recommendation)

Clinical Question Summary


Question

Answer

1 What is the initial evaluation and


management of patients with fecal
incontinence?

Evaluation consists of a focused history and


physical examination. Initial management is
medical in most cases.

5-14

2 What is the workup for patients being


considered for surgery to treat fecal
incontinence?

Endoanal ultrasound can diagnose anal


sphincter defects. Anal manometry and
PNTML may predict the outcome of repair.

5, 7, 15-22

3 What are the indications and outcomes


for anal sphincter repair?

Repair is indicated for highly selected patients


with sphincter defects. Short-term results
are reasonable.

5, 23-30

4 Is there a role for an artificial sphincter


or a sacral nerve stimulator (SNS)?

Both appear effective in treating fecal


incontinence, but remain largely
experimental.

31-37

5 What treatments are effective in the


medical management of constipation?

Management begins with dietary changes. PEG


and lubiprostone may be used if this fails.

38-46

6 What role do anorectal physiology and


colonic transit time studies have in
assessment of constipation?

In patients who fail dietary management, they


may help differentiate between slow-transit
and outlet obstruction.

47-51

7 What are the selection criteria and


outcomes for colectomy in slow-transit
constipation?

Colectomy should be considered for


refractory slow-transit constipation in
highly selected patients.

2, 47, 51-53

8 What is the management of PFD?

Biofeedback is appropriate treatment for


highly symptomatic patients.

54-57

9 What is the role of surgery for rectocele


with an associated defecation disorder?

Symptomatic rectoceles can be repaired via


transanal, transvaginal or transperineal
approaches.

38, 58-64

PMPH_CH36.indd 302

Grade of
Recommendation

References

5/22/2012 5:26:21 PM

Fecal Incontinence and Surgical Management of Constipation

REFERENCES
1. Higgins PD, Johanson JF. Epidemiology of constipation in
North America: A systematic review. Am J Gastroenterol.
2004;99(4):750-759.
2. Steele SR, Mellgren A. Constipation and obstructed defecation.
Clin Colon Rectal Surg. 2007;20(2):110-117.
3. Bharucha AE, Zinsmeister AR, Locke GR, et al. Prevalence
and burden of fecal incontinence: A population-based study in
women. Gastroenterology. 2005;129(1):42-49.
4. Tan JJ, Chan M, Tjandra JJ. Evolving therapy for fecal incontinence. Dis Colon Rectum.2007;50(11):1950-1967.
5. Tjandra JJ, Dykes SL, Kumar RR, et al. Practice parameters
for the treatment of fecal incontinence. Dis Colon Rectum.
2007;50(10):1497-1507.
6. Kamm MA. Obstetric damage and faecal incontinence. Lancet.
1994;344(8924):730-733.
7. Rao SS. Diagnosis and management of fecal incontinence. American College of Gastroenterology Practice Parameters Committee. Am J Gastroenterol. 2004;99(8):1585-1604.
8. Rockwood TH. Incontinence severity and QOL scales for fecal
incontinence. Gastroenterology. 2004;126(1 Suppl 1):S106-S113.
9. Jorge JM, Wexner SD. Etiology and management of fecal incontinence. Dis Colon Rectum. 1993;36(1):77-97.
10. Bliss DZ, Jung Hj, Savik K, et al. Supplementation with dietary
fiber improves fecal incontinence. Nurs Res. 2001;50(4):203-213.
11. Scarlett Y. Medical management of fecal incontinence. Gastroenterology. 2004;126(1 Suppl 1):S55-S63.
12. Cheetham M, Brazzelli M, Norton C, et al. Drug treatment for
faecal incontinence in adults. Cochrane Database Systematic
Review. 2003(3):CD002116.
13. Miner PB, Donnelly TC, Read NW. Investigation of mode of
action of biofeedback in treatment of fecal incontinence. Dig Dis
Sci. 1990;35(10):1291-1298.
14. Norton C, Cody JD, Hosker G. Biofeedback and/or sphincter exercises for the treatment of faecal incontinence in adults.
Cochrane Database Systematic Review. 2006;3:CD002111.
15. Karoui S, Savoye-Collet C, Koning E, et al. Prevalence of anal
sphincter defects revealed by sonography in 335 incontinent
patients and 115 continent patients. AJR Am J Roentgenol. 1999;
173(2):389-392.
16. Gearhart S, Hull T, Floruta C, et al., Anal manometric parameters: Predictors of outcome following anal sphincter repair?
J Gastrointest Surg. 2005;9(1):115-120.
17. Maeda Y, Norton C, Lundby L, et al. Predictors of the outcome of
percutaneous nerve evaluation for faecal incontinence. Br J Surg.
2010;97(7):1096-1102.
18. Zutshi M, Salcedo L, Hammel J, et al. Anal physiology testing
in fecal incontinence: is it of any value? Int J Colorectal Dis.
2010;25(2):277-282.
19. Kouraklis G, Andromanakos N. Evaluating patients with anorectal incontinence. Surg Today. 2004;34(4):304-312.
20. Ricciardi R, Mellgren AF, Madoff RD, et al. The utility of pudendal nerve terminal motor latencies in idiopathic incontinence.
Dis Colon Rectum. 2006;49(6):852-857.
21. Baig MK, Wexner SD. Factors predictive of outcome after surgery for faecal incontinence. Br J Surg. 2000;87(10):1316-1330.
22. Laurberg S, Swash M, Snooks SJ, et al. Neurologic cause of idiopathic incontinence. Arch Neurol. 1988;45(11):1250-1253.
23. Malouf AJ, Norton CS, Engel AF, et al. Long-term results of overlapping anterior anal-sphincter repair for obstetric trauma. Lancet.
2000;355(9200):260-265.

PMPH_CH36.indd 303

303

24. Halverson AL, Hull TL. Long-term outcome of overlapping anal


sphincter repair. Dis Colon Rectum. 2002;45(3):345-348.
25. Zutshi M, Tracey TH, Bast J, et al. Ten-year outcome after anal
sphincter repair for fecal incontinence. Dis Colon Rectum.
2009;52(6):1089-1094.
26. Vaizey CJ, Norton C, Thornton MJ, et al. Long-term results
of repeat anterior anal sphincter repair. Dis Colon Rectum.
2004;47(6):858-863.
27. Tjandra JJ, Han WR, Goh J, et al. Direct repair vs. overlapping
sphincter repair: A randomized, controlled trial. Dis Colon Rectum. 2003;46(7):937-942; discussion 942-943.
28. Morren GL, Hallbook O, Nystrom PO, et al. Audit of analsphincter repair. Colorectal Dis. 2001;3(1):17-22.
29. Evans C, Davis K, Kumar D. Overlapping anal sphincter repair
and anterior levatorplasty: Effect of patients age and duration of
follow-up. Int J Colorectal Dis. 2006;21(8):795-801.
30. Davis KJ, Kumar D, Poloniecki J. Adjuvant biofeedback following
anal sphincter repair: A randomized study. Aliment Pharmacol
Ther. 2004;20(5):539-549.
31. Mellgren A. Fecal incontinence. Surg Clin North Am. 2010;90(1):
185-194.
32. Michot F, Costaglioli B, Leroi AM, et al. Artificial anal sphincter in severe fecal incontinence: Outcome of prospective experience with 37 patients in one institution. Ann Surg. 2003;237(1):
52-56.
33. Carmona R, Alos Company R, Roig Vila JV, et al. Long-term results
of artificial bowel sphincter for the treatment of severe faecal incontinence. Are they what we hoped for? Colorectal Dis. 2009;11(8):
831-837.
34. Jarrett ME, Mowatt G, Glazener CM, et al. Systematic review of
sacral nerve stimulation for faecal incontinence and constipation. Br J Surg. 2004;91(12):1559-1569.
35. Tjandra JJ, Lim JF, Matzel K. Sacral nerve stimulation: An emerging treatment for faecal incontinence. ANZ J Surg. 2004;74(12):
1098-1106.
36. Wexner SD, Coller JA, Devroede G, et al. Sacral nerve stimulation for fecal incontinence: Results of a 120-patient prospective
multicenter study. Ann Surg. 2010;251(3):441-449.
37. Maeda Y, Lundby L, Buntzen S, et al. Suboptimal outcome following sacral nerve stimulation for faecal incontinence. Br J Surg.
2011;98(1):140-147. [Epub October 27, 2010.]
38. Ternent CA, Bastawrous AL, Morin NA, et al. Practice parameters for the evaluation and management of constipation. Dis
Colon Rectum. 2007;50(12):2013-2022.
39. Foxx-Orenstein AE, McNally MA, Odunsi ST. Update on constipation: One treatment does not fit all. Cleve Clin J Med. 2008;75(11):
813-824.
40. Anti M, Pignataro G, Armuzzi A, et al. Water supplementation
enhances the effect of high-fiber diet on stool frequency and
laxative consumption in adult patients with functional constipation. Hepatogastroenterology. 1998;45(21):727-732.
41. Voderholzer WA, Schatke W, Muhldorfer BE, et al. Clinical
response to dietary fiber treatment of chronic constipation. Am J
Gastroenterol. 1997;92(1):95-98.
42. Belsey JD, Geraint M, Dixon TA. Systematic review and meta
analysis: Polyethylene glycol in adults with non-organic constipation. Int J Clin Pract. 2010;64(7):944-955.
43. Lee-Robichaud H, Thomas K, Morgan J, et al. Lactulose versus
Polyethylene Glycol for Chronic Constipation. Cochrane Database Systematic Review. 2010(7):CD007570.
44. McKeage K, Plosker GL, Siddiqui MA. Lubiprostone. Drugs.
2006;66(6):873-879.

5/22/2012 5:26:21 PM

304

Surgery: Evidence-Based Practice

45. Ford AC, Suares NC. Effect of laxatives and pharmacological


therapies in chronic idiopathic constipation: Systematic review
and meta-analysis. Gut. 2011;60(2):209-218.
46. Ramkumar D, Rao SS. Efficacy and safety of traditional medical therapies for chronic constipation: Systematic review. Am
J Gastroenterol. 2005;100(4):936-971.
47. Knowles CH, Scott M, Lunniss PJ. Outcome of colectomy for
slow transit constipation. Ann Surg. 1999;230(5):627-638.
48. Denoya P, Sands DR. Anorectal physiologic evaluation of constipation. Clin Colon Rectal Surg. 2008;21(2):114-1121.
49. Savoye-Collet C, Koning E, Dacher JN. Radiologic evaluation of
pelvic floor disorders. Gastroenterol Clin North Am. 2008;37(3):
553-567, viii.
50. Voderholzer WA, Neuhaus DA, Klauser AG, et al. Paradoxical sphincter contraction is rarely indicative of anismus. Gut.
1997;41(2):258-262.
51. Minguez M, Herreros B, Sanchiz V, et al. Predictive value of
the balloon expulsion test for excluding the diagnosis of pelvic
floor dyssynergia in constipation. Gastroenterology. 2004;126(1):
57-62.
52. Pikarsky AJ, Singh JJ, Weiss EG, et al. Long-term follow-up of
patients undergoing colectomy for colonic inertia. Dis Colon
Rectum. 2001;44(2):179-183.
53. Redmond JM, Smith GW, Barofsky I, et al. Physiological tests
to predict long-term outcome of total abdominal colectomy
for intractable constipation. Am J Gastroenterol. 1995;90(5):
748-753.
54. Jorge JM, Habr-Gama A, Wexner DS. Biofeedback therapy in
the colon and rectal practice. Appl Psychophysiol Biofeedback.
2003;28(1):47-61.

PMPH_CH36.indd 304

55. Khaikin M, Wexner SD. Treatment strategies in obstructed defecation and fecal incontinence. World J Gastroenterol. 2006;12(20):
3168-3173.
56. Chiarioni G, Whitehead WE, Pezza V, et al. Biofeedback is superior to laxatives for normal transit constipation due to pelvic
floor dyssynergia. Gastroenterology. 2006;130(3):657-664.
57. Gilliland R, Heymen S, Altomare DF, et al. Outcome and predictors of success of biofeedback for constipation. Br J Surg. 1997;84(8):
1123-1126.
58. Zbar AP, Lienemann A, Fritsch H, et al. Rectocele: Pathogenesis
and surgical management. Int J Colorectal Dis. 2003;18(5):369-384.
59. Kudish BI, Iglesia CB. Posterior wall prolapse and repair. Clin
Obstet Gynecol. 2010;53(1):59-71.
60. Maher C, Feiner B, Baessler K, et al. Surgical management of
pelvic organ prolapse in women. Cochrane Database Systematic
Review. 2010(4):CD004014.
61. Ayabaca SM, Zbar AP, Pescatori M. Anal continence after rectocele repair. Dis Colon Rectum. 2002;45(1):63-69.
62. Schwandner O, Stuto A, Jaybe D, et al. Decision-making algorithm for the STARR procedure in obstructed defecation syndrome: Position statement of the group of STARR Pioneers. Surg
Innov. 2008;15(2):105-109.
63. Schwandner O, Furst A. Assessing the safety, effectiveness, and
quality of life after the STARR procedure for obstructed defecation: Results of the German STARR registry. Langenbecks Arch
Surg. 2010;395(5):505-513.
64. Jayne DG, Schwandner O, Stuto A. Stapled transanal rectal resection for obstructed defecation syndrome: One-year results of the
European STARR Registry. Dis Colon Rectum. 2009;52(7):12051212; discussion 1212-1214.

5/22/2012 5:26:21 PM

CHAPTER 37

Rectovaginal Fistula
Joshua D. Schulte, Kelly Ming, Michelle M. Olsen, and Philip F. Caushaj

obstructed delivery.4,5 Primiparity, midline episiotomies, increased


birth weight, and use of vaginal forceps are the confounding factors that along with the perineal laceration etiologically seem to be
causative in the formation of rectovaginal fistulas.6
Other causes for rectovaginal fistula are rare but must be
considered. Inflammatory bowel diseases, specifically Crohns
disease, is the next most frequent etiology.1 Other diseases that
may lead to this situation are given in Table 37.1. These conditions
include carcinoma, radiation therapy, diverticulitis, foreign body
including pessary, penetrating trauma, infectious processes, and
congenital anomalies; pelvic, perineal, and anorectal surgeries
that include vaginal hysterectomy, low anterior resection, stapled
hemorroidectomy, and recurrent rectocele repair, anorectal eroticism, chemotherapy, and neoadjuvant chemoradiation.7
Answer: The most common risk factor to develop rectovaginal fistula is obstetrical injuries occurring after vaginal deliveries.3
Inflammatory bowel disease is the next most common risk factor
occurring in up to 9% of patients with a history of Crohns disease.8 Other less common conditions include carcinoma, radiation
therapy, diverticulitis, penetrating trauma, infection, and pelvic
surgery.7 (Grade B recommendation)

INTRODUCTION
Rectovaginal fistula is a condition of protean manifestations for
the suffering patient. Unlike patients with anorectal fistula secondary to cryptoglandular etiology, these patients usually develop
this complication following an obstetrical delivery. This condition
not only affects the patients self-esteem and social relationships,
but also the patients quality of life significantly.
Most patients with rectovaginal fistula present clinically with
passage vaginally of stool, flatus, purulent discharge, or frequent
urinary tract infections. Patients may also note pain or pressure
referred to the perineum. This may be associated with dyspaurenia. Although not classically presenting symptoms, these patients
may experience tenesmus and frank fecal incontinence.
Rectovaginal fistulas are anatomically divided into low, mid,
and high; this may refer to either vagina or rectum. Nevertheless,
functionally and in regards to treatment, they should be considered either low or high based on anorectal sphincter complex.1
This classification is essentially determined by the relationship of
the fistula to the anorectal sphincter anatomy. Distal or lower fistulas are easily evident on physical examination which includes
digital rectal examination, anoscopy, and bi-manual examination
of the vagina and rectum. The surgical management is guided by
the anatomic basis of the rectovaginal fistula.

2. How do patients with rectovaginal fistulas present clinically?


Most patients will complain of flatus, feces, purulent discharge,
frequent urinary tract infections, or malordous vaginitis.

1. Who is at risk to develop a rectovaginal fistula?


Rectovaginal fistula commonly occurs following vaginal deliveries. Venkatesh et al.2 reviewed the incidence of rectovaginal fistulas
following vaginal deliveries in 20,500 women. Their study revealed
that 5% of all normal deliveries suffered an episiotomy associated
with either a third or fourth degree lacerations of the perineum. Of
the patients with fourth degree lacerations that were recognized
and repaired primarily, 10% of these cases disrupted following
primary repair usually secondary to sepsis. Eighty-eight percent
of rectovaginal fistulas occur following these injuries.3 Worldwide
the incidence to develop rectovaginal fistula following vaginal
delivery is 1%. The higher incidence of rectovaginal fistula in subSaharan Africa and South Asia is thought to be primarily due to
the limitation of access to quality obstetrical care and prolonged

Table 37.1 Etiology of Rectovaginal Fistula


Obstetrical

Penetrating trauma

Crohns disease

Infectious diseases

Colorectal carcinoma

Congenital anomalies

Anal carcinoma

Pelvic, perineal, and anorectal surgery

Radiation therapy

Low anterior resection

Diverticulitis

Stapled hemorrhoidectomy

Foreign body (pessary)

Anorectal eroticism

Recurrent rectocele repair

Neoadjuvant chemoradiation

Chemotherapy
305

PMPH_CH37.indd 305

5/22/2012 5:26:54 PM

306

Surgery: Evidence-Based Practice

A significant percentage of rectovaginal fistula patients may also


experience tenesmus and/or fecal incontinence. This may be difficult to elucidate historically as some patients may not recognize
the source of the incontinence primarily due to debility, etc. The
differential diagnosis for patients that present with these complaints is all inclusive of the conditions previously reviewed.
Rectovaginal fistulas are most often found at the dentate line.
The usual communication occurs between the posterior fornix of
the vagina and the anus.9 Most colorectal surgeons define the fistula
based on the anorectal opening. Gynecologists will define this fistulous opening based on the vaginal opening. This specialty-specific
consideration not withstanding, the anatomic basis for the fistula
must be clearly defined to determine what types of treatment strategies are needed. Consequently, depending on the specialty of the
examiner, there will be low, mid, and high fistulas and multiple
combinations thereof. There can be a high vaginal fistula with mid
rectal opening as well as a high rectal fistula with vaginal opening. Fistulas that are low and in close proximity to the vaginal
fourchette and involve the anus are primarily anovaginal fistulas. High rectovaginal fistulas include an internal orifice that is
proximal to the dentate line in regards to the anorectal sphincter
mechanism (i.e., puborectalis muscle) or in relation to the orifice
opening and proximity to the cervix.
Answer: Most patients will complain of flatus, feces, purulent discharge, frequent urinary tract infections, or malordous
vaginitis. A significant percentage of rectovaginal fistula patients
may also experience tenesmus and/or fecal incontinence. (Grade
B recommendation)
3. How do we establish the diagnosis of rectovaginal fistulas?
A thorough and focused history should be obtained, which will
allow the clinician to pinpoint the diagnostic modalities to search
for the cause of a patients rectovaginal fistulas. The patients medical history and medical records, to include previous diagnostic studies, should be carefully reviewed. This focused approach will allow
the further elucidation of whether carcinoma, radiation therapy, or
inflammatory bowel disease may be the precipitating disorder.
The physical examination of the rectovaginal fistula patient
should include a thorough assessment of the vagina, perineum,
and anorectal sphincter mechanism. A bivalved speculum examination of the posterior vaginal wall must be carried out with careful attention for any anatomical deficits that may be present. The
examiner should look carefully for evidence of vaginitis, malodorous discharge, nonspecific inflammation, mucopurulent drainage,
and of course feces.1-7 An inspection of the perineum to include
anal and vaginal orifices as well as the perineal body needs to be
performed. Obvious external evidence of fistulization, fissures, or
other processes can be visualized. The presence or absence of the
perineal body can be ascertained. The presence of cloacae demonstrates severe previous injury to the area. This external inspection will also allow for recognition of other diseases that may be
associated with rectovaginal fistula such as cystoceles, uterine
prolapse, and pelvic inflammatory disease. Tender, inflamed, and
otherwise uncomfortable processes may require an examination
under anesthesia to allow proper classification and diagnosis. The
examiner should be able to perform a digital rectal examination as
well as an examination that places a digit into both the vagina and
the rectum. This bi-digital examination will allow for evaluation of the compliance of the rectovaginal septum, the integrity,
and mass of the perineal body, and whether or not the anorectal
sphincter mechanism has been violated by the usual antecedent

PMPH_CH37.indd 306

obstetrical injury. The perineal body is often thin on nonexistent


in rectovaginal fistula patients that have a concomitant sphincter
injury. These patients may present with a functional cloacae combining the genitourinary and gastrointestinal points of egress.
Careful documentation of these injuries needs to occur to plan
appropriate treatment. Rectovaginal fistula may on bi-manual
examination demonstrate an irregularity of the posterior vaginal
wall that is usually a small area of irregular mucosa and the site of
the vaginal orifice. This area can usually be gently probed.
All patients should undergo an anoscopy (preferably with
a slotted side viewing scope), rigid proctosigmoidoscopy, and
fiberoptic colonoscopy to eliminate possible proximal causes for
the rectovaginal fistula. Most low fistulas can be easily diagnosed
whereas most high fistulas are problematic diagnostically. Some
investigators utilize a probing method to establish the diagnosis
whereas others would not attempt to blindly probe these areas.
When the fistula cannot be established by these simple methods, other diagnostic modalities need to be considered. The simplest method to establish the presence of a rectovaginal fistula is
the methylene blue retention enema test. This requires the placement of a vaginal tampon into the vagina and the administration
of a methylene blue retention enema via the anus. The patient is
allowed to rest in the left lateral position. After 1 h of observation, the presence of methylene blue on the tampon is the sine qua
non for the presence of an abnormal communication between the
vagina and the anorectum. A less commonly performed technique
required placing the patient into the lithotomy position with a
slight trendelenburg position and instilling the vagina with warm
saline. Thereupon, a rigid proctoscope is inserted into the rectum
and gentle insufflation is carried out. The presence of air bubbles
in the vagina usually establishes the presence of the rectovaginal
fistula, but may not confirm the anatomic classification.
When the previously discussed diagnostic evaluations have
not identified the site or the presence of a rectovaginal fistula, the
need to consider other adjunctive tests to establish the diagnosis
must be utilized. Diagnostic imaging has been useful in this regard.
Studies have reported a role for computerized axial tomography
that has a sensitivity of 60% in defining the location of the fistula.11
The introduction of coiled endoanal MRI has changed the evaluation of these disorders by demonstrating an excellent visualization of the pelvis, vagina, and anorectum. Some studies have a
remarkable specificity approaching 100%.12 Anal ultrasound is
also a useful adjunct in the diagnosis and the assessment of the
anorectal sphincter status. Unfortunately, the literature demonstrates a variable success rate for this modality ranging from 7% to
78%.13,14 This modality is operator-specific and consequently less
accurate than endoanal MRI. Some authors have recommended
the addition of hydrogen peroxide prior to the performance
of an endoanal ultrasound to improve the accuracy of the anal
ultrasound.14,15 Most colorectal surgeons still employ an ultrasound not as a diagnostic tool, but for preoperative assessment of
the anorectal sphincter which is often compromised.
Other contrast studies may be indicated and are easily
obtained. These studies include proctography, vaginography, and
cinedefecography.10 Although these adjunctive procedures have
been used more significantly in the past, there remains a role for
them currently.
Answer: The diagnosis of a rectovaginal fistula begins with a
through history and physical examination. The physical examination should include a rectal examination, bi-manual rectovaginal
examination, and anoscopy. All patients should undergo a

5/22/2012 5:26:55 PM

Rectovaginal Fistula

fiberoptic colonoscopy to eliminate proximal causes. Other helpful maneuvers include the vaginal methylene blue tampon test
and water bubble test as previously described. Radiologic studies
include CT scan which has a sensitivity of 60% and MRI which
has a sensitivity of 100% in some studies.11,12 Anal ultrasound is
helpful for preoperative assessment of the anorectal sphincter
complex and has a sensitivity of 7% to 78% in diagnosing fistulas if
used with hydrogen peroxide.13,14 Other contrast studies are available which include proctography, vaginography, and cinedefecography. (Grade C recommendation)
4. What are the indications for abdominal repair of rectovaginal fistula?
The traditional approach for rectovaginal fistula treatment is surgical. Medical therapy for the treatment of non-Crohnsrelated
fistulas is nonexistent. Following the evaluation of the patient and
diagnosis of a rectovaginal fistula, a careful preoperative assessment of the patient and related comorbid conditions is necessary.
The anatomic classification of the fistula as well as the etiology
will significantly determine the operative approach and ultimate
outcome. High fistulas are generally approached transabdominally and may require bowel resection especially if the fistula was
caused by a colonic or rectal disorder. Small bowel fistulas to the
vagina are exceedingly rare but may possibly require small bowel
resection. If the rectovaginal fistula is secondary to a previous
hysterectomy, then a resection of bowel may not be indicated provided the fistula is taken down and the openings closed and then
protected with an interposition of omentum, muscular flaps, or
fascial flaps to maintain isolation of the repair.15
Many approaches have been used to treat the variety of fistulas that can develop. Transanal, transperineal, transcoccygeal, and
transabdominal surgeries including pouch anal anastomosis with
or without interposition of tissue and with or without fecal diversion
and/or urinary diversion are all valid surgical options dependent on
the cause and the anatomic location of the fistula.7,15-17 Hampton and
Bacon favored abdominal anal pull-through with perineal repair.18
Turner-Warwick preferred an abdominal operation with interposition of omentum.19 Others have interposed gracilis muscle.20 Mason
exposed the area by dividing the rectum and the sphincters, closing
the fistula in layers and reconstructing the rectum.21
High rectovaginal fistulas are approached through the abdomen. The preoperative strategy requires understanding of the
anatomic location, etiology, and the confounding issues relating
to the surrounding tissues such as radiation to the area, sepsis,
and arborization of the fistula. The issue surgically is to develop
and conduct the operation in minimally involved tissue planes.
Rectal resection with coloanal anastomosis procedure was
initially developed by Sir Allen Parks et al.22 This procedure
includes dissection and mobilization of the rectum below the fistula site and furthermore requires mobilization of the proximal
colon from its lateral attachments including takedown and mobilization of the splenic flexure. Thus, by definition, this includes
proctectomy and coloanal anastomosis. The anastomosis has been
performed with either a mucosectomy or a double-staple technique. Nowacki reported functionally good results in 18 out of
23 patients undergoing this procedure for radiation-induced rectovaginal fistula.23 Additional studies have reported similar success for radiation-induced rectovaginal fistula.24,25 Other studies
have advocated interposition and/or on-lay grafts utilizing omentum to treat the fistula with varied success. Steichen pioneered the

PMPH_CH37.indd 307

307

use of staplers as an adjunct in the treatment for these disorders.26


Recent literature has advocated laparoscopic, hand-assisted, and
robotic treatments for the management of these complicated
fistulas.27-29 However, further evidence is lacking regarding the
superiority of these newer technical advances.
Answer: High fistulas are traditionally repaired through an
abdominal approach. These may include primary repair with
an interposition flap or repair with bowel resection.15 Many
techniques have been described including abdominal anal pullthrough with perineal repair, abdominal operation with interposition of omentum or interposition of gracilis muscle, and
reconstruction of the rectum after dividing the rectum and the
sphincters. If bowel resection is performed, the coloanal anastomosis can be constructed with either a mucosectomy or a doublestaple technique.23 (Grade B recommendation)
5. What are the indications for perineal and transanal repair
of rectovaginal fistula?
Any attempt to repair a rectovaginal fistula must primarily address
the anorectal openings, even though the fistula may have primarily originated from a vaginal source (i.e., obstetrical trauma).
Many surgeons, both general and gynecological, prefer to repair
the fistula using the transvaginal approach.30 The transvaginal
repair is not recommended based on the literature because the
high-pressure zone is in the rectum and the consequent direction
of flow is from rectum to vagina. If the repair of the rectal opening is accomplished surgically, it is often unnecessary to deal with
the vagina and allow the vagina to close secondarily. Conversely,
no matter how meticulously the technique is performed via the
vagina, if the rectal closure fails so does the repair.
The initial report regarding the use of anterior rectal advancement flap in a posttraumatic rectovaginal fistula was described by
Noble in 1902 with subsequent modification of that technique by
Laird in 1948.31,32 This technique is for low-lying rectovaginal fistula
without associated sphincter injury, and this variant is not common.
The theoretical advantages of this repair and the various techniques
and principles of this procedure have remained relatively constant,
which is to address the high-pressure defect in the anorectum.
The basic technique for the transanal flap repair is to create a
flap of substantive depth including mucosa, submucosa, and even
partial thickness of the internal sphincter. This flap is raised in
the cephalad direction for at least a length of 5 cm. The dissection
is usually completed at the point when the rectal opening is easily
reached and the defect can be covered without any tension. After
the flap has been created, the internal opening is cored out and
any defect in the sphincter complex or in the rectovaginal septum is sutured primarily. Then the flap is sutured in a tension-free
place. The vaginal opening is left open to drain.
Previous studies that have evaluated outcomes for this repair
have had inconsistent results.33-42 These studies have had heterogeneous populations, different surgical techniques, and inconsistent follow-up. Consequently, a wide range of success is seen in
larger studies. However, as there is no contrary Level 1 evidence to
refute, this is the standard treatment for traumatic low rectovaginal fistula without coexisting incontinence.
The other commonly used approach for simple or low rectovaginal fistulas is to perform a mucosal advancement flap with
an overlapping sphincteroplasty. Generally, these patients have
partial incontinence to flatus or feces or frank fecal incontinence.
Secondarily, these patients have suffered from an obstetrical

5/22/2012 5:26:55 PM

308

Surgery: Evidence-Based Practice

injury that has violated the anorectal sphincter mechanism. The


sphincteroplasty is carried out in a manner that allows repair of
the primary defect, corrects the underlying sphincter injury, and
interposes musculature by repairing the muscles of the perineal
body thereby buttressing the repair.
Kodner et al. from Washington University in St. Louis reported
on 107 endorectal advancement flaps for repair of rectovaginal fistula secondary to all causes (obstetrics, septic, traumatic, postoperative, and Crohns).33 Sixteen percent had persistent rectovaginal
fistula following repair. Eight percent required a second procedure.
Continence was unchanged in 80% and improved in 18%. The
authors concluded that endorectal flap improved or cured 93% of
the rectovaginal fistulas avoiding fecal diversion and stoma.
A Cleveland Clinic trial reviewed the endorectal advancement
flap performed during a 5-year period for rectovaginal fistula in 52
patients.34 Immediate fistula occurred in 6%. Statistically significant higher recurrence rates occurred following previous history
of surgical repair. A subsequent study from the same institution
reviewed 34 patients with rectovaginal fistula and reported a success rate of 63.6%.35 The only factor that seemed to alter the outcome negatively was an association with Crohns disease.
The University of Minnesota reviewed their data with
endorectal advancement flaps in 81 patients.33 The etiology in
this series was obstetrical in 74%, perineal infection in 10%,
operative trauma in 7%, and unknown in 8%. Overall success
rate was 83%. The main predictors for success were correlated to
the number of pervious repairs. If no previous repairs had been
carried out, then the success rate was 88%. If two or more previous repairs were attempted, then the success rate was 55%. Th is
study demonstrates that one should be hesitant to offer endorectal advancement flap repair to those who had undergone two or
more previous repairs.
A follow-up study by Tsang from the same group at University
of Minnesota analyzed the outcomes of RVF repairs based on preoperative sphincter status.37 Fift y-two women underwent 62 repairs
for simple obstetrical rectovaginal fistulas. Forty-eight percent
had varying degrees of fecal incontinence before surgery. There
were 27 endorectal advancement flaps and 35 sphincteroplasties
(28 with and 8 without levatorplasty). Success rates were 41% with
endorectal advancement flap and 80% with sphincteroplasties
(96% with and 33% without levatorplasty). An endorectal advancement flap was successful in 50% of patients with normal anorectal sphincter function, but in only 33% of patients with abnormal
sphincter function. Results were better in patients with sphincter
defects. Poor outcomes in this study were associated with increased
number of previous endorectal advancement flap repairs, but not
with previous sphincteroplasties.
Khanduja reported on the effectiveness of combining an
endorectal advancement flap with sphincteroplasty for symptomatic patients with rectovaginal fistula and anorectal sphincter
injury.38 In addition to the mucosal advancement flap, 65% of patients
underwent a two-layer repair of anorectal sphincter with reapproximation of the puborectalis muscle in eight patients. Thirty percent
of patients underwent a one-layer repair of the anorectal sphincter
and 5% underwent reapproximation of the anorectal sphincter alone. Continence was restored in 70% and incontinence was
improved in 30% with the only manifestation of incontinence
being to liquid stool or flatus.
In summary, the use of mucosal advancement flap repair is
appropriate for most simple rectovaginal fistulas. Obviously, the
success is determined based upon the etiology of fistula, anatomical

PMPH_CH37.indd 308

considerations, the number of previous attempted repairs, and the


presence of an associated anorectal sphincter injury.
Answer: Low vaginal fistulas are repaired via a transanal
or a perineal approach. The common procedures used include an
anterior rectal advancement flap and mucosal advancement flap
with or without an overlying sphincteroplasty.33-42 Anterior rectal
flaps have a varied success rates reported in the literature and do
not have any Level 1 evidence to support its use. However, mucosal
advancement flaps have been studied much further.33-38 The success
rate ranges from 33% to 83% depending on the number of previous repairs and if an associated sphincter injury is present.38 This
procedure is most successful in patients who have never had a previous repair and present with a simple rectovaginal without sphincter
injury. (Grade B recommendation)
6. What is the management for rectovaginal fistulas secondary
to Crohns disease?
Following obstetrical injury, Crohns disease is the second most
common etiology of rectovaginal fistula with a risk rate of 35%
in a population-based cohort study regarding 9% of all fistulous
etiologically.8 Rectovaginal fistula is related to the frequency
and severity of large bowel involvement.43-49 Rectovaginal fistula
occurs in 3.5% and 23% of patients with Crohns disease involving the small and large bowel, respectively.50,51 The presence of
a rectovaginal fistula increases the risk of proctectomy during a
patients course with the disease.46,52
Patients with no or mild symptoms may not require any
treatment.8,43,45,53 Symptomatic patients should be treated with
a multidisciplinary approach, in which medical treatment and
drainage of local sepsis are the initial steps before any definitive
treatment attempts are considered.1,43,54 Although immunosuppressants including biological agents are effective in intestinal
Crohns disease, their role in fistulous perianal Crohns disease
is controversial.55-62 Attempts at surgical closure have had varying degrees of success.63 A previously failed repair does not mean
that a subsequent repair will be unsuccessful. Although a primary
repair will fail 29% to 54%, the chance of a secondary repair being
successful does not change for Crohns disease patients.64 This
contrasts with non-Crohns disease patients.37,65
A minority of patients suffering from Crohns disease with rectovaginal fistula and associated anal ulcers/fistula may be candidates for surgical therapy. The majority undergo proctocolectomy.
The University of Minnesota retrospectively evaluated 51 patients
with Crohns disease that underwent 65 procedures.66 These procedures included seton drainage (n = 35), endorectal advancement
flap (n = 8), fibrin glue injection (n = 8), transperineal repair (n = 6),
collagen plug placement (n = 4), and bulbocavernosus flap (n = 4).
All patients were being treated for Crohns disease at the time of
surgery and 26/51 (51%) had received preoperative infliximab treatmentminimum of three infusions, 5 mg/kg. Ten patients underwent preoperative diversion. At a mean follow-up of 38.6 months,
27/51 (53%) fistulas healed and in 24/51 (47%) it reoccurred. Sixty
percent of diverted patients healed, whereas 51% of nondiverted
repairs were successful. Neither active proctitis nor infliximab therapy significantly altered tissue healing. Twenty-seven percent of the
patients eventually required proctectomy. The presence of a stoma
was not found statistically to improve healing after repair.
In patients with complex rectovaginal fistulas and anal ulcerations secondary to Crohns that are refractory to traditional therapy,
the Cleveland Clinic has proposed the transanal sleeve advancement flap. Transanal sleeve advancement flap is a circumferential

5/22/2012 5:26:55 PM

Rectovaginal Fistula

dissection of the anal mucosa/submucosa beginning at the dentate


line and proceeding cephalad until the rectum is mobilized. The
anal ulcerations and the opening of the low-lying rectovaginal fistula are excised. The rectal lining is then anastomosed to the dentate line.67 The Cleveland Clinic reported their initial experience
with 12 points68; 1-year following surgery, two-thirds of the patients
had healed their fistula, although 38% of the prior group required
additional surgery to prompt healing. Their recommendation based
on this initial small volume trial was to offer this alternative only to
selected patients when their other alternative would be total proctocolectomy and permanent stoma.
Answer: Treatment for rectovaginal fistulas secondary to
Crohns disease depends on the severity of symptoms. Patients
with minimal symptoms may not require any treatment although
patients with severe disease may be treated with medical therapy or
surgery, or a combination of both modalities. Surgical treatment is
usually reserved for patients that have complex fistulas with associated ulcerations. The procedures described include seton drainage,
endorectal advancement flap, transperineal repair, collagen plugs,
bulbocavernosus flap, and transanal sleeve advancement, all of
which have varying success rates.66 If all of the above measures fail,
then most patients will need to undergo a total proctocolectomy.
(Grade B recommendation)
7. Are there miscellaneous procedures used in the surgical
armamentarium to treat rectovaginal fistulas?
There are a variety of situations that require alternative surgical
approaches in the management of rectovaginal fistula. Not all of
these alternative situations are successful. When a substantive
destruction of the perineal body and sphincter/rectovaginal septum exists, as in a cloaca, occasionally a more complex repair is
required. A perineoproctectomy may be indicated in such a situation. This is usually only performed for a low rectovaginal fistulas.
The rectovaginal fistula tract is excised or divided; the anorectal
sphincter, vagina, and rectal mucosa are all identified, mobilized,
and repaired primarily. The Ferguson Clinic reported on a series of
95 patients operated on for rectovaginal fistula via a septal repair
after conversion to a fourth degree perineal laceration.69 Excellent
to good functional results occurred in 97% of patients.
Fistulotomy has been advocated in the literature to treat
rectovaginal fistulas. This procedure is mentioned to definitively

309

proscribe its use and is only mentioned as an abhorrent historical


procedure.9
Fibrin glue and a variety of plugs, primarily Surgis, have
anecdotally been used to treat rectovaginal fistulas.64,70,71 There is a
dearth of literature to support their use.72 This literature is limited
to small series reporting results. When these agents are applied
to long fistulas secondary to cryptoglandular sepsis/abscess, there
may be data to support their use. With regard to rectovaginal fistula, there is a paucity of data to recommend the use of these products except for exceptional causes.
The transvaginal repair of rectovaginal fistula is well described
in the literature.73 This procedure has not gained in popularity
because of the superiority of transanal repair. By occluding and
repairing the rectal orifice (the high-pressure zone is ablated),
there is no requirement to repair the vagina in these repairs.
The purpose of tissue transfer procedures for patients with
rectovaginal fistulas is to provide healthy tension-free, wellvascularized tissue in the area of the repair. A number of tissue
transfers such as gracilis, rectus, gluteus, and bulbocavernosus are
available and have been used extensively for rectovaginal fistulas
secondary to radiation injury.73
Rectovaginal fistula secondary to gynecologic and rectal
malignancy or radiation therapy for these tumors typically are
typically high or at least anatomically above the sphincter mechanism. The first issue is whether definitive surgical therapy can be
performed for cure. If only palliative options are to be considered,
the options are diverting stoma, endoscopic stent placement, and
other alternative treatments.
Answer: There are a variety of situations and techniques that
require alternative approaches. A perineoproctectomy can be utilized when there is substantive destruction of the perineal body
and sphincter complex.69 Fibrin glue and a variety of plugs have
been described in the literature, but are mostly successful in long
fistulas secondary to cryptoglandular sepsis/abscess.64,70-72 Tissue transfer techniques provide a tension-free, well-vascularized
repair, which are most successful in patients that have undergone radiation therapy. Transvaginal repair and fistulotomy have
been described, but are not recommended due to the high failure rate. Palliative options include stents and diverting stomas
which can be utilized in patients with rectal malignancy. (Grade
C recommendation)

Clinical Question Summary


Question

Answer

Grade of
Recommendation

References

1 Who is at risk to develop a


rectovaginal fistula?

The most common risk factors include obstetric trauma,


inflammatory bowel disease, cancer, diverticulitis,
penetrating trauma, and pelvic surgery.

1-7

2 How do patients with


rectovaginal fistulas present
clinically?

Most patients will complain of flatus, feces, purulent


discharge, frequent urinary tract infections, or
malordous vaginitis.

3 How do we establish the


diagnosis of rectovaginal
fistulas?

History and physical examination, colonoscopy, endoanal


MRI, CT scan, anal ultrasound, and contrast studies
(proctography, vaginography, and cinedefecography).

10-14

4 What are the indications


for abdominal repair of
rectovaginal fistula?

High fistulas above the sphincter complex. Repairs


include bowel resection with coloanal anastomosis and
primary repairs with interposition flaps.

7, 15-29

(Continued)

PMPH_CH37.indd 309

5/22/2012 5:26:55 PM

310

Surgery: Evidence-Based Practice

(Continued)
Question

Answer

5 What are the indications for


perineal and transanal repair
of rectovaginal fistula?

Low fistulas below the sphincter complex. Repair


options include an anterior rectal advancement flap
and mucosal advancement flap with or without an
overlying sphincteroplasty.

6 What is the management


for rectovaginal fistulas
secondary to Crohns disease?

Multimodal including medical and surgical treatments.


Surgery ranges from seton drainage to
proctocolectomy depending on severity of symptoms.

8, 37, 43-68,
74, 75

7 Are there miscellaneous


procedures used in the
surgical armamentarium to
treat rectovaginal fistulas?

Multiple procedures can be used with varying success


including perineoproctectomy, fibrin glue, plugs, and
tissue transfer procedures.

9, 15, 71

REFERENCES
1. Saclarides TJ. Rectovaginal fistula. Surg Clin North Am. 2002;82:
1261-1272.
2. Venkatesh KS, Ramanujam PS, Larson DM, et al. Anorectal complications of vaginal delivery. Dis Colon Rectum. 1989;32:1039.
3. Senatore PJ Jr. Anovaginal fistulae. Surg Clin North Am. 1994;74:
1361-1375.
4. World Health Organization. Prevention and treatment of obstetric fistulae: Report of a technical working group. Geneva: World
Health Organization; 1989.
5. Murray C, Lopez A. World Health Organization: Health dimensions of sex and reproduction. Geneva: World Health Organization; 1989.
6. Angioli R, Gomez-Marin O, Cantuaria G, et al. Severe perineal
lacerations during vaginal delivery: The University of Miami
experience. Am J Obstet Gynecol. 2000;182:1083-1085.
7. Goligher JC. Surgery of the Anus, Rectum and Colon. 4th ed. New
York: Macmillan; 1980:193.
8. Schwartz DA, Loftus EV, Jr., Tremaine WJ, et al. The natural history of fistulizing Crohns disease in Olmsted County, Minnesota. Gastroenterology. 2002;122:875-880.
9. Rivadeneira DE, Ruffo B, Amrani S, et al. Rectovaginal fistulas: Current surgical management. Clin Colon Rectal Surg. 2007;20:96-101.
10. Robert PL. Rectovaginal and Rectourethral fistulas. In: Pemberton J, ed. Shackelfords Surgery of the Alimentary Tract. 6th ed.
Philadelphia, PA: WB Saunders/Elsevier; 2007:1945-1957.
11. Kuhlman JE, Fishman EK. CT evaluation of enterovaginal and
vesicovaginal fistulas. J Comput Assist Tomogr. 1990;14:390-394.
12. Dwarkasing S, Hussain SM, Hop WC, et al. Anovaginal fistulas: Evaluation with endoanal MR imaging. Radiology. 2004;231:
123-128.
13. Sudol-Szopinska I, Jakubowski W, Szczepkowski M. Contrastenhanced endosonography for the diagnosis of anal and anovaginal fistulas. J Clin Ultrasound. 2002;30:145-150.
14. Choen S, Burnett S, Bartram CI, et al. Comparison between anal
endosonography and digital examination in the evaluation of
anal fistulae. Br J Surg. 1991;78:445-447.
15. Visser BC, McAninch JW, Welton ML. Rectourethral fistulae:
The perineal approach. J Am Coll Surg. 2002;195:138.
16. Celebrezze JP, Jr., Medich DS. Rectal ulceration as a result of
prostatic brachytherapy: A new clinical problem. Report of three
cases. Dis Colon Rectum. 2003;46:1277.
17. Nyam DCNK, Pemberton JH. Management of iastrogenic rectourethral fistula. Dis Colon Rectum. 1999;42:994.

PMPH_CH37.indd 310

Grade of
Recommendation

References
30-42

18. Hampton JM, Bacon HE. Diagnosis and surgical management of


rectourethral fistulas. Dis Colon Rectum. 1961;4:177.
19. Turner-Warwick R. The use of pedicle grafts in the repair of urinary tract fistulae. Br J Urol. 1972;44:644.
20. Zmora O, Potenti FM, Wexner SD, et al. Gracilis muscle transposition for iatrogenic rectourethral fistula. Ann Surg. 2003;237:483.
21. Mason AY. The place of local resection in the treatment of rectal
carcinoma. Proc R Soc Med. 1970;63:1259.
22. Parks AG, Allen CL, Frank JD, McPartlin JF. A method of treating
post-irradiation rectovaginal fistulas. Br J Surg. 1978;65:417-421.
23. Nowacki MP. Ten years of experience with Parks coloanal sleeve
anastomosis for the treatment of post-irradiation rectovaginal
fistula. Eur J Surg Oncol. 1991;17:563-566.
24. Cooke SA, Wellsted MD. The radiation-damaged rectum: Resection with coloanal anastomosis using the endoanal technique.
World J Surg. 1986;10:220-227.
25. Bricker EM, Johnston WD. Repair of postirradiation rectovaginal fistula and stricture. Surg Gynecol Obstet. 1979;148:499-506.
26. Steichen FM, Barber HK, Loubeau JM, Iraci JC. Bricker-Johnston
sigmoid colon graft for repair of postradiation rectovaginal fistula and stricture performed with mechanical sutures. Dis Colon
Rectum. 1992;35:599-603.
27. Schwenk W, Bohm B, Grundel K, Muller J. Laparoscopic resection of high rectovaginal fistula with intracorporeal colorectal
anastomosis and omentoplasty. Surg Endosc. 1997;11:147-149.
28. Pelosi MA III, Pelosi MA. Transvaginal repair of recurrent
rectovaginal fistula with laparoscopic-assisted rectovaginal
mobilization. J Laparoendosc Adv Surg Tech. 1997;7:379-383.
29. Kumaran SS, Palanivelu C, Kavalakat AJ, Parthasarathi R,
Neelayathatchi M. Laparoscopic repair of high rectovaginal
fistula: Is it technically feasible? BMC Surg. 2005;5:20.
30. Tancer ML, Lasser D, Rosenblum N. Rectovaginal fistula or
perineal and anal sphincter disruption, or both, after vaginal
delivery. Surg Gynecol Obstet. 1990;171:43.
31. Noble GH. A new operation for complete laceration of the
perineum designed for the purpose of eliminating danger of infection from the rectum. Trans Am Gynecol Soc. 1902;27:357-363.
32. Laird DR. Procedures used in the treatment of complicated fistulas. Am J Surg. 1948;76:701-708.
33. Kodner IJ, Mazor A, Shemesh EI, Fry RD, Fleshman JW, Birnbaum EH. Endorectal advancement flap repair of rectovaginal and
other complicated anorectal fistulas. Surgery. 1993;114:682-689.
34. Ozuner G, Hull TL, Cartmill J, Fazio VW. Long-term analysis
of the use of transanal rectal advancement flaps for complicated
anorectal/vaginal fistulas. Dis Colon Rectum. 1996;39:10-14.

5/22/2012 5:26:55 PM

Rectovaginal Fistula

35. Sonoda T, Hull T, Piedmonte MR, Fazio VW. Outcomes of primary


repair of anorectal and rectovaginal fistulas using the endorectal
advancement flap. Dis Colon Rectum. 2002;45:1622-1628.
36. Lowry AC, Thorson AG, Rothenberger DA, Goldberg SM. Repair
of simple rectovaginal fistulas. Influence of previous repairs. Dis
Colon Rectum. 1988;31:676-678.
37. Tsang CB, Madoff RD, Wong WD, et al. Anal sphincter integrity
and function influences outcome in rectovaginal fistula repair.
Dis Colon Rectum. 1998;41:1141-1146.
38. Khanduja KS, Padmanabhan A, Kerner BA, Wise WE, Aguilar
PS. Reconstruction of rectovaginal fistula with sphincter disruption by combining rectal mucosal advancement flap and anal
sphincteroplasty. Dis Colon Rectum. 1999;42:1432-1437.
39. Hilsabeck JR. Transanal advancement of the anterior rectal
wall for vaginal fistulas involving the lower rectum. Dis Colon
Rectum. 1980;23:236.
40. Jones IT, Fazio VW, Jagelman DG. The use of transanal rectal
advancement flaps in the management of fistulas involving the
anorectum. Dis Colon Rectum. 1987;30:919.
41. Rothenberger DA, Christenson CE, Balcos EG, et al. Endorectal
advancement flap for treatment of simple rectovaginal fistula.
Dis Colon Rectum. 1982;25:297.
42. Shemesh EI, Kodner IJ, Fry RD, et al. Endorectal sliding flap
repair of complicated anterior anoperineal fistulas. Dis Colon
Rectum. 1988;31:22.
43. Radcliffe AG, Ritchie JK, Hawley PR, Lennard-Jones JE, Northover
JM. Anovaginal and rectovaginal fistulas in Crohns disease.
Dis Colon Rectum. 1988;31:94-99.
44. Sher ME, Bauer JJ, Gelernt I. Surgical repair of rectovaginal fistulas in patients with Crohns disease: Transvaginal approach.
Dis Colon Rectum. 1991;34:641-648.
45. Tuxen PA, Castro AF. Rectovaginal fistula in Crohns disease.
Dis Colon Rectum. 1979;22:58-62.
46. Heyen F, Winslet MC, Andrews H, Alexander-Williams J, Keighley MR. Vaginal fistulas in Crohns disease. Dis Colon Rectum.
1989;32:379-383.
47. Rankin GB, Watts HD, Melnyk CS, Kelley ML. National Cooperative Crohns Disease Study: Extraintestinal manifestations
and perianal complications. Gastroenterology. 1979;77:914-920.
48. Hesterberg R, Schmidt WU, Muller F, Roher HD. Treatment of
anovaginal fistulas with an anocutaneous flap in patients with
Crohns disease. Int J Colorectal Dis. 1993;8:51-54.
49. Fry RD, Shemesh EI, Kodner IJ, Timmcke A. Techniques and
results in the management of anal and perianal Crohns disease.
Surg Gynecol Obstet. 1989;168:42-48.
50. Van Patter WN, Bargen JA, Dockerty MB, et al. Regional enteritis. Gastroenterology. 1954;26:347-450.
51. Ritchie JK, Lennard-Jones JE. Crohns disease of the distal large
bowel. Scand J Gastroenterol. 1976;11:433-436.
52. Scott NA, Nair A, Hughes LE. Anovaginal and rectovaginal fistula
in patients with Crohns disease. Br J Surg. 1992;79:1379-1380.
53. Morrison JG, Gathright JB, Ray JE, Ferrari BT, Hicks TC, Timmcke AE. Results of operation for rectovaginal fistula in Crohns
disease. Dis Colon Rectum. 1989;32:497-499.
54. Nicholls RJ, Dozois RR. Surgery of the Colon and Rectum. London: Churchill Livingstone; 1997:632-633.
55. Regueiro M, Mardini H. Treatment of perianal fistulizing Crohns
disease with infliximab alone or as an adjunct to exam under
anesthesia with seton placement. Inflamm Bowel Dis. 2003;9:
98-103.
56. Talbot C, Sagar PM, Johnston MJ, Finan PJ, Burke D. Infliximab
in the surgical management of complex fistulating anal Crohns
disease. Colorectal Dis. 2005;7:164-168.

PMPH_CH37.indd 311

311

57. Van Assche G, Vanbeckevoort D, Bielen D, et al. Magnetic resonance imaging of the effects of infliximab on perianal fistulizing
Crohns disease. Am J Gastroenterol. 2003;98:332-339.
58. van der Hagen SJ, Baeten CG, Soeters PB, Russel MG, Beets-Tan
RG, van Gemert WG. Anti-TNF-alpha (infliximab) used as induction treatment in case of active proctitis in a multistep strategy
followed by definitive surgery of complex anal fistulas in Crohns
disease: A preliminary report. Dis Colon Rectum. 2005;48:758-767.
59. Present DH, Rutgeerts P, Targan S, et al. Infliximab for the treatment of fistulas in patients with Crohns disease. N Engl J Med.
1999;340:1398-1405.
60. Sands BE, Anderson FH, Bernstein CN, et al. Infl iximab maintenance therapy for fistulizing Crohns disease. N Engl J Med.
2004;350:876-885.
61. Topstad DR, Panaccione R, Heine JA, Johnson DR, MacLean AR,
Buie WD. Combined seton placement, infliximab infusion, and
maintenance immunosuppressives improve healing rate in fistulizing anorectal Crohns disease: A single center experience. Dis
Colon Rectum. 2003;46:577-583.
62. Hyder SA, Travis SP, Jewell DP, McC Mortensen NJ, George BD.
Fistulating anal Crohns disease: Results of combined surgical
and infliximab treatment. Dis Colon Rectum. 2006;49:1837-1841.
63. Abel ME, Chiu YS, Russell TR, Volpe PA. Autologous fibrin glue
in the treatment of rectovaginal and complex fistulas. Dis Colon
Rectum. 1993;36:447-449.
64. Penninckx F, Moneghini D, DHoore A, Wyndaele J, Coremans
G, Rutgeerts P. Success and failure after repair of rectovaginal fistula in Crohns disease: Analysis of prognostic factors.
Colorectal Disease. 2001;3:406-411.
65. MacRae HM, McLeod RS, Cohen Z, Stern H, Reznick R. Treatment of rectovaginal fistulas that has failed previous repair
attempts. Dis Colon Rectum. 1995;38:921.
66. Gaertner WB, Madoff RD, Spencer MP, et al. Results of combined
medical and surgical treatment of rectovaginal fistula in Crohns
disease. Colorectal Dis. February 15, 2010. [Ahead of print.]
67. Hull TL, Fazio VW. Surgical approaches to low anovaginal fistula in Crohns disease. Am J Surg. 1997;173:95-98.
68. Marchesa P, Hull TL, Fazio VW. Advancement sleeve flaps for treatment of severe perianal Crohns disease. Br J Surg. 1998;85:1695-1698.
69. Mazier WP, Senagore AJ, Schiesel EC. Operative repair of anovaginal and rectovaginal fistulas. Dis Colon Rectum. 1995;38:4-6.
70. Cintron JR, Park JJ, Orsay CP, et al. Repair of fistulas-in-ano
using fibrin adhesive: Long-term follow-up. Dis Colon Rectum.
2000;43:944-949; discussion 949-950.
71. Champagne BJ, OConnor LM, Ferguson M, Orangio GR,
Schertzer ME, Armstrong DN. Efficacy of anal fistula plug in
closure of cryptoglandular fistulas: Long-term follow-up. Dis
Colon Rectum. 2006;49:1817-1821.
72. Loungnarath R, Dietz DW, Mutch MG, Birnbaum EH, Kodner
IJ, Fleshman JW. Fibrin glue treatment of complex anal fistulas
has low success rate. Dis Colon Rectum. 2004;47:432-436.
73. Rahman MS, Al-Suleiman SA, El-Yahia AR, Rahman J. Surgical
treatment of rectovaginal fistula of obstetric origin: A review of
15 years experience in a teaching hospital. J Obstet Gynaecol.
2003;23:607-610.
74. Targan SR, Hanauer SB, van Deventer SJ, et al. A short-term
study of chimeric monoclonal antibody cA2 to tumor necrosis factor alpha for Crohns disease. Crohns Disease cA2 Study
Group. N Engl J Med. 1997;337:1029-1035.
75. Gaertner WB, Decanini A, Mellgren A, Lowry AC, Goldberg
SM, Madoff RD, Spencer MP. Does infliximab infusion impact
results of operative treatment for Crohns perianal fistulas? Dis
Colon Rectum. 2007;50:1754-1760.

5/22/2012 5:26:55 PM

Commentary on
Rectovaginal Fistula
Patricia L. Roberts

The chapter entitled Rectovaginal Fistula by Schulte, Ming,


Olsen and Caushaj provides an excellent review of a difficult clinical problem. Overall, the literature on treatment of rectovaginal
fistulas is characterized by small single-institution series without Level 1 evidence. The authors address several important
questions including (1) Who is at risk to develop a rectovaginal
fistula? (2) How do patients with rectovaginal fistulas present
clinically? (3) How do we establish the diagnosis of rectovaginal fistulas? (4) What are the indications for abdominal repair
of rectovaginal fistula? (5) What are the indications for perineal
and transanal repair of rectovaginal fistulas? (6) What is the
management for rectovaginal fistulas secondary to Crohns disease? and (7) Are there miscellaneous procedures used in the
surgical armamentarium to treat rectovaginal fistulas?
Although the most common cause of rectovaginal fistula is
obstetric trauma, it is important to recognize the global differences
in the presentation and subsequent treatment of rectovaginal fistulas in developed and underdeveloped nations. Obstetric fistulas
are relatively common in underdeveloped countries.1 In Africa
and Asia, an estimated 3.5 million women are affected by obstetric fistula and an estimated 130,000 new cases occur each year.2
Although hospitals have been built solely to treat women with
obstetric fistula in these regions, the backlog of cases in Northern
Niger alone is believed to be at least one million women.1,3 The primary cause of these fistulas is obstructed labor in women or girls
who have no access to obstetric care and have obstructed labor for
3 or 4 days. The birth is often associated with a stillborn and the
necrotic tissue and rectovaginal septum slough resulting in a large
fistula. For every maternal death from obstructed labor it is estimated that there are 1.8 obstetric fistulas.4 Such fistulas are often
quite large and may also involve adjacent organs such as the bladder with associated bowel and urinary incontinence. The social
consequences of such fistulas are significant and there are high
rates of divorce, separation, abandonment, and social isolation.
Rectovaginal fistulas may also occur as a complication of a
variety of other pelvic and anorectal procedures including hysterectomy, low anterior resection, ileal pouch anal anastomosis, the
procedure for prolapse and hemorrhoids (PPH), and the stapled
transanal rectal resection (STARR) procedure. Anastomotic vaginal fistulas may occur in up to 13% of women after low anterior
resection for rectal cancer; risk factors include low anastomosis
and preoperative radiation.5 Ileoanal pouch vaginal fistulas may
occur in 4% to 14% of women and typically have an early or late
presentation.6 Early fistulas are commonly a manifestation of

anastomotic leakage or pelvic sepsis while late fistulas are more


likely to be caused by unsuspected Crohns disease. Meticulous
surgical technique and ensuring that the perivaginal tissues are
separated from the rectum when performing a stapled anastomosis
help avoid such fistulas.
As the authors point out, the signs and symptoms of rectovaginal fistula are generally straightforward; passage of stool
or air per vagina. Despite straightforward signs and symptoms,
most patients delay in seeking medical attention. The commonly
offered reasons are social embarrassment, the desire to have more
children prior to embarking upon a repair, or the belief (propagated by some gynecologists) that their symptoms are to be
expected after childbirth. Women with a colovaginal fistula from
diverticular disease often have few abdominal symptoms and may
initially present to a gynecologist with complaints of malodorous
vaginal discharge.
The authors have provided a stepwise approach to establishing the diagnosis of a rectovaginal fistula. Low rectovaginal fistulas are fairly easy to diagnose in the clinic/hospital if one knows
what they are looking for and other adjuncts such as vaginal tampons and methylene blue and sigmoidoscopy with insufflations or
air with water in the vagina are rarely needed. If a fistula is diagnosed with the methylene blue method, the location of the fistula
still needs to be determined by the surgeon in vivo. On digital rectal examination, an anterior dimple or roughening is commonly
palpated. Anoscopy confirms this finding and a probe can usually be passed into the vagina quite easily in the office. Obstetric
fistulas are generally quite distal and easy to demonstrate in this
way. Pouch vaginal or anastomotic vaginal fistulas may require
contrast examinations to demonstrate and are difficult to visualize endoscopically. The key points to evaluate in the office are the
integrity of the anal sphincter with determination of continence,
the status of the tissue around the fistula, and the location of the
fistula (in relation to the anal verge). Assessment of these factors
is mandatory to establish the optimal treatment for a rectovaginal
fistula; for instance, rectovaginal fistulas with total sphincter disruption require sphincter repair in addition to repair of the fistula
itself. Recurrent fistulas with tissue loss in patients who have had
multiple repairs may require tissue transfer techniques such as a
gracilis muscle transposition or a bulbocavernosus flap.
The authors have thoroughly outlined the indications for
abdominal repair of rectovaginal fistulasgenerally high fistulas,
more appropriately termed colovaginal fistulas, will require an
abdominal repair and resection of the affected segment of bowel.
312

PMPH_CH37.indd 312

5/22/2012 5:26:55 PM

Rectovaginal Fistula

The optimal initial procedure for an obstetric fistula is the


source of an ongoing debate. The most commonly performed procedures include endorectal advancement flaps with or without concomitant sphincter repair and transperineal procedures including
episioproctotomy and layered repair. A substantial number of
women with obstetric fistulas have an underlying sphincter defect
and some degree of fecal incontinence. Such patients require repair
of the underlying sphincter defect in addition to fistula repair.
Performing an endorectal advancement flaps without addressing
the underlying sphincter defect is associated with a failure rate of
up to 60%.7 Episioproctotomy (or essentially conversion of the fistula to a fourth degree perineal laceration with layered repair) is
another option.8 It is difficult to compare the results of the various
procedures; there have been no randomized trials and the available literature contain relatively small single-institution series with
heterogeneous groups of patients. There has been renewed interest
in collagen fistula plugs or a modification of the plug with a special
button; the results have generally been disappointing. These procedures have the advantage of having minimal morbidity.
Patients with Crohns disease are particularly challenging to
treat. Up to 10% of women with Crohns disease develop a rectovaginal fistula during the course of their disease. The optimal
repair should be put into context with the symptoms (some women
with rectovaginal fistulas and Crohns are remarkably minimally
symptomatic), and any associated anal or rectal disease. Local
repairs are not advisable in the setting of active Crohns proctitis. The ultimate rate of success with Crohns fistulas is about 50%
and a cohort of women ultimately requires fecal diversion and/or
proctectomy.9 It is helpful to discuss this with the patient to frame
the initial discussion.
The failure rate after repair of rectovaginal fistula is significant.10,11
Failed repairs are often associated, at least initially, with a larger and
more symptomatic fistula. Failure of a repair is often emotionally
devastating to the patient. Multiply recurrent or persistent fistulas,
particularly those associated with tissue loss may then require tissue transfer techniques such as a gracilis muscle transposition or a
bulbocavernosus flap. With the extent of the dissection and mobilization with these repairs, temporary fecal diversion, with a laparoscopic loop colostomy may be performed. Although fistula healing
is eventually achieved in the majority of patients, concerns about
self-image (from the cosmetic appearance of the site) in addition to
alterations in sexual function, including dyspareunia, may impact
overall satisfaction with the procedure and quality of life.

PMPH_CH37.indd 313

313

Treatment for rectovaginal fistulas is challenging. The operative approach depends on a variety of factors including the size,
location, condition of the surrounding tissues, and association
with concomitant disease, such as Crohns disease. Although
a successful repair is ultimately achieved in the majority of
patients, ultimate healing may require a number of different
repairs.

REFERENCES
1. Donnay F, Weil L. Obstetric fistula: The international response.
Lancet. 2004;363:71-72.
2. Browning A, Allsworth JE, Wall LL. The relationship between
female genital cutting and obstetric fistulae. Obstet Gynecol.
2010:115:578-582.
3. Kelly J. Outreach programmes for obstetric fistulae. J Obstet
Gynecol. 2004;24:117.
4. Abou Zahr C. Global burden of maternal death and disability. Br
Med Bull. 2003;67:1-11.
5. Matthiessen P, Hansson L, Sjodahl R, Rutegard J. Anastomoticvaginal fistula (AVF) after anterior resection of the rectum
for cancer occurrence and risk factors. Colorectal Disease.
2010;12:351-357.
6. Heriot AG, Tekkis PP, Smith JJ, et al. Management and outcome
of pouch-vaginal fistulas following restorative proctocolectomy.
Dis Colon Rectum 2005;48:451-458.
7. Tsang CB, Madoff RD, Wong WD, et al. Anal sphincter integrity
and function influences outcome in rectovaginal fistula repair.
Dis Colon Rectum. 1998;41:1141-1146.
8. Hull TL, El-Gazzaz G, Gurland B, et al. Surgeons should not
hesitate to perform episioproctotomy for rectovaginal fistula
secondary to cryptoglandular or obstetric origin. Dis Colon Rectum. 2011;54(1):54-59.
9. El-Gazzaz G, Hull T, Mignanelli E, et al. Analysis of function
and predictors of failure in women undergoing repair of Crohns
related rectovaginal fistulas. J Gastrointest Surg. 2010;14:
824-829.
10. Pinto RA, Peterson RV, Shawki S, et al. Are there predictors of
outcome following rectovaginal fistula repair? Dis Colon Rectum. 2010:53:1240-1247.
11. Roberts PL. Rectovaginal and rectourethral fistulas. In: J Pemberton, ed. Shacklefords Surgery of the Alimentary Tract. 6th ed.
Philadelphia, PA: WB Saunders/Elsevier; 2007:1945-1957.

5/22/2012 5:26:55 PM

CHAPTER 38

Lower Gastrointestinal Bleeding


Kerry G. Bennett and Steven Schwaitzberg

INTRODUCTION

of patients thought to have LGIB are found to actually be bleeding


from proximal to the ileocecal valve.4,5 In brief, for brisk active
bleeding, the available modality that best provides rapid diagnosis and treatment of the bleeding is the first choice. Colonoscopy
may be the most accurate single-stage evaluation as suggested
by the only RCT done on LGIB.5,6 Unfortunately, it is not always
readily available. Angiography is emerging as a possible first-line
diagnostic test in facilities with a strong interventional radiology
department but can be associated with a relatively low diagnostic
yield unless the bleeding is brisk enough.7,8 In slow active bleeding, current recommendations include beginning with a colonoscopy or a tagged red blood cell scan (TRBC). If the scintigraphy is
positive, proceed with an angiogram. If the scintigraphy is negative, proceed with either angiography, colonoscopy, or repeat TRBC
scan.9,10 As LGIB patients are often volumedepleted, complications from dye injection must also be considered. These problems
can be minimized by using scintigraphy or computer tomography
(CT) before angiogram.9-12 Surgery is the best approach for severe
bleeding.13
Answer: Colonoscopy is the most accurate modality and
allows for diagnosis and treatment of LGIB. (Grade C recommendation) The ideal order of tests in the setting of LGIB is
best determined by available resources in each treatment setting
although traditionally colonoscopy is used first and angiography has been used if colonoscopy fails or cannot be performed.
(Grade C and D recommendations). Scintigraphy and CT prior
to angiography can help further differentiate proximal sources
and decrease complications secondary to angiography. (Grade C
recommendation)

Lower gastrointestinal bleeding (LGIB) represents 20% to 33%


of gastrointestinal (GI) bleeding but is likely underreported.1
Advances in diagnosis and treatment have led to a shift from surgery as the mainstay of treatment to less invasive modalities. An
important principle in approaching LGIB in patients is to differentiate the site of bleeding; that is, determine if the bleeding source
is proximal or distal to the ileocecal valve. The major causes of
LGIB are diverticulosis, inflammatory bowel disease (IBD), colitis,
carcinoma, angiodysplasia, and anorectal disease with diverticulosis, and IBD as the most common etiologies.2
Evidence in LGIB contains few prospective studies.3 The
majority of evidence is historical, based on small case series,
cohorts, and expert opinion. The time-tested tool of colonoscopy
for diagnosis and treatment is used widely at present with varying results. When taking into account the rating scheme for the
strength of recommendations, much of the current clinical practice is based on Level 4 evidence. That is, the evidence level is
based on case reports or case series or expert opinion and not on
high-quality meta-analyses, randomized controlled trials (RCT)
or RCT with a very low risk of bias. This is likely due to challenges secondary to the nature of LGIB, which does not easily lend
itself to RCT or large analyses. Further, there are many challenges
to diagnosis and therapy secondary to differences in individual
patients, services, and treating facilities. Availability of coordinating services such as nuclear medicine and interventional radiology
often determines what options are available for an individual at a
specific institution.
In this chapter, we review the available literature regarding
LGIB. We focus on the diagnostic and therapeutic options and
the supporting data and end with a summary of current recommendations in the management of LGIB.

2. What is the diagnostic accuracy of colonoscopy, radionuclide scanning, and angiography in the setting of LGIB?
Scintigraphy and angiography alone are often not sufficient
to guide surgical resection, and recommendations are based
on Level 4 evidence. Hunter and Pezim14 found that up to 42%
of patients can have an undesirable result if limited surgical resection is planned on the basis of Tc-99labeled RBCs
alone. Although angiogram alone is of limited sensitivity and

1. What is the ideal order of diagnostic testing in the setting


of LGIB?
First, evaluation to differentiate an upper from a lower source of
GI bleeding must be done. Data reveal that approximately 10%
314

PMPH_CH38.indd 314

5/22/2012 5:27:32 PM

Lower Gastrointestinal Bleeding

specificity, the yield of positive arteriography can be improved


with the use of scintigraphy prior to angiogram.7,15 In a study
done in 1998, the complication rate of angiogram has been
found to be 11%, whereas the diagnostic yield of site localization for colon resection was 12%.16 More recent data in a small
(34 patients) retrospective study at one institution reveal that
angiogram results in bleeding localization in 33% of patients
tested with 10/11 patients controlled successfully with embolization. One embolization-related complication occurred
(perforation of ischemic bowel) and no hemorrhagic complications occurred.17
Helical CT and CT angiogram (CTA) have emerged as a
good diagnostic tool for identification of LGIB. CTA has been
found to be 70% to 80% sensitive and 100% specific for evaluation of LGIB.18-22 Multidetector row helical CT (MDCT) has
been found to be 92% accurate in detection and localization of
hemorrhage. 23 Twenty-six patients with massive GI hemorrhage
were studied with MDCT and the result showed that MDCT
was 90% sensitive and 99% specific with an accuracy of 88%
and a negative predictive value of 98%.11 Th is suggests that arteriogram may not be indicated in negative MDCT studies. In
2005, it became accepted that MDCT is readily available and
sufficiently sensitive as an initial screen for LGIB.12 As early
as 1976, studies comparing arteriogram and pan endoscopy
in LGIB were done. Chaudry5 found that 63% of 55 patients
studied with active bleeding were endoscopically controlled
even when the colon was unprepped. Haykir et al. 24 evaluated
magnetic resonance (MR) and CT colonography with conventional colonoscopy and found that MR was slightly more
accurate than CT and similar to conventional colonoscopy in
sensitivity with discovery of the lesion 96% of the time. (Level 3b
evidence).
Answer: Colonoscopy has traditionally been the most accurate method of diagnosing LGIB and considered to be the gold
standard for diagnosis and treatment of LGIB. (Grade C recommendation) Angiography is emerging as a possible firstline for diagnosis and treatment. (Grade D recommendation)
Scintigraphy may help determine candidates for angiographic
screening, but it is not sufficient for operative planning alone.
(Grade D recommendation) MDCT appears very promising as
a readily available, noninvasive modality for operative planning;
however, further studies are needed to confi rm this. (Grade D
recommendation)

3. Does urgent colonoscopy need to be performed for LGIB?


Historically, colonoscopy has been believed to be the most accurate method of diagnosing LGIB. At least two studies25 have
questioned this claim. Both studies suggest that urgent colonoscopy may confer little benefit for patients hospitalized for LGIB.
Unfortunately, small sample size (up to 85 patients studied) limits
interpretative power. Further, the majority of patients who bleed
less than four units of packed red blood cells (PRBC) cease bleeding without intervention.26 Colonoscopy is the diagnostic and
therapeutic treatment of choice for acute LGIB and the timing
must be determined by the clinical setting.13
Answer: Recently, a question has emerged regarding considering colonoscopy to be the traditional gold standard for the
diagnosis and treatment of LGIB. (Grade D recommendation) The
exact timing of colonoscopy needs further study.

PMPH_CH38.indd 315

315

4. What is the diagnostic accuracy of Tc-99 sulfur colloid


(Tc-99m) injection versus Tc-99 TRBCs?
In 1982, Alavi27 reported on the intravenous administration of
Tc-99m (Level 4 evidence) and the ability to detect bleeding rates
as low as 0.05 mL/min even in patients with negative arteriography. In the next year, Winn et al.28 presented a retrospective
review in 63 patients that suggested that if the Tc-99m study
were negative, then 15% of arteriograms were positive. At the
same time, other authors suggested 85% sensitivity, 100% specificity, and 91% accuracy with the Tc-TRBC, but sensitivity for
scintigraphy reported in the literature ranges from 20 to 95.29-31
Markisz et al.30 found no occurrences of positive arteriogram
when scintigraphy was negative. There is very little comparative
literature except in 1985 when Siddiqui et al.32 compared Tc-99m
and TRBC in 27 patients prospectively (Level 1b evidence) and
found a greater sensitivity in the TRBC group. Both Tc-99m and
TRBC produced positive results in 70% of patients studied.32
Other retrospective studies show a high sensitivity and less specificity with TRBC suggesting that its greatest utility may be as a
screen for patients in whom angiography will be helpful. Localization for surgical resection using either form of Tc-99 study
without a different localization study is often not possible.33-43
Further, arteriogram may be most helpful in those patients with
early positivity on scintigraphy.31
Answer: TRBC appears superior to Tc-99m injection. (Grade B
recommendation) In both tests, only about half of the patients
will have positive results. If scintigraphy is positive, further
localization studies are needed to improve anatomic accuracy
in surgical resection. (Grade C recommendation) Arteriography
has its greatest utility when scintigraphy is negative. (Grade C
recommendation) Overall, both these nuclear medicine scans
lack accuracy and should not be used in brisk bleeding. (Grade C
recommendation)
5. What is the clinical effectiveness of intra-arterial vasopressin infusion versus transcatheter embolization?
Vasopressin has been found to be highly (92%) effective in
stopping LGIB. (Level 4 evidence ) Athanasoulis et al.44 studied 24 patients hemorrhaging from colonic diverticulosis and
found that in 22 of the patients studied bleeding ceased with
the selective infusion of intra-arterial vasopressin. Fourteen
(58%) of the patients in this study underwent surgical resection
for persistent hemorrhage, rebleeding, or planned resection.
This is consistent with prior reporting.44,45 (Level 4 evidence)
Intra-arterial vasopressin has also been found to control colonic
hemorrhage in 63% of the patients studied with a rebleed rate of
16%.46 As practitioners have become more effective and adept with
intra-arterial infusion, rates of bleeding control have increased
up to at least 90%, but the rebleed rate can approach 50%.47
Since 1965 when Baum et al.48 described mesenteric angiography for the diagnosis of GI bleeding, this modality has
become well utilized in the diagnosis and treatment of LGIB.48
Although angiography was first employed as a diagnostic tool
where bleeding was identified by the identification of extravasation of contrast material, it is now used therapeutically by
selective and super-selective embolization. Numerous reports
show that intra-arterial embolization has similar-to-improved success rates than that of vasopressin, and concomitant increased
complications.45,49-52 (Level 4 evidence) Hemorrhage control

5/22/2012 5:27:33 PM

316

Surgery: Evidence-Based Practice

appears to be similar, but the rebleed rate is less with embolization than with vasopressin infusion.53 More recently, superselective embolization has been utilized with great success
and less necrotic complications.54-60 There appears to be no
difference between specific etiologies of LGIB and the rate of
embolization success.61
Answer: Intra-arterial vasopressin infusion is excellent (approximately 90% success rate) for cessation of bleeding, but has significant drawbacks such as systemic vasoconstriction, coronary
ischemia, and a variable but significant rate of rebleeding after
therapy cessation. (Grade D recommendation) Embolization is
as effective as vasopressin infusion, improves selective control,
avoids systemic side effects, and has less early rebleeding but an
increased rate of colon ischemia. (Grades C and D recommendations) Super-selective therapy does not eliminate ischemia risk
to the colon, has decreased the amount of surgery performed for
acute LGIB, and appears to decrease complication rates. (Grade D
recommendation) Late rebleeding rate is the same with vasopressin and embolic therapy.
6. What is the role of capsule and push endoscopy in LGIB?
The established diagnostic tools of endoscopic gastroduodenoscopy (EGD) and colonoscopy for GI bleeding have often been
used prior to angiography and scintigraphy. Capsule endoscopy
offers a novel approach to hemodynamically stable patients
and can lead to changes in clinical management and improved
outcomes.62-67 A meta-analysis showed that diagnostic yield is
greater for capsule than for push endoscopy and small bowel
radiography.68 Recent innovations and an increase in use of
capsule endoscopy have found its greatest utility in obscure GI
bleeding.69 Obscure GI bleeding occurs in up to 10% to 20% of
patients with LGIB and is most commonly defined as bleeding
that is persistent or recurrent without a source identified on standard investigational diagnostic tools such as upper endoscopy
and colonoscopy.70 Push enteroscopy has not been well studied
secondary to lack of widespread availability and patient stability
at the time of presentation.70
Answer: Patients with obscure (persistent, recurrent bleeding without an identified source) GI bleeding are candidates for
capsule or push endoscopy after negative upper endoscopy and
colonoscopy. (Grade D recommendation)
7. What are the criteria for surgical intervention in LGIB and
what operation should be done?
Patients who require more than two units of blood should receive
an evaluation to localize the source of bleeding expeditiously.71,72
Surgery is best for those patients that continue to bleed, those that
rebleed, and is ideally done after localization to minimize bowel
resection. Testing can delay therapy, but this must be balanced
with ensuring the source of bleeding, especially if it is proximal
or distal to the ileocecal valve.71 (Level 4 evidence) Localization
of massive LGIB lowers perioperative mortality when compared
with blind resection, but remains high at about 8% to 18%.8,11,23
(Level 4 evidence) The literature reveals great variability in rates

PMPH_CH38.indd 316

of morbidity of emergent subtotal colon resection. Many do


report that it is well tolerated and associated with a low rebleeding
rate although conflicting reports of high morbidity and mortality exist.16,73-76 Many patients with LGIB do not require colonic
resection. McGuire26 showed that bleeding stopped in 99% of
cases where four or less units of PRBC were transfused. (Level 4
evidence) Many authors have noted that up to 97% of patients
with LGIB are managed nonoperatively.77-79 Transfusion requirements have been offered as a reason to resect.80 Various recommendations have been made. Those who present to the emergency
department in a hypotensive state can benefit from earlier resection and has been found to be associated with lower perioperative
mortality.75,81
Answer: Grade C recommendations currently are that patients
who present with LGIB and receive more than two units of blood
should undergo expeditious localization of the bleeding source.
Persistent bleeding with anatomic localization can facilitate segmental resection. Patients who are not candidates for segmental
resection (i.e., unable to localize preoperatively) and continued
bleeding or rebleed are well treated with a subtotal colectomy and
ileo-rectostomy. (Grade C recommendations)

CONCLUSIONS
Due to the nature of LGIB, it is important to remember that
almost all recommendations are considered to be Grade C or D
defi ned by the U.S. Preventive Services Task Force as Grade C:
At least fair scientific evidence suggests that there are benefits
provided by the clinical service, but the balance between benefits
and risks are too close for making general recommendations.
Clinicians need not offer it unless there are individual considerations and Grade D: At least fair scientific evidence suggests
that the risks of the clinical service outweigh potential benefits.
Clinicians should not routinely offer the service to asymptomatic patients. Obviously, patients with LGIB are symptomatic
and the risk:benefit ratio is in favor of localization and bleeding
cessation.
Current recommendations in the management of LGIB:
Localization:
1. Localization of the bleeding with the cause and site should be
determined by the early use of colonoscopy, and use of CT,
CTA, or digital subtraction angiography.
2. Nuclear scintigraphy assists in the localization of bleeding and
the use of angiography.
Interventions:
1. Colonoscopic bleeding to control hemorrhage is best used in
acute diverticular and postpolypectomy bleeding.
2. Angiographic embolization.
3. Surgery-localized resection or subtotal colectomy. The extent
of surgical resection is often largely based on preoperative
localization studies and their accuracy.

5/22/2012 5:27:33 PM

Lower Gastrointestinal Bleeding

317

Clinical Question Summary


Question

Answer

Grade of
Recommendation

Level of
evidence

References

1 What is the ideal


order of diagnostic
testing in the setting
of LGIB?

Differentiate upper gastrointestinal (UGI)


from lower gastrointestinal (LGI) sources.
Colonoscopy may be the single best test at
most institutions and angiography at others. In
slow active bleeding starting with colonoscopy,
scintigraphy or CT followed by angiogram is
scintigraphy is positive.

CC, DC

444

4, 12, 54-88

2 What is the
diagnostic accuracy
of colonoscopy,
radionuclide
scanning, and
angiography in the
setting of LGIB?

Colonoscopy is the gold standard for diagnosis and


treatment of LGIB. Angiography is emerging as
a possible first-line for diagnosis and treatment.
Scintigraphy may help determine candidates
for angiographic screening and is not adequate
for preoperative planning alone. CT appears
promising as a highly available, noninvasive
modality for operative planning.

CD, DD

4444

2, 5-7, 11-24,
46

3 Does urgent
colonoscopy need
to be performed for
LGIB?

Urgent colonoscopy may not be needed as the


natural history for most bleeding is to stop.
The exact timing of colonoscopy needs
further study.

25

4 What is the
diagnostic accuracy
of Tc-99sulfur
colloid injection vs.
Tc-99 TRBCs?

Tc-99 labeled RBCs appear superior to Tc-99 sulfur


colloid injection. For all comers, about half of
the scans will be positive. Angiography is not
indicated when scintigraphy is negative. Positive
scans should be followed up with further
localization studies to improve
anatomic accuracy.

BC

1b4

27-43

5 What is the clinical


effectiveness of
intra-arterial
vasopressin infusion
vs. transcatheter
embolization?

Intra-arterial vasopressin ceases bleeding in the


majority of patients. Embolization is as effective
as vasopressin infusion, improves selective
control, avoids systemic effects, has less early
rebleeding but an increased rate of colon
ischemia. The late rebleeding risk is 10% to
15% with either technique.

D, C, D

444

44-46,
48-61, 53

6 What is the role of


capsule and push
endoscopy in LGIB?

Patients with obscure (persistent, recurrent


bleeding without an identified source) LGIB
are candidates for capsule or push endoscopy
after negative upper endoscopy and
colonoscopy, if available.

62-67, 70

7 What are the


criteria for surgical
intervention in LGIB
and what operation
should be done?

Patients who bleed two or more units of blood


should receive an evaluation to localize the
source of bleeding expeditiously. Clinical
stability and available testing options will
determine to what extent localization studies
can be performed so that excessive transfusion
is avoided (>10 units). Persistent bleeding
with true anatomic localization may allow
for segmental resection otherwise subtotal
colectomy with ileo-rectostomy should be
performed.

CCC

444

8, 11, 16, 23,


26, 71-81

PMPH_CH38.indd 317

5/22/2012 5:27:33 PM

318

Surgery: Evidence-Based Practice

REFERENCES
1. Talley NJ, Jones M. Self-reported rectal bleeding in the United
States community: Prevalence, risk factors, and health care seeking. Am J Gastroenterol. 1998;93(11):2179-2183.
2. Vernava AM, Longo WE, Virgo KS. A nationwide study of the
incidence and etiology of lower gastrointestinal bleeding. Surg Res
Commun. 1996;18:113-120.
3. Vorburger SA, Candinas D, Egger B. Acute lower gastrointestinal bleedingAn evidence-based algorithm for diagnosis and
treatment. Ther Umsch. 2006;63(5):301-309 (medline).
4. Levy R, Barto W, Gani J. Retrospective study of the utility of
nuclear scintigraphic-labelled red cell scanning for lower gastrointestinal bleeding. ANZ J Surg. 2003;73:205-209.
5. Chaudry V, Hyser MJ, Gracias VH, Gau FC. Colonoscopy: The initial test for acute lower gastrointestinal bleeding. Am Surg. 1998;64:
723-728.
6. Green BT, Rockey DC, Portwood G, et al. Urgent colonoscopy
for evaluation and management of acute lower gastrointestinal
hemorrhage: A randomized controlled trial. Am J Gastroenterol.
2005;100:2395-2402.
7. Whitaker SC, Gregson RH. The role of angiography in the investigation of acute or chronic gastrointestinal hemorrhage. Clin
Radiol. 1993;47:382-388.
8. Leitman IM, Paull DE, Shires GT, 3rd. Evaluation and management of massive lower gastrointestinal hemorrhage. Ann Surg.
1989;209:175-180.
9. Howarth DM, Tang K, Lees W. The clinical utility of nuclear
medicine imaging for the detection of occult gastrointestinal
hemorrhage. Nucl Med Commun. 2002;23:591-594.
10. Emslie JT, Zarnegar K, Siegel ME, Beart RW, Jr. Technetium-99mlabeled red blood cell scans in the investigation of gastrointestinal bleeding. Dis Colon Rectum. 1996;39:750-754.
11. Yoon W, Jeong YY, Shin SS, et al. Acute massive gastrointestinal
bleeding: Detection and localization with arterial phase multidetector row helical CT. Radiology. 2006;239:160-167.
12. Duchesne J, Jacome T, Serou M, et al. CT-angiography for the
detection of lower gastrointestinal bleeding source. Am Surg.
2005;71:392-397.
13. Barnert J, Messmann H. Diagnosis and management of lower gastrointestinal bleeding. Nat Rev Gastroenterol Hepatol. 2009;6(11):
637-646.
14. Hunter JM, Pezim ME. Limited value of technetium-99mlabeled red cell scintigraphy in localization of lower gastrointestinal bleeding. Am J Surg. 1990;159:504-506.
15. Syed MI, Shaikh A. Accurate localization of life threatening
colonic hemorrhage during nuclear medicine bleeding scan as
an aid to selective angiography. World J Emerg Surg. 2009;4(20):
1749-1752.
16. Cohn SM, Moller BA, Zieg PM, et al. Angiography for preoperative evaluation in patients with lower gastrointestinal bleeding:
Are the benefits worth the risks? Arch Surg. 1998;133:50-55.
17. Kim CY, Suhocki PV, Miller MJ, Jr., et al. Provocative mesenteric angiography for lower gastrointestinal hemorrhage: Results
from a single-institution study. J Vasc Interv Radiol. 2010;21(4):
477-483.
18. Junquera F, Quiroga S, Saperas E, et al. Accuracy of helical computed tomographic angiography for the diagnosis of colonic
angiodysplasia. Gastroenterology. 2000;119:293-299.
19. Ernst O, Bulois P, Saint-Drenant S, et al. Helical CT in
acute lower gastrointestinal bleeding. Eur Radiol. 2003;13:
114-117.

PMPH_CH38.indd 318

20. Yamaguchi T, Yoshikawa K. Enhanced CT for initial localization of active lower gastrointestinal bleeding. Adom Imaging.
2003;28:634-636.
21. Miller FH, Hwang CM. An initial experience: Using helical CT
imaging to detect obscure gastrointestinal bleeding. Clin Imaging.
2004;28(4):245-251.
22. Sabharwal R, Vladica P, Chou R, Law WP. Helical CT in the diagnosis of acute lower gastrointestinal haemorrhage. Eur J Radiol.
2006;58(2):273-279.
23. Jaeckle T, Stuber G, Hoffman MH, et al. Detection and localization of acute upper and lower gastrointestinal bleeding with arterial
phase multi-detector row helical CT. Eur Radiol. 2008;18:1406-1413.
24. Haykir R, Karakose S, Karabacakoglu A, et al. Three dimensional
MR and axial CT colonography versus conventional colonoscopy for detection of colon pathologies. World J Gastroenterol.
2006;12:2345-2350.
25. Laine L, Shah A. Randomized trial of urgent vs. elective colonoscopy in patients hospitalized with lower GI bleeding. Am J Gastroenterol. 2010;105(12):2636-2641.
26. McGuire HH, Jr. Bleeding colonic diverticula: A reappraisal of
natural history and management. Ann Surg. 1994;220:653-656.
27. Alavi A. Detection of gastrointestinal bleeding with 99mTcsulfur colloid. Semin Nucl Med. 1982;12(2):126-138.
28. Winn M, Weissmann HS, Sprayregen S, Freeman LM. The radionuclide detection of lower gastrointestinal bleeding sites. Clin
Nucl Med. 1983;8:389-395.
29. Miskowiak J, Nielsen SL, Munck O. Scintigraphic diagnosis of
gastrointestinal bleeding with 99mTc-labeled blood-pool agents.
Radiology. 1981;141:499-504.
30. Markisz JA, Front D, Royal HD, Sacks B, Parker JA, Kolodny
GM. An evaluation of 99mTc-labeled red blood cell scintigraphy
for the detection and localization of gastrointestinal bleeding
sites. Gastroenterology. 1982;83:394-398.
31. Ng DA, Opelka FG, Beck DE, et al. Predictive value of technetium Tc 99m-labeled red blood cell scintigraphy for positive
angiogram in massive lower gastrointestinal hemorrhage. Dis
Colon Rectum. 1997;40:471-477.
32. Siddiqui AR, Schauwecker DS, Wellman HN, Mock BH. Comparison of technetium-99m sulfur colloid and in vitro labeled
technetium-99m RBCs in the detection of gastrointestinal bleeding. Clin Nucl Med. 1985;10:546-549.
33. Friedman HI, Hilts SV, Whitney PJ. Use of technetium-labeled
autologous red blood cells in detection of gastrointestinal bleeding. Surg Gynecol Obstet. 1983;156:449-452.
34. Rosenkilde Olsen P, Nielsen L, Dyrbye M, Kuld Hansen L.
Colorectal bleeding localized with gamma camera. Acta Chir
Scand. 1983;149:793-795.
35. Orecchia PM, Hensley EK, McDonald PT, Lull RJ. Localization of lower gastrointestinal hemorrhage. Experience with red
blood cells labeled in vitro with technetium Tc 99m. Arch Surg.
1985;120:621-624.
36. Bearn P, Persad R, Wilson N, Flanagan J, Williams T.
99mTechnetium-labelled red blood cell scintigraphy as an alternative to angiography in the investigation of gastrointestinal
bleeding: Clinical experience in a district general hospital. Ann
R Coll Surg Engl. 1992;74:192-199.
37. Wang CS, Tzen KY, Huang MJ, Wang JY, Chen MF. Localization
of obscure gastrointestinal bleeding by technetium 99m-labeled
red blood cell scintigraphy. J Formos Med Assoc. 1992;91:63-68.
38. Dusold R, Burke K, Carpentier W, Dyck WP. The accuracy of
technetium-99m-labeled red cell scintigraphy in localizing gastrointestinal bleeding. Am J Gastroenterol. 1994;89:345-348.

5/22/2012 5:27:33 PM

Lower Gastrointestinal Bleeding

39. Rantis PC, Jr., Harford FJ, Wagner RH, Henkin RE. Technetiumlabelled red blood cell scintigraphy: Is it useful in acute lower
gastrointestinal bleeding? Int J Colorectal Dis. 1995;10:210215.
40. Dolezal J, Vizda J, Bures J. Detection of acute gastrointestinal
bleeding by means of technetium-99m in vivo labelled red blood
cells. Nucl Med Rev Cent East Eur. 2002;5:151-154.
41. Howarth DM, Tang K, Lees W. The clinical utility of nuclear
medicine imaging for the detection of occult gastrointestinal
haemorrhage. Nucl Med Commun. 2002;23:591-594.
42. Emslie JT, Zarnegar K, Siegel ME, Beart RW, Jr. Technetium-99mlabeled red blood cell scans in the investigation of gastrointestinal bleeding. Dis Colon Rectum. 1996;39:750-754.
43. Gutierrez C, Mariano M, Vander Laan T, Wang A, Faddis DM,
Stain SC. The use of technetium-labeled erythrocyte scintigraphy in the evaluation and treatment of lower gastrointestinal
hemorrhage. Am Surg. 1998;64:989-992.
44. Athanasoulis CA, Baum S, Rosch J, et al. Mesenteric arterial
infusions of vasopressin for hemorrhage from colonic diverticulosis. Am J Surg. 1975;129:212-216.
45. Waltman AC. Transcatheter embolization versus vasopressin infusion for the control of arteriocapillary gastrointestinal
bleeding. Cardiovasc Interv Radiol. 1980;3:289-295.
46. Sherman LM, Shenoy SS, Cerra FB. Selective intra-arterial vasopressin: Clinical efficacy and complications. Ann Surg. 1979;189:
298-302.
47. Browder W, Cerise EJ, Litwin MS. Impact of emergency angiography in massive lower gastrointestinal bleeding. Ann Surg.
1986;204:530-536.
48. Baum S, Nusbaum M, Blakemore WS, Finkelstein AK. Demonstration of intra-abdominal bleeding by selective arteriography.
Surgery. 1965;58(5):797-805.
49. Rahn NH, 3rd, Tishler JM, Han SY, Russinovich NA. Diagnostic
and interventional angiography in acute gastrointestinal hemorrhage. Radiology. 1982;143:361-366.
50. Rosenkrantz H, Bookstein JJ, Rosen RJ, Goff WB, 2nd, Healy JF.
Postembolic colonic infarction. Radiology. 1982;142:47-51.
51. Patel TH, Cordts PR, Abcarian P, Sawyer MA. Will transcatheter
embolotherapy replace surgery in the treatment of gastrointestinal bleeding?(2)(2). Curr Surg. 2001;58:323-327.
52. DeBarros J, Rosas L, Cohen J, Vignati P, Sardella W, Hallisey M.
The changing paradigm for the treatment of colonic hemorrhage:
Super selective angiographic embolization. Dis Colon Rectum.
2002;45:802-808.
53. Gomes AS, Lois JF, McCoy RD. Angiographic treatment of gastrointestinal hemorrhage: Comparison of vasopressin infusion
and embolization. AJR Am J Roentgenol. 1986;146:1031-1037.
54. Nawawi O, Young N, So S. Superselective coil embolization
in gastrointestinal haemorrhage: Early experience. Australas
Radiol. 2006;50:21-26.
55. Kickuth R, Rattunde H, Gschossmann J, Inderbitzin D, Ludwig
K, Triller J. Acute lower gastrointestinal hemorrhage: Minimally
invasive management with microcatheter embolization. J Vasc
Interv Radiol. 2008;19:1289-1296, e2.
56. Chin AC, Singer MA, Mihalov M, et al. Super selective mesenteric embolization with microcoils in a porcine model. Dis Colon
Rectum. 2002;45:212-218.
57. Burgess AN, Evans PM. Lower gastrointestinal haemorrhage and
super selective angiographic embolization. ANZ J Surg. 2004;74:
635-638.
58. Ledermann HP, Schoch E, Jost R, et al. Superselective coil embolization in acute gastrointestinal hemorrhage: Personal experience

PMPH_CH38.indd 319

59.

60.

61.

62.

63.

64.

65.

66.

67.

68.

69.

70.

71.

72.

73.

74.

75.

319

in 10 patients and review of the literature. J Vasc Interv Radiol.


1998;9(5):753-760.
Frodsham A, Berkmen T, Ananian C, Fung A. Initial experience using N-butyl cyanoacrylate for embolization of lower
gastrointestinal hemorrhage. J Vasc Interv Radiol. 2009;20(10):
1312-1319.
Kuo Wt, Lee De, Saad WE, et al. Superselective microcoil embolization for the treatment of lower gastrointestinal hemorrhage.
J Vasc Interv Radiol. 2003;14(2):1503-1509.
Khanna A, Ognibene SJ, Koniaris LG. Embolization as fi rst-line
therapy for diverticulosis-related massive lower gastrointestinal
bleeding: Evidence from a meta-analysis. J Gastrointest Surg.
2005;9(3):343-352.
Carey EJ, Leighton JA, Heigh RI, et al. A single-center experience of 260 consecutive patients undergoing capsule endoscopy for obscure gastrointestinal bleeding. Am J Gastroenterol.
2007;102(1):89-95.
Laine L, Sahota A, Shah A. Does capsule endoscopy improve
outcomes in obscure gastrointestinal bleeding? Gastroenterology. 2010;138(5):1673-1680.
Albert JG, Schulbe R, Hahn L, et al. Impact of capsule endoscopy
on outcome in mid-intestinal bleeding: A multicentre cohort
study of 285 patients. Eur J Gastroenterol Hepatol. 2008;20:
971-977.
Pennazio M, Santucci R, Rondonotti E, et al. Outcome of patients
with obscure gastrointestinal bleeding after capsule endoscopy:
Report of 100 consecutive cases. Gastroenterology. 2004;126:
643-653.
Neu B, Ell C, May A, et al. Capsule endoscopy versus standard
tests in influencing management of obscure digestive bleeding:
Results from a German multicenter trial. Am J Gastroenterol.
205;100:1736-1742.
Estevez E, Gonzalez-Conde B, Vazquez-Iglesias JL, et al. Diagnostic yield and clinical outcomes after capsule endoscopy in 100
consecutive patients with obscure gastrointestinal bleeding. Eur
J Gastroenterol Hepatol. 2006;18:881-888.
Triester SL, Leighton JA, Leontiadis GI, et al. A meta-analysis
of the yield of capsule endoscopy compared to other diagnostic
modalities in patients with obscure gastrointestinal bleeding.
Am J Gastroenterol. 2005;100:2407-2418.
Raju GS, Gerson L, Das A, et al. American Gastroenterological
Association (AGA) technical review on obscure gastrointestinal bleeding. Gastroenterology. 2007;133:1697-1717.
de Leusse A, Vahedi K, Edery J, et al. Capsule endoscopy or push
enteroscopy for first-line exploration of obscure gastrointestinal
bleeding? Gastroenterology. 2007;132:855-862.
Rozycki GS, Tremblay L, Feliciano DV, et al. Three hundred consecutive emergent celiotomies in general surgery patients: Influence of advanced diagnostic imaging techniques and procedures
on the diagnosis. Ann Surg. 2002;235:681-688; discussion 8-9.
Strate LL, Saltzman JR, Ookubo R, et al. Validation of a clinical
prediction rule for severe acute lower intestinal bleeding. Am J
Gastroenterol. 2005;100:1821-1827.
Baker R, Senagore A. Abdominal colectomy offers safe management for massive lower GI bleed. Am Surg. 1994;60:578-581;
discussion 82.
Farner R, Lichliter W, Kuhn J, et al. Total colectomy versus limited colonic resection for acute lower gastrointestinal bleeding.
Am J Surg. 1999;187:587-591.
Field RJ, Sr., Field RJ, Jr., Shackleford S. Total abdominal colectomy for control of massive lower gastrointestinal bleeding.
J Miss State Med Assoc. 1994;178:587-591.

5/22/2012 5:27:33 PM

320

Surgery: Evidence-Based Practice

76. Setya V, Singer JA, Minken SL. Subtotal colectomy as a last resort
for unrelenting, unlocalized, lower gastrointestinal hemorrhage:
Experience with 12 cases. Am Surg. 1992;58:295-299.
77. Bokhari M, Vernava AM, Ure T, Longo WE. Diverticular hemorrhage in the elderlyis it well tolerated? Dis Colon Rectum.
1996;39:191-195.
78. Al Qahtani AR, Satin R, Stern J, Gordon PH. Investigative
modalities for massive lower gastrointestinal bleeding. World J
Surg. 2002;26:620-625.

PMPH_CH38.indd 320

79. Buttenschoen K, Buttenschoen DC, Odermath R, Beger HG.


Diverticular disease-associated hemorrhage in the elderly. Langenbecks Arch Surg. 2001;386:8-16.
80. Rios A, Montoya MJ, Rodriguez JM, et al. Severe acute lower gastrointestinal bleeding: Risk factors for morbidity and mortality.
Langenbecks Arch Surg. 2007;392:165-171.
81. Clarke CS, Afifi AY. Impact of blood transfusion on outcome in
patients admitted for gastrointestinal hemorrhage. Curr Surg.
2000;57:493-496.

5/22/2012 5:27:33 PM

PART V

THE LIVER

PMPH_CH39.indd 321

5/22/2012 5:28:06 PM

PMPH_CH39.indd 322

5/22/2012 5:28:06 PM

CHAPTER 39

Newer Techniques in Liver Surgery


Anil S. Paramesh, Robert Cannon, Joseph F. Bue11

1. What is new in liver imaging studies?

showing lower attenuation. Three-dimensional (3D) reconstructions are also possible, allowing assessment of spatial relationships with vascular structures to plan resection.
Magnetic resonance imaging (MRI) has recently gathered popularity as a valuable imaging tool for liver masses/resection. Triplephase MRI scans using gadolinium-based contrast are possible,
with a much lower volume of contrast. Two newer MRI contrast
agents gadobenate dimeglumine (MultiHANCE) and gadoxetate
disodium (Eovist) have a dual route of excretion, both by renal and
hepatobiliary means. Five percent of the injected dose is taken up
by hepatocytes and excreted through the biliary system. Thus, liver
lesions which contain hepatocytes will take up contrast and can be
differentiated from ones that do not on delayed films.

Imaging of the liver is integral to a liver surgeons perspective,


allowing for proper planning of liver resections. Imaging studies have continued to improve over the past few years and have
increased the scope of liver surgery preparation.
Although ultrasound is the current standard for surveillance
of chronic hepatitis patients for hepatocellular carcinoma (HCC),1
it is with the knowledge that it has decreased sensitivity for smaller
lesions and it may be difficult to find smaller masses in a cirrhotic liver.
Newer techniques such as contrast-enhanced ultrasound use carbon dioxide microbubbles to help detect and differentiate liver
masses and evaluate flow differences in the liver parenchyma.2
Contrast ultrasound detects liver masses better than routine
ultrasound. One study determined that the sensitivity and specificity of contrast-enhanced ultrasonography for discrimination of
benign versus malignant lesions was 100% and 63%, respectively.3
This modality is useful for detecting smaller lesions (<1 cm), very
superficial lesions, or lesions close to the ligamentum teres.
One of the advances in ultrasonic evaluation of the liver is
transient elastography measurement. With this technique, we can
measure the stiffness of the liver noninvasively, which would correlate with the level of fibrosis, without the need for a liver biopsy.4
Such assessment would be an adjunct in preoperative assessment
for a liver resection.
Intraoperative ultrasound remains a valuable tool during a
liver resection. Smaller lesions (between 2 and 5 mm) can be picked
up with this technique, all of these smaller lesions may have been
missed by other imaging studies.5 With the advent of laparoscopic
liver resections, laparoscopic probes are available to be used in
such cases as well.
The most commonly performed test for liver masses remains
a triple-phase (unenhanced, arterial, and portal phases) contrast CT scan. With newer 64 row scanners, we are now able to
see much better anatomy. Arterial enhancing masses with rapid
portal washout are diagnostic for primary liver cancers and have
almost done away with the need for biopsies. Metastatic lesions are
also well visualized, with some (renal cell/breast primary) showing arterial enhancement, while others (colon/stomach primary)

2. Is ablation equivalent to resection for liver tumors?


Ablation of liver tumors, especially radiofrequency ablation (RFA),
has rapidly gained popularity in the last decade. By this technique,
high-frequency alternating current is applied via probes into
tumor tissue. Ions in the tissue that attempt to follow the change
in direction of current become heated. As the temperature of the
local tissue elevates beyond 60C, necrosis occurs.6 Most RFA
probes are placed by ultrasound guidance. A zone of necrosis usually 0.5 to 1 cm around the mass is typically sought-after to ensure
complete tumor necrosis. RFA was initially tested for HCC, but it
is now being used for colorectal cancer (CRC) metastases as well
with good outcomes reported.7
However, RFA has its limitations. Recurrence has been quoted
to be one of the biggest drawbacks of this technique when compared with resection.8 RFA is harder to accomplish with lesions
>4 cm.9 Although larger probes are being developed, in most
instances multiple overlapping ablations may be necessary to get
the entire tumor with margins. This leads to the risk of small nests
of tumor cells being left behind. Another risk is the presence of
large vessels adjoining the tumor. This may offer thermal protection from the heat of the probes, and adjacent tumor may not heat
enough to necrose. RFA is also difficult to perform on lesions near
the dome or adjacent stomach or colon, for risk of injury. Finally,
recurrence may be dependent on operator technical expertise.
323

PMPH_CH39.indd 323

5/22/2012 5:28:06 PM

324

Surgery: Evidence-Based Practice

Despite this, RFA has been reported to have up to 46% 3-year


survival rates post treatment of CRC metastases,7 which was similar to surgical resection results. Five-year survival rates of 54%
have been reported post primary RFA treatment for HCC.10
So, is RFA a better option than surgical resection? RFA is certainly a minimally invasive option and can be done percutaneously
or laparoscopically these days. The overall complication rate would
be lower with RFA versus resection. RFA can spare uninvolved
liver, which may need to be removed as part of a resection. RFA
is certainly cheaper than a liver resection. RFA can be performed
again for local recurrence with good local control.10
However, the comparison may not be that easy. To our knowledge, there are no good prospective randomized clinical trials
comparing RFA with resection for CRC metastases. Many comparisons are retrospective, and in most cases, the RFA patients
tend to have lesser disease than their resection controls. In the
setting of HCC, there have few studies doing actual comparisons.
One study from China11 randomized 180 patients to ablation versus resection. When analyzed for intention-to-treat, there was
no difference in overall and disease-free survival rates at 4 years
between the groups. Another study from Italy12 showed sustained
(median follow-up 31 months) complete response rate of 97.2%
and 5-year survival rates of 68.5% when small (2 cm) HCC were
treated with RFA as primary therapy in 218 patients. These results
are comparable to resection candidates.
Thus, there is some evidence that RFA can effectively treat
and control small malignancies of the liver when done appropriately. It may be considered as possible therapy for a small, centrally
located lesion, with no significant adjacent vessels, and meticulous
follow-up to watch for local recurrence. However, given the lack
of long-term studies comparing RFA with resection, the gold standard probably remains a partial resection.
3. Is percutaneous ablation equivalent to operative ablation?
RFA via a less invasive route such as a percutaneous ablation in a
radiology suite may be attractive compared with a surgical procedure requiring general anesthesia in the operating room. Indeed,
previous studies had shown a higher morbidity rate with open
surgical RFAs compared with radiological percutaneous RFAs.13
However, when looking at the most important variables, including local recurrence and survival rates, most of the literature would
support a surgical approach, whether it is open or laparoscopic.
There is little doubt that an intraoperative ultrasound would
be able to pick up additional smaller lesions that may be missed by
transcutaneous ultrasound or even a CT/MRI scan (CT, computed
tomography).5 This may even change the planned procedure.
In a large meta-analysis of 5224 patients that had RFA for primary and secondary lesions in the liver, a surgical approach (open
or laparoscopic) was shown in a multivariate analysis to be an
independent factor associated with reduced recurrence rates compared with a percutaneous approach.14 Another study looked at
228 HCC tumors undergoing percutaneous versus surgical RFA.
The surgical group had better 1- and 3-year survival compared
with the percutaneous group for tumors >3 cm in size.15
Also, with minimally invasive laparoscopy, the perceived benefit
of a percutaneous approach may be less. The reported complication
rates after a laparoscopic RFA are no different from a percutaneous
approach.13 Aside from laparoscopic ultrasonography, retractors
may be placed to push the liver away from adjacent viscera, reducing the chance of injury and ensuring a better ablation.

PMPH_CH39.indd 324

There is some evidence that temporary occlusion of hepatic


flow (Pringle maneuver) during the RFA, either via open or laparoscopic approach, may increase the success rates of RFA.14 As discussed previously, large blood vessels adjacent to tumor may act as
a heat sink, not allowing the tumor to reach necrosis temperatures.
However, with temporary occlusion of blood flow, this risk factor
is negated. This concept is even being attempted for percutaneous
RFAs, via balloon catheter occlusion.16
Thus, while the success rates of RFA via any approach is very
operator-dependant, based on current literature, it seems that a
surgical approach may be able to visualize tumors better, protect
against visceral injury and treat lesions better. With a laparoscopic
approach, surgical morbidities may be further reduced.
4. Is there a benefit to resecting noncolorectal liver metastases?
There is little doubt that surgical resection, when possible, offers
good chance of long-term tumor-free survival. It has also been
demonstrated that surgical resection of metastatic neuroendocrine tumors, including the possibility of liver transplantation,
also offers a proven survival benefit.17 However, it is the possibility
of survival benefit for resection of noncolorectal, nonneuroendocrine (NCRNNE) liver metastases that appears controversial and
undecided as yet.
Most published studies had few patients and are not comparable with each other as they may include some NCRNNE metastases, different primary malignancies, different chemotherapy
regimens, and different time-to-onset of tumor (synchronous vs.
metachronous).18
Despite this, it remains clear that certain patients with
NCRNNE liver metastases may actually benefit from resection.19 In
general, there should be no extrahepatic spread; a single metastatic
lesion would fare better than multiple lesions; synchronous metastases fare worse than metachronous ones. In the case of metachronous tumors, a longer disease-free interval (DFI) from the primary
tumor (usually at least 1224 months) indicates a better prognosis.
As would be expected, a proper surgical resection with adequate
margins would lend to better results, and debulking procedures
without the benefit of adjuvant therapies may be wasted.
With regard to individual primary cancers, foregut primaries
tend to have worse prognoses than non-foregut ones.20 Resecting
hepatic metastases from pancreas, stomach, and cutaneous melanomas generally have bad prognoses; metastases from kidney,
adrenal, ovarian, and testicular tumors tend to have better prognoses; and metastases from sarcomas, breast, and uveal melanomas tend to have an intermediate prognosis.18,19
Pancreatic cancer: Among pancreatic primary tumors, a pancreatic cystadenocarcinoma may have better prognosis than a
pancreatic ductal adenocarcinoma.21 A DFI >24 months would be
preferable. In a series reported by Yamada et al.,22 six patients underwent curative hepatectomy for metastatic pancreatic cancer. Five of
these patients died of recurrence between 4 and 52 months, the nonrecurrence occurring in a patient who had cystadenocarcinoma; and
the 1-, 3- and 5-year survival rates were 66.7%, 33.3%, and 16.7%.
Stomach cancer: Synchronous en-bloc resection of a stomach
cancer in contact with the left lobe of the liver may result in longterm survival.23 In a metaanalysis by Shirabe et al,24 there were 12
studies and a total of 229 patients who underwent hepatic resection for metastatic gastric cancer. More than 50% of patients died
of recurrence within 2 years, and only 11% (25 patients) made it
beyond 5 years. Interestingly, 11 of these 25 survivors underwent

5/22/2012 5:28:06 PM

Newer Techniques in Liver Surgery

synchronous resections. Some of the possible prognostic factors implicated were the presence of lymphovascular invasion in
the primary tumor and surgical margins <10 mm (both poor
prognosis).
Breast cancer: Liver resection has been associated with increased survival for metastatic breast carcinoma in selected patients.25
In a review of nine published studies of metastatic breast cancer to
the liver (all over 10 patients), Elias et al.26 noted that the median
overall survival of the 296 patients was 57 months with curative
resection. Aside from undergoing a curative resection, a hormonenegative breast tumor was a significant (poor) prognostic factor in
older women.
Ovarian cancer: Cytoreductive surgery, in combination with
chemotherapy, is the standard treatment for advanced ovarian
carcinoma. The inclusion of hepatic resection as part of a cytoreductive surgical plan has been shown to improve survival.27 In a
review of 35 patients with liver resections for metastatic ovarian
cancer, the Mayo Clinic showed that overall median disease-free
survival was 27.4 months. Among patients with no macroscopic
residual disease, the median survival has increased to 41.3 months.
Residual tumor <1 cm was associated with significantly improved
survival, as was a DFI of >1 year. Number, distribution, or tumor
grade of liver metastases did not appear to affect survival.
Renal cancer: Over the course of their disease, up to 50% of
renal cancer patients will develop metastases, and 20% of these
will develop hepatic metastases (although may not be confined to
liver). Most of these cancers are adenocarcinomas which tend to be
aggressive. In a report of their experience with review of the literature, Aloia et al.28 described 19 patients who underwent liver resection for metastatic renal cell carcinoma. Five-year disease-free and
overall survival rates were 25% and 26%, respectively. Significant
positive prognostic factors were a DFI of >24 months, metastases size <5 cm, R0 resection, and male sex. With the introduction
of newer chemotherapeutic agents for renal cancer,29 it is possible
that survival rates will continue to improve.
Other cancers: There have been reports of liver metastatic resections from other primary tumors, including lung,30 sarcomas,31 and
adrenocortical32 cancers, but most of these series are very small. A
longer DFI between primary cancer and metastases development,
as well as trying to complete an R0 resection remain the best prognostic factors.
5. Is HCC better treated with resection or transplant?
HCC accounts for 80% to 90% of primary liver cancers. Worldwide,
the incidence of new cases ranges between 500,000 and 1 million
annually.33 The treatment regimen for HCC is controversial in
different parts of the world for several reasons; the etiology of
HCC, extent of liver disease and availability of donor organs
for transplant.
The etiology of the liver cancer is important; in the Far East
nations, where Hepatitis B is rampant, HCC tends to occur in a
younger demographic, usually in the fourth to fift h decade. In the
western countries, however, other etiologies tend to cause HCC
more commonly, especially Hepatitis C and more recently, nonalcoholic steatohepatitis. These diseases tend to cause HCC in an
older demographic, usually the sixth or seventh decade and are
also associated with more comorbidities. Hence, a liver resection
may be harder among this group.
Improved surgical care has demonstrated improved outcomes
after liver resections for HCC. For patients with well-preserved

PMPH_CH39.indd 325

325

liver function, minimal portal hypertension, tumor confined


within the liver and with no major vascular involvement, liver
resection can provide good results. Currently, overall 5-year
survival after liver resection ranges between 30% and 60% with
only a 3% operative mortality rate.33 In very early cases of HCC,
5-year survival rates >90% has even been reported after resection.34 Unfortunately, <30% of patients with diagnosed HCCs can
undergo an operative resection. This is usually because of preexisting advanced liver disease (cirrhosis) precluding this. In a study
looking at 295 patients undergoing resection for HCC, 4-year
survival rates were twofold higher among noncirrhotic patients
than among cirrhotic patients.35 Another problem with resection
for HCC is recurrence of tumor. HCC will likely recur in up to
50% to 80% of patients after resection within 2 years, based on the
size and characteristics of the tumor.36 But re-resection is always
an option in a healthy patient.
A normal liver can tolerate up to 80% resection and will likely
regenerate within 4 to 6 weeks. However, HCC tends to occur in
the background of cirrhosis. Resection ability may depend on the
Child-Pugh class of the patient; in general, class A patients may
tolerate a 50% resection of their liver and a class B may tolerate
up to 25%. A Childs class C patient usually cannot tolerate a liver
resection.
There are no large randomized trials that have directly compared resection with transplant in cirrhotic patients. There have
been several single-center studies that have shown overall and
recurrence-free survival benefits for transplantation compared
with resection in Childs A and B patients with early HCC.37,38
Liver transplantation for HCC came to the forefront after a
landmark study by Mazzaferro et al.39 in 1996. The study demonstrated that patients transplanted with small hepatic tumors (single
tumor <5 cm or up to three tumors, each <3 cm) could have excellent outcomes with 4-year survival rates of 75%. These rates are
equivalent to transplantation for cirrhosis without tumor. In addition, recurrence rates post transplant are low.40 These criteria are
now known as the Milan criteria and are widely used across the
world as criteria for transplantation.
In the United States today, HCC candidates are given priority for transplant as long as they fall within the Milan criteria.
Liver transplant candidates are currently ranked by a scoring system called the MELD (Model for Endstage Liver Disease) score
(in pediatrics, called the PELD score).41 The score is calculated
by a logarithmic formula using the patients creatinine, INR,
and bilirubin levels. The calculated score ranges between 6 and
40 (capped at 40 maximum). Patients with HCC are automatically given a priority score of 22 (unless their calculated score is
already higher), with a 10% increase in score every 3 months that
they are not transplanted. This is because in most cases, these
patients may not have enough hepatic dysfunction to derive a
higher score by themselves, and the value of transplanting these
patients while they are still in the window period of being within
the Milan criteria is recognized.
The last point to discuss is the availability of donor livers. Many countries do not have a nationwide allocation system
that would allow transplantation to happen in a consistent fashion. Even in the United States, although these patients are given
priority points on their MELD score, this may not be enough to
get them transplanted in a timely fashion. Based on geographic
region, there is a huge variability in average MELD scores in the
United States.42 In regions with large cities (and consequent large
waiting lists), the average MELD score of patients transplanted is

5/22/2012 5:28:06 PM

326

Surgery: Evidence-Based Practice

higher than the 22 priority points assigned to HCC candidates.


This may be in contrast to regions with smaller lists, where HCC
patients may be transplanted faster. Although there appears to be
some discrepancy in nationwide allocation, this system currently
exists to allow local organs to stay locally, so as to reduce ischemic
time. Unfortunately, the current dropout rate for HCC candidates
on the waitlist is about 20%,43 usually because of progression of
their cancer.
Because of this scarcity of available organs, transplant centers
frequently will consider bridging therapy to allow more wait time
on the list. This may include initial resection, chemoembolization,
or ablation therapies.1 The benefit of such therapies is debatable,
although there is some evidence that this will reduce tumor burden and reduce dropout rates on the waiting list, although it may
not increase overall survival.
There are other factors to consider as well; studies looking at
newer immunosuppression therapies and chemotherapy agents
that may reduce the recurrence rates of HCC either after resection
or transplant are currently underway and may impact the decision of transplant versus resection in the future.
In summary, there is probably little argument that resection
for HCC is probably beneficial for a healthier individual, with
early tumor and no cirrhosis. Likewise, transplantation is probably the logical choice in a frankly cirrhotic patient with early
tumor. The controversy still lies within the best treatment option
for early HCC in a mildly cirrhotic patient. There is some evidence
that transplant may offer better overall and recurrence-free survival. However, this decision must be weighed with consideration
of local availability of livers and waiting time. The benefit of bridging therapies and newer agents to prevent recurrence is yet to be
determined.
6. Is laparoscopic liver surgery going to become the standard
of care?
With the advent of laparoscopic surgery in various surgical arenas, it seems logical that liver surgery may be approached through
laparoscopic means as well. Laparoscopic liver resection was
first reported in 1992.44 Most of the initial reports on this procedure usually involved minor wedge resections. However, since
that time, several centers have reported large laparoscopic series
containing major liver resections for cancer, posterior liver resections, and even living donor hepatectomies. With the introduction of devices such as the Harmonic Scalpel , LigaSureTM, and
staplers which can be used to divide the liver laparoscopically, this
procedure is becoming commonplace.
The benefits of a laparoscopic liver resection are similar to
the benefits of any other laparoscopic procedure; studies have
shown less pain, less chance of intra-abdominal adhesions,
better cosmesis, shorter hospital length of stay with equivalent complication rates compared with open resections, and
maintenance of oncologic goals when necessary.45,46 In a review
of 2804 laparoscopic liver resections performed worldwide,46
overall mortality was 0.3% and morbidity was 10.5%. The most
common procedure was a wedge resection (45%); hemihepatectomies accounted for about 20% of all the operations. Conversion to an open procedure occurred in 4.1% of procedures.

PMPH_CH39.indd 326

In a single-center review of 300 laparoscopic liver resections


compared with 100 contemporaneous open resections that were
matched for age, type of resection, cirrhosis, and malignancy,
Koff ron et al.47 showed that the laparoscopic group had shorter
operative times, less blood loss and transfusions, less complications, and shorter hospital length of stay. There is also evidence
that a laparoscopic resection in a cirrhotic patient may lead to
less post operative ascites.48
Some of the early concerns of laparoscopic liver resection
relate to a significant learning curve, for example, increased
risk of bleeding which may be difficult to control and gas embolism. Although laparoscopic liver resection remains a complicated procedure, there is gaining acceptance of this procedure
throughout the world. Concerns of possible carbon dioxide
embolism have largely been put to rest, as no significant complications from this have been noted. There have been no significant reports of port site metastases. There have been isolated
reports of argon gas embolization when it used for coagulation
on large vessels,49 and caution is advised on such cases. Because
most reports have shown decreased operative and recovery
times, in most cases, a laparoscopic approach may be cheaper
as well.
There have been no randomized trials comparing laparoscopic with open resections for cancer resections of the liver. A
meta-analysis of eight retrospective studies comparing laparoscopic versus open liver resections for malignancy between 1998
and 200550 showed similar overall and disease-free survival at
5 years post surgery; operative blood loss and hospital length of
stay were significantly shorter in the laparoscopic group. In the
reported literature, surgical margins free of cancer were attained
in 81% to 100% of cases.46
Laparoscopic surgery is now being performed routinely in
some centers for living donation as well. Th is pushes the envelope
even further, for now the liver resection needs to be performed
with meticulous preservation of the vessel length and the bile
ducts. The parenchymal dissection is typically performed without division of the blood supply, thus lending to increased risk of
bleeding. However, centers have started reporting good outcomes
with left lateral segmentectomies51 and even right hepatectomies
in donors,52 compared with open surgeries. The ethical question
of whether this is safe for a donor arises. But similar to the acceptance of laparoscopic donor nephrectomies, laparoscopic donor
hepatectomies can offer the same advantages of better cosmesis,
recovery, and acceptance.
So, will laparoscopy replace open procedures in the setting
of liver resections? There is no doubt that there is a steep learning
curve and the mindset of established surgeons may need revision.
However, there is no evidence that this approach is any riskier,
and in many instances, may even be better than open procedures.
Similar to other major laparoscopic surgeries today, there may
be a time of growing pains when this transition occurs. Future
training of surgeons may start with laparoscopic resections, with
open procedures limited to complicated patients, similar to the
cholecystectomies performed today. At the Northwestern University in Chicago, laparoscopic liver resections have increased from
10% to 80% of all liver resection procedures performed between
2002 to 2007.47

5/22/2012 5:28:06 PM

Newer Techniques in Liver Surgery

327

Clinical Question Summary


Question

Answer

1 What is appropriate radiologic screening for


HCC among high risk patients

Ultrasound q 6 to 12 months

2 What is the appropriate radiologic study for


suspicious HCC lesions

Contrast CT/MRI/US. At least one


study for lesions >2 cm. At least two
concordant studies for lesions between
1 and 2 cm.

3 Is RFA equivalent to resection for liver tumors?

Although there are good retrospective


results from selected RFA candidates,
resection, when possible, remains
standard of care.

7-12

4 Is percutaneous RFA equivalent to operative


RFA?

While there may be some operator


dependency, several large series have
shown better survival and decreased
recurrence rates with operative RFA.

14, 15

5 Is there a benefit to resection for non


colorectal, non neuroendocrine metastases to
the liver?

May be of benefit in selected patients.


Only limited case series are reported.

18-32

6 Is early HCC with mild cirrhosis better treated


with resection or transplant?

While there is some evidence that


recurrence rates may be lower with
transplant, availability of livers locally
needs to be figured. Benefit of adjuvant
therapy needs to be determined.

37, 38

7 Is laparoscopic resection for liver cancer


equivalent to open resections?

No RCTs. However, several series have


shown equivalent oncologic results
with less surgical morbidity. Data
suggests it may be equivalent in
selected patients.

46, 50

8 Is laparoscopic living donor hepatectomy


equivalent to open procedures?

No RCTs. Limited case series show good


results.

51, 52

REFERENCES
1. Bruix J, Sherman M; Practice Guidelines Committee, American Association for the Study of Liver Diseases. Management of
hepatocellular carcinoma. Hepatology. 2005;42(5):1208-1236.
2. Nicolai C, Bru C. Focal liver lesions: Evaluation with contrastenhanced ultrasonography. Abdom Imaging. 2004;29:348.
3. Von Herbay A, Vogt C, Haussinger D. Late phase pulse-inversion
sonography using the contrast agent levovist; differentiation
between benign and malignant focal lesions of the liver. Am J
Roentgenol. 2002;179:1273.
4. Foucher J, Chanteloup E, Vergniol J, et al. Diagnosis of cirrhosis
by transient elastography (Fibroscan): A prospective study. Gut.
2006;55:403-408.
5. Kruskal JB, Kane RA. Intraoperative ultrasound of the liver. Crit
Rev Diagn Imaging. 1995;36:175.
6. McGahan JP, Brock JM, Tesluk H, et al. Hepatic ablation with
use of radio-frequency electrocautery in the animal model.
J Vasc Interv Radiol. 1992;3:291.
7. Solbiati L, Livraghi T, Goldberg SN, et al. Percutaneous radiofrequency ablation of hepatic metastases from colorectal cancer: Long term results in 117 patients. Radiology. 2001;221:
159-166.

PMPH_CH39.indd 327

Grade of
References
Recommendation

8. Tanabe KK, Kulu Y. Radiofrequency ablation for colon and rectal


carcinoma liver metastases: Whats missing? Gastrointest Cancer
Res. 2007;1(4)(Suppl 2):S42-S46.
9. Tanabe KK, Curley SA, Dodd GD, Siperstein AE. Radiofrequency
ablation. Cancer. 2004;100:641.
10. Tateishi R, Shiina S, Teratani T, et al. Percutaneous radiofrequency for hepatocellular carcinoma. Cancer. 2005;103:1201.
11. Chen MS, Li JQ, Zheng Y, et al. A prospective randomized trial
comparing percutaneous local ablative therapy and partial hepatectomy for small hepatocellular carcinoma. Ann Surg. 2006;
243:321.
12. Livraghi T, Meloni F, Di Stasi M, Rolle E, Solbiati L, Tinelli C,
Rossi S. Sustained complete response and complication rates
after radiofrequency ablation of very early hepatocellular carcinoma in cirrhosis: Is resection still the treatment of choice?
Hepatology. 2008;47(1):82-89.
13. Burdio F, Mulier S, Navarro A, et al. Influence of approach on
outcome in radiofrequency ablation of liver tumors. Surg Oncol.
2008;17:295-299.
14. Mulier S, Ni Y, Jamart J, Ruers T, Marchal G, Michel L. Local
recurrence after hepatic radiofrequency coagulation. Multivariate
analysis and review of contributing factors. Ann Surg. 2005;242(2):
158-171.

5/22/2012 5:28:06 PM

328

Surgery: Evidence-Based Practice

15. Khan MR, Poon RTP, Ng KK, et al. Comparison of percutaneous and surgical approaches for radiofrequency ablation of small
and medium hepatocellular carcinoma. Arch Surg. 2007;142(12):
1136-1143.
16. Yamasaki T, Kurokawa F, Shirahashi H, Kusano N, Hironaka K,
Okita K. Percutaneous radiofrequency ablationtherapy for patients
with hepatocellular carcinoma during occlusion of hepatic blood
flow. Comparison with standard percutaneous radiofrequency
ablation therapy. Cancer. 2002;95(11):2353-2360.
17. Lang H, Oldhafer KJ, Weimann A, et al. Liver transplanta
tion for metastatic neuroendocrine tumors. Ann Surg. 1997;225:
347-354.
18. DiCarlo I. Liver surgery for noncolorectal nonneuroendocrine
metastases. HPB. 2006;8:83-84.
19. Detry O, Warzee F, Polus M, DeRoover A, Meurisse M, Honor
P. Liver resection for noncolorectal, nonneuroendocrine metastases. Acta Chir Belg. 2003;103:458-462.
20. Earle SA, Perez EA, Gutierrez JC, et al. Hepatectomy enables
prolonged survival in select patients with isolated noncolorectal
liver metastasis. J Am Coll Surg. 2006;203(4):436-446.
21. Berney T, Mentha G, Roth AD, Morel P. Results of surgical resection of liver metastases from non-colorectal primaries. Br J Surg.
1998;85:1423-1427.
22. Yamada H, Hirano S, Tanaka E, Shichinohe T, Kondo S. Surgical treatment of liver metastases from pancreatic cancer. HPB.
2006;8:85-88.
23. Bines S, England G, SDeziel D, et al. Synchronous, metachronous and multiple hepatic resections of liver tumors originating
from primary gastric tumors. Surgery. 1993;114:799-805.
24. Shirabe K, Wakiyama S, Gion T, et al. Hepatic resection for the
treatment of liver metastases in gastric carcinoma: Review of
the literature. HPB. 2006;8:89-92.
25. Elias D, Maisonnette F, Druet-Cabanac M, et al. An attempt to
clarify indications for hepatectomy for liver metastases from
breast cancer. Am J Surg. 2003;185:158-164.
26. Elias D, Di Pietroantonio D. Surgery for liver metastases from
breast cancer. HPB. 2006;8:97-99.
27. Bosquet JG, Meredith MA, Podratz KC, Nagorney DM. Hepatic
resection for metachronous metastases from ovarian carcinoma.
HPB. 2006;8:93-96.
28. Aloia TA, Adam R, Azoulay D, Bismuth H, Castaing D. Outcome
following hepatic resection of metastatic renal tumors: The Paul
Brousse Hospital experience. HPB. 2006;8:100-105.
29. Cooney MM, Remick SC, Vogelzang NJ. Novel agents for the
treatment of advanced kidney cancer. Clin Adv Hematol Oncol.
2004;2:664-670.
30. Ercolani G, Ravaioli M, Grazi GL, et al. The role of liver resections for metastases from lung carcinoma. HPB. 2006;8:114-115.
31. Stavrou G, Flemming P, Oldhafer KJ. Liver resection for metastases due to malignant mesenchymal tumors. HPB. 2006;8:110-113.
32. DiCarlo I, Toro A, Sparatore F, Cordio S. Liver resection for
hepatic metastases from adrenocortical carcinoma. HPB. 2006;8:
106-109.
33. Lau W, Lai ECH. Hepatocellular carcinoma: Current management and recent advances. Hepatobiliary Pancreat Dis Int. 2008;
7:237-257.
34. Takayama T, Makuuchi M, Hirohashi S, et al. Early hepatocellular carcinoma as an entity with a High rate of surgical cure.
Hepatology. 1998;28(5):1241-1246.

PMPH_CH39.indd 328

35. Yamanaka N, Okamoto E, Toyosaka A, et al. Prognostic factors


after hepatectomy for hepatocellular carcinomas. A univariate
and multivariate analysis. Cancer. 1990;65(5):1104-1110.
36. Poon RT, Ng IO, Fan ST, et al. Clinicopathologic features of
long-term survivors and disease-free survivors after resection of
hepatocellular carcinoma: A study of a prospective cohort. J Clin
Oncol. 2001;19(12):3037-3044.
37. Bismuth H, Chiche L, Adam R, Castaing D, Diamond T, Dennison A. Liver resection versus transplantation for hepatocellular
carcinoma in cirrhotic patients. Ann Surg. 1993;218(2):145-151.
38. Colella G, Bottelli R, De Carlis L, et al. Hepatocellular carcinoma:
Comparison between liver transplantation, resective surgery,
ethanol injection, and chemoembolization. Transpl Int. 1998;
11(Suppl 1):S193-S196.
39. Mazzaferro V, Regalia E, Doci R, et al. Liver transplantation for
the treatment of small hepatocellular carcinomas in patients
with cirrhosis. N Engl J Med. 1996;334(11):693-699.
40. Zimmerman MA, Ghobrial RM, Tong MJ, et al. Recurrence of Hepatocellular Carcinoma Following Liver Transplantation - A Review
of Preoperative and Postoperative Prognostic Indicators. Arch
Surg. 2008;143(2):182-188.
41. Wiesner R, Edwards E, Freeman R, et al. Model for end-stage liver
disease (MELD) and allocation of donor livers. Gastroenterology
2003;124(1):91-96.
42. Trotter JF, Osgood MJ. MELD scores of liver transplant recipients
according to size of waiting list: Impact of organ allocation and
patient outcomes. JAMA. 2004;291(15):1871-1874.
43. Yao FY, Bass NM, Nikolai B, et al. Liver transplantation for
hepatocellular carcinoma: Analysis of survival according to the
intention-to-treat principle and dropout from the waiting list.
Liver Transpl. 2002;8(10):873-883.
44. Gagner M, Rheault M, Dubuc J. Laparoscopic partial hepatectomy for liver tumor [abstract]. Surg Endosc. 1992;6:99.
45. Buell JF, Cherqui D, Geller DA, et al. The international position
paper on laparoscopic liver surgery. The Louisville statement,
2008. Ann Surg. 2009;250(5):825-830.
46. Nguyen KT, Gamblin C, Geller DA. World review of laparoscopicty liver resection 2804 patients. Ann Surg. 2009;250(5):
831-841.
47. Koff ron AJ, Auffenberg G, Kung R, et al. Evaluation of 300 minimally invasive liver resections at a single institution; less is more.
Ann Surg. 2007;246:385-392.
48. Belli G, Fantini C, DAgostino A, et al. Laparoscopic versus open
liver resection for hepatocellular carcinoma with histologically
proven cirrhosis short and middle term results. Surg Endosc.
2007;21:619-624.
49. Min SK, Kim JH, Lee SY. Carbon dioxide and argon gas embolism
during laparoscopic liver resection. Acta Anaesthesiol Scand. 2007;
51:949-951.
50. Similis C, Constantinidis VA, Tekkis PP, et al. Laparoscopic
versus open hepatic resections for benign and malignant neoplasms a meta-analysis. Surgery. 2007;141:203-211.
51. Soubrane O, Cherqui D, Scatton O, et al. Laparoscopic left
lateral sectionectomy in living donors; safety and reproducibility of the technique in a single center. Ann Surg. 2006;244(5):
815-820.
52. Baker TB, Jay CL, Ladner DP, et al. Laparoscopy-assisted and
open living donor right hepatectomy: A comparative study of
outcomes. Surgery. 2009;146(4):817-823.

5/22/2012 5:28:06 PM

CHAPTER 40

Hepatic Infections: Pyogenic


Abscess, Amebic Abscess,
and Hydatid Cyst
David M. Levi and Andreas G. Tzakis

especially hepatobiliary and pancreatic cancer.4 Does this change


reflect a true increase in incidence or simply the experience of
the authors? The inherent bias of case series limits a high-level
evidence-based answer. A group from Alberta, Canada reported
an incidence of 2.3 per 100,000 hospital admissions, the majority
of patients being male and older.5 In most large case series, the
average patient age is between 50 and 70 years, with a slight predominance of men.4-6 Pyogenic liver abscesses are more common
in Asia; a retrospective series from Taiwan reported an incidence
of 446 cases per 100,000 hospital admissions.7 The authors speculate that different bacteriology and patient factors accounts for
this variation.
Amebiasis results from infection with the protozoan parasite
Entamoeba histolytica. The liver is the most common extraintestinal site of ameba infection. The organism is found throughout the
world and the infection is common in places with inadequate sanitation. In the United States, the amebic liver disease is seen predominantly in individuals that have traveled to or have emigrated
from endemic areas.8 In adults, men are more commonly affected
than women; while children, boys and girls are equally affected.
Possibly, the higher rate of alcohol use by men contributes to this
difference.8 Conditions that affect cell-mediated immunity, such
as extremes of age, pregnancy, corticosteroid therapy, malignancy,
and malnutrition, may also increase the chances that E. histolytica
infection results in invasive disease with liver involvement.
Hydatid cystic disease of the liver results from infection
with the larval form of the tapeworm Echinococcus granulosus.
Although found worldwide, it is endemic in South America, Mediterranean countries, the Middle East, Australia, and Asia.9 The
liver is the most common site for echinococcal infection followed
by the lung. In children, pulmonary involvement is more common
than liver disease.10
Answer: Pyogenic abscess, amebic abscess, and hydatid cyst
of the liver are distinct disease entities, with varying epidemiologic characteristics. Not surprisingly, their pathophysiology and
clinical characteristics vary as well. (Grade C recommendation).

INTRODUCTION
Among the various infectious diseases that affect the liver, pyogenic abscess, amebic abscess, and hydatid cyst have historically
been the concern of the surgeon. The fundamentals of our understanding of pyogenic and amebic abscesses can be traced to two
seminal publications by Ochsner et al.1,2 in the 1930s. For hydatid
liver disease, which was rare in the United States, a publication
in English of equivalent importance had to wait until Moreno
Gonzalez et al.s 1991 report describing their 25-year experience
in Spain.3 Though separated by more than half a century, each
publication made a major impact on the surgical management of
liver abscess and hydatid cyst.
From the critical perspective of evidence-based practice,
these papers present Class IIIIV data with Grade C recommendations. Liver abscess and hydatid disease are relatively uncommon and occur in a heterogeneous patient population, making
rigorous, high-level clinical studies difficult to perform. The high
morbidity and mortality associated with these hepatic infections
is proof that clinically relevant questions regarding their management remain inadequately answered. Recently, trials that address
specific aspects of the diagnosis and treatment of liver abscess
and hydatid cyst have emerged, which meet the high standards of
evidence-based practice and will be noted in this chapter.
1. What are the incidence and epidemiologic characteristics of
liver abscesses and hydatid cysts?
Pyogenic liver abscesses are relatively uncommon, but their incidence varies by geographic region and patient population. In 1938,
Ochsner et al.2 reported an incidence of 8 per 100,000 admissions to New Orleans Charity Hospital.2 A large series from the
Johns Hopkins Hospital in Baltimore reported that from 1973 to
1993 the incidence increased from 13 to 20 per 100,000 hospital
admissions and that the increase was attributed to the increase in
patients seen during that time interval with malignant disease,
329

PMPH_CH40.indd 329

5/22/2012 5:28:39 PM

330

Surgery: Evidence-Based Practice

2. What is the pathophysiology of liver abscesses and hydatid


cysts? For pyogenic abscesses, what are the most common causative organisms? What are the clinical features of the three
entities?
Most pyogenic liver abscesses arise secondary to an infection that
originates elsewhere in the body. They can be categorized by the
mode of spread to the liver. Knowledge of these routes and mechanisms aids in diagnosis and often dictates treatment. The liver can
become the site of abscess formation via (1) the biliary tree, from
ascending cholangitis; (2) the portal vein, as in pylephlebitis resulting from appendicitis or diverticulitis; (3) the hepatic artery, as in
bacteremia from endocarditis or an oral cavity abscess; (4) direct
extension, from a contiguous disease process; and (5) post traumatic, from penetrating injuries or iatrogenic events. Rarely no
source is found.
In the past, the most common underlying etiology was acute
appendicitis.2 However, biliary tract pathology is now the most common cause of pyogenic liver abscess, accounting for 40% to 60% of
cases.11,12 Malignant biliary obstruction has been noted to be a more
frequent cause than in the past.4,13 Pyogenic abscess formation as
a procedure-related complication is being reported anecdotally in
recent years. These iatrogenic events include abscess formation after
biliary intervention, hepatic tumor ablation or embolization, hepatic
artery thrombosis after liver transplantation, and hepatic artery or
biliary injury during laparoscopic cholecystectomy.12,14-17
The microbiology of pyogenic liver abscesses varies and
often reflects the underlying etiology and route of liver involvement. Many liver abscesses are polymicrobial, including anaerobes. Klebsiella pneumoniae, Escherichia coli, and Enterococcus
species predominate in series where biliary tract pathology is
the common etiology.11 In case series from New York and San
Diego, K. pneumoniae was the most common cause of pyogenic
liver abscess and was noted to be particularly virulent.11,18 Staphylococcus aureus, Streptococcus species, Pseudomonas aeruginosa,
and Candida species are among the important but less common
causative organisms.19,20 Because the causative organism is unpredictable and may be due to resistant organisms, cultures from the
abscess (including those for anaerobic bacteria) are important for
determining the optimal antimicrobial therapy.
Liver abscesses due to E. histolytica infection occur most commonly in patients that have spent time in an endemic area. Infection
occurs when individuals ingest food or water contaminated with
feces containing E. histolytica; sexual transmission is uncommon.
Although E. histolytica infection occurs in men and women equally,
invasive amebic diseases, such as abscess formation, predominate
in men at a ratio of about 3:1.21 Once ingested, the organism penetrates the intestinal mucosa eventually reaching the portal venous
system. In the liver, E. histolytica has membrane-based molecules
that shield it from complement mediated lysis and releases proteases
that destroy host IgA and IgG allowing abscess formation.8
Hydatid cyst of the liver is caused by the parasitic tapeworm,
E. granulosus. Infected canines (definitive host) harbor the adult
tapeworm in their small intestine and shed eggs in their feces. Livestock (intermediate host), including sheep, goats, pigs, and cows, are
exposed to the parasite when they ingest eggs in fecally contaminated food or water. Humans are considered accidental intermediate
hosts.22 Ingested eggs release oncospheres which penetrate the small
intestine, access the circulation, and are carried to various organs,
usually the liver or lungs. In the target organ, a cystic structure
develops and gradually enlarges stimulating a host inflammatory

PMPH_CH40.indd 330

response. A fibrous, sometimes calcified pericyst forms and within


the cyst protoscolices and daughter cysts proliferate.
Answer: Pyogenic abscess, amebic abscess, and hydatid cyst
are distinct infectious diseases with different pathophysiological
aspects. This accounts for their differences in clinical presentation, diagnosis, and treatment. (Grade C recommendation).
3. What are the clinical features of the three entities and how
can these be distinguished?
Many of the clinical characteristics of the three entities are similar, nonspecific, and inconsistently present making their respective
diagnosis dependent on adjunct testing and imaging. Pain, localized to the right-upper quadrant and epigastrium, is common.
Pain radiating to the right scapular region suggests right hemidiaphragm irritation. Fever is common to all three entities; nausea and
vomiting, weight loss, malaise, anorexia, chills are less common,
but equally nonspecific.22-25 Patients with amebiasis can have diarrhea from colitis and symptoms from a simultaneous liver abscess.8
Elderly patients may present with only fever of unknown origin.
The physical examination findings of fever, upper abdominal tenderness, and hepatomegaly may suggest the diagnosis of
a hepatic infection. These coupled with relevant historical clues
may suggest the diagnosis. Travel to an endemic area may suggest
amebic abscess or hydatid cyst, whereas a recent biliary procedure
or history of diverticulitis suggests pyogenic abscess. In a large
series of adults in Pakistan, distinguishing between patients with
pyogenic and amebic abscess was studied. The authors found that
patients with pyogenic abscess were usually older with a history
of diabetes, more likely to present with jaundice, and pulmonary
findings. Patients with amebic abscess were younger with epigastric pain, lower serum albumin levels, and positive amebic titres.26
It is unlikely that their conclusions hold true for patients in nonendemic areas.
Complications resulting from hepatic infections can be dramatic and life-threatening. Patients with pyogenic abscess can
present with systemic sepsis and hemodynamic instability. Hydatid
cysts can rupture into the peritoneal cavity causing anaphylaxis, the
biliary tree causing cholangitis, the pleural space, or pericardial sac.
To complicate diagnostic efforts further, hydatid cysts can become
secondarily bacterially infected yielding a mixed pyogenic abscess.
Answer: Because the signs and symptoms of hepatic infection
are nonspecific, we depend heavily on adjunct testing and imaging for making a diagnosis and distinguishing between the three
entities. (Grade C recommendation).
4. What diagnostic tests and imaging studies are useful for
obtaining an accurate diagnosis?
Diagnosis of and differentiation between pyogenic abscess, amebic abscess, and hydatid cyst on clinical grounds may be impossible. Other diagnoses in the differential, including cystic liver
neoplasms, can further complicate diagnostic efforts. Serologic
tests are available to assist in the diagnosis of E. histolytica or
E. granulosus infection. In the setting of a mass lesion of the liver,
both are highly sensitive and specific.8,22 Microscopic examination of stool samples or a rectal mucosal biopsy may yield amebic trophozoites, supporting but not proving the diagnosis of
an amebic abscess. Once a cystic liver lesion is identified to be a
pyogenic abscess, computerized tomography (CT) or ultrasound
(US) guided aspiration of the cyst fluid is important to isolate the
causative organism(s) and tailor antimicrobial therapy.

5/22/2012 5:28:39 PM

Hepatic Infections: Pyogenic Abscess, Amebic Abscess, and Hydatid Cyst

Modern imaging studies that exploit the nuanced radiographic


characteristics of liver abscesses and hydatid cysts have become
imperative for an accurate diagnosis. CT is slightly superior to US
for detecting small liver abscesses; both have made radionucleotide
scans second-line studies.4 Magnetic resonance imaging (MRI)
can occasionally be useful but does not add greatly to CT imaging.
Also, CT offers the potential for guided, diagnostic aspiration and
therapeutic drainage; a distinct advantage over MRI.
Pyogenic liver abscesses are solitary or multifocal discrete
masses, usually round or lobulated. Contrast-enhanced CT demonstrates peripheral rim enhancement with central low attenuation.
They can be complex with loculations and may have an air-fluid
level. In a recent study of 58 patients with pyogenic abscess, CT
was the imaging study used for making the diagnosis in 56 cases,
or 97% of the time.13
CT and US are the imaging modalities of choice for diagnosing
amebic liver abscess. Both are very sensitive but lack needed specificity. Imaging may reveal solitary or multiple lesions, usually less complex than pyogenic abscesses. Ultrasonographic features include a
smooth wall, hypoechoic center with internal echoes. Because many
amebic abscesses are treated nonoperatively, serial US examinations
are ideal for tracking the progress of medical therapy.
The distinct morphologic features of hydatid cysts are usually detected on imaging studies and can help discriminate them
from liver abscesses. Hydatid cysts may be single or large with
multiple satellites. The wall can appear thickened, surrounded by
pericyst of compressed liver parenchyma. Calcification of the wall,
when present, is seen on CT and produces vivid acoustic shadows
on US. Septations and lobulations are readily detected by both
modalities. US is especially good for detecting the sediment-like
cyst contents made of scolices. The sediment moves, remaining in
the dependent portion of the cyst as the patients position is varied
for the exam.
Answer: The use of serologic testing for amebic abscess and
hydatid cyst, and bacterial and fungal cultures of aspirates from a
suspected pyogenic abscess are important tools aided in accurate
diagnosis. CT and US have become very important for detecting
and characterizing liver abscesses and cysts and guiding treatment. (Grade C recommendation).
5. What is the treatment for liver abscess and hydatid cyst?
In recent years, evidenced-based studies have emerged and address
controversies regarding the treatment of pyogenic abscess, amebic abscess, and hydatid cyst. The fundamental precepts guiding
the treatment of pyogenic liver abscess are the administration of
appropriate antibiotics and/or antifungal agents, drainage of the
abscess, and treatment of the root cause. When the abscess is secondary to biliary obstruction, biliary drainage via the transhepatic or endoscopic retrograde route may be required.
Historically, open surgery with either abscess drainage or
hepatic resection was regarded as the treatment of choice.4 Percutaneous drainage has developed with the emergence of interventional radiology, and has almost replaced open surgical drainage.13
Correspondingly, there has been a significant decrease in mortality related to pyogenic abscess.11 There are still proponents of
open surgical drainage as more effective for patients with large,
multifocal, and multiloculated abscesses, but the trend toward
percutaneous drainage is clear. 27 In a nonrandomized series
comparing open drainage with percutaneous drainage, Ferraioli
et al. 28 reported no percutaneous drainage failures while open

PMPH_CH40.indd 331

331

surgical drainage was associates with longer hospitalization and


greater morbidity.28
Small abscesses, defined as >3 cm may be treated with antibiotics alone in selected patients.29 In a randomized trial of 64
patients, Yu et al.30 demonstrated that percutaneous aspiration
without catheter placement was as effective as percutaneous drainage, with no difference in length of hospitalization or mortality.30
The mainstay of treatment for amebic liver abscess is metronidazole. The majority of patients will respond needing no
drainage. There is controversy regarding the role of percutaneous drainage in the management of amebic liver abscess. It has
been suggested that aspiration or drainage of large abscesses may
shorten the time to resolution. In one detailed review, the author
suggests that intervention beyond medical management should be
reserved for patients for whom the diagnosis is uncertain, seriously ill patients that may benefit from more rapid treatment, and
patients that have not responded as expected with resolution of
fever and decreased abdominal pain within 4 days.8
The treatment of echinococcal cyst depends on the patients
condition and on the presence or absence of complicating factors.
In two reports, Dziri et al.31,32 carefully conducted a comprehensive review of the evidence-based treatment strategies for hydatid
cyst. For uncomplicated cysts, randomized trials have shown
that management alone with albendazole and/or mebendazole is
superior to placebo, but is not dependable for achieving complete
resolution of all cysts.
Surgery remains the standard treatment for hydatid cysts
in the liver, although the optimal procedure remains a question.
The main surgical options include hepatic resection and pericystectomy (radical procedures) and cyst fenestration and evacuation of the cyst contents (conservative procedures). Although
there is no randomized comparison, most series suggest that radical resection is associated with lower mortality. In general, radical
procedures probably pose greater perioperative risks and conservative procedures have a higher associated cyst recurrence rate.31
Placement of the omentum in the surgical bed after resection or
pericystectomy or in the residual cyst after fenestration results in
fewer complications such as abscess formation.33 Laparoscopic
fenestration has been demonstrated to be safe, but high level data
are lacking with regard to its effectiveness compared with an open
procedure. Percutaneous drainage plus albendazole or mebendazole is a strategy for treating uncomplicated hydatid cysts that has
gained the support of some researchers.31 Complicated hydatid
liver cysts includes those that rupture or fistulize into adjacent
organs or cavities. In general, more radical procedures have been
advocated for complicated cysts. Because of the complexity and
variability of complicated hydatid cysts, it is difficult to generalize from existing series as to the best approach.32 A combination
of medical treatment and an individualized, aggressive surgical
approach is usually warranted.
Answer: The treatment of pyogenic liver abscess has become
less invasive over the years, with percutaneous drainage and antibiotics supplanting surgery in most cases. Amebic liver abscess
respond to medical treatment with metronidazole, although there
is occasionally a role for percutaneous drainage or aspiration. Surgery is the standard therapy for hydatid liver cysts. Radical procedures are effective, but carry higher perioperative risk than more
conservative procedures. More study is needed before laparoscopic and percutaneous procedures can be advocated. (Grade B
recommendation).

5/22/2012 5:28:39 PM

332

Surgery: Evidence-Based Practice

Clinical Question Summary


Question

Answer

1 What are the incidence


and epoidemiologic
characteristics of liver
abscesses and hydatid cysts?

Pyogenic abscess, amebic abscess, and hydatid cyst


of the liver are distinct diseases. Epidemiologic,
pathophysiologic, and clinical characteristics vary
as well.

2, 4-10

2 What is the pathophysiology


of liver abscesses and hydatid
cysts? For pyogenic abscesses,
what are the most common
causative organisms? What
are the clinical features?

Pyogenic abscess, amebic abscess, and hydatid cyst


are distinct infectious diseases with different
pathophysiological aspects. This accounts for their
differences in clinical presentation, diagnosis, and
treatment.

2, 4, 11-22

3 What are the clinical features


of the three entities and how
can these be distinguished?

Because the signs and symptoms of hepatic infection are


nonspecific, we depend heavily on adjunct testing and
imaging for diagnosis and distinguishing between the
3 entities.

8, 22-26

4 What diagnostic tests and


imaging studies are useful
for obtaining an accurate
diagnosis?

Serologic testing for amebic abscess and hydatid cyst,


and bacterial and fungal cultures of aspirates from
a suspected pyogenic abscess are aids to accurate
diagnosis. CT and US are important for detecting and
characterizing liver abscesses and cysts, and guiding
treatment.

4, 8, 13, 22

5 What is the treatment for liver


abscess and hydatid cyst?

Percutaneous drainage and antibiotics have supplanted


surgery in most cases. Amebic liver abscess are treated
with metronidazole and occasionally with percutaneous
drainage or aspiration. Surgery is the standard therapy
for hydatid liver cysts, More study is needed before
laparoscopic and percutaneous procedures can be
advocated.

4, 8, 11, 13,
27-33

REFERENCES
1. Ochsner A, DeBakey M. Liver abscess part I: Amebic abscess
analysis of 73 cases. Am J Surg. 1935;29:173-194.
2. Ochsner A, DeBakey M, Murray S. Pyogenic abscess of the liver:
II. An analysis of 47 cases with review of the literature. Am J
Surg. 1938;40:292-319.
3. Moreno Gonzalez E, Rico Selas P, Marinez B, et al. Results of
surgical treatment of hepatic hydatidosis: Current therapeutic
modifications. World J Surg. 1991;15:254-263.
4. Huang CJ, Pitt HA, Lipsett PA, et al. Pyogenic hepatic abscess.
Ann Surg. 1996;223:600-607.
5. Kaplan GG, Gregson DB, Laupland KB. Population-based study
of the epidemiology of and risk factors for pyogenic liver abscess.
Clin Gastroenterol Hepatol. 2004;2:1032-1038.
6. Alvarez Perez JA, Gonzalez JJ, Baldonedo RF, et al. Clinical
course, treatment, and multivariate analysis of risk factors for
pyogenic liver abscess. Am J Surg. 2001;181:177-186.
7. Chan KS, Chen CM, Cheng KC, et al. Pyogenic liver abscess: A
retrospective analysis of 107 patients during a 3-year period. Jpn
J Infect Dis. 2005;58:366.
8. Stanley SL. Amoebiasis. Lancet. 2003;361:1025-1034.
9. Sayek I, Bulent Tirnaksiz M, Dogan R. Cystic hydatid disease:
Current trends in diagnosis and management. Surg Today.
2004;34:987-996.

PMPH_CH40.indd 332

Grade of
References
Recommendation

10. Balci AE, Eren N, Eren S, Ulku R. Ruptured hydatid cysts of the
lung in children: Clinical review and results of surgery. Ann Thorac Surg. 2002;74:889-892.
11. Rahimian J, Wilson T, Oram V, et al. Pyogenic liver abscess: Recent
trends in etiology and mortality. Clin Infect Dis. 2004;39:1654-1659.
12. Lam YH, Wong SK, Lee DW, et al. ERCP and pyogenic liver
abscess. Gastrointest Endosc. 1999;50:340-344.
13. Mezhir JJ, Fong Y, Jacks LM, et al. Current management of pyogenic liver abscess: Surgery is now second-line treatment. J Am
Coll Surg. 2010;210:975-983.
14. Mezhir JJ, Fong Y, Fleischer D, et al. Pyogenic abscess after hepatic
artery embolization: A rare but potentially lethal complication.
J Vasc Interv Radiol. 2011;22:177-182.
15. Kong WT, Zhang WW, Qiu YD, et al. Major complications
after radiofrequency ablation for liver tumors: Analysis of 255
patients. World J Gastroenterol. 2009;15:2651-2656.
16. Nikeghbalian S, Salahi R, Salahi H, et al. Hepatic abscesses after
liver transplant: 19972008. Exp Clin Transplant. 2009;7:256-260.
17. Stewart L, Robinson TN, Lee CM, et al. Right hepatic artery injury
associated with laparoscopic bile duct injury: Incidence, mechanism, and consequences. J Gastrointest Surg. 2004;8:523-530.
18. Lederman ER, Crum NF. Pyogenic liver abscess with a focus
on Klebsiella pneumoniae as a primary pathogen: An emerging
disease with unique clinical characteristics. Am J Gastroenterol.
2005;100:322-331.

5/22/2012 5:28:40 PM

Hepatic Infections: Pyogenic Abscess, Amebic Abscess, and Hydatid Cyst

19. Smith BM, Zyromski NJ, Allison DC. Community-acquired


methicillin-resistant Staphylococcus aureus liver abscess requiring resection. Surgery. 2007;141:110-111.
20. Ulug M, Gedik E, Girgin S, et al. Pyogenic liver abscess caused by
community acquired multidrug resistance Pseudomonas aeruginosa. Braz J Infect Dis. 2010;14:218.
21. Acuna-Soto R, Maguire JH, Wirth DF. Gender distribution in
asymptomatic and invasive amebiasis. Am J Gastroenterol. 2000;95:
1277-1283.
22. Eckert J, Deplazes P. Biological, epidemiological, and clinical
aspects of echinococcosis, a zoonosis of increasing concern. Clin
Microbiol Rev. 2004;17:107-135.
23. Conter RL, Pitt HA, Tompkins RK, et al. Differentiation of pyogenic
from hepatic abscess. Surg Gynecol Obstet. 1986;162:114-120.
24. Akgun Y, Tacyildiz IH, Celik Y. Amebic liver abscess: Changing
trends over 20 years. World J Surg. 1999;23:102-106.
25. Hoffner JR, Kilaghbian T, Esekogwu VI, et al. Common presentations of amebic liver abscess. Ann Emerg Med. 1999;34:351-355.
26. Lodhi S, Sarwari AR, Muzammil M, et al. Features distinguishing amebic from pyogenic liver abscess: A review of 577 adult
cases. Tropic Med Inter Health. 2004;9:718-723.

PMPH_CH40.indd 333

333

27. Tan YM, Chung AY, Chow PK, et al. An appraisal of surgical and
percutaneous drainage for pyogenic liver abscesses greater than
5 cm. Ann Surg. 2005;241:485-490.
28. Ferraioli G, Garlaschelli A, Zanaboni D, et al. Percutaneous
and surgical treatment of pyogenic liver abscesses: Observation
over a 21-year period in 148 patients. Dig Liver Dis. 2008;40:
690-696.
29. Hope WW, Vrochides DV, Newcomb WL, et al. Optimal treatment of hepatic abscess. Am Surg. 2008;74:178-182.
30. Yu SC, Ho SS, Lau WY, et al. Treatment of pyogenic liver abscess:
Prospective randomized comparison of catheter drainage and
needle aspiration. Hepatology. 2004;39:932-938.
31. Dziri C, Haouet K, Fingerhut A. Treatment of hydatid cyst
of the liver: Where is the evidence? World J Surg. 2004;28:
731-736.
32. Dziri C, Haouet K, Fingerhut A, Zaouche A. Management of
cystic echinococcosis complications and dissemination: Where
is the evidence? World J Surg. 2009;33:1266-1273.
33. Balik AA, Basoglu M, Celebri F, et al. Surgical treatment of
hydatid disease of the liver: Review of 304 cases. Arch Surg. 1999;
134:166-169.

5/22/2012 5:28:40 PM

CHAPTER 41

Malignant Liver Tumors


Susanne Carpenter and Yuman Fong

INTRODUCTION

Although there is widespread acceptance of the use of American


Joint Committee on Cancer (AJCC) staging criteria for colorectal hepatic metastases,15 there exists a fair amount of controversy
regarding the staging of hepatocellular carcinoma (HCC). The
most obvious weakness is that AJCC staging focuses on anatomic
extent of disease without considering treatment or liver function
status, and is based on survival of patients undergoing resection, which makes it of limited use to the majority of patients
with HCC who present with late stage disease or with medical
contraindications to resection.16,17 Several different centers have
developed their own tumor staging and classification systems,
but these have yielded disparate survival rates at supposedly
early stages, likely secondary to the construction of these staging systems with patients at advanced tumor stage or disparate
liver functions.18-21 To circumvent some of these deficiencies, a
consensus conference originating from the American Hepato
Pancreato Biliary Association (AHPBA) concluded that a one
or both of the AJCC and Cancer of the Liver Italian Program
(CLIP) stages should be reported.17 Nonetheless, the most frequently used and most accurate algorithm to predict outcomes
following resection or transplantation in the United States is
currently the AJCC.22-24

Primary and metastatic hepatic malignancies present aggressive


disease processes that require aggressive management strategies.
Although chemotherapy has an important role, particularly in
patients presenting with unresectable disease, surgical and ablative
therapies are mainstays of both palliative and curative treatments.1-6
Since the advent of true anatomic hepatic resections in the 1950s,
surgical indications for hepatectomy have gradually expanded with
increasing safety and decreasing mortality rates.7,8 Recent decades
have seen the advent of segmental hepatic resections, which have
ushered in a new era of expanded definitions of respectability.8 Historically, bilobar disease was considered unresectable.9 However,
some authors are now advocating aggressive resection strategies
for bilobar disease with neoadjuvant chemotherapy and portal vein
embolization with subsequent attempts at curative and palliative
surgical interventions with the intent of prolonging survival.9,10
Although the evidence for partial hepatectomy is not based on any
randomized controlled trial (RCT), the conclusions of many single
and multi-institutional experiences have shaped current trends in
hepatobiliary surgery, and have produced outcomes that obviate
continued pursuit of currently accepted surgical management.11,12
One example of this is the Milan criteria, which originated from
a single-institutional series and has been built upon by many different institutions.13,14 Unfortunately, practice patterns in the treatment of malignant liver tumors can vary widely by institution, and
there is a glut of institutional protocols in todays literature in the
absence of RCTs.
This chapter will review the evaluation, diagnosis, and management of primary and secondary hepatic malignancies with
specific attention to optimal use of imaging modalities, staging
criteria, treatment algorithms, as well as minimally invasive, ablative, and palliative therapies.

2. What imaging techniques are best used for delineating


malignant from benign liver lesions?
Preoperative work-up of patients with hepatic malignancies typically includes some combination of ultrasound (US), computed
tomography (CT), magnetic resonance imaging (MRI), and [18F]
fluorodeoxyglucose (FDG)-positron emission tomography (PET).
For metastatic disease, a combination of PET and MRI or PET/CT
is frequently used.25 The superiority of PET scanning in detecting
extrahepatic disease over CT scanning has been demonstrated
by an RCT wherein patients were scanned with either CT alone
or with CT with a separate PET scan, which demonstrated that
futile laparotomy could be avoided for one in every six patients
with enhanced detection of extrahepatic disease.26 Similarly, the
superiority of MRI over CT has been demonstrated by multiple

INITIAL EVALUATION AND DIAGNOSIS


1. What is the standard staging of patients prior to hepatic
resection?
334

PMPH_CH41.indd 334

5/22/2012 5:29:14 PM

Malignant Liver Tumors

strenuous comparisons and meta-analyses thereof.27,28 However,


much debate surrounds the optimal combination of preoperative
and surveillance imaging techniques, with many concluding that
while on a per-patient basis, PET has consistent sensitivity greater
than 90% and superior to MRI sensitivity around 75%, the two
modalities perform similarly on a per-patient basis.27-30 Regardless
of the level of support for PET scanning, in preoperative imaging,
a surgical map detailing proximity of lesions to major vascular structures, and in relationship to segmental liver anatomy is
required, such that PET cannot be employed as a sole means of
imaging. It must be used in concert with either CT or MRI.
The primary weakness of PET is its inability to detect lesions
<1 cm in diameter.31,32 Chemotherapy further complicates imaging choices by producing changes in liver parenchyma that can
obscure otherwise obvious differences in tissue echogenicity
or enhancement.33-37 For instance, hepatic lesions exhibit impaired FDG uptake following chemotherapy when compared with
chemotherapy-nave patients, likely as a result of diminished
hexokinase activity.38 CT accuracy also suffers following chemotherapy in that chemotherapy-induced steatohepatitis results in a
darker-appearing liver, which results in obscured views of residual hypovascular (thus hypoattenuated) residual tumors.29,33,39 US
can be useful intraoperatively for guidance as to lesion location,
and to ensure that lesions have been completely resected; however,
fatty livers can also impair US accuracy, particularly in lesions
<1 cm in diameter.40 In the delineation of HCC, PET is less useful,
and US, CT, and MRI are more commonly employed.41
Radiologists assert that contrast-enhanced imaging is essential in the differentiation of malignant from benign hepatic
lesions, and point out that uniform features can be used to accurately establish malignant diagnoses via imaging.42 These features
relate to tumor appearance (heterogeneity vs. homogeneity with
contrast) and the interface of a tumor with its surroundings, and
were established by comparing the findings of multiple experts
over multiple images and modalities with pathologic confirmation.42 Many imagers also value liver-specific MRI contrast agents
like mangafodipir trisodium, ferucarbotran, and superparamagnetic iron oxide (SPIO) as superior tools in lesion delineation, with
some authors asserting the superiority of SPIO in the detection of
small HCC tumors in single-institutional series.43-45 It is generally
accepted that MRI provides the most accurate lesion characterization in the differentiation of benign from malignant lesions.46

MANAGEMENT
3. What are the treatment algorithms for various liver
malignancies?
Over the past decade, many conferences have been convened, and
consensus statements and expert opinions have been offered to
support consistency in treatment protocols and indications for
resection of malignant liver lesions. These statements are based
primarily on compilations of data from several large series combined with expert opinion. One such statement defines colorectal
lesions as absolutely unresectable in patients who have untreatable or extensive extrahepatic disease, those who are unfit for surgery, or those with involvement of >70% of the liver (or more than
six segments).47 Others take a more simple approach, describing
resectable tumors as those that would leave behind an adequate
liver remnant if completely resected.48 For otherwise healthy

PMPH_CH41.indd 335

335

noncirrhotic patients, the minimum published safe volume for a


future liver remnant (FLR) is 20% of preoperative liver volume,
whereas cirrhotics must have at least 40% FLRs.49
Several authors have proposed that liver resection should be
considered the treatment of choice for single lesions arising in
noncirrhotic livers or those with Child Pugh A cirrhosis with wellpreserved liver function, whereas transplantation should be considered the standard for curative therapy in patients with Childs B
and C cirrhosis who meet the Milan criteria.4,50 In terms of surgical technique, small tumors (<5 cm) located at the periphery of the
hepatic parenchyma, or well-away from major vascular structures,
are considered amenable to segmentectomy. Major hepatectomy,
typically defined as resection of three or more hepatic segments based
on Couinaud classifications, is recommended for tumors >5 cm,
or those invading hilar structures.51

HCC
Several major debates have emerged regarding the treatment
schemes for HCC. They primarily concern transplantation versus
resection as curative therapy.9,52 Among the most argued principles are the parameters for resection versus transplantation versus combinations of resection and ablation for curative therapy,
particularly in both early and bilobar HCC.4,5,9,52 The biggest step
in the development of treatment algorithms for HCC came with
the evolution of the Milan criteria. This originated with a study
by Mazzaferro et al.13 published in the New England Journal of
Medicine in 1996, detailing the treatment of 48 cirrhotic patients
with small HCC that was deemed unresectable by virtue of tumor
location, by multifocality, or by advanced hepatic insufficiency
related to cirrhosis.13 The study noted superior disease-free and
overall survival in those patients in patients who had one tumor
<5 cm in diameter or no more than three tumors with none
>3 cm in diameter.13 Although there has yet to be an RCT directly
comparing resection and transplantation, there are many strong
advocates in both camps and many retrospective comparative
reviews, and it is clear that patients with smaller less extensive
tumors fair much better than their counterparts with large multinodular or diff use tumors following transplantation.53 One of the
drawbacks of transplantation is the significant loss of patients
from the transplant waiting list secondary to death or disease
progression, and some argue that this should preclude transplant
as the first consideration of therapy.54 Yet, some centers advocate
expansion of the Milan criteria to include more patients, but again
this is based on single-institutional reviews.55 In addition to the
absence of an RCT, other obstacles in the currently available literature include a dearth of retrospective case control studies which
compare patients based on liver function, as well as a selection
bias in studies comparing transplant and resection wherein transplant patients typically have more severe cirrhosis and smaller
tumors.56 Thus, more robust evidence and uniform treatment criteria are needed to elucidate the optimal surgical management of
this disease.

COLORECTAL HEPATIC METS


Colorectal carcinoma has a poor prognosis when metastatic to the
liver, obviating the need for aggressive surgical therapy for the 20%

5/22/2012 5:29:14 PM

336

Surgery: Evidence-Based Practice

to 30% of patients who are potentially resectable.57,58 Treatment


algorithms for colorectal hepatic metastases fi nd themselves in
more agreed-upon territory, with most surgeons and practitioners
acknowledging that surgical treatment is the keystone of curative
therapy and that resection should be attempted so long as patients
exhibit adequate functional status, are free of extrahepatic disease,
and will maintain an adequate liver remnant.2,59-61 Transplantation for hepatic metastasis was investigated in the 1990s, but it
is generally not offered secondary to poor outcomes seen then.62
While it is clear that small metastases confined to small liver area
should be resected, resection criteria for more extensive disease
are constantly changing.57 In patients with bilobar hepatic metastases, several specialized centers have reported 5-year survival as
high as 60% in properly selected patients undergoing a two-stage
resection strategy combined with chemotherapy and portal vein
embolization, with completeness of resection and response to chemotherapy being essential to favorable outcome.60,63,64 It should be
mentioned that even in the previous study with highly selected
patients at a high-volume center, 28% of patients who entered the
multi-stage treatment were not able to complete the second stage
of complete resection, either secondary to disease progression or
medical decompensation.60 This aggressive treatment style warrants further investigation and optimization.
Management of synchronous colorectal primary and metastatic
hepatic disease is controversial with strategies that vary regarding
timing of chemotherapy, liver, and colonic resections.61 Most of the
data consist of single institution reports or retrospective reviews
which has prompted many senior investigators to demand further
evidence.65 The order and extent of resection can vary widely between
institutions and patients, especially in patients with more extensive
disease. While the more traditional approach of stepwise resection of
the primary lesion followed by a second operation to remove a hepatic
metastasis has yielded relatively favorable outcomes, several centers
have reported relatively favorable outcomes with synchronous resection of synchronous metastases.59,66-68 However, Capussotti et al.67
have reported that patients with more than three metastases exhibit
more favorable overall survival with delayed resection of hepatic
lesions with the appropriate adjuvant therapy. Other investigators
advocate a reverse approach with the first operation being resection of hepatic disease followed by a second colon resection, with
most reporting survival results similar to those of synchronous and
traditional two-stage procedures, but again this data is generated
only by small single-institutional series.40,61,69 Further investigations
are needed to optimize the current treatment strategies for extensive
colorectal hepatic metastases.

NEUROENDOCRINE
The treatment of neuroendocrine liver metastases (NLM) is also
confounded by a wide array of institutional treatment strategies
combined with indolent disease processes and a lack of RCTs
to mandate uniform treatment strategies.70 Most data is generated from single- or multi-institutional reviews, and the lack of
RCTs seems insurmountable in this relatively rare, histologically
inconsistent disease process.70 Moreover, as Gurusamy et al.71,72
point out, there are many obstacles to RCT investigations of
NLM treatment, including that to show a statistically significant
10% difference in survival between surgical debulking and other

PMPH_CH41.indd 336

palliative treatment, 776 patients would be required. Because of


the indolent nature of the disease, even incomplete resection of
NLM can confer survival and symptom benefits.73,74 In patients
with unresectable disease, hormonal suppression, ablative therapies, or loco-regional chemotherapies can be of some benefit, but
in the absence of data to compare the relative efficacy of these various techniques, it seems that choices of nonsurgical intervention
is largely based on institutional preference.70,75 Transplantation
for NLM is almost never indicated secondary to the high risk of
rapid disease recurrence and diminished actuarial survival compared with patients receiving transplants for all indications.75-79
Nonetheless, there is some evidence to suggest that transplantation may be beneficial to patients with hepatic metastases from
gastroenteropancreatic carcinoid disease.80 Thus, it is apparent
that patients with NLM amenable to removal of 80% to 90% of the
tumor volume should undergo surgical resection, whereas algorithms for other treatment strategies are less clear.70

TECHNIQUES OF RESECTION
Surgical techniques as well as amount of tissue manipulation have
been indirectly associated with patient survival.81 Moreover, positive margins of resection have been associated with recurrence,
which can be correlated with surgical technique.82 It is important
that hepatic resections be undertaken by experienced surgeons
at high-volume centers. The same principles apply to minimally
invasive resection in which advanced laparoscopic experience, in
addition to HPB experience, is essential to favorable outcomes.
4. How are ablative therapies used in the treatment of liver
malignancies?
Currently, ablative therapies are performed via laparoscopic, open,
and percutaneous approaches.6 Many different ablative technologies have been suggested and investigated. Many involve some sort
of thermal injury to the diseased tissue. Among the most frequently
employed in clinical practice are radiofrequency ablation (RFA)
and microwave coagulation therapy (MCT), which are discussed in
detail below. Several other ablative technologies are under investigation. For example, laser ablation, which has demonstrated favorable
results in 432 HCC patients with unresectable disease or with medical contraindications to resections.83 Irreversible electroporation
(IRE) increases cell membrane permeability via alteration of transmembrane potential and resultant disruption of lipid bilayer integrity and induce cell death.84 IRE has shown promise in preclinical
studies, but remains unestablished in clinical practice.85 Thus, without the support of RCT evidence and significant clinical experience,
that is afforded RFA and MCT, many new techniques have roles in
the therapy of hepatic malignancy that are still to be decided.
MCT has been studied primarily with hepatic colorectal
cancer metastases. One RCT evaluated 30 patients with multiple
(2-9) hepatic colorectal metastases.86 The patients were randomly
assigned to either MCT (14) or hepatectomy (16), but investigators
found differences in 1-, 2-, and 3-year survival percentages to be
statistically insignificant with 71%, 57%, and 14% in the microwave group versus 69%, 56%, and 23% in the hepatectomy group
(p = .83).86 Although these results suggest that MCT should be
considered in patients with multiple lesions, the power of this study
is very small and survival of 14% at 3 years is low when compared

5/22/2012 5:29:14 PM

Malignant Liver Tumors

with more recent studies.87 The remainder of currently available


literature consists of primarily retrospective series.88-90 One useful study evaluated the use of MCT in combination with resection in patients with bilobar colorectal metastases to the liver.90
Tanaka et al.90 reviewed 53 patients undergoing hepatectomy with
microwave ablation employed only in cases where complete resection was not possible (16), and found similar survival percentages
between patients treated with resection alone and resection with
MCT.90 Although some heterogeneity among adjuvant therapies
between the patient groups may confound the data, the suggestion
remains that with aggressive treatment, ablation of unresectable
lesions in conjunction with partial resection can confer survival
benefits similar to those seen in complete resection.
RFA is perhaps the most established ablative therapy. The
most recent ASCO consensus statement regarding RFA for the
treatment of hepatic metastases from colorectal cancer called
for more evidence and survival data, and presented an evidence
review rather than a practice guideline.6 There are no published
RCTs for the use of RFA for colorectal hepatic metastases. Many
recognize its use as adjunctive to surgical therapy, or as palliative
in cases of patients who are medically unfit for surgery or those
with extrahepatic disease for whom the risks and morbidity of
surgery outweigh any potential survival benefit.6,59
However, much more debate is held in the arena of HCC
treatment. Many investigators disagree with the pursuit of studies
paralleling RFA and surgery, contending that resection should be
employed over RFA whenever possible to provide definitive treatment rather than risk local recurrence with RFA as indicated by
multiple retrospective reviews.91-93 For instance, one prospective
database analysis out of MD Anderson revealed that local recurrence rates can increase as much as 32% when RFA is employed
when compared with resection, even in lesions that are <3 cm in
diameter.94 One RCT that was performed in China studied 230
patients with HCC meeting the Milan criteria and receiving RFA
or lesion resection, and confirmed the suspicions of many retrospective reviews that overall and recurrence-free survival are significantly better when resection is employed.95 In this particular
trial, 5-year recurrence rates were 63% in the RFA group and 42%
in the resection group (p = .017), and overall survival was 76% in
the resection group versus 55% in the RFA group. Unfortunately,
although RFA is attractive in unresectable patients, some of the
same features that make lesions unresectable, like proximity to
large vessels, as well as large lesion size, are the same features that
are thought to affect higher recurrence rates.91 Mazzaferro et al.
do advocate the use of RFA in cirrhotics with small HCC lesions
who are awaiting transplant; however, they caution that this strategy is not as useful when lesions are >3 cm or when >1 year elapses
from ablation to transplant secondary to the risk of recurrence.96
Thus, while ablative approaches are helpful in cases when resection is not possible, or is medically contraindicated, resection is
widely accepted as the gold standard of treatment for resection of
colorectal liver metastases.91,96
5. What are the palliative procedures performed by surgeons
in relation to liver malignancies?
Unfortunately, despite increased clinical awareness and surveillance programs in the United States, the majority of HCC patients
present with late stage disease that is beyond the abilities of
currently available curative therapies.97 Moreover, even patients
who present in early stages can recur following radical therapies

PMPH_CH41.indd 337

337

at rates as high as 50%.98,99 The case is similar for patients with colorectal carcinoma, with 15% of patients presenting with stage IV
disease, and up to 50% developing metastases to the liver at some
point during their course of treatment.100,101 Finally, some patients
with neuroendocrine hepatic metastases require mere debulking
measures to achieve a long-term sustainable quality of life.70 Thus,
a surgeons armamentarium must contain knowledge of techniques
for palliative management of unresectable hepatic metastases.
Putting aside the afore-discussed debate regarding the efficacy of RFA versus resection for resectable disease, RFA alone
or in combination with hepatic resection has shown benefit in
patients with unresectable hepatic tumors.87,102 One large study
examined 234 patients with colorectal hepatic metastases which
had progressed despite chemotherapy or were associated with
extrahepatic disease.103 Laparoscopic RFA was employed and
showed a median survival of 24 months with increased efficacy
in patients with tumors <3 cm in diameter or with <3 hepatic
lesions.103 Although the available literature on MCT is less extensive, it has also shown benefit in patients with unresectable hepatic
metastases of various histologies.104 However, direct comparison of
percutaneous RFA and MCT in a retrospective setting has shown
that RFA yields decreased local recurrence with increased ablative
efficacy after fewer treatments.105
Other interventions that were considered in past literature include percutaneous ethanol injection for local ablation of
HCC. However, five RCTs have demonstrated the superiority of
local control conferred by RFA versus ethanol injection for the
treatment of early stage HCC.106-108 Newer techniques like laser
ablation, and use of ablative technologies in combination with
loco-regional chemotherapeutic and gene therapy interventions
warrant further investigation.109
Cytoreductive surgery also has a place in the palliation of
various liver tumors. Perhaps the most effective arena for cytoreduction of hepatic metastases is that of neuroendocrine tumors.
Several authors have shown improved symptom control and
survival even with incomplete resections of hepatic neuroendocrine metastases.70,73,74 Until recently, cytoreductive intervention
in peritoneal carcinomatosis of colorectal origin was thought
to be contraindicated in the presence of lymph node or hepatic
metastases. However, some investigators are now showing favorable results with a combination of liver resection, peritoneal debulking, and intraperitoneal chemotherapy.110-112 Furthermore,
while it has been studied in small numbers, several centers report
favorable results following cytoreductive surgery for advanced or
unresectable HCC, with one study of 72 patients demonstrating a
5-year survival of 62% with the use of cytoreduction and sequential resection for initially unresectable HCC.113-115
6. How are minimally invasive therapies used in the treatment
of liver malignancies?
Proponents of laparoscopic resection point to benefits shown in
several series like shorter hospital lengths of stay, lower rates of
blood transfusion, lower overall operative complications, cost
effectiveness, and equivocal efficacy.91,116-119 Since the first studies in the 1990s demonstrating feasibility of laparoscopic resections were reported, more than 2800 laparoscopic liver resections
have been reported in 127 articles.120 Of those, at least half
were performed to remove malignant lesions and 35% of those
malignancies were colorectal hepatic metastases. Of the 2800
cases, 75% were performed with pure laparoscopy, 17% were

5/22/2012 5:29:14 PM

338

Surgery: Evidence-Based Practice

hand-assisted, and 2% were laparoscopic-assisted or hybrid.120


Procedures described included wedge resection or segmentectomy
(45%), anatomic left lateral sectionectomy (20%), right hepatectomy (9%), and left hepatectomy (7%).120 In malignant resections,
negative margins were achieved in 82% to 100% of cases.120
Given the novelty of many currently employed techniques of
laparoscopic resections as well as the inevitable absence of longterm follow-up data, perhaps the best data available are consensus opinion from the International Position on Laparoscopic
Liver Surgery, which reflects the recommendations of 45 invited
experts in both laparoscopic and open liver surgery.121 According
to the consensus conference, laparoscopic liver resections can be
divided into three subtypes: (1) biopsies and small wedge resections, (2) resections of the left lateral or anterior hepatic segments
(4b, 5, and 6), and (3) hemihepatectomies, trisectionectomies, and
resections of the difficult posterior segments (4a, 7, 8). 121
Major centers are experimenting with many novel surgical
approaches like single-incision hepatic resections.122 Techniques and
devices utilized for parenchymal transection and ligation of major
vascular structures, and hemostasis are also rapidly evolving, and
there does not seem to be a uniform opinion as to which is best; how-

ever, a recent meta-analysis of RCTs indicates that all new methods


should be compared against the clamp-crush technique as a gold
standard.123 The data for these techniques is still in early phases of
feasibility and efficacy reports.121 Indications for laparoscopic resection established by consensus were lesions 5 cm, located in peripheral segments 2 through 6.121 The coming years will likely bring with
them longer-term data on larger scales which are likely to establish
laparoscopic resections as the standard of care.121

DISCUSSION
Data for the treatment of malignant liver tumors are based on
a culmination of the experiences and opinions of many great
surgeons at excellent medical centers. However, treatment algorithms and guidelines remain driven largely by Level 2 and 3
evidence, and are unconfirmed by RCTs. Many new technologies
are emerging that will likely provide excellent adjuncts or alternatives to surgical resection. Minimally invasive liver surgery
represents the future of liver resection and warrants vigorous
investigative focus.

Clinical Question Summary


Question

Answer

Levels of
Evidence

1 What is the standard


staging of patients
prior to hepatic
resection?

Several systems available

Consensus
Conference
based on
1-3

18-24

2 What imaging
techniques are best
used for delineating
malignant from
benign liver lesions?

MRI best overall at analyzing liver and


lesion characteristics
PET most sensitive (90%) for
metastases pre-chemo
Clinically, MRI or CT used in
conjunction with PET

1-3

27-30, 46

1-2

3 What are the


treatment algorithms
for various liver
malignancies?

HCC Milan criteria for transplantation


though some propose changes.
Otherwise, resection where possible
Colorectal completely resect
synchronous and metachronous
lesions whenever possible
Neuroendocrine cytoreduction
can be of benefit

2-3

HCC: 4, 5, 9, 13,
14, 52

Colorectal: 2, 57,
59-61

NLM: 70, 72-74

4 How are ablative


therapies used in the
treatment of liver
malignancies?

RFA variable strategies:


I. As combo with resection for
extensive bilateral disease
II. Percutaneously or laparoscopically
in cirrhotics with poor resection
tolerance

RFA: 6, 91-93

5 What are the


palliative procedures
performed by
surgeons in relation
to liver malignancies?

RFA

1c

RFA: 87, 102, 104

Cytoreduction

1c

Cytoreduction:
110, 111, 113,
114

6 How are minimally


invasive therapies
used in treatment of
liver malignancies?

Laparoscopic liver resection should be


considered whenever possible

1c

Laparoscopy:
91, 116-120

PMPH_CH41.indd 338

AJCC is US standard

Grade of
Recommendations

References

5/22/2012 5:29:14 PM

Malignant Liver Tumors

REFERENCES
1. Nordlinger B, Quilichini MA, Parc R, Hannoun L, Delva E,
Huguet C. Hepatic resection for colorectal liver metastases.
Influence on survival of preoperative factors and surgery for
recurrences in 80 patients. Ann Surg. 1987;205:256-263.
2. Morris KT, Song TJ, Fong Y. Recent advancements in diagnosis
and treatment of metastatic colorectal cancer to the liver. Surg
Oncol. 2006;15:129-134.
3. Adam R, Avisar E, Ariche A, et al. Five-year survival following
hepatic resection after neoadjuvant therapy for nonresectable
colorectal. Ann Surg Oncol. 2001;8:347-353.
4. Morris-Stiff G, Gomez D, de Liguori Carino N, Prasad KR. Surgical management of hepatocellular carcinoma: Is the jury still out?
Surg Oncol. 2009;18:298-321.
5. Ye SL, Takayama T, Geschwind J, Marrero JA, Bronowicki JP.
Current approaches to the treatment of early hepatocellular carcinoma. Oncologist. 2010;15 Suppl 4:34-41.
6. Wong SL, Mangu PB, Choti MA, et al. American Society of Clinical Oncology 2009 clinical evidence review on radiofrequency
ablation of hepatic metastases from colorectal cancer. J Clin
Oncol. 2010;28:493-508.
7. Foster JH, Berman MM. Solid liver tumors. Major Probl Clin
Surg. 1977;22:1-342.
8. Jarnagin WR, Gonen M, Fong Y, et al. Improvement in perioperative outcome after hepatic resection: Analysis of 1,803 consecutive
cases over the past decade. Ann Surg. 2002;236:397-406; discussion 406-407.
9. Liu CL, Fan ST, Lo CM, et al. Hepatic resection for combined hepatocellular and cholangiocarcinoma. Arch Surg. 2003;138:86-90.
10. Wakabayashi H, Ishimura K, Okano K, et al. Is preoperative portal vein embolization effective in improving prognosis after major
hepatic resection in patients with advanced-stage hepatocellular
carcinoma? Cancer. 2001;92:2384-2390.
11. Braillon A. Hepatocellular carcinoma and evidence-based surgery. World J Gastroenterol. 2009;15:5371.
12. Tsim NC, Frampton AE, Habib NA, Jiao LR. Surgical treatment
for liver cancer. World J Gastroenterol. 2010;16:927-933.
13. Mazzaferro V, Regalia E, Doci R, et al. Liver transplantation for
the treatment of small hepatocellular carcinomas in patients
with cirrhosis. New Engl J Med. 1996;334:693-699.
14. Yao FY, Ferrell L, Bass NM, et al. Liver transplantation for hepatocellular carcinoma: Expansion of the tumor size limits does
not adversely impact survival. Hepatology. 2001;33:1394-1403.
15. Compton C, Fenoglio-Preiser CM, Pettigrew N, Fielding LP. American Joint Committee on Cancer Prognostic Factors Consensus Conference: Colorectal Working Group. Cancer. 2000;88:1739-1757.
16. Helton WS, Strasberg SM. AHPBA/AJCC consensus conference
on staging of hepatocellular carcinoma: Rationale and overview
of the conference. HPB (Oxford). 2003;5:238-242.
17. Henderson JM, Sherman M, Tavill A, Abecassis M, Chejfec G,
Gramlich T. AHPBA/AJCC consensus conference on staging of
hepatocellular carcinoma: Consensus statement. HPB (Oxford).
2003;5:243-250.
18. Chevret S, Trinchet JC, Mathieu D, Rached AA, Beaugrand M,
Chastang C. A new prognostic classification for predicting survival in patients with hepatocellular carcinoma. Groupe dEtude
et de Traitement du Carcinome Hepatocellulaire. J Hepatol. 1999;
31:133-141.
19. Llovet JM, Bruix J. Prospective validation of the Cancer of the
Liver Italian Program (CLIP) score: A new prognostic system for
patients with cirrhosis and hepatocellular carcinoma. Hepatology. 2000;32:679-680.

PMPH_CH41.indd 339

339

20. Leung TW, Tang AM, Zee B, et al. Construction of the Chinese
University Prognostic Index for hepatocellular carcinoma and
comparison with the TNM staging system, the Okuda staging
system, and the Cancer of the Liver Italian Program staging system: A study based on 926 patients. Cancer. 2002;94:1760-1769.
21. Llovet JM, Fuster J, Bruix J. The Barcelona approach: Diagnosis,
staging, and treatment of hepatocellular carcinoma. Liver Transpl.
2004;10:S115-S120.
22. Kee KM, Wang JH, Lee CM, et al. Validation of clinical AJCC/
UICC TNM staging system for hepatocellular carcinoma: Analysis of 5,613 cases from a medical center in southern Taiwan.
Int J Cancer. 2007;120:2650-2655.
23. Nathan H, Mentha G, Marques HP, et al. Comparative performances of staging systems for early hepatocellular carcinoma.
HPB (Oxford). 2009;11:382-390.
24. Vauthey JN, Dixon E, Abdalla EK, et al. Pretreatment assessment
of hepatocellular carcinoma: Expert consensus statement. HPB
(Oxford). 2010;12:289-299.
25. Carpenter S, Fong Y. Management of disappearing colorectal
hepatic metastases. Adv Surg. 2010;44:269-279.
26. Ruers TJ, Wiering B, van der Sijp JR, et al. Improved selection of
patients for hepatic surgery of colorectal liver metastases with (18)
F-FDG PET: A randomized study. J Nucl Med. 2009;50:1036-1041.
27. Bipat S, van Leeuwen MS, Comans EF, et al. Colorectal liver metastases: CT, MR imaging, and PET for diagnosismeta-analysis.
Radiology. 2005;237:123-131.
28. Kinkel K, Lu Y, Both M, Warren RS, Thoeni RF. Detection of
hepatic metastases from cancers of the gastrointestinal tract by
using noninvasive imaging methods (US, CT, MR imaging, PET):
A meta-analysis. Radiology. 2002;224:748-756.
29. Tan MC, Linehan DC, Hawkins WG, Siegel BA, Strasberg SM.
Chemotherapy-induced normalization of FDG uptake by colorectal liver metastases does not usually indicate complete pathologic response. J Gastrointest Surg. 2007;11:1112-1119.
30. Ward J, Robinson PJ, Guthrie JA, et al. Liver metastases in candidates
for hepatic resection: Comparison of helical CT and gadoliniumand SPIO-enhanced MR imaging. Radiology. 2005;237:170-180.
31. Fong Y, Saldinger PF, Akhurst T, et al. Utility of 18F-FDG positron
emission tomography scanning on selection of patients for resection of hepatic colorectal metastases. Am J Surg. 1999;178:282-287.
32. Kelloff GJ, Hoff man JM, Johnson B, et al. Progress and promise
of FDG-PET imaging for cancer patient management and oncologic drug development. Clin Cancer Res. 2005;11:2785-2808.
33. Cho CS, Curran S, Schwartz LH, et al. Preoperative radiographic
assessment of hepatic steatosis with histologic correlation. J Am
Coll Surg. 2008;206:480-488.
34. Fong Y, Bentrem DJ. CASH (Chemotherapy-Associated Steatohepatitis) costs. Ann Surg. 2006;243:8-9.
35. Peppercorn PD, Reznek RH, Wilson P, Slevin ML, Gupta RK.
Demonstration of hepatic steatosis by computerized tomography
in patients receiving 5-fluorouracil-based therapy for advanced
colorectal cancer. Br J Cancer. 1998;77:2008-2011.
36. Vauthey JN, Pawlik TM, Ribero D, et al. Chemotherapy regimen
predicts steatohepatitis and an increase in 90-day mortality after
surgery for hepatic colorectal metastases. J Clin Oncol. 2006;
24:2065-2072.
37. Zorzi D, Laurent A, Pawlik TM, Lauwers GY, Vauthey JN,
Abdalla EK. Chemotherapy-associated hepatotoxicity and surgery for colorectal liver metastases. Br J Surg. 2007;94:274-286.
38. Akhurst T, Kates TJ, Mazumdar M, et al. Recent chemotherapy
reduces the sensitivity of [18F]fluorodeoxyglucose positron emission tomography in the detection of colorectal metastases. J Clin
Oncol. 2005;23:8713-8716.

5/22/2012 5:29:14 PM

340

Surgery: Evidence-Based Practice

39. Valls C, Iannaccone R, Alba E, et al. Fat in the liver: Diagnosis


and characterization. Eur Radiol. 2006;16:2292-2308.
40. Verhoef C, van der Pool AE, Nuyttens JJ, Planting AS, Eggermont AM, de Wilt JH. The liver-first approach for patients with
locally advanced rectal cancer and synchronous liver metastases.
Dis Colon Rectum. 2009;52:23-30.
41. Teefey SA, Hildeboldt CC, Dehdashti F, et al. Detection of primary hepatic malignancy in liver transplant candidates: Prospective comparison of CT, MR imaging, US, and PET. Radiology.
2003;226:533-542.
42. Seltzer SE, Getty DJ, Pickett RM, et al. Multimodality diagnosis of liver tumors: Feature analysis with CT, liver-specific and
contrast-enhanced MR, and a computer model. Acad Radiol.
2002;9:256-269.
43. Choi JY, Kim MJ, Kim JH, et al. Detection of hepatic metastasis:
Manganese- and ferucarbotran-enhanced MR imaging. Eur J
Radiol. 2006;60:84-90.
44. Kim MJ, Kim JH, Lim JS, et al. Detection and characterization of
focal hepatic lesions: Mangafodipir vs. superparamagnetic iron
oxide-enhanced magnetic resonance imaging. J Magn Reson
Imag. 2004;20:612-621.
45. Oudkerk M, Torres CG, Song B, et al. Characterization of liver
lesions with mangafodipir trisodium-enhanced MR imaging:
Multicenter study comparing MR and dual-phase spiral CT.
Radiology. 2002;223:517-524.
46. Hann LE, Winston CB, Brown KT, Akhurst T. Diagnostic imaging approaches and relationship to hepatobiliary cancer staging
and therapy. Semin Surg Oncol. 2000;19:94-115.
47. Poston GJ, Adam R, Alberts S, et al. OncoSurge: A strategy for
improving resectability with curative intent in metastatic colorectal cancer. J Clin Oncol. 2005;23:7125-7134.
48. Berri RN, Abdalla EK. Curable metastatic colorectal cancer:
Recommended paradigms. Curr Oncol Rep. 2009;11:200-208.
49. Shirabe K, Shimada M, Gion T, et al. Postoperative liver failure
after major hepatic resection for hepatocellular carcinoma in the
modern era with special reference to remnant liver volume. J Am
Coll Surg. 1999;188:304-309.
50. Bruix J, Sherman M. Management of hepatocellular carcinoma.
Hepatology. 2005;42:1208-1236.
51. Fan ST, Lo CM, Liu CL, et al. Hepatectomy for hepatocellular
carcinoma: Toward zero hospital deaths. Ann Surg. 1999;229:
322-330.
52. Facciuto ME, Koneru B, Rocca JP, et al. Surgical treatment of
hepatocellular carcinoma beyond Milan criteria. Results of liver
resection, salvage transplantation, and primary liver transplantation. Ann Surg Oncol. 2008;15:1383-1391.
53. Bismuth H, Chiche L, Adam R, Castaing D, Diamond T, Dennison A. Liver resection versus transplantation for hepatocellular
carcinoma in cirrhotic patients. Ann Surg. 1993;218:145-151.
54. Llovet JM, Fuster J, Bruix J. Intention-to-treat analysis of surgical treatment for early hepatocellular carcinoma: Resection versus transplantation. Hepatology. 1999;30:1434-1440.
55. Toso C, Kneteman NM, James Shapiro AM, Bigam DL. The estimated number of patients with hepatocellular carcinoma selected
for liver transplantation using expanded selection criteria.
Transpl Int. 2009;22:869-875.
56. Kim RD, Hemming AW. Hepatocellular carcinoma: Resection
or transplantation. J Gastrointest Surg. 2009;13:1023-1025.
57. Abdalla EK, Adam R, Bilchik AJ, Jaeck D, Vauthey JN, Mahvi D.
Improving resectability of hepatic colorectal metastases: Expert
consensus statement. Ann Surg Oncol. 2006;13:1271-1280.
58. Pawlik TM, Choti MA. Surgical therapy for colorectal metastases to the liver. J Gastrointest Surg. 2007;11:1057-1077.

PMPH_CH41.indd 340

59. Aloia TA, Vauthey JN. Management of colorectal liver metastases: Past, present, and future. Updates Surg. 2011;63(1):1-3.
60. Brouquet A, Abdalla EK, Kopetz S, et al. High survival rate after
two-stage resection of advanced colorectal liver metastases:
Response-based selection and complete resection defi ne outcome. J Clin Oncol. 2011;29(8):1083-1090.
61. Brouquet A, Mortenson MM, Vauthey JN, et al. Surgical strategies for synchronous colorectal liver metastases in 156 consecutive patients: Classic, combined or reverse strategy? J Am Coll
Surg. 2010;210:934-941.
62. Foss A, Adam R, Dueland S. Liver transplantation for colorectal liver metastases: Revisiting the concept. Transpl Int. 2010;23:
679-685.
63. Chun YS, Vauthey JN, Ribero D, et al. Systemic chemotherapy
and two-stage hepatectomy for extensive bilateral colorectal liver
metastases: Perioperative safety and survival. J Gastrointest Surg.
2007;11:1498-1504; discussion 1504-1505.
64. Wicherts DA, Miller R, de Haas RJ, et al. Long-term results of
two-stage hepatectomy for irresectable colorectal cancer liver
metastases. Ann Surg. 2008;248:994-1005.
65. Khatri VP. Synchronous colorectal liver metastases: Triumph of
prospective randomized trials over observational bias leads to
paradigm shift. Ann Surg Oncol. 2009;16:1762-1764.
66. Capussotti L, Ferrero A, Vigano L, Ribero D, Lo Tesoriere R,
Polastri R. Major liver resections synchronous with colorectal
surgery. Ann Surg Oncol. 2007;14:195-201.
67. Capussotti L, Vigano L, Ferrero A, Lo Tesoriere R, Ribero D,
Polastri R. Timing of resection of liver metastases synchronous
to colorectal tumor: Proposal of prognosis-based decisional
model. Ann Surg Oncol. 2007;14:1143-1150.
68. de Santibanes E, Lassalle FB, McCormack L, et al. Simultaneous
colorectal and hepatic resections for colorectal cancer: Postoperative and longterm outcomes. J Am Coll Surg. 2002;195:196-202.
69. Mentha G, Roth AD, Terraz S, et al. Liver first approach in the
treatment of colorectal cancer with synchronous liver metastases.
Dig Surg. 2008;25:430-435.
70. Reddy SK, Clary BM. Neuroendocrine liver metastases. Surg Clin
North Am. 2010;90:853-861.
71. Gurusamy KS, Pamecha V, Sharma D, Davidson BR. Palliative cytoreductive surgery versus other palliative treatments
in patients with unresectable liver metastases from gastroentero-pancreatic neuroendocrine tumours. Cochrane Database
Syst Rev. 2009;(1):CD007118.
72. Gurusamy KS, Ramamoorthy R, Sharma D, Davidson BR. Liver
resection versus other treatments for neuroendocrine tumours
in patients with resectable liver metastases. Cochrane Database
Syst Rev. 2009;(2):CD007060.
73. Osborne DA, Zervos EE, Strosberg J, et al. Improved outcome
with cytoreduction versus embolization for symptomatic hepatic
metastases of carcinoid and neuroendocrine tumors. Ann Surg
Oncol. 2006;13:572-581.
74. Sarmiento JM, Heywood G, Rubin J, Ilstrup DM, Nagorney DM,
Que FG. Surgical treatment of neuroendocrine metastases to the
liver: A plea for resection to increase survival. J Am Coll Surg.
2003;197:29-37.
75. Sutcliffe R, Maguire D, Ramage J, Rela M, Heaton N. Management
of neuroendocrine liver metastases. Am J Surg. 2004;187:39-46.
76. Florman S, Toure B, Kim L, et al. Liver transplantation for neuroendocrine tumors. J Gastrointest Surg. 2004;8:208-212.
77. Lang H, Schlitt HJ, Schmidt H, et al. Total hepatectomy and liver
transplantation for metastatic neuroendocrine tumors of the
pancreas - a single center experience with ten patients. Langenbecks Arch Surg. 1999;384:370-377.

5/22/2012 5:29:14 PM

Malignant Liver Tumors

78. Le Treut YP, Delpero JR, Dousset B, et al. Results of liver transplantation in the treatment of metastatic neuroendocrine tumors.
A 31-case French multicentric report. Ann Surg. 1997;225:355-364.
79. Lehnert T. Liver transplantation for metastatic neuroendocrine
carcinoma: An analysis of 103 patients. Transplantation. 1998;
66:1307-1312.
80. van Vilsteren FG, Baskin-Bey ES, Nagorney DM, et al. Liver
transplantation for gastroenteropancreatic neuroendocrine cancers: Defining selection criteria to improve survival. Liver Transpl.
2006;12:448-456.
81. Zhang XF, Meng B, Qi X, et al. Prognostic factors after liver resection
for hepatocellular carcinoma with hepatitis B virus-related cirrhosis:
Surgeons role in survival. Eur J Surg Oncol. 2009;35:622-628.
82. Poon RT, Fan ST, Ng IO, Wong J. Significance of resection margin in hepatectomy for hepatocellular carcinoma: A critical reappraisal. Ann Surg. 2000;231:544-551.
83. Pacella CM, Francica G, Di Lascio FM, et al. Long-term outcome
of cirrhotic patients with early hepatocellular carcinoma treated
with ultrasound-guided percutaneous laser ablation: A retrospective analysis. J Clin Oncol. 2009;27:2615-2621.
84. Lencioni R, Cioni D, Della Pina C, Crocetti L. Hepatocellular carcinoma: New options for image-guided ablation. J Hepatobiliary
Pancreat Sci. 2010;17:399-403.
85. Guo Y, Zhang Y, Klein R, et al. Irreversible electroporation therapy in the liver: Longitudinal efficacy studies in a rat model of
hepatocellular carcinoma. Cancer Res. 2010;70:1555-1563.
86. Shibata T, Niinobu T, Ogata N, Takami M. Microwave coagulation therapy for multiple hepatic metastases from colorectal carcinoma. Cancer. 2000;89:276-284.
87. Rocha FG, DAngelica M. Treatment of liver colorectal metastases: Role of laparoscopy, radiofrequency ablation, and microwave
coagulation. J Surg Oncol. 2010;102:968-974.
88. Jagad RB, Koshariya M, Kawamoto J, et al. Laparoscopic microwave ablation of liver tumors: Our experience. Hepatogastroenterology. 2008;55:27-32.
89. Ogata Y, Uchida S, Hisaka T, et al. Intraoperative thermal
ablation therapy for small colorectal metastases to the liver.
Hepatogastroenterology. 2008;55:550-556.
90. Tanaka K, Shimada H, Nagano Y, Endo I, Sekido H, Togo S.
Outcome after hepatic resection versus combined resection and
microwave ablation for multiple bilobar colorectal metastases to
the liver. Surgery. 2006;139:263-273.
91. Buell JF, Thomas MT, Rudich S, et al. Experience with more
than 500 minimally invasive hepatic procedures. Ann Surg.
2008;248:475-486.
92. Guglielmi A, Ruzzenente A, Valdegamberi A, et al. Radiofrequency
ablation versus surgical resection for the treatment of hepatocellular carcinoma in cirrhosis. J Gastrointest Surg. 2008;12:192-198.
93. White RR, Avital I, Sofocleous CT, et al. Rates and patterns of
recurrence for percutaneous radiofrequency ablation and open
wedge resection for solitary colorectal liver metastasis. J Gastrointest Surg. 2007;11:256-263.
94. Aloia TA, Vauthey JN, Loyer EM, et al. Solitary colorectal
liver metastasis: Resection determines outcome. Arch Surg.
2006;141:460-466; discussion 466-467.
95. Huang J, Yan L, Cheng Z, et al. A randomized trial comparing
radiofrequency ablation and surgical resection for HCC conforming to the Milan criteria. Ann Surg. 2010;252:903-912.
96. Mazzaferro V, Battiston C, Perrone S, et al. Radiofrequency
ablation of small hepatocellular carcinoma in cirrhotic patients
awaiting liver transplantation: A prospective study. Ann Surg.
2004;240:900-909.

PMPH_CH41.indd 341

341

97. Kim WR, Gores GJ, Benson JT, Therneau TM, Melton LJ, 3rd.
Mortality and hospital utilization for hepatocellular carcinoma
in the United States. Gastroenterology. 2005;129:486-493.
98. Poon RT, Fan ST, Ng IO, Lo CM, Liu CL, Wong J. Different risk
factors and prognosis for early and late intrahepatic recurrence
after resection of hepatocellular carcinoma. Cancer. 2000;89:
500-507.
99. Cucchetti A, Piscaglia F, Caturelli E, et al. Comparison of recurrence of hepatocellular carcinoma after resection in patients
with cirrhosis to its occurrence in a surveilled cirrhotic
population. Ann Surg Oncol. 2009;16:413-422.
100. Leonard GD, Brenner B, Kemeny NE. Neoadjuvant chemotherapy before liver resection for patients with unresectable
liver metastases from colorectal carcinoma. J Clin Oncol.
2005;23:2038-2048.
101. Stangl R, Altendorf-Hofmann A, Charnley RM, Scheele J. Factors
influencing the natural history of colorectal liver metastases.
Lancet. 1994;343:1405-1410.
102. Choi D, Lim HK, Rhim H. Concurrent and subsequent radiofrequency ablation combined with hepatectomy for hepatocellular carcinomas. World J Gastrointest Surg. 2010;2:
137-142.
103. Siperstein AE, Berber E, Ballem N, Parikh RT. Survival after
radiofrequency ablation of colorectal liver metastases: 10-year
experience. Ann Surg. 2007;246:559-565; discussion 565-567.
104. Liang P, Dong B, Yu X, et al. Prognostic factors for percutaneous
microwave coagulation therapy of hepatic metastases. AJR Am J
Roentgenol. 2003;181:1319-1325.
105. Ohmoto K, Yoshioka N, Tomiyama Y, et al. Comparison of
therapeutic effects between radiofrequency ablation and
percutaneous microwave coagulation therapy for small hepatocellular carcinomas. J Gastroenterol Hepatol. 2009;24:
223-227.
106. Brunello F, Veltri A, Carucci P, et al. Radiofrequency ablation
versus ethanol injection for early hepatocellular carcinoma: A
randomized controlled trial. Scand J Gastroenterol. 2008;43:
727-735.
107. Lencioni RA, Allgaier HP, Cioni D, et al. Small hepatocellular
carcinoma in cirrhosis: Randomized comparison of radiofrequency thermal ablation versus percutaneous ethanol
injection. Radiology. 2003;228:235-240.
108. Shiina S, Teratani T, Obi S, et al. A randomized controlled trial
of radiofrequency ablation with ethanol injection for small
hepatocellular carcinoma. Gastroenterology. 2005;129:122-130.
109. Lencioni R. Loco-regional treatment of hepatocellular carcinoma
in the era of molecular targeted therapies. Oncology. 2010;78
Suppl 1:107-112.
110. Carmignani CP, Ortega-Perez G, Sugarbaker PH. The management of synchronous peritoneal carcinomatosis and hematogenous metastasis from colorectal cancer. Eur J Surg Oncol.
2004;30:391-398.
111. Franko J, Gusani NJ, Holtzman MP, et al. Multivisceral resection
does not affect morbidity and survival after cytoreductive
surgery and chemoperfusion for carcinomatosis from colorectal
cancer. Ann Surg Oncol. 2008;15:3065-3072.
112. Cao C, Yan TD, Black D, Morris DL. A systematic review and
meta-analysis of cytoreductive surgery with perioperative
intraperitoneal chemotherapy for peritoneal carcinomatosis of
colorectal origin. Ann Surg Oncol. 2009;16:2152-2165.
113. Shimada M, Takenaka K, Kawahara N, et al. Surgical treatment
strategy for patients with stage IV hepatocellular carcinoma.
Surgery. 1996;119:517-522.

5/22/2012 5:29:14 PM

342

Surgery: Evidence-Based Practice

114. Tang ZY, Yu YQ, Zhou XD, et al. Three decades experience in
surgery of hepatocellular carcinoma. Gan To Kagaku Ryoho.
1997;24 Suppl 1:126-133.
115. Usatoff V, Isla AM, Habib NA. Liver resection in advanced hepatocellular carcinoma. Hepatogastroenterology. 2001;48:46-50.
116. Belli G, Fantini C, DAgostino A, et al. Laparoscopic versus
open liver resection for hepatocellular carcinoma in patients
with histologically proven cirrhosis: Short- and middle-term
results. Surg Endosc. 2007;21:2004-2011.
117. Koffron A, Geller D, Gamblin TC, Abecassis M. Laparoscopic
liver surgery: Shift ing the management of liver tumors.
Hepatology. 2006;44:1694-1700.
118. Koff ron AJ, Auffenberg G, Kung R, Abecassis M. Evaluation of
300 minimally invasive liver resections at a single institution:
Less is more. Ann Surg. 2007;246:385-392; discussion 392-394.

PMPH_CH41.indd 342

119. Lesurtel M, Cherqui D, Laurent A, Tayar C, Fagniez PL. Laparoscopic versus open left lateral hepatic lobectomy: A case-control
study. J Am Coll Surg. 2003;196:236-242.
120. Nguyen KT, Gamblin TC, Geller DA. World review of laparoscopic liver resection-2,804 patients. Ann Surg. 2009;250:
831-841.
121. Buell JF, Cherqui D, Geller DA, et al. The international position
on laparoscopic liver surgery: The Louisville Statement, 2008.
Ann Surg. 2009;250:825-830.
122. Gaujoux S, Kingham TP, Jarnagin WR, DAngelica MI, Allen
PJ, Fong Y. Single-incision laparoscopic liver resection. Surg
Endosc. 2010;25(5):1489-1494.
123. Pamecha V, Gurusamy KS, Sharma D, Davidson BR. Techniques
for liver parenchymal transection: A meta-analysis of randomized
controlled trials. HPB (Oxford). 2009;11:275-281.

5/22/2012 5:29:14 PM

PART VI

PORTAL HYPERTENSION

PMPH_CH42.indd 343

5/22/2012 5:29:47 PM

PMPH_CH42.indd 344

5/22/2012 5:29:47 PM

CHAPTER 42

Management of Portal
Hypertension: A Surgical
Perspective
Sukru Emre and Manuel I. Rodriguez-Davalos

The PV is formed behind the upper border of the head of the


pancreas by the junction of the SV and the SMV, in front of the
vena cava. The IMV drains into the SV in most of the population
but may also join the SV and the SMV to form trifurcation, or drain
into the SMV.8,9 The PV enters into the liver, divides into branches
through its substance, and ends in capillaries. The PV branches are
single and destitute of valves. In most cases, the PV divides just
prior to entering the liver into left (LPV) and right PV branches
and then follows Couinauds segmental anatomy.
The anatomy of the LPV is the particular interest in this chapter because the meso-Rex bypass (previously called REX shunt)
became possible due to the anatomy at the recessus of Rex, where
the umbilical vein (UV) enters into the LPV. During fetal development, the UV carries oxygenated blood form mothers placenta
to the LPV. Recessus of Rex is located between the lateral and the
medial segments of the left lobe.9,10

1. What is the incidence and risk of mortality in patients with


portal hypertension (PHTN) and bleeding gastroesophageal
varices (GEV)?
PHTN can be one of the most severe complications of cirrhosis.
This hemodynamic abnormality is related to bleeding from GEV,
ascites, and hepatic encephalopathy (HE). Despite advances in its
treatment and control, the 6-week mortality of variceal hemorrhage is still 10% to 20%.1,2
GEV are present in 30% of well-compensated cirrhotics, and
60% of patients with decompensated cirrhosis, the increase in varix
size is 10% to 20% from small to large in 1 year and the index of
variceal bleeding is >20% in patients with large varices.3,4 PHTN can
also occur as a result of occlusion of the extrahepatic portion of the
portal vein (EHPVO). This entity is also called noncirrhotic PHTN.
This entity has particular importance in this chapter because this
vascular disorder of the liver is the second leading cause of PHTN
in developing countries following cirrhosis in developed countries,
and the most common etiology in children. In the case of EHPVO,
surgical intervention may be the first-line treatment.5-7
Despite advances in medical, endoscopic, and interventional
techniques as well as the improvements made in the field of liver
and multivisceral transplantation, surgical shunts are still important in the armamentarium of transplant and hepatobiliary surgeons who are often confronted with a select group of patients that
are not candidates for other modalities.

3. What is the definition and pathophysiology of PHTN?


PHTN is defined as a pathological increase of PV pressure above
the normal range of 5 to 8 mmHg. Hepatic vein pressure gradient
(HVPG) is one of the standard measurements utilized in assessing
the degree of PHTN. HVPG is the difference between the wedged
and the free hepatic venous pressures, normally between 3 and
6 mmHg. HVPG > 10 mmHg is considered as PHTN. Although
it is a general belief that esophageal variceal bleeding occurs on
patients with gradients >12 mmHg,4,11,12 there are reports indicating that the degree of PHTN does not always predict bleeding.13
Patients with presinusoidal PHTN HPVG measurement is not
increased because sinusoidal pressure is within normal limits.
Portal pressure is proportionally related to blood flow in the
portal system and resistance to the portal flow. The most important and common initial abnormality is the compromised vascular lumen secondary to fibrosis formation as seen in cirrhosis. The
obstruction can also be mechanical in nature, caused by a thrombus
or an outflow obstruction at the level of the sinusoids or hepatic

2. How do you describe the anatomy of the portomesenteric


system?
The portal venous system collects venous flow from the superior
mesenteric vein (SMV), inferior mesenteric vein (IMV), and splenic
vein (SV). The trunk formed by their union is the portal vein (PV),
which enters into the liver and divides into branches through its
substance and ends in capillaries. The branches are single and destitute of valves.
345

PMPH_CH42.indd 345

5/22/2012 5:29:47 PM

346

Surgery: Evidence-Based Practice

venous obstruction. This leads to the formation of collateral vessels that divert the portal blood to systemic circulation in addition
to a splanchnic vasodilation secondary increase in PV flow. Nitric
oxide, glucagon, and prostacyclin, along with other compounds,
are involved in this phenomenon, which aggravates this hyperdynamic state. The pathophysiology of PHTN is important in this
surgical chapter because of the reduced arterial pressure, increase
in cardiac output preceded by expanded plasma volume in relation to renal sodium retention and changes in peripheral resistance that may be directly correlated with the candidacy of this
patient for the different procedures to be discussed.8,14
4. What is the classification and treatment algorithm for bleeding PHTN?
PHTN can be classified as
a. prehepatic
b. hepatic
i. presinusoidal
ii. sinusoidal
iii. postsinusoidal
c. posthepatic
Based on this standard classification, therapeutic modalities
can be addressed depending on the level where the obstruction
to the portal flow occurs. Among the most common prehepatic
causes are EHPVO, which many time is idiopathic; portal vein
thrombosis (PVT) that can be related to cirrhosis; cancer; inherited or acquired prothrombotic conditions; focal inflammation; or
postoperative complications. The most common causes of hepatic
PHTN are cirrhosis, schistosomiasis (presinusoidal), congenital
hepatic fibrosis, and sarcoidosis. Postsinusoidal reasons are hepatic
vein thrombosis (Budd-Chiari syndrome) and veno-occlusive disease (VOD); and posthepatic usually related to cardiac etiologies or
inferior vena caval web.5,7,12,15
In May 2010, a workshop on the methodology of diagnosis
and therapy in PHTN met for the Baveno V consensus meeting.
This group of experts, who were responsible for most of the major
achievements in the field and authors of Level 1 and 2 studies on
the topic, reviewed therapeutic modalities in patients with PHTN
and issued evidenced-based recommendations for the conduct of
trials and guidelines for patient management.2 This position paper
divides their therapeutic options into five categories: pre-primary
prophylaxis, prevention of first bleeding episode, treatment of
acute bleeding, management of treatment failures, and prevention
of rebleeding.
The last two categories are of special interest in this review
since the patients that fail endoscopic and pharmacological therapy are the groups in which surgical treatment may be of use.
In adult patients with cirrhosis, the recommendations for
fi rst-line treatment to prevent rebleeding is a combination of
-blockers and endoscopic variceal band ligation (EBL), and a
follow-up of hemodynamic response. Isosorbide mononitrate
can be added to patients that do not have an adequate hemodynamic response or are unable to be treated with EBL. According
to the Baveno V therapeutic options, patients who fail to respond
to endoscopic and pharmacological treatment should then be
assessed for transjugular intrahepatic portosystemic shunt (TIPS)
or surgical shunting (SS) in appropriate candidates. Transplantation provides good long-term outcomes when available, in which
case TIPS are often used as a bridge.2

PMPH_CH42.indd 346

5. TIPS versus SS: When is surgery the treatment of choice and


when is a surgical procedure is considered for the treatment of
bleeding PHTN?
Patients with PHTN and/or liver cirrhosis would benefit from
surgical interventions when medical treatment fails, as TIPS procedure cannot be performed secondary to patients anatomical
problems, small children with long-term life expectancy without
transplantation especially biliary atresia patients with successful
Kasai procedure or children with EHPVO.1,16
The concept of selectivity was described by Warren and Zeppa16
in their landmark paper, in which they utilized a distal splenorenal
shunt (DSRS) and emphasized maintaining portal pressure and
flow to the liver (hepatopedal) via the SMV while alleviating the
portal pressure in the gastroesophageal and splenic areas. DSRS
was the first selective shunt described that addresses the left-sided
PHTN including esophageal and gastric varices. DSRS is largely
used when medical treatments failed in patients with cirrhosis and
PHTN. Preventing recurrent episodes of bleeding and diminishing
HE and liver failure should be a priority in SS.16,17
Another important concept to be discussed in the particular
case of extra hepatic vein obstruction is the physiologic redirection of blood from the spleno-mesenteric system into the LPV.
Due to the physiologic advantages it offers, the meso-Rex bypass
has become the standard procedure in such cases wherein this is
a concern.6,18 TIPS is a minimally invasive percutaneous way to
decompress the portal system. This procedure is performed by
placing a stent within the liver that connects the PV to the hepatic
vein therefore decompressing the portal system and preventing
bleeding. Compared with other medical therapies, TIPS is an
effective tool in decreasing portal pressures and it significantly
decreases recurrent variceal hemorrhage, but increases the likelihood of HE and has no effect on survival.19 In a 2009 update
of the American Association for the Study of Liver Diseases
practice guidelines, Boyer and Haskal 20 reviewed the role of TIPS
in the management of PHTN-related variceal bleeding. They suggested that TIPS should only be utilized when pharmacologic and
endoscopic therapies fail. Their recommendation also included the
use of expanded polytetrafluoroethylene (ePTFE)-covered stents
to decrease the incidence of shunt thrombosis. The authors also
showed that TIPS are slightly more cost-effective than DSRS at 5
years, but the two approaches are equal in efficacy.20 On the other
hand, multicenter trial conducted by Henderson et al.21 showed that
thrombosis, stenosis, and reintervention rates were significantly
higher in the TIPS group with the p < .001.21 There are also absolute
contraindications for TIPS procedure including congestive heart
failure, multiple hepatic cyst, sepsis, unrelieved biliary obstruction,
severe pulmonary hypertension and hepatoma. The obstruction of
PV or all hepatic veins used to be considered relative contraindications in the past. In patients with portal hypertension, anticoagulation therapy is the first line treatment. TIPS procedure should be
preferred over liver transplantation when patients fail to improve
with anticoagulation since the current data indicates that 1- and 10
years transplant free survival rates after TIPS is favorable.20
Porto-caval shunt (PCS) surgery is a very effective tool in preventing rebleeding from GEV in patients with PHTN, but it is
controversial as there is an increased risk of HE and no survival
advantage.22 A large multicenter trial of TIPS versus DSRS was
conducted in well-selected patients who failed to medical and
endoscopic treatments.21 In this study, the rebleeding incidence
was 5.5% in DSRS group versus 10.5% in TIPS; not significant

5/22/2012 5:29:47 PM

Management of Portal Hypertension: A Surgical Perspective

with no difference in survival. On the other hand, this study also


showed that TIPS group had very high reintervention rates (DSRS,
11%; TIPS, 82%; p < .001). This study also revealed that ascites formation, need for transplant, quality of life, and costs were not significantly different between two groups.21
One of the undesired side effects of TIPS procedure is the development of worsening encephalopathy. In some published series the
incidence of worsening encephalopathy reached 20% to 31%.23 In
contrast, the incidence of encephalopathy occurred 10% to 15% of the
cases after selective shunt surgery in many different series.16,17,21,23,24
As it was mentioned at the beginning of this chapter, there are
patients who will not be candidates for TIPS or medical therapy
and therefore it is important to understand the value of SS and the
meso-Rex bypass and their specific indications.
Surgical shunts can be classified into totally diverting shunts,
partially diverting shunts, and selective shunts.
6. What would be the proper evaluation for shunt surgery?
Perhaps the most important factor in the decision-making of the
surgical treatment of PHTN is the status of the patient. Evaluating
the hepatic reserve, by utilizing either Child-Turcotte-Pugh score
(CPT) or the Model for End-Stage Liver Disease (MELD) facilitates the surgical decision making.25 A complete medical evaluation of comorbidities such as obesity, age, preexisting cardiac or
pulmonary conditions, and active infection are essential. Patients
with PHTN may develop porto-pulmonary hypertension. When
shunt surgery is considered in patients with PHTN, measuring the
pulmonary artery pressure is paramount important since shunt
surgery is contraindicated in patients with pulmonary hypertension. We also suggest that evaluation of socioeconomic issues such
as medical access, treatment cost and distance to the center should
be taken into account before any type of therapy is chosen.18,21,24-27
Evaluation should also include liver biopsy, liver function
tests, complete blood count, tests for hypercoaguable states, and
adequate imaging to define the anatomy of the portomesenteric
system as well as confirming the patency of the intrahepatic PV
branches, in the case of the meso-Rex bypass for EHPVO in children. Superina et al.31 have contended that not identifying the patency of the intrahepatic PV branches on preoperative imaging is
not a contraindication for the procedure since in most cases the
patency of the intrahepatic PV branches will be identified intraoperatively after dissection of the recessus of Rex. On the other
hand, in one of our cases, an intraoperative portagraphy revealed
the totally collapsed intrahepatic PVs, which led us to abandon
the operation. Since then, it has been our policy to perform a preoperative transhepatic portography to confirm the patency of the
intrahepatic PV branches. The operative technique includes performing an intraoperative portal venography after LPV is accessed
via the UV. The neck of the patient should be prepared in case the
internal jugular (IJ) vein is needed for the bypass. Often there is
a septum of liver encountered between segments III and IV, and
needs to be divided to get adequate exposure to the LPV.28-31
Different conduits have been described for the creation of
the bypass, IJ, the UV in which case if recanalized with adequate
length only one anastomosis may be needed, if the length of the
UV is not enough this can be used as a bridge with the IMV.10
Others have used the SV, saphenous vein, coronary vein, synthetic
vascular, or cryopreserved vein grafts.18,32 After patency has been
confirmed, the LPV to IJ anastomosis is performed with running
6-0 monofilament sutures. After the conduit is brought to the

PMPH_CH42.indd 347

347

vicinity of the SMV via transmesocolic fashion, the end-to-side


anastomosis is performed between the conduit and the SMV using
6-0 polypropylene sutures. Patients are usually anticoagulated
postoperatively with low molecular weight heparin (LMWH) and
depending on their coagulation work-up preoperatively they will
either continue on LMWH or transitioned to aspirin. In special
cases a heparin drip will be started intraoperatively.
7. What procedure is indicated based on the etiology, classification, and anatomy of PHTN as well as spleno-mesenteric system?
In 1945, Whipple and Blakemore introduced the clinical use of
the portosystemic shunting concept that Eck had experimentally
performed and published in 1877.33-35 It was in the following years
that the concept of preservation of portal flow was recognized as
key to the survival of these patients. Although PCS were demonstrated to be an excellent way to decompress the GEV, most
patients developed hepatic decompensation and the long-term
survival rates were poor.18,25,36
These procedures were usually done by anastomosing the PV,
SV, or SMV to the vena cava, the left renal vein or another major
tributary of the vena cava. All are considered total shunts.37,38

PCS AND LOW DIAMETER H-GRAFTS


The original PCSs were performed in an end-to-side fashion, and were
effective in decreasing the portal pressures, but deprived the liver
from the portal blood flow. Julian and Metcalf39 described the sideto-side PCS as preserving hepatopedal flow. These shunts have high
patency rates. Severe encephalopathy is the major complication
of this type of shunt procedure with variable long-term survival.38
The outcomes of elective shunts such as Warrens DSRS to total
shunts such as the PCSs were compared in clinical trials. These
studies revealed that postshunt encephalopathy was seen in 18% to
30% of patients with excellent long-term shunt patency.33,40,41 PCS
are still utilized by many groups and the concept of low diameter
has been utilized in the mesocaval position.36,40

MESOCAVAL SHUNT
The concept of shunting SMV flow to vena cava began in the 1950s.
It was further developed by Lord et al.42 in the 1960s and then championed by Drapanas41 in 1972. His original series, presented at the
American Surgical Association, consisted of 25 patients with 8%
mortality using the procedure on Childs class B and C patients.40
His technique utilized knitted Dacron 19- to 22-mm graft. His
description and results have been cited by many groups. There were
discussions and disagreements in terms of selectivity and higher
incidence of HE due to the size of the graft. These discussions
played an important role for the development of the small diameter (810 mm) synthetic vascular interposition graft utilization
for mesocaval shunt (MCS). Before analyzing other evidence-based
studies, it is important to mention that prior to the Drapanas paper,
others had described the use of interposition grafts between vessels,
mainly homografts like jugular vein from the patient and Teflon
grafts.43 Scudamore et al.35 published their experience of using a
medium size (10 mm) PTFE graft in Vancouver. They reported 0%
rebleeding rate, 0% mortality, and 20% postshunt encephalopathy
in their patients with a mean follow-up time of 14 months.35

5/22/2012 5:29:47 PM

348

Surgery: Evidence-Based Practice

A prospective German study conducted by Paquet et al.44


analyzed 100 patients with mesocaval interposition shunts with
12.2% incidence of encephalopathy, 10% incidence of early mortality, and 65% long-term survival. Interestingly, it was shown in
sequential portal perfusion studies that 75% of the patients maintained their hepatopedal flow immediately after the MCS shunt
procedure. When this study was repeated 2 years later, only 38%
of the patients maintained hepatopedal flow, indicating that the
MCS becomes nonselective shunt by time. A prospective controlled study comparing the outcomes of DSRS with MCS performed using 10-mm interposition synthetic grafts was published
by Mercado et al.45 This study showed that DSRS group had significantly less shunt thrombosis rates than MCS (4.3% vs. 27%).
There was also a higher incidence of HE in the MCS group compared with DSRS group (36% vs. 4.3%).
From a technical point of view, all the authors agreed that the
vena cava to interposition graft anastomosis should be performed
as a first step to eliminate exposure problems at the level of the
vena cava and to avoid the prolonged occlusion of the SMV. The
length of the PTFE graft is crucial; it must be long enough to pass
inferiorly around the lower border of the third portion of the duodenum; it should also not be redundant since redundancy could
cause kinking of the graft.

SPLENORENAL AND
SPLENOCAVAL SHUNTS
The original concept of utilizing the SV as a conduit for shunting the portal blood flow in patients with PHTN goes back to the
beginning of PHTN surgery.35 In early reports, the proximal SV
was anastomosed to the renal vein due to their proximity, diverting blood flow from the high-pressure portal system into the lowpressure systemic venous circulation.39 This operation is called a
proximal or central splenorenal shunt.
Warren et al.16 at the University of Miami developed a new
shunt procedure in which the distal end of the SV was anastomosed to the side of the left renal vein. They were able to show
a decrease in incidence of HE while maintaining hepatic function by preserving the SMV flow to the liver. Although some of
the results were not reproducible by all groups and the utility
in patients with advance liver disease especially Childs class C
cirrhotics has been challenged by many, until the development
of liver transplantation (LT), many of these patients had limited options for treatment once medical therapy had failed.17,21
A variant of these selective shunts is the splenocaval, which,
as described by Orozco, has the same principal as the Warren.
However, when the renal vein is not feasible for anastomosis
secondary to large spleen pushing the left kidney distally, this
variation of DSRS may be a reasonable option by anastomosing
the distal SV to the vena cava directly.46 Other efforts were done
to insure the preservation of hepatopedal flow and prevent Ecks
syndrome, such as the left gastric venous caval shunt described
by Inokuchi et al.47,48
Generally, all portosystemic shunts have more than 90% control preventing variceal bleedings. In terms of their outcomes,
selective shunts are superior to the nonselective shunts with fewer
incidences of postshunt encephalopathy and other morbidities.23
One of the major complications of the DSRS is the development of the PVT. Total and partial PVT were seen on 13 (10.5%)
and 22 (17.7%) patients.49

PMPH_CH42.indd 348

MESENTERIC TO LEFT PV BYPASS


(MESO-REX BYPASS)
The concept of the mesenteric to LPV bypass (MLPVB) was originally reported by de Ville de Goyet for the treatment of PVT after
LT by accessing the PV in the fissure between the medial and
lateral segment of the left lobe of the liver (recessus of Rex). The
original technique uses the patients left IJ vein anastomosed endto-side to the LPV and implanted in the SMV.28,29 The term Rex
shunt is no longer used because this bypass redirects portal flow
from the spleno-mesenteric system into the liver restoring physiologic hepatopedal flow. Therefore, this procedure is not only anatomically a bypass, but also functionally a bypass.
The meso-Rex bypass has shown to improve growth and
neuro-cognition in children with EHPVO especially if it is performed earlier.18,30
The meso-Rex bypass has also been described in adult patients
that have developed PVT as a complication of other procedures and as
described originally for PVT posttransplant with good success.15,50-52

DEVASCULARIZATION PROCEDURES
Nonshunting procedures were developed as a life-saving temporary measure to control/prevent esophageal and gastric variceal
bleeding in patients with total thrombosis of portomesenteric system. Between 1967 and 1973, Hassab53 in Egypt and Sugiura and
Futagawa54 in Japan described devascularization procedures of the
esophagus in one or two stages depending on the patients overall medical condition. Multiple modifications have been made to
the original procedures in both pediatric and adult patients with
good results especially in patients with Childs class A and B. These
changes include modifications to the esophageal transection, avoiding splenectomy and performing single stage operation. We prefer
to perform this procedure after variceal bleeding is controlled and
patient becomes hemodynamically stable. Performing the surgery
with transabdominal approach, ligating and dividing of the splenic
artery rather than splenectomy, ligating varices entering the distal
5 cm of the esophagus via abdominal approach,, minimizing the gastric devascularization to proximal 1/3 of the stomach, exploring the
posterior aspects of the stomach wall for large variceal veins entering to the stomach and ligating them, using appropriate size circular
intraluminal stapler device for esophageal transection are important steps for successful outcomes. In most cases now we reserve
devascularization in patients in whom there are no shunt or bypass
options with life-threatening variceal bleeding. In selected groups,
the mortality rate in these cases is between 6% and 7% with approximately10% rebleeding rate and no HE.54-56
8. What is the role of emergency shunt surgery and transplantation in patients with bleeding esophageal varices?
In a randomized clinical trial, Orloff et al. 57 showed excellent
results with PCS in emergency situations over the years with only
6% of their patients were referred to LT secondary to progression
of liver disease. Other groups have not been able to reproduce the
similar results; they have reported 13% to 40% mortality in emergency cases. The report from the Baveno V consensus conference
on PHTN suggests utilizing medical, endoscopic, and radiologic
alternatives as a first-line treatment for patients with variceal
bleedings. Emergent surgery is used as a last resource in case of
failure of other treatment modalities.2

5/22/2012 5:29:47 PM

Management of Portal Hypertension: A Surgical Perspective

The definitive procedure for the treatment of PHTN in decompensated cirrhotic patients is LT. The current survival data
reported on patients receiving LT are higher than the observed on
nonresponders to the medical treatment for recurrent esophageal
variceal bleeding. As a result, the success of LT has revolutionized the field.10,22 We believe that LT is a life-saving procedure for
decompensated cirrhotic patient with gastroesophageal variceal
bleeding. On the other hand, availability of cadaveric organs is
unpredictable and current organ allocation system in the United
States does not prioritize patients with bleeding esophageal varices.
This will create a dilemma to perform timely transplantation in
this patient population.
9. When balloon tamponade is indicated using Sengstaken
Blakemore tube and how is this performed?
Balloon tamponade of the bleeding esophageal varices using
SengstakenBlakemore tube is indicated when overwhelming

349

bleeding cannot be controlled with medical treatment and/or TIPS


procedure. Development of new technologies such as TIPS, more
aggressive secondary prophylaxis with variceal band ligation procedures resulted in less and less use of balloon tamponade. On the
other hand, we do believe that knowing how to use Sengstaken
Blakemore tube appropriately is still important especially large
hepatology/liver transplant units. SengstakenBlakemore tube
is more complex form of nasogastric tube with several internal
channels and two inflatable balloons, one for the stomach and
more elongated one for the esophagus. It is important to make
sure that the stomach balloon is in the stomach. Stomach balloon
is inflated and approximately 1 kg traction is applied. This is very
important to compress the gastroesophageal junctional varices. If
bleeding cannot be controlled, inflating the esophageal balloon
is indicated to control bleeding. Known complications of this
procedure are development of pressure ulcers and rupture of the
esophagus and stomach.58-60

Clinical Question Summary


Question

Answer

Levels of
Evidence

Grade of
References
Recommendation

1 What is the incidence


and risk of mortality
in patients with
portal hypertension
(PHTN) and bleeding
gastro-esophageal
varices (GEV)

The 6-week mortality of variceal haemorrhage is


still 10% to 20%. GEVs are present in 30% of
well-compensated cirrhotics, and 60% of patients
with decompensated cirrhosis. The increase in
varix size is 10% to 20% from small to large in
1 year and the index of variceal bleeding is >20%
in patients with large varices.

1A, 1C

1-4

2 How do you describe


the anatomy of the
portomesenteric
system?

The portal venous system collects venous flow from


the SMV, IMV, and SV. The trunk formed by their
union is the PV, which enters into the liver and
divides into branches through its substance and
ends in capillaries. The branches are single and
destitute of valves. The PV is formed behind the
upper border of the head of the pancreas by the
junction of the SV and SMV, in front of the vena
cava. The IMV drains into the SV in most of the
population but may also enter the junction of
the SV and the SMV or drain into the SMV.

1A

8-10

3 What is the definition


and pathophysiology
of PHTN?

PHTN is defined as a pathological increase of PV


pressure (PVP) above the normal range of 5 to
8 mmHg. PVP is proportionally related to blood
flow in the portal system and resistance to the
portal flow. The most important and common
initial abnormality is the compromised vascular
lumen secondary to fibrosis formation as seen in
cirrhosis. The obstruction can also be mechanical
in nature, caused by a thrombus or an outflow
obstruction at the level of the sinusoids or hepatic
veins. This leads to the formation of collateral
vessels that divert the portal blood to systemic
circulation, in addition to a splanchnic vasodilation
secondary increase in PV flow. Nitric oxide,
glucagon, and prostacyclin, along with other
compounds, are involved in this phenomenon that
aggravates this hyperdynamic state.

1A, 1B

4, 11-13

(Continued)

PMPH_CH42.indd 349

5/22/2012 5:29:47 PM

350

Surgery: Evidence-Based Practice

(Continued)
Question

Answer

4 What is the
classification and
treatment algorithm
for bleeding PHTN?

PHTN can be classified as (1) prehepatic; (2) hepatic


(i. presinusoidal, ii. sinusoidal, and iii. postsinusoidal); (3) post hepatic. Treatment algorithm
includes medical treatment (-blocker,
octreotide) and endoscopic band ligation. When
medical treatment fails, TIPS or shunt surgery is
indicated. Patient with decompensated cirrhosis
LT will be the ideal operation. In patients with
EHPVO, the meso-Rex bypass is the treatment
of choice.

5 TIPS versus shunt:


When is surgery
the treatment of
choice and when is
a surgical procedure
is considered for
the treatment of
bleeding PHTN?

Levels of
Evidence

Grade of
References
Recommendation

1A, 1B

2, 5, 7, 12,
16

Patients with PHTN and/or liver cirrhosis


would benefit from surgical interventions when
medical treatment fails. If TIPS procedure
cannot be performed secondary to patients
anatomical problems, patients with Childs
class A cirrhosis and small children with
long-term life expectancy without transplantation or children with EHPVO surgical
treatment is indicated.

2B

1, 6, 17-25

6 What would be the


proper evaluation
for shunt surgery?

Evaluation should include complete blood count,


liver function tests, tests for hypercoaguable
states, and liver biopsy. A complete medical
evaluation of comorbidities such as obesity, age,
preexisting cardiac or pulmonary conditions,
and active infection are essential. Evaluation of
socioeconomic issues such as medical access,
treatment cost, and distance to the center
should also be taken into account. Obtaining
adequate imaging of the porto-mesenteric
system including the patency of the intrahepatic
PV branches in case of EHPVO is crucial.
Evaluating the hepatic reserve, by utilizing either
CPT or MELD facilitates the surgical decision
making.

3B

19, 22,
25-33

7 What procedure
is indicated based
on the etiology,
classification,
and anatomy of
PHTN as well as
spleno-mesenteric
system?

A patient with cirrhosis, selective shunt such as


DSRS is preferred to minimize the postshunt
encephalopathy. A variety of the PCS are also
used successfully. For a patient with EHPVO,
the meso-Rex shunt is the treatment of
choice. Patients with porto-mesenteric
thrombosis, nonshunt surgical procedure
such as Sugiura operation will be life-saving.

1B, 3B, 4

19, 24, 26,


37-50

8 What is the role


of emergency
shunt surgery
and transplantation
in patients with
bleeding esophageal
varices?

Only one group has shown excellent results with


PCS in emergency situations with only 6% of the
patients were referred to LT. Other groups have
not been able to reproduce the similar results;
they have reported 13% to 40% mortality in
emergency cases. The report from the Baveno V
suggests that emergent surgery is used as a
last resource. LT is the ultimate treatment
with decompensated cirrhotic patients with
bleeding GEV.

1A, 2B

2, 10, 23,
58

(Continued)

PMPH_CH42.indd 350

5/22/2012 5:29:47 PM

Management of Portal Hypertension: A Surgical Perspective

351

(Continued)
Question

Answer

9 When balloon
tamponade is
indicated using
Sengstaken
Blakemore tube
and how is this
performed?

Balloon tamponade of the esophageal varices using


SengstakenBlakemore tube is indicated when
bleeding cannot be controlled with medical
treatment and/or TIPS procedure. Many times it
is necessary to stabilize a patient for endoscopic
or surgical procedure. Appropriate application
of SengstakenBlakemore tube is important.
It is a more complex form of nasogastric tube
with several internal channels and two inflatable
balloons, one for the stomach and more elongated
one for the esophagus. It is important to make
sure that the stomach balloon is in the stomach.
Stomach balloon is inflated and approximately
1 kg traction is applied. This is very important
to compress the gastroesophageal junctional
varices. If bleeding cannot be controlled,
inflating the esophageal balloon is indicated to
control bleeding.

REFERENCES
1. Shneider B, Emre S, Groszmann R, et al. Expert pediatric opinion
on the Report of the Baveno IV consensus workshop on methodology of diagnosis and therapy in portal hypertension. Pediatr
Transplant. 2006;10:893-907.
2. de Franchis R. Revising consensus in portal hypertension: Report
of the Baveno V consensus workshop on methodology of diagnosis
and therapy in portal hypertension. J Hepatol. 2010;53:762-768.
3. Lebrec D, De Fleury P, Rueff B, Nahum H, Benhamou JP. Portal
hypertension, size of esophageal varices, and risk of gastrointestinal bleeding in alcoholic cirrhosis. Gastroenterology. 1980;79:
1139-1144.
4. Wadhawan M, Dubey S, Sharma BC, Sarin SK. Hepatic venous
pressure gradient in cirrhosis: Correlation with the size of varices,
bleeding, ascites, and childs status. Dig Dis Sci. 2006;51:2264-2269.
5. Sarin SK, Kumar A. Noncirrhotic portal hypertension. Clin Liver
Dis. 2006;10:627-651, x.
6. Superina R, Shneider B, Emre S, Sarin S, de Ville de Goyet J. Surgical guidelines for the management of extra-hepatic portal vein
obstruction. Pediatr Transplant. 2006;10:908-913.
7. Sarin SK, Sollano JD, Chawla YK, et al. Consensus on extrahepatic portal vein obstruction. Liver Int. 2006;26:512-519.
8. Marvin MR, Emond JC. Cirrhosis and Portal Hypertension
(Chapter 58)Greenfields surgery: Scientific principles and practice. Philadelphia, PA: Wolters Kluwer Health/Lippincott
Williams & Wilkins; 2011.
9. Couinaud C. Liver anatomy: Portal (and suprahepatic) or biliary
segmentation. Dig Surg. 1999;16:459-467.
10. Facciuto ME, Rodriguez-Davalos MI, Singh MK, et al. Recanalized
umbilical vein conduit for meso-Rex bypass in extrahepatic portal
vein obstruction. Surgery. 2009;145:406-410.
11. Garcia-Tsao G, Groszmann RJ, Fisher RL, Conn HO, Atterbury
CE, Glickman M. Portal pressure, presence of gastroesophageal
varices and variceal bleeding. Hepatology. 1985;5:419-424.

PMPH_CH42.indd 351

Levels of
Evidence
4

Grade of
References
Recommendation
B

59-61

12. Groszmann RJ, Bosch J, Grace ND, et al. Hemodynamic events in


a prospective randomized trial of propranolol versus placebo in
the prevention of a first variceal hemorrhage. Gastroenterology.
1990;99:1401-1407.
13. Rigau J, Bosch J, Bordas JM, et al. Endoscopic measurement of
variceal pressure in cirrhosis: Correlation with portal pressure and
variceal hemorrhage. Gastroenterology. 1989;96:873-880.
14. Bosch J, Pizcueta P, Feu F, Fernandez M, Garcia-Pagan JC. Pathophysiology of portal hypertension. Gastroenterol Clin North Am.
1992;21:1-14.
15. Reichman TW, Anthony T, Testa G. Treatment of extrahepatic
portal hypertension following a Whipple procedure with a Rex
shunt: Report of a case. Surg Today. 2011;41:292-296.
16. Warren WD, Zeppa R, Fomon JJ. Selective trans-splenic decompression of gastroesophageal varices by distal splenorenal shunt.
Ann Surg. 1967;166:437-455.
17. Livingstone AS, Koniaris LG, Perez EA, Alvarez N, Levi JU, Hutson DG. 507 WarrenZeppa distal splenorenal shunts: A 34-year
experience. Ann Surg. 2006;243:884-892; discussion 892-894.
18. Emre S, Dugan C, Frankenberg T, et al. Surgical portosystemic
shunts and the Rex bypass in children: A single-centre experience. HPB (Oxford). 2009;11:252-257.
19. Groszmann RJ, Bosch J. Portal Hypertension in the 21st Century. Dordrecht/Boston: Kluwer Academic Publishers; 2004:
xviii, 370.
20. Boyer TD, Haskal ZJ. The role of transjugular intrahepatic portosystemic shunt (TIPS) in the management of portal hypertension:
Update 2009. Hepatology. 2010;51:306.
21. Henderson JM, Boyer TD, Kutner MH, et al. Distal splenorenal
shunt versus transjugular intrahepatic portal systematic shunt
for variceal bleeding: A randomized trial. Gastroenterology.
2006;130:1643-1651.
22. Garcia-Tsao G, Sanyal AJ, Grace ND, Carey W. Prevention and
management of gastroesophageal varices and variceal hemorrhage in cirrhosis. Hepatology. 2007;46:922-938.

5/22/2012 5:29:48 PM

352

Surgery: Evidence-Based Practice

23. Rikkers LF, Rudman D, Galambos JT, et al. A randomized, controlled trial of the distal splenorenal shunt. Ann Surg. 1978;
188:271-282.
24. Orozco H, Mercado MA. Rise and downfall of the empire of
portal hypertension surgery. Arch Surg. 2007;142:219-221.
25. Boyer TD, Henderson JM, Heerey AM, et al. Cost of preventing variceal rebleeding with transjugular intrahepatic portal
systemic shunt and distal splenorenal shunt. J Hepatol. 2008;48:
407-414.
26. Orozco H, Mercado MA, Rodriguez-Davalos M. Surgery in elderly
patients in Mexico. Portal hypertension surgery as an example.
Arch Surg. 1997;132:1126-1128.
27. Malinchoc M, Kamath PS, Gordon FD, Peine CJ, Rank J, ter
Borg PC. A model to predict poor survival in patients undergoing transjugular intrahepatic portosystemic shunts. Hepatology.
2000;31:864-871.
28. Bambini DA, Superina R, Almond PS, Whitington PF, Alonso E.
Experience with the Rex shunt (mesenterico-left portal bypass)
in children with extrahepatic portal hypertension. J Pediatr Surg.
2000;35:13-18; discussion 18-19.
29. de Ville de Goyet J, Gibbs P, Clapuyt P, Reding R, Sokal EM, Otte
JB. Original extrahilar approach for hepatic portal revascularization and relief of extrahepatic portal hypertension related to
later portal vein thrombosis after pediatric liver transplantation.
Long term results. Transplantation. 1996;62:71-75.
30. Mack CL, Zelko FA, Lokar J, et al. Surgically restoring portal
blood flow to the liver in children with primary extrahepatic
portal vein thrombosis improves fluid neurocognitive ability.
Pediatrics. 2006;117:e405-e412.
31. Superina R, Bambini DA, Lokar J, Rigsby C, Whitington PF.
Correction of extrahepatic portal vein thrombosis by the mesenteric to left portal vein bypass. Ann Surg. 2006;243:515-521.
32. Chiu B, Pillai SB, Sandler AD, Superina RA. Experience with
alternate sources of venous inflow in the meso-Rex bypass operation: The coronary and splenic veins. J Pediatr Surg. 2007;42:
1199-1202.
33. Blakemore AH, Lord JW. The technic of using vitallium tubes
in establishing portacaval shunts for portal hypertension. Ann
Surg. 1945;122:476-489.
34. Whipple AO. The problem of portal hypertension in relation to
the hepatosplenopathies. Ann Surg. 1945;122:449-475.
35. Scudamore CH, Erb SR, Morris C, et al. Medium aperture mesocaval shunts reliably prevent recurrent variceal hemorrhages.
Am J Surg. 1996;171:490-494.
36. Clark W, Hernandez J, McKeon B, et al. Surgical shunting versus transjugular intrahepatic portasystemic shunting for bleeding varices resulting from portal hypertension and cirrhosis: A
meta-analysis. Am Surg. 2010;76:857-864.
37. Linton RR, Hardy IB, Jr., Volwiler W. Portacaval shunts in the
treatment of portal hypertension; an analysis of 15 cases with
special reference to the suture type of end-to-side splenorenal
anastomosis with splenectomy and preservation of the kidney.
Surg Gynecol Obstet. 1948;87:129-144.
38. Costa G, Cruz RJ, Jr., Abu-Elmagd KM. Surgical shunt versus
TIPS for treatment of variceal hemorrhage in the current era
of liver and multivisceral transplantation. Surg Clin North Am.
2010;90:891-905.
39. Julian OC, Metcalf W. Nonobstructive lateral portal vein-vena
cava anastomosis. Arch Surg. 1946;59(3):433-436.
40. Rosemurgy AS, Bloomston M, Clark WC, Thometz DP, Zervos
EE. H-graft portacaval shunts versus TIPS: Ten-year follow-up of
a randomized trial with comparison to predicted survivals. Ann
Surg. 2005;241:238-246.

PMPH_CH42.indd 352

41. Drapanas T. Interposition mesocaval shunt for treatment of portal hypertension. Ann Surg. 1972;176:435-448.
42. Goetz RH, Lord JW Jr. Crossover femoropopliteal shunt. Surgery
1968;64(3):681-684.
43. Austin MT, Pinson W. The Mesocaval Shunt: Technique (Chapter 99)
Surgery of the Liver, Biliary Tract, and Pancreas. Philadelphia,
PA: Saunders Elsevier; 2007:1647-1653.
44. Paquet KJ, Mercado MA, Kalk JF, Koussouris P, Siemens F,
Muting D. Analysis of a prospective series of 100 mesocaval
interposition shunts for bleeding portal hypertension. Hepatogastroenterology. 1990;37:115-120.
45. Mercado MA, Morales-Linares JC, Granados-Garcia J, GomezMendez TJ, Chan C, Orozco H. Distal splenorenal shunt versus
10-mm low-diameter mesocaval shunt for variceal hemorrhage.
Am J Surg. 1996;171:591-595.
46. Orozco H, Mercado MA. The evolution of portal hypertension
surgery: Lessons from 1000 operations and 50 Years experience.
Arch Surg. 2000;135:1389-1393; discussion 1394.
47. Inokuchi K, Kobayashi M, Kusaba A, et al. New selective decompression of esophageal varices by a left gastric venous-caval shunt.
Arch Surg. 1970;100:157-162.
48. Yoshida H, Mamada Y, Taniai N, Tajiri T. New trends in surgical
treatment for portal hypertension. Hepatol Res. 2009;39:1044-1051.
49. Jin GL, Rikkers LF. The significance of portal vein thrombosis after
distal splenorenal shunt. Arch Surg. 1991;126:1011-1015; discussion
1015-1016.
50. Gibelli NE, Tannuri AC, Tannuri U, et al. Rex shunt for acute
portal vein thrombosis after pediatric liver transplantation in
children with biliary atresia. Transplant Proc. 2011;43:194-195.
51. Yamamoto S, Sato Y, Oya H, et al. Splenic-intrahepatic left portal
shunt in an adult patient with extrahepatic portal vein obstruction
without recurrence after pancreaticoduodenectomy. J Hepatobiliary Pancreat Surg. 2009;16:86-89.
52. Krebs-Schmitt D, Briem-Richter A, Grabhorn E, et al. Effectiveness of Rex shunt in children with portal hypertension following
liver transplantation or with primary portal hypertension. Pediatr Transplant. 2009;13:540-544.
53. Hassab MA.Gastroesophageal decongestion and splenectomy. A
method of prevention in treatment of bleeding from esophageal
varices associated with bilharzial fibrosis:Preliminary report. J
Internat Coll Surg. 1964;41:232-248.
54. Sugiura M, Futagawa S. A new technique for treating esophageal
varices. J Thorac Cardiovasc Surg. 1973;66:677-685.
55. Superina RA, Weber JL, Shandling B. A modified Sugiura operation
for bleeding varices in children. J Pediatr Surg. 1983;18:794-799.
56. Selzner M, Tuttle-Newhall JE, Dahm F, Suhocki P, Clavien PA.
Current indication of a modified Sugiura procedure in the management of variceal bleeding. J Am Coll Surg. 2001;193:166-173.
57. Orloff MJ, Isenberg JI, Wheeler HO, et al. Liver transplantation in a randomized controlled trial of emergency treatment of
acutely bleeding esophageal varices in cirrhosis. Transplant Proc.
2010;42(10):4101-4108.
58. Bauer JJ, Kreel I, Kark AE. The use of SengstakenBlakemore
tube for immediate control of bleeding esophageal varices. Ann
Surg. 1974;179(3):273-277.
59. Hobolth L, Krag A, Malchow-Mller A, et al. Adherence to guidelines in bleeding oesophageal varices and effects on outcome:
Comparison between a specialized unit and a community hospital.
Eur J Gastroenterol Hepatol. 2010;22(10):1221-1227.
60. Chien JY, Yu CH. Malposition of a SengstakenBlakemore tube.
N Engl J Med. 2005;352:e7.
61. DeLeve LD, Valla DC, Garcia-Tsao G. Vascular disorders of the
liver. Hepatology. 2009;49:1729-1764.

5/22/2012 5:29:48 PM

CHAPTER 43

Management of Intractable
Ascites: The Evidence
Mark I. E. Cockburn and Adora Fou-Cockburn

to 400 mg/d.11 Loop diuretics, such as furosemide, are frequently


used as an adjunct to spironolactone therapy. The initial oral dose
of furosemide is usually 20 to 40 mg/d, and is generally adjusted
upward every few days up to a maximum of 160 mg/d.
Answer: There is Level 1b evidence that spironolactone is the
diuretic of choice in the management of patients with liver cirrhosis and ascites. (Grade A recommendation).

INTRODUCTION
Ascites can be the result of liver cirrhosis (75% of patients presenting with ascites), malignancy (10%), cardiac failure (3%), pancreatitis (1%), tuberculosis (2%), or other rarer causes.1 When ascites
develops as a complication of liver disease, it is associated with a
poor prognosis.2 Almost 10% of patients with cirrhosis and ascites
develop intractable/refractory ascites3 and medically intractable/
refractory ascites is generally thought to presage death due to liver
failure with relatively small 1- and 2-year survival rates.4,5 According to the International Ascites Club, refractory ascites is defined
as ascites that cannot be mobilized or whose early recurrence
after paracentesis cannot be satisfactorily prevented by medical
therapy.6

2. What is the role of paracentesis in the management of intractable ascites?


There are many randomized controlled trials comparing therapeutic paracentesis with diuretics in cirrhotic patients with ascites12-16
and these have shown that paracentesis was more effective than diuresis in eliminating ascites and shortened the duration of hospitalization. Tito et al.17 showed that total paracentesis was as effective
and as safe as repeated partial paracentesis. Paracentesis-induced
circulatory dysfunction (PICD) is the most common complication
of large-volume paracentesis and refers to the effective hypovolemia and renal impairment seen after large-volume paracentesis.
When the ascites is tapped, the intraabdominal pressure rapidly
falls, thereby improving the venous blood return to the right heart
and a transient increase in cardiac output may occur.18 There is
also resultant hyponatremia, which suggests that the paracentesisinduced decrease in effective volume may also stimulate vasopressin
secretion with subsequent retention.19 Logically the use of plasma
expanders has been suggested to prevent PICD. Gines et al.16 conducted a large multicenter RCT that demonstrated the superiority
of albumin compared with synthetic plasma expanders in preventing PICD.
There are no absolute contraindications to perform paracentesis, but the procedure should be avoided in patients with disseminated intravascular coagulopathy and extreme caution should
be taken in patients with intraabdominal adhesions or with a
distended urinary bladder.
Answer: There is Level 1b evidence that shows paracentesis
superior to diuretics in eliminating ascites and shortening hospital stay in patients with ascites. (Grade A recommendation).

1. What is the medical management of ascites?


The aim of treatment of ascites is to improve sodium balance or
circulatory function until liver transplantation or until the disease
runs its natural course. Upright posture activates sodium-retaining
systems and impairs renal perfusion and sodium excretion. In one
study bed rest improved the response to diuretics7 but no clinical
trials have shown that bed rest actually improves the efficacy of
medical treatment.
Although the use of low-salt diets is almost universally recommended, this approach is not backed by the results of controlled clinical trials. In one controlled study, a slightly reduced
salt diet (120 mmol/d) was equally effective in patients with
ascites when compared with low-salt diet (50 mmol/d).8 There are
no significant differences in survival between patients receiving
salt-restricted or -unrestricted diets, although the survival of
patients with previous gastrointestinal bleeding was better in the
low-salt group.9
There are both controlled and uncontrolled clinical trials
showing that spironolactone is the drug of choice for the initial treatment of secondary hyperaldosteronism. 8-11 The recommended dose of spironolactone is 100 to 200 mg once daily. When
severe hyperaldosteronism is present, the dosage may be increased
353

PMPH_CH43.indd 353

5/22/2012 5:30:22 PM

354

Surgery: Evidence-Based Practice

3. What is the role of transjugular intrahepatic portosystemic


shunt (TIPS) in the management of intractable ascites?
There are five RCTs that have compared the effects of TIPS versus
large-volume paracentesis as a treatment for intractable ascites.20-24
One hundred and sixty-two patients out of a total of 330 enrolled
underwent TIPS, while 168 underwent large-volume paracentesis. TIPS eliminated ascites in two-thirds (67%) of the patients
undergoing this procedure (TIPS), while large-volume paracentesis controlled ascites in 23% of the patients undergoing largevolume paracentesis.25 This meta-analysis also showed that TIPS
is associated with a trend toward improved survival. In another
meta-analysis on the data from four of the RTCs,21-24 it was shown
that patients receiving TIPS showed a significantly better survival
than patients receiving paracentesis.26 Of note is that not all the
patients in those four RTCs had refractory ascites, but recidivant
ascites. The RTC20 that was not included in this meta-analysis was
that with the worst outcomes. In this study,20 none of the ChildPugh class C patients eliminated their ascites and their overall
survival was significantly worse in the TIPS group. Hence, TIPS
is generally considered to be contraindicated in Child-Pugh class
C patients.
The superiority of TIPS in the treatment of refractory ascites
is still controversial and paracentesis remains the first option in
treating refractory ascites.27 When the frequency of paracentesis is
greater than three times per month TIPS insertion should be considered, but this decision depends on practical and patient issues
and the informed discussion about the risks of encephalopathy
and consent with the patient.28
The major complications of TIPS include (1) stenosis which
can occur in up to 70% of patients with bare stents within the
fi rst year.29 The advent of polytetrafluoroethylene (PTFE) covered stents has significantly reduced the incidence of TIPS
stenosis. 30-32 (2) Dislocation of the stent or its migration to the
right heart or lungs. (3) Onset of or worsening hepatic encephalopathy which can occur 20% to 30% of the time after TIPS
insertion. In a meta-analysis of RCTs comparing TIPS with
large-volume paracentesis as a treatment for refractory ascites,
the patients who received TIPS had an odds ratio of 2.26 for the
development of hepatic encephalopathy after TIPS. 25 In those
patients whose encephalopathy is refractory to medical management, occlusion or reduction in shunt size is mandatory. 33
(4) TIPS can induce intravascular hemolysis, occurring in 10%
of patients (especially those with bare stents). (5) Development of
cardiac failure (2.5%), renal failure (4.3%), and liver failure (1.9%).
(6) Bacterial infection of the stent itself or endotipsitis which
often cannot be treated successfully. 34

Answer: There are Level 1a and 1b evidence that supports TIPS


in the control of intractable ascites in carefully selected patients.
Potential complications of TIPS need to be considered when being
used in patients. (Grade A recommendation).
4. How does TIPS compare with peritoneovenous shunts in the
treatment of medically intractable ascites?
Rosemurgy et al.35 conducted a prospective randomized trial comparing TIPS to peritoneovenous shunts in the treatment of medically
intractable ascites. In this study 32 patients were prospectively randomized to undergo TIPS or peritoneovenous (Denver) shunts and
all had failed medical therapy. After TIPS versus peritoneovenous
shunts, median (mean SD) duration of shunt patency was similar:
4.4 months (6 6.6 months) versus 4 months (5 4.6 months).
Assisted shunt patency was longer after TIPS: 31.1 months (41 25.9
months) versus 13.1 months (19 17.3 months) (p < .01, Wilcoxon
test). After TIPS 19% of patients had irreversible shunt occlusion
versus 38% of patients after peritoneovenous shunts. Survival after
TIPS was 28.7 months (41 28.7 months) versus 16.1 months
(28 29.7 months) after peritoneovenous shunts. Control of ascites
was achieved sooner after peritoneovenous shunts than after
TIPS (73% vs. 46% after 1 month), but longer-term efficacy favored
TIPS (85% vs. 40% at 3 years). They concluded that peritoneovenous
shunts control ascites sooner, but TIPS provides better long-term
efficacy and this study promoted the use of TIPS for medically
intractable ascites if patients undergoing TIPS have prospects
beyond short-term survival.
Answer: There is Level 1b evidence that proves that TIPS provides better long-term efficacy in the management of medically
intractable ascites compared with peritoneovenous shunts. (Grade A
recommendation).
5. When should liver transplantation be considered in patients
with intractable ascites?
Ascites is usually an indicator of advanced cirrhosis and we know
that patients with ascites refractory to medical management have
a particularly poor prognosis. Liver transplantation is not a treatment for refractory ascites per se, but rather needs to be considered
for all patients who present with end-stage hepatic failure whether
or not it is accompanied by refractory ascites. Liver transplantation in patients who have developed refractory ascites secondary
to cirrhosis is the only therapy that addresses the underlying liver
disease in addition to improving the ascites.27
Answer: There is no data that supports liver transplantation
solely for the management of intractable ascites. No recommendations can be made based on the evidence.

Clinical Question Summary


Question

Answer

Levels of
Evidence

Grade of
Recommendation

1 What is the medical management


of ascites?

2 What is the role of paracentesis


in the management of intractable
ascites?

References

Spironolactone is the diuretic of


choice in the management of
patients with liver cirrhosis
and ascites.

1b

11

Paracentesis superior to diuretics


in eliminating ascites and
shortening hospital stay in patients
with ascites.

1b

12-17

(Continued)

PMPH_CH43.indd 354

5/22/2012 5:30:22 PM

Management of Intractable Ascites: The Evidence

355

(Continued)
Question

Answer

Levels of
Evidence

Grade of
Recommendation

3 What is the role of transjugular


intrahepatic portosystemic
shunt (TIPS) in the management
of intractable ascites?

TIPS can control intractable ascites in


carefully selected patients.

1b

20-24

4 How does TIPS compare with


peritoneovenous shunts in
the treatment of medically
intractable ascites?

TIPS provides better long-term


efficacy in the management
of medically intractable ascites
compared to peritoneovenous
shunts.

1b

35

5 When should liver transplantation


be considered in patients with
intractable ascites?

There is no data that supports liver


transplantation solely for the
management of intractable ascites.

REFERENCES
1. Reynold TB. Ascites. Clin Liver Dis. 2000;4:151-168.
2. Vadeyer HJ, Doran JD, Charnley R, Ryder SD. Saphenoperitoneal shunts for patients with intractable ascites associated with
chronic liver disease. Brit J Surg. 1999;86:882-885.
3. Elcheroth J, Vons C, Franco D. Role of surgical therapy in management of intractable ascites. World J Surg. 1994;18:240-245.
4. Cappone RR, Buhac I, Kohberger RC, et al. Resistant ascites in alcoholic liver cirrhosis: Course and prognosis. Dig Dis Sci. 1987;23:
867-871.
5. Guardiola J, Xiol X, Escribia JM, et al. Prognosis assessment of
cirrhotic patients with refractory ascites treated with a peritoneovenous shunt. Am J Gastroenterol. 1995;90:2097-2102.
6. Arroyo V, Gines P, Gerbes AL, et al. Definition and diagnostic criteria of refractory ascites and hepatorenal syndrome in cirrhosis.
International ascites club. Hepatology. 1996;23:164-176.
7. Ring-Larsen H, Henrikcen JH, Wilken C, Clausen J, Pals H,
Christensen NJ. Diuretic treatment in decompensated cirrhosis
and congestive heart failure: Effect of posture. Br Med J. 1986;292:
1351-1353.
8. Bernardi M, Laffi G, Salvagnini M, et al. Efficacy and treatment
of the stepped care medical treatment of ascites in liver cirrhosis: A randomized control clinical trial comparing two diets with
different sodium content. Liver. 1993;13:156-162.
9. Gauthier A, Levy VG, Quinton A, et al. Salt or no salt in the treatment of cirrhotic ascites: A randomized study. Gut. 1986;27:705-709.
10. Fogel MR, Saughney VK, Neal EA, Miller RG, Knauer CM, Gregory PB. Diuresis in the ascites patient: A randomized control trial
of three regimens. J Clin Gastroenterol. 1981;3(Suppl 1):73-80.
11. Perez-Ayuso RM, Arroyo V, Planas R, et al. Randomized comparative study of efficacy of furosemide versus spironolactone in
patients with liver cirrhosis and ascites. Gastroenterology. 1983;84:
961-968.
12. Gines P, Arroyo V, Quintero E, et al. Comparison of paracentesis and diuretics in the treatment of cirrhotics with tense ascites.
Results of a randomized study. Gastroenterology. 1987;93:234-341.
13. Sola R, Vila MC, Anreu M, et al. Total paracentesis with Dextran
40 vs diuretics in the treatment of ascites in cirrhosis: A randomized control study. J Hepatol. 1994;20:282-288.
14. Cotrim HP, Garrido V, Parana R, et al. Paracentesis associated
to dextran-70 in the treatment of ascites in patients with chronic
liver disease: A randomized therapeutic study. Arq Gastroenterol.
1994;31:125-129.

PMPH_CH43.indd 355

References

15. Hagengue H, Ink O, Ducreux M, Pelletier G, Buffet C, Etienne


JP. Treatment of ascites in patients with liver cirrhosis without
neither hyponatremia nor renal insufficiency. Results of a randomized study comparing diuretics and punctures compensated
by albumin. Gastroenterol Clin Biol. 1992;16:751-755.
16. Gines A, Fernandez-Esperrach G, Monescillo A, et al. Randomized trial comparing albumin, dextran 70, polygeline in cirrhotic
patients with ascites treated by paracentesis. Gastroenterology.
1996;111:1002-1010.
17. Tito L, Gines P, Arroyo V, et al. Total paracentesis associated
with intravenous albumin management of patients with cirrhosis and ascites. Gastroenterology. 1999;98:146-151.
18. Pozzi M, Osculati G, Boari G, et al. Time course of circulatory
and humoral effects of rapid total paracentesis in cirrhotic patients with tense, refractory ascites. Gastroenterology. 1994;106:
709-719.
19. Gines P, Tito L, Arroyo V, et al. Randomized comparative study
of therapeutic paracentesis with and without intravenous albumin in cirrhosis. Gastroenterology. 1998;94:1493-1502.
20. Lebrec D, Givily N, Hadengue A, et al. Transjugular intrahepatic
portosystemic shunts: Comparison with paracentesis in patients
with cirrhosis and refractory ascites: Randomized trial. J Hepatol.
1996;25:135-144.
21. Rossle M, Ochs A, Gulberg V, et al. A comparison of paracentesis
and transjugular intrahepatic portosystemic shunting in patients
with ascites. N Eng J Med. 2000;342:1701-1707.
22. Gines P, Uriz J, Calahorra B, et al. Transjugular intrahepatic
portosystemic shunting versus paracentesis plus albumin for
refractory ascites in cirrhosis. Gastroenterology. 2002;123:
1839-1847.
23. Sanyal AJ, Gening C, Redy KR, et al. The North American study
for the treatment of ascites. Gastroenterology. 2003;124:634-641.
24. Salerno F, Merli M, Roggio O, et al. Randomized control study
of TIPS versus paracentesis plus albumin in cirrhosis with severe
ascites. Hepatology. 2004;40:629-635.
25. DAmico G, Luca A, Morabito A, et al. Uncovered transjugular
intrahepatic portosystemic shunt for refractory ascites: a metaanalysis. Gastroenterology. 2005;129:1282-1293.
26. Salerno F, Camma C, Enea M, et al. Transjugular intrahepatic
portosystemic shunt for refractory ascites: A meta-analysis of individual patient data. Gastroenterology. 2007;133:825-834.
27. Salerno F, Guevara M, Bernardi M, et al. Refractory ascites:
pathogenesis, definition, and therapy of a severe complication in
patients with cirrhosis. Liv Int. 2010;30(7):937-947.

5/22/2012 5:30:22 PM

356

Surgery: Evidence-Based Practice

28. Moore KP, Wong F, Gines P, et al. The management of ascites in


cirrhosis: Report on the Consensus Conference of the International Ascites Club. Hepatology. 2003;38:258-266.
29. Sanyal AJ, Freedman AM, Luketic VA, et al. The natural history
of portal hypertension after transjugular intrahepatic portosystemic shunts. Gastroenterology. 1997;112:889-898.
30. Barris J, Ripoll C, Banares R, et al. Comparison of transjugular
intrahepatic portosystemic shunt dysfunction in PTFE-covered
stent grafts versus bare stents. Eur J Radiol. 2005;55:120-124.
31. Angermayer B, Cejna M, Koenig F, et al. Survival in patients
undergoing transjugular intrahepatic portosystemic shunt:
ePTFE-covered stent grafts versus bare stents. Hepatology.
2003;38:1043-1050.

PMPH_CH43.indd 356

32. Rossi P, Salvatori F, Fanelli F, et al. Polytetrafluoroethylene covered nitinol stent graft for transjugular intrahepatic portosystemic
shunt creation: 3 year experience. Radiology. 2004;231:820-823.
33. Kerlan RK, Jr., LaBerge JM, Baker EL, et al. Successful reversal of
hepatic encephalopathy with intentional occlusion of transjugular intrahepatic portosystemic shunts. J Vasc Interv Radiol. 1995;
6:917-921.
34. Mizrahi M, Adar T, Shouval D, et al. Endoptipsitis-persistent infection of transjugular intrahepatic portosystemic shunt: Pathogenesis, clinical features and management. Liv Int. 2010;30:175-183.
35. Rosemurgy AS, Zervos EE, Clark WC, et al. TIPS versus peritoneovenous shunt in the treatment of medically intractable ascites.
A prospective randomized trial. Ann Surg. 2004;239(6):883-891.

5/22/2012 5:30:22 PM

CHAPTER 44

Hepatic Encephalopathy
Terence OKeeffe and Tun Jie

INTRODUCTION

Hyponatremia is of particular concern as it is both an independent risk factor for the development of encephalopathy, and is
also predictive of poor prognosis in patients whose levels are below
135 mmol/L.2,3 Additionally, it has been suggested that patients with
cirrhosis who are malnourished or have diabetes mellitus appear to
have a higher frequency of developing hepatic encephalopathy.4 A
larger study did not provide evidence that nutritional status was an
independent risk factors for the presence of encephalopathy, thus
this risk factor remains unclear.5
In a study from 2007, 257 patients were assessed for risk factors for the onset of severe encephalopathy. Elevated ammonia
levels, serum bilirubin, lactate level, a prolonged International
Normalized Ratio (INR), and the Model for End-Stage Liver Disease (MELD) score were all shown to be predictive factors on univariate analysis, and on multivariate logistic regression analysis
elevated ammonia levels, and the MELD score remained independent predictors of the development of encephalopathy.6
Also, in a report from 2010, it was suggested that there might
be genetic variations in the clinical expression of encephalopathy, which appears to be related to variations in the promoter
region of the glutaminase gene.7 This may be a promising area of
future study.
Summary: There are specific precipitating factors that are
associated with the development of hepatic encephalopathy, and
these need to be taken into account and treated as part of the
management of this disease.
Answer: Identification of precipitating factors can help identify
those patients at risk to develop encephalopathy. (Level 2b evidence;
Grade B recommendation).

Hepatic encephalopathy continues to be a difficult problem for


many patients with liver disease, but rigorous scientific evidence
for a clear therapeutic approach is sorely lacking. Even the classification and grading of this condition are exceptionally difficult
to define, with no single test or examination having been shown to
be sufficiently precise or accurate in the diagnosis and/or assessment of the severity of encephalopathy. The purpose of this chapter is to present what evidence exists in the context of a number of
relevant clinical questions.
1. Who is at risk of developing hepatic encephalopathy?
Generally the more advanced the degree of liver disease the more
likely that hepatic encephalopathy will develop. However, precipitating factors are critically important in not only the diagnosis, but
also the management of hepatic encephalopathy. It would be fair
to say that most episodes of encephalopathy are associated with a
precipitant. There are a multitude of possible precipitants, which
include the following:1

Drugs: sedatives, analgesics, tranquilizers


Infectious: spontaneous bacterial peritonitis, UTI, pneumonia
Nutritional: constipation, excessive dietary protein
Electrolyte abnormalities: hypokalemia, hyponatremia
Acid-base disorders: acidosis, alkalosis
Renal: dehydration, renal failure
Hematological: gastrointestinal bleeding
Metabolic: hypoglycemia
Surgery
Transjugular intrahepatic portosystemic shunt (TIPS)
Superimposed liver injury: hepatitis, toxin-induced liver injury
Hepatocellular carcinoma

2. Are there different types of hepatic encephalopathy?


Hepatic encephalopathy has been classified clinically into three
major categories according to the underlying hepatic condition.8
Type A occurs in patients with acute liver failure. Type B occurs in
patients with encephalopathy caused by portosystemic shunting,
but without intrinsic liver disease. Type C is encephalopathy of

Patients with portal vein thrombosis and extensive portosystemic shunting without significant parenchymal liver disease can
also occasionally develop encephalopathy.

357

PMPH_CH44.indd 357

5/22/2012 5:30:56 PM

358

Surgery: Evidence-Based Practice

cirrhosis associated with portosystemic shunting, and this is also


the most common form. It may be episodic or persistent. Episodic
encephalopathy is often caused by precipitating factors (such as
those addressed above), which should be treated by addressing
these underlying causative factors. However, it may also occur
without a recognizing precipitating factor, when is known as
spontaneous episodic hepatic encephalopathy. When the encephalopathy lasts beyond 4 weeks or is recurrent it is considered persistent. Both episodic and persistent forms of encephalopathy may
be adequately suppressed by therapy, but will often reappear after
discontinuation of medications.
Minimal or subclinical hepatic encephalopathy represents
a particular problem as it has been used in the past to describe
patients who have no recognizable clinical symptoms of brain
dysfunction. Diagnosis was made on the basis of abnormal psychometric tests in the face of normal neuropsychiatric examination. It is difficult, therefore, to ascertain whether this is a separate
clinical entity or whether it is merely an early manifestation of the
disease process.
Unfortunately, there have been no attempts to assess the scientific validity of the above classification, but it is impossible to
discuss this aspect of hepatic encephalopathy in isolation, without considering how to grade the severity of the encephalopathy,
which we deal with in the following section.
Summary: No clear scientific data exists to assess the validity
of the current classification of hepatic encephalopathy.
Answer: The classification above, which is based on expert
opinion alone, is recommended as it is endorsed by the majority of the national and international hepatology societies. (Level 5
Evidence; Grade D recommendation).
3. Can you grade the severity of hepatic encephalopathy?
One of the major barriers to timely diagnosis and treatment of
hepatic encephalopathy is the lack of well-validated and standardized assessment tools. The first grading system to assess the severity of hepatic encephalopathy was developed in the 1970s and has
become known as the West Haven system of classification. This
uses a five-point grading scale starting at grade 0 and ending at
grade 4.9,10 This grading system is based purely on the physicians
clinical judgment by assessment of the patients mental status, and
takes approximately 15 min to complete. One of the drawbacks
of this system is the lack of objective measures regarding level of
consciousness, as it is based on changes of intellectual functioning
and behavior. The Glasgow Coma Scale (GCS), for example, does
not form part of this scoring system. Neither has this grading tool
being subjected to a large well-validated trial.
Other authors have proposed different scoring systems such
as the hepatic encephalopathy scoring algorithm (HESA). In their
paper from 2008, Hassanein et al.11 noted a moderately strong
association between the two grading systems (r = 0.60), and they
felt that their results suggested that HESA could be more sensitive to mental status impairment in the middle grades of hepatic
encephalopathy than the West Haven criteria.
This algorithm was prospectively tested in 70 adult patients
with cirrhosis, suffering from hepatic encephalopathy grades III
and IV, who were randomized to standard medical therapy or standard therapy with albumin dialysis. A total of 597 evaluations were
performed in these patients over 5 days, and the investigators noted
that there was significant separation between the grades using the
parameters that they had adopted. They also noted that the GCS was

PMPH_CH44.indd 358

less precise in defining minimal hepatic encephalopathy (i.e., grades


to III).12 They concluded that the HESA was a useful extension of
the West Haven criteria, which merited further validation.
Other tools that have been used to assess hepatic encephalopathy include the portal-systemic encephalopathy (PSE) index,
which was originally described in 1977 and combines the patients
mental state with the arterial ammonia levels, degree of asterixis,
electroencephalogram (EEG) findings, and the results of the
number connection test.10 However, in acute encephalopathy, the
PSE index has not been determined to be superior to other clinical grading systems and is associated with high rates of error.13
Psychometric tests can also be performed, but require adjustment
for demographic and cultural confounding variables. EEG and/or
evoked responses are nonspecific and are incapable of providing a
specific diagnosis of hepatic encephalopathy.14 Neuropsychological testing, likewise, does not provide significant specificity for the
diagnosis.15 Current recommendations for episodic or persistent
hepatic encephalopathy are that a clinical grading system should
be used for quantification. GCS may also provide additional useful information in stages III and IV.8
There are also well-known associations between encephalopathy and abnormalities seen via neuroimaging techniques such as
magnetic resonance imaging (MRI), computed tomography (CT),
or positron emission tomography. For example, cirrhotic patients
often demonstrate high signal intensity on T1-weighted MRI
images in the basal ganglia.16 This is thought to be related to deposition of manganese as the causative factor.17 Although these MRI
abnormalities correlate with EEG findings, they may not correlate
with clinical signs or symptoms and so may not be as useful to
assist in grading of severity of encephalopathy.
The bispectral index (BIS) has also been used to help differentiate severity. Dahaba et al.18 reported in a study in 2008 that they
were able to achieve an 87% agreement across grades 1 to 4 of hepatic
encephalopathy (West Haven criteria) using specific BIS value cutoffs
for the various grades. Not surprisingly, the discriminative power of
the BIS index was higher for high-grade (grades 3 and 4) than for
low-grade (grades 1 and 2) hepatic encephalopathy. Mean BIS values
correlated well with West Haven grades (r 2 = 0.90).
Another study looked at critical flicker frequency in diagnosing hepatic low-grade encephalopathy. The involved subjects
trying to discriminate the threshold frequencies at which life was
perceived as fused or flickering light. They found using a CFF cutoff value of 39 Hz, a 100% separation of patients with manifest
hepatic encephalopathy from noncirrhotic controls was achieved.
Statistically significant correlations were found between CFF and
psychometric testing. They concluded that CFF was a sensitive,
simple and reliable parameter for quantification of low-grade
headache encephalopathy severity in cirrhotic patients, and it may
be useful in the detection and monitoring of subclinical hepatic
encephalopathy.19 One of the advantages of this study was the relatively large patient population with cirrhosis92 patients, which
is larger than most of the other reports mentioned in this section.
However, like BIS testing, it does not lend itself to simple bedside
examination, as it requires sophisticated machinery, a dedicated
examiner, and 90 min to complete the entire test.
Summary: The West Haven criteria is the most common
grading system used to classify hepatic encephalopathy, although
the discriminatory power is not especially high, and the criteria
were developed over 30 years ago. Newer scoring systems and/or
objective tests hold promise, but await validation in large, welldesigned clinical trials.

5/22/2012 5:30:57 PM

Hepatic Encephalopathy

Answer: Grades III and IV of hepatic encephalopathy can


probably be accurately classified, by a number of the methods
listed above, whereas grades I and II are less accurately defined.
(Level 2b evidence; Grade B recommendation).
4. How can you differentiate hepatic encephalopathy from other
causes of confusion?
The confusion that is seen with hepatic encephalopathy may
clinically be similar to many other disease states and a thorough history and physical, together with a comprehensive evaluation is required to make sure that other diagnoses can be reliably
excluded.1
The differential diagnosis may include the following:
1. Intracranial pathology: hemorrhage, stroke, mass-occupying
lesion
2. Central nervous system infection: meningitis, encephalitis,
abscess
3. Metabolic endocrinopathies: ketoacidosis, uremia, thyroid
dysfunction
4. Alcohol withdrawal, delirium tremens or Wernickes
encephalopathy
5. Postictal following a seizure
Unfortunately, it is all too common for patients with liver
disease to have multiple causes of confusion, such as uremia associated with hepatorenal syndrome in addition to hepatic encephalopathy. It is also usually necessary to perform neuroimaging such
as a head CT or an MRI scan to exclude the presence of structural lesions as a cause of the abnormal mental state. However, as
detailed in the section above, these scans may not be specific for
the diagnosis, but may reveal important findings such as cerebral
edema or even uncial herniation.
The role of ammonia in diagnosing hepatic encephalopathy
remains controversial, with previously reported discrepancies
between blood levels and hepatic encephalopathy severity due in
part to problems with the chemical assays used for ammonia. If
properly processed (i.e., drawn in a nonheparinized container,
immediately placed on ice, and assayed within 30 min) there is
good correlation between blood ammonia levels and severity of
encephalopathy.20,21 In a study from 2003, Ong et al.22 found significant correlations between the arterial total ammonia level,
venous total ammonia level and venous partial pressures of
ammonia to the severity of hepatic encephalopathy. The West
Haven criteria were used to classify encephalopathy, and fasting blood samples were drawn immediately after testing, sent to
the lab on ice, and analyzed within 30 min. Although all four
measures of ammonia increased with the severity of hepatic
encephalopathy, the correlation coefficients were relatively weak
(r 2 = 0.520.61), and there was substantial overlap in the total
ammonia levels by grade of hepatic encephalopathy.22 In a study
from 2005, the correlation between plasma ammonia levels and
hepatic encephalopathy was only sufficiently discriminatory in
patients with acute liver failure and not in those with chronic liver
disease (r 2 = 0.91 vs. 0.30).23
Therefore it is felt that a normal blood ammonia level in a
patient with severe mental status changes is not supportive of the
diagnosis of hepatic encephalopathy. However, ammonia levels
may be moderately elevated in patients with cirrhosis without
encephalopathy, and an elevated serum ammonia level in a comatose patient is not sufficient alone to exclude coexistent conditions
that may contribute to a decreased level of consciousness.

PMPH_CH44.indd 359

359

A recent study looked at the discriminatory power of serum


nitric oxide in diagnosing hepatic encephalopathy, and Papadopoulos et al.24 showed that compared with serum ammonia,
nitric oxide was more sensitive than ammonia (96.8% vs. 87.9%),
using a cutoff of 75 g/dL for ammonia levels.24 However, it was
less specific overall than ammonia, and it performed better as an
exclusionary test. The combination of nitric oxide and ammonia
levels holds some promise to help in the diagnosis as well as in the
assessment of severity of encephalopathy.
Summary: An elevated blood ammonia level in the absence
of structural lesions on neuroimaging studies supports the diagnosis of hepatic encephalopathy.
Answer: Thorough diagnostic evaluation is required to exclude alternative diagnoses, including brain imaging. Ammonia
levels can help to discriminate hepatic encephalopathy, (Level 2b
evidence; Grade B recommendation).
5. What is the underlying cause of hepatic encephalopathy?
Hepatic encephalopathy represents the clinical manifestation of
functional disturbances in cells involved in neurotransmission,
and there are a number of competing hypotheses regarding these
pathways and the cells involved.
One theory proposes that encephalopathy is due to a disorder of astrocyte function, secondary to edema within these
cells interfering with cerebral neurotransmission. Th is theory is
attractive due to the associated cerebral edema and raised intracranial pressure that are often seen in these patients. However,
it is unclear if the astrocyte edema is related to the neurotoxicity of other substances, which may precipitate encephalopathy
(such as hyponatremia, inflammatory cytokines, and benzodiazepines) or whether it is the astrocyte itself that is the cause of the
problem.25
Not surprisingly, ammonia has also been suggested as the
principal neurotoxin causing encephalopathy. It has multiple neurotoxic effects, such as impairing amino acid metabolism, inhibiting the generation of postsynaptic potentials, and altering the
transit of amino acids, water and electrolytes in astrocytes and
neurons. Elevated ammonia levels may be directly neurotoxic,
and/or sensitize the astrocytes and neurons to injury by other
pathways and mediators.1 The clinical improvement that is often
seen with treatment that decreases serum ammonia levels also
lends credence to this theory. Unfortunately for this hypothesis
up to 10% of patients with significant encephalopathy have normal serum ammonia levels, as well as patients with chronic liver
disease having hyperammonemia without encephalopathy.
Seminal work in the 1980s suggested that -aminobutyric
acid (GABA), which is a neuroinhibitory substance produced in
the gastrointestinal tract, was responsible for encephalopathy due
to increased GABAergic tone in the brain.26 Proposed mechanisms
have included increased brain GABA content, altered integrity
of the receptor complex or the presence of increased amounts of
substances with modulatory activity at the receptor site.27 Experimental models have shown that neurotoxins can increase the
production of receptors, which can then stimulate the conversion
of cholesterol to neurosteroids.28 Neurosteroids are subsequently
released from the astrocyte that are capable of binding to their
receptor within the neuronal GABA receptor complex and can
increase inhibitory neurotransmission. This possible role for neurosteroids with GABA receptor modulatory properties might offer
a plausible alternative explanation of the increased GABAergic
tone in HE.29

5/22/2012 5:30:57 PM

360

Surgery: Evidence-Based Practice

It is however clear that at this time, that there is no single unifying theory to explain the development of encephalopathy from
the current available theories. Due to space constraints we cannot deal exhaustively with all the theories in this text, but a more
extensive review is available in Zakim and Boyers Hepatology.30
More research will be required to fully elucidate the underlying
mechanisms causing encephalopathy.
Summary: No single theory exists that completely explains
the pathogenesis of hepatic encephalopathy.
Answer: Neurotoxins likely play a role, but it is likely the cause
is multifactorial. (Level 4 evidence; Grade C recommendation).
6. Which therapies are effective for hepatic encephalopathy?
Despite decades of research, there is still lack of a clear understanding of the pathogenesis of hepatic encephalopathy. Current treatments for hepatic encephalopathy are largely based on
clinical experience, and some are extrapolated from experimental animal models. Only recently have data started to accumulate from evidence-based clinical trials in the treatment of hepatic
encephalopathy.
The management of patients with hepatic encephalopathy
depends on the precipitating factors, the severity, and the etiology
of the liver failure. In patients with chronic hepatic dysfunction,
encephalopathy often occurs episodically, caused by precipitating
factors previously discussed.31 In patients with acute liver failure,
hepatic encephalopathy is often a late finding, and is associated
with cerebral edema and extensive hepatocellular damage. Drugrelated hepatotoxicity, such as acetaminophen toxicity and other
idiosyncratic drug reactions, accounts for more than 50% of the
acute liver failure cases in the United States. Therefore, the first step
in management of patients with hepatic encephalopathy is to identify the cause in a timely fashion. Treatment of the precipitating
factors and administration of pharmacologic agents can often
improve the symptoms of encephalopathy.

LACTULOSE
Lactulose is a synthetic disaccharide (galacto-fructose), which
passes through small bowel unmodified because of the lack of specific disaccharidase in human enterocytes. It is metabolized by the
colonic bacteria enzymes into gas and acids. Ammonia is trapped
in the acidic colonic environment by forming nonabsorbable NH4+.
The osmotic diarrhea induced by lactulose further increases the
fecal nitrogen clearance32 and reduces colonic bacterial load. Elkington et al.33 first demonstrated the therapeutic value of the lactulose in patients with chronic hepatic encephalopathy in a small
double-blind clinical trial in early the 1970s. Since then, lactulose
has become a mainstay treatment for patient with hepatic encephalopathy. However, there is limited data to show the efficacy of
lactulose from well-designed, randomized, controlled clinical trials. A Cochrane review of the beneficial effects of nonabsorbable
disaccharides did not show statistical significant in improvement
of encephalopathy or mortality.34 The study was based on data of
10 trials with high methodological quality; however, most of the
early trials were underpowered because of the relative small sample
size. More recently it was shown that lactulose help improve
cognitive functions and quality of life in patients with minimal
hepatic encephalopathy,35,36 and to prevent encephalopathy after
acute variceal bleed.37

PMPH_CH44.indd 360

The optimal dose of lactulose has yet to be verified in clinic


trials. Patients are often instructed to titrate the dose of lactulose
to achieve 2 to 4 loose stools daily. Lactulose can be administrated
as a retention enema in comatose patients. The utility of lactulose
in patients with encephalopathy associated with acute fulminant
liver failure is uncertain.

ANTIBIOTICS
The rationale of using antibiotics in patients with hepatic encephalopathy is to eliminate the colonic bacteria, in particular, the
urease-producing bacteria, to reduce the production of ammonia. Neomycin was first proposed as the long-term treatment for
hepatic encephalopathy based on a small clinical trial with 20
patients.38 A double-blinded, randomized, placebo-controlled trial
compared neomycin and placebo in the treatment of patients with
hepatic encephalopathy (n = 39) and failed to demonstrate any
improvement.39 Blanc et al.40 also failed to show the efficacy of the
combination of lactulose-neomycin versus placebo in a larger trial
(n = 80). Recently, neomycin has fallen out of favor because of the
ototoxicity and nephrotoxicity risks associated with long-term
aminoglycoside use.
Rifaximin, a nonabsorbable derivative of rifampin, received
an orphan drug status as a treatment for hepatic encephalopathy
in 2005. In a Phase 3, multicenter, randomized, double-blind,
placebo-controlled trial with 299 patients in remission for recurrent hepatic encephalopathy, Bass et al.41 showed treatment of a
dose of 550 mg rifaximin twice daily was more effective than placebo to maintain remission. In this study, patients in both treatment and control groups also received lactulose. The episodes of
recurrent encephalopathy causing hospitalization were 22% in
the treatment group and 46% in the placebo group (p < .01). In
another recent randomized trial of patients with minimal hepatic
encephalopathy, Bajaj et al.42 reported improvements seen in driving simulator performance and cognitive performance, but not in
ammonia levels or MELD scores.

DIETARY MODIFICATION
To reduce the intestinal ammonia production, low-protein diets
have been routinely advocated as part of the management of hepatic
encephalopathy in the past. This practice was not supported by
strong evidence. Moreover, protein intake restriction may lead to
worsening nutritional status, which has been recognized as a serious problem as encephalopathy. A randomized study conducted
in cirrhotic patients admitted for an episode of encephalopathy
showed no benefit from receiving a low-protein diet. Those patients
on the low-protein diet had evidence for exacerbating protein
breakdown compared with those who received normal diet.43
Summary: The principle of management of hepatic encephalopathy is to correct the precipitating factors and to reduce the
ammonia level. Lactulose and rifaximin should be considered
as the first-line treatment for hepatic encephalopathy. Lactulose
can be administrated orally or as retention enema. The dose of
lactulose should be titrated to two to four loose stools daily, while
rifaximin can be taken orally 550 mg twice daily.
Answer: Lactulose and rifaximin are effective treatments for
hepatic encephalopathy. Low-protein diets have no beneficial

5/22/2012 5:30:57 PM

Hepatic Encephalopathy

361

effects. (Lactulose: Level 2b evidence, rifaximin: Level 1b evidence;


Grade B recommendation).

MELD = 3.78 [Ln serum bilirubin [mg/dL]) + 11.2 (Ln INR) +


9.57 (Ln serum creatinine [mg/dL]) + 6.43

7. What predicts the development of hepatic encephalopathy


following TIPS?

Because listing for transplantation now almost exclusively uses


the MELD score, no priority is given to patients with severe recurrent or resistant hepatic encephalopathy. However, the CTP has
also been shown to correlate with survival, and this scoring system does include encephalopathy.56 This score is calculated by
assigning points to different levels of total serum bilirubin, serum
albumin, INR, ascites, and the presence of hepatic encephalopathy giving a severity of liver disease. It has largely been superseded
by the MELD score at this time.
Failure to demonstrate that encephalopathy independently
predicts survival could be related to the close association that it
has with liver function status. Additionally, difficulties with accurately staging hepatic encephalopathy may have influenced older
data regarding the failure of encephalopathy to predict survival in
advanced liver disease.
More recent work, such as a study performed by Stewart
et al.57 in 2007 suggests that the presence of hepatic encephalopathy is an independent risk factor for mortality in the two patient
populations that they studied; post-TIPS procedure and those hospitalized for hepatic decompensation. This study also confirmed
that the MELD score and the CTP score were markedly associated
with survival. Also, a systematic review of factors associated with
survival from 2006 showed that encephalopathy was predictive
of mortality in 7 out of 31 studies that were assessed to have good
methodology.58 It may be that further refinements to the grading
system for encephalopathy will enable us to make accurate predictions regarding survival in these patients.
It is interesting to note that there is a subpopulation of patients
who have severe hepatic encephalopathy and poor survival but
have a moderately low MELD score.59 A more detailed analysis of
the effect of the subtypes of hepatic encephalopathy (severe, disabling, chronic, or recurrent versus precipitated, self-limited, and
episodic) is probably necessary to add more predictive power to
the MELD score for this subgroup of patients.1
Summary: There is evidence that hepatic encephalopathy
increases mortality and can improve the accuracy of the MELD
score. More work remains to be done to establish whether there
are specific subtypes of encephalopathy that affect mortality more
than others.
Answer: Mortality is likely increased by the presence of hepatic
encephalopathy. (Level 3 evidence; Grade C recommendation).

TIPS is an effective treatment modality in controlling variceal


bleeding, improving refractory ascites, and lately, treatment of
patients with Budd-Chiari syndrome.44-46 However, from 20%
to 77% of the patients undergoing TIPS can develop hepatic
encephalopathy, with the rate of development of encephalopathy
dependent on the patients residual liver function.47-49 The severity of encephalopathy following TIPS is variable and ranges from
encephalopathy that is easily controllable with antibiotics or
disaccharides to chronic encephalopathy, which can be refractory
to conventional treatment.
In a retrospective review of 136 patients who underwent TIPS
placement, followed up at 3 to 6 months postplacement, 47 patients
(34.5%) developed new or worsening encephalopathy. Of these, 69%
developed new encephalopathy and 31% had worsening of preexisting encephalopathy. Analysis showed that preexisting encephalopathy was the only significantly predictive variable in determining
the subsequent development of encephalopathy following TIPS
insertion.50
A recent meta-analysis showed that patients who underwent
TIPS despite much better in control of ascites, these patients, had
significantly more episodes of encephalopathy and more severe
episodes.51 Older age and lower mean arterial pressure (MAP)
were found to be independent risk factors for the development of
encephalopathy following TIPS. Other studies have shown that in
addition to age, patients who had an episode of hepatic encephalopathy prior to a TIPS procedure were more likely to develop
encephalopathy following placement of the shunt.52,53 The ChildTurcotte-Pugh (CTP) score has also been shown to be associated
with risk of encephalopathy post-TIPS placement.
The new generation of polytetrafluoroethylene-covered (PTFE)
stents has been shown to significantly reduce the risks of stent
dysfunction, but it has not shown to significantly alter the risk of
developing encephalopathy post-TIPS placement. In an observational study of 78 patients who received PTFE-covered stent, 44.8%
developed a episodic hepatic encephalopathy during the postprocedure follow-up. Only 8% of the patients developed encephalopathy refractory to conventional treatments and required reduction
of the stent diameter. The risk factors that were found associated
with post-TIPS encephalopathy included advanced age, high creatinine levels, and low serum sodium and albumin levels.54
Summary: There incidence of episodic hepatic encephalopathy after TIPS, is high but only a small percentage of the encephalopathy is refractory to medical management. The complete list
of risk factors associated with post-TIPS hepatic encephalopathy
remain to be elucidated.
Answer: Certain predictive factors exist that can predict
hepatic encephalopathy post-TIPS placement. (Level 3a evidence;
Grade B recommendation).
8. Does hepatic encephalopathy increase mortality?
When considering hepatic encephalopathy and its effects on mortality, it is important to remember that encephalopathy is not
included in the calculation of the MELD score.55 The MELD score
is calculated using the following formula:

PMPH_CH44.indd 361

9. Will hepatic encephalopathy resolve following transplantation?


Transplantation is probably the single most effective treatment for
hepatic encephalopathy. Most studies have demonstrated that the
encephalopathy is reversed by the transplantation, even in very
severe cases such as acquired hepatocerebral degeneration.60-63
It is nonetheless unfortunate that the presence of hepatic
encephalopathy pretransplant may well affect the cognitive
function in the same patient posttransplant. In a study from
2011, 52 consecutive patients with cirrhosis completed neuropsychological testing prior to transplantation, and then 6 and 12
months following the transplant. Following transplantation,
both global cognitive function and brain volume as assessed by
MRI scanning were poorer in those patients who had hepatic
encephalopathy prior to receiving their transplant. Th is is consistent with at least two other studies and shows that cognitive

5/22/2012 5:30:57 PM

362

Surgery: Evidence-Based Practice

defects can linger following transplantation.60,64 Th is may be


due to neurotoxic affects contributing to the encephalopathy, or
may be related to the underlying cause of the liver disease, for
example alcoholism. Long-term studies have shown significant
differences in MRI imaging lesions in liver transplant patients
compared with healthy controls, which may account for these
long-term differences.65
There have also been two case reports describing hepatic
encephalopathy in patients who had undergone liver transplantation. In the first report, two patients who had distal splenorenal shunts placed following transplantation to treat portal vein
thrombosis, and two other patients had spontaneous splenorenal
shunts, all developed hepatic encephalopathy despite their liver
transplant.66 The other report dealt with a single patient, whose
retroperitoneal varices were the cause of the portosystemic shunt-

ing.67 This demonstrates that portosystemic shunting in liver


transplant recipients, even with stable graft function, can be associated with hepatic encephalopathy. The authors all recommended
restoration of portal blood flow by either surgical or angiographic
means to treat the encephalopathy.
Summary: Hepatic encephalopathy is reversed by liver transplantation, usually completely in most patients, although increasing evidence suggests that continuing subtle cognitive defects may
remain. These may be related to the underlying cause of the disease, or poorly understood mechanisms of neurological damage.
Answer: Encephalopathy usually reverses following transplantation, although subtle cognitive deficits may remain. (Level 4
evidence; Grade C recommendation).
Level of evidence: 4
Strength of Recommendation: C

Clinical Question Summary


Question

Answer

Level of
Evidence

Grade of
Recommendation

References

1 Who is at risk of developing


hepatic encephalopathy?

Identification of precipitating factors can


help identify those patients at risk to
develop encephalopathy.

2b

1, 2, 3, 4, 5, 6

2 Are there different types of


hepatic encephalopathy?

Hepatic Encephalopathy is usually


classified into one of three forms.

3 Can you grade the severity of


hepatic encephalopathy?

Grades III and IV of hepatic


encephalopathy can probably be
accurately classified, but grades I and II
are less accurately defined.

2b

8, 9, 10, 11,
12, 13, 18,
19

4 How can you differentiate


hepatic encephalopathy from
other causes of confusion?

Thorough diagnostic evaluation is


required to exclude alternative
diagnoses. Ammonia levels help to
discriminate hepatic encephalopathy.

2b

20, 21, 22, 23,


24

5 What is the underlying cause of


hepatic encephalopathy?

Neurotoxins likely play a role in the


pathogenesis, but it is likely the cause
is multifactorial.

25, 26, 27, 28,


29, 30

6 Which therapies are effective


for hepatic encephalopathy?

Lactulose and rifaximin are effective


treatments for hepatic encephalopathy.

2b, 1b

33, 34, 35, 36,


37, 39, 40,
41, 42, 43

7 What predicts the development


of hepatic encephalopathy
following TIPS?

Specific predictive factors exist that can


predict hepatic encephalopathy postTIPS placement.

3a

47, 48, 49, 50,


51, 52, 53,
54

8 Does hepatic encephalopathy


increase mortality?

Mortality is likely increased by the


presence of hepatic encephalopathy.

57, 58, 59

9 Will hepatic encephalopathy


resolve following
transplantation?

Encephalopathy usually reverses following


transplantation, although subtle
cognitive deficits may remain.

60, 61, 62, 63,


64

REFERENCES
1. Munoz SJ. Hepatic encephalopathy. Med Clin North Am. 2008;
92:795-812, viii.
2. Guevara M, Baccaro ME, Torre A, et al. Hyponatremia is a risk
factor of hepatic encephalopathy in patients with cirrhosis: A
prospective study with time-dependent analysis. Am J Gastroenterol. 2009;104:1382-1389.

PMPH_CH44.indd 362

3. Jenq CC, Tsai MH, Tian YC, et al. Serum sodium predicts prognosis in critically ill cirrhotic patients. J Clin Gastroenterol. 2010;
44:220-226.
4. Kalaitzakis E, Olsson R, Henfridsson P, et al. Malnutrition and
diabetes mellitus are related to hepatic encephalopathy in patients
with liver cirrhosis. Liver Int. 2007;27:1194-1201.
5. Soros P, Bottcher J, Weissenborn K, Selberg O, Muller MJ.
Malnutrition and hypermetabolism are not risk factors for the

5/22/2012 5:30:57 PM

Hepatic Encephalopathy

6.

7.

8.

9.

10.

11.

12.

13.

14.

15.

16.

17.

18.

19.

20.

21.

22.

PMPH_CH44.indd 363

presence of hepatic encephalopathy: A cross-sectional study.


J Gastroenterol Hepatol. 2008;23:606-610.
Bernal W, Hall C, Karvellas CJ, Auzinger G, Sizer E, Wendon J.
Arterial ammonia and clinical risk factors for encephalopathy
and intracranial hypertension in acute liver failure. Hepatology.
2007;46:1844-1852.
Romero-Gomez M, Jover M, Del Campo JA, et al. Variations in the
promoter region of the glutaminase gene and the development
of hepatic encephalopathy in patients with cirrhosis: A cohort
study. Ann Intern Med. 2010;153:281-288.
Ferenci P, Lockwood A, Mullen K, Tarter R, Weissenborn K, Blei
AT. Hepatic encephalopathydefinition, nomenclature, diagnosis, and quantification: Final report of the working party at
the 11th World Congresses of Gastroenterology, Vienna, 1998.
Hepatology. 2002;35:716-721.
Atterbury CE, Maddrey WC, Conn HO. Neomycin-sorbitol and
lactulose in the treatment of acute portal-systemic encephalopathy. A controlled, double-blind clinical trial. Am J Dig Dis.
1978;23:398-406.
Conn HO, Leevy CM, Vlahcevic ZR, et al. Comparison of lactulose and neomycin in the treatment of chronic portal-systemic
encephalopathy. A double blind controlled trial. Gastroenterology. 1977;72:573-583.
Hassanein TI, Hilsabeck RC, Perry W. Introduction to the
Hepatic Encephalopathy Scoring Algorithm (HESA). Dig Dis Sci.
2008;53:529-538.
Hassanein T, Blei AT, Perry W, et al. Performance of the hepatic
encephalopathy scoring algorithm in a clinical trial of patients
with cirrhosis and severe hepatic encephalopathy. Am J Gastroenterol. 2009;104:1392-1400.
Kircheis G, Fleig WE, Gortelmeyer R, Grafe S, Haussinger D.
Assessment of low-grade hepatic encephalopathy: A critical
analysis. J Hepatol. 2007;47:642-650.
Guerit JM, Amantini A, Fischer C, et al. Neurophysiological
investigations of hepatic encephalopathy: ISHEN practice guidelines. Liver Int. 2009;29:789-796.
Saxena N, Bhatia M, Joshi YK, Garg PK, Dwivedi SN, Tandon
RK. Electrophysiological and neuropsychological tests for the
diagnosis of subclinical hepatic encephalopathy and prediction
of overt encephalopathy. Liver. 2002;22:190-197.
Solomou E, Terzis G, Tsota I, et al. Brain MRI findings in cirrhotic patients: correlation with Doppler US hepatic flow parameters. Hepatogastroenterology. 2005;52:420-424.
Solomou E, Velissaris D, Polychronopoulos P, et al. Quantitative
evaluation of magnetic resonance imaging (MRI) abnormalities
in subclinical hepatic encephalopathy. Hepatogastroenterology.
2005;52:203-207.
Dahaba AA, Worm HC, Zhu SM, et al. Sensitivity and specificity
of bispectral index for classification of overt hepatic encephalopathy: A multicentre, observer blinded, validation study. Gut.
2008;57:77-83.
Kircheis G, Wettstein M, Timmermann L, Schnitzler A, Haussinger D. Critical flicker frequency for quantification of low-grade
hepatic encephalopathy. Hepatology. 2002;35:357-366.
Nicolao F, Efrati C, Masini A, Merli M, Attili AF, Riggio O. Role
of determination of partial pressure of ammonia in cirrhotic
patients with and without hepatic encephalopathy. J Hepatol.
2003;38:441-446.
Kramer L, Tribl B, Gendo A, et al. Partial pressure of ammonia
versus ammonia in hepatic encephalopathy. Hepatology. 2000;31:
30-34.
Ong JP, Aggarwal A, Krieger D, et al. Correlation between
ammonia levels and the severity of hepatic encephalopathy. Am J
Med. 2003;114:188-193.

363

23. Kundra A, Jain A, Banga A, Bajaj G, Kar P. Evaluation of plasma


ammonia levels in patients with acute liver failure and chronic
liver disease and its correlation with the severity of hepatic encephalopathy and clinical features of raised intracranial tension.
Clin Biochem. 2005;38:696-699.
24. Papadopoulos N, Soultati A, Goritsas C, et al. Nitric oxide,
ammonia, and CRP levels in cirrhotic patients with hepatic
encephalopathy: is there a connection? J Clin Gastroenterol.
2010;44:713-719.
25. Haussinger D, Kircheis G, Fischer R, Schliess F, vom Dahl S.
Hepatic encephalopathy in chronic liver disease: A clinical manifestation of astrocyte swelling and low-grade cerebral edema?
J Hepatol. 2000;32:1035-1038.
26. Schafer DF, Jones EA. Hepatic encephalopathy and the gammaaminobutyric-acid neurotransmitter system. Lancet. 1982;1:18-20.
27. Ahboucha S, Butterworth RF. Pathophysiology of hepatic encephalopathy: A new look at GABA from the molecular standpoint.
Metab Brain Dis. 2004;19:331-343.
28. Butterworth RF. The astrocytic (peripheral-type) benzodiazepine receptor: Role in the pathogenesis of portal-systemic
encephalopathy. Neurochem Int. 2000;36:411-416.
29. Ahboucha S, Pomier-Layrargues G, Butterworth RF. Increased
brain concentrations of endogenous (non-benzodiazepine)
GABA-A receptor ligands in human hepatic encephalopathy.
Metab Brain Dis. 2004;19:241-251.
30. Mullen KD. Zakim and Boyers Hepatology: A Textbook of Liver
Disease. 5th ed. Philadelphia: Saunders-Elsevier; 2006.
31. Fessel JM, Conn HO. Lactulose in the treatment of acute hepatic
encephalopathy. Am J Med Sci. 1973;266:103-110.
32. Mortensen PB. The effect of oral-administered lactulose on
colonic nitrogen metabolism and excretion. Hepatology. 1992;16:
1350-1356.
33. Elkington SG, Floch MH, Conn HO. Lactulose in the treatment of
chronic portal-systemic encephalopathy. A double-blind clinical
trial. N Engl J Med. 1969;281:408-412.
34. Als-Nielsen B, Gluud LL, Gluud C. Nonabsorbable disaccharides
for hepatic encephalopathy. Cochrane Database Systematic Review.
2004:CD003044.
35. Prasad S, Dhiman RK, Duseja A, Chawla YK, Sharma A, Agarwal R.
Lactulose improves cognitive functions and health-related quality of life in patients with cirrhosis who have minimal hepatic
encephalopathy. Hepatology. 2007;45:549-559.
36. Sharma P, Sharma BC, Puri V, Sarin SK. An open-label randomized controlled trial of lactulose and probiotics in the treatment
of minimal hepatic encephalopathy. Eur J Gastroenterol Hepatol.
2008;20:506-511.
37. Sharma P, Agrawal A, Sharma BC, Sarin SK. Prophylaxis of
Hepatic encephalopathy in acute variceal bleed: A randomized
controlled trial of lactulose versus no lactulose. J Gastroenterol
Hepatol. 2011;6:996-1003.
38. Dawson AM, Mc LJ, Sherlock S. Neomycin in the treatment of
hepatic coma. Lancet. 1957;273:1262-1268.
39. Strauss E, Tramote R, Silva EP, et al. Double-blind randomized
clinical trial comparing neomycin and placebo in the treatment
of exogenous hepatic encephalopathy. Hepatogastroenterology.
1992;39:542-545.
40. Blanc P, Daures JP, Liautard J, et al. Lactulose-neomycin combination versus placebo in the treatment of acute hepatic encephalopathy. Results of a randomized controlled trial. Gastroenterol
Clin Biol. 1994;18:1063-1068.
41. Bass NM, Mullen KD, Sanyal A, et al. Rifaximin treatment in
hepatic encephalopathy. N Engl J Med. 2010;362:1071-1081.
42. Bajaj JS, Heuman DM, Wade JB, et al. Rifaximin improves driving simulator performance in a randomized trial of patients with

5/22/2012 5:30:57 PM

364

43.

44.

45.

46.

47.

48.
49.

50.

51.

52.

53.

54.

Surgery: Evidence-Based Practice

minimal hepatic encephalopathy. Gastroenterology. 2011;140:


478-487 e1.
Cordoba J, Lopez-Hellin J, Planas M, et al. Normal protein diet for
episodic hepatic encephalopathy: results of a randomized study.
J Hepatol. 2004;41:38-43.
Boyer TD, Haskal ZJ. The role of transjugular intrahepatic portosystemic shunt (TIPS) in the management of portal hypertension: Update 2009. Hepatology. 2010;51:306.
Garcia-Pagan JC, Heydtmann M, Raffa S, et al. TIPS for BuddChiari syndrome: Long-term results and prognostics factors in
124 patients. Gastroenterology. 2008;135:808-815.
Narahara Y, Kanazawa H, Fukuda T, et al. Transjugular intrahepatic portosystemic shunt versus paracentesis plus albumin in
patients with refractory ascites who have good hepatic and renal
function: A prospective randomized trial. J Gastroenterol. 2011;
46:78-85.
Boyer TD, Haskal ZJ. The role of transjugular intrahepatic portosystemic shunt in the management of portal hypertension.
Hepatology. 2005;41:386-400.
Runyon BA. Management of adult patients with ascites due to
cirrhosis. Hepatology. 2004;39:841-856.
Garcia-Tsao G, Sanyal AJ, Grace ND, Carey W. Prevention and
management of gastroesophageal varices and variceal hemorrhage in cirrhosis. Hepatology. 2007;46:922-938.
Masson S, Mardini HA, Rose JD, Record CO. Hepatic encephalopathy after transjugular intrahepatic portosystemic shunt
insertion: A decade of experience. QJM. 2008;101:493-501.
Salerno F, Camma C, Enea M, Rossle M, Wong F. Transjugular
intrahepatic portosystemic shunt for refractory ascites: A metaanalysis of individual patient data. Gastroenterology. 2007;133:
825-834.
Bai M, Qi X, Yang Z, et al. Predictors of hepatic encephalopathy
after transjugular intrahepatic portosystemic shunt in cirrhotic
patients: A systematic review. J Gastroenterol Hepatol. 2011;12:
1440-1746.
Riggio O, Masini A, Efrati C, et al. Pharmacological prophylaxis of hepatic encephalopathy after transjugular intrahepatic
portosystemic shunt: A randomized controlled study. J Hepatol.
2005;42:674-679.
Riggio O, Angeloni S, Salvatori FM, et al. Incidence, natural history, and risk factors of hepatic encephalopathy after transjugular
intrahepatic portosystemic shunt with polytetrafluoroethylenecovered stent grafts. Am J Gastroenterol. 2008;103:2738-2746.

PMPH_CH44.indd 364

55. Kamath PS, Wiesner RH, Malinchoc M, et al. A model to predict survival in patients with end-stage liver disease. Hepatology.
2001;33:464-470.
56. Pugh RN, Murray-Lyon IM, Dawson JL, Pietroni MC, Williams R.
Transection of the oesophagus for bleeding oesophageal varices.
Br J Surg. 1973;60:646-649.
57. Stewart CA, Malinchoc M, Kim WR, Kamath PS. Hepatic encephalopathy as a predictor of survival in patients with end-stage liver
disease. Liver Transpl. 2007;13:1366-1371.
58. DAmico G, Garcia-Tsao G, Pagliaro L. Natural history and prognostic indicators of survival in cirrhosis: A systematic review of
118 studies. J Hepatol. 2006;44:217-231.
59. Thornton JG, Mullen KD. The role of hepatic encephalopathy in
the era of MELD. Liver Transpl. 2007;13:1364-1365.
60. Mechtcheriakov S, Graziadei IW, Mattedi M, et al. Incomplete
improvement of visuo-motor deficits in patients with minimal
hepatic encephalopathy after liver transplantation. Liver Transpl.
2004;10:77-83.
61. Mattarozzi K, Stracciari A, Vignatelli L, DAlessandro R, Morelli
MC, Guarino M. Minimal hepatic encephalopathy: Longitudinal effects of liver transplantation. Arch Neurol. 2004;61:
242-247.
62. Atluri DK, Asgeri M, Mullen KD. Reversibility of hepatic encephalopathy after liver transplantation. Metab Brain Dis. 2010;25:
111-113.
63. Stracciari A, Guarino M, Pazzaglia P, Marchesini G, Pisi P.
Acquired hepatocerebral degeneration: full recovery after
liver transplantation. J Neurol Neurosurg Psychiatry. 2001;70:
136-137.
64. Sotil EU, Gottstein J, Ayala E, Randolph C, Blei AT. Impact of
preoperative overt hepatic encephalopathy on neurocognitive function after liver transplantation. Liver Transpl. 2009;15:
184-192.
65. Garcia-Martinez R, Rovira A, Alonso J, et al. Hepatic encephalopathy is associated with posttransplant cognitive function and
brain volume. Liver Transpl. 2011;17:38-46.
66. Braun MM, Bar-Nathan N, Shaharabani E, et al. Postshunt hepatic
encephalopathy in liver transplant recipients. Transplantation.
2009;87:734-739.
67. Herrero JI, Bilbao JI, Diaz ML, et al. Hepatic encephalopathy after
liver transplantation in a patient with a normally functioning
graft: Treatment with embolization of portosystemic collaterals.
Liver Transpl. 2009;15:111-114.

5/22/2012 5:30:57 PM

Commentary on Hepatic
Encephalopathy
Jamal J. Hoballah

In this chapter, Drs. OKeeffe and Jie from the University of Arizona have done a remarkable job examining the etiology, diagnosis,
and treatment of hepatic encephalopathy. The task is particularly
difficult as the etiology, diagnosis, and treatment of this condition are so poorly understood. The evidence that they present is
compelling and may lead one to a feeling of utter hopelessness as
there remains so much that needs to be understood. It seems that
we are left utilizing the West Haven criteria to grade severity of
disease and acute management, as adjuncts to the system remain
to be adequately validated.
For Grade I and II hepatic encephalopathy, most patients
require computed tomography to exclude other causes of altered
mental status. This may demonstrate signs of cerebral edema;
however, signs of intracranial hypertension (midline shift, ventricular compression, etc.) are generally not present. Treatment
of the condition remains largely supportive. Lactulose has been
utilized to decrease intestinal absorption of ammonia, but its utility in improving outcome and decreasing encephalopathy has yet
to be demonstrated. Similarly, evidence to advocate antibiotics
and dietary modification in this patient population does not exist.
Furthermore, sedatives dependent upon hepatic clearance, such
as benzodiazepines, should routinely be avoided. If sedation is
necessary, then haloperidol is generally the agent of choice.
For Grade III and IV hepatic encephalopathy, patients should
be intubated for airway protection and monitored closely in the
intensive care unit. Like other aspects in the management of this
disease, type of sedation and anesthesia utilized for intubation
has not been clearly delineated. Propofol, which clears rapidly
from the body by multiple mechanisms, is usually the sedative of
choice as it does not increase intracranial pressure (ICP). Utilizing ICP monitors in these patients remains an area of controversy.

Proponents advocate that ICP monitors will help to guide therapy,


whereas detractors argue that the risks of invasive procedures far
outweigh their benefit. Furthermore, little evidence exists that
demonstrates survival benefit with ICP monitors in patients with
hepatic encephalopathy. As with traumatic brain injury, target
goals of ICP of less than 20 and cerebral perfusion pressure (CPP)
of greater than 60 should be met.
Simple techniques to meet ICP and CPP goals include head of bed
elevation to greater than 30 degrees, minimizing agitation, optimizing oxygenation, and maintaining a partial pressure of carbon dioxide
between 35 and 40. Prolonged periods of hyperventilation may lead
to cerebral vasoconstriction and associated hypoxia. Analgesic support should be minimized. If necessary, dilaudid or fentanyl should
be utilized instead of morphine or meperidine as the latter have active
metabolites that can worsen hepatic and renal failure. Mannitol has
been shown to reduce ICP and mortality in this population, whereas
hypertonic saline and dexamethasone have not. Hypertonic saline
has been shown to decrease ICP, but no survival benefit could be
demonstrated. Fevers should be treated with cooling blankets and
other environmental techniques. Non-steroidal antiinflammatory
drugs and acetaminophen should be avoided as these have been
shown to worsen renal and hepatic failure, respectively. Refractory
ICP elevations may require barbiturate coma to manage.
Prophylaxis for seizures should be aggressively pursued as
seizure occurrence markedly increases cerebral oxygen consumption and ICP, and reduces CPP. Phenytoin should be used preferentially over benzodiazepines.
Overall, hepatic encephalopathy remains a difficult entity to
manage. As the disease is poorly understood, new modalities for
treatment have been slow to develop. The authors of the article
should be commended for their excellent overview of the topic.

365

PMPH_CH44.indd 365

5/22/2012 5:30:57 PM

CHAPTER 45
CHAPTERSurgery
2
Elective Non-hepatic
in
Cirrhotic Patients
Jared C. Brandenberger and Vafa Ghaemmaghami

INTRODUCTION

over OC with respect to operative time, morbidity, resumption of


diet, need for blood transfusion, and hospital stay.9
When comparing patients with cirrhosis to those without,
there is a greater need for conversion to an open procedure, longer
operative time, increased need for transfusion, increased overall
morbidity, and increased incidence of acute cholecystitis.7,10
Answer: In compensated cirrhotics with symptomatic gallstone disease, LC is safe, resulting in less overall morbidity than an
open approach. Operative risks in these patients are significantly
higher than in the general population without cirrhosis. Conclusions cannot be made about LC in patients with CTP C cirrhosis,
since most authors deem their operative risks to be unacceptably
high. (Grade B recommendation)

Though first noted as hardening of the liver associated with jaundice in ancient times by Hippocrates and Galen, the term cirrhosis was first coined by Ren Leannec in the 18th century. In
Greek kirrhos means orange-yellow; cirrhosis is the end result
of repeated hepatocellular injury, which leads to replacement of
hepatic parenchyma with scar and regenerative nodules. Its hallmarks are hepatocellular failure and portal hypertension, both of
which remain after the causative agent has been removed.
Liver transplantation has become the mainstay of definitive treatment, and remains the only option for cure in certain
patients. As medical therapy for complications of cirrhosis has
improved, the available number of organs for transplantation has
failed to keep pace. This has resulted in an ever growing population of patients living with cirrhosis. It has been estimated that
10% of cirrhotics undergo at least one operative procedure during
the last 2 years of their lives. This reality mandates that general
surgeons become informed regarding proper preoperative optimization, therapeutic approaches, and perioperative considerations in this high-risk population.

2. When and how should abdominal wall defects be repaired


in cirrhotics?
Umbilical hernia (UH) is very common in cirrhotics. Although
cirrhotics may also have inguinal (IH) and ventral hernias (VH),
UH is by far the most common and significant abdominal wall
defect in this population. Increased intraabdominal pressure,
relative malnutrition, and recanalization of the umbilical vein
are all thought to play a role in the prevalence of this condition
in the cirrhotic population. Historically, indications for operation included incarceration, ulceration, and leak; however, conservative management leads to increased emergent intervention,
with an expected increase in morbidity and mortality.11 There is
significant data in the recent literature to support elective UH
repair (UHR) in the well-compensated cirrhotic population with
minimal increase in morbidity over the noncirrhotic population
(Table 45.2).11-17 One study from the Netherlands showed that in
patients with UH and ascites (median MELD = 23, range 1827),
elective repair in 17 patients carried a 0% risk of mortality or
hepatic decompensation, an 18% wound complication rate, and a
24% rate of recurrence. Contrastingly, a similar group managed
expectantly had a 77% complication rate9 incarcerations, and
one spontaneous rupture and evisceration. Almost half (46%)
underwent emergent repair, with a 40% complication rate. Th is

1. Is laparoscopic cholecystectomy (LC) safe in cirrhotics with


symptomatic gallstone disease?
Multiple retrospective studies (Table 45.1) have been published
describing various authors experience with LC in the setting of
compensated cirrhosis (Child-Turcotte-Pugh [CTP] A/B, Model
for end-stage liver disease [MELD] < 8). Although not controlled,
the conclusion from these small series was that LC is a safe option
with lower morbidity than open cholecystectomy (OC). Postoperative morbidity ranged from 15% to 36%.1-6 The most common
complications associated with LC in cirrhosis include bleeding,
bile leak, wound infection, worsening of ascites, port site infection, and pulmonary infection.2,7,8
El-Awadi et al.9 performed a prospective randomized study
comparing the risks of OC with LC in cirrhotic patients. This
study confirmed significant benefits to patients undergoing LC
366

PMPH_CH45.indd 366

5/22/2012 5:32:15 PM

Elective Non-hepatic Surgery in Cirrhotic Patients

367

Table 45.1 Retrospective Studies on Safety of Laparoscopic Cholecystectomy in Cirrhotics with Symptomatic
Gallstone Disease
Authors

Procedure

Cucinotta et al.1

CTP

Number of Patients

Morbidity (%)

Mortality (%)

Level of Evidence

LC

A/B

22

36

LC

A/B

20

15

LC

A/B/C

25

52

2b

Tuech et al.4

LC

A/B

26

27

LC

A/B

34

17.6

2b

Shaikh et al.
Urban et al.

Leandros et al.
Leone et al.

Puggioni et al.7
Delis et al.

El-Awadi et al.9
10

Clark et al.

LC

A/B

24

20.8

2b

LC

A/B/C

351

20.86

0.59

2a

LC

A/B

220

19

2b

LC/OC

A/B

110 (55/55)

23.6 (12.7/34.5)

0 (0/0)

1b

LC

15

Table 45.2 Repair of Abdominal Wall Defects in Cirrhotics


Authors

Procedure

Gray et al.11

UHR

127

9.5

NR

0.8

2b

Marsman et al.12

UHR

17

18

24

AWH

32022

16.5

NR

2.5

2b

13

Carbonell et al.

Number of
Patients

Complications
(%)

Recurrence
(%)

Mortality
(%)

Level of
Evidence

14

Ammar et al.

UHR (trad/mesh)

80 (40/40)

39 (40/37.5)

15 (14.2/2.7)

0 (0/0)

1b

Youssef et al.15

UHR (mesh/trad)

40 (20/20)

30 (40/20)

25.7 (10/35)

87.5 (10/15)

1b

UHR (drain/none)

24 (12/11)

69.6 (25/100)

16.6 (8.3/27.3)

0 (0/0)

1b

16

Elsebae et al.
17

Belli et al.

Lap UHR/IHR

14

78.5

Park et al.18

UHR/IHR/VHR

53

NR

1.9

1.9

Mesh IHR

32

6.3

Preoperative TIPS

29

NA

14%

Patti et al.

20

Schlenker et al.19

NR = not reported; AWH = abdominal wall hernia repair (excluding IHR); Trad = traditional.

group had an overall mortality of 15%.12 One small retrospective


study by Park et al.18 shows no increase in operative risk for CTP
C patients undergoing hernia repair. Th is group, however, has
been sparsely studied.
Perioperative control of ascites is mandatory in the cirrhotic
undergoing UHR. The most common measure is aggressive medical management with diuretics and intermittent paracentesis. The
use of drains in selected patients has been advocated, and with
appropriate antibiotic prophylaxis, does not appear to increase
the rate of infection.15,16 However, the most effective treatment for
refractory ascites in patients with UH is the transjugular intrahepatic portosystemic shunt (TIPS).18,19 This should be considered
and aggressively pursued prior to elective repair in patients with
ascites refractory to conservative management.
The traditional method for UHR a primary repair, overlapping in a vest over pants fashion if feasible,14-16 However, there
has been recent data showing safety and increased durability of
repair with prosthetic mesh.14,15 Both studies cited show a significant decrease in recurrence of UH. There was a small but significant increase in superficial infection with the mesh groups. In
neither study did this mandate mesh removal or precipitate recurrence. With regard to IH, a study by Patti et al.20 demonstrated
that Lichtenstein repair in cirrhotics is not only safe and effective,
but also significantly improves quality of life.

PMPH_CH45.indd 367

Laparoscopy for abdominal wall defects in the cirrhotic patient


has been addressed in reports and small case series; it is a versatile
and widely applied modality and may become more prevalent as
more studies are published. A review by Belli et al.17 followed up
14 patients over 8 months after laparoscopic incisional repair and
URH. No recurrences or deaths were reported in their small series.
Answer: Elective UHR and IH repair (IHR) is safe early in the
course of cirrhosis, with morbidity comparable to the noncirrhotic
population. The use of mesh is associated with a slight increase in
wound complications, but significantly less recurrence. Ascites
should be aggressively controlled in the perioperative period, with
consideration given to TIPS. The use of laparoscopy is promising
for abdominal wall defects. (Grade B recommendation)
3. How can cirrhotic patients be optimized preoperatively to
improve outcomes?
Complications of portal hypertension, especially ascites and
hemorrhage, have the most profound impact on outcomes after
operation. Perioperative control of coagulopathy is of the utmost
importance in these patients, as bleeding and transfusion have
been shown to have a direct effect in mortality.21 Vitamin K
administration, although usually ineffective due to synthetic
dysfunction, can help correct deficiency due to malabsorption in

5/22/2012 5:32:16 PM

368

Surgery: Evidence-Based Practice

Table 45.3 Mortality Risk in Cirrhotic Patients With or Without Portal Hypertension
Procedure

No Portal Hypertension (HR [CI])

Portal Hypertension (HR [CI])

Cholecystectomy

3.4 (2.35.0)

12.3 (7.619.9)

Colectomy

3.7 (2.65.2)

14.3 (9.721.2)

CABG

8.0 (5.013.0)

22.7 (10.053.8)

AAA Repair

5.0 (2.68.0)

7.8 (2.326.5)

HR = hazard ratio; CI = confidence interval; CABG = coronary artery bypass grafting; AAA = abdominal aortic aneurysm.

cholestatic patients. In a study of invasive procedures in cirrhotics being evaluated for liver transplant, Gianni et al.22 showed an
increase in postprocedure bleeding with severe thrombocytopenia
(platelets < 75 K/mm3). Coagulopathy as assessed by international
normalized ratio (INR) was not associated with postprocedure
bleeding.22 However, in a study of trauma patients with blunt
splenic injury, coagulopathy was associated with increased
mortality, and aggressive correction was recommended.23 Franzetta et al.24 found coagulopathy, as well as tense ascites and
hypoalbunemia, to be a significant predictor of mortality in their
retrospective review.24 Although studied in the transplant literature, agents such as tranexamic acid, aprotenin, desmopression,
and recombinant factor VIIa have not been studied in this patient
population. Thromboelastography has been shown to be a useful
adjunct in transplantation for cirrhotics, and can help precisely
direct component therapy.25 This advantage has yet to be demonstrated in those patients undergoing nonhepatic surgery.
Early and aggressive diuresis is the fi rst step in achieving
control of ascites. A strict low-sodium diet combined with potassium sparing and loop diuretics are the most common initial
therapeutic measures.18 Paracentesis can also be used acutely to
control large volume ascites in the perioperative period; however,
it is important to replace albumin to maintain intravascular
volume.15,16,18,19,26 Performance of a TIPS procedure addresses both
portal decompression and ascites, and strong consideration
for elective placement should be given prior to any significant
intraabdominal undertaking, as it is preferred to peritoneovenous shunting.19,26
Prophylaxis and treatment of hepatic encephalopathy had
also been investigated extensively for liver resection and TIPS.27,28
Lactulose titrated to three bowel movements daily remains the
mainstay of treatment, and may be supplemented with rifaximin,
metronidazole, and neomycin as indicated.28 A recent doubleblind placebo controlled trial confirmed the benefits of rifaximin not just in treatment, but remission from encephalopathy
over a 6-month study period. In this study by Bass and et al.,29
299 patients were randomized to rifaximin treatment versus placebo. The group treated with rifaximin had significantly less hospitalizations and less incidents of breakthrough encephalopathy.
Adverse events did not differ between the groups.29
Answer: Early and aggressive therapy directed at control of
ascites and coagulopathy should be instituted to improve perioperative outcomes in cirrhotics. More study is needed to assess
whether strategies effective in the transplant population are effective in this population as well. (Grade C recommendation)
4. What factors are most predictive of morbidity/mortality
risk in the cirrhotic population?
One of the most important perioperative factors predictive of
morbidity and mortality in the cirrhotic population is the need for
emergency surgery. Farnsworth et al.30 described in their study a

PMPH_CH45.indd 368

significant increase in both 1- and 3-month mortality in an emergent surgical group versus an elective cohort. In the elective and
emergent groups, 1- and 3-month mortality increased from 17%
and 21% to 19% and 44%, respectively. Both differences were statistically significant.30 Neeff et al.31 reported a fivefold difference in
mortality for those patients undergoing emergent versus elective
surgery (47% vs. 9%). This highlights the need for an aggressive,
appropriate, elective intervention in certain clinical situations,
some of which are highlighted above.
The presence of portal hypertension also has a large impact
on patient survival and complication rates. A study by Csikesz
et al.32 highlighted this risk factor, showing that within a sample
of 22,569 patients with cirrhosis, those with signs of portal hypertension were at significantly increased risk for increased mortality from four index operations (Table 45.3). This mortality risk
was confirmed in a study of 4000 patients undergoing colorectal
procedures. Wound, urinary, and pulmonary complications were
also noted in this study.33
Answer: The presence of portal hypertension and need for
emergency surgery are strong predictors of increased perioperative morbidity and mortality in the cirrhotic patient population.
(Grade C recommendation)
5. Which scoring system most accurately predicts perioperative morbidity and mortality in patients with cirrhosis?
The CTP score was developed in 1964 to stratify risk for patients
undergoing portosystemic shunting. It awards 1 to 3 points in five
categories: encephalopathy, ascites serum bilirubin, albumin, and
coagulopathy. Patients are then divided into classes based on their
scores: A (56 points), B (79), and C (1015).
The main drawback to the CTP scoring system cited repeatedly
in the literature is its inherent use of subjective measures (ascites
and encephalopathy). When applied consistently in a homogenous patient population, this can predict morbidity/mortality
risk quite well; however, interpretations hamper its use between
investigators.30,32,34,35 The MELD score was initially developed to
stratify short-term risk in cirrhotic patients undergoing TIPS, but
was quickly adapted for use in organ allocation by UNOS and
Eurotransplant. It uses three parameters: serum bilirubin, INR,
and creatinine; and has the advantage of objectivity.
Northrup et al.36 performed a retrospective study of 140 postoperative cirrhotic patients (50% were intraabdominal). Though not
compared with CTP score, MELD was the only significant independent predictor of 30-day mortality. Befeler et al.37 concluded
that a MELD score >14 more accurately predicted outcomes than
CTP class C, while a second group identified the threshold as 8 for
major morbidity.37,38 Few studies exist that compare prognostication of morbidity using MELD versus CTP scoring systems. Results
from these single-institution experiences were mixedone showed
that the MELD score was more accurate in the setting of gallbladder
disease,8 while two showed correlation between the two.30,35

5/22/2012 5:32:16 PM

Elective Non-hepatic Surgery in Cirrhotic Patients

The largest retrospective study to examine this issue involved 772


postoperative cirrhotic patients. Two control groups were included
in the analysis. The authors concluded that the most important predictors of both short- and long-term mortality were age, ASA score,
and hepatic dysfunction as measured by MELD score.39

369

Answer: The MELD scoring system appears to be superior to


the CTP system in a population of cirrhotic patients undergoing
major surgery. Contributions of comorbidities, such as from ASA
scoring, may help refine this system to better stratify surgical risk.
(Grade D recommendation)

Clinical Question Summary


Question

Answer

1 Is laparoscopic cholecystectomy
(LC) safe in cirrhotics with
symptomatic gallstone disease?

In compensated cirrhotics, LC is safe, with less


morbidity than OC. More study is needed to
define the role of LC in patients with CTP C
cirrhosis.

1-10

2 When and how should


abdominal wall defects be
repaired in cirrhotics?

Elective UHR and IHR is safe in compensated


cirrhosis. Mesh repair is associated with
an increase in wound complications, but
significantly less recurrence. Ascites should
be aggressively controlled, with TIPS if
refractory. Laparoscopic repair may become
more prevalent.

11-20

3 How can cirrhotic patients be


optimized preoperatively to
improve outcomes?

Early and aggressive control of ascites and


coagulopathy improves perioperative
outcomes in cirrhotics.

15, 16, 18, 19,


21-29

4 What factors are most


predictive of morbidity/
mortality risk in the cirrhotic
population?

Portal hypertension and emergent intervention


are predictors of increased morbidity and
mortality in cirrhotics.

30-33

5 Which scoring system


most accurately predicts
perioperative morbidity and
mortality in patients with
cirrhosis?

The MELD scoring system appears to be superior


to the CTP.

30-39

REFERENCES
1. Cucinotta E, Lazzara S, Melita G. Laparoscopic cholecystectomy
in cirrhotic patients. Surg Endosc. 2003;17:1958-1960.
2. Shaikh AR, Muneer A. Laparoscopic cholecystectomy in cirrhotic patients. JSLS. 2009;13(4):592-596.
3. Urban L, Eason GA, ReMine S, et al. Laparoscopic cholecystectomy in patients with early cirrhosis. Curr Surg. 2001;58(3):
312-315.
4. Tuech JJ, Pessaux P, Regenet N, et al. Laparoscopic cholecystectomy in cirrhotic patients. Surg Laparosc Endosc Percutan Tech.
2002;12(4):227-231.
5. Leandros E, Albanopoulos K, Tsigris C, et al. Laparoscopic
cholecystectomy in cirrhotic patients with symptomatic gallstone disease. ANZ J Surg. 2008;78(5):363-365.
6. Leone N, Garino M, De Paolis P, et al. Laparoscopic cholecystectomy in cirrhotic patients. Dig Surg. 2001;18(6):449-452.
7. Puggioni A, Wong LL. A metaanalysis of laparoscopic cholecystectomy in patients with cirrhosis. J Am Coll Surg. 2003;197(6):
921-926.
8. Delis S, Bakoyiannis A, Madariaga J, et al. Laparoscopic cholecystectomy in cirrhotic patients: the value of MELD score and
Child-Pugh classification in predicting outcome. Surg Endosc.
2010;24(2):407-412.

PMPH_CH45.indd 369

Grade of
Recommendation

References

9. El-Awadi S, El-Nakeeb A, Youssef T, et al. Laparoscopic versus


open cholecystectomy in cirrhotic patients: A prospective randomized study. Int J Surg. 2009;7(1):66-69.
10. Clark JR, Wills VL, Hunt DR. Cirrhosis and laparoscopic cholecystectomy. Surg Laparosc Endosc Percutan Tech. 2001;11(3):165-169.
11. Gray SH, Vick CC, Graham LA, et al. Umbilical herniorrhaphy
in cirrhosis: improved outcomes with elective repair. J Gastrointest Surg. 2008;12(4):675-681.
12. Marsman HA, Heisterkamp J, Jalm JA, et al. Management of
patients with liver cirrhosis and an umbilical hernia. Surgery.
2007;142(3):372-375.
13. Carbonell AM, Wolfe LG, DeMaria EJ. Poor outcomes in cirrhosis-associated hernia repair: A nationwide study of 32,033
patients. Hernia. 2005;9(4):353-357.
14. Ammar SA. Management of complicated umbilical hernias in
cirrhotic patients using permanent mesh: Randomized clinical
trial. Hernia. 2010;14(1):35-38.
15. Youssef YF, El Ghannam M. Mesh repair of non complicated
hernias in ascitic patients with liver cirrhosis. J Egypt Soc Parasitol. 2007;37(Suppl 3):1189-1197.
16. Elsebae MM, Nafeh AI, Abbas M, et al. New approach in surgical management of complicated umbilical hernia in the cirrhotic patient with ascites. J Egypt Soc Parasitol. 2006;36(Suppl
2):11-20.

5/22/2012 5:32:16 PM

370

Surgery: Evidence-Based Practice

17. Belli G, DAgostino A, Fantini C, et al. Laparoscopic incisional


and umbilical hernia repair in cirrhotic patients. Surg Laparosc
Endosc Percutan Tech. 2006;16(5):330-333.
18. Park JK, Lee SH, Yoon WJ, et al. Evaluation of hernia repair
operation in Child-Turcotte-Pugh class C cirrhosis and refractory ascites. J Gastroenterol Hepatol. 2007;22(3):377-382.
19. Schlenker C, Johnson S, Trotter JF. Preoperative transjugular intrahepatic portosystemic shunt (TIPS) for cirrhotic patients undergoing
abdominal and pelvic surgeries. Surg Endosc. 2009;23(7):1594-1598.
20. Patti R, Almasio PL, Buscemi S, et al. Inguinal hernioplasty
improves the quality of life in patients with cirrhosis. Am J Surg.
2008;196(3):373-378.
21. Telem DA, Schiano T, Goldstone R, et al. Factors that predict
outcome of abdominal operations in patients with advanced cirrhosis. Clin Gastroenterol Hepatol. 2010;8(5):451-457.
22. Giannini EG, Greco A, Marenco S, et al. Incidence of bleeding following invasive procedures in patients with thrombocytopenia and advanced liver disease. Clin Gastroenterol Hepatol.
2010;8(10):899-902.
23. Fang JF, Chen RJ, Lin BC, et al. Liver cirrhosis: An unfavorable
factor for nonoperative management of blunt splenic injury. J
Trauma. 2003;54(6):1131-1136.
24. Franzetta M, Raimondo D, Giammanco M, et al. Prognostic factors of cirrhotic patients in extra-hepatic surgery. Minerva Chir.
2003;58(4):541-544.
25. Wang SC, Shieh JF, Chang KY, et al. Thromboelastographyguided transfusion decreases intraoperative blood transfusion
during orthotopic liver transplantation: Randomized clinical
trial. Transplant Proc. 2010;42(7):2590-2593.
26. Telem DA, Schiano T, Divino CM. Complicated hernia presentation in patients with advanced cirrhosis and refractory ascites:
Management and outcome. Surgery. 2010;148(3):538-543.
27. Riggio O, Masini A, Efrati C, et al. Pharmacological prophylaxis of hepatic encephalopathy after transjugular intrahepatic
portosystemic shunt: A randomized controlled study. J Hepatol.
2005;42(5):674-679.
28. Lawrence KR, Klee JA. Rifaximin for the treatment of hepatic
encephalopathy. Pharmacotherapy. 2008;28(8):1019-1032.

PMPH_CH45.indd 370

29. Bass NM, Mullen KD, Sanyal A, et al. Rifaximin treatment in


hepatic encephalopathy. N Engl J Med. 2010;362(12):1071-1081.
30. Farnsworth N, Fagan SP, Berger DH, et al. Child-Turcotte-Pugh
versus MELD as a predictor of outcome after elective and emergent surgery in cirrhotic patients. Am J Surg. 2004;188(5):580583.
31. Neeff H, Mariaskin D, Spangenberg HC, et al. Perioperative
mortality after non-hepatic general surgery in patients with liver
cirrhosis: an analysis of 138 operations in the 2000s using Child
and MELD Scores. J Gastrointest Surg. 2011;15(1):1-11.
32. Csikesz NG, Nguyen LN, Tseng JF, et al. Nationwide volume and
mortality after elective surgery in cirrhotic patients. J Am Coll
Surg. 2009;208(1):96-103.
33. Nguyen, GC, Correia AJ, Thuluvath PJ. The impact of cirrhosis
and portal hypertension on mortality following colorectal surgery: A nationwide, population based study. Dis Colon Rectum.
2009;52(8):1367-1374.
34. Kim SY, Yim HJ, Park SM, et al. Validation of a post-operative
mortality risk prediction model in Korean cirrhotic patients.
Liver Int. 2011;31(2):222-228.
35. Hoteit M, Ghazale AH, Bain AJ, et al. Model for end-stage liver
disease score versus Child score in predicting the outcome of
surgical procedures with patients with cirrhosis. World J Gastroenterol. 2008;14(11):1774-1780.
36. Northup PG, Wanamaker RC, Lee VD, Adams RB, Berg CL.
Model for end-stage liver disease (MELD) predicts nontransplant surgical mortality in patients with cirrhosis. Ann Surg.
2005;242(2):244-251.
37. Befeler AS, Palmer DE, Hoff man W, et al. The safety of intraabdominal surgery in patients with cirrhosis: Model for end stage
liver disease is superior to Child-Turcotte-Pugh classification in
predicting outcome. Arch Surg. 2005;140(7):650-654.
38. Perkins L, Jeff ries M, Patel T. Utility of perioperative scores for
predicting morbidity after cholecystectomy in patients with cirrhosis. Clin Gastroenterol Hepatol. 2004;2(12):1123-1128.
39. Teh SH, Nagorney DM, Stevens SR, et al. Risk factors for mortality after surgery in patients with cirrhosis. Gastroenterology.
2007;132(4):1261-1269.

5/22/2012 5:32:16 PM

Commentary on Elective
Non-hepatic Surgery in
Cirrhotic Patients
Bruce Gelb and H. Leon Pachter

Dr. Brandenberger and Ghaemmaghamis chapter on Elective


non-hepatic surgery in cirrhotic patients addresses important
issues in the high risk and complicated subset of surgical patients
with end stage liver disease. While liver transplantation is the ultimate surgical therapy in cirrhotic patients, the paucity of donor
organs and factors that preclude the feasibility of transplantation
in certain patients leaves surgeons with a non-ideal population that
requires surgical intervention. As the authors appropriately note,
improved medical management and understanding of patients
with cirrhosis has led to an increased prevalence of patients living
with cirrhosis in need of surgical intervention.
Sufficient literature exists addressing the level of safety and
expected morbidities in Childs A, and to a lesser extent Childs B cirrhosis. Unfortunately, studies are insufficient and underpowered in
Childs C cirrhosis to provide the high level evidence needed for firm
recommendations on how to best manage these patients surgically.
Operative risk is not absolute with cirrhosis, but rather correlates with the degree of physiologic derangement associated with
hepatic synthetic function, hepatic reserve, and portal hypertension.
Indeed, patients with Childs A cirrhosis have relatively preserved
hepatic function, and therefore carry a lower risk of morbidity and
mortality compared to less compensated cirrhotic patients.
As the authors show, multiple studies suggest that Laparoscopic Cholecystectomy in Childs A and select Childs B patients
is a relatively safe procedure when performed electively. Morbidity increases in Childs B and C. Performing LC on Childs B is
technically more difficult than in Childs A patients and is associated with significantly longer operative times, increased intraoperative blood loss, and requires more frequent conversion to
an open procedure1. Portal hypertension increases the risk of
bleeding from the liver bed. Judicious preoperative correction of
coagulopathy and platelets is essential. In cirrhotic patients, we
recommend liver transplantation evaluation and listing prior to
cholecystectomy.
It is imperative to avoid certain technical pitfalls when performing LC in the cirrhotic patient. 1) Avoid abdominal wall
varices during laparoscopic port placement. We advocate the use
of placing an umbilical or infra-umbilical Hassan port first via
direct cutdown and peritoneal entry under direct visualization.
Supraumbilical port placement is particularly hazardous due to
frequent umbilical vein recanalization secondary to portal hypertension. Transillumination of the abdominal wall will assist in

helping avoid subcutaneous abdominal wall varices during the


placement of additional laparoscopic ports. 2) Utilization of harmonic scalpel as an adjunct to diathermy during the gallbladder
dissection helps decrease intraoperative bleeding, gall bladder
perforation, and operative times using this technique in Childs
A and B patients.2 3) Exercise a lower threshold for performing
subtotal cholecystectomy. Bleeding from the liver bed can be
particularly troublesome during dissection and retraction of the
gallbladder. Chronic inflammation, portal venous collaterals, and
robust lymphatics can make dissection of the cystic duct especially hazardous as one nears the porta hepatis. It may be necessary to use a stapling device to ligate and divide the cystic duct
and artery. 4) Argon Beam Coagulation and judicious utilization
of topical hemostatic agents (avateen, cyano-acrylate, or oxidized
cellulose agents) are particularly useful in aiding hemostasis. 5)
Drain placement in the gallbladder fossa is advocated.
Regarding repair of abdominal wall defects in cirrhotics, the
reviewed literature supports the elective repair of umbilical hernias, as they are quite common in this population for the reasons
noted by the authors. Deferring surgery until urgent or emergent
presentation (incarceration, ulceration or rupture) is a misstep
and leads to unacceptable outcomes compared to earlier operative management. The authors are correct in advocating for the
control of ascites. Medical management with diuretics is the first
step in management with intermittent paracentesis as warranted.
For patients who fail to improve significantly with medical management, TIPS should be considered prior to elective repair. All
patients with ascites must be evaluated for spontaneous bacterial peritonitis (SBP) prior to surgery by diagnostic paracentesis
and cell count, and elective surgery must be deferred until SBP
is adequately treated. As the authors note, adequate nutrition is
essential for healing in these patients.
The decision to use mesh in UHR must be balanced with the
risk of infection. We recommend avoiding the use of nonabsorbable mesh in patients who have had multiple bouts or a recent
history of SBP. Ascites reaccumulation in the repair bed can be
particularly problematic and difficult to manage and is associated
with a high rate of recurrence. Our experience in using mesh as
an overlay or underlay of a primary tissue repair has been helpful
in reducing subcutaneous fluid accumulation. Drain placement
in the subcutaneous tissues in patients with moderate to large
umbilical hernia defects or redundant skin is also helpful.

371

PMPH_CH45.indd 371

5/22/2012 5:32:16 PM

372

Surgery: Evidence-Based Practice

The third question addresses the preoperative optimization


of cirrhotic patients, and is arguably the most important factor in
limiting perioperative morbidity and mortality.
Preoperative optimization of ascites has been addressed earlier, but it is important to emphasize (though only briefly mentioned by the authors) that aggressive diuretic therapy or large
volume paracentesis can precipitate circulatory collapse and acute
renal failure. This is best temporized by the administration of concentrated salt-poor albumin for plasma expansion3 +/ splanchnic
vasoconstrictors (i.e. octreotide and midodrine).
Intraoperative hemorrhage as a consequence of portal hypertension and coagulopathy is a major obstacle in cirrhotic patients.
Directed correction of coagulopathy can be better guided with
the use of thromboelastography. Thrombopoietin (TPO) receptor agonists are currently under investigation as a potential future
therapy for thrombocytopenia.4
The use of protease inhibitors in liver transplantation is mainly
due to increased fibrinolysis associated with resection of diseased
liver and to ischemia/reperfusion injury of a transplanted liver5,6
and therefore, has minimal application outside of hepatic resection in cirrhotic patients or liver transplantation.
The factors most predictive of morbidity/mortality in cirrhotic patients clearly parallel the extent of portal hypertension
and the need for emergency surgery. The key to offsetting these
factors is early recognition of patients likely to require surgery.
Obviously little can be done in unexpected situations such as
trauma, but keen forethought can prevent an elective umbilical
hernia repair or bowel surgery from becoming a perioperative
disaster. Cooling off a bout of acute cholecystitis or appendicitis
with antibiotic therapy and percutaneous drainage is more likely
to allow the surgeon the opportunity of coming back another day
and winning the fight rather than dooming the patient.
The final topic of which scoring system is best to predict poor
outcomes in cirrhotic patients is a tricky one. Yes, the ChildTurcotte-Pugh scoring system is partly based on subjective measures and neglects the contribution of renal function to risk of
morbidity/mortality, but it is the most recognized system across
the medical specialties. The MELD scoring system is clearly a
more objective measuring tool in patients with end stage liver disease (the same can be said of the APACHE-II), but is less understood outside the realm of transplant surgery and hepatology.

PMPH_CH45.indd 372

On a final note, it is important to consider the following questions prior to performing an operation in a cirrhotic patient: Is
the procedure technically feasible and relatively safe? Does the
risk of performing surgery outweigh the risk of nonsurgical or
less invasive management? Is the patient sufficiently optimized
with regard to ascites, renal function, cardiac status, pulmonary
function, and infection control? Is there adequate expertise and
specialty assistance (Hepatology, Nephrology, Critical Care,
Anesthesia, Transplant and Hepatobiliary Surgery) available to
assist in the perioperative management of a patient with highly
complex problems and diminished reserve and limited ability to handle complications? Cirrhotic patients are clearly an
exceptionally difficult group of patients to treat, especially when
undergoing the physiological insult of operative intervention.
Their management requires a highly specialized and multidisciplinary approach, often best suited in a tertiary care setting.

REFERENCES
1. Nguyen KT, Kitisin K, Steel J et al. Cirrhosis is not a contraindication to laparoscopic cholecystectomy: results and practical recommendations. HPB. 2011;13:192-197.
2. Bessa SS, Abdel-Razek AH, Aharaan MA et al. Laparoscopic
cholecystectomy in cirrhotic: A prospective randomized study
comparing the conventional diathermy and the harmonic scalpel
for gallbladder dissection. J Lap Adv Surg Tech. 2011;21(1):1-5.
3. Sola-Vera J, Minana J, Ricart E et al. Randomized trial comparing albumine and saline in the prevention of paracentesis-induced
circulatory dysfunction in cirrhotic patients with ascites. Hepatology. 2003;37:1147-1153.
4. A Phase 2, Randomized, Multicenter, Placebo-Controlled, DoubleBlind, Parallel-Group Study to Evaluate the Efficacy, Safety, and
Population Pharmacokinetics of Once-Daily Oral E5501 Tablets
Used Up to 7 Days in Subjects With Chronic Liver Diseases and
Thrombocytopenia Prior to Elective Surgical or Diagnostic Procedures. www.clinicaltrials.gov, identifier: NCT00914927.
5. Ryu HG, Jung CW, Lee CS, Lee J. Nafamostat Mesilate Attenuates Postreperfusion Syndrome during Liver Transplantation. Am
J Transplant. 2011;(11):977-983.
6. Makwana J, Paranjape S, Goswami J. Antifibrinolytics in liver surgery. Indian J Anaesth. 2010 Nov-Dec;54(6):489-495.

5/22/2012 5:32:16 PM

PART VII

THE GALLBLADDER AND


BILE DUCTS

PMPH_CH46.indd 373

5/22/2012 5:32:52 PM

PMPH_CH46.indd 374

5/22/2012 5:32:53 PM

CHAPTER 46

Silent Gallstones
Abdul Saied and James C. Doherty

INTRODUCTION

Laparoscopic surgery and its well-documented advantages


in terms of length of hospital stay, decreased postoperative pain,
faster recovery, and shorter time to full preoperative activity, have
facilitated the transition from open-cholecystectomy as the gold
standard treatment of symptomatic gallstones to laparoscopic
cholecystectomy. In the United States, laparoscopic cholecystectomy is the most common abdominal operation with 750,000 surgeries performed annually.4

Cholelithiasis constitutes one of the most common causes of gastrointestinal symptoms requiring hospital admission and emergency department visits and thus also represents a major source
of healthcare system expenditure. The defi nitive treatment of
symptomatic or complicated cholelithiasis is cholecystectomy. For
many years, open-cholecystectomy was reserved for symptomatic
cases or case involving significant complications. The introduction of laparoscopic surgery and the availability of laparoscopic
cholecystectomy as a less-invasive alternative to the open procedure have fueled a new discussion about the current indications
for cholecystectomy.
The aim of this review is to present the available data and to
provide recommendations that could be used as guidelines in the
treatment of asymptomatic cholelithiasis.

2. What is the natural history of asymptomatic (silent) gallstones?


To better understand this pathology we should first define
the entity. Asymptomatic cholelithiasis exists when gallstones
are detected in the absence of gallstone-related symptoms. These
symptoms are grouped as biliary colic or gallstone-related complications such as acute cholecystitis, acute cholangitis, or biliary
pancreatitis.
The understanding of the natural history of gallstones has
changed significantly in the past 100 years. In 1904, William J. Mayo
offered the famous dictum There is no innocent gallstone. On
the other hand, several studies from the 1980s established the
innocuous nature of asymptomatic gallstones when subjected to
long-term follow-up.5-7 Much controversy still exists about the
role of silent gallstones in gallbladder cancer, cholangitis, and
choledocolithiasis.
Most of the Western population with silent gallstones will
remain asymptomatic throughout their lives and do not require
any treatment.8 The rates of conversion of asymptomatic cholelithiasis varies within a range of 10% to 18% in 10- to 15-year
follow-up, and with an annual risk of developing biliary pain of
1% to 4%.7,9 On the other hand, symptomatic gallstones have a
1.2% risk of developing complications and a 50% risk of developing biliary pain.10 The cumulative probability of developing complications after 10 years is 3% in asymptomatic patients and 7% in
symptomatic patients.11
Ransohoff and Gracie12 published conversion rates of 10%,
15%, and 18% for 5, 10, and 15 years, respectively. These findings
correlate with the observations by Hermann,13 who concluded that
the longer the patient lives with gallstones the more likely they are

1. How frequent is this problem in the general population?


Gallstones are not a new disease entity. Autopsy reports from
Egyptian and Chinese mummies have demonstrated the presence
of gallstones more than 3500 years ago. In the 20th century, gallstone disease represents a major health problem with about 10% to
15% of the adults of the Western population having gallstones, more
than 20 million in the United States alone. One million patients
are newly diagnosed every year (about 1 in 200). In the United
States, gallstone disease represents approximately USD 6.2 billion
in direct and indirect healthcare costs, with a 20% increase since
1980, and it accounts for 1.8 million ambulatory visits.1
The majority of patients with gallstones will remain asymptomatic (the so-called silent gallstones) with only 1% to 4% of
patients presenting symptoms per year. Approximately 10% will
present symptoms 5 years after the diagnosis, and 20% in the next
20 years.2 In Western Europe the prevalence of gallstones is similar to that in the United States, ranging from 5.9% to 21.9%.3 Interestingly, despite the similar incidence, there are six times as many
cholecystectomies in the United States compared with Europe, a
difference primarily due to variation in the indications for surgical treatment among different nations.
375

PMPH_CH46.indd 375

5/22/2012 5:32:53 PM

376

Surgery: Evidence-Based Practice

to experience pain or complications. Other authors have found


similar rates of conversion with increasing numbers of symptomatic patients with age and longer follow-up.13-16 Conversely, other
investigators have observed that the longer patients were asymptomatic, the less likely they were to develop symptoms.8,17-19
It is important to note that the risk of having a life-threatening
complication (cholangitis with sepsis or severe pancreatitis) from
asymptomatic gallstones is extremely low with almost no reported
cases without first experiencing biliary symptoms.
3. Is prophylactic cholecystectomy justifiable from a risk
benefit standpoint?
As presented in the previous section, the risk of having biliary
symptoms or complications from asymptomatic gallstones ranges
from 1% to 4% annually with higher rates in the elderly population. Prophylactic cholecystectomy to prevent biliary symptoms
and associated complications, such as pancreatitis, cholangitis,
choledocolithiasis, or cholcystitis, requires consideration of the
operative risks of laparoscopic cholecystectomy as well as those of
general anesthesia.
The establishment of the minimally invasive technique of laparoscopic cholecystectomy as the standard treatment of gallstone
disease has certainly broadened the indications for cholecystectomy and decreased the surgical threshold. However, laparoscopic
cholecystectomy still carries a 1% risk of common bile duct injury,
a 3% risk of bile leaks, and a 1% or 2% risk of wound infection,
bleeding, retained stones, and sludge or bile spillage with/without
abscess formation. Moreover, one must also consider the complications associated with general anesthesia such as venous thromboembolism, pneumonia, myocardial infarction, and atelectasis.20
One argument in support of prophylactic cholecystectomy
centers around the increased technical challenges associated
with surgery for symptomatic and/or complicated disease. Such
surgery has been shown to require increased operative time
(106 vs. 92 min), and to increase the laparoscopic-to-open conversion rate (8.8% vs. 4.72%, p < .005). Moreover, while overall
operative mortality is only around 0.6%, it increases with age
(0.140.4% in patients <50 years and 4.5% in those >65 years).21
This observation argues for early aggressive treatment of the
asymptomatic young patient rather than waiting until the patient
becomes symptomatic at an advanced age. To this day, no randomized controlled trial exists that compares cholesystectomy
with expectant management for silent gallstones. Furthermore,
additional observational studies would be necessary to determine
whether a randomized trial would even be justified. Thus, at present, the evidence-based recommendation for the general population of patients with gallstones is that cholecystectomy should only
be offered to patients with symptomatic gallstones. No compelling
high-level clinical data exists that would support the practice of
prophylactic cholecystectomy. (Grade B recommendation).
4. Is prophylactic cholecystectomy cost-effective?
In the absence of strong clinical data, some might argue that early
operative management versus expectant management of asymptomatic gallstones might be justified based on economic considerations. Certainly, early operative management would reduce
the healthcare costs associated with some of the highly morbid
complications of gallstones such as acute cholecystitis, acute cholangitis, and biliary pancreatitis. Also, cholecystectomy could be

PMPH_CH46.indd 376

offered to younger low-risk patients rather than older high-risk


patients with associated decreased healthcare costs.
Nevertheless, such healthcare savings are likely to be dwarfed
by the costs associated with the performance of an expensive and
not totally benign intervention (laparoscopic cholecystectomy) on
a large population of patients for a highly prevalent disease. While
it remains an intriguing question, no detailed economic analysis
has been published that examines the cost-effectiveness of prophylactic cholecystectomy versus expectant management. In the
absence of such data, there is no compelling economic argument
that would support the practice of prophylactic cholecystectomy
in the general population. (Grade C recommendations)
5. Should prophylactic cholecystectomy be performed in certain
at-risk populations?
The concept of selective prophylactic cholecystectomy (cholecystectomy for specific groups of presumably high-risk patients) has been
evaluated by different authors with no clear consensus. Among
the specific high-risk groups that have been studied are diabetics,
solid-organ transplant recipients, patients with sickle cell disease
(SCD), and patients at increased risk for gallbladder cancer.

DIABETES MELLITUS
Diabetic patients are considered a high-risk group because the
complications of gallstone disease are more severe those of the
general population in part due to the masking of the symptoms
of acute cholecystitis by autonomic neuropathy.22 Nevertheless, the value of prophylactic cholecystectomy in this group of
patients remains controversial. Some authors have shown major
complications rates similar to those of nondiabetic patients with
low 14.5% rates of biliary symptoms.23 Chapman et al.24 found
an increase in the prevalence of gallstones in diabetics compared
with controls, but only noninsulin-dependent diabetes was
found to be an independent factor in multivariate analysis. In
a Swedish study, Persson and Thulin25 found similar prevalence
rates in diabetics compared with nondiabetic controls (14.4% vs.
12.5%).
Friedman et al. compared expectant management versus prophylactic cholecystectomy in diabetic patients using a model of
probability and outcomes estimates. Th is analysis showed no
benefit derived from prophylactic cholecystectomy and improved
life expectancy with conservative management.26 Thus, there is no
evidence to support selective cholecystectomy in diabetic patients,
and expectant management is recommended.

TRANSPLANT PATIENTS
Prophylactic cholecystectomy for patients with silent gallstones
waiting for solid-organ transplantation has been strongly recommended.27 The authors argue that such patients are at-risk to
develop more severe complications secondary to their immunosuppression making surgery more difficult and raising their
morbidity and mortality. Conversely, several other authors have
presented data supporting expectant management of silent gallstones in transplanted patients.28-31 Thus, no clear consensus exists
as to the role of prophylactic cholecystectomy in transplantation
patients.

5/22/2012 5:32:53 PM

Silent Gallstones

SICKLE CELL DISEASE


The accelerated hemolysis in SCD patients produce an increase in the
formation of pigmented gallstones. The prevalence of gallstones is
approximately 52% in homozygous patients and 20% in heterozygous
patients with a 50% incidence of complications, a significantly higher
rate than that of non-SCD patients.32 Symptomatic gallstone disease
can present a significant diagnostic dilemma in the SCD patient
because biliary symptoms can be confused with vaso-occlusive crisis
producing a delay in diagnosis and treatment in this population.
Most authors agree that this group of patients benefit from
prophylactic cholecystectomy.20,33,34 The recommended approach
is laparoscopic with perioperative partial exchange transfusion to
lower the Hb-S to <50%. This specific approach has been associated with a lower risk of veno-occlusive crises.35

GALLBLADDER CANCER AND


GALLSTONES
There have been several attempts to associate gallstones with
gallbladder cancer. However, no causative relationship has been

377

proven. Specific populations have an increase risk of gallbladder


cancer including North American Native American women,
and the people of certain South America countries (Colombia,
Chile, Bolivia). 20 This cancer risk has been estimated to be
increased as much as fourfold in the presence of gallstones,
and about 80% to 90% of the patients with gallbladder cancer
have gallstones. 36 On the other hand, only 0.01% of the patients
with symptomatic gallstones will develop cancer.18 Large stones
>3 cm have showed stronger association with gallbladder cancer. 36 For general populations, prophylactic cholecystectomy is
not indicated to prevent gallbladder cancer. Nevertheless, individuals from the aforementioned high-risk populations with
>3 cm gallstones might benefit from a more aggressive approach,
especially given the fact that gallbladder cancer usually presents
at an advanced stage with poor survival. Such patients should
be treated on a case-by-case basis with acknowledgement of the
absence of strong supporting evidence. Other recommendations
for cholecystectomy to prevent cancer are porcelain gallbladder
and gallbladder polyps >10 mm. 37 These recommendations are C
(exception being recommendation for prophylactic cholecystectomy in the setting of sickle cell disease where this is a Grade B
recommendation).

Clinical Question Summary


Question

Answer

1 How frequent is this problem in


the general population?

10% to 15% of Western population

1-4

2 What is the natural history of


asymptomatic (silent) gallstones?

14% of patients will develop


symptoms yearly

1-4

3 Is prophylactic cholecystectomy
justifiable from riskbenefit
standpoint?

Not justifiable, risk of complications is


lower than morbidity and mortality
from the procedure

2b

5, 18, 19

4 Is prophylactic cholecystectomy
cost-effective?

Not for general population

22

5 Should prophylactic
cholecystectomy be performed
in certain at-risk populations?

Only recommended for sickle cell


patients, not recommend for
diabetic, transplant patients or to
prevent gallbladder cancer

2b

24, 25, 27,


29-32, 33

REFERENCES
1. Everhart JE, Ruhl CE. Burden of digestive diseases in the United
States part II: Lower gastrointestinal diseases. Gastroenterology.
2009;136:741-754.
2. Gallstones and laparoscopic cholecystectomy. NIH Consens
Statement. 1992;10:1-28.
3. Aerts R, Penninckx F. The burden of gallstone disease in Europe.
Aliment Pharmacol Ther. 2003;18(Suppl 3):49-53.
4. Stinton LM, Myers RP, Shaffer EA. Epidemiology of gallstones.
Gastroenterol Clin North Am. 2010;39:157-169, vii.
5. Ransohoff DF, Gracie WA, Wolfenson LB, Neuhauser D. Prophylactic cholecystectomy or expectant management for silent gallstones. A
decision analysis to assess survival. Ann Intern Med. 1983;99:199-204.

PMPH_CH46.indd 377

Levels of
Evidence

Grade of
Recommendation

Reference

6. McSherry CK, Glenn F. The incidence and causes of death following surgery for nonmalignant biliary tract disease. Ann Surg. 1980;
191:271-275.
7. Gracie WA, Ransohoff DF. The natural history of silent gallstones: The innocent gallstone is not a myth. N Engl J Med. 1982;
307:798-800.
8. Friedman GD. Natural history of asymptomatic and symptomatic gallstones. Am J Surg. 1993;165:399-404.
9. Zubler J, Markowski G, Yale S, Graham R, Rosenthal TC. Natural history of asymptomatic gallstones in family practice office
practices. Arch Fam Med. 1998;7:230-233.
10. Beckingham IJ, Krige JE. ABC of diseases of liver, pancreas, and
biliary system: Liver and pancreatic trauma. Br Med J. 2001;322:
783-785.

5/22/2012 5:32:53 PM

378

Surgery: Evidence-Based Practice

11. Prevalence of gallstone disease in an Italian adult female population. Rome Group for the Epidemiology and Prevention of
Cholelithiasis (GREPCO). Am J Epidemiol. 1984;119:796-805.
12. Ransohoff DF, Gracie WA. Treatment of gallstones. Ann Intern
Med. 1993;119:606-619.
13. Hermann RE. The spectrum of biliary stone disease. Am J Surg.
1989;158:171-173.
14. Glenn F. Silent gallstones. Ann Surg. 1981;193:251-252.
15. Glenn F. Surgical management of acute cholecystitis in patients
65 years of age and older. Ann Surg. 1981;193:56-59.
16. McSherry CK, Ferstenberg H, Calhoun WF, Lahman E, Virshup M.
The natural history of diagnosed gallstone disease in symptomatic
and asymptomatic patients. Ann Surg. 1985;202:59-63.
17. Lowenfels AB, Domellof L, Lindstrom CG, Bergman F, Monk
MA, Sternby NH. Cholelithiasis, cholecystectomy, and cancer: A
case-control study in Sweden. Gastroenterology. 1982;83:672-676.
18. Attili AF, De Santis A, Capri R, Repice AM, Maselli S. The natural history of gallstones: The GREPCO experience. The GREPCO
Group. Hepatology. 1995;21:655-660.
19. NIH Consensus conference. Gallstones and laparoscopic cholecystectomy. JAMA. 1993;269:1018-1024.
20. Sakorafas GH, Milingos D, Peros G. Asymptomatic cholelithiasis:
Is cholecystectomy really needed? A critical reappraisal 15 years
after the introduction of laparoscopic cholecystectomy. Dig Dis
Sci. 2007;52:1313-1325.
21. McSherry CK. Cholecystectomy: The gold standard. Am J Surg.
1989;158:174-178.
22. Schwesinger WH, Diehl AK. Changing indications for laparoscopic cholecystectomy. Stones without symptoms and symptoms
without stones. Surg Clin North Am. 1996;76:493-504.
23. Del Favero G, Caroli A, Meggiato T, et al. Natural history of gallstones in non-insulin-dependent diabetes mellitus. A prospective
5-year follow-up. Dig Dis Sci. 1994;39:1704-1707.
24. Chapman BA, Wilson IR, Frampton CM, et al. Prevalence of gallbladder disease in diabetes mellitus. Dig Dis Sci. 1996;41:2222-2228.

PMPH_CH46.indd 378

25. Persson GE, Thulin AJ. Prevalence of gallstone disease in patients with
diabetes mellitus. A case-control study. Eur J Surg. 1991;157:579-582.
26. Friedman LS, Roberts MS, Brett AS, Marton KI. Management of
asymptomatic gallstones in the diabetic patient. A decision analysis. Ann Intern Med. 1988;109:913-919.
27. Kao LS, Kuhr CS, Flum DR. Should cholecystectomy be performed for asymptomatic cholelithiasis in transplant patients?
J Am Coll Surg. 2003;197:302-312.
28. Jackson T, Treleaven D, Arlen D, DSa A, Lambert K, Birch DW.
Management of asymptomatic cholelithiasis for patients awaiting renal transplantation. Surg Endosc. 2005;19:510-513.
29. Greenstein SM, Katz S, Sun S, et al. Prevalence of asymptomatic
cholelithiasis and risk of acute cholecystitis after kidney transplantation. Transplantation. 1997;63:1030-1032.
30. Lord RV, Ho S, Coleman MJ, Spratt PM. Cholecystectomy in cardiothoracic organ transplant recipients. Arch Surg. 1998;133:73-79.
31. Melvin WS, Meier DJ, Elkhammas EA, et al. Prophylactic cholecystectomy is not indicated following renal transplantation. Am
J Surg. 1998;175:317-319.
32. Walker TM, Hambleton IR, Serjeant GR. Gallstones in sickle cell
disease: Observations from The Jamaican Cohort study. J Pediatr.
2000;136:80-85.
33. Fall B, Sagna A, Diop PS, Faye EA, Diagne I, Dia A. Laparoscopic
cholecystectomy in sickle cell disease. Ann Chir. 2003;128:702-705.
34. Vecchio R, Cacciola E, Di Martino M, Gambelunghe AT, Murabito P, Cacciola RR. Laparoscopic surgery in sickle cell disease.
Surg Endosc. 2002;16:1807-1808.
35. Al-Mulhim AS, Al-Mulhim FM, Al-Suwaiygh AA. The role of laparoscopic cholecystectomy in the management of acute cholecystitis
in patients with sickle cell disease. Am J Surg. 2002;183:668-672.
36. Godrey PJ, Bates T, Harrison M, King MB, Padley NR. Gall stones
and mortality: A study of all gall stone related deaths in a single
health district. Gut. 1984;25:1029-1033.
37. Tewari M. Contribution of silent gallstones in gallbladder cancer.
J Surg Oncol. 2006;93:629-632.

5/22/2012 5:32:53 PM

Commentary on
Silent Gallstones
David H. Livingston

The management of patients with asymptotic cholelithiasis has


been a classic surgical question for decades. Prior to the introduction of ultrasonography only patients with symptoms underwent
oral cholecystography and the presence of silent gallstones could
only be determined by autopsy series. Asymptomatic cholelithiasis is increasingly being diagnosed today, mainly as a result of the
widespread use of abdominal ultrasonography and other forms
of imaging for the evaluation of patients for unrelated or vague
abdominal complaints. The widespread use of computed tomography for the evaluation of abdominal pain has also added to
our knowledge about the prevalence in the population. Though
the true prevalence is unknown and difficult to determine, large
ultrasound-based studies from Europe suggest a prevalence of 19%
in female and 10% in males.1 Those studies also found that the
incidence was about 5% for new stones when the population was
studied at 5- and 10-year intervals. Overall, a reasonable estimate
cholelithiasis in most Western countries is 10% to 20% depending
upon gender with two-thirds being asymptomatic at the time of
diagnosis. Given the prevalence of stones it is no wonder why cholecystectomy is one of the most common operation in the United
States. During the open-cholecystectomy era, the riskbenefit ratio
for prophylactic cholecystectomy came down strongly on the side
of leaving asymptotic stones alone.2 As laparoscopic cholecystectomy is now entering its third decade following its introduction the
authors rightly ask whether this has changed the equation. As well
described, the authors have examined the issue from riskbenefit
and cost-effectives standpoints and found that there continues to
be no compelling evidence for performing routine cholecystectomy
for asymptomatic cholelithiasis.
A more aggressive approach leaning toward elective cholecystectomy has been suggested in certain high-risk groups. The role of
diabetes should play no role in the management of asymptomatic
gallstones. Similarly, although there is a marked increased incidence of stones in patients with sickle cell disease, the low incidence
of conversion to symptomatic stones in this group does not justify

cholecystectomy unless there is clear documentation of benefit.


Although others have suggested that patients with hematologic
diseases who had emergency cholecystectomies had increased
complication, the actual need for emergent cholecystectomy is not
increased and should not change the recommendation that asymptomatic stones should be left alone.3
Lastly, while most patients with gall bladder cancer have stones,
no causative link has ever been determined. This is not surprising
since the incidence of gall bladder cancer is exceedingly small, especially compared with the population of patients with gallstones.
Again, there is no justification for using prophylactic cholecystectomy in the general population as a cancer deterrent.
In summary, the authors have found no compelling evidence
to change the standard practice of observation for asymptomatic cholelithiasis. As with most issues general surgery, choosing
appropriate patients for surgical intervention are usually met with
superior outcomes. Despite its safety and efficacy, laparoscopic
cholecystectomy still carries a 1% risk of bile duct injury and an
overall perioperative morbidity of 5%. Laparoscopic cholecystectomy is also one of the top 10 reasons for litigation. Thus although
a diligent questioner could probably elicit gastrointestinal symptoms in anyone with asymptomatic cholelithiasis, the data tell us
that these are the type of stones that should neither be overturned
nor removed.

REFERENCES
1. Attili AF, De Santis A, Capri R, Repice AM, Maselli S. The natural history of gallstones: The GREPCO experience. The GREPCO
Group. Hepatology. 1995;21:655-660.
2. Glenn F. Silent gallstones. Ann Surg. 1981;193:251-252.
3. Walker TM, Hambleton IR, Serjeant GR. Gallstones in sickle cell
disease: Observations from The Jamaican Cohort study. J Pediatr.
2000;136:80-85.

379

PMPH_CH46.indd 379

5/22/2012 5:32:53 PM

CHAPTER 47

Acute Cholecystitis
John S. Oh

EPIDEMIOLOGY AND HISTORY

palpation of the right-upper quadrant. During deep inspiration,


palpation of a tender, inflamed gallbladder will cause the patient
to cease inspiration. This sign, along with a consistent history, is
used to diagnose cholecystitis to this day.
In 2002, Trowbridge4 conducted a comprehensive review on
the literature to find which components of the history, physical
exam, or basic laboratory measures identified patients requiring diagnostic imaging for acute cholecystitis. The authors only
included studies with a control group that did not have cholecystitis. Finally, the studies had to base the diagnosis of cholecystitis on
an adequate gold standard. Of the 195 studies identified, 17 were
included in the final study.
There was no single clinical or laboratory finding with a significantly high positive or significantly low negative likelihood
ratio to rule in or rule out the diagnosis of acute cholecystitis. In
the study by Trowbridge, Murphys sign had the highest positive
likelihood ratio at 2.8, and right-upper quadrant tenderness had
a negative likelihood ratio of 0.4. However, both 95% confidence
intervals (CI) included 1.0. Furthermore, verification bias in the
studies reviewed could have artificially elevated the rule out power
of right-upper quadrant tenderness. Murphys sign, on the other
hand, may have had a higher than estimated rule in power due to
verification bias.
The authors then concluded that it would be logical to assume
a combination of fi ndings, or clinical gestalt, would have a
higher likelihood ratio for diagnosing acute cholecystitis. To identify the impact of the clinical gestalt, the authors estimated the
likelihood ratio of diagnosing acute cholecystitis based on two
randomized trials of early versus delayed cholecystectomy. Using
estimations of the impact of clinical gestalt on findings at laparotomy, the likelihood ratio increased to 25 to 30. Unfortunately,
the available literature could not identify the specific combination
of findings that could account for this likelihood ratio.4
In 2005, Mills et al.5 examined predictive capability of various laboratory and physical examination measures to distinguish
acute cholecystitis from normal gallbladders using ultrasonic
diagnosis of acute cholecystitis. This retrospective study included
177 patients, and identified four significant predictors including

Acute cholecystitis is a common disease that affects up to 10% of


patients worldwide. Cholelithiasis accounts for 90% to 95% of all
causes of acute cholecystitis, while acalculous cholecystitis accounts
for the remaining 5% to 10%.1 While the majority of patients with
gallstones are asymptomatic, 1% to 4% will develop acute cholecystitis annually. With a prevalence of 20 million patients in the
United States with gallstones, this results in 1 million hospitalizations with 700,000 operative procedures, and an annual cost of
USD 5 billion.2,3
Over the past few decades, there have been major advancements in the use of radionuclide technology, ultrasound (US), and
laparoscopic surgery. The purpose of this chapter is to provide
evidence-based recommendations for the diagnosis and treatment
of this common disease.

PATHOPHYSIOLOGY
In the majority of cases, the pathogenesis of acute cholecystitis
results from an obstruction of the cystic duct by a gallstone. The
subsequent increase in intraluminal pressure within the gallbladder causes an acute inflammatory response. Secondary bacterial
infections may also occur with enteric organisms (most commonly Escherichia coli, Klebsiella, and Streptococcus faecalis).
Acute acalculous cholecystitis usually occurs in the setting of
prolonged critical illness. The main causes for acalculous cholecystitis are thought to be gallbladder ischemia (during periods of
shock or trauma) and biliary stasis (during prolonged fasting or
parenteral nutrition).
1. What are the clinical criteria required for the diagnosis of
acute cholecystitis?
Over 100 years ago, John Benjamin Murphy fi rst described the
eponymous Murphys sign. This sign, often used as a confirmatory physical fi nding in acute cholecystitis, is elicited upon
380

PMPH_CH47.indd 380

5/22/2012 5:33:29 PM

Acute Cholecystitis

elevated levels of alkaline phosphatase, total bilirubin, and white


blood cell count, and a clinical or ultrasonic Murphys sign. Based
on this relatively small retrospective study, the authors suggested
using these clinical predictors to determine which patients will
require emergent imaging for acute cholecystitis.
Answer: There is no single or set of clinical criteria that can
reliably predict or rule out cholecystitis. The best clinical predictor
is the clinical gestalt, using a combination of history and exam
findings. (Grade C recommendation).
2. What is the optimal imaging study for the diagnosis of acute
cholecystitis?
Imaging studies for gallbladder diseases have undergone many
changes since the oral cholecystography was introduced in 1924.
This modality remained the standard for diagnosis of acute cholecystitis for decades until US technology was introduced. Today,
US is the most commonly used imaging modality for suspected
cholecystitis. In 1994, a meta-analysis of the diagnostic performance of dynamic, real-time US showed an unadjusted sensitivity
of 94% and specificity of 78% in the diagnosis of acute cholecystitis.6 These investigators also included adjustments for verification
bias in their estimates, resulting in a decrease of sensitivity to 88%
and increase in specificity to 80%.
For acute cholecystitis, radionuclide imaging has improved
test characteristics with a sensitivity of 97%, and a specificity of
90% in the same meta-analysis.6 Finally, oral cholecystography
had a sensitivity of 63% with a specificity of 100% for acute cholecystitis. However, these studies lack portability, and include exposure to radiation.
Computed tomography (CT) scan is often used to evaluate
other abdominal pathology, and in a retrospective review of 117
patients with right-upper quadrant abdominal pain, the use of CT
was compared with US as the initial imaging modality.7 The CT
scan had inferior test characteristics compared with US, with a
sensitivity of 39% and a specificity of 93%. However, depending on
the CT findings used to diagnose acute cholecystitis, the sensitivity
can be improved to 92%, with a specificity of 97% according to a
more recent retrospective review of 75 patients.8 Anecdotally, CT
scan has been used to diagnose complications of cholecystitis such
as perforation of the gallbladder, or other pathologic conditions
that may mimic acute cholecystitis.
Answer: Radionuclide imaging is the preferred study to
diagnose acute cholecystitis based on sensitivity and specificity;
however, dynamic US is the preferred initial imaging study based
on availability and portability. (Grade B recommendation) CT
scan may be useful for diagnosing alternate conditions. (Grade D
recommendation).

MANAGEMENT
3. Should laparoscopic or open cholecystectomy be performed
in acute and complicated acute cholecystitis?
Carl August Langenbuch, a German surgeon, performed the
first series of successful open cholecystectomies in 1886, and this
remained the standard of care for 100 years. After the first laparoscopic cholecytectomy was performed in 1986, it rapidly became
the new standard of care over the next two decades with little supporting evidence.9 The Cochrane group performed a systematic

PMPH_CH47.indd 381

381

review of randomized trials comparing laparoscopic to opencholecystectomy in patients with symptomatic cholelithiasis.10
This review excluded trials of patients exclusively diagnosed with
acute cholecystitis, and only included trials where the majority of
patients (more than half) had symptomatic cholelithiasis. A total
of 38 trials were included in the analysis with over 2300 patients
in this analysis. There were no significant differences in mortality or complications to include intraoperative, minor, and bile
duct injuries. There was also no significant difference in operating
time. However, hospital stay (3.15 days, 95% CI 3.94 to 2.35)
and convalescent days (22.51 days, 95% CI 36.89 to 8.13) were
significantly shorter in the laparoscopic surgery group. It is worth
mentioning that only three studies in this review reported on convalescent days, and that none of the trials could be classified as low
risk of bias.
In the early 1970s, small incision cholecystectomy was introduced as a method to reduce the morbidity and cost of cholecystectomy.11 Another Cochrane group systematic review was performed
to compare the small incision to open-cholecystectomy.12 For
purposes of the review, a small incision cholecystectomy was arbitrarily defined as <8 cm, and could be performed through a subcostal or midline incision. This review included seven randomized
trials with a total of 571 patients. Again, none of the trials could
be classified as low bias risk. There was no overall difference in
complications or operative time. There were no common bile duct
injuries in either group. There was, however, a significantly shorter
hospital stay for the small incision group (1.97 days, 95% CI 2.56
to 1.39). None of the trials reported on mortality.
Another Cochrane review compared laparoscopic with small
incision cholecystectomy.13 This review included 13 randomized
trials with 2337 patients. As in the previous reviews, trials of
patients exclusively with acute cholecystitis were not included. The
overall methodological quality of these trials was high. There were
no differences between groups in terms of complications, convalescence, or mortality. However, the small incision cholecystectomy group had a significantly shorter operative time (16.4 min,
95% CI 8.923.8). They concluded that both minimally invasive
techniques (small incision or laparoscopic) were equivalent, and
the choice is up to individual surgeon preference.
A meta-analysis of results of laparoscopic cholecystectomy
for severe versus nonsevere acute cholecystitis was performed by
Borzellino et al.14 Severe acute cholecystitis was defined as gangrenous, empyematous, or perforated cholecystitis. All other causes
of gallbladder inflammation, to include mucoceles or hydrops,
were considered nonsevere. This study evaluated seven case series
with a total of 1408 patients, of which 469 had severe acute cholecystitis and 939 had nonsevere acute cholecystitis. There were
no randomized trials included in this review. Of the seven studies, six were retrospective and one was prospective. The relative
risk for conversion to an open cholecystectomy was found to be
3.22 (95% CI 2.484.18) in favor of nonsevere cholecystitis. The
relative risk for overall complications was 1.61 (95% CI 1.172.21),
again in favor of nonsevere cholecystitis. There was only one study
that reported on bile duct injury, with no significant differences
between the two groups, and only two studies reported on mortality, again with no significant differences. Three of the included
studies reported on local complications, with no significant differences between the groups with a relative risk of 1.16 (95%
CI 0.592.26). This particular finding was surprising, considering
the increased complexity of performing laparoscopy in the setting

5/22/2012 5:33:29 PM

382

Surgery: Evidence-Based Practice

of severe cholecystitis. This was explained by surgeons having a


lower threshold for conversion to an open procedure in severe
cases, thereby mitigating the negative consequences of laparoscopy in complex cases.
In 1998, a single-center, prospective randomized trial of laparoscopic versus open cholecystectomy for cholecystitis was published. This study enrolled and randomized 63 patients.15 The two
groups were similar in preoperative characteristics, with similar
proportions of patients with gangrene or empyema of the gallbladder. Perioperative antibiotics and management of suspected common bile duct stones were standardized in both groups. This trial
had similar operative times for both procedures, with no deaths in
either group. The laparoscopic group, however, had a significantly
shorter hospital stay (median 4 [IQR 25] vs. 6 [58] days, p = .0063).
The open group had a significantly higher rate of major complications (0 vs. 7 [23%] p = .0048) to include severe wound infections,
sepsis, and incisional hernias. In addition, the mean duration
of sick leave for those patients that were employed at the time
was significantly shorter for patients in the laparoscopic group
(13.9 vs. 30, p < .0001).
In 2005, Johansson et al.16 published a prospective, randomized trial of open versus laparoscopic cholecystectomy for
acute cholecystitis. To minimize treatment bias, the patients
and staff involved in the post operative care were blinded to the
type of operation. In addition to peri-operative antibiotics and
operative management, postoperative pain management and
sick leave were standardized between the two groups. Seventy
patients were randomized in this study, with post operative hospital stay as the primary endpoint. There were no common bile
duct injuries or mortality in either group. There was no difference in postoperative complications, pain score at discharge, or
sick leave. Operative times were shorter in the open group, and
direct costs were equivalent. The postoperative hospital stay was
significantly shorter in the laparoscopic group (median 2 days
for both groups), with more patients discharged on the day after
surgery in the laparoscopic group. Th is study showed how preconceptions regarding postoperative pain and convalescence
can influence outcomes.
Answer: Minimally invasive techniques, whether small incision or laparoscopic, are favored over open-cholecystectomy based
on shorter hospital stay, although the meta-analysis is based mainly
on studies of patients without cholecystitis. There appear to be no
differences in terms of bile duct injury or mortality, and most studies also show no differences in terms of major postoperative complications. (Grade B recommendation).
4. What should the timing of surgical intervention be?
The risks and benefits of early versus delayed cholecystectomy
for acute cholecystitis have been controversial. Complications
related to early cholecystecomy result from acute inflammation
obscuring the anatomy, whereas complications from late cholecystectomy result from fibrous adhesions. To resolve this controversy, the Cochrane group performed a review in 2006 analyzing
early versus late laparoscopic cholecystectomy for acute calculous
cholecystitis.17 The early intervention group was defined as having
surgery within 7 days of symptom onset, versus the late group,
defined as the intention to perform cholecystectomy 6 weeks after
symptom onset. After screening the literature, five studies were
included with a total of 451 patients. Although none of the studies

PMPH_CH47.indd 382

could be conducted in a double-blinded fashion, four out of five


studies were of high methodological quality.
When comparing early with late cholecystectomy, there
were no deaths in any of the trials. There were no differences
with respect to common bile duct injury, wound infection, and
bile leak. There were also no differences in terms of conversion
to open-cholecystectomy. A repeat analysis using only the highquality trials again revealed no difference in these outcomes. In
addition, the incidence of nonresolving or recurrent cholecytitis
was 17.5% in the delayed group. These patients had to undergo
emergent cholecystectomy, and the conversion to open rate was
45% in this group. Hospital stay was 4.5 days shorter on average
for the early group, and two patients in the delayed group developed cholangitis.
In many of the studies reviewed, common bile duct stones,
gallbladder perforation, or severe concommittent cardiopulmonary disease were exclusionary criteria. This should be bore in
mind whenever the riskbenefit analysis of early laparoscopic
intervention is considered.18 In addition, many of the outcomes
such as common bile duct injury had low event rates making detection of patterns difficult.
In 2004, a meta-analysis by Papi et al.19 also reviewed early
versus late cholecystectomy in acute cholecystitis. This analysis
reviewed open and laparoscopic techniques, with early surgery
performed within 7 days of symptom onset versus 6 to 12 weeks
in the delayed group. This meta-analysis found no differences in
perioperative morbidity, with the only difference being shorter
hospital stays. The difference in hospital stay was even more pronounced when only open-cholecsytectomy was considered (10.6
1.8 days in the early group versus 20.4 4.2 days in the delayed
group; p = .006).
Answer: Early intervention is preferred in acute cholecystitis based on shortened hospital stays and avoidance of readmissions for recurrent cholecystitis. However, the risks and benefits
of urgent versus delayed surgery must be weighed carefully in
patients who are at high-operative risk. The choice of procedures
in patients with common bile duct stones is important, and should
based on the experience of the surgeon performing the procedure.
(Grade A recommendation).
5. What is the role of nonoperative management?
Only one randomized controlled trial compared cholecystectomy
(open or laparoscopic) with nonoperative management.20 A total
of 64 patients were randomized to delayed surgery versus observation. The delayed surgery group was operated on at a median of
3.6 months after randomization, and 4/31 patients (13%) refused
operation based on freedom from symptoms. Of the patients
randomized to the observation group, 10/33 patients underwent
surgery (30% failure rate). A significantly greater proportion of
patients randomized to the operative group underwent cholecystectomy compared with the observation group after 8 years of
follow-up (87% vs. 30%, p < .0001).
There were no significant differences between the groups in
either gallstone-related complications or admissions for pain (6/31
[19%] with cholecystectomy vs. 12/33 [36%] with observation, p = .16).
Gallstone-related complications included acute cholecystitis, pancreatitis, and common bile duct stones. There were no deaths in
either group related to gallstone disease. Three out of 27 patients
that underwent operation had a major operative complication, to

5/22/2012 5:33:29 PM

Acute Cholecystitis

include one common bile duct injury that required a biliaryenteric anastamosis, and one patient who was initially randomized to the nonoperative group that had two major complications
(11% vs. 10%, p = 1.00).
Based on this single trial, it appears that a large proportion of
patients with acute cholecystitis can be managed without surgery.
It bears mentioning that this trial excluded patients with severe
comorbid diseases, as well as patients with evidence of gallbladder perforation or gangrene. The median age of patients in the
observation group was also relatively young compared with the
surgery group (47 and 58 years, respectively). While most recent
randomized controlled trials on acute cholecystitis focused on the
timing of surgery, this trial offered some insight into the tradeoffs
between operative versus nonoperative intervention.21
Answer: Nonoperative management of patients with acute
cholecystitis has significant tradeoffs in terms of morbidity,
with a large proportion of patients eventually requiring surgery
despite best practices of nonoperative management. (Grade D
recommendation).
6. Do percutaneous cholecystostomy tubes improve outcomes
in high-risk surgical patients?
A systematic review of percutaneous cholecystostomy procedures
for acute cholecystitis was performed by Winbladh et al.22 in 2009.
The vast majority of the studies evaluated were retrospective case
series, and inclusion criteria varied widely. Most of the case series
defined a successful cholecystostomy procedure as a decrease
in fever, symptom resolution, and reduction in white blood cell
count. In these terms, percutaneous cholecystostomy was successful in 85% of patients.
Morbidity related to the procedure itself was not well-defined
in most case series, and there was wide variability in the quality of reporting. Catheter slippage was reported in 9% of patients;
however, this may be under-reported as follow-up was mainly
restricted to in hospital stays. Procedure-related mortality was
0.36%, and all-cause mortality in the high-risk patient population
was 15.4%.
Over 40% of patients eventually underwent cholecystectomy
with an overall operative mortality rate of 2.08%. None of the prospective studies in this review directly compared cholecystostomy
with cholecystectomy, and it was difficult to make comparisons
among many of the case series. As a result, no definitive conclusions can be drawn. It does appear that the overall mortality rate
for high-risk patients with cholecystitis is high. However, this may
be a reflection of the overall poor prognosis.
One prospective, randomized trial evaluated percutaneous
cholecystostomy followed by early laparoscopic cholecystectomy
versus medical management followed by late cholecystectomy.23
Seventy high-surgical-risk patients were randomized. Patients in
the first group were treated with a percutaneous cholecystostomy
within 8 h of admission followed by laparoscopic cholecystectomy within 96 h if they achieved symptom resolution and an
APACHE II score <12. In the second group, delayed cholecystectomy was performed 8 weeks after recovery.

PMPH_CH47.indd 383

383

The group undergoing percutaneous cholecystostomy and


early laparoscopic cholecystectomy had reduced total hospital stays
(mean 5.3 days vs. 15.2 days, p = .001), and earlier resolution of
symptoms (mean 15 vs. 55 h, p = .001). The adverse events related to
the percutaneous catheter were minor. There were no procedurerelated deaths in either group. Based on this single study, percutaneous cholecystostomy may be beneficial in terms of symptom
resolution and decreased length of hospital stay. The conclusions
drawn from this study, however, are biased due to nonblinding,
lack of an intent-to-treat analysis, and small sample size.
Answer: Percutaneous cholecystostomy tubes may offer
symptomatic resolution in high-risk patients with relatively low
procedural-related complications. (Grade D recommendation).
7. What is the role of antibiotics in acute cholecystitis?
There are no randomized controlled trials comparing antibiotics
with no antibiotics in patients with acute cholecystitis. Although
acute cholecystitis is primarily an inflammatory process, systemic
antibiotics are routinely prescribed with very little evidence to
support their efficacy.
A study of control subjects versus patients with gallstones was
published in 1996 describing the yield of positive bile cultures for a
variety of biliary tract conditions.24 None of the control subjects had
positive cultures, whereas 46% of patients with acute cholecystitis
had positive cultures. That percentage increased to 58% with the
presence of common bile duct stones, and 98% with cholangitis.
The most frequent isolates identified were E. coli, Streptococcus,
Klebsiella, and Enterobacter. Current prophylactic regimens are
tailored against these pathogens. However, with obstruction of
the cystic duct, bacterial infection is thought to be a secondary,
and not primary, process in acute cholecystitis. Furthermore,
there is no evidence that antibiotics penetrate into the bile in
cholecystitis.
One retrospective study of 302 patients in 1975 found no
benefit from antibiotics in terms of local complications such as
empyema or pericholecystic abcess formation. However, there did
appear to be a lower incidence of wound infection and sepsis with
periopertive use of broad spectrum antibiotics.25 The patients that
were most susceptible to infectious complications were >60 years
of age with a history of diabetes.
Various randomized controlled trials have evaluated differing antibiotic regimens in acute cholecystitis and cholangitis,
although no conclusions can be drawn regarding duration of antibiotics, or the role of routine biliary tract cultures.26-28 A recent
consensus statement recommended guiding antibiotic therapy
based on severity of disease and changing empirically administered antimicrobials according to identified causative organisms
and susceptibility testing.29
Answer: Perioperative antibiotics may decrease the incidence
of wound infection in cholecystectomy. Empiric antibiotics should
cover the most common organisms based on local pathogen profi les, and tailored based on causative organisms and susceptibility testing. Antibiotic use should be based on severity of disease.
(Grade C recommendation).

5/22/2012 5:33:29 PM

384

Surgery: Evidence-Based Practice

Clinical Question Summary


Question

Answer

Level of
Evidence

Grade of
Recommendation

1 What are the clinical


criteria required
for the diagnosis of
acute cholecystitis?

There is no single or set of clinical criteria that


can reliably predict or rule out cholecystitis.
The best clinical predictor is the clinical
gestalt.

3, 2b

4, 5

2 What is the optimal


imaging study for the
diagnosis of acute
cholecystitis?

Radionuclide imaging is the preferred study based


on sensitivity and specificity; however, dynamic
US is the preferred initial imaging study based
on availability and portability.

2a, 3b

6-8

3 Should laparoscopic
or open
cholecystectomy be
performed in acute
and complicated
acute cholecystitis?

Minimally invasive techniques are favored over


open cholecystectomy based on shorter
hospital stay. The systematic reviews are
mainly based on mixed population studies of
symptomatic cholelithiasis and cholecystitis.

1a, 3a, 2b

10, 12-16

4 What should the


timing of surgical
intervention be?

Early intervention is preferred in acute


cholecystitis. However, the risks and benefits
of urgent versus delayed surgery must be
weighed carefully in patients who are high
operative risk.

1a

17, 19

5 What is the role


of nonoperative
management?

Nonoperative management of patients with acute


cholecystitis has significant tradeoffs in terms
of morbidity.

1b

20

6 Do percutaneous
cholecystostomy
tubes improve
outcomes in highrisk surgical patients?

Percutaneous cholecystostomy tubes may


offer symptomatic resolution in high-risk
patients with relatively low procedure-related
complications.

3a, 1b

22, 23

7 What is the role of


antibiotics in acute
cholecystitis?

Perioperative antibiotics can help decrease


the incidence of wound infection in
cholecystectomy. Empiric antibiotics should be
tailored based on the causative organism and
susceptibility testing. Antibiotic use should be
based on severity of disease.

3a, 2b

25-28

REFERENCES
1. Kimura Y, Takada T, Karawada Y, et al. Definitions, pathophysiology, and epidemiology of acute cholangitis and cholecystitis:
Tokyo Guidelines. J Hepatobiliary Pancreat Surg. 2007;14:15-26.
2. Indar A, Beckingham I. Acute cholecystitis. Br Med J. 2002;325:
639-643.
3. Riall TS, Zhang D, Townsend CM, Jr., et al. Failure to perform
cholecystectomy for acute cholecystitis in elderly patients is associated with increased morbidity, mortality, and cost. J Am Coll
Surg. 2010;210(5):668-677.
4. Trowbridge RL, Rutkowski NK, Shojania KG. Does this patient
have acute cholecystitis? JAMA. 2003;289(1):80-86.
5. Mills LD, Mills T, Foster B. Association of clinical and laboratory variables with ultrasound findings in right upper quadrant
abdominal pain. South Med J. 2004;97:155-161.
6. Shea JA, Berlin JA, Escarce JJ, et al. Revised estimates of diagnostic test sensitivity and specificity in suspected biliary tract disease.
Arch Intern Med. 1994;154:2573-2581.

PMPH_CH47.indd 384

References

7. Harvey RT, Miller WT, Jr. Acute biliary disease: Initial CT and
follow up US versus initial US and follow up CT. Radiology. 1999;
213:831-836.
8. Bennett GL, Rusinek H, Lisi V, et al. CT findings in acute gangrenous
cholecystitis. AJR Am J Roentgenol. 2002;178:275-281.
9. NIH Consensus. 1993 NIH Consensus Development Panel on
Gallstones and Laparoscopic Cholecystectomy. JAMA. 1993;
269(8):1018-1024.
10. Keus F, de Jong J, Gooszen HG, Laarhoven CJHM. Laparoscopic versus open cholecystectomy for patients with symptomatic cholecystolithiasis. Cochrane Database of Systematic Reviews. 2009;Issue 1.
11. Goco IR, Chambers LG. Mini-cholecystectomy and operative
cholangiography. A means of cost containment. Am Surg. 1983;
49(3):143-145.
12. Keus F, de Jong J, Gooszen HG, Laarhoven CJ. Small incision versus open cholecystectomy for patients with symptomatic cholecystolithis. Cochrane Database of Systematic Reviews. 2009;Issue 1.
13. Keus F, de Jong JA, Gooszen HG, van Laarhoven CJ. Laparoscopic versus small-incision cholecystectomy for patients with

5/22/2012 5:33:29 PM

Acute Cholecystitis

14.

15.

16.

17.

18.

19.

20.

21.

PMPH_CH47.indd 385

symptomatic cholecystolithiasis. Cochrane Database Systematic


Reviews. 2009;Issue 1.
Borzellino G, Sauerland S, Minicozzi AM, et al. Laparoscopic
cholecystectomy for severe acute cholecystitis. A meta-analysis
of results. Surg Endosc. 2008;22(1):8-15.
Kiviluoto T, Sirn J, Luukkonen P, Kivilaakso E. Randomised
trial of laparoscopic versus open cholecystectomy for acute and
gangrenous cholecystitis. Lancet. 1998;351(9099):321-325.
Johansson M, Thune A, Nelvin L, et al. Randomized clinical trial
of open versus laparoscopic cholecystectomy in the treatment of
acute cholecystitis. Br J Surg. 2005;92(1):44-49.
Gurusamy KS, Samraj K. Early versus delayed laparoscopic cholecystectomy for acute cholecystitis. Cochrane Database of Systematic Reviews. 2006;Issue 4: CD005440. DOI: 10.1002/14651858.
CD005440.pub2.
Yamashita Y, Takada T, Kawarada Y, et al. Surgical treatment of
patients with acute cholecystitis: Tokyo Guidelines. J Hepatobiliary Pancreat Surg. 2007;14(1):91-97.
Papi C, Catarci M, DAmbrosio L, et al. Timing of cholecystectomy for acute calculous cholecystitis: A meta-analysis. Am J
Gastroenterol. 2004;99(1):147-155.
Vetrhus M, Sreide O, Nesvik I, Sndenaa K. Acute cholecystitis: Delayed surgery or observation. A randomized clinical trial.
Scand J Gastroenterol. 2003;38(9):985-990.
Fialkowski E, Halpin V, Whinney RR. Acute cholecystitis. Clin
Evid (Online). December 4, 2008. pii: 0411.

385

22. Winbladh A, Gullstrand P, Svanvik J, et al. Systematic review


of cholecystostomy as a treatment option in acute cholecystitis.
HPB (Oxford). 2009;11(3):183-193.
23. Akyrek N, Salman B, Yksel O, et al. Management of acute calculous cholecystitis in high-risk patients: Percutaneous cholecystotomy followed by early laparoscopic cholecystectomy. Surg
Laparosc Endosc Percutan Tech. 2005;15(6):315-320.
24. Csendes A, Burdiles P, Maluenda F, et al. Simultaneous bacteriologic assessment of bile from gallbladder and common bile duct
in control subjects and patients with gallstones and common
duct stones. Arch Surg. 1996;131(4):389-394.
25. Kune GA, Burdon JG. Are antibiotics necessary in acute cholecystitis? Med J Aust. 1975;2(16):627-630.
26. Muller EL, Pitt HA, Thompson JE, Jr., et al. Antibiotics in
infections of the biliary tract. Surg Gynecol Obstet. 1987;165(4):
285-292.
27. Chacon JP, Criscuolo PD, Kobata CM, et al. Prospective randomized comparison of pefloxacin and ampicillin plus gentamicin in
the treatment of bacteriologically proven biliary tract infections.
J Antimicrob Chemother. 1990;26(Suppl B):167-172.
28. Thompson JE, Jr., Bennion RS, Roettger R, et al. Cefepime
for infections of the biliary tract. Surg Gynecol Obstet.
1993;177(Suppl):30-34; discussion 35-40.
29. Yoshida M, Takada T, Kawarada Y, et al. Antimicrobial therapy
for acute cholecystitis: Tokyo Guidelines. J Hepatobiliary Pancreat Surg. 2007;14(1):83-90.

5/22/2012 5:33:29 PM

CHAPTER 48

Common Bile Duct Stones


Adrian W. Ong and Charles F. Cobb

INTRODUCTION

further review. Nonsystematic review articles, expert opinion


without explicit critical appraisal, noncontrolled case series were
excluded from forming evidence-based recommendations.
Suitable articles were graded according to the levels of
evidence developed by the Oxford Centre for Evidence-Based
Medicine, and grades of recommendations formulated based on
the same system. The two authors reviewed the suitable articles
independently initially and a consensus was reached as to building evidence-based recommendations.

Common bile duct stones (CBDS) are a common problem faced by


general surgeons. The diagnosis and surgical treatment of CBDS
has evolved in the past three decades, with the advent of minimally invasive methods for the treatment of biliary tract problems. We reviewed the literature in order to answer commonly
encountered questions.

QUESTIONS TO BE ADDRESSED

1. Are less invasive methods such as magnetic resonance cholangiopancreatography (MRCP) and endoscopic ultrasound
(EUS) as accurate as endoscopic retrograde cholangiopancreatography (ERCP), intraoperative cholangiography or surgery
in the detection of CBDS?

1. Are less invasive methods such as magnetic resonance cholangiopancreatography (MRCP) and endoscopic ultrasound
(EUS) as accurate as endoscopic retrograde cholangiopancreatography (ERCP), intraoperative cholangiography or surgery
in the detection of CBDS?
2. What is the best approach to evaluate and treat patients with
suspected CBDS?
3. If choledochotomy is done during common bile duct exploration
(CBDE), is T-tube drainage (TT) necessary?
4. What is the optimal method of biliary decompression and
timing of decompression in patients with acute cholangitis due
to CBDS?
5. Is cholecystectomy necessary after endoscopic treatment of
CBDS in patients both with and without gallstones (GS)?

With the widespread use of laparoscopic cholecystectomy (LC),


there has been more attention paid to diagnosing CBDS preoperatively. Detection of CBDS preoperatively has several perceived
advantages: it may allow for better preoperative planning with
management of patient expectations, possibly shorten operative time by eliminating intraoperative common bile duct (CBD)
evaluation, and may facilitate treatment planning in the event of
laparoscopic and endoscopic failure to extract CBDS. ERCP, the
established modality for detection of CBDs preoperatively, has
been associated with significant risks and complications. In a
large registry of more than 11,000 ERCPs, the postoperative complication rate was 9.8%, with low-volume centers having a higher
complication rate compared with high-volume centers. In this
large database, pancreatitis occurred in 2.7%, bleeding in 0.9%,
and perforation in 0.3%.1 Less invasive modalities for preoperative
CBDS detection might avert some of these complications. MRCP
and EUS have emerged as the two most popular modalities for the
detection of CBDS and will be the focus of this review. We examined studies comparing the performance of MRCP and EUS using
ERCP, IOC, surgery as the reference standard. Papers where clinical follow-up and/or where transabdominal ultrasound served as
a reference standard were excluded.

METHODS
An online search of PubMed and the Cochrane library was
performed for all English language articles on human subjects
from 1998 up to December 2010, using the terms common bile
duct stones, endoscopic retrograde cholangiopancreatography,
magnetic retrograde cholangiopancreatography, endoscopic
ultrasound, T-tube, and common bile duct exploration. Abstracts were reviewed and selected for suitability with regard to the
questions being examined, and the relevant articles obtained for
386

PMPH_CH48.indd 386

5/22/2012 5:34:10 PM

Common Bile Duct Stones

387

MRCP

2. What is the best approach to evaluate and treat patients with


suspected CBDS?

MRCP is highly sensitive and specific in general when compared


with the other invasive imaging modalities.2-7 However, its performance in the detection of small stones may be inferior to
these other imaging modalities. Zidi et al.8 found that MRCP was
not sensitive enough for the detection of small CBDS (<7 mm).
Others9,10 have similarly found decreased sensitivity for small
stones (<5 mm) with MRCP compared with EUS. In a systematic
review with meta-analysis, Romagnuolo et al.11 compared the performance of MRCP with the gold standard, which included surgery, IOC, percutaneous transhepatic cholangiography (PTHC),
computed tomography, ERCP, and EUS. They found that MRCP
was less sensitive in the detection of CBDS (sensitivity 92%) or
differentiating between malignant from benign disease (sensitivity 88%), but had excellent sensitivity (9798%) in detecting the
presence of obstruction or the level of obstruction. It was recommended that the role of MRCP should be further assessed through
large outcome studies in selected patient samples.

As preoperative ERCP is an invasive procedure with risks and


complications outlined above, and as laparoscopic methods of
CBD clearance are gaining acceptance among surgeons, there is
continued debate as to the optimal approach to patients with suspected CBDS. The general aims of any approach are to minimize
complications, reduce redundant procedures, shorten in-hospital
length of stay and avoiding long-term procedural complications.
Several issues are debated in the current literature: in patients
with suspected CBDS, should the endoscopic or surgical approach
be utilized first? If surgery is to be utilized first, should ERCP be
done intraoperatively or postoperatively for retained stones, or
should it be considered only for laparoscopic stone extraction failures. Should MRCP or EUS be employed first prior to any invasive
procedure?
Clayton et al.20 performed a meta-analysis of 12 randomized
trials with both open and laparoscopic methods, and found that
there were no differences in morbidity, mortality, duct clearance
success rate or need for additional procedures between the endoscopy (ERCP) and surgery versus the surgery alone group. Martin et al.21 similarly in a meta-analysis separately analyzed open
surgery and laparoscopic surgery. Open surgery was superior to
ERCP for CBD clearance, but there were no differences in morbidity or mortality rates between open surgery and ERCP. When the
laparoscopic first approach was compared with both preoperative
and postoperative ERCP, there were no differences in morbidity
or mortality, although single-stage treatment with LC followed
by intraoperative ERCP was a safer approach with less morbidity
compared with preoperative ERCP in one randomized controlled
trial.22 In mild-to-moderate gallstone pancreatitis, a randomized
trial demonstrated that if a surgery-first approach was utilized,
postoperative ERCP would be necessary in only 24% of patients.23
However, it should also be noted that in another randomized trial,
when laparoscopic CBDE (LCBDE) was done instead of postoperative ERCP, although long-term complication rates were low, there
was a higher incidence of postoperative bile leak (6/41 vs. 0/45).24 In
general, LCBDE is safe: Riciardi et al.25 found that of 346 patients
who had LCBDE in whom 78% had transcystic exploration (TC)
and 22% had laparoscopic choledochotomy (LCD), 95.6% had no
long-term biliary complications after a mean follow-up period of
43 months. Possible obstacles to a wider adoption of LCBDE by
surgeons include the need to acquire advanced laparoscopic skills
and the timely availability of the appropriate instruments in the
emergency setting.
The role of MRCP and EUS in the approach to patients with
suspected CBDS has been evaluated in several studies: For MRCP,
a policy of selective imaging based on risk stratification for CBDS
appears to have reduced redundant procedures and preserved a
low rate of missed CBDS (<1.5%).26 In another study, the selective
use of MRCP based on risk stratification of CBDS reduced the
potential negative ERCP rate from 48% to 16%.27 That ERCP
should be selectively done based on clinical assessment is highlighted by Sharma et al.28: Of 200 consecutive ERCPs, 102 (51%)
had positive pathology. If there were more than one indication to
do ERCP based on clinical, ultrasound findings, and liver serum
profi le, the yield was >85%, while if there was one indication, the
yield was 25%.
The use of EUS preoperatively has also reduced the need for
ERCP. A systematic review with meta-analysis of four randomized

EUS
EUS demonstrates excellent sensitivity and specificity in the detection of CBDS as compared with ERCP.12-14 Its performance may
even better than ERCP as suggested by Karakan et al.13 and Ney
et al.14 mainly due to its ability to detect small stones (<5 mm). A
consensus panel composed of sonographers has deemed EUS to be
as sensitive and more specific than ERCP in detection of CBDS.15
A systematic review by Tse et al.16 concluded that EUS was safe
and accurate in the detection of CBDS with Level 2a evidence.

MRCP VERSUS EUS


In a systematic review, with seven randomized controlled trials,
EUS was found to be more sensitive than MRCP in the detection
of CBDS (151/163 vs. 133/163, 93% vs. 82%; odds ratio 0.34, 95%
confidence interval [CI] 0.170.70), but similar in specificity and
overall accuracy. The authors concluded that both tests were reliable low-risk substitutes for diagnostic ERCP.17 Of the seven trials,
only one showed a difference in sensitivity.18 In this trial, EUS was
more sensitive than MRCP in the detection of choledocholithiasis
(80% vs. 40%). The overall accuracy of MRCP for any abnormality
was 61% (95% CI 0.410.78) compared with 89% (CI 0.720.98) for
EUS. Kondo et al.9 found that for small stones, EUS was more sensitive than MRCP. Another systematic review has found MRCP to
be less sensitive than EUS for CBDS detection.19

Recommendations
In the detection of CBDS, MRCP is reliable, but is less sensitive than ERCP particularly for small calculi (5 mm). (Grade B
recommendation)
EUS is as sensitive and specific compared with ERCP. (Grade A
recommendation)
EUS is a more invasive procedure than MRCP, but it is more
sensitive than MRCP particularly for small calculi. (Grade B
recommendation)

PMPH_CH48.indd 387

5/22/2012 5:34:10 PM

388

Surgery: Evidence-Based Practice

trials of EUS-guided ERCP versus ERCP alone concluded that the


use of EUS-guided ERCP reduced the need for ERCP in 67% of
patients, and also reduced the overall complication rate, and also of
post-ERCP pancreatitis (relative risk = 0.21 [95% CI 0.060.85]).29
In a randomized trial comparing the EUS-guided approach with
an ERCP-first approach, the use of EUS potentially would have
spared 75% of patients from ERCP at the expense of 25% unnecessary EUS examinations.30 Other randomized trials have supported
using EUS before ERCP with reduction in complication rates and
higher rates of successful initial biliary imaging.31,32
Because both MRCP and EUS have the potential to limit
unnecessary ERCPs, the choice between the two tests may be
determined by several practical factors: EUS is invasive, usually
requires patient sedation, relies on the availability of specialized
practitioners, is user-dependent, but logically can be combined
with ERCP at the same endoscopy session if indicated, thereby
improving efficiency. MRCP on the other hand is not very userdependent, is noninvasive, but requires a separate endoscopy session if preoperative CBD clearance is contemplated.
Based on the above data, a strategy using routine preoperative ERCP would result in many unnecessary invasive procedures.
Some studies therefore support the selective use of ERCP based on
the pretest probability of having CBDS,27,33 with the patients with
the highest pretest probability proceeding directly to ERCP preoperatively. However, some randomized trials where the prevalence
of CBDS was high (>70%) found that a surgery-first approach was
just as efficient as an endoscopy-first approach with similar complication rates.34-37 Williams et al.38 has recommended (Grade B
recommendation) that in patients with low or uncertain suspicion
of CBDS, EUS, or MRCP be used as the initial diagnostic modality
instead of ERCP. They also recommended against using ERCP as a
purely diagnostic modality if the clinician was not confident that
CBDS was present.

Recommendations
Routine preoperative ERCP for suspected CBDS is not
recommended.
If preoperative imaging is contemplated, EUS or MRCP
should be used in place of ERCP if there is not a high likelihood of
CBDS. (Grade B recommendation)
In patients with suspected CBDS, a surgery-fi rst strategy
(LC + LCBDE or open CBDE if indicated) is equivalent to an
endoscopy-first strategy in terms of safety and efficacy. The surgeryfirst strategy may shorten the hospital length of stay and reduce
redundant procedures. (Grade A recommendation)
3. If choledochotomy is done during common bile duct exploration (CBDE), is T-tube drainage (TT) necessary?
The insertion of a T-tube after choledochotomy and stone extraction is widely practiced. The objectives of TT are purportedly to
prevent uncontrolled bile leakage from the choledochotomy, and to
facilitate subsequent removal of retained stones. However, complications involving T-tubes have been described. In a retrospective
study, Wills et al.39 found that of 274 patients with T-tubes, 42 (15%)
had a total of 60 complications. The median duration of TT was
16 days in complicated T-tube insertions and 14 days in uncomplicated T-tube insertions. Complications included acute cholangitis, fluid and electrolyte problems and leak when the T-tube was in
situ; prolonged fistula, localized pain, and biliary peritonitis when

PMPH_CH48.indd 388

T-tubes were removed. Complications were similar between open


and laparoscopic cases, and resulted in two deaths.
Primary closure of the CBD (PC) has therefore been proposed
as a safe and efficient alternative to TT. A retrospective series of
100 LCD with PC found that PC after CBDE had a low rate of biliary complications: documented bile leak requiring laparoscopic
re-intervention occurred in only two patients, one from the PC
suture line and the other from the cystic bed.40
In a meta-analysis of randomized trials of PC versus TT after
open CBDE, Gurusamy and Samraj41 found no differences in bile
peritonitis, bile fistula, bile leak requiring reoperation, residual
stones and perioperative mortality. When PC versus TT were
compared after LCD, several randomized controlled trials found
no differences in biliary complications, with shorter hospital stays
and operative time seen in the primary closure groups. The size of
the CBD was not a factor in these trials, being equivalent in the
PC and TT groups.42-44
Some authors have advocated using TT in certain situations
when evidence of cholangitis with pus in the CBD is seen, or when
more than four or five stones are cleared.45,46 Alternatively, in these
situations, primary closure after placement of a biliary stent could
be a simpler option than TT.47
The abovementioned trials suggest that after verification of
duct clearance intraoperatively, primary duct closure with or
without biliary stenting is safe compared with TT. Long-term
follow-up of patients after primary closure or TT are needed to
address the incidence of biliary strictures.

Recommendation
In open or laparoscopic surgery, after verification of duct clearance intraoperatively, primary closure of the choledochotomy
with or without biliary stenting is a safe alternative to TT with no
differences in complication rates. Routine T-tube placement after
choledochotomy is not necessary. (Grade A recommendation)
4. What is the optimal method of biliary decompression and
timing of decompression in patients with acute cholangitis due
to CBDS?
The mortality rate of acute cholangitis due to CBDS is approximately 1% to 5%.48-50 Bile and serum endotoxin levels correlate
with the presence of the components of Charcots triad, and biliary decompression is effective as it has been shown to promptly
decrease bile and serum endotoxin levels.50
Whether surgery or endoscopic drainage should be performed
was addressed by Lai et al.51 in a randomized trial: 82 patients were
randomized at the time of ERCP to surgery or endoscopic sphincterotomy (ES). In this study, however, surgery was undertaken after
a mean of 2 h after randomization whereas in the endoscopic group,
ES was done at the same setting as ERCP. The findings were that
endoscopic drainage with ES was associated with lower mortality and shorter duration of ventilatory support. Mode of drainage
was an independent predictor of mortality in their analysis, along
with serum albumin, creatinine, leukocyte count, platelet count,
age, serum urea nitrogen, and concomitant medical problems. The
authors concluded that urgent endoscopic drainage should be considered for patients with adverse prognostic factors. Another study
involving only 22 patients with cholangitis randomized patients
after emergency ERCP. This study failed to show any advantage
of endoscopic decompression in terms of immediate mortality or

5/22/2012 5:34:11 PM

Common Bile Duct Stones

morbidity.52 That endoscopic drainage is the preferred method


over surgery is also noted by a systematic reviews and a consensus
conference38,53 as well as nonrandomized studies.48,49,54,55
There is no high-level evidence in the literature comparing the
effectiveness and safety of percutaneous transhepatic biliary drainage (PTBD) to endoscopic and surgical drainage. Pessa et al.56 studied 42 patients who underwent percutaneous transhepatic drainage
for acute cholangitis. Only 12 patients with cholangitis were due to
bile duct stones. All were successful despite a 17% incidence of nondilated ducts. Sepsis began to resolve in 22 of 24 after 24 h. There
was a 7% complication rate. PTBD is considered an alternative
option if endoscopic drainage is not possible. A consensus conference has considered both these modes of drainage (endoscopic and
PTBD) as preferred therapies over open surgery.53
The timing of biliary decompression has not been clearly
established in the literature. Rather, the literature has focused on
identifying risk factors for clinical deterioration. Yeung et al.57
in a study of 171 patients with cholangitis, found 31 that did not
respond to conservative measures after 6 h and underwent emergent biliary drainage. Logistic regression demonstrated five factors: age > 75 years, history of smoking, prothrombin time, size
of the CBD, and blood glucose as predictors for need for emergent
ERCP. The authors concluded that age > 75 years and smoking history were important factors that predicted failure of conservative
management, and that this subset of patients could benefit from
emergent intervention before deterioration. Another prospective
study of 142 consecutive patients with acute cholangitis found that
31 (21.8%) patients required emergency ERCP, with four factors
heart rate > 100/min, albumn < 30 g/dL, bilirubin > 50 mcmol/L,
and prothrombin time > 14 sthat were associated with failure
of medical treatment. However, the timing of emergency ERCP
was not explicitly stated in their paper.58 Another retrospective
study found that 13 patients who did not respond to antibiotics
after 24 h did not have complications if they were drained <72 h.
On the other hand, 12 patients where antibiotics had failed and
who were not drained until >3 days later developed a 33% rate
of septic complications. The authors conclude that urgent biliary
decompression was indicated in patients who do not respond early
(<24 h) to antibiotics.59 At the Tokyo consensus conference, no
evidence-based recommendations were produced with regard to
timing of drainage.53

Recommendations
In patients with acute cholangitis, urgent endoscopic biliary decompression is preferred over surgery. (Grade B recommendation)
There is insufficient evidence to recommend a specific time
period where urgent biliary decompression should occur.
5. Is cholecystectomy necessary after endoscopic treatment of
CBDS in patients both with and without gallstones (GS)?
There are differences of opinion among authors regarding the need
to remove the gallbladder (GB) after ERCP/endoscopic sphincterotomy (ES) in retrospective studies: In a large series of 371 patients
with both GS and CBDS who underwent endoscopic extraction,
Saito et al.60 found that acute cholecystitis occurred in 5.9%, and
recurrent CBDs in 9.7%, with a mean duration of follow-up of 7.7
years. They advocated that cholecystectomy may not always be
necessary. Kwon et al.61 found that in 146 patients followed-up for

PMPH_CH48.indd 389

389

a mean of 24 months after endoscopic removal of CBDS, the overall rate of cholecystectomy was 4.8%: it was 6.8% (4/59) in patients
with GS and 3.4% (3/87) in patients without GS. The authors recommended that cholecystectomy be reserved for patients with
symptoms. Other nonrandomized studies similarly concluded
that in elderly patients, postponement of surgery after ERCP and
stone clearance in asymptomatic patients was justified.62,63
On the other hand, in a systematic review, McAlister et al.64
examined the role of cholecystectomy after ES. The authors recommended proceeding with cholecystectomy due to the higher rate
of biliary complications in patients managed expectantly. Method
of cholecystectomy or patient ASA class did not seem to influence
outcome. Boerma et al.65 randomized patients with proven GS to
expectant management versus laparoscopic cholecystectomy after
ES. The wait-and-see group had a 47% incidence of recurrent biliary complications versus 2% in the cholecystectomy group, with
a median follow-up time of 30 months (range 1567 months). In
terms of higher-risk patients, Lau et al.66 similarly randomized
older patients (>60 years) with proven GS and found that at 5
years, 5.8% of patients in the cholecystectomy group versus 25.4%
in the group with GB in situ had recurrent biliary events. Taragona
et al.67 randomized 100 high-risk patients to open surgery with
CBDE if needed versus ERCP with ES only, and found that survival
at 1 year was similar, with the surgery group developing significantly less biliary complications (6% vs. 12%). In a retrospective
study, octogenarians who underwent ERCP/ES without LC had
a significantly higher rate of recurrent biliary events compared
with those with underwent ERCP/ES and LC (48% vs. 10%) with a
mean follow-up of 126 months.68 The authors recommended that
age not be the sole factor in selecting patients for elective LC after
ERCP/ES. Williams38 reached a similar conclusion in another systematic review, recommending cholecystectomy for all patients
with CBDS and symptomatic gallbladder stones unless there are
specific reasons for considering surgery inappropriate. (Grade B
recommendation)
On the other hand, in patients with no GS in the GB in situ,
the evidence for elective cholecystectomy is less convincing. Ando
et al.69 followed up 1042 patients prospectively with a median
follow-up of 7.5 years, and found that patients with acalculous
gallbladders had a lower risk of recurrent stones when cholecystectomy was not performed than patients with calculous gallbladders (11.3% vs. 23.9% in 15 years, relative risk = 2.16, 95% CI
1.213.87). In a retrospective study of 100 patients who underwent
ERCP with GB in situ, 28% developed biliary complications. In
patients with GS, 13 of 52 (25%) had acute cholecystitis versus
4 of 48 (8.3%) of those without GS (p = .02). However, the presence of GS was not an independent risk factor for predicting acute
cholecystitis in multivariate analysis in this study.70 Other studies involving long-term follow-up after ES have also shown that
the presence or absence of the GB did not affect the likelihood
of recurrent CBDS, and that acute cholecystitis was uncommon
when there were no GS in the GB in situ.71-74 In a consensus conference, Nagino et al.53 concluded that patients with acalculous
gallbladders need not have cholecystectomy after sphincterotomy
as the incidence of cholecystitis was low, about 1%. Of note, the
recurrent stones observed by some of these studies71,73 were mostly
of the brown pigment type rather than cholesterol stones, raising
the possibility that these were primary bile duct stones related to
bile stasis and bacterial infection of the bile duct.75 Thus, prophylactic cholecystectomy would not necessarily be of benefit.

5/22/2012 5:34:11 PM

390

Surgery: Evidence-Based Practice

Recommendation
In patients with proven gallstones, prophylactic cholecystectomy
should be offered to patients after ES unless there are specific
unfavorable patient-related factors. (Grade A recommendation)

In patients with no gallstones after ES, acute cholecystitis is


uncommon, and recurrent biliary complications may be a result
of primary bile duct stones. Routine cholecystectomy therefore
may not be necessary. (Grade C recommendation)

Clinical Question Summary


Question

Answer

Level of
Evidence

Grade of
Recommendation

1 Are less invasive methods


such as magnetic resonance
cholangiopancreatography
(MRCP) and endoscopic
ultrasound (EUS) as accurate
as endoscopic retrograde
cholangiopancreatography
(ERCP), intraoperative
cholangiography or surgery in
the detection of CBDS?
2 What is the best approach to
evaluate and treat patients
with suspected CBDS?

In the detection of CBDS, MRCP is


reliable, but is less sensitive than ERCP
particularly for small calculi (5 mm)
EUS is as sensitive and specific compared
to ERCP
EUS is a more invasive procedure than
MRCP, but it is more sensitive than
MRCP particularly for small calculi

2b

2-11, 76, 77

1b

12-19, 29

2b

9, 17-19, 78

Routine preoperative ERCP is not


recommended. If preoperative imaging
is contemplated, EUS or MRCP should
be used in place of ERCP if there is not
a high likelihood of CBDS
In patients with suspected CBDS, a
surgery-first strategy (LC + LCBDE or
open CBDE if indicated) is equivalent
to an endoscopy-first strategy in terms
of safety and efficacy. The surgery-first
strategy may shorten hospital length of
stay and reduce redundant procedures

2b

26-33, 85,
88

1b

20-24,
34-38, 79,
80, 81,
82, 83,
84, 86,
87, 89,
90, 96

3 If choledochotomy is done
during common bile duct
exploration (CBDE), is T-tube
drainage necessary?

In open or laparoscopic surgery,


after verification of duct clearance
intraoperatively, primary closure of
the choledochotomy with or without
biliary stenting is a safe alternative to
T-tube drainage with no differences
in complication rates. Routine T-tube
placement after choledochotomy is not
necessary

1b

40-47, 91,
97-98

4 What is the optimal method


of biliary decompression and
timing of decompression in
patients with acute cholangitis
due to CBDS?

In patients with acute cholangitis, urgent


endoscopic biliary decompression is
preferred over surgery
There is insufficient evidence to
recommend a specific time period
where urgent biliary decompression
should occur

2b

48-55, 92,
93

5 Is cholecystectomy necessary
after endoscopic treatment of
CBDS in patients both with
and without gallstones (GS)?

Prophylactic cholecystectomy should be


offered to patients after ES unless there
are unfavorable by patient-related factors,
in patients with proven gallstones
After ES, in patients with no gallstones,
acute cholecystitis is uncommon, and
recurrent biliary complications may be
a result of primary bile duct stones.
Routine cholecystectomy therefore may
not be necessary

1b

64-68, 94,
99

2b

53, 69-74,
95

PMPH_CH48.indd 390

References

5/22/2012 5:34:11 PM

Common Bile Duct Stones

REFERENCES
1. Enochsson L, Swahn F, Arnelo Y, et al. Nationwide, populationbased data from 11,074 ERCP procedures from the Swedish
Registry for Gallstone Surgery and ERCP. Gastrointest Endosc.
2010;72:1175-1184.
2. Shanmugam V, Beattie GC, Yule SR, et al. Is magnetic resonance
cholangiopancreatography the new gold standard in biliary
imaging? Br J Radiol. 2005;78:888-893.
3. Pamos S, Benages A, Medina E, Martinez Sanjuan V. Prospective
evaluation of magnetic resonance cholangiopancreatography in
patients with biliary disease: Comparative study with conventional ultrasonography and endoscopic retrograde cholagiopancreatography. Dig Liver Dis. 2003;35:186-192.
4. Tekin A, Saygili M, Hafta A, Suleyman O. Biliary stones and
stenoses: Diagnostic value of magnetic resonance cholangiography. Turk J Gastroenterol 2002;13:139-145.
5. Shamiyeh A, Lindner E, Schwarzenlander K, Wayand W. Shortvs long-sequence MRI cholangiography for the preoperative
imaging of the common bile duct in patients with cholecystolithiasis. Surg Endosc. 2005;19:1130-1134.
6. Demartines N, Eisner L, Schnabel K, et al. Evaluation of magnetic resonance cholangiography in the management of bile duct
stones. Arch Surg. 2000;135:148-152.
7. Hallal AH, Amortegui JD, Jeroukhimov IM, et al. Magnetic resonance cholangiopancreatography accurately detects common
bile duct stones in resolving gallstone pancreatitis. J Am Coll
Surg. 2005;200:869-875.
8. Zidi SH, Prat F, Le Guen O, et al. Use of magnetic resonance
cholangiography in the diagnosis of choledocholithiasis: A prospective comparison with a reference imaging method. Gut.
1999;44:118-122.
9. Kondo S, Isayama H, Akahane M, et al. Detection of common bile
duct stones: Comparison between endoscopic ultrasonography,
magnetic resonance cholangiography, and helical computed
tomographic cholangiography. Eur J Radiol. 2005;54:271-275.
10. Scaffidi MG, Luigiano C, Consolo P, et al. Magnetic resonance
cholangio-pancreatography versus endoscopic retrograde
cholangio-pancreatography in the diagnosis of common bile
duct stones: A prospective comparative study. Minerva Med.
2009;100:341-347.
11. Romagnuolo J, Bardou M, Rahme E, et al. Magnetic resonance
cholangiopancreatography: A meta-analysis of test performance
in suspected biliary disease. Ann Intern Med. 2003;139:547-557.
12. Aljebreen A, Azzam N, Eloubeidi A. Prospective study of endoscopic ultrasound performance in suspected choledocholithiasis. J Gastroenterol Hepatol. 2008;23:741-745.
13. Ney M, Maluf-Filho F, Sakai P, et al. Endoscopic ultraosund
versus endoscopic retrograde cholangiography for the diagnosis of choledocholithiasis: The influence of the size of the stone
and diameter of the common bile duct. Arq Gastroenterol. 2005;
42:239-243.
14. Karakan T, Cindoruk M, Alagozlu H, et al. EUS versus endoscopic retrograde cholangiography for patients with intermediate probability of bile duct stones: A prospective randomized
trial. Gastrointest Endosc. 2009;69:244-252.
15. Maluf-Filho F, Dotti CM, Halwan B, et al. An evidence-based
consensus statement on the role and application of endosonography in clinical practice. Endoscopy. 2009;41:979-987.
16. Tse F, Barkun JS, Barkun AN. The elective evaluation of patients
with suspected choledocholithiasis undergoing laparoscopic
cholcystectomy. Gastrointest Endosc. 2004;60:437-448.

PMPH_CH48.indd 391

391

17. Ledro-Cano D. Suspected choledocholithiasis: Endoscopic ultrasound or magnetic resonance cholangio-pancreatography? A systematic review. Eur J Gastroenterol Hepatol. 2007;19:1007-1011.
18. Scheiman JM, Carlos RC, Barnett JL, et al. Can endoscopic ultrasound or magnetic resonance cholangiopancreatography replace
ERCP in patients with suspected biliary disease? A prospective
trial and cost analysis. Am J Gastroenterol. 2001;96:2900-2904.
19. Mark DH, Flamm CR, Aronson N. Evidence-based assessment
of diagnostic modalities for common bile duct stones. Gastrointest Endosc. 2002;56:S190-S194.
20. Clayton ESJ, Connor S, Alexakis N, Leandros E. Meta-analysis of
endoscopy and surgery versus surgery alone for common bile duct
stones with the gallbladder in situ. Br J Surg. 2006;93:1185-1191.
21. Martin DJ, Vernon D, Toouli J. Surgical versus endoscopic
treatment of bile duct stones. Cochrane Database of Systematic
Reviews. 2006: Issue 2.
22. Ribago LR, Vicente C, Soler F, et al. Two-stage treatment wth
preoperative endoscopic retrograde cholangiopancreatography
(ERCP) compared with single-stage treatment with intraoperative ERCP for patients with symptomatic cholelithiasis with possible choledocholithiasis. Endoscopy. 2006;38:779-786.
23. Chang L, Stabile BE, Lewis RJ, et al. Preoperative versus postoperative endoscopic retrograde cholangiopancreatography in mild
to moderate gallstone pancreatitis. Ann Surg. 2000;231:82-87.
24. Nathanson LK, ORourke NA, Martin IJ, et al. Postoperative
ERCP versus laparoscopic choledochotomy for clearance of
selected bile duct calculi. A randomized trial. Ann Surg. 2005;242:
188-192.
25. Riciardi R, Islam S, Canete JJ, et al. Effectiveness and long-term
results of laparoscopic common bile duct exploration. Surg
Endosc. 2003;17:19-22.
26. Liu TH, Consorti ET, Kawashima A, et al. Patient evaluation and
management with selective use of magnetic resonance cholangiography and endoscopic retrograde pancreatography before
laparoscopic cholecystectomy. Ann Surg. 2001;234:33-40.
27. Mercer S, Singh S, Paterson I. Selective MRCP in the management
of suspected common bile duct stones. HPB. 2007;9:125-130.
28. Sharma SK, Larson KA, Adler Z, Goldfarb MA. Role of endscopic retrograde cholangiopancreatography in the management
of suspected choledocholithiasis. Surg Endosc. 2003;17:868-871.
29. Petrov MS, Savides TJ. Systematic review of endoscopic ultrasonography versus endoscopic retrograde cholangiopancreatography for suspected choledocholithiasis. Br J Surg. 2009;96:
967-974.
30. Lee YT, Chan FKL, Leung WK, et al. Comparison of EUS and
ERCP in the investigation with suspected biliary obstruction
caused by choledocholithiasis: A randomized study. Gastrointest
Endosc. 2008;67:660-668.
31. Liu CL, Fan ST, Lo CM, et al. Comparison of early endoscopic
ultrasonography and endoscopic retrograde cholangiopancreatography in the management of acute biliary pancreatitis:
A prospective randomized study. Clin Gastroenterol Hepatol.
2005;3:1238-1244.
32. Polkowski M, Regula J, Tilszer A, Butruk E. Endoscopic ultrasound versus endoscopic retrograde cholangiography for patients
with intermediate probability of bile duct stones: A randomized trial comparing two management strategies. Endoscopy.
2007;39:296-303.
33. Parnaby CN, Jenkins JT, Ferguson JC. Prospective validation study
of an algorithm for triage to MRCP or ERCP for investigation
of suspected pancreatico-biliary disease. Surg Endosc. 2008;22:
1165-1172.

5/22/2012 5:34:11 PM

392

Surgery: Evidence-Based Practice

34. Noble H, Tranter S, Chesworth T, et al. A randomized clinical


trial to compare endoscopic sphincterotomy and subsequent
laparoscopic cholecystectomy with primary laparoscopic bile
duct exploration during cholecystectomy in higher risk patients
with choledocholithiasis. J Laparoendosc Adv Surg Tech. 2009;19:
713-720.
35. Cuschieri A, Lezoche E, Morino M, et al. E.A.E.S. multicenter
prospective randomized trial comparing two-stage vs single
stage management of patients with gallstone disease and ductal
calculi. Surg Endosc. 1999;13:952-957.
36. Suc B, Escat J, Cherqui D, et al. Surgery vs endoscopy as primary
treatment in symptomatic patients with suspected common bile
duct stones. Arch Surg. 1998;133:702-708.
37. Sgourakis G, Dedemadi G, Stamatelopoulos A, et al. Predictors
of common bile duct lithiasis in laparoscopic era. World J Gastroenterol. 2005;11:3267-3272.
38. Williams EJ, Green J, Beckingham I, et al. Guidelines on the
management of common bile duct stones (CBDS). Gut. 2008;57:
1004-1021.
39. Wills VL, Gibson K, Karthaloot C, Jorgensen JO. Complications
of biliary T-tubes after choledochotomy. ANZ J Surg. 2002;72:
177-180.
40. Decker G, Borie F, Millat B, et al. One hundred laparoscopic choledochotomies with primary closure of the common bile duct.
Surg Endosc. 2003;17:12-18.
41. Gurusamy KS, Samraj K. Primary closure versus T tube drainage after open common bile duct exploration (review). Cochrane
Database of Systematic Reviews. 2007: Issue 1.
42. Zhang L, Bie P, Wang S, He Y. A randomized comparison of primary closure and t-tube drainage of the common bile duct after
laparoscopic choledochotomy. Surg Endosc. 2008;22:1595-1600.
43. Zhang WJ, Xu GF, Wu GZ, et al. Laparoscopic exploration of
common bile duct with primary closure versus T-tube drainage:
A randomized controlled trial. J Surg Res. 2009;157:e1-e5.
44. El-Geidie A. Is the use of t-tube necessary after laparoscopic choledochotomy? J Gastrointest Surg. 2010;14:844-848.
45. Yamazaki M, Yasuda H, Tsukamoto S, et al. Primary closure of
the common bile duct in open laparotomy for common bile duct
stones. J Hepatobiliary Pancreat Surg. 2006;13:398-402.
46. Jameel M, Darmas B, Baker A. Trend towards primary closure
following laparoscopic exploration of the common bile duct.
Ann R Coll Engl. 2008;90:29-35.
47. Martin IJ, Bailey IS, Rhodes M, et al. Towards T-tube free laparoscopic bile duct exploration. Ann Surg. 1998;228:29-34.
48. Sugiyama M, Atomi Y. Treatment of acute cholangitis due to
choledocholithiasis in elderly and younger patients. Arch Surg.
1997;132:1129-1133.
49. Leese T, Neoptolemos JP, Baker AR, Carr-Locke DL. Management of acute cholangitis and the impact of endoscopic sphincterotomy. Br J Surg. 1986;73:988-992.
50. Lau JYW, Ip SM, Chung SCS, et al. Endoscopic drainage aborts
endotoxaemia in acute cholangitis. Br J Surg. 1996;83:181-184.
51. Lai ECS, Mok FPT, Tan ESY, et al. Endoscopic biliary drainage
for severe acute cholangitis. N Engl J Med. 1992;326:1582-1586.
52. Khuroo MS, Mahajan R, Zargar SA, et al. Endoscopic vs surgical
drainage of biliary tract in acute pyogenic cholangitis: A controlled study. Indian J Gastroenterol. 1989;8:119-122.
53. Nagino M, Takada T, Kawarada Y, et al. Methods and timing of
biliary drainage for acute cholangitis: Tokyo Guidelines. J Hepatobiliary Pancreat Surg. 2007;14:68-77.
54. Lai ECS, Paterson IA, Tam PC, et al. Severe acute cholangitis: The
role of emergency nasobiliary drainage. Surgery. 1990;107:268-272.

PMPH_CH48.indd 392

55. Chijiiwa K, Kozaki N, Naito T, et al. Treatment of choice for choledocholithiasis in patients with acute obstructive suppurative
cholangitis and liver cirrhosis. Am J Surg. 1995;170:356-360.
56. Pessa ME, Hawkins IF, Vogel SB. The treatment of acute cholangitis. Percutaneous transhepatic biliary drainage before definitive therapy. Ann Surg. 1987;205:389-392.
57. Yeung YP, Yip AWC. Predictors for emergency biliary decompression in acute cholangitis. Eur J Gastroent Hepatol. 2006;18:
727-731.
58. Hui CK, Lai KC, Yuen MF, et al. Acute cholangitis-predictive
factors for emergency ERCP. Aliment Pharmacol Ther. 2001;15:
1633-1637.
59. Boender J, Nix GAJJ, de Ridder MAJ, et al. Endoscopic sphincterotomy and biliary drainage in patients with cholangitis due to
common bile duct stones. Am J Gastroenterol. 1995;90:233-238.
60. Saito M, Tsuyuguchi T, Yamaguchi T, et al. Long-term outcome
of endoscopic papillotomy for choledocholithiais with cholecystolithiasis. Gastrointest Endosc. 2000;51:540-545.
61. Kwon SK, Lee BS, Kim NJ, et al. Is cholecystectomy necessary
after ERCP for bile duct stones in patients with gallbladder in
situ? Kor J Int Med. 2001;16:254-259.
62. Schreurs WH, Vles WJ, Stuifbergen WHNM, Oostvogel HJM.
Endoscopic management of common bile duct stones leaving the
gallbladder in situ. Dig Surg. 2004;21:60-65.
63. Kullman E, Borch K, Dahlin LG, Liedberg G. Long-term follow
up of patients with gallbladder in situ after endoscopic sphincterotomy for choledocholithiasis. Eur J Surg. 1991;157:131-135.
64. McAlister VC, Davenport E, Renouf E. Cholecystectomy deferral
in patients with endoscopic sphincterotomy. Cochrane Database
of Systematic Review. 2007;(4):CD006233.
65. Boerma D, Rauws EAJ, Keulemans YCA, et al. Wait-and-see
policy or laparoscopic cholecystectomy after endoscopic sphincterotomy for bile-duct stones: A randomized trial. Lancet. 2002;
360:761-765.
66. Lau JYW, Leow CK, Fung TMK, et al. Cholecystectomy or gallbladder in situ after endoscopic sphincterotomy and bile duct
stone removal in Chinese patients. Gastroenterol. 2006;130:
96-103.
67. Taragona EM, Perez Ayuso RM, Bordas JM, et al. Randomised
trial of endoscopic sphincterotomy with gallbladder left in situ
versus open surgery for common bile duct calculi in high-risk
patients. Lancet. 1996;347:926-929.
68. Costi R, DiMauro A, Mazeo A, et al. Routine laparoscopic
cholecystectomy after endoscopic sphincterotomy for choledocholithiasis in octogenarians: Is it worth the risk? Surg Endosc.
2007;21:41-47.
69. Ando T, Tsuyuguchi T, Okugawa T, et al. Risk factors for recurrent bile duct stones after endoscopic papillotomy. Gut. 2003;52:
116-121.
70. Lee JK, Ryu JK, Park JK, et al. Risk factors of acute cholecystitis
after endoscopic common bile duct stone removal. World J Gastroenterol. 2006;12:956-960.
71. Sugiyama M, Atomi Y. Risk factors predictive of late complications after endoscopic sphincterotomy for bile duct stones: long
term (more than 10 years) follow-up study. Am J Gastroenterol.
2002;97:2763-2767.
72. Costomagna G, Tringali A, Shah SK, et al. Long term follow-up of
patients after endoscopic sphincterotomy for choledocholithiasis, and risk factors for recurrence. Endoscopy. 2002;34:273-279.
73. Tanaka M, Takahata S, Konomi H, et al. Long-term consequence
of endoscopic sphincterotomy for bile duct stones. Gastrointest
Endosc. 1998;48:465-469.

5/22/2012 5:34:11 PM

Common Bile Duct Stones

74. Lai K-H, Lin L-F, Lo G-H, et al. Does cholecystectomy after endoscopic sphincterotomy prevent the recurrence of biliary complications? Gastrointest Endosc. 1999;49:483-487.
75. Vitetta L, Sali A, Little P, et al. Primary brown pigment bile
duct stones. HPB Surgery. 1991;4:209-222.
76. Hintze RE, Adler A, Veltzke W, et al. Clinical significance of
magnetic resonance cholangiopancreatography (MRCP) compared to endoscopic retrograde cholangiopancreatography
(ERCP). Endoscopy. 1997;29:182-187.
77. Eshghi F, Abdi R. Routine magnetic resonance cholangiopancreatography compared to intra-operative cholangiography in
patients with suspected common bile duct stones. Hepatobiliary
Pancreat Dis Int. 2008;7:525-528.
78. Ainsworth AP, Rafaelson SR, Wamberg PA, et al. Is there a difference in diagnostic accuracy and clinical impact between
endoscopic ultrasonography and magnetic resonance cholangiopancreatography? Endoscopy. 2003;35:1029-1032.
79. Parra-Membrives P, Diaz-Gomez D, Vilegas-Portero R, et al.
Appropriate management of common bile duct stones: A RAND
Corporation/UCLA Appropriateness Method statistical analysis. Surg Endosc. 2010;24:1187-1194.
80. Wright BE, Freeman ML, Cumming JK, et al. Current management
of common bile duct stones: Is there a role for laparoscopic cholecystectomy and intra-operative endoscopic retrograde cholangiopancreatography as a single stage procedure? Surgery. 2002;132:729-737.
81. Catheline JM, Turner R, Paries J. Laparoscopic ultrasonography
is a complement to cholangiography for the detection of choledocholithiasis at laparoscopic cholecystectomy. Br J Surg. 2002;
89:1235-1239.
82. Tranter SE, Thompson MH. A prospective single-blinded controlled
study comparing laparoscopic ultrasound of the common bile duct
with operative cholangiography. Surg Endosc. 2003;17:216-219.
83. Stiegmann GV, Goff JS, Mansour A, et al. Precholecystectomy
endoscopic cholangiography and stone removal is not superior
to cholecystectomy, cholangiography, and common duct exploration. Am J Surg. 1992;163:227-230.
84. Rogers SJ, Cello JP, Horn JK, et al. Prospective randomized trial
of LC+LCBDE vs ERCP/S + LC for common bile duct stone disease. Arch Surg. 2010;145:28-33.
85. Bahram M, Gaballa G. The value of pre-operative magnetic resonance cholangiopancreatography (MRCP) in management of
patients with gallstones. Int J Surg. 2010;8:342-345.
86. Rhodes M, Sussman L, Cohen L, Lewis MP. Randomised
trial of laparoscopic exploration of common bile duct versus

PMPH_CH48.indd 393

87.

88.

89.

90.

91.

92.
93.

94.

95.
96.

97.
98.

99.

393

postoperative retrograde cholangiography for common bile duct


stones. Lancet. 1998;351:156-161.
Bansal VK, Misra MC, Garg P, Prabhu M. A prospective randomized trial comparing two-stage versus single-stage management of patients with gallstone disease and common bile duct
stones. Surg Endosc. 2010;24:1986-1989.
Artifon ELA, Kumar A, Eloubeidi MA, et al. Prospective randomized trial of EUS verusus ERCP-guided common bile duct
stone removal: An interim report (with video). Gastrointest
Endosc. 2009;69:238-243.
Varadarajulu S, Eloubeidi A, Wilcox CM, et al. Do all patients with
abnormal intraoperative cholangiogram merit endoscopic retrograde cholangiopancreatography? Surg Endosc. 2006;20:801-805.
Hong D-F, Xin Y, Chen D-W. Comparison of laparoscopic cholecystectomy combined with intraoperative endoscopic sphincterotomy and laparoscopic exploration of the common bile duct
for cholecystolithiasis. Surg Endosc. 2006;20:424-427.
Sheen-Chen S-M, Chou F-F. Choledochotomy for biliary lithiasis: Is routine T-tube drainage necessary? A prospective Randomized trial. Acta Chir. 1990;156:387-390.
Lai ECS, Tam PC, Paterson IA, et al. Emergency surgery for severe acute cholangitis. Ann Surg. 1990;211:55-59.
Sharma BC, Agarwal DK, Baijal SS, et al. Endoscopic management of acute calculous cholangitis. J Gastroenterol Hepatol.
1997;12:874-876.
Lee KM, Paik CN, Chung WC, et al. Risk factors for cholecystectomy in patients with gallbladder stones after endoscopic
clearance of common bile duct stones. Surg Endosc. 2009;23:
1713-1719.
Hui CK, Lai KC, Yuen MF, et al. Role of cholecystectomy in preventing recurrent cholangitis. Gastrointest Endosc. 2002;56:55-60.
Cuschieri A, Croce E, Faggioni A. E.A.E.S. ductal stone study:
Preliminary findings of multicenter prospective randomized
trial comparing two-stage versus single-stage management. Surg
Endosc. 1996;10:1130-1135.
Payne RA, Woods WGA. Primary suture or T-tube drainage after
choledochotomy. Ann Royal Coll Surg Engl. 1986;68:196-198.
Perez G, Escalona A, Jarufe N, et al. Prospective randomized study
of T-tube versus biliary stent for common bile duct decompression
after open choledochotomy. World J Surg. 2005;29:869-872.
Hammarstrom LE, Holmin T, Stridbeck H, Ihse I. Long-term
follow-up of a prospective randomized trial of endoscopic versus
surgical treatment of bile duct calculi in patients with gallbladder in situ. Br J Surg. 1995;82:1516-1521.

5/22/2012 5:34:11 PM

CHAPTER 49

Benign Biliary Strictures


Demetrius Pertsemlidis and David S. Pertsemlidis

INTRODUCTION

hepatico-jejunostomy has become the principal bilio-enteric


reconstruction leading to a success rate of 90%.1

Benign bile duct strictures are the consequence of severe iatrogenic injuries occurring mostly during biliary surgery. Since the
introduction of laparoscopic cholecystectomy in the United States
in 1988, the incidence of biliary strictures doubled and the longterm mortality tripled, in comparison with patients undergoing
uncomplicated cholecystectomy.1-6
In a cohort of 42,474 open cholecystectomies, the incidence of
biliary strictures was 0.2%. Rapid rise of iatrogenic biliary strictures in the early laparoscopic era was followed by gradual decline
reflecting experience with the new approach and improvement of
technology in optics and instruments; however, a national Australian audit in 1995 revealed a high rate of 0.7%. The majority of
the biliary strictures caused by iatrogenic injuries are discovered
late in the postoperative period.7 In four large series, the mean
interval between open or laparoscopic cholecystectomy and diagnosis of the biliary stricture was 18.7 months. Biliary strictures
resulting from anastomotic failure occur in about 10% of patients
after pancreatico-duodenectomy.8-10
The value of selective or routine intraoperative cholangiography (IOC) is still debated. The surgical literature indicates
that use of C-arm fluoroscopy and improvement in the quality of
imaging have virtually eliminated the concern of misinterpretation of cholangiograms. The threshold of indications for IOC has
been lowered substantially, when problems of biliary anatomy or
suspected injury arise during surgery. Clinical studies strongly
suggest superiority of IOC in the prevention of iatrogenic biliary
injuries.11,12 Differentiation between benign and malignant biliary strictures has markedly improved. The clinical, biochemical,
molecular, and pathologic parameters augmented by imaging
and genetic analysis have advanced the differential diagnostic
accuracy.
In the remote past (19301960), the dominant approach for
bilio-enteric reconstructions was duct-to-duct and duodenobiliary anastomoses, whereas Roux-Y hepatico-jejunostomy was
seldom utilized. In the past three to four decades, the Roux-Y

1. What predisposes to benign biliary strictures?


The vast majority of benign bile duct strictures are iatrogenic,
either from injury during biliary surgery or anastomotic failures
during pancreatico-duodenectomy. Rarely, direct injury to the bile
duct can also occur during subtotal pancreatectomy for neoplasm
or nesidioblastosis, coring out of the pancreatic head for chronic
pancreatitis, or pancreatic necrosectomy. The intrapancreatic
portion of the bile duct can be occluded in chronic alcoholic or
autoimmune pancreatitis and ampullary strictures may be caused
by endoscopic sphincterotomy, chronic use of one or more biliary
stents, or surgical ampullectomy.
Intraoperative distinction between a normal bile duct and
the cystic duct can be difficult in the open and especially in laparoscopic cholecystectomy because of similar size, color, and loss
of three-dimensional perception. The gallbladder infundibulum
and the cystic duct are in very close proximity to the main bile
duct, right hepatic duct and artery. The millimeter distance
separating these structures can be obscured from bleeding or
obliterated by acute or chronic inflammation, especially in a
contracted gallbladder. Aberrant right hepatic artery arising
from the superior mesenteric can distort the normal anatomy.
The common site of ligation, transection, burn, or resection
is the mid-portion of the 7-cm-long extrahepatic bile duct. Excessive vertical traction of the fundus, rather than lateral traction
of Hartmanns pouch, may produce tenting and accidental resection of the entire cystic-bile duct junction. Failure to identify the
junction and isolate the cystic duct may result in loss of a major
segment of the bile duct corresponding to Bismuth Type 2 or
Strasberg Type E2 injury.13,14
Thermal injuries to the bile duct and other organs in the subhepatic region can be treacherous because of difficult intraoperative recognition and common underestimation of the extent
of damage.

394

PMPH_CH49.indd 394

5/22/2012 5:34:45 PM

Benign Biliary Strictures

Aside from direct instrumental or thermal injuries, caused by


surgical, endoscopic or percutaneous transhepatic interventions,
anastomotic failures from ischemia, tension or infection are common causes of biliary strictures. These strictures are almost always
short, allowing distinction from the longer cholangiocarcinomas.
Extrinsic compression by cysts, large impacted gallstones
(Mirizzi syndrome), tumors, and chronic alcoholic or autoimmune pancreatitis can cause stenosis or occlusion of the bile duct,
leading to cholestasis-induced stones, cholangitis, jaundice, and
severe pruritus. Inherited choledochal cysts are treated with radical resection because of malignant predisposition and the reconstructions are similar and as complex as in the severe biliary
iatrogenic strictures.
Primary sclerosing cholangitis (PSC) has a strong association
with ulcerative colitis and affects two-thirds of young men with
this disease. This association correlates with the extent of colitis,
being tenfold higher (5.5%) with pancolitis, compared with 0.5%
with disease confined to the left side. Overall, 3.7% of patients with
ulcerative colitis will have PSC, and conversely colitis coexists in
70% to 100% of patients with PSC. The biology and clinical course
of the two diseases are independent, both in the time of clinical
expression and response to treatment. Cholangiocarcinoma develops in 10% to 20%, chronic pancreatitis in 20% to 40% in PSC
patients and the risk of pancreatic cancer is 15-fold higher than
in the unaffected population. The risk of colon cancer is fivefold
higher when the two diseases coexist over a follow-up period of
25 years. Long periods of indolent quiescence are not uncommon
and impossible to predict. Neither the time of progression to cirrhosis nor the transformation to biliary carcinoma are predictable.
Secondary sclerosing cholangitis is rare and affects patients treated
with hepatic arterial chemotherapy with 5-fluorodeoxyuridine for
metastatic colon cancer, or accidental leak of scolicidal agents into
intrahepatic bile ducts during infusion into echinococcal cysts.
Predilection of dominant biliary strictures to the hilar
region can be treated successfully with intrahepatic Roux-Y
bilio-enterostomy after partial central hepatectomy (segments 4
and 5). Progression to cirrhosis in PSC is treated with liver transplantation. The chemical injury induced by hepatic arterial chemotherapy can be minimized by prophylactic use of steroids. The
chemical toxicity of anti-echinococcal scolicidal agents leading
to secondary sclerosing cholangitis may be treated with bypass
alone, if the stricture is confined to the hilum; extensive intrahepatic involvement can be combined with hemi-hepatectomy or
use of liver transplantation.
Answer: The common major risk factors which predispose to
biliary strictures are iatrogenic bile duct injuries during biliary and
pancreatic surgery. Clinical evidence indicates that laparoscopic
cholecystectomy is associated with twofold higher rate of injuries
than open cholecystectomy. Other risk factors leading to bile duct
stenosis include inborn anatomic anomalies such as choledochal
cysts, autoimmune syndromes especially PSC, chronic inflammatory diseases such as alcoholic pancreatitis, extrinsic compression
by cystic or neoplastic lesions, and abdominal trauma. (Grade C
recommendation)
2. Can IOC prevent or minimize bile duct injuries?
The routine or selective performance of IOC during open or
laparoscopic cholecystectomy has been debated ever since its

PMPH_CH49.indd 395

395

introduction by P. L. Mirizzi in 1937. The rapid dissemination of


laparoscopic biliary surgery without the customary review and
scientific validation and the initial suboptimal technology has lead
to doubling of bile duct injuries, in comparison to open cholecystectomy. The need of IOC to define the surgical anatomy during
laparoscopic surgery has risen substantially.15 The acquisition of
skills necessary to perform IOC during laparoscopic cholecystectomy was included in the guidelines of the consensus conference
of the National Institutes of Health in 1993.16
A retrospective nationwide cohort study of nearly 1.6 million
cholecystectomies (laparoscopic 76%) was derived from records of
a Medicare Database over a period of 7 years (19921999). Overall
bile duct injuries amounting to 0.5% were defined as reoperation
to repair the biliary injury within 1 year from cholecystectomy.
IOC was performed in 39% of all patients. The use of IOC reduced
bile duct injuries to 0.39%, compared with 0.58% in those without
IOC (p < .001). The authors concluded that routine use of IOC may
decrease the rate of bile duct injury.4
A retrospective population-based study in Western Australia was based on hospital records and postoperative endoscopic
retrograde cholangio-pancreatographies (ERCPs) for bile leaks.
Of 19,187 colecystectomies, open and laparoscopic, 456 patients
with intraoperative intestinal, vascular, and bile duct injuries
were identified. After the introduction of laparoscopic cholecystectomy in 1991, the percentage of iatrogenic injuries doubled
from 0.67 in 19881990 to 1.33 in 19931994. The odds ratio for
laparoscopic to open cholecystectomy was 1.79. In this important population study, intraoperative cholangiographic imaging
lowered substantially the risk of all injuries. IOC reduced the
rate or injuries from 2.7 to 1.0/1,000 cases comparing laparoscopic with open cholecystectomy. Laparoscopic cholecystectomy combined with IOC lowered the number of injuries from
4.3 to 2.1, and from 16.9 in patients with acute cholecystitis,
pancreatitis, jaundice, or cholangitis, to 2.2 in patients without severe comorbidities. The authors estimated that combining surgery with routine IOC might have prevented one-third
of bile duct injuries.12
A respectable study of IOC published in 1995, did not find
beneficial protective effect from IOC.17
The sequence in the performance of IOC during laparoscopic colecystectomy offers certain protective advantages:
(a) trans-cystic cholangiography is performed before transecting
the cystic duct and dissection around the gallbladder; (b) biliary
anomalies can be detected before deeper dissection; (c) cholangiographic imaging is clearer as the continuous flow of contrast
fi lls sequential segments of the biliary system under C-arm fluoroscopy; and (d) the early detection of injury, which is critical.
The time for performance of the IOC and the low cost are trivial
in comparison with the gain of anatomic surgical precision and
prevention of injuries, especially in severe cholecystitis and coexisting morbidities.
Answer: IOC is a highly valuable tool in biliary surgery and
in operations adjacent to the biliary system, such as pancreatic,
duodenal, or liver surgery. The C-arm fluoroscopic cholangiography has vastly improved the accurate interpretation of cholangiograms, compared with the cumbersome and less precise
suboptimal radiography of the past. On the other hand, the
current preoperative imaging offers a great deal of information
regarding the biliary anatomy and the extent of inflammatory

5/22/2012 5:34:45 PM

396

Surgery: Evidence-Based Practice

changes surrounding the bile duct. The national trend appears to


favor selective use of IOC; however, the threshold for use of IOC
must be substantially low and the special equipment immediately
available. (Grade C recommendation)
3. How accurate is the differential diagnosis of benign and
malignant biliary strictures?
Progress in the distinction between benign biliary strictures and
cholangiocarcinoma has been significant, but not spectacular.
The history and clinical findings, the prior surgical, endoscopic or
percutaneous interventions, and conventional plus scintigraphic
imaging are the hallmarks of differential diagnosis. Tumor markers in the serum, cytology of bile, and fine needle tissue aspirations
(percutaneous or intraoperative) have not yielded major advancements. The severe desmoplastic response in cholangiocarcinoma
often precludes cytologic or histologic diagnosis.
PSC is often indistinguishable radiographically from cholangiocarcinoma. Choledochal cysts also have the potential of malignant transformation. After resection of a choledochal cyst, the
reconstruction is similar to that of iatrogenic strictures. Long-term
close follow-up of these premalignant syndromes is essential.
Bile samples collected during ERCP, percutaneous transhepatic and intraoperative were studied for methylation profi le of
6 of 19 tumor suppressor genes, assays of Ki-ras gene point mutations and brush cytology in bile and lumen of the stricture yielded
only qualitative trends.18-20 Fine needle aspiration (FNA) cytology
of percutaneously accessible strictures has been recommended to
only strengthen concomitant diagnostic data.21
Table 49.1 shows an overview of the features of the two entities. A history of biliary or pancreatic surgery, endoscopic or
percutaneous transhepatic interventions, chronic cholelithiasis or
choledocholithiasis, chronic pancreatitis, surgery for choledochal
cyst, abdominal trauma, radiotherapy, human immunodeficiency
viral (HIV) cholangiopathy, and cystic or neoplastic diseases
compressing the bile duct should raise suspicion of benign stricture. The presence of paraneoplastic symptoms, pruritus, episodic
cholangitis, chronic or autoimmune pancreatitis, and cholangitis,
augmented by a precise history, facilitate the distinction between
benign and malignant biliary disease. Bacterial cholangitis is
absent in cholangiocarcinoma unless there was endoscopic, percutaneous, or surgical intervention.
Imaging, noninvasive and invasive, is critical in differentiating
benign from malignant biliary strictures. Computed tomography
in 50 patients with strictures (32 malignant) showed length of 17.9
mm in malignant versus 8.9 mm in benign strictures (p < .001),
upstream bile duct caliber of 22.0 versus 17.8 mm (p = .003), thickness >1.5 mm in 26 malignancies, and greater enhancement during
arterial and portal venous phases in malignancies compared with
those with benign strictures.22
Intraductal ultrasonography (IDUS) utilizes a cholangioscope combined with an ultrasound probe. The 4.9-mm instrument is inserted through the duodenal ampulla or percutaneous
transhepatic route. This endoscopic approach was used in 93
patients with biliary strictures (38 cholangiocarcinoma, 23 pancreatic cancer, 7 gallbladder malignancy, 9 benign structure,
6 chronic pancreatitis, 6 autoimmune pancreatitis, and 4 sclerosing cholangitis). In 42 patients with malignant stricture, the
diagnosis was confirmed by examination of the surgical specimen. The diagnosis of 26 malignant strictures and 9 benign

PMPH_CH49.indd 396

strictures was made from the histology of biopsy specimens. The


sensitivity and specificity were 89.7% and 84%, respectively, for
the differential diagnosis of biliary strictures. Direct visualization of the biliary mucosa using the percutaneous transhepatic
endoscopic approach facilitated the distinction between benign
and malignant disease. Untreated autoimmune pancreatitis and
the intrapancreatic bile duct proved difficult in distinguishing
benign from malignant strictures.23
Magnetic resonance cholangiopancreatography (MRCP) was
compared with ERCP. Fift y patients with strictures (23 benign, 27
cholangiocarcinoma) were retrospectively reviewed after definitive
diagnosis with surgical biopsy. Irregular margins and asymmetric
narrowing were present in 89% of cholangiocarcinomas and in
30% in benign strictures. The sensitivity, specificity, and accuracy
were 81 versus 74, 70 versus 70 and 76 versus 72 for MRCP versus
ERCP, respectively. The mean length of cholangiocarcinomas was
30 mm and for benign strictures 13.6 (p < .001). The conclusion
from this small retrospective study was that a long stricture with
irregular margins and asymmetric stenosis suggests the presence
of cholangiocarcinoma.24
Rare entities which must be differentiated from benign biliary strictures are primary and secondary sclerosing cholangitis
and polycystic liver disease.
Answer: The distinction between benign and malignant biliary
strictures is not difficult. The history of previous surgery or abdominal trauma, the absence of paraneoplastic symptoms, the presence
of episodic cholangitis, combined with the imaging characteristics permit the differentiation in the majority of cases. Short symmetrical strictures especially on cholangiographic images strongly
support the diagnosis of benign disease. Serum tumor markers,
cytology, and molecular and genetic studies of bile have not yielded
sufficient diagnostic power. (Grade C recommendation)
4. What is the optimal surgical treatment of benign biliary
strictures?
The gold standard for evaluation of biliary stricture is cholangiography. MRCP offers a diagnostic overview of the biliary and
pancreatic duct integrity or disruption. ERCP will confirm the site
and degree of stenosis or occlusion of the bile duct. If the proximal
biliary system is not visualized with retrograde contrast injection,
percutaneous transhepatic cholangiography (PTC) is necessary to
define the site and extent of discontinuity of the bile ducts.
The types of surgical reconstruction include transduodenal
sphincteroplasty for ampullary stricture, supraduodenal choledocho-duodenostomy for ampullary or intrapancreatic strictures
and Roux-Y hepatico-jejunostomy. The reconstruction of the bile
duct with duct-to-duct anastomosis is limited to cases when the
loss of duct is <1 cm.
In both choledochoduodenostomy and transduodenal
sphincteroplasty, the choledocho-duodenal anastomosis should
be at least 2.5 cm. The requisite for successful patency for choledocho-duodenostomy is a bile duct caliber of at least 10 mm,
and 3 cm distal common duct above the fi rst portion of the duodenum. The transduodenal sphincterotomy should not be longer
than 2.5 cm from the ampulla to avoid duodenal perforation.
The suture material must be absorbable.
The technique of constructing a Roux-Y hepatico-jejunostomy
is using a segment of proximal jejunum with sufficient mesenteric length to reach the hilum of the liver through retrocolic path.

5/22/2012 5:34:45 PM

Benign Biliary Strictures

A side-to-side anastomosis avoids excessive mobilization of the


normal bile duct remnant above the stricture and the risk of devascularization, or injury to the hepatic artery or portal vein. For bilioenteric anastomosis above the confluence of the two hepatic ducts
it may be possible to join the two ducts and perform a technically
easier 2-cm intrahepatic reconstruction. The length of the biliary
limb should measure 40 to 50 cm to prevent reflux of gastroduodenal contents beyond the anastomosis and into intrahepatic ducts.
Postoperative peri-hilar strictures (Bismuth Types III and IV,
Strasberg Types E3 and E4) are technically the most challenging.
Peri-hilar or hemi-hepatectomy may be necessary to restore bilioenteric continuity but the incidence of recurrent strictures is in
the range of 12% to 45%.25
In the remote prelaparoscopic era covering the period from
1930 to 1960 biliary and bilio-enteric reconstructions were mostly
duct-to-duct and duct-to-duodenum. Three large series from
premier academic institutions in New York (Presbyterian and
Memorial Hospitals),26 Massachusetts General in Boston,27 and
Minnesota (Mayo Clinic)28 performed only 17% Roux-Y hepaticojejunostomies in 331 reconstructions. The duct-to-duct reconstructions were successful only in 56% of the 331 operations.
The Roux-Y hepatico-jejunostomy has become the gold
standard for restoration of bilio-enteric continuity for strictures

397

reaching the proximal common hepatic duct and peri-hilar


region. Lillemoe et al.1 reported successful immediate and longterm outcome in 98% of 156 patients covering a 10-year period
from 1990 to 1999. Similar successful outcomes have been
reported by Sikora et al.9 in 300 patients with postcholecystectomy biliary strictures (open 262, laparoscopic 38) with low
surgical mortality of 1.3%, excellent 5-year outcome and failure
rate of 5.4%. Millis et al.29 used two types of bilio-enteric reconstructions, Roux-Y choledocho-jejunostomy in 101 patients and
hepatico-jejunostomy in 45 patients over the period from 1955
to 1990. Recurrent strictures developed in 22% of the patients
during the 3-year follow-up.29
The superiority of Roux-Y hepatico-jejunostomy for high bilioenteric anastomosis was proven by Tocchi et al.30 in both early
and late outcomes. Fift y-four patients treated with Roux-Y reconstruction by Schmidt et al.31 yielded high success rate (93%) with a
mean follow-up of 62 months (range 2.6154).31
Answer: The surgical gold standards for benign biliary
strictures are Roux-Y hepatico-jejunostomy for proximal and
choledocho-duodenostomy (supra- or transduoenal) for distal
strictures. With the exception of the rare supra-hilar intrahepatic
strictures, the success rate of these surgical approaches is about 90%.
(Grade C recommendation)

Clinical Question Summary


Question

Answer

1 What predisposes
to benign biliary
strictures?

Biliary surgery, laparoscoic more than open. Surgery


in the pancreatic head. Surgical or endoscopic
interventions in the duodenal ampulla. Gallstones,
cysts or tumors compressing the bile duct. Chronic
pancreatitis. Primary or secondary sclerosing
cholangitis.

1-12

2 Can IOC prevent or


minimize bile duct
injuries?

Selective rather than routine IOC appears to be the


national trend. This attitude of surgeons has been
greatly influenced by strong reliance on endoscopic
diagnosis and treatment of choledocholithiasis and
iatrogenic injuries. Precise preoperative imaging,
familiarity with and low threshold of performance of
IOC are protective of injuries

4, 7, 12,
15-17

3 How accurate is
the differential
diagnosis of benign
and malignant biliary
strictures?

A history of biliary or pancreatic surgical, endoscopic


or transhepatic interventions, episodic bacterial
cholangitis, absence of paraneoplastic symptoms and
associations with inflammatory, autoimmune and
inherited syndromes strongly favor benign disease.
Imaging (conventional, endoscopic) depicting short
homogeneous strictures reinforce the diagnosis
of benign entity. Differentiation from primary or
secondary sclerosing cholangitis is difficult.

18-24

4 What is the optimal


surgical treatment
of benign biliary
strictures?

Duct-to-duct anastomosis should be avoided if


the segmental loss of bile duct is greater than
1 cm. Choledocho-duodenostomy (supra- or
transduodenal) and Roux-Y hepatico-jejunostomy
are the surgical gold standards for distal and
proximal strictures, respectively.

1, 9, 25-31

PMPH_CH49.indd 397

Level of Grade of
References
Evidence Recommendation

5/22/2012 5:34:45 PM

398

Surgery: Evidence-Based Practice

REFERENCES
1. Lillemoe KD, Melton GB, Cameron JL, et al. Postoperative bile
duct strictures; Management and outcome in the 1990s. Ann
Surg. 2000;232:430-441.
2. A prospective analysis of 1518 laparoscopic cholecystectomies.
The Southern Surgeons Club. N Engl J Med. 1991;324:1973-1078.
3. Windsor JA, Pong J. Laparoscopic biliary surgery: More than a
learning curve problem. Aust NZ Surg. 1998;68:186-189.
4. Flum DR, Cheadle A, Prelac C, et al. Bile duct injury during
cholecystectomy and survival in medicare beneficials. JAMA.
2003;290:2168-2173.
5. Roslyn JJ, Bins GS, Hughes EF, et al. Open cholecystectomy. A
contemporary analysis of 42,474 patients. Ann Surg. 1993;218:
129-137.
6. Dolan JP, Diggs BS, Sheppard BC, Hunter JG. Ten-year trend
in the national volume of bile duct injuries requiring operative
repair. Surg Endosc. 2005;19:967-973.
7. Lillemoe KD. Repair of common bile duct injuries. uptodate.com/
contents/repair-of-common-bile-duct-injuries, September 2010.
8. Kahn MH, Howard TJ, Fogel EL, et al. Frequency of biliary complications after laparoscopic cholecystectomy detected by ERCP:
Experience in a large tertiary referral center. Gastrointest Endosc.
2007;65:247-252.
9. Sikora SS, Pottakkat B, Srikanth G, et al. Postcholecystectomy
benign biliary strictures: Long-term results. Dig Surg. 2006;23:
304-312.
10. Cantwell CP, Pena CS, Gervais DA, et al. Thirty years experience
with balloon dilatation of benign postoperative biliary strictures:
Long-term outcomes. Radiology. 2008;249:1050-1057.
11. Pitt HA, Miyamoto T, Parapatis SK, et al. Factors influencing
outcome in patients with postoperative biliary strictures. Am J
Surg. 1982;144:14-21.
12. Fletcher DR, Hobbs MST, Tan P, et al. Complications of cholecystectomy: Risks of the laparoscopic approach and protective effects of operative cholangiography. Ann Surg. 1999;229:
449-457.
13. Jarnagin WR, Blumgart LH. Benign biliary strictures. In: Blumgart LH, ed. Surgery of the Liver, Biliary Tract, and Pancreas. 4th
ed. Philadelphia, PA: Saunders; 2007:634.
14. Strasberg SM, Hertl M, Soper NJ. An analysis of the problem of
biliary injury during laparoscopic cholecystectomy. J Am Coll
Surg. 1995;180:101-105.
15. Massarweh NN, Flum DR. Role of intraoperative cholangiography
in avoiding bile duct injury. J Am Coll Surg. 2007;204:656-664.

PMPH_CH49.indd 398

16. National Institutes of Health. Consensus Development Conference Statement on gallstones and laparoscopic cholecystectomy.
Am J Surg. 1993;165:390-396.
17. Lorimer JW, Fairfull-Smith RJ. Intraoperative cholangiography
is not essential to avoid duct injuries during laparoscopic cholecystectomy. Am J Surg. 1995;169:344-347.
18. Zhang Y, Yang B, Du Z, et al. Identification and validation of specific methylation profile in bile for differential diagnosis of malignant biliary stricture. Clin Biochem. 2010;43:1340-1344.
19. Saurin J-C, Joly-Pharoboz M-O, Pernas P, et al. Detection of
Ki-ras gene point mutations in bile for differential diagnosis of
malignant and benign biliary strictures. Gut. 2000;47;357-361.
20. Mansfield JC, Griffin SM, Wadehra V, et al. A prospective
evaluation of cytology from biliary strictures. Gut. 1997;40:
617-677.
21. Desa LA, Acosa AB, Lazzara S, et al. Cytodiagnosis in the
management of extrahepatic biliary stricture. Gut. 1991;32:
1188-1191.
22. Choi SH, Han JK, Lee JM, et al. Differentiating malignant from
benign common bile duct stricture with multiphasic CT. Radiology. 2005;236:178-183.
23. Inui K, Yoshino J, Miyoshi H. Differential diagnosis and treatment of biliary strictures. Clin Gastroenterol Hepatol. 2009;7:
579-583.
24. Park MS, Kim TK, Kim KW, et al. Differentiation of extrahepatic
bile duct cholangiocarcinoma from benign stricture: Findings at
MRCP versus ERCP. Radiology. 2004;233:234-240.
25. Larghi A, Tringal A, Lecca PG, et al. Management of hilar biliary
strictures. Am J Gastroenterol. 2008;103:458-473.
26. OMalley RD, Auses AH, Jr., Whipple AO. Benign extrahepatic
biliary tract obstruction. Ann Surg. 1951;134:797-807.
27. Donaldson GA, Allen AW, Bartlett MK. Postoperative bileduct strictures: their etiology and treatment. N Engl J Med.
1956;254:50-56.
28. Walters W, Ramsdell JA. Study of three hundred eight operations for stricture of bile ducts. JAMA. 1959;171:872-276.
29. Millis JM, Tompkins RK, Zinner, et al. Management of bile
duct strictures: An evolving strategy. Arch Surg. 1992;127:
1077-1084.
30. Tocchi A, Costa G, Lepre L, et al. The long-term outcome of
hepatico-jejunostomy in the treatment of benign bile duct strictures. Ann Surg. 1996;224:162-167.
31. Schmidt SC, Langrehr JM, Hintze RE. Long-term results and
risk factors influencing outcome of major bile duct injuries following cholecystectomy. Brit J Surg. 2005;92:76-82.

5/22/2012 5:34:45 PM

CHAPTER 50

Gallstone Ileus
David W. Smith and Ara J. Feinstein

INTRODUCTION

Although it is difficult to make conclusions about morbidity and mortality due to the poor quality and small sample sizes
of these studies, the complication rate of one-stage procedures
involving enterolithotomy, cholecystectomy, and repair of choleenteric fistula appears to be higher than enterolithotomy alone.
Th is is likely due to a longer operative time in these series. Mortality figures are small, but also seemingly higher in the one-stage
procedures.
The incidence of biliary complications after the enterolithotomy alone was low in all studies, and most resolved with
conservative or endoscopic management. Although follow-up
time was variable, there was only one case that required interval
cholecystectomy.3
Answer: Recent retrospective case series show that gallstone
ileus continues to have a substantial complication rate, likely due
to significant comorbidities of the affected population. In the
past, it was argued that, in addition to enterolithotomy, a cholecystectomy and fistula repair should be performed at the initial
procedure or later due to the subsequent risk of biliary complications. In more recent series, however, the incidence of biliary
complications in patients who received enterolithotomy alone was
quite low. The risks of cholecystectomy and fistula repair seemingly outweigh the benefits. Enterolithotomy alone should be the
treatment of choice, especially in older, less stable patients with
significant comorbidities. (Grade C recommendation).

Gallstone ileus is a rare complication of cholelithiasis characterized by small bowel obstruction from a biliary calculus migrated
through a biliary enteric fistula. Bowel obstruction from an enterolith accounts for 1% to 3% of all bowel obstructions, but may
account for 25% of bowel obstructions in those above age 65 years
of age. The most common locations for obstructing enteroliths
are: the ileum (60.5%), followed by the jejunum (16.5%), stomach
(14.2%), colon (4.1%), and duodenum (3.5%).1 Bouverets syndrome
is a clinical entity described as impaction of a stone within the
duodenal bulb and thus leads to gastric outlet obstruction.
Although gallstone ileus was first described by Erasmus Bartholin in 1654, current morbidity and mortality figures remain
high due to the often vague, nonspecific symptoms, delayed diagnosis, and elderly preponderance. Less than 50% of patients have
a known history of biliary disease.1,2 With current diagnostic and
management strategies, mortality from gallstone ileus remains as
high as 12% to 27% in most published studies. Questions remain
regarding optimal diagnosis and treatment for this clinical entity.
1. Is enterolithotomy alone sufficient treatment for gallstone
ileus?
Operative strategies for gallstone ileus are controversial. Current
surgical options include: (1) one-stage procedure involving enterolithotomy to relieve bowel obstruction, cholecystectomy, and repair
of chole-enteric fistula; (2) two-stage procedure involving enterolithotomy and interval cholecystectomy; and (3) Eenterolithotomy
alone. To date, there are no randomized or prospective clinical trials comparing the various treatment strategies.
In 1993, Reisner and Cohen conducted a review of 1001 cases
of surgically treated gallstone ileus in the literature.1 They reported
significantly less mortality in those patients treated with enterolithotomy alone versus a one-stage procedure (11.7% vs. 16.9%).
More recently, several smaller retrospective have been published
comparing the two modalities (Table 50.1).

2. Can gallstone ileus be treated laparoscopically?


Laparoscopic management of gallstone ileus is a variation of a
staged procedure or enterolithotomy alone. Ideally, laparoscopic
techniques would minimize the physiologic burden and high morbidity and mortality figures associated with an open procedure.
The use of laparoscopy to treat gallstone ileus was first published by Montgomery in 1993.13 In a subsequent case series by
Soto et al.,14 two patients underwent laparoscopic management for
gallstone ileus. An obstructing stone in the proximal jejunum was
identified and laparoscopically brought through an extended port

399

PMPH_CH50.indd 399

5/22/2012 5:35:18 PM

400

Surgery: Evidence-Based Practice

Table 50.1 Operative Strategies


Author

Procedure

Number of
Patients

Complications
(%)

Yakan et al.3
Moberg and Montgomery4

Open enterolithotomy
One-stage

11
2

1 (9.1)
0 (0)

2 (18.2)
0 (0)

Laparoscopic-assisted
enterolithotomy
Open enterolithotomy

19

6 (31.5)

13

5 (38.5)

Lobo et al. 5

Open enterolithotomy
One-stage

14
1

5 (35.7)
1 (100)

0
1 (100)

Ayantunde and Agrawal6

Open enterolithotomy
One-stage

20
2

12 (60)
2 (100)

4 (20)
1 (50)

Pavlidis et al.7

Open enterolithotomy
One-stage

3
6

1 (33.3)
2 (33.3)

0 (0)
1 (16.7)

Doko et al.8

Open enterolithotomy
One-stage

11
18

3 (27.3)
15 (83.3)

1 (9.1)
2 (11.1)

Tan et al.9

Open enterolithotomy
One-stage

7
12

4 (57.1)
7 (58.3)

0 (0)
0 (0)

Muthukumarasamy et al.10

Open enterolithotomy
One-stage

10
3

3 (30.0)
1 (33.3)

0 (0)
0 (0)

Riaz et al.11

Open enterolithotomy
One-stage

5
5

3 (60.0)
1 (20.0)

0 (0)
0 (0)

Martnez Ramos and


Daroca Jos12

Enterolithotomy (Open and


laparoscopic)
One-stage

27

13 (48.1)

5 (18.5)

2 (50.0)

1 (25.0)

site. Extracorporeal enterolithotomy was completed with the small


bowel defect closed in two layers. The second patient underwent
laparoscopic decompression of the stone into the large bowel. The
biliary enteric fistula was not addressed in either case and both
patients subsequently discharged on the 4th and 6th postoperative day, respectively. There was no morbidity or mortality noted
in either of the patients.
Total laparoscopic management has been cited in recent literature. Ferraina et al.15 retrospectively reviewed six patients who
underwent video-assisted laparoscopic enterolithotomy. Four out
of the six patients were treated with extracorporeal enterolithotomy and enterorrhaphy via a mini mini-incision after laparoscopic identification. One patient underwent total laparoscopic
enterolithotomy and enterorrhaphy. The remaining patient underwent laparoscopy alone, as the enterolith had subsequently passed
into the large bowel. The overall complication rate was 33% with
a mortality of 16.6%. Owera et al.16 reported three patients who
underwent total laparoscopic enterolithotomy without intraoperative or postoperative complications. The authors reported no
morbidity or mortality at follow-up.
In the largest study to date addressing laparoscopic management, Moberg and Montgomery4 retrospectively compared laparoscopically assisted and open enterolithotomy on 32 consecutive
patients with gallstone ileus. Once the impacted stone was identi-

PMPH_CH50.indd 400

Mortality
(%)

Level of
Evidence

fied laparoscopically, it was delivered extracorporeally for subsequent enterolithotomy through an extended port site or separate
lower abdominal incision. Two out of 19 patients treated initially
with laparoscopy were converted to open due to technical difficulties. No significant difference in patient demographics, duration of
operation, length of hospitalization, morbidity, or mortality was
identified between the two groups.
Answer: Current literature supporting feasibility and success of laparoscopic enterolithotomy comes from small retrospective case series. Technical feasibility has been demonstrated with
similar morbidity and mortality in the largest retrospective study
to date. With careful patient selection and laparoscopic expertise, laparoscopy with enterolithotomy alone may be a treatment
option in gallstone ileus. (Grade C recommendation).
3. What is the best diagnostic modality?
Abdominal radiographs were initially the diagnostic modality of
choice for gallstone ileus. Radiographic criteria included pneumobilia, presence of an ectopic gallstone, and mechanical bowel
obstruction. Review of retrospective studies reveals the presence
of this classic Riglers triad in less than 50% of cases of surgically
proven gallstone ileus with sensitively at a modest 70%.1,2,17 Ultrasonography and computed tomography have been reviewed as separate diagnostic modalities or in combination with plain abdominal

5/22/2012 5:35:18 PM

Gallstone Ileus

radiographs. To date, there are no prospective trials comparing


abdominal radiographs, ultrasound, or computed tomography in
accurately diagnosing gallstone ileus.
Yakan et al.3 retrospectively reviewed 12 patients with surgically confirmed gallstone ileus. Correct preoperative diagnosis by
imaging was made in five patients. Four patients had CT imaging
correctly identifying gallstone ileus, while and the remaining individual had an ultrasound finding of ectopic stone. The remaining
seven patients were taken to the operating room without preoperative imaging due to clinical presentation requiring urgent
exploration. Plain fi lms were obtained in all 12 patients with the
authors noting nonspecific findings in all 12 to suggest a correct
preoperative diagnosis of gallstone ileus.
Lassandro et al.18 retrospectively studied the radiologic findings of 27 patients with gallstone ileus and found air fluid levels
and dilated bowel loops were the most common findings with
abdominal plain fi lm, ultrasound, and computed tomography.
The complete Riglers triad was found 4 times (15%) with plain
abdominal fi lm, 3 times (11%) with ultrasound, and 21 times (78%)
with CT. The study suggests that plain abdominal films and ultrasound may be adequate screening tools, but CT is clearly superior
for identifying gallstone ileus. Reimann et al.19 also showed the
usefulness in CT scanning to evaluate gallstone ileus. In a retrospective review of three patients, all had the complete triad identified on CT scan prior to operative intervention.
Diagnostic sensitivity with separate or combined radiologic
approaches with abdominal radiographs and ultrasonography was
retrospectively reviewed by Ripolls et al.20 Alone, ultrasonography had a statistically significant increased sensitivity in definitive
diagnosis of gallstone ileus over plain abdominal radiographs. The
sensitivity of ultrasonography alone or in combination with plain
abdominal radiographs still only approached 74%. Combined
imaging results were also reported by Ayantunde and Agrawal.6
A 77% preoperative diagnosis of gallstone imaging was achieved
with various combinations of abdominal plain fi lm, ultrasonography, and computed tomography. Sensitivity of individual imaging
modalities were not reported, but the addition of CT imaging in
combination or alone achieved 100% sensitivity in preoperative
diagnosis of gallstone ileus.
Yu et al.21 retrospectively reviewed 151 patients with small
bowel obstruction with 14 patients in that group known to have surgically confirmed gallstone ileus. Thirteen had correct preoperative

401

CT diagnosis of gallstone ileus, one false positive case, and no false


negatives. Lassandro et al.22 in their retrospective review showed
that in 40 patients, correct preoperative diagnosis was made in 35
patients based on clinical and CT findings on presentation. The
remaining five patients had stones found at surgery.
Answer: Prompt definitive diagnosis of gallstone ileus is rarely
from clinical history and physical examination alone. Used solely
or in combination with ultrasound and plain films, computed
tomography may add to a higher diagnostic sensitivity, specificity,
and accuracy essential to early diagnosis and surgical management.
CT may also add location, size, and presence of additional stones.
This may warrant a CT scan in all elderly patients with small bowel
obstructions or in patients in whom there is a clinical suspicion of
gallstone ileus. (Grade C recommendation).
4. Can gallstone ileus be treated with other techniques?
Endoscopy and novel lithotriptic techniques alone or in combination with endoscopy have been successfully utilized in extracting impacted stones. Various lithotriptic techniques have been
described and include laser lithotripsy, extracorporeal shockwave
lithotripsy, intracorporeal electrohydraulic lithotripsy, and endoscopic mechanical lithotripsy. Current literature reveals several
case studies of endoscopic removal of proximally located gastric stones as well as those located in small and large bowel.23,24
One report utilized shockwave lithotripsy before endoscopic
removal,25 two cases reported using a YAG laser to assist in stone
fragmentation prior to removal,26,27 and one case utilized both.28
Some authors have utilized various combinations of lithotriptic
modalities to assist in stone disintegration for subsequent endoscopic removal. There are case studies which have shown spontaneous passage of disintegrated stones as well. The technique of
lithotripsy fragmentation for endoscopic retrieval or spontaneous passage may avoid exposure of the aged patient with medical
comorbidities to invasive operations and general anesthesia.
Answer: Less invasive techniques to treat gallstone ileus
have been described in several case reports. These reports demonstrate success in various modalities of lithotriptic procedures
alone or in combination with endoscopy. Less invasive modalities
to treat gallstone ileus await further investigation, but may be an
option in patients too ill to undergo abdominal surgery. (Grade D
recommendation).

Clinical Question Summary


Question

Answer

1 Is enterolithotomy alone
sufficient?

Yes, operating times are shorter and the risk of subsequent


biliary complications is low.

1-11

2 Can gallstone ileus be


treated laparoscopically?

Yes, it is technically feasible with similar morbidity and


mortality to open enterolithotomy.

12-16

3 What is the best diagnostic


modality?

CT scan, due to improved sensitivity in preoperative


diagnosis compared to with plain films and ultrasound.

17-23

4 Can gallstone ileus be


treated with different
techniques?

Possibly. The use of endoscopy and lithotripsy has been


successful in selected patients but larger studies are
needed.

24-28

PMPH_CH50.indd 401

Grade of
Recommendation

References

5/22/2012 5:35:18 PM

402

Surgery: Evidence-Based Practice

REFERENCES
1. Reisner RM, Cohen JR. Gallstone ileus: A review of 1001 reported
cases. Am Surg. 1994;60(6):441-446.
2. Clavien, PA, Richon J, Burgan S, Rohner A. Gallstone ileus. Br J
Surg. 1990;77:737-742.
3. Yakan S, Engin O, Tekeli T. Gallstone ileus as an unexpected
complication of cholelithiasis: Diagnostic difficulties and treatment. Ulus Travma Acil Cerrahi Derg. 2010;16(4):344-348.
4. Moberg AC, Montgomery A. Laparoscopically assisted or open
enterolithotomy for gallstone ileus. Br J Surg. 2007;94(1):53-57.
5. Lobo DN, Jobling JC, Balfour TW. Gallstone ileus: Diagnostic
pitfalls and therapeutic successes. J Clin Gastroenterol. 2000;
30(1):72-76.
6. Ayantunde AA, Agrawal A. Gallstone ileus: Diagnosis and management. World J Surg. 2007;31(6):1292-1297. Epub 15 April, 2007.
7. Pavlidis TE, Atmatzidis KS, Papaziogas BT. Management of gallstone ileus. J Hepatobiliary Pancreat Surg. 2003;10(4):299-302.
8. Doko M, Zovak M, Kopljar M. Comparison of surgical treatments
of gallstone ileus: Preliminary report. World J Surg. 2003;27(4):
400-404.
9. Tan YM, Wong WK, Ooi LL. A comparison of two surgical strategies for the emergency treatment of gallstone ileus. Singapore
Med J. 2004;45(2):69-72.
10. Muthukumarasamy G, Venkata SP, Shaikh IA. Gallstone ileus: Surgical strategies and clinical outcome. J Dig Dis. 2008;9(3):156-161.
11. Riaz N, Khan MR, Tayeb M. Gallstone ileus: Retrospective review
of a single centres experience using two surgical procedures.
Singapore Med J. 2008;49(8):624-626.
12. Martnez Ramos D, Daroca Jos JM. Gallstone ileus: Management options and results on a series of 40 patients. Escrig Sos J,Rev
Esp Enferm Dig. 2009;101(2):117-120, 121-124.
13. Montgomery A. Laparoscopic-guided enterolithotomy for gallstone ileus. Surg Laparosc Endosc. 1993;4:310-314.
14. Soto DJ, Evan SJ, Kavic MS. Laparoscopic management of gallstone ileus. JSLS. 2001;5:279-285.
15. Ferraina P, Gancedo MC, Elli F. Video-assisted laparoscopic
enterolithotomy: New technique in the surgical management of
gallstone ileus. Surg Laparosc Endosc Percuntan Tech. 2003;13(2):
83-87.

PMPH_CH50.indd 402

16. Owera A, Low J, Ammori BJ. Laparoscopic enterolithotomy


for gallstone ileus. Surg Laparosc Endosc Percuntan Tech. 2008;
18(5):450-452.
17. Maglinte DD, Reyes BL, Harmon BH, et al. Reliability and role
of plain fi lm radiography and CT in the diagnosis of small-bowel
obstruction. Am J Roentgenol. 1996;167:1451-1455.
18. Lassandro F, Gagliardi N, Scuderi M. Gallstone ileus analysis of
radiologic findings in 27 patients. Eur Radiol. 2004;50:23-29.
19. Reimann AJ, Yeh, BM, Breiman RS. Atypical cases of gallstone ileus
evaluated with multidetector computed tomography. J Comput
Assist Tomogr. 2004;28(4):523-527.
20. Ripolls T, Miguel-Dasit A, Errando J. Gallstone ileus: Increased
diagnostic sensitivity by combining plain fi lm and ultrasound.
Abdom Imaging. 2001;26:401-405.
21. Yu CY, Lin CC, Shyu RY, Hsieh CB. Value of CT in the diagnosis
and management of gallstone ileus. World J Gastroenterol. 2005;
11(14):2142-2147.
22. Lassandro F, Romano S, Ragozzino A. Role of helical CT in the
diagnosis of gallstone ileus and related conditions. AJR. 2005;
185:1159-1165.
23. DePalma, GD, Mastrobuoni G, Benassai. Gallstone ileus: Endoscopic removal of a gallstone obstructing the lower ileum. Dig
Liver Dis. 2009;41:446.
24. Heinzow, HS, Meister T, Wessling J. Ileal gallstone obstruction:
Single-balloon enteroscopic removal. World J Gastrointest Endosc.
2010;2(9):321-324.
25. Zielinski MD, Ferreira LE, Baron TH. Successful endoscopic treatment of colonic gallstone ileus using electrohydraulic lithotripsy.
World J Gastroenterol. 2010;15(12):1533-1536.
26. Alsolaimana MM, Chrisoph R, Nawras AT. Bouverets syndrome
complicated by distal ileus after laser lithotripsy using Holmium:
YAG laser. BMC Gastroenterol. 2002;2(15).
27. Goldstein EB, Savel RH, Pachter HL. Successful treatment of Bouveret Syndrome using holmium: YAG laser Lithotripsy. Am Surg.
2005;71(10):882-885.
28. Gemmel C, Weikert U, Eickhoff A. Successful treatment of
gallstone ileus (Bouverets syndrome) by using extracorporeal
shock wave lithotripsy and argon plasma coagulation. Gastrointest Endosc. 2007;65(1):173-175.

5/22/2012 5:35:18 PM

CHAPTER 51

Bile Duct and Gallbladder Tumors


T. Peter Kingham and Michael DAngelica

Bile duct and gallbladder tumors are rare. Traditionally these have
been thought of as diseases with poor prognoses. This viewpoint
exists because these diseases frequently present late in their disease course and often with disseminated disease. Furthermore,
chemotherapy has traditionally had limited successes. In late
stage bile duct and gallbladder cancer this pessimism is warranted
given the low survival rates. Surgical treatments, however, have
been shown to be effective in some well selected patients. In these
patients, given improvements in staging, imaging, and surgical
resections, long-term disease-free survival is now possible.

factor for cholangiocarcinoma is primary sclerosing cholangitis (PSC). In a Swedish series of 305 patients with PSC followed
up for more than 5 years, 8% of patients eventually developed
cancer while occult cholangiocarcinoma has been reported in
40% of autopsy specimens and 36% of liver explants from
patients undergoing orthotopic liver transplantation for PSC.7
The incidence of bile duct cancer in patients with congenital
biliary cystic disease is also substantial (1520%), especially in
patients who are not treated until after age 20 years.8 Biliary
para sites (Clonorchis sinensis, Opisthorchis viverrini) are endemic
in parts of Asia and are also associated with an increased risk
of cholangiocarcinoma.9
There are three common clinical scenarios for gallbladder
cancer: (1) final pathology after routine cholecystectomy identifies gallbladder cancer; (2) gallbladder cancer is discovered
intraoperatively; and (3) gallbladder cancer is suspected prior to
surgery.10 Gallbladder cancer is notorious for being asymptomatic
in its early stages. Careful history taking often shows a history of
constant right-upper quadrant pain rather than the typical pain
of biliary colic. The diagnosis of gallbladder cancer should be
considered in an elderly patient with constant right-upper quadrant pain with weight loss or anorexia, or both. The presence of
a palpable mass or jaundice is an ominous fi nding and predicts a
high rate of unresectability. In a report by Hawkins et al.,11 82 of
240 patients (34%) presented with jaundice. Of these 82 patients,
only 6 (7%) were resectable, and all 6 of these patients experienced recurrence or died of disease within 2 years. The early
symptoms of cholangiocarcinoma are nonspecific. Abdominal
pain, anorexia, weight loss, and pruritus are the most common,
but are seen only in about one-third of patients. Fever is rarely
seen at initial presentation, but is common once biliary manipulation is initiated. Ultimately, most patients come to medical
attention with the development of jaundice. Intrahepatic cholangiocarcinomas are often diagnosed after an intrahepatic mass is
found on abdominal imaging.
Laboratory examination often is not helpful except for the typical signs of advanced disease, such as anemia, hypoalbuminemia,
leukocytosis, and elevated alkaline phosphatase or bilirubin.12,13

1. How do patients with biliary tract tumors commonly present?


Gallbladder cancer and bile duct tumors have distinct risk factors
that can help raise suspicion of their diagnosis. The risk factors for
gallbladder cancer are cholelithiasis, chronic inflammation, calcification in the wall of the gallbladder, and gallbladder polyps.
Cholelithiasis exists in 75% to 90% of patients with gallbladder
cancer.1,2 Gallstones and gallbladder cancer may, however, simply
have similar risk factors instead of a cause-and-effect relationship.
Although most (close to 90%) gallbladder cancer specimens contain stones, the incidence of gallbladder cancer in the population
of patients with stones is only 0.3% to 3%. Longstanding inflammation (porcelain gallbladder) traditionally has been associated with a
higher risk of gallbladder cancer, but modern series have shown an
incidence of less than 10%.3,4 There is some suggestion of an adenoma to carcinoma progression in the development of gallbladder
cancer. There is often severe dysplasia and carcinoma in situ adjacent to gallbladder carcinomas.1 Most small gallbladder polyps,
however, are pseudopolyps with no increased risk of malignancy.
Adenomatous gallbladder polyps are distinctly uncommon. 5,6
Current practice recommendations are to perform cholecystectomy for gallbladder polyps >1 cm in size because of a small but
significantly increased risk of malignancy.
Although the great majority of cholangiocarcinoma cases in
the United States have no underlying risk factors, there are several conditions associated with an increased incidence of cholangiocarcinoma. In the United States, the most common risk
403

PMPH_CH51.indd 403

5/22/2012 5:35:55 PM

404

Surgery: Evidence-Based Practice

The only tumor markers studied that are of any potential value
with gallbladder cancer are carcinoembryonic antigen (CEA) and
CA 19-9. An elevated CEA tends to be specific for gallbladder
cancer (90%), but lacks sensitivity (50%) when used as a screening test in cancer patients compared with patients with benign
gallbladder diseases.14 CA 19-9 is more consistent as a marker for
gallbladder cancer with sensitivities and specificities of approximately 75% at a level greater than 20 U/mL.15 Many cholangiocarcinomas express CEA and CA 19-9 as well. CA 19-9 can be
falsely elevated in a jaundiced patient. Serum levels of these markers, although elevated in some patients, have had little diagnostic
value, but can be helpful in follow-up after complete resection and
normalization.9 In jaundiced patients, the total bilirubin level may
suggest an etiology. In patients with obstruction from cholangiocarcinoma, the serum bilirubin level often achieves levels greater
than 10 mg/dL and averages 18 mg/dL, whereas patients with
obstruction from choledocholithiasis have bilirubin levels 2 to
4 mg/dL and rarely greater than 15 mg/dL; however, there is likely
to be considerable overlap.
2. What are the optimal diagnostic tests to evaluate patients
with biliary tract masses?
It is important to try to establish the diagnosis and the extent of
disease with imaging to minimize the number of patients who have
to undergo a nontherapeutic surgical exploration. In addition to
the modalities available for examining the liver, chest radiographs,
or computed tomography (CT) scan should be obtained during
the complete staging work-up to rule out pulmonary metastases.
It is, however, rare to find pulmonary metastases without locally
advanced or intra-abdominal metastatic disease.16 Percutaneous biopsy is generally only used to provide a tissue diagnosis in
patients who have metastatic or unresectable disease.
Ultrasonography is an excellent first imaging modality for
the gallbladder and liver. Findings such as discontinuous mucosa,
echogenic mucosa, and submucosal echolucency are more common in early gallbladder malignancy compared with benign
gallbladder disease. Doppler assessment of blood flow can help
to differentiate early malignancy from benign disease.17 One retrospective study reported that in 203 patients with gallbladder
cancer, a mass was identified in 177 (87%) of patients on preoperative ultrasound (US).18 US was limited, however, in identifying regional lymph node metastases. For cholangiocarcinomas,
experienced ultrasonagraphers can show the level of biliary ductal
obstruction, but also may be able to provide information regarding tumor extension within the bile duct and in the periductal
tissues.
Cross-sectional imaging with CT or magnetic resonance imaging (MRI) is an important part of the preoperative assessment
of biliary tumors. These techniques provide crucial information
about the local extent of disease and whether distant metastases
are present. In patients with gallbladder cancer, the most common finding on CT is a mass involving all or part of the gallbladder. Extension into local organs, particularly the liver, usually can
be discerned. In one study of patients with gallbladder masses,
asymmetrical wall thickening was found in 45% of patients, a
mass replacing the gallbladder was found in 35% of patients, and
an intraluminal mass was found in 20% of patients.19 Assessment
of regional and distant lymph nodes is important and can be done
with CT. CT is also an important study for evaluating patients with

PMPH_CH51.indd 404

biliary obstruction. A high-quality CT scan can provide valuable


information regarding level of obstruction, vascular involvement,
and liver atrophy.
The utility of MRI for evaluating patients with biliary tract
tumors has increased in recent years. Invasive diagnostic cholangiography has largely been replaced by MRI cholangiography in
most high-volume centers.20 Analyses of MRI for the assessment
of gallbladder cancer have shown sensitivities of 70% to 100% for
hepatic invasion and 60% to 75% for lymph node metastases.20,21
MRI cholangiopancreatography (MRCP) is a powerful investigative tool for initial preoperative assessment of hilar cholangiocarcinoma. Several studies have shown its utility in evaluating
patients with biliary obstruction.22 MRCP not only may identify
the tumor and the level of biliary obstruction, but also may reveal
obstructed and isolated ducts not appreciated at endoscopic or
percutaneous study. MRCP also provides information regarding
the patency of hilar vascular structures, the presence of nodal or
distant metastases, and the presence of lobar atrophy.
Most gallbladder cancers are positron emission tomography
(PET)-avid, so theoretically PET imaging can help differentiate
between benign and malignant tumors and diagnose extra-hepatic
spread.23 PET is more accurate in diagnosing metastatic disease
than CT scan. In 41 patients with gallbladder cancer at Memorial Sloan-Kettering Cancer Center (MSKCC) PET results altered
surgical management in 23% of patients during preoperative staging (for either the initial operation or reresection after incidental
finding of cancer after cholecystectomy) compared with 10% in
patients evaluated for recurrent disease.24
Endoscopic retrograde cholangiopancreatography (ERCP) is
a useful tool for assessing biliary tract strictures in patients who
require biliary interventions, as it permits visualization of the biliary tree with the opportunity for treatment and collection of specimens for cyto- and histopathologic evaluation. Obstruction of the
midbile duct with occlusion of the cystic duct on endoscopic cholangiography all suggest gallbladder carcinoma. The finding of a
smooth, tapered stricture on cholangiography suggests a benign
stricture. Biliary drainage should not be routinely performed in
patients who are surgical candidates.
3. How does staging of biliary tract tumors affect preoperative
planning and treatment options?
When a biliary tract malignancy is suspected on clinical and radiographic grounds, patient evaluation should turn toward an assessment of tumor resectability and assessment of hepatic and overall
physiologic reserve. The presence of significant comorbid conditions, chronic liver disease, or portal hypertension generally precludes resection. Over the years, a variety of staging systems for
gallbladder cancer have been proposed based on prognostic clinical
and pathologic factors. The American Joint Committee on Cancer
(AJCC) tumor depth nodal status metastases (TNM) staging (6th
edition) of gallbladder cancer is the standard reporting mechanism
for gallbladder cancer studies in the literature. In 2010, the 7th edition was published and is the preferred staging system clinically.
Preoperative staging assists the surgeon in deciding optimal surgical management. The standard resection for most patients with
resectable gallbladder cancer is an extended cholecystectomy that
includes segments 4b and 5 with a regional portal lymphadenectomy. In some cases major hepatic resection or bile duct resection
may be necessary due to the location of the tumor.

5/22/2012 5:35:55 PM

Bile Duct and Gallbladder Tumors

For gallbladder cancer, the AJCC 6th edition uses TNM to


place patients into four stages based on pathologic and radiographic
findings. The 7th edition was issued in 2010 with stage groupings
that were changed to correlate with resectability and patient outcome. Locally unresectable T4 tumors are classified as Stage IV and
the N classification has changed. N1 nodes are lymph nodes adjacent to the cystic duct, bile duct, hepatic artery, and portal vein.
Regional (N2) nodes are celiac, periduodenal, and peripancreatic
nodes. Lymph node metastasis is now classified as Stage IIIB (N1)
or stage IVB (N2) disease.
For cholangiocarcinoma, historically there was no clinical
staging system available that stratifies patients preoperatively
into subgroups based on potential for resection. This preoperative
stratification is important for intraoperative planning, because
intrahepatic cholangiocarcinomas are generally treated with liver
resections, hilar cholangiocarcinomas are treated with liver and
bile duct resections, and distal cholangiocarcinomas are treated
with a pancreaticoduodenectomy. AJCC TNM stage system is
based largely on pathologic criteria and has little relevance to preoperative staging since it can only be determined after resection.
Jarnagin et al. proposed a system to predict resectability, one that
improves characterization of tumor extent by taking into account
all of the available preoperative data related to local tumor extent,
as follows: (1) extent of tumor within the biliary tree, (2) vascular
involvement, and (3) lobar atrophy.25 Using such an approach, it
is possible to stage tumors preoperatively in a way that correlates
with resectability and outcome.
This preoperative clinical staging system has been compared
with the AJCC classification in a series of 87 patients with cholangiocarcinoma, with no correlation seen between AJCC stage and
resectability or median survival.25 By contrast, the proposed staging system correlated closely to resectability, which was highest in
the T1 group (59%) and decreased progressively to 0 in the T3
group. The clinical T stage grouping was an independent predictor not only of resectability, but also of the likelihood of achieving
an R0 resection.
Patients with hilar cholangiocarcinoma almost always require
major liver resection, so their preoperative management must be
optimized carefully. Obstructive jaundice associated with perihilar cholangiocarcinoma differs from that associated with middle or
distal bile duct cancer, as it often requires multiple biliary drainage
catheters of the future liver remnant (FLR). For both endoscopic
and percutaneous biliary drainage, the intrahepatic bile duct of
the predicted FLR is intubated or punctured and subsequently
drained. Preoperative biliary drainage improves cholestatic liver
disease and affords safer performance of hepatectomy for hilar
cholangiocarcinoma; especially when the FLR is small.
Portal vein embolization (PVE) has become an important
consideration in the presurgical management of patients who are
to undergo extensive liver resection. The ultimate goal of PVE is
to minimize postoperative liver dysfunction by inducing compensatory hypertrophy in the FLR. In right PVE, the volume of
the left lobe increases by 130 cm3 on average within 2 weeks after
embolization, and the estimated resection volume decreases by
an average of approximately 10%.26 Biliary drainage is required
prior to PVE if the biliary tree is obstructed on the side to be
embolized. A single prospective trial in nonjaundiced patients
undergoing right hemi-hepatectomy demonstrated a benefit in
patients with chronic liver disease, but no benefit in patients with
normal livers.27

PMPH_CH51.indd 405

405

4. What are the results for resection of biliary tract tumors?


If the diagnosis of gallbladder cancer is made by frozen section
in the operating room the operating surgeon should prepare for a
curative resection or arrange for transfer of the patient to an experienced hepatobiliary surgeon. Although not ideal management, this
situation may not affect outcomes, as a study at MSKCC reported
outcomes after prior noncurative resection were similar to outcomes from primary curative resections.28 Ouchi et al. reported on
498 patients diagnosed with gallbladder cancer after laparoscopic
cholecystectomy. They found 34% had T1a tumors, 14% had T1b,
41% had T2, 8% had T3, and 2% had T4 tumors.29

T1 GALLBLADDER TUMORS
T1 tumors typically are diagnosed incidentally in cholecystectomy
specimens because they are usually not obvious on imaging studies. By definition, they do not penetrate through the muscular layer,
and because a simple cholecystectomy dissects the perimuscular
layer, this operation theoretically should be curative. With T1a
tumors, if margins are negative, standard cholecystectomy cures
85% to 100% of patients.30,31 Controversy exists with T1b tumors.
Principe et al. demonstrated a 50% 1-year survival in patients with
T1b gallbladder cancers after simple cholecystectomy.31,32 Other
series, however, report cure rates for T1b tumors as 90% to 100%
at 5 years.33 These results are similar to the published results for
extended cholecystectomies for T1b tumors, with survival rates
above 90% at 5 years. In a fit patient, it is recommended to treat
T1b tumors with a segment 4b,5 liver resection with portal node
dissection given the association of liver resection with improved
survival in some retrospective series.

T2 TUMORS
T2 tumors are those most likely to benefit from an extended resection of the liver and porta hepatis lymph nodes. Because these tumors
also are difficult to diagnose preoperatively, they also are commonly
diagnosed incidentally at cholecystectomy. The dissection plane of
a simple cholecystectomy in the subserosal plane is often involved
with tumor, resulting in a positive margin for many T2 tumors. In
addition, approximately one-third of patients with T2 tumors have
nodal metastases, reinforcing the need for a regional lymphadenectomy for diagnostic and potentially therapeutic purposes.34 Based
on retrospective comparisons survival is significantly improved in
patients undergoing an extended cholecystectomy and this operation is standard of care for patients with T2 tumors.

T3/T4 TUMORS (LOCALLY ADVANCED)


The most controversial aspect of the surgical treatment of gallbladder cancer involves patients with nonmetastatic locally advanced
tumors. Since the 1990s, numerous small series have documented
that with varying levels of extended resections, long-term survival
is possible in highly selected patients. Japanese surgeons liberally
use extensive surgical procedures, including vascular resection,
bile duct resection, extended hepatectomy, and hepatopancreaticuoduodenectomy.35-37 When extensive lymph node dissections are

5/22/2012 5:35:55 PM

406

Surgery: Evidence-Based Practice

performed routinely and survival results may be a reflection of


stage migration and better staging. Western series also have begun
to show similar results with an aggressive surgical approach. In
our own series from MSKCC, approximately 25% of patients with
locally advanced disease had long-term survival; usually accounted
for by the significant fraction of patients with lymph node negative
disease.28 These results, however, must be interpreted in the proper
context. Overall, most patients with locally advanced disease have
metastatic disease, and resection is usually not possible. The most
important associated prognostic factor in every series is the status
of lymph nodes. Long-term survival is rarely found with distant
lymph node disease and is uncommon with hepatoduodenal lymph
node metastases.
Long-term survival can be achieved with an acceptable operative mortality in patients with cholangiocarcinoma as well.25
For distal cholangiocarcinomas a pancreaticoduodenectomy
required for adequate resection. For hilar cholangiocarcinomas
and intrahepatic cholangiocarcinomas a liver resection is required to obtain an R0 resection. Five-year survival rates range
from 25% to 40%. The status of the resection margins is crucial to
obtaining such results. Several studies have shown that patients
resected with negative histologic margins survive significantly
longer than patients with involved margins.25,38 Hepatic resection is often critical to obtain negative margins. In addition to
the status of the resection margin, many other variables have
been shown to have an impact on outcome, including tumor
involvement of regional lymph nodes, tumor grade or differentiation, and tumor morphology (e.g., papillary versus nodularsclerosing). 25 The issue of lymph node metastases, their impact
on outcome, and extent of lymphadenectomy remain points of
some controversy.
5. What is the role of adjuvant therapies for biliary tract tumors?
The rarity of biliary tract cancers has precluded large randomized
trials, and the literature abounds in small case series and retrospective comparisons that attempt to address whether adjuvant
chemotherapy or radiation or both are beneficial after complete
resection. In a study from MSKCC, it was found that 85% of gallbladder cancer recurrences included distant sites, 39 highlighting
the importance of systemic therapies.
There is only one randomized trial that examined the role
of adjuvant chemotherapy in patients with biliary tract tumors.
Takada et al.40 reported a phase III multi-institutional trial of
adjuvant chemotherapy performed in Japan. This trial included
508 patients with biliary and pancreatic cancers. On subset analysis, this study included 140 gallbladder cancer patients who were
randomized to receive surgical resection alone or resection plus
adjuvant mitomycin C and 5-fluorouracil (5-FU). In considering
only the gallbladder cancer patients, the actuarial 5-year diseasefree survival favored the adjuvant chemotherapy group (20.3%)
when compared with the surgery alone group (11.6%, p = .02).
Data regarding the extent of surgical resection and histologic
staging were lacking, making it difficult to interpret these encouraging results. Th is study found no benefit of adjuvant systemic
5-FU and mitomycin compared with surgery alone for patients
with bile duct carcinoma.
There are several retrospective studies that have examined
the outcomes of patients treated with adjuvant chemotherapy
after resection for gallbladder cancer. One such series from Mayo

PMPH_CH51.indd 406

Clinic analyzed 73 patients, 25 of whom received adjuvant chemoradiotherapy.41 Adjuvant chemoradiotherapy was a significant
predictor of improved overall survival after adjusting for prognostic factors (T and N stage, histologic findings). In general, the use
of adjuvant chemotherapy and or radiation after complete resection of gallbladder or biliary cancers is unproven and used on an
individualized basis.
6. What palliative options are available to treat patients with
unresectable biliary tract tumors?
A common problem in patients with biliary tract cancers is palliation, because most patients are unresectable at presentation. The
most common symptoms to palliate include pain, jaundice, and
bowel obstruction. In the past, operative approaches provided
the most effective relief of obstruction in well-selected patients. In patients who are symptomatic from obstructive jau ndice
with pruritus or cholangitis, endoscopic or percutaneous interventions are the preferred approach for palliation and minimizing morbidity.42-44 Saluja et al.45 randomized 44 patients
with gallbladder cancer and obstructive jaundice to percutaneous or endoscopic stenting. They found successful stenting
occurred in 89% of patients in the percutaneous group compared with 41% in the endoscopic group (p < .001). Cholangitis was also significantly higher (48%) in the endoscopic stent
group compared with the percutaneous group (11%; p = .002).
The mortality rates were similar between the two groups, but
percutaneous stenting in this trial offered better palliation than
endoscopic stenting. Radiation therapy may also be an effective palliative therapy for locally advanced disease. Radiation
therapy is generally well tolerated, may have an impact on
local symptoms, and is usually combined with chemotherapy.46
Operative biliary-enteric bypass, either open or laparoscopic,
will provide excellent relief of jaundice and can be performed
with an acceptably low morbidity and mortality. Patients who
are found to have unresectable disease at the time of operative
exploration should be considered for a biliary-enteric bypass if
technically feasible.
When considering biliary drainage in patients with malignant
biliary obstruction in the proximal biliary tree the overall prognosis, specific symptoms and complexity of the stricture must be
considered. If symptoms are manageable and the overall prognosis
is poor, management without drainage should be considered. In
general, patients in need of palliative procedures for symptomatic proximal biliary tract cancers have high rates of complications and early death due to disease after palliative procedures. A
prospective study from MSKCC included 109 patients undergoing
percutaneous biliary drainage for malignant obstruction.47 There
was a 58% rate of major complication and two procedure-related
deaths. The 8-week postprocedure mortality rate was 28%; largely
due to progressive malignancy. These palliative procedures were
successful in alleviating pruritus, but quality of life measures continued to decline.
Recently, gemcitabine-based regimens, often combined with
a platinum agent, have become the drug of choice for oncologists
in treating patients with unresectable and metastatic gallbladder or biliary cancer. A recently published randomized Phase III
study of gemcitabine compared with gemcitabine with cisplatin
for metastatic gallbladder and bile duct cancer has established
the combination therapy as standard of care.48 Four hundred and

5/22/2012 5:35:55 PM

Bile Duct and Gallbladder Tumors

ten patients with locally advanced or metastatic gallbladder cancer, cholangiocarcinoma, or ampullary cancers were randomized
with overall survival as the primary endpoint. After a median
follow-up of 8 months, there was a significant difference in overall
survival, with a hazard ratio of 0.57 to 0.61 for in patients with
gallbladder cancer, intrahepatic cholangiocarcinoma, and extrahepatic cholangiocarcinomas favoring the gemcitabine-cisplatin
combination.48 Another smaller randomized controlled trial of
gemcitabine combined with oxaliplatin (GEMOX) showed a similar benefit when the gemcitibine regimen was compared with 5-FU
or best supportive care in patients with unresectable or metastatic
gallbladder cancer. Median overall survival was 9.5 months in the
GEMOX group (n = 27), compared with 4.6 months in the 5-FU
group (n = 28; p = .039).

407

SUMMARY
Improvements in high-resolution cross-sectional imaging have
permitted better patient selection and preoperative planning and
preparation prior to the safe performance of operations for hilar cholangiocarcinoma. Long-term disease-free survival, rather than palliation, is now the primary goal of operative resection. Judicious use
of adjunctive preoperative interventions including biliary drainage
and PVE may help to improve outcomes especially when major periand postoperative complications are anticipated. Selection of appropriate nonoperative therapies for palliation of unresectable tumors
arising from the proximal and distal bile ducts should be tailored
according to the patients expected longevity and technical expertise of
the multi-disciplinary team charged with treating bile duct cancer.

Clinical Question Summary


Question

Answer

1 How do patients with


biliary tract tumors
commonly present?

Early symptoms are nonspecific. Jaundice,


weight loss, and abdominal pain are all
presenting symptoms.

1-15

2 What are the optimal


diagnostic tests to
evaluate patients with
biliary tract masses?

US is a useful first imaging study. This can


be followed by CT scan or MRI. MRCP
has replaced ERCP for diagnostic imaging.
Biopsy of resectable tumors is not
necessary.

16-25

3 How does staging of


biliary tract tumors
affect preoperative
planning and treatment
options?

Staging determines which patients are


candidates for surgical resection.
Staging tests can provide information to
determine the need for PVE or biliary
drainage.

26, 27

4 What are the results


for resection of biliary
tract tumors?

For patients with resectable gallbladder


cancers, median survival is approximately
13 months. Lymph node positivity is an
important prognostic factor. For hilar
cholangiocarcinomas and intrahepatic
cholangiocarcinomas a liver resection is
required to obtain an R0 resection. Fiveyear survival rates after resection range
from 25% to 40%.

28-38

5 What is the role of


adjuvant therapies for
biliary tract tumors?

There are limited data to recommend


adjuvant therapy. One randomized
trial showed a benefit with adjuvant
chemotherapy. Several retrospective
series suggest an association between
adjuvant therapy and improved survival.

1B

39-41

6 What palliative options


are available to
treat patients with
unresectable biliary
tract tumors?

Gemcitabine-based regimens, often


combined with a platinum chemotherapy,
have shown an improved overall survival
in patients with metastatic biliary tract
tumors. There are endoscopic and
percutaneous biliary drainage procedures
that can successfully palliate most
patients.

1B

42-48

PMPH_CH51.indd 407

Level of
Evidence

Grade of
Recommendation

References

5/22/2012 5:35:55 PM

408

Surgery: Evidence-Based Practice

REFERENCES
1. Lazcano-Ponce EC, et al. Epidemiology and molecular pathology
of gallbladder cancer. CA Cancer J Clin. 2001;51(6):349-364.
2. Serra I, Diehl AK. Number and size of stones in patients with
asymptomatic and symptomatic gallstones and gallbladder carcinoma. J Gastrointest Surg. 2002;6(2):272-273; author reply 273.
3. Berk RN, Armbuster TG, Saltzstein SL. Carcinoma in the porcelain gallbladder. Radiology. 1973;106(1):29-31.
4. Kim JH, et al. Should we perform surgical management in all
patients with suspected porcelain gallbladder? Hepatogastroenterology. 2009;56(93):943-945.
5. Ito H, et al. Polypoid lesions of the gallbladder: Diagnosis and
follow up. J Am Coll Surg. 2009;208(4):570-575.
6. Zielinski MD, et al. Comparison of surgically resected polypoid
lesions of the gallbladder to their pre-operative ultrasound characteristics. J Gastrointest Surg. 2009;13(1):19-25.
7. Broome U, et al. Natural history and prognostic factors in 305
Swedish patients with primary sclerosing cholangitis. Gut. 1996;
38(4):610-615.
8. Lipsett PA, et al. Choledochal cyst disease. A changing pattern of
presentation. Ann Surg. 1994;220(5):644-652.
9. Pitt HA, et al. Malignancies of the biliary tree. Curr Probl Surg. 1995;
32(1):1-90.
10. Miller G, Jarnagin WR. Gallbladder carcinoma. Eur J Surg Oncol.
2008;34(3):306-312.
11. Hawkins WG, et al. Jaundice predicts advanced disease and early
mor tality in patients with gallbladder cancer. Ann Surg Oncol.
2004;11(3):310-315.
12. Grobmyer SR, Lieberman MD, Daly JM. Gallbladder cancer in the
twentieth century: Single institutions experience. World J Surg.
2004;28(1):47-49.
13. Thorbjarnarson B, Glenn F. Carcinoma of the gallbladder. Cancer.
1959;12:1009-1015.
14. Strom BL, et al. Serum CEA and CA 19-9: Potential future
diagnostic or screening tests for gallbladder cancer? Int J Cancer.
1990;45(5):821-824.
15. Ritts RE, Jr., et al. Comparison of preoperative serum CA19-9 levels
with results of diagnostic imaging modalities in patients undergoing laparotomy for suspected pancreatic or gallbladder disease.
Pancreas. 1994;9(6):707-716.
16. Lee JH, et al. Clinical usefulness of preoperative and postoperative
chest computed tomography for colorectal cancer. J Korean Soc
Coloproctology. 2010;26(5):359-364.
17. Sato M, et al. Localized gallbladder carcinoma: Sonographic fi ndings. Abdom Imaging. 2001;26(6):619-622.
18. Pandey M, et al. Carcinoma of the gallbladder: Role of sonography
in diagnosis and staging. J Clin Ultrasound. 2000;28(5):227-232.
19. Kalra N, et al. MDCT in the staging of gallbladder carcinoma. AJR
Am J Roentgenol. 2006;186(3):758-762.
20. Schwartz LH, et al. Gallbladder carcinoma: Findings at MR imaging with MR cholangiopancreatography. J Comput Assist Tomogr.
2002;26(3):405-410.
21. Kim JH, et al. Preoperative evaluation of gallbladder carcinoma:
Efficacy of combined use of MR imaging, MR cholangiography, and contrast-enhanced dual-phase three-dimensional MR
angiography. J Magn Reson Imaging. 2002;16(6):676-684.
22. Schwartz LH, et al. MRI as an alternative to CT-guided biopsy
of adrenal masses in patients with lung cancer. Ann Thorac Surg.
1998;65(1):193-197.
23. Petrowsky H, et al. Impact of integrated positron emission tomography and computed tomography on staging and management

PMPH_CH51.indd 408

24.

25.

26.

27.
28.

29.

30.
31.

32.

33.

34.

35.
36.
37.
38.

39.

40.

41.

42.

43.

44.

of gallbladder cancer and cholangiocarcinoma. J Hepatol. 2006;


45(1):43-50.
Corvera CU, et al. 18F-fluorodeoxyglucose positron emission
tomography influences management decisions in patients with
biliary cancer. J Am Coll Surg. 2008;206(1):57-65.
Jarnagin WR, et al. Staging, resectability, and outcome in 225
patients with hilar cholangiocarcinoma. Ann Surg. 2001;234(4):
507-517; discussion 517-519.
Nagino M, et al. Changes in hepatic lobe volume in biliary tract
cancer patients after right portal vein embolization. Hepatology.
1995;21(2):434-439.
Farges O, et al. Portal vein embolization before right hepatectomy: Prospective clinical trial. Ann Surg. 2003;237(2):208-217.
Fong Y, Jarnagin W, Blumgart LH. Gallbladder cancer: Comparison of patients presenting initially for definitive operation with
those presenting after prior noncurative intervention. Ann Surg.
2000;232(4):557-569.
Ouchi K. Mikuni J, Kakugawa Y. Laparoscopic cholecystectomy
for gallbladder carcinoma: Results of a Japanese survey of 498
patients. J Hepatobiliary Pancreat Surg. 2002;9(2):256-260.
Yamaguchi K. Tsuneyoshi M. Subclinical gallbladder carcinoma.
Am J Surg. 1992;163(4):382-386.
Shirai Y, et al. Inapparent carcinoma of the gallbladder. An appraisal
of a radical second operation after simple cholecystectomy. Ann
Surg. 1992;215(4):326-331.
Principe A, et al. Radical surgery for gallbladder carcinoma:
Possibilities of survival. Hepatogastroenterology. 2006;53(71):
660-664.
Kang CM, et al. Does clinical R0 have validity in the choice of
simple cholecystectomy for gallbladder carcinoma? J Gastrointest
Surg. 2007;11(10):1309-1316.
Chijiiwa K, et al. Surgical treatment of patients with T2 gallbladder carcinoma invading the subserosal layer. J Am Coll Surg. 2001;
192(5):600-607.
Nakamura S, et al. Aggressive surgery for carcinoma of the gallbladder. Surgery. 1989;106(3):467-473.
Shirai Y, et al. Radical surgery for gallbladder carcinoma. Longterm results. Ann Surg. 1992;216(5):565-568.
Onoyama H, et al. Extended cholecystectomy for carcinoma of
the gallbladder. World J Surg. 1995;19(5):758-763.
Endo I, et al. Clinical significance of intraoperative bile duct margin assessment for hilar cholangiocarcinoma. Ann Surg Oncol.
2008;15(8):2104-2112.
Jarnagin WR, et al. Patterns of initial disease recurrence after
resection of gallbladder carcinoma and hilar cholangiocarcinoma:
Implications for adjuvant therapeutic strategies. Cancer. 2003;
98(8):1689-1700.
Takada T, et al. Is postoperative adjuvant chemotherapy useful
for gallbladder carcinoma? A phase III multicenter prospective
randomized controlled trial in patients with resected pancreaticobiliary carcinoma. Cancer. 2002;95(8):1685-1695.
Gold DG, et al. Adjuvant therapy for gallbladder carcinoma:
The Mayo Clinic Experience. Int J Radiat Oncol Biol Phys. 2009;
75(1):150-155.
Naitoh I, et al. Unilateral versus bilateral endoscopic metal stenting for malignant hilar biliary obstruction. J Gastroenterol Hepatol. 2009;24(4):552-557.
Lung PF, et al. Novel deployment of a covered duodenal stent in
open surgery to facilitate closure of a malignant duodenal perforation. World J Surg Oncol. 2009;7:79.
Togawa O, et al. Management of occluded uncovered metallic stents in patients with malignant distal biliary obstructions

5/22/2012 5:35:55 PM

Bile Duct and Gallbladder Tumors

using covered metallic stents. J Clin Gastroenterol. 2008;42(5):


546-549.
45. Saluja SS, et al. Endoscopic or percutaneous biliary drainage for
gallbladder cancer: A randomized trial and quality of life assessment. Clin Gastroenterol Hepatol. 2008;6(8):944-950 e3.
46. Lin LL, et al. A phase II study of alternating cycles of split course
radiation therapy and gemcitabine chemotherapy for inoperable

PMPH_CH51.indd 409

409

pancreatic or biliary tract carcinoma. Am J Clin Oncol. 2005;


28(3):234-241.
47. Robson PC, et al. Prospective study of outcomes after percutaneous
biliary drainage for malignant biliary obstruction. Ann Surg Oncol.
2010;17(9):2303-2311.
48. Valle J, et al. Cisplatin plus gemcitabine versus gemcitabine for
biliary tract cancer. N Engl J Med. 2010;362(14):1273-1281.

5/22/2012 5:35:55 PM

CHAPTER 52

Obstructive Jaundice: Transhepatic


and Endoscopic Interventions
Brian J. Dunkin

INTRODUCTION

guide therapy. Patients with no CBD stones could proceed directly


to cholecystectomy, while those with stones could have preoperative ERCP. An accurate prediction strategy would assure that
patients who do not require ERCP will not get it and those that
do have CBD stones are not inadvertently missed or discovered
incidentally during surgery.
Single-risk factors examined to predict CBD stones focus on
individual factors such as elevated liver function tests (LFTs) or a
dilated CBD on ultrasound. Elevated bilirubin alone as a risk factor has a 31% to 56% sensitivity, 48% to 99% specificity, with a positive predictive value (PPV) of 42% to 95%.4-12 Studies vary as to
what is considered an abnormal bilirubin ranging from anything
above the normal limits as listed by a laboratory to greater than
three times normal. In general, the sensitivity of an elevated bilirubin is low, specificity higher, and PPV relatively high. Abnormal
LFTs other than bilirubin have also been studied and sensitivities,
specificity, and PPV values vary. A tradeoff is noted between sensitivity and specificity with cutoff values chosen for high specificity decreasing sensitivity. Dilated CBD (range >5 to >10 mm) on
ultrasound has a sensitivity of 28% to 94%, specificity of 72% to
98%, and PPV of 28% to 91%.12,13
Four studies have looked at multivariable risk factors for
predicting CBD stones with one of the four testing their model
prospectively.10,12-14 This prospective evaluation by Trondsen et al.
demonstrated 94% sensitivity and 84% specificity.12 Despite the
apparent excellent performance of multivariable risk-factor models
for predicting CBD stones, they have not become widely integrated
into clinical practice.
Studies have also investigated the ability to predict the
absence of CBD stones by studying whether or not an absence
of any of the previously mentioned factors predicts no CBD
stones.9,11-16 In the absence of any abnormality in LFTs or CBD
dilation by ultrasound there is 0.25% to 7% chance of having
a CBD stone which is comparable with the 0% to 17% negative
predictive value of ERCP.
Answer: The best method of predicting CBD stones based on
laboratory and ultrasound evidence is to look for an absence of
abnormality (i.e., normal LFTs and no CBD dilation on ultrasound).

A large variety of biliary and pancreatic pathologies cause jaundice. Gallstones, pancreatic malignancy, and biliary malignancy
are the most common reasons for obstructive jaundice and this
chapter focuses on the nonsurgical management of these diseases.
It is estimated that 6 per 100,000 people in the United States are
afflicted with common bile duct (CBD) stones and that over 57,000
people per year are diagnosed with malignancy of the pancreas or
biliary tree.1
Percutaneous transhepatic cholangiography (PTC) was described as early as 1937 by Huard-Du, but did not gain widespread
use until the introduction of the Chiba needle by Okuda of Japan
in 1973.2 In 1968 McCune, Shorb, and Moskovitz performed the
first endoscopic retrograde cholangiopancreatography (ERCP).3
The procedure was done intraoperatively using a modified endoscope with an external channel taped to the side. Their cannulation success rate was 25%. Since these initial reports both PTC and
ERCP have evolved into sophisticated diagnostic and therapeutic
procedures. This chapter reviews the best evidence for using these
modalities to manage jaundice from obstruction of the biliary tree.
It is divided into three sections: CBD Stones, benign CBD strictures,
and malignant CBD strictures.

CBD STONES
Stones in the CBD are the most common cause of obstructive
jaundice. ERCP has become a mainstay of management with successful clearance of the CBD approaching 100% in experienced
programs. Questions that arise around CBD stone management
center on how to predict their presence, discerning effective noninvasive and invasive modalities for visualizing them, and when
to apply endoscopic or surgical therapies.
1. What is the best way to predict CBD stones?
In patients with symptomatic cholelithiasis it would be useful to
have a noninvasive way to predict the presence of CBD stones to help
410

PMPH_CH52.indd 410

5/22/2012 5:36:36 PM

Obstructive Jaundice: Transhepatic and Endoscopic Interventions

This strategy has a negative predictive value comparable with diagnostic ERCP. Multivariable risk-factor formulas also have good
accuracy, but have not been widely adopted in clinical practice.
Univariate factor analysis has the least accuracy. (Level 2 evidence;
Grade B recommendation).
2. How does diagnostic ERCP compare with noninvasive imaging
modalities?
Understanding the accuracy of diagnostic ERCP in identifying CBD stones serves as a comparison with other noninvasive
modalities. However, most studies compare various diagnostic
modalities with ERCP making it impossible to know the real
performance of ERCP itself. Sugiyama et al.17 compared diagnostic ERCP with ERCP with endoscopic sphincterotomy (ES)
and balloon sweep of the CBD in the same patient when a stone
was thought to be found, and demonstrated a 100% sensitivity
of ERCP to discover stones. This compares with 90% sensitivity
for magnetic resonance cholangiopancreatography (MRCP) and
endoscopic ultrasound (EUS), and 71% for computed tomographic
cholangiography (CTC).17-31
Answer: Because of the excellent performance of MRCP and
EUS in detecting CBD stones, ERCP should not be used as a diagnostic tool alone particularly given its associated higher rate of
potential complications. CTC is not as accurate as MRCP and
EUS. (Level 2 evidence; Grade B recommendation).
3. What is the optimal treatment of suspected CBD stones: pre/
postoperative ERCP or intraoperative cholangiogram (IOC)
and CBD exploration?
There are essentially three options for managing patients with a
possible CBD stone: (1) preoperative ERCP and duct clearance
followed by cholecystectomy; (2) IOC with laparoscopic common bile duct exploration (LCBDE) if stones are found; and
(3) IOC with cholecystectomy and postoperative ERCP if CBD
stones are found.
Chung et al.32 investigated preoperative ERCP versus postoperative ERCP in gallstone pancreatitis patients who had CBD
stones identified on IOC. Both strategies were successful at clearing CBD stones in all patients, but over half of the patients having
preoperative ERCP had a negative study. Hospital length of stay
(LOS) and costs were higher in the preoperative ERCP group.
Cuschieri et al. 33 prospectively studied preoperative ERCP
versus IOC and LCBDE. Both techniques were successful in
clearing the CBD stones 84% of the time, but hospital LOS
was shorter in the LCBDE group. Interestingly, more patients
were converted to open surgery in the LCBDE group compared with preoperative ERCP, but this did not reach statistical
significance.
Rhodes et al. compared LCBDE with postoperative ERCP on
those patients with CBD stones identified on IOC.36 Both techniques had similar success in clearing the CBD of stones, but the
LOS was three times longer for the postoperative ERCP group.
Answer: The efficacy to clear CBD stones and procedurerelated morbidity are essentially equal among the three strategies.
However, IOC followed by LCBDE is more cost effective with
shorter LOS. Given that both ERCP and LCBDE are operatordependent procedures, the strategy employed at any single institution must take into account the available expertise. (Level 2
evidence; Grade B recommendation).

PMPH_CH52.indd 411

411

4. What is the expected success rate of ERCP to clear CBD stones


and can it serve as definitive therapy?
The success rate to clear CBD stones using ERCP and sphincterotomy plus balloon or basket extraction is operator dependent. Carr-Locke et al. 34 reported a greater than 90% success
rate with a 5% complication rate and 1% mortality. Other studies reported 75% to 100% initial success with a 93% to 100% final
clearance rate. 33,35,36 The National Institutes of Health (NIH)
state-of-the-science statement on ERCP reports that successful ERCP and CBD clearance should be achieved 90% of the
time. 37
Some have investigated if ERCP, ES, and clearance of the CBD
alone can serve as definitive therapy without having to remove the
gallbladder in patients that are at high surgical risk. Targarona
et al.38 compared ERCP with open cholecystectomy and CBDE
and found that ERCP had a higher initial failure rate, readmission
rate, and more complications from biliary disease, but the LOS
and immediate morbidity was lower in the ERCP group. Trias
et al.39 compared ERCP with laparoscopic cholecystectomy in the
same cohort of high-risk patients and found more long-term biliary complications, readmissions, and need for repeat surgery in
the ERCP group. LOS and immediate morbidity were similar
in both groups.
Answer: ERCP for removal of CBD stones should be successful at least 90% of the time (Level 2 evidence; Grade B recommendation). There is limited data on the use of ERCP alone for
definitive management of CBD stones, but the available studies
indicate that open or laparoscopic cholecystectomy can be performed in these high-risk patients with acceptable short-term
morbidity and equivalent mortality to that of ERCP with less
recurrence of biliary symptoms or need for hospital readmission.
It is reasonable to choose ERCP for definitive therapy in patients
deemed too ill to tolerate cholecystectomy understanding that
some studies put the 5-year risk of serious biliary complications
leading to cholecystectomy at 10% to 15%.40 (Level 3 evidence;
Grade C recommendation).
5. Can ERCP with ES and/or stent placement be used to palliate pregnant women with CBDS until after delivery?
Gallstone disease is common during pregnancy with acute
cholecystitis being the second most common nonobstetric
emergency in pregnant women. Given that there is an increased
risk for complications to the fetus when these patients undergo
general anesthesia and abdominal surgery, it may be advantageous to clear the CBD using ERCP and rely on ES and/or biliary
stenting to protect the patient from further attacks of choledocholithiaisis or gallstone pancreatitis until after delivery of the
baby. There are no prospective studies examining this question
and the literature reports only institution case series. However,
there is some data on using ERCP with ES and stone extraction versus placement of a biliary stent to manage CBD stones
in high-risk, nonpregnant patients that may support using these
techniques during pregnancy. Chopra et al.41 compared biliary
stent placement to ES and stone extraction in patients with CBD
stones that were at high risk for surgery because of advanced
age or debilitating dementia. The rate of immediate complications was similar in each group, but the ES group had less biliary complications at 20 months compared with the stent group
(14% vs. 36% p < .03).41

5/22/2012 5:36:36 PM

412

Surgery: Evidence-Based Practice

Since ERCP uses ionizing radiation, which can lead to chromosomal mutations, neurologic abnormalities, mental retardation, and increased risk of childhood leukemia, exposure to the
fetus must be minimized. Fetal mortality is greatest when exposure occurs in the first week of conception and it is recommended
that the cumulative radiation dose to the conceptus during pregnancy be less than 5 to 10 rads.42 Multiple studies report that
ERCP can be accomplished using little to no radiation at minimal
risk to the fetus.43
Answer: There is no high-level data investigating the use of
ERCP with ES and/or stent placement to palliate CBD obstruction from stones until after pregnancy. Case series data on managing nonobstetric high-risk surgery patients with ERCP and ES
or stenting report a significant rate of biliary complications at 20
months with less from ES than stenting. When performing ERCP
in pregnant patients, radiation exposure should be kept below
10 rads which can be easily accomplished in experienced hands.
(Level 3 evidence; Grade C recommendation).
6. What is the best method of gaining access to the CBD
following Roux-en-Y gastric bypass?
The rise of the surgical management of morbid obesity has resulted
in an increase in the number of patients with CBD pathology that
have altered foregut anatomy. The Roux-en-Y gastric bypass is the
most common procedure for morbid obesity and gaining endoscopic access to the ampulla of Vater after this procedure is challenging. There is no consensus as to the best approach to manage
these patients with the literature reporting only case reports and
small case series.
There are four approaches to gaining access to the biliary
system post Roux-en-Y gastric bypass. Intraoperative ERCP with
laparoscopic access to either the gastric remnant or the pancreaticobiliary limb is perhaps the most common method. The surgeon
gains access to the gastric remnant, creates a gastrotomy, and then
works with the endoscopist to introduce the duodenoscope across
the gastric wall, through the gastrotomy, and into the duodenum.
A similar approach can be used via the jejunum either at a de novo
jejunostomy or through an opening created in the entero-entrostomy. In nonemergent conditions, percutaneous access to the gastric remnant has also been described with placement of a temporary
gastrostomy tube. Over a number of weeks, the tube is upsized
until the opening is large enough to accommodate a pediatric
duodenoscope. ERCP is then performed working through the
gastrostomy tube tract.44 Another method of access is via PTC.
A skilled interventional radiologist can often gain access even to
nondilated bile ducts, but is limited in the therapy that can be
delivered for managing CBD stones. Finally, enteroscopy can be
used to gain retrograde access to the ampulla of Vater. Th is may
be performed with a pediatric colonoscope, push enteroscope,
balloon enteroscope, or with the assistance of an overtube
device. Th is method is limited not only by the difficulty in navigating the entero-enterostomy and traveling retrograde up to
the second portion of the duodenum, but also by the limitation
of ERCP tools that can be introduced through a colonoscope or
enteroscope.
Answer: There is no consensus in the literature about the best
approach to the papilla of Vater after Roux-en-Y gastric bypass.
The choice of approach depends on available expertise and equipment. (No Level 1, 2, or 3 evidence; no recommendation).

PMPH_CH52.indd 412

BENIGN CBD STRICTURES


Benign strictures of the CBD may occur from a variety of causes,
but most frequently from chronic pancreatitis with compression
of the intrapancreatic portion of the CBD or from anastomotic
strictures related to bilio-enteric or bilio-bilio anastomosis.
This section explores the endoscopic management of these
strictures.
7. Can biliary strictures from chronic pancreatitis be relieved
with dilation and stenting?
Bile duct strictures from chronic pancreatitis can be difficult to
manage endoscopically with long-term success rates as low as
10%.45,46 Calcifications in the pancreatic head predicts an even
worse outcome of 7.7% at 1 year.46 Results can be improved if multiple stents are placed side-by-side and left in place for up to 14
months.47 Using this technique with up to five stents placed simultaneously, Catalano et al.47 were able to achieve 100% success in
normalizing LFTs with no episodes of cholangitis at 4.2 years mean
follow-up. More recently, self-expanding metal stents (SEMS) have
been used to treat benign biliary strictures. These stents may be
uncovered, partially covered, or fully covered. Published data
using SEMS for benign biliary strictures is limited to small singleinstitution series and case reports. However, based on this available
data, plastic stents appear to be more efficacious than uncovered
SEMS, and partially or fully covered SEMS seem to be equivalent
to multiple plastic stents in patency and require less endoscopic
interventions.48-50
Answer: Benign biliary strictures from chronic pancreatitis
can be successfully managed with endobiliary stenting, but require
placement of multiple plastic stents left in place for up to 14 months.
The data is evolving on the use of partially or fully covered SEMS
for this purpose, but preliminary reports are encouraging. Calcifications at the head of the pancreas may predict worse outcomes.
(Level 2 evidence; Grade B recommendation).
8. How does management of biliary strictures from primary
sclerosing cholangitis (PSC) and anastomotic stenosis compare
with chronic pancreatitis?
Biliary strictures from PSC respond well to endoscopic management
and seem less refractory to treatment than those from chronic pancreatitis. They may be treated with dilation alone or in combination
with stenting. Limited published data is available on the optimal
approach, but it appears that balloon dilation alone is most effective
with stent placement increasing the risk of complications and cholangitis.51 Dilation of dominant strictures from PSC may improve
survival, but does not delay liver transplant.52 These strictures should
also be brushed or biopsied to ensure they are not malignant.53
Benign postoperative anastomotic bile duct strictures are also
amenable to endoscopic management and are less refractory to
treatment than those from chronic pancreatitis. Long-term clinical
success rates range from 55% to 88% overall.54 Bile duct strictures
after liver transplant may be successfully managed with dilation
and stenting with reports as high as 91% to 100% success early after
transplant; however, late postoperative strictures may be less amenable to this mode of treatment with as low as 8% success.55-57
Answer: Biliary strictures from PSC are more amenable to
dilation alone when compared with strictures from chronic pan-

5/22/2012 5:36:36 PM

Obstructive Jaundice: Transhepatic and Endoscopic Interventions

creatitis, and placement of a stent in these patients is not necessary


and may increase the risk of cholangitis. Postoperative anastomotic biliary strictures are also amenable to a combination of
dilation and stenting with early strictures responding better than
late. (Level 3 evidence; Grade C recommendation).

MALIGNANT CBD STRICTURES


Approximately 30,000 new cases of pancreatic cancer and 7000
cases of biliary tract cancer are diagnosed each year in the United
States. Many of these patients are not candidates for surgery and
decompression of the biliary tree may be an important factor in
their palliation. This chapter explores issues related to stenting of
the bile duct in the face of malignant obstruction.
9. Should a patient with jaundice from pancreas cancer undergo
biliary drainage prior to surgical resection?
Patients with pancreatic cancer often present with obstructive
jaundice. Radiologic imaging is the first step in evaluation to
determine if they have a lesion in the head of the pancreas. If so,
and if they are surgical candidates, will preoperative ERCP and
biliary decompression improve their outcome? Seven studies in
the literature examine this question, four nonrandomized and
three randomized.58-64 Some of the evidence is of poor methodologic quality, but all data fail to show a benefit of preoperative
ERCP and stenting. The technical success of gaining decompression appears to be high at 87% to 94% with all studies showing a
significant decrease in hyperbilirubinemia.64 Hospital LOS either
overall or pre/postoperatively is not clearly impacted by preoperative biliary decompression. Perioperative mortality is not impacted
either. Van der Gaag et al.63 demonstrated that total complications
(pre/postoperative, endoscopic, surgical) were increased in the
preoperative stent group (47% vs. 37%). Sewnath et al.59 reported
no difference in postoperative complications, but a 6% stent complication rate. Heslin et al.61 reported only postoperative complications with a higher rate in the stent group (59% vs. 34%).
Answer: There is no benefit to preoperative stenting of the
obstructed biliary tree in pancreatic cancer patients who are surgical candidates. In fact, the data suggest that preoperative stenting may result in more perioperative complications and should
not be performed routinely, but only in those cases with acute
cholangitis or intense pruritis in whom surgery may be delayed.
(Level 2 evidence; Grade B recommendation).
10. What is the best method of biliary decompression in hilar
cholangiocarcinoma: ERCP or PTC?
Primary cholangiocarcinoma is an uncommon malignancy in
the United States with poor prognosis. Hilar cholangiocarcinomas (Klatskins tumor) account for up to half of these cases and
are often inoperable. As a result, treatment centers on decompression of the obstructed biliary tree to improve liver function and
relieve pruritis so as to improve quality of life. Biliary decompression may be complex in these patients because of the high
location of the lesion and potential involvement of intrahepatic
ducts. Endoscopic retrograde biliary drainage (ERBD), external percutaneous transhepatic biliary drainage (EPTBD), and
internal biliary stenting via the PTBD tract (IPTBD) are the

PMPH_CH52.indd 413

413

strategies commonly reported in the literature and all have been


used successfully. The optimal method of drainage, however, is
not universal, but rather depends on the Bismuth classification
of the tumor involvement.65 Lee et al.66 investigated the outcomes
of these three drainage techniques based on the Bismuth classification of tumor. Th is retrospective review did not include any
patients with Bismuth Type I tumors. Overall, 91% of patients
gained relief from their jaundice with no differences among the
procedures or Bismuth types (IIIV) for Bismuth Type II lesions.
There was no difference in patency rates among the three methods of drainage. ERBD and IPTBD were superior to EPTBD for
Bismuth Type III, and IPTBD was best for Bismuth Type IV, particularly when factoring quality of life and patency of drainage
in comparison to EPTBD. The authors recommend ERCP with
stent as the first line of therapy for Bismuth Types II and III and
IPTBD for Bismuth Type IV.
There is debate in the literature on whether bilateral or unilateral drainage is better for Klatskins tumor. The worst outcomes
are in patients who have cholangiographic opacification of both
lobes of the liver, but drainage of only one side leading to cholangitis.67 If care is taken to drain ducts that have been opacified
on cholangiography, the data do not show a clear superiority to
placement of more than one stent. As a result, it has been argued
that unilateral stent placement using the easiest duct that can be
accessed for drainage should be selected.68
Answer: The best method of drainage of the biliary tract in
Klatskins tumor is dependent on the Bismuth classification. Bismuth Type II lesions are equally amenable to endoscopic or percutaneous transhepatic drainage. Bismuth Type III lesions are best
managed with either endoscopic drainage or a hybrid approach
using both PTC and ERCP to place an endobiliary stent. Bismuth
Type IV lesions do best with the hybrid approach. (Level 3 evidence;
Grade C recommendation).
11. Are plastic or metal stents best for biliary decompression
in malignancy?
Two randomized controlled trials have investigated this question.
Davids et al.69 randomized 115 patients with distal CBD obstruction from malignancy to receive plastic stents or SEMS. Prat et al.70
randomized 101 patients into three arms: SEMS, plastic stent with
scheduled changes, and plastic stent with change as needed based
on LFTs and symptoms. Both studies report excellent results in
all arms in placing the stents and gaining initial relief of jaundice.
Both studies demonstrated longer patency rates with SEMS versus plastic with Davids et al.69 reporting SEMS remaining patent
twice as long (9.1 vs. 4.2 months). Periprocedure mortality related
to biliary complications was similar in both groups as was survival and stent exchange as needed versus on a scheduled basis
did not seem to increase morbidity or mortality. Use of SEMS also
required less ERCPs than plastic and less hospital days with Prat
et al.70 reporting lower cost.
Answer: Use of SEMS for CBD obstruction from malignancy
results in longer patency rates, less procedures, hospital days,
and cost, and similar periprocedure morbidity and long-term
mortality when compared with routine plastic stent exchange.
Change of plastic stents on an as needed basis seems to be equivalent to scheduled exchange with the potential for fewer procedures and hospital days and lower cost. (Level 1 evidence; Grade A
recommendation).

5/22/2012 5:36:36 PM

414

Surgery: Evidence-Based Practice

12. Is per oral choledochoscopy helpful in defining CBD


strictures of unclear etiology?
Per oral choledochoscopy is a method of directly visualizing the
lumen of the CBD during ERCP. A choledochoscope is introduced
through the working channel of the ERCP duodenoscope and
passed under direct and fluoroscopic vision across the papilla of
Vater and into the CBD. In this configuration the duodenoscope
is called the mother scope and the choledochoscope the daughter scope. Traditionally this has been a demanding procedure
requiring two expert endoscopists and use of a daughter scope
that is expensive and fragile. Recent technologic developments
have enabled the procedure to be performed by one endoscopist
with a more functional choledochoscope which even enables
pinch biopsy.
The ability to directly examine the lumen of the CBD has led some
to investigate whether this approach could help better define strictures of unknown etiology. Choledochoscopy with biopsy, brushing, and even augmented visualization using chromoendoscopy

and narrow-band imaging have all been utilized to evaluate CBD


strictures.71 Unfortunately, all reports in the literature about choledochoscopy are case reports or small case experience series with
no comparative data.
Answer: Although per oral choledochoscopy is feasible, can be
performed safely, and holds promise as a better method of evaluating CBD strictures of unclear etiology, there is no high-level comparative data that proves it is superior to other any other modality.
(No Level 1, 2, or 3 evidence; no recommendation).

SUMMARY
This chapter examined the literature around 12 important questions
related to managing CBD obstruction and jaundice from either
stone disease or stricture. It should serve as a guide to managing
patients with these disease processes and as a basis for identifying
gaps in the literature where further study is needed.

Clinical Question Summary


Question

Answer

Level of
Evidence

Grade of
Recommendation

1 What is the best way to


predict CBD stones?

References

Look for an absence of abnormality


(i.e. normal LFTs and no CBD dilation
on ultrasound). Negative predictive
value comparable to diagnostic ERCP.
Multivariable risk-factor formulas also
have good accuracy. Univariate factor
analysis has the least accuracy.

4-16

2 How does diagnostic


ERCP compare with
noninvasive imaging
modalities?

Because of the excellent performance of


MRCP and EUS in detecting CBD stones,
ERCP should not be used as a diagnostic
tool alone particularly given its associated
higher rate of potential complications.
CTC is not as accurate as MRCP and EUS.

17-31

3 What is the optimal


treatment of suspected
CBD stones: pre/
postoperative ERCP
or IOC and CBD
exploration?

IOC followed by LCBDE is more cost


effective with shorter lengths of stay than
pre/postoperative ERCP. Given that both
ERCP and LCBDE are operator-dependent
procedures, the strategy employed at any
single institution must take into account
the available expertise.

32-33

4 What is the expected


success rate of ERCP
to clear CBD stones
and can it serve as
definitive therapy?

ERCP for removal of CBD stones should


be at least 90% successful. Open or
laparoscopic cholecystectomy is preferred
in high-risk patients because of acceptable
short-term morbidity and equivalent
mortality to that of ERCP with less
recurrence of biliary symptoms or need
for hospital readmission.

2, 3

B, C

33-39

5 Can ERCP with ES and/


or stent placement
be used to palliate
pregnant women
with CBDS until after
delivery?

Data limited to case reports. ERCP can


be done safely in pregnant women with
minimal risk to the fetus.

41-43

(Continued)

PMPH_CH52.indd 414

5/22/2012 5:36:36 PM

Obstructive Jaundice: Transhepatic and Endoscopic Interventions

415

(Continued)
Question

Answer

Level of
Evidence

Grade of
Recommendation

6 What is the best method


of gaining access to the
CBD following Rouxen-Y gastric bypass?

There is no consensus in the literature about


the best approach to the papilla of Vater
after Roux-en-Y gastric bypass. The choice
of approach depends on available expertise
and equipment.

44

7 Can biliary strictures


from chronic
pancreatitis be relieved
with dilation and
stenting?

Benign biliary strictures from chronic


pancreatitis can be successfully managed
with endobiliary stenting but require
placement of multiple plastic stents left in
place for up to 14 months. Calcifications
at the head of the pancreas predict worse
outcomes.

45-50

8 How does management


of biliary strictures
from PSC and
anastomotic stenosis
compare with chronic
pancreatitis?

Biliary strictures from PSC are more


amenable to dilation alone when
compared to strictures from chronic
pancreatitis, and placement of a stent in
these patients may increase the risk of
cholangitis. Postoperative anastomotic
biliary strictures are also amenable to a
combination of dilation and stenting
with early strictures responding better
than late.

51-57

9 Should a patient
with jaundice from
pancreas cancer
undergo biliary
drainage prior to
surgical resection?

There is no benefit to preoperative stenting


of the obstructed biliary tree in pancreatic
cancer patients who are surgical
candidates.

58-65

10 What is the best


method of biliary
decompression in hilar
cholangiocarcinoma:
ERCP or PTC?

Bismuth type II lesions are equally


amenable to endoscopic or percutaneous
transhepatic drainage. Bismuth type III
lesions are best managed with either
endoscopic drainage or a hybrid
approach using both PTC and ERCP to
place an endobiliary stent. Bismuth IV
lesions do best with the hybrid
approach.

65-68

11 Are plastic or metal


stents best for biliary
decompression in
malignancy?

Metal stents result in longer patency


rates, less procedures, hospital days,
and cost, and similar periprocedure
morbidity and long-term mortality
when compared to routine plastic
stent exchange.

69, 70

12 Is per oral
choledochoscopy
helpful in defining CBD
strictures of unclear
etiology?

Per oral choledochoscopy is feasible,


can be performed safely, and holds
promise as a better method of
evaluating CBD strictures of unclear
etiology. There is no high-level
comparative data that proves it is
superior to other any other modality.

N/A

N/A

PMPH_CH52.indd 415

References

71

5/22/2012 5:36:36 PM

416

Surgery: Evidence-Based Practice

REFERENCES
1. Aronson N, Flamm CR. Endoscopic retrograde cholangiopancreatography. Evidence report/technology assessment. 2002;50
(02-E017):1-360.
2. Huard-Du XH. La ponction transhepatique des canaux biliares.
Bull Soc Med Clin. 1937;15:1090-1100.
3. McCune WS, Shorb PE, Moskovitz H. Endoscopic cannulation
of the ampulla of Vater: A preliminary report. Ann Surg. 1968;
167:752-756.
4. Alponat A, Kum CK, Rajnakova A, et al. Predictive factors for synchronous common bile duct stones in patients with cholelithiasis.
Surg Endosc. 1997;11(9):928-932.
5. Barkun AN, Barkun JS, Fried GM, et al. Useful predictors of bile
duct stones in patients undergoing laparoscopic cholecystectomy.
McGill gallstone treatment group. Ann Surg. 1994;220(1):32-39.
6. Bergamaschi R, Tuech JJ, Braconier L, et al. Selective endoscopic
retrograde cholangiography prior to laparoscopic cholecystectomy for gallstones. Am J Surg. 1999;178(1):46-49.
7. Hauer-Jensen M, Karesen R, Nygaard K, et al. Predictive ability
of choledocholithiasis indicators. A prospective evaluation. Ann
Surg. 1985;202(1):64-68.
8. Kim KH, Kim W, Lee HI, et al. Prediction of common bile duct stones:
Its validation in laparoscopic cholecystectomy. Hepatogastroenterology. 1997;44(18):1574-1579.
9. Koo KP, Traverso LW. Do preoperative indicators predict the
presence of common bile duct stones during laparoscopic cholecystectomy? Am J Surg. 1996;171(5):495-499.
10. Menezes N, Marson LP, Debeaux AC, et al. Prospective analysis of a scoring system to predict cholelithiasis. Br J Surg. 2000;
87(9):1176-1181.
11. Santucci L, Natalini G, Sarpi L, et al. Selective endoscopic retrograde cholangiography and preoperative bile duct stone removal
in patients scheduled for laparoscopic cholecystectomy: A prospective study. Am J Gastroenterol. 1996;91(7):1326-1330.
12. Trondsen E, Edwin B, Reiertsen O, et al. Prediction of common bile
duct stones prior to cholecystectomy: A prospective validation of a
discriminant analysis function. Arch Surg. 1998;133(2):162-166.
13. Trondsen E, Edwin B, Reiertsen O, et al. Selection criteria for
endoscopic retrograde cholangiopancreatography (ERCP) in
patients with gallstone disease. World J Surg. 1995;19(6):852-856.
14. Hawasli A, Lloyd L, Pozios V, et al. The role of endoscopic retrograde cholangiopancreaticogram in laparoscopic cholecystectomy. Am Surg. 1993;59(5):285-288.
15. Carroll BJ, Phillips EH, Rosenthal R, et al. One hundred consecutive laparoscopic cholangiograms. Results and conclusions.
Surg Endosc. 1996;10(3):319-323.
16. Khaira HS, Ridings PC, Gompertz RH, et al. Routine laparoscopic cholangiography: A means of avoiding unnecessary endoscopic retrograde cholangiopancreatography. J Laparoendosc
Adv Surg Tech. 1999;9(1):17-22.
17. Sugiyama M, Atomi Y, Hachiya J. Magnetic resonance cholangiography using half-Fourier acquisition for diagnosis choledocholithiasis. Am J Gastroenterol. 1998;93(10):1886-1890.
18. Varghese JC, Liddell RP, Farrell MA, et al. Diagnostic accuracy
of magnetic resonance cholangiopancreatography and ultrasound compared with direct cholangiography in the detection of
choledocholithiasis. Clin Radiol. 2000;55(1):25-35.
19. Varghese JC, Farrell MA, Courtney G, et al. A prospective comparison of magnetic resonance cholangiopancreatography with
endoscopic retrograde cholangiopancreatography in the evaluation of patients with suspected biliary tract disease. Clin Radiol.
1999;54(8):513-520.

PMPH_CH52.indd 416

20. Prat F, Amouyal G, Amouyal P, et al. Prospective controlled study


of endoscopic ultrasonography and endoscopic retrograde cholangiography in patients with suspected common bile duct lithiasis.
Lancet. 1996;347(8994):75-79.
21. Burtin P, Palazzo L, Canard JM, et al. Diagnostic strategies for
extrahepatic cholestasis of indefinite origin: Endoscopic ultrasonography or retrograde cholangiography? Results of a prospective study. Endoscopy. 1997;29(5):349-355.
22. Canto MI, Chak A, Stellato T, et al. Endoscopic ultrasonography
versus cholangiography for the diagnosis of choledocholithiasis.
Gastrointest Endosc. 1998;47(6):439-448.
23. Norton SA, Alderson D. Prospective comparison of endoscopic
ultrasonography and endoscopic retrograde cholangiopancreatography in the detection of bile duct stones. Br J Surg. 1997;84(10):
1366-1369.
24. Dancygier H, Nattermann C. The role of endoscopic ultrasonography in biliary tract disease: Obstructive jaundice. Endoscopy.
1994;26(9):800-802.
25. Polkowski M, Palucki J, Regula J, et al. Helical computed tomographic cholangiography versus endosonography for suspected
bile duct stones: A prospective blinded study in non-jaundiced
patients. Gut. 1999;45(5):744-749.
26. Sugiyama M, Atomi Y. Endoscopic ultrasonography for diagnosing
choledocholithiasis: A prospective comparative study with ultrasonography and computed tomography. Gastrointest Endosc. 1997;
45(2):143-146.
27. Chak A, Hawes RH, Cooper GS, et al. Prospective assessment
of the utility of EUS in the evaluation of gallstone pancreatitis.
Gastrointest Endosc. 1999;49(5):599-604.
28. Soto JA, Alvarez O, Munera F, et al. Diagnosing bile duct stones:
Comparison of unenhanced helical CT, oral contrast-enhanced
CT cholangiography, and MR cholangiography. AJR Am J Roentgenol. 2000;175(4):1127-1134.
29. Soto JA, Barish MA, Alvarez O, et al. Detection of choledocholithiasis with MR cholangiography: Comparison of threedimensional fast spin-echo and single- and multisection
half-Fourier rapid acquisition with relaxation enhancement
sequences. Radiology. 2000;215(3):737-745.
30. Soto JA, Velez SM, Guzman J. Choledocholithiasis: Diagnosis
with oral-contrast-enhanced CT cholangiography. AJR Am J
Roentgenol. 1999;172(4):943-948.
31. Ishikawa M, Tagami Y, Toyota T, et al. Can three-dimensional helical CT cholangiography before laparoscopic cholecystectomy be a
substitute study for endoscopic retrograde cholangiography? Surg
Laparosc Endosc Percutan Tech. 2000;10(6):351-356.
32. Chang KJ, Katz KD, Durbin TE, et al. Endoscopic ultrasoundguided fine-needle aspiration. Gastrointest Endosc. 1994;40(6):
694-699.
33. Cuschieri A, Lezoche E, Morino M, et al. E.A.E.S. multicenter prospective randomized trial comparing two-stage vs single-stage management of patients with gallstone disease and ductal calculi. Surg
Endosc. 1999;13(10):952-957.
34. Carr-Locke DL. Therapeutic role of ERCP in the management
of suspected common bile duct stones. Gastrointest Endosc.
2002;56(Suppl 6):S170-174.
35. Chang L, Lo S, Stabile BE, et al. Preoperative versus postoperative endoscopic retrograde cholangiopancreatography in mild to
moderate gallstone pancreatitis: A prospective randomized trial.
Ann Surg. 2000;231(1):82-87.
36. Rhodes M, Sussman L, Cohen L, et al. Randomized trial of laparoscopic exploration of common bile duct versus postoperative
endoscopic retrograde cholangiography for common bile duct
stones. Lancet. 1998;351(9097):159-161.

5/22/2012 5:36:36 PM

Obstructive Jaundice: Transhepatic and Endoscopic Interventions

37. NIH state-of-the-science statement on endoscopic retrograde


cholangiopancreatography (ERCP) for diagnosis and therapy.
NIH Consensus State Sci Statements 2002;19:1-26.
38. Targarona EM, Ayuso RM, Bordas JM, et al. Randomised trial
of endoscopic sphincterotomy with gallbladder left in situ versus
open surgery for common bile duct calculi in high-risk patients.
Lancet. 1996;347(9006):926-929.
39. Trias M, Targarona EM, Ros E, et al. Prospective evaluation of a
minimally invasive approach for treatment of bile-duct calculi in
the high-risk patient. Surg Endosc. 1997;11(6):632-635.
40. Yi SY. Recurrence of biliary symptoms after endoscopic sphincterotomy for choledocholithiasis in patients with gallbladder
stones. J Gastroenterol Hepatol. 2000;15:661-664.
41. Chopra KB, Peters RA, OToole PA, et al. Randomized study of
endoscopic biliary endoprosthesis versus duct clearance for bile
duct stones in high-risk patients. Lancet. 1996;348(9030):791-793.
42. Karam PA. Determining and reporting fetal radiation exposure
from diagnostic radiation. Health Phys. 2000;79:S85-90.
43. Chong VH, Jalihal A. Endoscopic management of biliary disorders during pregnancy. Hepatobiliary Pancreat Dis In. 2010;9(2):
180-185.
44. Martinez J, Guerrero L, Byers P, et al. Endoscopic retrograde cholangiopancreatography and gastroduodenoscopy after Roux-en-Y
gastric bypass. Surg Endosc. 2006;20:1548-1550.
45. Barther M, Bernard JP, Duval JL, Aff riat C, Sahel J. Biliary stenting in benign biliary stenosis complicating chronic calcifying
pancreatitits. Endoscopy. 1994;26:569-572.
46. Kahl S, Zimmermann S, Genz I, et al. Risk factors for failure
of endoscopic stenting of biliary strictures in chronic pancreatitis: A prospective follow-up study. Am J Gastroenterol. 2003;98:
2448-2453.
47. Catalano MF, Linder JD, George S, Alcocer E, Geenen JE. Treatment of symptomatic distal common bile duct stenosis secondary to chronic pancreatitis: Comparison of single versus multiple
simultaneous stents. Gastrointest Endosc. 2004;60:945-952.
48. Van Boeckel, Vleggaar FP, Siersema PD. Plastic or metal stents
for benign extrahepatic biliary strictures: A systematic review.
BMC Gastroenterol. 2009;9:96-111.
49. Behm B, Brock A, Clarke BW, et al. Partially covered selfexpandable metallic stents for benign biliary strictures due to
chronic pancreatitis. Endoscopy. 2009;41(6):547-551.
50. Mahajan A, Ho H, Sauer B, et al. Temporary placement of fully
covered self-expandable metal stents in benign biliary strictures:
Midterm evaluation. Gastrointest Endosc. 2009;70(2):303-309.
51. Kaya M, Petersen BT, Angulo P, et al. Balloon dilation compared
to stenting of dominant strictures in primary sclerosing cholangitis. Am J Gastroenterol. 2001;96:1059-1066.
52. Baluyut AR, Sherman S, Lehman GA, Hoen H, Chalasani N.
Impact of endoscopic therapy on the survival of patients with primary sclerosing cholangitis. Gastrointest Endosc. 2001;53:308-312.
53. Adler DG, Baron TH, Davila RE, et al. ASGE guideline: The
role of ERCP in diseases of the biliary tract and the pancreas.
Gastrointest Endosc. 2005;62(1):1-8.
54. Draganov P, Hoff man B, Marsh W, Cotton P, Cunningham J. Longterm outcome in patients with benign biliary strictures treated
endoscopically with multiple stents. Gastrointest Endosc. 2002;55:
680-686.

PMPH_CH52.indd 417

417

55. Park JS, Kim MH, Lee SK, et al. Efficacy of endoscopic and percutaneous treatments for biliary complications after cadaveric and
living donor liver transplantation. Gastrointest Endosc. 2003;
57:78-85.
56. Rerknimitr R, Sherman S, Fogel EL, et al. Biliary tract complications after orthotopic liver transplantation with choledochocholedochostomy anastomosis: Endoscopic findings and results of
therapy. Gastrointest Endosc. 2002;55:224-231.
57. Thethy S, Thomson BN, Pleass H, et al. Management of biliary
tract complications after orthotopic liver transplantation. Clin
Transplant. 2004;18:647-653.
58. Lygidakis NJ, van der Heyde MN, Lubbers MJ. Evaluation of preoperative biliary drainage in the surgical management of pancreatic head carcinoma. Acta Chir Scand. 1987;153(11-12):665-668.
59. Sewnath ME, Birjmohun RS, Rauws EA, et al. The effect of preoperative biliary drainage on postoperative complications after
pancreaticoduodenectomy. J Am Coll Surg. 2001;192(6):726-734.
60. Karsten TM, Allema JH, Reinders M, et al. Preoperative biliary
drainage, colonization of bile and postoperative complications
in patients with tumors of the pancreatic head: A retrospective
analysis of 241 consecutive patients. Eur J Surg. 1996;162(11):
881-888.
61. Heslin MJ, Brooks AD, Hochwald SN, et al. A preoperative biliary stent is associated with increased complications after pancreatoduodenectomy. Arch Surg. 1998;133(2):149-154.
62. ten Hoopen-Neumann H, Gerhards MF, van Gulik TM, et al.
Occurrence of implantation metastases after resection of
Klatskin tumors. Dig Surg. 1999;16(3):209-213.
63. Van der Gaag NA, Rauws EA, Van Eijck CH, et al. Preoperative
biliary drainage for cancer of the head of the pancreas. N Engl J
Med. 2010;362(2):129-137.
64. Lai EC, Mok FP, Fan ST, et al. Preoperative endoscopic drainage for
malignant obstructive jaundice. Br J Surg. 1994;81:1195-1198.
65. Bismuth H, Castaing D, Traynor O. Resection or palliation: Priority of surgery in the treatment of hilar cancer. World J Surg.
1988;12:39-47.
66. Lee SH, Park JK, Yon WJ, et al. Optimal biliary drainage for
inoperable Klatskins tumor based on Bismuth type. World J
Gastroeterol. 2007;13(29):3948-3955.
67. Chang WH. Kortan P, Haber GB. Outcome in patients with
bifurcation tumors who undergo unilateral versus bilateral
hepatic duct drainage. Gastrointest Endosc. 1998;47:354-362.
68. DePalma GD, Galloro G, Siciliano S, Iovino P, Catanzano C. Unilateral vs. bilateral endoscopic hepatic duct drainage in patients
with malignant hilar biliary obstruction: Results of a prospective,
randomized, and controlled study. Gastrointest Endosc. 2001;53:
547-553.
69. Davids PH, Groen AK, Rauws EA, et al. Randomized trial of
self-expanding metal stents versus polyethylene stents for distal malignant biliary obstruction. Lancet. 1992;340(8834-8835):
1488-1492.
70. Prat F, Chapat O, Ducot B, et al. A randomized trial of endoscopic drainage methods for inoperable malignant strictures of
the common bile duct. Gastrointest Endosc. 1998;47(1):1-7.
71. Kozarek R, Kodama T, Tatsumi Y. Direct cholangioscopy and
pancreatoscopy. Gastrointest Endosc Clin N Am. 2003;13(4):
593-608.

5/22/2012 5:36:36 PM

PMPH_CH52.indd 418

5/22/2012 5:36:36 PM

PART VIII

THE PANCREAS

PMPH_CH53.indd 419

5/22/2012 5:37:13 PM

PMPH_CH53.indd 420

5/22/2012 5:37:13 PM

CHAPTER 53

Acute Pancreatitis
Stephen W. Behrman

In an early study, Neoptolemos et al.5 prospectively randomized 121 patients with presumed biliary pancreatitis to early (< 72 h
from admission) ERCP with sphincterotomy and stone extraction
if necessary versus conservative management alone with selective ERCP if it was indicated. The severity of pancreatitis was
assessed via the modified Glascow criteria with severe disease
defined as a score of 3 or higher. Interpretation of the data in this
study is somewhat clouded by the fact that gallstones could not be
confirmed in 18 patients despite the availability of ultrasound and
computed tomography. With this limitation in mind, early ERCP
was successful in 52 of 59 (88%) patients and choledocholithiasis
was confirmed in 19 (32%) (vs. 3 of 14 (21%) in the conservative
group) and successful stone extraction was accomplished in all.
Early ERCP was associated with a statistically significant decrease
in disease-related complications (pseudocyst, organ failure) and a
reduction (not significant) in mortality in those with severe, but
not mild, pancreatitis.
Fan et al.6 studied the role of ERCP in AP of all causes (predominantly biliary) in a prospective randomized trial of 197
patients. The purpose of this study was to compare the efficacy
of early (<24 h) ERCP with papillotomy if stones were identified
versus initial conservative treatment with ERCP papillotomy
reserved for those with clinical deterioration defi ned as fever,
tachycardia, worsening leukocytosis and/or an increase in bilirubin. Outcome was assessed on the basis of local and systemic
complications as well as death. Severe pancreatitis was defined
as a Ranson score of 4 or higher. Impacted stones were found in
37 of 97 (38%) patients having early ERCP. In contrast, 27 of 98
patients initially followed conservatively required ERCP for deterioration, with stones found in the common bile duct or ampulla
in only 12 (12%) confirming that the vast majority of stones pass
spontaneously. Complications were higher in those with initial
conservative treatment (29% vs. 18%), but this difference was not
significant. With the exception of those developing cholangitis in
the conservative group (8 vs. 0 patients), other complications did
not differ dramatically. Mortality was higher in those treated conservatively (9 vs. 5 patients), but did not reach statistical significance. All deaths occurred in those with severe pancreatitisthe

Acute pancreatitis (AP) is responsible for approximately 250,000


hospital admissions per year in the United States alone and its
incidence is rising.1 The spectrum of disease can range from a
self-limiting process to one that may be life-threatening and
require therapeutic intervention. Recurrent disease can occur
on the basis of two less common disorders: Sphincter of Oddi
dysfunction (SOD) and pancreas divisum (PD). 2 These entities
are often difficult to assess, diagnose and treat. Further, results
of endoscopic and surgical intervention for these diseases are
often less than enviable and thus it is imperative that patient
selection for therapeutic intervention be defi ned as precisely as
possible.
Severe pancreatitis defi ned as organ failure persisting
beyond 72 h and/or complications, such as pseudocyst formation
or necrosis occurs in 10% to 20%, may require surgical intervention and is associated with substantial morbidity and mortality. 3,4
Early therapeutic intervention in this cohort is imperative to
stabilize the patient physiologically, mitigate infectious complications and maintain metabolic needs. Although management
relative to the early resuscitative phase of severe AP has been well
delineated, other treatment algorithms have more recently been
challenged.
1. What is the role of early endoscopic retrograde cholangiopancreatography (ERCP) in acute biliary pancreatitis?
The need for, and timing of ERCP in biliary pancreatitis has been
a subject of controversy. The vast majority of stones will pass
spontaneously into the duodenum and thus will neither worsen
the degree of pancreatitis nor present a risk for the development
of concurrent cholangitis. Indeed, early ERCP may exacerbate
pancreatitis. However, cholangitis developing in the face of severe
AP would most certainly contribute to morbidity and mortality
favoring early endoscopic evaluation. Perhaps more controversial
is the role of early ERCP in ameliorating the degree of pancreatitis
and the ensuing inflammatory cascade. Several randomized controlled studies and meta-analyses have addressed these issues that
are key in the management of these often critically ill patients.
421

PMPH_CH53.indd 421

5/22/2012 5:37:13 PM

422

Surgery: Evidence-Based Practice

vast majority of who had no stone found on endoscopic evaluation. Early ERCP did not seem to either worsen or improve
the progression of pancreatitis. The authors, surprisingly, conclude that emergency ERCP is indicated in all patients with AP
although their data seems to suggest otherwise.
Folsch et al.7 conducted a prospective, randomized, multicenter study comparing early ERCP (<72 h) versus conservative
management in those with acute biliary pancreatitis without evidence of obstructive jaundice. Disease severity was measured by
the modified Glasgow criteria (>3 severe). Indications for ERCP in
the conservatively managed group were similar to those described
by Fan et al. Fift y-eight of 126 patients undergoing early ERCP had
documented bile duct stones versus 13 of 112 in the conservative
group. Of note, 22 of 112 patients in the conservatively managed
group developed indications for ERCP and the incidence of choledocholithiasis in this cohort was 60 percent. Overall, morbidity
and mortality did not differ between groups including the risk of
developing pancreatic-related complications such as pseudocyst
and necrosis. The authors conclude that early ERCP is not indicated in those with acute biliary pancreatitis in the absence of
clinical evidence of biliary obstruction or sepsis.
In a more recent study, Oria et al.8 examined the role of early
(<4872 h) ERCP in those presenting with acute gallstone pancreatitis and evidence of biliopancreatic obstruction defined as a
common bile duct >8 mm or serum bilirubin >1.2 mg/dL. Importantly, patients with clinical evidence of cholangitis (Charcots
triad) were excluded as this condition was felt to mandate early
ERCP in this randomized, prospective study. Severe pancreatitis
was defined as an APACHE II score >6. The specific aims of this
study were to determine if early ERCP could reduce the severity
of pancreatitis and thereby limit organ failure and complications
of pancreatitis. The safety of early endoscopy was also assessed.
Fift y-one of 103 patients were randomized to early ERCP with
choledocholithiasis noted on 47 (72%) successful cannulations
with minimal complications. When comparing the two groups,
early clearance of the common duct did not reduce organ failure,
local complications of the pancreas or mortality in either mild or
severe pancreatitis. The authors concluded that early ERCP did
not alter the course of acute gallstone pancreatitis and was not
indicated in the absence of cholangitis. Two recent meta-analyses
have yielded the same conclusions while recognizing the heterogeneity of patient populations, enrollment criteria, the arbitrary
assignment of mild and severe pancreatitis and the definition of
early ERCP.9,10
A slightly contrasting conclusion was reached in a group of
patients with severe AP (Apache II score > 8) reported by van
Santvoort et al.11 in a prospective multicenter observational study
where the decision to perform early ERCP (within 72 h) was at the
discretion of the treating physician. Patients with frank cholangitis were excluded. Those with cholestasis (defined as bilirubin
>2.3 mg/dL and/or dilated common bile duct with a temperature
<38.6C) were compared with those without. Although overall
complications were significantly reduced in the early ERCP group
with cholestasis, organ failure, infected pancreatic necrosis, the
need for operation, and hospital length of stay were no different.
This study was a subset analysis of a previous report from these
institutions on the utilization of probiotics in AP that demonstrated a negative outcome. This authors conclusion that complications are reduced with early ERCP in severe AP associated with
cholestasis is somewhat misleading.

PMPH_CH53.indd 422

In conclusion, ERCP has proven to be safe when performed


in the face of acute biliary pancreatitis. If performed early, the
incidence of choledocholithiasis is substantially higher than if
ERCP is performed selectively when there is evidence of persistent biliary obstruction based on routine radiologic and chemical analysis. However, in studies to date, early clearance of the
common bile duct has not correlated with a reduction in organ
failure, pancreatic-related complications or mortality. For these
reasons, in the absence of cholangitis or evidence of persistent biliary obstruction, early ERCP in gallstone pancreatitis is not recommended. (Grade A recommendation)
2. What is the role of prophylactic antibiotics in severe AP?
Severe pancreatitis, by any grading system, is associated with a
substantial risk for the development of pancreatic fluid collections
and/or pancreatic necrosis as defi ned by the Atlanta Classification.3 If these processes remain sterile, there is a good probability
that patients will recover without the need for operative intervention. In contrast, secondary pancreatic infections mandate
the need for operative drainage and debridement, markedly
increase hospital length of stay and are associated with significant morbidity and mortality.12 In theory, prophylactic antibiotics in those with severe AP might prevent the progression of a
sterile process into an infected milieu. Questions remain if this
mode of therapy is chosen. When should antimicrobial therapy
be initiated and for how long? In addition, there may be a price
to pay for such a strategy including antibiotic associated colitis
and the potential selection of resistant or fungal organisms given
prolonged therapy that may increase, rather than protect against,
the risk for mortality.13 The fluoroquinolones and carbapenems
have formed the basis of clinical studies investigating the role
of antimicrobial prophylaxis in severe pancreatitis due to their
superior concentrations in the pancreatic bed likely on the basis
of their liposolubility.14,15
In an early, small multicenter, nonblinded trial, Penderzoli
et al.16 randomized 74 patients with pancreatic necrosis noted on
CT scan within 72 h of admission to medical management alone
versus the addition of prophylactic imipenem-cilastin for 14 days
(41 patients). Mean Ranson criterion for all patients was 3.7 and
about one-half had pancreatitis on the basis of biliary disease.
More patients receiving prophylaxis had >50% necrosis (14 vs. 2).
Only five patients receiving antimicrobial prophylaxis developed
pancreatic sepsis (confirmed by culture) statistically different than
those medically managed. However, mortality and the need for
surgical debridement of the pancreas did not differ. Curiously, in
addition to the five septic patients in the prophylaxis group, seven
additional patients had laparotomy for reasons not stated. Culture
data on those with pancreatic sepsis suggests that prophylaxis did
not select out resistant organisms.
Isenmann et al.17 performed a multicenter, randomized,
placebo-controlled, double-blind study on the effect of ciprofloxacin and flagyl, administered for a minimum of 14 days, in preventing infected pancreatic necrosis and thereby reducing mortality.
One hundred and fourteen patients with severe AP defi ned as a
C-reactive protein level (CRP) >150 mg/L and or the presence of
pancreatic necrosis on contrast-enhance CT and entering within
72 h of admission were analyzed. Study patients were converted
to open antibiotic therapy if extra or de novo pancreatic sepsis
was documented, multiple organ failure developed or CRP levels

5/22/2012 5:37:13 PM

Acute Pancreatitis

increased. The etiology of pancreatitis was predominantly biliary


and alcohol related. Of the 58 patients randomized to antibiotic prophylaxis, only 16 required conversion to open antimicrobial administration versus 26 in the placebo groupa
significant difference. However, the incidence of secondary and
extra-pancreatic infections was not different nor was the mortality rate. Approximately one-half of the isolates in both groups
with infected necrosis were gram positive organisms. However,
it was not noted how these isolates were obtainedopen versus percutaneous. Thus, because empiric antibiotic treatment
did not lead to development of resistant or fungal organisms, it
failed to prevent pancreatic and systemic infections and it did
not reduce mortality in this study. It should be noted, however,
that the initial power analysis called for a study population of 200
patients assuming an incidence of pancreatic infection of 40%.
Surprisingly, this study was terminated after an interim analysis
because, the authors state, infected pancreatic necrosis occurred
in 7/53 treated patients versus 5/52 receiving placebo and this was
a reverse trend. Certainly it could be argued that study recruitment should have continued.
Dellinger et al.18 reported a multicenter similarly designed
study and patient population to that of Isenmann comparing
prophylactic meropenem infusion to placebo in 40 patients each
within 5 days of onset of severe AP and delivered for 7 to 21 days.
In contrast to the study by Isenmann, most patients in this study
had >30% pancreatic necrosis documented by CT consistent with
severe disease. The incidence of pancreatic infection, the number
of operative interventions on the pancreas and the mortality rate
was not different between groups. The utilization of prophylaxis
did not increase the incidence of resistant organisms with gram
positive and negative flora predominating. The authors concluded
the antibiotic prophylaxis did not reduce septic pancreatic infections in those with severe AP. This study again did not reach its
desired power analysis assuming an incidence of pancreatic infection of 40% and it was not continued to reach the desired number
of patients due to a restriction of resources.
Two small, randomized controlled studies yielded conclusions similar to Dellinger et al. Garcia-Barrasa et al.19 randomized 46 antibiotic naive patients with severe AP (defined by the
Atlanta criteria3) and CT documented pancreatic necrosis to ciprofloxacin (within 10 days) for a 10-day treatment versus placebo
in a double-blind, placebo-controlled trial. Of the 41 patients
completing study protocol-infected necrosis, organ failure, the
need for operation and mortality was not different between
groups. Fourteen patients crossed over to open antibiotic therapy
due to established infected necrosis or progressive organ dysfunction diminishing meaningful results in this study. Xue et al.20
randomized 56 well-matched patients with severe AP defined as
>30% necrosis by CT criteria reaching study entry within 72 h of
presentation to imipenem-cilastin for 7 to 14 days or placebo. The
incidence of pancreatic infection, the need for operative debridement, organ failure, and mortality were not different between
groups. Recent meta-analyses have yielded similar findings that
antibiotic prophylaxis does not reduce the morbidity and mortality of severe AP.21-23
To summarize, the utilization of prophylactic antibiotics
in severe necrotizing pancreatitis is well tolerated and may alter
the flora recovered if infection ensues, but is not associated with
the development of resistant organisms. Randomized, doubleblinded, placebo-controlled studies to date have failed to recruit

PMPH_CH53.indd 423

423

enough patients to establish a statistically significant difference, if


it exists, between prophylaxis and placebo. It is unlikely that given
the low incidence of severe pancreatitis as well as the heterogeneity of patients and the treatment they receive that future studies might improve on those reported to date. Given the inherent
morbidity and mortality associated with infected necrosis, antimicrobial prophylaxis could be considered in those with severe
pancreatitis and significant necrosis if there is evidence of organ
failure or hemodynamic instability, but studies to date would suggest otherwise. (Grade B recommendation)
3. Is gastric feeding safe and equivalent to jejunal feeding
in AP?
Nutritional support in severe AP is vital due to the local and systemic inflammatory response that increases metabolic demands
resulting in hypercatabolism.24 In an attempt to rest the pancreas and not worsen its severity, hyperalimentation has traditionally provided the backbone of therapy. It has long been recognized
that enteral nutrition is superior to the parenteral route in terms
of immune competence, metabolic homeostasis and reducing the
overall cost of therapy.25 Most recently, the paradigm that enteral
feeding in severe pancreatitis exacerbates the disease or will not
be tolerated has been challenged. The utilization of this mode of
nutritional support, however, must not present its own set of complications and it must demonstrate superior outcomes to standard
therapy with TPN. Nasojejunal tube placement typically requires
either radiologic or endoscopic advancement, and bedside placement can be utilized but is cumbersome and time consuming.26
When compared with TPN, careful utilization of jejunal feedings
is well tolerated, reduces the inflammatory response of AP, reduces
infectious morbidity and is less expensive. Clinical improvement
with respect to the need for operative intervention, a shorter hospital length of stay and disease-related mortality remains sparse
but promising due to the small number of patients reported in
comparative studies to date.27-30 With the aforementioned benefits
of jejunal feedings a reasonable extrapolation would be to simplify the limitations of tube placement by feeding directly into
the stomach. Several clinical trials have compared these routes of
administration.
Eatock et al.31 randomized 49 well-matched patients with
severe AP defi ned as an Imrie score >3, and APACHE II score >6
or a CRP >150 mg/dl to nasogastric (NG) versus endoscopically
placed nasojejunal (NJ) feedings beginning within 72 h of onset
of symptoms. All but one patient tolerated the enteral route and
the majority of patients in both groups were receiving at least
75% of goal calories within 48 h of initiation of feedings. Groups
did not differ with respect to follow-up APACHE II scores, CRP
levels or pain analog scales and mortality was not statistically
different (24.5% of study population). Gastrointestinal complications were equivalent between groups. One patient required
repeat endoscopy to replace an NJ tube. The authors conclude
that NG feeding is simpler, less expensive and equivalent to the
NJ route.
Kumar et al.32 randomized 31 evenly matched patients with
severe AP defined as organ failure and an APACHE II score >8 or
Balthazar score >7 to NG (n = 15) or NJ (placed endoscopically)
feedings. Importantly, patients in shock (systolic blood pressure
< 90 mmHg) were appropriately excluded and feedings were
gradually increased over a 7-day period. Patients were assessed for

5/22/2012 5:37:13 PM

424

Surgery: Evidence-Based Practice

study accrual up to 4 weeks after onset of symptomsa delay in


initiation that might allow better tolerance of feedings. No patient
required TPN once the goal rate of feeding was achieved (day 7).
When compared with the NJ route, NG feedings were associated
with similar rates of pancreatic infection and operative intervention and the LOS and mortality was not different between groups.
Anthropometric and nutritional parameters declined regardless of
route of administration and complications were similar. Neither
modality exacerbated pancreatitis. The authors conclude that both
routes of administration when gradually delivered are well tolerated but fail to reverse the catabolism associated with the disease.
Eckerwall et al. 33 compared NG feedings with TPN in 48
well-matched patients with severe AP defined as an APACHE II
score >8 and/or a CRP level >150 mg/dL. The goal of the study
was to assess the impact of nutrient delivery on the inflammatory response of AP during the first 10 days of illness. Nutritional support was started within 24 h of admission with a target
goal reached in 66% of the entire population with no difference
between groups. No patient receiving NG feeds had aspiration.
EndoCab concentrations decreased equally in both groups during
the study period. Only one patient in the entire series required
operative pancreatic surgery. The authors concluded that NG
feedings were tolerated well in those with predicted severe AP, but
did not attenuate the inflammatory response associated with the
disease when compared with TPN.
In summary, these preliminary studies suggest that NG
feeding seems to be tolerated as well as NJ feeding in those with
severe AP in the hemodynamically stable patient without exacerbating the disease process provided close assessment of tolerance
is made. Tube placement is easier and less costly. The relationship
of NG feedings to a decline in secondary pancreatic infections
and disease-related mortality has yet to be ascertained. (Grade B
recommendation)
4. Can preoperative testing accurately predict the success of
biliary sphincteroplasty and pancreatic septoplasty in sphincter of Oddi dysfunction?
SOD is defi ned as a benign outflow obstruction of the ampulla
of Vater resulting in episodic abdominal pain mimicking more
common biliary and pancreatic pathology. The derangements
associated with the sphincter may be anatomic (stenosis) or
physiologic (incomplete relaxation) in nature. The exact etiology
is unknown but commonly implicated factors include choledocholithiasis, instrumentation of the ampulla and pancreatitis to
name a few. The discomfort may be accompanied by abnormal

laboratory and radiologic fi ndings such as elevated liver function


tests or amylase and dilation or delayed drainage of the biliary
or pancreatic ductal system. A classification of SOD based on
clinical and radiologic paramenters was proposed by Hogan and
Geenen (Table 53.1).34 Sphincter of Oddi manometry (SOM) is
invasive and has a defined rate of iatrogenic induced pancreatitis of 10% to 25%, but is the most accurate tool for diagnosis. A
variety of provocative tests have been studied as potential less
invasive substitutes for SOM. Because the diagnosis can be elusive, it is crucial that a methodical preoperative assessment utilizing discrete clinical, laboratory, radiologic, and manometric
findings be undertaken prior to consideration of therapeutic
intervention. If not, results will be less than satisfactory and
recurrent painpredictable.

CLINICAL ASSESSMENT
Sherman et al.35 assessed the accuracy of clinical parameters in
diagnosing pancreatic SOD when compared with manometric
findings defined as a sphincter pressure greater than 40 mmHg.
In 168 patients with idiopathic pancreatitis, abnormal manometry correlated with Type 1 pancreatic clinical parameters in 92%.
In contrast, Type II and III cohorts had abnormal measurements
in only 58% and 32% of patients, respectively, suggesting this procedure was an important adjunct in directing therapy in these
groups. Although the results of therapeutic intervention were not
reported, these data suggest that in those patients meeting type 1
criteria for SODmanometry is unnecessary prior to proceeding
with therapeutic intervention.

PROVOCATIVE ASSESSMENT
Noninvasive testing has been utilized in an attempt to diagnose
SOD and predict the success of therapeutic intervention while
reducing the need of SOM and its inherent risk of pancreatitis.
The morphine neostigmine provocative (Nardi) test was assessed
in 290 of 446 patients having surgical sphincteroplasty by Madura
et al.36 A positive test was defined as reproduction of symptoms or
a fourfold increase in enzyme levels. In this study, postinjection
symptoms occurred in 91%, but only 47% had lipase levels elevated
fourfold or greater. Of the 71 patients having repeat testing postoperatively, enzyme levels were reduced significantly from baseline. While a subset analysis specific to those having the Nardi test

Table 53.1 Contemporary Classification of Sphincter of Oddi Dysfunction


Type

Biliary SOD

Pancreas SOD

Type I

1. Biliary type pain (lasting 30 min and occurring at least


once per year)

1. Recurrent pancreatitis or pain suspected of pancreatic


origin

2. Elevated AST/ALT on two occasions

2. Elevated amylase or lipase

3. Dilated CBD (>12 mm), or delayed biliary drainage


(>45 min)

3. Dilated pancreatic duct or delayed emptying of the


pancreatic duct

Type II

Biliary type pain and at least one additional factor above

Presumed pancreatic pain and at least one additional factor


above

Type III

Biliary type pain alone

Pancreatic type pain alone

PMPH_CH53.indd 424

5/22/2012 5:37:13 PM

Acute Pancreatitis

was not performed, overall 83% of the patients in this series had
an excellent or good result following surgical decompression. In
a study by Geenen et al.,37 only 7 of 35 patients with type II SOD
documented by manometry had a positive Nardi testfour of
who had normal manometric pressures suggesting that this study
may be inaccurate in this cohort.
The accuracy of secretin-stimulated magnetic resonance pancreatography (MRP) defined as pancreatic duct dilation 1 mm
from baseline when compared with SOM to accurately diagnose
SOD was assessed by Aisen et al.38 Thirty patients with type II and
III classification had both MRP and SOM. MRP was unreliable in
predicting SOD and demonstrated poor sensitivity and specificity
when compared with SOM. Although data with respect to therapeutic intervention in those with SOD in this study was lacking,
these findings suggest that secretin-stimulated MRP is not helpful
in the assessment of this disease.
Cholecystokinin scintigraphy has been utilized to assess postcholecystectomy pain. Parameters analyzed have included hepatic
hilum to duodenal transit time and percentage of common bile
duct emptying. Craig et al.39 studied 29 patients with type II and III
criteria following cholecystectomy subjected to both scintigraphy
and SOM. When compared with the eight with positive SOM, scintigraphy lacked sensitivity and specificity. The authors concluded
that scintigraphy was not useful in the diagnosis of SOD.

MANOMETRIC ASSESSMENT
The ability of abnormal manometry to predict success following
endoscopic sphincterotomy for SOD was assessed by Geenen et al. 37
Entry criteria for this prospective, randomized, double-blind
study included type II SOD criteria, previous cholecystectomy,
abdominal pain, and clinical evidence suggestive of biliary
obstruction. Patients received endoscopic retrograde cholangiopancreatography (ERCP) and either true or sham biliary
sphincterotomy and most were followed for 4 years. Sphincterotomy resulted in improvement in pain scores in 10 of 11
patients with elevated pressures. In contrast, only 3 of 12 with
elevated pressure receiving the sham procedure improved. Th is
latter group of patients crossed over to formal sphincterotomy
at 1 year. At study completion, 17 of 18 with SOD verified by
manometry remained pain-free. In patients with normal sphincter pressure, sphincterotomy offered no advantage over those
receiving placebo. These data suggest that SOM is effective at
distinguishing those that will and will not improve following
endoscopic sphincterotomy.
In conclusion, provocative testing in those with suspected
SOD cannot reliably differentiate those that will and will not benefit from therapeutic intervention. In those meeting type I criteria
for this disease, endoscopic or surgical sphincter disruption may
be comfortably recommended without further diagnostic testing. In contrast, those meeting type II and III criteria should have
therapy guided by results of SOM. (Grade B recommendation)
5. Endoscopic versus surgical management of sphincter of
Oddi dysfunction: Which is superior?
Once the diagnosis of SOD has been established, options for
management include endoscopic sphincterotomy versus surgical
sphincteroplasty. Proponents of sphincterotomy site the relative

PMPH_CH53.indd 425

425

ease of the outpatient procedure, rapid recovery, and durable early


results. Detractors note the complications of perforation, bleeding, and pancreatitis (reduced with empiric stent placement),
the potential for sphincter fibrosis and recurrent disease and the
lack of reliable long-term follow-up studies. Those favoring surgical intervention promote the durability of the patulous drainage provided. Critics observe the invasiveness of the procedure,
the potential for duodenal leak and the extended recovery. There
are no randomized controlled trials of endoscopic versus surgical treatment of SOD. Indeed, randomized trials to address the
efficacy of one form of treatment or another are difficult to define,
execute and interpret.40 Despite these limitations, scrutiny of the
literature helps balance and better delineate the therapeutic management of SOD.
In the previously noted study by Geenen et al.,37 17 of 18 with
SOD and abnormal manometry had a good outcome following
endoscopic biliary sphincterotomy alone with the majority followed up for 4 years postprocedure.
In a carefully performed study, Park et al.41 reviewed 313
patients having dual pancreaticobiliary sphincterotomy for type II
and III SOD following manometric confirmation of the disease
in at least one sphincter during a 5-year time period with a mean
length of follow-up of 43 months. Patients with prior sphincter
intervention were excluded and dual sphincterotomy was performed in a single session with the placement of a prophylactic
pancreatic stent for 10 to 14 days to prevent postprocedure pancreatitis. The purpose of this study was to assess the impact of routine dual sphincterotomy on the need for re-intervention due to
recurrent symptoms versus biliary sphincterotomy alone. Importantly, over one-half of these patients in this series had elevated
pressures in both the biliary and pancreatic sphincters Forty-five
patients had immediate postendoscopic pancreatitis that was
severe in three patients. Seventy-seven patients (25%) required
re-intervention (defined as the need for repeat ERCP) due to recurrent symptoms and this did not differ between SOD types. The
majority underwent repeat sphincter ablation. Of the 77 patients
having re-intervention, 26 (33%) had further ERCP procedures.
When compared with historical controls from this same institution having biliary sphincterotomy alone, dual sphincterotomy
had a lower rate of re-intervention.
Similar observations were noted by Wehrmann42 in a series
of 37 patients with manometrically documented SOD and acute
relapsing pancreatitis followed at least 10 years postintervention.
At least one episode of recurrent pancreatitis occurred in 19 (51%)
patients. When analyzed relative to single (biliary or pancreatic)
versus dual sphincterotomy, those receiving the latter therapeutic
intervention had significantly fewer episodes of recurrent pancreatitis (12/13 vs. 7/24).
Toouli et al.43 reported a series of 26 patients having surgical
sphincteroplasty and septectomy for recurrent idiopathic pancreatitits followed for a median of 24 months including 7 with failed
prior sphincterotomy. SOD was confirmed by manometry but
subtype analysis was not performed. Dual duct sphincteroplasty
resolved further pain in 23 and was recommended by the authors
as important for optimal results.
Madura et al.36 assessed the results of surgical sphincteroplasty in 372 patients with SOD100 of who had undergone
previous sphincter ablation by either endoscopic (73) or surgical
(27) means. Stratification according to SOD type was not done in
this retrospective review of a 25-year experience. Intraoperative

5/22/2012 5:37:13 PM

426

Surgery: Evidence-Based Practice

manometry was performed pre- and postprocedure and both


sphincters were surgically disrupted in the majority. Long-term
(not defined) outcome was considered excellent or good in 86.8%
with a trend toward improved results in the procedure nave
cohort. Manometric pressures of both ducts were significantly
reduced. Patency of the major ampulla was uniformally observed
but re-stenosis of the pancreatic duct requiring re-intervention
was an occasional problem.
Morgan et al.44 evaluated results of dual duct sphincteroplasty in 51 patients during a recent 5-year time period when
classification of SOD type existed and endoscopic therapy with
sphincterotomy predominated. Unfortunately, if preoperative
manometry was performed, these results were not reported relative to SOD types (predominantly II and III). Those patients
with an accessible ampulla all had at least one prior endoscopic
sphincterotomy (median 2, range 110) prior to surgical referral. In those with an inaccessible ampulla due to prior gastric
surgery, SOD was evaluated preoperatively with magnetic resonance cholangiopancreatography. The morbidity rate in this
group of predominantly type II and III SOD was 10% including one patient with pancreatic necrosis and two patients with
intraabdominal abscess. One patient required re-do sphincteroplasty. Follow-up surveys were completed in two-thirds at a
median length of 3.5 years. Sixty-six percent reported favorable
outcomes with a trend toward better results in those without
prior endoscopic intervention.
In conclusion, surgical studies are hindered by poor reporting of preoperative manometric findings with regard to SOD
subtypes. Patients are often referred for surgery as a salvage procedure frequently resulting in outcomes inferior to those reported
in endoscopic cohorts. Studies to date suggest that surgery is equal
to endoscopic therapy if performed as the index procedure. Endoscopic sphincterotomy is associated with a higher rate of recidivism versus sphincteroplasty. Data from both the endoscopic
and surgical literature favors up-front ablation of both the biliary and pancreatic sphincters when therapeutic intervention
is performed. A randomized trial is sorely needed, stratified for
type of SOD and manometric findings, to better identify patients
that may benefit from one form of therapy or the other. (Grade B
recommendation)
6. Can preoperative testing accurately assess those that will
benefit from surgical intervention in pancreas divisum?
Pancreas divisum (PD) occurs in 5% to 10% of the population,
based on autopsy studies, but is associated with pathology in
only a minority. The pain and recurrent episodes of pancreatitis
are felt to originate from impedence to flow of pancreatic secretions at the minor papilla leading to elevated ductal pressures.
In addition, a significant proportion of those with symptomatic
PD harbor genetic mutations associated with chronic pancreatitis.45 Therapeutic intervention on the minor papilla is technically
more demanding than its sister the ampulla of Vater. Further,
some reports in both the endoscopic and surgical literature suggest therapeutic results for PD are inferior to those performed for
SOD. Thus, patient selection is paramount if meaningful procedure-related success is to be anticipated.

PMPH_CH53.indd 426

RADIOLOGIC EVALUATION
Warshaw et al.46 examined the utility of the ultrasound-secretin
test in accurately predicting the success of surgical sphincteroplasty in 100 patients with PDthe majority of who were followed
for at least 4 years. Outcomes were evaluated relative to whether
patients presented with recurrent AP (49) or pain alone (51) with
results globally favoring the former group. Ninety-two percent
of 72 patients with a positive ultrasound-secretin test had a
good outcome following sphincteroplasty versus only 42% with
a negative test. In the subset with pain but not pancreatitis and a
negative secretin provocative test, only 21% had a good result.
This study suggests that provocative testing, in conjunction with
clinical presentation, is a helpful adjunct in predicting surgical
success.

ENDOPROSTHETIC ASSESSMENT
Lans et al.47 performed a prospective, nonblinded, randomized
controlled trial of 19 patients with PD and at least two prior episodes of AP comparing dorsal duct stenting (10) to control (9).
Mean follow-up was approximately 30 months and pancreatic
stents were removed at 1 year. In those with an endoprosthesis in
place, symptomatic improvement, emergency room visits, hospitalizations, and documented episodes of AP were significantly
reduced as compared with control. Four patients in the control
arm crossed over to stenting of the minor papilla with no episodes of hospitalization or AP at follow-up ranging from 6 to 53
months. No patient in this study had either endocopic or surgical sphincteroplasty. It should be noted that stenting of either
pancreatic duct has been associated with the early development
of chronic pancreatitis and/or ductal strictures on follow-up
imaging.48
Siegel et al.49 studied 31 consecutive patients with symptomatic PD including 5 with prior surgical sphincteroplasty found
to have postoperative stenosis. Minimal criteria for study entry
included pain, either serum amylase or lipase values greater
than three times normal and/or evidence of pancreatitis via CT
scan.49 Twenty-six patients (84%) had symptomatic improvement following stent placement though data relative to any
further episodes of AP was not reported. During an average
follow-up of 2 years the authors noted progressive stricturing
and ductal dilation of the minor duct suggesting that the stent
itself induced chronic pancreatitis if left for prolonged periods.
Seventeen patients ultimately had surgical intervention including dual duct sphincteroplasty (11) or resection with 15 having
a good outcome.
In summary, PD should be considered an incidental finding
unless accompanied by documented episodes of AP. Both secretin provocative testing and temporary stent placement though the
minor papilla are predictive of success following surgical or endoscopic sphincteroplasty though neither is firmly recommended
prior to proceeding to therapeutic intervention. Long-term endoprosthesis placement should be avoided to prevent deterioration
to chronic pancreatitis. (Grade B recommendation)

5/22/2012 5:37:13 PM

Acute Pancreatitis

427

Clinical Question Summary


Question

Answer

1 What is the role of ERCP in acute


biliary pancreatitis?

Only if evidence of cholangitis or


biliary obstruction exists

1a

5-7

2 What is the role of prophylatic


antibiotics in severe AP?

Studies do not show a benefit.


May be reasonable in severe AP

1b

16-23

3 Is gastric feeding safe and equivalent


to jejunal feeding in AP?

Safe in hemodynamically stable


patients if tolerated

2b

31-33

4 Can the success of intervention for


SOC be predicted?

Yes, when correlated with SOD


type

1b

35-39

5 Is therapeutic endoscopy or surgery


superior for SOD?

Equal early results, surgery may


reduce recidivism

2c

36, 37, 41-44

6 Can the success of intervention for


PD be predicted?

Yes, when correlated with AP

2c

46, 47, 49

REFERENCES
1. Whitcomb DC. Acute pancreatitis. N Engl J Med. 2006;354(20):
2142-2150.
2. Madura JA, Madura JA. Diagnosis and management of sphincter of Oddi dysfunction and pancreas divisum. Surg Clin N Am.
2007;87:1417-1429.
3. Bradley EL. A clinically based classification system for acute
pancreatitis. Summary of the International Symposium on Acute
Pancreatitis. Arch Surg. 1993;128(5):586-590.
4. Gloor B, Muller CA, Worni M, et al. Late mortality in patients
with severe acute pancreatitis. Br J Surg. 2001;88(7):975-979.
5. Neoptolemos JP, Carr-Locke DL, London NJ, et al. Controlled
trial of urgent endoscopic cholangiopancreatography and endoscopic sphincterotomy versus conservative treatment of acute
pancreatitis due to gallstones. Lancet. 1988;2(8618):979-983.
6. Fan S, Lai E, Mok F, et al. Early treatment of acute biliary pancreatitis by endoscopic papillotomy. N Engl J Med. 1993;328(4):
228-232.
7. Folsch UR, Nitsche R, Ludtle R, et al. Early ERCP and papillostomy compared with conservative treatment for acute biliary
pancreatitis. The German Study Group on acute biliary pancreatitis. N Engl J Med. 1997;336(4):237-242.
8. Oria A, Cimmino D, Ocampo C, et al. Early endoscopic intervention versus early conservative management in patients with
acute gallstone pancreatitis and billiopancreatic obstruction: A
randomized clinical trial. Ann Surg. 2007;245(1):10-17.
9. Behrns KE, Ashley SW, Hunter JG, et al. Early ERCP for gallstone pancreatitis: For whom and when? J Gastrointest Surg.
2008;12(4):629-633.
10. Petrov MS, van Santvoort HC, van der Heijden GJ, et al. Early
endoscopic retrograde cholangiopancreatography versus conservative management in acute biliary pancreatitis: A meta-analysis
of randomized trials. Ann Surg. 2008;247(2):250-257.
11. Van Santvoort HC, Besselink MG, de Vries AC, et al. Early endoscopic retrograde cholangiopancreatography in predicted sever
acute biliary pancreatitis. Ann Surg. 2009;250:68-75.
12. Rodriguez JR, Razo AO, Targarona J, et al. Debridement and
closed packing for sterile or infected necrotizing pancreatitis:
Insights into indications and outcomes in 167 patients. Ann Surg.
2008;247(2):294-299.

PMPH_CH53.indd 427

Level of
Evidence

Grade of
Recommendation

References

13. Hoerauf A, Hammer S, Muller Myhsok B, et al. Intra-abdominal


Candida Infection during acute necrotizing pancreatitis has a
high prevalence and is associated with increased mortality. Crit
Care Med. 1998;26(12):2010-2015.
14. Buchler M, Malfertheiner P, Friess H, et al. Human pancreatic
tissue concentration of bactericidal antibiotics. Gastroenterology.
1992;103(6):1902-1908.
15. Bassi C, Pederzoli P, Vesentini S, et al. Behavior of antibiotics
during human necrotizing pancreatitis. Antimicrob Agents Chemother. 1994;38(4):830-836.
16. Penderzoli P, Bassi C, Vesentini S, et al. A randomized multicenter
clinical trial of antibiotic prophylaxis of septic complications
in acute necrotizing pancreatitis with imipenem. Surg Gynecol
Obstet. 1993;176(5):480-483.
17. Isenmann R, Runzi M, Kron M, et al. Prophylactic antibiotic
treatment in patients with predicted severe acute pancreatitis: a
placebo-controlled, double-blind trial. Gastroenterology. 2004;
126(4):997-1004.
18. Dellinger EP, Tellado JM, Soto NE, et al. Early antibiotic treatment for severe necrotizing pancreatitis: A randomized, doubleblind, placebo-controlled study. Ann Surg. 2007;245(5):674-683.
19. Garcia-Barrasa A, Borobia FG, Pallares R, et al. A double-blind,
placebo-controlled trial of ciprofloxacin prophylaxis in patients
with acute necrotizing pancreatitis. J Gastrointest Surg. 2009;
13:768-774.
20. Xue P, Deng LH, Zhang ZD, et al. Effect of antibiotic prophylaxis
on acute necrotizing pancreatitis: Results of a randomized controlled trial. J Gastroenterol Hepatol. 2009;24:736-742.
21. Bai Y, Gao J, Zou D, et al. Prophylactic antibiotics cannot reduce
infected pancreatic necrosis and mortality in acute necrotizing
pancreatitis: Evidence from a meta-analysis of randomized controlled trials. Am J Gastroenterol. 2008;103(1):104-110.
22. Hart PA, Bechtold ML, Marshall JB, et al. Prophylactic antibiotics in necrotizing pancreatitis: A meta-analysis. South Med J.
2008;101:1126-1131.
23. Jafri NS, Mahid SS, Idstein ST, et al. Antibiotic prophylaxis is not
protective in severe acute pancreatitis: A systematic review and
meta-analysis. Am J Surg. 2009;197:806-813.
24. Dickerson RN, Vehe KL, Mullen JL, et al. Resting energy expenditure in patients with pancreatitis. Crit Care Med. 1991;19:
484-490.

5/22/2012 5:37:13 PM

428

Surgery: Evidence-Based Practice

25. Kudsk KA. Beneficial effect of enteral nutrition. Gastrointest


Endosc Clin N Am. 2007;17:647-662.
26. Zaloga GP. Bedside method for placing small bowel feeding
tubes in critically ill patients. A prospective study. Chest. 1991;
100:1643-1646.
27. Windsor A, Kanwar S, Li A, et al. Compared with parenteral
nutrition enteral feeding attenuates the acute-phase response
and improves disease severity in acute pancreatitis. Gut. 1998;
42(3):431-435.
28. McClave SA, Greene LM, Snider HL, et al. Comparison of the
safety of early enteral vs parenteral nutrition in mild acute pancreatitis. J Parenter Enter Nutr. 1997;21(1):14-20.
29. Kalfarentzos F, Kehagias J, Mead N, et al. Enteral nutrition is
superior to parenteral nutrition in severe acute pancreatitis: results of a randomized prospective trial. Br J Surg. 1997;
84(12):1665-1669.
30. Petrov MS, Kukosh MV, Emelyanov NV. A randomized controlled trial of enteral versus parenteral feeding in patients with
predicted severe acute pancreatitis shows a significant reduction
in mortality and in infected pancreatic complications with total
enteral nutrition. Dig Sug. 2006;23(5-6):336-345.
31. Eatock F, Chong P, Menezes N, et al. A randomized study of early
nasogastric versus nasojejunal feeding in severe acute pancreatitis. Am J Gastroenterol. 2005;100(2):432-439.
32. Kumar A, Singh N, Prakash S, et al. Early enteral nutrition in
severe acute pancreatitis: A prospective randomized controlled
trial comparing nasojejunal and nasogastric routes. J Clin Gastroenterol. 2006;40(5):431-434.
33. Eckerwall G, Axelsson J, Andersson R. Early nasogastric feeding in predicted severe acute pancreatitis: a clinical randomized
study. Ann Surg. 1006;244(6):959-967.
34. Hogan WJ, Greenen JE. Biliary dyskinesia. Endoscopy. 1988;
20(Suppl 1):179-783.
35. Sherman S, Iroiano FP, Hawes RH, et al. Frequency of abnormal sphincter of Oddi manometry compared with the clinical
suspicion of sphincter of Oddi dysfunction. Am J Gastroenterol.
1991;86:586-590.
36. Madura JA, Madura JA, II , Sherman S, et al. Surgical sphincteroplasty in 446 patients. Arch Surg. 2005;140:504-512.
37. Geenen JE, Hogan WJ, Dodds WJ, et al. the efficacy of endoscopic sphincterotomy after cholecystectomy in patients with
spincter-of-Oddie dysfunction. N Engl J Med. 1989;320:82-87.

PMPH_CH53.indd 428

38. Aisen AM, Sherman S, Jennings SG, et al. Comparison of


secretin-stimulated pancreatography and manometry results in
patients with suspected sphincter of ODDI dysfunction. Acad
Radiol. 2008;15:601-609.
39. Craig AG, Peter D, Saccone GTP, et al. Scintigraphy versus
manometry in patients with suspected biliary spincter of Oddi
dysfunction. Gut. 2003;52:352-357.
40. Cotton PB, Durkalski V, Orrell KB, et al. Challenges in planning
and initiating a randomized clinical study of sphincter of Oddi
dysfunction. Gastrointest Endosc. 2010;72:986-991.
41. Parks SH, Watkins JL, Fogel EL, et al. Long-term outcome of
endoscopic dual pancreatobiliary sphincterotomy in patients
with manometry-documented sphincter of Oddi dysfunction and normal pancretogram. Gastrointest Endosc. 2003;57:
483-491.
42. Wehrmann T. Long-term results (10 years) of endoscopic
therapy for sphincter of Oddi dysfunction in patients with acute
recurrent pancreatitis. Endoscopy. 2011;43:202-207.
43. Toouli J, diFrancesco V, Kollias SJ, et al. Divison of the sphincter
of Oddi for treatment of dysfunction associated with recurrent
pancreatitis. Br J Surg. 1996;83:1205-1210.
44. Morgan KA, Ramognuolo J, Adams DB. Transduodenal sphincteroplasty in the management of sphincter of Oddi dysfunction and pancreas divisum in the modern era. J Am Coll Surg.
2008;206:901-914.
45. Garg PK, Khajuria R, Kabra M, et al. Association of SPINK
1 gene mutation and CFTR gene polymorphisms in patients with
pancreas divisum presenting with idiopathic pancreatitis. J Clin
Gastroenterol. 2009;43:848-852.
46. Warshaw AL, Simeone JF, Schapiro RH, et al. Evaluation and
treatment of the dominant dorsal duct syndrome (pancreas
divisum redefined). Am J Surg. 1990;159:59-66.
47. Lans JI, Geenen JE, Johanson JF, et al. Endoscopic therapy in
patients with pancreas divisum and acute pancreatitis: A prospective, randomized, controlled clinical trial. Gastrointest Endosc.
1992;38:430-434.
48. Attasaranya SA, Abdel Aziz AM, Lehman GA. Endoscopic management of acute and chronic pancreatitis. Surg Clin N Am. 2007;
87:1379-1402.
49. Siegel JH, Ben-Zvi JS, Puliano W, et al. Effectiveness of endoscopic drainage for pancreas divisum: Endoscopic and surgical
results in 31 patients. Endoscopy. 1990;22:129-133.

5/22/2012 5:37:13 PM

Commentary on
Acute Pancreatitis
Wayne H. Schwesinger

As illustrated in this excellent review, acute pancreatitis covers a


broad clinical spectrum: from common and straightforward to
rare and esoteric. In each case, optimal management requires a
multidisciplinary approach often involving gastroenterology, radiology, and surgery. Continuing advances in technology in each of
these specialties has generally promoted less invasive methodologies, often with minimal supportive evidence. Increasingly,
the role of surgery has been relegated to the prevention of recurrence (laparoscopic cholecystectomy) or the treatment of major
complications (pancreatic necrosectomy). Accordingly, few open
sphincterotomies are currently performed and even the volume of
necrosectomies has dramatically decreased. Whether the fledgling enthusiasm for laparoscopic retroperitoneal necrosectomy

will alter the latter experience is far from clear. Much more operative experience with this apparently demanding procedure is necessary. Meanwhile, improvements in radiological and endoscopic
methods are regularly reported.
It seems clear that the potentially lethal nature of acute pancreatitis will continue to stimulate re-newed attempts at improved
diagnosis and control. Surgeons are integral to these efforts
and must remain active members of the management and
research teams. Ongoing critical assessments such as the current
review represent an important step in our evolving understanding of acute pancreatitis and will ultimately help us to select
the most appropriate and efficacious of the new management
strategies.

429

PMPH_CH53.indd 429

5/22/2012 5:37:13 PM

CHAPTER 54

Infected Pancreatic Collections


Nader N. Massarweh and Karen D. Horvath

INTRODUCTION

Pancreatitis

In the United States, acute pancreatitis is among the three most


common gastrointestinal diseases requiring hospitalization, with an
annual cost of over USD 2 billion.1,2 The majority (75%) of patients
have a mild form of acute pancreatitis and resolve their symptoms
without event. The rest may go on to develop a more severe form of
acute pancreatitis with consequent organ failure and/or the development of organ necrosis. Despite modern medical advances, severe
acute pancreatitis is still currently associated with significant rates
of morbidity and mortality approaching 15% to 30%.3
While management of mild acute pancreatitis has changed
little in the past decade, appropriate management of patients
with severe acute pancreatitis and its associated complications
has undergone significant changes. Sterile peripancreatic collections can usually be treated conservatively. However, secondary
infection of peripancreatic collections occurs in about 33% of
patientsgenerally 3 to 4 weeks after the onset of disease. Without timely and effective treatment of these patients, mortality
approaches 100%.4
A body of data is currently emerging to help clarify some of
the questions regarding the optimal management of patients with
severe acute pancreatitis who develop infectious complications. A
potential source of much of the uncertainty surrounding management has been the use of different terminology in the literature and amongst clinicians over the last 20 years. For example,
the term pancreatic abscess may be used by some clinicians to
describe a process others might refer to as infected necrosis and
yet others might call an infected pseudocyst. Although the distinction may seem subtle, these differences can markedly affect
the selection of proper management algorithms and thus treatment outcomes. The current confusion surrounding severe acute
pancreatitis nomenclature developed over time as we have come to
understand more about this disease. The Atlanta Symposium Consensus Conference5 terminology, which provided a key advance
in 1993, is now outdated. In this regard, an international working group has updated the Atlanta classification system to create
a common framework for discussing the hierarchy and severity

IEP
Early

*APFC

Necrotizing
Late

*Pseudocyst

Early

*APNC

Late

*WON

Figure 54.1 Updated Atlanta classification


Note that the term pancreatic abscess is not used in this classification.
Early is <4 weeks. Late is >4 weeks.
*Each of these entities can be infected or noninfected.
IEPInterstital edematous pancreatitis; APFCAcute peripancreatic fluid
collection; APNCAcute postnecrotic collection; WONWalled-off
necrosis.

of pancreatitis-associated complications.6 It is anticipated that the


uniform adoption of the new classification will both aid and simplify clinical management, research methodology, and providerto-provider communication. For these reasons, we describe the
updated terminology below and use these terms throughout the
remainder of this chapter (Figure 54.1).

NOMENCLATURE
1. How should infectious complications associated with pancreatitis be described?
Throughout the inflammatory phase, fluid collections may develop
within or around the pancreas. In the early stage of the disease
(<4 weeks), the pancreatitis can be morphologically characterized
as interstitial edematous pancreatitis (IEP), or an acute peripancreatic fluid collection (APFC) when a separate collection is present.
By comparison, if the computed tomography (CT) morphology
demonstrates necrotizing pancreatitis (with necrosis of the gland
and/or the peripancreatic tissues), the collection is referred to as
an acute postnecrotic collection (APNC) of the pancreatic parenchyma and the peripancreatic tissues, the pancreatic parenchyma
430

PMPH_CH54.indd 430

5/22/2012 5:38:02 PM

Infected Pancreatic Collections

alone, or the peripancreatic tissue alone. In all cases, the APNC


contains variable amounts of both fluid and necrotic pancreatic
or peripancreatic tissue.
In the later stages of the disease (4 weeks), an APFC may
develop a defined inflammatory wall and now be considered a
pseudocyst. An APNC may also develop a defined wall at which
point it is classified as walled-off necrosis (WON). In all cases
(early/late, APFC/APNC, or pseudocyst/WON), these collections may be considered sterile or infected. The term pancreatic
abscess, as it has been traditionally used, is not specific enough to
adequately characterize the infectious complications. For example, under the prior Atlanta Classification, the term referred to
a purulent fluid collection in the absence of significant necrotic
tissue (an uncommon finding). Therefore, the term was not incorporated into the new classification and will not be used herein.
Answer: To provide a more uniform framework for the classification of infectious sequelae associated with pancreatitis and
to facilitate communication between providers caring for these
patients and better standardization of research terminology, the
New International Working Group Consensus Classification system (an update of the former Atlanta Classification) should be
used. (Grade D recommendation).

DIAGNOSIS
2. Should fine-needle aspiration biopsy (FNAB) be used?
Early and accurate identification of infection in a pancreatic or
peripancreatic collection is crucial for providing patients with
timely and appropriate care since patients with severe acute pancreatitis who have infectious complications have a significantly
higher mortality rate. Although the use of modern, contrastenhanced imaging has greatly facilitated the diagnosis and classification of severe acute pancreatitis and its associated complications,
imaging modalities alone are not always sufficient for establishing
the diagnosis of infection. The presence of gas in a peripancreatic
collection is considered pathognomonic for the presence of infection, but in its absence a practitioner must rely on clinical judgment and physiologic parameters to establish the diagnosis of
infection. However, this can be challenging since common signs
of infection may be elusive or absent or may simply indicate the
presence of other sources of infection (urinary tract infections,
pneumonia, bacteremia from line sepsis, etc.), which are common
in this patient population. In these cases tissue/fluid sample with
subsequent gram stain and bacterial culture can be very useful.
Although there is a lack of high-level evidence to direct the
use of image-guided fine-needle aspiration biopsy (FNAB), there
is general consensus that when the presence of infection is suspected, but not clearly demonstrated on imaging, FNAB is indicated to help direct therapy. In the past decade, only one study
has provided data regarding the sensitivity and specificity of
FNAB. Evaluating 98 patients with necrotizing pancreatitis using
ultrasound-guided FNAB, Rau et al.7 described an 88% and 90%
sensitivity and specificity, respectively. Although CT-guided
approaches are likely more common, there are no contemporary
studies that have evaluated this method of tissue sampling. However, one would suspect the sensitivity/specificity would at least
be similar to (if not better than) the ultrasound-guided method.
Endoscopic methods introduce the potential for iatrogenic infection by gut flora and should be avoided.

PMPH_CH54.indd 431

431

Answer: Despite a lack of high-level evidence, there is general consensus that FNAB has a role in establishing the diagnosis
of infection when imaging alone is not adequate. When FNAB is
used, it has high sensitivity/specificity and can be repeated multiple times in the same patient with a very low risk of iatrogenic
infection. (Grade C recommendation).

NUTRITION
3. Should enteral or parenteral feeding be used?
Enteral nutrition (via the nasoenteral route or through direct
enteric access) is an integral tool in the management of critically
ill general surgical patients. But, in patients with severe acute pancreatitis, there exists a longstanding dogma that enteral nutrition
has the potential to stimulate exocrine pancreatic function thereby
exacerbating the peripancreatic inflammatory process. There is,
however, little evidence to suggest that this actually occurs. To the
contrary, there are data demonstrating that these patients are benefited when enteral nutrition is used.
Several randomized controlled trials (RCTs) have been performed comparing the use of enteral and parenteral nutrition
among patients with pancreatitis. Various measures of morbidity
and mortality have been evaluated, including the occurrence of
infectious complications. Data from these trails have been analyzed in three meta-analysesall of which found statistically
significant reductions in the risk of systemic and pancreatic infectious complications. Marik et al.8 found the use of enteral nutrition to be associated with a significantly lower risk of infectious
complications, need for surgical intervention, and length of hospital stay, but failed to identify a decreased risk of mortality or
noninfectious complications. However, this study had two important limitations to consider. First, the study had a relatively small
sample size and a small number of mortality events, which likely
underpowered the analysis. Second, the study included patients
who were heterogenous in terms of the severity of acute pancreatitis, which may have biased the assessment of mortality toward
the null.
In a subsequent analysis, Petrov et al. restricted their inclusion criteria to only those studies evaluating patients with severe
acute pancreatitis.9 When patients with mild pancreatitis were
excluded (limiting the analysis to a more homogeneous group of
patients with severe acute pancreatitis), use of enteral nutrition
was associated with statistically significant reductions in the risk
of total and pancreatic infectious complications, need for surgical interventions, and mortality (without significant heterogeneity across the included studies). Similarly, in a recent Cochrane
review updating the analysis performed by Marik et al., Al-Omran
et al.10 included three additional studies which increased the sample size from 263 to 348 patients. While the prior analysis failed
to identify a decreased risk of mortality, these additional patients
may have provided adequate power to allow identification of a
statistically significant reduction in the risk of mortality associated with enteral nutrition. However, since some of the studies
included in these analyses were performed prior to the current
emphasis placed on the prevention of sepsis associated with temporary central lines, it is possible that these data may be biased
in favor of enteral nutrition. Nonetheless, providing supplemental
nutrition in a manner that minimized possible routes of infection
is an appealing general principle of treating critically ill patients.

5/22/2012 5:38:02 PM

432

Surgery: Evidence-Based Practice

Given that enteral nutrition appears to provide consistent


and measurable benefit, another important question to consider is
whether the route of enteral administration (i.e., gastric vs. jejunal)
makes a difference? Two small randomized studies have addressed
this issue. First, Eatock et al.11 sought to evaluate whether nasogastric feeding is equivalent to the nasojejunal route in terms of
safety, effectiveness, complications, and exacerbation of inflammation and pain. Because the authors did not use a noninferiority design, this study was underpowered to establish equivalence.
However, nasojejunal feeding was not found to be superior when
compared with nasogastric feeding in terms of clinical and physiologic parameters. In a second pilot study, the authors again did
not find evidence that jejunal feeding was superior to intragastric
feeding.12 Although the results of these two studies are compelling and appear to support the conclusion that jejunal feeding is
not superior to nasogastric feeding, both suffer from small sample
size, which may have unpowered these analyses to detect significant differences in the primary outcome or in other meaningful
complications such as aspiration (which is a highly relevant issue
in this patient population). Future work will, therefore, be necessary to confirm these findings. In the meantime, it seems prudent
to advise that nasogastric tube feeding be considered in patients
with severe acute pancreatitis who either are capable of protecting
or have a protected airway.
Answer: There exists consistent evidence that using enteral,
as compared with parenteral, nutrition results in clinically and
statistically significant decreases in the risk of pancreatitisassociated morbidity and mortality. Furthermore, there currently
does not appear to be evidence that jejunal administration of
enteral nutrition offers any significant advantage over the gastric
route. As such, enteral feeding should be the preferred method
of providing nutrition to patients with pancreatitis and the route
(jejunal vs. gastric) should be tailored to each patient and/or dictated by the practitioners best clinical assessment and judgment.
(Grade A recommendation).
4. Should feeding begin early or be delayed?
The management paradigm for patients with pancreatitis has traditionally included resting the gastrointestinal tract and providing intravenous fluids until the initial signs and symptoms of
pancreatic inflammation, such as lab markers (amylase and lipase)
and abdominal pain subside. The rationale for this approach has
been to minimize passage of food through the duodenum and
past the pancreas which was thought to stimulate pancreatic exocrine function and potentially drive the inflammatory response.
Although this form of management may be appropriate for the
75% of patients who present with a self-limited attack of garden variety mild acute pancreatitis, for patients diagnosed with
severe acute pancreatitis, more aggressive management is needed.
In these patients prolonged fasting can have important negative
ramifications for the patient including bacterial overgrowth in the
small bowel with atrophy of the mucosal barrier increasing the
risk of bacterial translocation leading to infectious complications, progressive and profound protein calorie malnutrition, and
immune compromise. As the enteral route, when tolerated, is the
preferred method for providing nutrition to patients with severe
acute pancreatitis, remaining questions are what to feed patients
and when to begin feeding (immediately vs. delayed)?
The inflammatory process associated with severe acute pancreatitis can create a profound, chronic, catabolic state in a patient who

PMPH_CH54.indd 432

is generally unable to consistently consume enough calories to keep


up with their caloric needs, so nutritional supplementation through
the use of tube feeding will often be required. However, selecting
from the many and varied nutritional formulations can be challenging. A recent meta-analysis performed by Petrov et al.13 compared
the use of (semi)elemental with polymeric formulations in terms of
infectious complications and mortality. The authors found no identifiable difference in the risk of mortality, feeding intolerance, or
infectious complications associated with either formulation.
To date, only one randomized trial has compared an early with
a delayed approach to feeding.14 In this study, Eckerwall et al.14
randomized 60 patients with mild acute pancreatitis to an early
feeding strategy where patients received immediate oral feeding
(as tolerated) or a delayed feeding approach where patients underwent immediate fasting with administration of intravenous fluids. They found no differences in pain, inflammatory markers, or
gastrointestinal symptoms. However, those randomized to immediate oral feeding spent shorter periods of time on intravenous
fluids, reintroduced solid food much earlier, and had a significantly shorter length of hospital stay. There are several important
considerations when reflecting on the results of this trial. First,
and most importantly, this study only included patients with mild
pancreatitis. Because it is very likely the severity of the inflammatory process that influences a patients ability to tolerate oral
intake, the results of this work may not necessarily generalize to
those with severe acute pancreatitis. Second, this study was powered to detect a difference in length of stay. Therefore, although
there were potentially important differences between the early
and delayed feeding strategies (e.g., in many cases, the proportion
of gastrointestinal symptoms in the early feeding group was lower,
but not statistically significant) the small sample size may not have
been sufficient. Finally, the authors did not evaluate whether one
strategy resulted in more infectious complications.
Answer: For patients who either have or are predicted to have
severe acute pancreatitis, it is advised that supplemental, enteral
nutrition through an appropriate nasoenteric route (tailored to
the patient) begin in those not expected to resume adequate oral
intake within approximately 3 days. There does not appear to be
any clinical difference between using a (semi)elemental or a polymeric formulation. For patients who can tolerate oral nutrition and
who are able to consume the required number of calories, a liberalized diet (beyond normal step-wise advancement) may result in
a shorter length of hospital stay. (Grade B recommendation).
5. Should probiotics be used?
The idea of using of probiotics as a nutritional supplement for the
purpose of reducing bacterial overgrowth, protecting the integrity of the barrier function of the bowel, and modulating immune
function has gained in popularity. However, two important studies reporting results from the PROPATRIA trial conducted by the
Dutch Acute Pancreatitis Study Group have helped to clarify the
role for probiotics.15 In the first, 296 patients predicted to have a
severe course of acute pancreatitis were randomized to a blinded
28-day course of either a multispecies probiotic preparation or a
placebo.16 All patients received concurrent nasojejunal tube feeding. Although the results of this trial did not suggest probiotic
prophylaxis resulted in any reduction in the risk of infectious complications, the authors unexpectedly found higher rates of organ
failure, bowel ischemia, and mortality in the probiotic group.
In a second study, the authors evaluated a subgroup of patients

5/22/2012 5:38:02 PM

Infected Pancreatic Collections

from the PROPATRIA trial in whom assessments of enterocyte


damage, intestinal permeability, and bacterial translocation were
made.17 Probiotic administration was associated with decreased
bacterial translocation and increased enterocyte damage in
the overall cohort. However, when the subgroup with organ failure
was evaluated, probiotics appeared to increase bacterial translocation and enterocyte damage. Thus, it appeared that the adverse
effect of probiotics on mortality was limited to patients with organ
failure. Although the mechanism responsible for the unexpected
findings in both studies is unclear, probiotics should be avoided
until the reason for these adverse outcomes is resolved or until
subgroups in whom probiotics are beneficial can be identified.
Answer: Probiotics should not be used in the management of
patients with severe pancreatitis because they appear to be associated with an increased risk of organ failure, bowel ischemia, and/
or mortality. (Grade A recommendation).

MANAGEMENT
6. Are prophylactic antibiotics recommended?
Local and/or systemic infections are common complications in
patients with severe acute pancreatitis. Early infections, such as
pneumonia or bacteremia, usually develop within the first week
following hospital admission while infected pancreatic necrosis develops later (a median of 26 days following symptoms).3
Whether early or late, the mortality of infectious complications in severe acute pancreatitis portends a worse prognosis.
Clinicians may, therefore, perceive antibiotic prophylaxis as a reasonable method of decreasing the risk of infectious complications
thereby lowering the risk of death.
However intuitive this reasoning may seem, current data do
not appear to support the use of prophylactic antibiotics in patients
who have, or are predicted to have, severe acute pancreatitis. A
recent Cochrane review evaluated the effectiveness of prophylactic antibiotics in the setting of severe necrotizing pancreatititis.18
The results did not indicate any significant benefit (in terms of
mortality or infection) associated with the use of prophylactic
antibiotics. There are several important issues to consider. First,
these seven trials evaluated various types of antibiotics, some of
which are uncommonly used for pancreatitis (i.e., quinolones), so
the lack of a significant benefit may have been a function of the
disparate types of antibiotics used. Of the seven trials included in
this study, five assessed the use of -lactam antibiotics (commonly
used for pancreatitis); however, the combined data did not identify
a significant difference. A second important consideration is that
only the two most recent trials included in this analysis were
double-blind. Finally, in the overall analysis and the subgroup
analyses, the use of antibiotics was associated with notably lower
rates of each outcome assessed (mortality [8.4% vs. 14.4%]; infected
pancreatic necrosis [19.7% vs. 24.4%]; nonpancreatic infection
[23.7% vs. 36.0%]; all sites of infection [37.5% vs. 51.9%]). Therefore, although there may not have been a statistically significant
decrease in the risk of infectious complications and mortality, this
meta-analysis simply may not have been adequately powered to
evaluate the effectiveness of a current and appropriate prophylactic antibiotic regimen. However, a recently updated systematic
review and meta-analysis including seven additional randomized
studies (with a total of 841 patients) provides similar results that
appear to confirm these findings.19

PMPH_CH54.indd 433

433

Answer: There is insufficient evidence to support routine use


of antibiotics as a means of preventing infectious complications
in patients with necrotizing pancreatitis. The use of antibiotics
for targeted therapy should be based on a thorough assessment of
clinical and laboratory data. (Grade A recommendation).
7. How should infected pancreatic collections be managed?
Traditionally, the presence of an infected pancreatic or peripancreatic collection has been an indication for open necrosectomy.
However, the different methods for open surgical necrosectomy
in these often moribund patients are associated with a high rate
of morbidity and mortality,20-22 so less invasive interventions are
increasingly being utilized with quality data supporting their
use. However, there may be instances where immediate surgical
intervention is still required because percutaneous drainage alone
is either inadequate or not feasible (e.g., multi-loculated fluid collections; mixed fluid and solid components of the collection; no
direct path for placement of a drainage catheter; and a patient
with visceral ischemia). In these cases, an open transabdominal
necrosectomy affords ample access to the pancreatic bed and surrounding tissues and allows drains to be properly positioned. For
these reasons, open necrosectomy techniques should not disappear from the armamentarium of the general surgeon.
The use of percutaneous drains for infected pancreatic collections was first described in the mid-1980s. Drains are usually placed
percutaneously using ultrasound or CT guidance; however, transluminal catheters can also be placed by an endoscopist via the transgastric or transduodenal route. Once considered only an adjunctive
therapy or a temporizing measure to delay surgical intervention, there
is evidence that drains alone may represent adequate therapy and in
many cases and may be the only therapeutic intervention needed.
For example, a recent systematic review of 11 nonrandomized studies suggested that catheter drainage might be the only treatment
needed in up to 55% of patients with necrotizing pancreatitis.23
Even when percutaneous drainage is not successful, the drain
tract can be used as roadmap for minimally invasive necrosectomy.
The best studied approach for minimally invasive necrosectomy is
known as the Step-Up approach. This involves placement of percutaneous drains followed by a minimally invasive necrosectomy
procedure when drainage alone is unsuccessful. Step-Up has been
classically paired with the technique of video-assisted retroperitoneal debridement (VARD), a simple, low-tech technique requiring minimal resource utilization, which involves a 4- to 5-cm left
flank incision with debridement through a 10-mm laparoscopic
port followed by postoperative lavage.24,25 In a recent multicenter,
prospective Phase II study of 40 patients with infected necrosis,26
the Step-Up approach was associated with an excellent safety profi le (7.5% bleeding complication rate, a 17.5% enteric fistula rate
and a 2.5% mortality) and a high success rate (60% of patients who
required intervention were successfully managed by percutaneous drains with/without a VARD and 23% of patients were managed with percutaneous drains alone). In another case-matched
study of open necrosectomy with continuous postoperative lavage
compared with retroperitoneal debridement with continuous
postoperative lavage, the authors found no difference in the rate of
postoperative complications, but a lower rate of in-hospital mortality and a significantly lower rate of organ failure in the retroperitoneal debridement group.27
A recent trial performed by the Dutch Pancreatitis Study group
(the PANTER trial) provides data comparing the Step-Up approach

5/22/2012 5:38:02 PM

434

Surgery: Evidence-Based Practice

with open necrosectomy.28 In this RCT, 88 patients with infected


pancreatic or peripancreatic tissue were randomized to either
open necrosectomy (45 patients) or a minimally invasive Step-Up
approach (43 patients). Patients randomized to the Step-Up arm had
a lower rate of incidence of postoperative organ failure, a lower rate
of major complications, and a lower risk of death or complications
(primarily attributable to the decreased complication rate). Postoperatively, the Step-Up arm had fewer incisional hernias (7% vs.
24%), less diabetes (16% vs. 38%), less exocrine insufficiency with
need for pancreatic enzyme supplementation (7% vs. 33%), and
lower healthcare utilization and medical costs (12% cheaper as
compared with open necrosectomy). In addition, surgical intervention (other than drain placement) was avoided in one-third of
patients in the Step-Up arm. These results provide compelling data
in favor of the minimally invasive Step-Up approach.
Although Step-Up offers a clear advance in the management of
patients with infectious complications of pancreatitis, there are other
minimally invasive options that can be considered. Another widely
used minimally invasive approach also associated with decreased
mortality as compared with open necrosectomy is minimal access
percutaneous retroperitoneal necrosectomy (MAPRN).29 This technique involves dilating a percutaneous drainage tract (to approximately 30 F) and use of an operating nephroscope to accomplish
debridement. Postoperative lavage is then used with repeated necrosectomies at 7 to 10 day intervals using local anesthetic. If VARD
or MAPRN is not feasible, a transluminal endoscopic approach
(NOTES) can be considered.30-32 Although early results seem promising, the lack of wide availability of this technique and the need for
randomized data should argue against its routine use at this time.
Pending the results of randomized comparisons, patients
with severe acute pancreatitis should likely be referred to centers,
or treated by practitioners, that can confidently offer the full gambit of multidisciplinary therapeutic options and have experience
making the complex management decisions so often associated
with this complicated disease process.
Answer: Infected pancreatic collections should be managed
using a Step-Up approach, which involves the placement of percutaneous or transluminal drains with or without subsequent minimally invasive necrosectomy. When compared with traditional
open necrosectomy, minimally invasive approaches can be as efficacious with lower rates of complications and healthcare utilization. (Grade A recommendation).

8. What is the optimal timing for surgical intervention in


patients with infected necrosis?
Over the past 50 years, the timing of surgical intervention for
patients with infected necrosis has changed significantly. From
the 1960s to the mid-1990s, surgeons performed radical, open
necrosectomies or even total pancreatectomies on patients within
the first few days of presentation with mortality rates approaching 70%. The use of percutaneous drains in combination with
significant improvements in intensive care unit (ICU) care during the mid-1980s, introduced a new paradigm of delaying surgical intervention and led to a decrease in mortality rates. Among
patients with necrotizing pancreatitis, an early (within 72 h of
admission) interventional approach has been compared with a
late approach (after 12 days) in an RCT.33 This study terminated
prematurely due to a significant difference in mortality (58% for
early necrosectomy vs. 27% for late). Although a major criticism
of this study is that all patients underwent necrosectomy (not just
those who were infected), it nonetheless instituted a new era of
delayed intervention.
In 2007, Besselink et al.34 reported a systematic review of
11 studies including 1136 patients correlated mortality rate with
timing of first intervention. The authors found a median time to
first surgical intervention of 26 days with a 25% mortality rate.
Most striking, the authors found a statistically significant inverse
association between mortality (decreasing) and time to surgical
intervention (delayed). Although the data only allowed an evaluation of intervention out to 30 days, there may have been a continued trend past this time point.34
It is postulated that the delay in intervention permits the acute
inflammatory process associated with the initial SIRS (Systemic
Inflammatory Response Syndrome) response to abate, the wall
of the collection to encapsulate and the tissues to demarcate making the surgery safer. Exceptions to the delayed approach should
be considered when patients manifest hemodynamic instability
despite aggressive resuscitation for 24 h or those patients for whom
an intra-abdominal catastrophe cannot be ruled. Otherwise, current International Association of Pancreatology guidelines advise
against operating in the acute phase of the disease.35
Answer: Delaying surgical intervention to at least 30 days
from onset of symptoms, when feasible, is likely associated with a
decreased risk of mortality. (Grade B recommendation).

Clinical Question Summary


Question

Answer

Level of
Evidence

Grade of
Recommendation

1 How should infectious


complications
associated with
pancreatitis be
described?

References

The new International Working Group


Consensus Classification system should be
used.

2 Should fine-needle
aspiration biopsy
(FNAB)be used?

Yes, in cases where imaging alone cannot


establish a diagnosis of infection.

3 Should enteral or
parenteral feeding
be used?

Enteral nutrition decreases the risk of


morbidity and mortality and should be the
preferred method of nutritional support.

1a

8-12

(Continued)

PMPH_CH54.indd 434

5/22/2012 5:38:02 PM

Infected Pancreatic Collections

435

(Continued)
Question

Answer

Level of
Evidence

Grade of
Recommendation

4 Should feeding begin


early or be delayed?

Supplemental tube feeding should begin early


in patients who have or are predicted to
have severe acute pancreatitis. Either
(semi)elemental or polymeric formulations
can be used.

1b

13-14

5 Should probiotics be
used?

No.

1b

15-17

6 Are prophylactic
antibiotics
recommended?

No.

1a

18-19

7 How should infected


pancreatic collections
be managed?

Drain placement subsequent minimally


invasive interventions (VARD) appears
to be an efficacious and safe approach to
the management of infected pancreatic
collections.

1b

23-28

8 What is the optimal


timing for surgical
intervention in
patients with infected
pancreatic necrosis?

Delaying surgical intervention to at least


30 days from symptom onset, when feasible,
is likely associated with a decrease in the
risk of mortality.

2a

33-34

REFERENCES
1. Fagenholz PJ, Fernandez-del Castillo C, Harris NS, Pelletier AJ,
Camargo CA, Jr. Direct medical costs of acute pancreatitis hospitalizations in the United States. Pancreas. 2007;35(4):302-307.
2. Shaheen NJ, Hansen RA, Morgan DR, et al. The burden of gastrointestinal and liver diseases, 2006. Am J Gastroenterol. 2006;
101(9):2128-2138.
3. Besselink MG, van Santvoort HC, Boermeester MA, et al. Timing and impact of infections in acute pancreatitis. Br J Surg. 2009;
96(3):267-273.
4. Banks PA, Freeman ML. Practice guidelines in acute pancreatitis. Am J Gastroenterol. 2006;101(10):2379-2400.
5. Bradley EL, 3rd. A clinically based classification system for acute
pancreatitis. Summary of the International Symposium on Acute
Pancreatitis, Atlanta, Ga, September 11 through 13, 1992. Arch
Surg. 1993;128(5):586-590.
6. Banks PA, Bollen T, Dervenis C, et al. Revision of the Atlanta
Classification of Acute Pancreatitis. Ongoing.
7. Rau B, Pralle U, Mayer JM, Beger HG. Role of ultrasonographically
guided fine-needle aspiration cytology in the diagnosis of infected
pancreatic necrosis. Br J Surg. 1998;85(2):179-184.
8. Marik PE, Zaloga GP. Meta-analysis of parenteral nutrition versus
enteral nutrition in patients with acute pancreatitis. BMJ. 2004;
328(7453):1407.
9. Petrov MS, van Santvoort HC, Besselink MG, et al. Enteral nutrition
and the risk of mortality and infectious complications in patients
with severe acute pancreatitis: A meta-analysis of randomized
trials. Arch Surg. 2008;143(11):1111-1117.
10. Al-Omran M, Albalawi ZH, Tashkandi MF, Al-Ansary LA. Enteral
versus parenteral nutrition for acute pancreatitis. Cochrane Database Systematic Reviews. 2010(1):CD002837.

PMPH_CH54.indd 435

References

11. Eatock FC, Chong P, Menezes N, et al. A randomized study of early


nasogastric versus nasojejunal feeding in severe acute pancreatitis.
Am J Gastroenterol. 2005;100(2):432-439.
12. Kumar A, Singh N, Prakash S, Saraya A, Joshi YK. Early enteral
nutrition in severe acute pancreatitis: A prospective randomized
controlled trial comparing nasojejunal and nasogastric routes.
J Clin Gastroenterol. 2006;40(5):431-434.
13. Petrov MS, Loveday BP, Pylypchuk RD, et al. Systematic review
and meta-analysis of enteral nutrition formulations in acute pancreatitis. Br J Surg. 2009;96(11):1243-1252.
14. Eckerwall GE, Tingstedt BB, Bergenzaun PE, Andersson RG.
Immediate oral feeding in patients with mild acute pancreatitis is
safe and may accelerate recoverya randomized clinical study.
Clin Nutr. 2007;26(6):758-763.
15. Besselink MG, Timmerman HM, Buskens E, et al. Probiotic
prophylaxis in patients with predicted severe acute pancreatitis
(PROPATRIA): Design and rationale of a double-blind, placebocontrolled randomised multicenter trial [ISRCTN38327949].
BMC Surg. 2004;4:12.
16. Besselink MG, van Santvoort HC, Buskens E, et al. Probiotic prophylaxis in predicted severe acute pancreatitis: A randomised,
double-blind, placebo-controlled trial. Lancet. 2008;371(9613):
651-659.
17. Besselink MG, van Santvoort HC, Renooij W, et al. Intestinal barrier dysfunction in a randomized trial of a specific probiotic composition in acute pancreatitis. Ann Surg. 2009;250(5):712-719.
18. Villatoro E, Mulla M, Larvin M. Antibiotic therapy for prophylaxis against infection of pancreatic necrosis in acute pancreatitis. Cochrane Database Systematic Reviews. 2010;(5):CD002941.
19. Wittau M, Mayer B, Scheele J, et al. Systematic review and metaanalysis of antibiotic prophylaxis in severe acute pancreatitis.
Scand J Gastroenterol. 2011;46(3):261-270.

5/22/2012 5:38:02 PM

436

Surgery: Evidence-Based Practice

20. Besselink MG, de Bruijn MT, Rutten JP, et al. Surgical intervention in patients with necrotizing pancreatitis. Br J Surg. 2006;
93(5):593-599.
21. Rau B, Bothe A, Beger HG. Surgical treatment of necrotizing pancreatitis by necrosectomy and closed lavage: Changing patient
characteristics and outcome in a 19-year, single-center series.
Surgery. 2005;138(1):28-39.
22. Rodriguez JR, Razo AO, Targarona J, et al. Debridement and
closed packing for sterile or infected necrotizing pancreatitis:
Insights into indications and outcomes in 167 patients. Ann Surg.
2008;247(2):294-299.
23. van Baal MC, van Santvoort HC, Bollen TL, et al. Systematic
review of percutaneous catheter drainage as primary treatment
for necrotizing pancreatitis. Br J Surg. 2011;98(1):18-27.
24. Horvath KD, Kao LS, Wherry KL, Pellegrini CA, Sinanan MN.
A technique for laparoscopic-assisted percutaneous drainage of
infected pancreatic necrosis and pancreatic abscess. Surg Endosc.
2001;15(10):1221-1225.
25. van Santvoort HC, Besselink MG, Horvath KD, et al. Videoscopic assisted retroperitoneal debridement in infected necrotizing pancreatitis. HPB (Oxford). 2007;9(2):156-159.
26. Horvath KD, Freeny P, Escallon J, et al. Safety and efficacy of
video-assisted retroperitoneal debridement (VARD) for infected
pancreatic collections: A multicenter, prospective, single-arm
phase II study. Arch Surg. 2010;145(9):817-825.
27. van Santvoort HC, Besselink MG, Bollen TL, et al. Case-matched
comparison of the retroperitoneal approach with laparotomy for
necrotizing pancreatitis. World J Surg. 2007;31(8):1635-1642.

PMPH_CH54.indd 436

28. van Santvoort HC, Besselink MG, Bakker OJ, et al. A step-up
approach or open necrosectomy for necrotizing pancreatitis.
N Engl J Med. 2010;362(16):1491-1502.
29. Raraty MG, Halloran CM, Dodd S, et al. Minimal access retroperitoneal pancreatic necrosectomy: Improvement in morbidity and mortality with a less invasive approach. Ann Surg. 2010;
251(5):787-793.
30. Papachristou GI, Takahashi N, Chahal P, Sarr MG, Baron TH. Peroral endoscopic drainage/debridement of walled-off pancreatic
necrosis. Ann Surg. 2007;245(6):943-951.
31. Seifert H, Biermer M, Schmitt W, et al. Transluminal endoscopic
necrosectomy after acute pancreatitis: A multicentre study with
long-term follow-up (the GEPARD Study). Gut. 2009;58(9):
1260-1266.
32. Voermans RP, Bruno MJ, van Berge Henegouwen MI, Fockens P.
Review article: Translumenal endoscopic debridement of organized pancreatic necrosisthe first step towards natural orifice
translumenal endoscopic surgery. Aliment Pharmacol Ther. 2007;
26(Suppl 2):233-239.
33. Mier J, Leon EL, Castillo A, Robledo F, Blanco R. Early versus
late necrosectomy in severe necrotizing pancreatitis. Am J Surg.
1997;173(2):71-75.
34. Besselink MG, Verwer TJ, Schoenmaeckers EJ, et al. Timing of
surgical intervention in necrotizing pancreatitis. Arch Surg. 2007;
142(12):1194-1201.
35. Uhl W, Warshaw A, Imrie C, et al. IAP guidelines for the surgical management of acute pancreatitis. Pancreatology. 2002;2(6):
565-573.

5/22/2012 5:38:02 PM

CHAPTER 55

Pancreatic Pseudocysts
Olga N. Tucker and Raul J. Rosenthal

INTRODUCTION

A classification system for acute pancreatitis was published in


1993 following the International Symposium on Acute Pancreatitis
in Atlanta, Georgia in September 1992.7,15 This was an attempt at
reaching an international agreement to create a uniform set of
accepted clinically based definitions for acute pancreatitis, and
associated local complications including pancreatic pseudocysts.
(Level 5 evidence).
Definitions of local complications of acute pancreatitis according to the Atlanta Criteria (Level 5 evidence)7 are as follows:

A pancreatic pseudocyst is a collection of pancreatic secretions


enclosed in a fibrous tissue layer in the absence of a lining epithelium.1 In the majority of cases, pancreatic pseudocysts arise as a
complication of acute or chronic pancreatitis. Pancreatic pseudocysts are caused by pancreatic ductal disruption due to pancreatic
parenchymal necrosis or injury, or following increased pancreatic
ductal pressure due to obstruction of the pancreatic ductal system
by calculi, protein plugs, stenosis or localized fibrosis usually complicating chronic pancreatitis leading to extravasation of pancreatic secretions.2-5 Pancreatic pseudocysts, therefore, communicate
either indirectly through the injured pancreatic parenchyma or
directly with the pancreatic ductal system.
There are several important issues to consider in the management of pancreatic pseudocysts including exclusion of other
causes of pancreatic cystic lesions by appropriate clinical and
radiological features, consideration of the optimal management
approach, determination of the optimal timing for intervention,
and awareness and management of potential complications.

Acute fluid collections (Figure 55.1): Occur early in the course


of acute pancreatitis, are located in or near the pancreas, and
always lack a wall of granulation of fibrous tissue. Spontaneous

1. What is the definition of acute fluid collections, pancreatic


necrosis, pancreatic abscess, and pancreatic pseudocyst?
Accurate definition of a pancreatic pseudocyst with differentiation
from other local complications of pancreatitis, including pancreatic and peripancreatic fluid collections, pancreatic necrosis, and
pancreatic abscess, is essential for optimal management and outcome. Expert opinion from a number of international conferences
and published guidelines from specialty groups have attempted
to clarify the terminology for acute pancreatitis and its complications, including the international conferences in Marseilles
(1963), Cambridge (1983), Marseilles (1984), Marseilles-Rome
(1988), and Atlanta (1992), and guidelines of the Intractable Pancreatic Disease Investigation and the Research Group of the Japanese Ministry of Health, and Welfare (1987), the British Society
of Gastroenterology 1998), the Japanese Working Group (2006),
and most recently the Japanese guidelines for the management of
acute pancreatitis Working Group (2010).6-14

Figure 55.1 CECT demonstrating an acute peripancreatic fluid


collection in a patient with acute pancreatitis.
437

PMPH_CH55.indd 437

5/22/2012 5:38:38 PM

438

Surgery: Evidence-Based Practice

Figure 55.2 CECT demonstrating pancreatic necrosis, acute


peripancreatic fluid collection and ascites in a patient with acute
pancreatitis.

regression occurs in approximately 50% of cases. In the other


50%, an acute fluid collection progresses to a pancreatic abscess
or pseudocyst.
Pancreatic necrosis (Figure 55.2): Diff use or focal area(s) of nonviable pancreatic parenchyma, typically associated with peripancreatic fat necrosis. Nonenhanced pancreatic parenchyma
>3-cm diameter or involving >30% of the area of the pancreas
is required.
Acute pancreatic pseudocyst (Figure 55.3): (1) A collection of
pancreatic juice enclosed by a wall of fibrous or granulation tissue, which arise as a consequence of acute pancreatitis, pancreatic
trauma, or chronic pancreatitis. (2) Their formation requires 4 or
more weeks from onset of pancreatitis. (3) Pseudocyst contents
should consist of clear pancreatic fluid with no pus or necrotic
debris. (4) Pseudocyst are round or ovoid and most often sterile. In the presence of pus the lesion is termed a pancreatic
abscess.
Pancreatic abscess: Circumscribed, intra-abdominal collection
of pus, usually in proximity to the pancreas, containing little or
no pancreatic necrosis, which arises as a consequence of acute
pancreatitis or pancreatic trauma. Usually arises >4 weeks after
onset of symptoms.

However, in 2008, a review by Bollen et al.16 on 447 articles retrieved using a Medline literature search on studies published after
1993 on the use of the Atlanta Criteria in the defi nition and
management of acute pancreatitis, including 3 meta-analysis,
34 randomized controlled trials, 12 guidelines, and 82 reviews,
suggested the existence of a large variation in the utilization
and interpretation of the Atlanta defi nitions of local complications including pancreatic pseudocysts. (Level 2 evidence)
Alternative or nonuniform definitions were frequency used.
The authors suggested the need for a revision of the criteria.16
In 38 reviewed articles, pseudocysts were defi ned as collections
containing both fluid and solid necrotic debris.16 According to
Bradley,17 the defi nition of a pancreatic pseudocyst should be an

PMPH_CH55.indd 438

Figure 55.3 CECT demonstrating a large pancreatic pseudocyst causing external compression of the stomach anteriorly.
encapsulated homogenous fluid collection without necrotic contents, and should be carefully distinguished from a peripancreatic
acute fluid collection to determine appropriate management. The
authors concluded that treatment outcome in many published
studies could not be interpreted with accuracy due to the inconsistencies in reporting techniques of distinctions between acute
fluid collections and pancreatic pseudocysts, and acute and
chronic pseudocysts.16
In 2010, a revised edition of the Japanese guidelines for the
management of acute pancreatitis was published. The publication
of revised guidelines so soon after the previous guidelines in 2006
was justified on the basis of a summary of activities and reports of
shared studies conducted in 2008. The revised guidelines included
a new severity classification and clinical indicators (pancreatitis
bundles) to improve the quality of management, and new terminology was proposed for pancreatitis and its complications.14
Definitions of local complications of acute pancreatitis according to the revised Japanese guidelines are as follows:14

Acute fluid collections: Exudate collection that often occurs


within the pancreas or in the parapancreatic tissue in the early
phase of the disease. It may progress as far as the anterior
paraphrenic cavity, the mesocolon, and beyond the inferior
renal portion. It is also characterized by lack of the fibrous wall.
Pleural fluid, ascites, and fluid collection as far as the cavity of the
omental bursa occur as a reaction against inflammation, so these
features are not defined as acute exudate collection.
Pancreatic necrosis: Diff use or localized necrosis of the pancreatic parenchyma and is differentiated from necrosis occurring
around the pancreas and that of extrapancreatic adipose tissue.
The detection of a poorly visualized area by contrast-enhanced
computed tomography (CECT) does not necessarily suggest the
presence of necrosis in all the cases involved and that detection
of an area that is not visualized, particularly in the acute phase,
may suggest the presence of temporary ischemia, which can be
reversible.
Acute pancreatic pseudocyst: The type of pseudocyst with a wall
structure of granulation or fibrotic tissue. It is accompanied

5/22/2012 5:38:38 PM

Pancreatic Pseudocysts

by the collection of pancreatic juice and the tissue of liquefaction necrosis, and often occurs 4 weeks after the onset of acute
pancreatitis. It may resolve spontaneously, although it may
persist for a long time. It may be complicated by infections or
bleeding.
Pancreatic abscess: An abscess accompanied by localized pus
collection in the pancreas and adjacent organs. However, there is
usually no necrosis within the pancreas, or there is only a small
amount if any. Because pancreatic abscess consists of necrotic
tissue as well as liquid components, there are indications that it
is induced by the liquefaction of tissue necrosis.

Answer: The Atlanta Classification system provides a definition for pancreatic pseudocyst and local complications of acute
pancreatitis. (Level 5 evidence; Grade D recommendation) The
revised Japanese guidelines provide further definitions of terminology associated with acute pancreatitis. However, neither of the
systems has been validated, or used uniformly or consistently, to
allow accurate comparison of management outcome between centers. (Level 2 evidence; Grade C recommendation).
2. What is the true incidence of pancreatic pseudocysts?
The reported incidence of pancreatic pseudocysts is low at 0.5 to
1 per 100,000 adults per year or 1.6% to 4.5%.18,19 However, the true
incidence of pancreatic pseudocysts is unknown due to inconsistencies in the application of a uniform definition, the timing of
diagnosis, differing techniques of clinical monitoring, the use
of varying diagnostic modalities, the use of varying clinical and
radiological severity grading systems, the complexity and variety
of the underlying pathology, and the wide use of multiple interventional techniques with poor reporting of treatment outcomes.
Incidence rates of pseudocyst formation after acute pancreatitis,
trauma, iatrogenic injury, and autoimmune pancreatitis have been
reported in case reports, multiple case series, and review articles.
(Level 4 evidence).
Many reported case series on the incidence and management
of pseudocysts are limited by population heterogeneity, small
patient numbers, and mixed data on patients with mild acute
nonnecrotizing and severe acute pancreatitis, and/or the inclusion of patients with varying etiology, including acute and chronic
pancreatitis. Mild acute and severe acute pancreatitis represent
contrasting ends of a wide spectrum of disease severity with significant differences in complication and survival rates. In addition,
there are many cases of severe acute pancreatitis, which are not
found until autopsy.18,19 The incidence of pancreatic pseudocysts
in patients with chronic pancreatitis is probably grossly underestimated as long-term follow-up is poor. In contrast to patients with
acute pancreatitis, those with chronic pancreatitis have a greater
potential to develop local complications over the prolonged period
of their illness.
The majority of acute fluid collections complicating acute
nonnecrotizing pancreatitis will resolve spontaneously with
pseudocyst formation in a minority.20 The incidence of acute
pancreatic pseudocysts is higher after severe acute pancreatitis,
with higher morbidity and mortality rates related to a higher
incidence of complications.21 The use of inaccurate and imprecise defi nitions of acute pancreatic pseudocysts has resulted in
inaccurate representation of data. One of the most common difficulties is the differentiation of organized pancreatic and peripancreatic necrosis with associated fluid sequestration from an

PMPH_CH55.indd 439

439

acute pancreatic pseudocyst with pancreatic necrosis.22 These


clinical entities are very different in terms of treatment approach
and prognosis.
Answer: The true incidence of pancreatic pseudocyst is
unknown, due to the heterogeneity of published reports and inconsistencies in the published literature. (Level 4 evidence; Grade C
recommendation).
3. Does the etiology of pancreatitis influence the likelihood of
pancreatic pseudocyst formation?
Pancreatic pseudocyst formation is more commonly seen in
patients with chronic pancreatitis compared with acute pancreatitis with a reported incidence of 30% to 40% and is more
common after alcohol-induced than nonalcohol-related pancreatitis.4,5,18,19,23-25 Alcohol-related pancreatitis is the major cause
documented in most series and is responsible for a reported 59%
to 78% of all pseudocysts.4,18,23 Patients with chronic pancreatitis who develop acute exacerbations have a higher incidence of
pseudocyst formation than patients with acute pancreatitis,
whereas patients with gallstone-related acute pancreatitis appear
to have a lower incidence. (Level 4 evidence). However, pancreatic
pseudocysts can occur with any condition that results in pancreatic ductal disruption or injury, or following increased pancreatic ductal pressure due to obstruction of the pancreatic ductal
system.26 In the case of acute pancreatitis, approximately 20% of
patients develop a severe form with local and systemic complications. Pancreatic enzymatic activation leads to the formation of
pancreatic and peripancreatic fluid collections. These fluid collections represent an exudative or serous reaction to pancreatic
injury and occur in approximately 50% of patients with moderate to severe pancreatitis. Approximately 50% of these collections spontaneously resolve within 6 weeks of acute presentation.
In approximately 5% to 15% a profound inflammatory response
along the serosal surfaces of the adjacent organs results in the formation of a fibrous pseudocapsule and a pseudocyst develops.4,25,27
Approximately 20% to 40% of pancreatic pseudocysts develop in
patients with chronic pancreatitis related to chronic and progressive ductal obstruction, dilation, and disruption.4,25 Other
causes of pancreatic pseudocyst formation include blunt trauma,
penetrating trauma, iatrogenic surgical pancreatic injury, pancreatic ductal adenocarcinoma, very rarely hemorrhagic pancreatic pseudocysts associated with autoimmune pancreatitis, and
idiopathic.28-32
Answer: Pancreatic pseudocyst formation is more commonly
seen in patients with chronic pancreatitis compared with acute
pancreatitis, and is more common after alcohol-induced than
nonalcohol-related pancreatitis. However, pancreatic pseudocysts
can occur with any condition that results in pancreatic ductal disruption or injury, or following increased pancreatic ductal pressure due to obstruction of the pancreatic ductal system. (Level 4
evidence; Grade C recommendation).
4. Can an increased risk of pancreatic pseudocyst formation be
predicted at admission in patients presenting with pancreatitis?
A small number of published retrospective and prospective cohort
studies and case series have suggested that the presence of ascites,
pleural eff usion, a high Ranson score, and pancreatic necrosis
increases the risk of acute pancreatic pseudocyst formation.33-35 In
a retrospective cohort study, Diculescu et al.36 examined predictive

5/22/2012 5:38:38 PM

440

Surgery: Evidence-Based Practice

factors for pancreatic pseudocyst formation in 62 patients admitted over a 1-year period with acute pancreatitis.36 A predominance of alcoholic acute pancreatic was seen in 58.1%, followed
by biliary (22.6%), hypertriglyceridemia (8.1%), following endoscopic retrograde cholangiopancreatography (ERCP) (3.2%) and
unknown etiology (8%). Splenic or portal vein thrombosis was
present in 5%, 30.6% had surgical intervention, and two patients
(3.2%) died. Twenty-two patients (35.5%) developed pancreatic
pseudocysts, which were multiple in 12 patients. Alcoholic etiology of acute pancreatitis was the only categorical variable in
univariate analysis significantly associated with pseudocyst presence. Multivariate analysis demonstrated alcoholic etiology and
lower values of serum alkaline phosphatase predicted the occurrence of pseudocysts.36 Acute pseudocyst formation could be predicted with a specificity of >90% if serum alkaline phosphatase
was less than a cutoff value of two times the upper normal limit
(UNL = 185 U/L). Lower levels of serum alkaline phosphatase
were seen as a risk factor for pseudocyst formation in patients
with nonalcoholic etiology. However, 48.4% of the patients had
concomitant chronic liver disease with either fatty liver disease or
cirrhosis on imaging.36
In a recent publication, Poornachandra et al.37 prospectively analyzed clinical, biochemical, and radiologic parameters
at admission in 65 of 75 recruited patients with acute pancreatitis. Twenty-four (36.9%) patients had alcohol-related, whereas
18 (27.7%) had gallstone-related acute pancreatitis. CECT demonstrated pancreatic necrosis in 38 (58.46%) patients, and acute
fluid collections in 34 (52.3%). Thirty-four (52.3%) patients developed an acute pancreatic pseudocyst, with multiple pseudocysts
in the majority (52.9%) after 4 weeks of follow-up. Identified
factors associated with pseudocyst formation after acute pancreatitis on univariate analysis included male sex, a palpable
mass, blood sugar >150 mg%, presence of necrosis, acute fluid collection, ascites, pleural eff usion, a high grade of pancreatitis, sepsis, elevated C-reactive protein, acute fluid collection at 2 weeks,
and a high CT severity index (CTSI) score. The etiology of the
acute pancreatitis was not significantly associated with pseudocyst development. On multivariate logistic regression analysis, it
was shown that male sex, a palpable abdominal mass, ascites, and
a high CTSI score were significantly associated with formation the
an acute pseudocyst.37
Answer: There is limited published data on the factors at
admission that predict the development of a pseudocyst following acute pancreatitis. Male sex, a palpable abdominal mass,
ascites, and a high CTSI score on admission may predict pseudocyst development in acute pancreatitis. (Level 2 evidence; Grade C
recommendation).
5. What is the risk of nonoperative expectant management of a
pancreatic pseudocyst?
Current evidence supports intervention in patients who are symptomatic or who develop a pseudocyst-related complication (Levels
3 and 4 evidence). Controversies exist with regard to the need for
intervention in those pseudocysts that are asymptomatic. In these
patients, factors such as the diameter, location, persistence of the
pseudocyst during follow-up and the length of time the pseudocyst has been present have been used as indicators to determine
the need for intervention because of a potential higher risk of
complications. Some authors advocate elective intervention in
all patients with uncomplicated acute pancreatic pseudocysts

PMPH_CH55.indd 440

greater than 6 cm in size, which persist for longer than 6 weeks


regardless of symptoms due to reduction in the possibility of
spontaneous resolution and a reported increase in complications
(rupture, abscess, jaundice, and hemorrhage) during extended
periods of observation.21,38,39 Complications can arise in the presence of a pancreatic pseudocyst including pseudocyst infection with
abscess formation, intracystic hemorrhage, rapid expansion with
increasing abdominal pain, extrinsic compression, and obstruction
of adjacent organs including the esophagus, stomach, duodenum,
jejunum, colon, biliary tree, or retroperitoneal structures.21,40,41 If
the pseudocyst extends into the mediastinum, patients may present
with dyspnea, chest pain, palpitations, dysphagia, hemoptysis, acute
respiratory compromise, or cardiogenic shock depending on the
location and size of the lesion. Cyst rupture can occur into an adjacent hollow viscus such as the stomach, duodenum, or colon, or an
adjacent body cavity.21,41,46 Rupture into the peritoneal cavity results
in pancreatic ascites, while rupture into the pleural space results in
a pleural effusion or haemothorax.42-44 In addition, rupture into the
pericardium, or bronchus with fistula formation can occur.45
Abdominal wall pseudocyst fluid collection have been
reported in isolated case series due to a slow leak or rupture of
a pancreatic pseudocyst.47 Pseudocysts may erode into an adjacent major artery, more commonly the splenic artery, resulting
in a pseudoaneurysm and/or hemorrhage.48 A massive gastrointestinal bleed can occur if the pseudoaneurysm communicates
with the main pancreatic duct, a condition known as hemosuccus pancreaticus. Portal and splenic vein thrombosis have been
reported in patients with pancreatic pseudocysts with a persistent
inflammatory response with development of gastric varices. Rarer
complications including pancreatic pseudocyst-inferior vena cava
fistula, pancreaticorenal fistula, pancreaticocholedochal fistula,
gastric necrosis, and perforation have been described.49-52
In a series by Vitas et al.53 of 114 patients with the diagnosis
of a pancreatic pseudocyst followed up over a period of 5 years, 46
patients had primary surgical intervention, with 13% requiring
emergency operative treatment for pseudocyst-related complications. The remaining 68 patients were managed expectantly.
Of these, severe life-threatening complications developed only
in six patients (9%) over a mean period of 46 months. Nineteen
patients required elective surgery over a 5-year period related to
the pseudocyst or pancreatitis-associated complications. Of 24
patients treated conservatively with satisfactory radiological followup, spontaneous resolution was seen in 57%, with 38% resolving
more than 6 months following initial diagnosis.53 Operative intervention was more common in large pancreatic pseudocysts >6.9-cm
diameter; however, no serious complications occurred in seven
patients with pseudocysts >10-cm diameter treated expectantly.53
Cooperman et al.54 advocated expectant management of asymptomatic pseudocysts due to the natural history of spontaneous
resolution. Yeo et al.46 also support a conservative approach in
asymptomatic patients able to tolerate oral intake, with a reported
spontaneous resolution rate of 60% at 1 year with stability or size
reduction in 40% treated nonoperatively in the absence of pseudocyst-related mortality. Again, large pseudocyst size predicted the
need for surgical intervention, with operative drainage required
in 67% of those greater than 6-cm diameter, whereas only 40% less
than 6-cm diameter required operative intervention.46
Answer: Nonoperative expectant management of asymptomatic pseudocysts is feasible due to the natural history of spontaneous resolution and the low incidence of severe life-threatening
complications. (Level 4 evidence; Grade C recommendation).

5/22/2012 5:38:38 PM

Pancreatic Pseudocysts

6. What is the incidence of complicated pancreatic pseudocysts?


Although the occurrence of complications is uncommon, no accurate figures are available from the published literature on the true
incidence of morbidity associated with pancreatic pseudocysts.
Available data has been extracted from multiple case series, case
reports, and review articles.18,21,38,46,47,49-52,55-60 (Level 4 evidence).
The incidence of complicated pseudocysts is higher in patients
following severe acute pancreatitis, as the majority of acute fluid
collections in patients with mild acute nonnecrotizing pancreatitis resolve without pseudocyst formation.20 Ocampo et al.,21
reported complications in 43 (59%) of 73 patients over a 10-year
period with an acute pancreatic pseudocyst following severe acute
pancreatitis including infection in 74%, perforation in 21%, and
bleeding in 4.6%.
Answer: Complications of pancreatic pseudocysts are uncommon; however, no accurate figures of the true incidence are
available in the published literature. (Level 4 evidence; Grade C
recommendation).
7. What is the optimal time for intervention once the diagnosis
of pancreatic pseudocyst has been confirmed?
Once identified, the timing of intervention for pancreatic pseudocysts remains controversial.18-20,38,39,46,53,54,61,62 Experimental studies
by Warren et al.63 suggest a minimum period of 6 weeks to allow
cyst wall maturation. As it is not always possible to date the onset
of pseudocyst formation a wait period of 6 weeks from the time
of diagnosis has been recommended.38,64 Intervention is indicated
in the presence of symptoms directly attributable to the pancreatic pseudocyst and in the presence of pseudocyst-related complications. Others advocate a nonoperative, noninterventional
approach in selected patients.46,53,54
Warshaw et al.39 defi ned clinical and biochemical criteria in
a series of 42 patients, of whom 28 had underlying chronic pancreatitis, to guide the time of optimal drainage in patients with
pancreatic pseudocyst. They observed differences in the natural
history and treatment requirements dictated by etiology. Spontaneous resolution occurred in only three patients following antecedent acute pancreatitis, although it was not seen in any patient
with chronic pancreatitis. They suggested that a pseudocyst is
unlikely to resolve when persistent for greater than 6 weeks, in the
presence of chronic pancreatitis, a thick cyst wall on ultrasound,
and a pancreatic duct abnormality other than communication
with the pseudocyst.39 In the setting of chronic pancreatitis, the
authors concluded that internal drainage procedures should be
performed at the time of diagnosis to avoid unnecessary additional expense and potential increased complications. 39 Serum
levels of old amylase may help guide the optimal drainage time
indicating a mature pseudocyst.39 In a small series of study of
37 patients with chronic pancreatitis and pancreatic pseudocysts by Talar-Wojnarowska et al.65 spontaneous regression was
observed in seven pseudocysts (18.9%). The mean size of the
pancreatic pseudocysts in the series was 7.8 cm (range 216 cm).
Stable lesions in terms of diameter and absence of symptoms were
reported in nine patients (24.3%). Therapeutic intervention was
required in 21 patients (56.8%) involving percutaneous drainage
in 10.8%, endoscopic drainage in 27.1%, and surgical intervention in 18.9%. The overall recurrence rate in the series was high
at 33.3%. Again the size of the primary pseudocyst was a statistically significant factor determining outcome where the mean
size for those patients requiring interventional management was

PMPH_CH55.indd 441

441

9.6 cm compared with 4.2cm for patients undergoing successful


conservative management.65
Answer: There are no published randomized controlled
trials in the literature that defi ne the optimal time of intervention for pancreatic pseudocysts. Evidence from highly selected
multiple case series, case reports, and review articles support an
expectant approach in patients with asymptomatic pseudocysts
following acute pancreatitis regardless of size for a minimum
of 6 weeks after diagnosis. In the setting of chronic pancreatitis, immediate intervention to drain the cyst is feasible and may
reduce postoperative complications. (Level 3 evidence; Grade C
recommendation).
8. What are the optimal imaging modalities for diagnosis of a
pancreatic pseudocyst?
A variety of radiological techniques are utilized in diagnosis,
monitoring, and planning of therapeutic intervention for pancreatic pseudocysts, including transabdominal ultrasonography
(USS), CECT, magnetic resonance imaging (MRI), and magnetic
resonance cholangiopancreatography (MRCP); combined radiological and endoscopic modalities include ERCP and endoscopic
USS (EUS). Prospective data from randomized controlled trials
and large patient series comparing currently available imaging
modalities is lacking. CECT is the preferred and most commonly utilized modality to facilitate the accurate diagnose,
define extent of disease, and plan percutaneous intervention if
appropriate.66,67 Balthazars CTSI, based on combined assessments of peripancreatic inflammatory collections and degree of
pancreatic necrosis, can be used to predict morbidity and mortality in patients with severe acute pancreatitis.68 However, the CT
appearances cannot characterize the local complications of acute
pancreatitis, and in the acute phase cannot predict the development
or extent of pseudocyst formation. Controversies exist regarding
interobserver variability in interpretation of CECT, and the varying definitions used to define acute peripancreatic fluid collections
including pancreatic pseudocysts. A recent study performed to
assess the interobserver agreement of categorizing peripancreatic
collections on CECT using the Atlanta Classification in patients
with acute necrotizing pancreatitis, who underwent surgery from
2000 to 2003, involving five radiologists from 11 hospitals demonstrated poor concordance despite the radiologists awareness
of the clinical condition of the patient and the timing of the scan.
All five radiologists agreed in only 4% of 70 cases, four of the five
agreed in 19%, and three agreed in 60% using terminology defined
by the Atlanta Criteria to define CECT findings.69 In most of the
published series, the differentiation between an acute fluid collection and a pseudocyst was determined 4 weeks from onset of
disease; however, different time periods have been described from
3 to 8 weeks.35,70-72 In further publications, pseudocysts have been
defined as collections containing fluid and necrotic debris.73-75 As
previously stated pseudocysts should be devoid of solid necrotic
debris. Controversy also exists in correct differentiating pseudocysts and pancreatic abscesses as CECT has a low sensitivity in the
detection of necrotic debris in collections predominantly containing
fluid, and poor discriminatory ability between sterile and infected
collections.22,66,76-78 Misinterpretation of CECT findings may result
in instrumentation of sterile collections causing infection, or a delay
in appropriate intervention. MRI and EUS can more accurately
detect the presence of necrotic debris, and may be of additional
benefit in guiding appropriate intervention (Figure 55.4).66,76,79

5/22/2012 5:38:38 PM

442

Surgery: Evidence-Based Practice

Figure 55.4 MRCP demonstrating a large pancreatic pseudocyst with no communication with the pancreatic ductal system
and normal biliary tree.
Demonstration of communication with the pancreatic ductal
system at ERCP is thought to differentiate a pancreatic cystic neoplasm from a pseudocyst. However, pitfalls can occur as reported
by Ellis et al.,80 who reported two cases of mucinous cystadenomas
with main pancreatic duct communication. Upper gastrointestinal
endoscopy can be performed to plan endoscopic or surgical drainage (Figure 55.5). EUS is an excellent diagnostic and therapeutic
tool due to its high resolution and the proximity of the transducer
to the pancreas.81,82 However, it has limited efficacy in diagnosis and management of lesions in the distal body and tail of the
pancreas. Additional limitations include the operator-dependent
nature of the investigation. Radial and linear EUS, elastography,
contrast-enhanced EUS, and fine-needle aspiration (FNA-EUS)
can be performed, where diagnostic dilemmas exist to differentiate a focal pancreatic mass.81
Answer: There are no published randomized controlled trials in the literature to directly compare currently available imaging modalities in the diagnosis and management of pancreatic
pseudocysts. Evidence from multiple case series, case reports,
reviews, and nonrandomized trials support CECT as the imaging
modality of choice. Prior to anticipated intervention, an MRI scan
or an EUS should be performed to exclude necrotic debris in the
collection. (Level 3 evidence; Grade C recommendation).
9. What is the optimal method of therapeutic intervention for
symptomatic pancreatic pseudocysts?
Indications for intervention include symptomatic, large (>6-cm
diameter), enlarging, and complicated pseudocysts, and where
there is a suspicion of an underlying malignancy.25,38,39,62 Therapeutic intervention takes the form of internal or external drainage, or surgical resection. Options include percutaneous external

PMPH_CH55.indd 442

Figure 55.5 Upper gastrointestinal endoscopy demonstrating


protrusion of a large pancreatic pseudocyst into the posterior
wall of the stomach.
drainage, ERCP with transpapillary pancreatic duct stenting,
endoscopic internal drainage, laparoscopic, laparoscopic-assisted,
or open surgical internal drainage and/or resection.74,83-90 To
date, no prospective randomized trials have directly compared
percutaneous, endoscopic, and surgical approaches. The management is therefore usually based on the existing local or regional
expertise. Clinical decisions are based on available clinical evidence from retrospective cohort studies, case series, and isolated case reports. (Level 2 evidence) Factors that determine the
approach and timing of intervention include etiology, maturity of
the cyst wall, cyst location, the presence or absence of complications, and the availability of local expertise.38,39,63,64,85 The optimal
approach and timing of intervention should be determined in a
multidisciplinary setting incorporating the experience of radiologists, gastroenterologists, and surgeons.
Percutaneous external drainage can be achieved using a
transabdominal USS or CT-guided approach. A drainage pigtail
catheter is placed percutaneously under local anaesthetic into the
fluid cavity of the pseudocyst and left in situ until confirmation of
minimal output and/or cavity resolution. Contrast material can be
injected into the cyst cavity to confirm resolution. This approach
is associated with a low risk of mortality, but a complication rate
of 16% in some series with a risk of hemorrhage in 1% to 2%,
infection in 9%, traversing of viscera/pleural space in 1% to
2%, and pancreaticocutaneous fistula formation. Percutaneous
external drainage is contraindicated in patients with cysts containing solid or bloody material, and in patients with an obstructed
ductal system.84,91 Three-dimensional USS and color Doppler can
be used to visualize structures such as blood vessels, small bowel
and colon, and guide the accurate placement of the catheter. Percutaneous drainage is generally performed under CT guidance,

5/22/2012 5:38:39 PM

Pancreatic Pseudocysts

with successful outcomes reported in the diagnosis and drainage


of sepsis in infected pancreatic pseudocysts, and in patients with
symptomatic or complicated pseudocysts that are too unwell to
undergo a more definitive procedure.21,87 In a series by Ocampo
et al.,21 CT-guided percutaneous and endoscopic drainage were
successful in controlling sepsis in 11 of 13 patients (85%) with
severe organ failure and facilitated subsequent definitive surgical
management.21,87
Open surgical drainage as an initial therapeutic option has
been largely replaced by minimally invasive techniques including endoscopic and laparoscopic approaches.83,85,92-98 Endoscopic
drainage can be performed transmurally through the wall of
the stomach or duodenum, or transpapillary via the pancreatic
duct.74,59,99,100 Transpapillary drainage is performed when the
pancreatic pseudocyst is demonstrated to communicate with
the main pancreatic duct at ERCP, or in the presence of a distal
pancreatic duct stricture. In some reported series endoscopic
transmural drainage has been associated with a higher success
rate, with lower morbidity, mortality, and recurrence rates than
surgery.73,99 Cavallini et al.100 recently reported a retrospectively
analyzed case series of 55 patients who underwent endoscopic
drainage of symptomatic pancreatic pseudocysts with proven
direct contact between the pseudocyst itself and the gastric wall
following pancreatitis or pancreatic resection from January
1999 to June 2008.100 In this study, a postprocedural CTET was
performed at 1 week, and an USS or CECT at 1 and 6 months,
with an annual USS thereafter. Of the 55 patients studied, 28
had a pseudocyst complicating acute and two complicating
chronic pancreatitis. The overall technical success rate was
73.3% in the patients with pseudocyst following pancreatitis
compared with 84% in those following pancreatic resection. The
procedure complication rate was much higher in the pancreatitis group at 23.3% compared with 8% in the resection group.
Six patients in the pancreatitis group developed a pseudocyst
infection and underwent surgical intervention within 2 weeks.
Two deaths occurred due to major hemorrhage with a mortality rate 1.8%. Pseudocyst recurrence was seen in 6 of 43 treated
patients (13.9%) all of whom were in the pancreatitis group. Th is
study demonstrated the high technical success rate of transmural endoscopic drainage, but a high rate of procedure-related
complications and recurrences particularly in patients with
pancreatitis-associated pseudocysts.100 Baron et al.101 previously
reported a higher resolution rate in with chronic (92%) than
acute pseudocysts (74%) or necrosis (72%) with endoscopic
drainage, with a higher rate of complications in patients with
necrosis (37%) than chronic (17%) or acute pseudocysts (19%).
Recurrence rates at a median follow-up of 2.1 years were 16%,
and were more commonly seen in patients with necrosis (29%)
than acute (9%) or chronic pseudocysts (12%).These fi ndings suggest outcome differences following endoscopic drainage dependent on the nature of the pseudocyst.101 The role of concomitant
transpapillary duct stenting at endoscopic drainage is unclear
and benefit has only been demonstrated in one study.102
The use of EUS-guided pseudocyst drainage with improved
visualization of the contact area between the pseudocyst and
the gastric wall on EUS and delineation of anatomy with avoidance of injury to major vessels or bowel has been advocated over
conventional endoscopy Two small randomized controlled trials demonstrated a significantly higher success rate in patients

PMPH_CH55.indd 443

443

randomized to EUS.93,103 In a study by Varadarajulu et al.103 100%


of patients randomized to EUS-guided drainage underwent successful drainage compared with 33.3% randomized to drainage
guided by esophagogastroduodenoscopy (EGD). Based on astreated analysis, treatment was successful in 23 patients managed by EUS and in four patients treated by EGD (80%) with
no significant difference in rates of treatment success between
EUS and EGD after stenting. Although not statistically significant, EUS was associated with less complications than EGD.103
Park et al., randomized 60 consecutive patients with pancreatic
pseudocysts to undergo either EUS- (n = 31) or EGD-guided
(n = 29) drainage of pancreatic pseudocysts.93 The technical success rate was higher for EUS (94%) than EGD (72%). Complications occurred in 7% of the EUS compared with 10% of the EGD
group. Long-term follow-up showed no significant difference in
clinical outcomes between EUS and EGD.103 The major limitations in these studies is small patient numbers with inclusion of
only 29 patients and 60 patients, respectively.93,103
Laparoscopic techniques include endogastric, transgastric,
or exogastric cystgastrostomy, Roux en Y or loop cystjejunostomy.83 A review of the published literature on laparoscopic
and endoscopic approaches to internal drainage of pancreatic
pseudocysts from 1974 to 2005 was published by Aljarabah
et al.83 Forty-four cohort series were identified. No randomized control or comparative studies were identified. Laparoscopic procedures were performed in 118 patients in 19 reports,
and endoscopic procedures in 583 patients in 25 reports. The
reporting of data related to the underlying etiology, patient
demographic data, pseudocyst size, duration of the procedure,
procedural complications including estimated blood loss, and
hospital stay was better in the laparoscopic group. The endoscopic approach was more widely employed with fivefold
greater number of reported patients. The mean cyst diameter
was significantly smaller in the endoscopic group with a mean
cyst diameter of 7 cm compared with 13 cm in the laparoscopic
group. In three reports, pancreatic pseudocysts as small as 1.5- to
2-cm diameter were drained endoscopically.104-106 The success rate
in achieving pseudocyst drainage and resolution was higher
after the laparoscopic (98.3%) compared with the endoscopic
(80.8%) approach. Postprocedural complications were observed
in 4.2% of patients after laparoscopic versus 12% after endoscopic drainage. Two patients died after endoscopic drainage
resulting in a mortality rate of 0.35%, with no deaths after laparoscopic drainage. The mean follow-up period was longer at
24 months (range 0.570) after endoscopic than laparoscopic
drainage at 13 months (range 159) with reported recurrence in
14.4% and 2.5%, respectively.83
Answer: The optimal approach to pancreatic pseudocyst
drainage remains controversial. Minimally invasive internal
drainage techniques by endoscopic and laparoscopic approaches
are commonly employed. These two approaches are safe with
minimal morbidity and mortality. Although laparoscopic drainage has a higher success rate in achieving pseudocyst drainage
and resolution, a lower postprocedural complication rate, and a
lower recurrence rate, reported follow-up periods are significantly
shorter. The heterogeneity of the published reports and the lack of
consistency in the reporting of data limit direct comparison between the two techniques. Only two small randomized controlled
trails have been performed comparing EUS- with EGD-guided

5/22/2012 5:38:39 PM

444

Surgery: Evidence-Based Practice

drainage of pancreatic pseudocysts. (Level 2 evidence; Grade C


recommendation).
10. Do delays in surgical intervention affect outcome?
Initial interventions in the management of pancreatic pseudocysts
are increasingly directed toward nonsurgical therapies including percutaneous external drainage or endoscopic approaches
due to the perceived benefits of reduced invasiveness, and lower
morbidity and mortality rates. However, these techniques can be
associated with significant failure rates and complications.75,107,108
Subsequent surgical intervention is often required as a salvage
procedure to treat persistent or recurrent pseudocysts, or complications such as infection following percutaneous drainage.75,107,109
Some authors have suggested that primary nonoperative intervention with delayed surgery is associated with a higher incidence of postoperative complications, readmission, morbidity,
and mortality.107,110 Rao et al.107 retrospectively reviewed outcome
in 52 patients who underwent early surgical intervention compared with 18 who underwent delayed surgery after failed CT and
endoscopic drainage. Perioperative morbidity was twice as frequent in the delayed surgery group (33% vs. 14%), with increased
time to pancreatic pseudocyst resolution from the initial drainage attempt.107 In a study by Ito et al.,110 284 consecutive patients
admitted with pancreatic pseudocysts over a 15.5-year period

were identified retrospectively, of which 46 underwent initial


operative intervention.110 Percutaneous drainage was performed
in 89 patients of whom 42 required subsequent surgical intervention for failure, while endoscopic drainage was performed in 73
patients of whom 33 required subsequent surgical intervention
for failure. There was no significant difference in patient demographics, etiology of pancreatitis, location, number, and diameter of pseudocysts, or morphology of the main pancreatic duct
in patients treated with initial surgery versus those undergoing
delayed surgery. However, the median time from diagnosis to
surgery was three times longer in the delayed surgery group. The
main indication for intervention in the delayed group was pseudocyst infection in 43% versus 13% in the early group. The delayed
surgery group had a significantly higher incidence of postoperative pancreatic complications, infectious complications, perioperative morbidity, and readmission rates. Five patients died in the
postoperative period, of whom two died secondary to sepsis and
three due to organ failure. On univariate analysis failure of nonsurgical intervention was associated with pseudocyst diameter
>6 cm, main pancreatic duct stricture >2, nonsurgical interventional procedures, and pseudocyst infection.110
Answer: Surgical intervention after failed nonoperative drainage procedures is associated with higher incidences of postoperative infection, pancreatic complications, morbidity, mortality, and
readmission rates. (Level 3 evidence; Grade C recommendation).

Clinical Question Summary


Question

Answer

1 What is the definition of acute fluid


collections, pancreatic necrosis,
pancreatic abscess, and pancreatic
pseudocyst?

There is a lack of uniformity and consistency


in the use of terminology to define
pancreatitis-related local complications

7, 14

2 What is the incidence of pancreatic


pseudocysts?

The true incidence of pancreatic pseudocyst


is unknown

4, 20-22

3 Does the etiology of pancreatitis influence


the likelihood of pancreatic pseudocyst
formation?

Pancreatic pseudocysts can occur with


any condition that causes pancreatic
ductal disruption or injury, or following
increased pancreatic ductal pressure

4, 26

4 Can an increased risk of pancreatic


pseudocyst formation be predicted at
admission?

Male sex, palpable mass, ascites and high


CTSI score on admission may predict
pseudocyst development

36, 37

5 What is the risk of nonoperative


management?

Nonoperative expectant management of


asymptomatic pseudocysts is feasible

46, 53, 54

6 What is the incidence of complicated


pancreatic pseudocysts?

The true incidence of complicated pancreatic


pseudocyst is unknown

21, 55

7 What is the optimal time for intervention


for pancreatic pseudocyst?

The optimal time for intervention is


unknown

39, 65

8 What are the optimal imaging modalities


for diagnosis of a pancreatic pseudocyst?

CECT is the imaging modality of choice

68, 111

9 What is the optimal method of therapeutic


intervention?

The optimal approach to pancreatic


pseudocyst drainage remains controversial

83

10 Do delays in surgical intervention affect


outcome?

Yes

107, 110

PMPH_CH55.indd 444

Grade of
Recommendation

References

5/22/2012 5:38:39 PM

Pancreatic Pseudocysts

445

Levels of Evidence
Subject

Year

References

Level of
Evidence

Strength of
Recommendation

1 What is the definition of


acute fluid collections,
pancreatic necrosis,
pancreatic abscess, and
pancreatic pseudocyst?

1993
2010

2 What is the incidence of


pancreatic pseudocysts?

7, 14

Widespread adoption of an
uniform terminology for
local complications of acute
pancreatitis is required

1999
2007
2008

4, 20-22

The true incidence is unknown

3 Does the etiology of


pancreatitis influence the
likelihood of pancreatic
pseudocyst formation?

1988
1990

4, 26

Pancreatic pseudocysts can be


caused by multiple pathologies

4 Can an increased risk of


pancreatic pseudocyst
formation be predicted
at admission?

2005
2010

36, 37

Male sex, palpable mass, ascites and


high CTSI score on admission
may predict pseudocyst
development

46, 53, 54

Expectant management of
asymptomatic pseudocysts is
feasible

5 What is the risk


of nonoperative
management?

Findings

6 What is the incidence of


complicated pancreatic
pseudocysts?

2001
2007

21, 55

The true incidence is unknown

7 What is the optimal


time for intervention of
pancreatic pseudocyst?

1985
2011

39, 65

Timing of intervention is determined


by etiology, symptoms, and
complications

8 What are the optimal


imaging modalities for
diagnosis of a pancreatic
pseudocyst?

1990
2010

68, 111

CECT is the imaging modality of


choice. Prior to intervention, an
MRI or EUS should be performed
to exclude necrotic debris

9 What is the optimal


method of therapeutic
intervention?

2007

83

Minimally invasive internal drainage


by endoscopic and laparoscopic
approaches are safe

10 Do delays in surgical
intervention affect
outcome?

1993
2007

107, 110

Surgical intervention after failed


nonoperative drainage is
associated with a worse outcome

REFERENCES
1. Lehman GA. Pseudocysts. Gastrointest Endosc. 1999;49(3 Pt 2):
S81-S84.
2. Bradley EL, Gonzalez AC, Clements JL, Jr. Acute pancreatic
pseudocysts: Incidence and implications. Ann Surg. 1976;184(6):
734-737.
3. Gumaste VV, Pitchumoni CS. Pancreatic pseudocyst. Gastroenterologist. 1996;4(1):33-43.
4. Imrie CW, Buist LJ, Shearer MG. Importance of cause in the
outcome of pancreatic pseudocysts. Am J Surg. 1988;156(3 Pt 1):
159-162.
5. Kloppel G. Pseudocysts and other non-neoplastic cysts of the
pancreas. Semin Diagn Pathol. 2000;17(1):7-15.
6. United Kingdom guidelines for the management of acute pancreatitis. British Society of Gastroenterology. Gut. 1998;42(Suppl 2):
S1-S13.

PMPH_CH55.indd 445

7. Bradley EL, III. A clinically based classification system for acute


pancreatitis. Summary of the International Symposium on Acute
Pancreatitis, Atlanta, GA, September 11 through 13, 1992. Arch
Surg. 1993;128(5):586-590.
8. Matsuno M. Intractable Pancreatic Disease Investigation and
Research Group of the Japanese Ministry of Health, and Welfare.
Guidance of management of severe pancreatitis (in Japanese).
Tokyo: Japanese Ministry of Health, and Welfare. 1997;2-3.
9. Sarles H. Proposal adopted unanimously by the participants of the
symposium on pancreatitis at Marseille, 1963. 7, VII-VIII. 1965.
10. Sarles H, Adler G, Dani R, et al. The pancreatitis classification
of Marseilles-Rome 1988. Scand J Gastroenterol. 1989;24(6):
641-642.
11. Sarner M, Cotton PB. Classification of pancreatitis. Gut. 1984;
25(7):756-759.
12. Singer MV, Gyr K, Sarles H. Revised classification of pancreatitis.
Report of the Second International Symposium on the Classification

5/22/2012 5:38:39 PM

446

13.

14.

15.
16.

17.
18.

19.

20.

21.

22.

23.

24.
25.

26.

27.
28.

29.

30.

31.

32.

33.

Surgery: Evidence-Based Practice

of Pancreatitis in Marseille, France, March 28-30, 1984. Gastroenterology. 1985;89(3):683-685.


Takada T, Kawarada Y, Hirata K, et al. JPN Guidelines for the
management of acute pancreatitis: Cutting-edge information. J
Hepatobiliary Pancreat Surg. 2006;13(1):2-6.
Takada T, Hirata K, Mayumi T, et al. Cutting-edge information for
the management of acute pancreatitis. J Hepatobiliary Pancreat
Sci. 2010;17(1):3-12.
Bradley EL, III. A clinically based classification system for acute
pancreatitis. Ann Chir. 1993;47(6):537-541.
Bollen TL, van Santvoort HC, Besselink MG, et al. The Atlanta
Classification of acute pancreatitis revisited. Br J Surg. 2008;95(1):
6-21.
Bradley EL, III. Atlanta redux. Pancreas. 2003;26(2):105-106.
Kourtesis G, Wilson SE, Williams RA. The clinical significance
of fluid collections in acute pancreatitis. Am Surg. 1990;56(12):
796-799.
Andren-Sandberg A, Dervenis C. Pancreatic pseudocysts in the
21st century. Part I: Classification, pathophysiology, anatomic
considerations and treatment. JOP. 2004;5(1):8-24.
Lenhart DK, Balthazar EJ. MDCT of acute mild (nonnecrotizing) pancreatitis: Abdominal complications and fate of fluid collections. AJR Am J Roentgenol. 2008;190(3):643-649.
Ocampo C, Oria A, Zandalazini H, et al. Treatment of acute pancreatic pseudocysts after severe acute pancreatitis. J Gastrointest
Surg. 2007;11(3):357-363.
Baron TH, Morgan DE, Vickers SM, Lazenby AJ. Organized pancreatic necrosis: Endoscopic, radiologic, and pathologic features
of a distinct clinical entity. Pancreas. 1999;19(1):105-108.
Maringhini A, Uomo G, Patti R, et al. Pseudocysts in acute nonalcoholic pancreatitis: Incidence and natural history. Dig Dis Sci.
1999;44(8):1669-1673.
Apostolou C, Krige JE, Bornman PC. Pancreatic pseudocysts. S
Afr J Surg. 2006;44(4):148-155.
Rosao EL, Sonnenday CJ, Lillemoe KD, Yeo CJ. Pseudocysts and
other complications of pancreatitis. In: Yeo CJ, ed. Shackelfords
Surgery of the Alimentary Tract. 6th ed. Philadelphia: Saunders;
2007:1329-1357.
Walt AJ, Bouwman DL, Weaver DW, Sachs RJ. The impact of
technology on the management of pancreatic pseudocyst. Fift h
annual Samuel Jason Mixter Lecture. Arch Surg. 1990;125(6):
759-763.
Whitcomb DC. Clinical practice. Acute pancreatitis. N Engl J
Med. 2006 18;354(20):2142-2150.
Kawakami H, Kuwatani M, Shinada K, et al. Autoimmune pancreatitis associated with hemorrhagic pseudocysts: A case report
and literature review. Intern Med. 2008;47(7):603-608.
Inagi E, Shimodan S, Amizuka H, et al. Pancreatic cancer initially
presenting with a pseudocyst at the splenic flexure. Pathol Int.
2006;56(9):558-562.
Welsch T, Kleeff J, Esposito I, Buchler MW, Friess H. Autoimmune pancreatitis associated with a large pancreatic pseudocyst.
World J Gastroenterol. 2006;12(36):5904-5906.
Lin BC, Fang JF, Wong YC, Liu NJ. Blunt pancreatic trauma and
pseudocyst: Management of major pancreatic duct injury. Injury.
2007;38(5):588-593.
Yamaguchi T, Takahashi H, Kagawa R, et al. Huge pseudocyst
of the pancreas caused by poorly differentiated invasive ductal
adenocarcinoma with osteoclast-like giant cells: Report of a case.
Hepatogastroenterology. 2007;54(74):599-601.
Behrman SW, Melvin WS, Ellison EC. Pancreatic pseudocysts
following acute pancreatitis. Am J Surg. 1996;172(3):228-231.

PMPH_CH55.indd 446

34. Neoptolemos JP, London NJ, Carr-Locke DL. Assessment of main


pancreatic duct integrity by endoscopic retrograde pancreatography in patients with acute pancreatitis. Br J Surg. 1993;80(1):
94-99.
35. Maringhini A, Ciambra M, Patti R, et al. Ascites, pleural, and
pericardial eff usions in acute pancreatitis. A prospective study of
incidence, natural history, and prognostic role. Dig Dis Sci. 1996;
41(5):848-852.
36. Diculescu M, Ciocirlan M, Ciocirlan M, Stanescu D, Ciprut T,
Marinescu T. Predictive factors for pseudocysts and peripancreatic collections in acute pancreatitis. Rom J Gastroenterol. 2005;
14(2):129-134.
37. Poornachandra KS, Bhasin DK, Nagi B, et al. Clinical, biochemical, and radiologic parameters at admission predicting formation
of a pseudocyst in acute pancreatitis. J Clin Gastroenterol. 2011;
45(2):159-163.
38. Bradley EL, Clements JL, Jr., Gonzalez AC. The natural history of
pancreatic pseudocysts: A unified concept of management. Am J
Surg. 1979;137(1):135-141.
39. Warshaw AL, Rattner DW. Timing of surgical drainage for pancreatic pseudocyst. Clinical and chemical criteria. Ann Surg.
1985;202(6):720-724.
40. Ben MM, Feki MN, Baraket O, Chaari M, Saidani A, Bouchoucha S. Pancreatic pseudocysts complicated by hemorrhage and
infection. Tunis Med. 2011;89(4):383-385.
41. Nayar MK, Leiper KL, Lombard MG. Pseudocystoesophagitis:
Esophagitis and stricture due to pancreatic pseudocyst. JOP.
2005;6(2):194-196.
42. Boudaya MS, Alifano M, Baccari S, Regnard JF. Hemothorax as
the clinical presentation of a pancreaticopleural fistula: Report
of a case. Surg Today. 2007;37(6):518-520.
43. Rigaux J, Poreddy V, Al-Kawas F. Intraperitoneal and retroperitoneal hemorrhage associated with coumadin-induced bleeding into a pancreatic pseudocyst. Clin Gastroenterol Hepatol.
2007;5(6):A32.
44. Yoon SE, Lee YH, Yoon KH, Choi CS, Kim HC, Chae KM. Spontaneous pancreatic pseudocyst-portal vein fistula presenting
with pancreatic ascites: Strength of MR cholangiopancreatography. Br J Radiol. 2008;81(961):e13-e16.
45. Buelta C, Velayos B, Abril C, et al. Pancreaticobronchial fistula as
the first manifestation of pancreatic pseudocysts. Gastroenterol
Hepatol. 2006;29(4):274-275.
46. Yeo CJ, Bastidas JA, Lynch-Nyhan A, Fishman EK, Zinner MJ,
Cameron JL. The natural history of pancreatic pseudocysts documented by computed tomography. Surg Gynecol Obstet. 1990;
170(5):411-417.
47. Subhas G, Bakston D, Ferguson L, Silapaswan S, Mittal VK.
Abdominal wall pseudocyst fluid collection: A complication of
pancreatic pseudocyst. Am Surg. 2010;76(12):1428-1429.
48. Araki K, Shimura T, Watanabe A, et al. Gastric bleeding from
a penetrating pancreatic pseudocyst with pseudoaneurysm
of the splenic artery. Hepatogastroenterology. 2009;56(94-95):
1411-1413.
49. Porcu A, Tilocca PL, Pilo L, Ruiu F, Dettori G. Pancreatic
pseudocyst-inferior vena cava fistula causing caval stenosis, left
renal vein thrombosis, subcutaneous fat necrosis, arthritis and
dysfibrinogenemia. Ann Ital Chir. 2010;81(3):215-220.
50. Ting CF, Chou JW, Yii CY, Huang WH. Spontaneous pancreaticorenal fistula. Intern Med. 2010;49(20):2271-2272.
51. Hsu CY, Lee KC, Chan CC, Lee FY, Lin HC. Gastric necrosis and
perforation as a severe complication of pancreatic pseudocyst. J
Chin Med Assoc. 2009;72(11):603-606.

5/22/2012 5:38:39 PM

Pancreatic Pseudocysts

52. Al Ali JA, Chung H, Munk PL, Byrne MF. Pancreatic pseudocyst
with fistula to the common bile duct resolved by combined biliary
and pancreatic stentinga case report and literature review. Can J
Gastroenterol. 2009;23(8):557-559.
53. Vitas GJ, Sarr MG. Selected management of pancreatic pseudocysts: Operative versus expectant management. Surgery. 1992;
111(2):123-130.
54. Cooperman AM. An overview of pancreatic pseudocysts: The
emperors new clothes revisited. Surg Clin North Am. 2001;
81(2):391-397, xii.
55. Beger HG, Rau B, Isenmann R. Prevention of severe change in
acute pancreatitis: Prediction and prevention. J Hepatobiliary
Pancreat Surg. 2001;8(2):140-147.
56. Ho HS, Frey CF. Gastrointestinal and pancreatic complications associated with severe pancreatitis. Arch Surg. 1995;130(8):
817-822.
57. Fedorak IJ, Ko TC, Djuricin G, McMahon M, Thompson K, Prinz
RA. Secondary pancreatic infections: Are they distinct clinical
entities? Surgery. 1992;112(4):824-830.
58. Bassi C, Vesentini S, Nifosi F, et al. Pancreatic abscess and other
pus-harboring collections related to pancreatitis: A review of 108
cases. World J Surg. 1990;14(4):505-511.
59. Lumsden A, Bradley EL, III. Secondary pancreatic infections.
Surg Gynecol Obstet. 1990;170(5):459-467.
60. Belinkie SA, Russell JC, Deutsch J, Becker DR. Pancreatic
pseudocyst. Am Surg. 1983;49(11):586-590.
61. Aghdassi AA, Mayerle J, Kraft M, Sielenkamper AW, Heidecke
CD, Lerch MM. Pancreatic pseudocystswhen and how to treat?
HPB (Oxford). 2006;8(6):432-441.
62. Bradley EL, III, Clements LJ. Spontaneous resolution of pancreatic pseudocysts: Implications for timing of operative intervention. Am J Surg. 1975;129(1):23-28.
63. Warren WD, Marsh WM, Mullen WH, Jr. Experimental production of pseudocysts of the pancreas with preliminary observations on internal drainage. Surg Gynaecol Obstet. 1957;105:385.
64. Frey CF. Pancreatic pseudocystoperative strategy. Ann Surg.
1978;188(5):652-662.
65. Talar-Wojnarowska R, Wozniak B, Pazurek M, Malecka-Panas
E. Outcome of pseudocysts complicating chronic pancreatitis.
Hepatogastroenterology. 2010;57(99-100):631-634.
66. Bollen TL, van Santvoort HC, Besselink MG, van Es WH,
Gooszen HG, van Leeuwen MS. Update on acute pancreatitis: Ultrasound, computed tomography, and magnetic resonance imaging features. Semin Ultrasound CT MR. 2007;28(5):
371-383.
67. Koo BC, Chinogureyi A, Shaw AS. Imaging acute pancreatitis.
Br J Radiol. 2010;83(986):104-112.
68. Balthazar EJ, Robinson DL, Megibow AJ, Ranson JH. Acute pancreatitis: Value of CT in establishing prognosis. Radiology. 1990;
174(2):331-336.
69. Besselink MG, van Santvoort HC, Bollen TL, et al. Describing
computed tomography findings in acute necrotizing pancreatitis with the Atlanta classification: An interobserver agreement
study. Pancreas. 2006;33(4):331-335.
70. De WJ, Vogelaers D, Decruyenaere J, De VM, Colardyn F. Infectious complications of acute pancreatitis. Acta Clin Belg. 2004;
59(2):90-96.
71. Soliani P, Franzini C, Ziegler S, et al. Pancreatic pseudocysts following acute pancreatitis: Risk factors influencing therapeutic
outcomes. JOP. 2004;5(5):338-347.
72. Zhou ZG, Zheng YC, Shu Y, et al. Laparoscopic management of
severe acute pancreatitis. Pancreas. 2003;27(3):e46-e50.

PMPH_CH55.indd 447

447

73. Andren-Sandberg A, Ansorge C, Eiriksson K, Glomsaker T,


Maleckas A. Treatment of pancreatic pseudocysts. Scand J Surg.
2005;94(2):165-175.
74. Kruger M, Schneider AS, Manns MP, Meier PN. Endoscopic
management of pancreatic pseudocysts or abscesses after an
EUS-guided 1-step procedure for initial access. Gastrointest
Endosc. 2006;63(3):409-416.
75. Nealon WH, Walser E. Surgical management of complications
associated with percutaneous and/or endoscopic management of
pseudocyst of the pancreas. Ann Surg. 2005;241(6):948-957.
76. Merkle EM, Gorich J. Imaging of acute pancreatitis. Eur Radiol.
2002;12(8):1979-1992.
77. Morgan DE, Baron TH, Smith JK, Robbin ML, Kenney PJ. Pancreatic fluid collections prior to intervention: Evaluation with
MR imaging compared with CT and US. Radiology. 1997;203(3):
773-778.
78. Baron TH, Morgan DE. The diagnosis and management of
fluid collections associated with pancreatitis. Am J Med. 1997;
102(6):555-563.
79. Robinson PJ, Sheridan MB. Pancreatitis: Computed tomography and magnetic resonance imaging. Eur Radiol. 2000;10(3):
401-408.
80. Ellis CT, Barbour JR, Shary TM, Adams DB. Pancreatic cyst:
Pseudocyst or neoplasm? Pitfalls in endoscopic retrograde cholangiopancreatography diagnosis. Am Surg. 2010;76(7):725-730.
81. Seicean A. Endoscopic ultrasound in chronic pancreatitis: Where
are we now? World J Gastroenterol. 2010;16(34):4253-4263.
82. Ross AS, Kozarek RA. Therapeutic pancreatic endoscopy. Dig
Liver Dis. 2010;42(11):749-756.
83. Aljarabah M, Ammori BJ. Laparoscopic and endoscopic approaches for drainage of pancreatic pseudocysts: A systematic
review of published series. Surg Endosc. 2007;21(11):1936-1944.
84. Froeschle G, Meyer-Pannwitt U, Brueckner M, Henne-Bruns D.
A comparison between surgical, endoscopic and percutaneous
management of pancreatic pseudocystslong term results. Acta
Chir Belg. 1993;93(3):102-106.
85. Palanivelu C, Senthilkumar K, Madhankumar MV, et al. Management of pancreatic pseudocyst in the era of laparoscopic
surgeryexperience from a tertiary centre. Surg Endosc. 2007;
21(12):2262-2267.
86. Pryor A, Means JR, Pappas TN. Laparoscopic distal pancreatectomy with splenic preservation. Surg Endosc. 2007;21(12):
2326-2330.
87. Stiles GM, Berne TV, Thommen VD, Molgaard CP, Boswell WD.
Fine needle aspiration of pancreatic fluid collections. Am Surg.
1990;56(12):764-768.
88. Vosoghi M, Sial S, Garrett B, et al. EUS-guided pancreatic pseudocyst drainage: Review and experience at Harbor-UCLA Medical
Center. MedGenMed. 2002;4(3):2.
89. Zein CO, Baron TH, Morgan DE. Endoscopic pancreaticoduodenostomy for treatment of pancreatic duct disconnection because of
severe acute pancreatitis. Gastrointest Endosc. 2003;58(1):130-134.
90. Sheng QS, Chen DZ, Lang R, et al. Laparoscopic cystogastrostomy for the treatment of pancreatic pseudocysts: A case report.
World J Gastroenterol. 2008;14(30):4841-4843.
91. Heider R, Meyer AA, Galanko JA, Behrns KE. Percutaneous
drainage of pancreatic pseudocysts is associated with a higher
failure rate than surgical treatment in unselected patients. Ann
Surg. 1999;229(6):781-787.
92. Underwood RA, Soper NJ. Current status of laparoscopic surgery of the pancreas. J Hepatobiliary Pancreat Surg. 1999;6(2):
154-164.

5/22/2012 5:38:39 PM

448

Surgery: Evidence-Based Practice

93. Park DH, Lee SS, Moon SH, et al. Endoscopic ultrasoundguided versus conventional transmural drainage for pancreatic
pseudocysts: A prospective randomized trial. Endoscopy. 2009;
41(10):842-848.
94. Hamza N, Ammori BJ. Laparoscopic drainage of pancreatic
pseudocysts: A methodological approach. J Gastrointest Surg. 2010;
14(1):148-155.
95. Pallapothu R, Earle DB, Desilets DJ, Romanelli JR. NOTES((R))
stapled cystgastrostomy: A novel approach for surgical management of pancreatic pseudocysts. Surg Endosc. 2011;25(3):
883-889.
96. Melman L, Azar R, Beddow K, et al. Primary and overall success
rates for clinical outcomes after laparoscopic, endoscopic, and
open pancreatic cystgastrostomy for pancreatic pseudocysts.
Surg Endosc. 2009;23(2):267-271.
97. Evans KA, Clark CW, Vogel SB, Behrns KE. Surgical management
of failed endoscopic treatment of pancreatic disease. J Gastrointest Surg. 2008;12(11):1924-1929.
98. Johnson MD, Walsh RM, Henderson JM, et al. Surgical versus
nonsurgical management of pancreatic pseudocysts. J Clin
Gastroenterol. 2009;43(6):586-590.
99. Cahen D, Rauws E, Fockens P, Weverling G, Huibregtse K,
Bruno M. Endoscopic drainage of pancreatic pseudocysts: Longterm outcome and procedural factors associated with safe and
successful treatment. Endoscopy. 2005;37(10):977-983.
100. Cavallini A, Butturini G, Malleo G, et al. Endoscopic transmural
drainage of pseudocysts associated with pancreatic resections
or pancreatitis: A comparative study. Surg Endosc. 2011;25(5):
1518-1525.
101. Baron TH, Harewood GC, Morgan DE, Yates MR. Outcome
differences after endoscopic drainage of pancreatic necrosis, acute
pancreatic pseudocysts, and chronic pancreatic pseudocysts.
Gastrointest Endosc. 2002;56(1):7-17.
102. Trevino JM, Tamhane A, Varadarajulu S. Successful stenting
in ductal disruption favorably impacts treatment outcomes

PMPH_CH55.indd 448

103.

104.

105.

106.

107.

108.

109.

110.

111.

in patients undergoing transmural drainage of peripancreatic fluid collections. J Gastroenterol Hepatol. 2010;25(3):
526-531.
Varadarajulu S, Christein JD, Tamhane A, Drelichman ER,
Wilcox CM. Prospective randomized trial comparing EUS
and EGD for transmural drainage of pancreatic pseudocysts
(with videos). Gastrointest Endosc. 2008;68(6):1102-1111.
Cremer M, Deviere J, Engelholm L. Endoscopic management
of cysts and pseudocysts in chronic pancreatitis: Long-term
follow-up after 7 years of experience. Gastrointest Endosc. 1989;
35(1):1-9.
Fockens P, Johnson TG, van Dullemen HM, Huibregtse K,
Tytgat GN. Endosonographic imaging of pancreatic pseudocysts
before endoscopic transmural drainage. Gastrointest Endosc.
1997;46(5):412-416.
Binmoeller KF, Seifert H, Walter A, Soehendra N. Transpapillary
and transmural drainage of pancreatic pseudocysts. Gastrointest
Endosc. 1995;42(3):219-224.
Rao R, Fedorak I, Prinz RA. Effect of failed computed
tomography-guided and endoscopic drainage on pancreatic
pseudocyst management. Surgery. 1993;114(4):843-847.
Bartoli E, Delcenserie R, Yzet T, et al. Endoscopic treatment
of chronic pancreatitis. Gastroenterol Clin Biol. 2005;29(5):
515-521.
Jacobson BC, Baron TH, Adler DG, et al. ASGE guideline: The
role of endoscopy in the diagnosis and the management of cystic
lesions and inflammatory fluid collections of the pancreas.
Gastrointest Endosc. 2005;61(3):363-370.
Ito K, Perez A, Ito H, Whang EE. Pancreatic pseudocysts: Is
delayed surgical intervention associated with adverse outcomes?
J Gastrointest Surg. 2007;11(10):1317-1321.
Seicean A, Stan-Iuga R, Badea R, et al. The safety of endoscopic
ultrasonography-guided drainage of pancreatic fluid collections
without fluoroscopic control: A single tertiary center experience.
J Gastrointestin Liver Dis. 2011;20(1):39-45.

5/22/2012 5:38:39 PM

CHAPTER 56

Chronic Pancreatitis
Katherine A. Morgan and David B. Adams

INTRODUCTION

Nevertheless, insight, intuition, and individual surgeon innovation have brought us to the modern era of pancreatic surgery,
and the future of surgery for chronic pancreatitis remains exciting
and challenging as we move the field forward with careful examination of the science of chronic pancreatitis and the published
evidence of its surgical management.

It is the merit of the common law that it decides the case first
and determines the principle afterwards. The life of the law
has not been logic; it has been experience.
Oliver Wendell Holmes
Chronic pancreatitis is a disorder whose pathogenesis is poorly
understood. Most likely, chronic pancreatitis does not have a single
causative agent but is multifactorial, related to genetic, anatomic,
and environmental factors. The disease itself has protean manifestations, and when we consider surgical treatment of chronic
pancreatitis we are speaking of more than one disease entity.
Peripancreatic fibrosis and inflammation may result in obstruction of the biliary tree, the duodenum, the transverse colon, the
splanchnic vasculature, as well as the pancreatic duct. Erosion of
peripancreatic vessels may cause life-threatening hemorrhage.
The number one indication for surgery in chronic pancreatitis
remains, however, intractable pain, and the complex splanchnic,
peripheral, and central pathways involved in chronic pancreatitis
pain is a promising focus of future research.
Surgical management of chronic pancreatitis is steeped in history
and tradition. Eponyms for pancreatic tumor resection procedures
Kausch, Whipple, Longmirehave their counterparts in the 20th
century eponyms of pancreatic resection and drainage procedures
for chronic pancreatitisDuval, Puestow, Gillesby, Partington,
Rochelle, Frey, Beger. The eponymic status of chronic pancreatitis
surgery points to the fact that though chronic pancreatitis is an
increasingly common cause of hospitalization, surgery for chronic
pancreatitis is not. The life of pancreatic surgery has not been
evidence, it has been experience. Individual surgeon and centerbased experiences dominate the literature, and because surgery for
chronic pancreatitis is uncommon compared with other abdominal
operations, large cohorts of randomized and case-controlled series
are hard to find. The merit of the conventional wisdom regarding
pancreatic surgery is that surgeon experience decides the case first
and the evidence is found afterwards.

1. What is the current understanding of the underlying pathophysiology of chronic pancreatitis?


The underlying pathophysiology of chronic pancreatitis is not well
defined. Although risk factors, most notably excessive alcohol consumption, are recognized, direct causation has not been delineated.
A more complex etiologic mechanism likely exists involving the
interaction of multiple factors including environmental stressors,
genetic predisposition, and individual immunologic response. Several theories on the pathophysiologic sequence of chronic pancreatitis have been proposed. The necrosisfibrosis hypothesis contends
that pancreatic injury from repeated bouts of acute pancreatitis
results in fibrosis and chronic pancreatitis.1 An alternate modern
theory is the Sentinel Acute Pancreatitis Event (SAPE) hypothesis,
described by Whitcomb. In this model, it is theorized that a significant acute pancreatitis event results in an inflammatory response.
This inflammatory injury incites an immune response that then
effects the development of fibrosis and pain.2
A genetic basis for susceptibility to pancreatitis is recognized.
In 1996, Whitcomb et al.3 described the cationic trypsinogen gene
(PRSS1), which is the mutational locus responsible in several family cohorts of hereditary pancreatitis. The mutated genes code
for a trypsin that is inappropriately activated.3 Hereditary pancreatitis is an autosomal dominant disorder with incomplete penetrance (8093%) and phenotype. Approximately, half of patients
affected develop chronic pancreatitis and 40% of those develop
pancreatic cancer. Importantly, the discovery of PRSS1 led to
an understanding of the potential for genetic polymorphisms
as a mechanism for susceptibility to pancreatitis. It also led to
the concept that inappropriate activation of trypsin or failure to

449

PMPH_CH56.indd 449

5/22/2012 5:39:16 PM

450

Surgery: Evidence-Based Practice

eliminate activated trypsin is a primary cause of acute and chronic


pancreatitis.3-6
Other susceptibility genes have been recognized, including the
anionic trypsinogen gene (PRSS2), serine protease inhibitor Kazal 1
(SPINK1; pancreatic secretory trypsin inhibitor), cystic fibrosis
transmembrane conductance regulator (CFTR), chymotrypsinogen C (CTRC), and calcium-sensing receptor (CASR).
The anionic trypsinogen gene (PRSS2) has been associated
with a loss of function mutation resulting in a form of trypsin that
is rapidly eliminated by autolysis. This mutation, found more commonly in healthy controls than in pancreatitis patients, is felt to be
protective against the development of pancreatitis.7
SPINK1 is an acute phase protein, upregulated during pancreatic inflammation, which plays an inhibitory role toward trypsin
by competitively binding to its active site. Its mutation is associated
with increased risk of pancreatitis.8 The cystic fibrosis transmembrane conductance regulator gene (CFTR) is thought to play a duct
flushing role in the prevention of pancreatitis. Mutation conveys a
loss of this protective function.9
Chymotrypsin C (CTRC) destroys prematurely activated
trypsin, protecting against pancreatitis. Its mutation is found in
4.8% to 12% of pancreatitis versus 0.7% to 1.1% of healthy controls,
translating to an increased risk of pancreatitis of fivefold.10,11 The
CASR is a membrane-bound G protein coupled receptor that is
involved in calcium homeostasis, mutation of which has been associated with chronic pancreatitis. The hypercalcemic state favors
trypsin activation and inhibits trypsin autolysis.12
An altered immune response has been noted in the development of chronic pancreatitis. HLA antigens have been examined
at length with inconclusive results. HLAA1 and HLABw39 are
expressed more frequently in cases of chronic pancreatitis.13
On a cellular level, pancreatic stellate cells (PSCs) have been
implicated in chronic pancreatitis. PSCs are homologous to
hepatic stellate cells and in the quiescent state are involved in fat
storage. In response to pancreatic injury, however, PSCs alter their
phenotype to extracellular matrix-producing cells, contributing
to the fibrotic parenchymal changes characteristic of chronic
pancreatitis.14
More complete understanding of the pathophysiology of pancreatitis on a genetic and cellular level is ongoing. This developing knowledge in the context of the SAPE hypothesis promises
to unveil potential points of early intervention in the future. For
example, understanding and avoidance of certain environmental
stressors can prevent triggering of the acute inf lammation.
Enzyme pathway inhibition could potentially allow for intervention in the case of the presence of a known genetic susceptibility
and minimize an inflammatory response. Modulation of the
immune response may potentially prevent the development of
fibrosis (Level 5 evidence).
2. What is the etiology of pain in chronic pancreatitis?
The clinical hallmark of chronic pancreatitis is severe intractable
abdominal pain. Pain is reported by 80% to 94% of chronic pancreatitis patients.15 Unfortunately, the mechanisms of pain in chronic
pancreatitis are poorly understood. The pain is complex, likely multifactorial, and certainly of disparate causes in individual patients.
Pancreatic ductal hypertension has been implicated as an
etiology of pain in some patients with chronic pancreatitis.

PMPH_CH56.indd 450

Intraoperative measurements of main pancreatic duct pressures


have been shown to be significantly elevated in patients with
dilated duct pancreatitis. The ductal hypertension is theorized
to result from exocrine secretion against a proximal obstruction
due to stricture or stone. In theory, the pain results from ductal
distention. Th is theory is supported in part by successful pain
relief after lateral pancreaticojejunostomy (LPJ) for dilated duct
pancreatitis.16
Pancreatic tissue hypertension, analogous to extremity or
abdominal compartment syndrome, occurs when gland fibrosis
is constricting, notably during times of glandular exocrine secretion. Elevated pancreatic tissue pressures have been observed and
reported (>30 vs. 7 mm Hg in normal gland). This compartment
syndrome is evidenced by a decreased pain with pancreatectomy
during surgical pancreaticojejunostomy.17
Strong evidence exists that neural mechanisms, both peripheral nociceptive neural pathways and neuroplastic/neuropathic
pathways, are fundamental to the pain of chronic pancreatitis.
Peripancreatic neurons are clearly involved in the inflammatory
process of pancreatitis.18 Histologically, there are focal disruptions of the perineural sheath with infi ltration of inflammatory
cells within and around the pancreatic tissue.19 The degree of this
inflammatory neuritis is directly related to the frequency and
intensity of the pancreatitis pain.20
There is an increase in the number and diameter of intralobular and interlobular pancreatic nerve bundles in patients
with chronic pancreatitis, likely due to the activation of multiple
neurotrophic factors.19 Nerve growth factor (NGF) and its highaffinity receptor tyrosine kinase A (TrkA) are overexpressed in
chronic pancreatitis and the degree of TrkA expression correlates
to pain intensity.21 Similarly, brain-derived neutrotrophic factor (BDNF)22 and the glial cell-linederived neurotrophic factor
artemin are upregulated in chronic pancreatitis and relate directly
to pain levels.20
There is upregulation of several of the nociceptive neuropeptides such as Substance P and calcitonin gene-related peptide in the
pancreatic tissue of pancreatitis patients.23 Substance P is a tachykinin involved in nociception and is a component in both the neural and immune systems. There is increased Substance P mRNA
expression in chronic pancreatitis. Substance P receptors NK-1R,
NK-2R, and PPT-A are also upregulated in chronic pancreatitis
tissue. NK-2R expression correlates with intensity and duration of
pain and NK-1R levels relate to the degree of pancreatic fibrosis.24,25
The Substance P degradation enzyme neutral endopeptidase
(NEP) is not upregulated in pancreatitis and may exacerbate the
associated pain syndrome. Interestingly, after pancreatic resection, the majority of chronic pancreatitis patients have decreased
Substance P serum levels.24
In addition to these histologic and biochemical neural changes
in pancreatitis, there are alterations in the overall autonomic innervation and glial cell activation response, resulting in general neural
remodeling.26 This neuroplastic effect represents pancreatic neuropathy. These changes are certainly consistent with the clinical
pattern of chronic pancreatitis pain, with its degree of recidivism
despite many therapies.
Although much has been elucidated about pain mechanisms
in pancreatitis, they are still incompletely understood. A clearer
definition of the pain mechanisms at work would better direct
therapeutic efforts (Level 5 evidence).

5/22/2012 5:39:16 PM

Chronic Pancreatitis

3. What is the role of endoscopic intervention for chronic


pancreatitis?
Medical therapies, including ethanol abstinence, nutritional
optimization, oral pancreatic enzyme supplementation, and
analgesics are the initial approaches to chronic pancreatitis. In
patients with continued symptomatology, endoscopic assessment
and management are warranted. Therapeutic endoscopic interventions for chronic pancreatitis are aimed at improving drainage in obstructive pancreatopathy, and include sphincterotomy,
stone extraction with possible lithotripsy, stricture dilation, and
stenting.
In practice, the endoscopic approach is generally exhausted
prior to committing a patient to surgery, given the perceived risks
and increased potential morbidity of surgical intervention. This
algorithm holds in most centers despite now two randomizedcontrolled trials demonstrating an increased effectiveness of pain
relief in patients after surgical intervention versus therapeutic
endoscopy. Dite et al.27 in the Czech Republic reported on 72
patients with pancreatic duct obstruction and pain who were randomized to endoscopic or surgical therapy. Endoscopic therapy
included 52% sphincterotomy and stenting or 23% stone removal,
whereas operative management consisted of drainage (20%)
and resective (80%) procedures. At 5-year follow-up, the surgical group had a greater number of patients who were pain-free
(34% vs. 15%), whereas the rate of partial pain relief was equivalent between the groups (52% surgery, 46% endotherapy).27 Cahen
et al.28 in 2007 reported on 39 patients with dilated duct (>7 mm
main pancreatic duct) pancreatitis randomized to surgery (pancreaticojejunostomy) versus endoscopic therapy (sphincterotomy/stent) for the management of pain. At 2-year follow-up,
surgically managed patients had significantly lowered Izbicki
pain scores (25 vs. 51) as well as better quality of life as measured
by their SF-36 physical health scores. Pain relief was achieved in
32% of endoscopically managed patients as compared with 75% of
patients who underwent surgery (p = .007). Complications, hospital stay, and pancreatic exocrine and endocrine function were
equivalent between groups. The endoscopically managed group
required a median of eight procedures versus three for the surgical group (p < .001).28
Endoscopic evaluation and intervention remain an essential
component to the management of chronic pancreatitis. In some
patients, early surgical intervention may be beneficial rather than
repeated endocopic procedures (Level 1 evidence).
4. What are the indications and outcomes in the surgical management of chronic pancreatitis?
The primary indication for surgery in patients with chronic pancreatitis is intractable pain, unresponsive to medical or endoscopic
therapies. Less common indications include anatomic complications, including duodenal or biliary obstruction or concern for
malignancy. In all, approximately half of patients with chronic
pancreatitis meet the criteria for surgery at some point in their
disease process.
As described earlier in this chapter, the mechanisms for
pain in pancreatitis are poorly understood, thus making surgical
decision-making for chronic pancreatitis challenging. The goals
of surgery are to effectively and durably relieve pain, to minimize
morbidity and mortality, and to preserve endocrine and exocrine

PMPH_CH56.indd 451

451

function when possible. In general, the main pancreatic duct


anatomy is the most essential determinant of surgical planning.
Patients with a large main pancreatic duct (greater than 67mm
in size) are suited to a drainage procedure. Those with a small
pancreatic duct do less well with drainage and are better managed
with resection.29 If a portion of the organ is primarily affected,
this part can be resected with expected benefit. An inflammatory
mass in the head of the pancreas or a densely fibrotic tail can be
resected by partial pancreatectomy with good results. If however,
the gland is diff usely involved, total pancreatectomy (TP) may be
indicated. This operation has received new enthusiasm given the
development of islet autotransplantation.

LATERAL PANCREATICOJEJUNOSTOMY
In 1957, at the Western Surgical Association, Puestow and Gillesby16
presented their series of 21 patients with chronic pancreatitis who underwent pancreaticojejunostomy. Their technique
included a splenectomy and pancreatic tail resection followed by
opening of the main pancreatic duct along its length. The pancreas was then mobilized to the level of the superior mesenteric
vessels, and the gland was invaginated into a defunctionalized
limb of jejunum.16 In 1960, Partington and Rochelle30 presented
their modification of Puestows procedure, with exposure of the
anterior aspect of the pancreas and a lateral side-to-side anastomosis between the fibrotic parenchymal edge of the pancreatic
duct and the opened small bowel Roux limb. Since these initial
descriptions, multiple series have examined the outcomes of LPJ
for dilated duct pancreatitis.31-38 Pain relief is achieved in 48%
to 91% of patients on follow-up. Morbidity is reported on average to be 20%, and preservation of endocrine and exocrine pancreatic function is optimized.39 LPJ remains the procedure of
choice in dilated duct pancreatitis (duct diameter >7 mm), without an inflammatory mass in the head of the pancreas (Level 4
evidence).
With LPJ, a secondary failure rate exists. The primary mechanism for failure of the Puestow procedure, marked by recurrent
pain, is likely related to the disease within the head of the pancreas.
Some of this disease represents intraductal stone disease and can
be addressed with the addition of intraoperative pancreatoscopy
with electrohydraulic lithotripsy. In selected patients, this procedure decreases re-admissions and pain recurrence from 9% to 0%.
Pain relief rates of 90% are reported.40 Alternatively, in patients
with dilated duct disease with an inflammatory pancreatic head,
drainage alone may be doomed to failure and a localized pancreatic head resection combined with a pancreaticojejunostomy
(LR-LPJ) is an option.

LOCAL RESECTION OF THE PANCREATIC


HEAD WITH LONGITUDINAL
PANCREATICOJEJUNOSTOMY
In 1987, Frey et al.41 described a localized pancreatic head resection with a longitudinal pancreaticojejunostomy (LR-LPJ). The
goal of this surgery is pain relief due to both ductal drainage and
resection of damaged and poorly drained tissue in the head of the

5/22/2012 5:39:16 PM

452

Surgery: Evidence-Based Practice

pancreas, at the same time preserving the duodenum and maintaining pancreatic function. Certainly, this addresses the issue of
late failure of LPJ due to residual disease in the head of the pancreas. Frey reported initially on 50 patients, describing a morbidity of 22% and a pain relief rate of 84%.41 In modern series, pain
relief rates of 62% to 88% are reported, with morbidity of 20% to
30% (Level 1 evidence).42-45

in patients with chronic pancreatitis. Long-term pain relief


rates and the development of postoperative diabetes mellitus are
equivalent.48 Some studies have reported improved professional
rehabilitation53 and improved quality of life after PPPD (Level 1
evidence).55

DUODENAL PRESERVING PANCREATIC


HEAD RESECTION

PANCREATICODUODENECTOMY
Pancreaticoduodenectomy (PD) for chronic pancreatitis was described as early as 1946 by Whipple, and thus has greater history than
any other procedure in the management of chronic pancreatitis.46
Despite this, the enthusiasm for PD for benign disease was limited
really until the 1970s, given a prohibitive mortality rate around
30%. In the modern era, with acceptable operative morbidity
and mortality, patients with chronic pancreatitis pain and an
inflammatory mass in the head of the pancreas benefit from this
operation. In these patients, the head acts as the described pacemaker of disease. Outcomes for PD include pain relief in 70% to
89% of patients, morbidity in 16% to 53%, and mortality in less than
5% in high volume centers.38,47-50
In an effort to maintain the potential physiologic benefits of
a functional pylorus, the pylorus preserving pancreaticoduodectomy (PPPD) was popularized by Traverso and Longmire51 in 1978.
This modification has been well adopted into pancreatic surgery.
Disappointingly, the postulated nutritional benefits have not been
evidenced, with similar long-term weight status being reported
between the two techniques.48,52-54 PPPD may have a higher
rate of delayed gastric emptying postoperatively, particularly

In an effort to limit the associated long-term morbidity of duodenal resection, the duodenal preserving pancreatic head resection (DPPHR) was developed by Beger et al.56-58 in the 1970s. In
this surgery, the afflicting inflammatory mass in the head of the
pancreas is resected, leaving the bile duct and duodenum intact
and decompressed. Morbidity and mortality are acceptable at
28.5% and 1%, respectively. Pain relief is reported in 77% to 88%
of patients, with professional rehabilitation rates of 63% to 69%
(Level 1 evidence).56-59

COMPARATIVE RANDOMIZED CLINICAL


TRIALS OF PANCREATIC HEAD
RESECTION
Over the past several years, there has been great interest from
multiple surgical groups, particularly in Europe, in methodically
evaluating the comparative efficacy of the various approaches to
pancreatic head resection in chronic pancreatitis.60-66 The results
of the Level 1 studies are summarized in Table 56.1.

Table 56.1 Level 1 studies comparing techniques of pancreatic head resection


Study

Comparison

Morbidity (%)

Mortality (%)

Pain Relief (%)

Klempa et al.60

PD
DPPHR

21
22

51
54

0
5

70
100

Buchler et al.61

PPPD
DPPHR

20
20

20
15

0
0

77
94

Muller et al.*62

PPPD

14

DPPHR

15

Farkas et al.63

PPPD
DPPHR

20
20

40
0

0
0

100
100

Izbicki et al.64

DPPHR
LR-LPJ

20
22

20
9

0
0

70
70

Izbicki et al.65

LR-LPJ
PPPD

31
30

19
53

3
0

80
75

Strate et al.**66

LR-LPJ

31

82

PPPD

30

81

*Long-term follow-up of Buchlers study.


**Long-term follow-up of Izbickis study.

PMPH_CH56.indd 452

5/22/2012 5:39:16 PM

Chronic Pancreatitis

DISTAL PANCREATECTOMY
In patients with chronic pancreatitis with disease localized to
the body and tail of the pancreas or in patients with obliteration
of the pancreatic duct in the neck or body, distal pancreatectomy (DP) is an effective means of pain relief. Pain relief rates
of 57% to 84% are reported with occupational rehabilitation in
29% to 73%. Morbidity and mortality are reasonable, reported
in 15% to 32% and 2% to 2.2% of cases, respectively.37,67-69 The
postoperative pancreatic fistula remains a challenge and appears
to be related to patient-specific factors rather than operative
technique.70 In the modern era, some cases can be performed
safely laparoscopically, with expected decreased length of hospital stay and at least equivalent morbidity.71,72 DP appears to be
applicable in approximately 9% to 25% of patients in larger series
of patients undergoing surgery for chronic pancreatitis (Level 4
evidence).37,73

TP WITH ISLET AUTOTRANSPLANTATION


In selected patients with chronic pancreatitis, TP is a good option
for pain relief. Specifically, patients with diff use small duct pancreatitis, patients who have failed lesser surgeries, and those with
hereditary pancreatitis are good candidates for TP. TP was first
performed in 1894 by Theodore Billroth and in 1944 for pancreatitis by Clagett. Excellent pain relief rates of 72% to 100% have
been described, with morbidity rates of 22% to 54% and mortality 0% to 14%. There is a requisite pancreatogenic diabetes that
accompanies TP; however, with severe diabetic control problems
in 15% to 75% of patients, and in one series half of late postoperative deaths were due to hypoglycemia.74-76 The problem is not only
a loss of insulin, but also of the counter-regulatory islet hormones,
such as glucagon and pancreatic polypeptide. As a result, patients
have wide swings in blood glucose, an unpredictable response
to insulin therapy, and importantly may develop hypoglycemic
unawareness, which increases morbidity sixfold.77 The largest
modern series of TP is from the Mayo Clinic. There were three late
deaths from hypoglycemia and 26% of patients were rehospitalized for glycemic control.76 Thus, it seems that TP is a good option
for pain relief, but the resultant diabetes is exceptionally morbid.
In the 1970s, the concept arose at the University of Minnesota for the autotransplantation of islet cells immediately after
pancreas resection, in order to preserve endocrine function and
allow for the analgesic benefits of extensive pancreatectomy. In
1978, Sutherland reported the first TP with islet autotransplantation (TPIAT) in a young woman with hereditary pancreatitis.78
Since that time, the science of islet harvest has progressed, and
over the past decade, the procedure has received new interest with
now multiple centers offering this therapy.
Pain relief rates of 72% to 86% have been reported with TPIAT,
with 47% to 55% morbidity and 1.4% to 6% mortality, and insulin
independence in 10% to 40% after islet transplant.74,79-82 Improved
posttransplant endocrinologic function correlates with the number
of islet equivalents per kilogram harvested74,81,82 and transplanted
islet function appears to be durable, with outcomes reported for
greater than 13 years.83 Although this therapy holds promise, longterm pain relief data are currently lacking (Level 4 evidence).

PMPH_CH56.indd 453

453

LAPAROSCOPIC SURGERY FOR


CHRONIC PANCREATITIS
Laparoscopic pancreatic surgery began as early as 1911 when
Bernkeim described the use of organoscopy, which involved a
headlight and a proctoscope to evaluate for metastatic disease in
pancreatic cancer. Modern laparoscopic staging for pancreatic
cancer was introduced by Cuschieri in Europe in 1978 and Warshaw in the United States in 1986. It was not until the development of laparoscopic stapler technology in the early 1990s that
more complex procedures could be undertaken. In 1996, Gagner
reported on five patients with benign tumors on which he performed a laparoscopic DP.84 Currently, laparoscopic DP is the
most commonly reported laparoscopic pancreatic surgery. The
largest single-institution experience is from the Mayo Clinic,
with 100 laparoscopic DPs compared with 100 open historical
controls. The authors reported lower blood loss and length of
stay postoperatively in the laparoscopic group, but equivalent
operative time and morbidity including pancreatic fistula rate.
Importantly, only one of the laparoscopic cases was for chronic
pancreatitis.71 The largest multicenter group included 667 DPs
with 159 (24%) laparoscopic. Fourteen (9%) were for chronic
pancreatitis. The authors reported lower blood loss, length of
stay and morbidity with laparoscopy, and equivalent operative
times and pancreatic fistula rates.72 Laparoscopic DP is a technique in progress, rapidly approaching the standard in benign
or low-grade neoplasms. Given the difficult dissection and loss
of planes in chronic pancreatitis, its application in this patient
group is still currently in evolution. Laparoscopic LPJ has been
described by several authors and is technically feasible.85 Success rate for the minimally invasive approach in this operation
increases as the size of the pancreatic duct increases. The laparoscopic PD is being performed at a few centers, mostly for malignant disease, but in a few with chronic pancreatitis. The authors
report reasonable operative times (median 357368 min), blood
loss (75240 mL), morbidity (26.742%), and pancreatic fistula
rates (6.718%).86,87 The limitations of laparoscopic PD in chronic
pancreatitis are similar to those with laparoscopic DP, with loss
of planes and distortion of anatomy due to the fibrotic changes
of disease (Level 3 evidence).

CONCLUSIONS
The British physicist Sir Ernest Rutherford quipped that all science is either physics or stamp-collecting. The evidence behind
the surgical science of chronic pancreatitis management is
clearly more akin to stamp-collecting than physics. Level 1 evidence, when uncovered, is handicapped by confounders related
to surgeon experience and selection, difficult to measure outcome variables, and time which grows old in itself. Yet, large
advances have been made in the standardization of operations
for chronic pancreatitis and in defi ning the salient outcome
measurements. Therefore, it cannot be long before we have better understanding of the disease of chronic pancreatitis and new
surgical science to continue to improve the safety and efficacy
of surgical treatments.

5/22/2012 5:39:16 PM

454

Surgery: Evidence-Based Practice

Clincial Question Summary


Question

Answer

Level of
Evidence

Recommendation

References

1 What is the current


understanding of the
underlying pathophysiology
of chronic pancreatitis?

There is an interaction of multiple


factors including environmental
stressors, genetic predisposition,
and individual immunologic
response.

1-14

2 What is the etiology of pain in


chronic pancreatitis?

Neural mechanisms, both peripheral


nociceptive neural pathways and
neuroplastic/neuropathic pathways,
are fundamental to the pain of
chronic pancreatitis.

15-26

3 What is the role of endoscopic


intervention for chronic
pancreatitis?

Endoscopic evaluation and


intervention remain an essential
component to the management
of chronic pancreatitis. In patients
with obstructive pancreatopathy,
early surgical intervention may be
beneficial rather than repeated
endocopic procedures.

27-28

5 Lateral pancreaticojejunostomy
(LPJ)

LPJ remains the procedure of choice


for pain relief in dilated duct
pancreatitis (duct diameter >7 mm),
without an inflammatory mass in
the head of the pancreas.

30-39

6 Pancreaticoduodenectomy (PD)

PD is an effective means of pain relief


in patients with an inflammatory
mass in the head of the pancreas
and debilitating pain from chronic
pancreatitis.

60-63, 65-66

7 Duodenal preserving pancreatic


head resection (DPPHR)

DPPHR is an effective means of


pain relief in patients with an
inflammatory mass in the head of
the pancreas and debilitating pain
from chronic pancreatitis.

60-64, 66

8 Local resection of the head


of the pancreas with lateral
pancreaticojejunostomy
(LR-LPJ)

LR-LPJ achieves pain relief in chronic


pancreatitis by both ductal
drainage and resection of damaged
and poorly drained tissue in the
head of the pancreas.

64-66

9 Distal pancreatectomy

Distal pancreatectomy is an effective


means of pain relief in patients with
disease localized to the tail of the
pancreas.

37, 67-70

10 Total pancreatectomy with


islet autotransplantation

Total pancreatectomy is an effective


means of pain relief in selected
patients with chronic pancreatitis.

74, 79-82

11 Laparoscopic pancreas surgery

Laparoscopy is a safe and effective


approach in selected cases of
pancreatic resection.

71, 72

4 What are the indications and


outcomes in the surgical
management of chronic
pancreatitis?

PMPH_CH56.indd 454

5/22/2012 5:39:16 PM

Chronic Pancreatitis

REFERENCES
1. Comfort MW, Gambrill EE, Baggenstoss AH. Chronic relapsing pancreatitis. A study of twenty-nine cases without associated
disease of the biliary or gastro-intestinal tract. Gastroenterology.
1968;54:706-705.
2. Whitcomb DC. Hereditary pancreatitis: New insights into acute
and chronic pancreatitis. Gut. 1999;45:317-322.
3. Whitcomb DC, Gorry, MC, Preston RA, et al. Hereditary pancreatitis is caused by a mutation in the cationic trypsinogen gene.
Nat Genet. 1996;14:141-145
4. Gorry MC, Gabbaizedeh D, Furey W, et al. Mutations in the cationic trypsinogen gene are associated with recurrent acute and
chronic pancreatitis. Gastroenterology. 1997;113:1063-1068.
5. Witt H, Luck W, Becker M. A signal peptide cleavage site mutation in the cationic trypsinogen gene is strongly associated with
chronic pancreatitis. Gastroenterology. 1999;117:7-10.
6. Rebours V, Boutron-Ruault MC, Schnee M, et al. The natural
history of hereditary pancreatitis: A national series. Gut. 2009;
58:97-103.
7. Santhosh S, Witt H, te Morsche RH, et al. A loss of function polymorphism (of anionic trypsinogen (PRSS2) confers protection
against chronic pancreatitis. Pancreas. 2008;36:317-320.
8. Auoun E, Chang CC, Greer JB, et al. Pathways to injury in chronic
pancreatitis: Decoding the role of the high-risk SPINK1 N34S
haplotype using meta-analysis. PLoSONE. 2008;3:2003.
9. Audrezet MP, Dabricot A, Le Marechal C, et al. Validation of
high resolution DNA melting analysis for mutation scanning of
the cystic fibrosis transmembrane conductance regulator (CFTR)
gene. J Mol Diagn. 2008;10:424-434.
10. Rosendahl J, Witt H, Szmola R, et al. Chymotrypsin C (CTRC)
variants that diminish activity or secretion are associated with
chronic pancreatitis. Nat Genet. 2008;40:78-82.
11. Masson E, Chen JM, Scotet V, et al. Association of rare chymotrypsinogen C (CTRC) gene variation in patients with idiopathic chronic pancreatitis. Hum Genet. 2008;123:83-91.
12. Maddana V, Lamb J, Greer JB, et al. Association between calcium
sensing receptor gene polymorphisms and chronic pancreatitis
in a US population. World J Gastroenterol. 2008;14:4486-4491.
13. Anderson RJ, Dyer PA, Donnai D, Klouda PT, Jennison R, Braganza JM. Chronic pancreatitis, HLA, and autoimmunity. Internat J Pancreatol. 1988;3:83-90.
14. Bachem MG, Zhou Z, Zhou S, Siech M. Role of stellate cells in
pancreatic fibrogenesis associated with acute and chronic pancreatitis. J Gastroenterol Hepatol. 2006;21(Supp 3):S92-S96.
15. Demir IE, Tieft runk E, Maak M, Friess H, Ceyhan GO. Pain
mechanisms in chronic pancreatitis: Of a master and his fire.
Langenbecks Arch Surg. 2011;396:151-160.
16. Puestow CB, Gillesby WJ. Retrograde surgical drainage of pancreas for chronic relapsing pancreatitis. Arch Surg. 1958;76:
898-907.
17. Karanjia ND, Widdison AL, Leung F, et al. Compartment syndrome in experimental chronic obstructive pancreatitis: Effect
of decompressing the main pancreatic duct. Br J Surg. 1994;81:
259-264.
18. Keith RG, Keshavjee SH, Kerenyi NR. Neuropathology of chronic
pancreatitis in humans. Can J Surg. 1985;28:207-211.
19. Bockman DE, Buchler M, Malfertheiner P, Beger GH. Analysis of nerves in chronic pancreatitis. Gastroenterology. 1988;94:
1459-1469.
20. Ceyhan GO, Bergmann F, Kadihasanoglu M, et al. The neurotrophic factor artemin influences the extent of neural damage
and growth in chronic pancreatitis. Gut. 2007;56:534-544.

PMPH_CH56.indd 455

455

21. Friess H, Zhu ZW, di Mola FF, et al. Nerve growth factor and its
high affi nity receptor in chronic pancreatitis. Ann Surg. 1999;230:
615-624.
22. Zhu ZW, Friess H, Wang L, Zimmermann A, Buchler MW. Brain
derived neurotrophic factor is upregulated and associated with
pain in chronic pancreatitis. Dig Dis Sci. 2001;46:1633-1639.
23. Buchler M, Weihe E, Friess H, et al. Changes in peptidergic innervation in chronic pancreatitis. Pancreas. 1992;7:183-192.
24. Michalski CW, Shi X, Reiser C, et al. Neurokinin-2 receptor levels correlate with intensity, frequency, and duration of pain in
chronic pancreatitis. Ann Surg. 2007:246:786-793.
25. Shrikhande SV, Friess H, Di Mola FF, et al. NK-1 receptor gene
expression is related to pain in chronic pancreatitis. Pain. 2001;91:
209-217.
26. Ceyhan GO, Demir IE, Rauch U, et al. Pancreatic neuropathy
results in neural remodeling and altered pancreatic innervation in chronic pancreatitis and pancreatic cancer. Am J Gastroenterol. 2009;104:2555-2565.
27. Dite P, Ruzicka M, Zboril V, Novotny I. A prospective randomized trial comparing endoscopic and surgical therapy for chronic
pancreatitis. Endoscopy. 2003;35:553-558.
28. Cahen DL, Gouma DJ, Nio Y, et al. Endoscopic versus surgical
drainage of the pancreatic duct in chronic pancreatitis. NEJM.
2007;356:676-684.
29. Rios GA, Adams DB, Yeoh KG, Tarnasky PR, Cunningham JT,
Hawes RH. Outcome of lateral pancreaticojejunostomy in the
management of chronic pancreatitis with nondilated pancreatic
ducts. J Gastrointes Surg. 1998;2:223-229.
30. Partington PF, Rochelle RE. Modified Puestow procedure for
retrograde drainage of the pancreatic duct. Ann Surg. 1960;152:
1037-1048.
31. Nealon WH, Matin S. Analysis of surgical success in preventing
recurrent acute exacerbations in chronic pancreatitis. Ann Surg.
2001;233:793-800.
32. Greenlee HB, Prinz RA, Aranha GV. Longterm results of sideto-side pancreaticojejunostomy. World J Surg. 1990;14:70-76.
33. Sato T, Miyashita E, Yamauchi H, Matsuno S. The role of surgical
treatment for chronic pancreatitis. Ann Surg. 1986;203:266-271.
34. Bradley EL. Long term results of pancreaticojejunostomy in
patients with chronic pancreatitis. Am J Surg. 1987;153:207-213.
35. Holmberg JT, Isaksson G, Ihse I. Longterm results of pancreaticojejunostomy in chronic pancreatitis. Surg Gynecol Obstet.
1985;160:339-346.
36. Sarles JC, Nacchiero M, Garani F, Salasc B. Surgical treatment of
chronic pancreatitis. Report of 134 cases treated by resection or
drainage. Am J Surg. 1982;144:317-321.
37. Adams DB, Ford MC, Anderson MC. Outcome after lateral
pancreaticojejunostomy for chronic pancreatitis. Ann Surg.
1994;219:481-489.
38. Schnelldorfer T, Lewin DN, Adams DB. Operative management
of chronic pancreatitis: Longterm results in 372 patients. JACS.
2007;204:1039-1045.
39. Nealon WH, Thompson JC. Progressive loss of pancreatic function in chronic pancreatitis is delayed by man pancreatic duct
decompression. A longitudinal prospective analysis of the modified puestow procedure. Ann Surg. 1993;217:458-468.
40. Rios G, Adams DB. Does intraoperative EHL improve outcome
in the surgical management of chronic pancreatitis? Am Surg.
2001;67:534-538.
41. Frey CF, Smith GJ. Description and rationale of a new operation
for chronic pancreatitis. Pancreas. 1987;2:701-707.
42. Frey CF, Amikura K. Local resection of the head of the pancreas combined with longitudinal pancreaticojejunostomy in

5/22/2012 5:39:16 PM

456

43.

44.
45.

46.

47.

48.

49.
50.

51.
52.

53.

54.

55.

56.

57.

58.

59.

60.

Surgery: Evidence-Based Practice

the management of patients with chronic pancreatitis. Ann Surg.


1994;220:492-504.
Keck T, Wellner UF, Riediger H, et al. Long term outcome after
92 duodenum preserving pancreatic head resections for chronic
pancreatitis: Comparison of Beger and Frey procedures. J Gastrointest Surg. 2010;14:549-556.
Negi S, Singh A, Chaudhary A. Pain relief after Freys procedure
for chronic pancreatitis. Br J Surg. 2010;97:187-195.
Pessaux P, Kianmanesh R, Regimbequ JM, et al. Frey procedure in the treatment of chronic pancreatitis. Pancreas. 2006;33:
354-358.
Whipple AO. Radical surgery for certain cases of pancreatic
fibrosis associated with calcareous deposits. Ann Surg. 1946;124:
991-1006.
Sakorafas GH, Farnell, MB, Nagorney DM, et al. Pancreatoduodenctomy for chronic pancreatitis: Long term results in 105 patients.
Arch Surg. 2000;135:517-523.
Jimenez RE, Fernandez-Del Castillo C, Rattner DW, et al. Pylorus preserving pancreaticoduodenectomy in the treatment of
chronic pancreatitis. World J Surg. 2003;27:1211-1216.
Russell RC, Theis BA. Pancreatoduodenectomy in the treatment
of chronic pancreatitis. World J Surg. 2003;27:1203-1210.
Vickers SM, Chan C, Heslin MJ, et al. The role of pancreaticoduodenectomy in the treatment of severe chronic pancreatitis. Am
Surg. 1999;65:1108-1111.
Traverso LW, Longmire WP, Jr. Preservation of the pylorus in pancreaticoduodenectomy. Surg Gynecol Obstet. 1978;146:959-962.
Tran KT, Smeenk HG, van Eijck CH, et al. Pylorus preserving
pancreaticoduodenectomy versus standard Whipple procedure:
A prospective, randomized, multicenter analysis of 170 patients
with pancreatic and periampullary tumors. Ann Surg. 2004;
240:738-745.
Seiler CA, Wagner M, Bachmann T, et al. Randomized clinical trial
of pylorus-preserving duodenopancreatectomy versus classical
Whipple resectionlong term results. Br J Surg. 2005;92:547-556.
Iqbal N, Lovegrove RE, Tilney HS, et al. A comparison of pancreaticoduodenectomy with pylorus preserving pancreaticoduodenectomy: A meta-analysis of 2822 patients. EJSO. 2008;34:
1237-1245.
Ohtsuka T, Yamaguchi K, Ohuchida J, et al. Comparison of quality of life after pylorus preserving pancreatoduodenectomy and
Whipple resection. Hepatogastroenterology. 2003;50:846-850.
Beger HG, Krautzberger W, Bittner R, Buchler M, Limmer J.
Duodenum-preserving resection of the head of the pancreas
in patients with severe chronic pancreatitis. Surgery. 1985;97:
467-475.
Beger HG, Buchler M, Bittner RR, Oettinger W, Roscher R.
Duodenum-preserving resection of the head of the pancreas in
severe chronic pancreatitis. Early and late results. Ann Surg. 1989;
209:273-278.
Beger HG, Schlosser W, Friess HM, Buchler MW. Duodenum preserving head resection in chronic pancreatitis changes the natural course of the disease: A single-center 26 year experience. Ann
Surg. 1999;230:512-519.
Buchler MW, Friess H, Bittner R, et al. Duodenum preserving
pancreatic head resection: Long term results. J Gastrointest Surg.
1997;1:13-19.
Klempa I, Spatny M, Menzel J, et al. Pancreatic function and quality of life after resection of the head of the pancreas in chronic
pancreatitis. A prospective, randomized comparative study after
duodenum preserving resection of the head of the pancreas versus
Whipples operation. Chirurg. 1995;66:350-359.

PMPH_CH56.indd 456

61. Buchler MW, Friess H, Muller MW, et al. Randomized trial of


duodenum preserving pancreatic head resection versus pylorus
preserving Whipple in chronic pancreatitis. Am J Surg. 1995;
169:65-69.
62. Muller MW, Friess H, Leitzbach S, et al. Perioperative and follow
up results after central pancreatic head resection in a consecutive series of patients with chronic pancreatitis. Am J Surg. 2008;
196:364-372.
63. Farkas G, Leindler L, Daroczi M, Farkas G. Prospective randomized comparison of organ preserving pancreatic head resection
with pylorus preserving pancreaticoduodenectomy. Langenbecks
Arch Surg. 2006;391:338-342.
64. Izbicki JR, Bloechle C, Knoefel WT, et al. Duodenum preserving
resection of the head of the pancreas in chronic pancreatitis. A
prospective randomized trial. Ann Surg. 1995;221:350-358.
65. Izbicki JR, Bloechle C, Broering DC, et al. Extended drainage
versus resection in surgery for chronic pancreatitis: A prospective randomized trial comparing the longitudinal pancreaticojejunostomy combined with local pancreatic head excision with
the pylorus preserving pancreatoduodenectomy. Ann Surg. 1998;
228:771-779.
66. Strate T, Taherpour Z, Bloechle C, et al. Long term follow up
of a randomized trial comparing the Beger and Frey procedures
for patients suffering from chronic pancreatitis. Ann Surg. 2005;
241:591-598.
67. Hutchins RR, Hart RS, Pacifico M, et al. Long term results of distal pancreatectomy for chronic pancreatitis in 90 patients. Ann
Surg. 2002;236:612-618.
68. Schoenberg MH, Schlosser W, Ruck W, Beger HG. Distal pancreatectomy in chronic pancreatitis. Dig Surg. 1999;16:130-136.
69. Sakorafas GH, Sarr MG, Rowland CM, Farnell MB. Postobstructive chronic pancreatitis: Results with distal resection. Arch Surg.
2001;136:643-648.
70. Schnelldorfer T, Mauldin PD, Lewin DN, Adams DB. Distal pancreatectomy for chronic pancreatitis: Risk factors for postoperative pancreatic fistula. J Gastrointest Surg. 2007;11:991-997.
71. Vijan SS, Ahmed KA, Harmsen WS, et al. Laparoscopic vs open distal pancreatectomy: A single institution comparative study. Arch
Surg. 2010;145:16
72. Kooby DA, Gillespie T, Bentrem D, et al. Left-sided pancreatectomy:
A multicenter comparison of laparoscopic and open approaches.
Ann Surg. 2008;248(3):438-446.
73. Riediger H, Adam U, Fischer E, et al. Long-term outcome after resection for chronic pancreatitis in 224 patients. J Gastrointest Surg.
2007;11:949-960.
74. Gruessner RWG, Sutherland DER, Dunn DL, Najarian JS. Transplant options for patients undergoing total pancreatectomy for
chronic pancreatitis. JACS. 2004;198:559-567.
75. Dresler CM, Fortner JG, McDermott K, Bajorunas DR. Metabolic
consequences of regional (total) pancreatectomy. Ann Surg. 1991;
214:131-140.
76. Billings BJ, Christein JD, Hermsen WS, et al. Quality of life after
total pancreatectomy: Is it really that bad on long term follow up?
J Gastrointest Surg. 2005;9:1059-1066.
77. Rickles MR, Schutta MF, Markmann JF, Barker CF, Naji A, Teff
KL. Beta cell function following human islet transplantation for
type I diabetes. Diabetes. 2005;54:100-106.
78. Sutherland DER, Matas AJ, Najarian JS. Pancreatic islet cell
transplantation. Surg Clin North Am. 1978;58:365-382.
79. Argo JL, Contreras JL, Wesley MM, Christein JD. Pancreatic resection with islet cell autotransplant for the treatment of severe chronic
pancreatitis. Am Surg. 2008;74:530-536.

5/22/2012 5:39:16 PM

Chronic Pancreatitis

80. Garcea G, Weaver J, Phillips J, et al. Total pancreatectomy with


and without islet cell transplantation for chronic pancreatitis. A
series of 85 consecutive patients. Pancreas. 2009;38:1-7.
81. Jie T, Hering B, Ansite J, Gilmore T. Pancreatectomy and auto
islet transplantation in patients with chronic pancreatitis. JACS.
2005;201:S14.
82. Rilo HR, Ahmad SA, DAlessio D. Total pancreatectomy and
autologous islet cell transplant as a means to treat severe chronic
pancreatitis. J Gastrointest Surg. 2003;7:978-989.
83. Robertson RP, Lanz KJ, Sutherland DER, Kendall DM. Prevention
of diabetes for up to 13 years by autoislet transplantation after pancreatectomy for chronic pancreatitis. Diabetes. 2001;50:47-53.

PMPH_CH56.indd 457

457

84. Gagner M, Pomp A, Herrera MF. Early experience with laparoscopic resections of islet cell tumors. Surgery. 1996;120:1051-1054.
85. Tantia O, Jindal MK, Khanna S, Sen B. Laparoscopic lateral pancreaticojejunostomy: Our experience of 17 cases. Surg Endosc. 2004;
18:1054-1057.
86. Palanivelu C, Rajan RS, Rangarajan M, Vaithiswaran V. Evolution in techniques of laparoscopic pancreaticoduodenectomy. A
decade long experience from a tertiary center. J HBP Surg. 2009;
16:731-740.
87. Kendrick ML, Cusati D. Total laparoscopic pancreaticoduodenectomy: Feasibility and outcome in an early experience. Arch Surg.
2010;145:19-23.

5/22/2012 5:39:16 PM

CHAPTER 57

Pancreatic Adenocarcinoma
Jamii St. Julien, Alexander A. Parikh, and Nipun B. Merchant

INTRODUCTION

providing high-image resolution and fast image acquisition. The


optimal protocol for imaging includes injection of 100120 mL
of nonionic iodinated contrast, with image acquisition in both an
arterial and a venous phase to visualize the relationship of the primary tumor to the mesenteric vasculature, as well as the detection of metastatic deposits as small as 35 mm. The addition of
CT angiography with 3D reconstruction of the mesenteric vessels has been shown to markedly improve the ability to predict
resectability.5

Pancreatic cancer remains a major healthcare problem. In 2010,


there were an estimated 43,140 new cases and 36,800 deaths
from pancreatic cancer, making it the 10th most common cause of
cancer and the 4th most common cause of cancer-related death in
the United States.1 Over the past four decades, we have made minimal impact on survival outcomes for this disease, with overall
1- and 5-year relative survival rates for all stages combined being
25% and 6%, respectively.
The nonspecific symptoms associated with early pancreatic
cancer and the aggressiveness of the tumors make it one of the most
challenging diseases to treat. Numerous studies have attempted to
guide treatment strategies and improve outcomes. However, due
to the lack of standardized approaches for diagnosis, staging, surgery, and multimodality therapy, it has been challenging to interpret data from clinical trials in a meaningful way.
This chapter will discuss pretreatment assessment and
combined-modality therapy for pancreatic cancer through a
review of the current data and with the goal of clarifying and
standardizing the important components of pancreatic cancer
management.

ENDOSCOPIC ULTRASONOGRAPHY
Endoscopic ultrasound (EUS) is accurate for local tumor (T) staging and in predicting vascular invasion of the tumor. Compared
with CT imaging, EUS has been found to be as accurate in determining resectability, and is more sensitive for detecting vascular
invasion.5,6 This assessment may, however, be less accurate when
a biliary stent is present.7 The accuracy of this staging modality
depends largely upon the experience of the operator and results
may vary. In addition, EUS is not routinely available at all hospitals and therefore has not been widely adopted as a routine test for
staging pancreatic cancer.
However, EUS is an essential tool for patients enrolled in neoadjuvant therapy trials because it provides relatively easy access to
the pancreas for tissue diagnosis by fine-needle aspiration (FNA).
Lymph nodes, liver lesions, and ascites can all be sampled via
EUS-FNA.
The three modalities most commonly utilized for preoperative
tissue diagnosis are endoscopic retrograde cholangiopancreatography (ERCP) with cytologic brushing, image-guided percutaneous FNA, or EUS-guided FNA. Of these, EUS-FNA has the highest
sensitivity (96%), specificity (100%), and diagnostic accuracy (98%)
for pancreatic cancer (especially for tumors 25 mm), and has
been shown to be the most cost-effective approach.8-10 If indicated,
celiac ganglion blockade can also be performed under EUS guidance for pain control. In patients requiring stent placement, EUS
and ERCP can be performed during the same sedation.

DIAGNOSIS
1. What is the optimal pretreatment staging assessment of
patients with pancreatic adenocarcinoma?

COMPUTED TOMOGRAPHY
Accurately staging and selecting the appropriate patients for surgical
therapy is crucial to minimize nontherapeutic surgical exploration.
Computed tomography (CT) remains the primary method for staging pancreatic malignancy and determining tumor resectability.2-4
The current state-of-the-art system uses a 64-slice multidetector CT scanner, which allows for very thin (35 mm) collimation,
458

PMPH_CH57.indd 458

5/22/2012 5:39:55 PM

Pancreatic Adenocarcinoma

Diagnostic adjuncts, such as magnetic resonance imaging


(MRI) and positron emission tomography (PET) scanning, may
play a role in select instances when results from CT or EUS are
not definitive.3

STAGING LAPAROSCOPY
The major limitation of CT and EUS is their low sensitivity for
detecting occult liver metastases or peritoneal implants, which
may lead to unnecessary laparotomy. The value of staging laparoscopy (SL) remains controversial. Proponents argue that SL is quick
and efficient, does not affect subsequent resection, and imparts
minimal morbidity. Further supporting the use of SL, hospital
stays, costs, and morbidity are reduced when unnecessary laparotomies are avoided.11 Opponents argue that with the current high
quality of noninvasive imaging, only a small minority of patients
actually benefits, and the cited value of SL is overestimated.
A comprehensive review pointed out that the literature on SL
for pancreatic cancer was difficult to interpret due to the inconsistent use of high-quality CT, poor definitions of surgical resectability, inclusion of nonpancreatic periampullary malignancies
and benign disease, inclusion of patients with locally advanced
disease, and poor analysis of surgical margins after resection,
making it impossible to correlate laparoscopic findings with R0/
R1/R2 resection rates.12 However, they concluded that due to the
high accuracy of CT imaging for predicting resectability based on
objective criteria, the proportion of potentially resectable patients
in whom SL may prevent unnecessary laparotomy is only 4% to
13%. Based on the results of this study and two subsequent retrospective analyses, selective use of SL at the time of planned laparotomy for potentially resectable patients should be reserved for
patients at higher risk for occult metastatic disease. These include
patients with (1) primary tumors >3 cm, (2) lesions in the neck,
body, or tail, (3) equivocal radiographic findings suggestive of
occult M1 disease (low volume ascites, possible carcinomatosis,
small liver lesions not amenable to biopsy), or (4) CA 19-9 levels
>100 U/mL.12-14
SL may be beneficial in patients with locally advanced disease
without evidence of distant metastases by CT imaging by helping
to determine the need for radiation therapy. In this group, up to
34% of patients have been found to have occult metastatic disease
by SL, thereby avoiding the inappropriate use of radiotherapy.15
Answer: CT imaging with a specific pancreas protocol with
thin cuts (35 mm) through the pancreas is the optimal preoperative diagnostic and staging tool to determine resectability in
all patients with suspected pancreatic adenocarcinoma. Objective
criteria to determine resectability based on CT imaging should
be utilized (see next section). For potentially resectable pancreatic
adenocarcinoma, SL may be used selectively in patients considered
high risk for occult metastatic disease: (1) primary tumors >3 cm,
(2) all tumors of the neck, body, and tail, (3) equivocal findings on
MDCT, (4) or high CA 19-9 levels (>100 U/mL). SL may also be used
in patients with locally advanced (Stage III) pancreatic adenocarcinoma without evidence of distant metastases on CT imaging to rule
out occult metastatic disease and avoid unnecessary radiotherapy.
Patients being considered for neoadjuvant therapy require a
preoperative tissue diagnosis. EUS-FNA is the best modality for
obtaining a tissue diagnosis, and can add additional staging information to CT imaging.

PMPH_CH57.indd 459

459

2. What defines resectable, borderline resectable, and locally


advanced pancreatic adenocarcinoma?
Most patients with pancreatic adenocarcinoma present with advanced
disease at the time of diagnosis, and only 15% to 20% are candidates
for potentially curative resection.16-18 Surgical resection remains the
only potentially curative option. Therefore, accurate disease staging
and selection of appropriate patients for surgical therapy are crucial
to maximizing the benefits of surgery and minimizing nontherapeutic surgical explorations.
The definition of resectable disease has been an area of controversy. Several efforts have been made to utilize cross-sectional
imaging to objectively classify pancreatic adenocarcinoma as resectable, borderline resectable, or locally advanced, unresectable. Each
classification emphasizes the distinction between potentially resectable tumors, where a negative margin resection can reasonably
be expected, and locally advanced and metastatic disease, where
a curative resection is unlikely. These definitions are based upon
properly performed CT imaging (as detailed above).
Varadhachary et al.19,20 from MD Anderson Cancer Center
(MDACC), in an attempt to standardize inclusion criteria into
clinical trials for neoadjuvant chemotherapy, proposed the following definitions (of tumors without evidence of metastatic disease): resectable tumors (Stage I/II) were those with normal tissue
planes between the tumor and the superior mesenteric artery
(SMA) and celiac axis, and a patent superior mesenteric vein
(SMV)/portal vein (PV) confluence; locally advanced, unresectable tumors (Stage III) were those with encasement (>180 of the
vessel circumference) of the SMA or the common hepatic artery,
or an occluded SMV/PV confluence with no technical option
for reconstruction; borderline resectable tumors were defined as
those with tumor abutment (180 of the circumference) of the
SMA, abutment or short-segment encasement of the common
hepatic artery, or short-segment occlusion of the SMV/PV confluence with patent vessel above and below the occlusion suitable for
venous reconstruction.19,20
The National Comprehensive Cancer Network (NCCN)
defines a tumor in the head or in the body of the pancreas as borderline resectable if there is abutment (i.e., 180) of the SMA,
reconstructible abutment or encasement of the hepatic artery,
severe impingement of the PV and/or SMV, or reconstructible
SMV occlusion.21
In the 2009 American Hepato-Pancreato-Biliary Association
(AHPBA)/Society of Surgical Oncology (SSO) Consensus Report
on pretreatment assessment of resectable and borderline resectable pancreatic cancers, borderline resectable tumors differ from
potentially resectable tumors anatomically by having either venous
involvement (abutment with or without impingement and narrowing of the SMV) or limited involvement of the gastroduodenal
artery, hepatic artery, or SMA.22
Answer: Tumor resectability must be based on objective criteria from multidetector CT imaging. Resectable disease should
be arrowly defined as the absence of tumor extension to the SMA,
celiac, or common hepatic arteries, and the absence of any tumorinduced unilateral shift or narrowing of any aspect of the PVSMV confluence. 23,24 A surgery fi rst approach is acceptable in
these patients.
Borderline resectable disease is defined as tumors with abutment (180 of the circumference) of the SMA, abutment or shortsegment encasement of the common hepatic artery, tumor-induced
unilateral shift or narrowing of the PV-SMV, or short-segment

5/22/2012 5:39:55 PM

460

Surgery: Evidence-Based Practice

occlusion of the SMV/PV confluence with patent vessel above


and below the occlusion suitable for venous reconstruction.20,22,23
These patients should be treated with neoadjuvant therapy prior
to surgery, preferably within a clinical trial to enhance the possibility of a negative margin resection.
Locally advanced tumors are those with encasement (>180
of the circumference) of the SMA or common hepatic artery, or
an occluded SMV/PV confluence with no technical option for
reconstruction.20,21

MANAGEMENT
3. What is the role of adjuvant therapy after resection for pancreatic adenocarcinoma?
Despite attempted curative resection, the majority of patients with
pancreatic adenocarcinoma will recur and ultimately die of their
disease, presumably due to the presence of micrometastatic disease
present at the time of diagnosis. As a result, significant research has
been devoted to evaluating adjuvant treatment strategies including systemic chemotherapy, radiation therapy, and combined
chemoradiation therapy (CRT).

HISTORICAL PERSPECTIVE
In the United States, the standard of care for adjuvant therapy
in pancreas cancer has been based largely on a number of relatively small trials, the first being the Gastrointestinal Tumor
Study Group (GITSG) trial published in 1985. This small trial
of 49 patients showed a survival benefit for fluorouracil (5-FU)
based CRT followed by 5-FU chemotherapy versus surgery alone
(median survival 20 vs. 11 months, p = .035) and established adjuvant CRT and chemotherapy as the standard of care for resected
pancreas cancer.25
Unfortunately, most subsequent trials have been limited
due to size or other methodologic problems. The European
Organization for Research and Treatment of Cancer (EORTC)
trial published in 1999 consisted of 218 patients with pancreatic
and periampullary tumors randomized to adjuvant 5-FUbased
CRT alone or observation.26 Although there was a trend for survival in the adjuvant group, for the entire cohort, as well as for
the subset with pancreatic cancer, this did not reach statistical
significance.

MODERN TRIALS
Since that time, several large studies have been published in hopes
of better defining the role of adjuvant therapy in pancreatic cancer.
The European Study Group of Pancreas Cancer (ESPAC-1) trial
published their complex trial over several years starting in 1997.
This trial utilized a 2 2 factorial design to compare adjuvant
CRT, adjuvant chemotherapy (6 mos), both of these, or observation.27 The median survival for the chemotherapy and no-chemotherapy groups was 20.1 and 15.5 months, respectively (p = .009),
with 5-year survival rates of 21% and 8%, respectively (p = .05). The
median survival for the CRT and no CRT groups was 16 and 18
months, respectively (p = .05), with 5-year survival rates of 10%

PMPH_CH57.indd 460

and 20%, respectively (p = .05). The authors concluded that adjuvant chemotherapy was beneficial, whereas adjuvant CRT was
actually detrimental. Criticisms of this trial include possible suboptimal CRT regimen, poor compliance, a lack of quality control,
and the complex 2 2 design of the trial. Despite these limitations,
this trial helped establish a clear role for systemic chemotherapy in
the adjuvant setting, but introduced controversy regarding the role
of radiation therapy.
In 2009, composite data from the ESPAC-1, ESPAC-1 Plus,
and the ESPAC-3(v1) trial were reported, which recapitulated the
results reported in 2004.28 The ESPAC-1 Plus trial was a cohort of
192 patients from ESPAC-1 that were entered into a direct randomized comparison between 5-FU and observation alone (with
clinicians choice of background CRT if indicated). The ESPAC3(v1) trial was initially a three-arm study of adjuvant 5-FU versus
gemcitabine versus observation alone. Following the publication
of the results of the larger ESPAC-1 trial, 27 the observation
arm was dropped. The eligibility criteria and treatment schedules were identical for all three studies. A meta-analysis of the
pooled data again showed the benefit of adjuvant chemotherapy,
with a superior overall survival in patients randomized to 5-FU
compared with observation (hazard ratio [HR] = 0.70, p = .003).
The composite 2- and 5-year survival rates were 49% and 24%
for the 5-FU arm, compared with 37% and 14% for the observation arm. 28 A direct comparison between the gemcitabine and
5-FU arms was recently reported as well and showed no difference in the median overall survival (23.6 vs. 23 months, respectively; p = .39).
The CONKO-001 trial compared adjuvant gemcitabine chemotherapy (six cycles) to observation alone in 368 patients.29 Estimated disease-free survival was 13.4 months in the gemcitabine
arm compared with 6.9 months in the control arm (p = .001),
regardless of margin status, tumor size, or nodal involvement.
Updated results presented at the American Society of Clinical
Oncology (ASCO) meeting in 2008 showed estimated 5-year survival rates of 21% in the gemcitabine arm, and 9% in observation
arm (p = .005), further establishing the role of adjuvant gemcitabine chemotherapy in resected pancreatic cancer.30
The Radiation Therapy Oncology Group (RTOG 97-04) trial
was a large intergroup trial of 451 patients randomized to either
5-FU- or gemcitabine-based chemotherapy followed by identical
5-FUbased CRT regimens. The final analysis revealed no difference in overall or disease-free survival between the treatment
groups.31 A subgroup analysis of patients with tumors in the head
of the pancreas showed a nonsignificant trend toward improved
survival with gemcitabine, with a median survival of 20.5 months
and a 3-year survival rate of 31% in the gemcitabine group versus
16.9 months and 22% in the 5-FU group (HR = 0.82, p = .09). However, after adjusting for prespecified stratification variables of nodal
status, tumor diameter, and surgical margin status, the treatment
effect yielded an HR of 0.80 (p = .05). Although this trial was well
designed and adequately powered, it did not clearly demonstrate an
advantage for gemcitabine over 5-FU in the adjuvant setting, and
certainly did not address the more pressing issue of whether or not
CRT added any benefit to adjuvant chemotherapy.
A new EORTC randomized Phase II trial (40013) comparing
gemcitabine alone to gemcitabine with radiation using modern
CRT techniques was reported at the 2009 ASCO annual meeting.
Ninety patients were randomized to either chemotherapy or CRT.
There was a significant improvement in the local control with the

5/22/2012 5:39:55 PM

Pancreatic Adenocarcinoma

use of CRT, but the overall survival was the same for both arms
(24 months).32

RETROSPECTIVE STUDIES
Several retrospective reviews have also attempted to investigate the
role of adjuvant therapy in pancreatic cancer. Although they suffer
from the inherent biases of retrospective studies, they do provide
additional evidence for the use of adjuvant therapy in resected
pancreatic cancer. A study from the Mayo Clinic showed a significant survival benefit with the use of adjuvant CRT.33 In this study,
274 patients received adjuvant CRT, most receiving concomitant
5-FU, whereas 180 were observed after surgical resection. Radiation doses varied widely from 13.8 to 60 Gy. Those receiving CRT
had improved median survival (25.2 vs. 19.2 months, p = .001)
with improved 2- and 5-year overall survival.
Another review from Johns Hopkins Hospital compared 271
patients who received adjuvant 5-FUbased CRT with 345 patients
who underwent surgery alone.34 Patients who received CRT had an
improved median, 2- and 5-year survival compared with surgery
alone (21.2 vs. 14.4 months; 43.9% vs. 31.9%, and 20.1% vs. 15.4%,
respectively, for adjuvant CRT vs. no CRT; p < .001). The beneficial
effect of CRT remained even after adjustment for prognostic factors including tumor size, grade, margin, and nodal status. A subset analysis suggested that lymph node-positive patients showed a
benefit, whereas node-negative patients did not.
Finally, a recent retrospective, multicenter study from the
Central Pancreas Consortium (CPC) compared 374 patients who
had surgery alone with 299 patients who underwent adjuvant
CRT (predominantly 5-FUbased).35 Patients receiving CRT after
surgery had significantly improved survival (20 vs. 14.5 months,
p = .001). On subset and multivariate analysis, patients with lymph
node-positive disease had a significantly improved survival with
adjuvant CRT (19.4 vs. 10.4 months, p < .01), whereas patients that
were lymph node-negative did not show any benefit from CRT
(22.9 vs. 24.2 months, p = .774) regardless of margin status (R0
or R1).
Answer: Randomized trials have clearly established a role for
adjuvant systemic chemotherapy in the treatment of resected pancreatic cancer. Both adjuvant 5-FU and gemcitabine have led to a
survival advantage compared with surgery alone, though neither
agent has conclusively been shown to be superior. The role of adjuvant radiation therapy is less clear. Although randomized trials
have failed to show a clear benefit, well-designed retrospective studies have suggested a role for adjuvant CRT in high-risk patients
that is, patients with lymph node-positive disease. Well-designed
randomized trials are therefore needed to better establish the
role of adjuvant CRT. Furthermore, as these trials are designed,
it will be important to stratify patients by high-risk features such
as lymph node status to better define which patients may benefit
the most.
4. What is the role of neoadjuvant therapy in the management
of pancreatic adenocarcinoma?
Another option in the treatment of resectable pancreatic cancer
is systemic and locoregional therapy prior to resection. The use of
neoadjuvant therapy has several potential advantages over adjuvant strategies including (1) downsizing of the tumor leading to

PMPH_CH57.indd 461

461

improved margin-negative resection and local recurrence rates;


(2) the ability to identify patients who would most benefit from
pancreatectomy and avoid unnecessary surgery in patients with
progressive disease; (3) the ability to deliver treatment to better
perfused and oxygenated tissues prior to surgical manipulation,
thereby enhancing the effects of chemotherapy and/or radiation;
(4) improved patient tolerance and a decrease in complication
rates; and (6) the early delivery of systemic therapy to treat occult
metastases.
The use of neoadjuvant therapy also has potential disadvantages. For example, to receive neoadjuvant therapy, a pathologic
confirmation of pancreatic adenocarcinoma is required. This often
requires invasive techniques such as EUS-FNA, which is not without risk, highly operator-dependent, and not available at all hospitals. In addition, patients who are jaundiced require preoperative
biliary drainage such as stenting prior to receiving chemotherapy.
The use of preoperative stents, however, has been associated with
increased perioperative complications.36,37 Also, ineffective neoadjuvant strategies may lead to local progression of an initially
resectable tumor.

HISTORICAL PERSPECTIVE
The first reported trials utilizing neoadjuvant therapy for resectable pancreatic cancer were published in the early 1990s. Initial
experience involved the use of 5-FUbased CRT. Most patients
tolerated these regimens well, and approximately 60% ultimately
underwent successful resection with median survival of approximately 2 years.38-40 It is important to note that the reported survival
was only for the patients who subsequently underwent successful
resection. Two other trials included neoadjuvant regimens of continuous infusion (CI) 5-FU, mitomycin C (MMC), and concurrent
radiotherapy.39,41 They both included patients with both resectable
and locally advanced disease, and one also included duodenal as
well as pancreatic cancer. Their results were unimpressive, but
these studies highlighted the importance of designing trials that
separate patients by well-defined resectability criteria as discussed
earlier.

MODERN TRIALS
A French single-institution trial of 101 patients with resectable
or borderline resectable pancreatic adenocarcinoma undergoing
neoadjuvant 5-FU-cisplatin-CRT reported a resectability rate
of 62% (92% R0) and a median survival of 23 months for those
who underwent resection versus 11 months for those who did not
(p = .002).42 This group has also published a Phase II trial of 34
patients with resectable or borderline resectable pancreatic adenocarcinoma who underwent docetaxel-based CRT. This regimen
resulted in a 50% resectability rate (100% R0) and a median survival of 32 months for those resected compared with 11 months
for those who were not (p < .001). 43
Gemcitabine has also been used in combination with radiation in the neoadjuvant setting. The group at MDACC published a
Phase II trial of preoperative gemcitabine-based CRT in 86 patients
with Stage I/II pancreatic head adenocarcinoma. Although 41%
required hospital admission, 74% were successfully resected (89%

5/22/2012 5:39:55 PM

462

Surgery: Evidence-Based Practice

R0). Of the patients whose disease progressed, all developed metastatic disease rather than local progression. The median survival for
resected patients was 34 months, compared with 7 months for those
who were not resected (p < .001). The 5-year survival was 36% and
0% for those who did and did not undergo resection, respectively.44
These results led to gemcitabine-based CRT being considered the
preferred neoadjuvant approach for pancreatic cancer.
The MDACC group also reported on the addition of systemic cisplatin to the gemcitabine-based chemoradiation regimen in a Phase II trial of 90 patients with Stage I/II pancreatic
adenocarcinoma. A total of 79 patients (88%) completed the preoperative regimen and 52 (58%) were successfully resected (96%
R0). Median overall survival was 17.4 months for all 90 patients,
and 31 months for the patients successfully resected compared
with 10.5 months for those who were not (p < .001).45 Although

these results were also encouraging, the authors noted that the
addition of systemic gemcitabine and cisplatin did not appear to
improve survival when compared with gemcitabine-based CRT
alone.
Answer: Neoadjuvant CRT, regardless of the regimen, is well
tolerated and can lead to excellent margin-negative resection rates
and significantly improved survival in patients eligible for resection. A minority of patients will have disease progression during
therapy, with the majority of these being due to distant metastases rather than local progression alone suggesting that patients
with biologically aggressive disease may be spared from unnecessary surgery. The applicability of neoadjuvant strategies outside of
major academic centers is unclear. Randomized-controlled trials
are needed to determine whether neoadjuvant therapy offers any
advantage over the adjuvant approach.

Clinical Question Summary


Question

Answer

1 What is the optimal


pretreatment staging
assessment of patients with
pancreatic adenocarcinoma?

CT with pancreas protocol


is the preferred method for
diagnosis, staging, and determining
resectability.
EUS-FNA is the best modality for
obtaining a tissue diagnosis.
SL should be used selectively in
patients at high risk for occult
metastatic disease, and in those
with Stage III disease to avoid
unnecessary radiotherapy.

2 What defines resectable,


borderline resectable, and
locally advanced pancreatic
adenocarcinoma?

Resectable disease is defined as the


absence of tumor extension to the
SMA, celiac, or common hepatic
arteries, and the absence of any
tumor-induced unilateral shift or
narrowing of any aspect of the
PV-SMV confluence.
Borderline resectable disease is
defined as tumors with abutment
of the SMA, abutment or
short-segment encasement of
the common hepatic artery,
tumor-induced unilateral shift
or narrowing of the PV-SMV,
or short-segment occlusion of
the SMV/PV confluence with
patent vessel above and below
the occlusion suitable for venous
reconstruction.
Locally advanced tumors are those
with encasement of the SMA or
common hepatic artery, or an
occluded SMV/PV confluence
with no technical option for
reconstruction.

Level of
Evidence

Grade of
Recommendation

References

2c

2, 4-6

2c

11, 13-15

2b, 3a

7-10

2c, 5

19-24

(Continued)

PMPH_CH57.indd 462

5/22/2012 5:39:55 PM

Pancreatic Adenocarcinoma

463

(Continued)
Question

Answer

3 What is the role of adjuvant


therapy after resection for
pancreatic adenocarcinoma?

Adjuvant chemotherapy is clearly


beneficial.
The role of adjuvant CRT is unclear,
though may benefit the subset of
patients with nodal involvement.
Gemcitabine should be used with
adjuvant CRT regimens.

4 What is the role of neoadjuvant


therapy in the management of
pancreatic adenocarcinoma?

Neoadjuvant CRT is well tolerated


and results in excellent marginnegative resection rates and
significantly improved survival in
patients who undergo resection.
It can also identify the subset of
patients with aggressive disease
who will not benefit from surgical
therapy.

REFERENCES
1. ACS. American Cancer Society. Cancer Facts and Figures 2010.
[February 7, 2011]; Available from: http://www.cancer.org/
Research/CancerFactsFigures/CancerFactsFigures/cancer-factsand-figures-2010.
2. Tamm E, Charnsangavej C, Szklaruk J. Advanced 3-D imaging
for the evaluation of pancreatic cancer with multidetector CT.
Int J Gastrointest Cancer. 2001;30(1-2):65-71.
3. Faria SC, Tamm EP, Loyer EM, Szklaruk J, Choi Haesun, Charnsangavej C. Diagnosis and staging of pancreatic tumors. Semin
Roentgenol. 2004;39(3):397-411.
4. Kulig J, Popiela T, Zajc A, Kk S, Koodziejczyk P. The value
of imaging techniques in the staging of pancreatic cancer. Surg
Endosc. 2005;19(3):361-365.
5. Raptopoulos V, Steer ML, Sheiman RG, Vrachliotis TG, Gougoutas
CA, Movson JS. The use of helical CT and CT angiography to predict vascular involvement from pancreatic cancer: Correlation with
findings at surgery. AJR Am J Roentgenol. 1997;168(4):971-977.
6. House MG, Yeo CJ, Cameron JL, Campbell KA, Schulick RD,
et al. Predicting resectability of periampullary cancer with
three-dimensional computed tomography. J Gastrointest Surg.
2004;8(3):280-288.
7. Bao PQ, Johnson JC, Lindsey EH, Schwartz DA, Arildesen RC, et al.
Endoscopic ultrasound and computed tomography predictors of
pancreatic cancer resectability. J Gastrointest Surg. 2008;12(1):10-16.
8. Chang KJ. State of the art lecture: Endoscopic ultrasound (EUS) and
FNA in pancreatico-biliary tumors. Endoscopy. 2006;38(Suppl 1):
S56-S60.
9. Jhala NC, Jhala D, Eltoum I, Vickers SM, Wilcox CM, Chhieng DC,
et al. Endoscopic ultrasound-guided fine-needle aspiration biopsy:
A powerful tool to obtain samples from small lesions. Cancer.
2004;102(4):239-246.
10. Harewood GC, Wiersema MJ. A cost analysis of endoscopic
ultrasound in the evaluation of pancreatic head adenocarcinoma. Am J Gastroenterol. 2001;96(9):2651-2656.
11. Holzman MD, Reintgen KL, Tyler DS, Pappas TN. The role of
laparoscopy in the management of suspected pancreatic and
periampullary malignancies. J Gastrointest Surg. 1997;1(3):236243; discussion 243-244.

PMPH_CH57.indd 463

Level of
Evidence

Grade of
Recommendation

References

1b
2c

37-45

31-34

2b

2b

12. Pisters PW, Lee JE, Vauthey JN, Charnsangavej C, Evan DB.
Laparoscopy in the staging of pancreatic cancer. Br J Surg.
2001;88(3):325-337.
13. Karachristos A, Scarmeas N, Hoff man JP. CA 19-9 levels predict
results of staging laparoscopy in pancreatic cancer. J Gastrointest
Surg. 2005;9(9):1286-1292.
14. Vollmer CM, Drebin JA, Middleton WD, Teefey SA, Linehan DC,
Soper NJ, et al. Utility of staging laparoscopy in subsets of peripancreatic and biliary malignancies. Ann Surg. 2002;235(1):1-7.
15. Liu RC, Traverso, LW. Diagnostic laparoscopy improves staging
of pancreatic cancer deemed locally unresectable by computed
tomography. Surg Endosc. 2005;19(5):638-642.
16. Golcher H, Brunner T, Grabenbauer G, Merkel S, Papadopoulos T,
Hohenberger W, et al. Preoperative chemoradiation in adenocarcinoma of the pancreas. A single centre experience advocating a
new treatment strategy. Eur J Surg Oncol. 2008;34(7):756-764.
17. Kim HJ, Czischke K, Brennan MF, Conlon KC. Does neoadjuvant chemoradiation downstage locally advanced pancreatic
cancer? J Gastrointest Surg. 2002;6(5):763-769.
18. Tse RV, Dawson LA, Wei A, Moore M. Neoadjuvant treatment
for pancreatic cancera review. Crit Rev Oncol Hematol.
2008;65(3):263-274.
19. Varadhachary GR, Tamm EP, Crane C, Evans DB, Wolff RA.
Borderline resectable pancreatic cancer. Curr Treat Options Gastroenterol. 2005;8(5):377-384.
20. Varadhachary GR, Tamm EP, Abbruzzese JL, Xiong HQ, Crane CH,
et al. Borderline resectable pancreatic cancer: Definitions, management, and role of preoperative therapy. Ann Surg Oncol.
2006;13(8):1035-1046.
21. National Comprehensive Cancer Network. Clinical practice
guidelines in oncology v.2.2010. Pancreatic adenocarcinoma.
[cited February 18, 2011]; Available from: http://www.nccn.org/
professionals/physician_gls/default.asp.
22. Abrams RA, Lowy AM, OReilly EM, Wolff RA, Picozzi VJ,
et al. Combined modality treatment of resectable and borderline
resectable pancreas cancer: Expert consensus statement. Ann
Surg Oncol. 2009;16(7):1751-1756.
23. Chun YS, Mileston BN, Watson JC, Cohen SJ, Burtness B, et al.
Defining venous involvement in borderline resectable pancreatic
cancer. Ann Surg Oncol. 2010;17(11):2832-2838.

5/22/2012 5:39:55 PM

464

Surgery: Evidence-Based Practice

24. Callery MP, Chang KJ, Fishman EK, Talamonti MS, Traverso LW,
et al. Pretreatment assessment of resectable and borderline resectable pancreatic cancer: Expert consensus statement. Ann Surg
Oncol. 2009;16(7):1727-1733.
25. Kalser MH, Ellenberg SS. Pancreatic cancer. Adjuvant combined
radiation and chemotherapy following curative resection. Arch
Surg. 1985;120(8):899-903.
26. Klinkenbijl JH, Johannes J, Tarek S, vanPel R, Couvreur ML,
Veenhof CH, et al. Adjuvant radiotherapy and 5-fluorouracil after
curative resection of cancer of the pancreas and periampullary
region: Phase III trial of the EORTC gastrointestinal tract cancer cooperative group. Ann Surg. 1999;230(6):776-782; discussion
782-784.
27. Neoptolemos JP, Stocken DD, Friess, H, Bassi C, Dunn JA,
Hickey H, et al. A randomized trial of chemoradiotherapy and
chemotherapy after resection of pancreatic cancer. N Engl J Med.
2004;350(12):1200-1210.
28. Neoptolemos JP, Stocken DD, Smith CT, Bassi C, Ghaneh P,
Owen E, et al. Adjuvant 5-fluorouracil and folinic acid vs observation for pancreatic cancer: Composite data from the ESPAC-1
and -3(v1) trials. Br J Cancer. 2009;100(2):246-250.
29. Oettle H, Post S, Neuhaus P, Gellert K, Langrehr J, Ridwelski K,
et al. Adjuvant chemotherapy with gemcitabine vs observation
in patients undergoing curative-intent resection of pancreatic cancer: A randomized controlled trial. JAMA. 2007;297(3):
267-277.
30. Neuhaus P, Riess H, Post S, Gellert K, Ridwelski K, Schramm H,
et al. CONKO-001: Final Results of the randomized, prospective,
multicenter phase III trial of adjuvant chemotherapy with gemcitabine versus observation in patients with resected pancreatic
cancer (PC). J Clin Oncol. 2008;26:abstr LBA4504.
31. Regine WF, Winter KA, Abrams RA, Safran H, Hoff man JP,
Konski A, et al. Fluorouracil vs gemcitabine chemotherapy before
and after fluorouracil-based chemoradiation following resection
of pancreatic adenocarcinoma: A randomized controlled trial.
JAMA. 2008;299(9):1019-1026.
32. Van Laethem JO, Hammel P, Mornex F, Azria D, Van Tienhoven G,
Vergauwe P, et al. Adjuvant gemcitabine alone versus gemcitabine-based chemoradiation after curative resection for pancreatic
cancer: Updated results of a randomized EORTC/FFCD/GERCOR phase II study (40013-22012/9203). Proceedings of the
American Society of Clinical Oncology. 2009 (Abstract 4527).
33. Corsini MM, Miller RC, Haddock MG, Donohue JH, Farnell MB,
Nagorney DM, et al. Adjuvant radiotherapy and chemotherapy
for pancreatic carcinoma: The Mayo Clinic experience (19752005). J Clin Oncol. 2008;26(21):3511-3516.
34. Herman JM, Swartz MJ, Hsu CC, Winter J, Pawlik TM, Sugar E,
et al. Analysis of fluorouracil-based adjuvant chemotherapy
and radiation after pancreaticoduodenectomy for ductal adenocarcinoma of the pancreas: Results of a large, prospectively

PMPH_CH57.indd 464

35.

36.

37.

38.

39.

40.

41.

42.

43.

44.

45.

collected database at the Johns Hopkins Hospital. J Clin Oncol.


2008;26(21):3503-3510.
Merchant NB, Rymer J, Koehler E, Ayers GD, Castellano J,
Kooby DA, et al. Adjuvant chemoradiation therapy for pancreatic adenocarcinoma: Who really benefits? J Am Coll Surg.
2009;208(5):829-838; discussion 838-841.
Povoski SP, Karpeh MS, Conlon KC, Blumgart LH, Brennan MF.
Association of preoperative biliary drainage with postoperative outcome following pancreaticoduodenectomy. Ann Surg.
1999;230(2):131-142.
Velanovich V, Kheibek T, Khan M. Relationship of postoperative
complications from preoperative biliary stents after pancreaticoduodenectomy. A new cohort analysis and meta-analysis of
modern studies. JOP. 2009;10(1):24-29.
Evan DB, Rich TR, Byrd DR, Cleary KR, Connelly JH, Levin B,
et al. Preoperative chemoradiation and pancreaticoduodenectomy for adenocarcinoma of the pancreas. Arch Surg. 1992;127(11):
1335-1339.
Yeung RS, Weese JL, Hoff man JP, Solin LJ, Paul AR, Engstrom PF,
et al. Neoadjuvant chemoradiation in pancreatic and duodenal
carcinoma. A Phase II Study. Cancer. 1993;72(7):2124-2133.
Pisters PW, Abbruzzese JL, Janjan NA, Cleary KR, Charnsangavej C, Goswitz MS, et al. Rapid-fractionation preoperative
chemoradiation, pancreaticoduodenectomy, and intraoperative
radiation therapy for resectable pancreatic adenocarcinoma.
J Clin Oncol. 1998;16(12):3843-3850.
Hoff man JP, Lipsitz S, Pisansky T, Weese JL, Solin L, Benson AB.
Phase II trial of preoperative radiation therapy and chemotherapy for patients with localized, resectable adenocarcinoma of the
pancreas: An Eastern Cooperative Oncology Group Study. J Clin
Oncol. 1998;16(1):317-323.
Turrini O, Viret F, Moureau-Zabotto L, Guiramand J, Moutardier V,
Lelong B, et al. Neoadjuvant 5 fluorouracil-cisplatin chemoradiation effect on survival in patients with resectable pancreatic
head adenocarcinoma: A ten-year single institution experience.
Oncology. 2009;76(6):413-419.
Gurrini O, Ychou M, Moureau-Zabotto L, Rouanet P, Giovannini M,
Moutardier V, et al. Neoadjuvant docetaxel-based chemoradiation for resectable adenocarcinoma of the pancreas: New neoadjuvant regimen was safe and provided an interesting pathologic
response. Eur J Surg Oncol. 2010;36(10):987-992.
Evans DB, Varadhachary GR, Crane CH, Sun CC, Lee JE,
Pisters PWT, et al. Preoperative gemcitabine-based chemoradiation for patients with resectable adenocarcinoma of the pancreatic head. J Clin Oncol. 2008;26(21):3496-502.
Varadhachary GR, Wolff RA, Crane CH, Sun CC, Lee JE,
Pisters PWT, et al. Preoperative gemcitabine and cisplatin followed by gemcitabine-based chemoradiation for resectable adenocarcinoma of the pancreatic head. J Clin Oncol. 2008;26(21):
3487-3495.

5/22/2012 5:39:55 PM

Commentary on
Pancreatic Adenocarcinoma
Matthew H. G. Katz, Jason B. Fleming, Jeffrey E. Lee, Peter W. T. Pisters

The first multi-institutional trial of multimodality therapy for


localized pancreatic adenocarcinoma, published over 25 years ago,
demonstrated that the overall survival duration of patients who
received chemoradiation after resection was longer than that of
patients who underwent surgery alone.1 Although numerous trials have since evaluated novel pre- and postoperative therapy regimens for patients with pancreatic cancer, no meaningful further
improvement in survival has been achieved. The median overall
survival duration of patients with resectable pancreatic tumors
just under 2 yearshas essentially stagnated.2-4 In this chapter,
Dr. St. Julien and colleagues provide a concise, thoughtful overview of four critical, yet controversial issues in the multimodality care of patients with localized pancreatic cancer and highlight
several reasons for our incredibly slow progress. In doing so, the
authors suggest several key areas for improvement in the design
and conduct of future clinical trials of novel therapies for localized pancreatic cancer.
Although adjuvant therapy maximizes oncologic outcomes
following surgery for localized pancreatic cancer, the only treatment that has been proven to be curative itself is the complete
resection of the primary tumor and regional lymph nodes. To
the extent that microscopically complete (R0) resection has been
associated with longer survival times than either macroscopically complete (R1) or incomplete (R2) resection,5 fundamental
components of a multidisciplinary approach to treating patients
with resectable pancreatic cancer include accurate staging of the
disease to properly identify patients for whom complete resection
is feasible, use of a meticulous surgical technique with specific
attention given to oncologically critical soft tissue margins, and
precise histopathologic analysis of the surgical specimen prior to
(or, if neoadjuvant therapy is used, following) adjuvant therapy.
Unfortunately, as Dr. St. Julien and colleagues note, no
standard system of staging localized pancreatic cancer exists.
Although most institutions use computed tomography images
to determine whether a tumor can be technically resected, the
interpretation of such images to differentiate potentially resectable, borderline resectable, and unresectable primary cancers
varies among institutions, radiologists, pancreatic surgeons, general surgeons, and other oncologists. Over the past decade, the
National Comprehensive Cancer Network,6 the Society of Surgical Oncology-American Hepatopancreaticobiliary Association,7
and The University of Texas MD Anderson Cancer Center8 have
developed systems of classifying localized pancreatic cancer that

focus on the primary cancers radiographic relationship to the


superior mesenteric vein and superior mesenteric artery. However,
none of these systems have been validated or uniformly adopted,
and all are open to subjectivity with regard to their use in clinical
practice.
After disease stage and tumor resectability have been precisely
determined, a meticulous technical operation must be performed
to achieve a macroscopically and microscopically complete resection. Several studies have reported high rates of microscopic residual disease at the SMA margin, suggesting that surgeons need to
pay critical attention to this margin during pancreaticoduodenectomy. However, in a recent analysis of patients enrolled in an
American College of Surgeons Oncology Group (ACOSOG) trial
who received chemoradiation following presumably complete
pancreaticoduodenectomy performed by experienced pancreatic
surgeons at high-volume sites, the ACOSOG found that only a
minority of surgeons may actually utilize a standard operation
that is designed to minimize residual cancer at the SMA margin.9
At least 60% of patients analyzed in that study may have undergone an operation in which the retroperitoneal tissues were incompletely resected, suggesting that residual cancer may be left in
many patients in the United States who undergo tumor resection
an outcome that might be prevented or reduced with a more thorough, standardized operation.
Finally, a systematic histopathologic analysis of the surgical
specimen is critical, particularly proper stratification of patients
by margin status. The College of American Pathologists10 suggests
using a standardized method to examine and report the status of
the surgical margins. However, as anatomic nomenclature varies,11
many centers may not routinely evaluate the oncologically significant SMA margin,9 and many surgeons dispute the definition of a
positive margin (tumor cells at the margin vs. tumor cells within
1 mm of the margin), which leads to inconsistency in documentation.12 Determining the role of specific therapies based on surgical
margin status is therefore difficult, if not impossible.13
The variability or absence of quality control in disease staging, surgical methods, and pathologic analysis may thus cause
significant heterogeneity in patient populations in clinical trials
of multimodality therapy for pancreatic cancer. Notwithstanding
the limitations and methodological problems that Dr. St. Julien
and colleagues note are inherent to previous trialsthe inclusion
of patients with varied periampullary diagnoses, the use of complicated study designs, the use of suboptimal treatment regimens,
465

PMPH_CH57.indd 465

5/22/2012 5:39:55 PM

466

Surgery: Evidence-Based Practice

etc.the absence of standardization and quality control specifically related to the selection and conduct of surgery suggests that
past trials have enrolled a heterogenous group of patients who had
a variety of disease stages, underwent variable operations that
needlessly left many with residual cancer, and were subsequently
stratified by margin status by variable methods. Is it any wonder,
then, that we have not been able to come to a consensus on the
use of adjuvant chemoradiation, let alone refine the staging system, define the precise role (or absence thereof) of preoperative
therapy, or demonstrate the efficacy of novel targeted therapeutics
in pancreatic cancer?
The pancreatic research community must give considerable
thought to these critical issues, as well as to the general research
strategy that we pursue for this disease. Although large, phase III
randomized trials have been appropriately encouraged in other
solid tumors in support of evidence-based decision making, it
must be recognized that in the setting of localized pancreatic cancer, large phase III trials take years to accrue, cost millions of dollars, and utilize valuable patient resources that might be directed
instead to a more efficientand perhaps more effectiveresearch
strategy. For example, the last American phase III trial, RTOG
9704, took 4 years to accrue 388 patients with pancreatic cancer
and the outcome data were published a decade after the trial was
opened.4 The current phase III trial, RTOG 0848 has a sample size
of 856 patients and will take more than 5 years to complete accrual
at the protocol specified accrual rate of 14 patients per month.
These valuable patient resources might instead be invested in nine
phase II trials enrolling 100 patients each, each designed to evaluate the efficacy of novel targeted agents administered preoperatively, and each coupled with basic and translational correlative
research studies. It is quite possible that by adopting this alternate
strategy we would make more progress in understanding the biology and potential therapeutic efficacy of the broad spectrum of
targeted agents that will come to market in the time that we are
locked into a national commitment to a phase III trial that is very
unlikely to yield much gain on the less than 2-year median survival outlined by St. Julien and colleagues.
Dr. St. Julien and colleagues highlight the limitations of the
limited number of high-level studies that provide the data we use
as the basis for the multidisciplinary care of patients with localized pancreatic cancer. To accelerate progress, we must recognize
the opportunity costs of continuing an expensive and resourceintensive research strategy that has failed to yield a meaningful
improvement in patient outcome over the last 25 years. Focus on
a more efficient program of smaller and better-designed clinical
trialswith attention to the methodological problems described
by Dr. St. Julien and colleagues and described abovewould be a
major step forward.

PMPH_CH57.indd 466

REFERENCES
1. Kalser MH, Ellenberg SS. Pancreatic cancer. Adjuvant combined
radiation and chemotherapy following curative resection. Archives
of surgery. 1985;120(8):899-903.
2. Klinkenbijl JH, Jeekel J, Sahmoud T, et al. Adjuvant radiotherapy
and 5-fluorouracil after curative resection of cancer of the pancreas and periampullary region: phase III trial of the EORTC gastrointestinal tract cancer cooperative group. Annals of surgery.
1999;230(6):776-782; discussion 782-784.
3. Oettle H, Post S, Neuhaus P, et al. Adjuvant chemotherapy with
gemcitabine vs observation in patients undergoing curativeintent resection of pancreatic cancer: a randomized controlled
trial. JAMA: the journal of the American Medical Association.
2007;297(3):267-277.
4. Regine WF, Winter KA, Abrams RA, et al. Fluorouracil vs Gemcitabine Chemotherapy Before and After Fluorouracil-Based
Chemoradiation Following Resection of Pancreatic Adenocarcinoma. A Randomized Controlled Trial. JAMA: the journal of the
American Medical Association. 2008;299(9):1019-1026.
5. Katz MH, Hwang R, Fleming JB, et al. Tumor-node-metastasis
staging of pancreatic adenocarcinoma. CA Cancer J Clin. 2008;
58(2):111-125.
6. Pancreatic adenocarcinoma. NCCN Clinical Practice Guidelines
in Oncology. 2009.
7. Callery MP, Chang KJ, Fishman EK, et al. Pretreatment assessment
of resectable and borderline resectable pancreatic cancer: expert
consensus statement. Ann Surg Oncol. 2009;16(7):1727-1733.
8. Katz MH, Pisters PW, Evans DB, et al. Borderline resectable pancreatic cancer: the importance of this emerging stage of disease.
J Am Coll Surg. 2008;206(5):833-846; discussion 846-848.
9. Katz MH, Merchant NB, Brower S, et al. Standardization of surgical and pathologic variables is needed in multicenter trials of
adjuvant therapy for pancreatic cancer: results from the ACOSOG Z5031 trial. Ann Surg Oncol. 2011;18(2):337-344.
10. Pancreas (exocrine) [College of American Pathologists Cancer Protocols web site]. 2009. Available at: http://www.cap.org/apps/docs/
committees/cancer/cancer_protocols/2009/PancreasExo_09
protocol. pdf. Accessed March 25, 2011, 2010.
11. Raut CP, Tseng JF, Sun CC, et al. Impact of resection status on
pattern of failure and survival after pancreaticoduodenectomy for
pancreatic adenocarcinoma. Annals of surgery. 2007;246(1):52-60.
12. Verbeke CS, Menon KV. Variability in reporting resection margin status in pancreatic cancer. Annals of surgery. 2008;247(4):
716-717.
13. Neoptolemos JP, Stocken DD, Dunn JA, et al. Influence of resection margins on survival for patients with pancreatic cancer
treated by adjuvant chemoradiation and/or chemotherapy in
the ESPAC-1 randomized controlled trial. Annals of surgery.
2001;234(6):758-768.

5/22/2012 5:39:55 PM

CHAPTER 58

Unusual Pancreatic Tumors


David W. Rittenhouse, Charles J. Yeo, and Samuel D. Gross

INTRODUCTION

neoplasms which arise from a different progenitor cell than the


pancreatic ductal cell, share mixed histology or are a result of
metastasis from another organ.2 Cystic tumors comprise up to
15% of pancreatic neoplasms and present many challenges with
diagnosis and management.1 Neuroendocrine tumors, both functional (Table 58.2) and nonfunctional, account for approximately
1% to 2% of pancreatic tumors.3 Adenosquamous carcinoma,
acinar cell carcinoma, lymphoma, rare sarcomas, pancreatoblastoma, metastatic disease, and others make up the remaining rare
neoplasms of the pancreas. The relative rarity of these tumors has
made it difficult to gather substantial epidemiologic data. Their
scarcity has also made it challenging to develop any guidelines
on the best diagnostic and management strategies. In this chapter, we discuss key elements of many of the unusual pancreatic
tumors.

With greater quality and availability of cross-sectional imaging,


more patients with pancreatic neoplasms are being found than
in the past. Pancreatic ductal adenocarcinoma (PDA) is the 10th
most commonly diagnosed cancer in the United States and the
4th leading cause of cancer-related death. Much work is currently
being done to develop biomarkers that are both prognostic and
predictive for adjuvant therapy. Th is work will help to identify
which populations carry the highest risk for the development of
PDA, which patients will benefit from early screening, and which
therapies will be optimal for any specific patient. However, PDA
only comprises approximately 85% of all pancreatic neoplasms.1
The remaining pancreatic neoplasms (Table 58.1) consist of other
Table 58.1 Unusual Pancreatic Tumors

Pancreatic Endocrine Neoplasms

A. Neuroendocrine tumors
Nonfunctional tumors
Functional tumors
Insulinoma
VIPoma
Gastrinoma
Glucagonoma
Somatostatinoma

Insulinoma
1. Is endoscopic ultrasound (EUS) the most sensitive test used
for preoperative localization of insulinoma and other pancreatic neuroendocrine neoplasms?
Insulinomas occur with an annual incidence in the general population of approximately one to four cases per 1,000,000.4 They
comprise the majority of functional neuroendocrine tumors and
typically occur sporadically.4,5 Patients can present with the classic Whipples triad: (1) symptoms of hypoglycemia during fasting,
(2) documentation of hypoglycemia, with a serum glucose level
below 50 mg/dL, and (3) relief of hypoglycemic symptoms following administration of exogenous glucose. The typical symptoms include neuroglycopenic symptoms (confusion, personality
change, and coma) and catecholamine surge symptoms (trembling, diaphoresis, and tachycardia). The classic diagnostic test is
a monitored fast, done under supervision, where blood is drawn
for glucose and insulin levels at the time of symptoms. Fasting glucose and insulin measurements aid in the diagnosis of insulinoma,
but do not help in the localization. Insulinomas occur with an even

B. Cystic tumors
Serous cystadenoma
Serous cystadenocarcinoma
Mucinous cystadenoma
Mucinous cystadenocarcinoma
Intraductal papillary mucinous neoplasm
C. Adenosquamous carcinoma
D. Acinar cell carcinoma
E. Solid pseudopapillary tumor
F. Pancreatoblastoma
G. Metastases from a nonpancreatic primary
467

PMPH_CH58.indd 467

5/22/2012 5:40:30 PM

468

Surgery: Evidence-Based Practice

Table 58.2 Symptoms, Diagnostic Tests and Anatomic Localization of Functional Neuroendocrine Neoplasms of
the Pancreas

Symptoms

Diagnostic tests

Insulinoma

Gastrinoma

VIPoma

Glucagonoma

Neuroglycopenia causes
confusion, personality
change, coma
Catecholamine surge
causes trembling,
diaphoresis, tachycardia
Anabolic state: weight gain

Peptic ulcer disease

Watery diarrhea

Necrolytic migratory
erythema

Diarrhea

Weakness

Stomatitis

Esophagitis

Lethargy
Nausea

Angular chelosis, glossitis


Catabolic state: weight loss

Monitored fast

Serum gastrin
measurement
Gastric acid analysis
Secretin stimulation test

Hypokalemia

Hyperglycemia

Achlorhydria
Metabolic acidosis
Elevated serum
VIPoma

Hypoproteinemia
Serum glucagon measurement
Serum amino acid profile

Duodenum and head of


pancreas (gastrinoma
triangle)

Most in body and


tail of pancreas,
with liver
metastases

Most in body and tail of


pancreas, with liver
metastases

Insulin-to-glucose ratio

Anatomic localization

Evenly distributed through


pancreas

Source: Ref. [19].

distribution throughout the head, body, and tail of the pancreas.


Approximately, 90% of insulinomas are less than 2 cm in diameter,
and 30% are less than 1 cm in diameter.
Localization techniques include noninvasive studies or invasive studies. Noninvasive cross-sectional imaging such as computed tomography (CT) scanning and magnetic resonance imaging
(MRI) is commonly performed. CT scanning has a sensitivity of
detecting insulinomas ranging only from 30% to 66%.6 Much of
this variability comes from the differences in imaging quality,
interpreter variability, and also is related to the small size of many
insulinomas. CT scanning can be helpful in staging the tumor, in
the case of local spread or metastasis.7 MRI is another form of
cross-sectional imaging that has been used to localize insulinomas.
Druce et al. studied the accuracy of MRIs in correctly identifying
tumors that were preoperatively presumed to be insulinoma.8 Of
the 28 patients who had preoperative MRIs, the correct diagnosis
was made in 75% of patients.
Invasive tests occasionally used for localization are angiography or transhepatic portal-venous sampling (THPVS). For the
latter, a catheter is placed percutaneously through the liver into
the portal vein. Insulin levels are measured from blood samples
that are collected from the splenic vein, superior mesenteric vein,
and portal vein.9 Another invasive procedure, the EUS uses a 7.5 to
10 MHz transducer and detects lesions as small as 5 mm.9 Zimmer
et al. prospectively evaluated the sensitivity and specificity of EUS
in localizing insulinomas and gastrinomas.10 EUS identified 13 of
14 insulinomas in 10 patients that were expected to have the diagnosis of insulinoma based on clinical characteristics. All patients
underwent resection. Sensitivity of detecting the insulinoma was
93%. Rosch et al. compared the sensitivities of EUS with angiography in identifying pancreatic endocrine neoplasms.11 Thirtyseven patients with 39 CT negative pancreatic neuroendocrine
neoplasms (31 insulinomas) were analyzed by EUS and angiography. EUS had a sensitivity of 85% and a specificity of 95%, whereas
angiography had a sensitivity of only 27%.

PMPH_CH58.indd 468

Answer: Yes. EUS has a greater sensitivity than CT scanning and MRI and should be used in the localization of suspected
insulinomas.

VIPoma
2. Are somatostatin analogs (octreotide) helpful in controlling symptoms in patients with VIPoma (vasoactive intestinal
polypeptide-oma)?
VIPoma, first described by Verner and Morrison in 1958, occurs
with a frequency of 1 in 10 million people.12,13 Synonyms for VIPoma
include the VernerMorrision syndrome, pancreatic cholera, and
the WDHA syndrome (watery diarrhea, hypokalemia, and achlorhydria/acidosis). VIPomas secrete high levels of VIP, a neuropeptide that activates adenylate cyclase and cAMP on the intestinal
lumen, inhibiting absorption and stimulating intestinal secretion.
Patients frequently present with intermittent voluminous watery
diarrhea. Laboratory analysis typically reveals hypokalemia, gastric achlorhydria, metabolic acidosis, and elevated serum VIP levels. Ghaferi et al.14 reviewed three patients at a single institution
and the world literature and found that the mean serum VIP levels
were 683 pg/mL (range 2931500 pg/mL). The differential diagnosis can include a host of gastrointestinal diseases from infectious
to malignant (Table 58.3). Over 90% of VIPomas are confined to
the pancreas (Fig. 58.1), with less than 10% occurring in the retroperitoneum or in the chest. The body or tail of the pancreas is
the most common location of the primary VIPoma. More than
60% of VIPomas have metastasized (typically to lymph nodes or
to liver) by the time of diagnosis.13 Fluid replacement and electrolyte correction are critical, given the severe dehydration that
can occur with the voluminous diarrhea. In addition to surgery to
remove the tumor, treatment with somatostatin analogs (typically
octreotide 50250 g SQ q8 h) has shown excellent results in ameliorating the diarrhea. Oberg et al.15 reported a tumor response
(diminution in size) in less than 5% of patients, but symptomatic

5/22/2012 5:40:30 PM

Unusual Pancreatic Tumors

relief from diarrhea in more than 60% of patients. Maton et al.16


reported less than 20% of patients having a response in tumor
size, with 83% of patients experiencing symptomatic relief. The
inhibitory effect of somatostain analogs has been hypothesized to

469

act by reducing tumor blood flow or by a direct hormonal inhibitory effect.17


Answer: Yes. Management of VIPoma patients with longacting somatostatin analogs can be effective at controlling symptoms, but should not be anticipated to affect tumor size or bulk.

Table 58.3 Differential Diagnosis of the


VernerMorrison Syndrome (VIPoma)

Gastrinoma

Entity

Workup

3. Should you ever electively operate on a gastrinoma patient


without first controlling their gastric acid secretion?

Villous adenoma

Lower GI endoscopy

Laxative abuse

Stool examination for


phenolphthalein

Celiac disease

Fecal fat measurement


D-xylose tolerance test
Small bowel biopsy
HLA typing

Parasitic and
infectious diseases

Stool culture
Ovum and parasite analysis
Clostridium difficile toxin assay

Inflammatory bowel
disease

Lower GI endoscopy
Upper GI and small bowel series

Carcinoid syndrome

Urinary 5'-HIAA
Upper GI and small bowel series
Abdominal CT
Serum serotonin measurement

Gastrinoma

Serum gastrin measurement


Gastric acid analysis
Secretin stimulation test

CT, computed tomography; GI, gastrointestinal; 5'-HIAA,


5'-hydroxyindoleacetic acid
Source: Ref. [19].

Figure 58.1 An abdominal MRI of a 64-year-old female presented with voluminous watery diarrhea and hypokalemia. The
MRI reveals a 5.1 5.8 cm mass (arrow) with central necrosis
in the head and uncinate process of the pancreas. The serum
VIP level was markedly elevated. The patient underwent a pylorus preserving pancreaticoduodenectomy, and the pathology
revealed a pancreatic endocrine neoplasm. Immunohistochemical staining for VIP was strongly positive.

PMPH_CH58.indd 469

Gastrinomas are the second most common functional pancreatic neuroendocrine tumor (Fig. 58.2). These gastrin-producing
tumors typically present with peptic ulcer disease and diarrhea.
Approximately, 75% of tumors are sporadic, with nearly 25%
occurring as part of the multiple endocrine neoplasia-1 syndrome
(MEN1). MEN1 is an autosomal dominant inherited disorder
that consists of parathyroid hyperplasia, pituitary neoplasms,
and pancreatic neuroendocrine neoplasms (most commonly gastrinoma). More than 90% of gastrinoma patients have abdominal pain and some form of upper gastrointestinal ulceration. The
fi nding of a serum gastrin level elevated to more than 1000 pg/mL
(far above the upper limit of normal which is 100 pg/mL) is nearly
pathognomonic for gastrinoma in a patient with gastric acid
excess. Other causes of ulcerogenic hypergastrinemia (Table 58.4)
include antral G-cell hyperplasia/hyperfunction, gastric outlet
obstruction, renal insufficiency, and retained excluded gastric
antrum (a surgical mishap).18 Patients with gastrinoma have elevated acid secretion with a basal acid output (BAO) above 15 mEq/h
or a ratio of BAO to maximal acid output (MAO) exceeding
0.6. One common mistake is to falsely consider the diagnosis of
gastrinoma in a patient on proton pump inhibitors (PPIs) with
mildly elevated serum gastrin levels. As PPIs cause reduced acid
secretion, the gastrin elevation is a result of the achlorhydria and
does not indicate a neoplastic process. Provocative testing can
be done to differentiate the causes of ulcerogenic hypergastrinemia. An intravenous bolus of secretin is given after a baseline

Figure 58.2 An abdominal CT scan of a 69-year-old man presented with abdominal pain and peptic ulcer disease. The patient
had an elevated serum gastrin level of greater than 1000 pg/mL.
The CT scan shows a 2.0 2.4 cm hypervascular mass in the
body of the pancreas (arrow) with a dilated pancreatic duct in the
tail of the gland. The patient underwent a distal pancreatectomy
and splenectomy. The pathology revealed a pancreatic endocrine
neoplasm, with the immunohistochemical stain for gastrin being
strongly positive.

5/22/2012 5:40:30 PM

470

Surgery: Evidence-Based Practice

Table 58.4 Disease States Associated with


Hypergastrinemia
Nonulcerogenic causes (normal to low gastric acid secretion)
Atrophic gastritis
Pernicious anemia
Previous vagotomy
Renal failure
Short-gut syndrome
Ulcerogenic causes (excess gastric acid secretion)
Antral G-cell hyperplasia or hyperfunction
Gastric outlet obstruction
Retained excluded gastric antrum
ZollingerEllison syndrome
Source: Ref. [19].

level of gastrin is drawn and gastrin levels are collected at 5-min


intervals for 30 min. An increase by more than 200 pg/mL above
the basal level is indicative of the diagnosis of gastrinoma.19
Answer: No. Gastric acid should be suppressed with appropriate (often high dose) PPIs prior to elective exploration for gastrinoma. To operate without controlling the ulcer diathesis risks
problems with perioperative gastrointestinal ulceration.

Glucagonoma
4. In patients with glucagonoma, does resection of advanced
local disease or metastases increase overall survival?
Glucagonomas have a characteristic clinical presentation of a skin
rash (necrolytic migratory erythema), glossitis, angular chelosis,
diabetes, and a catabolic state resulting in weight loss. Laboratory
analyses reveal hyperglycemia and elevated fasting levels of serum
glucagon. The disease is also accompanied by a dramatic amino
acid profi le disruption. Most glucagonomas are in the body or tail
of the pancreas. Compared with other neuroendocrine tumors of
the pancreas, these tumors are often large (>4 cm) at the time of
diagnosis.18 The rate of malignancy in glucagonomas is 60%
to 80% in tumors that are larger than 5 cm. Thus, the majority
of patients have lymph node or hepatic metastases at the time of
diagnosis.18 There are data to support resection for patients for
locally advanced and metastatic disease.
Wermers et al.20 performed a large single institution review
of 21 patients with glucagonoma. All of the patients had metastatic disease at presentation. Twelve patients underwent surgical debulking via distal pancreatectomy and splenectomy with
or without resection of hepatic metastases. Eight of the patients
had major reduction in serum glucagon levels from preoperative
levels. Serum glucagon levels decreased by a mean of 1442 pg/mL.
Six patients had remission of the necrolytic migratory erythema.
Chu et al.21 reviewed 12 patients that had resection of a glucagonoma at a single institution. Eight of the patients underwent
surgical exploration. Three of the eight had diff use metastatic
spread to the liver. Of those with metastatic disease found at
operation, only one underwent a hepatic resection (left lateral
segmentectomy) after extensive tumor embolization procedures.
Sarmiento et al.22 reviewed 170 patients that had hepatic metastases from neuroendocrine tumor (9 glucagonomas, 5.3%) and
found that hepatic resection of metastases was associated with a
5-year survival of 61%. The authors further reviewed the literature
and found 5-year survival of 47% to 76% after hepatic resection

PMPH_CH58.indd 470

for neuroendocrine metastases.22 Hill et al.23 analyzed the SEER


database to look for a survival advantage with surgery in all pancreatic neuroendocrine tumors, including glucagonoma. Patients
with all stages of disease showed a survival advantage with surgery compared with no surgery.
Answer: Large prospective multi-institutional trials are lacking, but there is some evidence to suggest that aggressive resection
of a locally advanced primary tumor and metastatic deposits can
improve survival in patients with glucagonoma.

Adenosquamous Carcinoma
5. Are there any means to preoperatively differentiate between
adenosquamous carcinoma and adenocarcinoma?
Pancreatic adenosquamous carcinoma (PASC) is a rare morphologic variant of PDA which comprises only 1% to 4% of exocrine
pancreatic tumors. PASC is distinguished from PDA on a histologic basis by having more than 30% squamous cell carcinoma
mixed with the underlying ductal adenocarcinoma.24 It has been
thought that PASC carries a worse prognosis than PDA.25 Katz
et al.26 sought to look for differences in clinical features between
PASC and PDA by performing a historical analysis of cases in the
California Cancer Registry Database. A total of 14,746 cases of PDA
and 95 cases of PASC were included in the analysis. Median age at
diagnosis, gender, race, socioeconomic status, and clinical stage of
patients were all similar between the PDA and PASC patients. More
than 50% of patients from both groups had metastatic disease at
the time of diagnosis. Patients with locoregional PASC were more
likely to undergo resection (61.9%) than those patients with PDA
(35.6%); however, the two groups had similar rates of adjuvant therapy. The two cohorts had a similar median survival (approximately
4 months) when examined by a Cox proportional hazard model.
PASC and PDA have been compared on a molecular level.
Brody et al.27 examined the molecular characteristics of eight cases
of PASC and found that these patients had similar molecular alterations to that of PDA in KRAS2, tp53, and DPC4. However, p63
staining was helpful in identifying those cases of adenosquamous
differentiation with an acantholytic growth pattern.27 However,
this is not helpful in preoperatively differentiating between the
two tumors, due to the paucity of tissue retrieved by standard
needle biopsy techniques.
It would be ideal to have a radiologic method of distinguishing
between PASC and PDA. Currently, no such method exists. Izuishi
et al.28 published a case report of a patient with PASC. The patient
underwent a fluorodeoxyglucose positron emission tomography
(FDG-PET), which showed strong FDG uptake with a maximum
standardized uptake volume (SUV) of 15.8. The mean SUV of PDA
is somewhat lower at 4.7 2.5. Yet, as most patients with resectable
pancreatic tumors are not recommended to undergo PET scanning,
its value in differentiating PASC from PDA remains unproven.
Answer: There is currently no evidence to support any
means of effectively differentiating between PASC and PDA
preoperatively.
6. Are there molecular markers for adenosquamous cancers
that can be used for prognosis or the prediction of response to
adjuvant therapy?
A better understanding of biomarkers is critical as we progress toward the goal of personalized cancer therapy. It has been
shown that adjuvant chemoradiation therapy (typically either

5/22/2012 5:40:30 PM

Unusual Pancreatic Tumors

gemcitabine or 5-FU with radiation) is associated with a slight


improvement in overall survival in patients with resected PDA.
Voong et al.24 showed that the proportion of squamous differentiation (<30% vs. >30%) was not a factor which influenced the median
overall survival.24 It would be helpful if we could identify biomarkers that could be prognostic or be predictive in patients with PASC.
One biomarker that is currently used is carbohydrate antigen 19-9
(CA19-9). CA19-9 has been used to preoperatively help to distinguish between benign disease and PDA, with a sensitivity ranging from only 41% to 86%, and a specificity ranging from 33% to
100%.29 Another emerging biomarker is the mRNA binding protein
HuR. High expression levels of HuR have been shown to carry a
poor prognosis in patients with PDA and other cancers. However,
high cytoplasmic levels of HuR have been shown to be predictive
of a favorable response to gemcitabine-based adjuvant therapy in
patients with PDA.30,31 Patients with higher levels of HuR expression had a seven fold increase in survival after gemcitabine treatment, when compared with patients with low HuR expression.
Answer: There is no evidence currently available that shows
that molecular markers for PASC can predict response to adjuvant
therapy.

Acinar Cell Carcinoma


7. Are there clinical measures to preoperatively differentiate
between acinar cell carcinoma of the pancreas and adenocarcinoma of the pancreas?
Pancreatic acinar cell carcinoma (ACC) is a rare cancer that
accounts for only 1% of the non-neuroendocrine pancreatic tumors, despite approximately 80% of the pancreas volume
being composed of acinar cells. Matos et al.32 performed a multiinstitutional study looking at the clinical characteristics of 17
patients with pathologically confirmed ACC between the years
1988 and 2008.32 Patients with ACC did not present with the typical symptoms of painless obstructive jaundice, but rather they
presented with pain and weight loss (Fig. 58.3). None of the ACC
patients had elevated CA19-9 levels on preoperative laboratory
analysis, a tumor marker which is commonly elevated in PDA.
Schmidt et al.33 reviewed 865 patients with ACC and 367,999
patients with PDA in the National Cancer Database from 1985
to 2005. Patients with ACC were more likely to be male, white,
have a larger tumor size (>4 cm), no nodal involvement, and to
have pancreatic tail tumors. Wisnoski et al.34 reviewed the Surveillance, Epidemiology, and End Results (SEER) database and
compared 672 patients with ACC with 58,526 patients with PDA.
Patients with ACC were more likely to be younger at diagnosis
(56 vs. 70 years), white and male when compared to patients with
PDA. Another institutional review looked at 14 patients with ACC
and reported a median age of diagnosis of 57 years.35
Answer: Large retrospective database reviews have shown
that patients with ACC tend to be white, male, and present at
an earlier age as compared with PDA patients. However, these
demographic features lack accuracy in predicting the ultimate
pathology in patients with pancreatic neoplasia.
8. Is the overall prognosis worse for patients with acinar cell
carcinoma of the pancreas when compared with adenocarcinoma of the pancreas?
Schmidt et al.33 reported that stage-specific 5-year survivals were
significantly better for resected ACC versus resected PDA with

PMPH_CH58.indd 471

471

Figure 58.3 MRI showing a dilated pancreatic duct (arrow) in a


34-year-old white female who presented with a history of seven
bouts of pancreatitis in 1 year. Her CA19-9 level was normal at
28 U/mL. The patient underwent a distal pancreatectomy and
splenectomy. Pathology revealed a multifocal acinar cell carcinoma (T2N0M0) with positive surgical margins. The patient
returned for a completion right-sided pancreatectomy 2 months
later, with the second pancreatectomy specimen also revealing a
scattered multifocal acinar cell carcinoma.
Stage I: 52.4% versus 28.4%, Stage II: 40.2% versus 9.8%, Stage III:
22.8% versus 6.8%, and Stage IV disease: 17.2% versus 2.8%. By
multivariate analysis, age less than 65 years, well-differentiated
tumors, and negative resection margins were the independent
favorable prognostic markers for ACC. Wisnoski et al.34 also
reported overall better survivals for patients with ACC when
compared with PDA, as their overall 5-year survival was 42.8%
(median, 47 months) for ACC and only 3.8% for PDA (median,
4 months). When analyzing only those patients who did not
undergo resection, the 5-year survival rates were 22% for ACC
and 2% for PDA. When comparing only patients that underwent
surgical resection, the 5-year survival rates were 72% for ACC
and only 16.3% for PDA.
Answer: Large retrospective database reviews have shown
that patients with ACC have a better overall prognosis when compared with similarly treated and staged patients with PDA.

Mucinous Cystic Neoplasm (MCN)


9. What is the most important element, between surgery and
pathology, in ensuring the proper diagnosis of MCN? What
information is essential to predict prognosis?
MCN are cystic tumors of the pancreas that are characterized
by an ovarian-like stroma underlying the cyst epithelium.36 It is
hypothesized that this ovarian-like tissue arises from the rest of
the embryologic ovarian tissue in the pancreas.1 Patients with these
tumors have an average age at diagnosis of 48 years, and a female
to male ratio that exceeds 20:1. MCNs most commonly arise in the
body and tail of the pancreas and consist of uni- or multilocular
components that typically do not communicate with the pancreatic ductal system (Figs. 58.4A and B).36 Microscopically, the wall
may have velvety papillations with trabeculations.1 Less than 20%
of MCNs harbor an invasive component consisting of elements
that resemble PDA, or even rarer forms such as undifferentiated

5/22/2012 5:40:30 PM

472

Surgery: Evidence-Based Practice

variable and difficult identification of small malignant areas in


large neoplasms.
Answer: The most important aspect in managing the pathology specimen in patients with MCN is the clear communication
between the surgeon and the pathologist about the importance
of a complete pathologic analysis of the tumor, given the propensity of MCNs to undergo scattered malignant transformation. We
recommend that all MCNs be completely submitted and entirely
pathologically examined.

Intraductal Papillary Mucinous Neoplasm (IPMN)


10. Should you perform a partial or a total pancreatectomy for a
main duct intraductal papillary mucinous neoplasms (IPMN)?

Figure 58.4A An abdominal CT scan from a 32-year-old female


presented to the hospital 4 days after her first episode of severe
left upper quadrant pain. The patient was treated for pancreatitis
and diagnosed as having a pseudocyst of the pancreas, as this
CT scan showed a 3.3 3.9 cm cystic lesion in the tail of the
pancreas (arrow).

Figure 58.4B The lesion failed to disappear after 1 year of


follow-up, at which time the patient sought a second opinion.
Repeat CT scanning showed an increased size cystic lesion in
the tail of the pancreas, now 4.5 4.5 cm (arrow). The patient
underwent a distal pancreatectomy and splenectomy with the
final pathology on the entirely submitted specimen revealing a
mucinous cystic neoplasm with a low-grade dysplasia.
carcinoma with osteoclast-like giant cells, adenosquamous carcinoma, choriocarcinoma, or sarcoma.36 Zamboni et al.37 studied
the clinicopathologic features of 56 patients with MCNs. Malignancy correlated with multilocularity and presence of papillary
projections or mural nodules, loss of ovarian-like stroma, and
p53 immunoreactivity. Complete pathologic analysis should
be done on all MCNs given their malignant potential and the

PMPH_CH58.indd 472

IPMNs, which account for 10% of pancreatic tumors, consist of


cystic dilatation of the main and/or branch pancreatic ducts in
which intraductal proliferation of neoplastic mucin-producing
cells project as papillations.1 These tumors, which typically present in the 7th to 8th decade of life, are most commonly found in
the head and uncinate process of the pancreas. 38 Macroscopically, an IPMN communicates with the ductal system. Microscopically, the cystic lesions will consist of papillae with three
distinct patterns: intestinal, pancreatobiliary, and gastric.1
Branch duct IPMNs are more commonly found in younger
patients and those resected have lower rates of malignancy,
ranging from 6% to 46%.39 Resected main duct IPMNs have
been shown to harbor malignancy in 58% to 92% of tumors.38
Given the increased risk for malignancy, especially in main
duct IPMNs, it is imperative to develop clearly defi ned treatment strategies. Sohn et al.40 reviewed 60 patients that underwent pancreatic resections for IPMNs and compared them with
702 concurrent patients with PDA. The mean age at presentation was 67.4 years in patients with IPMNs. Most IPMNs were in
the head of the pancreas or diff usely throughout the gland. Of
the operations for IPMN, 70% underwent pancreaticoduodenectomy, 22% underwent total pancreatectomy, and 8% underwent
distal pancreatectomy. Infi ltrating adenocarcinoma was found
in association with 37% of the IPMNs. Adsay et al.41 examined
the clinicopathologic characteristics of 28 IPMN lesions. Four of
these patients died of disease at the follow-up time of 35 months.
Two of the four patients only had borderline atypia with no evidence of invasive carcinoma. One of the patients had intraductal carcinoma extending to the distal pancreatic ductal margin.
It is unclear whether this was a missed disease at the original
operation or inadequate analysis because the entire tumor
specimen was not analyzed. Chari et al.42 analyzed 113 IPMNs
and categorized them based on the presence of an invasive
component. Forty patients had an element of invasive carcinoma, whereas 73 patients did not. Recurrence rates were similar
after partial pancreatectomy (18/27; 67%) when compared with
total pancreatectomy (8/13; 62%). Recurrence occurred within
3 years of resection in 91% of the patients. Among the patients
with noninvasive IPMNs, 5 of 60 (8%) recurred after partial pancreatectomy and 0 of 13 had recurrence after total pancreatectomy. Of those noninvasive IPMNs that recurred, the median
time to recurrence was 40 months.
Answer: At this time, there is no evidence to support a routine total pancreatectomy over a partial resection (typically rightsided) for an isolated main duct IPMN. Total pancreatectomy

5/22/2012 5:40:31 PM

Unusual Pancreatic Tumors

renders the patient an obligate insulin-dependent diabetic. If partial pancreatectomy is performed, the remnant pancreas needs to
be kept under radiographic surveillance.
11. What is the appropriate surveillance for patients who have
undergone resection for IPMN?
The question of surveillance arises when dealing with IPMNs.
Sohn et al.40 in a review of 60 patients found that 37% of patients
had an associated infi ltrating IPMN and that the 5-year survival
rate for all patients with IPMNs was 57%. Chari et al.42 reported
that 91% of the patients that had resection for an IPMN with
an invasive component recurred within 3 years, whereas recurrence was seen at a mean of 3.25 years in resected IPMNs that
did not have an invasive component. Verbesey et al. 38 suggested
re-evaluation every 6 months with radiologic imaging.
Answer: There is no Level 1 evidence to suggest the exact
timing of surveillance screening for patients who have had an
IPMN resected. Patients who have had a main duct IPMN are
at an increased risk for recurrence; so we believe they should be
screened more frequently than those with branch duct IPMNs.
We recommend yearly or every other year MRI/MRCP for such
surveillance.

Serous Cystic Neoplasm (SCN)


12. Can you reliably differentiate preoperatively between SCN,
MCN, and IPMN?
SCNs are benign tumors of the pancreas that are composed of
uniform small cysts containing serous fluid.1 These tumors are
seen with a female to male ratio of 3:1 and present at a mean age of
61 years. SCNs often present as large masses in the body and tail
of the pancreas with either a microcystic or a macrocystic appearance. Microcystic forms contain multiple small cysts fi lled with
a clear fluid in a characteristic honeycomb pattern that can have a
central stellate scar. Macrocystic SCNs form unilocular cystic
structures with fewer but larger loculi.
Lee et al.43 retrospectively reviewed the CT scans of 52 patients
to try to determine the preoperative accuracy of CT scanning in
correctly identifying SCNs. The accuracy varied based on the
structure of the cysts. The accuracy of CT scanning in predicting
unilocular macrocystic SCNs was only 35.7%. However, the accuracy of CT scanning in correctly predicting honeycombed microcystic SCNs and multilocular macrocystic SCNs was 81% and
87.5%, respectively. Shah et al.44 reported the frequency of which
certain characteristic SCN features appear on CT imaging. Of the
28 patients reviewed who had SCN, 22 (78%) had a microcystic
appearance, 25 (89%) had surface lobulations, and 9 (32%) had the
presence of a central scar. Based on a stepwise logistic regression
analysis, a microcystic appearance was the only finding statistically predictive for the diagnosis of SCN.
Cyst fluid can also be used to help diagnose SCN and to
help to differentiate it from MCNs and IPMNs. Frossard et al.
reviewed 113 patients who had cystic tumors of the pancreas who
underwent EUS-FNA.45 The cyst fluid analysis and EUS results
were compared with the final pathology from surgery or postmortem examination. EUS appearance alone correctly diagnosed
SCNs, MCNs, and IPMNs with a sensitivity/specificity of
43%/76%, 65%/84%, and 100%/100%, respectively. FNA cyst fluid
analysis correctly diagnosed SCNs, MCNs, and IPMNs with a

PMPH_CH58.indd 473

473

sensitivity/specificity of 100%/98%, 94%/100%, and 100%/98%,


respectively. Allen et al.46 reported using a biomarker panel for
the analysis of cyst fluid. When comparing SCN, MCN, and
IPMN, there was an error in classification of tumors in 27% of
cases; however, this number fell to only 8% when only comparing
SCN with IPMN.
Answer: Preoperative differentiation of cystic tumors of the
pancreas is a challenge. The literature is lacking in large randomized prospective trials. The use of cross-sectional imaging with
CT scanning, along with EUS visualization and cyst fluid analysis,
can provide information to differentiate between these tumors;
however, the accuracy is not 100%.
13. Do serous cystic neoplasms ever undergo malignant
transformation?
Unlike MCNs (up to 20% can harbor malignancy), SCNs are usually benign. Galanis et al.47 reviewed 158 patients with SCN that
underwent surgical resection at a single institution. Two percent of
the patients had the final diagnosis of serous cystadenocarcinoma.
Strobel et al.48 reviewed the literature from 1989 to 2003 and found
673 cases of SCN with 19 cases reporting malignant transformation, for an overall malignant prevalence of 3%. Nine of the 19
patients were diagnosed as a malignant SCN because of the infiltrative pattern of tumor growth, whereas three of the patients presented with metastatic disease.
Answer: Although of low risk, SCNs do have malignant
potential, with the capacity for infi ltrative tumor growth and
metastatic disease.

Solid Pseudopapillary Tumor (SPT)


14. Are there genetic markers that can predict phenotype and
prognosis for solid pseudopapillary tumors (SPTs)?
SPTs account for less than 1% of pancreatic tumors. They are
usually benign, are commonly found in young women, and have
a nearly 95% to 100% 5-year survival after surgical resection.49
Nearly 20% of the tumors examined in a review by Papavramidis
et al.49 had some element of invasion or metastases suggesting a
capacity of these tumors to have a malignant component. Prognosis is much better than other pancreatic tumors, given that the
5-year survival is 95%. Kim et al.50 reviewed the medical records
of patients with pathologically confirmed SPTs over a 12-year
period in search for clinical predictors of malignancy. Clinical fi ndings that suggested malignancy were metastasis at the
first operation, invasion of adjacent structures, large tumor size
(>13 cm), younger age at presentation, tumor rupture, and inadequate resection.50 Even though patients with malignant SPTs
do better than patients with other pancreatic tumors, it would
be beneficial to identify biomarkers that could possibly predict
phenotype or prognosis of disease. SPTs show diff use nuclear
and cytoplasmic staining for -catenin. Tanaka et al.51 examined
18 patients and found all of them to have diff use nuclear and
cytoplasmic staining for -catenin, and suggested that nuclear
accumulation of -catenin with consequent overexpression of
cyclin D may lead to cell arrest and a more favorable phenotype
of disease.
Answer: There is some evidence to suggest that measuring
the differential subcellular expression of -catenin and cyclin
D may be useful in elucidating which patients will have a more

5/22/2012 5:40:31 PM

474

Surgery: Evidence-Based Practice

favorable phenotype. Greater evidence is needed before tissue or


serum biomarkers can be used for accurate prediction of prognosis in SPTs.

Pancreatoblastoma
15. What is the most common age and presentation of pancreatoblastoma?
Pancreatoblastomas are rare pancreatic tumors that usually arise
in children, although some tumors present in adulthood. Those
affected typically present with vague gastrointestinal symptoms
of weight loss and abdominal pain.52 On gross examination, these
lesions are typically large and globular, consisting of discrete
masses with pseudocapsules of compressed tissue.53 Histologically,
these tumors resemble fetal tissue. Dhebri et al.54 searched the literature for all cases of pancreatoblastoma and found 158 patients

with a median age at presentation of 5 years (range from <1 to


60 years). Seventeen of the patients had metastatic disease at the
time of diagnosis and the median survival was 48 months. Factors
associated with a worse prognosis were synchronous (p = .05) or
metachronous metastases on presentation (p < .001), unresectable
disease at presentation (p < .001), and age >16 years at diagnosis
(p = .02). Rajpal et al.53 reviewed the literature for adult patients
with pancreatoblastoma. Of the 15 adult patients, ages ranged
from 19 to 78 years. Thirteen of the 15 patients underwent surgical
resection. Eight of the patients died with recurrent disease with a
mean survival of only 12 months.
Answer: The median age at presentation of patients with
pancreatoblastoma is 5 years; however, this disease can present in
adults as well. Increased age at the time of diagnosis carries a poor
prognosis. Patients typically present with weight loss and abdominal pain.

Clinical Question Summary


Question

Answer

Grade of
Recommendation

References

1 Is endoscopic ultrasound the most


sensitive test used for preoperative
localization of insulinoma and
other pancreatic neuroendocrine
neoplasms?

Yes. EUS has a greater sensitivity than CT scanning


and MRI and should be used in the localization
of suspected insulinomas.

4-11

2 Are somatostatin analogs (octreotide)


helpful in controlling symptoms in
patients with VIPoma (vasoactive
intestinal polypeptide-oma)?

Yes. Management of VIPoma patients with longacting somatostatin analogs can be effective
at controlling symptoms, but should not be
anticipated to affect tumor size or bulk.

12-17

3 Should you ever electively operate


on a patient with a gastrinoma
without first controlling their
gastric acid secretion?

No. Gastric acid should be suppressed with


appropriate (often high dose) PPIs prior to
elective exploration for gastrinoma. To operate,
without controlling the ulcer diathesis risk
problems with perioperative gastrointestinal
ulceration.

18, 19

4 In patients with glucagonoma, does


resection of advanced local disease
or metastases increase overall
survival?

Large prospective multi-institutional trials are


lacking, but there is some evidence to suggest
that aggressive resection of a locally advanced
primary tumor and metastatic deposits can
improve survival in patients with glucagonoma.

18, 20-23

5 Are there any means to


preoperatively differentiate
between adenosquamous
carcinoma and adenocarcinoma?

There is currently no evidence to support any


means of effectively differentiating between
PASC and PDA preoperatively.

24-28

6 Are there molecular markers for


adenosquamous cancers that
can be used for prognosis or the
prediction of response to adjuvant
therapy?

There is no evidence currently available that shows


that molecular markers for PASC can predict
response to adjuvant therapy.

24, 29-31

7 Are there clinical measures to


preoperatively differentiate
between acinar cell carcinoma of
the pancreas and adenocarcinoma
of the pancreas?

Large retrospective reviews of databases have


shown that patients with ACC tend to be
white, male, and present at an earlier age
as compared with PDA patients. However,
these demographic features lack accuracy in
predicting the ultimate pathology in patients
with pancreatic neoplasia.

32-35

(Continued)

PMPH_CH58.indd 474

5/22/2012 5:40:31 PM

Unusual Pancreatic Tumors

475

(Continued)
Question

Answer

8 Is the overall prognosis worse for


patients with acinar cell carcinoma
of the pancreas when compared
with adenocarcinoma of the
pancreas?

Large retrospective database reviews have shown


that patients with ACC have a better overall
prognosis when compared with similarly treated
and staged patients with PDA.

33, 34

9 What is the most important


element, between surgery
and pathology, in ensuring the
proper diagnosis of MCN? What
information is essential to predict
prognosis?

The most important aspect in managing the


pathology specimen in patients with MCNs is
the clear communication between the surgeon
and the pathologist about the importance of a
complete pathologic analysis of the tumor, given
the propensity of MCNs to undergo scattered
malignant transformation. We recommend that
all MCNs be completely submitted and entirely
pathologically examined.

1, 36, 37

10 Should you perform a partial or a


total pancreatectomy for a main
duct IPMN?

At this time, there is no evidence to support a


routine total pancreatectomy over a partial
resection (typically right-sided) for an isolated
main duct IPMN. Total pancreatectomy renders
the patient an obligate insulin-dependent
diabetic. If partial pancreatectomy is performed,
the remnant pancreas needs to be kept under
radiographic surveillance.

1, 38-42

11 What is the appropriate


surveillance for patients who have
undergone resection for IPMN?

There is no level 1 evidence to suggest the exact


timing of surveillance screening for patients
who have had an IPMN resected. Patients who
have had a main duct IPMN are at an increased
risk for recurrence; so we believe they should
be screened more frequently than those with
branch duct IPMNs. We recommend yearly
or every other year MRI/MRCP for such
surveillance.

38, 40, 42

12 Can you reliably differentiate


preoperatively between SCN,
MCN, and IPMN?

Preoperative differentiation of cystic tumors of the


pancreas is a challenge. The literature is lacking
in large randomized prospective trials. The use
of cross-sectional imaging with CT scanning,
along with EUS visualization and cyst fluid
analysis, can provide information to differentiate
between these tumors; however, the accuracy is
not 100%.

1, 43-46

13 Do serous cystic neoplasms ever


undergo malignant transformation?

Although very low risk, SCNs do have malignant


potential with the capacity for infiltrative tumor
growth and metastatic disease.

47, 48

14 Are there genetic markers that can


predict phenotype and prognosis
for solid pseudopapillary tumors?

There is some evidence to suggest that measuring


the differential subcellular expression of
-catenin and cyclin D may be useful in
elucidating which patients will have a more
favorable phenotype. Greater evidence is needed
before tissue or serum biomarkers can be used
for accurate prediction of prognosis in SPTs.

49-51

15 What is the most common


age and presentation of
pancreatoblastoma?

The median age at presentation of patients with


pancreatoblastoma is 5 years; however, this
disease can present in adults as well. Increased
age at the time of diagnosis carries a poor
prognosis. Patients typically present with weight
loss and abdominal pain.

52-53

PMPH_CH58.indd 475

Grade of
Recommendation

References

5/22/2012 5:40:31 PM

476

Surgery: Evidence-Based Practice

REFERENCES
1. Adsay N. Cystic lesions of the pancreas. Mod Pathol. 2007;20:
s71-s93.
2. Sheehan M, Latona C, Aranha G, Pickleman J. The increasing
problem of unusual pancreatic tumors. Arch Surg. 2000;135(6):
644-648; discussion 648-650.
3. Heithz PU, Komminoth P, Perren A, et al. Tumors of the endocrine pancreas. In: DeLellis RA, Lloyd RV, Heitz PU, et al, eds.
Pathology and Genetics of Tumors of Endocrine Organs. Lyon,
France: IARC Press; 2004:175-208.
4. Halfdanarson TR, Rubin J, Farnell MB, et al. Pancreatic endocrine
neoplasms: Epidemiology and prognosis of pancreatic endocrine
tumors. Endocr Relat Cancer. 2008;15(2):409-427.
5. Service FJ, McMahon MM, OBrien PC, Ballard DJ. Functioning
insulinomaincidence, recurrence, and long-term survival of
patients: A 60-year study. Mayo Clin Proc. 1991;66(7):711-719.
6. Doppman JL, Miller DL, Chang R, et al. Insulinomas: Localization with selective intraarterial injection of calcium. Radiology.
1991;178(1):237-241.
7. Horton KM, Hruban RH, Yeo C, Fishman EK. Multi-detector
row CT of pancreatic islet cell tumors. Radiographics. 2006;26(2):
453-464.
8. Druce MR, Muthuppalaniappan VM, OLeary B, et al. Diagnosis
and localisation of insulinoma: The value of modern magnetic
resonance imaging in conjunction with calcium stimulation
catheterisation. Eur J Endocrinol. 2010;162(5):971-978.
9. Vaidakis D, Karoubalis J, Pappa T, et al. Pancreatic insulinoma:
Current issues and trends. Hepatobiliary Pancreat Dis Int. 2010;
9(3):234-241.
10. Zimmer T, Stolzel U, Bader M, et al. Endoscopic ultrasonography
and somatostatin receptor scintigraphy in the preoperative localisation of insulinomas and gastrinomas. Gut. 1996;39(4):562-568.
11. Rosch T, Lightdale CJ, Botet JF, et al. Localization of pancreatic
endocrine tumors by endoscopic ultrasonography. N Engl J Med.
1992;326(26):1721-1726.
12. Verner JV, Morrison AB. Islet cell tumor and a syndrome of
refractory watery diarrhea and hypokalemia. Am J Med. 1958;
25(3):374-380.
13. Warner RR. Enteroendocrine tumors other than carcinoid: A
review of clinically significant advances. Gastroenterology. 2005;
128(6):1668-1684.
14. Ghaferi AA, Chojnacki KA, Long WD, et al. Pancreatic VIPomas: Subject review and one institutional experience. J Gastrointest Surg. 2008;12(2):382-393.
15. Oberg K. Chemotherapy and biotherapy in the treatment of neuroendocrine tumours. Ann Oncol. 2001;12(Suppl 2):S111-S114.
16. Maton PN, Gardner JD, Jensen RT. Use of long-acting somatostatin analog SMS 201-995 in patients with pancreatic islet cell
tumors. Dig Dis Sci. 1989;34(Suppl 3):28S-39S.
17. Cho KJ, Vinik AI. Effect of somatostatin analogue (octreotide)
on blood flow to endocrine tumors metastatic to the liver: Angiographic evaluation. Radiology. 1990;177(2):549-553.
18. Morgan KA, Adams DB. Solid tumors of the body and tail of the
pancreas. Surg Clin North Am. 2010;90(2):287-307.
19. Kennedy EP, Brody JR, Yeo CJ. Neoplasms of the endocrine pancreas. In: Mulholland MW, Lillemoe KD, Doherty GM, et al.
eds. Greenfields Surgery: Scientific Principles and Practice. 5th
ed. Lippincott Williams & Wilkins; 2010:857-871.
20. Wermers RA, Fatourechi V, Wynne AG, et al. The glucagonoma
syndrome. Clinical and pathologic features in 21 patients. Medicine (Baltimore). 1996;75(2):53-63.
21. Chu QD, Al-kasspooles MF, Smith JL, et al. Is glucagonoma of the
pancreas a curable disease? Int J Pancreatol. 2001;29(3):155-162.

PMPH_CH58.indd 476

22. Sarmiento JM, Heywood G, Rubin J, et al. Surgical treatment of


neuroendocrine metastases to the liver: A plea for resection to
increase survival. J Am Coll Surg. 2003;197(1):29-37.
23. Hill JS, McPhee JT, McDade TP, et al. Pancreatic neuroendocrine
tumors: The impact of surgical resection on survival. Cancer.
2009;115(4):741-751.
24. Voong KR, Davison J, Pawlik TM, et al. Resected pancreatic adenosquamous carcinoma: Clinicopathologic review and evaluation of adjuvant chemotherapy and radiation in 38 patients. Hum
Pathol. 2010;41(1):113-122.
25. Yamaguchi K, Enjoji M. Adenosquamous carcinoma of the
pancreas: A clinicopathologic study. J Surg Oncol. 1991;47(2):
109-116.
26. Katz MH, Taylor TH, Al-Refaie WB, et al. Adenosquamous versus adenocarcinoma of the pancreas: A population-based outcomes analysis. J Gastrointest Surg. 2011;15(1):165-174.
27. Brody JR, Costantino CL, Potoczek M, et al. Adenosquamous
carcinoma of the pancreas harbors KRAS2, DPC4 and TP53
molecular alterations similar to pancreatic ductal adenocarcinoma. Mod Pathol. 2009;22(5):651-659.
28. Izuishi K, Takebayashi R, Suzuki Y. Electronic image of the
month. Adenosquamous carcinoma of the pancreas. Clin Gastroenterol Hepatol. 2010;8(4):e40.
29. Bunger S, Laubert T, Roblick UJ, Habermann JK. Serum biomarkers for improved diagnostic of pancreatic cancer: A current
overview. J Cancer Res Clin Oncol. 2011;137(3):375-389.
30. Costantino CL, Witkiewicz AK, Kuwano Y, et al. The role of HuR
in gemcitabine efficacy in pancreatic cancer: HuR Up-regulates
the expression of the gemcitabine metabolizing enzyme deoxycytidine kinase. Cancer Res. 2009;69(11):4567-4572.
31. Richards NG, Rittenhouse DW, Freydin B, et al. HuR status is a
powerful marker for prognosis and response to gemcitabine-based
chemotherapy for resected pancreatic ductal adenocarcinoma
patients. Ann Surg. 2010;252(3):499-505; discussion 505-506.
32. Matos JM, Schmidt CM, Turrini O, et al. Pancreatic acinar cell
carcinoma: A multi-institutional study. J Gastrointest Surg.
2009;13(8):1495-1502.
33. Schmidt CM, Matos JM, Bentrem DJ, et al. Acinar cell carcinoma of the pancreas in the United States: Prognostic factors
and comparison to ductal adenocarcinoma. J Gastrointest Surg.
2008;12(12):2078-2086.
34. Wisnoski NC, Townsend CM, Jr., Nealon WH, et al. 672 patients
with acinar cell carcinoma of the pancreas: A population-based
comparison to pancreatic adenocarcinoma. Surgery. 2008;144(2):
141-148.
35. Seth AK, Argani P, Campbell KA, et al. Acinar cell carcinoma
of the pancreas: An institutional series of resected patients and
review of the current literature. J Gastrointest Surg. 2008;12(6):
1061-1067.
36. Roggin KK, Chennat J, Oto A, et al. Pancreatic cystic neoplasm.
Curr Probl Surg. 2010;47(6):459-510.
37. Zamboni G, Scarpa A, Bogina G, et al. Mucinous cystic tumors
of the pancreas: Clinicopathological features, prognosis, and
relationship to other mucinous cystic tumors. Am J Surg Pathol.
1999;23(4):410-422.
38. Verbesey JE, Munson JL. Pancreatic cystic neoplasms. Surg Clin
North Am. 2010;90(2):411-425.
39. Woo SM, Ryu JK, Lee SH, et al. Branch duct intraductal papillary
mucinous neoplasms in a retrospective series of 190 patients. Br J
Surg. 2009;96(4):405-411.
40. Sohn TA, Yeo CJ, Cameron JL, et al. Intraductal papillary mucinous neoplasms of the pancreas: An increasingly recognized clinicopathologic entity. Ann Surg. 2001;234(3):313-321; discussion
321-322.

5/22/2012 5:40:31 PM

Unusual Pancreatic Tumors

41. Adsay NV, Conlon KC, Zee SY, et al. Intraductal papillarymucinous neoplasms of the pancreas: An analysis of in situ and
invasive carcinomas in 28 patients. Cancer. 2002;94(1):62-77.
42. Chari ST, Yadav D, Smyrk TC, et al. Study of recurrence after surgical resection of intraductal papillary mucinous neoplasm of the
pancreas. Gastroenterology. 2002;123(5):1500-1507.
43. Lee SE, Kwon Y, Jang JY, et al. The morphological classification
of a serous cystic tumor (SCT) of the pancreas and evaluation of
the preoperative diagnostic accuracy of computed tomography.
Ann Surg Oncol. 2008;15(8):2089-2095.
44. Shah AA, Sainani NI, Kambadakone AR, et al. Predictive value
of multi-detector computed tomography for accurate diagnosis
of serous cystadenoma: Radiologic-pathologic correlation. World
J Gastroenterol. 2009;15(22):2739-2747.
45. Frossard JL, Amouyal P, Amouyal G, et al. Performance of
endosonography-guided fine needle aspiration and biopsy in the
diagnosis of pancreatic cystic lesions. Am J Gastroenterol. 2003;
98(7):1516-1524.
46. Allen PJ, Qin LX, Tang L, et al. Pancreatic cyst fluid protein expression profi ling for discriminating between serous cystadenoma
and intraductal papillary mucinous neoplasm. Ann Surg. 2009;
250(5):754-760.
47. Galanis C, Zamani A, Cameron JL, et al. Resected serous cystic neoplasms of the pancreas: A review of 158 patients with

PMPH_CH58.indd 477

48.

49.

50.

51.

52.

53.

54.

477

recommendations for treatment. J Gastrointest Surg. 2007;11(7):


820-826.
Strobel O, ZGraggen K, Schmitz-Winnenthal FH, et al. Risk of
malignancy in serous cystic neoplasms of the pancreas. Digestion. 2003;68(1):24-33.
Papavramidis T, Papavramidis S. Solid pseudopapillary tumors
of the pancreas: Review of 718 patients reported in English literature. J Am Coll Surg. 2005;200(6):965-972.
Kim CW Han D, Kim J, Kim YH, Park JB, Kim SC. Solid pseudopapillary tumor of the pancreas: Can malignancy be predicted?
Surgery. 2011;149(5):625-634.
Tanaka Y, Kato K, Notohara K, et al. Frequent beta-catenin
mutation and cytoplasmic/nuclear accumulation in pancreatic
solid-pseudopapillary neoplasm. Cancer Res. 2001;61(23):84018404.
Levey JM, Banner BF. Adult pancreatoblastoma: A case report
and review of the literature. Am J Gastroenterol. 1996;91(9):18411844.
Rajpal S, Warren RS, Alexander M, et al. Pancreatoblastoma in
an adult: Case report and review of the literature. J Gastrointest
Surg. 2006;10(6):829-836.
Dhebri AR, Connor S, Campbell F, et al. Diagnosis, treatment and
outcome of pancreatoblastoma. Pancreatology. 2004;4(5):441-451;
discussion 452-453.

5/22/2012 5:40:31 PM

PMPH_CH58.indd 478

5/22/2012 5:40:31 PM

PART IX

THE SPLEEN

PMPH_CH59.indd 479

5/22/2012 5:41:03 PM

PMPH_CH59.indd 480

5/22/2012 5:41:03 PM

CHAPTER 59

Hematologic Indications
for Splenectomy
Mark T. Muir

INTRODUCTION

transfusions over six months, and patients with pain related to


splenomegaly also had symptomatic improvement.6 Splenectomy
had no impact on survival. Sadamori and Sandberg studied a
cohort of 53 subjects in blastic phase and divided them according
to karyotype.7 They report that splenectomy improved survival as
well as increased the response to chemotherapy in subjects with
certain chromosomal abnormalities in addition to the Philadelphia chromosome. A large retrospective review of 358 patients
undergoing allogeneic bone marrow transplant, 68 of whom had
previously undergone splenectomy, found no effect of splenectomy on acute or chronic graft-versus-host disease or infectious
complications.8 There was a significant increase in the rate of leukemic relapse in the splenectomy group, but splenomegaly is likely
the major risk factor for relapse rather that the splenectomy itself.
Finally, there is anecdotal evidence that splenectomy following
bone marrow transplant may hasten engraftment.9 In a series of
four patients with splenomegaly and poor engraftment following
allogeneic bone marrow transplant, all four patients had rapid
hematologic recovery and ceased to require transfusions within a
month of splenectomy.

Traumatic injury to the spleen, including both external and iatrogenic injuries, accounts for the most common indication for splenectomy. Of the remaining indications for splenectomy, staging
for Hodgkins lymphoma has historically been the most common
reason for splenectomy.1 More recently, idiopathic thrombocytopenic purpura (ITP) has become the most common indication
for elective splenectomy.2,3 This chapter discusses the evidence for
splenectomy in various hematologic diseases, both malignant and
benign. A discussion of the evidence regarding the role of laparoscopic splenectomy in hematologic diseases is also included.
1. What are the indications for splenectomy in chronic myeloid
leukemia?
Chronic myeloid leukemia (CML) is a disorder of pluripotent
bone marrow stem cells, and is characterized by the presence of
a chromosomal translocation resulting in a fusion between the
abl gene on chromosome 22 and the bcr gene on chromosome 9.
This Philadelphia chromosome encodes for a tyrosine kinase
implicated in the leukemic transformation.4 As the disease progresses, patients may experience splenomegaly with subsequent
sequestration of platelets and erythrocytes. Patients may also
experience mechanical symptoms directly related to the splenomegaly, including left upper quadrant pain and early satiety. The
role of splenomegaly has been investigated in both early- and latestage CML. A prospective trial of 189 patients in the early indolent phase of CML randomized them to either splenectomy or no
surgery, and all of the patients underwent the same chemotherapy
regimen. Splenectomy did not influence either disease progression
or survival, but patients in the splenectomy group had a higher
incidence of both thrombotic and hemorrhagic complications.5
Based on this data, the authors conclude that splenectomy is not
indicated in the early stage of CML. A retrospective review of 53
patients with CML in either the accelerated phase or the blastic
phase found that patients with transfusion-dependent thrombocytopenia has a significant reduction in both red cell and platelet

RECOMMENDATIONS
1) In the early indolent stage of CML, splenectomy does not
provide a survival benefit and is associated with thrombotic and
hemorrhagic complications. This recommendation is supported
by a randomized controlled trial. (Grade A recommendation)
2) In the accelerated or blastic phase of CML, patients with
transfusion-dependent thrombocytopenia, anemia, or symptomatic splenomegaly experienced symptomatic relief but no survival benefit after splenectomy. (Grade B recommendation)
2. What are the indications for splenectomy in nonleukemic
chronic myeloid disorders?
Myelofibrosis with myeloid metaplasia (MMM) refers to a group
of related bone marrow disorders including agnogenic myeloid
481

PMPH_CH59.indd 481

5/22/2012 5:41:04 PM

482

Surgery: Evidence-Based Practice

metaplasia, postthrombocythemic myeloid metaplasia (advancedstage essential thrombocythemia), and postpolycythemic myeloid
metaplasia (advanced-stage polycythemia vera). MMM is associated with varying degrees of splenomegaly, as extramedullary
hematopoiesis increases to compensate for the diminished marrow hematopoiesis caused by progressive fibrosis of the bone
marrow.10 Splenectomy in patients with MMM has been reserved
for those patients experiencing significant sequelae of splenomegaly, including thrombocytopenia, transfusion-dependent anemia, and pain. All of the literature in support of splenectomy for
MMM comes in the form of case series. In the largest published
series, the results of 314 patients undergoing splenectomy over a
28-year period were described.11 Indications for splenectomy in
this group of patients included pain (49%), transfusion-dependent
anemia (25%), symptoms related to portal hypertension (15%),
and thrombocytopenia (11%). The authors report that for those
patients undergoing splenectomy within the most recent decade,
76% of patients experienced symptomatic improvement for a
median duration of 12 months. Twenty-eight percent of patients
experienced postoperative morbidity, including infection (10%),
thrombosis (10%), and bleeding (14%), as well as a 7% mortality. Of
the factors evaluated by the authors, only preoperative thrombocytopenia had a statistically significant association with decreased
survival. Another series of 71 patients undergoing splenectomy
for MMM found similar rates of morbidity (39%) and mortality
(8%).12 The authors report that hemorrhagic and/or thrombotic
complications occurred in 17% of patients, and rapidly progressive hepatomegaly occurred in 24% of patients. They report
improved survival in younger patients (<45 years) and in patients
with a baseline leukocyte count <10 106/mL count. There is some
evidence that patients undergoing splenectomy for MMM may
have a higher incidence of leukemic transformation.12-14 However,
there does not seem to be any decrease in the survival for splenectomized patients with leukemic transformation compared to
splenectomized patients without transformation.15 It is important
to note that while the literature with regard to splenectomy in the
setting of MMM results in a high rate of symptomatic improvement, splenectomy has not been shown to increase survival.16
Essential thrombocythemia (ET) and polycythemia vera (PV)
are myeloproliferative disorders of the megakaryocyte and erythrocyte cell lineages, respectively.10 Splenomegaly may occur during the course of either disease, but splenectomy has not been
recommended in the early phase of these diseases due to a risk
hemorrhage.17,18 However, in the limited number of subjects with
either ET or PV who progress to myeloid metaplasia (postthrombocytocythemic myeloid metaplasia for ET and postpolycythemic
myeloid metaplasia for PV), progressive splenomegaly may
accompany bone marrow fibrosis.19 Since both of these conditions
are extremely rare, and progression to the myeloid metaplasia
stage even more uncommon, there is little evidence evaluating the
role of splenectomy. It is generally agreed that splenectomy plays
no role in the early course of either ET or PV, but may be considered for relief of symptoms due to splenomegaly in the myeloid
metaplasia phase.20

RECOMMENDATIONS
1) Splenectomy in the setting of MMM is recommended only for
palliation of symptoms attributable to splenomegaly, as there

PMPH_CH59.indd 482

is no evidence for survival benefit following splenectomy. The


decision regarding whether to perform elective splenectomy
for any of the above symptoms of splenomegaly should be
individualized, based on a discussion of the patients degree
of symptoms and preoperative surgical risk factors. (Grade B
recommendation)
2) The results of small series of patients suggest that splenectomy
is contraindicated in the early course of PV and ET. (Grade C
recommendation)
3. What are the indications for splenectomy in non-Hodgkins
lymphoma (NHL) and nonmyeloid leukemia?
Historically, laparotomy (including splenectomy) was performed
as a routine part of the staging evaluation for patients with
Hodgkins lymphoma.1 More recently, laparotomy and splenectomy were applied selectively in those cases in which intraabdominal or splenic involvement would change the planned
therapy.21 Recent advances in imaging technology, most notably
the use of PET scans, have relegated splenectomy and staging
laparotomy to historical significance only.22-24 Current recommendations do not include a role for splenectomy in the staging of
Hodgkins lymphoma.
NHL is the term encompassing all malignancies of lymphoid
origin exclusive of classical Hodgkins lymphoma. Two 10-year
retrospective reviews totaling 81 patients with NHL undergoing
splenectomy identified the indications as diagnostic (5 patients),
hematologic abnormalities (28 patients), painful splenomegaly
(8 patients), and combined hematologic abnormalities and splenomegaly (40 patients).25,26 The authors report postsplenectomy
response rates for leukopenia, thrombocytopenia, and anemia
of 82% to 94%, 68% to 89%, and 50% to 64%, respectively. These
studies show a mortality rate of 3% to 9% and morbidity of 21%
to 37%, primarily infectious complications. The introduction of
rituximab (anti-CD20 monoclonal antibody) has changed the
indications for splenectomy in patients with responsive variants of NHL, including diff use large B-cell CD20-positive NHL,
follicular or low-grade CD20-positive B-cell NHL, and splenic
marginal zone B-cell NHL.27,28 Where previously splenectomy
was indicated for the B-cell lymphomas for the indications discussed above, rituximab has been shown to induce response
rates of 88% to 100% and sustained responses of 60% to 88%.28,29
Currently splenectomy is reserved for patients with symptomatic splenomegaly or cytopenias not responsive to rituximab.
There are, however, no direct comparisons of rituximab and
splenectomy.
Chronic lymphocytic leukemia (CLL) and hairy cell leukemia (HCL) are malignancies of the lymphocyte cell lineage.
The traditional indications for splenectomy in CLL and HCL are
similar to those discussed for NHL, namely to correct refractory
cytopenia or to treat symptomatic splenomegaly. Indications for
splenectomy in a retrospective review of 77 patients with CLL
over a 24-year period included hypersplenism (51%), immune
thrombocytopenia (21%), hemolytic anemia (20%), and miscellaneous (9%).30 Hematologic response was most pronounced
in those patients with the lowest baseline values, with 69% of
subjects with baseline hemoglobin < 10 g/dL having an excellent response, and 61% of subjects with platelet counts < 50
106 per mL also having an excellent response. The authors also
identified a survival advantage for splenectomized patients with

5/22/2012 5:41:04 PM

Hematologic Indications for Splenectomy

hemoglobin < 10 g/dL or platelets < 50 106 per mL. But like
many forms of NHL, CD20-positive B-cell forms of CLL are now
routinely treated with rituximab as fi rst-line therapy, with splenectomy reserved for treatment failures.27 HCL is an uncommon
B-cell lymphocytic leukemia, and like CLL retrospective studies have shown splenectomy to be effective in alleviating symptoms of splenomegaly and improving cytopenias in 40% to 70%
of patients.31,32 The introduction of purine nucleoside analogs
has since transformed the treatment of HCL with many patients
achieving a durable response, such that splenectomy is rarely
indicated.33

RECOMMENDATIONS
1) Multiple retrospective studies demonstrate splenectomy to result
in a durable improvement in cytopenia in patients with NHL.
However, in patients with CD20-positive B-cell NHL, rituximab
is the first-line therapy and splenectomy should be reserved for
treatment failures. (Grade C recommendation)
2) Consistent retrospective data demonstrate an improvement in
cytopenia following splenectomy for CLL or HCL. But similar
to NHL, immuno- or chemotherapy are considered first-line
treatments for most patients with CLL or HCL, and splenectomy
is reserved for treatment failures. (Grade C recommendation)
4. What are the indications for splenectomy in immune
thrombocytopenia?
Immune thrombocytopenia (ITP) is an uncommon disorder, so it
has only been within the past decade that rigorous evidence has
emerged directly comparing the effectiveness of various treatment regimens. Since the widespread use of glucocorticoids began
in the 1960s, steroids have been the standard first-line therapy for
ITP, and splenectomy has been the standard therapy for patients
who fail to have an adequate response to steroids.34 The introduction of rituximab (monoclonal antibody to CD20), and even more
recently the thrombopoietin-mimetics (romiplostim and eltrombopag), have provided additional medical alternatives for those
patients who fail steroid therapy. As opposed to glucocorticoids
and the thrombopoietin-mimetic agents, which require ongoing maintenance therapy, both rituximab and splenectomy are
capable of inducing complete remission. Splenectomy was the first
treatment for ITP, and remains the most effective. A systematic
review of splenectomy for ITP spanning 58 years and including
2623 patients demonstrated a partial or complete response of 86%.
Two-thirds of patients underwent complete remission (essentially
cured) and recurrences were rare.35 There are also data demonstrating that the risk of severe infection is no greater in patients
with ITP after splenectomy than in ITP patients who have not had
a splenectomy.36 A recent international consensus report on the
management of ITP gives splenectomy a Grade C recommendation regarding the level of evidence.37

RECOMMENDATIONS
Splenectomy is effective in achieving a lasting improvement in
thrombocytopenia in ITP, but given the extensive evidence (including multiple RCTs) in favor of medical treatment as first-line therapy,

PMPH_CH59.indd 483

483

splenectomy should be reserved for patients who have failed treatment with at least two classes of medication. Unfortunately, little
data exist comparing splenectomy to the newer therapeutic agents
as second- or third-line therapy. (Grade C recommendation)
5. What are the indications for splenectomy in thrombotic
thrombocytopenic purpura?
Thrombotic thrombocytopenic purpura (TTP) is a rare but serious disorder characterized by microangiopathic hemolytic anemia, consumptive thrombocytopenia, neurological deficits, and
renal insufficiency.38 First-line therapy is plasma exchange with
fresh frozen plasma, and both rituximab (anti-CD20 monoclonal
antibody) and splenectomy have been recommended as secondline for plasma-refractory or relapsing cases.39 A recent systematic review of the literature identified 18 studies with 87 subjects
undergoing splenectomy for relapsing TTP, and 15 studies with
74 subjects undergoing splenectomy for refractory TTP.40 Patients
undergoing splenectomy for relapsing TTP had a lower mortality
(1.2% vs. 5%) and morbidity (6% vs. 10%). The relapse rate following splenectomy was low in both groups, but relapsing patients
had a higher rate of recurrence (17% vs. 8%) than patients with
refractory TTP.

RECOMMENDATIONS
Although there are multiple retrospective reviews demonstrating
the effectiveness of splenectomy in relapsing or refractory TTP,
there are no data comparing splenectomy directly to rituximab
in this setting. (Grade D recommendation)
6. What are the indications for splenectomy in hemolytic
anemia?
Sickle cell disease (SCD) is an inherited disorder of hemoglobin, and results in polymerization of deoxygenated hemoglobin
molecules and subsequent sickling of the erythrocyte. Splenic
sequestration of erythrocytes and resultant splenomegaly are
typical, and many patients eventually undergo splenic infarction
and autosplenectomy.41 In three retrospective reviews including a total of 343 patients with sickle cell anemia undergoing
splenectomy, indications for splenectomy were major sequestration crisis or recurrent minor sequestration crises (6077%),
hypersplenism (1336%), splenic abscess (511%), and massive
splenic infarction (12%).42-44 All three studies demonstrated
improvement in hemoglobin level following splenectomy in
those patient with hypersplenism, in addition to eliminating
the risk of future splenic sequestration crises. There were no
recorded mortalities and the rate of postoperative complications was 6% to 9%, confi rming the safety of splenectomy in
this patient population.
Thalassemias are a group of inherited disorders of hemoglobin
production resulting in microcytic anemia, and in the homozygous form (-thalassemia major) require regular transfusion to
maintain hemoglobin levels above 10 g/dL.45 The data regarding the role of splenectomy in thalassemia are largely retrospective in nature, and much of it arises from the Arabian Peninsula
and southeastern Asia due to the high prevalence of thalassemia (the thalassemia belt). A retrospective study of 24 children with -thalassemia major who had undergone splenectomy

5/22/2012 5:41:04 PM

484

Surgery: Evidence-Based Practice

demonstrated a significant increase in mean hemoglobin level


in those patients who were not transfusion-dependent preoperatively, and a significant decrease in transfusion requirements in
those patients who were transfusion-dependent prior to surgery.46
Among three other retrospective reviews involving a total of 67
patients with -thalassemia major, the indications for splenectomy were increased transfusion requirements (> 200 mL/kg/
year) with or without massive splenomegaly in all patients, except
for one patient presenting with splenic abscess.42,44,45 These studies
all demonstrated a significant increase in postsplenectomy hemoglobin level and a significant decrease in postsplenectomy transfusion requirements. Vaccines against encapsulated organisms were
administered when available, and prophylaxis with oral penicillin
was given in the absence of vaccines. There were no mortalities
and no cases of postsplenectomy sepsis. This demonstrates that
splenectomy is effective in increasing mean hemoglobin concentration in -thalassemia major without significant morbidity. One
prospective cross-sectional study found a significant association
between prior splenectomy and pulmonary hypertension in thalassemia patients (75.8% of patients with pulmonary hypertension
had splenectomy vs. 25.6% of patients without pulmonary hypertension, 95% confidence interval 3.027.7).47 The study design precludes a determination of causality; however, splenectomy may
simply be an indicator of more advanced disease.
Hereditary spherocytosis (HS) is an inherited disorder of the
erythrocyte membrane resulting in hemolytic anemia and subsequent splenomegaly.48 A systematic review of the literature concluded that the evidence supports splenectomy in patients with
severe HS (hemoglobin < 8 g/dL), and that splenectomy should be
considered on an individual basis in patients with moderate HS
(hemoglobin 812 g/dL and reticulocyte count > 6%).49 Splenectomy is not recommended in patients with mild HS (hemoglobin
> 11 g/dL and reticulocyte count < 6%). Patients with symptomatic
cholelithiasis should undergo splenectomy at the time of cholecystectomy. Patients with cholelithiasis but without symptoms may
be considered for stone removal rather than cholecystectomy.
There is also increasing data on the role of partial splenectomy,
but the level of evidence at this time is poor.
Autoimmune hemolytic anemia (AIHA) is an autoantibodymediated hemolytic anemia that frequently presents with splenomegaly as well. Initial treatment is typically with corticosteroids,
with IVIG as second-line treatment. 50 Splenectomy may be
considered for patients who have failed medical therapy, as
the spleen both produces the autoantibodies and sequesters
the coated erythrocytes.51 The data in support of splenectomy
for AIHA are derived from small case series, demonstrating a
response to splenectomy ranging from 54% to 100%.42,44,50,52,53

RECOMMENDATIONS
1) Splenectomy is safe and results in significant improvement
in hemoglobin levels in sickle cell patients with recurrent
splenic sequestration crises or hypersplenism. (Grade C
recommendation)
2) Splenectomy leads to increases in hemoglobin level and
decreased transfusion requirements in -thalassemia, and is
indicated in patients receiving blood transfusion greater than
200 mL/kg/year. (Grade C recommendation)

PMPH_CH59.indd 484

3) Splenectomy should be performed in patients with severe HS


and avoided in patients with mild HS. Patients with concurrent
cholelithiasis should have surgical intervention at the same
time as splenectomy. (Grade B recommendation)
4) Data for the benefit of splenectomy in AIHA come from small
case series but are consistent. (Grade C recommendation)
7. What are the benefits of splenectomy for Feltys syndrome
(rheumatoid arthritis, splenomegaly, and neutropenia)?
Feltys syndrome comprises a well-known triad of rheumatoid
arthritis, splenomegaly, and neutropenia, although it occurs in
less than 1% of patients with rheumatoid arthritis.54 The splenomegaly of Feltys syndrome may result in portal venous hypertension
and its sequelae, including bleeding complications from varices.
Splenectomy in patients with portal hypertension and esophageal varices has been shown to reduce portal venous pressure
and prevent future episodes of variceal bleeding.55,56 In patients at
higher risk of surgical complications, the risks of serious adverse
events from variceal hemorrhage must be weighed against the risk
of splenectomy. For this reason, some authors advocate medical
management alone in elderly patients or those with significant
surgical risk factors.57

RECOMMENDATIONS
Splenectomy can reduce portal venous pressure and prevent episodes of variceal bleeding, but this data is based on small case
series. (Grade C recommendation)
8. What is the role of laparoscopic splenectomy in hematologic
disease?
There are no randomized trials comparing the safety and effectiveness of laparoscopic and open splenectomy in hematologic
disorders. The data therefore are either in the form of retrospective reviews or prospective cohort studies. A retrospective review
of data from the Italian Registry of Laparoscopic Surgery of the
Spleen (IRLSS) identified 309 patients undergoing laparoscopic
splenectomy since 1993.58 The size of spleens removed ranged from
85 to 4500 g, and they experienced a 7% conversion rate, 0.6% rate
of perioperative mortality, and 18% morbidity (fever, pleural eff usion, and hemorrhage), with 2% of patients undergoing re-operation
for bleeding. They conclude that laparoscopic splenectomy may
represent the new gold standard, but this is difficult to support
without comparison to an open splenectomy group. Another
retrospective review of 186 patients undergoing laparoscopic
splenectomy for hematologic disease focused on long-term hematologic outcomes.59 The study had a mean follow-up period of 35
months, and the most common indications for splenectomy were
ITP, followed by hematologic malignancy, HS, and autoimmune
hemolytic anemia. The authors report a remission rate of 89% for
ITP, 100% for HS, and 70% for autoimmune hemolytic anemia.
Mortality was 22% in the malignancy group and 5% for the other
disorders. These results are comparable to those for open splenectomy. In an attempt to compare results between laparoscopic
and open splenectomy, Sapucahy et al. compared a prospectively
studied group undergoing laparoscopic splenectomy (n = 30) to a
retrospective group undergoing open splenectomy (n = 28).60 The

5/22/2012 5:41:04 PM

Hematologic Indications for Splenectomy

conversion rate was 13%, and laparoscopic procedures took significantly longer to complete (261 83 vs. 184 71 min, p = 0.0004),
but there was no significant difference in postoperative hospital
stay, amount of blood transfused, or postoperative complications.
Rate of recurrence of the original disorder was similar in the two
groups. In another series of 137 patients undergoing splenectomy for hematologic disease (63 laparoscopic and 74 open), the
authors similarly found a longer operative time for laparoscopic
procedures, but they also noted a significantly decreased postoperative hospital stay in the laparoscopic group (3.5 2.3 vs. 6.7
3.1 days, P < 0.01), as well as significantly increased blood transfusions in the open group.61 Complications were again similar in the
two groups. Two additional retrospective studies including 258
patients undergoing laparoscopic splenectomy for hematologic
disease evaluated the impact of preoperative diagnosis on postoperative outcomes.3,62 Both studies found that patients undergoing

485

laparoscopic splenectomy for ITP, and perhaps for hemolytic anemia, had significantly shorter hospital stays compared to other
indications. The larger study also found significantly shorter
operative times, less intraoperative blood loss, and lower conversion rates for patients with ITP.3

RECOMMENDATIONS
Laparoscopic splenectomy may result in shorter hospital stays,
and has similar rates of complications compared to open splenectomy. More data are needed to determine if laparoscopic or open
splenectomy is preferred for certain hematologic diseases. Laparoscopic splenectomy is an acceptable approach for hematologic
disease, including patients with massive splenomegaly. (Grade B
recommendation)

Clinical Question Summary


Question

Answer

Grade of
Recommendation

1 What are the indications


for splenectomy
in chronic myeloid
leukemia?

1. Splenectomy is not indicated during the early indolent phase


2. Splenectomy is indicated for symptomatic splenomegaly (pain,
thrombocytopenia, or anemia) in the accelerated or blastic phase

A
B

2 What are the indications


for splenectomy in
nonleukemic chronic
myeloid disorders?

1. MMMSymptomatic splenomegaly (persistent pain, thrombocytopenia,


or anemia)
2. PV/ETNo role for splenomegaly until spent phase, then treat as MMM

3 What are the indications


for splenectomy in
non-Hodgkins lymphoma
and nonmyeloid
leukemia?

1. NHLSymptomatic splenomegaly or refractory cytopenia in non


CD20-positive patients
2. CLL/HCLSymptomatic splenomegaly or refractory cytopenia in non
CD20-positive patients (CLL) or medically refractory disease (HCL)

4 What are the


indications for
splenectomy in immune
thrombocytopenia?

Splenectomy should be reserved for patients that have failed at least


two medical treatments (corticosteroids and either rituximab or a
thrombopoietin-mimetic agent)

5 What are the indications


for splenectomy
in thrombotic
thrombocytopenic
purpura?

Splenectomy or rituximab may be used in relapsing or plasma exchangerefractory thrombotic thrombocytopenic purpura

6 What are the indications


for splenectomy in
hemolytic anemia?

1. SCAHypersplenism or recurrent splenic sequestration crises


2. -Thalassemia majorTransfusion > 200 mL/kg/year
3. HSSevere disease (Hgb < 8 g/dL), or patients undergoing
cholecystectomy for symptomatic cholelithiasis
4. AIHAMedically refractory disease

C
C
B

7 What are the benefits of


splenectomy for Feltys
syndrome?

Splenectomy reduces portal venous pressure and future variceal bleeding in


patients with portal hypertension

8 What is the role of


laparoscopic splenectomy
in hematologic disease?

Laparoscopic splenectomy is an acceptable approach for hematologic disease,


including patients with massive splenomegaly

Abbreviations: Hgb, hemoglobin; SCA, sickle cell anemia.

PMPH_CH59.indd 485

5/22/2012 5:41:04 PM

486

Surgery: Evidence-Based Practice

REFERENCES
1. Schwartz SI, Cooper RA Jr. Surgery in the diagnosis and treatment of Hodgkins disease. Adv Surg. 1972;6:175-203.
2. Katkhouda N, Hurwitz MG, Rivera RT, et al. Laparoscopic splenectomy: Outcome and efficacy in 103 consecutive patients. Ann
Surg. 1998;228:568-578.
3. Rosen M, Brody F, Walsh RM, et al. Outcome of laparoscopic
splenectomy based on hematologic indication. Surg Endosc. 2002;
16:272-279.
4. Daley G, Van Etten R, Baltimore D. Induction of chronic myelogenous leukemia in mice by the P210bcr/abl gene of the Philadelphia chromosome. Science. 1990;247:824-830.
5. The Italian Cooperative Study Group on Chronic Myeloid Leukemia. Results of a prospective randomized trial of early splenectomy in chronic myelogenous leukemia. Cancer. 1984;54:
333-338.
6. Bouvet M, Babiera G, Termuhlen P, et al. Splenectomy in the accelerated or blastic phase of chronic myelogenous leukemia: A singleinstitution, 25-year experience. Surgery. 1997;122:20-25.
7. Sadamori N, Sandberg A. Chromosome changes and splenectomy
in Ph1-positive chronic myeloid leukemia. I. Predictive parameters in the blastic phase. Cancer. 1984;54:2456-2459.
8. Kalhs P, Schwarzinger I, Anderson G, et al. A retrospective analysis of the long-term effect of splenectomy on late infections,
graft-versus-host disease, relapse, and survival after allogeneic
marrow transplantation for chronic myelogenous leukemia. Blood.
1995;86:2028-2032.
9. Richard C, Romn I, Perez-Encinas M, et al. Splenectomy for
poor graft function after allogeneic bone marrow transplantation in patients with chronic myeloid leukemia. Leukemia. 1996;
10:1615-1618.
10. Mesa R, Elliott M, Tefferi A. Splenectomy in chronic myeloid
leukemia and myelofibrosis with myeloid metaplasia. Blood Rev.
2000;14:121-129.
11. Mesa R, Nagorney D, Schwager S, et al. Palliative goals, patient
selection, and perioperative platelet management: Outcomes
and lessons from 3 decades of splenectomy for myelofibrosis
with myeloid metaplasia at the Mayo Clinic. Cancer. 2006;107:
361-370.
12. Barosi G, Ambrosetti A, Buratti A, et al. Splenectomy for patients
with myelofibrosis with myeloid metaplasia: Pretreatment variables and outcome prediction. Leukemia. 1993;7:200-206.
13. Barosi G, Ambrosetti A, Centra A, et al. Splenectomy and risk
of blast transformation in myelofibrosis with myeloid metaplasia. Italian Cooperative Study Group on Myeloid with Myeloid
Metaplasia. Blood. 1998;91:3630-3636.
14. Akpek G, McAneny D, Weintraub L. Risks and benefits of splenectomy in myelofibrosis with myeloid metaplasia: A retrospective analysis of 26 cases. J Surg Oncol. 2001;77:42-48.
15. Tefferi A, Mesa R, Nagorney D, et al. Splenectomy in myelofibrosis with myeloid metaplasia: A single-institution experience with
223 patients. Blood. 2000;95:2226-2233.
16. Benbassat J, Gilon D, Penchas S. The choice between splenectomy and medical treatment in patients with advanced agnogenic myeloid metaplasia. Am J Hematol. 1990;33:128-135.
17. Jacobs P, Wood L, Dent D. Splenectomy in the chronic myeloproliferative syndromes. A retrospective risk-versus-benefit analysis. S Afr Med J. 1992;81:499-503.
18. Ravich R, Gunz F, Thompson I, Reed C. The dangers of surgery
in uncontrolled haemorrhagic thrombocythaemia. Med J Aust.
1970;1:704-708.

PMPH_CH59.indd 486

19. Wang J, Wang A. Plasma soluble interleukin-2 receptor in patients


with primary myelofibrosis. Br J Haematol. 1994;86:380-382.
20. Silverstein M. The evolution into and the treatment of late stage
polycythemia vera. Semin Hematol. 1976;13:79-84.
21. Armitage J. Early-stage Hodgkins lymphoma. N Engl J Med. 2010;
363:653-662.
22. Isasi C, Lu P, Blaufox M. A metaanalysis of 18F-2-deoxy-2-fluoroD-glucose positron emission tomography in the staging and restaging of patients with lymphoma. Cancer. 2005;104:1066-1074.
23. Juweid M. Utility of positron emission tomography (PET) scanning in managing patients with Hodgkin lymphoma. Hematology Am Soc Hematol Educ Prog. 2006:259-265.
24. Podoloff D, Advani R, Allred C, et al. NCCN task force report:
Positron emission tomography (PET)/computed tomography (CT)
scanning in cancer. J Natl Compr Canc Netw. 2007;5(Suppl 1):
S1-S22.
25. Lehne G, Hannisdal E, Langholm R, et al. A 10-year experience
with splenectomy in patients with malignant non-Hodgkins
lymphoma at the Norwegian Radium Hospital. Cancer. 1994;74:
933-939.
26. Kehoe J, Daly J, Straus D, DeCosse J. Value of splenectomy in
non-Hodgkins lymphoma. Cancer. 1985;55:1256 -1264.
27. Keating G. Spotlight on rituximab in chronic lymphocytic leukemia, low-grade or follicular lymphoma, and diff use large B-cell
lymphoma. BioDrugs. 2011;25:55-61.
28. Thieblemont C, Felman P, Berger F, et al. Treatment of splenic
marginal zone B-cell lymphoma: An analysis of 81 patients. Clin
Lymphoma. 2002;3:41-47.
29. Bennett M, Schechter G. Treatment of splenic marginal zone
lymphoma: Splenectomy versus rituximab. Semin Hematol. 2010;
47:143-147.
30. Cusack J Jr, Seymour J, Lerner S, et al. Role of splenectomy in
chronic lymphocytic leukemia. J Am Coll Surg. 1997;185:237-243.
31. Golomb H, Vardiman J. Response to splenectomy in 65 patients
with hairy cell leukemia: An evaluation of spleen weight and
bone marrow involvement. Blood. 1983;61:349-352.
32. Magee M, McKenzie S, Filippa D, et al. Hairy cell leukemia. Durability of response to splenectomy in 26 patients and treatment of
relapse with androgens in 6 patients. Cancer. 1985;56:2557-2562.
33. Grever M. How I treat hairy cell leukemia. Blood. 2010;115:
21-28.
34. George J. Management of immune thrombocytopenia - something old, something new. N Engl J Med. 2010;363:1959-1961.
35. Kojouri K, Vesely S, Terrell D, George J. Splenectomy for adult
patients with idiopathic thrombocytopenic purpura: A systematic literature review to assess long-term platelet count responses,
prediction of response, and surgical complications. Blood. 2004;
104:2623-2634.
36. Thomsen R, Schoonen W, Farkas D, et al. Risk for hospital contact with infection in patients with splenectomy: A populationbased cohort study. Ann Intern Med. 2009;151:546-555.
37. Provan D, Stasi R, Newland A, et al. International consensus
report on the investigation and management of primary immune
thrombocytopenia. Blood. 2010;115:168-186.
38. George J. Clinical practice. Thrombotic thrombocytopenic purpura. N Engl J Med. 2006;354:1927-1935.
39. Kremer Hovinga J, Meyer S. Current management of thrombotic thrombocytopenic purpura. Curr Opin Hematol. 2008;15:
445-450.
40. Dubois L, Gray D. Case series: Splenectomy: Does it still play a
role in the management of thrombotic thrombocytopenic purpura? Can J Surg. 2010;53:349-355.

5/22/2012 5:41:04 PM

Hematologic Indications for Splenectomy

41. Schwartz S. Role of splenectomy in hematologic disorders. World


J Surg. 1996;20:1156-1159.
42. Al-Salem A, Naserullah Z, Qaisaruddin S, et al. Splenectomy for
hematological diseases: The Qatif Central Hospital experience.
Ann Saudi Med. 1999;19:325-330.
43. Al-Salem A. Indications and complications of splenectomy for
children with sickle cell disease. J Pediatr Surg. 2006;41:19091915.
44. Machado N, Grant C, Alkindi S, et al. Splenectomy for haematological disorders: A single center study in 150 patients from Oman.
Int J Surg. 2009;7:476-481.
45. al Hawsawi Z, Hummaida T, Ismail G. Splenectomy in thalassaemia major: Experience at Madina Maternity and Childrens
Hospital, Saudi Arabia. Ann Trop Paediatr. 2001;21:155-158.
46. Hoe T, Lammi A, Webster B. Homozygous beta-thalassaemia: A
review of patients who had splenectomy at the Royal Alexandra
Hospital for Children, Sydney. Singapore Med J. 1994;35:59-61.
47. Phrommintikul A, Sukonthasarn A, Kanjanavanit R, Nawarawong W. Splenectomy: A strong risk factor for pulmonary hypertension in patients with thalassaemia. Heart. 2006;92:1467-1472.
48. Shah S, Vega R. Hereditary spherocytosis. Pediatr Rev. 2004;25:
168-172.
49. Bolton-Maggs P, Stevens R, Dodd N, et al. Guidelines for the diagnosis and management of hereditary spherocytosis. Br J Haematol. 2004;126:455-474.
50. Hill J, Walsh R, McHam S, et al. Laparoscopic splenectomy for
autoimmune hemolytic anemia in patients with chronic lymphocytic leukemia: A case series and review of the literature. Am
J Hematol. 2004;75:134-138.
51. Diehl L, Ketchum L. Autoimmune disease and chronic lymphocytic
leukemia: Autoimmune hemolytic anemia, pure red cell aplasia,
and autoimmune thrombocytopenia. Semin Oncol. 1998;25:80-97.

PMPH_CH59.indd 487

487

52. Bowdler A. The role of the spleen and splenectomy in autoimmune hemolytic disease. Semin Hematol. 1976;13:335-348.
53. Coon W. Splenectomy in the treatment of hemolytic anemia.
Arch Surg. 1985;120:625-628.
54. Stock H, Kadry Z, Smith J. Surgical management of portal hypertension in Feltys syndrome: A case report and literature review.
J Hepatol. 2009;50:831-835.
55. Klofkorn R, Steigerwald J, Mills D, Smyth C. Esophageal varices
in Feltys syndrome: A case report and review of the literature.
Arthritis Rheum. 1976;19:150-154.
56. Cohen M, Ginsburg W, Allen G. Nodular regenerative hyperplasia of the liver and bleeding esophageal varices in Feltys syndrome: A case report and literature review. J Rheumatol. 1982;
9:716-718.
57. DeCoux Jr R, Achord J. Portal hypertension in Feltys syndrome.
Am J Gastroenterol. 1980;73:315-318.
58. Casaccia M, Torelli P, Squarcia S, et al. Laparoscopic splenectomy for hematologic diseases: A preliminary analysis performed on the Italian Registry of Laparoscopic Surgery of the
Spleen (IRLSS). Surg Endosc. 2006;20:1214-1220.
59. Balagu C, Targarona EM, Cerdn G, et al. Long-term outcome
after laparoscopic splenectomy related to hematologic diagnosis.
Surg Endosc. 2004;18:1283-1287.
60. Sapucahy M, Faintuch J, Bresciani C, et al. Laparoscopic versus
open splenectomy in the management of hematologic diseases.
Rev Hosp Clin Fac Med Sao Paulo. 2003;58:243-249.
61. Friedman R, Hiatt J, Korman J, et al. Laparoscopic or open splenectomy for hematologic disease: Which approach is superior?
J Am Coll Surg. 1997;185:49-54.
62. Trias M, Targarona E, Espert J, et al. Impact of hematological
diagnosis on early and late outcome after laparoscopic splenectomy: An analysis of 111 cases. Surg Endosc. 2000;14:556-560.

5/22/2012 5:41:04 PM

CHAPTER 60

Tumors, Cysts, and Abscesses


of the Spleen
Robert Benjamin

INTRODUCTION

2. Level 3 evidenceFor metastatic disease to the spleen, retrospective studies show splenectomy is palliative.

Over the past 20 years, medical management has evolved to the


point where splenectomy is no longer the first line therapy or a
diagnostic tool for many disorders involving the spleen. In addition, the laparoscopic approach has supplanted open splenectomy
as the procedure of choice. But there are disorders in which a
splenectomy is still a relevant therapy. This chapter explores the
changing medical and surgical management of malignant and
infectious diseases involving the spleen.
There are many questions which remain controversial when
discussing the surgical management of splenic diseases: (1) Is
splenectomy as effective as medical therapy? (2) Is percutaneous
drainage the preferred therapy for splenic abscess? and (3) How
does splenic size affect surgical approach?

SPLENIC ABSCESS
2. Is percutaneous drainage the preferred therapy for splenic
abscess?
Splenic abscess is a rare condition that is manifested in immunodeficiency disease, intravenous drug use, trauma, use of steroids,
immunosuppressive therapy, diabetes mellitus, and an infective
endocarditis.5,6 Computer tomographic-guided drainage with catheter placement has proven safe and efficacious with 70% to 100%
resolution.7,8 Catheters will initially resolve up to 70% of the abscess
with the rest being drained for up to two weeks. In addition, failure
at drainage can mean multiple attempts will be performed because
of the low rates of morbidity and mortality associated with the procedure.8 In infective endocarditis, splenic abscess occur as a result
of septic emboli that produce a splenic infarct which over time
becomes an abscess. Patients with severe valve disease will need
expedited heart valve replacement when the infection is cleared.
Percutaneous drainage of a splenic abscess will be effective in 75%
of patients and fails to be definitive in patients who need a prosthetic
heart valve.9 The consequence of continuous infection with heart
valve replacement was described by Robinson et al., who reported
a retrospective study of 27 patients with endocarditis and splenic
abscess. Ten patients did not have a splenectomy and of those ten,
five received prosthetic heart valves. This groups mortality rate was
100%. The remaining 17 patients had a splenectomy and after valve
replacement sustained a survival rate of 82%.10 This study confirms
the findings by Johnson et al.11 who studied 37 patients with infective endocarditis and splenic abscess. They reported 100% mortality
in those patients not treated with splenectomy.11

MALIGNANCIES
1. Is splenectomy as effective as medical therapy?
Splenic tumors are divided into nonlymphoid and lymphoid tumors.
Malignant nonlymphoid tumors are rare and most commonly vascular hemangiosarcomas. Clinical features include splenomegaly,
left upper quadrant pain, sequestration, pleural effusions, and hypersplenism. These spleens can be massive and weigh over 3000 grams.
When diagnosed, splenectomy is indicated, but these patients present
with advanced disease and have a poor prognosis.1,2
Metastatic disease to the spleen is usually from the lung, breast,
and melanoma.3 Retroperitoneal and pancreatic tumors can directly
extend into the spleen.4 These patients can present with splenomegaly or splenic ruptures. In these patients, splenectomy is palliative
due to their advanced disease.1

Recommendations
Recommendations

These recommendations are based on retrospective Class III data:

These recommendations are based on retrospective Class III data:

1. Level 2 evidenceMany retrospective articles confirm that


splenectomy is indicated for primary splenic malignancy
despite poor outcomes for advanced disease.

1. Level 3 evidenceRetrospective studies show that retained


splenic infection in the face of prosthetic valve replacement has
488

PMPH_CH60.indd 488

5/22/2012 5:41:36 PM

Tumors, Cysts, and Abscesses of the Spleen

a 100% mortality rate. Splenectomy is recommended prior to


valve replacement.
2. Level 2 evidenceprospective and retrospective studies show
that computer tomographic-guided drainage of a splenic abscess
is safe and efficacious. Drains can be placed and used until
resolution of the abscess.

SPLENOMEGALY: DOES SIZE MATTER?


3. How does splenic size affect the surgical approach?
The laparoscopic approach has gained wide acceptance as the procedure of choice for splenectomy. Compared with open splenectomy, laparoscopic splenectomy has demonstrated decreased pain,
shorter length of hospital stay, and similar rates of morbidity and
mortality.12 However, some authors allude to increased morbidity and higher conversion rates in patients with large spleens.13,14
Massive splenomegaly refers to a spleen weighing >600 g or with a
craniocaudal length >17 cm. Super massive spleens weigh >1600 g
and have a length >22 cm.15 A significant portion of malignancies
involve massive and super massive splenomegaly and need a larger
orifice to remove the specimen intact in order to maintain oncologic principles.16 Heniford et al.17 performed a prospective study on
142 patients who underwent laparoscopic splenectomy. One group
included 82 patients with normal-sized spleens, whereas the second
group included 60 patients with splenomegaly and 58% of those

489

were due to a malignancy. Ten patients with a malignancy had super


massive splenomegaly and needed laparoscopy via a hand-assisted
technique. The only significant factor between the two groups was
operative time and blood loss. The groups had similar conversion to
laparotomy, length of stay, and complication rates.17 Another prospective study by Kercher et al. evaluated 177 patients who underwent laparoscopic splenectomy. Forty-nine patients (28%) had
massive splenomegaly, and 12 patients had super massive splenomegaly. These 12 patients underwent laparoscopic splenectomy with a
hand-assisted technique. The majority of massive splenomegaly was
due to malignancies. There were no conversions to laparotomy.15

Recommendations
These recommendations are based on retrospective Class II data:
1. Level 2 evidenceLaparoscopic splenectomy is the standard of
care and leads to less pain and morbidity for patients.
2. Level 2 evidenceSplenomegaly is amendable to laparoscopic
splenectomy. Many authors use a lateral approach to aid in the
dissection.
3. Level 2 evidenceSuper massive spleens have similar morbidity
rates and conversion rates to open splenectomy as normal sized
spleens. Super massive spleens (>22 cm) are removed with
hand-assisted techniques.

Clinical Question Summary


Question

Answer

Level of Evidence References

1 Is splenectomy as
effective as
medical therapy?

Splenectomy is indicated for primary splenic malignancy despite poor


outcomes for advanced disease (LoE 2). For metastatic disease
splenectomy is palliative (LoE 3).

2 and 3

1-4

2 and 3

5-11

12-17

Retained splenic infection after prosthetic valve replacement has 100%


2 Is percutaneous
mortality. Splenectomy is recommended before valve replacement
drainage the
(LoE 3). Computer tomographic-guided drainage of a splenic abscess is
preferred therapy
safe and efficacious (LoE 2).
for splenic abscess?
3 How does splenic
size affect surgical
approach?

Laparoscopic splenectomy is the standard of care. Splenomegaly is amenable


to laparoscopic splenectomy. Super massive spleens (>22 cm) have similar
morbidity and conversion to open splenectomy rates as normal spleens.

REFERENCES
1. Katz S, Pachter HL. Indications for splenectomy. Am Surg. 2006;
72:565-579.
2. Morgenstern L, Rosenberg J, Geller SA. Tumors of the spleen.
World J Surg. 1985;9:468-476.
3. Berge T. Splenic metastases: Frequency and patterns. Acta Pathol
Microbiol Scand A. 1974;82:499-506.
4. Marymount JH, Gross S. Patterns of metastatic carcinoma in the
spleen. Am J Clin Pathol. 1963;40:58-64.
5. Kim HS, Cho MS, Hwan S, et al. Splenic abscess associated with
endocarditis in a patient on hemodialysis: A case report. J Korean
Med Sci. 2005;20(2):313-315.
6. Fotiadis C, Lavranos G, Patopis P, et al. Abscesses of the spleen:
Report of three cases. World J Gastroenterol. 2008;14(19):3088-3091.
7. Nelken N, Ignatlus J, Skinner M, et al. Changing clinical spectrum of splenic abscess: A multicenter study and review of the
literature. Am J Surg. 1987;154:27-34.
8. Thanos L, Dailiana T, Papaioannou G, et al. Percutaneous CTguided drainage of splenic abscess. Am J Roen. 2002;179:629-632.
9. Simsir SA, Cheeseman SH, Lancey RA, et al. Staged laparoscopic splenectomy and valve replacement in splenic

PMPH_CH60.indd 489

10.
11.
12.

13.
14.
15.
16.

17.

abscess and infective endocarditis. Ann Thorac Surg. 2003;75:


1635-1637.
Robinson SL, Saxe JM, Lucas CE, et al. Splenic abscess associated
with endocarditis. Surgery. 1992;112:781-787.
Johnson JD, Raff MJ, Barnell PA, et al. Splenic abscess complicating infectious endocarditis. Arch Intern Med. 1983;143:906-912.
Kucuk C, Sozuer E, Ok E, et al. Laparoscopic versus open splenectomy in the management of benign and malign hematologic
diseases: A ten-year single-center experience. J Laparo Adv Surg
Tech. 2005;15:135-139.
Park A, Gagner M, Pomp A. The lateral approach to laparoscopic
splenectomy. Am J Surg. 1997;173:126-130.
Rege RV, Merriam LT, Joehl RJ. Laparoscopic splenectomy. Surg
Clin North Am. 1996;3:459-468.
Kercher KW, Matthews BD, Walsh RM, et al. Laparoscopic splenectomy for massive splenomegaly. Am J Surg. 2002;183:192-196.
Rosen M, Brody F, Ponsky MJ. Outcome of laparoscopic splenectomy based on hematologic indication. Surg Endosc. 2002;16:
272-279.
Heniford BT, Park A, Walsh RM, et al. Laparoscopic splenectomy in patients with normal-sized spleens versus splenomegaly:
Does size matter? Am Surg. 2001;67(9)854-858.

5/22/2012 5:41:37 PM

CHAPTER 61

Splenic Salvage
Dror Soffer and Daniel Abraham

reliability in grading the injury.4 An appropriate screening test is


crucial to help the physician decide which patients will succeed
on NOM. However, until one is decided upon, most physicians
have come to rely on predictors of NOM success as seen in the
literature.
In 1995, Schurr et al.5 published a retrospective study reviewing 309 BSI patients to better characterize failures of NOM. Results
showed that contrast blush was noted in 8 of 12 (67%) patients
who failed NOM and in 5 of 77 (6%) of those managed successfully (p < .0001). These results suggest that contrast blush is an
important indicator of NOM failure. In 2000, Peitzman et al.6
from 27 trauma centers published a large retrospective study
evaluating 1488 patients with BSI, 61.5% of which were admitted for NOM. Among patients that were treated conservatively,
the failure rate was 10.8%, with the failure rate increasing with
increasing graded splenic injuries. The failure rates were 5% for
grade I, 10% for grade II, 20% for grade III, 33% for grade IV, and
75% for grade V. Most (61%) of the failures occurred in the first 24 h
and were correlated with the quantity of hemoperitoneum present. Successful NOM was correlated with a higher blood pressure
and hematocrit. The success of lesser injuries was based on ISS,
GCS, grade of injury, and quantity of hemoperitoneum.6 In 2009,
Cohn et al.7 published a paper proposing a new grading system
for splenic injuries in an attempt to create a better predictor of
the need for intervention. Results showed that the following were
highly predictive of the need for intervention: devascularization
of half or more of the splenic parenchyma, contrast blush that
could be attributed to active extravasation or pseudoaneurysm
(PSA), and a large hemoperitoneum.7
Recommendations: There are no set criteria to predict the success of NOM. While the AAST grading scale for splenic injury is
commonly used as a predictor, there is sufficient Level 2 and 3 evidences to demonstrate that it can be inconsistent. Although there
are no official predictors of NOM success, contrast extravasation,
amount of hemoperitoneum, and grade of injury might assist in
making a decision about the need for intervention. (Level 3 evidence; Grade C recommendation)

INTRODUCTION
The spleen is the intraabdominal organ most frequently injured
in blunt trauma. Because the spleen is a highly vascular organ, an
injury could have devastating consequences. This fear led to the
universal belief that most splenic injuries should be managed with
splenectomy, a practice that began in the early 19th century and
continued up until four decades ago. The mortality risk associated
with not operating was said to be 90%, compared with 30% to 40%
following an uncomplicated removal. Over the last few decades,
however, the influx of data concerning the disadvantages of splenectomy and its relation to overwhelming sepsis has led to a more
conservative approach. Today, nonoperative management (NOM)
is considered safe in selected patients. More than 70% of all stable
patients are being treated by means of a nonoperative approach.1
The movement toward a nonoperative scheme has exposed a whole
new set of questions. This chapter highlights the questions that
have arisen from this new trend and attempt to present the current state of NOM of the patient that presents with blunt splenic
injury (BSI) using up-to-date clinical data.
1. Is there a way to predict the success of NOM?
Physicians are progressively opting for splenic preservation procedures in a patient that presents with a BSI. In contrast to the
alternative, a more conservative approach raises several issues
which make appropriate patient care more challenging. The
Organ Injury Scaling Committee of the AAST released a set of
criteria in 1989 for the grading of a splenic injury that would
help facilitate clinical investigation and treatment.2 The grading
ranges from I to V and can be based on computed tomography
(CT) scan fi ndings. While the intention of the AAST grading system is to standardize the description of the injury, many physicians will use these grading systems to direct their clinical plan.3
The actual ability of these grading systems in predicting the need
for intervention, however, is a controversial topic. Radiologists
evaluating abdominal CT scans demonstrate poor accuracy and
490

PMPH_CH61.indd 490

5/22/2012 5:42:30 PM

Splenic Salvage

2. When is splenic angiographic embolization recommended?


While NOM has become the current gold standard of practice for
most splenic injuries, there is still no set protocol for a physician
treating a patient with BSI. The procedure of splenic artery embolization (SAE) has emerged as an exciting adjunct to the practice
of NOM. It is therefore vital that there exists a set of screening criteria for the use of SAE for patients with BSI. A PubMed search of
splenic angiographic embolization over the past 30 years yielded
several studies, mostly with Level 2 and 3 evidences.
In the 1980s, Sclafani8 introduced a novel treatment of splenic
angiographic embolization in the management of splenic trauma.
Subsequently, his group described the use of admission CT to characterize splenic injury, and its value as a screening tool for SAE.9 In
1995, Sclafani et al.10 prospectively evaluated 172 patients with BSI.
SAE was performed in the hemodynamically stable patients that
had any CT-graded splenic injury or stable patients that demonstrated angiographic extravasation. Fift y-six of the 60 treated by
embolization had a successful outcome.10
Hagiwara et al.,11 who followed the study design set out by
Sclafani et al., led their own group of studies. The first one, in 1996,
was a prospective study that included 228 patients. Their protocol
called for a CT on all BSI patients and subsequent SAE performed on
all patients that showed contrast extravasation and/or an (arteriovenous) AV fistula. The study reported a 93% salvage rate with
these criteria.11 A second prospective study by Hagiwara et al.,12
published in 2005, involved 104 patients. SAE was performed
on patients with CT-graded injuries of III or higher and patients
with contrast extravasation. The results demonstrated that even
patients that present with hemodynamic instability, a parameter
typically used as an indication for laparatomy, could be amenable
to SAE if they respond to fluid resuscitation.12
In 2001, Haan et al.13 conducted a retrospective study that
demonstrated 92% effectiveness in the 40 patients who underwent SAE. They concluded that CT-graded injuries were a useful
predictor of SAE success.13 In a larger retrospective study conducted in 2004 involving 140 patients, Haan et al.14 demonstrated
a splenic salvage rate of 87%, which decreased with increasing
injury grade. Over 80% of the splenic injury grades IV and V,
however, were successfully managed nonoperatively.14 In 2005,
Haan et al.15 retrospectively evaluated 132 patients who underwent
SAE. The authors established a 90% splenic salvage rate using SAE
in patients with CT-graded injuries of level IV and V, patients
with hemoperitoneum and contrast extravasation, and in patients
with a splenic artery PSA.15
In 2004, a prospective observational study by Liu et al.16 demonstrated an 89% success rate using SAE in patients with hemoperitoneum, CT extravasation, and CT- graded IV and V injuries.
In 2006, Bessoud et al.17 published a retrospective study involving 67 NOM patients, 37 of whom underwent proximal splenic
embolization (PSE). Splenic injuries were notably more severe in
the NOM PSE group than in the NOM group without embolization with regards to the mean splenic injury CT grade (3.7 vs. 2,
respectively; p < .0001), active contrast extravasation or blush (38%
[14/37] vs. 3% [1/30], respectively; p = .0005), and hemoperitoneum
grade (1.6 vs. 0.8, respectively; p = .0006). Secondary splenectomy
rate was lower in the nonoperative management PSE group (2.7%
[1/37] vs. 10% [3/30]).17
Most recently, Sabe et al.18 retrospectively looked at all patients
presenting with BSI over 16 years. They found a 97% success rate

PMPH_CH61.indd 491

491

for NOM and a 90% success rate for SAE management. Their criteria for SAE included contrast extravasation, PSA, and level III
and IV CT-graded injuries.18
Recommendations: There is sufficient Level 2 and 3 data to
suggest that SAE might be considered as an adjunct to conservative treatment in stable patients, where no other indication for
laparotomy exists. It is recommended in patients with contrast
material extravasation (Blush on CT); in patients with a PSA;
and it might also be recommended in patients with high CTgraded splenic injuries (Levels IIIV). (Level 3 evidence; Grade C
recommendation)
3. Should postsplenic angiographic embolization patients need
to be vaccinated against Pneumococus and Hemophilus influenza infections?
The spleen has historically been linked with its connection to
the immune system, and more recently, upon total splenectomy,
with overwhelming sepsis. Unfortunately, the immune function
in patients that have undergone SAE has not been well studied. A
PubMed search of immune function and splenic embolization
yielded only four retrospective studies.
In 2007, Bessoud et al.19 published a retrospective study, evaluating 24 patients that had undergone SAE with an objective of
ascertaining the impact it had on immune function. The study
looked at the presence of HowellJolly bodies, and serum antibody titer determinations (Pneumococcus and H. influenzae B).
All patients who were assessed for exposure-driven immunity
against H. influenza B had sufficient immunity. Seventeen of the
18 patients (94%) assessed for exposure-driven immunity against
Pneumococcus had sufficient immunity.19
In 2009, Tominaga et al. 20 conducted a retrospective study
on 17 patients who underwent SAE and compared results with
controls (blunt abdominal trauma patients with negative abdominal CT scans) and splenectomy patients. The objective was to
define any immunologic differences by comparing levels of IgM,
IgG, C3 complement, complement factor B, helper T cells (CD3,
CD4), suppressor T cells (CD8), complete blood counts, and HIV
status. Their results show that the immune capability tested 3
months following SAE was preserved in embolized patients.20 In
2009, Nakae et al.21 conducted a large retrospective study involving seven trauma centers, which focused on the immunocompetence of patients who underwent splenic salvage procedures
compared with patients who underwent splenectomy. Eighty-one
of the patients underwent SAE. The study group did not show a
discernible advantage over the group who underwent splenectomy according to immunologic indices, which included IgM and
14 serotypes of anti-S. pneumonia antibodies, suggesting that prophylactic measures and close follow-up are necessary after both
treatments.21
In 2010, Shih et al.22 conducted a study that compared a
group of five BSI patients who had undergone SAE with 11
patients that were closely monitored. The study followed the
endotoxin responses of peripheral blood mononuclear cells in
the two groups. The results demonstrate that SAE may induce
alterations of immune response and culminate in infectious
vulnerability.22
Recommendations: There is not yet enough data to support
any recommendation against vaccination in patients who underwent SAE. (Level 4 evidence; Grade C recommendation)

5/22/2012 5:42:31 PM

492

Surgery: Evidence-Based Practice

4. What is the role of repeated splenic imaging in conservatively treated patients and what should the preferred imaging
technique be in such patients?
While it has become a gold standard to use a CT scan upon admission of BSI, the use of follow-up repeated imaging is a controversial
topic. Delayed splenic hemorrhage is theorized to be associated
with latent PSA formation, which may not be picked up on admission CT, and can subsequently lead to late rupture.
In 2007, Weinberg et al.23 conducted a retrospective study
evaluating 341 patients on NOM. The purpose of the study was
to re-evaluate the practice of serial CT imaging in the clinical setting. They discovered that close to one-half of the PSAss identified
in this study were captured on a follow-up CT scan performed
24 to 48 h after the initial scan.23 In 2008, a study by Savage et al.24
included 637 patients with BSI retrospectively reviewed with attention to their follow-up CT scans. The purpose of the study was
to document the progression of splenic recovery using CT scans.
The results indicate that most tissue healing occurred within
2 months but about 20% had not healed after 3 months. Using
these results, they concluded that a close observation of patients
with BSI, which could include serial CT scanning, should continue until healing can be confirmed, although the clinical significance of such a survey is not clear.24
Despite the various studies that have demonstrated the value
of serial CT scanning, there are equally as many conflicting articles. In 2005, Fata et al.25 published a retrospective study on surgeons practices that confirm the schism. They found that 85.5%
of surgeons would not usually perform predischarge abdominal
CT scans in the absence of clinical deterioration, extravasation
on initial CT, or high-grade injury.25 In 1996, Allins et al.26 published a retrospective study on 99 NOM patients with an objective of determining the utility of a second CT scan. None of the
follow-up scans showed any major progression of injury, and scan
findings had no influence over decisions for additional operations in any patients.26 Thaemert et al.s27 retrospective study in
1997 focused on 62 follow-up abdominal CT scans obtained in
49 patients. Information that affected management was evident
on only one follow-up CT scan performed in the absence of clinical indications.27 In a more recent retrospective study by Sharma
et al.,28 NOM was undertaken in 221 patients with spleen/liver
trauma, with 65 patients having an additional CT scan after their
admission scan. NOM failed in 11 patients. Only 4.9% of repeat
scans, which accounted for four patients, were done prior to surgery. Two of these patients had hemodynamic instability and, in
the other two, there were clinical signs of peritonitis. The remaining seven patients who failed NOM had delayed surgery due to
hemodynamic instability.28
There are thus proponents on both sides of the serial CT scan
issue with regards to its use for NOM patients. Major arguments
from those that oppose repeat CT scans include unnecessary
radiation, contrast material complications, and expensive costs.
This has driven surgeons to adopt different means of imaging for
identification of PSA, potential delayed splenic rupture, and documentation of healing.
In 1996, Goletti et al.29 conducted a trial with 10 patients to
determine the efficacy of Doppler ultrasound (DUS) on patient
follow-up and its ability to detect splenic PSA. Echo Doppler and
echo color Doppler evaluations were scheduled at 24 h and at 3
and 6 days after trauma. In eight cases, no complications were

PMPH_CH61.indd 492

observed; in two cases, PSAs were diagnosed.29 In 2005, Doody


et al.30 conducted a review of the practices in their hospital and
found that ultrasound is an important modality in assessing
success of embolization and in evaluating for recurrence during
follow-up. Within the review, they also documented what a PSA
might look like when imaged by DUS.30 In 2007, in a retrospective study by Bessoud et al.19 of 24 embolized patients, DUS was
also used to follow-up after the procedure and demonstrates that
splenic size and measurements were in the normal range. Within
this paper, they documents the size and measurements seen on
a DUS after an embolization with no complications.19 More
recently, Soffer et al. conducted a prospective study on 38 patients
using DUS to follow-up. Grading of splenic injury demonstrated
19 (50%) patients with grade I, 16 (42%) with grade II, and three
patients (8%) with grade III injuries. Two patients (5%) had
PSAs. Splenic injury was detected by ultrasound in 35/38 (92%);
hence, the DUS examination had a sensitivity of 94.6% with a
positive predictive value of 100%.31 While DUS has shown some
promise as an adjunct to NOM, it is extremely user-dependent
and therefore has limitations.
Recommendations: There is not enough data to support the
need for repeated imaging in patients with BSI after conservative treatment. Repeated imaging might be considered in patients
with high-grade injuries. DUS might be a safer, cheaper, and easily
repeated modality to be used in such patients. (Level 3 evidence;
Grade C recommendation)
5. Which technique is preferred, proximal SAE or distal selective embolization?
Proximal SAE involves inserting embolic coils to the splenic
artery at a point proximal to branching. Selective embolization,
also known as distal embolization, involves inserting embolic
coils closer to locations of vascular injury, thereby limiting the
fraction of the spleen that will be ischemic. Recent studies have
started to tease apart the advantages of using proximal embolization, and some have even reported fewer complications than the
distal approach.
In 2004, in his letter to the editor, Bessoud reported an invasive intrasplenic arterial pressure monitoring during main SAE in
two patients in order to demonstrate the effects of this technique
on splenic blood pressure. Patients experienced a pressure reduction of 47% and 58% within the splenic artery. He concluded that
main SAE significantly reduces the intrasplenic blood pressure,
a condition that may help the clot to organize in splenic injuries
and permit the conservative management of the patient.32 Sclafani
et al.s10 1995 prospective study involving 60 embolized patients set
out to determine if coil embolization of the proximal splenic artery
provides effective hemostasis. The results showed that embolization decreased the splenic blood flow and arterial blood pressure
by occluding the main arterial channel to the spleen. The rich
network of collateral circulation provided sufficient blood flow
to the spleen, thus maintaining the viability of the spleen, which
explained the absence of splenic infarction after SAE. Therefore,
Sclafani et al.10 concluded that coil embolization of the proximal
splenic artery is an effective method of hemostasis in stabilized
patients with splenic injury. Bessoud et al.17 conducted another
study that mirrored this sentiment in 2006. This retrospective
study included 67 NOM patients, 37 of whom underwent PSE. No
procedure-related complications were encountered during early

5/22/2012 5:42:31 PM

Splenic Salvage

and delayed clinical follow-up.17 In 2009, Zmora et al.33 published a


prospective observational study that included 11 patients. Their aim
was to assess the efficacy of PSE in the cessation of bleeding without
the formation of PSAs. The results showed that after embolization,
splenic parenchymal blood flow was found on Doppler sonography
in 82% of the patients and no PSAs were demonstrated. During follow-up, no patient underwent surgery or repeated embolization.33
There are also some studies that have tried both embolization approaches. In 2006, Smith et al. 34 conducted a trial of 38
embolized patients and found that selective distal embolization
failed in 3 of 9 (33%) patients whereas proximal SAE failed in
about 6 of 27 (22%) patients. Another study conducted in 2001
by Killeen et al.35 using 80 embolized patients found that splenic
infarcts occurred in 63% of patients after main SAE and in 100%
of patients after selective distal embolization. Infarcts after distal
embolization were larger, whereas infarcts after proximal embolization were smaller.35
In 2004, Haan et al.14 published a retrospective study where
they used combined therapy embolization. In patients that were
candidates for combined embolization therapy, the largest vascular injuries were selectively embolized and the smaller injuries were
treated with a proximal coil. Although only nine patients in this
trial were treated with combined therapy, it was found that failure
rates were higher when compared with main coil or distal embolization, though it never reached statistical significance (p = .057).14
Most recently in 2011, a systematic review and meta-analysis
was released by Schnuriger et al.36 to assess the impact of the different embolization techniques. Pooled outcomes from 15 evaluated studies totaling 479 embolized patients were compared based
on proximal and distal embolizations. The overall failure rate of
angioembolization was 10.2%. Although rebleeding was the most
common reason for failure, it did not differ statistically between
the two techniques. Minor complications, which included rebleeding, infarction, and infection not requiring splenectomy, occurred
statistically and clinically more often after distal than after proximal embolization. Major complications, which included rebleeding, infarction, and infection requiring splenectomy, had an
equivalent rate in both techniques.36
Recommendations: There is insufficient Level 1 and 2 data supporting the use of a particular embolization technique. Therefore,
proximal artery embolization cannot be stated to be preferred to
distal embolization. There are, however, some clues which show
that PSE might be an easier, more feasible approach associated
with less splenic tissue damage, despite adequate data to support
its success over distal embolization. (Level 3 evidence; Grade C
recommendation)
6. What is the natural history of traumatic splenic PSAs?
PSAs of splenic artery branches after blunt abdominal trauma
are commonly considered as the most probable cause of delayed
splenic rupture, and therefore its detection is crucial.31 This detection is the basis of the argument for serial imaging in a patient that
presents with splenic injury. The association with PSA formation
and delayed splenic rupture has never been properly studied, and
as a result, little is known about the natural progression of a splenic
PSA. Thus, the recommended approach to any detected PSA is by
angiographic embolization or by surgery. Spontaneous obliteration
and resolution of a post-traumatic splenic PSA was described in a
case report by Dror et al.37

PMPH_CH61.indd 493

493

In 2010, Soffer et al.31 conducted a prospective study regarding the use of DUS as a tool for follow-up and reported a spontaneous obliteration and resolution of two out of two detected splenic
PSAs. Further, it was showed that PSA formation is an unexpected
phenomenonone that can form regardless of grade of injury.31
Recommendation: There is insufficient data to support any
assumption regarding the natural history of traumatic splenic
PSAs. While some consider them to be infrequent and benign, they
might present with a significant risk if ruptured. (Level 4 evidence;
Grade D recommendation)
7. How long should one follow a conservatively treated BSI
patient?
The importance of follow-up observation for the nonsurgically
treated patient has already been expressed. Successful inpatient
management of splenic injuries leads to an outpatient follow-up
dilemma. The debate over serial imaging is only one facet of this
matter. Another equally important and equally debated topic is
the length of follow-up time, which begs the question: how long
is long enough?
In 2008, Savage et al.24 released a study which was based on
637 patients with BSI. The purpose of their study was to determine
the time course and natural progression of BSI. Ninety-seven
patients discharged with BSI had outpatient CT scans. Nine of the
discharged patients had worsening of BSI as outpatients and two
required splenectomies. Thirty-three outpatients were followed
up to complete healing. Lesser injuries had a shorter mean healing
time compared with severe injuries (12.5 vs. 37.2 days, p < .001).
Most healing occurred within 2 months but approximately 20% of
each group (mild vs. severe injury) had not healed after 3 months.
The results thus elucidated that the majority of those who will heal
from their injury will do so within 2 to 2.5 months, regardless of the
severity at presentation. These findings led to the conclusion that
close outpatient follow-up for at least 8 to 10 weeks after presentation should be considered.24 In 2008, McCray et al.38 released a
retrospective study involving 449 NOM patients with the purpose
of finding out what is an appropriate length of time is safe for observation of the injured spleen. The protocol for the study called for
serial hemoglobin measurements until stable measurements warranted discharge. Using this protocol, the study had a 96% success
rate. They found that hospital stay was reduced to 3 days 0.8 days,
and believed that late failures were rare and did not justify inpatient
observation beyond the point when their hemoglobin stabilizes.38
Recommendation: Not enough Level 1 and 2 evidences exist
regarding the length of follow-up time in a splenic injury patient.
Because many hospitals follow their own protocol, there are no set
criteria to ascertain how long a conservatively treated patient should
be followed up. (Level 5 evidence; Grade D recommendation)
8. When can full physical activity be resumed?
Regardless of the management that is used to treat blunt splenic
trauma, full splenic recovery is an end point that all physicians
have in mind. This point, however, comes with a necessary amount
of activity restriction for the patient. Limited activity is an important concern in a young and otherwise healthy population, especially for individuals whose quality of life and return to work may
be compromised by such limitations. While most physicians follow their hospital protocol when it comes to activity restriction,

5/22/2012 5:42:31 PM

494

Surgery: Evidence-Based Practice

there is no national standard for these considerations, and little


research in the literature regarding it.
In 1987, Dulchavsky et al.39 published a study evaluating
canine and porcine wound healing and breaking strength following splenorrhaphy postsplenic injury. The results showed that
there was no difference in the wound breaking strength after
observant therapy in pigs after 3 weeks, and in both species after
6 weeks. These results suggest that prolonged rest following splenic
injury appears unjustified.39 In 2005, Fata et al.25 released a study
that used membership of the EAST, drawing on answers compiled
from a survey constructed to collect data on surgeons current
practices related to NOM of isolated blunt trauma to the spleen. It
was found that 62.9% of the sample chose to allow patients with
grade I and II injuries to return to light activity (defined as light
house work, office work, or low impact aerobic activity) within
2 weeks of their initial trauma. For grade III injury, 29.6% of
respondents chose to allow light activity at 2 weeks. For grades IV
and V, 22.4% of the sample chose to lift light activity restriction as
early as 2 weeks from the start of their injury. Two-thirds of the
sample chose to allow patients with grades III or higher to return

to light activities within 4 weeks to 2 months of their injury. For


full activity including contact sports, no detectable patterns
emerged. For grade I and II injuries, 37.6% reported less than 6
weeks restriction, 39.9% reported less than 3 months, and 19.7%
between 4 and 6 months. For grades IV and V, 45.8% reported 2 to
3 months full activity restriction, 31% reported 4 to 6 months, and
5% for longer than 6 months.25
In 2008, Savage et al.24 published a study with 637 splenic
injury patients, intent on finding out more information on the progression and resolution of splenic injury. The results showed that
by 2 to 2.5 months, over 80% of patients with blunt spleen injuries
had improved or healed their injury. This represented a starting
point for their recommendations and they therefore optioned to
limit physical activity for at least 8 to 10 weeks.24
Recommendation: There is not enough evidence concerning a
return to full physical activity after blunt splenic trauma. While
there is a similarity across protocols offered within different hospitals, there is not enough data to support these recommendations.
Therefore, no recommendations based solely on the literature can
be made. (Level 5 evidence; Grade D recommendation)

Clinical Question Summary


Question

Answer

1 Is there a way to predict the


success of NOM?

Although there are no set criteria to predict the


success of NOM, contrast extravasation, amount
of hemoperitoneum, and grade of injury might
assist in making a decision about the need for
intervention.

2-7

2 When is splenic angiographic


embolization recommended?

It is recommended in patients that are


hemodynamically stable with no other indications
for laparotomy; in patients with CT extravasation
or blush; in patients with a PSA; and might be
recommended in patients with high CT-graded
splenic injuries (levels IIIV).

8-18

3 Should postsplenic angiographic


embolization patients need
to be vaccinated against
Pneumococus and Hemophilus
influenza infections?

There are not yet enough data to support any


recommendation against vaccination in patients
who underwent SAE.

19-22

4 What is the role of


repeated splenic imaging in
conservatively treated patients
and what should the preferred
imaging technique be in such
patients?

There is no set role or set modality for repeat imaging


in conservatively treated BSI patients. Common
practices include using CT to grade injuries upon
admission. Some physicians use CT to follow up
or to detect PSA formation. Doppler US might be
used for follow-up in contrast to CT.

19, 23-31

5 Which technique is preferred,


proximal SAE or selective
embolization?

There are not enough data to support the use or


benefit of either approach.

10, 14, 17,


32-36

6 What is the natural history of


traumatic splenic PSAs?

Little is known about the natural history of traumatic


splenic PSA because there are not enough data
regarding its progression.

31, 37

7 How long should one follow


a conservatively treated BSI
patient?

There are not enough data to suggest how long


follow-up should be in a patient with BSI, if at all.

24, 38

8 When can full physical activity


be resumed?

There are not enough data to suggest when a patient


with BSI can return to full physical activity.

24, 25, 39

PMPH_CH61.indd 494

Grade of
Recommendation

References

5/22/2012 5:42:31 PM

Splenic Salvage

REFERENCES
1. Richardson JD. Changes in the management of injuries to the
liver and spleen. J Am Coll Surg. 2005;200(5):648-669.
2. Moore EE, Shackford SR, Pachter HL, et al. Organ injury scaling:
Spleen, liver, and kidney. J Trauma. 1989;29(12):1664-1666.
3. Moore EE, Jurkovitch GJ, Cogbill TH, et al. Organ injury scaling:
Spleen and liver (1994 revision). J Trauma. 1995;38(3):323-324.
4. Barquist ES, Pizano LR, Feuer W, et al. Inter- and intrarater reliability in computed axial tomographic grading of splenic injury:
Why so many grading scales? J Trauma. 2004;56(2):334-338.
5. Schurr MJ, Fabian TC, Gavant M, et al. Management of blunt
splenic trauma: Computed tomographic contrast blush predicts
failure of nonoperative management. J Trauma. 1995;39(3):507-512;
discussion 512-513.
6. Peitzman AB, Heil B, Rivera L, et al. Blunt splenic injury in adults:
Multi-institutional Study of the Eastern Association for the Surgery of Trauma. J Trauma. 2000;49(2):177-187; discussion 187-189.
7. Cohn SM, Arango JI, Myers JG, et al. Computed tomography
grading systems poorly predict the need for intervention after
spleen and liver injuries. Am Surg. 2009;75(2):133-139.
8. Sclafani SJ. The role of angiographic hemostasis in salvage of the
injured spleen. Radiology. 1981;141(3):645-650.
9. Sclafani SJ, Weisberg A, Scalea TM, Phillips TF, Duncan AO.
Blunt splenic injuries: Nonsurgical treatment with CT, arteriography, and transcatheter arterial embolization of the splenic artery.
Radiology. 1991;181(1):189-196.
10. Sclafani SJ, Shaftan GW, Scalea TM, et al. Nonoperative salvage
of computed tomography-diagnosed splenic injuries: Utilization of angiography for triage and embolization for hemostasis. J
Trauma. 1995;39(5):818-825; discussion 826-827.
11. Hagiwara A, Yukioka T, Ohta S, Nitatori T, Matsuda H, Shimazaki
S. Nonsurgical management of patients with blunt splenic injury:
Efficacy of transcatheter arterial embolization. AJR Am J Roentgenol. 1996;167(1):159-166.
12. Hagiwara A, Fukushima H, Murata A, Matsuda H, Shimazaki S.
Blunt splenic injury: Usefulness of transcatheter arterial embolization in patients with a transient response to fluid resuscitation. Radiology. 2005;235(1):57-64.
13. Haan JM, Scott J, Boyd-Kranis RL, Ho S, Kramer M, Scalea TM.
Admission angiography for blunt splenic injury: Advantages and
pitfalls. J Trauma. 2001;51(6):1161-1165.
14. Haan JM, Biffl W, Knudson MM, et al. Splenic embolization
revisited: A multicenter review. J Trauma. 2004;56(3):542-547.
15. Haan JM, Bochicchio GV, Kramer N, Scalea TM. Nonoperative
management of blunt splenic injury: A 5-year experience. J Trauma.
2005;58(3):492-498.
16. Liu PP, Lee WC, Cheng YF, et al. Use of splenic artery embolization
as an adjunct to nonsurgical management of blunt splenic injury.
J Trauma. 2004;56(4):768-772; discussion 773.
17. Bessoud B, Denys A, Calmes JM, et al. Nonoperative management
of traumatic splenic injuries: Is there a role for proximal splenic
artery embolization? AJR Am J Roentgenol. 2006;186(3):779-785.
18. Sabe AA, Claridge JA, Rosenblum DI, Lie K, Malangoni MA. The
effects of splenic artery embolization on nonoperative management of blunt splenic injury: A 16-year experience. J Trauma. 2009;
67(3):565-572; discussion 571-572.
19. Bessoud B, Duchosal MA, Siegrist CA, et al. Proximal splenic artery
embolization for blunt splenic injury: Clinical, immunologic, and
ultrasound-Doppler follow-up. J Trauma. 2007;62(6):1481-1486.
20. Tominaga GT, Simon FJ, Jr, Dandan IS, et al. Immunologic function after splenic embolization, is there a difference? J Trauma.
2009;67(2):289-295.

PMPH_CH61.indd 495

495

21. Nakae HT, Shimazu T, Miyauchi H, et al. Does splenic preservation


treatment (embolization, splenorrhaphy, and partial splenectomy)
improve immunologic function and long-term prognosis after
splenic injury? J Trauma. 2009;67(3):557-563; discussion 563-564.
22. Shih HC, Wang CY, Wen YS, et al. Spleen artery embolization
aggravates endotoxin hyporesponse of peripheral blood mononuclear cells in patients with spleen injury. J Trauma. 2010;8(3):
532-537.
23. Weinberg JA, Magnotti LJ, Croce MA, Edwards NM, Fabian
TC. The utility of serial computed tomography imaging of blunt
splenic injury: Still worth a second look? J Trauma. 2007;62(5):
1143-1147; discussion 1147-1148.
24. Savage SA, Zarzaur BL, Magnotti LJ, et al. The evolution of blunt
splenic injury: Resolution and progression. J Trauma. 2008;64(4):
1085-1091; discussion 1091-1092.
25. Fata P, Robinson L, Fakhry SM. A survey of EAST member practices in blunt splenic injury: A description of current trends and
opportunities for improvement. J Trauma. 2005;59(4):836-841;
discussion 841-842.
26. Allins A, Ho T, Nguyen TH, Cohen M, Waxman K, Hiatt JR. Limited
value of routine followup CT scans in nonoperative management
of blunt liver and splenic injuries. Am Surg. 1996;62(11):883-886.
27. Thaemert BC, Cogbill TH, Lambert PJ. Nonoperative management of splenic injury: Are follow-up computed tomographic
scans of any value? J Trauma. 1997;43(5):748-751.
28. Sharma OP, Oswanski MF, Singer D. Role of repeat computerized tomography in nonoperative management of solid organ
trauma. Am Surg. 2005;71(3):244-249.
29. Goletti O, Ghiselli G, Lippolis PV, et al. Intrasplenic posttraumatic pseudoaneurysm: Echo color doppler diagnosis. J Trauma.
1996;41(3):542-545.
30. Doody O, Lyburn D, Geoghegan T, Govender P, Munk PL,
Torreggiani WC. Blunt trauma to the spleen: Ultrasonographic
findings. Clin Radiol. 2005;60(9):968-976.
31. Soffer D, Wiesel O, Schulman CI, Ben Haim M, Klausner JM,
Kessler A. Doppler ultrasound for the assessment of conservatively treated blunt splenic injuries: A prospective study. Eur J
Trauma Emerg Surg. 2010;37(2):197-202.
32. Bessoud B, Denys A. Main splenic artery embolization using coils
in blunt splenic injuries: Effects on the intrasplenic blood pressure. Eur Radiol. 2004;14(9):1718-1719.
33. Zmora O, Kori Y, Samuels D, et al. Proximal Splenic Artery
Embolization in Blunt Splenic Trauma. Eur J Trauma Emerg Surg.
2009;35(2):108-114.
34. Smith HE, Biffl WL, Majercik SD, Jednacz J, Lambiase R, Cioffi
WG. Splenic artery embolization: Have we gone too far? J Trauma.
2006;61(3):541-544; discussion 545-546.
35. Killeen KL, Shanmuganathan K, Boyd-Kranis R, Scalea TM,
Mirvis SE. CT findings after embolization for blunt splenic
trauma. J Vasc Interv Radiol. 2001;12(2):209-214.
36. Schnuriger B, Inaba K, Konstantinidis A, Lustenberger T, Chan
LS, Demetriades D. Outcomes of proximal versus distal splenic
artery embolization after trauma: A systematic review and metaanalysis. J Trauma. 2011;70(1):252-260.
37. Dror S, Dani BZ, Ur M, Yoram K. Spontaneous thrombosis of a
splenic pseudoaneurysm after blunt abdominal trauma. J Trauma.
2002;53(2):383-385.
38. McCray VW, Davis JW, Lemaster D, Parks SN. Observation
for nonoperative management of the spleen: How long is long
enough? J Trauma. 2008;65(6):1354-1358.
39. Dulchavsky SA, Lucas CE, Ledgerwood AM, Grabow D. Wound
healing of the injured spleen with and without splenorrhaphy. J
Trauma. 1987;27(10):1155-1160.

5/22/2012 5:42:31 PM

Commentary on
Splenic Salvage
Sherry Sixta, John B. Holcomb, and Jack H. Mayfield

In Chapter 61 entitled Splenic Salvage, the authors have thoroughly reviewed the recent debates and controversies surrounding nonoperative management (NOM) of blunt splenic injury
(BSI). NOM, whether simple observation or angioembolization
(AE) followed by observation, has become the standard of care
for hemodynamically stable patients with BSI. BSI management
has changed significantly over the past 30 years. It was not until
the middle of the 20th century that we began to understand the
immunologic risks associated with splenectomy. Prior to that
time, splenectomies were performed with impunity as the spleen
was felt to serve no function. Historically, there was a notion that
the spleen did not have the ability to heal, and if injured it would
ultimately bleed, resulting in lethal hemorrhage. The management
subsequently transitioned to laparotomy and splenorraphy. Even
up until the 1980s and the early 1990s, the term splenic salvage,
which now refers to observation AE, referred to splenorraphy.
The practice of splenorrhaphy was fraught with its own issues
including technical difficulty, rebleeding, splenic abscesses, and
morbidity. Once the CT scanner emerged and imaging became
more accurate, management evolved toward observation and AE.1
In 2000, a multicenter trial was published through EAST, which
documented the evolution of NOM of BSI. The percentages of adult
trauma patients admitted for observation of BSI increased from
48% in 1993 to 61% in 1997. Given the rapid transition in practice
patterns, this was the first large study to document outcomes in
correlation with splenic injury grade, ISS, hemoperitoneum, initial hematocrit, and blood pressure.2 By 2008, the transition was
profound. Data from the NTDB documented that over a 5-year
span, in 23,532 adult patients with BSI, only 10.3% were taken for
laparotomy within the first 2 h of hospital arrival.3 As management trends, including practice patterns associated with AE, have
evolved so quickly, historical reviews are difficult to meaningfully
analyze. Some of the literature is cohort in nature but all is retrospective. As the pendulum swung toward NOM, an entirely new
group of clinical problems evolved.
In 2008, Moore et al.4 from WTA published a consortium
statement as well as an algorithm for treatment of BSI and AE
based on a review of the literature, all Level 2 and 3 data, as well as
expert opinion. Management is dependent on hemodynamic instability (Graded 15), response to resuscitation, FAST examination,
and CT findings as available. Although the grade of splenic injury
does correlate with successful outcomes of NOM, it is not accurate enough to predict outcomes in individual patients. Therefore,
the CT may contribute to the decision-making process, but it is in

no way an absolutely reliable indication for either method. Hemodynamic response to initial resuscitation always takes precedence.
If the patient is a transient responder, AE can be considered; however, the surgeon must consider site-specific availability of AE,
balancing transfusion requirements/risks with immediate operative intervention. Nonresponders should proceed to the operating room for splenectomy. If the patient is normotensive and a
blush is seen on the CT scan, angiography is recommended. The
use of angiography is variable. There is a controversy over the
optimal use of AE due to the labor intensity, and multiple studies
have denoted a surprisingly high rate of complications. The WTA
multi-institutional data reported 140 patients who underwent
arterial embolization, of which 27 (20%) suffered major complications including 16 (11%) failure to control bleeding (requiring nine
splenectomies and seven repeat AE), 4 (3%) missed injuries, 6 (4%)
splenic abscesses, and 1 (1%) iatrogenic vascular injury. However,
more aggressive use of angiography is associated with the highest
rates of NOM (>80%) and the lowest rates of failure (25%).4
The evidence regarding the technique of AE, proximal versus
distal embolization, is inconclusive. The Schnuriger meta-analysis
referenced in the chapter claims that both methods were equivalent in respect to complications that ultimately led to failure of
AE, defined as splenectomy. However, complications that did not
ultimately lead to failure, such as rebleeding, infection, and infarction, were more often seen in distal embolizations. There is also a
concern that distal embolizations are more time consuming and
resource intensive. Moreover, the incidence of pseudoaneurysm
(PSA) after proximal embolization is unknown. Unless improved,
prospective studies can better elucidate the best technique,
local expertise and consensus, should dictate which AE technique
is utilized.
The mechanism behind delayed splenic bleeding is thought
to be a PSA or a pericapsular hematoma that subsequently ruptures. Delayed splenic bleeding is usually categorized as early or
late (>48 h). There is a small incidence of late bleeding (24%),
and most tend to occur within 4 to 8 days of injury. There have
been reports of hemorrhage weeks removed from injury.4 The
Weinberg data referenced in the chapter found that 7% (25/341)
of their patient population had PSAs on CT imaging: 4% were
seen on initial imaging and 3% were seen on follow-up CT scans
(2448 h after admission). They also found that PSAs were associated with splenic injury grade: 24% were in grade I and II injuries
and 76% were in high-grade injuries. Once identified, PSAs can
then be managed by AE. This is an area where prospective studies
496

PMPH_CH61.indd 496

5/22/2012 5:42:31 PM

Splenic Salvage

would be extremely helpful in clinical decision-making. However,


given the thought process of correlation of grade of injury with
development of PSA, regardless of the small incidence, it may be
worth repeating follow-up imaging depending upon the clinical
scenario.
The evidence of whether or not to vaccinate after AE is mixed.
The majority of the studies, other than the Nakae et al. paper, are
underpowered, and all are retrospective in nature. Although rare,
the lifetime risk of overwhelming postsplenectomy sepsis is 1% to
2% with a mortality rate of 33%,5 versus the very low risk of vaccinations. The current recommendation is to vaccinate AE patients
for encapsulated organisms. Large prospective controlled trials
are needed prior to alteration of vaccination recommendations.
Most trauma center protocols for NOM BSI call for serial monitoring of hemoglobins as well as continuous noninvasive hemodynamic monitoring for 24 to 48 h. The average stay from McCray
et al.s study is 3 0.8 days, though other reported observation rates
vary widely and range from 4.1 to 12 days.3 Smith et al. reported
that 95% of patients who fail NOM will do so within the first 3 days
post injury, and 2 more days of monitoring only identifies another
1.5%.3 Therefore, it seems reasonable to discharge the patient once
hemoglobins remain stable.
Savage et al. helped to delineate splenic healing times, revealing that lower grade injuries healed faster than higher grade injuries. Though the study was underpowered, it did help to defi ne an
average timeframe of 2 to 2.5 months for healing of 80% of injuries. Therefore, an outpatient following of 8 to 10 weeks seems reasonable. Patients can be followed clinically and symptomatically
without any need for further radiographic imaging. In general,
outpatient CTs are not usually necessary or helpful unless being
used to confirm healing for patients with high-risk activities such
as contact sports.4
Evidence supporting the notion that restriction of activity
after NOM prevents delayed splenic rupture is also lacking. However, it seems intuitive that there should be some timeframe of
activity limitation. The American Pediatric Surgical Association

PMPH_CH61.indd 497

497

recommendations are based on the grade of BSI. (Grade I: 3 weeks


restriction, Grade II: 4 weeks, Grade III: 5 weeks, and Grade IV:
6 weeks).6 In a survey of EAST surgeons, 95% of surgeons who
responded limited activity of NO-SBI patients, with the majority requiring 2 to3 months, but >20% mandating 4 to 6 months
restriction.7
This review points out the lack of quality data in this field,
and although recommendations are made the grade of those recommendations never rises above a C. For such a common injury,
this lack of quality data represents an outstanding opportunity for
a young surgeon to perform extremely important, high-quality
clinical research.

REFERENCES
1. Peitzman, Andrew B. MD; Richardson, J. David MD. Surgical treatment of injuries to the solid abdominal organs: A 50-year perspective from the Journal of Trauma. J Trauma. 2010;69(5):1011-1021.
2. Peitzman AB, Heil B, Rivera L, et al. Blunt splenic injury in adults:
Multi-institutional study of the Eastern Association for the Surgery of Trauma. J Trauma. 2000;49:177-189.
3. Smith J et al. Blunt splenic injuries: Have we watched long enough?
J Trauma. 2008;64:656-665.
4. Moore FA, Davis JW, Moore EE, Jr., Cocanour CS, West MA, McIntyre RC, Jr. Western Trauma Association critical decisions in trauma:
Management of adults splenic trauma. J Trauma. 2008;65:1007-1011.
5. Davies JM, Barnes R, Milligan D; British Committee for Standards in Haematology. Update of guidelines for the prevention and
treatment of infection in patients with an absent or dysfunctional
spleen. Clin Med. 2002;2(5):440-443.
6. Zarzaur BL, Vashi S, Magnotti LJ, Croce MA, Fabian TC. The real
risk of splenectomy after discharge home following nonoperative
management of blunt splenic injury. J Trauma. 2009;66(6):1531-1538.
7. Fata P, Robinson L, Fakhry SM. A survey of EAST member practices in blunt splenic injury: A description of current trends and
opportunities for improvement. J Trauma. 2005;59:836-842.

5/22/2012 5:42:31 PM

CHAPTER 62

Postsplenectomy Sepsis
Regan J. Berg and Kenji Inaba

INTRODUCTION

differences account for this variation including variable disease


definition, follow-up duration, population size and age distribution, prevalence of underlying disease, and time-related changes
in antibiotic and immunoprophylaxis utilization.
ONeal and McDonald11 reported a 2.7% incidence of lethal
sepsis in a 10-year, retrospective matched cohort study, a rate
similar to Singers, but they did not report nonlethal infections. Schwartz et al.12 reviewed 193 splenectomy patients from
the Mayo Clinic over 25 years, reporting incidence of mortality,
fulminant sepsis, and severe infection requiring hospitalization.
Mortality was 0.9 per 1000 person-years of follow-up, whereas the
rates of fulminant sepsis and severe infection were 0.18 and 7.16
per 100 person-years, respectively. The authors contrast their low
mortality rate with that reported by ONeal (7.3 per 1000 personyears), attributing this to unspecified differences in the two populations. An Australian study noted a 2.2% incidence of severe, late
postsplenectomy infection (0.42 per 100 person-years) and overall
mortality rate of 0.4% in 1490 patients over a 13-year period. This
risk of septicemia was 12.6 times that of an age-matched, nonsplenectomized cohort.13
Since Singers initial review, two more recent studies have
extensively examined the literature. Holdsworth et al.14 generated
a cohort of 12,514 patients from 59 series between 1952 and 1987.
Using only those series with full documentation (5902 patients),
they found the overall severe infection and mortality rates to be
2.9% and 1.5%. Similarly, Bisharat15 found an invasive infection
rate of 3.2% and a mortality rate of 1.4% in a review of 19,680 postsplenectomy patients (78 series) reported in the literature between
1966 and 1996. More recently, Kyaw et al.16 reviewed 1648 splenectomy patients, aged 1 to 94 years, during an 11-year period with
a mean follow-up of 4.5 years. Twenty-one percent developed a
severe infection requiring hospitalization, an overall incidence
of 7.0 per 100 person-years. Overwhelming infection (defined as
septicemia or meningitis) occurred at an incidence of 0.89 per 100
person-years. Variable inclusion of less severe infections as postsplenectomy sepsis is one factor responsible for the disparity in
incidence noted across the literature.

Over the first half of the 20th century, understanding of the spleen
shifted from its classical perception as an organ of relative inconsequence to one of key immunologic function, whose removal
risks rare but devastating infectious sequelae.1 In 1919, Morris
and Bullock 2 first recognized the spleens immunologic role after
noting high rates of fatal infection in splenectomized rats. King
and Shumacker3 reported the first case series of severe postsplenectomy sepsis in 1952, describing five infants with congenital
hemolytic anemia and concluding, the subsequent development
of serious infection was so constant as to suggest a cause-effect
relationship. In 1969, Diamond4 coined the term overwhelming postsplenectomy infection (OPSI) to describe an increasingly
noted syndrome of rapidly progressive and often fatal sepsis in
children. The syndrome was subsequently demonstrated in the
adult population.5-8 Growing awareness of this rare but devastating complication has prompted increasingly conservative surgical management of splenic disorders, particularly in traumatic
injury and malignant disease.9 Although diagnostic acceptance
of postsplenectomy sepsis is well established, the conditions rarity makes the determination of its true incidence, precipitants,
associated features, prevention, and management difficult,
despite considerable literature. Th is chapter reviews the current evidence regarding postsplenectomy sepsis, addressing key
questions of epidemiology, clinical presentation, treatment, and
prophylaxis.
1. Who is at risk for postsplenectomy sepsis, what is the magnitude of risk and what factors affect it?
Singer published the first major review of the literature in 1973,
reporting 2795 patients from 24 series, defining the condition
as septicaemia, meningitis, or pneumonia days to years after
splenectomy.10 The overall incidence of sepsis and mortality was
4.3% and 2.5%, respectively. Subsequent studies have reported
incidence rates both higher and lower. Significant methodologic

498

PMPH_CH62.indd 498

5/22/2012 5:43:51 PM

Postsplenectomy Sepsis

499

INTERVAL BETWEEN SPLENECTOMY


AND ONSET OF SEPSIS

RISK RELATED TO UNDERLYING


PATHOLOGY

The risk of postsplenectomy sepsis appears greatest in the first


2 to 3 years postsurgery in pediatric patients.4,17-19 Adult populations demonstrate a similar peak in early incidence.14 ONeal
reported average onset of lethal sepsis at 26 months (range
172).11 Schwartz found that the greatest risk of severe infection
occurred in the fi rst year.12 Holdsworths extensive literature
review found 32% of cases occurred within the fi rst year and
52% within the second year. Additionally, younger patients at
the time of splenectomy had a shorter interval to infection. Onethird of all cases, however, still occurred at least 5 years after
splenectomy.14
In Bisharats more recent review, mean time to infection was
22.6 months. Patients with spherocytosis and thalassemia had the
shortest mean time to infection (15 and 18 months, respectively)
and traumatically injured patients the longest (49.7 months).15 In
Kyaws clinical series, more than 50% of severe infections presented within the first year and 84% within 3 years. The interval
to first infection was also shorter in patients with malignancy,
both hematologic and nonhematologic.16 Although these studies
suggest an early peak in incidence, multiple case reports document infection 10 to 50 years postsplenectomy, suggesting lifelong
risk.20-23 Waghorn collected 40 cases of postsplenectomy sepsis
through microbiologic surveillance. The interval from operation
to infection ranged from 24 days to 64 years; 60% of these cases
occurred 10 to 30 years postsurgery.24 Cullingford noted that
42% of cases occurred more than 5 years after splenectomy.13 It is
unclear whether the often cited early peak in incidence is biased
by studies with shorter follow-up durations or over-representation
of pediatric populations. The risk appears to extend across the
lifespan.

Underlying pathology, as well as concomitant therapies, may influence the risk of sepsis. Risk of infection is higher in pediatric patients
after splenectomy for thalassemia major, portal hypertension, and
malignancy when compared with traumatic injury or nonmalignant
conditions.4,10,17,28 In adults, the presence of underlying hematologic
or malignant disease doubled the rate of fatal sepsis.11 Underlying
malignancy has been associated with five times greater risk of severe
postsplenectomy infection than of traumatic injury.12 The same
study also found that splenectomy for malignant disease or incidental to other abdominal surgery had significantly higher cumulative
risk of infection at 1 year, compared with splenectomy secondary
to trauma or nonmalignant hematologic and primary splenic disorders. Other studies have also found higher risk for severe infection with malignancy (hematologic or otherwise) and lower rates for
trauma.15,16 The increased incidence in malignant conditions likely
reflects both immunosuppressive aspects of the underlying disease and the impact of concomitant therapy. Schwartz found that
chemotherapy increased the relative risk of infection by a factor of
3.7, immunosuppressive therapy by a factor of 3.2, and radiation
treatment by a factor of 2.3.12 Many have also conjectured that the
decreased rates with traumatic splenectomy might reflect retained
immunologic function secondary to splenosis.9,13,29

EFFECT OF AGE GROUP ON RISK


Most series demonstrate a higher risk of infection in children
relative to adults.19,25,26 The youngest pediatric age group (<5
years) appears particularly vulnerable. Splenectomy in patients
below 4 years of age conferred a 2.5 time greater risk of infection than seen in older patients.27 Similarly, Walker et al.17 found
significantly increased risk in patients younger than 5 years.
Holdsworth also found a higher incidence of postsplenectomy
infection (4.4%) and mortality (2.2%) in children generally as
compared with adults (0.9% and 0.8%, respectively).14 Incidence
of infection did not differ between children and adults in Bishrats collective review (3.3 vs. 3.2%), but pediatric mortality was
slightly higher (1.7% vs. 1.3%). Children with thalassemia major
and sickle cell anemia suffered significantly higher mortality
than similarly affected adults (7.3% and 6.1% vs. 3.7% and 3.8%,
respectively).15
In contrast, one review found the risk of severe infection
highest in those who are older than 50 years of age (914 per
100 person-years). These patients were also more likely to have
second and third episodes of severe infection. Severe infection risk correlated with advancing age.16 These results may be
confounded by a very small number of patients (six) less than
5 years old.

PMPH_CH62.indd 499

RISK IN TRAUMA PATIENTS


The risk of severe infection after traumatic splenectomy is lower
than in patients with underlying disorders.10,12,14,25 Cullingford
determined an incidence of 0.21 per 100 person-years exposure.13
Despite a lower incidence, postsplectomy sepsis after trauma
has significant clinical consequence. A review of 47 published
case reports of severe infection following traumatic splenectomy described a mortality rate of 59%, 84% dying within 48 h of
symptom onset.30 Green et al. prospectively followed 18 patients
less than 15 years old undergoing splenectomy for trauma over
a mean duration of 5.8 years (range 112). They reported an 11%
incidence of overwhelming late infection (septicemia or meningitis), a rate four- to twentyfold greater than previous reports and
150 times that of the general pediatric population.31 The same
group prospectively followed 144 adults undergoing splenectomy
for trauma or intraoperative injury for a mean duration of 61
months (12114). Major infection occurred in 5.9% of those suffering trauma and 18.5% of those with splenectomy secondary to
intraoperative injury. The overall rate of major infection was 8.3%,
166 times that expected in the general population.32 These are the
only prospective studies to date but are weakened by small sample
size, in the first study, and loss of a significant number of patients
to follow-up (37%) in the latter.

SUMMARY
Despite considerable literature, the true incidence of postsplenectomy sepsis remains unknown due to considerable heterogeneity
across published studies. The literature suggests an incidence of
1.4% to 4.3% with an associated mortality incidence of 0.4% to

5/22/2012 5:43:51 PM

500

Surgery: Evidence-Based Practice

2.5%. (Grade B recommendation) Risk may be greater in the first


2 to 3 years postsplenectomy, particularly in children <5 years of
age, but extends across the lifespan. (Grade B recommendation)
Risk is also higher in those with underlying malignant or hematologic disease, or those undergoing immunosuppressive therapies,
and is lowest in trauma patients. (Grade B recommendation)
2. What are the typical pathogens involved and are there emerging pathogens?
The encapsulated bacteria Streptococcus pneumoniae, Hemophilius
influenzae, and Neisseria meningitidis are traditionally most frequently implicated in overwhelming postsplenectomy sepsis.9,10,33
A major component of the reticuloendothelial system, the spleen,
is important in both the specific and the nonspecific immune
response.29,34 Splenic recognition of carbohydrate antigens, early
IgM production, and removal of poorly opsionized particles likely
account for the increased rate of sepsis due to encapsulated bacteria in asplenic individuals.9,35,36 These polysaccharide-coated bacteria resist antibody binding and subsequent clearance by hepatic
kupffer cells, increasing the relative importance of splenic sequestrative and phagocytic function.35,37-39 The importance of intact
splenic function is increased in infants and very young children,
who lack specific antibody function, or in adults with concomitant disease or therapy modifying immune response.
Pathogen frequency varies between series, and may be altered
by increasing utilization of immunoprophylactic strategies, but
S. pneumoniae historically accounts for 50% to 90% of postsplenectomy infection.9,10,24,37,40,41 In the two most recent large collective
reviews, examining 349 and 356 episodes of postsplenetomy infection, S. pneumoniae accounted for 56.7% and 66%, respectively.14,15
H. influenza is the second most frequent pathogen, although far
less common, accounting for 5% to 6% of infections.13,14 N. meningitidis is cited as the third most common, although rare. Less commonly thought of as an encapsulated bacterium, Escherichia coli,
which possesses an acidic polysaccharide microcapsule, may be
more frequent than usually recognized. An Australian review of
1490 splenectomized patients found that E. coli accounted for 26%
of positive cultures and S. pneumoniae only 32%.13 Not all studies support such frequency; Holdsworths review of 349 reported
episodes of infection found E. coli far less common (3.7%) but still
equal in incidence to N. meningitidis.14 A Danish population-based
review of 40 bacteremias, in 538 splenectomized patients between
1983 and 1994, also found E. coli to be the most common single
pathogen, accounting for 25% of infections. Gram-negative rods
as a group comprised 45% of all bacteremias, whereas traditional
encapsulated bacteria were infrequent.42 Sixty percent of this population was vaccinated against S. pneumoniae but direct association between immunoprophylaxis and the evolving bacteriology
of postsplenectomy sepsis has not been clearly demonstrated.
Although encapsulated bacteria predominate in most studies,
postsplenectomy sepsis can occur secondary to any bacterial, viral,
fungal, or protozoan organism. Other implicated bacterial species
include Pseudomonas aeruginosa, B-hemolytic strep, Strep sanguis,
Salmonella species, Staphylcoccus aureus, bacteroides species, and
Plesiomonas shigelloides.9,13,14,23,33,37,43,44
Capnocytophaga canimorsus and Bordetella holmesii are two
more recently recognized pathogens. Capnocytophaga canimorsus
(formerly DF2) is a gram-negative rod found primarily in dog, but
also in cat saliva, first reported as a human pathogen in 1976.45 Rarely

PMPH_CH62.indd 500

virulent in healthy hosts, as many as 90% of affected patients have


underlying immunosuppression, most often secondary to asplenia or to alcohol use.45-47 Bordetella holmseii, a gram-negative rod
first characterized in 1995, is a cause of bacteremia, endocarditis,
and respiratory illness in children and adults.48,49 Eighty-five percent of affected patients in one series were functionally or anatomically asplenic.50
Splenic function is also important in defense against protozoal disease. Babesiosis is a zoonotic infection transmitted by
the Ixodes tick that produces mild or subclinical infections in
healthy hosts but severe and potentially fatal disease in asplenic
individuals.51,52 A review of 22 reported cases of babesiosis in
splenectomized patients, from 1957 to 1984, found an overall
mortality rate of 27%.53 Splenectomized patients accounted for
18% of clinical and subclinical infections in one review, 41% of
clinical infections in another.54,55 The role of the spleen in defense
against malaria is less clear. Animal and human studies indicate
splenic importance in the removal of parasitized erythrocytes and
the generation and maintenance of immune response.56 It is still
unclear, however, if splenectomized patients have an increased
risk of infection or disease severity.56,57
Increased susceptibility of asplenic patients to viral infections is not well documented. Influenza, however, is important as
a precipitant of secondary bacterial infection and sepsis in this
population.8

SUMMARY
Encapsulated bacteria, S. pneumoniae (5566%), H. Influenza (35%),
and N. meningitidis (34%), have traditionally accounted for the
majority of postsplenectomy infections. (Grade B recommendation). The effect of immunnoprophylaxis on subsequent pathogen
frequency is unclear. Increasing evidence, however, indicates growing importance of gram-negative rods, particularly E. coli. (Grade B
recommendation). Capnocytophaga canimorsus and B. holmesii are
important emerging pathogens. (Grade C recommendation). Tickborne babesiosis can result in severe and fatal disease in asplenic
patients. (Grade C recommendation)
3. What are the clinical presentations and outcomes of postsplenectomy sepsis?
The classic presentation is based on Singers initial categorization of the syndrome as a fulminant bacteremia, meningitis, or
pneumonia following splenectomy. A brief nonspecific prodrome
may occur, variably followed by headache, photophobia, nausea,
emesis, diarrhea, malaise, fever, rigors, and confusion.9,5,40,41,58
Adults typically present without an obvious septic focus whereas
meningitis is the predominant presentation in children.14,19,37,43
Rapid progression to septic shock occurs and death can ensue
within 2448 h. Disseminated intravascular coagulation (DIC),
adrenal hemorrhage (WaterhouseFriderichsen syndrome), purpura fulminans, and bilateral extremity gangrene accompany
severe cases.5,8,30,38,59-62 Patients demonstrate massive bacteremia
(greater than 106/mL), in contrast to nonsplenectomized individuals who rarely show more than 103 organisms per mL.40 Capnocytophaga canimorsus sepsis can present a similar fulminant
course but is also associated with arthritis, meningitis, and endocarditis.45 Peripheral extremity gangrene and purpura fulminans

5/22/2012 5:43:51 PM

Postsplenectomy Sepsis

are common.45,47,63 Severe abdominal pain can occur, mimicking


peritonitis.46,64,65 Bortedella holmseii is associated with bacteremia,
endocarditis, and pneumonia but tends to have a milder course,
even in asplenic patients.48,50

Outcomes
Although a rare complication, the true significance of postsplenectomy sepsis derives from associated mortality rates of 50% to 70%
despite appropriate antibiotic therapy.9,37 Singers initial study concluded that death from sepsis was 200 times more likely in asplenic
individuals.10 Subsequent reviews suggested rates of lethal sepsis
300 to 540 times greater in splenectomized patients compared with
the general population11,13 Mortality rates are influenced by the age
of the patient, the presence of underlying disease, and the etiologic
agent. Patients with hematologic or malignant disease demonstrate
mortality rates from postsplenectomy sepsis nearly double those
without underlying illness.11 Waghorn found postsplenectomy
sepsis highest in those with coexistent hematologic malignancy
(69%), lower in trauma patients (46%), and lowest in patients without traumatic injury, hematologic disease or malignancy (38%).24
Thalassemia major and sickle cell anemia are also associated with
higher morality rates than trauma or spherocytosis.15
Patient age is also a factor. Mortality incidence among thalassemia major and sickle cell anemia pediatric patients is significantly
higher than in similarly affected adults (7.3% and 6.9% vs. 3.7% and
3.8%, respectively).15 Holdsworth found a significantly higher incidence of postsplenectomy sepsis mortality in children (2.2%) compared with adults (0.9%).14 This study also suggests a more complex
interaction between age, clinical presentation, and mortality. Pure
meningitis without bacteremia had a relatively low (22%) mortality
rate and 83% of these cases occurred in patients younger than 15.
Bacteremia alone produced 65% mortality and occurred in all age
groups. Bacteremia and meningitis, however, occurred primarily
in adult patients, associated with a 76% mortality rate.
The type of pathogen may also affect survival. Bisharat found
slight mortality rate variation between S. pneumoniae (55%),
gram-negative bacteria (62%), and N. meningitidis (59%).14 More
convincingly, E. coli and N. meningitidis produced greater mortality rates than S. pneumoniae and H. influenzae (77% and 78% vs.
57% and 32%, respectively).14 The factors contributing to mortality in postsplenectomy sepsis likely interact in a complex manner
that has not been adequately assessed.

SUMMARY
Severe postsplenectomy infection is associated with fatality rates of
50% to 70% despite antibiotic treatment. (Grade B recommendation). Young children, particularly those less than 5 years old, have
increased mortality, as do those with coexistent malignant or hematologic disease or those undergoing concomitant immunosuppressive
therapy. (Grade B recommendation). Traumatic splenectomy is associated with the lowest mortality rates. (Grade B recommendation)
4. What is the role of antibiotics in preventing postsplenectomy sepsis?
The use of prophylactic antibiotics in splenectomized or functionally asplenic patients is widely advocated but good evidence to

PMPH_CH62.indd 501

501

support efficacy is scarce. The PROPS study, a multicenter, randomized, double-blind, placebo-controlled trial, reported an 84%
reduction in the rate of infection in children with sickle cell disease
given daily oral penicillin. Three deaths occurred in the placebo
but none in the treatment group and the trial terminated early, after
an average of 15 months follow-up.66 A Cochrane review has since
supported the efficacy of penicillin prophylaxis in this patient population.67 Sickle cell disease is associated with functional asplenia
but no prospective trial has specifically examined splenectomized
pediatric patients or adult populations. A more recent retrospective review of 318 patients (mean age 14 years; range 526), who
underwent splenectomy between 1985 and 1997, suggested a significant difference in infection between those taking regular penicillin (2.7%) and those who did not (20%) over a median follow-up
of 10.5 years. The overall incidence of postsplenectomy infection in
this patient group was 5.7%.68 Oral penicillin or amoxicillin is the
traditionally advocated prophylactic agent but growing pneumococcal antibiotic resistance has prompted the utilization of broader
spectrum drugs such as fluroquinolones, macrolides, amoxicillin/
clavulanate, and trimethoprim/sulframethoxazole.37,39
The appropriate duration of antibiotic prophylaxis is also
uncertain. In a follow-up to the original PROPS study, the
authors concluded that prophylaxis could be safely discontinued
by 5 years of age. However, this recommendation was directed
at patients with sickle cell disease, receiving concomitant vaccination and regular medical care, who did not have a surgical
splenectomy or a prior severe episode of pneumococcal sepsis.69
Prophylaxis in pediatric patients, particularly those younger than
5, for at least 2 years following surgery is frequently advocated,
reflective of the perceived increased risk during this period.44,70,71
The American Academy of Pediatrics suggest continuation until
at least age 5.72 Some suggest continuation into adulthood for
high-risk patients.73
The use of prophylactic antibiotics in adult populations has
not been prospectively studied and is controversial. Given the risk
of lifelong sepsis, some authors advocate continual prophylaxis.74-76
Short duration (23 years) prophylaxis, particularly in high-risk
patients, has also been suggested.9,77-79 Others contend that no prophylaxis is needed.80,81
Failures of antibiotic prophylaxis have been reported.82-85
Patient adherence to daily prophylaxis can be problematic in both
children and adults.77,86,87 The variable contributions of patient
adherence, evolving resistance patterns, and changing frequency
of causative pathogens to these cases of failure are unclear. In one
review of 77 cases of postsplenectomy sepsis, 14% of cases were
taking regular prophylaxis at the time of infection.24
The use of stand-by antibiotics, for early patient-initiated
treatment, is also widely advocated.9,41,43 This practice has not been
studied but is consistent with known benefits of early antibiotic
therapy in the setting of sepsis.88 However, no evidence exists to
suggest that outcomes are improved or that benefit exists over
patient education and prompt presentation to medical attention,
at first sign of illness, alone.

SUMMARY
Antibiotic prophylaxis benefits pediatric patients with functional
asplenia secondary to sickle cell disease. (Grade A recommendation). The use in anatomically asplenic children and adults has not

5/22/2012 5:43:51 PM

502

Surgery: Evidence-Based Practice

been studied but recommendations for prophylaxis in the first


2 years following splenectomy in all pediatric patients until adulthood and long-term use in high-risk populations are common
despite clear evidence of efficacy. (Grade C/D recommendation).
Similarly, patient-initiated stand-by antibiotics therapy is often
advocated, although efficacy and risk benefit have not been established by trials. (Grade D recommendation)
5. What vaccines should be given, and when, and how effective
are they in preventing postsplenectomy sepsis?
Vaccines exist for three of the pathogens implicated in postsplenectomy sepsis: S. pneumoniae, H. influenzae type b, and N. meningiditis. Ninety known different serotypes of S. pneumoniae exist.34
In the U.S., two vaccines are available. A 23-valent polysaccharide
vaccine (PPSV23) covers 85% to 90% of serotypes responsible for
invasive pneumococcal infection.89 A new 13-valent conjugate
vaccine (PCV13), approved by the FDA in 2010, replaces the previous 7-valent vaccine for pediatric use. PCV13 is approved for
all children aged 2 to 59 months and in those up to 18 years with
conditions increasing the risk of invasive disease.90,91
Six serotypes (Af) of H. influenzae cause invasive disease.
The current conjugated vaccine covers serotype B (HiB), known to
cause 95% of invasive disease in children younger than 5.92 Vaccination is recommended for all asplenic children and adults.93 Both
pneumococcal and the HiB vaccine have been shown immunogenic in asplenic patients.94
Neiserria meningitidis has 13 serotypes, with 90% of infections worldwide caused by serotypes A, B, and C.34 Two current
U.S. vaccines protect against serotypes A, C, Y and W-135, a tetravalent polysaccharide vaccine (MPSV) and a newer tetravalent conjugate vaccine (MCV4).95 MCV4 is approved for patients
between 11 and 55 years old. Children 2 to 10 years of age, or
persons older than 55, should receive MPSV. Asplenic individuals
should receive a two-dose primary series, administered 2 months
apart, and a booster dose every 5 years.95
Although asplenic individuals are not known to be at increased risk for influenza, yearly immunization is widely recommended due to the diseases association with secondary bacterial
infection.8,96,97
Optimal timing of vaccination has been debated. Serum
IgM antibodies become detectable around 9 days following a
novel antigen exposure, leading to wide recommendation for
vaccination at least 2 weeks prior to elective splenectomy. 34,96,98
The timing of vaccination after emergent splenectomy has
been addressed in two prospective, randomized trials by Shatz
et al.99,100 Trauma patients undergoing splenectomy demonstrated greatest functional antibody response to pneumococcal polysaccharide vaccine given 14 days postoperatively
when compared with days 1 or 7. Antibody response was not
improved by delaying vaccination to 28 days. A Danish study
examined pneumococcal antibody levels in a population of
remotely splenectomized patients.101 Need for re-immunization
was more likely in those vaccinated less than 14 days before
or after splenectomy, lending support to these parameters for
timing vaccination in both elective and emergent cases. Vaccination is generally performed 6 months after treatment for
patients undergoing chemo- or radiotherapy to allow reconstitution of adequate immune response.96,102

PMPH_CH62.indd 502

Revaccination is required as antibody levels diminish with


time.98 Antibody concentrations decreased linearly by 24% to
32% from peak levels in splenectomized adults and children in
the fi rst year following vaccination.103 In patients with splenectomy for trauma or Hodgkins disease, antibody levels declined
to prevaccination levels over 3 years.104 Revaccination has been
shown to be safe and effective.105,106 The U.S. Advisory Committee on Immunization Practice (ACIP) currently recommends
pneumococcal revaccination at 5 years for adult patients with
functional or anatomic asplenic but does not currently support
multiple revaccinations without further evidence of clinical
benefit.90
Few studies have examined the efficacy of vaccination in
splenectomized patients. A recent Cochrane meta-analysis
of 15 randomized controlled trials and 7 observational studies suggested an overall pneumococcal vaccine efficacy of 74%
but did not specifically address asplenic patients.107 An earlier indirect cohort analysis of 2837 pneumococcal infections,
however, found 77% vaccine efficacy in anatomically asplenic
individuals.108 Observational studies in splenectomized children and adults also suggest benefit for immunoprophylactic
strategies.73,109
Despite considerable efficacy in preventing invasive disease,
vaccination failure occurs.82,110-112 Vaccine failure may reflect waning antibody response, coexistent immunosuppressive conditions,
or rare infection with a serotype not covered by the vaccine.24,98 Also
contributing to the persistent risk of infection is evidence of highly
variable adherence to immunoprophylaxis guidelines among clinicians and minimal patent awareness of infectious risks following
splenectomy. A review of 70 cases of postsplenectomy infection,
87% secondary to S. pneumoniae, found only a 31% vaccination
rate against this organism.113 Other studies report pneumococcal
vaccination rates from 11% to 88% with much lower overall rates for
H. influenzae and meningococcus.79,114-117 Splenectomy for traumatic or intraoperative injury was associated with far lower vaccination rates compared with splenectomy for other etiologies
in one study.118 A survey of North American surgeons found
wide variation in immunization practice, particularly regarding
revaccination.119
Patient education and awareness of risk have been found
deficient in multiple studies, a potentially correctable barrier to
immunoprophylactic strategy adherence and early detection and
treatment of disease.68,115,116,120-122

SUMMARY
Pneumoccocal vaccine is effective in asplenic patients and vaccination against S. pneumoniae, H. influenzae type b, and N. meningitidis should be performed at least 2 weeks prior to elective
splenectomy. (Grade C recommendation). Vaccination administration at 14 days after emergent splenectomy appears optimal.
(Grade A recommendation). Antibody levels in asplenic patients
diminish over time and re-immunization with meningococcal
and pneumococcal vaccine at 5 years is recommended. (Grade C
recommendation). Yearly influenza vaccine may have benefit in
preventing secondary bacterial infection and sepsis in asplenic
patients. (Grade C/D recommendation)

5/22/2012 5:43:51 PM

Postsplenectomy Sepsis

6. What role do splenosis, autotransplantation, conservative


splenic surgery, and splenic embolization have in preventing
postsplenectomy sepsis?
Splenosis, ectopic tissue found in patients following splenic rupture,
has been proposed as an explanation for the lower rates of postsplenectomy sepsis seen after traumatic injury.34 In 1978, Pearson
et al.123 found that 59% of 22 children with traumatic splenectomy
had low counts of pitted erythrocytes, similar to eusplenic individuals, in contrast to the much higher counts seen in patients splenectomized for hematologic indications. One study found increased
tuftsin activity and decreased pitted erythrocytes in patients undergoing traumatic rather than elective splenectomy, felt suggestive
of residual splenic function attributed to splenosis.124 One patient
found to have post-traumatic splenosis incidentally, at subsequent
abdominal surgery, had some residual spleen-associated phagocytic and immunologic activity.125 This concept of the born again
spleen has persisted in the literature despite lack of strong evidence
of its efficacy in preventing infection. In one series of 47 cases of
severe postsplenectomy infection following trauma, four fatal cases
were found to have ectopic splenic tissue at autopsy.9 Another series
noted the presence of splenosis or accessory splenic tissue in 26%
of 39 fatal infections.126 Multiple case reports and series describe
fatal infection despite the presence of splenic nodules or accessory
splenic tissue, weighing as much as 92 gm.9,127-129 Although there is
a definite publication bias in favor of failed cases of splenosis, one
large review of the literature to date concluded that little evidence
exists of any protective benefit.130
Re-implantation of splenic tissue at the time of operation, autotransplantation, is also thought to preserve some degree of immune
function. Initial animal models suggested more rapid bacterial
clearance and improved survival than in splenectomy, but required
omental as opposed to subcutaneous, intramuscular, or intraperitoneal placement.131-133 Regeneration of autotransplanted splenic tissue was also better in young rats compared with old rats, suggesting
a possible requisite developmental window.134 Re-implantation of at
least 50% of original splenic tissue was necessary to produce benefit
in rodent models, but linear increases in immunologic function to
a maximal 80% were observed.132,135 In contrast, a porcine model
found that the size of implants or mass of grafted tissue has no effect
on the regenerated splenic mass at 6 months.136
In humans, splenic reticuloendothelial function was prospectively assessed in 51 trauma patients with suspected splenic injury
over 2 to 5 years. Splenorrhaphy or partial splenectomy was associated with lower levels of pocked erythrocytes and greater clearance
of antibody-labeled autologous cells than splenectomy, suggesting a
maintained splenic function. Autotransplantation resulted in better
function than splenectomy though not as good as when the spleen
was conserved.137 A comparison of 10 autotransplanted patients
with 14 patients undergoing splenectomy alone, found significantly
greater rises in antibody titers (both IgM and IgG) in response to
pneumococcal vaccination, suggesting potential immunologic benefit.138 This study, however, lacked a control group. Subsequently,
the same authors found a similar antibody increase (in response to
23-valent pneumococcal vaccine) in autotransplanted and shamoperated rats, compared with a significantly lower rise in those
splenectomized.139 The capacity of splenic autotransplants to restore
immunologic and phagocytic function has been questioned in two
more recent animal models.140,141

PMPH_CH62.indd 503

503

Failure of autotransplantation has been documented in


human case reports.142,143 Determining the significance of these
rare events is difficult and there is clearly a relevant publication
bias. In one case, implantation occurred in the anterior rectus and
in the other, less than 2 gm of tissue was implanted. One study
examined 90 postsplenectomy patients with scintigraphy and red
cell microscopy, suggesting 20 to 30 cm3 of retained splenic tissue
necessary for immunologic function but no human trial has adequately assessed this issue.144 Moore et al.145 reviewed 43 splenic
autotransplantation patients over a 6-year period, noting one
death from postsplenectomy sepsis. The most recent systematic
review of the autotransplantation literature, published in 1994,
concluded that the procedure was relatively safe and easy to perform with some normalization of the measured immunologic and
reticuloendothelial function but with unknown effect on morbidity and mortality.146 Although the bulk of evidence hints potential
benefit, this has been established or quantified in any large, welldesigned trial.
Immunologic function appears retained in animals undergoing partial splenectomy.133,147-149 In children with hereditary
spherocytosis, partial splenectomy preserved immunologic and
phagocytic function when assessed by peripheral blood smear,
red blood cell microscopy, immunoglobulin levels, and specific
antibody titers.149 A group of 12 thalassemia patients with partial splenectomies did not suffer any fatality due to sepsis over a
4-year follow-up period in contrast to five deaths in a group of
30 patients having total splenectomy. These patients did not have
access to immunoprophylaxis.150 In both these studies, partial
splenectomy provided adequate control of the underlying disease. In trauma patients, subtotal splenectomy was associated
with the absence of HowellJolly bodies and normal antibody
levels. No early complications occurred specifically related to
the splenic surgery but clinical correlation of the observed functional parameters was limited by the lack of follow-up beyond
the late postoperative period.151
Although not studied by any large prospective trials, a small
body of evidence suggests that splenic artery embolization preserves immunologic function. Thirty-seven adult trauma patients
successfully managed with proximal splenic artery embolization
retained normal splenic size, homogeneity, and immunologic
function, as measured by the exposure-driven antibody responses
to H. influenza and S. pneumoniae.152 Splenic volume and a variety of cellular counts, complement, and immunoglobulin levels
were evaluated in patients with abdominal trauma who either had
a CT scan without evidence of significant injury, a splenectomy,
or had a splenic artery embolization. No significant difference
was found between these groups.153 Other authors have reported
similar preservation of immunologic function.154,155 These studies
have been variably subject to criticism for retrospective methodology, lack of appropriate controls, lack of tests able to discriminate
the asplenic state, or utilization of tests examining organ viability rather than immunologic function. To address these limitations, Malhotra et al.156 examined CD4+ T cell counts and specific
CD4+ subpopulations in patients having embolization comparing
them with control groups of splenectomized and normal individuals. Splenic embolization preserved immunologic function, as
measured by the T cell subset, tests that allowed the detection of
asplenic individuals.156

5/22/2012 5:43:51 PM

504

Surgery: Evidence-Based Practice

SUMMARY

CONCLUSION

Evaluation of a protective benefit for splenosis and splenic autotransplantation is hampered by the lack of large studies and a publication
bias favoring failed cases. A benefit for post-traumatic splenosis is
not established by the literature to date. (Grade D recommendation). Autotransplantation of splenic tissue may preserve some
immunologic capability, but demonstration of reduced infectious
risk has not clearly been shown and the amount of tissue required
to preserve function is unclear. (Grade C recommendation). Partial
splenectomy, in suitable patients, appears to preserve immunologic
function and is associated with reduced infection risk. (Grade C
recommendation). Similarly, immune function appears preserved
after splenic embolization, although a reduction in the subsequent
risk of infection has not been studied. (Grade C recommendation)

A paucity of high-quality evidence exists to aid in the prevention and management of postsplenectomy sepsis, largely due
to the inherent difficulty of studying a rare clinical condition.
Although the true incidence is unknown, the risk clearly extends
across the lifespan, is influenced by underlying disease and concomitant therapy, and produces high mortality. The efficacy of
immunoprophylactic strategies may improve with greater clinician adherence to management guidelines and patient education.
Particularly in trauma, where the limits of nonoperative management of splenic injury are being continually tested, a real understanding of this disease is necessary to balance its risk with the
equally real, and potentially greater, risks of failure of nonoperative management.157

Clinical Question Summary


Question

Answer

Level of
Evidence

1 What is the
epidemiology of
postsplenectomy
sepsis (PSS)?

True incidence remains unknown but ranges


from 1.4% to 4.3% with mortality incidence
of 0.4% to 2.5% are reported.
Age-related incidence of overwhelming
infection ranges 0.18 to 0.89 per
100 person-years.
Risk may be higher in the first 2 to 3 years
postsplenectomy but persists lifelong and is
higher after splenectomy for malignant and
hematologic disease compared with trauma.

2b3

10-17

2b4

12, 16

4, 11-14, 17-19,
20-25

Streptococcus pneumoniae (5566%),


H. influenzae (35%), and
N. meningitidis (34%) are traditionally the
most frequently implicated pathogens.
Gram-negative rods (particularly E. coli)
may be implicated more frequently than
often recognized.
C. canimorsus and B. holmesii are important
emerging pathogens.

2b4

13-15, 42

2b, 2c

13, 42

3b, 4

45-51

Morality ranges from 50% to 70% despite


treatment.
Young children (less than 5) may have higher
mortality.
Coexistent malignant and hematologic disease
increase mortality.

2b, 2c

11, 15, 24

2b

14, 15

2a, 3b

11, 15, 24

Antibiotic prophylaxis benefits functionally


asplenic, sickle cell pediatric patients.
Use in pediatric patients, or in all patients
during the 2-year period following
splenectomy, and long-term use in highrisk populations are frequently advocated
despite lack of clear evidence for benefit.
Patient-initiated stand-by antibiotic
treatment for early infection is often
advocated but efficacy has not been
examined.

1a, 1b

66, 67, 69

2 What pathogens
cause PSS?

3 What mortality is
associated with PSS?

4 Do antibiotics prevent
PSS?

45

Grade of
Recommendation

References

C/D

9, 70-79

9, 41, 43

(Continued)

PMPH_CH62.indd 504

5/22/2012 5:43:51 PM

Postsplenectomy Sepsis

505

(Continued)
Question

Answer

Level of
Evidence

5 What vaccinations
should be given and
when?

Vaccination appears effective in asplenic


patients. Meningococcal, pneumococcal,
and H. influenzae B vaccines should be
given at least 2 weeks prior to elective
splenectomy.
Vaccination following emergent splenectomy
appears optimal at 14 days.
Re-immunization at 5 years is recommended
for pneumococcal and meningococcal
vaccines as antibody levels diminish with
time.
Yearly influenza vaccination is recommended
to reduce the risk of secondary bacterial
infection and sepsis.

2b5

34, 71, 90, 108,


109

1b

99, 100

2b5

90, 95, 98, 103,


104

4, 5

C/D

A protective benefit of post-traumatic


splenosis is not supported by the current
literature.
Splenic autotransplantation may preserve
some residual immunologic function but
subsequent risk reduction for sepsis or
mortality has not been established.
Partial splenectomy retains some immunologic
function and may reduce the risk of sepsis.
Splenic embolization appears to preserve
immunologic function but long-term clinical
impact has not been studied.

2b5

30, 123-130

2b4

137, 138, 145,


146

2b, 3b

137, 138, 149-151

2b, 3b

152-156

6 Does splenosis,
splenic
autotransplantation,
partial splenectomy,
or splenic
embolization prevent
infection?

REFERENCES
1. McClusky DA, Skandalakis LJ, Colborn GL, Skandalakis JE.
Tribute to a triad: History of splenic anatomy, physiology, and
surgery-part 2. World J Surg. 1999;23(5):514-526.
2. Morris DH, Bullock FD. The Importance of the spleen in resistance to infection. Ann Surg. 1919;70:513-521.
3. King H, Shumacker HB, Jr. Splenic studies: I. Susceptibility to
infection after splenectomy performed in infancy. Ann Surg. 1952;
136(2):239.
4. Diamond LK. Splenectomy in childhood and the hazard of overwhelming infection. Pediatrics. 1969;43(5):886-889.
5. Gopal V, Bisno AL. Fulminant pneumococcal infections in normal asplenic hosts. Arch Int Med. 1977;137(11):1526.
6. Dickerman JD. Bacterial infection and the asplenic host: A
review. J Trauma. 1976;16(8):662.
7. Robinette CD, Fraumeni JF. Splenectomy and subsequent mortality in veterans of the 1939-45 war. Lancet. 1977;2(8029):127-129.
8. Roberts GT, Roberts JT. Postsplenectomy sepsis due to influenzal viremia and pneumococcemia. Can Med Assoc J. 1976;115(5):
435-437.
9. Lynch AM, Kapila R. Overwhelming postsplenectomy infection.
Infect Dis Clin North Am. 1996;10(4):693-707.
10. Singer DB. Postsplenectomy sepsis. Perspect Pediatr Pathol. 1973;
1:285-311.
11. ONeal BJ, McDonald JC. The risk of sepsis in the asplenic adult.
Ann Surg. 1981;194(6):775.
12. Schwartz PE, Sterioff S, Mucha P, Melton LJ, III, Offord KP.
Postsplenectomy sepsis and mortality in adults. JAMA. 1982;
248(18):2279.

PMPH_CH62.indd 505

Grade of
Recommendation

References

96, 97

13. Cullingford GL, Watkins DN, Watts AD, Mallon DF. Severe late
postsplenectomy infection. Br J Surg. 1991;78(6):716-721.
14. Holdsworth RJ, Irving AD, Cuschieri A. Postsplenectomy sepsis
and its mortality rate: Actual versus perceived risks. Br J Surg.
1991;78(9):1031-1038.
15. Bisharat N, Omari H, Lavi I, Raz R. Risk of infection and death
among post-splenectomy patients. J Infect. 2001;43(3):182-186.
16. Kyaw MH, Holmes EM, Toolis F, et al. Evaluation of severe infection and survival after splenectomy. Am J Med. 2006;119(3):276.
e1-e7.
17. Walker W. Splenectomy in childhood: A review in England and
Wales, 1960-4. Br J Surg. 1976;63(1):36-43.
18. Di Cataldo A, Puleo S, Li Destri G, Racalbuto A, Trombatore G,
Latteri F, et al. Splenic trauma and overwhelming postsplenectomy infection. Br J Surg. 1987;74(5):343-345.
19. Shaw JH, Print CG. Postsplenectomy sepsis. Br J Surg. 1989;
76(10):1074-1081.
20. Hollis N, Marsh RH, Marshall RD, Robertson PC. Overwhelming
pneumococcal sepsis in healthy adults years after splenectomy.
Lancet. 1987;1(8524):110.
21. Evans DI. Postsplenectomy sepsis 10 years or more after operation. J Clin Pathol. 1985;38(3):309.
22. Nelson J, McMullin MF. Postsplenectomy sepsis: A lifelong risk.
Ulster Med J. 1993;62(2):166-169.
23. White BP, Aanning HL. Overwhelming postsplenectomy sepsis
twenty-two years after operation risks management and prevention. S D J Med. 1991;44(11):317-320.
24. Waghorn DJ, Mayon-White RT. A study of 42 episodes of overwhelming post-splenectomy infection: Is current guidance for
asplenic individuals being followed? J Infect. 1997;35(3):289-294.

5/22/2012 5:43:52 PM

506

Surgery: Evidence-Based Practice

25. Ellison EC, Fabri PJ. Complications of splenectomy. Etiology,


prevention, and management. Surg Clin North Am. 1983;63(6):
1313-1330.
26. Chaikof EL, McCabe CJ. Fatal overwhelming postsplenectomy
infection. Am J Surg. 1985;149(4):534-539.
27. Eraklis AJ, Kevy SV, Diamond LK, Gross RE. Hazard of overwhelming infection after splenectomy in childhood. N Engl J Med.
1967;276(22):1225-1229.
28. Eraklis AJ, Filler RM. Splenectomy in childhood: A review of
1413 cases. J Pediatr Surg. 1972;7(4):382-388.
29. Likhite VV. Immunological impairment and susceptibility to
infection after splenectomy. JAMA. 1976;236(12):1376-1377.
30. Zarrabi MH, Rosner F. Serious infections in adults following
splenectomy for trauma. Arch Int Med. 1984;144(7):1421-1424.
31. Green JB, Shackford SR, Sise MJ, Powell RW. Postsplenectomy
sepsis in pediatric patients following splenectomy for trauma:
A proposal for a multi-institutional study. J Pediatr Surg. 1986;
21(12):1084-1086.
32. Green JB, Shackford SR, Sise MJ, Fridlund P. Late septic complications in adults following splenectomy for trauma: A prospective analysis in 144 patients. J Trauma. 1986;26(11):999-1004.
33. Davidson RN, Wall RA. Prevention and management of infections in patients without a spleen. Clin Microbiol Infect. 2001;
7(12):657-660.
34. Howdieshell TR, Heffernan D, Dipiro JT. Surgical infection society guidelines for vaccination after traumatic injury. Surg Infect.
2006;7(3):275-303.
35. Wara DW. Host defense against streptococcus pneumoniae: The
role of the spleen [with discussion]. Rev Infect Dis. 1981;3(2):
299-309.
36. Altamura M, Caradonna L, Amati L, Pellegrino NM, Urgesi G,
Miniello S. Splenectomy and sepsis: The role of the spleen in the
immune-mediated bacterial clearance. Immunopharmacol Immunotoxicol. 2001;23(2):153-161.
37. Sumaraju V, Smith LG, Smith SM. Infectious complications in
asplenic hosts. Infect Dis Clin North Am. 2001;15(2):551-565:x.
38. Okabayashi T, Hanazaki K. Overwhelming postsplenectomy
infection syndrome in adults a clinically preventable disease.
World J Gastroenterol. 2008;14(2):176-179.
39. Hansen K, Singer DB. Asplenic-Hyposplenic overwhelming sepsis: Postsplenectomy sepsis revisited. Pediatr Dev Pathol. 2001;
4(2):105-121.
40. Leonard AS, Giebink GS, Baesl TJ, Krivit W. The overwhelming postsplenectomy sepsis problem. World J Surg. 1980;4(4):
423-432.
41. Brigden ML, Pattullo AL. Prevention and management of overwhelming postsplenectomy infectionan update. Crit Care
Med. 1999;27(4):836-842.
42. Ejstrud P, Kristensen B, Hansen JB, Madsen KM, Schnheyder
HC, Srensen HT. Risk and patterns of bacteraemia after splenectomy: A population-based study. Scand J Infect Dis. 2000;
32(5):521-525.
43. Melles DC, de Marie S. Prevention of infections in hyposplenic
and asplenic patients: An update. Netherlands J Med. 2004;
62(2):45-52.
44. Williams DN, Kaur B. Postsplenectomy care. Strategies to
decrease the risk of infection. Postgrad Med. 1996;100(1):195-198,
201, 205.
45. Kullberg BJ, Westendorp RG, vant Wout JW, Meinders AE. Purpura fulminans and symmetrical peripheral gangrene caused
by capnocytophaga canimorsus (formerly DF-2) septicemiaa
complication of dog bite. Medicine. 1991;70(5):287-292.

PMPH_CH62.indd 506

46. Sawmiller CJ, Dudrick SJ, Hamzi M. Postsplenectomy capnocytophaga canimorsus sepsis presenting as an acute abdomen.
Arch Surg. 1998;133(12):1362-1365.
47. Findling JW, Pohlmann GP, Rose HD. Fulminant gram-negative
bacillemia (DF-2) following a dog bite in an asplenic woman. Am
J Med. 1980;68(1):154-156.
48. Panagopoulos MI, Saint Jean M, Brun D, et al. Bordetella holmesii bacteremia in asplenic children: Report of four cases initially misidentified as Acinetobacter lwoffii. J Clin Microbiol.
2010;48(10):3762-3764.
49. Weyant RS, Hollis DG, Weaver RE, et al. Bordetella holmesii sp.
nov., a new gram-negative species associated with septicemia.
J Clin Microbiol. 1995;33(1):1-7.
50. Shepard CW, Daneshvar MI, Kaiser RM, et al. Bordetella holmesii
bacteremia: A newly recognized clinical entity among asplenic
patients. Clin Infect Dis. 2004;38(6):799-804.
51. Rowin KS, Tanowitz HB, Rubinstein A, Kunkel M, Wittner M.
Babesiosis in asplenic hosts. Trans Roy Soc Trop Med Hyg. 1984;
78(4):442-444.
52. Browne S, Ryan Y, Goodyer M, Gilligan O. Fatal babesiosis in an
asplenic patient. Br J Haematol. 2010;148(4):494.
53. Rosner F, Zarrabi MH, Benach JL, Habicht GS. Babesiosis in
splenectomized adults: Review of 22 reported cases. Am J Med.
1984;76(4):696-701.
54. Dammin GJ, Spielman A, Benach JL, Piesman J. The rising incidence of clinical babesia microti infection. Human Pathol. 1981;
12(5):398-400.
55. Benach JL, Habicht GS. Clinical characteristics of human babesiosis. J Infect Dis. 1981;144(5):481.
56. Engwerda CR, Beattie L, Amante FH. The importance of the
spleen in malaria. Trends Parasitol. 2005;21(2):75-80.
57. Looareesuwan S, Suntharasamai P, Webster HK, Ho M. Malaria
in splenectomized patients: Report of four cases and review. Clin
Infect Dis. 1993;16(3):361-366.
58. West KW, Grosfeld JL. Postsplenectomy sepsis: Historical background and current concepts. World J Surg. 1985;9(3):477-483.
59. Locker GJ, Wagner A, Peter A, et al. Lethal WaterhouseFriderichsen syndrome in posttraumatic asplenia. J Trauma. 1995;
39(4):784-786.
60. Rizzo M, Magro G, Castaldo P. OPSI (overwhelming postsplenectomy infection) syndrome: A case report. Forens Sci Int. 2004;
146(Suppl):S55-S56.
61. Fogarty AB, Small JO, Colville J. Postsplenectomy pneumococcal septicaemia resulting in gangrene of the extremities. Ulster
Med J. 1991;60(1):117-119.
62. Taylor MD, Genuit T, Napolitano LM. Overwhelming postsplenectomy sepsis and trauma: Time to consider revaccination?
J Trauma. 2005;59(6):1482.
63. Wald K, Martinez A, Moll S. Capnocytophaga canimorsus
infection with fulminant sepsis in an asplenic patient: Diagnosis by review of peripheral blood smear. Am J Hematol. 2008;
83(11):879.
64. Pers C, Gahrn-Hansen B, Frederiksen W. Capnocytophaga canimorsus septicemia in Denmark, 1982-1995: Review of 39 cases.
Clin Infect Dis. 1996;23(1):71-75.
65. Deprs-Brummer P, Buijs J, van Engelenburg KC, Oosten HR.
Capnocytophaga canimorsus sepsis presenting as an acute
abdomen in an asplenic patient. Netherlands J Med. 2001;59(5):
213-217.
66. Gaston MH, Verter JI, Woods G, et al. Prophylaxis with oral penicillin in children with sickle cell anemia. A randomized trial.
N Engl J Med. 1986;314(25):1593-1599.

5/22/2012 5:43:52 PM

Postsplenectomy Sepsis

67. Hirst C, Owusu-Ofori S. Cochrane review: Prophylactic antibiotics for preventing pneumococcal infection in children
with sickle cell disease. Evid Based Child Health. 2007;2(3):
993-1009.
68. El-Alfy MS, El-Sayed MH. Overwhelming postsplenectomy
infection: Is quality of patient knowledge enough for prevention?
Hematol J. 2004;5(1):77-80.
69. Falletta JM, Woods GM, Verter JI, Buchanan GR, Pegelow CH,
Iyer RV, et al. Discontinuing penicillin prophylaxis in children
with sickle cell anemia. Prophylactic penicillin study II. J Pediatr.
1995;127(5):685-690.
70. Lortan JE. Management of asplenic patients. Br J Haematol. 1993;
84(4):566-569.
71. Konradsen HB, Henrichsen J. Pneumococcal infections in splenectomized children are preventable. Acta Paediatr Scand. 1991;
80(4):423-427.
72. American Academy of Pediatrics. Committee on Infectious Diseases. Policy Statement: Recommendations for the Prevention of
Pneumococcal Infections, Including the Use of Pneumococcal Conjugate Vaccine (Prevnar), Pneumococcal Polysaccharide Vaccine,
and Antibiotic Prophylaxis. Pediatrics. 2000;106(2 Pt 1):362-366.
73. Castagnola E, Fioredda F. Prevention of life-threatening infections due to encapsulated bacteria in children with hyposplenia
or asplenia: A brief review of current recommendations for practical purposes. European J Hematol. 2003;71(5):319-326.
74. Newland A. Preventing severe infection after splenectomy. Br
Med J. 2005;331(7514):417-418.
75. McMullin M, Johnston G. Long term management of patients
after splenectomy. Br Med J. 1993;307(6916):1372.
76. Barnes JN. Long term management after splenectomy. A national
problem. Br Med J. 1994;308(6824):338.
77. de Montalembert M, Lenoir G. Antibiotic prevention of pneumococcal infections in asplenic hosts: Admission of insufficiency.
Ann Hematol. 2004;83(1):18-21.
78. Spelman D, Buttery J, Daley A, et al. Guidelines for the prevention of sepsis in asplenic and hyposplenic patients. Intern Med J.
2008;38(5):349-356.
79. Kyaw MH, Holmes EM, Chalmers J, Jones IG, Campbell H. A
survey of vaccine coverage and antibiotic prophylaxis in splenectomised patients in Scotland. J Clin Pathol. 2002;55(6):472-474.
80. Makris M, Greaves M, Winfield DA, Preston FE, Lilleyman JS.
Long-Term management after splenectomy. Lifelong penicillin
unproved in trials. Br Med J. 1994;308(6921):131.
81. Finch RG. Long term management after splenectomy . . . and
may be ineffective. Br Med J. 1994;308(6921):132.
82. Klinge J, Hammersen G, Scharf J, Ltticken R, Reinert RR. Overwhelming postsplenectomy infection with vaccine-type streptococcus pneumoniae in a 12-year-old girl despite vaccination and
antibiotic prophylaxis. Infection. 1997;25(6):368-371.
83. Zarrabi MH, Rosner F. Rarity of failure of penicillin prophylaxis
to prevent postsplenectomy sepsis. Arch Int Med. 1986;146(6):
1207-1208.
84. Shetty N. Failure of pneumococcal vaccine and prophylactic penicillin in a splenectomized patient. J Infect. 1998;1:87-88.
85. Brivet F. Fatal post-splenectomy pneumococcal sepsis despite
pneumococcal vaccine and penicillin prophylaxis. Lancet. 1984;
2(8398):356-357.
86. Teach SJ, Lillis KA, Grossi M. Compliance with penicillin prophylaxis in patients with sickle cell disease. Arch Pediatr Adolesc
Med. 1998;152(3):274-278.
87. Bitares EL, Oliveira BM, Viana MB. Compliance with antibiotic
prophylaxis in children with sickle cell anemia: A prospective
study. J Pediatr. 2008;84(4):316-322.

PMPH_CH62.indd 507

507

88. Dellinger RP, Levy MM, Carlet JM, et al. Surviving sepsis
campaign: International guidelines for management of severe
sepsis and septic shock: 2008. Crit Care Med. 2008;36(1):296-327.
89. Nuorti JP, Whitney CG; Centers for Disease Control and
Prevention (CDC). Prevention of pneumococcal disease among
infants and children - use of 13-valent pneumococcal conjugate
vaccine and 23-valent pneumococcal polysaccharide vaccine recommendations of the advisory committee on immunization
practices (ACIP). MMWR Recomm Rep. 2010;59(RR-11):1-18.
90. Advisory Committee on Immunization Practices. Updated
recommendations for prevention of invasive pneumococcal
disease among adults using the 23-valent pneumococcal
polysaccharide vaccine (PPSV23). Morb Mort Wkly Rep. 2010;
59(34):1102-1106.
91. Nuorti JP, Whitney CG; Centers for Disease Control and
Prevention (CDC). Prevention of pneumococcal disease among
infants and children use of 13-valent pneumococcal conjugate
vaccine and 23-valent pneumococcal polysaccharide vaccine recommendations of the advisory committee on immunization
practices (ACIP). MMWR. Recommendations and Reports:
Morbidity and Mortality Weekly Report. Recommendations and
Reports/Centers for Disease Control. 2010;59(RR-11):1-18.
92. Centers for Disease Control and Prevention (CDC). Progress
toward elimination of haemophilus influenzae type b invasive
disease among infants and children United States, 1998-2000.
Morb Mort Wkly Rep. 2002;51(11):234-237.
93. Kroger AT, Atkinson WL, Marcuse EK, Pickering LK; Advisory
Committee on Immunization Practices (ACIP) Centers for
Disease Control and Prevention (CDC). General recom mendations on immunization: Recommendations of the advisory
committee on immunization practices (ACIP). MMWR Recomm
Rep. 2006;55(RR-15):1-48.
94. Meerveld-Eggink A, de Weerdt O, van Velzen-Blad H, Biesma
DH, Rijkers GT. Response to conjugate pneumococcal and
haemophilus influenzae type b vaccines in asplenic patients.
Vaccine. 2011;29(4):675-680.
95. Centers for Disease Control and Prevention (CDC). Updated
recommendations for use of meningococcal conjugate vaccines
advisory committee on immunization practices (ACIP), 2010.
Morb Mort Wkly Rep. 2011;60(3):72-76.
96. Davies JM, Barnes R, Milligan D; British Committee for
Standards in Haematology. Working Party of the Haematology/
Oncology Task Force Update of guidelines for the prevention
and treatment of infection in patients with an absent or
dysfunctional spleen. Clin Med. 2002;2(5):440-443.
97. Centers for Disease Control and Prevention (CDC). Bacterial
coinfections in lung tissue specimens from fatal cases of 2009
pandemic influenza A (H1N1) United States, may-august
2009. Morb Mort Wkly Rep. 2009;58(38):1071-1074.
98. Shatz DV. Vaccination considerations in the asplenic patient.
Expert Rev Vaccines. 2005;4(1):27-34.
99. Shatz DV, Schinsky MF, Pais LB, Romero-Steiner S, Kirton
OC, Carlone GM. Immune responses of splenectomized
trauma patients to the 23-valent pneumococcal polysaccharide
vaccine at 1 versus 7 versus 14 days after splenectomy. J Trauma.
1998;44(5):760-765; discussion 765-766.
100. Shatz DV, Romero-Steiner S, Elie CM, Holder PF, Carlone
GM. Antibody responses in postsplenectomy trauma patients
receiving the 23-valent pneumococcal polysaccharide vaccine
at 14 versus 28 days postoperatively. J Trauma. 2002;53(6):1037.
101. Konradsen HB, Rasmussen C, Ejstrud P, Hansen JB. Antibody
levels against streptococcus pneumoniae and haemophilus
influenzae type b in a population of splenectomized individuals

5/22/2012 5:43:52 PM

508

102.

103.

104.

105.

106.

107.

108.

109.

110.

111.
112.

113.

114.

115.

116.

117.

118.

119.

Surgery: Evidence-Based Practice

with varying vaccination status. Epidemiol Infect. 1997;


119(02):167-174.
Siber GR, Weitzman SA, Aisenberg AC. Antibody response of
patients with Hodgkins disease to protein and polysaccharide
antigens. Rev Infect Dis. 1981;3:144-159.
Giebink GS, Le CT, Cosio FG, Spika JS, Schiffman G. Serum
antibody responses of high-risk children and adults to
vaccination with capsular polysaccharides of streptococcus
pneumoniae. Rev Infect Dis. 1981;3 Suppl:S168-178.
Grimfors G, Sderqvist M, Holm G, Lefvert AK, Bjrkholm M.
A longitudinal study of class and subclass antibody response to
pneumococcal vaccination in splenectomized individuals with
special reference to patients with Hodgkins disease. European J
Hematol. 1990;45(2):101-108.
Landgren O, Bjrkholm M, Konradsen HB, et al. A prospective
study on antibody response to repeated vaccinations with
pneumococcal capsular polysaccharide in splenectomized
individuals with special reference to Hodgkins lymphoma.
J Int Med. 2004;255(6):664-673.
Rutherford EJ, Livengood J, Higginbotham M, et al. Efficacy
and safety of pneumococcal revaccination after splenectomy
for trauma. J Trauma. 1995;39(3):448-452.
Moberley SA, Holden J, Tatham DP, Andrews RM. Vaccines
for preventing pneumococcal infection in adults. Cochrane
Database of Systematic Reviews (Online). 2008;1.
Butler JC, Breiman RF, Campbell JF, Lipman HB, Broome CV,
Facklam RR. Pneumococcal polysaccharide vaccine efficacy: An
evaluation of current recommendations. JAMA. 1993;270(15):1826.
Jugenburg M, Haddock G, Freedman MH, Ford-Jones L, Ein
SH. The morbidity and mortality of pediatric splenectomy:
Does prophylaxis make a difference? J Pediatr Surg. 1999;34(7):
1064-1067.
Chaikof EL, Goodson JD, McCabe CJ. Postsplenectomy pneumococcemia in a healthy vaccinated adult. Am J Emerg Med. 1984;
2(2):141-143.
Gonzaga RA. Fatal post-splenectomy pneumococcal sepsis
despite prophylaxis. Lancet. 1984;2(8404):694.
Zarrabi MH, Rosner F. Pneumococcal sepsis and meningitis in
vaccinated subjects: A review of 55 reported cases. J Natl Med
Assoc. 1987;79(4):372-376.
Waghorn DJ. Overwhelming infection in asplenic patients:
Current best practice preventive measures are not being
followed. J Clin Pathol. 2001;54(3):214.
Brigden ML, Pattullo A, Brown G. Pneumococcal vaccine
administration associated with splenectomy: The need for
improved education, documentation, and the use of a practical
checklist. Am J Hematol. 2000;65(1):25-29.
Meerveld-Eggink A, de Weerdt O, Rijkers GT, van VelzenBlad H, Biesma DH. Vaccination coverage and awareness of
infectious risks in patients with an absent or dysfunctional
spleen in the Netherlands. Vaccine. 2008;26(52):6975-6979.
Kind EA, Craft C, Fowles JB, McCoy CE. Pneumococcal
vaccine administration associated with splenectomy: Missed
opportunities. Am J Infect Control. 1998;26(4):418-422.
Kinnersley P, Wilkinson CE, Srinivasan J. Pneumococcal
vaccination after splenectomy: Survey of hospital and primary
care records. Br Med J. 1993;307(6916):1398.
Bruni L, Bayas JM, Vilella A, Conesa A. Vaccination coverage
in adults undergoing splenectomy: Evaluation of hospital
vaccination policies. Epidemiol Infect. 2006;134(4):837-844.
Shatz DV. Vaccination practices among North American trauma
surgeons in splenectomy for trauma. J Trauma. 2002;53(5):
950-956.

PMPH_CH62.indd 508

120. Lammers AJ, Veninga D, Lombarts MJ, Hoekstra JB, Speelman


P. Management of post-splenectomy patients in the Netherlands.
European J Clin Microbiol Infect Dis. 2010;29(4):399-405.
121. Hegarty PK, Tan B, OSullivan R, Cronin CC, Brady MP.
Prevention of postsplenectomy sepsis: How much do patients
know? Hematol J. 2000;1(5):357-359.
122. Kotsanas D, Al-Souffi MH, Waxman BP, King RW, Polkinghorne
KR, Woolley IJ. Adherence to guidelines for prevention of
postsplenectomy sepsis. Age and sex are risk factors: A five-year
retrospective review. ANZ J Surg. 2006;76(7):542-547.
123. Pearson HA, Johnston D, Smith KA, Touloukian RJ. The bornagain spleen. Return of splenic function after splenectomy for
trauma. N Engl J Med. 1978;298(25):1389.
124. Zoli G, Corazza GR, DAmato G, Bartoli R, Baldoni F, Gasbarrini G. Splenic autotransplantation after splenectomy: Tuftsin
activity correlates with residual splenic function. Br J Surg.
1994;81(5):716-718.
125. Hathaway JM, Harley RA, Self S, Schiffman G, Virella G. Immunological function in post-traumatic splenosis. Clin Immunol
Immunopathol. 1995;74(2):143-150.
126. Van Wyck DB. Overwhelming postsplenectomy infection
(OPSI): The clinical syndrome. Lymphology. 1983;16(2):107-114.
127. Rice HM, James PD. Ectopic splenic tissue failed to prevent
fatal pneumococcal septicaemia after splenectomy for trauma.
Lancet. 1980;1(8168 Pt 1):565-566.
128. Reay DT, Nakonechny D. Sudden death and sepsis after
splenectomy. J Forens Sci. 1979;24(4):757-761.
129. Sass W, Bergholz M, Kehl A, Seifert J, Hamelmann H.
Overwhelming infection after splenectomy in spite of some
spleen remaining and splenosis. A case report. Klin Wochenschr.
1983;61(21):1075-1079.
130. Connell NT, Brunner AM, Kerr CA, Schiff man FJ. Splenosis
and sepsis: The born-again spleen provides poor protection.
Virulence. 2011;2(1):4-11.
131. Livingston CD, Levine BA, Sirinek KR. Preservation of splenic
tissue prevents postsplenectomy pulmonary sepsis following
bacterial challenge. J Surg Res. 1982;33(4):356-361.
132. Iinuma H, Okinaga K, Sato S, Tomioka M, Matsumoto K. Optimal
site and amount of splenic tissue for autotransplantation. J Surg
Res. 1992;53(2):109-116.
133. Patel JM, Williams JS, Naim JO, Hinshaw JR. The effect of site
and technique of splenic tissue reimplantation on pneumococcal
clearance from the blood. J Pediatr Surg. 1986;21(10):877-880.
134. Willfhr KU, Westermann J, Pabst R. Splenic autotransplantation provides protection against fatal sepsis in young but not
in old rats. J Pediatr Surg. 1992;27(9):1207-1212.
135. Steely WM, Satava RM, Brigham RA, Setser ER, Davies RS. Splenic
autotransplantation: Determination of the optimum amount
required for maximum survival. J Surg Res. 1988;45(3):327-332.
136. Pabst R, Kamran D. Autotransplantation of splenic tissue. J
Pediatr Surg. 1986;21(2):120-124.
137. Traub A, Giebink S, Smith C, et al. Splenic reticuloendothelial
function after splenectomy, spleen repair, and spleen autotransplantation. N Engl J Med. 1987;317(25):1559-1564.
138. Leemans R, Manson W, Snijder JA, et al. Immune response
capacity after human splenic autotransplantation: Restoration
of response to individual pneumococcal vaccine subtypes. Ann
Surg. 1999;229(2):279-285.
139. Leemans R, Harms G, Rijkers GT, Timens W. Spleen autotransplantation provides restoration of functional splenic
lymphoid compartments and improves the humoral immune
response to pneumococcal polysaccharide vaccine. Clin Exp
Immunol. 1999;117(3):596-604.

5/22/2012 5:43:52 PM

Postsplenectomy Sepsis

140. Smith E, J De Young N, Drew PA. Decreased phagocytic capacity


of autotransplanted splenic tissue. ANZ J Surg. 2003;73(11):
894-896.
141. Tang WH, Wu FL, Huang MK, Friess H. Splenic tissue autotransplantation in rabbits: No restoration of host defense.
Langenbecks Archives of Surgery/Deutsche Gesellschaft Fr
Chirurgie. 2003;387(9-10):379-385.
142. Tesluk GC, Thomas CG, Benjamin JT, McMillan CW. Fatal
overwhelming postsplenectomy sepsis following autologous
splenic transplantation in severe congenital osteopetrosis.
J Pediatr Surg. 1984;19(3):269-272.
143. Moore GE, Stevens RE, Moore EE, Aragon GE. Failure of splenic
implants to protect against fatal postsplenectomy infection. Am
J Surg. 1983;146(3):413-414.
144. Corazza GR, Tarozzi C, Vaira D, Frisoni M, Gasbarrini G.
Return of splenic function after splenectomy: How much tissue
is needed? Br Med J. 1984;289(6449):861.
145. Moore FA, Moore EE, Moore GE, Millikan JS. Risk of splenic
salvage after trauma: Analysis of 200 adults. Am J Surg. 1984;
148(6):800-805.
146. Pisters PW, Pachter HL. Autologous splenic transplantation for
splenic trauma. Ann Surg. 1994;219(3):225.
147. Okinaga K, Giebink GS, Rich RH, Baesl TJ, Krishnanaik D,
Leonard AS. The effect of partial splenectomy on experimental
pneumococcal bacteremia in an animal model. J Pediatr Surg.
1981;16(5):717-724.
148. Haque AU, Hudson P, Wood G, Lindsey NJ. Splenic autotransplant and residual partial spleen: Prevention of septicemia.
Jpn J Surg. 1984;14(5):407-412.
149. Rice HE, Oldham KT, Hillery CA, Skinner MA, OHara SM,
Ware RE. Clinical and hematologic benefits of partial splen-

PMPH_CH62.indd 509

150.

151.

152.

153.

154.

155.

156.

157.

509

ectomy for congenital hemolytic anemias in children. Ann Surg.


2003;237(2):281-288.
Sheikha AK, Salih ZT, Kasnazan KH, et al. Prevention of
overwhelming postsplenectomy infection in thalassemia
patients by partial rather than total splenectomy. Can J Surg.
2007;50(5):382-386.
Resende V, Petroianu A. Functions of the splenic remnant after
subtotal splenectomy for treatment of severe splenic injuries.
Am J Surg. 2003;185(4):311-315.
Bessoud B, Duchosal MA, Siegrist CA, et al. Proximal splenic artery
embolization for blunt splenic injury: Clinical, immunologic, and
ultrasound-Doppler follow-up. J Trauma. 2007;62(6):1481-1486.
Tominaga GT, Simon FJ, Dandan IS, et al. Immunologic function
after splenic embolization, is there a difference? J Trauma.
2009;67(2):289-295.
Falimirski M, Syed A, Prybilla D. Immunocompetence of the
severely injured spleen verified by differential interference
contrast microscopy: The red blood cell pit test. J Trauma.
2007;63(5):1087-1091; discussion 1091-1092.
Walusimbi MS, Dominguez KM, Tran EP, et al. Immunologic
integrity after splenic embolization for trauma (abstract).
J Trauma. 2007;67:471.
Malhotra AK, Carter RF, Lebman DA, et al. Preservation of splenic
immunocompetence after splenic artery angioembolization for
blunt splenic injury. J Trauma. 2010;69(5):1126-1130; discussion
1130-1131.
Peitzman AB, Harbrecht BG, Rivera L, Heil B; Eastern Association for the Surgery of Trauma Multiinstitutional Trials
Workgroup. Failure of observation of blunt splenic injury in
adults: Variability in practice and adverse consequences. J Am
Coll Surg. 2005;201(2):179-187.

5/22/2012 5:43:52 PM

PMPH_CH62.indd 510

5/22/2012 5:43:52 PM

PART X

HERNIA

PMPH_CH63.indd 511

5/22/2012 5:44:23 PM

PMPH_CH63.indd 512

5/22/2012 5:44:24 PM

CHAPTER 63

Inguinal Hernias
George Kasotakis and Marc A. de Moya

Fitzgibbons et al.4 followed prospectively 720 men with reducible, minimally symptomatic or asymptomatic inguinal hernias
after randomizing them to watchful waiting versus a Lichtenstein
tension-free repair with a prosthetic mesh. Primary outcome measures included worsening pain that interfered with daily activities at
2 years and a change in quality of life metrics. Both groups developed
pain interfering with activities in equal proportions (5.1% for the
watchful waiting group vs. 2.2% for the surgically treated, p = .52)
in the intention-to-treat analysis. However, 23% of watchful waiting
patients had crossed over to the intervention group by 2 years, citing
worsening symptomatology, whereas 17% of the men assigned to
hernia repair crossed over to watchful waiting. Acute hernia accidents (bowel obstruction without strangulation) were very rare at
a cumulative accident rate of 0.0018 events per patient year, and the
patients that presented with those were managed successfully with
urgent or semielective repairs without significant complications.
Contrary to popular belief, there appears to be no penalty
for delaying operation in the men with minimally symptomatic
or asymptomatic hernias. A follow-up study on the same cohort
of patients assessed a range of objective measures after grouping
patients in an immediate (<6 months) or delayed (>6 months)
repair group.6 Operative time (64 vs. 67 min, p = .382), complication (17% vs. 21.5%, p = .375), and recurrence rates (1% vs. 3.1%),
as well as patient satisfaction scores were similar between the
two groups. Watchful waiting also appears to be a cost-effective
approach in managing minimally symptomatic patients with hernias7 that does not overburden patients caregivers.8
The second trial took place in the United Kingdom and
included 160 males over the age of 55 years with asymptomatic inguinal hernias.5 The primary outcome was pain at 1 year
and was similar between the two groups. Twenty-three of the
80 observation patients crossed over to repair due to pain or
increase in size. Hernia-related adverse events occurred only in
three of those, and those were successfully managed with either
an urgent or an elective herniorrhaphy after reduction. When
the same cohort of watchful waiting patients were followed for a
period of 8 years, more than half (46 of 80, 57.5%) elected to have
their hernias repaired, with most citing pain or increase in size as

INTRODUCTION
Inguinal hernia repair represents the most commonly performed
procedure by General Surgeons in the United States with more
than 770,000 repairs performed annually according to the National
Center for Health Statistics.1 These figures have significant socioeconomic ramifications, as both the condition and the operation
are associated with significant costs, morbidity, and afflict caregivers with a nontrivial burden. Until recently, a belief commonly held
among surgical training programs worldwide purported that all
inguinal hernias be repaired at diagnosis. The reasoning behind
this principle was twofold: early intervention helped prevent complicating events with unacceptably high morbidity and mortality and allowed a less technically challenging operation later on.
However, a growing body of evidence suggests that the incidence
of long-term complications after herniorrhaphy might be higher
than previously thought, whereas little is known about the natural history of hernias in men who elect to not have an operation.
Other controversial issues surrounding inguinal hernias include
the routine use of mesh and neurectomies to prevent recurrences
and postoperative groin pain, respectively; the role of laparoscopy
in unilateral, bilateral, and recurrent hernias; optimal anesthesia
selection for elective herniorrhaphies; as well as factors predisposing recurrence are among the topics discussed in this chapter.
1. Should asymptomatic hernias be repaired?
Although the question of whether to intervene in a patient with a
symptomatic hernia is easily answered, defining whether asymptomatic or minimally symptomatic patients warrant herniorrhaphy
is much more difficult to tackle. The difficulty in this undertaking
lies in estimating the incidence of potentially life-threatening hernia accidents, which appears to be lower than initially thought.2,3
Two prospective-randomized controlled clinical trials have
been published in the past few years testing the hypothesis that a
strategy of watchful waiting is an acceptable alternative to routine
repair at diagnosis of an asymptomatic or minimally symptomatic hernia in men.4,5
513

PMPH_CH63.indd 513

5/22/2012 5:44:24 PM

514

Surgery: Evidence-Based Practice

an indication.9 Over the course of the 8 years, only two patients


presented with acute hernia accidents, but neither required bowel
resection.
Given the aforementioned data, it appears that watchful waiting can be safely offered as an option in asymptomatic or minimally symptomatic men with inguinal hernias. This is a Grade A
recommendation.
2. Are mesh repairs better than native tissue repairs?
Since the late 19th century, when Bassini introduced the musculoaponeurotic approximation for inguinal hernias,10 the primary
tissue repair dominated the general surgical circles with multiple variations (Shouldice, McVay) for over a century. However, it
was unacceptably high recurrence rates in the 10% to 15% that
prompted surgeons to look for new approaches. The concept of
tension-free repair that was first introduced by Lichtenstein in
the 1980s11 attracted attention early on, as the addition of a mesh
allowed reconstruction without the need to pull layers of tissue
under tension. This technically easier approach continued to gain
ground and multiple prospective-randomized trials demonstrated
its advantages over the more traditional native tissue approaches,
including a significant reduction in recurrence, the ability to perform under local anesthesia, less pain and more rapid return to
work, and routine physical activities.
In 2001, the Cochrane Collaboration identified 20 prospective-randomized or quasi-randomized clinical trials comparing
open mesh with nonmesh repairs.12 Of these, 17 used a flat mesh,
two plug-and-mesh, and one a mesh placed preperitoneally. The
control groups in all studies included a variety of primary tissue
repairs. Despite the marked heterogeneity of the studies included,
it appeared that tension-free approaches required on average 7 to
10 min less to perform than Shouldice repairs, but 1 to 4 min longer than the Bassini or McVay approach. There was no statistically
significant difference in minor postoperative complications, such
as hematoma and seroma formation, or wound infections. Serious
complications, including femoral neurovascular bundle, spermatic
cord, and visceral injuries were rare in both groups. Hospital stay
was slightly shorter in the mesh-based repairs (OR 0.28, 95%
CI 0.35 to 0.22), but significant heterogeneity was noted, likely
reflecting variability in the local discharge practices. Return to
usual activities/work was also shorter in the mesh-repair group
(OR 0.81, 95% CI 0.730.91), and persisting pain or numbness in
the genitofemoral area also favored the tension-free approach (OR
0.68, 95% CI 0.470.98 and OR 0.7, 95% CI 0.291.72, respectively).
Most importantly, recurrence after mesh repair was consistently
less frequently reported and overall was reduced by between 50%
and 75% (OR 0.37, 95% CI 0.260.51).
Subsequent prospective trials reported similar findings, favoring mesh-based repairs over primary herniorrhaphies.13-15 Given
the aforementioned data, we conclude that the use of mesh during
open inguinal hernia repair is associated with a significant reduction in the recurrence risk and may act favorably in reducing postoperative groin pain and numbness and allowing earlier return to
preoperative functioning status. We therefore recommend routine
application of mesh-based tension-free inguinal herniorrhaphy.
This is a Grade A recommendation.
3. Open or laparoscopic repair for inguinal hernias?
With the improved outcomes and subsequent popularization of
the Lichtenstein herniorrhaphy in the 1980s and the explosion of

PMPH_CH63.indd 514

laparoscopic surgery in the 1990s, minimally invasive attempts


at tension-free repairs yielded encouraging results. The dominating techniques for laparoscopic inguinal hernia repair include the
transabdominal preperitoneal (TAPP) and the totally extraperitoneal (TEP) approaches.
In an attempt to answer what patient population might benefit the most from a laparoscopic approach, multiple prospective
clinical trials have been conducted. The EU Hernia Trialists Collaboration meta-analysis16 included 41 prospective-randomized
trials, comparing laparoscopic with open tension-free herniorrhaphies. This meta-analysis, which included a total of 7294 patients,
demonstrated no significant difference in the recurrence rates
between the laparoscopic and the open approach (2.2% vs. 1.7%;
OR 1.26, 95% CI 0.762.08), but showed a benefit in chronic groin
pain in the former (OR 0.64, 95% CI, 0.520.78, p < .001).
Another meta-analysis,17 which included 29 prospectiverandomized trials comprising a total of 5588 patients, concluded
that short-term hernia recurrence was higher in the laparoscopically managed by about 50%, although this result was not statistically significant (OR 1.51, 95% CI, 0.812.79). Postoperative
complications were fewer in the laparoscopic group (OR 0.62,
95% CI, 0.460.84), discharge from the hospital occurred earlier (3.43 h, 95% CI, 0.356.5 h) and patients returned sooner to
normal activity by about 5 days (95% CI 3.515.96). The downside was a slightly longer operative time (15.2 min longer; 95% CI
7.7822.63 min).
The Veterans Affairs cooperative trial,18 one of the most commonly cited trials in the United States, included 1983 patients
that were randomized to a tension-free open versus a laparoscopic repair and had some contradictory findings: recurrence at
2 years was lower in the open group (10.1% vs. 4.9%; OR 2.2; 95%
CI, 1.53.2), but when surgeon experience was taken into account
(>250 laparoscopic hernia repairs) the recurrence was low and
below 5% with either approach. Complication rate was slightly
higher in the laparoscopic group (39% vs. 33.4%; OR 1.3, 95% CI,
1.11.3). The laparoscopically treated patients reported less groin
pain at 2 weeks postoperatively and were able to return to their
usual activity 1 day earlier than the open group.
A more recent meta-analysis from Dedemadi and associates19 that comprised 1542 patients undergoing laparoscopic
versus Lichtenstein repair reported similar postoperative complications and recurrence risk, but noted more recurrences when
the TAPP group was compared with the TEP (RR 3.25, 95%
CI 1.327.9, p = .01).
With regard to the optimal approach for managing recurrent
hernias, the Danish Hernia Database analysis,20 which included
a total of 67,306 prospectively recorded herniorrhaphies, demonstrated a reduced re-reoperation rate if a laparoscopic approach
was used for the first recurrence (1.3%, 95% CI 0.43.0) compared
with Lichtenstein (11.3%, 95% CI 8.215.2). Another randomized trial comparing laparoscopic with open repairs for recurrent
inguinal hernias21 demonstrated no difference in operative time
or recurrence rates at 5 years (18% for TAPP vs. 19% for Lichtenstein), but postoperative pain and time to return to work were less
with the former.
In summary, laparoscopic repairs offer an equivalent recurrence risk and a slightly earlier return to normal activity compared with open repairs, at the expense of longer operative times,
a greater equipment cost, and the need for general anesthesia.
Laparoscopic hernia repairs may offer an advantage for managing
recurrent hernias. This is a Grade A recommendation.

5/22/2012 5:44:24 PM

Inguinal Hernias

4. Risk factors for hernia recurrence.


Inguinal hernia recurrence is estimated at <5% with either a Lichtenstein or a laparoscopic repair when performed by experienced
surgeons. Using a mesh to re-approximate the tissues in a tensionfree fashion is one of the best known methods employed to significantly reduce recurrence; however, other risk factors for hernia
recurrence are not clearly delineated.
Mayagoitia et al.22 looked at 551 open hernia repairs performed with either a flat mesh (Licthenstein), a Prolene Hernia
System, or a Mesh-Plug and concluded that recurrence was greater
with the latter (2.5%, RR 4.35, 95% CI 0.8522.23); yet the difference was not statistically significant. Previous herniorrhaphy, an
internal ring >4.5 cm, and postoperative complications were also
found to be predictive of future recurrence.
Data from the Veterans Affairs trial23 demonstrated that independent predictors of recurrence in the open repair group were
recurrent hernia, lack of a caregiver, and operating time <72 min.
Among the patients treated laparoscopically, low surgeon volume,
active lifestyle, and a BMI <25 were the independent predictors.
While the aforementioned data provide an idea for what
might contribute to a hernia recurrence, larger studies designed
to detect risk factors for recurrence should be performed. This is a
Grade C recommendation.
5. Should neurectomy for prevention of postoperative groin
pain be done routinely?
Inguinodynia or chronic groin pain is one of the most dreaded
complications following inguinal hernia repair and one that
affects the quality of life significantly.24 It is usually attributed to
intraoperative nerve damage or postoperative mesh-related fibrosis. Although traditional surgical teaching holds that the nerves
(ilioinguinal, iliohypogastric, and genitofemoral) should be identified and preserved during repair, recent cohort studies demonstrate that routine ilioinguinal nerve sacrifice is associated with
less chronic groin pain, whereas subjective paresthesia is usually
only temporary.25-27 In addition, ilioinguinal neurectomy appears
to be the effective treatment for chronic groin pain relief after open
herniorrhaphy.28,29 However, results of prospective-randomized
clinical trials comparing the preservation versus routine ilioinguinal neurectomy during open tension-free herniorrhaphies are
conflicting.30-33 In light of the contradictory evidence and while a
Cochrane review is underway, preservation or routine resection of
the inguinal nerves should be left to the discretion of the treating
surgeon. This is a Grade D recommendation.
6. What is the optimal approach for bilateral hernias: open or
laparoscopic?
Concurrent repair of bilateral hernias may best be accomplished
laparoscopically. Long-term data demonstrate no difference in
recurrence between bilateral open compared with bilateral laparoscopic repairs34 and it appears that overall recurrence (0.63%
vs. 0.42%), postoperative complications (1.9% vs. 1.4%), need for
reoperation (0.5% vs. 0.43%), and time to return to previous activity (14 vs. 14 days) are the same for unilateral versus bilateral laparoscopic hernia repair.35
Postoperative complications, operative time, length of stay,
and groin pain appear to favor the laparoscopic approach compared with a Licthenstein repair in three prospective-randomized
trials.36-38 With these data in mind, and given the easier concurrent

PMPH_CH63.indd 515

515

access to both groins with the laparoscopic approach, we recommend the TAPP or the TEP for bilateral inguinal hernias if the
operating surgeon is expert in laparoscopic repairs, otherwise the
open repair is acceptable.
7. Is a plug necessary for mesh repairs?
Due to the rising popularity of tension-free herniorrhaphies over
the last few decades, plug-and-patch prostheses, which consist
of a plug covering the abdominal wall defect and a flat mesh
reinforcing the inguinal canal floor, were developed. The unique
feature of these repairs is that they require minimal dissection,
theoretically allowing for shorter operating times, reducing postoperative pain, and affording earlier recovery. However, these
claims were not confirmed in rigorous clinical trials.
Dalenbck et al.39 randomized 472 men undergoing tension-free
herniorrhaphy to a Lichtenstein, Prolene Hernia System, or plugand-patch repair and followed them for 3 years. Although operative time was slightly shorter in the latter two groups (40.4 1 min
vs. 37.4 1 min, and 35.5 1 min, respectively), the difference was
not clinically significant. Postoperative complications, groin pain,
return to full functional ability, and incidence of recurrence did not
differ between the groups. Similarly, Nienhuijs et al.40 randomized
334 patients to the same procedures, and assessed the quality of life
and pain with the SF-36 and the visual analog scale at 2, 12, and
60 weeks after surgery. There were no differences noted in either.
When the plug-and-patch repair was compared against the
Lichtenstein herniorrhaphy 595 patients undergoing a total of 700
primary or recurrent herniorrhaphies by Frey et al., no differences
were noted in recurrence rates or postoperative complications.41
Similarly, postoperative pain and time to recovery did not differ in
141 individuals studied by Kingsworth and colleagues.42 Operative
time was significantly shorter in the plug-and-patch group (32 vs.
37.6 min, p = .01), but the difference was not clinically significant.
Based on the above findings, mesh type selection should be left
to the surgeons discretion. This is a Grade B recommendation.
8. Local versus general anesthesia: does one confer a better outcome than the other?
There have been several case series that have described the feasibility and safety of performing inguinal hernia repairs utilizing local
anesthesia.43,44 Other larger database retrospective series have also
suggested that local anesthesia is underutilized as a method for
inguinal hernia repair.45 This has been extended to include laparoscopic hernia repairs,46 as well as open. We focus our question
on the use of local anesthesia versus general anesthesia in open
hernia repairs.
In 2001, Gonullu et al.47 performed a randomized clinical
trial directly comparing the use of local anesthesia with the use of
general anesthesia with a primary outcome of pulmonary effects,
postoperative pain and fatigue, morbidity, and patient satisfaction. They demonstrated a significant difference in pain relief but
only at one time point (8 h postop) but significantly improved
CO2 clearance and oxygenation in the local anesthesia group.
There was no significant difference in patient satisfaction. They
concluded that local anesthesia provided slightly better pain control and improve pulmonary function.
In 2003, ODwyer et al. performed a randomized trial comparing local and general anesthesia on 279 patients with ultimately 138 in each group. They found that intraoperative pain led
to patient dissatisfaction but postoperative pain was better at 6 h

5/22/2012 5:44:24 PM

516

Surgery: Evidence-Based Practice

than the general anesthesia group. In addition, they noted that


open repair using general anesthesia was 4% more in cost than
local anesthesia. They concluded that there were no major differences in patient recovery after local or general anesthesia and
patients could be presented with both options.48
Th is was followed by a randomized clinical trial comparing local, regional, and general anesthesia in Sweden. There were
approximately 200 patients in each arm of this multi-institutional
trial. The investigators seemed to standardize methods across the
institutions with favorable outcomes for those undergoing local
anesthesia. There was a significant decrease in admission duration, less immediate postoperative pain, and fewer problems with
urinary retention. However, there was no difference in pain after
the first day and no difference in time to normal activity. This
study suggested that local anesthesia should be utilized more frequently for open inguinal hernia repairs.49
There is currently a meta-analysis being performed by the
Cochrane Collaboration to evaluate the randomized trials concerning this question. However, after review of the evidence, it
appears as though the use of local anesthesia is safe and effective and in some patients may be a better alternative to general
anesthesia. However, there is no compelling data that suggest that
local anesthesia is superior to general anesthesia. This is a Grade B
recommendation.

REFERENCES
1. Rutkow IM, Robbins AW. Demographic, classificatory, and socioeconomic aspects of hernia repair in the United States. Surg Clin
North Am. 1993;73:413-426.
2. Hair A, Paterson C, Wright D, Baxter JN, ODwyer PJ. What
effect does the duration of an inguinal hernia have on patient
symptoms? J Am Coll Surg. 2001;193:125-129.
3. Gallegos NC, Dawson J, Jarvis M, Hobsley M. Risk of strangulation in groin hernias. Br J Surg. 1991;78:1171-1173.
4. Fitzgibbons RJ, Jr., Giobbie-Hurder A, Gibbs JO, et al. Watchful
waiting vs repair of inguinal hernia in minimally symptomatic
men: a randomized clinical trial. JAMA. 2006;295:285-292.
5. ODwyer PJ, Norrie J, Alani A, Walker A, Duff y F, Horgan P. Observation or operation for patients with an asymptomatic inguinal
hernia: A randomized clinical trial. Ann Surg. 2006;244:167-173.
6. Thompson JS, Gibbs JO, Reda DJ, et al. Does delaying repair of an
asymptomatic hernia have a penalty? Am J Surg. 2008;195:89-93.
7. Stroupe KT, Manheim LM, Luo P, et al. Tension-free repair versus watchful waiting for men with asymptomatic or minimally
symptomatic inguinal hernias: A cost-effectiveness analysis. J
Am Coll Surg. 2006;203:458-468.
8. Gibbs JO, Giobbie-Hurder A, Edelman P, McCarthy M, Jr., Fitzgibbons RJ, Jr. Does delay of hernia repair in minimally symptomatic men burden the patients family? J Am Coll Surg. 2007;
205:409-412.
9. Chung L, Norrie J, ODwyer PJ. Long-term follow-up of patients
with a painless inguinal hernia from a randomized clinical trial.
Br J Surg. 2011;98(4):596-599.
10. Bassini E. Nuovo metodo sulla cura radicale dell ernia inguinale. Arch Soc Ital Chir. 1887;4:380.
11. Lichtenstein IL, Shulman AG, Amid PK, Montllor MM. The tension-free hernioplasty. Am J Surg. 1989;157:188-193.
12. Scott N, Graham P, McCormack K, Orss S, Grant A. Open mesh
versus non-mesh for groin hernia repair Review. Cochrane
Database Syst Rev. 2002;(4):CD002197.

PMPH_CH63.indd 516

13. Miedema BW, Ibrahim SM, Davis BD, Koivunen DG. A prospective trial of primary inguinal hernia repair by surgical trainees.
Hernia. 2004;8:28-32.
14. Koninger J, Redecke J, Butters M. Chronic pain after hernia
repair: A randomized trial comparing Shouldice, Lichtenstein
and TAPP. Langenbecks Arch Surg 2004;389:361-365.
15. Nordin P, Bartelmess P, Jansson C, Svensson C, Edlund G. Randomized trial of Lichtenstein versus Shouldice hernia repair in
general surgical practice. Br J Surg. 2002;89:45-49.
16. McCormack K, Scott NW, Go PM, Ross S, Grant AM. Laparoscopic techniques versus open techniques for inguinal hernia
repair. Cochrane Database Systematic Reviews. 2003:CD001785.
17. Memon MA, Cooper NJ, Memon B, Memon MI, Abrams KR.
Meta-analysis of randomized clinical trials comparing open
and laparoscopic inguinal hernia repair. Br J Surg. 2003;90:
1479-1492.
18. Neumayer L, Giobbie-Hurder A, Jonasson O, et al. Open mesh
versus laparoscopic mesh repair of inguinal hernia. N Engl J Med.
2004;350:1819-1827.
19. Dedemadi G, Sgourakis G, Radtke A, et al. Laparoscopic versus
open mesh repair for recurrent inguinal hernia: A meta-analysis
of outcomes. Am J Surg. 2010;200:291-297.
20. Bisgaard T, Bay-Nielsen M, Kehlet H. Re-recurrence after operation for recurrent inguinal hernia. A nationwide 8-year follow-up
study on the role of type of repair. Ann Surg. 2008;247:707-711.
21. Eklund A, Rudberg C, Leijonmarck CE, et al. Recurrent inguinal
hernia: Randomized multicenter trial comparing laparoscopic
and Lichtenstein repair. Surg Endosc. 2007;21:634-640.
22. Mayagoitia JC, Prieto-Diaz Chavez E, Suarez D, Cisneros HA, Tene
CE. Predictive factors comparison of complications and recurrences in three tension-free herniorraphy techniques. Hernia.
2006;10:147-151.
23. Matthews RD, Anthony T, Kim LT, et al. Factors associated with
postoperative complications and hernia recurrence for patients
undergoing inguinal hernia repair: A report from the VA Cooperative Hernia Study Group. Am J Surg. 2007;194:611-617.
24. van Hanswijck de Jonge P, Lloyd A, Horsfall L, Tan R, ODwyer
PJ. The measurement of chronic pain and health-related quality
of life following inguinal hernia repair: A review of the literature. Hernia. 2008;12:561-569.
25. Zieren J, Tsigris C, Menenakos C. Open tension-free hernia repair
in soccer players: Preservation or primary neurectomy of the ilioinguinal nerve? Clin J Sport Med. 2007;17:398-400.
26. Dittrick GW, Ridl K, Kuhn JA, McCarty TM. Routine ilioinguinal nerve excision in inguinal hernia repairs. Am J Surg. 2004;
188:736-740.
27. Tsakayannis DE, Kiriakopoulos AC, Linos DA. Elective neurectomy during open, tension free inguinal hernia repair. Hernia.
2004;8:67-69.
28. Loos MJ, Scheltinga MR, Roumen RM. Tailored neurectomy
for treatment of postherniorrhaphy inguinal neuralgia. Surgery.
2010;147:275-281.
29. Aasvang EK, Kehlet H. The effect of mesh removal and selective
neurectomy on persistent postherniotomy pain. Ann Surg. 2009;
249:327-334.
30. Malekpour F, Mirhashemi SH, Hajinasrolah E, Salehi N, Khoshkar A, Kolahi AA. Ilioinguinal nerve excision in open mesh
repair of inguinal herniaresults of a randomized clinical trial:
Simple solution for a difficult problem? Am J Surg. 2008;195:
735-740.
31. Mui WL, Ng CS, Fung TM, et al. Prophylactic ilioinguinal neurectomy in open inguinal hernia repair: A double-blind randomized
controlled trial. Ann Surg. 2006;244:27-33.

5/22/2012 5:44:24 PM

Inguinal Hernias

32. Picchio M, Palimento D, Attanasio U, Matarazzo PF, Bambini


C, Caliendo A. Randomized controlled trial of preservation or
elective division of ilioinguinal nerve on open inguinal hernia
repair with polypropylene mesh. Arch Surg. 2004;139:755-758;
discussion 759.
33. Ravichandran D, Kalambe BG, Pain JA. Pilot randomized controlled study of preservation or division of ilioinguinal nerve
in open mesh repair of inguinal hernia. Br J Surg. 2000;87:
1166-1167.
34. Kald A, Fridsten S, Nordin P, Nilsson E. Outcome of repair of
bilateral groin hernias: A prospective evaluation of 1,487 patients.
Eur J Surg. 2002;168:150-153.
35. Wauschkuhn CA, Schwarz J, Boekeler U, Bittner R. Laparoscopic
inguinal hernia repair: Gold standard in bilateral hernia repair?
Results of more than 2800 patients in comparison to literature.
Surg Endosc. 2010;24:3026-3030.
36. Feliu X, Claveria R, Besora P, et al. Bilateral inguinal hernia
repair: Laparoscopic or open approach? Hernia. 2011;15:15-18.
37. Sarli L, Iusco DR, Sansebastiano G, Costi R. Simultaneous repair
of bilateral inguinal hernias: A prospective, randomized study of
open, tension-free versus laparoscopic approach. Surg Laparosc
Endosc Percutan Tech. 2001;11:262-267.
38. Mahon D, Decadt B, Rhodes M. Prospective randomized trial
of laparoscopic (transabdominal preperitoneal) vs open (mesh)
repair for bilateral and recurrent inguinal hernia. Surg Endosc.
2003;17:1386-1390.
39. Dalenback J, Andersson C, Anesten B, et al. Prolene Hernia System,
Lichtenstein mesh and plug-and-patch for primary inguinal hernia repair: 3-year outcome of a prospective randomised controlled
trial. The BOOP study: Bi-layer and connector, on-lay, and onlay with plug for inguinal hernia repair. Hernia. 2009;13:121-129;
discussion 231.

PMPH_CH63.indd 517

517

40. Nienhuijs S, van Oort I, Keeners-Gels M, Strobbe L, Rosman C.


Randomized clinical trial comparing the Prolene Hernia System,
mesh plug repair and Lichtenstein method for open inguinal
hernia repair. Br JSurg. 2005;92:33-38.
41. Frey DM, Wildisen A, Hamel CT, Zuber M, Oertli D, Metzger
J. Randomized clinical trial of Lichtensteins operation versus
mesh plug for inguinal hernia repair. Br J Surg. 2007;94:36-41.
42. Kingsnorth AN, Porter CS, Bennett DH, Walker AJ, Hyland
ME, Sodergren S. Lichtenstein patch or Perfi x plug-and-patch
in inguinal hernia: A prospective double-blind randomized controlled trial of short-term outcome. Surgery. 2000;127:276-283.
43. Callesen T, Bech K, Kehlet H. Feasibility of local infi ltration
anaesthesia for recurrent groin hernia repair. Eur J Surg. 2001;
167:851-854.
44. Makuria T, Alexander-Williams J, Keighley MRB. Comparison
between general and local anaesthesia for repair of groin hernias.
Ann Roy Coll Surg Engl. 1979;61:291-294.
45. Kehlet H, Nielsen M. Anaesthetic practice for groin hernia repair a
nation-wide study in Denmark 1998-2003. Acta Anaesthesiol
Scand. 2005;49:143-146.
46. Frezza E, Ferzli G. Local and general anesthesia in the laparoscopic preperitoneal hernia repair. J Soc Lap Surg. 2000;4:
221-224.
47. Gonullu NN, Cubukcu A, Alponat A. Comparison of local and
general anesthesia in tension-free (Lichtenstein) hernioplasty: A
prospective randomized trial. Hernia. 2002;6:29-32.
48. ODwyer PJ, Serpell MG, Millar K, et al. Local or general anesthesia for open hernia repair: A randomized trial. Ann Surg. 2003;
237:574-579.
49. Nordin P, Zetterstrom H, Gunnarsson U, Nilsson E. Local,
regional, or general anaesthesia in groin hernia repair: Multicentre randomised trial. Lancet. 2003;362:853-858.

5/22/2012 5:44:24 PM

CHAPTER 64

Recurrent Inguinal Hernia


H. R. Nanda Kumar and Kent R. Van Sickle

INTRODUCTION

of Lichtenstein tension-free repair as the gold standard for open


anterior approaches. Lichtenstein tension-free repair is the most
commonly used prosthetic repair by open technique.1
There are limited prospective, randomized data comparing the recurrence rate of open prosthetic repairs versus open
nonprosthetic (i.e., primary tissue) repairs. A review of 26,000
inguinal hernia repairs from Denmark found that mesh repairs
had a lower reoperation rate than conventional open repairs.3
Meta-analysis studies have also demonstrated that mesh repair
was associated with fewer overall recurrences with 1.4% in mesh
group and 4.4% in nonmesh group9,10 and a reduction in the risk
of recurrence by 5075%.

Approximately, 770,000 inguinal hernia repairs are performed


annually in the United States.1 An analysis in Sweden noted that 15%
of all inguinal hernia repairs were performed to treat recurrence.2
Although recurrence has been regarded as the most important measure of success of an inguinal hernia repair, newer measures also
focus on the quality of life and return to normal activities. Recurrent
inguinal hernia repairs are technically more difficult than primary
repairs because of inflammation and scarring from prior repair.
1. What is the rate of recurrence for inguinal hernia?

Primary Repair
Laparoscopic versus Open Repairs

The rate of recurrence following primary repair of an inguinal hernia has been variable and is reported between <1% and 17%.2-4 This is
attributable to the multiple techniques of repair involving open versus
laparoscopy, tissue versus prosthetic repairs, and so on. Recurrence
after primary inguinal hernia repair is higher in females compared
with males,2,3 mainly due to occurrence of femoral hernias.
Recurrence rates of tissue-based repairs vary according to procedure. The Shouldice repair has been regarded as the most superior
by large-scale analyses. Surgeons who perform a large volume of the
Shouldice repair are able to demonstrate recurrence rates around
1%.5 In less experienced hands, such low recurrence rates have
not been demonstrated. However, recurrence rates for the Shouldice repair are consistently lower than those of the Bassini or
McVay repair. A multicenter controlled trail of 1578 patients from
France demonstrated that the Shouldice repair, even with a recurrence rate near 6%, is superior to the Bassini repair (8.6% recurrence
rate) and McVay repair (11.2%).6
The introduction of prosthetic repairs reduced recurrence
rates to a consistently low level. In a multi-institutional series, 3019
inguinal hernias were repaired using Lichtenstein tension-free
repair with a reported recurrence rate of 0.2%.7 In another series
of 3175 inguinal hernias repaired by Lichtenstein tension-free
repair, the recurrence rate was 0.5% with a follow-up period of up
to 5 years.8 Consistently low recurrence rates led to the acceptance

In the early 1990s, the application of laparoscopic surgery to the


repair of inguinal hernias gained interest. Evidence available to
date has not been definitive while comparing laparoscopic with
open methods of repair. A meta-analysis of 41 randomized trials in 2000 found no significant difference in the risk of recurrence between the two approaches.11 In 2004, the results of the
VA cooperative study, a multicenter randomized trial that analyzed long-term hernia results in over 2000 patients in 14 Veterans
Affairs hospitals, were published. A total of 1983 patients underwent either a laparoscopic (TAPP or TEP) or an open (Lichtenstein tension-free) operation. It showed a significantly increased
recurrence rate of primary unilateral hernias at 2 years within the
laparoscopic group versus the open group (10.1% vs. 4.0%). However, a posthoc evaluation of the surgeons self-reported experience compared with recurrence rates demonstrated a statistically
significant reduced recurrence rate (5.1%) for those that had performed >250 laparoscopic procedures. It has been suggested that
the recurrence rates noted in this study may be more reflective of
the general population outside of specialty centers. In the hands
of very experienced laparoscopic surgeons, with more than 250
cases, the recurrence rates may approach that of open technique.
However, other studies including a systematic review published in
2000 have shown equivalent recurrence rates in both groups.12-18
518

PMPH_CH64.indd 518

5/22/2012 5:44:59 PM

Recurrent Inguinal Hernia

Recurrent Inguinal Hernia Repair


The recurrence rate for repair of a recurrent inguinal hernia varies
depending on the method of primary hernia repair and method of
the recurrent hernia repair. Bisgaard et al. reported in 2008 that the
re-recurrence rate after a primary open hernia repair ranged from
1% to 2% and 7% to 17% with a laparoscopic technique and open
repair for recurrence, respectively.4 In other randomized trials
including a total of 337 recurrent groin hernias, the re-recurrence
rate ranged from 8% to 19% and 2% to 18% in patients undergoing laparoscopic and open repair, respectively.22-24 The follow-up
ranged from 2.5 to 5 years but again no information on the technique of previous repair was provided. Two of the trials23,24 showed
no significant differences whereas one trial22 reported significantly
lower re-recurrence rate after open repair (Stoppa) compared with
a laparoscopic repair (2% vs. 13%).
Answer: Operations using mesh result in fewer recurrences
than nonmesh techniques. (Grade A) The recurrence rate following
Lichtenstein repair of primary inguinal hernia is around 4%. There
is a significantly higher recurrence rate with laparoscopic repair of
inguinal hernias. Highly experienced surgeons may have a recurrence
rate which approaches that seen with the open repair. (Grade B)
2. What is the pattern of recurrence?
Studies suggest that most hernia recurrences are medial22,24 and
around 10% of them are femoral.4 There is a higher incidence of
recurrence in the form of a femoral hernia in women. In a very large
Swedish hernia registry, where around 7000 women with inguinal
hernias were followed prospectively, 42% of women with a recurrent groin hernia were noted to have a femoral hernia at the time of
reoperation. This number was less than 5% in men. It is important
that surgeons are aware of the incidence of femoral hernia, especially in women and make appropriate considerations while planning the operation for recurrent inguinal hernias.25
Answer: Women have a higher incidence of recurrence following the repair of inguinal hernia due to occurrence of femoral
hernias. (Grade C)
3. Is there a role for nonoperative management of recurrent
inguinal hernias?
Recurrent inguinal hernias on presentation are most often asymptomatic or minimally symptomatic. The traditional teaching has
been to repair these hernias considering the risk of incarceration
or strangulation. However, the risk of incarceration of an asymptomatic inguinal hernia is low (estimated to be 0.33% per year).26
An emergency operation due to a strangulated inguinal hernia
has a higher associated mortality than an elective operation (>5 vs.
<0.5%).3,27 A large randomized multicenter clinical trial published
in 2006 compared patients with primary or recurrent inguinal hernias with a minimum follow-up of 2 years.28 They were randomized
to watchful waiting versus Lichtenstein repair. The rate of hernia accident (defined as a strangulation or bowel obstruction) for
all patients (both primary and recurrent) was calculated at 0.0018
events/patient/year in that study. The main conclusions after 2 years
of follow-up were 23% crossover rate from watchful waiting to operation, one acute incarceration without strangulation within 2 years,
and one incarceration with bowel obstruction within 4 years. There
were no significant differences between the two groups in pain interfering with activities or performance on the physical component of
the SF-36. There were no consequences for delaying operation, and

PMPH_CH64.indd 519

519

outcomes were the same when compared with those randomized


to receive immediate operation. In another trial,29 in which 80 men
(over 55 years of age) were randomized to operation and 80 to watchful waiting, the main conclusions after 1 year of follow-up were 29%
of patients crossed over to operation and three serious hernia-related
adverse events occurred in the watchful waiting. The results demonstrated no difference in outcomes or costs between groups and
concluded that surgery can be safely delayed and watchful waiting is
an acceptable option for men who are asymptomatic.
Answer: Watchful waiting is an acceptable option for men
with minimally symptomatic or asymptomatic hernias. (Grade A)
4. What is the preferred method of repair for recurrent inguinal hernias?
Recurrence following a repair of an inguinal hernia can occur
early within days of or late after a few years after the initial repair.
It is advisable to delay the repair of a reducible recurrent inguinal
hernia for at least 6 weeks after the primary repair whenever feasible to allow for the inflammatory process to subside.
Recurrent inguinal hernias can be repaired via an open or
a laparoscopic approach. The method of the initial hernia repair
should be given due consideration before deciding the type of
repair for the recurrent inguinal hernia. The operative report
should be obtained and any other intervening procedures like retropubic prostatectomy and pelvic fi xation should also be considered. The goal of repair of a recurrent inguinal hernia should be
to identify the failure in coverage of the myopectineal orifice from
the initial repair, and attempt to provide a definitive coverage.

Open Approach for a Recurrent Inguinal Hernia


The open approach for repair of a recurrent inguinal hernia is
currently based on prosthetic repairs. The conventional nonprosthetic repairs are out of favor for recurrent inguinal hernias.2
If the initial repair was a prosthetic repair, then the repair
of the recurrent hernia should be approached via a virgin plane.
An anterior approach should be used following a failed posterior
repair and a posterior approach should be used following a failed
anterior repair.30
The choice of the procedure will also depend on the individual
experience of the surgeon. The most common open anterior mesh
procedure is the Lichtenstein repair. Orchiectomy should be discussed with the patient although it is rarely necessary. Attempts to
remove the mesh are discouraged but the anatomy should be carefully delineated to include the femoral canal and repair performed
adhering to the principles as discussed earlier. When a femoral
hernia is encountered, the mesh should be anchored to Coopers
ligament. This technique has also been modified by the addition
of a mesh plug into the hernia defect after reduction of the hernia
sac, prior to placement of the mesh onlay.
Multiple trials have demonstrated the effectiveness of open
anterior mesh repair for recurrent inguinal hernias. Open preperitoneal repair can be performed by posterior approach. It
requires the placement of a mesh posterior to transversalis fascia.
This approach is less popular than the other repairs, but some
studies have reported low recurrence and complication rates.22
Many recent randomized controlled trails have shown
advantages with laparoscopic approach for the repair of recurrent
inguinal hernias and are more commonly used when a posterior
approach is being considered.

5/22/2012 5:45:00 PM

520

Surgery: Evidence-Based Practice

Laparoscopic Approach for a


Recurrent Inguinal Hernia
Laparoscopic repair offers the benefit of visualizing the myopectineal orifice and thereby all the potential defects. The two
most common types of laparoscopic repair for an inguinal hernia
are the transabdominal preperitoneal repair (TAPP) and totally
extraperitoneal repair (TEP).
There are no multicenter randomized controlled trails comparing TEP and TAPP repairs. Most studies show equivalent
results in terms of outcomes (pain, recovery, complications, and
recurrence). Some groups favor the TEP repair, as it does not
violate the peritoneum and a reported lower incidence of serious
complications.11,31,32

Open versus Laparoscopic Repair


The debate of open versus laparoscopic repair for recurrent inguinal hernias has been addressed by randomized controlled trials22-24
over the last few years. The majority of these studies have excluded
patients with prior mesh repair. They have shown equivalent results
in terms of recurrence when mesh repair has been used in the open
approach. The laparoscopic approach has certain advantages mainly
with reduced complications like seroma, early postoperative pain,
analgesic consumption, and return to activities.2,4,21-24,33,34
Some of the relative contraindications for laparoscopic repair
are similar to that of a primary inguinal hernia. A large scrotal
hernia and patients with ascites are also better served with an
open repair.

Answer: A recurrent inguinal hernia after conventional open


repair should be repaired by an open or a laparoscopic preperitoneal approach. (Grade A). The preperitoneal approach (open or
laparoscopic) is recommended for the repair of recurrent inguinal
hernia following previous open Lichtenstein repair. (Grade B). An
open anterior approach is recommended for a recurrent inguinal
hernia after previous posterior approach. (Grade D)
5. What is the incidence of chronic pain following repair of
primary and recurrent inguinal hernias?
Chronic postoperative groin pain syndromes have emerged as the
major complication being faced by hernia surgeons today. Systematic reviews of prospective studies of inguinal hernia repair
report the incidence of moderate to severe chronic pain to be
around 1012%.35-37 A study of 226 patients found that there was
a fourfold increase in the risk of developing chronic pain for an
operation of recurrent inguinal hernia as compared with a primary inguinal hernia.37,38 Also, a Danish prospective study of 419
patients noted a higher risk of moderate or severe pain after repair
of recurrent than primary hernias (14% vs. 3%).39 There are no randomized controlled trials comparing the incidence chronic pain
following open versus laparoscopic repairs of recurrent inguinal
hernias. However, studies have reported higher incidence of pain
with open as compared with laparoscopic repair of initial inguinal hernias.35,36,40
Answer: The overall incidence of moderate to severe pain
after hernia surgery is around 1012%. There is a fourfold increase
in the rate of moderate to severe chronic pain following surgery
for recurrent inguinal hernia. (Grade B)

Randomized controlled trials comparing laparoscopic versus open repairs


Study

Year

Intervention

No. of Pts

Follow-up

Recurrence

Comments

Eklund et al.19

2009

TEP vs. TFR

1365

5 years

TEP: 3.5% *
TFR: 1.2%

PIH only
When one surgeon with a
33% recurrence rate in the
TEP group was excluded,
the cumulative recurrence
rate dropped to 2.4% in the
TEP group (not significant
statistically)

Hallen et al.12

2008

TEP vs. TFR

80: TEP
87: TFR

1 year

TEP: 4.3%
TFR: 5.1 %

PIH & RIH

Lau et al.13

2006

TEP vs. TFR

100: TEP
100: TFR

1 year

None in both
groups

PIH only

Arvidsson et al.20

2005

TAPP vs.
Shouldice

1183

5 years

TAPP: 6.6%
Shouldice: 6.7%

PIH only

Neumayer et al.21

2004

TFR vs. TEP/


TAPP

756: TFR
781: TEP
and
TAPP

2 years

TFR: 4.9%*
TEP/TAPP: 10.1%

PIH (4%, 10.1%)


RIH (14.1%, 10%)
Difference in recurrence rate
between TEP and TAPP not
reported. Recurrence rate
in lap group was 5.1 % in
experienced surgeon group
(self-reported experience of
>250 repairs)
(Continued)

PMPH_CH64.indd 520

5/22/2012 5:45:00 PM

Recurrent Inguinal Hernia

521

(Continued)
Study

Year

Intervention

No. of Pts

Follow-up

Recurrence

Comments

Liem et al.14

2003

TEP vs.
Conventional
open anterior
repair

487: TEP
507: open

2 and
4 years

2 years
TEP: 3.8
Open: 6.3
4 years
TEP: 4.9*
Open: 10

PIH and RIH


Open Bassini and Non-Bassini
(Including Mesh)

Douek et al.15

2003

TAPP vs. TFR

TAPP: 122
TFR: 120

5 years

TAPP: 1.6%
TFR: 2.5%

PIH and RIH

Colak et al.16

2003

TEP vs. TFR

TEP: 67
TFR: 67

1 year

TEP: 2.9%
TFR: 5.9%

PIH and RIH

Bringman et al.17

2003

TEP vs. Mesh


plug vs. TFR

TEP: 92
M-P: 104
TFR: 103

19.8 8.6
months

TEP: 2.2
M-P: 1.9
TFR: 0

PIH and RIH

Andersson et al.18

2003

TEP vs. TFR

TEP: 76
TFR: 85

1 year

TEP: 2.6%
TFR: 0

PIH and RIH

TAPP, Transabdominal Preperitoneal Patch; TEP, Totally Extraperitoneal Patch; TFR, Tension-Free Repair; PIH, Primary Inguinal Hernia;
RIH, Recurrent Inguinal Hernia.

Summary Table
Question

Recommendation

1 What is the rate of recurrence


for repair of an inguinal hernia?

Operations using mesh result in fewer recurrences than nonmesh techniques.


The recurrence rate following Lichtenstein repair of primary inguinal hernia is around
4%. There is a significantly higher recurrence rate with laparoscopic repair of
inguinal hernias. Highly experienced surgeons may have a recurrence rate similar to
that seen with open repair.

A
B

2 What is the pattern of


recurrence?

Women have a higher incidence of recurrence following repair of inguinal hernia due
to occurrence of femoral hernias.

3 Is there a role for nonoperative management of


recurrent inguinal hernias?

Watchful waiting is an acceptable option for men with minimally symptomatic or


asymptomatic hernias.

4 What is the preferred method


of repair for a recurrent
inguinal hernia?

A recurrent inguinal hernia after conventional open repair should be repaired by an


open or laparoscopic preperitoneal approach.
The preperitoneal approach is recommended for repair of recurrent inguinal hernia
following previous open Lichtenstein repair.
An open anterior approach is recommended for a recurrent inguinal hernia after
previous posterior approach.

5 What is the incidence of


chronic pain following repair of
primary or recurrent inguinal
hernia?

The overall incidence of moderate to severe pain after hernia surgery is around
1012%.
There is a fourfold increase in rate of moderate to severe chronic pain following
surgery for recurrent inguinal hernia.

REFERENCES
1. Rutkow IM. Demographic and socioeconomic aspects of hernia
repair in the United States in 2003. Surg Clin North Am. 2003;
83(5):1045-1051, v-vi.
2. Haapaniemi S, Gunnarsson U, Nordin P, Nilsson E. Reoperation after recurrent groin hernia repair. Ann Surg. 2001;234(1):
122-126.
3. Bay-Nielsen M, Kehlet H, Strand L, et al. Quality assessment of
26,304 herniorrhaphies in Denmark: A prospective nationwide
study. Lancet. 2001;358(9288):1124-1128.

PMPH_CH64.indd 521

Grade

B
D
B
C

4. Bisgaard T, Bay-Nielsen M, Kehlet H. Re-recurrence after operation for recurrent inguinal hernia. A nationwide 8-year follow-up
study on the role of type of repair. Ann Surg. 2008;247(4):707-711.
5. Glassow F. The Shouldice Hospital technique. Int Surg. 1986;
71(3):148-153.
6. Hay JM, Boudet MJ, Fingerhut A, et al. Shouldice inguinal hernia
repair in the male adult: The gold standard? A multicenter controlled trial in 1578 patients. Ann Surg. 1995;222(6):719-727.
7. Shulman AG, Amid PK, Lichtenstein IL. The safety of mesh repair
for primary inguinal hernias: Results of 3,019 operations from five
diverse surgical sources. Am Surg. 1992;58(4):255-257.

5/22/2012 5:45:00 PM

522

Surgery: Evidence-Based Practice

8. Kark AE, Kurzer MN, Belsham PA. Three thousand one hundred seventy-five primary inguinal hernia repairs: Advantages of
ambulatory open mesh repair using local anesthesia. J Am Coll
Surg. 1998;186:447-455; discussion 456.
9. Scott NW, McCormack K, Graham P, Go PM, Ross SJ, Grant AM.
Open mesh versus non-mesh for repair of femoral and inguinal hernia. Cochrane Database Systematic Reviews. 2002(4):CD002197.
10. Grant A. Mesh compared with non-mesh methods of open
groin hernia repair: Systematic review of randomized controlled trials. Br J Surg. 2000;87(7):854-859.
11. Grant A. Laparoscopic compared with open methods of groin
hernia repair: Systematic review of randomized controlled trials.
Br J Surg. 2000;87(7):860-867.
12. Hallen M, Bergenfelz A, Westerdahl J. Laparoscopic extraperitoneal inguinal hernia repair versus open mesh repair: Longterm follow-up of a randomized controlled trial. Surgery. 2008;
143(3):313-317.
13. Lau H, Patil NG, Yuen WK. Day-case endoscopic totally extraperitoneal inguinal hernioplasty versus open Lichtenstein hernioplasty
for unilateral primary inguinal hernia in males: A randomized
trial. Surg Endosc. 2006;20:76-81.
14. Liem MS, van Duyn EB, van der Graaf Y, van Vroonhoven TJ. Recurrences after conventional anterior and laparoscopic inguinal hernia
repair: A randomized comparison. Ann Surg. 2003;237(1):136-141.
15. Douek M, Smith G, Oshowo A, Stoker DL, Wellwood JM. Prospective randomised controlled trial of laparoscopic versus open
inguinal hernia mesh repair: Five year follow up. Br Med J. 2003;
326(7397):1012-1013.
16. Colak T, Akca T, Kanik A, Aydin S. Randomized clinical trial comparing laparoscopic totally extraperitoneal approach with open
mesh repair in inguinal hernia. Surg Laparosc Endosc Percutan
Tech. 2003;13(3):191-195.
17. Bringman S, Ramel S, Heikkinen TJ, Englund T, Westman B,
Anderberg B. Tension-free inguinal hernia repair: TEP versus
mesh-plug versus Lichtenstein: A prospective randomized controlled trial. Ann Surg. 2003;237(1):142-147.
18. Andersson B, Hallen M, Leveau P, Bergenfelz A, Westerdahl J.
Laparoscopic extraperitoneal inguinal hernia repair versus open
mesh repair: A prospective randomized controlled trial. Surgery.
2003;133(5):464-472.
19. Eklund AS, Montgomery AK, Rasmussen IC, Sandbue RP, Bergkvist
LA, Rudberg CR. Low recurrence rate after laparoscopic (TEP) and
open (Lichtenstein) inguinal hernia repair: A randomized, multicenter trial with 5-year follow-up. Ann Surg. 2009;249(1):33-38.
20. Arvidsson D, Berndsen FH, Larsson LG, et al. Randomized clinical trial comparing 5-year recurrence rate after laparoscopic versus Shouldice repair of primary inguinal hernia. Br J Surg. 2005;
92(9):1085-1091.
21. Neumayer L, Giobbie-Hurder A, Jonasson O, et al. Open mesh
versus laparoscopic mesh repair of inguinal hernia. N Engl J Med.
2004;350(18):1819-1827.
22. Beets GL, Dirksen CD, Go PM, Geisler FE, Baeten CG, Kootstra G.
Open or laparoscopic preperitoneal mesh repair for recurrent

PMPH_CH64.indd 522

23.

24.

25.

26.
27.

28.

29.

30.

31.

32.
33.
34.

35.
36.

37.
38.

39.
40.

inguinal hernia? A randomized controlled trial. Surg Endosc.


1999;13(4):323-327.
Dedemadi G, Sgourakis G, Karaliotas C, Christofides T, Kouraklis G. Comparison of laparoscopic and open tension-free repair
of recurrent inguinal hernias: A prospective randomized study.
Surg Endosc. 2006;20(7):1099-1104.
Eklund A, Rudberg C, Leijonmarck CE, et al. Recurrent inguinal
hernia: randomized multicenter trial comparing laparoscopic
and Lichtenstein repair. Surg Endosc. 2007;21(4):634-640.
Koch A, Edwards A, Haapaniemi S, Nordin P, Kald A. Prospective evaluation of 6895 groin hernia repairs in women. Br J Surg.
2005;92(12):1553-1558.
Gallegos NC, Dawson J, Jarvis M, Hobsley M. Risk of strangulation in groin hernias. Br J Surg. 1991;78(10):1171-1173.
Nilsson H, Stylianidis G, Haapamaki M, Nilsson E, Nordin P.
Mortality after groin hernia surgery. Ann Surg. 2007;245(4):
656-660.
Fitzgibbons RJ, Jr., Giobbie-Hurder A, Gibbs JO, et al. Watchful
waiting vs repair of inguinal hernia in minimally symptomatic
men: A randomized clinical trial. JAMA. 2006;295(3):285-292.
ODwyer PJ, Norrie J, Alani A, Walker A, Duff y F, Horgan P.
Observation or operation for patients with an asymptomatic
inguinal hernia: A randomized clinical trial. Ann Surg. 2006;
244(2):167-173.
Simons MP, Aufenacker T, Bay-Nielsen M, et al. European Hernia
Society guidelines on the treatment of inguinal hernia in adult
patients. Hernia. 2009;13(4):343-403.
Ramshaw BJ, Tucker JG, Mason EM, et al. A comparison of transabdominal preperitoneal (TAPP) and total extraperitoneal approach (TEPA) laparoscopic herniorrhaphies. Am Surg. 1995;61(3):
279-283.
Felix EL, Michas CA, Gonzalez MH, Jr. Laparoscopic hernioplasty. TAPP vs TEP. Surg Endosc. 1995;9(9):984-989.
Felix EL. A unified approach to recurrent laparoscopic hernia
repairs. Surg Endosc. 2001;15(9):969-971.
Kumar S, Nixon SJ, MacIntyre IM. Laparoscopic or Lichtenstein
repair for recurrent inguinal hernia: one units experience. J Roy
Coll Surg Edinb. 1999;44(5):301-302.
Aasvang E. Chronic postoperative pain: The case of inguinal
herniorrhaphy. Br J Anaesth. 2005;95(1):69-76.
Nienhuijs S, Staal E, Strobbe L, Rosman C, Groenewoud H, Bleichrodt R. Chronic pain after mesh repair of inguinal hernia: A
systematic review. Am J Surg. 2007;194(3):394-400.
Poobalan AS, Bruce J, Cairns W, et al. A review of chronic pain
after inguinal herniorrhaphy. Clin J Pain. 2003;19:48-54.
Poobalan AS, Bruce J, King PM, Chambers WA, Krukowski
ZH, Smith WC. Chronic pain and quality of life following open
inguinal hernia repair. Br J Surg. 2001;88(8):1122-1126.
Callesen T, Bech K, Kehlet H. Prospective study of chronic pain
after groin hernia repair. Br J Surg. 1999;86(12):1528-1531.
Eklund A, Montgomery A, Bergkvist L, Rudberg C. Chronic pain
5 years after randomized comparison of laparoscopic and Lichtenstein inguinal hernia repair. Br J Surg. 2010;97(4):600-608.

5/22/2012 5:45:00 PM

Commentary on
Recurrent Inguinal Hernia
Thomas E. Knuth

Considering that 24% of all males and 2% of females will develop


an inguinal hernia in their lifetime1 and considering that 750,000
inguinal hernia repairs are performed each year in the United
States, even a low recurrence rate, if a second operation is needed,
represents a significant medical and economic concern. The authors
in this chapter take a critical look at the literature related to the
two best measures of a successful hernia repairrecurrence rates
and chronic postoperative painand offer some best-evidencebased insight into one of the most commonly performed general
surgery procedures.
Unfortunately, despite a steady evolution in repair techniques and hernia repair products,2 not much has changed since
the father of modern inguinal hernia surgery, Edoardo Bassini
(18441924), recognized these same issues over 150 years ago.
Even then, he understood that there was something intrinsically
wrong with the tissue of patients who required repeated hernia
repairs. We now understand that connective tissue disorders,3
malnutrition, steroid use, chronic renal failure, and smoking4
contribute to recurrences. We are getting closer to understanding the DNA of poor protoplasm. Someday soon, we may have
the ability to regenerate a strong, healthy native tissue in an area
of injury or tissue breakdown and finally achieve the ideal hernia
repair. Meanwhile, there is still no agreement on the best way to
initially repair an inguinal hernia or to repair its recurrence after
repair. By addressing relevant questions and outlining the current
best-evidence-based data, this chapter is an essential reading for
every hernia surgeon.
The authors point out that the rate of recurrence following primary repair of an inguinal hernia varies depending on
technique. It is anywhere from 1% to 17% and slightly higher in
females who have more concomitant femoral hernias. In comparing tissue-based or tension repairs (Bassini, Shouldice, and McVay
repairs) to mesh or tension-free repairs (Lichtenstein repair), the
authors are clear: recurrence rates are lower with the use of mesh.
Th is fi nding has been generally accepted since the 1990s but
perhaps now the question can be put to rest. The continuing debate,
however, is which tension-free mesh technique is best. The Lichtenstein repair is the current gold standard against which others
are measured but the Plug and Patch repair popularized by
Rutkow5 and the Prolene Hernia System (PHS) repair popularized
by Gilbert6 deserve mention. Gilberts claim that his bilayer polypropylene mesh device (BPMD) reduces recurrence rates to onetenth that of other common repairs is significant. He claims that

the PHS has the practical advantage over other products, in that
it covers the entire myopectineal orifice and that the dual layers
above and below the abdominal wall with a connector traversing
the defect will not move. Because recurrence rates are known to be
so low with all of these mesh techniques, head-to-head randomized controlled trials of sufficient size may not be feasible. For the
time being, we may have to accept that any one of these techniques
is best in the personal experience of any given surgeon.
In comparing open mesh repairs to laparoscopic mesh repairs,
the authors find no significant difference in risk of recurrence. This
is surprising when considering the inherent benefit of preperitoneal reinforcement versus extraperitoneal. Several common sense
explanations are offered to explain laparoscopic recurrences
the mesh can fold during evacuation of insufflation, the mesh
may shrink or migrate, or there may be incomplete reduction of
a hernia sack. The authors point out that the overall laparoscopic
recurrence rate is significantly higher than that of open repairs
but, in the hands of surgeons who have performed more than 250
cases, recurrence rates approach those of open repairs. This of
course emphasizes errors in technique. The take-home message
may be alluded to in the brief mention of the Shouldice Clinic
experience. The fact that very low Shouldice recurrence rates have
not been duplicated elsewhere makes a point that has become thematic at the Harvard Business School as the concept of Focused
Factory whereby a best practice may be the practice of perfecting one, clearly defined process and pursuing it to the point or
excellence.7
Next, the authors address a pattern of inguinal hernia recurrence and state that most are medial, at the pubic tubercle.
Three related conditions are undoubtedly responsible for these
recurrences. First is the insufficient underlap in a laparoscopic
repair or an overlap in a Lichtenstein repairan error in technique. Second, as the mesh shrinks in the process of inflammation and scar formation, it can pull away from its anchora
problem with product. Th ird is the inherent deficiency in tissue
that contributed to hernia formation in the first placeproblem
with the patient. The fi nding that 10% of recurrent hernias are
femoral carries an important lesson although it would be helpful
to know how many of the 42% of femoral recurrences in women
and the 10% in men were actually missed concomitant hernias
or whether the fi rst repair somehow contributes to the development of the femoral hernia. Nevertheless, the important lesson is
that the entire myopectineal orifice, including the femoral canal,
523

PMPH_CH64.indd 523

5/22/2012 5:45:00 PM

524

Surgery: Evidence-Based Practice

should be covered in the initial repair. The Lichtenstein, Plug and


Patch, and Bassini repairs cover only medial and lateral inguinal
triangles but laparoscopic and open PHS techniques cover the
femoral triangle as well.
The question always arises as to whether a coincidentally
found recurrent hernia needs to be fi xed. The authors point out
that one-quarter to one-third of these patients needed an operation
within 12 years and that only 12 per thousand cases developed a
potentially catastrophic event requiring emergency surgery. Given
the exceedingly small risk of incarceration, the authors conclude
that watchful waiting is acceptable for asymptomatic patients. On
a more practical level, active patients who frequently run, jump,
or lift heavy weights may need to be repaired sooner while it may
be prudent to watch very sedentary patients and those at high
surgical risk due to comorbidities. The take-home message is that
recurrences probably do need to be fi xedbut selectively.
Finally, the authors discuss chronic pain after inguinal hernia repair, typically defined as lasting longer than 3 months after
surgery. The authors quote a 1012% incidence of chronic pain
after initial repair that rises fourfold after a recurrent repair.
While recurrences can be asymptomatic or relatively easily fi xed,
chronic pain can be debilitating and difficult to treat. Various factors including age, body habitus, preoperative pain, use of mesh,
and workmans compensation have been examined to predict
who will develop postoperative pain but numerous scoring algorithms have not been found to be reliable.8 Prevention remains
the key and this involves a meticulous technique to avoid inadvertent injury to the ilioinguinal, genitofemoral, and iliohypogastric
nerves in the open repair and the lateral femoral cutaneous nerve
in the laparoscopic repairs. Reports that some surgeons routinely
sacrifice the ilioinguinal nerve in open repairs and tuck the cut
ends in soft tissue away from the surgical field where it will not
become entrapped in scar tissue9 suggest that this is also the solution for a neuroma after inadvertent injury.10 This point may be
of particular importance in recurrent hernia surgery where dissection through distorted tissue planes increases the risk to small
nerves that are already trapped in scar tissue. Again, prevention is

PMPH_CH64.indd 524

the key and warrants consideration for approaching through the


undisturbed planesanterior for a previous laparoscopic repair,
and laparoscopic for a previous open anterior repair.
In conclusion, the best evidence is that recurrent inguinal hernias should be repaired with the benefit of mesh. The question of
which technique to use for recurrent inguinal hernia repair remains
unanswered although the authors point out that the Lichtenstein
and Plug and Patch techniques are the most popular. The conventional wisdom is that an approach through virgin, uninterrupted
planes is best for minimizing the risk of injury to cord structures
and nerves and thereby avoid chronic postoperative pain.

REFERENCES
1. Abramson JH, Golin J, Hopp C, et al. The epidemiology of inguinal hernia. A survey in western Jerusalem. J Epidemiol Community Health. 1991;27:300.
2. DeBord JR. The historical development of prosthetics in hernia
surgery. Surg Clin North Am. 1998;78(6):973-1006.
3. Uden A, Lindhagen T. Inguinal hernia in patients with congenital dislocation of the hip. A sign of general connective tissue disorder. Acta Orthop Scand. 1988;59:667.
4. Cannon DJ, Read RC. Metastatic emphysema: A mechanism for
acquiring inguinal herniation. Ann Surg. 1981;194:270.
5. Robbins AW, Rutkow IM. The mesh-plug hernioplasty. Surg Clin
North Am. 1993;73:501-512.
6. Gilbert AI, Young j, Graham MF, et al. Combined anterior and
posterior inguinal hernia repair: Intermediate recurrence rates
with three groups of surgeons. Hernia. 2004;8:203-207.
7. Shouldice Hernia Clinic website: www.shouldice.com
8. Dickinson KJ, Thomas M, Fawole AS, et al. Predicting chronic
post-operative pain following laparoscopic inguinal hernia
repair. Hernia. 2008;12:597-601.
9. Dittrick GW, Ridl K, Kuhn JA, et al. Routine ilioinguinal nerve
excision in inguinal hernia repairs. Ann Surg. 2004;188:736-740.
10. Ferzli GS, Edwards ED, Khoury GE. Chronic pain after inguinal
herniorrhaphy. J Am Coll Surg. 2007;205(2):333-341.

5/22/2012 5:45:00 PM

CHAPTER 65

Epigastric and Umbilical Hernia


Rachel Beard and Steven D. Schwaitzberg

Umbilical hernias and epigastric hernias comprise 10% of all


primary hernias.1 Umbilical hernias develop at the site of the
umbilical ring, the fascial defect via which the umbilical vessels
pass. This defect is physiologic and the vast majority will close
spontaneously during childhood. A minority, however, remains
open and becomes symptomatic, requiring repair in childhood or
adulthood.2 Umbilical hernias can also be acquired in adulthood
when herniation occurs through an umbilical canal, defined by
the umbilical fascia posteriorly, the linea alba anteriorly, and the
medial edge of the two rectus sheaths on either side. Epigastric
hernias protrude through the midline of the linea alba, most commonly above the level of the umbilicus. Whether they are congenital or acquired remains a topic of controversy. They occur in
35% of the population and 20% of patients have multiple hernias.
They are usually repaired at the time of diagnosis because smaller
defects are likely to become incarcerated or strangulated with
preperitoneal fat, whereas larger defects tend to become unsightly
and symptomatic.1
Spigelian hernias are protrusions of fat or abdominal organs,
usually small bowel or colon, through a defect in the spigelian
aponeurosis, defined as the aponeurosis of the transverse abdominal muscle limited by the linea semilunaris laterally and the lateral
edge of the rectus muscle medially.3 It was first described by Josef.
T Klinkosh in 1764 and named after Adrian van der Spieghel.
Affected patients are primarily in the fift h and sixth decades of
life, and obesity, multiple pregnancies, COPD, chronic constipation, ascites, trauma, and prior surgeries are all significant risk
factors. They are rare, affecting less than 1% of the population4
and comprising only 12% of all hernia repairs. The majority
occur in 6 cm wide transverse zone above the interspinal plane,
known as the spigelian hernia belt.3 The weakness of this area is
secondary to the intersection of the fibers of the transverse and
internal oblique muscles, the transition below the arcuate line of
the anterolateral abdominal wall becoming anterior and only the
external oblique muscle fibers maintaining their firmness, and
the penetration of the epigastric artery through the transversalis
fasia at the lateral border of the rectus sheath muscle.4

Lumbar hernias are a rare defect in the posterior abdominal


wall defined as a protrusion of the preperitoneal or intra-abdominal
contents that occur in the lumbar region, which is defined superior by the 12th rib, medically by the erector spinae muscle, inferiorly by the iliac crest, and laterally by the external oblique. These
hernias were first suggested by Barbette in 1672 and first described
in publication in 1731 by Garangeot. Lumbar hernias may be congenital, appearing initially during infancy, or acquired. Acquired
lumbar hernias are usually primary and associated with strenuous physical activity. Predisposing factors to spontaneous lumbar
hernias include extremes of weight, older age, chronic coughing,
debilitating disease, muscle atrophy, and sepsis. All these factors
lead to a loss of fatty tissue that leads to a rupture of the neurovascular orifices which penetrate the lumbodorsal fascia. The most
common location for spontaneous lumbar hernias is the superior lumbar triangle, dubbed the GrynfelttLesshaft triangle by
Macready in 1890 after Grynfeltt described the superior spaced
in 1866. It is an inverted space defined by the base of the 12th rib,
the lower edge of the serratus posterior inferior muscle, posteriorly by the sacrospinal muscle, anteriorly by the internal oblique
muscle with a roof formed by the external oblique and latissmus
dorsi, and the floor formed by the transversalis fascia and aponeurosis of the transversalis muscle. Another common location for
herniation is the Petit triangle, after Petit who described the inferior space in 1783. It is smaller and bordered by the crest of the
iliac, laterally by the external oblique, medially by latissmus dorsi
and the lumbodorsal fascia which forms the floor. Secondary
acquired lumbar hernias can be caused by blunt injury, iliac crest
fractures, infection, or abscesses, all of which alter the integrity
of the lumbodorsal fascia. Previous surgery can also lead to incisional acquired lumbar hernias, particularly after lumbotomy,
nephrectomy, abdominal aortic aneurysm surgery iliac bone
donation, and latissmus dorsi flaps.5 Traumatic acquired lumbar
hernias have also been described after seatbelt injuries after motor
vehicle crashes.6
Obturator hernias were first described by Amaud de Ronsil in
1724 with the first repair having been done by Obre in 1851.7 They

525

PMPH_CH65.indd 525

5/22/2012 5:45:41 PM

526

Surgery: Evidence-Based Practice

are rare pelvic hernias that account for less than 1% of all abdominal hernias8 though high incidences, 1.6%, have been reported
in Asian populations.7 The obturator foramen lies between the
ischium and the pubis. It is defined by the superior pubic ramus as
the upper margin, the body and inferior pubic ramus as the inferior margin, the ramus of the ischium as the lower margin, and
the anterior margin is the body of the ischium. Obturator hernias
protrude through the obturator canal through which the obturator vessels and nerve pass to reach the hip and the sac usually
contains small bowel, particularly ileum.7 The patient population
tends to be women greater than 70 years of age8 and slight build,
multiparity, cachexia, chronic lung disease and constipation and
kyphoscoliosis are all risk factors.7 Occurrences are more often on
the right than on the left and presenting symptoms include groin
pain radiating down the medial side of the thigh to the knee due
to compression of the obturator nerve (the HowshipRomberg
sign), small bowel obstruction and strangulation.8 Occasionally,
a palpable mass can be appreciated on rectal or vaginal examination.7 Diagnosis is often delayed and presentation is often of
acute or of recurrent obstruction, bowel incarceration or perforation, with high mortality rates from 12% to 70%.7 Unlike
most other hernias, CT is generally utilized for diagnosis of obturator hernias.9
1. What do we know about the natural history of umbilical hernias in adults and children?
All children possess a defect in the abdominal wall at the time
of birth through which the umbilical vessels pass. Closure of
the umbilical ring is spontaneous and affected by genetics, with
African Americans and African children having higher rates of
umbilical hernias, as well as patients with genetic disorders such
as BeckwithWiedemann and Downs syndrome. Closure rates are
variable and closure can become arrested, resulting in an umbilical hernia which may become clinically significant. Incarceration
and symptoms such as pain or obstruction are clear indications
for repair. As children grow, umbilical hernias which remain open
are less likely to close, though studies from Nigeria have demonstrated that closure does continue to occur until the age of 14.10
However, repair is generally undertaken for lesions greater than
1.5 cm in diameter after 23 years of age, defects greater than
1.0 cm which fail to decrease in size of 612 months, and defects
that persist after the age of 5.2 In adults, 10% of umbilical hernias,
as persistent from childhood and the other 90% are acquired,
represented an indirect herniation through an umbilical canal.
Factors that increase intra-abdominal pressure predispose adults
to umbilical hernias, such as obesity, multiple pregnancies, and
large abdominal tumors.1 Umbilical hernias are more likely to
occur in patients who are obese, as well as those with cirrhosis
and ascites, and early repair is often endorsed to prevent later
complications.11
Answer: The majority of umbilical hernias in children will
close spontaneously, yet those that fail to close by adolescence
are likely to remain open. In adults, the vast majority of umbilical hernias are acquired rather than persistent from childhood.
2. Do umbilical and epigastric hernia repairs require mesh?
Although there are multiple options for open repair of umbilical
and epigastric hernias, there is a widespread agreement on the

PMPH_CH65.indd 526

need for tension-free repair, regardless of the technique used.


The simplest open technique utilizes simple interrupted sutures.
Another option is the Mayo technique which involves placing
two rows of sutures and overlapping the upper edge with the
lower edge by 23 cm. Another method, the Keel technique, is
a two-layered interrupted closure method in which the medial
1.52 cm of the rectus sheath is on either side.12 Mesh repair
techniques, which utilize either a flat mesh or a mesh plug, are
increasingly popular. Possible locations for mesh placement
include preapneurotic placement using an onlay technique, fi xation to the ring on the fascial layer, on the retromuscular prefasical space or on the preperitoneal space.13 There are many options
for the types of mesh to use. Unprotected synthetic lightweight
mesh is appropriate for open repairs with no exposure of mesh to
viscera. Tissue-separating mesh is appropriate for repairs with a
high risk of dehiscence and exposure of the viscera to prevent ingrowth into the mesh. Polypropylene prosthetic mesh is overall
the most commonly used.14 Biologic meshes are another option
which are becoming more widely used, particularly when contact with bowel is unavoidable. Long-term data are still pending
but studies have demonstrated fewer bowel adhesions to repair
sites in comparison with polypropylene mesh (8% vs. 70%) with
similar strength.15 Reported recurrence rates after umbilical
hernia repair are variable and have been reported to exceed 40%
for repairs done without mesh. Studies have consistently shown
lower recurrence rates after mesh repair. One single center retrospective study reported a recurrence rate at 6 years of 7.7% in
patients undergoing open suture repair and 3% in those who had
undergone mesh repair.16 Another randomized controlled trial
reported even more disparate recurrence rates of 1% for mesh
repairs and 11% for suture repairs at 5 years.17 Yet another series
showed a recurrence rate at 5 years of 14.7% after suture repair
and 3.1% after repair with mesh.18 A meta-analysis reported a
10-fold decreased recurrence rate in mesh repair patients compared with suture repair patients with similar rates of complication.19 Disadvantages of the use of mesh include increased
cost and higher infection rates reported in some studies, which
can require reoperation for mesh removal. A single center study
specifically examined differences in infection rates between
mesh and suture repair and reported an overall infection rate
of 19% with higher rates of infection in the mesh group, though
they did fi nd lower recurrence rates with the mesh group (1.5%
vs. 9.2%).20 Other studies, however, have reported similar infection rates.17
Answer: Utilizing mesh in the repair of umbilical and epigastric
hernias clearly decreases recurrence rates, though increased infectious risks and subsequent complications need to be considered.
3. Is there a role for laparoscopic repair of umbilical and epigastric hernias?
Laparoscopic repair of ventral hernias has been described for
many years as a feasible and safe option. Initial studies described
it as an option for recurrent ventral hernias, including those with
failed open mesh repairs.21 Studies that followed demonstrated
low conversion rates (01.9%), low rates of recurrence (33.4%),
low rates of complication (1314%) and quick recovery times,
and recommended it as a reasonable option for repair of primary
hernias.22,23 Subsequent studies specifically examined its role in the
repair of umbilical hernias and found similarly acceptable results,

5/22/2012 5:45:42 PM

Epigastric and Umbilical Hernia

particularly for larger umbilical hernias. Shorter hospital stays,


faster operative times, and lower recurrence rates were demonstrated, particularly for those with previous repairs and hernias
larger than 3 cm.24 Subsequent prospective studies have consistently
demonstrated shorter hospital stays, less postoperative pain, fewer
wound complications, and no recurrence rates with laparoscopic
onlay patch hernioplasty for umbilical hernias compared with
open techniques including Mayo repair, suture herniorrhaphy,
and mesh hernioplasty.25 Open repair remains the technique of
choice, however, for smaller defects (<2 cm), as these can be performed under local anesthetic, rather than the general anesthesia required for laparoscopic repair.26 Retrospective reviews have
demonstrated shorter operative times and lower costs for open
repair of small defects as compared with laparoscopic repairs,
though prospective series comparing the techniques for umbilical
and epigastric hernia repairs have not yet been done.26
Answer: Laparoscopic repair of umbilical and epigastric hernias is an acceptable alternative to open repair and can be considered the procedure of choice for larger defects.
4. Is there an optimal method for repairing spigelian, lumbar,
and obturator hernias?
The most widespread repair of Spigelian hernias utilizes an open
anterior herniorraphy with approximation of the muscle and
mesh placement into the preperitoneal space.3 Th is approach
offers good results but can require wide dissection of the muscle
layers, general anesthesia, and postoperative admission. Recently,
a new technique has been described involving preperitoneal and
subfascial prosthetic repair using Prolene Hernia System (PHS)
mesh. In this repair, the bottom underlay portion of the PHS mesh
lies flat in the preperitoneal space, the connector portion obliterates the hernia orifice, and the overlay covers the internal oblique
with four cardinal sutures placed to secure the connection portion. Studies have shown a quicker recovery time and this repair
can more often be done with local, rather than general anesthesia.4
Laparoscopic repair has also been done using a transabdominal
approach with preperitoneal mesh placement 27 as well as utilizing
the scroll technique, wherein the mesh is rolled and attached to
the anterior abdominal wall prior to being unrolled. Laparoscopic
suture repair has also been reported as feasible and safe.28
There are multiple options for repair of lumbar hernias. The
anterior approach requires a major dissection but allows for a complete parietal reconstruction. Laparoscopic approach has become
more popular for its obvious benefits of less pain and shorter
hospital stays. Benefits include improved visualization and exact
localization of the hernia lesion; however, laparoscopic repair does
not permit parietal reconstruction. A transabdominal approach
can also be used. One study reviewed the literature and recommended an anterior approach or extraperitoneal laparoscopy on
small defects with extraperitoneal contents, a transabdominal
approach for moderate defects with paraperitoneal or intraperitoneal hernias, and an anterior repair with double mesh for cases

PMPH_CH65.indd 527

527

of recurrences or diff use hernias larger than 15 cm.5 Small series


have reported open anterior repair with mesh to be easy and safe
with no recurrences, with a recovery to normal function in about
2 weeks for most patients.29 Laparoscopic repairs of acquired and
traumatic lumbar hernias have been reported in small series to be
safe and feasible with no recurrences and a faster return to normal function and the majority of these cases can be done on an
outpatient basis.30-32
Surgical repair of obturator hernias, particularly primary
repair, is difficult because the tissues around obturator hernias
are not easily mobilized.33 Because the presenting symptoms are
often bowel obstruction or incarceration, many repairs are done
on an urgent or on an emergent basis. Open repair has traditionally been done, most frequently with a lower midline laparotomy incision. Retropubic, obturator, and inguinal approaches
have also been described. After the hernia is reduced and bowel
viability is confirmed, the repair can either be performed primarily34 or with mesh.7 There are many case reports of successful laparoscopic repairs, both elective and emergent, of obturator
hernias, 35-37 but prospective and retrospective comparisons have
not been done.
Answer: Anterior repair with mesh is the most widely utilized technique for spigelian hernias, though laparoscopic repair is
becoming more widespread. Anterior, transabdominal, and laparoscopic repairs are all acceptable options for lumbar hernia
repairs, and the choice is most dependent on the size of the defect.
Open repair of obturator hernias is traditional but successful laparoscopic repair has been reported, though large trials have not
yet been done.
5. Does umbilical or epigastric hernia repair require preoperative antibiotics?
There is controversy over whether preoperative antibiotics are
useful for prevention of infection prior to repair of umbilical
and epigastric hernias. Some authors assert that hernias are
clean cases and should therefore not require preoperative antibiotic dosing. 38 One randomized control trial showed significantly
lower infection rates of both umbilical (11% vs. 40%) and incisional (0% vs. 50%) hernia repairs after a one-time preoperative dose of cefonicid. 39 There are ample studies which examine
preoperative antibiotic dosing for repair of inguinal hernias,
some of which demonstrate a decreased wound infection after
antibiotic dosing,40 whereas others fail to show a difference in
outcomes.41,42 More studies are needed, however, to determine
if preoperative antibiotics for umbilical and epigastric hernias are necessary, though postoperative antibiotics have been
shown to decrease the incidence of cellulitis in patients who
developed seromas after laparoscopic ventral hernia repair (40%
vs. 100%).43
Answer: Though more studies are needed, small series have
demonstrated decreased wound infections with preoperative antibiotics for umbilical and epigastric hernia repair.

5/22/2012 5:45:42 PM

528

Surgery: Evidence-Based Practice

Clinical Question Summary


Question

Answer

1 What do we know about


the natural history of
umbilical hernias in adults
and children?

The majority of umbilical hernias in children


will close spontaneously, yet those that
fail to close by adolescence are likely to
remain open. In adults, the vast majority of
umbilical hernias are acquired rather than
persistent from childhood.

Grade C

2c, 4

1, 2, 10, 11

2 Do umbilical and epigastric


hernia repairs require
mesh?

Utilizing mesh in the repair of umbilical


and epigastric hernias clearly decreases
recurrence rates, though increased
infectious risks and subsequent
complications need to be considered.

Grade B

1b, 2b, 4

12-20

3 Is there a role for


laparoscopic repair of
umbilical and epigastric
hernias?

Laparoscopic repair of umbilical and epigastric


hernias is an acceptable alternative to open
repair and can be considered the procedure
of choice for larger defects.

Grade B

2b, 4

21-26

4 Is there an optimal method


for repairing spigelian,
lumbar, and obturator
hernias?

Anterior repair with mesh is the most widely


utilized technique for spigelian hernias,
though laparoscopic repair is becoming
more widespread.
Anterior, transabdominal, and laparoscopic
repairs are all acceptable options for
lumbar hernia repairs, and the choice is
most dependent on the size of the defect.
Open repair of obturator hernias is traditional
but successful laparoscopic repair has been
reported, though large trials have not yet
been done.

Grade C

2, 4, 27, 28

Grade C

5, 29-32

Grade C

7, 33-37

Though more studies are needed, small series


have demonstrated decreased wound
infections with preoperative antibiotics for
umbilical and epigastric hernia repair.

Grade B

1b, 4

38-43

5 Does umbilical or
epigastric hernia repair
require preoperative
antibiotics?

REFERENCES
1. Muschaweck U. Umbilical and epigastric hernia repair. Surg Clin
North Am. 2003;83(5):1207-1221.
2. Brandt M. Pediatric hernias. Surg Clin North Am. 2008;88(1):27-43.
3. Skandalakis PN, Zoras O, Skandalakis JE, Mirilas P. Spigelian
hernia: surgical anatomy, embryology, and technique of repair.
Am Surg. 2006;72(1):42-48.
4. Campanelli G, Pettinari D, Nicolosi FM, Avesani EC. Spigelian
hernia. Hernia. 2005;9(1):3-5.
5. Moreno-Egea A, Baena E, Calle M, Martnez JA, Albasini JL.
Controversies in the current management of lumbar hernias.
Arch Surg. 2007;142(1):82-88.
6. Thompson NS, Date R, Charlwood AP, Adair IV, Clements WD.
Seat-belt syndrome revisited. Int J Clin Pract. 2001;55(8):573-575.
7. Rodrguez-Hermosa JI, Codina-Cazador A, Maroto-Genover A,
et al. Obturator hernia: clinical analysis of 16 cases and algorithm
for its diagnosis and treatment. Hernia. 2008;12(13):289-297.
8. Pandey R, Maqbool A, Jayachandran N. Obturator hernia: A
diagnostic challenge. Hernia. 2009;13:97-99.
9. Igari K, Ochiai T, Aihara A, Kumagai Y, Iida M, Yamazaki S. Clinical presentation of obturator hernia and review of the literature.
Hernia. 2010;14(4):409-413.

PMPH_CH65.indd 528

Grade of
Recommendation

Level of
Evidence

References

10. Meier D, OlaOlorun D, Omodele R, Nkor S, Tarpley J. Incidence


of umbilical hernia in African children: Redefinition of normal and reevaluation of indications for repair. World J Surg.
2001;25(5):645-648.
11. McKay A, Dixon E, Bathe O, Sutherland F. Umbilical hernia
repair in the presence of cirrhosis and ascites: Results of a survey
and review of the literature. Hernia. 2009;13(5):461-468.
12. Mushaweck U. Umbilical and epigastric hernia repair. Surg Clin
North Am. 2003;83(5):1207-1221.
13. Velasco M, Gracia-Urena M, Hidalgo M, Vega V, Carnerol F. Current concepts on adult umbilical hernia. Hernia. 1999;3:233-239.
14. Bachman S, Ramshaw B. Prosthetic material in ventral hernia repair: How do I choose? Surg Clin North Am. 2008;88(1):
101-112.
15. Burns NK, Jaffari M, Rios CN, Mathur AB, Butler CE. Noncross-linked porcine acellular dermal matrices for abdominal
wall reconstruction. Plast Reconstr Surg. 2010;125(1):167-176.
16. Asolati M, Huerta S, Sarosi G, Harmon R, Bell C, Anthony T. Predictors of recurrence in veteran patients with umbilical hernia:
Single center experience. Am J Surg. 2006;192(5):627-630.
17. Arroyo A, Garcia P, Prez F, Andreu J, Candela F, Calpena R.
Randomized clinical trial comparing suture and mesh repair of
umbilical hernia in adults. Br J Surg. 2001;88(10):1321-1323.

5/22/2012 5:45:42 PM

Epigastric and Umbilical Hernia

18. Stabilini C, Stella M, Frascio M, et al. Mesh versus direct suture


for the repair of umbilical and epigastric hernias. Ten-year experience. Ann Ital Chir. 2009;80(3):183-187.
19. Aslani N, Brown C. Does mesh offer an advantage over tissue in
the open repair of umbilical hernias? A systematic review and
meta-analysis. Hernia. 2010;14(5):455-462.
20. Farrow B, Awad S, Berger DH, et al. More than 150 consecutive
open umbilical hernia repairs in a major Veterans Administration Medical Center. Am J Surg. 2008;196(5):647-651.
21. Costanza M, Heniford B, Arca M, Gagner M. Laparoscopic repair
of recurrent ventral hernias. Am Surg. 1998;64(12):1121-1125.
22. Heniford B, Ramshaw B. Laparoscopic ventral hernia repair: A
report of 100 consecutive cases. Surg Endosc. 2000;14(5):419-423.
23. Heniford BT, Park A, Ramshaw BJ, Voeller G. Laparoscopic ventral and incisional hernia repair in 407 patients. J Am Coll Surg.
2000;190(6):645-650.
24. Gonzalez R, Mason E, Duncan T, Wilson R, Ramshaw BJ. Laparoscopic versus open umbilical hernia repair. J Soc Laparoendosc Surg. 2003;7(4):323-328.
25. Lau H, Patil N. Umbilical hernia in adults. Surg Endosc. 2003;
17(12):2016-2020.
26. Wright BE, Beckerman J, Cohen M, Cumming JK, Rodriguez JL. Is
laparoscopic umbilical hernia repair with mesh a reasonable alternative to conventional repair? Am J Surg. 2002;184(6):505-508.
27. Majeski J. Open and laparoscopic repair of Spigelian hernia. Int
Surg. 2004;94(4):365-369.
28. Bittner JG 4th, Edwards MA, Shah MB, MacFadyen BV Jr,
Mellinger JD. Mesh-free laparoscopic spigelian hernia repair.
Am Surg. 2008;74(8):713-720.
29. Cavallaro G, Sadighi A, Miceli M, Burza A, Carbone G,
Cavallaro A. Primary lumbar hernia repair: The open approach.
Eur Surg Res. 2007;39(2):88-92.
30. Yavuz N, Ersoy YE, Demirkesen O, Tortum OB, Erguney S.
Laparoscopic incisional lumbar hernia repair. Hernia. 2009;
13(3):281-286.
31. Madan AK, Ternovits CA, Speck KE, Pritchard FE, Tichansky DS.
Laparoscopic lumbar hernia repair. Am Surg. 2006;74(4):318-321.
32. Moreno-Egea A, Guzman P, Girela E, Corral M, Aguayo
Albasini JL. Laparoscopic hernioplasty in secondary lumbar
hernias. J Laparoendosc Adv Surg Tech. 2006;16(6):572-576.

PMPH_CH65.indd 529

529

33. Bergstein JM, Condon RE. Obturator hernia: Current diagnosis


and treatment. Surgery. 1996;119(2):133-136.
34. Shipkov CD, Uchikov AP, Grigoriadis E. The obturator hernia: Difficult to diagnose, easy to repair. Hernia. 2004;8(2):
155-157.
35. Fakeye VO, John AR, Jambulingam PS, Vidya R. Pre-operative
diagnosis of obturator hernia: A report of two cases. Ann Roy
Coll Surg Engl. 2010;92(1):W1-W2.
36. Haith LR Jr, Simeone MR, Reilly KJ, Patton ML, Moss BE,
Shotwell BA. Obturator hernia: Laparoscopic diagnosis and
repair. J Soc Laparoendosc Surg. 1998;2(2):191-193.
37. Perry CP, Hantes JM. Diagnosis and laparoscopic repair of type
I obturator hernia in women with chronic neuralgic pain. J Soc
Laparoendosc Surg. 2005;9(2):138-141.
38. Bohnen J. Antimicrobial prophylaxis in general surgery. Can J
Surg. 1991;34(6):548-550.
39. Classen DC, Evans RS, Pestotnik SL, Horn SD, Menlove RL,
Burke JP. The timing of prophylactic administration of antibiotics and the risk of surgical-wound infection. N Engl J Med. 1992;
326(5):281-286.
40. Abramov D, Jeroukhimov I, Yinnon AM, et al. Antibiotic prophylaxis in umbilical and incisional hernia repair: A prospective
randomised study. Eur J Surg. 1996;162(12):945-948.
41. Yerdel MA, Akin EB, Dolalan S, et al. Effect of single-dose prophylactic ampicillin and sulbactam on wound infection after
tension-free inguinal hernia repair with polypropylene mesh:
The randomized, double-blind, prospective trial. Ann Surg. 2001;
233(1):26-33.
42. Aufenacker TJ, van Geldere D, van Mesdag T, et al. The role of
antibiotic prophylaxis in prevention of wound infection after
Lichtenstein open mesh repair of primary inguinal hernia: A
multicenter double-blind randomized controlled trial. Ann Surg.
2004;240(6):955-960.
43. Perez AR, Roxas MF, Hilvano SS. A randomized, double-blind,
placebo-controlled trial to determine effectiveness of antibiotic
prophylaxis for tension-free mesh herniorrhaphy. J Am Coll
Surg. 2005;200(3):393-397.
44. Edwards C, Angstadt J, Whipple O, Grau R. Laparoscopic ventral hernia repair: Postoperative antibiotics decrease incidence of
seroma-related cellulitis. Am Surg. 2005;71(11):931-935.

5/22/2012 5:45:42 PM

CHAPTER 66

Incisional and Ventral Hernias


Rachel Beard and Steven D. Schwaitzberg

INTRODUCTION

repair which consisted of closing the anterior and posterior rectus sheaths separately along with the associated rectus muscles.
They reported a recurrence rate of 4.5%.5 Current practice suggests that mesh repair decreases recurrence rates to 09%.1 There
are many different ways to perform a mesh repair, including prefascial onlay, subfascial or preperitoneal sublay or inlay, in which
the fasica is not approximated but a mesh is used to bridge the
gap. There are various different types of synthetic mesh which can
be used. Type I mesh is a macroporous prosthesis consisting of
a monofilament or a double fi lament polypropylene. Type II is a
completely microporous prosthesis, such as expanded polytetrafluoroethylene (PTFE) whereas Type III is a macroporous prosthesis with microporous prosthesis, such as PTFE mesh.4 Recurrence
rates with prosthetic mesh have been reported as high as 24% over
a 3 year period for primary hernia and 20% for a recurrent hernia.6 A number of studies have looked at the differences in the
recurrence based on the type of mesh used. One study from 2005
reported improved recurrence rates at 2 years (7% vs. 17%) when
using a standard polyester or a polypropylene mesh as opposed to
the lightweight composite mesh.7 Another study from 2002 compared polypropylene mesh repair with skin autograft repair in
both simple and complex hernias, with complex hernias defined
as those larger than 10 cm in diameter or a re-recurrence. Both
methods utilized an onlay technique and results demonstrated
a recurrence rate of 8.6% for the mesh group and 12.3% for the
skin autograft group.8 A study published in 2003 prospectively
looked at patients undergoing open intraperitoneal underlay
repair with a bilayer prosthetic mesh and found no recurrences
at a median follow-up of 28 months.9 Another option is the use
of a biologic mesh. A recent study published in 2011 looked at a
group of 57 patients who underwent ventral hernia repair with
an underlay of XenMatrix biologic porcine mesh. Their overall
recurrence rate was 7.2% and all recurrences occurred within the
first 3 weeks, with no additional recurrences at 1 year.10 Another
study published in 2011 examined the differences between various biologic meshes used in repair of ventral abdominal defects,
and demonstrated lower rates of infection (37.9%) but higher rates
of hernia recurrence (28.6%) when using human-derived mesh

Incisional hernias occur through a scar from a prior surgery and


are a serious complication of abdominal wall surgery. Occurrence
rates have been reported in up to 11% of patients undergoing
abdominal wall surgery and up to 23% in those who develop postoperative wound infections.1 Studies have been done to compare
the rate of herniation between different abdominal incisions. One
review of 11 studies found that midline incisions had the highest
rate of ventral hernias at 10.5%, followed by transverse incisions
at 7.5% and paramedian incisions at 2.5%.2 Another study, performed in the prelaparoscopic era of the late 1970s, randomized
150 patients to undergo cholecystectomy either via a midline or via
a transverse incision, and demonstrated lower rates of incisional
hernias when utilizing a transverse incision, 2% versus 14%.3 Incisional hernias are fraught with complications. Even hernias which
are initially small and asymptomatic can enlarge overtime and
lead to pain, bowel obstruction, and ultimately life threatening
complications such as incarceration or strangulation. Risk factors for developing incisional hernias include suture technique,
wound infection, increased abdominal wall tension from obesity
or pregnancy, and connective tissue disorders.4 Minimizing the
occurrence and recurrence of incisional hernias is not only in
the interest of the patients, but also in the interest of the healthcare system overall. There are numerous options available for
repair of such hernias, including open or laparoscopic techniques,
suture hernioplasty, mesh repair, and component separation. Each
technique has benefits and drawbacks, and familiarity with such
repairs is an essential knowledge for the general surgeon.
1. What are the available data on recurrence rates (>2 years)
for hernias repaired with biologic versus synthetic mesh?
Aside from very small incisional and ventral hernias, the majority of ventral hernia repairs require the use of a mesh to prevent
recurrence. The recurrence rates after open suture repair have
been found to be unacceptably high, reported at 3149%.1 Of
note, one recent paper advocated utilizing a two-layered suture
530

PMPH_CH66.indd 530

5/22/2012 5:46:16 PM

Incisional and Ventral Hernias

as compared with porcine cross-linked meshes, which demonstrated a 60% infection rate and 20% hernia recurrence rate. Porcine noncrosslinked meshes had the worst outcome with 29.4%
recurrence rates and 54.2% infection rates.11
Answer: Recurrence rates following mesh repair depend not
only on the type of mesh but also on the technique. Truly longterm data do not exist comparing biologic and synthetic meshes,
but there is a likely trend toward higher recurrence rates with biologic meshes (Grade C recommendation).
2. Are there any data available on watchful waiting for incisional hernias?
Conventional teaching is that once an incisional hernia occurs, it
should be repaired as soon as possible to prevent complications
such as bowel obstruction, strangulation, or loss of abdominal
domain. Over time, the lateral abdominal muscles retract and
become fibrotic, thereby enlarging the hernia and increase the
complexity of future repairs, as well as the risk of recurrence after
repair. The only recommended delay in the repair of an incisional
hernia is to allow for weight loss in obese patients.12 In a review
of the literature, there are plentiful data on watchful waiting
and the natural history of other types of hernias. Several randomized clinical trials have been done studying watchful waiting of
patient with asymptomatic or minimally symptomatic inguinal
hernias. These studies have concluded that delaying repair has no
adverse effect on the eventual operation and no statistically significant increased complications in the interim.13-15 A recent study
examined watchful waiting in the case of traumatic lumbar
hernia and reported good outcomes with two patients who were
managed with just an observation for several months.16 Watchful
waiting of abdominal wall hernias has been studied in the pregnant population and delay of repair until the postpartum period
has been shown to be a safe management strategy.17 However,
similar studies have not been done on patients with incisional or
with ventral hernias.
Answer: There are no data available to promote or reject
the watchful waiting concept for incisional hernias (Grade D
recommendation).
3. What is the recurrence rate for repairing hernias where the
rectus cannot be closed comparing component separation versus bridging the defect with mesh?
When an abdominal wall hernia is so large and there is such a
loss of domain that the fascia cannot be approximated, often the
mesh is used to close the gap.18 A recent study published in 2007
looked at a series of patients with incisional hernias at least 20 cm
in length that could not be closed primarily and randomized them
to either component separation repair or PTFE repair using the
mesh to bridge the fascial gap. They found similar recurrence rates,
56% versus 58%, between the component separation and the mesh
group, respectively, at 1 year, and recurrences occurred at a mean
of 7 months postoperatively.19 An earlier study in 2005, however,
compared two different methods of bridging a fascial gap. Patients
with large recurrent ventral hernias either underwent repair with
an onlay polypropylene mesh or double mesh intraperitoneal
repair in which a Vicryl and polypropylene mesh were sutured
together and placed intraperitoneally with the closure of the hernia sac over the mesh. They reported a recurrence rate of 27% in the
onlay group and 0% in the double mesh group at 1 year.20 A recent
retrospective review published in 2011 attempted component

PMPH_CH66.indd 531

531

separation of 133 ventral hernia repair patients over 4 years with


placement of a mesh underlay when the fascia could not be closed.
They reported a recurrence rate of 16% in patients undergoing
component separation alone and 8% in those undergoing component separation with the mesh bridging the fascial gap, with the
median recurrence occurring at 10.4 months.21 A small case series
published in 2010 described a novel method of repair of complex
incisional hernias in which a laparoscopic component separation was performed to facilitate a RivesStoppa repair with mesh
placement in the retromuscular position without skin flaps. They
reported no recurrences in short-term follow-up.22
Answer: Intraperitoneal mesh repair and combined component separation and mesh repair yield the lower recurrence
rates when compared with component separation alone (Grade C
recommendation).
4. Are there available data comparing open hernia repairs to
laparoscopic repairs with respect to recurrence?
Laparoscopic repair has been demonstrated to be a safe and feasible option for incisional hernias, with recurrence rates reported at
09%.1 There are plentiful data comparing open and laparoscopic
repairs, particularly with respect to complications and recurrence rates. One meta-analysis of randomized controlled trials
in the literature demonstrated no difference in recurrence rates
between techniques, but a shorter mean hospital stay with laparoscopic repair. This study also showed that laparoscopic repair
was associated with fewer wound infections and fewer hemorrhagic complications.23 Another prospective study demonstrated
similar findings, namely shorter postoperative stays, less pain,
fewer complications, and lower recurrence rates (2% vs. 10%) with
patients undergoing laparoscopic repair with the mesh for ventral
hernias as compared with open repair with the mesh.24 A multicenter cohort study published in 2010 concluded that underlay
mesh technique, regardless of whether or not it is performed open
or laparoscopically, leads to lower recurrence rates after several
years than suture repairs, inlay, or only mesh techniques.25 However, another randomized trial demonstrated higher recurrence
rates at 2 years with patients undergoing laparoscopic repairs for
ventral hernias as compared with open, 12.5% vs. 8.2%, though
laparoscopic repair was associated with fewer complications and
faster recovery.26 The majority of studies comparing recurrence
rates have a follow-up time of 23 years. One retrospective study
followed up 331 patients for 5 years and found similar recurrence rates between laparoscopic and open repairs. With respect
to surgical technique, the only patients in the study that demonstrated higher recurrence rates were those who underwent laparoscopic repairs that required conversion to open procedures.
Larger defects demonstrated higher recurrence rates, regardless
of the technique.27 The majority of studies published in the U.S.
seem to demonstrate few complications with laparoscopic repair,
though one study published in 2009 out of Denmark demonstrated higher rates of major complications (4.8% versus. 2.8%)
and overall morbidity (11.8% vs. 10.1%) with laparoscopic repair
as compared with open.28
Answer: The literature demonstrates similar recurrence rates
with laparoscopic and open repair of incisional hernias. Laparoscopic repairs are associated with faster recovery times and lower
rates of wound infection, but some studies have demonstrated
higher rates of major complications and morbidity (Grade C
recommendation).

5/22/2012 5:46:17 PM

532

Surgery: Evidence-Based Practice

5. What influence does technique have on wound infection


rate?
In addition to hernia recurrence, wound infection is one of the
most important complications of hernia repair. Studies have demonstrated certain comorbidities which independently increase the
rate of wound infection following hernia repair, including steroid
use, diabetes, obesity, advanced age, chronic obstructive pulmonary disease, coronary artery disease, and poor nutritional status.
In addition, long operative times and use of synthetic mesh have
demonstrated higher wound infection rates. Importantly, infected
fields involving synthetic mesh often require reoperation for mesh
removal, whereas those done with suture repairs can more often
resolve with conservative management. There is evidence that the
technique used when utilizing mesh techniques affects the rate
of occurrence of wound infections. One study from 2010 demonstrated that patients who underwent incisional hernia repair with
a prosthetic mesh sublay technique had lower wound infection

rates than those who underwent repair with an onlay technique


with the same prosthetic mesh.29 In patients where the risk of surgical site infection is thought to be high, biologic mesh may be
used in place of synthetic mesh. These materials are more resistant
to infection, and, should infection occur, remain intact without
requiring mesh removal.30,31 Some case-controlled studies have
demonstrated higher rates of wound infection with open ventral
hernia repair with mesh when compared with laparoscopic repair,
22.1% versus 3.4% in one study. Open repairs were also associated with higher blood loss and were more likely to require drain
placement.32 In addition to overt wound infection, seromas also
occurred more often with open repair than with laparoscopic
repair, 23.3% versus 6.8%, and were associated with drain placement and more preoperative abdominal incisions.18
Answer: The risk of wound infection can be lessened by utilizing a sublay technique for repairs utilizing mesh and laparoscopic techniques when possible (Grade B recommendation).

Clinical Question Summary


Question

Answer

1 What are the available


data on recurrence rates
(>2 years) for hernias
repaired with biologic
versus synthetic mesh?

Recurrence rates following mesh


repair depend not only on
the type of mesh but also on
technique. Truly long-term data
do not exist comparing biologic
and synthetic meshes, but there
is a likely trend toward higher
recurrence rates with biologic
meshes.

Grade C

1a, 1b, 2b, 2c, 3b, 4

1, 4-11

2 Are there any data


available on watchful
waiting for incisional
hernias?

There are no data available to


promote or reject the watchful
waiting concept for incisional
hernias.

Grade D

1b, 4, 5

12-17

3 What is the recurrence


rate for repairing hernias
where the rectus cannot
be closed comparing
component separation
versus bridging the
defect with mesh?

Intraperitoneal mesh repair


and combined component
separation and mesh repair
yield the lower recurrence
rates when compared with
component separation alone.

Grade C

1b, 2b, 2c, 4

18-22

4 Are there available data


comparing open hernia
repairs to laparoscopic
repairs with respect to
recurrence?

The literature demonstrates


similar recurrence rates with
laparoscopic and open repair of
incisional hernias. Laparoscopic
repairs are associated with faster
recovery times and lower rates
of wound infection, but some
studies have demonstrated
higher rates of major
complication and morbidity.

Grade C

1a, 1b, 2b, 2c, 3b, 4

1, 22-28

5 What influence does


technique have on
wound infection rate?

The risk of wound infection


can be lessened by utilizing a
sublay technique for repairs
utilizing mesh and laparoscopic
techniques when possible.

Grade B

1a, 1b, 4

18, 29-32

PMPH_CH66.indd 532

Grade of
Recommendation

Level of
Evidence

References

5/22/2012 5:46:17 PM

Incisional and Ventral Hernias

REFERENCES
1. Cassar K, Munro A. Surgical treatment of incisional hernias. Br
J Surg. 2002;89:534-545.
2. Carlson MA, Ludwig K, Condon RE. Ventral hernia and other
complications of 1,000 midline incisions. Southern Med J. 1995;
88(4):450-453.
3. Halm JA, Lip H, Schmitz PI, Jeekel J. Incisional hernia after
upper abdominal surgery: A randomised controlled trial of midline versus transverse incision. Hernia. 2009;13(3):275-280.
4. den Hartog D, Dur A, Tuinebreijer WE, Kreis RW. Open surgical
procedures for incisional hernias. Cochrane Database of Systemic
Reviews. 2011;16(3):CD006438.
5. Dur AH, den Hartog D, Tuinebreijer WE, Kreis RW, Lange JF. Low
recurrence rate of a two-layered closure repair for primary and
recurrent midline incisional hernia without mesh. Hernia. 2009;
13(4):421-426.
6. Luijendijk RW, Hop W, van den Tol MP, et al. A comparison of
suture repair with mesh repair for incisional hernia. N Engl J
Med. 2000;343(6):392-398.
7. Conze J, Kingsnorth A, Flament JB, et al. Randomized clinical trial comparing lightweight composite mesh with polyester
or polypropylene mesh for incisional hernia repair. Br J Surg.
2005;92(12):1488-1493.
8. Korenkov M, Sauerland S, Arndt M, Bograd L, Neugebauer EA,
Troidl H. Randomized clinical trial of suture repair, polypropylene mesh or autodermal hernioplasty for incisional hernia. Br J
Surg. 2002;89(1):50-56.
9. Millikan KW, Baptista M, Amin B, Deziel DJ, Doolas A. Intraperitoneal underlay ventral hernia repair utilizing bilayer expanded
polytetrafluoroethylene and polypropylene mesh. Am Surg. 2003;
69(4):287-291.
10. Byrnes MC, Irwin E, Carlson D, et al. Repair of high-risk incisional hernias and traumatic abdominal wall defects with porcine mesh. Am Surg. 2011;77(2):144-150.
11. Shah BC, Tiwari M, Goede MR, et al. Not all biologics are equal!
Hernia. 2011;15(2):165-171.
12. Voeller G. Ventral abdominal hernia. In: Fischer JE, ed. Mastery
of Surgery. Philadelphia, PA, Lippincott Williams and Williams;
2006:1947-1957.
13. Thompson JS, Gibbs J, Reda DJ, et al. Does delaying repair of an
asymptomatic hernia have a penalty? Am J Surg. 2008;195(1):89-93.
14. Fitzgibbons RJ, Jr, Giobbie-Hurder A, Gibbs JO, et al. Watchful
waiting vs repair of inguinal hernia in minimally symptomatic
men: A randomized clinical trial. JAMA. 2006;295(3):285-292.
15. Stroupe KT, Manheim L, Lou P, et al. Tension-free repair versus watchful waiting for men with asymptomatic or minimally
symptomatic inguinal hernias: A cost-effectiveness analysis. J
Am Coll Surg. 2006;203(4):458-468.
16. Bathla L, Davies E, Fitzgibbons RJ Jr., Cemaj S. Timing of traumatic lumbar hernia repair: is delayed repair safe? Report of two
cases and review of the literature. Hernia. 2011;15(2): 205-209.

PMPH_CH66.indd 533

533

17. Buch KE, Tabrizian P, Divino CM. Management of hernias in


pregnancy. J Am Coll Surg. 2008;207(4):539-542.
18. Kaafarani HM, Hur K, Hirter A, et al. Seroma in ventral incisional herniorrhaphy: Incidence, predictors and outcome. Am J
Surg. 2009;198(5):639-644.
19. de Vries Reilingh TS, van Goor J, Charbon JA, et al. Repair of
giant midline abdominal wall hernias: Components separation
technique versus prosthetic repair. World J Surg. 2007;31(4):
756-763.
20. Afifi R. A prospective study between two different techniques for
the repair of a large recurrent ventral hernia: A double mesh intraperitoneal repair versus onlay mesh repair. Hernia. 2005;9(4):
310-315.
21. Hadeed JG, Walsh M, Pappas TN, et al. Complex abdominal wall
hernias: A new classification system and approach to management based on review of 133 consecutive patients. Ann Plast Surg.
2011;66(5):497-503.
22. Cox TC, Pearl J, Ritter EM. Rives-Stoppa incisional hernia repair
combined with laparoscopic separation of abdominal wall components: A novel approach to complex abdominal wall closure.
Hernia. 2010;14(6):561-567.
23. Forbes SS, Eskicioglu C, McLeod RS, Okrainec A. Meta-analysis
of randomized controlled trials comparing open and laparoscopic
ventral and incisional hernia repair with mesh. Br J Surg. 2009;
96(8):851-858.
24. Lomanto D, Iyer S, Shabbir A, Cheah WK. Laparoscopic versus
open ventral hernia mesh repair: A prospective study. Surg
Endosc. 2006;20(7):1030-1035.
25. Hawn MT, Snyder C, Graham LA, Gray SH, Finan KR, Vick CC.
Long-term follow-up of technical outcomes for incisional hernia
repair. J Am Coll Surg. 2010;210(5):648-655.
26. Itani KM, Hur K, Kim LT, et al. Comparison of laparoscopic and
open repair with mesh for the treatment of ventral incisional
hernia: A randomized trial. Arch Surg. 2010;145(4):322-328.
27. Ballem N, Parikh R, Berber E, Siperstein A. Laparoscopic versus
open ventral hernia repairs: 5 year recurrence rates. Surg Endosc.
2008;22(9):1935-1940.
28. Bisgaard T, Kehlet H, Bay-Nielsen MB, et al. Nationwide study
of early outcomes after incisional hernia repair. Br J Surg. 2009;
96(12):1452-1457.
29. Venclauskas L, Maleckas A, Kiudelis M. One-year follow-up after
incisional hernia treatment: Results of a prospective randomized
study. Hernia. 2010;14(6):575-582.
30. Breuing K, Butler C, Ferzoco S, et al. Incisional ventral hernias:
review of the literature and recommendations regrading the
grading and technique of repair. Surgery. 2010;148(3):544-558.
31. Kim H, Bruen K, Vargo D. Acellular dermal matrix in the management of high-risk abdominal wall defects. Am J Surg. 2006;
192(6):705-709.
32. Kaafarani HM, Kaufman D, Reda D, Itani KM. Predictors of surgical site infection in laparoscopic and open ventral incisional
herniorrhaphy. J Surg Res. 2010;163(2):229-234.

5/22/2012 5:46:17 PM

PMPH_CH66.indd 534

5/22/2012 5:46:17 PM

PART XI

ENDOCRINE GLANDS

PMPH_CH67.indd 535

5/22/2012 5:46:47 PM

PMPH_CH67.indd 536

5/22/2012 5:46:47 PM

CHAPTER 67

Adrenocortical Tumors and


Incidentalomas
Paul Karanicolas and Murray Brennan

INTRODUCTION

8.1% during that time frame despite purported improvements in


surgical techniques such as the proliferation of laparoscopic surgery. Given this apparent increase in the use of adrenalectomy for
benign disease and the increase in postoperative complications,
an evidence-based appraisal of the current approach to diagnosis
and management of adrenal tumors is warranted.
We attempt to identify the highest level of evidence available
for each seminal question using a systematic, comprehensive search
of the published literature. Very few RCTs have been performed
directly addressing the issues we consider to be most relevant to the
practicing surgeon managing patients with adrenocortical tumors.
When high-quality evidence is not available, we present the best
available evidence and our recommendations. Finally, we discuss
the opportunities for surgical research to contribute to the evidence that is accumulating for the management of these patients.

In a text of evidence-based surgical practice, summarizing data


and providing recommendations on the relatively uncommon,
diverse collection of diseases encompassing the topic of adrenocortical tumors is challenging. The threat to evidence-based surgery
is the paucity of high-quality evidence to guide decision-making.
Although evidence from randomized controlled trials (RCTs)
provides surgeons with the most confidence in the effectiveness
of an intervention, several challenges limit the ability to perform
RCTs of surgical interventions.1 RCTs commonly include highly
selected patients treated in a highly controlled environment that
limits the applicability of these findings to a broader population.2
In situations where RCTs cannot be performed due to practical
considerations, surgeons must still make decisions and treatment
recommendations based on suboptimal evidence.
The optimal disease to study with an RCT is one which (1) is
common, to allow sufficient accrual in a reasonable amount of
time, (2) has a relatively homogeneous presentation, severity, and
prognosis, (3) has limited treatment optionsideally only two, and
(4) has a predictable, measurable outcome in a short interval. Adrenal tumors satisfy none of these requirements. They are uncommon (except for benign incidentalomas), and may be characterized
as benign or malignant, primary or metastatic, functional or nonfunctional, and cortical or medullary. Taken together, these permutations represent 16 potential categories of diseases, each with
unique challenges and treatment considerations. Thus, management of the patient with an adrenal tumor must be individualized,
incorporating the patients expectations, the surgeons experience,
and the best available evidence to guide decision-making.
The majority of adrenal tumors are asymptomatic and do
not require treatment, identified in 2.3% of patients at autopsy.3
Despite a consistent prevalence at autopsy over the past several
decades, the annual volume of adrenalectomies has nearly doubled
in the United States from 3241 in 1998 to 5323 in 2006.4 The vast
majority of this increase occurred in patients with benign diseases
of the adrenal, predominantly adenomas. More concerning, the
rate of major postoperative complications increased from 5.9% to

EVALUATION OF ADRENAL
INCIDENTALOMAS
As cross-sectional imaging has become increasingly sophisticated
and more patients are investigated for a variety of abdominal
symptoms; incidentally detected adrenal tumors (adrenal incidentalomas) have become a common diagnostic challenge. An
effective management strategy requires the surgeon to consider
four issues: (1) Is the tumor of cortical or medullary origin? (2) Is
the tumor benign or malignant? (3) Is the tumor primary or metastatic? (4) Is the tumor functional or nonfunctional? If the physician is able to confidently reconcile these four considerations, a
clear pathway for management may be developed.
1. What are the indications for resection of an adrenal
incidentaloma?
The neglected question is whether the anticipated benefits of
resection outweigh the risks associated with operative intervention. There are two primary indications for adrenalectomy: malignant lesions (or sufficient concern that a tumor is malignant) and
functional tumors.
537

PMPH_CH67.indd 537

5/22/2012 5:46:47 PM

538

Surgery: Evidence-Based Practice

Several single-center series have attempted to identify factors


that can confidently discriminate benign from malignant tumors
of the adrenal, including patient characteristics, laboratory investigations, and radiographic features. Unfortunately, even with the
most comprehensive evaluation possible, the ability to identify
malignant tumors is poor. A major limitation in interpreting
studies of adrenocortical cancer is the variability in definitions
of malignancy between studies. Histopathology alone is a poor
discriminator of malignancy and the only definitive criteria for
malignancy are the distant metastasis or local invasion. Several
classification systems have been developed based on histopathologic criteria found to be associated with tumors that had metastasized or had locally recurred; the most utilized being the Weiss
system.5 In this classification system, nine criteria are assessed
including nuclear grade, mitotic rate, atypical mitoses, clear cells,
architecture, necrosis, and invasion of venous, sinusoidal, or capsular
structures. Irrespective of the histopathologic characteristics, the
only definitive evidence of malignancy remains the presence of
metastatic disease.
Clinically, patients who present with pain, weight loss, anemia, fever, virilization, or feminization are more likely to have
malignancy.6,7 Patients with these symptoms should be offered
exploration if the adrenal tumor appears resectable on imaging
studies. (Grade C recommendation)
The most consistent radiologic characteristic identified that
correlates with the risk of malignancy is size. In one institutional
series of 51 adrenalectomies for nonfunctional incidentalomas,
adrenocortical carcinoma was identified in only three patients
(6%).8 In this cohort of patients, the size was the only significant
characteristic associated with malignancy. The three carcinomas
in this group measured 3.8, 6.1, and 8.1 cm, whereas the mean
size of the benign adenomas was 3.9 cm. However, malignancy in
these patients was defined by the histopathologic criteria and was
not confirmed by metastatic disease.
Another single-institution series included 342 patients with
incidentalomas of whom 55 underwent adrenalectomy.9 Four
patients were diagnosed with adrenocortical carcinoma based on
histopathology, with the smallest tumor measuring 5 cm. A minimum of 1 year of follow-up was obtained in 88% of patients who
did not undergo operation, with no patients exhibiting signs of
clinical progression to suggest malignancy.
In a multicenter retrospective analysis, the Italian Society of
Endocrinology reported data collected from 26 referral centers
over a 15 year period.10 The study included 1004 patients with
adrenal incidentalomas, including 388 who underwent adrenalectomy. The prevalence of adrenocortical carcinoma based on histopathologic assessment was 12% (47 patients). Although the size of
the tumor was significantly associated with the risk of malignancy,
four adrenocortical tumors in this study measured less than 4 cm.
The authors concluded that a cutoff of 5 cm maximized the area
under the ROC curve; however, this strategy yields a sensitivity
of only 81% with a specificity of 63%. Sensitivity increased to 93%
with a 4 cm cutoff but specificity fell to 42%.
Due to the inability of size alone to accurately discriminate
between benign and malignant tumors, radiologists have examined other characteristics that may be helpful. The simplest of
these characteristics is the measurement of noncontrast CT attenuation (Hounsfield Units). In a series of 290 patients, the mean
value in adenomas or cases of adrenal hyperplasia was 16.2, significantly less than the mean values in adrenocortical cancers (36.9),
metastases (39.2), and pheochromocytomas (38.6).11 In this group

PMPH_CH67.indd 538

of patients, a management strategy of resecting tumors greater


than 4 cm or with Hounsfield Unit measurement greater than 20
would yield a sensitivity of 100% and a specificity of 42.1% for one
of these three conditions.
Smaller studies have measured the washout characteristics
of tumors at contrast-enhanced CT.12,13 These studies concluded
that the optimal threshold for the relative percentage washout is
40% to 50%, with all malignant lesions demonstrating less than
this and nearly all benign tumors demonstrating more than 50%
enhancement loss. These results have not been validated in larger
studies.
Due to the limited ability of CT scanning to differentiate
between benign and malignant adrenal tumors, other imaging
modalities have been explored. Consistent MRI features of adrenocortical carcinoma include internal hemorrhage, central necrosis, and peripheral enhancing nodules.14 In a study of 229 adrenal
masses in 204 patients, MRI achieved a sensitivity of 89%, a specificity of 99%, and an accuracy of 94% at distinguishing malignancy based on the histopathologic criteria.15
In patients with indeterminate tumors after cross-sectional
imaging with CT and MRI, FDG-PET/CT scanning may assist
in better characterizing malignant potential. Among 37 patients
with no personal history of cancer, PET/CT achieved a sensitivity
of 100% and a specificity of 86% for malignancy.16 PET/CT may be
particularly helpful in patients who have a prior history of nonadrenal malignancy. In a study of 80 adrenal tumors in patients with
prior cancers, PET was 93% sensitive and 96% specific for adrenal
metastases.17 Visual interpretation was as accurate as quantitative
analysis with SUV in both of these studies.
In summary, several single-center studies and one multicenter study identify a strong association between size of adrenal
tumor and risk of malignancy. Additional imaging characteristics
on CT including noncontrast attenuation and contrast-enhanced
washout appear to provide additional, complementary information. MRI and FDG-PET/CT may assist in better characterizing
indeterminate lesions. Tumors greater than 4 cm in size; with
Hounsfield Units greater than 20 or less than 40% to 50% washout at contrast-enhanced CT; with internal hemorrhage, central
necrosis, or peripheral enhancing nodules on MRI; or with visual
adrenal uptake on PET/CT should be considered for resection.
(Grade C recommendation)
Independent of the risk of malignancy, patients with symptomatic, functional adrenal tumors are typically offered resection.
In this group of patients, adrenalectomy offers the only curative
option and is appropriate. The role of adrenalectomy in patients
with asymptomatic, functional adrenocortical tumors is less clear.
One RCT compared surgical with conservative management in
patients with subclinical Cushing syndrome (a cortisol-producing
tumor).18 Patients were eligible for the study if they had an adrenal
incidentaloma, a positive dexamethasone suppression test, and no
clinical signs of hormonal excess. The trial randomized 23 patients
to laparoscopic adrenalectomy and 22 patients to observation, of
whom 3 patients crossed over to the surgical group due to their
adrenal masses increasing in size during follow-up. Urinary cortisol and the dexamethasone suppression test normalized in all
patients who underwent adrenalectomy. Furthermore, patients
who underwent surgery experienced improvements in diabetes
(5 of 8 patients), hypertension (12 of 18 patients), hyperlipidemia
(3 of 8 patients), and obesity (3 of 6 patients). No patients in the
observation group experienced improvements in these comorbidities, and several worsened.

5/22/2012 5:46:48 PM

Adrenocortical Tumors and Incidentalomas

In summary, patients with an adrenal incidentaloma and


subclinical Cushing syndrome should consider adrenalectomy,
particularly if they suffer from diabetes, hypertension, hyperlipidemia, or obesity. (Grade B recommendation)
2. What is the most cost-effective approach to adrenal
incidentalomas?
Given the paucity of evidence to guide surgeons in the diagnostic
evaluation of patients with adrenal incidentalomas and the current economic climate, the costs of various approaches should be a
consideration to clinicians managing these patients. When several
management strategies appear to yield similar clinical outcomes,
a reasonable approach is to select the strategy that is least costly.
Unfortunately, a dearth of high-quality research exists to determine the relative costs of selected management strategies. Our
search for research examining the cost-effectiveness of treatment
algorithms for adrenal incidentalomas yielded only three publications, all published in nonsurgical subspecialty journals.
The three decision analyses that we identified reached different conclusions, depending on the management strategies compared, underlying assumptions, and healthcare systems. Kievit
concluded that in the Dutch system, the most cost-effective strategy was to screen patients for medullary secreting tumors using
urinary metanephrines, with more extensive investigations such
as hormonal testing, imaging, and biopsy reserved for subgroups
of patients.19 In an analysis from Italy, MRI in conjunction with
image-guided fine needle aspiration biopsy was felt to be most
cost-effective in patients with adrenal incidentalomas greater than
2 cm.20 From the American perspective, Dwamena et al.21 concluded that a strategy involving one-time evaluation with 131I-6
beta-iodomethylnorcholesterol (NP-59) scintigraphy yielded the
optimal combination of high diagnostic accuracy and low cost.
These markedly opposing conclusions highlight the limitations of cost-effectiveness analyses and the importance of carefully defining the context of the decision-maker. Based on the
existing evidence, no recommendation can be made with respect
to the most cost-effective approach to patients with adrenal incidentalomas. However, it is important to emphasize that the 64%
increase in the use of adrenalectomy, without proven evidence of
function or malignancy, is costly and accompanied by unnecessary complications. Common sense dictates that clinicians should
only employ investigations likely to yield clinically useful information and that the frequency of surveillance should be kept to
a minimum. Further contemporary economic analyses of this
increasingly prevalent diagnostic challenge are needed.
With respect to the operative approach, one study compared
the cost of transperitoneal, lateral retroperitoneal, and posterior
laparoscopic adrenalectomy in the American system.22 The average costs ranged from $2850 to $3219 with no significant difference between the approaches.

OPERATIVE APPROACH TO
ADRENAL TUMORS
3. What is the optimal operative approach to adrenal tumors?
Once a decision has been made to attempt resection of an adrenal
tumor, the surgeon must select one of several operative approaches
to the adrenal gland. For relatively small (less than 78 cm) adrenocortical tumors felt to be benign, the historically preferred approach

PMPH_CH67.indd 539

539

was a posterior adrenalectomy through the bed of the 11th or 12th


rib. Over the past two decades, laparoscopy has emerged as the overwhelmingly preferred approach, supported by several retrospective
single-center comparative studies.23-25 Taken together, these studies suggest that the laparoscopic approach yields longer operative
time, less blood loss, lower postoperative narcotic requirement,
shorter postoperative hospital stay, and less overall morbidity than
open adrenalectomy. These findings were replicated in a population-based study from the National Surgical Quality Improvement
Program database.26 Therefore, laparoscopic adrenalectomy is the
preferred approach for patients with small adrenocortical tumors
felt to be benign. (Grade B recommendation)
In a small prospective RCT, Tiberio et al.27 compared laparoscopic with open adrenalectomy for sporadic pheochromocytoma.
Although the number of patients was small (22 total), no significant differences were seen in hemodynamic instability, operative
time, or morbidity. The laparoscopic procedure yielded less blood
loss and a shorter length of hospitalization. Although patients
with pheochromocytoma must be carefully monitored intraoperatively for hemodynamic instability, laparoscopy appears to be
a safe operative approach. (Grade B recommendation)
During the development of laparoscopic adrenalectomy,
surgeons were appropriately cautious applying these results
to patients with malignancies. Over time, some surgeons have
expanded their indications for this operative approach to selected
patients with metastases to the adrenal or primary adrenocortical
carcinoma. Strong et al.28 reported their experience with 63 open
adrenalectomies and 31 laparoscopic adrenalectomies for isolated
metastases to the adrenal. In their series, the laparoscopic approach
yielded shorter operative time, lower blood loss, shorter length of
hospital stay, and fewer total complications than the open approach
with no difference in margin status, local recurrence, or overall survival. In a multicenter study from Germany, Brix et al.29 compared
laparoscopic with open resection in 152 patients with stage IIII
adrenocortical carcinoma less than 10 cm in maximal diameter.
Laparoscopy was attempted in 35 patients, with 12 patients converted to an open approach. There was no difference in the recurrencefree or in the disease-specific survival in these selected patients. In
summary, laparoscopic resection is an alternative to open adrenalectomy in selected patients with metastases to the adrenal or to the
early adrenocortical carcinoma. (Grade C recommendation)
Two approaches to laparoscopic adrenalectomy have been compared in a RCT.30 Transperitoneal adrenalectomy yielded similar
operative time, blood loss, analgesic requirements, hospital stay, and
complication rate as retroperitoneal adrenalectomy. Transperitoneal
and retroperitoneal laparoscopic adrenalectomy can be performed
safely and effectively. (Grade B recommendation)

ADRENOCORTICAL CARCINOMA
4. What is the role of adjuvant therapy for completely resected
adrenocortical carcinoma?
Adrenocortical carcinoma is a rare tumor that most commonly
presents with advanced disease precluding complete resection.
For patients with localized disease, surgical resection offers the
only chance of cure, with median survival of 101 months for those
with early stage I or II disease.31 In contrast, patients with incomplete primary resection have a uniformly poor prognosis, with
median survival of 12 months.

5/22/2012 5:46:48 PM

540

Surgery: Evidence-Based Practice

Mitotane is an adrenocorticolytic drug that has efficacy in


patients with ACC. For patients with unresectable or incompletely
resected functional adrenocortical carcinoma, primary treatment
with mitotane is indicated. The main benefit in this setting is a
reduction in the excess hormone production and a reduction in the
symptoms of hypercortisolism in up to 75% of patients.32 Unfortunately, the duration of efficacy is limited and survival does not
appear to be improved compared with untreated patients.33 For
the patient with a nonfunctional tumor, mitotane should be considered. However, many patients experience significant side effects
from higher doses of mitotane, requiring a balance between the
minimal uncertain benefit and the significant side effects.
The role of adjuvant mitotane in patients undergoing complete
resection of adrenocortical carcinoma is controversial, with no
RCTs to guide the clinicians. Several small series have been published with conflicting results due to variability in patient characteristics and dosing regimens in individual studies. The strongest
evidence in support of adjuvant mitotane is derived from a multicenter retrospective analysis including 177 patients treated at 55
centers in Italy and Germany.34 The authors compared 47 patients
who received adjuvant mitotane in Italy for stage IIII carcinoma
with 55 Italian patients and 75 German patients who did not. The
groups were relatively well balanced with the exception of a higher
frequency of older patients with earlier stage disease in the German group than in the other two groups. Treatment with adjuvant
mitotane was associated with prolonged recurrence-free survival
(median recurrence-free survival 42 months compared with 10 to
25 months in the control groups). Mitotane treatment was well tolerated with a temporary dose reduction needed in 13% of patients.
This study is limited by the inherent selection bias in a retrospective study, however, given the poor prognosis in patients with
adrenocortical carcinoma and the lack of other efficacious agents;
this study recommends adjuvant mitotane therapy to patients who
have undergone resection of adrenocortical carcinoma, regardless
of stage or completeness of resection. (Grade B recommendation)
Several investigators have explored the role of mitotane in
combination with other agents, but none have demonstrated consistent improvements in outcomes.
The role of adjuvant radiation therapy in patients undergoing
complete resection of adrenocortical carcinoma is controversial.
Polat et al.35 reviewed 10 studies published between 1960 and 2008
including 64 patients. Rates of local control varied widely from
0% to 86% without clear indications or complete information on
the completeness of resection in these studies. Given the lack of
evidence, adjuvant radiation therapy is not recommended.
5. What is the optimal treatment of persistent or recurrent
adrenocortical carcinoma?
Recurrent or metastatic adrenocortical carcinoma is common
even for patients who undergo complete resection. Given the paucity of evidence to guide the management of primary disease, it is
not surprising that little data exist to support strategies for persistent or for recurrent adrenocortical carcinoma. In the absence
of strong evidence, surgeons who manage patients with adrenocortical cancer will be faced with this dilemma and must make
treatment recommendations based on the limited data available.
Treatment modalities including surgery, radiation therapy, radiofrequency ablation (RFA), and chemotherapy should be tailored
to the individual patient depending on the extent of disease and
goals of therapy.

PMPH_CH67.indd 540

The largest surgical series includes 47 patients who underwent


resection of locally recurrent or distant metastatic adrenocortical
carcinoma at Memorial Sloan-Kettering Cancer Center.31 As in
primary resection, the most important prognostic factor in these
patients was the ability to achieve a complete resection. Patients
who had a complete second resection had a median survival of
74 months, compared with 16 months in patients who underwent
an incomplete second resection. Similar results were observed in
a retrospective comparison of 18 patients treated with chemotherapy and 15 patients treated with resection plus chemotherapy.36 In
addition to the possible survival benefit, resection provides excellent palliation in patients with symptomatic steroid production
that cannot be controlled with medical therapy.
Other local therapies including external beam radiation and
RFA may have a role in patients with unresectable recurrent disease, particularly in those with local symptoms. In several small
studies, the administration of mitotane to previously untreated
patients with unresectable disease results in tumor response in up
to one-third of patients, but no improvement in survival has been
consistently reported.
In summary, patients with locally recurrent or metastatic disease should be offered reoperation if they have potentially completely resectable disease and are in otherwise good health. (Grade C
recommendation)

METASTASES TO THE ADRENAL


6. What is the optimal management of metastases to the
adrenal?
Adrenal metastases occur most commonly in patients with lung,
renal, breast, and gastrointestinal carcinoma. 37,38 Only 4% of
patients with adrenal metastases develop symptoms, usually due to
large tumors compressing adjacent structures.37 Most patients with
adrenal metastases present with multiple synchronous metastases
at other sites, and in approximately half of patients both adrenal
glands are involved.37 Local therapies have no role in asymptomatic
patients with extra-adrenal synchronous metastases; these patients
may be offered palliative systemic therapy based on the primary
site or supportive care.
When metastatic disease is limited to the adrenal gland,
local treatment may be associated with a survival advantage.
Several small retrospective studies have examined the role of
adrenalectomy in patients with isolated adrenal metastases, most
commonly from nonsmall cell lung cancer.37,39-41 In these highly
selected patients, adrenalectomy could be safely performed with
minimal morbidity and short length of hospital stay. Resection
was associated with an improved survival in these patients compared with patients who underwent nonoperative management,
although patient selection was likely a primary determinant of
this difference. Overall 3-year survival in patients who underwent adrenalectomy was approximately 30%, compared with no
long-term survivors in patients treated nonoperatively.28,37,39,41 In a
systematic review comparing adrenalectomy for isolated synchronous (6 months or less disease-free interval) versus metachronous (greater than 6 months) metastases from nonsmall cell lung
cancer, the median overall survival was shorter for patients with
synchronous metastasis (12 vs. 31 months, p = .02).42 However, a
durable long-term survival was achieved in approximately 25% of
patients in both groups. Therefore, adrenalectomy is appropriate in

5/22/2012 5:46:48 PM

Adrenocortical Tumors and Incidentalomas

highly selected patients with isolated adrenal metastasis. (Grade C


recommendation)
Several small case series have reported clinicians experiences
with less invasive percutaneous ablative techniques for adrenal metastasis, including RFA,43 microwave ablation,44 chemical
ablation,45 and RFA combined with arterial chemoembolization
(for hepatocellular carcinoma).46 Although these reports demonstrate promising early local results, the data are far too limited to
generate meaningful conclusions. Until further evidence emerges,
percutaneous ablation of adrenal metastasis should be viewed as
experimental.
For patients with unresectable or widely metastatic symptomatic adrenal metastasis, radiotherapy may improve pain. In
one study of 16 symptomatic patients treated with external beam
radiotherapy, the overall response rate was 75% and all patients
who responded experienced moderate to complete pain relief.47 In
eight of these patients, the pain relief persisted until death. External

541

beam radiation is an option for patients with unresectable or


widely metastatic symptomatic adrenal metastasis. (Grade C
recommendation)

SUMMARY
Management of patients with adrenocortical tumors provides
many challenges to surgeons due to the rarity and paucity of
evidence to guide decision-making. Treatment must be individualized, with patients fully aware of the uncertainty and explicitly involved in the decision-making. Further research is sorely
needed, particularly in the areas of cost-effectiveness and in the
assessment of new technologies including percutaneous ablation. Until then, surgeons must rely on the combination of scant
evidence, clinical intuition, and biologic rationale to guide their
patients with adrenocortical tumors.

Clinical Question Summary


Question

Answer

1 What are the indications


for resection of an
adrenal incidentaloma?

Patients who present with pain, weight loss,


anemia, fever, virilization, or feminization
should be offered exploration if the
adrenal tumor appears resectable.
Tumors greater than 4 cm in size; with
Hounsfield Units greater than 20 or less
than 40% to 50% washout at contrastenhanced CT; with internal hemorrhage,
central necrosis, or peripheral enhancing
nodules on MRI; or with visual adrenal
uptake on PET/CT should be considered
for resection.
Patients with an adrenal incidentaloma and
subclinical Cushing syndrome should
consider adrenalectomy, particularly if
they suffer from diabetes, hypertension,
hyperlipidemia, or obesity.

2 What is the most costeffective approach to


adrenal incidentalomas?

No Level 13 evidences exist.

3 What is the optimal


operative approach to
adrenal tumors?

Laparoscopic adrenalectomy is the preferred


approach for patients with small
adrenocortical tumors felt to be benign.
Although patients with pheochromocytoma
must be carefully monitored
intraoperatively for hemodynamic
instability, laparoscopy appears to be a
safe operative approach.
Laparoscopic resection is an alternative to
open adrenalectomy in selected patients
with metastases to the adrenal or small
suspected adrenocortical carcinoma.
Transperitoneal and retroperitoneal
laparoscopic adrenalectomy can be
performed safely and effectively.

Levels of
Evidence

Grade of
Recommendation

References

6, 7

34

8-17

18

23-26

27

28, 29

30

(Continued)

PMPH_CH67.indd 541

5/22/2012 5:46:48 PM

542

Surgery: Evidence-Based Practice

(Continued)
Question

Answer

4 What is the role of


adjuvant therapy for
completely resected
adrenocortical
carcinoma?

Patients who have undergone resection of


proven adrenocortical carcinoma should
be considered for adjuvant mitotane
therapy.

34

5 What is the optimal


treatment of
persistent or recurrent
adrenocortical
carcinoma?

Patients with locally recurrent or metastatic


disease should be offered reoperation
if they have potentially completely
resectable disease and are in otherwise
good health.

31, 36

6 What is the optimal


management of
metastases to the
adrenal?

Adrenalectomy should be offered to


selected patients with isolated adrenal
metastasis.
External beam radiation is an option for
patients with locally unresectable solitary
or multisite metastatic symptomatic
adrenal metastasis.

28, 37, 39-42

47

REFERENCES
1. Farrokhyar F, Karanicolas PJ, Thoma A, et al. Randomized controlled trials of surgical interventions. Ann Surg. 2010;251(3):
409-416.
2. Karanicolas PJ, Montori VM, Devereaux PJ, et al. A new
Mechanistic-Practical Framework for designing and interpreting randomized trials. J Clin Epidemiol. 2009;62(5):479-484.
3. Barzon L, Sonino N, Fallo F, et al. Prevalence and natural history of
adrenal incidentalomas. Eur J Endocrinol. 2003;149(4):273-285.
4. Murphy MM, Witkowski ER, Ng SC, et al. Trends in adrenalectomy:
A recent national review. Surg Endosc. 2010;24(10):2518-2526.
5. Weiss LM. Comparative histologic study of 43 metastasizing
and nonmetastasizing adrenocortical tumors. Am J Surg Pathol.
1984;8(3):163-169.
6. Pommier RF, Brennan MF. An eleven-year experience with adrenocortical carcinoma. Surgery. 1992;112(6):963-970; discussion
970-971.
7. Soreide JA, Brabrand K, Thoresen SO. Adrenal cortical carcinoma in Norway, 1970-1984. World J Surg. 1992;16(4):663-667;
discussion 668.
8. Grogan RH, Mitmaker E, Vriens MR, et al. Adrenal incidentaloma: Does an adequate workup rule out surprises? Surgery. 2010;
148(2):392-397.
9. Herrera MF, Grant CS, van Heerden JA, et al. Incidentally discovered adrenal tumors: An institutional perspective. Surgery. 1991;
110(6):1014-1021.
10. Mantero F, Terzolo M, Arnaldi G, et al. A survey on adrenal
incidentaloma in Italy. Study Group on Adrenal Tumors of the
Italian Society of Endocrinology. J Clin Endocrinol Metab. 2000;
85(2):637-644.
11. Hamrahian AH, Ioachimescu AG, Remer EM, et al. Clinical
utility of noncontrast computed tomography attenuation value
(hounsfield units) to differentiate adrenal adenomas/hyperplasias
from nonadenomas: Cleveland Clinic experience. J Clin Endocrinol Metab. 2005;90(2):871-877.

PMPH_CH67.indd 542

Levels of
Evidence

Grade of
Recommendation

References

12. Pena CS, Boland GW, Hahn PF, et al. Characterization of indeterminate (lipid-poor) adrenal masses: Use of washout characteristics at contrast-enhanced CT. Radiology. 2000;217(3):798-802.
13. Szolar DH, Korobkin M, Reittner P, et al. Adrenocortical carcinomas and adrenal pheochromocytomas: Mass and enhancement
loss evaluation at delayed contrast-enhanced CT. Radiology. 2005;
234(2):479-485.
14. Schlund JF, Kenney PJ, Brown ED, et al. Adrenocortical carcinoma: MR imaging appearance with current techniques. J Magn
Reson Imaging. 1995;5(2):171-174.
15. Honigschnabl S, Gallo S, Niederle B, et al. How accurate is MR
imaging in characterisation of adrenal masses: update of a longterm study. Eur J Radiol. 2002;41(2):113-122.
16. Tessonnier L, Sebag F, Palazzo FF, et al. Does 18F-FDG PET/CT add
diagnostic accuracy in incidentally identified non-secreting adrenal
tumours? Eur J Nucl Med Mol Imaging. 2008;35(11):2018-2025.
17. Jana S, Zhang T, Milstein DM, et al. FDG-PET and CT characterization of adrenal lesions in cancer patients. Eur J Nucl Med
Mol Imaging. 2006;33(1):29-35.
18. Toniato A, Merante-Boschin I, Opocher G, et al. Surgical versus
conservative management for subclinical Cushing syndrome in
adrenal incidentalomas: A prospective randomized study. Ann
Surg. 2009;249(3):388-391.
19. Kievit J, Haak HR. Diagnosis and treatment of adrenal incidentaloma. A cost-effectiveness analysis. Endocrinol Metab Clin North
Am. 2000;29(1):69-90, viii-ix.
20. Lumachi F, Basso SM, Borsato S, et al. Role and cost-effectiveness
of adrenal imaging and image-guided FNA cytology in the management of incidentally discovered adrenal tumours. Anticancer
Res. 2005;25(6C):4559-4562.
21. Dwamena BA, Kloos RT, Fendrick AM, et al. Diagnostic
evaluation of the adrenal incidentaloma: Decision and costeffectiveness analyses. J Nucl Med. 1998;39(4):707-712.
22. Farres H, Felsher J, Brodsky J, et al. Laparoscopic adrenalectomy:
A cost analysis of three approaches. J Laparoendosc Adv Surg Tech
A. 2004;14(1):23-26.

5/22/2012 5:46:48 PM

Adrenocortical Tumors and Incidentalomas

23. Brunt LM, Doherty GM, Norton JA, et al. Laparoscopic adrenalectomy compared to open adrenalectomy for benign adrenal
neoplasms. J Am Coll Surg. 1996;183(1):1-10.
24. Hallfeldt KK, Mussack T, Trupka A, et al. Laparoscopic lateral
adrenalectomy versus open posterior adrenalectomy for the treatment of benign adrenal tumors. Surg Endosc. 2003;17(2):264-267.
25. MacGillivray DC, Shichman SJ, Ferrer FA, et al. A comparison of open vs laparoscopic adrenalectomy. Surg Endosc. 1996;
10(10):987-990.
26. Lee J, El-Tamer M, Schifft ner T, et al. Open and laparoscopic
adrenalectomy: Analysis of the National Surgical Quality Improvement Program. J Am Coll Surg. 2008;206(5):953-959; discussion 959-961.
27. Tiberio GA, Baiocchi GL, Arru L, et al. Prospective randomized
comparison of laparoscopic versus open adrenalectomy for sporadic pheochromocytoma. Surg Endosc. 2008;22(6):1435-1439.
28. Strong VE, DAngelica M, Tang L, et al. Laparoscopic adrenalectomy for isolated adrenal metastasis. Ann Surg Oncol. 2007;
14(12):3392-3400.
29. Brix D, Allolio B, Fenske W, et al. Laparoscopic versus open
adrenalectomy for adrenocortical carcinoma: Surgical and oncologic outcome in 152 patients. Eur Urol. 2010;58(4):609-615.
30. Rubinstein M, Gill IS, Aron M, et al. Prospective, randomized
comparison of transperitoneal versus retroperitoneal laparoscopic
adrenalectomy. J Urol. 2005;174(2):442-445; discussion 445.
31. Schulick RD, Brennan MF. Long-term survival after complete
resection and repeat resection in patients with adrenocortical
carcinoma. Ann Surg Oncol. 1999;6(8):719-726.
32. Lubitz JA, Freeman L, Okun R. Mitotane use in inoperable adrenal cortical carcinoma. JAMA. 1973;223(10):1109-1112.
33. Macfarlane DA. Cancer of the adrenal cortex; the natural history, prognosis and treatment in a study of fift y-five cases. Ann R
Coll Surg Engl. 1958;23(3):155-186.
34. Terzolo M, Angeli A, Fassnacht M, et al. Adjuvant mitotane treatment for adrenocortical carcinoma. N Engl J Med. 2007;356(23):
2372-2380.
35. Polat B, Fassnacht M, Pfreundner L, et al. Radiotherapy in adrenocortical carcinoma. Cancer. 2009;115(13):2816-2823.

PMPH_CH67.indd 543

543

36. Jensen JC, Pass HI, Sindelar WF, et al. Recurrent or metastatic
disease in select patients with adrenocortical carcinoma. Aggressive resection vs chemotherapy. Arch Surg. 1991;126(4):457-461.
37. Lam KY, Lo CY. Metastatic tumours of the adrenal glands: A
30-year experience in a teaching hospital. Clin Endocrinol (Oxf).
2002;56(1):95-101.
38. Uberoi J, Munver R. Surgical management of metastases to the
adrenal gland: Open, laparoscopic, and ablative approaches. Curr
Urol Rep. 2009;10(1):67-72.
39. Higashiyama M, Doi O, Kodama K, et al. Surgical treatment of
adrenal metastasis following pulmonary resection for lung cancer: Comparison of adrenalectomy with palliative therapy. Int
Surg. 1994;79(2):124-129.
40. Kim SH, Brennan MF, Russo P, et al. The role of surgery in the
treatment of clinically isolated adrenal metastasis. Cancer. 1998;
82(2):389-394.
41. Luketich JD, Burt ME. Does resection of adrenal metastases
from non-small cell lung cancer improve survival? Ann Thorac
Surg. 1996;62(6):1614-1616.
42. Tanvetyanon T, Robinson LA, Schell MJ, et al. Outcomes of
adrenalectomy for isolated synchronous versus metachronous
adrenal metastases in non-small-cell lung cancer: A systematic
review and pooled analysis. J Clin Oncol. 2008;26(7):1142-1147.
43. Mouracade P, Dettloff H, Schneider M, et al. Radio-frequency
ablation of solitary adrenal gland metastasis from renal cell carcinoma. Urology. 2009;74(6):1341-1343.
44. Wang Y, Liang P, Yu X, et al. Ultrasound-guided percutaneous
microwave ablation of adrenal metastasis: Preliminary results.
Int J Hyperthermia. 2009;25(6):455-461.
45. Xiao YY, Tian JL, Li JK, et al. CT-guided percutaneous chemical ablation of adrenal neoplasms. AJR Am J Roentgenol. 2008;
190(1):105-110.
46. Yamakado K, Anai H, Takaki H, et al. Adrenal metastasis from
hepatocellular carcinoma: Radiofrequency ablation combined
with adrenal arterial chemoembolization in six patients. AJR Am
J Roentgenol. 2009;192(6):W300-W305.
47. Soffen EM, Solin LJ, Rubenstein JH, et al. Palliative radiotherapy
for symptomatic adrenal metastases. Cancer. 1990;65(6):1318-1320.

5/22/2012 5:46:48 PM

CHAPTER 68

Pheochromocytoma
Raymon H. Grogan and Quan-Yang Duh

INTRODUCTION

high ratio of studies to cases demonstrates how interesting the


physiology of these tumors is to physicians, and bodes well for
the future of evidence-based medical practice for this rare, but
fascinating disease.

Pheochromocytomas are tumors that arise in neuroectodermal


chromaffin cells within the adrenal medulla that cause excess
catecholamine secretion. When these tumors develop in neuroectodermal cells outside the adrenal gland, they are known
as paragangliomas. Studies from the Swedish National Cancer
Registry and the Mayo Clinic are the most commonly quoted
studies on pheochromocytoma incidence. They show that pheochromocytomas occur with an incidence of two to eight people
per million per year.1,2 Pheochromocytoma is considered to be a
rare disease in the United States according to the Office of Rare
Diseases Research at the National Institutes of Health. Studying
and making any evidence-based recommendations for pheochromocytoma is difficult because it is such a rare disease.
New developments in genetic analysis, laboratory science,
imaging technology, and surgical techniques over the last 2
decades have significantly altered our approach to the diagnosis,
management, and treatment of these tumors. In 2001, it was discovered that mutations of the succinate dehydrogenase subunit B
gene (SDHB) were associated with pheochromocytoma formation.3 This lead to a complete redefinition of our understanding of the genetics of pheochromocytomas that is still ongoing.
Laboratory techniques have continually improved over the last 10
years and newer assays have been developed that are more sensitive and specific than those that used to be considered the gold
standard. Similarly, imaging technology has evolved, particularly
in the ability to detect functional catecholamine-producing tissues. Laparoscopic resection of adrenal tumors was first reported
in 1992, and since that time it has become the standard of care
for the majority of adrenal tumors.4 Laparoscopic adrenalectomy
represents a paradigm shift in the way adrenal tumors, and pheochromocytomas, are treated because the postoperative recovery
time and pain are significantly shorter than with open surgery.
A quick PubMed search reveals over 11,500 studies that address
pheochromocytoma or paraganglioma. In comparison with that,
based on current population data, there are likely no more than
6800 new cases of pheochromocytoma worldwide per year. The

GENETICS
1. What genetic mutations are associated with pheochromocytoma development, and how often are pheochromocytomas
caused by inherited mutations?
Until recently, it has always been accepted that 10% of pheochromocytomas are caused by a hereditary genetic mutation.
This presumption was based on multiple case series observations (Level 4 evidence) starting in the 1960s that found 10% of
pheochromocytomas were diagnosed in patients who had one of
three genetic syndromes: von-Hippel Lindau (VHL) caused by
mutations in the VHL gene, Multiple Endocrine Neoplasia Type
2 (MEN 2) caused by mutations in the RET gene, and neurofibromatosis type 1 (NF1) caused by mutations in the NF1 gene.
This dogma was challenged in 2001 when a germline mutation
in the SDHB gene was found to be associated with pheochromocytoma development in the absence of one of these three genetic
syndromes.3 Since 2001, at least seven more gene mutations
(SDHA, SDHAF1, SDHAF2, SDHC, SDHD, TMEM127, and KIF1BBeta) have been linked to pheochromocytoma formation in
patients with no obvious personal or family history of a genetic
syndrome other than pheochromocytoma formation.5,6 Debate
remains as to the exact prevalence of germline mutations associated with pheochromocytoma. Shortly after the discovery of the
association between SDHB mutations and pheochromocytoma, a
nonconsecutive cohort study (Level 3b evidence) of 271 patients
from a German cancer registry found that 24% of nonsyndromic,
apparently sporadic pheochromocytoma patients, had germline
mutations that were responsible for pheochromocytoma development.7 Similar studies from other European cancer registries
have found varying results. Apparent sporadic pheochromocytomas
544

PMPH_CH68.indd 544

5/22/2012 5:47:22 PM

Pheochromocytoma

were found to have causative germline mutations with a frequency of 7.5% in the Netherlands8 (Level 3b evidence), 10.8% in
Italy 9 (Level 1b evidence), 11.9% in France10 (Level 3b evidence),
and 14% in Spain11 (Level 3b evidence). All of these studies
attempted to exclude patients with a genetic syndrome or family history of pheochromocytoma. The variation in mutation
frequency is likely caused by the ethnic and geographic population differences in the studies as well as in the inclusion of some
patients with genetic syndromes that were clinically unrecognizable at the time of initial presentation. Regardless of the variation
in the frequency of mutations found, all these studies support
the fi nding that the prevalence of germline, inheritable genetic
mutations that cause pheochromocytoma, is much higher than
the originally identified 10% prevalence. Combining the pheochromocytomas caused by genetic syndrome mutations with
apparently sporadic pheochromocytomas caused by germline
mutations, the overall rate of inheritable genetic mutations in
pheochromocytoma patients ranges from 17.5% to 34%. Th is
does not take into account the more recently identified mutations
SDHA, SDHAF1, SDHAF2, TMEM127, and KIF1BBeta because
these mutation rates have yet to be studied.
Answer: Pheochromocytomas are caused by inherited genetic
mutations in the RET, VHL, NF1, SDHB, SDHC, or SDHD genes
between 17.5% and 34% of the time (Grade B recommendation).
However, the overall rate of inherited mutations in pheochromocytoma patients is likely to be higher because the incidence and
prevalence of SDHA, SDHAF1, SDHAF2, TMEM127, and KIF1BBeta mutations are unknown.

DIAGNOSIS/IMAGING
2. What is the best laboratory test to diagnose pheochromocytoma?
Excess catecholamine production is the hallmark of pheochromocytomas, and is responsible for the morbidity and mortality
associated with these tumors. Catecholamines are amine compounds that have a catechol group attached to them. The three
most common catecholamines in the human body are epinephrine, norepinephrine, and dopamine. The production and metabolism of catecholamines is a complex process that is not specific
to the adrenal medulla and this complexity and nonspecificity has
resulted in the development of multiple serum and urine tests for
both the parent catecholamines and their metabolites.
Production of catecholamines occurs in the sympathetic
nervous system neurons as well as in the adrenal medulla. Measurement of plasma or urine catecholamines was one of the first
diagnostic tests for pheochromocytoma. However, both sympathetic neurons and the adrenal medulla contribute to the circulating levels of these parent catecholamine compounds with only a
minor percentage of the normal levels being contributed by the
adrenal medulla. Excess catecholamine production by the adrenal
medulla increases the amount of circulating catecholamines dramatically and is what allows catecholamine secretion to be used as
a diagnostic tool. But the contribution of sympathetic neurons to
the circulating levels of catecholamines reduces the sensitivity and
specificity of this test for diagnosing pheochromocytoma. Once
this was understood, tests were developed to detect the downstream metabolites of the parent catecholamines. Catecholamine
metabolites that can be used to diagnose pheochromocytoma are

PMPH_CH68.indd 545

545

vanillylmandelic acid (VMA), free plasma metanephrines, and


fractionated urine metanephrines.
Metabolism and conversion of the parent catecholamines into
downstream metabolites occur by deamination, O-methylation,
and/or sulfate conjugation. These processes occur in multiple locations including the sympathetic neurons, adrenal medulla, liver,
kidney, and/or the gastrointestinal tract. However, the majority of
catecholamine metabolism occurs within the tissue where the catecholamines are produced, that is, the sympathetic neurons and
adrenal medulla.12 These two tissues metabolize catecholamines
via two distinct processes and that is what accounts for the differences in sensitivity and specificity of the various tests that are
currently used to diagnose pheochromocytoma.
Sympathetic neurons deaminate the parent catecholamines to
an intermediate compound 3,4 dihydroxyphenolglycol (DHPG)
using the enzyme monoamine oxidase. Deamination by sympathetic neurons accounts for the majority of DHPG found in
the circulation. The liver converts DHPG in the circulation into
other intermediate compounds and eventually into VMA. Thus,
the majority of VMA normally found in the circulation and in
the urine is a reflection of the sympathetic neuronal production
of catecholamines, rather than catecholamine production from
the adrenal medulla. Circulating catecholamines that have been
secreted by the adrenal medulla can also be converted to DHPG
and eventually VMA by the liver, and is the likely source of
elevated levels of VMA that are associated with pheochromocytomas. Similar to the tests that measure parent catecholamines,
the sensitivity and specificity of VMA to diagnose pheochromocytoma are decreased because VMA is a downstream metabolite
of catecholamines from both the sympathetic neurons and the
adrenal medulla.
The adrenal medulla metabolizes catecholamines by Omethylation of the parent catecholamine compound, a process that
produces metanephrine (from epinephrine), normetanephrine
(from norepinephrine), and methoxytyramine (from dopamine).
The adrenal medulla contains high concentrations of membrane
bound catechol-O-methyltransferase, the enzyme required for
O-methylation of catecholamines. The adrenal medulla is responsible for 94% of elevated levels of metanephrines in the circulation.
O-methylation of catecholamines in the adrenal medulla occurs
continuously and independently of fluctuations in catecholamine
release. This causes a steady state of elevated plasma metanephrines
even if catecholamine production fluctuates. Because the majority
of plasma free metanephrines are continuously produced by the
adrenal medulla, this makes plasma-free metanephrines a highly
sensitive and specific test for pheochromocytoma diagnosis.
Catecholamines and metanephrines are cleared from the circulation by the kidney only after they undergo sulfate conjugation.
The sulfate conjugates remain in the circulation for an extended
period of time before the kidney excretes them. This build-up
makes them a poor serum test for the diagnosis of pheochromocytoma because levels can be very high in the normal circulation.
However, they are excreted in urine at a constant rate that changes
as serum concentration increases, thus making metanephrine
sulfate conjugates an ideal urine test. In humans, sulfotransferase
isoenzyme is responsible for the majority of sulfate conjugation.
The highest concentrations of this enzyme are found in the human
small intestine. Thus, metanephrines are released by the adrenal
medulla into the circulation, undergo sulfate conjugation in the
small intestine, and are then excreted in the urine. Measurement
of urinary fractionated metanephrine sulfate conjugates (urinary

5/22/2012 5:47:23 PM

546

Surgery: Evidence-Based Practice

fractionated metanephrines) is also a highly sensitive and specific test for pheochromocytoma formation. The sulfate conjugation step by the small intestine might make it slightly less reliable
than plasma-free metanephrine measurements because of the temporary changes in gut physiology and sulfate conjugation of
nonadrenal catecholamines. These biochemical pathways suggest
that levels of plasma-free metanephrines are the best diagnostic
choice for pheochromocytoma because the adrenal medulla contains high levels of the enzyme catechol-O-methyltransferase and
thus produce metanephrines. In contrast, the sympathetic neurons contain high levels of monoamine oxidase and almost no
catechol-O-methyltransferase and produce DHPG and VMA.
This theory is supported by clinical studies.
Multiple clinical studies on the sensitivity and specificity of
assays for VMA, catecholamines, plasma-free metanephrines,
and urinary fractionated metanephrines have been conducted,
and have repeatedly shown that plasma-free metanephrines have
the highest sensitivity and specificity of any currently available
single test. A high sensitivity, that is, a low false-negative rate, is
particularly important for any pheochromocytoma diagnostic
test because an undiagnosed pheochromocytoma can be a potentially lethal problem. A rigorous meta-analysis of the literature in
2004 revealed 36 papers that specifically studied the sensitivity
and specificity of plasma-free metanephrines.13 In the final analysis, heterogeneity in methods and statistical analysis excluded
all but three papers.14-16 The meta-analysis found the sensitivity
of plasma-free metanephrines to be 96% to 100% and the specificity to be 85% to 100% (Level 1a evidence). Plasma-free metanephrine values within the normal range essentially rules out the
diagnosis of pheochromocytoma. This is important in pheochromocytoma because with this test, any patient with normal plasma
metanephrine values can be safely diagnosed as not having a
pheochromocytoma. However, given the lower specificity, that is,
increased false-positive rate, a positive test does not necessarily
rule in the diagnosis of pheochromocytoma, meaning more testing needs to be done if there is any clinical concern or suspicion
for misdiagnosis.
Urinary fractionated metanephrines are the second best
option with a slightly lower sensitivity of 88% to 97% and a slightly
lower specificity of 69% to 95% (Levels 1b and 2b evidence).14,17,18
Again with this test, the sensitivity is so high that a negative test
safely rules out the diagnosis of pheochromocytoma. Although
the literature indicates that serum-free metanephrines have
slightly better sensitivity and specificity compared with fractionated urine metanephrines, there is little evidence in the literature
to support the use of one over the other. In the past, plasma and
urinary metanephrines were measured by high-pressure liquid
chromatography. Newer assays using liquid chromatographytandem mass spectrometry are now preferred because they eliminate false-positive results caused by drug interference.19 Both
types of assays have been used in the literature making it difficult to determine whether plasma-free or urinary fractionated
metanephrines are superior. Direct comparison of the specificity
of plasma-free metanephrines to urinary fractionated metanephrines combined with urinary catecholamines shows that the combination of the two urinary tests has a specificity of 98% compared
with a specificity of 85% for plasma-free metanephrines (Level 1c
evidence).16 It is clear that either plasma or urinary metanephrine tests are superior to plasma or urinary catecholamine tests

PMPH_CH68.indd 546

(sensitivity 84% and 86%, specificity 81% and 88%) or urinary


VMA (sensitivity 64%, specificity 95%) for the diagnosis of pheochromocytoma (Level 1b evidence).14
Answer: Plasma-free metanephrines are the best choice for
diagnosing pheochromocytoma because of their high sensitivity.
(Grade A recommendation). If plasma-free metanephrines are not
available, urinary fractionated metanephrines can be used as an
alternative. (Grade B recommendation). If there is concern over
a possible false-positive plasma-free metanephrine result, then
urinary fractionated metanephrines combined with urinary catecholamines can be considered due to the high specificity of the
combination of the two tests. (Grade B recommendation). Urinary
fractionated metanephrines are now widely available so plasma
or urinary catecholamines and urinary VMA should no longer
be used as the first-line diagnostic tests for pheochromocytoma.
(Grade A recommendation)
3. What is the best imaging study for pheochromocytoma?
There are many types of imaging technologies for pheochromocytoma. It is helpful to divide them into two categories, anatomical
and functional. Anatomical studies of pheochromocytoma include
computed tomography (CT) and magnetic resonance imaging
(MRI) and can be used for anatomic evaluation, localization, identification of metastasis, and preoperative resection planning. Functional imaging techniques take advantage of the catecholamine
synthesis, storage, and secretion pathways of pheochromocytomas.
These techniques include [123/131I]metaiodobenzylguanidine
(MIBG) scintigraphy, 6-[18F]fluoro-L-3,4-dihydroxy-phenylalanine
(DOPA) positron emission tomography (PET), 6-[18F]fluorodopamine (FDA) PET, and 2-[18F]fluoro-2-deoxy-D-glucose (FDG)
PET and are superior to anatomical studies in identifying extraadrenal sites of metastasis.
Anatomical imaging studies are known to have high sensitivity but low specificity for detecting pheochromocytomas.
Many studies have been done to determine the sensitivity and
specificity of both CT and MRI to detect pheochromocytomas.
Both are better at detecting primary intra-adrenal tumors
than they are at detecting extra-adrenal metastasis. Postoperative changes can also hinder their ability to detect recurrence in the adrenal bed. The sensitivity of CT for detecting
primary, intra-adrenal pheochromocytomas is between 95%
and 100%, and is decreased to 73% for extra-adrenal locations (Level 1b evidence).20-22 MRI sensitivity for detecting
primary intra-adrenal pheochromocytomas is similar to CT
and is 91% to 100%, and is decreased to 85% for extra-adrenal
locations (Level 1b evidence). 21,23-25 The specificity of CT and
MRI for detecting pheochromocytoma is poor because they
cannot distinguish functional from nonfunctional adrenal
tumors. The specificity for both CT and MRI to exclude intraadrenal primary tumors and extra-adrenal recurrence is 30%
to 50% (Level 1b evidence). 23,25-27 When evaluating pheochromocytomas with CT, a triphasic scan should be used. A classic pheochromocytoma will show an adrenal nodule with
Hounsfield units of 40 to 50 on the noncontrast image with
an enhancement to over 100 Hounsfield units on the contrast
phase followed by <50% washout on the venous phase. For MRI,
both T1 and T2 weighted images with in- and out-of-phase
sequences and contrast enhancement should be used. A typical

5/22/2012 5:47:23 PM

Pheochromocytoma

pheochromocytoma on MRI will be bright on T2 with no signal


dropout on out-of-phase imaging. Many pheochromocytomas
will also show an intense contrast enhancement on MRI.
Functional imaging studies are much more specific than anatomical imaging studies for pheochromocytoma. Their ability
to identify catecholamine-producing tissue makes them ideal at
distinguishing functional from nonfunctional masses. For many
years, MIBG has been the gold standard for functional imaging
of pheochromocytomas with I123 being preferred over I131 for its
higher sensitivity. A recent meta-analysis of 15 studies found a combined specificity of 98% and a sensitivity of 96% in detecting primary
intra-adrenal pheochromocytomas (Level 1a evidence). There were
very little data on extra-adrenal metastasis in this study. A separate
meta-analysis showed that the sensitivity of 123I-MIBG decreases to
79% for metastatic pheochromocytoma (Level 1a evidence). Adrenal cortical cancer and infectious lesions can cause false-positive
results, and dedifferentiated malignant pheochromocytomas can
cause false-negative results. Even though MIBG is similar to CT and
MRI in sensitivity, and superior in specificity, it is generally not the
first-line imaging study preferred in patients with a clear biochemical diagnosis of pheochromocytoma. This is because CT and MRI
provide superior anatomic detail that is necessary for preoperative
planning (Level 5 evidence). Furthermore, in patients with a single
unilateral tumor on anatomic imaging, a clear biochemical diagnosis, and no clinical or family history suspicious for metastatic or
multifocal disease, the addition of MIBG to a CT or MRI does not
add any significant information (Level 2b evidence).28
Recent advances in PET technology have allowed for the
development of a number of promising functional studies that may
perform better than MIBG in certain situations. A recent validating prospective cohort study compared the diagnostic accuracy
of 18F-DOPA PET, 18F-FDG PET, 18F-FDA PET, and 123I-MIBG
in patients with both primary and metastatic pheochromocytoma
(Level 1b evidence).29 There was no statistically significant difference in sensitivity between any of the functional studies (7788%)
for primary intra-adrenal lesions. For extra-adrenal metastatic
disease, 18F-FDA had the highest sensitivity (76%), followed by
18F-FDG (74%), 123I-MIBG (57%), and 18F-DOPA (45%). Even
though the sensitivities of 18F-FDA and 18F-FDG are similar,
the specificity of 18F-FDG for pheochromocytoma is much lower
because it is a marker of increased tissue glucose metabolism,
rather than a marker of catecholamine synthesis. The final results
of this study indicate that 18F-FDA is the best functional test for
diagnosing pheochromocytoma metastasis or recurrence (Level 1b
evidence).
Answer: The high sensitivity and anatomic detail of CT or
MRI make them the diagnostic imaging test of choice in biochemically proven pheochromocytoma in patients with no clinical history concerning for multifocal or for metastatic disease. (Grade A
recommendation). CT and MRI perform equally well for localizing
pheochromocytomas; however, MRI is preferred in pediatric, pregnant, and iodinated-contrast allergic patients. In patients with metastatic pheochromocytoma or in patients with genetic syndromes
associated with pheochromocytoma, 123I-MIBG is the functional
test of choice and should be added to the anatomic imaging as part
of the workup. (Grade B recommendation). 18F-FDA PET is a newer
modality that is not widely available at this time, but initial studies
show that 18F-FDA PET may be the functional imaging study of
choice if it is available. (Grade B recommendation)

PMPH_CH68.indd 547

547

PROGNOSIS/MALIGNANT POTENTIAL
4. Does pheochromocytoma size predict malignant potential?
According to the World Health Organization definition, there is
no single structural or cellular finding that distinguishes malignant from benign pheochromocytoma. The only criterion that
distinguishes malignant pheochromocytoma from benign pheochromocytoma is distant metastasis.30 Even local invasion is not
considered a marker of malignancy in pheochromocytoma. There
are scant evidence-based data on the incidence, prevalence, diagnosis, and prognosis of malignant pheochromocytoma. There are
no randomized clinical trials or systematic reviews. Almost all the
data to date come from a collection of retrospective case series
that contain heterogeneous populations of tumor types with lack
of control data.
The malignancy rate of pheochromocytoma ranges between
3% and 36% depending on the genetic background of the tumor as
well as intra- versus extra-adrenal location (Level 4 evidence).31-33
The overall 5-year survival rate of patients with malignant pheochromocytoma is reported to be between 34% and 60% (Level 4
evidence).31,33 Some case reports have also shown that malignant pheochromocytoma may not metastasize for decades, with
the longest reported disease-free interval being 20 years (Level 4
evidence).34,35 The poor survival, difficulty in diagnosis, and the
need for long-term follow-up are all the factors that make it important to develop a method of identifying the difference between
benign and malignant pheochromocytomas.
The general consensus has typically been that pheochromocytomas greater than 5 cm in size are at an increased risk for
being malignant. Th is belief was based mostly on data from a
few different single institution case series of roughly 100 patients
each (Level 4 evidence).31,36-39 The majority of these studies were
descriptive in nature and many were focused more on the histopathology of malignant pheochromocytoma, rather than on
the size. These studies were further hindered by the inclusion
of intra- as well as extra-adrenal pheochromocytomas, were not
designed as cohort studies, and had no predictive value for sensitivity or specificity.
To address this question more rigorously, investigators at the
University of California San Francisco designed a retrospective
exploratory cohort study of the Surveillance Epidemiology and
End Results database (Level 2b evidence).40 Their study confirmed
the belief that at the time of presentation, malignant pheochromocytomas are larger than benign pheochromocytomas (7.6 vs.
5.3 cm). However, a subgroup analysis showed that there was no
difference in the size between benign and malignant pheochromocytomas that presented initially without distant metastasis. An
analysis of sensitivity and specificity confirmed that pheochromocytoma size could not reliably predict malignancy in the absence
of metastasis at the time of diagnosis.
Answer: Malignant pheochromocytomas are larger than
benign pheochromocytomas at initial presentation. However, the
size difference is not significant in pheochromocytomas that do
not have a distant metastasis at the time of initial presentation.
Size should not be used as a criterion for predicting the malignant
potential of a pheochromocytoma. (Grade C recommendation).
Distant metastasis at the time of presentation remains the only
preoperative criterion for predicting malignancy.

5/22/2012 5:47:23 PM

548

Surgery: Evidence-Based Practice

TREATMENT
5. Is laparoscopic adrenalectomy the procedure of choice for
pheochromocytoma?
Surgical removal is the treatment of choice for pheochromocytoma. Charles Mayo reported the first adrenalectomy in the
United States in 1927.41 Historical records indicate that the mortality of pheochromocytoma resection from that time until the
1950s was as high as 26%.42 The morbidity and mortality of pheochromocytoma resection are a result of uncontrolled surges of
catecholamines causing vasoconstriction, severe hypertension,
cardiovascular shock, and stroke. After the tumor is excised,
patients can experience hypotension and cardiovascular collapse
because of the sudden withdrawal of the excess catecholamines. In
the 1950s, investigators at the Mayo Clinic greatly decreased the
operative mortality of pheochromocytoma resection by introducing alpha-adrenergic blockade and volume expansion as a routine
part of preoperative preparation of these patients.43 This intervention reduced the operative mortality of open pheochromocytoma
resection to between 0% and 6.7% and the postoperative stay to
approximately 1 week according to multiple case series (Level 4
evidence).44-46
In 1992, Michel Gagner reported the fi rst series of laparoscopic transperitoneal adrenalectomies.4 Laparoscopic adrenalectomy rapidly became the procedure of choice for the removal
of small, benign adrenal tumors such as aldosteronomas
and cortisol-secreting tumors. Surgeons also began to experiment with laparoscopic techniques for pheochromocytoma
resection. In the beginning, there was a concern that pneumoperitoneum, increased operative times, and the larger size

of pheochromocytomas made laparoscopic resection unsafe.


However, by 1996, Gagner had accumulated a case series of 82
laparoscopic adrenalectomies that included 23 pheochromocytoma patients (Level 4 evidence).47 Th is series showed that
in comparison with other adrenal tumors, pheochromocytomas were larger, required longer operative times, had more
intraoperative hemodynamic instability, and were associated
with longer hospital stays. Th is series also showed that in comparison with open adrenalectomy, laparoscopic adrenalectomy
was just as safe, but had much shorter postoperative hospitalizations and much less postoperative pain. A large number of
case series from around the world have since been accumulated showing that laparoscopic adrenalectomy is as safe as
open adrenalectomy (Level 4 evidence). 34,40,48-51 The fi ndings in
these case series have been corroborated by a few prospective
and retrospective cohort studies; however, the small numbers
of patients in these studies mean that most are not sufficiently
powered to detect differences in the populations being studied
(Level 2b evidence). 52-55 There have been no systematic reviews
of the literature on laparoscopic versus open pheochromocytoma resection.
Answer: Laparoscopic adrenalectomy is considered superior
to open adrenalectomy for pheochromocytoma resection. (Grade C
recommendation). This is based on the data from multiple case
series and expert opinion that has been generated by these case
series. Because of the low prevalence of pheochromocytoma, most
cohort studies do not have the power to detect significant differences in outcomes. A high-quality systematic review of what is
already available in the literature needs to be done to improve our
understanding of the safety and efficacy of laparoscopic adrenalectomy for pheochromocytoma.

Clinical Question Summary


Question

Answer

Grade of
Recommendation

References

1 What genetic mutations


are associated with
pheochromocytoma
development, and how often
are pheochromocytomas
caused by inherited mutations?

Pheochromocytomas are caused by inherited


genetic mutations in the RET, VHL, NF1, SDHB,
SDHC or SDHD genes between 17.5% and 34%
of the time. However, the overall rate of inherited
mutations is likely to be higher because the
incidence and prevalence of SDHA, SDHAF1,
SDHAF2, TMEM127, and KIF1B Beta mutations
are unknown.

3, 5-11

2 What is the best laboratory


test to diagnose
pheochromocytoma?

Plasma-free metanephrines are the best choice


for diagnosing pheochromocytoma (Grade A).
Alternatively, if these are not available, urinary
fractionated metanephrines can be used
(Grade B). If a false-positive plasma free
metanephrine result seems possible, then urinary
fractionated metanephrines combined with urinary
catecholamines can be considered (Grade B).
Plasma or urinary catecholamines and urinary
VMA should no longer be used as first-line diagnostic
tests for pheochromocytoma (Grade A).

A and B

12-19

(Continued)

PMPH_CH68.indd 548

5/22/2012 5:47:23 PM

Pheochromocytoma

549

(Continued)
Question

Answer

Grade of
Recommendation

References

3 What is the best imaging study


for pheochromocytoma?

CT or MRI are the diagnostic imaging tests of choice in


biochemically proven pheochromocytoma in patients
with no clinical history concerning multi-focal or
metastatic disease (Grade A). CT and MRI perform
equally well for localizing pheochromocytomas, but
MRI is preferred in pediatric, pregnant, and iodinatedcontrast allergic patients. In patients with metastatic
pheochromocytoma or in patients with genetic
syndromes associated with pheochromocytoma 123IMIBG is the functional test of choice and should be
added to the anatomic imaging as part of the workup
(Grade B). 18F-FDA PET is not widely available at this
time, but initial studies show that 18F-FDA PET may
be the functional imaging study of choice when it is
available (Grade B).

A and B

20-29

4 Does pheochromocytoma size


predict malignant potential?

Malignant pheochromocytomas are larger than


benign pheochromocytomas at initial presentation,
but the size difference is not significant in
pheochromocytomas lacking distant metastasis at
initial presentation. Size should not be used as a
criterion for predicting the malignant potential of a
pheochromocytoma (Grade C). Distant metastasis at
presentation remains the only preoperative criterion
for predicting malignancy.

30-40

5 Is laparoscopic adrenalectomy
the procedure of choice for
pheochromocytoma?

Laparoscopic adrenalectomy is superior to open


adrenalectomy for pheochromocytoma resection
(Grade C). Because of the low prevalence of
pheochromocytoma, most cohort studies cannot
detect significant differences in outcomes.

41-55

REFERENCES
1. Beard CM, Sheps SG, et al. Occurrence of pheochromocytoma
in Rochester, Minnesota, 1950 through 1979. Mayo Clin Proc.
1983;58:802-804.
2. Stenstrm G, Svrdsudd K. Pheochromocytoma in Sweden 19581981. An analysis of the National Cancer Registry Data. Acta
Med Scand. 1986;220:225-232.
3. Astuti D, Latif F, Dallol A, et al. Gene mutations in the succinate
dehydrogenase subunit SDHB cause susceptibility to familial
pheochromocytoma and to familial paraganglioma. Am J Hum
Genet. 2001;69:49-54.
4. Gagner M, Lacroix A, Bolt E. Laparoscopic adrenalectomy in
Cushings syndrome and pheochromocytoma. N Engl J Med. 1992;
327:1033.
5. Hensen EF, Bayley JP. Recent advances in the genetics of SDHrelated paraganglioma and pheochromocytoma. Fam Cancer.
2010;10:355-363.
6. Schlisio S, Kenchappa RS, Vredeveld LC, et al. The kinesin KIF1Bbeta acts downstream from EglN3 to induce apoptosis and is a
potential 1p36 tumor suppressor. Genes Dev. 2008;22:884-893.
7. Neumann HP, Bausch B, McWhinney SR, et al. Germ-line mutations in nonsyndromic pheochromocytoma. N Engl J Med.
2002;346:1459-1466.

PMPH_CH68.indd 549

8. Korpershoek E, Van Nederveen FH, Dannenberg H, et al. Genetic


analyses of apparently sporadic pheochromocytomas: The Rotterdam experience. Ann N Y Acad Sci. 2006;1073:138-148.
9. Mannelli M, Castellano M, et al. Clinically guided genetic screening in a large cohort of italian patients with pheochromocytomas
and/or functional or nonfunctional paragangliomas. J Clin Endocrinol Metab. 2009;94:1541-1547.
10. Gimenez-Roqueplo AP, Favier J, Rustin P, et al. Mutations in the
SDHB gene are associated with extra-adrenal and/or malignant
phaeochromocytomas. Cancer Res. 2003;63:5615-5621.
11. Cascon A, Ruiz-Llorente S, Fraga MF, et al. Genetic and epigenetic profi le of sporadic pheochromocytomas. J Med Genet.
2004;41:e30.
12. Eisenhofer G, Goldstein D, Kopin I, Richard Crout J. Pheochromocytoma: Rediscovery as a catecholamine-metabolizing tumor.
Endocr Pathol. 2003;14:193-212.
13. Sawka AM, Prebtani AP, Thabane L, Gafni A, Levine M, Young WF.
A systematic review of the literature examining the diagnostic
efficacy of measurement of fractionated plasma-free metanephrines in the biochemical diagnosis of pheochromocytoma. BMC
Endocr Disord. 2004;4:2.
14. Lenders JW, Pacak K, Walther MM, et al. Biochemical diagnosis of pheochromocytoma: Which test is best? JAMA. 2002;287:
1427-1434.

5/22/2012 5:47:23 PM

550

Surgery: Evidence-Based Practice

15. Raber W, Raffesberg W, Bischof M, et al. Diagnostic efficacy of


unconjugated plasma metanephrines for the detection of pheochromocytoma. Arch Intern Med. 2000;160:2957-2963.
16. Sawka AM, Jaeschke R, Singh RJ, Young WF. A comparison of
biochemical tests for pheochromocytoma: Measurement of
fractionated plasma metanephrines compared with the combination of 24-hour urinary metanephrines and catecholamines. J
Clin Endocrinol Metab. 2003;88:553-558.
17. Hickman PE, Leong M, Chang J, Wilson SR, McWhinney B.
Plasma free metanephrines are superior to urine and plasma catecholamines and urine catecholamine metabolites for the investigation of phaeochromocytoma. Pathology. 2009;41:173-177.
18. Gardet V, Gatta B, Simonnet G, et al. Lessons from an unpleasant
surprise: A biochemical strategy for the diagnosis of pheochromocytoma. J Hypertens. 2001;19:1029-1035.
19. Taylor RL, Singh RJ. Validation of liquid chromatographytandem mass spectrometry method for analysis of urinary
conjugated metanephrine and normetanephrine for screening
of pheochromocytoma. Clin Chem. 2002;48:533-539.
20. Quint LE, Glazer GM, Francis IR, Shapiro B, Chenevert TL.
Pheochromocytoma and paraganglioma: Comparison of MR
imaging with CT and I-131 MIBG scintigraphy. Radiology.
1987;165:89-93.
21. Neumann HP, Berger DP, Sigmund G, et al. Pheochromocytomas, multiple endocrine neoplasia type 2, and von HippelLindau disease. N Engl J Med. 1993;329:1531-1538.
22. Lumachi F, Tregnaghi A, Zucchetta P, et al. Sensitivity and positive predictive value of CT, MRI and 123I-MIBG scintigraphy in
localizing pheochromocytomas: A prospective study. Nucl Med
Commun. 2006;27:583-587.
23. Maurea S, Cuocolo A, Reynolds JC, Neumann RD, Salvatore M.
Diagnostic imaging in patients with paragangliomas. Computed
tomography, magnetic resonance and MIBG scintigraphy comparison. Q J Nucl Med. 1996;40:365-371.
24. Varghese JC, Hahn PF, Papanicolaou N, Mayo-Smith WW,
Gaa JA, Lee MJ. MR differentiation of phaeochromocytoma
from other adrenal lesions based on qualitative analysis of T2
relaxation times. Clin Radiol. 1997;52:603-606.
25. Timmers HJ, Eisenhofer G, Carrasquillo JA, et al. Use of 6-[18F]fluorodopamine positron emission tomography (PET) as
first-line investigation for the diagnosis and localization of nonmetastatic and metastatic phaeochromocytoma (PHEO). Clin
Endocrinol (Oxf). 2009;71:11-17.
26. Peplinski GR, Norton JA. The predictive value of diagnostic tests
for pheochromocytoma. Surgery. 1994;116:1101-1109; discussion
1109-1110.
27. Maurea S, Cuocolo A, Reynolds JC, et al. Iodine-131-metaiodobenzylguanidine scintigraphy in preoperative and postoperative
evaluation of paragangliomas: Comparison with CT and MRI.
J Nucl Med. 1993;34:173-179.
28. Miskulin J, Shulkin BL, Doherty GM, Sisson JC, Burney RE,
Gauger PG. Is preoperative iodine 123 meta-iodobenzylguanidine
scintigraphy routinely necessary before initial adrenalectomy
for pheochromocytoma? Surgery. 2003;134:918-922.
29. Timmers HJ, Chen CC, Carrasquillo JA, et al. Comparison of 18Ffluoro-L-DOPA, 18F-fluoro-deoxyglucose, and 18F-fluorodopamine
PET and 123I-MIBG scintigraphy in the localization of pheochromocytoma and paraganglioma. J Clin Endocrinol Metab. 2009;94:
4757-4767.
30. DeLellis RA. Pathology and genetics of tumours of endocrine
organs. World Health Organization; 2004.

PMPH_CH68.indd 550

31. John H, Ziegler WH, Hauri D, Jaeger P. Pheochromocytomas: Can


malignant potential be predicted? Urology. 1999;53:679-683.
32. ORiordain DS, Young WF, Grant CS, Carney JA, van Heerden JA.
Clinical spectrum and outcome of functional extraadrenal
paraganglioma. World J Surg. 1996;20:916-921; discussion 922.
33. Mundschenk J, Lehnert H. Malignant pheochromocytoma. Exp
Clin Endocrinol Diabetes. 1998;106:373-376.
34. Tanaka S, Ito T, Tomoda J, Higashi T, Yamada G, Tsuji T. Malignant pheochromocytoma with hepatic metastasis diagnosed 20
years after resection of the primary adrenal lesion. Intern Med.
1993;32:789-794.
35. Shen WT, Grogan R, Vriens M, Clark OH, Duh QY. One hundred two patients with pheochromocytoma treated at a single
institution since the introduction of laparoscopic adrenalectomy. Arch Surg. 2010;145:893-897.
36. Linnoila RI, Keiser HR, Steinberg SM, Lack EE. Histopathology
of benign versus malignant sympathoadrenal paragangliomas:
Clinicopathologic study of 120 cases including unusual histologic features. Hum Pathol. 1990;21:1168-1180.
37. Thompson LD. Pheochromocytoma of the Adrenal gland Scaled
Score (PASS) to separate benign from malignant neoplasms: A
clinicopathologic and immunophenotypic study of 100 cases.
Am J Surg Pathol. 2002;26:551-566.
38. Strong VE, Kennedy T, Al-Ahmadie H, et al. Prognostic indicators of malignancy in adrenal pheochromocytomas: Clinical,
histopathologic, and cell cycle/apoptosis gene expression analysis. Surgery. 2008;143:759-768.
39. Kimura N, Watanabe T, Noshiro T, Shizawa S, Miura Y. Histological grading of adrenal and extra-adrenal pheochromocytomas
and relationship to prognosis: A clinicopathological analysis of
116 adrenal pheochromocytomas and 30 extra-adrenal sympathetic paragangliomas including 38 malignant tumors. Endocr
Pathol. 2005;16:23-32.
40. Shen WT, Sturgeon C, Clark OH, Duh QY, Kebebew E. Should
pheochromocytoma size influence surgical approach? A comparison of 90 malignant and 60 benign pheochromocytomas.
Surgery. 2004;136:1129-1137.
41. Mayo CM. Paroxysmal Hypertension With Tumor of Retroperitoneal Nerve. JAMA. 1927;89:1047.
42. Welbourn RB. The adrenal glands. The History of Endocrine Surgery. New York: Praeger; 1990:147-215.
43. Kvale WF, Manger WM, Priestley JT, Roth GM. Pheochromocytoma. Circulation. 1956;14:622-630.
44. Orchard T, Grant CS, van Heerden JA, Weaver A. Pheochromocytomacontinuing evolution of surgical therapy. Surgery.
1993;114:1153-1158; discussion 1158-1159.
45. Pruszczyk P, Januszewicz W, Feltynowski T, et al. Long term followup after surgical removal of pheochromocytomaobservations
in 61 patients. Clin Exp Hypertens A. 1991;13:1179-1194.
46. Plouin PF, Duclos JM, Soppelsa F, Boublil G, Chatellier G. Factors
associated with perioperative morbidity and mortality in patients
with pheochromocytoma: Analysis of 165 operations at a single
center. J Clin Endocrinol Metab. 2001;86:1480-1486.
47. Gagner M, Breton G, Pharand D, Pomp A. Is laparoscopic
adrenalectomy indicated for pheochromocytomas? Surgery.
1996;120:1076-1079; discussion 1079-1080.
48. Li QY, Li F. Laparoscopic adrenalectomy in pheochromocytoma:
Retroperitoneal approach versus transperitoneal approach. J
Endourol. 2010;24:1441-1445.
49. Perry KA, El Youssef R, Pham TH, Sheppard BC. Laparoscopic adrenalectomy for large unilateral pheochromocytoma:

5/22/2012 5:47:23 PM

Pheochromocytoma

Experience in a large academic medical center. Surg Endosc.


2010;24:1462-1467.
50. Smith CD, Weber CJ, Amerson JR. Laparoscopic adrenalectomy:
New gold standard. World J Surg. 1999;23:389-396.
51. Cheah WK, Clark OH, Horn JK, Siperstein AE, Duh QY. Laparoscopic adrenalectomy for pheochromocytoma. World J Surg.
2002;26:1048-1051.
52. Kim HH, Kim GH, Sung GT. Laparoscopic adrenalectomy for
pheochromocytoma: Comparison with conventional open
adrenalectomy. J Endourol. 2004;18:251-255.

PMPH_CH68.indd 551

551

53. Mellon MJ, Sundaram CP. Laparoscopic adrenalectomy for pheochromocytoma versus other surgical indications. J Soc Lap Surg.
2008;12:380-384.
54. Inabnet WB, Pitre J, Bernard D, Chapuis Y. Comparison
of the hemodynamic parameters of open and laparoscopic
adrenalectomy for pheochromocytoma. World J Surg. 2000;24:
574-578.
55. Tiberio G, Baiocchi G, Arru L, et al. Prospective randomized
comparison of laparoscopic versus open adrenalectomy for
sporadic pheochromocytoma. Surg Endosc. 2008;22:1435-1439.

5/22/2012 5:47:23 PM

CHAPTER 69

Thyroid Nodules
Gerard M. Doherty

INTRODUCTION

One special circumstance is the discovery of hypermetabolic


nodules in the thyroid on fluorodeoxyglucose-positron emission
tomography (FDG-PET) scan done in patients with an unrelated
cancer. In these patients, the standardized uptake values (SUV)
can be helpful in guiding the evaluation. In general, the higher
the SUV, the more likely the lesion is to be malignant. In general,
however, the nodule is evaluated along the usual algorithm of history, examination, ultrasound characterization and fi ne needle
aspiration (FNA), to determine the best course of action.1

The risk of malignancy drives the evaluation of a solitary thyroid


nodule; however, selective management is important due to the
preponderance of benign lesions. Between 4% and 7% of the adult
population in the United States have a palpable thyroid nodule,
the vast majority of these patients have benign disease, and do
not require surgical resection. The nodule must be evaluated in
its context, in light of the status of the remainder of the thyroid
gland, and the patients overall health and risk factors. The accuracy of various diagnostic techniques must be considered in light
of the risk the lesion, to synthesize an overall plan for the patient.
The management of thyroid nodules has been thoroughly
evaluated based on available evidence by the both the American
Thyroid Association and the National Comprehensive Cancer
Network.1,2 In addition, a National Cancer Institute State of the
Science Conference in October 2007 evaluated the available evidence focused specifically on thyroid cytology for diagnostic
evaluation of nodules.3 This chapter summarizes some of the
information available in those more detailed documents.

2. What is the appropriate clinical evaluation for thyroid


nodules?
Evaluation of a nodule in the thyroid gland should begin with a
comprehensive history and physical examination (Figure 69.1).
It is not usually possible to distinguish a benign nodule from a
malignant nodule by palpation; however, there are some clinical findings that are indicative of malignant disease. A history
of a hard and relatively fast-growing nodule is associated with a
higher risk for malignancy when compared with a soft and slowly
growing nodule. Furthermore, the presence of a solitary nodule is
more indicative of malignancy, whereas a multinodular thyroid
is more consistent with benign disease. High-risk aspects of the
patient history include a history of thyroid cancer in one or more
first-degree relatives; history of external beam radiation as a child;
exposure to ionizing radiation in childhood or adolescence; prior
hemithyroidectomy with discovery of thyroid cancer; 18FDG
avidity on PET scanning; or a known activating RET protooncogene mutation.1
The measurement of serum thyroid stimulating hormone
(TSH) to characterize thyroid function is important. The risk of
thyroid carcinoma increases as the serum TSH increases. Though
most patients with thyroid cancer have a normal serum TSH, an
elevated TSH seems to confer increased risk, and a suppressed
TSH level is suggestive of benign pathology, as it is uncommon
for malignant lesions to cause thyrotoxicosis.5 This is an important first branch point in any algorithm for thyroid nodule management. In particular, nuclear scintigraphy of the thyroid gland

1. How are thyroid nodules identified?


Thyroid nodules are identified by local symptoms, visible asymmetry, self-palpation, physical examination, or incidental finding
on unrelated imaging.
The majority of nodules are small (<15 mm) and so do not
typically cause local symptoms nor are they noticed by patients.
However, if they are in a position that is relatively superficial,
then they may be palpated on screening physical examination.
Predominantly, recent increases in the number of imaging tests
performed for unrelated issues in the neck and chest especially,
have led to a large number of incidentally discovered thyroid nodules, and by extension, an increase in the number of small thyroid
cancers discovered.4 Some common imaging scenarios include
carotid duplex examination and magnetic resonance (MR) examinations of the cervical spine, on which previously unsuspected
thyroid nodules can be noted.
552

PMPH_CH69.indd 552

5/22/2012 5:47:55 PM

Thyroid Nodules
Patient with a thyroid nodule

History, Physical Examination, TSH level


Characterize personal risk

TSH Low

TSH normal or
high

Nuclear scintigraphy
distinguish Graves with
nodule from toxic adenoma

Characterize nodule by
ultrasound

Treat for hyperthyroidism


No indication
for tissue
sampling

Patient/nodule
meet criteria
for tissue
sampling
(Table 1)

Interval follow-up by clinical


exam with or without
ultrasound
FNA (with ultrasound
guidance if available)

Indeterminate

Repeat

Benign

Malignant
Insufficient
sample

Resection for Cancer

Consider diagnostic resection or


close interval follow-up

Figure 69.1 Algorithm for the management of a patient with a


thyroid nodule.
is no longer considered useful for the routine management of
people with thyroid nodules, except for that subgroup that has a
suppressed TSH.
The utility of serum measurement of calcitonin in the evaluation of thyroid nodules is not clear. This has been proposed as a
way to detect small medullary thyroid carcinomas, and to select
patients for biopsy of otherwise benign appearing nodules. This
has been evaluated in prospective, nonrandomized trials, and
appears to lead to the earlier diagnosis of medullary cancer. However, these trials have been done mainly in Europe, and the prevalence of medullary thyroid cancer in the US appears to be lower.
This may diminish the value of such testing in the United States,
and so it is not generally recommended here.6
3. What is the role of thyroid ultrasound to evaluate thyroid
nodules?
Further evaluation of the solitary nodule is best done by ultrasound and selective use of FNA. The ultrasound examination of
the thyroid gland allows accurate characterization of the thyroid
nodule and the surrounding thyroid tissue. Characterization of
the nodule includes the size, shape, vascularity, solid versus cystic
nature, the presence of calcifications in the nodule, and the nature
of the borders (smooth, irregular, etc.). Ultrasound examinations
can be performed in a variety of settings, and are now frequently
performed in the outpatient clinic by thyroid specialists. Any
subsequent tissue sampling can then often be performed during t
he same visit in order to compress the patients episode of care to
the greatest extent possible.

PMPH_CH69.indd 553

553

The ultrasound findings can then be combined with the clinical history, to determine whether the nodule should be aspirated.1,2
Suspicious ultrasound findings include microcalcifications in the
nodule, hypoechogenicity, increased nodular vascularity, infi ltrative margins and a nodule that is taller than wide on transverse
view. These findings must be interpreted in light of the clinical
situation of the patient. The ultrasound findings can also be the
baseline study for any subsequent follow-up examinations for
patients who need this. It is very useful to have direct access to the
images from prior examinations for the follow-up studies, rather
than merely the reports.
The combination of the clinical findings and the ultrasound
observations can be used to recommend which nodules should be
biopsied (Table 69.1).
4. What is the role of FNA thyroid cytology to evaluate thyroid
nodules?
FNA cytology is the main evaluative technique available to separate the many benign thyroid nodules from the few malignant
nodules. FNA has the desirable features of being relatively welltolerated, inexpensive, and widely available. The negative aspects
of FNA are the substantial proportion of patients who receive
nondefinitive results, either due to insufficient samples in about
15%, or to indeterminate results in about 15% of those who have
sufficient samples.7 These weaknesses are the subject of ongoing
investigations to try to develop molecular testing approaches
on the FNA samples that can resolve these issues. The use of
ultrasound-guided FNA can improve the diagnostic accuracy, but
interpretation of the aspirate for definitive diagnosis may still not
be possible.
In patients with follicular neoplasia (follicular adenoma, follicular carcinoma), FNA is limited in its ability to distinguish
malignant from benign disease, since the diagnosis of follicular
carcinoma is based upon histologic features. Specifically, identification of capsular or vascular invasion is necessary for diagnosis
of malignant disease. This scenario is similar when dealing with
patients who have Hrthle cell neoplasms.
The 2007 NCI conference regarding thyroid FNA provided
some new guidelines with respect to reporting the results of the
FNA.3 This should help to standardize the interpretation of thyroid FNA results, and aid the clinician in translating the cytology
report into clinical action. The categories expand the usual four
categories of benign, malignant, indeterminate, and insufficient,
to include three intermediate categories representing various
levels of risk of malignancy (Table 69.2). Standardized reporting categories should improve the consistency of communication
between cytologists and clinicians.
5. What is the appropriate nonoperative follow-up for thyroid
nodules?
After determination that a thyroid nodule is not at high risk of
being malignant, the clinical course of action may be to follow
the patient and the nodule over time. This is necessary even for
nodules that appear to be benign on evaluation, as there is a low
but real rate of false-negative evaluation including FNA cytology.
Follow-up should generally occur at intervals of 3 to 12 months,
depending on the risk of the nodule being malignant. A common
paradigm is to re-check the nodule at 6 months, and if still consistent with a benign lesion, to follow-up at 12-month intervals

5/22/2012 5:47:55 PM

554

Surgery: Evidence-Based Practice

Table 69.1 Indications for FNA of a Thyroid Nodule


Patient/Nodule Characteristics

Threshold Nodule Size to Trigger Tissue Sampling

ATA Recommendationsa
High-Risk historyb
Nodule with multiple suspicious sonography featuresc

>5 mm

Nodule without suspicious sonography

>5 mm

Abnormal cervical lymph nodes

FNA lymph node and/or thyroid nodule

Microcalcifications in nodule

>10 mm

Solid nodule
AND hypoechoic

>10 mm

AND isoechoic or hyperechoic

>1015 mm

Mixed cystic-solid nodule


WITH any suspicious ultrasound features

>1520 mm

WITHOUT any suspicious ultrasound features

>20 mm

Spongiform nodule

>20 mm

Purely cystic nodule

FNA not indicated

NCCN Recommendationsd
Solid nodule
With suspicious sonographic features

>10 mm

Without suspicious sonographic features

>15 mm

Mixed cystic-solid nodule


With suspicious sonographic features

>1520 mm

Without suspicious sonographic features

>20 mm

Spongiform nodule

>20 mm

Simple cyst

FNA not indicated

Suspicious cervical lymph node

FNA lymph node and/or thyroid nodule

Revised American Thyroid Association Management Guidelines for Patients with Thyroid Nodules and Differentiated Thyroid Cancer (Ref. [1]).
High-risk history: History of thyroid cancer in one or more first-degree relatives; history of external beam radiation as a child; exposure to ionizing
radiation in childhood or adolescence; prior hemithyroidectomy with discovery of thyroid cancer, 18FDG avidity on PET scanning; MEN-2 = FMTCassociated RET protooncogene mutation, calcitonin >100 pg/mL; MEN, multiple endocrine neoplasia; FMTC, familial medullary thyroid cancer.
c
Suspicious features: microcalcifications; hypoechoic; increased nodular vascularity; infiltrative margins; taller than wide on transverse view.
d
National Comprehensive Cancer Network Guidelines: Thyroid Carcinoma version 1.2011 (Ref. [2]).
e
Suspicious sonographic features: Hypoechoic, microcalcifications, increased central vascularity, infiltrative margins, taller than wide in transverse plane.
b

Table 69.2 Bethesda System for Thyroid Cytopathology


Diagnostic Category

Risk of Malignancy

Nondiagnostic or unsatisfactory

14%

Benign

03%

Atypical lesion of undetermined significance or follicular lesion of undetermined significance

515%

Follicular neoplasm or suspicious for follicular neoplasm

1530%

Suspicious for malignancy

6075%

Malignant

9799%

Source: From the National Cancer Institute State of the Science Conference, Bethesda, Maryland, October 2223, 2007. 3

PMPH_CH69.indd 554

5/22/2012 5:47:55 PM

Thyroid Nodules

555

thereafter. Follow-up examinations should include the same


modalities as the initial evaluation including sonography.
Benign nodules do enlarge, though often slowly. Thus, enlargement by itself is not a reason for removal, but rather, enlargement
should prompt repeat FNA cytology. The evaluation is then similar
to the initial decision-making process. The definition of enlargement used in the ATA guidelines is growth by at least 20% (minimum 2 mm) in two perpendicular dimensions.

life. The potential complications of thyroid operations include


the immediate complication of cervical hematoma, as well as the
more chronic complications of hypoparathyroidism, nerve injury,
and injuries to the aerodigestive tract. Finally, chronic problems
can arise from iatrogenic hyper- or hypothyroidism.

6. What is the role of operative management of thyroid nodules?

Neck hematoma requiring re-operation develops after operation


in about 1 of every 150 thyroidectomies12-16 and nearly always
appears within the initial 6 h after the completion of the procedure. The hematoma is manifest by increasing pain, neck swelling,
and often marked anxiety. It can collect either between the platysma muscle and the sternohyoid muscles (superficial), or deep to
the strap muscles along the larynx (deep). Deep hematomas are
the more dangerous due to collection on one side of the larynx
causing a shift and compression of the airway.
A minority of patients with postoperative hematomas develop airway compromise requiring emergent evacuation at the
bedside, but this possibility exists with every neck hematoma.
Patients with a significant hematoma of the neck should not
be left alone until the hematoma has been evacuated. For most
patients, the hematoma is less immediately threatening, and the
patient can be urgently returned to the operating room, placed
under anesthesia, and the hematoma then evacuated and bleeding controlled. Careful hemostasis during the initial operation
is justified, with particular attention to the superior pole vascular pedicle, and the vessels of the ligament of Berry, adjacent
to the recurrent laryngeal nerve insertion, to try to prevent this
complication.
The risk of cervical hematoma has led some to question
the safety of outpatient thyroidectomy, because there would be
some possibility of the hematoma developing after discharge.12-16
The current experience with outpatient thyroid surgery by experts
in the field has demonstrated that this can be done safely, though
postoperative observation for 6 h is routine, to detect this complication prior to facility discharge.

Thyroid nodules with a significant risk of being malignant after


evaluation, or that are symptomatic, should be treated by thyroidectomy. The most typical procedure is a unilateral diagnostic thyroid lobectomy, although a bilateral procedure may be indicated if
there are contralateral abnormalities. The minimum appropriate
procedure for assessing the nature of a potentially malignant thyroid lesion is lobectomy and isthmusectomy.8-11 This must include
a gross margin of normal thyroid gland between the line of division and the lesion at risk.
Removing one side of the thyroid gland carries complications
that are similar to the complications of total thyroidectomy, with
some important distinctions. First, as a single functional parathyroid gland is sufficient to maintain normal parathyroid control
of calcium flux, and there are parathyroid glands on each side of
the larynx, it is not possible to produce permanent hypoparathyroidism by thyroid lobectomy. Second, injury to the ipsilateral
recurrent laryngeal nerve can produce permanent voice changes,
but thyroid lobectomy does not carry a risk of bilateral recurrent nerve injury and consequent airway occlusion. Finally, most
patients who require only unilateral thyroidectomy have no need
for thyroid hormone replacement therapy, thus eliminating the
possibility of iatrogenic hyper- or hypothyroidism.
7. What operative approaches are available for operative
management?
The most commonly applied standard thyroidectomy is performed under general anesthesia, through a low cervical incision
and encompasses an overnight facility stay. However, each of these
features can be altered. Some surgeons commonly perform this
procedure under local anesthesia as an outpatient, for example.
Others perform the procedure through very small neck incisions
with videoscopic guidance, or even axillary or breast incisions
using video or robotic technology. Any of these approaches are
acceptable as long as the basic principles are respected: the ipsilateral lobe should be completely resected, and the adjacent structures should be carefully protected.
Currently, improved cosmesis is the only advantage of the
procedures that use new technology to change the standard thyroidectomy. There is no advantage in terms of complications, cost,
or recovery time. However, cosmesis is extremely important to
some patients, and in spite of the relatively good cosmesis of most
standard thyroidectomy scars, the additional cost and effort of the
newer approaches may be important to them.
8. What is the operative morbidity of thyroidectomy?
Thyroid operations are generally very safe but not minor surgery.
The risk of death is minimal; however, the risk of life-altering
complications is significant, and as they occur in people with a
long life-expectancy, they can have a lasting effect on quality of

PMPH_CH69.indd 555

NECK HEMATOMA

HYPOPARATHYROIDISM
Permanent hypoparathyroidism can occur after bilateral thyroid
procedures, but is not a risk of unilateral thyroid lobectomy. The
parathyroid glands are small, delicate structures that share a blood
supply with the thyroid gland. Their diminutive size (normal
3060 mg) and fragile nature make them particularly prone to
damage during thyroidectomy. Patients who have markedly
diminished or absent parathyroid function after thyroidectomy
have severe hypocalcemia that requires replacement. If permanent, this complication can be palliated by calcium supplements,
but this requires multiple doses each day, and uncomfortable
symptoms occur if doses are late or missed. In addition, there is
cumulative bone damage over time.
Temporary hypocalcemia occurs in about 10% of patients
after total thyroidectomy, and permanent hypocalcemia in about
1%.17-23 The temporary hypocalcemia can be severe, and requires
intravenous and oral supplementation for the duration of the
effect. Permanent hypoparathyroidism requires lifelong support with calcium supplements and vitamin D analogs. Missing

5/22/2012 5:47:55 PM

556

Surgery: Evidence-Based Practice

doses of the supplements usually produces symptoms of varying


severity, which, although manageable, are quite bothersome for
patients. In addition to the discomfort and inconvenience of the
supplements, patients develop low-turnover bone disease, which
resembles osteomalacia. Though dysmorphic, bone mass is generally preserved or increased in hypoparathyroidism, and fracture risk is not apparently increased.24 Finally, the calcium and
vitamin D supplements with low parathyroid hormone (PTH)
lead to an increased daily urinary excretion of calcium, and significant risk or nephrolithiasis.
The recent availability of pharmacologic PTH for exogenous
administration has opened the opportunity to replace PTH in
patients with postoperative hypoparathyroidism. The experience
with this to date is limited, but early results demonstrate that
PTH delivered subcutaneously twice daily can maintain serum
calcium levels in the same range as oral calcium and vitamin D
supplements, and decreases the amount of hypercalciuria.25,26
Further experience with this strategy will be necessary before the
full long-term effects are clear.

NERVE INJURIES
There are several nerves adjacent to the thyroid gland that can
be deliberately or inadvertently affected during thyroidectomy. These
include the recurrent laryngeal nerve (RLN) immediately adjacent
to the thyroid, and the vagus nerve, which is slightly more removed,
but causes the same symptoms when damaged. The external branch
of the superior laryngeal nerve can be injured during dissection of
the upper pole of the thyroid gland, and the sympathetic chain
and stellate ganglion can be injured near the posterior aspect of
the upper pole of the gland as well.
Damage to the RLN causes unilateral paralysis of the muscles
that control ipsilateral vocal cord tension. Unilateral RLN injury
changes the voice substantially in most patients, and also significantly affects the swallowing mechanism. The voice can range
from a soft, whispery voice, with the inability to increase the volume at all, to a nearly normal-sounding voice which cannot be
raised to a yell. The difference between these is based on the ability
of the contralateral vocal cord to cross the midline and appose the
affected cord. If the cords cannot meet, then the voice will be soft
and breathy. If the cords can meet, then the speaking voice will
be more normal in timbre, but the affected cord prolapses with
increased airway pressure, and the ability to yell is lost. Swallowing is affected also, and the aspiration of liquids is a mark of severe
RLN paresis. This improves with time and can be helped by swallowing training.
Bilateral RLN injury causes paralysis of both cords, and usually results in a very limited airway lumen at the cords. These
patients usually have a normal-sounding speaking voice, but
severe limitations on inhalation velocity because of upper airway obstruction. They often require re-intubation to maintain
ventilation.
RLN paresis is usually temporary, and resolves over days
to months.17-23 There is no known method of aiding or speeding
recovery. If a unilateral paresis proves to be permanent, then palliation of the cord immobility and voice changes can be achieved
with vocal cord injection or laryngoplasty. These procedures
stiffen and medialize the paralyzed cord, to allow the contralateral
cord to appose the paralyzed cord during speech. If both cords
are affected, then the palliative procedures are more limited, and

PMPH_CH69.indd 556

involve creating an adequate airway for ventilation; improvements


in voice quality are not likely, as there is no muscular control of
the cord function.
9. What methods are available to limit the operative morbidity of
thyroidectomy?
As hypoparathyroidism is not a risk of thyroid lobectomy, the
most effective strategy to avoid this complication is to limit procedures as appropriate. Preservation of each parathyroid on its
native blood supply is critical. In the case of parathyroid glands
that are devascularized during dissection, autograft of the parathyroid tissue into the sternocleidomastoid muscle can restore
normal function.27
As with hypoparathyroidism, avoidance of RLN injury is far
superior to palliation. Great care must be taken during the dissection of the nerve to protect it. In some clinical situations, the
RLN is sacrificed to allow and adequate tumor resection. Absent
this unusual circumstance, though, careful dissection can generally preserve cord function. The principles of the dissection are as
follows:
1. Avoid dividing any structures in the tracheoesophageal groove
until the recurrent laryngeal nerve is definitively identified.
Small branches of the inferior thyroid artery may seem like
they can clearly be safely transected; however, the distortion
of tumor, retraction, or previous scar may lead the surgeon to
mistakenly divide a branch of the RLN. The identifying feature
of the RLN is that the more it is dissected, the more it looks
like the correct structure. This is based on the morphologic
appearance and the anatomic course. The nerve can tolerate
manipulation, but not cutting. Once cut, repair of the nerve is
of unproven benefit.
2. Identify the recurrent laryngeal nerve low in the neck, well below
the inferior thyroid artery, at the level of the lower pole of the
thyroid gland, or below. This allows dissection of the nerve at a
site where it is not tethered by its attachments to the larynx or
its relation to the inferior thyroid artery. Traction injuries to
the nerve can occur when the nerve is manipulated near a site
of fi xation.
3. Keep the recurrent laryngeal nerve in view during the subsequent
dissection of the thyroid away from the larynx. Once the nerve
is identified, the dissection can generally proceed from inferior
to superior along the nerve, dividing the inferior thyroid artery
branches and preserving the parathyroid glands. This allows
careful dissection of the tissues with minimal manipulation of
the RLN.
4. Minimize the use of powered dissection posterior to the thyroid.
Although the electrocautery and high-frequency ultrasonic
scalpel are useful tools in dissection, they have some risk of
lateral thermal spread, which can damage adjacent tissues.
Careful cold dissection and hemostasis with ligatures or clips
will avoid this risk. This is particularly important at the entry
of the RLN to the larynx, immediately adjacent to the ligament
of Berry and its vessels.
The use of nerve stimulators and laryngeal muscle potential monitors has recently been investigated as a tool to try to limit or avoid
nerve injuries.28,29 The data do not currently support the mandatory use of these devices, as the risk of nerve injury is related to
several factors. 30-32 However, many experienced surgeons now
routinely use a nerve monitoring system intraoperatively. This

5/22/2012 5:47:55 PM

Thyroid Nodules

may be because they merely help to identify the nerve, while the
portion of the operation most likely to produce damage in experienced hands is the dissection of the RLN at the fi xed point of the
cricopharyngeus.

557

About 10% of patients have some evidence of RLN paresis after thyroidectomy; however, this resolves in most patients.
About 1% or fewer patients have permanent nerve injury when
total thyroidectomy is performed by experienced surgeons.

Clinical Question Summary


Question

Answer

1 How are thyroid nodules identified?

By local symptoms, visible asymmetry, self-palpation,


physical examination, or incidental finding on
unrelated imaging.

1, 2, 6

2 What is the appropriate clinical


evaluation for thyroid nodules?

Comprehensive history and physical examination,


TSH measurement, ultrasound evaluation,
possibly FNA as per Fig. 69.1.

1-3, 5, 6

3 What is the role of thyroid


ultrasound to evaluate thyroid
nodules?

Thyroid ultrasound is mandatory for the evaluation


of thyroid nodules.

1-3, 5

4 What is the role of FNA thyroid


cytology to evaluate thyroid
nodules?

FNA is used selectively for the evaluation of thyroid


nodules.

1-3, 5

5 What is the appropriate


nonoperative follow-up for thyroid
nodules?

Follow-up should generally occur at intervals of 3 to


12 months, depending upon the risk of the nodule
being malignant.

6 What is the role of operative


management of thyroid nodules?

Thyroid nodules with a significant risk of


being malignant after evaluation, or that
are symptomatic, should be treated by
thyroidectomy.

1, 2, 6, 8

7 What operative approaches are


available for operative management?

Standard thyroidectomy is performed under general


anesthesia, through a low cervical incision and
encompasses an overnight facility stay, but can also
be done under local anesthesia as an outpatient,
through very small neck incisions with videoscopic
guidance, or through axillary or breast incisions
using video or robotic technology.

1, 2, 13

8 What is the operative morbidity of


thyroidectomy?

Thyroid operations are generally very safe but not


minor. The risk of life-altering complications is
significant, and as they occur in people with a long
life-expectancy, they can have a lasting effect on
quality of life.

1, 12-20

9 What methods are available to


limit the operative morbidity of
thyroidectomy?

Proper surgical technique can limit the risk of


morbidity, including liberal use of parathyroid
autograft and consideration of intraoperative
laryngeal nerve monitoring.

12-20

REFERENCES
1. Cooper DS, Doherty GM, Haugen BR, et al. Revised American
Thyroid Association management guidelines for patients with
thyroid nodules and differentiated thyroid cancer.[see comment]. Thyroid. 2009;19:1167-1214.
2. Tuttle RM, Ball DW, Byrd D, et al. Thyroid carcinoma. J Natl
Compr Cancer Net. 2010;8:1228-1274.
3. Cibas ES, Ali SZ. The Bethesda System for Reporting Thyroid
Cytopathology. Thyroid. 2009;19:1159-1165.
4. Davies L, Welch HG. Increasing incidence of thyroid cancer in
the United States, 19732002. JAMA. 2006;295:2164-2167.
5. Kumar H, Daykin J, Holder R, Watkinson JC, Sheppard MC,
Franklyn JA. Gender, clinical findings, and serum thyrotropin

PMPH_CH69.indd 557

Grade of
References
Recommendation

measurements in the prediction of thyroid neoplasia in 1005


patients presenting with thyroid enlargement and investigated
by fine-needle aspiration cytology. Thyroid. 1999;9:1105-1109.
6. AACE/AME Task Force on Thyroid Nodules. American Association of Clinical Endocrinologists and Associazione Medici
Endocrinologi medical guidelines for clinical practice for the
diagnosis and management of thyroid nodules. Endoc Prac.
2006;12:63-102.
7. Gharib H, Goellner JR. Fine-needle aspiration biopsy of the thyroid: An appraisal. Ann Intern Med. 1993;118:282-289.
8. Udelsman R, Westra WH, Donovan PI, Sohn TA, Cameron JL.
Randomized prospective evaluation of frozen-section analysis for follicular neoplasms of the thyroid. Ann Surg. 2001;233:
716-722.

5/22/2012 5:47:55 PM

558

Surgery: Evidence-Based Practice

9. Mazzaferri EL. An overview of the management of papillary and


follicular thyroid carcinoma. Thyroid. 1999;9:421-427.
10. Hay ID, Grant CS, Bergstralh EJ, Thompson GB, van Heerden
JA, Goellner JR. Unilateral total lobectomy: Is it sufficient
surgical treatment for patients with AMES low-risk papillary
thyroid carcinoma? Surgery. 1998;124:958-964; discussion
964-966.
11. Chen H, Nicol TL, Zeiger MA, et al. Hurthle cell neoplasms of
the thyroid: Are there factors predictive of malignancy? Ann
Surg. 1998;227:542-546.
12. Burkey SH, van Heerden JA, Thompson GB, Grant CS, Schleck
CD, Farley DR. Reexploration for symptomatic hematomas after
cervical exploration. Surgery. 2001;130:914-920.
13. Adler JT, Sippel RS, Schaefer S, Chen H. Preserving function
and quality of life after thyroid and parathyroid surgery. Lancet
Oncol. 2008;9:1069-1075.
14. Leyre P, Desurmont T, Lacoste L, et al. Does the risk of compressive hematoma after thyroidectomy authorize 1-day surgery?
Langenbecks Arch Surg. 2008;393:733-737.
15. Rosenbaum MA, Haridas M, McHenry CR. Life-threatening
neck hematoma complicating thyroid and parathyroid surgery.
Am J Surg. 2008;195:339-343; discussion 343.
16. Harding J, Sebag F, Sierra M, Palazzo FF, Henry J-F. Thyroid surgery: Pstoperative hematomaprevention and treatment. Langenbecks Arch Surg. 2006;391:169-173.
17. Farrar WB, Cooperman M, James AG. Surgical management of
papillary and follicular carcinoma of the thyroid. Ann Surg. 1980;
192:701-704.
18. Thompson NW, Nishiyama RH, Harness JK. Thyroid carcinoma:
Current controversies. Curr Probl Surg. 1978;15:1-67.
19. Schroder DM, Chambous A, France CJ. Operative strategy for
thyroid cancer, is total thyroidectomy worth the price? Cancer.
1986;58:2320-2328.
20. Clark OH, Levin K, Zeng QH, Greenspan FS, Siperstein A. Thyroid cancer: The case for total thyroidectomy. Eur J Cancer Clin
Oncol. 1988;24:305-313.
21. Ley PB, Roberts JW, Symmonds J, et al. Safety and efficacy of
total thyroidectomy for differentiated thyroid carcinoma: A 20year review. Am Surg. 1993;59:110-114.

PMPH_CH69.indd 558

22. Tartaglia F, Sgueglia M, Muhaya A, et al. Complications in total


thyroidectomy: Our experience and a number of considerations.
Chirurgia Italiana. 2003;55:499-510.
23. Rosato L, Avenia N, Bernante P, et al. Complications of thyroid
surgery: Analysis of a multicentric study on 14,934 patients operated on in Italy over 5 years. World J Surg. 2004;28:271-276.
24. Rubin MR, Dempster DW, Zhou H, et al. Dynamic and structural properties of the skeleton in hypoparathyroidism. J Bone
Miner Res. 2008;23:2018-2024.
25. Winer KK, Ko CW, Reynolds JC, et al. Long-term treatment of
hypoparathyroidism: A randomized controlled study comparing parathyroid hormone-(1-34) versus calcitriol and calcium.
J Clin Endocrinol Metab. 2003;88:4214-4220.
26. Winer KK, Sinaii N, Peterson D, Sainz B, Jr., Cutler GB, Jr. Effects
of once versus twice-daily parathyroid hormone 1-34 therapy in
children with hypoparathyroidism. J Clin Endocrinol Metab.
2008;93:3389-3395.
27. Olson JA, Jr, DeBenedetti MK, Baumann DS, Wells SA, Jr. Parathyroid autotransplantation during thyroidectomy. Results of
long-term follow-up.[see comment]. Ann Surg. 1996;223:472-478;
discussion 478-480.
28. Rea JL, Khan A. Clinical evoked electromyography for recurrent
laryngeal nerve preservation: Use of an endotracheal tube electrode and a postcricoid surface electrode. Laryngoscope. 1998;108:
1418-1420.
29. Otto RA, Cochran CS. Sensitivity and specificity of intraoperative recurrent laryngeal nerve stimulation in predicting postoperative nerve paralysis. Ann Otol, Rhinology & Laryngology.
2002;111:1005-1007.
30. Dralle H, Sekulla C, Lorenz K, Brauckhoff M, Machens A, German ISG. Intraoperative monitoring of the recurrent laryngeal
nerve in thyroid surgery. World J Surg. 2008;32:1358-1366.
31. Dralle H, Sekulla C, Haerting J, et al. Risk factors of paralysis
and functional outcome after recurrent laryngeal nerve monitoring in thyroid surgery. Surgery. 2004;136:1310-1322.
32. Thomusch O, Sekulla C, Machens A, Neumann H-J, Timmermann W, Dralle H. Validity of intra-operative neuromonitoring signals in thyroid surgery. Langenbecks Arch Surg. 2004;389:
499-503.

5/22/2012 5:47:56 PM

CHAPTER 70

Hyperthyroidism, Thyroiditis,
and Nontoxic Goiter
Prashant Khullar and Geeta Lal

et al.2 noted good correlation between the studies in a retrospective review of 273 patients. (Level 2b evidence) I-123 is both
trapped and organified by the thyroid, has less salivary activity, is
orally administered, and is more effective than Tc-99m at identifying pyramidal lobes and thyroglossal duct remnants. However,
it is more expensive and takes longer to complete. In contrast to
Graves disease, thyroid scans typically demonstrate single or multiple hot nodules with suppression of the remaining gland in STN
and toxic MNG, respectively.
Answer: Hyperthyroidism is diagnosed using a combination
of laboratory tests and, in some cases, thyroid scan and uptake
to determine etiology. The latter can be performed with I-123 or
with pertechnetate. (Grade B recommendation)

INTRODUCTION
Hyperthyroidism is a clinical syndrome resulting from excessive
production and release of thyroid hormone into the circulation.
The commonest cause is Graves disease, accounting for nearly
60% to 80% of the cases of hyperthyroidism in North America.
Other causes which are common and are relevant to surgeons are
toxic multinodular goiter (MNG) and solitary toxic nodule (STN).
Thyroiditis is defined as an inflammatory disorder of the thyroid
gland. It may result from a myriad of etiologies and is classified
into acute, subacute, and chronic forms, each associated with a
distinct clinical presentation and histology. Nontoxic goiters are
diff use or nodular enlargements of the thyroid. Thyroid nodules
occur in about 4% of the general population; however, most are
benign and do not require any intervention. In this chapter, we
review and provide evidence-based guidelines for the diagnosis
and management of these common conditions, with particular
focus on surgical therapy.

2. What is the optimal defi nitive treatment of hyperthyroidism?


Three treatment options are available for Graves disease: (1) antithyroid drug (ATD) therapy, (2) surgery, and (3) RAI ablation.
ATDs (propylthiouracil (PTU) and methimazole) act by decreasing thyroid hormone production by inhibiting the organification
of iodine and the coupling of iodotyrosines. PTU also acts by
inhibiting the peripheral conversion of T4 to T3. ATDs can be used
as both a primary treatment and an adjunct modality in preparation for RAI or for surgery. When used as primary therapy, they
are given for 6 months to 2 years, till spontaneous remission of
the disease occurs. At the initiation of treatment, higher doses are
used till a clinically euthyroid state is achieved and the dose is
then titrated according to FT4 levels. Drawbacks of ATDs include
a high relapse rate (4050% by 1 year and 6070% by 10 years),
once therapy is stopped. Adverse effects include rash, agranulocytosis (less than 0.5%), and possible hepatocellular toxicity. Thus,
ATD treatment is suitable for small glands (<40 g), patients with
mild hyperthyroidism, and those who show substantial decrease
in gland size and remission of symptoms soon after therapy is initiated. ATDs are often used in conjunction with -blockers to help
control symptoms related to catecholamine release and have the
added benefit of decreasing peripheral conversion of T4 to T3.

1. What are the appropriate diagnostic tests (laboratories and


imaging) for hyperthyroidism?
A suppressed TSH with or without elevation of free T3 and/or
T4 levels is sufficient to make a diagnosis of hyperthyroidism.1
The presence of eye signs of Graves ophthalmopathy (GO, clinically apparent in 2025% of patients) along with consistent labs
is sufficient to make a diagnosis without further testing. Antithyroglobulin (Tg) and anti-thyroid peroxidase (TPO) antibodies
are present in both Graves disease and autoimmune thyroiditis
and are therefore nonspecific. TSH receptor antibodies (TRAb)
are specific to Graves disease and elevated levels are diagnostic.
TRAb levels are useful in patients who present with unilateral eye
signs or minimal clinical manifestations of hyperthyroidism. In
the absence of eye signs, an I-123 radioactive iodine scan showing
an elevated uptake with diff use enlargement of the gland confirms
the diagnosis. Either I-123 or technetium-99m pertechnetate
(Tc-99m) may be used for thyroid imaging. There are no randomized studies comparing these modalities; however, Reschini
559

PMPH_CH70.indd 559

5/22/2012 5:48:26 PM

560

Surgery: Evidence-Based Practice

RAI therapy is the most common modality used for the


defi nitive treatment of Graves disease in the United States. Frequently, patients require pretreatment with ATDs to achieve
a euthyroid state prior to treatment. Euthyroidism is usually
achieved in 70% of patients by 2 months, although it can take
up to 6 months to see the full therapeutic effect. Almost 80%
of adequately treated patients eventually develop hypothyroidism. Advantages of RAI therapy include lower cost, avoidance
of surgery, and long-term drug treatment and their associated
side effects. RAI therapy is contraindicated in pregnant and
breast-feeding patients. Surgery is recommended for treatment of
Graves disease when other modalities of treatment are contraindicated. Absolute indications for surgery include patients with
nodules suspicious of or proven to be cancer, pregnant patients
not controlled with ATDs, women desiring pregnancy within 6 to
12 months, patients who had severe allergic/adverse reactions to
ATDs, and patient preference. Large glands are less responsive
to ATDs or RAI are best treated surgically, especially if they are
causing compressive symptoms.
The question of which treatment is optimal has been a matter of considerable debate. Torring et al.3 attempted to address
this issue and randomized 179 patients with Graves disease to
3 groupsATDs, RAI, or thyroidectomy. (Level 2b evidence) All
modalities normalized the mean serum hormone levels within
6 weeks. The risk of relapse was highest in the medically treated
and lowest in surgically treated patients. The majority of patients
in each group (90%) were satisfied with the treatment received.
Thus, the results from this study did not allow any definitive recommendations regarding the optimal therapy for Graves disease.
Although a recent cost-effectiveness study (Level 3b evidence)
demonstrated that thyroidectomy was the best option compared
with lifelong ATD and RAI (till the cost of surgery exceeds $19,300)
for patients who failed to achieve euthyroidism after initial medical treatment,4 additional studies are clearly needed in this area.
In contrast, several investigators have evaluated the optimal
therapy in patients with Graves disease and GO. In a prospective, randomized study, Tallstedt et al.5 reported progression of
GO more frequently following RAI when compared with patients
undergoing surgery (32% vs. 16%), respectively. This led to the recommendation for avoidance of RAI in patients with GO (Level
2b evidence), although later studies showed that glucocorticoid
therapy ameliorates this worsening after either therapy. Another
randomized study (Level 2b evidence) of 60 patients showed that
complete thyroid ablation (with near-total thyroidectomy (TT)
and RAI) was superior to near-TT in terms GO outcomes (0% vs.
25% progression, respectively, at 9 months); however, no long-term
data are available.6 Other casecontrol studies also show similar
results. In terms of novel therapies, small uncontrolled studies show that Rituximab (RTX, a monoclonal chimeric human/
mouse antibody directed against the surface molecule CD20)
resulted in significant eye symptom improvement in addition
to reduced relapse rates and that it may be particularly useful in
steroid resistant GO.
There are no prospective randomized studies in the English literature evaluating the various treatment options for toxic
MNG and STN; however, retrospective case series support the
role of both RAI and surgery for these diseases. The doses of RAI
needed in toxic MNG are much higher than those needed for
Graves and ATDs may be needed concurrently. The results for
RAI therapy of STNs are variable and larger nodules typically

PMPH_CH70.indd 560

require higher doses, which can lead to hypothyroidism in about


12% of patients. Long-term success rates for RAI ablation have
been reported at 88% for toxic MNG and 100% for STN.7 (Level 4
evidence) Surgery is indicated for large, symptomatic goiters, as
it not only provides rapid control of the hyperthyroidism but also
relieves compressive symptoms. Surgery also provided early and
late cure rates of 99% to 100%.7 (Level 4 evidence) For younger
patients, surgery may also be a more cost-effective option (Level 3b
evidence), provided complication rates are low. 8 Percutaneous
ethanol injection (PEI) has been described in a prospective nonrandomized multicenter trial of 429 patients with STNs and
reported success rates of 67% for patients with toxic nodules
and 83% of patients with pretoxic nodules at 1 year follow-up.9
Another randomized-controlled trial compared US-PEA + RAI
with RAI alone in 22 patients with large STN (>4cm) who were
not operative candidates or refused surgery. The former was
associated with a success rate of 79% versus 57% with the latter,
supporting the adjuvant role of US-PEA in a selected group of
patients.10 (Level 2b evidence) Of note, no studies directly compare this modality with surgery.
Answer: There is insufficient evidence to recommend surgery over RAI or ATDs for Graves disease and treatment should
be based on the general guidelines in combination with patient
preference. However, patients with GO are best treated by surgery. (Grade B recommendation) Toxic MNGs and STNs may
be treated with thyroidectomy or RAI. (Grade C recommendation) PEA may have a role in selected patients with STN. (Grade C
recommendation)
3. What is the optimal extent of surgery for hyperthyroidism?
Graves disease may be treated by TT or subtotal (ST) resection
(leaving a 47 g remnant). The latter may be accomplished by
bilateral ST or a unilateral total lobectomy combined with a
contralateral ST thyroidectomy (HartleyDunhill procedure).
The debate around the extent of surgery is primarily related
to the previously reported higher complication rates with TT.
However, Palit et al.,11 in a meta-analysis of 35 studies including 7241 patients, noted that there was no difference in the permanent recurrent laryngeal nerve injury rates (0.9% vs. 0.7%) or
hypoparathyroidism (0.9% vs. 1%) between patients undergoing
TT or ST, respectively. (Level 3a evidence) None of the patients
undergoing TT had persistent or recurrent disease, but the recurrence rate was 7.9% in the ST group. Witte et al.12 performed a
prospective randomized trial on 150 patients with Graves disease comparing bilateral ST resection-total remnant <4 mL,
unilateral hemithyroidectomy with contralateral ST resection
remnant <4 mL and TT with respect to outcomes. Recurrent
hyperthyroidism occurred in two patients with ST resections,
and although early postoperative hypoparathyroidism was more
frequently detected in patients with TT than in those with ST
resection (28% vs. 12%; p < .002), there was no difference in the
permanent complication rates. (Level 2b evidence) Multiple retrospective casecontrol studies have also shown equivalent complication rates for TT and ST in Graves disease.13-15 (Level 3a
evidence) Of note, small randomized-control studies do not
show any difference in outcomes between the two types of ST
resections.16,17 (Level 2b evidence) Although several retrospective studies have shown less GO progression with TT, most likely
as a result of removal of the antigenic stimulus, only two small

5/22/2012 5:48:26 PM

Hyperthyroidism, Thyroiditis, and Nontoxic Goiter

prospective, randomized studies (Level 2b evidence) have evaluated this question and did not show any difference between ST
and TT.12,18 There are no Level 1 or 2 studies evaluating the optimal extent of thyroidectomy in the treatment of toxic MNG or
STN although retrospective case series (Level 4 evidence) show
that more limited resections have lower complication rates.7 This
must, however, be balanced with higher recurrence rates. Lugols
iodine is often used preoperatively and has been shown to reduce
the rate of blood flow, thyroid vascularity, and blood loss during
thyroidectomy for Graves disease in a small randomized, prospective study of 36 patients (Level 2b evidence),19 although other
retrospective studies fail to show any benefit.
Answer: Based on the available evidence regarding cure and
complication rates, TT has become the procedure of choice for
Graves disease. (Grade B recommendation) In GO, TT does not offer
any advantage over ST resection and either procedure is appropriate. (Grade B recommendation) Toxic MNG may be treated by TT
or ST resection, whereas lobectomy is preferred for STN. (Grade C
recommendation) Iodine may be used to decrease the vascularity of
the gland. (Grade E recommendation)
4. What are the appropriate diagnostic tests (laboratories and
imaging) for thyroiditis?
Acute suppurative thyroiditis is often characterized by leukocytosis and an elevated ESR. Serial thyroid function studies may
show transient elevations of T3 and T4 as a result of the release of
preformed hormone from the inflamed gland. RAI uptake scans
are usually normal or there is a decreased uptake due to suppression of TSH by the release of thyroid hormones. Ultrasound
is helpful to distinguish solid from cystic lesions. Fine needle
aspiration biopsy (FNAB) for gram stain, culture, and cytology
confirms the diagnosis, helps guide antibiotic therapy, and diagnose underlying malignancy. Contrast-enhanced CT scans may
help to identify abscesses and delineate the extent of infection. A
persistent pyriform sinus fistula should always be suspected in
recurrent acute thyroiditis. In a consecutive series of 18 patients,
Kim et al.20 reported that the sensitivity of identification of fistulae in the acute setting was lowest for barium esophagography
(50%) and best for direct endoscopy (100%), with CT scans being
intermediate (80%). In addition, another series of 21 patients
showed that both barium esophagogram and CT scans (especially if the trumpet maneuver is used) have improved sensitivity once the acute inflammation has resolved (100% and 83%,
respectively), with CT being better at defi ning the accurate anatomic pathway and its relationship to the thyroid gland.21 (both
Level 3b evidence)
Subacute thyroiditis can occur in the painful (DeQuervains
thyroiditis) or painless forms, the latter may occur sporadically or
in the postpartum period. In the early stages, TSH is decreased,
and Tg, T4, and T3 levels are elevated due to the release of preformed
thyroid hormone from destroyed follicles. Thyroid antibody titers
(anti-Tg, anti-microsomal, and TSH receptor antibody) are also
elevated in 10% to 20% of patients. Titers of anti-TPO antibodies are often elevated in painless thyroiditis and these patients
have a risk of recurrence of approximately 70% following a subsequent pregnancy. ESR is typically greater than 100 mm/h in
painful thyroiditis, but is generally normal or only mildly elevated
(<40 mm/h) in painless thyroiditis and the leukocyte counts are
generally normal. RAI uptake is also decreased (<2% at 24 h) in

PMPH_CH70.indd 561

561

subacute thyroiditis. FNAB may be useful in equivocal cases or


to rule out malignancy or acute thyroiditis. Thyroid ultrasound
shows areas of hypoechogenicity which reflect the degree of
inflammation. A recent study showed that RAI uptake (and thyroid scintigraphy) was more sensitive than ultrasound (100% vs.
36%) for the diagnosis.22 (Level 2b evidence)
Chronic thyroiditis includes Lymphocytic (Hashimotos)
thyroiditis and a rare variant known as Riedels struma (invasive fibrous thyroiditis). When Hashimotos thyroiditis is suspected clinically, an elevated TSH, reduced T4 and T3 levels,
and the presence of thyroid autoantibodies confi rm the diagnosis. Antibodies are directed against three main antigensTg
(60 %), TPO (95 %), the TSH-R (60 %), and less commonly to
the sodium/iodine symporter (25%). FNAB is indicated in
patients who present with a solitary suspicious nodule or a rapidly enlarging goiter as thyroid lymphoma is a rare but ominous
complication of chronic autoimmune thyroiditis. Reidels thyroiditis is characterized by the replacement of all or part of the
thyroid parenchyma by fibrous tissue, which also invades into
the adjacent tissues. As fibrosis progresses, an elevated TSH and
hypocalcemia may be apparent. Anti-thyroid antibodies and a
mild eosinophilia may be present. FNAB is generally inadequate
due to the fibrotic nature of the gland.
Answer: Thyroiditis can be diagnosed with a careful examination of laboratory tests and imaging. Due to the increased risk
of recurrence, screening for anti-TPO antibodies should be considered in women who are at high risk and pregnant. (Grade B recommendation) RAI uptake scans, ultrasound (FNAB), and CT
scans are often necessary (Grade B recommendation) to delineate
the diagnosis and define anatomic abnormalities.
5. What are the indications for surgical intervention in
thyroiditis?
There are no randomized clinical trials directing therapy. Once
the diagnosis of acute suppurative thyroiditis is suspected,
empiric, broad spectrum antibiotic therapy must be initiated.
In the absence of airway complications, surgical intervention is
guided by the presence, persistence, or recurrence of abscesses.
Some case reports suggest that in the absence of an abscess on
CT imaging, acute thyroiditis may be successfully treated with
antibiotics alone or in combination with US surveillance and
drainage of small abscesses. Surgery may include open surgical
drainage or thyroidectomy (lobectomy, near-TT, or TT). In case a
pyriform sinus fistula is identified, complete surgical or nonsurgical obliteration is needed to reduce the risk of recurrence.23,24
(Level 3b evidence) Transnasal flexible fiberoptic laryngoscopy
is being increasingly used to identify the internal opening of the
pyriform sinus tract and may also allow electrocauterization of
the tract and success rates similar to open surgery have been
reported.
Subacute thyroiditis is typically self-limited and therefore
treatment is primarily symptomatic. Aspirin and other nonsteroidal anti-inflammatory drugs are used for pain relief, but steroids
may be indicated in more severe cases of deQuervains thyroiditis. Short-term thyroid replacement may shorten the duration of
symptoms and is advised for patients with TSH >10 and pregnant women as pregnancies in women with subclinical hypothyroidism were 3 times more likely to be complicated by placental
abruption (relative risk 3.0, 95% confidence interval [CI] 1.18.2)

5/22/2012 5:48:27 PM

562

Surgery: Evidence-Based Practice

and preterm birth, (relative risk, 1.8, 95% CI 1.12.9).25 (Level 2b


evidence) In a retrospective review of 17 patients who underwent thyroidectomy for deQuervains thyroiditis, the most common indication was an indeterminate thyroid nodule; however,
many of these patients were operated on prior to the era of routine
FNAB.26 (Level 4 evidence)
Thyroid hormone replacement therapy is also indicated in
overtly hypothyroid patients with chronic thyroiditis, with a goal
of maintaining normal TSH levels. The management of patients
with subclinical hypothyroidism (normal T4 and elevated TSH)
is controversial. The condition is typically asymptomatic; however, a systematic review of cohort studies shows that in age and
sex-adjusted analyses, subclinical hypothyroidism is associated
with a hazard ratio (HR) for coronary heart disease events of 1.89
(95% CI 1.282.80, p < .001) and coronary heart disease mortality of 1.58 (95% CI 1.102.27, p = .005) for a TSH level of 10 to
19.9 mIU/L.27 (Level 2a evidence) The data for TSH levels 5 to
10 were less convincing. An evaluation of the 12 randomizedcontrolled trials in this area only showed a trend toward improvement of some lipid parameters (Level 1a evidence) and none of the
included trials evaluated overall mortality or cardiac morbidity.28
Patients with anti-TPO antibodies are at highest risk of progression to overt hypothryodism. When needed, thyroidectomy for
Hashimotos thyroiditis can be performed with low complication
rates.29 (Level 4 evidence) Surgery is the mainstay of the treatment
of Reidels thyroiditis, with the chief goal being decompression of
the trachea by wedge excision of the thyroid isthmus and for tissue
diagnosis. Corticosteroids, tamoxifen and more recently mycophenolate mofetil have been used to attenuate the inflammatory
process and have led to dramatic symptom improvements in small
case series.
Answer: Treatment for thyroiditis is primarily medical and
Levothyroxine is recommended for all patients with TSH levels
>10 uIU/mL and patients with levels of 5 to 10 uIU/mL in the
presence of a goiter or anti-TPO antibodies. (Grade B recommendation) Surgical treatment may be emergently needed in acute
thyroiditis for airway compromise, persistence or progression
of abscess(es) or concern for malignancy. Thyroidectomy is preferred for poorly defined, multiple abscesses that progress despite
antibiotics or if open drainage fails. A pyriform sinus fistula, if
present, should be obliterated electively by surgical or nonsurgical means (Grade B recommendation) to avoid recurrence. For
subacute and chronic thyroiditis, thyroidectomy is reserved for
patients with proven cancer/indeterminate nodules. Thyroidectomy is also indicated for the rare patient who has a prolonged
course not responsive to medical measures or for recurrent,
symptomatic disease. (Grade C recommendation) In addition,
open surgical biopsy may be needed to diagnose lymphoma or
Reidels thyroiditis.
6. What are the essential diagnostic tests in the workup of a
nontoxic goiter?
A complete history and physical examination is warranted with
specific questions directed toward determining a patients risk for
thyroid malignancy. A history of childhood radiation exposure
for head and neck tumors or lymphomas, whole body irradiation
for hematologic malignancy, family history of thyroid cancer or
cancer syndrome and a history of rapid growth, or a hoarse voice

PMPH_CH70.indd 562

at presentation increases likelihood of malignancy. Careful attention should be paid toward evaluating compressive symptoms
such as dysphagia and dyspnea. Physical examination showing
fi xation, enlarged cervical lymph nodes, and vocal cord palsy also
suggests cancer.
A TSH measurement is indicated as nontoxic goiters may
develop autonomous function. If low or suppressed, free T4
and T3 levels should be measured. High serum TSH is associated with a higher risk of malignancy in nodular goiters as
reported by Boelaert et al.30 who noted adjusted odds ratios for
cancer ranging from 2.72 to 3.88 for TSH levels >0.45.5 mU/L
at presentation in their cohort study of 1500 patients. (Level 2b
evidence) Serum Tg levels can be elevated in many benign thyroid conditions and are an insensitive and nonspecific indicator
of malignancy. In contrast, calcitonin levels are a sensitive and
specific marker of medullary thyroid cancer (MTC) with several
prospective, nonrandomized cohort studies indicating that routine screening results in early detection of MTC and improved
cure rates.31-33 (Level 2b evidence) Another study has also demonstrated the cost-effectiveness of this approach and shown that
screening for MTC in the United States would yield an additional
113,000 life years at a cost increase of 5.3% (Level 3b evidence),
a level which is comparable with mammography and colonoscopy.34 However, the prevalence estimates in this analysis also
included patients with C-cell hyperplasia/microscopic carcinomas which are of unclear clinical significance and the tests may
be falsely positive.
Thyroid ultrasonography is crucial in the evaluation of a
nontoxic goiter and can be used not only to determine the size,
nature, and number of nodules but also to evaluate for cervical lymphadenopathy. Ultrasound characteristics are also being
increasingly used to predict the risk of malignancy and thus
select nodules for FNAB. In a multicenter study of 831 patients by
Moon et al., 35 statistically significant (p < .05) fi ndings of malignancy were a taller-than-wide shape, a spiculated margin, marked
hypoechogenicity, microcalcification, and macrocalcification
(Level 2b evidence). Similar fi ndings were noted in other studies
which also noted that the absence of a halo and the presence of
suspicious cervical lymphadenopathy also predict a higher risk
of malignancy.36,37 (Level 2b evidence) Ultrasound can also direct
image-guided FNAB and improve diagnostic yield especially in
nonpalpable nodule or nodules with a mixed solid-cystic component. It has been shown to alter the management of up to 63%
of patients referred after a palpable abnormality was found on
physical examination.38 (Level 2b evidence) CT scans and MRI
scans may be needed to evaluate for substernal extent, changes
in size, and evidence of tracheal compression. The latter may also
be assessed by pulmonary function tests or by direct laryngoscopy. Barium esophagograms may be needed for evaluation of
dysphagia. (Level 4 evidence)
Answer: A serum TSH level should be the first laboratory
test obtained in any patient with an apparent nontoxic goiter.39 If
the serum TSH is suppressed, a radionuclide thyroid scan should
be performed to determine whether there are hyperfunctioning nodules. (Grade B recommendation) Routine Tg measurements are not recommended, but there is insufficient evidence
for or against routine calcitonin measurement. A diagnostic thyroid ultrasound is also recommended in all patients who present with MNG. (Grade B recommendation) Additional imaging

5/22/2012 5:48:27 PM

Hyperthyroidism, Thyroiditis, and Nontoxic Goiter

may be obtained to assess for extent or as indicated by symptoms.


(Grade E recommendation)
7. What is the role of FNAB in the diagnosis of nontoxic
goiter?
FNAB has emerged as an accurate and low-cost diagnostic test
in the workup of nontoxic goiters. Several retrospective studies
have shown that ultrasound-guided FNAB has a lower rate of
nondiagnostic and false-negative cytology compared with those
using palpation alone.30,40,41 (Level 2b evidence) Optimum cytology specimens should have at least six follicles each containing at
least 10 to 15 cells from at least two aspirates. Patients with multiple thyroid nodules have the same risk of malignancy as those
with STN and the risk is independent of nodule size.36,38 (Level 2b
evidence) Therefore, ultrasound characteristics can be used for
selection of nodules for FNAB. Of note, the sensitivity of microcalcifications as a predictor of malignancy in nodules 1cm or
less in size is lower than that in larger nodules (36.6% vs. 51.4%,
p < .05).35 This finding should prompt the evaluation of lateral
lymph nodes and possibly biopsy as even small papillary cancers
can be clinically significant and prone to recurrence. FNAB is not
needed in patients with nonnodular goiters. Ultrasound elastography provides an estimation of tissue stiff ness and has shown
high sensitivity and specificity (92% and 90%, respectively) in predicting malignancy in thyroid nodules in a recent meta-analysis
(Level 2a evidence) and may be comparable with FNAB.42
Answer: FNAB is the diagnostic procedure of choice in the
evaluation of nodular nontoxic goiters. (Grade B recommendation) Ultrasound elastography has potential utility as a predictor
of malignancy. (Grade C recommendation)
8. What are the indications for thyroidectomy in nontoxic goiter and is there a role for molecular markers in clinical decisionmaking?
Typically, FNAB results are divided into four categories
malignant (risk of cancer at excision >95%), indeterminate/suspicious for neoplasm, benign or nondiagnostic. A recent NCI state of
the Science conference proposed two additional classifications
suspicious for malignancy (risk of cancer 5075%) and follicular lesion of undetermined significance (risk of cancer 510%).
In addition, the term indeterminate was replaced by follicular
or Hrthle cell neoplasm (risk of cancer 2030%). In the latter
situation, the diagnosis of malignancy requires the demonstration
of capsular and/or vascular invasion. Some authors recommend
obtaining a radionuclide scan in patients with a follicular neoplasm and low or low-normal TSH (level 5), as the risk of cancer
is lower in this setting. The use of molecular markers can improve
diagnostic accuracy of FNAB, particularly for indeterminate
nodules, as shown in several recent prospective nonrandomized
studies. Bartolazzi et al.43 showed that galectin-3 expression was
78% (95% CI 7482) sensitive and 93% (95% CI 9095) specific
for the diagnosis of cancer. Nikiforov et al.44 evaluated mutations
in BRAF, RAS, RET/PTC, and PAX8/PPAR gamma and noted
that any of these mutations was a strong predictor of malignancy
(97%). (Level 2b evidence)
Patients with nodular nontoxic goiter in whom FNAB shows
benign findings do not need additional treatment, but should be
followed up with serial ultrasounds and physical examinations, due

PMPH_CH70.indd 563

563

to the inherent risk of false-negative biopsies (up to 5%),41,45 which


some studies report is higher for nodules larger than 4 cm. (Level 3b
evidence) Nodule growth by itself is not diagnostic of malignancy,
but nodules which increase in size warrant a repeat FNA. There is no
consensus on what constitutes a nodule growth or what thresholds
should be used for a re-biopsy; however, some investigators suggest that a 50% increase in nodule volume is the minimally significant and reproducible change in size. Additional studies show that
repeat biopsy increases the diagnostic accuracy of benign lesions to
98%.46 (Level 3b evidence) Biopsies that yield nondiagnostic results
are typically complex nodules, and repeat biopsy under ultrasound
guidance has been shown to yield adequate specimens in 91% series
of patients with initial nondiagnostic biopsy. Up to 7% of nodules
may be nondiagnostic despite repeat biopsies and some will be
malignant at excision.47 (Level 4 evidence)
Although RAI has been shown to decrease gland volume
and improve compressive symptoms in some patients with MNG
(Level 4 evidence),48 symptomatic patients are best treated by thyroidectomy. The presence of a significant substernal component
is also considered to be an indication for thyroidectomy by many
authors (Level 5 evidence) due to the general inaccessibility of
nodules in this area for FNAB and the ongoing ultrasound surveillance. The extent of surgery has remained a matter of debate;
however, results from a randomized-control study of 600 patients
showed that TT, although associated with higher transient complication rates, leads to significantly lower recurrence and reoperation rates when compared with ST resections (HartleyDunhill
or bilateral ST).49 (Level 1b evidence)
Answer: Nontoxic goiters in which the FNAB is diagnostic
or suspicious for malignancy should undergo thyroidectomy.39 If
the FNAB is consistent with a follicular neoplasm (with a normal
TSH) or a Hurthle cell neoplasm, either a lobectomy or a TT may
be performed, depending on lesion size, presence of other nodules, and risk factors.39 Molecular markers may be considered in
selected cases to guide the management of indeterminate cytology. (Grade C recommendation) Cytologically benign nodules
should be followed up by serial ultrasounds 6 to 18 months after
initial FNA. Enlarging nodules that are benign on repeat FNA or
those that yield persistently nondiagnostic results should be considered for surgical removal especially if clinically concerning.
(Grade C recommendation) Symptomatic goiters are best treated
by thyroidectomy, with TT becoming the preferred approach.
(Grade B recommendation)
9. Is there a role for medical therapy in the treatment of nontoxic nodular goiter?
TSH suppression may have a role in medical management of nontoxic goiter in areas with a high prevalence of iodine deficiency
and can decrease nodule size and potentially prevent the growth
of new nodules. In iodine-sufficient populations, the data have
been less impressive. Randomized-controlled trial analyses have
shown that less than 25% of benign nodules shrink more than 50%
with TSH suppression in iodine-replete populations.50,51 (Level 1b
evidence) In addition, a meta-analysis of existing randomizedcontrolled trials did not show a significant effect of levothyroxine
treatment.52 (Level 1a evidence)
Answer: Routine TSH suppression for nodular nontoxic goiters is not recommended. (Grade A recommendation)

5/22/2012 5:48:27 PM

564

Surgery: Evidence-Based Practice

Clinical Question Summary


Question

Answer

1 What are the


appropriate
diagnostic tests
(laboratories
and imaging) for
hyperthyroidism?

TSH, free T3, T4 levels are the essential tests.

2 What is the optimal


definitive treatment
of hyperthyroidism?

Levels of
Evidence

Grade of
References
Recommendation

Published
guidelines
2b

A*

Insufficient evidence to recommend surgery


over RAI or ATDs for Graves disease.
Patients with GO are best treated by
surgery.
Toxic MNGs and STNs may be treated with
thyroidectomy or with RAI.
PEA may have a role in selected patients
with toxic STN.

2b

2b

5, 6

2b

9, 10

TT is the procedure of choice in Graves


disease.
Either total or sub-TT may be performed in
patients with GO.
Toxic MNG can be treated with TT or
ST resection, whereas lobectomy is
preferred for STN.

2b and 3a

11-15

2b

12, 18

4 What are the


appropriate
diagnostic tests
(laboratories
and imaging) for
thyroiditis?

CBC, serum TSH, Free T3 and T4, antithyroid antibodies (particularly anti-TPO),
ESR
RAI uptake, ultrasound (FNAB), and CT
scans are helpful for diagnosis and the
delineation of anatomic abnormalities.

Published
guidelines

A*

2b and 3b

20-22

5 What are the


indications
for surgical
intervention in
thyroiditis?

Treatment is primarily medical and


Levothyroxine is recommended for all
patients with TSH levels >10 uIU/mL and
patients with levels of 5 to 10 uIU/mL
in the presence of a goiter or anti-TPO
antibodies.
A pyriform sinus fistula, if identified, should
be obliterated surgically or nonsurgically
to reduce recurrence.
Patients with clinico-pathologic features
concerning for malignancy or
indeterminate nodules should be treated
with surgery thyroidectomy, which may
also be needed to diagnose Reidels
thyroiditis or lymphoma.
Thyroiditis refractory to prolonged medical
treatment or recurrent, symptomatic
thyroiditis may be treated with
thyroidectomy.

1a and 2b

25, 27, 28

3b

23, 24

Published
guidelines

A*

26, 29

Serum TSH is recommended for all patients.

Published
guidelines
2b

A*

39, 30

30

2b

35-38

3 What is the
optimal extent
of surgery for
hyperthyroidism?

6 What are the


essential diagnostic
tests in the workup
of a nontoxic
goiter?

RAI uptake scan or pertechnetate scans


to determine the presence of hot or
cold nodules and delineate etiology of
hyperthyroidism.

A radionuclide scan should be performed in


patients with a low-normal TSH.
Neck ultrasound should be performed in all
patients to evaluate for any nodularity
and assess its characteristics.

(Continued)

PMPH_CH70.indd 564

5/22/2012 5:48:27 PM

Hyperthyroidism, Thyroiditis, and Nontoxic Goiter

565

(Continued)
Question

Answer

7 What is the role


of FNAB in the
diagnosis of
nontoxic goiter?

FNAB is the diagnostic procedure of choice


for thyroid nodules.
Ultrasound elastography may have potential
utility in predicting malignancy.

2b

40, 41

2a

42

8 What are the


indications for
thyroidectomy in
nontoxic goiter and
is there a role for
molecular markers
in decision-making?

Thyroid nodules with cytopathologic


evidence diagnostic or concerning for
malignancy, Hurthle cell neoplasm, or
follicular neoplasm (with a normal TSH)
should be treated with thyroidectomy.
Molecular markers can be useful in decisionmaking for selected patients with
indeterminate nodules; however, the
exact combination of markers remains to
be determined.
Cytologically benign nodules increasing
in size (benign by repeat FNA) may be
considered for thyroidectomy based
on clinical concern (due to risk of falsenegative biopsy).
Nodules with persistent nondiagnostic
cytology should be considered for
excision.
Symptomatic nontoxic goiters are best
treated by thyroidectomy in patients who
are surgical candidates, with TT being the
procedure of choice.

Published
guidelines

A*

39

2b

43, 44

3b

41, 45

47

1b

49

Routine TSH suppression with levothyroxine


is not recommended for benign nodules.

1a and 1b

50-52

9 Is there a role for


medical therapy in
the treatment of
nontoxic nodular
goiter?

Levels of
Evidence

Grade of
References
Recommendation

* This does not necessarily imply Level 1 evidence but rather the recommendation is based on accepted practice and published guidelines regarding
these conditions.

REFERENCES
1. Baskin HJ, Cobin RH, Duick DS, et al. American Association of
Clinical Endocrinologists medical guidelines for clinical practice for the evaluation and treatment of hyperthyroidism and
hypothyroidism. Endocr Pract. 2002;8:457-469.
2. Reschini E, Catania A, Ferrari C, Bergonzi M, Paracchi A, Raineri P. Comparison of pertechnetate and radioiodine thyroid scintiscans in thyroid disease. J Nucl Biol Med. 1993;37:12-17.
3. Torring O, Tallstedt L, Wallin G, et al. Graves hyperthyroidism:
Treatment with antithyroid drugs, surgery, or radioiodinea
prospective, randomized study. Thyroid Study Group. J Clin Endocrinol Metab. 1996;81:2986-2993.
4. In H, Pearce EN, Wong AK, Burgess JF, McAneny DB, Rosen JE.
Treatment options for Graves disease: A cost-effectiveness analysis. J Am Coll Surg. 2009;209:170-179; e171-172.
5. Tallstedt L, Lundell G, Torring O, et al. Occurrence of ophthalmopathy after treatment for Graves hyperthyroidism. The Thyroid Study Group. N Engl J Med. 1992;326:1733-1738.
6. Menconi F, Marino M, Pinchera A, et al. Effects of total
thyroid ablation versus near-total thyroidectomy alone
on mild to moderate Graves orbitopathy treated with intravenous glucocorticoids. J Clin Endocrinol Metab. 2007;92:
1653-1658.

PMPH_CH70.indd 565

7. Porterfield JR, Jr., Thompson GB, Farley DR, Grant CS, Richards
ML. Evidence-based management of toxic multinodular goiter
(Plummers Disease). World J Surg. 2008;32:1278-1284.
8. Vidal-Trecan GM, Stahl JE, Eckman MH. Radioiodine or surgery for toxic thyroid adenoma: Dissecting an important decision. A cost-effectiveness analysis. Thyroid. 2004;14:933-945.
9. Lippi F, Ferrari C, Manetti L, et al. Treatment of solitary autonomous thyroid nodules by percutaneous ethanol injection: Results
of an Italian multicenter study. The Multicenter Study Group.
J Clin Endocrinol Metab. 1996;81:3261-3264.
10. Zingrillo M, Modoni S, Conte M, Frusciante V, Trischitta V. Percutaneous ethanol injection plus radioiodine versus radioiodine
alone in the treatment of large toxic thyroid nodules. J Nucl Med.
2003;44:207-210.
11. Palit TK, Miller CC, 3rd, Miltenburg DM. The efficacy of thyroidectomy for Graves disease: A meta-analysis. J Surg Res.
2000;90:161-165.
12. Witte J, Goretzki PE, Dotzenrath C, et al. Surgery for Graves
disease: Total versus subtotal thyroidectomy-results of a prospective randomized trial. World J Surg. 2000;24:1303-1311.
13. Barakate MS, Agarwal G, Reeve TS, Barraclough B, Robinson B,
Delbridge LW. Total thyroidectomy is now the preferred option
for the surgical management of Graves disease. ANZ J Surg.
2002;72:321-324.

5/22/2012 5:48:27 PM

566

Surgery: Evidence-Based Practice

14. Lal G, Ituarte P, Kebebew E, Siperstein A, Duh QY, Clark OH.


Should total thyroidectomy become the preferred procedure
for surgical management of Graves disease? Thyroid. 2005;15:
569-574.
15. Wilhelm SM, McHenry CR. Total thyroidectomy is superior to
subtotal thyroidectomy for management of Graves disease in the
United States. World J Surg. 2010;34:1261-1264.
16. Andaker L, Johansson K, Smeds S, Lennquist S. Surgery for hyperthyroidism: hemithyroidectomy plus contralateral resection or
bilateral resection? A prospective randomized study of postoperative complications and long-term results. World J Surg. 1992;
16:765-769.
17. Muller PE, Bein B, Robens E, Bein HS, Spelsberg F. Thyroid surgery according to Enderlen-Hotz or Dunhill: A comparison of two
surgical methods for the treatment of Graves disease. Int Surg.
2001;86:112-116.
18. Jarhult J, Rudberg C, Larsson E, et al. Graves disease with moderate-severe endocrine ophthalmopathy-long term results of a
prospective, randomized study of total or subtotal thyroid resection. Thyroid. 2005;15:1157-1164.
19. Erbil Y, Ozluk Y, Giris M, et al. Effect of lugol solution on thyroid
gland blood flow and microvessel density in the patients with
Graves disease. J Clin Endocrinol Metab. 2007;92:2182-2189.
20. Kim KH, Sung MW, Koh TY, Oh SH, Kim IS. Pyriform sinus fistula:
Management with chemocauterization of the internal opening.
Ann Otol Rhinol Laryngol. 2000;109:452-456.
21. Miyauchi A, Tomoda C, Uruno T, et al. Computed tomography
scan under a trumpet maneuver to demonstrate piriform sinus
fistulae in patients with acute suppurative thyroiditis. Thyroid.
2005;15:1409-1413.
22. Espinoza PG, Guendelman CL, Quevedo Limon LN, Fernandez RJ. A comparison between two imaging techniques for the
diagnosis of subacute thyroiditis (de Quervain thyroiditis): Brief
communication. Clin Nucl Med. 2010;35:862-864.
23. Yamada H, Fujita K, Tokuriki T, Ishida R. Nine cases of piriform
sinus fistula with acute suppurative thyroiditis. Auris Nasus Larynx. 2002;29:361-365.
24. Sai Prasad TR, Chong CL, Mani A, et al. Acute suppurative thyroiditis in children secondary to pyriform sinus fistula. Pediatr
Surg Int. 2007;23:779-783.
25. Casey BM, Leveno KJ. Thyroid disease in pregnancy. Obstet
Gynecol. 2006;108:1283-1292.
26. Duininck TM, van Heerden JA, Fatourechi V, et al. de Quervains
thyroiditis: surgical experience. Endocr Pract. 2002;8:255-258.
27. Rodondi N, den Elzen WP, Bauer DC, et al. Subclinical hypothyroidism and the risk of coronary heart disease and mortality.
JAMA. 2010;304:1365-1374.
28. Villar HC, Saconato H, Valente O, Atallah AN. Thyroid hormone
replacement for subclinical hypothyroidism. Cochrane Database
of Systematic Reviews. 2007:CD003419.
29. Shih ML, Lee JA, Hsieh CB, et al. Thyroidectomy for Hashimotos thyroiditis: Complications and associated cancers. Thyroid.
2008;18:729-734.
30. Boelaert K, Horacek J, Holder RL, Watkinson JC, Sheppard MC,
Franklyn JA. Serum thyrotropin concentration as a novel predictor of malignancy in thyroid nodules investigated by fi ne-needle
aspiration. J Clin Endocrinol Metab. 2006;91:4295-4301.
31. Elisei R, Bottici V, Luchetti F, et al. Impact of routine measurement of serum calcitonin on the diagnosis and outcome of medullary thyroid cancer: Experience in 10,864 patients with nodular
thyroid disorders. J Clin Endocrinol Metab. 2004;89:163-168.

PMPH_CH70.indd 566

32. Costante G, Meringolo D, Durante C, et al. Predictive value of


serum calcitonin levels for preoperative diagnosis of medullary
thyroid carcinoma in a cohort of 5817 consecutive patients with
thyroid nodules. J Clin Endocrinol Metab. 2007;92:450-455.
33. Hahm JR, Lee MS, Min YK, et al. Routine measurement of serum
calcitonin is useful for early detection of medullary thyroid
carcinoma in patients with nodular thyroid diseases. Thyroid.
2001;11:73-80.
34. Cheung K, Roman SA, Wang TS, Walker HD, Sosa JA. Calcitonin measurement in the evaluation of thyroid nodules in the
United States: A cost-effectiveness and decision analysis. J Clin
Endocrinol Metab. 2008;93:2173-2180.
35. Moon WJ, Jung SL, Lee JH, et al. Benign and malignant thyroid
nodules: US differentiationmulticenter retrospective study.
Radiology. 2008;247:762-770.
36. Papini E, Guglielmi R, Bianchini A, et al. Risk of malignancy
in nonpalpable thyroid nodules: Predictive value of ultrasound
and color-Doppler features. J Clin Endocrinol Metab. 2002;87:
1941-1946.
37. Cappelli C, Castellano M, Pirola I, et al. The predictive value of
ultrasound findings in the management of thyroid nodules. Q J
Med. 2007;100:29-35.
38. Marqusee E, Benson CB, Frates MC, et al. Usefulness of ultrasonography in the management of nodular thyroid disease. Ann
Intern Med. 2000;133:696-700.
39. Cooper DS, Doherty GM, Haugen BR, et al. Revised American
Thyroid Association management guidelines for patients with
thyroid nodules and differentiated thyroid cancer. Thyroid. 2009;
19:1167-1214.
40. Danese D, Sciacchitano S, Farsetti A, Andreoli M, Pontecorvi A.
Diagnostic accuracy of conventional versus sonography-guided
fine-needle aspiration biopsy of thyroid nodules. Thyroid. 1998;8:
15-21.
41. Carmeci C, Jeff rey RB, McDougall IR, Nowels KW, Weigel RJ.
Ultrasound-guided fine-needle aspiration biopsy of thyroid
masses. Thyroid. 1998;8:283-289.
42. Bojunga J, Herrmann E, Meyer G, Weber S, Zeuzem S, FriedrichRust M. Real-time elastography for the differentiation of benign
and malignant thyroid nodules: A meta-analysis. Thyroid;20:
1145-1150.
43. Bartolazzi A, Orlandi F, Saggiorato E, et al. Galectin-3-expression
analysis in the surgical selection of follicular thyroid nodules with
indeterminate fine-needle aspiration cytology: A prospective
multicentre study. Lancet Oncol. 2008;9:543-549.
44. Nikiforov YE, Steward DL, Robinson-Smith TM, et al. Molecular testing for mutations in improving the fine-needle aspiration
diagnosis of thyroid nodules. J Clin Endocrinol Metab. 2009;94:
2092-2098.
45. Ylagan LR, Farkas T, Dehner LP. Fine needle aspiration of the
thyroid: A cytohistologic correlation and study of discrepant
cases. Thyroid. 2004;14:35-41.
46. Oertel YC, Miyahara-Felipe L, Mendoza MG, Yu K. Value of
repeated fine needle aspirations of the thyroid: An analysis of over
ten thousand FNAs. Thyroid. 2007;17:1061-1066.
47. Yeh MW, Demircan O, Ituarte P, Clark OH. False-negative fineneedle aspiration cytology results delay treatment and adversely
affect outcome in patients with thyroid carcinoma. Thyroid.
2004;14:207-215.
48. Baczyk M, Pisarek M, Czepczynski R, et al. Therapy of large multinodular goitre using repeated doses of radioiodine. Nucl Med
Commun. 2009;30:226-231.

5/22/2012 5:48:27 PM

Hyperthyroidism, Thyroiditis, and Nontoxic Goiter

49. Barczynski M, Konturek A, Hubalewska-Dydejczyk A, Golkowski


F, Cichon S, Nowak W. Five-year follow-up of a randomized clinical trial of total thyroidectomy versus Dunhill operation versus
bilateral subtotal thyroidectomy for multinodular nontoxic goiter. World J Surg. 2010;34:1203-1213.
50. Zelmanovitz F, Genro S, Gross JL. Suppressive therapy with
levothyroxine for solitary thyroid nodules: A double-blind controlled clinical study and cumulative meta-analyses. J Clin Endocrinol Metab. 1998;83:3881-3885.

PMPH_CH70.indd 567

567

51. Wemeau JL, Caron P, Schvartz C, et al. Effects of thyroidstimulating hormone suppression with levothyroxine in reducing
the volume of solitary thyroid nodules and improving extranodular nonpalpable changes: A randomized, double-blind, placebocontrolled trial by the French Thyroid Research Group. J Clin
Endocrinol Metab. 2002;87:4928-4934.
52. Castro MR, Caraballo PJ, Morris JC. Effectiveness of thyroid
hormone suppressive therapy in benign solitary thyroid nodules:
A meta-analysis. J Clin Endocrinol Metab. 2002;87:4154-4159.

5/22/2012 5:48:27 PM

CHAPTER 71

Hyperparathyroidism
Jason D. Prescott and Robert Udelsman

INTRODUCTION

Several familial syndromes predisposing individuals to parathyroid hyperplasia have been described, the most common of
which are multiple endocrine neoplasias type 1 and type 2A. In
addition to hyperparathyroidism, these genetic syndromes predispose affected kindreds to additional syndromic manifestations.
Careful assessment of family history is thus crucial in the workup
of suspected hyperparathyroidism and genetic testing should precede treatment in suspicious cases.
Familial hypocalciuric hypercalcemia (FHH) is a mild hypercalcemic condition. Affected patients present with mild hypercalcemia, hypocalciuria, and normal or slightly elevated serum
iPTH levels.6 This condition is rarely symptomatic and does not
generally require treatment. FHH can be distinguished from conventional 1 HPTH by careful assessment of family history and by
the measurement of 24-h urinary calcium level, which is almost
always less than 30 mg in FHH.
Hypocalcemia is a cardinal feature of CRF. It is largely due
to impaired vitamin D production and virtually all patients with
a glomerular fi ltration rate of less than 60 mL/min/1.73 m2 will
develop some degree of four-gland parathyroid hyperplasia.7 The
severity of 2 HPT is linearly related to the degree of renal dysfunction and thus the frequency of hyperparathyroidism screening in CRF patients depends on the degree of renal disease. Stage 3
chronic renal disease patients (GFR 3059 mL/min/1.73 m2) should
undergo annual biochemical testing for 2 HPTH whereas more
advanced renal dysfunction merits screening every 3 months.8
3 HPT is diagnosed when hypercalcemia and hyperparathyroidism develop after definitive treatment of 2 HPT. It can also occur
following renal transplantation.
Answer: Hyperparathyroidism is diagnosed when serum
iPTH levels are persistently elevated or inadequately suppressed in
the context of persistent hypercalcemia. (Grade A recommendation). The subtype of hyperparathyroidism, for example, primary,
secondary, or tertiary, is determined on the basis of medical history and biochemical testing. A careful family history is required
in all hyperparathyroidism patients and suspicious findings mandate genetic evaluation. A 24-h urinary calcium screen can help

Hyperparathyroidism is the result of excessive serum parathyroid


hormone (PTH) production and the resultant hypercalcemia.
The multiorgan pathologic effects of hyperparathyroidism were
first recognized in the 1950s and include neuropsychiatric and
neuromuscular disturbances, osteoporosis, peptic ulcer disease,
pancreatitis, nephrolithiasis, nephrocalcinosis, and cardiovascular disease.1-4 Three major causes of hyperparathyroidism have
been identified. Primary hyperparathyroidism (1 HPT) is due
to an intrinsic abnormality of one or more parathyroid glands. It
is caused by (1) adenomas, (2) four-gland parathyroid hyperplasia, or (3) parathyroid carcinoma. Single adenomas account for
approximately 85% of cases, whereas hyperplasia is responsible
for 15%. Parathyroid carcinoma is extremely rare.
Secondary hyperparathyroidism (2 HPT) results from fourgland hyperplasia. Unlike 1 HPT, 2 HPT occurs in response
to continuous stimulation of normal parathyroid cells by chronic
hypocalcemia, most commonly in the context of chronic renal failure
(CRF). Tertiary hyperparathyroidism (3 HPT) is a rare condition
in which chronically overstimulated parathyroid tissue associated
with 2 HPT becomes autonomous, continuing to overproduce
PTH despite correction of underlying hypocalcemia . Patients with
3 HPT almost always have four-gland enlargement although one of
these glands is almost always asymmetrically enlarged.5
1. How is hyperparathyroidism diagnosed?
Calcium-mediated negative feedback inhibition normally maintains a tight inverse relationship between serum PTH and calcium
levels. Disruption of this relationship produces hyperparathyroidism. The diagnosis of hyperparathyroidism is thus biochemical, hinging on the presence of inappropriately elevated intact
parathyroid hormone (iPTH) levels (normal 1065 pg/mL) in the
context of persistent hypercalcemia (normal 8.510.5 mg/dL). Signs
and symptoms of hyperparathyroidism help to confirm the diagnosis, but are not required, as some patients are asymptomatic.

568

PMPH_CH71.indd 568

5/22/2012 5:49:54 PM

Hyperparathyroidism

rule out FHH, which may mimic the biochemical features of conventional 1 HPTH. (Grade B recommendation)
2. What are the optimal treatments for hyperparathyroidism?
Surgical resection of hyperfunctioning parathyroid tissue is the
only means of curing 1 HPT and cure rates greater than 95%
are common when parathyroid surgery is performed by an experienced endocrine surgeon.9,10 Thus, virtually all symptomatic
patients should undergo either resection of causative parathyroid
adenoma(s) or, if four-gland disease is present, subtotal parathyroidectomy. En bloc parathyroidectomy is indicated in rare cases of
parathyroid carcinoma. In contrast, the benefits of surgery remain
unclear in patients having biochemically proven 1 HPT but for
whom symptoms and additional signs of hyperparathyroidism
are absent. Expert consensus regarding such cases was recently
refined at the third international workshop on asymptomatic primary hyperparathyroidism. This workshop identified a high probability of benefit from surgery in all asymptomatic 1 HPT patients
less than 50 years old, those unable or unwilling to comply with
biannual biochemical surveillance, in those having a total serum
calcium greater than 1.0 mg/dL above the upper limit of normal, a
creatinine clearance of less than 60 mL/min, a bone mineral density
T score is less than 2.5 at any site, or for whom a history of fragility
fracture was present.11 Asymptomatic 1 HPT patients meeting one
or more of these criteria should be referred for surgery.
Recent data indicate that the majority of supposedly asymptomatic 1 HPT patients actually experience symptoms, most
com monly resorptive bone disease and subtle neurocognitive
disturbances, and that these symptoms often improve following
parathyroid surgery.12-15 Further, progression to symptomatic disease is common and approximately 27% of asymptomatic patients
eventually undergo parathyroid surgery.16 All asymptomatic, nonoperative cases of 1 HPT therefore merit biannual re-evaluation
for both disease progression and the need for surgery. Bone densitometry should be performed annually in these cases.
Medical therapy for 1 HPT offers inferior disease control relative to surgical resection and is only indicated in those few patients
who cannot tolerate surgery.17 Bisphosphonates and hormone
replacement therapy can decrease bone turnover and improve
bone mineral density in 1 HPT patients, although these findings
have not been correlated with decreased fracture rates.18,19 Calcimimetics can decrease serum calcium and iPTH levels in 1 HPT,
although these medications do not improve bone mineral density
or bone turnover.20 Studies examining the effects of these medications on the other pathologic features of hyperparathyroidism
have not been reported.
Management of 2 HPT focuses on re-establishing normocalcemia. This involves the correction of the underlying disease when
possible, or medical therapy to normalize serum calcium levels
when it is not. Oral activated vitamin D analogs and calcimimetics provide good control of serum calcium and iPTH levels in the
majority of CRF patients, although the optimal therapy for such
patients remains renal transplantation.21,22 Surgery for 2 HPT is
rarely indicated, being reserved for symptomatic cases refractory
to medical therapy and for patients developing calciphylaxis. The
optimal operative intervention for those 2 HPT patients who
do require surgery remains controversial. Subtotal (3.5 gland)
parathyroidectomy and total parathyroidectomy with autotransplantation are associated with recurrence rates of up to 33%.

PMPH_CH71.indd 569

569

Although some groups have employed total parathyroidectomy


without autotransplantation, this technique has not gained wide
acceptance.23,24 Long-term prospective outcome data comparing
these three techniques have not been reported and the choice of
surgical procedure continues to depend on surgeon preference.
Calciphylaxis is a rare, life-threatening complication of hyperparathyroidism characterized by progressive calcification of cutaneous tissues. This process manifests as painful purpuric skin
lesions that respond poorly to local wound care, become chronically infected, and lead to sepsis and death in up to 92% of cases.25
Although data supporting parathyroid surgery for calciphylaxis
are mixed, the absence of effective alternative therapies mandates
aggressive surgical intervention.
Surgery for asymptomatic 2 HPT patients demonstrating
persistent biochemical evidence of hyperparathyroidism despite
medical therapy remains controversial. Although several reports
describe biochemical indications for surgery in 2 HPT patients,
no long-term data evaluating outcomes, with and without surgery,
have been reported.26 It is thus reasonable to apply the same criteria
established for the surgical management of asymptomatic 1 HPT
when assessing asymptomatic 2 HPT patients for surgery.11
The management of 3 HPT is similar to that of 1 HPT, as
resection of parathyroid adenoma(s) or subtotal/total parathyroidectomy, when four-gland disease is present, offers the only opportunities for cure.27,28 Indications for surgery among asymptomatic
3 HPT patients are controversial for the same reasons associated
with the surgical management of asymptomatic 1 HPT. Thus,
it is reasonable to apply the same recommendations guiding the
surgical management and surveillance of asymptomatic 1 HPT
patients to asymptomatic 3 HPT patients.11
Answer: Symptomatic 1 HPT and 3 HPT patients should be
referred for surgery (Grade A recommendation), whereas 2 HPT
patients should undergo parathyroid resection if medical management fails (Grade A recommendation) or if calciphylaxis develops. (Grade B recommendation) Medical therapy for 1 HPT and
3 HPT should be limited to patients unable to tolerate parathyroid
surgery. Asymptomatic 1 HPT patients should be evaluated for
surgery according to the guidelines published by the third international workshop on asymptomatic primary hyperparathyroidism.11
(Grade A recommendation) These same guidelines may be extrapolated to asymptomatic 3 HPT or medically refractory asymptomatic 2 HPT patients, because no evidence-based recommendations
specifying surgical management in such cases have been reported.
3. What is the preoperative management of hyperparathyroidism?
The depth of the surgeons operating experience is the most important determinant in maximizing the probability of cure and minimizing morbidity in patients requiring endocrine surgery.29-31
Although data comparing surgical outcomes to case volume following parathyroid surgery have not yet been reported, the clear
association between experience and surgical outcome following
thyroid surgery merits referral of these patients to an experienced
endocrine surgeon.
Optimal surgical outcomes require careful preoperative evaluation of all hyperparathyroidism patients. In addition to a
thorough history and physical examination, all cases should be
biochemically verified. Previous medical management should be
reviewed with the referring endocrinologist for all 2 HPT patients

5/22/2012 5:49:54 PM

570

Surgery: Evidence-Based Practice

and cases in which medical treatment options remain should not


undergo surgery. Surgical indications should be reviewed for
asymptomatic patients and surgery should be deferred in favor of
careful surveillance when these indications are absent.11 Preoperative laryngoscopy can help stratify surgical risk by alerting the
surgeon to preexisting recurrent laryngeal nerve (RLN) dysfunction. Laryngoscopy should be considered for all uncomplicated
hyperparathyroidism patients and is mandatory when the risk of
operative RLN injury is high, as in reoperative cases. Preoperative
localization imaging must be obtained when minimally invasive
parathyroidectomy (MIP) is planned and when reoperative surgery, which involves a distorted and scarred reoperative field, is
necessary.
Answer: All hyperparathyroidism patients requiring surgery
should be referred to an experienced endocrine surgeon. (Grade B
recommendation) Preoperative evaluation should include a complete history and physical examination as well as biochemical
confirmation of hyperparathyroidism. Failure of medical management should be confirmed preoperatively in cases of 2 HPT.
Preoperative laryngoscopy should be considered for all patients
and is mandatory in reoperative cases. (Grade D recommendation) Preoperative localization imaging is required when MIP or
reoperative surgery is planned.
4. Which imaging studies should be used preoperatively for
localization of abnormal parathyroid tissue?
The purpose of preoperative imaging in hyperparathyroidism is
to localize hyperfunctioning parathyroid tissue to create a road
map for the surgeon. Such a preoperative knowledge allows the
surgeon to restrict the extent of neck exploration to area(s) harboring abnormal parathyroid tissue, which minimizes operative
morbidity. Therefore, cases in which the scope of neck exploration
cannot be focused (e.g., four-gland disease) do not generally benefit from preoperative imaging. Imaging studies should never be
employed for diagnostic purposes, as available imaging modalities lack the sensitivity and specificity of biochemical tests used to
detect hyperparathyroidism.
Initial preoperative imaging for hyperparathyroidism should
be noninvasive and inexpensive. Invasive and/or costly techniques should be reserved for reoperative cases in which initial
imaging is nonlocalizing. Cervical ultrasound (US) and sestamibi
scintigraphy (MIBI) are relatively inexpensive, readily available,
and safe modalities that are suitable first-line imaging choices.
Computed tomography (CT) scanning, 4-dimensional CT (4D
CT) scanning, and magnetic resonance imaging (MRI) are more
expensive studies and CT scanning necessitates exposure to ionizing radiation. Positron emission tomography (PET) and combined PET-CT scanning are additional imaging options, although
both studies are more expensive than either MRI or CT alone.
Selective arteriography with venous blood iPTH sampling (SVS)
is expensive and invasive. Arteriography with SVS should thus
be reserved for reoperative cases in which adequate preoperative
localization cannot be obtained using less invasive and less expensive imaging techniques. Individual imaging studies often fail to
adequately localize causative hyperfunctioning parathyroid tissue
and sequential imaging, employing multiple different modalities,
should be employed in such cases.
The quality of preoperative imaging is highly variable. Variables include the quality of the imaging study, the patients body
habitus as well as his or her ability to tolerate the study, the presence

PMPH_CH71.indd 570

of implants or prostheses, and the quality of the interpreting radiologist. Consequently, reported positive-predictive values for each
preoperative imaging modality are extremely variable and difficult to compare.32 Given such variability, the potential for accurate preoperative imaging should be optimized by performing
these studies at experienced centers.
Answer: Multiple preoperative imaging studies can accurately localize causative hyperfunctional parathyroid tissue, thus
allowing the surgeon to limit the extent of operative exploration.
Nonetheless, preoperative imaging accuracy is highly variable
among institutions and no gold standard modality has been identified. The choice of preoperative imaging study is therefore based
on the morbidity and cost associated with each technique. (Grade
A recommendation) US and MIBI are inexpensive, benign, and
often accurate. Either test is appropriate for the initial preoperative evaluation of hyperparathyroidism. CT, MRI, and PET scans
are progressively more expensive studies, with CT and PET scans
necessitating ionizing radiation exposure. These imaging modalities can be considered if US and MIBI fail to localize aberrant
parathyroid tissue. Alternatively, an experienced parathyroid
surgeon can explore nonlocalized patients with a high degree of
operative success. Arteriography with SVS is expensive and should
be reserved for reoperative cases in which less morbid studies fail
to localize causative parathyroid lesions. (Grade A recommendation) US-guided fine needle aspiration for PTH should also be
considered when feasible in the reoperative patient.
5. What are the surgical approaches for treating hyperparathyroidism and what intraoperative adjuncts best facilitate parathyroid surgery?
The choice of surgical approach for hyperparathyroidism depends
on the location(s) of causative hyperfunctioning parathyroid tissue. Until recently, identification of aberrant parathyroid glands
required four-gland exploration under general anesthesia. This
approach is still preferred in known cases of four-gland disease and in patients for whom preoperative localization studies
are inadequate. With the advent of intraoperative serum iPTH
(IoPTH) monitoring, refinements in cervical nerve block technique, and the development of accurate preoperative imaging
modalities, MIP became possible. This technique involves focused
neck exploration via a small cervical incision under regional anesthesia, thus minimizing operative morbidity.10
Hyperfunctional parathyroid tissue is usually cervical and can
be readily excised via an abbreviated Kocher incision. Excellent
exposure can be obtained by either a standard medial approach or
by using a lateral approach. The latter approach involves mobilization lateral to the strap muscles and allows the surgeon to avoid
previously explored operative fields in reoperative cases. Because
most hyperfunctioning parathyroid tissue is found at one or more
normal (eutopic) anatomic parathyroid gland positions, the medial
approach generally affords the most direct means of resection. The
selection of operative approach is influenced by multiple factors,
including surgeon preference, preoperative localization, and history of previous neck surgery. Intraoperative circumstances may
merit the use of both techniques.
Cases in which preoperative localization studies identify
extracervical hyperfunctional parathyroid tissue may necessitate
alternative surgical approaches. Intrathymic or retroesophageal
disease can often be resected via a cervical approach, although
rare patients will require exposure via partial or complete median

5/22/2012 5:49:54 PM

Hyperparathyroidism

sternotomy and/or by transthoracic techniques, 33,34 including


thorascopic approaches.
Parathyroid adenomas should be excised carefully to avoid
capsule rupture and seeding of the operative field with abnormal
or potentially malignant parathyroid cells. Parathyromatosis, an
iatrogenic form of recurrent hyperparathyroidism, in which parathyroid nodules develop in the previous surgical field, can develop
after accidental seeding. Great care must be taken to preserve adequate postoperative parathyroid function. Inferior thyroid artery
branches supplying residual parathyroid tissues must be carefully
protected during surgery. Suspected intraoperative devitalization
of normal parathyroid glands should prompt autotransplantation
of parathyroid tissue.
Intraoperative adjuvant techniques can facilitate identification
of abnormal parathyroid tissue and may help minimize the risk of
injury to the recurrent and superior laryngeal nerves. Intraoperative US has been used successfully to guide parathyroid resection,
although this technique has generally given way to accurate preoperative imaging. Intraoperative Geiger counter (gamma probe)
detection of radiolabeled sestamibi, which localizes preferentially
to hyperfunctioning parathyroid tissue, can also be used to guide
cervical exploration. Background signal from normal tissues,
however, can limit the intraoperative sensitivity of this technique
and no data demonstrating additive value, relative to preoperative
MIBI scanning alone, have been reported.
Intraoperative monitoring of serum iPTH levels allows verification of disease extirpation by demonstrating a decline in serum
iPTH levels after resection of hyperfunctioning parathyroid tissue. For 1 HPT, IoPTH monitoring is rapid and a serum iPTH
decrease of at least 50% after resection is associated with cure in
up to 99.4% of cases.10 The value of IoPTH monitoring for 2 HPT
and 3 HPT is less clear, because PTH clearance is impaired by
renal dysfunction. Further, the specificity of some commercially
available iPTH assays is insufficient to identify intraoperative cure
in some renal failure patients.35 Finally, the percent decline in preexcisional IoPTH level correlating with cure is unclear in the context of renal failure, with some studies demonstrating a need for
at least a 90% drop in IoPTH level.36,37
Used in combination, preoperative imaging studies, intraoperative adjuncts, and meticulous operative technique allow for
successful identification and resection of hyperfunctioning parathyroid tissue in almost all cases. Nonetheless, rare cases occur in
which these techniques fail to achieve a cure. In such instances,
neck exploration may reveal three eutopically localized normal
parathyroid glands, with the fourth gland remaining undiscovered. The surgeon must be cognizant of and capable of exploring both eutopic and ectopic parathyroid locations; these include
retroesophageal, retrophargeal, intrathymic, intrathyroidal, and
carotid sheath locations.
Techniques for minimizing the risk of intraoperative nerve
injury during surgery include real-time intraoperative electromyographic nerve monitoring, direct visualization of one or both
RLNs, and direct assessment of nerve function, as measured by
intraoperative voice quality testing in conscious patients receiving regional anesthesia. A recent prospective study assessing the
protective potential of intraoperative electromyographic nerve
monitoring demonstrated no improvement in nerve injury rates
when this technique was employed for remedial 1 HPT.38 Further, recent guidelines for the standardization and application of
this technique suggest a maximum positive- predictive value of

PMPH_CH71.indd 571

571

only 76%.39 No data assessing the impact of direct intraoperative


voice quality monitoring on RLN injury risk have been reported.
It is clear, however, that surgical experience is paramount to safe
surgery.
Answer: Surgeon preference, preoperative disease localization, and history of previous neck surgery influence the selection
of operative approach and both medial and lateral techniques may
be employed. All patients should be prepped and draped to facilitate either operative approach and, when regional anesthesia is
employed, efficient conversion to general anesthesia may be necessary. Resection of extracervical parathyroid tissue may rarely
require partial or complete median sternotomy or transthoracic
exposure. The rapid IoPTH assay appears to be the most useful
intraoperative adjunct facilitating a curative operation and its use
should be considered in all operative 1 HPT patients. (Grade A
recommendation) The value of IoPTH monitoring during parathyroid resection in 2 HPT and 3 HPT patients is less clear,
because renal dysfunction alters the kinetics of PTH clearance
and impairs the specificity of some IoPTH assays. In this context,
the use of IoPTH monitoring should be used with caution. Intraoperative neuromonitoring of recurrent nerve function has not
been demonstrated to protect nerves at risk.
6. How are the common complications of parathyroid surgery
managed?
Complications following parathyroid surgery most commonly
result from injury to the RLN and/or parathyroid devitalization.
The RLNs provide innervation of all laryngeal muscles with the
exception of the cricothyroideus. A unilateral RLN will most
likely compromise the voice quality, although some patients will
be asymptomatic. Bilateral RLN injury results in bilateral vocal
cord medialization, which will severely compromise the airway
and can result in airway obstruction. Unilateral postoperative
RLN dysfunction generally manifests as hoarseness and is usually temporary, resulting from minor traction or crush injuries.
Permanent RLN injury is rare when parathyroid surgery is performed by an experienced endocrine surgeon, occurring in less
than 1% of cases.10,40
Transient hypocalcemia resulting from temporary postoperative parathyroid dysfunction can occur following either subtotal
parathyroidectomy or total parathyroidectomy with autotransplantation. Normalization of serum calcium level should be verified in the immediate postoperative period using serial serum
calcium checks and a short course of prophylactic oral calcium
carbonate, with or without activated vitamin D supplementation.41 Transient hypocalcemia may also develop when extensive exploration of all four glands is required and serum calcium
levels should also be monitored in these patients. Hungry bone
syndrome, which involves rapid absorption of serum calcium by
the chronically demineralized bone after normalization of serum
PTH, is associated with curative parathyroid resection in renal
failure patients and can produce profound hypocalcemia. For
this reason, postoperative 2 HPT and 3 HPT patients must be
carefully monitored for hypocalcemia and must be aggressively
treated when hypocalcemia is discovered. Inadvertent parathyroid devitalization resulting in permanent hypocalcemia is rare,
occurring in less than 1% of cases performed by an experienced
surgeon.10,28,40 All cases of permanent postoperative hypocalcemia
require life-long calcium supplementation, with or without vitamin D administration.

5/22/2012 5:49:55 PM

572

Surgery: Evidence-Based Practice

Answer: Hypocalcemia following subtotal parathyroidectomy, or following cases involving exploration of all four parathyroid glands, is usually transient. Normalization of serum calcium
levels should be verified. (Grade C recommendation) Patients
demonstrating persistent postoperative hypocalcemia may require

life-long calcium and vitamin D supplementation. Postoperative 2 HPT and 3 HPT patients must be carefully monitored for
hungry bone syndrome and hypocalcemia must be aggressively
treated in these patients. The RLN(s) must be identified and carefully preserved during surgical exploration.

Clinical Question Summary


Question

Answer

1 How is hyperparathyroidism
diagnosed?

Primary hyperparathyroidism is diagnosed when the serum


PTH level is inappropriately elevated relative to the serum
calcium level in the setting of normal renal function.
Secondary hyperparathyroidism most commonly occurs in
renal failure patients who have elevated serum PTH levels.

7, 8, 17

2 What are the


causes of primary
hyperparathyroidism?

Parathyroid adenoma (85%)


Parathyroid hyperplasia (15%)
Parathyroid carcinoma (<1%)

11, 16

3 What is the optimal


treatment for symptomatic
primary hyperthyroidism?

Surgery

10, 16

4 What is a minimally invasive


parathyroidectomy (MIP)?

It is a focused operation based on preoperative localization


that utilizes an intraoperative adjunct such as an
intraoperative parathyroid hormone assay to confirm the
adequacy of resection.

10

5 What are the complications


associated with
parathyroidectomy?

The overall complication rate should be less than 3%; within


this context complications include recurrent nerve injury,
symptomatic hypocalcemia, and neck hematoma.

9, 10

REFERENCES
1. Cope O, Culver PJ, Mixter CG, et al. Pancreatitis, a diagnostic
clue to hyperparathyroidism. Ann Surg. 1957;145:857-863.
2. Fitz TE, Hallman BL. Mental changes associated with hyperparathyroidism; report of two cases. AMA Arch Intern Med. 1952;
89:547-551.
3. Kretschmer HL. Parathyroid adenoma and renal calculi. J Urol.
1950;63:947-958.
4. St Goar WT. Gastrointestinal symptoms as a clue to the diagnosis of primary hyperparathyroidism: A review of 45 cases. Ann
Intern Med. 1957;46:102-118.
5. Kilgo MS, Pirsch JD, Warner TF, et al. Tertiary hyperparathyroidism after renal transplantation: Surgical strategy. Surgery. 1998;
124:677-683; discussion 683-674.
6. Law WM, Jr., Heath H, 3rd. Familial benign hypercalcemia
(hypocalciuric hypercalcemia). Clinical and pathogenetic studies in 21 families. Ann Intern Med. 1985;102:511-519.
7. Brossard JH, Lepage R, Cardinal H, et al. Influence of glomerular fi ltration rate on non-(1-84) parathyroid hormone (PTH)
detected by intact PTH assays. Clin Chem. 2000;46:697-703.
8. K/DOQI clinical practice guidelines for bone metabolism and
disease in chronic kidney disease. Am J Kidney Dis. 2003;42:
S1-201.
9. Lo Gerfo P. Bilateral neck exploration for parathyroidectomy
under local anesthesia: A viable technique for patients with
coexisting thyroid disease with or without sestamibi scanning.
Surgery. 1999;126:1011-1014; discussion 1014-1015.

PMPH_CH71.indd 572

Grade of
References
Recommendation

10. Udelsman R, Lin Z, Donovan P. The superiority of minimally


invasive parathyroidectomy based on 1650 consecutive patients with primary hyperparathyroidism. Ann Surg. 2011;253:
585-591.
11. Bilezikian JP, Khan AA, Potts JT, Jr. Guidelines for the management of asymptomatic primary hyperparathyroidism: Summary
statement from the third international workshop. J Clin Endocrinol Metab. 2009;94:335-339.
12. Ambrogini E, Cetani F, Cianferotti L, et al. Surgery or surveillance for mild asymptomatic primary hyperparathyroidism: A
prospective, randomized clinical trial. J Clin Endocrinol Metab.
2007;92:3114-3121.
13. Rao DS, Phillips ER, Divine GW, et al. Randomized controlled
clinical trial of surgery versus no surgery in patients with mild
asymptomatic primary hyperparathyroidism. J Clin Endocrinol
Metab. 2004;89:5415-5422.
14. Pasieka JL, Parsons LL, Demeure MJ, et al. Patient-based surgical
outcome tool demonstrating alleviation of symptoms following
parathyroidectomy in patients with primary hyperparathyroidism. World J Surg. 2002;26:942-949.
15. Roman SA, Sosa JA, Pietrzak RH, et al. The effects of serum
calcium and parathyroid hormone changes on psychological
and cognitive function in patients undergoing parathyroidectomy for primary hyperparathyroidism. Ann Surg. 2011;253:
131-137.
16. Silverberg SJ, Shane E, Jacobs TP, et al. A 10-year prospective
study of primary hyperparathyroidism with or without parathyroid surgery. N Engl J Med. 1999;341:1249-1255.

5/22/2012 5:49:55 PM

Hyperparathyroidism

17. Khan A, Grey A, Shoback D. Medical management of asymptomatic primary hyperparathyroidism: Proceedings of the third
international workshop. J Clin Endocrinol Metab. 2009;94:
373-381.
18. Khan AA, Bilezikian JP, Kung AW, et al. Alendronate in primary
hyperparathyroidism: A double-blind, randomized, placebocontrolled trial. J Clin Endocrinol Metab. 2004;89:3319-3325.
19. Grey AB, Stapleton JP, Evans MC, et al. Effect of hormone replacement therapy on bone mineral density in postmenopausal
women with mild primary hyperparathyroidism. A randomized, controlled trial. Ann Intern Med. 1996;125:360-368.
20. Shoback DM, Bilezikian JP, Turner SA, et al. The calcimimetic
cinacalcet normalizes serum calcium in subjects with primary hyperparathyroidism. J Clin Endocrinol Metab. 2003;88:
5644-5649.
21. Block GA, Martin KJ, de Francisco AL, et al. Cinacalcet for secondary hyperparathyroidism in patients receiving hemodialysis.
N Engl J Med. 2004;350:1516-1525.
22. Jean G, Vanel T, Terrat JC, et al. Prevention of secondary hyperparathyroidism in hemodialysis patients: The key role of native
vitamin D supplementation. Hemodial Int. 2010;14:486-491.
23. Hargrove GM, Pasieka JL, Hanley DA, et al. Short- and long-term
outcome of total parathyroidectomy with immediate autografting versus subtotal parathyroidectomy in patients with end-stage
renal disease. Am J Nephrol. 1999;19:559-564.
24. Chan HW, Chu KH, Fung SK, et al. Prospective study on dialysis patients after total parathyroidectomy without autoimplant.
Nephrology (Carlton). 2010;15:441-447.
25. Lal G, Nowell AG, Liao J, et al. Determinants of survival in patients with calciphylaxis: A multivariate analysis. Surgery. 2009;
146:1028-1034.
26. Dumasius V, Angelos P. Parathyroid surgery in renal failure
patients. Otolaryngol Clin North Am. 2010;43:433-440, x-xi.
27. Pitt SC, Sippel RS, Chen H. Secondary and tertiary hyperparathyroidism, state of the art surgical management. Surg Clin North
Am. 2009;89:1227-1239.
28. Nichol PF, Starling JR, Mack E, et al. Long-term follow-up of
patients with tertiary hyperparathyroidism treated by resection
of a single or double adenoma. Ann Surg. 2002;235:673-678; discussion 678-680.
29. Stavrakis AI, Ituarte PH, Ko CY, et al. Surgeon volume as a predictor of outcomes in inpatient and outpatient endocrine surgery. Surgery. 2007;142:887-899; discussion 887-899.

PMPH_CH71.indd 573

573

30. Gourin CG, Tufano RP, Forastiere AA, et al. Volume-based


trends in thyroid surgery. Arch Otolaryngol Head Neck Surg. 2010;
136:1191-1198.
31. Sosa JA, Bowman HM, Tielsch JM, et al. The importance of surgeon experience for clinical and economic outcomes from thyroidectomy. Ann Surg. 1998;228:320-330.
32. Prescott JD, Udelsman R. Remedial operation for primary hyperparathyroidism. World J Surg. 2009;33:2324-2334.
33. Chae AW, Perricone A, Brumund KT, et al. Outpatient videoassisted thoracoscopic surgery (VATS) for ectopic mediastinal
parathyroid adenoma: A case report and review of the literature.
J Laparoendosc Adv Surg Tech A. 2008;18:383-390.
34. Gold JS, Donovan PI, Udelsman R. Partial median sternotomy:
An attractive approach to mediastinal parathyroid disease. World
J Surg. 2006;30:1234-1239.
35. Bieglmayer C, Kaczirek K, Prager G, et al. Parathyroid hormone monitoring during total parathyroidectomy for renal
hyperparathyroidism: Pilot study of the impact of renal function and assay specificity. Clin Chem. 2006;52:1112-1119.
36. de Vos tot Nederveen Cappel R, Bouvy N, de Herder W, et al.
Novel criteria for parathyroid hormone levels in parathyroid
hormone-guided parathyroid surgery. Arch Pathol Lab Med.
2007;131:1800-1804.
37. Kara M, Tellioglu G, Bugan U, et al. Evaluation of intraoperative parathormone measurement for predicting successful surgery in patients undergoing subtotal/total parathyroidectomy
due to secondary hyperparathyroidism. Laryngoscope. 2010;120:
1538-1544.
38. Yarbrough DE, Thompson GB, Kasperbauer JL, et al. Intraoperative electromyographic monitoring of the recurrent laryngeal
nerve in reoperative thyroid and parathyroid surgery. Surgery.
2004;136:1107-1115.
39. Randolph GW, Dralle H, Abdullah H, et al. Electrophysiologic
recurrent laryngeal nerve monitoring during thyroid and parathyroid surgery: International standards guideline statement.
Laryngoscope. 2011;121(Suppl 1):S1-16.
40. Kebebew E, Duh QY, Clark OH. Tertiary hyperparathyroidism:
Histologic patterns of disease and results of parathyroidectomy.
Arch Surg. 2004;139:974-977.
41. Wang TS, Cheung K, Roman SA, et al. To supplement or not to
supplement: A cost-utility analysis of calcium and vitamin D
repletion in patients after thyroidectomy. Ann Surg Oncol. 2010.
[Epub ahead of print.]

5/22/2012 5:49:55 PM

PMPH_CH71.indd 574

5/22/2012 5:49:55 PM

PART XII

THE BREAST

PMPH_CH72.indd 575

5/22/2012 5:50:36 PM

PMPH_CH72.indd 576

5/22/2012 5:50:36 PM

CHAPTER 72

Screening, Breast Biopsy,


Benign Disease
Adora Fou-Cockburn and Mark I. Cockburn

INTRODUCTION

Clinical discussion with women about screening must include


accurate risk assessment. There are two broad categories: normal risk and increased risk. Women at increased risk have different screening regimens that are beyond the investigation of
this discussion. High-risk women include (1) women who have
received thoracic or mantle radiation,9 (2) women aged 35 or
older who have a 5-year risk of invasive cancer >1.7% based on
risk calculation models, such as the Gail model,10 (3) women
with a >2025% lifetime risk of breast cancer due to family
history, (4) women who are BRCA 1 or 2 carriers, (5) women
with a previous biopsy of atypical ductal hyperplasia (ADH) or
lobular carcinoma in situ (LCIS), and (6) women with a previous
breast cancer.
Routine risk factors for asymptomatic women include age
>50, female gender, ethnicity, family history of premenopausal
breast cancer in a first degree relative, early menarche <12 years,
late menopause >55 years, late age at first full-term pregnancy
(>30 years), nulliparity, breast density, history of breast biopsies
(with ADH, LCIS), no breast feeding, obesity, exogenous hormone
use, and alcohol use.11-13 These risk factors may increase a womans
risk up to a RR of 4.0 and must be discussed with her at the time
of screening.

Breast cancer is the second leading cause of cancer death after


lung cancer among women in North America. A womans lifetime risk of developing breast cancer is 1 in 8.2, with the incidence
increasing with age. One in 30 will die from their disease. Death
rates finally began to decrease in the 1990s, with larger decreases
in women younger than 50 (3.2% per year) than in women over
50 (2.0% per year).1,2 These death rates are indicative of progress
in screening for earlier detection, treatments, and most recently,
decreased overall incidence.

SCREENING FOR BREAST CANCER


Breast cancer screening is performed on asymptomatic patients
with the goal of early detection. Overall and disease-free survival
is improved in treating an early stage cancer compared with treating a late stage cancer. Breast cancer screening includes breast
self-examination (BSE), clinical breast examination (CBE) by a
physician, risk assessment, and screening mammography. 3 In
well-selected cases, screening may also include breast ultrasound
or breast MRI. Patient with an existing clinical problem, such as a
new breast mass, needs a diagnostic evaluation and is no longer a
screening candidate.
When investigating population-based screening practices,
the magnitude of benefit should outweigh harms. In breast
cancer screening, absolute risk reduction increases and harm
decreases with increasing age.4 Th is is due to multiple factors.
The accuracy of mammography screening5,6 is affected by age
and density of the breast. There are higher false-positive rates in
the younger screening population because of breast density and
higher rates of BSE practice.7 The younger population apparently also tolerates a higher risk of false-positive results.8 All
patients should be informed of the risks as well as the benefits
of screening.

Breast SelfExamination
1. Is the breast self-examination (BSE) a useful screening tool?
The traditional goal of BSE was to detect palpable tumors. The
newer philosophy of BSE is its role in increasing self-awareness
of a womans normal breast so that there is improved ability to
detect changes and seek appropriate clinical help.14 Greenwald
et al. in the premammography era showed that those who practiced BSE had their cancers detected at a smaller size and earlier
stage.15 Interestingly, even among the BSE patients, cancers were
most commonly detected incidentally, giving weight to the association between patients who practice BSE and increased body
awareness.

577

PMPH_CH72.indd 577

5/22/2012 5:50:36 PM

578

Surgery: Evidence-Based Practice

Although there are actual recommendations published


against BSE16 (Level 1b evidence), the American Cancer Society,
American College of Obstetricians and Gynecologists, and The
American Cancer Society still recommend that women practice
BSE especially after age 40. Hackshaw et al.17 (Level 2a evidence)
in a meta-analysis of 20 observational studies and three clinical
trials determined that BSE had no effect on breast cancer mortality. In a well-conducted RCT of over 200,000 women investigating
whether or not instruction of BSE had an effect on breast cancer
mortality over 10 to 11 years of follow-up in Shanghai, Thomas et al.
showed no reduction of mortality, RR 1.04 (95% CI 0.821.33).18
(Level 1b evidence) Although this study was one about the
instruction of BSE rather than about execution and efficacy of
the BSE, one important point to consider is that 40% of the study
population and their matched cohorts were in their 30s. A 10-year
follow-up may not have been enough time to see the true number
of cancer deaths in that population. The Russian study19 (Level 1b
evidence), which included 120,000 women aged 40 to 64 years,
suggested that there were more cancers found in the BSE group
(24% more); however, the risk of dying was, once again, not significantly higher in the BSE group. The pooled RR of dying was
1.01 (95% CI .921.12, p = .79). Notably, there was an increase in
the number of benign breast biopsies in the BSE group, 53% more
biopsies in the BSE group from data from both trials combined,
RR 1.53 (95% CI 1.441.63, P < .001), as well as increased anxiety
among that same group.
Answer: Current evidence indicates that women may practice
BSE knowing the risks as well as benefits. There is insufficient consistent data to recommend for or against BSE. Level 1b evidence
reveals no mortality benefit. More cancers were found in the BSE
groups; however, they were found incidentally and not during the
formal monthly BSE.20,21 (Level 1b evidence) This suggests that
an increase in a womans self-awareness may lead her to seek a
clinical follow-up sooner for incidentally found masses. Beginning in their 20s, women should be told about the limitations and
benefits of BSE. Those who want to learn should be instructed,
with regular reinforcements of teaching and counseling. (Grade B
recommendation)

Clinical Breast Examination


2. Is the clinical breast examination (CBE) a useful screening
tool?
The data from randomized controlled trials did not evaluate
CBE as an individual screening tool, but as an adjunct to mammography. There is sparse evidence of CBE independently having
statistically significant effect on breast cancer detection or on mortality. CBE detected only 3% to 45% of breast cancers found that
were missed by screening mammography. Barton et al. reviewed
all controlled trials and casecontrol studies where CBE was at
least part of the screening modality. Analysis of all the trial data
showed that CBE sensitivity is estimated at 54% (95% CI 4860%)
and specificity at 94% (95% CI 9097%) if done systematically and
with training.22 (Level 2a evidence) A randomized trial from Edinburgh and one from the United States showed a reduction in mortality with screening with CBE and mammography of 29% and
14%, respectively4 (Level 2a evidence), but could not conclude how
much of that benefit was due to CBE. The randomized trial from
Canada, which studied women 50 to 59 years of age, compared

PMPH_CH72.indd 578

groups with CBE and BSE versus CBE, BSE plus mammography,
without comparing data from a control group who received no
screening. Though mortality rates were not statistically different
between the two groups, the CBE, BSE plus mammography group
detected more smaller and lymph node-negative breast cancers.23
(Level 1b evidence) The likelihood ratios of finding a breast cancer
on physical examination were higher in the carefully controlled
RCT setting compared with community settings (10.6 vs. 2.1),
suggesting that excellent CBE technique is important.
Answer: There is no direct evidence for or against CBE in
screening for breast cancer. However, the high specificity of abnormal findings increases the probability of breast cancer detection.
The American Cancer Society review panel suggested that rather
than discarding the CBE, clinicians use it as another opportunity
to raise awareness about the early detection of breast cancer.24
(Level 2a evidence; Grade B recommendation)

Mammography for Screening


3. When is mammography efficacious as a screening tool?
Breast cancer screening in the United States is estimated to have
reduced the death rate from breast cancer 7% to 23% (median
15%). Screening recommendations for women are divided into
recommendations for 40 to 49 year olds, 50 to 69 year olds, and
>70 year olds. Across the RCTs of mammography, the mortality reduction parallels the reduction in node-positive cancers.25
(Level 2a evidence) The largest benefit is in the 50 to 69 age group.
Screening the 40 to 49 groups has shown lower mortality benefit.
The power of the studies for this group was low. Mortality benefit
may also be skewed as the incidence of cancers is lower in this
group. However, it is argued that in young women, cancer is usually more aggressive and women benefit mostly from appropriate
treatment at the earliest stage possible. The 9.8% annual falsepositive rate of mammography should be discussed with the 40 to
49 groups. The >70 age group has limited data. Most of the RCTs
studied patients up to age 65. Mammograms are the most sensitive in this age group and detection rates are better, but the decision to screen in this elderly age group must take into account
the patients overall health, comorbidities, longevity, and their
personal wishes. Mammography may miss up to 30% of cancers;
cancers found by mammography may still have poor prognosis.26
Other downsides to mammography for screening include false
positives, benign breast biopsies, anxiety, and increased repeated
imaging after an abnormal reading.
In the last 10 years, there have been no RCTs for breast cancer screening with mammography, CBE, or BSE with death from
breast cancer as the outcome measured. In general, the eight historic RCTs that all occurred before 1981 have shown that mammography screening has significantly reduced breast cancer
mortality.27 (Level 1b evidence) The debate rages on over who
benefits most, and what the true mortality benefit might be. The
eight well-established trials are from the United States, Edinburgh, Gothenburg, Stockholm, Malmo, Sweden, and Canada.
The recruitment of patients, mammography protocol, control
groups, size, and follow-up all varied. The ages ranged from 40
to 74. There were flaws in all the studies, which have all been rigorously dissected and subsequently defended multiple times in
subsequent years.4,24,28 (Levels 1a and 2a evidence) In 2000, a controversial meta-analysis by Gotzsche and Olsen29 questioned the

5/22/2012 5:50:36 PM

Screening, Breast Biopsy, Benign Disease

validity of these historic RCTs. This sparked re-analysis and more


meta-analyses of the data to answer these criticisms. Follow-up
data also matured and became available.
In the 5069 year old group, meta-analyses that included
all the trials demonstrated statistically significant 20% to 35%
reductions in mortality from breast cancer.30 (Level 2a evidence)
Humphrey et al.4 (Level 1a evidence) independently reviewed all
RCTs and concluded that the methodologic flaws in each trial did
not negate the effectiveness of mammography for screening especially in women 50 to 69. Their newer analyses of the old data supported breast cancer screening as efficacious in reducing breast
cancer mortality, albeit with a wide range: 7% to 44% overall
studies. Fletcher and Elmore in a similarly exhaustive review of
the literature demonstrated statistically significant reductions in
mortality from breast cancer of 20% to 35% for women 50 to 69
who were routinely screened.30 Blanks et al. in 2000 designed a
highly significant age cohort model (p < .001) in a communitybased screening setting in the UK. Th is study revealed that the
mortality dropped across all ages with mammography for screening, with the largest drop in mortality seen in the 5569 groups.
This study attempted to differentiate the effects of screening from
that of improved therapy upon mortality. Their estimate of the
effects of screening is a 6.4% reduction in mortality, and the estimate of the effects of therapy on reduction in mortality is 14.9%.31
(Level 2b evidence)
The benefit of screening women 40 to 49 is slower to appear
as the incidence of cancer is lower in that age group, requiring a
longer follow-up to show benefit or harm. The HIP study out of
the United States showed a possible 25% reduction in mortality
benefit after 10 to 18 years of follow-up.32 (Level 1a evidence) The
data gathered from the eight RCTs in regards to women 40 to 49
were re-analyzed by Smart et al. in 1995 using a MantelHaenszel
estimator method to assess the follow-up data. This showed a 16%
benefit from screening mammography without statistical significance at the 95% confidence level. However, when the data from
the Canadian trial was excluded, which actually showed no benefit
to this group, the analysis showed 24% benefit with statistical significance.33 (Level 1a evidence) Most of the RCTs individually did
not have a statistical power to evaluate 40 to 49 year olds. Hendrick
et al. in 1997 re-analyzed the eight RCTs in another meta-analysis
when follow-up reached 18 years (avg time 12.7 years), and found
a statistically significant 18% mortality reduction among women
screened (RR 0.82; 95% CI 0.710.95). When they looked only at
the Swedish trials, the mortality reduction was significant and
even higher at 29% (RR 0.71; 95% CI 0.570.89).34 (Level 1a evidence) The USPSTF meta-analysis4 calculated a number needed
to screen of 1385 (Cr I 6596060) to prevent one death. A historical prospective cohort study of 4482 women aged 40 to 49, with a
follow-up to 9.9 years (median 4.7 years) examined prognostic differences in cancers detected by mammography versus BSE, CBE,
or incidental patient findings. In the mammography group, the
mean tumor size detected was smaller (p < .002), more likely to be
localized (p < .0001), and had better survival even after adjustment
for lead-time bias (p < .0001).21 (Level 1b evidence) The question
of benefit versus harm is important especially in this age group.
Annual mammography of 100,000 women starting at age 40, for
10 consecutive years, will result in at most eight cancer deaths due
to the radiation exposure during the lifetime of these women.35
Denser breast tissue in women under 50 decreases the sensitivity of mammography leading to higher false-positive rates, more

PMPH_CH72.indd 579

579

benign breast biopsies, scarring, and anxiety. In this age group,


there is lower incidence of disease, and faster growing cancers.5
Conversely, in older women, when density decreases, false positives decrease; cancers are less aggressive and have longer sojourn
times. The point at which the trade-off between harm and benefit
becomes acceptable is subjective and therefore both the benefits
and harms of screening must be discussed with patients.
For women greater than age 70, screening is also controversial. Of the eight RCTs, only two included women 70 and older and
is of limited power for strong screening recommendations. Relative risk reductions of death from breast cancer in women 65 to 74
years were reported as 0.68 (CI 0.510.89) from the Malmo trial,
and 0.78 (CI 0.620.99) in women 70 to 74 years in the updated
analysis of the Swedish trial.36 (Level 1b evidence) In 2000, a study
out of California investigated 690,000 women aged 66 to 79 years
and found that in situ, local, and regional breast cancers were
more likely to be detected among women who underwent screening mammography, with a relative risk of detection in the screened
group of 3.3 (95% CI 3.13.5). Overall, the risk of metastatic
disease was reduced by 43% among women who were screened
(p < .0001).37 (Level 2b evidence) Screening in this population
was successful in that cancers were detected at an early stage.
Twenty-four early stage breast cancers were diagnosed for each
metastatic case prevented. However, in this age group, detecting
early stage breast cancer may not have as high an impact on breast
cancer mortality due to comorbidities from which patients are
more likely to succumb. Over-treating early stage breast cancer in
elderly should be avoided. In a small cohort study, 17% of screened
women experienced burden, including false-positive results, further workup requirements, anxiety, the need to refuse workup,
and identification of clinically insignificant lesions. Forty-two
percent of women had clinically documented anxiety, pain, and
depressive symptoms after screening.38 (Level 2b evidence) Satariano, in a longitudinal observational study of over 900 women, 40
to 84 years old concluded that trials for appropriate assessment
of the efficacy of screening should include comorbidity measures.
Their results also supported the finding that women with severe
comorbid conditions have uniformly higher mortality rates, in
whom there was no survival advantage in detecting early stage
breast cancers.39 (Level 1b evidence) Annual mammography is
still recommended for women >70 years of age, but individual
decisions to screen must be made carefully for elderly individuals
based on comorbidities and overall life expectancy.
Answer: Current recommendations based on the available
data support starting annual mammographic screening of asymptomatic normal risk women at age 40. CBE and BSE are to be used
with clinical judgment and with appropriate counseling. Annual
mammography for women should continue until age 70. After age
70, screening may continue with consideration of comorbid medical conditions that may affect the life expectancy and whether
a patient would accept further treatment and procedures after
screening detects abnormalities. Patients need to be aware of the
potential harms associated with breast cancer screening including false positives and false negatives, biopsies for benign results,
scarring on future imaging studies after biopsies, and associated
anxiety. When cancers are detected at an earlier stage, there is
the possibility of less aggressive therapy, and more treatment
options.26 At this time, breast ultrasound and breast MRI are not
used for screening asymptomatic average-risk women. (Grade B
recommendations)

5/22/2012 5:50:37 PM

580

Surgery: Evidence-Based Practice

Percutaneous Breast Biopsies


4. Is the sensitivity and specificity of percutaneous breast
biopsy sufficient to replace open surgical biopsy for initial diagnosis of a breast abnormality?
Modern breast care strives to minimize the number of open surgical
biopsies performed for initial diagnosis on patients with abnormal
imaging or with clinically apparent lesions. The goal is to optimize
the accuracy of atypical or malignant diagnoses and minimize
the underestimation rates of the percutaneous procedure of choice,
although taking into account patient comfort, complication rates,
and days until return to work. There are pitfalls of benign diagnoses such as false negatives, radiologichistologic discordance, and
lack of appropriate follow-up, as well as patient anxiety. Percutaneous biopsies have increasingly become the alternative to open
surgical biopsy for initial diagnosis of breast abnormality.40 Percutaneous biopsy can be obtained stereotactically, with ultrasound
guidance, or with clinical palpation and can be performed with
large core needle biopsy (CNB) usually 14-, 11-, or 8-gauge or with
fine-needle aspiration (FNA), usually 22-gauge.

is no correlation between the sonographically placed marker to


the mammographic finding on the postprocedure film, then stereotactic biopsy must still be undertaken for adequate sampling of
the mammographic finding.

ULTRASOUND-GUIDED BIOPSIES
Ultrasound-guided automated core biopsies have also been studied
and shown to have up to 100% concordance rates compared with
open surgical excision. Dillon et al. in 2005 published the largest retrospective study at that time, which included all patients undergoing
biopsy in a 5-year period with the goal of assessing which method
of core biopsy is the most accurate: ultrasound-guided, stereotactic, or clinical palpation. The study is limited by its 3-year follow-up;
however, the false-negative rate was only 1.7% for ultrasound-guided
biopsies, whereas it was 13% for clinical and 8.9% for stereotactic
biopsies.44 (Level 2b evidence) In addition, studies have also shown
the cost-effectiveness, relative comfort, and safety of ultrasoundguided biopsies. There is no exposure to ionizing radiation and in
live time, all parts of the breast and axilla are accessible.41

STEREOTACTIC BIOPSIES
FINE-NEEDLE ASPIRATION
Stereotactic biopsy is based on the principle that the precise
location of a lesion in three dimensions can be determined based
on its apparent change in position on two angled (stereotactic)
images.41 Brenner et al. in a multi-institutional prospective trial
showed that stereotactic CNBs have statistically significant concordance with surgical excision for obtaining accurate histopathologic diagnosis of breast lesions.42 (Level 1b evidence) CNBs
on 1003 patients after selection criteria were applied, had results
validated at surgery or clinical and mammographic follow-up
based on the American College of Radiology Breast Imaging Reporting and Data System (BIRADS), as well as a clearly
defined scoring scale for histopathologic samples obtained at
CNB or at surgery. Median follow-up was 24 months. The strict
sensitivity was 91% + 1.9% and the strict specificity was 100%.
Accuracy was 97%, with potential for improvement with careful recommendation of surgery based on image-discordant histology, and for suspected sampling error. Another multicenter,
prospective study from the COBRA group (Core Biopsy after
RAdiologic localization) studied 1029 lesions in five centers for
the assessment of diagnostic accuracy of stereotactic large CNB
in women aged 29 to 85 (mean 58). All patients were offered surgery after biopsy, whereas benign patients who refused surgery
were referred to a nationwide database registry to track them.
Discordant results were cored again or surgically excised. Benign
patients had 24 months of imaging follow-up. High-risk lesions
(ADH, ALH, LCIS) had open surgery. DCIS patients had wide
local excision. Invasive cancers were offered appropriate therapy.
The sensitivity was 97% (95% CI 9598%) and specificity 99%
(95% CI 97100%).43 (Level 1b evidence)
Stereotactic biopsies are suitable for mammographic microcalcifications or mammographic densities that have no ultrasound
correlate. If there is sonographic correlation of a mammographic
finding, biopsy should be attempted with an ultrasound guidance
and a radiopaque marker placed at the time of biopsy. A completion mammogram is then warranted to ensure concordance
between the mammographic and the sonographic lesions. If there

PMPH_CH72.indd 580

FNA may be used to sample palpable or nonpalpable breast


abnormalities; however, there are higher underestimation rates.
One NCI study determined that physician training was contributory to the accuracy of fine-needle aspiration biopsies (FNAB).
One thousand and forty-three cases were reviewed and analyzed
over 1 year to show that physicians formally trained in FNAB
had a 2% missed cancer rate and physicians with no training
in FNAB had a 25% missed cancer rate (p < .0001).45 (Level 2b
evidence) The sensitivity of FNAB in the trained group was 98%
versus 75% in the untrained group (p = 00014). In 2001, Pisano
et al. in a large multicentered trial that was double-armed and
randomized to stereotactic CNB versus stereotactic FNAB for
BIRADS 4 or 5 images closed the FNAB arm early because of the
high underestimation rates. Results showed sensitivity of 85% to
88% and specificity of 55% to 91%. FNAB was better for solid
masses than for microcalcifications, accuracy 67% versus 53%
(p = .006). FNAB with ultrasound was better than with stereotactic guidance with accuracy 77% versus 58.9% (p = .002).46 (Level 1b
evidence)
Percutaneous biopsy has documented advantages of being
faster, less costly, causing less mammographic scarring subsequently, and has lower complications rates than open surgical
biopsy.41 Because of the smaller needle size, FNAB can be more
suitable for patients that cannot stop anticoagulation or who have
other comorbid conditions that preclude large core biopsies. However, the practitioner must recognize the high underestimation
rate and convey this to the patient. Approximately, 70% to 80%
of lesions with sufficient concern to warrant biopsy will result in
benign findings. If there is imaging-pathologic concordance, the
patient avoids open surgery. If the diagnosis is malignant, surgical
planning can decrease the number of operations performed on
a patient, 2.01 operations in women with open surgical biopsies versus 1.25 operations in percutaneously diagnosed cancer (p < .001).47
(Level 2b evidence) Multicentricity of malignancy, proven before
open surgery, improves surgical planning. This allows for more

5/22/2012 5:50:37 PM

Screening, Breast Biopsy, Benign Disease

accurate recommendations for breast conservation versus total


mastectomy.48
Answer: Yes, use large gauge percutaneous CNB whenever
possible and biopsy using the modality with which the abnormality was detected. (Grade B recommendations)

SURGICAL MANAGEMENT OF
BENIGN BREAST DISEASES
A literature review of the last 10 years for surgical management
of benign breast diseases revealed three benign breast diseases of
importance: lobular neoplasia (LN), radial scars (RSs), and benign
breast papillomas. Because of the overall low incidence of these
lesions, there are many studies that, although well conducted, are
retrospective with small cohorts. Therefore, there is insufficient
power for Grade A recommendations. However, the following
reviews are currently available data.

Radial Scar
5. When is RS diagnosed by percutaneous CNB safe to follow
without complete surgical excision (CSE)?
RS, otherwise known as radial sclerosing lesion, sclerosed elastic
lesion, indurative mastopathy, nonencapsulated sclerosing lesion,
sclerosing papillary proliferative lesion, and if >1 cm, a complex
sclerosing lesion, are often incidental findings on autopsy but also
present as masses, architectural distortions with or without calcifications, and sonographic abnormalities. Strict adherence to
mammographic criteria for RS cannot differentiate between RS
and carcinoma, thus sampling of a suspected RS is still required49
(Level 2a evidence); however, which lesions need CSE is debatable.
Brenners retrospective multi-institutional study from 2002, which
included data from 11 institutions, studied 157 RSs diagnosed by
image-guided CNB for nonpalpable mammographically detected
lesions and concluded that RS could be reliably diagnosed on
CNB.49 Furthermore, for RS associated with atypia, 28% of these
patients had malignancy at CSE whereas only 4% had malignancy
in the RS group without atypia (p < .0001). At CSE of this latter RS
group without atypia, carcinoma was missed in 9% of lesions biopsied with a spring-loaded device but was missed in 0% of biopsies
performed with a vacuum-assisted device (p = .01); carcinoma was
missed in 8% of lesions sampled with <12 specimens per lesion but
in 0% of lesions sampled by >12 specimens (p = .015). Of the RS
patients that did not proceed to surgery, all 55 cases were followed
up mammographically; none showed progression for a median of
38 months. Brenner concluded that CNB of sufficient specimens
(>12), absence of atypia, and concordant imaging was reliable to
diagnose RS with no expectation of underestimation. Unless all
these criteria are met, surgical excision is still recommended.
Another systematic review of eight studies found 341 RSs and evaluated them for malignancy upgrade. None of the RS diagnosed by
11- or larger gauge devices had underestimated the lesion.50 (Level 4
evidence) Becker et al., in 2006 from Canada, had the only other
large single-center retrospective trial on RS which included 144
RS lesions of 15,986 consecutive CBNs. Although there were study
limitations, they paralleled the findings of Brenner et al. in that
large (11- or 8-) gauge biopsies of RS that were imaging concordant
and revealed no atypia or other finding that would independently

PMPH_CH72.indd 581

581

require CSE, was safe for interval imaging follow-up to a 2-year


stability endpoint to ensure benignity.51 (Level 2b evidence)
Answer: RS diagnosed by CNB with >12 cores from an 11or larger gauge needle, without atypia, with imaging concordance
(pathology discordant to the imaging must proceed to CSE), and
with no other independent finding that would require CSE was
safe for nonoperative management if followed up with imaging
and CBE for 2 years to demonstrate continued stability. (Grade B
recommendation)

Lobular Neoplasia
6. What is the appropriate surgical management of LN?
Definitive management of LN, previously known as Lobular Carcinoma In Situ (LCIS), and Atypical Lobular Hyperplasia (ALH)
also have no current Level 1 evidence. The incidence of LN is
unknown in the general population and occurs in up to 2.7% on
biopsy of benign lesions. LCIS confers a lifetime risk of cancer of
30% to 40% in either breast, versus 12.5% in average-risk women.
ALH is defined as a precursor lesion as well as a high-risk marker
conferring a 10% to 20% risk of subsequent carcinoma and a
>3-fold risk of invasive carcinoma (lobular > ductal type) in the
ipsilateral breast.52 (Level 2b evidence) LN found on CNB must
be followed by CSE if the core also yields cancer, DCIS, or other
high-risk lesions (ADH or RS). LN on a CNB alone but performed
for BIRAD 4 or 5 finding needs CSE, as 33% of BIRAD 4 lesions
and 90% of BIRAD 5 lesions yield carcinoma when excised.53
(Level 2b evidence) In a small study from Columbia University
Breast Department, Bauer et al.54 (Level 2b evidence) proposed
that isolated LN with benign fibrocystic changes concordant with
imaging findings, adequately sampled by multiple large CNB
specimens, and with no synchronous or prior breast cancer can
be managed with close follow-up consisting of yearly mammograms with high-risk discussions and biannual CBE. However,
the authors cautioned that larger prospective studies were needed.
Another small prospective study55 (Level 2b evidence) adds to the
above conclusions that LN associated with a mass must be completely excised, as the LN is likely an incidental finding to the
mass. Brem et al. in 200856 (Level 3b evidence) showed statistically
significant underestimation rates when LN was associated with
a mass (p < .001), with high BIRAD scores (4 or 5) (p < .01), with
small specimen sizes (p < .01), and with fewer than 10 specimens
(p = .0027). They concluded that patients with LN at CNB should
be completely excised. ALH of fewer than three foci with no additional findings that independently require excision and with no
imaging discordance may be managed nonoperatively with close
follow-up. This was shown in a 2010 study from Johns Hopkins
that found 117 isolated ALH cases of 10,024 breast CNBs done
during the 10-year study period for women 40.6 to 85.5 years with
up to 10-year followup.57 (Level 2b evidence) In this study, LN
was defined by the Page criteria58,59 and a focus was defined as
involvement by one or more (up to 3) adjacent lobules. In the
group with <3 foci of ALH, all patients went on for CSE and none
had higher risk lesions or malignancy. Only two women developed minimal DCIS at 1.5 and 2.3 years later during follow-up.
Answer: Current practice of CSE of LN found on CNB has
no evidence to support or contradict it. Multiple studies (Level 2b
evidence and worse) recommend CSE, especially when associated
with a mass, associated with BIRADS 4 or BIRADS 5 imaging, or
if there is insufficient sampling (less than 1012 specimens from a

5/22/2012 5:50:37 PM

582

Surgery: Evidence-Based Practice

large bore needle). However, careful consideration for risk stratification as well as adherence to imaging concordance is crucial
for surgical decision-making; otherwise close follow-up with CBE
and imaging, as well as the use of chemoprophylaxis, remains
acceptable until larger clinical trials are available. Minimal ALH
may be managed nonoperatively with close clinical and imaging
follow-up. (Grade BC recommendations)

Benign Papillary Lesions


7. Should all benign papillary lesions diagnosed on breast CNB
be completely excised?
Papillary lesions pose similar management dilemmas, as the
overall incidence is low (5% of breast biopsy specimens). There
are little radiologic signs that distinguish benign from atypical or
malignant papillary lesions. Papillary lesions, also named sclerosing papillomas, benign sclerosing lesions, have a 7-fold risk of subsequent development of carcinoma if they demonstrate atypia.60
Atypical lesions are excised due to the atypical finding. No Level 1
data support defi nitive surgical management of nonpalpable
benign papillary lesions found on breast CNB. Jacobs et al.61 (Level 3a
evidence) looked at data from five retrospective studies that
ranged from having 734 to 1327 consecutive CNBs that yielded
12 to 28 benign papillomatous lesions each. They concluded that
there is a small but definite chance of atypia or malignancy on
CSE. The incidence of carcinoma or ADH at CSE can be as high
as 36%.62 (Level 2b evidence) Liberman et al.63 (Level 2b evidence)
found malignancy in 14% (5/35, 1 IDC, and 4 DCIS) of concordant
benign papillary lesions in a retrospective study of 3864 lesions,

which yielded 50 papillomas on CNB, 35 of which had CSE or


2-year follow-up. Four DCIS were found at interval follow-up at a
median of 22 months (range 725 months), the one invasive cancer was found immediately at CSE. The frequency of cancer (14%)
is higher than the <2% frequency of cancer in lesions interpreted
as probably benign designated as BIRADS 3. This suggests treating benign papillomas as a lesion between a BIRAD 3 which could
be managed nonoperatively with a 6-month imaging follow-up,
and BIRADS 4 lesion which requires excision. Th is study also
found 17% high-risk lesions on CSE (ADH, RS, LN) with multiple versus solitary papillomas giving higher frequency of cancer
(p = .02). Data from Cheng et al. support this and also found statistical significance in age >45 (p = .0008) as a predictor of upgrading
a benign papilloma to malignancy.64 (Level 2b evidence) Renshaw
showed in a study of 62 papillary lesions that the rate of carcinoma
in severely atypical papillomas was greater than in benign papillomas (p < .0001). Also, no upgrades to malignancy were found at
surgery for benign papillomas with florid hyperplasia, and no or
minimal atypia, leading them to be the only study to recommend
that this group be managed nonoperatively.65 (Level 2b evidence)
Answer: Current recommendations are for CSE of benign
nonpalpable papillary lesions found on breast CNB. (Grade BC
recommendations)
Benign surgical findings found at biopsy of abnormal imaging can be treated according to the above recommendations. The
keys to successful management are accurate tissue sampling,
accurate pathologic assessment, and strict adherence to imagingpathologic concordance, and minimum 2-year follow-up with
imaging and CBE for every percutaneously diagnosed benign
breast lesion.

Clinical Question Summary


Questions

Answers

Grade of
Recommendation

References

1 Is the breast selfexamination (BSE) a


useful screening tool?

Although BSE has limitations, and no mortality benefit is


clearly demonstrated, there is current evidence that women
who practice BSE have more awareness of their bodies.
Extrapolation from these data leads current decisionmaking bodies to continue to recommend that clinicians
continue to teach BSE for those women who wish to proactively participate in their own breast health maintenance.

14-21

2 Is the clinical breast


examination (CBE) a
useful screening tool?

The high specificity of the CBE and absence of data


demonstrating any negative effects of CBE allow us to
continue to recommend it for breast cancer screening.

22-24

3 When is
mammography
efficacious as a
screening tool?

Annual screening mammography should start at age 40 and


continue to 70. After 70, it may continue as long as the
clinicians also take into account the patients comorbidities,
life expectancy, and personal acceptance of possible therapy
that may result from the screening.

25-39

4 Is the sensitivity and


the specificity of
percutaneous breast
biopsy sufficient to
replace open surgical
biopsy for initial
diagnosis of a breast
abnormality?

Yes, using large gauge percutaneous CNB whenever possible


using the modality with which the abnormality was
detected.

40-46

(Continued)

PMPH_CH72.indd 582

5/22/2012 5:50:37 PM

Screening, Breast Biopsy, Benign Disease

583

(Continued)
Questions

Answers

5 When is RS
diagnosed by
percutaneous
CNB safe to follow
without complete
surgical excision
(CSE)?

RS diagnosed by CNB with >12 cores from an 11- or larger


gauge needle, without atypia, with imaging concordance,
and with no other independent finding that would require
CSE was safe for nonoperative management if followed up
with imaging and CBE for 2 years to demonstrate continued
stability.

49-51

6 What is the
appropriate surgical
management of LN?

LN associated with a breast mass, BIRADS 4, or BIRADS


5 imaging must have CSE. LN, if not discordant, may be
observed with close follow-up. Chemoprophylaxis may be
discussed with oncologist. Minimal ALH may be managed
nonoperatively.

BC

52-59

7 Should all benign


papillary lesions
diagnosed on breast
CNB be completely
excised?

Yes, all benign papillomas on core should be surgically excised.

BC

60-65

REFERENCES
1. American Cancer Society. Breast Cancer Facts and Figures 20092010. Atlanta: American Cancer Society, Inc.
2. American Cancer Society. Cancer Facts and Figures 2010. Atlanta:
American Cancer Society; 2010.
3. NCCN Clinical Practice Guidelines in Oncology. Breast Cancer
Screening and Diagnosis. Version 1. 2011.
4. Humphrey LL, Helfand M, Chan BKS, et al. Breast Cancer
Screening: A Summary of the Evidence for the U.S. Preventative
Services Task Force. Ann Intern Med. 2002; 137:347-360.
5. Carney PA, Miglioretti DL, Yankaskas BC. Individual and combined effects of age, breast density, and hormone replacement therapy use on the accuracy of screening mammography. Ann Intern
Med. 2003;138:168-175.
6. Griggs K. Ultrasound as a secondary screening tool in mammographically dense breasts. Radiographer. 2006;53(1):20-23.
7. Smith EM, Burns TL. The effects of breast self-exam practice and
physician examination on extent of disease at diagnosis. Prev
Med. 1980;9:409-417.
8. Schwartz LM, Woloshin S, Sox HC, et al. US womens attitudes
to false positive mammography results and a detection of ductal carcinoma in-situ: Cross sectional survey. Br Med J. 2000;320:
1635-1640.
9. Childrens Oncology Group. Long Term Follow-up Guidelines.
Version 2010.
10. Gail MH. Projecting individualized probabilities of developing
breast cancer for white females who are being examined annually. J Natl Cancer Inst. 1989;81:1879-1886.
11. Sur D, Holly M. Breast cancer prevention and Tx: An evidencebased guide. J Fam Prac. 2010:59(10):575-581.
12. Boyle P, Boffetta P. Alcohol consumption and breast cancer risk.
Breast Cancer Res. 2009;11(Suppl 3):S3.
13. Duff y CM, Assaf A, Cyr M, et al. Alcohol and folate intake and
breast cancer risk in the WHI Observational Study. Breast Cancer Res Treat. 2009;116:551-562.
14. Harris R, Kinsinger LS. Routinely teaching breast self-examination
is dead. What does this mean? J Natl Canc Inst. 2002;94:1420-1421.

PMPH_CH72.indd 583

Grade of
Recommendation

References

15. Greenwald P, Nasca PC, Lawrence CE, et al. Estimated effect


of breast self examination and routine physical examination
on breast cancer mortality. N Engl J Med. 1978;299:271-273.
16. Baxter N. Preventative health care, 2001 update: Should women
be routinely taught self-examination to screen for breast cancer?
Can Med Assoc J. 2001;164(13):1837-1846.
17. Hackshaw AK, Paul EA. Breast self-examination and death from
breast cancer: A meta-analysis. Br J Cancer. 2003;88:1047-1053.
18. Thomas DB, Gao DL, Ray RM, et al. Randomized trial of breast
self-examination in Shanghai: Final results. J Natl Cancer Inst.
2002;94(2):1445-1457.
19. Semiglazov VF, Moiseyenko VM, Bavli IL, et al. The role of
breast self-examination in early breast cancer detection (Results
of the 5-years USSR/WHO randomized trial). Eur J Epidemiol.
1992;8:498-502.
20. Miller AB, Baines CJ, To T, Wall C. Canadian National Breast
Screening Study: 2. Breast cancer detection and death rates among
women aged 50-59 years. Can Med Assoc J. 1992;147:1477-1488.
21. McPherson CP, Swenson KK, Jolitz G, Murray CL. Survival of
women ages 40-49 years with breast carcinoma according to
method of detection. Cancer. 1997;79:1923-1932.
22. Barton MB, Harris R, Fletcher SW. Does this patient have breast
cancer? The screening clinical breast examination: should it be
done? How? JAMA. 1999;282(13):1270-1280.
23. Miller AB, To T, Baines CJ, Wall C. Canadian National Breast
Screening Study-2: 13-year results of a randomized trial in women
aged 50-59 years. J Natl Cancer Inst. 2000;92(18):1490-1499.
24. Smith RA, Saslow D, Sawyer KA, et al. American Cancer Society
Guidelines for Breast Cancer Screening: Update 2003. CA Cancer J Clin. 2003;53:141-169.
25. Smith RA, Duff y SW, Gabe R, Tabar L, Yen AM, Chen TH. The
randomized trials of breast cancer screening: What have we
learned? Radiol Clin N Am. 2004;42(5):793-806.
26. Smith RA, Cokkinides V, EyreHJ. American Cancer Society
Guidelines for the Early Detection of Cancer, 2006. CA Cancer J
Clin. 2006;56:11-25.
27. Tabar L, Vitak B, Chen TH, Yen MF, DuffySW, Smith RA. Beyond
randomized controlled trials: Organized mammographic screening

5/22/2012 5:50:37 PM

584

28.
29.
30.
31.

32.

33.

34.

35.

36.

37.

38.

39.

40.

41.
42.

43.

44.

45.

46.

Surgery: Evidence-Based Practice

substantially reduces breast carcinoma mortality. Cancer. 2001;


91(9):1724-1731.
Elmore JG, Armstrong K, Lehman CD, Fletcher SW. Screening
for breast cancer. JAMA. 2005;293:1245-1256.
Gotzsche PC, Olsen O. Is screening for breast cancer with mammography justifiable? Lancet. 2000;355:129-134.
Fletcher SW. Mammographic screening for breast cancer. N Engl
J Med. 2003;348:1672-1680.
Blanks RG, Moss SM, McGahan CE, et al. Effect of NHS breast
screening programme on mortality from breast cancer in England and Wales, 1990-8: Comparison of observed with predicted
mortality. Br Med J. 2000;321:665-669.
Fletcher SW, Black W, Harris R, Rimer BK, Shapiro S. Report
of the International Workshop on Screening for Breast Cancer.
J Natl Cancer Inst. 1993;85(20):1644-1656.
Smart CR, Hendrick RE, Rutledge JH, 3rd, Smith RA. Benefit
of mammography screening in women ages 40-49 years. Current evidence from randomized controlled trials. Cancer. 1995;
75(5):1619-1626.
Hendrick RE, Smith RA, Rutledge JH, 3rd, Smart CR. Benefit
of mammography screening in women ages 40-49: A new metaanalysis of randomized controlled trials. J Natl Cancer Inst
Monogr. 1997;22:87-92.
Moss S. A trial to study the effect on breast cancer mortality of
annual mammographic screening in women starting at age 40. J
Med Screen. 1999;6:144-148.
Nystrom L, Andersson I, Bjurstam N, et al. Long term effects
of mammography screening: Updated overview of the Swedish
randomized trials. Lancet. 2002;359:909-919.
Bindman RS, Kerlikowske K, Gebretsadik T, Newman J. Is
screening mammography effective in elderly women? Am J Med.
2000;108:112-119.
Walter LC, Eng C, Coviesky KE. Screening mammography for
frail older women. What are the burdens? J Geri Intern Med. 2001;
16(11):779-784.
Satariano WA, Ragland DR. The Effect of comorbidity on 3-year
survival of women with primary breast cancer. Ann Intern Med.
1994;120:104-110.
Duijm LEM, Groenewoud JH, Roumen RMH, et al. A decade of
breast cancer screening in The Netherlands: Trends in the preoperative diagnosis of breast cancer. Breast Cancer Res Treat. 2007;
106:113-119.
Liberman L, Percutaneous image-guided core breast biopsy. Rad
Clin N Am. 2002;40:483-500.
Brenner RJ, Bassett LW, Fajardo LL, et al. Stereotactic coreneedle breast biopsy: A multi-institutional prospective trial.
Radiology. 2001;218:866-872.
Verkooijen HM, Diagnostic accuracy of stereotactic large-core
needle biopsy for nonpalpable breast disease: Results of a multicenter prospective study with 95% surgical confi rmation. Int J
Cancer. 2002;99:853-859.
Dillon MF, Hill ADK, Quinn CM, et al. The accuracy of ultrasound, stereotactic, and clinical core biopsies in the diagnosis of
breast cancer, with an analysis of false negative cases. Ann Surg.
2005;242:701-707.
Ljung BM, Drejet A, Chiampi N, et al. Diagnostic accuracy of
fine-needle aspiration biopsy is determined by physician training
in sampling technique. Cancer (Cancer Cytopathol). 2001;93:
263-268.
Pisano ED, Fajardo LL, Caudry DJ, et al. Fine-needle aspiration
biopsy of nonpalpable breast lesions in a multicenter clinical

PMPH_CH72.indd 584

47.
48.

49.

50.

51.

52.

53.
54.

55.

56.

57.

58.

59.

60.

61.

62.

63.

64.

65.

trial: Results from the Radiologic Diagnostic Oncology Group V.


Radiology. 2001;219:785-792.
Smith DN, Christian R, Meyer JE. Large-core needle biopsy of
nonpalpable breast cancers. Arch Surg. 1997;132:256-259.
Liberman L, Dershaw DD, Rosen PP, et al. Core needle biopsy of
synchronous ipsilateral breast lesions: Impact on treatment. AJR
Am J Roentgenol. 1996;166:1429-1432.
Brenner RJ, Jackman RJ, Parker SH, et al. Percutaneous core needle biopsy or radial scars of the breast: When is excision necessary? AJR Am J Roentgenol. 2002;179:1179-1184.
Sohn VY, Causey MW, Steele SR, et al. The treatment of radial
scars in the modern era-surgical excision is no required. Am Surg.
2010;76:522-525.
Becker L, Trop I, David J, et al. Management of radial scars found
at percutaneous breast biopsy. Can Assoc Radiol J. 2006;57(2):
72-78.
Page DL, Schuyler PA, Dupont WD, et al. Atypical lobular hyperplasia as a unilateral predictor of breast cancer risk: A retrospective cohort study. Lancet. 2003;361:125-129.
Orel SG, Kay N, Reynolds C, et al. BI-RADS categorization as a
predictor of malignancy. Radiology. 1999;211:845-850.
Bauer VP, Ditkoff, Schnabel F, et al. The management of lobular
neoplasia identified on percutaneous core breast biopsy. Breast J.
2003;9(1):4-9.
Elsheikh TM, Silverman JF. Follow-up surgical excision is indicated when breast core needle biopsies show atypical lobular
hyperplasia or lobular carcinoma in situ. A correlative study of
33 patients with review of the literature. Am J Surg Pathol. 2005;
29:534-543.
Brem R, Lechner MC, Jackman RJ, et al. Lobular neoplasia at
percutaneous breast biopsy: Variables associated with carcinoma
at surgical excision. AJR Am J Roentgenol. 2008;190:637-641.
Subhawong AP, Subhawong TK, Khouri N, et al. Incidental minimal atypical lobular hyperplasia on core needle biopsy: Correlation with findings on follow-up excision. Ann J Surg Pathol. 2010;
34:822-828.
Page DL, Dupont WD, Rogers LW, et al. Atypical hyperplastic
lesions of the female breast. A long-term follow-up study. Cancer.
1985;55:2698-2708.
Page DL, Kidd TE, Dupont WD, et al. Lobular neoplasia of the
breast: Higher risk for subsequent invasive cancer predicted by
more extensive disease. Hum Pathol. 1991;22:1232-1239.
Page DL, Salhany KE, Jensen RA, Dupont WD. Subsequent
breast carcinoma risk after biopsy with atypia in breast papilloma. Cancer. 1996;78:258-266.
Jacobs TW, Connoly JL, Schnitt SJ. Nonmalignany lesions in
breast core needle biopsies. To excise or not to excise? Am J Surg
Pathol. 2002;26(9):1095-1110.
Mercado CL, Hamele-Bena D, Oken S, et al. Papillary lesions of
the breast at percutaneous core-needle biopsy. Radiology. 2006;
238(3):801-808.
Liberman L, Tornos C, Huzjan R, et al. Is surgical excision warranted after benign, concordant diagnosis of papilloma at percutaneous breast biopsy? AJR Am J Roentgenol. 2006;186:1328-1334.
Cheng TY, Chen CM, Lee MY, et al. Risk factors associated with
conversion from nonmalignant to malignant diagnosis after surgical excision of breast papillary lesions. Ann Surg Oncol. 2009;
16:3375-3379.
Renshaw AA, Derhagopian RP, Tizol-Blanco DM, Gould EW.
Papillomas and atypical papillomas in breast core needle biopsy
specimens. Am J Clin Pathol. 2004;122:217-221.

5/22/2012 5:50:37 PM

Commentary on
Screening, Breast Biopsy,
Benign Disease
Richelle Williams and David P. Winchester

In the chapter entitled Screening, Breast Biopsy, Benign Disease,


Adora Fou-Cockburn and Mark Cockburn provide an excellent
review of the current literature on seven key areas of controversy
in breast cancer screening and the management of certain benign
breast diseases. The questions addressed were (1) Is the Breast
Self-Examination (BSE) a useful screening tool? (2) Is the Clinical
Breast Examination (CBE) a useful screening tool? (3) When is
mammography efficacious as a screening tool? (4) Is the sensitivity
and specificity of percutaneous breast biopsy sufficient to replace
open surgical biopsy for initial diagnosis of a breast abnormality?
(5) When is radial scar (RS) diagnosed by percutaneous core needle biopsy safe to follow without complete surgical excision (CSE)?
(6) What is the appropriate surgical management of Lobular Neoplasia (LN)? and (7) Should all benign papillary lesions diagnosed
on breast core needle biopsy be completely excised?
These are all extremely important topics to discuss and to
provide guidance and education, not only for us as physicians
but also for the public. In fact, several of these same questions
(BSE, when to start mammography, percutaneous vs. open surgical breast biopsy) have garnered much media attention in recent
years. As the authors highlight in this chapter, the evidence associated with many of these issues is quite variable and at times even
conflicting, but they do a great job of sorting through the noise
to find a signal and come up with meaningful recommendations.
Throughout all of this is the idea that while guidelines are helpful, there has to be a certain amount of tailoring of ones care to
address the specific circumstances of the patient in front of you
the art, not so much the science, of medicine. The chapter focuses
mainly on women at average risk undergoing screening.
Regarding the fi rst question, BSE as a screening tool has
been a subject of great debate over the past 2 years. There have
been recommendations both for and against BSE. Proponents
argue that women who perform BSE had more cancers detected
at a smaller size and earlier stage, which may be due to increased
self-awareness. Despite this, the best evidence shows no effect
on breast cancer mortality.1 Thus, these earlier stage detections
do not seem to translate to improved mortality. Opponents add
that BSE leads to increased numbers of benign breast biopsies
with the attendant increase in unnecessary resource utilization
and patient stress.2,3 The authors highlight the largest study providing evidence against routine BSE, the Shanghai randomized

controlled trial of over 200,000 women (their reference 18), and


caution that one of the limitations was that a significant number
of these patients were under 40 and thus the 1011-year follow-up
period may not be adequate to see the true number of cancer
deaths. While that may be a fair criticism, the Russian study the
authors cite (their reference 19) ultimately followed about 120,000
women aged 40 to 64 years for 15 years and also did not find a
difference in mortality.3 The final analysis of that study, however,
did show a significantly higher detection rate of both benign and
malignant tumors in the BSE group.3 As a result, it is difficult to
advocate against teaching a woman BSE, particularly if she wants
to learn and is aware of the risks outlined above. It seems to us
that a womans response may depend on her personality and life
philosophy, and the authors rightly conclude that the decision to
practice BSE should be individualized based on a discussion of
the risks/benefits between patient and physician.
Unlike BSE, CBE has no direct evidence to support a firm
stand either for or against its use in breast cancer screening. The
best studies do not isolate the effect of CBE independent of the
effect of mammography, which has been shown to decrease breast
cancer mortality.3 However, as the authors point out, while CBE is
not very sensitive (54%), it is quite specific (94%) for breast cancer
detection. In addition, since the likelihood of finding an abnormality increases with excellent technique, routine CBE provides an
opportunity for the physician to perfect his/her technique as well
as educate the patient about the importance of early detection.
The third question about when mammography should be utilized as a screening tool has also been hotly debated, especially
after the publication of the U.S. Preventive Services Task Force
report in 2009 recommending biennial mammogram starting at
age 50.3 The controversy is mostly regarding the appropriateness of
screening mammography in 4049year-old women. While there
is no doubt that mammography decreases breast cancer mortality
in this group, the reduction is less than for 5069-year-old women.
Furthermore, it comes at a price since the greater breast density in
younger women results in a greater number of false positives with
increased benign breast biopsies and anxiety. Also, because the
incidence of breast cancer is lower in this age group, the number
of women needing to be screened to prevent one cancer death is
likely higher.3 Given that the point at which the benefit of preventing breast cancer mortality outweighs these risks is subjective, the
585

PMPH_CH72.indd 585

5/22/2012 5:50:37 PM

586

Surgery: Evidence-Based Practice

authors advocate (and we agree) having the discussion with the


patient. Similarly, but at the other end of the spectrum, screening mammography in women over 70 is also controversial. In this
group, the issue of life expectancy related to other comorbidities
starts to play a significant role in treatment decisions, and should
therefore also impact decisions about screening.
Regarding the sensitivity and specificity of percutaneous
breast biopsy versus open surgical biopsy, many studies have been
published examining this issue. Percutaneous needle biopsy techniques have improved in accuracy with increased experience and
also utilization of image guidance and vacuum-assisted methods.
Overall, large core needle biopsy seems to have some advantages
over fine-needle aspiration in terms of both sensitivity and specificity. In a study commissioned by the Agency for Healthcare
Research and Quality, Bruening et al. reviewed several studies
and found that core needle biopsy was as accurate as open surgical
biopsy and also associated with fewer serious complications and
a reduced number of surgical procedures.4 Given these and other
advantages of percutaneous needle biopsy over surgical biopsy
(improved cosmesis, decreased cost, improved patient satisfaction, better treatment planning), we strongly advocate the use of
needle biopsy, and specifically core needle biopsy when possible,
as the initial diagnostic procedure of choice for breast lesions.
Finally, in their analysis of the three controversial benign
lesions they chose to examine, Fou-Cockburn and Cockburn do
an excellent job of outlining the major considerations. For RS,
there is a risk of associated malignancy, which cannot be determined by imaging characteristics. However, finding no atypia on
at least 12 core needle biopsy specimens taken with 11-gauge or
larger needles was reliable criteria on which to base a surveillance
approach. Absent these, a diagnosis of RS should prompt surgical
excision to rule out an associated malignancy. With respect to the
optimal management of LN, which includes lobular carcinoma in
situ (LCIS) and atypical lobular hyperplasia (ALH), we agree with
the authors that there is not enough evidence to support or refute

PMPH_CH72.indd 586

CSE as the primary management strategy and that close surveillance with chemoprophylaxis may be a reasonable approach to
these benign lesions. Although, since there is no good evidence,
we believe it is best to err on the side of caution and perform CSE
unless contraindicated. This stance is supported by NCCN guidelines.5 Lastly, regarding whether all papillary lesions diagnosed on
core needle biopsy should be excised, the authors quote a 7-fold
increase in the risk of subsequent carcinoma. In addition, there is
a high rate of carcinoma or atypical ductal hyperplasia (ADH) at
the time of CSE (somewhere between about 15% and 35%). Therefore, the most conservative approach is to recommend CSE for
all these lesions until we are better able to distinguish high-risk
from low-risk lesions. From our review of the literature, we are
not there yet.

REFERENCES
1. Ksters JP, Gtzsche PC. Regular self-examination or clinical
examination for early detection of breast cancer. Cochrane Database of Systematic Reviews. 2003;(2):CD003373.
2. Baxter N; Canadian Task Force on Preventive Health Care. Preventive health care, 2001 update: Should women be routinely
taught breast self-examination to screen for breast cancer? Can
Med Assoc J. 2001;164(13):1837-1846.
3. Nelson HD, Tyne K, Naik A, Bougatsos C, Chan BK, Humphrey L;
U.S. Preventive Services Task Force. Screening for breast cancer:
An update for the U.S. Preventive Services Task Force. Ann Intern
Med. 2009;151(10):727-737, W237-W242.
4. Bruening W, Fontanarosa J, Tipton K, Treadwell JR, Launders J,
Schoelles K. Systematic review: Comparative effectiveness of coreneedle and open surgical biopsy to diagnose breast lesions. Ann
Intern Med. 2010;152(4):238-246.
5. NCCN Clinical Practice Guidelines in Oncology, Breast Cancer.
Version 2. 2011. Available at: http://www.nccn.org/professionals/
physician_gls/pdf/breast.pdf [Accessed April 15, 2011].

5/22/2012 5:50:37 PM

CHAPTER 73

Breast Cancer: Surgical Therapy


Alyssa Gillego, Manjeet Chadha, Beth Freedman, and Susan K. Boolbol

INTRODUCTION

The acceptance of less radical surgery coincided with advances in


breast irradiation and chemotherapy, and multimodality treatment for breast cancer emerged. Trials examining the various
operative techniques in combination with these adjuvant therapies
lay the foundation of modern-day breast cancer management.
Invasive carcinoma is pathologically defined as malignant
cells which have spread beyond the basement membrane of the
ductulobular unit of the breast. The two main histologic types are
invasive ductal carcinoma and invasive lobular carcinoma. Invasive ductal carcinoma is the more common type, and can present
as a radiographic lesion or a palpable mass. Frequently invasive
lobular carcinoma is difficult to detect clinically or radiographically, leading to an underestimation of size and extent of disease
in the breast. Invasive lobular carcinoma is more likely to be multifocal. Despite these differences, histologic type does not appear
to be a prognostic factor for overall clinical outcome. Mersin et al.9
performed a retrospective study of 510 women with stage I and II
breast cancer. Rates of hormone positive tumors and lymph node
metastasis were not statistically different between the two histologic subtypes. Five-year overall survival was 90% in patients with
invasive ductal carcinoma and 94% in patients with invasive lobular carcinoma (p = NS). Multivariate analysis showed histologic
subtype was not an independent risk factor for outcome.
All patients diagnosed with breast carcinoma should have
a complete history and comprehensive physical examination.
The history should pay special attention to associated risk factors for breast cancer, such as early menarche, nulliparity, age
of first delivery, late menopause, and use of unopposed estrogen
therapy. A thorough breast examination should be performed,
along with careful inspection for skin changes and palpation of
the regional lymph node basins in the axilla and supraclavicular
fossa. Blood tests should comprise of a complete blood cell count,
routine chemistry, liver function tests, and alkaline phosphatase
level. Complete investigation of other suspicious areas in the ipsilateral breast and the contralateral breast should be performed
to exclude synchronous disease. In addition, women diagnosed
with breast cancer who have a strong family history of breast cancer or ovarian cancer should be evaluated for a mutation of the

Breast cancer remains the most common cancer worldwide with


high rates of survival in developed countries. According to the
American Cancer Society there were 232,620 new cases of invasive
breast cancer in the United States in 2011.1 The surgical management of invasive breast cancer has evolved from William S. Halsteads first radical mastectomy in 1882 to todays skin-sparing
mastectomy (SSM) and breast-conserving therapy. Over the past
several decades, the surgical management of breast cancer has
undergone many advances. This is due to a multitude of national
and international randomized clinical trials that have been performed. This chapter addresses the surgical treatment of invasive
ductal carcinoma and invasive lobular carcinoma of the breast.
Historically, surgery involved excision of the entire breast,
underlying muscles, and axillary lymph nodes. Halstead2 published his series from the Johns Hopkins Hospital, in which he
reported a 3% local recurrence rate after 3-year follow-up. He was
a pioneer in both surgical education and technique, and radical mastectomy remained the standard operation for nearly 100
years. The surgery often resulted in disfigurement, disability,
and morbidity such lymphedema and chronic pain. In the 1940s,
Haagensen3 questioned the appropriateness of Halsteads radical
mastectomy. In 1948, Patey and Dyson4 reported their technique
of performing a mastectomy with preservation of the pectoralis
major muscle. In 1955, Crile, Jr.5 also challenged the established
practice and explored more conservative operations. Debate over
the appropriate surgical treatment of breast cancer led to a call
for scientific evidence. In 1977, Fisher et al.6 published the initial
results of the National Surgery Adjuvant Breast and Bowel Project
(NSABP) B-04 trial. The NSABP B-04 trial was a randomized trial
which compared radical mastectomy with total mastectomy with
or without radiation. This practice-changing trial showed there
was no survival difference between women who were randomized
to radical mastectomy and those who had a total mastectomy.7 Surgical clinical trials with large numbers of patients continued and
innovators in breast conserving operations emerged. Veronesi8 in
Milan promoted the safety of quadrantectomy and lumpectomy.
587

PMPH_CH73.indd 587

5/22/2012 5:51:25 PM

588

Surgery: Evidence-Based Practice

breast cancer susceptibility genes, BRCA 1 and BRCA 2. Referral


to a genetic specialist for counseling and testing should follow the
practice guidelines of the National Comprehensive Cancer Network (NCCN).10 Additional workup is indicated in symptomatic
patients with bone pain, abdominal pain, pulmonary complaints,
or neurologic findings. A bone scan should be obtained in any
patient with bone pain or an elevated alkaline phospatase level.
The sensitivity and specificity of alkaline phospatase in detecting bone metastasis is 80% and 98%, respectively.11 The sensitivity
and specificity of bone scan in detecting skeletal metastasis is 92%
and 96%, respectively. Computed tomography (CT) scan of the
chest, abdomen, and pelvis is necessary in patients with abdominal pain, pulmonary symptoms, or abnormal liver function tests.
Brain MRI is indicated in patients with neurologic symptoms.
In patients with clinical stage IIIA disease or higher, the NCCN
recommends considering preoperative bone scan, abdominal CT
scan, and chest imaging for staging purposes.
1. What is the appropriate management for early-stage (stages
I and II) invasive ductal carcinoma and invasive lobular carcinoma of the breast?
Answer: Most patients with early-stage breast cancer can be
treated with breast-conserving surgery (BCS) followed by wholebreast radiation therapy (RT). Numerous large trials have demonstrated that survival rates of mastectomy and BCS with adjuvant
RT is equivalent. The first important trial was the NSABP B-06,
which randomized patients with invasive breast cancer to receive
mastectomy or partial mastectomy plus radiation.12 The trial randomized 1851 women with tumors up to 4 cm in size. Twenty-year
follow-up demonstrated that women who underwent mastectomy
or partial mastectomy plus radiation had similar rates of survival. Local recurrence rate in the mastectomy arm was 10.2%,
and local recurrence rate in the partial mastectomy plus radiation
arm was 14.3%. Veronesi et al.13 from the Milan Cancer Institute
also demonstrated equivalent overall survival in women who had
mastectomy compared with women who had partial mastectomy
followed by whole-breast radiation. The Milan trial randomized
women with tumors up to 2 cm in size. Once again, a twenty-year
follow-up reported no difference in overall survival. The European
Organization for Research and Treatment of Cancer (EORTC)
10801 trial similarly reported equivalent survival rates between
the two groups.14 Equivalent long-term survival rates in women
who received mastectomy compared with women who underwent
BCS and RT was also demonstrated in a meta-analysis by the Early
Breast Cancer Trialists Collaborative Group (EBCTCG).15
The goal of BCS is removal of the cancer with negative margins while achieving an acceptable cosmetic result. BCS is removal
of the primary tumor along with a rim of normal breast tissue,
allowing preservation of the gland. BCS has been described in various terms such as lumpectomy, partial mastectomy, segmentectomy, quadrantectomy, and tumorectomy. Determining whether
a patient is a candidate for BCS involves thorough consideration
of patient factors and careful evaluation of extent of disease. Multifocal disease, defined as multiple foci of disease in one quadrant,
can be treated with BCS. Multicentric disease, defined as disease
in more than one quadrant of the breast, is a contraindication to
BCS and requires mastectomy. Relative contraindications to BCS
include prior RT to the breast or chest wall, pregnancy, diff use
malignant appearing calcifications, and inability to achieve negative margins, active connective tissue disease and tumors greater

PMPH_CH73.indd 588

than 5 cm in size. Large tumor to breast ratio is also a relative


contraindication to BCS because of poor cosmetic outcome from
resection of a large breast volume relative to breast size. BCS is
generally performed by making a skin incision above the tumor
to avoid tunneling. The specimen removed should be oriented to
allow for reexcision of close or positive surgical margins. At least
two margins of the breast tissue excised should be oriented, either
with sutures or clips, in order for the pathologist to identify specific margins. Larger breast defects may require a local advancement flap or an oncoplastic procedure for best cosmetic outcome,
therefore preoperative referral to a plastic and reconstructive surgeon is recommended in these cases.
The evaluation of surgical margins when performing BCS
has been reported in numerous studies. The NCCN describes
several requirements for optimal margin evaluation. These are
orientation of the specimen, description of the gross and microscopic margin status, and reporting of the distance to the closest
margin. Margin status is an important predictor of local control.
Lagios et al.16 reported a series of patients who underwent BCS
without radiation. Mean follow-up was only 24 months; however, local recurrence was 9% in women with negative margins
compared with 45% in women with positive margins. Schnitt
et al.17 examined margins status in women who underwent BCS
followed by whole-breast radiation. Five-year recurrence rate was
0% in women with margins >1 mm, 4% in women with margins
<1 mm, 6% in women with a focally positive margin, and 21% in
women with positive margins. The definition of a negative margin
differs among institutions and differs among the trials examining impact of margin status in breast cancer. Therefore, the greatest limitation of the literature examining surgical margins is that
a negative margin is defined differently in these studies. Most
surgeons attempt to achieve a negative margin of 1 to 3 mm of
normal breast tissue around the tumor, while some remove up to
1 cm. Mansfield et al.18 studied the impact of margin status on
local control. In their series of 1070 patients who underwent
lumpectomy and radiation, a positive margins status was a highly
significant risk factor for local recurrence in multivariate analysis.
Smitt et al.19 also reviewed the impact of margins on local control.
The retrospective study reviewed 289 women with 303 invasive
breast cancers. A positive margin was defined as tumor at the
inked surface, a close margin was defined as 2 mm, and a negative margin was >2 mm. The 10-year actuarial local control rate
was 98% in women who had negative margins and 82% in women
with close or positive margins (p = 0.007). Patient and tumor characteristics were analyzed, and final margin status was the most
significant determinant of local recurrence in univariate analysis.
In this series, patients who underwent reexcision for close or positive margins had a 10-year local control rate of 97%, and patients
who did not have a reexicision had a 10-year local control rate
of 84% (p = 0.0001). Therefore, when performing BCS, reexcision
should be performed for close or positive margins. Orientation of
the original lumpectomy specimen helps facilitate reexcision of
only the involved margin instead of excising the entire cavity. If
margins continue to remain positive, especially if multiple reexcision margins are involved, mastectomy may be required.
2. What is the approach to the axilla in patients with earlystage breast cancer?
Answer: The current NCCN recommendation for patients with
early-stage breast cancer is sentinel lymph node (SLN) biopsy and

5/22/2012 5:51:25 PM

Breast Cancer: Surgical Therapy

axillary lymph node dissection (ALND) in a patient with sentinel node metastasis.10 Surgical management of the axilla historically involved routine excision of all draining lymph nodes of
the breast. Morton20 first introduced the use of lymphatic mapping and SLN biopsy in the treatment of melanoma. Giuliano
et al.21,22 used methylene blue dye and applied Mortons technique
to the treatment of breast cancer. The identification of SLNs in the
axilla as the first site of lymphatic drainage of the breast redefined
breast surgery. Krag et al.23 described the use of radioisotope to
help localize the SLNs. The accuracy of identifying the SLNs of
the breast is enhanced by combining the injection of blue dye with
injection of technetium-99m (Tc99) sulfur colloid. Tafra et al.24
reviewed 529 patients who underwent 535 sentinel node biopsies
using peritumoral injection of Tc99 and isosulfan blue. The identification rate of finding a SLN in this series was 87% and the falsenegative rate was 13%.
In the United States, Guiliano et al.25 reported the safety of
omitting ALND in women with negative SLNs. In Italy, Veronesi
et al.26 further validated the safety and accuracy of performing
and SLN biopsy. Women with breast cancers 2 cm were randomized to either SLN biopsy with axillary dissection or SLN biopsy
with axillary dissection only if the node was positive for metastasis. The sensitivity of SLN biopsy in this study was 91.2% and the
specificity was 100%. Currently, the American Society of Clinical
Oncology recommends that a positive SLN should be routinely
followed by completion ALND.27
The largest prospective randomized trial examining the role
of SLN biopsy in breast cancer is the NSABP B-32 study.28 The
phase III trial randomized 5611 women with invasive breast cancer to sentinel node resection and axillary dissection or to sentinel node resection alone with completion dissection for positive
SLNs. Blue dye and radioactive tracer were used to identify the
SLNs. The 8-year overall survival was 91.8% in the group who
underwent sentinel node resection and axillary dissection compared with 90.3% in the group who had sentinel node resection
and dissection only in cases of positive SLN. The hazard ratio was
1.20 (95% confidence interval; p = 0.12). The study concluded that
SLN biopsy is safe and accurate, and no axillary dissection is necessary when the sentinel node is negative. Kim et al.29 published a
review of 69 observational studies, which included 8059 patients
who underwent SLN biopsy followed by ALND. The sentinel node
identification rate was 96%, and the false-negative rate was 7%.
The Axillary Lymphatic Mapping against Nodal Axillary Clearance (ALMANAC) trial had a sentinel node identification rate of
95% and a false-negative rate of 5%.30
The most common mapping agents used when performing
SLN biopsy are 1% lymphazurin (isosulfan blue) dye, methylene
blue dye, and Tc99 sulfur colloid. Blue dye and Tc99 sulfur colloid
are preferentially taken up by the lymphatic system. The American Society of Clinical Oncology (ASCO) recommends using both
blue dye and radioisotope to increase the rate of sentinel node identification. Typically a dose of 1 mCi of filtered technetium sulfur
colloid is injected into the breast prior to the surgery. Unfiltered
isotope should be used in patients who are injected the day prior
to surgery. Unfiltered radioisotope doses contain larger particles
and result in longer time interval for migration to the axilla. Various injection techniques and injection sites have been described
for both blue dye and the radioisotope. Successful lymphatic mapping with Tc99 sulfur colloid has been seen with intraparenchymal injection, intradermal injection, subareolar injection, and
peritumoral injection.31,32 Lymphoscintigraphy is not necessary

PMPH_CH73.indd 589

589

after injection of radioisotope. Similarly various successful injections sites for blue dye have also been described. The volume of
blue dye injected is 1 to 3 mL, and the breast is gently massaged for
5 minutes. Montgomery et al.33 examined 2392 patients who underwent SLN biopsy using isosulfan blue dye and reported a 1.6% rate
of allergic reactions. The incision for SLN biopsy is made at the
inferior aspect of the hair-bearing area of the axilla. This incision
can be readily extended for a completion axillary dissection if the
SLN is positive for metastasis. A hand-held gamma probe placed
within a sterile cover is used to locate the area with the highest
counts. SLNs are stained blue or have the highest counts per unit
(CPU) as indicated by the gamma probe. Following excision of
the node or nodes, ex vivo radioactive counts should be obtained
away from the surgical field to avoid counts from the injection
site of the breast. The CPU should be recorded. The gamma probe
should then be place in the axillary incision and positioned in
multiple directions to identify any additional areas of increased
uptake. Any lymph node with a count more than 10% of the node
with the highest CPU should be excised.34 The axilla should then
be palpated, and any abnormal or suspicious node should also be
excised. The sentinel node or nodes can be examined by touch
prep or intraoperative frozen section. If one cannot identify a
SLN, an axillary dissection should be performed.
Traditionally, the finding of a SLN with metastasis is followed
by a completion axillary dissection. The known morbidities of
axillary surgery, low rates of axillary recurrence, and impact on
overall survival have led clinicians to question the need for axillary dissection in women with positive SLNs. Many studies have
described the short- and long-term complications of both SLN
biopsy and ALND. Complications include paresthesia, chronic
pain, restricted range of motion, and lymphedema. Fleissig et al.35
reported arm swelling in 7% of patients who had an SLN biopsy
compared with 14% of patients who had axillary dissection 18
months after surgery (p = 0.002). Arm numbness was 19% in the
axillary dissection group and 8.7% in SLN biopsy alone group. Several studies have attempted to identify a subset of women who may
not benefit from axillary dissection despite having involved sentinel nodes. Bilimoria et al.36 retrospectively reviewed the National
Cancer Database and identified women with positive SLNs who
did not undergo completion axillary dissection. Although these
women tended to be older and had smaller tumors, there was no
difference in axillary recurrence compared with women who had
an axillary dissection.
Recently, Guiliano et al.37 published the results of the American College of Surgeons Oncology Group (ACOSOG) Z0011
study. They randomized 891 SLN positive patients who underwent
lumpectomy to completion, ALND, or no dissection. The primary
end point was overall survival. Patients were women with clinical
T1 to T2 N0 M0 invasive breast cancers. Lymph node metastasis was detected by frozen section, touch prep, or hematoxilyneosin stain on permanent section. Exclusion criteria included T3
tumors, clinically positive axillae, multicentric disease, and three
or more positive SLNs. All lumpectomy margins were negative and
all women received whole-breast radiation. In the ALND arm, a
level I and II lymph node dissection was performed, with removal
of at least 10 lymph nodes. Patient and tumor characteristics were
similar between 445 women randomized to axillary dissection
and 446 women with no dissection. Median number of SLNs was
2, and median number of lymph nodes removed in the dissection
group was 17. At a median follow-up of 6.3 years, 5-year overall
survival was 91.8% in women who had an ALND and 92.5% in

5/22/2012 5:51:25 PM

590

Surgery: Evidence-Based Practice

women who had no dissection. The hazard ratio was 0.87 when
adjusted for age and adjuvant therapy. There was no statistically
significant difference in locoregional recurrence. The locoregional
recurrence was 3.6% in the ALND arm, and the locoregional recurrence was 1.8% in the SLND alone arm. Adjuvant systemic therapy
was administered to 96% of the patients in the ALND arm and
to 97% of the patients in the SLND arm. All patients in the study
received whole-breast irradiation with standard tangential fields.
Therefore, it is likely that the axilla in many patients was treated in
both arms of the trial. The Z0011 trial concluded that SLND may
offer local control in early-stage breast cancer patients who undergo
breast conservation with adjuvant therapy, and that perhaps local
control is not improved by ALND.
3. What is the role of RT in the treatment of early-stage invasive breast cancer?
Answer: Multiple randomized trials have demonstrated that RT
following BCS decreases local recurrence and improves overall
survival. As previously mentioned, the NASBP B-06 was a large
phase III trial, which randomized 1851 women with stage I or II
invasive breast cancer into three treatment arms: (1) modified
radical mastectomy; (2) BCS with axillary dissection followed
by whole-breast radiation; and (3) BCS with axillary dissection
without radiation.12 Twenty-year follow-up was published in 2002,
which showed a statistically significant lower rate of recurrence
in the group of women who received BCS plus RT, compared
with BCS without RT. The incidence of ipsilateral breast recurrence in the group who had BCS plus RT was 14.3%, and the incidence of ipsilateral breast recurrence in the group who had BCS
without RT was 39.2% (p < 0.001). Among the three arms of the
study, there was no significant difference in disease-free survival,
distant-disease-free survival, and overall survival. The EBCTCG
performed a meta-analysis of 7300 women randomized to receive
either BCS or BCS plus RT.38 In women who had negative lymph
nodes, the 10-year local recurrence rate was 29% in the group who
had BCS, compared with a 10-year local recurrence of 10% in the
group who had BCS plus RT. In women who had positive lymph
nodes, 10-year local recurrence rate was 47% in the group who
had BCS, compared with a 10-year local recurrence rate of 13% in
the group who had BCS plus RT. In women with negative lymph
nodes who had BCS, the mortality was 31% compared with 26%
in those who had BCS plus RT. In women with positive lymph
nodes who had BCS, the mortality was 55% compared with 48%
in those who had BCS plus RT. The 15-year overall death rate was
41% in the group who had BCS, and the 15-year overall mortality
rate was 35% in the group who had BCS plus RT (p = 0.005). The
absolute survival advantage attributable to RT was 5.1% in nodenegative women. The absolute survival advantage attributable to
RT was 7.1% in node-positive women. Therefore, the decrease in
local recurrence associated with the addition of RT translated to
an absolute reduction in death.
Conventional radiotherapy is administered is to the whole
breast using electron beam. The most common complications are
fatigue and local skin changes such as dermatitis, erythema, and
hyperpigmentation. The standard dose is 45 to 50 Gy in 1.8 to
2.0 Gy daily fractions, given over 6 weeks. A boost dose of 10 to
16 Gy is delivered to the lumpectomy site, and radiation of regional
nodal basins is given when indicated. Patients who undergo BCS
with negative SLN biopsy typically do not need radiation to the
regional lymph nodes. Patients who are treated with BCS with

PMPH_CH73.indd 590

four or more positive axillary lymph nodes should receive radiation to regional lymph nodes. Whole-breast radiation following
BCS may also be administered to patients with high risk of locoregional recurrence. Radiation is administered after surgery, unless
a patient is recommended chemotherapy. Patients who receive
chemotherapy are scheduled for radiation several weeks after
the last dose. A randomized trial examining the optimal sequence
of chemotherapy and radiation compared patients who received
doxorubicin-based chemotherapy followed by RT with patients
who received RT followed by the same chemotherapy. There was
no difference in local recurrence or overall survival in the two
groups.39
Multiple trials have attempted to identify a subset of patients
undergoing BCS in whom radiation can be omitted. In these
studies, women who did not receive radiation had higher rates
of local recurrence, despite favorable disease characteristics such
as smaller size and positive hormone status. Lim et al.40 prospectively studied patients with tumors 2 cm or less in size. Patients
with extensive lymphovascular invasion or extensive intraductal
component were excluded from the study, and all margins were
1 cm or greater. Patients were randomized to BCS with no RT or
BCS alone. The trial was closed due to high recurrence rates in the
arm that did not receive RT. The local recurrence rate in this arm
was 23% after 86-month follow-up. The possibility of eliminating
the need for RT in a select group of patients with favorable tumor
characteristics was also examined in the NSABP B-21 trial.41 The
study randomized women with estrogen-positive tumors 1 cm
to receive tamoxifen alone, placebo, placebo plus RT, or tamoxifen
plus RT after BCS. After 8-year median follow-up, the arm which
received tamoxifen alone had a local recurrence rate of 16.5%. The
arm which received placebo plus RT had a local recurrence rate of
9.3%, and the arm which received tamoxifen plus RT had a local
recurrence rate of 2.8%. This study reinforced the role of combining BCS with RT to decrease local recurrence rate, despite the
addition of hormonal therapy. The elimination of RT in a cohort
of elderly patients with breast cancer was examined in the Cancer
and Leukemia Group B (CALGB) 9343 trial.42 The study enrolled
636 women over age 70 years with clinically negative axilla and
hormone-positive tumors 2 cm or less in size. After lumpectomy,
the women were randomized to receive tamoxifen with radiation
or tamoxifen alone. At 5 years, the group who received tamoxifen had a local recurrence rate of 4%, and the group who received
tamoxifen with radiation had a local recurrence rate of 1%
(p < 0.001). However, there was no difference in overall survival in
the two arms of the study. The breast cancer mortality rate was 1%
at 5 years, and the mortality rate from all other causes was 17%.
Careful consideration of comorbidities and risk of disease recurrence must be examined to provide the best treatment plan for
the elderly patient with breast cancer. The omission of radiation
in patients receiving breast conserving therapy is best managed
by a multidisciplinary team and reserved for carefully selected
patients.
4. What is the management of locally advanced breast cancer
(LABC)?
Answer: Patients who present with LABC should first undergo
additional imaging to determine stage and resectability. LABCs
include tumors greater than 5 cm (T3) or tumors which extend
to the chest wall (T4a) or skin (T4b). Disease is confined to the
breast and regional lymph nodes. Patients with LABC usually

5/22/2012 5:51:25 PM

Breast Cancer: Surgical Therapy

present with a large palpable mass, which can be associated with


overlying skin changes, nipple retraction, and lymphadenopathy.
Patients with large tumors or axillary involvement are at high risk
for systemic disease and local recurrence. The National Cancer
Data Base of the American College of Surgeons reported 8.7% of
patients with breast cancer in 2008 presented as stage III disease.43
Neoadjuvant chemotherapy should be considered in all patients
with LABC. Neoadjuvant chemotherapy can be used in patients
with large, fi xed tumors or in patients with bulky lymphadenopathy. In some cases, it can convert a patient with an unresectable
tumor to a patient with operable disease. Workup for metastatic
disease is appropriate in all patients with LABC. Bone scan and
CT scan of the chest, abdomen, and pelvis should be obtained.
Systemic metastasis is detected in up to 30% of patients with
LABC who undergo imaging for staging purposes.44
Work done earlier by Haagensen and Stout45 described their
experience with LABCs treated with radical mastectomy, and
described a 6% 5-year survival rate. Results of the NSABP B-04
trial demonstrate radical mastectomy is not indicated.46 The trial
randomized 1079 women. In the node-negative group, the hazard ratio for death among women treated with total mastectomy
and radiation compared with women treated with radical mastectomy was 1.08 (95% confidence interval, 0.911.28; p = 0.38).
In the node-positive group, the hazard ratio for death among
women treated with total mastectomy with radiation compared
with women treated with radical mastectomy was 1.06 (95% confidence interval, 0.871.23; p = 0.72). Twenty-five-year follow-up
comparing radical mastectomy, total mastectomy with postoperative radiation, and total mastectomy with axillary dissection
for nodal involvement did not show survival advantage of radical
mastectomy.
Most patients with LABC undergo mastectomy. Total or simple mastectomy involves complete removal of the gland including the nipple and areola. The underlying pectoralis muscles are
left intact. Mastectomy is removal of the entire breast to the anatomic borders of the gland. The superior border of the breast is
the clavicle, the medial border is the sternum, the lateral border
is the latissimus muscle, and the inferior border is the inframammary fold. The posterior border is the pectoralis major muscle and
the breast specimen is removed along with the pectoralis major
fascia. Mastectomy flaps can be raised using various techniques
including sharp dissection with a scalpel or scissors, or with electrocautery. The thickness of the mastectomy flaps should ensure
both removal of the entire gland and maintenance of a thin layer
of subcutaneous fat beneath the dermis. Meticulous dissection in
the plane between the gland and this layer of subcutaneous fat
improves the vascular supply of the mastectomy skin flaps, minimizing the risk of flap necrosis. Mastectomy can be performed
with or without reconstruction. In patients who undergo mastectomy with no reconstruction, an elliptical incision is made from
the lateral border of the sternum to the latissimus. The incision
should allow for adequate, tension-free closure of the two wound
edges, while avoiding redundant skin. Redundant skin should be
excised and the chest wall should be flat. Avoiding redundant skin
on the chest wall helps facilitate comfortable wear of a breast prosthesis postoperatively. To decrease risk of seroma formation, the
skin should be reapproximated over two closed suction drains.
One drain should be placed beneath the superior mastectomy
flap and the other placed beneath the inferior mastectomy flap. In
cases in which an axillary dissection is also performed, one drain
can be placed on the chest wall and a second placed in the axilla.

PMPH_CH73.indd 591

591

Drains are removed when the drainage volume is less that 30 mL


in 24 hours. The deep dermal layer is reapproximated with interrupted sutures and the skin is closed with continuous subcuticular suture. Possible complications following mastectomy include
surgical site infection, hematoma, seroma, and chronic pain. El
Tamer et al.47 reviewed the National Surgical Quality Improvement Program (NSQIP) data and reported an infection rate of
4.34% after mastectomy. In this analysis, the only significant
independent predictors of wound complications including surgical site infection were obesity (BMI > 30), preoperative albumin,
and hematocrit level.
5. What is the approach to the axilla in LABC?
Answer: Many patients with LABC are treated with neoadjuvant
chemotherapy, and performing SLN biopsy prior to or after neoadjuvant chemotherapy is under debate. There is concern that
SLN biopsy after chemotherapy is inaccurate due to sterilization
of axillary nodes. In addition, some clinicians prefer to know the
status of the axilla prior to chemotherapy when planning adjuvant
radiation. Sabel et al.48 reported a small series of 24 patients who
underwent SLN biopsy prior to neoadjuvant chemotherapy. The
SLN identification rate was 100%, and 10 out of 24 (42%) patients
had sentinel node metastasis. All 10 patients who had positive sentinel nodes underwent a completion axillary dissection at the time
of definitive surgical management after chemotherapy. Three out
of ten (30%) patients had metastasis in additional lymph nodes.
The NSABP B-27 studied the role neoadjuvant chemotherapy in
breast cancer management. In this trial, a subgroup of 428 women
underwent SLN biopsy after receiving chemotherapy. Mamounas
et al.49 reported an SLN identification rate of 85% and a falsenegative rate of 11%. The ACOSOG Z1071 trial is a phase II study
evaluating sentinel node biopsy in patients receiving preoperative
chemotherapy. This is a single-arm study examining patients with
known SLN involvement prior to the start of neoadjuvant chemotherapy. Patients enrolled in the trial will undergo sentinel node
biopsy and completion axillary dissection after chemotherapy.
The primary goal of the study is to determine the false-negative
rate of SLN biopsy performed after neoadjuvant chemotherapy.
Results of the study are pending. One of the advantages of doing
SLN biopsy after chemotherapy is that it allows for a single operative procedure, compared with performing sentinel node biopsy
before neoadjuvant chemotherapy and then performing the final
operative procedure after chemotherapy. Most importantly, SLN
biopsy after chemotherapy may also allow the group of women
with positive SLNs prior to chemotherapy whose axillae are sterilized by chemotherapy to avoid unnecessary axillary dissection
and the morbidity associated with dissection.
If an axillary dissection is to be performed, the NCCN recommends excision of level I and II lymph nodes. Lymph nodes in the
axilla are divided into three levels according to their relationship
to the pectoralis minor muscle. Level I nodes are located lateral
to the muscle, level II nodes are located beneath the muscle, and
level III nodes are located medial to the muscle. The lymph nodes
located between the pectoralis major and pectoralis minor muscles are known as Rotters nodes. Rotters nodes are not removed
when performing an axillary dissection, unless grossly involved.
When performing an axillary dissection, the clavipectoral fascia
is incised and the axilla is exposed. The pectoralis major and pectoralis minor muscles are retracted upward and the axillary vein
is identified. The apex of the axilla is the axillary vein. The axillary

5/22/2012 5:51:25 PM

592

Surgery: Evidence-Based Practice

content inferior to the vein is excised when performing an axillary dissection, paying close attention to small branches from the
axillary artery and vein. NCCN guidelines state a level III axillary
dissection can be considered in patients who have gross disease of
the level II nodes. The thoracodorsal and long thoracic vessels and
nerves should be identified and preserved.
6. What is the role of postmastectomy radiation in locally
advanced breast cancer?
Answer: Postmastectomy radiation therapy (PMRT) is administered to patients with locally advanced breast cancer to improve
locoregional control and survival following mastectomy. Several
studies have demonstrated that these women are at high risk for
local and regional recurrence.50,51 The Danish Breast Cooperative
Group (DBCG) 82b trial randomized 1708 premenopausal women
with Stage II and III breast cancer who underwent mastectomy
to receive chemotherapy or chemotherapy with radiation.52 The
trial demonstrated a local recurrence rate of 9% in patients who
received PMRT with chemotherapy, and a local recurrence rate
of 32% in patients who did not receive PMRT (p < 0.001). Overall
survival in the PMRT group was 45%, compared with 54% in the
group with no PMRT (p < 0.001). The DBCG 82c trial randomized
1375 postmenopausal women undergoing mastectomy to receive
tamoxifen alone or tamoxifen with radiation.53 The trial showed
a local recurrence rate of 8% in the patients who received PMRT
with tamoxifen, and a local recurrence rate of 35% in patients
who received tamoxifen alone (p < 0.001). Overall survival in the
PMRT group was 45%, compared with 38% in the group with no
PMRT (p < 0.03). In 2005, Ragaz et al.54 reported the results of
the British Columbia Trial which examined the role of PMRT in
premenopausal women who received chemotherapy. Median follow-up was 14.5 years, and PMRT was shown to decrease locoregional recurrence and improve disease-free survival and overall
survival. PMRT is routinely indicated in patients with tumors
>5 cm, metastasis to four or more lymph nodes, or positive surgical margins.55 Others factors that increase risk of recurrence such
as extensive lymphovascular invasion and extranodal extension of
disease are also considered when determining if a patient is a candidate for PMRT. PMRT can also be considered in women with
one to three involved axillary nodes. Overgaard et al.56 reported
a survival benefit when PMRT was administered to women with
one to three involved axillary nodes.
7. What is the role of neoadjuvant therapy in breast cancer?
Answer: In some cases, neoadjuvant chemotherapy can be used to
convert the operative management of disease from mastectomy to
BCS. Systemic chemotherapy or hormonal therapy prior to surgery
can decrease the size of the tumor, while also providing the clinician the ability to assess tumor response to therapy. The patient
should be assessed periodically during therapy, and surgery is
planned three to four weeks after the last dose of chemotherapy. To
assess response to therapy, repeat breast imaging prior to surgery
is necessary. This is especially useful in patients who will undergo
breast conserving therapy as the imaging can help guide surgical
planning. The current NCCN guidelines recommend performing SLN biopsy before beginning neoadjuvant chemotherapy. If
the SLN is negative, an axillary dissection at the time of mastectomy is not indicated. If the SLN is positive, a level I and II axillary
dissection is performed at the time of definitive surgery. Complete pathologic response to neoadjuvant chemotherapy has been

PMPH_CH73.indd 592

demonstrated with cases of no residual tumor found in the excised


specimen. Therefore, placement of a marking clip in the primary
tumor should be performed prior to initiation of therapy.
The NSABP B-18 trial examined the use of neoadjuvant chemotherapy and showed no difference in disease-free survival
and overall survival in patients who received preoperative versus
postoperative chemotherapy.57 In this trial, 1523 patients with
T1-T3N0M0 and T1-T3N1M0 disease were treated preoperatively
or postoperatively with four cycles of doxorubicin and cyclophosphamide (AC). All patients over age 50 years received tamoxifen.
The patients treated with preoperative chemotherapy had higher
rates of breast conserving therapy. The survival rates were similar
in the two groups. However, the women randomized to the neoadjuvant arm had more lumpectomies compared with the group
who had surgery prior to chemotherapy. The lumpectomy rate was
67% among those who received neoadjuvant therapy, compared
with 60% in those who had adjuvant chemotherapy (p = 0.002).
The clinical response rate in women who received neoadjuvant
chemotherapy was 80% and the complete clinical response rate
was 36%. The complete pathologic response rate was 13%. In the
NSABP B-18 trial, the survival rate of women whose tumor had
a complete pathologic response was 75% at 9 years, compared
with a survival rate of 58% in women who had residual disease.
The study showed no difference between the neoadjuvant and the
adjuvant chemotherapy arms in terms of disease-free survival. The
9-year disease-free survival rate in the neoadjuvant arm was 55%
compared with 53% in the adjuvant arm (p = NS). The study also
showed no difference in overall survival (OS). The overall survival
in the neoadjuvant arm was 69% and the OS in the adjuvant arm
was 70% (p = NS). Another NSABP trial which explored the benefit of neoadjuvant chemotherapy was the NSABP B-27 trial. The
B-27 trial was a randomized phase III study with three arms which
included 2411 women with invasive breast cancer.58 The women
were randomized to receive preoperative doxorubicin and cyclophosphamide (AC) for four cycles followed by surgery, preoperative AC followed by docetaxel for four cycles followed by surgery,
or AC followed by surgery and four cycles of postoperative docetaxel. The highest complete pathologic response rate was seen in
women who received preoperative AC followed by docetaxel prior
to surgery. Approximately 25% of patients in the neoadjuvant arms
were converted from mastectomy to BCS.59 Gianni et al. 60 also
examined the use of neoadjuvant chemotherapy and randomized
1335 patients with invasive breast cancer into three arms. The first
arm involved surgery followed by adjuvant doxorubicin, followed
by cyclophosphamide, methotrexate, and 5-FU (CMF). The second arm involved surgery followed by adjuvant doxorubicin and
paclitaxel (AT) followed by CMF. Lastly, the third arm was neoadjuvant AT followed by CMF, followed by surgery. The lumpectomy
rate in the neoadjuvant arm was 63% compared with 34% in the
adjuvant arms (p < 0.001). Overall survival at 5 years was similar
in the neoadjuvant and adjuvant arms (p = 0.81).
Use of neoadjuvant endocrine therapy for hormone positive
LABC is increasing. Three to four months of therapy is indicated
prior to surgery. Similar to neoadjuvant chemotherapy, periodic
clinical examination and imaging is necessary to assess response
to therapy. One of the initial reports describing the use of neoadjuvant endocrine therapy was by Preece et al.61 who prescribed
tamoxifen prior to surgery. Dixon et al.62 recommend administering an aromatase inhibitor (letrozole, anastrozole, or exemestane)
for 3 months prior to surgery. They report the ability to convert
inoperable tumors to resectable tumors, and performing BCS in

5/22/2012 5:51:26 PM

Breast Cancer: Surgical Therapy

patients initially thought to require mastectomy. The P024 trial


randomized 337 postmenopausal women with estrogen-positive
breast cancer to receive letrozole (2.5 mg daily) or tamoxifen
(20 mg daily).63 All women at the start of the trial were not candidates for BCS and 14% were considered inoperable. The group
who received letrozole had a 55% clinical response rate compared with a 36% clinical response rate in the tamoxifen group
(p < 0.001). The rate of conversion to BCS was 45% in the letrozole
arm and the rate of conversion to BCS was 35% in the tamoxifen arm (p = 0.022). The PROACT trial randomized 451 postmenopausal women to anastrozole for 3 months or tamoxifen for
3 months prior to surgery.64 The rate of BCS was 43% in the anastrolze group and 35.4% in the tamoxifen group.
8. What is the role of SSM and nipple- and areola-SSM in breast
cancer?
Answer: SSM is removal of the entire gland, while leaving the
majority of the skin as an envelope for breast reconstruction. If
immediate breast reconstruction is to be performed, the skin
allows coverage over an implant or autologous tissue flap. The primary concern is the oncologic safety of the procedure. Numerous
studies have established that rates of local recurrence in patients
who had a SSM are similar to those in patients who had a non-SSM.
A series from the MD Anderson Cancer Center with four-year follow-up described a 4.2% rate of recurrence in 95 patients who had
an SSM with immediate reconstruction.65 A series from Emory
with 6-year follow-up reported a local recurrence rate of 5.5% in
patients who had an SSM and immediate breast reconstruction.66
The use of nipple-SSM (NSM) and areolar-SSM is increasing.
Among the various incisions used in nipple-sparing and areolarSSM are a single lateral incision, a vertical incision inferior to the
areola, a periareolar incision with lateral extension, an inframammary incision, and a transareolar incision. Similar to the oncologic concerns surrounding SSM, there is debate as to the safety of
these procedures because of the concern of leaving breast or ductal tissue behind. When performing a NSM procedure, the nipple
should be inverted and a core of tissue should be sharply excised
and submitted for intraoperative frozen section analysis. If the
frozen section is positive for disease, the nippleareolar complex
(NAC) should be excised. Crowe et al.67 reviewed 149 NSMs performed on 110 patients. In this series, no cancer was detected in
the cored nipple tissue in patients who underwent prophylactic
mastectomy. In patients who underwent NSM for breast cancer,
11% were found to have involvement of the NAC. Chen et al.68
published a series of 115 consecutive patients at Memorial Sloan
Kettering Cancer Center undergoing NSM or areola-SSM with
immediate tissue expander placement. The nipple was involved
in 5.2% of patients, requiring excision. Rate of nipple loss due to
wound-healing problems was 3.5%. Petit et al.69 reported a series
of 579 NSMs performed at the European Institute of Oncology in
Milan. Intraoperative analysis of the NAC was performed in all
patients. Intraoperative radiotherapy using electron beam (total
dose 16 Gy) was administered to the NAC after mastectomy to
decrease risk of local recurrence. In this series, no local recurrences occurred in the area of the NAC and local relapse rate was
0.9% per year. A series of 99 NSMs with 5-year follow-up reported
by Jensen et al.70 from the John Wayne Cancer Center reported
three recurrences and no mortalities.
NSM and areola-SSM procedures are being performed more
frequently. Patients who undergo these procedures should be

PMPH_CH73.indd 593

593

counseled on the need to resect the NAC if the cored tissue has
disease. They should be informed of the possible complications of
the procedure, which include partial nipple necrosis and nipple
loss. Appropriate and careful patient selection remains the key
element when performing these procedures.
9. What is the approach to stage IV breast cancer?
Answer: Fewer than 10% of patients with breast cancer will present with stage IV disease. The 5-year overall survival is approximately 15%. The most common sites of breast cancer metastasis
are to the bones, lungs, liver, and central nervous system. Women
with bone metastasis and no visceral involvement have more
favorable prognosis and longer survival. Metastasis to the central
nervous system is more commonly associated with HER-2 positive
cancers.71 Biopsy of a presumed area of metastasis should be performed, and tissue diagnosis established. Hormonal therapy can
be used in patients with estrogen-positive tumors. Chemotherapy
is used in patients with estrogen-negative tumors, extensive visceral involvement, or rapidly growing tumors. Trastuzumab can
safely be administered to patients with HER-2 positive metastatic
breast cancer. Vogel et al.72 reported trastuzumab therapy is well
tolerated as first-line treatment and described a response rate of
26%. The goals of therapy of stage IV breast cancer are to improve
or stabilize disease, palliate symptoms, and minimize treatmentrelated toxicities.
Excision of the primary tumor in patients with metastatic
breast cancer has been examined in several series. Rapiti et al.73
reviewed 300 patients with metastatic breast cancer included in
the Geneva Cancer Registry from 1977 to 1996. Women who
had complete excision of the primary tumor with negative margins had improved survival compared with women who did not
have surgery. Risk of death was decreased by 40% in the group
who underwent surgery (95% confidence interval, HR 0.6). Khan
et al.74 examined 16,023 patients presenting with stage IV breast
cancer in the National Cancer Data Base between 1990 and 1993.
They reported 42.8% of patients did not have surgery, and 57.2% of
patients had either a partial or total mastectomy. Negative surgical
margins were associated with improved 3-year survival. Women
who had resection of their primary tumor with negative margins
had improved survival compared with women who did not have
resection (HR 0.61). The Eastern Cooperative Oncology Group
(ECOG) 2108 study is an ongoing phase III trial evaluating the
value of early local therapy in patients with metastatic breast cancer. Patients enrolled in the study will receive systemic therapy,
and those who demonstrate response to therapy are randomized
to either continued systemic therapy or surgery. Patients randomized to the surgery arm will either undergo BCS or mastectomy,
according to patient or physician preference. The trial opened in
2011 and the results will help guide future management of patients
with stage IV disease.

CONCLUSION
There has been a dramatic shift in the surgical management of breast
cancer from radical surgery to BCS. Multiple large prospective randomized trials have established the current treatment modalities
as safe and effective. The treatment of invasive breast carcinoma
requires shared-decision making with the patient, as well as a multidisciplinary approach to achieve the best clinical outcome.

5/22/2012 5:51:26 PM

594

Surgery: Evidence-Based Practice

Clinical Question Summary


Question

Answer

Levels of
Evidence

1 What is the appropriate


management for early-stage
(stages I and II) invasive
ductal carcinoma and
invasive lobular carcinoma
of the breast?

The majority of patients can be treated


with BCS. BCS followed by wholebreast irradiation when compared with
total mastectomy has been shown to
have equivalent overall survival.

Ia
Ib

A
A

15
8, 14-16

2 What is the approach to the


axilla in early-stage breast
cancer?

Current guidelines recommend


performing SLN biopsy, followed
by completion axillary dissection in
patients with positive SLNs.
The results of the ACOSOG Z11 trial
indicate ALND may be omitted in
early-stage breast cancer patients who
undergo BCS plus RT.

Ib
IIa
IIb
IB

A
B
B
A

30, 32
31
27, 28
39

3 What is the role of RT in the


treatment of early-stage
invasive breast cancer?

RT decreases local recurrence and


improves overall survival.

Ia
Ib

A
A

15, 40
12

4 What is the management


of locally advanced breast
cancer (LABC)?

Modified radical mastectomy in patients


with resectable disease.
Consider neoadjuvant chemotherapy in all
patients with LABC.

Ib
See question 7

48

5 What is the approach to the


axilla in LABC?

ALND.

6 What is the role of


postmastectomy radiation
(PMRT) in breast cancer?

PMRT improves locoregional control and


improves disease free survival and
overall survival.

Ib

54-56

7 What is the role of


neoadjuvant therapy in
breast cancer?

Neoadjuvant chemotherapy can


potentially convert a patient for
mastectomy to BCS.

Ib

58-60

8 What is the role of SSM and


nipple- and areola-SSM in
breast cancer?

Several patient series report safety and


low rates of local recurrence.

IV

67-72

9 What is the approach to


stage IV breast cancer?

Patients with metastasis receive palliative


therapy.
Some series report improved survival with
resection of the primary tumor.

IIa
IIb

B
B

74
73

REFERENCES
1.
2.

3.

4.

5.

American Cancer Society. Cancer Facts and Figures 2011. Atlanta:


American Cancer Society; 2011.
Halstead WS. The results of operations for the cure of cancer of
the breast performed at the Johns Hopkins Hospital from June,
1889, to January, 1894. Ann Surg. 1894;20(5):497-555.
Haagensen CD. In: Harris JR, Lippman ME, Morrow M, et al.,
eds. Diseases of the Breast. 2nd ed. Philadelphia: WB Saunders
Co.; 1971:394-395.
Patey DH, Dyson WH. The prognosis of carcinoma of the breast
in relation to the type of operation performed. Br J Cancer.
1948;2:7-13.
Crile G Jr. A Biological Consideration of Treatment of Breast
Cancer. Springfield, IL: Charles C. Thomas; 1967.

PMPH_CH73.indd 594

6.

7.

8.

9.

Grade of
References
Recommendation

Fisher B, Montague E, Redmond C, et al. Comparison of radical mastectomy with alternative treatments for primary breast
cancer. A first report of results from a prospective randomized
clinical trial. Cancer. 1977;39:2827-2839.
Fisher B, Redmond C, Fisher ER, et al. Ten-year results of a
randomized clinical trial comparing radical mastectomy and
total mastectomy with or without radiation. N Engl J Med.
1985;312:674-681.
Veronesi U, Cascinelli N, Mariani L, et al. Twenty-year follow
up of a randomized study comparing breast-conserving surgery
with radical mastectomy for early breast cancer. N Engl J Med.
2002;347:1227-1232.
Mersin H, Yildirim E, Gulben K, Berberoglu U. Is invasive lobular carcinoma different from invasive ductal carcinoma? Eur J
Surg Oncol. 2003;29:390-395.

5/22/2012 5:51:26 PM

Breast Cancer: Surgical Therapy

10. NCCN: National Comprehensive Cancer Network. Clinical


Guidelines in Oncology: Breast Cancer. http://www.nccn.org/
professionals/physician_gls/PDF/breast.pdf. Version 2.2011,2011.
11. Brar HS, Sisley JF, Johnson RH Jr. Value of preoperative bone
and liver scans and alkaline phosphatase in the evaluation of
breast cancer patients. Am J Surg. 1993;165:221-224.
12. Fisher B, Anderson S, Bryant J, et al. Twenty-year follow up of a
randomized trial comparing total mastectomy, lumpectomy and
lumpectomy plus irradiation for the treatment of invasive breast
cancer. N Engl J Med. 2002;347:1233-1241.
13. Veronesi U, Marubini E, Mariani L, et al. Radiotherapy after
breast conserving-surgery in small breast carcinoma: long-term
results of a randomized trial. Ann Oncol. 2001;12:997-1003.
14. Van Dongen JA, Voogd AC, Fentiman IS, et al. Long-term results
of a randomized trial comparing breast conserving-surgery
with mastectomy. European Organization for Research and
Treatment of Cancer 10801 trial. J Natl Cancer Inst. 2000;92:
1143-1150.
15. Early Breast Cancer Trialists Collaborative Group. Effects of
radiotherapy and surgery in early breast cancer. An overview of
the randomized trials. N Engl J Med. 1995;333:1444-1455.
16. Lagios MD, Richards VE, Rose MR, Yee E. Segmental mastectomy without radiotherapy. Short-term follow-up. Cancer.
1983;52(11):2173-2179.
17. Schnitt SJ, Abner A, Gelman R, et al. The relationship between
microscopic margins of resection and the risk of local recurrence
in patients with breast cancer treated with breast-conserving
surgery and radiation therapy. Cancer. 1994;74:1746-1751.
18. Mansfield CM, Komarnicky LT, Schwatrz GF, et al. Tenyear results in 1,070 patients with stage I and II breast cancer
treated by conservative surgery and radiation therapy. Cancer.
1995;75:2328-2336.
19. Smitt MC, Nowels KW, Zdeblick MJ, et al. The importance of
lumpectomy surgical margin status in long-term results of breast
conservation. Cancer. 1995;76:259-267.
20. Morton DL, Wen DR, Wong JH, et al. Technical details of intraoperative lymphatic mapping for early stage melanoma. Arch
Surg. 1992;127:392-399.
21. Giuliano AE, Kirgan DM, Guenther JM, Morton DL. Lymphatic
mapping and sentinel lymphadenectomy in breast cancer. Ann
Surg. 1994;220:391-398.
22. Guiliano AE, Jones RC, Brennan M, et al. Surgical lympadenectomy in breast cancer. J Clin Oncol. 1997;15:2345.
23. Krag D, Weaver D, Ashikaga T, et al. The sentinel node in
breast cancer a multicenter validation study. N Engl J Med.
1998;339:941-946.
24. Tafra L, Lannin D, Swanson M. Multicenter trial of sentinel node
biopsy for breast cancer using both technetium sulfur colloid
and isosulfan blue dye. Ann Surg. 2001;233:51-59.
25. Giuliano AE, Haigh PI, Brennan MB, et al. Prospective observational study of sentinel lymphadenectomy without further
axillary dissection in patients with sentinel node-negative breast
cancer. J Clin Oncol. 2000;18:2553-2559.
26. Veronesi U, Paganelli G, Viale G, et al. A randomized comparison of sentinel-node biopsy with routine axillary dissection in
breast cancer. N Engl J Med. 2003;349:546-53.
27. Lyman GH, Giuliano AE, Somerfield MR, et al. American Society of Clinical Oncology guideline recommendations for sentinel lymph node biopsy in early-stage breast cancer. J Clin Oncol.
2005;23:7703-7720.
28. Krag, D. Anderson S, Julian T, et al. Sentinel-lymph-node resection compared with conventional axillary-lymph-node dissection in clinically node-negative patients with breast cancer:

PMPH_CH73.indd 595

29.

30.

31.

32.

33.

34.

35.

36.

37.

38.

39.

40.

41.

42.

43.

44.

45.

595

overall survival findings from the NSABP B-32 randomised


phase 3 trial. Lancet Oncol. 2010;11(10):927-933.
Kim T, Guiliano AE, Lyman GH. Lymphatic mapping and sentinel lymph node biopsy is early-stage breast carcinoma. Cancer.
2006;106:4-16.
Mansel RE, Fallowfield L, Kissin M, et al. Randomized multicenter trial of sentinel lymph node biopsy versus standard axillary treatment in operable breast cancer: The ALMANAC trial.
J Natl Cancer Inst. 2006;98:599-609.
Johnson CB, Boneti C, Korourian S. Intraoperative injection of
subareolar or dermal radioisotope results in predictable identification of sentinel lymph nodes in breast cancer. Ann Surg.
2011;254(4):612-618.
Cox CE, Pendas, Cox JM, et al. Guidelines for sentinel lymph
node biopsy and lymphatic mapping of patients with breast cancer. Ann Surg. 1998;227(5):645-651.
Montgomery LL, Thorne AC, Van Zee KJ, et al. Isosulfan blue
reactions during sentinel lymph node mapping for breast cancer.
Anesth Analg. 2002;95(2):385-388.
Martin RC 2nd, Edwards MJ, Wong SL, et al. Practical guidelines for optimal gamma probe detection of sentinel lymph
nodes in breast: Results of a multi-institutional study. Surgery.
2000;128;(2)139-144.
Fleissig A, Fallowfield LJ, Langridge CI, et al. Post-operative arm
morbidity and quality of life. Results of the ALMANAC randomized trial comparing sentinel node biopsy with standard axillary
treatment in the management of patients with early breast cancer. Breast Cancer Res Treat. 2006;95:279293.
Bilimoria KY, Bentrem DJ, Hansen NM, et al. Comparison of
sentinel lymph node biopsy alone and completion axillary lymph
node dissection for node-positive breast cancer. J Clin Oncol.
2009;27:2946-2953.
Guiliano AR, McCall L, Beitsch P, et al. Locoregional recurrence
after sentinel lymph node dissection with or without axillary
dissection in patients with sentinel lymph node metastases. Ann
Surg. 2010;252:426-433.
Early Breast Cancer Trialists Collaborative Group. Effects of
radiotherapy and of differences in the extent of surgery for early
breast cancer on local recurrence and 15-year survival: an overview of the randomized trials. Lancet. 2005;366:2087-2106.
Bellon JR, Come SE, Gelman RS, et al. Sequencing of chemotherapy and radiation therapy in early stage breast cancer:
updated results of a prospective randomized trial. J Clin Oncol.
2005;23:1934-1940.
Lim M, Bellon JR, Gelman R, et al. A prospective study of conservative surgery without radiation therapy in select patients
with stage I breast cancer. Int J Radiat Oncol Biol Phys. 2006;65:
1149-1154.
Fisher B, Bryant J, Dignam JL, et al. Tamoxifen, radiation therapy, or both for prevention of ipsilateral breast tumor recurrence
after lumpectomy in women with invasive breast cancers of one
centimeter or less. N Eng J Med. 2002:20:4141-4149.
Hughes KS, Schnaper LA, Berry D, et al. Lumpectomy plus tamoxifen with or without irradiation in women 70 years of age or older
with early breast cancer. N Engl J Med. 2004;351:971-977.
National Cancer Database, 2008. Available at: https://cromwell.
facs.org/BMarks/BMCmp/ver10/Docs/#sxs_2008. Accessed January 14, 2012.
Samant R, Ganguly P. Staging investigation in patients with
breast cancer: the role of bone scan and liver imaging. Arch Surg.
1999;134:551-553.
Haagensen C, Stout A. Carcinoma of the breast II. Criteria of
operability. Ann Surg. 1943;118:859.

5/22/2012 5:51:26 PM

596

Surgery: Evidence-Based Practice

46. Fisher B, Jeong JH, Anderson S. Twenty-five year follow-up of a


randomized trial comparing radical mastectomy, total mastectomy, and total mastectomy followed by irradiation. N Engl J
Med. 2002;347(8):567-575.
47. El-Tamer MB, Ward BM, Schifft ner T, et al. Morbidity and mortality following breast cancer surgery in women: national benchmarks for standard of care. Ann Surg. 2007:245(5):665-671.
48. Sabel MS, Schott AF, Kleer CG, et al. Sentinel node biopsy prior
to neoadjuvant chemotherapy. Am J Surg. 2003;186:102-105.
49. Mamounas EP, Brown A, Anderson S, et al. Sentinel node biopsy
after neoadjuvant chemotherapy in breast cancer: results from
National Surgical Adjuvant Breast and Bowel Project Protocol
B-27. J Clin Oncol. 2005;23:2694-2702.
50. Taghian A, Jeong JH, Mamounas E, et al. Patterns of locoregional
failure in patients with operable breast cancer treated with mastectomy and adjuvant chemotherapy with or without tamoxifen
and without radiotherapy: results from five National Surgical
Adjuvant Breast and Bowel Project randomized clinical trials. J
Clin Oncol. 2004;22:4247-4254.
51. Nielsen HM, Overgaard M, Grau C, et al. Study of failure pattern among high-risk breast cancer patients with or without
postmastectomy radiotherapy in addition to adjuvant systemic
therapy: long term results from the Danish Breast cancer Cooperative Group DBCG 82b and c randomized studies. J Clin Oncol.
2006;24:2268-2275.
52. Overgaard M, Hansen PS, Overgaard J, et al. Postoperative radiotherapy in high risk premenopausal women with breast cancer
who receive adjuvant chemotherapy. Danish Breast cancer Cooperative Group DBCG 82b Trial. N Engl J Med. 1997:337;949-955.
53. Overgaard M, Jensen MB, Overgaard J, et al. Postoperative radiotherapy in high risk postmenopausal breast cancer patients given
adjuvant tamoxifen: Danish Breast cancer Cooperative Group
DBCG 82c Trial. Lancet. 1999;353:1641-1648.
54. Ragaz J, Olivotto, Spinelli J, et al. Locoregional radiation therapy
in patients with high-risk breast cancer receiving adjuvant chemotherapy: 20 year results of the British Columbia randomized
trial. J Natl Cancer Inst. 2005:97:116-126.
55. Recht A, Edge SB, Solin LJ, et al. Postmastectomy radiotherapy:
guidelines of the American Society of Clinical Oncology. J Clin
Oncol. 2001;19:1539-1569.
56. Overgaard M, Nielsen HM, Overgaard J. Is the benefit of postmastectomy irradiation limited to patient with four or more
positive nodes, as recommended in international consensus
reports? A subgroup analysis of the DBCG 82 b&c randomized
trials. Radiother Oncol. 2007;82:247-253.
57. Fisher B, Bryant J, Wolmark N, et al. Effect of preoperative chemotherapy on the outcome of women with operable breast cancer. J Clin Oncol. 1998;16:2672-2685.
58. Bear HD, Anderson S, Smith RE, et al. Sequential preoperative
or postoperative chemotherapy docetaxel added to preoperative
doxorubicin plus cyclophosphamide for operable breast cancer:
National Surgical Adjuvant Breast and Bowel Project Protocol
B-27. J Clin Oncol. 2006;24:2019-2027.
59. Bear HD, Anderson S, Brown A, et al. The effect of tumor response
of adding sequential preoperative docetaxel to preoperative

PMPH_CH73.indd 596

60.

61.

62.

63.

64.

65.

66.

67.

68.

69.

70.

71.

72.

73.

74.

doxorubicin and cyclophosphamide: preliminary results from


National Surgical Adjuvant Breast and Bowel Project Protocol
B27. J Clin Oncol. 2003;21:4165-4174.
Gianni L, Baselga J, Eiermann W, et al. Phase III trial evaluating
the addition of paclitaxel to doxorubicin followed by cyclophosphamide, methotrexate, and fluorouracil, as adjuvant or primary
systemic therapy: European Cooperative Trial in Operable Breast
Cancer. J Clin Oncol. 2009;27:2474-2481.
Preece PE, Wood RA, Mackie CR, et al. Tamoxifen as initial sole
treatment of localized breast cancer in elderly women: a pilot
study. Br Med J. 1982;284:869-870.
Dixon JM, Anderson TJ, Miller WR. Neoadjuvant endocrine
therapy of breast cancer: A surgical perspective. Eur J Cancer.
2002;38:2214-2221.
Eiermann W, Paepke S, Appfelstaedt J, et al. Preoperative treatment of postmenopausal breast cancer patients with letrozole:
A randomized double-blind multicenter study. Ann Oncol.
2001;12:1527-1532.
Cataliotti L, Buzdar AU, Noguchi S, et al. Comparison of anastrozole versus tamoxifen as preoperative therapy in postmenopausal women with hormone receptor-positive breast cancer: the
Pre-Operative Arimidex Compared to Tamoxifen (PROACT)
trial. Cancer. 2006;106:2095-2103.
Singletary SE. Skin-sparing mastectomy with immediate breast
reconstruction: the M. D. Anderson Cancer Center experience.
Ann Surg Onco. 1996;3:411-416.
Carlson GW, Styblo TM, Lyles RH, et al. The use of skin-sparing
mastectomy in the treatment of breast cancer: the Emory experience. Surg Oncol. 2003;12:265-269.
Crowe JP, Patrick RJ, Yetman RJ, et al. Nipple-sparing mastectomy: one hundred forty-nine procedures and clinical outcomes.
Arch Surg. 2008;143(11):1106-1110.
Chen CM, Dissa JJ, Sacchini V, et al. Nipple-sparing mastectomy
and immediate tissue expander/implant breast reconstruction.
Plast Reconstr Surg. 2009;124(6):1772-1780.
Petit JY, Veronesi U, Rey P. Nipple-sparing mastectomy: risk of
nipple-areolar recurrences in a series of 579 cases. Breast Cancer
Res Treat. 2009;114:97-101.
Jensen JA, Orringer JS, Guiliano AE. Nipple-sparing mastectomy in 99 patients with mean follow-up of 5 years. Ann Surg
Oncol. 2011;18(6):1665-1670.
Altundag K, Bondy JR, Mirza N, et al. Clinicopathologic characteristics and prognostic factors in 420 metastatic breast cancer patients with central nervous system metastasis. Cancer.
2007;110:2640-2647.
Vogel CL, Cobleigh MA, Tripathy D, et al. Efficacy and safety
of trastuzumab as a single agent in first-line treatment of
HER2-overexpressing metastatic breast cancer. J Clin Oncol.
2002;20(3);719-726.
Rapiti E, Verkooijen HM, Vlastos G, et al. Complete excision of
primary breast tumor improves survival of patients with metastatic breast cancer at diagnosis. J Clin Oncol. 2006;24:2743-2749.
Khan SA, Stewart AK, Morrow M. Does aggressive local therapy improve survival in metastatic breast cancer? Surgery.
2002;132(4):620-626.

5/22/2012 5:51:26 PM

CHAPTER 74

Breast Cancer: Lymphatic Mapping


and Sentinel Lymph Node Biopsy
Abigail S. Caudle, Elizabeth A. Mittendorf, and Henry M. Kuerer

INTRODUCTION

in clinically node-negative breast cancer patients. Although


its original use was limited to patients presenting with unifocal, small breast cancers, its application has since broadened to
include patients with larger tumors or even those with muticentric or multifocal disease. Additionally, the technique has
evolved from one offered only in highly specialized cancer institutions, to one that is taught to all general surgery residents and
is available in most communities. Thus, all clinicians caring for
breast cancer patients should have an understanding of the principles of SLN biopsy and when it is indicated in the management
of breast cancer patients as well as its benefits and limitations.

Great strides have been made toward the goal of personalizing


therapy for breast cancer patients. While this concept is often
mentioned when discussing the evolution of one size fits all
cytotoxic chemotherapy to the current standard of tailoring
agents with targeted therapies and endocrine approaches based
on tumor biology and patient characteristics, the same is true
regarding surgical management of the disease. The acceptance of
breast conservation therapy in selected patients is one example of
personalized surgical therapy. Another major milestone in this
effort was the acceptance of sentinel lymph node (SLN) biopsy
for nodal assessment in patients with clinically node-negative
breast cancer. The SLN technique provides accurate staging
while limiting morbidity associated with a complete axillary
lymph node dissection (ALND). Although the concept of SLN
biopsy is straightforward and validated, there remain questions
about its application and interpretation of results. Th is chapter
addresses the clinical questions still debated among surgeons
regarding SLN biopsy in breast cancer and the evidence available to answer those questions.
SLN surgery is based on the concept that the breast has an
orderly pattern of lymphatic drainage with sentinel nodes
being the fi rst nodes to drain the breast followed by the rest
of the nodal basin. The concept was fi rst conceived 100 years
ago by Braithwaite after noticing the lymphatic drainage of
gangrenous appendicitis. After much investigation, the fi rst
clinical application was reported in the 1970s for penile cancer; however, the initial technique was cumbersome which
limited its widespread use. SLN biopsy as we know it today
became clinically relevant in 1991 when Morton et al.1 refi ned
the technique for melanoma patients. Th is prompted considerable interest from the breast cancer community as ALND with
its inherent morbidities such as lymphedema, pain, and functional limitations was the standard of care at that time. Once
technical aspects were established and results were validated,
SLN biopsy became the standard technique for nodal evaluation

SLN TECHNIQUE
1. What is the best method for identifying an SLN?
There are basic tenets to SLN biopsy; however, there are many
variations in the materials and techniques used by clinicians. The
guiding principle is that an agent such as dye, radioisotope, or
combination of the two is injected into the breast and allowed to
drain into the nodal basins. At the time of operation, any node
that is blue, hot (i.e., has increased radioisotope counts by handheld Geiger probe), or is suspiciously palpable is classified as an
SLN and should be removed for pathologic evaluation. A 10%
rule is applied to determine hot nodesall nodes with counts at
least 10% of the hottest node should be removed. If the SLN is
negative for disease, then the remaining lymph nodes can be left
in place. If metastasis is found in the SLN, the standard approach
has been to perform a completion ALND. However, as will be discussed below, recent studies have shown that this may not always
be necessary in a select group of patients.
In the United States, technetium-99m sulfur colloid is the
radioisotope usually employed if a radioisotope is used. It
can be injected at low doses (0.30.5 mCi) perioperatively or
at higher doses (12.5 mCi) up to 24 h before surgery. If the
radioisotope is injected prior to arrival to the operating room,

597

PMPH_CH74.indd 597

5/22/2012 5:52:01 PM

598

Surgery: Evidence-Based Practice

lymphoscintigraphy can be performed to confirm drainage and


indicate the site of drainage. However, preoperative injection
can be painful and requires scheduling with nuclear medicine.
Blue dyes used as mapping agents include isosulfan blue, methylene blue, and vital blue dye. Although the dye is easily accessible and easy to inject in the operating room, drawbacks include
skin tattooing, unclear safety profi le in pregnancy, and the risk
of anaphylactic reaction. Although early studies showed higher
identification rates when blue dye was used with radioisotope, 2
more recent studies reflecting the general learning curve of this
procedure show no difference in success rates with the use of
one agent alone versus a combination when performed by experienced surgeons.3,4
Similarly, the optimal site of injection has not been determined. The original reports relied on peritumoral injections;
however, this can be difficult in nonpalpable tumors. In one
early study of five patients with muticentric breast cancers who
had blue dye injections in one tumor and radioisotope injection
of another, SLNs were identified which were both blue and hot
suggesting that the entire breast drains into the same SLNs,
rather than each specific area of the breast having different
drainage patterns. Th is allowed for the possibility of alternative
injection sites such as subdermal or subareolar. 5 The data supporting subdermal injection includes one study of 200 patients
who had peritumoral blue dye injections who then were randomized to receive either peritumoral or intradermal radioisotope injection. Overall SLN identification rates were excellent in
both groups with concordance between blue and hot nodes in
both groups. The intradermal injection patients had a slightly
higher identification rate (100% vs. 92%) possibly related to the
more rapid lymphatic drainage.6 Similar studies have evaluated
subareolar injections, including a prospective, multicenter study
from France that randomized 449 patients to receive either peritumoral injection of blue dye and radioisotope or subareolar
injection of the combination. The rate of SLN identification was
similar between the two groups, and blue/hot concordance
was 95.6% in the subareolar group versus 91.5% in the peritumoral cohort.7 Although subdermal and subareolar injection
sites are accepted as valid, extra-axillary drainage sites such as
internal mammary nodes are rarely seen when these routes are
employed which has led some surgeons to advocate peritumoral
injections.
The number of SLNs removed can varyin one retrospective review of 777 patients, the median number of SLNs removed
was 2.9 (range 113 nodes). The authors found that >99% of the
positive SLNs were identified within the fi rst five SLNs removed.8
Importantly, studies show that the morbidity of SLN biopsy does
not increase with an increasing number of nodes removed.9
When an SLN cannot be identified, the recommendation has
been for complete ALND. Patients should be counseled preoperatively about the risk of nonidentification; so their desires as
to whether or not to proceed to full ALND can be considered in
this circumstance.
In conclusion, the best technique for SLN biopsy should
be based on the surgeons experience, the patients characteristics, and institutional resources. Radioisotope or blue dye or
the combination can be injected in peritumoral, subdermal,
or subareolar locations. The surgeon should then explore the
axilla and remove any nodes that are blue, hot, or clinically
suspicious.

PMPH_CH74.indd 598

ACCURACY OF SLN BIOPSY


2. Is SLN biopsy as accurate as ALND for breast cancer?
Although the concept of a minimally invasive technique is attractive, the primary goal of clinicians caring for breast cancer patients
is optimization of diagnostic and therapeutic modalities. Therefore, less invasive techniques such as SLN biopsy must be tested
rigorously against standard therapies to ensure equivalency for
both accuracy and outcomes. The SLN concept has undergone
such an evaluation and has been shown to be even more sensitive than ALND in identifying lymph nodes harboring metastatic
disease, particularly small volume metastases, because evaluating
fewer lymph nodes allows for more rigorous pathologic evaluation
of each node.
Th is question was fi rst addressed in a single-institution
study published in by Giuliano et al. in 1995.10 In this study,
they compared 134 patients who underwent ALND versus 162
who underwent SLN biopsy followed by ALND. Nodal metastasis was identified in 29.1% of the ALND group versus 42% in the
group undergoing SLN biopsy followed by ALND (p < .03). Of
the patients with metastasis, only 10% of the ALND group were
micrometastasis (tumor focus of 2 mm) compared with 16% in
the SLN biopsy group (p < .0005) suggesting that SLN biopsy may
actually be more accurate than ALND in staging the axilla. This
was validated in a broader population in 1998 with a multicenter
trial that included 11 sites.11 This study enrolled 443 patients who
underwent SLN biopsy followed by full ALND to assess the accuracy of the SLN in determining the presence of SLN metastasis.
They found SLN biopsy to have an accuracy rate of 97% and a specificity of 100%. A single-institution randomized study from Italy
confirmed these results in 516 patients who were also randomized
to SLN biopsy followed by ALND versus SLN biopsy with ALND
only if metastasis was found. They reported an accuracy rate of
96.9% for SLN biopsy, and importantly had no cases of axillary
recurrence in the SLN biopsy only group after a median follow-up
of 46 months.12 The results of these large, randomized trials led
to the 2005 American Society of Clinical Oncology Guidelines
recommending SLN biopsy for nodal evaluation in patients with
clinically node-negative, early-stage breast cancer.13
Building on these data, the National Surgical Adjuvant Breast
and Bowel Project (NSABP) B-32 trial was designed to determine
whether SLN biopsy achieves the same cancer outcomes as ALND
with decreased morbidity. This was a randomized, controlled,
phase 3 trial performed at 80 centers in the United States and
Canada with end points that included survival, regional control,
and morbidity. A total of 5611 women were randomized to either
SLN biopsy followed by ALND or SLN biopsy followed by ALND
only if the SLN(s) were positive. SLN biopsy was performed
with blue dye and radioisotope, and participating surgeons were
required to show adequate experience with the technique. With a
mean follow-up of 95 months, there was no difference seen in the
overall survival (OS), disease-free survival, or locoregional recurrence in the two groups, thus further validating this technique in
the management of breast cancer patients.14
In conclusion, SLN biopsy is at least as accurate, if not more
accurate, as complete ALND for diagnosing the presence of nodal
metastasis as long as it is performed by an experienced surgeon.
Additionally, using this method to evaluate for nodal metastasis,
thus eliminating the need for ALND in node-negative patients,

5/22/2012 5:52:01 PM

Breast Cancer: Lymphatic Mapping and Sentinel Lymph Node Biopsy

has been shown to have equivalent survival and locoregional


recurrence rates as the previous standard of ALND with the significant benefit of decreased morbidity.

SURGICAL MANAGEMENT OF
AN SLN METASTASIS
3. Do all patients need a completion ALND when an SLN metastasis is identified?
Current recommendations from the American Society of Clinical Oncology and the National Comprehensive Cancer Network
(NCCN) recommend performing a completion ALND in patients
that have evidence of metastases in their SLN. This practice is supported by data from a meta-analysis of over 8000 patients who
underwent SLN biopsy and completion ALND where 53% of the
patients with a positive SLN were found to have additional axillary disease.15 As the number of women with small volume nodal
disease to include micrometastases has increased, the need to
routinely perform a completion ALND after a positive SLN has
been questioned. Recent publications analyzing National Cancer
Data Base (NCDB) and the Surveillance, Epidemiology, and End
Results (SEER) data have demonstrated a trend toward omitting
cALND in selected patients.16,17 The NCDB data from 1998 to 2005
showed that 21% of patients with a positive SLN did not undergo
ALND. Analysis of the SEER data demonstrated that 16% of
patients with a positive SLN did not undergo ALND. This trend
was seen predominantly in older women with low-grade, estrogen
receptor (ER) positive tumors. When only patients with a micrometastasis in their SLN were considered, the proportion undergoing SLN biopsy alone increased from 21% to 38% during the study
period. Taken together, these data suggest that many clinicians do
not believe that completion ALND plays an important role in the
management of breast cancer patients with small volume metastases in their SLN.
Recently published results from the American College of
Surgeons Oncology Group Z0011 trial support this trend of
omitting ALND in a select group of patients with a positive SLN.
The Z0011 trial enrolled patients with clinical T1 or T2 invasive breast cancer with clinically negative lymph nodes treated
with breast conserving surgery who had one or two SLNs containing metastases identified by hematoxylin and eosin staining. Patients were randomized to undergo ALND or no further
surgery; all patients received whole breast irradiation. The trial
opened in 1999 with a planned accrual of 1900 patients and
closed in 2004 after 891 patients enrolled due to slow accrual and
a lower than expected event rate. The trials primary end point
was OS and at a median follow-up of 6.3 years, 5-year OS was
91.8% with ALND and 92.5% with SLN biopsy alone; a 0.7%
absolute difference favoring the SLN biopsy alone group. Both
cohorts had substantially better OS than the 80% that had been
anticipated at protocol design.18 Study investigators also evaluated locoregional recurrences. At the 6.3-year median follow-up,
local recurrences were reported in 3.6% of patients in the ALND
group versus 1.8% in the SLN biopsy alone group. Ipsilateral axillary recurrences were identified in 0.5% of patients after ALND
and 0.9% of patients in the SLN biopsy alone arm.19
It is anticipated that the Z0011 data will be practice changing; however, there are several considerations with respect to data

PMPH_CH74.indd 599

599

interpretation and implementation.20 The trial enrolled a select


group of patients with 70% having T1 tumors, 83% ER positive
tumors, and 35% having only micrometastases identified in their
SLN. Adjuvant systemic therapy to include chemotherapy, endocrine therapy, or both was administered to 96% of women in the
ALND group and 97% in the SLN biopsy group. In addition, it is
important to remember that whole breast irradiation with standard tangential fields was given to patients as a component of their
breast conserving therapy. Whole breast irradiation administers
95% of the prescribed dose to approximately 50% of the level 1 and
25% of the level 2 axilla.21 Despite these caveats, the trial did have
strict eligibility criteria which resulted in a homogeneous population of patients allowing clinicians to translate the study findings
into clinical practice in a clearly defined population of patients
who will not derive benefit from undergoing ALND after identification of a positive SLN.20
The Z0011 trial did not enroll patients undergoing mastectomy, those treated with accelerated partial breast irradiation, or
those receiving neoadjuvant therapy; therefore, in those patients,
ALND remains the standard practice when the SLN shows evidence of disease. For patients such as these and others who do not
fulfi ll eligibility criteria for the Z0011 trial, the use of predictive
models including nomograms to predict non-SLN status in the setting of a positive SLN has become popular. The most widely used
nomogram is the one developed at the Memorial Sloan-Kettering
Cancer Center.22 This nomogram is based on eight clinicopathologic variables including tumor size, histology, nuclear grade, the
presence of lymphovascular invasion, multifocality, ER status,
method of SLN metastasis detection, number of positive SLN(s),
and number of negative SLN(s). From these data, the nomogram
generates a numerical probability of finding additional disease in
non-SLNs. The appeal of nomograms is that they provide reliable
prognostic information that is individualized to a given patient.
There is no cut-off below which a completion ALND can be omitted with absolute certainty that additional disease will not be
missed; however, the information obtained from nomograms can
be valuable in counseling a patient and in guiding the decision as
to whether an ALND will be performed.

SEQUENCING OF AXILLARY SURGERY


IN PATIENTS RECEIVING PREOPERATIVE
CHEMOTHERAPY
4. Should SLN biopsy be performed prior to receiving preoperative systemic therapy or after systemic therapy?
There is increasing use of preoperative chemotherapy for breast
cancer. Use of SLN biopsy after preoperative chemotherapy has
the potential to reduce the morbidity associated with breast cancer staging and decrease the need for ALNDs. Preoperative chemotherapy is generally utilized in women whose breast cancers
are larger and more likely to have stage II and III diseases. Women
in this category would be expected to have axillary lymph node
metastases approximately 40% to 80% of the time. As discussed
above, the current NCCN Guidelines recommend that ALND be
performed in patients known to have metastatic axillary nodes.23
Preoperative chemotherapy has now been shown to eradicate documented axillary nodal metastases in 25% to 70% of
patients.24,25 Because of this, some groups strongly advocate and

5/22/2012 5:52:01 PM

600

Surgery: Evidence-Based Practice

utilize SLN biopsy only after preoperative chemotherapy to assess


and surgically manage the axilla. On the other hand, some clinicians have suggested that SLN biopsy be performed before
preoperative chemotherapy, and that patients in whom axillary
metastases are detected be treated with ALND after preoperative chemotherapy. Clinicians who prefer to perform initial SLN
biopsy state that they would prefer to have information about
axillary metastases for prognostic purposes before preoperative
chemotherapy is begun. A potential problem with this approach
is that many patients are then committed to two surgical procedures, the SLN biopsy before preoperative chemotherapy and then
an ALND following the chemotherapy if the SLN is found to have
metastases during the first procedure. Even if the SLN is negative
during the first procedure, the patient will still need to have a second surgery for management of the breast primary after completion of chemotherapy.
A possible way of minimizing the need for multiple surgeries
is using a potential alternative to SLN biopsy before preoperative
chemotherapy. At clinical presentation, many centers are increasingly utilizing nodal ultrasound staging and ultrasound-guided
fine needle aspiration (FNA) biopsy to detect and document axillary nodal disease without the need for surgery.25 For patients
with FNA biopsy-proven axillary metastases, standard ALND
is performed after preoperative therapy and can provide a measure of the response of axillary metastases to chemotherapy. For
patients where the axillary ultrasound is normal, SLN biopsy can
be performed after preoperative chemotherapy and patients
can be spared an ALND if the SLN is found to be normal. Axillary node dissection can be performed if disease is identified
in the SLN(s) after preoperative chemotherapy.26 Furthermore,
residual disease in the axillary nodes after preoperative chemotherapy has powerful independent prognostic value.25 Two main
questions to be answered are (1) whether and (2) how the patients
systemic therapy would change if information about metastatic
nodal disease were available before, rather than after, preoperative chemotherapy. Currently, in most circumstances, decisions
about systemic therapy would be based on the primary tumor
characteristics and therefore would not be altered. Proceeding
with upfront SLN biopsy prior to neoadjuvant chemotherapy
may also be appropriate if nodal ultrasound is not available or if a
patients particular multidisciplinary treatment team is uncomfortable because of disagreement regarding the timing of the
procedure.
There have now been many investigations and three published systematic reviews evaluating the accuracy and safety of
SLN biopsy in patients treated with preoperative therapy.10,27,28
In the largest reported series of patients receiving SLN biopsy
after preoperative chemotherapy, Hunt et al. 29 compared 575
patients who underwent SLN biopsy after chemotherapy with 3171
patients who underwent surgery first. Preoperative chemotherapy
patients were younger and had more clinical T2T3 tumors at
diagnosis. SLN identification rates were 97.4% in the preoperative
group and 98.7% in the surgery first group. False-negative rates
were similar between groups. Analyzed by presenting T stage,
there were significantly fewer positive SLNs in the neoadjuvant
group. Adjusting for clinical stage revealed no differences in local
regional recurrences, disease-free, or OS between groups. This
study concluded that SLN biopsy after chemotherapy is as accurate for axillary staging as SLN surgery prior to chemotherapy.

PMPH_CH74.indd 600

SLN biopsy after chemotherapy results in fewer positive SLNs and


decreases unnecessary ALNDs.
Twenty-seven studies, with a total of more than 2100 patients,
have evaluated SLN after preoperative chemotherapy for breast
cancer.10,28 Systematic review demonstrates robust estimates of
successful identification rates and false-negative rates of SLN
biopsy after preoperative therapy for early-stage breast cancer
patients.10 Overall, an SLN can be identified approximately 90%
of the time with a false-negative rate of approximately 8%. These
results compare favorably with other large prospective clinical trials which have demonstrated identification rates of approximately
97% and an overall false-negative rate of 9.5% for patients with
early-stage breast cancer receiving SLN biopsy in the absence of
preoperative chemotherapy.30
One subgroup of patients where there remains controversy on
the accuracy of SLN biopsy is in women with documented known
axillary nodal metastases at presentation.28,31,32 To address this
group of patients, the American College of Surgeons Oncology
Group is currently accruing 660 patients to a prospective phase
II trial evaluating the role of SLN biopsy and ALND after preoperative chemotherapy in women with documented known nodepositive breast cancer at initial diagnosis.
Overall, SLN biopsy after preoperative chemotherapy appears
to be a reliable tool for planning treatment and as an alternative
to axillary surgery before preoperative chemotherapy among
patients who present with a clinically negative axilla.

ROLE OF SLN BIOPSY IN PATIENTS WITH


DUCTAL CARCINOMA IN SITU (DCIS)
5. Should SLN biopsy be performed in patients with Ductal
Carcinoma In Situ (DCIS)?
By definition, DCIS of the breast is a noninvasive lesion that does
not have the ability to metastasize. For the most part, this disease is
treated to prevent the occurrence of invasive breast cancer. Given
this background, ALND or SLN biopsy should be considered
inappropriate in patients with DCIS. However, patients with DCIS
sometimes also have microinvasive or frankly invasive carcinoma
that can be missed with image-guided biopsy. In fact, patients
treated for DCIS sometimes, although very rarely, die of metastatic breast cancer, most likely as a result of an otherwise missed
invasive component of disease. It would be difficult to justify SLN
biopsy in the majority of patients treated for DCIS with segmental
resection, as this tissue can be thoroughly evaluated for the presence of invasive disease and SLN biopsy can be performed as a
subsequent surgical procedure. However, for patients undergoing
mastectomy for the treatment of DCIS, the ability to perform SLN
biopsy is lost if the breast is removed and invasive carcinoma is
identified. In this situation, ALND is indicated to assess the critical staging information. In a large series from the University of
Texas MD Anderson Cancer Center, 399 patients with an initial
diagnosis of DCIS were identified to determine which factors were
associated with finding invasive carcinoma on final pathologic
evaluation.33 On multivariate analysis, significant independent
predictors of finding invasive carcinoma were age younger than
or equal to 55 years; diagnosis made with a core biopsy; mammographic primary tumor size greater than 4 cm; and high-grade

5/22/2012 5:52:01 PM

Breast Cancer: Lymphatic Mapping and Sentinel Lymph Node Biopsy

tumor status. Overall, 20% of the patients with an initial diagnosis of DCIS were found to have invasive carcinoma, and 35%
of these patients underwent an SLN biopsy. Patients in this series
were more often offered SLN biopsy if they had a mastectomy.
Ten percent of the patients were found to have a positive SLN and
the only independent predictor of finding axillary metastases in
patients initially believed to have only DCIS was the presence of
a palpable tumor at diagnosis. On the basis of this analysis, the
investigators from that study do not routinely perform SLN biopsy
on all patients with an initial diagnosis of DCIS. Instead, the risks
and benefits of SLN biopsy are discussed with patients scheduled to undergo mastectomy, younger patients, and patients with
large or high-grade DCIS. At the European Institute of Oncology
(Milan, Italy), only 1.4% of 854 unselected patients with pure DCIS
were found to have a positive SLN and no patients were found to
have additional positive nodes on ALND.34 Finally, a recent metaanalysis of 22 published studies found that approximately 3.7% of
patients with a postoperative pathologic diagnosis of DCIS were
found to have SLN metastases.35
In 2009, the NIH Consensus Conference on DCIS made some
important points regarding SLN biopsy use and DCIS: the clinical
significance of positive SLN metastases in patients who have DCIS
is indeterminate, given that the majority of them are micrometastases or isolated tumor cells; existing studies of SLN biopsy have
been reported in highly selected patient populations that may
not represent the general population of women who have DCIS;
studies of the impact of SLN biopsy for DCIS on subsequent treatments have been limited to descriptions of single (not multicenter)
practices, and although SLN biopsy is less invasive than ALND,

601

it is associated with some risk of complications, including lymphedema and pain in some women.36
Taken together, the results of published uncontrolled studies
indicate that lymphatic mapping and SLN biopsy for DCIS should
not be routinely done in all patients. Patients with a diagnosis
of DCIS who are scheduled to undergo mastectomy, and other
patients considered at high risk for having invasive disease based
on suspicion of invasion on core biopsy, can be offered SLN biopsy
as part of their initial surgical management.

CONCLUSIONS
In conclusion, SLN biopsy has become a standard technique in the
management of patients with breast cancer. It offers accurate staging of the axilla with much less morbidity than previous standard
of ALND. It should be offered as a component of initial surgical
management to clinically node-negative breast cancer patients,
even those who receive neoadjuvant chemotherapy, and patients
with DCIS who are either undergoing mastectomy or have a high
risk of discovering invasive cancer on pathologic evaluation of
their surgical specimen. Although the exact technical aspects may
vary, experienced surgeons can safely perform the procedure with
reliable results. Although the standard approach has been to perform ALND on all patients with SLN metastasis, new data from
the ACOSOG Z0011 trial shows that ALND can safely be omitted
in carefully selected patients. Clinicians caring for breast cancer
patients should be familiar with the technique and understand its
benefits and limitations.

Clinical Question Summary


Question

Answer

Level of
Evidence

Grade of
Recommendation

References

1 What is the best


technique for SLN
biopsy?

Blue dye, radioisotope, or a combination of the


two should be injected in either a peritumoral,
intradermal, or subareolar location. In exploring
the axilla, the surgeon should remove any node
that is blue, hot, or suspicious. Although no
technique has been shown to be superior to
another, the success of SLN biopsy is reliant on
surgeon experience.

2b

2-7

2 Is SLN biopsy
as accurate as
ALND?

Yes. In fact, it may be more accurate as it allows for


more thorough evaluation of lymph nodes.

1a

10-14

3 Do all patients
need a completion
ALND if SLN
metastasis is
identified?

No. Omission of ALND in the setting of a positive


SLN is appropriate for early-stage patients (T1/
T2, clinically node-negative) treated with breast
conserving therapy to include whole breast
irradiation who have one or two positive SLN(s)
identified by H&E. Other patients, to include
those undergoing mastectomy, those treated
with accelerated partial breast irradiation, or
those receiving neoadjuvant therapy should
undergo ALND as standard practice when the
SLN shows evidence of disease.

1b

15-19

(Continued)

PMPH_CH74.indd 601

5/22/2012 5:52:01 PM

602

Surgery: Evidence-Based Practice

(Continued)
Question

Answer

4 Should SLN biopsy


be performed
prior to receiving
preoperative
systemic therapy
or after therapy?

SLN biopsy after preoperative chemotherapy


appears to be a reliable tool for planning
treatment and as an alternative to axillary
surgery before preoperative chemotherapy.
Additionally, it may decrease unnecessary axillary
dissections in patients presenting with clinically
negative axilla.

2b

10, 27-30

5 Should patients
with DCIS have an
SLN biopsy?.

Not routinely. Those who are scheduled to undergo


mastectomy and other patients considered at
high risk for having invasive disease based can be
offered SLN biopsy as part of their initial surgical
management.

2b

33-36

REFERENCES
1. Sato K, Shigenaga R, Ueda S, et al. Sentinel lymph node biopsy
for breast cancer. J Surg Oncol. 2007;96:322-329.
2. McMasters K, Tuttle T, Carlson D, et al. Sentinel lymph node
biopsy for breast cancer: A suitable alternative to routine axillary
dissection in multi-institutional practice when optimal technique
is used. J Clin Oncol. 2000;18:2560-2566.
3. Morrow M, Rademaker A, Bethke K, et al. Learning sentinel
node biopsy: Results of a prospective randomized trial of two
techniques. Surgery. 1999;126:714-720.
4. Kang T, Yi M, Hunt K, et al. Does blue dye contribute to success of sentinel node mapping for breast cancer? Ann Surg Oncol.
2010;17:S280-S285.
5. Jin Kim H, Heerdt A, Cody H, et al. Sentinel lymph node drainage in multicentric breast cancers. Breast J. 2002;8:356-361.
6. Linehan D, Hill A, Akhurst T, et al. Intradermal radiocolloid and
intraparenchymal blue dye injection optimize sentinel node identification in breast cancer patients. Ann Surg Oncol. 1999;6:450-454.
7. Rodier J, Velten M, Wilt M, et al. Prospective multicentric randomized study comparing periareolar and peritumoral injection
of radiotracer and blue dye for the detection of sentinel lymph
node in breast sparing procedures: FRANSENODE trial. J Clin
Oncol. 2007;25:3664-3669.
8. Yi M, Meric-Bernstam F, Ross M, et al. How many sentinel lymph
nodes are enough during sentinel lymph node dissection for
breast cancer? Cancer. 2008;113:30-37.
9. Goldberg J, Wiechmann L, Riedel E, et al. Morbidity of sentinel
node biopsy in breast cancer: The relationship between the number of excised lymph nodes and lymphedema. Ann Surg Oncol.
2010;17:3278-3286.
10. Kelly A, Dwamena B, Cronin P, et al. Breast cancer sentinel node
identification and classification after neoadjuvant chemotherapysystematic review and meta analysis. Acad Radiol. 2009;16:
551-563.
11. Krag D, Weaver D, Ashikaga T, et al. The sentinel node in breast
cancera multicenter validation study. N Engl J Med. 1998;
339:941-946.
12. Veronesi U, Paganelli G, Viale G, et al. A randomized comparison of sentinel-node biopsy with routine axillary dissection in
breast cancer. N Engl J Med. 2003;349:546-553.

PMPH_CH74.indd 602

Level of
Evidence

Grade of
Recommendation

References

13. Lyman G, Giuliano A, Somerfield M, et al. American Society


of Clinical Oncology guideline recommendations for sentinel
lymph node biopsy in early-stage breast cancer. J Clin Oncol.
2005;23:7703-7720.
14. Krag D, Anderson S, Julian T, et al. Sentinel-lymph-node resection compared with conventional axillary-lymph-node dissection
in clinically node-negative patients with breast cancer: Overall
survival findings from the NSABP B-32 randomised phase 3 trial.
Lancet Oncol. 2010;11:927-933.
15. Kim T, Guiuliano A, Lyman G. Lymphatic mapping and sentinel
lymph node biopsy in early-stage breast carcinoma: A metaanalysis. Cancer. 2006;106:4-16.
16. Bilimoria K, Bentrem D, Hansen N, et al. Comparison of sentinel
lymph node biopsy alone and completion axillary lymph node
dissection for node-positive breast cancer. J Clin Oncol. 2009;
27:2946-2953.
17. Yi M, Giordano S, Meric-Bernstam F, et al. Trends in and outcomes from sentinel lymph node biopsy (SLNB) alone vs. SLNB
with axillary lymph node dissection for node-positive breast cancer patients: Experience from the SEER database. Ann Surg Oncol.
2010;17.
18. Giuliano A, Hunt K, Ballman K, et al. Axillary dissection vs no
axillary dissection in women with invasive breast cancer and
sentinel node metastasis: A randomized clinical trial. JAMA.
2011;305:569-575.
19. Giuliano A, McCall L, Beitsch P, et al. Locoregional recurrence
after sentinel lymph node dissection with or without axillary
dissection in patients with sentinel lymph node metastases: The
American College of Surgeons Oncology Group Z0011 randomized trial. Ann Surg. 2010;252:426-432.
20. Caudle A, Hunt K, Kuerer H, et al. Multidisciplinary considerations concerning implementation of the fi ndings from the
American College of Surgeons Oncology Group (ACOSOG)
Z0011 study: A practice-changing trial. Ann Surg Oncol. 2011.
21. Reznik J, Cicchetti M, Degaspe B, et al. Analysis of axillary
coverage during tangential radiation therapy to the breast. Int J
Radiat Oncol Biol Phys. 2005;61:163-168.
22. Van Zee K, Manasseh D, Bevilacqua J, et al. A nomogram for
predicting the likelihood of additional nodal metastases in
breast cancer patients with a positive sentinel node biopsy. Ann
Surg Oncol. 2003;10:1140-1151.

5/22/2012 5:52:02 PM

Breast Cancer: Lymphatic Mapping and Sentinel Lymph Node Biopsy

23. Network, NCC. Surgical Axillary Staging Algorithm from the


NCCN Invasive Breast Cancer Clinical Practice Guidelines.
Vol. 2011. 2011.
24. Dominici L, Negron-Gonzalez V, Buzdar A, et al. Cytologically
proven axillary lymph node metastases are eradicated in patients
receiving preoperative chemotherapy with concurrent trastuzumab
for HER2-positive breast cancer. Cancer. 2010;116:2884-2889.
25. Kuerer H, Sahin A, Hunt K, et al. Incidence and impact of
documented eradication of breast cancer axillary lymph node
metastases before surgery in patients treated with neoadjuvant
chemotherapy. Ann Surg. 1999;230:72-78.
26. Breslin T, Cohen L, Sahin A, et al. Sentinel lymph node biopsy is
accurate after neoadjuvant chemotherapy for breast cancer. J Clin
Oncol. 2000;18:3480-3486.
27. Xing Y, Foy M, Cox S, et al. Meta-analysis of sentinel lymph node
biopsy after preoperative chemotherapy in patients with breast
cancer. Br J Surg. 2006;93:539-546.
28. van Duerzen C, Vriens B, Tjan-Heijnen V, et al. Accuracy of sentinel node biopsy after neoadjuvant chemotherapy in breast cancer
patients: A systematic review. Eur J Cancer. 2009;45:3124-3130.
29. Hunt K, Mittendorf E, Guerrero C, et al. Sentinel lymph node surgery after neoadjuvant chemotherapy is accurate and reduces the
need for axillary dissection in breast cancer patients. Ann Surg.
2009;4:558-566.
30. Krag D, Ashikaga T, Harlow S, et al. Surgeon training, protocol
compliance, and technical outcomes from breast cancer sentinel

PMPH_CH74.indd 603

31.

32.

33.

34.

35.

36.

603

lymph node randomized trial. J Natl Cancer Inst. 2009;101:


1356-1362.
Shen J, Gilcrease M, Babiera G, et al. Feasibility and accuracy
of sentinel lymph node biopsy after preoperative chemotherapy
in breast cancer patients with documented axillary metastases.
Cancer. 2007;109:1255-1263.
Classe J, Bordes V, Campion L, et al. Sentinel lymph node biopsy
after neoadjuvant chemotherapy for advanced breast cancer:
Results of Ganglion Sentinelle et Chimiotherapie Neoadjuvante,
a French prospective multicentric study. J Clin Oncol. 2009;
27:726-732.
Yen T, Hunt K, Ross M, et al. Predictors of invasive breast cancer
in patients with an initial diagnosis of ductal carcinoma in situ: A
guide to selective use of sentinel lymph node biopsy in management
of ductal carcinoma in situ. J Am Coll Surg. 2005;200:516-526.
Intra M, Rotmensz N, Veronesi P, et al. Sentinel node biopsy is not
a standard procedure in ductal carcinoma in situ of the breast: The
experience of the European institute of oncology on 854 patients
in 10 years. Ann Surg. 2008;247:315-319.
Ansari B, Ogsten S, Purdie C, et al. Meta-analysis of sentinel
node biopsy in ductal carcinoma in situ of the breast. Br J Surg.
2008;95:547-554.
Allegra C, Aberle D, Ganschow P, et al. NIH state-of-the-science
conference statement: Diagnosis and management of ductal
carcinoma in situ (DCIS). NIH Consens State Sci Statements.
2009;26:1-27.

5/22/2012 5:52:02 PM

CHAPTER 75

Systemic Treatment Strategies for


Early-Stage Breast Cancers
Jennifer K. Litton and Kelly K. Hunt

INTRODUCTION

The indications for recommending adjuvant chemotherapy are


based not only on the stage of disease at presentation, but also
increasingly on the underlying biology of the tumor. There are
a number of important clinical, pathologic, and biologic factors
that determine prognosis and indications for treatment. These
factors include age, medical comorbidities, tumor grade, involvement of regional lymph nodes, and hormone receptor (HR) status
and HER2 status of the primary tumor.1,2 There have been several
tools developed to guide clinicians in the assessment of risk of
recurrence and potential benefit from therapeutic interventions
when discussing adjuvant therapies with their patients. These
include the Adjuvant! Online computer program, 3 Mammaprint,4
and the 21-gene recurrence score assay (Oncotype DxTM).5 These
are all discussed in more detail below. In addition, there are
treatment guidelines, such as those developed by the National
Comprehensive Cancer Network (NCCN)1 that provides expert
opinion synthesizing the available evidence for treatment recommendations largely based on the stage of disease at presentation.
These guidelines are generally updated annually and are accessible via www.nccn.org.
Much of the evidence for adjuvant treatment decisions in
early-stage breast cancer comes from the Early Breast Cancer Trialists Collaborative Group (EBCTCG).2 Th is group meets every
5 years and reviews the clinical trial data from studies around the
world that initiated accrual by 1995. The last published update
was in 2005 and the most recent data have been reviewed by
the EBCTCG panel and a forthcoming publication is anticipated
in the near future. A series of important questions have been
asked by clinical trialists, the fi rst of which was the value of singleagent chemotherapy versus polychemotherapy in the adjuvant
setting. The ECBCTG has data from 4000 women who received
single-agent chemotherapy and 29,000 women who received
polychemotherapy as part of a clinical trial. Polychemotherapy
regimens included agents such as CMF (cyclophosphamide,
methotrexate, and 5-fluorouracil), FEC, (5-fluorouracil, epirubicin, cyclophosphamide), FAC (5-fluorouracil, doxorubicin,
and cyclophosphamide), and AC (doxorubicin and cyclophosphamide). Some regimens also included taxanes such as docetaxel

There have now been significant data accumulated demonstrating


an improvement in breast cancer-specific survival rates when adjuvant systemic therapies are employed following the local-regional
management of breast cancer patients. Chemotherapy, endocrine
therapy, and, more recently, the addition of biologic therapies have
all contributed to improved outcomes. The treatment of breast cancer
has evolved into a more integrated multidisciplinary program with
differences in the sequencing of local-regional and systemic therapies determined by individual patient and tumor characteristics. The
estrogen and progesterone receptor status of the primary tumor have
been used to personalize breast cancer treatment for decades and
HER2 status has been shown to be an important prognostic marker
and predictive factor for HER2-directed therapies. In addition to
improvements in overall (OS) and disease-free survival (DFS) with
the use of systemic therapies, these agents can also result in downsizing of the primary tumor to allowing for breast-conserving therapy
and response to therapy can provide important biologic information.
Therefore, it is paramount that all of the disciplines involved in the
treatment of breast cancer patients understand the indications for
systemic therapy and the available therapeutic strategies.
In this review, we will summarize the current recommendations and ongoing controversies in the systemic treatment of breast
cancer. The indications for chemotherapy and hormonal therapy and
emerging biologic therapies are reviewed, as well as the option for
utilizing these agents in the neoadjuvant setting. Information that
is used to improve patient selection for systemic treatment will be
discussed in an effort to avoid toxicity in those patients who would
likely derive little or no benefit. The goal is to improve our ability
to provide more targeted therapies and more effective, personalized
local and systemic treatments to our breast cancer patients.

DECISION-MAKING IN THE USE OF


ADJUVANT SYSTEMIC THERAPY
1. What are the indications for adjuvant chemotherapy in breast
cancer and which chemotherapy regimens should be used?
604

PMPH_CH75.indd 604

5/22/2012 5:52:42 PM

Systemic Treatment Strategies for Early-Stage Breast Cancers

(D) or paclitaxel (T) among many other agents. Although


single-agent chemotherapy did lower the risk of recurrence, the
use of polychemotherapy more substantially decreased recurrence
and breast cancer mortality hazard ratios (HRs) (0.77 and 0.83,
respectively). Polychemotherapy was found in subgroup analyses to be almost three times more effective in younger women
(<50 years old) compared with older women (ages 5069 years).
In addition, the benefit of polychemotherapy for reducing breast
cancer recurrence and mortality remained significant in the subgroups of both hormone receptor-positive and negative tumors
in younger women. In older women, this benefit of polychemotherapy did not maintain statistical significance in the setting
of tamoxifen usage in patients with hormone receptor-positive
breast cancer but did show a 5-year gain of 9.6% with respect
to disease recurrence (logrank = p < .00001) in older women
(ages 5069 years) with hormone receptor-negative tumors. Th is
overview analysis focused largely on age and hormone receptor status and did not take into account other factors such as
HER2 status, grade, lymphovascular invasion, or molecular
subtypes.
There are many systemic chemotherapy agents that have
shown activity in breast cancer and the NCCN guidelines1 list 14
different polychemotherapy regimens and at least 6 trastuzumabcontaining regimens (for HER2-positive disease) that are considered appropriate choices for therapy. The choice is often based
on institutional preference but may also be impacted by patient
factors. Most regimens contain an anthracycline as part of the
therapy (doxorubicin or epirubicin) based on historical data demonstrating that there is reduction in the risk of recurrence with
anthracycline- containing regimens over nonanthracycline-based
therapies. A selection of these pivotal studies is described below;
however, an excellent history of all of the randomized chemotherapy trials is detailed on www.cancer.gov/clinicaltrials/results/
type/breast.
The INT-01026 trial evaluated CMF versus CAF with and
without tamoxifen for high-risk, node-negative breast cancer
patients. CAF did not improve DFS but did trend toward the
improved OS. The use of tamoxifen was beneficial only in those
women with hormone receptor-positive disease. NSABP-B157
compared six cycles of CMF with four cycles of AC and showed
no difference in DFS and OS. The EBCTCG2 also compared
anthracycline-containing regimens with nonanthracyclinecontaining regimens and confi rmed a modest benefit for the
anthracycline-containing regimens of 3.4% for recurrence and
3.3% for mortality at 15 years. With the addition of taxanes to
systemic therapy regimens, additional gains in breast cancer
outcomes have been realized. CALGB 93448 evaluated AC
4 paclitaxel every 3 weeks in patients with node-positive disease and demonstrated significant improvements in both DFS
and OS. Similar results were demonstrated in NSABP B-289
with slightly higher doses of paclitaxel. More recently, investigators have evaluated the concept of replacing the anthracycline
with a taxane. The US Oncology Clinical Trials Group randomized 1016 women with stage III breast cancer to TC (docetaxel
and cyclophosphamide) 4 versus AC 4. The TC regimen
resulted in a 6% (p = .018) improvement in DFS and 4% (p = .045)
improvement in OS at 7 years.10 The schedule and dosing of both
docetaxel and paclitaxel were evaluated in ECOG 1199.11 Th is
study had four arms and compared paclitaxel 175 mg/m 2 every
3 weeks 4 versus paclitaxel 80 mg/m 2 weekly 12 weeks versus

PMPH_CH75.indd 605

605

docetaxel 100 mg/m 2 every 3 weeks 4 versus docetaxel 35 mg/m 2


weekly 12 weeks. The every 3-week docetaxel dosing and the
weekly paclitaxel dosing were similar in terms of improved
DFS and OS but with differing toxicity profi les. Docetaxel had
increased hematologic toxicities, whereas the weekly paclitaxel
was associated with an increased incidence of peripheral neuropathy. There are multiple ongoing studies currently accruing
to further address the benefit of taxanes in high-risk nodenegative disease settings as well as taxane combinations without
anthracyclines versus anthracycline-containing regimens.
There continues to be significant debate as to which patient
populations may derive additional benefit from anthracyclinebased chemotherapy and which patients can avoid the potential
cardiotoxicity of this class of agents. Several expert opinions
have emerged regarding this debate; however, clinicians are
forced to address this issue on a daily basis.12,13 In addition to the
potential for cardiotoxicity, other significant long-term toxicity
concerns with the use of many of these regimens include treatment-related leukemia, peripheral neuropathy, and infertility.
The risk of congestive heart failure from adjuvant anthracycline
regimens when cumulative doses of 240 mg/m 2 or less were used
ranges from 0.5% to 2.1% in 5 years.14 Estimates from the SEER
database have been reported as HR of 1.26; 96% confidence interval [CI] 1.121.42) for women aged 66 to 70 years. Cardiac risk
factors such as hypertension, diabetes, and coronary artery disease are predictors of developing congestive heart failure.15 The
risk of treatment-related leukemia at 10 years after treatment is
estimated at 1.8% versus 1.2% in breast cancer patients who did
not receive chemotherapy.16 Infertility related to chemotherapy
is difficult to measure and amenorrhea is often used as a surrogate marker. In general, the closer a woman is to her natural
age of menopause, the more likely she is to experience premature
ovarian failure from systemic chemotherapy.17 Therefore, women
who are candidates for adjuvant chemotherapy and/or tamoxifen who are of child-bearing potential and desirous of future
pregnancies should discuss their risks and options for fertility
preservation prior to initiating therapy.
Answer: The NCCN guidelines recommend consideration of
adjuvant chemotherapy for women with node-positive disease.
For women with HER2-negative tumors, and tumors that are
>1 cm and node-negative or 0.6 to 1 cm with unfavorable features
such as high grade and lymphovascular invasion, the NCCN
guidelines recommend consideration of further information that
may impact treatment decisions, such as the 21-gene recurrence
score assay (Oncotype DxTM). Patients who can avoid adjuvant
chemotherapy or be considered for adjuvant endocrine therapy
alone are those patients with hormone receptor-positive tumors
<0.6 cm.

USE OF TUMOR MARKERS IN ADJUVANT


THERAPY DECISIONS
2. What are the indications for adjuvant endocrine therapy in
breast cancer?
Women with hormone receptor-positive tumors (estrogen receptor and/or progesterone receptor positive) should be considered
for adjuvant endocrine therapy. The EBCTCG overview analysis
showed a significant benefit for the use of adjuvant tamoxifen in

5/22/2012 5:52:42 PM

606

Surgery: Evidence-Based Practice

women with hormone receptor-positive tumors but no benefit


for women with hormone receptor-negative tumors. In terms of
the length of tamoxifen therapy, 5 years was superior compared
with 2 years. For patients with hormone receptor-positive disease,
5 years of tamoxifen therapy resulted in a recurrence ratio of 0.59
and breast cancer mortality ratio of 0.66 when compared with no
endocrine therapy. Albain et al.18 evaluated the timing of tamoxifen therapy, either concurrent with chemotherapy or after the
completion of chemotherapy (sequentially) in a phase III parallel
randomized trial. The use of sequential therapy trended toward
superiority with a DFS HR of 0.84 (95% CI 0.701.01, p = .61) and
OS HR of 0.90 (95% CI 0.731.10, p = .30). Importantly, tamoxifen has demonstrated efficacy in both pre- and postmenopausal
women with hormone receptor-positive tumors.
Aromatase inhibitors (AI) have been compared with tamoxifen in postmenopausal women with hormone receptor-positive
breast cancer and have been shown to be superior in terms of DFS
and toxicity profi les. The NCCN guidelines recommend considering that a woman is postmenopausal if she has had an oophorectomy, she is 60 years old, or 60 years old with amenorrhea for
12 or more months without exposure to chemotherapy, tamoxifen, or ovarian suppression, and FSH and plasma estradiol levels
are in the postmenopausal range. If a patient is taking tamoxifen,
it is important not to rely on FSH alone, but also on plasma estradiol levels. As ovarian function can be quite pulsatile and women
may regain ovarian function after chemotherapy-induced amenorrhea after 1 year, following these levels in women exposed to
chemotherapy is paramount. Women who are premenopausal or
perimenopausal at the time of initiation of chemotherapy should
be started on tamoxifen even if they experience treatment-related
ovarian failure.19
The American Society of Clinical Oncology (ASCO)19 has
recently released published guidelines regarding endocrine therapy. These guidelines have been completed after an exhaustive
literature search and evaluation of several randomized clinical trials. These trials include the ATAC,20 BIG-1-98,21 and ABCSG-1222
studies. The ATAC and BIG 1-98 trials evaluated tamoxifen versus an AI. The ATAC trial showed an improved DFS HR of 0.85;
95% CI 0.760.94, p = .003 for anastrozole when compared with
tamoxifen after 100 months of follow-up.20 There was no benefit
identified to administering tamoxifen concurrently with an AI.
The BIG 1-98 trial again supported this finding. There have been
multiple trials addressing the question of sequencing of tamoxifen
with aromatase inhibitors (BIG 1-98, MA-17,23 and TEAM24 trials). Each AI-based therapy was superior to tamoxifen alone but
there was no statistically significant difference in the timing or
sequencing if tamoxifen and an AI were used. Therefore, the current ASCO guidelines support using an AI at some point during
the first 5 years of therapy, if the patient is postmenopausal. There
is currently no evidence to support using an AI for more than
5 years. A multicenter, randomized clinical trial to evaluate this
question is ongoing through the NSABP.
Tamoxifen requires CYP2D6 to metabolize the drug into
its active form. This knowledge has led to significant debate as
to the utility of CYP2D6 isoenzyme testing in patients planned
for tamoxifen therapy. Clinical observations have been made
that women taking SSRI drugs have decreased the efficacy of
tamoxifen because of the interference with this metabolic pathway.25 However, there has yet to be convincing evidence that
changes in the CYP2D6 isoenzyme affects outcomes; therefore

PMPH_CH75.indd 606

ASCO guidelines state that there is insufficient evidence to recommend testing for CYP2D6 isoenzyme to determine if a patient
should or should not receive tamoxifen therapy.19
Answer: Tamoxifen should be given to women with hormone
receptor-positive tumors and who are pre- or perimenopausal.
Women who are postmenopausal should receive an AI alone or in
sequence with tamoxifen for at least 5 years of adjuvant endocrine
therapy.

INCORPORATION OF BIOLOGIC
THERAPIES INTO SYSTEMIC
THERAPY REGIMENS
3. What are the indications for adjuvant biologic therapies in
breast cancer?
The use of trastuzumab in the adjuvant setting has made a significant impact on survival for patients with HER2-positive
breast cancer. After multiple studies demonstrated its efficacy in
the metastatic setting, trastuzumab was tested in several large
adjuvant trials, and these all demonstrated impressive antitumor
activity. These studies included the North Central Cancer Treatment Group (NCCTG) Intergroup N9831 trial, the National Surgical Adjuvant Breast and Bowel Project (NSABP) B-31 trial, the
Herceptin Adjuvant trial (HERA), BCIRG 006, and FinHER.26-30
The HERA trial28,31 evaluated more than 5000 women who
received adjuvant chemotherapy from a list of acceptable chemotherapy regimens who were then randomized to receive either 1 or
2 years of trastuzumab therapy versus observation and no further
therapy. Trastuzumab was delivered upon completion of chemotherapy and not given concurrently with chemotherapy. The HR
for recurrence after the completion of chemotherapy and 1 year of
trastuzumab was 0.64 (p < .001). The results of extension of trastuzumab to 2 years have not yet been reported.
The NSABP (B-31) and the NCCTG (N9831) trials had similar designs and data were pooled together once the HERA trial
fi rst reported. The only difference between the studies was the
schedule of paclitaxel administration of weekly versus every 3
weeks dosing. In one arm of the N9831 trial, trastuzumab was
given during the taxane portion of the chemotherapy and in the
other arm trastuzumab was administered after completion of
chemotherapy. Significant differences were noted in DFS in the
patients who received trastuzumab versus those who did not
(85.3% vs. 67.1%, p .0001). There was also a significant difference in OS of 91.4% versus 86.6%, p = .015, favoring trastuzumab
therapy.
The BCIRG 006 compared AC 4 followed by docetaxel 4
(AC-T) alone or with trastuzumab (AC-TH) versus TCH (docetaxel, carboplatinum, and trastuzumab). In the third planned
efficacy analysis,30 which was presented at the San Antonio Breast
Cancer Symposium in 2009, AC-TH was associated with a DFS
of 84%, TCH of 81%, and AC-T of 75% at 60 months. When compared with AC-T, AC-TH had a HR of 0.64, p < .001 and TCH
had a HR of 0.75, p = .04. When compared with AC-T in terms of
OS, AC-TH had a HR of 0.63, p < .001 and TCH had a HR of 0.77,
p = .038. The FinHER study27 was a smaller trial that randomized 1010 women with unselected primary breast cancer to receive
three cycles of vinorelbine or docetaxel, followed by FEC. HER2
amplification was identified in 232 tumors and these women were

5/22/2012 5:52:42 PM

Systemic Treatment Strategies for Early-Stage Breast Cancers

further randomized to receive 9 weeks of trastuzumab with either


the vinorelbine or docetaxel. Trastuzumab was administered only
during treatment with vinorelbine or docetaxel. The patients who
received trastuzumab had a better 3-year recurrence-free survival
(RFS) when compared with those who did not receive trastuzumab
(89% vs. 78%, p = .01).
With the success of trastuzumab therapy in the adjuvant
treatment of patients with HER2-positive breast cancer, several
groups have initiated studies evaluating different regimens in the
neoadjuvant setting. Regimens that have been demonstrated to
be effective in the adjuvant setting are often used in the neoadjuvant setting and several have been listed as potential options
in the NCCN guidelines. Although concerns have been raised
when giving anthracyclines concurrent with trastuzumab in the
metastatic setting, Buzdar et al.32 have demonstrated impressive pathologic response rates when combining trastuzumab
with weekly paclitaxel (80 mg/m2) followed by FEC (with epirubicin given at a lower dose of 75 mg/m2). After assessment of
the first 42 patients randomized on this trial, the data and safety
monitoring committee recommended that the trial be stopped
early as the pathologic complete response (pCR) rate for patients
receiving chemotherapy plus trastuzumab was 66.7% versus
25% in patients treated with chemotherapy alone. Given these
high pCR rates, this regimen of concurrent chemotherapy with
trastuzumab is being studied in an ongoing multi-institutional,
randomized trial through the American College of Surgeons
Oncology Groups (ACOSOG).
Given the known aggressive biology of HER2-positive disease
and the efficacy and tolerability of trastuzumab, the NCCN guidelines recommend trastuzumab-based chemotherapy for all patients
with node-positive tumors and for those with node-negative
tumors >1 cm. Several groups have also studied the outcomes of
patients with HER2-positive tumors less than 1 cm. GonzalezAngulo et al.33 have reported that patients with HER2-positive
tumors that do not receive trastuzumab or chemotherapy, the
5-year RFS rates are as low as 77.1%. Therefore, trastuzumab-based
chemotherapy regimens should be considered for patients with
tumors 0.6 to 1 cm in size. Further evidence is being collected for
tumors <0.6 cm before further modifications to the guidelines can
be recommended.
Multiple other biologic therapies have been studied in the
metastatic setting including bevacizumab, pertuzumab, and lapatinib. There are now several ongoing studies evaluating these and
other biologic therapies in the adjuvant and neoadjuvant settings;
however, to date, there is no indication for use of these agents
outside of a clinical trial in the adjuvant or in the neoadjuvant
setting.
Answer: Trastuzumab therapy should be used in combination
with chemotherapy for all patients with HER2-positive tumors
that are node-positive or node-negative and 0.6 cm in size.

SEQUENCING OF SYTEMIC AND


LOCAL-REGIONAL THERAPIES
4. What is the role for neoadjuvant therapies in breast cancer?
Neoadjuvant (or preoperative) chemotherapy has traditionally
been utilized in cases of inoperable or locally advanced disease

PMPH_CH75.indd 607

607

to improve or facilitate local-regional treatment with surgery and


radiation. Substantial clinical response rates (complete and partial)
have been documented in patients with locally advanced disease
leading many to opt for this approach in patients with operable
breast cancer. The landmark National Surgical Adjuvant Breast and
Bowel Project (NSABP) B-18 trial clearly demonstrated increased
rates of breast conservation in women randomized to neoadjuvant
chemotherapy.34 Patients underwent surgery first or received four
cycles of doxorubicin plus cyclophosphamide (AC) every 3 weeks.
The use of neoadjuvant AC increased the proportion of patients
able to undergo lumpectomy by 12%. At a mean follow-up of 9.5
years, no significant differences in DFS and OS rates were noted
between the two groups (69% vs. 70%, p = .80; 55% vs. 53%, p = .50,
respectively). Similar results have been published from other randomized studies, and a recent pooled meta-analysis revealed that
survival outcomes are equivalent between the surgery first and
neoadjuvant approaches.35 Therefore, neoadjuvant chemotherapy
is a safe alternative to the use of adjuvant chemotherapy, especially
in patients who prefer breast conservation therapy and may not be
candidates if surgery is the initial approach.
Another advantage of shift ing chemotherapy to the neoadjuvant setting is the ability to assess clinical and pathologic response
rates of the primary tumor and any nodal metastases to systemic
treatment. Both clinical and pathologic response rates have been
proposed to be surrogate markers for the traditional endpoints of
DFS and OS typically measured in adjuvant trials. In the European
Organisation for Research and Treatment of Cancer (EORTC)
10902 trial, patients with operable breast cancer were randomized
to pre- or postoperative therapy with four cycles of 5-fluorouracil,
epirubicin, and cyclophosphamide (FEC) every 3 weeks.36 At 56
months of follow-up, no significant differences were seen in DFS
and OS. There were 13 patients in the preoperative therapy arm
who achieved a pCR, defined as the absence of invasive cancer in
the breast and axillary nodes. This was associated with a statistically improved OS compared with patients who did not achieve a
pCR, with a HR of 0.86 (95% CI 0.770.96; p = .008). The use of
pCR after neoadjuvant chemotherapy has been suggested as a surrogate endpoint for DFS and OS. Evaluation of agents in the neoadjuvant setting using pCR as a measure of efficacy can provide a
more rapid evaluation of treatments for patients with early-stage
breast cancer. Symmans et al. developed a prognostic model that
quantifies residual disease in the breast and lymph nodes after
neoadjuvant chemotherapy to estimate distant relapse rates at
5 years.37 This residual cancer burden index is a tool that has been
validated and is considered a useful surrogate marker for longterm survival endpoints.
As described above, the neoadjuvant approach can facilitate
testing of novel biologic agents alone and in combination with
systemic chemotherapy. In addition to targeted treatment trials
with specific biologic agents, investigators are currently using
gene-expression profiling to estimate the likelihood of achieving a pCR with specific regimens. There have now been several
reports of in vitro chemosensitivity gene- expression signatures,
which can predict response to neoadjuvant chemotherapy with
high degrees of sensitivity and specificity.38 A similar concept is
being tested with neoadjuvant endocrine therapy in patients with
hormone receptor-positive disease in the American College of
Surgeons Oncology Group Z1031 trial. Patients are randomized
to receive one of three AIs for 16 weeks prior to surgical resection.
Pre- and posttreatment samples are obtained and gene-expression

5/22/2012 5:52:43 PM

608

Surgery: Evidence-Based Practice

profi ling is being performed with the goal of developing profi les
that can predict resistance to endocrine therapies.
The neoadjuvant therapy approach has the potential to allow
individualization of cancer therapies through the use of geneexpression profi ling of tumors and assessment of pathologic
response rates. Th is approach can expedite evaluation of novel
therapies with the surrogate endpoints for survival of pCR and
residual cancer burden, without the need for long-term follow-up
typically required in adjuvant treatment trials. Using clinical
response early in the course of neoadjuvant therapies may spare
some patients the toxicities of ineffective therapies because they
can be discontinued if there is no response or progression of disease. The major question that remains is what to recommend for
patients who continue to have significant residual disease despite
the use of neoadjuvant chemotherapy. This is a group of patients at
significantly increased risk for systemic disease recurrence.
The NSABP B-18 and EORTC trials showed that breastconserving surgery was safe following neoadjuvant chemotherapy. There is no consensus on the volume of tissue that should be
resected after chemotherapy; however, placing radiopaque clips at
the primary tumor site prior to chemotherapy can facilitate the
appropriate targeting of any residual nidus of disease after systemic
treatment.39 Investigators from the MD Anderson Cancer Center
have reported 5-year actuarial ipsilateral breast tumor RFS and
local-regional RFS rates of 95% and 91%, respectively, in patients
treated with breast conservation after chemotherapy.40 Factors correlating with ipsilateral breast tumor recurrence were clinical N2
or N3 disease, pathologic residual tumor size >2 cm, a multifocal
pattern of residual disease, and lymphovascular space invasion.
Axillary lymph node dissection (ALND) has been routinely
performed for the management of the axilla following neoadjuvant
chemotherapy. As sentinel lymph node (SLN) surgery has increasingly gained acceptance and reduced the need for ALND in earlystage node-negative patients, surgeons have begun to incorporate
this technique in the management of patients after chemotherapy.
This allows for one operative procedure at the completion of chemotherapy with the advantage of pathologic response data from
the breast and nodes. A number of institutions have demonstrated
feasibility of SLN surgery after neoadjuvant chemotherapy, and a
meta-analysis reported an overall accuracy rate of 94%, sensitivity of 88%, negative predictive value of 90%, and SLN identification rate of 90%. In addition to increasing the breast preservation
rates, the use of neoadjuvant chemotherapy appears to reduce the
need for completion ALND because fewer patients will have positive lymph nodes after chemotherapy.41
Answer: Neoadjuvant chemotherapy is an established approach for patients who present with inoperable and locally breast
cancer. It has the potential to improve surgical options in some
patients and is an important tool for evaluating novel therapeutics
in select subsets. The neoadjuvant approach allows for the pathologic assessment of systemic therapies on tumor biology which
can also provide information on expected outcomes for individual patients.
5. What is the role of the Adjuvant! Online computer program
in systemic treatment planning?
Adjuvant! Online (www.adjuvantonline.com) is a validated computer model designed to help physicians determine the 10-year
risk of recurrence and death due to breast cancer for an individual

PMPH_CH75.indd 608

patient. It also informs the clinician how specific interventions,


chemotherapy, hormone therapy, or no therapy are expected to
impact survival.3 These estimates of prognosis are largely based
on Surveillance, Epidemiology, and End-Results (SEER) registry
estimates. The primary factors used in this model include age,
comorbidity, estrogen receptor status, grade, tumor size, and
nodal status. HER2 status has not yet been added in the model
but is expected in future updated versions. The computer program
provides patient-friendly graphics to depict 10-year RFS and OS
estimates as well as estimates adjusted for chemotherapy or endocrine interventions.
The Adjuvant! Online program does have several limitations.
It is based on registry information and therefore relapse data and
cause of death may be inaccurate. It does not incorporate data for
HER2-positivity or for breast cancer in women younger than 35
years old. In some cases, especially when estimating recurrence
and breast cancer-related deaths in women with locally advanced
disease, the estimates of survival and recurrence despite extensive
systemic therapy may still be distressing to patients. However, the
overall concept of this design allows the physician and patient to
have a more interactive discussion regarding the risks and benefits of therapies. Several additions to this website have included
diagrams and other patient education tools that may also enhance
this dialog.
Answer: The Adjuvant! Online computer program is an effective web-based tool to inform discussions between physicians and
patients regarding expected recurrence rates due to breast cancer and the potential for improvements in outcome with current
interventions.
6. What molecular tools are available to assess the risk of recurrence from breast cancer?
It is widely believed that the use of anatomic staging and assessment of clinical and pathologic factors to guide adjuvant systemic
treatment decisions likely results in the undertreatment and overtreatment of some patients. In an effort to develop a more individualized approach for defining risk and determining therapy,
multigene tumor assays have been developed and tested for use in
breast cancer patients. The 21-gene recurrence score assay (Oncotype Dx) has been validated to quantify the risk of recurrence (low,
intermediate, and high) in patients with estrogen receptor-positive,
node-negative breast cancer.42 It also predicts the potential for
chemotherapy benefit.43 In a recent analysis of the 21-gene recurrence score in clinical practice, the knowledge of the recurrence
score altered treatment recommendations by the oncologist
in 31.5% of cases and 27% of patients changed their decision to
undergo treatment.44 There was a significant reduction in anxiety
and decisional conflict by incorporating the 21-gene recurrence
score results into decisions regarding systemic treatment. Another
multigene assay for determining prognosis is the Mammaprint
assay. The Mammaprint assay analyzes data from 70 genes to
develop a risk profile and provides a readout of low risk or high risk.
This tool can be used for risk assessment in patients with estrogen
receptor-positive and estrogen receptor-negative tumors.
Answer: Multigene assays are available to assist clinicians in
assessing the risk of recurrence in early-stage breast cancer. These
molecular tools can help to individualize treatment decisions and
may increase confidence in clinical decision-making and reduce
anxiety among patients.

5/22/2012 5:52:43 PM

Systemic Treatment Strategies for Early-Stage Breast Cancers

609

Clinical Question Summary


Question

Answer

1 What are the


indications
for adjuvant
chemotherapy in
breast cancer and
which chemotherapy
regimens should be
used?

The NCCN guidelines recommend consideration


of adjuvant chemotherapy for women with node
positive disease. For women with HER2-negative
tumors, and tumors that are >1 cm and node
negative or 0.6 to 1 cm with unfavorable features
such as high grade and lymphovascular invasion,
the NCCN guidelines recommend consideration
of further information that may impact treatment
decisions, such as the 21 gene recurrence score
assay (Oncotype Dx). Patients who can avoid
adjuvant chemotherapy or be considered for
adjuvant endocrine therapy alone are those patients
with hormone receptor-positive tumors <0.6 cm.

1-17

2 What are the


indications for
adjuvant endocrine
therapy in breast
cancer?

Tamoxifen should be given to women with hormone


receptor-positive tumors and who are pre- or
perimenopausal. Women who are postmenopausal
should receive an AI alone or in sequence
with tamoxifen for at least 5 years of adjuvant
endocrine therapy.

2, 18-25

3 What are the


indications for
adjuvant biologic
therapies in breast
cancer?

Trastuzumab therapy should be used in combination


with chemotherapy for all patients with HER2positive tumors that are node positive or node
negative and 0.6 cm in size.

26-33

4 What is the role


of neoadjuavnt
therapies in breast
cancer?

Neoadjuvant chemotherapy is an established


approach for patients who present with inoperable
and locally breast cancer. It has the potential to
improve surgical options in some patients and is an
important tool for evaluating novel therapeutics
in select subsets. The neoadjuvant approach
allows for the pathologic assessment of systemic
therapies on tumor biology, which can also
provide information on expected outcomes for
individual patients.

34-41

5 What is the role of


Adjuvant! Online
computer program
is systemic therapy
treatment planning?

The Adjuvant! Online computer program is an


effective web-based tool to inform discussions
between physicians and patients regarding
expected recurrence rates due to breast cancer
and the potential for improvements in outcome
with current interventions.

6 What molecular
tools are available
to assess the risk
of recurrence from
breast cancer?

Multigene assays are available to assist clinicians


in assessing risk of recurrence in early stage
breast cancer. These molecular tools can help to
individualize treatment decisions and may increase
confidence in clinical decision making and reduce
anxiety among patients.

42-44

REFERENCES
1. NCCN: National Comprehensive Cancer Network. Clinical
Guidelines in Oncology: Breast Cancer. http://www.nccn.org/
professionals/physician_gls/PDF/breast.pdf. Version 3.2010,
2010. Last accessed 3/1/2011.
2. Early Breast Cancer Trialists Collaborative Group. Effects of
chemotherapy and hormonal therapy for early breast cancer on

PMPH_CH75.indd 609

Levels of Grade of
References
Evidence Recommendations

recurrence and 15-year survival: An overview of the randomised


trials. Lancet. 2005;365:1687-1717.
3. Ravdin PM, Siminoff LA, Davis GJ, et al. Computer program to
assist in making decisions about adjuvant therapy for women
with early breast cancer. J Clin Oncol. 2001;19:980-991.
4. Ross JS, Hatzis C, Symmans WF, et al. Commercialized multigene predictors of clinical outcome for breast cancer. Oncologist.
2008;13:477-493.

5/22/2012 5:52:43 PM

610

Surgery: Evidence-Based Practice

5. Paik S, Tang G, Shak S, et al. Gene expression and benefit of


chemotherapy in women with node-negative, estrogen receptor
positive breast cancer. J Clin Oncol. 2006;24:3726-3734.
6. Hutchins LF, Green SJ, Ravdin PM, et al. Randomized, controlled
trial of cyclophosphamide, methotrexate, and fluorouracil versus cyclophosphamide, doxorubicin, and fluorouracil with and
without tamoxifen for high-risk, node-negative breast cancer:
Treatment results of Intergroup Protocol INT-0102. J Clin Oncol.
2005;23:8313-8321.
7. Fisher B, Brown AM, Dimitrov NV, et al. Two months of doxorubicin-cyclophosphamide with and without interval reinduction therapy compared with 6 months of cyclophosphamide,
methotrexate, and fluorouracil in positive-node breast cancer
patients with tamoxifen-nonresponsive tumors: Results from the
National Surgical Adjuvant Breast and Bowel Project B-15. J Clin
Oncol. 1990;8:1483-1496.
8. Henderson IC, Berry DA, Demetri GD, et al. Improved outcomes from adding sequential paclitaxel but not from escalating doxorubicin dose in an adjuvant chemotherapy regimen for
patients with node-positive primary breast cancer. J Clin Oncol.
2003;21:976-983.
9. Mamounas EP, Bryant J, Lembersky B, et al. Paclitaxel after
doxorubicin plus cyclophosphamide as adjuvant chemotherapy
for node-positive breast cancer: Results from NSABP B-28. J Clin
Oncol. 2005;23:3686-3696.
10. Jones SE, Savin MA, Holmes FA, et al. Phase III trial comparing
doxorubicin plus cyclophosphamide with docetaxel plus cyclophosphamide as adjuvant therapy for operable breast cancer. J
Clin Oncol. 2006;24:5381-5387.
11. Sparano JA WM, Martino S, et al. Phase III study of doxorubicin-cyclophosphamide followed by paclitaxel or docetaxel given
every 3 weeks or weekly in operable breast cancer: Results of
Intergroup Trial E1199. Proc Am Soc Clin Oncol. 2007;25:6s.
12. Henderson I. Can we abandon anthracyclines for early breast
cancer patients? Oncology. 2011;25:115-127.
13. Gianni L, Norton L, Wolmark N, et al. Role of anthracyclines
in the treatment of early breast cancer. J Clin Oncol. 2009;27:
4798-4808.
14. Trudeau M, Charbonneau F, Gelmon K, et al. Selection of adjuvant chemotherapy for treatment of node-positive breast cancer.
Lancet Oncol. 2005;6:886-898.
15. Pinder MC, Duan Z, Goodwin JS, et al. Congestive heart failure
in older women treated with adjuvant anthracycline chemotherapy for breast cancer. J Clin Oncol. 2007;25:3808-3815.
16. Patt DA, Duan Z, Fang S, et al. Acute myeloid leukemia after
adjuvant breast cancer therapy in older women: Understanding
risk. J Clin Oncol. 2007;25:3871-3876.
17. Goodwin PJ, Ennis M, Pritchard KI, et al. Risk of menopause
during the first year after breast cancer diagnosis. J Clin Oncol.
1999;17:2365.
18. Albain KS, Barlow WE, Ravdin PM, et al. Adjuvant chemotherapy and timing of tamoxifen in postmenopausal patients with
endocrine-responsive, node-positive breast cancer: A phase 3,
open-label, randomised controlled trial. Lancet. 2009;374:
2055-2063.
19. Burstein HJ, Griggs JJ, Prestrud AA, et al. American society of
clinical oncology clinical practice guideline update on adjuvant
endocrine therapy for women with hormone receptor-positive
breast cancer. J Oncol Pract. 2010;6:243-246.
20. The Arimidex, Tamoxifen, Alone or in Combination (ATAC)
Trialists Group. Effect of anastrozole and tamoxifen as adjuvant
treatment for early-stage breast cancer: 100-month analysis of
the ATAC trial. Lancet Oncol. 2008;9:45-53.

PMPH_CH75.indd 610

21. Crivellari D, Sun Z, Coates AS, et al. Letrozole compared


with tamoxifen for elderly patients with endocrine-responsive
early breast cancer: The BIG 1-98 Trial. J Clin Oncol. 2008;26:
1972-1979.
22. Gnant M, Mlineritsch B, Schippinger W, et al. Endocrine therapy plus zoledronic acid in premenopausal breast cancer. N Engl
J Med. 2009;360:679-691.
23. Goss PE, Ingle JN, Pater JL, et al. Late extended adjuvant treatment with letrozole improves outcome in women with earlystage breast cancer who complete 5 years of tamoxifen. J Clin
Oncol. 2008;26:1948-1955.
24. Rea D, Hasenburg A, Seynaeve C, et al. Five years of exemestane as initial therapy compared to 5 years of tamoxifen followed
by exemestane: The TEAM Trial, a prospective, randomized,
phase iii trial in postmenopausal women with hormone-sensitive early breast cancer. San Antonio Breast Cancer Symposium.
2009;Abstract #11.
25. Kelly CM, Juurlink DN, Gomes T, et al. Selective serotonin
reuptake inhibitors and breast cancer mortality in women
receiving tamoxifen: A population based cohort study. Br Med J.
2010;340:1-8.
26. Hudis CA. Trastuzumab Mechanism of action and use in clinical practice. N Engl J Med. 2007;357:39-51.
27. Joensuu H, Kellokumpu-Lehtinen P-L, Bono P, et al. Adjuvant
docetaxel or vinorelbine with or without trastuzumab for breast
cancer. N Engl J Med. 2006;354:809-820.
28. Piccart-Gebhart MJ, Procter M, Leyland-Jones B, et al. Trastuzumab after adjuvant chemotherapy in HER2-positive breast
cancer. N Engl J Med. 2005;353:1659-1672.
29. Romond EH, Perez EA, Bryant J, et al. Trastuzumab plus adjuvant chemotherapy for operable HER2-positive breast cancer. N
Engl J Med. 2005;353:1673-1684.
30. Slamon D, Eiermann W, Robert N, et al. BCIRG 006: Phase III
trial comparing AC-T with AC-TH and with TCH in the adjuvant treatment of HER-2-amplified early breast cancer patients:
Th ird planned efficacy analysis. San Antonio Breast Cancer
Symposium. 2009:Abstract #62.
31. Smith I, Procter M, Gelber RD, et al. 2-year follow-up of trastuzumab after adjuvant chemotherapy in HER2-positive breast
cancer: A randomised controlled trial. Lancet. 2007;369:29-36.
32. Buzdar AU, Ibrahim NK, Francis D, et al. Significantly higher
pathologic complete remission rate after neoadjuvant therapy with
trastuzumab, paclitaxel, and epirubicin chemotherapy: Results of
a randomized trial in human epidermal growth factor receptor
2-positive operable breast cancer. J Clin Oncol. 2005;23:3676-3685.
33. Gonzalez-Angulo AM, Litton JK, Broglio KR, et al. High risk of
recurrence for patients with breast cancer who have human epidermal growth factor receptor 2-positive, node-negative tumors
1 cm or smaller. J Clin Oncol. 2009;27:5700-5706.
34. Fisher B, Brown A, Mamounas E, et al. Effect of preoperative
chemotherapy on local-regional disease in women with operable
breast cancer: Findings from National Surgical Adjuvant Breast
and Bowel Project B-18. J Clin Oncol. 1997;15:2483-2493.
35. Mauri D, Pavlidis N, Ioannidis JP. Neoadjuvant versus adjuvant
systemic treatment in breast cancer: A meta-analysis. J Natl Cancer Inst. 2005;97:188-194.
36. van der Hage JA, van de Velde CJH, Julien J-P, et al. Preoperative
chemotherapy in primary operable breast cancer: Results from
the European Organization for Research and Treatment of Cancer trial 10902. J Clin Oncol. 2001;19:4224-4237.
37. Symmans WF, Peintinger F, Hatzis C, et al. Measurement of
residual breast cancer burden to predict survival after neoadjuvant chemotherapy. J Clin Oncol. 2007;25:4414-4422.

5/22/2012 5:52:43 PM

Systemic Treatment Strategies for Early-Stage Breast Cancers

38. Hess KR, Anderson K, Symmans WF, et al. Pharmacogenomic


predictor of sensitivity to preoperative chemotherapy with paclitaxel and fluorouracil, doxorubicin, and cyclophosphamide in
breast cancer. J Clin Oncol. 2006;24:4236-4244.
39. Oh JL, Nguyen G, Whitman GJ, et al. Placement of radiopaque
clips for tumor localization in patients undergoing neoadjuvant chemotherapy and breast conservation therapy. Cancer.
2007;110:2420-2427.
40. Chen AM, Meric-Bernstam F, Hunt KK, et al. Breast conservation after neoadjuvant chemotherapy: the M.D. Anderson cancer
center experience. J Clin Oncol. 2004;22:2303-2312.
41. Hunt KK, Yi M, Mittendorf EA, et al. Sentinel lymph node surgery after neoadjuvant chemotherapy is accurate and reduces the

PMPH_CH75.indd 611

611

need for axillary dissection in breast cancer patients annals of


surgery. Ann Surg. 2009;250(4):10.
42. Paik S, Shak S, Tang G, et al. A multigene assay to predict recurrence of tamoxifen-treated, node-negative breast cancer. N Engl J
Med. 2004;351:2817-2826.
43. Paik S, Tang G, Shak S, et al. Gene expression and benefit of chemotherapy in women with node-negative estrogen receptor-positive breast cancer. J Clin Oncol 2006;24:
3726-3734.
44. Lo S, Mumby PB, Norton J, et al. Prospective multicenter study
of the impact of the 21-gene recurrence score assay on medical
oncologist and patient adjuvant breast cancer treatment selection. J Clin Oncol. 2010;28:1671-1676.

5/22/2012 5:52:43 PM

CHAPTER 76

In Situ Carcinoma of the Breast:


Ductal and Lobular Carcinoma
Jane E. Mndez

INTRODUCTION

Characteristically, LCIS is multifocal and bilateral in a large proportion of cases. Over 50% of patients diagnosed with LN contain
multiple foci in the ipsilateral breast and about 30% of cases will
have further LCIS in the contralateral breast.8,9 The most important, and the most difficult, differential diagnosis of LCIS is from
low-nuclear-grade, solid DCIS. Correct identification is essential
as there are different management implications. However, distinction of LCIS from low-grade solid DCIS can be very difficult
because morphologically they can be strikingly similar.10

Ductal carcinoma in situ (DCIS) is a noninvasive breast cancer


that encompasses a wide spectrum of diseases ranging from lowgrade lesions that are nonlife threatening to high-grade lesions
that may harbor foci of invasive breast cancer. DCIS is typically
classified according to architectural pattern, tumor grade, and the
presence or absence of comedo necrosis.1 The incidence of DCIS
has increased dramatically since the early 1970s largely attributed to the widespread use of mammography. Before mammography was common, DCIS was rare representing less than 1% of
all breast cancer. Today, DCIS is common representing 27% of
newly diagnosed breast cancer cases and as many as 30% to 50%
of breast cancer cases diagnosed by mammography. In 2009, more
than 67,000 new instances of DCIS were diagnosed in the United
States alone, making noninvasive breast cancer the fourth most
commonly diagnosed malignancy in women.2
The first description of lobular carcinoma in situ (LCIS) was
by Foote and Stewart in 1941.3 There are no specific clinical abnormalities that a patient will present with, in particular no palpable
mass and usually no associated microcalcifications on mammography. Therefore, the diagnosis of LCIS is often made as an incidental, microscopic finding in breast biopsy performed for other
indications. For these reasons, many asymptomatic women go
undiagnosed, and the true incidence of LCIS in the general population remains unknown. The incidence of LCIS in otherwise
benign breast biopsy is between 0.5% and 3.8%.4,5
The term atypical lobular hyperplasia (ALH) was subsequently
introduced to describe morphologically similar but less welldeveloped lesions. Lobular neoplasia (LN) was a term introduced by
Haagensen in 1978 to cover the full range of proliferation, including both ALH and LCIS within the spectrum.6 (LAK) ALH and
LCIS have since become well-established histopathologic entities
in the classification of breast neoplasia, but it has become clear over
the past 60 years that they are not precursor lesions for invasive
carcinoma in the same way as high-grade DCIS of comedo type.7
A diagnosis of ALH/LCIS today is often seen as a risk indicator for subsequent carcinoma rather than a true precursor.

1. What is the appropriate surgical management for DCIS?


DCIS is a heterogeneous disease associated with high rates of longterm, disease-free survival (9698%) when treated with available
therapies. Given the heterogeneity, there is considerable controversy regarding how best to surgically manage patients with
DCIS.11 Current management options for DCIS include mastectomy, breast-conserving surgery (BCS), or BCS followed by whole
breast radiation therapy (BCS + RT).
Only one trial has compared mastectomy with breast conservation for patients with DCIS, and the data were only incidentally accrued.12 The National Surgical Adjuvant Breast Project
(NSABP) performed protocol B-06, a prospective randomized
trial for patients with invasive breast cancer with three treatment
arms: total mastectomy, excision of the tumor plus radiation, or
excision alone. A subgroup of 78 patients were found to have pure
DCIS without evidence of invasion. After 83 months of follow-up,
the percent of patients with local recurrences were 0% for mastectomy, 7% for excision plus radiation, and 43% for excision alone.
Despite the differences in rates of local recurrence, there was no
difference among the three groups in terms of breast cancerspecific survival. (Level 2b evidence)
Clinical factors associated with increased risk of local recurrence following breast-conserving treatment for DCIS are symptomatic presentation (palpable mass) and young age at diagnosis.
To further evaluate the impact of age at diagnosis on the clinical
and pathologic features of DCIS, the influence of age on outcome
after lumpectomy and RT, and the impact of age on outcome after
mastectomy, Vicini and Recht conducted a search of MEDLINE
612

PMPH_CH76.indd 612

5/22/2012 5:53:15 PM

In Situ Carcinoma of the Breast: Ductal and Lobular Carcinoma

and CancerLit databases and reviewed the available studies.13 DCIS


in younger patients more frequently contained adverse prognostic pathologic factors and extended over a greater distance in the
breast than in older patients. In series with adequate follow-up,
younger patients treated with lumpectomy and RT had a significantly higher rate of local recurrence than older patients, especially for invasive local recurrences. Some studies have suggested
that careful attention to margin status and excising larger volumes
of tissue can reduce this difference substantially. No available data
showed that younger patients have better long-term cancer-free
survival rates if treated by mastectomy rather than by lumpectomy and RT. On the basis of this review, Vicini et al. concluded
that successful treatment of younger patients with DCIS with
lumpectomy and RT requires careful attention to patient evaluation, selection, and surgical technique. Age at diagnosis should
not be a contraindication to breast-conserving therapy.
There is no consensus on what constitutes an adequate surgical margin in patients receiving BCS and postoperative RT for
DCIS. Inadequate margins may result in high local recurrence,
and excessively large resections may lead to poor cosmetic outcome without oncologic benefit. Dunne et al.14 conducted a comprehensive search for published trials that examined outcomes
after adjuvant RT after BCS for DCIS was performed using MEDLINE and cross-referencing available data. Reviews of each study
were conducted, and data were extracted. Primary outcome was
ipsilateral breast tumor recurrence (IBTR) related to surgical margins. A total of 4660 patients were identified from trials examining BCS and RT for DCIS. Patients with negative margins were
significantly less likely to experience recurrence than patients
with positive margins after RT (odds ratio [OR] = 0.36; 95% CI
0.270.47). A negative margin significantly reduced the risk of
IBTR when compared with a close (OR = 0.59; 95% CI 0.420.83)
or unknown margin (OR = 0.56; 95% CI 0.360.87). When specific margin thresholds were examined, a 2-mm margin was superior to a margin less than 2 mm (OR = 0.53; 95% CI 0.260.96);
however, we saw no significant difference in the rate of IBTR with
margins between 2 mm and more than 5 mm (OR = 1.51; 95% CI
0.515.0; p > .05).
The authors concluded that negative surgical margins should
be obtained after BCS for DCIS. A margin threshold of 2 mm
seems to be as good as a larger margin when BCS for DCIS is combined with RT.
Answer: Negative surgical margin is a critical component
of breast-conserving surgery in patients with DCIS. (Grade B
recommendation)
2. What is the role of RT in the treatment of DCIS?
The main risk of inadequately removing all the DCIS is either a
recurrence of DCIS or the development of invasive breast cancer
at a later time with the risk that this can progress to metastatic
disease. Giving RT after BCS is thought to reduce the risk of developing recurrent disease (either DCIS or invasive breast cancer).
Strong Level 1 evidence from four randomized prospective trials
on the value of RT after BCS for DCIS.15 These trialsNSABP and
Bowel Project-B17 (NSABP-B17),16,17 European Organisation for
Research and Treatment of Cancer 10853 (EORTC 10853),18 the
United Kingdom, Australia, and New Zealand trial (UKAusNZ),19
and the Swedish (SweDCIS) trial20have consistently demonstrated a risk reduction of approximately 50% in IBTR among
women randomly assigned to the BCS plus RT arms.

PMPH_CH76.indd 613

613

Goodwin et al.15 conducted a search of Cochrane Breast Cancer Group Specialised Register (January 2008), Cochrane Central
Register of Controlled Trials (CENTRAL) (The Cochrane Library
2008, Issue 1), MEDLINE (February 2008), and EMBASE (February 2008) of the randomized clinical trials (RCTs) of breastconserving surgery with and without radiotherapy in women
at first diagnosis of pure DCIS (no invasive disease present). Four
RCTs involving 3925 women were identified and included in this
review. All were high quality with minimal risk of bias. Three trials compared the addition of RT with BCS. One trial was a two-bytwo factorial design comparing the use of RT and tamoxifen, each
separately or together, in which participants were randomized in at
least one arm. Analysis confirmed a statistically significant benefit
from the addition of radiotherapy on all ipsilateral breast events
(hazards ratio (HR) 0.49; 95% CI 0.410.58, p < .00001), ipsilateral
invasive recurrence (HR 0.50; 95% CI 0.320.76, p = .001) and ipsilateral DCIS recurrence (HR 0.61; 95% CI 0.390.95, p = .03). All
the analyzed subgroups benefited from addition of radiotherapy.
No significant long-term toxicity from radiotherapy was found.
From their review, Goodwin et al. concluded that their findings
confirmed the benefit of adding radiotherapy to breast-conserving
surgery for the treatment of all women diagnosed with DCIS.
Although the randomized trials have demonstrated an overall reduction in IBTR in all patients with DCIS treated with BCS +
RT, the absolute benefit of whole breast radiotherapy may be
smaller in subsets of patients based on their age, tumor size, histology, grade, and margin status.21 To date, subset analyses have not
been able to identify any patient or tumor characteristic groups in
which radiation could be omitted for DCIS. In a series published
by Silverstein et al., patients with small lesions, with favorable histologies, and of low to intermediate grade with widely negative
margins (>1 cm) treated by BCS alone reported an IBTR rate as
low as 6% at 5 years.22 There are, however, conflicting retrospective data demonstrating higher local relapse rates with the omission of RT in even these favorable patient groups.23
In an effort to address this issue, a trial initiated by the Eastern Cooperative Oncology Group (ECOG) enrolled patients with
DCIS into ECOG 5194, a single-arm, multi-institutional, prospective study of observation after BCS, with the end points of ipsilateral and contralateral breast relapse.24 Patient eligibility included
low- or intermediate-grade (LIG) DCIS lesions measuring from 0.3
to 2.5 cm in size with margins 3 mm, or high-grade (HG) DCIS
lesions measuring from 0.3 to 1.0 cm in size with margins 3 mm.
With a median follow-up of 6.2 years, they reported 5- and 7-year
IBTR rates of 6.1% and 10.5% in the LIG cohort and 15.3% and
18% in the HG cohort, respectively. The authors concluded that
the LIG cohort had an acceptable rate of IBTR, although they
acknowledged that further follow-up is warranted and that the
HG cohort had an unacceptably high relapse rate of 15.3% at
5 years, suggesting BCS alone may be inadequate treatment in this
subgroup of patients. (Level 1b evidence)
Motwani et al.21 set to evaluate the outcomes in a large cohort
of DCIS patients who met the eligibility criteria for ECOG 5194
(E5194), but were treated with BCS and adjuvant whole breast
radiotherapy, to compare the ipsilateral and contralateral breast
tumor recurrence in these patients treated with radiation with
those treated with observation alone in the ECOG study. A total of
263 patients with DCIS were treated between 1980 and 2009 who
met the enrollment criteria for E5194: (1) LIG with size >0.3 cm
but <2.5 cm and margins >3 mm (n = 196), or (2) HG, size <1 cm

5/22/2012 5:53:15 PM

614

Surgery: Evidence-Based Practice

and margins >3 mm (n = 67). All patients were treated with


lumpectomy and whole breast RT with a boost to a median total
tumor bed dose of 6400 cGy. With a follow-up time of 6.9 years,
the 5- and 7-year IBTR for the LIG cohort in this study were 1.5%
and 4.4% compared with 6.1% and 10.5% in E5194, respectively.
The 5- and 7-year IBTR for the HG cohort were 2.0% and 2.0% in
this study compared with 15.3% and 18% in E5194, respectively.
Motwani et al. concluded that adjuvant RT reduces the risk of an
IBTR by more than 70% for both LIG and HG DCIS compared
with the results seen in ECOG 5194. Taking into consideration
the significant local failure after 5 years of follow-up, the authors
recommend both LIG and HG DCIS patients be considered for
whole breast radiation until longer follow-up of ECOG 5194 is
reported and a low-risk group of DCIS patients can be identified.
(Level 2b evidence)
Recent studies also demonstrate favorable results in selected
patients with BCS followed by partial breast irradiation. Goyal
et al. studied the IBTR in DCIS patients treated in the American
Society of Breast Surgeons MammoSite Breast Brachytherapy Registry Trial who met the criteria for E5194 treated with local excision and adjuvant accelerated partial breast irradiation (APBI).25
A total of 194 patients with DCIS were treated between 2002 and
2004 in the Mammosite registry trial; of these, 70 patients met
the enrollment criteria for E5194: (1) LIG-pathologic size >0.3
but <2.5 cm and margins 3 mm (n = 41) or (2) HG-pathologic
size <1 cm and margins 3 mm (n = 29). All patients were treated
with lumpectomy followed by adjuvant APBI using MammoSite.
Median follow-up was 52.7 months (range 088.4). In the LIG
cohort, the 5-year IBTR was 0%, compared with 6.1% at 5 years
in E5194. In the HG cohort, the 5-year IBTR was 5.3%, compared
with 15.3% at 5 years in E5194. The overall 5-year IBTR was 2%,
and there were no cases of elsewhere or regional failures in the
entire cohort. The 5-year contralateral breast event rate was 0%
and 5.6% in LIG and HG patients, respectively (compared with
3.5% and 4.2%, respectively, in E5194). This study found that
patients who met the criteria of E5194 treated with APBI had
extremely low rates of recurrence. (Level 1b evidence)
Answer: Radiation is an important adjuvant treatment
after breast-conserving surgery in patients with DCIS. (Grade A
recommendation)
3. What is the role for axillary sentinel lymph node biopsy in
DCIS?
Only 1.5 % of patients with DCIS have axillary node metastases
identified by conventional pathologic assessment.26,27 By longstanding consensus, there is no role for ALND in DCIS.28 In five
series of SLN biopsy for patients with DCIS, SLN metastases were
found in 3% to 12% of cases,29-33 but even for series of pure DCIS
SLN metastases were present in 3% to 5%.32,34
The current American Society of Clinical Oncology guidelines for SLNB in early-stage breast cancer recommend SLNB for
patients undergoing mastectomy for DCIS because of the technical
difficulty of performing SLNB after mastectomy, but the routine
use of SLNB in patients who undergo BCS is not recommended.
In circumstances of high-risk DCIS or large tumors, SLNB should
be considered on a case-by-case basis.35 The 2001 Sentinel Lymph
Node Biopsy Consensus Conference recommendations were similar. However, the Consensus generated a clearer statement recommending the use of SNB in patients who have DCIS with the

PMPH_CH76.indd 614

presence of any type of invasive breast cancer but against its use in
patients with DCIS without any type of invasive breast cancer.36
Answer: The routine use of SLNB in all patients with pure
DCIS is not warranted. For patients with proven invasive or
microinvasive disease with DCIS, SLNB is supported. In those
who undergo mastectomy for DCIS, SLNB is recommended at
the time of mastectomy. A case-by-case decision should be made
for the use of SLNB in patients who have high-risk DCIS or large
tumors. (Grade B recommendation)
4. What is the appropriate surgical management for LCIS?
The surgical management of LCIS has to be addressed considering
two different scenarios: (1) when LCIS pathology is found on core
biopsy and (2) when LCIS is found incidentally in a lumpectomy
specimen or at the resection margin of a patient with known invasive carcinoma.
The significance of LIN diagnosed at core needle biopsy (CNB)
is unclear. To determine the incidence of malignancy (invasive
carcinoma or DCIS) in patients diagnosed with LN (B3) on CNB
of breast lesions, Hussain et al.37 reviewed the published literature on Medline, Embase, OVID-database, and reference lists to
identify and review all English-language articles addressing the
management of LN diagnosed on CNB. Of 1229 LN diagnosed
on CNB, 789 (64%) underwent surgical excision; 211 (27%) of
excisions contained either DCIS or invasive disease; 280 of the
excision specimens were classified as ALH, 241 as LCIS, 22 as
pleomorphic LCIS (PLCIS), and 246 unspecified LN on the original CNB. After surgical excision, 19% of the ALH cases, 32% of
the LCIS cases, and 41% of the PLCIS cases contained malignancy; 29% of the unspecified LNs were upgraded to malignancy.
The higher incidence of malignancy within excision specimens
for LCIS and PLCIS compared with ALH was significant (p < .04
and .003, respectively). The authors concluded that there is a significant underestimation of malignancy in patients diagnosed
with breast LN on CNB and recommended that all patients diagnosed with LN on CNB should be considered for surgical excision
biopsy. (Level 2b evidence)
In contrast, Bowman et al. performed a PubMed search to
identify all published addressing management of LN diagnosed at
CNB.38 The 19 studies that form the basis of this report included a
total of 504 subjects. Limitations of the reviewed studies included
their retrospective nature, small number of subjects, inconsistent
inclusion criteria, and selection bias regarding surgical excision.
Based on the reviewed literature, the authors concluded that it
was difficult to reach a firm evidence-based conclusion regarding
optimal management of LN diagnosed at CNB. At this time, the
available retrospective literature does not support a routine excision for all patients. (Level 2b evidence)
To address the second scenario, when LCIS is found incidentally in the lumpectomy or at the resection margin of a patient
with known invasive carcinoma, Ciocca et al. studied 2894 patients
treated with BCT for DCIS, stage I or II breast cancer.39 Group A
had 290 patients with LCIS in the lumpectomy; 84 had LCIS at the
final margin. Group B included 2604 patients with no evidence
of LCIS. The histologic distribution of tumor types in group A
was lobular in 47.2%, ductal in 34.5%, DCIS in 11.4%, and other
invasive histologies in 6.9%, compared with 4.1%, 76.3%, 13.6%,
and 6.0% for group B, respectively (p < .0001). The crude rate of LR
was 4.5% in group A and 3.8% in group B (p = NS); 5- and 10-year

5/22/2012 5:53:15 PM

In Situ Carcinoma of the Breast: Ductal and Lobular Carcinoma

actuarial LR rates for LCIS at the margin were 6% and 6%, 1% and
15% for LCIS present but not at the margin, and 2% and 6% for no
LCIS (p = NS), for groups A and B, respectively. LCIS, either in the
specimen or at the margin, was not significantly associated with
LR. Presence of LCIS, even at the margin, in BCT specimens does
not have an impact on LR. Re-excision is not indicated if LCIS is
present or close to margin surfaces. Hence, when LCIS is found
incidentally at the surgical margin, re-excision is not necessary.
(Level 2b evidence)
Answer: The necessity of routine surgical excision for LCIS
remains controversial due to conflicting opinions on the biologic
behavior of lobular lesions, diagnostic confusion regarding histopathology, and uncertainty of their association with high-risk
lesions. (Grade C recommendation)
5. What is the role for chemoprevention in the treatment of
LCIS?
Although it is clear that LCIS is not an obligate precursor to invasive
lobular carcinoma, many studies have shown that a proportion

615

of women with LCIS go on to develop invasive carcinoma, with a


risk of 6.9 times to about 12 times that of women without LCIS.6,40
The time taken to develop invasive cancer after diagnosis of LCIS
is unclear. In the NSABP Breast Cancer Prevention Trial (BCPT),
the reduction in risk of noninvasive breast cancer was 50%.41 The
BCPT randomized 13,000 healthy women at increased risk for
breast cancer to receive 20 mg of tamoxifen daily or a placebo for
a 5-year period.42 Through 7 years of follow-up, the cumulative
incidence of noninvasive breast cancer among the placebo group
was 15.8 per 1000 women versus 10.2 per 1000 women in the
tamoxifen group.43 The cumulative rate of invasive breast cancer
was reduced 43% in the tamoxifen group and the cumulative rate
of noninvasive carcinoma was reduced 37%. The BCPT revealed
that significant net benefit accrues for women with a diagnosis of
either LCIS or atypical hyperplasia who take tamoxifen. Among
women with a history of LCIS, the reduction in risk was 46%.
(Level 1b evidence)
Answer: Chemoprevention is recommended for patients at
high risk of breast cancer including patients with LCIS. (Grade A
recommendation)

Clinical Question Summary


Question

Answer

1 What is the appropriate surgical


management for DCIS?

Negative surgical margin is a critical component of


breast conserving surgery in patients with DCIS.

12-14

2 What is the role of RT in the


treatment of DCIS?

Radiation is an important adjuvant treatment after


breast-conserving surgery in patients with DCIS.

15-20

3 What is the role for axillary


sentinel lymph node biopsy in
DCIS?

The routine use of SLNB in all patients with pure


DCIS is not warranted. For patients with proven
invasive or microinvasive disease with DCIS, SLNB
is supported. In those who undergo mastectomy
for DCIS, SLNB is recommended at the time of
mastectomy. A case-by-case decision should be
made for the use of SLNB in patients who have
high-risk DCIS or large tumors.

35-36

4 What is the appropriate surgical


management for LCIS?

The necessity of routine surgical excision for LCIS


remains controversial due to conflicting opinions
on the biologic behavior of lobular lesions,
diagnostic confusion regarding histopathology,
and uncertainty of their association with high-risk
lesions.

37-39

5 What is the role for


chemoprevention in the
treatment of LCIS?

Chemoprevention is recommended for patients at


high risk of breast cancer including LCIS.

41-43

REFERENCES
1. Virnig BA, Tuttle T, Shamliyan T, et al. Ductal carcinoma in situ
of the breast: A systematic review of incidence, treatment and
outcomes. J Natl Cancer Inst. 2010;102(3):170-178.
2. Jemal A, Siegel R, Ward E, et al. Cancer Statistics 2009. CA Cancer J Clin. 2009;59:225-249.
3. Foote FW, Jr., Stewart FW. Lobular carcinoma in situ. A rare
form of mammary cancer. Am J Pathol. 1941;17:491-496.
4. Fulford LG, Reis-Filho JS, Lakhani SR. Lobular in situ neoplasia.
Curr Diagn Pathol. 2004;10:183-192.

PMPH_CH76.indd 615

Grade of
References
Recommendation

5. Page DL, Kidd TE, Jr., Dupont WD, et al. Lobular neoplasia of
the breast: Higher risk for subsequent invasive cancer predicted
by more extensive disease. Hum Pathol. 1991;22:1232-1239.
6. Haagensen CD, Lane N, Lattes R, et al. Lobular neoplasia of the
breast. Cancer. 1978;42:737-769.
7. Skinner KA, Silverstein MJ. The management of ductal carcinoma in situ of the breast. Endocr Relat Cancer. 2001;8:33-45.
8. Urban JA, Bilaterality of cancer of the breast. Biopsy of the opposite breast. Cancer. 1967;20:1867-1870.
9. Rosen PP, Braun DW, Jr., Lynholm B, et al. Lobular carcinoma in
situ of the breast: Preliminary results of treatment by ipsilateral

5/22/2012 5:53:15 PM

616

10.

11.
12.

13.

14.

15.

16.

17.

18.

19.

20.

21.

22.

23.

24.

25.

Surgery: Evidence-Based Practice

mastectomy and contralateral breast biopsy. Cancer. 1981;47:


813-819.
Simpson PT, Gale T, Fulford LG, et al. Review the diagnosis and
management of pre-invasive breast disease: Pathology of atypical
lobular hyperplasia and lobular carcinoma in situ. Breast Cancer
Res. 2003;5(5):258-262.
Leonard GD, Swain SM. Ductal Carcinoma in situ, complexities
and challenges. J Natl Cancer Inst. 2004;96(12):906-920.
Fisher E, Sass R, Fisher B. Pathologic findings from the National
Surgical Adjuvant Breast Project (Protocol 6). Intraductal Carcinoma (DCIS). Cancer. 1986;57:197-208.
Vicini FA, Recht A. Age at diagnosis and outcome for women
with ductal carcinoma-in-situ of the breast: A critical review of
the literature. J Clin Oncol. 2002;20(11):2736-2744.
Dunne C, Burke JP, Morrow M, et al. Effect of margin status on
local recurrence after breast conservation and radiation therapy
for ductal carcinoma in situ. J Clin Oncol. 2009;27(10):1615-1620.
Goodwin A, Parker S, Ghersi D, Wilcken N. Post-operative radiotherapy for ductal carcinoma in situ of the breasta systematic
review of the randomised trials. Breast. 2009;18(3):143-149.
Fisher B, Constantino J, Redmond C, et al. Lumpectomy compared with lumpectomy and radiation for the treatment of intraductal cancer. N Engl J Med. 1993;328:1581-1586.
Fisher B, Dignam J, Wolmark N, et al Lumpectomy and radiation
therapy for the treatment of intraductal breast cancer: Findings
from the National Surgical Adjuvant Breast and Bowel Project
B-17. J Clin Oncol. 1998;16:441-452.
Bijker N, Meijen P, PeterseJL, et al. Breast conserving treatment
with or without radiotherapy in ductal carcinoma in situ. Ten
year results of European Organization for Research and Treatment of Cancer randomized phase III trial 10853-a study by the
EORTC Breast Cancer Cooperative Group and EORTC Radiotherapy Group. J Clin Oncol. 2006;24:3381-3387.
Houghton J, Parker S, Ghersi D, et al. Radiotherapy and tamoxifen in women with completely excised ductal carcinoma in situ
of the UK, Australia and New Zealand: Randomised controlled
trial. Lancet. 2003:362:95-102.
Emdin SO, Granstrand B, Ringberg A, et al. SweDCIS: Radiotherapy after resection for ductal carcinoma in situ of the breast:
Results of a randomized trial in a population offered mammography screening. Acta Oncol. 2006:45;536-543.
Motwani SB, Goyal S, Moran MS, et al. Ductal carcinoma in situ
treated with breast conserving surgery and radiotherapy: A comparison with ECOG study 5194. Cancer. 2011;117:1156-1162.
Silverstein MJ, Lagios MD, Groshen S, et al. The influence of
margin width on local control of ductal carcinoma in situ of the
breast. N Engl J Med. 1999;340:1455-1461.
Wong JS, Kaelin CM, Troyan SL, et al. Prospective study of wide
excision alone for ductal carcinoma in situ of the breast. J Clin
Oncol. 2006;24:1031-1036.
Hughes LL, Wang M, Page DL, et al. Local excision alone without
irradiation for ductal carcinoma in situ of the breast: A trial of the
Eastern Oncology Group. J Clin Oncol. 2009;27(32):5319-5324.
Goyal S, Vicini F, Beitsch PD, et al. Ductal carcinoma in situ treated
with breast-conserving surgery and accelerated partial breast irradiation: Comparison of the Mammosite registry trial with intergroup study E5194. Cancer. 2011;117:1149-1155.

PMPH_CH76.indd 616

26. Cody HS. Sentinel lymph node biopsy for DCIS: Are we approaching consensus? Ann Surg Oncol. 2007;14(8): 2179-2181.
27. Leonard GD, Swain SM. Ductal carcinoma in situ, complexities
and challenges. JNCI Cancer Spectrum. 2004;96:906-920.
28. Schwartz GF, Solin LJ, Olivotto IA, et al. The consensus conference on the treatment of in situ ductal carcinoma of the breast,
April 2225, 1999. Breast J. 2000;6:4-13.
29. Pendas S, Dauway E, Giuliano AE, et al. Sentinel node biopsy in
duct carcinoma in situ patients. Ann Surg Oncol. 2000;7:15-20.
30. Klauber-DeMore N, Tan LK, Liberman L, et al. Sentinel lymph
node biopsy: Is it indicated in patients with high-risk ductal
carcinoma-in-situ and ductal carcinoma-in-situ with microinvasion? Ann Surg Oncol. 2000;7:636-642.
31. Cox CE, Nguyen K, Gray RJ, et al. Importance of lymphatic mapping in ductal carcinoma in situ (DCIS): Why map DCIS? Am
Surg. 2001;67:513-519.
32. Intra M, Veronesi P, Mazzarol G, et al. Axillary sentinel lymph
node biopsy in patients with pure ductal carcinoma in situ of the
breast. Arch Surg. 2003;138:309-313.
33. Yen TW, Hunt KK, Ross MI, et al. Predictors of invasive breast cancer in patients with an initial diagnosis of ductal carcinoma in situ:
A guide to selective use of sentinel lymph node biopsy in management of ductal carcinoma in situ. J Am Coll Surg. 2005;200:516-526.
34. Wilkie C, White L, Dupont E, et al. An update of sentinel lymph
node mapping in patients with ductal carcinoma in situ. J Am
Coll Surg. 2005;200:516-526.
35. Lyman GH, Giuliano AE, Somerfield MR, et al. American Society of Clinical Oncology guideline recommendations for sentinel lymph node biopsy in early-stage breast cancer. J Clin Oncol.
2005;23(30):7703-7720.
36. Schwartz GF, Giuliano AE, Veronesi U, et al. Proceedings of the
consensus conference on the role of sentinel lymph node biopsy
in carcinoma of the breast, April 1922, 2001, Philadelphia, Pennsylvania. Cancer. 2002;94(10):2532-2551.
37. Hussain M, Cunnick GH. Management of lobular carcinoma insitu and atypical lobular hyperplasia of the breastA review. Eur
J Surg Oncol. 2011;37:279-289.
38. Bowman K, Munoz A, Mahvi DM, et al. Lobular neoplasia diagnosed at core biopsy does not mandate surgical excision. J Surg
Res. 2007:142(2);275-280.
39. Ciocca RM, Li T, Freedman GM, et al. Presence of lobular carcinoma
in situ does not increase local recurrence in patients treated with
breast-conserving therapy. Ann Surg Oncol. 2008;15(8):2263-2271.
40. Andersen JA. Lobular carcinoma in situ. A long term follow up
in 52 cases. Acta Pathol Microbiol Scand. 1974;82:519-533.
41. Vogel VG, Constatino JP, Wickerham DL, et al. Carcinoma in
situ outcomes in National Surgical Adjuvant Breast and Bowel
Project Breast Cancer Chemoprevention Trials. J Natl Cancer Inst
Monogr. 2010;2010(41):181-186.
42. Fisher B, Constantino J, Wickerman DL, et al. Tamoxifen for prevention of breast cancer: Report of the National Surgical Adjuvant Breast and Bowel Project P-1 study. J Natl Cancer Inst. 1998;
90:1371-1378.
43. Fisher B, Constantino J, Wickerman DL, et al. Tamoxifen for the
prevention of breast cancer: Current status of the National Surgical Adjuvant Breast and Bowel Project P-1 study. J Natl Cancer
Inst. 2005;97:1652-1662.

5/22/2012 5:53:15 PM

CHAPTER 77

Male Breast Cancer


Jessica Keto and Paul Ian Tartter

INTRODUCTION

the VA study was 17 after adjustment of other medical conditions.


This increases their lifetime risk of breast cancer to 1.5%, still far
below the risk of a low-risk woman.
Gynecomastia was associated with breast cancer in the VA
study independent of obesity. Most likely, gynecomastia is a reflection of an abnormal hormonal milieu resulting in a high estrogen
to androgen ratio causing breast development, simply increasing
the amount of tissue at risk for malignancy.
The association of obesity with breast cancer risk noted in the
VA study was previously reported in men6,7 and, in women, high
body weight is an accepted risk and prognostic factor for breast
cancer.8 Obesity was found to be a significant risk factor for male
breast cancer in a casecontrol study conducted in Canada of 119
male breast cancer cases age-matched to 1905 controls identified
between 1994 and 1998.6 The risk of breast cancer increased with
increasing weight and increasing body mass index, with men over
90 kg having double the risk of men weighing less than 73 kg. In
Scandinavia, a similar casecontrol study of 468 male breast cancers and 780 controls noted a similar increased risk for men with
BMIs over 30.7 These findings have been attributed to increased
production of estrogen by adipose tissue. Serum estrogen in men
increases with age, whereas testosterone declines resulting in high
estrogen to testosterone ratios in obese men.
The same Scandinavian and Canadian casecontrol studies
referred to above noted that family history of breast cancer was
also a significant risk factor for the development of breast cancer
in men. Family history of breast cancer in a fi rst degree relative was associated with a 3.3 to 3.6 times higher risk of breast
cancer compared with men with no family history. Certainly,
some of these men with breast cancer were carriers of mutations
in BRCA1 or BRCA2.9 The majority of inherited breast cancers in men are related to the BRCA2 gene mutation, accounting for approximately 15% of male breast cancer.10 The lifetime
risk of breast cancer for a man carrying the BRCA2 mutation
is increased 80 to 100 times with 6.9% diagnosed by the age
of 80.4 The BRCA1 gene mutation is associated with a lifetime
risk of 1.8%, approximately 15 times the risk in men without a
mutation.11 Other cancers including prostate, pancreatic, colon,

The incidence of breast cancer in men in the United States has


more than doubled in the last 20 years from 900 cases in 1987 to
an estimated 1970 cases in 2010.1,2 Despite the dramatic increase in
this uncommon cancer, the mortality for men has declined from
33% to 20%. Breast cancer incidence has increased in women 18%
during the same interval and mortality for women has declined
from 25% to 19%. In 2010, male breast cancer will account for
0.25% of all cancers in men, 0.13% of cancer deaths in men, and
0.94% of all breast cancers. These observations are supported by
a recent analysis of SEER data comparing male and female breast
cancer incidence and mortality.3 They also observed that male
breast cancers present later in life, at median age of 67 compared
with 62 for women.
1. What are the Risk Factors for Male Breast Cancer?
Risk factors for male breast cancer can be divided into those that
are accepted, confirmed with good statistical information, and
those that are suspected risk factors for which the supporting data
are anecdotal, generally based on case studies. Confirmed risk
factors include Klinefelters syndrome, obesity, testicular disorders, family history of breast cancer, and mutation in BRCA2. The
largest study of risk factors for male breast cancer comes from the
analysis of the Patient Treatment File of the U.S. Veterans Affairs
medical system.4 Cases of male breast cancer (652) were identified
by ICD-9 codes among 4,501,578 patients hospitalized between
1969 and 1996. Additional medical conditions identified by ICD-9
codes and their frequency among breast cancer patients were compared with their frequency among patients without breast cancer.
Using this method, men with breast cancer had a significantly
higher likelihood of also being diagnosed with Klinefelters syndrome, gynecomastia, obesity, and orchitis/epidydimitis.
Klinefelters syndrome is due to an extra X chromosome present in an otherwise XY male.5 Men with Klinefelters have low
levels of testosterone and a high ratio of estrogens to androgens.
The relative risk of breast cancer among Klinefelters patients in
617

PMPH_CH77.indd 617

5/22/2012 5:53:45 PM

618

Surgery: Evidence-Based Practice

stomach, and melanoma have been associated with BRCA mutations. Due to the significant association for the BRCA gene
mutations with MBC, the diagnosis of breast cancer in a male is
an indication for genetic testing.
Radiation exposure is not proven to affect the risk of breast
cancer in men. Tumor registries for the survivors of the atomic
bomb in Japan were established in 1958 including 32,411 male
survivors with known radiation exposure.12 They were compared
with 10,491 male residents of Hiroshima and Nagasaki who were
not in the cities at the time of the bombings. There were 1.8 cases
of male breast cancer per 100,000 person years in the radiation
exposed men compared with 0.5 per 100,000 person years among
those not exposed. The small number of patients developing breast
cancer12 precludes statistical significance. A study of men exposed
to diagnostic and therapeutic ionizing radiation also observed
an increased risk of breast cancer which did not reach statistical
significance.13
Exogenous estrogen taken by male to female trans-sexuals or
to treat prostate cancer has also been associated with the development of male breast cancer.
Numerous other suspected but statistically unproven risk
factors for male breast cancer include germline mutations in the
PTEN tumor suppressor gene found in Cowdens syndrome,14 germline mutation in the Androgen Receptor gene thought to influence estrogen/androgen ratios,15 polymorphism in the CYP17 gene
which codes for an enzyme of estrogen and androgen synthesis,16
and mutation of the gene for CHEK2, a kinase which mediates
DNA repair.17
2. What is the most common presentation for Male Breast
Cancer?
Male breast cancer most commonly presents as a mass under the
nipple/areolar complex and occasionally in the upper outer quadrant. Small numbers of male breast cancers also present in the
same miscellaneous ways that breast cancers present in women:
nipple discharge, skin or nipple ulceration, palpable axillary mass
or with distant metastases.
The diagnosis of male breast cancer is often suspected from
physical examination alone: a hard, irregular mass is appreciated
under the nipple. Mammography may be helpful to differentiate
benign from malignant disease, but recent series indicate that
most, if not all, breast cancers in men are suspected on clinical
examination alone and mammography may be unnecessary in
men with clinical gynecomastia.18,19 Screening mammography
is not used in men because the yield would be exceedingly low.
When malignancy is suspected, a mammogram should be done
prior to core needle biopsy because hemorrhage from the core
needle biopsy will affect the mammographic appearance of the
lesion. The mammogram may be helpful to assess the extent of
disease. Tissue diagnosis is often possible with fine needle aspiration.20 When a definitive diagnosis cannot be made by fine needle
aspiration, ultrasound-guided core needle biopsy is indicated.21,22
The ultrasound confirms that the tissue removed comes from the
mass and the core needle is used because the volume of tissue
obtained gives a more reliable diagnosis than fine needle biopsy.
In addition, estrogen receptors and her2/neu can be measured on
the core needle biopsy. If the presentation is with excoriation of
the skin or the nipple, the diagnosis can be made with a punch
biopsy.

PMPH_CH77.indd 618

Breast MRI studies have been done in men with breast cancer, but it is not clear that this adds to the information obtained
by clinical examination, mammography, and ultrasound-guided
biopsy.23
If axillary nodes or other sites of distant disease are suspected, these should be biopsied for confirmation before proceeding to surgery. If the cancer is early, less than 4 cm, not invading
the skin, not fi xed to underlying muscle, and without evidence
of lymph node or distant metastases, there is no need for preoperative imaging with whole body PET/CT. If any of the foregoing
signs of advanced disease are present, distant metastases should
be ruled out.
3. What Histopathological Types are seen in Male Breast
Cancer?
Virtually, all breast cancers in men arise from ductal cells and
85% of cancers in men are infi ltrating ductal.24 All pathologic
subtypes of ductal cancers reported in women have been reported
in men. As men are not screened with mammography and the
majority of in situ ductal cancers are detected with mammography in women, in situ ductal cancer is much less frequently
seen in men. The incidence of invasive papillary carcinoma in
men is twice than that in women, although still only accounting
for 4% of male breast cancers. The absence of lobular cells in the
male breast has been attributed to the absence of high estrogen
levels needed for lobular differentiation. The few lobular cancers
reported in men have been associated with Klinefelters syndrome and other clinical situations associated with high estrogen/androgen ratios.
Breast cancers in men tend to be more poorly differentiated
than cancers in age-matched women, 25,26 although the majority of breast cancers in men are intermediate or low grade and
they are more frequently estrogen receptor and progesterone
receptor-positive. More than 80% are estrogen receptor-positive and more than 75% progesterone receptor-positive. In fact,
breast cancers in men resemble those found in postmenopausal
women, not surprising because 97% of men with breast cancer
are over 50.27
The rates of human epidermal growth factor receptor 2 (her2/
neu) overexpression in male breast cancer reported in the literature ranges from 1.7% to 15%28,29 compared with approximately
25% of breast cancers in women. SEER database statistics support lower her2/neu in cancers in men compared with cancers in
women, but this is not a universal finding.11
Breast cancers in men are diagnosed at more advance stage
than in women because men are not screened with mammography. As a consequence, ductal carcinoma in situ, a common
finding on mammography in women, is unusual in men. In addition, invasive breast cancers in men are generally more advanced
because many invasive cancers in women are found on screening
mammography before a palpable mass can be appreciated. More
than 40% of breast cancers in men are over 2 cm compared with
one-third of breast cancers in women and 39% have involved axillary nodes.3
4. What is the best approach to Local Control for Male Breast
Cancer?
Mastectomy continues to be the standard surgical management of
male breast cancer because the cancers are usually located under

5/22/2012 5:53:46 PM

Male Breast Cancer

the nipple and areola. In the absence of invasion of the pectoralis


muscle, modified radical mastectomy is preferred over radical
mastectomy. When breast cancer in men is not adherent or invading the nipple, breast conservation can be attempted. However,
adjuvant radiation therapy will be required to reduce the risk of
local recurrence to acceptable levels. Surgical treatment without
mastectomy is possible in many cases with excellent local control.30 Mastectomy should be accompanied by axillary lymph
node sampling using sentinel node biopsy or axillary node dissection. Sentinel node biopsy is now an accepted alternative to axillary lymph node dissection in men with clinically node-negative
breast cancer.31,32
Radiation should be considered in patients with locally advanced disease not amenable to surgical removal.
5. Is Adjuvant Therapy Indicated for Male Breast Cancer?
Adjuvant hormonal therapy with Tamoxifen has been extensively used in estrogen receptor-positive male breast cancers
because of the proven benefits of Tamoxifen in randomized
trials among women with breast cancer. Several retrospective
studies, but not all, have found a benefit in men.31-34 Aromatase
inhibitors are also being given to men with advanced estrogen
receptor-positive breast cancer. Doyen et al. 35 administered aromatase inhibitors to 15 men with metastatic breast cancer with
complete response in two, partial response in four, stable disease in two, and progressive disease in seven. In men, circulating estrogens are derived both from peripheral aromatization
of testosterone and by direct synthesis in the testes. Aromatase
inhibitors do not affect estrogen production by the testes and
measurable levels of serum estrogen are present in men taking
aromatase inhibitors. In addition, as aromatase inhibitors in
healthy men cause increases in leutenizing hormone and follicle
stimulating hormone, the resultant increased synthesis of testosterone would partially over-ride the inhibition of aromatase.
Perhaps, combining an aromatase inhibitor with Tamoxifen or
with the rHLH would result in better outcomes.
Chemotherapy is used in men with estrogen receptor-negative
cancers, especially those with nodal involvement or distant disease, and with estrogen receptor-positive cancers that progress on
hormonal therapy. Less than one-quarter of patients receive chemotherapy alone and less than one-third receive chemotherapy in
addition to hormonal therapy.34-36 This is because there are no studies of survival benefit for chemotherapy in men with breast cancer.
There are no randomized trials of chemotherapy in MBC. Walshe
et al.33 reported on the 20-year follow-up for 31 node-positive
patients given cyclophosphamide, methotrexate, and fluorouracil
after mastectomy at the National Cancer Institute. Tamoxifen was
given to only four patients despite a high rate of estrogen receptor
positivity (22/23 with known receptor status). Twenty-one of the 31
patients have died, one from a treatment-related complication.
As previously noted, HER2 overexpression occurs in up to
15% of MBC. As data from randomized controlled trials clearly
show an improved DFS in women with HER2-positive cancers
treated with trastuzumab independent of age, hormonal status,
nodal status, and tumor size, adjuvant trastuzumab therapy
should be considered in node-positive or high-risk node-negative
HER2-positive MBC.
Adjuvant radiation therapy should be considered in patients
undergoing breast conservation, for patients with large cancers,

PMPH_CH77.indd 619

619

with invasion of the pectoralis muscle, and in patients with


multiple-positive nodes. These are the same determinants of radiation therapy used in women and, in the absence of contradicting
data for men, these indications must be accepted. Men undergoing mastectomy more frequently receive radiation than women
because the pectoralis muscle or the skin is more frequently
involved. Radiation is thought to reduce the local recurrence rate
as it does in women.30,37,38
6. What is the Prognosis for Male Breast Cancer?
Breast cancer-specific survival in men is influenced by the same
factors that affect survival in women: lymph node involvement
and tumor size. However, as men with breast cancer are diagnosed at more advanced age than women, and life expectancy
for men is in general is lower than for women, men with breast
cancer have a higher risk of noncancer death than women with
breast cancer.39-41 As a consequence, one of the most significant
prognostic factors for men is age: the risk of dying from breast
cancer is 15 times higher for men over 80 years than for men 65
to 69 years.40 This observation comes from the analysis of the 510
men in the SEER database. After age, stage is the next most significant variable affecting outcome, followed by comorbidities and
race. This analysis found no survival benefit for chemotherapy or
radiation therapy. This is well illustrated by a study by EL-Tamer
et al.39 who matched 53 male breast cancer cases by age, date of
diagnosis, stage, and pathology to female cases. Ten-year breast
cancer-specific survival for men was 90% compared with 70% for
women. Overall survivals at 10 years for men and women were
57% and 51%. These findings were replicated in a similar study by
Marchal et al.41These seeming contradictory results were due to a
fourfold higher rate of noncancer death among the men.
The only additional significant prognostic factors in men
besides age and stage are estrogen receptor status, and expression of her2/neu and p53. However, these observations come from
studies with small numbers of patients.
Men treated for breast cancer have higher risks of developing
a second breast malignancy.42-44 The largest available study assembled data for 3409 men with breast cancer from 13 cancer registries
in different countries.44 Excess risk of a second breast cancer, and
cancers of the small bowel, rectum, pancreas, lymphatics, prostate,
and nonmelanoma skin were also noted. Analysis of SEER data43
and data from the California Cancer Registry42 noted increase risk
of melanoma and, in California, stomach cancer. Male breast cancer patients need genetic testing for BRCA 1 and 2 mutations, and
these patients need to be followed up closely because the risk of a
second breast cancer is increased at least 30 times.
The prognosis for men with breast cancer is highly influenced
by race.40 Using SEER data, Crew et al. found that Black men presented with significantly more advanced stage, with significantly
lower socioeconomic status, and were half as likely to receive chemotherapy than White male patients. Breast cancer mortality was
three times higher among Black patients.
Breast cancers of men are frequently compared with cancers
of postmenopausal women.39-41,45 However, breast cancers in men
differ from those in women in many ways: risk factors, presentation, pathology, treatment, and outcome. Reproductive risk factors,
age at menarche, first birth, and menopause are absent in men.
More than half of breast cancers in women are detected on screening mammography when nonpalpable and often in situ, whereas

5/22/2012 5:53:46 PM

620

Surgery: Evidence-Based Practice

in men almost all cancers present as a palpable mass. We are not


advocating screening mammography in men, but the absence of
screening causes cancers to present when larger, more frequently
with nodal involvement, and seldom when in situ. Hormone
receptor positivity is much more frequent in men and they should
be receiving Tamoxifen. Lumpectomy is not commonly used in

men as it is in women and, consequently, fewer men with early


disease receive radiation. Despite optimistic reports by oncologists concerning adjuvant chemotherapy, there are no substantive
data to support this optimism. Prognosis for men is worse than
for women because they are diagnosed later in life and at more
advanced stage than women.

Clinical Question Summary


Question

Answers

What are the risk factors?

BRCA mutation
Family History
Testicular disorders
Klinefelters
Obesity

Presentation?

Hard Subareolar Mass 2a

Diagnosis Made by?

Needle biopsy

Pathology, ER, Her2/neu?

Infiltrating Ductal
Estrogen-Positive
Her2/neu-Negative

Surgical Management?

Mastectomy
Node Sampling

2a

Best Hormonal Therapy?

Tamoxifen

2a

Role of chemotherapy?

Estrogen-Negative
Tamoxifen Failure

Role of Radiation?

Breast Conservation
Advanced Local Stage

REFERENCES
1. Jemal A, Siegel R, Xu J, Ward E. Cancer statistics, 2010. CA Cancer J Clin. 2010;60:277-300.
2. Steinberg E, Boring CC, Squires TS. Cancer statistics 1990. CA
Cancer J Clin. 1990;40:9-26.
3. Anderson WF, Althuis MD, Brinton LA, Devesa SS. Is male breast
cancer similar or different than female breast cancer? Breast
Cancer Res Treat. 2004;83:77-86.
4. Brinton LA, Carreon JD, Gierach GL, McGlynn KA, Gridley G.
Etiologic risk factors for male breast cancer in the U.S. Veterans
Affairs medical care system database. Breast Cancer Res Treat.
2010;119:185-192.
5. Swerdlow AJ, Schoemaker MJ, Higgins CD, Wright AF, Jacobs
PA. Cancer incidence and mortality in men with Klinefelter syndrome: A cohort study. J Natl Cancer Inst. 2005;97:1204-1210.
6. Johnson KC, Pan S, Mao Y. Risk factors for male breast cancer in
Canada, 1994-1998. Eu J Ca Prev. 2002;11:253-263.
7. Ewertz M, Holmberg L, Tretli S, Pedersen BV, Kristensen A. Risk
factors for male breast cancera case-control study from Scandinavia. Acta Oncol. 2001;40:467-471.
8. Ewertz M, Jensen M-J, Gunnarsdottir KA, et al. Effect of obesity on prognosis after early-stage breast cancer. J Clin Oncol.
2011;29:25-31.
9. Tai YC, Domchek S, Parmigiani G, Chen S. Breast cancer risk
among male BRCA1 and BRCA2 mutation carriers. J Natl Cancer Inst. 2007;99:1811-1814.

PMPH_CH77.indd 620

Level of
Evidence

Grade of
Recommendation

References

4, 9-11

2a

20-22

2a
25, 26
28, 29

24

31-34

2a
6, 7
4
4, 5
4, 6, 7

30, 37, 38

10. Liede A, Karlan BY, Narod SA. Cancer risks for male carriers of
germline mutations in BRCA1 or BRCA2: A review of the literature. J Clin Oncol. 2004;22:735-742.
11. Korde LA, Zujewski JA, Kamin L, et al. Multidisciplinary meeting on male breast cancer: Summary and research recommendations. J Clin Oncol. 2010;28:2114-2122.
12. Ron E, Ikeda T, Preston DL, Tokuoda S. Male breast cancer incidence among atomic bomb survivors. J Natl Cancer Inst. 2005;
97:603-605.
13. Thomas DB, Rosenblatt K, Jimenez M, et al. Ionizing radiation
and breast cancer in men (United States). Cancer Causes Con.
1994;5:9-14.
14. Olopade O. Male breast cancer in Cowden syndrome patients
with germline PTEN mutations. J Med Genet. 2001;38:159-164.
15. MacLean HE, Brown RW, Beilin J, Warne GL, Zajac JD. Increased
frequency of long androgen receptor CAG repeats in male breast
cancers. Breast Cancer Res Treat. 2004;88:239-246.
16. Young IE, Kurian KM, Annink C, et al. Br J Cancer. 1999;81:
141-143.
17. Wasielewski M, den Bakker MA, van den Ouweland A, et al.
CHEK2 1100delC and male breast cancer in the Netherlands.
Breast Cancer Res Treat. 2009;116:397-400.
18. Hanavadi S, Monypenny IJ, Mansel RE. Is mammography overused in male patients. Breast. 2006;15:123-126.
19. Hines SL, Tan WW, Yasrebi M, DePeri ER, Perez EA. The role
of mammography in male patients with breast symptoms. Mayo
Clin Proc. 2007;82:297-300.

5/22/2012 5:53:46 PM

Male Breast Cancer

20. Wauters CA, Kooistra BW, de Kievit-van der Heijden IM, Strobbe
LJ. Is cytology useful in the diagnostic workup of male breast
lesions? A retrospective study of over a 16-year period and review
of recent literature. Acta Cytol. 2010;54:259-264.
21. Rosen DG, Laucirica R, Verstovesek G. Fine needle aspiration of
male breast lesions. Acta Cytol. 2009;53:369-374.
22. Westenend PJ. Core needle biopsy in male breast lesions. J Clin
Path. 2003;56:863-865.
23. Morakkabati-Spitz N, Schild HH, Leutner CC, von Falkenhausen
M, Lutterby G, Kuhl CK. Dynamic contrast-enhanced breast MR
imaging in men: Preliminary results. Radiology. 2005;238:438-445.
24. Burga AM, Fadare O, Lininger RA, Tavassoli FA. Invasive carcinomas of the male breast: A morphologic study of the distribution of histologic subtypes and metastatic patterns in 778 cases.
Virchows Arch. 2006:449;507-512.
25. Evans GF, Anthony T, Turnage RH, et al. The diagnostic accuracy of mammography in the evaluation of male breast disease.
Am J Surg. 2001;181:96-100.
26. Patterson SK, Helvie MA, Aziz, K, Nees AV. Outcome of men
presenting with clinical breast problems: The role of mammography and ultrasound. Breast J. 2006;12:418-423.
27. Anderson WF, Jatoi I, Tse J, Rosenberg PS. Male breast cancer:
A population based comparison with female breast cancer. J Clin
Oncol. 2010;28:232-239.
28. Curigliano G, Colleoni M, Renne G, et al. Recognizing features
that are dissimilar in male and female breast cancer: Expression
of p21Waf1 and p27Kip1 using an immunohistochemical assay.
Ann Oncol. 2002;13:895-902.
29. Bloom KJ, Govil H, Gattuso P, Reddy V, Francescatti D. Status
of HER-2 in male and female breast carcinoma. Am J Surg. 2001;
182:389-392.
30. Golshan M, Rusby J, Dominguez F, Smith BL. Breast conservation for male breast carcinoma. Breast. 2007;16:653-656.
31. Boughey JC, Bedrosain I, Meric-Bernstam F, et al. Comparative
analysis of sentinel lymph node operation in male and female
breast cancer patients. J Am Coll Surg. 2006;203:475-480.
32. Gentilini O, Chagas E, Zurrida S, et al. Sentinel lymph node
biopsy in male patients with early breast cancer. Oncologist. 2007;
12:512-515.

PMPH_CH77.indd 621

621

33. Harlan LC, Zujewski JA, Goodman MT, Stevens JL. Breast cancer in men in the United States. Cancer. 2010:116;3558-3568.
34. Giordano SH, Perkins GH, Broglio K, et al. Adjuvant systemic
therapy for male breast carcinoma. Cancer. 2005;104:2359-2364.
35. Doyen J, Italiano A, Largillier R, Ferrero J-M, Fontana X,
Thyss A. Aromatase inhibition in male breast cancer patients:
Biological and clinical implications. Ann Oncol. 2010;21:
1243-1245.
36. Liukkonen S, Saarto T, Maenpaa H, Sjostrom-Mattson J. Male
breast cancer: A survey at the Helskinki University Central Hospital during 1981-2006. Acta Oncol. 2010;49:322-327.
37. Chakravarthy A, Kim CR. Post-mastectomy radiation in male
breast cancer. Radiother Oncol. 2002;65:99-103.
38. Zabel A, Milker-Mabel S, Zuna I, Wannenmacher M, Debus
J. External beam radiotherapy in the treatment of male breast
carcinoma: Patterns of failure in a single institution experience.
Tumori. 2005;91:151-155.
39. El-Tamer MB, Komenaka IK, Troxel A, et al. Men with breast cancer have better disease-specific survival than women. Arch Surg.
2004;139:1079-1082.
40. Crew KD, Neugut AI, Wang X, et al. Racial disparities in the
treatment and survival of male breast cancer. J Clin Oncol.
2007;25:1089-1098.
41. Marchal F, Salou M, Marchal C, Lesur A, Desandes E. Men with
breast cancer ave same disease-specific and event-free survival as
women. Ann Surg Oncol. 2009;16:972-978.
42. Satram-Hoag S, Ziogas A, Anton-Culver H. Risk of second
primary cancer in men with breast cancer. Breast Cancer Res.
2007;9:R10.
43. Wemberg JA, Yap J, Murekeyisoni C, et al. Multiple primary
tumors in men with breast cancer diagnoses: A SEER database
review. J Surg Oncol. 2009:99:16-19.
44. Grenader T, Goldberg A, Shavit L. Second cancers in patients
with male breast cancer: A literature review. J Cancer Surviv.
2008;2:73-78.
45. Speirs V, Ball G; the Male Breast Cancer Consortium. Male versus female breast cancer: A comparative study of 523 matched
cases reveals differences behind similarity. Breast Cancer Res.
2010;12(Suppl 1):O1.

5/22/2012 5:53:46 PM

CHAPTER 78

Breast Reconstruction
Following Mastectomy
David M. Adelman and Steven J. Kronowitz

INTRODUCTION

65 were the independent risk factors for perioperative complications following expander/implant breast reconstruction. Smoking,
obesity, and hypertension were similarly associated with reconstructive failure.9 These complications are not only found in patients
undergoing implant-based reconstructions, however. In a study
of pedicled transverse rectus abdominis myocutaneous (TRAM)
flap reconstructions, both active and former smokers had a higher
incidence of multiple flap-related complications. Active smokers
had a statistically significant higher rate of TRAM infection compared with nonsmokers. Former smokers were also found to have
a higher rate of TRAM-related delayed wound healing compared
with nonsmokers.10 With the advent of microsurgical breast reconstruction, most studies found that smokers again had increased
rates of complications. Free TRAM flap breast reconstruction in
smokers was not associated with a significant increase in the rates
of vessel thrombosis, flap loss, or fat necrosis compared with rates
in nonsmokers. However, smokers were at significantly higher
risk for mastectomy skin flap necrosis, abdominal flap necrosis,
and hernia compared with nonsmokers. Patients with a smoking
history of greater than 10 pack years were at especially high risk
for perioperative complications, suggesting that this should be
considered a relative contraindication for free TRAM flap breast
reconstruction. Smoking-related complications were significantly
reduced when the reconstruction was delayed or when the patient
stopped smoking at least 4 weeks before surgery.11 Additional
data advocate that microsurgical complications are not directly
increased by smoking, but rather the donor site and overall healing
abilities are affected.12 Perforator free flaps are part of the continued
evolution of autologous breast reconstruction. In one large study,
active smoking (within 1 month of surgery) was considered a specific contraindication to perforator flap surgery.13 Current smokers
with a large flap weight were also shown to have decreased intraflap
blood flow and more severe flap complications.14
Answer: Most studies demonstrate that smoking increases
the rates of mastectomy flap necrosis, implant and autologousbased infections, delayed wound healing, and generally worse
outcome, and patients should be counseled as such prior to any
postmastectomy breast reconstruction.

Breast reconstruction is a very important component of the multidisciplinary care of patients with breast cancer. Although in some
patients, especially those who will require postmastectomy radiation therapy (PMRT), it may be preferable to delay breast reconstruction, immediate breast reconstruction offers aesthetic and
technical benefits over delayed reconstruction.1,2 Immediate breast
reconstruction also provides psychologic benefits.3 Patients who
undergo immediate breast reconstruction do not have to experience
the psychologic trauma of not having a breast because they awake
from anesthesia with a breast mound. Despite these benefits, many
patients continue to present for delayed reconstruction, which is
often more complex than immediate breast reconstruction and
which may be associated with an increased risk of complications.
In patients undergoing breast reconstruction after total mastectomy, choosing the best method of reconstruction is essential
to optimize the aesthetic outcome and minimize the potential for
postoperative complications. Patient desires are extremely important in selecting the reconstructive technique. Unfortunately,
sometimes patients desires cannot be fulfi lled because of patient
anatomy or other clinical realities.
1. Does smoking add increased risk to outcome in breast reconstruction following mastectomy?
The detrimental effects of smoking on the outcome of any reconstructive surgery in which flaps are utilized are well documented.4-6
Smoking has been found to adversely affect outcomes in many
types of breast surgery, including breast reduction. In 2007, Bikhchandani et al. suggested that stoppage of smoking in the perioperative period should be adopted as an essential eligibility criterion
for breast reduction, given the significantly increased complication rate.7 In breast cancer patients undergoing reconstruction, the
complication rate is similarly increased compared with nonsmokers. In implant-based reconstructions, smoking was found to be
an independent predictor of increased surgical site infection rates.8
In a separate study, smoking, obesity, hypertension, and age over
622

PMPH_CH78.indd 622

5/22/2012 5:54:18 PM

Breast Reconstruction Following Mastectomy

2. Do neoadjuvant or adjuvant therapies increase risk in breast


reconstruction following mastectomy?
An increasing number of patients are receiving chemotherapy
or radiation therapy as adjuncts to surgical treatment for breast
cancer. Various protocols exist that differ in both the types of
therapy, as well as the timing of the intervention. The goals
of these therapies may be to shrink primary tumor size, and/or
to treat nodal or metastatic disease. Common concerns of reconstructive surgeons are related to these therapies. First, might
performing breast reconstruction following mastectomy lead to
delayed implementation of these adjuvant therapies? Two studies found that immediate breast reconstruction did not appear
to lead to omission of adjuvant chemotherapy, but was associated with a statistically significant delay in initiating treatment.
For most patients, however, it was unlikely that, or unclear if,
this delay had any clinical significance.15,16 Second, might use
of these therapies before (neoadjuvant) or after (adjuvant) surgery lead to increased morbidity related to the reconstruction?
Most surgeons will wait to perform mastectomy and reconstruction until a patient has recovered from neoadjuvant treatments.
Likewise, most patients will not be initiated on adjuvant therapies until sufficient healing from reconstruction has occurred.
One recent study looked at the prevalence of complications after
breast reconstruction following mastectomy in cohorts that did
and did not receive adjuvant chemotherapy. Although the highest rate of surgical site infections was in the adjuvant chemotherapy group, there were no differences between groups with respect
to unplanned return to the operating room, expander loss, or
donor-site complications. Neither the inclusion of chemotherapy
nor the timing of its administration seemed to significantly affect
the complication rates after mastectomy and immediate breast
reconstruction.17 A separate study asked if adjuvant chemotherapy and radiation therapy had effect on immediate postmastectomy autologous breast reconstructions. Patient-specific factors,
including diabetes mellitus and smoking, were found to increase
the risk of post-irradiation parenchymal changes, and neoadjuvant chemotherapy was associated with a greater than twofold
increase in skin complications. However, they concluded that
overall, autologous breast reconstruction followed by irradiation
can be successful, but patients with specific risks factors should
be aware of increased complication rates.18
Answer: The data suggest that although these neoadjuvant
and adjuvant therapies may contribute to increased morbidity, they
are generally not a contraindication to performing breast reconstruction following mastectomy.
3. Does surveillance for locoregional recurrence necessitate
delay of breast reconstruction following mastectomy?
The primary aim of surgery in breast cancer is to achieve a local
control of disease. Secondly, patient satisfaction and improvement in quality of life are of utmost importance. Although the
positive effects of breast reconstruction following mastectomy
on the psychologic well-being of women with breast cancer has
been demonstrated, evidence-based data on its oncologic safety
remain sparse. There is concern that the presence of an implant
or autologous tissue may mask locoregional recurrence. However, this concern has not been borne out in the literature thus
far. For example, immediate breast reconstruction after mastectomy was determined to be an oncologically safe approach in

PMPH_CH78.indd 623

623

a study by Knottenbelt et al.19 The majority of patients in this


study had stage I or II cancers, and was reconstructed with tissue expanders and implants. A later study by McCarthy et al.
agreed with these fi ndings.20 They found that prosthetic breast
reconstruction neither hinders detection of locoregional cancer
recurrence, nor does it often necessitate removal of the reconstruction for management of the disease. Similar fi ndings exist
in a retrospective study of patients reconstructed with autologous tissue in the form of a TRAM flap.21 A literature review of
the impact of breast reconstruction on the incidence and detection of locoregional recurrence and treatment options was also
recently performed, 22 and highlights some of the aforementioned
studies and fi ndings.
Answer: Taken together, the data suggest that surveillance
for breast cancer recurrence may be performed sufficiently even
in reconstructed breasts, and should not delay reconstruction.
4. Is patient anatomy important in the selection of reconstructive technique?
As with most reconstructive or aesthetic procedures, patient
anatomy plays a critical role in breast reconstruction following mastectomy. Anatomy may be divided into three categories:
body habitus, breast size/shape, and donor site, each with its own
importance. Patients may be too thin or too obese, but preferably fall somewhere in between. Thin patients may be malnourished from chemotherapy. They may lack adequate donor sites
for autologous reconstruction, and/or may have thin mastectomy
flaps through which implants are easily palpated and poorly protected. On the contrary, obese patients tend to have high rates
of perioperative complications. However, it is generally safe to
perform autologous reconstruction in obese patients, and they
tend to have high rates of satisfaction.23,24 The largest implants
available are frequently too small for the body frame of an obese
patient, and therefore may not look appropriate. Ideally, a patient
should reach her goal weight and be stable at that weight prior to
surgery if possible. However, given that these are cancer operations, rather than merely aesthetic operations, one often must
proceed under less than ideal circumstances. Patient education
becomes critical in such situations so that expectations can be
appropriately met.
Preoperative and postoperative breast size plays an important role. Tissue expansion may allow a small-breasted woman
to achieve a larger cup size postoperatively. An autologous reconstruction from the abdomen may provide enough tissue for a
unilateral breast reconstruction, but not for a bilateral reconstruction. Similarly, a breast lift (mastopexy), reduction, or implant
augmentation may be required to enable symmetry between a
native and reconstructed breast. It is important to have discussions preoperatively regarding patient desires, and what can be
accomplished with safety, efficacy, and the best possible aesthetic
outcomes.
Donor and recipient site anatomy also play critical roles in
breast reconstruction. Free (microvascular) flaps are not possible
without adequate recipient vessels. The decision between harvesting a superficial inferior epigastric artery (SIEA), TRAM, or deep
inferior epigastric perforator (DIEP) flap requires a patients vasculature to be favorable toward one or all types of flap.25 Number,
caliber, and location of perforating vessels, as well as perfusion
through these vessels, must be evaluated in each patient.

5/22/2012 5:54:18 PM

624

Surgery: Evidence-Based Practice

Answer: Patient anatomy is critical in both selection of


tech nique and execution of breast reconstruction following
mastectomy.
5. Is there an age limit on breast reconstruction following
mastectomy?
Elective surgery in the elderly is not without its associated risks.
Although the benefits of breast reconstruction following mastectomy have been fully validated, do these benefits become limited
with advanced age? One study suggested that all types of reconstruction should be an option for women older than 60 years of age
and that age as an isolated factor should not deter physicians from
offering these women the option of breast reconstruction.26 This
may be true for implant-based reconstruction, but can the same
be said for a free microvascular autologous tissue flap reconstruction? In recent years, many studies have looked at this question
and all arrived at the same conclusionsthat free-tissue transfer
in the elderly population has demonstrated similar success rates to
those of the general population. In other words, age alone should
not be considered a contraindication or an independent risk factor for free-tissue transfer.27,28 More recent studies have had similar findings.29,30 In the study by Selber et al., despite higher rates
of hypertension, higher American Society of Anesthesiologists
status, higher body mass index, and higher rates of blood transfusion, the 65 years and older group had outcomes equal to those of
the general population.
Answer: Breast reconstruction following mastectomy, and in
particular free microvascular autologous tissue flap breast reconstruction, in patients of advanced age is safe, and should be offered
when indicated.
6. Which patients should receive immediate breast reconstruction as opposed to delayed reconstruction after radiation
therapy?
PMRT can improve survival and locoregional control in patients
with invasive breast cancer. The optimal timing and techniques
of breast reconstruction in patients requiring PMRT are controversial. Kronowitz and Robb recently performed a review of
the literature and published their findings.31 The indications for
radiation vary, but in general are favored in patients with more
advanced disease (larger tumor size, increased number of positive nodes, extra-nodal disease, etc.).32-34 In multiple studies,
radiation has been shown to adversely affect the aesthetic outcome and increase complication rates of both implant-based35-38
and autologous tissue reconstructions.39-42 A study by Cordeiro
et al.43 suggests that if a tissue expander is exchanged for a permanent implant prior to radiation therapy, the outcomes can be
more favorable. However, other studies demonstrate that the presence of a fully inflated implant, or an autologous tissue flap, may
even compromise the design and delivery of radiation treatment
fields.44-51 The suggestion is therefore made that in patients who
are likely, or at least at high risk, for requiring PMRT, reconstruction of any kind be delayed until therapy is completed. There is a
population of patients, however, for whom the need for radiation
at the time of surgery is unclear (mostly stage II and some stage I).
For these patients, a delayed- immediate type approach may be
of benefit.46,52
Answer: In patients known prior to mastectomy to require
radiation, delay breast reconstruction until after radiation; in

PMPH_CH78.indd 624

patients who are considered at an increased risk for requiring postmastectomy radiation perform a delayed-immediate approach.
7. Is a breast implant or an autologous tissue flap preferable for
reconstruction following mastectomy?
An old adage in reconstructive surgery says to replace like with
like. As the adult breast is mostly skin and fat, it is sensible to
replace it with skin and fat. Autologous tissue flap options, including those from the lower abdomen (SIEA, TRAM, DIEP), would
adhere to this adage well. But there are advantages and disadvantages to such types of reconstruction. Benefits of autologous
reconstruction may include the placement of well-vascularized
tissue into a previously irradiated chest wall, which may alleviate
some of the negative effects of the radiation. Autologous tissue is
able to prevent and treat infection more so than implants, given
its ample blood supply. It can also provide a better match with the
native breast in a unilateral reconstruction. And, it can remain
viable for the life of the patient. The disadvantages of autologous
reconstruction include additional donor-site morbidity (such as
scars, abdominal hernia or bulge, delayed wound healing, infection), prolonged operating times, insufficient quantity or quality
of tissue, and the need to change the position of the patient during
the operation (e.g., latissimus dorsi or gluteal flaps).
By contrast, a breast implant can be the best reconstructive choice for many women. Surgery is confined to the breast,
requires minimal recovery, does not depend on the availability or
the quality of donor tissue, and implants are available in different
sizes and shapes. Sounds perfect, so why dont all women choose
implants? Well, implants are foreign bodies, and incite a reaction
from the body to wall it off (i.e., a capsule). In many women, this
capsule is thin and soft, but in others it can be hard, painful, and
even distort the breast.53 This is particularly seen after radiation.35
Implants are also prone to infection, as they do not have a blood
supply capable of bringing immune agents to the affected region.
They may rupture, undergo positional changes, or otherwise
require replacement, even multiple times throughout the length
of a womans life.
Answer: Both implants and autologous tissues have their
advantages and disadvantages. The decision regarding each ones
use is best made when all factors regarding patient anatomy, therapies, and desires are taken into account.
8. Is a free microvascular TRAM flap better than a pedicle
TRAM flap?
The reconstructive ladder is used by plastic surgeons to define
reconstructions from the simple to complex.54 In this paradigm,
pedicle flaps are generally lower on the ladder than free microvascular flaps, suggesting that pedicle flaps are somehow easier
or less complex. The reconstructive ladder also implies a certain
evolutionary quality, in that the higher up the ladder, the more
recent the innovation. This is certainly true with microsurgery, in
that it has only existed in any serious form since the 1970s.55 However, many pedicle flaps are far more difficult to plan and execute
than their free flap counterparts, and may be less ideal forms of
reconstruction. Currently, many reconstructive surgeons take the
reconstructive elevator to the top and perform free flap reconstructions as a primary means.56
There are many advantages to a pedicle TRAM flap for breast
reconstruction. 57 These flaps follow a well-defined anatomy,

5/22/2012 5:54:18 PM

Breast Reconstruction Following Mastectomy

making them straightforward to harvest. Special microsurgical


expertise is not required, making this technique readily available
to most surgeons. Because recipient vessels do not need to be prepared, their availability and quality does not factor into the success of the reconstruction. The harvest and inset of a pedicle flap
can be fast, minimizing operative time and cost.
However, there are problems associated with some pedicle
TRAM flaps. By definition, they are based off the superior epigastric blood supply, itself a continuation of the internal mammary vessels. In many patients, this pedicle is less robust when
compared with the deep inferior epigastric system. When based
on the superior system, there may be a higher rate of fat necrosis, especially in larger flaps. With pedicle TRAM flaps, minimal
efforts traditionally have been made to spare muscle and fascia.
In bilateral pedicle TRAM flap reconstructions more than unilateral reconstructions, there is a higher incidence of abdominal
wall weakness, bulge, and hernia.58-60 Although free TRAM and
DIEP flap operative times are considerably longer than with pedicle TRAM flaps, many surgeons still prefer the free TRAM flap
because of the better blood supply and decreased abdominal wall

625

morbidity. Microsurgical experience plays a much larger role in


free TRAM survival and outcomes. Also, community hospitals
without specialized equipment and trained staff may not be prepared to support microsurgical breast reconstruction.
Answer: The benefits of a free microvascular TRAM (and its
variations, such as the DIEP flap) probably outweigh those of the
pedicle TRAM flap. However, the pedicle TRAM flap still has its
role and can provide a long-lasting reconstruction with minimal
donor-site morbidity.

CONCLUSIONS
Breast reconstruction following mastectomy is not a one size
fits all endeavor. Multiple techniques may be appropriate for a
given patient, and achieving the best outcome requires thoughtful planning and execution. Various factors including anatomy,
comorbidity, and associated therapies must be considered. As our
expectations for outcomes continue to evolve, so must our understanding of the factors surrounding breast reconstruction.

Clinical Question Summary


Question

Answer

Level of Grade of
References
Evidence Recommendation

1 Does smoking add


increased risk to outcome
in breast reconstruction
following mastectomy?

Most studies demonstrate that smoking increases


rates of mastectomy flap necrosis, implant and
autologous-based infections, delayed wound
healing, and generally worse outcome, and
patients should be counseled as such prior to
any postmastectomy breast reconstruction.

3b

4-14

2 Do neoadjuvant or
adjuvant therapies
increase risk in breast
reconstruction following
mastectomy?

The data suggest that although these neoadjuvant


and adjuvant therapies may contribute to
increased morbidity, they are generally not
a contraindication to performing breast
reconstruction following mastectomy.

2b

15-18

3 Does surveillance for


locoregional recurrence
necessitate delay of breast
reconstruction following
mastectomy?

Taken together, the data suggest that surveillance


for breast cancer recurrence may be
performed sufficiently even in reconstructed
breasts, and should not delay reconstruction.

3b

19-22

4 Is patient anatomy
important in the selection
of reconstructive
technique?

Patient anatomy is critical in both selection


of technique and execution of breast
reconstruction following mastectomy.

3b

23-25

5 Is there an age limit on


breast reconstruction
following mastectomy?

Breast reconstruction following mastectomy, and


in particular free microvascular autologous
tissue flap breast reconstruction, in patients
of advanced age is safe, and should be offered
when indicated.

2b

26-30

6 Which patients should


receive immediate breast
reconstruction as opposed
to delayed reconstruction
after radiation therapy?

In patients known prior to mastectomy to


require radiation, delay breast reconstruction
until after radiation and in patients who are
considered at an increased risk for requiring
postmastectomy radiation perform a delayedimmediate approach.

3b

31-52

(Continued)

PMPH_CH78.indd 625

5/22/2012 5:54:18 PM

626

Surgery: Evidence-Based Practice

(Continued)
Question

Answer

7 Is a breast implant or
an autologous tissue
flap preferable for
reconstruction following
mastectomy?

Both implants and autologous tissues have their


merits and demerits. The decision regarding
each ones use is best made when all factors
regarding patient anatomy, therapies, and
desires are taken into account.

3b

53

8 Is a free microvascular
TRAM flap better than a
pedicle TRAM flap?

The benefits of a free microvascular TRAM (and


its variations, such as the DIEP flap) probably
outweigh those of the pedicle TRAM flap.
However, the pedicle TRAM still has its role
and can provide a long-lasting reconstruction
with minimal donor site morbidity.

3b

54-60

REFERENCES
1. Kroll SS, Coffey JA, Jr, Winn RJ, Schusterman MA. A comparison of factors affecting aesthetic outcomes of TRAM flap breast
reconstructions. Plast Reconstr Surg. 1995;96(4):860-864.
2. Miller MJ, Rock CS, Robb GL. Aesthetic breast reconstruction using
a combination of free transverse rectus abdominis musculocutaneous flaps and breast implants. Ann Plast Surg. 1996;37(3):258-264.
3. DeBono R, Thompson A, Stevenson JH. Immediate versus delayed
free TRAM breast reconstruction: an analysis of perioperative
factors and complications. Br J Plast Surg. 2002;55(2):111-116.
4. Chang LD, Buncke G, Slezak S, Buncke HJ. Cigarette smoking,
plastic surgery, and microsurgery. J Reconstr Microsurg. 1996;12(7):
467-474.
5. Reus WF, Robson MC, Zachary L, Heggers JP. Acute effects
of tobacco smoking on blood flow in the cutaneous microcirculation. Br J Plast Surg. 1984;37(2):213-215.
6. van Adrichem LN, Hoegen R, Hovius SE, et al. The effect of cigarette smoking on the survival of free vascularized and pedicled
epigastric flaps in the rat. Plast Reconstr Surg. 1996;97(1):86-96.
7. Bikhchandani J, Varma SK, Henderson HP. Is it justified to refuse
breast reduction to smokers? J Plast Reconstr Aesthet Surg. 2007;
60(9):1050-1054.
8. Olsen MA, Lefta M, Dietz JR, et al. Risk factors for surgical site
infection after major breast operation. J Am Coll Surg. 2008;207(3):
326-335.
9. McCarthy CM, Mehrara BJ, Riedel E, et al. Predicting complications following expander/implant breast reconstruction: an
outcomes analysis based on preoperative clinical risk. Plast
Reconstr Surg. 2008;121(6):1886-1892.
10. Spear SL, Ducic I, Cuoco F, Hannan C. The effect of smoking
on flap and donor-site complications in pedicled TRAM breast
reconstruction. Plast Reconstr Surg. 2005;116(7):1873-1880.
11. Chang DW, Reece GP, Wang B, et al. Effect of smoking on complications in patients undergoing free TRAM flap breast reconstruction. Plast Reconstr Surg. 2000;105(7):2374-2380.
12. Mehrara BJ, Santoro TD, Arcilla E, Watson JP, Shaw WW, Da
Lio AL. Complications after microvascular breast reconstruction: experience with 1195 flaps. Plast Reconstr Surg. 2006;118(5):
1100-1109; discussion 1110-1111.
13. Granzow JW, Levine JL, Chiu ES, LoTempio MM, Allen RJ.
Breast reconstruction with perforator flaps. Plast Reconstr Surg.
2007;120(1):1-12.

PMPH_CH78.indd 626

Level of Grade of
References
Evidence Recommendation

14. Booi DI, Debats IB, Boeckx WD, van der Hulst RR. Risk factors
and blood flow in the free transverse rectus abdominis (TRAM)
flap: smoking and high flap weight impair the free TRAM flap
microcirculation. Ann Plast Surg. 2007;59(4):364-371.
15. Kontos M, Lewis RS, Lchtenborg M, Holmberg L, Hamed H.
Does immediate breast reconstruction using free flaps lead to
delay in the administration of adjuvant chemotherapy for breast
cancer? Eur J Surg Oncol. 2010;36(8):745-749.
16. Alderman AK, Collins ED, Schott A, et al. The impact of breast
reconstruction on the delivery of chemotherapy. Cancer. 2010;
116(7):1791-1800.
17. Warren Peled A, Itakura K, Foster RD, et al. Impact of chemotherapy on postoperative complications after mastectomy and
immediate breast reconstruction. Arch Surg. 2010;145(9):880-885.
18. Albino FP, Koltz PF, Ling MN, Langstein HN. Irradiated autologous breast reconstructions: effects of patient factors and treatment variables. Plast Reconstr Surg. 2010;126(1):12-16.
19. Knottenbelt A, Spauwen PH, Wobbes T. The oncological implications of immediate breast reconstruction. Eur J Surg Oncol.
2004;30(8):829-833.
20. McCarthy CM, Pusic AL, Sclafani L, et al. Breast cancer recurrence following prosthetic, postmastectomy reconstruction:
incidence, detection, and treatment. Plast Reconstr Surg. 2008;
121(2):381-388.
21. Howard MA, Polo K, Pusic AL, et al. Breast cancer local recurrence
after mastectomy and TRAM flap reconstruction: incidence and
treatment options. Plast Reconstr Surg. 2006;117(5):1381-1386.
22. Kropf N, McCarthy CM, Disa JJ. Breast cancer local recurrence
after breast reconstruction. Handchir Mikrochir Plast Chir. 2008;
40(4):219-224.
23. Atisha DM, Alderman AK, Kuhn LE, Wilkins EG. The impact of
obesity on patient satisfaction with breast reconstruction. Plast
Reconstr Surg. 2008;121(6):1893-1899.
24. Chang DW, Wang B, Robb GL, et al. Effect of obesity on flap and
donor-site complications in free transverse rectus abdominis
myocutaneous flap breast reconstruction. Plast Reconstr Surg.
2000;105(5):1640-1648.
25. Lipa JE. Breast reconstruction with free flaps from the abdominal donor site: TRAM, DIEAP, and SIEA flaps. Clin Plast Surg.
2007;34(1):105-121; abstract vii.
26. Bowman CC, Lennox PA, Clugston PA, Courtemanche DJ.
Breast reconstruction in older women: should age be an exclusion criterion? Plast Reconstr Surg. 2006;118(1):16-22.

5/22/2012 5:54:18 PM

Breast Reconstruction Following Mastectomy

27. Lipa JE, Youssef AA, Kuerer HM, Robb GL, Chang DW. Breast
reconstruction in older women: advantages of autogenous tissue.
Plast Reconstr Surg. 2003;111(3):1110-1121.
28. Serletti JM, Higgins JP, Moran S, Orlando GS. Factors affecting
outcome in free-tissue transfer in the elderly. Plast Reconstr Surg.
2000;106(1):66-70.
29. Coskunfirat OK, Chen HC, Spanio S, Tang YB. The safety of
microvascular free tissue transfer in the elderly population. Plast
Reconstr Surg. 2005;115(3):771-775.
30. Selber JC, Bergey M, Sonnad SS, Kovach S, Wu L, Serletti JM.
Free flap breast reconstruction in advanced age: is it safe? Plast
Reconstr Surg. 2009;124(4):1015-1022.
31. Kronowitz SJ, Robb GL. Radiation therapy and breast reconstruction: a critical review of the literature. Plast Reconstr Surg.
2009;124(2):395-408.
32. Harris JR, Halpin-Murphy P, McNeese M, Mendenhall NP,
Morrow M, Robert NJ. Consensus Statement on postmastectomy radiation therapy. Int J Radiat Oncol Biol Phys. 1999;44(5):
989-990.
33. Recht A, Edge SB, Solin LJ, et al. Postmastectomy radiotherapy:
clinical practice guidelines of the American Society of Clinical
Oncology. J Clin Oncol. 2001;19(5):1539-1569.
34. Truong PT, Olivotto IA, Whelan TJ, Levine M; Steering Committee on Clinical Practice Guidelines for the Care and Treatment
of Breast Cancer. Clinical practice guidelines for the care and
treatment of breast cancer: 16. Locoregional post-mastectomy
radiotherapy. CMAJ. 2004;170(8):1263-1273.
35. Ascherman JA, Hanasono MM, Newman MI, Hughes DB.
Implant reconstruction in breast cancer patients treated with
radiation therapy. Plast Reconstr Surg. 2006;117(2):359-365.
36. Behranwala KA, Dua RS, Ross GM, Ward A, Ahern R, Gui GP.
The influence of radiotherapy on capsule formation and aesthetic
outcome after immediate breast reconstruction using biodimensional anatomical expander implants. J Plast Reconstr Aesthet
Surg. 2006;59(10):1043-1051.
37. Benediktsson K, Perbeck L. Capsular contracture around salinefi lled and textured subcutaneously-placed implants in irradiated and non-irradiated breast cancer patients: five years of
monitoring of a prospective trial. J Plast Reconstr Aesthet Surg.
2006;59(1):27-34.
38. Spear SL, Onyewu C. Staged breast reconstruction with salinefi lled implants in the irradiated breast: recent trends and
therapeutic implications. Plast Reconstr Surg. 2000;105(3):
930-942.
39. Rogers NE, Allen RJ. Radiation effects on breast reconstruction
with the deep inferior epigastric perforator flap. Plast Reconstr
Surg. 2002;109(6):1919-1924; discussion 1925-1926.
40. Spear SL, Ducic I, Low M, Cuoco F. The effect of radiation on
pedicled TRAM flap breast reconstruction: outcomes and implications. Plast Reconstr Surg. 2005;115(1):84-95.
41. Tran NV, Chang DW, Gupta A, Kroll SS, Robb GL. Comparison
of immediate and delayed free TRAM flap breast reconstruction
in patients receiving postmastectomy radiation therapy. Plast
Reconstr Surg. 2001;108(1):78-82.
42. Williams JK, Carlson GW, Bostwick J 3rd, Bried JT, Mackay G.
The effects of radiation treatment after TRAM flap breast reconstruction. Plast Reconstr Surg. 1997;100(5):1153-1160.

PMPH_CH78.indd 627

627

43. Cordeiro PG, Pusic AL, Disa JJ, McCormick B, VanZee K. Irradiation after immediate tissue expander/implant breast reconstruction: outcomes, complications, aesthetic results, and satisfaction
among 156 patients. Plast Reconstr Surg. 2004;113(3):877-881.
44. Buchholz TA, Strom EA, Perkins GH, McNeese MD. Controversies regarding the use of radiation after mastectomy in breast
cancer. Oncologist. 2002;7(6):539-546.
45. Kronowitz SJ. Immediate versus delayed reconstruction. Clin
Plast Surg. 2007;34(1):39-50; abstract vi.
46. Kronowitz SJ, Hunt KK, Kuerer HM, et al. Delayed-immediate
breast reconstruction. Plast Reconstr Surg. 2004;113(6):1617-1628.
47. Kronowitz SJ, Kuerer HM. Advances and surgical decisionmaking for breast reconstruction. Cancer. 2006;107(5):893-907.
48. Kronowitz SJ, Robb GL. Breast reconstruction with postmastectomy radiation therapy: current issues. Plast Reconstr Surg.
2004;114(4):950-960.
49. Motwani SB, Strom EA, Schechter NR, et al. The impact of
immediate breast reconstruction on the technical delivery of
postmastectomy radiotherapy. Int J Radiat Oncol Biol Phys. 2006;
66(1):76-82.
50. Schechter NR, Strom EA, Perkins GH, et al. Immediate breast
reconstruction can impact postmastectomy irradiation. Am J
Clin Oncol. 2005;28(5):485-494.
51. Woodward WA, Strom EA, Tucker SL, et al. Locoregional recurrence after doxorubicin-based chemotherapy and postmastectomy: Implications for breast cancer patients with early-stage
disease and predictors for recurrence after postmastectomy
radiation. Int J Radiat Oncol Biol Phys. 2003;57(2):336-344.
52. Kronowitz SJ. Delayed-immediate breast reconstruction: technical and timing considerations. Plast Reconstr Surg. 2010;125(2):
463-474.
53. Embrey M, Adams EE, Cunningham B, Peters W, Young VL, Carlo
GL. A review of the literature on the etiology of capsular contracture and a pilot study to determine the outcome of capsular contracture interventions. Aesthetic Plast Surg. 1999;23(3):197-206.
54. Janis JE, Kwon RK, Attinger CE. The new reconstructive ladder: modifications to the traditional model. Plast Reconstr Surg.
2011;127(Suppl 1):205S-212S.
55. Buncke HJ. Microsurgeryretrospective. Clin Plast Surg. 1986;
13(2):315-318.
56. Gottlieb LJ, Krieger LM. From the reconstructive ladder to
the reconstructive elevator. Plast Reconstr Surg. 1994;93(7):
1503-1504.
57. Larson DL, Yousif NJ, Sinha RK, Latoni J, Korkos TG. A comparison of pedicled and free TRAM flaps for breast reconstruction
in a single institution. Plast Reconstr Surg. 1999;104(3):674-680.
58. Man LX, Selber JC, Serletti JM. Abdominal wall following free
TRAM or DIEP flap reconstruction: a meta-analysis and critical
review. Plast Reconstr Surg. 2009;124(3):752-764.
59. Selber JC, Fosnot J, Nelson J, et al. A prospective study comparing the functional impact of SIEA, DIEP, and muscle-sparing free
TRAM flaps on the abdominal wall: Part II. Bilateral reconstruction. Plast Reconstr Surg. 2010;126(5):1438-1453.
60. Selber JC, Nelson J, Fosnot J, et al. A prospective study comparing the functional impact of SIEA, DIEP, and muscle-sparing free
TRAM flaps on the abdominal wall: Part I. unilateral reconstruction. Plast Reconstr Surg. 2010;126(4):1142-1153.

5/22/2012 5:54:18 PM

PMPH_CH78.indd 628

5/22/2012 5:54:18 PM

PART XIII

CHEST WALL, MEDIASTINUM,


TRACHEA

PMPH_CH79.indd 629

5/22/2012 5:54:50 PM

PMPH_CH79.indd 630

5/22/2012 5:54:50 PM

CHAPTER 79

Lung Cancer Staging


Joe B. Putnam, Jr.

THE NEW (2010) AMERICAN JOINT


COMMITTEE ON CANCER LUNG
CANCER STAGING SYSTEM

of definitive therapy. Pathologic stage is the determination of the


physical extent of the disease based on the resected specimen,
including the hilar and mediastinal lymph nodes. The choice
of therapy depends on the patients clinical stage of the disease
and the unique anatomic, mechanical, and biological properties
of the specific lung cancer and the specific patient. Optimal clinical staging is essential for selecting optimal therapy.
Current staging models use anatomic characteristics of
the tumor or lymph nodes (Table 79.1). These characteristics
include size, location, or presence of metastatic disease. Clinically
these characteristics, although crude, are used as a surrogate for
the biological behavior of the tumor. Treatment usually based on
anatomic characteristics will yield results that are highly variable. Biological characteristics including genetic mutations may

Staging of lung cancer or any cancer attempts to quantitatively


describe the extent of the disease process so as to compare patient
selection, results of therapy, and survival outcomes. Staging provides clinicians a shorthand description of the tumor, lymph
node, and metastatic characteristics to (1) facilitate selection of
patients for specific therapy, (2) apply optimal therapy for individual patients, and (3) evaluate outcome based on the clinical stage
and the pathologic stage.
Clinical stage is the best determination of the extent of the
disease, based on all available information, prior to the initiation
Table 79.1 TNM Descriptors
TPrimary Tumor
TX

Primary tumor cannot be assessed; or tumor proven by the presence of malignant cells in sputum or bronchial washings but
not visualized by imaging or bronchoscopy

T0

No evidence of primary tumor

Tis

Carcinoma in situ

T1

Tumor 3 cm or less in greatest dimension, surrounded by lung or visceral pleura, without bronchoscopic evidence of
invasion more proximal than the lobar bronchus (e.g., not in the main bronchus)

T1a

Tumor 2 cm or less in greatest dimension

T1b

Tumor more than 2 cm but not more than 3 cm in greatest dimension

T2

Tumor more than 3 cm but 7 cm or less, or tumor with any of the following features (note: T2 tumors with these features
are classified as T2a if 5 cm or less):
Invades visceral pleura (PL1 or PL2)
Involves main bronchus, 2 cm or more distal to the carina
Associated with atelectasis or obstructive pneumonitis that extends to the hilar region but does not involve the entire
lung
T2a tumor is more than 3 cm but 5 cm or less in greatest dimension
T2b tumor is more than 5 cm but 7 cm or less in greatest dimension
(Continued)
631

PMPH_CH79.indd 631

5/22/2012 5:54:50 PM

632

Surgery: Evidence-Based Practice

Table 79.1 (Continued)


TPrimary Tumor
T3

Tumor more than 7 cm; or tumor that directly invades any of the following: parietal pleura (PL3), chest wall (including
superior sulcus tumors), diaphragm, phrenic nerve, mediastinal pleura, parietal pericardium; or tumor in the main
bronchus less than 2 cm distal to the carina1 but without involvement of the carina; or associated atelectasis or
obstructive pneumonitis of the entire lung or separate tumor nodule(s) in the same lobe as the primary.

T4

Tumor of any size that invades any of the following: mediastinum, heart, great vessels, trachea, recurrent laryngeal nerve,
esophagus, vertebral body, carina; separate tumor nodule(s) in a different ipsilateral lobe to that of the primary.a

NRegional Lymph Nodes


NX

Regional lymph nodes cannot be assessed

N0

No regional lymph node metastasis

N1

Metastasis in ipsilateral peribronchial and/or ipsilateral hilar lymph nodes and intrapulmonary nodes, including involvement
by direct extension

N2

Metastasis in ipsilateral mediastinal and/or subcarinal lymph node(s)

N3

Metastasis in contralateral mediastinal, contralateral hilar, ipsilateral or contralateral scalene, or supraclavicular lymph node(s)

MDistant Metastasis
M0

No distant metastasis

M1

Distant metastasis

M1a Separate tumor nodule(s) in a contralateral lobe; tumor with pleural nodules or malignant pleural or pericardial
effusion
M1b Distant metastasis (in extrathoracic organs)

From Goldstraw et al.7

provide a more sensitive measure of the tumor biology but is not


in common use at this time. For now, clinical stage should be as
accurate as possible to guide selection of therapy, and pathologic
stage, to guide adjuvant therapy and decisions based on the survival associated with specific-stage disease.
The American Joint Committee on Cancer (AJCC) 5th edition
(1997) lung cancer staging model1,2 consisted of 5319 patients, who
were mostly treated at the University of Texas M. D. Anderson
Cancer Center. In that iteration, stage I was subdivided into IA
(T1 N0 M0) and IB (T2 N0 M0). Stage II was divided into IIA (T1
N1 M0) and IIB (T2 N1 M0 and T3 N0 M0). Satellite nodules (ipsilateral, same lobe) were characterized as T4 and separate nodules
(ipsilateral, different lobe) as M1.
The AJCC 6th edition (2002) lung cancer staging had no
changes recommended. There was a valid need to obtain larger
patient numbers from different data sets, to evaluate survival
based on various treatment strategies (alone or in combination),
and to develop proposals for future changes in the AJCC and the
Union for International Cancer Control (UICC) lung cancer staging model. The International Association for the Study of Lung
Cancer (IASLC) embarked on its lung cancer staging project to
include all treatment and diagnostic groups, and to collect data for
analysis and reform future revisions.3 The current AJCC 7th edition lung cancer staging model reflects the impact of the IASLC
lung cancer staging project.4,5
The IASLC collected over 100,000 non-small cell lung cancer
(NSCLC) cases treated between 1990 and 2000. Each patient had
a minimum of 5 years of follow-up and all treatment modalities
were included. Over 81,000 cases were submitted and eligible for
analysis. These included 67,725 patients with NSCLC and 13,290

PMPH_CH79.indd 632

patients with small cell carcinoma. Survival was calculated


by the Kaplan-Meier method. Prognostic groups were created
using Cox regression analysis and results were both internally
and externally validated. Stage groupings were revised to reflect
these analyses, and they were internally and externally validated.7 Selected changes in the AJCC 7th edition are shown in Table
79.2. TNM stage groupings are shown in Table 79.3.

AJCC 7TH EDITION NSCLC TUMOR (T)


Over 18,000 patients had a T1-T4 tumor with N0 lymph node dissection and an R0 resection.8 A running long-rank test was performed to assess cut points by tumor size. T1 was divided into T1a
(2 cm) and T1b (>23 cm). T2 was divided into T2a (>35 cm)
and T2b (>57 cm). T2c would have been >7 cm; however,
patients with a tumor size >7 cm had a survival that was statistically similar to survival of T3 patients. Therefore, lung cancers
>7 cm were categorized as T3. Survival curves based on tumor size
had excellent separation between each T designator (Figure 79.1).
Other T2 descriptors such as visceral pleural invasion and
partial atelectasis (less than the entire lung) could not be evaluated because of small number of patients and inconsistent data. In
the AJCC 7th edition, nodules in the same lobe were categorized
as T3; nodules in a different lobe were categorized as T4; a nodule in a contralateral lobe was designated as M1a unless there was
compelling evidence to suggest a management strategy for synchronous primary tumors.
T3 tumors may also be characterized as a tumor with invasion into the pleura, pericardium, or diaphragm; an endobronchial

5/22/2012 5:54:50 PM

Lung Cancer Staging

633

Table 79.2 Selected Changes in the AJCC 7th Edition NSCLC Tumor (T) and Metastasis (M) Characteristics
T/M Descriptor

T/M

N0

N1

T1 (2 cm)

T1a

IA

IIA

IIIA

IIIB

T1 (>23 cm)

T1b

IA

IIA

IIIA

IIIB

T2 (5 cm)

T2a

IB

IIA

IIIA

IIIB

T2 (>57 cm)

T2b

IIA

IIB

IIIA

IIIB

T2 (>7 cm)

T3

IIB

IIIA

IIIA

IIIB

T3 invasion

IIB

IIIA

IIIA

IIIB

T4 (same lobe nodules)

IIIB

IIIA

IIIA

IIIB

IIIA

IIIA

IIIB

IIIB

IIIA

IIIA

IIIB

IIIB

IV

IV

IV

IV

IV

IV

IV

IV

IV

IV

IV

IV

T4 (extension)

T4

M1 (ipsilateral lung)
T4 (pleural effusion)

M1a

M1 (contralateral lung)
M1 (distant)

M1b

N2

N3

Table 79.3 AJCC 7th Edition TNM Stage Groupings


Occult Ca

TX

N0

M0

Stage 0

Tis

N0

M0

Stage IA

T1a/b

N0

M0

Stage IB

T2a

N0

M0

Stage IIA

T2b

N0

M0

T1a/b; T2a

N1

M0

T2b

N1

M0

T3

N0

M0

Any T1; T2

N2

M0

T3

N1/N2

M0

T4

N0/N1

M0

Stage IIB

Stage IIIA

Stage IIIB

Stage IV

T4
Any T

N3

M0

Any T

Any N

M1a/b

tumor less than 2 cm from the carina; or an obstructing tumor


causing atelectasis of the entire lung; and as mentioned before,
two nodules in the same lobe.
T4 tumors would involve the mediastinal structures such as
the heart, great vessels, esophagus, and trachea, as well as the vertebral body or the carina. Two nodules, one each in two separate
ipsilateral lobes, would also be characterized as T4.
The characteristic of pleural metastases was changed from
T4 to M1. Patients previously categorized as a clinical T4 based
on a malignant pleural eff usion, malignant pericardial eff usions,
or pleural nodules, are now categorized as clinical M1 on the basis of
poor survival, which more closely resembles patients with metastatic disease.

PMPH_CH79.indd 633

M0

AJCC 7TH EDITION NSCLC LYMPH


NODES (N)
The nodal characteristic and designations did not change in the
AJCC 7th edition.9 Over 67,000 patients had T, N, and M characteristics as well as histologic type and survival. A total of 38,265
patients had cN and 28,371 patients had pN staging information. Clinical staging studies included tests such as diagnostic
imaging, computed tomography, and mediastinoscopy. Thoracotomy for staging was excluded. Positron emission tomography
(PET) was not widely used internationally in this cohort during
this time. A new international lymph node map was proposed
combining the integral aspects of the Japanese/Naruke and the

5/22/2012 5:54:50 PM

634

Surgery: Evidence-Based Practice


A m e r i c a n

J o i n t

C o m m i t t e e

o n

C a n c e r

Lung Cancer Staging

7t h E D I T I O N

Regional Lymph Nodes (N)

( Memorial Sloan-Kettering
Cancer Center, 2009.)

Financial support for AJCC 7th Edition Staging Posters


provided by the American Cancer Society

Co p y r i g h t 2 0 0 9 A m e r i c a n J o i n t Co m m i t t e e o n C a n ce r

ILLUSTRATION
The IASLC lymph node map shown
with the proposed amalgamation
of lymph into zones.

Printe d with per mission f rom the A JCC.

NX Regional lymph nodes


cannot be assessed
N0 No regional lymph
node metastases
N1 Metastasis in ipsilateral
peribronchial and/or
ipsilateral hilar lymph nodes
and intrapulmonary nodes,
including involvement
by direct extension
N2 Metastasis in ipsilateral
mediastinal and/or
subcarinal lymph node(s)
N3 Metastasis in contralateral
mediastinal, contralateral
hilar, ipsilateral or
contralateral scalene, or
supraclavicular lymph node(s)

2 of 2

Figure 79.1 American Joint Committee on Cancer 7th Edition Lung Cancer Staging Regional Lymph Node Map.
North American/Mountain lymph node maps.10 Of special note,
the authors proposed radiographic regions, particularly for integration with computed tomography, to guide the radiologic staging of patients with NSCLC.

AJCC 7TH EDITION NSCLC


METASTASES (M)
Metastases were divided into M1a and M1b. Patients with metastasis to the contralateral lung only were designated as M1a;
metastases to regions outside the lung/pleura were designated as
M1b. A second nodule in the nonprimary ipsilateral lobe, previously designated as M1, was changed to T4 M0. In this situation,
the patient received the benefit of the doubt approach as this
might represent a second primary.

COMMENTS ON THE AJCC 7TH EDITION


NSCLC STAGING
The AJCC 7th edition is based on a large international series
including centers from Europe, Japan, and North America. The
surgical foundation continues; however, other treatment categories are included and will be further refi ned. The data collected
were both internally and externally validated with training and

PMPH_CH79.indd 634

validation sets. External validation was assessed against the


Surveillance, Epidemiology, and End Results (SEER) program
database. The data collected were retrospective, and an audit of
the data was not performed. However, information was provided
by credible centers that facilitated data collection and analysis
of a large patient population. Future directions will most certainly include prospective data collection11 and proteomic and
genomic characteristics. Additional revisions should include
more rare cancer types, pleural malignancies such as mesothelioma, different treatment strategies, and even wider geographical representation.
In general, clinical stage or pathologic stage does not dictate treatment; it guides treatment. This critical statement must
be considered in application of the AJCC 7th edition NSCLC
staging system to patients. Optimal therapy depends on optimal staging and the clinicians best efforts to achieve optimal outcomes. A visual schematic has been created for the lymph nodes
(Figure 79.1)12 and T characteristics.13
Lung cancer clinical stage is an integral part of all patient care
and clinical trials. Clinical stage and pathologic stage is collected as
part of the Society of Thoracic Surgeons National General Thoracic
Surgery Database. Lung cancer clinical stage has been proposed
and approved by the Steering Committee of the National Quality
Forum as a Physician Level Perioperative Quality Indicator.
The AJCC 8th edition is planned for 2018. The IASLC staging
project papers are available to the public at http://www.jto.org.

5/22/2012 5:54:50 PM

Lung Cancer Staging

REFERENCES
1. Mountain CF, Dresler CM. Regional lymph node classification
for lung cancer staging. Chest. 1997;111(6):1718-1723.
2. Mountain CF. Revisions in the international system for staging
lung cancer. Chest. 1997;111(6):1710-1717.
3. Goldstraw P, Crowley JJ, IASLC International Staging Project. The
international association for the study of lung cancer international
staging project on lung cancer. J Thor Oncol. 2006;1:281-286.
4. Edge SB, Byrd DR, Compton CC, Fritz AG, Greene FL, Trotti AI.
AJCC Cancer Staging Manual. 7th ed. New York: Springer; 2009.
5. Lung. In: Edge SB, Byrd DR, Compton CC, Fritz AG, Greene FL,
Trotti AI, eds. AJCC Cancer Staging Manual. 7th ed. Springer;
2009:253-270.
6. Groome PA, Bolejack V, Crowley JJ, et al. The IASLC lung cancer staging project: validation of the proposals for revision of
the T, N, and M descriptors and consequent stage groupings in
the forthcoming (seventh) edition of the TNM classification of
malignant tumours. J Thorac Oncol. 2007;2:694-705.
7. Goldstraw P, Crowley J, Chansky K, et al. The IASLC Lung Cancer Staging Project: proposals for the revision of the TNM stage

PMPH_CH79.indd 635

8.

9.

10.

11.

12.
13.

635

groupings in the forthcoming (seventh) edition of the TNM Classification of malignant tumours. J Thorac Oncol. 2007;2(8):706-714.
Rami-Porta R, Ball D, Crowley J, et al. The IASLC Lung Cancer
Staging Project: proposals for the revision of the T descriptors in
the forthcoming (seventh) edition of the TNM classification for
lung cancer. J Thorac Oncol. 2007;2(7):593-602.
Rusch VW, Crowley J, Giroux DJ, et al. The IASLC Lung Cancer
Staging Project: proposals for the revision of the N descriptors
in the forthcoming seventh edition of the TNM classification for
lung cancer. J Thorac Oncol. 2007;2(7):603-612.
Rusch VW, Asamura H, Watanabe H, et al. The IASLC lung
cancer staging project: a proposal for a new international lymph
node map in the forthcoming seventh edition of the TNM classification for lung cancer. J Thorac Oncol. 2009;4(5):568-577.
Giroux DJ, Rami-Porta R, Chansky K, et al. The IASLC Lung
Cancer Staging Project: data elements for the prospective project. [Review] [25 refs]. J Thorac Oncol. 2009;4(6):679-683.
Lung Cancer Staging. AJCC 7th Edition. 2010. http://www.
cancerstaging.org/staging/posters/lung8.5x11.pdf.
Rice TW, Murthy SC, Mason DP, Blackstone EH. A cancer staging primer: lung. J Thorac Cardiovasc Surg. 2010;139(4):826-829.

5/22/2012 5:54:51 PM

CHAPTER 80

Primary Chest Wall Tumors


Adam H. Lackey, Joseph B. Levin, and Harvey I. Pass

wide resection and reconstruction.8-10 There are exceptions, and


multimodality treatments involving chemotherapy and radiation
therapy have shown to be more effective than surgery alone in certain malignant primary tumors.

INTRODUCTION
Primary chest wall tumors are a heterogeneous group of neoplasms arising from the bone, cartilage, or soft tissue of the thoracic cage. The differential diagnosis of these tumors is broad,
because the group encompasses a wide range of malignant and
benign diseases. In contrast to secondary chest wall tumors, which
can include local invasion from lung, breast, pleura, or even distant metastases, primary tumors originate from tissues of the
chest wall exclusively.1 Owing to their rarity and heterogeneity,
primary tumors of the chest wall present challenges in both accurate diagnosis and effective treatment methods.
The most common chest wall tumors are secondary, representing invasion from underlying adjacent malignancies or distant metastases. Primary tumors are rare, with an incidence of
1% to 2% of the population and accounting for only 0.04% of all
newly diagnosed cancers.2,3 Further, they represent only 5% of
all thoracic neoplasms and less than 2% of all primary tumors.4
Approximately 50% to 80% of chest wall tumors are malignant,
with about 55% originating from bone or cartilage and 45% from
soft tissue.2,5,6 Although primary chest wall tumors are diagnosed
in every age group, they are more common at the ends of the spectrum, in the elderly and children/adolescents. Patients developing malignant primary chest wall tumors tend to be older than
patients with benign neoplasms.2
In most cases involving neoplasms of the chest wall, surgery is
considered the gold standard. The first case of chest wall resection
to be published was in 1778, when Airman removed an osteosarcoma lesion from the ribs. Later, in 1820, Cittadini also reported
a case of a bony chest wall tumor resection. The first large series
was published by Parham in 1899, which described a perioperative mortality rate of 30% of malignant tumors involving the bony
chest wall.7 Since that time, advancements in anesthesia, surgical
techniques, and perioperative care have dramatically reduced the
morbidity and mortality rates associated with chest wall resection and reconstruction. Today, mortality rates for the procedure
range from 2% to 7%, and most chest wall tumors are treated with

CLINICAL PRESENTATION
Most patients with chest wall tumors present with a palpable,
enlarging mass. Pain is the most common symptom, but many
patients can also be asymptomatic. Chest wall tumors may also be
found incidentally on routine x-ray, although this occurs in less
than 20% of all cases.11 Pain resulting from a primary neoplasm can
be attributed to local invasion into adjacent structures, which most
commonly represents bony lesions invading the periosteum. Compared to lesions arising from bone, patients with soft-tissue tumors
are generally pain-free. Nearly all malignant tumors will cause
pain, but studies show that almost two-thirds of benign lesions
will also become painful as a result of musculoskeletal damage due
to growth and compression.2,5 Weakness and paresthesias are also
common symptoms among patients with chest wall tumors, resulting from the mass involvement with neurological structures, such
as the brachial plexus and spinal cord. Systemic symptoms such
as fever, malaise, fatigue, and weight loss can also be present
in patients with certain types of tumors such as Ewings sarcoma.
Differential diagnosis can be difficult based on the clinical
presentation of a patient with a primary chest wall neoplasm.
Owing to their rarity, pain is often attributed to musculoskeletal
conditions such as arthritis or physical trauma. This can result
in a significant amount of time between the onset of symptoms
and diagnosis. When a palpable mass is present, clinicians often
associate pain with malignancy although, as mentioned previously, the symptom does not possess high diagnostic value as both
malignant and benign lesions may cause pain. Clinical presentation assessment alone is not a reliable tool in the diagnosis of primary chest wall tumors, as there are no true signs or symptoms
that will determine if a lesion is malignant or benign.
636

PMPH_CH80.indd 636

5/22/2012 5:55:29 PM

Primary Chest Wall Tumors

IMAGING
Following physical examination, the diagnostic workup of a patient
presenting with a chest wall tumor should proceed to imaging studies to identify characteristic radiological features of the lesion. Chest
radiography (x-ray) is often the first imaging technique to be used
to determine the size, location, and rate of growth of the mass. In
addition, characteristic features such as calcification, ossification,
and bone involvement can be obtained. When employing chest
x-ray as a diagnostic tool, it is important to use the appropriate technique so that the desired clinical information will be ascertained.
Low-kilovoltage x-ray for bone imaging will more accurately define
calcifications and soft-tissue planes when compared with highkilovoltage techniques and should be used when assessing bony
involvement and tumor matrix characteristics.12 Although x-ray
should not be used as the only diagnostic study when evaluating a
chest wall mass, it can provide essential clinical data and in some
cases is the source of incidental tumor discovery.
Computed tomography (CT) is essential in the radiological evaluation of a patient with a chest wall tumor. The technique
possesses greater sensitivity than chest x-ray, and allows for more
detailed assessment of size, location, and morphology of the lesion.
Tumor involvement with local and adjacent structures such as
pleura, mediastinum, lymph nodes, and lungs can also be visualized with high quality.12,13 When intravenous contrast is used in
conjunction with CT scan, tumor angiogenesis and vascularity
can be evaluated. Owing to their sensitivity and ability to capture a
diverse set of tumor characteristics, CT scans are also used extensively to evaluate response to neoadjuvant therapy and during the
postoperative period to monitor tumor recurrence.
Magnetic resonance imaging (MRI) provides high-contrast
resolution, multiplanar imaging that is currently the preferred
imaging technique to assess tumors of the chest wall. This modality is ideal for distinguishing soft-tissue planes and can provide
detailed information on tumor relationship with the vascular system, neural invasion including the spine and epidural space, and
extension to the thoracic inlet. To obtain optimal image quality,
MRI techniques such as cardiac gating and respiratory compensation can reduce artifacts on images of anterior chest wall masses.
In addition, surface coils are employed to obtain detailed imaging
of surface chest wall neoplasms; in cases in which lesions have
greater intrathoracic extension, torso coils may be used.12
In many cases, the findings of CT and MRI imaging can determine if a chest wall lesion is benign or malignant based on characteristic radiographic features. The presence of a well-defined mass
lacking extension into adjacent structures, bone erosion rather than
destruction, and slow growth over a period of time are all indicative
of a benign lesion. Malignant processes, while in many cases possessing a nonspecific imaging profile, often present as large, poorly
marginated masses that infiltrate surrounding tissues. In addition,
these masses generally have low density necrotic areas and local bony
destruction.14 There are times when tumor characteristics, in both
benign and malignant lesions, can suggest a tissue of origin based
on imaging. Most chest wall masses, however, will require tissue
excision with histological analysis to confirm a diagnosis.
More recently, practitioners have turned to the use of positron
emission technology (PET) to improve the diagnostic capability of
CT scans. Although further studies are needed to evaluate the efficacy of PET/CT in the diagnosis of chest wall lesions specifically,
there are data that illustrate superior abilities of PET and PET/CT

PMPH_CH80.indd 637

637

in detecting, staging, restaging, and evaluating treatment response


in primary bone and soft-tissue sarcomas when compared with CT
alone.2,15-18 Other studies, however, recognize the limits of PET in
malignancy detection and staging, recommending that PET should
not replace CT/MRI for diagnosis or staging, but can help confirm
malignancy when the findings of conventional imaging methods
are equivocal.19

TECHNIQUES OF BIOPSY FOR


TISSUE DIAGNOSIS
In a patient presenting for evaluation for a chest wall mass, tissue diagnosis is of paramount importance as this will allow further decisions regarding imaging choices and further treatment.
The techniques of biopsy range from simple core needle biopsy or
fine needle aspiration done in the office under local sedation to
ultrasound-guided and CT-guided needle biopsies. The particular
technique used varies from patient to patient, and largely depends
on whether or not the mass is palpable and easily discernable from
underlying structures. In selected cases in which it is unclear if
the mass arises from the lung or intrathoracic structures versus
the actual chest wall, a video-assisted thoracic approach may be
more appropriate and may be done through one thoracoscopic
port.20 In all instances in which a thoracic mass is biopsied with
percutaneous techniques, the patient should be counseled that this
approach does raise the potential risk of recurrence along the needle track. Prior to any biopsy, the patient should be counseled about
the potential risk, and the approach should be oriented so that the
entire site can be removed en bloc at a later date if necessary.
Given the relative rarity of primary chest wall neoplasms,
immunohistochemical techniques may need to be employed to
determine the exact cell line from which the tumor arises. Although
almost all patients with primary chest wall neoplasms will have
surgical management of their disease, proper identification of the
tumor will allow tailoring of the surgical and chemotherapeutic
approach to the tumor, as well as allow identification of the patients
that are appropriate for neoadjuvant techniques.

TYPES OF PRIMARY CHEST WALL TUMORS


A variety of primary chest wall tumors, both malignant and benign,
can arise from various tissues of the thoracic cage. Table 80.1,
although not inclusive, lists some of the more common primary
chest wall lesions with respective appropriate treatments. Approximately 55% of primary chest wall tumors will arise from bone
or cartilage. The most common benign bony tumors are fibrous
dysplasia (3050%), osteochondroma (3050%), and chondroma
(1025%).3 Of the malignant bony lesions, chondrosarcoma, Ewings
sarcoma, osteosarcoma, and solitary plasmacytoma are most frequently observed. Primary tumors may also arise from soft tissue of
the thorax, accounting for approximately 45% of all primary chest
wall neoplasms. Common benign soft-tissue tumors are lipoma,
desmoid, giant cell tumors, and Schwannoma. Soft-tissue sarcomas
are the most frequent soft-tissue malignancies of the chest wall,
accounting for 45% of all primary malignant chest tumors.6 This
group includes malignant fibrous histiocytoma, synovial sarcomas,
rhabdomyosarcoma, and fibrosarcoma. Other sarcomas, such as
leiomyosarcoma and neuroblastomas, are uncommon in the chest.

5/22/2012 5:55:30 PM

638

Surgery: Evidence-Based Practice

Table 80.1 Types of Primary Chest Wall Tumors


Tumor (Benign)

Cell Type

Features

Treatment

Fibrous dysplasia

Osteoblast

Usually monostotic

Resection if painful

Osteochondroma

Osteoblast

Frequently ostochondral junction

Wide resection if symptomatic

Chondroma

Chondrocyte

Well-defined osteolytic lesion

Wide resection (cannot differentiate from


chondrosarcoma)

Langerhans cell
histiocytosis

Langerhans cell

Systemic symptoms (fever, malaise)

Excisional if single lesion, systemic treatment


if multiple

Osteoid osteoma

Osteoblast

Dull pain, more severe at night

Radiofrequency ablation

Osteoblastoma

Osteoblast

Pain most common symptom, not


worse at night

Complete resection if locally aggressive

Giant cell tumor

Giant cell Stromal


cells

Soap bubble appearance on


chest-ray

Excision vs. radiation, radiation may cause


degeneration to malignant

Lipoma

Lipomatous cells

May be intra- and extra thoracic

Resection if large or disfiguring

Lipoblastoma

Embryonal fat cells

Exclusively in infants and children

Wide resection

Desmoid tumor

Connective Tissue

Can grow intrathoracic

Surgical resection with negative margins

Schwannoma

Schwann cells

Extremely painful when biopsied

Surgical resection when possible

Tumor (Malignant)

Cell Type

Features

Treatment

Chondrosarcoma

Chondrocytes

Bone destruction, irregular


contours, intratumoral
mineralization

Resection, adjuvant radiotherapy

Osteosarcoma

Osteoblasts

Sunburst pattern on chest x-ray

Wide surgical resection, adjuvant


chemotherapy

Ewings sarcoma

Round cell type

Onion peel on chest x-ray

Neoadjuvant chemotherapy, wide resection,


possible adjuvant radiation

Soft tissue sarcoma

Mesoderm

Painless mass

Wide surgical resection, >5-cm margins

Solitary plasmacystoma

Plasmacytes

Lytic lesion with cortical thinning


on chest x-ray

Radiation therapy is primary treatment

The location of a neoplasm can often aid in the diagnosis of


a tumor, as many primary chest wall tumors are common to a
particular structure. Rib lesions comprise 50% of bony malignant
tumors and a majority of benign bony tumors of the chest wall.3
They can be derived from bone, cartilage, bone marrow, vascular, or neural structures. The most common benign primary chest
wall tumors occurring in the rib are fibrous dysplasia (usually
limited to one rib) and chondroma. Chondrosarcoma and osteosarcoma are the malignant rib lesions with the highest incidence;
however, Ewings sarcoma is the most common rib malignancy
among pediatric patients.3 Tumors located in the sternum are usually malignant, the most common types being osteosarcoma and
chondrosarcoma. Primary scapular tumors make up to 30% of all
malignant bony chest wall lesions and are a common site for the
rare, benign soft-tissue neoplasm elastofibroma dorsi.3 Bony lesions
can also occur in the clavicle, but this is relatively uncommon.

treatment may be withheld unless a patient experiences pain or


discomfort due to an enlarging mass, such is the case with fibrous
dysplasia, osteochondroma, and lipoma. In other tumor types,
combination chemoradiation therapy or ablation techniques have
proven to be more effective than surgery alone. When the primary
tumor is an Ewings sarcoma, or locally invasive breast or lung
cancer, neoadjuvant therapy may be indicated prior to definitive
resection, and the approach should be multidisciplinary.4 Solitary plasmacytoma is treated solely with radiation. Of the benign
tumor types, osteoid osteoma is treated with radiofrequency
ablation and rarely requires surgical resection. Although the role
of chemotherapy and radiation has been established in certain
tumor types, further study is necessary to describe the benefits in
others such as soft-tissue sarcomas.1-3

CONSIDERATIONS PRIOR TO RESECTION


TREATMENT
Most benign and malignant chest wall tumors are treated with
surgical resection due to the propensity for local invasion, recurrence, or resistance to chemotherapy. In certain benign types,

PMPH_CH80.indd 638

Prior to undergoing resection for a chest wall mass, all patients


should undergo workup for metastatic disease if the initial mass
is malignant. Patients who have extrathoracic spread of malignant neoplasms are not appropriate for resection for curative
intent, but resection may be indicated for palliation (i.e., in the

5/22/2012 5:55:30 PM

Primary Chest Wall Tumors

case of a painful or fungating lesion).21 In such cases, if the primary lesion is radiosensitive, consideration for radiation therapy
should be part of the treatment plan as this may provide local
control of symptoms without the morbidity of a chest wall resection. In prepubertal females, if the lesion is proven benign and
resection may impinge on the breast bud, consideration should
be given to deferring defi nitive treatment until after breast development in order to preserve symmetry. Finally, as with any
large resectional therapy, careful attention should be paid to the
patients nutritional status in the pre-, peri-, and postoperative
period and aggressive efforts should be undertaken to correct
any nutritional deficit.

TECHNICAL CONSIDERATIONS
OF RESECTION
The margin of resection depends on the type of tumor encountered. For benign tumors, grossly negative margins are adequate,
unless the tissue type of the tumor is prone to recurrence. In those
cases, a 2-cm margin of histologically negative tissue should be
the goal. Th is is also an appropriate margin for metastatic tumors
and locally invasive breast and lung cancer, although a histologically R0 resection is more important for survival, and frozen sections should be obtained if there is any doubt to the completeness
of the resection.22 Desmoid tumors may be the exception to this
rule as their high rate of recurrence may mandate a more generous resectional margin.23 Primary malignant tumors of the chest
should be resected with generous margins, with the primary
concern being the prevention of locally recurrent disease. Pathologically clear margins continue to be the ultimate goal, and
some studies have shown that this is more important than the
number of ribs resected or empiric margins.23 Other studies have
shown a 5-year survival benefit to resection with 4-cm versus
2-cm margins; correspondingly, for primary malignant tumors
of the chest a 4-cm margin is preferred.24,25 The entirety of any
directly involved rib should be resected, along with partial resection of adjacent ribs to accomplish control with margins noted
above. Any structures attached directly to the tumor should be
excised.24 Any directly involved lung should be resected en bloc
as an R0 resection is the goal. However, the surgeon should be
aware that resection of lung has been shown to increase morbidity and mortality, especially concordant chest wall resection and
pneumonectomy.26

639

WHEN TO RECONSTRUCT THE BONY


THORAX AFTER RESECTION
The decision to reconstruct the bony thorax is a point of controversy and depends to some extent on the location of the resection
and patient preference. As a general rule, resections of two ribs or
less can be constructed with local techniques and tissue transposition. Many authors agree that segmental resection of three ribs
or less does not require reconstruction of the bony thorax and that
defects of less than 5 cm may be treated similarly, although such
resections may be reconstructed for cosmetic reasons, especially
when anterior. Resections of four or more ribs are generally recommended to be reconstructed, although there is some debate on
this subject. As a general rule, the more compromised the patients
underlying pulmonary function, the more aggressive an approach
should be taken to stabilize the thorax, as this will optimize postoperative lung function and pulmonary reserve. Resections that
encompass the posterior aspect of the first four ribs do not need to
be reconstructed. Alternately, posterior resections that are more
inferior should be evaluated for the risk of entrapping the scapular
tip and reconstructed if this is a possibility.

METHODS OF CHEST WALL


RECONSTRUCTION
Muscle flaps are an attractive option for chest wall reconstruction
as they are derived from the patients own tissue, are well vascularized, and can be used to fill space left from the resection. Depending on the particular flap used, skin and subcutaneous tissues can
be harvested with the flap, aiding in reconstruction (Table 80.2).
Appropriate drainage of the spaces created by muscle flap transfer
is paramount to avoid postoperative seroma and poor flap apposition to the graft site.
When rigid stabilization of the chest is required, several options are available. Using methylmethacrylate sandwiched between
polyproylene mesh has been well described for reconstruction of
large chest wall defects, including situations in which part of the
entire sternum has been resected. This technique has the benefit
of being versatile and uses a replacement for the chest wall that
is initially malleable into almost any configuration in the operating room before the methylmethacrylate hardens into its final
shape. This technique requires either local tissue advancement
techniques to cover the prosthetic or coverage with local or free

Table 80.2 Local Musculocutaneous Flaps in the Thorax


Flap

Vascular Supply

Territory Covered

Latissiumus dorsi

Thoracodorsal artery

Entire ipsilateral chest wall

Pectoralis major

Thoracoacromial artery

Anterior chest

Rectus abdominis

Epigastric artery

Anterior and anteriolateral

Serratus anterior

Serratus artery (Thoracodorsal)

Lateral

External oblique

Posterior intercostal arteries

Lateral and inferior anterior

Pedicled omental flap

Gastroepiploic arteries

Limited by length only

Free flaps

Varied

Limited by arterial supply

Other

PMPH_CH80.indd 639

5/22/2012 5:55:30 PM

640

Surgery: Evidence-Based Practice

flaps and a skin graft for final coverage. As this is a foreign body,
it should not be used in situations where there is active infection
or questions about the sterility of the operative field (i.e. resection
of an open, fungating chest lesion) exists. Postoperative studies of
functional outcomes and patient satisfaction have shown this to
be a very effective, durable repair for patients undergoing chest
wall resection and reconstruction.8
Placement of titanium struts to bridge the bony gap has also
been described. This technique offers excellent stability due to
the rigidity of the metal used. Mesh can be attached to the titanium struts for improved cosmetic result and stability between
the struts. Because the titanium struts are not as occlusive as the
methylmethacrylate sandwich, fluid egress from the operative site
may be improved. This technique, much like the methylmethacrylate method described above, represents placement of a foreign
substance into the body, and should be used with caution in situations where infection is suspected. Outcomes following titanium
strut reconstruction of the thorax appear to be similarly excellent
from a functional standpoint.27
In patients in whom concerns for infection exist, other techniques must be used to avoid infection of the material used to
reconstruct the chest. A potential solution to this is now possible
with the advent of bioprosthetic materials that can be placed into
infected spaces with less concern of infection of the prosthetic.
The currently available bioprosthetics do not have the strength of
the rigid prosthesis discussed previously, but do have the benefit of
being somewhat more resistant to infection.27 In particular, using
these techniques may allow for staged repairs in which infected tissue is removed and formal resection is undertaken at a later time.
In current incarnation, given the lack of rigidity of the available
bioprosthetics, the use of these materials for resection of large chest
wall defects has not been studied in large patient series.
A definitive list of all available materials that can be used to
reconstruct the bony thorax is large and evolving, limited only
by the imagination of the surgeons reconstructing the chest and
the biomedical companies developing the products. In any chest
reconstruction scenario, the two goals of restoration of function
in the setting of adequate cosmetic result are paramount, and the
techniques available for reconstruction are often used in a complementary fashion to achieve both of these goals for the patient. The
ideal materials and methods for chest wall reconstruction have not
been found, and as more experience is being obtained with this
clinical situation, the standards of care will continue to evolve.

MANAGEMENT OF THE POSTRESECTION


PLEURAL SPACE
Following resection of a chest wall neoplasm, consideration must
be given to management of the pleural space. Any residual blood
or fluid should be evacuated from the pleural space as the chest is
closed, and the lung should be fully expanded. This is a separate consideration from the drainage systems employed to prevent seroma
formation under the flaps and grafts that are employed to close
the chest wall defect. In any case of chest wall resection in which
the pleural is violated, a closed suction drainage system should be
employed. In cases where lung has been resected or inadvertently
damaged during the resection, the closed drainage system allows
for management of any resultant air leaks. The particular drainage
system used can be at the discretion of the operating surgeon, but

PMPH_CH80.indd 640

should accomplish the dual goals of complete inflation of the lung


with full apposition of the parietal and visceral pleura, management of any air leaks, and complete drainage of any postoperative
fluid draining into the pleural space. In the event that the lung is
not able to fill the entire thoracic cavity after resection, pedicled
muscle flap or omental transposition into the intrathoracic cavity
should be undertaken to obliterate any space expected to remain
after chest wall reconstruction. This will help prevent postoperative accumulation of fluid into the space, assist in healing any air
leaks from the lung parenchyma, and avoid infection of the space.

PERI- AND POSTOPERATIVE CARE


The most common postoperative complications following chest
wall resection are pulmonary in nature, and significant effort
should be used to minimize this risk.9-10 An attempt should be
made to extubate the patient in the operating room or as soon
after the operation as possible. The concept of delayed extubation
for concerns of chest wall stability reflects a poor operative stabilization, and it is highly unlikely that any additional stability
would be generated in the first few days postoperatively. Standard
approaches to improving pulmonary function postoperatively
should be undertaken aggressively, such as incentive spirometry,
early ambulation, and aggressive pain control. Thoracic epidural
catheters should be used judiciously to this end. Risk stratification
and appropriate treatment for perioperative -blockade and deep
vein thrombosis (DVT) prophylaxis is appropriate in all patients.

COMPLICATIONS OF CHEST
WALL RESECTION
Chest wall resection for primary chest wall tumors yields good
oncologic results and adequate patient satisfaction in most patients.
The most common complication in the postoperative period is usually respiratory in nature in most patients. In a review of 197 patients
from the Mayo Clinic undergoing chest wall resection, postoperative
respiratory complications were noted in 24.4% of the patients, and
were the proximate cause of death in three patients.10 Similarly, in
a review of 200 patients over 25 years of chest wall resection,
Mansour et al. noted that 20% of patients developed a respiratory
complication (14% pneumonia, 6% acute respiratory distress syndrome [ARDS]).9 In this study, although the rate of pneumonia
was observed to be higher in the patients undergoing concomitant
lung and chest wall resection, mortality and intensive care unit
(ICU) length of stay did not differ between the two groups.
Prolonged air leak is uncommon outside the setting of concurrent lung and chest wall resection. Other complications particular to chest wall resection and reconstruction include seroma
due to the large potential spaces created during reconstruction,
flap or graft failure, and donor site complications such as hernias
for muscle flaps or large wounds in the setting of skin grafts.

FOLLOW-UP POSTRESECTION
All patients should be followed as dictated by their underlying
oncologic diagnosis. A postoperative visit should be scheduled on
discharge for evaluation of wound healing and seroma formation.

5/22/2012 5:55:30 PM

Primary Chest Wall Tumors

This is typically 10 to 14 days postdischarge, but may be sooner


for patients who have a wound issue or are discharged with active
wound care such as negative pressure wound therapy. A repeat CT
of the chest should be obtained in all patients once the inflammatory response has subsided (typically 36 months) to define a new
baseline for chest anatomy.
In the initial postoperative period, recurrent masses at the
resection site will most likely be seromas or other fluid collections.
However, depending on the type of reconstruction used, a change
in chest contour may represent a failure of the rigid prosthesis.
If the diagnosis of seroma is not immediately and definitively
evident on clinical examination, a CT scan should be obtained
to further delineate the problem. After the initial postoperative
period has passed, changes in the chest wall contour may represent recurrence of the underlying malignancy and will require a
CT scan and biopsy if a solid mass is found. The role of PET scan
in this situation is unclear as the operative site will be PET avid as
long as there is ongoing tissue inflammation, which may be prolonged as the body incorporates prosthetic material.
Patients who have stable pulmonary function do not need any
specific follow-up of their chest wall reconstruction. Serial pulmonary function tests are not indicated. In patients who present
with decreased pulmonary function, especially acutely, failure of
the rigid prosthesis or accumulation of fluid in the pleural space
should be suspected. While chest x-ray may be helpful, this situation will likely require CT scan to fully delineate the diagnosis.

REFERENCES
1. Kim JY, Hofstetter WL. Tumors of the mediastinum and chest
wall. Surg Clin North Am. 2010;90:1019-1040.
2. Shah AA, DAmico TA. Primary chest wall tumors. J Am Coll Surg.
2010;210:360-366.
3. Smith SE, Keshavjee S. Primary chest wall tumors. Thorac Surg
Clin. 2010;20:495-507.
4. Incarbone M, Pastorino U. Surgical treatment of chest wall tumors.
World J Surg. 2001;25:218-230.
5. Hsu PK, Hsu HS, Lee HC, et al. Management of primary chest
wall tumors: 14 years clinical experience. J Chin Med Assoc. 2006;
69:377-382.
6. Burt M. Primary malignant tumors of the chest wall. The Memorial Sloan-Kettering Cancer Center experience. Chest Surg Clin
N Am. 1994;4:137-154.
7. Parham FW. Thoracic resecion for tumors growing from the bony
wall of the chest. Trans South Surg Gynecol Assoc. 1898;11:223-363.
8. Weyant MJ, Bains MS, Venkatraman E, et al. Results of chest wall
resection and reconstruction with and without rigid prosthesis.
Ann Thorac Surg. 2006;81:279-285.
9. Mansour KA, Thourani VH, Losken A, et al. Chest wall resections
and reconstruction: a 25-year experience. Ann Thorac Surg. 2002;
73:1720-1725; discussion 5-6.

PMPH_CH80.indd 641

641

10. Deschamps C, Tirnaksiz BM, Darbandi R, et al. Early and longterm results of prosthetic chest wall reconstruction. J Thorac Cardiovasc Surg. 1999;117:588-591; discussion 91-92.
11. Sabanathan S, Salama FD, Morgan WE, Harvey JA. Primary
chest wall tumors. Ann Thorac Surg. 1985;39:4-15.
12. Tateishi U, Gladish GW, Kusumoto M, et al. Chest wall tumors:
radiologic findings and pathologic correlation: part 1. Benign
tumors. Radiographics. 2003;23:1477-1490.
13. Tateishi U, Gladish GW, Kusumoto M, et al. Chest wall tumors:
radiologic findings and pathologic correlation: part 2. Malignant
tumors. Radiographics. 2003;23:1491-1508.
14. Schaefer PS, Burton BS. Radiographic evaluation of chest-wall
lesions. Surg Clin North Am. 1989;69:911-945.
15. Volker T, Denecke T, Steffen I, et al. Positron emission tomography for staging of pediatric sarcoma patients: results of a prospective multicenter trial. J Clin Oncol. 2007;25:5435-5441.
16. Tateishi U, Hosono A, Makimoto A, et al. Comparative study of
FDG PET/CT and conventional imaging in the staging of rhabdomyosarcoma. Ann Nucl Med. 2009;23:155-161.
17. Piperkova E, Mikhaeil M, Mousavi A, et al. Impact of PET and CT
in PET/CT studies for staging and evaluating treatment response in
bone and soft tissue sarcomas. Clin Nucl Med. 2009;34:146-150.
18. Petermann D, Allenbach G, Schmidt S, et al. Value of positron
emission tomography in full-thickness chest wall resections for
malignancies. Interact Cardiovasc Thorac Surg. 2009;9:406-410.
19. Schuetze SM. Utility of positron emission tomography in sarcomas. Curr Opin Oncol. 2006;18:369-373.
20. McDonald JM, Freeman RK. Thoracoscopic localization of nonpalpable rib tumors for excisional biopsy. Ann Thorac Surg. 2000;
70:318-319.
21. Warzelhan J, Stoelben E, Imdahl A, Hasse J. Results in surgery
for primary and metastatic chest wall tumors. Eur J Cardiothorac
Surg. 2001;19:584-588.
22. Magdeleinat P, Alifano M, Benbrahem C, et al. Surgical treatment
of lung cancer invading the chest wall: results and prognostic
factors. Ann Thorac Surg. 2001;71:1094-1099.
23. Walsh GL, Davis BM, Swisher SG, et al. A single-institutional,
multidisciplinary approach to primary sarcomas involving the
chest wall requiring full-thickness resections. J Thorac Cardiovasc Surg. 2001;121:48-60.
24. Athanassiadi K, Kalavrouziotis G, Rondogianni D, Loutsidis A,
Hatzimichalis A, Bellenis I. Primary chest wall tumors: early and
long-term results of surgical treatment. Eur J Cardiothorac Surg.
2001;19:589-593.
25. King RM, Pairolero PC, Trastek VF, Piehler JM, Payne WS, Bernatz PE. Primary chest wall tumors: factors affecting survival. Ann
Thorac Surg. 1986;41:597-601.
26. Lardinois D, Muller M, Furrer M, et al. Functional assessment of
chest wall integrity after methylmethacrylate reconstruction. Ann
Thorac Surg. 2000;69:919-923.
27. Butler CE, Langstein HN, Kronowitz SJ. Pelvic, abdominal, and
chest wall reconstruction with AlloDerm in patients at increased
risk for mesh-related complications. Plast Reconstr Surg. 2005;
116:1263-1275; discussion 76-77.

5/22/2012 5:55:30 PM

CHAPTER 81

Tracheostomy: Timing and


Techniques
Matthew O. Dolich

INTRODUCTION

and pain in patients unlikely to survive more than a few days in


the ICU, and is therefore undesirable in that subpopulation of
patients requiring an artificial airway.
In a recent prospective, randomized, multicenter trial of 600
patients, Terragni and colleagues2 examined the effect of early
tracheostomy (after 68 days of translaryngeal intubation) versus late tracheostomy (after 1315 days of translaryngeal intubation) in reducing the incidence of pneumonia. Although there
were statistically significant reductions in ICU days and days
on mechanical ventilation in the early tracheostomy group,
long-term outcomes did not differ and there was no difference in the rate of ventilator-associated pneumonia. Given that
39% of the patients in each group experienced a tracheostomyrelated adverse event, the authors concluded that tracheostomy
should not be performed prior to 13 to 15 days of translaryngeal
intubation.
Sugerman and colleagues, 3 in a prospective study of 127
patients randomized to either early tracheostomy (days 35) or prolonged translaryngeal intubation, found no difference in mortality
rate, ICU length of stay, or pneumonia. However, this study was
notable for its limitations, including physician bias and randomization issues.
In another randomized, prospective, multicenter trial, Blot
and colleagues4 compared outcomes in patients undergoing early
tracheostomy (within 4 days) or prolonged translaryngeal intubation. Interim analysis after enrollment of 123 patients revealed no
differences in 28-day or 60-day mortality, duration of mechanical
ventilation, incidence of pneumonia, or other infectious complications. Tracheostomy appeared to afford greater patient comfort,
though this was not objectively evaluated in the study.
Zagli and colleagues5 retrospectively analyzed their experience with 506 percutaneous tracheostomies performed at a single
institution. They found no difference in mortality, pneumonia
incidence, or hospital length of stay in patients undergoing early
(1.9 +/ 0.9 days) versus late (6.8 +/ 3.8 days) tracheostomy. However, duration of mechanical ventilation and ICU length of stay
were significantly shorter in the group of patients who underwent

Tracheostomy is one of the most commonly performed procedures in the intensive care unit (ICU), and it is estimated that
as many as 10% of patients who require mechanical ventilation
for 3 or more days will undergo this procedure.1 Tracheostomy
provides a stable airway in patients who require prolonged
mechanical ventilation. It provides relatively easy access to the
lower airways for suctioning of tracheobronchial secretions,
and may help protect the oropharynx and larynx from damage related to prolonged translaryngeal intubation via either the
orotracheal or nasotracheal routes. Reduced work of breathing,
enhanced patient comfort, lower sedation requirements, more
rapid weaning from mechanical ventilation, diminished risk
of ventilator-associated pneumonia, and shorter length of stay
in the hospital or ICU have all been proposed as benefits of
transitioning from translaryngeal intubation to tracheostomy.
Despite descriptions of tracheostomy performed thousands of
years agoand common performance for almost a century
considerable controversy persists regarding selection of appropriate candidates for the procedure, as well as timing and
techniques. Th is chapter will review the literature regarding
these issues and will provide recommendations based on the
available evidence.
1. What is the optimal timing of tracheostomy?
Considerable debate has persisted regarding the appropriate timing of tracheostomy in patients requiring mechanical ventilation. Advocates of early tracheostomy cite improved patient
comfort, reduced complications from prolonged translaryngeal
intubation, better pulmonary toilet, and improved ability to
communicate. However, the procedure itself is not without risk,
and routine performance of early tracheostomy incurs the possibility of per forming unnecessary procedures in patients who
might have otherwise been extubatable if given more time for
recovery. In addition, early tracheostomy incurs extra cost, risk,

642

PMPH_CH81.indd 642

5/22/2012 5:56:04 PM

Tracheostomy: Timing and Techniques

early tracheostomy (13.3 vs. 16.7 days and 16.9 vs. 20.8 days, respectively; P < .0001).
In the systematic review and meta-analysis of randomized
and quasi-randomized controlled studies, Griffiths 6 found that
early tracheostomy (defi ned as within the fi rst week of mechanical ventilation) did not alter the risk of either mortality or pneumonia. However, early tracheostomy did significantly reduce
the duration of mechanical ventilation (8.5 days, 95% confidence interval 15.3 to 1.7) and ICU length of stay (15.3 days,
24.6 to 6.1). In another meta-analysis of trauma patients by
Dunham,7 the authors found that early tracheostomy had no
influence on mortality, pneumonia, or rates of laryngotracheal
injury.
Recommendation: Early tracheostomy does not appear to
influence mortality or incidence of pneumonia (Grade B). Conflicting evidence exists regarding any beneficial effect of early
tracheostomy on ICU length of stay and duration of mechanical ventilation. Considerable variation exists in the defi nition of
early tracheostomy (17 days).
2. What patient populations benefit from tracheostomy?
Despite several intuitive physiologic benefits from tracheostomy,
active debate continues regarding patient selection for performance of this procedure. Indeed, many of the recommendations
regarding tracheostomy are based on expert opinion or consensus
statements, rather than on credible evidence. The primary reason
for this relates to treatment selection bias confounding many of
the prospective studies of the relationship between tracheostomy
and outcome.
In an attempt to control for this type of physician bias, Clech
and colleagues8 utilized propensity scoring in a multicenter, prospective, observational cohort study of 2186 unselected medical and surgical patients requiring mechanical ventilation for
more than 48 hours. Two different models of propensity scores
were created using multivariate logistic regression. One hundred
seventy-seven (8.1%) received a tracheostomy. In both propensity
score models, tracheostomy did not improve ICU survival, and
there was no difference in survival whether tracheostomy was
performed early (within 7 days) or late (after 7 days). In fact, tracheostomy was associated with increased post-ICU mortality in
patients who were not decannulated on discharge from the ICU
(odds ratio 3.73 4.63, P = .003.008).
In a single-center, prospective, randomized study, Barquist9
studied adult trauma patients both with and without brain injury.
Patients requiring tracheostomy for facial trauma or neck injuries were excluded, and patients were randomized to early (before
day 8) or late (after day 28) tracheostomy. The study was halted
after enrollment of 60 patients when interim analysis revealed no
differences in duration of ventilatory support, ICU length of stay,
pneumonia rate, or mortality.
Although many practitioners recommend tracheostomy for
patients with severe traumatic brain injuries (TBIs), surprisingly
little data exist to support this practice. Ahmed10 performed a retrospective review of 55 patients with severe TBI who underwent
early (within 1 week) or late (after 1 week) tracheostomy. Early
tracheostomy was beneficial with regard to ICU length of stay
(19.0 +/ 7.7 days vs. 25.8 +/ 11.8 days, P = .008). However, no
differences were noted in hospital length of stay, pneumonia, or
mortality.

PMPH_CH81.indd 643

643

In a prospective, randomized study, Bouderka11 evaluated


trauma patients with isolated head injury and Glasgow Coma
Scale (GCS) score < 8. Patients were randomized on day 5 to
receive a tracheostomy within the next 24 hours, or prolonged
translaryngeal intubation. Although there was no difference in
mortality or rate of pneumonia, early tracheostomy was associated
with shorter duration of mechanical ventilation (14.5 +/ 7.3 days
vs. 17.5 +/ 10.6 days, P = .02). An additional benefit was noted
in the subgroup of patients who developed pneumonia, in that
the duration of mechanical ventilation was significantly shorter
after diagnosis of pneumonia (6.0 +/ 4.7 days vs. 11.7 +/ 6.7
days, P = .01).
Goettler et al.12 utilized multivariate logistic regression to
determine predictors for the need for tracheostomy in adult
trauma patients. They retrospectively reviewed 992 patients, 430
of who underwent tracheostomy. Injury severity score (ISS), age,
and pulmonary/neurologic risk factors were found to correlate
with the need for tracheostomy. Specifically, they found > 90%
risk of requiring tracheostomy in patients of any age with ISS
> 54, patients > 40 years old with ISS > 40, patients > 55 years old
with GCS = 3 after 24 hours, patients > 40 years old with paralysis
at any level, and patients > 55 years old with bilateral pulmonary
contusions.
Recommendation: In unselected patients, tracheostomy does
not appear to affect mortality (Grade C). Tracheostomy may offer
modest benefits in patients with brain injury in terms of duration
of mechanical ventilation and ICU length of stay (Grade C).
3. Is open or percutaneous tracheostomy preferable?
Tracheostomy was popularized by Jackson almost a century ago,
and for decades the procedure was typically performed in the
operating room. Though many tracheostomies are still performed
in this setting, improvements in ICU care and monitoring have
increasingly allowed performance at the bedside in the ICU.
Bedside tracheostomy avoids the time, cost, and risks associated
with transfer to the operating room. In 1985, Ciaglia13 described
bedside percutaneous dilatational tracheostomy as an alternative
to the traditional open technique. Although several variations
exist, the procedure is generally performed by insertion of a needle into the trachea, followed by a flexible guidewire. Dilatation
is achieved by passage of a series of plastic dilators of increasing
diameter, or by a single large conical dilator (Blue RhinoTM technique). After dilatation, a cuffed tracheostomy tube is inserted
over the guidewire using modified Seldinger technique. The procedure may be performed with or without bronchoscopic guidance. Percutaneous bedside tracheostomy has enjoyed increasing
popularity due to its relatively short learning curve, low complication rate, and short procedure time.
In a recent retrospective review of 1000 bedside percutaneous tracheostomies, Kornblith and colleagues14 observed a very
low complication rate in both low-risk (1.2%) and high-risk (1.7%)
patients. There was only one procedure failure (0.1%) requiring
cricothyroidotomy, and there were no deaths related to performance of tracheostomy. The authors concluded that based on
the safety of the procedure, bedside percutaneous tracheostomy
should be considered the gold standard.
Silvester and colleagues15 prospectively analyzed perioperative
and long-term complications in patients randomly assigned to percutaneous or open surgical tracheostomy. The overall procedural

5/22/2012 5:56:04 PM

644

Surgery: Evidence-Based Practice

complication rate was low (3.5%), and there were no significant


differences in short- or long-term complications between the two
groups. The authors concluded that both procedures were safe and
equivalent.
In another prospective randomized study, Wu et al.16 found
no difference in complication rates between percutaneous and
open tracheostomy, but did observe significantly shorter procedure times in the percutaneous group (22.0 +/ 12.1 minutes vs.
41.5 +/ 5.9 minutes, P < .001).
In a meta-analysis of open versus percutaneous tracheostomy,
Higgins and Punthakee17 reviewed 15 prospective randomized
controlled trials and found no significant difference in complication rates between open and percutaneous techniques. Case
length was shorter by almost 5 minutes (not including time for
transport) for percutaneous tracheostomy, and costs were lower
by US$456. In another systematic review and meta-analysis of
randomized controlled trials, Delaney et al.18 found a decreased
rate of wound infection favoring percutaneous tracheostomy
(OR = 0.28; 95% CI 0.160.49; P < .0005). Early and late complications were otherwise equivalent between the two groups.
Recommendation: Both open and percutaneous tracheostomy may be performed safely with low complication rates, and
both short- and long-term complication rates are roughly equivalent (Grade A). Percutaneous tracheostomy appears to have advantages with regard to both cost and time efficiency.
4. Is bronchoscopic guidance necessary to perform percutaneous tracheostomy safely?
Percutaneous tracheostomy may be performed with bronchoscopic
visualization via the endotracheal tube, or may be performed
blind without fiberoptic visualization. Proposed advantages of
bronchoscopic guidance include avoidance of posterior tracheal
injury, avoidance of endotracheal tube puncture, minimizing risk
of premature extubation, and decreasing risk of paratracheal insertion. However, bronchoscopy does add additional cost, and it may
result in hypercarbia in patients with smaller caliber endotracheal
tubes. In addition, operative times may be marginally increased,
and costly damage to the bronchoscope may be incurred by inadvertent needle puncture.
In a retrospective review of 183 patients, Tomsic et al.19 analyzed complications felt to have been correctable by utilization
of bronchoscopic guidance. These included needle puncture of
the endotracheal tube (2.2%), passage of the guidewire through
the Murphy eye of the endotracheal tube (1.6%), and injury of the
posterior trachea (1.1%). Although the overall complication rate
was low, the findings prompted the authors to adopt routine use
of bronchoscopic guidance for all percutaneous tracheostomies at
their institution.
In another retrospective study, Paran and colleagues20 reviewed their experience with blind percutaneous tracheostomy
in 61 patients. Three procedures were aborted due to anatomical
problems, and one patient required operative revision secondary
to bleeding. There were no other procedure-related complications,
and the authors deemed blind insertion a safe practice.
Capnography has been evaluated as an alternative to bronchoscopy for confirmation of endotracheal needle position prior
to guidewire insertion. In a small prospective study, Mallick
et al. 21 found no difference in operative times or complication
rates in patients undergoing tracheostomy with capnographic

PMPH_CH81.indd 644

confirmation of needle placement as compared with bronchoscopic guidance. In two cases where bronchoscopic guidance was
utilized, the bronchoscope was damaged by needle puncture and
required repair.
Recommendation: On the basis of limited data, percutaneous tracheostomy may be safely performed without bronchoscopic
guidance (Grade C). When bronchoscopic guidance is utilized,
care should be taken to avoid needle puncture of the bronchoscope and subsequent costly repairs.
5. Is routine chest x-ray necessary after performance of open or
percutaneous tracheostomy?
Because performance of a tracheostomy may be associated with
immediate complications such as bleeding, pneumothorax, hemothorax, or tube malposition, many centers obtain routine chest
radiographs after conclusion of the procedure. However, major
postoperative complications are relatively rare, and are frequently
detectable on physical examination or bedside assessment of the
ventilator. Thus, the utility of routine postprocedure chest radiography has been called into question.
Hoehne and colleagues22 retrospectively reviewed records
from 73 trauma patients who underwent percutaneous tracheostomy for prolonged mechanical ventilatory support. All patients underwent routine postprocedure chest radiography. One
patient required conversion to an open procedure secondary to
bleeding, and no immediate complications were diagnosed by
chest x-ray.
In a similar retrospective study, Datta et al.23 examined records
of 60 patients who underwent percutaneous tracheostomy. Two
patients (3.3%) had complications diagnosed by postprocedure
chest x-ray. Both procedures were noted to be technically difficult.
The remainder of postprocedure chest x-rays showed no complications and did not influence patient management.
In a more recent prospective cohort study of 239 patients,
Haddad et al. 24 compared posttracheostomy chest radiographs
with preprocedure fi lms and recorded management modifications based on radiographic findings. Atelectasis was the only new
posttracheostomy radiographic finding, occurring in 24 (10%)
patients. Although the finding of atelectasis spurred changes in
management such as ventilator alteration and chest physiotherapy
in 4% of the total patient population, the authors concluded that
routine chest x-ray had low diagnostic yield and changed management in a minority of patients.
In another prospective observational study, Kumar and colleagues25 evaluated the utility of chest x-ray after both routine and
technically difficult percutaneous tracheostomies performed in a
cohort of 384 patients. In the 93 patients who had a procedure
described as technically difficult, the rate of new findings on chest
x-ray was 7.5%. In the 252 patients who underwent uncomplicated
tracheostomy, the detection rate for new abnormalities was only
0.4%. From this data, the authors concluded that routine chest
radiography is justified after technically difficult or complicated
tracheostomy, but may be safely omitted after uncomplicated
procedures.
Recommendation: Routine postprocedure chest radiography
may be safely omitted in patients who undergo uncomplicated,
technically uneventful tracheostomy (Grade B). In cases where
technical difficulty is noted, postprocedure chest x-rays should be
obtained.

5/22/2012 5:56:04 PM

Tracheostomy: Timing and Techniques

6. What is the impact of tracheostomy on swallowing and


aspiration?
The presence of an artificial airway, either translaryngeal or transtracheal, may have an effect on both deglutition and incidence of
aspiration. This relationship may be impacted by other variables,
including the specific indication for prolonged mechanical ventilation and duration of pulmonary failure.
In a nonrandomized study, Sharma and colleagues26 prospectively assessed the incidence of swallowing dysfunction in trauma
and nontrauma patients with and without tracheostomy. Bedside
swallow studies and videofluoroscopy were used to assess deglutition and incidence of aspiration. The overall aspiration rate was
38% (36% in patients with tracheostomy, and 40% in patients without tracheostomy). Dysphagia was common, but rates were similar
in patients with and without tracheostomy (71% vs. 77%, P > .05).
In an attempt to clarify a causal relationship between tracheostomy and aspiration, Leder and Ross27 prospectively evaluated pre- and postprocedure aspiration data in 20 adult patients
undergoing tracheostomy. Nineteen of 20 patients (95%) exhibited
the same aspiration status before and after tracheostomy. All 12
patients who aspirated prior to tracheostomy continued to aspirate afterwards, and 7 of 8 (88%) patients who did not aspirate
before tracheostomy also did not aspirate afterwards (P > .05).
In a more recent follow-up study of 25 consecutive patients,
the same authors confirmed similar results.28 Twenty-two (88%)
patients demonstrated the same aspiration status or resolved aspiration posttracheostomy. Three patients were felt to exhibit new
aspiration posttracheostomy secondary to worsening medical
conditions, and four patients demonstrated resolution of aspiration after tracheostomy. On the basis of the results from these two
studies, the authors concluded that the presence or absence of a
tracheostomy is irrelevant to swallowing success or failure.
Recommendation: Patients requiring artificial airways are
at high risk for both swallowing dysfunction and aspiration. The
presence of a tracheostomy does not appear to significantly influence these risks (Grade C). Formal bedside swallowing evaluation
should be performed in patients who have undergone recent tracheostomy, prior to institution of oral intake.
7. Is conversion to tracheostomy necessary after emergent
cricothyroidotomy?
In certain emergent circumstances, it may be necessary to access
the airway via the cricothyroid membrane. Cricothyroidotomy
may be performed in the setting of severe maxillofacial trauma,
pharyngeal injury, or airway obstruction when intubation by either
the orotracheal or nasotracheal routes is difficult or impossible,

645

and ventilation is unachievable by bag valve mask (BVM) techniques. In these circumstances, the cricothyroid membrane is
the access point of choice due to its relatively superficial anatomical position and technically easy surgical approach. After
establishment of an emergency airway via cricothyroidotomy,
the traditional approach has been conversion to a tracheostomy,
particularly if the need for prolonged mechanical ventilation is
anticipated. However, this is not universally accepted, and several
authors have challenged this approach.
In a small retrospective study, Wright and colleagues29 reviewed
the records of 15 surviving trauma patients who had undergone
surgical cricothyroidotomy. Eight patients underwent subsequent
conversion to tracheostomy, whereas seven underwent prolonged
ventilation via cricothyroidotomy until decannulation. Mean duration of ventilation via cricothyroidotomy was 14.1 days (range, 241
days). There was no difference in acute complications between the
two groups, but there was a trend toward increased infectious complications in the group that underwent conversion to tracheostomy.
These findings suggest that routine conversion to tracheostomy
may not be necessary in all patients who have undergone emergent
cricothyroidotomy.
In another retrospective study, Hawkins et al.30 reviewed
records of 26 patients who underwent cricothyroidotomy and survived to hospital discharge. Seven patients underwent decannulation of the cricothyroidotomy without further airway procedures,
and 19 patients underwent conversion to tracheostomy. No patients
experienced cricothyroidotomy-related morbidity, irrespective of
subsequent conversion to tracheostomy.
Cricothyroidotomy has also been utilized in the elective management of critically ill patients requiring prolonged mechanical
ventilation. Rehm et al.31 retrospectively compared 18 patients
who underwent elective cricothyroidotomy in the ICU for prolonged ventilatory support with a matched cohort of patients who
underwent tracheostomy. One patient from the cricothyroidotomy
group required silver nitrate treatment for excess granulation
tissue; no other complications were noted. Rates of minor voice
change deemed insignificant by patients were identical between
the two groups.
In a more recent review32 of complication rates after cricothyroidotomy in trauma patients, Talving and colleagues found little
data to support the contention that cricothyroidotomy was associated with more frequent or severe complications, and concluded
that routine conversion to tracheostomy was not supported by the
available evidence.
Recommendation: Patients who undergo emergent cricothyroidotomy may not require routine conversion to tracheostomy
(Grade C).

Clinical Question Summary


Question

Answer

Grade of
Recommendation

References

1 What is the optimal timing of


tracheostomy?

Unclear. Early tracheostomy does not appear to


influence rates of mortality or pneumonia.

2-7

2 What patient populations benefit


from tracheostomy?

Patients with brain injuries may experience


modest benefits.

8-12

(Continued)

PMPH_CH81.indd 645

5/22/2012 5:56:04 PM

646

Surgery: Evidence-Based Practice

(Continued)
Question

Answer

3 Is open or percutaneous
tracheostomy preferable?

Open and percutaneous approaches are safe and


equivalent in terms of complication rates.

13-18

4 Is bronchoscopic guidance necessary


to perform percutaneous
tracheostomy safely?

No

19-21

5 Is routine chest x-ray necessary


after performance of open or
percutaneous tracheostomy?

No

22-25

6 What is the impact of tracheostomy


on swallowing and aspiration?

Minimal, though all patients with artificial airways


are at high risk for swallowing dysfunction.

26-28

7 Is conversion to tracheostomy
necessary after emergent
cricothyroidotomy?

No

29-32

REFERENCES
1. Durbin CG. Tracheostomy: why, when, and how? Resp Care.
2010;55(8):1056-1068.
2. Terragni PP, Antonelli M, Fumagalli R, et al. Early vs late tracheotomy for prevention of pneumonia in mechanically ventilated
adult ICU patients: a randomized controlled trial. JAMA. 2010;
303(15):1483-1489.
3. Sugerman HJ, Wolf L, Pasquale MD, et al. Multicenter, randomized prospective trial of early tracheostomy. J Trauma. 1997;
43(5):741-747.
4. Blot F, Similowski T, Trouillet JL, et al. Early tracheotomy versus
prolonged endotracheal intubation in unselected severely ill ICU
patients. Intensive Care Med. 2008;34(10);1779-1787.
5. Zagli G, Linden M, Spina R, et al. Early tracheostomy in intensive care unit: a retrospective study of 506 cases of video-guided
Ciaglia Blue Rhino tracheostomies. J Trauma. 2010;68(2):
367-372.
6. Griffiths J, Barber VS, Morgan L, et al. Systematic review and metaanalysis of studies of timing of tracheostomy in adult patients
undergoing artificial ventilation. BMJ. 2005;330(7502):1243.
7. Dunham CM, Ransom KJ. Assessment of early tracheostomy
in trauma patients: a systematic review and meta-analysis. Am
Surg. 2006;72(3): 276-281.
8. Clech C, Alberti C, Vincent F. Tracheostomy does not improve
the outcome of patients requiring prolonged mechanical ventilation: a propensity analysis. Crit Care Med. 2007;35(1):132-138.
9. Barquist EQ, Amortegui J, Hallal A, et al. Tracheostomy in ventilator dependent trauma patients: a prospective, randomized,
intention-to-treat study. J Trauma. 2006;60(1):91-97.
10. Ahmed N, Kuo YH. Early versus late tracheostomy in patients with severe traumatic head injury. Surg Infect. 2007;8(3):
343-347.
11. Bouderka MA, Fakhir B, Bouaggad A, et al. Early tracheostomy
versus prolonged endotracheal intubation in severe head injury.
J Trauma. 2004;57(2):251-254.

PMPH_CH81.indd 646

Grade of
Recommendation

References

12. Goettler CE, Fugo JR, Bard MR, et al. Predicting the need for
early tracheostomy: a multifactorial analysis of 992 intubated
trauma patients. J Trauma. 2006;60(5):991-996.
13. Ciaglia P, Firsching R, Syniec C. Elective percutaneous dilatational tracheostomy: a simple new bedside procedure; preliminary report. Chest. 1985;87(6):715-719.
14. Kornblith LZ, Cothren Burlew C, Moore EE, et al. One thousand
bedside percutaneous tracheostomies in the surgical intensive
care unit: time to change the gold standard. J Am Coll Surg. 2011;
212(2):163-170.
15. Silvester W, Goldsmith D, Uchino S, et al. Percutaneous versus
surgical tracheostomy: a randomized controlled study with longterm followup. Crit Care Med. 2006;34(8):2145-2152.
16. Wu JJ, Huang MS, Tang GJ, et al. Percutaneous dilatational tracheostomy versus open tracheostomya prospective, randomized, controlled trial. J Chin Med Assoc. 2003;66(8):467-473.
17. Higgins KM, Punthakee X. Meta-analysis comparison of open
versus percutaneous tracheostomy. Laryngoscope. 2007;117(3):
447-454.
18. Delaney A, Bagshaw SM, Nalos M. Percutaneous dilatational tracheostomy versus surgical tracheostomy in critically ill patients: a
systematic review and meta-analysis. Crit Care. 2006;10(2):R55.
19. Tomsic JP, Connolly MC, Joe VC, et al. Evaluation of bronchoscopic assisted percutaneous tracheostomy. Am Surg. 2006;
72(10):970-972.
20. Paran H, Butnaru G, Hass I, et al. Evaluation of a modified percutaneous tracheostomy technique without bronchoscopic guidance. Chest. 2004;126(3):868-871.
21. Mallick A, Venkatanath D, Elliot C, et al. A prospective randomized
controlled trial of capnography vs. bronchoscopy for Blue RhinoTM
percutaneous tracheostomy. Anaesthesia. 2003;58(9):864-868.
22. Hoehne F, Ozaeta M, Chung R. Routine chest x-ray after percutaneous tracheostomy is unnecessary. Am Surg. 2005;71(1):51-53.
23. Datta D, Onyirimba F, McNamee MJ. The utility of chest radiographs following percutaneous dilatational tracheostomy. Chest.
2003;123(5):1603-1606.

5/22/2012 5:56:05 PM

Tracheostomy: Timing and Techniques

24. Haddad SH, Aldawood AS, Arabi YM. The diagnostic yield and
clinical impact of a chest x-ray after percutaneous dilatational
tracheostomy: a prospective cohort study. Anaesth Intensive
Care. 2007;35(3):393-397.
25. Kumar VM, Grant CA, Hughes MW, et al. Role of routine chest
radiography after percutaneous dilatational tracheostomy. Br J
Anesth. 2008;100(5):663-666.
26. Sharma OP, Oswanski MF, Singer D, et al. Swallowing disorders in trauma patients: impact of tracheostomy. Am Surg. 2007;
73(11):1173-1180.
27. Leder SB, Ross DA. Investigation of the causal relationship
between tracheotomy and aspiration in the acute care setting.
Laryngoscope. 2000;110(4):641-644.

PMPH_CH81.indd 647

647

28. Leder SB, Ross DA. Confirmation of no causal relationship between tracheotomy and aspiration status: a direct confirmation
study. Dysphagia. 2010;25(1):35-39.
29. Wright MJ, Greenberg DE, Hunt JP, et al. Surgical cricothyroidotomy in trauma patients. South Med J. 2003;96(5):465-467.
30. Hawkins ML, Shapiro MB, Cue JI, et al. Emergency cricothyrotomy: a reassessment. Am Surg. 1995;61(1):52-55.
31. Rehm CG, Wanek SM, Gagnon EB, et al. Cricothyroidotomy for
elective airway management in critically ill trauma patients with
technically challenging neck anatomy. Crit Care. 2002;6:531-535.
32. Talving P, Dubose J, Inaba K, et al. Conversion of emergent cricothyrotomy to tracheotomy in trauma patients. Arch Surg. 2010;
145(1):87-91.

5/22/2012 5:56:05 PM

Commentary on
Tracheostomy: Timing and
Techniques
Matthew E. Lissauer and Thomas M. Scalea

In this well-constructed evidence-based review of the tracheostomy literature, Dr Dolich tackles several questions vital to the
care of the mechanically ventilated patient. Unfortunately, the
data can be contradictory and the findings divergent. It is for this
reason that there are many varied opinions on the role, timing,
and method of tracheostomy. Dr Dolich summarizes the available
literature well, but there are still significant gaps in knowledge
that need to be overcome.
Much has been written on the subject of when a tracheostomy
should be performed, but few randomized studies have been performed. The recent study by Terragni et al.1 is the largest randomized study to evaluate the timing of tracheostomy in the intensive
care unit (ICU). As pointed out in the review, there was no difference in ventilator-associated pneumonia or 28-day survival
with early tracheostomy. However, 31% of the early tracheostomy
group and 43% of the late group did not get a tracheostomy as
assigned. In a study by Blot et al.,2 16 of 61 patients assigned to the
prolonged intubation group received a delayed tracheostomy.
Determining the risk of prolonged ventilation early in the
clinical course of a patient in the ICU is difficult. Ventilator asynchrony as measured by ineffective triggering predicts increased
duration of mechanical ventilation.3 In surgical patients, a lung
injury score of 1 may predict duration of ventilation >15 days.4
However, none of these indicators has the sensitivity or specificity needed to be useful to predict the need for prolonged ventilation. These results suggest that prior to studying the question of
when a tracheostomy is indicated, we need much better studies to
understand who requires prolonged mechanical ventilation and,
therefore, may benefit from early tracheostomy.
Trauma patients are the most studied group in terms of
identifying who should get a surgical airway, but the results are
still disappointing. Prolonged mechanical ventilation in patients
with cervical spine injury is common, particularly if the level is
high.5,6 Therefore, this is one subgroup of patients that may benefit from tracheostomy. Brain injured patients may benefit from
an early surgical airway. That may be because some patients with
traumatic brain injury (TBI) do not need mechanical ventilation
but do require access to the trachea for suctioning secondary to
their depressed mental state. In the medical ICU, patients who
were deemed to require mechanical ventilation for more than
14 days also seemed to benefit from early tracheostomy.7 Once

again, 17% of those randomized to late tracheostomy avoided one,


again underscoring the difficulty in predicting who will need a
tracheostomy. More rigorous studies are needed to develop scores
predictive of prolonged ventilation in potential ICU survivors.
Ideally these would take into account both diagnosis and physiology. Once developed, a multicenter randomized trial evaluating
early tracheostomy (or tracheostomy in general) using this score
as either entry criteria or an independent variable can be developed. In the meantime, given the current state of the literature,
the individual clinician must evaluate the known risks and benefits of tracheostomy and decide if the procedure is beneficial for
their patient.
Once the decision to place a tracheostomy is made, the next
decision is method. The equivalence of percutaneous and open
tracheostomy in terms of outcomes and complications is nicely
described in the review. As it does not require the operating room
and has shorter operative time, percutaneous tracheostomy costs
less. With percutaneous tracheostomy now being more commonly
used, general surgery residents and trauma fellows perform fewer
open tracheostomies. They may become less competent in performing open surgical airways in general. In the course of most
general surgeons career, there is a high likelihood they will need
to perform an emergent cricothyroidotomy. Although cadaver,
animal, and simulation labs allow for training in noncritical situations, there is still a difference between simulated procedures
and real life experience. The first open surgical airway performed
by a physician should not be an emergent airway. Open tracheostomy allows a resident to gain experience with the feel of surgical
airways in a controlled situation with an attending physician to
teach them. Performing at least some open tracheostomies during residency has the potential to improve education and possibly
save lives.
If the percutaneous route of tracheostomy is chosen, most
would favor bronchoscopic guidance as it helps confirm placement and adds to surgeon comfort. Visualizing the airway and the
subsequent needle stick and guidewire placement alleviates much
concern. As with other questions in this review, the literature on
the subject is remarkably lacking. There are no randomized or controlled trials. Only descriptive data exist. In a recent prospective
observational cohort, the risks related to blind placebo controlled
trial (PCT) remained slim but real.8 In this carefully selected
648

PMPH_CH81.indd 648

5/22/2012 5:56:05 PM

Tracheostomy: Timing and Techniques

group of patients, there was a 10% risk of periprocedural complications including bleeding, transient oxygen desaturation, and
perforation of the endotracheal tube cuff. These were mostly minor
and the rate was similar to the described rate of complications
using the bronchoscope.9 There is almost no work comparing both
strategies head to head in a controlled manner. Dr Dolich is correct in stating the blind procedure is safe, but if the main risk of
bronchoscopic guidance is damage to the scope, most of us would
gladly trade the cost of fi xing a scope for potentially avoiding a
major airway complication on a patient.
The remainder of the questions in the review focus on management after the surgical airway has been obtained. Well explained is the Grade B recommendation that a chest x-ray is not
required after tracheostomy unless the procedure is technically
difficult. We would add that any clinical change in respiratory
status after tracheostomy requires x-ray evaluation as well, even
if the procedure was uncomplicated, though this would be good
clinical care even in the absence of airway manipulation. In
terms of dysphagia after artificial airway management, patients
who have not had a tracheostomy, but have been managed with
prolonged translaryngeal intubation should also have a bedside
swallow as dysfunction is common in patients intubated for
greater than 48 hours.10
Finally in regards to the question of an emergent cricothyroidotomy requiring conversion to formal tracheostomy, the literature suggests this is not necessary. Although we concur with
the literature and this review, the events surrounding an emergent cricothyroidotomy are often chaotic. The cricothyroidotomy
is often performed with an endotracheal tube, not a tracheostomy
tube. A trip to the operating room if and when the patient has
stabilized, affords the opportunity to ensure the surgical airway is
secure with an appropriate tube and that hemostasis is complete,
though of course we have no data to back this recommendation.
In summary, the literature provides a reasonable guide to
clinical questions surrounding the surgical airway. These questions are adeptly answered by Dr Dolich. Unfortunately, the literature is incomplete and unsatisfactory for many of these questions,

PMPH_CH81.indd 649

649

especially need and timing of tracheostomy. As such, future work


should be directed toward better answering these questions.

REFERENCES
1. Terragni PP, Antonelli M, Fumagalli R, et al. Early vs. late tracheotomy for prevention of pneumonia in mechanically ventilated
adult ICU patients: a randomized controlled trial. JAMA. 2010;
303(15):1483-1489.
2. Blot F, Similowski T, Trouillet JL, et al. Early tracheotomy versus
prolonged endotracheal intubation in unselected severely ill ICU
patients. Intensive Care Med. 2008;34(10);1779-1787.
3. de Wit M, Miller KB, Green Da, et al. Ineffective triggering predicts increased duration of mechanical ventilation. Crit Care Med.
2009;37(10):2740-2745.
4. Troche G, Moine P. Is the duration of mechanical ventilation
predictable? Chest 1997;112(3):745-751.
5. Romero J, Vari A, Gambarutta C, Oliveiro A. Tracheostomy timing
in traumatic spinal cord injury. Eur Spine J. 2009;18:1452-1457.
6. Como JJ, Sutton ER, McCunn M, et al. Characterizing the need
for mechanical ventilation following cervical spinal cord injury
with neurological deficit. J Trauma. 2005;59:912-916.
7. Rumbak MJ, Newton M, Truncale T, et al. A prospective, randomized, study comparing early percutaneous dilational tracheotomy
to prolonged translaryngeal intubation (delayed tracheotomy) in
critically ill medical patients. Crit Care Med. 2004;32:1689-1694.
8. Ahmed R, Rady RS, Siddique JIM, Iqbal M. Percutaneous tracheostomy in critically ill patients: 24 months experience at a
tertiary care hospital in the United Arab Emirates. Ann Thoracic
Med. 2010;5(1):26-29.
9. Polderman KH, Spijkstra JJ, de Bree R, et al. Percutaneous dilatational tracheostomy in the ICU: optimal organization, low complication rates, and description of a new complication. Chest.
2003;123:1595-1602.
10. Leder SB, Cohn SM, Moller BA. Fiberoptic endoscopic documentation of the high incidence of aspiration following extubation in
critically ill trauma patients. Dysphagia. 1998;13:208.

5/22/2012 5:56:05 PM

CHAPTER 82

Pneumothorax and Hemothorax


Joseph J. DuBose

INTRODUCTION

phenomena. In a recent examination conducted by Ball et al.,


investigators found that this entity may be present in up to 15% of
trauma patients undergoing CT imaging of the chest. The majority of these individuals prove asymptomatic and have no physical exam findings that would arouse suspicion for the presence
of pneumothorax, such as subcutaneous emphysema.1 Over the
past 5 years, it has subsequently become an accepted practice to
manage appropriately selected patients with occult pneumothorax without drainage.
In a study of 338 trauma patients with paired plain fi lms of
the chest and a chest CT after trauma, Ball and colleagues found
that among 103 identified pneumothoraces, 55% (57 of 103) were
occult. In their series, no complications were observed when
observation of the pneumothorax was chosen as management.
In contrast, 23% of patients undergoing thoracostomy had tuberelated complications or required repositioning of their tube on
follow-up imaging. Interestingly, when this group of investigators compared patients with occult pneumothorax to those with
overt collections visible on plain radiography, they found that
the CT estimated size of the collections in both groups were similar, suggesting that a number of overt pneumothoraces would also
have been appropriate for observation without the potential risk
associated with thoracostomy tube placement.2
In the largest review on the topic, Yadav and colleagues
conducted an evidence-based review of 411 articles on occult
pneumothorax management. They identified three randomized trials for consideration, comprising 101 patients. This group
found that there was no difference between observation and tube
thoracostomy placement for patients with occult pneumothorax
with regards to progression of pneumothorax, risk of pneumonia, hospital length of stay (LOS), intensive care unit (ICU) LOS,
empyema, or mortality. These findings suggest that observation
is an appropriate and safe management approach to patients with
occult pneumothorax.3
There remains, however, a concern for the potential increase
in size of an occult or small pneumothorax if observation is
selected in the setting of positive pressure mechanical ventilation requirement. Although this topic has not been particularly

Pneumothorax and hemothorax are commonly encountered


in both trauma and elective surgical practice. The identification and treatment of these entities remain a demanding task
that requires both effective utilization of diagnostic tools and
timely employment of therapeutic interventions. Many issues
surrounding the optimization of outcomes following these fi ndings, however, are either not well defi ned or remain controversial. Do all pneumothoraces require treatment? Are antibiotics
necessary for thoracostomy tube placement? What is the optimal
management algorithm for thoracostomy tube use? When can
these tubes be discontinued, and how should they be removed?
What is the significance and optimal treatment of a retained
hemothorax? These questions continue to warrant debate and
investigation.
1. Does every pneumothorax require treatment?
Just as there are multiple potential causes of pneumothorax, the
clinical manifestations of this entity can vary significantly. At
present, it remains unclear what amount of air within the pleural space requires drainage. It is clearly important to provide for
prompt drainage of any pneumothorax contributing to tension
physiology. Although the definition of symptomatic in this setting has not been particularly well defined, it is also prudent that
any pneumothorax causing significant pain or pulmonary compromise should also be evacuated. The optimal management of
small asymptomatic collections, however, remains controversial.
At present, no system for estimation of the size of pneumothorax has been adequately validated. As such, even the definition of size of pneumothorax remains a matter of contention. The
best examinations of this topic are found in the evolving trauma
literature. In recent years occult pneumothorax, or those pleural
space air collections visible on computed tomography (CT) of the
chest, but not on plain radiography, have merited some attention. With the increased utilization of CT in imaging for trauma,
the occult pneumothorax may be an increasingly appreciated
650

PMPH_CH82.indd 650

5/22/2012 5:56:37 PM

Pneumothorax and Hemothorax

well examined, Ouelet et al.4 have reported their experience with


a small trial of 24 stable trauma patients with occult pneumothorax who were randomized to observation or tube thoracostomy
drainage. These investigators found that, between these two management choices, there was no difference in the rates of respiratory distress, mortality, or median ICU or hospital LOS. The topic
requires additional study, however, and at present it remains prudent to place a thoracostomy tube in the treatment of occult pneumothorax when positive pressure ventilation is required, when
very close observation cannot be achieved, or when air transport
of the injured patient is required.
Answer: Although available literature suggests that occult
pneumothoraces can be safely observed without tube thoracostomy or percutaneous drainage, it remains difficult to extrapolate
this experience to all small air collections in the pleural space. It
is likely that small, asymptomatic pneumothoraces may be safely
observed in appropriately selected patients and settings. The role
of follow-up imaging for patients under observation, however,
has not been adequately examined (Level 2 evidence, Grade C
recommendation).
2. Are antibiotics necessary for thoracostomy tube placement?
The use of antibiotic prophylaxis for tube thoracostomy has been
examined in several populations, including both trauma patients
and patients requiring elective general surgery. The larger body
of examination, however, exists in the trauma literature. Demetriades et al., in 1991, compared penetrating trauma patients who
received a single dose of ampicillin at the time of thoracostomy
tube placement to counterparts who had antibiotics continued
until thoracostomy removal. These investigators found that the
incidence of pneumonia and empyema were the same between
these groups, concluding that there was no benefit to prolonged
antibiotics after trauma thoracostomy placement.5 Subsequent
studies that followed examined the importance of periprocedural
antibiotics for posttraumatic thoracostomy tube placement. The
study of this topic has remained convoluted, however, due to the
wide range and types of antibiotics utilized, as well as differences
in dosing. In addition, the outcome measures from these small
studies proved inconsistent in comparison. Two separate metaanalytic studies reported in 19926 and 1995,7 however, concluded
that prophylactic antibiotics made a significant impact on the
incidence of empyema when utilized for thoracostomy tube placement after trauma.
The Eastern Association for the Surgery of Trauma contributed to available knowledge of antibiotic prophylaxis when they,
in 2000, published a comprehensive review of the literature and
their accompanying guidelines on the topic. Th is group8 could
find no Level 1 or Level 2 evidence on the topic, and were left to
conclude that a first generation cephalosporin should be utilized,
but no longer than 24 hours after thoracostomy placement. Based
on Level 3 evidence, these investigators noted that available data
suggested that there may be a reduction in the incidence of pneumonia, but not empyema in trauma patients receiving antibiotics
with tube thoracostomy placement. Their review was followed by
a prospective, randomized, double-blind trial reported by Maxwell and colleagues in 2004,9 who found that, among 224 patients
requiring thoracostomy tube placement, presumptive antibiotic use did not significantly effect the incidence of pneumonia

PMPH_CH82.indd 651

651

after trauma. The investigators in this examination were not,


however, able to reach conclusion on the effects of antibiotic use
on empyema, as the rate of this infectious manifestation in both
of the arms of their study was too small to facilitate adequate
examination.
The question of prophylactic antibiotic utilization in the general thoracic surgery population has also been examined. Although
these studies are comparatively smaller in overall number, several
have been fairly well-designed prospective randomized trials. In a
study of 127 patients conducted by Aznar and colleagues in 1991,10
investigators randomized 127 patients undergoing thoracic surgical procedures to receive either a single perioperative dose of
cefazolin or placebo. These researchers found that cefazolin significantly reduced the subsequent incidence of wound infection
rate, but not that of empyema or pneumonia. Another examination by Olak et al. examined 228 consecutive patients admitted for
elective thoracotomy and lung resection, randomizing patients to
either a single perioperative dose of cefazolin or a six dose continuation regimen of the same antibiotic. This group found that
additional doses of cefazolin did not confer any clinically significant benefit with regards to any infectious complication beyond
that obtained from a single prophylactic dose.11
The use of antibiotic prophylaxis for thoracostomy treatment
of spontaneous pneumothorax has also bee studied. In 2006,
Olgac and colleagues12 examined 119 of these patients who did not
receive antibiotics. Although the investigators reported that 25%
of patients developed some induration around the entry site of the
chest tube, none of these required further treatment. They also
noted that no bacterial cultures from suspected sites of infection
revealed significant growth.
Answer: Available data from small prospective randomized
trials suggest that a single perioperative dose of cefazolin confers
adequate protection for elective thoracic surgery patients with
thoracostomy tubes. The trauma literature on the topic remains
inadequate, but presently available data suggest that utilization of a single generation cephalosporin for no more than 24
hours postprocedure may confer some protection against subsequent pneumonia development (Level 3 evidence, Grade C
recommendation).
3. What is the optimal management of a thoracostomy tube?
Once a thoracostomy tube has been placed, for either trauma or
elective surgical indications, a management plan to facilitate the
subsequent safe removal of the tube must be implemented. To this
end, the variables that are commonly manipulated include the
duration of use for phases of suction and water seal cycles until the
requirement for the tube has resolved. Practice patterns remain
variable, with mixed literature on the role and duration of the suction phase following placement.
In a prospective randomized trial reported by Davis and colleagues in 199413 investigators conducted a prospective randomized trial of 80 patients requiring tube thoracostomy for various
indications. They found that patients continued on suction until
thoracostomy removal required a shorter duration of tube use
(72.2 vs. 92.5 hours, P = .013), and there were more patients
requiring prolonged removal times (defi ned as >36 hours) in
the water seal group (P = .009). A more recent prospective randomized trial of 100 patients requiring tube thoracostomy after

5/22/2012 5:56:37 PM

652

Surgery: Evidence-Based Practice

penetrating thoracic trauma found that continuous low pressure suction promoted shorter duration of tube requirement,
increased the number of patients achieving full lung expansion,
prevented the development of clotted hemothorax or empyema
requiring intervention, and resulted in shorter hospital stays.14
In contrast, a number of authors have suggested that early
water seal use after tube thoracostomy placement may be beneficial. Martino et al.15 reported the results of a prospective randomized trial of 205 blunt and penetrating trauma patients in 1999.
Among patients who had thoracostomy output on suction at less
than 150 cc per 24 hours, the investigators randomized to either
immediate tube removal under suction or a 6-hour trial of water
seal followed by chest x-ray (CXR) prior to removal. These researchers found that there was no difference between the two groups with
regards to tube duration or hospital LOS. A subsequent prospective study reported in 200216 examined 68 patients undergoing
elective thoracic surgery procedures. All patients in this study
were subjected to a brief initial period of suction, followed by randomization to continued suction at 20 cm of water or water seal.
The water seal group was found to have shorter durations of air
leak (mean 1.5 vs. 3.3 days, P = .05) and shorter durations of tube
requirement (mean 3.3 vs. 5.5 days, P = .06). Another smaller study
conducted by Reed et al.17 randomized 29 patients with iatrogenic
or spontaneous pneumothorax to either 20-cm water, 10-cm
water, or water seal after all were initially treated with 1 hour of
suction at 20 cm water. This group found that there was no difference between the three groups with regards to successful removal
rates at 48 hours or the need for pleurodesis. The investigators concluded that early water seal is safe in the treatment of iatrogenic or
spontaneous pneumothorax.
Answer: The role of suction and water seal cycling following
thoracostomy tube placement requires additional examination.
Limited available evidence suggests that prolonged suction use
may decrease the number of clotted hemothoraces or empyema
requiring intervention after trauma. The preponderance of existing data suggests that early water seal is safe for use in patients
with pneumothorax and following elective procedures. Early
water seal may also decrease the duration of postoperative air leak
following elective thoracic surgical procedures (Level 1b evidence,
Grade B recommendation).
4. What is the appropriate approach to thoracostomy tube
removal?
Once the need for a thoracostomy tube has resolved, the tube
must be safely removed. The appropriate conduct of this seemingly simple action has inspired considerable debate. In particular, questions remain about the optimal phase or respiration
during which to remove the tube, the relative safety of removing a thoracostomy tube in a patient requiring positive pressure
ventilation, and the role of follow-up imaging once the tube has
been removed.
Traditional teaching has provided rationales for thoracostomy tube removal at either end-inspiration or end-expiration
during the respiratory cycle. In an effort to reconcile existing
opinions on the matter, Bell and colleagues at Yale18 conducted
a prospective randomized study of 102 trauma patients undergoing thoracostomy tube removal. They found that, when randomized to either end of the respiratory cycle spectrum, there was no

PMPH_CH82.indd 652

subsequent difference in postremoval recurrent pneumothorax


or enlargement of small pneumothorax (8% vs. 6%, P = 1.0). In
addition, there was no difference between the two methods with
regard to the need for subsequent repeat thoracostomy. Their findings suggest that the phase of the respiratory cycle during which
a thoracostomy tube is removed has no significant influence on
outcome.
The safety of removing a thoracostomy tube from a patient
requiring positive pressure ventilation has traditionally represented a concern for both trauma providers and practitioners of
elective surgery. Although prospective studies on the topic have
not yet been conducted, Tawil et al. did report the results of a sizeable retrospective review in 2010.19 In their examination of 234
thoracostomy removals, they compared the removal under positive pressure ventilation for 58% of patients to those of the 42%
of patients who underwent tube discontinuations while breathing
spontaneously unassisted. These researchers found that there was
no difference between the two groups with regard to pneumothorax rates or need for tube reinsertion, regardless of ventilatory
support requirement.
Postthoracostomy tube removal imaging has traditionally
been a mainstay of practice at most centers. Most commonly, this
consists of plain radiography follow-up at a prescribed interval.
Two groups, however, have suggested that the routine utilization of postremoval imaging may not be required for all patients.
In an initial report by Palesty et al.,20 investigators conducted a
5-year retrospective examination of 73 trauma patients undergoing thoracostomy tube removal. The researchers found that only
eight patients had postprocedure imaging reports that differed
from those of pre-removal fi ndings. Only two patients in this
study required subsequent reinsertion of tube thoracostomy and,
in each of these instances, the reinsertion was conducted based
on clinical appearance of the patient, and not radiographic data
alone. A more recent retrospective review conducted by Goodman and colleagues21 at the University of Cincinnatis level I
trauma center found that patients undergoing tube removal without subsequent radiographic imaging were less severely injured
and were less likely to have suffered penetrating thoracic injuries.
They found, however, that patients who did not undergo imaging
after thoracostomy removal had fewer CXRs overall and shorter
LOS after chest tube removal. These investigators estimated that
foregoing CXR in selected patients resulted in an annual decrease
in charges of $16,280 for their institution.
The role of postremoval radiographic follow-up has also been
examined among patients who require mechanical ventilation.
Pizano and colleagues22 examined 75 patients requiring ventilation who underwent removal and subsequent CXR at approximately 1, 10, and 36 hours. They found that, although postremoval
pneumothorax was identified in 12% of patients, all of these were
identified on the initial fi lms obtained between 1 and 3 hours. The
investigators concluded that plain radiography within this time
frame effectively identified postremoval pneumothorax among
mechanically ventilated patients.
Answer: The phase of the respiratory cycle during which a
thoracostomy tube is removed has not been shown to have any
impact on subsequent outcome. Available data supports the position that patients requiring positive pressure ventilation can
safely undergo thoracostomy tube removal. Limited retrospective
data suggest that appropriately selected patients may not require

5/22/2012 5:56:37 PM

Pneumothorax and Hemothorax

routine imaging after thoracostomy tube removal. Further prospective validation and randomized study is required before the
latter practice should be routinely advocated (Level 1b evidence,
Grade B recommendation).
5. What is the optimal management of retained hemothorax?
The diagnosis and optimal management of retained posttraumatic
hemothorax remains a problematic issue. It is generally accepted
that persistent blood within the thoracic cavity represents a concerning finding; primarily due to concern for the subsequent development of fibrothorax (trapped lung) and empyema. Although a
link between uncomplicated retained hemothorax and trapped
lung has been less well established,23 the presence of retained blood
within the chest has more clearly been identified as a risk factor for
the development for empyema.24-27 Although empyema remains
an infrequent complication of thoracic trauma,25,27 diagnosis and
management of this infectious process remains controversial,28-31
and the occurrence of empyema may be associated with significant
morbidity and mortality.32 For this reason, the establishment of the
ideal modality for the effective evacuation of retained hemothorax
has remained an area of active investigation.
The identification of individuals at greatest risk for subsequent
complications due to retained hemothorax is, however, problematic. Although liquefied hemothorax can frequently be effectively
drained with the placement of an initial or secondary thoracostomy tube, clotted and loculated collections may be more likely to
require more aggressive management for evacuation. The natural
history of retained hemothoraces, particularly smaller collections,
has also not been well defined.33,34 Although largely dependent on
the screening modality used, even the incidence of this entity has
not been well defined; although small studies have reported rates as
high as 10%.35 In addition, although CT appears a more sensitive and
specific modality by which to characterize and quantify retained
hemothorax,35 the effective use of radiographic assessment to stratify
risk and guide therapeutic decisions has remained elusive.29,33,34
Despite these controversies, several evacuation strategies for
retained hemothorax have been effectively used, including open
thoracotomy, thoracostomy, video-assisted thoracoscopy (VATS),
and the use of intrapleural fibrinolytics. Thoracotomy remains
the gold standard to which newer approaches are compared, but
this surgical approach can be associated with significant morbidity. Less invasive modalities are more commonly employed in the
modern era. VATS has emerged as an increasingly used modality
in recent years, although the use of intrapleural fibrinolytics has
garnered some interest.
Increasing experience with thoracoscopy has increased the
enthusiasm for the use of this modality to evacuate retained

PMPH_CH82.indd 653

653

hemothoraces.35-47 Although early VATS appears to be beneficial,


the definition of early has varied among available retrospective series and there is a paucity of prospective studies available
in the literature.37,38,40,44-47 However, in one prospective, randomized report of VATS use within 72 hours of initial thoracostomy
tube placement, Meyer et al.37 found that VATS at this interval
was associated with shorter hospital stays and lower hospital costs
compared to individuals randomized to additional thoracostomy
tube placement.
The use of intrapleural fibrinolytics for the degradation
and subsequent drainage of retained hemothorax has also been
investigated. The safe use of this approach for the treatment of
organized hemothorax and infectious collections of the pleural space has already been reported by several groups.29,35,48-65
In a review of studies from the Cochrane Database of Systematic Reviews reported in 2008, Cameron and Davies29 identified
seven randomized control trials examining the use of fibrinolytics for empyema and parapneumonic eff usions. These investigators found that fibrinolytics utilized in these settings safely
resulted in a significant decrease in the risk of requiring subsequent surgical drainage. Although these fi ndings appear to
demonstrate the utility of intrapleural fibrinolytic use, none of
these studies included posttraumatic retained hemothorax in
their examinations.
At present, no Level 1 evidence supporting the use of fibrinolytics for the treatment of posttraumatic retained hemothorax
exists. However, in one small, prospective observational study
reported by Kimbrell and colleagues, the use of fibrinolytics
resulted in successful resolution of residual hemothorax in 92%
of patients.59 Another limited retrospective examination conducted by Oguzkaya et al.35 compared the use of VATS to intrapleural streptokinase for management of posttraumatic retained
hemothorax, finding that the use VATS resulted in shorter hospital
stay and a decreased need for subsequent thoracotomy. Given the
documented success of these adjuncts in the treatment of pleural
space infections, the utility of fibrinolytics for the treatment of
retained hemothorax warrants further examination.
Answer: Reflective of the complexities in diagnosis and risk
stratification for patients with the fi nding of retained hemothorax, no single therapeutic approach has emerged as a superior
modality. At present, VATS appears to hold the most promise,
having been shown to decrease hospital stay and cost in one
small, randomized trial comparing the use of this approach to
additional tube thoracostomy placement. To date, however, no
prospective examination of less invasive techniques has shown
any treatment modality to prove superior in decreasing the
need for subsequent thoracotomy (Level 2b evidence, Grade C
recommendation).

5/22/2012 5:56:37 PM

654

Surgery: Evidence-Based Practice

Clinical Question Summary


Question

Answer

Level of
Evidence

1 Does every
pneumothorax
require treatment?

Occult pneumothoraces can be safely observed


without tube thoracostomy or percutaneous
drainage.

2-4

2 Are antibiotics
necessary for
thoracostomy
tube placement?

A single perioperative dose of cefazolin may


confer adequate protection for elective
thoracic surgery patients with thoracostomy
tubes. Among trauma patients, the use of
a single generation cephalosporin for no
more than 24 hours postprocedure may
confer some protection against subsequent
pneumonia development.

6-11

3 What is the optimal


management of a
thoracostomy
tube?

Thoracostomy suction may reduce clotted


hemothoraces or empyema requiring
intervention after trauma. Early water seal is
safe for use in patients with pneumothorax
and following elective proceduresand may
decrease the duration of postoperative air
leak.

1b

13, 14, 16, 17

4 What is the
appropriate approach
to thoracostomy tube
removal?

Thoracostomy tubes can safely be removed


anywhere in the respiratory cycle.
Patients requiring positive pressure ventilation can
safely undergo thoracostomy tube removal.

1b

18, 19

5 What is the optimal


management of
retained hemothorax?

Early VATS for retained hemothorax may


decrease hospital stay and cost.

2b

43, 44, 45

REFERENCES
1. Ball CG, Kirkpatrick AW, Feliciano DV. The occult pneumothorax: what have we learned? Can J Surg. 2009;52(5):E173-179.
2. Ball CG, Kirkpatrick AW, Laupland KB, et al. Incidence, risk factors, and outcomes for occult pneumothoraces in victims of major
trauma. J Trauma. 2005;59(4):917-924; discussion 924-925.
3. Yadav K, Jalili M, Zehtabchi S. Management of traumatic occult
pneumothorax. Resuscitation. 2010;81(9):1063-1068.
4. Ouellet JF, Trottier V, Kmet L, et al. The OPTICC trial: a multiinstitutional study of occult pneumothoraces in critical care. Am
J Surg. 2009;197(5):581-586.
5. Demetriades D, Breckon V, Breckon C, et al. Antibiotic prophylaxis in penetrating injuries of the chest. Ann R Coll Surg Engl.
1991;73(6):348-351.
6. Fallon WF, Jr, Wears RL. Prophylactic antibiotics for the prevention of infectious complications including empyema following
tube thoracostomy for trauma: results of meta-analysis. J Trauma.
1992;33(1):110-116; discussion 116-117.
7. Evans JT, Green JD, Carlin PE, Barrett LO. Meta-analysis of antibiotics in tube thoracostomy. Am Surg. 1995;61(3):215-219.
8. Luchette FA, Barrie PS, Oswanski MF, et al. Practice management
guidelines for prophylactic antibiotic use in tube thoracostomy
for traumatic hemopneumothorax: the EAST practice management guidelines work group eastern association for trauma.
J Trauma. 2000;48(4):753-757.
9. Maxwell RA, Campbell DJ, Fabian TC, et al. Use of presumptive antibiotics following tube thoracostomy for traumatic

PMPH_CH82.indd 654

10.

11.

12.

13.

14.

15.

16.

17.

Grade of
Recommendation

References

hemopneumothorax in the prevention of empyema and


pneumoniaa multi-center trial. J Trauma. 2004;57(4):742-748;
discussion 748-749.
Aznar R, Mateu M, Miro JM, et al. Antibiotic prophylaxis in
non-cardiac thoracic surgery: cefazolin versus placebo. Eur J Cardiothorac Surg. 1991;5(10):515-518.
Olak J, Jeyasingham K, Forrester-Wood C, Hutter J, al-Zeerah
M, Brown E. Randomized trial of one-dose versus six-dose cefazolin prophylaxis in elective general thoracic surgery. Ann Thorac Surg. 1991;51(6):956-958.
Olgac G, Aydogmus U, Mulazimoglu L, Kutlu CA. Antibiotics are
not needed during tube thoracostomy for spontaneous pneumothorax: an observational case study. J Cardiothorac Surg. 2006;1:43.
Davis JW, Mackersie RC, Hoyt DB, Garcia J. Randomized study
of algorithms for discontinuing tube thoracostomy drainage.
J Am Coll Surg. 1994;179(5):553-557.
Muslim M, Bilal A, Salim M, Khan MA, Baseer A, Ahmed M.
Tube thoracostomy: Management and outcome in patients with
penetrating chest trauma. J Ayub Med Coll Abbottabad. 2008;
20(4):108-111.
Martino K, Merrit S, Boyakye K, et al. Prospective randomized trial of thoracostomy removal algorithms. J Trauma. 1999;
46(3):369-371; discussion 372-373.
Marshall MB, Deeb ME, Bleier JI, et al. Suction vs water seal after
pulmonary resection: a randomized prospective study. Chest.
2002;121(3):831-835.
Reed MF, Lyons JM, Luchette FA, Neu JA, Howington JA. Preliminary report of a prospective, randomized trial of underwater

5/22/2012 5:56:38 PM

Pneumothorax and Hemothorax

18.

19.

20.
21.

22.

23.
24.

25.

26.
27.

28.

29.

30.
31.
32.
33.
34.
35.

36.

37.

38.

PMPH_CH82.indd 655

seal for spontaneous and iatrogenic pneumothorax. J Am Coll


Surg. 2007;204(1):84-90.
Bell RL, Ovadia P, Abdullah F, Spector S, Rabinovici R. Chest
tube removal: end-inspiration or end-expiration? J Trauma. 2001;
50(4):674-677.
Tawil I, Gonda JM, King RD, Marinaro JL, Crandall CS. Impact
of positive pressure ventilation on thoracostomy tube removal. J
Trauma. 2010;68(4):818-821.
Palesty JA, McKelvey AA, Dudrick SJ. The efficacy of X-rays after
chest tube removal. Am J Surg. 2000;179(1):13-16.
Goodman MD, Huber NL, Johannigman JA, Pritts TA. Omission of routine chest x-ray after chest tube removal is safe in
selected trauma patients. Am J Surg. 2010;199(2):199-203.
Pizano LR, Houghton DE, Cohns SM, Frisch MS, Grogan RH.
When should a chest radiograph be obtained after chest tube
removal in mechanically ventilated patients? A prospective study.
J Trauma. 2002 Dec;53(6):1073-1077.
Doelken P, Sahn SA. Trapped lung. Semin Respir Crit Care Med.
2001;22(6):631-636.
Aguilar MM, Battistella FD, Owings JT, Su T. Posttraumatic
empyema: risk factor analysis. Arch Surg. 1997;132(6):647-650;
discussion 650-651.
Eren S, Esme H, Sehitogullari A, Durkan A. The risk factors and
management of posttraumatic empyema in trauma patients.
Injury. 2008;39(1):44-49.
Sriussadaporn S, Poomsuwan P. Post-traumatic empyema thoracis in blunt chest trauma. J Med Assoc Thai. 1995;78(8):393-398.
Mandal AK, Thadepalli H, Mandal AK, Chettipalli U. Posttraumatic empyema thoracis: a 24-year experience at a major trauma
center. J Trauma. 1997;43(5):764-771.
Wong KS, Lin TY, Huang YC, Chang LY, Lai SH. Scoring system
for empyema thoracis and help in management. Indian J Pediatr.
2005;72(12):1025-1028.
Cameron R, Davies HR. Intra-pleural fibrinolytic therapy versus conservative management in the treatment of adult parapneumonic eff usions and empyema. Cochrane Database Syst Rev.
2008;(2)(2):CD002312.
Bilgin M, Akcali Y, Oguzkaya F. Benefits of early aggressive management of empyema thoracis. ANZ J Surg. 2006;76(3):120-122.
Molnar TF. Current surgical treatment of thoracic empyema in
adults. Eur J Cardiothorac Surg. 2007;32(3):422-430.
Wehr CJ, Adkins RB, Jr. Empyema thoracis: a ten-year experience. South Med J. 1986;79(2):171-176.
Stafford RE, Linn J, Washington L. Incidence and management
of occult hemothoraces. Am J Surg. 2006;192(6):722-726.
Meyer DM. Hemothorax related to trauma. Thorac Surg Clin.
2007;17(1):47-55.
Oguzkaya F, Akcali Y, Bilgin M. Video thoracoscopy versus
intrapleural streptokinase for management of post traumatic
retained haemothorax: a retrospective study of 65 cases. Injury.
2005;36(4):526-529.
Velmahos GC, Demetriades D, Chan L, et al. Predicting the need
for thoracoscopic evacuation of residual traumatic hemothorax:
chest radiograph is insufficient. J Trauma. 1999;46(1):65-70.
Meyer DM, Jessen ME, Wait MA, Estrera AS. Early evacuation of
traumatic retained hemothoraces using thoracoscopy: a prospective, randomized trial. Ann Thorac Surg. 1997;64(5):1396-1400;
discussion 1400-1401.
Morales Uribe CH, Villegas Lanau MI, Petro Sanchez RD. Best
timing for thoracoscopic evacuation of retained post-traumatic
hemothorax. Surg Endosc. 2008;22(1):91-95.

655

39. Cetindag IB, Neideen T, Hazelrigg SR. Video-assisted thoracic


surgical applications in thoracic trauma. Thorac Surg Clin. 2007;
17(1):73-79.
40. Casos SR, Richardson JD. Role of thoracoscopy in acute management of chest injury. Curr Opin Crit Care. 2006;12(6):584-589.
41. Carrillo EH, Richardson JD. Thoracoscopy in the management
of hemothorax and retained blood after trauma. Curr Opin Pulm
Med. 1998;4(4):243-246.
42. Navsaria PH, Vogel RJ, Nicol AJ. Thoracoscopic evacuation of
retained posttraumatic hemothorax. Ann Thorac Surg. 2004;
78(1):282-285; discussion 285-286.
43. Tomaselli F, Maier A, Renner H, Smolle-Juttner FM. Thoracoscopical water jet lavage in coagulated hemothorax. Eur J Cardiothorac Surg. 2003;23(3):424-425.
44. Velmahos GC, Demetriades D. Early thoracoscopy for the
evacuation of undrained haemothorax. Eur J Surg. 1999;165(10):
924-929.
45. Vassiliu P, Velmahos GC, Toutouzas KG. Timing, safety, and efficacy of thoracoscopic evacuation of undrained post-traumatic
hemothorax. Am Surg. 2001;67(12):1165-1169.
46. Heniford BT, Carrillo EH, Spain DA, Sosa JL, Fulton RL, Richardson JD. The role of thoracoscopy in the management of
retained thoracic collections after trauma. Ann Thorac Surg. 1997;
63(4):940-943.
47. Brutsche MH, Tassi GF, Gyorik S, et al. Treatment of sonographically stratified multiloculated thoracic empyema by medical
thoracoscopy. Chest. 2005;128(5):3303-3309.
48. Khalil BA, Corbett PA, Jones MO, et al. Less is best? The impact
of urokinase as the first line management of empyema thoracis.
Pediatr Surg Int. 2007;23(2):129-133.
49. Agarwal R, Aggarwal AN, Gupta D. Intrapleural fibrinolysis in
clotted haemothorax. Singapore Med J. 2006;47(11):984-986.
50. Aye RW, Froese DP, Hill LD. Use of purified streptokinase in
empyema and hemothorax. Am J Surg. 1991;161(5):560-562.
51. Misthos P, Sepsas E, Konstantinou M, Athanassiadi K, Skottis I,
Lioulias A. Early use of intrapleural fibrinolytics in the management of postpneumonic empyema. A prospective study. Eur J
Cardiothorac Surg. 2005;28(4):599-603.
52. Basile A, Boullosa-Seoane E, Dominguez Viguera L, et al. Intrapleural fibrinolysis in the management of empyemas and haemothoraces: our experience. Radiol Med (Torino). 2003;105(1-2):12-16.
53. Lim TK, Chin NK. Empirical treatment with fibrinolysis and
early surgery reduces the duration of hospitalization in pleural
sepsis. Eur Respir J. 1999;13(3):514-518.
54. Sahn SA. Use of fibrinolytic agents in the management of complicated parapneumonic effusions and empyemas. Thorax. 1998;
53(Suppl 2):S65-72.
55. De Gregorio MA, Ruiz C, Alfonso ER, Fernandez JA, Medrano J,
Arino I. Transcatheter intracavitary fibrinolysis of loculated
pleural eff usions: experience in 102 patients. Cardiovasc Intervent Radiol. 1999;22(2):114-118.
56. Davies RJ, Traill ZC, Gleeson FV. Randomised controlled trial of
intrapleural streptokinase in community acquired pleural infection. Thorax. 1997;52(5):416-421.
57. Davies CW, Lok S, Davies RJ. The systemic fibrinolytic activity
of intrapleural streptokinase. Am J Respir Crit Care Med. 1998;
157(1):328-330.
58. Jerjes-Sanchez C, Ramirez-Rivera A, Elizalde JJ, et al. Intrapleural fibrinolysis with streptokinase as an adjunctive treatment
in hemothorax and empyema: a multicenter trial. Chest. 1996;
109(6):1514-1519.

5/22/2012 5:56:38 PM

656

Surgery: Evidence-Based Practice

59. Kimbrell BJ, Yamzon J, Petrone P, Asensio JA, Velmahos GC.


Intrapleural thrombolysis for the management of undrained
traumatic hemothorax: a prospective observational study.
J Trauma. 2007;62(5):1175-1178; discussion 1178-1179.
60. Bouros D, Schiza S, Patsourakis G, Chalkiadakis G, Panagou P,
Siafakas NM. Intrapleural streptokinase versus urokinase in the
treatment of complicated parapneumonic effusions: a prospective,
double-blind study. Am J Respir Crit Care Med. 1997;155(1):291-295.
61. Bouros D, Schiza S, Siafakas N. Fibrinolytics in the treatment of
parapneumonic eff usions. Monaldi Arch Chest Dis. 1999;54(3):
258-263.
62. Bouros D, Schiza S, Tzanakis N, Drositis J, Siafakas N. Intrapleural urokinase in the treatment of complicated parapneumonic

PMPH_CH82.indd 656

pleural eff usions and empyema. Eur Respir J. 1996;9(8):16561659.


63. Diacon AH, Theron J, Schuurmans MM, Van de Wal BW, Bolliger CT. Intrapleural streptokinase for empyema and complicated parapneumonic eff usions. Am J Respir Crit Care Med.
2004;170(1):49-53.
64. Maskell NA, Davies CW, Nunn AJ, et al. U.K. controlled trial
of intrapleural streptokinase for pleural infection. N Engl J Med.
2005;352(9):865-874.
65. Tuncozgur B, Ustunsoy H, Sivrikoz MC, et al. Intrapleural
urokinase in the management of parapneumonic empyema:
a randomised controlled trial. Int J Clin Pract. 2001;55(10):
658-660.

5/22/2012 5:56:38 PM

Commentary on
Pneumothorax and Hemothorax
J. D. Richardson

20022 recommending a 24-hour course of narrow-spectrum antibiotics aimed at Gram-positive organisms seems prudent. However, if empyema is the endpoint, the incidence will be low and the
value of prophylaxis will be very difficult to prove except by a very
large contemporaneous multi-institutional study.
The third question revolves around the use of water seal or suction prior to chest tube removal. Not surprisingly, data from trials
using both techniques provide contradictory results. All of the trials use a variety of patients with different indications for tube placement. I am convinced (but cannot prove) that suction is preferential
for patients with a recent air leak or for the potential of a visceral
pleura that is not firmly opposed to the parietal pleura of the chest
wall. One can determine this fairly well clinically, by simply observing the fluctuation in the fluid column of a chest tube with deep
inspiration. If there is little variance on deep inspiration, the patient
could go to water seal (or likely have the tube pulled), whereas a significant fluctuation should cause continued suction to be employed.
Pulling a tube in such a scenario frequently leads to a pneumothorax and the lung may not stay fully expanded on water seal. I have
observed this in over 35 years of chest tube management but it is
hardly the stuff on a randomized trial.
Does the cycle of respiration matter when a tube is pulled,
and is post-pull imaging necessary? Studies show no difference
in which ventilating cycle a tube is pulled and preference for one
method over another is likely mythical. The articles cited regarding post-pull imaging display a problem with medical literature
that is often not recognized.3 A study with 73 trauma patients
treated over 5 years by chest tube would have little validity to me
(barely over one per month). In addition, an 11% rate of change
seems significant to me even though Goodman and colleagues
apparently did not.4 They noted a whooping annual savings of
$16,280 by foregoing a post-pull radiograph.1 I would submit that
one adverse outcome for lack of a follow-up film will likely negate
such miniscule saving.
Finally, the author reviews the question of the optimal management of retained hemothorax. Our group has published extensively on the use of video-assisted thoracoscopy (VATS) for this
problem as we believe in its use.5-7 Having admitted our bias, we
would hasten to add we have an extremely busy unit where retained
collections are a potential problem; we have surgeons skilled in the
procedure, and we use it reasonably aggressively. The EAST recently
issued guidelines on VATS for retained hemothorax that noted
class II and III evidence supporting the use of VATS.8 However, I

The chapter entitled Pneumothorax and Hemothorax by Dr


Joseph J. Dubose provides excellent insight into the level of evidence supporting clinical decisions made in five fundamental
areas relevant to the management of pleural space problems; that
is, pneumothorax and hemothorax. The questions addressed
include (1) Does every pneumothorax require treatment? (2) Are
antibiotics necessary for thoracotomy tube placement? (3) What
is the optimal management of a thoracostomy tube? (4) What is
the appropriate approach to thoracostomy tube removal? and (5)
What is the optimal management of retained hemothorax?
Although these are all important questions, the level of evidence supporting recommendations on each of these questions
is often inadequate to permit firm conclusions. Reports on these
questions are generally experiential and, with the possible exception of the antibiotic question (#2), have rarely been subjected to
randomized clinical trials. In addition, many of the adverse events
that could be attributed to a particular treatment method are
uncommon: thus, studies would have to be enormously powered,
with many subjects, to show a statistical difference. Having stated
this, the author does an excellent job in reviewing the available
literature supporting, refuting, or being an agnostic on these fundamental questions.
Regarding the necessity to treat every pneumothorax, the
reviewed literature suggests that observation is safe with most
small pneumothoraces. The author, however, offers a note of caution (properly in my opinion) about observing and not inserting a tube in a patient with positive pressure ventilation (PPV).
The exacerbation of a small pneumothorax by PPV is not a myth;
I have observed it several times in my career including a nearfatal event in an operating room for an orthopedic procedure.
However, statistically the occurrence is likely fairly small. Thus
the conundrum: most small studies will disclose no problems;
however, do we offer no treatment to prevent known chest tube
complications at the peril of an occasional (perhaps rare) lifethreatening event?
The second question about the use of antibiotics for chest
tube insertion has long been a subject of interest to me. I was a coauthor of the first randomized, double blind control trial on this
subject published in 1997.1 Although several recent reports imply
that no randomized trials have occurred, our study of 75 patients
with penetrating trauma (38 antibiotics and 37 placebos) demonstrated positive effects in the antibiotic group. Perhaps the results
would differ in blunt trauma patients, but the EAST guidelines of
657

PMPH_CH82.indd 657

5/22/2012 5:56:38 PM

658

Surgery: Evidence-Based Practice

concur with Dr Duboses summary statement that this is an area


that can be managed by multiple strategies and no evidence to the
superiority of one over another has been forthcoming to date.

REFERENCES
1. Grover FL, Richardson JD, Fewel JG, Arom KV, Trinkle JK, Webb
GE. Prophylactic antibiotics in the treatment of penetrating chest
woundsa prospective double blind study. J Thorac Cardiovasc
Surg. 1977;74(4):528-536.
2. Luchette FA, Barrie PS, Oswanski ME, et al. Practice management
guidelines for prophylactic antibiotic use in tube thoracostomy
for traumatic hemopneumothorax: the EAST practice management guidelines work group. Eastern Association for Trauma. J
Trauma. 2000;48(4):753-757.

PMPH_CH82.indd 658

3. Palesty JA, McKelvey AA, Dudrick SJ. The efficacy of X-rays after
chest tube removal. Am J Surg. 2000;179(1):13-16.
4. Goodman MD, Huber NL, Johannigman JA, Pritts TA. Omission
of routine chest x-ray after chest tube removal is safe in selected
trauma patients. Am J Surg. 2010;199(2):199-203.
5. Carrillo EH, Heniford BT, Richardson JD, et al. Video-assisted
thoracic surgery in trauma patients. J Am Coll Surg. 1997;184:
316-324.
6. Heniford BT, Carrillo EH, Richardson JD, et al. The role of thoracoscopy in the management of retained thoracic collections after
trauma. Ann Thorac Surg. 1997;63:940-943.
7. Carrillo EH, Richardson JD. Thoracoscopy for the acutely injured
patient. Am J Surg. 2005;190:234-238.
8. Mowery NT, Gunter OL, Collier BR, et al. Practice management
guidelines for management of hemothorax and occult pneumothorax. J Trauma. 2011;70:510-518.

5/22/2012 5:56:38 PM

PART XIV

VASCULAR SYSTEM

PMPH_CH83.indd 659

5/22/2012 5:57:07 PM

PMPH_CH83.indd 660

5/22/2012 5:57:08 PM

CHAPTER 83

Abdominal Aortic Aneurysm


Boulos Toursarkissian

death was reduced by 68%.2 The third trial was the Multicenter
Aneurysm Screening Study that randomized 33,839 men aged 65
to 74 years to screening and 33,961 to a control group. AAA-related
mortality was reduced in the screened patients after 4 years.3
There is Level I evidence that AAA screening is effective in
reducing AAA-related mortality. Screening, however, should be
directed at individuals who are at higher risk for the disease. The
US Preventive Services Task Force recommends screening men
aged 65 to 74 years who have ever smoked (Grade B recommendation). Women with a family history of AAA should also likely be
included (Grade B recommendation).

Abdominal aortic aneurysm (AAA) remains a common vascular surgical problem, with roughly 40,000 annual repairs in the
United States alone. It is also one of the most well-studied problems, both in terms of natural history and procedures for repair.
This chapter will summarize the evidence available regarding the
most relevant clinical issues related to AAAs.
1. What are the risk factors for AAA development?
In a Veterans Administration study, the relative risk of developing
an aneurysm greater than 4 cm in diameter was five-folds higher
in smokers as opposed to nonsmokers.1 Men were also 5.6 times
more likely than women to have an AAA. Caucasians had a twofold higher risk than non-Caucasians and persons with a positive
family history were also twice as likely to have an AAA versus
individuals with a negative family history. Diabetes is negatively
correlated with AAA development.2,3 AAA is mostly a diseased of
advanced age, with few cases reported in individuals under the
age of 50 years.
The mean growth rate for AAAs less than 5.5 cm ranges from
2.6 to 3.2 mm per year, being higher in larger aneurysms.3 AAA
expansion is most strongly associated with diameter at baseline.
Other risk factors include continued smoking.
There is Level I evidence that smoking, male sex, white race,
and positive family history increase the likelihood of a patient harboring an AAA.

3. Are there any proven medical therapies for AAAs?


The UK Small Aneurysm Trial showed in a multivariate analysis
that ongoing smoking was linked to increased AAA expansion
rates4 (Level I evidence). Continued smoking has been associated
with more rapid aneurysm expansion.5 Patients diagnosed with
AAA should therefore be counseled to stop smoking (Grade A
recommendation).
-Blockers have been shown not to be beneficial in terms of
reducing AAA growth rates in prospective randomized trials6,7
(Level I evidence). They are not therefore routinely recommended
unless indicated for other cardiovascular indications (Grade A
recommendation).
Some retrospective studies have suggested that statin use is
associated with slower AAA expansion rates8-10 (Level II evidence).
However, there are no prospective randomized trials proving that
point. Statin use should therefore be decided on its own cardiovascular merits for secondary atherosclerosis prevention and not
for AAA management per se (Grade B recommendation).
Three small randomized trials have shown that doxycycline
and roxithromycin reduce AAA expansion rates, likely through
inhibition of wall degrading matrix metalloproteinases.11-13 The
total number of patients in these studies was rather small (Level IB
evidence). It is therefore reasonable to consider the use of such
medications in patients who can otherwise tolerate them (Grade B
recommendation).

2. Should screening for AAA be carried out?


There have been three randomized controlled trials addressing
this question. The first such study was carried out in Chichester
in the United Kingdom and enrolled 6058 individuals aged 65 to
80 years.1 AAAs greater than 3 cm were detected in 7.6%. AAArelated death was reduced by 41% at 5 years, when compared to
men not offered screening. No such benefit was seen in women
in whom the prevalence of AAA was only 1.3%. The second trial
was carried out in Denmark and involved 12,658 men aged 65 to
73 years.2 AAA was diagnosed in 4% and in-hospital AAA-related
661

PMPH_CH83.indd 661

5/22/2012 5:57:08 PM

662

Surgery: Evidence-Based Practice

4. What is the risk of AAA rupture and when should repair be


recommended?
The Aneurysm Detection and Management trial (ADAM trial)
randomized veterans aged 50 to 79 years with AAA from 4.0 to
5.4 cm to either immediate open surgical in 569 patients repair
or to ultrasound surveillance in 567 patients, with repair if the
AAA reaches 5.5 cm or becomes symptomatic. With an operative mortality of 2.7%, there was no significant difference between
the two groups in the primary outcome of long-term mortality,
with a mean follow-up of 4.9 years. Sixty-two percent of patients
in the observation group ended up with repair by the end of the
observation period. Only 0.6% per year of the observation patients
suffered a ruptured AAA. The likelihood of needing repair in
the observation group varied with the initial size of the AAA at
enrollment, with 81% of those in the 5- to 5.4-cm group needing repair by the end of the study. Ultrasound follow-up was done
every month in patients randomized to observation.
The UK small aneurysm trial randomized 1090 patients aged
60 to 76 years with AAA of 4.0 to 5.5 cm to either open surgical
repair or periodic ultrasound surveillance.14 For those observed,
repair was done if the AAA exceeded 5.5 cm on follow-up ultrasound, or grew by more than 1 cm in a year. Follow-up averaged
4.6 years. Ninety-three percent of patients adhered to the assigned
treatment. The 30-day operative mortality averaged 5.8%, and all
repairs were open. At 2, 4, and 6 years, there was no difference
in the main endpoint mortality between the two groups. Sixty
percent of the patients in the observation group ended up getting
the AAA repaired at a median of 2.9 months after enrollment. The
operative mortality in that group was 7%. The initial size of the
AAA did not correlate with the need for operative repair, a finding
different form the ADAM trial.
The UK trial also followed a cohort of patients with AAAs
who were not randomized for a variety of reasons including
patient refusal or unfitness. This cohort was followed prospectively.15 Seventy-six percent of the ruptures occurred in AAA larger
than 5 cm. The risk of rupture was associated with female sex,
larger initial diameter, current smoking, and higher mean blood
pressure. These risk factors have been reported in other retrospective studies. A low forced expiratory volume has also been associated with risk of rupture.
There were no women in the ADAM trial and only 17% of
participants in the UK trial were female. Some retrospective studies have suggested that at a given size, women have higher risk of
rupture, and a higher mortality when rupture occurs. Therefore
the results of the above studies should be interpreted with caution
in women.
Since both the ADAM trial and the UK small aneurysm trial
were of open repair versus observation, another trial was conducted of EVAR versus observation in patients with AAA sized
4 to 5 cm. The PIVOTAL (Positive Impact of Endovascular Options
for Treating Aneurysms Early) trial randomized 366 such patients
to early EVAR and 362 patients to periodic ultrasound surveillance.16 After a mean of 12 months, there was no difference in
overall mortality between the two groups (4.1%) or AAA-related
mortality (0.6%).
Retrospective studies have shown that the risk of rupture
increases exponentially once the diameter exceeds 6 cm.
In summary, in men, it seems reasonable to delay surgical treatment until AAA diameter size reaches 5.5 cm (Level I

PMPH_CH83.indd 662

evidence, Grade A recommendation). Follow-up should be on a 6


months basis in patients being observed. In some cases, justification can be made for intervening on smaller sizes (greater than
5 cm) (Grade B recommendation). Women may warrant intervention at smaller sizes than men (Level II evidence, Grade B
recommendation).

CHOICE OF REPAIR: OPEN VERSUS


ENDOVASCULAR
The UKs EVAR1 trial randomized 1252 patients aged 60 years or
older with 5.5 cm or greater AAA with anatomy suitable to either
open repair (OR) or endovascular repair (EVAR) to either treatment, at 1of 34 hospitals.17,18 Ninety-four percent of the patients
completed their allocated treatment. The 30-days mortality was
lower in EVAR (1.8% vs. 4.3%). After 4 years, all cause mortality
was similar in the two groups, although AAA-related mortality
was less in the EVAR group (4% vs. 7%). However, late complications were more frequent in the EVAR group (41% vs. 9%), even
after exclusion of the benign type 2 endoleak, with increased need
for reinterventions. At 10 years of follow-up, there was still no difference in all cause related mortality and there was no longer any
difference in AAA-related mortality between the two groups. That
difference evened out at 6 years. There was a 30% reintervention
rate in the EVAR group due to an increase in graft complications
compared to the OR group. The obvious criticism leveled at the
study has been that many of the grafts used were first or second
generation devices more prone to late complications than later
devices.
The Dutch Randomised Aneurysm Management trial
(DREAM) randomized 351 patients with AAA greater than 5 cm
suitable for both techniques to either OR or EVAR.19,20 Operative
mortality was lower for EVAR (1.2%) as opposed to OR (4.6%), as
were early complications. However, at 6 years of follow-up, there
was again no difference in all cause related mortality. The survival
curves crossed at 2 years. At 6 years, there was a 30% reintervention rate in the EVAR group versus 18% in the OR group.
The OVER trial (open vs. endovascular repair) was conducted
by the US Department of Veterans affairs and randomized 881
patients from 42 VA Hospitals. Patients had to be candidates for
OR and EVA R.21 The 2-year outcomes have been reported. Once
again, 30-day mortality was lower for EVAR than OR (0.5% vs. 3%)
with less blood loss and shorter hospital stays. Overall, mortality
again was similar at 2 years (7% vs. 9.8%), with similar reintervention rates between the two groups. This latter result is different
than that reported by the UK and Dutch trials. It likely reflects the
inclusion of incisional hernias as a complication as well as a more
conservative approach toward type 2 endoleaks.
A French randomized prospective trial of open versus EVAR
for AAA dubbed ACE has been completed, but the results have
not been released as of the date of this writing. From these studies, it is clear that EVAR is associated with less operative blood
loss, lower 30-days mortality, but not any lesser long-term mortality, and it is associated with a higher rate of late interventions
(Level I data). For patients with relatively limited long-term life
expectancy, EVAR will likely be the procedure of choice (Grade A
recommendation). For younger healthier patients the jury is still
out as to what the preferred treatment modality should be.

5/22/2012 5:57:08 PM

Abdominal Aortic Aneurysm

PATIENTS UNFIT FOR OR


The EVAR 2 trial Endovascular Aneurysm Repair evaluated the
results of EVAR in patients judged to be unfit for OR.22,23 From
1999 through 2004, 404 patients in the United Kingdom with
AAA > 5.5 cm considered unfit for open AAA repair were randomized to either EVAR or observation. Aneurysm related mortality was lower in the EVAR group, but this did not result in an
improvement or benefit in total mortality. Twenty-seven percent
of patients in the EVAR subgroup required reintervention in the
first 6 years. The trial has been criticized because of the long delays
in getting patients to the OR in the subgroup randomized to
EVAR, the relatively high 30-days mortality of 7.3%, as well as the
high degree of crossover into EVAR from the group randomized
initially to observation. It is, however, the only Level I evidence
available in this field.
The Eurostar registry, which collects data from 101 European
sites, published a report on 399 patients considered unfit for open
surgery for anatomic or physiologic reasons.24 The early and late
mortality rates were higher in this group compared to patients
considered fit (3-year survival of 68% vs. 83%). On the basis of a
mathematical model, the authors concluded that EVAR is of benefit in prolonging life if the life expectancy is longer than 1 year.
Finally, a retrospective US study looking at EVAR in patients
considered at high risk for open surgery evaluated outcomes in
565 such patients.25 The 30-days mortality was 2.9%, the AAArelated death rate at 4 years was 4.2%, and overall mortality at
4 years was 56%. The authors concluded that EVAR in such
patients affords protection against AAA-related mortality. Some
have suggested that the patients in this study may not have been
as unfit as those in the Eurostar registry.
From these studies, one may conclude that EVAR in patients
at high risk for open surgery does result in a decrease in AAArelated mortality (Level I evidence). It may not improve overall
mortality over time and must therefore be offered only to nonmoribund patients (Grade A recommendation).

ENDOVASCULAR REPAIR:
CHOICE OF DEVICES
There are currently six devices for EVAR on the market in the
United States (AneuRx, Talent and Endurant from Medtronic,
Excluder from Gore, Zenith from Cook , and Endologix). Direct
randomized controlled trials comparing devices directly have not
been performed. All devices were FDA approved via clinical trials
comparing EVAR with the device under study versus open AAA
repair.
The Eurostar registry (European Collaborators on Stent
Graft Techniques for Aortic Aneurysm Repair) enrolled patients
from 1996 to 2005. It includes a majority of patients treated
with AneuRx, Talent, Excluder, and Zenith, but hardly any with
Endologix. Distal migration of the stent graft was most common
with AneuRx and Talent and least common with Zenith. Zenith
had the highest rate of limb occlusion. Excluder had the lowest
rate of AAA size shrinkage. Some of these findings such as migration differences have been reported in single center retrospective
studies, but such studies typically have had low number of patients
and are therefore subject to potential errors.26,27

PMPH_CH83.indd 663

663

The Lifeline Registry is pooled data from industry sponsored trial comparisons of OR versus EVAR of AAA.28 It does not
include any Zenith cases. It is not useful in comparing individual
devices. Individual center series are usually limited by the number of cases.
A retrospective review of 565 elective EVAR cases from Boston showed that reintervention rates were comparable between
devices, with all devices performing worse if used outside the
described indications for use.29
It appears that devices with active fixation (hooks or barbs) and
devices with suprarenal fixation may have less migration than others
(Level II evidence). Limb occlusion rates may be different between
various devices as well, as well as rates of AAA sac shrinkage. One
should choose the device that fits best a given patient anatomy.

ENDOVASCULAR REPAIR:
MANAGEMENT OF ENDOLEAKS
A type 1 endoleak, which represents an attachment site failure, is
widely recognized to need treatment in order to prevent aneurysm
sac rupture. Type 1 endoleaks will happen 5% to 10% of the time
on follow-up, depending on the population study and how well the
device indications for use (IFU) are followed. Endovascular options
(cuff, angioplasty, bare stent) or open surgical conversion should be
entertained (Level IIA evidence, Grade A recommendation).
A type 2 endoleak indicates flow around the stent graft in the
aneurysm sac, usually from the lumbar or inferior mesenteric arteries. A majority of these will resolve with observation, and a period
of observation can be recommended in the absence of aneurysm
enlargement. A recent review of such endoleaks from the United
Kingdom showed that only 24% were associated with AAA sac
size expansion. However, other reviews have suggested that up to
20% of ruptures after EVAR are due to type 2 endoleaks. There are
no prospective randomized trials in this regard, but mostly retrospective reviews and a few short-term prospective series. There is,
however, agreement that, in the case of AAA enlargement (more
than 5 mm in 6 months or more than 10 mm from pre-EVAR
size), intervention should be considered. The endoleak can often
be addressed by endovascular means (coil embolization, obliteration of the cavity); other options include surgical control of the
feeding vessels, or plain stent graft explantation and open conversion (Level IIA evidence, Grade A recommendation).30-35
A recent review of 1768 EVAR cases from one institution
over an 8-year period showed that 19.2% of patients required
secondary interventions, with type 2 endoleaks accounting for
40% of those interventions.36 That institution, however, followed
an aggressive program of interventions for type 2 endoleaks present at 6 months without decrease in AAA sac diameter. Mortality
data were provided but the long-term durability of the secondary
interventions was not clarified.

OR: TRANSABDOMINAL VERSUS


RETROPERITONEAL
Both the retroperitoneal (RP) and transabdominal or transperitoneal (TP) approaches to open AAA repair have had their proponents over the years.

5/22/2012 5:57:08 PM

664

Surgery: Evidence-Based Practice

Sicard et al. randomized 145 patients between the two approaches and found that the incidence of prolonged ileus and small
bowel obstruction was less in the RP group, whereas incisional pain
was more frequent. There was a trend toward shorter hospitalization
in the RP group. However, in that study, the percentage of patients
with COPD was significantly higher in the RP group.37
Cambria et al. randomized 113 patients between the RP
and TP approaches. They found no differences between the two
approaches in terms of operative or postoperative complications.
Nearly 80% of the patients had epidural anesthesia.38
A very small prospective randomized trial in 35 patients
showed improved intestinal function and shorter hospital stay
with the use of the RP approach.39
Most others studies in this subject have been retrospective
series by proponents of the retroperitoneal approach, suggesting its superiority in terms of fluid requirements, resumption of
intestinal function, and length of intensive care unit (ICU) stay.
A recent retrospective review has suggested that the RP approach
may be beneficial for suprarenal clamping in the treatment of juxtarenal aneurysms.40-44
The best analysis of all this data is that both approaches are
clearly acceptable and likely equivalent today (Level II evidence,
Grade B recommendation). There are clear-cut indications for the
RP approach such as the presence of multiple prior laparotomies,
right-sided stomas, inflammatory aneurysms, horseshoe kidneys,
and massive morbid obesity. Control of the suprarenal aorta may
be relatively easier with that approach, whereas exposure of the
right renal artery may be quite difficult. A large right iliac aneurysm may also be challenging to control with the RP approach.

PRESERVATION OF COLLATERAL FLOW


IN OR AND EVAR
A major complication of AAA repair by any modality is the
development of mesenteric ischemic complications, especially
ischemic colitis (IC). A ruptured aneurysm and prolonged

hypotension are two accepted risk factors for the development of


this complication.
A number of retrospective studies, single center and multicenter, have examined the role of the inferior mesenteric artery
(IMA) and the internal iliac arteries (IIAs) in the development
of this complication in open aneurysm repair.45-47 The results are
conflicting, with some studies suggesting that failing to preserve
flow to a patent IMA may lead to IC, with others failing to find
a correlation. Some studies have also found a decreased incidence of IC in cases where flow can be preserved to at least one
IIA, whereas others have not. Broad recommendations cannot be
made therefore, although it appears prudent to preserve flow to a
patent (but poorly backbleeding) IMA, or to at least one IIA (Level
II evidence, Grade B recommendation), especially in cases where
the normal collateral network may have been disrupted, such as
from prior colon operations.
Buttock claudication is fairly common after IIA embolization prior to stent graft placement, occurring in nearly one-third
of cases. More severe complications such as buttock necrosis or
ischemic colitis are far less common. Different retrospective series
have reported conflicting results on the risk associated with bilateral IIA embolization for stent graft placement.48-50 If possible, it
makes sense to try to preserve flow to at least one IIA (since the
IMA is routinely covered in EVAR). This could be accomplished
by open reimplantation of the IIA on the distal external iliac
artery (EIA) or by the use of branches of endografts now becoming available (Level IIb evidence, Grade B recommendation).
5. Should EVAR be used for ruptured aneurysms?
There are no randomized controlled trials comparing OR and
EVAR for ruptured aneurysms. Many retrospective and cohort
studies suggest that in patients with ruptured aneurysms who are
EVAR candidates, the use of EVAR may be associated with a lower
mortality and morbidity (Level IIb evidence).51,52 The big issue to
consider is whether the facility is equipped with all that is needed
for EVAR repair. EVAR for ruptured AAA should be considered
when possible (Grade B recommendation).

Summary Table
Clinical Issue

Level of
Evidence

Grade of
Recommendation

Risk factors for AAA are smoking, age, white race, and family history.

NA

Screening is effective in reducing AAA-related mortality.

Smoking cessation should be recommended for patients with AAA.

AAA repair should be considered when the diameter exceeds 5 to 5.5 cm.

EVAR is associated with decreased early morbidity and mortality, but similar to open
repair on longer-term follow-up.

NA

EVAR is preferable in patients with numerous comorbidities.

Stent grafts with active fixation mechanisms seem to be less prone to migration.

II

Type II endoleaks with increase in AAA size should be treated.

II

The transabdominal and the retroperitoneal approaches for open repair produce
equivalent results.

II

It is preferable to try to maintain flow to at least one hypogastric artery during EVAR.

IIb

EVAR for AAA should be considered whenever possible.

IIb

PMPH_CH83.indd 664

5/22/2012 5:57:08 PM

Abdominal Aortic Aneurysm

REFERENCES
1. Lederle FA, Johnson GR, Wilson SE, et al. Prevalence and associations of abdominal aortic aneurysm detected through screening. Aneurysm Detection and Management (ADAM) Veterans
Affairs Cooperative Study Group. Ann Intern Med. 1997;126:
441-449.
2. United Kingdom small aneurysm trial participants. Long term
outcomes of immediate repair compared with surveillance of
small abdominal aortic aneurysms. N Engl J Med. 2002;346:
1445-1452.
3. Lederle FA, Wilson SE, Johnson GR, et al. Immediate repair compared with surveillance of small abdominal aortic aneurysms.
N Engl J Med. 2002;346:1437-1444.
4. Powell JT, Brown LC, Forbes JF, et al. Final 12-year follow up of
surgery vs surveillance in the UK Small Aneurysm Trial. Br J
Surg. 2007;94:702-708.
5. Chang JB, Stein TA, Liu JP, et al. Risk factors associated with
rapid growth of small abdominal aortic aneurysms. Surgery.
1997;121:117-122.
6. Propranolol Aneurysm Trial Investigators. Propranolol for small
abdominal aortic aneurysms: results of a randomized trial. J Vasc
Surg. 2002;35(1):72-79.
7. Wilmink AB, Hubbard CS, Day NE, Quick CR. Effect of propranolol on the expansion of abdominal aortic aneurysms: a randomized study. Br J Surg. 2000;87:499.
8. Schouten O, van Laanen JH, Boersma E, et al. Statins are associated with a reduced infrarenal abdominal aortic aneurysm
growth. Eur J Vasc Endovasc Surg. 2006;32:21-26.
9. Schlosser FJ, Tangelder MJ, Verhagen HJ, et al. Growth predictors and prognosis of small abdominal aortic aneurysms. J Vasc
Surg. 2008;47:1127-1133.
10. Sukhija R, Aronow WS, Sandhu R, et al. Mortality and size of
abdominal aortic aneurysm at long-term follow-up of patients
not treated surgically and treated with and without statins. Am J
Cardiol. 2006;97:279-280.
11. Baxter BT, Pearce WH, Waltke EA, et al. Prolonged administration of doxycycline in patients with small asymptomatic
abdominal aortic aneurysms: report of a prospective (Phase II)
multicenter study. J Vasc Surg. 2002;36:1-12.
12. Mosorin M, Juvonen J, Biancari F, et al. Use of doxycycline to
decrease the growth rate of abdominal aortic aneurysms: a randomized, double-blind, placebo-controlled pilot study. J Vasc
Surg. 2001;34:606-610.
13. Vammen S, Lindholt JS, Ostergaard L, et al: Randomized doubleblind controlled trial of roxithromycin for prevention of abdominal aortic aneurysm expansion. Br J Surg 2001; 88:1066-1072.
14. Mortality results for randomised controlled trial of early elective
surgery or ultrasonographic surveillance for small abdominal
aortic aneurysms. The UK Small Aneurysm Trial Participants.
Lancet 1998;352:1649-1655.
15. Brown LC, Powell JT. Risk factors for aneurysm rupture in
patients kept under ultrasound surveillance. The UK Small
Aneurysm Trial. Ann Surg. 1999;230(3):289-296.
16. Ouriel K, Clair DG, Kent KG, Zarins CK. Endovascular repair
compared with surveillance for patients with small abdominal
aortic aneurysms. J Vasc Surg. 2010;51:1081-1087.
17. Endovascular aneurysm repair versus open repair in patients
with abdominal aortic aneurysm (EVAR trial 1): randomised
controlled trial. Lancet. 2005;365:2179-2186.
18. The United Kingdom EVAR trial investigators. Endovascular
versus open repair of Abdominal Aortic Aneurysm. N Engl J
Med. 2010;362:1863-1871.

PMPH_CH83.indd 665

665

19. Prinssen M, Verhoeven EL, Buth J, et al. A randomized trial


comparing conventional and endovascular repair of abdominal
aortic aneurysms. N Engl J Med. 2004;351:1607-1618.
20. De Bruin JL for the DREAM study group. Long term outcome
of open or endovascular repair of abdominal aortic aneurysm.
N Engl J Med. 2010;362:1881-1889.
21. Lederle FA for the Open vs Endovascular Repair (OVER) Veterans
Affairs Cooperative Study Group. Outcome following endovascular vs open repair of abdominal aortic aneurysm: a randomized trial. JAMA. 2009;302:1525-1542.
22. The EVAR trial participants. Endovascular aneurysm repair and
outcome in patients unfit for open repair of abdominal aortic
aneurysm (EVAR trial 2): randomized controlled trial. Lancet.
2005;365:2187-2192.
23. The United Kingdom EVAR trial Investigators. Endovascular
repair of Aortic Aneurysm in patients physically ineligible for
open repair. NEJM. 2010;362(20):1872-1880.
24. Buth J, van Marrewijk CJ, Marris PL, et al. Outcome of endovascular abdominal aortic aneurysm repair in patients with conditions considered unfit for an open procedure: a repsot on the
Eurostar experience. J Vasc Surg. 2002;35:211-221.
25. Sicard GA, Zwolak RM, Sidawy AN, et al. Endovascular abdominal aortic aneurysm repair: long term outcome measures in
patients at high risk for open surgery. J Vasc Surg. 2006;44:
229-236.
26. Ouriel K, Clair DG, Greenberg RK, et al. Endovascular repair
of abdominal aortic aneurysms: device specific outcome. J Vasc
Surg. 2003;37(5):991-998.
27. Greenberg RK, Deaton D, Sullivan T, et al. Variable sac behavior
after endovascular repair of abdominal aortic aneurysm: analysis of core laboratory data. J Vasc Surg. 2004;39:95-101.
28. Siami FS. Lifeline Registry of EVAR Publications Committee:
Lifeline registry of endovascular aneurysm repair: long-term
primary outcome measures. J Vasc Surg. 2005;42:1-10.
29. Abbruzzsese TA, Kwolek CJ, Brewster DC, et al. Outcomes following endovascular aortic aneurysm repair (EVAR): an anatomic and device specific analysis. J Vasc Surg. 2008;48:19-28.
30. Van Marrewijk C, Buth J, Harris PL, et al. Significance of
endoleaks after end vascular repair of abdominal aortic aneurysms: the EUROSTAR experience. J Vasc Surg. 2002;35:461-473.
31. Resch T, Ivancev K, Lindh M, et al. Persistent collateral perfusion
of abdominal aortic aneurysm after endovascular repair does not
lead to progressive change in aneurysm diameter. J Vasc Surg.
1998;28:242-249.
32. Brewster DC, Jones JE, Chung TK, et al. Long-term outcomes
after endovascular abdominal aortic aneurysm repair: the first
decade. Ann Surg. 2006;244:426-438.
33. Rayt HS, Sandford RM, Salem M, et al. Conservative management of type 2 endoleaks is not associated with increased risk of
aneurysm rupture. Eur J Vasc Endovasc Surg. 2009;38:718-723.
34. Conrad MF, Adams AB, Guest JM, et al. Secondary intervention
after endovascular abdominal aortic aneurysm repair. Ann Surg.
2009;250:383-389.
35. Mehta M, Sternbach Y, Taggert JB, et al. Long term outcomes
of secondary procedures after endovascular aneurysm repair. J
Vasc Surg. 2010;52:1442-1448.
36. Jones JE, Atkins, MD, Brewster DC, et al. Persistent type 2
endoleak after endovascular repair of abdominal aortic aneurysm is associated with adverse late outcomes. J Vasc Surg.
2007;46:1-8.
37. Sicard GA, Reilly JM, Rubin BG, et al. Transabdominal versus
retroperitoneal incision for abdominal aortic surgery: report of a
prospective randomized trial. J Vasc Surg. 1995;21:174-181.

5/22/2012 5:57:08 PM

666

Surgery: Evidence-Based Practice

38. Cambria RP, Brewster DC, Abbott WM, et al. Transperitoneal


versus retroperitoneal approach for aortic reconstruction: a randomized prospective study. J Vasc Surg. 1990;11:314-353.
39. Arya N, Sharif MA, Lau, LL, et al. Retroperitoneal repair of
abdominal aortic aneurysm reduces bowel dysfunction. Vasc
Endovasc Surg. 2009;43(3):262-270.
40. Todd GJ, DeRose Jr JJ. Retroperitoneal approach for repair of
inflammatory aortic aneurysms. Ann Vasc Surg. 1995;9:525-534.
41. Arko FR, Bohannon WT, Mettauer M, et al. Retroperitoneal
approach for aortic surgery: is it worth it? Cardiovasc Surg. 2001;
9:20-26.
42. Darling 3rd C, Shah DM, Chang BB, et al. Current status of the
use of retroperitoneal approach for reconstructions of the aorta
and its branches. Ann Surg. 1996;224:501-506.
43. Sieunarine K, Lawrence-Brown MM, Goodman MA. Comparison
of transperitoneal and retroperitoneal approaches for infrarenal
aortic surgery: early and late results. Cardiovasc Surg. 1997;5:71-76.
44. Wahlgren C, Piano G, Desai T, et al. Transperitoneal vs retroperitoneal suprarenal cross clamping for repair of abdominal aortic
aneurysm with a hostile infrarenal arotic neck. Ann Vasc Surg.
2007;21(6):687-694.
45. Pittaluga P, Batt M, Hassen-Khodja R, et al. Revascularization
of internal iliac arteries during aortoiliac surgery: a multicenter
study. Ann Vasc Surg. 1998;12:537-543.

PMPH_CH83.indd 666

46. Van Damme H, Creemers E, Limet R. Ischaemic colitis following


aortoiliac surgery. Acta Chir Belg. 2000;100:21-27.
47. Bast TJ, van der Biezen JJ, Scherpenisse J, et al. Ischaemic disease
of the colon and rectum after surgery for abdominal aortic aneurysm: a prospective study of the incidence and risk factors. Eur J
Vasc Surg. 1990;4:253-257.
48. Rayt HS, Bown MJ, Lambert KV, et al. Buttock claudication and
erectile dysfunction after internal iliac artery embolization in
patients prior to endovascular aortic aneurysm repair. Cardiovasc Intervent Radiol. 2008;31:728-734.
49. Karch LA, Hodgson KJ, Mattos MA, et al. Adverse consequences
of internal iliac artery occlusion during endovascular repair of
abdominal aortic aneurysms. J Vasc Surg. 2000;32:676-683.
50. Mehta M, Veith FJ, Ohki T, et al. Unilateral and bilateral hypogastric artery interruption during aortoiliac aneurysm repair
in 154 patients: a relatively innocuous procedure. J Vasc Surg.
2001;33:S27-S32.
51. Veith FJ, Lachat M, Mayer D, et al. Collected world and single center experience with endovascular treatment of ruptured abdominal aortic aneurysms. Ann Surg. 2009;250:818-824.
52. Holst J, Resch T, Ivancev K, et al. Early and intermediate outcome
of emergency endovascular aneurysm repair of ruptured infrarenal aortic aneurysm: a single center experience of 90 consecutive
patients. Eur J Vasc Endovasc Surg. 2009;7:413-419.

5/22/2012 5:57:08 PM

Commentary on
Abdominal Aortic Aneurysm
Gregorio Sicard

The chapter titled Abdominal Aortic Aneurysm by Dr Boulos


Toursarkissian provides an outstanding review of the risk factors,
diagnosis, and treatment of infrarenal abdominal aortic aneurysms based on current levels of evidence. These selected reviews
of levels of evidence and recommendations will serve as practice
guidelines for healthcare providers who care for patients with this
condition.
Abdominal aortic aneurysm (AAA) is a common vascular
condition, the incidence of which increases as the population
ages. As a disease of the elderly, this condition has undergone
extensive scrutiny of its pathobiology, diagnosis, and best therapeutic options. As with any other complex disease, many questions remain unanswered by lack of sufficient strong evidence
and reflects the need for further work to strengthen or modify the
recommendations. This chapter by Dr Toursarkissian addresses
some of the main issues that are important for the diagnosis and
treatment of this condition, and the level of scientific evidence
currently available to support the practice recommendations.
In this chapter Dr Toursarkissian presents strong levels of evidence that identifies those important risk factors such as smoking,
gender, and family history that have been clearly associated with
the development of AAA. Dr Toursarkissian addresses the frequently controversial issue of which patients should be screened
and what the patient characteristics are that best defines the population that may benefit from such screening to yield a high incidence of early detection.
An important and related area of investigation addressed in
this chapter is whether pharmacologic treatment can influence
the progression or stabilization of the growth of an AAA. Considerable progress has been made over the past 20 years defi ning the pathologic processes responsible for the development
of AAA.
However, there is a general agreement among investigators that one of the most important factors in the formation and
continued growth of aneurysms is the destruction of the medial
lamellar structure by elastolytic tissue enzymes known as metalloproteinases (MMPs). Many studies using animal models have
confi rmed the positive effect of an MMP inhibitor such as doxycycline in AAA growth. Dr Toursarkissian reviews some of the
recent studies that have attempted to pharmacologically affect
the growth rate of small aneurysms. A few interesting studies

have demonstrated reduction in rate of growth of AAA in short


courses of tetracycline both in intact aneurysms and in endografttreated patients. The lack of robust randomized control trials
has made it difficult to generate strong levels of evidence and
therapeutic recommendations at this point. Phase I trials have
established the safety profi le of this pharmacologic treatment in
a clinical setting. A novel medical therapy trial for small AAA
is currently being reviewed by the National Institute of Aging,
which anticipates to receive funding for the trial. The noninvasive treatment for abdominal aneurysm clinical trial (N-TA3CT)
is a randomized, double blind, and placebo controlled study that
is aimed at detecting a significant reduction in the growth rate
of an aneurysm of between 3.5 and 5.0 cm. Th is study is scheduled to enroll over 240 patients selected in 15 centers. The vascular community eagerly awaits this trial as it may defi ne the
importance of early detection and the pharmacologic treatment
of small aortic aneurysms.
Since the first report in 1991 by Parodi and collaborators,
endovascular repair of abdominal aortic aneurysm (EVAR) has
achieved worldwide acceptance. The levels of evidence and recommendations of this innovative therapeutic approach is based
on well-constructed randomized trials performed in the United
States, United Kingdom, and Europe. All of these studies have
clearly and universally demonstrated the early benefits in mortality and morbidity of the endovascular approach over the standard
open surgical repair. Longer-term follow-up of both therapeutic
approaches also show similar outcomes. In his chapter, Dr Toursarkissian nicely reviews the levels of evidence and grades of recommendation to the most common problems associated with this
new less-invasive therapeutic approach for treating AAA. Similarly, because of the paucity of high levels of evidence for comparisons of EVAR for ruptured aneurysms, Dr Toursarkissian
identifies the need for further research in this group of patients
with symptomatic aneurysms.
In summary, this excellent chapter by Dr Toursarkissian
demonstrates the significant progress that has been achieved
in the diagnosis and treatment of this common condition. By
identifying the levels of evidence in selected areas, he has provided practice recommendations, and as a result of the analysis,
areas that need further research to advance the field have been
identified.

667

PMPH_CH83.indd 667

5/22/2012 5:57:08 PM

668

Surgery: Evidence-Based Practice

REFERENCES
1. Chaikof EL, Brewster DC, Dalman RL, et al. The care of patients with abdominal aortic aneurysm: the Society for Vascular
Surgery practice guidelines. J Vasc Surg. 2009; October
(Supplement).
2. Lindeman JHN, Abdul-Hussien H, van Bockel JH, Wolterbeek
R, Kleeman R. Clinical trial of doxycycline for matrix metalloproteinase-9 inhibition in patients with an abdominal aneurysm:

PMPH_CH83.indd 668

doxycycline selectively depletes aortic wall neutrophils and cytotoxic T cells. Circulation. 2009;119:2209-2216.
3. Masorin M, Juvonen J, Biancari F, et al. Use of doxycycline
to decrease the growth rate of abdominal aortic aneurysms: a
randomized, double-blind, placebo-controlled pilot study. J Vasc
Surg. 2001;34:606-610.
4. Hackman AE, Rubin BG, Sanchez LA, Geraghty PA, Thompson RW,
Curci JA. A randomized, placebo-controlled trial of doxycycline
after endoluminal aneurysm repair. J Vasc Surg. 2008;48:519-526.

5/22/2012 5:57:08 PM

CHAPTER 84

Aortic Dissection
Benjamin J. Pearce

INTRODUCTION

behind the evaluation and treatment of acute type B dissection in


the evolving era of advanced imaging and endovascular therapy.

Aortic dissection is both a misunderstood and fearful entity. Often


referred to as a dissecting aneurysm, aortic dissection can occur
both in the setting of atherosclerotic degenerative aneurysm and
in normal sized aorta. More recent attempts have been made to
classify the anatomic and physiologic response of the acute aortic syndrome. It is clear that the risk of aortic dissection goes
beyond the anatomic malady and is a systemic process. This likely
accounts for the relatively high mortality of even the most benign
presentations.
The nomenclature of aortic dissection is critical to understanding treatment recommendations. The most important distinction is related to the site of the initial entry tear as any
dissection proximal to the left subclavian artery is considered a
surgical emergency, whereas others may be considered for medical management. The simplest classification is the Stanford system
that uses the treatment recommendations and is Stanford type A
Dissection (type A dissection) for any dissection involving the
aortic arch proximally and type B Dissection (type B dissection)
for those at or distal to the left subclavian artery.1 Aortic dissection is defined temporally as well, owing to Dr Crawfords description that 74% of all mortalities with a new dissection occur within
14 days.2 Hence, acute aortic dissections are of <14-days duration
and chronic aortic dissections are of >14-days duration.
Acute aortic dissection is the most common aortic emergency, nearly doubling the incidence of ruptured aneurysm.3 Left
untreated, acute aortic dissection carries a mortality of 1% to 2%
per hour with a >90% mortality at 1 week.4 Acute type A dissection is the most lethal presentation. The mortality is mainly related
to dissection into the aortic root leading to valvular dysfunction,
coronary ischemia, tamponade, free rupture, or some combination of these events. Surgical replacement of the ascending aorta
with valve and coronary artery repair as indicated has been proven
as the definitive treatment of acute type A dissection with a surgical mortality as low as 9%.5 The decision making for acute type A
dissection is clear as the majority of untreated patients will die.
As such, the focus of this chapter will be on the decision making

EPIDEMIOLOGY
1. Who gets aortic dissection and how do they present?
The largest contemporary attempt at evaluating aortic dissection is
the International Registry of Acute Aortic Dissection (IRAD) database. This registry is composed of 21 international referral centers
for aortic pathology and published the initial analysis of the registry
in 2000.6 Multiple subsequent cohort analyses have been published
to evaluate nuances of aortic dissection and long-term survival, and
the most recent publication encompasses 1417 patients.7
The peak incidence of type B dissection is in the seventh
decade of life and the most common risk factor is hypertension,
which is present in 72% of patients prior to presentation of dissection.6 Males are predominately at risk, accounting for 70% of all
type B dissection. Personal history of atherosclerotic disease has
not been described as an absolute risk factor for dissection;8 however, in a large series of open repairs done to treat acute aortic dissection, atheroma or plaque was found in the suspected lead point
of the dissection in up to 83% of patients.9 Likewise, Marfans
disease is a well-described risk factor for all aortic pathology,10
yet only 4.9% of patients in the IRAD database had documented
Marfans disease.6 The incidence of aortic dissection in pregnancy
is quite small, but in pre-eclampsia it can be as high as 13%.11 The
overwhelming majority of these are type A dissections initiating at the sinotubular junction in the absence of atherosclerosis.
Another infrequent overall population is iatrogenic dissection
from another endovascular procedure. However, with the number
of endoluminal procedures for all arterial pathology increasing
yearly, a history of recent catheterization cannot be overlooked.
The most common presentation is acute onset of severe and
localized pain.8 However, the presence of chest, back, or abdominal
pain has been reported as low as 63%, 64%, and 43% respectively
in acute type B dissection8 versus pain as a presenting symptom in
85% of type A dissections.6 Therefore, not all patients with aortic
669

PMPH_CH84.indd 669

5/22/2012 5:57:42 PM

670

Surgery: Evidence-Based Practice

dissection will have pain, and clinical suspicion should remain


high in evaluation of the hypertensive patient with other vascular
deficits. Other manifestations of type B dissection include transient, or persistent, spinal cord ischemia in 2% to 10%,12 anuria,
mesenteric ischemia, lower extremity pulse deficit, and acute
congestive heart failure (CHF) from severe afterload increase in
patients with significant proximal true lumen collapse.6 In addition to important clinical signs leading to diagnosis, these features are critical signs of malperfusion and will aid in directing
subsequent therapy. An often overlooked presenting feature of
acute dissection is syncope. As high as 20% of patients with type A
dissection can have syncope from involvement of the aortic valve
or cerebral perfusion. Yet even true type B dissection patients can
present with syncope from activation of aortic baroreceptors.8
Careful evaluation for presenting symptoms and physical exam
findings of malperfusion will lead to expeditious workup and initial therapy. The cost of misdiagnosis is high with 20% mortality in
acute type B dissection at day 2 if presenting with malperfusion.8

DIAGNOSIS
2. What modalities are appropriate for diagnosis and therapeutic planning?
Unfortunately, acute chest pain is one of the most frequent presentations in emergency visits. Contrast-based imaging is not
necessary in the majority of these cases, thus careful correlation
between history and physical findings as described above can
prompt efficient diagnostic evaluation.
Plain chest x-ray is often included in the initial workup for
chest pain. This is not adequate for confirmation of diagnosis;
however, close observation for potential aortic pathology on plain
radiograph can lead a clinician to an appropriate imaging study.
Widening of the mediastinum or left-sided pleural eff usion should
be assessed on chest x-ray.13 Historically, patients with a question
of dissection underwent conventional catheter-based aortography.14 Intravascular ultrasound (IVUS) performed at the time of
angiography can yield useful information regarding the location
of the entry tear in relation to branch vessels, location of fenestrations, the nature of malperfusion to visceral and extremity
branches, and sizing for potential therapy. With the improvement
in computed tomography angiography (CTA) and magnetic resonance angiography (MRA), conventional angiography has largely
been replaced for diagnostic purposes.
CTA is the overwhelming diagnostic study of choice with sensitivity and specificity reported as high as 95%. One continuous scan
of the chest, abdomen, and pelvis can not only confirm the diagnosis, but can provide essential information for planning of potential
interventions. Evaluation of a helical scan of the aorta from the root
to the femoral bifurcation can determine the true lumen from the
false one by the continuity to the undissected arch segment. In addition, the false lumen is the larger of the two lumens in acute type B dissection in >90% of cases. These features allow the surgeon to evaluate
the perfusion of the visceral and extremity branches and determine
potential need for intervention.15 In addition, the nature of the flap
can yield important information regarding chronicity of the injury.
Acute dissection flaps are bowed in over 60% of patients due to pressure differential between the two lumens. Conversely, the flap is flat
in chronic dissection as the pressures have equalized over time.15 As
patients can have delayed presentation, or occasionally incidental
findings of dissection, these features may aid in determining acuity.

PMPH_CH84.indd 670

Similar to CT scanning, MRA is an excellent noninvasive


test for evaluation of aortic dissection with sensitivity and specificity also approaching 100%.16,17 Similar to CTA, MRA can provide valuable detail regarding branch vessel involvement and
operative planning. In addition, there is hope that the emerging
technique of dynamic MR may provide important information
to predict success of medical versus surgical treatment owing to
improved evaluation of the nature of obstruction by the aortic
flap. Unfortunately, MR requires longer scan times and decreased
access to the patient during the scan, in addition to exclusion of
patients with implants.
All contrast-based imaging modalities (angiography, CTA,
MRA) increase the risk of nephropathy in patients already at
increased risk for renal failure due to malperfusion, chronic hypertension, or acute shock. Thus, additional diagnostic modalities may
be considered before administration of contrast in patients with elevated creatinine or documented anaphylactic reactions to contrast.
Echocardiography, either transthoracic (TTE) or transesophageal
(TEE), was the first diagnostic test in 33% of patients in the initial
IRAD database.6 Sensitivity and specificity of TEE in diagnosis of
acute dissection have been reported as >90%.18 Ultimately, an average of two diagnostic studies were utilized in confirming the diagnosis and formulating a treatment plan in the IRAD patients.6

MANAGEMENT
3. Which patients are appropriate for medical management
and which require operative intervention?
All patients diagnosed with acute type B dissection require immediate initiation of anti-impulse therapy (unless presenting in
hypotensive shock) and movement to a critical care setting. Placement of right arm arterial line, central venous access, and urinary
drainage catheterization is warranted. Patients should be maintained nil per os (NPO). The next critical decision is whether the
patients can be maintained successfully on medical therapy or
warrant surgical intervention.
Indications for immediate intervention include rupture, acute
aneurismal degeneration of the aorta (greater than 4-cm false
lumen or 5-cm total aortic diameter), and, most commonly, end
organ ischemia presenting as malperfusion syndrome. Malperfusion in the setting of acute aortic dissection is a complicated
mechanism dependent on the location of the entry tear, extent of
intimal flap, presence/absence/location of distal re-entry tears,
cardiac output, dissection of branch vessels, and thrombosis. The
two main types of obstruction are static and dynamic.
Static obstruction is relatively straightforward. As dissection
is carried to a visceral or extremity branch, the flap may propagate
into the branch itself. If no re-entry exists within the branch vessel, the false lumen will often thrombose, leading to constriction
of flow into the branch and occlusion or severe hypoperfusion.
Conversely, the ostium of a branch vessel (commonly the left renal
artery) can be completely fed from the false lumen. In this scenario, as long as flow is maintained within the false lumen, the
end organ remains perfused.19 However, if the false lumen is
thrombosed, or excluded by intervention, preservation of the end
organ perfusion will be dependent on reperfusion of the branch
via bypass or stenting across the intimal flap.
Dynamic obstruction is the most common cause of malperfusion in acute type B dissection.20 Flow into the false lumen creates
a pressure differential between the two lumens. The most obvious

5/22/2012 5:57:42 PM

Aortic Dissection

form of dynamic obstruction is when no re-entry tear exists in the


distal aorta. In this scenario, the false lumen pressure will greatly
exceed the true lumen and essentially compress the true lumen to
the extent that the entire aortic segment is hypoperfused. In this
setting, the afterload on the cardiac pump becomes so great that
it precipitates acute heart failure. Conversely, dissections often
have multiple sites of re-entry, or natural fenestration, between
the two lumens. During the cardiac cycle, the flow through the false
lumen can cause pulsation of the intimal flap to cover the ostium
of a branch vessel in systole; effectively creating hypoperfusion.
In an experimental model, it has been demonstrated that the
degree of dynamic obstruction is governed by cardiac output,
blood pressure, peripheral resistance, and location and size of the
entry and re-entry tears.21 An understanding of how these factors
combine to create malperfusion are essential for appropriate management of acute type B dissection.

Medical Therapy
In the uncomplicated, hypertensive patient, medical management
continues to be the mainstay of therapy for acute type B dissection.
In all patients with acute type B dissection, except those presenting with hypotensive shock, immediate control of heart rate, cardiac
output, and peripheral resistance can (1) prevent progression of the
entry tear; (2) prevent progression of the dissection flap; and (3) allow
for equalization of pressures between the true and false lumens.
No randomized trial has specifically compared pharmacologic
strategies for treatment of acute type B dissection. The goals of therapy are to reduce systolic blood pressure to less than 120 mm Hg.
This is best achieved with a combination of pain control, inotropic/
chronotropic blockade, and afterload reduction. Often overlooked is
the importance of adequate analgesia. The contribution of acute pain
to the cascade of catecholamine release is significant, and appropriate treatment with intravenous opiates is warranted. One of the most
feared complications of acute type B dissection is paraplegia from
spinal cord infarction either spontaneously or from treatment. As
such, the use of intrathecal analgesia is relatively contraindicated.
Initiation of therapy with -blockade is the standard of care.22
The goal heart rate is <80 bpm. By initiating -blockade early, reflex
tachycardia and catecholamine release are blunted with the initiation of afterload reducing agents. Choice of a -blocker, for example,
labetalol, with and adrenergic effects may achieve appropriate
cardiac and peripheral effects with a single agent. However, this may
precipitate pulmonary complications in at-risk patients and thus initiation of therapy with a short acting agent, for example, esmolol,
may be more prudent in selected cases.19 Initiation of sodium nitroprusside infusion after -blockade has been an effective agent. However, recent data have shown similar efficacy of nicardipine infusion
in treatment of acute type B dissection without the risk of cyanide
toxicity.23 Conversion to oral -blockade and oral afterload reducing
agents such as angiotensin converting enzyme inhibitors or minoxidil are appropriate after patient is pain free for 48 hours.

Open Surgical Therapy


Historically, surgical therapy for complicated acute type B dissection required central aortic replacement with either obliteration
of false lumen flow via distal anastomosis, or open fenestration.
The largest early series of central aortic repair carried a mortality of 45% in 1986.9 And yet, in the most recent IRAD update on
surgically repaired type B dissection, the in-hospital mortality

PMPH_CH84.indd 671

671

remained at 30%.24 In addition, the indication of acute type B dissection carries the highest risk of spinal cord ischemia in series of
open thoracic aortic surgery.25,26 Central aortic repair has essentially been reserved only for acute rupture or acute aneurysm formation that cannot be excluded by endograft technology.

Endovascular Therapy
Endovascular therapy for acute type B dissection is aimed at both
treatment of malperfusion and stabilization of the aortic true
lumen. The most straightforward technique to achieve both goals
is thoracic endovascular aortic repair (TEVAR) with commercially
available covered stent grafts. Theoretically, TEVAR can exclude
the entry tear thus reducing pressure in the false lumen. Simultaneously, the stent supports the true lumen and flow is directed
preferentially into the true lumen to improve distal organ perfusion. A few essential components unique to TEVAR warrant special attention. Deployment of the stent graft within the true lumen
is perhaps the most important tenet in TEVAR for acute type B
dissection. Although this may seem intuitive, wire selection from
the femoral artery into the ascending aorta may pass from lumen
to lumen through multiple fenestrations. The adjunctive use
of IVUS can assure placement and prevent distal ischemia. Use of
right brachial access can also aid in appropriate wire access.
In addition, complete exclusion of the proximal entry tear is
necessary for both acute and long-term success. This requires coverage of the left subclavian artery in up to 46% of cases in registry
data.27 Whenever coverage of the left subclavian artery is necessary, careful evaluation of vertebral artery anatomy and presence
of internal mammary to coronary bypass is warranted. This also
is an independent risk factor for postoperative neurologic deficit.28
However, the incidence of permanent paraplegia is reported as low
as 0.8% with TEVAR for acute type B dissection29 and overall spinal cord ischemia/stroke rate as low as 2.9%.30 As such, expectant
management of spinal cord complications with postoperative lumbar drainage and hypertensive management is well supported.31,32
Alternative, and/or adjunctive, endovascular techniques exist
to treat malperfusion. In addition to the malperfusion syndrome
of type B dissection, distal malperfusion persists in up to 25% of
patients after successful proximal correction of type A dissection.33
Endovascular correction of malperfusion can be achieved through
fenestration, aortic stenting, and branch artery stenting. However,
unlike TEVAR, endovascular management of malperfusion will not
have a long-term impact on the remodeling of the aorta or subsequent aneurysm formation. Endovascular management of malperfusion requires wire access to each lumen and is often aided by
IVUS and brachial access. Power injection into the true lumen can
spuriously identify flow in the setting of dynamic obstruction and
should not be utilized for diagnostic purposes. To accurately assess
hypoperfusion, selection of the target vessel must be obtained and a
pressure gradient determined between the ascending aorta proximal
to the entry tear and the actual pressure within the branch vessel.34
Fenestration of the intimal flap immediately proximal to the
area of malperfusion can result in equalization of pressures and
restoration of flow in areas of dynamic obstruction. Confirmation of wire access in both lumens and fenestration from the true
to false lumen with the use of various needles is the basic technique. Use of the Pioneer catheter (Medtronic Inc., Santa Rosa,
CA) allows for visualization of the flap and false lumen during
needle placement using IVUS. Once wire access is gained, the
fenestration is widened with a standard angioplasty balloon.

5/22/2012 5:57:42 PM

672

Surgery: Evidence-Based Practice

Conversely, an endo-scissors technique has been described in


which a sheath is advanced over both the true and false lumen
wires after fenestration.35
Stenting of branch arteries involved in static obstruction is
straightforward. If the dissection flap progresses into the branch,
true lumen access into the branch and use of standard stents
can support true lumen flow and restore perfusion. In cases of
dynamic obstruction, branch artery stenting usually requires
fenestration of the flap from the true to false lumen, expansion of
the fenestration with balloon angioplasty, and, ultimately, stenting from the branch vessel across the flap into the true lumen.
Conversely, restoration of perfusion can be achieved with support of true lumen flow in the distal thoracic aorta using larger
stents36 and placement of appropriate branch stents distal to the
supported true lumen.

OUTCOME
4. What is the natural history of patients treated medically
versus surgically? Are there predictors of success with either
modality?
Acute aortic dissection continues to have significant morbidity
and mortality even in its most benign presentation. In the IRAD
database, 80% of type B dissection patients had no signs of malperfusion and received medical therapy. And yet these patients still
had an in-hospital mortality of 10.7%6 and the long-term sequelae
of dissection in survivors is not benign. Onitsuka et al.37 demonstrated a 13% rate of conversion to surgery in a group of 76 patients
randomized to medical therapy and 23% incidence of dissectionrelated events including rupture and sudden death in the medical
cohort studied for 10 years. Others have reported a rupture risk of
18% during follow-up of medically treated type B dissection38 and
that 20% of all thoracoabdominal aneurysms have chronic dissection as the etiology.39 The overall 3-year mortality of patients
with type B dissection in the IRAD database who survived initial
hospitalization was 24.9%.40 In fact, in the interim analysis of the
IRAD data, the patients with the best 1- and 3-year survival are
those who survive open surgical intervention during their initial
presentation.7
Numerous factors have been evaluated for improved outcome
in patients with type B dissection. Successful surgical correction
in the acute phase likely leads to improved long-term survival via
obliteration of the false lumen while restoring normal perfusion
to the distal aortic segment and stabilizing aortic size. Akutsu
et al.41 demonstrated that in medically managed type B dissections followed over 10 years, patent false lumen was a significant
predictor of dissection-related death (HR = 5.6) and dissectionrelated event (HR = 7.6). Mean aortic size and aortic growth rate
have been demonstrated as independent predictors of dissectionrelated events during follow-up of medically treated patients.37
These observations support the potential benefit of excluding the
entry point of the aortic dissection with stent grafts. In a nonrandomized cohort, Dialetto42 demonstrated a thrombosed false
lumen rate of 75% in stent grafted patients versus 10.7% in medically treated patients. The same cohort showed aneurysmal degeneration in only 3.5% of the stent grafted patients versus 28.5% in
the medically treated group.
Stent graft technology is quickly replacing open graft replacement as the treatment of choice for complicated type B dissection.

PMPH_CH84.indd 672

Ideally, TEVAR should offer patients improvement in hospital


mortality compared to traditional repair, while maintaining the
survival benefit demonstrated by surgical correction. Multiple
generations of thoracic endografts are now commercially available in various sizes, thus increasing the number of patients eligible for endovascular treatment of complicated type B dissections.
Technical success in device implantation covering the proximal
entry tear is between 86% and 95%.43-46
Perioperative 30-day mortality rates of TEVAR for complicated type B dissection have been reported ranging from 3.2% to
13.3%.27,29-30,42-44 This mortality rate corresponds to reported mortality of medically treated patients in historic controls and is better than the expected open surgical mortality for these patients.
Effectively, endovascular techniques have shifted these patients
from the higher mortality associated with complicated dissection treated surgically to a lower mortality commensurate with
uncomplicated dissection.
Significant complications of permanent dialysis, spinal cord
ischemia, bowel ischemia, limb ischemia, and stroke can occur
in both medically and endovascularly treated patients. A metaanalysis of stent grafts for type B dissection demonstrated a major
complication rate of 11.2% with neurologic deficits persisting in
only 2.9% of patients.30
In addition to acute improvement in outcomes, the goal of
endovascular therapy is to improve long-term remodeling of the
aorta and prevent late complications. Success in thrombosing
the segment of false lumen within the stented segment may not
change the overall risk. Nathanson et al.46 noted no reduction in
aortic size in TEVAR patients despite absence of endoleak in 71%
and bring into question the fate of the uncovered abdominal aorta as
demonstrated by a 56% rate of increase in abdominal false lumen
diameter. Likewise, Schoder et al.44 demonstrated that in TEVAR
treated patients, the false lumen thrombosed in the stented segment
in 90% of patients with significant diameter reduction in the aorta
at 2 years. However, in the segment of aorta adjacent to the stent, the
thrombosis rate dropped to 60% and in the distal aorta it was 22%.
In the distal segment a significant increase in aortic size was noted.
5. Has endovascular technology changed the treatment paradigm for acute type B dissection?
To assess the potential for TEVAR to improve outcomes in patients
with type B dissection, a randomized trial comparing best medical therapy to elective TEVAR in uncomplicated type B dissection
was performed in Europe.47 The INvestigation of STEnt grafts in
patients with type B Aortic Dissection (INSTEAD) trial randomized patients 2 weeks after presentation to chronic medical therapy
versus TEVAR. This excluded patients who may have benefited from
TEVAR for acute malperfusion syndrome, but it was designed to
evaluate if freedom from chronic complications could be provided
by TEVAR. However, at 1 year, the all cause mortality was higher
in the TEVAR cohort than medical therapy (8.6% vs. 3%, p NS).48
This effect persisted at 2 years with survival in the medical cohort
of 96% and in the TEVAR cohort of 89% (p NS), although it is partially attributed to the surgical mortality of 3%.49 Other complications included spinal cord ischemia in one medical patient and two
TEVAR, and one stroke in the TEVAR group. The trial did demonstrate a significant rate of false lumen thrombosis and stabilization
of aortic size in 91% (P < .05) of TEVAR patients, whereas 16% of
medically treated patients required crossover into surgery due to
aneurysm expansion >6 cm during follow-up.49 Despite this effect,

5/22/2012 5:57:42 PM

Aortic Dissection

however, the all cause endpoints are not significantly different and
cannot justify the empiric use of TEVAR for uncomplicated dissection at this time. This is reflected in the 3-year follow-up analysis
of IRAD patients stratified by treatment. Survival in the medically
treated patients was 77% versus 76% in those treated with stent
graft.7 In essence, TEVAR can alter the risk of a complicated dissection into the same category as the uncomplicated dissection, but
current medical therapy has reached a threshold at which patients
with acute aortic pathology cannot be improved.
Likewise, alternative endovascular management of malperfusion has mixed results. In one of the largest series of balloon
fenestration, the technical success rate of resolving malperfusion
was 93%; yet the 30-day mortality was still 25%.50 In addition,
the aortic segments in patients treated with fenestration or traditional stents remain pressurized and have demonstrated expansion in the postoperative period.51 In fact, these patients may be at
increased risk as fenestration should lead to persistent false lumen
flow and a decreased rate of thrombosis.

Chronic Dissection
The most common complication of survivors of acute dissection
is degeneration of dissected segment into aneurysm, occurring

673

in 20% to 40% of chronic type B dissections.52-53 More importantly, up to 60% of late mortality in patients with chronic type
B dissection is attributed to aortic-related death, either from
rupture, ischemia, or complications of subsequent repair of thoracoabdominal aneurysm.7 Among those suffering late mortality, independent risk factors for dissection-related death were
identified, these include female gender, atherosclerosis, aneurysm at time of dissection, renal failure, and in-hospital shock.

CONCLUSION
Acute aortic dissection continues to be a significant source of diagnostic and therapeutic anxiety for patients and medical practitioners. The spectrum of disease is severe with major morbidity and
mortality. Because of the acute nature of the disease process, very
little randomized data exist to guide optimal treatment practice.
However, a large amount of registry data have now been accumulated to assess presentation, evaluation, and subsequent management. Despite significant advances in endovascular technology,
the mainstay of treatment for uncomplicated dissection continues
to be medical therapy. Endovascular technique has improved the
outcome in patients requiring intervention.

Clinical Question Summary


Question

Answer

1 Who gets aortic


dissection and how
do they present?

Predominently males in seventh decade of life


with history of hypertension. Acute pain and
hypertensive episode is the most common
presentation. Can also present with syncope,
pulse deficit, visceral ischemia, and paraplegia.

1b

6-12

2 What modalities
are appropriate
for diagnosis and
therapeutic planning?

CTA and MRA are first-line studies. In patients


with acute renal failure or contrast allergy,
TEE can confirm diagnosis before making
decision about adding contrast study.

3b, 4

6, 15-18

3 Which patients are


appropriate for
medical management
and which
require operative
intervention?

Unless presenting in shock, all patients warrant


initiation of anti-impulse therapy with
-blockade and afterload reduction. Indication
for operative intervention are recurrent pain
on appropriate anti-impulse therapy, acute
aneurismal expansion, rupture, end organ
malperfusion.

2a, 3b, 4

6, 8, 22, 27,
29-30

4 What is the natural


history of patients
treated medically
versus surgically? Are
there predictors of
success with either
modality?

The best outcome is in patients with successful


surgical correction. Thrombosis of the
false lumen with either medical or surgical
management is the best long-term predictor
of freedom from aortic-related morbidity/
mortality.

1b, 4

7, 37-41

5 Has endovascular
technology changed
the treatment
paradigm for acute
type B dissection?

Yes and no. Endovascular management has


essentially replaced open central aortic
repair when surgery is indicated. This has
dramatically improved the morbidity and
mortality of treatment for complicated
dissection. However, endovascular therapy
has not improved medical management in
uncomplicated patients.

1b, 2a,
2b, 4

27, 29, 30,


42-44,
48-49

PMPH_CH84.indd 673

Levels of
Evidence

Grade of
Recommendation

References

5/22/2012 5:57:42 PM

674

Surgery: Evidence-Based Practice

REFERENCES
1. Daily PO, Trueblood HW, Stinson EB, Wuerflein RD, Shumway
NE. Management of acute aortic dissections. Ann Thorac Surg.
1970;10(3):237-247.
2. Crawford ES. The diagnosis and management of aortic dissection. JAMA. 1990;264(19):2537-2541.
3. Prtre R, Von Segesser LK. Aortic dissection. Lancet. 1997;349
(9063):1461-1464.
4. von Kodolitsch Y, Schwartz AG, Nienaber CA. Clinical prediction
of acute aortic dissection. Arch Intern Med. 2000;160(19):29772982.
5. Bavaria JE, Pochettino A, Brinster DR, et al. New paradigms and
improved results for the surgical treatment of acute type A dissection. Ann Surg. 2001;234(3):336-342.
6. Hagan PG, Nienaber CA, Isselbacher EM, et al. The International
Registry of Acute Aortic Dissection (IRAD): new insights into an
old disease. JAMA. 2000;283(7):897-903.
7. Tsai TT, Fattori R, Trimarchi S, et al. for the International Registry
of Acute Aortic Dissection. Long-term survival in patients presenting with type B acute aortic dissection: insights from the International Registry of Acute Aortic Dissection. Circulation. 2006;114(21):
2226-2231.
8. Nienaber CA, Eagle KA. Aortic dissection: new frontiers in diagnosis and management: Part I: from etiology to diagnostic strategies. Circulation. 2003;108(5):628-635.
9. Jex RK, Schaff HV, Piehler JM, et al. Early and late results following repair of dissections of the descending thoracic aorta. J Vasc
Surg. 1986;3(2):226-237.
10. Black JH. Aneurysms caused by connective tissue abnormalities.
In: Cronenwett J and Wayne Johnston K. Rutherfords Vascular
Surgery, 7th ed. Chapter 140. Philadelphia, PA: Elsevier Saunders; 2010.
11. Januzzi JL, Isselbacher EM, Fattori R, et al. for the International
Registry of Aortic Dissection (IRAD). Characterizing the young
patient with aortic dissection: results from the International Registry of Aortic Dissection (IRAD). J Am Coll Cardiol. 2004;43(4):
665-669.
12. Syed MA, Fiad TM. Transient paraplegia as a presenting feature
of aortic dissection in a young man. Emerg Med J. 2002;19(2):
174-175.
13. Hata N, Tanaka K, Imaizumi T, et al. Clinical significance of pleural effusion in acute aortic dissection. Chest. 2002;121(3):825-830.
14. Petasnick JP. Radiologic evaluation of aortic dissection. Radiology. 1991;180(2):297-305.
15. LePage MA, Quint LE, Sonnad SS, Deeb GM, Williams DM.
Aortic dissection: CT features that distinguish true lumen from
false lumen. AJR Am J Roentgenol. 2001;177(1):207-211.
16. Fruehwald FX, Neuhold A, Fezoulidis J, et al. Cine-MR in dissection of the thoracic aorta. Eur J Radiol. 1989;9(1):37-41.
17. Tomiguchi S, Morishita S, Nakashima R, et al. Usefulness of turbo-FLASH dynamic MR imaging of dissecting aneurysms of the
thoracic aorta. Cardiovasc Intervent Radiol. 1994;17(1):17-21.
18. Keren A, Kim CB, Hu BS, et al. Accuracy of biplane and multiplane
transesophageal echocardiography in diagnosis of typical acute
aortic dissection and intramural hematoma. J Am Coll Cardiol.
1996;28(3):627-636.
19. Conrad MF, Cambria RP. Aortic dissection. In: Cronenwett J
and Wayne Johnston K. Rutherfords Vascular Surgery, 7th ed.,
Chapter 135. Philadelphia, PA: Elsevier Saunders; 2010.
20. Williams DM, Lee DY, Hamilton BH, et al. The dissected aorta:
percutaneous treatment of ischemic complicationsprinciples
and results. J Vasc Interv Radiol. 1997;8(4):605-625.

PMPH_CH84.indd 674

21. Chung JW, Elkins C, Sakai T, et al. True-lumen collapse in aortic dissection: part II. Evaluation of treatment methods in phantoms with pulsatile flow. Radiology. 2000;214(1):99-106.
22. Nienaber CA, Eagle KA. Aortic dissection: new frontiers in diagnosis and management: Part II: therapeutic management and
follow-up. Circulation. 2003;108(6):772-778.
23. Kim KH, Moon IS, Park JS, Koh YB, Ahn H. Nicardipine hydrochloride injectable phase IV open-label clinical trial: study on
the anti-hypertensive effect and safety of nicardipine for acute
aortic dissection. J Int Med Res. 2002;30(3):337-345.
24. Trimarchi S, Nienaber CA, Rampoldi V, et al. for the IRAD Investigators. Role and results of surgery in acute type B aortic dissection: insights from the International Registry of Acute Aortic
Dissection (IRAD). Circulation. 2006;114(1 Suppl):I357-I364.
25. Estrera AL, Miller CC 3rd, Huynh TT, et al. Preoperative and
operative predictors of delayed neurologic deficit following repair
of thoracoabdominal aortic aneurysm. J Thorac Cardiovasc Surg.
2003;126(5):1288-1294.
26. Safi HJ, Miller CC 3rd, Reardon MJ, et al. Operation for acute and
chronic aortic dissection: recent outcome with regard to neurologic
deficit and early death. Ann Thorac Surg. 1998;66(2):402-411.
27. Pearce BJ, Passman MA, Patterson MA, et al. Early outcomes of
thoracic endovascular stent-graft repair for acute complicated
type B dissection using the Gore TAG endoprosthesis. Ann Vasc
Surg. 2008;22(6):742-749.
28. Buth J, Harris PL, Hobo R, et al. Neurologic complications associated with endovascular repair of thoracic aortic pathology:
Incidence and risk factors. a study from the European Collaborators on Stent/Graft Techniques for Aortic Aneurysm Repair
(EUROSTAR) registry. J Vasc Surg. 2007;46(6):1103-1110.
29. Leurs LJ, Bell R, Degrieck Y, Thomas S, Hobo R, Lundbom J, for
the EUROSTAR; UK Thoracic Endograft Registry collaborators.
Endovascular treatment of thoracic aortic diseases: combined
experience from the EUROSTAR and United Kingdom Thoracic
Endograft registries. J Vasc Surg. 2004;40(4):670-679.
30. Eggebrecht H, Nienaber CA, Neuhuser M, et al. Endovascular
stent-graft placement in aortic dissection: a meta-analysis. Eur
Heart J. 2006;27(4):489-498.
31. Tiesenhausen K, Amann W, Koch G, Hausegger KA, Oberwalder
P, Rigler B. Cerebrospinal fluid drainage to reverse paraplegia after
endovascular thoracic aortic aneurysm repair. J Endovasc Ther.
2000;7(2):132-135.
32. Chiesa R, Melissano G, Marrocco-Trischitta MM, Civilini E,
Setacci F. Spinal cord ischemia after elective stent-graft repair of
the thoracic aorta. J Vasc Surg. 2005;42(1):11-17.
33. Elefteriades JA, Hammond GL, Gusberg RJ, Kopf GS, Baldwin
JC. Fenestration revisited. A safe and effective procedure for
descending aortic dissection. Arch Surg. 1990;125(6):786-790.
34. Barnes DM, Williams DM, Dasika NL. A single-center experience treating renal malperfusion after aortic dissection with
central aortic fenestration and renal artery stenting. J Vasc Surg.
2008;47(5):903-910; discussion 910-911.
35. Beregi JP, Prat A, Gaxotte V, Delomez M, McFadden EP. Endovascular treatment for dissection of the descending aorta. Lancet. 2000;356(9228):482-483.
36. Patel HJ, Williams DM, Meerkov M, Dasika NL, Upchurch GR
Jr, Deeb GM. Long-term results of percutaneous management of
malperfusion in acute type B aortic dissection: implications for
thoracic aortic endovascular repair. J Thorac Cardiovasc Surg.
2009;138(2):300-308.
37. Onitsuka S, Akashi H, Tayama K, et al. Long-term outcome and
prognostic predictors of medically treated acute type B aortic
dissections. Ann Thorac Surg. 2004;78(4):1268-1273.

5/22/2012 5:57:42 PM

Aortic Dissection

38. Juvonen T, Ergin MA, Galla JD, et al. Risk factors for rupture
of chronic type B dissections. J Thorac Cardiovasc Surg. 1999;
117(4):776-786.
39. Black JH, Cambria RP. Aortic Dissection: perspectives for the
vascuar/endovascular surgeon. In: Rutherfords Vascular Surgery.
6th ed., Chapter 104. Philadelphia, PA: Elsevier Saunders; 2005.
40. Tsai TT, Evangelista A, Nienaber CA, et al.for the International
Registry of Acute Aortic Dissection. Partial thrombosis of the
false lumen in patients with acute type B aortic dissection. N
Engl J Med. 2007;357(4):349-359.
41. Akutsu K, Nejima J, Kiuchi K, et al. Effects of the patent false
lumen on the long-term outcome of type B acute aortic dissection. Eur J Cardiothorac Surg. 2004;26(2):359-366.
42. Dialetto G, Covino FE, Scognamiglio G, et al. Treatment of type
B aortic dissection: endoluminal repair or conventional medical
therapy? Eur J Cardiothorac Surg. 2005;27(5):826-830.
43. Xu SD, Huang FJ, Yang JF, et al. Endovascular repair of acute type
B aortic dissection: early and mid-term results. J Vasc Surg. 2006;
43(6):1090-1095.
44. Schoder M, Czerny M, Cejna M, et al. Endovascular repair of acute
type B aortic dissection: long-term follow-up of true and false
lumen diameter changes. Ann Thorac Surg. 2007;83(3):1059-1066.
45. Palma JH, de Souza JA, Rodrigues Alves CM, Carvalho AC, Buffolo
E. Self-expandable aortic stent-grafts for treatment of descending
aortic dissections. Ann Thorac Surg. 2002;73(4):1138-1141; discussion 1141-1142.
46. Nathanson DR, Rodriguez-Lopez JA, Ramaiah VG, et al. Endoluminal stent-graft stabilization for thoracic aortic dissection. J Endovasc Ther. 2005;12(3):354-359.

PMPH_CH84.indd 675

675

47. Nienaber CA, Zannetti S, Barbieri B, Kische S, Schareck W, Rehders TC, for the INSTEAD study collaborators. INvestigation of
STEnt grafts in patients with type B Aortic Dissection: design of
the INSTEAD triala prospective, multicenter, European randomized trial. Am Heart J. 2005;149(4):592-599.
48. Nienaber CA, Kische S, Akin I, et al. Strategies for subacute/
chronic type B aortic dissection: the Investigation Of Stent
Graft s in Patients with type B Aortic Dissection (INSTEAD)
trial 1-year outcome. J Thorac Cardiovasc Surg. 2010;140(6 Suppl):
S101-S108.
49. Nienaber CA, Rousseau H, Eggebrecht H, et al.for the INSTEAD
Trial. Randomized comparison of strategies for type B aortic
dissection: the INvestigation of STEnt Grafts in Aortic Dissection (INSTEAD) trial. Circulation. 2009;120(25):2519-2528.
50. Slonim SM, Miller DC, Mitchell RS, Semba CP, Razavi MK, Dake
MD. Percutaneous balloon fenestration and stenting for lifethreatening ischemic complications in patients with acute aortic
dissection. J Thorac Cardiovasc Surg. 1999;117(6):1118-1126.
51. Beregi JP, Haulon S, Otal P, et al. for the Societ Fraaise dImagerie Cardio-Vasculaire (SFICV) Research Group on Aortic
Dissection. Endovascular treatment of acute complications associated with aortic dissection: midterm results from a multicenter
study. J Endovasc Ther. 2003;10(3):486-493.
52. Panneton JM, Hollier LH. Dissecting descending thoracic and
thoracoabdominal aortic aneurysms: Part II. Ann Vasc Surg.
1995;9(6):596-605.
53. Hollier LH, Symmonds JB, Pairolero PC, Cherry KJ, Hallett JW,
Gloviczki P. Thoracoabdominal aortic aneurysm repair. Analysis of postoperative morbidity. Arch Surg. 1988;123(7):871-875.

5/22/2012 5:57:42 PM

Commentary on
Aortic Dissection
Karthikeshwar Kasirajan

In the chapter titled Aortic Dissection, Benjamin J. Pearce has


attempted to summarize the available data on acute and chronic
type A and type B dissection. A significant portion is dedicated to
the evolving stent graft technology to treat dissections. Although
no devices are specifically approved in the United States for dissections, this is a rapidly evolving field with interest now extending into the development of percutaneous aortic valve with stent
grafts for the treatment of acute type A dissection. Although little is dedicated to the treatment of chronic dissections, various
authors have described promising results with use of stents grafts
for patients with chronic dissections.

hypertension if any one of the renals has a partial or complete


false lumen blood supply. This technique involves placement of
self-expanding nitinol stents extending from the true lumen to
the involved renal vessel. Endovascular therapy with stent grafts
has rapidly replaced open surgery in patients with complicated
type B dissections. However, current data and available devices
do not support their use in patients who can be managed with
anti-impulse therapy. Regarding fenestration, in my experience
this technique is rarely required to relieve end-organ ischemia
and should be avoided if possible. Extensive areas of fenestration
in the thoracic aorta and the visceral segment often result in
rapid false lumen enlargement during follow-up. In these patients with prior fenestration and false lumen aneurysm few
endovascular options are available, converting this to a complex
open procedure. In my practice, fenestration is limited to patients
presenting with acute paraplegia. Follow-up imaging on a yearly
basis is important as a significant number of the patients with
developed false lumen dilatation require delayed intervention
despite adequate blood pressure control.
4. Outcomes: Although no long-term data are available for stent
graft therapy for acute dissections, mid-term data are very
promising. I have noticed that a significant number of patients
treated in the first few months have the capacity for total aortic
remodeling, that is, the aorta reverts to the predissection state
once the entry tear has been covered. It is important to note
that long-segment (subclavian to celiac) coverage should be
avoided in acute dissections, as this substantially increases
the risk of paraplegia with no added benefit. I typically would
use a 15-cm thoracic stent graft in most patients with minimal
oversizing (size to the normal nondissected proximal aorta).
5. Chronic dissection: The authors have dedicated little to this less
understood entity. In spite of the resistance to the use of stent
grafts for dissections, I have found this to be very effective
in causing false lumen thrombosis across the stented segment of
the aorta, despite distal perfusion of the false lumen. Based on
my experience, I believe that chronic dissections, despite having
a false lumen aneurysm, do not behave in a similar fashion to
degenerative aneurysms. Elimination of the primary entry tear
by stent grafts results in false lumen remodeling in most patients,
suggestive of a different mechanism for aneurysm formation
other than pressure. Factors such as shear stress and flow velocity
tend to play a more important role in false lumen dilatation of
these patients.

1. Who gets aortic dissection and how do they present? Aortic


dissection is most commonly seen in patients with uncontrolled
hypertension. It is important to note that most often patients
are thought to present with an acute myocardial infarction or
a pulmonary embolism. A CT or a MRA is obtained usually
after these have been ruled out. Early diagnosis and treatment
cannot be stressed enough, given the high mortality with each
passing hour.
2. What modalities are appropriate for diagnosis and therapeutic
planning? Importance of early contrast-enhanced CTA or MRA
are well described by the author. It is also important to get a
complete imaging of the chest/abdomen and pelvis, as various
visceral vessels can be involved and may need to be incorporated
into the treatment plan. Although indication for a CTA and
MRA often overlap, I have found CTA to be more user-friendly
with regard to evaluation of the branch vessels, and obtaining
the required measurements for stent graft placement. ECHO
is often limited to the unstable patient or an unreliable CTA
or MRA due to excessive patient movement. I have found the
intraoperative use of transesophageal ECHO to be a valuable
tool to exclude a type A conversion after stent graft placement.
3. Which patients are appropriate for medical management and
which patients require operative interventions? Typically all
patients with proximal dissections require operative interventions. Dissections distal to the left subclavian artery are best
managed with anti-impulse therapy unless they have complications. Indications for nonmedical intervention in type B dissections include rupture, rapid false lumen enlargement,
persistent back pain despite blood pressure control, and endorgan hypoperfusion (visceral or limbs). I have also found
renal stenting to be useful in patients with difficult to control
676

PMPH_CH84.indd 676

5/22/2012 5:57:42 PM

CHAPTER 85

Arterial Pseudoaneurysms and


Arteriovenous Fistulae
J. Leigh Eidson III and Marvin D. Atkins

undoubtedly result in considerable economic impact.1-2,7-10 Consequently, numerous products and devices have been developed in
an attempt to reduce these complications while curtailing procedure times.
The following questions were selected to provide practitioners
with clinically useful information collected from the most current
and validated scientific literature.

INTRODUCTION
As the number of percutaneous arterial catheter-based treatments
has surged over the past two decades, complications of such interventions including arteriovenous fistulae (or arteriovenous fistulas
[AVFs]) and arterial pseudoaneurysms (PAs) have become common. Pseudoaneurysms, sometimes referred to as pulsatile hematomas, differ from aneurysms in that the boundaries of the aneurysm
sac do not contain elements of the arterial wall but rather consist of surrounding soft tissue, muscle, compressed thrombus, or
even dermis. Penetrating trauma, usually iatrogenic, frequently
initiates the PA. As arterial blood is exposed to the extralumenal
tissues, it dissects and forms a cavity that remains pressurized by
its continuity with the intravascular space. Unlike a hematoma,
continuous flow within a PA prevents clot formation and sealing
of the arteriotomy.
Femoral PAs typically present as a pulsatile mass and bruit
that forms at the arterial puncture site within 48 hours of an arterial procedure. Pain and swelling is the most common presentation. Infrequently, patients may also experience femoral vein and
nerve compressionleading to deep vein thrombosis or neuropathy. Occasionally a large PA will result in uncontrolled hemorrhage, hypotension, and even skin necrosis and free rupture.
An AVF is a communication between an artery and vein. An
AVF is similar to PA in that it is typically the result of vascular
trauma or percutaneous intervention, occurring in 0.001% to
2.8% of catheter-based treatments.1-5 Such communications result
in abnormal arterial to venous flow, which may be palpated as a
thrill or auscultated as a bruit. Although usually clinically insignificant at the femoral vessels, AVF can rarely lead to high cardiac
output, resulting in heart failure.
According to a recent report from the American Heart Association, one in three Americans will die from cardiovascular disease and half of these specifically from coronary artery disease. In
the year 2007 alone, nearly 1,059,000 cardiac catheterizations with
622,000 interventions were performed in the United States.6 With
the incidence of iatrogenic PA reported from 0.1% to as high as 5.5%
following percutaneous arterial procedures, such complications

RISK FACTORS
1. What are the risk factors for the development of PAs and
AVFs?
At the origin of every PA and AVF is the initiating vascular injury.
Many variables exist that may predispose patients to complications of vascular access. In addition to recognizable factors such
as catheter size, other considerations include catheter removal and
hemostatic techniques, location of arteriotomy, gender, hypertension, anticoagulation, vascular calcification, and obesity. Frequently, the location of the arteriotomy is either too high (distal
external iliac) or too low (bifurcation of the superficial femoral
and profunda femoris or the superficial femoral itself).
In a large prospective study of 11,992 consecutive patients
undergoing cardiac catheterization, Popovic et al.9 reported 76
(0.6%) PAs. Significant risk factors included catheter size (>6 fr;
P = 0.03), percutaneous intervention (OR, 1.99; P < .05), and left
femoral access site (OR, 4.65; P < .05) (Level 2a evidence). Ates
et al.7 in their series of 41,322 catheterizations causing 630 PAs
(1.5%) found hypertension, coronary artery disease, diabetes,
catheter size, body-mass index (OR, 2.21), procedure room volume, and catheter size (>7 fr; OR, 2.82) to all be statistically significant risk factors (Level 2b evidence).
Ohlow and colleagues8 similarly report PA and AVF rates
of 1.2% and 0.6%, respectively, in their series of 18,165 patients
undergoing cardiac catheterization. In addition to hypertension,
they discovered a statistically significant increased risk for women
(OR, 1.65) and in patients treated with emergency procedures
(OR, 2.13) (Level 2a evidence).
677

PMPH_CH85.indd 677

5/22/2012 5:58:15 PM

678

Surgery: Evidence-Based Practice

Juergens et al.11 in their prospective, randomized controlled


trial found no difference in vascular complication rates between
6-fr and 7-fr catheters used to treat 414 patients (Level 1b evidence). Another study found that ambulation at 4 hours, after
manual compression was used for hemostasis, does not increase
the risk of vascular complications12 (Level 3b evidence). Arterial
access over the femoral head is less likely to have associated complications because greater seal is attained when the artery is compressed against this structure13 (Level 2b evidence).
Perings and colleagues5 in analyzing 88 patients with AVFs
following 10,271 cardiac catheterizations found that female gender, left groin access, hypertension, and systemic anticoagulation
all increased the risk of fistula formation. Other factors such as
sheath size, interventions, and body-mass index did not increase
the risk of AVF (Level 2b evidence).
Answer: Catheter size, percutaneous intervention, female
gender, left groin access, body-mass index, procedure room volume, diabetes mellitus, and hypertension are all risk factors for
developing femoral artery PA. Female gender, left groin access,
hypertension, and systemic anticoagulation increase ones risk of
developing an AVF (Grade A statement).

DIAGNOSIS
2. What is the optimal imaging study to diagnose PAs and
AVFs?
Duplex ultrasound has superseded angiography as the diagnostic
study of choice for detecting femoral PAs and AVFs over the past
25 years. Duplex ultrasound is noninvasive, avoids radiation exposure and intravascular contrast, and has become widely accessible
with many clinicians using them in their daily practices.
In the mid-1980s, Doppler ultrasound was first reported to
be 94% sensitive and 95% specific14 (Level 4 evidence). The flow
characteristics of the PAs, initially known as the to-and-fro,
distinguished them from hematomas.15 Ultimately, ultrasound
technology and experience improved that led to a widespread
acceptance despite the lack of data characterizing these intrinsic
qualities. Additional advantages in this setting include the quick
ability to detect local disease in the arterial system and deep
venous thrombus.
Computed tomography angiography (CTA), although it exposes
patients to radiation and IV contrast, can be a useful tool for detecting PAs. It is a valuable study when there is a concern of proximal
arterial injury with associated retroperitoneal hemorrhage. CTA
can also differentiate vascular calcifications, thus characterizing
the severity of underlying peripheral atherosclerotic disease.
Magnetic resonance angiography (MRA) is an alternative
form of angiography that is less commonly used in this setting.
Although cumbersome and expensive, it may be clinically useful
in the setting of complex AVFs or PAs being evaluated for surgical repair. There has been some recent concern regarding the
use of gadolinium contrast in patients with chronic renal disease
as they are at increased risk for developing nephrogenic systemic
fibrosis.16
Regardless of which imaging modality is used, the patients
history and physical examination remain essential to good practice. Documenting symptoms of peripheral vascular disease, distal pulse characteristics, and ankle-brachial indices both before

PMPH_CH85.indd 678

and after any invasive procedure will aid significantly in the recognition of complications.
Answer: Despite the lack of formal studies comparing duplex
ultrasound to angiography, clinicians were quick to adopt this
pragmatic tool given its inherent attributes. There remains a
secondary role for catheter-based, CTA and MRA (Grade C
statement).

TREATMENT
3. When do AVFs require surgical repair and when can they
be observed?
Iatrogenic AVFs are difficult to study because they are somewhat
rare and mostly asymptomatic. Small cohorts have reported a 66%
to 81% spontaneous closure rate for AVFs not selected for immediate repair17,18 (Level 4 evidence). Kelm and colleagues reported
a rate of 0.86% for AVF formation for 10,271 consecutive femoral
catheterization patients that they evaluated with duplex ultrasound. Interestingly, more than one-third of these spontaneously
closed within 12 months, and all remained asymptomatic. In their
series, none of the AVFs resulted in cardiac dysfunction or limb
threat4 (Level 2b evidence).
Answer: Most small and asymptomatic AVFs can safely be
observedawaiting spontaneous closure. If persistent, large, or
symptomatic, open surgical repair is recommended. Stent graft
repair of such lesions, although several case reports suggest this
is technically feasible, is not recommended in the femoral location except in the most extenuating of circumstances (Grade C
recommendation).
4. What is the optimal treatment of femoral PAs: thrombin
injection, ultrasound compression, or surgical repair?
Traditionally, the standard treatment for arterial PAs was surgical
repair with direct closure of the arteriotomy. By the nature of their
underlying vascular disease, patients undergoing surgical repair
for PAs have complication and mortality rates reported as high as
21% and 3.3%, respectively19-20 (Level 2c evidence).
Compression of the PA, either by hand or with a mechanical
device, can disrupt the flow characteristics within the aneurysm
sac leading to thrombosis and cure. One early report demonstrated
that manual compression of up to 1 hour was successful in nearly
87% of 85 patients selected for this method (74% on first attempt).
Of the 96 patients enrolled, 10 were treated with primary surgical
repair, as were all study failures21 (Level 3b evidence).
Treatment with compression dressing alone is less useful and
has a reported 25.8% success rate, useful only for small PA with low
neck velocities.22 Furthermore, for small PA (<2 cm), there is some
evidence that with observation alone 56% to 87% could resolve
within a few weeks17-18 (Level 3b evidence).
As ultrasound devices became more widespread, clinicians
were quick to utilize them as a compression tool so that PA could
be observed during the process. Ultrasound-guided compression
(UGC) has reported success rates of 74% to 95%, with much variation in compression times and the number of attempts23-25 (Level
3b evidence). In one study, UGC was 95% successful in 57 patients;
however, only 83% of those were cured on first attempt. In that
study, heparinized patients were more likely to fail treatment.25

5/22/2012 5:58:16 PM

Arterial Pseudoaneurysms and Arteriovenous Fistulae

Paschalidis et al. in a prospective randomized study, selected 168


of 185 PAs for treatment by either manual compression or UGC
with 98% and 99% success rates, respectively. In their experience,
only a handful required more than one treatment session and success was defined as clinical resolution26 (Level 2b evidence).
Compression techniques have lost favor because they are labor
intensive and uncomfortable for patients, often requiring multiple
attempts at time intervals as long as 41.5 minutes in one study.23
With compression, there is a small risk for developing arterial and
deep venous thrombosis.
Direct thrombin injection into the PA sac, typically guided by
ultrasound, results in rapid sac thrombosis with minimal patient
discomfort. This technique is usually accompanied by some degree
of compression, which promotes thrombus formation. Eleven case
series, reporting 757 cases of iatrogenic femoral PAs treated with
ultrasound-guided thrombin injection (UGTI) were reviewed. The
overall success rate of UGTI ranged from 90% to 100%. The rates
for a successful first injection, or primary treatment, varied from
71% to 100%, with the majority of authors indicating that additional
treatments were commonly required23-24,27-35 (Level 3a evidence).
Complications of UGTI were unusual. Five arterial embolic
complications were reported, but none caused significant morbidity or limb loss. 28-29,33,36 There were 18 reports of PA recurrence following UGTI, underscoring the importance of follow-up
imaging for patients with this complication. Many recurrences
occurred early but some were detected as late as 4 weeks following
treatment.24,33,36
The dosage of thrombin injected varied greatly between studies. In the largest series reviewed, Krueger et al. reported that a
mean thrombin dose of 425 IU (1000 IU/mL concentration) was
used to attain a 93.8% primary, and 99.6 secondary success rates.
Large, multilobulated PAs were more likely to require a larger
injection and secondary treatments.33
Embolization by ultrasound-guided coil placement is an alternative to thrombin injection, though not well studied. In one small
series, Kobeiter et al. reported a 100% embolization rate inserting an average 3.8 (size: 415 cm 10 mm) stainless steel spring
coils containing synthetic fibers. Two of the 17 patients (11.7%) had
recurrences discovered at 1 and 4 weeks postprocedure37 (Level 4
evidence). In a recent case report, Bellmunt et al. described a
new technique of PA treatment using a single 20-cm Inconel coil
(IMWCE; Cook Medical Inc., Bloomington Ind), part of which was
left externally exposed and later removed.38
Answer: Manual or UGC is still a viable treatment for uncomplicated PAs and can be performed at bedside. Thrombin injection is
more rapid, causes less discomfort, and ought to be considered firstline for large or complex PAs. Distal perfusion should be assessed
before and after any intervention so that limb-threatening complications can be rapidly detected. Direct surgical repair should only
be considered for patients who repeatedly fail less-invasive methods, have hemodynamic instability, or those at high risk for PA
rupture, or when embolic complications are encountered (Grade B
recommendation).
5. Is there are role for endovascular therapy in the management of femoral PAs and AVFs?
At this time, there are only case reports and small case series
describing the successful placement of covered stent grafts in the
femoral artery to exclude AVF and PA.39,40

PMPH_CH85.indd 679

679

Waigand and colleagues reported treating 47 of 53 patients


(30 PAs, 21 AVFs, 2 combined lesions) with either covered stents
(32) or endovascular embolization (14) and one by a combined
procedure. Six failures were treated surgically. Ultimately, the
authors report four late stent occlusions (12.5%) with a followup of 301 ( 280 days)40 (Level 4 evidence).
In their small series, Onal et al. reported successfully treating
9/10 AVFs involving the profunda femoris artery using covered
stent grafts. Mean follow-up was 18.5 months, at which time Doppler ultrasound demonstrated 100% stent graft patency.41 Thalhammer et al. repaired 16 PAs, 9 AVFs, and 1 combined lesion
using 26 covered stent grafts. In addition to three immediate failures, four occluded stents (17%) were detected at 1 year42 (Level 4
evidence).
Answer: The use of covered stent grafts as primary repair for
femoral PAs and AVFs is not recommended as a routine first-line
treatment. Stent occlusion and fracture remain a concern and longterm patency has not been reported (Grade C recommendation).

PREVENTION
6. What is the role of vascular closure devices in the management of arterial access catheterization sites?
Attaining natural hemostasis by direct pressure over arteriotomy
site is labor intensive and can be unpredictable in the setting
of antiplatelet and anticoagulation therapy. Numerous devices
and products, known as vascular closure devices (VCDs), have
been brought to market over the past two decades with intent to
facilitate arteriotomy seal while reducing the risk of access site
complications. VCDs work through a variety of methods including mechanized external compression, deposition of hemostatic
material near arteriotomy, the use hemostatic dressings, and
reapproximation of the arteriotomy. With the global market
of such devices expected to exceed $1 billion by 2013, further
refi nement and ongoing development is expected.43 Table 85.1
lists several common devices, their manufacturers, and specific
functions.
With a huge commercial interest in VCDs, the evidence supporting their use is somewhat mixed. Lack of standardization of
manual compression controls and lack of intent-to-treat methodology obscures results. As technology and devices rapidly evolve,
there remains lag between reported results and the generation of
device currently in use.
Mechanical compression devices are the simplest VCDs.
Clamp-style or inflatable (pressure regulated) appliances are
usually anchored to the bed or counter-supported by the patients
body, replacing manual digital compression. In a recent prospective study involving 908 patients, the risk of PA formation with
mechanical compression (3.3%) was found to be statistically
equivalent to manual compression with two different hemostatic
dressings (calcium ion 4.3%, thrombin 3.3%).44 Although quite
popular given their ease of use, compressive devices limit early
ambulation and can result in substantial discomfort.
Implanted substances, such as collagen, polyethylene glycol,
and thrombin, are thought to aid in hemostasis directly at the
arteriotomy site. Two of the earliest devices that utilized this concept, Duett and VasoSeal, are no longer available secondary to
concerns about increased complication rates. The meta-analysis

5/22/2012 5:58:16 PM

680

Surgery: Evidence-Based Practice

Table 85.1 Vascular Closure Devices


Device

Manufacturer

Function

AngioSeal

St. Jude Medical, St. Paul, MN

Implanted collagen plug

VasoSeal*

Maquet, Montvale, NJ

Implanted collagen plug

Duett*

Vascular Solutions, Minneapolis, MN

Implanted thrombin and collagen

Mynx

AccessClosure, Mountain View, CA

Implanted polyethylene glycol

D-Stat Dry

Vascular Solutions, Minneapolis, MN

Topical thrombin dressing

Syvek

Marine Polymer Technologies, Danvers, MA

Topical poly-N-acetyl glucosamine

Prostar

Abbott Vascular, Redwood City, CA

Direct suture closure

Perclose

Abbott Vascular, Redwood City, CA

Direct suture closure

Starclose

Abbott Vascular, Redwood City, CA

Direct clip closure

Catalyst III

Cardiva Medical, Mountainview, CA

Removable internal clip with protamine-coated wire

FemoStop

St. Jude Medical, St. Paul, MN

External compression

Safeguard

Maquet, Mahwah, NJ

External compression

CompressAR

Advanced Vascular Dynamics, Portland, OR

External compression

Clamp Ease

ClampEase, Portland, OR

External compression

*Device is no longer produced.

by Koreny et al. that investigated 30 prospective randomized trials


(N = 4000) primarily investigated Angioseal, Vasoseal, and Perclose. In their analysis, they found that the use of VCDs shortened procedure times by a mean of 17 minutes and resulted in
no significant difference in the number of PA or AVF compared
to manual compression. However, the subanalysis of the only two
trials to utilize an intention-to-treat methodology and blindedoutcomes had a significant increase in the relative risk of PA formation (RR 5.4, P = .02)45 (Level 1a evidence).
Nikolsky et al., in another meta-analysis, analyzed data from
30 prospective and retrospective studies, representing 37,066
patients treated with AngioSeal, VasoSeal, and Perclose. Again,
no significant differences in risk were detected for all device
groups except for VasoSeal, which was found to have a significant
association with vascular complications (OR 2.27, 95% CI = 1.35
3.8). Several of the reports in this analysis were published early
in the endovascular experience, using first-generation products46
(Level 2a evidence).
A more recent meta-analysis by Biancari et al. studied 7528
patients from 31 prospective, randomized studiesmany of which,
the authors comment, excluded high-risk patients. Comparable to
earlier meta-analyses, there was no statistically significant difference in occurrence of PA. They did report a significant risk of groin
infection with VCD use (RR 2.48, P = .02) as well as a trend toward
increased risk of limb ischemic complications (RR 3.28, P = .07),
though the VasoSeal data contributed positively toward both of
these complications47 (Level 1a evidence).
The StarClose device is less studied as it is newer to the
marketplace. To date, major complication rates of 0.4% to 3.4%
have been reported. It is currently approved for closure of 5- to
6-fr catheter sites and has the advantage of providing a suitable site
for future arterial access. Device deployment success is reported
between 87% and 95.7%.48-50
Answer: VCDs reduce procedure times by supplanting
manual compression; however, there is no evidence that they

PMPH_CH85.indd 680

decrease vascular access complications. Further studies are


needed to characterize the advantages of newer products and to
clearly establish whether or not they are cost-effective (Grade A
recommendation).

SPECIAL CIRCUMSTANCES
7. How does one manage the infected femoral PA?
Almost always related to intravenous drug abuse, infected femoral PAs traditionally require operative debridement with arterial
ligation and aneurysmectomy. When ligation results in limbthreatening ischemia, surgeons should attempt to revascularize
the extremity.
Johnson and colleagues reported that 32% of 28 patients
with ligation of the common femoral artery or femoral bifurcation had limb-threatening ischemia. Six of these required arterial
bypass. Distal stump arterial pressure may be useful for determining adequacy of collateral circulation. Extra-anatomic bypass
through the obturator foramen using Dacron or polytetrafluoroethylene (PTFE) should be considered in these circumstances,
though with a high risk of graft infection or thrombosis51-53 (Level
4 evidence).
Alternatively, CryoVein (CryoLife, Kennesaw, GA) human
allografts have been used in the reconstruction of infected arterial graft s and might be considered in this setting54 (Level 5
evidence).
Endovascular repair using a stent graft has been reported, but
should only be considered as a temporizing measure in moribund
patients who cannot undergo immediate ligation or reconstruction55 (Level 5 evidence).
Answer: Uncommon, infected femoral PAs should be excised
with or without extra-anatomic bypass. Endovascular stenting is
not recommended (Grade D recommendation).

5/22/2012 5:58:16 PM

Arterial Pseudoaneurysms and Arteriovenous Fistulae

681

Clinical Question Summary


Question

Answer

Level of
Evidence

Grade of
Recommendation

References

1 What are the risk factors


for the development of
PAs and AVFs?

PA risk: catheter size, PCI, female


gender, left groin access, HTN,
obesity, diabetes
AVF risk: anticoagulation, female
gender, left groin access, HTN

1b

5, 7-13

2 What is the optimal


imaging study to diagnose
PA and AVF?

Duplex ultrasound, angiography

14-15

3 When do AVF require


surgical repair and when
can they be observed?

If asymptomatic and small, AVF can be


observed

2b

17-18

4 What is the optimal


treatment for femoral PA?

Thrombin injection is faster and


moderately more effective than
compression

3a

19-38

5 Is there a role for


endovascular therapy
in the management of
femoral PA and AVF?

Endovascular repair is not


recommended

39-42

6 What is the role of vascular


closure devices in the
management of arterial
access catheterization
sites?

Closure devices decrease procedure


times but they do not reduce access
site complications, their costeffectiveness is unknown

1a

44-50

7 How does one manage the


infected femoral PA?

Femoral artery ligation with


aneurysmectomy, reconstruction in
selected patients

51-55

PA = pseudoaneurysm, AVF = arteriovenous fistula, HTN = hypertension.

REFERENCES
1. Castillo-Sang M, Tsang AW, Almaroof B, et al. Femoral artery
complications after cardiac catheterization: a study of patient profile. Ann Vasc Surg. 2010;24:328-335.
2. Kresowik TF, Khoury MD, Miller BV, et al. A prospective stud
of the incidence and natural history of femoral vascular complications after percutaneous translumenal coronary angioplasty. J
Vasc Surg. 1991;13(2):328-333.
3. Ricci MA, Trevisani GT, Pilcher DB. Vascular complications of
cardiac catheterization. Am J Surg. 1994;167(4):375-378.
4. Kelm M, Perings SM, Jax T, et al. Incidence and clinical outcome
of iatrogenic femoral arteriovenous fistulas: implications for risk
stratification and treatment. J Am Coll Cardiol. 2002 Jul 17;0(2):
291-297.
5. Perings SM, Kelm M, Jax T, Strauer BE. A prospective study on
incidence and risk factors of arteriovenous fistulae following
transfemoral cardiac catheterization. Int J Cardiol. 2003;88(2-3):
223-228.
6. Roger VL, Go AS, Lloyd-Jones DM, et al. Heart disease and stroke
statistics2011 update: a report from the American Heart Association. Circulation. 2011;123(4):e18-e209. Epub 2010 Dec 15.
7. Ates M, Sahin S, Konuralp C, Gullu U, et al. Evaluation of risk
factors associated with femoral pseudoaneurysms after cardiac
catheterization. J Vasc Surg. 2006;43(3):520-524.

PMPH_CH85.indd 681

8. Ohlow MA, Secknus MA, Von Korn H, et al. Incidence and outcome
of femoral vascular complications among 18,165 patients undergoing cardiac catheterization. Int J Cardiol. 2009;135(1):66-71.
9. Popovic B, Freysz L, Chometon F, et al. Femoral pseudoaneurysms
and current cardiac catheterization: evaluation of risk factors and
treatment. Int J Cardiol. 2010;141(1):75-80.
10. Sulzebach-Hoke LM, Ratcliffe SJ, Kimmel SE, et al. Predictors of
complications following sheath removal with percutaneous coronary intervention. J Cardiovasc Nurs. 2010;25(3):E1-E8.
11. Juergens CP, Hallani H, Leung DY, et al. Comparison of 6 and 7
French guiding catheters for percutaneous coronary intervention:
results of a randomized trial with a vascular ultrasound endpoint.
Catheter Cardiovasc Interv. 2005;66(4):528-534.
12. Schiks IE, Schoonhoven L, Aengevaeren WR, et al. Ambulation
after femoral sheath removal in percutaneous coronary intervention: a prospective comparison of early vs. late ambulation. J Clin
Nurs. 2009;18(13):1862-1870.
13. Gutzeit A, Graf N, Schoch E, Sautter T, Jenelten R, Binkert CA.
Ultrasound-guided antegrade femoral access: comparison between
the common femoral artery and the superficial femoral artery.
Eur Radiol. 2010 Dec 28 [Epub ahead of print].
14. Helvie MA, Rubin JM, Silver TM, Kresowik TF. The distinction
between femoral artery pseudoaneurysms and other causes of
groin masses: value of duplex Doppler sonography. AJR. 1988;
150:1177-1180.

5/22/2012 5:58:16 PM

682

Surgery: Evidence-Based Practice

15. Abu-Yousef MM, Wiese JA, Shamma AR. The to-and-fro sign:
duplex Doppler evidence of femoral artery pseudoaneurysm.
AJR. 1988;150:632-634.
16. Marckmann P, Skov L, Rossen K, et al. Nephrogenic systemic
fibrosis: suspected causative role of gadodiamide used for contrastenhanced magnetic resonance imaging. J Am Soc Nephrol. 2006;
17(9):2359-2362.
17. Toursarkissian B, Allen BT, Petrinec D, et al. Spontaneous closure of selected iatrogenic pseudoaneurysms and arteriovenous
fistulae. J Vasc Surg. 1997;25:803-808.
18. Kent KC, McArdle CR, Kennedy B, et al. A prospective study of the
clinical outcome of femoral pseudoaneurysms and arteriovenous
fistulas induced by arterial puncture. J Vasc Surg. 1993;17:125-131.
19. Lumsden AB, Miller JM, Kosinski AS, et al. A prospective evaluation of surgically treated groin complications following percutaneous cardiac procedures. Am Surg. 1994;60:132-137.
20. Borioni R, Garofalo M, De Paulis R, et al. Surgical treatment of
femoral artery pseudoaneurysms after cardiac catheterization.
Minerva Chir. 2008;63(4):277-282.
21. Theiss W, Schreiber K, Schomig A. Manual compression repair of
post-catheterization femoral pseudoaneurysms: an alternative to
ultrasound guided compression repair? Vasa. 2002;31(2):95-99.
22. Duszanska A, Dziobek B, Streb W, et al. Predictors of successful
iatrogenic pseudoaneurysm compression dressing repair. Cardiol J. 2010;17(2):179-183.
23. Paulson EK, Sheafor DH, Kliewer MA, et al. Treatment of iatrogenic
femoral arterial pseudoaneurysms: comparison of US-guided
thrombin injection with compression repair. Radiology. 2000;
215(2):403-408.
24. Weinmann EE, Chayen D, Kobzantzev ZV, Zaretsky M, Bass A.
Treatment of post-catheterization false aneurysms: ultrasoundguided compression vs ultrasound-guided thrombin injection.
Eur J Vasc Endovasc Surg. 2002;23(1):68-72.
25. Hajarizadeh H, LaRosa CR, Cardullo P, Rohrer MJ, Cutler BS.
Ultrasound-guided compression of iatrogenic femoral pseudoaneurysm failure, recurrence, and long-term results. J Vasc Surg.
1995;22(4):425-430.
26. Paschalidis M, Theiss W, Kolling K, Busch R, Schomig A. Randomized comparison of manual compression repair versus ultrasound
guided compression repair of postcatheterization femoral pseudoaneurysms. Heart. 2006;92(2):251-252.
27. Kang SS, Labropoulos N, Mansour MA, Baker WH. Percutaneous
ultrasound guided thrombin injection: a new method for treating post catheterization femoral pseudoaneurysms. J Vasc Surg.
1998;27(6):1120-1121.
28. Sackett WR, Taylor SM, Coffey CB, et al. Ultrasound-guided
thrombin injection of iatrogenic femoral pseudoaneurysms: a prospective analysis. Am Surg. 2000;66(10):937-940.
29. Calton WC Jr, Franklin DP, Elmore JR, Han DC. Ultrasoundguided thrombin injection is a safe and durable treatment for
femoral pseudoaneurysms. Vasc Surg. 2001;35(5):379-383.
30. Sultan S, Nicholls S, Madhavan P, Colgan MP, Moore D, Shanik
G. Ultrasound guided human thrombin injection. A new modality in the management of femoral artery pseudo-aneurysms. Eur
J Vasc Endovasc Surg. 2001;22(6):542-545.
31. Lonn L, Olmarker A, Geterud K, Risberg B. Prospective randomized study comparing ultrasound-guided thrombin injection
to compression in the treatment of femoral pseudoaneurysms.
J Endovasc Ther. 2004;11(5):570-576.

PMPH_CH85.indd 682

32. Olsen DM, Rodriguez JA, Vranic M, Ramaiah V, Ravi R, Diethrich


EB. A prospective study of ultrasound scan-guided thrombin
injection of femoral pseudoaneurysm: a trend toward minimal
medication. J Vasc Surg. 2002;36(4):779-782.
33. Krueger K, Zaehringer M, Strohe D, Stuetzer H, Boecker J,
Lackner K. Post catheterization pseudoaneurysm: results of USguided percutaneous thrombin injection in 240 patients. Radiology. 2005;236(3):1104-1110.
34. Etemad-Rezai R, Peck DJ. Ultrasound-guided thrombin injection
of femoral artery pseudoaneurysms. Can Assoc Radiol J. 2003;
54(2):118-120.
35. Vlachou PA, Karkos CD, Bains S, McCarthy MJ, Fishwick G,
Bolia A. Percutaneous ultrasound-guided thrombin injection for
the treatment of iatrogenic femoral artery pseudoaneurysms. Eur
J Radiol. 2009 Aug 4. [Epub ahead of print]
36. Madaric J, Mistrik A, Vulev I, et al. The recurrence of iatrogenic
femoral artery pseudoaneurysm after occlusion by ultrasound
guided percutaneous thrombin injection. EuroIntervention. 2009;
5(4):443-447.
37. Kobeiter H, Lapeyre M, Becquemin J, Mathieu D, Melliere D, Desgranges P. Percutaneous coil embolization of post catheterization
arterial femoral pseudoaneurysms. J Vasc Surg. 2002;36:127-131.
38. Bellmunt S, Dilme J, Barros A, Escudero JR. Compression
assisted by removable coils as a new treatment for iatrogenic
femoral pseudoaneurysms. J Vasc Surg. 2010 Sept 1 [Epub ahead
of print]
39. Kendrick AS, Sprouse LR. Repair of a combined femoral pseudoaneurysm and arteriovenous fistula using a covered stent graft.
Am Surg. 2007;73(3):227-229.
40. Waigand J, Ulich F, Gross CM, Thalhammer C, Dietz R. Percutaneous treatment of pseudoaneurysms and arteriovenous fistulas
after invasive vascular procedures. Catheter Cardiovasc Interv.
1999;47(2):165-166.
41. Onal B, Kosar S, Gumus T, Ilgit ET, Akpek S. Post catheterization femoral arteriovenous fistulas: endovascular treatment with
stent-grafts. Cardiovasc Intervent Radiol. 2004;27(5):453-458.
42. Thalhammer C, Kirchherr AS, Uhlich F, Waigand J, Gross CM.
Post catheterization pseudoaneurysms and arteriovenous fistulas: repair with percutaneous implantation of endovascular covered stents. Radiology. 2000;214(1):127-131.
43. Life Science Intelligence, Market Report. http://www.lifescienceintelligence.com/market-reports-page.php?id=LSI-WW081VA.
44. Schwarz T, Rastan A, Pochert V, et al. Mechanical compression
versus haemostatic wound dressing after femoral artery sheath
removal: a prospective, randomized study. Vasa. 2009;38(1):
53-59.
45. Koreny M, Riedmuller E, Nikfardjam M, Siostrzonek P, Mullner
M. Arterial puncture closing devices compared with standard
manual compression after cardiac catheterization. JAMA. 2004;
291(3):350-357.
46. Nikolsky E, Mehran R, Halkin A, et al. Vascular complications
associated with arteriotomy closure devices in patients undergoing percutaneous coronary procedures. J Am Coll Cardiol. 2004;
44:1200-1209.
47. Biancari F, DAndrea V, Di Marco C, et al. Meta-analysis of randomized trials on the efficacy of vascular closure devices after diagnostic angiography and angioplasty. Am Heart J. 2010;159:518-531.
48. McTaggart RA, Raghavan D, Haas RA, Jayaraman MV. StarClose vascular closure device: safety and efficacy of deployment

5/22/2012 5:58:16 PM

Arterial Pseudoaneurysms and Arteriovenous Fistulae

49.

50.

51.
52.

PMPH_CH85.indd 683

and reaccess in a neurointerventional radiology service. AJNR


Am J Neuroradiol. 2010;31(6):1148-1150.
Branzan D, Sixt S, Rastan A, et al. Safety and efficacy of the StarClose
vascular closure system using 7-F and 8-F sheath sizes: a consecutive single-center analysis. J Endovasc Ther. 2009;16(4):475-482.
Hermiller JB, Simonton C, Hinohara T, et al. The StarClose
Vascular Closure System: interventional results from the CLIP
study. Catheter Cardiovasc Interv. 2006;68(5):677-683.
Johnson JR, Ledgerwood AM, Lucas CE. Mycotic aneurysm:
new concepts in therapy. Arch Surg. 1983;118(5):577-582.
Hu ZJ, Wang SM, Li XX, Li SQ, Huang XL. Tolerable hemodynamic changes after femoral artery ligation for the treatment of

683

infected femoral artery pseudoaneurysms. Ann Vasc Surg. 2010;


24(2):212-218.
53. Peirce C, Coffey JC, OGrady H, Aly S, OMalley K, ODonohoe
M. The management of mycotic femoral pseudoaneurysm in
intravenous drug abusers. Ann Vasc Surg. 2009;23(6):824.
54. Brown KE, Heyer K, Rodriguez H, et al. Arterial reconstruction
with cryopreserved human allografts in the setting of infection:
a single-center experience with midterm follow-up. J Vasc Surg.
2009;49(3):660-666.
55. Klonaris C, Katsargyris A, Matthaiou A, et al. Emergent stenting of ruptured infected anastomotic femoral pseudoaneurysm.
Cardiovasc Intervent Radiol. 2007;30(6):1238-1241.

5/22/2012 5:58:16 PM

CHAPTER 86

Carotid Occlusive and


Aneurysmal Disease
Matthew J. Sideman and Lori L. Pounds

INTRODUCTION

3000 symptomatic patients. They were randomized into a medical arm (aspirin) and a surgical arm (carotid endarterectomy)
with plans to follow them for a minimum of 5 years. Patients
were subdivided into 30% to 69% stenosis and 70% to 99% stenosis. A total of 659 patients were enrolled between January 1, 1988
and February 21, 1991. The trial was stopped early by the oversight committee due to a clear benefit to the patients randomized
into the surgery arm.
The results of the NASCET trial are as follows. There was a
26% risk of stroke or death over a 2-year period for patients randomized into the medical arm. The risk of stroke was 9% over
the same 2-year period for patients in the surgical arm. This
translated into a 17% absolute risk reduction and a 65% relative
risk reduction for endarterectomy. Post hoc analysis showed that
the surgical benefit remains down to 50% stenosis, but there was
no benefit for less than 50% stenosis.2 The benefit from carotid
endarterectomy (CEA) was also shown to be greatest with the
most severe symptoms.2 In other words, the benefit of CEA is
greatest in a patient whose symptom was a previous stroke than a
patient whose symptom was TIA that was greater than a patient
whose symptom was amaurosis fugax. Benefits of endarterectomy
were also found to be greater for men than women, although this
may be a result of the greater number of men compared to women
who were enrolled in the study.2
The European Carotid Surgery Trial (ECST)3 was similarly
designed to answer the question of optimal treatment for symptomatic carotid occlusive disease. It enrolled 3024 patients between
1981 and 1994 who had experienced symptoms within the previous 6 months. There were unequal groups with 1811 patients allocated to the surgical arm and 1213 patients in the control group.
Analysis of the data showed a 26.5% risk of major stroke or death
in the control group versus a 14.9% risk in the surgical group at
3 years for an absolute benefit from surgery of 11.6%. The investigators recommended surgery for symptomatic lesions greater
than about 80%.
Several differences should be highlighted between NASCET
and ECST. The largest difference was the method for determining the percentage of stenosis. NASCET used the normal distal

Stroke is the third leading killer in the United States. Strokes cost
over $29 billion annually in disability and lost productivity. Eightyfive percent of strokes are ischemic and 15% are hemorrhagic.
Ischemic strokes are caused by carotid artery stenosis, cardiac
thromboembolism, hypertension, and diabetes but the majority
of ischemic strokes are due to carotid artery occlusive disease. The
pathophysiology of ischemic strokes from carotid artery occlusive
disease is distal emboli. The emboli can be atheroemboli, platelet
aggregates, or thrombus. Symptoms of carotid occlusive disease
include amaurosis fugax, transient ischemic attacks (TIA), and
strokes. Amaurosis fugax is a transient monocular blindness that
is often described as a shade coming down over eye that then
resolves. TIAs are motor or sensory deficits, aphasia, or dysarthria
that lasts less than 24 hours by definition. A stroke is any neurologic deficit that lasts for greater than 24 hours.
Risk factors for carotid occlusive disease include smoking,
hypertension, hyperlipidemia, age, gender, genetic predisposition,
and diabetes. There are usually no specific physical findings for
carotid occlusive disease although a carotid bruit may be heard.
Bruits must be differentiated from transmitted cardiac murmurs.
A complete neurologic examination should be part of the physical
examination to detect any deficits. If a carotid bruit is present, it
is only 50% predictive of a surgically significant stenosis. In addition, critical stenoses can have such low flow that the bruits often
disappear.

STROKE RISKS
1. What is the stroke risk for symptomatic carotid stenosis?
Differences in opinion about the optimal treatment of carotid
occlusive disease led to the North American Symptomatic Carotid
Endarterectomy Trial (NASCET).1
It was a prospective, randomized trial conducted at 50 centers across the United States and Canada that planned to enroll
684

PMPH_CH86.indd 684

5/22/2012 5:58:48 PM

Carotid Occlusive and Aneurysmal Disease

internal carotid artery diameter as the denominator when calculating percent stenosis. ECST used the estimated diameter of the
internal carotid at the level of the disease. This difference resulted in
approximately 350 patients being included in the severe stenosis group that would have been classified as less than 70% in the
NASCET study. There was also a longer lead in period for symptoms in ECST compared to NASCET (6 months vs. 120 days) and
more patients in the surgical arm compared to the medical arm
raising questions of study design. Despite these differences, ECST,
like NASCET, showed benefit for surgical intervention compared
to medical management for severe symptomatic carotid occlusive
disease.
Answer: The risk of stroke or death for a symptomatic carotid
lesion of 70% stenosis or greater is 26% over a 2-year period
with medical management alone compared to 9% for surgical
management.
2. What is the stroke risk for asymptomatic carotid stenosis?
Just as NASCET was designed to answer the question of the
optimal treatment for symptomatic carotid artery occlusive
disease, and the enrolling and completing its data acquisition,
the Asymptomatic Carotid Artery Study (ACAS)4 was designed
to answer the question What about asymptomatic stenosis?.
It was a prospective, randomized trial conducted at 39 centers
across the United States and Canada that planned to enroll 1500
patients to daily aspirin or carotid endarterectomy. The initial
endpoint of the study was TIA but was later amended to include
stroke and death. Between December 1987 and December 1993,
1662 patients were enrolled. The inclusion criteria were asymptomatic carotid lesions greater than or equal to 60% stenosis. The
study was completed in 1994 after the planned enrollment had
been reached. The results of ACAS showed an 11% risk of stroke
or death over the 5-year study period for the patients randomized
into the medical arm of the study. The risk of stroke or death in
the same 5-year period was 5% for patients randomized into the
surgical arm. Th is equates to a 5.9% absolute reduction, and a
53% aggregate risk reduction.
There were other lessons learned from ACAS as well. There
was a 1.2% stroke rate from the diagnostic arteriogram performed
as part of the study. The perioperative stroke rate in the trial was
1.5%. There is a transient increase in the risk of stroke by operating on an asymptomatic patient. For a patient to enjoy the preventative benefit from surgery, they need to live 3 years afterwards.
Another asymptomatic carotid trial was undertaken in
Europe. The Asymptomatic Carotid Surgery Trial (ACST)5
enrolled 3120 asymptomatic patients between 1993 and 2003.
They were randomized into an immediate carotid endarterectomy
arm and an indefinite deferral arm and followed for up to 5 years.
The 30-day risk of stroke or death in the after CEA was 3.1%. The
5-year stroke risks were 6.4% for the immediate CEA group and
11.8% for the deferred group. The investigators recommended
immediate CEA for asymptomatic patients younger than 75 years
with 70% stenosis by ultrasound. The findings in this study support those of ACAS.
The significance of symptoms from a carotid lesion was
clearly illustrated by the results of NASCET and ACAS. Both
studies looked at patients with 70% carotid stenosis; however, the
stroke risks were significantly different between the two studies.
The stroke risk in NASCET is 26% over a 2-year period compared
with an 11% risk over 5 years in ACAS. Annualized, the risk of

PMPH_CH86.indd 685

685

stroke is 13% versus 2.2% for symptomatic versus asymptomatic


patients. This clearly demonstrates that the symptomatic plaques
are more unstable and much more dangerous or virulent. These
facts all need to be taken into consideration when recommending medical management or intervention for patients with carotid
occlusive disease.
Answer: The risk of stroke for an asymptomatic 70% stenosis
of the carotid artery is 11% over a 5-year period for medical management compared with a 5% risk in the same 5-year period for
patients undergoing surgical management.

DIAGNOSIS
3. What is the optimal diagnostic test for carotid occlusive
disease?
There are four modalities available for imaging the extracranial
carotid arteries. They are duplex ultrasonography, computed
tomographic angiography (CTA), magnetic resonance angiography (MRA), and conventional digital angiography. Each modality is associated with its own advantages and limitations. The
gold standard continues to be conventional digital angiography;
however, this is the most invasive method and carries the largest
risk of complications including the risk of stroke as was shown
in ACAS.4 Despite these risks, conventional digital angiography
is performed when there is discordance between less invasive
modalities or as part of the treatment of the patient when endovascular intervention is planned.
CTA provides direct imaging of the extracranial carotid
arteries and can measure the severity of stenosis.6 Calcification,
metallic artifacts, and patient compliance can all limit the quality
of the imaging and thus the accuracy of the exam. In addition, the
need for intravenous contrast will limit its applicability to patient
with borderline renal function. Magnetic resonance imaging
similarly images the carotid arteries and can estimate the severity
of stenosis.6 MRA does not expose patients to radiation like CTA
and conventional digital angiography. Calcium is not a limitation
either but patient size and compliance are. MRA tends to overestimate the degree of stenosis and it has difficulty discriminating
between near occlusions and complete occlusions. Again, patients
with borderline renal function are not candidates due to the risk
of nephrogenic systemic fibrosis from the gadolinium contrast
agents.7
Duplex ultrasonography is inexpensive, noninvasive, and
reproducible. It uses measurement of blood flow velocities to calculate categories of stenosis. Its accuracy was developed through
extensive comparison research looking at duplex exams, angiographic findings, and operative specimens.8,9 Based on this, duplex
criteria were established. The most commonly used criteria are
those reported by Strandness.10 In this scheme, carotid arteries are
classified as either normal, 1% to 15% stenosis, 16% to 49% stenosis, 50% to 79% stenosis, 80% to 99% stenosis, or occluded. The
major determining factors are the peak systolic velocity (PSV) in
the internal carotid artery and the end diastolic velocity (EDV). A
PSV > 125 cm/s qualifies for a 50% to 79% stenosis, and an EDV >
140 cm/s qualifies for a stenosis of 80% to 99%. While proven and
reliable, the Strandness criteria do not fit well with the indications
for intervention as outlined in the NASCET and ACAS trials. To
address these concerns, other authors have looked for more specific criteria to define a 70% stenosis more in line with the trials.11

5/22/2012 5:58:49 PM

686

Surgery: Evidence-Based Practice

Internal validation of any vascular lab is crucial to verify accuracy


of its own test results and criteria.
Answer: Duplex ultrasonography is the optimal test for diagnosis carotid occlusive disease based on its cost-effectiveness, and
it is the least invasive modality; however, there is no strong evidence to support one modality over another and the best resources
available in a given setting should be used.

TREATMENT
4. What are the indications for intervention of the carotid
artery?
The fi rst decision point on whether or not to intervene on a
patient with carotid occlusive disease centers around the presence
or absence of symptoms. There is a significant difference in the
stroke risks between these two groups as outlined above. Utilizing
the results from NASCET and ECST, symptomatic patients with
50% or greater carotid stenosis should undergo treatment. There
is no benefit for symptomatic patients with less than 50% stenosis.
According to ACAS and ACST, asymptomatic patients with 60%
or greater stenosis benefit from surgical intervention, although the
benefit was small for lesser stenosis and may no longer exist with
todays maximal medical management compared to the medical
management given in those trials. More recently, multispecialty
consensus guidelines have been published by the American Heart
Association.12 This group examined all available evidence and
recommended the following indications with regards to carotid
endarterectomy:

Patients at average or low surgical risk who experience


nondisabling ischemic stroke or transient cerebral ischemic
symptoms, including hemispheric events or amaurosis fugax,
within 6 months (symptomatic patients) should undergo
CEA if the diameter of the lumen of the ipsilateral internal
carotid artery is reduced more than 70% as documented by
noninvasive imaging (Level of evidence: 1) or more than 50%
as documented by catheter angiography (Level of evidence: 2)
and the anticipated rate of perioperative stroke or mortality is
less than 6%.12
It is reasonable to perform CEA in asymptomatic patients who
have more than 70% stenosis of the internal carotid artery if the
risk of perioperative stroke, myocardial infarction (MI), and
death is low (Level of evidence: 1).12
Prophylactic carotid artery stenting (CAS) might be considered
in highly selected patients with asymptomatic carotid stenosis
(minimum 60% by angiography, 70% by validated Doppler
ultrasound), but its effectiveness compared with medical
therapy alone in this situation is not well established (Level of
evidence: 2).12

Answer: Symptomatic patients with >50% carotid stenosis and


asymptomatic patients with greater than 60% stenosis.
5. What is the best medical treatment for carotid occlusive
disease?
The medical management for carotid occlusive disease has
advanced since the days of NASCET and ACAS. At the time of those
trials, medical management consisted of varying doses of daily
aspirin. Since that time, recognition of carotid artery occlusive

PMPH_CH86.indd 686

disease as a part of atherosclerosis overall has led to more aggressive risk factor modification and medical management. Hypertensive patients should be treated to maintain their blood pressure
below 140/90 mm Hg.13 Patients who smoke cigarettes should be
advised to quit.14 Patients with hyperlipidemia should be treated
with statins to lower their low-density lipoprotein (LDL) cholesterol to below 100 mg/dL at a minimum.15 Some authors suggest
lowering the LDL to below 70 mg/dL.16 Diabetics should have their
blood sugars controlled with a goal of a glycosylated hemoglobin
A1c level less than 7.0%.17
There have been multiple studies looking into the type of antiplatelet agents used to treat carotid artery occlusive disease with
varied results. Aspirin, 75 to 325 mg daily, is recommended for the
treatment of carotid occlusive disease and to prevent myocardial
infarctions.15 In patients who have suffered symptoms from their
carotid disease, aspirin or clopidogrel (75 mg daily)18 or a combination of aspirin and dipyridamole (25 and 200 mg twice daily)19
is recommended. The use of aspirin and clopidogrel as dual agents
is not recommended20 nor is the use of warfarin.21 Of course, if the
patient has atrial fibrillation or a mechanical heart valve necessitating treatment with warfarin, this medical need outweighs the
recommendations for carotid artery occlusive disease.
Answer: Carotid artery occlusive disease should be treated
medically with antiplatelet therapy consisting of either aspirin
(75325 mg daily), clopidogrel (75 mg daily), or a combination of
aspirin and dipyridamole (25 and 200 mg twice daily). Aggressive risk factor modification should also be employed including
the use of statins, blood pressure control, smoking cessation, and
diabetic control.
6. What are the best surgical techniques for carotid endarterectomy?
The first report of a successful carotid operation for a symptomatic
patient was by Eastcott22 in 1954. They performed an end to end
anastomosis of the common carotid artery to the distal internal
carotid artery in a patient who was having TIAs. The first carotid
endarterectomy was likely performed by Dr DeBakey23 in 1953.
Since its introduction, the surgical technique has been modified and refined. Its utilization has also changed over time with
decreasing volume during the 1980s followed by an increase after
publication of the randomized trials in the 1990s. Despite its long
history of success, there are multiple variables in how a carotid
endarterectomy is performed.
One of the first variations in technique begins with the anesthetic choice. The majority of surgeons perform CEA under general anesthesia but some choose local anesthesia and/or a cervical
block. The benefits of general anesthesia are decreased cerebral
metabolism, a controlled airway, and a quiet operating theater. The
drawback is the inability to directly monitor the neurologic function of the patient. Those that prefer regional anesthesia propose
that it is the safest modality for monitoring intraoperative neurologic status of the patient; however, one recent, large randomized
trial showed no difference in outcomes between the two groups.24
The best anesthetic option is therefore the one with which the
operating surgeon and anesthesiologist are most adept.
The surgical incision can be performed in a longitudinal
direction parallel to the sternocleidomastoid muscle or transversely in the mid-portion of the neck. The longitudinal incision
provides superior exposure but can lead to excessive scarring
and contractures. The horizontal incision may limit exposure to

5/22/2012 5:58:49 PM

Carotid Occlusive and Aneurysmal Disease

the distal internal carotid artery and/or the proximal common


carotid artery. It does provide for improved wound healing and
a more cosmetic scar. There is little evidence to support one incision over the other25 and the best option is likely again surgeon
familiarity and preference.
The standard surgical technique involves exposure and control of the common carotid, internal carotid, and external carotid
arteries followed by clamping and longitudinal arteriotomy. At
this point a vascular shunt can be used to maintain antegrade
blood flow around the operative site, or the surgery can be done
without a shunt, relying on collateral blood flow from the other
carotid and vertebrals. Routine shunters argue that is the safest
way to maintain blood flow and prevent ischemic events. Selective
shunters use either back pressure in the internal carotid artery
(stump pressure) and/or neurologic monitoring to decide when
to use a shunt.26 Neurologic monitoring is done with continuous
intraoperative electroencephalogram (EEG) with or without the
addition of somatosensory evoked potentials. The arguments for
selective shunting are that only 10% to 15% of patients will require
a shunt, it avoids potential embolization or intimal injury during
the placement of a shunt, and that it makes the operation technically easier if the shunt is not used. Despite strong beliefs of superiority held by surgeons supportive of both methods, there is little
evidence to support either approach.27-29 Like the choice of anesthesia and incision, the choice of routine versus selective shunting
is best left to surgeon familiarity and comfort.
There is good evidence to support three techniques for the
performance of carotid endarterectomy. Kresowik et al.,30 demonstrated in their review of surgical outcomes for CEA in 10 states
that preoperative use of antiplatelet agents, intraoperative use of
heparin, and patch angioplasty closure were associated with lower
stroke and death rates. Furthermore, they showed that quality
improvement initiatives and adoption of preoperative antiplatelet
agents and routine patch angioplasty resulted in improved results
for CEA in all states studied.31
Answer: The preoperative use of antiplatelet agents and
routine patch angioplasty closure for CEA are associated with
improved stroke and death rates. Choice of anesthetic, incision,
and routine versus selective use of shunts do not have sufficient
evidence to be supported one way or another. These options are
best left to the comfort of the operating surgeon.
7. When should you do carotid stenting versus carotid endarterectomy?
Carotid artery angioplasty and stenting is the newest treatment
modality for carotid occlusive disease. There has been much
debate and research over the past 10 years comparing CAS
with CEA. Most of the data has been industry driven and registry data lending very little evidence on which to base treatment
decisions.32-36 The earliest trial comparing the two modalities was
the Carotid and Vertebral Artery Transluminal Angioplasty Study
(CAVATAS).37 It was a multicenter trial in Europe, Australia, and
Canada that randomized 504 patients from March 1, 1992 to July
31, 1997 into either angioplasty/stent or CEA arms. The inclusion
criteria were rather lax and described as a stenosis that a physician thought needed treatment. The results were a 10.0% stroke
and/or death rate for the angioplasty/stent arm and a 9.9% stroke/
death rate for CEA. They concluded that there were similar major
risks and effectiveness with decreased complications compared to
surgery. There were multiple critiques of CAVATAS, mainly the

PMPH_CH86.indd 687

687

exceedingly high stroke and death rate in the surgical arm of the
trial but carotid stenting was gaining ground.
The breakthrough study for CAS was Stenting and Angioplasty
with Protection in Patients at High Risk for Endarterectomy study
(SAPPHIRE).38 SAPPHIRE randomized high-risk patients into
surgical or stent arms. Medical and/or anatomic criteria were used
to define high risk. The medical criteria included congestive heart
failure (CHF), recent MI, unstable angina, coronary revascularization, chronic obstructive pulmonary disease (COPD), chronic
renal insufficiency (CRI), or age greater than 80 years. Anatomic
criteria included prior radical neck dissection, neck radiation
therapy, recurrent stenosis, high carotid lesions, or lesions below
the clavicle. A total of 307 high-risk patients were randomized to
stent or surgery at 29 institutions. Patients turned down for surgery
were entered into the stent registry. One stent and distal embolic
protection device combination was used for the study. The traditional study endpoints of stroke and death were expanded to
include cardiac events. The 30-day results showed a 5.8% death/
stroke/MI rate in the stent arm and a 12.6% death/stroke/MI rate
in the CEA arm (P = .05). When subdivided into medical and anatomic comorbidities, the results were 2.8% versus 15.5% (CAS vs.
CEA) (P = .02) for the medical high-risk patients and 5.5% versus 10.0% (CAS vs. CEA) (P = .47) for the anatomic high-risk
patients. The investigators concluded that CAS was not inferior to
endarterectomy in high-risk patients. The results can be criticized
by the expanded endpoints, the definitions of high risk, and the
large number of registry data; however, the stroke rate in surgical
arm cannot be denied. Based on SAPPHIRE data, the Food and
Drug Administration and the Centers for Medicare and Medicaid
Services (CMS) approved CAS for high-risk patients.
SAPPHIRE was followed by larger randomized controlled trials
without registry data. They include two studies performed in Europe
and one in the United States. The European trials were Endarterectomy versus Stenting in Patients with Symptomatic Severe Carotid
Stenosis (EVA 3S)39 and Stent-Supported Percutaneous Angioplasty
of the Carotid Artery versus Endarterectomy (SPACE).40 EVA 3S was
a multicenter, randomized, noninferiority trial looking at symptomatic carotid lesions of at least 60% stenosis. Multidisciplinary teams
including a neurologist, a vascular surgeon (who had performed
at least 25 CEAs), and an interventionalist (who had performed at
least 12 CASs) were used to treat the patients. They enrolled 527
patients between November 2000 and September 2005. The study
was stopped early by the safety committee. The results showed a
30-day stroke/death rate of 3.9% versus 9.6% (CEA vs. CAS) and
a 6-month stroke/death rate of 6.1% versus 11.7% (CEA vs. CAS).
They concluded that CEA was superior. SPACE was a multinational,
multicenter, randomized trial looking at symptomatic carotid lesions
of at least 60% stenosis. Multidisciplinary teams including a neurologist, a vascular surgeon (who had performed at least 25 CEAs), and
an interventionalist (who had performed at least 25 CASs) were used
to treat the patients. SPACE enrolled 1183 patients between March
2001 and February 2006. The study was stopped due to futility and
lack of funds. The results showed a 30-day stroke/death of 5.64% versus 6.95% (CEA vs. CAS).
The long-awaited results of the American study were published in the Carotid Revascularization Endarterectomy versus
Stent Trial (CREST).41 CREST randomized 2502 symptomatic and
asymptomatic patients to CAS or CEA over a 10-year period. Primary endpoints were 30-day stroke, death, or MI and ipsilateral
stroke within 4 years postprocedure. The results of overall 30-day

5/22/2012 5:58:49 PM

688

Surgery: Evidence-Based Practice

endpoints were not significantly different between the two groups


with 5.2% for CAS and 4.5% for CEA. The 30-day stroke/death rate
was 4.1% versus 2.3% (CAS vs. CEA) and the 30-day MI rate was
1.1% versus 2.3% (CAS vs. CEA). There were no statistical differences between symptomatic and asymptomatic patients. The investigators concluded that CAS and CEA were equivalent treatments.
The International Carotid Stenting Study (ICSS) is another ongoing
randomized trial in Europe looking into the relative risks of CAS
versus CEA. The enrollment criteria are symptomatic patients with
>50% stenosis. An interim report of 1713 symptomatic patients
randomized to CAS or CEA was published in the Lancet.42
The results showed a 30-day stroke, death, and MI rate of
8.5% for CAS and 5.2% for CEA. They concluded that long-term
follow-up was needed to determine the appropriate role for CAS
but that CEA remained the treatment of choice.
In summary, there have been many studies over the past
decade dedicated to comparing CAS and CEA. Some have favored
CAS whereas others have favored CEA. Critiques of all of the
studies abound. Taken as a whole, the body of evidence seems to
support the current recommendations of CMS. That is to say that
CEA is the treatment of choice for carotid occlusive disease and
CAS is indicated for high-risk, symptomatic patients.
Answer: CAS is indicated for the treatment of high-risk
patients with symptomatic carotid occlusive disease.

CAROTID ANEURYSMAL DISEASE


Aneurysms of the extracranial carotid artery are exceedingly
rare. Intracranial carotid artery aneurysms are more common
but fall under the scope of the neurosurgeon and the neurointerventional radiologist. Extracranial carotid aneurysms include
both true aneurysms and false aneurysms (pseudoaneurysms).
Etiologies include atherosclerosis, fibromuscular dysplasia,
trauma (penetrating and nonpenetrating), dissection, postsurgical, and infection. Syphilis was once the most common cause
of carotid aneurysms but is seldom seen today. It is generally
felt that extracranial carotid aneurysms should be treated when
found to prevent the occurrence of neurologic events from embolism. Surgical treatment involves resection of the aneurysm and
bypass to restore cerebral blood flow.43 Th is approach is good for
aneurysms of the common carotid artery or the carotid bifurcation but may be impossible for lesions extending to the skull
base. Endovascular treatment with covered stents 44 or embolization45 is likely the preferred method for distal lesions where vascular control would be problematic or in reoperative fields where
dissection would carry unacceptable risks. Given the rarity of
extracranial aneurysms, there is very little literature to delineate the natural history of the disease or the best management
strategies.

Clinical Question Summary


Question

Answer

1 What is the stroke


risk for symptomatic
carotid stenosis?

Medical management 26% over 2 years


Surgical management 9% over 2 years

A
A

1-3

2 What is the stroke


risk for asymptomatic
carotid stenosis?

Medical management 11% over 5 years


Surgical management 5.1% over 5 years

A
A

4-5

3 What is the optimal


diagnostic test for
carotid stenosis?

Duplex ultrasonography

4 What are the


indications for
intervention of the
carotid artery?

Symptomatic >70%
Symptomatic >50%
Asymptomatic >70%
Asymptomatic 60%

A
B
A
B

1-5

5 What is the best


medical treatment
for carotid occlusive
disease?

Antiplatelet therapy: aspirin (75 to 325 mg daily),


clopidogrel (75 mg daily) or combination aspirin and
dipyridamole (25 and 200 mg twice daily)
Statin use
Risk factor modification

12-21

6 What are the best


surgical techniques
for carotid
endarterectomy?

Preoperative use of antiplatelet agents and routine patch


angioplasty closure for CEA

30-31

7 When should you


do carotid stenting
versus carotid
endarterectomy?

CAS is indicated for the treatment of high-risk patients


with symptomatic carotid occlusive disease

38

PMPH_CH86.indd 688

Grade of
Recommendation

References

5/22/2012 5:58:49 PM

Carotid Occlusive and Aneurysmal Disease

REFERENCES
1. North American Symptomatic Carotid Endarterectomy Trial Collaborators. Beneficial effect of carotid endarterectomy in symptomatic patients with high-grade carotid stenosis. NEJM. 1991;
325(7):445-453.
2. Barnett HJ, Taylor DW, Eliasziw M, et al. Benefit of carotid endarterectomy in patients with symptomatic moderate or severe stenosis. North American Symptomatic Carotid Endarterectomy Trial
Collaborators. N Engl J Med. 1998;339(20):1415-1425.
3. Randomised trial of endarterectomy for recently symptomatic
carotid stenosis: final results of the MRC European Carotid Surgery Trial (ECST). Lancet. 1998;351:1379-1387.
4. Executive Committee for the Asymptomatic Carotid Atherosclerosis Study. Endarterectomy for asymptomatic carotid artery
stenosis. JAMA. 1995;273:1421-1428.
5. Halliday A, Mansfield A, Marro J, et al. MRC Asymptomatic
Carotid Surgery Trial (ACST) Collaborative Group. Prevention of
disabling and fatal strokes by successful carotid endarterectomy
in patients without recent neurological symptoms: randomised
controlled trial. Lancet. 2004;363(9420):1491-1502.
6. Long A, Lepoutre A, Corbillon E, et al. Critical review of non- or
minimally invasive methods (duplex ultrasonography, MR- and
CT-angiography) for evaluating stenosis of the proximal internal
carotid artery. Eur J Vasc Endovasc Surg. 2002;24:43-52.
7. Cowper SE, Kuo PH, Bucala R. Nephrogenic systemic fibrosis
and gadolinium exposure: association and lessons for idiopathic
fibrosing disorders. Arthritis Rheum. 2007;56:3173-3175.
8. Blackshear WM Jr, Phillips DJ, Thiele BL, et al. Detection of carotid
occlusive disease by ultrasonic imaging and pulsed Doppler spectrum analysis. Surgery. 1979;86(5):698-706.
9. Fell G, Phillips DJ, Chikos PM, Harley JD, Thiele BL, Strandness
DE. Ultrasonic duplex scanning for disease of the carotid artery.
Circulation. 1981;64:1191-1195.
10. Strandness DE Jr. Extracranial arterial disease. In: Strandness
DE Jr. ed., Duplex Scanning in Vascular Disorders. 2nd ed. New
York: Raven; 1993:113-158.
11. Moneta GL, Edwards JM, Chitwood RW, et al. Correlation of North
American Symptomatic Carotid Endarterectomy Trial (NASCET)
angiographic definition of 70% to 99% internal carotid artery
stenosis with duplex scanning. J Vasc Surg. 1993;17(1):152-157.
12. Brott TG, Halperin JL, Abbara S, et al. 2011 ASA/ACCF/AHA/
AANN/AANS/ACR/ASNR/CNS/SAIP/SCAI/SIR/SNIS/SVM/
SVS Guideline on the Management of Patients with Extracranial
Carotid and Vertebral Artery Disease: Executive Summary A
Report of the American College of Cardiology Foundation/American Heart Association Task Force on Practice Guidelines, and the
American Stroke Association, American Association of Neuroscience Nurses, American Association of Neurological Surgeons,
American College of Radiology, American Society of Neuroradiology, Congress of Neurological Surgeons, Society of Atherosclerosis
Imaging and Prevention, Society for Cardiovascular Angiography
and Interventions, Society of Interventional Radiology, Society of
NeuroInterventional Surgery, Society for Vascular Medicine, and
Society for Vascular Surgery Developed in Collaboration With the
American Academy of Neurology and Society of Cardiovascular
Computed Tomography. J Am Coll Cardiol. 2011;57(8):1002-1044.
13. Rashid P, Leonardi-Bee J, Bath P. Blood pressure reduction and
secondary prevention of stroke and other vascular events: a systematic review. Stroke. 2003;34(11):2741-2748.
14. Wolf PA, DAgostino RB, Kannel WB, Bonita R, Belanger AJ.
Cigarette smoking as a risk factor for stroke. The Framingham
Study. JAMA. 1988;259(7):1025-1029.

PMPH_CH86.indd 689

689

15. Adams RJ, Albers G, Alberts MJ, et al.; American Heart Association; American Stroke Association. Update to the AHA/
ASA recommendations for the prevention of stroke in patients
with stroke and transient ischemic attack. Stroke. 2008;39(5):
1647-1652.
16. Amarenco P, Bogousslavsky J, Callahan A 3rd, et al.; Stroke Prevention by Aggressive Reduction in Cholesterol Levels (SPARCL)
Investigators. High-dose atorvastatin after stroke or transient
ischemic attack. N Engl J Med. 2006;355(6):549-559.
17. ADVANCE Collaborative Group, Patel A, MacMahon S, et al.
Intensive blood glucose control and vascular outcomes in patients
with type 2 diabetes. N Engl J Med. 2008;358(24):2560-2572.
18. CAPRIE Steering Committee. A randomised, blinded, trial of
clopidogrel versus aspirin in patients at risk of ischaemic events
(CAPRIE). Lancet. 1996;348(9038):1329-1339.
19. Diener HC, Cunha L, Forbes C, Sivenius J, Smets P, Lowenthal A.
European Stroke Prevention Study. 2. Dipyridamole and acetylsalicylic acid in the secondary prevention of stroke. J Neurol Sci.
1996;143(1-2):1-13.
20. Diener HC, Bogousslavsky J, Brass LM, et al.; MATCH Investigators. Aspirin and clopidogrel compared with clopidogrel alone
after recent ischaemic stroke or transient ischaemic attack in highrisk patients (MATCH): randomised, double-blind, placebocontrolled trial. Lancet. 2004;364(9431):331-337.
21. Mohr JP, Thompson JL, Lazar RM, et al.; Warfarin-Aspirin Recurrent Stroke Study Group. A comparison of warfarin and aspirin for
the prevention of recurrent ischemic stroke. N Engl J Med. 2001;
345(20):1444-1451.
22. Eastcott HHG, Pickering GW, Rob C. Reconstruction of internal
carotid artery in a patient with intermittent attacks of hemiplegia. Lancet. 1954;2:994-996.
23. DeBakey ME, Crawford ES, Cooley DA, et al. Surgical considerations of occlusive disease of innominate, carotid, subclavian,
and vertebral arteries. Ann Surg. 1959;149:690-710.
24. GALA Trial Collaborative Group, Lewis SC, Warlow CP, et al.
General anaesthesia versus local anaesthesia for carotid surgery
(GALA): a multicentre, randomised controlled trial. Lancet.
2008;372(9656):2132-2142.
25. Skillman JJ, Kent KC, Anninos E. Do neck incisions influence
nerve deficits after carotid endarterectomy? Arch Surg. 1994;
129(7):748-752.
26. Hans SS, Jareunpoon O. Prospective evaluation of electroencephalography, carotid artery stump pressure, and neurologic changes
during 314 consecutive carotid endarterectomies performed in
awake patients. J Vasc Surg. 2007;45(3):511-515.
27. Aburahma AF, Stone PA, Hass SM, et al. Prospective randomized
trial of routine versus selective shunting in carotid endarterectomy based on stump pressure. J Vasc Surg. 2010;51(5):1133-1138.
Epub 2010 Mar 29.
28. Woodworth GF, McGirt MJ, Than KD, Huang J, Perler BA,
Tamargo RJ. Selective versus routine intraoperative shunting
during carotid endarterectomy: a multivariate outcome analysis.
Neurosurgery. 2007;61(6):1170-1176; discussion 1176-1177.
29. Rerkasem K, Rothwell PM. Routine or selective carotid artery
shunting for carotid endarterectomy (and different methods of
monitoring in selective shunting). Cochrane Database Syst Rev.
2009;(4):CD000190.
30. Kresowik TF, Bratzler D, Karp HR, et al. Multistate utilization,
processes, and outcomes of carotid endarterectomy. J Vasc Surg.
2001;33(2):227-234.
31. Kresowik TF, Bratzler DW, Kresowik RA, et al. Multistate
improvement in process and outcomes of carotid endarterectomy.
J Vasc Surg. 2004;39(2):372-380.

5/22/2012 5:58:49 PM

690

Surgery: Evidence-Based Practice

32. Gray WA, Hopkins LN, Yadav S, et al.; ARCHeR Trial Collaborators. Protected carotid stenting in high-surgical-risk patients:
the ARCHeR results. J Vasc Surg. 2006;44(2):258-268.
33. CARESS Steering Committee. Carotid revascularization using
endarterectomy or stenting systems (CARESS): phase I clinical
trial. J Endovasc Ther. 2003;10(6):1021-1030.
34. Fairman R, Gray WA, Scicli AP, et al.; for the CAPTURE Trial
Collaborators. The CAPTURE registry: analysis of strokes resulting from carotid artery stenting in the post approval setting: timing, location, severity, and type. Ann Surg. 2007;246(4):551-556;
discussion 556-558.
35. Katzen BT, Criado FJ, Ramee SR, et al.; CASES-PMS Investigators. Carotid artery stenting with emboli protection surveillance
study: thirty-day results of the CASES-PMS study. Catheter Cardiovasc Interv. 2007;70(2):316-323.
36. Iyer SS, White CJ, Hopkins LN, et al.; BEACH Investigators.
Carotid artery revascularization in high-surgical-risk patients
using the Carotid WALLSTENT and FilterWire EX/EZ: 1-year
outcomes in the BEACH Pivotal Group. J Am Coll Cardiol. 2008;
51(4):427-434.
37. Endovascular versus surgical treatment in patients with carotid
stenosis in the Carotid and Vertebral Artery Transluminal
Angioplasty Study (CAVATAS): a randomised trial. Lancet. 2001;
357(9270):1729-1737.
38. Yadav JS, Wholey MH, Kuntz RE, et al.; Stenting and Angioplasty
with Protection in Patients at High Risk for Endarterectomy Investigators. Protected carotid-artery stenting versus endarterectomy
in high-risk patients. N Engl J Med. 2004;351(15):1493-1501.

PMPH_CH86.indd 690

39. Mas JL, Chatellier G, Beyssen B, et al.; EVA-3S Investigators.


Endarterectomy versus stenting in patients with symptomatic
severe carotid stenosis. N Engl J Med. 2006;355(16):1660-1671.
40. SPACE Collaborative Group, Ringleb PA, Allenberg J, et al.
30 day results from the SPACE trial of stent-protected angioplasty versus carotid endarterectomy in symptomatic patients:
a randomised non-inferiority trial. Lancet. 2006;368(9543):
1239-1247.
41. Silver FL, Mackey A, Clark WM, et al.; for the CREST Investigators. Safety of Stenting and Endarterectomy by Symptomatic
Status in the Carotid Revascularization Endarterectomy Versus
Stenting Trial (CREST). Stroke. 2011;42(3):675-680. Epub 2011
Feb 9.
42. International Carotid Stenting Study investigators, Ederle J,
Dobson J, et al. Carotid artery stenting compared with endarterectomy in patients with symptomatic carotid stenosis (International Carotid Stenting Study): an interim analysis of a
randomised controlled trial. Lancet. 2010;375(9719):985-997.
43. Moreau P, Albat B, Thvenet A. Surgical treatment of extracranial internal carotid artery aneurysm. Ann Vasc Surg. 1994;8(5):
409-416.
44. Yi AC, Palmer E, Luh GY, Jacobson JP, Smith DC. Endovascular
treatment of carotid and vertebral pseudoaneurysms with covered stents. AJNR Am J Neuroradiol. 2008;29(5):983-987.
45. Cox MW, Whittaker DR, Martinez C, Fox CJ, Feuerstein IM,
Gillespie DL. Traumatic pseudoaneurysms of the head and
neck: early endovascular intervention. J Vasc Surg. 2007;46(6):
1227-1233.

5/22/2012 5:58:49 PM

CHAPTER 87

Aortoiliac Occlusive Disease


Jeffrey S. Horn and William D. Jordan, Jr.

segment was reported in 1979.4 Early results with angioplasty


were disappointing due to poor recanalization rate, high failure
rate, and higher than expected complication rates. The introduction of stents and better delivery platforms as well as increased
operator skill has made endovascular therapy the chosen treatment for a great majority of patients with AOID. Indeed, the
number of direct reconstructions has decreased in recent years as
endovascular means are increasingly used to treat even the most
complex forms of disease.5

INTRODUCTION
Aortoiliac occlusive disease (AOID) is a common entity in modern
vascular surgery practice as an obstructive disease of the terminal
aorta and iliac arteries. Patients present over many age groups
with the concentration in the 50 to 60 year olds. The patients
report varying levels of symptomatology from claudication to
critical limb ischemia depending on concomitant infrainguinal
disease. Successful treatments include both open and endovascular reconstruction. Best medical therapy may reduce overall risk
from systemic disease or halt the progression of the process, but
medical therapy does not reverse the presence of atherosclerosis
in this region. Controversy continues to exist over the best course
of management for patients with this disease.

EPIDEMIOLOGY
Peripheral arterial disease (PAD) is defined by an ankle-brachial
index (ABI) of 0.9. In epidemiologic samples, PAD affects over
8 million men and women over age 40.6 This disease worsens with
age, with a 12% to 20% incidence in those over 65.7 The ratio of
symptomatic to asymptomatic disease is independent of age and
on the order of 5% to 33%.8 PAD is caused by advancing atherosclerosis which has numerous risk factors including smoking,
hypertension, diabetes mellitus, dyslipidemia, age, race, gender,
and chronic renal disease.
There are several patterns of distribution of atherosclerosis
in patients with PAD. The most common description, based on
arteriographic study separates the periphery into (1) aortoiliac,
(2) femoropopliteal, and (3) tibioperoneal disease. Aortoiliac
inflow disease can be subdivided into (1) isolated aortoiliac
disease, (2) abdominal inflow disease extending into external
iliacs, and (3) multilevel disease with extensive atherosclerosis of infrainguinal vessels.9 Those with type I AOID tend to be
younger with fewer risk factors and present with claudication.
Severe symptoms are usually absent owing to the brisk collateralization around this segment. Symptoms on presentation may
include hip and buttock claudication, impotence in men, and
diminished femoral pulses (classically, the Leriche syndrome).
Women make up a good part of this group, especially those with
extensive smoking history and those with a small, hypoplastic
aorta. Isolated AOID is seen in 5% to 10% of patients undergoing

HISTORY
Pathology of the aortoiliac segment was fi rst addressed in the
medical community in the early 1900s. Leriche made a series of
observations on young patients with a specific constellation of
symptoms, namely claudication and absence of femoral pulses
on exposure.1 Subsequently, impotence and leg atrophy were
also noted in many of these patients. He reported his fi ndings
on a cohort of 20 patients in 1948 and went so far as to recommend arteriography and arterial resection for likely obliterative
disease of the aorta. Cid Dos Santos in 1946 developed techniques of thromboendarterectomy by accidentally entering the
wrong dissection plane while attempting femoral embolectomy.2 Once the plaque was excised he was met with brisk flow,
and later coined the term disobliteration. His pioneering techniques were then applied to the aortoiliac segment by Wylie in
the early 1950s.3 Direct replacement of the aortoiliac segment
was devised after the development of prosthetic vascular grafts
in the 1950s. Further refinements in technique over the ensuing decades have made direct reconstruction with aortofemoral grafts, the standard to which newer therapies are compared.
The first successful percutaneous treatment of pathology in this
691

PMPH_CH87.indd 691

5/22/2012 5:59:24 PM

692

Surgery: Evidence-Based Practice

intervention, whereas multilevel disease is present in 20% to 50%.


Patients suffering from multilevel disease tend to be older and
present with critical limb ischemia. Hansen studied the arteriograms of patients younger and older than 50 to see if there were
differences in distribution of atherosclerosis. Of the 59 patients
younger than 50, 42% had disease of the aortoiliac segment, and
there was isolated disease in 54%. In contrary, 140 patients older
than 50 had similar percentage aortoiliac disease but was isolated
in only 16%.10
The natural history of those with PAD indicates increased
risk of cardiac events, stroke, and death. Cardiovascular causes
account for almost 70% of mortality in this population. Several
investigations indicate that degree of ABI reduction corresponds
to risk for cardiovascular death. Patients with symptomatic PAD
have shortened life spans compared to age-matched controls.

DIAGNOSIS
Symptomatic patients with AOID may complain of either hip
and buttock or sometimes calf claudication depending on the
distribution of atherosclerosis as discussed. Examination may
reveal absent or diminished femoral pulses, as well as bruits or
thrills over the groins. These fi ndings may be present only after
brisk exercise on a treadmill. History of impotence should also
be ascertained, as up to 30% of males with AOID may have this
symptom on questioning. History and physical exam should be
followed by noninvasive studies. ABI may be mildly reduced
and pressure waveform reduction at the thigh level may indicate
stenosis or occlusion on that site. Exercise testing may unmask
disease in a patient with otherwise normal ABI measurements.
Duplex ultrasound can be used to determine levels of disease
and velocity across stenoses, but is rarely used alone in preoperative preparation. Computed tomography and magnetic
resonance give the most detailed noninvasive information.
Advances in technology such as multislice scanners and development of iso-osmotic contrast agents make these tests highly
accurate with minimal but not negligible risk to the patient.
Runoff studies can give high resolution to the tibial vessels for
operative planning. Arteriography remains the gold standard
for diagnosis and preoperative planning in AOID. Although
invasive, it shows volume flow in real time. Pressure measurements can also be taken across lesions to determine their physiologic significance.
1. Which lesions in aortoiliac occlusive disease are favorable
for endovascular repair?
Since the first iliac angioplasty and subsequent development of
stents, there have been many progressive reports regarding successful treatment of various lesions. Originally published in 2000,
the TASC (Trans-Atlantic Inter-Society Consensus Document on
Management of Peripheral Arterial Disease) document has been
updated in 2007.8 This guide gives recommendations on treatment
of various risk factors of PAD as well as treatment algorithms for
the lower extremities. Aortoiliac disease is broken down into four
subgroups based on severity of disease. These groupings are based
on successful treatments with either modality. Endovascular success is defined as technical ability to cross and treat the offending

PMPH_CH87.indd 692

lesion, as well as long-term patency after intervention. TASC A


encompasses short focal stenoses of the common or external iliac
artery. Total occlusions, longer stenoses, tandem, and bilateral
stenoses progress through TASC B and C to TASC D, the most
severe pattern of disease, which is best treated by open reconstruction. Although TASC-II recommendation 36 states that the
treatment of choice for A is endovascular therapy and for D is
open therapy,8 treatments for B and C are less clear and depend
on comorbidities, informed patient decision making, and operator skill.
Timaran studied this less clearly defined group of TASC B
and C and compared open against endovascular revascularization
for each.11 One hundred eighty-eight patients were treated: 136
endovascularly with 178 angioplasty and stent procedures, and
52 patients undergoing 60 direct surgical reconstructions. Patient
comorbidities were well matched, but more were treated for claudication in the open surgery group. The percutaneous group was
more likely to be TASC B and have stenoses, whereas the open
group more commonly had occlusions and fit into TASC C. There
was no difference in runoff to the lower extremity. Primary patency at 1, 3, and 5 years was 89%, 86%, and 86%, respectively, after
surgical bypass compared with 85%, 72%, and 64%, respectively,
after angioplasty and stenting. Patency was not affected by TASC
classification in this study. Survival was not significantly different
between the groups.
As technology and experience improve, more research is
looking at safety and efficacy of endovascular treatment of TASC
C and D lesions. Leville in 2005 published results from a series
of percutaneous treatment of varying degrees of unilateral or
bilateral iliac occlusion.12 Eighty-nine patients underwent 92
procedures for symptomatic lesions. Technical success was 94%
and 86% for TASC C and D patients, respectively. Primary patency ranged from 73% to 80% and did not differ significantly
between TASC groups. Secondary patency rates where 93% and
83%. Claudicants had better secondary patency than did those
with critical limb ischemia. Th is study showed good patency and
limb salvage with similar to open repair but with lower complication rate.
Hans compared results of 32 aortofemoral bypass (ABF)
patients to 40 aortoiliac stenting (AIS) patients.13 There was no
difference between TASC groups or indications for procedure for
open or percutaneous methods. Stented patients were older in this
cohort. Primary patency was 93% for ABF and 69% in AIS. Complications occurred in 10% of AIS, including dissections, thrombosis, and access-related problems. Secondary patency was 100%
in ABF and 89% in AIS, suggesting that open repair had better
patency and clinical results.
These studies indicate that all TASC lesions can be approached
percutaneously with moderate success. One may be trading the
difficulty of open aortic reconstruction (hospital stay, pulmonary
and cardiac complications) with reduced durability of the intervention. So, the clinician should consider both the comorbidities
of the patient and the anatomic distribution of disease when planning open versus endovascular repair.
There are no large prospective randomized trials comparing
the various modalities. Most series are retrospective reviews of
nonrandomized cohorts or case series (Level II evidence, Grade B
recommendation).

5/22/2012 5:59:24 PM

Aortoiliac Occlusive Disease

2. Is primary stenting better than selective stenting?


Aortoiliac angioplasty and stenting has proven itself for TASC A
and most TASC B lesions, with increasing use in more advanced
lesions. Prior investigations have tried to identify which combinations of angioplasty and stenting give the best results, whether it
is angioplasty alone with selective stenting or primary stent placement. Selective stent placement is used after angioplasty if there is
a residual angiographic lesion 30% or a persistent mean pressure
gradient of 5 mm Hg.
The Dutch Iliac Stent Trial (DIST) in 2006 attempted to
answer this question in a prospective randomized trial.14 Two
hundred seventy-nine patients were randomized to either primary stent (143) or PTA and selective stent (136). Of the selective
group, 43% ultimately required stents. Mortality, hemodynamic
success, patency, and reintervention rates were similar between
groups. Symptomatic success, defined by quality of life tools, was
improved for the selective group at 5 years, and approached that
of age-matched controls. Equivalence of treatments and perhaps
improved quality of life in selective stenting led them to recommend selective stenting as the procedure of choice for iliac stenosis and occlusion. The implications of their findings were reduced
costs with selective stenting, and introduction of less foreign
material into the body.
Kudo reviewed his institutions 11-year experience with
151 limbs in 104 patients treated with angioplasty and selective
stenting.15 Demographics were similar, with a trend toward more
male smokers in the PTA-only group. The distribution of TASC
A through D was 26%, 47%, 24%, and 3%. The stent group had
more patients being treated in higher TASC categories. Primary,
assisted, and secondary patencies were not significantly different
between the two groups. Technical success was high (99%) and
complication rates were low (0.7%). Distal SFA stenosis and Type
C and D TASC lesions were associated with PTA failure. Although
equivalence of treatment arms was reported, overall patency was
low at 59% and 49% at 3 and 5 years, respectively.
De Roeck in 2006 published results of a cohort undergoing
primary stenting in complex aortoiliac lesions.16 Thirty-eight
patients were analyzed, twelve TASC B, ten TASC C and sixteen
TASC D. Reported initial success was 97% and clinical improvement was noted in 96% of patients. Primary patency was 94%,
89%, and 77% at 1, 3, and 5 years. Secondary patency was 100%,
94%, and 94% at the same interval. Complication rate was 5.4%.
As in the Dutch Iliac Stent Trial, AbuRahma compared selective with primary stenting in a cohort of 110 consecutive patients
with 149 lesions.17 Unlike the DIST, this study included TASC C
and D lesions. Technical success was 100%. Clinical success was
defined by symptom resolution, increased ABI, and low residual
stenosis. This was achieved in 97% of the primary group and 83%
of the selective group, which was clinically significant. When
evaluated by TASC grouping, type A and B lesion outcomes were
similar regardless of treatment. TASC C and D lesions, however,
fared much better with primary stenting, with 84% clinical success compared to 46% in the selective group, again statistically
significant. This trend held true for patency data as well. Patency
for C and D lesions at 1, 2, and 3 years was 96%, 90%, and 72%
whereas selective stenting had 46%, 46%, and 28% at similar intervals. Overall complication rate in this sample was 2.6%.

PMPH_CH87.indd 693

693

Whereas TASC II does not give specific recommendations


on which technique to use, the above reports may indicate better
results with primary stenting in longer, more advanced aortoiliac
lesions, leaving selective stenting for shorter focal stenoses (Level I b
evidence, Grade B recommendation).
3. Does open reconstruction trump endovascular repair?
The decision to intervene for AOID is similar to other vascular interventions: lifestyle limiting claudication, rest pain, and
gangrene. For asymptomatic patients, lifestyle modification and
medical management should be initial treatment. Once intervention is deemed necessary to improve pain or heal tissue, one must
decide on how to treat the lesion based on anatomic distribution and patient-specific risk factors. Anatomic factors include
lesion location, severity, distal extent, stenosis or occlusion, and
calcium burden. Patient factors include age, medical comorbidities, preference to one type of operation, and allergies (contrast).
Open reconstruction, the gold standard, is durable and successful if the above technical factors are followed. However, not all
are candidates for major open reconstruction, especially the
elderly, medically infirmed patient in need of inflow to the lower
extremity.
Although not randomized, several studies have compared
direct reconstruction to endovascular repair. Burke in 2009 performed a retrospective review comparing 118 aortobifemoral
bypasses to 174 aortoiliac angioplasty and stent procedures.18
Operations that included both open femoral endarterectomy and
proximal endovascular treatment were excluded from the study.
The endovascular group was older, suffered more from CHF and
COPD and was more likely to claudicate. The ABF group was more
likely to have undergone prior intervention and more commonly
presented with critical limb ischemia. TASC D classification was
more common in the ABF group. Outcomes with regard to mortality, myocardial infarction, or stroke were no different between the
groups. The ABF group had more complications postoperatively
and had longer stays in hospital and the ICU. After controlling for
several variables, the change in postprocedural ABI was higher in
the ABF group. Freedom from amputation or reintervention was
no different, however, between the two cohorts at mean follow-up
of 32 months. The two cohorts were not exactly similar with
respect to TASC distribution, but the single center study indicated
no difference in the study endpoints of freedom from amputation
or reintervention.
A similar study was published by Kashyap in 2008.19 Their
study included 86 patients with ABF or ileofemoral bypass (IFB),
and 83 patients with percutaneous angioplasty and stenting
(PTAS). Demographics were similar except that ABF patients
were younger with higher cholesterol and more frequent tobacco
abuse. General anesthesia was more common in ABF. A significant difference in primary patency at 3 years was reported at 93%
for ABF and 74% for PTAS (P = .002). Assisted patency, secondary patency, limb salvage, and survival were similar between the
groups. On univariate analysis diabetes, age less than 60, and
requirement of distal bypass were found to adversely affect patency. Although no patency data were presented in the fi rst study,
they both indicate that excellent results can be achieved with
both methods. In the second study although initial patency was

5/22/2012 5:59:24 PM

694

Surgery: Evidence-Based Practice

not as good, assisted patency matched open repair and patients


were better matched as far as TASC classification is concerned.
In light of these studies, patients should be made aware of all
options for revascularization with attendant risks: ICU stay,
hospital stay, transfusion requirements, durability, and need for
future treatments to maintain patency of the intervention. With
a skilled surgeon at hand, they can then assist in the process
of informed decision making to agree on the most appropriate
method of revascularization.
4. What technical factors result in successful direct reconstruction of AOID?
Aortofemoral graft ing has become the gold standard of aortoiliac reconstruction, but many technical controversies emerged
during its development.20 Although certain generalizations can
be made, operations should be tailored to the patient and clinical
scenario as appropriate.

Approach
Transperitoneal exposure of the abdominal aorta is the preferred
route for direct reconstruction. This incision is rapid and provides
ample access to pathology in question. A retroperitoneal incision
may be used for alternative access and has been reported to reduce
postoperative ileus and pulmonary complications. Retroperitoneal
incisions are also advantageous in that they can avoid a hostile
abdomen from prior aortic surgery or intestinal catastrophes. Left
renal and visceral arterial approaches are also facilitated by this
technique, making it the incision of choice for combined revascularizations to those beds. The right renal, iliac, and femoral systems are sometimes difficult to access during this approach.
Several reports have compared the two operative exposures
with attention toward pulmonary and gastrointestinal (GI) morbidity. Some studies included both aneurysmal and occlusive
disease together. One prospective randomized trial by Cambria
evaluated outcomes after transperitoneal or retroperitoneal exposures in 113 patients.21 Baseline characteristics between the groups
were similar. The study found no difference between cohorts for
operative times, transfusion requirements, respiratory morbidity,
recovery of GI function, narcotic requirements, or hospital stay.
Other case series demonstrated significantly more ileus and longer hospital stay. No series demonstrated increased mortality with
one method. In another randomized study of 145 patients undergoing aortic reconstruction for aneurysmal or occlusive disease of
the aorta, Sicard discovered higher incidence of ileus, small bowel
obstruction, and longer hospital stay in the transperitoneal exposure cohort.22 The surgeon can thus choose either approach (Level Ib
evidence, Grade B recommendation).

Proximal Anastomoses
End to end and end to side are the two possible configurations
for the proximal anastomosis. Both are considered acceptable
and effective, and each has its place in certain types of occlusive
distributions in the aortoiliac segment. End-to-end configuration should be sought when there is complete degeneration of the
aorta or there is occlusion of the infrarenal segment. Benefits of
the end-to-end anastomoses include straight in-line flow without
hemodynamic alterations, as well as lack of competing flow into

PMPH_CH87.indd 694

the native distal aortic segment. With heavy calcification of the


aorta, end-to-side connections may be much more difficult than
being able to endarterectomize a circumferential cuff of distal
aorta prior to suturing end to end. Other considerations include
maintaining patency of inferior mesenteric artery (IMA) and
accessory renal vessels, best suited by end-to-side technique. For
patients with occluded external iliac arteries, end-to-end configurations may devoid the pelvis of antegrade blood flow if additional
hypogastric grafting is not sought and lead to disabling hip and
buttock claudication or impotence.
Amelli in 1991 prospectively studied 120 patients with aortofemoral bypass, 42.5% with end-to-end and 57.5% with end-toside anastomoses.23 Cumulative survival and patency were similar
in both groups, whereas the end-to-end group experienced more
operative death. The end-to-end cohort was older, suffered more
from coronary disease, and contained a higher proportion of
aneurismal aortic disease. Dunn in 1982 compared proximal
technique in 192 patients undergoing aortofemoral bypass, 101
end to end and 91 end to side.24 The cohorts achieved similar
5-year patency, 87% in end to end and 85% in end to side. There
were two aortoduodenal fistulas in their study, both in the end-toside group although not reaching statistical significance. Grafts
secured by end-to-end proximal anastomosis fit better anatomically in the native aorta bed, allowing for adequate coverage by
retroperitoneal tissue. End-to-side configurations are more difficult to cover, possibly exposing them to duodenal adherence and
fistulization.
Interestingly, in computed tomography follow-up of end-toside anastomoses, Mikati revealed distal aortic occlusion in 48 of
52 patients (92%).25 These investigations were done 5 to 10 years
after bypass, potentially allowing time for collateralization prior
to thrombosis. These findings were confirmed in nine patients
studied by OConnor, of which six had distal aortic occlusion
within 8 months.26 No patients in either series developed symptoms of pelvic ischemia, regardless of aortic patency.
Although no recent data exist on this topic, most proximal
anastomoses are constructed according to surgeon preference
and training, taking into account aortic calcium burden, pelvic
blood flow, ease of retroperitoneal coverage, and inferior mesenteric and accessory branch blood flow (Level II evidence; Grade B
recommendation).

Distal Anastomosis
Because atherosclerosis is a progressive disease, distal anastomoses should be brought to the femoral system in the majority of
cases. Obese or diabetic patients with unsanitary groins would
be an obvious exception, where all graft material should be kept
in the abdomen. Baird demonstrated increase in graft failure
when not bringing the graft to the femoral arteries.27 The femoral
exposure is straightforward and allows direct examination of the
outflow vessels. When groin infection is a concern, autogenous
reconstruction with deep femoral veins can be done to minimize
risk of infection.

Outflow Vasculature
For a vascular bypass to be successful, there must be adequate outflow. Profundaplasty, including removal of an obstructing plaque
at the profunda origin has been shown to be a vital component

5/22/2012 5:59:24 PM

Aortoiliac Occlusive Disease

of reconstruction in patients with disease at the anastamotic site.


Reconstruction of the profunda can be performed with endarterectomy, autogenous patch angioplasty, or extension of the graft
limb onto the profunda itself. Malone identified profunda disease
as the sole source for aortofemoral graft limb occlusion to 71 of
522 femoral arteries in his series.28 Occlusions were treated with
angioplasty of diseased profunda arteries, with 100% patency. In
this series, autogenous angioplasty was 2.5 times more likely to be
successful than on-lay Dacron patch (Level IIb evidence).
5. What adjunctive medication and follow-up protocol support
successful AOID interventions?
Successful arterial bypass requires meticulous anastomoses to
prevent short-term graft failures. Long-term, adjunctive antithrombotic agents likely assist with patency. Prior studies have
shown that failure rates of lower extremity bypasses can be as high
as 45% without these medications. Tangelder in 1999 performed a
meta-analysis of randomized control trials of aspirin and warfarin in lower extremity bypass grafts.29 The relative risk for occlusion with aspirin was 0.78. This was reduced to 0.38 if aspirin was
combined with oral anticoagulation. No reductions in the combined endpoints of myocardial infarction, stroke, or death were
seen in this meta-analysis.
Two Cochrane reviews evaluating antiplatelet in peripheral intervention have been performed. In 2008, Brown showed
improved patency with antiplatelet medications of both venous
and artificial grafts compared to placebo.30 Those with prosthetic
grafts were more likely to benefit than those with vein grafts.
Again, the outcomes related to cardiac morbidity and mortality
were not different between antiplatelet and placebo. In 2011 Drffler-Melly noted that addition of aspirin and dipyridamol reduced
recurrent obstruction by 60% after percutaneous angioplasty.31

695

As a result of these findings and level of evidence, recommendation 41 of the TASC-II document suggests adding antiplatelet
therapy to all those undergoing open or endovascular revascularization procedures for both atherosclerotic and nonatherosclerotic
disease, that is, trauma.8 Antiplatelet agents should be continued
indefinitely unless contraindicated by bleeding risk (Level I evidence; Grade A recommendation).
Clinical follow-up of any intervention should be multifaceted and incorporate interval history, physical exam findings,
and warranted imaging or noninvasive studies. New claudication
after intervention should raise suspicion of graft or stent problems
or development of distal disease. Graft surveillance can be performed in a dedicated vascular laboratory with duplex examination to examine stent or ABF limb velocities. Early elevations in
velocities can indicate anastomotic problems with the intervention and need for revision. With elevated velocities at anastomotic
sites, patients should be placed in a more frequent surveillance
protocol if reintervention is not pursued. Patients with AOID
intervention should be evaluated every 6 months with history,
physical, and duplex of the graft or stent. Although there are
several nonrandomized studies evaluating the efficacy of duplex
exam after infrainguinal bypass, data are lacking for similar studies on aortoiliac intervention. Rigorous evaluation of interventions
may identify the failing graft needing intervention prior to thrombosis. These exams may indicate pending aortofemoral limb
thrombosis, in-stent restenosis, new dissection from prior angioplasty, or anastomotic intimal hyperplasia at graft connections.
The vascular noninvasive lab can also detect new lesions in the
femoropopliteal or tibioperoneal region that may restrict outflow
of a prior functioning inflow procedure. All these are reasons to
enroll patients in an intense follow-up protocol and prevent future
limb problems.

Clinical Question Summary


Question

Answer

1 Which lesions in AOID


are favorable for
endovascular repair?

All lesions can be approached and treated


with moderate success.

II

11-13

2 Is primary stenting
better than selective
stenting?

Selective stenting should be used in TASC


A and B short focal lesions, whereas
primary stenting should be used on
TASC C and D lesions.

Ib

14-17

3 Does open
reconstruction trump
endovascular repair?

Patency is better with open


reconstruction, while postoperative
complications may be higher.

II

18, 19

4 What technical factors


result in successful
direct reconstruction
of AOID?

Via transperitoneal or retroperitoneal


approach, bypass should be taken to
the femoral arteries with end-to-end
or end-to side proximal anastomoses,
depending on clinical situation.
Profunda femoris disease should be
addressed during femoral anastomosis.

II

21-28

5 What adjuncts
and follow-up are
recommended after
intervention?

Antiplatelet agents should be used after


open or percutaneous revascularization.
Follow-up every 6 to 12 months with
history, physical, and noninvasive lab.

A
D

29-31

PMPH_CH87.indd 695

Level of
Evidence

Grade of
Recommendation

References

5/22/2012 5:59:24 PM

696

Surgery: Evidence-Based Practice

REFERENCES
1. Kieny R. Ren Leriche and his work as time goes by. Ann Vasc
Surg. 1990;4(2):105-111.
2. Connolly J, Price T. Aortoiliac endarterectomy: a lost art? Ann
Vasc Surg. 2006;20(1):56-62.
3. Wylie J. Thromboendarterectomy for arteriosclerotic thrombosis of major arteries. Surgery. 1952;32:275-292.
4. Tegtmeyer C, Moore T, Chandler J, Wellons H, Rudolf L. Percutaneous transluminal dilatation of a complete block in the right
iliac artery. AJR. 1979;133:532-535.
5. Upchurch G, Dimick J, Eliason J, et al. Diff usion of new technology in health care: the case of aorto-iliac occlusive disease.
Surgery. 2004;136(4):812-818.
6. Selvin E, Erlinger T. Prevalence of and risk factors for peripheral
arterial disease in the United States: National Health and Nutrition Examination Survey, 1999-2000. Circulation. 2004;110:
738-743.
7. Becker G, McClenny T, Kovacs M, Raabe R, Katzen B. The importance of increasing public and physician awareness of peripheral
arterial disease. J Vasc Interv Radiol. 2002;13:7-11.
8. Norgen L, Hiatt W, Dormandy J, et al. Inter-society consensus
for the management of peripheral arterial disease (TASC II). Eur
J Endovasc Surg. 2007;33:S1-70.
9. Haimovici, H. Arteriographic patterns of atherosclerotic occlusive disease of the lower extremity. In: Ascher E., ed. Haimovicis
Vascular Surgery. 5th ed. Malden, MA: Blackwell Publishing;
2004.
10. Hansen M, Valentine J, McIntire D, et al. Age-related differences
in the distribution of peripheral atherosclerosis: when is atherosclerosis truly premature? Surgery. 1995;118(5):834-839.
11. Timaran C, Prault T, Stevens S, et al. Iliac artery stenting versus
surgical reconstruction for TASC (TransAtlantic Inter-Society
Consensus) type B and C iliac lesions. J Vasc Surg. 2003;38(2):
272-278.
12. Leville C, Kashyap V, Clair D, et al. Endovascular management
of iliac artery occlusions: extending treatment to TransAtlantic Inter-Society Consensus class C and D patients. J Vasc Surg.
2006;43(1):32-39.
13. Hans S, DeSantis D, Siddiqui R, Khoury M. Results of endovascular therapy and aortobifemoral graft ing for Transatlantic InterSociety type C and D aortoiliac occlusive lesions. Surgery. 2008;
144(4):583-590.
14. Klein W, van der Graaf Y, Seegers J, et al. Dutch Iliac Stent Trial:
Long-term results in patients randomized for primary or selective stent placement. Radiology. 2006;238:734-744.
15. Kudo T, Chandra F, Ahn S. Long term outcomes and predictors of
iliac angioplasty with selective stenting. J Vasc Surg. 2005;42(3):
466e1-466e13.

PMPH_CH87.indd 696

16. De Roeck A, Hendricks J, Delrue F. Long term results of primary


stenting for long and complex iliac artery occlusions. Acta Chir
Belg. 2006;106:187-192.
17. AbuRahma A, Hayes D, Flaherty S, Peery W. Primary iliac stenting versus transluminal angioplasty with selective stenting.
J Vasc Surg. 2007;46(5):965-970.
18. Burke C, Henke P, Hernandez R, et al. A contemporary comparison of aortofemoral bypass and aortoiliac stenting in the treatment
of aortoiliac occlusive disease. Ann Vasc Surg. 2010;24(1):4-13.
19. Kashyap V, Pavkov M, Bena J, et al. The management of severe aortoiliac occlusive disease: Endovascular therapy rivals open reconstruction. J Vasc Surg. 2008;48(6):1451-1457.
20. Brewster D. Current controversies in the management of aortoiliac occlusive disease. J Vasc Surg. 1997;25:365-379.
21. Cambria R, Brewster D, Abbott W, et al. Transperitoneal versus
retroperitoneal approach for aortic reconstruction: a randomized prospective study. J Vasc Surg. 1990;12(4):505-506.
22. Sicard G, Reilly J, Rubin B, et al. Transabdominal versus retroperitoneal incision for abdominal aortic surgery: report of a prospective randomized trial. J Vasc Surg. 1995;21(2):174-181.
23. Ameli F, Stein M, Aro L, et al. End-to-end versus end-to-side
proximal anastomosis in aortobifemoral bypass surgery: does it
matter? Can J Surg. 1991;34(3):243-246.
24. Dunn D, Downs A, Lye C. Aortoiliac reconstruction for occlusive disease: comparison of end-to-end and end-to-side proximal
anastomoses. Can J Surg. 1982;25(4):382-384.
25. Mikati A, Marache P, Watel A, et al. End-to-side aortoprosthetic
anastomoses: long-term computed tomography assessment. Ann
Vasc Surg. 1990;4(6):584-591.
26. OConnor S, Walsh D, Zwolack R, et al. Pelvic blood flow following aortobifemoral bypass with proximal end-to-side anastomoses. Ann Vasc Surg. 1992;6(6):493-498.
27. Baird R, Feldman P, Miles J, et al. Subsequent down-stream repair
after aorta-iliac and aorta-femoral bypass operations. Surgery.
1997;82:785-793.
28. Malone J, Goldstone J, Moore W. Autogenous profundaplasty:
The key to long-term patency in secondary repair of aortofemoral
graft occlusion. Arch Surg. 1982;117:1593-1600.
29. Tanfelder M, Lawson J, Algra A, Eikelboom B. Systematic review
of randomized controlled trials of aspirin and oral anticoagulants
in the prevention of graft occlusion and ischemic events after
infrainguinal bypass surgery. J Vasc Surg. 1999;30(4):701-709.
30. Brown J, Lethaby A, Maxwell H, Wawrzyniak A, Prins M. Antiplatelet agents for preventing thrombosis after peripheral arterial bypass surgery. Cochrane Library. 2008; Issue 4.
31. Drffler-Melly J, Koopman M, Prins M, Bller H. Antiplatelet
and anticoagulant drugs for prevention of restenosis/reocclusion
following peripheral endovascular treatment. Cochrane Library.
2011; Issue 2.

5/22/2012 5:59:24 PM

Commentary on
Aortoiliac Occlusive Disease
Alan B. Lumsden

Perhaps no area of vascular intervention has undergone such


radical changes in approach than aortoiliac occlusive disease.
Frequency of aortobifemoral bypass has dwindled to almost nonsustainable training levels in most programs, while endovascular
enthusiasts tout that no lesion in this segment should be excluded
from catheter-based intervention! Drs Horn and Jordan have provided an excellent and traditional review of therapy for aortoiliac
occlusive disease. In this commentary I will be deliberately provocative and hope to stimulate the reader into considering more
forward thinking and nontraditional interventions.
Let us first consider that all procedures consist of a delivery
system and a therapeutic component to the operation. The latter
consists of inflating the balloon, deploying the stent, or sewing in
the bifurcated graft. Although the aortoiliac segment may be the
only area in which real debate could exist, few would argue that
the successful placement of the Dacron conduit reigns supreme.
But the delivery system for placement of an aortoiliac Dacron
graft leaves a lot to be desired, and that is an understatement.
Who in the right mind would not give up some patency for the
more acceptable catheter-based delivery of balloons, stent, and
stent grafts? However, the implication is that we surgeons have
sacrificed durability for delivery and we have been more than
complicit in enhancing public acceptance of this approach. Had
we placed as much emphasis on maintaining the core therapeutic
part of our operation while changing the delivery system to make
it more acceptable, we could have experienced the best of both
worlds. Hence the ongoing, although limited flirtation with robotics. Several groups both in the United States, Canada, and Europe
continue to flirt with robotic aortofemoral bypass grafting and the
more challenging repair of aneurysms. Criticisms abound: takes too
long, technically too difficult, too many ports, not enough vascular surgeons with laparoscopic skills, and not enough right instruments. Perhaps too many of the surgeons have moved too far away
from traditional surgery to even objectively consider laparoscopic
or robotic bypass grafting. Let us not forget the rapid development
of highly sophisticated hybrid rooms, where blending endovascular
techniques and open skills are the order of the day. I watch our cardiac surgeons perform left internal mammary artery (LIMA) to left
anterior descending artery (LAD) bypass through a thoracotomy
on a beating heart or sew a vein graft on to the aorta through the
same incision using anastamotic staplers, while cardiologists provide catheter interventions for difficult to access lesions. Where are
the anastamotic staplers in our world? Such devices could transform

our ability to perform minimally invasive bypass. Similarly direct


aortic access through a mini-sternotomy can be used for a tanscatheter aortic valve implant (TAVI) placement. So the vascular surgical
community that has led so much of the minimally invasive vascular
community must continue to re-evaluate our delivery systems and
preserve durability of our interventions. Optimal therapy for aortoiliac occlusive disease has yet to be defined and we should keep an
open mind; all options should be on the table and vascular surgeons
alone could objectively evaluate hybrid approaches.
In talking about hybrid procedures, I wanted to address an
area that has been a particular concern of vascular surgeons,
namely the interface between a heavily diseased common femoral/
distal external iliac and an iliac system relined with stent grafts
or bare metal stents. Here is the typical scenario: surgeon opts to
cut down on the common femoral artery, perform an endarterectomy, and use catheter-based techniques to open the ipsilateral
iliac system. The sequence is usually the following: expose the
common femoral artery, direct puncture with needle as far distally as possible within the common femoral, pass wire, and work
through sheath. Once the iliac system has been opened, endoluminal control is obtained with a balloon. This is easy for a stent graft;
a balloon placed close to the distal end will provide good control,
not so with a bare metal stent, where collaterals are often preserved
and the balloon can be ruptured on the stent edges. Control of
all the common femoral branches is necessary. The artery is then
opened and if possible the distal end of the stent/stent graft directly
visualized. This will allow the interface between the endoluminal
and open reconstruction to be inspected directly. The endarterectomy is then performed, and the plaque sharply transected at the
stent edge, ensuring no residual plaque protruding into the lumen.
Often, however, this is an area that is difficult to visualize when
the stent ends higher in the external iliac artery. A patch should be
initiated as close to the end of the stent as possible, necessitating
the endoluminal control to move a little higher. If the endoluminal
open interface is not directly visualized, then a completion retrograde arteriogram should be performed and the stent extended
across any residual plaque if required.
Regarding the totally occluded aorta, an open end-to-end
aortobifemoral bypass is often required in our opinion. This
procedure, which preserves renal flow by preventing thrombus extension, has excellent durability and is tolerated by many
because the aorta is already occluded. An axillobifemoral graft is
a poor second choice, although there remains a role in patients
697

PMPH_CH87.indd 697

5/22/2012 5:59:24 PM

698

Surgery: Evidence-Based Practice

with prohibitive cardiac or pulmonary comorbidities. Likewise,


although rarely necessary, thoracofemoral bypass graft ing from
the distal descending thoracic aorta is an operation vascular
surgeons should continue to maintain in their armamentarium,
and perhaps this is an application for an endoscopic proximal
anastomosis.
Finally, as the romance with stent grafting continues, we
can expect even more aggressive endoluminal interventions. The

PMPH_CH87.indd 698

technical challenges that we continue to face are controlled navigation through an occluded segment with deliberate and precise
re-entry into perfused segments. Solutions for these technical challenges are all currently being evaluated. So I am certainly not proposing to throw away the hard-earned catheter, wire, and imaging
skills. What I am proposing is that we surgeons continue to innovate toward optimal delivery and optimal durability, and that we
continue to evaluate and refine all the skills at our disposal.

5/22/2012 5:59:25 PM

CHAPTER 88

Femoropopliteal and
Tibioperoneal Occlusive
and Aneurysmal Disease
Luke X. Zhan and Joseph L. Mills

CLASSIFICATION OF INFRAINGUINAL
ARTERIAL DISEASE AND INDICATION FOR
REVASCULARIZATION INTERVENTION

systolic pressure <50 mm Hg or toe systolic pressure <30 mm Hg


(or absent pedal pulses in diabetic patients).2
PAD is clinically staged based on symptoms using Fontaine or
Rutherford classifications (Table 88.1).1,3 They are useful for standardized communication between clinicians. The two primary
indications for revascularization intervention are claudication
and CLI. Patients who are significantly disabled by claudication,
such that they are unable to perform their primary occupations
or activities of daily life, or those whose lifestyles are significantly
limited are potential candidate for infrainguinal intervention. A
trial of risk factor modifications, including smoking cessation,
blood sugar control in diabetic patients, lifestyle change, and
exercise, in conjunction with medical therapy, is usually indicated
before operative intervention. The primary goal of intervention on
a patient with lifestyle limiting claudication is to achieve symptomatic relief and improvement in quality of life.

Atherosclerotic peripheral arterial disease (PAD) is responsible


for almost all lower limb arterial insufficiency. The incidence and
prevalence is likely to rise as the population ages. Patients with
PAD are faced not only with disability affecting their legs and risk
of limb loss, but also with increased risk of cardiac and cerebrovascular morbidity and mortality due to a systemic disease burden. PAD can occur at any level and patients with severe forms
of PAD commonly have multilevel lesions. Involvement of the
infrainguinal segment is often the rule in those patients. Infrainguinal disease is often termed as outflow disease. The treatment
of femoropopliteal and tibioperoneal occlusive disease arguably
presents one of the most evolutionary and perhaps also most challenging and controversial areas of vascular medicine. We will primarily focus our discussion on management of occlusive disease.
However, many principles and techniques described herein may
also be applied to patients with aneurysmal conditions.
Patients with PAD can clinically present from asymptomatic
to different degrees of claudication to critical limb ischemia (CLI).
This spectrum of manifestations reflects the severity of disease.
The Transatlantic Inter-Society Consensus (TASC II) defines
claudication as muscle discomfort in the lower limb reproducibly
produced by exercise and relieved by rest within 10 min.1 However, absence of claudication does not preclude the diagnosis of
PAD because patients may have comorbidities such as severe cardiopulmonary or musculoskeletal disease that prevent sufficient
activity to produce claudication. CLI is a manifestation of PAD
that describes patients with typical chronic ischemic rest pain
or with ischemic tissue loss. The most recent European consensus document defines CLI as (1) persistent, recurring ischemic
rest pain requiring opiate analgesia for >2 weeks and (2) ankle

Table 88.1 Fontaine and Rutherford Classification


of PAD
Fontaine

Rutherford

Stage Clinical

Grade Clinical

Asymptomatic

Asymptomatic

II a

Mild claudication

Mild claudication

II b

Moderate to severe
claudication

Moderate
claudication
Severe
claudication

3
III

Ischemic rest pain

Ischemic rest pain

IV

Ulceration or gangrene

5
6

Minor tissue loss


Major tissue loss

699

PMPH_CH88.indd 699

5/22/2012 5:59:58 PM

700

Surgery: Evidence-Based Practice

Patients with CLI generally require interventions. Such


patients typically fall into Fontaine III and IV and Rutherford 4 to
6 categories. Wolfe and Wyatt further subdivided patients with CLI
into subgroups of subcritical and critical ischemia based on subsequent amputation risk.4 Subcritical ischemia describes patient with
rest pain and ankle pressure >40 mm Hg, whereas critical ischemia
refers to patient with rest pain and tissue loss or ankle pressure
<40 mm Hg. At 1 year, 27% of patients with subcritical ischemia
achieved limb survival without revascularization, in contrast to
only 5% in the group of patients with critical ischemia. In practice,
these data suggest that revascularization intervention is imperative
for limb salvage in patients with critical ischemia.

and cerebral vascular morbidity and mortality. The greater risk


of open surgery must be balanced against the lower durability of
endovascular treatment. Patients with unreconstructable vascular
disease, severe joint contractures, extensive tissue loss, or severe
infection are much less likely to be amenable to any revascularization attempt, thus requiring primary amputation. The optimal
strategy must be tailored to individual patient; it should be aimed
to minimize the mortality and morbidity and maximize favorable
and durable outcomes.

SELECTION OF REVASCULARIZATION
INTERVENTION: ENDOVASCULAR
THERAPY VERSUS OPEN BYPASS
SURGERY

Regardless of the choice of interventions, subjective and preferably


quantifiable measurements are necessary to evaluate outcomes of
revascularization. The outcomes of infrainguinal bypass surgery
have been traditionally reported in terms of graft patency, limb
salvage, and patient survival rate. In comparison, a wider variety of outcome measurements are used to define the successful
infrainguinal EVT interventions, including symptom resolution,
limb salvage, technical success, primary patency, assisted primary
patency, secondary patency, and target lesion revascularization.
Clinical responses are defined using various combinations
of resolution of symptoms, limb salvage, and patient survival.
Among these criteria, patient survival is the least relevant because
intervention for infrainguinal disease only addresses one aspect
of a systemic disease. Clinical response, when expressed in terms
of symptomatic reliefincluding relief from rest pain, resolution of claudication, 50% increase in claudication distance, and so
onhas been notoriously subjective and unreliable. Limb salvage
has been defined using various combinations of clinical criteria,

Infrainguinal revascularization in CLI is traditionally accomplished by open bypass surgery (OBS). Recent rapid advances in
our understanding of arterial occlusive disease and technological
developments in endovascular therapy (EVT) have had a tremendous impact on patient care. Initial support for a role of EVT in
CLI treatment came from a randomized trial that showed no significant difference in outcome after a median of 4 years between
EVT and OBS for iliac or femoropopliteal disease and for claudication or rest pain.5 The Bypass versus Angioplasty in Severe Ischemia of the Leg (BASIL) trial challenged the traditional notion
that patients with CLI due to infrainguinal disease should be
treated with bypass surgery. The BASIL trial is the only randomized controlled trial to date comparing open surgical bypass with
EVT for CLI. In their initial 2005 report, the BASIL investigators
reported that the main clinical outcomes (overall survival and
amputation-free survival) were no different at 2 years after randomization to angioplasty-first or bypass-first revascularization
strategies. However, beyond 2 years there appeared to be a benefit
for OBS, providing impetus for an extension study.6
The 2005 ACC/AHA guidelines and the 2007 TASC II consensus document on the management of PAD recommend initial
revascularization with surgery only when the arterial anatomy
is not favorable for a percutaneous approach for infrainguinal
disease.7 The TASC II uses angiographic criteria to determine
treatment preference for infrainguinal disease based on anatomy
and severity of lesions. (See Table 88.2.) Simple lesions (TASC-A)
should be treated with EVT, and TASC-D should be treated with
OBS first due to prohibitive EVT failure rate. The TASC-B and
TASC-C lesion represents the middle of the spectrum, and the
treatment bias toward EVT first for TASC-B and OBS first for
TASC-D.2 There is evidence that in patients with TASC-C or D disease who are facing imminent limb loss but are poor candidates for
surgery, treatment first with EVT may be beneficial.8,9 In addition,
failed EVT usually does not preclude subsequent OBS according to
TASC II. We view both approaches as complementary rather than
competitive to each other. Treatment strategy must be individualized to the specific patient. The decision to proceed with EVT first
or OBS first for infrainguinal disease requires a thorough riskto-benefit analysis. The advanced disease present in CLI patients
represents a systemic disease burden with significant cardiac

PMPH_CH88.indd 700

DETERMINATION OF OUTCOMES OF
REVASCULARIZATION

Table 88.2 TASC II Classification of Femoropopliteal


Lesions

Type A

Type B

Type C
Lesion type

Type D
Description

Single stenosis 10 cm in length


Single occlusion 5 cm in length

Multiple lesions (stenoses or occlusions), each 5 cm


Single stenosis or occlusion 15 cm not involving the infrageniculate popliteal artery
Single or multiple lesions in the absence of continuous tibial vessels to improve inflow
for a distal bypass
Heavily calcified occlusion 5 cm in length
Single popliteal stenosis

Multiple stenoses or occlusions totaling >15 cm with or without heavy calcification


Recurrent stenoses or occlusion that need treatment after two endovascular interventions

Chronic total occlusions of CFA or SFA (>20 cm, involving the popliteal artery)
Chronic total occlusion of popliteal artery and proximal trifurcation vessels

From Norgren et al.1

5/22/2012 5:59:58 PM

Femoropopliteal and Tibioperoneal Diseases

including healing of ulcers and gangrene, no or minor amputation


at 6 weeks, freedom from major amputation, and so on. Again, the
presence of a wide variety of definitions reflects lack of a standardized staging system to gauge the clinical response. The revised
Society for Vascular Surgery lower extremity ischemia reporting
standards (Rutherford guidelines) recommend the use of specific
clinical criteria, as well as noninvasive vascular laboratory data
to evaluate outcomes from interventions for infrainguinal PAD.3
This guideline provides uniform criteria for evaluating change in
clinical status and facilitates standardization of report outcomes
of revascularization and clinician communications (Table 88.3).
Technical success is defined as the presence of antegrade flow
through the treated lesion at the completion of the intervention.
Commonly used methods for assessing whether intervention is
successful technically includes (1) distal pulse palpation and Doppler flow assessment; (2) intraoperative duplex scanning, with and
without papaverine administration; and (3) completion arteriography. The distal pulse palpation and Doppler flow assessment are
noninvasive and offers ease of use. However, it lacks sensitivity.
Intraoperative duplex scanning has been used to evaluate flow
velocity and characteristics after revascularization. Bandyk and
Chavanpun10 and Tinder et al.11 have reported that intraoperative
duplex scanning allows identification of characteristics of infrainguinal vein bypass graft that is predictive of graft stenosis and failure. Completion angiography is considered as the gold standard
and is routinely performed at the conclusion of endovascular intervention. Technical success has also been defi ned as the absence of
occlusion or less than 30% to 50% residual stenosis in the treated
vessel on completion angiography. Flow-limiting dissections and
residual stenosis of greater than 30% occur in 10% of EVT.12 These
characteristics predict poor immediate and long-term outcomes.
The convenience of performing angiography in the same setting
as EVT intervention allows detection of these intraprocedural
factors that adversely affect the outcome of EVT. Intraoperative
angiography has also been increasingly used to evaluate the graft,
anastomosis, and outflow for significant graft-threatening lesions
in open bypass reconstructive surgery. Immediate assessment
with completion angiography improves the outcome by identifying any graft-threatening lesion that can be addressed. Mills and
associates prospectively evaluated 214 consecutive infrainguinal

701

bypass graft s with completion angiography and identified significant lesions requiring revision in 8% of the grafts.13 All those
reports support the notion that immediate assessment of the technical success of revascularization can improve the outcome.
The ultimate goal for any revascularization intervention is to
improve the distal perfusion. In a clinical setting, distal perfusion
can be measured with hemodynamic parameters such as anklebrachial pressure index (ABI), segmental pressure, and Doppler
duplex waveform. When used as a measurement of revascularization outcome, an intervention is often considered as a hemodynamic success with an increase in ABI of 0.10 to 0.15 or greater.
The extent of improvement in ABI can serve as a gauge of the
immediate hemodynamic response as well as a prognostic indicator. Improvement of the ABI after iliac percutaneous transluminal angioplasty has been shown to be strongly predictive of
sustained patency.14,15 However, limited data exist for hemodynamic response of EVT intervention in infrainguinal diseases.
The Rutherford guideline requires both hemodynamic success
and improved clinical response for a clinical success. More studies
are needed to compare the hemodynamic improvement of EVT
versus OBS in infrainguinal disease.
Durability of interventions can be measured by long-term
clinical response, or more mechanically, by closely surveying the
patency of conduit or graft for revascularization. Patency is especially relevant in OBS since various grafts are used to accomplish
revascularization with different long-term patency. In EVT, patency is often referred to the patency of native vessels that were
subjected to intervention or stents placed in native vessels. There
are several types of patency. Patency may be primary, assisted
primary (most applicable to vein grafts), or secondary. Primary
patency describes the duration of a graft or arterial segment that
is free of occlusion or significant stenosis within the treated segment. Various articles define stenosis as an anatomical lesion that
occludes greater than 30% to 50% of the segment on angiography,
or using velocity ratio greater than 2, or ABI decrease greater than
0.15.16-19 Primary patency is typically reported in the context of
Kaplan-Meier life-table analysis. A graft with primary patency has
been continuously patent without any actions being performed to
maintain graft patency. Assisted primary patency is primary patency requiring a subsequent intervention to maintain patency.

Table 88.3 Recommended Scale for Gauging Changes in Clinical Status


+3

Markedly improved: No ischemic symptoms, and any foot lesions completely healed; ABI essentially normalized (increased to
more than 0.90)

+2

Moderately improved: No open foot lesions; still symptomatic but only with exercise and improved by at least one category*;
ABI not normalized but increased by more than 0.10

+1

Minimally improved: Greater than 0.10 increase in ABI but no categorical improvement or vice versa (i.e., upward categorical shift
without an increase in ABI of more than 0.10)

No change: No categorical shift and less than 0.10 change in ABI

Mildly worse: No categorical shift but ABI decreased more than 0.10, or downward categorical shift with ABI decrease of less
than 0.10

Moderately worse: One category worse or unexpected minor amputation

Markedly worse: More than one category worse or unexpected major amputations

*Categories refer to Rutherford classification (Table 1). 3


In cases where the ABI cannot be accurately measured, an index based on the toe pressure, or any measurable pressure distal to the site of
revascularization, may be substituted.

PMPH_CH88.indd 701

5/22/2012 5:59:58 PM

702

Surgery: Evidence-Based Practice

Secondary patency refers to patency that has been restored after


occlusion of the treated segment. Graft surveillance is critical to
the long-term outcome of infrainguinal bypass.

Table 88.4 Angiographic Criteria for Runoff Scoring


Angiographic Finding

Score

Scoring of the popliteal, anterior and posterior tibial,


and peroneal arteries

DETERMINANTS OF OUTCOMES

Normal or minimal evidence of disease

Characteristics of Vascular Lesions


Affect Outcomes

2049% Stenosed

5099% Stenosed

In general, the location of the lesion, stenosis versus occlusion,


lesion length, extent of calcification, and the number of segments involved can affect the outcome of an intervention. For
infrainguinal disease, outcomes improve following the treatment
of femoropopliteal lesion compared with infrapopliteal lesions,
presumably due to larger caliber of vessel and improved inflow
after intervention. The length of the lesion has been thought to
inversely correlate with outcome as determined in terms of technical success, clinical response, and patency. Various data support the conclusion that clinical responses are more favorable for
stenosis compared with occlusion. This may be due to the fact that
occlusive lesions occupy a severe end of the disease spectrum. In
multilevel disease, each lesion has its own failure rate and can
collectively have an adverse impact on the outcome. In addition,
patients with multisegment disease tend to have more comorbidities, which increase the risk for perioperative complication and
decreased long-term survival. All those are considered adverse
factors for the outcome of revascularization. Accordingly, most
recent TASC II document takes those characteristics into consideration with its recommendation.2
In addition to target lesion characteristics, the runoff status
also correlates with outcome of infrainguinal EVT or OBS. The
outflow vessel quality is commonly evaluated with perioperative
angiography and assigned a weighted score using the Society for
Vascular Surgery and the International Society for Cardiovascular
Surgery (SVS/ISCVS ) runoff score standard.20,21 The SVS/ISCVS
system was revised in 1997.3 This scoring system grades both the
degree of occlusion and the relative contribution to outflow of
each runoff vessel, from 0 to 3, then adds 1 to the product of these
two grades, resulting in a decimal scoring system that assigns 1 to
a widely patent runoff and 10 to an isolated, blind segment with no
major vessel runoff (Table 88.4). Some evidence suggests that the
success of OBS correlates directly with runoff status. Poor outflow
vessels result in poor outcomes in femoropopliteal or infrapopliteal bypass from the open bypass literature.22-24 Similar to what
is reported in the open surgical bypass literature, poor patency
rates are also associated with poor runoff scores in EVT for superficial femoral artery disease,25,26 femoropopliteal angioplasty,27
and infrapopliteal interventions.23 These findings suggest that
runoff status is an important determinant in outcome.

Occluded for less than half its length

2.5

Occluded throughout most of its length

The Choice of Interventions Can Impact


Outcomes: Comparison of Open Surgical
versus Endovascular Approaches
Despite extensive development and application of EVT, there has
only been one large-scale prospective randomized controlled trial
comparing the outcome of EVT, primarily percutaneous transluminal balloon angioplasty (PTA) with OBS in patients with CLI.
The BASIL trial was a landmark study that compared clinical

PMPH_CH88.indd 702

Scoring of the pedal arch


Completely patent arch

Partially occluded

1.5

Little or no arch visualized

Adopted from Rutherford et al. 3

efficiency and cost effectiveness of a revascularization using bypass


surgery first versus EVT first for CLI due to infrainguinal disease.
The main measurements of clinical outcomes were amputationfree survival and overall survival. Secondary outcomes were postprocedural morbidity, reinterventions, health-related quality of
life, and the consumption of hospital resource. The initial analysis
of the BASIL trial in 2005 reported no statistically significant difference in amputation-free survival or health-related quality of
life after 2 years of follow-up. However, OBS has over one-third
higher hospital costs during the first year than the EVT.6
The final analysis of the long-term outcomes from BASIL
study at 5 years confirms the initial conclusion that OBS or EVT
offers equal amputation-free survival or overall survival rate. For
patients surviving for 2 years or more (70% of the BASIL cohort),
health-related quality-of-life measures were not significantly different overall. However, the initial difference in hospital-related
costs of the two interventions equalized in the long term, possibly
attributable to higher occurrence of reintervention in EVT group
and its associated cost. Another interesting finding is that those
who were randomized to bypass surgery first and who survived
for at least 2 years had a significant increase in subsequent overall
survival and a trend toward improved amputation-free survival.
It is still unclear whether this gain was due to increased surveillance, durability of results, or improved healing of wounds.28-30
There are many controversies surrounding the BASIL trial
and its interpretation. These include the choice of study population, endpoints examined, and the nature of procedures performed. In addition, very little data exist to compare the outcome
of EVT and OBS when outcomes are measured in terms of clinical
response, hemodynamic improvement, and long-term durability
of those interventions. These are currently active areas of clinical
investigation.

Types of Grafts Affect Outcomes of OBS


Various choices of conduit exist for OBS performed for infrainguinal occlusive. Autogenous vein options include great saphenous
vein, small saphenous, femoral vein, and basilic and cephalic vein.
Prosthetic options include cryopreserved vein or synthetic graft
such as Dacron, polytetrafluoroethylene (PTFE). The patency rates

5/22/2012 5:59:58 PM

Femoropopliteal and Tibioperoneal Diseases

of various grafts are summarized in Tables 88.5A, B, and C that


are based on recent review.31,32 Even though some reports describe
saphenous vein grafts and PTFE grafts as having comparable patency rates, autogenous vein is felt by most to be superior to all prostheses, even in the above-knee popliteal segment. A study by Veith
et al. has shown that the amputation rate following vein graft occlusion was lower than that with PTFE graft (12% vs. 26%).33 Furthermore, when an amputation was to be performed, the above-knee
amputation was performed less frequently in patients with failed
vein bypass grafts as opposed to patients with failed prosthetic
bypass grafts (20% vs. 80%).34 ACC/AHA guidelines recommend
autogenous vein as first choice for bypass to the above-knee and
below-knee popliteal and femorotibial bypass. When no autogenous
vein is available, it is reasonable to use prosthetic grafts.7

Comorbidities Influence Outcomes


Diabetes is a major etiological factor for PAD. Diabetes also
adversely influences the outcome of infrainguinal revascularization, possibly due to a systemic disease burden. Diabetic patients
commonly have multilevel lesion and tibial involvement; this indicates a poor prognosis by itself. In additional, diabetic patients
present with decreased primary patency, equal associated primary
assisted patency, and a significantly decreased 5-year survival and
limb salvage on univariate analysis. A retrospective review of

703

infrainguinal EVT by Lazaris et al. found a decreased technical


success rate and significant increased complication rate.16 Comparative analysis by Abularrage et al. analyzed 920 patients who
had undergone infrainguinal EVT, half with diabetes and half
without, over a 5-year period. The diabetic cohort had a higher
incidence of CLI than the nondiabetic cohort.35
End-stage renal disease (ESRD) adversely affects outcomes of
OBS. A systematic review of 16 retrospective surgical case series of
infrainguinal bypass in ESRD patients reported pooled estimates
of 79%, 77%, and 42% for 1-year graft patency, 1-year limb salvage,
and 2-year patient survival, respectively,36 compared with 89%, 94%,
and 76% respectively in non-ESRD control groups.37 In addition, the
mortality rate and overall 5-year survival rate were lower, and the
percentage of amputation due to failure of foot salvage was higher in
patients with ESRD.38-40 Limited data exist regarding the outcomes
of EVT in patients with ESRD. Study from Graziani et al. suggest
that EVT can achieve technical success.41 Brosi et al. reported that
in infrapopliteal angioplasty in patients with ESRD, the primary
technical success reached 97%, whereas hemodynamic improvement was more limited (in only 50% of the limbs treated). The pedal
arteries were severely diseased in all, and complete occlusion of the
pedal arch was found in 58% of limbs on completion.42 Given the
high operative mortality for OBS in patients with ESRD, EVT offers
a viable alternative. However, further study is required to evaluate
the long-term clinical response to EVT in patients with ESRD.

Table 88.5A Patency of Above-Knee Femoropopliteal Grafts


Primary Patency (%)
Graft Type

1 Month

6 Months

1 Year

2 Years

3 Years

4 Years

Reversed saphenous vein

99

91

84

82

73

69

Arm vein

99

82

65

60

60

Human umbilical vein

95

90

82

82

70

70

PTFE

89

79

74

66

60

Table 88.5B Patency of Below-Knee Femoropopliteal Grafts


Patency (%)
Graft and Patency Type

1 Month

2 Months

1 Year

2 Years

3 Years

4 Years

Primary Patency
Reversed saphenous vein

98

90

84

79

78

77

In situ vein bypass

95

87

80

76

73

68

Secondary Patency
In situ vein bypass

97

96

96

89

86

81

Arm vein

97

83

83

73

70

Human umbilical vein

88

82

77

70

61

60

PTFE

96

80

68

61

44

40

Limb Salvage
Reversed saphenous vein

100

92

90

88

86

75

In situ vein bypass

97

96

94

84

83

PMPH_CH88.indd 703

5/22/2012 5:59:58 PM

704

Surgery: Evidence-Based Practice

Table 88.5C Patency of Infrapopliteal Grafts


Patency (%)
Graft and Patency Type

1 Month

6 Months

1 Year

2 Years

3 Years

4 Years

Primary Patency
Reversed saphenous vein

92

81

77

70

66

62

In situ vein bypass

94

84

82

76

74

68

Secondary Patency
Reversed saphenous vein

93

89

84

80

78

76

In situ vein bypass

95

90

89

87

84

81

Arm vein

94

73

62

58

Huma umbilical vein

80

65

52

46

40

37

PTFE

89

58

46

32

21

Limb Salvage
Reversed saphenous vein

95

88

85

83

82

82

In situ vein bypass

96

91

88

83

83

PTFE

76

68

60

56

48

Based on review of data from published series since 1981.

32

REFERENCE
1. Norgren L, Hiatt WR, Dormandy JA, et al. Inter-Society Consensus for the Management of Peripheral Arterial Disease (TASC
II). Eur J Vasc Endovasc Surg. 2007;33(Suppl 1):S1.
2. Lyden SP, Smouse HB. TASC II and the endovascular management of infrainguinal disease. J Endovasc Ther. 2009;16:II5.
3. Rutherford RB, Baker JD, Ernst C, et al. Recommended standards for reports dealing with lower extremity ischemia: revised
version. J Vasc Surg. 1997;26:517.
4. Wolfe JH, Wyatt MG. Critical and subcritical ischaemia. Eur J
Vasc Endovasc Surg. 1997;13:578.
5. Wolf GL, Wilson SE, Cross AP, et al. Surgery or balloon angioplasty for peripheral vascular disease: a randomized clinical trial.
Principal investigators and their Associates of Veterans Administration Cooperative Study Number 199. J Vasc Interv Radiol.
1993;4:639.
6. Adam DJ, Beard JD, Cleveland T, et al. Bypass versus angioplasty
in severe ischaemia of the leg (BASIL): multicentre, randomised
controlled trial. Lancet. 2005;366:1925.
7. Hirsch AT, Haskal ZJ, Hertzer NR, et al. ACC/AHA 2005 Practice guidelines for the management of patients with peripheral
arterial disease (lower extremity, renal, mesenteric, and abdominal aortic): a collaborative report from the American Association for Vascular Surgery/Society for Vascular Surgery, Society
for Cardiovascular Angiography and Interventions, Society for
Vascular Medicine and Biology, Society of Interventional Radiology, and the ACC/AHA Task Force on Practice Guidelines
(Writing Committee to Develop Guidelines for the Management
of Patients With Peripheral Arterial Disease): endorsed by the
American Association of Cardiovascular and Pulmonary Rehabilitation; National Heart, Lung, and Blood Institute; Society for
Vascular Nursing; Trans Atlantic Inter-Society Consensus; and
Vascular Disease Foundation. Circulation. 2006;113:e463.

PMPH_CH88.indd 704

8. Conrad MF, Cambria RP, Stone DH, et al. Intermediate results of


percutaneous endovascular therapy of femoropopliteal occlusive
disease: a contemporary series. J Vasc Surg. 2006;44:762.
9. Brosi P, Dick F, Do DD, et al. Revascularization for chronic critical lower limb ischemia in octogenarians is worthwhile. J Vasc
Surg. 2007;46:1198.
10. Bandyk DF, Chavanpun JP. Duplex ultrasound surveillance can
be worthwhile after arterial intervention. Perspect Vasc Surg
Endovasc Ther. 2007;19:354.
11. Tinder CN, Chavanpun JP, Bandyk DF, et al. Efficacy of duplex
ultrasound surveillance after infrainguinal vein bypass may be
enhanced by identification of characteristics predictive of graft
stenosis development. J Vasc Surg. 2008;48:613.
12. Pentecost MJ, Criqui MH, Dorros G, et al. Guidelines for peripheral percutaneous transluminal angioplasty of the abdominal
aorta and lower extremity vessels. A statement for health professionals from a special writing group of the Councils on Cardiovascular Radiology, Arteriosclerosis, Cardio-Thoracic and
Vascular Surgery, Clinical Cardiology, and Epidemiology and
Prevention, the American Heart Association. Circulation. 1994;
89:511.
13. Mills JL, Fujitani RM, Taylor SM. Contribution of routine intraoperative completion arteriography to early infrainguinal bypass
patency. Am J Surg. 1992;164:506.
14. Carnevale FC, De Blas M, Merino S, et al. Percutaneous endovascular treatment of chronic iliac artery occlusion. Cardiovasc
Intervent Radiol. 2004;27:447.
15. Sullivan TM, Childs MB, Bacharach JM, et al. Percutaneous
transluminal angioplasty and primary stenting of the iliac arteries in 288 patients. J Vasc Surg. 1997;25:829.
16. Lazaris AM, Tsiamis AC, Fishwick G, et al. Clinical outcome
of primary infrainguinal subintimal angioplasty in diabetic
patients with critical lower limb ischemia. J Endovasc Ther.
2004;11:447.

5/22/2012 5:59:58 PM

Femoropopliteal and Tibioperoneal Diseases

17. Spinosa DJ, Leung DA, Harthun NL, et al. Simultaneous antegrade and retrograde access for subintimal recanalization
of peripheral arterial occlusion. J Vasc Interv Radiol. 2003;
14:1449.
18. Tisi PV, Mirnezami A, Baker S, et al. Role of subintimal angioplasty in the treatment of chronic lower limb ischaemia. Eur J
Vasc Endovasc Surg. 2002;24:417.
19. Myers KA. Reporting guidelines for open and endovascular
surgery: why the current recommendations should be revised.
J Endovasc Surg. 1995;2:321.
20. Rutherford RB, Flanigan DP, Gupta SK, et al. Suggested standards for reports dealing with lower extremity ischemia. J Vasc
Surg. 1986;4:80.
21. Peterkin GA, Manabe S, LaMorte WW, Menzoian JO. Evaluation of a proposed standard reporting system for preoperative
angiograms in infrainguinal bypass procedures: angiographic
correlates of measured runoff resistance. J Vasc Surg. 1988;7:379.
22. Seeger JM, Pretus HA, Carlton LC, et al. Potential predictors of
outcome in patients with tissue loss who undergo infrainguinal
vein bypass graft ing. J Vasc Surg. 1999;30:427.
23. Alback A, Roth WD, Ihlberg L, et al. Preoperative angiographic
score and intraoperative flow as predictors of the mid-term patency of infrapopliteal bypass graft s. Eur J Vasc Endovasc Surg.
2000;20:447.
24. Alback A, Biancari F, Saarinen O, Lepantalo M. Prediction of
the immediate outcome of femoropopliteal saphenous vein
bypass by angiographic runoff score. Eur J Vasc Endovasc Surg.
1998;15:220.
25. Davies MG, Saad WE, Peden EK, et al. Impact of runoff on superficial femoral artery endoluminal interventions for rest pain and
tissue loss. J Vasc Surg. 2008;48:619.
26. Ihnat DM, Duong ST, Taylor ZC, et al. Contemporary outcomes
after superficial femoral artery angioplasty and stenting: the
influence of TASC classification and runoff score. J Vasc Surg.
2008;47:967.
27. Clark TW, Groffsky JL, Soulen MC. Predictors of long-term patency after femoropopliteal angioplasty: results from the STAR
registry. J Vasc Interv Radiol. 2001;12:923.
28. Forbes JF, Adam DJ, Bell J, et al. Bypass versus Angioplasty in
Severe Ischaemia of the Leg (BASIL) trial: health-related quality of life outcomes, resource utilization, and cost-effectiveness
analysis. J Vasc Surg. 2010;51:43S.
29. Bradbury AW, Adam DJ, Bell J, et al. Bypass versus Angioplasty in Severe Ischaemia of the Leg (BASIL) trial. J Vasc Surg.
2010;51:5S.

PMPH_CH88.indd 705

705

30. Bradbury AW, Adam DJ, Bell J, et al. Multicentre randomised


controlled trial of the clinical and cost-effectiveness of a bypasssurgery-first versus a balloon-angioplasty-first revascularisation
strategy for severe limb ischaemia due to infrainguinal disease.
The Bypass versus Angioplasty in Severe Ischaemia of the Leg
(BASIL) trial. Health Technol Assess. 2010;14:1.
31. Dalman RL, Taylor LM, Jr. Infrainguinal revascularization procedures. In: Porter JM, Taylor LM, Jr., eds. Basic Data Underlying Clinical Decision Making in Vascular Surgery. St. Louis, MO:
Quality Medical Publishing; 1994:141.
32. Mills JL. Infrainguinal disease: surgical treatment. In: Cronenwett JL, Johnston W, eds. Rutherfords Vascular Surgery. 7th ed.
Philadelphia, PA: Saunders; 2010:1682.
33. Veith FJ, Gupta SK, Ascer E, et al. Six-year prospective multicenter randomized comparison of autologous saphenous vein
and expanded polytetrafluoroethylene grafts in infrainguinal
arterial reconstructions. J Vasc Surg. 1986;3:104.
34. Wilson, YG, Wyatt, MG, Currie, IC, et al. Preferential use of vein
for above-knee femoropopliteal grafts. Eur J Vasc Endovasc Surg.
1995;10:220.
35. Abularrage CJ, Conrad MF, Hackney LA, et al. Long-term outcomes of diabetic patients undergoing endovascular infrainguinal interventions. J Vasc Surg. 2010;52:314.
36. Albers M, Romiti M, Braganca Pereira CA, et al. A meta-analysis
of infrainguinal arterial reconstruction in patients with endstage renal disease. Eur J Vasc Endovasc Surg. 2001;22:294.
37. OHare A, Johansen K. Lower-extremity peripheral arterial disease among patients with end-stage renal disease. J Am Soc Nephrol. 2001;12:2838.
38. Whittemore AD, Donaldson MC, Mannick JA. Infrainguinal
reconstruction for patients with chronic renal insufficiency. J
Vasc Surg. 1993;17:32.
39. Johnson BL, Glickman MH, Bandyk DF, Esses GE. Failure of
foot salvage in patients with end-stage renal disease after surgical revascularization. J Vasc Surg. 1995;22:280.
40. Reddan DN, Marcus RJ, Owen WF, Jr., et al. Long-term outcomes
of revascularization for peripheral vascular disease in end-stage
renal disease patients. Am J Kidney Dis. 2001;38:57.
41. Graziani L, Silvestro A, Bertone V, et al. Percutaneous transluminal angioplasty is feasible and effective in patients on chronic
dialysis with severe peripheral artery disease. Nephrol Dial
Transplant. 2007;22:1144.
42. Brosi P, Baumgartner I, Silvestro A, et al. Below-the-knee angioplasty in patients with end-stage renal disease. J Endovasc Ther.
2005;12:704.

5/22/2012 5:59:58 PM

Commentary on
Femoropopliteal and Tibioperoneal
Occlusive and Aneurysmal Disease
Frank J. Veith

Dr Zhan and Dr Mills have done a creditable job of describing the


management of infrainguinal arteriosclerosis and the symptoms
and problems it produces. There are a few areas in which I would
like to provide some additional information and a few points of
minor disagreement that I would like to raise.
First, it has become more common for vascular specialists
to treat intermittent claudication with aggressive invasive treatments, and this is particularly true in the endovascular treatment era. However, I still believe that most (>80%) of patients
with intermittent claudication are best managed conservatively.
Many of these patients have serious comorbidities, and it is well
known that intermittent claudication only rarely (~1% per year)
progresses to critical ischemia. Many, if not most, patients can live
with their claudication if they are adequately reassured.
Second, we have always had a very aggressive approach to
limbs salvage when a patient is facing an imminent amputation
which not all patients with critical limb ischemia (CLI) do. This
aggressive approach and all its components are well described in
three articles that stress the following points: (1) the importance
of reintervention when a primary procedure fails; (2) the value of
prosthetic bypass when autologous vein is unavailable or of poor
quality; (3) the fact that limb salvage techniques can work quite
well in diabetic patients; (4) the fact that most patients with threatened limbs have a pattern of arterial disease suitable for revascularization with a percutaneous transluminal balloon angioplasty
(PTA) or bypass; and (5) the interplay of endovascular techniques
and bypasswith both often being required in the same patient,
particularly after a primary procedure fails.1-3 Such failure occurs
in at least 35% of patients after the primary procedure at some
point of time in the patients course.
Third, although the authors point out the value of the Bypass
versus Angioplasty in Severe Ischemia of the Leg (BASIL) trial, and
the relative equipoise of surgery-first and endovascular-first
approaches, the endovascular therapies (EVTs) used in BASIL are
now very outmoded. Endovascular treatments for infrainguinal
lesions, including very distal lesions in the lower leg and foot are

improving dramatically and changing the landscape dramatically.


Better balloons, stents, and 0.014 inch-based systems are facilitating EVTs that were not in use during the BASIL trial. Drug eluting
stents and balloons have become available, and give promise of
preventing restenosis and intimal hyperplasia. Retrograde access
is proving feasible for crossing difficult lesions. Although all these
techniques are costly and still have to be evaluated, it is my belief
that EVTs will have markedly improved outcomes in the near
future. Other improvements with new techniques, gene therapy,
and cell therapy may also prove to have value for the treatment of
CLI. Accordingly, I believe the pendulum will swing further away
from open bypass procedures than it currently has. On the other
hand, some patientsat some time in their course will always
require an open procedure to salvage their limb. I estimate this
to be from 25% to 35%. Just as in the past with the bypasses we
pioneered, most of these treatments for CLI will have to address
very distal lesions, and many of them will be difficult and time
consuming to administer.
Despite these challenges, the treatment of CLI will be one of
the most interesting and rewarding areas in vascular surgery during the coming years with rapidly improving technology and outcomes. In addition, there will be a continuing need for specialized
skills to administer these old and new treatments.

REFERENCES
1. Veith FJ, Gupta SK, Samson RH, et al. Progress in limb salvage by
reconstructive arterial surgery combined with new or improved
adjunctive procedures. Ann Surg. 1981;194:186-197.
2. Veith FJ, Gupta SK, Ascher E, et al. Changing arteriosclerotic disease patterns and management strategies in lower limb threatening ischemia. Ann Surg. 1990;212:402-414.
3. Veith FJ, Gupta SK, Ascher E, et al. Improved strategies for secondary operations on infrainguinal arteries. Ann Vasc Surg. 1990;
4:85-93.

706

PMPH_CH88.indd 706

5/22/2012 5:59:58 PM

CHAPTER 89

Thoracic Outlet Syndromes


Robert W. Thompson

INTRODUCTION

ETIOLOGY

Thoracic outlet syndrome (TOS) represents a group of heterogeneous and potentially disabling upper extremity disorders caused
by extrinsic compression of neurovascular structures passing
through the thoracic outletthe anatomic space immediately
posterior to the clavicle and overlying the first rib. Three types of
TOS are defined by the anatomic structures involved and the distinct clinical syndromes that result: (1) neurogenic TOS, caused by
congenital and/or posttraumatic forms of brachial plexus nerve
compression within the scalene triangle or subcoracoid (pectoralis minor) space; (2) venous TOS, characterized by subclavian
vein compression within the costoclavicular space at the level of
the first rib, leading to the effort thrombosis (Paget-Schroetter)
syndrome; and (3) arterial TOS, caused by subclavian artery
compression, usually in association with a cervical rib or first rib
anomaly, resulting in subclavian artery aneurysm formation and
distal thromboembolism.1,2
All three forms of TOS are relatively rare, and the overall
prevalence of these conditions is unknown. The best information
comes from analysis of the Nationwide Inpatient Sample database,
from which Chang et al. estimated that approximately 1900 operations are performed for neurogenic TOS each year in the United
States.3 Based on the proportion of patients with neurogenic TOS
who typically require surgical treatment and the prevalence of
neurogenic TOS compared with other forms of TOS, the total
number of patients treated for TOS each year is approximately
10,000 to 20,000. Although the different forms of TOS are distinct, they often share clinical features that make them interrelated but potentially confusing, thereby increasing the challenge
of clinical diagnosis. These disorders can also produce substantial
disability; because they occur in relatively young, active, and otherwise healthy individuals, they are often unrecognized and/or
inadequately treated, and they can cause chronic pain syndromes
or limb-threatening complications requiring long-term management. Despite several decades of debate and controversy regarding proper diagnosis and optimal treatment, TOS continues to be
poorly understood and understudied.

1. What causes the different types of TOS?

Neurogenic TOS
Neurogenic TOS constitutes approximately 85% to 95% of all
patients with TOS.1,4 It is characterized by compression of the
brachial plexus nerve roots (C5 to T1) within the scalene triangle
and/or subpectoral space, with symptoms consisting of pain and
paresthesias in the neck, shoulder, arm, and hand. These symptoms can be somewhat variable and are typically dynamic, with
marked positional exacerbation during arm abduction, elevation,
and other maneuvers. Neurogenic TOS can be especially debilitating as it can result in chronic pain syndromes refractory to
conservative management.
Current concepts indicate that neurogenic TOS is caused by
a combination of predisposing anatomical factors and previous
trauma. Indeed, the normal anatomy of the thoracic outlet serves
as a predisposing factor in the development of neurogenic TOS,
because the neurovascular structures that traverse this region are
prone to compression even during regular daily activities that
may place undue tension on the scalene muscles, such as sustained
or repeated elevation of the arm, heavy lifting, or vigorous turning of the neck. The anatomical predisposition to brachial plexus
compression may be increased by congenital structural variants,
such as scalene muscle variations and abnormal tendinous bands,
which have been observed during surgical operations and in
cadaver studies.5,6 For example, the posterior aspect of the anterior scalene muscle (ASM) is frequently quite firm and tendinous
in nature and may give origin to fascial bands that extend from
the posterior surface of the muscle to the thickened extrapleural
fascia over the dome of the pleura (Sibsons fascia), potentially
exerting pressure on adjacent nerve roots. The most common
muscular variation in this region is the scalene minimus muscle,
a structure that originates within the plane of the middle scalene
muscle, passes between various nerve roots of the brachial plexus,
and joins the ASM to insert upon the first rib. Additional softtissue variations include fascial bands that pass across or between
707

PMPH_CH89.indd 707

5/22/2012 6:00:29 PM

708

Surgery: Evidence-Based Practice

individual nerve roots, subsequently attaching to either the first


rib or the extrapleural fascia, and dense fascial bands that cross
over the T1 nerve root where it passes from underneath the first
rib to contribute to the brachial plexus. Roos has classified these
variations into at least nine different types of soft-tissue abnormalities that can affect the brachial plexus.5
Cervical ribs occur in approximately 0.45% to 1.5% of the
population and in 5% to 10% of patients with neurogenic TOS.7
They arise from C7 in the plane of the middle scalene muscle and
typically attach to the midlateral portion of the first rib either as
an immobile bony fusion or a fully developed joint.8 Incomplete
cervical ribs may also occur, arising as a bony or cartilaginous
extension from the seventh cervical vertebrae, with extension to
the first rib by a band of ligamentous tissue. Rudimentary first
ribs are not as frequently recognized as cervical rib anomalies
and their actual incidence is unknown. These structures consist
of a first rib that tends to lie higher in the neck than normal and
often inserts into the second rib rather than the sternum. Previous
trauma may also result in abnormalities of the fi rst rib or clavicle,
such as fractures with formation of thickened callous at the site of
bony healing.
Because many individuals harbor anatomical variations in
the scalene triangle in the absence of symptoms of neurogenic
TOS, anatomical factors are only considered a predisposing factor
rather than a distinct and separate cause, appearing only to lower
the threshold for development of symptoms following injury.
Most patients with neurogenic TOS describe some form of previous trauma to the head, neck, or upper extremity, followed by a
variable interval before the onset of progressive upper extremity
symptoms.1 Hyperextension injury of the anterior scalene and/or
pectoralis minor muscles is thought to lead to acute and chronic
inflammation and a reparative process that includes fibrosis and
persistent muscle spasm, with nerve root irritation that eventually
results in compressive neurological symptoms. Chronic changes
in the scalene musculature also include fibrotic contracture and
stiffening, as well as histopathological alterations that reflect persistent muscle injury such as type I muscle fiber predominance
and endomesial fibrosis.9,10 Prolonged nerve root compression and
irritation may also cause degenerative neural changes, such as
perineural fibrosis.11 Persistent use of the upper extremity in
activities that promote brachial plexus compression may further
aggravate symptom progression over time to produce progressive disability, and even low-grade repetitive trauma, such as
prolonged work at computer consoles that can contribute to this
disorder as a form of repetitive strain injury. Intermittent exacerbations of neurogenic symptoms may also occur as a result of
secondary scalene/pectoralis muscle injury producing additional
local inflammation and spasm. These exacerbations are often
interspersed with periods where symptoms are relatively quiescent, such that many patients do not seek medical attention until
symptoms are sustained and well advanced.
The interval between injury and the onset of symptoms may
thereby range from days and weeks to several years, serving to
obscure the relationship between a specific injury and development of neurogenic TOS; in some patients, the inciting injury has
been long forgotten and a history of trauma may be overlooked if
not specifically sought by the examining physician.1 Conversely,
not all patients with TOS have their condition brought on by a
specific traumatic event or activity. In these situations, it is postulated that extrinsic neural compression is caused by age-related

PMPH_CH89.indd 708

changes in posture (e.g., slumping of the shoulders and stooping


of the neck) superimposed upon congenital variations of scalene
musculature.

Venous TOS
Venous TOS is present in approximately 10% to 15% of all patients
with TOS.1 It is characterized by subclavian vein compression
between the clavicle and first rib within the costoclavicular space,
immediately anterior to the ASM and underneath the subclavius
muscle. Patients with venous TOS most commonly present with
axillary-subclavian vein effort thrombosis (Paget-Schroetter
syndrome), an acute clinical event characterized by the sudden
development of hand and arm edema, upper extremity cyanosis,
enlarged subcutaneous collateral veins, and early forearm fatigue
in the absence of arterial compromise. Although this condition
may occur in the setting of an acute overhead or lift ing strain to
the upper extremity, it frequently appears to develop as an unprovoked spontaneous event in the absence of a specific traumatic
injury. Some patients with venous TOS may exhibit a low-grade,
chronic form of subclavian vein compression, with or without
a superimposed effort thrombosis event. It is also important to
distinguish effort thrombosis from secondary subclavian vein
thrombosis caused by indwelling central venous catheters, pacemakers, malignancy, or other forms of intrathoracic pathology.
Venous TOS typically occurs in young, otherwise healthy
patients, who are often involved in vigorous occupational or recreational use of the upper extremity. In those with predisposing
anatomy, these activities can cause subclavian vein compression
between the first rib and clavicle. Scalene or subclavius muscle
hypertrophy, particularly in athletes and others involved in weight
lifting programs, may also contribute to subclavian vein compression. The pathogenesis of effort thrombosis involves repetitive
extrinsic venous compression with chronic injury and progressive fibrous stenosis and scar tissue encasement of the vein, along
with collateral vein expansion.12,13 Stenosis of the subclavian vein,
stagnant blood flow, and local tissue responses eventually lead to
thrombosis, which may cause abrupt occlusion of collateral vessels as well as the subclavian vein. Pulmonary embolism may also
occur but is less frequent compared to lower extremity deep vein
thrombosis. Although hematological abnormalities and hypercoagulable states have been suspected to potentially contribute to the
development of effort thrombosis, strong evidence for this is lacking; venous TOS is fundamentally considered to be a mechanical problem of extrinsic venous compression.

Arterial TOS
Arterial TOS is the least frequent type of TOS, present in approximately 2% to 5% of patients.14-18 It is caused by extrinsic compression of the subclavian artery between the ASM and the first rib,
and is almost always associated with the presence of a congenital
cervical rib or other bony anomaly. Sustained compression of the
subclavian artery leads to pathological changes in the arterial wall,
which may include intimal thickening with atherosclerotic plaque
formation or the development of flow-related poststenotic subclavian artery aneurysms. In either case, the principal complications of subclavian artery pathology are related to mural thrombus
and acute thromboembolism to the upper extremity, resulting in

5/22/2012 6:00:29 PM

Thoracic Outlet Syndromes

hand and/or digital ischemia. Mural thrombus can develop on


even small intimal plaques or ulcerations and it occurs early in
development of subclavian artery aneurysms, before they reach
a size when rupture would be a concern. Because the subclavian
artery is subject to frequent and substantial motion during upper
extremity activity, mural thrombus in this vessel appears particularly prone to cause distal embolism. In some cases, complete
occlusion of the subclavian artery due to focal intimal thickening
may also result in chronic arterial insufficiency; however, because
subclavian occlusions associated with TOS occur distal to the origin of the vertebral artery, TOS is not associated with the subclavian steal syndrome. As a consequence of distal embolism and
hand ischemia, severe digital vasospasm and complex regional
pain syndrome (causalgia or reflex sympathetic dystrophy) may
accompany and further complicate arterial TOS.
Answer: The different types of TOS are closely related to congenital and acquired variations in the underlying anatomy of the
scalene triangle and/or subpectoral space, combined with injury
and/or upper extremity overuse. All three types of TOS are distinguishable with respect to the specific neurovascular structure subject to compression and their clinical presentation, with minimal
but occasional overlap in symptoms and physical findings. Beyond
observational studies, there have been few investigations intended
to specifically address various aspects of etiology or pathophysiology of the various types of TOS. (Grade B recommendation.)

DIAGNOSIS
2. What are the best criteria to establish a clinical diagnosis of
neurogenic TOS?
The diagnosis of neurogenic TOS is based on clinical pattern recognition, often suggested by a stereotypical history and description of symptoms.19-21 The principal symptoms of TOS include
neck, hand or arm pain, dysesthesias, numbness, and weakness.
The symptoms may be bilateral, but they most commonly have
their greatest effect on the dominant upper extremity. The distribution of symptoms does not follow typical patterns referable
to a single cervical nerve root or peripheral nerve, allowing neurogenic TOS to be distinguished from compressive nerve root
conditions (cervical disc disease or arthritis), the ulnar nerve at
the elbow (cubital compression syndrome), the median nerve at
the wrist (carpal tunnel syndrome), or other related disorders.
Occipital headache is a common complaint associated with TOS,
most likely caused by secondary spasm within the trapezius and
paraspinous muscles. Periscapular pain is also common, often as
a result of poor posture and disturbed shoulder girdle mechanics,
and anterior chest wall or axillary pain may accompany neurogenic TOS associated with pectoralis minor syndrome. Symptoms
of neurogenic TOS are typically reproduced or exacerbated by
activity requiring elevation or sustained use of the arms or hands,
such as reaching for objects overhead, lifting, carrying, prolonged
work at keyboards, driving, speaking on hand-held phones, shaving, or combing or brushing the hair. Positional complaints may
also be brought on by lying supine, resulting in pain and difficulty sleeping. Many patients with neurogenic TOS are affected to
a mild and tolerable degree, yet the majority of patients consulting the surgeon have developed progressively disabling symptoms
that effectively prevent them from working or carrying out simple

PMPH_CH89.indd 709

709

daily activities. In some cases, the symptoms of neurogenic TOS


may have progressed to resemble the acuity of those observed in
complex regional pain syndrome, with persistent digital vasospasm, disuse edema in the hand, hypersensitivity, and withdrawal
to light touch of the affected extremity. The clinical course of
patients with neurogenic TOS is often accompanied by multiple
physician consultations, nonspecific testing results, and numerous partial or ineffective treatments.
Physical examination is directed toward excluding alternative conditions, eliciting particular factors that exacerbate hand
and arm complaints, and defi ning the degree of neurogenic disability. A thorough peripheral nerve examination is performed
to exclude ulnar nerve entrapment, carpal tunnel syndrome,
and other etiologies. The neck is examined to identify the extent
of local muscle spasm and to localize areas where focal digital
compression reproduces the individual patients symptom pattern, with additional maneuvers (e.g., Spurlings sign) to identify
degenerative cervical spine disease. The presence of supraclavicular tenderness over the scalene triangle serves to reinforce the
diagnosis of neurogenic TOS, with tenderness over the infraclavicular subcoracoid space used to identify the presence of neural compression at the level of the pectoralis minor tendon.22-24
Ablation of the palpable radial pulse while the patient elevates the
arm, inspires deeply, and turns the neck away from the affected
extremity (i.e., the Adson sign) is used to identify any degree of
positional subclavian artery compression. Since a positive Adson
sign does not specifically reveal brachial plexus compression and
is also common in asymptomatic individuals, this maneuver
(or similar tests in the noninvasive vascular laboratory) can only
serve to support, but not prove, the diagnosis of TOS; similarly,
negative fi ndings of the Adson maneuver do not exclude a diagnosis of neurogenic TOS.20 Perhaps the most useful components
of physical examination are the upper limb tension test (ULTT)
and the 3-minute elevated arm stress test (EAST). The ULTT is
performed with the arms outstretched 90o in abduction, with the
wrists sequentially flexed and extended; a positive finding occurs
when the abduction position reproduces symptoms, with exacerbation during wrist extension and alleviation during wrist flexion. The EAST is performed with the arms elevated in a 90o-90o
surrender position, with the patient asked to repetitively open
and close the hands. Most patients with neurogenic TOS report
the rapid reproduction of upper extremity symptoms with EAST,
often being unable to complete the exercise beyond 60 to 120 seconds. Conversely, those able to readily complete the 3-minute
EAST are unlikely to have a diagnosis of neurogenic TOS or have
relatively mild symptoms.
There are no specific diagnostic tests or imaging studies that
can replace the clinical diagnosis of neurogenic TOS, but plain
radiographs of the neck are helpful in determining if an osseous
cervical rib or prolonged C7 transverse process is present. The
results of conventional computed tomography (CT), magnetic resonance imaging (MRI), and electrodiagnostic (nerve conduction)
studies are frequently negative or nondiagnostic, even in patients
with pronounced disability. Nonetheless, inclusion of these tests
is important in the overall evaluation to help exclude other conditions considered in the differential diagnosis, such as degenerative
cervical spine or disc disease, neoplasms or other masses, intracranial pathology, and specific peripheral neuropathies.
Answer: The diagnosis of neurogenic TOS is based predominantly on the exclusion of alternative conditions and the presence

5/22/2012 6:00:29 PM

710

Surgery: Evidence-Based Practice

of a consistent set of clinical criteria, which have been established


over a period of many years by experts with experience in management of this condition. No formal, systematic, or specific
consensus diagnostic criteria for neurogenic TOS have yet been
developed for clinical or investigative purposes, and there have
been no comprehensive studies specifically addressing the accuracy, utility, or validity of different diagnostic findings. Indeed,
a recent review by the Cochrane Collaboration concluded that
evidenced-based analysis of treatments for neurogenic TOS is
complicated by the lack of generally accepted criteria for diagnosis and that development of distinct diagnostic criteria for this
condition is a high priority for future investigations.25 (Grade D
recommendation.)

DIAGNOSIS
3. What adjunctive studies are most useful in the diagnosis and
prognosis of neurogenic TOS?
Many of the controversies regarding neurogenic TOS have focused
on the indications for surgical management in the context of difficulty predicting individual responses to treatment and longterm prognosis. Longstanding effort has therefore been exerted to
establish a defined, objective, diagnostic approach to neurogenic
TOS, as well as to identify adjunctive testing procedures that can
better predict the outcomes of treatment.

Scalene Muscle Blocks


Infi ltration of local anesthetics into the ASM was introduced over
10 years ago by Jordan and colleagues, as a means to confirm the
clinical diagnosis of neurogenic TOS and to help predict outcomes of treatment.26 This approach was based on the recognition
that interruption of muscle spasm during ASM block can temporarily decompress neural elements passing through the thoracic
outlet, through relief of brachial plexus nerve root compression
exerted directly by the ASM, as well as through indirect effects
on the first rib and associated muscles that cause narrowing of
the costoclavicular space. Successful ASM blocks are associated
with immediate improvement in symptoms of numbness, pain,
and fatigability of the affected limb in patients with neurogenic
TOS, and are thereby thought to strongly confirm the diagnosis
otherwise based on clinical criteria.
Techniques for ASM block initially used surface landmarks.
However, spread of anesthetic into surrounding tissues can result in
inadvertent somatic and/or sympathetic blockade, impairing
interpretation of outcome. Jordan and colleagues have developed
increasingly rigorous testing protocols to improve the sensitivity
and specificity of this technique. To help improve localization,
percutaneous insertion of an insulated hypodermic needle into
the ASM is guided by electromyographic (EMG) monitoring
from the needle tip, with stimulation used to verify that the needle is not in the brachial plexus.26 After instillation of local anesthetic into the ASM, the intensity of pain induced by positional
maneuvers designed to exacerbate thoracic outlet compression
is assessed, and compared to the intensity of pain induced by
the same maneuvers following control injections with saline
alone. In an initial series of 122 patients with a clinical diagnosis of neurogenic TOS, the ASM was successfully identified

PMPH_CH89.indd 710

by EMG monitoring in each case with no instances of inadvertent somatic (brachial plexus) or sympathetic blockade.26,27 Of 38
patients that subsequently underwent surgical decompression, 30
of 32 (94%) with a positive ASM block had a good response to
operation, compared to only 3 of 6 (50%) that had had a negative
ASM block. This study thereby demonstrated for the first time
that electrophysiological guidance facilitates accurate needle
tip placement during performance of ASM blocks and that the
results of these blocks appear to correlate well with surgical outcomes. Jordan and colleagues subsequently combined the use of
EMG guidance with fluoroscopic monitoring and test-injections
with radiographic contrast to further improve needle tip localization during ASM injections.27 Using this approach in a series
of 22 patients with a clinical diagnosis of neurogenic TOS, they
encountered no significant side effects or technical complications
related to ASM injection. Favorable experience with this technique has led to extension of the tissues targeted to include the
middle scalene, subclavius, and pectoralis minor muscles.
Most recently, Jordan and colleagues have further refined
the technique for ASM blocks, to examine the potential use of
ultrasound (US) guidance rather than fluoroscopy (F) to enhance
needle tip localization.28 In a retrospective clinical analysis of 245
procedures in which patients were treated using one of the two
different targeting techniques (EMG/US, n = 77; EMG/F, n = 168),
there were no complications using EMG/US and three complications (1.8%) using EMG/F. There was also a good clinical outcome
in 70 of 77 (91%) procedures conducted with EMG/US and in 136
of 168 (81%) procedures conducted with EMG/F (not significant).
These findings reinforce the successful use of electrophysiologic
monitoring and image-guidance in the conduct of scalene/pectoralis muscle blocks for patients with neurogenic TOS, and indicate
that US imaging is equivalent in efficacy to fluoroscopy but may
have advantages with respect to diminished costs and radiation
exposure.

Clinical Predictors of Response to Treatment


To improve prediction of responses to treatment for neurogenic
TOS, Jordan and colleagues examined a variety of clinical features and the results of a patient self-assessment survey they
developed for this purpose, the Cervical Brachial Symptom Questionnaire (CBSQ).27 The CBSQ was initially validated in a group
of 50 patients, with a test-retest reliability of 0.87 and an internal
consistency of 0.93 (Cronbachs alpha). In 140 patients the correlation between CBSQ and the Brief Pain Inventory (BPI; total
of pain and interference scales) was 0.70, with a strong responsiveness to change when comparing patients before and after successful treatment. A total of 85 patients that underwent targeted
treatment for neurogenic TOS were then evaluated prospectively
for at least 6 months after treatment. A positive scalene block
was significantly associated with a treatment-responsive condition. Significant predictors of treatment-resistant TOS included
(1) a coexisting widespread pain syndrome, (2) past surgery for a
cervical-brachial condition other than TOS, (3) widespread sensory symptoms involving the upper arm and/or face, (4) weakness extending beyond the lower trunk dermatomes, and (5) the
presence of depression. In addition, 92% of treatment-resistant
patients had the combination of either a negative scalene block or
a coexisting widespread pain syndrome, as compared to 12% of

5/22/2012 6:00:29 PM

Thoracic Outlet Syndromes

the treatment-responsive patients. Finally, pretreatment CBSQ


scores were significantly higher in the treatment-resistant patients
as compared to the treatment-responsive group.

Medial Antebrachial Cutaneous Nerve


Conduction Studies
Despite the recognized limitations in the use of conventional electrophysiologic testing for the diagnosis of neurogenic TOS, it has been
suggested that nerve conduction (NC) studies of the medial antebrachial cutaneous (MAC) nerve may provide a more sensitive and
specific test for this condition. The MAC nerve is derived from the
C8 and T1 nerve roots and it arises from the medial cord of the brachial plexus immediately proximal to the origin of the ulnar nerve.
It passes along the medial aspect of the upper arm and emerges from
the deep fascia, along with the basilic vein, just proximal to the antecubital space. The sensory distribution of the MAC nerve encompasses the medial forearm to the wrist. The proximal origin of the
MAC nerve within and just distal to the scalene triangle, and the
correspondence of its distribution to a pattern of sensory symptoms
frequently observed in patients with neurogenic TOS, suggested
that this nerve might serve as a useful target for diagnostic studies.
Seror first reported the use of MAC studies in 2004, in 16 patients
with neurogenic TOS in whom conventional Electromyography
and nerve conduction studies (EMG/NCS) were normal.29
The use of MAC NC studies in patients with neurogenic TOS
was recently examined further by Machanic and Sanders. 30 They
evaluated a series of 41 consecutive patients with a clinical diagnosis of neurogenic TOS for whom thoracic outlet decompression
was planned, and compared the fi ndings with MAC NC studies
conducted in 19 asymptomatic healthy volunteers. They found
that MAC sensory latency measurements ranged from 1.5 to 2.4
millisecond (ms) for 79 control limbs, compared to a range of 2.2
to 2.8 ms for the symptomatic limbs; MAC latency values above
or below a threshold of 2.4 ms thereby had a specificity of 99% and
a sensitivity of 73% in the diagnosis of neurogenic TOS. The difference in MAC latency between the right and left limbs was less
than 0.2 ms in 17 of 19 volunteers (89%), whereas the difference
in latency measures between the symptomatic and asymptomatic limbs was greater than 0.3 ms in 31 of 41 patients (sensitivity 89%, specificity 63%). Measurements of MAC NC amplitude
revealed values of 10 V or greater in 77 control limbs and
values less than 10 V in 28 patients; amplitude measures above
or below a threshold of 10 V thereby had a specificity of 97%
and a sensitivity of 68% in the diagnosis of neurogenic TOS. The
amplitude ratio between right and left limbs was less than 1.7
in all control subjects, whereas ratios between the symptomatic
and asymptomatic limbs were greater than 2.0 in 25 patients, for
a specificity of 100% and a sensitivity of 61%. Importantly, there
were 10 patients in whom MAC NC studies were conducted
before and 3 months after thoracic outlet decompression for disabling neurogenic TOS. Eight of these patients had good clinical
improvement and two had no improvement. In comparing preand postoperative MAC studies, latency was reduced in all eight
patients with good clinical outcomes (range 0.20.6 ms); in two
patients with no clinical improvement, latency was unchanged
in one and increased in the other. MAC amplitude measurements were increased in seven of the eight patients with good
clinical outcome and unchanged in one; in the two patients with

PMPH_CH89.indd 711

711

no clinical improvement, amplitude measures were decreased in


one and unchanged in the other.
This study therefore suggests that MAC NC studies may provide a novel and reliable electrophysiological test by which to help
confirm the clinical diagnosis of neurogenic TOS and to predict
outcomes of treatment. It is emphasized that these techniques
must be standardized for each electrophysiologic laboratory and
that a combination of criteria should be used rather than a single measure. Machanic and Sanders recommend four diagnostic criteria for neurogenic TOS based on their MAC NC studies:
(1) latency greater than 2.4 ms; (2) latency difference between
symptomatic and asymptomatic limbs of 0.3 ms or greater; (3)
amplitude measures less than 10 V; and (4) amplitude ratios
greater than 2.0 between limbs.

Imaging Assessment of the Brachial Plexus


It has been considered for many years that radiographic imaging
might be applied to help define distinct anatomic features associated with a clinical diagnosis of neurogenic TOS, as a means
of diagnostic confirmation prior to surgical intervention or even
to guide different surgical approaches. Unfortunately, anatomic
variations in the thoracic outlet region are relatively frequent, and
previous imaging approaches used for this condition have been
of insufficient resolution to yield satisfactory diagnostic information. At present, many surgeons do not consider the presence of
radiographically evident abnormalities to be either necessary or
sufficient for the existence of clinically apparent neurogenic TOS,
because the extent of correlation between results from imaging
and functional clinical assessment has not yet been assessed
adequately enough to guide treatment. Recent approaches using
contrast-enhanced MRI appear to provide far greater anatomic
detail of the thoracic outlet region than available in the past,
and several investigators have described the successful use of
MRI in patients with neurogenic (and vascular) TOS. 31-36 It has
been reported that anatomic alterations observed with MRI
correspond with clinical symptoms and the results of surgical
treatment, but it has not yet been firmly established whether the
detection of specific anatomical fi ndings by MRI will be helpful
in the diagnosis or prediction of outcomes in patients with neurogenic TOS.
Answer: The use of EMG and radiographically guided scalene/pectoralis muscle blocks has emerged as a valuable adjunctive test in patients with a clinical diagnosis of neurogenic
TOS, both to lend support to the diagnosis and to provide prognostic information regarding response to treatment. Further
studies are needed to validate the wider use of this technique,
and to establish its general utility as a diagnostic/prognostic tool.
(Grade B recommendation.) MAC NC studies appear to demonstrate considerable promise as a diagnostic adjunct for patients
with neurogenic TOS, and may offer useful prognostic information regarding the responsiveness of patients to treatment.
Th is specific electrophysiological test appears to be a substantial
improvement over conventional NC studies but has yet to be
completely evaluated. (Grade C recommendation.) At this point
there does not appear to be substantial evidence that brachial
plexus imaging studies offer useful information in diagnosis,
prognosis, or treatment planning for patients with neurogenic
TOS. (Grade D recommendation.)

5/22/2012 6:00:29 PM

712

Surgery: Evidence-Based Practice

DIAGNOSIS
4. How does one make the diagnosis of the vascular forms of
TOS?

Venous TOS
The diagnosis of venous TOS is initially suggested by a history of
sudden and substantial arm swelling and cyanotic discoloration
in an otherwise healthy active individual. Venous Duplex studies may be used to confirm a clinical suspicion of upper extremity deep vein thrombosis, but it is important to recognize that
Duplex imaging is usually technically inadequate to visualize
the proximal subclavian vein at the location where it is typically
obstructed in venous TOS, with an overall accuracy of approximately 70%. Thus, venous Duplex studies do not add substantially to clinical evaluation in making a diagnosis of venous TOS
and they cannot be used to exclude a diagnosis of subclavian vein
effort thrombosis. Any patient in whom venous TOS is suspected
should therefore be studied promptly by contrast venography,
particularly in the context of an effort thrombosis event. Th is
may be performed using CT, MR or catheter-based approaches,
but catheter-based approaches carry the advantage of immediate access for thrombolytic treatment. Venographic studies in
patients with venous TOS typically reveal focal occlusion of the
subclavian vein at the angle between the first rib and clavicle; in
some cases, the absence of venous branch fi lling may give the
appearance of segmental venous thrombosis. Following spontaneous recanalization or thrombolysis of the subclavian vein,
the vein may appear patent with the exception of a focal stenosis
in the expected location. With or without complete occlusion,
a dense network of cervical collaterals is typically present, providing a route for venous drainage from the affected extremity
and reflecting the chronic nature of venous TOS. It is valuable to
conduct these venographic studies with positional maneuvers to
detect subclavian vein occlusion at the level of the first rib, and
it is often helpful to examine the contralateral side to detect evidence of symmetrical abnormalities.

Arterial TOS
Patients with arterial TOS may present with acute ischemia of the
hand or digits, accompanied by pain, paresthesias, and weakness
of the affected upper extremity. The brachial and axillary artery
pulses may also be absent to palpation in the presence of extensive
thromboembolism; however, the presence of palpable pulses does
not exclude the possibility of a more proximal source of arterial
embolism. One should seek additional evidence of arterial compromise to the upper extremity, such as sympathetic overactivity
with vasospasm, digital or hand ischemia, cutaneous ulceration
or emboli, forearm claudication, or the pulsatile supraclavicular
mass or bruit characteristic of a subclavian artery aneurysm.
Because the anatomical abnormalities underlying TOS are often
bilateral, it may also be informative to examine the contralateral
extremity for evidence of subclavian artery compression, occlusion, or aneurysm. In cases where acute intervention is necessary
to restore flow to the brachial artery and its branches, the diagnosis of arterial TOS may be entertained after initial brachial artery
thrombectomy or when thrombolysis has allowed more defi nitive evaluation of the proximal vessels by contrast arteriography.

PMPH_CH89.indd 712

A subset of patients may present with chronic symptoms related


to subclavian artery compression, including hand or arm weakness, early fatigue with use, and digital ulcerations or periodic
discoloration reminiscent of Raynauds phenomenon. In these
cases, compression of the subclavian artery is often incomplete,
with total occlusion being apparent only with positional maneuvers (i.e., the Adson test). In other situations, arterial TOS is suspected by the finding of an asymptomatic pulsatile mass in the
anterolateral neck. Patients with features suggesting arterial TOS
may be evaluated by positional noninvasive vascular laboratory
studies (segmental arterial pressures, waveform analysis, and
Duplex imaging), but it is often difficult to adequately visualize
the subclavian artery behind the clavicle. Radiographic studies
are useful to determine the presence of bony abnormalities, such
as a cervical rib. The definitive examination necessary to completely exclude or prove the existence of a fi xed arterial lesion is
provided by contrast arteriography. This may be accomplished by
traditional catheter-based arteriography, or by contrast-enhanced
CT or MRI techniques. In these examinations, it is important to
specifically alert the vascular radiologist to the potential need for
positional maneuvers and to consider bilateral studies even in
the absence of contralateral symptoms.
Answer: The diagnosis of subclavian vein effort thrombosis
is usually made on the basis of clinical presentation. Although
the presence of upper extremity deep vein thrombosis may be
confi rmed by Duplex US studies, such studies are often unnecessary and inaccurate in excluding venous TOS. Direct contrastenhanced imaging studies remain the most effective means to
confi rm the clinical diagnosis and catheter-based venography
offers the potential to initiate thrombolytic treatment in the
same setting. Evaluation without contrast-enhanced venography may lead to significant delays in diagnosis and treatment,
potentially impairing outcomes of care. (Grade A recommendation.) The clinical diagnosis of arterial TOS is often suspected
on the basis of the clinical presentation, but contrast-enhanced
arteriography is required to evaluate the subclavian and axillary arteries, as well as the presence or absence of distal emboli
within the arm and hand. Evaluation without contrast-enhanced
arteriography may lead to significant delays in diagnosis and
treatment, potentially impairing outcomes of care. (Grade A
recommendation.)

TREATMENT
5. How effective are nonsurgical approaches for the treatment
of neurogenic TOS?

Physical Therapy
In almost all situations, neurogenic TOS is initially treated with
physical therapy to relax and stretch the scalene and pectoralis
minor muscles, to improve relevant postural disturbances and
limb mobility, and to correct muscle imbalance in the cervicoscapular region.37-41 Occupational rehabilitation approaches are also
used to diminish repetitive strain exposure in the workplace. It
is important to recognize that physical therapy and occupational
therapy approaches otherwise used for related disorders of the
neck, shoulder, or upper extremity may not be appropriate for
neurogenic TOS, and may frequently aggravate symptoms.

5/22/2012 6:00:29 PM

Thoracic Outlet Syndromes

It is generally considered that the majority of patients with


neurogenic TOS experience substantial improvement in symptoms following appropriate conservative management, but the
exact proportion of such subjects and the extent of improvement
remains difficult to define for patients who exhibit widely different
degrees and duration of symptoms. It is also notable that there are
many patients with relatively mild and nondisabling symptoms
of neurogenic TOS that can be expected to improve with physical
therapy, whereas surgical treatment is typically considered only
for patients with longstanding and disabling symptoms in whom
conservative measures have failed to produce sufficient clinical
improvement.40,42 It therefore appears that valid comparisons of
nonsurgical and surgical approaches to the treatment of neurogenic TOS are unlikely, as these treatments are offered sequentially and to different patient populations.
Rigorous studies regarding various approaches to the conservative management of neurogenic TOS are quite limited despite
the fact that this approach is applied in almost all patients. In one
prospective observational study, Lindgren reported the outcomes
for 119 patients with neurogenic TOS that were treated with an
inpatient rehabilitation program and home exercises.43 There was
an 88% rate of patient satisfaction with the inpatient treatment
and consistent improvement during long-term follow-up, but
many limitations to the study with respect to standardization of
the diagnosis and extent of symptoms, the absence of a comparison control group, and use of qualitative outcome measures. In a
later study of 34 patients treated with physical therapy for a form
of neurogenic TOS, Glbahar et al. reported significantly better
results with regard to pain and patient satisfaction for those who
had completed the exercise program as compared with those with
irregular compliance.44 In the single randomized clinical trial
of conservative management for neurogenic TOS, Taskaynatan
et al. performed a prospective trial to compare results following
cervical traction plus standard treatment (exercise and heat pack
therapy) in 40 patients.45 At 3 weeks there was a similar degree
of improvement in pain and the ability to perform provocative
maneuvers in both groups, but a significant difference in favor of
cervical traction was only regarding numbness scores. The limitations common to these studies therefore reinforce the difficulty
in establishing any form of comparison between different forms
of conservative management and the challenges in assessing outcomes of conservative versus surgical management.

Botulinum Toxin Chemodenervation


In recent years there has been increasing interest in extending conservative management approaches to neurogenic TOS to include
injection of the scalene muscles with botulinum toxin (BTX). This
application of BTX is based on its FDA-approved use in cervical
dystonia, an uncommon condition characterized by severe and
sustained contraction of the neck muscles in which BTX can substantially diminish muscle spasm.46 Based on experience with the
use of scalene/pectoralis muscle blocks in evaluation of patients
with neurogenic TOS, Jordan and colleagues suggested that BTX
might provide a novel treatment strategy based on chemodenervation of muscles thought to be responsible for brachial plexus
nerve root compression.28,47
Because complication rates with BTX for cervical dystonia
can be as high as 10% to 20% due to inadvertent spread of toxin

PMPH_CH89.indd 713

713

into adjacent muscles, Jordan and colleagues combined the use


of EMG and fluoroscopic monitoring to guide ASM injections
in a series of 22 patients with a clinical diagnosis of neurogenic
TOS.28 They found that treatment with lidocaine and steroids
produced only temporary improvement (reduction in symptoms by 50% using a 101-point self-assessment scale), and that
effects lasting for at least 1 month were observed in only four
patients (64%). In contrast, BTX injection extended the beneficial effects of ASM block to a mean duration of 88 days, and
there were no significant side effects of BTX chemodenervation
except for mild transient dysphagia in two cases. Th is favorable experience has led to increasing exploration of US-guided
BTX chemodenervation as a potential treatment for neurogenic
TOS, along with extension of the tissues targeted to include the
middle scalene, subclavius, and pectoralis minor muscles.48,49
Another recent study examined the extent of pain relief following BTX administration in 27 patients with neurogenic TOS that
had previously failed physical therapy.50 In this study, BTX was
directed into the ASM using a single CT-guided injection, with
reassessment of patients at 3 months after treatment. Although
there were substantial reductions in pain using several different measures, it is notable that longer-term follow-up was not
reported and it is unclear how many of the BTX-treated patients
subsequently underwent surgical treatment. Indeed, this is a
crucial issue in assessing whether there are any substantive
effects of BTX in neurogenic TOS, because the effects of BTX
are not typically expected to extend beyond 90 to 120 days and
patients with neurogenic TOS typically have had longstanding symptoms for which a more durable treatment is needed.
To date only one randomized, prospective, controlled study to
compare BTX and placebo has been conducted in patients with
neurogenic TOS, but the results have not yet been reported. It is
therefore premature to advocate the use of BTX in this condition
except for specific individualized situations where a temporary
effect on neurogenic symptoms might be desirable prior to more
defi nitive (surgical) treatment.
Answer: Conservative treatment with physical therapy is recommended as initial management for virtually all patients with
neurogenic TOS, but there remain few studies documenting the
overall response and effectiveness of this approach. Valid comparisons between conservative treatment and surgical approaches are
unlikely, given that these treatments are generally recommended
in different clinical situations and for different patient populations. (Grade B recommendation.) Because the value of treatment
approaches to neurogenic TOS utilizing scalene/pectoralis muscle
injection with BTX remain uncertain and likely temporary at best,
this approach cannot be recommended in the absence of further
evidence. (Grade D recommendation.)

TREATMENT
6. What is the optimal surgical approach for treatment of neurogenic TOS?

Primary Operations for Neurogenic TOS


Surgical thoracic outlet decompression for neurogenic TOS is
generally recommended for patients with disabling symptoms in
whom the clinical diagnosis is well founded, and when appropriate

5/22/2012 6:00:29 PM

714

Surgery: Evidence-Based Practice

conservative management has provided insufficient improvement


in symptoms. The two principal operations currently used for neurogenic TOS are (1) transaxillary first rib resection and partial scalenectomy, and (2) supraclavicular first rib resection and complete
scalenectomy. In each approach, decompression is usually coupled
with removal of a cervical rib when present, along with any anomalous ligaments, fascial bands, aberrant scalene muscles, and/or
scar tissue found encompassing the brachial plexus nerve roots
(external neurolysis).19,51-55 It is crucial to emphasize that severe
injuries can occur during operations for neurogenic TOS, including injury or thrombosis of the axillary and subclavian arteries and
veins, hemothorax, chylothorax, sympathetic nerve injury (resulting in Horners syndrome), and transient or permanent paralysis
of the brachial plexus, long thoracic or phrenic nerves.56-59 These
complications can occur even with experienced surgeons using
either the supraclavicular or transaxillary approaches. Operations
for neurogenic TOS should be considered only when the diagnosis
is certain and the surgeon has considerable experience with the
regional anatomy, and the operation must be conducted with an
adequate approach to achieve good visualization of the relevant
structures, along with meticulous technique.
Successful results for the surgical treatment of neurogenic
TOS have been described in many single-center reports over the
past several decades, yet the majority of these reports involve relatively small numbers of patients.53,60-87 One exceptional report is
a series of 5102 patients who underwent primary thoracic outlet
decompression procedures for all types of TOS over a 50-year
period by Urschel and Kourlis.88 For these patients with neurogenic TOS, 70% to 95% were reported to have improved early
after surgery, with 50% to 90% having sustained improvement at
5-year follow-up. Hempel et al. described a series of 770 consecutive patients undergoing supraclavicular operations for TOS over
a 28-year period, of whom 705 had neurogenic TOS.86 Results were
described as excellent in 59%, good in 27%, fair in 13%, and poor
in 1%. There were no brachial plexus injuries and <1% had other
complications, and only 2% required reoperations for recurrent
TOS. Sanders has also reported an extensive personal experience with greater than 1000 operations for neurogenic TOS, both
in a 1991 textbook and in more recent publications.1,89-91 Overall results in these and other series indicate that approximately
70% to 80% of patients have successful outcomes for at least to
1 to 2 years after operation, with approximately 50% maintaining successful results at 5 to 10 years. Unfortunately, the results
described in these reports represent the outcomes attained by a
few highly experienced surgeons with nationwide referral practices accumulated over long periods of time, making it unlikely
that these results can be more widely translated. Second, the outcome measures used in most of these reports have been confi ned
to general retrospective assessments of the success or failure of
operation, typically evaluated in a subjective, nonsystematic
manner.
To date there has been only one prospective, randomized clinical trial evaluating different surgical approaches to neurogenic
TOS.92 This study compared the outcomes of patients undergoing
either transaxillary first rib resection (n = 25) or supraclavicular scalenotomy and brachial plexus neurolysis (n = 24), with no
difference in postoperative complications but significantly lower
pain and numbness ratings for the transaxillary first rib resection
group. These results were interpreted to indicate improved results

PMPH_CH89.indd 714

for operations including first rib resection; unfortunately, the limited extent of scalene muscle resection and the absence of first rib
resection in the supraclavicular group make it difficult to evaluate the results of this study with more contemporary techniques
for supraclavicular decompression (which include complete
scalenectomy and first rib resection, along with brachial plexus
neurolysis). Although there remain strong advocates and distinct
advantages/disadvantages for each of these operative approaches,
previous studies have shown no consistent differences between
transaxillary and supraclavicular operations with regard to the
rate of postoperative complications, successful relief of symptoms,
or long-term outcomes.

Operations for Recurrent Neurogenic TOS


Patients failing to improve during the first 3 months after an
operation for neurogenic TOS are considered to have persistent
neurogenic TOS (or a treatment-resistant cervical-brachial syndrome). Patients who have initially had a successful result, but
subsequently develop disabling symptoms similar to the original condition, are considered to have recurrent neurogenic TOS.
Published rates of recurrence for patients undergoing surgical
treatment for neurogenic TOS range from 2% to 40%.22,93,94 The
majority of such recurrences appear to occur within the fi rst
2 years after operation, and are due to formation of scar tissue
surrounding and fi xing the brachial plexus nerve roots, reattachment of residual unresected ASM, and nerve root compression
by retained bony structures (e.g., unresected portions of the first
rib). The likelihood of developing recurrent neurogenic symptoms therefore depends in part on the original procedure performed, the use of adjunctive methods to suppress postoperative
fibrosis, and subsequent trauma. In the only prospective study
specifically addressing recurrence rates, Altobelli et al. reported
a recurrence rate of 40% in patients undergoing transaxillary
fi rst rib resection, for whom subsequent treatment was provided
by a supraclavicular approach. Another study of reoperations for
neurogenic TOS indicated that fibrotic scar tissue surrounding
the brachial plexus and incomplete scalenectomy were the most
frequent causes of recurrence. In addition, this study and others
have emphasized the importance of pectoralis minor tenotomy
in treating recurrent neurogenic TOS, as well as treatment of
this condition in patients undergoing primary thoracic outlet
decompression procedures by either transaxillary or supraclavicular approaches. Thus, although ongoing conservative
therapy with physical therapy, pain management, trigger point
injections, and other techniques is essential in the management
of recurrent neurogenic TOS, carefully selected patients may be
candidates for reoperative treatment. The clinical reports available indicate that such reoperations can be surprisingly successful, with rates of improvement comparable to those for primary
operations.1,89-91
Answer: Thoracic outlet decompression for neurogenic TOS
may be effectively and safely performed by either transaxillary or
supraclavicular approaches, with no high-quality evidence indicating any significant differences between these two approaches
in early or long-term outcomes. Valid comparisons between different operations are nonetheless complicated by substantial
variations in technique, as well as a lack of consistent criteria for
reporting results. Limited observations indicate that reoperations

5/22/2012 6:00:29 PM

Thoracic Outlet Syndromes

for recurrent neurogenic TOS can also be equally effective, but the
optimal approach must be individualized based on the previous
procedure(s) performed and other patient-specific factors. (Grade C
recommendation.)

TREATMENT
7. What is the optimal treatment strategy for venous TOS?
Conservative management of venous TOS and subclavian vein
effort thrombosis with long-term anticoagulation is associated
with chronic arm swelling and significant functional limitations
in approximately 50% to 70% of patients, thereby leading many to
favor surgical approaches.95-100 The introduction of thrombolytic
therapy in the 1980s offered a new strategy, based on early subclavian vein thrombolysis followed at various intervals by thoracic outlet decompression. The more recent use of endovascular
techniques (i.e., balloon angioplasty) has added to the treatment
options available following surgical decompression, while at the
same time surgical techniques have become more refined. It is
notable that utilization of endovascular stents following subclavian vein thrombolysis, in the absence of surgical decompression,
is associated with extremely high failure rates and is not recommended in current practice.101-103
Unfortunately, analysis of outcomes for current surgical
treatment of venous TOS is complicated by the recognition that
not all patients presenting with this condition are clinically equivalent, as some may have either stenosis or occlusion of the subclavian vein, with or without superimposed acute thrombosis, and
others may have only intermittent (exertional) symptoms of venous
congestion with positional subclavian vein obstruction. Another
difficulty that complicates comparisons between clinical series is
that there are currently several different treatment algorithms and
operative approaches used in the management of venous TOS.108-124
Each of these strategies has its proponents and advantages/disadvantages, and excellent results have been reported for each of
these approaches in retrospective single-center studies. However,
it remains difficult to achieve any meaningful comparison of outcomes for the various treatment approaches to venous TOS, due
to relatively small numbers of patients at any single center and the
absence of consistent measures used to judge long-term results.
Molina et al. recently described treatment of a consecutive
series of 114 patients with acute subclavian vein thrombosis, of
whom 97 were seen within 2 weeks of the onset of symptoms and
17 were referred a mean of 34 days after previous treatment elsewhere.104 All patients underwent subclavian vein thrombolysis
followed by a one-stage operation including subclavicular first rib
resection, scalenectomy, and resection of the subclavius muscle
and tendon, along with vein patch angioplasty of the stenotic segment of the subclavian vein. All of the patients presenting in the
early group had restoration of subclavian vein patency and normal
flow, although 7 subsequently required balloon angioplasty and
stent placement. The surgical approach used in this study was less
successful in patients presenting later than 2 weeks after symptoms, in whom chronic subclavian vein obstruction had developed due to progressive vein wall fibrosis; only 5 of these patients
were considered operable and the remaining 12 remained
disabled. Although this report indicates that outstanding results

PMPH_CH89.indd 715

715

can be obtained with urgent surgical management of subclavian vein effort thrombosis, it fails to address the more usual
clinical scenario in which diagnosis and referral of patients with
venous TOS is often delayed more than 2 weeks after the onset of
symptoms.
More recently, our group at Washington University in St.
Louis demonstrated that a similarly high degree of success can be
achieved in more typical patients with venous TOS, even in those
with chronic subclavian vein occlusion.13 This was accomplished
using a comprehensive surgical approach based on paraclavicular thoracic outlet decompression, consisting of scalenectomy and
complete first rib resection, along with frequent use of direct subclavian vein reconstruction by either patch angioplasty or interposition vein bypass. In this series of 36 highly competitive athletes
presenting after a mean interval of 20.2 5.6 days between symptoms and a definitive venographic diagnosis (range, 1120 days),
all patients recovered sufficiently to have symptom-free and unrestricted use of the upper extremity. Moreover, the median interval
between operation and the return to participation in competitive
athletics was 3.5 months (range, 210 months), and the overall
duration of management was significantly correlated with the
time interval from venographic diagnosis to operation. This study
thereby reinforces the notion that optimal outcomes for venous
TOS are obtained with early diagnosis and comprehensive individualized surgical management, and demonstrates one approach
by which almost all patients with symptomatic venous TOS can be
successfully treated.
Answer: Early thrombolytic therapy is preferred for patients
with venous TOS and subclavian vein effort thrombosis, but treatment with chronic anticoagulation alone is not as effective as that
with surgical approaches. Several different surgical approaches
are currently used for venous TOS, but there are no suitable studies by which to make valid comparisons between approaches, and
excellent outcomes are generally obtained. Results from studies
in which thoracic outlet decompression with first rib resection is
coupled with direct subclavian vein reconstruction appear to be
among the best described, and such approaches can be offered to
almost all patients with venous TOS. (Grade B recommendation.)

TREATMENT
8. What are the functional outcomes and quality of life following surgical treatment for neurogenic TOS?
One of the major gaps in the literature on neurogenic TOS is the
absence of prospective quantitative data regarding functional
outcomes and quality of life (QOL) in patients undergoing surgical treatment. Freischlag and colleagues have begun to address
this issue, with the recent completion of a prospective, observational, clinical study to quantify the degree and characteristics of
disability in patients with neurogenic and venous TOS.105 They
used two validated patient-reported QOL instruments, the Medical Outcomes Study Short Form-12 (SF-12) and the Disabilities
of the Arm, Shoulder, and Hand (DASH) survey, which were
completed on initial presentation and during postoperative office
visits up to 24 months after operation.106,107 There were 30 women
and 9 men with a mean age of 39 years, and the median preoperative duration of symptoms was 36 months (range, 4 months to

5/22/2012 6:00:29 PM

716

Surgery: Evidence-Based Practice

45 years). Twenty-nine patients (74%) had symptoms secondary


to occupations or activities that required repetitive motion, and
11 (28%) experienced symptoms as a result of an injury (workrelated, motor vehicle collision, and/or head trauma). One-third
of the patients had undergone previous surgery related to upper
extremity symptoms, including procedures involving the cervical
spine, shoulder, or elbow. Preoperative treatment had included
generalized physical therapy in 33 (85%), TOS-specific physical therapy in 21 (54%), and scalene muscle blocks in 30 (77%)
patients. All patients were considered to have substantial ongoing
disability from neurogenic TOS and to have failed appropriate
attempts at conservative management before surgical treatment
was recommended.
All patients in this study underwent transaxillary thoracic
outlet decompression consisting of first rib resection and partial scalenectomy. There were neither brachial plexus injuries
nor any patients with long thoracic nerve or phrenic nerve palsy.
Postoperative physical therapy was initially prescribed for 2
months for all patients, and continued on an individual basis as
needed for an average of 5.3 months (range, 218 months). Secondary interventions for recurrent symptoms occurring during the follow-up period included additional physical therapy
(10 patients, 26%; beginning an average of 15 months after surgery); botulinum toxin injections (8 patients, 21%; typically performed at 1 year after surgery); and/or referral for comprehensive
pain management (9 patients, 23%; typically within the first year
after surgery).
Fourteen patients (36%) were disabled or unemployed upon
initial presentation, whereas 26 (67%) eventually returned to fulltime work or activity at some time during the 24-month period
of follow-up. The time-course of the return to full-time work or
activity revealed that half of the patients had returned to full-time
work or activity by 3 months after surgery, and more than 75%
by 6 months. There was no significant difference in the time to
return to work between patients with neurogenic TOS and a parallel cohort of patients with venous TOS.
Immediately prior to surgery, the mean SF-12 Physical Component Score (PCS) value for patients with neurogenic TOS was
34.2, which is approximately 1.5 standard deviations below the
population norm (lower than at least 80% of the population). Th is
is worse than PCS values reported for men with chronic prostatitis and similar to values reported for patients with chronic heart
failure. The baseline PCS values for patients with neurogenic
TOS were also significantly lower than those in patients with
venous TOS. The mean preoperative SF-12 Mental Component
Score (MCS) values in patients with neurogenic TOS was not significantly different from the normal population or patients with
venous TOS. In accord with the results of the SF-12 surveys, the
mean baseline DASH score in patients with neurogenic TOS was
51.0, somewhat worse than the mean DASH score of 43.7 reported
for patients with rotator cuff tears, and also significantly worse
than the mean baseline DASH score of 25.0 in patients with venous
TOS (P < .001). During postoperative follow-up, SF-12 PCS scores
for patients with neurogenic TOS improved by an average of 0.28
points per month (P < .001), and SF-12 MCS scores improved
an average of 0.16 points per month (P = .03). DASH scores for
patients with neurogenic TOS also improved an average of
0.99 points per month (P < .001). Importantly, the median time for
patients with neurogenic TOS to recover normal QOL measures

PMPH_CH89.indd 716

was as long as 25 months for physical function and up to 12 months


for mental function. It is notable that these recovery times were
significantly longer than the time to return to full-time work or
activity in the same patients, suggesting that many patients may
be able to return to work or usual activities before achieving full
recovery of QOL.
Answer: Initial QOL studies indicate that the long-term
results for surgical treatment of neurogenic TOS are probably
considerably better than commonly understood or described in
the literature, but this is dependent on relatively long periods of
recovery and ongoing patient management. The application of
current QOL instruments for longitudinal assessment is feasible
and valid in this patient population (i.e., internally consistent,
reliable, and responsive to change) and can yield a great deal of
new information regarding the long-term outcomes of patients
with neurogenic TOS. (Grade B recommendation.)

TREATMENT
9. What are the outcomes of treatment for arterial TOS?
Arterial TOS is characterized by occlusion or poststenotic dilatation and aneurysm formation of the subclavian artery, almost
always in association with a cervical rib or rudimentary first rib;
such patients typically present with a pulsatile neck mass and/or
symptoms of arterial thromboembolism to the arm, often with
evidence of digital ischemia and vasospasm. Although the diagnosis of arterial TOS requires arteriography, this can also underestimate the extent of luminal irregularity. Prompt operative
intervention is usually indicated, with scalenectomy and resection
of the cervical and first ribs and vascular reconstruction when
necessary to correct or prevent occlusion of the arterial supply to
the hand due to repeated embolization. In most cases this requires
direct subclavian artery repair with creation of an interposition
bypass graft.
Although there are limited published studies focusing on
arterial TOS, a 1991 review of the literature by Sanders and Haug
summarized experience with a total of 200 patients.14 Good results
from treatment were reported in 84% of 137 patients reported
between 1970 and 1991, with 3% requiring amputation and 3% suffering cerebral emboli. Significant delays in recognition and treatment were identified as the primary reasons for the most severe
arterial complications. More recent series reinforce the conclusions
that severe upper limb ischemia is frequently the initial presentation in young patients with arterial TOS, and that almost all patients
with subclavian aneurysms exhibit associated bony abnormalities. There remain several unresolved questions regarding optimal
treatment for patients with arterial TOS, as well as significant gaps
in our understanding of long-term outcomes and QOL in this rare
patient population.
Answer: Prompt surgical treatment with thoracic outlet
decompression and interposition bypass graft reconstruction is
recommended for all patients with arterial TOS and subclavian
artery aneurysms, regardless of size or the presence of symptoms,
with a high rate of excellent functional outcome. Delays in diagnosis and/or treatment may be associated with distal embolism
and hand ischemia, complicating treatment and diminishing
functional outcomes. (Grade B recommendation.)

5/22/2012 6:00:29 PM

Thoracic Outlet Syndromes

717

Clinical Question Summary


Question

Answer

1 What adjunctive studies can be useful


in diagnosing neurogenic TOS?

Scalene muscle block.


Median antebrachial cutaneous NCV.
Contrast-enhanced brachial plexus MRI

II
IIB
III

B
C
D

2 What are the best tests for diagnosis


the vascular types of TOS?

Contrast venography for venous TOS.


Contrast arteriography for arterial TOS.

II
II

A
A

3 How effective is physical therapy for


neurogenic TOS?

Should be tried in all patients.

III

4 Should botox anterior scalene


injection be tried in neurogenic
TOS?

Not enough data are available.

III

5 Is the transaxillary or supraclavicular


approach better for neurogenic
TOS?

They are in a state of equipoise.

II

6 What is the optimal treatment


strategy for venous TOS?

Early thrombolysis followed by surgical


decompression with or without vein
repair or later endogenous treatment.

II

REFERENCES
1. Sanders RJ. Thoracic Outlet Syndrome: A Common Sequelae of
Neck Injuries. Philadelphia, PA: J. B. Lippincott Company; 1991.
2. Melby SJ, Thompson RW. Diseases of the great vessels and the
thoracic outlet. In: Norton JA, Bollinger RR, Chang AE, et al.,
eds. Surgery: Scientific Principles and Current Practice. 2nd ed.
New York, NY: Springer Verlag; 2008:1375-1395.
3. Chang DC, Lidor AO, Matsen SL, Freischlag JA. Reported inhospital complications following rib resections for neurogenic
thoracic outlet syndrome. Ann Vasc Surg. 2007;21(5):564-570.
4. Thompson RW, Driskill M. Thoracic outlet syndrome: neurogenic. In: Cronenwett JL, Johnston KW, eds. Rutherfords Vascular Surgery. 7th ed. Philadelphia, PA: Elsevier; 2010:1878-1898.
5. Roos DB. Congenital anomalies associated with thoracic outlet
syndrome. Am J Surg. 1976;132:771-778.
6. Juvonen T, Satta J, Laitala P, Luukkonen K, Nissinen J. Anomalies
at the thoracic outlet are frequent in the general population. Am
J Surg. 1995;170(1):33-37.
7. Makhoul RG, Machleder HI. Developmental anomalies at the
thoracic outlet: an analysis of 200 consecutive cases. J Vasc Surg.
1992;16(4):534-542.
8. Sanders RJ, Hammond SL. Management of cervical ribs and
anomalous first ribs causing neurogenic thoracic outlet syndrome.
J Vasc Surg. 2002;36(1):51-56.
9. Machleder HI, Moll F, Verity MA. The anterior scalene muscle in
thoracic outlet compression syndrome: histochemical and morphometric studies. Arch Surg. 1986;121(10):1141-1144.
10. Sanders RJ, Jackson CG, Banchero N, Pearce WH. Scalene muscle
abnormalities in traumatic thoracic outlet syndrome. Am J Surg.
1990;159(2):231-236.
11. Tubbs RS, Louis RGJ, Wartmann CT, et al. Histopathological
basis for neurogenic thoracic outlet syndrome. J Neurosurg Spine.
2008;8:347-351.
12. Molina JE. Need for emergency treatment in subclavian vein
effort thrombosis. J Am Coll Surg. 1995;181(5):414-420.

PMPH_CH89.indd 717

Level of Evidence

Grades of
Recommendation

13. Melby SJ, Vedantham S, Narra VR, et al. Comprehensive surgical


management of the competitive athlete with effort thrombosis
of the subclavian vein (Paget-Schroetter syndrome). J Vasc Surg.
2008;47(4):809-820.
14. Sanders RJ, Haug C. Review of arterial thoracic outlet syndrome
with a report of five new instances. Surg Gynecol Obstet. 1991;
173:415-425.
15. Durham JR, Yao JS, Pearce WH, Nuber GM, McCarthy WJr. Arterial injuries in the thoracic outlet syndrome. J Vasc Surg. 1995;
21:57-69.
16. Nehler MR, Taylor LMJ, Moneta GL, Porter JM. Upper extremity
ischemia from subclavian artery aneurysm caused by bony abnormalities of the thoracic outlet. Arch Surg. 1997;132:527-532.
17. Criado E, Berguer R, Greenfield L. The spectrum of arterial compression at the thoracic outlet. J Vasc Surg. 2010;52:406-411.
18. Duwayri YM, Emery VB, Driskill MR, et al. Positional compression of the axillary artery causing upper extremity thrombosis
and embolism in the elite overhead throwing athlete. J Vasc Surg.
2011;53:1329-1340.
19. Thompson RW, Petrinec D. Surgical treatment of thoracic outlet
compression syndromes. I. Diagnostic considerations and transaxillary first rib resection. Ann Vasc Surg. 1997;11(3):315-323.
20. Sanders RJ, Hammond SL, Rao NM. Diagnosis of thoracic outlet
syndrome. J Vasc Surg 2007; 46(3):601-604.
21. Emery VB, Rastogi R, Driskill MR, Thompson RW. Diagnosis of
neurogenic thoracic outlet syndrome. In: Eskandari MK, Morasch MD, Pearce WH, eds. Vascular Surgery: Therapeutic Strategies.
Shelton, CT: Peoples Medical Publishing House; 2010:129-148.
22. Ambrad-Chalela E, Thomas GI, Johansen KH. Recurrent neurogenic thoracic outlet syndrome. Am J Surg. 2004;187(4):505-510.
23. Sanders RJ. Pectoralis minor syndrome. In: Eskandari MK, Morasch MD, Pearce WH, Yao JST, eds. Vascular Surgery: Therapeutic
Strategies. Shelton, CT: Peoples Medical Publishing House; 2010:
149-160.
24. Sanders RJ, Rao NM. The forgotten pectoralis minor syndrome: 100 operations for pectoralis minor syndrome alone or

5/22/2012 6:00:29 PM

718

25.

26.

27.

28.

29.

30.

31.

32.
33.

34.

35.

36.

37.

38.
39.
40.

41.
42.

43.

Surgery: Evidence-Based Practice

accompanied by neurogenic thoracic outlet syndrome. Ann Vasc


Surg. 2010;24:701-708.
Povlsen B, Belzberg A, Hansson T, Dorsi M. Treatment for
thoracic outlet syndrome. Cochrane Database Syst Rev. 2010;
20:CD007218.
Jordan SE, Machleder HI. Diagnosis of thoracic outlet syndrome
using electrophysiologically guided anterior scalene blocks. Ann
Vasc Surg. 1998;12(3):260-264.
Jordan SE, Ahn SS, Gelabert HA. Differentiation of thoracic outlet syndrome from treatment-resistant cervical brachial pain syndromes: development and utilization of a questionnaire, clinical
examination and ultrasound evaluation. Pain Physician. 2007;
10(3):441-452.
Jordan SE, Ahn SS, Gelabert HA. Combining ultrasonography and
electromyography for botulinum chemodenervation treatment of
thoracic outlet syndrome: comparison with fluoroscopy and electromyography guidance. Pain Physician. 2007;10(4):541-546.
Seror P. Medial antebrachial cutaneous nerve conduction study,
a new tool to demonstrate mild lower brachial plexus lesions: a
report of 16 cases. Clin Neurophysiol. 2004;115:2316-2322.
Machanic BI, Sanders RJ. Medial antebrachial cutaneous nerve
measurements to diagnose neurogenic thoracic outlet syndrome.
Ann Vasc Surg. 2008;22(2):248-254.
Demondion X, Boutry N, Drizenko A, Paul C, Francke JP, Cotten
A. Thoracic outlet: anatomic correlation with MR imaging. AJR
Am J Roentgenol. 2000;175:417-422.
van Es HW. MRI of the brachial plexus. Eur Radiol. 2001;11:
325-336.
Demondion X, Bacqueville E, Paul C, Duquesnoy B, Hachulla
E, Cotten A. Thoracic outlet: assessment with MR imaging in
asymptomatic and symptomatic populations. Radiology. 2003;
227:461-468.
Charon JP, Milne W, Sheppard DG, Houston JG. Evaluation of
MR angiographic technique in the assessment of thoracic outlet
syndrome. Clin Radiol. 2004;59:588-595.
Demondion X, Herbinet P, Van Sint Jan S, Boutry N, Chantelot
C, Cotten A. Imaging assessment of thoracic outlet syndrome.
Radiographics. 2006;26:1735-1750.
Demirbag D, Unlu E, Ozdemir F, et al. The relationship between
magnetic resonance imaging findings and postural maneuver
and physical examination tests in patients with thoracic outlet
syndrome: results of a double-blind, controlled study. Arch Phys
Med Rehabil. 2007;88(7):844-851.
Wilbourn AJ, Porter JM. Neurogenic thoracic outlet syndrome:
surgical versus conservative therapy. J Vasc Surg. 1992;15(5):
880-882.
Aligne C, Barral X. Rehabilitation of patients with thoracic outlet syndrome. Ann Vasc Surg. 1992;6(4):381-389.
Walsh MT. Therapist management of thoracic outlet syndrome.
J Hand Ther. 1994;7(2):131-144.
Novak CB, Collins ED, Mackinnon SE. Outcome following conservative management of thoracic outlet syndrome. J Hand Surg
Am. 1995;20(4):542-548.
Novak CB. Conservative management of thoracic outlet syndrome. Semin Thorac Cardiovasc Surg. 1996;8(2):201-207.
Edgelow P. Neurovascular consequences of cumulative trauma
disorders affecting the thoracic outlet: a patient centered treatment approach. In: Donatelli R, ed. Physical Therapy of the
Shoulder. New York: Churchill Livingston; 2004:205-238.
Lindgren KA. Conservative treatment of thoracic outlet syndrome: a 2-year follow-up. Arch Phys Med Rehabil. 1997;78(4):
373-378.

PMPH_CH89.indd 718

44. Glbahar S, Akalin E, Baydar M, Sahin E, Manisali M, Kizil R.


Regular exercise improves outcome in droopy shoulder syndrome:
a subgroup of thoracic outlet syndrome. J Muscle Pain. 2005;
13(4):21-26.
45. Taskaynatan MA, Balaban B, Yasar E, Ozgul A, Kalyon TA. Cervical traction in conservative management of thoracic outlet syndrome. J Muscle Pain. 2004;15:89-94.
46. Swope D, Barbano R. Treatment recommendations and practical
applications of botulinum toxin treatment of cervical dystonia.
Neurol Clin. 2008;26 Suppl 1:54-65.
47. Jordan SE, Ahn SS, Freischlag JA, Gelabert HA, Machleder HI.
Selective botulinum chemodenervation of the scalene muscles
for treatment of neurogenic thoracic outlet syndrome. Ann Vasc
Surg. 2000;14(4):365-369.
48. Torriani M, Gupta R, Donahue DM. Sonographically guided
anesthetic injection of anterior scalene muscle for investigation of neurogenic thoracic outlet syndrome. Skeletal Radiol.
2009;38(11):1083-1087.
49. Torriani M, Gupta R, Donahue DM. Botulinum toxin injection
in neurogenic thoracic outlet syndrome: results and experience using a ultrasound-guided approach. Skeletal Radiol. 2010;
39:973-980.
50. Christo PJ, Christo DK, Carinci AJ, Freischlag JA. Single CTguided chemodenervation of the anterior scalene muscle with
botulinum toxin for neurogenic thoracic outlet syndrome. Pain
Med. 2010;11(4):504-511.
51. Roos DB. Transaxillary approach for first rib resection to relieve
thoracic outlet syndrome. Ann Surg. 1966;163:354-358.
52. Hempel GK, Rusher AH Jr., Wheeler CG, Hunt DG, Bukhari HI.
Supraclavicular resection of the first rib for thoracic outlet syndrome. Am J Surg. 1981;141(2):213-215.
53. Reilly LM, Stoney RJ. Supraclavicular approach for thoracic outlet decompression. J Vasc Surg. 1988;8:329-334.
54. Thompson RW, Petrinec D, Toursarkissian B. Surgical treatment of
thoracic outlet compression syndromes. II. Supraclavicular exploration and vascular reconstruction. Ann Vasc Surg. 1997;11(4):
442-451.
55. Sanders RJ, Hammond SL. Supraclavicular fi rst rib resection
and total scalenectomy: technique and results. Hand Clin. 2004;
20:61-70.
56. Dale A. Thoracic outlet compression syndrome: critique in 1982.
Arch Surg. 1982;117:1437-1145.
57. Wilbourn AJ. Thoracic outlet syndrome surgery causing severe
brachial plexopathy. Muscle Nerve. 1988;11(1):66-74.
58. Melliere D, Becquemin JP, Etienne G, Le Cheviller B. Severe injuries resulting from operations for thoracic outlet syndrome: can
they be avoided? J Cardiovasc Surg (Torino). 1991;32(5):599-603.
59. Leffert RD. Complications of surgery for thoracic outlet syndrome. Hand Clin 2004;20(1):91-98.
60. Sanders RJ, Monsour JW, Baer SB. Transaxillary first rib resection for the thoracic outlet syndrome. Arch Surg. 1968;97:
1014-1023.
61. Roeder DK, Mills M, McHale JJ, Shepard BM, Ashworth HE.
First rib resection in the treatment of thoracic outlet syndrome:
transaxillary and posterior thoracoplasty approaches. Ann Surg.
1973;178:49-52.
62. Hoofer WD, Burnett AD. Thoracic outlet relief. J Kansas Med Soc.
1973;74:329-331.
63. Dale WA. Management of thoracic outlet syndrome. Ann Surg.
1975;181:575-585.
64. Kremer RM, Ahlquist REJ. Thoracic outlet compression syndrome. Am J Surg. 1975;130:612-616.

5/22/2012 6:00:30 PM

Thoracic Outlet Syndromes

65. McGough EC, Pearce MB, Byrne JP. Management of thoracic


outlet syndrome. J Ther Card Med. 1979;77:169-174.
66. Youmans CRJ, Smiley RH. Thoracic outlet syndrome with negative Adsons and hyperabduction maneuvers. Vasc Surg. 1980;
14:318-329.
67. Roos DB. The place for scalenectomy and first-rib resection in
thoracic outlet syndrome. Surgery. 1982;92:1077-1085.
68. Batt M, Griffet J, Scotti L, LeBas P. Le syndrome de la transversee
cervico-brachiale: a proposde 112 cas: vers une attitude tactique
plus nuancee. J Chir Paris. 1983;120:687-691.
69. Sallstrom J, Gjores JE. Surgical treatment of the thoracic outlet
syndrome. Acta Chir Scand. 1983;149:555-560.
70. Heughan C. Thoracic outlet syndrome. Can J Surg. 1984;27:
35-36.
71. Qvarfordt PG, Ehrenfeld WK, Stoney RJ. Supraclavicular radical scalenectomy and transaxillary first rib resection for the thoracic outlet syndrome: a combined approach. Am J Surg. 1984;
148(1):111-116.
72. Narakas A, Bonnard C, Egloff DV. The cervico-thoracic outlet
syndrome. Ann Chir Main. 1986;5:185-207.
73. Takagi K, Yamaga M, Morisawa K, Kitagawa T. Management
of thoracic outlet syndrome. Arch Orthop Trauma Surg. 1987;
106:78-81.
74. Davies AL, Messerschmidt W. Thoracic outlet syndrome: a therapeutic approach based on 115 consecutive cases. Del Med J. 1988;
60:307.
75. Selke FW, Kelly TR. Thoracic outlet syndrome. Am J Surg. 1988;
156:54-57.
76. Stanton PEJ, Vo NM, Haley T, Shannon J, Evans J. Thoracic outlet syndrome: a comprehensive evaluation. Am Surg. 1988;54:
129-133.
77. Wood VE, Twito R, Verska JM. Thoracic outlet syndrome: the
results of fi rst rib resection in 100 patients. Orthop Clin North
Amer. 1988;19:131-146.
78. Cikrit DF, Haefner R, Nichols WK, Silver D. Transaxillary or
supraclavicular decompression for the thoracic outlet syndrome:
a comparison of the risks and benefits. Am Surgeon. 1989;55:
347-352.
79. Lindgren SHS, Ribbe EB, Norgren LEH. Two year follow-up of
patients operated on for thoracic outlet syndrome: effects on
sick-leave incidence. Eur J Vasc Surg. 1989;3:411-415.
80. Lepantalo M, Lindgren KA, Leino E, et al. Long term outcome
after resection of the fi rst rib for thoracic outlet syndrome. Br J
Surg. 1989;76(12):1255-1256.
81. Jamieson WG, Chinnick B. Thoracic outlet syndrome: fact or fancy?
A review of 409 consecutive patients who underwent operation.
Can J Surg. 1996; 39(4):321-326.
82. Graham GG, Lincoln BM. Anterior resection of the first rib for
thoracic outlet syndrome. Am J Surg. 1973;126:803-806.
83. Thompson JB, Hernandez IA. The thoracic outlet syndrome: a
second look. Am J Surg. 1979;138:251-253.
84. Thomas GI, Jones TW, Stavney LS, Manhas DR. The middle scalene muscle and its contribution to the TOS. Am J Surg. 1983;
145:589-592.
85. Loh CS, Wu AVO, Stevenson IM. Surgical decompression for
thoracic outlet syndrome. J R Coll Surg Edin. 1989;34:66-68.
86. Hempel GK, Shutze WP, Anderson JF, Bukhari HI. 770 consecutive supraclavicular first rib resections for thoracic outlet syndrome. Ann Vasc Surg. 1996;10(5):456-463.
87. Axelrod DA, Proctor MC, Geisser ME, Roth RS, Greenfield LJ.
Outcomes after surgery for thoracic outlet syndrome. J Vasc Surg.
2001;33(6):1220-1225.

PMPH_CH89.indd 719

719

88. Urschel HC, Kourlis H. Thoracic outlet syndrome: a 50-year


experience at Baylor University Medical Center. Proc Bayl Univ
Med Ctr. 2007;20:125-135.
89. Sanders RJ, Pearce WH. The treatment of thoracic outlet syndrome: a comparison of different operations. J Vasc Surg. 1989;10:
626-634.
90. Sanders RJ, Haug CE, Pearce WH. Recurrent thoracic outlet
syndrome. J Vasc Surg. 1990;12(4):390-398.
91. Sanders RJ, Hammond SL, Rao NM. Observations on the use
of seprafi lm on the brachial plexus in 249 operations for neurogenic thoracic outlet syndrome. Hand. 2007;2:179-183.
92. Sheth RN, Campbell JN. Surgical treatment of thoracic outlet
syndrome: a randomized trial comparing two operations.
J Neurosurg Spine. 2005;3(5):355-363.
93. Cheng SW, Stoney RJ. Supraclavicular reoperation for neurogenic thoracic outlet syndrome. J Vasc Surg. 1994;19(4):565-572.
94. Altobelli GG, Kudo T, Haas BT, Chandra FA, Moy JL, Ahn
SS. Thoracic outlet syndrome: pattern of clinical success after
operative decompression. J Vasc Surg. 2005;42(1):122-128.
95. Adams JT, McEvoy RK, DeWeese JA. Primary deep venous
thrombosis of the upper extremity. Arch Surg. 1965;91:29-42.
96. Tilney NL, Griffiths HJG, Edwards E. Natural history of major
venous thrombosis of the upper extremity. Arch Surg. 1970;
101:792-795.
97. Donayre CE, White GH, Mehringer SM, Wilson SE. Pathogenesis determines late morbidity of axillosubclavian vein thrombosis. Am J Surg. 1986;152(2):179-184.
98. Gloviczki P, Kazmier FJ, Hollier LH. Axillary-subclavian
venous occlusion: the morbidity of a nonlethal disease. J Vasc
Surg. 1986;4(4):333-337.
99. Lindblad B, Tengborn L, Bergqvist D. Deep vein thrombosis
of the axillary-subclavian veins: epidemiologic data, effects of
different types of treatment and late sequelae. Eur J Vasc Surg.
1988;2:161-165.
100. Aziz S, Straehley CJ, Whelan TJ, Jr. Effort-related axillosubclavian vein thrombosis: a new theory of pathogenesis and a plea
for direct surgical intervention. Am J Surg. 1986;152(1):57-61.
101. Meier GH, Pollak JS, Rosenblatt M, Dickey KW, Gusberg RJ.
Initial experience with venous stents in exertional axillarysubclavian vein thrombosis. J Vasc Surg. 1996;24(6):974-981.
102. Rutherford RB. Primary subclavian-axillary vein thrombosis:
the relative roles of thrombolysis, percutaneous angioplasty,
stents, and surgery. Semin Vasc Surg. 1998;11(2):91-95.
103. Urschel HC, Jr., Patel AN. Paget-Schroetter syndrome therapy:
failure of intravenous stents. Ann Thorac Surg. 2003;75(6):
1693-1696.
104. Molina JE, Hunter DW, Dietz CA. Paget-Schroetter syndrome
treated with thrombolytics and immediate surgery. J Vasc Surg.
2007;45(2):328-334.
105. Chang DC, Rotellini-Coltvet L, Mukherjee D, DeLeon R, Freischlag JA. Surgical intervention for thoracic outlet syndrome
improves patients quality of life. J Vasc Surg. 2009;49:630-635.
106. Ware J, Jr., Kosinski M, Keller SD. A 12-Item Short-Form
Health Survey: construction of scales and preliminary tests of
reliability and validity. Med Care. 1996;34(3):220-233.
107. Hudak PL, Amadio PC, Bombardier C. Development of an
upper extremity outcome measure: the DASH (disabilities of the
arm, shoulder and hand). The Upper Extremity Collaborative
Group (UECG). Am J Ind Med. 1996;29(6):602-608.
108. Kunkel JM, Machleder HI. Treatment of Paget-Schroetter
syndrome: a staged, multidisciplinary approach. Arch Surg.
1989;124(10):1153-1157.

5/22/2012 6:00:30 PM

720

Surgery: Evidence-Based Practice

109. Machleder HI. Evaluation of a new treatment strategy for PagetSchroetter syndrome: spontaneous thrombosis of the axillarysubclavian vein. J Vasc Surg. 1993;17(2):305-315.
110. Lee WA, Hill BB, Harris EJ, Jr., Semba CP, Olcott CI. Surgical
intervention is not required for all patients with subclavian
vein thrombosis. J Vasc Surg. 2000;32(1):57-67.
111. Lee JT, Karwowski JK, Harris EJ, Haukoos JS, Olcott Ct. Longterm thrombotic recurrence after nonoperative management of
Paget-Schroetter syndrome. J Vasc Surg. 2006;43(6):1236-1243.
112. Urschel HC Jr., Razzuk MA. Improved management of the
Paget-Schroetter syndrome secondary to thoracic outlet
compression. Ann Thorac Surg. 1991;52(6):1217-1221.
113. Angle N, Gelabert HA, Farooq MM, et al. Safety and efficacy
of early surgical decompression of the thoracic outlet for PagetSchroetter syndrome. Ann Vasc Surg. 2001;15(1):37-42.
114. Caparrelli DJ, Freischlag J. A unified approach to axillosubclavian venous thrombosis in a single hospital admission. Semin
Vasc Surg. 2005;18(3):153-157.
115. Urschel HC, Jr., Razzuk MA. Paget-Schroetter syndrome:
what is the best management? Ann Thorac Surg. 2000;69(6):
1663-1668.
116. Schneider DB, Dimuzio PJ, Martin ND, et al. Combination
treatment of venous thoracic outlet syndrome: open surgical
decompression and intraoperative angioplasty. J Vasc Surg.
2004;40(4):599-603.
117. Kunkel JM, Machleder HI. Spontaneous subclavain vein thrombosis: a successful combined approach of local thrombolytic

PMPH_CH89.indd 720

118.

119.

120.

121.

122.

123.

124.

therapy followed by first rib resection. Surgery. 1989;


106(1):114.
Kreienberg PB, Chang BB, Darling RC 3rd, et al. Long-term
results in patients treated with thrombolysis, thoracic inlet
decompression, and subclavian vein stenting for Paget-Schroetter
syndrome. J Vasc Surg. 2001;33(2 Suppl):S100-105.
Divi V, Proctor MC, Axelrod DA, Greenfield LJ. Thoracic outlet
decompression for subclavian vein thrombosis: experience in
71 patients. Arch Surg. 2005;140(1):54-57.
Thompson RW, Schneider PA, Nelken NA, Skioldebrand CG,
Stoney RJ. Circumferential venolysis and paraclavicular thoracic outlet decompression for effort thrombosis of the subclavian vein. J Vasc Surg. 1992;16(5):723-732.
Azakie A, McElhinney DB, Thompson RW, Raven RB, Messina
LM, Stoney RJ. Surgical management of subclavian vein effort
thrombosis secondary to thoracic outlet compression. J Vasc
Surg. 1998;28:777-786.
Cassada DC, Lipscomb AL, Stevens SL, Freeman MB, Grandas
OH, Goldman MH. The importance of thrombophilia in the
treatment of Paget-Schroetter syndrome. Ann Vasc Surg. 2006;
20(5):596-601.
Green RM, Waldman D, Ouriel K, Riggs P, Deweese JA. Claviculectomy for subclavian venous repair: long-term functional
results. J Vasc Surg. 2000;32(2):315-321.
Doyle A, Wolford HY, Davies MG, et al. Management of effort
thrombosis of the subclavian vein: todays treatment. Ann Vasc
Surg. 2007;21(6):723-729.

5/22/2012 6:00:30 PM

Commentary on
Thoracic Outlet Syndromes
Julie Ann Freischlag

Thoracic outlet syndrome is thought to be a rare entity that occurs


in a rare patient. Some physicians do not even believe that neurogenic thoracic outlet is a cause of pain or discomfort in patients,
and therefore they opt to offer only conservative treatments such
as physical therapy to the patients. After reading this complete
review of thoracic outlet syndromes by Robert W. Thompson, the
surgeon who decides to treat these patients can certainly appreciate the spectrum of presentation and the need for an individual
approach for each patient. There is a need for more prospective
studies regarding the treatment of these patients, and a centralized
database would be helpful to achieve that. Long-term follow-up

is virtually unknown in all forms of thoracic outlet syndrome


arterial, neurogenic, and venous.
Dr Thompson has been singularly responsible for uniting physicians, surgeons, and healthcare professionals who treat thoracic
outlet syndrome. He organized and hosted a 2-day symposium at
Washington University in 2009. As more of us publish and discuss
these patients and their prognosis and outcomes, more treatment
options will be on the radar screen of primary-care physicians
who initially see these patients.
If you have knowledge, let others light their candles in it.
(Margaret Fuller)

721

PMPH_CH89.indd 721

5/22/2012 6:00:30 PM

CHAPTER 90

Peripheral Arterial Embolus


Shane OKeeffe and Eric Endean

INTRODUCTION

the extremity. Basic concepts can be applied to other vascular


beds such as the mesenteric vessels.
The original approach in treating patients with acute ischemia
was to surgically remove the thrombus by directly exposing the
occluded vessel.1 Reoperation to remove additional thrombus was
often necessary, and frequently the end result was amputation.
In 1963, Thomas Fogarty reported the use of catheter extraction
for arterial thrombus.2 The use of a balloon catheter to remove an
embolus and associated thrombus revolutionized the treatment of
patients with acute arterial occlusions.
The use of thrombolytic agents is based on an understanding of the inherent fibrinolytic system, a system that is complex
and not completely understood. Thrombus formation is essential
for hemostasis and is precisely controlled through the coagulation
cascade. The final step in the formation of thrombus (clot) is the
conversion of fibrinogen to fibrin, a step catalyzed by thrombin.
Balancing the deposition of thrombus is the fibrinolytic system.
The dissolution of thrombus is catalyzed by plasmin, a proteolytic
enzyme formed from the inactive proenzyme, plasminogen. Plasminogen is produced in the liver and its activation to plasmin
is controlled by both activators and inhibitors (Table 90.1). The

Acute arterial ischemia from an embolus often develops in patients


with otherwise normal arterial flow. Cessation of arterial flow results
in the classic findings of pulselessness, pain, pallor, paresthesias, and
paralysis. Onset of symptoms is usually sudden and often severe.
The nervous tissue is most susceptible to ischemia, and permanent
damage can occur within 2 hours of ischemia. Muscle necrosis can
develop after 4 hours of ischemia. It is therefore crucial to diagnose
and treat patients with an arterial embolus expediently.
A number of clinically relevant questions arise in the treatment
of patients with arterial embolus. What is the role of thrombolytic
therapy for ischemia caused by an embolus? Since a majority of
emboli arise from a cardiac source, specifically in patients with
atrial fibrillation, should all patients with atrial fibrillation be
anticoagulated in an effort to prevent peripheral embolization?
Finally, restoration of flow to profoundly ischemic tissue results in
ischemia-reperfusion syndrome. One manifestation of this syndrome is tissue edema that can result in compartment syndrome,
especially in the lower extremity. Should fasciotomies routinely be
done in patients with arterial embolization at the time of restoration of flow? This chapter will review literature related to these
questions in an attempt to provide clinically relevant answers.

Table 90.1 Activators and Inhibitors of Plasmin


Formation

1. Is thrombolysis indicated for acute arterial ischemia resulting from an embolus?

Activators
Activated factor XII
Tissue activator (t-PA)
Urokinase
Streptokinase-plasminogen complex

When a patient presents with acute ischemia, the practitioner is


often faced with a therapeutic decision: surgical approach versus
thrombolysis for restoration of flow. As with other areas of medicine, there has been an increased emphasis on minimally invasive treatment in vascular surgery. Thrombolysis is one potential
minimally invasive treatment for acute ischemia that may obviate
the need for surgical intervention. The question becomes, what
are the benefits, risks, and outcomes for surgical revascularization
and thrombolysis in the setting of arterial embolization? Acute
ischemia can develop in many and diverse vascular beds; however,
the focus of this discussion will be on treatment of emboli affecting

Inhibitors
2-Antiplasmin
2-Macroglobulin
1-Antitrypsin
Antithrombin III
Complement 2 inhibitor

722

PMPH_CH90.indd 722

5/22/2012 6:01:01 PM

Peripheral Arterial Embolus

primary substrate for plasmin is fibrin or fibrinogen. Plasmin


catalyzes the hydrolysis of the chemical cross-links between
fibrin chains. Plasminogen activators have been used pharmacologically to effect thrombolysis. Commercially available t-PA
has been produced by the Bowes melanoma cells, a cell line that
efficiently produces large amounts of t-PA. Recombinant DNA
technology has also been utilized to produce commercially available t-PA. The half-life of t-PA is estimated to be between 4 and 7
minutes. t-PA does not directly dissolve clot; rather, it must bind
with plasminogen at the site of clot. It also should be emphasized
that plasminogen is nonspecific and can degrade fibrin/fibrinogen
wherever it is found, whether in the circulation or at any site of
intravascular clot. The implication of this fact is that intravenous
administration of t-PA would degrade circulating fibrinogen and
small amounts of drug would come into contact with the end of
the arterial thrombus. For these reasons, a thrombolytic agent
is usually given at the site of thrombosis with catheter-directed,
intrathrombus infusion. This provides the highest concentration
of the activator where it is needed, ideally resulting in better efficacy in thrombus dissolution and less bleeding complications.3
In general, patients who present with acute ischemia have abrupt
onset of symptoms. This is in comparison to patients who develop
thrombosis of underlying disease. Such patients often develop collateral flow, and thrombosis of the diseased vessel does not result in
very severe symptoms. It is therefore imperative for the clinician to
determine the severity of the ischemia. It is recommended that acute
ischemia be classified as Viable (Level I), Threatened (Level IIa
marginally threatened or Level IIbimmediately threatened), and
Irreversible (Level III).4 The severity of ischemia often affects
treatment decisions. It is recommended that the more severely ischemic (but viable) extremities undergo revascularization strategies
to assure that perfusion is quickly re-established.
Three randomized trials investigated thrombolysis versus
surgical thromboembolectomy for patients presenting with
acute arterial ischemia. In the first, Ouriel et al. randomized 114
patients with acute limb ischemia to receive either urokinase (57)
or surgical revascularization (57).5 The duration of the limb ischemia was fewer than 7 days. Embolus was the cause of ischemia
in 11 patients (19%) in the thrombolytic group and in 13 patients
(23%) in the surgical group. At 1-year follow-up, the amputation
rate was 18% in both groups of patients. When evaluating outcomes based on the etiology of the ischemia (embolus vs. thrombosis) the in-hospital results were similar. As a group, there was a
significant (62%) reduction in mortality at 1 year in patients who
underwent thrombolysis. Interestingly, when specifically evaluating patients with embolus as the cause for ischemia, the 1-year
survival of patients appeared to be greater in those patients with
embolic causes and treated with thrombolytics (100%) compared
with patients with embolus treated surgically (51%). The authors
suggest that the improved survival in patients treated with thrombolytics was a reflection of the increased frequency of in-hospital
cardiopulmonary complications in the surgically treated group.
However, use of thrombolytics was not without complications.
They noted that approximately 20% of patients undergoing thrombolysis had hemorrhagic complications, half of which were major.
One patient died of intracranial hemorrhage.
The Surgery versus Th rombolysis for Ischemia of the Lower
Extremity (STILE) trial is often quoted as demonstrating benefit
in use of thrombolytics for acute limb ischemia.6 A total of 393
patients with lower extremity ischemia were randomized either to

PMPH_CH90.indd 723

723

catheter-directed thrombolysis (using urokinase or recombinant


tissue plasminogen activator) or to surgical revascularization.
Entry into the study required patients to have ischemia of less
than 6 months duration and as such, included patients with
chronic ischemia. Results of the entire group demonstrated that
a higher percentage of patients receiving thrombolysis compared
with the surgery group had ongoing ischemia (54% vs. 25.7%,
respectively; P < .001) and a worse composite clinical outcome
(61.7% vs. 36.1%, respectively; P < .001). These fi ndings most
likely reflect that many patients entering the study had chronic
ischemia as opposed to acute arterial occlusions. When patients
were stratified according to the length of ischemia, those with
acute ischemia (defined as ischemia present for less than 14-days
duration) had a better 6-month limb salvage (89% vs. 70%, P = .02)
and amputation-free survival (84.7% vs. 62.5%, P = .01) when
treated with thrombolysis versus open surgical thrombectomy. It
should be noted that an absolute exclusion criteria for inclusion in
this study was acute ischemia caused by an embolic occlusion.
The Th rombolysis or Peripheral Artery Surgery (TOPAS)
trial7 randomized 544 patients with acute limb ischemia to receive
either intra-arterial urokinase (272 patients) or surgical treatment
(272 patients). Most patients had thrombosis of either a bypass
graft or a native artery (464/544, 85%). Embolism was the cause
of acute ischemia in 78 (14%) patients, of which 37 were randomized to urokinase therapy and 41 to surgical therapy. The primary
endpoint was amputation-free survival 6 months after treatment.
Secondary endpoints were: survival free of open surgical procedures, amputation-free survival at 1 year, increase in anklebrachial blood pressure index, and rates of adverse effects of
treatment. Amputation-free survival at 6 months was 71.8% in the
urokinase group and 74.8% in the surgery group (P = .43). Anklebrachial index measurements increased significantly in both
groups (P < .001) with no significant difference between groups
(P = .23). Mortality for the urokinase group and surgery group at
6 months was 16.0% and 12.3%, respectively (P = .22), and at 1
year was 20.0% and 17.0%, respectively (P = .39). Major hemorrhagic complications were higher in the urokinase group (12.5%)
compared with the surgery group (5.5%, P = .005). Similarly,
hemorrhagic complications occurring in an area other than the
vascular access sites was higher in the urokinase group compared
with the surgery group (P = .04). Although the authors discussed
patient characteristics associated with success rates of thrombolysis, no mention was made of embolus as being a factor associated
with success or failure of thrombolysis.
Since the above studies have been reported, other randomized trials have been conducted to determine optimal dose of
thrombolytics, method of infusion, and type of thrombolytic
(systemic vs. intra-arterial, pulse spray vs. infusion, low dose vs.
high dose).8-12 Yet, as in the above studies, most patients are identified as having a graft thrombosis or thrombosis of a native artery.
Little, if any, mention is made regarding ischemia resulting from
embolus. Newer techniques have also been introduced to enhance
thrombolysis. Saline injected through a high-pressure catheter
(AngioJet, Medrad) creates turbulence that causes mechanical
lysis of thrombus. The lysed thrombus and small clot fragments
are aspirated through a separate channel in the catheter. Thrombolytic agents can be added to this catheter system combining both
mechanical and chemical dissolution of thrombus. Segmental
pharmacomechanical thrombolysis has been introduced. A catheter with proximal and distal balloons (Trellis; Bacchus Vascular,

5/22/2012 6:01:01 PM

724

Surgery: Evidence-Based Practice

Santa Clara, CA) is inserted into the clot. Thrombolytic agent is


injected between the two balloons and the catheter between the
balloons rotates to mechanically fragment the clot. Another catheter (EKOS, Bothell, WA) uses ultrasound waves as an adjunct to
thrombolytic agents to accelerate thrombolysis.
Answer: Controlled, randomized studies do not definitively
answer the question. Most patients enrolled in these studies had
acute arterial ischemia related to graft thrombosis or thrombosis
of the native vessel. Thrombolysis requires infusion of the thrombolytic agent over time. Many, if not most, patients with an arterial
embolus have severe ischemia and require immediate restoration
of blood flow. Therefore, the degree of ischemia is a primary determinant of whether thrombolysis should be attempted. It would
appear that thrombolytics are effective against emboli and that if
a patients ischemic symptoms can tolerate 24 to 48 hours required
for thrombolysis, such an approach can be attempted. However,
if the ischemic symptoms are profound and the limb is immediately threatened (Level IIb), surgical therapy may be the most
expedient method to restore perfusion (Level Ia data; Grade B
recommendation).
2. What is the role of chronic anticoagulation for prevention of
embolus in patients with atrial fibrillation?
Emboli can arise from multiple potential sources. Cardiac sources
include atrial fibrillation, acute myocardial infarction, ventricular aneurysm, endocarditis, cardiac tumor (atrial myxoma), and
valvular heart disease. Noncardiac sources include peripheral
aneurysms (aortic, popliteal), mural thrombus in the aorta, and
thrombus in the deep venous system. A thrombus from the deep
venous system can cause arterial emboli when it embolizes and traverses a patent foramen ovale to gain access to the arterial system.
Of all these sources, embolization most often is cardiac in origin,
and of the cardiac causes, atrial fibrillation is the most frequent.13
In atrial fibrillation, thrombus develops in the left atrial appendage
as a result of uncoordinated atrial contractions. Since such patients
are at risk for embolization, it would therefore appear reasonable
to attempt to prevent formation of thrombus in the atrium and/
or embolization of thrombus to the peripheral arterial tree. One
such preventative measure is to cardiovert patients with new onset
atrial fibrillation into normal sinus rhythm. Another strategy is
to treat patients who have atrial fibrillation with anticoagulation.
Since chronic anticoagulation carries risk of bleeding complications, this raises the question as to the risk benefit for chronic
anticoagulation in patients with atrial fibrillation, especially
in an elderly population.
The fi rst issue is the incidence of embolization in patients
with atrial fibrillation. Emboli arising from the atrial appendage
can lodge in any peripheral vascular bedcerebral, mesenteric/
renal, extremity. Much of the literature that addresses use of anticoagulation uses stroke as an endpoint. Since stroke caused by
embolization of thrombus from the atrium is similar to embolization to an extremity, extrapolation of these results can provide
information regarding use of anticoagulation for the prevention of peripheral arterial embolus. One such study by Nieuwlatt
et al. used the Euro Heart Survey to evaluate 1-year prognosis of
patients with atrial fibrillation.14 In 4192 patients evaluated, they
found that 1.6% suffered an ischemic stroke and 4.3% had some
thromboembolic complication. In the latter group of patients, 61%
were on oral anticoagulation at baseline; at 1 year, anticoagulation
had been discontinued in 16% of the patients, but started in 19%.

PMPH_CH90.indd 724

Thus the incidence of embolization and the effect that anticoagulation has on embolization is not clear.
A number of strategies have been proposed to predict risk of
stroke in patients with atrial fibrillation. Using a National Registry of Atrial Fibrillation (NRAF) that consisted of 1733 Medicare
patients who had nonrheumatic atrial fibrillation and were not
treated with warfarin, Gage et al. described a simple stroke-risk
scheme (CHADS2) that quantified the risk for stroke.15 In this
scheme, patients were assigned one point for presence of congestive heart failure (C), hypertension (H), age of 75 or greater (A),
and diabetes mellitus (D). Two points were assigned for patients
with a past history of stroke or transient ischemic attack (S2).
A CHADS2 score of 0 had an adjusted stroke rate of 1.9%/year.
As the CHADS2 score increased, the adjusted stroke rate also
increased (0 = 1.9; 1 = 2.8; 2 = 4.0; 3 = 5.9; 4 = 8.5; 5 = 12.5;
6 = 18.2).
One study, however, did specifically investigate the incidence
of peripheral thromboembolism in patients with atrial fibrillation.16 Out of 29862 patients studied, 621 (2.1%) had peripheral
arterial embolization. These emboli were distributed in the aorta
(7%), renal artery (2%), mesenteric arteries (29%), pelvic arteries
(9%), and upper or lower extremities (61%). In this study, atrial
fibrillation increased the relative risk for peripheral thromboembolic events by 4.0 in men and 5.7 in women. The authors also
noted that the highest risk occurred in the first year after the
incident diagnosis of atrial fibrillation and declined thereafter. A
meta-analysis by Andersen et al. reviewed 15 randomized controlled trials to determine the efficacy of warfarin in preventing
systemic embolization (to the legs or visceral vessels) in patients
with atrial fibrillation.17 They found that warfarin reduces risk of
stroke and is better than placebo and antiplatelet agents in preventing systemic embolization in patients with atrial fibrillation
(Level I evidence).
Perhaps the greatest concern regarding routine use of anticoagulation is the risk of bleeding complications, especially
intracranial hemorrhage. A case-control study conducted by Fang
et al. showed that the risk for intracranial hemorrhage in elderly
patients was not lower in patients with an International Normalized Ratio (INR) of less than 2.0 compared with those with an
INR between 2.0 and 3.0.18 Oden et al. reviewed the literature from
1980 to 2004 identifying 21,967 patients with atrial fibrillation.19
They performed a record linkage analysis with death hazard estimated as a continuous function of INR and found a U-shaped distribution of risk of death with the lowest risk of death associated
with an INR of 2.4. With high INRs, the risk for death increased
2.3 times for each one-unit increase in INR. Their conclusions
were that an INR of 2.0 to 2.5 provided the lowest risk for stroke
and death in patients with atrial fibrillation.
The practice guidelines of the American College of Cardiology/American Heart Association/European Society of Cardiology
(ACC/AHA/ESC) recommend antithrombotic therapy to prevent
thromboembolism for all patients with AF, except those with
lone AF or contraindications.20 Using the CHADS risk stratification scheme, the ACC/AHA/ESC practice guidelines suggest that
patients with no risk factors be treated with aspirin (81325 mg
daily). Those with one moderate-risk factor (age equal to or greater
than 75, hypertension, heart failure, left ventricular ejection fraction 35% or less or diabetes mellitus) be treated with either aspirin
or warfarin with a target INR of 2.0 to 3.0. Patients with any highrisk factor (prior stroke or embolism, mitral stenosis, or prosthetic

5/22/2012 6:01:02 PM

Peripheral Arterial Embolus

heart valve) or more than one moderate-risk factor should be


treated with warfarin, targeting an INR between 2.0 and 3.0.
Answer: Most of the information regarding use of warfarin
in patients with atrial fibrillation uses stroke as an endpoint. Since
the mechanism of stroke in the face of atrial fibrillation is assumed
to be embolic, it appears appropriate that these studies can be
used to extrapolate recommendations in patients with peripheral
embolization. Patients are at highest risk for embolization with
onset of atrial fibrillation and/or cardioversion to normal sinus
rhythm. With good control of the INR (INR 2.03.0), risk of
bleeding (intracranial hemorrhage) appears to be no higher than
observed in patients with an INR less than 2.0. Therefore, based on
the evidence, anticoagulation with warfarin is recommended for
prevention of embolization (stroke) unless there are contraindications for chronic anticoagulation (Grade A recommendation).
3. Should fasciotomies routinely be done with lower extremity revascularization for acute ischemia resulting from an
embolus?
In general, patients with acute ischemia from an arterial embolus
present with more severe ischemic symptoms than those who
develop thrombosis of an underlying atherosclerotic lesion. This
finding is related to an embolus causing sudden obstruction of
arterial flow in a relatively normal vessel while collateral vessels
have often developed prior to thrombosis when related to an atherosclerotic lesion. With loss of arterial blood supply, the metabolic demand of the tissue quickly exceeds its supply resulting
in tissue hypoxia and microcirculation stasis. At the tissue level,
there is a drop in adenosine triphosphate (ATP). This in turn
results in disturbances of cellular ion homeostasis, activation of
hydrolases, and increased permeability of the cell membrane. One
key event that occurs with ischemia is the conversion of xanthine
dehydrogenase to xanthine oxidase. Xanthine oxidase uses oxygen as an electron acceptor and generates superoxide anions and
hydrogen peroxide during the oxidation of hypoxanthine and
xanthine.21 With reperfusion, oxygen rich blood is supplied to the
tissue resulting in a complex tissue response including formation
of oxygen free radicals, complement activation, development of
tissue edema, vasoconstriction mediated by endothelin, compartment syndrome, and rhabdomyolysis. As a result, restoration of
flow (reperfusion) can exacerbate the injury initiated by ischemia,
and the degree to which it exacerbates the injury is a function of
the duration of ischemia.
It is well recognized that patients with acute arterial occlusion are at risk for the development of compartment syndrome.
The lower leg contains four muscular compartments (superficial
posterior, deep posterior, lateral, and anterior compartments),
and as edema within the compartments develops, the intra-compartment pressure rises.22 Blood flow can be expressed according
to Poiseuilles law, F = Pr4/8L (F = flow, P = pressure difference between two ends of a tube, r = radius of the tube, L = length,
and = viscosity). In the capillary circulation, P is the pressure
difference between the arterial and venous ends of the capillary.
As intracompartmental pressure rises, the extrinsic compartment
pressure is transmitted to the capillary and the pressure difference
between the arterial and venous ends of the capillary decreases.
The compartment pressure also compresses the capillaries reducing the radius of the capillary. (Note that flow is related to the
fourth power of the radius.) Changes caused by increased intracompartmental pressure in both P and diameter, according to

PMPH_CH90.indd 725

725

Poiseuilles law, result in decreased blood flow at the capillary level


within the compartment, with the end result being risk of tissue
ischemia despite restoration of flow at the macrovessel level.
Simply stated, as intracompartmental pressure rises, tissue
perfusion decreases. The tissue most sensitive to ischemia is nervous tissue, with permanent damage developing in 2 hours or less.
Muscle is somewhat more resistant to ischemia, with irreversible
muscle destruction developing after approximately 4 hours of
ischemia. The development of paresthesia and pain followed by
weakness of the muscles is explained by these tissues sensitivity
to blood flow. It is well known that compartment syndrome can be
present despite palpable pulses and Doppler signals. It is also well
established that palpation of the compartment, while suggestive,
is not accurate in determining the presence of compartment syndrome. As a result, measurement of compartment pressures using
modification of a blood pressure manometer,23 the wick catheter
technique,24 or the Stryker manometer25 is imperative to make the
diagnosis of compartment syndrome.
Although compartment pressure measurement has become
an important tool for determining the need for fasciotomy, there is
debate as to what intracompartmental pressure mandates decompression.26 As discussed above, the pathophysiology is related to
muscle perfusion at the capillary level. Some have suggested that
intracompartmental pressures need to be compared with arterial
pressure. McQueen and Court-Brown reported on 116 patients
with tibial fractures, in whom a difference between diastolic
pressure and compartment pressure of 30 mm Hg was used as a
threshold for fasciotomy.27 The authors continuously monitored
compartment pressures for 24 hours postoperatively. Arterial
pressures were recorded at intervals, calculating the pressure difference between diastolic and compartment pressures. In their
series, three fasciotomies were done. No patient had any sequelae of compartment syndrome, leading the authors to the conclusion that their method of monitoring compartment pressures and
their indication for fasciotomy resulted in no missed compartment syndromes. In their study, if an absolute compartment pressure of 30 mm Hg had been used as an indication for fasciotomy,
50 patients would have undergone fasciotomy. A threshold of 40
mm Hg would have resulted in 27 fasciotomies. This study suggests that a dynamic compartment pressure relative to arterial
pressure may be more appropriate as a guide for selecting patients
for fasciotomy as opposed to an absolute compartment pressure
(Level IIb evidence).
Some have advocated prophylactic fasciotomy for patients
at high risk for the development of compartment syndrome. In a
report by Papalambros et al., 43 fasciotomies were performed following 194 procedures done for acute arterial ischemia in the lower
extremity.28 They suggested that fasciotomy should be considered
in patients with an ischemic time of greater than 6 hours, when the
arterial occlusion is precipitous, back-flow is inadequate intraoperatively after thromboembolectomy, and in young patients
without sufficient collateral circulation. Jensen and Sandermann
reported the need for fasciotomy in 8.7% of cases undergoing
revascularization for thromboembolism.29 In a series reporting
on lower extremity fasciotomies done for acute compartment syndrome from all causes, acute revascularization for acute ischemia
was the second most likely etiology (after trauma) consisting of
22.5% of cases.30 They also compared outcome between patients
requiring fasciotomy after a vascular operation and those needing the fasciotomy following trauma. Vascular patients were older,

5/22/2012 6:01:02 PM

726

Surgery: Evidence-Based Practice

had a higher mortality, and less often had a functional leg. Interestingly, they were unable to correlate the severity of the compartment syndrome or its treatment to the clinical outcome.
Another consideration regarding routine fasciotomy is to
consider the potential risks associated with surgical fasciotomy.
Known complications include increased rates of soft tissue infection, renal failure, osteomyelitis, and ultimate amputation.31,32
Bermudez and Knudson found that development of poor calf
muscle function develops following fasciotomy.33 This in turn
can lead to chronic venous insufficiency. Although it is true that
complications can develop after a surgical procedure, it is unclear
whether all of the described complications can be attributed to
the fasciotomy, or if they are a result of the underlying compartment syndrome. Ernst addresses this issue stating, Exposure of
viable muscle after timely fasciotomy has rarely, if ever, resulted

in amputation or serious infectious complications, whereas late


fasciotomy that exposes necrotic muscle has.34
Answer: There are no randomized controlled trials in which
surgical decompression of lower extremity compartments is compared to nonsurgical treatment. Such a study will not be performed
as it would be unethical. It appears that compartment syndrome
develops in approximately 20% of patients who have revascularization following acute arterial ischemia. Because ischemia due
to embolism is more severe, it is possible that the incidence of
compartment syndrome is higher in such patients. The literature
does not support routine or prophylactic fasciotomy in patients
after perfusion has been restored. However, the clinician must
be aware that compartment syndrome is a possible complication
and be prepared to diagnose and decompress the lower extremity
compartments (Grade C recommendation).

Clinical Question Summary


Question

Answer

Level of
Evidence

1 Is thrombolysis
indicated for acute
arterial ischemia
resulting from an
embolus?

Thrombolysis can be effective for dissolution


of emboli, but because it takes time to be
effective, immediate surgical revascularization
may be more appropriate for profound
ischemia (Level IIb).

IIb

5-7

2 What is the role of


chronic anticoagulation
for prevention of
embolus in patients
with atrial fibrillation?

Warfarin should be prescribed for patients with


atrial fibrillation unless there is a specific
contraindication to chronic anticoagulation.

17-20

3 Should fasciotomies
routinely be done
with lower extremity
revascularization for
acute ischemia resulting
from an embolus?

Fasciotomy is not routinely needed following


lower extremity revascularization.

III

26-28

REFERENCES
1. Fogarty T. Historical reflections on the management of acute
limb ischemia. Sem Vasc Surg. 2009;22:3-4.
2. Fogarty TJ, Cranley JJ, Krause RJ, Strasser ES, Hafner CD. A
method for extraction of arterial emboli and thrombi. Surg Gynecol Obstet. 1963;116:241-244.
3. Angle N, Quinones-Baldrich. Thrombolytic therapy for vascular
disease. In: Moore WS, ed. Vascular and Endovascular Surgery.
A Comprehensive Review. 7th ed. Philadelphia, PA: Saunders
Elsevier; 2006:414-442.
4. Rutherford RB, Baker JD, Ernst C, et al. Recommended standards for reports dealing with lower extremity ischemia: revised
version. J Vasc Surg. 1997;26:517-538.
5. Ouriel K, Shortell CK, DeWeese J, et al. A comparison of thrombolytic therapy with operative revascularization in the initial
treatment of acute peripheral arterial ischemia. J Vasc Surg. 1994;
19:1021-1030.
6. The STILE Investigators. Results of a prospective randomized
trial evaluating surgery versus thrombolysis for ischemia of the
lower extremity. The STILE trial. Ann Surg. 1994;220:251-266.
7. Ouriel K, Veith FJ, Sasahara AA, for the Thrombolysis or Peripheral Artery Surgery (TOPAS) Investigators. A comparison of

PMPH_CH90.indd 726

8.

9.

10.

11.

12.

Grade of
Recommendation

References

recombinant urokinase with vascular surgery as initial treatment for acute arterial occlusion of the legs. N Engl J Med. 1998;
338:105-111.
Berridge DC, Gregson RH, Hopkinson BR, Makin GS. Randomized trial of intra-arterial recombinant tissue plasminogen activator, intravenous recombinant tissue plasminogen activator and
intra-arterial streptokinase in peripheral arterial thrombolysis.
Br J Surg. 1991;78:988-995.
Braithwaite BD, Buckenham TM, Galland Rb, Heather BP, Earnshaw JJ. Prospective randomized trial of high-dose bolus versus
low-dose tissue plasminogen activator in the management of
acute limb ischemia. Br J Surg. 1997;84:646-650.
Cragg AH, Smith TP, Corson JD, et al. Two urokinase dose regimens in native arterial and graft occlusions: initial results of a prospective, randomized clinical trial. Radiology. 1991;178:681-686.
Kandarpa K, Chopra PS, Aruny JE, et al. Intraarterial thrombolysis of lower extremity occlusions: prospective, randomized
comparison of forced periodic infusion and conventional slow
continuous infusion. Radiology. 1993;188:861-867.
Yusuf SW, Whitaker SC, Gregson RH, Wehnam PW, Hopkinson
BR, Makin GS. Prospective randomized comparative study of
pulse spray and conventional local thrombolysis. Eur J Endovasc
Surg. 1995;10:136-141.

5/22/2012 6:01:02 PM

Peripheral Arterial Embolus

13. Earnshaw JJ. Acute ischemia: evaluation and decision making.


In: Cronenwett JL, Johnston KW, eds. Rutherfords Vascular
Surgery. 7th ed. Philadelphia, PA: Saunders Elsevier; 2010:23892398.
14. Nieuwlatt R, Prins MH, Heuzey J-YL, et al. Prognosis, disease
progression, and treatment of atrial fibrillation during 1 year:
follow-up of the Euro Heart Survey on Atrial Fibrillation. Eur
Heart J. 2008;29:1181-1189.
15. Gage BF, Waterman AD, Shannon W, Boelchler M, Rich MW,
Radford MJ. Validation of clinical classification schemes for
predicting stroke. Results from the National Registry of Atrial
Fibrillation. JAMA. 2001;285:2864-2870.
16. Frost L, Engholm G, Johnsen S, Moller H, Henneberg EW, Husted S. Incident thromboembolism in the aorta and the renal, mesenteric, pelvic, and extremity arteries after discharge from the
hospital with a diagnosis of atrial fibrillation. Arch Inten Med.
2001;161:272-276.
17. Andersen LV, Vestergaard P, Deichgraeber P, Lindholt JS, Mortensen LS, Frost L. Warfarin for the prevention of systemic
embolism in patients with non-valvular atrial fibrillation: a metaanalysis. Heart. 2007;94:1607-1613.
18. Fang MC, Chang Y, Hylek EM, et al. Advanced age, anticoagulation intensity, and risk for intracranial hemorrhage among patients
taking warfarin for atrial fibrillation. Ann Inten Med. 2004;
141:745-752.
19. Oden A, Fahlen M, Hart RG. Optimal INR for prevention of stroke
and death in atrial fibrillation, a critical appraisal. Thromb Res.
2006;117:493-499.
20. Fuster V, Ryden LE, Crijns HJ, et al. ACC/AHA/ESC 2006 Guidelines for the Management of Patients with Atrial Fibrillation. A
report of the American College of Cardiology/American Heart
Association task force on practice guidelines and the European
Society of Cardiology Committee for practice guidelines. J Am
Coll Cardiol. 2006;48:e179-e190.

PMPH_CH90.indd 727

727

21. Crawford RS, Watkins MT. Ischemia-reperfusion. In: Cronenwett JL, Johnston KW, eds. Rutherfords Vascular Surgery. 7th ed.
Philadelphia, PA: Saunders Elsevier; 2010:89-100.
22. Hyde GL, Peck D, Powell DC. Compartment syndromes. Early
diagnosis and a bedside operation. Am Surg. 1983;49:563-568.
23. Whitesides TE, Haney TC, Harada H, Holmes HE, Morimoto K.
A simple method for tissue pressure determination. Arch Surg.
1975;110:1311-1313.
24. Russell Wl, Apyan PM, Burns RP. An electronic technique for
compartment pressure measurement using the wick catheter.
Surg Gynec Obst. 1985;161:173-175.
25. Boody AR, Wongworawat MD. Accuracy in the measurement of
compartment pressures: a comparison of three commonly used
devices. J Bone Joint Surg Am. 2005;87:2415-2422.
26. Mars M, Hadley GP. Raised intracompartmental pressure and
compartment syndromes. Injury. 1998;29:403-411.
27. McQueen MM, Court-Brown CM. Compartment monitoring in
tibial fractures. J Bone Joint Surg [Br]. 1996;78:99-104.
28. Papalambros EL, Panayiotopoulos YP, Bastounis E, Zavos G,
Balas P. Prophylactic fasciotomy of the legs following acute arterial occlusion procedures. Int Angiol. 1989;8:120-124.
29. Jensen SL, Sandermann J. Compartment syndrome and fasciotomy
in vascular surgery. A review of 57 cases. Eur J Vasc Endovasc Surg.
1997;13:48-53.
30. Heemskerk J, Kitslaar P. Acute compartment syndrome of the
lower leg: retrospective study on prevalence, technique, and outcome of fasciotomies. World J Surg. 2003;27:744-747.
31. Sheridan GW, Mutsen FA. Fasciotomy in the treatment of acute
compartment syndrome. J Bone Joint Surg Am. 1976;58:112-115.
32. Blaisdell FW. Is there a reason for controversy regarding fasciotomy? J Vasc Surg. 1989;9:828.
33. Bermudez KM, Knudson MM. Long term results of lower
extremity venous injuries. Arc Surg. 1997;132:963-967.
34. Ernst CB. Fasciotomyin perspective. J Vasc Surg. 1989;9:829-830.

5/22/2012 6:01:02 PM

Commentary on
Peripheral Arterial Embolus
Jamal J. Hoballah

The chapter on Peripheral Arterial Embolus by Drs Shane OKeeffe


and Eric Endean presents a valuable insight into the management
of acute limb ischemia due to embolization, a serious condition
associated with significant mortality and morbidity. The duration
and severity of ischemia along with the provided management
often determine the outcomes of acute limb ischemia. The surgeon
typically has no control over the duration and severity of the ischemia when the patient presents. The surgeon will have to deal with
the situation at hand using all the available armamentarium, to
prevent limb loss or the complications associated with the selected
treatment. The management options include open revascularization, thrombolytic therapy, the performance of fasciotomy, and
chronic anticoagulation. The authors provide a summary and critique of the existing evidence addressing debatable issues in the
management options by answering the following questions:

small infrainguinal incision. With the availability of fluoroscopic


guidance and over the wire embolectomy catheters, almost all the
tibial vessels can be accessed through a groin incision. Thromboembolectomy under fluoroscopic guidance will change the
procedure from a blind embolectomy to a well-controlled thromboembolectomy. Lytic therapy can also be administered in small
doses directly into the affected bed if there are residual thrombi
that could not be extracted. Such approach avoids the issues and
costs of intensive care unit monitoring while thrombolytic therapy
is being administered. In addition, it also avoids the costs of blood
testing for fibrinogen and fibrinogen split products that need to
be monitored. Alternatively, if while performing an endovascular
intervention, an embolus occurred distally, suction embolectomy
or thrombolytic therapy may be very appealing and appropriate.
Finally, surgical thromboembolectomy provides a specimen that
should be send for pathologic examination to rule out the presence of tumors such as in the very rare cases of atrial myxoma.

1. Is thrombolysis indicated for acute arterial ischemia resulting from an embolus?

2. What is the role of chronic anticoagulation for prevention of


embolus in patients with atrial fibrillation?

The authors showed elegantly that the existing controlled trials do not provide a definitive answer. They suggested that if the
patients ischemic symptoms can tolerate 24 to 48 hours of ischemia, thrombolysis can be attempted. One may argue that even if
a patient can tolerate the duration needed for thrombolytic therapy to provide perfusion, an open surgical thromboembolectomy
may be more appropriate when dealing with acute ischemia due to
embolization. In our practice, we have rarely used thrombolytic
therapy for acute ischemia due to embolization. In selecting the
treatment option, one should always weigh the complexity and
risks of thrombolysis versus those of the proposed surgical intervention. In addition, the costs of the procedures and the needed
resources should be taken into consideration. As such, each case
should be individualized. An acute embolus to the brachial artery
in a frail lady with atrial fibrillation may be best managed by a
small incision over the anticubital area under local anesthesia with
expeditious extraction of the clot rather than subjecting her to a
percutaneous procedure with a catheter traveling from the femoral artery through the aortic arch into her brachial artery. Clearly
in this scenario, thrombolytic therapy will be more likely to be
associated with risks of bleeding and complications related to the
procedure. Similarly, it will require multiple trips to the angiography suite with injection of nephrotoxic contrast. Even in the
lower extremity, most surgical thromboembolectomy procedures
can be performed very safely under local anesthesia through a

I fully agree with the conclusion outlined by the authors. One


should keep in mind that newer forms of oral anticoagulants such as
dabigatran , rivaroxaban, and others are becoming available and
may soon replace the use of warfarin, which was the main oral
anticoagulant used in the quoted studies.1-4 These new oral anticoagulants may offer protection from embolization with lower rates
of bleeding complications and without the need for blood tests to
monitor the INR levels. In addition, the combination of aspirin
and clopidogrel should be considered as an option in patients with
atrial fibrillation who cannot take warfarin but are at increased
risk of embolization.5,6
3. Should fasciotomies routinely be done with lower extremity revascularization for acute ischemia resulting from an
embolus?
I fully agree with the authors that the literature does not support
routine prophylactic fasciotomy following revascularization for
acute ischemia from an embolus. The authors presented data supporting the concept that the perfusion pressure gradient rather
than the absolute compartment pressure should guide the decision to perform a fasciotomy. Nevertheless, in cases of severe and
prolonged ischemia, it may be prudent to perform a fasciotomy
728

PMPH_CH90.indd 728

5/22/2012 6:01:02 PM

Peripheral Arterial Embolus

at the completion of the revascularization rather than to wait for


meeting the pressure criteria for performing it. It is also important to stress that if a fasciotomy is not performed after revascularization of an ischemic limb, frequent careful monitoring of the
extremity and its compartments should be performed to ensure
that the need of a fasciotomy is not missed.

REFERENCES
1. Bounameaux H. The novel anticoagulants: entering a new era.
Swiss Med Wkly. 2009;139:60-64.
2. Turpie AG. New oral anticoagulants in atrial fibrillation. Eur
Heart J. 2008;29:155-165.

PMPH_CH90.indd 729

729

3. Camm AJ. The RE-LY study: Randomized Evaluation of Longterm anticoagulant therapY: dabigatran vs. warfarin. Eur Heart J.
2009;30:2554-2555.
4. Connolly SJ, Ezekowitz MD, Yusuf S, et al. Dabigatran versus warfarin in patients with atrial fibrillation. N Engl J Med. 2009;361:
1139-1151.
5. The ACTIVE Investigators. The effect of clopidogrel added to aspirin in patients with atrial fibrillation. N Engl J Med. 2009: DOI:
10.1056/NEJMoa0901301.
6. ACTIVE Writing Group of the ACTIVE Investigators. Clopidogrel
plus aspirin versus oral anticoagulation for atrial fibrillation in
the Atrial fibrillation Clopidogrel Trial with Irbesartan for prevention of Vascular Events (ACTIVE W): a randomised controlled trial. Lancet. 2006;367:1903-1912.

5/22/2012 6:01:02 PM

CHAPTER 91

Vascular Syndromes
John E. Campbell and Ali F. Aburahma

compared to warm climates.3 Raynauds syndrome can be classified as primary or secondary. Primary Raynauds syndrome is
an idiopathic vasospastic disorder with no identifiable underlying
cause, whereas secondary Raynauds syndrome is a vasospastic
disorder secondary to some underlying disease or condition commonly seen in patients with connective tissue disorders. The first
line of therapy for Raynauds syndrome is behavioral modification such as avoiding exposure to cold and wearing gloves or mittens when exposed to a cold environment; this is often all that is
needed in most patients with primary Raynauds syndrome. Pharmacologic therapy for the treatment of Raynauds syndrome is
initiated when avoidance of precipitating factors does not result in
significant relief of vasospastic attacks. It is important to explain
to patients that the initiation of medications is not to cure the
underlying cause of the vasospasm but to decrease the frequency
and severity of the vasospastic attacks.

INTRODUCTION
Though atherosclerotic vascular disease is the most common
disease entity encountered in a vascular surgeons practice, it is
important to have an understanding of other vascular syndromes.
Recent focus has shifted toward prevention of atherosclerosis
with medical therapies such as HMG-CoA reductase inhibitors
for hyperlipidemia, antihypertensives for hypertension, multiple
classes of medication for the treatment of diabetes mellitus, and of
course pharmacologic therapy for smoking cessation. However, an
understanding of the pathophysiology, diagnosis, and especially
the treatment of nonatherosclerotic vascular syndromes such as
Raynauds syndrome and thromboangiitis obliterans (TAO) is
indispensable in a vascular surgeons practice. This chapter will
focus on an evidence-based evaluation of the medical treatments
of both Raynauds syndrome and TAO with a review of the current literature.

1. Are calcium channel blockers effective in the treatment of


Raynauds syndrome?

RAYNAUDS SYNDROME

Nifedipine is the most well studied calcium channel blocker for


the treatment of Raynauds syndrome and has been demonstrated
to be effective in decreasing the frequency and severity of vasospastic attacks.4-8 Other calcium channel blockers that have been
shown to have benefit include nicardipine,9,10 felodipine,11,12 and
diltiazem.13,14 There have been two meta-analyses evaluating the
use of calcium channel blockers. The first evaluated the efficacy of
calcium channel blockers for primary Raynauds syndrome. Seventeen total randomized controlled trials (n = 348) were included
in the analysis and the results demonstrated a small reduction in
the frequency and severity of attacks. There was an average of 5.0
or 2.8 (after two trials with the most extreme positive and negative point estimates were removed) fewer attacks per week and a
33% reduction in severity with treatment with calcium channel
blockers (Level 1a evidence).15 The second meta-analysis evaluated
the efficacy of calcium channel blocker in patients with secondary Raynauds syndrome with underlying scleroderma. Eight randomized, double-blinded, placebo-controlled trials (n = 109) were

Maurice Raynaud first described what is now known as Raynauds


syndrome in 1862.1 The symptoms of Raynauds syndrome can
generally be described as an exaggerated response to a cold environment or emotions, and it usually involves the digits, though the
syndrome can affect the nose, ear lobes, and nipples as well. The
classic symptoms involve the affected digits turning white from
ischemia, then blue from deoxygenation, and finally red with reperfusion. However, the classic triad of color changes does not need
to be present for the diagnosis and often only one or two color
changes occur. Raynauds syndrome is more common in women
than in men,2 and the prevalence in the general population is not
well known secondary to varied reports in the literature. A study
using the Framingham cohort reported the prevalence to be 9.6%
in women and 8.1% in men. There is no doubt that the prevalence of Raynauds syndrome changes based on the geographical
region, with colder climates having an increased prevalence when
730

PMPH_CH91.indd 730

5/22/2012 6:01:44 PM

Vascular Syndromes

included in the analysis; the results indicated that calcium channel blockers lead to significant clinical improvement in both the
frequency (8.31 fewer attacks weekly) and severity (>35% improvement) of ischemic attacks in patients with scleroderma (Level 1a
evidence).16
Answer: Although calcium channel blockers appear to only
have a small reduction in the frequency or severity of attacks,
they are beneficial and should be used in patients with primary
Raynauds syndrome when conservative measures fail (Grade A
recommendation). Calcium channel blockers lead to a significant
improvement in both the frequency and severity of vasospastic
attacks in patients with secondary Raynauds syndrome with scleroderma (Grade A recommendation).
2. Are nitrates effective in the treatment of Raynauds
syndrome?
Nitrate preparations are potent vasodilators that have been classically used to treat coronary angina but are also used to treat
Raynauds syndrome. Nitrates can be given as oral, topical, or
intravenous preparations. Treatment with topical glyceryl trinitrate has been demonstrated to increase microvascular blood flow
to the digits in patients with primary Raynauds syndrome and
secondary Raynauds syndrome in patients with scleroderma.17
However, the use of nitrates is limited secondary to their side
effects, which includes hypotension, headache, and flushing. Th is
was demonstrated in a randomized, double-blind study by Teh
et al. comparing the use of sustained-release glyceryl trinitrate
patches to placebo for the treatment of both primary Raynauds
syndrome and secondary Raynauds syndrome in patients with
scleroderma.18 The glyceryl trinitrate patches were found to
decrease the number and severity of attacks significantly; however, the objective assessments by infrared thermography did
not show any significant improvement when the patients were
treated with glyceryl trinitrate patches (Level 1b evidence). More
importantly, approximately 80% of patients had the side effect
of headaches, and 8 of the 42 patients enrolled in the trial withdrew from the trial during treatment with glyceryl trinitrate
patches, secondary to headaches refractory to treatment.18 There
is a paucity of evidence for the use of oral nitrate preparations
to decrease the frequency or severity of vasospastic attack, and
generally nitrates are not considered a fi rst-line therapy for Raynauds syndrome.
Answer: Topical nitrates have been demonstrated to significantly decrease the severity and frequency of vasospastic attacks
in Raynauds syndrome; however, the use of topical nitrates is
limited by significant side effects, particularly headaches (Grade B
recommendation). There is insufficient data regarding the use
of oral nitrates to give a recommendation regarding use in Raynauds syndrome.
3. Are prostaglandins effective in the treatment of Raynauds
syndrome?
Prostaglandins are potent vasodilators and platelet aggregation
inhibitors that have been used to treat severe Raynauds syndrome.
Intravenous iloprost, alprostadil, and epoprostenol are examples
of prostaglandins that have been used to treat severe Raynauds
syndrome in patients with systemic sclerosis. Iloprost has been
investigated in multiple prospective studies and has been shown
to improve frequency and severity of vasospastic attacks19-23

PMPH_CH91.indd 731

731

and improve healing of digital ulcers19,22,23 (Level 1a evidence).


Iloprost has also been found to have durable results with maintenance infusions on the prevention of ischemic ulcers in patients
with scleroderma (Level 4 evidence).24 The Cochrane Review
performed a meta-analysis on the effect of intravenous iloprost
in the treatment of secondary Raynauds syndrome in patients
with scleroderma. Iloprost was found to decrease the frequency
and severity of vasospastic attacks and to be effective in preventing and healing digital ulcers (Level 1a evidence).25 Oral iloprost
was evaluated in a multicenter, randomized, parallel-group
comparison where patients were treated for 6 weeks (Level 1b
evidence).26 The treatment was found to significantly decrease the
duration of attacks at 6 weeks, but the severity and frequency of
attacks was not decreased significantly. At the end of follow-up at
12 weeks, there was a significant decrease in the duration of
attacks and the severity of attacks, but not in the frequency of
attacks.26 A second multicenter randomized, parallel-group,
placebo-controlled double-blind study comparing oral iloprost
and placebo found no difference between the treatment groups
in the duration or frequency of attacks (Level 1b evidence). 27
Epoprostenol is another prostaglandin that has been studied in
the treatment of severe Raynauds syndrome. It is administered
intravenously and has been demonstrated in studies to decrease
the frequency and duration of vasospastic attacks28 as well as
increase fi ngertip skin temperature and laser Doppler flow28,29
in patients with severe Raynauds syndrome, but the effects do
not appear to be sustained long term (Level 1a evidence). 28,29 Two
other oral prostaglandins have been studied for the treatment
of severe Raynauds syndrome. One is beraprost, which was not
found to have a statistically significant benefit when compared
to placebo in decreasing the number or severity of vasospastic
attacks in patients with primary Raynauds syndrome (Level 1b
evidence). 30 It was also compared to placebo in the efficacy in
the prevention of digital ulceration in patients with scleroderma.
Though there was a trend toward fewer digital ulcerations and
the median survival without recurrence of digital ulcerations
was longer in the group treated with beraprost, it did not reach
statistical significance (Level 1b evidence). 31 Another oral prostaglandin that has been studied is cicaprost. Cicaprost has been
evaluated for effectiveness in the treatment of Raynauds syndrome in patients with systemic sclerosis and was not found to
have a statistically significant impact on the clinical and laboratory parameters of digital vasospasm when compared to placebo (Level 1b evidence). 32 Unfortunately, given the evidence, it
appears that intravenous formulations of prostaglandins should
be used rather than oral formulations. Th is can be a barrier to
treatment in some individuals secondary to the time involved to
administer the medication.
Answer: There is Level 1a evidence that intravenous iloprost
decreases the frequency and severity of vasospastic attacks in
patients with severe Raynauds syndrome and systemic sclerosis
and that it is effective in preventing and healing digital ulcers in
patients with systemic sclerosis (Grade A recommendation). Epoprostenol appears to have short-term benefits in decreasing the
frequency and severity of vasospastic attacks but the benefit does
not appear to be sustained in the long term (Grade A recommendation). Secondary to the benefits not being sustained, iloprost is a
preferred treatment over epoprostenol. Oral prostaglandins have
not been demonstrated to be efficacious in the treatment of Raynauds syndrome (Grade D recommendation).

5/22/2012 6:01:44 PM

732

Surgery: Evidence-Based Practice

4. Is sildenafi l effective in the treatment of Raynauds


syndrome?
Sildenafi l is a selective inhibitor of cyclic guanosine monophosphate (cGMP)specific phosphodiesterase type 5 (PDE5) that
increases levels of nitric oxide that is a potent vasodilator. Sildenafi l is indicated for use in the treatment of erectile dysfunction but
has also been used for the treatment of pulmonary hypertension.
It has also been used for the treatment of refractory Raynauds
syndrome. The efficacy of sildenafi l in treatment of patients with
scleroderma and secondary Raynauds has been demonstrated
in a retrospective study that evaluated 10 patients. There was an
improvement in the severity and frequency of vasospastic attacks
and six of eight patients ulcers healed (Level 4 evidence).33 The
effectiveness of sildenafi l in the treatment of digital ulcers in
patients with systemic sclerosis was demonstrated in a prospective nonrandomized study. Nineteen patients had 49 digital ulcers
prior to treatment and this was decreased to 17 ulcers (P < .001)
after treatment (Level 4 evidence).34 Sildenafi l was also investigated in a double-blinded, placebo-controlled, fi xed-dose, crossover randomized trial to investigate its efficacy in the treatment of
Raynauds syndrome that was resistant to previous conventional
vasodilatory therapy.35 Treatment with sildenafi l was demonstrated to significantly decrease the duration and frequency of
vasospastic attacks and was demonstrated to increase capillary
blood flow as measured with a laser Doppler anemometer (Level 1b
evidence).
Answer: There is Level 1b evidence that sildenafi l is an effective treatment of refractory Raynauds syndrome (Grade B recommendation). However, more evidence is needed to confi rm the one
randomized study that has been completed and to obtain Level 1a
evidence.

THROMBOANGIITIS OBLITERANS
(BUERGERS DISEASE)
Thromboangiitis obliterans (TAO) is a nonatherosclerotic segmental inflammatory disease that most commonly affects the
small and medium-sized arteries and veins of the extremities. TAO is distinguished from other forms of vasculitis by the
patient usually having a normal erythrocyte sedimentation rate
and the pathologic examination demonstrating highly inflammatory thrombus involving both the arteries and veins with
inflammation involving all three layers of the arterial wall. Classically TAO affects young men before the age of 45 years; however, the incidence in women has been increasing compared to
the original reports by Buerger.36,37 The cause of TAO is unknown
but the use of tobacco is essential in the onset and progression
of disease. Patients with TAO can have claudication (64%), rest
pain (81%), ischemic ulcers (76%), Raynauds syndrome (44%),
or thrombophlebitis (38%) as presenting signs and symptoms.37
There is not a consensus on diagnostic criteria for TAO and several different criteria have been published. Shionoya proposed the
following criteria for the diagnosis of TAO: (1) smoking history;
(2) onset before the age of 50 years; (3) infrapopliteal arterial occlusions; (4) either upper limb involvement or phlebitis migrans; and
(5) absence of atherosclerotic risk factors other than smoking.38
Usually a biopsy is not needed to confirm the diagnosis unless the

PMPH_CH91.indd 732

patient has an atypical presentation. The natural history of TAO


involves significant morbidity with a risk of amputation of 25% at
5 years, 38% at 10 years, and 46% at 20 years.38 The risk of amputation is decreased with smoking cessation and is eliminated by
8 years after smoking cessation.39
5. Is autologous stem cell therapy effective for the treatment
of TAO?
Autologous stem cell therapy has been used for the treatment of
peripheral arterial disease, coronary artery disease, and TAO.
Endothelial progenitor cells (EPCs) are involved in the promotion of new blood vessel growth, and these can be isolated from
the bone marrow, circulating mononuclear cells, and cord blood.
Therapy involves isolation of the EPCs and injection into the gastrocnemius muscle, sole of the foot, or even the hand. Miyamoto
et al. performed a prospective unblinded pilot study with injection of bone marrow mononuclear cells into 11 limbs of 8 patients
with TAO.40 Eight of the eleven treated limbs had ulcers at baseline
and in all eight limbs there was improvement, with complete healing in seven of the eight limbs (88%) (Level 4 evidence). However,
the durability and safety of the procedure was questionable with
four out of the eight patients having clinical events on follow-up
including one individual having sudden death, another with
worsening of skin ulcers on follow-up, one with worsening rest
pain, and the last developing an arteriovenous shunt in the foot
that resolved spontaneously during follow-up.40 The best review to
date is a meta-analysis performed by Fadini et al. evaluating the
efficacy of autologous stem cell therapy for peripheral arterial disease.41 Twelve trials with 147 patients were included that evaluated
stem cell therapy for TAO. Two types of stem cells were used in the
clinical trials: bone marrow aspiration and apheresis of peripheral
blood after stimulation with granulocyte colony stimulating factor (G-CSF). Unfortunately, there were no controlled trials in the
TAO patients. However, there was a significantly larger benefit in
patients with TAO compared to atherosclerosis obliterans (ASO)
when using ankle-brachial index (ABI), transcutaneous oxygen
(TcO2), pain scale, and pain-free walking distance. Also, when
all patients with both TAO and ASO (701 patients) were pooled
together there was a significant improvement in ABI, TcO2, painfree walking distance, and improvement in pain (Level 3a evidence with TAO).41
Answer: There is mounting evidence that autologous stem
cell therapy is an effective treatment for TAO (Grade B recommendation). However, prospective randomized trials are needed
to confirm the benefits and safety of stem cell therapy before
widespread use.
6. Is intravenous iloprost efficacious for the treatment of TAO?
Intravenous iloprost has been demonstrated to be an effective
treatment for severe Raynauds syndrome in patients with systemic sclerosis as discussed earlier. However, the pathogenesis
of systemic sclerosis and TAO is quite different though both can
lead to Raynauds syndrome. Intravenous iloprost was evaluated
by Fiessinger and Schafer in a randomized prospective doubleblind trial of 133 patients with TAO where it was compared to
aspirin.42 Iloprost treatment was found to be associated with a
significantly increased relief of ischemic pain and ulcer healing
when compared to the aspirin-treated group (Level 1b evidence).

5/22/2012 6:01:44 PM

Vascular Syndromes

More specifically, 85% of iloprost-treated patients demonstrated


clinical benefit compared to 17% of the aspirin-treated group.42
A randomized prospective trial comparing intravenous iloprost
to lumbar sympathectomy (LS) in patients with TAO and rest
pain or ulcers was reported by Bozkurt et al.43 There were 200
patients randomized with 162 patients available in follow-up
for evaluation. The complete healing rate was 61.9% in the iloprost group compared to 40% in the LS group (P = 0.012) at the

733

4th week and 85.3% versus 52.3%, respectively, in the 24th week
(P < .01). The size of the ulcers also significantly decreased more
in the iloprost group at the 4th and 24th week as well (Level 1b
evidence).43
Answer: There is good quality evidence that the use of intravenous iloprost is superior to both aspirin and lumbar sympathectomy for the treatment of ischemic rest pain and ulcers in
patients with TAO (Grade A recommendation).

Clinical Question Summary


Question

Answer

1 Are calcium channel


blockers effective
in the treatment of
Raynauds syndrome?

Calcium channel blockers (CCBs) give a small reduction


in the frequency and severity of vasospastic attacks in
patients with primary Raynauds syndrome. CCBs cause a
significant improvement in both the frequency and severity
of vasospastic attacks in patients with secondary Raynauds
syndrome with scleroderma.

4-16

2 Are nitrates effective


in the treatment of
Raynauds syndrome?

Topical nitrates significantly decrease the severity and frequency


of vasospastic attacks in Raynauds syndrome; however, the
use of topical nitrates is limited by significant side effects.

17, 18

3 Are prostaglandins
effective in the
treatment of Raynauds
syndrome?

Intravenous iloprost decreases the frequency and severity


of vasospastic attacks in patients with severe Raynauds
syndrome and systemic sclerosis and is effective in preventing
and healing digital ulcers in patients with systemic sclerosis.

19-32

4 Is sildenafil effective
in the treatment of
Raynauds syndrome?

Sildenafil is an effective treatment of refractory Raynauds


syndrome.

33-35

5 Is autologous stem cell


therapy effective for
the treatment of TAO?

Autologous stem cell therapy is an effective treatment for TAO.

40, 41

6 Is intravenous iloprost
efficacious for the
treatment of TAO?

Intravenous iloprost is superior to both aspirin and lumbar


sympathectomy for the treatment of ischemic rest pain and
ulcers in patients with TAO.

42, 43

REFERENCES
1. Raynaud M. Local Asphyxia and Symmetrical Gangrene of the
Extremities 1862. New Researches on the Nature and Treatment
of Local Asphyxia of the Extremities 1874. [Translated by Barlow.]
London: New Sydenham Society; 1888.
2. Brand FM, Larson MG, Kannel WB, McGuirk JM. The occurrence of Raynauds phenomenon in a general population: the
Framingham Study. Vasc Med. 1997;2:296-301.
3. Maricq HR, Carpentier PH, Weinrich MC, et al. Geographic
variation in the prevalence of Raynauds phenomenon: a 5 region
comparison. J Rheumatol. 1997;24(5):879-889.
4. Kahan A, Weber S, Amor B, Menks CJ, Hodara M, Degeorges
M. Nifedipine and Raynauds phenomenon associated with connective tissue diseases. Int Angiol. 1985;4(2):221-223.
5. White CJ, Phillips WA, Abrahams LA, Watson TD, Singleton
PT Jr. Objective benefit of nifedipine in the treatment of Raynauds phenomenon. Double-blind controlled study. Am J Med.
1986;80(4):623-625.

PMPH_CH91.indd 733

Grade of
References
Recommendation

6. Costantini A, Martelli E, Bavera P, Agus GB. Slow release nifedipine in the treatment of Raynauds phenomenon. Int Angiol.
1987;6(4):359-363.
7. Smith CD, McKendry RJ. Controlled trial of nifedipine in the
treatment of Raynauds phenomenon. Lancet. 1982;2(8311):12991301.
8. Gjrup T, Kelbaek H, Hartling OJ, Nielsen SL. Controlled doubleblind trial of the clinical effect of nifedipine in the treatment
of idiopathic Raynauds phenomenon. Am Heart J. 1986;111(4):
742-745.
9. Ferri C, Cecchetti R, Cini G, et al. Slow-releasing nicardipine in
the treatment of Raynauds phenomena without underlying diseases. Clin Rheumatol. 1992;11(1):76-80.
10. Kahan A, Amor B, Menks CJ, Weber S, Gurin F, Degeorges
M. Nicardipine in the treatment of Raynauds phenomenon: a
randomized double-blind trial. Angiology. 1987;38(4):333-337.
11. Schmidt JF, Valentin N, Nielsen SL. The clinical effect of felodipine and nifedipine in Raynauds phenomenon. Eur J Clin
Pharmacol. 1989;37(2):191-192.

5/22/2012 6:01:44 PM

734

Surgery: Evidence-Based Practice

12. Kallenberg CG, Wouda AA, Meems L, Wesseling H. Once daily


felodipine in patients with primary Raynauds phenomenon. Eur
J Clin Pharmacol. 1991;40(3):313-315.
13. Rhedda A, McCans J, Willan AR, Ford PM. A double blind placebo controlled crossover randomized trial of diltiazem in Raynauds phenomenon. J Rheumatol. 1985;12(4):724-727.
14. Kahan A, Amor B, Menks CJ. A randomized double-blind trial
of diltiazem in the treatment of Raynauds phenomenon. Ann
Rheum Dis. 1985;44(1):30-33.
15. Thompson AE, Pope JE. Calcium channel blockers for primary
Raynauds phenomenon: a meta-analysis. Rheumatology. 2005;
44:145-150.
16. Thompson AE, Shea B, Welch V, Fenlon D, Pope JE. Calciumchannel blockers for Raynauds phenomenon in systemic sclerosis. Arthritis Rheum. 2001;44(8):1841-1847.
17. Anderson ME, Moore TL, Hollis S, Jayson MIV, King TA, Herrick AL. Digital vascular response to topical glyceryl trinitrate, as
measured by laser Doppler imaging, in primary Raynauds phenomenon and systemic sclerosis. Rheumatology. 2002;41:324-328.
18. Teh LS, Manning J, Moore T, Tully MP, OReilly D, Jayson MI.
Sustained-release transdermal glyceryl trinitrate patches as a
treatment for primary and secondary Raynauds phenomenon.
Br J Rheumatol. 1995;34(7):636-641.
19. Kawald A, Burmester GR, Huscher D, Sunderktter C, Riemekasten G. Low versus high-dose iloprost therapy over 21 days in
patients with secondary Raynauds phenomenon and systemic sclerosis: a randomized, open, single-center study. J Rheumatol. 2008;
35:1830-1837.
20. Milio G, Corrado E, Genova C, et al. Iloprost treatment in patients
with Raynauds phenomenon secondary to systemic sclerosis and
the quality of life: a new therapeutic protocol. Rheumatology.
2006;45:999-1004.
21. McHugh NJ, Csuka M, Watson H, et al. Infusion of iloprost, a
prostacyclin analogue, for treatment of Raynauds phenomenon
in systemic sclerosis. Ann Rheumatic Dis. 1988;47(1):43-47.
22. Torley HI, Madhok R, Cappell HA, et al. A double blind randomized, multicentre comparison of two doses of intravenous iloprost
in the treatment of Raynauds phenomenon secondary to connective tissue diseases. Ann Rheumatic Dis. 1991;50(11):800-804.
23. Rademaker M, Cooke ED, Almond NE, et al. Comparison of
intravenous infusions of iloprost and oral nifedipine in treatment
of Raynauds phenomenon in patients with systemic sclerosis: a
double blind randomized study. BMJ. 1989;298:561-564.
24. Bettoni L, Geri A, Air P, et al. Systemic sclerosis therapy with
iloprost: a prospective observational study of 30 patients treated
for a median of 3 years. Clin Rheumatol. 2002;21:244-250.
25. Pope J, Fenlon D, Thompson A, et al. Iloprost and cisaprost for
Raynauds phenomenon in progressive systemic sclerosis. Cochrane Database Syst Rev. 1998;(2):CD000953.
26. Black CM, Halkier-Srensen L, Belch JJ, et al. Oral iloprost in
Raynauds phenomenon secondary to systemic sclerosis: a multicentre, placebo-controlled, dose-comparison study. Br J Rheumatol. 1998;37(9):952-960.
27. Wigley FM, Korn JH, Csuka ME, et al. Oral iloprost treatment
in patients with Raynauds phenomenon secondary to systemic

PMPH_CH91.indd 734

28.

29.

30.

31.

32.

33.

34.

35.

36.

37.
38.
39.

40.

41.

42.

43.

sclerosis: a multicenter, placebo-controlled, double-blind study.


Arthritis Rheum. 1998;41(4):670-677.
Belch JJ, Newman P, Drury JK, et al. Intermittent epoprostenol
(prostacyclin) infusion in patients with Raynauds syndrome. A
double-blind study. Lancet. 1983;1(8320):313-315.
Kigma K, Wollersheim H, Th ien T. Double-blind, placebocontrolled study of intravenous prostacyclin on hemodynamics
in severe Raynauds phenomenon: the acute vasodilatory effect is
not sustained. J Cardiovasc Cardiol. 1995;26(3):388-393.
Vayssairat M. Controlled multicenter double blind trial of an oral
analog of prostacyclin in the treatment of primary Raynauds
phenomenon. French Microcirculation Group for the Study of
Vascular Acrosyndromes. J Rheumatol. 1996;23(11):1917-1920.
Vayssairat M. Preventive effect of an oral prostacyclin analog,
beraprost sodium, on digital necrosis in systemic sclerosis. French
Microcirculation Society Multicenter Group for the Study of
Vascular Acrosyndromes. J Rheumatol. 1999;26(10):2173-2178.
Lau CS, Belch JJ, Madhok R, et al. A randomized, double-blind
study of cicaprost, an oral prostacyclin analogue, in the treatment
of Raynauds phenomenon secondary to systemic sclerosis. Clin
Exp Rheumatol. 1993;11(1):35-40.
Gore J, Silver R. Oral sildenafi l for the treatment of Raynauds
phenomenon and digital ulcers secondary to systemic sclerosis.
Ann Rheumatic Dis. 2005;64:1387.
Brueckner CS, Becker MO, Kroencke T, et al. Effect of sildenafi l
on digital ulcers in systemic sclerosis: analysis from a single centre pilot study. Ann Rheumatic Dis. 2010;69:1475-1478.
Fries R, Shariat K, Wilmowsky HV, Bhm M. Sildenafi l in the
treatment of Raynauds phenomenon resistant to vasodilatory
therapy. Circulation. 2005;112:2980-2985.
Olin JW, Young JR, Graor RA, Ruschhaupt WF, Bartholomew
JR. The changing clinical spectrum of thromboangiitis obliterans (Buergers Disease). Circulation. 1990;82(5 Suppl):IV3-8.
Buerger L. The Circulatory Disturbances of the Extremities. Philadelphia, PA: WB Saunders; 1924.
Shionoya S. Diagnostic criteria of Buergers disease. Int J Cardiol.
1998;66(Suppl 1):S243-S245.
Cooper LT, Tse TS, Mikhail MA, Mc Bane RD, Stanson AW,
Ballman KV. Long-term survival and amputation risk in thromboangiitis obliterans (Buergers Disease). Circulation. 2004;44(12):
2410-2411.
Miyamoto K, Nishigami K, Nagaya N, et al. Unblinded pilot
study of autologous transplantation of bone marrow mononuclear cells in patients with thromboangiitis obliterans. Circulation. 2006;114:2679-2684.
Fadini GP, Agostini C, Avogara A. Autologous stem cell therapy
for peripheral arterial disease: meta-analysis and systemic review
of the literature. Atherosclerosis. 2010;209(1):10-17.
Fiessinger JN, Schafer M. Trial of iloprost versus aspirin treatment for critical limb ischaemia of thromboangiitis obliterans.
Lancet. 1990;335:555-557.
Bozkurt AK, Kksal C, Demirbas MY, et al. A randomized trial
of intravenous iloprost (a stable prostacyclin analogue) versus
lumbar sympathectomy in the management of Buergers disease.
Int Angiol. 2006;25:162-168.

5/22/2012 6:01:44 PM

CHAPTER 92

Management of Graft Occlusion


David Vogel and Anthony J. Comerota

INTRODUCTION

patients have bypasses to arteries that would not admit a 1-mm


dilator or were bypassed to isolated arterial segments with discontinuous arterial continuity to the tarsal or plantar arteries.4,7
Poor conduitthat is, a vein that has sclerotic segments, synectiae, or has required venovenostomy, vein patch angioplasty, or
prosthetic interpositionhas increased risk of early occlusion.7
Vein conduits <3-mm diameter have a 30-day primary patency of
85.7%, compared to veins >3.5 mm, which have a 93% 30-day primary patency. Generally, composite vein grafts have poorer 30-day
patency (84.1%) whereas a single-segment, good quality great
saphenous vein graft has a 30-day primary patency of 92.2%.8
Stept et al.4 found that atheroma and thromboembolism were
associated with approximately 10% of early graft occlusions. Twothirds of patients with early graft occlusion were found to have
well-organized thrombotic debris. Once these grafts were treated
with thrombectomy and the underlying cause of occlusion corrected, patency was maintained in 83% of the grafts.4
Hypercoagulability increases the rate of both early and late
graft failure and has been found to contribute to 16% of early graft
failures.4 Some authors have speculated that, with the increased
use of arterial reconstructions by endovascular procedures, there
will be more patients who develop immune-related hypercoagulable states secondary to repeated heparin exposure.9,10
Singh et al.,3 in their review of the Veterans Affairs National
Surgical Quality Improvement Program (NSQIP) database, identified a number of important factors responsible for early graft
occlusion. Patients younger than 50 have an 8.2% rate of early
graft failure compared to 5.3% for those aged 51 to 60 and 4.4% for
those over 70 (P < .001). African Americans have a significantly
increased risk of early graft occlusion (6.7%) whereas all other
races combined have a rate of 4.5%. Not surprisingly, patients
with a history of smoking had a higher (5.4%) rate of early graft
failure than nonsmokers (4.4%).3
Answer: Early graft occlusion is usually due to technical considerations; however, other factors such as hypercoagulability,
African-American race, and tobacco use play important roles in
early occlusion (Level II evidence).

Graft thrombosis remains one of the more challenging aspects


of vascular surgery. In patients with occluded infrageniculate
bypass grafts, over 50% undergo major amputation at 1 year.1,2
The remaining patients will suffer from ischemic rest pain and
nonhealing ischemic ulcers, and over 25% will die. Successfully restoring perfusion to the acutely ischemic extremity preserves limbs, improves quality of life, and may prolong survival.
Although the causes of graft thrombosis are multifactorial, it is
useful for the management of occluded grafts to categorize the
occlusive event temporally: early (<30 days) and late (30 days)
graft occlusion.

PHYSIOLOGY
1. What are the common causes of early graft occlusion?
Early graft failure occurs in 0.3% to 10% of arterial reconstructions.1,3
Technical errors, which commonly include intimal flaps, anastomotic narrowing, twisting or kinking of the graft, or thrombus
formation during the operation,4 historically accounted for up to
55% of early failures. However, improved techniques have reduced
these technical failures to approximately 10% of early failures.
This improvement can be attributed to completion imaging of the
reconstructive procedure including intraoperative duplex ultrasound, angioscopy, and completion arteriography.5-7 These techniques visualize technical defects, permitting correction before
the patient leaves the operating room. If the graft occludes following a technically good reconstruction, the etiology is likely due to
a poor conduit or poor distal runoff (in which case the patient is a
poor candidate for reconstruction).
Occlusive disease in the vessels beyond the reconstructed
segment, known as compromised distal runoff, is a recognized
cause of early graft occlusion. In fact, 87% of patients with graft
occlusion have only one-vessel runoff. Furthermore, up to 20% of

735

PMPH_CH92.indd 735

5/22/2012 6:02:54 PM

736

Surgery: Evidence-Based Practice

2. What are the common causes of late graft occlusion?


Many authors would have you believe that the main difference
between late and early graft occlusion is the elimination of early
technical errors. However, operative technique and poor choice
of conduit continue to play a significant role in late graft failure
because some surgeons choose not to use the best available conduit, an autogenous vein graft, and settle for polytetrafluoroethylene (PTFE) or other prosthetic to an above-the-knee popliteal
artery target, or they choose other poor target vessels.11
Multiple studies have shown the superiority of autogenous
vein graft over PTFE or polyester, with veins demonstrating a primary patency of 64%67% at 4 to 5 years compared with 54% and
47%51% at 4 years for dacron and PTFE, respectively.12 Common
reasons given for using prosthetics are to save the vein for more
distal bypasses or for future coronary bypass.13 However, when
prosthetic conduit occludes, patients are more likely to have SVS/
ISCVS (Society for Vascular Surgery/International Society for
Cardiovascular Surgery) grade II acute ischemia (critical limb
ischemia), resulting in more emergent bypasses, distal bypass
grafting, or amputation.14 Further, only 3% to 5% of all patients
will require the vein used for infrainguinal revascularization for a
coronary artery bypass.15
A common technical error that decreases graft patency is targeting a diseased artery as the distal outflow.11 Common examples
include using an above-knee proximal popliteal artery versus a
below-knee popliteal, or forcing a femoropopliteal bypass rather
than using a nondiseased tibial vessel. Anastomosing to a vessel with poor or disadvantaged runoff increases the risk of graft
occlusion.16
Neointimal hyperplasia plays an important role in late
graft occlusion by causing stenosis of the bypass graft or anastomotic stenoses, which lead to graft occlusion. It usually occurs at
anastomotic sites but can also occur within a vein graft at valve
cusps. Neointimal hyperplasia begins at the time of arterial
trauma, resulting in smooth muscle cell (SMC) migration and
proliferation. The SMCs deposit extracellular matrix as a response
to injury of the artery. Although this response slows after several
weeks, SMCs continue to proliferate.17 Eventually the ongoing proliferation can cause a critical stenosis, leading to graft occlusion.
Progression of disease also plays an important role in graft
occlusion. Most patients with symptomatic atherosclerosis have
multiple risk factors, including but not limited to smoking, diabetes, hypertension, and hypercholesterolemia. Despite public
awareness of the risks of tobacco, many patients continue to smoke
as it is a powerful addiction. Unfortunately, the likelihood of successful smoking cessation in those attempting to quit is only 20%
to 40% at 1 year.18 The female gender and the African-American
race are also significant risk factors for graft occlusion.19
Answer: The causes of late graft failure are multifactorial;
technical error, progression of disease, and neointimal hyperplasia all play a role (Level II evidence).

MANAGEMENT
3. What is the treatment for early graft occlusion?
Some have argued that if a failed bypass was constructed with the
only available suitable conduit to the only available target vessel,

PMPH_CH92.indd 736

and there is no evidence of anastomotic or intrinsic conduit problem, further attempts at thrombectomy, revision, replacement, or
substitution of target artery are only likely to increase morbidity,
mortality, and expense, with little hope of limb salvage.20 Secondary patency and limb salvage rates for early occluded grafts are
poor, with patency rates of 70%, 37%, 27%, and 23% and limb salvage rates of 74%, 54%, 40%, and 31% at 1 month, 1 year, 3 years,
and 5 years, respectively.21
Despite these observations, most surgeons perform open graft
thrombectomy with or without revision. If an underlying abnormality is found and corrected, patency rates approximate primary patency.20 The important fact remains that limb salvage at
1 month is improved in patients who undergo take back operations (74%) compared to those who were not taken back for revascularization (12%). However, in patients who require multiple
take backs, there is considerable morbidity and poor limb salvage
at 1 month (33%).21 When reviewing long-term outcomes after
early graft occlusion, Robinson et al.21 noted that patients placed
on anticoagulation after their index procedure who went on to
occlude within the first 30 days had significantly lower limb salvage and secondary patency rates.21 This was thought to be not
due to negative effects of anticoagulation itself but because these
patients were considered to be at higher risk of graft failure in
the first place (as the grafts were more likely to be PTFE grafts),
have significant cardiac disease, or have repeat leg bypass procedures. Thrombolysis is mentioned simply to discourage its use for
early graft occlusion.
Answer: The treatment of early graft occlusion is thrombectomy with operative revision and correction of technical errors
(Level II evidence, Grade B recommendation).
4. What is the treatment for late graft occlusion?
When planning treatment for late graft occlusion, the initial step
is to evaluate the degree of ischemia following graft occlusion,22
which determines the need for and urgency of intervention.
Patients with grade I ischemia may need nothing done whereas
proper treatment of the patient with grade III ischemia may
be primary amputation, as the systemic effects of reperfusion of
an irreversibly ischemic limb may cause multiorgan system failure and death. This section will focus on the treatment of patients
with grades IIa and IIb ischemia (threatened limb).
Patients presenting with grades IIa and IIb ischemia should
be immediately anticoagulated to prevent thrombus propagation. Importantly, anticoagulation has been shown to increase
ischemic tolerance time by preventing small vessel thrombosis,
minimizing the inflammatory response, and maintaining collateral blood flow.23-27
Patients with acute graft occlusion have traditionally been
treated with open surgical thrombectomy and graft revision. In
more recent years, catheter-directed thrombolysis has been used
with improved results. The results of randomized trials evaluating catheter-directed thrombolysis versus primary surgical
revascularization have demonstrated important principles in the
management of patients with acute graft occlusion. The STILE
trial (Surgery versus Thrombolysis for Ischemia of the Lower
Extremity)28 clearly demonstrated that patients with acute occlusion (<14 days) had significantly better limb salvage and amputation-free survival with catheter-directed thrombolysis than an

5/22/2012 6:02:54 PM

Management of Graft Occlusion

initial surgical approach. However, surgical revascularization for


chronic occlusion resulted in significantly better limb salvage.
Interestingly, diabetics treated with catheter-directed thrombolysis had better overall survival than those randomized to surgery.
Other randomized trials have also observed a survival advantage
in patients treated with plasminogen activators for an ischemic
lower extremity.29,30
The TOPAS (Thrombolysis or Peripheral Arterial Surgery)
study30 failed to show a limb salvage benefit of catheter-directed
thrombolysis versus surgery; however, similar to STILE, patients
treated with thrombolysis required fewer open surgical procedures,
and when surgery was performed, it was of a lesser magnitude.
Bleeding complications were more frequent with catheterdirected thrombolysis, and the STILE investigators noted a correlation of bleeding with fibrinogen levels, which was likely a
reflection of a systemic fibrinolytic effect of the infused plasminogen activator. Intracranial bleeds occurred in 1.2% of the lytic
patients in the STILE trial and 1.6% to 2.1% of patients in the
TOPAS trial. When it occurred, intracranial bleeding was associated with a 50% mortality rate.
Bleeding complications have notably decreased during the
past decade as pharmacomechanical techniques (rheolytic, isolated segmental thrombolysis, ultrasound accelerated) have been
integrated, speeding lytic results. In addition, the hourly dose of
plasminogen activator has decreased while substantially increasing the volume of infusate. It is now common to infuse 0.5 to
1.0 mg of recombinant tissue plasminogen activator (rt-PA) in 50
to 100 cc saline per hour.
Important predictors of success of catheter-directed thrombolysis are the duration of occlusion (as noted previously) and the
ability to pass a guidewire through the occluded vessel. If a guidewire cannot be advanced through the occluded vessel, it is unlikely
that thrombolysis will be successful and should not be pursued.
An important indicator of sustained patency following successful thrombolysis is the identification and correction of an
underlying (culprit) lesion. If a lesion is not found or, if found, is
not corrected, rethrombosis in the short term is likely.
The method used to correct an underlying lesion is generally
endovascular. However, the type of lesion (neointimal fibroplasia
versus atherosclerosis), its location, length, degree of calcification,
and other characteristics should be considered, as some patients
will fare better with definitive surgical correction.
Answer: Thrombolysis plays a major role in the treatment
of late graft occlusion that is less than 14 days of duration. After
thrombolytic treatment is completed, underlying lesions can be
treated with endovascular techniques or surgical revision (Level I
evidence, Grade A recommendation).

PREVENTION
5. Does platelet inhibition affect graft patency?
Platelet inhibitors are often a part of the medical management of
patients with atherosclerotic disease. Consequently, a significant
number of patients with bypass grafts are on aspirin, cilostazol,
aspirin/dipyridamole, and/or clopidogrel. There is evidence that
platelet inhibition is beneficial in treating patients with bypass
grafts.

PMPH_CH92.indd 737

737

Aspirin is the most common platelet inhibitor used for the


treatment of patients with atherosclerotic disease. Numerous studies have shown increased graft patency with the use of aspirin and
the combination of aspirin with dipyridamole. In a Cochrane metaanalysis, Brown et al.31 found that at 1 year patients who underwent infrainguinal bypass surgery with both vein and prosthetic
conduit benefited from the use of aspirin or aspirin/dipyridamole
compared with those not receiving platelet inhibition (OR 0.59;
95% CI: 0.450.79). Other meta-analyses in the past have shown a
22% to 38% reduction in occlusion rate with patients receiving aspirin therapy.32,33 The Dutch Bypass Oral Anticoagulants or Aspirin
(BOA) Study Group34 found that patients with venous bypass conduit received less benefit from aspirin than those with nonvenous
bypass conduit (OR: 1.26). Johnson et al.35 reported the results of the
Veterans Affairs Cooperative Trial in which 831 patients were randomized to receive either warfarin plus aspirin (INR target of 1.4
2.8) or aspirin alone following bypass with either vein or prosthetic
grafts. Analysis of outcomes in the prosthetic group demonstrated
that the 6-mm bypass subgroup of patients receiving warfarin plus
aspirin had improved patency at 1 year (71.4% vs. 57.9%, P = .02) compared to those who had 8-mm bypass. This observation also reflects
the fact that an 8-mm prosthetic functions better as a femoralpopliteal graft than a 6-mm prosthetic. The same authors noted the
benefit of warfarin plus aspirin versus aspirin alone.
Cilostazol is a weak platelet inhibitor used in the medical
management of patients with intermittent claudication. It functions by inhibiting phosphodiesterase III. However, the more
important effects of cilostazol may be from its nonplatelet effects,
which include vasodilation and inhibition of SMC migration and
proliferation, thereby reducing neointimal fibroplasia.36,37 As discussed previously, neointimal hyperplasia plays a critical role in
eventual graft occlusion not only by causing stenosis at anastomotic sites but also within vein grafts, particularly at valve sites.
The pathway for inhibition of neointimal hyperplasia is not
fully understood but several aspects have been studied. Cilostazol increases intracellular cAMP which inhibits proliferation of
vascular SMC (VSMC) by inducing apoptosis via p53 and p21
pathways. Increased cAMP also inhibits phosphorylation of
pRB which regulates a nuclear protein E2F causing arrest in the
cell cycle of VSMC.37 Cilostazol also suppresses the tenascin-C
(TN-C) mRNA expression induced by platelet-derived growth
factor (PDGF)-BB. TN-C blocks adhesion of SMCs to fibronectin and mediates SMC migration. By blocking TN-C neointimal hyperplasia is suppressed.38 Ishizaka et al.36 demonstrated
the inhibitor effect of cilostazol on neointimal hyperplasia in rat
models by inducing carotid artery injury with balloon angioplasty
and locally applying cilostazol topically. At 14 days the rats that
had local application were found to have significantly less intimal
thickness. Similar studies have been done with rat aortic and free
artery graft models and anastomotic models in dogs with similar
results.17,38,39 Clinical studies in humans of cilostazol used following coronary and peripheral vascular intervention show reduction
in neointimal hyperplasia, which ultimately improves patency.
The combination of aspirin plus clopidogrel has been shown
to improve the patency of infrainguinal prosthetic grafts compared to aspirin alone. There was no benefit of this combination
in patients having autogenous bypass.40 Ticlopidine, an adenosine diphosphate (ADP) receptor inhibitor that has fallen out of
favor because of its side effects of neutropenia and thrombotic

5/22/2012 6:02:54 PM

738

Surgery: Evidence-Based Practice

thrombocytopenia, has been shown to improve 24-month patency


versus placebo from 63% to 82% (P = .002).41
Answer: Aspirin, the most popular antiplatelet agent, should
be used in all patients who undergo lower extremity bypass (Level I
evidence, Grade A recommendation). Cilostazol, another platelet
inhibitor often used in vascular patients, may benefit patients not
so much as an antiplatelet agent but rather for its role in preventing
neointimal hyperplasia (Level II evidence, Grade B recommendation). The addition of clopidogrel to aspirin has been shown to
benefit patients who undergo prosthetic bypass grafting (Level Ia
evidence, Grade A recommendation).
6. Does anticoagulation affect graft patency?
Vitamin K antagonist (VKA) therapy has been shown to improve
autogenous bypass graft patency versus placebo.42 The Dutch
BOA (Bypass Oral anticoagulants or Aspirin) trial studied 2690
patients who had undergone infrainguinal bypass and were randomized to either aspirin or VKA.34 Overall, the graft patency rate
was nearly equivalent; however, subgroup analysis demonstrated
that vein bypasses had a lower risk of occlusion if VKAs were used
(hazard ratio 0.69), but had a higher rate of major bleeding (hazard ratio 1.96).43
High-risk grafts (marginal venous conduit, poor arterial runoff, and prior failed bypass grafts) may also benefit from VKA
therapy. Sarac et al.44 performed a randomized trial of aspirin

versus aspirin plus warfarin in 56 patients at high risk. The cumulative 3-year primary, primary-assisted, and secondary patency
rates were significantly improved in the aspirin plus warfarin
group (74% vs. 51%, P = .04; 77% vs. 56%, P = .05; 81% vs. 56%,
P = .02) as were limb salvage rates (81% vs. 31%, P = .03). Brumberg
et al.45 demonstrated that VKA therapy is important in low-flow
prosthetic grafts (midgraft velocity 45 cm/s), showing that therapeutic anticoagulation with warfarin demonstrated improved
patency at 3 years. Higher-flow grafts did not benefit from VKA.
Jackson et al.46 demonstrated that patients with both PTFE and
vein infrainguinal bypass grafts benefited from aspirin plus warfarin. They further demonstrated that patients with prosthetic femoralpopliteal bypasses who were on aspirin plus warfarin at the time of
graft occlusion had less risk of critical limb ischemia compared to
patients taking aspirin alone (27.8% vs. 55.3%, P = .057).
It appears that the combination of aspirin plus warfarin improves lower extremity bypass graft patency, especially in patients
with compromised grafts (low flow). Aspirin plus warfarin also
reduces the risk of critical limb ischemia at the time of prosthetic
graft occlusion. However, aspirin plus warfarin is associated with
an increased risk of bleeding.
Answer: Vitamin K antagonists should be considered in
all patients who undergo compromised bypass graft ing. Vitamin K antagonists lower the risk of grade II ischemia when the
prosthetic graft eventually occludes (Level Ia evidence, Grade B
recommendation).

Clinical Question Summary


Question

Answer

Level of
Evidence

Grade

References

1 What are the common


causes of early graft
occlusion?

Technical issues, hypercoagulable state, poor


conduit, African-American race, tobacco use.

II

N/A

1, 3-10

2 What are the common


causes of late graft
occlusion?

Technical errors, progression of disease,


neointimal hypoplasia.

II

N/A

11, 13-19

3 What is the treatment for


early graft occlusion?

Surgical revision.

II

20, 21

4 What is the treatment for


late graft occlusion?

Catheter-directed thrombolysis with correction


of underlying lesion.

28-30

5 Does platelet inhibition


affect graft patency?

Aspirin and cilostazol are beneficial for all


patients (vein or graft) and clopidogrel has
shown benefit in patients with prosthetics.

Ia
II
Ia

Aspirin-A
Cilostazol-B
Clopidogrel-A

31-35
36-39
40, 41

6 Does anticoagulation
affect graft patency?

Warfarin improves graft patency and


consequences of occlusion.

Ia

32, 42-46

REFERENCES
1. Giswold ME, Landry GJ, Sexton GJ, et al. Modifiable patient
factors are associated with reverse vein graft occlusion in the era
of duplex scan surveillance. J Vasc Surg. 2003;37(1):47-53.
2. Watson HR, Schroeder TV, Simms MH, et al. Relationship of
femorodistal bypass patency to clinical outcome. Iloprost Bypass
International Study Group. Eur J Vasc Endovasc Surg. 1999;
17(1):77-83.

PMPH_CH92.indd 738

3. Singh N, Sidawy AN, DeZee KJ, Neville RF, Akbari C, Henderson W. Factors associated with early failure of infrainguinal
lower extremity arterial bypass. J Vasc Surg. 2008;47(3):556-561.
4. Stept LL, Flinn WR, McCarthy WJ III, Bartlett ST, Bergan JJ, Yao
JS. Technical defects as a cause of early graft failure after femorodistal bypass. Arch Surg. 1987;122(5):599-604.
5. Liebman PR, Menzoian JO, Mannick JA, Lowney BW, Logerfo
FW. Intraoperative arteriography in femoropopliteal and femorotibial bypass grafts. Arch Surg. 1981;116(8):1019-1021.

5/22/2012 6:02:54 PM

Management of Graft Occlusion

6. Bandyk DF, Jorgensen RA, Towne JB. Intraoperative assessment


of in situ saphenous vein arterial graft s using pulsed Doppler
spectral analysis. Arch Surg. 1986;121(3):292-299.
7. Rzucidlo EM, Walsh DB, Powell RJ, et al. Prediction of early graft
failure with intraoperative completion duplex ultrasound scan.
J Vasc Surg. 2002;36(5):975-981.
8. Schanzer A, Hevelone N, Owens CD, et al. Technical factors
affecting autogenous vein graft failure: observations from a large
multicenter trial. J Vasc Surg. 2007;46(6):1180-1190.
9. Curi MA, Skelly CL, Baldwin ZK, et al. Long-term outcome of
infrainguinal bypass grafting in patients with serologically proven hypercoagulability. J Vasc Surg. 2003;37(2):301-306.
10. Donaldson MC, Weinberg DS, Belkin M, Whittemore AD, Mannick JA. Screening for hypercoagulable states in vascular surgical
practice: a preliminary study. J Vasc Surg. 1990;11(6):825-831.
11. Baldwin ZK, Pearce BJ, Curi MA, et al. Limb salvage after infrainguinal bypass graft failure. J Vasc Surg. 2004;39(5):951-957.
12. Mamode N, Scott RN. Graft type for femoro-popliteal bypass
surgery. Cochrane Database Syst Rev. 2000;(2):CD001487.
13. AbuRahma AF, Robinson PA, Holt SM. Prospective controlled study of polytetrafluoroethylene versus saphenous vein in
claudicant patients with bilateral above knee femoropopliteal
bypasses. Surgery. 1999;126(4):594-601.
14. Klinkert P, Schepers A, Burger DH, van Bockel JH, Breslau PJ. Vein
versus polytetrafluoroethylene in above-knee femoropopliteal
bypass graft ing: five-year results of a randomized controlled
trial. J Vasc Surg. 2003;37(1):149-155.
15. Poletti LF, Matsuura JH, Dattilo JB, et al. Should vein be saved
for future operations? A 15-year review of infrainguinal bypasses
and the subsequent need for autogenous vein. Ann Vasc Surg.
1998;12(2):143-147.
16. Tangelder MJ, Algra A, Lawson JA, Eikelboom BC. Risk factors
for occlusion of infrainguinal bypass grafts. Eur J Vasc Endovasc
Surg. 2000;20(2):118-124.
17. Kudo FA, Kondo Y, Muto A, et al. Cilostazol suppresses neointimal hyperplasia in canine vein graft s. Surg Today. 2009;39(2):
128-132.
18. American Heart Association. Smoking cessation. http://www.
americanheart.org/presenter.jhtml?identifier=4731 2011. Accessed
January 24, 2011.
19. Chew DK, Nguyen LL, Owens CD, et al. Comparative analysis
of autogenous infrainguinal bypass grafts in African Americans
and Caucasians: the association of race with graft function and
limb salvage. J Vasc Surg. 2005;42(4):695-701.
20. Stone DH, Walsh DB. Local complications: graft thrombosis. In:
Rutherford RB, ed. Rutherfords Vascular Surgery. 7th ed. Philadelphia, PA: Saunders Elsevier; 2010:685-695.
21. Robinson KD, Sato DT, Gregory RT, et al. Long-term outcome
after early infrainguinal graft failure. J Vasc Surg. 1997;26(3):
425-437.
22. Rutherford RB, Baker JD, Ernst C, et al. Recommended standards for reports dealing with lower extremity ischemia: revised
version. J Vasc Surg. 1997;26(3):517-538.
23. Blaisdell FW, Steele M, Allen RE. Management of acute lower
extremity arterial ischemia due to embolism and thrombosis.
Surgery. 1978;84(6):822-834.
24. Blaisdell FW. The pathophysiology of skeletal muscle ischemia and the reperfusion syndrome: a review. Cardiovasc Surg.
2002;10(6):620-630.
25. Becker M, Menger MD, Lehr HA. Heparin-released superoxide
dismutase inhibits postischemic leukocyte adhesion to venular
endothelium. Am J Physiol. 1994;267(3 Pt 2):H925-H930.

PMPH_CH92.indd 739

739

26. Wright JG, Kerr JC, Valeri CR, Hobson RW. Heparin decreases
ischemia-reperfusion injury in isolated canine gracilis model.
Arch Surg. 1988;123(4):470-472.
27. Shin CS, Han JU, Kim JL, et al. Heparin attenuated neutrophil
infi ltration but did not affect renal injury induced by ischemia
reperfusion. Yonsei Med J. 1997;38(3):133-141.
28. The STILE Investigators. Results of a prospective randomized trial evaluating surgery versus thrombolysis for ischemia
of the lower extremity. The STILE trial. Ann Surg. 1994;220(3):
251-266.
29. Comerota AJ, Weaver FA, Hosking JD, et al. Results of a prospective, randomized trial of surgery versus thrombolysis for occluded
lower extremity bypass grafts. Am J Surg. 1996;172(2):105-112.
30. Ouriel K, Veith FJ, Sasahara AA. A comparison of recombinant
urokinase with vascular surgery as initial treatment for acute
arterial occlusion of the legs. Thrombolysis or Peripheral Arterial Surgery (TOPAS) Investigators. N Engl J Med. 1998;338(16):
1105-1111.
31. Brown J, Lethaby A, Maxwell H, Wawrzyniak AJ, Prins MH.
Antiplatelet agents for preventing thrombosis after peripheral arterial bypass surgery. Cochrane Database Syst Rev.
2008;(4):CD000535.
32. Tangelder MJ, Lawson JA, Algra A, Eikelboom BC. Systematic
review of randomized controlled trials of aspirin and oral anticoagulants in the prevention of graft occlusion and ischemic
events after infrainguinal bypass surgery. J Vasc Surg. 1999;30(4):
701-709.
33. Collaborative overview of randomised trials of antiplatelet
therapyII: Maintenance of vascular graft or arterial patency
by antiplatelet therapy. Antiplatelet Trialists Collaboration.
BMJ. 1994;308(6922):159-168.
34. Tangelder MJ, McDonnel J, Van Busschbach JJ, et al. Quality of
life after infrainguinal bypass graft ing surgery. Dutch Bypass
Oral Anticoagulants or Aspirin (BOA) Study Group. J Vasc Surg.
1999;29(5):913-919.
35. Johnson WC, Williford WO. Benefits, morbidity, and mortality associated with long-term administration of oral anticoagulant therapy to patients with peripheral arterial bypass
procedures: a prospective randomized study. J Vasc Surg. 2002;
35(3):413-421.
36. Ishizaka N, Taguchi J, Kimura Y, et al. Effects of a single local
administration of cilostazol on neointimal formation in ballooninjured rat carotid artery. Atherosclerosis. 1999;142(1):41-46.
37. Kim MJ, Park KG, Lee KM, et al. Cilostazol inhibits vascular
smooth muscle cell growth by downregulation of the transcription factor E2F. Hypertension. 2005;45(4):552-556.
38. Yamamoto K, Onoda K, Sawada Y, et al. Locally applied cilostazol suppresses neointimal hyperplasia and medial thickening
in a vein graft model. Ann Thorac Cardiovasc Surg. 2007;13(5):
322-330.
39. Fujinaga K, Onoda K, Yamamoto K, et al. Locally applied cilostazol suppresses neointimal hyperplasia by inhibiting tenascin-C
synthesis and smooth muscle cell proliferation in free artery
grafts. J Thorac Cardiovasc Surg. 2004;128(3):357-363.
40. Belch JJ, Dormandy J, Biasi BM, et al. Results of the randomized, placebo-controlled clopidogrel and acetylsalicylic acid in
bypass surgery for peripheral arterial disease (CASPAR) trial.
J Vasc Surg. 2010;52(4):825-833.
41. Clagett GP, Sobel M, Jackson MR, Lip GY, Tangelder M, Verhaeghe R. Antithrombotic therapy in peripheral arterial occlusive
disease: the Seventh ACCP Conference on Antithrombotic and
Thrombolytic Therapy. Chest. 2004;126(3 Suppl):609S-626S.

5/22/2012 6:02:54 PM

740

Surgery: Evidence-Based Practice

42. Kretschmer G, Herbst F, Prager M, et al. A decade of oral anticoagulant treatment to maintain autologous vein grafts for femoropopliteal atherosclerosis. Arch Surg. 1992;127(9):1112-1115.
43. Algra A, Tangelder MJ, Lawson JA, Eikelboom BC. Interpretation of Dutch BOA trial. Dutch Bypass Oral anticoagulants or
Aspirin study group. Lancet. 2000;355(9210):1186-1187.
44. Sarac TP, Huber TS, Back MR, et al. Warfarin improves the outcome of infrainguinal vein bypass grafting at high risk for failure. J Vasc Surg. 1998;28(3):446-457.

PMPH_CH92.indd 740

45. Brumberg RS, Back MR, Armstrong PA, et al. The relative
importance of graft surveillance and warfarin therapy in infrainguinal prosthetic bypass failure. J Vasc Surg. 2007;46(6):11601166.
46. Jackson MR, Johnson WC, Williford WO, Valentine RJ, Clagett
GP. The effect of anticoagulation therapy and graft selection on
the ischemic consequences of femoropopliteal bypass graft occlusion: results from a multicenter randomized clinical trial. J Vasc
Surg. 2002;35(2):292-298.

5/22/2012 6:02:54 PM

Commentary on
Management of Graft Occlusion
Daniel B. Walsh

Question 3: The authors emphasis that technical errors only


account for a minority of early graft failures is critically important
and, even today, rarely recognized in surgical writing. However,
even in this text, the focus on technical error as a cause of graft
failure is a problem. The authors, in answering Question 3 (What
is the treatment for early graft occlusion?) recommend thrombectomy with operative revision and correction of technical error.
In my opinion, the reasons most grafts fail are not because the
procedure was performed incorrectly. Early failure is very often
predictable, particularly among patients operated upon for critical limb ischemia, because a surgeons ability to evaluate vein conduit and target artery preoperatively remains imperfect. Typically
one or the other or the combination of the two is not adequate to
achieve successful long-term revascularization. Often, this can be
anticipated at operation. Completion studies at the time of bypass
placement are vital to ensure technical adequacy and to decide
at operation whether early failure is expected.1,2 Unexpected failure should provoke immediate return to the operating room (OR)
for thrombectomy and revision. However, the authors themselves
point out the morbidity and mortality associated with early return
to the OR. A risk burdened patient with a poor vein and/or poor,
calcified single tibial runoff vessel would be unlikely to benefit
from further attempts at limb salvage using arm vein, composite vein, or prosthetic to bypass to the same or even less appealing targets. Clearly, a no return policy to the OR leads to a high
amputation rate. I would argue that many of those patients would
have been better served by amputation when the conduit and/or
the target artery were demonstrated to be inadequate either preoperatively or after the anticipated initial bypass failure.
Question 4: My own view of the role of thrombolysis in the
treatment of late graft failure differs somewhat from the authors,
despite their excellent review of the pertinent randomized trials.
You could lyse a thrombosed femoral above-knee popliteal bypass
using PTFE; but, why would you want to? Surgical thrombectomy is quicker, cheaper, and allows immediate treatment of the
causative lesion using standard open or intraoperative endovascular techniques. In the case of late thrombosis of a vein graft,
the calculation is more complex. In the review of our experience,
grafts patent for more than 12 months, particularly in nondiabetic
patients, had a predicted patency of 44% after successful lysis,
compared with grafts patent for less than 12 months in diabetic
patients (~0%).3 For patients with vein available for use as conduit
and reasonable bypass targets, patency and limb salvage with a

repeat or secondary bypass have been reported to be at least as


good as these.4,5 Unfortunately, in our experience, such patients
are rare. Therefore, we recommend lysis in vein grafts that have
been patent for a prolonged length of time who have no other
bypass options. In patients whose grafts fail in less than a year,
patency rates are very poor, no matter how salvage is attempted.
Happily, despite poor long-term patency, limb salvage in the late
failure lysis group is relatively good (6367%).3,6 Of course, 90% of
these patients require intervention after successful lysis.3,6
Question 5: To the authors discussion I would add this quote
from a recent review article on the topic: The studies looking at
the clinical effect of statins in patients with claudication or after
infrainguinal bypass surgery are all observational and provide
a variable picture, but the majority reports a beneficial effect on
walking performance, patency, and survival in the statin users.7
Question 6: I cannot quibble with the authors review of the literature. However, it is clear from the collected experience of vascular
surgeons that not all bypasses need be anticoagulated to achieve lifetime patency. As with everything in surgery, patient selection is the
key. We would anticoagulate with warfarin any patient with a disadvantaged graft, that is, any graft that had previously failed or had
disadvantaged conduit or target artery as defined by the surgeon.

REFERENCES
1. Johnson BL, Bandyk DF, Back MR, Avino AJ, Roth SM. Intraoperative duplex monitoring of infrainguinal vein bypass procedures.
J Vasc Surg. 2000;31(4):678-690.
2. Rzucidlo EM, Walsh DB, Powell RJ, et al. Prediction of early graft
failure with intraoperative completion duplex ultrasound scan.
J Vasc Surg. 2002;36(5):975-981.
3. Nackman GB, Walsh DB, Fillinger MF, et al. Thrombolysis of
occluded infrainguinal vein grafts: predictors of outcome. J Vasc
Surg. 1997;25:1023-1032.
4. Whittemore AD, Clowes AW, Couch NP, Mannic K JA. Secondary
femoropopliteal reconstruction. Ann Surg. 1981;193:35-42.
5. Hye RJ, Turner C, Valji K, et al. Is thrombolysis of occluded popliteal
and tibial bypass grafts worthwhile? J Vasc Surg. 1994;20:588-597.
6. Belkin M, Donaldson MC, Whittemore AD, et al. Observations
on the use of thrombolytic agents for thrombotic occlusion of
infrainguinal vein grafts. J Vasc Surg. 1990;11:289-296.
7. Stalenhoef AFH. The benefit of statins in non-cardiac vascular
surgery patients. J Vasc Surg. 2009;49:260-265.
741

PMPH_CH92.indd 741

5/22/2012 6:02:54 PM

CHAPTER 93

Vascular Access for Hemodialysis


Yazan Duwayri and Brian G. Rubin

INTRODUCTION

catheters and multiple phlebotomies that can potentially damage


the cephalic vein that should be preserved for access creation.
Close collaboration between the nephrologist and surgeon is
of utmost importance. KDOQI guidelines call for creation of an
autologous AVF at least 6 months before hemodialysis initiation.
This allows for adequate time for AVF maturation or for secondary procedures. If a primary fistula cannot be created, arteriovenous graft (AVG) placement should be performed 3 to 6 weeks
prior to hemodialysis initiation, due to the limited life span of the
prosthetic grafts.
In surgical planning, it has been found that preoperative
sonographic vascular mapping increases the total proportion of
patients dialyzing with fistulae.9-12 Vein diameter threshold of
2 to 2.5 mm12-14 and arterial diameter threshold of 1.6 to 2 mm12,15
should guide the placement of AVFs (Level 2b evidence; Grade B
recommendation).

Since the advent of hemodialysis as treatment for renal failure


in the 1940s, establishing vascular access to deliver this treatment has posed challenges for the treating renal specialists and
surgeons. Vascular access evolved initially from the impractical
separate arterial and venous cannulation to the indwelling external arteriovenous shunt, popularized by Scribner et al. in 1960. In
1966, Brescia and colleagues1 published their technique of arteriovenous fistula (AVF) creation to allow for frequent puncture of the
arterialized vein while at the same time providing high blood flow
into the hemodialysis machine. Tunneled prosthetic grafts subsequently appeared to provide an alternative when primary AVF
construction is not feasible. The choice of the optimal method for
hemodialysis delivery as well as the creation and maintenance
of vascular access are important aspects that add to the difficult
management of patients with end-stage renal disease (ESRD). The
literature is rich in material that compares the effectiveness of each
method and provides guidelines for the management of complications that may arise. The National Kidney Foundation, Kidney
Disease Outcomes Quality Initiative (KDOQI), has published a
set of clinical practice recommendations and guidelines to guide
the management decisions in patients with ESRD. These guidelines were last updated in 2006 and were taken into consideration
while writing this text.2 The primary goal of this chapter is to provide an evidence-based overview of dialysis vascular access.

AUTOLOGOUS FISTULA VERSUS


PROSTHETIC GRAFT
The ideal vascular access can be cannulated repeatedly, provides
enough flow for adequate hemodialysis, has good long-term patency, and low risk of complications. The autologous AVF comes
closest to fulfi lling these criteria. The first KDOQI guidelines
published in 1997 established the importance of placing autologous AVFs in long-term dialysis patients. This was subsequently
referred to as the fistula first initiative. When compared to
AVGs, AVFs have a lower incidence of thrombosis,16 lower rates
of infection,17 and an overall lower access-event rate.16,18 Therefore,
AVFs require fewer secondary interventions16,18 and last longer
than AVG.16,18,19 In general, AVF creation and maintenance cost
less than AVG.20,21 The use of AVF has also been found to reduce
mortality in ESRD patients compared to AVG22-25 (Level 2a evidence; Grade B recommendation).
Despite many attempts to overcome the disadvantages of
prosthetic grafts, no significant improvements have been achieved

PREOPERATIVE PLANNING
Planning for dialysis access can start prior to surgical referral.
When dialysis is anticipated in a patient with worsening renal
function, subclavian vein catheterization3-5 and PICC line (Peripherally Inserted Central Catheters) placement6-8 should be avoided.
This is due to the high incidence of venous outflow thrombosis or
stenosis secondary to intimal hyperplasia at the site of line insertion. The same principle applies to standard peripheral intravenous
742

PMPH_CH93.indd 742

5/22/2012 6:03:49 PM

Vascular Access for Hemodialysis

in the patency or complication rates. So far, no material, including


polyurethane26 and cryopreserved vein,27,28 has shown improved
patency rates over polytetrafluoroethylene (PTFE). Polyurethane
AVGs have the advantage of early cannulation.26 Biografts, on the
other hand, have a higher risk of aneurysmal degeneration.28 AVG
patency is also independent of the manufacturer26,29,30 or graft
thickness,31 but modest improvement of patency rates have been
achieved by the presence of a venous outflow cuff32-34 and with
arterial taper (68-mm graft vs. 6-mm straight graft)35 (Level 1b
evidence; Grade B recommendation).
1. Where is the best anatomic location for dialysis access
creation?
There are no trials comparing different anatomic locations for
AVF and AVG placement. However, KDOQI guidelines state that
access should be placed distally in the nondominant upper extremity whenever possible. AVF and AVG construction in the lower
extremity is associated with greater morbidity and is therefore
considered only after exhaustion of upper extremity options.36-38
The wrist radiocephalic AVF is the conduit of first choice. It
is simple to create and has a lower rate of late complications (steal
and infection) than other forms of access.39,40 The disadvantage of
this location, however, is a high primary failure rate.23
The elbow brachiocephalic AVF should be the second choice,
and a transposed brachiobasilic AVF is the third choice. Similarly,
AVG placement should follow a similar pattern of preferentially
using a more distal location to preserve proximal vessels for later
use (Level 4 evidence; Grade C/D recommendation).
2. When are AVFs and AVGs considered mature and ready
for use?
Fistulae should show evidence of maturation at 4 to 6 weeks after
construction. Failure of maturation by this time should be investigated by ultrasound, which has a higher likelihood of detecting
correctible abnormalities than clinical evaluation.41,42 Accessing
fistulae before maturation increases the likelihood of long-term
failure.43 Robbin et al. found that the predictive power of AVF
maturation can be increased by ultrasound evaluation using a
venous diameter of 4 mm and a flow volume of 500 mL/min42
(Level 2b evidence; Grade B recommendation).
In the management of nonmature AVFs, arterial inflow problems can be corrected with percutaneous angioplasty.44 Ligation
of multiple venous branches can also improve the maturation of
the main outflow vein. Isometric hand exercises can increase the
diameter of forearm veins and are therefore encouraged in the
postoperative period.45-47
AVGs should be ready for use within 3 weeks from placement.
This time allows for the postoperative swelling to resolve.
3. What is the role of postoperative surveillance of dialysis
access?
Loss of vascular access usually occurs as a result of thrombosis.
Thrombosis usually results from stenosis that develops over variable periods of time. Surveillance and correction of this problem
can therefore prevent access loss. In AVG, the stenosis occurs at
the venous anastomosis as a result of intimal hyperplasia.48-50
Measurement of intra-access pressure in the AVFs and AVGs
provides an accurate assessment of the degree of stenosis and

PMPH_CH93.indd 743

743

therefore is useful in dialysis access surveillance.51-54 KDOQI


workgroup therefore recommends measuring static intra-access
pressures serially at 2-week intervals to detect trends, since elevation of static pressures has been shown to correlate with a higher
likelihood for access thrombosis.
Doppler flow measurements in the AVFs and AVGs55-58 and
intradialytic access flow measurements59-66 are also excellent determinants of stenosis and thrombosis risk. Overall, the access is at
a higher risk when access blood flow is <600 to 800 mL/min in
AVG,59-66 or when there is a decrease of 25% in blood flow in otherwise stable grafts.57,58,62,66
Preemptive percutaneous balloon angioplasty is indicated in
stenoses >50% on angiogram, when the stenosis is associated with
hemodynamic, functional, or clinical abnormailty.54,67
Nonrandomized studies and studies that used historical control data showed that prospective surveillance of AVFs and AVGs
and intervention can reduce the rate of access thrombosis.54,67-70
Two randomized controlled trials (RCTs) showed improved vascular access survival with surveillance and preemptive intervention
in AVFs.71,72 RCTs, however, failed to show a similar effect in AVGs
surveillance73 (Level 1b evidence; Grade A recommendation).
4. How should each of the following common complications of
vascular access be treated?
(a) Swelling: Early postoperative swelling, within the first month,
is common and usually resolves with arm elevation. Late
swelling, on the other hand, is secondary to venous outflow
obstruction resulting from neointimal hyperplasia at the
venous anastomosis or from central venous obstruction from
prior catheter use. Percutaneous balloon angioplasty is the
treatment of choice. Stent grafts placed at the treated venous
anastomosis in AVGs have been shown to improve long-term
patency in a recent RCT.74 Stents after balloon angioplasty
have also been shown to improve long-term patency when
used in central venous stenosis75-77 (Level 1b evidence; Grade
B recommendation).
(b) Thrombosis: This is the most common etiology of access loss.
Thrombectomy should be attempted in thrombosed AVF and
AVG to salvage the vascular access.78 In a retrospective study,
primary patency after AVF thrombectomy was 50% (14% in
AVG) and secondary patency was 80% (83% in AVG).79 An
underlying stenosis in the venous outflow is detected and treated
in almost all cases. Thrombectomy can be performed in an open
or percutaneous fashion. Studies comparing both methods are
conflicting and do not show superiority of one method over the
other80-84 (Level 4 evidence; Grade C recommendation).
(c) Steal: This complication results from compromising arterial
flow distal to the fistula or graft. Symptoms can range from cold
sensitivity and pain to severe ischemia and digital ulceration.
These changes can be reversed after ligation of the vascular
access. However, satisfactory results can also be obtained
using the DRIL (Distal Revascularization and Interval
Ligation) procedure in normal flow AVFs and AVGs85,86 and
the proximal anastomosis technique in low flow AVFs and
AVGs.87 These procedures allow for preservation of the dialysis
access (Level 4 evidence; Grade C recommendation).
(d) Pseudoaneurysm formation: This complication results from
graft or vein degeneration as a result of repeated single site
cannulation. This can be treated by resection of the involved

5/22/2012 6:03:50 PM

744

Surgery: Evidence-Based Practice

segment and graft interposition.88,89 Successful results have also


been reported with stent graft placement in the aneurysmal
segment.89
(e) Infection: KDOQI guidelines recommend that treatment
of infected primary AVFs is nonsurgical and constitutes of
antibiotic therapy for 6 weeks in a manner similar to bacterial

endocarditis. On the other hand, infected prosthetic AVGs


should be surgically excised. Total excision is recommended in the presence of systemic sepsis, whereas subtotal
or partial graft excision can be sufficient in the absence
of systemic bacteremia90,91 (Level 4 evidence; Grade C/D
recommendation).

Clinical Question Summary


Question

Answer

Levels of
Evidence

Grade of
Recommendation

References

What is the preferred


location for
placement of
temporary or
tunneled central
venous catheters
in renal failure
patients?

Right internal jugular catheters are the


preferred choice. This prevents the high
incidence of subclavian vein stenosis or
thrombosis associated with subclavian
vein catheters, and the high incidence
of arm vein thrombosis with PICC line
placement. These veins can therefore be
preserved for vascular access creation.

2b
Multiple
retrospective
cohort studies
and casecontrol studies

3-8

How long prior to


start of dialysis
should dialysis
access be created?

Autologous AVFs should be created 6


months prior to start of dialysis. AVGs,
on the other hand, should be placed
3 to 6 weeks prior to dialysis initiation.

5
Expert opinion

KDOQI
workgroup
consensus

Is preoperative
sonographic vein
mapping necessary
for surgical
planning?

Sonographic evaluation of the size and


patency of the upper extremity veins
can increase the proportion of patients
with functional autologous AVF. The
vein diameter threshold for fistula
creation is 2 to 2.5 mm. The arterial
diameter threshold is 1.5 mm.

2b
Retrospective
cohort studies

9-15

Which vascular access


modality is better:
AVF or AVG?

AVF is better due to a lower complication


rate, fewer secondary intervention rate,
and better overall patency.

2a
Systemic review
of cohort
studies

16-25

In AVG construction,
is the patency
improved by using
one type of graft
over another?

AVG patency is the same regardless of


graft material, graft thickness, or
manufacturer.
AVG patency is modestly improved with a
venous outflow cuff or with the use of
tapered grafts (e.g., 6 to 8 mm)

1b
Individual RCTs

26-35

Where is the best


anatomic location
for vascular access
creation?

Access should be placed distally in the


nondominant upper extremity whenever
possible.
Lower extremity AVFs and AVGs have a
higher complication rate.

5 (Expert
opinion)

DOQI
guidelines

4 (Case series)

36-38

How should AVF


readiness for use be
assessed?

The predictive power of AVF maturation


can be increased by sonographic
evaluation using a venous diameter of
4 mm and a flow volume of 500mL/min

2b
Individual cohort
study

42

How can maturation


of the AVF be
improved?

Percutaneous angioplasty of the arterial


inflow or the vein can improve
maturation rate of AVF. Isometric hand
exercises have also been found to be
beneficial.

4
(Case series)

44-47

(Continued)

PMPH_CH93.indd 744

5/22/2012 6:03:50 PM

Vascular Access for Hemodialysis

745

(Continued)
Question

Answer

Levels of
Evidence

Is surveillance of
functioning
dialysis access
recommended?

Prospective surveillance of AVFs and AVGs


and intervention can reduce the rate of
access thrombosis and improve patency.

1b
(RCTs)

67-72

What is the best


treatment modality
for ipsilateral
venous outlet
obstruction?

Central venous stenosis is best treated


by percutaneous balloon angioplasty
followed by stent placement.
In AVG, stenosis at the venous anastomosis
is best treated by angioplasty followed
by stent graft placement.

4 (Case series)

75-77

1b (RCT)

74

Should failed
(thrombosed) AVF
and AVG undergo
thrombectomy?

Thrombectomy, percutaneous or open,


followed by treatment of inflow or
outflow lesions can prolong the
functional use of the AVF or AVG.

4 (Case series)

78-84

What is the
recommended
treatment for
arterial steal
secondary to AVF
or AVG placement?

Satisfactory results have been reported


using the DRIL procedure. Refractory
steal should be treated by ligation of the
vascular access.

4 (Case series)

85-86

What is the
recommended
treatment for
AVF or AVG
pseudoaneurysms?

Operative resection and stent graft


placement have had equally acceptable
results in preventing rupture and
maintaining access patency.

4 (Case series)

88-89

How should infection


of autologous AVF
be treated?

This is treated in a manner similar to


bacterial endocarditis with a 6-week
course of IV antibiotics.

5 (Expert
opinion)

KDOQI
guidelines

How should infection


of prosthetic AVG
be treated?

In the presence of systemic sepsis, all


prosthetic material should be removed.
However, localized graft infections can
be treated with partial resection

4 (Case series)

90-91

REFERENCES
1. Brescia MJ, Cimino JE, Appel K, Hurwich BJ. Chronic hemodialysis using venipuncture and a surgically created arteriovenous
fistula. N Engl J Med. 1966;275(20):1089-1092.
2. National Kidney Foundation. KDOQI Clinical Practice Guidelines and Clinical Practice Recommendations for 2006 Updates:
Hemodialysis Adequacy, Peritoneal Dialysis Adequacy and Vascular Access. Am J Kidney Dis. 2006(Suppl 1);48:S1-S322,
3. Barrett N, Spencer S, McIvor J, Brown EA. Subclavian stenosis:
a major complication of subclavian dialysis catheters. Nephrol
Dial Transplant. 1988;3(4):423-425.
4. Marx AB, Landmann J, Harder FH. Surgery for vascular access.
Curr Probl Surg. 1990;27(1):1-48.
5. Schwab SJ, Quarles LD, Middleton JP, Cohan RH, Saeed M,
Dennis VW. Hemodialysis-associated subclavian vein stenosis.
Kidney Int. 1988;33(6):1156-1159.
6. Gonsalves CF, Eschelman DJ, Sullivan KL, DuBois N, Bonn J.
Incidence of central vein stenosis and occlusion following upper
extremity PICC and port placement. Cardiovasc Intervent Radiol.
2003;26(2):123-127.

PMPH_CH93.indd 745

Grade of
Recommendation

References

7. Abdullah BJ, Mohammad N, Sangkar JV, et al. Incidence of upper


limb venous thrombosis associated with peripherally inserted
central catheters (PICC). Br J Radiol. 2005;78(931):596-600.
8. Allen AW, Megargell JL, Brown DB, et al. Venous thrombosis
associated with the placement of peripherally inserted central
catheters. J Vasc Interv Radiol. 2000;11(10):1309-1314.
9. Allon M, Lockhart ME, Lilly RZ, et al. Effect of preoperative
sonographic mapping on vascular access outcomes in hemodialysis patients. Kidney Int. 2001;60(5):2013-2020.
10. Ascher E, Gade P, Hingorani A, et al. Changes in the practice
of angioaccess surgery: impact of dialysis outcome and quality
initiative recommendations. J Vasc Surg. 2000;31(1 Pt 1):84-92.
11. Gibson KD, Caps MT, Kohler TR, et al. Assessment of a policy to
reduce placement of prosthetic hemodialysis access. Kidney Int.
2001;59(6):2335-2345.
12. Silva MB, Jr., Hobson RW, 2nd, Pappas PJ, et al. A strategy for increasing use of autogenous hemodialysis access procedures: impact
of preoperative noninvasive evaluation. J Vasc Surg. 1998;27(2):
302-307; discussion 307-308.
13. Mendes RR, Farber MA, Marston WA, Dinwiddie LC, Keagy
BA, Burnham SJ. Prediction of wrist arteriovenous fi stula

5/22/2012 6:03:50 PM

746

14.

15.

16.

17.
18.

19.

20.

21.

22.

23.

24.

25.

26.

27.

28.

29.

30.

Surgery: Evidence-Based Practice

maturation with preoperative vein mapping with ultrasonography. J Vasc Surg. 2002;36(3):460-463.
Malovrh M. Approach to patients with end-stage renal disease
who need an arteriovenous fistula. Nephrol Dial Transplant.
2003;18(Suppl 5):v50-52.
Wong V, Ward R, Taylor J, Selvakumar S, How TV, Bakran A. Factors associated with early failure of arteriovenous fistulae for haemodialysis access. Eur J Vasc Endovasc Surg. 1996;12(2):207-213.
Perera GB, Mueller MP, Kubaska SM, Wilson SE, Lawrence PF,
Fujitani RM. Superiority of autogenous arteriovenous hemodialysis access: maintenance of function with fewer secondary
interventions. Ann Vasc Surg. 2004;18(1):66-73.
Nassar GM, Ayus JC. Infectious complications of the hemodialysis access. Kidney Int. 2001;60(1):1-13.
Huber TS, Carter JW, Carter RL, Seeger JM. Patency of autogenous and polytetrafluoroethylene upper extremity arteriovenous hemodialysis accesses: a systematic review. J Vasc Surg.
2003;38(5):1005-1011.
Pisoni RL, Young EW, Dykstra DM, et al. Vascular access use in
Europe and the United States: results from the DOPPS. Kidney
Int. 2002;61(1):305-316.
Mehta S. Statistical summary of clinical results of vascular access
procedures for hemodialysis. In: Sommer BG, Henry ML, eds.
Vascular Access for Hemodialysis-II. 2nd ed. Chicago, IL: Gore;
1991:145-157.
The Cost Effectiveness of Alternative Types of Vascular Access and
the Economic Cost of ESRD. Bethesda, MD: National Institute of
Health, National Institute of Diabetes and Digestive and Kidney
Diseases; 1995:139-157.
Dhingra RK, Young EW, Hulbert-Shearon TE, Leavey SF, Port
FK. Type of vascular access and mortality in U.S. hemodialysis
patients. Kidney Int. 2001;60(4):1443-1451.
Woods JD, Port FK. The impact of vascular access for haemodialysis on patient morbidity and mortality. Nephrol Dial Transplant. 1997;12(4):657-659.
Xue JL, Dahl D, Ebben JP, Collins AJ. The association of initial
hemodialysis access type with mortality outcomes in elderly
Medicare ESRD patients. Am J Kidney Dis. 2003;42(5):1013-1019.
Polkinghorne KR, McDonald SP, Atkins RC, Kerr PG. Vascular
access and all-cause mortality: a propensity score analysis. J Am
Soc Nephrol. 2004;15(2):477-486.
Glickman MH, Stokes GK, Ross JR, et al. Multicenter evaluation
of a polytetrafluoroethylene vascular access graft as compared
with the expanded polytetrafluoroethylene vascular access graft
in hemodialysis applications. J Vasc Surg. 2001;34(3):465-472;
discussion 472-463.
Matsuura JH, Johansen KH, Rosenthal D, Clark MD, Clarke KA,
Kirby LB. Cryopreserved femoral vein grafts for difficult hemodialysis access. Ann Vasc Surg. 2000;14(1):50-55.
Bosman PJ, Blankestijn PJ, van der Graaf Y, Heintjes RJ, Koomans
HA, Eikelboom BC. A comparison between PTFE and denatured
homologous vein grafts for haemodialysis access: a prospective
randomised multicentre trial. The SMASH Study Group. Study
of Graft Materials in Access for Haemodialysis. Eur J Vasc Endovasc Surg. 1998;16(2):126-132.
Hurlbert SN, Mattos MA, Henretta JP, et al. Long-term patency
rates, complications and cost-effectiveness of polytetrafluoroethylene (PTFE) grafts for hemodialysis access: a prospective study
that compares Impra versus Gore-tex grafts. Cardiovasc Surg.
1998;6(6):652-656.
Kaufman JL, Garb JL, Berman JA, Rhee SW, Norris MA,
Friedmann P. A prospective comparison of two expanded

PMPH_CH93.indd 746

31.

32.

33.

34.

35.

36.

37.

38.

39.

40.

41.

42.

43.

44.

45.

46.

polytetrafluoroethylene graft s for linear forearm hemodialysis access: does the manufacturer matter? J Am Coll Surg. 1997;
185(1):74-79.
Lenz BJ, Veldenz HC, Dennis JW, Khansarinia S, Atteberry
LR. A three-year follow-up on standard versus thin wall ePTFE
grafts for hemodialysis. J Vasc Surg. 1998;28(3):464-470; discussion 470.
Freischlag JA. Regarding the effect of venous anastomosis Tyrell
vein patch collar on the primary patency of arteriovenous grafts
in patients undergoing hemodialysis and effects of a venous
cuff at the venous anastomosis of polytetrafluoroethylene grafts
for hemodialysis vascular access. J Vasc Surg. 2000;32(6):12351236.
Lemson MS, Tordoir JH, van Det RJ, et al. Effects of a venous cuff
at the venous anastomosis of polytetrafluoroethylene grafts for
hemodialysis vascular access. J Vasc Surg. 2000;32(6):1155-1163.
Sorom AJ, Hughes CB, McCarthy JT, et al. Prospective, randomized evaluation of a cuffed expanded polytetrafluoroethylene graft for hemodialysis vascular access. Surgery. 2002;132(2):
135-140.
Garcia-Pajares R, Polo JR, Flores A, Gonzalez-Tabares E, Solis
JV. Upper arm polytetrafluoroethylene grafts for dialysis access.
Analysis of two different graft sizes: 6 mm and 6-8 mm. Vasc
Endovascular Surg. 2003;37(5):335-343.
Cull JD, Cull DL, Taylor SM, et al. Prosthetic thigh arteriovenous
access: outcome with SVS/AAVS reporting standards. J Vasc
Surg. 2004;39(2):381-386.
Gradman WS, Cohen W, Haji-Aghaii M. Arteriovenous fistula
construction in the thigh with transposed superficial femoral
vein: our initial experience. J Vasc Surg. 2001;33(5):968-975.
Geenen IL, Nyilas L, Stephen MS, Makeham V, White GH, Verran DJ. Prosthetic lower extremity hemodialysis access grafts
have satisfactory patency despite a high incidence of infection.
J Vasc Surg. 2010;52(6):1546-1550.
Kherlakian GM, Roedersheimer LR, Arbaugh JJ, Newmark KJ,
King LR. Comparison of autogenous fistula versus expanded
polytetrafluoroethylene graft fistula for angioaccess in hemodialysis. Am J Surg. 1986;152(2):238-243.
Kinnaert P, Vereerstraeten P, Toussaint C, Van Geertruyden J.
Nine years experience with internal arteriovenous fistulas for
haemodialysis: a study of some factors influencing the results. Br
J Surg. 1977;64(4):242-246.
Malik J, Slavikova M, Malikova H, Maskova J. Many clinically
silent access stenoses can be identified by ultrasonography.
J Nephrol. 2002;15(6):661-665.
Robbin ML, Chamberlain NE, Lockhart ME, et al. Hemodialysis arteriovenous fistula maturity: US evaluation. Radiology.
2002;225(1):59-64.
Rayner HC, Pisoni RL, Gillespie BW, et al. Creation, cannulation and survival of arteriovenous fistulae: data from the Dialysis
Outcomes and Practice Patterns Study. Kidney Int. 2003;63(1):
323-330.
Guerra A, Raynaud A, Beyssen B, Pagny JY, Sapoval M, Angel
C. Arterial percutaneous angioplasty in upper limbs with vascular access devices for haemodialysis. Nephrol Dial Transplant.
2002;17(5):843-851.
Leaf DA, MacRae HS, Grant E, Kraut J. Isometric exercise
increases the size of forearm veins in patients with chronic renal
failure. Am J Med Sci. 2003;325(3):115-119.
Oder TF, Teodorescu V, Uribarri J. Effect of exercise on the diameter of arteriovenous fistulae in hemodialysis patients. ASAIO J.
2003;49(5):554-555.

5/22/2012 6:03:50 PM

Vascular Access for Hemodialysis

47. Rus RR, Ponikvar R, Kenda RB, Buturovic-Ponikvar J. Effect


of local physical training on the forearm arteries and veins in
patients with end-stage renal disease. Blood Purif. 2003;21(6):
389-394.
48. Hakim R, Himmelfarb J. Hemodialysis access failure: a call to
action. Kidney Int. 1998;54(4):1029-1040.
49. Haruguchi H, Teraoka S. Intimal hyperplasia and hemodynamic
factors in arterial bypass and arteriovenous graft s: a review.
J Artif Organs. 2003;6(4):227-235.
50. Kelly BS, Heffelfinger SC, Whiting JF, et al. Aggressive venous
neointimal hyperplasia in a pig model of arteriovenous graft
stenosis. Kidney Int. 2002;62(6):2272-2280.
51. Sullivan KL, Besarab A, Bonn J, Shapiro MJ, Gardiner GA, Jr.,
Moritz MJ. Hemodynamics of failing dialysis grafts. Radiology.
1993;186(3):867-872.
52. Besarab A, al-Saghir F, Alnabhan N, Lubkowski T, Frinak S.
Simplified measurement of intra-access pressure. ASAIO J.
1996;42(5):M682-687.
53. Besarab A, Dorrell S, Moritz M, Michael H, Sullivan K. Determinants of measured dialysis venous pressure and its relationship to true intra-access venous pressure. ASAIO Trans.
1991;37(3):M270-271.
54. Schwab SJ, Raymond JR, Saeed M, Newman GE, Dennis PA,
Bollinger RR. Prevention of hemodialysis fistula thrombosis.
Early detection of venous stenoses. Kidney Int. 1989;36(4):
707-711.
55. Bay WH, Henry ML, Lazarus JM, Lew NL, Ling J, Lowrie EG.
Predicting hemodialysis access failure with color flow Doppler
ultrasound. Am J Nephrol. 1998;18(4):296-304.
56. Shackleton CR, Taylor DC, Buckley AR, Rowley VA, Cooperberg
PL, Fry PD. Predicting failure in polytetrafluoroethylene vascular access grafts for hemodialysis: a pilot study. Can J Surg.
1987;30(6):442-444.
57. Wang E, Schneditz D, Levin NW. Predictive value of access
blood flow and stenosis in detection of graft failure. Clin Nephrol. 2000;54(5):393-399.
58. May RE, Himmelfarb J, Yenicesu M, et al. Predictive measures
of vascular access thrombosis: a prospective study. Kidney Int.
1997;52(6):1656-1662.
59. Lindsay RM, Blake PG, Malek P, Posen G, Martin B, Bradfield E.
Hemodialysis access blood flow rates can be measured by a differential conductivity technique and are predictive of access
clotting. Am J Kidney Dis. 1997;30(4):475-482.
60. Rehman SU, Pupim LB, Shyr Y, Hakim R, Ikizler TA. Intradialytic serial vascular access flow measurements. Am J Kidney Dis.
1999;34(3):471-477.
61. Hoeben H, Abu-Alfa AK, Reilly RF, Aruny JE, Bouman K, Perazella MA. Vascular access surveillance: evaluation of combining
dynamic venous pressure and vascular access blood flow measurements. Am J Nephrol. 2003;23(6):403-408.
62. Neyra NR, Ikizler TA, May RE, et al. Change in access blood
flow over time predicts vascular access thrombosis. Kidney Int.
1998;54(5):1714-1719.
63. McCarley P, Wingard RL, Shyr Y, Pettus W, Hakim RM, Ikizler
TA. Vascular access blood flow monitoring reduces access morbidity and costs. Kidney Int. 2001;60(3):1164-1172.
64. Mercadal L, Challier E, Cluzel P, et al. Detection of vascular
access stenosis by measurement of access blood flow from ionic
dialysance. Blood Purif. 2002;20(2):177-181.
65. Sands JJ, Jabyac PA, Miranda CL, Kapsick BJ. Intervention based
on monthly monitoring decreases hemodialysis access thrombosis. ASAIO J. 1999;45(3):147-150.

PMPH_CH93.indd 747

747

66. Schwab SJ, Oliver MJ, Suhocki P, McCann R. Hemodialysis


arteriovenous access: detection of stenosis and response to
treatment by vascular access blood flow. Kidney Int. 2001;59(1):
358-362.
67. Besarab A, Sullivan KL, Ross RP, Moritz MJ. Utility of intraaccess pressure monitoring in detecting and correcting venous
outlet stenoses prior to thrombosis. Kidney Int. 1995;47(5):
1364-1373.
68. Lok CE, Bhola C, Croxford R, Richardson RM. Reducing vascular access morbidity: a comparative trial of two vascular access
monitoring strategies. Nephrol Dial Transplant. 2003;18(6):11741180.
69. Cayco AV, Abu-Alfa AK, Mahnensmith RL, Perazella MA.
Reduction in arteriovenous graft impairment: results of a vascular access surveillance protocol. Am J Kidney Dis. 1998;32(2):
302-308.
70. Sands JJ, Miranda CL. Prolongation of hemodialysis access survival with elective revision. Clin Nephrol. 1995;44(5):329-333.
71. Tessitore N, Mansueto G, Bedogna V, et al. A prospective controlled trial on effect of percutaneous transluminal angioplasty
on functioning arteriovenous fistulae survival. J Am Soc Nephrol. 2003;14(6):1623-1627.
72. Tessitore N, Lipari G, Poli A, et al. Can blood flow surveillance
and pre-emptive repair of subclinical stenosis prolong the useful life of arteriovenous fistulae? A randomized controlled study.
Nephrol Dial Transplant. 2004;19(9):2325-2333.
73. Lumsden AB, MacDonald MJ, Kikeri D, Cotsonis GA, Harker
LA, Martin LG. Prophylactic balloon angioplasty fails to prolong
the patency of expanded polytetrafluoroethylene arteriovenous
grafts: results of a prospective randomized study. J Vasc Surg.
1997;26(3):382-390; discussion 390-382.
74. Haskal ZJ, Trerotola S, Dolmatch B, et al. Stent graft versus balloon angioplasty for failing dialysis-access grafts. N Engl J Med.
2010;362(6):494-503.
75. Maskova J, Komarkova J, Kivanek J, Danes J, Slavikova M.
Endovascular treatment of central vein stenoses and/or occlusions in hemodialysis patients. Cardiovasc Intervent Radiol. 2003;
26(1):27-30.
76. Oderich GS, Treiman GS, Schneider P, Bhirangi K. Stent placement for treatment of central and peripheral venous obstruction: a long-term multi-institutional experience. J Vasc Surg.
2000;32(4):760-769.
77. Shoenfeld R, Hermans H, Novick A, et al. Stenting of proximal
venous obstructions to maintain hemodialysis access. J Vasc
Surg. 1994;19(3):532-538; discussion 538-539.
78. Rajan DK, Clark TW, Simons ME, Kachura JR, Sniderman K.
Procedural success and patency after percutaneous treatment of
thrombosed autogenous arteriovenous dialysis fi stulas. J Vasc
Interv Radiol. 2002;13(12):1211-1218.
79. Turmel-Rodrigues L, Pengloan J, Rodrigue H, et al. Treatment of
failed native arteriovenous fistulae for hemodialysis by interventional radiology. Kidney Int. 2000;57(3):1124-1140.
80. Green LD, Lee DS, Kucey DS. A metaanalysis comparing surgical thrombectomy, mechanical thrombectomy, and pharmacomechanical thrombolysis for thrombosed dialysis grafts. J Vasc
Surg. 2002;36(5):939-945.
81. Marston WA, Criado E, Jaques PF, Mauro MA, Burnham SJ,
Keagy BA. Prospective randomized comparison of surgical versus endovascular management of thrombosed dialysis access
grafts. J Vasc Surg. 1997;26(3):373-380; discussion 380-371.
82. Sands JJ, Patel S, Plaviak DJ, Miranda CL. Pharmacomechanical thrombolysis with urokinase for treatment of thrombosed

5/22/2012 6:03:50 PM

748

83.

84.

85.

86.

Surgery: Evidence-Based Practice

hemodialysis access grafts. A comparison with surgical thrombectomy. ASAIO J. 1994;40(3):M886-888.


Schwartz CI, McBrayer CV, Sloan JH, Meneses P, Ennis WJ.
Thrombosed dialysis grafts: comparison of treatment with transluminal angioplasty and surgical revision. Radiology. 1995;194(2):
337-341.
Beathard GA, Litchfield T. Effectiveness and safety of dialysis
vascular access procedures performed by interventional nephrologists. Kidney Int. 2004;66(4):1622-1632.
Schanzer H, Schwartz M, Harrington E, Haimov M. Treatment of
ischemia due to steal by arteriovenous fistula with distal artery
ligation and revascularization. J Vasc Surg. 1988;7(6):770-773.
Knox RC, Berman SS, Hughes JD, Gentile AT, Mills JL. Distal
revascularization-interval ligation: a durable and effective treatment for ischemic steal syndrome after hemodialysis access.
J Vasc Surg. 2002;36(2):250-255; discussion 256.

PMPH_CH93.indd 748

87. Zanow J, Petzold M, Petzold K, Kruger U, Scholz H. Diagnosis and


differential treatment of ischemia in patients with arteriovenous
vascular access. In: Henry ML, ed. Vascular Access for HemodialysisVII. Chicago, IL: Gore; 2001:201-209
88. Ballard JL, Bunt TJ, Malone JM. Major complications of angioaccess surgery. Am J Surg. 1992;164(3):229-232.
89. Hausegger KA, Tiessenhausen K, Klimpfinger M, Raith J, Hauser
H, Tauss J. Aneurysms of hemodialysis access grafts: treatment
with covered stents: a report of three cases. Cardiovasc Intervent
Radiol. 1998;21(4):334-337.
90. Ryan SV, Calligaro KD, Scharff J, Dougherty MJ. Management
of infected prosthetic dialysis arteriovenous grafts. J Vasc Surg.
2004;39(1):73-78.
91. Deneuville M. Infection of PTFE grafts used to create arteriovenous fistulas for hemodialysis access. Ann Vasc Surg. 2000;
14(5):473-479.

5/22/2012 6:03:50 PM

CHAPTER 94

Atherosclerotic
Renovascular Disease
Eric Hager and Luke Marone

INTRODUCTION TO RENAL
ARTERY STENOSIS

PATHOPHYSIOLOGY OF RENOVASCULAR
HYPERTENSION

Hypertension is exceedingly common in the United States


and is often the principal reason for patients to seek medical
attention from their primary care physician. Analysis of the
National Health and Nutrition Examination Survey over a
10-year period estimated the incidence of hypertension to be
approximately 29% in adults living in the United States.1 The
vast majority of these patients suffer from essential hypertension, and are managed with medical therapy. The presence of
severe or resistant hypertension, malignant hypertension, or onset
of high blood pressure at an early age (less than 30) may be
indicative of renal artery disease as an underlying cause of the
hypertension.
Renal artery stenosis remains the most common cause of
secondary hypertension in the adult population with an incidence of approximately 7% to 32% in hypertensive adults in the
United States.2,3 Because of the insidious nature of the disease,
and the potential for curative intervention, the American College
of Cardiology in conjunction with the American Heart Association released screening guidelines in 2005 for the identification
of renal artery stenosis. These guidelines include (1) the presence
of severe or stage I hypertension (blood pressure above 160/100
mm Hg in patients older than 55); (2) hypertension in patients
with an atrophic kidney (<9 cm in size) or with a size discrepancy of >1.5 cm between the two kidneys; (3) hypertension in
the setting of recurrent episodes of flash pulmonary edema;
(4) unilateral or lateral abdominal bruit; or (5) an acute elevation
of the plasma creatinine in a patient after initiating mandate
screening of angiotensin converting enzyme therapy for renal
artery stenosis.4 It is important to note that there is most likely a
large number of patients who do not suffer from these sequelae
despite the presence of renal artery stenosis, thus are never followed or treated for the disease.5

The underlying cause of renovascular hypertension is hypoperfusion of the kidney. This is caused by a stenosis most commonly
found at the ostium and proximal third of the renal artery.6 As
the juxtaglomerular cells sense the decrease in perfusion pressure to the kidney, they respond by releasing renin, which in
turn causes the cleavage of angiotensinogen to angiotensin I.
This is then converted by angiotensin converting enzyme (found
primarily in the lung) to angiotensin II. Angiotensin II causes
systemic vasoconstriction and stimulates the release of aldosterone from the adrenal cortex, resulting in volume expansion
via retention of renal sodium.7 Persistent hypoperfusion of the
kidney with activation of the aforementioned cascade eventually results in ventricular hypertrophy as well as hyperplasia of
the medial layer of the resistance vessels through the body that
further contributes to hypertension. This is in stark contrast to nonrenal artery stenosis induced hypertension where the vessel remodeling is not marked by increased mass of the wall resistance.8
1. What is the natural history of renal artery stenosis?
Many patients with severe hypertension have renal artery stenosis. This said, there is a large population of patients who are identified with incidental lesions seen either during angiography
for coronary intervention or during ultrasound screening who
do not suffer from symptoms related to the lesion. The decision
to intervene therefore must be based on a thorough understanding of the disease progression and associated sequelae so that the
risks of the procedure do not outweigh the potential benefits. Thus
it is important to understand how often patients experience progression of disease, as well as be cognizant of the risk factors that
increase the likelihood of disease progression.
In an early study, Zierler and colleagues stratified 80 patients
with renovascular hypertension into three groups: those with

749

PMPH_CH94.indd 749

5/22/2012 6:04:28 PM

750

Surgery: Evidence-Based Practice

normal renal arteries, those with a stenosis less than 60%, and
those with greater than 60% stenosis. They found that over a 3-year
period of surveillance with duplex, 51% of patients classified as
normal or with less than a 60% stenosis actually progressed to a
stenosis greater than 60%. The rate of occlusion of the renal artery
over this period was found to be 0.7% in patients with less than
60% stenosis. The progression was found to be related to smoking,
patient age, female gender, and poorly controlled hypertension.9
In 2006, a longitudinal population-based study found that of 119
patients screened by ultrasound, only 13 had significant renal
artery stenosis (>60%) and none of these progressed to occlusion
over the 8-year follow-up period.10
Despite the fact that most patients with renal artery stenosis
do not progress to occlusion, the evolution of the stenotic lesion
can be associated with worsening hypertension and renal dysfunction due to loss of renal mass. Interestingly, the degree of
end-organ damage from the hypertension related to renal artery
stenosis is worse than that found in patients with essential hypertension of the same severity. The effects of this culminate in left
ventricular hypertrophy, proteinuria, and progressive renal dysfunction with the need for dialysis.11 These data were later mirrored by Conlon and colleagues who looked at 1235 patients with
documented renal artery stenosis by angiography; they found that
the patients who had >50% stenosis had a statistically significant
reduction in 4-year survival (88% vs. 67%).12 In 2004 Edwards and
associates screened 834 independent people over the age of 65 for
renal artery stenosis and found that the rates of angina and myocardial infarction were statistically higher in patients with renal
artery stenosis than those with normal renal arteries.13
In conclusion, there is no clearcut evidence that renal artery
stenosis necessarily progresses to renal artery occlusion (Level II
evidence). There is, however, evidence of association with cardiovascular disease sequelae and possibly reduced survival (Level II
evidence).

it has the advantage of no ionizing radiation and no nephrotoxic


contrast. The images can be reconstructed and viewed in two- or
three-dimensional planes but, unfortunately, tend to overestimate
the degree of stenosis. The major disadvantage of MRA is the length
of the test, the potential for movement artifact, inability to image
with metal implants, and the rare condition of gadolinium-induced
nephrogenic systemic fibrosis.19
In summary, there is no Level I evidence to recommend one
test over another. Level II evidence suggests that duplex sonography and CTA are the best available modalities.

2. What is the best way to diagnose renal artery stenosis?

4. Is angioplasty better than medical therapy for treatment of


renal artery stenosis?

All patients who have symptoms suggestive of renal artery stenosis


should be evaluated by a combination of laboratory blood work and
imaging studies. Initial tests include serum creatinine and urinalysis that serve as appropriate starting points for the evaluation of
baseline renal function. The three noninvasive imaging modalities
most commonly used are duplex ultrasonography, magnetic resonance angiography (MRA), and computed tomographic angiography (CTA). Duplex ultrasonography is useful as a screening tool as
it is noninvasive, inexpensive, and does not subject the patient to a
contrast insult or ionizing radiation. Unfortunately, a technically
inadequate study can occur in as many as 20% due to bowel gas,
obesity, or operator inexperience.14 However, in those patients who
have technically successful ultrasounds, the sensitivity and specificity range from 93% to 98%.15 Critical stenosis, defined as >60% luminal obstruction, is predicted by the presence of high peak systolic
velocity (>180 cm/s), poststenotic turbulence, and/or a ratio of renal
artery velocity to aortic velocity >3.5.16,17 The routine use of CTA has
increased as technological advances have improved the speed and
accuracy of the scanner. The three-dimensional reconstructions
prove invaluable in the planning of possible intervention as well as
identifying other intra-abdominal lesions that may be responsible
for the hypertension (adrenal mass etc.). The primary drawbacks of
CTA are the use of nephrotoxic contrast as well as the high doses
of ionizing radiation.18 MRA is used less than CTA or duplex, but

PMPH_CH94.indd 750

3. Is angioplasty better than open reconstruction for the treatment of renal artery stenosis?
Historically it was believed that hypertension with renal artery
stenosis could be effectively remedied by operative intervention.
Open surgical bypass was the mainstay of treatment for those who
failed medical management. With the dawn of the endovascular
era, there was an immediate interest in a percutaneous solution to
renal artery stenosis. In 1993, Weibull and associates compared
percutaneous transluminal renal angioplasty with open reconstruction and found a primary and secondary patency rate of 75%
and 90% in the angioplasty group versus a 96% and 97% in the
surgical group. Hypertension was cured or improved in 83% of
the angioplasty group and 89% in the surgical cohort (P = not
significant). Neither group showed any change in the renal function after the intervention. From this data, the authors concluded
that percutaneous angioplasty was effective, and could be used as
a first-line treatment for unilateral renal artery stenosis.20 Once
it became established that percutaneous intervention was efficacious, a comparison of best medical management to angioplasty
needed to be studied. In summary, Level II evidence suggests that
angioplasty is inferior to open surgery in terms of renal patency
and blood pressure control.

Near the turn of the century, there were three major prospective
randomized trials that compared angioplasty to best medical therapy. The first was the Essai Multicentrique Medicaments vs Angioplastie (EMMA) trial, which included patients who had a diastolic
blood pressure of >95 mm Hg, angiographically documented unilateral renal artery stenosis of >60%, and a creatinine clearance of
>50 mL/min/m2. These patients were then stratified into the percutaneous angioplasty group and the medical management group.
The investigators found that there was no significant difference in
blood pressure or renal function between the groups at 6 months.
They did note that the angioplasty group achieved blood pressure
control with fewer daily antihypertension medications, though this
had no significant change in long-term cardiovascular outcomes.21
The second trial was the Scottish and Newcastle Renal Artery
Stenosis Collaborative Group (SNRASCG) that included patients on
two medications for hypertension, with a persistent diastolic blood
pressure >95 mm Hg and renal artery stenosis >50% by angiogram.
These patients were stratified into two groups: those who underwent
percutaneous angioplasty and those who underwent medical therapy only. At 6 months the authors failed to identify a statistically significant difference in blood pressure between the two groups. They
concluded that angioplasty should only be used as a bail out procedure for renal artery stenosis refractory to medical management.22

5/22/2012 6:04:28 PM

Atherosclerotic Renovascular Disease

The DRASTIC study was the third and largest of these studies and included 102 hypertensive patients with a single renal
artery stenosis of >50% by angiogram and a creatinine of 2.3
mg/dL or less. Again patients were randomized to best medical therapy or percutaneous angioplasty. They looked at blood
pressure outcomes after 1 year of follow-up. Interestingly 44%
of the medically managed patients crossed over to the percutaneous arm due to progression of disease or failure to control
blood pressure adequately. When the data were analyzed, there
again was no significant difference in blood pressure control
between the two groups. 23 These three studies have led clinicians to conclude that there was no clear advantage to percutaneous angioplasty over maximized medical therapy with regard
to blood pressure control or renal function, although adequate
blood pressure control does seem to be achieved with less medication after angioplasty.
In summary, Level I evidence indicates that angioplasty and
best medical management both achieve similar blood pressure
control. Angioplasty cannot be recommended as a routine treatment (Grade A recommendation).
5. Is angioplasty and stenting better than medical therapy for
treatment of renal artery stenosis?
During the mid-1990s there was an evolution in stent technology, and their use became more ubiquitous throughout the body.
Naturally, clinicians sought to apply this technique to the renal
arteries (Figure 94.1). The Stent Placement and Blood Pressure and
Lipid-lowering for the Prevention of Progression of Renal Dysfunction Caused by Atherosclerotic Ostial Stenosis of the Renal Artery
(STAR) trial was a prospective randomized study designed to
compare the treatment of ostial renal artery lesions with medical
therapy alone against outcomes after angioplasty and stenting. The
study included 140 patients with a creatinine clearance less than
80 mL/min per 1.73 m2 and a renal artery stenosis of 50% or greater.
The primary endpoint was a reduction of 20% or more in the creatinine clearance at 2 years. This reduction was seen in 16% of the stent
group and 22% of the medical therapy group (P = NS). There were
two peri-procedural deaths and one late death in the stent group
as well as one patient that required dialysis due to a cholesterol
embolism during the procedure. The authors concluded that stent
placement did not alter the progressive loss of renal function and
with the added risk of procedural complications, the conservative
approach to renal artery stenosis should be used.24
The Angioplasty and Stenting for Renal Artery Lesions
(ASTRAL) trial began recruiting patients in 2000, and is to date
the largest prospective randomized trial comparing maximal
medical therapy against angioplasty and stenting. The study
enrolled 806 patients across 57 medical centers around the
world. It randomized 403 patients to each group with the primary endpoint being the rate of change in renal function. The
secondary endpoints were hypertension control, cardiovascular events, renal events, and death. The mean follow-up of the
cohort was 33.6 months. There was no significant difference in
the rates of decline of the patients renal function or the amount
their blood pressure was controlled. Nearly 7% of the percutaneous intervention group experienced major complications during
or immediately following the procedure. Thus the results of the
ASTRAL trial are similar to the STAR trial in that there appears
to be no significant improvement in patients who are intervened
upon with stenting, and there is a significantly higher risk of

PMPH_CH94.indd 751

751

Figure 94.1 Bilateral renal artery stenosis. (A) Initial angiogram


showing bilateral renal artery stenosis. (B) Angiogram after left
renal artery stent. (C) Angiogram after right renal artery stent.
morbidity and mortality associated with the procedure.25 To
date, there are no randomized controlled trials comparing percutaneous angioplasty and stenting against medical therapy
with purely clinical endpoints.
The Cardiovascular Outcomes in Renal Atherosclerotic
Lesions (CORAL) trial is currently enrolling patients with refractory hypertension (>155 mm Hg on two or more antihypertensive
medications) and renal artery stenosis of 60% with a pressure
gradient of at least 20 mm Hg or 80% with no gradient. These
patients are then being randomized to either medical management or stenting with embolic protection and medical management. The trial was designed to examine clinical outcomes such as
death from renal or cardiovascular causes, myocardial infarction,
stroke, congestive heart failure, and need for dialysis.26 To date no
results have been reported.
In summary, current Level I evidence shows no benefit for
routine stenting as opposed to medical management in patients
with renal artery stenosis. Routine stenting cannot therefore be
recommended (Grade A recommendation).

CONCLUSION
The treatment of renal artery stenosis has evolved over the past
20 years. The emergence of new endovascular technologies has
brought promise of a definitive renal artery stenosis treatment
paradigm. Unfortunately comparative studies have not shown
there to be any advantage to intervention in these patients. Despite
the lack of evidence in support of angioplasty and stenting over
medical management, percutaneous intervention in the setting of
stenotic renal arteries is commonly practiced in the United States.
Although some retrospective data do suggest a role for intervention in particular clinical scenariosincluding patients who
suffer from recurrent flash pulmonary edema or hypertension
refractory to three drugs including a diuretic (these patients were
excluded from the ASTRAL trial, a major criticism of the trial)
prospective randomized data are lacking.23,27

5/22/2012 6:04:28 PM

752

Surgery: Evidence-Based Practice

Clinical Question Summary


Question

Answer

1 What is the natural


history of renal artery
stenosis?

Stenosis progresses slowly over time, and in


patients with greater than 50% stenosis,
there is an increased risk of worsening
hypertension, renal dysfunction, and
myocardial infarction.

II

N/A

9-13

2 What is the best way to


diagnose renal artery
stenosis?

Duplex ultrasonography is most useful as a


screening tool as it is noninvasive, cheap,
and does not use potentially toxic contrast.

II

N/A

14-19

3 Is angioplasty better than


open reconstruction for
the treatment of renal
artery stenosis?

Angioplasty is as effective as open surgical


reconstruction for treatment.

II

20

4 Is angioplasty better
than medical therapy
for treatment of renal
artery stenosis?

There is neither difference in hypertension


control nor progression of renal dysfunction
between patients treated with the best
medical therapy and those treated with
angioplasty.

21-23

5 Is angioplasty and stenting


better than medical
therapy for treatment of
renal artery stenosis?

Stenting does not alter the progression of


renal disease, but does subject the patient
to increased complication rates from the
procedure.

24-25

REFERENCES
1. Cutler JA, Sorlie PD, Wolz M, Thom T, Fields LE, Roccella EJ.
Trends in hypertension prevalence, awareness, treatment, and
control rates in United States adults between 1988-1994 and
1999-2004. Hypertension. 2008;52(5):818-827.
2. Edwards MS, Hansen KJ, Craven TE, et al. Relationships between
renovascular disease, blood pressure, and renal function in the
elderly: a population-based study. Am J Kidney Dis. 2003;41:
990-996.
3. Davis RP, Pearce JD, Craven TE, et al. Atherosclerotic renovascular disease among hypertensive adults. J Vasc Surg. 2009;50:
564-570. e1-3; discussion 571.
4. Hirsch AT, Haskal ZJ, Hertzer NR, et al. ACC/AHA 2005 Practice Guidelines for the management of patients with peripheral
arterial disease. Circulation. 2006;113:e463.
5. Safian RD, Textor SC. Renal-artery stenosis. N Engl J Med. 2001;
344:431-442.
6. Sawicki PT, Kaiser S, Heinemann L, Frenzel H, Berger M. Prevalance of renal artery stenosis in diabetes mellitusan autopsy
study. Ann Intern Med. 1991;229:489-492.
7. Williams G. Aldosterone biosynthesis, regulation and classical
mechanism of action. Heart Fail Rev. 2005;10:7-13.
8. Rizzoni D, Porteri E, Guefi D, et al. Cellular hypertophy in subcutaneous small arteries in patients with renovascular hypertension. Hypertension. 2000;35:931-935.
9. Zierler RE, Bergelin RO, Isaacson JA, Strandness DE Jr. Natural history of atherosclerotic renal artery stenosis: a prospective
study with duplex ultrasonograpy. J Vasc Surg. 1994;19:250-257.
10. Pearce JD, Craven BL, Craven TE, et al. Progression of atherosclerotic renovascular disease: a prospective population-based study.
J Vasc Surg. 2006;44:955-962.

PMPH_CH94.indd 752

Level of
Evidence

Grade of
Recommendation

References

11. Losito A, Fagugli RM, Zampi I, et al. Comparison of target organ


damage in renovascular and essential hypertension. Am J Hypertens. 1996;9:1062-1067.
12. Conlon PJ, Athirakul K, Kovalik E, et al. Survival in renal vascular disease. J Am Soc Nephrol. 1998;9:252-256.
13. Edwards MS, Hansen KJ, Craven TE, et al. Associations between
renovascular disease and prevalent cardiovascular disease in the
elderly: a population-based study. Vasc Endovasc Surg. 2004;38:
25-35.
14. Drelich-Zbroja A, Jargiello T, Drelich G, Lewandowska-Stanek H,
Szczerbo-Trojanowska M. Renal artery stenosis: value of contrastenhanced ultrasonography. Abdom Imaging. 2004;29(4):518-524.
15. Zierler, R. Is duplex scanning the best screening test for renal
artery stenosis? Sem Vasc Surg. 2001;14(3):177-185.
16. Hua HT, Hood DB, Jensen CC, Hanks SE, Weaver FA. The use
of color flow duplex scanning to detect significant renal artery
stenosis. Ann Vasc Surg. 2000;14:118-124.
17. De Haan MW, Kroon AA, Flobbe K, et al. Renovascular disease
in patients with hypertension: detection with duplex ultrasound.
J Hum Hypertens. 2002;16:501-507.
18. Leiner T, de Haan MW, Nelemans PJ, van Engelshoven JM,
Vasbinder GB. Contemporary imaging techniques for the diagnosis of renal artery stenosis. Eur Radiol. 2005;15:2219-2229.
19. Marcos HB, Choyke PL. Magnetic resonance angiography of the
kidney. Sem Nephrol. 2000;20:450-455.
20. Weibull H, Bergqvist D, Bergentz SE, Jonsson K, Hulthn L,
Manhem P. Percutaneous transluminal renal angioplasty versus
surgical reconstruction of atherosclerotic renal artery stenosis:
a prospective randomized study. J Vasc Surg. 1993;18:841-850.
21. Plouin P, Chatellier G, Darn B, Raynaud A. Blood pressure outcome of angioplasty in atherosclerotic renal artery stenosis: a randomized trial. (EMMA trial). Hyptertension. 1998;31:823-829.

5/22/2012 6:04:29 PM

Atherosclerotic Renovascular Disease

22. Webster J, Marshall F, Abdalla M, et al. Randomised comparison of percutaneous angioplasty vs. continued medical therapy
for hypertensive patients with atheromatous renal artery stenosis. Scottish and Newcastle Renal Artery Stenosis Collaborative
Group. J Hum Hypertens. 1998;12:329-335.
23. Van Jaarsveld BC, Krijnen P, Pieterman H, et al. The effect of balloon angioplasty on hypertension in atherosclerotic renal-artery
stenosis. Dutch Renal Artery Stenosis Intervention Cooperative
Study Group (DRASTIC). N Engl J Med. 2000;14:1007-1014.
24. Bax L, Mali WP, Buskens E, et al. The benefit of STent placement and blood pressure and lipid-lowering for the prevention
of progression of renal dysfunction caused by Atherosclerotic
ostial stenosis of the Renal artery. J Nephrol. 2003;16:807-812.

PMPH_CH94.indd 753

753

25. Wheatley K, Ives N, Gray R, et al. Revasularization versus medical therapy for renal-artery stenosis (ASTRAL). N Engl J Med.
2009;361:1953-1962.
26. Cooper CJ, Murphy TP, Matsumoto A, et al. Stent revascularization for the prevention of cardiovascular and renal events
among patients with renal artery stenosis and systolic hypertension: rationale and design of the CORAL trial. Am Heart J.
2006;152:59-66.
27. Gray BH, Olin JW, Childs MB, Sullivan TM, Bacharach JM.
Clinical benefit of renal artery angioplasty with stenting for the
control of recurrent and refractory congestive heart failure. Vasc
Med. 2002;7:275-279.

5/22/2012 6:04:29 PM

Commentary on
Atherosclerotic Renovascular Disease
Daniel G. Clair

The chapter Atherosclerotic Renovascular Disease by Marone


and Hager reviews very well the current state of understanding
regarding renal artery disease. Importantly, the rise in performance of percutaneous interventions to treat renal artery stenosis
has led to confusion and misunderstanding regarding who benefits from revascularization and why. The cost of this increase in
interventions is, not surprisingly, a concern for healthcare systems
globally. Added to this is the understanding that renovascular
disease itself appears to confer an increased risk of mortality and
cardiovascular events in general, which not only has an impact on
patient longevity but also increases healthcare costs. The authors
set out to answer the following questions:

(CTA) provides anatomic information and can give excellent


information regarding degree of stenosis and renal parenchymal
size and abnormalities as well, but has the limitations as noted
of contrast and radiation, and magnetic resonance angiography
(MRA) in our hands has not proven as beneficial.
3. Is angioplasty better than open reconstruction for the treatment of renal artery stenosis?
This question has largely been supplanted by the use of stents for
the percutaneous treatment of renal artery stenosis. The only situation in which angioplasty remains a mainstay of therapy for renal
artery disease is in the treatment of fibromuscular dysplasia, and
in these patients, angioplasty has similar outcomes to surgery.

1. What is the natural history of renal artery stenosis?


The authors note that there appears to be a small group of patients
with progression of the renal artery stenotic lesions to occlusion,
and in addition, that patients appear to experience progression of
end-organ damage and higher rates of myocardial infarction and
angina. There are truly three parts to the answer to this question:
What happens to the stenosis? What happens to the renal function?
And finally, what happens to the overall health of the patient? The
authors correctly note that over time the stenosis tends to progress,
and in most current studies at a slower rate than reported before.
Initial studies appear to reveal a benefit with statin use in reducing the rate of progression and possibly in reversing the degree of
stenosis.1 The progression of renal dysfunction over time is interestingly unrelated to a stenosis proximal to the kidney parenchyma,
as renal dysfunction is present in kidneys contralateral to a severe
stenosis, and this dysfunction tends to progress over time as well.2
This progression appears to vary over time as noted in the ASTRAL
trial where initial creatinine rise in the medical arm was noted to
be 10% over a year with subsequent rise of only 1% over the next
3 years.3 The increase in mortality and cardiovascular morbidity is
well outlined by the authors. By any measure, this is a devastating
problem with extensive deleterious effects.

4. Is angioplasty better than medical therapy for treatment of


renal artery stenosis?
Again here the authors have noted that randomized trials have
failed to prove any benefit of angioplasty alone versus medical
therapy in the treatment of renal artery stenosis. In review of this
topic, however, in the Cochrane database, the authors noted that
with blood pressure refractory to medical therapy, balloon angioplasty appears to lower blood pressure more effectively, and additionally concluded, Balloon angioplasty appears to be safe and
shows fewer cardiovascular and renovascular complications.4
Although it is not easy to identify patients, there do appear to be a
subset who may benefit from intervention.
5. Is angioplasty and stenting better than medical therapy for
treatment of renal artery stenosis?
The authors correctly note the results of the STAR and ASTRAL
trials,3,5 both of which fail to identify benefit from stenting for renal
artery stenosis, but they have failed to point out some significant
shortcomings of the studies. The STAR trial randomized a small
number of patients into each group and fewer than 50 patients
ultimately underwent stent placement. Patients needed only a
50% stenosis of the renal artery to be enrolled along with renal
dysfunction; in addition, there were significant numbers of procedural complications including two procedural deaths. This study
included patients without hemodynamically significant stenosis
and interventionalists who had higher than anticipated complication rates. The ASTRAL trial, the larger of the two randomized trials, was more flawed. The main flaw in this study is in the eligibility

2. What is the best way to diagnose renal artery stenosis?


The authors correctly note ultrasound as probably the best way to
diagnose renal artery stenosis in the setting of appropriate clinical and laboratory abnormalities, along with reasonable ability
to image the vessels. Other methods of testing including angiotensin converting enzyme (ACE) inhibitor nephrograms have not
proven clinically useful. Computed tomographic angiography
754

PMPH_CH94.indd 754

5/22/2012 6:04:29 PM

Atherosclerotic Renovascular Disease

requirements for the study. Patients were not eligible if the physician felt they would have definite worthwhile benefit from revascularization. Inherent in this study was a bias against stenting. These
were patients for whom revascularization was of questionable benefit. Also excluded were those in whom physicians foresaw a need for
revascularization within 6 months. The group of patients included
in this trial were those for whom revascularization might not have
even been normally considered. In addition, a large percentage of
the population enrolled had less than 70% stenosis in the artery in
question leading to randomization of patients in whom no hemodynamically significant lesion was demonstrated. Although the
study does refute the trend toward widespread application of percutaneous renal revascularization, it does not truly answer the question of whether renal artery stenting is better than medical therapy
except in those patients in whom the benefits of the procedure are
in question in the first place. More data still need to be obtained to
help identify the subgroup of patients who may benefit significantly
from renal artery intervention.

PMPH_CH94.indd 755

755

REFERENCES
1. Cheung MV, Patel A, Shaheen N, et al. The effects of statins on the
progression of atherosclerotic renovascular disease. Nephron Clin
Pract. 2007;107:c35-c42.
2. Farmer CK, Cook GJ, Blake GM, et al. Individual kidney function in atherosclerotic nephropathy is not related to the presence of renal artery stenosis. Nephrol Dial Transplant. 1999;14:
2880-2884.
3. ASTRAL Investigators, Wheatley K, Ives N, et al. Revascularization versus medical therapy for renal-artery stenosis. N Engl J
Med. 2009;361(20):1953-1962.
4. Nordmann AJ, Logan AG. Balloon angioplasty versus medical
therapy for hypertensive patients with renal artery obstruction.
Cochrane Database Syst Rev. 2003;Issue 3.
5. Bax L, Woittiez AJ, Jouwenberg HJ, et al. Stent placement in
patients with atherosclerotic renal artery stenosis and impaired
renal function: a randomized trial. Ann Intern Med. 2009;150(12):
840-848.

5/22/2012 6:04:29 PM

CHAPTER 95

Acute and Chronic


Mesenteric Ischemia
Luke P. Brewster and Elliot L. Chaikof

ANATOMY AND PHYSIOLOGY

dial infarction, cardiomyopathy, structural heart defects, cardiac


tumors, and endocarditis. Mural thrombus from both aneurysmal
and nonaneurysmal aorta may also be the source of embolism.
Thrombotic causes are the next most frequent cause,3 and nonocclusive mesenteric ischemia or mesenteric venous thrombosis are
less likely causes.
Answer: Persons at risk for cardiogenic thrombus, preexisting atherosclerosis of their mesenteric vessels, or with prothrombotic disorders are at the greatest risk of developing AMI. These
patients should receive lifelong anticoagulation or undergo appropriate surgical procedures electively in order to lessen their risk.
Aortic dissections and trauma are other known causes that are
usually discussed separately. In addition, there may be a slight
gender preference for women. Level II evidence.

With its rich blood supply, the intestines rarely develop chronic
ischemia. The major vessels supplying the intestines are the celiac
artery for the foregut, the superior mesenteric artery for the midgut,
and the inferior mesenteric artery for the hindgut. In addition, collateral circulation can occur between the inferior mesenteric artery
and the internal iliac artery. Further the intestines can survive with
as little as 20% of maximal capillary flow.1 With chronic occlusion
of one major artery, the mesenteric network of blood flow may be
adequate, and symptoms usually require stenosis or occlusion in
two or more of the three major vessels. If only one vessel is causing
symptoms, it is usually the superior mesenteric artery.

HISTORY AND EPIDEMIOLOGY

2. Who is at risk for CMI?

Mesenteric angina from chronic ischemia can occur with symptoms of bloating, weight loss, nausea, vomiting, diarrhea, and/or
constipation, and it usually occurs within the first hour after eating. The symptoms are often severe enough to cause the patient
to restrict food intake (food fear), and the subsequent weight
loss may lead to lengthy and nondiagnostic workups. Given the
debilitating symptoms and the malignant nature of the disease,
revascularization attempts should not be delayed.
The first clinical presentation may be acute mesenteric ischemia (AMI) in 15% to 50% of the patients, with a mortality rate of
about 15% to 70%. AMI can also present subsequent to an embolic event, poor perfusion due to heart failure, or venous thrombosis. With AMI, the patients are frequently fulminantly ill and
may have peritonitis on examination with signs and symptoms of
shock. To date, there are no randomized controlled trials comparing therapies for AMI or chronic mesenteric ischemia (CMI).

CMI has been estimated to occur at a frequency of 1/100,000


persons/year.4 Consecutive mesenteric duplex identified that 17.5%
of persons greater than age 65 had at least a critical stenosis of one
vessel,5 and autopsy findings suggest mesenteric atherosclerosis in
about 10% of dead persons.6
Answer: Aged persons with atherosclerosis risk factors (smokers, dyslipidemia) are at risk. Here too there may be a female gender preference.7-10 Level II evidence.

INITIAL EVALUATION AND DIAGNOSIS


3. What is the evaluation pathway for AMI?
Because of the paucity of early laboratory and diagnostic fi ndings with positive predictive value and the error rate that is
present even with experienced clinicians,11 patients being considered for AMI with peritonitis need to be volume resuscitated,
have antibiotics initiated, and taken emergently to the operating room for laparotomy.12 There, any frankly necrotic bowel
should be resected, and revascularization performed. In the
absence of peritonitis, either conventional arteriography or

RISK FACTORS
1. Who is at risk for AMI?
Embolic causes of AMI are frequently cardiac in nature. 2 Cardiogenic thrombus may be due to atrial dysrhythmias, myocar756

PMPH_CH95.indd 756

5/22/2012 6:05:27 PM

Acute and Chronic Mesenteric Ischemia

computed tomography arteriography should be considered in


patients who are hemodynamically normal with normal renal
function. Mesenteric vessels with fi ndings of stenosis or occlusion should then undergo revascularization by endovascular or
open surgery.
Answer: The algorithm is predicated on the presence or absence
of peritonitis and/or sepsis. If these are present, the patient is transferred expediently to the operating room. If they are not present,
then an arterial imaging modality should be initiated prior to surgical intervention. The natural history of untreated AMI is almost
universally fatal (Level II evidence; Grade B recommendation).
4. What is the evaluation pathway for CMI?
Mesenteric duplex in a fasting patient with food challenge
should provide support for or against the diagnosis of CMI.13-14
Prior to intervention (endovascular or open), mesenteric arteriogram should be performed. During angiography, endovascular
attempts to revascularization may be performed. Open surgery
may also be chosen or deferred to patients who fail endovascular
management.15
Answer: Mesenteric duplex is the best screening examination
(Level II evidence). If positive, a confirmatory study using conventional or CT arteriography should be performed. Endovascular
intervention can be performed at time of arteriography or open
surgery may be utilized (Grade B recommendation).

MANAGEMENT
Prevention of Thrombotic and Embolic Events
5. What is the role of antiplatelet, anticoagulants, and lipid
lowering therapies in the prevention of CMI?
Although there are no clear benefits of antiplatelet therapy in prevention of mesenteric ischemia or enhancing the patency of
mesenteric bypasses, antiplatelet therapy has benefit in primary
prevention for cardiovascular events, and it is an accepted part of
therapy.16 For patients who have had embolic events from a defi ned
thrombus (cardiac or aortic wall) or a thrombophilia, lifelong anticoagulation should be maintained. Adding Coumadin to antiplatelet therapy may decrease cardiovascular events, but it comes at a cost
of increased bleeding risks that may limit any potential benefits.17
Similarly statin therapy has had both perioperative and longterm benefits limiting cardiovascular events,18-19 but may not have
a direct benefit to the durability of these procedures.
Answer: Patients with CMI meet criteria for both statin and
antiplatelet therapy for primary prevention of cardiovascular
events and death. This is sufficient to recommend therapy for these
patients (Level II evidence; Grade B recommendation).
6. What steps can be made in the evaluation, diagnosis, and
treatment of AMI to limit morbidity and mortality?
The prompt diagnosis and consultation with a surgeon are the
bottle-neck steps regarding outcome for AMI as morbidity and
mortality increase rapidly from time of onset. 20 Surgeons then
provide rapid access to the operating room or for imaging studies that will guide therapy and limit the systemic illness of these
patients. More recent approaches may limit need for laparotomy
in 30% of patients and may limit mortality.21

PMPH_CH95.indd 757

757

Answer: High level of suspicion, early transfer, and acceptance


of patients who cannot be treated at their presenting facility, ready
access to a surgeon, and hybrid operating rooms with arteriography teams all can decrease time to therapy and optimize therapeutic approaches (Level II evidence; Grade B recommendation).
7. Does elective therapy for CMI limit morbidity and
mortality?
Prophylactic intervention on patients with CMI is important since
86% of untreated patients will go on to fulminant symptoms or
death. Only one study has looked at asymptomatic patients with
mesenteric stenoses/occlusion. The study found that 4/15 patients
developed ischemia over a 2.5-year follow-up; all had three-vessel mesenteric disease.22 Other studies have suggested that up to
50% of patients develop AMI without prior symptomatology23,24
with some risk of acute ischemia during preoperative parenteral
nutrition.25
Answer: Although both endovascular and open surgical therapy carries a morbidity rate of 20% to 30% along with a mortality
rate of ~10%,17 the lethality of fulminant presentation and ready
availability of therapy options to match pathology and patients
operative risk mandate therapy in symptomatic patients (Level II
evidence; Grade B recommendation).
8. How should patients with CMI be selected for endovascular
or open surgery?
The acute technical success rates reported in endovascular series
are 90% or better. These expectations should be understood in
the context of patients selected for endovascular therapy, and
they cannot be generalized to all persons presenting with CMI.
Furthermore, technical success alone is not a good endpoint
for evaluating therapy. Palliation of symptoms is very good and
similar for both endovascular and open surgery.26 The benefits of
endovascular therapy include at least a trend toward decreased
perioperative morbidity and mortality. These lead to markedly
shortened hospital stays.27 There is conflicting data with regard
to whether treating two vessels is superior to one vessel.28,29 The
benefits of open surgery include better patency rates at longer
follow-up periods, the ability to treat chronic occlusions and recurrent disease via a variety of operative approaches, and acceptable
perioperative mortality.28,30 Most reassuring is the fact that the
long-term survival of patients treated by each approach is similar to each other31,32 as well as to that of patients with peripheral
artery disease and claudication.17 There are no prospective randomized trials comparing open and endovascular treatments for
CMI; only retrospective comparisons and prospective case series
are available.
Answer: In appropriately selected patients (anatomically
and pathologically suitable lesions), endovascular therapy carries a high likelihood of success and effectiveness with decreased
morbidity and mortality, but it comes at the cost of less durability and increased likelihood of more interventions. We would
strongly recommend this approach in patients at prohibitive
surgical risk and weakly recommend it for patients as initial
therapy because of these benefits, and it does not confer worse
open outcomes if needed in the future. For suitable operative
candidates with significant burden or complexity of disease,
open surgery is an excellent option (Level II evidence; Grade B
recommendation).

5/22/2012 6:05:27 PM

758

Surgery: Evidence-Based Practice

Clinical Question Summary


Question

Answer

Levels of Grade of
References
Evidence Recommendation

1 Who is at risk for AMI?

Persons at risk for cardiogenic thrombus,


preexisting atherosclerosis of their mesenteric
vessels, older persons, female gender, or with
prothrombotic disorders.

2b

N/A

2, 3

2 Who is at risk for CMI?

Aged persons with atherosclerosis risk factors


(smokers, dyslipidemia) and female gender are
at risk.

2b

N/A

4-10

3 What is the evaluation


pathway for AMI?

For patients with peritonitis: Focused history


and physical examination, resuscitation, and
expediently to the operating room.
For stable patients without peritonitis: Focused
history and physical examination, imaging study,
and then endovascular or open operation as
dictated by findings and patients condition.

2b

12

4 What is the evaluation


pathway for CMI?

History and physical examination. Mesenteric


duplex as a screening test.*
Arteriography as a confirmatory test.
Endovascular or open operation as dictated by
findings and patients condition.

*1b

13, 14

2b

15

Antiplatelet therapy is effective in primary


prevention of myocardial infarction (MI),
cerebrovascular accident (CVA), and mortality
in patients with peripheral arterial disease
(PAD).
Anticoagulants carry a significant bleeding risk that
limits its recommendation in these patients.
Statin therapy is of significant benefit in these
patients for both cardiovascular morbidity and
mortality.

1b

16

2a

17

1a, b

18, 19

6 What steps can be made in


the evaluation, diagnosis,
and treatment of AMI
to limit morbidity and
mortality?

High level of suspicion, early transfer, and


acceptance of patients who cannot be treated
at their presenting facility, ready access to a
surgeon, and hybrid operating rooms with
arteriography teams all can decrease time to
therapy and optimize therapeutic approaches.

20, 21

7 Does elective therapy for


CMI limit morbidity and
mortality?

This has never been tested. However, given the


lethal nature of CMI and ready therapy, elective
treatment is recommended.

22-25

8 How should patients with


CMI be selected for
endovascular or open
surgery?

In appropriately selected patients (anatomically


and pathologically suitable lesions),
endovascular therapy carries a high likelihood
of success and effectiveness with decreased
morbidity and mortality, but it comes at the
cost of less durability and increased likelihood
of more interventions. We would strongly
recommend this approach in patients at
prohibitive surgical risk and weakly recommend
it for patients as initial therapy because of
these benefits, and it does not confer worse
open outcomes if needed in the future. For
suitable operative candidates with significant
burden or complexity of disease, open surgery
is an excellent option.

2b

26-32

5 What is the role


of antiplatelet,
anticoagulant, and lipid
lowering therapies on
the prevention of CMI?

PMPH_CH95.indd 758

5/22/2012 6:05:27 PM

Acute and Chronic Mesenteric Ischemia

REFERENCES
1. Granger DN, Richardson PD, Kvietys PR, Mortillaro NA. Intestinal blood flow. Gastroenterology. 1980;78:837-863.
2. Acosta S, Ogren M, Sternby NH, Bergqvist D, Bjorck M. Clinical implications for the management of acute thromboembolic
occlusion of the superior mesenteric artery: autopsy findings in
213 patients. Ann Surg. 2005;241:516-522.
3. Mansour MA. Management of acute mesenteric ischemia. Arch
Surg. 1999;134:328-330; discussion 31.
4. McMillan WD, McCarthy WJ, Bresticker MR, et al. Mesenteric
artery bypass: objective patency determination. J Vasc Surg.
1995;21:729-740; discussion 40-41.
5. Wilson DB, Mostafavi K, Craven TE, Ayerdi J, Edwards MS,
Hansen KJ. Clinical course of mesenteric artery stenosis in
elderly americans. Arch Intern Med. 2006;166:2095-2100.
6. Foley MI, Moneta GL, Abou-Zamzam AM, Jr., et al. Revascularization of the superior mesenteric artery alone for treatment of
intestinal ischemia. J Vasc Surg. 2000;32:37-47.
7. Acosta S. Epidemiology of mesenteric vascular disease: clinical
implications. SeminVasc Surg. 2010;23:4-8.
8. Hertzer NR, Beven EG, Humphries AW. Acute intestinal ischemia. Am Surg. 1978;44:744-749.
9. Ottinger LW, Austen WG. A study of 136 patients with mesenteric infarction. Surg Gynecol Obstet. 1967;124:251-261.
10. Derrow AE, Seeger JM, Dame DA, et al. The outcome in the
United States after thoracoabdominal aortic aneurysm repair,
renal artery bypass, and mesenteric revascularization. J Vasc
Surg. 2001;34:54-61.
11. Chaikof EL. Developing a discriminant noninvasive test for early
mesenteric ischemia: measuring the basic rhythms of life. J Vasc
Surg. 1999;30:367-369.
12. Jamieson WG, Pliagus G, Marchuk S, et al. Effect of antibiotic
and fluid resuscitation upon survival time in experimental intestinal ischemia. Surg Gynecol Obstet. 1988;167:103-108.
13. Gentile AT, Moneta GL, Lee RW, Masser PA, Taylor LM, Jr., Porter JM. Usefulness of fasting and postprandial duplex ultrasound
examinations for predicting high-grade superior mesenteric
artery stenosis. Am J Surg. 1995;169:476-479.
14. Moneta GL, Lee RW, Yeager RA, Taylor LM, Jr., Porter JM. Mesenteric duplex scanning: a blinded prospective study. J Vasc Surg.
1993;17:79-84; discussion 5-6.
15. Biebl M, Oldenburg WA, Paz-Fumagalli R, McKinney JM,
Hakaim AG. Surgical and interventional visceral revascularization for the treatment of chronic mesenteric ischemiawhen to
prefer which? World J Surg. 2007;31:562-568.
16. A randomised, blinded, trial of clopidogrel versus aspirin in
patients at risk of ischaemic events (CAPRIE). CAPRIE Steering
Committee. Lancet. 1996;348:1329-1339.
17. Hirsch AT, Haskal ZJ, Hertzer NR, et al. ACC/AHA 2005 Practice guidelines for the management of patients with peripheral
arterial disease (lower extremity, renal, mesenteric, and abdominal aortic): a collaborative report from the American Association for Vascular Surgery/Society for Vascular Surgery, Society
for Cardiovascular Angiography and Interventions, Society for

PMPH_CH95.indd 759

18.

19.

20.

21.

22.

23.

24.
25.

26.

27.

28.

29.

30.

31.

32.

759

Vascular Medicine and Biology, Society of Interventional Radiology, and the ACC/AHA Task Force on Practice Guidelines
(Writing Committee to Develop Guidelines for the Management
of Patients With Peripheral Arterial Disease): endorsed by the
American Association of Cardiovascular and Pulmonary Rehabilitation; National Heart, Lung, and Blood Institute; Society for
Vascular Nursing; TransAtlantic Inter-Society Consensus; and
Vascular Disease Foundation. Circulation. 2006;113:e463-654.
Winchester DE, Wen X, Xie L, Bavry AA. Evidence of preprocedural statin therapy a meta-analysis of randomized trials.
J Am Coll Cardiol. 2010;56:1099-1109.
MRC/BHF Heart Protection Study of cholesterol lowering with
simvastatin in 20,536 high-risk individuals: a randomised placebo-controlled trial. Lancet. 2002;360:7-22.
Kougias P, Lau D, El Sayed HF, Zhou W, Huynh TT, Lin PH.
Determinants of mortality and treatment outcome following
surgical interventions for acute mesenteric ischemia. J Vasc Surg.
2007;46:467-474.
Arthurs ZM, Titus J, Bannazadeh M, et al. A comparison of endovascular revascularization with traditional therapy for the treatment of acute mesenteric ischemia. J Vasc Surg. 2011;53(3):698-705.
Thomas JH, Blake K, Pierce GE, Hermreck AS, Seigel E. The clinical course of asymptomatic mesenteric arterial stenosis. J Vasc
Surg. 1998;27:840-844.
Levy PJ, Krausz MM, Manny J. Acute mesenteric ischemia:
improved results--a retrospective analysis of ninety-two patients.
Surgery. 1990;107:372-380.
Stoney RJ, Cunningham CG. Acute mesenteric ischemia. Surgery. 1993;114:489-490.
Rheudasil JM, Stewart MT, Schellack JV, Smith RB, 3rd, Salam
AA, Perdue GD. Surgical treatment of chronic mesenteric arterial insufficiency. J Vasc Surg. 1988;8:495-500.
Kasirajan K, OHara PJ, Gray BH, et al. Chronic mesenteric ischemia: open surgery versus percutaneous angioplasty and stenting. J Vasc Surg. 2001;33:63-71.
Atkins MD, Kwolek CJ, LaMuraglia GM, Brewster DC, Chung
TK, Cambria RP. Surgical revascularization versus endovascular
therapy for chronic mesenteric ischemia: a comparative experience. J Vasc Surg. 2007;45:1162-1171.
Peck MA, Conrad MF, Kwolek CJ, LaMuraglia GM, Paruchuri V,
Cambria RP. Intermediate-term outcomes of endovascular treatment for symptomatic chronic mesenteric ischemia. J Vasc Surg.
2010;51:140-147.e1-2.
Malgor RD, Oderich GS, McKusick MA, et al. Results of singleand two-vessel mesenteric artery stents for chronic mesenteric
ischemia. Ann Vasc Surg. 2010;24:1094-1101.
Rawat N, Gibbons CP. Surgical or endovascular treatment for
chronic mesenteric ischemia: a multicenter study. Ann Vasc Surg.
2010;24:935-945.
Jimenez JG, Huber TS, Ozaki CK, et al. Durability of antegrade
synthetic aortomesenteric bypass for chronic mesenteric ischemia. J Vasc Surg. 2002;35:1078-1084.
Silva JA, White CJ, Collins TJ, et al. Endovascular therapy for
chronic mesenteric ischemia. J Am Coll Cardiol. 2006;47:
944-950.

5/22/2012 6:05:27 PM

Commentary on
Acute and Chronic
Mesenteric Ischemia
Michael J. Sise

2. Who is at risk for CMI?

This chapter addresses various questions related to an uncommon


but often deadly process, mesenteric ischemia. There remains a
general lack of understanding of the clinical entity and vascular
surgeons are frequently called to sort out patients suspected of
having this diagnosis. It is ironic that the widespread use of computed tomography (CT) of the abdomen with intravenous contrast identifies increasing numbers of patients with asymptomatic
mesenteric arterial stenosis while at the same time many patients
with a classical clinical presentation of acute mesenteric ischemia
(AMI) go unrecognized. Despite many significant advances in
clinical medicine, the basic disease process, treatment, and outcome of mesenteric ischemia has changed little over many decades
with the exception of the addition of endovascular therapy. There
remain four major causes of AMI: acute cardiac source embolism to the superior mesenteric artery (50%), acute thrombosis of
an atherosclerotic lesion with previous partial occlusion (20%),
splanchnic vasoconstriction leading to low flow and regional ischemia (commonly called nonocclusive mesenteric ischemia) (20%),
and mesenteric venous thrombosis (10%).1 Therefore, AMI occurs
as a secondary phenomenon that results from other major disease
processes and is always associated with high risk for a poor outcome.2 Diagnosis must be timely and treatment effective to insure
the best possible result.
Chronic mesenteric ischemia (CMI) remains a disease of insidious onset and is associated with diff use atherosclerosis. It is
often mistaken for other etiologies of gastrointestinal disease and
delay in diagnosis is the rule. Early CT scan mitigates this tendency and improves the likelihood of a timely diagnosis.

The authors identify the association with advanced age and atherosclerosis. They also mention the association with female gender. This is an important factor. There is a peculiar tendency for
women to develop mid-abdominal aortic atherosclerosis with
involvement of the visceral vessel origins and this may present in
younger patients.7,8 In general, the diagnosis of CMI is also frequently delayed and the vast majority of patients with acute mesenteric arterial thrombosis have a long-standing history of the
symptoms of chronic disease.8
3. What is the evaluation pathway for AMI?
The authors discuss a variety of diagnostic factors. There are
two key findings. One is the classical early finding of pain out of
proportion to physical exam findings and the other is profound
leukocytosis.9,10 Early use of CT is an excellent means of prompt
diagnosis of AMI. The authors note that patients with peritoneal
findings need immediate laparotomy. Although it is true that during emergency exploratory laparotomy the pattern of ischemic
bowel and palpation of the proximal mesenteric arteries will yield
the underlying cause of acute ischemia if preoperative CT was not
obtained, it is very helpful to have a preoperative CT.
4. What is the evaluation pathway for CMI?
The most important first step in the workup of CMI is a high index
of suspicion. Unfortunately, many clinicians do not entertain this
etiology of postprandial pain and most diagnoses are delayed.
Many patients undergo endoscopy with nonspecific findings prior
to an effective workup that identifies the true source of symptoms.
The authors focus on mesenteric duplex scanning. In experienced
noninvasive laboratories, this is an important first adjunct. However, it is not universally available and CT scan is frequently the
first diagnostic study. The authors correctly identify the need for
catheter angiography in the workup.

1. Who is at risk for AMI?


The authors touch on some of the more common etiologies. A
more extensive list includes the following etiologies3-6:
Arterial embolism or thrombosis. Cardiac disease (atrial fibrillation, recent myocardial infarction, congestive heart failure,
digitalis therapy), previous arterial emboli, hypercoagulable state,
hypovolemia, shock.
Venous thrombosis. Portal hypertension, bowel inflammation, postoperative period, trauma, prothrombotic state, chronic
renal failure. There are, therefore, a wide variety of clinical settings in which AMI occurs and the risk of delay in diagnosis
remains high.

5. What is the role of antiplatelet, anticoagulants, and lipid


lowering therapies in the prevention of CMI?
The authors appropriately identify the role of these therapies.
Unfortunately, CMI causing significant symptoms is associated
760

PMPH_CH95.indd 760

5/22/2012 6:05:27 PM

Acute and Chronic Mesenteric Ischemia

with a high risk of death from associated coronary artery disease


in addition to the potential mortality from mesenteric infarction.9 These medical interventions are important in not only halting progression of disease but also limiting recurrence following
treatment.
6. What steps can be made in the evaluation, diagnosis, and
treatment of AMI to limit morbidity and mortality?
The authors effectively identify the key factors in successful treatment. It can be summarized by the fundamental rule for successful management of all surgical emergenciesearly diagnosis and
prompt treatment.9,10
7. Does elective therapy for CMI limit morbidity and
mortality?
The authors discuss the key factors and identify the need to treat
symptomatic patients prior to bowel infarction. This is a high-risk
group of patients because of the generalized nature of atherosclerosis and the associated cardiac and cerebrovascular disease
risks.
8. How should patients with CMI be selected for endovascular
or open surgery?
The authors very effectively identified the appropriate shift toward
endovascular therapy. Open mesenteric arterial surgery has
become very uncommon. Surgery should be reserved for failures of endovascular treatment or in those few settings where it is
either unavailable or not appropriate. The collaboration between

PMPH_CH95.indd 761

761

specialists treating these patients should ensure that both modes


of therapy are readily available.

REFERENCES
1. Edwards MS, Cherr GS, Craven TE, et al. Acute occlusive mesenteric ischemia: surgical management and outcomes. Ann Vasc
Surg. 2003;17:72-79.
2. Schoots IG, Koffeman GI, Legemate DA, et al. Systematic review
of survival after acute mesenteric ischemia according to disease
aetiology. Br J Surg. 2004;91:17-27.
3. Boley SJ, Kaleya RN, Brandt LJ. Mesenteric venous thrombosis.
Surg Clin North Am. 1992;72:183-201.
4. Phee RY, Gloviczki P, Mendonca CT, et al. Mesenteric venous
thrombosis: still a lethal disease in the 1990s. J Vasc Surg. 1994;
20:688-697.
5. Weil J, Sen Gupta R, Harfarth H. Nonocclusive mesenteric ischemia induced by digitalis. Int J Colorectal Dis. 2004;19:277-280.
6. Burns BJ, Brandt LJ. Intestinal ischemia. Gastroenterol Clin
North Am. 2003;32:1127-1143.
7. Kougias P, Lau D, Saed HF, et al. Determinants of mortality and
treatment outcome following surgical interventions for acute
mesenteric ischemia. J Vasc Surg. 2007;46:465-466.
8. Moawad J, Gewertz BL. Chronic mesenteric ishcemia. Clinical
presentation and diagnosis. Surg Clin N Am. 1997;77:357-369.
9. McKinsey J, Gewertz B. Acute mesenteric ischemia. Surg Clin N
Am. 1997; 77:307-318.
10. Par WM, Gloviczki P, Cherry KJ, et al. Contemporary management of acute mesenteric ischemia: factors associated with survival. J Vasc Surg. 2002;35:445-452.

5/22/2012 6:05:27 PM

CHAPTER 96

The Diabetic Foot


Alexandra A. MacLean and Maria Codreanu

impairment.8 Ndip et al. studied 326 patients with diabetes and


chronic kidney disease and compared the outcomes of those
patients on dialysis versus nondialysis patients. Dialysis patients
had a higher prevalence of peripheral neuropathy, prior amputations, prior foot ulceration, and current foot ulceration.
Patients who develop peripheral neuropathy have a high
chance of presenting with foot ulceration. Abbott et al. looked at
factors that can predict which diabetic patients with neuropathy
will develop foot ulceration.9 They followed 1035 patients (with
NIDDM and IDDM) and an established peripheral neuropathy
diagnosis (by measuring vibration perception threshold [VPT]).
After 1 year, 7.2% of this population developed their first foot ulcer.
When the investigators looked at the relationship between baseline VPT scores and Michigan diabetic polyneuropathy (DPN),
they found that for each 1 U increase in each score, the hazard of
the first foot ulcer increased by 5.6% and 5%, respectively.
Answer: There is a lack of recent (within last 10 years) comprehensive epidemiological data to validate those risk factors
identified in the past and to uncover new ones. However, there
is evidence that diabetic patients on dialysis and those with an
established peripheral neuropathy diagnosis have an increased
rate of developing foot complications (Level II evidence, Grade B
recommendation).

INTRODUCTION
The incidence of diabetes mellitus (DM) (type 1 and 2) is continuing to increase worldwide. Boyle et al. modeled the growth of DM
and projected that the prevalence of DM will increase from 14%
of the US population in 2010 to 25% to 28% by 2050.1 The risk
rate of a patient with DM developing a foot ulcer in their lifetime
is 25% and the recurrence rate is greater than 50% after 3 years.2
DM has complications that affect almost every organ and system
but one of the most burdensome effects is on the peripheral vascular system and sensory system. Foot complications occur in
this population at high rates due to the prevalence of peripheral
vascular disease (PVD) and neuropathies: peripheral neuropathy
(2040% in patients with DM), PVD (2040%), foot ulceration
(5% per year), foot infection and osteomyelitis (2266%), amputation (0.5% per year), charcots neuroarthropathy (0.10.4% per
year).3 The development of foot ulcers in this population is usually
from a combination of ischemia and neuropathy. Sensory defects
alone can lead to deformities and traumatic injuries and ulcers.

RISK FACTORS
1. Which diabetic patients are at greatest risk for developing
diabetic foot complications?

DIAGNOSIS

There are many factors that impact the development of the diabetic
foot. The well-known factors include greater than 10-year duration of diabetes, older age, male sex, poor glycemic control, smoking, obesity, and cardiovascular, renal, and retinal comorbidities.4-7
All of these risk factors impact the development of ischemia, neuropathy, and infection. There has not been a recent comprehensive
analysis of factors with a risk-related ranking for the development
of diabetic foot complications. There have, however, been some
specific investigations into risk factors to help identify patients
prior to the development of complications.
Dialysis treatment is an independent risk factor for the development of foot ulceration in patients with diabetes and renal

2. When the vascular supply to the foot needs to be imaged due


to foot pain/infection, which is the best modality?
Imaging of the vessels in the leg and foot in the setting of diabetes
and a foot complication (pain, infection, etc.) poses specific challenges. Generally, these patients often have extensive tibial disease that may spare the pedal vessels. In addition, many patients
have some degree of impaired renal function. Therefore, imaging
studies need to be able to accurately depict the anterior and posterior circulations, collateral pathways, and the patency of pedal
vessels to allow the physician to choose the appropriate method
762

PMPH_CH96.indd 762

5/22/2012 6:06:12 PM

The Diabetic Foot

(open bypass and/or endovascular) to establish inline blood flow.


And, these studies should enable permit preservation of renal
function.
The gold standard imaging study to which all others are compared is digital subtraction angiography (DSA). As the literature
demonstrates, the other modalities, especially magnetic resonance
angiography (MRA), are threatening the dominance of DSA.
Hofmann et al. prospectively studied 37 patients comparing DSA
with contrast enhanced-MR and duplex ultrasound for pedal
artery imaging. Two independent radiologists reviewed the images
and scored the visualization of the pedal vessels. These investigators concluded that duplex and MRA proved superior to DSA for
the prediction of distal outflow vessels.10 In an additional study,
the same investigators examined the accuracy of preoperative
high-frequency duplex scanning of potential pedal target vessels.
The population consisted of 33 diabetics with critical limb ischemia. The results were compared to DSA, MRA, intraoperative
findings, and early postoperative results. Duplex scanning predicts more pedal target segments than DSA. In addition, duplex
had the highest agreement in prediction of distal anastomotic site
(kappa 0.82).11
MRA is noninvasive and avoids the use of iodinated contrast
material and exposure to ionizing radiation used in DSA. Two
studies assessed the accuracy of MRA and MRI studies for detection of foot vasculature occlusions: sensitivities were 86% and 79%
and specificities were 27% and 87%, respectively.12 Enthusiasm for
contrast enhanced MRA in the diabetic population must be tempered with the recent Food and Drug Administration (FDA) warning regarding gadolinium and the risk of developing nephrogenic
systemic fibrosis or nephrogenic fibrosing dermopathy in patients
with underlying renal failure. A study by Langer et al. examined
3D MRA, DSA, and duplex scanning of the distal calf and pedal
vasculature in critical limb ischemia in 29 patients prospectively.13
These investigators found that MRA could identify more patent
pedal vessels than the other modalities.
The use of computed tomographic angiography (CTA) in the
diagnosis of peripheral arterial disease (PAD) has been extensively
examined and recently reviewed as a systematic review and metaanalysis.14 Bipat et al. concluded that CTA is an accurate study for
PAD assessment in patients with intermittent claudication. The
authors note that the group of studies examined in this systematic
review is weak with respect to study methodology and therefore
it is difficult to make definitive conclusions. There is a paucity of
studies looking at the utility of CTA as a diagnostic and preoperative mapping modality for the pedal vessels.
Answer: MRA and duplex ultrasonography can replace DSA
as the imaging modalities to be used for assessment of the diabetic
foot (Level III evidence, Grade B recommendation).

TREATMENT
3. Are there pharmacologic agents that can effectively treat
neuropathic symptoms?
Small-nerve-fiber neuropathy can present as pain and lead to foot
ulceration. The large-nerve-fiber neuropathies lead to numbness,
unbalance, and thus increase the potential for falls and fractures.
The symptoms of the most common form of diabetic neuropathy
result from abnormalities in both types of nerve fibers. A wide

PMPH_CH96.indd 763

763

range of treatments has been evaluated to provide relief from the


neuropathic pain.
Consensus guidelines from different groups exist that outline the appropriate tiers of treatment options based on clinical
evidence. Argoff et al. published guidelines in 2006.15 This group
consisted of pain specialists who recommend the use of duloxetine,
oxycodone, pregabalin, and the tricyclic antidepressants as the firsttier approach to the treatment of diabetic peripheral neuropathy. The
next tier approach includes the anticonvulsants carbamazepine and
gabapentin. In addition, the International Association for the Study
of Pain (IASP) Neuropathic Pain Special Interest Group undertook
an evaluation of the literature surrounding the management of neuropathic pain.16 Results from randomized trials supported the use of
certain antidepressants, calcium channel ligands (gabapentin and
pregabalin), and topical lidocaine as first-line treatments. Second-line
treatments included opioid analgesics and tramadol. Third-line treatments include other antidepressant and antiepileptic medications,
topical capsaicin, mexilentine, and N-methyl-d-aspartate receptor
antagonists. As you can see, there is a lack of consensus between these
guidelines as far as tier ranking.
In a double blinded, placebo controlled study, Jude et al.
examined the effect of l-arginine on the microcirculation in the
neuropathic diabetic foot.17 The endpoints included foot transcutaneous oxygen pressure (TcPO2), neuropathy disability score
(NDS), and vibration perception threshold (VPT) at baseline and
after 3 months of treatment. l-arginine did not have an effect on
clinical neuropathy.
Another approach to the neuropathic pain is to use cutaneous anesthesia. The thinking behind this is that the cutaneous anesthesia works by inducing a silent area in the primary
somatosensory cortex thereby allowing adjacent cortical areas to
expand resulting in enhanced sensory processing. Lundborg et al.
studied this phenomenon in a double blind study with 37 patients
assigned to either an anesthetic cream or placebo that was applied
to the skin of the lower leg for 1.5 hours.18 The endpoint of sensibility at five points of the sole was assessed as was the change in
touch threshold at the first metatarsal head from pretreatment to
1.5 hours. The application of the anesthetic cream improved the
touch thresholds significantly at 1.5 and 24 hours. In addition,
there was also a decreased vibration threshold.
Answer: A wide range of pharmacologic agents has been evaluated for the alleviation of neuropathic foot pain. Guidelines
drawn from examination of the literature point out that duloxetine and pregabalin have Level 1 evidence to support their use
(Grade A recommendation).
4. How should osteomyelitis of the diabetic foot be diagnosed
and imaged?
In most cases, osteomyelitis of a diabetic foot occurs secondary to the inability of an ischemic barrier to stop the spread
of soft tissue infection to the bone. Diagnosis of osteomyelitis
in the diabetic foot based on physical fi ndings alone is notoriously unreliable. The exception to this rule is the probe to the
bone (PTB) sign. Th is sign has a sensitivity of 66%, specificity
of 85%, and positive predictive value (PPV) of 89%.19 However,
in another study, the PTB test had a lower PPV at 57% and a
higher negative predictive value (NPV) at 98%. 20 A larger study
of this diagnostic test would shed light on the true predictive
power of PTB.

5/22/2012 6:06:12 PM

764

Surgery: Evidence-Based Practice

Diagnostic value of blood laboratory results is poor as these are


nonspecific for osteomyelitis. However, there are many studies that
demonstrate a correlation between a high ESR and the presence
of osteomyelitis.21-23 There is variability around the ESR threshold
value to predict the diagnosis.
The imaging test of choice is MRI.24 If MRI is contraindicated,
F-18 fluorodeoxyglucose (FDG)-positron emission tomography is
a highly sensitive and specific test (81% and 93% respectively).25
Answer: Imaging test of choice is MRI (Level II evidence,
Grade B recommendation).

5. Which antibiotics should be used to treat osteomyelitis?


The two main components for treatment of diabetic foot osteomyelitis (DFO) are (1) medical, consisting of wound care and
off loading, antibiotics, and specialized footwear; and (2) surgical, including soft tissue debridement, bone excision/resection, and vascular interventions. Due to the complexity of the
comorbidities that these patients also have, a multidisciplinary
teamcomprising surgeons (vascular surgeon/plastic surgeon),
orthopedist/podiatrist, infectious disease specialists, endocrinologists, and wound care nursesshould treat DFO. Both
approaches (medical and surgical) have their indications and
specific recommendations.
Once osteomyelitis is diagnosed if no emergent vascular intervention is mandated (e.g., critically ischemic limb), the wound
should be cleaned and visible necrotic tissue debrided; glycemic
control and nutritional status should be optimized. If the infection is considered mild (purulence and inflammation with localized infection that does not extend more than 2 cm beyond ulcer
margins), antibiotics should be started.
Traditionally it was thought that the antibiotics to be chosen should achieve a good concentration in the bone and should
be administered via parenteral route. Both concepts have been
under attack lately, studies showing that it is more important
to choose an antibiotic that has high serum concentration and
that there are oral regimens that are effective if administered
for a longer period. In terms of the right antibiotics there are few
clear-cut criteria to be used. Lazzarini et al. reviewed 93 trials
published over three decades and concluded that (1) acute osteomyelitis fares better than chronic osteomyelitis; (2) intravenous
(IV) drugs are no better than oral antibiotic therapy (although
there were few trials randomizing IV versus oral therapy); (3) there
is no specific antibiotic that is better than another in terms of
outcomes.26
Guidelines regarding antibiotic treatment regimens have
been published (i.e., by Infectious Diseases Society of America)
and these include recommendations for both route and duration of antibiotic therapy. 27,28 Most studies treated patients with
4 to 6 weeks of antibiotic therapy, although some used longer
durations.
Together with systemic antibiotic therapy the wound should
be addressed with either topical anti-infective or with VACtherapy. The role of local antibiotic elution devices (e.g., beads or
cement) in DFO is not yet established, although they may achieve
high local (without systemic) antibiotic levels and assist with
management of the surgical dead space (antibiotic-impregnated
polymethylmethacrylate (PMMA) beads; gentamicin-loaded
PMMA beads, antibiotic calcium sulfate pellets, antibiotic polylactic microspheres).

PMPH_CH96.indd 764

Answer: No specific antibiotic has been shown to be superior


to another (Level II evidence, Grade B recommendation).
6. If the diabetic foot requires revascularization, do patients
do better with operative bypass versus angioplasty intervention
as the first line?
Many studies have examined the utility of angioplasty for revascularization in patients who are not candidates for bypass surgery.29,30
The overall conclusion is that this method provides an acceptable rate of limb salvage in the highest-risk population. However,
the choice between bypass and angioplasty in severe ischemia
of the secondary to infrainguinal disease has been the subject
of a large randomized controlled trial conducted in the United
Kingdom (BASIL trial).31 Although only less than half (42%) of
the patients registered in the trial had diabetes, the location of the
vascular disease was similar (tibial vessels). The conclusions from
this trial were that (1) amputation-free survival was similar in
both groups (bypass vs. angioplasty); (2) if the patient has less than
10% chance of being alive at 2 years or a vein is not available for
bypass a more conservative approach (angioplasty or, according
to some critics of the study, primary amputation or symptomatic
best medical treatment) should be taken. It has to be taken into
account that renal failure poses an important problem for these
patients. However, if the patient has a life expectancy greater than
2 years, an infrainguinal bypass with vein should be attempted.
In most cases that would require that the outflow vessel be located
in the foot/ankle (dorsalis pedis). The perioperative morbidity and
mortality is higher than in the angioplasty group; however, after
2 years the rates become lower.
Answer: Either angioplasty or open revascularization is an
appropriate method to enhance blood flow to the diabetic foot.
The physician should consider the overall fitness of the patient
and likelihood of being alive at 2 years following the procedure
(Level Ib evidence, Grade A recommendation).

7. Is hyperbaric oxygen therapy an effective healing adjunct for


diabetic foot wounds?
Many investigators have examined the efficacy of different
wound healing adjuncts to treat challenging diabetic foot ulcers.
Oxygen therapy has a theoretical utility based on the failure of
wounds to heal in hypoxic environments. This is one reason why
hyperbaric oxygen therapy (HBOT) has a long history of investigation for chronic wounds. Some studies have shown that the
use of HBOT can help heal ulcers and decrease the major amputation rate.32-34
A prospective, double blind, randomized, controlled clinical trial comparing standard wound care with adjunctive HBOT
to standard wound care for the treatment of chronic, nonhealing
ulcers of the lower limb in patients with DM has been posted in
clinical trials.gov.35 This trial will randomize 59 patients to each
arm and follow them for 12 weeks to evaluate the primary endpoint of freedom from major amputation. This trial should provide Level 1 evidence regarding the effectiveness of HBOT for
wounds of the diabetic foot.
Answer: HBOT of chronic wounds may reduce the risk of
major amputation. More definitive evidence will be available after
the double blind randomized controlled trial (Current Level III
evidence, Grade C recommendation).

5/22/2012 6:06:12 PM

The Diabetic Foot

765

Clinical Question Summary


Question

Answer

Level of
Evidence

Grade of
Recommendation

1 Who is at risk for diabetic foot ulcer


(DFU) formation?

Diabetic for more than 10 years, advanced age,


smoking, obesity, poor glycemic control,
cardiovascular, renal and retinal comorbidities.
Dialysis treatment is an independent risk factor.
Increase in vibration perception threshold by
one unit.

IIb

IIb

2 What is the best test to image the foot


vascularity?

MRA and duplex ultrasound can rival


angiography in some cases.

III

3 Which pharmacologic agents can treat


neuropathic symptoms?

Duloxetine and pregabalin are preferred.

Ia

4 What is the test of choice to diagnose


osteomyelitis?

MRI.

IIB

5 What is the preferred antibiotic regimen?

None has been shown to be superior.

IIb

6 Is angioplasty or bypass preferred to


revascularize the diabetic foot?

Either depending on other comorbidities and


availability of conduit.

Ia

7 Is adjunctive hyperbaric oxygen beneficial?

No convincing current evidence.

III

REFERENCES
1. Boyle JP, Thompson TJ, Gregg EW, et al. Projection of the year
2050 burden of diabetes in the US adult population: dynamic
modeling of incidence, mortality, and prediabetes prevalence.
Popul Health Metrics. 2010;8:1-12.
2. Boulton AJM, Vileikyte L, Ragnarson-Tennvall G, et al. The global
burden of diabetic foot disease. Lancet. 2005;366:1719-1724.
3. Cheer K, Shearman C, Jude EB. Managing complications of the
diabetic foot. BMJ. 2009;339:b4905.
4. Kalish J, Hamdan A. Management of diabetic foot problems.
JVS. 2010:51(2):476-486.
5. Stuck RM, Song M-W, Budiman-Mak E, et al. Charcot arthropathy risk elevation in the obese diabetic population. Am J Med.
2008;121(11):1008-1014.
6. Urbancic-Rovan V. Causes of diabetic foot lesions. Lancet. 2005;
366:1675-1676.
7. Pataky Z, Vischer U. Diabetic foot disease in the elderly. Diabetes
Metab. 2007;33:S56-S65.
8. Ndip A, Rutter MK, Vileikyte L, et al. Dialysis treatment is
an independent risk factor for foot ulceration in patients with
diabetes and stage 4 or 5 chronic kidney disease. Diabetes Care.
2010;33(8):1811-1816.
9. Abbott CA, Vileikyte L, Williamson S, et al. Multicenter study
of the incidence of and predictive risk factors for diabetic neuropathic foot ulceration. Diabetes Care. 1998;21(7):1071-1075.
10. Hofmann WJ, Forstner R, Kofler B, et al. Pedal artery imaginga
comparison of selective digital subtraction angiography, contrast
magnetic resonance angiography and duplex ultrasound. Eur J
Vasc Endovasc Surg. 2002;24(4):287-292.
11. Hofmann WJ, Walter J, Ugurluoglu A, et al. Preoperative highfrequency duplex scanning of potential pedal target vessels. JVS.
2004;39(1):169-175.
12. Burch CR, Aguiar-Ibez R, Craig D, et al. Duplex ultrasonography, magnetic resonance angiography, and computed tomography angiography for diagnosis and assessment of symptomatic,
lower limb peripheral arterial disease: systematic review. BMJ.
2007;334:1257.

PMPH_CH96.indd 765

13. Langer S, Krmer N, Mommertz G, et al. Unmasking pedal


arteries in patients with critical ischemia using time-resolved
contrast-enhanced 3D MRA. JVS. 2009;49(5):1196-1202.
14. Met R, Bipat S, Legemate DA, et al. Diagnostic performance of computed tomography angiography in peripheral arterial disease: a
systematic review and meta-analysis. JAMA. 2009;301(4):415-424.
15. Argoff CE, Backonja MM, Belgrade MJ, et al. Consensus guidelines: treatment planning and options. Diabetic peripheral neuropathic pain. Mayo Clin Proc. 2006;81:S12-S25.
16. OConnor AB, Dworkin RH. Treatment of neuropathic pain: an
overview of recent guidelines. Am J Med. 2009;122:S22-S32.
17. Jude EB, Dang C, Boulton AJM. Effect of L-arginine on the microcirculation in the neuropathic diabetic foot in type 2 diabetes
mellitus: a double-blind, placebo-controlled study. Diabetic Med.
2009;27:113-116.
18. Lundborg GN, Bjrkman ACG, Rosn BN, et al. Cutaneous
anaesthesia of the lower leg can improve sensibility in the diabetic
foot. A double-blind, randomized clinical trial. Diabetic Med.
2010;27:823-829.
19. Grayson ML, Gibbons GW, Balogh K, et al. Probing to bone in
infected pedal ulcers. A clinical sign of underlying osteomyelitis
in diabetic patients. JAMA. 1995;273(9):721-723.
20. Lavery LA, Armstrong DG, Peters EJ, et al. Probe-to-bone test
for diagnosing diabetic foot osteomyelitis: reliable or relic?
Diabetes Care. 2007;30(2):270-274.
21. Ertugrul BM, Savk O, Ozturk B, et al. The diagnosis of diabetic
foot osteomyelitis: examination findings and laboratory values.
Med Sci Mont. 2009;15(6):CR307-CR312.
22. Malabu UH, Al-Rubeaan KA, Al-Derewish M. Diabetic foot
osteomyelitis: usefulness of erythrocyte sedimentation rate in its
diagnosis. West Afr J Med. 2007;26(2):113-116.
23. Rabjohn L, Roberts K, Troiano M, et al. Diagnostic and prognostic value of erythrocyte sedimentation rate in contiguous osteomyelitis of the foot and ankle. J Foot Ankle Surg. 2007;46(4):
230-237.
24. Kapoor A, Page S, Lavalley M, et al. Magnetic resonance imaging
for diagnosing foot osteomyelitis: a meta-analysis. Arch Intern
Med. 2007;167(2):125-132.

5/22/2012 6:06:12 PM

766

Surgery: Evidence-Based Practice

25. Nawaz A, Torigian DA, Siegelman ES, et al. Diagnostic performance of FDG-PET, MRI, and plain fi lm radiography (PFR) for
the diagnosis of osteomyelitis in the diabetic foot. Mol Imaging
Biol. 2010;12(3):335-342.
26. Lazzarini L, Lipsky BA, Mader JT. Antibiotic treatment of osteomyelitis: what have we learned from 30 years of clinical trials? In
J Infect Dis. 2005;9(3):127-138.
27. Malloy KM, Davis GA. Summary of ASHP/IDSA/SIDP vancomycin monitoring recommendations: a focus on osteomyelitis.
Orthopedics. 2009;32(7):499.
28. Berendt AR, Peters EJ, Bakker K, et al. Specific guidelines for
treatment of diabetic foot osteomyelitis. Diabetes Metab Res Rev.
2008;24:S190-S191.
29. Zhu YQ, Zhao JG, Liu F, et al. Subintimal angioplasty for belowthe-ankle arterial occlusions in diabetic patients with chronic
critical limb ischemia. J Endovasc Ther. 2009;16(5):604-612.
30. Werneck CC, Lindsay TF. Tibial angioplasty for limb salvage
in high-risk patients and cost analysis. Ann Vasc Surg. 2009;
23(5):554-559.

PMPH_CH96.indd 766

31. Bradbury AW, Adam DJ, Bell J, et al. Bypass versus angioplasty
in severe ischaemia of the leg (BASIL) trial: analysis of amputation free and overall survival by treatment received. JVS. 2010;51:
S18-S31.
32. Kranke P, Bennett M, Roeckl-Wiedmann I, et al. Hyperbaric
oxygen therapy for chronic wounds. Cochrane Database Syst Rev.
2004;2:CD004123.
33. Lndahl M, Katzman P, Nilsson A, et al. Hyperbaric oxygen
therapy facilitates healing of chronic foot ulcers in patients with
diabetes. Diabetes Care. 2010;33(5):998-1003.
34. Roeckl-Wiedmann I, Bennett M, Kranke P. Systematic review of
hyperbaric oxygen in the management of chronic wounds. Br J
Surg. 2005;92(1):24-32.
35. OReilly DJ, Linden R, Fedorko L, et al. A prospective, doubleblind, randomized, controlled clinical trial comparing standard wound care with adjunctive Hyperbaric Oxygen Therapy
(HBOT) to standard wound care only for the treatment of
chronic, non-healing ulcers of the lower limb in patients with
diabetes mellitus: a study protocol. Trials. 2011;12(1):69.

5/22/2012 6:06:12 PM

Commentary on
The Diabetic Foot
Jodi Walters and David G. Armstrong

The chapter The Diabetic Foot by MacLean and Codreanu provides a concise summary of the lower extremity risk and complications relating to diabetes. In addition, it addresses the key
factors pertaining to imaging modalities and preferred treatment
of peripheral arterial disease, osteomyelitis, and neuropathic
wounds.
Diabetes is estimated to double worldwide within the next 18
to 20 years. Fifteen to twenty-five percent of patients with diabetes
have a lifetime risk of developing a diabetic foot ulcer and approximately 15% of those will result in some level of amputation.1,2
Foot complications are associated with greatest number of hospital admissions in patients with diabetes.3,4
A systemic approach to identifying risk and underlying disease coupled with targeted treatment/interventions is critical. It is
also important to develop standardized guidelines for imaging of
associated peripheral vascular disease, with the intent of improving outcomes. It is with this diagnostic and therapeutic spirit that
the authors attempt to answer the following questions:

universal hemodynamic and anatomic perfusion information.


Therefore, a combination of modalities is generally the order of
the day.
3. Are there pharmacologic agents that can effectively treat
neuropathic symptoms?
We tend to treat symptomatic diabetic polyneuropathy similar
to the algorithm outlined by the American Diabetes Association
(ADA). Our stepwise approach includes initially excluding nondiabetic etiologies, then stabilizing glycemic control. If symptoms
continue, we find it most beneficial to begin with antidepressant
drugs such as amitriptyline and other tricyclics and then progress
to anticonvulsants (gabapentin, pregabalin), alpha-lipoic acids,
and only additionally add opioid or opioid-like drugs if necessary for pain control. Furthermore, nonpharmacologic, topical,
or physical modalities can be used as supplements at nearly any
stage.
4. How should osteomyelitis of the diabetic foot be diagnosed
and imaged?

1. Who is at greatest risk for developing diabetic foot


complications?

Since the accuracy of the probe to bone test is based on pretest


probability, one may have a higher predictive value when patients
have a moderate or severe foot infection and are hospitalized.8,9
The probe to bone test in the clinic or outpatient setting may have
a lower predictive value as they tend to have much less severe infections as well as less tissue necrosis or loss. However, if exposed
bone is clearly evident, there is a high likelihood that osteomyelitis is present and additional noninvasive evaluation may not be
required. Many expensive modalities can be performed such as
bone scan, MRI, and CT, which may aid in higher-level surgical
planning. Yet by treating infection and/or removing bone surgically that is underlying a chronic ulcer and preventing it from
healing may be the best option, as it removes both the infectious
nidus and often the deforming force causing the index wound in
the first place.

Factors significantly associated with diabetic foot ulceration


include male sex, duration of diabetes (>10 years), poor glycemic control, neuropathy (leading to foot deformity, limited joint
mobility, and high plantar pressures), and history of amputations.5 It is of critical importance to identify these factors early
and stratify patients for evidence-based medicine. Recent studies
suggest that peripheral vascular disease significantly increases the
risk for foot ulceration.6 There is increased morbidity and mortality associated with the diabetic population with peripheral neuropathy and previous ulceration. Patients who have a neuropathic
ulcer have a 45% mortality rate in 5 years and the patients with
peripheral arterial disease have a comparable rate of mortality to
major forms of cancer.7 Prevention by identifying individuals at
risk is most effective in improving health-related outcomes in this
population.

5. Which antibiotics should be used to treat osteomyelitis?


2. When the vascular supply to the foot needs to be imaged due
to foot pain/infection, which is the best modality?

One point to consider when choosing antibiotic therapy is the evidence of acute versus chronic osteomyelitis and if surgical resection
is performed or even an option. Empiric antibiotics are essential to
control cellulitis and soft tissue infection with or without the presence of bone infection. Bone biopsy should be completed to identify

The choice of diagnostic modality depends on the context, that is,


diagnosis (or prevention), assessing wound healing, and surgical
planning. At present, there is no ideal test that gives both precise
767

PMPH_CH96.indd 767

5/22/2012 6:06:12 PM

768

Surgery: Evidence-Based Practice

a specific pathogen and susceptibility data should be obtained when


feasible. As it may be impractical to obtain a bone biopsy in some
circumstances, since the biopsy site may heal poorly in the setting
of advanced vascular disease, empiric management may, in some
cases, be pursued.8,10 If bone consistent with osteomyelitis is resected
to a clear proximal margin, as in amputations, a much shorter
course of antibiotic therapy may be appropriate. In such cases, antibiotic therapy can usually be discontinued when operative wounds
are healing without signs of infection, at least until debrided bone
has been covered by vascularized soft tissue. However, if surgery is
not a viable option or if the clinical picture is less ominous and the
debrided bone is covered by viable soft tissue that has a good vascular supply, then osteomyelitic suppression with antibiotics for 4 to
6 weeks is optimal. Antibiotic selection should be based on biopsy
specification and/or wound infection response.
6. If the diabetic foot requires revascularization, do patients
do better with operative bypass versus angioplasty intervention
as the first line?
Although endovascular versus open surgical repair remains a subject
for debate and discussion, it may be not only the type of lesion but
also the demand based on the size of the index soft tissue defect.11
7. Is hyperbaric oxygen therapy an effective healing adjunct for
diabetic foot wounds?
Hyperbaric oxygen (HBO) therapy has been used as an adjunct to
good ulcer care, including debridement, antibiotic therapy, and
revascularization in the treatment of chronic, nonhealing wounds
associated with diabetes or nondiabetic vascular insufficiency. It
has been reported that hyperbaric therapy can serve an adjunctive role healing diabetic wounds.12 However, cost-effectiveness
has to be considered and these associated costs and risks may not
be appropriate for all people or all wounds. Perhaps even more
important than any potential clinical utility is the fact that HBO
therapy is not a substitute for a physician or nurses experience
and skill-set. A diabetic foot center, although it may contain an
HBO unit, should not be built around an HBO unit.

PMPH_CH96.indd 768

REFERENCES
1. Dyck PJ. Detection, characterization, and staging of polyneuropathy: assessed in diabetics. Muscle Nerve. 1988;11:21-32.
2. Singh N, Armstrong DG, Lipsky BA. Preventing foot ulcers in
patients with diabetes. JAMA. 2005;293(2):217-228.
3. Reiber GE. Epidemiology of foot ulcers and amputations in the
diabetic foot. In: Bowker JH, Pfeifer MA, eds. The Diabetic Foot.
St. Louis, MO: Mosby; 2001:13-32.
4. Reiber GE, Vileikyte L, Boyko EJ, et al. Causal pathways for incident lower-extremity ulcers in patients with diabetes from two
settings. Diabetes Care. 1999;22(1):157-162.
5. Lavery LA, Armstrong DG, Vela SA, Quebedeaux TL, Fleischli
JG. Practical criteria for screening patients at high risk for diabetic foot ulceration. Arch Intern Med. 1998;158:158-162.
6. Lavery LA, Peters EJ, Williams JR, Murdoch DP, Hudson A,
Lavery DC. Reevaluating the way we classify the diabetic foot:
restructuring the diabetic foot risk classification system of the
International Working Group on the Diabetic Foot. Diabetes
Care. 2008;31(1):154-156.
7. Armstrong DG, Wrobel J, Robbins JM. Guest editorial: are diabetes-related wounds and amputations worse than cancer? Int
Wound J. 2007;4(4):286-287.
8. Lipsky BA. A report from the international consensus on diagnosing and treating the infected diabetic foot. Diabetes Metab
Res Rev. 2004;(20 Suppl 1):S68-77.
9. Lavery LA, Armstrong DG, Peters EJ, Lipsky BA. Probe-to-bone
test for diagnosing diabetic foot osteomyelitis: reliable or relic?
Diabetes Care. 2007;30(2):270-274.
10. Lipsky BA, Berendt AR, Embil J, De Lalla F. Diagnosing and
treating diabetic foot infections. Diabetes Metab Res Rev. 2004;
20(Suppl 1):S56-64.
11. Zhan L, Bharara M, White M, et al. Comparison of early hemodynamics after endovascular therapy and open surgical bypass
in patients with diabetes and critical limb ischemia: does mode
of revascularization matter? Vascular Annual Meeting. Chicago,
IL: Society for Vascular Surgery; 2011.
12. Londahl M, Katzman P, Nilsson A, Hammarlund C. Hyperbaric oxygen therapy facilitates healing of chronic foot ulcers in
patients with diabetes. Diabetes Care. 2010;33(5):998-1003.

5/22/2012 6:06:12 PM

CHAPTER 97

Varicose Veins and


Venous Insufficiency
M. K. Sheehan

Proreflux and obstruction


Pnno pathophysiology identified

Varicose veins and chronic venous insufficiency are common


problems. Varicose veins are subcutaneous veins dilated to 3 mm
or more in a standing position. Venous insufficiency is a result of
delayed valvular closure leading to pooling of blood and venous
distension. Prevalence estimates of each differ in studies and vary
based on study population, imaging techniques, definition, and
methodology. For varicose veins, studies estimate a prevalence range
from 2% to 56% in men and <1% to 60% in women with 2-year
incidence rates of 39.4 per 1000 men and 51.9 per 1000 women,
whereas with respect to venous insufficiency, the estimated prevalence ranges from <1% to 17% in men and <1% to 40% in women.1,2
Venous insufficiency, which may or may not include varicose
veins, can present in and progress to a wide spectrum that can
vary the treatment algorithm. Therefore, a classification system,
CEAP, outlined below, was developed that helps to describe the
disease process affecting each patient.

Treatment algorithms and outcomes may vary depending on


the location and severity of the disease.

RISK FACTORS
1. What are the risk factors for venous insufficiency?

Gender
Many studies have shown a higher prevalence of varicose veins in
women than in men although selection bias cannot be completely
excluded as women are more likely to present for evaluation and
treatment of venous problems.

CClinical classification
C0no visible or palpable signs of venous disease
C1telangiectasis or reticular vein; veins less than 3 mm
C2varicose veins; veins greater than 3 mm
C3edema
C4apigmentation or eczema
C4blipodermatosclerosis or atrophie blanche
C5healed venous ulcer
C6active venous ulcer

Age
A majority of studies have found that prevalence increases with
age. The Framingham Study found a varicose vein prevalence in
persons less than 30 years of age of 1% in men and 10% in women
whereas in persons over age 70 the prevalence was 57% in men and
77% in women.

Family History

EEtiology
Epprimary
Essecondary
Enno venous cause identified

A family history of varicose veins has been shown to increase


ones risk to develop varicose veins by 21.5 times; however, the
same study did not find family history to be a significant risk factor in chronic venous insufficiency.

AAnatomy
Assuperficial veins
Apperforator veins
Addeep veins
Anno location identified

Pregnancy
Pregnancy leads to increased intra-abdominal pressure, as well
as upregulation of hormones such as relaxin, estrogen, and
progesterone all of which can contribute to venous distension. One study found that increased parity increased risk of
varicose vein development with a prevalence of 32% in women

PPathophysiology
Prreflux
Poobstruction
769

PMPH_CH97.indd 769

5/22/2012 6:07:22 PM

770

Surgery: Evidence-Based Practice

with no pregnancies up to 59% in women with four or more


pregnancies. 2
Answer: Family history, age, and parity are all significant
risk factors for development of varicose veins and venous insufficiency2 (Level II evidence).

shown to prevent or slow disease progression. However, compression has been shown to be effective for treatment of venous ulcers
with multilayer compression being more effective than single layer
(Level I evidence; Grade A recommendation).

INVASIVE PROCEDURES
IMAGING
2. How is venous insufficiency diagnosed?
Venous duplex has become the gold standard test for venous insufficiency. The test should be performed with the patient standing with
most of their weight on the leg that is not being examined. The great
and small saphenous are examined throughout their length with
intermittent compression and release of the calf. In the deep system, at the minimum, the common femoral, femoral, and popliteal
veins should all be examined. Reversal of flow greater than 500 ms
in the saphenous veins is indicative of reflux. In the common femoral, femoral, and popliteal veins reflux is defined as reversal of flow
greater than 1000 ms, while in perforating veins reflux is defined as
reversal of flow greater than 350 ms in a vein a perforator at least
4mm in size. Duplex examination is operator dependent.3
Answer: Duplex ultrasonography is the diagnostic test of
choice for determining presence of reflux within the venous system (Level III evidence).

COMPRESSION
3. Is compression an effective treatment for venous insufficiency?
Compression either via wrapping or hose, since noninvasive,
is the first-line treatment for varicose veins and venous insufficiency. Compression hose has graduated compression, with highest compression at the foot and less compression as they progress
proximal. This graduation improves venous hemodynamics and
keeps blood and fluids moving more proximal and discourages
pooling of the same. Many individuals will find some relief with
compression, but few find full relief. In addition, compression
helps to treat the symptoms but does not treat the underlying condition of valvular reflux. As a result, when the compression hose
are removed the veins are still insufficient and symptoms recur.
In a recent review assessing the utility of compression hose for
treatment of uncomplicated varicose veins, data were found to be
insufficient. Only one systematic review was found that included
three randomized control trials. The review concluded that although
compression hose may control some symptoms, there is no good evidence that they help slow progression of disease.4 Another review
by Palfreyman et al.5 found 25 studies addressing the use of compression hose for uncomplicated venous disease. They found that
although compression hose may help with symptoms, there was
not enough data to support the theory that compression hose helps
to slow progression of disease or prevent recurrence. On the other
hand, a Cochrane meta-analysis of 22 trials examining effectiveness
of compression for venous ulcers found that compression was more
effective than no compression in healing venous ulcers with the rate
of healing ranging from 23% to 84% at 3 months to 1 year. Multilayer
compression is more effective than single-layer compression.6
Answer: For uncomplicated venous insufficiency and varicose
veins, compression may help alleviate symptoms but has not been

PMPH_CH97.indd 770

4. What are the best invasive procedures for venous insufficiency?


To more permanently treat varicose veins or venous insufficiency
in the superficial system, the involved veins need to be eliminated.
Many methods of destruction have been developed in recent
years. The traditional method had been stripping and ligation,
which involved a groin incision for ligation of the saphenofemoral junction and removal of the greater saphenous vein (GSV)
via avulsion. This procedure, although effective, generally has a
painful and somewhat prolonged recovery period. The procedure
involves an avulsion of the GSV or short saphenous vein (SSV)
for a distance. Tributaries are avulsed from the main trunk leading to significant bruising and hematoma that leads to increased
postoperative pain. More recently, noninvasive methods of ablation using radiofrequency (RFA) or laser energy (EVLT), and
foam sclerotherapy have been introduced. All of these procedures
have decreased morbidity and recovery periods when compared
with stripping and ligation. One study comparing RFA and EVLT
found decreased ecchymosis and pain at 2 weeks following RFA
compared to EVLT, but no difference at 1 month.7 A meta-analysis
looking at minimally invasive therapies versus surgical ligation
and stripping found 64 eligible studies in which 12,320 limbs were
treated. They found that estimated pooled success rates for stripping were 78%, for foamed sclerotherapy 77%, for RFA 94%, and
for EVLT 84%. After adjusting for follow-up they concluded that
EVLT was superior to stripping and ligation whereas RFA and
sclerotherapy were both as effective as stripping and ligation.8
Answer: EVLT appears to be most effective in the treatment
of venous insufficiency of the superficial axial system, although
RFA, foamed sclerotherapy, and stripping and ligation all have
acceptable results. Stripping and ligation have higher morbidity
associated with the procedure than other methods (Level Ib and II
evidence).

VALVE RECONSTRUCTION
5. When is venous reconstruction indicated?
Venous insufficiency of the deep system requires separate consideration from insufficiency of the superficial system. When both deep
and superficial systems have evidence of insufficiency, treatment of
the superficial system helps to offload overall venous hypertension
and the deep system insufficiency. This can result in correction of
deep venous reflux in half of patients and ulcer healing in 77% of
patients in one study. When only deep system insufficiency exists
or remains, compression dressings and graded compression hose
become the mainstay of treatment for venous ulcers and symptoms. In patients with severe recalcitrant ulceration due to deep
system reflux, open surgical intervention can be performed on
the incompetent valves to restore competency and reduce venous
hypertension. Interventions available include internal or external

5/22/2012 6:07:22 PM

Varicose Veins and Venous Insufficiency

valvuloplasty, external banding, valve transplantation, and valve


transposition. A Cochrane review found three studies involving 233 patients who had mild to moderate symptoms but no
ulcerations. Although there was a decrease of 15 mm Hg in veins
treated with external plication, ambulatory venous pressures still
remained high. They concluded that although surgery may produce

771

moderate and sustained results, overall there was insufficient data


to recommend surgery for patients without ulcerations.9
Answer: Valvular reconstruction may be beneficial in complicated venous disease when compression is not adequate; however, data at this time are insufficient (Level III evidence; Grade C
recommendation).

Clinical Question Summary


Question

Answer

1 How is venous insufficiency diagnosed?

Venous duplex is the most frequent


modality.

III

2 Is compression an effective treatment


modality?

Yes.

3 Which invasive modality is best for


superficial reflux treatment?

EVLT.

Ib/II

REFERENCES
1. Lim CS and Davies AH. Pathogenesis of primary varicose veins.
Br J Surg. 2009;96(11):1231-1242.
2. Robertson CE, Fowkes FG. Epidemiology of chronic venous disease. Phlebology. 2008;23(3):103-111.
3. Labropoulos N, Leon LR. Evaluation of chronic venous disease.
In: Mansour MA, Labropoulous N, ed. Vascular Diagnosis. Philadelphia, PA: Elsevier Saunders; 2005.
4. Tisi PV. Best Practice Compression Stockings. On line http://preview.
bestpractice-bmj-com.external.bmjgroup.com/best-practice/
evidence/intervention/0212/0/sr-0212-i1.html
5. Palfreyman SJ, Michaels JA. A systematic review of compression hosiery for uncomplicated varicose veins. Phlebology. 2009;
24:13-33.

PMPH_CH97.indd 771

Level of Evidence

Strength of
Recommendation

6. OMeara S, Cullum NA, and Nelson EA. Compression for venous


leg ulcers. Cochrane Database Syst Rev. 2009;1:26.
7. Almeida JI, Kaufman J, Gckeritz O, et al. Radiofrequency endovenous ClosureFAST versus laser ablation for the treatment of
great saphenous reflux: a multicenter, single-blinded, randomized
study (RECOVERY study). J Vasc Intervent Radiol. 2009;20(6):
752-759.
8. van den Bos R, Arends L, Kockaert M, Neumann M, Nijsten T.
Endovenous therapies of lower extremity varicosities: a metaanalysis. J Vasc Surg. 2009;49(1):230-239.
9. Hardy GR, Abidia A. Surgery for deep venous incompetence.
Cochrane Database Syst Rev. 2009;1:9.

5/22/2012 6:07:22 PM

Commentary on Varicose Veins and


Venous Insufficiency
Seshadri Raju

Varicose veins have been known since antiquity. Some old concepts
and dogma frozen in time over millennia still survive. Only with the
advent of modern investigative techniques is the disease beginning
to be understooda task made difficult by its protean manifestations. A confusingly variable terminology has also accreted over
time. A standardized new terminology has now been proposed.1
Chronic venous disease denotes any or all clinical manifestations
and varicose veins denotes that particular clinical feature. The
CEAP classification has been widely adopted. C1-2 represents various forms of varicose veins that may or may not be associated with
more severe forms of disease represented by C3-6. The disease is usually assigned the highest C Grade present. Venous insufficiency
should be restricted to C3-6. Unfortunately, insufficiency popularized by long currency is confusing as it is often used to denote valve
reflux. We now know that the pathophysiology of chronic venous
disease may be due to reflux, obstruction, and often both. Aetiologically, most chronic venous disease is either due to primary (cryptogenic) or postthrombotic causes, which cannot be differentiated
by clinical signs (C Grade) alone. The CEAP classification is precise
but allows 288 permutation combinations. But this can be reduced
to a handful of clinically usable categories as some CEAP combinations occur more commonly.2
Randomized control studies (Level 1 evidence) are sparse in
the venous literature. This is due to variable expression of the disease, absence of a reliable classification till recently, and inadequacy
of current diagnostic techniques. Many age-old concepts that still
survive in practice remain to be tested on evidence. The recent
introduction of many new technologies and their rapid adoption
because they are minimally invasive has outpaced our ability for
critical assessment. Current practice largely rests on Level II or
more often Level III evidence.

Even so, it is only a qualitative test, and quantification of reflux


remains elusive. Reflux time, often referred to as valve closure
time (VCT), has a poor clinical correlation.3 More importantly,
clinically significant obstruction that occurs predominantly in
the iliac veins is impervious to routine duplex. Even venography
is only ~50% sensitive to obstructive lesions in this location.4
Intravascular ultrasound (IVUS) is the emerging gold standard
in this arena.5 This too is a morphologic method and a clinically
usable test to assess hemodynamic severity of obstruction is as
yet unavailable.
There is increasing recognition that not only the presence of
reflux but also the size of the refluxing vein is significant in assessing its clinical importance. Reversal of flow occurs normally in
perforators during the calf emptying cycle. Most authors do not
consider perforator reflux to be clinically important unless the
perforator is at least 4 mm in size.6
3. Is compression effective treatment for venous insufficiency?
Venous symptoms may be relived or ameliorated by compression
therapy. The mechanism remains poorly understood. It has been
difficult to document hemodynamic improvement commensurate
with clinical improvement.7 Edema, an important component of
advanced venous disease, may be controlled yielding clinical benefit. Competence may be restored to previously reflexive valves
by mechanical compression of the refluxing vein.8 Compression
stockings come in different grades of effective pressure. Hospital
stockings commonly used for deep vein thrombosis (DVT) prophylaxis have an effective pressure of 10 to 12 mm at the ankle.
This may be adequate to control edema. Advanced skin manifestations require 30 mm Hg or even higher pressure for control.
Most patients will fi nd this degree of pressure uncomfortable.
Noncompliance with prescribed stockings is very high (>50%)
even in patients with advanced disease manifestations due to the
pressure or binding, warm weather, cosmetics, cost, poor fit, or
simply lifestyle considerations.9 Contact dermatitis or inability
to apply the compression hosiery due to arthritis, frailty, or fragility of the skin is also factors in many patients.10 Noncompliance with stockings is a major source of symptom recurrence.11
It has been shown that compression stockings often work in the
short term but fail in the long term (12 years). Specific correction of venous pathology, that is, saphenous reflux by ablation
has been shown to be the answer in such cases.12 The emergence

1. What are the risk factors for venous insufficiency?


Venous disease has a predilection for women and the left lower
limb roughly by a factor of 3X in each case. This demographic
imbalance in the literature is so striking and uniform that there is
little chance of a selection bias. Appearance of varicosities in men
in the family tree suggests a genetic predisposition. In women, hormonal influences with age and pregnancy confuse the picture.
2. How is venous disease diagnosed?
Modern duplex equipment that emerged only in the last three
decades has been a great advance in understanding reflux pathology.
772

PMPH_CH97.indd 772

5/22/2012 6:07:22 PM

Varicose Veins and Venous Insufficiency

of minimally invasive ablation techniques that can be carried out


in the office allows early correction of offending pathology more
attractive than before. Efficacy of compression stockings in deep
venous disease of different pathology has not been adequately
examined.
4. What are the best invasive procedures for venous insufficiency?
Owing to the recent introduction of minimally invasive technologies and their rapid adoption, a paradigm shift in the way venous
patients are initially approached has occurred, as indeed is the
case in other fields. The various techniques in popular use appear
to be effective and safe in the short term, but long-term outcome
is unknown. As venous disease manifestations are indolent with
spontaneous remissions, it is generally recognized that follow-up
data spanning 5 years, preferably 10 years is necessary to establish long-term efficacy. It is unlikely that such data in controlled
settings for any of the many techniques in current use will be
available any time soon. Compounding the difficulty, venous disease characteristically has multifocal pathology (see CEAP class
3-6). Correction of one component alone (partial correction) may
yield symptom remission. Choice of optimal technique for durable efficacy will therefore depend on intertechnique comparison
trialsa daunting task given the variety of treatment combinations possible. Even though Level I evidence to support durable
outcome is lacking, the initially minimalistic approach appears
justified as it is safe; all treatment options including more complex invasive and open procedures are preserved in case of initial treatment failure. Currently available treatment options are
listed below in order of increasing complexity/invasiveness and
potential morbidity.
1. Saphenous ablation: Dr Sheehan has provided a balanced review
of this technique. Technological improvements continue to be
introduced in laser (endovenous laser treatment [EVLT]) as well
as radiofrequency ablation (RFA) technologies with promise of
even better outcome and lesser morbidity. Varices in the medial
side of the leg are often associated with saphenous reflux.
Removal of the varices through microincisions (phlebectomy)
should be combined with saphenous ablation to minimize the
chance of recurrence. Limb swelling beyond ankle edema is
not a clinical feature of saphenous insufficiency. Deep venous
pathology is invariably present in advanced limb swelling.
Saphenous ablation alone is unlikely to yield symptom relief in
such cases.
2. Perforator interruption: Endoscopic interruption (subfascial
endoscopic perforator surgery [SEPS]) is the most popular
technique. Recently, RFA as well as foam sclerotherapy have
been adapted for perforator interruption. SEPS when combined
with saphenous ablation can produce long-term (5 years) remission
of symptoms including healing of ulcers in >80% of cases
provided the pathology is primary.13 Results are unsatisfactory
in postthrombotic cases and SEPS is not recommended. Some
authorities believe that perforator interruption adds little
hemodynamic benefit to saphenous ablation alone.14
3. Iliac vein stenting: It has recently become clear that iliac vein
obstruction is common not only in postthrombotic cases but in
primary pathology as well.15 More surprisingly, stent correction
of the obstruction appears to yield excellent symptom remission
including healing of venous ulcers even in cases when severe

PMPH_CH97.indd 773

773

associated reflux remains uncorrected.4 The procedure is minimally invasive with high long-term patency and safety profile.16
This technique has the potential to revolutionize management
of advanced venous cases that are currently consigned to long
compression regimens.
4. Valve reconstruction: It is an intricate procedure and expertise is
sparse as few centers have marshaled enough experience to use
the procedure effectively. Although the results have been good
both in recalcitrant primary and postthrombotic cases,17,18
the technique is likely to see further decline in use due to the
emergence of venous stenting.

REFERENCES
1. Eklof B, Perrin M, Delis KT, Rutherford RB, Gloviczki P. Updated
terminology of chronic venous disorders: the VEIN-TERM
transatlantic interdisciplinary consensus document. J Vasc Surg.
[Practice Guideline]. 2009;49(2):498-501.
2. Raju S, Neglen P. Clinical practice. Chronic venous insufficiency
and varicose veins. N Engl J Med. [Review]. 2009;360(22):23192327.
3. Neglen P, Egger JF, 3rd, Olivier J, Raju S. Hemodynamic and
clinical impact of ultrasound-derived venous reflux parameters.
J Vasc Surg. [Comparative Study]. 2004;40(2):303-310.
4. Raju S, Darcey R, Neglen P. Unexpected major role for venous
stenting in deep reflux disease. J Vasc Surg. 2010;51(2):401-408;
discussion 8.
5. Neglen P, Raju S. Intravascular ultrasound scan evaluation of
the obstructed vein. J Vasc Surg. 2002;35(4):694-700.
6. Labropoulos N, Mansour MA, Kang SS, Gloviczki P, Baker WH.
New insights into perforator vein incompetence. Eur J Vasc
Endovasc Surg. 1999;18(3):228-234.
7. Mayberry JC, Moneta GL, DeFrang RD, Porter JM. The influence
of elastic compression stockings on deep venous hemodynamics. J Vasc Surg. [Comparative Study Research Support, Non-U.S.
Govt]. 1991;13(1):91-99; discussion 9-100.
8. Sarin S, Scurr JH, Coleridge Smith PD. Mechanism of action
of external compression on venous function. Br J Surg. 1992;
79(6):499-502.
9. Raju S, Hollis K, Neglen P. Use of compression stockings in chronic
venous disease: patient compliance and efficacy. Ann Vasc Surg.
2007;21(6):790-795.
10. Franks PJ, Oldroyd MI, Dickson D, Sharp EJ, Moffatt CJ. Risk
factors for leg ulcer recurrence: a randomized trial of two types
of compression stocking. Age Ageing. [Clinical Trial Comparative Study Randomized Controlled Trial Research Support, NonU.S. Govt]. 1995;24(6):490-494.
11. Mayberry JC, Moneta GL, Taylor LM, Jr., Porter JM. Fifteen-year
results of ambulatory compression therapy for chronic venous
ulcers. Surgery. [Research Support, Non-U.S. Govt]. 1991;109(5):
575-581.
12. Gohel MS, Barwell JR, Taylor M, et al. Long term results of compression therapy alone versus compression plus surgery in chronic
venous ulceration (ESCHAR): randomised controlled trial. BMJ.
[Comparative Study Multicenter Study Randomized Controlled
Trial Research Support, Non-U.S. Govt]. 2007;335(7610):83.
13. Kalra M, Gloviczki P. Surgical treatment of venous ulcers: role of
subfascial endoscopic perforator vein ligation. Surg Clin North
Am. [Review]. 2003;83(3):671-705.
14. ODonnell TF, Jr. The present status of surgery of the superficial venous system in the management of venous ulcer and the

5/22/2012 6:07:22 PM

774

Surgery: Evidence-Based Practice

evidence for the role of perforator interruption. J Vasc Surg.


[Review]. 2008;48(4):1044-1052.
15. Raju S, Neglen P. High prevalence of nonthrombotic iliac vein
lesions in chronic venous disease: a permissive role in pathogenicity. J Vasc Surg. 2006;44(1):136-143; discussion 44.
16. Neglen P, Hollis KC, Olivier J, Raju S. Stenting of the venous
outflow in chronic venous disease: long-term stent-related

PMPH_CH97.indd 774

outcome, clinical, and hemodynamic result. J Vasc Surg. 2007;


46(5):979-990.
17. Raju S, Hardy JD. Technical options in venous valve reconstruction. Am J Surg. 1997;173(4):301-307.
18. Raju S, Neglen P, Doolittle J, Meydrech EF. Axillary vein transfer
in trabeculated postthrombotic veins. J Vasc Surg. 1999;29(6):10501062; discussion 62-64.

5/22/2012 6:07:22 PM

CHAPTER 98

Deep Venous Thrombosis


Mark L. Ryan, Chad M. Thorson, Thai Vu,
Christian A. Otero, and Enrique Ginzburg

INTRODUCTION

SCREENING/DIAGNOSIS

Deep venous thrombosis (DVT) is a significant public health


problem in the United States, with an annual incidence of 350,000
cases per year, and approximately 100,000 deaths due to DVT/
pulmonary embolism (PE) each year.1 It is estimated that the
incidence could be as high as 600,000 cases per year due to undiagnosed asymptomatic, or silent, DVT.2 Although the overall
annual incidence of venous thromboembolism (VTE) is estimated to be 0.48 per 1000,3 surgical and trauma patients especially are at increased risk of thrombosis. Prior to the institution
of routine thromboprophylaxis, DVT rates were approximately
15% to 30% in general surgery patients and in excess of 50% in
trauma patients.4 Pharmacological prophylaxis has been shown to
decrease these risks by 60% to 70%.5 Current estimates of DVT
rates vary significantly (0.3650%) depending on whether or not
an institution employs an aggressive screening policy.6
In 1856, the German physician Rudolf Virchow postulated
that the pathogenesis of venous thrombosis was due to abnormalities in the endothelium, disruptions in blood flow, and alterations of the platelets as well as the coagulation and fibrinolytic
pathways.7 His theory has endured for the last several hundred
years and is now referred to as Virchows triad. Although opinions regarding the etiology of DVTs have remained constant,
those concerning the detection, prevention, and treatment of
the disease have continued to evolve. With advances in imaging technology, we are able to discern increasingly smaller clots
and occlusions, the clinical relevance of which is unknown. Prophylactic medications such as the low molecular weight heparins (LMWHs) have provided a method by which to reduce the
incidence of DVT, and therefore PE. However, the majority of
symptomatic VTE associated with hospital admissions occur
after hospital discharge;8 the duration of prophylaxis is therefore uncertain. As DVT falls under the umbrella of reasonably
preventable events as defined by the Centers for Medicare and
Medicaid Services,9 it is imperative that institutions implement
screening, prophylaxis, and treatment protocols based on the
best available evidence.

1. Who should be screened for DVT? (Risk assessment)


All patients should be evaluated on admission with a thorough
history and physical for factors that place them at risk for VTE.
The American College of Chest Physicians (ACCP) guidelines
for VTE prophylaxis identify multiple acquired risk factors for
VTE, such as age, immobility, previous VTE, cancer, surgery,
trauma, spinal cord injury, obesity, central venous catheter
insertion, inflammatory bowel disease, nephritic syndrome,
recent pregnancy, estrogen therapy, and acute medical illness.
Forty percent of hospitalized patients have three or more of these
risk factors.4 Several risk assessment scales have been proposed
in order to provide customized prophylaxis based on a patients
composite risk estimate;10-13 however, there are currently no standardized guidelines regarding the routine screening of asymptomatic trauma or surgical patients, although multiple strategies
have been explored.
Routine duplex ultrasound (DUS) screening of high-risk
trauma patients has been found to be more cost-effective than
prophylactic vena cava fi lter (VCF) placement in the prevention
of PE in patients with a length of stay of less than 2 weeks. A fourfold increase in the use of DUS screening in trauma patients has
been shown to be associated with a ten-fold increase in DVT diagnosis when solely high-risk patients are targeted for screening.6
There is no evidence to show that DUS screening of asymptomatic,
low-risk patients is cost-effective or prevents PE. In a study of 160
patients, of which 58 were designated as low risk, the incidence
of DVTs in the low-risk group was 0%.14 Another retrospective
analysis of 2529 patients (280 high risk, 2249 low risk) found that
the incidence of DVT was 5% and 0.1% in the high- and low-risk
groups, respectively.15
There is a large amount of disparity in the use of DUS screening
in different institutions in the United States. A survey of institutions
reporting to the National Trauma Database (NTDB) found that
only 28% of responding centers had a written policy regarding DUS
screening of asymptomatic trauma patients, 42% of which made
screening a mandatory policy.9 Currently there are no definitive
775

PMPH_CH98.indd 775

5/22/2012 6:07:55 PM

776

Surgery: Evidence-Based Practice

guidelines available regarding DUS screening. The Eastern Association for the Surgery of Trauma practice management guidelines
for the prevention of VTE in trauma patients recommend the use
of DUS for the diagnosis of DVT in symptomatic trauma patients
(Level 1A evidence). However, it goes on to state that serial DUS
screening may be a cost-effective way to decrease PE, but the sensitivity is relatively low compared to venography.16 The ACCP guidelines
recommend against routine DUS screening for asymptomatic DVT
unless patients are high risk (defined as spinal cord injury, lower
extremity or pelvic fracture, or major head injury) and have received
suboptimal thromboprophylaxis or no thromboprophylaxis.4
Answer: Routine screening of all asymptomatic surgery or
trauma patients is not justified by the evidence. DUS screening
of high-risk trauma patients is recommended (Grade 1C recommendation). There is insufficient evidence to recommend routine
screening of high-risk surgical patients.
2. What is the optimal diagnostic test for DVT?
DVT is frequently asymptomatic and clinical symptoms are often
unreliable in its diagnosis. Fortunately, there are several laboratory and imaging methods that are useful for the assessment of
the symptomatic patient.
D-dimers are degradation products of the fibrinolytic breakdown of a thrombus by plasmin. Depending on the type of assay
used, the test is very sensitive for the presence of thrombosis. This
test alone is insufficient due to lack of specificity; increases in
D-dimer are also seen in infection, inflammation, cancer, surgery,
trauma, burns, ischemic heart disease, stroke, peripheral vascular
disease, ruptured aneurysm, aortic dissection, or pregnancy.17 It is
therefore of minimal use in the surgical and trauma populations.
Contrast venography has been considered as the gold standard
for the diagnosis of DVT due to its high sensitivity and availability
of hard copies, which facilitates its use in clinical trials.18 However,
it is no longer routinely employed in the clinical setting due to
high costs, patient discomfort, moderate interobserver variability, incomplete or nondiagnostic rates of 20% to 40%, questionable relevance of small or distal thrombi, endothelial toxicity, and
limited availability.17,19 Also, since it is relatively invasive it is not
amenable to repeated testing.
Doppler ultrasound has become widely accepted as the primary diagnostic procedure in the evaluation of symptomatic
DVT. It is accurate for symptomatic DVT; it is widely available,
noninvasive, and repeatable.19 The sensitivity of the test is lower
in asymptomatic patients due to differences in DVT distribution
between symptomatic and asymptomatic patients. Two-thirds
of DVT are confined to the calf veins in asymptomatic patients,
whereas this is only the case in 15% of those who are symptomatic. Also, proximal DVT is often less extensive in asymptomatic
patients.20 With advances in imaging technology and standardization of DUS technique, the sensitivity of the test is improving.
Combined CT venography and CT pulmonary angiography
(CTVPA) was initially described in 1998 as a method to image
the deep venous system in patients being evaluated for a pulmonary embolus.21 It addresses several shortcomings of DUS, such
as operator independence and detailed imaging of areas that are
often technically inaccessible, such as above the inguinal ligament, below the knee, and in the adductor canal.22 However, since
this modality requires the patient to receive ionizing radiation
and contrast medium, its use is limited to an adjunct in the evaluation of PE.

PMPH_CH98.indd 776

Answer: Doppler ultrasound is both sensitive and specific in


the diagnosis of DVT in symptomatic patients (Grade 1A). Since
it is readily repeatable, inexpensive, and noninvasive, it is the preferred diagnostic modality.

COMPLICATIONS
3. What are the risks associated with asymptomatic DVT?
It was initially assumed that the majority of asymptomatic DVT
would spontaneously resolve without clinical consequence. However, a recent population-based cohort study found that among
516 patients who developed symptomatic VTE in association
with prior hospitalization, 67% of these events occurred during
the fi rst month after discharge from hospital, thereby suggesting
that many thrombi that are silent during hospitalization are destined to become clinically overt.23 Multiple clinical studies and
reviews of autopsy fi ndings have found that asymptomatic DVT
is of equal clinical importance to that of symptomatic DVT.24,25
The prevalence of asymptomatic DVT varies according to the
patient population being studied. Surgical intensive care unit
(SICU) patients are at particularly high risk. Harris et al. advocated for routine screening of SICU patients with high Apache II
scores (>12), who had undergone emergent procedures, and were
older than 65.26 Nonambulatory neurosurgical patients are also
vulnerable; approximately 23% will have a DVT despite mechanical and pharmacological prophylaxis.27
A challenging issue has been to determine the course and risk
of asymptomatic DVT, the most feared consequence of which is
pulmonary embolus leading to sudden death. Approximately half
of all DVT episodes produce few, if any, symptoms.28 Both symptomatic and asymptomatic proximal DVTs are closely correlated
with the risk of PE.29 Kearon showed that a majority of patients
presenting with symptomatic PE had asymptomatic DVT.30
Vaitkus et al. conducted a post hoc analysis of the patients
enrolled in the Prospective Evaluation of Dalteparin Efficacy for
Prevention of VTE in Immobilized Patients Trial (PREVENT),
looking at mortality rates in patients who were found to have
asymptomatic DVT. Of the 1738 patients who were asymptomatic on day 2, DUS screening for DVT identified 1540 patients free
of DVT, 118 with asymptomatic distal DVT, and 80 with asymptomatic proximal DVT yielding three distinct study groups. At
day 90, mortality was 1.9% in the group without DVT, 3.4% in
the group with distal DVT, and 13.8% in the group with proximal
DVT. The hazard ratio for death in the asymptomatic proximal
DVT group was 7.6% (95% CI: 3.815.3; P < 0.0001), even after
adjusting for differences in baseline demographics and clinical
variables.31 The mortality rate in the asymptomatic proximal DVT
group in this study (13.8%) was comparable to the 12.5% mortality rate observed in high-risk patients with symptomatic proximal
DVT in the PREPIC study,32 and the 17.4% mortality rate in a large
study of patients with PE.33 This further serves to reinforce the
clinical relevance of asymptomatic proximal DVT.
Answer: The incidence of symptomatic PE in the presence
of asymptomatic DVT and the high mortality rate associated in
patients with asymptomatic proximal DVT underscores the clinical relevance of recognizing the presence of this condition. Perhaps the strongest conclusion further supporting this comes from
the fact that the 8th American College of Chest Physicians (ACCP)
consensus recommendations (Grade 1A) do not distinguish

5/22/2012 6:07:55 PM

Deep Venous Thrombosis

between proximal and isolated distal DVT nor between symptomatic and asymptomatic events and recommend the same anticoagulant treatment for all these events.34
4. What are the complications of DVT and how often do they
convert to PE?
The most concerning complication of DVT is embolization of the
clot into the pulmonary arterial circulation. With early treatment,
patients with DVT can reduce their chances of developing a lifethreatening PE to less than 1%. One in every 100 patients who
develop DVT die from a PE, resulting in up to 200,000 deaths
annually. However, postthrombotic syndrome is also a significant
source of morbidity in these patients.

Symptomatic PE
It is estimated that 10% of symptomatic PE cause death within
1 hour of onset.35 Those patients with PE who do not die, acutely
often have nonspecific symptoms. For this reason, the diagnosis
of PE is often delayed or missed entirely. Consequently, most fatal
episodes of PE that occur in the hospital or the community are
diagnosed on autopsy. The highest risk period for postoperative
fatal PE appears to be 3 to 7 days after surgery.36
Barritt and Jordan37 found that 26% (5 of 19) of untreated
patients with clinically diagnosed PE (severe end of the spectrum) died of PE during a follow-up period of 2 weeks, and
another 26% of patients experienced nonfatal recurrences. In the
Prospective Investigation of PE Diagnosis (PIOPED) study, 10%
(2 of 20) of patients with PE in whom the diagnosis was missed
(less severe end of the spectrum) and consequently were not
treated with anticoagulants were judged to have had a recurrence
during 3 months of follow-up.38 It has been suggested that PE may
not always originate in the proximal deep veins of the lower and
upper extremities, as commonly believed, but instead may occur
de novo in the lungs.39
It is difficult to estimate the proportion of patients with
symptomatic proximal DVT who would progress to symptomatic PE if left untreated. Hull et al.40 studied patients with proximal DVT who were treated with 10 days of intravenous heparin
(adequate initial therapy) followed by 3 months of low-dose subcutaneous heparin (inadequate therapy); 47% (9 of 19) developed
recurrent VTE during this period.40 Of these 9 cases, 6 were
symptomatic and 1 was a PE. In contrast to this study, Nielsen
et al. repeated venography after 30 days in 30 fully ambulant
patients with symptomatic DVT (three quarters proximal)
treated with phenylbutazone alone.41 Progressive thrombosis in
the proximal veins was found in 27% of patients, with additional
patients having progression in the calf veins. Ventilation-perfusion lung scanning showed progression in 8% (3 of 39) after 10
days and 3% (1 of 30) at 60 days. During 3 months of follow-up,
one patient had a confi rmed episode of PE and eight patients
were suspected of having recurrent DVT (no objective testing
performed).41 These two studies, together with the high frequency of asymptomatic PE in patients with proximal DVT and
the high prevalence of recurrent PE in untreated PE patients, 38
suggest that approximately 50% of patients with untreated proximal DVT will develop symptomatic PE within 3 months. Th is
risk appears highest at the time of acute DVT, with a subsequent
rapid decline over a 3-month period.42

PMPH_CH98.indd 777

777

It is now evident that when anticoagulant therapy is stopped,


the risk of recurrence is much lower (3% per year) when VTE is
associated with a risk factor that has resolved than in patients with
idiopathic thrombosis or a persistent risk factor (approximately
10% per year, or more).43,44 Similarly, the risk of recurrent or progressive VTE in untreated patients is also expected to be greater in
those with idiopathic thrombosis or continuing risk factors such
as cancer compared with those in whom the initiating risk factor
rapidly resolves.

Postthrombotic Syndrome
Th rombosis damages the deep venous valves, which promote
venous return during contraction of leg muscles. This results in
venous reflux and venous hypertension in the lower limbs. Valvular incompetence may also occur in venous segments not involved
in the initial DVT. This type of reflux has a distinctive anatomic
distribution and is more likely to be temporary. However, venous
reflux associated with thrombosis and residual venous obstruction
are largely responsible for the development of postthrombotic syndrome, which is characterized by pain, heaviness, and swelling of
the leg aggravated by standing or walking. In its more severe form,
the postthrombotic syndrome results in skin and subcutaneous tissue changes that include varicose eczema, subcutaneous atrophy
(lipodermatosclerosis), hyperpigmentation, and chronic skin ulceration. Initially considered an unusual, long-term sequela, it actually occurs frequently in as many as 60% to 70% of people and can
develop within 2 months of DVT. Although its pathophysiological
sequence is generally accepted, there is a poor correlation between
the severity of the postthrombotic syndrome and either the extent
of previous DVT or associated hemodynamic changes.45,46
Prandoni and associates, in a prospective study of 355 consecutive patients with symptomatic DVT, all of whom were instructed
to wear graduated compression stockings for 2 years, observed a
cumulative incidence of classic postthrombotic syndrome of 17%
after 1 year, 23% after 2 years, 28% after 5 years, and 29% after 8
years of follow-up. The cumulative incidence of severe postthrombotic syndrome was 3% after 1 year and 9% after 5 years.47 Recurrent ipsilateral DVT during follow-up was associated with a six-fold
increase in the risk of developing postthrombotic syndrome.13
During long-term follow-up, symptoms of postthrombotic syndrome resolved in over half the affected patients, regardless of the
severity of initial symptoms.48
A separate syndrome of venous claudication, in which patients with previous extensive iliofemoral thrombosis develop a
bursting leg pain during exercise, has also been described but
is uncommon.49 This is thought to occur secondary to venous
hypertension caused by residual iliofemoral venous obstruction.50
Outcome is generally dependent on the rate and adequacy of collateral development.
Ginsberg and associates, in a study of 255 patients, examined
the association between asymptomatic DVT after hip or knee
arthroplasty treated for 6 to 12 weeks, and the subsequent risk
of the postthrombotic syndrome. After an average of 5 years, the
prevalence of the postthrombotic syndrome (moderate or severe
symptoms with venous reflux) was low and the same (~5%) in
patients who had isolated calf DVT (n = 66), proximal DVT (n = 25),
or no DVT (n = 164).46
Answer: The two main complications of DVT are PE and
the postthrombotic syndrome. Without proper treatment of an

5/22/2012 6:07:55 PM

778

Surgery: Evidence-Based Practice

initial DVT, approximately 50% can be expected to convert to a


PE (Level 2B evidence).

PROPHYLAXIS
5. How long should thromboprophylaxis be continued postoperatively?
Recommendations regarding the duration of DVT prophylaxis
vary according to the patients reason for admission to the hospital. For all patients undergoing major general surgery, the risk of
postoperative DVT is highest within the fi rst 2 weeks, although
VTE complications may occur later.4 There have been three clinical trials in general surgery patients, which have examined the
effects of extended prophylaxis on DVT. The fi rst, a study of 322
patients who had undergone major abdominal or pelvic surgery
for cancer, found that patients receiving enoxaparin for 28 days
rather than 9 days had a significant decrease in the incidence
of DVTs (12% vs. 5%, OR 0.36).51 Another study involving 427
patients who had undergone major abdominal surgery found
that patients receiving dalteparin for 4 weeks had a significant
reduction in DVT rates as well (16.3% vs. 7.3%, P = .012).52 A
third study of 118 patients undergoing major elective abdominal or noncardiac thoracic operations was unable to demonstrate a significant benefit to receiving tinzaparin for 4 weeks
instead of 1 week.53 A Cochrane review integrating the results
of these studies with that of previously unpublished data found
that extended prophylaxis resulted in a significant reduction in
the incidence of DVT in patients receiving prolonged administration of DVT prophlaxis (14.3% vs. 6.1%, OR 0.43) without an
increase in bleeding complications. Th is meta-analysis did not
demonstrate a reduction in PE or mortality, although the studies
included were not powered to do so.54 American College of Chest
Physicians guidelines currently recommend extended prophylaxis only in the event of major cancer surgery of the abdomen or
pelvis, since extended treatment with LMWH was not found to
be cost-effective in a thorough economic analysis conducted by
the National Institute for Health and Clinical Excellence.55
The duration of thromboprophylaxis in trauma patients has
yet to be clearly defined. Extended administration of LMWH has
been shown to decrease the time to hospital discharge in trauma
patients requiring warfarin for prolonged anticoagulation who
are not yet therapeutic.56 Current recommendations state that all
trauma patients should be placed on prophylaxis for the duration
of their hospitalization.4,16 Patients with prolonged immobility
undergoing rehabilitation should receive extended prophylaxis
with LMWH or warfarin.4
Patients with orthopedic injuries or procedures are at especially high risk for thrombotic complications throughout hospitalization and after discharge. In a study of 24,000 patients who
had undergone total hip replacement (THR), 76% of the thrombotic events were diagnosed after hospital discharge (mean time
to diagnosis: 17 days).8 These patients are consistently hypercoagulable after discharge and therefore require extended prophylaxis
as well. Patients who have undergone a THR, total knee replacement (TKR), or hip fracture surgery (HFS) should receive a minimum of 10 days of thromboprophylaxis.4 Multiple randomized
controlled trials have shown that extended prophylaxis in these
patients using fondaparinux, LMWH, or warfarin results in lower
rates of DVT and symptomatic VTE.57-59

PMPH_CH98.indd 778

Acute spinal cord injury (SCI) also places a patient at extremely high risk for VTE. All patients admitted for SCI should
receive routine thromboprophylaxis once primary hemostasis has
been obtained. Patients are also at risk during the rehabilitation
period; about 10% of SCI patients in rehab develop a symptomatic
DVT and 3% become symptomatic from a PE.60 Patients should be
maintained on LMWH or warfarin for 3 months following injury
or until completion of inpatient rehabilitation.4
Answer: Moderate high-risk general surgery patients should
be maintained on thromboprophylaxis for the duration of their
hospitalization (Grade 1A), which may be extended up to 28
days in extremely high-risk patients (Grade 2A). Trauma patients
should also receive mechanical and pharmacological prophylaxis
until discharge (Grade 1C). This should be extended if the patient
has continued immobility or requires rehabilitation (Grade 2C).
Patients who have undergone THR, TKR, or HFS should receive
10 to 35 days of thromboprophylaxis (Grade 1A), whereas those
with acute SCI should be maintained for 3 months or for the duration of inpatient rehabilitation (Grade 1C).

TREATMENT
6. What are the proper treatment protocols for initial and
recurrent DVT?
Anticoagulation is the mainstay of therapy for the treatment of
acute DVT. The goal of treatment is to prevent thrombus extension and recurrence of VTE. Treatment should be initiated as
soon as the diagnosis is confirmed by objective testing, though it
can be initiated immediately in patients with a high clinical suspicion. The conventional treatment has been continuous IV infusion of unfractionated heparin (UFH), with adjustments in dose
to keep activated partial thromboplastin time (APTT) > 1.5 times
normal.61 Multiple randomized control trials have compared
LMWHs to continuous UFH.62-64 The most recent meta-analysis
showed that LMWH was associated with fewer thrombotic complications, less major bleeding, and fewer deaths when compared
to UFH.65 An added benefit of LMWH is the potential to be safely
administered at home in select patients.66,67 Fondaparinux, a
synthetic inhibitor of factor Xa given once daily, has also been
shown to be equivalent to LMWH.68 Regardless of the drug chosen for treatment, the duration of initial heparin therapy should
be continued for at least 5 days.69 Concomitant administration of
vitamin-K antagonists (VKA) should begin on the first or second
day of treatment, and heparin therapy should continue until the
International Normalised Ratio is stable between 2.0 to 3.0.
Patients with acute DVTs require long-term anticoagulation
to prevent symptomatic extension or recurrence. Evidence-based
clinical practice guidelines from the ACCP provide extensive recommendations for treatment of venous thromboembolic disease, with Level 1 evidence existing for all treatment
recommendations.70,71 Patients with incidentally discovered DVTs
should be treated the same way as comparable patients with
symptomatic DVTs. The duration of long-term anticoagulation is
based on risk-stratification for extension and recurrence. For an
initial episode of DVT due to a transient event (surgery, trauma),
treatment with VKA should be continued for 3 months. For the
first episode of DVT in the absence of a known identifiable risk
factor, treatment with VKA should be extended for 3 to 6 months.
If the patient has an underlying thrombophilia, treatment for 6 to

5/22/2012 6:07:55 PM

Deep Venous Thrombosis

12 months is recommended. For patients with DVT and cancer,


LMWH should be continued for the first 3 to 6 months of treatment, followed by VKA indefinitely or until the cancer resolves.
Indefinite treatment should be carried out for all patients with
recurrent thrombosis (two or more objectively diagnosed DVTs)
and should be considered in patients with idiopathic DVTs or
underlying thrombophilia. In patients who receive indefinite
anticoagulant treatment, the riskbenefit of continuing treatment
should be reassessed at regular intervals to maximize patient
safety and decrease recurrence risk.
When patients with deep vein thrombosis have contraindications to, or complications from, anticoagulation treatment,
inferior vena cava (IVC) fi lters are indicated.71 In addition, IVC
fi lters are indicated in patients who have recurrent DVTs despite
adequate anticoagulation. To date, there are no randomized trials or prospective studies evaluating IVC fi lters as sole therapy
for DVT. In patients who receive fi lters, a conventional course of
anticoagulant therapy should be given if their risk of bleeding
resolves.

779

Removal of thrombus in patients with DVT has the potential


to reduce symptoms from clot burden and postthrombotic syndrome, though only Level 2 evidence exists.71 In select patients with
extensive acute DVT (iliofemoral DVT, symptoms for < 14 days,
life expectancy > 1 year) with a low risk of bleeding, catheterdirected thrombolysis (CDT) may reduce symptoms and postthrombotic morbidity. Patients receiving CDT should receive
pharmacologic thromboprophylaxis concurrently. For patients at
high risk of bleeding, operative venous thrombectomy can be used
as an alternative. In patients who receive either CDT or operative
thrombectomy, the same intensity and duration of anticoagulation therapy is recommended as for comparable patients who do
not undergo CDT.
Answer: The initial treatment of acute DVTs can be safely
accomplished with continuous infusion of UFH, subcutaneous
LMWH, or subcutaneous fondaparinux for at least 5 days. This treatment should be accompanied by VKA administration for at least 3
months and longer based on the clinical picture. Routine use of IVC
filters, CDT, or operative thrombectomy is not recommended.

Clinical Question Summary


Question

Answer

Grade of
Recommendation

References

1 Who should be screened


for DVT?

High-risk trauma patients. There is insufficient evidence to


recommend routine screening of general surgery patients.

4, 14, 15

2 What is the optimal


diagnostic test for DVT?

Doppler ultrasound is sensitive and specific for the diagnosis


of DVT in a symptomatic patient.

4, 10, 19

3 What are the risks


associated with
asymptomatic DVT?

The risk of PE is equivalent regardless of whether a DVT is


symptomatic or asymptomatic.

31, 32

4 What are the


complications of DVT,
and how often do they
convert to PE?

Symptoms of postthrombotic syndrome include swelling,


discoloration, sclerosis, and ulceration. Untreated DVT
can be expected to progress to PE in 50% of patients.

40, 45, 46

5 How long should


thromboprophylaxis
be continued
postoperatively?

Prophylaxis should be continued until patient discharge.


Prophylaxis may be extended 28 to 35 days in extremely
high-risk patients or those who require rehabilitation.
Patients with acute spinal cord injury should receive
3 months of LMWH.

51-54, 57-60

6 What are the proper


treatment protocols
for initial and recurrent
DVT?

The initial treatment of acute DVTs can be safely


accomplished with continuous infusion of UFH, LMWH,
or fondaparinux for at least 5 days. This treatment should
be accompanied by VKA administration for at least
3 months.

62-64, 70, 71

Evidence-Based Table
Study

Design

Intervention

Description

Results

Prandoni
1992
[47]

RCT

LMWH (fixed weight-based


dose) administered SQ BID
versus continuous infusion
of UFH for treatment of
symptomatic proximal
DVT.

170 patients enrolled. 85


randomized to UFH
and 85 to LMWH.

No difference in symptomatic
extension (14% UFH, 7%
LMWH), bleeding (3.5% UFH,
1% LMWH), or mortality (3.5%
in each group).
(Continued)

PMPH_CH98.indd 779

5/22/2012 6:07:55 PM

780

Surgery: Evidence-Based Practice

(Continued)
Study

Design

Intervention

Description

Results

Hull 1992
[63]

RCTplacebo
double
blind
(Multicenter)

LMWH (fixed weight-based


dose) administered SQ
daily versus continuous
infusion of UFH for
treatment of symptomatic
proximal DVT.

432 patients enrolled. 219


received UFH and SQ
placebo, 213 received
LMWH and continuous
placebo.

No difference in recurrence
(6.9% UFH, 2.8% LMWH) or
extension to PE (2.7% vs. 1.4%).
Significant reduced bleeding
(5% UFH, 0.5% LMWH) and
mortality (9.6% vs. 4.7%).

Levine 1996
[66]

RCT

LMWH (fixed weight-based


dose) administered SQ BID
at home versus inpatient
treatment with continuous
UFH; symptomatic
proximal DVT.

500 patients enrolled.


253 received UFH, 246
received LMWH.

No difference in symptomatic
recurrent DVT (6.7% UFH,
5.3% LMWH) or bleeding
(1.2% vs. 2%).

Buller [68]

RCT

Fondaparinux (factor Xa
inhibitor) given daily SQ
versus SQ LMWH BID
in patients with
symptomatic DVT.

2205 patients enrolled.


1101 received LMWH,
1091 received
fondaparinux.

No difference in recurrence (4.1%


LMWH, 3.9% fondaparinux),
bleeding (1.2% vs. 1.1%), or
mortality (3.0% vs. 3.8%).

Baursachs
2010
[72]

RCT

Rivaroxaban (oral factor Xa


inhibitor) BID for 3 weeks
then daily versus SQ
LMWH followed by VKA
in patients with
symptomatic DVT.

3449 patients enrolled.


1718 received
LMWH+VKA, 1731
received rivaroxaban.

Rivaroxaban had similar


recurrence (3% LMWH,
2.1% rivaroxaban) and similar
bleeding (8.1% in each group).

Bergqvist
et al.
2002
[51]

RCTplacebo
controlled
double blind

All patients received


enoxaparin 6-10 days,
then given enoxaparin or
placebo for an additional
21 days.

332 cancer surgery


patients; 167 placebo,
165 enoxaparin.

Increased rate of DVT in placebo


group (12% vs. 4.8%).

Rasmussen
et al.
2006
[52]

RCTopen
label

Dalteparin 7 days vs.


dalteparin 28 days.

427 major surgery


patients; 225 for shortterm prophylaxis,
205 for prolonged
prophylaxis.

Increased rate of VTE in


short-term group (16.3% vs.
7.3%). No increase in bleeding
events.

Lausen
et al.
1998
[53]

RCTopen
label

Tinzaparin 7 days vs.


Tinzaparin 28 days.

118 major surgery


patients; 60 for shortterm prophylaxis,
58 for prolonged
prophylaxis.

Higher DVT rate in short-term


group (10% vs. 5%), but study
underpowered to detect a
difference with adequate power.

REFERENCES
1. US Department of Health and Human Services. The Surgeon
Generals call to action to prevent deep venous thrombosis and
pulmonary embolism. 2008. Available at http://www.surgeon
general.gov/topics/deepvein/on. Accessed February 15, 2009.
2. Anderson FA, Jr., Wheeler HB, Goldberg RJ, et al. A populationbased perspective of the hospital incidence and case-fatality rates
of deep vein thrombosis and pulmonary embolism. The Worcester DVT Study. Arch Intern Med. 1991;151(5):933-938.
3. Silverstein MD, Heit JA, Mohr DN, et al. Trends in the incidence
of deep vein thrombosis and pulmonary embolism: a 25-year
population-based study. Arch Intern Med. 1998;158(6):585-593.
4. Geerts WH, Bergqvist D, Pineo GF, et al. Prevention of venous
thromboembolism: American College of Chest Physicians
Evidence-Based Clinical Practice Guidelines (8th Edition). Chest.
2008;133(6 Suppl):381S-453S.

PMPH_CH98.indd 780

5. Meissner MH. The effectiveness of deep vein thrombosis prevention. J Vasc Surg. 2010;52(5 Suppl):65S-67S.
6. Haut ER, Noll K, Efron DT, et al. Can increased incidence of deep
vein thrombosis (DVT) be used as a marker of quality of care in
the absence of standardized screening? The potential effect of
surveillance bias on reported DVT rates after trauma. J Trauma.
2007;63(5):1132-1135; discussion 5-7.
7. Lippi G, Franchini M. Pathogenesis of venous thromboembolism: when the cup runneth over. Semin Thromb Hemost.
2008;34(8):747-761.
8. White RH, Romano PS, Zhou H, et al. Incidence and time course
of thromboembolic outcomes following total hip or knee arthroplasty. Arch Intern Med. 1998;158(14):1525-1531.
9. Haut ER, Schneider EB, Patel A, et al. Duplex ultrasound screening for deep vein thrombosis in asymptomatic trauma patients: a
survey of individual trauma surgeon opinions and current trauma
center practices. J Trauma. 2011;70(1):27-33; discussion 4.

5/22/2012 6:07:55 PM

Deep Venous Thrombosis

10. Geerts WH, Heit JA, Clagett GP, et al. Prevention of venous
thromboembolism. Chest. 2001;119(1 Suppl):132S-175S.
11. Greenfield LJ, Proctor MC, Rodriguez JL, et al. Posttrauma
thromboembolism prophylaxis. J Trauma. 1997;42(1):100-103.
12. Gearhart MM, Luchette FA, Proctor MC, et al. The risk assessment profi le score identifies trauma patients at risk for deep vein
thrombosis. Surgery. 2000;128(4):631-640.
13. Kucher N, Koo S, Quiroz R, et al. Electronic alerts to prevent
venous thromboembolism among hospitalized patients. N Engl J
Med. 2005;352(10):969-977.
14. Cipolle MD, Wojcik R, Seislove E, et al. The role of surveillance
duplex scanning in preventing venous thromboembolism in
trauma patients. J Trauma. 2002;52(3):453-462.
15. Spain DA, Richardson JD, Polk HC, Jr., et al. Venous thromboembolism in the high-risk trauma patient: do risks justify
aggressive screening and prophylaxis? J Trauma. 1997;42(3):463467; discussion 7-9.
16. Rogers FB, Cipolle MD, Velmahos G, et al. Practice management guidelines for the prevention of venous thromboembolism
in trauma patients: the EAST practice management guidelines
work group. J Trauma. 2002;53(1):142-164.
17. Tan M, van Rooden CJ, Westerbeek RE, et al. Diagnostic management of clinically suspected acute deep vein thrombosis. Br J
Haematol. 2009;146(4):347-360.
18. Rabinov K, Paulin S. Roentgen diagnosis of venous thrombosis
in the leg. Arch Surg. 1972;104(2):134-144.
19. Geerts WH, Pineo GF, Heit JA, et al. Prevention of venous
thromboembolism: the Seventh ACCP Conference on Antithrombotic and Thrombolytic Therapy. Chest. 2004;126(3 Suppl):
338S-400S.
20. Kearon C. Noninvasive diagnosis of deep vein thrombosis in
postoperative patients. Semin Thromb Hemost. 2001;27(1):3-8.
21. Loud PA, Grossman ZD, Klippenstein DL, et al. Combined CT
venography and pulmonary angiography: a new diagnostic technique for suspected thromboembolic disease. AJR Am J Roentgenol. 1998;170(4):951-954.
22. Katz DS, Loud PA, Bruce D, et al. Combined CT venography and
pulmonary angiography: a comprehensive review. Radiographics. 2002;22(Spec No):S3-19; discussion S20-24.
23. Spencer FA, Lessard D, Emery C, et al. Venous thromboembolism in the outpatient setting. Arch Intern Med. 2007;167(14):
1471-1475.
24. Ro A, Kageyama N, Tanifuji T, et al. Pulmonary thromboembolism: overview and update from medicolegal aspects. Leg Med
(Tokyo). 2008;10(2):57-71.
25. Venet C, Berger C, Tardy B, et al. [Prevention of venous thromboembolism in polytraumatized patients. Epidemiology and
importance]. Presse Med. 2000;29(2):68-75.
26. Harris LM, Curl GR, Booth FV, et al. Screening for asymptomatic deep vein thrombosis in surgical intensive care patients.
J Vasc Surg. 1997;26(5):764-769.
27. Dermody M, Alessi-Chinetti J, Iafrati MD, et al. The utility of
screening for deep venous thrombosis in asymptomatic, nonambulatory neurosurgical patients. J Vasc Surg. 2011.
28. Piazza G, Goldhaber SZ. Acute pulmonary embolism: part II:
treatment and prophylaxis. Circulation. 2006;114(3):e42-e47.
29. Moser KM, Fedullo PF, LitteJohn JK, et al. Frequent asymptomatic pulmonary embolism in patients with deep venous thrombosis. JAMA. 1994;271(3):223-225.
30. Kearon C. Natural history of venous thromboembolism. Circulation. 2003;107(23 Suppl 1):I22-I30.
31. Vaitkus PT, Leizorovicz A, Cohen AT, et al. Mortality rates and
risk factors for asymptomatic deep vein thrombosis in medical
patients. Thromb Haemost. 2005;93(1):76-79.

PMPH_CH98.indd 781

781

32. Decousus H, Leizorovicz A, Parent F, et al. A clinical trial of vena


caval fi lters in the prevention of pulmonary embolism in patients
with proximal deep-vein thrombosis. Prevention du Risque
dEmbolie Pulmonaire par Interruption Cave Study Group.
N Engl J Med. 1998;338(7):409-415.
33. Goldhaber SZ, Visani L, De Rosa M. Acute pulmonary embolism:
clinical outcomes in the International Cooperative Pulmonary
Embolism Registry (ICOPER). Lancet. 1999;353(9162):1386-1389.
34. Bounameaux H, Perrier A, Righini M. Diagnosis of venous
thromboembolism: an update. Vasc Med. 2010;15(5):399-406.
35. Stein PD, Henry JW. Prevalence of acute pulmonary embolism among patients in a general hospital and at autopsy. Chest.
1995;108(4):978-981.
36. Rasmussen MS, Wille-Jorgensen P, Jorgensen LN. Postoperative
fatal pulmonary embolism in a general surgical department. Am
J Surg. 1995;169(2):214-216.
37. Barritt DW, Jordan SC. Anticoagulant drugs in the treatment of
pulmonary embolism. A controlled trial. Lancet. 1960;1(7138):
1309-1312.
38. Stein PD, Henry JW, Relyea B. Untreated patients with pulmonary embolism. Outcome, clinical, and laboratory assessment.
Chest. 1995;107(4):931-935.
39. Velmahos GC, Spaniolas K, Tabbara M, et al. Pulmonary embolism and deep venous thrombosis in trauma: are they related?
Arch Surg. 2009;144(10):928-932.
40. Hull R, Delmore T, Genton E, et al. Warfarin sodium versus lowdose heparin in the long-term treatment of venous thrombosis.
N Engl J Med. 1979;301(16):855-858.
41. Nielsen HK, Husted SE, Krusell LR, et al. Anticoagulant therapy
in deep venous thrombosis. A randomized controlled study.
Thromb Res. 1994;73(3-4):215-226.
42. Coon WW, Willis PW, 3rd. Recurrence of venous thromboembolism. Surgery. 1973;73(6):823-827.
43. Schulman S, Rhedin AS, Lindmarker P, et al. A comparison of six
weeks with six months of oral anticoagulant therapy after a first
episode of venous thromboembolism. Duration of Anticoagulation Trial Study Group. N Engl J Med. 1995;332(25):1661-1665.
44. Kearon C, Gent M, Hirsh J, et al. A comparison of three months
of anticoagulation with extended anticoagulation for a fi rst
episode of idiopathic venous thromboembolism. N Engl J Med.
1999;340(12):901-907.
45. Heldal M, Seem E, Sandset PM, et al. Deep vein thrombosis: a
7-year follow-up study. J Intern Med. 1993;234(1):71-75.
46. Ginsberg JS, Turkstra F, Buller HR, et al. Postthrombotic syndrome after hip or knee arthroplasty: a cross-sectional study.
Arch Intern Med. 2000;160(5):669-672.
47. Prandoni P, Lensing AW, Cogo A, et al. The long-term clinical course
of acute deep venous thrombosis. Ann Intern Med. 1996;125(1):1-7.
48. Prandoni P, Lensing AW, Prins MH, et al. Which is the outcome of the post-thrombotic syndrome? Thromb Haemost. 1999;
82(4):1358.
49. Cockett FB, Thomas ML. The iliac compression syndrome. Br J
Surg. 1965;52(10):816-821.
50. Christenson JT, Al-Hassan HK, Shawa NJ. Subcutaneous and
intramuscular pressures in the post-phlebitic limb. Scand J Clin
Lab Invest. 1986;46(2):137-141.
51. Bergqvist D, Agnelli G, Cohen AT, et al. Duration of prophylaxis
against venous thromboembolism with enoxaparin after surgery
for cancer. N Engl J Med. 2002;346(13):975-980.
52. Rasmussen MS, Jorgensen LN, Wille-Jorgensen P, et al. Prolonged
prophylaxis with dalteparin to prevent late thromboembolic
complications in patients undergoing major abdominal surgery:
a multicenter randomized open-label study. J Thromb Haemost.
2006;4(11):2384-2390.

5/22/2012 6:07:55 PM

782

Surgery: Evidence-Based Practice

53. Lausen I, Jensen R, Jorgensen LN, et al. Incidence and prevention


of deep venous thrombosis occurring late after general surgery:
randomised controlled study of prolonged thromboprophylaxis.
Eur J Surg. 1998;164(9):657-663.
54. Rasmussen MS, Jorgensen LN, Wille-Jorgensen P. Prolonged
thromboprophylaxis with low molecular weight heparin for
abdominal or pelvic surgery. Cochrane Database Syst Rev. 2009;
(1):CD004318.
55. National Institute for Health and Clinical Excellence. Reducing
the risk of venous thromboembolism (deep vein thrombosis and
pulmonary embolism) in inpatients undergoing surgery. NICE
clinical guideline no. 46:1-160. Available at http://www.nice.org.
uk/CG046. Accessed February 15, 2011.
56. Bridges GG, Lee MD, Jenkins JK, et al. Expedited discharge
in trauma patients requiring anticoagulation for deep venous
thrombosis prophylaxis: the LEAP Program. J Trauma. 2003;
54(2):232-235.
57. Eriksson BI, Lassen MR. Duration of prophylaxis against venous
thromboembolism with fondaparinux after hip fracture surgery:
a multicenter, randomized, placebo-controlled, double-blind
study. Arch Intern Med. 2003;163(11):1337-1342.
58. Dahl OE, Andreassen G, Aspelin T, et al. Prolonged thromboprophylaxis following hip replacement surgeryresults of a doubleblind, prospective, randomised, placebo-controlled study with
dalteparin (Fragmin). Thromb Haemost. 1997;77(1):26-31.
59. Comp PC, Spiro TE, Friedman RJ, et al. Prolonged enoxaparin
therapy to prevent venous thromboembolism after primary hip
or knee replacement. Enoxaparin Clinical Trial Group. J Bone
Joint Surg Am. 2001;83-A(3):336-345.
60. Prevention of venous thromboembolism in the acute treatment
phase after spinal cord injury: a randomized, multicenter trial
comparing low-dose heparin plus intermittent pneumatic compression with enoxaparin. J Trauma. 2003;54(6):1116-1124; discussion 25-26.
61. Basu D, Gallus A, Hirsh J, et al. A prospective study of the value
of monitoring heparin treatment with the activated partial
thromboplastin time. N Engl J Med. 1972;287(7):324-327.
62. Prandoni P, Lensing AW, Buller HR, et al. Comparison of subcutaneous low-molecular-weight heparin with intravenous standard

PMPH_CH98.indd 782

63.

64.

65.

66.

67.

68.

69.

70.

71.

72.

heparin in proximal deep-vein thrombosis. Lancet. 1992;


339(8791):441-445.
Hull RD, Raskob GE, Pineo GF, et al. Subcutaneous lowmolecular-weight heparin compared with continuous intravenous heparin in the treatment of proximal-vein thrombosis.
N Engl J Med. 1992;326(15):975-982.
Low-molecular-weight heparin in the treatment of patients with
venous thromboembolism. The Columbus Investigators. N Engl
J Med. 1997;337(10):657-662.
van Dongen CJ, van den Belt AG, Prins MH, et al. Fixed dose subcutaneous low molecular weight heparins versus adjusted dose
unfractionated heparin for venous thromboembolism. Cochrane
Database Syst Rev. 2004;4:CD001100.
Levine M, Gent M, Hirsh J, et al. A comparison of low-molecularweight heparin administered primarily at home with unfractionated heparin administered in the hospital for proximal
deep-vein thrombosis. N Engl J Med. 1996;334(11):677-681.
Koopman MM, Prandoni P, Piovella F, et al. Treatment of venous
thrombosis with intravenous unfractionated heparin administered in the hospital as compared with subcutaneous lowmolecular-weight heparin administered at home. The Tasman
Study Group. N Engl J Med. 1996;334(11):682-687.
Buller HR, Davidson BL, Decousus H, et al. Fondaparinux or enoxaparin for the initial treatment of symptomatic deep venous thrombosis: a randomized trial. Ann Intern Med. 2004;140(11):867-873.
Hull RD, Raskob GE, Rosenbloom D, et al. Heparin for 5 days
as compared with 10 days in the initial treatment of proximal
venous thrombosis. N Engl J Med. 1990;322(18):1260-1264.
Buller HR, Agnelli G, Hull RD, et al. Antithrombotic therapy for
venous thromboembolic disease: the Seventh ACCP Conference
on Antithrombotic and Thrombolytic Therapy. Chest. 2004;126(3
Suppl):401S-428S.
Kearon C, Kahn SR, Agnelli G, et al. Antithrombotic therapy
for venous thromboembolic disease: American College of Chest
Physicians Evidence-Based Clinical Practice Guidelines (8th
Edition). Chest. 2008;133(6 Suppl):454S-545S.
Bauersachs R, Berkowitz SD, Brenner B, et al: Oral rivaroxaban
for symptomatic venous thromboembolism. N Engl J Med. 2010;
363(26):2499-510.

5/22/2012 6:07:55 PM

Commentary on
Deep Venous Thrombosis
M. M. Knudson

The chapter on Deep Venous Thrombosis (DVT) by Ginzburg


and colleagues nicely summarizes the current state of understanding of this potentially lethal postsurgical complication. In
addition to the morbidity and mortality associated with venous
thromboembolic events (VTEs), hospitals with high VTE rates
face the potential for loss of revenue, as the Agency for Healthcare
Research and Quality, the Joint Commission, and the Centers for
Medicare and Medicaid consider VTE a marker of quality of care.
Although VTE has been recognized for decades, our understanding of this disease is still incomplete and thus our approach to
prevention is largely empirical and often imperfect. As pointed
out by the authors, risk factors for VTE can be considered innate
(as in inherited protein C deficiency) or acquired (obesity, surgery,
trauma, cancer, etc.). Although most authors agree on risk factors, there is much less consistency when it comes to prevention
beyond the consensus document provided by the American College of Chest Physicians (ACCP). 1 It is in this background that
Ryan and colleagues attempt to answer the following questions:

fi xators. The authors mention the potential combined role of CT


venography and CT pulmonary angiography, and although this
might be of interest in a research protocol, the radiation and
contrast exposure will limit its routine use as a screen in clinical
practice.

1. Who should be screened for DVT?

4. What are the complications of DVT and how often does DVT
convert to PE?

3. What are the risks associated with asymptomatic DVT?


Here the water gets even muddier as we discover more and more
patients with asymptomatic VTE. Among trauma patients, the
reported incidence of PE has more than doubled in recent years,
presumably due to liberal use of chest CT in trauma centers for
diagnosis of injuries and to assess postinjury complications.4 Yet,
despite this increase in PE rates, the mortality associated with
posttraumatic PE has decreased dramatically, presumably due to
earlier detection and appropriate treatment. Thus, I agree with
the authors that asymptomatic clots should be treated in the same
manner as those that produce symptoms.

Screening with duplex ultrasound (DUS) performed by an experienced vascular technologist has been advocated by some authors
as a measure to detect silent DVT in high-risk patients, especially
in those where prophylaxis is considered inadequate or there has
been some delay in initiation of therapy.2 We have used DUS in
multiple studies as an endpoint in evaluating the efficacy of measures aimed at VTE prophylaxis. DUS has the advantage of being
noninvasive and repeatable as well as capable of detecting upper
extremity DVT (which would be missed by routine venography
and is an increasingly recognized source of pulmonary embolism
[PE]).3 However, as pointed out by the authors, hospitals that routinely screen with DUS will have much higher rates of DVT, that
is, a surveillance bias. The same may be said for centers where
chest CT is used liberally with the resultant increased detection
of silent pulmonary emboli: the more you look, the more you
will find.

Here I differ somewhat from the rationale offered by this chapters


authors. While DVT and PE can, and often do, occur together,
it is no longer clear that DVT must precede PE. In fact, there is
fairly strong evidence that PE can occur de novo following chest
trauma, perhaps incited by inflammation.4 DVT and PE are a
manifestation of a hypercoagulable state and these two conditions, although related, may exist separately; however, the treatment is the same: administer an anticoagulant.
5. How long should thromboprophylaxis be continued postoperatively?
With shorter and shorter hospital stays, the risk for developing
postdischarge VTE complications may be increasing. I agree with
the authors that methods aimed at VTE prophylaxis should be
utilized until the acquired risk factors have resolved (i.e., prolonged bed rest, cancer therapy, etc.). There is increasing evidence,
however, that one size does not fit all in the arena of pharmacologic prophylaxis. Standard doses of low molecular weight heparins, for example, may be inadequate in some patients and doses
may need to be adjusted by monitoring anti-Xa activity or using
thromboelastography (TEG).5 One additional point is to consider

2. What is the optimal diagnostic test for DVT?


I agree with the authors that Doppler ultrasound is currently
the optimal test to rule-out proximal DVT. However, there are
limitations when querying the pelvic veins with this tool, as well
as direct visualization of veins under casts and around external
783

PMPH_CH98.indd 783

5/22/2012 6:07:55 PM

784

Surgery: Evidence-Based Practice

a screening DUS prior to stopping therapy in extremely high-risk


patients.
6. What are the proper treatment protocols for initial and
recurrent DVT?
As the authors point out, 3 to 6 months of therapy with a heparintype drug followed by an oral anticoagulant is considered adequate for the initial episode of VTE, provided that there are no
ongoing active risk factors. The cost associated with monitoring
warfarin therapy may eventually be offset with the development of
newer and more effective direct thrombin inhibitors given in fi xed
doses.6 Unfortunately, the use of inferior vena cava (IVC) fi lters
has increased dramatically with the increased ease of placement
and the potential ability to retrieve them. It must be remembered
that IVC fi lters predispose to DVT rather than prevent DVT and
they are associated with a considerable number of complications
including migration, fragmentation, caval thrombosis, and PE.
Finally, considering that PE can develop without lower extremity
DVT, the role of IVC filters as a prophylactic measure needs careful re-evaluation.

PMPH_CH98.indd 784

REFERENCES
1. Geerts WH, Bergqvist D, Pineao GF, et al. Prevention of venous
thromboembolism: American College of Chest Physicians
Evidence-Based Clinical Practice Guidelines (8th Edition). Chest.
2008;133(6 Suppl):381S-453S.
2. Knudson MM, Ikossi DG, Khaw L, et al. Thromboembolism after
trauma: analysis of 1602 episodes from the American College
of Surgeons National Trauma Data Bank. Ann Surg. 2004;240:
490-498.
3. Kucher N. Deep-vein thrombosis of the upper extremities. N Engl
J Med. 2011;364:861-869.
4. Knudson MM, Gomez D, Haas B, et al: Three thousand seven hundred thirty-eight posttraumatic pulmonary emboli: a new look at
an old disease. Ann Surg. 2011; 254(4):625-32.
5. Van PY, Cho SD, Underwood SJ, et al. Thromboelastography versus antifactor Xa in the assessment of prophylactic dose enoxaparin in critically ill patients. J Trauma. 2009;66:1509-1517.
6. Schulman S, Kearon C, Kakkar AK, RE-COVER Study Group.
Dabigatan versus warfarin in the treatment of acute venous
thromboembolism. N Engl J Med. 2009;361:2342-2352.

5/22/2012 6:07:55 PM

CHAPTER 99

Pulmonary Embolism
George C. Velmahos

INTRODUCTION

calculations and decisions about the administration of potentially


harmful thromboprophylaxis is unknown. Multiple risk factors
have been suggested: obesity, immobility, cancer, major abdominal or pelvic operations, trauma, oral contraceptives, increasing
age, previous thromboembolism, pregnancy and postpartum
period, smoking, coagulation abnormalities, and acute medical
illness including heart, renal, and respiratory failure. There is poor
evidence documenting the impact of each one of these risk factors
on the pathogenesis of PE and contradictory studies are common.
For example, it is unknown which exact level of obesity, exact
duration and level of immobility, exact age, type and stage of cancer, or severity of medical illness predisposes the patient for PE.
A systematic review and meta-analysis of the existing literature
among trauma patients underscores precisely this inconsistency. 3
Although gender, head injuries, spinal fractures, spinal cord injuries, long-bone fractures, and pelvic fractures were examined as
possible risk factors among studies of trauma patients, only spinal
fractures and spinal cord injuries were found on pooled analysis to affect the incidence of venous thromboembolism (Level 2b
evidence). The study also found that the likelihood of venous
thromboembolism increases with old age and higher injury severity score; the threshold at which the rate of the outcome increases
significantly could not be determined by the available literature.
The seventh ACCP conference created a stratification of risk
according to the presence of risk factors.4 This stratification makes
clinical sense but is based on variable levels of evidence (typically,
Level 3) and therefore should be considered with caution. Patients
younger than 40 years with no other risk factors and minor surgery
are at low risk for PE. Patients at moderate risk have only one of the
following: age 40 to 60 years, major surgery, or a major preexisting
risk factor. Patients at high risk are those who are either older than
60 years or older than 40 years but with major surgery and a major
preexisting risk factor present. At the highest risk are patients who
are older than 40 years of age and have major surgery and one of the
following: previous thromboembolic event or cancer or hypercoagulable condition, major trauma, spinal injury, hip/knee arthroplasty,
hip surgery. Major surgery was considered as a thoracic or abdominal operation under general anesthesia lasting over 30 minutes.

Pulmonary embolism (PE) is a national health problem, claiming


over 50,000 lives in the United States. PE has been found in 32%
of surgical patients who had autopsy, and in about half of these
cases PE was thought to be the causing or contributing factor for
death.1 Although the sample of patients in that study was not representative of the entire surgical population and was subject to
variable thromboprophylactic practices, the high figures indicate
the importance of the problem. Currently, the overall PE rates
are estimated to be lower but vary significantly (0.330%) due to
the inconsistent screening and diagnosis among centers. The exact
percentage of fatal PE is unknown for the same reasons.
The pathogenesis of PE is based on the theory of clot dislodgment from a lower extremity or pelvic deep venous thrombosis
(DVT). Neck and upper extremity veins contribute on occasions.
However, there is a consistent disconnect in the literature between
DVT and PE. Although one would expect that a lower extremity or
pelvic DVT would be found on patients with PE, this is only infrequently the case. In the past, this discrepancy was explained by the
inaccuracy of available diagnostic methods to detect DVT, particularly of pelvic origin. With the development of CT venography and
high-definition ultrasonography, this is no longer the case. These
tests accurately evaluate the pelvic and proximal extremity veins
and frequently fail to discover DVT associated with an existing PE.
Therefore, the original theory of PE pathogenesis may be incorrect.
It is possible that PE does not always originate from peripheral veins
but may be formed de novo in the pulmonary circulation.2
There are more unknowns than standards in PE. The optimal
diagnosis, prevention, and treatment are under constant debate.

RISK FACTORS
1. Who is at risk for PE?
The classic Virchows triad places the surgical patient at risk for
PE but the exact level of risk that allows intelligent risk-to-benefit
785

PMPH_CH99.indd 785

5/22/2012 6:08:24 PM

786

Surgery: Evidence-Based Practice

The authors calculated a risk of 2% to 4% for PE and 0.4% to 1% for


fatal PE in patients at high risk and 4% to 10% for PE and 0.5% to 5%
for fatal PE in patients at the highest risk.
Answer: There is inconsistent evidence about the exact risk
factors that predispose to PE. It seems that major traumaand
particularly spinal injuriesolder age, major surgery, previous
history of thromboembolism, and cancer increase the risk. The
effect of other factors such as immobility, obesity, and medical illness is ill-defined (Grade B recommendation).

DIAGNOSIS
2. What is the optimal diagnostic test for PE?
The ventilation-perfusion (V-P) scan and pulmonary angiography
(PA) have been the main tests for diagnosis of PE for more than
20 years. The PIOPED (Prospective Investigation Of Pulmonary
Embolism Diagnosis) study5 showed that V-P scan is 96% sensitive when the index of clinical suspicion is high. However, 75%
of the patients belong to the intermediate category in which V-P
scan is less sensitive. Pulmonary angiography may still remain
the standard of reference but is invasive and requires significant
time spent in the angiography suite, a major setback for critically
ill patients.
Over the last 10 years, computed tomographic pulmonary
angiography (CTPA) has evolved to become the preferred diagnostic method for PE in surgical patients. In a meta-analysis of
the diagnostic performance of CTPA and V-P scan, Hayashino et al.6 examined 12 studies from 1985 to 2003, which were
selected according to the following three criteria: (1) the tests
were performed for the diagnosis of acute PE, (2) PA was used
as the standard of reference, and (3) absolute numbers of true
positive, true negative, false positive, and false negative fi ndings
were given. On the basis of these studies, a random effects model
found CTPA to have 86% sensitivity (95% confidence intervals
[CI]: 80.2%, 92.1%) and 93.7% specificity (95% CI: 91.1%, 96.3%).
V-P scan was found to have low sensitivity (39%) and high specificity (97.1%) with high probability threshold but high sensitivity (98.3%) and low specificity (4.8%) with normal threshold.
The authors concluded that, although V-P scan and CTPA have
similar diagnostic ability for patients with a high probability
for PE, CTPA has higher discriminatory power than V-P scan
for patients with normal and near-normal probability (Level 1b
evidence).
In another systematic review of the literature, Quiroz et al.7
examined the clinical validity of a negative CTPA for suspected
PE. Of particular concern was the alleged low sensitivity of
CTPA for peripheral PE. To calculate the overall negative likelihood ratio of PE after a negative or inconclusive CTPA, the
authors included PE that was confi rmed by another diagnostic
test within 3 months of CTPA. Fifteen studies with a total population of 3500 patients were included from 1994 to 2002. Singleslice, multidetector, and electron-beam scanners were used in
the different studies. The negative predictive value of a normal
CTPA was 99.7% (95% CI: 98.7%, 99.5%) and the negative likelihood ratio of a PE after a normal CTPA was 0.7 (95% CI: 0.05,
0.11). There was no difference in the risk of PE based on the different type of CT scanner. The authors concluded that the clinical

PMPH_CH99.indd 786

validity of CTPA to rule out PE is similar to that reported for


conventional PA (Level 1b evidence). This study shows that, even
if the diagnosis of peripheral PE is the principal limitation of
CTPA, undiagnosed peripheral PE (which can exist in as many
as 30% of normal CTPA) is usually not clinically significant
and does not cause subsequent clinically detectable PE or death
from PE.
Finally, a meta-analysis of different diagnostic strategies for
PE by Roy et al.8 included 48 articles of 1012 examined from 1990
to 2003. The study attempted to determine the clinical application
of each test according to pretest clinical probability. In patients
with a high pretest probability, a high-probability V-P scan, a
positive CTPA, and a positive lower extremity venous ultrasound
was associated with a higher than 85% posttest probability of PE.
In patients with an intermediate or low pretest probability, a normal or near-normal V-P scan, a normal CTPA in combination
with normal lower extremity venous ultrasound, and a D-dimer
concentration of less than 500 g/L measured by quantitative
enzyme-linked immunosorbent assay was associated with a less
than 5% posttest probability of PE. CTPA alone, magnetic resonance angiography, a low-probability V-P scan, and a quantitative
latex or hemoagglutination D-dimer test could only exclude PE in
patients with low pretest probability. The authors concluded that
the accuracy of the different tests vary significantly and according
to the pretest clinical probability for PE (Level 2 evidence).
Answer: CTPA is convenient, safe, and accurate for the diagnosis of clinically significant PE. It is the preferred diagnostic
method for most emergency surgery and trauma patients (Grade A
recommendation).

PREVENTION
3. Are heparin and compression devices adequate for PE
prophylaxis?
The use of low-dose unfractionated heparin (UFH), usually
administered subcutaneously, for prevention of PE was established in the mid-1970s by the seminal study of Kakkar et al.9 That
study included only elective surgery patients; emergency surgery
and trauma patients were excluded. Despite this fact, thromboprophylaxis by UFH became common practice for all surgical
patients. An overview of randomized trials of general, orthopedic,
and urologic surgery patients concluded that UFH reduced symptomatic PE rates from 2% to 1.3% and fatal PE rates from 0.8% to
0.3% but the risk of perioperative bleeding increased from 3.8%
to 5.9%.10 However, the evidence about UFH in trauma is controversial, and the evidence about UFH in emergency nontraumatic
general surgery patients simply does not exist. Low molecular
weight heparin (LMWH), also administered subcutaneously, has
shown increased stability and bioavailability compared to UFH,
benefits possibly associated with improved effectiveness and
safety. There are multiple randomized studies and meta-analyses
in general surgery patients documenting equivalence or superiority of LMWH over UFH11,12 but, again, this evidence is only modestly applicable to the emergency surgery population because the
majority of included patients had elective operations.
Sequential compression devices (SCDs) have been used
extensively based on the assumption that they promote blood

5/22/2012 6:08:25 PM

Pulmonary Embolism

flow, simulate muscle function, and trigger the release of fibrinolytic agents from the vascular endothelium. The evidence on
their effectiveness is also questionable, and at least two studies
document poor compliance.13,14 Th is could be the ultimate drawback for their use, as it gives the physician a false sense of security, while the patient receives no benefit from the prescribed
treatment.
There is a number of noncontrolled studies and a few prospective randomized trials in trauma patients. Knudson et al. produced
three randomized trials (Level 1c evidence). In 1992 the authors randomized 113 trauma patients to UFH or SCD and found no significant difference in thromboembolic complications (5 patients with
DVT, 4 with PE, and 3 with DVT and PE) between the two groups.15
In 1994 the authors compared patients receiving UFH, SCD, or no
treatment and found similar VT rates in the three groups, except
for a mild advantage of SCD over no treatment in neurosurgical
patients.16 There were only two documented PEs, one in an SCD
patient and one in a patient who received no thromboprophylaxis.
In 1996 they randomized 181 patients to LMWH or SCD and failed
to find any significant difference in DVT.17 There were no documented cases of PE in any of the randomized groups.
In a study of LMWH against SCD in head and spinal trauma,
60 patients were randomized to LMWH and 60 to SCD.18 The incidence of PE was not different between the two groups, 7% in the
LMWH group and 3% in the SCD group. This high incidence of
PE could indicate a poor thromboprophylactic effect of LMWH
and SCD (Level 1c evidence). In another randomized study, spinal cord injury patients received either UFH with SCD or LMWH
and showed no difference in proximal DVT or PE rates.19 The
total number of thromboembolic events was very high and almost
identical in the two groups (65.5% for LMWH and 63.3% for UFH
with SCD, P = .81), placing again in doubt the effectiveness of
these regimens (Level 1c evidence).
Probably, the two best-designed randomized trials in trauma
patients examined LMWH versus SCD20 and LMWH versus
UFH.21 In both, DVT and not PE (or total thromboembolic events)
was the principal outcome. In the study by Ginzburg et al.20 the
DVT rates were similar between LMWH and SCD. There was
one PE in each group. There was no difference in thromboembolic events when a subanalysis of patients with injury severity
score higher than 19 was undertaken. The rate of bleeding was
not different either (Level 1b evidence). In the study by Geerts
et al.21 LMWH was associated with lower DVT rates compared
to UFH. There was only one patient with documented PE (a highprobability V-P scan) and he belonged to the LMWH group. The
rate of major bleeding was not different (0.6% vs. 2.9%, P = .12)
but of the six documented episodes, one was in the UFH group
and five in the LMWH group (Level 1b evidence). Two systematic
reviews of the existing evidence in trauma confirmed the low level
of evidence that exists about UFH, SCD, and LMWH, and the
uncertainty about their exact profi le of effectiveness and safety22,23
(Level 1b evidence).
Answer: Although general surgery patients with elective operations seem to benefit from the current thromboprophylactic
methods, the effectiveness of UFH, LMWH, and SCD in emergency surgery and trauma patients remains uncertain. An individual risk-to-benefit assessment should be made for each such
patient at risk of PE. LMWH is probably more effective than UFH
or SCD (Grade B recommendation).

PMPH_CH99.indd 787

787

4. Are PE and mortality from PE reduced by inferior vena cava


filters?
The effectiveness of inferior vena cava (IVC) fi lters relies on their
ability to capture clot originating from lower extremity or pelvic
veins. Three scenarios may hamper this ability. First, a misplaced
or tilted fi lter may not function adequately. A tilt of as little as 10o
in relationship to the IVC axis has been reported to compromise
optimal function.24 Second, the capture of a primary clot at the
apex of the fi lter may force blood circulation toward the periphery of the vessel and recurrent clots to escape in this way. Third,
clots may originate from upper extremity or neck veins25 or even
possibly form de novo in the pulmonary circulation,2 in which
case a device in the IVC is obviously of no use. One can argue
that a fi lter is never used therapeutically, since its effect never
involves a PE that has already occurred but only the embolus that
may follow. The use of IVC fi lters in patients with breakthrough
PE (occurring while the patient is fully anticoagulated) or with
primary PE and inability to anticoagulate is well accepted. Other
criteria are more controversial and include (1) a contraindication
for prophylactic anticoagulation in the presence of high risk for
PE, (2) added prophylaxis in patients at very high risk for PE even
if prophylactic anticoagulation is feasible, (3) added prophylaxis
in patients who have already sustained a significant PE and are
therapeutically anticoagulated but would be at risk of death, if a
breakthrough PE occurred26 (Level 3b evidence).
Currently, retrievable fi lters have replaced temporary fi lters
for most indications. Unless it is deemed that a fi lter needs to
remain in place for life, as it may happen with spinal cord injury
patients or very old patients with significant comorbidities, most
trauma and emergency surgery patients have only a finite period
of risk and therefore do not need a permanent device. Unfortunately, a multicenter study27 has shown that only 19% of these filters are being removed and, therefore, most are left permanently,
even if not designed for this purpose (Level 3a evidence).
There is not a single prospective randomized study on the use
of IVC fi lters in trauma and emergency surgery patients. Decousous et al.28 randomized a mixed population of 200 predominantly
medical patients with DVT into IVC fi lter versus no fi lter. After a
2-year follow-up those with filters had a significant decrease in PE
but a significant increase in DVT. There was no difference in mortality. When this population was followed up for 8 years,29 the results
remained unchanged: the IVC fi lter group had a lower incidence
of PE (6.2% vs. 15.1%, P = .008), higher incidence of DVT (35.7%
vs. 27.5%, P = .042), and no difference in mortality compared to
the no fi lter group (Level 1b evidence). Studies of trauma patients
have failed to consistently prove that the insertion of vena cava
fi lters resulted in a decrease of PE or death from PE30-32 (Level 3b
evidence).
IVC fi lters are not complication free. Morbidity related to
access (bleeding, thrombosis, arterial damage), catheter advancement (vessel damage), contrast material (anaphylaxis, renal failure), and the fi lter itself (vessel wall perforation, migration, IVC
thrombosis, DVT, misplacement) is detected in approximately 4%
to 7% of the cases, although the variability in rates among studies
is great26 (Level 3a evidence). A new class of complications is now
related to the removal of retrievable fi lters, including all of the
above problems as well as dislodgement of clot captured by the
fi lter, damage of the IVC wall, and inability to retrieve.

5/22/2012 6:08:25 PM

788

Surgery: Evidence-Based Practice

Answer: There is no convincing evidence that in emergency


surgery and trauma patients, IVC fi lters reduce the incidence of
PE and mortality from PE. Use of IVC fi lters should be made
based on an individual patient-by-patient risk-to-benefit analysis
(Grade C recommendation).

TREATMENT
5. Is LMWH as safe and effective as UFH for the treatment
of PE?
Dose-adjusted intravenous UFH is used for the treatment of PE.
However, subcutaneous LMWH at therapeutic doses presents
significant benefits over UFH, as monitoring is not required and
treatment can be self-administered at home. There are multiple
randomized studies in the literature and all of them include
either exclusively or predominantly medical patients. Therefore,
the evidence on emergency surgery and trauma patients is poor.

A meta-analysis of 12 randomized studies33 found that LMWH


was associated with a nonsignificant decrease of symptomatic
PE (1.7% vs. 2.3%) and asymptomatic PE (1.2% vs. 3.2%), while
offering a nonsignificant advantage in decreasing bleeding (1.3%
vs. 2.1%), compared to UFH (Level 1a evidence). The authors
concluded that LMWH was at least as safe and effective as UFH
for the initial treatment of PE. It is expected that in emergency
surgery and trauma patients the rates of the above outcomes
and specifically of bleedingmay be different but there is little
reason to believe that the equivalence between the two groups
will not be maintained. However, concerns about the inability
to reverse LMWH effectively by protamin if a high-risk patient
were to bleed, may still create discomfort in consistently using
LMWH over UFH.
Answer: LMWH is as safe and effective as UFH for the
treatment of PE. It may be the preferred treatment in patients
at lower risk of bleeding, based on the convenience of outpatient self-administration and no need for monitoring (Grade A
recommendation).

Clinical Question Summary


Question

Answer

Grade of
Recommendation

References

1 Who is at risk for PE?

Patients with spinal injuries, older age, major surgery or


trauma, previous history of thromboembolism, and cancer.

3, 4

2 What is the optimal


diagnostic test for
PE?

Computed tomographic angiography.

6-8

3 Are heparin and


compression devices
adequate for PE
prophylaxis?

The effectiveness of heparin and compression devices in


trauma and emergency surgery patients is unclear. LMWH
seems to perform better than UFH.

11-23

4 Are PE and mortality


from PE reduced by
IVC filters?

The effectiveness of IVC filters in reducing PE and mortality


from PE in trauma and emergency surgery patients is
unclear.

28-32

5 Is LMWH as safe and


effective as UFH for
the treatment of PE?

Yes.

33

Evidence-Based Table
Study

Design

Intervention

Description

Results

#15

RCT

Duplex within 24 hours of


admission and every 5 days.
Prophylaxis by SCD or UFH or
nothing.

113 patients included. 76


randomized to SCD and
37 to UFH.

VT developed in 12 (5 DVT, 4 PE,


3 both); 9 SCD and 3 UFH.
Risk factor for DVT was spinal
trauma.

#16

RCT

Duplex within 24 hours of


admission and then every 57
days. Prophylaxis by SCD or
UFH or nothing.

Division in 3 groups and


randomization within each
group. Group I: UFH or SCD
or nothing, Group II: UFH or
nothing, Group III: SCD or
nothing.

255 patients15 developed DVT;


unclear PE. No difference in
Group I (2.3% UFH, 14.2% SCD,
3.2% nothing) and Group II
(5.5% UFH, 8% nothing). Lower
DVT in Group III (0% SCD,
14.7% nothing).
(Continued)

PMPH_CH99.indd 788

5/22/2012 6:08:25 PM

Pulmonary Embolism

789

(Continued)
Study

Design

Intervention

Description

Results

#17

RCT

Duplex on admission and every


57 days. Prophylaxis by
LMWH, SCD, AVF.

487 patients included, 372


analyzed. 202 stratified to the
heparin group and randomized
to LMWH (120) or SCD
(61)/AVF (21). 170 stratified
to the no heparin group and
received SCD/AVF.

DVT in 9 (2.4%) and PE in 1. In


randomized patients 1 LMWH
and 2 SCD patients had DVT.
The other 6 DVT were in the
nonrandomized group.

#20

RCT

Duplex within 24 hours of


admission and then weekly.

294 moderately and 148 severely


injured patients randomized
separately into LMWH and
SCD.

DVT in 2.7% of SCD and 0.5% of


LMWH (P = .12). PE in 1 patient
per group.

#21

RCT

Venography 1014 days after


admission. Prophylaxis by UFH
or LMWH.

265 included; 136 randomized


to UFH and 129 to LMWH.

DVT (44% UFH, 31% LMWH,


P = .014).
Proximal DVT (15% UFH, 6%
LMWH, P = .012). 1 PE in
LMWH.

#28

RCT

V-P scan and, if necessary,


pulmonary angiography.

400 randomized: 200 to filter and


200 to no filter.

At day 12, PE developed in 1.1% of


filters and 4.8% of no filters
(P = .03). At 2 years 20.8% filter
and 11.6% no filter patients had
recurrent DVT (P = .02).

The intervention described refers to DVT. No study except #28 had a protocolized routine intervention for PE.
RCT: randomized controlled study, SCD: sequential compression device, UFH: unfractionated heparin, LMWH: low molecular weight heparin,
AVF: arteriovenous foot pumps, DVT: deep venous thrombosis, PE: pulmonary embolism, V-P scan: ventilation-perfusion scan.

REFERENCES
1. Lindblad B, Eriksson A, Bergqvist D. Autopsy-verified pulmonary embolism in a surgical department. Analysis of the period
from 1951 to 1988. Br J Surg. 1991;78:849-852.
2. Velmahos GC, Spaniolas K, Tabbara M, et al. The relationship of
pulmonary embolism and deep venous thrombosis in trauma.
Are they really related? Arch Surg. 2009;144:928-932.
3. Velmahos GC, Kern J, Chan LS, Oder D, Murray JA, Shekelle P. Prevention of venous thromboembolism after injury:
an evidence-based reportpart II: analysis of risk factors and
evaluation of the role of vena caval fi lters. J Trauma. 2000;49(1):
140-144.
4. Geerts WH, Pineo GF, Heit HA, et al. Prevention of venous thromboembolism: the seventh ACCP conference on antithrombotic
and thrombolytic therapy. Chest. 2004;126(3 Suppl):338S-400S.
5. The PIOPED Investigators. Value of the ventilation/perfusion
scan in the diagnosis of pulmonary embolism: results of the
prospective investigation for pulmonary embolism diagnosis
(PIOPED). JAMA. 1990;263:2753-2759.
6. Hayashino Y, Goto M, Noguchi Y, Fugul T. VentilationPerfusion scanning and helical CT in suspected pulmonary
embolism: meta-analysis of diagnostic performance. Radiology.
2005;234:740-748.
7. Quiroz R, Kucher N, Zou KH, et al. Clinical validity of a negative
computed tomography scan in patients with suspected pulmonary
embolism. A systematic review. JAMA. 2005;293:2012-2017.
8. Roy PM, Colombet I, Durieux P, Chatellier G, Sors H, Meyer G.
Systematic review and meta-analysis of strategies for the diagnosis of suspected pulmonary embolism. BMJ. 2005;331:1-9.

PMPH_CH99.indd 789

9. Kakkar VV, Corrigan TP, Fossard DP, et al. Prevention of fatal


postoperative pulmonary embolism by low doses of heparin. An
international multicentre trial. Lancet. 1975;2(7924):45-51.
10. Collins R, Scrimgeour A, Yusuf S, Peto R. Reduction in fatal
pulmonary embolism and venous thrombosis by perioperative
administration of subcutaneous heparin. Overview of results of
randomized trials in general, orthopedic, and urologic surgery.
N Engl J Med. 1988;318:1162-1173.
11. Koch A, Bouges S, Ziegler S, et al. Low molecular weight heparin and unfractionated heparin in thrombosis prophylaxis after
major surgical intervention: update of previous meta-analyses.
Br J Surg. 1997;84:750-759.
12. Mismetti P, Laporte S, Darmon JY, et al. Meta-analysis of low
molecular weight heparin for the prevention of venous thromboembolism in general surgery. Br J Surg. 2001;88:913-930.
13. Cornwell EE, Chang D, Velmahos G, et al. Compliance with
sequential compression device prophylaxis in at-risk trauma
patients: a prospective analysis. Am Surg. 2002;68:470-473.
14. Comerota AJ, Katz ML, White JV. Why does prophylaxis with
external pneumatic compression for deep vein thrombosis fail?
Am J Surg. 1994;164:265-268.
15. Knudson MM, Collins JA, Goodman SB, McCrory DW. Th romboembolism following multiple trauma. J Trauma. 1992;32:
2-11.
16. Knudson MM, Lewis FR, Clinton A, Atkinson K, Megerman J.
Prevention of venous thromboembolism in trauma patients.
J Trauma. 1994;37:480-487.
17. Knudson MM, Morabito D, Paiement GD, Schackleford S. Use of
low molecular weight heparin in preventing thromboembolism
in trauma patients. J Trauma. 1996;41:446-459.

5/22/2012 6:08:25 PM

790

Surgery: Evidence-Based Practice

18. Kurtoglou M, Yanar H, Bilsel Y, et al. Venous thromboembolism


prophylaxis after head and spinal trauma: intermittent pneumatic compression devices versus low molecular weight heparin.
World J Surg. 2004;28:807-811.
19. Merli G and the Spinal Cord Injury Thromboprophylaxis Investigators. Prevention of venous thromboembolism in the acute treatment phase after spinal cord injury: a randomized, multicenter
trial comparing low-dose heparin plus intermittent pneumatic
compression with enoxaparin. J Trauma. 2003;54:1116-1126.
20. Ginzburg E, Cohn SM, Lopez K, et al. Randomized clinical
trial of intermittent pneumatic compression and low molecular
weight heparin in trauma. Br J Surg. 2003;90:1338-1344.
21. Geerts WH, Jay RM, Code KI, et al. A comparison of low-dose
heparin with low-molecular-weight heparin as prophylaxis
against venous thromboembolism after major trauma. N Engl J
Med. 1996;335:701-707.
22. Velmahos GC, Kern J, Chan LS, Oder D, Murray JA, Shekelle P. Prevention of venous thromboembolism after injury: an
evidence-based reportpart I: analysis of risk factors and evaluation of the role of vena caval filters. J Trauma. 2000;49(1):132-138.
23. Rogers FB, Cipolle MD, Velmahos GC, Rozycki G, Luchette FA.
Practice management guidelines for the prevention of venous
thromboembolism in trauma patients: the EAST practice management guidelines work group. J Trauma. 2002;53:142-164.
24. Rogers FB, Stringberg G, Schackford GR, et al. Five-year followup of prophylactic vena cava fi lters in high-risk trauma patients.
Arch Surg. 1998;133:406-411.
25. Hingorani A, Ascher E, Lorenson E, et al. Upper extremity deep venous thrombosis and its impact on morbidity and

PMPH_CH99.indd 790

26.
27.

28.

29.

30.

31.

32.

33.

mortality rates in a hospital-based population. J Vasc Surg.


1997;26(5):853-860.
Martin MJ, Salim A. Vena cava fi lters in surgery and trauma.
Surg Clin N Am. 2007;87:1229-1252.
Karmy-Jones R, Jurkovich G, Velmahos GC, et al. Practice patterns and outcomes after retrievable vena cava fi lters in trauma
patients: a AAST multicenter study. J Trauma. 2007;62:17-25.
Decousous A, Leizorovicz S, Parent F, et al. for the PREPIC study
group. A clinical trial of vena cava fi lters in the prevention of
pulmonary embolism in patients with deep vein thrombosis.
N Engl J Med. 1998;338:409-415.
The PREPIC study group. Eight year follow-up of patients with
permanent vena cava fi lters in the prevention of pulmonary
embolism Circulation. 2005;112:416-422.
Antevil JL, Sise MJ, Sack DI, et al. Retrievable vena cava fi lters for
preventing pulmonary embolism in trauma patients: a cautionary tale. J Trauma. 2006;60:35-40.
Rogers FB, Shackford SR, Ricci MA, Wilson JT, Parsons S. Routine prophylactic vena cava fi lter insertion in severely injured
trauma patients decreases the incidence of pulmonary embolism. J Am Coll Surg. 1995;180:641-647.
McMurtry AL, Owings JT, Anderson JT, Battistella FD, Gosselin R. Increased use of prophylactic vena cava fi lters in trauma
patients failed to decrease overall incidence of pulmonary embolism. J Am Coll Surg. 1999;189:314-320.
Quinlan DJ, McQuillan AM, Eikelbloom JW. Low-molecularweight heparin compared with intravenous unfractionated
heparin for treatment of pulmonary embolism. Ann Intern Med.
2004;140:175-183.

5/22/2012 6:08:25 PM

Commentary on
Pulmonary Embolism
Kenneth L. Mattox

First, I commend Dr Velmahos on an excellent, in-depth review


of a complicated and controversial subject in his chapter entitled,
Pulmonary Embolism. Since he has been reviewing the literature
and reporting on his clinical and basic science interests in deep
venous thrombosis (DVT), pulmonary thromboembolism, and
fatal pulmonary embolism (PE) for almost two decades, it is appropriate that he should report on the relevance of evidence supporting the many biases and opinions relating to these three different
conditions. He has presented a complete analysis of current state
of all aspects surrounding pulmonary embolism. As Dr Velmahos
indicates, attempts to discover firm, scientific evidence (Class 1 or
2 data) to address the impact of prevention and treatment strategies have been difficult. In the initial statement of the problem, he
points out that the exact number of hospitalized patients who die as
a direct result of fatal pulmonary is not known. Including patients
with already terminal conditions who, at autopsy, are found to have
finally succumbed as a result of a pulmonary embolus would not be
appropriate in a book with evidence-based in its title.
With the introduction of devices and drugs to treat lower
extremity (and pelvic) thrombophlebitis, it was predictable that
various treatments would be promoted. The literature is rich in
modifiers relating to pulmonary emboli, from words such as minor,
segmental, silent, and chronic to more troublesome words such as
saddle, fatal, central, sudden, massive, and extensive. Attempts to
quantify these modifiers can only kindly be described as confusing.
As Dr Velmahos points out, a pulmonary embolus can originate
in the upper extremity or neck veins, the lower extremity and pelvic
veins, or the major source, the pulmonary arteries, but the specific
source of a fatal pulmonary embolism is rarely discernable. Without a foundation of understanding of these variances and just which
venous clots from which sites produce the clinically significant fatal
pulmonary emboli, any discussion of evidence-based continues
to be Class 3 (or worse) data. It is ironic that after over 100 years of
extensive research on this subject and the expenditure of millions
of dollars on research and therapy for these conditions of DVT and
fatal pulmonary embolism, our knowledge base on these subjects is
still lacking in evidence-based science.
In this chapter and in previous publications and presentations
(cited in his chapter), Dr Velmahos has suggested that the occurrence of lower extremity DVT in the hospitalized surgical patient
may have very little to do with the appearance of a large fatal
venous clot in the main pulmonary artery or in both main right

and left pulmonary arteries. Ironically, it is the linkage of lower


extremity DVT and the potential appearance of a fatal pulmonary
embolism that directs most of our pharmaceutical therapy. It is
also unknown, even in 2011, whether or not our usual and standard best pharmaceutical practice actually affects the deep veins
in the pelvis and the inferior vena cava.
Dr Velmahos has asked a series of provocative questions to
attempt to look for evidence-based answers.
1. Who is at risk for PE?
2. What is the optimal diagnostic test for PE?
3. Are heparin and compressive devices adequate for PE prophylaxis?
4. Are PE and mortality from PE reduced by IVC filters?
5. Is LMWH as safe and effective as UFH for the treatment of PE?
The answers leave the reader with more questions than answers, not because the author has not thoroughly and thoughtfully
sought the answers, but because this subject continues to need better science-based evidence to support our treatments. He does cite
that computed tomographic pulmonary angiography (CTPA) is a
convenient and safe diagnostic modality for PE and that LMWH
is as safe and effective as UFH. I was also pleased to see substantiated by Dr Velmahos the fact that most retrievable IVC fi lters
are NOT retrieved, even though these devices have been shown
to have a higher rate of clot formation on the fi lter than the nonretrievable fi lters.
The surgical historian might find interest in a presentation by
Dr Michael DeBakey to the Philadelphia County Medical Society on September 17, 1952, entitled, A Critical Evaluation of the
Problem of Thromboembolism.1 This article contains 447 references, virtually all the world literature on the subject at the time.
Dr DeBakeys conclusions then were almost identical to those of
Dr Velmahos in this chapter. Furthermore, Dr DeBakey made the
suggestion then that all our therapies for DVT had not altered the
incidence of in-hospital fatal PE. It appears that Dr Velmahos has
made the same suggestion again in 2011.

REFERENCE
1. DeBakey ME. A critical evaluation of the problem of thromboembolism. Surg Gynecol Obstet. 1954;98:1-27.

791

PMPH_CH99.indd 791

5/22/2012 6:08:25 PM

CHAPTER 100

Management of Lymphedema
Magdiel Trinidad-Hernandez and Peter Gloviczki

INTRODUCTION

primary are due to inborn abnormalities of the lymphatic system.


It is still unclear how or why the lymphatic system is damaged
or malformed in these cases. However, some light has been shed.
Investigators have linked certain genetic conditions in patients
with congenital lymphedema. For instance, Milroys disease was
mapped to the telomeric part of chromosome 5q, in the region
5q34-q35. This region contains the gene for vascular endothelial
growth factor receptor-3 (VEGFR-3), which encodes a receptor,
tyrosine kinase, specific for lymphatic vessels. Defective VEGFR-3
signaling seems to be the cause of congenital hereditary lymphedema linked to 5q34-q35.4 Lymphedema is associated with
other genetic syndromes such as Noonan syndrome, Pierre Robin
syndrome, Aegenaes syndrome, Turner syndrome, and Klinefelter
syndrome.5
Secondary lymphedema is much more common and occurs
when a recognized pathological process has disrupted the lymphatic system. In the United States, cancer operations are the most
frequent cause. Breast cancer surgery with regional lymph node
dissection is the most common and the most studied. The overall
risk of lymphedema associated with breast cancer therapy is estimated to be 20% to 30%. This number varies based on the type of
surgical resection and whether radiation therapy has been used or
not. Other cancers can also be associated with lymphedema. In
a recent meta-analysis that excluded breast cancer, the incidence
of lymphedema was 15.5% and it varied according to the type of
malignancy: it was 16% with melanoma (upper extremity, 5%;
lower extremity, 28%), 20% with gynecologic malignancies, 10%
with genitourinary tumors, 4% with head and neck cancers, and
30% with sarcomas. Increased lymphedema risk was also noted
for patients undergoing pelvic dissections (22%) and radiation
therapy (31%).3
In developing nations, parasitic infections remain the most
common cause of chronic lymphedema. Of the estimated 90.2
million people in the world who are infected, more than 90% suffer from fi lariasis caused by Wuchereria bancrofti.6
Answer: In the United States and in other developed nations
the most commonly seen form of chronic lymphedema is linked
to cancer therapy.

Lymphedema is the term given to swelling due to accumulation


of protein-rich interstitial fluid in the subcutaneous tissues. Lymphedema develops when lymph production exceeds the transport
capacity of the lymphatic system. Although lymphedema can
develop in many areas, the clinical problem is usually chronic
swelling of the limbs or genitalia. Chronic lymphedema is most
frequently the result of acquired or congenital obstruction of
the lymphatic system. Less frequently, lymphatic stasis develops
due to valvular incompetence in the lymphatic system. Finally,
in some patients, lymphedema is the result of excessive production of lymph and the inability of the lymphatic system to cope
with the problem (high output lymphatic failure).
Whether lymphatic insufficiency is a result of an intrinsic
defect in the vessel itself, a consequence of tumor, trauma, lymph
node dissection, irradiation, or scarring secondary to infection,
the end-result is the accumulation of different plasma proteins,
immunoglobulins, and cytokines leading to chronic inflammatory changes in the subcutaneous tissue.
There are very few reports in the literature to provide an
accurate estimate of the incidence of lymphedema. However, it is
clear that secondary lymphedema is more common and the most
frequent form in the world today is parasitic infection caused by
fi lariasis.1 Filariasis is endemic in some parts of the world and it is
estimated that one in every 20 people on earth has some form of
lymphedema. Primary lymphedema is rare and often seen in association with other infrequent congenital anomalies.2 In developed
nations, the most common cause of lymphedema is lymph node
dissection combined with cancer operations.3

RISK FACTORS
1. Who gets lymphedema?
Standard clinical classifications differentiate lymphedema based
on cause. The majority of chronic lymphedema cases classified as
792

PMPH_CH100.indd 792

5/22/2012 6:08:55 PM

Management of Lymphedema

CLASSIFICATION OF LYMPHEDEMA
2. What are the types of lymphedema?
Lymphedema can be classified according to cause into primary
(congenital) and secondary forms.7 Primary lymphedema is subdivided according to lymphatic anatomical abnormalities (obstruction or incompetence). In obstruction the lymphatic vessels may be
absent or hypoplastic. Alternatively, there may be numerical hyperplasia. In this case the lymphatics are hyperplastic but nonfunctional. When the lymphatic vessels are dilated and develop valvular
incompetence the term used to describe the disease is lymphangiectasia. Rupture of the dilated lymphatics can cause lymphatic or
chylous effusions, like chylous ascites or chylothorax.
Primary lymphedema is also categorized according to the
time of onset. When the swelling is present at birth or diagnosed
within the first year of life it is termed congenital. The most common time of presentation is, however, during puberty. Th is form
is called lymphedema praecox and it accounts for 94% of the cases
of primary lymphedema. The age of 35 has arbitrarily been determined as the cutoff for lymphedema praecox. When patients present after age 35 with primary lymphedema, the term we use for
the disease is lymphedema tarda. Frankly, this classification has
neither been very helpful in determining the cause of limb swelling nor has it been helpful in guiding therapy.
Answer: Lymphedema is classified by cause into primary
and secondary forms. In primary lymphedema there is an innate
defect, absence, or hypoplasia of the lymphatic vessels. Secondary
lymphedema develops as a consequence of damage and fibrotic
occlusion of the lymphatic channels. This is usually due to injury
or infection.

DIAGNOSIS
3. What diagnostic techniques are valuable?
In most patients with chronic lymphedema the diagnosis can be
made based on the history and physical examination. History
recording must pay attention to travel to endemic regions. Also a
history of trauma, cancer surgery, insect bite, and infection must
be noted. A family history of limb swelling must also be recorded.
Also, all other systemic or local causes of limb swelling must be
excluded. The physical examination of a patient with lymphedema
should include inspection for cutaneous and subcutaneous fibrosis and peau dorange. In chronic lymphedema swelling is nonpitting. The pathognomonic Stemmer sign is the inability to tent the
skin of the interdigital webs. In addition to this, the toes acquire
a square appearance and the dorsum of the foot has a puff y look
that resembles a buffalo hump. Any limb length discrepancy
should also be noted.
In cases of primary lymphedema, particularly those presenting later in life, the first efforts must be geared toward the exclusion of other common causes of limb swelling. The fi rst test
performed should be a venous duplex ultrasound because of the
high prevalence of venous insufficiency caused by venous valvular
incompetence or by venous obstruction.
Isotope lymphoscintigraphy is today the gold standard test
for lymphedema.8,9 Lymphoscintigraphy is a functional as well
as an anatomical study.10 This test is performed with injection of
Tc 99m-labeled human serum albumin between the fi rst and

PMPH_CH100.indd 793

793

second web-space of the toes or fingers. In normal limbs, several


superficial lymph vessels are visualized along the anteromedial
aspect of the leg, the calf, and along the medial aspect of the thigh.
Tracer activity is clear in the inguinal lymph nodes by 60 minutes
(range, 1560 minutes). The para-abdominal nodes are usually visible at 1 hour, and a faint activity may be seen in normal studies in the
liver and bladder as well. Three-hour images show intense uptake in
the lymph nodes of the groin, pelvis, and abdomen, and, occasionally, a tracer focus in the left supraclavicular area at the site of
the distal thoracic duct. At 3 hours, uptake in the liver and bladder
is well seen.
Typical findings of lymphatic stasis in lymphedema include
subcutaneous accumulation of the tracer, known as dermal backflow. Other findings include absent or delayed transport of tracer
with absent or delayed visualization of the regional lymph nodes.
Lymphatic collaterals may also be visible.
The asymmetry or delayed appearance of radiocontrast in the
proximal lymph nodes can be used as a semiquantitative measure
of the severity of lymphatic vascular insufficiency. The density of
the dermal backflow can also be quantitated, as can the ratio of
radioactivity in ipsilateral versus contralateral nodes in the setting
of unilateral limb edema. Quantitation has been used to predict the
likelihood of a beneficial response to therapeutic intervention.
Scintigraphic findings in lymphangiectasia consist of dilated
lymph channels with only mild or no delay in lymph transport.
Colloid injected into the unaffected lower extremity may reflux
into the affected lymphedematous leg because of lymphatic valvular incompetence. Similar reflux of the colloid may be seen
in the dilated mesenteric lymphatics or in the retroperitoneum,
perineum, or scrotum. Ruptured lymphatics cause extravasation
of the colloid into the abdominal cavity or the chest in patients
with chylous ascites or chylothorax. The images are generally helpful in determining the approximate site of the lymphatic leak.
Contrast lymphography is rarely used today to study the lymphatic system. However, in patients with lymphangiectasia and
chylous leaks, contrast lymphangiography may provide useful
information. Imaging is obtained through the direct injection of
iodine-based, lipid-soluble agents into subcutaneous lymphatics.
Contrast lymphography is technically difficult and may exacerbate lymphatic malfunction through the buildup and pooling of
the oil-based contrast medium. For these reasons, its use should
be limited in lymphedema to preoperative evaluation of selected
patients, and should be performed only in specialized centers.
Answer: Patient history and physical examination will establish the diagnosis in most cases. Lymphoscintigraphy with
Tc99m-labeled human serum albumin can differentiate lymphedema from edemas of other etiology. A normal lymphoscintigram excludes the diagnosis of lymphedema. The most frequently
used additional test geared to exclude underlying venous disease
is duplex scanning. Contrast lymphangiography is reserved for
use in select cases of lymphangiectasia or chylous eff usions.

TREATMENT
4. What is the optimal early management?
Lymphedema is more easily managed in the early stages before
subcutaneous fibrotic changes occur. Complex decongestive physical therapy is recommended today as the gold standard for treatment of chronic lymphedema. This program, first used in Europe,

5/22/2012 6:08:56 PM

794

Surgery: Evidence-Based Practice

has been modified in the last few decades and has been introduced
with success in most medical centers and lymphedema clinics across the United States.11,12 The program requires a life-long
commitment to lifestyle modifications and compression therapy
and it has been most successful in those patients who are highly
motivated and have a strong support network. The program consists of two phases. The first phase includes an intensive program
of volume reduction. It involves manual lymphatic drainage massage by certified therapists and the application of low-stretch multilayered wraps. In addition, the patients are educated on exercise,
skin care, and elevation principles.
The second phase of the program is dedicated to maintenance
therapy. This includes the daily use of compression garments, continued nightly wrapping, self-administered lymphatic massage,
continuous exercise, and skin care.
Answer: Manual lymphatic drainage massage and compression therapy are the mainstay of early lymphedema treatment.
In late cases, an intensive regimen can control limb swelling and
help patients cope with the process.
5. What is the best long-term treatment?
Chronic lymphedema is not curable. However, it can be managed and kept under control to help the patient adapt to a normal
life. Prevention of any problem is better than treating it. Satisfactory results can be obtained with a few prophylactic measures.
There is evidence that the routine use of an appropriately fitting
compression garment can control the volume of the extremity.13
It is a Class A recommendation in a recent consensus statement
of an expert panel, organized by the International Union of
Phlebology.8
Skin care is particularly important to avoid cellulitis and
lymphangitis. Should this occur, antibiotics with superior grampositive coverage are recommended. Examples include cephalexin,
penicillin, clindamycin, and cefadroxil.8
Exercise has been recommended as an integral part of the
regimen to control lymphedema. However, strong evidence to
support this recommendation is lacking.
Answer: The best long-term treatment regimen for chronic
lymphedema includes the routine use of compression garments to

control the limb volume variability and the prevention of infections (Level II evidence, Class B recommendation).

MICROVASCULAR SURGERY FOR


LYMPHEDEMA
6. Does microvascular surgery provide relief?
Microvascular surgical techniques for lymphedema require high
level of training and expertise. Mostly in uncontrolled and retrospective studies, specialized microsurgical centers have reported
good results with lymphatic microsurgery, using either a technique to anastomose lymph vessels to veins (lymphovenous anastomosis) or the use of free lymphatic grafts, harvested from the
superficial medial bundle of the nonaffected lower limb. Campisi
and coworkers reported results in 665 patients with obstructive
lymphedema using microsurgical lymphovenous anastomosis
with subjective improvement in 87% of their patients.14-15 Four
hundred forty-six patients were available for long-term follow-up
and volume reduction was observed in 69% with discontinuation of conservative measures in 85%. The authors concluded that
microsurgical reconstruction early in the course of lymphedema
is more effective, since intrinsic contractility of the lymphatics is
still maintained. Chances of normalization of the lymph circulation are better before significant chronic inflammatory changes in
the subcutaneous tissue develop.
Recent experience with supermicroscopic surgical techniques,
using high-power magnification, was reported by Narushima and
colleagues.16 Using stents to facilitate multiconfigurational lymphovenous anastomosis, these authors observed good early results
after performing 39 lymphovenous anastomoses in 14 patients.
Mean limb girth decreased by 3.6 cm, ranging from 1.5 to 7 cm
(11.3%). More importantly, the number of anastomoses performed
directly correlated with the degree of improvement.
Answer: Microvascular reconstructive surgery has been reported in uncontrolled single center experiences to have good results.
However, these operations on lymph vessels <1 mm in size require
high level of microsurgical training, expertise, and commitment.

Clinical Question Summary


Question

Answer

Levels of Grade of
References
Evidence Recommendation

1 Who gets
lymphedema?

In the United States and in other developed nations, the


most commonly seen form of chronic lymphedema is
linked to cancer therapy.

II

2 What diagnostic
techniques are
valuable?

Patient history and physical examination will establish the


diagnosis in a great portion of cases. Lymphoscintigraphy
with Tc99m-labeled human serum albumin can
differentiate lymphedema from edemas of other etiology.
A normal lymphoscintigram excludes the diagnosis.

II

8-10

3 What is the
optimal early
management?

Manual lymphatic drainage massage and compression


therapy are the mainstay of early lymphedema treatment.
In late cases, an intensive regimen can control limb
swelling and help patients cope with the process.

II

11-13

1-4

(Continued)

PMPH_CH100.indd 794

5/22/2012 6:08:56 PM

Management of Lymphedema

795

(Continued)
Question

Answer

Levels of Grade of
References
Evidence Recommendation

4 What is the
best long-term
treatment?

The best long-term treatment regimen for chronic


lymphedema includes the routine use of compression
garments to control the limb volume variability and the
prevention of infections.

II

7, 13

5 Does
microvascular
surgery
provide relief?

Microvascular reconstructive surgery has been reported to


have good results. However, these operations on lymph
vessels <1 mm in size require high level of microsurgical
training, expertise, and commitment.

III

48

REFERENCES
1. Taylor MJ, Hoerauf A, Bockarie M. Lymphatic filariasis and
onchocerciasis. Lancet. 2010;376(9747):1175-1185. Epub 2010
Aug 23. Review.
2. Hashem FK, Ahmed S. Idiopathic scrotal lymphoedema in
Downs syndrome. Aust N Z J Surg. 1999;69(1):75-77.
3. Cormier JN, Askew RL, Mungovan KS, Xing Y, Ross MI, Armer
JM. Lymphedema beyond breast cancer: a systematic review and
meta-analysis of cancer-related secondary lymphedema. Cancer.
2010;116(22):5138-5149. doi:10.1002/cncr.25458. Review.
4. Irrthum A, Karkkainen MJ, Devriendt K, Alitalo K, Vikkula
M. Congenital hereditary lymphedema caused by a mutation
that inactivates VEGFR3 tyrosine kinase. Am J Hum Genet.
2000;67(2):295-301. Epub 2000 Jun 9.
5. McGuiness CL, Burnand KG. Lymphedema. In: Hallet JW, ed.
Comprehensive Vascular and Endovascular Surgery. 2nd ed. Philadelphia, PA: Mosby; 2009
6. Mak JW. Epidemiology of lymphatic filariasis. Ciba Found Symp.
1987;127:5-14. Review.
7. Browse NL. The diagnosis and management of primary lymphedema. J Vasc Surg. 1986;3(1):181-184.
8. Lee B, Andrade M, Bergan J, et al. International Union of Phlebology. Diagnosis and treatment of primary lymphedema.
Consensus document of the International Union of Phlebology
(IUP)-2009. Int Angiol. 2010;29(5):454-470.
9. Gloviczki P, Calcagno D, Schirger A, et al. Noninvasive
evaluation of the swollen extremity: experiences with 190

PMPH_CH100.indd 795

10.

11.

12.

13.

14.

15.
15.

16.

lymphoscintigraphic examinations. J Vasc Surg. 1989;9(5):683689; discussion 90.


Cambria RA, Gloviczki P, Naessens JM, Wahner HW. Noninvasive evaluation of the lymphatic system with lymphoscintigraphy: a prospective, semiquantitative analysis in 386 extremities.
J Vasc Surg. 1993;18(5):773-782.
Boris M, Weindorf S, Lasinski B, Boris G. Lymphedema reduction by noninvasive complex lymphedema therapy. Oncology
(Williston Park). 1994;8(9):95-106; discussion 109-110.
Boris M, Weindorf S, Lasinkski S. Persistence of lymphedema
reduction after noninvasive complex lymphedema therapy. Oncology (Williston Park). 1997;11(1):99-109; discussion 110, 113-114.
Stout Gergich NL, Pfalzer LA, McGarvey C, Springer B, Gerber LH, Soballe P. Preoperative assessment enables the early
diagnosis and successful treatment of lymphedema. Cancer.
2008;112(12):2809-2819.
Campisi C, Boccardo F, Zilli A, Maccio A, Napoli F. Long-term
results after lymphatic-venous anastomoses for the treatment of
obstructive lymphedema. Microsurgery. 2001;21:135-139.
Campisi C, Boccardo F. Lymphedema and microsurgery. Microsurgery. 2002;22:74-80.
Campisi C, Bellini C, Campisi C, Accogli S, Bonioli E, Boccardo
F. Microsurgery for lymphedema: clinical research and longterm results. Microsurgery. 2010;30(4):256-260.
Narushima M, Mihara M, Yamamoto Y, Iida T, Koshima I, Mundinger GS. The intravascular stenting method for treatment of
extremity lymphedema with multiconfiguration lymphaticovenous
anastomoses. Plastic Reconstructive Surg. 2010;125(3):935-943.

5/22/2012 6:08:56 PM

PMPH_CH100.indd 796

5/22/2012 6:08:56 PM

PART XV

SKIN AND SOFT TISSUE

PMPH_CH101.indd 797

5/22/2012 6:09:30 PM

PMPH_CH101.indd 798

5/22/2012 6:09:30 PM

CHAPTER 101

Malignant Melanoma
Robert E. Roses and Daniel F. Roses

INTRODUCTION

Incidence reporting in the United States is largely dependent


on data collected by the National Program of Cancer Registries
(NPCR) and the Surveillance, Epidemiology and End Results
(SEER) Program that incompletely surveyed the US population.5
Inherent problems in accurate reporting may be particularly relevant as they relate to thin lesions and in situ melanomas, both
of which are increasingly treated in outpatient facilities, often
nonhospital ambulatory and office facilities. Registry data have
reflected a disproportionate increase in the diagnosis of thinner
lesionsa trend that in fact might be amplified if not for the limitations of reporting.
Conclusion: Some of the inadequacies in reporting and data
retrieval make the magnitude of the increasing incidence of malignant melanoma over the past three decades difficult to verify. The
disparity in the increasing rates of incidence compared with mortality, however, may also reflect the contribution of a disproportionate increase in the diagnosis of thin lesions, a paradoxically
favorable trend.

Cancer of the skin is the most frequently diagnosed malignant


disease in the United States. Basal cell and squamous cell carcinomas account for 95% of newly diagnosed cases and are generally associated with an excellent prognosis. Malignant melanomas
account for most of the remaining cases and have a far greater
potential for lethal outcomes. Although less common, malignant
melanoma has always been a focus of intense interest due not only
to its propensity for local, regional, and systemic recurrence but
because of its accessibility for clinical observation and the study
of tumor oncogenesis and behavior. This interest in melanoma as
a prototype of malignant disease has been greatly magnified over
the past three decades by a perceived escalation in incidence.
1. Is malignant melanoma increasing in incidence?
The American Cancer Society reported that in 2010, excluding in
situ melanoma, 68,078 new cases of invasive malignant melanoma
would be diagnosed in the United States (39,481 in men, 28,597
in women). The lifetime risk in 2004 to 2006 was estimated to be
1 in 37 for white men and 1 in 57 for white womena five-fold
increase from 1981. The reported incidence trend was reappraised
in 2008, noting that while during the 1970s the incidence had increased rapidly by about 6% per year, between 1981 and 2000 the
rate of increase had slowed to 3% and since 2000 had remained
stable. Also noted was a decrease in death rate in the male white
population younger than 50 years by 3.0% per year, and in the
female white population younger than 50 years by 2.3% per year
since 1985.1 Mortality from melanoma has increased during the
last three decades, but to a lesser extent than has incidence and,
after 1990, had reached a plateau. Coincident with these trends has
been a disproportionate increase in the diagnosis of thin lesions.2
Whether the observed change in incidence is real, and mortality
has decreased as a result of earlier intervention, or is partly an
artifact of increasing diagnosis of lesions predominantly in their
early stage of evolution and of uncertain clinical significance,
remains a subject of controversy (Level 2c evidence).3,4

2. Does sunlight cause melanoma?


Proposed risk factors for the development of melanoma include
skin type (fair complexion), blue or green eyes, blonde or red
hair, a multiplicity of nevi particularly dysplastic nevi, a history
of nonmelanoma skin cancer, familial dysplastic nevi, xeroderma
pigmentosum, and giant congenital nevi. More attention has been
directed at the role of excessive exposure to sunlight than any
other environmental or genetic factor. The presumed causative
role of ultraviolet (UV) light has come from circumstantial evidence linking malignant melanoma with the intensity and length
of exposure, including a higher incidence in adults who were
excessively exposed or who developed severe sunburns in childhood and adolescence, and individuals with a history of sunburns
or intermittent recreational sunlight exposure. Such risk appears
magnified in those with light skin pigmentation, blonde or red
hair, and blue or green eyes.6-8 Evidence for these associations is
obviously retrospective and uncontrolled (Level 2 and Level 3
evidence). Moreover, the recollection of sunburns and its further
799

PMPH_CH101.indd 799

5/22/2012 6:09:30 PM

800

Surgery: Evidence-Based Practice

quantification by the criterion of blistering is of questionable


reliability. More problematic is the occurrence of only a single
melanoma in the great majority of affected individuals when multiple melanomas might be expected if sunlight were causative,
and the overwhelming occurrence on less frequently exposed and
obviously nonexposed areas (e.g., the palms and soles, and vulvar
and perianal skin). A meta-analysis considering the melanoma
risk from intermittent sun exposure did demonstrate a modest
relationship (odds ratios of 1.7) (Level 2a evidence).6 Indoor tanning has also been implicated as contributing to a rising incidence
of melanoma. A study of 1167 melanoma patients compared with
a nonmelanoma control population of 1101, found that whereas
63% of melanoma patients had frequented indoor tanning facilities, just over 51% of control groups had tanned indoors, suggesting an indoor tanning epidemic more than a reliable causative
effect (Level 3b evidence).9
Conclusion: Although limiting intermittent intense sun exposure and using UV protection is laudatory in decreasing the incidence of nonmelanoma skin cancer, as well as other sequelae, the
evidence implicating a direct relationship between excessive UV
exposure and melanoma is inconclusive at this time.
3. What are the clinical signs of malignant melanoma?
The escalating incidence of diagnosing malignant melanomas
early in their evolution no doubt reflects a growing awareness
among professionals and the lay public of the clinical indicators of the disease. However, this increasing clinical sensitivity
is likely associated with a decreasing specificity for identifying
the subtle nuances of in situ and thin lesions and differentiating them from a myriad of benign pigmented lesions. Clinical
awareness of malignant melanoma has undoubtedly expanded
the frequent biopsy and diagnosis of benign simulators and even
nonsuspicious lesions such as dysplastic nevi, Spitzs nevi, congenital nevi, and even keratoses and lentigos. The popularization
of the A (asymmetry), B (border irregularity), C (color variegation), D (diameter greater that 6 mm), and the recently added E
(evolving) criteria and the ugly duckling sign (a pigmented
lesion different from others where a multiplicity of pigmented lesions are present) although useful, may overlook wellcircumscribed nodular melanomas as well as those less than
6 mm in diameter. In a study of 94 children with malignant
melanoma a clinical diagnosis of any kind was made in only 39
and in only 1 was the diagnosis of melanoma suspected clinically (Level 2b evidence).10 Data supporting the use of diagnostic aids such as serial photography and dermoscopy are lacking.
Patient awareness and attention to new or changing pigmented
lesions is self-recommending as are regular total body cutaneous examinations in high-risk patients (e.g., those with a history
of melanoma, family history of melanoma, increased number of
nevi or freckling, the rare xeroderma pigmentosum, retinoblastoma, or Li Fraumeni syndrome). General population screenings
to decrease mortality are largely untested and may be further
complicated by the nuances and variations in pathologic interpretations of dysplastic nevi, atypical melanocytic hyperplasias,
and in situ melanomas. Nevertheless, retrospective analyses
have found that physician detected malignant melanomas were
far more frequently thin than those lesions detected by patients
(Level 2b evidence).11,12
Conclusion: Increasing awareness of malignant melanoma
by all physicians and the lay public through education is likely

PMPH_CH101.indd 800

to be the most effective measure in reducing mortality. Screening efficacy, however, remains unproven. Nevertheless, when
confined to the visible epidermaldermal junction, a malignant
melanomaalthough in a proliferative phasecannot traverse
lymphatic and vascular channels. Therefore, the identification of
an often flat melanoma with a minimally atypical appearance at
its in situ inception is a compelling and achievable goal.
4. How is the patient with a malignant melanoma definitively
diagnosed?
The great majority of patients with a primary malignant melanoma present with no other evidence of regional or disseminated disease. Microstaging of the primary lesion is, therefore,
a component of the histologic diagnosis, providing essential
prognostic information. The diagnostic biopsy should, whenever possible, conservatively encompass the entire lesion and
be followed by serial sectioning of the specimen. The maximal
thickness of the lesion can thereby be determined, as thickness
may vary over the breadth of the lesion. When excisional biopsy
is not possible because of large lesion diameter or anatomic limitations, selecting the clinically thickest part of the lesion for
an incisional or punch biopsy is appropriate. The 2009 staging
system for melanoma by the American Journal Committee on
Cancer (AJCC) was based on a multivariate analysis of 30,946
patients with stages I, II, and III and 7972 patients with stage IV
melanoma (Level 2c evidence).13 The staging recommendations
confi rmed primary melanoma thickness (P < .0001) and ulceration (P < .0001) as the essential variables for prognosis. For
patients whose lesions were 1.0 mm in thickness the 10-year
survival was 92%, with lesions 1.01 to 2.00 mm it was 80%, with
lesions 2.01 to 4.00 mm it was 63%, and with lesions >4.00 mm
it was 50%. Mitotic rate, defi ned as the number of mitoses/mm2,
was evaluated for 4861 patients with T1 melanomas and emerged
as an independent prognostic variable for T1 lesions (1.0-mm
thick) without ulceration. The presence of 1 mitosis/mm2
replaced Clarks level for classifying a lesion as T1b. Ten-year
survival rates were 97% versus 95% for T1 lesions of 0.1 to 0.50
mm without or with 1 mitosis/mm2 respectively, and 93% versus 87% for T1 lesions of 0.51 to 1.00 mm without or with 1
mitosis/mm2 respectively.
Conclusion: Total excisional biopsy, or selective incisional or
punch biopsy of a primary malignant melanoma, with appropriate serial sectioning and histologic microstaging (tumor thickness, ulceration, and mitotic rate) is an essential components of
diagnosis and surgical planning.
5. How is the patient with a malignant melanoma staged?
Once the diagnosis of a malignant melanoma is established and
the primary lesion is microstaged, further staging is dependent
on assessment of whether there are regional nodal or systemic
metastases. The presence of regional nodal metastases is associated with a worse prognosis in every thickness category except
T4b (>4.0 mm with ulceration). The presence of nodal micrometastases may be more sensitively assessed with immunohistochemical staining (S-100, HMB-45, and Melan-A/MART I) of
serially sectioned sentinel nodes than with standard hematoxylin and eosin staining.
In the AJCC analysis of 3307 stage II patients, a low volume threshold for determining the prognostic significance of

5/22/2012 6:09:30 PM

Malignant Melanoma

microscopic metastatic disease has not been established. The


present staging system allows the acceptance of at least one
of the melanoma associated antigenic markers (HMB-45,
Melan-A/MART I) as indicative of nodal disease even in the
absence of other microscopic criteria, as all micrometastatic disease is associated with a worse prognosis compared to patients
with comparable tumor thickness and an absence of such nodal
metastases (Level 2c evidence).13 Indeed, data from the AJCC
heighten the significance of regional nodal staging and the
greater sensitivity of sentinel node histology in the staging of
melanoma patients. Conversely, the presence of nodal metastases that are clinically detectable or histologically demonstrate
gross extension beyond the nodal capsule is associated with a
worse prognosis than those exclusively detected microscopically and contained within the nodal capsule. An increasing
number of metastatic nodes, one node (N1), 2-3 nodes (N2), or 4
or more nodes (N3), are also associated with worsening prognosis. In the same prognostic N2 group is the presence of regional,
satellite, or in-transit metastases without nodal metastases
(N2c) that were associated with 5- and 10-year survival rates of
69% and 52%. Patients who are diagnosed with clinical nodal
metastases without a defi nable primary melanoma are grouped
with patients with comparable nodal metastases and a known
primary lesion.13
The presence of distant metastases was associated with decreasing 1-year survival rates as evaluated in 7972 stage IV patients.
if they were located at distant skin, subcutaneous, and nodal sites
(MIa), lung (M1b), or other locations (M1c), with 1-year survival
rates of 62%, 53%, and 33% respectively (P < .0001). An elevated
serum lactic dehydrogenase (LDH) is associated with an even
worse prognosis in those with distant metastases and relegates all
stage IV patients, regardless of the distant metastatic site, into the
poorest prognostic category (Mlc). Although the importance of a
clinical history and physical examination for assessing possible
metastatic disease remains essential, the use of imaging modalities such as positron emission tomography/computed tomography
(PET/CT) scanning in the asymptomatic patients with apparent
stage I disease is unclear. Constantinidou and associates found
PET scanning of no value in a limited series of 29 sequential
patients with sentinel node metastases. Two patients had a false
positive study. No patient had an alteration in management with
a median follow-up of 23 months (Level 2b evidence).14 In contrast, in a series of 124 high-risk patients, Strobel and associates
were able to identify 53 patients with metastases from a heterogeneous high-risk group of 124 patients using PET/CT. The
sensitivity, specificity, and accuracy were 80%, 96%, and 91%,
which increased with dedicated CT interpretation to 98%, 94%,
and 96% (P = .016) (Level 2b evidence).15 The use of PET/CT for
high-risk asymptomatic patients is evolving and certainly may
be appropriate for inclusion in investigative protocols for adjuvant
treatment.
Conclusion: In the 2009 AJCC Melanoma Staging Database of
over 30,000 patients with localized disease, regional nodal status
is clearly the dominant predictor of survival as evaluated by sentinel lymphadenectomy or elective complete lymphadenectomy.
Although survival rates with distant metastases show some variability by specific site, their presence remains a dire prognostic indicator. Nodal metastases negatively impact 10-year survival rates by
at least 20% affirming the role of sentinel lymphadenectomy as a
low-risk means of accurately staging malignant melanoma patients

PMPH_CH101.indd 801

801

where microstaging of the primary lesion meets threshold levels for


performing the procedure.
6. What is the appropriate extent of excision for a primary
malignant melanoma?
Wide excisions for malignant melanoma were historically predicated on a perception of centrifugal lymphatic permeation from
the primary lesion. Prior to microstaging, wide excisions were
performed around virtually all malignant melanomas, which
were often only diagnosed in advanced stages of evolution. The
rarity of histologic microsatellitosis and local recurrences for
malignant melanomas in the more recent microstaging era,
particularly for lesions 2.0 mm, led to a critical reassessment
of the appropriateness of wide excisions beyond histologically
free margins. Prospective randomized trials have confi rmed
the efficacy of narrower margins (1.0 cm) for melanoma 2-mm
thick and the equivalence of 2- and 4-cm margins for intermediate thickness melanomas (14 mm) (Level 1b evidence).16,17 In
one trial a slightly increased rate of local recurrence was noted
for lesions 1 mm and 2 mm when margins were 1 cm versus
3 cm but this did not achieve statistical significance.16 Another
study also demonstrated increasing locoregional recurrence
for lesions >2 mm in thickness with 1-cm versus 3-cm margins.18 These data have led to recommendations of 1-cm margins for lesions 1 mm, 1 to 2 m for lesions 1 to 2 mm, and
2 cm or greater for lesions 2 mm. Although these guidelines
are directed at locoregional control, no prospective study has
demonstrated a correlation between the extent of resection
margin and survival nor has there been any study comparing patients treated with a defi ned margin compared to those
treated with a margin, however minimal, that is histologically
free of tumor.
Conclusion: Although excisions with negative margins are
clearly required to prevent local recurrence, the extent of margin
excision beyond histologically negative margins to ablate presumed local satellitosis and reduced local recurrence is undefined.
Moreover, the extent of resection margin has no proven impact
on survival. Certainly wide excision to prevent the potential morbidity of local recurrence is of obvious value. However, margin
restraint for cosmetically or functionally important sites would
appear justified by level 1 evidence supporting prudent 1- to 2-cm
margins, particularly when a primary closure can be achieved,
a skin graft avoided, and normal function and relatively normal
appearance maintained.
7. Is sentinel node biopsy always indicated for a patient with
malignant melanoma?
As the proportion of malignant melanomas diagnosed at an early
stage increased, and skepticism on the role of elective regional
node dissection persisted, sentinel lymphadenectomy emerged
as an appealing means of selectively limiting the extent of surgery for the approximately 85% or more of patients without nodal
metastases. It also allowed a more accurate identification of
patients with regional nodal micrometastases through detailed
histologic evaluation of sentinel lymph nodes with serial sectioning and immunohistochemical staining. Although the presence
or absence of regional nodal metastases has proven to be a powerful prognostic indicator, it is less clear whether their ablation has
an impact on survival.

5/22/2012 6:09:30 PM

802

Surgery: Evidence-Based Practice

The threshold for elective regional node dissection and, more


recently, sentinel lymphadenectomy has been limited in most
series to patients with lesions 1.0 mm. For lesions 1.0 mm,
the incidence of nodal metastases was 5% in a study by Wright
and associates of 631 patients with thin melanomas. Those
instances of positivity most often occurred with lesions in the
0.76- to 1.0-mm range.19 In a study of 184 patients with thin
melanomas, Ranieri and associates found that in the 0.76- to
1.0-mm range, rates were as high as 10.2% (Level 2b evidence). 20
In a study of 181 patients with thin lesions, Kesmodel showed
that the rate of sentinel node positivity rose to 12.3% when
mitotic rate was >0 (Level 2b evidence). 21 Presently a threshold
of 0.76 mm, and certainly with a mitotic rate of 1.0/mm2 and
ulceration, would appear appropriate. Although, the occasional
patient with a thin melanoma and ulceration and, conversely,
regression, may enter into this category, supporting data are not
available.
Prior to the development of sentinel lymphadenectomy, the
long-standing controversy in melanoma treatment over the therapeutic benefit of any form of nodal dissection was approached
with retrospective analyses, claiming a survival benefit of approximately 25% for patients having elective lymphadenectomy in
whom nodal metastases were only detected pathologically compared with those having therapeutic lymphadenectomy for
clinically apparent nodal disease (Level 2b evidence).22 Other
retrospective analyses demonstrated the benefit to be limited to
intermediate thickness lesions broadly being inclusive of those
with a thickness of 1.0 to 4.0 mm (Level 2b evidence).23 However, prospective randomized trials did not offer convincing evidence for the therapeutic benefit of elective lymphadenectomy,
finding either no benefit or, in the Intergroup Melanoma Surgical Trial that randomized patients to elective lymph node dissection or observation, a benefit for only a selective group of patients
with nonulcerated melanomas of 1.0 to 2.0 mm thickness (P = .03)
particularly of 60 years age (P = .00006) (Level 1b evidence).24
In a WHO trial of elective lymphadenectomy versus therapeutic
lymphadenectomy for clinically apparent nodal disease developing on observation in patients with lesions 1.5 mm, an advantage
in 5-year survival of 48.2% versus 26.4% (P = .04) was observed
for patients with microscopic metastases in the elective group
compared to the therapeutic group (Level 1b evidence). However,
this study selected from the group having elective dissection those
with clinically occult nodal disease and compared them with a
subset of patients who developed potentially more virulent nodal
macrometastases.25
The Multicenter Selective Lymphadenectomy Trial I (MSLT I)
was designed to overcome such limitations in the existing retrospective and prospective studies that addressed the issue
of regional nodal ablation. MSLT I randomized patients with
lesions 1.0 mm in thickness to sentinel lymphadenectomy and
complete lymphectomy if nodal metastases were detected, or
to observation and therapeutic lymphadenectomy if clinically
apparent nodal disease was detected during subsequent observation. The 5-year melanoma specific survival rates for patients
with intermediate thickness melanomas (1.23.5 mm) were similar in the sentinel node group and the observation group (87.1%
1.3% or 86.6% 1.6%). Predictably, the presence of metastases
in the sentinel node group was the most significant prognostic
indicator, the 5-year survival rate being 72.3% 4.6% for patients

PMPH_CH101.indd 802

with positive sentinel nodes compared to 90.2% 1.3% without


sentinel node metastases (P < .001). Among patients with nodal
metastases, the 5-year survival rate was higher among those
who underwent immediate lymphadenectomy than for those
whose lymphadenectomy was delayed (72.3% 4.6% vs. 52.4%
5.9%, P = .004). Notably, the incidence of nodal metastases
detected in sentinel nodes (16.0%) was no different from the
incidence of subsequent clinically detected nodal metastases
in the observation group (15.6%), suggesting a clinical equivalence for sentinel node micrometastases and clinically detected
nodal disease (Level 1b evidence).26 A subsequent analysis on
morbidity demonstrated an increase in lymphedema incidence
when complete lymph node dissection was delayed compared
to immediate complete lymph node dissection for patients with
metastases detected in sentinel lymph nodes (20.4% vs 12.4%,
P = .04). 27
Although the MSLT I clearly demonstrates the prognostic
significance of nodal metastases, as in previous retrospective and
prospective studies, an advantage can only be inferred for patients
having lymphadenectomy for microscopic compared to clinically
apparent disease. The absence of nodal metastases in 84.0% of the
sentinel lymphadenectomy group has limited the potential of the
MSLT I to evaluate the possible survival benefit of sentinel lymphadenectomy and immediate completion lymphadenectomy for
nodal micrometastases.
Conclusion: The therapeutic advantage of elective nodal dissection and, more recently, sentinel lymphadenectomy, even in
well-designed studies, although suggested, remains unproven for
patients with intermediate thickness malignant melanoma. The
prognostic value of detecting nodal metastases, however, has been
confirmed and is furthermore of value in stratifying patients for
adjuvant therapeutic trials.
8. Do all patients with sentinel lymph node metastases require
complete regional lymph node dissections?
Among patients with sentinel lymph node metastases, additional nonsentinel lymph node metastases are not detected in
the majority of patients. Indeed, in the MSLT I trial, the mean
number of positive nodes was 1.6 in the sentinel node positive
and completion lymphadenectomy group. Routine completion
lymphadenectomy would therefore potentially overtreat the
majority of sentinel node positive patients, for whom prognostic information has already been achieved. As the therapeutic
advantage of regional nodal dissection, although suggested, is
unproven, observation has been proposed as an alternative to
completion lymph node dissection for patients with sentinel
nodal metastases. Observation may include serial sonography
capable of detecting nodal enlargement before palpable change.
In a prospective study of 373 melanoma patients by Machet
et al., a sensitivity of 92.9% was observed with sonography compared to 71.4% for clinical examination alone (P = .02) (Level 1b
evidence). 28 If sonographically detected, however, the burden
of disease would still be greater than if detected only microscopically, and would have the same disadvantage to clinical
observation alone if ablating a microscopic volume of nodal
metastases were to have any therapeutic impact. Scheri and associates reported that in patients with only sentinel nodal metastases of less than 0.02 mm, there was a 12% risk of additional

5/22/2012 6:09:30 PM

Malignant Melanoma

nonsentinel nodal metastases (Level 2b evidence).29 Although


immediate completion lymphadenectomy has been advocated,
supporting data are not available and the issue of whether there
is a therapeutic impact remains perhaps even more problematic
than that of sentinel lymphadenectomy. The MSLT II trial is
designed to address this question.
Conclusion: Although there is no definitive evidence supporting an advantage to completion lymphadenectomy, as with
nodal dissection of any kind, it can only be inferred from data
supporting the ablation of microscopic versus macroscopic nodal
metastases, and from the possible decreased morbidity of the procedure compared to delayed completion lymph node dissection
for clinically apparent disease.
9. Is there a role for adjuvant radiotherapy in the treatment of
malignant melanoma?
Radiotherapy has been used in selected patients as an adjuvant
following surgical therapy. Indications may include patients at
high risk for locoregional recurrence including those with lesions
where the ability to achieve adequate margins is compromised by
anatomic location, extensive nodal disease, particularly in head
and neck locations, melanomas exhibiting neurotropic or desmoplastic histopathology, and thick melanomas with ulceration or
satellitosis. Data on the therapeutic value of adjuvant radiotherapy are limited due to variability in surgical treatment, radiation
techniques, and small cohorts of patients (Level 2 evidence).30,31
As in previous retrospective and nonrandomized studies, the
prospective Intergroup Randomized Trial of 250 patients at high
risk of regional recurrence after lymphadenectomy, comparing radiotherapy and observation, reported an improvement in
regional disease control (20 radiotherapy patients versus 34 observation patients recurred, P = .041). However, no benefit in survival
(median survival for radiotherapy patients was 2.6 years and for
observation patients was 3.9 years, P = .14) could be demonstrated
(Level 1b evidence).32
Conclusion: Adjuvant radiotherapy to achieve additional
locoregional disease control appears justified in selected instances.
No impact on survival has been proven.
10. When is regional limb perfusion indicated in the treatment
of malignant melanoma?
As the frequency of diagnosing melanoma in a prognostically
favorable early phase increases, the incidence of in-transit and satellite metastases has decreased. Such recurrences may be treated
by surgery and selective adjuvant radiotherapy. Persistent or multiple in-transit metastases limited to an extremity may be treated
by hypothermic isolated limb perfusion (ILP) or isolated limb
infusion (ILI). Overall response rates as high as 80% to 100% have
been reported using melphalan, tumor neurosis factor- (TNF-),
and interferon- (IFN-) alone or in combinations by hyperthermic isolated limb perfusion (Level 1b and 1c evidence).33-35
Although this approach is often effective at palliating unresectable disease, no impact on survival has been demonstrated. ILP
has also been studied as an adjuvant to surgery in patients with
resectable high-risk melanomas of the extremities in several randomized trials, the largest of which was conducted by the European Organization for Research and Treatment of Cancer and the
World Health Organization.36 This study randomized 832 patients

PMPH_CH101.indd 803

803

to wide excision and ILP with melphalan or wide excision alone.


A modest reduction in in-transit metastases was seen in the ILP
group (3.3% vs. 6.6%); however, this did not translate into a survival advantage and probably does not justify the morbidity of the
procedure (Level 1b evidence).37 More recently, ILI, using percutaneously placed catheters, has been explored as an alternative to
the more technically complex perfusion procedure. Prospectively
collected data suggest outcomes similar to those achieved with
ILP (Level 1c evidence).38
Conclusion: Hyperthermic ILP and, more recently, ILI is of
efficacy in selected patients with locoregional recurrent disease
that is otherwise not amenable to surgery. Long-term complete
responses were reported with hyperthermic ILP and LIL. However,
the benefit of ILP or ILI as an adjuvant to surgical resection of
a primary melanoma in the absence of detectable satellite or intransit disease is unproven.
11. When is adjuvant systemic therapy indicated for a patient
with malignant melanoma?
The tenacious resistance of metastatic melanoma to cytotoxic
agents has encouraged investigators to use other treatment modalities to improve survival for patients at high risk of recurrence.
The well-recognized evidence of host immune responsiveness to
melanoma has led to a variety of immunotherapeutic approaches
as adjuvant therapy. Nonspecific immunotherapy, with bacillus
Calmette-Guerin (BCG), failed to improve survival in high-risk
patients in several randomized trials (Level 1b evidence).39 Interferon , which can inhibit angiogenesis and tumor proliferation
and activate immune responsiveness by a variety of mechanisms,
has been evaluated as an adjuvant in numerous randomized trials
for high-risk melanoma patients beginning in 1996. In the initial
Eastern Cooperative Oncology Group (ECOG) study randomizing
high-risk patients to either observation or high-dose interferon ,
a benefit was demonstrated in recurrence-free survival (P = .0023)
and overall survival (P = .0237) (Level 1b evidence).40 However, a
meta-analysis by Wheatley et al. of 12 randomized trials of adjuvant interferon revealed an improvement in recurrence-free
survival but failed to identify a clear benefit in overall survival
(P = .1) (Level 1a evidence).41 Disease-free relapse appears most
apparent in patients with only nodal micrometastatic disease and
may be related as well to higher interferon dosage.
Clinical trials of vaccine therapy for melanoma alone or
in combination with a variety of immunomodulating agents
have yet to demonstrate a survival benefit in the adjuvant setting (Level 1b evidence).42,43 A randomized trial of 689 patients
treated with an allogeneic tumor vaccine versus observation
by the Southern Oncology Group for patients with intermediate thickness melanomas (1.54 mm) failed to demonstrate a
difference in survival although, in a subset of patients with the
HLA-A2 or HLA-C3 haplotypes, a benefit in disease-free and
overall survival was seen.43
Conclusion: High-risk patients may derive some benefit
from interferon although an improvement in overall survival
is unproven. Adjuvant therapy trials, many using immunotherapeutic agents, have yet to show a benefit in the treatment of highrisk melanoma patients. However, subsets of patients may benefit
thereby identifying valuable directions for further research and
clinical applications.

5/22/2012 6:09:30 PM

804

Surgery: Evidence-Based Practice

Clinical Question Summary


Question

Answer

1 Is malignant melanoma
increasing in
incidence?

The magnitude of the increasing incidence of malignant melanoma


over the past three decades is difficult to verify. Although the
true incidence of malignant melanoma may be increasing, the
disparity in the increasing rates of incidence compared with
mortality may reflect the contribution of a disproportionate
increase in the diagnosis of thin lesions.

1-4

2 Does sunlight cause


melanoma?

Although limiting intermittent intense sun exposure and using


UV protection is laudatory in decreasing the incidence of
nonmelanoma skin cancer, as well as other sequelae, the
evidence implicating a direct relationship between excessive UV
exposure and melanoma is inconclusive at this time.

5-9

3 What are the clinical


signs of malignant
melanoma?

The popular ABCDE criteria, although useful, may overlook wellcircumscribed nodular melanomas as well as those less than
6 mm in diameter. While increasing awareness of malignant
melanoma is likely to be the most effective measure in reducing
mortality, screening efficacy remains unproven.

10-12

4 How is the patient


with a malignant
melanoma definitively
diagnosed?

Total excisional biopsy, or selective incisional or punch biopsy of a


primary malignant melanoma, with appropriate serial sectioning
and histologic microstaging (tumor thickness, ulceration, and
mitotic rate) are essential components of diagnosis and surgical
planning.

13

5 How is the patient


with a malignant
melanoma staged?

The 2009 AJCC Melanoma Staging system, based on a multivariate


analysis of over 30,000 patients affirms regional nodal status
as the dominant predictor of survival as evaluated by sentinel
lymphadenectomy or elective complete lymphadenectomy.
Although survival rates with distant metastases show some
variability by specific site, their presence remains a dire
prognostic indicator.

13-15

6 What is the
appropriate extent of
excision of a primary
malignant melanoma?

Although excision with 1- to 2-cm margins to prevent the potential


morbidity of local recurrence is justified, no prospective
study has demonstrated the extent of margin resection to be
associated with survival rates.

16-18

7 Is sentinel lymph
node biopsy always
indicated for a patient
with malignant
melanoma?

The therapeutic benefit of sentinel lymph node dissection remains


unproven for patients with intermediate thickness malignant
melanoma. The prognostic value of detecting nodal metastases,
however, has been supported and confirmed, and is therefore
essential in stratifying patients for adjuvant therapeutic trials.

19-27

8 Do all patients with


sentinel lymph node
metastases require
regional lymph node
dissections?

An advantage to completion lymphadenectomy can only be


inferred from data supporting the ablation of microscopic versus
macroscopic nodal metastases, and from the possible decreased
morbidity of the procedure compared to delayed complete
lymph node dissection for clinically apparent disease.

28, 29

9 Is there a role for


adjuvant radiotherapy
in the treatment of
melanoma?

Adjuvant radiotherapy to achieve additional locoregional disease


control appears justified in selected instances, though an impact
on survival is unproven at the present time.

30-32

10 When is regional limb


perfusion indicated
in the treatment of
malignant melanoma?

Regional limb perfusion is of efficacy in selected patients with


locoregional recurrent disease otherwise not amenable to
surgery.

33-38

11 When is adjuvant
systemic therapy
indicated for a patient
with malignant
melanoma?

High-risk patients may derive some benefit from interferon


although an improvement in overall survival is unproven. Subsets
of patient may benefit from immunotherapy in an investigational
setting.

39-43

PMPH_CH101.indd 804

Grade of
References
Recommendation

5/22/2012 6:09:30 PM

Malignant Melanoma

REFERENCES
1. American Cancer Society, Cancer Facts and Figures. www.
cancer.org
2. Lipsker DM, Hedelin G, Heid E, et al. Striking increase of thin
melanomas contrasts with stable incidence of thick melanomas.
Arch Dermatol. 1999;135:1451-1456.
3. Welch HG, Woloshin S, Schwartz LM. Skin biopsy rates and
incidence of melanoma: population based ecological study. BMJ.
2005;331:481.
4. Jemal A, Devesa SS, Hartge P, et al. Recent trends in cutaneous
melanoma incidence among whites in the United States. J Natl
Cancer Inst. 2001;93:678-683.
5. Wingo PA, Jamison PM, Hiatt RA, et al. Building the infrastructure for nationwide cancer surveillance and controla comparison between the National Program of Cancer Registries (NPCR)
and the Surveillance, Epidemiology, and End Results (SEER) Program (United States). Cancer Causes Control. 2003;14:175-193.
6. Elwood JM, Jopson J. Melanoma and sun exposure: an overview
of published studies. Int J Cancer. 1997;73:198-203.
7. Bataille V, Winnett A, Sasieni P, et al. Exposure to the sun and
sunbeds and the risk of cutaneous melanoma in the UK: a casecontrol study. Eur J Cancer. 2004;40:429-435.
8. Veierod MB, Weiderpass E, Thorn M, et al. A prospective study
of pigmentation, sun exposure, and risk of cutaneous malignant
melanoma in women. J Natl Cancer Inst. 2003;95:1530-1538.
9. Lazovich D, Vogel RI, Berwick M, et al. Indoor tanning and risk
of melanoma: a case-control study in a highly exposed population. Cancer Epidemiol Biomarkers Prev. 2010;19:1557-1568.
10. Mones JM, Ackerman AB. Melanomas in prepubescent children:
review comprehensively, critique historically, criteria diagnostically, and course biologically. Am J Dermatopathol. 2003;25:
223-238.
11. Carli P, De Giorgi V, Palli D, et al. Dermatologist detection and
skin self-examination are associated with thinner melanomas:
results from a survey of the Italian Multidisciplinary Group on
Melanoma. Arch Dermatol. 2003;139:607-612.
12. Epstein DS, Lange JR, Gruber SB, et al. Is physician detection
associated with thinner melanomas? JAMA. 1999;281:640-643.
13. Balch CM, Gershenwald JE, Soong SJ, et al. Final version of 2009
AJCC melanoma staging and classification. J Clin Oncol. 2009;
27:6199-6206.
14. Constantinidou A, Hofman M, ODoherty M, et al. Routine
positron emission tomography and positron emission tomography/computed tomography in melanoma staging with positive sentinel node biopsy is of limited benefit. Melanoma Res.
2008;18:56-60.
15. Strobel K, Dummer R, Husarik DB, et al. High-risk melanoma:
accuracy of FDG PET/CT with added CT morphologic information for detection of metastases. Radiology. 2007;244:566-574.
16. Veronesi U, Cascinelli N, Adamus J, et al. Thin stage I primary
cutaneous malignant melanoma. Comparison of excision with
margins of 1 or 3 cm. N Engl J Med. 1988;318:1159-1162.
17. Balch CM, Urist MM, Karakousis CP, et al. Efficacy of 2-cm
surgical margins for intermediate-thickness melanomas (1 to
4 mm). Results of a multi-institutional randomized surgical trial.
Ann Surg. 1993;218:262-267; discussion 267-269.
18. Thomas JM, Newton-Bishop J, AHern R, et al. Excision margins in high-risk malignant melanoma. N Engl J Med. 2004;350;
757-766.
19. Wright BE, Scheri RP, Ye X, et al. Importance of sentinel lymph
node biopsy in patients with thin melanoma. Arch Surg. 2008;
143:892-899; discussion 899-900.

PMPH_CH101.indd 805

805

20. Ranieri JM, Wagner JD, Wenck S, et al. The prognostic importance of sentinel lymph node biopsy in thin melanoma. Ann Surg
Oncol. 2006;13:927-932.
21. Kesmodel SB, Karakousis GC, Botbyl JD, et al. Mitotic rate as a
predictor of sentinel lymph node positivity in patients with thin
melanomas. Ann Surg Oncol. 2005;12:449-458.
22. Morton DL, Wen DR, Wong JH, et al. Technical details of intraoperative lymphatic mapping for early stage melanoma. Arch
Surg. 1992;127:392-399.
23. Reintgen DS, Cox EB, McCarty KS, Jr., et al. Efficacy of elective
lymph node dissection in patients with intermediate thickness
primary melanoma. Ann Surg. 1983;198:379-385.
24. Balch CM, Soong S, Ross MI, et al. Long-term results of a multiinstitutional randomized trial comparing prognostic factors and
surgical results for intermediate thickness melanomas (1.0 to
4.0 mm). Intergroup Melanoma Surgical Trial. Ann Surg Oncol.
2000;7:87-97.
25. Cascinelli N, Morabito A, Santinami M, et al. Immediate or
delayed dissection of regional nodes in patients with melanoma
of the trunk: a randomised trial. WHO Melanoma Programme.
Lancet. 1998;351:793-796.
26. Morton DL, Thompson JF, Cochran AJ, et al. Sentinel-node
biopsy or nodal observation in melanoma. N Engl J Med. 2006;355:
1307-1317.
27. Faries MB, Thompson JF, Cochran A, et al. The impact on morbidity and length of stay of early versus delayed complete lymphadenectomy in melanoma: results of the Multicenter Selective
Lymphadenectomy Trial (I). Ann Surg Oncol. 2010;17:3324-3329.
28. Machet L, Nemeth-Normand F, Giraudeau B, et al. Is ultrasound lymph node examination superior to clinical examination in melanoma follow-up? A monocentre cohort study of 373
patients. Br J Dermatol. 2005;152:66-70.
29. Scheri RP, Essner R, Turner RR, et al. Isolated tumor cells in the
sentinel node affect long-term prognosis of patients with melanoma. Ann Surg Oncol. 2007;14:2861-2866.
30. Burmeister BH, Mark Smithers B, Burmeister E, et al. A prospective phase II study of adjuvant postoperative radiation therapy
following nodal surgery in malignant melanoma-Trans Tasman
Radiation Oncology Group (TROG) Study 96.06. Radiother
Oncol. 2006;81:136-142.
31. Mendenhall WM, Amdur RJ, Grobmyer SR, et al. Adjuvant radiotherapy for cutaneous melanoma. Cancer. 2008;112:1189-1196.
32. Henderson BB, Thompson JF, Di Iulio J, et al. Adjuvant radiotherapy and regional lymph node field control in melanoma
patients after lymphadenectomy: Results of an intergroup randomized trial (ANZMTG 01.02/TROG 02.01). J Clin Oncol.
2009;27:18s(suppl;abstr LBA9084).
33. Lienard D, Ewalenko P, Delmotte JJ, et al. High-dose recombinant tumor necrosis factor alpha in combination with interferon
gamma and melphalan in isolation perfusion of the limbs for
melanoma and sarcoma. J Clin Oncol. 1992;10:52-60.
34. Fraker DL, Alexander HR, Andrich M, et al. Treatment of
patients with melanoma of the extremity using hyperthermic
isolated limb perfusion with melphalan, tumor necrosis factor,
and interferon gamma: results of a tumor necrosis factor doseescalation study. J Clin Oncol. 1996;14:479-489.
35. Cornett WR, McCall LM, Petersen RP, et al. Randomized multicenter trial of hyperthermic isolated limb perfusion with melphalan alone compared with melphalan plus tumor necrosis
factor: American College of Surgeons Oncology Group Trial
Z0020. J Clin Oncol. 2006;24:4196-4201.
36. Lens MB, Dawes M. Isolated limb perfusion with melphalan
in the treatment of malignant melanoma of the extremities: a

5/22/2012 6:09:30 PM

806

Surgery: Evidence-Based Practice

systematic review of randomised controlled trials. Lancet Oncol.


2003;4:359-364.
37. Koops HS, Vaglini M, Suciu S, et al. Prophylactic isolated limb
perfusion for localized, high-risk limb melanoma: results of a multicenter randomized phase III trial. European Organization for
Research and Treatment of Cancer Malignant Melanoma Cooperative Group Protocol 18832, the World Health Organization Melanoma Program Trial 15, and the North American Perfusion Group
Southwest Oncology Group-8593. J Clin Oncol. 1998;16:2906-2912.
38. Kroon HM, Moncrieff M, Kam PC, et al. Outcomes following
isolated limb infusion for melanoma. A 14-year experience. Ann
Surg Oncol. 2008;15:3003-3013.
39. Agarwala SS, Neuberg D, Park Y, et al. Mature results of a phase
III randomized trial of bacillus Calmette-Guerin (BCG) versus
observation and BCG plus dacarbazine versus BCG in the adjuvant therapy of American Joint Committee on Cancer Stage I-III
melanoma (E1673): a trial of the Eastern Oncology Group. Cancer. 2004;100:1692-1698.

PMPH_CH101.indd 806

40. Kirkwood JM, Strawderman MH, Ernstoff MS, et al. Interferon


alfa-2b adjuvant therapy of high-risk resected cutaneous melanoma: the Eastern Cooperative Oncology Group Trial EST 1684.
J Clin Oncol. 1996;14:7-17.
41. Wheatley K, Ives N, Hancock B, et al. Does adjuvant interferonalpha for high-risk melanoma provide a worthwhile benefit? A
meta-analysis of the randomised trials. Cancer Treat Rev. 2003;
29:241-252.
42. Morton DL, Mozzillo N, Thompson JF, et al. An international,
randomized, phase III trial of bacillus Calmette-Guerin (BCG)
plus allogeneic melanoma vaccine (MCV) or placebo after complete resection of melanoma metastatic to regional or distant
sites. 2007 ASCO Annual Meeting Proceedings Part I. J Clin
Oncol. 2007;25:18S(June 20 Supplement), 8508.
43. Sondak VK, Liu PY, Tuthill RJ, et al. Adjuvant immunotherapy of
resected, intermediate-thickness, node-negative melanoma with
an allogeneic tumor vaccine: overall results of a randomized trial of
the Southwest Oncology Group. J Clin Oncol. 2002;20:2058-2066.

5/22/2012 6:09:30 PM

CHAPTER 102

Soft Tissue Sarcoma


Keila E. Torres and Raphael E. Pollock

INTRODUCTION

is increased in patients in whom radiotherapy is used to treat


lymphoma, breast, testicular, ovarian, prostate, lung, or other
cancer.5,6 Radiation-induced sarcomas account for approximately
0.5% to 5.5% of all sarcomas.7,8

1. What are soft tissue sarcomas?


Soft tissue sarcomas (STSs) are a group of relatively rare malignancies that exhibit remarkable diversity regarding anatomic site
and histopathologic characteristics. STSs consist of more than 50
histologic subtypes.1,2 Although somatic mesenchymal origin tissues account for more than two-thirds of total body weight, sarcomas, which are tumors of putative mesenchymal origin, comprise
less than 1% of adult solid malignancy. Despite their rarity, a thorough understanding of these tumors is important because patient
outcomes are in large part dependent on and determined by the
efficacy of initial management.
Answer: Sarcomas comprise a multitude of malignancies
rather than a single entity.

Chemicals
Exposure to PVC, thorotrast use in angiography, exposure to
inorganic arsenic, and treatment with androgenic-anabolic steroids all are associated with hepatic angiosarcoma (HAS).9-11

Host Factors
Chronic irritation of tissues is a potential cause of sarcomas.
Chronic upper extremity lymphedema after mastectomy with
radiation increases the risk for subsequent sarcoma development,
primarily angiosarcoma (Stewart-Treves syndrome).12,13 Severe
and chronic immunosuppression following solid organ transplantation also represents another risk factor for the development
of sarcomas.14,15

EPIDEMIOLOGY
2. How often do STSs occur?
The annual US incidence of bone sarcoma and STS is approximately
15,000 cases, of which 10,520 are adult STSs.3 As such, STS constitutes 1% of all new solid tumor cases in the United States annually
with a projected incidence of 2.5 to 3.5 cases per 100,000 US inhabitants per year and an overall mortality rate of 30% to 50%.3,4
Answer: STS constitutes 1% of all new solid tumor cases in
the United States. The annual incidence of adult STS is 10,520.

Genetic Factors
Sarcomas can arise in persons with certain genetic predispositions
to cancer development, including predispositions within family clusters, such as Li-Faumeni syndrome; germline mutations
in the P53 tumor suppressor gene are observed in these patients.
Somatic mutations in the P53 gene are observed in 30% to 60% of
STSs.16,17
Cytogenetic analysis of STSs has identified distinct chromosomal translocations that code for oncoproteins associated with
certain histologic subtypes. Examples of these gene rearrangements have been found in Ewing sarcoma (EWSFLI-1 fusion),18
alveolar rhabdomyosarcoma (PAX3FHKR fusion),19 clear-cell
sarcoma (EWSATF1 fusion),20 myxoid liposarcoma (TLSCHOP
fusion),21 desmoplastic small round-cell tumor (EWSWT1
fusion),22 and synovial sarcoma (SSXSYT fusion).23

ETIOLOGY
3. What factors are associated with an increased risk of STSs?

Environmental Etiologies
Radiation
Exposure to radiation has been recognized to induce sarcomas.
It has been demonstrated that the risk for sarcoma development
807

PMPH_CH102.indd 807

5/22/2012 6:10:00 PM

808

Surgery: Evidence-Based Practice

Answer: The following factors are associated with an increased risk of STS development: radiation, certain chemicals, chronic irritation, immunosuppression, and genetic factors such as
mutations or gene translocations.

PATHOLOGY
4. Are all sarcomas the same?
The World Health Organization has defined approximately 50 STS
histologic subtypes.2 STSs are subcategorized according to their
apparent line of differentiation, which also may be etiologically
and therapeutically relevant (e.g., liposarcoma [fat], leiomyosarcoma [smooth muscle], rhabdomyosarcoma [skeletal muscle], and
fibrosarcoma [connective tissue]). Conversely, some sarcomas lack
recognizable differentiation/derivation from any mesenchymal
tissues, for example, Ewing sarcoma and synovial sarcoma.
Answer: All sarcomas are not the same. These tumors are a
diverse group of malignancies arising predominantly from the
embryonic mesoderm.

CLINICAL PRESENTATION
5. How do most of STSs present?
Patients with nonosseous sarcomas typically present with a painless mass that has been growing slowly for months or years. The
size at presentation usually depends on the location of the tumor.
Retroperitoneal STSs and tumors in the proximal extremities can
become quite large before they are symptomatic. Infrequently,
patients may initially exhibit obstructive gastrointestinal symptoms or neurologic symptoms related to compression of the pelvic
nerves.
Answer: STS presents as a growing mass. Infrequently patients may present with symptoms such as pain or gastrointestinal
obstruction.
6. Where in the body are STSs most likely to develop?
STSs can arise at virtually all anatomic sites. Based on a database
of 5113 STS patients seen at the University of Texas MD Anderson
Cancer Centers Sarcoma Center from 1996 to 2005, one-third of
STSs originate in the lower extremity (32%).24 Retroperitoneal sarcomas comprise 15% to 20% of all STSs. Visceral sarcomas make
up an additional 24%. Thoracic and head and neck locations are
uncommon (9% and 6%, respectively).
Answer: Sarcomas can develop anywhere in the body. The
most frequent STS site of presentation is the lower extremity.

DIAGNOSIS
7. How are STSs diagnosed?

preferred imaging technique for evaluating retroperitoneal sarcomas to avoid GI motion artifacts.25 Magnetic resonance imaging
(MRI) is the preferred imaging modality for extremity sarcomas
because it can more accurately distinguish between bone, vascular structures, muscle groups, and tumor.26,27 Generally, patients
with low- and intermediate-grade tumors or high-grade tumors
5 cm or less in diameter require only a chest radiograph for satisfactory staging of the chest.28 However, patients with high-grade
tumors larger than 5 cm should undergo more thorough staging
of the chest by CT due to the increased risk of presentation with
established metastatic disease in this group of patients.29

Biopsy
Fine-needle aspiration biopsy can be used in conjunction with
clinical and imaging studies. The diagnostic accuracy of fineneedle aspiration biopsy-based findings in patients with primary
tumors ranges from 60% to 96%.30 Core-needle biopsy is an economical procedure that has the advantage of providing enough
tissue for use in several diagnostic tests (e.g., cytogenetic analysis,
flow cytometry, and electron microscopy). The diagnostic accuracy of core-needle biopsy-based findings is reported to be 93%.31
Incisional biopsies are usually performed only when fineneedle aspiration biopsy or core-needle biopsy specimens are nondiagnostic. The biopsy incision should be oriented longitudinally
along the extremity to allow a subsequent wide local excision that
includes the biopsy site, scar, and tumor. Ideally this type of biopsy
should be performed in a sarcoma treatment center by the same
surgeon who will perform the definitive surgery. The benefits of an
excisional biopsy rarely exceed those of other biopsy techniques.
Answer: Accurate diagnosis requires a complete history and
physical examination coupled with radiographic studies. The
optimal imaging modality is determined by the location of the
primary tumor. Biopsy is essential to precisely determine the histologic subtype that may be an important determinant of specific
treatment.

TREATMENT
8. How are STSs treated?
Management of STSs requires a multidisciplinary approach as is
articulated in various national guidelines and protocols.32 Accurate preoperative histologic diagnosis is critical in crafting the
optimal primary treatment strategy for a patient with STS. The
treatment goals are to ensure long-term survival, avoid local or
distant recurrence while maximizing patient function, and minimizing therapy-related morbidity. This is accomplished by various combinations of surgery, radiotherapy, and systemic therapies
such as chemotherapy. Treatment decisions are best made by a
sarcoma multidisciplinary team of surgeons, medical oncologists,
pathologists, radiologists, and radiation oncologists working prospectively in concerted collaboration.

Diagnostic Imaging
Radiologic imaging is critical for defining the local extent of a
tumor and staging the disease. Optimal imaging modalities applicable for a given primary tumor are anatomic site dependent. Contrast computed tomography (CT) of the abdomen and pelvis is the

PMPH_CH102.indd 808

Surgery
Tumor location, tumor size, depth of invasion, involvement of
contiguous anatomic structures, and patient performance status
are variables that can impact on the specific surgical resection

5/22/2012 6:10:00 PM

Soft Tissue Sarcoma

performed. Surgical excision, either alone or in combination with


radiation therapy if negative final pathologic margins are constrained by anatomic considerations, is the approach taken for
patients with small (<5 cm) primary tumors without evidence of
distant metastatic disease.
Limb-sparing surgery is usually feasible even for patients
with high-grade extremity sarcoma.33 Such approaches typically
involve wide local excision with the goal of resecting the tumor
with a 2-cm margin of surrounding normal tissue if possible. It has
been demonstrated that positive microscopic surgical margins are
a strong risk factor for local recurrence.34,35 However, neither surgical margin status nor local recurrence has been shown to impact
overall survival where the key determinant is the presence of lifethreatening distant metastases, especially in the lungs.36 Such
natural history considerations are especially important in the situation where achieving negative surgical margins would require
amputation or substantial functional compromise of an extremity
in that substantially less than 50% of patients with microscopically positive margins ultimately develop local recurrence

Radiotherapy
Prospective and retrospective studies have shown that radiotherapy improves local control in surgically resectable disease.37-43
However, it is still unresolved whether or not radiotherapy should
be used before versus after surgical resection. A meta-analysis
of five studies that compared preoperative and postoperative
radiotherapy in localized resectable STSs identified a lower risk
of recurrence in the preoperative radiotherapy group without a
demonstrable overall survival benefit.44 A multicenter prospective randomized study that included 190 patients demonstrated
an increased incidence of wound complications in those receiving
preoperative radiotherapy (35% vs. 17%), but this heightened preoperative radiotherapy risk was ameliorated by the aggressive use
of postresection microsurgical tissue transfer reconstruction.45
A randomized clinical trial performed by the National Cancer
Institute of Canada Sarcoma Clinical Trials Group demonstrated
that wound complications occurred more frequently in preoperative radiation therapy than postoperative radiation therapy
patients.46 Both groups achieved similar high rates of local control
and progression-free survival at 3 years.45,46 The long-term effects
of radiation therapy exposure include fibrosis, necrosis, edema,
fractures, and contractures; these potential complications should
be discussed in detail with patients before treatment.

Neoadjuvant Chemotherapy
There is little survival data to support the routine use of preoperative
chemotherapy. A combined trial performed by the European Organization for Research and Treatment of Cancer and the National
Cancer Institute of Canada randomly assigned 134 patients to
receive either preoperative doxorubicin- and ifosfamide-based
chemotherapy followed by surgery or surgery alone.47 At a median
follow-up time of 7.3 years, the 5-year overall survival rate for
patients who were not treated with neoadjuvant therapy was estimated to be 64% versus 65% for patients who received preoperative
chemotherapy in addition to surgical resection (P = .22).
A retrospective study evaluated the rates of radiological evident response to anthracycline-based preoperative chemotherapy

PMPH_CH102.indd 809

809

by review of the prechemotherapy and postchemotherapy crosssectional imaging studies.48 Twenty-eight percent of the patients
treated with preoperative doxorubicin-based neoadjuvant therapy
demonstrated complete to partial radiologic response. However,
30% of the patients demonstrated tumor progression during preoperative chemotherapy. Similar results were obtained in a separate
study that evaluated objective radiologic response rate on patients
treated with doxorubicin-based and ifosfamide-based neoadjuvant therapy, demonstrating an objective radiologic response
rate of 34% and an objective rate of disease progression of 26% on
patients treated with neoadjuvant therapy.49 Therefore, the question of whether neoadjuvant chemotherapy can improve overall
and disease-free survival remains unanswered for the time being,
and further prospective randomized trials are needed.

Postoperative Chemotherapy
The evidence pertaining to the use of chemotherapy in patients
with resectable STS remains controversial. Randomized trials of
postoperative chemotherapy suggest that such treatment does
not improve disease-free and overall survival in STS patients.50-52
Conversely, the Sarcoma Meta-analysis Collaboration demonstrated in a series of 1568 patients with resectable extremity STS
that overall survival rates were significantly better in patients who
received doxorubicin-based chemotherapy versus the group that
did not receive chemotherapy after local treatment.53
Answer: Management of STS requires a multidisciplinary
approach. Prospective and retrospective studies have shown
that radiotherapy improves local control in surgically resectable
disease. The evidence pertaining to the use of chemotherapy in
patients with resectable STS remains controversial.

SURVEILLANCE
A single center study of 2123 patients demonstrated that 66% of
STS recurrences developed within 2 years of initial surgery. Furthermore, 9% of recurrences occurred after a disease-free interval
of 5 years.54 Chest radiography is the recommended technique
for screening detection of pulmonary metastases; suspicious
lesions are further investigated with CT scanning of the chest.
Follow-up physical examinations and radiologic surveillance
studies (primary tumor site and chest) should be performed every
3 months during the first 2 to 3 years, the peak time interval for
STS recurrence. CT scan and MRI are useful diagnostic tools to
assess equivocal changes found during physical exam. It has been
demonstrated that MRI is a preferred diagnostic imaging modality for extremity STS to distinguish recurrence from postoperative changes,55 whereas CT scan is especially useful for detecting
recurrences in the abdomen, retroperitoneum, and pelvis.

TREATMENT OF RECURRENT SARCOMAS


Following treatment of the primary STS, 40% to 60% of patients
will develop local and/or distant recurrence. Most STSs that recur
do so within 2 to 3 years of the completion of treatment of the
primary lesion. These patients should be managed at specialized
multidisciplinary sarcoma centers. The evaluation must include a
full review of previous therapy including prior surgery, pathology

5/22/2012 6:10:00 PM

810

Surgery: Evidence-Based Practice

reports, chemotherapy regimens, and previous radiotherapy (including volume treated, dose, and energy of radiation).
An isolated local recurrence should be treated aggressively with
margin-negative resection. Acceptable rates of local control can be
achieved with function-preserving resection combined with additional radiation therapy, with or without chemotherapy.56 Several
studies have demonstrated that limb-sparing conservative surgery
is possible in the majority of patients with isolated locally recurrent
STS.56-58 Durable local control can be established with individualized
local treatment strategies. Therefore, aggressive multimodality limbsparing treatment approaches for these patients are recommended.
Patients with retroperitoneal or intra-abdominal sarcomas tend to
develop local rather than distant recurrences that are unfortunately
frequently not resectable.59 The preferred treatment for locally recurrent retroperitoneal tumors is surgical resection, if possible.

TREATMENT OF METASTATIC DISEASE


A study of 1240 patients found that nearly 50% of STS patients will
develop metastatic disease.60 Pulmonary metastases are the most
typical site of dissemination in patients with extremity sarcomas;
80% of metastases in such patients are to the lungs. Other sites
of metastasis are much less frequent and include bone (7%), liver
(4%), and lymph nodes (<4%).60 Complete pulmonary metastasectomy is useful for patients who have limited metastatic disease

burden and a long disease-free interval (i.e., >2 years) between primary resection and the appearance of dissemination. A 40% postmetastasectomy survival rate can be anticipated in these selected
individuals.59,61

NEW APPROACHES
Systemic treatment options for advanced sarcoma remain limited.
A new era in cancer therapy has arrived with the development of
agents targeting cancer-related proteins. These targeted therapies
selectively inhibit protein kinases, which mediate most signal transduction pathways in malignant cells and result in increased proliferation, evasion of apoptosis, invasion, and metastasis. A significant
proportion of sarcomas have distinctive molecular changes potentially amenable to targeted therapy. For example, aberrant signaling of mammalian target of rapamycin (mTOR) signaling has been
implicated in gastrointestinal stromal tumor, leiomyosarcoma, and
rhabdomyosarcoma.62 Agents targeting various tyrosine kinase
receptors, such as tyrosine-protein kinase Kit, platelet-derived
growth factor receptor, insulin like growth factor-1 receptor and
vascular endothelial growth factor receptor have all shown efficacy
to some degree in advanced sarcoma.63-65 Better understanding of
the unregulated signaling pathways in each histological sarcoma
subtype is critical as it has the potential to bring an unprecedented
paradigm shift to the treatment of soft tissue sarcomas.

Clinical Question Summary


Question

Answer

1 What are STSs?

Sarcoma represents a multitude of malignancies rather than a


single entity.

NA

1-2

2 How often do STSs


occur?

STS constitutes 1% of all new solid tumor cases in the United


States. The annual incidence of adult STS is 10,520.

NA

3 What factors are


associated with an
increased risk of STSs?

The following factors are associated with an increased risk of STSs:


radiation, chemicals, chronic irritation, immunosuppression,
and genetic factors such as mutations or gene translocations.

NA

5-23

4 Are all sarcomas the


same?

All sarcomas are not the same. These tumors are a diverse group
of malignancies arising predominantly from the embryonic
mesoderm.

NA

5 How do most of STSs


present?

STSs present as a growing mass. Infrequently, patients may present


with symptoms such as pain or gastrointestinal obstruction.

NA

6 Where in the body are


STSs most likely to
develop?

Sarcomas can develop anywhere in the body. The majority of STSs


present in the lower extremity.

NA

24

7 How are STSs


diagnosed?

Accurate diagnosis requires a complete history and physical exam


coupled with radiographic studies. The optimal imaging modality
is determined by the location of the primary tumor. Early biopsy
is essential to determine the specific histologic subtype that may
be an important determinant of specific treatment.

25-29

8 How are STSs treated?

Management of STS requires a multidisciplinary approach.


Prospective and retrospective studies have shown that
radiotherapy improves local control in surgically resectable
disease. The evidence pertaining to the use of chemotherapy in
patients with resectable STS remains controversial.

51-53,
56-59

PMPH_CH102.indd 810

Grade of
References
Recommendation

5/22/2012 6:10:00 PM

Soft Tissue Sarcoma

811

Evidence-Based Table
Study

Design

Intervention

Description

Results

#34

Prospective
Cohort
Study

Wide local excision


with negative
margins.

1041 patients with localized


extremity sarcomas included.
Median follow-up time 3.95 years.

Microscopically, positive surgical margins


were an adverse prognostic factor.

#39

RCT

Adjuvant radiotherapy
following resection
of high-grade
sarcoma lesion of
the extremity.

Following resection, 91 patients with


high-grade sarcoma lesions of the
extremities were randomized; 47
to receive adjuvant radiotherapy
and 44 to not receive adjuvant
radiotherapy.

A highly significant decrease


(P = .0028) in the probability of local
recurrence was seen with radiation.
No difference in overall survival was
shown.

#40

RCT

Adjuvant
brachytherapy
(BRT).

164 patients were randomized


intraoperatively to receive either
adjuvant brachytherapy (BRT)
or no further therapy after
complete resection of STS of the
extremity or superficial trunk.

Patients with high-grade lesions had


local control rates of 89% (BRT) and
66% (no BRT) (P = .0025).

#45-46

RCT

Preoperative versus
postoperative
radiotherapy in
STS of the limbs.

94 patients randomized to receive


preoperative radiotherapy (50 Gy
in 25 fractions) and 96 to receive
postoperative radiotherapy
(66 Gy in 33 fractions).

Wound complications occurred more


frequently in preoperative radiation
therapy (35%) than postoperative
radiation therapy (17%) patients
(P = .01).

#53

Meta-analysis

Adjuvant
chemotherapy.

1568 patients from 14 trials of


doxorubicin-based adjuvant
chemotherapy were included
(median follow-up 9.4 years).

Hazard ratios of 0.73 (95% CI 0.56


0.94; P = .016) for local RFI, 0.70 (95%
CI 0.570.85, P = .0003) for distant
RFI, and 0.75 (95% CI 0.640.87,
P = .0001) for overall recurrencefree survival. For overall survival the
hazard ratio of 0.89 (0.761.03) was
not significant (P = .12) at 10 years.

The intervention described refers to STS.

REFERENCES
1. Clark MA, Fisher C, Judson I, Thomas JM. Soft-tissue sarcomas
in adults. N Engl J Med. 2005;353:701-711.
2. Fletcher C, Unii K, Mertens F. World Health Organization Classification of Tumors. Pathology and Genetics. Tumors of Soft Tissue and Bone. Lyon: IARC Press; 2002.
3. Jemal A, Siegel R, Xu J, Ward E. Cancer statistics, 2010. CA Cancer J Clin. 2010;60:277-300.
4. Toro JR, Travis LB, Wu HJ, Zhu K, Fletcher CD, Devesa SS. Incidence patterns of soft tissue sarcomas, regardless of primary site, in
the surveillance, epidemiology and end results program, 1978-2001:
an analysis of 26,758 cases. Int J Cancer. 2006;119:2922-2930.
5. Brady MS, Gaynor JJ, Brennan MF. Radiation-associated sarcoma of bone and soft tissue. Arch Surg. 1992;127:1379-1385.
6. Zahm SH, Fraumeni JF, Jr. The epidemiology of soft tissue
sarcoma. Semin Oncol. 1997;24:504-514.
7. Davidson T, Westbury G, Harmer CL. Radiation-induced softtissue sarcoma. Br J Surg. 1986;73:308-309.
8. Huvos AG, Woodard HQ, Cahan WG, et al. Postradiation osteogenic sarcoma of bone and soft tissues. A clinicopathologic study
of 66 patients. Cancer. 1985;55:1244-1255.
9. Falk H, Herbert J, Crowley S, et al. Epidemiology of hepatic
angiosarcoma in the United States: 1964-1974. Environ Health
Perspect. 1981;41:107-113.

PMPH_CH102.indd 811

10. Waxweiler RJ, Stringer W, Wagoner JK, Jones J, Falk H, Carter C.


Neoplastic risk among workers exposed to vinyl chloride. Ann N
Y Acad Sci. 1976;271:40-48.
11. Da Silva Horta J, Da Motta C, Tavares MH. Thorium dioxide
effects in man. Epidemiological, clinical, and pathological studies (experience in Portugal). Environ Res. 1974;8:131-159.
12. Stewart FW, Treves N. Lymphangiosarcoma in postmastectomy
lymphedema; a report of six cases in elephantiasis chirurgica.
Cancer. 1948;1:64-81.
13. Woodward AH, Ivins JC, Soule EH. Lymphangiosarcoma arising in
chronic lymphedematous extremities. Cancer. 1972;30:562-572.
14. Eltom MA, Jemal A, Mbulaiteye SM, Devesa SS, Biggar RJ.
Trends in Kaposis sarcoma and non-Hodgkins lymphoma incidence in the United States from 1973 through 1998. J Natl Cancer
Inst. 2002;94:1204-1210.
15. Penn I. Sarcomas in organ allograft recipients. Transplantation.
1995;60:1485-1491.
16. Das P, Kotilingam D, Korchin B, et al. High prevalence of p53
exon 4 mutations in soft tissue sarcoma. Cancer. 2007;109:
2323-2333.
17. Hieken TJ, Das Gupta TK. Mutant p53 expression: a marker of
diminished survival in well-differentiated soft tissue sarcoma.
Clin Cancer Res. 1996;2:1391-1395.
18. May WA, Gishizky ML, Lessnick SL, et al. Ewing sarcoma 11;22
translocation produces a chimeric transcription factor that

5/22/2012 6:10:00 PM

812

19.

20.

21.

22.

23.

24.

25.
26.
27.

28.

29.

30.
31.

32.
33.
34.

35.

36.

37.

38.

Surgery: Evidence-Based Practice

requires the DNA-binding domain encoded by FLI1 for transformation. Proc Natl Acad Sci U S A. 1993;90:5752-5756.
Sublett JE, Jeon IS, Shapiro DN. The alveolar rhabdomyosarcoma
PAX3/FKHR fusion protein is a transcriptional activator. Oncogene. 1995;11:545-552.
Pellin A, Monteagudo C, Lopez-Gines C, Carda C, Boix J,
Llombart-Bosch A. New type of chimeric fusion product between
the EWS and ATFI genes in clear cell sarcoma (malignant melanoma of soft parts). Genes Chromosomes Cancer. 1998;23:358-360.
Crozat A, Aman P, Mandahl N, Ron D. Fusion of CHOP to a
novel RNA-binding protein in human myxoid liposarcoma.
Nature. 1993;363:640-644.
Gerald WL, Rosai J, Ladanyi M. Characterization of the genomic
breakpoint and chimeric transcripts in the EWS-WT1 gene
fusion of desmoplastic small round cell tumor. Proc Natl Acad
Sci U S A. 1995;92:1028-1032.
de Leeuw B, Berger W, Sinke RJ, et al. Identification of a yeast
artificial chromosome (YAC) spanning the synovial sarcomaspecific t(X;18)(p11.2;q11.2) breakpoint. Genes Chromosomes
Cancer. 1993;6:182-189.
Lahat G, Lazar A, Lev D. Sarcoma epidemiology and etiology:
potential environmental and genetic factors. Surg Clin North Am.
2008;88:451-481, v.
Heslin MJ, Smith JK. Imaging of soft tissue sarcomas. Surg Oncol
Clin N Am. 1999;8:91-107.
Hanna SL, Fletcher BD. MR imaging of malignant soft-tissue
tumors. Magn Reson Imaging Clin N Am. 1995;3:629-650.
Demas BE, Heelan RT, Lane J, Marcove R, Hajdu S, Brennan MF.
Soft-tissue sarcomas of the extremities: comparison of MR and
CT in determining the extent of disease. AJR Am J Roentgenol.
1988;150:615-620.
Fleming JB, Cantor SB, Varma DG, et al. Utility of chest computed tomography for staging in patients with T1 extremity soft
tissue sarcomas. Cancer. 2001;92:863-868.
Porter GA, Cantor SB, Ahmad SA, et al. Cost-effectiveness of
staging computed tomography of the chest in patients with T2
soft tissue sarcomas. Cancer. 2002;94:197-204.
de Saint Aubain Somerhausen N, Fletcher CD. Soft-tissue sarcomas: an update. Eur J Surg Oncol. 1999;25:215-220.
Dupuy DE, Rosenberg AE, Punyaratabandhu T, Tan MH,
Mankin HJ. Accuracy of CT-guided needle biopsy of musculoskeletal neoplasms. AJR Am J Roentgenol. 1998;171:759-762.
Demetri GD, Antonia S, Benjamin RS, et al. Soft tissue sarcoma.
J Natl Compr Canc Netw. 2010;8:630-674.
Limb-Sparing Treatment of Adult Soft-Tissue Sarcomas and
Osteosarcomas. In: Online NCS (ed.). 1984;5:1-7.
Pisters PW, Leung DH, Woodruff J, Shi W, Brennan MF.
Analysis of prognostic factors in 1,041 patients with localized
soft tissue sarcomas of the extremities. J Clin Oncol. 1996;14:
1679-1689.
Herbert SH, Corn BW, Solin LJ, Lanciano RM, Schultz DJ,
McKenna WG, et al. Limb-preserving treatment for soft tissue
sarcomas of the extremities. The significance of surgical margins. Cancer. 1993;72:1230-1238.
Tanabe KK, Pollock RE, Ellis LM, Murphy A, Sherman N, Romsdahl MM. Influence of surgical margins on outcome in patients
with preoperatively irradiated extremity soft tissue sarcomas.
Cancer. 1994;73:1652-1659.
Lindberg RD, Martin RG, Romsdahl MM, Barkley HT, Jr. Conservative surgery and postoperative radiotherapy in 300 adults
with soft-tissue sarcomas. Cancer. 1981;47:2391-2397.
Suit HD, Russell WO. Radiation therapy of soft tissue sarcomas.
Cancer. 1975;36:759-764.

PMPH_CH102.indd 812

39. Yang JC, Chang AE, Baker AR, et al. Randomized prospective
study of the benefit of adjuvant radiation therapy in the treatment of soft tissue sarcomas of the extremity. J Clin Oncol. 1998;
16:197-203.
40. Pisters PW, Harrison LB, Leung DH, Woodruff JM, Casper ES,
Brennan MF. Long-term results of a prospective randomized
trial of adjuvant brachytherapy in soft tissue sarcoma. J Clin
Oncol. 1996;14:859-868.
41. Suit HD, Spiro I. Role of radiation in the management of adult
patients with sarcoma of soft tissue. Semin Surg Oncol. 1994;
10:347-356.
42. Barkley HT, Jr., Martin RG, Romsdahl MM, Lindberg R, Zagars
GK. Treatment of soft tissue sarcomas by preoperative irradiation and conservative surgical resection. Int J Radiat Oncol Biol
Phys. 1988;14:693-699.
43. Wilson AN, Davis A, Bell RS, et al. Local control of soft tissue
sarcoma of the extremity: the experience of a multidisciplinary
sarcoma group with definitive surgery and radiotherapy. Eur J
Cancer. 1994;30A:746-751.
44. Al-Absi E, Farrokhyar F, Sharma R, et al. A systematic review
and meta-analysis of oncologic outcomes of pre- versus postoperative radiation in localized resectable soft-tissue sarcoma. Ann
Surg Oncol. 2010;17:1367-1374.
45. OSullivan B, Davis AM, Turcotte R, et al. Preoperative versus
postoperative radiotherapy in soft-tissue sarcoma of the limbs: a
randomised trial. Lancet. 2002;359:2235-2241.
46. Davis AM, OSullivan B, Turcotte R, et al. Late radiation morbidity following randomization to preoperative versus postoperative radiotherapy in extremity soft tissue sarcoma. Radiother
Oncol. 2005;75:48-53.
47. Gortzak E, Azzarelli A, Buesa J, et al. A randomised phase II
study on neo-adjuvant chemotherapy for high-risk adult softtissue sarcoma. Eur J Cancer. 2001;37:1096-1103.
48. Pisters PW, Patel SR, Varma DG, et al. Preoperative chemotherapy for stage IIIB extremity soft tissue sarcoma: long-term results
from a single institution. J Clin Oncol. 1997;15:3481-3487.
49. Meric F, Hess KR, Varma DG, et al. Radiographic response to
neoadjuvant chemotherapy is a predictor of local control and
survival in soft tissue sarcomas. Cancer. 2002;95:1120-1126.
50. Frustaci S, De Paoli A, Bidoli E, et al.. Ifosfamide in the adjuvant therapy of soft tissue sarcomas. Oncology. 2003;65(Suppl 2):
80-84.
51. Brodowicz T, Schwameis E, Widder J, et al. Intensified Adjuvant
IFADIC Chemotherapy for Adult Soft Tissue Sarcoma: A Prospective Randomized Feasibility Trial. Sarcoma. 2000;4:151-160.
52. Petrioli R, Coratti A, Correale P, et al. Adjuvant epirubicin with
or without Ifosfamide for adult soft-tissue sarcoma. Am J Clin
Oncol. 2002;25:468-473.
53. Sarcoma Meta-analysis Collaboration. Adjuvant chemotherapy
for localised resectable soft-tissue sarcoma of adults: metaanalysis of individual data. Sarcoma Meta-analysis Collaboration. Lancet. 1997;350:1647-1654.
54. Stojadinovic A, Leung DH, Allen P, Lewis JJ, Jaques DP, Brennan
MF. Primary adult soft tissue sarcoma: time-dependent influence of prognostic variables. J Clin Oncol. 2002;20:4344-4352.
55. Vanel D, Shapeero LG, De Baere T, et al. MR imaging in the
follow-up of malignant and aggressive soft-tissue tumors: results
of 511 examinations. Radiology. 1994;190:263-268.
56. Singer S, Antman K, Corson JM, Eberlein TJ. Long-term salvageability for patients with locally recurrent soft-tissue sarcomas.
Arch Surg. 1992;127:548-553; discussion 53-54.
57. Midis GP, Pollock RE, Chen NP, et al. Locally recurrent soft tissue sarcoma of the extremities. Surgery. 1998;123:666-671.

5/22/2012 6:10:00 PM

Soft Tissue Sarcoma

58. Karakousis CP, Proimakis C, Rao U, Velez AF, Driscoll DL. Local
recurrence and survival in soft-tissue sarcomas. Ann Surg Oncol.
1996;3:255-260.
59. Whooley BP, Mooney MM, Gibbs JF, Kraybill WG. Effective
follow-up strategies in soft tissue sarcoma. Semin Surg Oncol.
1999;17:83-87.
60. Potter DA, Glenn J, Kinsella T, et al. Patterns of recurrence in
patients with high-grade soft-tissue sarcomas. J Clin Oncol. 1985;
3:353-366.
61. van Geel AN, Pastorino U, Jauch KW, et al. Surgical treatment
of lung metastases: the European Organization for Research and
Treatment of Cancer-Soft Tissue and Bone Sarcoma Group study
of 255 patients. Cancer. 1996;77:675-682.

PMPH_CH102.indd 813

813

62. Okuno S. Mammalian target of rapamycin inhibitors in sarcomas. Curr Opin Oncol. 2006;18:360-362.
63. McAuliffe JC, Hunt KK, Lazar AJ, et al. A randomized, phase II
study of preoperative plus postoperative imatinib in GIST: evidence of rapid radiographic response and temporal induction of
tumor cell apoptosis. Ann Surg Oncol. 2009;16:910-919.
64. Kurzrock R, Patnaik A, Aisner J, et al. A phase I study of weekly
R1507, a human monoclonal antibody insulin-like growth factor-I receptor antagonist, in patients with advanced solid tumors.
Clin Cancer Res. 2009;16:2458-2465.
65. Maki RG, DAdamo DR, Keohan ML, et al. Phase II study of
sorafenib in patients with metastatic or recurrent sarcomas.
J Clin Oncol. 2009;27:3133-3140.

5/22/2012 6:10:00 PM

CHAPTER 103

Solitary Neck Mass


Luc G. T. Morris and Ashok R. Shaha

BACKGROUND

The most common neoplastic processes presenting as a neck


mass are squamous cell carcinoma (SCC), lymphoma, thyroid
neoplasms (most of which are benign), salivary tumors, paragangliomas, schwannomas, lipomas, and sarcomas. In adults, the
leading diagnosis for a solitary neck mass is metastatic SCC from
a mucosal primary site, such as the oral cavity, oropharynx, larynx, hypopharynx, or nasopharynx.
In children and young adults, if an infectious etiology for
lymphadenopathy is suspected, observation or an empiric course
of antibiotics can be considered. Persistence of disease should
prompt imaging. In children, ultrasound is commonly used to
avoid radiation exposure. In adults, because malignant disease is
suspected, workup begins with physical examination including
flexible endoscopy and imaging, generally computed tomography
(CT) or magnetic resonance imaging (MRI), as these modalities
image the entire neck and potential primary sites for metastatic
disease. After imaging, the initial approach for tissue diagnosis is
fine-needle aspiration (FNA) for cytologic analysis.1,4,5
The workup of metastatic SCC of unknown primary origin
is discussed later in further detail. Metastatic adenocarcinoma
of unknown primary origin is less common. Adenocarcinoma
metastases in the upper neck are most likely to be of head and
neck salivary origin, whereas metastases in the lower neck are
most likely to originate from primary tumors below the clavicles
(e.g., breast, lung, stomach, colon, rectum, prostate, or ovary).
Other tumors identified in the neck may be challenging to
the pathologist, and many are classified as poorly differentiated
or undifferentiated carcinoma. These entities include lymphomas, medullary thyroid carcinoma, melanoma, paraganglioma,
small cell neuroendocrine carcinomas, sarcomas, and carcinoid
tumors. In these cases, immunohistochemistry becomes invaluable in first identifying cell lineage as epithelial, mesenchymal,
hematopoietic, or melanocytic and then narrowing the diagnosis
to a specific tumor type. In some cases, open biopsy may be necessary to obtain sufficient tissue for diagnosis. Accurate diagnosis
of neck masses demands close cooperation between the surgeon,
pathologist, and radiologist.

Neck masses are common in the adult and pediatric populations,


and represent a wide range of differential diagnoses in each group.
In many cases, a self-limited infectious or inflammatory process
may be responsible, but the differential diagnosis also includes
potentially surgical diseases such as neoplastic or congenital processes. Most pediatric neck masses represent benign lesions, such
as inflammatory disease or congenital masses. In contrast, these
entities are uncommon in adults, in whom most neck masses represent malignant disease.1
Evaluation begins with a careful history and physical examination. Age is the most important initial factor. Critical elements
of the medical history include the time course for development of
the mass, antecedent infection, possible immunodeficiency, and
prior travel, trauma, irradiation or surgery. Symptoms to evaluate include fever, pain, weight loss, swallowing or breathing
difficulty, and ear pain. Physical examination requires careful
examination of the head and neck, including the skin of the face
and scalp, and all mucosal surfaces of the upper aerodigestive
tract. In adults, endoscopic examination of the nasopharynx,
larynx, and hypopharynx is a necessary part of the physical
examination.
In children, inflammatory processes are at the top of the
list. These include lymphadenitis (commonly caused by Streptococcus and Staphylococcus), granulomatous processes such as
tuberculosis, atypical mycobacteria, and salivary processes such
as sialadenitis or salivary stones.2,3 Congenital masses in children and young adults can often be differentiated on physical
examination. The list includes torticollis (pseudotumor of the
sternocleidomastoid muscle in an infant), thyroglossal duct
cysts (located in the midline), branchial apparatus abnormalities (most commonly second branchial cleft cysts, located along
the anterior border of the sternocleidomastoid muscle), dermoid cysts and teratomas, lymphatic malformations, and vascular malformations such as hemangiomas and arteriovenous
malformations.
814

PMPH_CH103.indd 814

5/22/2012 6:10:33 PM

Solitary Neck Mass

1. What is the risk of malignancy in a neck mass?


In children, palpable lymph nodes are common, with prevalence
estimated at 55%.6 Even in pediatric patients with persistent masses
who go on to have fine-needle biopsy, the risk of malignancy remains
low, estimated at between 4% and 11%.7,8 In one recent large cohort
study of pediatric patients undergoing FNA for persistent lymphadenopathy, 86% of lesions were benign, most commonly reactive
lymphoid hyperplasia (49% of aspirates), granulomatous disease
(9%), and benign thyroid lesions (7%). Malignant lesions represented 4%, and indeterminate aspirates, 3% (Level 2b evidence).8
In adults, neck masses are less prevalent, but much more
likely to be malignant. A large population-based cohort study in
the United States established that 83.5% of adult neck masses were
neoplastic, of which 75.9% were malignant.9 A similar large study
in France established that 82% of adult neck masses were malignant.10 The most common malignancy in both cohorts was metastatic squamous cell carcinoma from a head and neck mucosal
primary site (Level 2b evidence).
Age is the most significant factor predicting malignancy in a
neck mass. In a retrospective analysis of adults undergoing open
biopsy for neck masses, multivariate analysis of patient and tumor
factors identified age as the only significant factor predictive of
malignancy11 (Level 2c evidence).
Answer: Few pediatric neck masses are malignant. In contrast, the majority of neck masses in adults represent metastatic
cancer, most commonly SCC from a head and neck primary site.
2. How accurate is FNA biopsy?
In children with persistent suspicious lymphadenopathy, and in
adults with neck masses, the first approach for tissue diagnosis is
generally FNA for cytologic analysis.
Several large retrospective cohort studies of FNA in the
workup of pediatric neck masses have confirmed high rates of
accuracy, with sensitivity and specificity for malignancy in the
range of 94% to 97% (Level 2b evidence).12-14
In adults, similar high rates of accuracy have been reported in
several large cohort studies, ranging from 94% to 97%.15-18 Retrospective analyses have demonstrated that performance of needle
biopsy by a cytopathologist decreases the rate of indeterminate
aspirates, and increases accuracy (Level 2b evidence).15,18
Answer: FNA for cytologic analysis is highly accurate in
identifying malignant processes.
3. Does metastatic cervical lymph node location predict the
site of primary tumor?
If metastatic cancer is confirmed on needle or open biopsy of a neck
mass, a comprehensive head and neck examination is required,
with a goal of identifying the site of primary tumor. The patterns
of nodal spread in head and neck squamous cell carcinoma were
first described in a large cohort of 1081 patients by Shah.19 These
data established that oral cavity tumors most commonly metastasized to levels I-III, whereas oropharyngeal, laryngeal, and
hypopharyngeal tumors most commonly spread to levels II-IV.
Accordingly, a level I neck mass would direct attention to the oral
cavity examination, whereas a level IV neck mass would emphasize the importance of thorough evaluation of the oropharynx,
larynx, and hypopharynx. These findings have been confirmed in
a contemporary cohort, which reported that metastatic disease in
level I is usually attributable to oral cavity tumors, uncommonly

PMPH_CH103.indd 815

815

oropharyngeal tumors, and rarely laryngeal or hypopharyngeal


tumors.20 Isolated level V (posterior triangle) nodal metastasis
is uncommonly associated with these primary sites but is commonly seen in nasopharyngeal carcinoma.21 Central compartment (level VI) nodal disease is most commonly attributable to
thyroid disease. The presence of nodal metastasis in the parotid
gland or external jugular basin is highly suggestive of cutaneous
malignancy of the face or scalp (Level 2b evidence).22
Cystic neck masses may represent benign processes such as
branchial cleft cysts or nodal metastases. In adults, lateral cystic masses most commonly represent metastatic SCC originating in the mucosa of Waldeyers ring (tonsil, base of tongue, and
nasopharynx).23 In the past, these were believed to be branchial
cleft carcinomas, an entity that is now believed to be nonexistent
or exceedingly rare. Metastatic papillary thyroid carcinoma is
another consideration. In contrast to solid cervical nodes, cytologic assessment by FNA is less reliable in cystic masses, with
false-negative rates ranging from 27% to 50%.23,24 Accordingly, a
high degree of suspicion for cancer must be maintained in adult
patients with lateral cystic neck masses, and the finding of SCC
should prompt a thorough evaluation for the primary site of
the tumor. Most cystic metastasis of SCC are likely to represent
human papillomavirus (HPV)-positive tumors.25
Answer: The level of nodal metastasis helps predict the site
of the primary head and neck tumor. Cystic metastases are most
commonly associated with HPV+ oropharyngeal tumors or papillary thyroid carcinoma.
4. What is the role of positron-emission tomography in metastatic SCC of unknown primary?
When the primary site of metastatic SCC to a cervical node is
not apparent on clinical examination (including flexible endoscopy) and radiographic imaging, it is considered a metastatic
cancer of unknown primary origin. Contemporary large cohort
studies4,26 reveal that approximately 50% of occult primary sites
are ultimately identified after additional diagnostic techniques
are performed. The most common sites for occult primaries in
these cohorts were the tonsil (4044%), followed by the base of
tongue (3344%) (Level 2b evidence).
The additional procedures used to identify occult primary
sites are panendoscopy with directed biopsies of suspicious sites,
tonsillectomy, and metabolic imaging with 18-fluorodeoxyglucose
positron-emission tomography (PET). PET should be performed
prior to biopsies. In the original systematic review of 16 studies of
PET data in unknown primary head and neck cancer, the sensitivity, specificity, and accuracy of PET in identifying occult primary
tumors were 88.3%, 74.9%, and 78.8%, respectively. In 24.5% of
unknown primary cases, PET was able to detect the primary
tumor when other modalities failed.27 In the era of fused PET/CT,
small series have reported slightly higher rates of occult primary
identification, ranging from 32% to 44%26,28 (Level 2a evidence).
Answer: PET/CT imaging is a useful adjunct in the identification of primary site in metastatic head and neck cancer of
unknown primary.
5. What is the role of panendoscopy in metastatic SCC of
unknown primary?
Retrospective cohort studies in the pre-PET era estimated that
panendoscopy identified primary tumors in 24% to 29% of patients

5/22/2012 6:10:33 PM

816

Surgery: Evidence-Based Practice

without suggestive findings on physical examination, or imaging (CT and/or MR).4,29 In the modern era, panendoscopy is performed after PET/CT, so that any areas with hypermetabolic foci
on PET can be subjected to directed biopsies at endoscopy (Level 2b
evidence).
After PET/CT imaging, panendoscopy is performed, with
directed biopsies of sites deemed suspicious on functional imaging (PET), anatomic imaging (CT or MRI), or on inspection at the
time of endoscopy. Panendoscopy should include careful examination of the oral cavity, oropharynx, larynx, and hypopharynx.
Careful attention should be directed to the tonsils and base of
tongue as these two sites represent 70% to 80% of occult primary
cancers. The nasopharynx should be evaluated in select patients
such as those from Southern China, or with posterior triangle
lymphadenopathy.
The yield of esophagoscopy and bronchoscopy are both low in
patients with SCC of unknown primary, and have been reported
to identify occult or second primary cancers in <1% of patients
followed in a prospective cohort study; therefore, these procedures need not be performed in all patients30 (Level 1b evidence).
Workup targeting the chest and mediastinum such as esophagoscopy, bronchoscopy and chest CT, likely have the highest yield in
patients with metastases to the supraclavicular region
Answer: In patients with cancer of unknown primary,
endoscopy of the upper aerodigestive tract under anesthesia

should be performed after PET/CT imaging. Directed biopsies


should be taken from areas deemed suspicious on inspection or
imaging.
6. What is the role of tonsillectomy in metastatic SCC of
unknown primary?
Because the tonsillar fossa is the most common site of occult
cancer, a tonsillectomy increases diagnostic yield at the time of
panendoscopy. In recent cohort studies, ipsilateral tonsillectomy
has identified occult cancer in 30% to 44% of patients.4,26 Yield
was much lower (3%) if deep tonsil biopsies were performed
(Level 2b evidence). Accordingly, it seems prudent to perform
at least ipsilateral tonsillectomy in patients with head and neck
cancer of unknown primary. Although one retrospective cohort
identified a 10% incidence of occult primary cancer in the contralateral tonsil31 (Level 3b), which would support the need for
bilateral tonsillectomy, these data have not been replicated in
other cohorts.
Answer: In patients with SCC of unknown primary, a tonsillectomy should be performed. Occult cancer in the contralateral
tonsil is rare. Bilateral tonsillectomy has not been consistently
supported by the existing evidence but can be considered if a
symmetrical oropharynx is desired in order to facilitate follow-up
examinations.

Clinical Question Summary


Question

Answer

1 What is the risk of


malignancy in a neck
mass?

Few pediatric neck masses are malignant diseases. In


contrast, the majority of neck masses in adults represent
metastatic cancer, most commonly SCC from a head and
neck primary site.

6-11

2 How accurate is FNA


biopsy?

FNA for cytologic analysis is highly accurate in identifying


malignant processes.

12-18

3 Does metastatic cervical


lymph node location
predict the site of
primary tumor?

The level of nodal metastasis helps predict the site of the


primary head and neck tumor. Cystic metastases are
most commonly associated with HPV+ oropharyngeal
tumors or papillary thyroid carcinoma.

19-25

4 What is the role of PET/


CT in metastatic SCC
of unknown primary
origin?

PET/CT imaging is a useful adjunct in the identification


of primary site in metastatic head and neck cancer of
unknown primary.

26-28

5 What is the role of


panendoscopy in
metastatic SCC of
unknown primary
origin?

In patients with cancer of unknown primary, endoscopy of


the upper aerodigestive tract
under anesthesia should be performed after PET/CT
imaging. Directed biopsies should be taken from areas
deemed suspicious on inspection or imaging.

29, 30

6 What is the role of


tonsillectomy in
metastatic SCC of
unknown primary
origin?

In patients with SCC of unknown primary, a tonsillectomy


should be performed. Occult cancer in the contralateral
tonsil is rare. Bilateral tonsillectomy has not been
consistently supported by the existing evidence, but can
be considered if a symmetrical oropharynx is desired in
order to facilitate follow-up examinations.

4, 26, 31

PMPH_CH103.indd 816

Grade of
Recommendation

References

5/22/2012 6:10:33 PM

Solitary Neck Mass

REFERENCES
1. Chen A, Otto KJ. Differential diagnosis of neck masses. In:
Charles WC, Bruce HH, Thomas JR, Lee AH (eds). Cummings
Otolaryngology Head & Neck Surgery. Philadelphia, PA: Elsevier
Mosby; 2004.
2. Leung AK, Davies HD. Cervical lymphadenitis: etiology, diagnosis, and management. Curr Infect Dis Rep. 2009;11(3):183-189.
3. Leung AK, Kellner JD. Group A beta-hemolytic streptococcal
pharyngitis in children. Adv Ther. 2004;21(5):277-287.
4. Cianchetti M, Mancuso AA, Amdur RJ, Werning JW, Kirwan J,
Morris CG, et al. Diagnostic evaluation of squamous cell carcinoma metastatic to cervical lymph nodes from an unknown head
and neck primary site. Laryngoscope. 2009;119(12):2348-2354.
5. Dickson PV, Davidoff AM. Malignant neoplasms of the head and
neck. Semin Pediatr Surg. 2006;15(2):92-98.
6. Park YW. Evaluation of neck masses in children. Am Fam Physician. 1995;51(8):1904-1912.
7. Torsiglieri AJ Jr, Tom LW, Ross AJ 3rd, Wetmore RF, Handler SD,
Potsic WP. Pediatric neck masses: guidelines for evaluation. Int J
Pediatr Otorhinolaryngol. 1988;16(3):199-210.
8. Anne S, Teot LA, Mandell DL. Fine needle aspiration biopsy: role
in diagnosis of pediatric head and neck masses. Int J Pediatr Otorhinolaryngol. 2008;72(10):1547-1553.
9. Gray SW, Skandalakis JE, Androulakis JA. Non-thyroid tumors
of the neck. Contemporary Surg. 1985;26:13-24.
10. Lefebvre JL, Coche-Dequeant B, Van JT, Buisset E, Adenis A.
Cervical lymph nodes from an unknown primary tumor in 190
patients. Am J Surg. 1990;160(4):443-446.
11. Bhattacharyya N. Predictive factors for neoplasia and malignancy in a neck mass. Arch Otolaryngol Head Neck Surg. 1999;
125(3):303-307.
12. Mobley DL, Wakely PE, Jr, Frable MA. Fine-needle aspiration
biopsy: application to pediatric head and neck masses. Laryngoscope. 1991;101(5):469-472.
13. Wakely PE, Jr, Kardos TF, Frable WJ. Application of fine needle
aspiration biopsy to pediatrics. Hum Pathol. 1988;19(12):1383-1386.
14. Liu ES, Bernstein JM, Sculerati N, Wu HC. Fine needle aspiration biopsy of pediatric head and neck masses. Int J Pediatr
Otorhinolaryngol. 2001;60(2):135-140.
15. Carr S, Visvanathan V, Hossain T, Uppal S, Chengot P, Woodhead CJ. How good are we at fine needle aspiration cytology? J
Laryngol Otol. 2010;124(7):765-766.
16. Shaha A, Webber C, Marti J. Fine-needle aspiration in the diagnosis of cervical lymphadenopathy. Am J Surg. 1986;152(4):420-423.
17. Shaha AR, Webber C, DiMaio T, Jaffe, BM. Needle aspiration
biopsy in salivary gland lesions. Am J Surg. 1990;160(4):373-376.
18. Wu M, Burstein DE, Yuan S, Nurse LA, Szporn AH, Zhang D,
et al. A comparative study of 200 fine needle aspiration biopsies

PMPH_CH103.indd 817

19.

20.

21.

22.

23.

24.

25.

26.

27.

28.

29.

30.

31.

817

performed by clinicians and cytopathologists. Laryngoscope.


2006;116(7):1212-1215.
Shah JP. Patterns of cervical lymph node metastasis from squamous carcinomas of the upper aerodigestive tract. Am J Surg. 1990;
160(4):405-409.
Ozer E, Karapinar U, Ryoo C, Agrawal A, Schuller DE. When
to address level I lymph nodes in neck dissections? Otolaryngol
Head Neck Surg. 2010;142(3):355-358.
King AD, Ahuja AT, Leung SF, Lam WW, Teo P, Chan YL, et al.
Neck node metastases from nasopharyngeal carcinoma: MR
imaging of patterns of disease. Head Neck. 2000;22(3):275-281.
Vauterin TJ, Veness MJ, Morgan GJ, Poulsen MG, OBrien CJ.
Patterns of lymph node spread of cutaneous squamous cell carcinoma of the head and neck. Head Neck. 2006;28(9):785-791.
Goldenberg D, Sciubba J, Koch WM. Cystic metastasis from head
and neck squamous cell cancer: a distinct disease variant? Head
Neck. 2006;28(7):633-638.
Sheahan P, Oleary G, Lee G, Fitzqibbon J. Cystic cervical metastases: incidence and diagnosis using fi ne needle aspiration biopsy.
Otolaryngol Head Neck Surg. 2002;127(4):294-298.
Goldenberg D, Bequm S, Westra WH, Khan Z, Sciubba J, Pai SI,
et al. Cystic lymph node metastasis in patients with head and
neck cancer: an HPV-associated phenomenon. Head Neck.
2008;30(7):898-903.
Waltonen JD, Ozer E, Hall NC, Schuller DE, Agrawal A. Metastatic carcinoma of the neck of unknown primary origin: evolution and efficacy of the modern workup. Arch Otolaryngol Head
Neck Surg. 2009;135(10):1024-1029.
Rusthoven KE, Koshy M, Paulino AC. The role of fluorodeoxyglucose positron emission tomography in cervical lymph node
metastases from an unknown primary tumor. Cancer. 2004;
101(11):2641-2649.
Roh JL, Kim JS, Lee JH, Cho KJ, Choi SH, Nam SY, et al. Utility of combined (18)F-fluorodeoxyglucose-positron emission
tomography and computed tomography in patients with cervical metastases from unknown primary tumors. Oral Oncol.
2009;45(3):218-224.
Mendenhall WM, Mancuso AA, Parsons JT, Stringer SP, Cassisi NJ.
Diagnostic evaluation of squamous cell carcinoma metastatic to
cervical lymph nodes from an unknown head and neck primary
site. Head Neck. 1998;20(8):739-744.
Davidson J, Gilbert R, Irish J, Witterick I, Brown D, Birt D, et al.
The role of panendoscopy in the management of mucosal head
and neck malignancya prospective evaluation. Head Neck.
2000;22(5):449-454; discussion 454-455.
Koch WM, Bhatti N, Williams MF, Eisele DW. Oncologic rationale for bilateral tonsillectomy in head and neck squamous cell
carcinoma of unknown primary source. Otolaryngol Head Neck
Surg. 2001;124(3):331-333.

5/22/2012 6:10:33 PM

CHAPTER 104

Peripheral Nerve Injury Repair


Marisa H. Amaral, Pranay Parikh, and Joseph H. Shin

INTRODUCTION

median and ulnar nerves in children, who can achieve near complete recovery.7
The goal of the answers to the following questions will provide a guide for treatment of traumatic peripheral nerve injuries
and identify the current issues regarding their management.

Much of our early knowledge about peripheral nerve injury and


subsequent management was developed during periods of war,
dating back to as early as the American Civil War.1 These combat
injuries are often related to blast injuries, complicated by vascular and soft tissue damage. Peripheral nerve injuries continue to
be encountered in emergency departments, with sources citing
up to 2% to 3% of patients admitted to Level I trauma centers.2
Traumatic peripheral nerve injuries most commonly result from
motor vehicle collisions, with the upper extremity as the most
commonly injured limb.2,3 A trauma patient entering the emergency department following a serious motor vehicle collision may
likely have other injuries requiring immediate attention. Therefore, early recognition of peripheral nerve injury is an important
hurdle to overcome.
The extent of nerve injury typically determines the treatment
modality. Two major classification schemes exist, one proposed
by Seddon and the other by Sunderland. Seddon used the terms:
neurapraxia, to describe mild myelin injury without axonal injury,
coinciding with Sunderlands description of first-degree injury;
axonotmesis, to describe axonal injury with damage to some or
all of the connective tissues, coinciding the Sunderlands description of second- through fourth-degree injury; and neurotmesis, to
describe a complete division of the nerve, coinciding with Sunderlands description of fift h-degree injury. Neurotmesis is a level of
injury that will not regenerate without surgical intervention.4
Once a peripheral nerve injury is recognized, the challenge
lies in determining the timing and the modality of repair. This
is a topic of utmost importance for the rehabilitation of patients
suffering from peripheral nerve injury. Military personnel and
trauma patients are typically younger; debilitating peripheral
nerve injuries, therefore, result in loss of work and handicap.
Their ability to rejoin the workforce is related to functional motor
and sensory recovery.5,6 Unfortunately, the injured adult peripheral nerve will never regain full function after repair. Age plays a
major role in nerve recovery; this is demonstrated by the repair of

1. What are important considerations when planning the timing of nerve repair?
In a review of peripheral nerve injury, Campbell identifies the
factors involved in deciding when to repair a peripheral nerve.
Surgical intervention is required for neurotmesis, and usually for
severe axonotmesis. Nerve injuries can be repaired immediately,
or the surgical repair may be delayed by several weeks. Ultimately,
the main consideration for timing of nerve repair appears to be
the mechanism of injury. The distinction is made between injuries
involving sharp transection of the nerve and those involving blunt
injury, including crush, avulsion, and stretch or blast injuries. For
the sharply transected nerve without significant surrounding tissue damage, the best results are demonstrated with immediate
repair of the injured nerve, by end-to-end neurorrhaphy.1
For injuries involving blunt or blast trauma, the extent of injury
may not be completely obvious, even when aided by a microscope.
In these situations, it is reported that the two nerve ends be tacked
together to prevent retraction and delay repair until at least 3 weeks.4
Initial assessment for peripheral nerve injury can be challenging in
blast injuries. These types of injuries are often associated with coexisting damage to soft tissue, bones and blood vessels, which require
immediate control for the salvage of an extremity. Delayed nerve
exploration and repair has been argued in these cases, due to the
possibility of spontaneous recovery of nerve function with lower
grades of injury. Frequent re-evaluation of functional recovery is
required in cases of delayed repair. If no evidence of regeneration is
witnessed, exploration of the injury within 3 months is required.8
A time-distance equation may facilitate determining timing of a surgical intervention for peripheral motor nerves: Reinnervation must occur before the muscle undergoes irreversible

818

PMPH_CH104.indd 818

5/22/2012 6:11:07 PM

Peripheral Nerve Injury Repair

atrophic changes at 12 to 18 months and before endoneurial tubes


will no longer support nerve regrowth, which remains viable for
18 to 24 months after injury.1
In summary: The repair of a damaged nerve is dependent on
both the extent of the injury and the mechanism of injury. Lower
grades of injury, including neurapraxia and less severe grades of
axonotmesis, may recover function within weeks of injury. More
serious grades of injury, including severe axonotmesis and neurotmesis, will require surgical intervention if any recovery of sensory
and/or motor function is to be achieved. Injuries resulting from
sharp transection may be repaired immediately, whereas those
injuries resulting from blunt or blast injuries may benefit from
delayed repair, as the extent of nerve damage may persist beyond
the inciting traumatic event (Grade B recommendation).
2. What is the preferred method of nerve repair?
A primary end-to-end nerve repair of proximal and distal nerve
stumps is the preferred method, if possible. Dahlin describes a
stepwise approach to the repair of a transected peripheral nerve9:
First, the nerve ends are prepared by removing any necrotic tissue
or debris; the nerve ends are then approximated, with minimal dissection if necessary, to avoid undue tension and stretch on the nerve
ends; the nerve ends are then coapted, and the repair maintained by
sutures placed at the epineurium, the connective tissue surrounding the peripheral nerve. Buckling of the nerve bundles, or fascicles,
may cause misdirection of the regenerating nerve, so care is taken
to ensure that the proximal and distal ends are aligned by trimming
the fascicles and by placing the sutures loosely.4
One of the guiding tenets of peripheral nerve repair involves
maintaining a tension-free link between the two end segments
of a damaged nerve. Studies from the latter half of the 20th century, discussed in a review by Trumble and McCallister, identify
a correlation between tension at a peripheral nerve and decreased
blood flow.10 In turn, ischemia at a peripheral nerve can have
direct effects on its function. It is suggested that the elongation
of a peripheral nerve at time of repair be limited to 8% to 10%
of the original length of the nerve. Furthermore, it is important
to consider stabilizing the extremity in a neutral position during the operative repair to determine if an end-to-end nerve
repair is even possible without inadvertently stretching the nerve
segments.4
Additional experimental methods of primary nerve repair
have been described for extenuating circumstances, where the
proximal nerve stump is unavailable or a significant nerve gap
exists. One of these experimental methods is an end-to-side nerve
repair. Using this technique, the distal stump of an injured nerve
is coapted to the side of an uninjured donor nerve. Pannucci et al.
summarized in a literature review in 2007 that end-to-side neurorrhaphy is a possible method of nerve repair, but experimental models up to that point had not yet proven this method to be
superior to existing methods of nerve repair.11
Another principle of peripheral nerve repair is to repair
the nerve segments in a matched modality manner, such that
proximal motor nerve segments are matched with their counterpart distal motor nerve segments, and proximal sensory nerve
segments matched in like fashion with distal sensory nerve segments. Several studies have investigated the importance of a
matched modality repair for successful regeneration. Brushart
proposed a preferential motor reinnervation model, and in 1993

PMPH_CH104.indd 819

819

demonstrated that motor axons preferentially reinnervate distal


motor pathways, rather than sensory pathways.12 Nichols et al.
supported this model by demonstrating that motor or mixed
nerve grafts, rather than sensory nerve grafts, promoted regeneration across a rat tibial mixed nerve defect.13
In the clinical situation, however, autologous sensory nerve
grafts are used for repair of peripheral nerves, especially those
with extended gap segments. Sensory donor nerves, commonly
the sural nerve or the medial antebrachial cutaneous nerve, can
be harvested without the morbidity associated with harvesting a
motor nerve.9 To challenge the matched modality repair model,
and to identify potential neurobiologic mechanisms to optimize
peripheral nerve repair, Lloyd et al. used a rodent sciatic nerve
injury model to suggest that the physically intact nerve graft was
an important factor in determining regeneration capabilities.14
Using experimental conduits that contained minced motor nerve
fragments, minced sensory nerve fragments, or minced mixed
nerve fragments to bridge the nerve gap, no significant difference was observed on histomorphometric analysis among the
conduits. The multitude of experiments that have demonstrated
modality matching have used mixed fascicular grafts. Kawamura
et al., in 2010, developed a more clinically applicable model to test
the matched modality repair theory by using unifascicular grafts
for nerve reconstruction, similar to what is employed in human
autologous nerve graft repair.15 When a pure motor or sensory
nerve graft was used for repair, the advantage of the motor graft
seen in previous studies was no longer there.
In summary: Primary end-to-end nerve repair in a tensionfree manner is the preferred method of repair for peripheral nerve
injury. Early studies suggest that motor nerves preferentially reinnervate distal nerve ends, owing to the theory of preferential
motor reinnervation. This theory is challenged by the current
clinical practice of using sensory autologous nerve grafts to bridge
gaps of nerve damage (Grade B recommendation).
3. What are the indications and options for repair of a peripheral nerve injury with a nerve graft?
An end-to-end, tension-free primary nerve repair is the ideal
management of a peripheral nerve injury; however, in traumatic
extremity injuries, the luxury of an ideal situation is seldom
offered. A nerve graft provides a bridge across a nerve gap that
may result after traumatic injury. To determine if the use of a graft
is necessary for nerve repair, Trumble and McCallister describe a
formula to calculate the percentage elongation when proximal and
distal nerve ends are marked with methylene blue before mobilization: [(AI BI) (A B) / (A B)] 100, where (A B) represents
the original length of the nerve and (AI BI) represents the length
after elongation.10 It is suggested that a nerve graft be utilized if
the percentage of elongation exceeds 10%. Historically, autologous sensory nerve grafts have served as donors for repair of nerve
gaps that cannot be repaired primarily. Owing to the morbidity of
loss of sensation associated with this method of repair, researchers have investigated alternatives for the repair of peripheral nerve
injuries. The ideal graft would provide a protective barrier to the
regenerating axons, as well as an environment rich in trophic factors to stimulate and promote directed growth.16
The research and development of grafts has been pursued,
in hopes of providing the regenerating nerve with nutrients
and protection for maintenance of growth. Depending on the

5/22/2012 6:11:07 PM

820

Surgery: Evidence-Based Practice

composition of the graft, the conduit may serve as a haven or


as a scaffold for the regenerating axons. The various options are
divided into biologic grafts and synthetic grafts, on the basis of
their origin.

Biologic Grafts

In summary: Each of the biologic graft options provides a


unique alternative to autologous sensory grafts, thereby limiting
the additional morbidity associated with the sensory graft harvesting. Although vein grafts provide a shelter for regenerating axons,
the nerve allograft provides a scaffold. Nerve transfers are another
innovative option for nerve repair and have shown promising
results for brachial plexus injuries (Grade B recommendation).

Vein Grafts
Among the first conduits to bridge gaps between severed nerve
ends was the autogenous vein graft. Advantages of the vein conduit over an autologous sensory nerve graft include the decreased
morbidity associated with harvesting a donor vein. Chiu et al.
demonstrated in 1981 that a femoral vein segment allowed
regenerating sciatic nerve fibers in a rat model to successfully
reach the distal stump.17 In order to confirm the successful regeneration, nerve conduction studies showed restoration of conduction, and necropsy showed nearly normal gastrocnemius muscle
fibers. Subsequent studies have used vein conduit grafts for the
repair of digital nerve injuries, with acceptable return of sensory
function.18

Synthetic Grafts
Nondegradable
Of the nondegradable inert polymers are silicone and expanded
polytetrafluoroethylene (PTFE). Ichihara et al. reviewed clinical studies of artificial nerve guides.16 It was determined that the
silicone conduit provided equivalent results to primary repair;
however, this conduit required removal in several patients within
3 years of repair due to local discomfort. The PTFE conduit has
also been evaluated in humans and was found to satisfy recovery
of short-segment nerve deficits, 15 to 40 mm.

Biodegradable
Allografts
Also minimizing the morbidity associated with autologous
graft s is the technique of nerve allografts. The advantages of
a cadaveric donor nerve are (1) the injured patient would not
be subjected to additional morbidity with the harvesting of an
autologous sensory nerve, and (2) the allograft would provide an
organized basal lamina for regenerating axons, which is an element not yet provided by most available synthetic graft s. Arguments against the use of nerve allografts are mainly related to
the concern surrounding the hosts immune response, and thus
the need for immunosuppression. Researchers have attempted
to develop methods to remove immunogenic components from
an allograft while maintaining an intact basal lamina structure.
Such methods were described and further tested in a rat sciatic
nerve model by Sondell et al. in 1998.19 Using a chemical extraction technique, the resultant acellular nerve allograft was shown
to serve as a guide for regenerating axons. Further studies have
compared an approved allograft to an approved collagen conduit. Whitlock et al. demonstrated that, although the graft alternatives did not show any advantage over the use of an isograft
in the repair of long-segment gaps, the allograft product proved
to be superior to the collagen conduit product for short-segment
gaps, in a rat sciatic nerve model.20

Nerve Transfers
An additional technique for the repair of peripheral nerve injuries includes nerve transfer. This technique involves coapting the
distal end of a nerve with minor function with the distal end of
an injured nerve that provides critical function. Examples include
techniques such as the Oberlin procedure, where fascicles of the
ulnar nerve are implanted in the musculocutaneous nerve or
the biceps muscle in order to gain elbow flexion.1 Tung and Mackinnon describe the indications for the use of nerve transfers and
focus on an appropriate patient selection.21 The advantages of the
nerve transfer technique can be seen in injuries where there is not
an adequate proximal nerve stump available for reconstruction,
such as in brachial plexus or other high nerve injures.

PMPH_CH104.indd 820

Biodegradable materials for use in nerve conduits include polyglycolic acid (PGA), polylactide-caprolactone (PLCL), polyglycolic
acid mesh (PGA-c), and collagen. Clinical evaluations of these
biodegradable conduits, again reviewed by Ichihara et al., have
varying success in the regeneration of injured nerve segments.16
Although these biodegradable conduits appear to provide a reasonable alternative to the standard autologous sensory nerve graft,
the functional recovery appears to be dependent on the length of
the graft.
In summary: Artificial nerve tubes serve as guides for the
regenerating nerve ends and can be categorized into nondegradable inert polymers and biodegradable materials. The evidence
indicates that both can serve as alternatives to autologous sensory
nerve grafts but are limited in their use by the length of the nerve
gap required to overcome (Grade B recommendation).
4. What are some therapeutic adjuncts for the treatment of
nerve injuries?
Several surgical options exist for the repair of damaged peripheral
nerves. These microsurgical techniques, however, do not guarantee
reinnervation. Despite the regenerative process of peripheral nerves,
the rate is slow, noted at 1 mm per day,22,23 and there has been evidence of a delay in regeneration after injury. This contributes to a
long-standing period of denervation for supportive cells and target
muscles.22 Even with the regeneration of axons across the site of injury,
the functional recovery of target motor complexes will not reach the
same level prior to injury. This discrepancy is thought to be due to
possible inappropriate reinnervation or reinnervation of antagonistic muscle groups, resulting in atrophy of target muscles.24 In order
to overcome these limiting factors, adjunctive modes of therapy have
been sought to enhance the rate of axonal regeneration.

Electrical Stimulation
Studies that investigate the role of electrical stimulation at the
site of peripheral nerve injury have yielded varying results. There
is agreement that brief 1-hour periods of electrical stimulation

5/22/2012 6:11:07 PM

Peripheral Nerve Injury Repair

stimulates axonal outgrowth toward distal nerve stumps.22,25-27


The impact of electrical stimulation on functional recovery of
motor complexes, however, is questionable. Asensio-Pinilla et al.
and Gordon et al. demonstrated improved muscle reinnervation
and functional recovery, respectively, with brief electrical stimulation at transected nerves in rodent models.22,25 Other studies,
however, were not able to demonstrate this enhanced effect and
further suggest that electrical stimulation may have a detrimental
effect on functional recovery. Results from a study by Lu et al.,
using different current intensities on a rodent model with a nerve
gap of 10 mm, indicate that increased current intensity may hinder the regenerative process.26 In addition, a study by Gigo-Benato
et al., using a crush nerve injury rodent model, demonstrates that
electrical stimulation may delay nerve regeneration.28 It should be
noted that these competing studies used a different protocol for
electrical stimulation, applying current every other day.

Exercise Training
Herbison et al. attempted to identify optimal levels of intensity
and timing of exercise following nerve injury in a rat sciatic nerve
crush model subjected to swimming exercise.29 Their findings supported the hypothesis that nerve regeneration is not enhanced by
higher intensity exercise in the early stages after injury. In a similar
rat sciatic nerve crush injury model, van Meeteren et al. compared
types of exercise, with swim exercise training and treadmill training.30 This study noted that the rats subjected to treadmill training
experienced impaired return of motor function, although sensory
function was not affected by the strenuous exercise.
Treadmill training was further evaluated by Sabatier et al.,
and showed, in a transected common fibular nerve rat injury
model, that treadmill training at both continuous and highintensity interval training increased the lengths of regenerating
axons, suggesting that postinjury exercise enhances the overall growth rate of axons.31 This was followed up by English et al.
with a sciatic nerve mouse injury model, using retrograde labeling of the transected nerve ends, to determine if the regenerating
axons displayed a misdirected reinnervation of targets.24 It was
concluded that, although treadmill training had a robust effect
on the regeneration of axons after injury, there was no increase of
misdirected reinnervation compared to untreated controls.

Neurotrophic Factors
In addition to the external factors noted above, studies have investigated biological factors that may overcome the slow regenerative
process of axons following injury, in order to limit the state of
chronic axotomy to which distal nerve stumps are subjected. Lundborg et al. surmise, in a silicone tubular model of reconstructing transected peripheral nerves, that injured peripheral nerves
need to be provided with an appropriate healing environment,
consisting of the localization of neurotrophic factors, such as
nerve growth factor, brain-derived growth factor, and ciliary neurotrophic factor, to aid in the regeneration of axonal stumps.32
Subsequent investigations in the field of peripheral nerve
injury have used this theory as a basis for innovative alternative
therapies. Walsh and Midha suggest the use of skin-derived precursor cells as a model of stem cell therapy to add to the regenerative milieu at the site of peripheral nerve injury.23 Other studies
have investigated the addition of neurotrophic factors directly to

PMPH_CH104.indd 821

821

the nerve repair site. This is demonstrated by Panaite and BarakatWalter with the addition of thyroid hormone to a nerve guide,
resulting in an enhanced reinnervation and functional recovery
of a transected rat sciatic nerve model.33 A nerve guide was also
used by Li et al. as a vehicle for the localized delivery of FK506,
which has been identified recently as a neurotrophic factor.34
In summary: Several different types of adjunctive therapies
have been investigated with the goal of identifying a method to
enhance axonal regeneration. Electrical stimulation has been
shown to stimulate the outgrowth of axons. Its effect on functional recovery after peripheral nerve injury may depend on several factors, including the mechanism of injury, the intensity of
applied current, and the timing of applied current. In a similar
fashion, the impact of exercise training on functional recovery
following peripheral nerve injury may be related to the type and
intensity of training. The varying results of these studies indicate
that a proven rehabilitation protocol using electrical stimulation
and/or exercise training as adjunctive therapies does not yet exist.
In contrast to these externally applied modes of therapy, investigations into alternative therapies that exploit the intrinsic physiologic properties of peripheral nerves are promising. (Grade C
recommendation).
5. When is it safe to mobilize and rehabilitate an extremity after
repair of a peripheral nerve injury?
Following repair of a peripheral nerve injury, the extremity is
immobilized for a variable number of weeks, between 2 and 6
weeks, often depending on the coexisting injuries. Lee et al. investigated the effects of mobilization at the site of neurorrhaphy in
an effort to provide a histomorphometric basis for recommendations.35 Based on the length of time for revascularization of a
nerve injury in a canine model, there is evidence in an isolated
nerve injury to immobilize the affected extremity for 3 weeks.
An avascular zone at the nerve injury site persisted up to 6 weeks
with mobilization, thereby delaying nerve regeneration. Evidence
of decreased angiogenesis by mobilization of a nerve repair was
confirmed by Schmidhammer et al.36 This group hypothesized
that an increased tension at the site of repair created by mobilization impeded nerve regeneration. They attempted to prove this
hypothesis by developing a silicone tube nerve splint to alleviate
tension at the site of neurorrhaphy.
To further evaluate the effects of mobilization in a nerve gap
model, Chao et al. demonstrated that nerve sutures did not tear
in cadaveric digital nerve gaps up to 5 mm when a dorsal finger
splint was used, despite early mobilization (<1 week after repair).
In addition, nerve sutures tolerated full range of motion in digital
nerve gaps up to 2.5 mm without the use of splints.37
In summary: Early mobilization of a repaired peripheral
nerve injury creates increased tension at the site of neurorrhaphy,
which appears to impede the regenerative process by formation
of scar tissue and decreased angiogenesis. Th ree weeks of immobilization allows sufficient time for revascularization across
the site of injury in a primary nerve repair. There is evidence,
however, to support that splinting of an extremity, and even at
the level of the nerve itself, may alleviate tension across a repair
site, such that mobilization earlier than 3 weeks may not have
such detrimental effects on the regenerative process (Grade B
recommendation).
The management of peripheral nerve injuries is clear when
it comes to the repair of a nerve damaged by sharp transection.

5/22/2012 6:11:07 PM

822

Surgery: Evidence-Based Practice

A nerve injury with a significant gap, however, has fueled the


development of innovative management alternatives, based on
the inherent healing properties of the injured peripheral nerve. A
variety of options exist, and most appear to provide a reasonable

alternative to the autologous sensory nerve graft , with the limiting factor being length of graft. Future studies are needed for
comparative analysis of which graft alternative is better than
others.

Clinical Question Summary


Question

Answer

Grade of
Recommendation

References

1 What are important considerations when


planning the timing of nerve repair?

Extent of damage to the nerve


structure and mechanism of
injury.

1, 4, 8

2 What is the preferred method of nerve


repair?

Primary end-to-end anastomosis

4, 9-12, 14, 15

3 What are the indications and options for


repair of a peripheral nerve injury with a
nerve graft?

Nerve graft: For gaps that require


elongation greater than 10% of
its original length.
Graft options:
Biologic alternatives
Synthetic alternatives

1, 10, 16-21

4 What are some therapeutic adjuncts for the


treatment of peripheral nerve injuries?

Electrical stimulation.
Exercise training.
Neurotrophic additives to site of
injury.

B
B
C

22-31, 33

5 When is it safe to mobilize and rehabilitate


an extremity after repair of a peripheral
nerve injury?

Three weeks after repair.

18, 36, 37

B
B

Levels of Evidence
Subject

Year

References

Timing of
repair

2008

IIC

Neurotmesis requires surgical intervention.


Injuries involving sharp transection may be
repaired immediately.

Method of
repair

2008

IIC

A tension-free primary end-to-end


anastomosis is the preferred method of
repair.

Repair with a
nerve graft

2000

10

IIB

1981,
1998,
2010
2008

17, 19, 21

IIB

Nerve grafts are required if elongation


exceeds 10% of original nerve length.
Various grafts are available as alternatives
to the autologous sensory graft, but are
limited by the length of the gap.

16

IIB

Adjunctive
therapies:
Electrical
stimulation
Exercise
training
Neurotrophic
additives to
site of injury
Time to
mobilization

PMPH_CH104.indd 822

Level of
Evidence

Strength of
Recommendation

2009

22

2009

24

IIB

2009,
2010

23, 33

IIC, IIB

1999

35

IIB

Findings

Brief, early electrical stimulation and exercise


training enhance axonal regeneration and
subsequent functional recovery. Additional
innovative alternatives to supplement the
injury site with neurotrophic factors are
promising.

Immobilization for a period of 3 weeks after


repair allows for sufficient regeneration.

5/22/2012 6:11:07 PM

Peripheral Nerve Injury Repair

REFERENCES
1. Campbell WW. Evaluation and management of peripheral nerve
injury. Neurophysiol Clin. 2008;119:1951-1965.
2. Robinson LR. Traumatic injury to peripheral nerves. Muscle
Nerve. 2000;23:863-873.
3. Noble J, Munro CA, Prasad VS, Midha R. Analysis of upper
and lower extremity peripheral nerve injuries in a population of
patients with multiple injuries. J Trauma. 1998;45:116-122.
4. Myckatyn TM, Mackinnon SE. Microsurgical repair of peripheral nerves and nerve grafts. In: Thorne CH, ed. Grabb and
Smiths Plastic Surgery. 6th ed. Baltimore, MD: Lippincott Williams and Wilkins; 2007:73-83.
5. Lundborg G, Rosen B. Hand function after nerve repair. Acta
Physiol. 2007;189:207-217.
6. Jaquet JB, Luijsterburg AJM, Kalmijn S, Kuypers PDL, Hofman
A, Hovius SER. Median, ulnar, and combined median-ulnar
nerve injuries: Functional outcome and return to productivity.
J Trauma. 2001;51:687-692.
7. Birch, R. Nerve repair. In: Green DP, ed. Greens Operative Hand
Surgery. Vol. 1, 5th ed. Philadelphia, PA: Elsevier Churchill
Livingstone; 2005: Chapter 30.
8. Secer HI, Daneyemez M, Tehli O, Gonul E, Izci Y. The clinical,
electrophysiologic, and surgical characteristics of peripheral
nerve injuries caused by gunshot wounds in adults: a 40-year
experience. Surg Neurol. 2008;69:143-152.
9. Dahlin LB. Techniques of peripheral nerve repair. Scand J Surg.
2008;97:310-316.
10. Trumble TE, McCallister, WV. Repair of peripheral nerve defects
in the upper extremity. Hand Clin. 2000;16:37-52.
11. Pannucci C, Myckatyn TM, Mackinnon SE, Hayashi A. End-toside nerve repair: review of the literature. Restor Neurol Neurosci.
2007;25:45-63.
12. Brushart TME. Motor axons preferentially reinnervate motor
pathways. J Neurosci. 1993;13:2730-2738.
13. Nichols CM, Brenner MJ, Fox IK, et al. Effect of motor versus
sensory nerve grafts on peripheral nerve regeneration. Exp Neurol. 2004;190:347-355.
14. Lloyd BM, Luginbuhl RD, Brenner MJ, et al. Use of motor nerve
material in peripheral nerve repair with conduits. Microsurgery.
2007;27:138-145.
15. Kawamura DA, Johnson PJ, Moore AM, et al. Matching of motorsensory modality in the rodent femoral nerve model shows no
enhanced effect on peripheral nerve regeneration. Exp Neurol.
2010;223:496-504.
16. Ichihara S, Inada Y, Nakamura. Artificial nerve tubes and their
application for repair of peripheral nerve injury: an update of
current concepts. Injury. 2008;39:529-539.
17. Chiu DTW, Janecka I, Krizek TJ, Wolff M, Lovelace RE. Autogenous vein graft as a conduit for nerve regeneration. Surgery.
1982;91:226-233.
18. Lee, YH and S-J Shieh. Secondary nerve reconstruction using
vein conduit grafts for neglected digital nerve injuries. Microsurgery. 2008;28:436-440.
19. Sondell M, Lundborg G, Kanje M. Regeneration of the rat sciatic
nerve into allografts made acellular through chemical extraction. Brain Res. 1998;795:44-54.
20. Whitlock EL, Tuffaha SH, Luciano JP, et al. Processed allografts
and Type I collagen conduits for repair of peripheral nerve gaps.
Muscle Nerve. 2009;39:787-799.

PMPH_CH104.indd 823

823

21. Tung TH, Mackinnon SE. Nerve transfers: indications, techniques, and outcomes. J Hand Surg. 2010;35A:332-341.
22. Gordon T, Chan KM, Sulaiman OAR, Udina E, Amirjani N,
Brushart TM. Accelerating axon growth to overcome limitations
in functional recovery after peripheral nerve injury. Neurosurgery. 2009;65:A132-A144.
23. Walsh S, Midha R. Use of stem cells to augment nerve injury
repair. Neurosurgery. 2009;65:A80-A86.
24. English AW, Cucoranu D, Mulligan A, Sabatier M. Treadmill
training enhances axon regeneration in injured mouse peripheral nerves without increased loss of topographic specificity.
J Compar Neurol. 2009;517:245-255.
25. Asensio-Pinilla E, Udina E, Jaramillo J, Navarro X. Electrical
stimulation combined with exercise increase axonal regeneration after peripheral nerve injury. Exp Neurol. 2009;219:
258-265.
26. Lu MC, Tsai CC, Chen SC, Tsai FJ, Yao CH, Chen YS. Use of
electrical stimulation at different current levels to promote
recovery after peripheral nerve injury in rats. J Trauma. 2009;67:
1066-1072.
27. Viv M, Puigdemasa A, Casals L, Asensio E, Udina E, Navarro
X. Immediate electrical stimulation enhances regeneration and
reinnervation and modulates spinal plastic changes after sciatic
nerve injury and repair. Exp Neurol. 2008;211:180-193.
28. Gigo-Benato D, Russo TL, Geuna S, Santa Rosa Domingues
NR, Salvini TF, Parizotto NA. Electrical stimulation impairs
early functional recovery and accentuates skeletal muscle
atrophy after sciatic nerve crush injury in rats. Muscle Nerve.
2010;41:685-693.
29. Herbison GJ, Jaweed MM, Ditunno JF. Effect of swimming on
reinnervation of rat skeletal muscle. J Neurol Neurosurg Psych.
1974;37:1247-1251.
30. van Meeteren NL, Brakkee JH, Helders PJ, Gispen WH. The
effect of exercise training on functional recovery after sciatic
nerve crush in the rat. J Peripher Nerv Syst. 1998;3:277-282.
31. Sabatier MJ, Redmon N, Schwartz G, English AW. Treadmill
training promotes axon regeneration in injured peripheral
nerves. Exp Neurol. 2008;211:489-493.
32. Lundborg G, Rosen B, Dahlin L, Danielson N, Holmberg J.
Tubular versus conventional repair of median and ulnar nerves
in the human forearm: early results from a prospective, randomized, clinical study. J Hand Surg. 1997;22A:99-106.
33. Panaite PA, Barakat-Walter, I. Thyroid hormone enhances
transected axonal regeneration and muscle reinnervation following rat sciatic nerve injury. J Neurosci Res. 2010;88:1751-1763.
34. Li X, Wang W, Wei G, Wang G, Zhang W, Ma X. Immunophilin FK506 loaded in chitosan guide promotes peripheral nerve
regeneration. Biotechnol Lett. 2010;32:1333-1337.
35. Lee WP, Constantinescu MA, Butler PEM. Effect of early mobilization on healing of nerve repair: histologic observations in a
canine model. Plast Reconstr Surg. 1999;104:1718-1725.
36. Schmidhammer R, Zandieh S, Hopf R, et al. Alleviated tension at the repair site enhances functional regeneration: the
effect of full range of motion mobilization on the regeneration of peripheral nerveshistologic, electrophysiologic,
and functional results in a rat model. J Trauma. 2004;56:
571-584.
37. Chao RP, Braun SA, Ta KT, et al. Early passive mobilization after
digital nerve repair and graft ing in a fresh cadaver. Plast Reconstr Surg. 2001;108:386-391.

5/22/2012 6:11:07 PM

Commentary on
Peripheral Nerve Injury Repair
Howard T. Wang

Peripheral nerve injury can be an extremely challenging issue to


manage. Nerve regeneration is very slow and can require significant patience on the part of both physician and patient. To further
complicate matters, peripheral nerve injury often is in association with other injuries that may take precedent over the nerve
injury and also mask the nerve injury. The authors in the chapter
summarize some of the major considerations that go into treating a patient with nerve injury. The topics touched upon include
type of injury, timing of repair, synthetic conduits, nerve grafting, and rehabilitation and recovery. Unfortunately, most studies
looking at injury in humans are not prospective randomized trials
for obvious reasons. Thus, the best level of evidence in most cases
will be Level II evidence. In terms of clinical significance, severe
axonotmesis or neurotmesis are the most important to recognize
early, as they will generally require surgical intervention.
As the authors have pointed out, the best possible scenario for
nerve repair is a primary end-to-end suture shortly after the time
of injury. Clean lacerations that transect the nerve will be more
amenable to repair than a blast injury with significant debris in
the wound. However, the type of nerve injured is also an important consideration. Clearly, a small superficial sensory nerve can
be sacrificed without much functional deficit as compared to a
larger motor nerve. The key to a successful nerve repair is tension-free anastomosis in a clean field. It is important to have good
approximation of the epineurium with loose coaptation of the
nerve ends. It is thought to be important, as well as possible, to try
to line up the nerve, but really it is the regeneration process that
helps the nerves find their way across gaps.1 In order for the repair
to be tension free, transposition of the nerve can be performed to
yield additional length in areas such as the upper arm.2
Should a gap exist that does not allow for primary suture, the
authors touch upon alternative techniques such as synthetic conduits and nerve grafting. In shorter gaps, synthetic conduits have
shown good results without the need for a donor site.3 For longer
gaps of large peripheral nerves, cable graft ing using peripheral
sensory nerves such as sural nerve graft has been the standard of
care.1 Although some have advocated that the polarity of nerve
grafts is important, studies show that polarity of the grafts does
not seem to affect regeneration.4,5 In many cases where there is a
need for nerve grafting, the surgery is performed in a delayed setting. A clean surgical field with good healthy noninfected tissue
is important for any successful repair. Thus, nerve graft ing often
occurs after multiple trips to the OR for washout and debridement
of the affected area.

Two further scenarios worth mentioning, however, are situations where it is not immediately apparent that a nerve injury has
occurred and what to do if repair of the motor nerve is not possible
or is not successful. The first situation may occur when there is a
closed injury that causes a neurological deficit. This can occur when
an extremity is injured (e.g., in a car accident) and causes a sheer
force to the peripheral nerve. These injuries may be quite significant and can involve major structures such as the brachial plexus.
Clearly, this is different than an injury from a table saw that has
cleanly cut through an extremity. In these situations, a careful neurological examination along with studies such as electromyography
(EMG)/nerve conduction tests is helpful.6,7 These injuries are particularly difficult to treat as the diagnosis of the type of nerve injury
may not be readily apparent (neuropraxia vs. axonotmesis). In the
event that a nerve injury cannot be adequately repaired or perhaps
is impractical, such as cases where the distal innervated muscle is
no longer available due to trauma or due to time (loss of the motor
endplate due to delay in repair), other alternatives are possible. As
well, even with a timely nerve repair, often times regeneration is
not complete or is inadequate. One alternative plan includes tendon
transfer for hand injury, and this is commonly employed in order to
use muscle groups from noninjured nerve distributions. While not
a repair of the nerve, this is an important tool in the treatment of
peripheral nerve injury in restoring function to the patient.8,9 Free
tissue transfer of muscle has also been employed in treating facial
nerve injury and reanimation of the face.10,11
Lastly, nerve and extremity injury can result in significant
pain long after most of the soft tissue has healed. This can be in
the form of a neuroma or complex regional pain syndrome. It is
important to remember that nerve injury repair is just the beginning of a very long recovery for the patient and may need the support of many specialists for pain management and physical and
occupational therapy.

REFERENCES
1. Dvali L, Mackinnon S. Nerve repair, graft ing, and nerve transfers. Clin Plast Surg. 2003;30(2):203-221.
2. Pfaeffle HJ, Waitayawinyu T, Trumble TE. Ulnar nerve laceration and repair. Hand Clin. 2007;23(3):291-299, v.
3. Siemionow M, Bozkurt M, Zor F. Regeneration and repair of
peripheral nerves with different biomaterials: review. Microsurgery. 2010;30(7):574-588.
824

PMPH_CH104.indd 824

5/22/2012 6:11:08 PM

Peripheral Nerve Injury Repair

4. Nakatsuka H, Takamatsu K, Koshimune M, Imai Y, Enomoto


M, Yamano Y. Experimental study of polarity in reversing cable
nerve grafts. J Reconstr Microsurg. 2002;18(6):509-515.
5. Stromberg BV, Vlastou C, Earle AS. Effect of nerve graft polarity
on nerve regeneration and function. J Hand Surg Am. 1979;4(5):
444-445.
6. Stewart JD. Electrodiagnostic techniques in the evaluation of
nerve compressions and injuries in the upper limb. Hand Clin.
1986;2(4):677-687.
7. Malessy MJ, Pondaag W, van Dijk JG. Electromyography,
nerve action potential, and compound motor action potentials in obstetric brachial plexus lesions: validation in the

PMPH_CH104.indd 825

8.
9.

10.
11.

825

absence of a gold standard. Neurosurgery. 2009;65(4 Suppl):


A153-A159.
Cooney WP. Tendon transfer for median nerve palsy. Hand Clin.
1988;4(2):155-165.
Hastings H 2nd, Davidson S. Tendon transfers for ulnar nerve
palsy. Evaluation of results and practical treatment considerations. Hand Clin. 1988;4(2):167-178.
Terzis JK, Olivares FS. Secondary surgery in adult facial paralysis reanimation. Plast Reconstr Surg. 2009;124(6):1916-1931.
Terzis JK, Olivares FS. Long-term outcomes of free-muscle
transfer for smile restoration in adults. Plast Reconstr Surg.
2009;123(3):877-888.

5/22/2012 6:11:08 PM

CHAPTER 105

Necrotizing Soft Tissue Infections


Including Gas Gangrene
Kevin M. Schuster and Erik S. Barquist

States.7 Although several reports have claimed an increasing incidence of NSTI, there appears to be little evidence for this.1,8 The
true incidence of NSTIs remains difficult to estimate due to the
nature of difficulty in diagnosis and the associated wide range of
diagnosis that fall under the category of NSTIs. Prior epidemiologic studies have focused on single entities such as necrotizing
fasciitis. Despite the low incidence, the clinician needs to always
consider the possibility of the presence of a necrotizing infection
due to the high rate of mortality that only increases with delays
to initiation of treatment. However, maintaining vigilance represents a significant challenge, as it has also been estimated that
family physicians, internists, infectious disease specialists, and
even surgeons may encounter just one of these infections in their
whole career.8
Answer: The incidence of NSTI is currently unclear.

INTRODUCTION
Necrotizing soft tissue infection (NSTI) is the current term used to
describe severe soft tissue infections with associated tissue necrosis regardless of the anatomic region or depth of tissue involvement. Th is term encompasses previously described multiple
categories of soft tissue infections including necrotizing cellulitis,
necrotizing fasciitis, myositis, myonecrosis, Fourniers gangrene,
and gas gangrene.1 NSTI is a rapidly progressive infection with
a high lethality that has been recognized for much of recorded
medical history. Hippocrates knew of necrotizing infections that
caused flesh, sinew and bones to fall away in great quantity.
British naval surgeons described gas gangrene in the 1700s; more
recently, gas gangrene was described in casualties of the civil
war and the world wars. Pasteur identified Clostridia in 1861.
Welch and Nuttall identified Clostridia perfringens (welchii) in
1892 from gangrenous wounds.2,3 The first to report a mortality
rate for NSTI was Joseph Jones in 1871; he reported a mortality rate
of 46%.4 Mortality has improved since the time of the civil war,
but it remains significant with a range of 17% to 43% for studies reported after 1990.5,6 Because of the lack of consensus, with
multiple definitions having been used to describe NSTI, interpretation of the literature remains difficult. In addition, the low
incidence of NSTI has created a body of literature consisting
mostly of small to moderate retrospective studies from which
few conclusions can be drawn regarding optimal diagnostic
and therapeutic strategies. With the advent of the term NSTI
as well as some uniformity of classification of NSTIs, this will
likely improve.

RISK FACTORS AND CAUSATIVE


ORGANISMS
2. What are the risk factors and most common causative organisms in NSTI?
NSTIs are typically classified based on the infecting organism, although these classifications are not universally agreed
upon.9 Type I represent 80% of NSTIs seen in practice and are
polymicrobial infections caused by both gram-positive aerobes,
gram-negative aerobes, and anaerobes.10 Type II infections represent somewhat less than 20% of NSTI and are generally monomicrobial with group A streptococci (GAS) predominating.8,10
Type II, however, includes all monomicrobial infections caused
by gram-positive organisms including the increasingly common
community-associated methicillin resistant Staphylococcus aureus
(CA-MRSA).11,12 Table 105.1 identifies the most common organisms isolated when type I and type II infections were considered
in a cohort of 73 patients.13
True gas gangrene, caused by a Clostridia species, is a rare
disease with only about 3000 cases reported in the United States

INCIDENCE
1. What is the incidence of NSTI?
The incidence of necrotizing fasciitis was estimated to be 0.04
cases per 1000 person-years in an insurance study of patients
residing predominantly in the mountain west region of the United
826

PMPH_CH105.indd 826

5/22/2012 6:11:42 PM

NSTIs Including Gas Gangrene

827

Table 105.1 Species Isolated from 73 Patients with NSTI


Organism

Number of Patients (Polymicrobial)

Number of Patients (Monomicrobial)

Streptococcus species*

19

12

Staphylococcus species

21

Enterococcus species

12

Klebsiella species

14

Burkholderia pseudomallei

Vibrio vulnificus

Aeromonas species

Acinetobacter baumanii

13

Eschericia coli

12

Pseudomonas aeruginosa

10

Enterobacter species

Proteus species

Bacteroides species

Candida species

Peptostreptococcus species

Bacillus species

Clostridium species

Other

* Streptococcus species includes: Streptococcus pyogenes, group-B streptococcus, group-G streptococcus, and Streptococcus milleri.
From Giuliano et al.10

per year.14 Up to 60% of these cases occur in traumatic wounds as


a late consequence of poor wound hygiene. Clostridia perfringens
is the predominant causative organisms, but others pathogens
include C. novyi, C. septicum, C. histolyticum, C. bifermentans,
and C. fallax. The clinical infection caused by these organisms is
unique in that several exotoxins are formed which rapidly lyse tissues, and little purulence is formed by the body in response to the
infection. Different Clostridia species elaborate a different array
of exotoxins. Those of C. perfringens are the best understood.
They include -toxin or phospholipase C (PLC), 2-toxin or perfringolysin O, 6-toxin or collagenase and sporulation-associated
enterotoxin. These toxins may have evolved as a consequence of
Clostridias normal activity as a saprophyte. The most lethal of
these toxins is the -toxin, which caused profound shock when
infused as a purified protein into experimental animals. It is
cardio-depressive, hemolytic, and cytotoxic to platelets and leukocytes and increases capillary permeability. Mutant C. perfringens
that lack this enzyme are much less pathogenic and immunization
against this protein conveys protection against some of the effects
of C. perfringens infection.15
Infections with Vibrio species and other monomicrobial
infections with gram-negative rods have been categorized as
type III infections by some authors.16 These infections are often
associated with sea water contamination of open wounds; liver
disease has been implicated as a significant risk factor.17 As demonstrated in Table 105.1 these infections are generally less common

PMPH_CH105.indd 827

and represent less than 1% of NSTIs in most series. Aeromonas


hydrophilia is an organism usually found in lakes and rivers. It
can cause a gas forming infection in immunocompetent individuals that appears clinically like clostridial gas gangrene. Although
type IV is not a universally agreed upon categorization, these
infections are fungal with candida being an uncommon organism
infecting the immunocompromised.18 Zygomyotic cases including
Mucor and Rhizopus species have also been classified as type
IV and generally affect the immunocompetent.19 An additional
pathogen that has become more common in both polymicrobial
and monomicrobial infection is Acinetobacter baumannii. 20 Only
small case series have currently been reported; however, more are
likely to appear as Acinetobacter has been identified as a common
pathogen in soldiers in Iraq and Afghanistan.21 Atypical mycobacteria may also cause slow growing indolent soft tissue infections.
Among these are Mycobacterium ulcerans and Mycobacterium
marinum. These rare infections are extremely difficult and require
poly-drug treatment.22
Independent of causative organisms immunocompromised
states including diabetes, HIV infection, peripheral vascular disease, malignancy, and alcoholism have been implicated as risk factors; however, they have been most closely associated with type I
infections. All types of NSTIs have been associated with age
and obesity.23,24 Often NSTI is identified without any precipitating factors and this may be indicative of infection with CAMRSA or GAS.8

5/22/2012 6:11:42 PM

828

Surgery: Evidence-Based Practice

Answer: Type I infections are polymicrobial with immunocompromised patients most at risk. Gram-positive bacteria
including MRSA predominate. Type II infections are monomicrobial, predominantly GAS. These occur in immunocompetent
individuals but there may be a recent history of remote streptococcal infection.

PATIENT PRESENTATION
3. What are the appropriate diagnostic tests for NSTI?
Patient presentation correlates loosely with the type of infection.
Polymicrobial or type I infections are generally slowly progressive
evolving over several days. It may follow abdominal or perineal
surgery where gut organisms have access to subcutaneous tissues.
In type II infections due to GAS, symptoms develop much more
rapidly; however, an obvious source for the infection is often lacking. Patients may develop infections after hematogenous spread
from a distal source such as the case with pharyngitis or vaginitis.
In a series of 37 patients with GAS NSTI, 21 patients presented
with gastrointestinal complaints, influenza-like symptoms, and
complaints of muscle strain.25 Later in the course of the disease,
the patient may present with signs of severe sepsis and septic
shock. Patients with fungal NSTI may provide a history of trauma
involving contaminated soil and foreign travel.19 Raw seafood
ingestion or wound exposure to seawater should raise suspicion
for infection with Vibrio spp.17
The presenting symptom most uniformly reported in series
of patients with NSTI is pain.1,16,25,26 The pain is commonly
described as being inconsistent with the degree of skin changes
that are present. It is usually severe, with anesthesia in the central
areas due to ischemic death of neurons supplying these areas.27
Other common physical examination fi ndings include erythema
and edema classically described as being present beyond the borders of the affected skin. A complete list of common presenting
symptoms is included in Table 105.2.23,24 A close association in
Table 105.2 Common Presenting Symptoms of NSTI
Symptom

% of Patients

Pain

85

Erythema

78

Edema/swelling

78

Discoloration

30

Cellulitis

75

Fever

49

Crepitus

31

Vesicles

20

Disorientation

16

Discharge

47

Induration

45

Skin slough/necrosis

31

Weakness/paraplegia

Numbness

PMPH_CH105.indd 828

children with recent varicella zoster often makes the early skin
findings difficult to interpret and as such there is often a delay in
diagnosis.28

Laboratory Findings
Findings from a laboratory examination are typical of those of the
systemic inflammatory response syndrome, severe sepsis or septic
shock. Leukocytosis and leukopenia are common as is thrombocytopenia, anemia, and coagulation abnormalities. Acute kidney
injury is also common due to both sepsis and, possibly in some cases,
myonecrosis as indicated by rising creatinine kinase. Hypocalcemia
is common as is hyponatremia, and if serum sodium is less than 135
mmol/L this suggests NSTI.29,30 Both hyponatremia and an elevated
serum lactate are predictive of death in cases of NSTI.30 In terms
of diagnosis, the most clinically relevant lab results are the components of the Laboratory Risk Indicator for Necrotizing Fasciitis
(LRINEC) score.31 This scoring system includes c-reactive protein,
leukocyte count, hemoglobin, sodium, creatinine, and glucose. On
the basis of each laboratory finding, a score is assigned and the total
score is calculated by addition. A score of 6 or more is suspicious for
NSTI and a score of 8 or more is strongly predictive.31 Blood cultures
are positive in monomicrobial NSTI in 11% to 60% of cases but yield
is lower in polymicrobial infections.16 The infecting organism(s) can
most likely be determined by tissue biopsy and culture of the tissue
at the advancing edge of the necrotic infection.

Imaging Findings
Magnetic resonance imaging (MRI) is often identified as the modality of choice for imaging of soft tissue infections.32-34 Computed
tomography (CT) has also been used but no study has directly
compared these modalities. Recently, one study of 67 patients estimated that sensitivity of CT for 16-slice or better CT scanners is
100% with a specificity of 75%.35 CT scanning will likely become
the preferred initial study not only because there is no direct comparison of CT and MRI, but also because of the greater availability
of CT scanners, the shorter time required for a CT scan, and the
lower patient compliance requirement.
Findings on both CT and MRI indicative of NSTI include
thickening of fascial planes and fluid tracking, and as the disease
progresses, small vessel thrombosis and gas in the deeper fascial
planes.36 On MRI, T1-weighted images demonstrate decreased signal
intensity within the subcutaneous tissues and T2-weighted images
demonstrate increased signal intensity. Fluid sensitive images
will identify increased signal intensity both above and below the
fascial level.37 Specific findings on CT indicative of NSTI include
asymmetrical thickening of the fascia and muscle, nonenhancing
tissues, fluid collections, and gas across tissue plains.35 Recently,
a scoring system has been developed based on CT findings. This
system identifies fascial air, muscle or fascial edema, and fluid
tracking in the fascial planes as predictors of NSTI.38 Because the
time for diagnosis is critically important, only studies that can
be performed rapidly should be considered as surgical exploration provides definitive diagnosis with only a minimal increase in
morbidity, discomfort, and cost.
Answer: History and physical examination are important. If
suspicion is high for NSTI or infection is progressing rapidly, diagnostic surgery is warranted. For more indolent cases, serum lab values, CT scanning, or MRI are helpful (Grade C recommendation).

5/22/2012 6:11:42 PM

NSTIs Including Gas Gangrene

MANAGEMENT
Resuscitation and Antibiotics
4. What is the appropriate approach to antibiotic therapy for
NSTI, and are bacterial cultures important?
If necessary, aggressive early resuscitation with IV fluids and
vasopressors and inotropes is essential to achieve appropriate
hemodynamic, urinary output, and mixed venous oxygen saturation targets. No specific protocols have been developed for the
resuscitation of NSTI patients; however, severe sepsis protocols
are applicable. Because physiology in these patients represents a
septic-type shock, a protocol-based approach is likely to improve
outcomes. 39,40 The surviving sepsis campaign guidelines represent
one approach to achieve these goals.41 Any systematic approach
that initially optimizes fluid resuscitation, while maintaining
a hemostatic approach to prevent dilutional coagulopathy in a
preoperative patient, is likely to be most effective while continuously assessing the response to resuscitation. Vasopressors and
inotropes can be added as necessary to improve vascular resistance and cardiac performance. Other adjuvant therapies for
severe sepsis including low-dose steroids and activated protein C
can be considered.
Initiation of appropriate broad spectrum antibiotic therapy is
essential as inadequate antibiotic therapy leads to excess mortality
in hospital-treated infections including sepsis and even NSTIs.42,43
Similarly, initial therapy that covers MRSA has been demonstrated
to improve outcomes for skin and soft tissue infections when this
organism is present.44 In one study of patients with MRSA infection who were treated with early incision and drainage, there were
no failures when antibiotics were active against MRSA compared
with failure in 67% of cases where inactive antibiotics were the initial treatment.45 Equally important is the timeliness of initiation of
appropriate antibiotics for patients experiencing septic shock. In
one study of sepsis with hypotension, it was reported that for every
hour delay in appropriate antimicrobial therapy, in hours 2 through
6 after presentation, mortality increased by 7.6%.46 Initial therapy
should therefore be given immediately with an IV agent active
against MRSA (vancomycin, linezolid, daptomycin).47 Consideration should also be given to the addition of clindamycin or linezolid that are known to inhibit toxin production by staphylococcal
and streptococcal organisms.9 Broad spectrum antibiotics covering gram-negative organisms and anaerobes especially those found
in Table 105.1 should be a part of the initial combination therapy. If
Vibrio spp. is suspected as a causative organism, antibiotic options
include combination doxycycline and ceftazidime or alternatively
ciprofloxacin can be used as monotherapy.16 Antibiotic selection in
cases where Acinetobacter baumannii is suspected is difficult due
to the often multidrug resistant nature of this organism.20 However, empiric coverage of this organism should be considered for
soldiers returning from combat operations in Afghanistan or Iraq,
or for those patients who had a recent infection with this organism
at a remote site, such as a pneumonia infection.20
Equally important is antibiotic de-escalation. Ideally, blood
cultures are obtained prior to initiation of antibiotic therapy. Tissue cultures are mandatory as well, in order to target the antibiotic
therapy to the exact causative organisms and sensitivities. Antibiotic therapy should be tailored when the culture results become
available.9

PMPH_CH105.indd 829

829

Answer: Broad spectrum antibiotics covering gram-negative


organisms, gram-positive organisms, including MRSA, and anaerobes are the initial antibiotics of choice. Antibiotic therapy should
be started immediately and suspicion for unusual organisms may
warrant alternative antibiotic choices. Both blood and tissue cultures at the time of surgery can help with antibiotic de-escalation
(Grade B recommendation).

Surgical Debridement
5. What is the appropriate timing and extent of surgery for
NSTI?
The most important component of therapy for NSTI is prompt,
aggressive surgical debridement. Multiple studies have demonstrated this important relationship. In one study, the difference
in time to initial debridement was 1.2 days in survivors versus
3.1 days in nonsurvivors.48 In another study, a delay in surgical
debridement, which was unfortunately poorly defined, resulted
in a 38% mortality compared with 4.2% (P < .001) with surgical
debridement immediately upon diagnosis.49 Delay as represented
by transfer from an outside institution resulted in a mortality of
75% compared with 7% in direct admissions in a third study.50
Although the evidence is not as strong as that for timing of debridement, completeness of debridement is also important. Debridement should encompass the entire area of skin changes and result
in boundaries of excision that consist of healthy bleeding tissue.1
Because, despite what appears to be complete initial debridement,
if infection persists, repeat debridements are usually required.48-50
Answer: Surgery should not be delayed for any reason and
should be aggressive to encompass all nonviable tissue (Grade B
recommendation).

Intravenous Immunoglobulin
6. Is intravenous immunoglobulin an optional adjuvant therapy for streptococcal or staphylococcal infections only?
Adjuvant therapy with intravenous immunoglobulin (IVIG) is controversial in gram-negative sepsis. Its use in NSTI has the theoretical basis of binding of gram-positive exotoxins. Clinical studies of
IVIG on NSTI have been completed in blinded but nonrandomized fashion, and there has been a suggestion of benefit.51,52 Use of
IVIG should only be contemplated if streptococcal or staphylococcal infection is suspected and subsequently proven by culture.53,54
Answer: IVIG is an optional adjuvant therapy for streptococcal or staphylococcal infections only (Grade B recommendation).

Hyperbaric Oxygen
7. What is the role of hyperbaric oxygen in the treatment of
NSTI?
Infections caused by Clostridia spp. whether isolated or as a synergistic organism are most likely to derive benefit from hyperbaric oxygen (HBO). HBO inhibits -toxin production and is
believed to enhance the bactericidal action of neutrophils. 55
However, studies on the use of HBO for NSTI have been confl icting in their results. One recent retrospective comparison demonstrated no benefit in a heterogenous patient population where
patient selection for HBO was based on the institution in which

5/22/2012 6:11:42 PM

830

Surgery: Evidence-Based Practice

they were treated.56 Other similar reports contain confl icting evidence in terms of mortality, need for additional debridements,
need for amputation, need for antibiotics, and hospital length
of stay.57-60 Unfortunately, all of these studies are retrospective
and suffer from significant selection bias. No study defines the
reasons why some patients received HBO and others did not,
except in the cases where some centers provided this therapy and
others did not. HBO may be of benefit in NSTI or some subgroups
of NSTI; however, without a prospective randomized study, the
definitive answer will not be known.
Answer: HBO is an optional therapy that may improve mortality, volume of debridement, or limb salvage (Grade D recommendation).

SUMMARY
NSTIs cover a wide range of pathologic virulence from the rapidly progressive gas gangrene due to Clostridium sp. or GAS to the
indolent ulcers of atypical mycobacterial infections. The treatment
of all of these variants depends on prompt diagnosis, ideally based
on physical examination, resuscitation of the patient, meticulous
surgical removal of all nonviable tissue, appropriate and timely
IV antibiotic therapy, and usually second-look surgery. While
the mortality from these infections has dropped significantly in
the last century, under-treated lesions still remain a source of
mortality; the most common error in the management of these
lesions is delayed or inadequate surgical debridement.

Clinical Question Summary


Question

Answer

Levels of
Evidence

Grade of
Recommendation

References

1 What is the incidence


of NSTI?

The incidence of NSTI is currently unclear.

N/A

1, 7, 8

2 What are the risk


factors and most
common causative
organisms in NSTI?

Type I infections are polymicrobial with


immunocompromised patients most at
risk. Gram-positive bacteria including
MRSA predominate. Type II infections
are monomicrobial, predominantly
GAS. These occur in immunocompetent
individuals but there may be a recent
history of remote streptococcal
infection.

N/A

8, 10-13

3 What are the


appropriate diagnostic
tests for NSTI?

History and physical examination are most


important. If suspicion is high for NSTI or
infection is progressing rapidly, diagnostic
surgery is warranted. For more indolent
cases, serum lab values, CT scanning, or
MRI are helpful.

29-31, 35, 38

4 What is the
appropriate approach
to antibiotic therapy
for NSTI, and are
bacterial cultures
important?

Broad spectrum antibiotics covering


gram-negative organisms, grampositive organisms, including MRSA,
and anaerobes are the initial antibiotics
of choice. Antibiotic therapy should
be started immediately, and suspicion
for unusual organisms may warrant
alternative antibiotic choices. Both blood
and tissue cultures at the time of surgery
can help with antibiotic de-escalation.

9, 16, 42-46

5 What is the appropriate


timing and extent of
surgery for NSTI?

Surgery should not be delayed for any


reason and should be aggressive to
encompass all nonviable tissue.

48-50

6 What is the role of


IVIG in treatment of
NSTI?

IVIG is an optional adjuvant therapy


for streptococcal or staphylococcal
infections only.

51-54

7 What is the role of


HBO in the treatment
of NSTI?

HBO is an optional therapy that may


improve mortality, volume of
debridement, or limb salvage.

55-60

NSTI, necrotizing soft tissue infection; IVIG, intravenous immunoglobulin; HBO, hyperbaric oxygen; MRSA, methicillin resistant Staphylococcus aureus;
GAS, group A streptococcus.

PMPH_CH105.indd 830

5/22/2012 6:11:42 PM

NSTIs Including Gas Gangrene

REFERENCES
1. Sarani B, Strong M, Pascua J, Schwab CW. Necrotizing fasciitis:
current concepts and review of the literature. J Am Coll Surg. 2009;
208:279-288.
2. Hitchcock CR: Gas gangrene. In: Gustilo RB, ed., Orthopedic
Infections. Philadelphia, PA: WB Saunders; 1989:190-201.
3. Lindsey D. Soft tissue infections. Emer Med Clin NA. 1992;10:
737-751.
4. Jones J. Surgical Memoirs of the War of the Rebellion: Investigation upon the Nature, Causes and Treatment of Hospital Gangrene
as Prevailed in the Confederate Armies 1861-1865. New York: US
Sanitary Commission, 1871.
5. Anaya DA, McMahon K, Nathens AB, et al. Predictors of mortality and limb loss in necrotizing soft tissue infections. Arch Surg.
2005;140:151-157.
6. McHenry CR, Piotrowski JJ, Petrinic D, Malangoni MA. Determinants of mortality for necrotizing soft-tissue infections. Ann
Surg. 1995;221:558-563.
7. Ellis Simonsen SM, van Orman ER, Hatch BE, et al. Cellulitis incidence in a defined population. Epidemiol Infec. 2006;134:293-299.
8. Anaya DA, Dellinger EP. Necrotizing soft-tissue infection: diagnosis and management. Clin Infect Dis. 2007;44:705-710.
9. Napolitano LM. Severe soft tissue infections. Infect Dis Clin N
Am. 2009;23:571-591.
10. Giuliano A, Lewis Jr F, Hadley K, et al. Bacteriology of necrotizing fasciitis. Am J Surg. 1977;134:52-57.
11. Young LM, Price Cs. Community-acquired methicillin-resistant
Staphylococcus aureus emerging as an important cause of necrotizing fasciitis. Surg Infect. 2008;9:469-474.
12. Miller LG, Perdreau-Remington F, Rieg G, et al. Necrotizing
fasciitis caused by community-associated methicillin-resistant
Staphylococcus aureus in Los Angeles. N Engl J Med. 2005;352:
1445-1453.
13. Wong CH, Chang HC, Pasupathy S, et al. Necrotizing fasciitis:
clinical presentation, microbiology, and determinants of mortality. J Bone Joint Surg Am. 2003;85:1454-1460.
14. Hart GB, Lamb RC, Strauss MB. Gas gangrene. J Trauma. 1983;
23:991.
15. Rood JI: Virulence genes of Clostridium perfringens. Annu Rev
Microbiol. 1998:52;333-360.
16. Morgan MS. Diagnosis and management of necrotizing fasciitis:
a multiparametric approach. J Hosp Infect. 2010;75:149-157.
17. Bross MH, Soch K, Morales R, Mitchell RB. Vibrio vulnificus infection: diagnosis and treatment. Am Fam Physician.
2007;76:539-544.
16. Cook DA, Heiner JP, Rao VK. Necrotizing candidal fasciitis following hip surgery. Orthopedics. 1990;13:768-770.
17. Jain D, Kumar Y, Vasishta RK, et al. Zygomycotic necrotizing
fasciitis in immunocompetent patients: a series of 18 cases. Mod
Pathol. 2006;19:1221-1226.
18. Guerrero DM, Perez f, Conger NG, et al. Acinetobacter baumannii-associated skin and soft tissue infections: recognizing a
broadening spectrum of disease. Surg Infect. 2010;11:49-57.
19. Scott P, Deye G, Srinivasan A, et al. An outbreak of multidrugresistant Acinetobacter baumannii-calcoaceticus complex infection in the US military health care system associated with
military operations in Iraq. Clin Infect Dis. 2007;44:1577-1584.
22. Kirschner RA, Parker BC, Falkinham JO. Epidemology of infection by nontuberculosis mycobacterium: Mycobacterium avium,
Mycobacterium intracellulare and Mycobacterium scrofulaceum
in acid, brown-water swamps of the southeastern United States

PMPH_CH105.indd 831

23.

24.

25.

26.
27.

28.

29.

30.

31.

32.

33.
34.

35.

36.
37.

38.

39.

40.

41.

42.

831

and their association with environmental variables. Am Rev


Respir Dis. 1992:145;271.
Childers BJ, Potyndy LD, Nachreiner R, et al. Necrotizing fasciitis: a fourteen-year retrospective study of 163 consecutive
patients. Am J Surg. 2002;68:109-116.
Elliott DC, Kufera JA, Myers RA. Necrotising soft tissue infections. Risk factors for mortality and strategies for management.
Ann Surg. 1996;224:672-683.
Bisno AL, Cockerill FR, Bermudez CT. The initial outpatientphysician encounter in Group A Streptococcal Necrotizing Fasciitis. Clin Infect Dis. 2000;31:607-608.
Bellapianta JM, Ljungquist K, Tobin E, Uhl R. Necrotizing fasciitis. J Am Acad Orthop Surg. 2009;17:174-182.
Chelsom J, Halstensen A, Haga T, et al. Necrotising fasciitis due
to group A streptococci in western Norway: incidence and clinical features. Lancet. 1994;344:3369-3374.
Fustes-Morales A, Gutierrez-Castrellon P, Duran-Mckinster C,
et al. Necrotizing fasciitis: report of 39 pediatric cases. Arch Dermatol. 2002;38:893-899.
Wall DB, de Virgilio C, Black S, et al. Objective criteria may assist
in distinguishing necrotizing fasciitis from non-necrotizing soft
tissue infection. Am J Surg. 2000;179:17-21.
Yaghoubian A, de Virgilio C, Dauphine C, Lewis RJ, Lin M. Use
of admission serum lactate and sodium levels to predict mortality in necrotizing soft tissue infections. Arch Surg. 2007;142:
840-846.
Wong CH, Khin LW, Heng KS, Tan KC, Low CO. The LRINEC
(laboratory risk indicator for necrotizing fasciitis from other soft
tissue infections). Crit Care Med. 2004;32:1535-1541.
Beltran J, Noto AM, McGhee RB, Freedy RM, McCalla MS.
Infections of the musculoskeletal system: high-field-strength
MR imaging. Radiology. 1987;164:449-454.
Beltran J. MR imaging of soft tissue infection. Magn Reson Imaging Clin N Am. 1995;3:743-751.
Beauchamp NJ, Scott WW Jr, Gottlieb LM, Fishman EK. CT
evaluation of soft tissue and muscle infection and inflammation: a systemic compartmental approach. Skel Rad. 1995;24:
317-324.
Zacharias N, Velmahos GC, Salama A, et al. Diagnosis of necrotizing soft tissue infections by computed tomography. Arch Surg.
2010;145:452-455.
Turecki MB, Taljanovic MA, Stubbs AY, et al. Imaging of musculoskeletal soft tissue infections. Skeletal Radiol. 2010;39:957-971.
Yu JS, Habib P. MR imaging of urgent inflammatory and infectious conditions affecting the soft tissues of the musculoskeletal
system. Emerg Radiol. 2009;16:267-276.
McGillicuddy EA, Lischuk AW, Schuster KM, et al. Development
of a CT-based scoring system for necrotizing soft tissue infections. J Trauma. 2011;70:894-899.
Micek SST, Roubinian N, Heuring T, et al. Before-after study of
a standardized hospital order set for the management of septic
shock. Crit Care Med. 2006;34:2707-2713.
Nguyen HB, Corbett SW, Steele R, et al. Implementation of a
bundle of quality indicators for the early management of severe
sepsis and septic shock is associated with decreased mortality.
Crit Care Med. 2007;35:1105-1112.
Dellinger RP, Levy MM, Carlet JM, et al. Surviving sepsis campaign: international guidelines for management of severe sepsis
and septic shock: 2008. Crit Care Med. 2008;36:296-327.
Kollef MH, Sherman G, Ward S, et al. Inadequate antimicrobial treatment of infections: a risk factor for hospital mortality
among critically ill patients. Chest. 1999;115:462-474.

5/22/2012 6:11:42 PM

832

Surgery: Evidence-Based Practice

43. Garnacho-Montero J. Impact of adequate empirical antibiotic


therapy on the outcome of patients admitted to the intensive care
unit with sepsis. Crit Care Med. 2003;31:2742-2751.
44. Ruhe JJ, Smith N, Bradsher RW, et al. Community-onset methicillin-resistant Staphylococcus aureus skin and soft tissue infections: impact of antimicrobial therapy on outcome. Clin Infect
Dis. 2007;44:777-784.
45. Chuck EA, Frazee BW, Lambert L, McCabe R. The benefit of
empiric treatment for methicillin-resistant Staphylococcus
aureus. J Emerg Med. 2010;38:576-571.
46. Kumar A, Roberts D, Wood KE, et al. Duration of hypotension
before initiation of effective antimicrobial therapy is the critical
determinant of survival in human septic shock. Crit Care Med.
2006;34:1589-1596.
47. Steven DL, Bisno AL, Chambers HR, et al. Infectious Disease Society of America. Practice guidelines for the diagnosis and management of skin and soft tissue infections. Clin Infect Dis. 2005;
41:1373-1406.
48. Elliot DC, Kufera JA, Myers RAM. Necrotizing soft tissue infections risk factors for mortality and strategies for management.
Ann Surg. 1996;224:672-683.
49. Bilton BD, Zibari GB, McMillan RW, et al. Aggressive surgical
management of necrotizing fasciitis serves to decrease mortality:
a retrospective study. Am Surg. 1998;64:397-400.
50. Voros D, Pissiotis C, Georgantas D, et al. Role of early and extensive surgery in the treatment of severe necrotizing soft tissue
infection. Br J Surg. 1993;80:1190-1191.
51. Kaul R, McGeer A, Norrby-Teglund A, et al. Intravenous immunoglobulin therapy for streptococcal toxic shock syndrome a
comparative observational study. The Canadian Streptococcal
Study Group. Clin Infec Dis. 1999;28:800-807.

PMPH_CH105.indd 832

52. Alejandria MM, Lansang MA, Dans LF, Mantaring JB. Intravenous immunoglobulin for treating sepsis and septic shock.
Cochrane Database Syst Rev. 2002;1:CD001090
53. Darabi K, Abdel-Wahab O, Dzik WH. Current usage of intravenous immune globulin and the rationale behind it: the Massachusetts General Hospital data and a review of the literature.
Transfusion. 2006;46:741-753.
54. Schrage B, Duan G, Yang LP, Fraser JD, Proft T. Different preparations of intravenous immunoglobulin vary in their efficacy to
neutralize streptococcal superantigens: implications for treatment of streptococcal toxic shock syndrome. Clin Infect Dis.
2006;43:743-746.
55. Escobar SJ, Slade JB Jr, Hunt TK, Cianci P. Adjuvant hyperbaric
oxygen therapy (HBO2) for treatment of necrotizing fasciitis
reduces mortality and amputation rate. Undersea Hyperb Med.
2005;32:437-443.
56. George ME, Rueth NM, Skarda DE, et al. Hyperbaric oxygen
does not improve outcome n patients with necrotizing soft tissue
infection. Surg Infect. 2009;10:21-28.
57. Brown DR, Davis NL, Lepawsky M, et al. A multicenter review
of the treatment of major truncal necrotizing infections with
and without hyperbaric oxygen therapy. Am J Surg. 1994;167:
485-489.
58. Shupak A, Shoshani O, Goldenberg I, et al. Necrotizing fasciitis:
an indication for hyperbaric oxygenation therapy? Surgery. 1995;
118:873-878.
59. Wilkinson D, Doolette D. Hyperbaric oxygen treatment and
survival from necrotizing soft tissue infection. Arch Surg. 2004;
139:1339-1345.
60. Mindrup SR, Kealey GP, Fallon B. Hyperbaric oxygen for the
treatment of Fourniers gangrene. J Urol. 2005;173:1975-1977.

5/22/2012 6:11:42 PM

CHAPTER 106

Necrotizing Soft Tissue Infections


Mark D. Sawyer

INTRODUCTION AND DEFINITIONS

NECROTIZING FASCIITIS

Necrotizing soft tissue infections (NSTIs) are a group of highly


morbid and lethal infections. Successful management entails
early recognition, aggressive operative debridement (which may
include amputation), aggressive antibiotic therapy, and supportive care in an intensive care unit (ICU). It is a difficult subject
to approach in an evidence-based manner; such an uncommon
and highly lethal disease process makes large, prospective, randomized trials difficult to design and implement. Further complicating the picture is the fact that a large proportion of current
practice is by necessity based on individual observations and
deductions concerning the disease, which, as one might expect,
can engender strong biases and suspicions that at times seem to
be in inverse proportion to the amount of available evidence.
Thus, some of the evidence-based discussions of questions concerning NSTIs have more of an appearance of a process of eliminationdeciding which tentative conclusions are probably not
justified because there is insufficient evidence to support them
in either quality, quantity, or both. The list of conclusions supported by solid statistical evidence is short. Although many have
called for large, cooperative multicenter efforts to glean more
substantial evidence from the thousand or so cases per year that
occur,1,2 this has yet to come to fruition.
Although NSTIs comprise a wide variety of clinical scenarios as reflected by the bewildering array of terms used in
the literature, a simple divisionpredominantly fascial versus
muscular involvementcategorizes these infections reasonably both in terms of their behavior and a pragmatic approach
to empiric treatment.3,4 Although anyone may be affected, the
immunocompromised and debilitatedmost commonly those
with advancing age and diabetes mellitusare disproportionately represented in terms of acquiring the disease and in suffering poorer outcomes.

The Disease
Necrotizing fasciitis is an infection involving the investing fascia of
muscle, primarily the superficial layer, and may secondarily involve
a modest amount of juxtaposed fat and muscle. It has a predilection
for the immunocompromised, in which it is more morbid and lethal
as well. Originally described as a streptococcal or streptococcalpredominant infection process, 5,6 it is usually a polymicrobial
infection, although monomicrobial forms of the disease (Vibrio,
Pseudomonas, Klebsiella, and others) exist as well. Studies carefully culturing the tissues may show a mix of gram-positive, gramnegative, and anaerobic bacteria, as well as candidal species in some.
Necrotizing fasciitis is described as a rapidly progressive process, but some patients may describe a relatively indolent period
prior to seeking medical attention, with subsequent decompensation giving the outward appearance of rapid progression.7,8

Diagnosis
Open biopsy of suspected tissues has been the standard of care
for diagnosis, although radiographic studies such as computed
tomography (CT) scans can provide useful data regarding the
location and extent of disease.

Mainstay Therapy
There are two cornerstones of initial therapy: expeditious and
complete debridement and broad-spectrum antimicrobial therapy. Immediate initiation of empiric broad-spectrum antimicrobial therapy based on an anti-streptococcal component is a
key; awaiting culture or even Gram stain data to guide therapy
would constitute an unnecessary and potentially dangerous

833

PMPH_CH106.indd 833

5/22/2012 6:12:11 PM

834

Surgery: Evidence-Based Practice

delay. With one study using careful culture techniques showing Candida species in a majority of patients and the dangers of
superinfection following potent broad-spectrum antimicrobial
therapy, many would advocate empiric therapy with an antifungal agent as well.
The other unequivocal cornerstone of therapy is expeditious
and complete excision of the infected and necrotic fascia to prevent further progression and begin the healing process, although
there is some controversy regarding whether a staged approach
versus a complete initial resection is superior. Regardless of initial philosophy, returning to the operating room for second-look
procedures to at least assess if not complete the resection process
is ubiquitous. Necrotizing fasciitis is a progressive disease, and
ensuring that progression has been halted is mandatory. Though
excision and debridement can be debilitating and disfiguring,
completeness is essential to halt the progression of disease and
maximize survival. Following the initial phase, a prolonged healing convalescent phase is usual in survivors, with care of open
wounds that may constitute a large percentage of the patients
body surface area. In addition to standard techniques for dressing
and closing such wounds, newer technologies, such as vacuumassisted wound closure devices, may be helpful.

Supplemental Therapy
There are a number of therapies that have been used in NSTIs to
try and improve outcome, such as hyperbaric oxygen and immunoglobulin therapy. The rationale for the former had its genesis
in the treatment of anaerobic NSTIs such as clostridial necrotizing myositis, and the latter as an attempt to improve treatment of
aggressive streptococcal infections and their complications, such
as streptococcal toxic shock syndrome. Although theoretically
attractive, neither has definitively proven itself as a mainstay of
treatment in NSTIs.

However, although the CT characteristics of NSTIs are well delineated, they may not be specific, as exemplified by elements such
as fascial thickening and edema without asymmetry. Other more
specific findings, such as gas within soft tissues, are not ubiquitous,
and therefore, their absence does not rule out the disease.10,11 MRI
findings have been found to be perhaps even more nonspecific;
one author found the MRI findings similar between necrotizing
fasciitis, dermatomyositis, and posttraumatic muscle injury.12 If
it does not delay definitive surgical therapy, CT may help in planning a thorough surgical intervention by showing the extent of
disease, but it should not be relied on to rule in or out the diagnosis of NSTI.
Answer: The standard for diagnosis in NSTI is clinical; confirmation by open biopsy (Grade C recommendation).
2. Which is a better approach to initial resection in NSTI,
staged or complete?
There has been little in the literature to suggest a standardized approach to NSTI. Certainly, the objective is to remove all
necrotic and infected tissue as quickly as possible. Whether this
may be achieved in one operative intervention, however, depends
heavily on the patients ability to tolerate extended, aggressive
resection. Regardless of whether the first procedure is considered
complete, nearly all patients will require at least one second-look
procedure to ensure a lack of disease progression. Recently, Wong
et al., advocated a standardized approach to resection, involving a
complete resection in the first procedure, with second-look operations to follow.13 Though the approach seems sensible in those who
will tolerate their complete approach to the initial procedure, it is
not on the basis of randomized data but the authors considered
approach to the problem.
Answer: As complete an approach as the patient will tolerate
(Grade C recommendation).
3. Is there convincing evidence for the use of hyperbaric oxygen therapy in the treatment of NSTIs?

NECROTIZING MYOSITIS
The most common eponyms for necrotizing muscle infections are
gas gangrene, clostridial/streptococcal myonecrosis, and necrotizing myositis. Necrotizing myositis is simple, descriptive, and
alliteratively associates the disease process with its fascial cousin.
The infection infects, spreads, and necroses entire muscle compartments with celerity; it is rapidly progressive and in contradistinction to necrotizing fasciitis has no recognized indolent
variants. Pragmatically, this means that exceptionally aggressive
surgery, such as proximal amputation, may be required to gain
control of the disease process before the patient succumbs, which
may occur within hours of presentation. In further contradistinction to necrotizing fasciitis, necrotizing myositis is usually
a monomicrobial infection, most commonly a toxin-producing
Clostridium or Streptococcus species.
1. Is open biopsy still the standard for diagnosis of NSTI, or
has it been supplanted by radiographic studies?
The standard of diagnosis in NSTIs is clinical diagnosis, confirmed
by open biopsy with frozen section,3,4,9 but imaging modalities
particularly CT and magnetic resonance imaging (MRI)have
developed ever finer resolution and sophistication in the software.

PMPH_CH106.indd 834

Hyperbaric oxygen (HBO) therapy was originally devised as a


way to treat decompression sickness after deep underwater diving
(the bends). At many atmospheres of depth, more nitrogen is solubilized in the bloodstream, and too-rapid ascent results in nitrogen desolubilizing out of the bloodstream as bubbles, which then
cause gas emboli. The use of HBO is currently unregulated, and
usage ranges from legitimate and proven use (treatment decompression sickness) to therapeutic and experimental use in medicine such as carbon monoxide poisoning and NSTIs to oxygen
barlike operations hawking sessions for their purported general
health benefits.
HBO therapy is a theoretically attractive potential therapy
for NSTIs, using oxygen as a direct toxin to combat anaerobic
bacteria.14-16,34 What comparative studies exist are generally not
randomized, and controls may be historical. A recent retrospective study compared patients at two tertiary academic centers in
a single city; one of the academic centers used HBO as a regular
part of therapy for NSTIs, and the other did not. There was no
statistically significant difference in outcomes. 36 Another recent
comparative but nonrandomized study in a small number of
patients showed a shorter length of treatment when using HBO.17
Most studies of NSTIs are observational in nature;18-21 there are
synergistic factors that make randomized controlled trials in the

5/22/2012 6:12:11 PM

Necrotizing Soft Tissue Infections

NSTIs difficult to complete. The first is the rarity of the infections.


It can take a single center decades to accrue a few dozen cases,
making adequate statistical analysis difficult as well as weakening
any conclusions by virtue of the rapid general advances made in
medicine over such a long period of timethe longer the study,
the less comparable the first patients entered into such a study are
to the last patients. The second factor is the zeal and evangelism
that proponents of HBO therapy for NSTIs maintain despite any
solid statistical evidence to support its use in these infections.
Some have gone so far as to say that randomized controlled trials
of its use are unethical because (in their minds) it is so clearly beneficial. This attitude may be compounded by the long timespan
between patients, in fact, there are studies spanning decades that
use the pre-hyperbaric chamber era as a historical control for
the post-hyperbaric chamber era. A further bias is likely engendered by the reasons for the purchase and use of these chambers.
Given their cost, it is unlikely to be purchased if it was not thought
to be efficaciousor profitablea priori. The same concerns apply
post facto. The strength with which HBO use is championed by its
supporters is at odds with the extant literature, which is bereft
of any solid objective evidence supporting its use as a standard
component of therapy.
Answer: No. HBO should not be part of the standard treatment for NSTIs. More rigorous trials are needed (Grade C
recommendation).
4. Is immunoglobulin therapy part of standard care for
NSTIs?
Because streptococcal species have been strongly implicated in
NSTIs since their initial description, and immunoglobulin has
been used in the treatment of Streptococcus pyogenes (group a
streptococcus) and streptococcal toxic shock syndrome, it is reasonable to examine whether immunoglobulin therapy has a salutary effect on NSTIs.22 Immunoglobulin therapy has been used
in an attempt to improve survival in cases of streptococcal toxic
shock syndrome.23,24 Although case reports and observational
studies have been encouraging, the efficacy of gamma globulin
in streptococcal toxic shock syndrome has not been confirmed by
randomized studies.
Answer: Debatable. Although intravenous immunoglobulin
treatment may be helpful, it has not been proven to improve outcomes (Grade C recommendation).
5. Is drotrecogin alpha a helpful adjunct in NSTIs?
Drotrecogin alpha (recombinant human activated protein C,
APC) has been shown in one large multicenter study to decrease
the mortality in patients with sepsis and a high risk of death,25 but
there has been considerable controversy regarding the validity of
the results.26,27 Although the study was stopped early because of

835

demonstration of efficacy, there has been no confirmatory trial,


and other trials of APC have either shown no effect on mortality in
other groups, or a higher incidence of bleeding complications.26-30
NSTIs were not examined as a subgroup in the initial study,
but they could be thought of as having a favorable profi le by the
parameters of the original study, namely sepsis with a high incidence of death. Rosing et al. retrospectively examined the use of
APC in NSTI patients over a 3-year period in a small (ten) group of
patients.31 Using a predictive model of mortality in NSTIs formulated at their institution as a comparison, there was no statistically
significant difference between the predicted mortality (18%) and
their patients (1/10, 10% P = .2). They did not note an increase in
bleeding complications in their patients, but this should be taken
in the context of the expected bleeding complication rate of APC
of only 2% to 4%.
Answer: Debatable. Although one could justify the use of
APC in NSTIs with sepsis and a high risk of death on the basis of
the original APC trial, the extant data regarding its use specifically for this disease process is very weak, especially considering
the potential for bleeding complications in patients undergoing
extensive resectional debridements (Grade C recommendation).
6. Are there new adjunctive therapies to consider?
In an interesting report, extracorporeal membrane oxygenation
was performed in two patients with NSTIs who almost certainly
would not have survived without its use; both survived.32 Another
report detailed the use of plasmapheresis as an adjunctive therapy
to surgery and antibiotics in a patient who also survived.33
Answer: New treatment strategies are being attempted and
reported, but there are no new therapies that have been proven to
affect the outcome in NSTIs by evidence-based criteria.

CONCLUSIONS
NSTIs are uncommon, highly lethal infections requiring rapid
diagnosis and treatment to achieve optimal outcomes. With only
a thousand or so cases a year across the United States, however,
prospective randomized trials are difficult, and in the case of a
single institution, near impossible. With agreement on the basis
of therapyaggressive surgical debridement and broad-spectrum
antimicrobials35the important questions presently involve secondary therapies, which at best remain unproven. To obtain quality
data for questions such as the use of HBO, polyclonal immunoglobulin administration, and activated protein C, multicenter studies
will almost certainly be required to obtain a level of evidence sufficient to recommend their use with confidence. Until such time,
the care of patients with these difficult infections will remain more
dogmatic than definitive, as much supposition as science.

Clinical Question Summary


Questions

Answer

Grade

1 What is the standard for diagnosis in NSTIs?

Clinical; confirmation by open biopsy.

2 What is the best approach to initial


resection?

As complete as the patient will tolerate.

C
(Continued)

PMPH_CH106.indd 835

5/22/2012 6:12:11 PM

836

Surgery: Evidence-Based Practice

(Continued)
Questions

Answer

Grade

3 Should hyperbaric oxygen be part of the


standard treatment for NSTIs?

No. More rigorous trials are needed.

4 Should intravenous immunoglobulin be part


of the standard treatment for NSTIs?

No; although it may be helpful, it has not been proven to improve


outcomes.

5 Is drotrecogin alpha useful in NSTIs?

While one could justify the use of APC in NSTIs with sepsis and a high
risk of death on the basis of the original APC trial, the data regarding
its use specifically for this disease process is weak, especially
considering the potential for bleeding complications in patients
undergoing extensive resectional debridements.

6 Are there new adjunctive strategies to


consider?

New treatment strategies are being attempted and reported, but there
are no new therapies that have been proven to affect the outcome in
NSTIs by evidence-based criteria.

Abbreviations: NSTI, necrotizing soft tissue infection; APC, activated protein C.

REFERENCES
1. Eke N. Fourniers gangrene: a review of 1726 cases. Br J Surg.
2000;87:718-728.
2. Cainzos M, Gonzalez-Rodriguez FJ. Necrotizing soft tissue
infections. Curr Opin Crit Care. 2007;13:433-439.
3. Sawyer MD, Dunn DL. Deep soft tissue infections. Curr Opin
Infec Dis. 1991;4:649-654.
4. Dunn DL, Sawyer MD. Deep soft-tissue infections. Curr Opin Infec
Dis. 1990;3:691-696.
5. Meleny FL. Hemolytic streptococcus gangrene. Arch Surg. 1924;
9:317-364.
6. Wilson B. Necrotizing fasciitis. Am Surg. 1952;18:416.
7. Wong CH, Tan SH. Subacute necrotizing fasciitis. Lancet. 2004;
364:1376.
8. Wong CH, Wang YS. What is subacute necrotizing fasciitis? A
proposed clinical diagnostic criteria. J Infection. 2006;52:415-419.
9. Stamenkovic I, Lew PD. Early recognition of potentially fatal
necrotizing fasciitis; the use of frozen section biopsy. N Engl J
Med. 1984;310:1689-1693.
10. Majeski JA, Majeski E. Necrotizing fasciitis: improved survival
with early recognition by tissue biopsy and aggressive surgical
treatment. South Med J. 1997;90:1065-1068.
11. Wong CH, Wang YS. The diagnosis of necrotizing fasciitis. Curr
Opin Infec Dis. 2005;18:101-106.
12. Levenson RB, Singh AK, Novelline RA. Fournier gangrene role
of imaging. Radiographics. 2008;28:519-528.
13. Wong CH, Yam AKT, Tan ABH, et al. Approach to debridement
in necrotizing fasciitis. Am J Surg. 2008;196:e19-e24.
14. Kaide CG, Khandelwal S. Hyperbaric oxygen: applications in
infectious disease. Emerg Med Clin N Amer. 2008;26:571-595.
15. Kornonen K. Hyperbaric oxygen therapy in acute necrotizing
infections with special reference to the effects on tissue gas tensions.
A clinical and experimental study. Ann Chirurg Gynaecol Suppl.
2000;214:3-36.
16. Kornonen K, Him M, Niinkoski J. Hyperbaric oxygen in the
treatment of Fourniers gangrene. Eur J Surg. 1998;164:251-255.
17. Jallali N, Withey MS, Butler PE. Hyperbaric oxygen as adjuvant
therapy in the management of necrotizing fasciitis. Am J Surg.
2005;189:462-466.

PMPH_CH106.indd 836

18. Barry W, Hudgins L, Donta S, et al. Intravenous immunoglobulin therapy for toxic shock syndrome. JAMA. 1992;267;33153316.
19. Kaul R, McGeer A, Norrby-Teglund A, et al. Intravenous immunoglobulin therapy for streptococcal toxic shock syndrome
a comparative observational study. Clinic Infec Dis. 1999;28:
800-807.
20. Sugihara A, Watanabe H, Oohashi M, et al. The effect of hyperbaric oxygen therapy on the bout of treatment for soft tissue
infections. J Infec. 2004;48:330-333.
21. Kornonen K, Klossner J, Hirn M, et al. Management of clostridial gas gangrene and the role of hyperbaric oxygen. Ann Chirurg
Gynaec. 1999;88:139-142.
22. Wysoki MG, Santora TA, Sha RM, et al. Necrotizing fasciitis:
CT characteristics. Radiology. 1997;203:859-863.
23. Arslan A, Pierre-Jerome C, Borthine A. Necrotizing fasciitis:
unreliable MRI findings in the preoperative diagnosis. Eur J
Radiol. 2000;36:139-143.
24. Stevens DL. Streptococcal toxic shock syndrome associated with
necrotizing fasciitis. Ann Rev Med. 2000;5l:271-288.
25. Bernard GR, Vincent JL, Laterre PF, et al. Efficacy and safety of
recombinant human activated protein C for severe sepsis. N Engl
J Med. 2001;344:699-709.
26. Laterre, Pierre-Francois. Clinical trials in severe sepsis with
drotrecogin alfa (activated). Crit Care. 2007;11(Suppl 5):S5-S12.
27. Levi M. Activated protein C in sepsis: a critical review. Curr Op
Hematol. 2008;15:481-486.
28. Vincent JL, Bernard GR, Beale R, et al. Drotrecogin alfa (activated) treatment in severe sepsis from the global open-lable trial
ENHANCE: further evidence for survival and safety and implications for early treatment. Crit Care Med. 2005;33:2266-2277.
29. Abraham AE, Laterre PF, Garg R, et al. Drotrecogin alfa (activated) for adult patients with severe sepsis and a low risk of
death. N Engl J Med. 2005;353:1332-1341.
30. Levi M, Levy M, Williams MD, et al. Prophylactic heparin in
patients with severe sepsis treated with drotrecogin alfa (activated). Am J Respir Crit Care Med. 2007;176:483-490.
31. Rosing DK, Malepati S, Yaghoubian A, et al. The use of drotrecogin alpha for necrotizing soft tissue infections. Am Surg. 2010;
76:1104-1107.

5/22/2012 6:12:11 PM

Necrotizing Soft Tissue Infections

32. Firstenberg MS, Abel E, Blais D, et al. The use of extracorporeal


membrane oxygenation in severe necrotizing soft tissue infections complicated by septic shock. Am Surg. 2010;76:1287-1289.
33. Kyles DM, Baltimore J. Adjunctive use of plasmapheresis and
intravenous immunoglobulin therapy in sepsis: a case report.
Am J Crit Care. 2005;14:109-112.
34. Kornonen K, Kuttila K, Niinkoski J. Tissue gas tensions in
patients with necrotizing fasciitis and healthy controls during

PMPH_CH106.indd 837

837

treatment with hyperbaric oxygen: a clinical study. Eur J Surg.


2000;166:530-534.
35. May AK, Stafford RE, Bulger EM, et al. Treatment of complicated skin and soft tissue infections, Surgical Infection Society
Guidelines. Surg Inf. 2009;10:467-499.
36. George ME, Rueth NM, Skarda DE, Chipman JG, Quickel RR, Beilman GJ. Hyperbaric oxygen does not improve outcome in patients
with necrotizing soft tissue infections. Surg Inf. 2009;10:21-28.

5/22/2012 6:12:11 PM

Commentary on
Necrotizing Soft Tissue Infections
A. Patchen Dellinger

Regarding the recommendation that initial debridement be


as complete as the patient will tolerate, there are no useful data,
but the clinical experience of surgeons who have managed this
disease suggests that this is the correct approach.
Dr. Sawyer correctly notes that there is an absence of data
supporting the use of hyperbaric oxygen for NSTI. One potential
harm that can come from efforts to use hyperbaric oxygen occurs
when attempts to get a patient to a hyperbaric chamber delay the
mandatory aggressive debridement of the infection, which is the
most important initial step along with antibiotic administration
and aggressive support of vital signs. The initial debridement
should never be postponed to get a patient to a chamber. On the
other hand, if a chamber is close by or if the opportunity is readily
available for transfer after the first operation and prior to the second look procedure that Dr. Sawyer correctly recommends, then
there is no harm in transfer. Because of the widespread belief in
the possible value of hyperbaric oxygen, surgeons and physicians
in some hyperbaric centers may have a greater experience with the
disease than the average practitioner. Thus the benefit may stem
from experience with the disease rather than the chamber.
The data on immunoglobulin therapy and drotrecogin alpha
are correctly noted to be quite weak, but they are probably not
harmful except to the medical budget. New, adjunctive therapies
would be nice, but I am not aware of any on the horizon.

This chapter nicely summarizes the clinical situation in NSTI and


the poor quality of data available to draw conclusions about the
management of this uncommon but devastating disease. NSTI
occur uncommonly enough that few physicians or surgeons outside of specialty referral centers ever see enough cases to become
comfortable managing the disease, and frequently enough that
most surgeons will encounter at least one or two cases during a
professional career. Dr. Sawyer correctly notes that the grades of
evidence for all of his recommendations are of the C grade due to
the absence of any prospective trials in the diagnosis or treatment
of this condition. All the information that we have comes from
case series.
The chapter recommends open biopsy for diagnosis of the
disease and correctly notes that radiological studies (CT and
MRI) lack specificity although sensitivity is rather good. Mention
is made of frozen section examination of biopsy, and while this
is not harmful, in my experience it is rarely needed. The greatest
risk to a patient with NSTI is failure of the medical and surgical
team to recognize the necrotizing nature of the condition and to
treat expectantly for cellulitis without necrosis. What is needed is
a high index of suspicion for the diagnosis of NSTI. Once the suspicion is raised, a small incision for biopsy usually reveals to the
naked eye the nature of the process, and adequate debridement
can be undertaken without waiting for a frozen section.

838

PMPH_CH106.indd 838

5/22/2012 6:12:11 PM

PART XVI

PREOPERATIVE AND
POSTOPERATIVE CARE

PMPH_CH107.indd 839

5/22/2012 6:12:43 PM

PMPH_CH107.indd 840

5/22/2012 6:12:43 PM

CHAPTER 107

Electrolytes and Fluids


Daun J. Milligan, Megan B. Steigelman, Tristan T. Lai, and Aaron M. Fields

of the sinoatrial or atrioventricular nodes, causing a reversal of the


myocyte depression resulting from cases of severe hyperkalemia.1
Calcium administration is fast acting, achieving membrane stabilization in approximately 1 minute and lasting from 30 to 60
minutes. This treatment, however, does nothing to lower serum
potassium levels. It should also be strictly avoided in patients taking digitalis as it can lead to toxic digitalis levels.2
The next treatment option includes those that create an intracellular shift of potassium, lowering serum concentration. The
most traditional is insulin, usually accompanied with dextrose
to prevent hypoglycemia. It is the most reliable monotherapy for
hyperkalemia.2 Insulin stimulates the sodium-potassium pump,
causing an exchange for sodium (extracellularly) for potassium
(intracellularly) in a 2:3 ratio. This effect, which has no relation
to insulins ability to lower blood glucose, is usually achieved by
the infusion of 10 units of regular, short-acting insulin with 55 cc
of 50% dextrose. The insulin usually achieves a 0.6 to 1.0 mEq/L
decrease in serum potassium within 10 to 20 minutes. In their
recommendations on the treatment of hyperkalemia, Kaeml and
Wei advocate not treating hyperkalemia in nondiabetic patients
with glucose and without insulin, because such patients may not
be able to produce the high levels of insulin necessary to lower
serum potassium.3
Since catecholamines activate the same sodium-potassium
pump through the stimulation of -2 receptors, this has been
explored as a novel approach in lowering serum potassium levels
in cases of severe hyperkalemia. It has traditionally been advocated as an excellent adjunct to insulin therapy, not a monotherapy.4 The earliest studies occurred using intravenous -agonists
like albuterol. For example, Montoliu and colleagues in 1987 discovered that the administration of 0.5 mg of intravenous albuterol
led to a nearly 1 mEq/L decrease in serum potassium in human
patients.5 Unfortunately, intravenous -agonists are not available
in the United States. This has led to a number of studies of nebulized or meter-dosed inhaled applications of -agonists for the
treatment of hyperkalemia. For nebulized albuterol, the dose must
be increased to as much as 40-fold to achieve a decrease in potassium level of 0.61 to 0.98 mEq/L. The onset of action, however,

The topic of fluids and electrolytes is a topic whose breadth is vast.


Much of what we know is based on the personal experience of
a few people who lived hundreds of years ago. Randomized controlled trials have been designed to answer few of the many questions that clinicians deal with on a daily basis. Below are some
of the questions that have some evidence to support the answers
that follow.

HYPERKALEMIA
1. What is the best treatment for hyperkalemia?
Hyperkalemia, defined as serum potassium greater than 5.5 mEq/L,
is a serious condition that may be deemed a medical emergency.
Potassium affects the depolarization of electrically excitable tissues to include cardiac and skeleton muscle; thus, its abundance
may lead to dangerous arrhythmias and even sudden death. As a
result, its rapid and efficient correction is of utmost importance.
It is necessary to institute correction of hyperkalemia when the
serum potassium reaches 6.5 mEq/L or greater in the presence of
electrocardiogram (EKG) changes. Unfortunately, there is a paucity of data on the best treatment. It seems that the most important
factors in determining which treatment to use for hyperkalemia
depends on the underlying circumstances surrounding the electrolyte abnormality and must be tailored on a patient-to-patient
basis. There are three major ways to treat elevated serum levels
of potassium, which include antagonism, intracellular shift, and
elimination.
Three-quarters of cases of hyperkalemia result from renal failure (RF). The remaining majority of cases stem from side effects
of medicines. In most cases, however, it is imperative to choose
a treatment that is fast acting. Calcium gluconate, administered
intravenously in the amount of 10 cc of 10% solution infused over
2 to 3 minutes, acts as a potassium antagonist. Calcium stabilizes
cell membranes, raising the threshold potential to a less negative
number and making it more difficult for membranes to become
depolarized. This is particularly important in cells such as those
841

PMPH_CH107.indd 841

5/22/2012 6:12:44 PM

842

Surgery: Evidence-Based Practice

according to Allon and Copkney, was immediate and lasted for up


to 2 hours.6 In a single center study also by Allon et al. in 1989, 10
patients with chronic hyperkalemia on maintenance hemodialysis
for end-stage renal disease (ESRD) were treated with either saline
or 10- to 20-mg nebulized albuterol. Serial serum potassium measurements revealed patents receiving the albuterol experienced
a significant reduction in serum potassium, with a maximum
decrease of 0.62 +/ 0.09 mmol/L for the 10-mg albuterol group
and 0.98 +/ 0.14 mmol/L for the 20-mg albuterol group. No
patient experienced significant increases in blood pressure (BP)
or heart rate (HR), known side effects of -agonists.7
Mandelberg and colleagues sought to prove that salbutamol
delivered via metered-dose-inhaler (MDI) could reduce serum
potassium early after drug delivery. They gave 17 chronic hyperkalemic hemodialysis patients MDI salbutamol 1200 mcg followed by placebo, or placebo followed by salbutamol 1200 mcg.
Serum potassium levels were measured at 1, 3, 5, 10, and 60 minutes. Results showed that potassium levels reached a significant
difference 5 minutes after treatment (P < .05). Reduction in serum
potassium was fi rst noted in the salbutamol groups within
3 minutes of therapy; there was no significant reduction in serum
potassium in the placebo group. Of note, there was a paradoxical
and transient rise in serum potassium in the salbutamol groups
within the first minute of therapy completion before the level
dropped. In fact, 10 of 17 patients experienced a rise of greater
than 0.1 mEq/L, which was not experienced in the placebo group
(P < .0001). This suggests that salbutamol may not be suitable as a
first-line approach to hyperkalemia treatment.8
The third mechanism for hyperkalemia correction is elimination. Rectal elimination can theoretically be achieved through the
use of sodium polystyrene sulfonate resin. Administered either
orally or rectally, the resin exchanges gut potassium for sodium in
the resin. The resins require an extensive amount of time to work,
resulting in approximately 1 mEq/L reduction in serum potassium over a 24-hour period. Thus, it is impossible to use them as
first-line treatment in hyperkalemic emergency. In addition, the
resins can cause severe constipation and even the devastating side
effect of intestinal necrosis. Resin is ordinarily given with laxatives, which hastens elimination of the potassium-filled resin and
may prevent such complications. There is little data on the efficacy
of resin for hyperkalemia treatment. Gruy-Kapral et al. found no
significant decrease in mean serum potassium up to 4 hours after
resin treatment versus placebo versus laxatives alone. In fact the
group discovered a trend of increasing serum potassium in all
groups throughout resin treatment, with the resin arm experiencing the least amount of rise in potassium.9 Resin has, in fact, fallen
out of favor due to the delayed onset, constipation, small risk of
intestinal necrosis, and lack of data to support its efficacy.
One traditional means of eliminating extra potassium is by
route of hemodialysis. This method, which is invasive, expensive,
and time consuming, has been used for this purpose since the
1970s. It is only used as a first-line treatment when the patient is
already a dialysis recipient and the hyperkalemia is life threatening. It has been proven that certain types of dialysate are more
effective at reducing serum potassium than others. In a study of
12 nondiabetic hemodialysis patients, Gutzwiller et al. performed
standard hemodialysis using dialysate with 0, 1, or 2 mmol/L
potassium. Partial dialysate collection revealed the potassium-free
dialysate to significantly enhance potassium elimination without
impairing quality of dialysis.10 The other traditional means of
eliminating body potassium include the renal route via diuretics.

PMPH_CH107.indd 842

Theoretically, the administration of NaHCO3 should decrease


the concentration of protons in the extracellular fluid compartment, leading to an efflux of protons out of cells in exchange for
potassium. However, NaHCO3 has not been shown to promote significant reductions in serum potassium in hyperkalemia and has
fallen out of favor as a mainstay treatment. In 1992, Blumberg and
colleagues randomized 12 dialysis patients with hyperkalemia to
receive a bicarbonate infusion of 390 mmol over a 6-hour period.
No reductions were seen in serum potassium until the 4-hour
postinfusion mark, and the maximum serum potassium reduction was a meager 0.35 mEq/L at the 6-hour mark. The treatment
was slow to work and unable to sustain reductions in potassium.11
Likewise, Allon and Shanklin in 1996 discovered no significant
decrease in serum potassium at 30 or 60 minutes when hemodialysis patients were infused bicarbonate versus placebo.12
The idea of multimodal hyperkalemia therapy has been
explored by many investigators with mixed results. Of interest,
Ngugi et al. in 1997 took 70 hemodialysis patients and divided
them randomly into seven groups of 10 patients each. They dosed
each group with different combinations of placebo, insulin, bicarbonate, and salbutamol, and discovered that the absolute best
combination of therapy consisted of glucose 25 g, insulin 10 units,
NaHCO3 50 mmol 8.4%, and 0.5 mg salbutamol. This was given
as a recommendation for therapy for all patients suffering hyperkalemia or as emergency treatment while awaiting definitive
potassium removal via hemodialysis.13
Answer: In conclusion, it appears that the best treatment
for hyperkalemia is dependent on the cause, the type of patient,
and the urgency. A combination of several therapies may even
be best.

TRIS-HYDROXYMETHYL AMINOMETHANE
2. What role does tris-hydroxymethyl aminomethane play
during resuscitation?
Tris-hydroxymethyl aminomethane, or THAM, is an organic buffer widely used in biochemical research; it found its way into medical use for correcting metabolic acidosis. It has been used in the
treatment of respiratory failure in conjunction with hypothermia
and controlled hypercapnea, diabetic and renal acidosis, aspirin
or barbiturate overdose, and increased intracranial pressures after
traumatic brain injury.14 When infused intravenously, THAM
acts as a proton acceptor and actively scavenges protons from the
bloodstream to increase the arterial pH. Since the pKa of THAM at
normal body temperature is 7.8, it may be a very effective buffer at
physiological blood pH when the patients acid-base homeostasis
becomes overwhelmed. The role THAM plays in resuscitation is
controversial and there is a paucity of data on the subject. Sodium
bicarbonate is the gold standard for alkalinization therapy for
metabolic acidosis but has the side effects of hypernatremia and
increased serum carbon dioxide (CO2). In 2005, Hoste et al. conducted a small randomized controlled trial to compare the traditional sodium bicarbonate infusion with THAM infusion in 18
intensive care units (ICUs) with mild metabolic acidosis (serum
bicarbonate <20 mmol/L). The infusions were given over 1 hour
and titrated to buffer excess acid load in the serum. They found
that sodium bicarbonate and THAM were equivalent in alkalinizing effect from start of infusion to a maximum effect of 4 hours
for the sodium bicarbonate and 3 hours for THAM. Though the

5/22/2012 6:12:44 PM

Electrolytes and Fluids

effects of the traditional treatment were longer lasting, the THAM


arm had no rise in serum sodium or arterial concentration of CO2,
thus THAM may be more beneficial in cases of mixed respiratory and metabolic acidosis. However, potassium decreased in the
sodium bicarbonate group and remained the same in the THAM
group, thus THAM may not be ideal for patients in renal failure or
those with hyperkalemia.15
During trauma, prevention or correction of the lethal triad of
hypothermia, acidosis, and coagulopathy is a major determinant
of survival. In vitro studies have shown that acidosis increases
clotting time and impairs clotting strength.16 When adult trauma
victims become acidotic, coagulation is greatly impaired and large
volume blood loss persists. Theoretically, if the acidosis can be
corrected, so will the coagulopathy. Martini and colleagues tested
this hypothesis using a swine model. They induced acidemia to a
pH of 7.1 using 0.2-M hydrochloric acid in pigs and, after stabilization, infused either THAM (to correct to a pH of 7.4) or an equal
volume of lactated ringers solution (LR). Another cohort was
identified as the THAM control group, which consisted of pigs
from the same vendor without induced acidemia that received an
equal amount of LR or THAM. Blood samples were taken from
each group at baseline, after induced acidosis, and 15 minutes after
pH correction (in the THAM group) for biochemical assays as
well as thromboelastography (TEG). The results showed acidosis
caused statistically significant reductions in total plasma protein,
fibrinogen, and serum albumin. It was also noted that there was
a 50% loss of platelets with acidosis. There was also prolongation
of prothrombin time (PT), partial thromboplastin time (PTT),
and activated clotting time (ACT). On TEG, the K value (clot formation time) was prolonged whereas the R value (time for initial
clot) remained at baseline. The MA (max clotting strength) was
also decreased during acidosis. In the THAM infusion group,
decreases in total protein and albumin were noted, but there were
no significant changes in fibrinogen concentration or platelet
count, excluding hemodilution as a cause. Similarly, there were
no changes in TEG parameters or clotting times when compared
with baseline in patients treated with THAM. It was noted in the
THAM infusion group that correction of pH resulted in a significant correction of base excess from 12 to +3. However, most
importantly, coagulation parameters did not correct with pH neutralization. In addition, there were no improvements in fibrinogen
concentration or platelet count with the use of THAM.17
Though the role of THAM in trauma resuscitation may be
unclear, it still appears to have merit in certain populations where
acidosis is life threatening and refractory to traditional therapy.
For example, Marfo and colleagues published an interesting case
study in 2009 where THAM was successfully used to correct
severe lactic acidosis in a 50-year female on nucleoside analogue
reverse transcriptase therapy for human immunodeficiency virus
(HIV) infection. These HIV drugs disrupt the mitochondrial
oxidative phosphorylation function whereby protons generated
by hydrolysis of adenosine triphosphate are removed. The hyperlactinemic metabolic acidosis induced by these drugs is notorious for being refractory to sodium bicarbonate therapy and the
mortality is very high. Not only was THAM quick and efficient
in correcting the severe lactic acidosis in this particular patient,
it also did not increase her arterial carbon dioxide concentration,
avoiding a mixed respiratory and metabolic acidosis.18
Answer: THAM may be useful for correction of metabolic acidosis in patients without causing a rise in serum CO2 as long as the
patient is not in anuric renal failure or has hyperkalemia. Certain

PMPH_CH107.indd 843

843

populations, to include patients with respiratory compromise or


refractory lactic acidosis, may benefit in particular; however, the
role of THAM in trauma resuscitation remains to be seen.

POSTOPERATIVE NAUSEA AND


VOMITING
3. Does liberal crystalloid decrease the incidence of postoperative nausea and vomiting?
In the United States, postoperative nausea and vomiting (PONV)
is the number one reason for unplanned hospital admissions after
surgery. There are many known risk factors, including type
of surgery, gender, duration of surgery, type of anesthetic used,
and smoking history. Prevention is key as many risk factors cannot be modified. Anatomically, nausea is controlled by the central
nervous system at the area postrema in the chemoreceptor trigger zone. This is why drugs such as serotonin (5-HT3) antagonists
are frequently used to eliminate this symptom at the level of the
receptor. Research has also revealed that PONV may be a consequence of intestinal hypoperfusion during surgery. A patient who
is adequately resuscitated should also have adequate perfusion to
the gut, which may reduce the risk of PONV.19 Chaudhary and
colleagues set out to validate this theory in a study of 60 female
patients undergoing elective open cholecystectomy. The patients
were equally divided into three cohorts and given various intravenous fluids and in varying amounts preoperatively. Group A, the
control, received 2 mL/kg LR, whereas Group B received 12 mL/
kg Ringer lactate, and Group C received 12 mL/kg 4.5% hydroxyethyl starch (HES). A blinded observer assessed the patients for
24 hours postoperation using a visual analogue scale (VAS) for
nausea. Rescue antiemetics were given to patients with a VAS >5.
Their results revealed a significantly reduced incidence of PONV
using rapid fluid supplementation (12 mL/kg) with crystalloid or
colloid as compared with the control group. This effect was more
significant in the early periods (04 hours) than in the later time
period. The number of patients requiring rescue antiemetics was
greater in the control than the two groups receiving liberal fluids. Overall incidence of PONV in the two test groups, colloid or
crystalloid, was the same. This study also revealed that intravascular preloading with colloids or crystalloids is equally effective
in reducing PONV.20
In 2009, Lambert et al. conducted a randomized, doubleblinded control trial to test if a preoperative fluid bolus of crystalloid could prevent PONV in 46 laproscopic gynecological surgery
patients. The patients were well matched for demographics and
risk factors for PONV; there was no statistically significant difference in opioid dosing between groups, a known contributor
to PONV. The test arm received a crystalloid bolus in an amount
up to 1 L (based on the 4-2-1 rule) whereas the control group
received the staff anesthesiologists routine crystalloid infusion.
No antiemetics were given preoperatively or postoperatively in
either group. Nurses, blinded to which treatment the patient had
received, assessed PONV in the postanesthesia care unit (PACU).
The incidence of PONV was significantly decreased in the test arm
(P = .046). Although episodic hypotension has been associated
with PONV, ironically, the test group had a significantly higher
incidence of perioperative hypotension than the control group.21
Laparoscopic Roux-en-Y gastric bypass (RYGBP) for morbid
obesity has a high incidence of PONV. Schuster et al. performed

5/22/2012 6:12:44 PM

844

Surgery: Evidence-Based Practice

a retrospective study of 180 RYGBP patients to determine the


amount of perioperative fluids given and the incidence of PONV.
It was discovered that patients who did not experience PONV
received a larger volume of intravenous fluid at a faster rate than
similar patients who experienced and were treated for PONV in
the PACU.22 An additional study by Chohedri and colleagues analyzed the effect of perioperative fluids on complications in ambulatory surgery, with PONV being a major outcome. This was a
prospective, double-blinded study of 200 ambulatory surgery
patients randomized to either 20 cc/kg or 2 cc/kg of intravenous
sodium chloride over a 30-minute period prior to induction of
anesthesia. The incidence of postoperative vomiting and thirst
significantly decreased in the test arm compared with the control arm (P = .014 and P = .029, respectively); there was, however,
no difference in the incidence of nausea or dizziness between the
two groups.23
Answer: There is mixed data on the efficacy of fluid administration for the prevention of PONV. It appears that liberal fluid for
surgeries prone to PONV, such as abdominal and gynecological
procedures, may decrease the incidence of PONV. Crystalloid or
colloid seem equivocal at this reduction. Fluids given early in the
perioperative period may be of particular benefit.

ALBUMIN
4. Does albumin infusion improve outcomes in sepsis?
The debate over which fluid is better, crystalloid versus colloid,
has been an on-going topic of research and discussion. The largest
study published to date is the saline versus albumin fluid evaluation study (SAFE) published in the New England Journal of Medicine in 2004.24 According to the SAFE researchers, there was no
difference in mortality outcomes when septic patients were resuscitated with saline versus 4% albumin. However, in a specific subgroup of patients with severe sepsis, it was suggested that albumin
may be beneficial. The Surviving Sepsis campaign guidelines published in 2008 recommended the early use of crystalloid or colloid
for resuscitation in patients with sepsis based on the results of the
SAFE study.25
In 2011, Delaney et al. conducted a meta-analysis of multiple
small studies to assess whether there truly was a mortality difference when resuscitating with albumin versus other fluids.26 They
conducted a search of randomized controlled trials that compared
resuscitation of septic patients with albumin versus alternative
fluid resuscitation regimens. Their search turned up 17 trials with
1977 patients where eight studies included patients only with sepsis whereas nine included patients with sepsis who were a subgroup of the study population; the SAFE study was among the 17
included studies in the meta-analysis. The results of these studies
showed mortality benefit from resuscitation with fluids containing albumin. The pooled odds ratio (OR) for mortality was 0.82
(95% confidence limits, 0.671.0; P = .047). To further validate
this finding, the SAFE study was removed and the pooled analysis remained similar with an estimated OR of 0.84 (95% confidence limits, 0.691.02; P = .08). They cited possible mechanisms
of this benefit to include better intravascular volume expansion,
replenishment of serum albumin to normal levels, and the role
of albumin as a biological transporter and free-radical scavenger.
Though the overall results may lean toward a mortality benefit,
Delaney and colleagues sited a weakness in their study: their data

PMPH_CH107.indd 844

are only as reliable as the data obtained from all the individual
studies. Though the pooled analysis showed favorable results, the
P-value fell short of statistical significance when using a random
effects model for pooling. 26 Though promising, their conclusions should be accepted cautiously as further research needs to
be accomplished before definitively answering the question of
whether albumin has a significant mortality advantage in patients
with sepsis.
Answer: In conclusion, the only study to show statistically significant mortality benefit of albumin over crystalloid is a pooled
analysis of septic patients. Otherwise, albumin appears to be
equivocal with crystalloid for resuscitation in all populations.

HES 130/0.4 (VOLUVEN )


5. What is the safety of HES 130/0.4 (Voluven) versus older
HESs?
HES 130/0.4 is a third-generation HES created as an alternative to more traditional synthetic colloid preparations that have
been associated with the side effects of allergic or anaphylactoid
reaction, renal failure, tissue accumulation, and, most notably,
coagulopathy. HES is derived from waxy maize amylopectin, a
branched polysaccharide that resembles glycogen. In comparison with the earliest artificial colloid, gelatin, HES forms have
very little allergic or anaphylactoid potential, and 130/0.4 is no
exception.27 It is known that the molecular weight and degree of
substitution (ratio of hydroxyethyl groups to glucose) are directly
proportional to the side effects of HES.28 For example, Hetastarch
(450/0.7) and pentastarch (200/0.5), with their larger molecular
weights and higher degree of substitution, have been implicated in
tissue accumulation, renal failure, and coagulopathy. HES 130/0.4,
with a low molecular weight and smaller degree of substitution,
was developed for the specific purpose of improving pharamacokinetics, thus making it safer, while at the same time preserving the efficacy of volume effect of first- and second-generation
HES. HES 130/0.4 is a novel formulation, demonstrating a lower
molecular weight (13,000 Da), a lower degree of substitution (0.4),
and a greater renal elimination resulting in less accumulation in
the plasma and tissues.
It has been shown that HES 130/0.4 has the same resuscitation efficacy as older HES preparations. According to a study in
rats by Bepperling et al. in 1999, its infusion provides a volume
effect of 100% of an infused volume and remains intravascular for
4 to 6 hours, comparable to pentastarch.29 Ghandi et al. in 2007
found that in major orthopedic surgery with an indication for
large blood loss and need for colloidal intravascular resuscitation,
the total volume of colloid solution necessary to maintain hemodynamic stability did not differ between traditional hetastarch or
the new HES 130/0.4.30
Though HES 130/0.4 is purely excreted by the kidney, it has
been shown to be safe and not lead to accumulation even in patients with severe renal failure. Data from a small trial of 19 volunteers with varying degrees of renal function, ranging from mild
to severe, were given a single 500 cc infusion of 6% HES 130/0.4
over 30 minutes and HES blood levels were subsequently determined until 72 hours postinfusion. Neither peak concentration
nor terminal half-life of HES 130/0.4 was affected by any degree of
renal impairment, given the patient was not anuric. In addition,
there was no decline in renal function after dosing in any patient,

5/22/2012 6:12:44 PM

Electrolytes and Fluids

as determined by repetitive creatinine clearance measurements.


Ironically, creatinine clearance on average increased during the
study, but this finding was not significant (P = .054).31 As recent
as 2010, Wittlinger et al. explored the effect of HES 130/0.4 on
renal function during sepsis, which has been a controversial issue.
Using an in vivo model of human proximal tubular epithelial cells
deranged to sepsis via stimulation with tumor necrosis factor-
(TNF-), the group treated the cells with either HES 130/0.4 or
pentastarch. They measured monocyte chemoattractant protein-1
(MCP-1) in culture, a marker for inflammation by neutrophils
and macrophages. The found that both types of HES attenuated
the inflammatory response in the renal cells.32
Unlike its larger and more highly substituted predecessors,
HES 130/0.4 has not been shown to accumulate in either plasma
or tissue, despite multiple, large bolus dosing. Bepperling et al.
demonstrated in 1999 that such bolus dosing of highly concentrated 10% solution of HES 130/0.4 over a period as long as 10
days failed to result in plasma accumulation in healthy adult volunteers.33 HES 130/0.4 is the first starch that has been shown to
not accumulate rapidly in the plasma. The most likely reason why
it does not has to do with its low degree of substitution, which
lends it to rapid degradation by serum -amylase. In addition, in
comparison to pentastarch, the administration of HES 130/0.4
resulted in a three-quarters reduction in tissue accumulation.29
Recently, Leuschner et al. injected radio-labeled daily bolus injections of both 0.7 g/kg of 10% HES 130/0.4 and 10% of pentastarch
for 18 consecutive days into 48 rats. Radio-labeled HES was measured in body, carcass, liver, kidney, spleen, lymph nodes, plasma,
and urine. They found that 52 days after the last injection, there
was almost four-fold higher concentrations of pentastarch than
HES 130/0.4 (P .05).34 The Waitzinger group demonstrated a
similar lack of accumulation in the plasma of 12 healthy male volunteers after 10 repeated daily boluses of 500 cc HES 130/0.4 that
same year.35
There would be no benefit to using HES 130/0.4 for the purpose of blood replacement in a resuscitation scenario if its use
results in the need for transfusion of more blood products. Hence,
coagulopathy, the most worrisome, lethal, and common side
effect of HES has been challenged repeatedly in HES 130/0.4.
The cheapest and earliest form of artificial colloid, gelatin, enjoys
the profi le of negligible coagulopathy in repeated trials. Poorly tolerated by many patients due to allergic or anaphylactoid reactions,
HES in general was prepared as a modern substitute. One of the
first studies to explore the effect of HES 130/0.4 on coagulation
challenged the novel HES against gelatin, the least coagulopathic
colloidal intravascular replacement. In 2000, Haisch et al. conducted a prospective double-blinded controlled trial and randomized patients undergoing major abdominal surgery to one of two
agents infused until postoperative day 1 to a target central venous
pressure (CVP) of 10 to 14 mm Hg. There were 21 patients who
received HES 130/0.4 and 21 who received gelatin as standard of
care. The serial thromboelastogram (TEG) results indicated that
patients receiving HES 130/0.4 incurred the need for the same
amount of allogeneic blood/blood products and led to the same
coagulation variables as those receiving gelatin.36
The coagulopathic potential of HES 130/0.4 has recently
been well studied against second-generation HES. More recent
evidence has come from research in fields of surgery that typically result in large blood loss and the need for blood transfusion.
In 2003, Sander et al. infused either HES 130/0.4 or the standard
pentastarch into a total of 60 gynecological surgery candidates

PMPH_CH107.indd 845

845

in a randomized, double-blinded clinical study. They found that


prothrombin time (PT) and international normalized ratio (INR)
were significantly higher in the patients who received HES 130/0.4
(P < .05).37 Orthopedics, another field of surgery that often requires
perioperative colloidal volume replacement, has contributed data
as well. As recent as 2007, Ghandi et al. conducted a prospective,
randomized, double-blind, multicenter trial of patients undergoing major orthopedic surgery. There were 49 patients in the HES
130/0.4 arm and 51 patients in the hetastarch arm. The study
showed a significantly lower nadir in factor VIII, 2 hours postoperatively in the hetastarch group versus HES 130/0.4, and the
same was true for von Willebrand factor 2 hours postoperatively
if the patients received greater than 1 L colloid.30
Perhaps the most compelling data come from an analysis
of pooled data from all available major surgeries by Kozek-Langenecker and colleagues in 2008. Recognizing that HES has the
potential to affect clot formation and polymerization, humoral
coagulation factors, and platelet aggregation, the group examined
estimated blood loss, drainage loss, calculated blood loss, transfused blood product volumes, and coagulation variables for 24
hours after the start of surgery. A total of 449 patients from seven
major clinical trials had received either 6% HES 130/0.4 or 6%
pentastarch. They discovered that there was a significant decrease
in estimated, drainage, and calculated blood loss as well as less
need for red blood cell transfusion in the HES 130/0.4 group. In
the earliest postoperative phase, HES 130/0.4 exerted less of a negative effect on PT and von Willebrand levels. In addition, there
was a slight but significant increase in the platelet numbers in the
HES 130/0.4 group.38
Answer: HES 130/0.4 has been shown to be safe and effective
for resuscitation. It has little risk of allergic or anaphylactic reaction, renal dysfunction, tissue accumulation, or coagulopathy.

MAGNESIUM ABNORMALITIES
6. What is the effect of abnormal serum magnesium levels on
respiratory muscle function?
Magnesium is the fourth most abundant mineral in the body and
only second to potassium for intracellular cations. It is a bivalent
ion that acts as a cofactor in several enzymatic systems as well as
regulating many other ion channels. It is a guardian of transmembrane and intracellular ionic flows. A dysregulation of magnesium
may lead to critical dysregulation of many other ions important
for cell physiology. A normal serum magnesium level is 1.7 to
2.3 mg/dL. When considering the role of abnormal serum magnesium concentrations in the pathogenesis of respiratory muscle
weakness, it is interesting that either a high or a low abnormality
is equally capable of causing the disorder. In order to keep respiratory muscles functioning properly the serum magnesium concentration must not be too high or too low, but just right.
Hypermagnesemia has levels that correlate with progressive symptoms. For example, a serum range of 4.8 to 7.2 mg/dL
correlates with nausea, lethargy, drowsiness, and reduced deep
tendons reflex (DTR). A range between 7.2 and 12.0 manifests as
somnolence, absent DTR, EKG changes, and hypotension. Finally,
a patient may demonstrate muscle paralysis, respiratory arrest,
and complete heart block followed by cardiovascular collapse at
serum magnesium of 12.0 mg/dL or above. The most common
side effect of magnesium toxicity is neuromuscular blockade, as

5/22/2012 6:12:44 PM

846

Surgery: Evidence-Based Practice

a lack of magnesium leads to the decreased transmission of nerve


impulses across the neuromuscular junction. The effect of hypermagnesemia resembles the paralytic effect of curare on the neuromuscular junction.39 On the cellular level, hypermagnesemia
inhibits the release of calcium from the sarcoplasmic reticulum
(SR) via the blockade of calcium channels. On a synaptic level,
high serum magnesium concentrations reduce the presynaptic
release of acetylcholine, which in turn decreases neuromuscular
signal transmission. In addition, the condition diminishes the
effect of acetylcholine on postsynaptic receptors, increasing the
threshold necessary to achieve an action potential.40
Although renal dysfunction is the most common cause of
high serum magnesium because magnesium is renally excreted,
hypermagnesemia may develop in nonrenal failure patients who
have a history of significant gastrointestinal (GI) disease or abuse
of over-the-counter medicines containing high levels of magnesium. A review of the literature yields an abundance of case
reports of respiratory dysfunction and need for intubation and
mechanical ventilation for the condition of hypermagnesemia.
In chronological order, the most relevant ones include a 62-year
female found to have a perforated duodenal ulcer who had consumed large amounts of magnesium citrate. The case report,
published by Mordes et al. in 1975, demonstrated that a peak
serum magnesium level of 15.3 mg/dL led to respiratory arrest,
coma, and need for intubation and mechanical ventilation.41 In
1981, Millette and Snodgrass reported a 51-year-old male with
no history of renal failure who developed acute onset renal stones
and subsequent renal failure from consuming large amounts of
magnesium hydroxide. At a peak serum magnesium level of 8.4
mg/dL, the patient developed respiratory depression and coma
requiring ventilator support.42 In 1989, Castelbaum et al. published a case report of a 56-year-old female who took magnesium
laxative and magnesium-containing antacids for GI upset and
experienced respiratory arrest with a serum magnesium level of
8.4 mg/dL.43
An impressive case of undiagnosed hypermagnesemia is
described by Fung et al. in 1995. A 60-year-old female with a past
medical history of coronary artery disease, diabetes, and GI and
orthopedic surgeries presented to the emergency room with a
24-hour history of progressive dysarthria and left upper extremity
weakness, which was assumed to be a cerebrovascular accident.
She had head imaging that was negative for acute intracranial processes. Her muscular weakness progressed rapidly to severe respiratory acidosis, respiratory arrest, and coma followed by intubation
and mechanical ventilation. She exhibited profound paralysis with
no DTR and fi xed, dilated pupils. Her family discussed withdrawing care as the best neurological diagnosis was brainstem stroke,
which was unable to be properly viewed on imaging. At this point
in her care, the serum magnesium level returned to a value of 16.2
mg/dL. Prompt treatment of her hypermagnesemia led to quick
resolution of her neuromuscular condition with zero neurological
sequelae. This case report speaks of the profound effect hypermagnesemia can have on respiratory muscle function.44
Hypomagnesemia can be as devastating to respiratory muscle integrity as hypermagnesemia. There are two reasons for this.
Initially, since magnesium functions on l-type calcium channels
in cell membrane and in the SR, its scarcity leads to hypercalcemia. This, in effect, leads to a rapid release of calcium from the
SR, inducing neuromuscular hyperexcitability. This increases the
threshold necessary to trigger an action potential. A low intracellular magnesium concentration also permits the efflux of potassium,

PMPH_CH107.indd 846

and this efflux of potassium from the cell creates hyperkalemia.


This state opens voltage-gated sodium channels, which eventually
shut down, creating a state refractory to nerve impulse transmission. The combination of effects leads to muscle weakness and
paralysis, especially in the respiratory muscles.40
As early as 1983, Rochester et al. described the role of hypomagnesemia in the development of respiratory muscle weakness
and failure to wean from mechanical ventilation. One of the earliest studies that cited hypomagnesemia as a major contributing
factor for skeletal muscle weakness and sought to correlate this
weakness with respiratory muscles was performed by Dhingra
et al. in 1984. This group measured both muscle power and lung
volumes in 16 control and 17 test subjects. The patients were randomized to either magnesium or dextrose (placebo) treatments
on day 1 and then crossed over to receive magnesium if they had
been given placebo or vice versa on day 2. The study was doubleblinded. They discovered that there was no significant difference in lung volumes between groups but there was a significant
improvement in muscle power in the patients after receiving magnesium.45 Rochester recognized hypomagnesemia as a significant
cause of respiratory muscle weakness.46 Several reviewers, including Rochester, have concluded that weakness of respiratory muscles,
particularly those of inspiration, predisposes patients to respiratory
arrest and failure to wean from mechanical ventilation.46-48 In 1988,
Fiaccadori et al. conducted needle-biopsy of the quadricep muscles
of 32 patients hospitalized with chronic obstructive pulmonary disease (COPD) and subsequent acute respiratory distress syndrome
(ARDS). They used atomic absorption spectrophotometry to compare the amount of magnesium in muscle tissue with the amount in
the serum. They found that 9.4% of patients had hypomagnesemia
in the presence of normal muscle magnesium levels, and 48.7% of
patients had low muscle magnesium values in the presence of normal serum magnesium concentration. It was the decreased values
of muscle magnesium, not serum, which were in turn associated
with ventilator dependence and prolonged stay in the ICU.49
An interesting study whose purpose was to determine the
effect of supplementing magnesium in patients who were on
mechanical ventilation for at least 6 days due to respiratory failure
was performed in 1993 by Johnson et al. Twelve patients received
6 mg MgSO4 via intravenous route over 16 hours on day 1 then
placebo infusion on day 2. Nine patients received the opposite, placebo first then magnesium second. The results demonstrated that
magnesium infusion in patients with normal serum magnesium
(1.72.3 mg/dL) on mechanical ventilation for respiratory failure
was not associated with increased respiratory muscle strength, as
measured by vital capacity (VC), maximal inspiratory pressure
(PImax), and maximal expiratory pressure (PEmax).50
Answer: The Johnson study, as well as studies of hyper- or
hypomagnesemia in respiratory failure, support the concept that
serum magnesium within normal levels (1.72.3 mg/dL) is optimal for respiratory muscle function. Values that are too high or
too low are equally devastating in terms of respiratory muscle
failure.
7. Does volume of fluid resuscitation affect anastomotic leak in
abdominal surgery?
The question of how much volume to give a patient both intraoperatively and postoperatively has been addressed for some
time. Initial studies indicated that low gut perfusion was associated with poor outcomes.51 However, randomized controlled

5/22/2012 6:12:44 PM

Electrolytes and Fluids

trials in elective colonic resections soon showed that positive


fluid balance associated with a weight gain of 3 kg or more was
no better and resulted in delayed return of gut function and longer hospital stays.52 Brandstrup et al. performed one of the better studies to answer this question. In a randomized, controlled
fashion, she showed that a restrictive fluid strategy aimed at
maintaining body weight in the perioperative period significantly
improved outcomes and decreased mortality.53 Supporting evidence soon emerged from the anesthesiology literature as well.
A randomized controlled trial was performed in which LR was
administered at a rate of 4 mL/kg/h in the restrictive group, and
this resulted in fewer postoperative complications.54 The most
recent study attempting to answer this question was performed
in 2010.55 Futier and his colleagues enrolled 70 patients and randomized them to 6 mL/kg/h of crystalloid (restrictive group) or
12 mL/kg/h of crystalloid (what they termed the conservative
group). Patients were monitored intraoperatively with a continuous aortic velocity Doppler probe. Those that were deemed hypovolemic by this monitor were given colloid fluid boluses. Those in
the conservative group had fewer anastomotic leaks than those in
the restrictive group. So, despite 15 years of research into the subject, and multiple, randomized, controlled trials, the exact fluid
strategy is still unknown. Too much fluid and too little fluid have
consistently shown to be detrimental, but the perfect mechanism
for determining how much is just right has yet to be elucidated.
Perhaps the answer lies not in restrictive or conservative management, but in using newer technologies to determine the optimal
fluid balance in a goal-directed fashion. The price of esophageal
Doppler probes continues to fall, and several studies have shown
that using them for fluid management improves outcomes.55,56
Answer: The amount of fluid given to a patient does change
the anastomotic leak rate, but the correct fluid dose is conflicting.
Goal-directed fluid resuscitation may be the answer.
8. Do hypertonic saline solutions have a role in managing traumatic brain injury?
Mannitol has been one of the mainstay treatment for traumatic
brain injury (TBI) for some time. However, it was never subjected
to the rigorous testing that newer drugs must undergo. In the
market, there are many hypertonic saline solutions (HTSs) now,
and there is emerging evidence regarding their use. HTS has been
used to treat hypotension in TBI57 and to treat increased intracranial pressures (ICPs);58-60 it has also been used to improve neurologic outcomes in TBI.61
In a randomized controlled prehospital study, Cooper et al.57
investigated whether TBI patients with hypotension had better
neurological outcomes after a single 250 mL bolus of 7.5% saline
(HTS) or LR who were then further resuscitated according to normal protocols. This single intervention showed no difference in
neurological outcomes at 6 months.
The best evidence in support of HTS is in the treatment of
increased ICP, where it has been shown to be better than mannitol
at lowering pressures. Two very similar studies, both published in
Critical Care Medicine showed that 7.5% saline improved elevated
ICPs better than 20% mannitol. Battison and colleagues used 100
mL of 7.5% saline, whereas Vialet et al. gave 2 mL/kg per dose.
Although they thus differed in the volume of saline given, they
both showed it was more effective than equal volumes of mannitol and the improved ICP measurements lasted longer in the HTS
groups than they did in the mannitol groups.58,59 In a retrospective

PMPH_CH107.indd 847

847

analysis, Kerwin et al. showed that 23.4% saline (HTS) was more
efficacious than mannitol in lowering increased ICPs. 60
In one of the largest randomized controlled trials to date
using HTS, Bulger et al. recruited over 1300 patients before the
study was terminated for utility.61 They found that in patients with
a Glascow Coma Scale of <8 who were not hypovolemic, a single
dose of 7.5% saline with 6% dextran (HTS) did no better than
those patients receiving 0.9% normal saline at 6 months.
Answer: HTS is better than mannitol at lowering elevated
ICPs, but it has not been shown to be beneficial in other settings.
9. Identify the electrolyte abnormalities seen in gastric outlet
obstruction and the best way to correct them.
Gastric outlet obstruction (GOO) is a pathophysiologic consequence of a mechanical blockage of the distal stomach, regardless of the cause, which may be benign or malignant, extrinsic
or intrinsic. Vomiting without the pancreatic, biliary, and intestinal secretions results only in the loss of gastric fluid, which is
high in chloride and hydrogen ions, resulting in a hypochloremic
alkalosis.62 Urinary bicarbonate initially elevates to compensate
for the alkalosis. Hydrogen and sodium reabsorption requires
potassium exchange and excretion, resulting in hypokalemia. The
hypokalemia is then countered with the exchange of hydrogen in
the urine, resulting in a paradoxical aciduria in the face of alkalosis. Regardless of the etiology, the initial medical management of
the hypokalemic, hypochloremic metabolic alkalosis is the same:
fluid resuscitation. Depending on the duration of the symptoms
and the age of the patient, varying degrees of dehydration will be
present.62
Initial gastric decompression, resuscitation with sodium chloride solution, and correction of the obstruction is the mainstay of
treatment. The alkalosis will respond readily to chloride replacement. After placement of a nasogastric tube and diagnostic imaging, the resuscitation begins immediately with a salt solution.
After correction of the alkalosis and hypochloremia, potassium is
then administered.62 If the condition was prolonged prior to presentation, or significant nutritional deficits exist, these should be
addressed prior to treating the underlying surgical causes.
Answer: Decompression, hydration, and correction of electrolyte abnormalities should be accomplished.
10. What are the most common causes of severe hypercalcemia, and what are the symptoms and the treatment?
Severe hypercalcemia is seen when calcium levels reach 14 mg/dL
or greater and is seen most often in hospitalized patients with a
malignancy. Elevations in calcium to a level of 14 or greater should
be treated regardless of symptoms.63 Calcium can be elevated by
several mechanisms. Malignancies such as multiple myeloma,
breast cancer, renal cell and lung cancer, in which bone resorption is increased or the renal excretion of calcium is decreased,
lead to significantly elevated calcium levels. Also, PTH-like hormone can be secreted and binds to parathyroid receptors, which
stimulates calcium absorption and decreases renal excretion of
calcium. Hodgkins and non-Hodgkins lymphomas increase calcitriol formation, which increases the absorption from the bone
and GI tract.
GI symptoms of hypercalcemia include anorexia, nausea,
vomiting, and abdominal pain. Neuromuscular symptoms include
weakness, confusion, coma, and bone pain. Cardiovascular
and renal effects include hypertension, arrhythmia, polyuria, and

5/22/2012 6:12:44 PM

848

Surgery: Evidence-Based Practice

polydypsia. EKG changes seen include a shortened QT interval,


prolonged PR and QRS intervals, increased QRS voltage, T-wave
widening, and AV block that can progress to complete block and
cardiac arrest. The severity of the symptoms are directly related to
calcium levels.62
The goals of treatment of hypercalcemia include stabilization
and reduction of the calcium levels via dilution with hydration,
increased urinary calcium excretion, and decreased release of calcium from the bones.62 Initial treatment centers on discontinuing
medications that are contributing to increased uptake, and treating the underlying malignancy.
Hydration with normal saline is the initial treatment, as these
patients are profoundly dehydrated. Calcium levels can be reduced
by dilutional effects. Volume expansion at an initial rate of 200 to
300 mL/h that is then titrated to maintain urine output at 100
to 150 mL/h. The extracellular volume increase also increases
renal calcium clearance in functioning kidneys. Hydration must
be carefully accomplished to avoid volume overload in cardiac
patients and is relatively ineffective in kidney failure patients.
Patients in renal failure will require dialysis to normalize calcium
levels. In patients with functioning kidneys, loop diuretics can be
used with hydration to increase renal calcium excretion and avoid
severe volume overloading. Thiazide diuretics increase the uptake
of calcium and should be avoided. Bisphosphonates will also assist
in inhibiting osteoclast activity for a month at a time to decrease
release of calcium from stores. Calcitonin is also used to promote
osteoblast function and enhance calcium retention in the bones.
Long-term use of zoledronic acid or pamidronate is useful in
patients with hypercalcemia related to malignancy to treat and
prevent skeletal complications in metastatic bone disease.
Answer: Severe hypocalcemia is usually caused by malignancy in a hospitalized patient. Rapid correction includes dilution
with normal saline, prevention of further release of calcium from
the bones, and increase in calcium excretion.
11. What electrolyte abnormalities are associated with seizures?
In patients with seizure activity, it is important to obtain history
and lab values to rule out electrolyte abnormalities, as many of
them can cause a lowering of the seizure threshold.
Answer: Patients may seize secondary to hyponatremia,64
hypernatremia,65 hypoglycemia,66 nonketotic hyperglycemia,66 or
hypomagnesemia.
12. How can you differentiate prerenal azotemia from intrinsic
renal failure?
Prerenal insufficiency is a functional response of structurally normal kidneys to hypoperfusion. Globally, prerenal insufficiency
accounts for approximately 70% of community-acquired cases of
acute renal failure and as many as 60% of hospital-acquired cases.
A decrease in circulatory volume evokes a systemic response
aimed at normalizing intravascular volume at the expense of
the glomerular fi ltration rate (GFR). The reabsorptive capacity
of tubular cells are preserved. Intrinsic renal failure is associated
with structural renal damage, and the tubular cells are impaired
due to this damage. This includes acute tubular necrosis (from
prolonged ischemia, drugs, or toxins), primary glomerular diseases, or vascular lesions.
When evaluating for acute kidney injury, serum creatinine is
the most commonly used marker. However, creatinine takes several hours to rise after acute kidney injury, and new biomarkers

PMPH_CH107.indd 848

such as plasma cystatin C, urinary NGAL, and urine netrin-1 are


being tested to predict the occurrence and extent of renal damage
within hours of an acute event.67-69
In order to distinguish prerenal azotemia from intrinsic renal
failure, the most common method used is urinalysis. In prerenal
failure, there is little cellularity, protein, heme, fine granular, or
hyaline casts observed. Heme positive urine without red cells suggests hemolysis or rhabdomyolysis. Urine specific gravity is high
(>1020). In intrinsic renal failure, the urine is highly cellular and
proteinaceous. The urine specific gravity is low.
Urinary indexes can be used to determine which entity is
causing low urine output. Measures of urinary and serum sodium,
creatinine, and osmolality are used commonly. In prerenal failure, the ratio of urinary to plasma creatinine is high (>40), the
ratio of urinary to plasma osmolality is high (>1.5), and the urinary sodium concentration is low (<20 mEq/L). The opposite is
seen in intrinsic renal failure, where the ratio of urine to plasma
creatinine is low (<20), the ratio of urine to plasma osmolality
is less than 1.1, and urine sodium concentration is greater than
40 mEq/L.
An elevated serum creatinine with a normal urinalysis gives
a very short differential in renal failure and includes prerenal
azotemia, hypertensive nephrosclerosis, obstructive nephropathy, interstitial nephritis, renal vascular diseases, hypokalemic
nephropathy, and hypercalcemia as well as other electrolyte
abnormalities.70
The fractional excretion of sodium (FENa) is also helpful to
determine prerenal from renal oliguria. It is the percentage of
fi ltered sodium that is excreted and is calculated with the formula %FENa = [(U/P)Na]/[(U/P)Cr] 100. In prerenal azotemia,
the % FENa is less than 1%, and greater than 1% in intrinsic renal
failure.
The FENa, along with the osmolality and urinalysis findings should be used in combination to identify the etiology of the
oliguria, and the proper treatment should be instituted with this
information.
Answer: Urinalysis, FENa, and urine osmolality are used to
differentiate prerenal azotemia from intrinsic renal failure.
13. What is the effect of serum albumin on calcium measurements?
Total serum calcium is directly related to the serum albumin concentration. The total serum calcium will increase by 0.8 mg/dL for
every 1 g/dL rise in serum albumin, and will fall by 0.8 mg/dL
for every 1 g/dL fall in serum albumin.
The ionized calcium is inversely related to serum albumin.
The lower the serum albumin, the higher the portion of the total
calcium that is present in ionized form. In the presence of hypoalbuminemia, although total calcium level may be low, the ionized
calcium level may be normal.62
Calculated serum calcium should not be used to determine
calcium levels in the critically ill or chronic dialysis patient, but
the ionized levels should be measured. The use of the correction formula in these patients results in inaccurate assessment
of the calcium state when compared with the ionized calcium. If
in doubt, the calcium should be evaluated using direct measurements of ionized calcium to determine hypocalcemia in the intensive care and dialysis settings.71-73
Answer: Serum calcium will increase by 0.8 mg/dL for every
1 g/dL rise in serum albumin, and will fall by 0.8 mg/dL for

5/22/2012 6:12:44 PM

Electrolytes and Fluids

every 1 g/dL fall in serum albumin. However, ionized calcium


levels are unaffected by albumin.
14. What is the current management of transfusion-related
acute lung injury?
Transfusion-related acute lung injury (TRALI) is now considered the most frequent cause of acute transfusion fatalities. If
recognized early, intensive pulmonary supportive measures can
improve the overall outcome. There are several mechanisms proposed that induce TRALI, all of which center around neutrophil
and antibody interactions in the pulmonary vasculature, leading
to pulmonary edema without systemic fluid overload. Regardless
of the etiology, early recognition is essential to begin institution
of supportive measures.
Like the treatment of pulmonary edema secondary to other
etiologies, the acute management of TRALI consists of supportive
measures. This may range from supplemental oxygen to ventilatory support with small tidal volumes and lung protective strategies of ventilation. Diuretic use has not been found beneficial in
the treatment of TRALI without concomitant fluid overload or
cardiogenic pulmonary edema.74 Blood transfusions can continue
with a different donor if necessary, and the blood bank should
be made aware that there is a concern for TRALI in the patient.
No evidence suggests that corticosteroids or antihistamines are
beneficial. A transfusion reaction workup should be initiated and
the HLA typing should be compared in the donor and recipient to
identify corresponding antibodies.74 Most patients respond to supportive management within 24 to 48 hours. Mortality has been
reported to be 11% to 45%.75
Answer: The management of TRALI includes recognition,
stopping the causative agent, and supporting ventilation and oxygenation as necessary.
15. Describe the symptoms of hyponatremia and the treatment of the syndrome of inappropriate antidiuretic hormone
(SIADH).
Hyponatremia is the most common electrolyte abnormality, and
SIADH is the most frequent underlying pathophysiology.65 Severe
hyponatremia is defined as a plasma sodium <120 mmol/L, and

849

mortality from hyponatremia reaches 50% as the levels go below


115 mmol/L.65
Symptoms of hyponatremia are dependent on the rate at
which the sodium has dropped and are thought to originate from
cerebral edema. Mild symptoms include headache, nausea, and
lethargy. The severity progresses to seizures, coma, and death.
Untreated mild and severe hyponatremia has been found to
increase overall inpatient mortality and should be addres sed
regardless of symptoms or severity.65
SIADH is a functional disorder of water regulation and leads
to hypervolemic hyponatremia. It most commonly develops from
injury to the hypothalamic-neurohyphophyseal system such as
brain tumors, abscess, head trauma, subarachnoid hemorrhage,
meningitis, encephalitis, and Guillian-Barre syndrome. Low concentrated urine output is seen along with decreased hematocrit and
hemoglobin due to dilutional status. Serum sodium and osmolality
are low as opposed to a high urine sodium and osmolality.
Once diagnosed, the initial treatment of SIADH is fluid
restriction between 800 and 1200 mL/day, which can be difficult
for patients to comply with given the excessive thirst that results
from the condition. Oral salt tablets can be given. Medications
such as demeclocycline and lithium can induce nephrogenic diabetes insipidus in 30% to 60% of patients to reverse hyponatremia.76
These effects are often unpredictable and have considerable side
effects. Lasix is also used in the short term as a diuretic, but longterm treatment induces natriuresis, and can actually worsen
hyponatremia. The correction of severe, symptomatic hyponatremia utilizes hypertonic saline using the Adrogue-Madias formula that corrects hyponatremia at <0.5 mmol/L per hour, slow
enough to avoid central pontine myelinolysis. The newest treatment option for mild to moderate hyponatremia is vasopressin-2
receptor antagonists, the vaptans.77 This class of drugs targets the
underlying pathophysiological cause of the disorder by preventing
the reabsorption of water from the renal tubules without affecting
solute excretion.77 Studies are underway to determine the safety of
this drug in subarachnoid hemorrhage patients.65
Answer: Mild hyponatremia can present with headache, nausea,
and lethargy, whereas severe hyponatremia presents with seizures,
coma, and death. Treatment for SIADH includes fluid restriction,
salt replacement, and vasopressin-2 receptor antagonists.

Clinical Question Summary


Question

Recommendation

1 What is the best treatment for hyperkalemia?

The single best treatment is dialysis; however, insulin/glucose


infusion, bicarbonate, and agonists can be used.

2 What role does tris-hydroxymethyl aminomethane (THAM)


play during resuscitation?

THAM may be useful for correction of metabolic acidosis in


patients without causing a rise in serum CO2 as long as the
patients are not in anuric renal failure or have hyperkalemia.
Certain populations, including patients with respiratory
compromise or refractory lactic acidosis, may benefit in
particular; however, the role of THAM in trauma resuscitation
remains to be elucidated.

3 Does liberal crystalloid decrease the incidence of postoperative


nausea and vomiting (PONV)?

Liberal fluid for surgeries prone to PONV, such as abdominal


and gynecological procedures, may decrease the incidence of
PONV.

4 Does albumin infusion improve outcomes in sepsis?

A single pooled analysis study showed benefit. However, other


studies have failed to do so.
(Continued)

PMPH_CH107.indd 849

5/22/2012 6:12:44 PM

850

Surgery: Evidence-Based Practice

(Continued)
5 What is the safety of HES 130/0.4 (Voluven) versus older
HESs?

HES 130/0.4 has been shown to be safe and effective for


resuscitation. It has little risk of allergic or anaphylactic
reaction, renal dysfunction, tissue accumulation, or
coagulopathy.

6 What is the effect of abnormal serum magnesium levels on


respiratory muscle function?

Serum magnesium within normal levels is optimal for respiratory


muscle function. Values that are too high or too low are
equally devastating in terms of respiratory muscle failure.

7 Does volume of fluid resuscitation affect anastomotic leak in


abdominal surgery?

The amount of fluid given to a patient does change the


anastomotic leak rate, but the correct fluid dose is unknown.
Goal-directed fluid resuscitation may be the answer.

8 Do hypertonic saline solutions (HTSs) have a role in managing


traumatic brain injury?

HTS is better than mannitol at lowering elevated intracranial


pressures, but it has not been shown to be beneficial in other
settings.

9 Identify the electrolyte abnormalities seen in gastric outlet


obstruction and the best way to correct them.

Hypochloremic, hypokalemic alkalosis with dehydration.

10 What are the most common causes of severe hypercalcemia,


and what are the symptoms and the treatment?

Severe hypocalcemia is usually caused by malignancy in a


hospitalized patient. Rapid correction includes dilution with
normal saline, prevention of further release of calcium from
the bones, and an increase in calcium excretion.

11 What electrolyte abnormalities are associated with seizures?

Hyponatremia, hypernatremia, hypoglycemia, nonketotic


hyperglycemia, or hypomagnesemia.

12 How can you differentiate prerenal azotemia from intrinsic


renal failure?

Urinalysis, fractional excretion of sodium (FENa), and urine


osmolality are used to differentiate prerenal azotemia from
intrinsic renal failure

13 What is the effect of serum albumin on calcium


measurements?

Serum calcium will increase by 0.8 mg/dL for every 1 g/dL rise in
serum albumin, and will fall by 0.8 mg/dL for every 1 g/dL fall in
serum albumin. However, ionized calcium levels are unaffected
by albumin.

14 What is the current management of transfusion-related acute


lung injury (TRALI)?

The management of TRALI includes recognition, stopping the


causative agent, and supporting ventilation and oxygenation as
necessary.

15 Describe the symptoms of hyponatremia and the treatment of


the syndrome of inappropriate antidiuretic hormone (SIADH).

Mild hyponatremia can present with headache, nausea, and


lethargy, whereas severe hyponatremia presents with
seizures, coma, and death. Treatment for SIADH includes
fluid restriction, salt replacement, and vasopressin-2 receptor
antagonists.

REFERENCES
1. Parham WA, Mehdirad AA, Biermann KM, et al. Hyperkalemia
revisited. Tex Heart Inst J. 2006;33(1):40-47.
2. Greenberg A. Hyperkalemia: treatment options. Semin Nephrol.
1998;18(1):46-57.
3. Kaeml KS, Wei C. Controversial issues in the treatment of hyperkalaemia. Nephrol Dial Transplant. 2003;18(11):2215-2218.
4. Flatman JA, Clausen T. Combined effects of adrenaline and insulin on active electrogenic Na+-K+ transport in rat soleus muscle.
Nature. 1979;281:580-581.
5. Montoliu J, Lens XM, Revert L. Potassium-lowering effect of
albuterol for hyperkalemia in renal failure. Arch Intern Med.
1987;147:713-717.
6. Allon M, Copkney C. Albuterol and insulin for treatment of hyperkalemia in hemodialysis patients. Kidney Int. 1990;38:869-872.
7. Allon M, Dunlay R, Copkney C. Nebulized albuterol for acute
hyperkalemia in patients on hemodialysis. Ann Int Med.
1989;110:426-429.

PMPH_CH107.indd 850

8. Mandelberg A, Krupnik Z, Huori S, et al. Sambutamol metereddose inhaler with space for hyperkalemia. Chest. 1999;115:617-622.
9. Gruy-Kapral C, Emmett M, Santa Ana CA, et al. Effect of single
dose resin cathartic therapy on serum potassium concentration in patients with end-stage renal disease. J Am Soc Nephrol.
1998;9:1924-1930.
10. Gutzwiller JP, Schneditz D, Huber AR, et al. Increasing blood
flow increases kt/V(urea) and potassium removal but fails to
improve phosphate removal. Clin Nephrol. 2003;59:130-136.
11. Blumberg A, Weidmann P, Ferrari P. Effect of prolonged bicarbonate administration on plasma potassium in terminal renal
failure. Kidney Int. 1992;41:369-374.
12. Allon M, Shanklin N. Effect of bicarbonate administration on
plasma potassium in dialysis patients: interactions with insulin
and albuterol. Am J Kidney Dis. 1996;28:508-514.
13. Ngugi NN, McLigeyo SO, Kayima JK. Treatment of hyperkalaemia by altering the transcellular gradient in patients with renal
failure: effect of various therapeutic approaches. East Afr Med J.
1997;74:503-509.

5/22/2012 6:12:44 PM

Electrolytes and Fluids

14. Capan L, Nahas GG, Manne J, et al. Guidelines for the treatment
of acidaemia with THAM. Drugs. 1998;55(12):191-224.
15. Hoste EA, Colpaert K, Vanhelder RC. Sodium bicarbonate versus THAM in ICU patients with mild metabolic acidosis. J Nephrol. 2005;18(3):303-307.
16. Engstrom M, Schott U, Rommer B, et al. Acidosis impairs the
coagulation: a thromboelastographic study. J Trauma. 2006;61:
624-628.
17. MartiniW, Dubick M, Wade C, Holcomb J. Evaluation of trishydroxymethylaminomethane on reversing coagulation abnormalities caused by acidosis in pigs. Crit Care Med. 2007;35(6):
1568-1574.
18. Marfo K, Garala M, Kvetan V. Case report: severe lactic acidosis
in highly active antiretroviral therapy (HAART) in HIV. J Clin
Pharm Ther. 2009;34(1):119-123.
19. Gan TJ, Mythen MG, Glass PS. Intraoperative gut hypoperfusion may be a risk factor for postoperative nausea and vomiting.
Br J Anaesth. 1997;78:476.
20. Chaudhary S, Sethi AK, Motiani P, Adatia C. Pre-operative
intravenous fluid therapy with crystalloids or colloids on
post-operative nausea and vomiting. Ind J Med Res. 2008;127:
577-581.
21. Lambert KG, Wakim JH, Lambert NE. Preoperative fluid
bolus and reduction of postoperative nausea and vomiting in
patients undergoing laparoscopic gynecologic surgery. AANAJ.
2009;77(2):110-114.
22. Schuster R, Alami RS, Curet MJ. et al. Intra-operative fluid volume influences postoperative nausea and vomiting after laparoscopic gastric bypass surgery. Obes Surg. 2006;16(7):848-851.
23. Chohedri AH, Matin M, Khosravi A. The impact of operative
fluids on the prevention of postoperative anesthetic complications in ambulatory surgeryhigh dose vs low dose. Middle East
J Anesthesiol. 2006;18(6):1147-1156.
24. The SAFE Study Investigators. A comparison of albumin and
saline for fluid resuscitation in the intensive care unit. N Engl J
Med. 2004;350:2247-2256.
25. Dellinger RP, Levy MM, Carlet JM, et al. Surviving Sepsis Campaign: international guidelines for management of severe sepsis
and septic shock: 2008. Intensive Care Med. 2008;34:17-60.
26. Delaney AP, Dan A, McCaff rey J, Finfer S. The role of albumin as
a resuscitation fluid for patients with sepsis: a systematic review
and meta-analysis. Critical Care Med. 2011;39(2):386-391.
27. Laxenaire MC, Charpentier C, Feldman L. Reactions anaphylactoides aux substituts colloidaux du plasma: incidence, facteurs de
risqu, mecanismes. Ann Fr Anesth Reanim. 1994;13:301-310.
28. Treib J, Haass A, Pindur G, et al. Influence of low molecular
weight hydroxyethyl starch (HES 40/0.5-0.55) on hemostasis and
hemorheaology. Haemostasis. 1996;26(5):258-265.
29. Bepperling F, Opitz J, Leuschner J. HES 130/0.4, a new HES specification: tissue storage after multiple infusions in rats. Crit Care.
1999;3(Suppl 1):76-77.
30. Ghandi S, Weiskopf R, Jungheinrich C, et al. Volume replacement therapy during major orthopedic surgery using Voluven
(Hydroxyethyl Starch 130/0.4) or hetastarch. Anesthesiology.
2007;106(6):1120-1127.
31. Jungheinrich C, Scharpf R, Wargenau M, et al. The pharmacokinetics and tolerability of an intravenous infusion of the new
hydroxyethyl starch 130/0.4 (6%, 500 mL) in mild-to-severe renal
impairment. Anesth Analg. 2002;95:544-551.
32. Wittlinger M, Schlapfer M, De Conne E, et al. The effect of
hydroxyethyl starches (HES 130/0.4 and HES 200/0.5) on activated renal tubular epithelial cells. Anesth Analg. 2010;110(2):
300-301.

PMPH_CH107.indd 851

851

33. Bepperling F, Opitz J, Waitzinger J, et al. HES 130/0.4, a new HES


specification: pharmacokinetics after multiple infusions of 10%
solutions in healthy volunteers. Crit Care. 1999;3(Suppl 1):P154.
34. Leuschner J, Opitz J, Winkler A, et al. Tissue storage of 14Clabelled hydroxyethyl starch (HES) 130/0.4 and HES 200/0.5
after repeated intravenous administration to rats. Drugs R & D.
2003;4(6):331-338.
35. Waitzinger J, Bepperling F, Pabst G, et al. Hydroxyethyl starch
(HES) [130/0.4], a new HES specification: pharmacokinetics and
safety after multiple infusions of 10% solution in healthy. Drugs
R & D. 2003;4(3):149-157.
36. Haisch G, Boldt J, Krebs C, et al. The influence of intravascular volume therapy with a new hydroxyethyl starch preparation
(6% HES 130/0.4) on coagulation in patients undergoing major
abdominal surgery. Anesth Analg. 2001;92:565-571.
37. Sander O, Reinhart K, Meier-Hellmann A. Equivalence of
hydroxyethyl starch HES 130/0.4 and HES 200/0.5 for perioperative volume replacement in major gynaecological surgery. Acta
Anaesthesiologica Scandinavica. 2003;47(9):1151-1158.
38. Kozek-Langenecker S, Jungheinrich C, Sauermann W, et al. The
effects of hydroxyethyl starch 130/0.4 (6%) on blood loss and use
of blood products in major surgery: a pooled analysis of randomized clinical trials. Anesth Analg. 2008;107(2):382-390.
39. Krendel DA. Hypermagnesemia and neuromuscular transmission. Semin Neurol. 1990;10:42.
40. Dube L, Granry J-C. The therapeutic use of magnesium in anesthesiology, intensive care and emergency medicine: a review.
Can J Anesth. 2003;50(7):732-746.
41. Mordes JP, Swartz R, Arky RA. Extreme hypermagnesemia as
a cause of refractoryhypotension. Ann Intern Med. 1975;83:
657-658.
42. Millette CH, Snodgrass GL. Acute renal failure associated with
chronic antacid ingestion. Am J Hosp Pharm. 1981;38:13521355.
43. Castelbaum AR, Donofrio PD, Walker FO, Troost BT. Laxative
abuse causing hypermagnesemia, quadriparesis, and neuromuscular junction defect. Neurology. 1989;39:746-747.
44. Fung MC, Weintraub M, Bowen DL. Hypermagnesemia. Arch
Fam Med. 1995;4(8):718-723.
45. Dinghra S, Solven F, Wilson A, et al. Hypomagnesemia and
respiratory muscle power. Am Rev Respir Dis. 1984;129(3):
497-498.
46. Rochester DF. Respiratory effects of respiratory muscle weakness
and atrophy. Am Rev Respir Dis. 1986;134:1083-1086.
47. Armenguard HN, Aubier M. Clinical important of inspiratory
muscle fatigue in acutely ill patients. J Crit Care. 1988;3:267-274.
48. Roussos C, Macklem PT. The respiratory muscles. N Engl J Med.
1982;307:786- 797.
49. Fiaccadori E, Del Canale S, Coffrini E, et al. Muscle and serum
magnesium in pulmonary intensive care patients. Crit Care Med.
1988;16:751-760.
50. Johnson D, Gallagher C, Cavanaugh C, et al. The lack of effect
of routine magnesium administration on respiratory function in
mechanically ventilated patients. Chest. 1993;104:536-541.
51. Mythen MG, Webb AR. Intra-operative gut mucosal hypoperfusion is associated with increased post-operative complications
and cost. Intensive Care Med. 1994;20:99-104.
52. Lobo DN, Bostock KA, Neal KR, Perkins AC, Rowlands BJ, Allison SP. Effect of salt and water balance on recovery of gastrointestinal function after elective colonic resection: a randomised
controlled trial. Lancet. 2002;359(9320):1812-1818.
53. Brandstrup B, Tnnesen H, Beier-Holgersen R, et al.; Danish Study Group on Perioperative Fluid Therapy. Effects of

5/22/2012 6:12:44 PM

852

54.

55.

56.

57.

58.

59.

60.

61.

62.
63.

64.

Surgery: Evidence-Based Practice

intravenous fluid restriction on postoperative complications:


comparison of two perioperative fluid regimens: a randomized assessor-blinded multicenter trial. Ann Surg. 2003;238(5):
641-648.
Nisanevich V, Felsenstein I, Almogy G, Weissman C, Einav S,
Matot I. Effect of intraoperative fluid management on outcome
after intraabdominal surgery. Anesthesiology. 2005;103(1):25-32.
Futier E, Constantin JM, Petit A, et al. Conservative vs restrictive
individualized goal-directed fluid replacement strategy in major
abdominal surgery: a prospective randomized trial. Arch Surg.
2010;145(12):1193-1200.
Noblett SE, Snowden CP, Shenton BK, Horgan AF. Randomized clinical trial assessing the effect of Doppler-optimized fluid
management on outcome after elective colorectal resection. Br J
Surg. 2006;93:1069-1076.
Cooper DJ, Myles PS, McDermott FT, et al. Prehospital hypertonic saline resuscitation of patients with hypotension and severe
traumatic brain injury: a randomized controlled trial. JAMA.
2004;291(11):1350-1357.
Vialet R, Albanese J, Thomachot L, et al. Isovolume hypertonic
solutes (sodium chloride or mannitol) in the treatment of refractory posttraumatic intracranial hypertension: 2 mL/kg 7.5%
saline is more effective than 2 mL/kg 20% mannitol. Crit Care
Med. 2003;31:1683-1687.
Battison C, Andrews PJD, Graham C, Petty T. Randomized, controlled trial on the effect of a 20% mannitol solution and a 7.5%
saline/6% dextran solution on increased intracranial pressure
after brain injury. Crit Care Med. 2005;33:196-202.
Kerwin AJ, Schinco MA, Tepas JJ, et al. The use of 23.4% hypertonic saline for the management of elevated intracranial pressure in patients with severe traumatic brain injury: a pilot study.
J Trauma. 2009;67(2):277-282.
Bulger EM, May S, Brasel KJ, et al. Out-of-hospital hypertonic
resuscitation following severe traumatic brain injury: a randomized controlled trial. JAMA. 2010;304(13):1455-1464.
Brunicardi FC. Schwartzs Principles of Surgery. 8th ed. New
York: McGraw-Hill Medical Publishing Division; 2005:49-51.
Hosking DJ, Cowley A, Bucknall CA. Rehydration in the treatment of severe hypercalcaemia. Q J Med. 1981;50(200):473-474,
nt.81.
Upadhyay UM, Gormley WB. Etiology and management of
hyponatremia in neurosurgical patients. J Intensive Care Med.
2011 Feb 23.

PMPH_CH107.indd 852

65. Sherlock M, Thompson CJ. The syndrome of inappropriate antidiuretic hormone: current and future management options. Eur
J Endocrinol. 2010;162(Suppl 1):S13-S18. Epub Feb 17, 2010.
66. Adams SM, Knowles PD. Evaluation of a first seizure. Am Fam
Physician. 2007;75(9):1342-1347.
67. Bennett M, Dent CL, Ma Q, et al. Urine NGAL predicts severity
of acute kidney injury after cardiac surgery: a prospective study.
Clin J Am Soc Nephrol. 2008;3(3):665-673. Epub 2008 Mar 12.
68. Soto K, Coelho S, Rodrigues B, et al. Cystatin C as a marker of
acute kidney injury in the emergency department. Clin J Am Soc
Nephrol. 2010;5(10):1745-1754. Epub 2010 Jun 24.
69. Ramesh G, Krawczeski CD, Woo JG, Wang Y, Devarajan P. Urinary
netrin-1 is an early predictive biomarker of acute kidney injury
after cardiac surgery. Clin J Am Soc Nephrol. 2010;5(3):395-401.
Epub 2009 Dec 10.
70. Hellman RN, Decker BS, Murray M. Elevated serum creatinine
and a normal urinalysis: a short differential diagnosis in the etiology of renal failure. Ren Fail. 2006;28(5):389-394.
71. Slomp J, van der Voort PH, Gerritsen RT, Berk JA, Bakker AJ.
Albumin-adjusted calcium is not suitable for diagnosis of
hyper- and hypocalcemia in the critically ill. Crit Care Med.
2003;31(5):1389-1393.
72. Clase CM, Norman GL, Beecroft ML, Churchill DN. Albumincorrected calcium and ionized calcium in stable haemodialysis
patients. Nephrol Dial Transplant. 2000;15(11):1841-1846.
73. Dickerson RN, Alexander KH, Minard G, Croce MA, Brown RO.
Accuracy of methods to estimate ionized and corrected serum
calcium concentrations in critically ill multiple trauma patients
receiving specialized nutrition support. JPEN J Parenter Enteral
Nutr. 2004;28(3):133-141.
74. Toy P, Gajic O. Transfusion-related acute lung injury. Anesth
Analg. 2004;99(6):1623-1624, table of contents.
75. Moore SB. Transfusion-related acute lung injury (TRALI):
clinical presentation, treatment, and prognosis. Crit Care Med.
2006;34(5 Suppl):S114-S117.
76. Verbalis JG, Goldsmith SR, Greenberg A, Schrier RW, Sterns
RH. Hyponatremia treatment guidelines 2007: expert panel recommendations. Am J Med. 2007;120:S1S21.
77. Soupart A, Gross P, Legros JJ, et al. Successful long-term treatment of hyponatremia in syndrome of inappropriate antidiuretic
hormone secretion with satavaptan (SR121463B), an orally active
nonpeptide vasopressin V2-receptor antagonist. Clin J Am Soc
Nephrol. 2006;1:1154-1160.

5/22/2012 6:12:44 PM

Commentary on
Electrolytes and Fluids
David B. Hoyt

The chapter by Milligan et al. covers several important topics in


fluids and electrolytes and creates a mosaic of current clinically
relevant topics in this area. In general, the evidence basis is clear
and makes support of the conclusions appropriate.
Hyperkalemia is presented from the standpoint of correction.
The principles of treatment with calcium gluconate and shifting
potassium intracellularly with insulin remain the basic fundamentals of treatment. A third traditional mechanism of treatment
through elimination using resins such as sodium-polystyrene
sulfonate are de-emphasized due to the slowness with which they
work and associated problems with constipation. Similarly, the
use of sodium bicarbonate does not stand up to recent scrutiny.
The use of -agonist therapy such as albuteral has been advocated
by some as an adjunct to calcium and insulin. Since intravenous
-agonists are not available in the United States, this technique
requires metered-dose-inhaled application. This is less popular
and is used inconsistently.
With regard to fluid therapy, early adequate crystalloid resuscitation may be associated with a decreased incidence of postoperative nausea and vomiting though there is very little definitive
data making this clear. Crystalloid and colloid are equal in this
effect. Albumin continues to show no difference in mortality outcomes even when septic patients are studied. New data regarding
albumins superiority in septic patients is still based on retrospective review and has not changed this perspective. HES 130/0.4
(Voluven) is a third-generation colloid designed to avoid allergic
reactions, renal failure, tissue accumulation, and coagulopathy.
It appears to have made significant progress in achieving these
goals though no data to date suggest its superiority to other colloid solutions. The effect of fluid volume on anastomotic leak rates
in abdominal surgery is still unclear. Getting the right amount of

fluid resuscitation appears to lower anastomotic leaks but agreement on what the right amount is continues to be a question.
Hypertonic saline continues to be evaluated in shock and head
injury. Hypertonic saline appears to be better than mannitol at
lowering elevated intracranial pressure (ICP) but has not been
shown to be beneficial in other areas.
The role of other specific electrolyte conditions continues to
be of interest. Hyper- or hypomagnesemia contributes to respiratory failure, and maintaining normal serum magnesium levels is
optimal for respiratory muscle function. Severe hypocalcemia continues to be caused by malignancies. Rapid correction is affected
by dilution with normal saline, the prevention of further release
from the bones, and increased excretion by diuretics. Hyponatremia (the most common electrolyte abnormality) is most often
caused by SIADH. Mild hyponatremia can present with headache,
nausea, and lethargy whereas severe hyponatremia can cause
seizures, coma, and death. The treatment of underlying SIADH
includes emphasis on fluid restriction, replacement of salt, and the
use of vaporpressin-2 receptor antagonists. Early recognition of
SIADH can avoid these severe complications of hyponatremia.
The effect of serum albumin on calcium measurements continues to be emphasized and serum calcium will increase by 0.8
mg/dL for every 1g/dL rise in serum albumin. Ionized calcium is
unaffected by albumin and this distinction should be made when
evaluating calcium levels clinically.
The approach of dividing the broad topic of fluids electrolytes
into specific questions about individual electrolyte abnormalities
or questions relative to fluid type is important. The authors have
clarified several issues that are seen in day-to-day practice. The
clear need for ongoing research will clarify these issues even more
going forward.

853

PMPH_CH107.indd 853

5/22/2012 6:12:44 PM

CHAPTER 108

Surgical Nutrition
Jayson D. Aydelotte

INTRODUCTION

nutrition index (PMI) did correlate with a higher pancreatic fistula rate, a BMI of less than 18.5 kg/m2 alone did not.4
A relatively large retrospective study investigating outcomes
after radical cystectomy in patients with a nutritional deficiency
found similar results. This study comprised 585 patients who were
divided into two groups: those who were nutritionally deficient
(the ND group, which meant they had preoperative albumin levels
less than 3.5 g/dL, BMI less than 18.5 kg/m2, or recent weight loss
of 5% or greater) and those who were not. The ND group had a
lower 90-day and a lower 3-year survival rate than those patients
who were not nutritionally deficient. However, when evaluated on
their own, neither BMI nor weight loss had a significant impact
on survival.2
Answer: A decreased BMI of less than 18 kg/m2 may be associated with increased risk for poor surgical outcome (Grade C
recommendation).

People who undergo surgery all have one thing in common, they
have to heal. To do this, they need energy to power their immune
system to fight infection and protein and other building blocks to
reconstruct the local environment manipulated by the surgery. It
is a delicate balance that is affected in both the preoperative and
postoperative time periods. For the most part, this is a microcosm
of life: consume or be consumed.
Although many other specialties are exposed to malnutrition, surgeons must approach patients nutritional status with
a careful eye and sound decisions, so they can maximize the
patients survival and minimize morbidity. To that end, several
questions can be asked that relate to the overall risk/diagnosis
of malnutrition, the preoperative treatment of the malnourished
patient, and the different treatment modalities for delivering calories and protein.

2. Is there a good, reliable laboratory test that helps predict


better/worse outcomes for patients who need surgery?

RISK/DIAGNOSIS OF MALNUTRITION

Variables in nutrition are very good scientific study media as they


are relatively consistent throughout the United States and the
world. Probably the most well studied, partially for this reason,
is serum albumin levels. Several studies have evaluated the predictive value of low serum albumin.1-3,5,6 Although not all studies found a positive correlation between low albumin and poor
outcome,1,4,5 several others found a strong correlation between
increased mortality and low albumin.2,3,6 The same study that
identified nutritionally deficient patients as having worse 90-day
and 3-year mortality rates found that a low albumin (25th percentile vs. 75th percentile) was an independent predictor of
all-cause mortality.2 Likewise, Mullen et al. found a statistically significant decrease in serum albumin concentration (3.64
mg/dL) linked to its underweight group, which had a five-fold
increase in mortality.3
However, one of the most often quoted and the largest study
done to evaluate preoperative albumin and outcome was the compiled results from the National VA Surgical Risk Study published

1. Are there quick, reliable clinical tests that can be done to


assess whether a patient is malnourished and at risk for a worse
outcome?
Ideally, a surgeon would like to be able to assess risk in the most
efficient, least invasive way possible. With regard to nutritional status, a history of recent weight loss and body-mass index (BMI) provide two relatively reliable clinical indicators of undernutrition.1-4
One study done to assess outcomes relative to BMI appeared to be
done to assess for poor outcomes with regard to obesity (increasing BMI). However, there were no significant differences in outcomes as patients weights:height went up. More importantly,
there was a five-fold increase in mortality for those patients with
a BMI of less than 18.5 kg/m2, that is, for those who were malnourished.3 Another study published in 2010 sought to evaluate
the effects of preoperative nutrition on outcomes after pancreatic surgery. Whereas a low BMI combined with a prognostic
854

PMPH_CH108.indd 854

5/22/2012 6:13:18 PM

Surgical Nutrition

in 1999. Th is was a huge, prospectively collected observational


study of over 52,000 patients. The investigators found a strong,
statistically significant correlate between a lower serum albumin
level and increasing mortality. In fact, of all the variables collected
in this cohort, including the American Society of Anesthesiologists
(ASA) class, hematocrit, blood urea nitrogen (BUN), functional
status, prothrombin time, and even age, serum albumin levels
were the strongest predictor of outcome. Whereas patients with
serum albumin levels above 4.6 mg/dL had a mortality of close
to 1%, those with serum albumin less than 2.1 mg/dL had a mortality of close to 30%. Interestingly, this study and the regression
analysis done to identify the correlation between serum albumin
and poor outcome also identified the serum level of 3.5 mg/dL as
the significant level at which mortality and other poor outcomes
begin to rise significantly. This study is perhaps the strongest data
set linking preoperative serum albumin levels and outcome.6
Answer: Low serum albumin is a strong indicator of malnutrition and predictor of poor outcome (Grade B recommendation).
3. Is there a test that raises concern for malnutrition for patients
already admitted to a hospital?
Many hospitals employ Registered Dieticians and Nutritionists to
perform complicated, thorough nutritional assessments that are
typically viewed as the gold standard.1,5,7,8 However, these assessments must be ordered by either the physicians or the nursing staff
after certain screening measures identify those patients who may
be at risk. These screens may not be the most reliable. There may be
patients in the hospital who are at risk for protein malnutrition
that is not picked up with a screen, and therefore may not get a
registered dietician to perform a thorough assessment.8 So, much
like the preoperative serum albumin, certain laboratory studies
such as prealbumin, retinol-binding protein (RBP), and transferrin have been investigated to try and produce a reliable indicator
for protein-energy malnutrition.1,5,8
One of the most heavily studied of these serum markers is
prealbumin, also known as transthyretin. Prealbumin is a good
screen for protein malnutrition because its half-life is 1.9 days.
Theoretically, declines in prealbumin levels will be a more reliable
indicator than albumin for early detection of protein malnutrition because of its shorter half-life.5 There are several studies that
substantiate this claim.
Devoto and colleagues set out to test this theory comparing
the detailed nutritional assessment tool (their gold-standard for
evaluating in-hospital protein-energy malnutrition) to several
other cheaper, less time-consuming screens in 108 general medicine patients admitted to their hospital. The three screens were the
subjective global assessment (a clinically driven assessment using
history of weight loss and specific physical findings associated
with malnutrition), the PINI scoring system (a calculation using
the serum levels of albumin, 1-acid glycoprotein, and CRP), and
prealbumin levels. Prealbumin levels showed the highest concordance with detailed nutritional assessment tool.8
Another study published in JAMA in 1999 prospectively evaluated nearly 500 patients admitted to both surgical and medical
services at a large VA medical center. The investigators followed
the patients from admission, throughout their hospitalization,
and 90 patients after discharge collecting complete, thorough
clinical and laboratory nutritional assessments including calorie/protein intake calculations and several serum markers. After
accrual of data, patients were divided into two groupsthe low

PMPH_CH108.indd 855

855

nutrient group (which comprised patients who took in less than


50% of their average daily nutrient intake) and all the others.
Several poor outcomes were attributed to the low nutrient group
including an eight-fold increase in in-hospital mortality and a
nearly three-fold increase in 90-day mortality. But two laboratory
testsserum albumin (less than 3) and serum prealbumin (less
than 20)correlated very well with identifying the low nutrient
group. This was a significant finding in this study because it validated simple lab tests as correlates for expensive, time-consuming
nutritional assessments.1
Another study published in 2003 examined the utility of a
nurse-delivered nutritional screen followed by a registered dieticians assessment for all patients found to be at risk or malnourished. Three serum markers, albumin, prealbumin, and RBP, were
drawn to try and correlate serum markers with dietician-verified
malnutrition. In this study, prealbumin levels less than 20 mg/dL
prompting a dieticians full assessment led to earlier (1.2 days earlier) and more reliable (94% vs. 69%) identification of malnourishment versus standard, nurse-driven screens or even screens
utilizing RBP and albumin.5
Answer: Yes, serum prealbumin less than 20 mg/dL is a good,
reliable test that can identify those patients at risk for malnutrition (Grade C recommendation).

INTERVENTION
So, if it is possible to identify patients who may be malnourished,
the questions then become the following:
4. Can it be fixed and how do we fi x it?
Several, more specific questions, may help develop an answer.
5. Is there any utility in providing patients preoperative enteral
nutrition?
The data surrounding preoperative delivery of enteral nutrition is
plentiful, but not very convincing. Many of the studies that were
carried out to answer this question have several limiting factors
that make this rather simple question difficult to answer.9
For example, many of the studies done on this topic evaluate
a specific type of feed. There are few large studies evaluating large
numbers of patients at risk for malnutrition with traditional
preoperative tube feeds.
One study published in Germany in 2007 evaluated the utility
of two different immune-modulating feeds given preoperatively
against a control group of immunonutrition given only postoperatively. The goals of this study were to identify trends in immunefunction serum markers, but it found a significant decrease in
ICU stay and overall hospital length of stay.10
Likewise, another study conducted in Milan, published in
2002, evaluated two preoperative regimens of another immunemodulating feed. One group received the preoperative feed with
the same postoperative regimen, another received only preoperative feedings with no postoperative feeds, and the control group
got neither pre- nor postoperative feedings. There were no differences in mortality in this study, but hospital length of stay as
well as overall infective complications were decreased in both
treatment groups compared with control.11 But this study did not
compare preoperative versus postoperative feedings and did not

5/22/2012 6:13:18 PM

856

Surgery: Evidence-Based Practice

provide a definitive answer on whether or not delaying surgery for


supplemental enteral feedings is worthwhile.
Another study compared standard enteral therapy (control)
given only postoperatively with two other groups. One was given
immunoenhanced feeds preoperatively with the standard enteral
regimen postoperatively. The other was given the immunomodulating feed pre- and postoperatively. Both of the treatment groups
had shorter hospital stays and less overall complications than the
control group.12 Again, this study did not provide standard enteral
therapy against the immunomodulating feeds in the same way,
making the results difficult to interpret.
A similar study published in The Lancet in 2001 evaluated
preoperative immunonutrition and preoperative standard enteral
feeds in high-risk patients undergoing cardiac surgery. The
aim of the study was to evaluate serum markers of the inflammatory response. However, its clinical conclusions showed a significant decrease in overall infection, pneumonia, volume of IV fluid
delivered, and blood transfusion requirements.13
Overall, the lack of a standardized control group and treatment arm evaluating the idea of preoperative feeds in general
versus no preoperative feeds in a uniformly established malnourished patient population leave this question without a definitive
answer.
Answer: There is no convincing data that establish a true benefit of preoperative enteral feeds versus no preoperative feeding
regimen (Class C recommendation).
6. Is there a benefit to total parenteral nutrition?
The current state of things still does not differ too much from a
landmark study done by the Veterans Affairs TPN Cooperative
Study Group in 1991. Twenty years and many studies later, the
conclusions are still roughly the same. The VA group prospectively gathered data in a random assignment trial of 459 patients,
separating the patients into two groups: the total parenteral nutrition (TPN) group, which received 1000 calories over their resting
metabolic expenditure for 7 to 15 days preoperatively then another
72 hours postoperatively, and the control group, which received
no preoperative TPN and no enteral or parenteral nutrition for
the first 72 hours postoperation. The significant outcomes of this
study were that the TPN group had more infectious complications
of all types, line related and otherwise. There was no mortality difference between the two groups. Interestingly, the patients who
were the most malnourished who received TPN had fewer noninfectious complications when compared with their control-group
counterparts.14
Although many other studies, some even prospective random
assignment trials, have been done to evaluate the utility of TPN,
the overall conclusions remain about the same.9,15,16 Two large
meta-analyses are useful to outline the salient studies done in the
past two decades.
One was published in JAMA in 1998. This was a compilation
of 26 prospective random assignment trials evaluating the use of
TPN versus oral diet with IV dextrose. This analysis found no significant difference in mortality between the two groups. Likewise,
as in the VA Cooperative study, it discovered that TPN may benefit the more-malnourished patients, especially in reducing noninfectious complication rates.15
The other was a meta-analysis published in 2005 looking specifically at studies comparing TPN and enteral feeding in medical,
surgical, and trauma patients. A total of 30 randomized controlled

PMPH_CH108.indd 856

trials were included. Overall, the conclusions of the study identified TPN as having increased risk of infective complications,
catheter-related blood stream infections, and hospital length of
stay. There was no identified mortality difference.16
Answer: The benefit of TPN is still unclear. It still appears
that TPN may have an increased risk of infective complications but may reduce noninfective complications in the moremalnourished (Grade C recommendation).
7. Is there any benefit to feeding beyond the pylorus?
A very common idea among surgeons across the country is that
feeding past the pylorus, or even past the ligament of Treitz, is
safer than feeding the stomach. For this reason, many surgeons
and nonsurgeons advocate the placement of a small bowel feeding catheter instead of feeding through a gastric tube. The difficulty lies in the actual placement of these tubes, some of which
require specialized equipment and facilities to place. Neuman
and colleagues randomized 60 patients to receive gastric feeds
or have a postpyloric tube placed. Patients receiving gastric feeds
had their feeds started sooner and had an earlier time to goal
feeding while having no increased aspiration as compared with
the patients randomized to the postpyloric group.17 Another
study with similar numbers in children came to an opposite conclusion. The postpyloric group achieved a higher percentage of
daily caloric goal, but had the same complication rate as stomach
feeding. However, in this study the investigators suffered from
the same problem that lead to the original question in that nearly
30% of the patients randomized to the postpyloric group could
not have their tube placed properly and were then switched to
the gastric feeding group.18 There were no significant differences
in complications between gastric and postpyloric tube feeding in
this study.
Answer: It is safe to feed a working stomach. Placing a small
bowel feeding tube delays time to goal feeds and does not lower
complication rates (Grade B recommendation).
8. If feeding the stomach is safe, is there a difference between
continuous tube feeds or intermittent tube feeds?
Continuous tube feeding in the stomach has raised several concerns
including early satiety, increasing the inability to take in oral food,
and decreasing the total number of delivered calories from stoppages of feeds.19,20 Several studies have evaluated these issues.
One study was aimed at identifying hormonal satiety mediators and total caloric intake after the addition of bolused tube
feeds in healthy men. The men were blinded to what was being
bolused into their stomachs. Their total caloric intake, their food
intake, and several serum markers for satiety/hunger were evaluated. The bolused tube feeds did cause a slight decrease in the
amount of oral food intake as compared with the placebo boluses,
but overall caloric intake was increased. As compared to continuous tube feeds, the intermittent feeds had a greater, not lesser,
effect on food-by-mouth appetite and consumption. Likewise, the
serum markers for satiety (ghrelin) were affected more with the
bolused tube feeds.19
Shimoni and colleagues evaluated four different types/delivery methods of tube feedings in elderly patients on a medicine service. The four groups consisted of intermittent feeds with/without
additional fiber and continuous feeds with and without additional
fiber. The only statistically significant result in this study was

5/22/2012 6:13:18 PM

Surgical Nutrition

a relative risk of diarrhea decrease in those patients who received


continuous tube feeds with additional fiber.21 This conclusion was
actually against conventional wisdom at the time that implicated
continuous tube feeds with more diarrhea versus intermittent,
bolused feeds.
A prospective random assignment trial evaluating continuous
versus intermittent tube feeds in critically ill trauma patients was
published in 2007. The study hypothesized that an intermittent

857

tube feeding regimen would lead to an optimized caloric intake


in the first 7 days of therapy versus continuous tube feeding. It
found that the total caloric intake for the first 7 days of ICU stay
was equivalent between the two groups. Likewise, there were no
significant increases in complications between the groups.20
Answer: There is no good evidence that identifies an advantage for either intermittent or continuous tube feeding regimen
(Grade C recommendation).

Clinical Question Summary


Question

Answer

Grade of
Recommendation

1 Are there quick, reliable clinical


tests that can be done to
assess whether a patient is
malnourished and at risk for a
worse outcome?

A decreased BMI of less than 18 kg/m2 may be


associated with increased risk for poor surgical
outcome.

2, 4, 5, 8

2 Is there a good, reliable laboratory


test that helps predict better/
worse outcomes for patients
who need surgery?

Low serum albumin is a strong indicator of


malnutrition and predictor of poor outcome.

2, 4, 5, 7-9

3 Is there a test that raises the


concern for malnutrition for
patients already admitted to the
hospital?

Yes, serum prealbumin less than 20 mg/dL is


a good, reliable test that can identify those
patients at risk for malnutrition.

1, 2, 3, 7

4 Is there any utility in providing


patients preoperative enteral
nutrition?

There is no convincing data that establish a true


benefit of preoperative enteral feeds versus no
preoperative feeding regimen.

6, 12, 15-17

5 Is there a benefit to total


parenteral nutrition?

The benefit of TPN is still unclear. It still appears


that TPN may have an increased risk of infective
complications but may reduce noninfective
complications in the more-malnourished.

6, 10, 11, 14

6 Is there any benefit to feeding


beyond the pylorus?

It is safe to feed a working stomach. Placing a small


bowel feeding tube delays time to goal feeds and
does not lower complication rates.

21, 22

7 If feeding the stomach is safe,


is there a difference between
continuous tube feeds or
intermittent tube feeds?

There is no good evidence that identifies an


advantage for either intermittent or continuous
tube feeding regimen.

18-20

REFERENCES
1. Sullivan D, Sun S, Walls RC. Protein-energy undernutrition among
elderly hospitalized patients. A prospective study. JAMA. 1999;
281(21):2013-2019.
2. Gregg J, Cookson M, Phillips S, et al. Effect of pre-operative nutritional deficiency on mortality after radical cystectomy for bladder
cancer. J Urol. 2011;185(1):90-96.
3. Mullen J, Davenport D, Hutter MM, et al. Impact of body mass
index on perioperative outcomes in patients undergoing major intraabdominal cancer surgery. Ann Surg Oncol. 2008;15(8):2164-2172.
4. Kanda M, Fujii T, Kodera Y, Nagai S, Takeda S, Nakao A. Nutritional predictors of postoperative outcome in pancreatic cancer.
Br J Surg. 2010;98:268-274.
5. Robinson M, Trujillo E, Mogensen KM, Rounds J, McManus K,
Jacobs DO. Improving nutritional screening of hospitalized

PMPH_CH108.indd 857

6.

7.

8.

9.

References

patients: the role of prealbumin. J Parenter Enter Nutr. 2003;27(6):


389-395.
Gibbs J, Cull W, Henderson WG, Daley J, Hur K, Khuri SF. Preoperative serum albumin level as a predictor of operative mortality
and morbidity: results from the National VA Surgical Risk Study.
Arch Surg. 1999;134:36-42.
Rambod M, Kovesdy C, Bross R, Kopple JD, Kalantar-Zadeh K.
Association of serum prealbumin and its changes over time with
clinical outcomes and survival in patients receiving hemodialysis.
Am J Clin Nutr. 2008;88:1485-1494.
Devoto G, Gallo F, Marchello C, et al. Prealbumin serum concentrations as a useful tool in the assessment of malnutrition in hospitalized patients. Clin Chem. 2006;52(12):2281-2285.
Hebbar R, Harte B. Do preoperative nutritional interventions
improve outcomes in malnourished patients undergoing elective
surgery? Cleve Clin J Med. 2007;74(September suppl):S8-S10.

5/22/2012 6:13:18 PM

858

Surgery: Evidence-Based Practice

10. Giger U, Buchler M, Farhadi J, et al. Preoperative immunonutrition suppresses perioperative inflammatory response in patients
with major abdominal surgerya randomized controlled pilot
study. Ann Surg Oncol. 2007;14(10):2798-2806.
11. Gianotti L, Braga M, Vignali A, Di Carlo V. A randomized
controlled trial of preoperative oral supplementation with a
specialized diet in patients with gastrointestinal cancer. Gastroenterology. 2002;122:1763-1770.
12. Braga M, Gianotti L, Nespoli L, Radaelli G, Di Carlo V. Nutritional approach in malnourished surgical patients. Arch Surg.
2002;137:174-180.
13. Tepaske R, Velthuis H, Oudemans-van Straaten HM, et al. Effect
of preoperative oral immune-enhancing nutritional supplement
on patients at high risk of infection after cardiac surgery: a randomized placebo-controlled trial. Lancet. 2001;358:696-701.
14. Veterans Affairs Cooperative Study Group.; Perioperative total
parenteral nutrition in surgical patients. NEJM. 1991;325(8):
525-532.
15. Heyland D, MacDonald S, Keefe L, Drover JW. Total parenteral nutrition in the critically ill patient. JAMA. 1998;280(23):
2013-2019.

PMPH_CH108.indd 858

16. Peter J, Moran J, Phillips-Hughes J. A metaanalysis of treatment


outcomes of early enteral versus early parenteral nutrition in
hospitalized patients. Crit Care Med. 2005;33(1):213-220.
17. Neumann D, Delegge M. Gastric versus small-bowel tube feeding in the intensive care unit: a prospective comparison of efficacy. Crit Care Med. 2002; 30(7):1436-1438.
18. Meert K, Daphtary K, Metheny N. Gastric vs small-bowel feeding in critically ill children receiving mechanical ventilation: a
randomized controlled trial. Chest. 2004;126(3):872-878.
19. Stratton R, Stubbs R, Elia M. Bolus tube feeding suppresses food
intake and circulating ghrelin concentrations in healthy subjects in a short-term placebo-controlled trial. Am J Clin Nutr.
2008;88:77-83.
20. MacLeod J, Lefton J, Houghton D, et al. Prospective randomized
control trial of intermittent versus continuous gastric feeds for
critically ill trauma patients. J Trauma. 2007;63(1):57-61.
21. Simoni Z, Averbuch Y, Shir E, et al. The addition of fiber and
the use of continuous infusion decrease the incidence of diarrhea in elderly tube-fed patients in medical wards of a general
regional hospital: a A controlled clinical trial. J Clin Gastroenterol. 2007;41(10):901-905.

5/22/2012 6:13:18 PM

CHAPTER 109

Preoperative Risk Factor


Assessment of the Surgical Patient
Edgar Joseph Pierre, Shawn Michael Cantie, and Faisal Huda

INTRODUCTION

1. What are the major factors that determine the preoperative


risk of the surgical patient?

The purpose of preoperative evaluation is not simply to just give


medical clearance but rather to perform an evaluation of the patients
current medical status while making recommendations concerning
the evaluation, management, and risk of cardiac (or medical) problems over the entire perioperative period. Furthermore, a complete
evaluation will provide a clinical risk profile that the patient, primary physician, anesthesiologist, and surgeon can use in making
treatment decisions that may influence short-term and long-term
cardiac outcomes. No evaluation is complete without a thorough and
complete physical examination. The physical examination should
include a brief neurological examination along with auscultation of
the heart and lungs and evaluation for peripheral pulses and edema
(see Table 109.1). The overall goal of the preoperative evaluation is
to assess medical problems in surgical patients, to determine how to
manage these problems preoperatively and intraoperatively, and to
provide recommendations for postoperative care. Preoperative risk
assessment is a critical part of this process. This chapter will discuss
the general principles of the preoperative evaluation.

The preoperative risk assessment begins by identifying the type of


surgery and the patient undergoing surgery. There are two major
factors that determine the risk of complications: (1) the type of
surgery to be performed and (2) the type of patient who is undergoing the procedure. For example, a patient with cardiac disease
is at relatively low risk for cataract surgery whereas a patient with
cardiac disease is at relatively high risk for a pneumonectomy. The
first of these factors is the surgical risk.
Answer: The two major factors are type of surgery and the type
of patient undergoing the surgery (Grade B recommendation).
2. What types of surgery have the greatest risk factors?
Emergency surgery of any kind carries the highest risk of mortality and postoperative complications. In general, operative death
is uncommon, occurring in only 0.3% of all operations. Average risk implies perioperative mortality of 1% or less. Significant
risk implies 1% to 10% and high risk is 10% to 20%. Procedures
associated with higher mortality and complication rates include
major vascular surgery, cardiac surgery, intraperitoneal surgery,
and craniectomy. As previously mentioned, emergency surgery
of any type carries a higher risk of mortality and postoperative
complications.
The second factor of the risk equation is determined by the
patients health. The American Society of Anesthesiology (ASA)
classification of the patients gives an impression of the patients
current clinical state that correlates with surgical outcomes. The
classifications range from 1 to 6, with 1 being healthy to 6 being
brain dead (Table 109.2).
Answer: The type of surgery with the greatest risk is emergency surgery of any kind (Grade B recommendation).

Table 109.1 Components of a Physical Examination


Central nervous system
Level of consciousness
Sensory or skeletal muscle dysfunction

Cardiovascular system
Auscultation of the heart (rate, rhythm, murmurs)
Blood pressure (supine and standing)
Peripheral pulses
Peripheral edema

Pulmonary system
Ausculation of lungs (rales, wheezing)

3. What are the guidelines for reducing cardiac complications


of noncardiac surgery?

Pattern of breathing
Anatomy of thorax (barrel chest)

a. Cardiac complications of noncardiac surgery


859

PMPH_CH109.indd 859

5/22/2012 6:13:49 PM

860

Surgery: Evidence-Based Practice

Table 109.2 ASA Physical Classifications


ASA Classifications

Description

ASA - 1

Healthy patient, no current medical problems

ASA - 2

Patient with mild systemic disease that is controlled, e.g., HTN, DM

ASA - 3

Patient with severe systemic disease that is uncontrolled, e.g., uncontrolled HTN, previous MI

ASA - 4

Patient with severe systemic disease that is constant threat to life, e.g., CHF, renal failure

ASA - 5

Patient not expected to survive without the operation, e.g., PE, ruptured aneurysm

ASA - 6

Brain dead patient whose organs will be donated

Emergency operation (E)

Any patient who requires an emergency operation, e.g., a healthy 21-year-old with acute appendicitis (1E)

Table 109.3 Evaluation of Cardiac Risk Factors


Step 1
What is the urgency of noncardiac surgery? Certain emergencies do not allow time for preoperative cardiac evaluation. Postoperative
risk stratification may be appropriate for some patients who have not had such an assessment before.

Step 2
Has the patient undergone coronary revascularization in the past 5 years? If so, and if clinical status has remained stable without
recurrent symptoms/signs of ischemia, further cardiac testing is generally not necessary.

Step 3
Has the patient had a coronary evaluation in the past 2 years? If coronary risk was adequately assessed and the findings were favorable,
it is usually not necessary to repeat testing unless the patient has experienced a change or new symptoms of coronary ischemia
since the previous evaluation.

Step 4
Does the patient have an unstable coronary syndrome or a major clinical predictor of risk? When elective noncardiac surgery is being
considered the presence of unstable coronary disease, decompensated HF, symptomatic arrhythmias, and/or severe valvular heart
disease usually leads to cancellation or delay of surgery until the problem has been identified and treated.

Step 5
Does the patient have intermediate clinical predictors of risk? The presence or absence of prior MI by history or ECG, angina pectoris,
compensated or prior HF, preoperative creatinine greater than or equal to 2 mg/dL, and/or diabetes mellitus helps to further
stratify clinical risk for perioperative coronary events. Consideration of functional capacity and level of surgery-specific risk allows a
rational approach to identify patients most likely to benefit from further noninvasive testing.

Step 6
Patients without major but with intermediate predictors of clinical risk and moderate or excellent EF or patients with two or more
intermediate predictors of risk.

Step 7
Noncardiac surgery is generally safe for patient with neither major nor intermediate predictors of clinical risk and moderate or
excellent functional capacity (4 METS or greater). Additional testing may be considered on an individual basis for patients without
clinical markers but with poor functional capacity who are facing higher-risk operations particularly those with several minor clinical
predictors of risk who are scheduled to undergo vascular surgery.

Step 8
The results of noninvasive testing can be used to determine the need for additional preoperative testing and treatment. In some
patients with documented CAD, the risk of coronary intervention or corrective cardiac surgery may approach or even exceed the
risk of the proposed noncardiac surgery. The approach may be appropriate, however, if it significantly improves the patients longterm prognosis. For some patients, a careful consideration of clinical, surgery-specific, and functional status attributes leads to a
decision to proceed to coronary angiography.

b. Evaluation of cardiac risk


c. Clinical predictors of cardiac risk (see Table 109.3).
The evaluation of cardiac risk always involves a history and physical examination, and a resting electrocardiogram (ECG). The initial history, physical examination, and ECG assessment should

PMPH_CH109.indd 860

focus on identification of potentially serious cardiac disorders,


including coronary artery disease (CAD), that is, prior myocardial infarction (MI) and angina pectoris, congestive heart failure (CHF), symptomatic arrhythmias, presence of pacemaker or
implantable cardioverter defibrillator (ICD), or a history of orthostatic intolerance.1 The presence of anemia may also place a patient

5/22/2012 6:13:50 PM

Preoperative Risk Factor Assessment

861

Table 109.4 Estimated Energy Requirements for Various Activities


1 MET*
Can you take care of yourself?
Eat, dress, or use the toilet?
Walk indoors around the house?
Walk a block or two on level ground at 2 to 3 mph or 3.2 to 4.8 km per hour

4 METs
Do light work around the house like dusting or washing dishes?
Climb a flight of stairs or walk up a hill?
Walk on level ground at 4 mph or 6.4 km per h?
Run a short distance?
Do heavy work around the house like scrubbing floors or lifting or moving heavy furniture?
Participate in moderate recreational activities like golf, bowling, dancing, doubles tennis, or throwing a baseball or football?

Greater than 10 METs


Participate in strenuous sports like swimming, singles tennis, football, basketball, or skiing?
*MET indicates metabolic equivalent.
Adapted from the Duke Activity Status Index20 and AHA Exercise Standards.96; and Eagle et al. ACC/AHA Perioperative Executive Summary 1261.

at higher perioperative risk.2-4 Other factors that help determine


cardiac risk include functional capacity, age, comorbid conditions
(e.g., diabetes mellitus, peripheral vascular disease, renal dysfunction, and chronic pulmonary disease), and surgical procedures (vascular procedures and prolonged, complicated thoracic, abdominal,
and head and neck procedures, which are all considered higher risk).
Numerous risk indices have been developed over the past 25 years
on the basis of multivariate analyses.5-14
In addition to the presence of CAD and CHF, a history of
cerebrovascular disease, preoperative elevated creatinine greater
than 2 mg/dL, insulin treatment for diabetes mellitus, and highrisk surgery have all been associated with increased perioperative cardiac morbidity. A high-risk patient with known coronary
artery disease would not need further evaluation if he/she is able
to exercise for 30 minutes daily without symptoms. On the other
hand, a sedentary person with clinical factors suggesting the possibility of increased operative risk may benefit from further testing even in the absence of known coronary artery disease.
As always, physical examination should be tailored to the
patients medical condition. The goal of the physical examination is to identify conditions that may increase operative risk. The
American College of Cardiology guidelines identify the following
components of the physical examination:
Functional capacity can be expressed in metabolic equivalent
(MET) levels. Multiples of the baseline-MET value can be used to
express aerobic demands for specific activities. Perioperative cardiac and long-term risks are increased in patients unable to meet
a 4-MET demand during most normal daily activities.15-17 Energy
expenditures for activities such as eating, dressing, walking around
the house, and dishwashing range from 1 to 4 METs; climbing a
flight of stairs, walking on level ground at 6.4 km/h, running a short
distance, scrubbing floors, or playing a game of golf represents 4 to
10 METs; participating in strenuous sports such as running, football, basketball, and so on or being sexually active represents greater
than 10 METs (Table 109.4).

PMPH_CH109.indd 861

Cardiac risk indices are routinely used when assessing the


preoperative patient. Cardiac risk is the most-feared and moststudied complication of surgery and is often an implicit request
to determine the perioperative risk of cardiac complications.
As no patient and no procedure is risk free, the internist should
avoid statements such as avoid hypoxia, hypotension and hypovolemia or cleared for surgery. The role of the anesthesiologist
is to determine the risk factors (patient and surgical), to identify
the patients risk for that procedure, and determined if further
testing is indicated. The original Goldman risk index was based
on a series of patients over the age of 40 undergoing noncardiac
surgery. Nine preoperative factors were found to be associated
with life-threatening cardiac complications and death. Myocardial
infarctions within 6 months, S3 gallop or jugular venous distension, age over 70, arrhythmias, aortic stenosis, poor general medical condition, emergency surgery, and major intraabdominal or
intrathoracic surgery.
More recently, Lee et al.5 validated a much simpler index for
patients undergoing nonurgent major noncardiac surgery known as
the Revised Cardiac Risk Index. Their analysis identified six independent risk factors: ischemic heart disease, congestive heart failure,
cerebral vascular disease, high-risk surgery, preoperative insulin
treatment for diabetes mellitus, and preoperative creatinine greater
than 2mg/dL. Rather than weighing each of these risk factors, they
designated risk classes by the number of risk factors. Patients without any risk factors are assigned to the lowest risk class (I) and
were found to have the cardiac complication rates of 0.5% and 0.4%
in the derivation and validation cohorts. In contrast, patients with
three or more risk factors are assigned to the highest risk class (IV)
and were found to have cardiac complication rates of 9.1% and 11%.
If a test is unlikely to effect posttest probability of poor outcome, it
is an unnecessary component of preoperative risk assessment.
Which patients are most likely to benefit from preoperative
coronary assessment and treatment? The lack of adequately controlled or randomized clinical trials to define the optimal evaluation

5/22/2012 6:13:50 PM

862

Surgery: Evidence-Based Practice

Table 109.5 Clinical Predictors of Increased Perioperative Cardiovascular Risk (Myocardial Infarction, Heart
Failure, Death)
Major
Unstable coronary syndromes

Acute or recent myocardial infarction with evidence of important ischemic risk by clinical symptoms or noninvasive study
Unstable or severe angina (Canadian class III or IV)
Decompensated heart failure
Significant arrhythmias

High-grade atrioventricular block


Symptomatic ventricular arrhythmias in the presence of underlying heart disease
Supraventricular arrhythmias with uncontrolled ventricular rate
Severe valvular disease

Intermediate
Mild angina pectoris (Canadian class I or II)
Previous myocardial infarction by history or pathological Q waves
Compensated or prior heart failure
Diabetes mellitus (particularly insulin-dependent)
Renal insufficiency

Minor
Advanced age
Abnormal ECG (left ventricular hypertrophy, left bundle-branch block, ST-T abnormalities)
Rhythm other than sinus (e.g., atrial fibrillation)
Low functional capacity (e.g., inability to climb one flight of stairs with a bag of groceries)
History of stroke
Uncontrolled systemic hypertension
ECG indicates electrocardiogram
*The American College of Cardiology National Database Library defines recent MI as greater than 7 days but less than or equal to 1 month (30 days);
acute MI is within 7 days.
May include stable angina in patients who are unusually sedentary.
Campeau L. Grading of angina pectoris. Circulation. 1976;54:522-523.

strategy led to the proposed algorithm based on collected observational data and expert opinion. Since publication of the guidelines in
1996, several studies have suggested that this stepwise approach to
the assessment of CAD is both efficacious and cost-effective. A stepwise Bayesian strategy that relies on assessment of clinical markers,
prior coronary evaluation and treatment, functional capacity, and
surgery-specific risk has been outlined. A framework for determining which patients are candidates for cardiac testing is presented in
algorithmic form. Successful use of the algorithm requires an appreciation of the different levels of risk attributable to certain clinical
circumstances, levels of functional capacity, and types of surgery.
The major clinical predictors of increased perioperative cardiovascular risk is a recent unstable coronary syndrome (such as an
acute MI [documented MI less than 7 days previously], recent MI
[more than 7 days but less than 1 month before surgery], unstable
or severe angina, and evidence of a large ischemic burden by clinical symptoms or noninvasive testing), decompensate heart failure,
significant arrhythmias (high-grade atrioventricular block, symptomatic arrhythmias in the presence of underlying heart disease,
or supraventricular arrhythmias with uncontrolled ventricular
rate), and severe valvular disease. (See Table 109.5.) Intermediate predictors of increased risk are mild angina pectoris, a more

PMPH_CH109.indd 862

remote prior MI (more than 1 month before planned surgery),


compensated heart failure, preoperative creatinine greater than
or equal to 2.0 mg/dL, and diabetes mellitus. Minor predictors of
risk are advanced age >65 years of age, abnormal ECG, rhythm
other than sinus, low functional capacity, history of stroke, and
uncontrolled systemic hypertension.
A history of MI or abnormal Q waves by ECG is listed as an
intermediate predictor, whereas an acute MI (defined as at least 1
documented MI less than or equal to 7 days before the examination)
or recent MI (more than 7 days but less than or equal to 1 month
before the examination) with evidence of important ischemic risk
by clinical symptoms or noninvasive study is a major predictor.
This definition reflects the consensus of the ACC Cardiovascular
Database Committee. In this way, the separation of MI into the traditional 3- and 6-month intervals has been avoided.6,16 If a recent
stress test does not indicate residual myocardium at risk, the likelihood of reinfarction after noncardiac surgery is low. Although
there are no adequate clinical trials on which to base firm recommendations, it appears reasonable to wait 4 to 6 weeks after MI to
perform elective surgery. Studies have shown that exercise tolerance is as good as exercise stress testing in predicting perioperative
complications in patients with stable coronary artery disease.

5/22/2012 6:13:50 PM

Preoperative Risk Factor Assessment

Answer: The guidelines for reducing cardiac complications


after noncardiac surgery include a number of patient-specific
factors obtained from the patients medical history and physical examination. Some of the cardiac complications include
MI, CHF, and arrhythmias. The clinical predictors of cardiac
risk are patients with a history of MI, CAD, CHF, arrhythmias, pacemakers, ICDs, and presence of anemia. Other factors
include functional capacity, age, and comorbid conditions such
as diabetes mellitus (DM), peripheral vascular disease (PVD),
renal dysfunction, and chronic pulmonary disease (Grade B
recommendation).
4. What are the management strategies for patients with high
preoperative risk factors?
a. Management of cardiac risk
b. Coronary angiography and revascularization
c. Medical management: -blockade
If preoperative evaluation suggests that a patient is at high risk
for cardiac complications from surgery, options include revascularization prior to surgery. Evidence from retrospective studies suggests that the reduction in risk from revascularization is
approximately equal to the risk from surgery itself. Therefore, the
decision to refer a patient for coronary revascularization in the
perioperative setting is identical to that in the nonoperative setting. It is not appropriate to propose surgery for coronary revascularization that would not otherwise have been indicated. Patients
undergoing elective noncardiac procedures who are found to have
prognostic high-risk coronary disease and in whom long-term
outcome would likely be improved by coronary artery bypass
graft (CABG)18 should generally undergo revascularization before
a noncardiac elective surgical procedure of high or intermediate
risk. Data to support percutaneous transluminal coronary angioplasty (PTCA) are also lacking. There are no controlled trials
comparing perioperative cardiac outcome after noncardiac surgery for patients treated with preoperative PTCA versus medical
therapy. Several small observational series have suggested that
cardiac death is infrequent in patients who have undergone PTCA
before noncardiac surgery.18-22

863

Several studies have also demonstrated a number of complications from angioplasty, including emergency CABG in some
patients. There is uncertainty regarding how much time should
pass between PTCA and noncardiac procedures. Delaying surgery
for at least 1 week after balloon angioplasty to allow for healing of
the vessel injury has theoretical benefits. If a coronary stent is used,
a delay of at least 2 weeks and ideally 4 to 6 weeks should occur
before noncardiac surgery to allow 4 full weeks of dual antiplatelet therapy and re-endothelialization of the stent to be completed,
or nearly so.23 There is some suggestion that patients undergoing
noncardiac surgery soon after PTCA are at higher risk for cardiac
complications, and elective surgery should generally be delayed
for at least 2 weeks after PTCA.
Since 1996, perioperative -blockade has become the mainstay of medical therapy for noncardiac surgical patients. Two
randomized, placebo-controlled trials of -blocker administration have been performed.13,14,24,25 One trial demonstrated
reduced perioperative cardiac events, and the other demonstrated
improved 6-month survival with perioperative -blocker usage.
Current studies, however, suggest that appropriately administered
-blockers reduce perioperative ischemia and may reduce the risk
of MI and death in high-risk patients. When possible, -blockers
should be started days or weeks before elective surgery, with the
dose titrated to achieve a resting heart rate between 50 and 60
beats per minute. Perioperative treatment with -2 agonists may
have similar effects on myocardial ischemia, infarction, and cardiac death. Further research is needed in this area. Although the
optimal dosing schedule for -blockers is unknown, it is recommended that -blockers should be started before hospitalization
or immediately upon hospitalization. Importantly, perioperative
-blockers should be continued through the hospitalization and
up to a month postoperatively (Table 109.6).
Answer: When assessing a patient for cardiac risk from surgery
it is important to evaluate the need for surgery versus attempting to revascularize the patient prior to surgery. Evidence suggests
that the reduction in risk from revascularization in the preoperative setting is identical to that in the nonoperative setting. A
patient should only undergo a CABG if the long-term outcome
will be improved. Current studies suggest that -blockers reduce

Table 109.6 Recommendations for Perioperative Medical Therapy


Class I
1. -Blockers required in the recent past to control symptoms of angina or patients with symptomatic arrhythmias or hypertension.
2. -Blockers: patients at high cardiac risk owing to the finding of ischemia on preoperative testing who are undergoing
vascular surgery.

Class IIa
l. -Blockers: preoperative assessment identifies untreated hypertension, known coronary disease, or major risk factors
for coronary disease.

Class IIb
1. -2 Agonist: perioperative control of hypertension, or known CAD or major risk factors for CAD.

Class III
1. -Blockers: contraindication to -blockade.
2. -2 Agonists: contraindication to -2 agonists.

PMPH_CH109.indd 863

5/22/2012 6:13:50 PM

864

Surgery: Evidence-Based Practice

perioperative ischemia and may reduce the risk of MI and death


in high-risk patients (Grade B recommendation).
5. What are the guidelines for reducing pulmonary complications after noncardiac surgery?
a. Pulmonary risk assessment
b. Procedure-specific risk factors
c. Patient-specific risk factors
Pulmonary function is altered in patients undergoing surgery.
Decreased functional residual capacity, productive coughs,
atelectasis are all frequent causes of operative morbidity. Pulmonary complications are defined as pneumonia, respiratory failure
with prolonged mechanical ventilation, bronchospasm, and exacerbation of chronic lung disease.
Preoperative assessment of patients undergoing pulmonary
resection is generally extensive. Patients with no history or symptoms of clinically significant lung disease and a normal lung
examination require no further studies. Routine preoperative
pulmonary function tests are not required, and routine preoperative chest x-ray has not shown to be helpful in improving patient
outcomes.26
Smoking increases the risk of pulmonary complications even
in the absence of chronic lung disease, and all patients undergoing
elective surgery should be counseled on smoking cessation and
improvement of perioperative risk. The risk declines if smoking is
stopped at least 8 weeks before surgery; one study actually showed
a higher risk among patients who stopped smoking for less than
8 weeks than those who never stopped. However, acute cessation
of smoking for 24 hours does reduce the quantity of methemoglobin and can improve oxygenation. Cessation between 24 hours
and 6 weeks is associated with an increased incidence of morbidity, presumably secondary to decreased mucociliary clearance.27
As a physician, every opportunity should be made to discuss the
importance of smoking cessation with all patients who smoke.
Patients with chronic obstructive pulmonary disease (COPD)
are complex. Determination of symptoms of wheezing is as
important as the patient with asthma. The onset of these symptoms may indicate the start of bronchitis or a pulmonary infection. For these patients, the surgery should be delayed until
wheezing has stopped. Symptoms of dyspnea and reduced exercise tolerance identify those who are at increased risk, whereas
the absence of such symptoms with moderate exercise is rarely
associated with advanced disease. Patients who were unable to
walk more than 100 yards, or breathless while talking, dressing, or unable to leave the house because of breathlessness had
a 53% incidence of complication compared with those having no
abnormal dyspnea.28

Patients who have a history of sleep apnea and/or who are


morbidly obese without systemic disease are acceptable for surgery, but hospitalization postoperatively is recommended. Sleep
apnea suggests intermittent airway obstruction and these patients
may have a difficult airway. A thorough evaluation is frequently
required to identify those patients in whom a fiberoptic intubation is necessary.
The management of patients with productive cough represents
a difficult dilemma. Retrospective series did not demonstrate an
increased risk of complications in patients with uncomplicated respiratory infections, whereas other studies did show a significant difference, especially if an endotracheal intubation was involved.29-33
Perioperative interventions have been shown to reduce the
incidence of pulmonary complications. Patients who stop smoking 2 months prior to surgery have significantly fewer pulmonary complications than those who continue to smoke or stop
less than 8 weeks before admission. Incentive spirometry and
chest physiotherapy have been shown to reduce pulmonary morbidity. Adequate analgesia and early mobilization are strongly
recommended.
Answer: Pulmonary function is altered in patients undergoing surgery. Decreased functional residual capacity, productive
coughs, atelectasis all are frequent causes of operative morbidity.
When assessing the pulmonary risk in patients, it is important to
obtain a complete pulmonary history, such as history of asthma,
wheezing, COPD, emphysema, and smoking. Patients undergoing
pulmonary resection with no history or symptoms of lung disease
do not need any further workup (Grade B recommendation).

CONCLUSION
In conclusion, it is evident that a thorough preoperative evaluation
of each and every patient is imperative. Good communication is
also an essential feature of preoperative evaluation. Findings and
recommendations should always be discussed with the referring
surgeon, ideally in person. Notes should be brief, focused, and specific. The goal of preoperative risk assessment is to determine if a
patient is at average or increased risk for a specific procedure, or to
recommend diagnostic testing if this determination cannot yet be
made. As no patient is clear of risk, the phrase medical clearance
is misleading and should not be used by a medical consultant.
The patient should understand that their surgeon, primary care
doctor, and anesthesiologist are all working together as a team to
optimize their care and that the final decision on whether or not
to operate will be made by the surgeon. Again, the most important
part of the evaluation will be a complete physical examination of
the patient.

Clinical Question Summary


Questions

Answers

Grade References

1 What are the major factors


that determine the
preoperative risk of the
surgical patient?

The two major factors are type of surgery and the type of patient
undergoing the surgery.

7-11, 14

2 What type of surgery has


the greatest risk factor?

The type of surgery with the greatest risk is emergency surgery of any kind.

5-8
(Continued)

PMPH_CH109.indd 864

5/22/2012 6:13:50 PM

Preoperative Risk Factor Assessment

865

(Continued)
3 What are the guidelines
for reducing cardiac
complications of
noncardiac surgery?

The guidelines for reducing cardiac complications after noncardiac


surgery include a number of patient-specific factors obtained from the
patients medical history and physical examination. Some of the cardiac
complications include MI, CHF, and arrhythmias. The clinical predictors
of cardiac risk in a patient are a history of MI, CAD, CHF, arrhythmias,
pacemakers, ICDs, and presence of anemia. Other factors include
functional capacity, age, and comorbid conditions such as DM, PVD, renal
dysfunction, and chronic pulmonary disease.

9-14, 17

4 What are the management


strategies for patients
with high preoperative
risk factors?

When assessing a patient for cardiac risk from surgery, it is important to


evaluate the need for surgery versus attempting to revascularize the
patient prior to surgery. Evidence suggests that the reduction in risk from
revascularization in the preoperative setting is identical to that in the
nonoperative setting. A patient should only undergo a CABG if the longterm outcome will be improved. Current studies suggest that -blockers
reduce perioperative ischemia and may reduce the risk of MI and death in
high-risk patients.

1-10, 23-25

5 What are the guidelines


for reducing pulmonary
complications after
noncardiac surgery?

Pulmonary function is altered in patients undergoing surgery. Decreased


functional residual capacity, productive coughs, and atelectasis are all
frequent causes of operative morbidity. When assessing the pulmonary
risk in patients, it is important to obtain a complete pulmonary history
such as history of asthma, wheezing, COPD, emphysema, and smoking.
Patients undergoing pulmonary resection with no history or symptoms of
lung disease do not need further workup.

16, 27-34

REFERENCES
1. Eagle KA, Brundage BH, Chaitman BR, et al. Guidelines for
perioperative cardiovascular evaluation for noncardiac surgery.
Report of the American College of Cardiology/American Heart
Association Task Force on Practice Guidelines. Committee on
Perioperative Cardiovascular Evaluation for Noncardiac Surgery. Circulation. 1996;93:1278-1317.
2. Hogue CW, Jr., Goodnough LT, Monk TG. Perioperative myocardial ischemic episodes are related to hematocrit level in patients
undergoing radical prostatectomy. Transfusion. 1998;38:924-931.
3. Hahn RG, Nilsson A, Farahmand BY, Persson PG. Blood haemoglobin and the long-term incidence of acute myocardial
infarction after transurethral resection of the prostate. Eur Urol.
1997;31:199-203.
4. Nelson AH, Fleisher LA, Rosenbaum SH. Relationship between
postoperative anemia and cardiac morbidity in high-risk vascular patients in the intensive care unit. Crit Care Med. 1993;21:
860-866.
5. Lee TH, Marcantonio ER, Mangione CM, et al. Derivation and
prospective validation of a simple index for prediction of cardiac
risk of major noncardiac surgery. Circulation. 1999;100:1043-1049.
6. Goldman L, Caldera DL, Nussbaum SR, et al. Multifactorial
index of cardiac risk in noncardiac surgical procedures. N Engl J
Med. 1977;297:845-850.
7. Ashton CM, Petersen NJ, Wray NP, et al. The incidence of perioperative myocardial infarction in men undergoing noncardiac
surgery. Ann Intern Med. 1993;118:504-510.
8. Cooperman M, Pflug B, Martin EW, Jr., Evans WE. Cardiovascular risk factors in patients with peripheral vascular disease.
Surgery. 1978;84:505-509.
9. Detsky AS, Abrams HB, McLaughlin JR, et al. Predicting cardiac
complications in patients undergoing non-cardiac surgery. J Gen
Intern Med. 1986;1:211-219.

PMPH_CH109.indd 865

10. Lette J, Waters D, Bernier H, et al. Preoperative and long-term


cardiac risk assessment. Predictive value of 23 clinical descriptors, 7 multivariate scoring systems, and quantitative dipyridamole imaging in 360 patients. Ann Surg. 1992;216:192-204.
11. Michel LA, Jamart J, Bradpiece HA, Malt RA. Prediction of risk
in noncardiac operations after cardiac operations. J Thorac Cardiovasc Surg. 1990;100:595-605.
12. Eagle KA, Coley CM, Newell JB, et al. Combining clinical and
thallium data optimizes preoperative assessment of cardiac
risk before major vascular surgery. Ann Intern Med. 1989;110:
859-866.
13. Poldermans D, Boersma E, Bax JJ, et al. The effect of bisoprolol on
perioperative mortality and myocardial infarction in high-risk
patients undergoing vascular surgery. Dutch Echocardiographic
Cardiac Risk Evaluation Applying Stress Echocardiography
Study Group. N Engl J Med. 1999;341:1789-1794.
14. Boersma E, Poldermans D, Bax JJ, et al. Predictors of cardiac
events after major vascular surgery. Role of clinical characteristics, dobutamine echocardiography, and beta-blocker therapy.
JAMA. 2001;285:1865-1873.
15. Reilly DF, McNeely MJ, Doerner D, et al. Self-reported exercise
tolerance and the risk of serious perioperative complications.
Arch Intern Med. 1999;159:2185-2192.
16. Older P, Hall A, Hader R. Cardiopulmonary exercise testing as a
screening test for perioperative management of major surgery in
the elderly. Chest. 1999;116:355-362.
17. Bartels C, Bechtel JF, Hossmann V, Horsch S. Cardiac risk stratification for high-risk vascular surgery. Circulation. 1997;95:24732475.
18. Guidelines and indications for coronary artery bypass graft surgery.
A report of the American College of Cardiology/American Heart
Association Task Force on Assessment of Diagnostic and Therapeutic Cardiovascular Procedures (Subcommittee on Coronary Artery
Bypass Graft Surgery). J Am Coll Cardiol. 1991;17:543-589.

5/22/2012 6:13:50 PM

866

Surgery: Evidence-Based Practice

19. Huber KC, Evans MA, Bresnahan JF, Gibbons RJ, Holmes DR,
Jr. Outcome of noncardiac operations in patients with severe
coronary artery disease successfully treated preoperatively with
coronary angioplasty. Mayo Clin Proc. 1992;67:15-21.
20. Elmore JR, Hallett JW, Jr., Gibbons RJ, et al. Myocardial revascularization before abdominal aortic aneurysmorrhaphy: effect of
coronary angioplasty. Mayo Clin Proc. 1993;68:637-641.
21. Allen JR, Helling TS, Hartzler GO. Operative procedures not
involving the heart after percutaneous transluminal coronary
angioplasty. Surg Gynecol Obstet. 1991;173:285-288.
22. Gottlieb A, Banoub M, Sprung J, Levy PJ, Beven M, Mascha EJ.
Perioperative cardiovascular morbidity in patients with coronary artery disease undergoing vascular surgery after percutaneous transluminal coronary angioplasty. J Cardiothorac Vasc
Anesth. 1998;12:501-506.
23. Posner KL, Van Norman GA, Chan V. Adverse cardiac outcomes after noncardiac surgery in patients with prior percutaneous transluminal coronary angioplasty. Anesth Analg. 1999;89:
553-560.
24. Wallace A, Layug B, Tateo I, et al. Prophylactic atenolol reduces
postoperative myocardial ischemia. McSPI Research Group.
Anesthesiology. 1998;88:7-17.
25. Mangano DT, Layug EL, Wallace A, Tateo I. Effect of atenolol
on mortality and cardiovascular morbidity after noncardiac
surgery. Multicenter Study of Perioperative Ischemia Research
Group. N Engl J Med. 1996;335:1713-1720.

PMPH_CH109.indd 866

26. Cain HD, Stevens PM, Adaniya R. Preoperative pulmonary


function and complications after cardiovascular surgery. Chest.
1979;76:130-135.
27. Warner MA, Offord KP, Warner ME, Lennon RL, Conover MA,
Jansson-Schumacher U. Role of preoperative cessation of smoking and other factors in postoperative pulmonary complications:
a blinded prospective study of coronary artery bypass patients.
Mayo Clin Proc. 1989;64:609-616.
28. Wong DH, Weber EC, Schell MJ, et al. Factors associated with
postoperative pulmonary complications in patients with severe
chronic obstructive pulmonary disease. Anesth Analg. 1995;80:
276-284.
29. Stein M, Cassara EL. Preoperative pulmonary evaluation and
therapy for surgery patients. JAMA. 1970;211:787-790.
30. Pien LC, Grammer LC, Patterson R. Minimal complications in
a surgical population with severe asthma receiving prophylactic
corticosteroids. J Allergy Clin Immunol. 1988;82:696-700.
31. Schuurmans M, Diacon A, Bolliger C. Functional evaluation
before lung resection. Clin Chest Med. 2002;23:159-172.
32. Gerson MC, Hurst JM, Hertzberg VS, Baughman R, Rouan GW,
Ellis K. Prediction of cardiac and pulmonary complications
related to elective abdominal and noncardiac thoracic surgery in
geriatric patients. Am J Med. 1990;88:101-107.
33. Kocabas A, Kara K, Ozgur G, Sonmez H, Burgut R. Value of preoperative spirometry to predict postoperative pulmonary complications. Respir Med. 1996;90:25-33.

5/22/2012 6:13:50 PM

Commentary on
Preoperative Risk Factor Assessment
of the Surgical Patient
Elliott Bennett-Guerrero

QUESTIONS DISCUSSED IN
THE CHAPTER

poorly supported by both the literature as well as anecdotal observations. Although it is likely that emergency status (vs. elective)
confers higher risk for a specific type of surgery, for example, emergency abdominal aortic aneurysm (AAA) surgery versus elective
AAA surgery, emergency status in and of itself is not a predictor of
adverse outcome. For example, emergency eye surgery is not associated with a significant risk of mortality and morbidity.
Question #3 relates to guidelines for reducing cardiac complications in patients undergoing noncardiac surgery. This section
contains some useful information, for example, a description of the
importance of functional capacity and types of activity associated
with at least a desirable 4-MET demand indicative of lower cardiac
risk. The introduction by the authors of the term myocardium at
risk is very important since clinicians often confuse it (and equate
it) with terms such as coronary artery disease and prior MI. For
example, a patient may carry a diagnosis of coronary artery disease
but may have undergone successful coronary artery bypass graft
(CABG) 2 years ago and not have any myocardium at risk.
It would have been helpful if the authors had explicitly defined
what types of cardiac complications they were addressing and the
definitions for each, since cardiac complications are sometime
lumped together leaving the reader to guess or assume what this
refers to. For example, it would have been helpful to address the
following cardiac complications: (1) myocardial infarction (MI),
(2) myocardial ischemia, congestive heart failure, or pulmonary
edema, and (3) arrhythmias, in particular, atrial fibrillation.
We recognize that this topic is difficult as even MI is difficult
to define. For example, a case of major MI with associated ventricular arrhythmias and/or evidence of ventricular dysfunction
(by Echocardiography (ECHO) or clinical evidence of new heart
failure) is of clear significance to most clinicians. However, most
studies also include the much more common complication of
isolated myocardial necrosis as evidenced by increased cardiac
enzymes. Although this is associated with increased longer-term
adverse outcome, it is not clear as to what extent it should be considered a bona fide postoperative complication because it is usually
silent and is not associated with any overt signs or symptoms.
Question #4 relates to management strategies to reduce cardiac risk, which is of most relevance to patients and clinicians. The
topic of need (or lack thereof) for preoperative revascularization
(i.e., CABG or PTCA) prior to elective surgery is summarized. It
would have been helpful to have more discussion on the issue of
how long elective surgery should be delayed after stent insertion
and whether potent antiplatelet therapy, that is, clopidogrel,

1. What are the major factors that determine the preoperative


risk of the surgical patient?
2. What types of surgery have the greatest risk factors?
3. What are the guidelines for reducing cardiac complications of
noncardiac surgery?
a. Cardiac complications of noncardiac surgery
b. Evaluation of cardiac risk
c. Clinical predictors of cardiac risk
4. What are the management strategies for patients with high
preoperative risk factors?
a. Management of cardiac risk
b. Coronary angiography and revascularization
c. Medical management: -blockade
d. Pulmonary risk reduction
i. Preoperative
ii. Intraoperative
iii. Postoperative
5. What are the guidelines for pulmonary complications of
noncardiac surgery?
a. Pulmonary risk assessment
b. Procedure-specific risk factors
c. Patient-specific risk factors
The chapter entitled Preoperative Risk Factor Assessment of the
Surgical Patient by E.J. Pierre, S.M. Cantie, and F. Huda addresses
risk factor assessment and guidelines to reduce perioperative cardiac and pulmonary complications. An important limitation of this
chapter is that there is no discussion of a large number of other perioperative complications including acute renal dysfunction/failure,
coagulopathy/bleeding, neurological complications (stroke, delirium, cognitive dysfunction), postoperative ileus, nausea and vomiting, starvation, surgical site infection, and deep venous thrombosis/
pulmonary embolism. Nevertheless, let us look at the authors focus
on the cardiac and pulmonary systems.
The first question addressed by the authors relates to risk factors that determine preoperative risk of complications. I agree
with their broad statement that this risk is generally determined
by the type of surgery and the health, that is, extent of comorbidities, of the patient. The authors response to Question #2 is somewhat problematic as it concludes that The type of surgery with the
greatest risk is emergency surgery of any kind. This statement is
867

PMPH_CH109.indd 867

5/22/2012 6:13:50 PM

868

Surgery: Evidence-Based Practice

should be discontinued prior to surgery. As summarized recently


in a joint position paper on this issue,1 there is growing evidence
that in many patients elective surgery should be delayed for a prolonged period of time; if necessary, antiplatelet therapy should
not be discontinued preoperatively in some patients even if this
increases surgical blood loss.
The authors review of -blockers did not include the 2008
POISE trial publication,2 which involved randomization of 8351
noncardiac surgical patients with (or with risk for) atherosclerotic
disease. Although there was a small reduction in the primary
composite endpoint of cardiac complications (5.8% metoprolol vs.
6.9% placebo, P = .0399), there were more deaths (3.1% vs. 2.3%,
P = .0317) and more strokes (1% vs. 0.5%, P = .0053) in metoprolol
treated patients. Authors of this landmark study concluded that their
results highlight the risk in assuming a perioperative beta-blocker
regimen has benefit without substantial harm, and the importance
and need for large randomised trials in the perioperative setting.
Therefore, the issue of -blockade is still unresolved with many
important questions including unknown risk/benefit profile, and
uncertainty over the optimal dosing regimen, for example, low dose
versus titration of -blocker to achieve low heart rate.
The authors then focus on guidelines to reduce pulmonary
complications after noncardiac surgery. They include pneumonia,
prolonged mechanical ventilation, bronchospasm, and exacerbation of chronic lung disease. The authors summarize issues related
to smoking cessation and the important role clinicians have in
encouraging patients to stop smoking. Regarding the risk of bronchospasm, the authors correctly state that elective surgery should
be postponed in a patient who has significant wheezing that is
worse than their optimal status. However, there is no discussion

PMPH_CH109.indd 868

of the potential role of preoperative administration of steroids


to optimize high-risk patients, for example, those with previous
ICU admission or tracheal intubation for bronchospasm. This and
related issues are summarized nicely by Licker et al.3
I agree with the authors concluding remarks, in particular,
the importance of good communication between different care
teams (e.g., anesthesia, surgery, cardiology). In addition, the concept of medical clearance is unclear and this term should be
avoided. I agree that the focus needs to be on whether a patient has
any medical issues for which further diagnostic testing or interventions would reduce the risk of perioperative complications.

REFERENCES
1. Korte W, Cattaneo M, Chassot PG, et al. Peri-operative management of antiplatelet therapy in patients with coronary artery
disease. Joint position paper by members of the working group
on Perioperative Haemostasis of the Society on Thrombosis and
Haemostasis Research (GTH), the working group on Perioperative Coagulation of the Austrian Society for Anesthesiology,
Resuscitation and Intensive Care (OGARI) and the Working
Group Thrombosis of the European Society for Cardiology (ESC).
Thromb Haemost. 2011;105:743-749.
2. Devereaux PJ, Yang H, Yusuf S, et al. Effects of extended-release
metoprolol succinate in patients undergoing non-cardiac surgery (POISE trial): a randomised controlled trial. Lancet. 2008;
371:1839-1847.
3. Licker M, Schweizer A, Ellenberger C, Tschopp JM, Diaper J, Clergue
F. Perioperative medical management of patients with COPD. Int
J Chron Obstruct Pulmon Dis. 2007;2:493-515.

5/22/2012 6:13:50 PM

CHAPTER 110

Perioperative Cardiac Monitoring


Steven G. Venticinque

INTRODUCTION

instantaneous blood pressure, the effect of positive pressure ventilation on the pulse pressure, and the central venous pressure
(CVP). In addition, several minimally invasive monitors of
cardiac output have become available to help guide resuscitative
efforts.5 In particular, arterial pulse contour analysis is an easily
applicable technology that can yield cardiac output information
from an existing radial arterial line. The purpose of this review is to
summarize the performance of the CVP and respiratory-induced
changes in the arterial waveform to determine preload and intravascular volume responsiveness. It will also review the agreement
of arterial pulse contour cardiac output devices with the standard
clinical reference, the pulmonary artery catheter (PAC).

There are numerous causes of intravascular volume depletion


surrounding surgery, including preoperative volume depletion,
bleeding, ongoing catabolic fluid requirements, and increased
microvascular permeability mediated by inflammation and surgical stress response.1,2 The increase in capillary permeability, which
is proportional to the amount of surgical stress, can not only cause
intravascular hypovolemia, but can also contribute to an expansion of the interstitial space.3 In addition, the vasodilating effects of
anesthetic agents can cause a relative loss of intravascular volume,
further contributing to the effective volume deficit. The ensuing
reduction in cardiac preload can lead to hypotension, deceased cardiac output, and inadequate oxygen delivery leading to end-organ
insult which, in turn, may further propagate inflammation.
In order to reverse perioperative hypovolemia, surgical
patients are administered parenteral crystalloid and/or colloid
solutions. Although there is ongoing debate surrounding liberal versus restrictive fluid resuscitation strategies surrounding surgery, it is implicit that an effective circulating blood volume
is restored.4 However, the dynamic nature of the intravascular
volume losses, particularly ongoing losses due to capillary permeability, can skew predictions of fluid requirements, leading to
significant volume and sodium gains. In fact, all of the factors
contributing to perioperative hypovolemia can confound estimates of need leading to indiscriminate fluid administration.
Since these patients have avidity for fluid and sodium retention,
they also become predisposed to hypervolemia due to aggressive
volume resuscitation.3 Excessive fluid therapy in this setting may
have deleterious effects, including an adverse impact on outcome.
Recognizing this important balance, perioperative physicians are increasingly focusing on administering fluids in a
goal or endpoint directed manner, particularly when large fluid
shifts are anticipated. Since standard monitoring such as blood
pressure, heart rate, and urine output can be unreliable in this
regard, many surgical patients also receive arterial lines and central venous catheters. These devices may yield potentially useful
data pertaining to the intravascular volume status such as the

1. Is CVP monitoring an effective measure of intravascular


volume or predictive of a response to intravascular volume
administration?
It has been a widely held belief that pressure in the central venous
circulation, namely the intrathoracic venae cavae or right atrium, is
proportional to the intravascular volume status of the patient. This
follows from classic teachings in physiology regarding the coupling
between the heart and vascular system that correlates cardiac output to central venous pressure (Pv), where Pv is expressed in mm
Hg.6 Although volume is recognized as the true preload variable,
the surrogate of vascular pressure remains commonly utilized
clinically. Although the validity of using CVP as an indicator of
intravascular volume has been increasingly questioned, it continues to be utilized and endorsed as a resuscitation parameter.7-9
If CVP is to be used as a measure of the intravascular volume status, it should hold that there is an association between
CVP and the circulating volume. Also, CVP should ideally be
predictive of responsiveness to volume expansion manifested by
an increase in cardiac output. Studies looking at either assertion
have not demonstrated this association. Shippy et al., measured
blood volume at various intervals in critically ill patients using
125
I-labeled human serum albumin and compared the measure
with several physiologic variables including CVP.10 The researchers found no correlation between CVP and blood volume,
869

PMPH_CH110.indd 869

5/22/2012 6:14:33 PM

870

Surgery: Evidence-Based Practice

reporting a correlation coefficient (r) of 0.10. Other groups have


reported similar fi ndings when comparing CVP to blood volume
measured by pulse-dye densitometry and dual indicator transcardiopulmonary dilution techniques, reporting r-values of 0.17
and 0.025, respectively.11,12
CVP is also not a good predictor of a positive cardiac output response to an intravascular volume challenge (e.g., volume
responsiveness). In a systematic review, Marik et al. reported on
19 studies that compared the CVP/CVP to the change in stroke
volume or cardiac index (CI) following a fluid challenge.13 In the
trials where the data was available (10 of 19), the r-value between
baseline CVP and change in stroke index/cardiac index was only
0.18 (95% confidence interval, 0.080.28). The reported pooled
area under the receiver operating curve (AUC) for the measure
was likewise very poor (0.56), grading the ability of CVP to predict volume responsiveness at just greater than chance. Likewise,
in numerous trials that compare indices such as CVP and pulmonary capillary wedge pressure (PCWP) to respiratory-induced
changes in the arterial waveform as measures of volume responsiveness, the predictive value of CVP is near chance.14-17
Despite the fact that CVP seems to perform poorly when
looked at as singular measure of preload, some studies that have
utilized CVP as one of the parameters in goal-oriented resuscitation strategy have demonstrated positive outcome. In a trial
involving patients with septic shock that compared standard
therapy to an immediate resuscitation protocol, which included a
CVP goal (812 mm Hg), researchers found that the protocol
group demonstrated a more rapid reversal of shock and improved
ICU and in-hospital mortality (50% vs. 67.2%, P = .009 and 53.7%
vs. 71.6%, P = .006, respectively).18 In a study assessing a goaldirected resuscitation strategy in 135 high-risk surgical patients,
patients assigned to a protocol group experienced fewer organ
failures and shorter hospital stays (27 failures vs. 9 failures,
P < .001, and 11.3 3.8 days vs. 13.4 6.1 days, P < .05, respectively).19
Although both groups in this trial were managed to achieve a CVP
goal of 8 to 12 mm Hg, the first prescribed intervention in the protocol group for not meeting the primary endpoint (an 02 extraction ratio estimate of <27%) was to increase CVP if it was less than
10 mm Hg. In the well-known trial by Rivers et al., researchers
randomized 263 patients with septic shock to either a nonprotocolized standard care (which included defined CVP, mean arterial
pressure, and urine output goals) or an intensive, goal-directed
resuscitation protocol conducted during a 6-hour period in the
emergency department.20 The first goal to achieve in the protocol
group was a CVP. In-hospital mortality was significantly reduced
in the protocol group when compared with the standard therapy
group (30.5% vs. 46.5%, P = .009, respectively). It was interesting to note that within the first 6 hours of therapy, the protocol
group had higher average CVP (13.8 4.4 mm Hg vs. 11.8 6.8,
P = .007) and had received more fluid (4981 2984 mL vs. 3499
2438, P .001). Although the results of these trials do not directly
support CVP as resuscitative endpoint, the study designs do suggest its utility. When this is combined with CVPs historic role in
physiology and patient management, it is easy to understand why
it still factors into resuscitative guidelines despite the controversy
surrounding its validity.7,21 Finally, in a recent small trial looking
at the predictive value of passive leg raise (PLR) induced changes
of cardiac output (CO) and pulse pressure (PP), a CVP with PLR
significantly increased the value of both PLR PP and PLRCO to
predict a response to an intravascular volume challenge (AUCs of
0.91 vs. 0.66 and 0.98 vs. 0.83; P < .05, respectively).22 The fact that

PMPH_CH110.indd 870

CVP increased the predictive value of the other indices further


suggests a supportive role of the parameter.
There are several factors that can affect accurate measurement
of CVP including the effect of respiration, transducer zeroing, and
the location of the transducer relative to the reference level.23,24
Likewise, the presence of positive end expiratory pressure (PEEP),
lung disease, cardiac tamponade, restrictive cardiac disease, rightsided cardiac dysfunction, tricuspid valvular disease, and pulmonary hypertension can all dim hope that the absolute value of
right-sided venous pressure will correlate to left ventricular volume
and or fluid responsiveness.23,24
Answer: As an isolated parameter, CVP measure performs
poorly as a measure of preload and intravascular volume responsiveness and should not be used in isolation in order to guide
fluid therapy (Grade B recommendation). The CVP is vulnerable
to several measurement artifacts, and can be affected by several
pathologic conditions. The results of randomized controlled trials
suggest that CVP may serve in a supportive role as part of a multiparameter goal-directed resuscitation strategy. On this basis then,
CVP may have merit (Grade C recommendation).
2. Do pulse contour cardiac output devices provide acceptable
accuracy and precision when compared with thermodilution
techniques?
Several devices are available to the clinician for measuring cardiac output. The most widely accepted method is intermittent
thermodilution (ITD) via a pulmonary artery catheter (PAC).
Although the PAC can provide valid cardiac output information
and other data such as mixed venous oximetry and static pressure
parameters, the practical aspects of insertion, its invasiveness, and
the potential risks of placing and maintaining the device often
discourage its use. Furthermore, several randomized prospective
trials have not consistently demonstrated its use, and its use is
often under scrutiny.25 These concerns, combined with increasing
evidence that resuscitation to defined hemodynamic goals may be
associated with improved outcome, have popularized several socalled minimally invasive methods and devices for the measurement of stroke volume and cardiac output. These methods include
arterial pulse contour (APCO) analysis, transesophageal aortic
Doppler, partial carbon dioxide rebreathing, and thoracic electrical bioimpedance.
Of these technologies, cardiac output by APCO is becoming
increasingly popular. Several reasons probably account for this,
but primarily it is because they are simple to apply, the sensor
that interfaces with the patient (the arterial line) is very familiar
to physicians, and they can be used easily in patients who are
not intubated (universally applicable). The devices work based
on the fundamental principle that the magnitude of the pulse
pressure is proportional to the stroke volume.26 Cardiac output
is the product of stroke volume and heart rate. The seemingly
simple relationship between arterial pulse area and stroke volume is complicated by several factors including the compliance
of the arterial tree, the vascular resistance, and the overall complexity of the pulse wave.26 The unique aspects of analysis and
factoring of these variables in actually converting the pulse wave
into a stroke volume measurement is what differentiates these
devices among manufacturers.
For the purposes of this review, the majority of the devices
included were the LiDCO (LiDCO Plus; LiDCO, Cambridge, UK)
and the FloTrac/Vigileo (Edwards Lifesciences, Irvine, CA) that

5/22/2012 6:14:33 PM

Perioperative Cardiac Monitoring

can be used with a peripheral arterial catheter. The FloTrac/Vigileo and the LiDCO systems both foundationally rely on the proportionality of the area under the pulse pressure wave to stroke
volume in their calculation of cardiac output. The FloTrac/Vigileo
device requires patient biometric information in order to factor
in an estimate of vascular compliance. It also assesses waveform
shape variables (kurtosis and skewness) in order to correct for
changes in vascular tone.27-29 The algorithm then provides measure
of stroke volume that does not require any external calibration.
The LiDCO device uses two proprietary systems in order to derive
cardiac output. It incorporates a pulse pressure analysis algorithm
(PulseCO) that is calibrated by a single injection lithium indicator
dilution calibration system (LiDCO). The device first calculates
a nominal value for cardiac output based on an analysis of the
pressure waveform, which is then multiplied by correction factor
derived from the lithium indicator dilution cardiac output.30
The standard approach to validating the performance of the
minimally invasive cardiac output devices, regardless of the technology, is to compare their performance to that of ITD, which is
generally felt to represent the bed-side gold standard for cardiac output measurement.31 The problem with this is that neither
ITD, nor continuous thermal cardiac output (CCO), provides
unequivocally correct measurement. Since comparisons to ITD
(and sometimes CCO) is the approach taken by most researchers
during validation studies, they are essentially comparing devices
that are measuring the same clinical variable, each with an inherent degree of inaccuracy. Since classic correlation and regression
analysis are deficient in this circumstance, researchers typically
apply Bland-Altman analysis in order to assess agreement between
the measurement methods.32,33
Numerous clinical trials and two meta-analyses have been
performed in order to validate the accuracy and precision of
APCO against the common standard, the PAC, or against transpulmonary thermodilution (TPTD) conducted via a central
venous catheter and a femoral arterial line. Peyton et al. through
meta-analysis reviewed the agreement of four separate minimally
invasive cardiac output techniques, including pulse contour, with
the PAC or TPTD.34 Statistical criteria for inclusion in the metaanalysis required that single, independent measurements from
each subject could be identified, allowing a pooling of study data.
Twenty-four pulse contour studies met the criteria for this pooled,
weighted meta-analysis. Overall, pulse contour methods had a
combined bias of 0.00 (+/ 0.09) and a percentage of error of
41.3 (+/ 2.7)%. The researchers concluded that none of the four
methods of minimally invasive cardiac output measurement that
they evaluated, including pulse contour, achieved agreement with
bolus thermodilution within the expected 30% error limit. In
another meta-analysis looking specifically at the performance of
the FloTrac/Vigileo device, the researchers found a bias, precision,
and % error of 0.32, 1.16, and 44%, respectively.35 However, when
looking specifically at studies that utilized devices with the more
recent soft ware version of the device (1.10), the values improved to
0.15, 0.67, and 30%, which suggests the performance of this technology capable of being improved. Data for studies not included
in these systematic reviews are presented in Table 110.1. It can
be seen that level of agreement between the APCO devices and
the PAC is highly variable among studies. Although the accuracy
of the devices (bias) is often acceptable, their precision (limits of
agreement [LOA]) is the factor that most often leads to an unfavorable assessment. For example, in the study conducted by Jeong,
the researchers found a low bias of 0.23 L/min. However, based on

PMPH_CH110.indd 871

871

their reported LOA, there was a 95% chance that the actual cardiac
output would be between 2.5 to +2.5 L/min of the reported value.39
Since the mean cardiac output in the population studied was
approximately 4 L/min, the reported degree of imprecision would
be clinically unacceptable by any standard. Knowing the mean
value of CO (or CI) is in a trial of this type when the LOA are
reported. The mean serves to put the degree of imprecision into
context. Unfortunately, several of the studies do not report a
mean CO or CI. The magnitude of imprecision can also be presented as the percentage error, which is also not always available
to the reader. A maximum percentage of error of approximately
30% has been suggested as the maximal limit of acceptability as
it pertains to devices that measure cardiac output.32 This is based
on the premise that the limit of precision of physiologic measurement in this regard is approximately +/ 20%, which is about the
percentage of error of the PAC ITD reference method. Combining the inherent errors of the reference method and a presumably valid cardiac output device with similar precision predicts an
error of approximately 30%.32
Although the measurement accuracy of a minimally invasive
cardiac output device is important, so too is its ability to track and
trend changes. Very few CO device validation studies address this
issue; in a recent review on the topic, the authors state that the
ability of the pulse contour devices to trend cardiac output is inferior to that of the Doppler-based devices.46 In a study evaluating
the performance of several minimally invasive cardiac output
techniques, the FloTrac/Vigileo demonstrated better agreement
with the reference method over esophageal Doppler in measuring
CO.47 However, in comparison to the Doppler-based device, the
pulse contour device overestimated changes in cardiac output. It
is possible that pulse contour analysis devices are vulnerable to
variations in vascular resistance. This is suggested by findings in
validation studies conducted during hepatic surgery or transplantation and in patients with sepsis where there is a trend toward
worse agreement as can be seen in Table 110.1.
Answer: Overall, pulse contour cardiac output devices demonstrate moderate performance when compared with thermodilution
and should not be used in isolation in order to guide resuscitative
therapy (Grade A recommendation). The results of numerous trials yield mixed results. Methodologic differences make pooling
these conclusions difficult. However, formal meta-analysis suggest
that APCO devices have poor agreement with reference methods.
APCO seems to be particularly challenged during hyperdynamic
conditions such as states of decreased systemic vascular resistance
(SVR) seen in sepsis and hepatic dysfunction (Grade B recommendation). The ability of these monitors to accurately track CO
changes has not been thoroughly evaluated. Evidence of enhanced
performance following software upgrade suggests that this technology is continuing to evolve and improve.
3. Are the dynamic indices good predictors of intravascular
volume responsiveness?
There are numerous resuscitative endpoints that can be utilized
in the perioperative setting including heart rate, blood pressure,
mental status, capillary refi ll, urine output, cardiac output, venous
oxygen concentration, lactate levels, and so on. When these endpoints are not being met, the question that always arises, in some
form, is will the patient respond in a positive manner to a fluid
bolus? The hope is that administering fluid will result in a significant increase in cardiac output by increasing the stroke vol-

5/22/2012 6:14:33 PM

872

Surgery: Evidence-Based Practice

Table 110.1 Studies Evaluating the Accuracy and Precision of FloTrac and LiDCO Against the Pulmonary Artery
Catheter That Are Not Included in Recent Meta-Analysis
Author

Year

Site

Device

Reference

CO/CI

Bias

Limits of
Agreement

% Error

Authors
Conclusion

McCoy
et al.36

2009

ICU CV

LiDCO

PAC (CCO)

CI

0.01 L/min/
m2

+/ 1.3 L/min/
m2

NA

Hadian
et al.37

2010

ICU CV

FloTrac
LiDCO

PAC (ITD/
CCO)
PAC (ITD/
CCO)

CO
CO

0.43 L/min
0.18 L/min

+/ 3.37 L/min
+/ 1.56 L/min

NA
NA

NR
NR

Hofer
et al. 38

2010

ICU &
OR
CV

FloTrac

PAC (ITD)

CO

0.2 +/ 2.1
L/min

NR

NA

NR

Jeong
et al.39

2010

OR CV

FloTrac

PAC (CCO)

CO

0.23 L/min

2.5 to 2.0 L/
min

57

Matthieu
et al.40

2008

OR Liver
TX

FloTrac

PAC (CCO)

CO

0.8 L/min

1.8 to 3.5 L/
min

43

Della
Rocca
et al.41

2008

ICU Liver
TX

FloTrac

PAC (ITD)

CO

0.95 L/min

1.88 to 3.77
L/min

25.6

Cecconi
et al.42

2010

ICU
Mixed

FloTrac
LiDCO

PAC (ITD)
PAC (ITD)

CO
CO

1.1 L/min
0.5 L/min

NR

55
40

Krejci
et al.43

2010

OR Liver
TX

FloTrac
LiDCO

PAC (ITD)

CI

1.78 L/min/
m2
0.99 L/min/
m2

0.99 to 4.56
L/min/m2
2.35 to 4.33
L/min/m2

68.5

75.6

Slagt
et al. 44

2010

ICU
Sepsis

FloTrac

PAC (ITD)

CO

1.2 L/min

3.5 to 1.0 L/
min

32

NR

Kim
et al.45

2006

Cath Lab
(peds)

LiDCO

PAC (ITD)

CO

0.19 L/min/
m2

0.09 to 0.47
L/min

NA

Authors conclusions about agreement: Acceptable, Moderate, Unacceptable. PAC, pulmonary artery catheter; CCO, continuous cardiac output;
ITD, intermittent thermodilution; CV, cardiac surgery; NR, not reported; NA, not available.

ume through augmentation of the preload. The common tools for


which the clinician has to determine the answer to the question
include the patients recent history (reasons for dehydration or
fluid losses, recent bleeding, etc.), the patients previous response
to previous fluid challenges, and the traditional static variables
that are thought to correspond to patients intravascular volume
status such as CVP and PCWP. In addition, portable ultrasonography can provide estimates of the intravascular volume status.48
It is now well recognized that fluid responsiveness can also
be assessed by quantifying the cyclic changes in left ventricular
stroke volume that are induced by positive pressure ventilation.49
This so-called dynamic variation in stroke volume (SVV) is also
manifest as systolic blood pressure variation (SPV) and pulse
pressure variation (PPV). These completely interrelated variables
all increase in direct proportion to the degree of the intravascular
volume deficit and/or the magnitude of ventilation. The mechanism that underlies this phenomenon involves a reduction in
vena caval blood flow during the inspiratory period secondary
to intrathoracic vena caval and right atrial compression.50 This
results in a transient decrease in right ventricular preload followed
by reduced pulmonary artery blood flow, and, following a lag of
a few heart beats, ultimately left ventricular preload and stroke

PMPH_CH110.indd 872

volume.51 The magnitude of this phenomenon is inversely proportional to the preload, and directly proportional to the positive
pressure tidal volume. As the preload decreases, the magnitude of
SVV, PPV, and SPV increases.
Studies that attempt to validate the hypothesis that SVV,
PPV, or SPV can predict fluid responsiveness have been conducted
similarly. They usually involve the recording of baseline parameters such as mean arterial pressure (MAP), heart rate (HR), CO
or CI, SVV/PPV/SPV, the static indices such as CVP and PCWP,
and possibly other indices of the intravascular volume status such
as left ventricular end-diastolic area (LVEDA) by transesophageal echocardiography. The patients are then administered an
intravascular fluid challenge and then subdivided into those who
responded to a fluid with an increase in CO or CI (usually 1215%)
and those who did not. Baseline and post-bolus in parameters are
then compared in order to determine the ability of the various
parameters to discriminate between patients who responded to
volume and those who did not. The data are typically presented in
the form of receiver operating curves, which allow an assessment
of the actual and relative abilities of each parameter to predict
the response to volume. Correlations and Bland-Altman analysis
comparing measures are also often presented.

5/22/2012 6:14:33 PM

Perioperative Cardiac Monitoring

In 2009, Marik et al. performed a systematic review of the literature and meta-analysis in order to determine the ability of the
respiratory-induced dynamic changes in the arterial waveform
to predict volume responsiveness relative to the traditional static
indices of fluid responsiveness.52 Studies included in their review
comprised those that reported correlation coefficients or receiver
operating characteristics between SPV, PPV, or SVV and a change
in stroke volume index (SVi) or CI following a fluid challenge.
Within the 29 studies that met their criteria, the pooled correlation
coefficients between baseline PPV, SVV, and SPV and the change
in SVi and/or CI were 0.78, 0.72, and 0.72, respectively. The area
under the receiver operating characteristic curves (AUROC) were
0.94, 0.84, and 0.86, respectively. As markers of volume responsiveness, CVP, left ventricular end-diastolic area index (LVEDAI) by
transesophageal echocardiography (TEE) and global end-diastolic
index by TPTD fared significantly worse, with AUROCs of 0.55,
0.56, and 0.64, in order. Recent studies have also demonstrated
similar results. Cannesson et al. studied the ability of SVV to detect
fluid responsiveness in mechanically ventilated patients during
cardiac surgery.17 The AUROC for SVV, PPV, CVP, and PCWP
were 0.871 +/ 0.085 for SVV, 0.857 +/ 0.084 for PPV, 0.533 +/
0.118 for CVP, and 0.338 +/ 0.126 for PCWP. A SVV of >10% in
this trial was satisfactory to discriminate responders from nonresponders. In a study conducted during hepatic transplantation,

873

Shin et al. was also able to demonstrate positive predictive ability


of the dynamic indices of volume responsiveness over static measures.53 Using fluids challenges of 10 mL/kg of 6% hydroxyethyl
starch solution, they noted AUROC of 0.894, 0.670, and 0.576 for
SVV, PCWP, and CVP, respectively. In their trial they found that
an SVV of >8% identified fluid responders with a sensitivity and
specificity of 89 and 80%.
An important characteristic regarding the patients in these
trials is that they are all intubated and on controlled modes of
ventilation. When researchers have looked at the ability of the
dynamic indices to predict volume responsiveness in spontaneously breathing patients, the performance of this index has been
poor.54,55 Another question that arises is, are the dynamic indices
useful during open chest conditions? The data is mixed on the
topic, but leans toward the conclusion that the dynamic indices
lose their predictive ability in this circumstance as well.56-60 Neither of these findings is surprising given the known physiology
associated with these indices.
Answer: The dynamic indices of SVV, PVV, and SPV are
excellent predictors of intravascular volume responsiveness in
intubated patients receiving controlled ventilation (Grade A recommendation). In spontaneously breathing patients and in situations where the chest is open, the predictive value of these indices
is poor (Grade B recommendation).

Clinical Question Summary


Questions

Answers

1 As an isolated measure, is
CVP an effective measure of
intravascular volume or of
volume responsiveness?

1. CVP correlates poorly to intravascular volume.


2. CVP is a poor predictor of intravascular volume responsiveness.
3. CVP monitoring may contribute in a positive manner as a parameter
within a multimodal resuscitation strategy.

B
B
C

10-12
14-17
18-20

2 Do the pulse contour cardiac


output devices provide
acceptable accuracy and
precision?

1. Overall, pulse contour cardiac output devices demonstrate moderate


performance when compared with thermodilution methods.

34-45

3 Are the dynamic indices good


predictors of a positive
cardiac output response
to intravascular volume
administration?

1. They are excellent predictors of intravascular volume responsiveness


in intubated patients receiving controlled ventilation.
2. They are poor predictors of intravascular volume responsiveness in
spontaneously breathing patients and during open chest conditions.

17, 52, 53

54-60

REFERENCES
1. Lobo DN, Macafee DA, Allison SP. How perioperative fluid
balance influences postoperative outcomes. Best Practice. 2006;
20(3):439-455.
2. Rosenthal MH. Intraoperative fluid managementwhat and
how much? Chest. 1999;115(5):106S-1012S.
3. Holte K. Pathophysiology and clinical implications of perioperative fluid management in elective surgery. Dan Med Bull. 2010
07;57(7).
4. Shields CJ. Towards a new standard of perioperative fluid management. Ther Clin Risk Manag. 2008;4(2):569-571.
5. Funk DJ, Moretti EW, Gan TJ. Minimally invasive cardiac
output monitoring in the perioperative setting. Anesth Analg.
2009;108(3):887-897.

PMPH_CH110.indd 873

Grade

References

6. Berne RM, Levy MN. Physiology. 2nd ed. St. Louis, MO: C.V.
Mosby; 1988.
7. Dellinger RP, Levy MM, Carlet JM, et al.Surviving sepsis campaign: International guidelines for management of severe sepsis
and septic shock: 2008. Crit Care Med. 2008;36(1):296-327.
8. Kastrup M, Markewitz A, Spies C, et al. Current practice of
hemodynamic monitoring and vasopressor and inotropic therapy in post-operative cardiac surgery patients in Germany:
Results from a postal survey. Acta Anaesthesiol Scand. 2007;51(3):
347-358.
9. Bilkovski RN, Rivers EP, Horst HM. Targeted resuscitation strategies after injury. Curr Opin Crit Care. 2004;10(6):529-538.
10. Shippy CR, Appel PL, Shoemaker WC. Reliability of clinical
monitoring to assess blood volume in critically ill patients. Crit
Care Med. 1984;12(2):107-112.

5/22/2012 6:14:33 PM

874

Surgery: Evidence-Based Practice

11. Oohashi S, Endoh H. Does central venous pressure or pulmonary


capillary wedge pressure reflect the status of circulating blood
volume in patients after extended transthoracic esophagectomy?
J Anesth. 2005;19(1):21-25.
12. Kntscher MV, Germann G, Hartmann B. Correlations between
cardiac output, stroke volume, central venous pressure, intraabdominal pressure and total circulating blood volume in resuscitation of major burns. Resuscitation. 2006;70(1):37-43.
13. Marik PE, Baram M, Vahid B. Does central venous pressure predict fluid responsiveness? A systematic review of the literature
and the tale of seven mares. Chest. 2008;134(1):172-178.
14. Kramer A, Zygun D, Hawes H, et al. Pulse pressure variation
predicts fluid responsiveness following coronary artery bypass
surgery. Chest. 2004;126(5):1563-1568.
15. Michard F, Boussat S, Chemla D, et al. Relation between respiratory changes in arterial pulse pressure and fluid responsiveness in
septic patients with acute circulatory failure. American journal of
respiratory and critical care medicine. 2000;162(1):134-138.
16. Hofer CK, Muller SM, Furrer L, et al. Stroke volume and pulse
pressure variation for prediction of fluid responsiveness in
patients undergoing off-pump coronary artery bypass grafting.
Chest. 2005;128(2):848-854.
17. Cannesson M, Musard H, Desebbe O, et al. The ability of stroke
volume variations obtained with Vigileo/FloTrac system to monitor fluid responsiveness in mechanically ventilated patients.
Anesth Analg. 2009;108(2):513-517.
18. Lin SM, Huang CD, Lin HC, et al. A modified goal-directed protocol improves clinical outcomes in intensive care unit patients
with septic shock: a randomized controlled trial. Shock (Augusta,
GA). 2006;26(6):551-557.
19. Donati A, Loggi S, Preiser J, et al. Goal-directed intraoperative
therapy reduces morbidity and length of hospital stay in highrisk surgical patients. Chest. 2007;132(6):1817-1824.
20. Rivers E, Nguyen B, Havstad S, et al. Early goal-directed therapy
in the treatment of severe sepsis and septic shock. N Engl J Med.
2001;345(19):1368-1377.
21. Carl M, Alms A, Braun J, et al. S3 guidelines for intensive care in
cardiac surgery patients: Hemodynamic monitoring and cardiocirculary system. GMS e-J. 2010;8.
22. Lakhal K, Ehrmann S, Runge I, et al. Central venous pressure
measurements improve the accuracy of leg raising-induced
change in pulse pressure to predict fluid responsiveness. Intensive Care Med. 2010;36(6):940-948.
23. Andritsos MJ, Park KW. Advantages and limitations of static
parameters of fluid loading. Int Anesthesiol Clin. 2010;48(1):1-21.
24. Magder S. Central venous pressure monitoring. Curr Opin Crit
Care. 2006;12(3):219-227.
25. Greenberg SB, Murphy GS, Vender JS. Current use of the pulmonary artery catheter. Curr Opin Crit Care. 2009;15(3):249-253.
26. Holmes TW, Williams DJ. Cardiac output measurement. Anaesth
Intensive Care Med. 2010;11(2):58-61.
27. Mayer J, Suttner S. Cardiac output derived from arterial pressure
waveform. Curr. Opin Anaesthesiol. 2009;22(6):804-808.
28. Hashim B, Lerner AB. The FloTrac systemmeasurement of
stroke volume and the assessment of dynamic fluid loading. Int
Anesthesiol Clin. 2010;48(1):45-56.
29. Cannesson M, Vallet B, Michard F. Pulse pressure variation and
stroke volume variation: From flying blind to flying right? Br J
Anaesth. 2009;103(6):896-897; author reply 897-899.
30. Sundar S, Panzica P. LiDCO systems. Int Anesthesiol Clin. 2010;
48(1):87-100.
31. Cecconi M, Grounds M, Rhodes A. Methodologies for assessing
agreement between two methods of clinical measurement: Are

PMPH_CH110.indd 874

32.

33.

34.

35.

36.

37.

38.

39.

40.

41.

42.

43.

44.

45.

46.

47.

48.

we as good as we think we are? Curr Opin Crit Care. 2007;13(3):


294-296.
Critchley LA, Critchley JA. A meta-analysis of studies using bias
and precision statistics to compare cardiac output measurement
techniques. J Clin Monit Comput. 1999;15(2):85-91.
Cecconi M, Rhodes A, Poloniecki J, et al. Bench-to-bedside
review: the importance of the precision of the reference technique
in method comparison studieswith specific reference to the
measurement of cardiac output. Crit Care. 2009;13:201.
Peyton PJ, Chong SW. Minimally invasive measurement of cardiac output during surgery and critical care: a meta-analysis of
accuracy and precision. Anesthesiology. 2010;113(5):1220-1235.
Mayer J, Boldt J, Poland R, et al. Continuous arterial pressure
waveform-based cardiac output using the FloTrac/Vigileo: a
review and meta-analysis. J Cardiothorac Vasc Anesth. 2009;23(3):
401-406.
McCoy JV, Hollenberg SM, Dellinger RP, et al. Continuous
cardiac index monitoring: a prospective observational study
of agreement between a pulmonary artery catheter and a calibrated minimally invasive technique. Resuscitation. 2009;80(8):
893-897.
Hadian M, Kim HK, Severyn DA, et al. Cross-comparison of
cardiac output trending accuracy of LiDCO, PiCCO, FloTrac
and pulmonary artery catheters. Crit Care. 2010;14(6):R212.
Hofer CK, Button D, Weibel L, et al. Uncalibrated radial and femoral arterial pressure waveform analysis for continuous cardiac
output measurement: An evaluation in cardiac surgery patients.
J Cardiothorac Vasc Anesth. 2010;24(2):257-264.
Jeong YB, Kim TH, Roh YJ, et al. Comparison of uncalibrated
arterial pressure waveform analysis with continuous thermodilution cardiac output measurements in patients undergoing elective off-pump coronary artery bypass surgery. J Cardiothorac
Vasc Anesth. 2010;24(5):767-771.
Matthieu B, Nouette-Gaulain k, Cottenceau V, et al. Cardiac output measurement in patients undergoing liver transplantation:
Pulmonary artery catheter versus uncalibrated arterial pressure
waveform analysis. Anesth Analg. 2008;106(5):1480.
Della Rocca G, Costa MG, Chiarandini P, et al. Arterial pulse cardiac output agreement with thermodilution in patients in hyperdynamic conditions. J Cardiothorac Vasc Anesth. 2008;22(5):
681-687.
Cecconi M, Dawson D, Casaretti R, et al. A prospective study of
the accuracy and precision of continuous cardiac output monitoring devices as compared to intermittent thermodilution. Minerva Anesthesiol. 2010;76(12):1010-1017.
Krejci V, Vannucci A, Abbas A, et al. uncalibrated arterial
pressure-based cardiac output monitors during orthotopic liver
transplantation. Liver Transpl. 2010;16(6):773-782.
Slagt C, Beute J, Hoeksema M, et al. Cardiac output derived from
arterial pressure waveform analysis without calibration vs. thermodilution in septic shock: evolving accuracy of soft ware versions. Eur J Anaesthesiol. 2010;27(6):550-554.
Kim JJ, Dreyer J, Chang AC, et al. Arterial pulse wave analysis:
An accurate means of determining cardiac output in children.
Pediatr Crit Care Med. 2006;7(6):532-535.
Critchley LA, Lee A, Ho AM. A critical review of the ability of
continuous cardiac output monitors to measure trends in cardiac output. Anesth Analg. 2010;111(5):1180-1192.
de Wilde RB, Geerts BF, Cui J, et al. Performance of three minimally invasive cardiac output monitoring systems. Anaesthesia.
2009;64(7):762-769.
Cowie BS. Focused transthoracic echocardiography in the perioperative period. Anaesth Intensive Care. 2010;38(5):823.

5/22/2012 6:14:33 PM

Perioperative Cardiac Monitoring

49. Michard F, Teboul JL. Using heart-lung interactions to assess


fluid responsiveness during mechanical ventilation. Crit Care.
2000;4(5):282-289.
50. Michard F. Changes in arterial pressure during mechanical ventilation. Anesthesiology. 2005;103(2):419-428; quiz 449-445.
51. Scharf SM, Brown R, Saunders N, et al. Hemodynamic effects of
positive-pressure inflation. J Appl Physiol. 1980;49(1):124-131.
52. Marik PE, Cavallazzi R, Vasu T, et al. Dynamic changes in arterial
waveform derived variables and fluid responsiveness in mechanically ventilated patients: a systematic review of the literature.
Crit Care Med. 2009;37(9):2642-2647.
53. Shin YH, Ko JS, Gwak MS, et al. Utility of uncalibrated femoral
stroke volume variation as a predictor of fluid responsiveness during the anhepatic phase of liver transplantation. Liver Transpl.
2011;17(1):53-59.
54. Soubrier S, Saulnier F, Hubert H, et al. Can dynamic indicators
help the prediction of fluid responsiveness in spontaneously
breathing critically ill patients? Intensive Care Med. 2007;33(7):
1117-1124.

PMPH_CH110.indd 875

875

55. Heenen S, De Backer D. Vincent JL. How can the response to volume expansion in patients with spontaneous respiratory movements be predicted? Crit Care. 2006;10(4):R102.
56. de Waal EE, Rex S, Kruitwagen CL, et al. Dynamic preload indicators fail to predict fluid responsiveness in open-chest conditions. Crit Care Med. 2009;37(2):510-515.
57. Reuter DA, Goepfert MS, Goresch T, et al. Assessing fluid responsiveness during open chest conditions. Br J Anaesth. 2005;
94(3):318-323.
58. Rex S, Schalte G, Schroth S, et al. Limitations of arterial pulse
pressure variation and left ventricular stroke volume variation
in estimating cardiac pre-load during open heart surgery. Acta
Anaesthesiol Scand. 2007;51(9):1258-1267.
59. Sander M, Spies CD, Berger K, et al. Prediction of volume
response under open-chest conditions during coronary artery
bypass surgery. Crit Care. 2007;11(6):R121.
60. Wyffels PAH, Sergeant P, Wouters PF. The value of pulse pressure
and stroke volume variation as predictors of fluid responsiveness
during open chest surgery. Anaesthesia. 2010;65(7):704-709.

5/22/2012 6:14:33 PM

CHAPTER 111

Bacteremia: An Evidence-Based
Review of Recommendations for
Elective General Surgery
Katherine Hetz, Kevin K. Chung, and Christopher E. White

is defined as sepsis-induced hypotension, defined as mean arterial pressure (MAP) <70 mm Hg, systolic blood pressure (SBP)
<90 mm Hg, or SBP decrease >40 mm Hg in the absence of other
causes of hypotension, that persists despite appropriate fluid
resuscitation. Severe septic shock refers to septic shock requiring
vasopressor infusion to maintain adequate blood pressure.4,10,11
The spectrum of infection to bacteremia to sepsis and on to
septic shock is of great concern to physicians as approximately
750,000 to 800,000 patients are admitted to hospitals in the United
States with one of these diagnoses every year.5,12,13 Because of the
increasing awareness and diagnosis of sepsis, the number of deaths
due to sepsis has increased, but mortality rates have decreased to
approximately 18% (from 28%) over a 20-year period.4,13,14 About
1.1 to 2.24 per 1000 people admitted to hospitals in the United
States are diagnosed as having sepsis.15,16 Out of every 1000 elective general surgery cases with a length of stay greater than 3 days,
11.6 patients are diagnosed with postoperative sepsis, accounting
for approximately one-third of the total number of patients diagnosed with sepsis.4 Approximately, 20% to 50% of total patients
diagnosed with sepsis die during hospitalization.7,12,15,16 In addition, it has been estimated that half of the patients diagnosed with
septic shock will die.17

Infection describes an inflammatory response to the invasion of


microorganisms into an area of the body that is usually sterile,1 for
example, the bloodstream. Presence of viable bacteria within the
bloodstream is defined as a bloodstream infection, or bacteremia 2,3
and has become a problem that is increasing in incidence in our
hospitals, and our surgical patients.4,5 Bacteremia increases the
cost, length of stay, and mortality.4,6-8 In fact, on average, in-hospital
length of stay for patients undergoing elective surgery who were
diagnosed with sepsis was three times greater than that of those
who were not diagnosed with sepsis (18 days vs. 3 days).4 Primary
bloodstream infections are those of unknown origin, whereas
secondary bloodstream infections are related to an infection at
another primary site, for example, the urinary tract or lungs.9
Viremia, fungemia, and parasitemia are defi ned as the presence
of viable viruses, fungi, and parasites, respectively, within the
bloodstream.10
Septicemia was classically defined as the presence of microorganisms or their toxins in the blood; however, this term has
fallen out of favor and is no longer commonly used.10 Instead,
the term sepsis is used, which refers to the systemic inflammatory response to an infection. The manifestations of sepsis are
defined in the Systemic Inflammatory Response Syndrome (SIRS)
criteria as follows: (1) body temperature less than 36Celsius or
greater than 38Celsius; (2) heart rate greater than 90 beats per
minute; (3) respiratory rate greater than 22 per minute or arterial
carbon dioxide less than 32 mm Hg; and (4) presence of greater
than 12,000 neutrophils/mm3, less than 4000 neutrophils/mm3,
or greater than 10% immature neutrophils.3,10 To qualify as being
septic, a patient must have an infection and at least two manifestations of the SIRS criteria. Many other clinical conditions can
result in SIRS, including trauma, burns, and pancreatitis; but these
do not qualify as sepsis since their root cause is not infection.10
Severe sepsis is associated with hypoperfusion, which is manifested by elevated lactate and/or organ dysfunction (i.e., oliguria, mental status changes, acute lung injury, etc.). Septic shock

1. What elective procedures have the highest incidence of


bacteremia/sepsis?
In a multicenter retrospective review performed by Vogel et al., 15
major classes of elective surgery were identified, and rates of postoperative sepsis as well as mortality were looked at. According
to this review, esophageal, pancreatic, gastric, and small-bowel
surgeries had the highest incidences of sepsis with hernia, thyroid, and breast surgeries having the lowest. Patients diagnosed
with sepsis had a significantly higher mortality rate (25.88%) than
those who were not diagnosed with sepsis (0.81%) in all surgical
groups. Mortality rates related to sepsis are highest in thoracic,
adrenal, and esophageal surgeries.4
876

PMPH_CH111.indd 876

5/22/2012 6:15:32 PM

Bacteremia

Many patients who undergo elective general surgery will have


central venous catheters placed, and these are the most common
cause of nosocomial infections.18 There are approximately 250,000
to 500,000 cases of bacteremia related to indwelling catheters in
the United States per year.19 Catheter-related infections are a
major cause of in-hospital morbidity and mortality for all patients,
including patients undergoing elective general surgery.18,19
Answer: The most frequent elective general surgery procedures
associated with bacteremia and/or sepsis are esophageal, pancreatic,
gastric, and small-bowel surgeries (Grade C recommendation).
2. What are the most common predispositions and comorbidities in patients who become bacteremic after elective general
surgery?
The risk of development of bacteremia is directly related to the
length of stay in an intensive care unit (ICU).2 Comorbidities
that are associated with higher mortality in patients who become
bacteremic include cancer, diabetes mellitus, and diseases of the
cardiovascular and respiratory systems.20 Anaerobic bacteremia
has an incidence of 1% to 17%; predisposing factors to this type of
infection include malignant neoplasms, organ transplants, hematologic disorders, intestinal obstruction, diabetes mellitus, splenectomy, recent gastrointestinal or obstetric gynecologic surgery,
the use of immunosuppressive drugs, or an undrained abscess.
Risk factors for increased mortality in anaerobic bacteremia
include presence of polymicrobial sepsis, compromised immune
status, malignancy, and advanced age.21 Vogels group has demonstrated that patients in different types of hospitals and different demographic groups undergoing elective general surgery have
varying rates of sepsis. Patients treated in hospitals that are larger,
in an urban setting, and in nonteaching hospitals have a higher
rate of sepsis compared with smaller, rural, and teaching hospitals. In addition, patients who are black or Hispanic are at higher
risk of sepsis when compared with white patients undergoing the
same procedure. Increasing age results in a linear increase in incidence of sepsis as well. Men have higher rates of sepsis for a given
type of surgery compared to women.4
Answer: Comorbidities associated with increased rates and
mortality in sepsis include an immunocompromised status (i.e.,
diabetes mellitus, cancer, organ transplant, splenectomy, or immunosuppressive drugs), diseases of the cardiovascular and respiratory systems, hematologic disorders, intestinal obstruction, and
advanced age (Grade C recommendation).
3. What are the optimal methods to diagnose bacteremia?
The gold standard for diagnosing bacteremia is the use of blood
cultures, which are typically obtained from a patient as part of
the so-called full-fever workup.7,9,22 These are typically obtained
after a patient as part of the so-called full-fever workup. Other
aspects of the postoperative full-fever workup include a white
blood cell count, urine culture, blood cultures, and a chest x-ray
as well as cultures of other suspected sites/sources of infection.
Mortality rates for patients diagnosed with bacteremia range from
20% to 35% and can be as high as 60% in patients admitted to the
ICU. However, the yield of full-fever workups are frequently low,
with ranges of bacteremia between 0% and 3% in postoperative
patients with fever.8,22-25
In order to diagnose a bloodstream infection, a pathogen must be isolated from at least one set of blood cultures. 2

PMPH_CH111.indd 877

877

A set of blood cultures is defined by the College of American


Pathologists as 1 or more specimens collected serially within a
24-hour period to detect a bacteremic episode.26 At least 10 to
20 mL22,27 and up to 30 mL of blood should be collected for adults,
1 to 2 mL for neonates, 2 to 3 mL for infants (124 months),
3 to 5 mL for older children, and 10 to 20 mL for adolescents;9,28
larger volumes of blood are associated with less contamination and more true positive growth.15,29 In fact, the single, most
important factor in sensitivity in blood cultures is the volume of
blood drawn.27 Blood should be sampled from at least two different sites7,22,27 and should be sent for both aerobic and anaerobic
cultures.7,21 The number of blood samples as well as volume of
blood collected can impact the sensitivity and specificity of blood
cultures.9,22 In addition, a sample should ideally be taken prior
to the patients use of antibiotics. The average estimated cost for
obtaining blood cultures is $70 to $230 per set obtained, and the
average estimated cost to identify a single patient with bacteremia is around $3000.22
Unfortunately, contamination of blood cultures3,22,26,30 and
false-positive blood cultures15 are common and can confuse the
clinical picture of a patient, which can lead to unnecessary further
testing, treatment, and length of stay.7,15,22 As many as 50% of all
positive blood cultures are actually falsely positive.5 The most
common contaminants, as described by the College of American
Pathologists, are as follows: coagulase-negative Staphylococcus
species, Propionibacterium acnes, Micrococcus species, viridansgroup Streptococci, Corynebacterium species, and Bacillus species. These are typically considered to be contaminants when
they are isolated from only one culture in the set.3,26,27 Several
antiseptic techniques have been shown to decrease the likelihood
of blood culture contamination: using gloves and antimicrobial
scrubs prior to venipuncture; having cultures obtained by dedicated phlebotomists rather than nursing staff; providing feedback
on contamination rates; and obtaining blood cultures from separate, new sitesnamely, those that do not already contain an
indwelling catheter.26,30 Blood cultures drawn from a preexisting indwelling catheter are more likely to be contaminated when
compared with fresh venipuncture sites.31 In addition, if only one
blood sample is obtained, 65% of the time it will yield a true positive result. If two blood samples are obtained, this increases to
80% when both return positive; and if three blood samples are
obtained, this increases to 96% when all bottles return the same
organism(s).9,32 Generally, two blood culture sets are sufficient to
establish bacteremia.22 Postoperative blood cultures drawn on
postoperative day 4 or greater are more likely to yield a true positive result than those drawn sooner. Patients who received preoperative antibiotics were significantly less likely to have subsequent
positive blood cultures postoperatively.7
Blood cultures typically take 12 to 24 hours to detect pathogens. Identification of microbes as well as susceptibilities to specific antibiotics usually takes approximately 6 to 24 hours after
the initial isolation of pathogens.15,16 In a study on Staphylococcus aureus bacteremia by Khatib et al., it was found that a time
to positivity of blood cultures of less than or equal to 14 hours
was associated with the highest sensitivity and specificity for
prediction of the source of bacteremia. These patients were also
at higher risk for extended bacteremia, metastatic infection, and
mortality.29
As previously mentioned, many surgical patients will end
up with a central venous catheter; it can be difficult to diagnose

5/22/2012 6:15:32 PM

878

Surgery: Evidence-Based Practice

catheter-related infections since there are typically no obvious


signs or symptoms specific to this that will lead a clinician to conclude infection at this site.18,19 A high degree of suspicion is needed.
Catheter-related bloodstream infections are defined by the following criteria: (1) clinical manifestations of infection, including fever,
chills, and hypotension, with no apparent source for bloodstream
infection except for a central venous catheter; (2) bacteremia or
fungemia in a patient with an indwelling central venous catheter
and greater than or equal to 1 positive result of culture of blood
samples obtained from a peripheral vein; and (3) positive difference in time to predict (DTP), or semiquantitative catheter culture (SQCC),33,34 meaning the catheter is removed and a portion
of the tip is placed in culture medium and sent to the laboratory
for analysis.19
In a study performed by Chen et al., paired blood cultures,
meaning one from a peripheral vein and one from the central line,
were obtained from febrile patients with indwelling central venous
catheters. When positive paired cultures were not related with a
possible source for bacteremia, they were evaluated for SQCC
and DTP. DTP was said to be positive if there was a culture result
120 minutes earlier in the blood drawn from the central venous
catheter. SQCC involves culturing the catheter tip, and significant
colonization was defined as greater than 15 colony-forming units
(CFUs) plus positive results from both cultures.26,33
Answer: Optimal methods to diagnose bacteremia are the use
of blood cultures, preferably two separate fresh venipunctures or
one fresh venipuncture and one from an indwelling central venous
catheter if this is a suspected source (Grade B recommendation).
4. What are the most common pathogens in bacteremia after
elective surgery?
Gram-positive cocci are the most common pathogens associated
with nosocomial bacteremia in ICU patients. A Canadian study
noted that most episodes of bacteremia are due to a single organism (67%) as opposed to a polymicrobial infection. In addition,
most bloodstream infections were due to gram-positive organisms (44%). It was also noted that an independent risk factor for
bacteremia-associated death is acquiring this diagnosis while the
patient is admitted to the ICU.2 A study performed in the United
States in 2002 by Karlowsky et al. found gram-positive organisms to be the most common pathogens as well. Specifically,
coagulase-negative Staphylococci were found to be associated
with 42% of all cases of bacteremia, followed by Staphylococcus
aureus (16.5%) and Enterococcus faecalis (8.3%).5
As previously mentioned, many patients undergoing general
surgery have central venous catheters. Some studies involving
critically ill patients in the ICU have demonstrated higher colonization rates at the internal jugular and femoral catheter sites
when compared with subclavian lines.18,35 Femoral vein catheters
were more likely to be colonized with non-Staphylococcus epidermidis species.36 In addition, femoral vein catheters have also been
demonstrated to have higher rates of infections than those in the
subclavian and jugular veins. Of patients diagnosed with central
line infections, those who had undergone surgery in the last 28
days were more likely to have gram-negative infections than those
who had not recently undergone surgery.37
Answer: Most patients with bacteremia have monomicrobial infections that are most commonly due to gram-positive

PMPH_CH111.indd 878

organisms, with the highest number accounted for by coagulasenegative Staphylococci (Grade B recommendation).
5. What are the optimal antibiotics to treat bacteremia, and
how long should treatment last?
Appropriate antibiotic regimens are of the utmost importance
when attempting to maximize patient survival in both sepsis and
septic shock cases. In addition, patients should be appropriately
resuscitated.13 In the Surviving Sepsis Campaign, Dellinger et al.
made multiple recommendations on how to resuscitate a patient
with sepsis. Patients should be provided hemodynamic support
through fluids, vasopressors, and inotropic agents as needed. The
goal of intravenous fluid therapy is to maintain a central venous
pressure (CVP) greater than 8 mm Hg, or greater than 12 mm Hg
if mechanically ventilated. Intravenous vasopressor support is
recommended for maintaining a MAP of greater than or equal to
65 mm Hg; inotropes should be used for myocardial dysfunction,
with dobutamine being the first choice in therapy. Blood transfusions are recommended to maintain a hemoglobin of 7.0 to 9.0 g/
dL.11 When a patient is in septic shock, it is recommended that initial empirical antibiotic therapy include one or more drugs with
activity against the most likely pathogens (bacterial and/or fungal). Broad spectrum antibiotics should be used until the organism is identified and antibiotic sensitivities are tested,11,15,16 which
on average takes about 24 to 36 hours.15,16 It is also important to be
aware of drug resistance, particularly methicillin-resistant Staphylococcus aureus. Full loading doses should be used as tolerated,
being aware of hepatic and renal dysfunction or failure. The antibiotic regimen should be reassessed daily. Combination therapy
(i.e., at least two agents) should be used for patients with known
or suspected pseudomonas infections or for neutropenic patients.
In addition, any intravascular access devices should be removed if
they are a possible source.11
In general, when patients are started on antibiotics, they are
empirically covered for the fi rst 24 to 36 hours on average. That
is, they are given antibiotics that will cover a wide range of possible pathogens prior to having proof of an infection because of
the lag time for growth of cultures and sensitivities to antimicrobials as previously discussed. It is preferable to begin these
antibiotics within 30 minutes of clinical suspicion of sepsis or
septic shock. Some studies cite a 15% absolute reduction in mortality for patients in septic shock who receive timely, appropriate empiric antibiotic therapy.38 Adequate empiric antibiotic
treatment is achieved if at least one of the drugs chosen before
culture/susceptibility results are known was effective against
the pathogen(s) isolated from blood culture. Th is has shown to
be associated with a significantly higher rate of survival than
those patients who did not receive adequate empiric antibiotic
treatment.38-40
When the decision is made to start a patient on empirical antibiotic therapy, several factors must be taken into account. First, the
most likely sources of infection should be considered. Second, the
most likely pathogens and their susceptibility to various antibiotics, based on recent ICU and/or hospital culture results and antibiograms, should be considered, which vary from institution to
institution. The third aspect that should be considered is patient
factors that include immune and nutritional status. If a patient has
diabetes, is on immunosuppressants, has an immunodeficiency, or

5/22/2012 6:15:32 PM

Bacteremia

is asplenic, then these factors should be known and addressed as


needed; that is, maintain tight glucose control and monitor closely
for signs of unusual types of infections. If a patient is malnourished
and does not have normal levels of protein in his or her blood, this
condition can affect the effective dose of drugs; and these patients
may not require the same doses as patients with normal protein
levels. Finally, any possibly virulent or resistant pathogens should
be covered by more than one antimicrobial agent in order to ensure
that at least one of the chosen drugs in the empiric coverage is effective against the offending pathogen.40
A pathogen is tested for sensitivity and specificity to antimicrobials by serially diluting a solution containing antibiotic until
the pathogen can be grown in the solution. Once the minimum
inhibitory concentration (MIC) is one-sixteenth to one-fourth of
the highest serum concentration achievable, the pathogen is termed
sensitive to this antimicrobial agent. It is important to keep in
mind, however, that the antibiotic needs to achieve this concentration at the target site. For bacteremia, this would mean the blood.
For central nervous system infections, this would require these levels to be reached in cerebrospinal fluid. The amount of drug that is
protein-bound must also be taken into consideration.13
Empiric combination therapy should not be continued for
more than 3 to 5 days if cultures remain negative. By this point,
cultures and sensitivities should be available. De-escalation to
the most appropriate single therapy, meaning the least powerful
drug the pathogen is susceptible to, should occur as soon as the
susceptibility profi le is known. The duration of therapy should
be approximately 7 to 14 days, depending on the patients clinical
response.11
Answer: Patients should be treated empirically with broad
spectrum antibiotics at the first sign of bacteremia or sepsis based
on most common species isolated and antibiograms specific to
the institution. Antibiotics should then be tailored to the specific
pathogen(s) isolated using a narrow spectrum, single agent when
possible to avoid development of resistant organisms (Grade B
recommendation).
6. What factors help us predict patient mortality in bacteremia?
The development of a bloodstream infection acquired in the
ICU2 as well as increasing Acute Physiology and Chronic Health
Evaluation (APACHE II) scores in patients with sepsis have been
shown to be independent risk factors for mortality.3 Patients
with more severe APACHE II scores, as well as those admitted
to either a trauma or a neurosurgical ICU, are at a higher risk of
developing bacteremia than other patients.2,3 Medical patients
with sepsis have a lower mortality than surgical patients despite
having a similar severity score.41 Secondary bloodstream infections have been shown to have a higher mortality than primary
bloodstream infections. Gram-positive and gram-negative bacteremia have similar mortality rates. Fungemia, however, has been
demonstrated to have higher mortality rates than bacteremia.20
The number one predictor of mortality in bacteremia and sepsis,
however, is a delay in administering effective empiric antimicrobial therapy.13 Nosocomial bacteremia has a worse outcome than

PMPH_CH111.indd 879

879

community-acquired bacteremia. In addition, bacteremia in the


setting of severe sepsis has been demonstrated as a risk factor for
early mortality. Likewise, initiation of early and adequate antimicrobial treatment is associated with a reduction in mortality
in bacteremic patients.40 It has also been concluded that for every
hour in delay to the administration of appropriate antibiotics,
there is an associated 10% increase in mortality.8
The presence of bacteremia in patients with sepsis was shown
to be associated with an increased risk of death; however, early
mortality did not appear to be affected by bacteremia. They were,
however, shown to have a higher 28- and 60-day mortality as well
as a higher in-hospital mortality rate. Patients with nosocomial
bacteremia were shown to have a significantly increased mortality rate when inadequate empirical antibiotic therapy was used.
Clearly, optimization of antibiotic treatment early is an important
step in the management of adults who are critically ill and carry
the diagnosis of sepsis. In severe sepsis, the use of recombinant
human-activated protein C (Xigris), along with appropriate resuscitation and antibiotic therapy, has been demonstrated to reduce
28-day mortality.11,40
Multiple-organ dysfunction syndrome (MODS) is an evolving clinical syndrome characterized by the development of
abnormalities of organ function that were previously healthy and
normally functioning in critically ill patients. MODS is frequently
seen in sepsis. Failure of one organ appears to establish an amplification process that leads to failure of subsequent organs.10 Mortality in the setting of MODS is quite high, with a rate of 40% with
one-organ system failure, 60% with two-organ system failure,
and 98% with three- or more-organ system failures lasting at least
3 days.42
Answer: Patients with more severe APACHE II scores, nosocomial bacteremia, secondary bacteremia, fungemia, MODS, or
those patients who are admitted to either a trauma or a neurosurgical ICU, or have a delay in empiric antibiotic therapy have
a higher mortality than other bacteremic patients (Grade B
recommendation).
In summary, the spectrum from infection to bacteremia to
sepsis, septic shock, and MODS is a major problem facing both
medical and surgical patients in our population today. 5,12,13 The
total average cost of sepsis in the United States is $17 billion
annually.12 It is important to have a high index of clinical suspicion for all of these diagnoses and to perform blood cultures
promptly.7,9,22 Antimicrobial therapy should be initiated within
the fi rst half hour to an hour after sepsis is recognized, and
patients should be appropriately resuscitated as outlined in the
Surviving Sepsis Campaign.11 Likely pathogens should be considered, and at least one antibiotic that has coverage over those
pathogens should be used.7,22,26,27 If appropriate empirical antibiotics are not initially used, its use has been shown in recent
studies to cause an increase in morbidity as well as mortality.12
If cultures remain negative for 3 to 5 days, antibiotics should be
discontinued to avoid resistant organisms. Once culture results
and sensitivities are available, antimicrobial therapy should be
tailored to monotherapy, if possible, using the least powerful
drug.11

5/22/2012 6:15:33 PM

880

Surgery: Evidence-Based Practice

Clinical Question Summary


Question

Answer

1 What elective
procedures have the
highest incidence of
bacteremia/sepsis?

Esophageal, pancreatic, gastric, and small-bowel surgery have


the highest incidences of sepsis with hernia, thyroid, and
breast surgery having the lowest.

4, 18, 19

2 What are the


most common
predispositions and
comorbidities in
patients who become
bacteremic after
elective general
surgery?

Comorbidities associated with bacteremia include


immunocompromised status, diseases of the cardiovascular
and respiratory systems, hematologic disorders, intestinal
obstruction, and advanced age.

2, 4, 20-21

3 What are the optimal


methods to diagnose
bacteremia?

Optimal methods to diagnose bacteremia are with blood


cultures, preferably two separate fresh venipunctures or
one fresh venipuncture and one from an indwelling central
venous catheter if this is a suspected source.

2, 3, 5, 7-9,
15-16, 18-19,
21-34

4 What are the most


common pathogens
in bacteremia after
elective surgery?

Most patients with bacteremia have monomicrobial infections


which are most commonly due to gram-positive organisms
with the highest number accounted for by coagulasenegative Staphylococci.

2, 5, 18, 35-37

5 What are the optimal


antibiotics to treat
bacteremia and
how long should
treatment last?

Patients should be treated empirically with broad spectrum


antibiotics at the first sign of bacteremia or sepsis based on
most common species isolated and antibiograms specific to
the institution. Antibiotics should then be tailored to the
specific pathogen isolated using a narrow spectrum, single
agent when possible to avoid development of resistant
organisms.

11, 13, 15-16,


38-40

6 What factors help


us predict patient
mortality in
bacteremia?

Patients with more severe APACHE II scores, nosocomial


bacteremia, secondary bacteremia, fungemia, MODS, those
that are admitted to either a trauma or neurosurgical ICU,
or those that have a delay in empiric antibiotic therapy
have a higher mortality than other bacteremic patients.

2-3, 8, 10-11, 13,


20, 40-42

REFERENCES
1. OLeary P, ed. The Physiologic Basis of Surgery. 4th ed. Philadelphia, PA: Lippincott Williams & Wilkins; 2008.
2. Laupland K, Kirkpatrick A, Church D, Ross T, Gregson D. Intensive-care-unit-acquired bloodstream infections in a regional
critically ill population. J Hosp Infect. 2004;58(2):137-145.
3. Karvellas CJ, Pink F, McPhail M, et al. Bacteremia, Acute Physiology and Chronic Health Evaluation II and Modified End Stage
Liver Disease are independent predictors of mortality in critically ill nontransplanted patients with acute on chronic liver failure. Crit Care Med. 2010;38(1):121.
4. Vogel TR, Dombrovskiy VY, Carson JL, Graham AM, Lowry SF.
Postoperative sepsis in the United States. Ann Surg. 2010;252(6):
1065-1071.
5. Karlowsky JA, Jones ME, Draghi DC, Thornsberry C, Sahm DF,
Volturo GA. Prevalence and antimicrobial susceptibilities of bacteria isolated from blood cultures of hospitalized patients in the
United States in 2002. Ann Clin Microbiol Antimicrob. 2004;3(1):7.
6. Silva E, de Figueiredo LFP, Colombari F. Prowess-shock trial: a
protocol overview and perspectives. Shock. 2010;34(7):48.

PMPH_CH111.indd 880

Grade of
Recommendation

References

7. Lee JJ, Martin DR. The efficacy of blood culture in postoperative


patients. Am Surg. 2010;76(10):1172-1175.
8. Reichley RM, Kollef MH. Inappropriate antibiotic therapy in
Gram-negative sepsis increases hospital length of stay. Sepsis.
2011;1:2.
9. Paolucci M LM, Sampri V. Conventional and molecular techniques for the early diagnosis of bacteremia. Int J Antimicrob
Agents. 2010;36(Suppl):S6-16.
10. Bone R, Balk R, Cerra F, et al. Definitions for sepsis and organ
failure and guidelines for the use of innovative therapies in sepsis.
The ACCP/SCCM Consensus Conference Committee. American
College of Chest Physicians/Society of Critical Care Medicine.
Chest. 1992;101(6):1644.
11. Dellinger RP, Levy MM, Carlet JM, et al. Surviving Sepsis Campaign: international guidelines for management of severe sepsis
and septic shock: 2008. Intensiv Care Med. 2008;34(1):17-60.
12. Fitousis K, Moore LJ, Hall J, Moore FA, Pass S. Evaluation
of empiric antibiotic use in surgical sepsis. Am J Surg. 2010;
200(6):776-782.
13. Kumar A. Optimizing antimicrobial therapy in sepsis and septic
shock. Critical care Clin. 2009;25(4):733-751.

5/22/2012 6:15:33 PM

Bacteremia

14. Martin GS, Mannino DM, Eaton S, Moss M. The epidemiology


of sepsis in the United States from 1979 through 2000. N Engl J
Med. 2003;348(16):1546.
15. Paxton A. Nipping contamination in the blood. CAP Today/Coll
Am Pathol. 2000;14(5):1.
16. Lisboa T, Waterer G, Rello J. We should be measuring genomic
bacterial load and virulence factors. Crit Care Med. 2010;
38(10):S656.
17. Lukaszewicz AC, Payen D. The future is predetermined in
severe sepsis, so what are the implications? Crit Care Med.
2010;38(10):S512.
18. Leonidou L, Gogos CA. Catheter-related bloodstream infections:
catheter management according to pathogen. Int J Antimicrob
Agents. 2010;36:26-32.
19. Safdar N, Fine JP, Maki DG. Meta-analysis: methods for diagnosing intravascular devicerelated bloodstream infection. Ann
Intern Med. 2005;142(6):451.
20. Pittet D, Li N, Wenzel R. Association of secondary and polymicrobial nosocomial bloodstream infections with higher mortality. Eur J Clin Microbiol Infect Dis. 1993;12(11):813-819.
21. Brook I. The role of anaerobic bacteria in bacteremia. Anaerobe.
2010;16(3):183-189.
22. Badillo AT, Sarani B, Evans S. Optimizing the use of blood cultures in the febrile postoperative patient. J Am Coll Surg. 2002;
194(4):477.
23. de la Torre SH, Mandel L, Goff BA. Evaluation of postoperative
fever: usefulness and cost-effectiveness of routine workup. Am J
Obstet Gynecol. 2003;188(6):1642-1647.
24. Schey D, Salom EM, Papadia A, Penalver M. Extensive fever
workup produces low yield in determining infectious etiology.
Am J Obstet Gynecol. 2005;192(5):1729-1734.
25. Lesperance R, Lehman R, Lesperance K, Cronk D, Martin M.
Early postoperative fever and the routine fever work-up: results
of a prospective study. J Surg Res. 2011;171(1):245-250.
26. Bekeris LG, Tworek JA, Walsh MK, Valenstein PN. Trends in
blood culture contamination: a College of American Pathologists Q-Tracks study of 356 institutions. Arch Pathol Lab Med.
2005;129(10):1222-1225.
27. Mermel LA, Maki DG. Detection of bacteremia in adults: consequences of culturing an inadequate volume of blood. Ann Intern
Med. 1993;119(4):270.
28. Paisley JW, Lauer BA. Pediatric blood cultures. Clin Lab Med.
1994;14(1):17.
29. Khatib R, Riederer K, Saeed S, et al. Time to positivity in Staphylococcus aureus bacteremia: possible correlation with the source
and outcome of infection. Clin Infect Dis. (An official publication
of the Infectious Diseases Society of America) 2005;41(5):594.

PMPH_CH111.indd 881

881

30. Caldeira D, David C, Sampaio C. Skin antiseptics in venous


puncture-site disinfection for prevention of blood culture contamination: systematic review with meta-analysis. J Hosp Infect.
2011;77(3):223-232.
31. Bryant J. Reliability of blood cultures collected from intravascular catheter vs. venipuncture. Ped Infect Dis J. 1988;7(9):679.
32. Cockerill III F, Wilson J, Vetter E, et al. Optimal testing parameters for blood cultures. Clin Infect Dis. 2004;38(12):1724-1730.
33. Chen WT, Liu TM, Wu SH, Tan TD, Tseng HC, Shih CC. Improving diagnosis of central venous catheter-related bloodstream
infection by using differential time to positivity as a hospital-wide
approach at a cancer hospital. J Infect. 2009;59(5):317-323.
34. Mermel LA, Farr BM, Sherertz RJ, et al. Guidelines for the management of intravascular catheter-related infections. Clin Infect
Dis. 2001;32(9):1249-1272.
35. Goetz AM, Wagener MM, Miller JM, Muder RR. Risk of infection due to central venous catheters: effect of site of placement
and catheter type. Infect Control Hosp Epidemiol. 1998;19(11):
842-845.
36. Gowardman JR, Robertson IK, Parkes S, Rickard CM. Influence of
insertion site on central venous catheter colonization and bloodstream infection rates. Intensiv Care Med. 2008;34(6):1038-1045.
37. Sreeramoju PV, Tolentino J, Garcia-Houchins S, Weber SG. Predictive factors for the development of central line-associated
bloodstream infection due to gram-negative bacteria in intensive
care unit patients after surgery. Infect Control Hosp Epidemiol.
2008;29(1):51-56.
38. Rivers E, Nguyen B, Havstad S, et al. Early goal-directed therapy
in the treatment of severe sepsis and septic shock. N Engl J Med.
2001;345(19):1368.
39. Paul M, Shani V, Muchtar E, Kariv G, Robenshtok E, Leibovici
L. Systematic review and meta-analysis of the efficacy of appropriate empiric antibiotic therapy for sepsis. Antimicrob Agents
Chemother. 2010;54(11):4851.
40. Garnacho-Montero J, Garcia-Garmendia JL, BarreroAlmodovar A, Jimenez-Jimenez FJ, Perez-Paredes C, OrtizLeyba C. Impact of adequate empirical antibiotic therapy on the
outcome of patients admitted to the intensive care unit with sepsis. Crit Care Med. 2003;31(12):2742.
41. Bernard G, Vincent J, Laterre P, et al. Recombinant human protein C Worldwide Evaluation in Severe Sepsis (PROWESS) study
group. Efficacy and safety of recombinant human activated protein C for severe sepsis. N Engl J Med. 2001;344(10):699-709.
42. Baue AE, Durham R, Faist E. Systemic inflammatory response
syndrome (SIRS), multiple organ dysfunction syndrome (MODS),
multiple organ failure (MOF): are we winning the battle? Shock.
1998;10(2):79.

5/22/2012 6:15:33 PM

Commentary on
Bacteremia: An Evidence-Based
Review of Recommendations for
Elective General Surgery
John C. Marshall

products such as endotoxin or bacterial DNA reflected infection,


and whether infection could arise in tissues normally colonized
with bacteria (e.g., the gut epithelium in Clostridium difficile colitis) was not addressed. When infection elicited a systemic host
response, sepsis was said to be present. But the conference recognized that a systemic response to infection was an appropriate
process to expedite the elimination of an infectious threat, and so
sepsis was not necessarily a deleterious process. For this reason,
the concept of severe sepsis was proposed to denote sepsis in association with a maladaptive host response leading to organ dysfunction, and that of septic shock to describe sepsis in association
with systemic cardiovascular collapse. Moreover, it was apparent
that a response indistinguishable from the response to infection
could arise in the absence of infection, and so a new term was
needed to describe the response, independent of its cause. The
neologism Systemic Inflammatory Response Syndrome (SIRS) was
coined. The conference itself posited SIRS as a concept, and did not
provide specific diagnostic criteria; these were added later as the
manuscript was prepared, and were based solely on the opinion of
those who developed the final version. The SIRS criteria identify a
population of patients at increased risk of adverse outcome (perhaps not surprising, since they represent 4 of the 12 domains of
the APACHE II score). Whether these generic clinical criteria are
the best markers of the clinical syndrome is controversial, and a
more recent consensus conference has recommended broadening
the criteria used to characterize SIRS as a clinical syndrome.4
So in trying to develop evidence-based guidelines for the diagnosis and management of bacteremia in surgical patients, one is
inevitably subverted by the imprecision of the data sources, which
conflate bacteremia and the generic concept of sepsis and which
derive from a highly heterogeneous group of studies that have
focused on diverse patient populations. Evidence-based recommendations are only as good as the quality and clarity of the evidence. When both are low, the role of a commentary is to frame
some caveats that should be considered in implementing recommendations for clinical management.
Bacteremia can arise from one of three sources. Primary bacteremia, as Dr Hetz notes, occurs in the absence of an identified

Dr Hetz and her colleagues have made a valiant attempt to provide


an evidence-based framework to assess the epidemiology, diagnosis, and management of bacteremia in patients who have undergone general surgical procedures. The effort is subverted not only
by the relative lack of strong evidence to inform the questions they
address, but even more importantly, by the shortcomings of the
conceptual framework within which this analysis is made. I will
focus on the latter as it provides the context for the former.
It is now two decades since the ACCP/SCCM consensus conference on definitions was held in Chicago in 1991.1 This meeting
sought to provide clarity to a field in which advances in biologic
understanding had rendered traditional concepts obsolete. It is
only within the last three decades that we have understood that
the morbidity of infection arises not directly through the release
of toxins by the infecting organism, but rather indirectly through
the activation of a host response that, while central to the containment of infection, can produce inadvertent bystander injury
to the host.2,3 The distinction is subtle, but the implications are
profound, for they imply at least three conclusions:
1. The characteristic clinical manifestations of a septic response are
not pathognomonic for an infectious etiology, and conversely,
infection may be present in the absence of the classical clinical
syndrome.
2. Control of infection alone may be inadequate to effect reversal
of the deleterious consequences for the host, and elements of
the host response may be promising targets.
3. Infection, including bacteremia, may be present in the absence
of a characteristic host response, whereas noninfectious insults
such as tissue injury can activate the same cellular responses
as are activated by pathogens, and so elicit a clinical septic
response in the absence of invasive infection.
Contemporary terminology partially addresses these, but does
so imperfectly. Infection was defined as the presence of microorganisms invading normally sterile tissues, independent of the
response evoked in the host. Thus transient bacteremias associated with colonoscopy, or with brushing teeth, could be considered as infections. Whether the presence of circulating bacterial
882

PMPH_CH111.indd 882

5/22/2012 6:15:33 PM

Bacteremia

focus of infection. Enterococcus is a common cause;5 other organisms that are frequently isolated include diphtheroids, coagulasenegative Staphylococci, and Candida. These episodes of bacteremia
may represent translocation from the gastrointestinal tract or contamination of the specimen; however, their origin is uncertain.
The most common type of bacteremia in the hospitalized patient,
that is a device-related infection, differs from primary bacteremia
because it arises from a focusa vascular catheter. Because of its
capacity to create a biofi lm, Staphylococcus epidermidis is the most
common isolate, although other organisms that share the capacity
to establish biofi lmsCandida, Enterococcus, Pseudomonas, for
exampleare also important causes of line infections. Establishing that an episode of bacteremia arises from an intravascular
device is important, for the infection can be cured by source control measures (device removal), and the role of adjuvant antibiotics is uncertain. Although catheter-related infections are readily
diagnosed and managed, they result in an increased risk of mortality for patients in an intensive care unit (ICU),6 and so their
prevention is a priority. Finally, secondary bacteremia represents
systemic dissemination from a focus of infection. Although certain focal infections are commonly associated with bacteremia
(e.g., endocarditis or pneumococcal pneumonia), bacteremia is
less common in those infections that are common in surgical
patients, and it has even been suggested that the absence of bacteremia is a useful criterion in establishing the diagnosis of intraabdominal infection following laparotomy.7 Bacteremia is common
in cholangitis, upper urinary tract infections, and some necrotizing soft tissue infections.
In summary, then sepsis, bacteremia, and infection in the
surgical patient are overlapping clinical entities. Successful treatment requires the rapid identification of the focus, and its eradication by source control measures where possible. Since the septic
response results in organ system dysfunction, which is itself lifethreatening, physiologic monitoring and supportive measures are
provided as appropriate. Systemic antibiotics play an important
adjuvant role; they should be initiated as rapidly as possible,8 but
it is equally important to narrow the spectrum of coverage, once

PMPH_CH111.indd 883

883

an infecting organism has been identified, and to discontinue


them altogether if cultures remain negative. A further synthesis
of the optimal management of sepsis and infection in the sickest
patients can be found in the published guidelines of the Surviving
Sepsis Campaign.9

REFERENCES
1. Bone RC, Balk RA, Cerra FB et al.; ACCP/SCCM Consensus Conference. Definitions for sepsis and organ failure and guidelines
for the use of innovative therapies in sepsis. Chest. 1992;101:16441655.
2. Michalek SM, Moore RN, McGhee JR, Rosenstreich DL, Mergenhagen SE. The primary role of lymphoreticular cells in the
mediation of host responses to bacterial endotoxin. J Infect Dis.
1980;141:55-63.
3. Marshall JC. Sepsis: rethinking the approach to clinical research.
J Leukoc Biol. 2008;83(3):471-482.
4. Levy MM, Fink MP, Marshall JC, et al. 2001 SCCM/ESICM/ACCP/
ATS/SIS International Sepsis Defi nitions Conference. Intensive
Care Med. 2003;29(4):530-538.
5. Barrall DT, Kenney PR, Slotman GJ, Burchard KW. Enterococcal
bacteremia in surgical patients. Arch Surg. 1985;120:57-63.
6. Siempos II, Kopterides P, Tsangaris I, Dimopoulou I, Armaganidis
AE. Impact of catheter-related bloodstream infections on the
mortality of critically ill patients: a meta-analysis. Crit Care Med.
2009;37(7):2283-2289.
7. Le Gall JR, Fagniez PL, Meakins JL, Buisson CB, Trunet P, Carlet
J. Diagnostic features of early high post-laparotomy fever: a prospective study of 100 patients. Brit J Surg. 1982;69:452-455.
8. Kumar A, Roberts D, Wood KE, et al. Duration of hypotension
before initiation of effective antimicrobial therapy is the critical
determinant of survival in human septic shock. Crit Care Med.
2006;34(6):1589-1596.
9. Dellinger RP, Levy MM, Carlet JM, et al. Surviving Sepsis Campaign: international guidelines for management of severe sepsis
and septic shock: 2008. Crit Care Med. 2008;36(1):296-327.

5/22/2012 6:15:33 PM

CHAPTER 112

Prevention of Central Venous


Catheter Infections
Antonio Aponte-Feliciano, J. Matthias Walz, and Stephen O. Heard

Although improvements in the prevention of catheter-related


bloodstream infection (CRBSI) have occurred in the last decade,
CRBSI still remains a significant problem. Approximately 250,000
cases of CRBSI occur per year in US hospitals and 80,000 of these
cases occur in the intensive care unit (ICU).1 Although the attributable mortality associated with CRBSI is debatable, the morbidity is high and the economic costs are profound.1
For the purposes of this chapter, a CRBSI is a clinical definition that must satisfy specific criteria (e.g., catheter cultures, time
to positivity of paired catheter blood, and peripheral blood cultures) as to whether or not the catheter is the cause of the bacteremia. A central line associated bloodstream infection (CLABSI) is
a surveillence definition and is defined as a primary bloodsteam
infection without any other obvious source in a patient who had
a centeral venous catheter (CVC) any time in the 48 h prior to the
bacteremia.2

have shown that the subclavian insertion site is associated with


the lowest rate of significant catheter colonization.7-9 Thus, the
preponderance of data supports the use of the subclavian site for
catheter insertion.
Maintenance of sterility during insertion of the catheter is
extremely important in preventing catheter infection. Most clinicians in the 1980s and early 1990s would wear only sterile gloves
and use a small drape when inserting CVCs. In a randomized,
controlled trial in cancer patients undergoing central venous catheterization, Raad et al. demonstrated that the use of maximum
barrier precautions reduces the incidence of CRBSI.10 Although
a recent prospective randomized study failed to demonstrate the
efficacy of maximum barrier precautions,11 the Raad study and
other observational studies12,13 (including those where this practice has been part of a bundle) support the use of this practice.13
Recommendations: The subclavian site is the preferred site
for catheter insertion (Grade B). Maximum barrier precautions
(cap, mask, sterile gown and gloves, and a sterile drape that completely covers the patient) should be used during catheter insertion (Grade B).

1. What is the best method and site to insert intravascular


catheters?
A significant amount of research has focused on these issues. A
number of randomized prospective trials and observational studies support the notion that use of the femoral insertion site is
associated with a higher risk of catheter colonization and CRBSI.
Merrer et al. randomized ICU patients to receive either a femoral
or a subclavian CVC.3 At the time of removal, catheters were cultured using a quantitative broth dilution technique. The authors
found that the femoral catheters had a much higher rate of significant colonization compared to the CVCs in the subclavian position. Furthermore, there was a trend toward a higher rate of CRBSI
in the femoral catheter group. The majority of (but not all) prospective observational studies have corroborated these findings.4-6
The subclavian site of insertion appears to be associated with a
lower risk of colonization (if not infection) compared to the internal jugular site. In a prospective observational study, Nagashima
et al. observed a two-fold increase in CRBSI when catheters were
inserted in the internal jugular vein compared to the subclavian
vein.4 Randomized, prospective studies evaluating the effectiveness of anti-infective catheters or the risk of multilumen catheters

2. What is the role of chlorhexidine in insertion and management of CVCs?


Skin antisepsis is an important component of preparation for catheter insertion and maintenance. Many studies have compared a
variety of skin antiseptics, most notably 10% povidone-iodine, 70%
isopropyl alcohol, and 2% aqueous chlorhexidine, and the majority of them have demonstrated that the aqueous or alcoholic chlorhexidine solutions are superior in reducing significant catheter
colonization.14,15 In a meta-analysis of eight randomized controlled
trials evaluating the effectiveness of chlorhexidine solutions compared to povidone-iodine solutions in preventing significant catheter colonization and CRBSI, Chaiyakunapruk et al. found that the
chlorhexidine solutions were the superior skin antiseptic agents.16
In a follow-up study, the authors found that the use of a chlorhexidine skin prep resulted in savings of $113 per catheter.17
Timsit et al.18 performed a single-blinded randomized control
trial evaluating chlorhexidine impregnated sponges to a standard
884

PMPH_CH112.indd 884

5/22/2012 6:16:06 PM

Prevention of CVC Infections


Major catheter-related infection
0.05

Dressing
Control
CHGIS

0.04
Cumulative risk

HR, 0.39 (95% Cl, 0.17-0.93); P = .03

0.03

0.02

0.01

0
2
No. of catheters at risk
1762
Control
1908
CHGIS

4
6
8
10
12
14
Duration of catheter maintenance, d

16

1378
1524

156
200

949
1070

678
750

482
538

325
386

228
272

Figure 112.1 Cumulative risk of major catheter-related infection as a function of duration of catheterization when using chlorhexidine impregnated sponges compared to a standard dressing.
The hazard ratio is significantly lower in the sponge group. From
Timsit et al.18 with permission.
dressing. Both catheter colonization and CRBSI were significantly
lower in the sponge group. Furthermore, the dressings could
remain safely in place for up to 7 days (Fig. 112.1).
Chlorhexidine bathing has been used in ICUs and preoperatively to reduce skin bacteria, thereby decreasing the risk of subsequent infections. Dixon and Carver19 performed an observational
cohort study using historical controls and found that daily cloth
bathing with 2% chlorhexidine gluconate significantly reduced
the rate of CRBSI from 12.07 to 3.17 per 1000 catheter-days. A
before and after intervention study20 with daily chlorhexidine
bathing demonstrated similar findings. In an observational study,
another group of investigators found no beneficial effect of chlorhexidine bathing on CLABSI in a surgical ICU21 but did find a
reduction in CLABSI when such an intervention was used in a
medical ICU.22 Clearly, randomized controlled trials are necessary to delineate the role of daily chlorhexidine bathing in the
reduction of catheter infection.
Recommendations: An aqueous or alcoholic chlorhexidine
solution is the preferred skin antiseptic prior to catheter insertion and during catheter maintenance (Grade A). Chlorhexidine
impregnated sponges reduce CRBSI (Grade A). Daily bathing with
chlorhexidine gluconate reduces the rates of CLABSI (Grade B).
3. Is education useful in preventing CRBSI?
Studies have demonstrated that catheterization by less experienced providers is associated with a higher risk of infection.23
The presumption regarding these observations was that it was the
technical skill of the provider performing the task rather than the
knowledge about basic infection control practices that was important in the development of infection. Sherertz et al.24 demonstrated that an education program that consisted of a didactic
program and a hands-on demonstration of insertion of both
arterial and central venous catheters and that was offered to
beginning PGY-1 physicians resulted in a steady and significant

PMPH_CH112.indd 885

885

reduction in catheter-related infection and primary bloodstream


infections (per 1000 patient-days) over time. Other investigators
have expanded the scope of infection control and best practices
education. Coopersmith et al.25 used a focused educational initiative that includes not only interns but also ICU nurses, residents,
and attending physicians. The educational aspect consisted of a
10-page self-study module with a pre- and posttest. When the
educational program was implemented, a sustained reduction in
CRBSI was documented in both surgical and medical ICUs. Similar educational programs have achieved success in nonteaching,
community hospitals.26
Simulation-based medical education is being used more frequently in medical schools and residency training programs. An
observational cohort study of medical and emergency medicine
residents rotating in a medical ICU using a simulation-based central venous catheter placement mastery learning program demonstrated a decrease in CRBSI from 3.2 to 0.5 per 1000 catheter-days.27
Khouli et al.28 performed a randomized controlled single-blinded
trial to evaluate sterile technique during central vein catheterization
with second- and third-year medical residents. The study assigned
residents to a video training alone or video training and simulation.
The authors found that previous to this intervention, the CRBSI was
3.5 infections per 1000 catheter-days whereas the postintervention
CRBSI rate decreased to 1 per 1000 catheter-days.
Recommendations: An education program for all staff including nurses, physicians, and physician extenders will reduce the
risk of CRBSI (Grade B). Simulation-based education in conjunction with video training will assist in decreasing CRBSI rates
(Grade B).
4. What other behavioral interventions are helpful in preventing or reducing CRBSI?
Although educational programs are helpful in reducing CRBSI,
compliance to the principles enumerated in the initiatives may
wane over time. Eighteen months after initiation of an education
program, Coopersmith et al.29 audited the compliance in their surgical ICU with the best practice principles to reduce CRBSI. They
discovered that hand hygiene before catheter insertion was poor:
only 17% of providers washed their hands prior to catheter insertion and a large sterile drape was used only in 50% of insertions.
Subsequently, they posted pictures of every step of catheter insertion (aimed at physicians) and maintenance (aimed at nurses) at
the bedside, throughout the ICU and in the orientation manuals
for the residents. In addition to the previously mentioned written educational modules, yearly lectures were given to the nurses
and monthly lectures to the residents as they rotated into the ICU.
Despite these interventions, hand hygiene before catheter insertion increased to only 30% and use of a large sterile drape during
the procedure increased to 80%. The authors observed an insignificant reduction in rates of CRBSI; from 3.4/1000 catheter-days
to 2.8/1000 catheter-days. In a prospective randomized trial, Xiao
and colleagues demonstrated that an online training course containing video clips of CVC insertion compared to a paper version
of the course or no training improved compliance with sterile
technique during catheter insertion by surgical and emergency
medicine residents.30 Nonetheless, even in the video study group,
compliance was only 74%. These studies highlight the difficulty
with adherence to best practice principles.
In contrast, Berenholtz et al.31 used a somewhat different intervention model to prevent CRBSI. Their intervention consisted of

5/22/2012 6:16:06 PM

886

Surgery: Evidence-Based Practice

five parts: (1) a web-based education program (preinsertion hand


hygiene, use of chlorhexidine skin preparation, full barrier precautions during insertion, the use of the subclavian vein as the
preferred insertion site, maintenance of sterile field during catheter placement and the proper care of the catheter after placement)
and test that all physicians or physician extenders were required
to take before they could insert CVCs; (2) creation of a CVC insertion cart where all necessary equipment and supplies were readily
available; (3) use of a daily checklist during patient rounds where
the need for central venous catheterization was questioned; (4) use
of a checklist by the bedside nurse during insertion of a CVC to
insure compliance to best practices; and (5) empowering nurses to
stop procedures if the guidelines were not followed. The control
ICU was one in which only an institutional educational initiative
was used to increase provider awareness about evidence-based
infection control practices for catheter insertion and maintenance. The authors found that their program resulted in sustained
reductions in CRBSI.
The results of Berenholtz et al. have been replicated in other
studies. Most recently, Pronovost et al.32 demonstrated that an
interventional program very similar to the one described by Berenholtz et al. and applied to 108 ICUs in the state of Michigan
resulted in a sustained reduction in CRBSI for up to 18 months.
Incidence rate ratios of CRBSI decreased continuously from 0.62
at the 0- to 3-month time interval to 0.34 at the 16- to 18-month
interval.
Recommendations: Use of catheter bundles or multimodal
interventional programs will reduce the incidence of CRBSI
(Grade B).
5. How long should catheters remain in situ?
Numerous observational studies have demonstrated that the
risk of significant catheter colonization or CRBSI increases with
the duration of catheterization. These data led to the hypothesis that scheduled changes of catheters, either by insertion at
a new site or changing over a guidewire, would reduce the risk
of infection. Small, relatively underpowered studies of CVCs,
peripheral arterial catheters, and pulmonary artery catheters
demonstrated that routine changes irrespective of method did
not result in a reduction of infection. 33-35 Cook et al. 36 performed
a meta-analysis of 12 prospective randomized trials evaluating
this question and determined that scheduled changes were associated with a higher risk of catheter colonization. Furthermore,
there was a trend toward a higher risk of CRBSI when guidewire exchange was utilized. On the basis of these data, scheduled changes of CVCs cannot be recommended and guidewire
exchanges should be avoided unless the risk for mechanical
complications associated with the insertion of CVC at a new
site exceeds the risk of infectious complications associated with
a guidewire change.
Recommendations: Catheters should not be routinely changed. If the catheter needs to be changed, a new insertion site
should be used (Grade B).
6. Should anti-infective catheters be utilized?
Three commercial anti-infective catheters are available with different protective coatings or impregnated materials: (1) chlorhexidine and silver sulfadiazine; (2) minocycline-rifampin; and (3)
silver in a carbon/platinum matrix.

PMPH_CH112.indd 886

Raad et al.37 and Darouiche et al.38 compared the minocyclinerifampin catheters to uncoated catheters and first-generation
chlorhexidine-silver sulfadiazine catheters, respectively, and demonstrated a significant reduction in CRBSI in the minocyclinerifampin catheter group.
First-generation chlorhexidine-silver sulfadiazine catheters
had the external surface coated with these antiseptics. Randomized prospective trials and meta-analyses demonstrated their efficacy and cost-effectiveness in reducing catheter colonization and
CRBSI.39-41 Second-generation chlorhexidine-silver sulfadiazine
catheters were subsequently developed. These catheters contain
more chlorhexidine on the catheter surface and chlorhexidine is
also present on the luminal surface. Recent large randomized controlled trials comparing them to uncoated catheters have failed to
demonstrate their efficacy in reducing CRBSI.42,43
Several recent randomized controlled trials have evaluated
the effectiveness of the silver/carbon/platinum catheter against
uncoated catheters. In general, catheter colonization is significantly reduced when the silver/carbon/platinum catheters are
used.7,44,45 In one study, the rate of CRBSI was significantly reduced
when the anti-infective catheter was used.44 One large prospective randomized study has been performed that compared the
silver/carbon/platinum catheter to the minocycline-rifampin
catheter.46 There was a trend toward reduced colonization in the
minocycline-rifampin catheter group, but overall there was no
difference in the rate of CRBSI between the two groups.
Since focused education and behavior modification programs reduce the risk of CRBSI, the use of anti-infective catheters may be ineffective in ICUs where the rate of CRBSI is low.
Many of the negative randomized, prospective controlled trials cited above used best practices during the conduct of the
study. Such practices may explain why no difference was noted
in CRBSI rates. Further support for this notion comes from an
observational study by Schuerer et al.,47 where they examined
CRBSI rates before and after the introduction of second-generation chlorhexidine-silver sulfadiazine catheters in their ICU.
An education program to reduce the risk of CRBSI was in place
during the entire study. The authors found that there was no significant difference in CRBSI between the control period when
uncoated catheters were used (3.3/1000 catheter-days) and the
intervention period when the anti-infective catheters were used
(2.1/1000 catheter-days.
Recommendations: In summary, the anti-infective catheters
appear to be efficacious in preventing significant catheter colonization that is often used as a proxy for efficacy. In ICUs where the rate
of CRBSI is high, they also reduce the risk of CRBSI (Grade A).
7. Do anti-infective catheters promote antibiotic resistance?
Despite the availability of anti-infective catheters, fears about
the development of antiseptic or antibiotic resistance when these
catheters are used (particularly the antimicrobial impregnated
catheters) persist. However, the preponderance of data suggests
that resistance to either antiseptic or antibiotic combinations does
not occur. For the chlorhexidine-silver sulfadiazine catheters, randomized prospective trials and observational studies suggest that
these catheters do not promote antiseptic resistance.39,42,43 Furthermore, following a cluster of coagulase negative staphylococcus (CNS) CRBSI after the introduction of chlorhexidine-silver
sulfadiazine CVCs, Rosato et al.48 did not observe any resistance
of the CNS isolates to the antiseptic combination in vitro.

5/22/2012 6:16:06 PM

Prevention of CVC Infections

Recommendations: The use of anti-infective catheters does


not promote bacterial resistance to either chlorhexidine-silver
sulfadiazine or minocycline-rifampin (Grade B).
Phase I
9

Phase II

Phase III

Phase IV

8.3

8
Incidence density
per 1000 patient days

In vitro studies of the minocycline-rifampin catheters suggest that bacterial resistance does not develop, but small increases
in the minimum inhibitory concentration of the bacterial
combination may occur with Staphylococcus epiderimidis.49-51
Randomized controlled trials have not demonstrated the development of bacterial resistance to minocycline or rifampin. 37,38
One prospective observational study demonstrated that when
the catheter was introduced into an ICU, there was a significant
decrease in nosocomial and multidrug-resistant bacteremias. 52
In a retrospective cohort study of the use of the minocyclinerifampin catheter in patients hospitalized for leukemia or bone
marrow transplantation, Chatzinikolaou et al. 53 demonstrated
no change in the susceptibility patterns of staphylococci to
either antibiotic over a 4-year period when compared to a
baseline time period. In a retrospective clinical cohort study,
Ramos et al. 54 evaluated catheters inserted during the period
from 1999 to 2006 (8009 patients) at a tertiary university-based
cancer center. The incidence of CLABSI gradually decreased
from 8.3 per 1000 catheter-days to 1.2 per 1000 catheter-days
after the implementation of an infection control bundle and
the use of minocycline-rifampin impregnated CVC. In addition, there was no evidence of increased resistance of staphylococcal isolates from their ICU to tetracycline or rifampin
(Fig. 112.2).

887

AIC use only

Infection control
bundle only * AIC
stopped

AIC and infection control bundle

6
5

P 0.001

4
3
2

1.2

1
0
FY 98 FY 99 FY 00 FY 01 FY 02 FY 03 FY 04 FY 05 FY 06

Figure 112.2 Incidence density of CLABSI as a function of time.


The baseline phase (Phase I) had a rate of 8.3 cases of CLABSI
per 1000 patient-days. During phase II, minocycline-rifampin
(MR) impregnated catheters were introduced. During phase III,
the MR catheters were phased out and infection control bundles
were used. Phase IV included both MR catheters and an infection
control bundle. From Ramos et al. 54 with permission.

Clinical Question Summary


Question

Answer

1 What is the best method


and site to insert
intravascular catheters?

The subclavian site is the preferred site for catheter


insertion. Maximum barrier precautions (cap,
mask, sterile gown and gloves, and a sterile drape
that completely covers the patient) should be used
during catheter insertion.

2c

3-13

2 What is the role of


chlorhexidine in insertion
and management of
CVCs?

A: An aqueous or alcoholic chlorhexidine solution


is the preferred skin antiseptic prior to catheter
insertion and during catheter maintenance.
B: Chlorhexidine impregnated sponges reduce CRBSI.
C: Daily bathing with chlorhexidine gluconate reduces
the rates of CLABSI.

A: 1a

A: A

14-22

B: 1b
C: 2a

B: B
C: B

3 Is education useful in
preventing CRBSI?

A: An education program for all staff including nurses,


physicians, and physician extenders will reduce the
risk of CRBSI
B: Simulation-based education in conjunction with
video training will assist in decreasing CRBSI rates.

A: 2a

A: B

B: 2a

B: B

4 What other behavioral


interventions are helpful
in preventing or reducing
CRBSI?

Use of catheter bundles or multimodal


interventional programs will reduce the incidence
of CRBSI.

2a

29-32

5 How long should catheters


remain in situ?

Catheters should not be routinely changed. If the


catheter needs to be changed, a new insertion site
should be used.

2a

33-36

6 Should anti-infective
catheters be utilized?

Anti-infective catheters prevent significant catheter


colonization. In ICUs where the rate of CRBSI is
high, they also reduce the risk of CRBSI.

1a

5, 37-47

7 Do anti-infective catheters
promote antibiotic
resistance?

The use of anti-infective catheters does not promote


bacterial resistance to either chlorhexidine-silver
sulfadiazine or minocycline-rifampin.

2a

39, 42, 43,


48-54

PMPH_CH112.indd 887

Level of
Evidence

Grade

References

23-28

5/22/2012 6:16:07 PM

888

Surgery: Evidence-Based Practice

REFERENCES
1. Raad I, Hanna H, Maki D. Intravascular catheter-related infections: advances in diagnosis, prevention, and management. Lancet Infect Dis. 2007;7:645-657.
2. OGrady N P, Alexander M, Burns LA, et al. Guidelines for the
prevention of intravascular catheter-related infections. Clin
Infect Dis. 2011;52:2162-93.
3. Merrer J, De Jonghe B, Golliot F, et al. Complications of femoral
and subclavian venous catheterization in critically ill patients: a
randomized controlled trial. JAMA. 2001;286:700-707.
4. Nagashima G, Kikuchi T, Tsuyuzaki H, et al. To reduce catheterrelated bloodstream infections: is the subclavian route better
than the jugular route for central venous catheterization? J Infect
Chemother. 2006;12:363-365.
5. Deshpande KS, Hatem C, Ulrich HL, et al. The incidence of infectious complications of central venous catheters at the subclavian,
internal jugular, and femoral sites in an intensive care unit population. Crit Care Med. 2005;33:13-20; discussion 234-235.
6. Goetz AM, Wagener MM, Miller JM, Muder RR. Risk of infection due to central venous catheters: effect of site of placement and
catheter type. Infect Control Hosp Epidemiol. 1998;19:842-845.
7. Moretti EW, Ofstead CL, Kristy RM, Wetzler HP. Impact of central venous catheter type and methods on catheter-related colonization and bacteraemia. J Hosp Infect. 2005;61:139-145.
8. Heard SO, Wagle M, Vijayakumar E, et al. Influence of triplelumen central venous catheters coated with chlorhexidine and
silver sulfadiazine on the incidence of catheter- related bacteremia. Arch Intern Med. 1998;158:81-87.
9. Templeton A, Schlegel M, Fleisch F, et al. Multilumen central
venous catheters increase risk for catheter-related bloodstream
infection: prospective surveillance study. Infection. 2008;36:
322-327.
10. Raad, II, Hohn DC, Gilbreath BJ, et al. Prevention of central
venous catheter-related infections by using maximal sterile barrier precautions during insertion. Infect Control Hosp Epidemiol.
1994;15:231-238.
11. Ishikawa Y, Kiyama T, Haga Y, et al. Maximal sterile barrier precautions do not reduce catheter-related bloodstream infections
in general surgery units: a multi-institutional randomized controlled trial. Ann Surg. 2010;251:620-623.
12. Mermel LA, Maki DG. Infectious complications of Swan-Ganz
pulmonary artery catheters. Pathogenesis, epidemiology, prevention, and management [published erratum appears in Am
J Respir Crit Care Med 1994;150(1):290]. Am J Respir Crit Care
Med. 1994;149:1020-1036.
13. Carrer S, Bocchi A, Bortolotti M, et al. Effect of different sterile
barrier precautions and central venous catheter dressing on the
skin colonization around the insertion site. Minerva Anestesiol.
2005;71:197-206.
14. Maki DG, Ringer M, Alvarado CJ. Prospective randomised trial
of povidone-iodine, alcohol, and chlorhexidine for prevention of
infection associated with central venous and arterial catheters.
Lancet. 1991;338:339-343.
15. Mimoz O, Pieroni L, Lawrence C, et al. Prospective, randomized
trial of two antiseptic solutions for prevention of central venous
or arterial catheter colonization and infection in intensive care
unit patients. Crit Care Med. 1996;24:1818-1823.
16. Chaiyakunapruk N, Veenstra DL, Lipsky BA, Saint S. Chlorhexidine compared with povidone-iodine solution for vascular catheter-site care: a meta-analysis. Ann Intern Med. 2002;136:792-801.
17. Chaiyakunapruk N, Veenstra DL, Lipsky BA, Sullivan SD, Saint
S. Vascular catheter site care: the clinical and economic benefits

PMPH_CH112.indd 888

18.

19.

20.

21.

22.

23.

24.

25.

26.

27.

28.

29.

30.

31.

32.

33.

34.

35.

of chlorhexidine gluconate compared with povidone iodine. Clin


Infect Dis. 2003;37:764-771.
Timsit JF, Schwebel C, Bouadma L, et al. Chlorhexidineimpregnated sponges and less frequent dressing changes for
prevention of catheter-related infections in critically ill adults: a
randomized controlled trial. JAMA. 2009;301:1231-1241.
Dixon JM, Carver RL. Daily chlorohexidine gluconate bathing
with impregnated cloths results in statistically significant reduction in central line-associated bloodstream infections. Am J
Infect Control. 2010;38:817-821.
Evans HL, Dellit TH, Chan J, Nathens AB, Maier RV, Cuschieri J.
Effect of chlorhexidine whole-body bathing on hospital-acquired
infections among trauma patients. Arch Surg. 2010;145:240-246.
Popovich KJ, Hota B, Hayes R, Weinstein RA, Hayden MK. Daily
skin cleansing with chlorhexidine did not reduce the rate of
central-line associated bloodstream infection in a surgical intensive care unit. Intensive Care Med. 2010;36:854-858.
Popovich KJ, Hota B, Hayes R, Weinstein RA, Hayden MK.
Effectiveness of routine patient cleansing with chlorhexidine
gluconate for infection prevention in the medical intensive care
unit. Infect Control Hosp Epidemiol 2009;30:959-963.
Armstrong CW, Mayhall CG, Miller KB, et al. Prospective study
of catheter replacement and other risk factors for infection of
hyperalimentation catheters. J Infect Dis. 1986;154:808-816.
Sherertz RJ, Ely EW, Westbrook DM, et al. Education of physicians-in-training can decrease the risk for vascular catheter
infection. Ann Intern Med. 2000;132:641-648.
Coopersmith CM, Rebmann TL, Zack JE, et al. Effect of an
education program on decreasing catheter-related bloodstream
infections in the surgical intensive care unit. Crit Care Med.
2002;30:59-64.
Warren DK, Zack JE, Elward AM, Cox MJ, Fraser VJ. Nosocomial
primary bloodstream infections in intensive care unit patients in
a nonteaching community medical center: a 21-month prospective study. Clin Infect Dis. 2001;33:1329-1335.
Barsuk JH, Cohen ER, Feinglass J, McGaghie WC, Wayne DB.
Use of simulation-based education to reduce catheter-related
bloodstream infections. Arch Intern Med. 2009;169:1420-1423.
Khouli H, Jahnes K, Shapiro J, et al. Performance of medical residents in sterile techniques during central vein catheterization:
randomized trial of efficacy of simulation-based training. Chest.
2011;139:80-87.
Coopersmith CM, Zack JE, Ward MR, et al. The impact of bedside behavior on catheter-related bacteremia in the intensive care
unit. Arch Surg. 2004;139:131-136.
Xiao Y, Seagull FJ, Bochicchio GV, et al. Video-based training
increases sterile-technique compliance during central venous
catheter insertion. Crit Care Med. 2007;35:1302-1306.
Berenholtz SM, Pronovost PJ, Lipsett PA, et al. Eliminating catheter-related bloodstream infections in the intensive care unit.
Crit Care Med. 2004;32:2014-2020.
Pronovost P, Needham D, Berenholtz S, et al. An intervention to
decrease catheter-related bloodstream infections in the ICU. N
Engl J Med. 2006;355:2725-2732.
Powell C, Kudsk KA, Kulich PA, Mandelbaum JA, Fabri PJ. Effect
of frequent guidewire changes on triple-lumen catheter sepsis.
JPEN J Parenter Enteral Nutr. 1988;12:462-464.
Eyer S, Brummitt C, Crossley K, Siegel R, Cerra F. Catheter-related
sepsis: prospective, randomized study of three methods of longterm catheter maintenance. Crit Care Med. 1990;18:1073-1079.
Cobb DK, High KP, Sawyer RG, et al. A controlled trial of scheduled replacement of central venous and pulmonary-artery catheters. N Engl J Med. 1992;327:1062-1068.

5/22/2012 6:16:07 PM

Prevention of CVC Infections

36. Cook D, Randolph A, Kernerman P, et al. Central venous catheter replacement strategies: a systematic review of the literature.
Crit Care Med. 1997;25:1417-1424.
37. Raad I, Darouiche R, Dupuis J, et al. Central venous catheters
coated with minocycline and rifampin for the prevention of
catheter-related colonization and bloodstream infections. A randomized, double-blind trial. The Texas Medical Center Catheter
Study Group. Ann Intern Med. 1997;127:267-274.
38. Darouiche RO, Raad II, Heard SO, et al. A comparison of two
antimicrobial-impregnated central venous catheters. Catheter
Study Group. N Engl J Med. 1999;340:1-8.
39. Maki DG, Stolz SM, Wheeler S, Mermel LA. Prevention of central venous catheter-related bloodstream infection by use of an
antiseptic-impregnated catheter. A randomized, controlled trial.
Ann Intern Med. 1997;127:257-266.
40. Veenstra DL, Saint S, Saha S, Lumley T, Sullivan SD. Efficacy of
antiseptic-impregnated central venous catheters in preventing
catheter-related bloodstream infection: a meta-analysis. JAMA.
1999;281:261-267.
41. Veenstra DL, Saint S, Sullivan SD. Cost-effectiveness of antiseptic-impregnated central venous catheters for the prevention of
catheter-related bloodstream infection. JAMA. 1999;282:554-560.
42. Brun-Buisson C, Doyon F, Sollet JP, Cochard JF, Cohen Y, Nitenberg G. Prevention of intravascular catheter-related infection with
newer chlorhexidine-silver sulfadiazine-coated catheters: a randomized controlled trial. Intensive Care Med. 2004;30:837-843.
43. Rupp ME, Lisco SJ, Lipsett PA, et al. Effect of a second-generation
venous catheter impregnated with chlorhexidine and silver sulfadiazine on central catheter-related infections: a randomized,
controlled trial. Ann Intern Med. 2005;143:570-580.
44. Corral L, Nolla-Salas M, Ibanez-Nolla J, et al. A prospective, randomized study in critically ill patients using the Oligon Vantex
catheter. J Hosp Infect. 2003;55:212-219.
45. Ranucci M, Isgro G, Giomarelli PP, et al. Impact of oligon central
venous catheters on catheter colonization and catheter-related
bloodstream infection. Crit Care Med. 2003;31:52-59.
46. Fraenkel D, Rickard C, Thomas P, Faoagali J, George N, Ware R. A
prospective, randomized trial of rifampicin-minocycline-coated

PMPH_CH112.indd 889

47.

48.

49.

50.

51.

52.

53.

54.

889

and silver-platinum-carbon-impregnated central venous catheters. Crit Care Med. 2006;34:668-675.


Schuerer DJ, Zack JE, Thomas J, et al. Effect of chlorhexidine/
silver sulfadiazine-impregnated central venous catheters in an
intensive care unit with a low blood stream infection rate after
implementation of an educational program: a before-after trial.
Surg Infect. (Larchmt). 2007;8:445-454.
Rosato AE, Tallent SM, Edmond MB, Bearman GM. Susceptibility of coagulase-negative staphylococcal nosocomial bloodstream
isolates to the chlorhexidine/silver sulfadiazine-impregnated
central venous catheter. Am J Infect Control. 2004;32:486-488.
Sampath LA, Tambe SM, Modak SM. In vitro and in vivo efficacy of catheters impregnated with antiseptics or antibiotics:
evaluation of the risk of bacterial resistance to the antimicrobials in the catheters. Infect Control Hosp Epidemiol. 2001;22:
640-646.
Aslam S, Darouiche RO. Prolonged bacterial exposure to minocycline/rifampicin-impregnated vascular catheters does not affect
antimicrobial activity of catheters. J Antimicrob Chemother.
2007;60:148-151.
Munson EL, Heard SO, Doern GV. In vitro exposure of bacteria
to antimicrobial impregnated-central venous catheters does not
directly lead to the emergence of antimicrobial resistance. Chest.
2004;126:1628-1635.
Hanna HA, Raad, II, Hackett B, et al. Antibiotic-impregnated
catheters associated with significant decrease in nosocomial and
multidrug-resistant bacteremias in critically ill patients. Chest.
2003;124:1030-1038.
Chatzinikolaou I, Hanna H, Graviss L, et al. Clinical experience
with minocycline and rifampin-impregnated central venous
catheters in bone marrow transplantation recipients: efficacy
and low risk of developing staphylococcal resistance. Infect Control Hosp Epidemiol. 2003;24:961-963.
Ramos ER, Reitzel R, Jiang Y, et al. Clinical effectiveness and
risk of emerging resistance associated with prolonged use of
antibiotic-impregnated catheters: more than 0.5 million catheter days and 7 years of clinical experience. Crit Care Med.
2011;39:245-251.

5/22/2012 6:16:07 PM

Commentary on
Prevention of Central Venous
Catheter Infections
Nicole J. Krumrei and Donald H. Jenkins

The use of antiseptic/antibiotic impregnated short-term


CVCs has shown promise in reducing the rate of catheter-related
blood stream infections. However, after the establishment of educational programs, the effect is mitigated. So although antibiotic
impregnated CVCs may help decrease CVC-related infections,
they should not be used in exclusion of good aseptic technique
and adequate training.
Antibiotics have been further investigated in an antibiotic lock
prophylaxis, which has demonstrated usefulness in neutropenic
patients.1 The rate of catheter-related infections was significantly
lower in the antibiotic lock groups (compared with heparin) and
the time to a first episode of bacteremia was longer. Conversely,
the rate of Vancomycin Resistant Enterococci was increased, so
this is not recommended as a standard practice.
We would like to emphasize that in order to acquire a CVCrelated infection, one must have (or recently had) a CVC. Therefore, daily scrutiny by the physician regarding the necessity of the
CVC should be performed and, if deemed nonessential, should be
promptly removed.2 This, in fact, is likely the most potent infection prevention technique of all. Routine placement of CVCs
should not be performed.
Although the authors do not expressly address infection
prevention in percutaneously placed peripheral venous catheters
that terminate in the central vasculature, a so-called peripherally
inserted central catheter (PICC), one can make inference from
their evidence-based review that infection prevention techniques
are similar. Again, the basic tenet is to establish a clear need for
the device and remove it as soon as the goals for its insertion have
been met and the PICC is no longer needed. Perhaps more evidence
in the future will highlight specific differences in the approach to
infection prevention between CVCs and PICC lines.

Dr Aponte et al. have provided an excellent treatise on prevention of


central venous catheter (CVC) infections and detailed their conclusions with the strongest available evidence. Intensivists have come to
heavily rely on CVCs to deliver life-saving medications and transfusions; thus, these catheters are absolutely indispensable in modern
day medical practice. Although CVCs provide necessary vascular
access, their use puts patients at risk for both local and systemic infections. The cost of such infections, both in morbidity and healthcare
expense, is significant. The authors have provided evidence-based
recommendations for preventing CVC-related infections. Their areas
of major emphasis included method and site of catheter insertion,
the role of chlorhexidine, education and behavioral interventions in
the training of healthcare workers, the avoidance of routine catheter
exchange, and the use of anti-infective catheters.
The density of local skin flora at the insertion site has been
demonstrated to be a major risk factor in developing a catheterrelated infection. Adult studies have demonstrated decreased rate
of colonization of catheters in the subclavian position compared
with the internal jugular or femoral approach. Other factors such
as the risk of subclavian stenosis, insertion skill, patient restrictions, and anatomy may factor into catheter placement decision.
Therefore, in adult patients, the subclavian approach, when available, is the preferred location of CVC insertion.
There are strong data to support the education of healthcare
workers regarding the indications for CVC use, proper procedures for insertion, and proper maintenance and inspection of
CVCs. With the increased availability of simulation centers and
web-based programming, there are multiple modes of instruction available for the modern adult learner. Scheduled education
modules for those who place CVCs may increase compliance and
maintenance with best practice techniques.
Moreover, central catheter bundles, where multiple interventions are grouped together to increase compliance, have been
shown to reduce the incidence of CVC-related infections. Similar
to a presurgical pause, a checklist consisting of hand washing, use
of maximal sterile barrier precautions, skin preparation, insertion
site selection, the indication for catheter placement, the immediate availability of necessary equipment, and aseptic technique can
be used successfully after appropriate education has been given.
Chlorhexidine skin preparation should be used prior to insertion as well as during catheter maintenance. The dressing should
be changed weekly at minimum and if the site becomes damp,
loosened, or soiled. Either sterile gauze or sterile transparent
dressing may be used. The skin insertion site should be monitored
daily for signs of infection.

REFERENCES
1. Carratala J, Niubo J, Fernandez-Sevilla A, et al. Randomized,
double-blind trial of an antibiotic lock technique for prevention
of gram-positive central venous catheter-related infection in
neutropenic patients with cancer. Antimicrob Agents Chemother.
1999;43:2200-2204.
2. Parenti CM, Lederle FA, Impola CL, Peterson LR. Reduction of
unnecessary intravenous catheter use: internal medicine house
staff participate in a successful quality improvement project. Arch
Intern Med. 1994;154:1829-1832.

890

PMPH_CH112.indd 890

5/22/2012 6:16:07 PM

CHAPTER 113

Surgical Site Infection (SSI) and


Prophylactic Antibiotics
Robert Krell and Lena M. Napolitano

INTRODUCTION

more than the duration cut-point in hours, where the cut-point


is the 75th percentile of the duration of surgery in minutes for
the procedure). Laparoscopic surgery is associated with decreased SSI risk.
For infection to occur in a surgical wound, bacteria must contaminate the wound. The sources of infection are the patients skin
and/or a hollow viscus entered during the case. There is a marked
increase in wound infection rates when wounds were contaminated with more than 105 bacteria/gram tissue.7 Risk factors for SSI
include patient, environmental, and treatment factors. Patient risk
factors for SSI include extremes of age, diabetes, tobacco use, corticosteroid use, malnutrition, prolonged preoperative hospitalization, and colonization with the infecting organism.8 Environmental
risk factors for SSI include inadequate disinfection/sterilization of
instruments, inadequate skin antisepsis, hair removal with shaving rather than clipping, inadequate ventilation, and contaminated
medications.9 Preincisional skin preparation with chlorhexidinecontaining solution rather than povidone-iodine was superior in
reducing superficial SSI in clean-contaminated cases in a recent
large randomized clinical trial.10 Treatment-related risk factors
for SSI include inadequate antibiotic prophylaxis, hypothermia,
prolonged operative time, emergency procedure, hyperglycemia,
drains, prolonged preoperative hospitalization, and supplemental
oxygen. Perioperative blood transfusion11 and hypothermia (core
temp. < 36C)12 have been associated with increased SSI risk. Maintaining blood glucose <200 mg/dL in both diabetics and nondiabetics during the intraoperative and first 48 postoperative hours
has also been shown to reduce SSI rates.13
Answer: Patient, environmental, and treatment risk factors all increase risk for SSIall preventive strategies to reduce
SSI should be implemented (Grade A recommendation, Level I
evidence).

Surgical site infections (SSIs) are a common problem faced by


the surgeon and surgical patient, with as many as 20% of abdominal surgery patients and 5% of clean nonabdominal surgery
patients experiencing an SSI.1,2 SSI results in longer length of stay,
additional cost, and is associated with higher mortality and readmission rates.3

SSI Definition
In 1992, The CDCs National Nosocomial Infection Surveillance
(NNIS), currently the National Healthcare Safety Network (NHSN)
system, standardized the classification of SSI into three categories:
superficial incisional SSI (affecting skin and subcutaneous tissues),
deep incisional SSI (affecting deep soft tissues including fascia and
muscle), and organ/space SSI (such as abdominal abscess or thoracic empyema).4 SSI was also defined as occurring within 30 days
of the index operation for procedures without implants, and within
1 year of the index operation if implants were placed. Using these
criteria the national reporting of SSI has become standardized,
allowing more meaningful research to be conducted regarding
epidemiology, risk factors, and treatment efficacy.
1. What are the risk factors for SSI?
SSI risk is strongly associated with wound classification, being
low for the clean (Class 1) and clean-contaminated (Class 2,
defi ned as gastrointestinal or genitourinary tract entered in a
controlled manner) incisions and high for the contaminated
(Class 3, defi ned as open traumatic wounds, infected urine or
bile, gross spillage from the gastrointestinal tract) and dirty-infected (Class 4) incisions. Within each wound classification, additional SSI risk is measured by the NNIS/NHSN SSI Risk Index, 5-6
which includes three variables: (1) American Society of Anesthesiologists (ASA) score > 2 (ASA score 3, 4, 5), (2) classification of
a surgical site as contaminated (Class 3) or dirty-infected (Class
4), and (3) prolonged operation (defi ned as operation lasting

2. What are the most common causative pathogens in SSI?


Staphylococcus aureus, coagulase-negative staphylococci, and
enterococci are the predominant organisms causing SSI, responsible for approximately 40% of these infections.10,11 Several important
891

PMPH_CH113.indd 891

5/22/2012 6:18:06 PM

892

Surgery: Evidence-Based Practice

pathogens are to be noted based on the site or type of operation. For transplant operations, enterococci are the predominant
SSI-causing organism and fungal infections have a much higher
prevalence. In abdominal procedures, 38% of isolates from SSI
are gram-positive and 34% are gram-negative. SSIs caused by
methicillin-resistant Staphylococcus aureus (MRSA) are increasing in prevalence nationwide.14 In a study of 8302 patients readmitted to 97 US hospitals with a culture-confirmed SSI, S. aureus was
the most common causative organism, with MRSA SSI increasing in prevalence from 16% to 20%.15 In the same study, MRSA
infections were associated with increased hospital costs and
length of stay.
Answer: Gram-positive pathogens, with S. aureus as the most
common SSI pathogen, with increasing rates of MRSA. In abdominal procedures, SSIs are caused roughly equally by gram-positive
and gram-negative organisms. In transplant recipients, enterococci are the most commonly isolated pathogens and fungal SSIs
are much more common.
3. What is the optimal timing of the initial dose of intravenous
(IV) antibiotics for SSI prevention?
Well-designed prospective studies showed that antibiotics given
prior to surgical incision were effective at reducing SSI, whereas
patients for whom antibiotics were started after wound closure
had the same SSI rate as patients who received no antibiotics.16-17
In a large multicenter study of cardiothoracic, hip arthroplasty,
and hysterectomy patients, the greatest benefit was derived from
antibiotics given within 60 minutes prior to incision, with a trend
toward reduced SSI rates when antibiotics were given within 30
min (SSI rates at 030 minutes and 3060 minutes were 1.6% and
2.4%, respectively). Patients receiving antibiotics after incision
were more than twice as likely to develop SSI as groups receiving
antibiotics prior to incision.18
Answer: Antibiotic administration should occur within 60
minutes prior to surgical incision. Due to recommended long
infusion time, vancomycin should be administered within 120
minutes of surgical incision (Grade A recommendation, Level I-II
evidence).
4. What is the optimal duration of IV antibiotics for SSI
prevention?
The shortest effective duration of antimicrobial administration
for prevention of SSI is indicated, and postoperative antimicrobial
administration is not necessary for most procedures. For most
procedures, the duration of antimicrobial prophylaxis should be
24 hours or less.
In vascular surgery, a 2007 meta-analysis of 35 trials confirmed no benefit of antimicrobial prophylaxis greater than 24
hours.19 A systematic review of single-dose versus multiple-dose
antimicrobial prophylaxis in major surgery demonstrated no
difference in SSI rates and, therefore, recommended single-dose
antimicrobial prophylaxis.20 Clean-contaminated procedures,
complex oncologic procedures, or flap reconstruction procedures
in the head and neck can have SSI rates as high as 87% without
antibiotic prophylaxis.21,22 Studies in clean-contaminated head
and neck procedures found no difference in efficacy between antimicrobial prophylaxis regimens of 24 hours and longer regimens
of 3, 5, or 7 days.23-24 Limited data exist on single-dose prophylaxis
in these procedures.

PMPH_CH113.indd 892

Randomized controlled trials and systematic reviews of gastric and upper gastrointestinal tract operations, including bariatric
procedures and percutaneous endoscopic gastrostomy, confirm efficacy from prophylactic antibiotics in SSI reduction but no benefit to
antimicrobial prophylaxis beyond the duration of the procedure.
No data have shown a benefit to extending the period of antimicrobial prophylaxis beyond the duration of the procedure.25-26
Studies examining single- versus multiple-dose antibiotics in
uncomplicated appendicitis have demonstrated equal efficacy of
both regimens. A randomized trial demonstrated no difference
between a single preoperative dose, three doses, and 5-day antimicrobial prophylaxis regimens, with SSI rates of 6.5%, 6.4%, and
3.6%, respectively.27 A large cohort study of 2139 surgical patients
found no difference in SSI rates between single-dose, 1-day and
multiple-day antimicrobial prophylaxis regimens of metronidazole/gentamicin.28 Two studies of colorectal patients compared
single- versus multiple-dose regimens of the same antibiotic. In
one randomized nonblinded study where doxycycline was the
antibiotic used for prophylaxis, there was no difference in SSI rates
between the single preoperative dose and 3-day prophylaxis groups
(10% SSI rate in both).29 Another randomized nonblinded study
using ampicillin/metronidazole also showed no difference between
single-dose and 3-day regimens (6% SSI rates in both groups).30
The most recent Cochrane review, with 182 trials and 30,880
patients reviewed, found no benefit to extending duration of prophylaxis (single preoperative dose vs. multiple dosing, OR 1.06,
95% CI 0.881.27, P = .58).31
A single preoperative dose of antibiotics with appropriate gramnegative and anaerobic coverage is sufficient antibiotic prophylaxis
for uncomplicated appendicitis and elective colorectal procedures,
with intraoperative redosing if the procedure extends beyond three
half-lives of the antibiotic or there is significant blood loss.
Answer: Antibiotic prophylaxis should terminate within
24 hours of operation. A single preoperative dose with appropriate
intraoperative redosing is adequate in most procedures. Antibiotics should not be continued beyond the duration of the procedure
(Grade A recommendation, Level I-II evidence).

National Efforts in Reducing SSIs


The Surgical Care Improvement Project (SCIP) was created in
2003 as a multi-institution collaborative effort to reduce the
morbidity and mortality of surgical patients. The Surgical Infection Prevention (SIP) Project was created in 2002 to reduce the
incidence, morbidity, and mortality of surgical infections by
implementing evidence-based guidelines as standard of care for
surgical patients. The collaboration of the two entities resulted in
the SCIP-INF quality measures, the first three of which were the
original SIP quality measures (Table 113.1). Adherence to these
measures has been shown to decrease the rate of SSI.32,33

Principles of Antibiotic Prophylaxis for


SSI Prevention
For an antibiotic to be effective prophylaxis for SSI, it should (1)
cover the most likely pathogens that will cause SSI for the particular procedure; (2) be administered such that tissue levels of
antibiotic will be sufficient to have antibiotic activity at the time of
incision; and (3) carry minimal risk to the patient in terms of cost
and side effects.

5/22/2012 6:18:07 PM

SSI and Prophylactic Antibiotics

893

Table 113.1 Individual SCIP Measures


INF-1

Prophylactic antibiotics received within 1 hour prior to surgical incision (2 hours if receiving vancomycin)

INF-2

Prophylactic antibiotic selectionpatients received appropriate recommended antibiotic for their specific surgical procedure

INF-3

Prophylactic antibiotics are discontinued within 24 hours after surgery end time (48 hours for cardiac surgery patients)

INF-4

Cardiac surgery patients with controlled 6 AM postoperative blood glucose level (200 mg/dL)

INF-6

Surgery patients with appropriate surgical site hair removal (clippers or depilatory or those not requiring surgical site
hair removal)

INF-7

Colorectal surgery patients with immediate postoperative normothermia (first recorded temperature was 96.8F within
first 15 minutes after leaving the operating room)

Summary of Principles of Antibiotic Prophylaxis for


SSI Prevention
Optimal SSI antimicrobial prophylaxis includes the following
principles:
1. Preoperative administration. High serum levels of an antimicrobial active against expected microorganisms can be achieved
with preoperative timing, using the IV route of administration,
and using the highest safe dose of the antimicrobial. Weightbased dosing should be considered in obese patients.
2. Redose during procedure. Serum levels of antimicrobials should be
adequate throughout the procedure. For patients with prolonged
procedures or large blood loss or fluid resuscitation, intraoperative
redosing of antibiotics for SSI prevention is necessary.
3. Duration of antimicrobial prophylaxis. Antimicrobials for SSI
prevention should be discontinued after the surgical site is
closed. Maximal duration should be 24 hours after the operative
procedure is completed.
Ultimately, the decision regarding optimal antimicrobial prophylaxis will also depend on local resistance patterns for the pathogens most likely to cause SSI for the particular procedure planned.
Clinicians should be aware of their local antibiotic resistance
profi les as well as their institutional antibiotic armamentarium
to choose the most effective and cost-effective antibiotic prophylaxis. National evidence-based recommendations regarding optimal antimicrobial prophylaxis selection for specific procedures
have been published by the SCIP.
5. What dosing modifications should be used for obese patients?
The physiologic effects of obesity, including insulin resistance and
decreased subcutaneous tissue oxygen tension, combined with the
pharmacokinetic effects of obesity place the obese patient at particularly high risk for developing SSI.34,35 In a recent study of colorectal surgical patient, obese patients (body-mass index [BMI] >
30) were twice as likely to develop SSI, especially superficial SSI,
than patients with BMI < 30.36 Obesity, defined as greater than
25% body fat, was associated with a five-fold increased SSI risk,
and percentage of body fat was found to be a more sensitive and
precise measurement of SSI risk than BMI.37
Studies have documented that in morbidly obese and superobese patients, adequate serum and tissue levels of antibiotic
(with concentrations over Minimum Inhibitory Concentation for
the most common pathogens) were rarely achieved when given a
1-g dose of cefazolin, even with intraoperative redosing according to current guidelines.38,39 Other studies reported a significant

PMPH_CH113.indd 893

decrease in SSI when obese patients were given higher doses of


antibiotics, with a decrease in SSI rates from 16.5% to 5.6% after
increasing the preoperative cefazolin dose from 1 to 2 g.40
Guidelines regarding optimal antimicrobial dosage for SSI
prevention in obese patients are lacking. In addition, the nonuniform effects of obesity on the pharmacokinetics of many antibiotics prevent standardized dosing regimens for most antibiotics
used for SSI prevention. In general, higher doses of antibiotics,
and potentially more frequent dosing, are necessary to maintain
adequate serum and tissue concentrations in obese patients.41
Considering the low cost and safety profi le of cephalosporins,
increasing the dose to 2 g for patients weight >80 kg and 3 g for
patients weight >120 kg can easily be justified.
Answer: In obese patients (BMI > 30), increased dosing and/or
dosing interval modifications may be required to achieve adequate
antibiotic prophylaxis. In general, cefazolin 2 g should be used for
patients greater than 80 kg, and 3 g given for patients heavier than
120 kg (Grade B recommendation, Level II-III evidence).
6. Are prophylactic antibiotics indicated for SSI prevention in
all general surgery procedures, including clean breast and hernia surgery?
Skin and soft tissue, including breast procedures without prosthetic implants, are considered clean cases. There is strong agreement in the literature that antibiotic prophylaxis does not confer
additional benefit to standard skin preparation in preventing SSI
for clean procedures. However, a 2006 Cochrane review including seven randomized, placebo-controlled trials (1984 patients)
undergoing breast cancer procedures (axillary lymph node dissection and primary, nonreconstructive surgery) demonstrated a
significant reduction in SSI rate with antimicrobial prophylaxis
(8% vs. 10.5%, relative risk 0.72 [95% CI: 0.530.97]).42
Hernia repairs are considered clean cases, with overall
reported SSI rates less than 5%. Since the use of prosthetic mesh
materials is common in hernia repair, many surgeons administer prophylactic antibiotics given the known severe morbidity
associated with mesh SSIs. A 2010 Cochrane review of 12 randomized clinical trials (6705 patients) undergoing elective open
inguinal hernia repair found a significant reduction in SSI rate
in the subgroup of patients who underwent hernia repair with
mesh prosthetic material (SSI rate in the prophylaxis group was
1.4% vs. 2.9% in the control group [OR 0.48; 95% CI: 0.270.85]).
In the group that underwent hernia repair without mesh, the SSI
rates for the prophylaxis and control groups were 3.5% and 4.9%,
respectively (OR 0.71; 95% CI: 0.511.00).43 On the basis of these
findings, a single dose of prophylactic antibiotic is recommended

5/22/2012 6:18:07 PM

894

Surgery: Evidence-Based Practice

for SSI prevention in elective hernia repair, particularly since surgeons cannot always determine whether mesh prosthetic material
will or will not be used for the hernia repair preoperatively.
A recent review of all meta-analyses for the effectiveness of
antimicrobial prophylaxis in SSI prevention concluded that antibiotic prophylaxis is an effective intervention for SSI prevention
over a broad range of different surgical procedures. The review
recommended that we should assume that antibiotic prophylaxis is effective in reducing SSI risk for all types of surgery, even
ones where no clinical trial data exist.44 Therefore, in clean surgical procedures, a single dose of antimicrobial prophylaxis is
recommended.
Answer: For elective clean breast and hernia surgery, a single
dose of antibiotic prophylaxis is recommended to reduce the risk
of SSI (Grade A recommendation, Level I, II evidence).

These two reports confirm the efficacy of oral antimicrobials


(acceptable regimens are neomycin/erythromycin base or neomycin/metronidazole) in reducing SSI in elective colorectal surgery
in addition to preoperative IV antibiotics covering facultative
gram-negative aerobes and anaerobes given within 60 minutes of
incision. However, oral antimicrobials are only effective in patients
undergoing MBP for reduction of fecal mass.
Answer: Oral antibiotics should be used in conjunction with
MBP and preoperative IV systemic antibiotics for elective colorectal surgery to achieve the lowest SSI rates. Oral antibiotics should
NOT be used if an MBP is not used (Grade A recommendation,
Level I-II evidence).

7. Are prophylactic antibiotics indicated for SSI prevention in


low-risk elective cholecystectomy?

A Cochrane review included 13 studies and 4777 patients, and


examined the effect of MBP for SSI prevention in elective colorectal surgery.50 No significant difference in SSI rates was identified
(9.6% for MBP patients vs. 8.3% for control patients [OR 1.19; 95%
CI: 0.981.45]). This analysis is limited in that almost all the studies analyzed included MBP without oral antibiotics, thus effectively studying the efficacy of MBP with IV antibiotics (no oral
antibiotics) versus IV antibiotics alone.
Another meta-analysis included 14 trials and 4859 patients.51
They found no statistical difference between the groups for anastomotic leakage (OR= 1.12; [0.821.53]; P = .46), pelvic or abdominal
abscess (P = .75), and wound sepsis (P = .11). When all SSIs were
considered, the meta-analysis favored no MBP (OR=1.40; [1.05
1.87], P = .02). Once again, there was no evaluation of whether oral
antibiotics were administered in those patients who underwent
MBP, and this was not included in the statistical analysis.
A third meta-analysis included five randomized trials to assess
the efficacy of MBP with polyethylene glycol (PEG) in prevention
of postoperative complications in elective colorectal surgery.52
The use of PEG for MBP did not significantly reduce the rate of
SSI (OR 1.39; 95% CI: 0.852.25; P = .19) including incisional SSI
(OR 1.44; 95% CI: 0.882.33; P = .15), organ/space SSI (OR 1.10;
95% CI: 0.432.78; P = .49), or anastomotic leak (OR 1.78; 95% CI:
0.953.33; P = .07). Similarly, the use of oral antimicrobials was
not assessed in this study, which is a significant limitation.
An additional limitation of these systematic reviews is that
some of the studies included in the analyses did not administer
preoperative IV antibiotics, some did not have adequate dosing
of preoperative IV antibiotics, some had inappropriate systemic
antibiotic use (metronidazole alone, ignoring the need for antimicrobial prophylaxis against facultative aerobic gram-negative
organisms common in colonic flora), and some studies gave oral
antimicrobials before MBP, rendering them ineffective. Clearly,
these factors do impact on SSI rates, and these studies should have
been excluded from the analysis.
The Cochrane review by Nelson et al. (182 studies, 30,880
patients) documented the statistically significant lower SSI rate for
colorectal surgery patients who received MBP and oral antibiotics
(MBP + PO + IV antibiotics) versus those who did not receive oral
antibiotics (MBP + IV antibiotics) (P = .000084). These results suggest that MBP without oral antibiotics is associated with higher
SSI rates than patients receiving no MBP (provided the patient
receives IV antibiotic prophylaxis before incision).53
A recent statewide best practices study in Michigan (1553 elective colectomy patients with preoperative MBP) evaluated the use

Risk factors for developing SSI after laparoscopic cholecystectomy


include emergent operations, conversion to open cholecystectomy,
acute cholecystitis or cholangitis, immune suppression, diabetes,
intraoperative gallbladder rupture, ASA score >3, bile spillage,
jaundice, pregnancy, and prolonged operative times.45 A multicenter study of 4477 patients in Germany showed a significant
risk reduction in wound infection for both open and laparoscopic
cholecystectomy when patients were given antibiotic prophylaxis
(1.6% wound infection rate without antibiotic prophylaxis vs. 0.1%
infection rate with antibiotic prophylaxis; and 3.3% overall infectious complication rate without prophylaxis vs. 0.6% with antibiotic prophylaxis).46 In contrast, a meta-analysis of 15 randomized
trials (2961 patients) examined the role of antibiotics in low-risk
elective laparoscopic cholecystectomy (patients without cholecystitis, pancreatitis, history of biliary surgery, or bile spillage).47 The
overall SSI rate was low (48/2961) with 1.47% infections occurring
in the group with antibiotic prophylaxis (primarily cephalosporin
antibiotics) and 1.77% in the control group (OR for prophylaxis
0.79, CI: 0.441.41).
Answer: Because of the limited ability to identify patients at
low risk for SSI, a single dose of antibiotic given within 60 minutes
of incision is recommended to reduce the risk of postoperative SSI
(Grade B recommendation, Level I-II evidence).
8. Are oral antibiotics necessary for SSI prevention in colorectal surgery?
A meta-analysis of oral versus systemic antibiotics in elective
colon surgery compared the efficacy of combined oral/systemic
antibiotics versus systemic antibiotics alone (13 randomized trials
from 1975 to 1995) in SSI prevention.48 Combined oral/systemic
antibiotics were superior to systemic antibiotics alone in SSI prevention (RR 0.51; 95% CI: 0.240.78; P < .001).
A Cochrane review by Nelson et al. (182 studies, 30,880
patients) examined the use of combination oral and IV antibiotics to IV alone for the prevention of SSI.49 They also confirmed
that combined oral/systemic antibiotics were superior to systemic
antibiotics alone in SSI prevention (RR 0.55; 95% CI: 0.410.74;
P < .001). Most studies in this systematic review included a
mechanical bowel preparation (MBP) for all patients and an oral
antibiotic regimen consisting of either oral neomycin/erythromycin combination or aminoglycoside (neomycin or kanamycin)
and metronidazole.

PMPH_CH113.indd 894

9. Is an MBP required for elective colorectal surgery? Does


MBP reduce SSI in colorectal surgery?

5/22/2012 6:18:07 PM

SSI and Prophylactic Antibiotics

of oral antibiotics in a propensity-matched analysis. Two groups


of 370 patients were identified after propensity-score matching; in
the group that received oral nonabsorbable antibiotics the overall
SSI rate was 4.6%; in the group that did not receive oral antibiotics the SSI rate was 12.4% (P < .001). 54 This was not a randomized
trial, however.
Answer: MBP is not required for elective colorectal surgery, but may be useful in cases with anticipated intraoperative
colonoscopy or rectal resections. If MBP is used, oral antibiotics must also be administered for maximal SSI reduction. MBP
without oral antibiotics is associated with higher SSI rates than IV
preoperative antibiotics alone (Grade A recommendation, Level
I-II evidence).
10. In patients colonized or infected with MRSA or VRE, should
IV prophylactic antibiotics be used to cover these pathogens?
MRSA and vancomycin-resistant Enterococcus (VRE) infections
have increased dramatically since the 1970s; today, nearly half of all
S. aureus SSI are caused by MRSA, and nearly 20% of Enterococcus
SSIs are caused by vancomycin-resistant strains.55 MRSA nasal colonization is a known risk factor for MRSA SSI, and MRSA carriers
are up to 10 times more likely to develop a staphylococcal infection
than those colonized with methicillin-sensitive S. aureus (MSSA)
or those who are not colonized.56,57 Similarly, individuals colonized
with VRE are at risk for VRE infections, including SSI. 58-59
MRSA SSI is associated with significantly increased mortality, length of stay, and costs.60,61 The use of vancomycin to reduce
MRSA SSIs has not been shown to reduce overall SSI rates, but it
is associated with decreased SSI rates due to MRSA which is more
difficult to eradicate (particularly in patients with prosthetic

895

implants) and is associated with adverse outcome. In a study examining vancomycin versus cefazolin for SSI prevention in cardiac
surgery patients in a single institution with a perceived high rate
of MRSA infection, 885 patients were randomized to antimicrobial
prophylaxis with cefazolin or vancomycin. There was no difference in SSI rates between the groups (9.0% cefazolin vs. 9.5% vancomycin, P = .8). However, patients who received cefazolin had a
higher MRSA SSI rate, and patients who received vancomycin had
a higher MSSA rate, documenting that the choice of antimicrobial
agent changed the flora of SSIs that occurred, but did not alter total
SSI rates.62 Other studies have confirmed similar findings.63
On the basis of these data, in 2004, an Advisory Statement
from the National Surgical Infection Prevention Project recommended the following: For patients with known MRSA colonization, vancomycin should be considered as the appropriate
antimicrobial agent for prophylaxis.64 Vancomycin use is a risk
factor for both colonization with and infection by VRE. Owing to
the emergence of VRE and vancomycin-resistance Staphylococcus
species, the routine use of vancomycin for antibiotic prophylaxis
is not recommended. However, if an institution has a high rate
of MRSA SSI or methicillin-resistant Staphylococcus epidermidis
(MRSE) SSI, vancomycin may be warranted for SSI prophylaxis.
But the threshold prevalence of MRSA at which switching from
nonglycopeptide to glycopeptide antibiotic prophylaxis might be
clinically effective and cost-effective is not known.65
Answer: Patients colonized or infected with MRSA or VRE
may require antibiotics that cover these pathogens for appropriate
antimicrobial prophylaxis for SSI prevention. If an institution has
a high rate of MRSA/MRSE SSI, vancomycin may be warranted for
SSI prophylaxis (Grade C recommendation, Level II evidence).

Clinical Question Summary


Question

Answer

Levels of
Evidence

Grade of
Recommendation

References

1 What are the risk


factors for SSI?

Patient, environmental, and treatment risk


factors all increase risk for SSI. All evidencebased preventive strategies to reduce SSI
should be implemented.

5-13

2 What are the most


common causative
pathogens in SSI?

Staphylococci are the most common


pathogens, with increasing rates of MRSA.
In abdominal procedures, SSIs are caused
roughly equally by gram-positive and gramnegative organisms. In transplant recipients,
enterococci are the most commonly isolated
pathogens and fungal SSIs are much more
common.

N/A

N/A

10-15

3 What is the optimal


timing of the initial
dose of IV antibiotics
for SSI prevention?

The first dose of preoperative antibiotics should


be given within 60 minutes prior to surgical
incision. If using vancomycin, the dose should
be given within 120 minutes of incision due
to prolonged infusion time required.

I, II

16-18

4 What is the optimal


duration of IV
antibiotics for SSI
prevention?

Antibiotic prophylaxis should terminate within


24 hours of operation. A single preoperative
dose with appropriate intraoperative
redosing is adequate in most procedures.

I, II

19-31

(Continued)

PMPH_CH113.indd 895

5/22/2012 6:18:07 PM

896

Surgery: Evidence-Based Practice

(Continued)
Question

Answer

Levels of
Evidence

Grade of
Recommendation

References

5 What dosing
modifications should
be used for obese
patients?

In obese patients (BMI > 30), increased dosing


and/or dosing interval modifications may
be required to achieve adequate antibiotic
prophylaxis. In general, cefazolin 2 g should
be used for patients greater than 80 kg, and
3g given for patients >120 kg.

II-III

34-41

6 Are prophylactic
antibiotics indicated
for SSI prevention in
all general surgery
procedures, including
clean breast and
hernia surgery?

For elective clean breast and hernia surgery,


a single dose of antibiotic prophylaxis is
recommended to reduce the risk of SSI.

I, II

42-44

7 Are prophylactic
antibiotics indicated
for SSI prevention
in low-risk elective
cholecystectomy?

Because of the limited ability to identify


patients at low risk for SSI, a single dose of
antibiotic given within 60 minutes of incision
is recommended to reduce the risk of
postoperative SSI.

I, II

45-47

8 Are oral antibiotics


necessary for
SSI prevention in
colorectal surgery?

Oral antibiotics should be used in conjunction


with MBP and preoperative IV systemic
antibiotics for elective colorectal surgery to
achieve the lowest SSI rates. Oral antibiotics
should not be used if an MBP is not used.

I, II

48-49

9 Is an MBP required for


elective colorectal
surgery?

MBP is not required, but may be useful in cases


with anticipated intraoperative colonoscopy
or rectal resections. If MBP is used, oral
antibiotics must also be administered for
SSI reduction. MBP without oral antibiotics
is associated with higher SSI rates than IV
preoperative antibiotics alone.

I, II

50-54

10 In patients colonized
or infected with
MRSA/VRE, should
IV prophylactic
antibiotics be used
to cover these
pathogens?

There is insufficient data to recommend


the routine use of IV antibiotics active
against MRSA or VRE as prophylaxis for all
procedures. Patients colonized or infected
with MRSA or VRE may require antibiotics
that cover these pathogens for appropriate
antimicrobial prophylaxis for SSI prevention.

II

55-65

SSI, surgical site infection; MRSA, methicillin-resistant Staphylococcus aureus; IV, intravenous; BMI, body-mass index; MBP, mechanical bowel preparation;
VRE, vancomycin-resistant Enterococcus.

REFERENCES
1. Auerbach AD. Prevention of surgical site infections. In: Shojania
KG, Duncan BW, McDonald KM, et al., eds. Making Health Care
Safer: A Critical Analysis of Patient Safety Practices. Evidence
Report/Technology Assessment No. 43. AHRQ Publication No.
01-E058, Rockville, MD: Agency for Healthcare Research and
Quality; 2001:221-244.
2. Bratzler DW, Houck PM. Antimicrobial prophylaxis for surgery:
an advisory statement from the national surgical infection prevention project. Clin Inf Dis. 2004;38:1706-1715.
3. Kirkland KB, Briggs JP, Trivette SL, et al. The impact of surgical
site infections in the 1990s: attributable mortality, excess length

PMPH_CH113.indd 896

of hospitalization, and extra costs. Infect Control Hosp Epidemiol. 1999;20:725-730.


4. Horan TC, Gaynes RP, Martone WJ, et al. CDC definitions of
nosocomial surgical site infections, 1992: a modification of CDC
definitions of surgical wound infections. Am J Infect Control.
1992;20:271-274.
5. Gaynes RP. Surgical site infections and the NNIS SSI Risk
Index: room for improvement. Infect Control Hosp Epidemiol.
2000;21:184-185.
6. Gaynes RP, Culver DH, Horan TC, et al. Surgical site infection
(SSI) rates in the United States, 1992-1998: the National Nosocomial Infections Surveillance System Basic SSI Risk Index. Clin
Infect Dis. 2001;33(Suppl 2):S69-S77.

5/22/2012 6:18:07 PM

SSI and Prophylactic Antibiotics

7. Krizek TJ, Robson MC. Evolution of quantitative bacteriology in


wound management. Am J Surg. 1975;130:579-584.
8. Malone DL, Genuit T, Tracy JK, Gannon C, Napolitano LM. Surgical site infections: reanalysis of risk factors. J Surg Res. 2002
Mar;103(1):89-95.
9. Mangram AJ, Horan TC, Pearson ML, et al. Guideline for prevention of surgical site infection 1999. Centers for Disease Control and Prevention (CDC) Hospital Infection Control Practices
Advisory Committee. Infect Control Hosp Epidemiol. 1999;
20:250-278.
10. Darouiche RO, Wall MJ Jr, Itani KM, et al. Chlorhexidine-alcohol versus povidone-iodine for surgical-site antisepsis. N Engl J
Med. 2010;362(1):18-26.
11. Campbell DA Jr, Henderson WG, Englesbe MJ, et al. Surgical
site infection prevention: the importance of operative duration
and blood transfusionresults of the first American College of
Surgeons-National Surgical Quality Improvement Program Best
Practices Initiative. J Am Coll Surg. 2008;207(6):810-820.
12. Kurz A, Sessler DI, Lenhardt R for the Study of Wound Infection
and Temperature Group. Perioperative normothermia to reduce
the incidence of surgical-wound infection and shorten hospitalization. N Engl J Med. 1996;334:1209-1215.
13. Latham R, Lancaster AD, Covington JF, et al. The association of
diabetes and glucose control with surgical-site infections among
cardiothoracic surgery patients. Infect Control Hosp Epidemiol.
2001;22(10):607-612.
14. Hidron AI, Edwards JR, Patel J, et al.; National Healthcare Safety
Network Team; Participating National Healthcare Safety Network
Facilities. NHSN Annual Update: antimicrobial-resistant pathogens associated with healthcare-associated infections: Annual
summary of data reported to the National Healthcare Safety Network at the Centers for Disease Control and Prevention, 2006
2007. Infect Control Hosp Epidemiol. 2008;29:996-1011.
15. Weigelt JA, Lipsky BA, Tabak UP, et al. Surgical site infections:
causative pathogens and associated outcomes. Am J Infect Control. 2010;38:112-120.
16. Classen DC, Evans RS, Pestotnik SL, Horn SD, Menlove RL,
Burke JP. The timing of prophylactic administration of antibiotics and the risk of surgical-wound infection. N Engl J Med.
1992;326(5):281-286.
17. Muoz Platn E, Jimnez Antoln JA, Brea Zubigaray S, Bravo
Garca P. [The effect of surgical antibiotic prophylaxis and the
timing of its administration on the risk of surgical wound infection.] Rev Clin Esp. 1995;195(10):669-673.
18. Steinberg JP, Braun BI, Hellinger WC, et al. Timing of antimicrobial prophylaxis and the risk of surgical site infections: results
from the trial to reduce antimicrobial prophylaxis errors. Ann
Surg. 2009;250:10-16.
19. Stewart AH, Eyers PS, Earnshaw JJ. Prevention of infection in
peripheral arterial reconstruction: a systematic review and metaanalysis. J Vasc Surg. 2007;46:148-155.
20. McDonald M, Grabsch E, Marshall C, Forbes A. Single- versus
multiple-dose antimicrobial prophylaxis for major surgery: a
systematic review. Aust N Z J Surg. 1998;68(6):388-396.
21. Simo R, French G. The use of prophylactic antibiotics in head
and neck oncological surgery. Curr Opin Otolaryngol Head Neck
Surg. 2006;14:55-61.
22. Saginur R, Odell PF, Poliquin JF. Antibiotic prophylaxis in head
and neck cancer surgery. J Otolaryngol. 1988;17:78-80.
23. Johnson JT, Myers EN, Thearle PB, et al. Antimicrobial prophylaxis for contaminated head and neck surgery. Laryngoscope.
1984;94:46-51.

PMPH_CH113.indd 897

897

24. Liu SA, Tung KC, Shiao JY, Chiu YT. Preliminary report of associated factors in wound infection after major head and neck neoplasm operationsdoes the duration of prophylactic antibiotic
matter? J Laryngol Otol. 2008;122:403-408.
25. Lipp A, Lusardi G. Systemic antimicrobial prophylaxis for percutaneous endoscopic gastrostomy. Cochrane Database Syst Rev.
2006;(4): CD005571. Updated 2008.
26. Watanabe A, Kohnoe S, Shimabukuro R, et al. Risk factors associated with surgical site infection in upper and lower gastrointestinal surgery. Surg Today. 2008;38:404-412.
27. Mui LM, Ng CS, Wong SK, et al. Optimum duration of prophylactic antibiotics in acute non-perforated appendicitis. ANZ J
Surg. 2005;75;425-428.
28. Kasatpibal N, Nrgaard M, Srensen HT, et al. Risk of surgical site infection and efficacy of antibiotic prophylaxis: a cohort
study of appendectomy patients in Thailand. BMC Infect Dis.
2006;6:111.
29. Goransson G, Nilsson-Ehle I, Olsson S, et al. Single- versus multipledose doxycycline prophylaxis in elective colorectal surgery. Acta
Chir Scand. 1984;150:245-249.
30. Juul PZ, Klaaborg KE, Kronborg O. Single or multiple doses of
metronidazole and ampicillin in elective colorectal surgery: a
randomized trial. Dis Colon Rectum. 1987;30:526-528.
31. Nelson RL, Glenny AM, Song F. Antimicrobial prophylaxis
for colorectal surgery. Cochrane Database Syst Rev. 2009;(1):
CD001181. DOI: 10.1002/14651858.CD001181.
32. Stulberg JJ, Delaney CP, Neuhauser DV, Aron DC, Fu P, Koroukian SM. Adherence to Surgical Care Improvement Project
(SCIP) measures and the association with postoperative infections. JAMA. 2010;303(24):2479-2485.
33. Dellinger EP, Hausmann SM, Bratzler DW, et al. Hospitals collaborate to decrease surgical site infections. Am J Surg. 2005;
190(1):9-15.
34. Barbour A, Schmidt S, Rout RW, Ben-David K, Burkhardt O, Derendorf H. Soft tissue penetration of cefuroxime determined by clinical mirodialysis in morbidly obese patients undergoing abdominal
surgery. Int J Antimicrobial Agents. 2009;34(3):231-235.
35. Anaya DA, Dellinger EP. The obese surgical patient: a susceptible
host for infection. Surg Infect. 2006;7(5):473-480.
36. Itani KMF, Jensen EH, Finn TS, Tomassini JE, Abramson MA.
Effect of body mass index and ertapenem versus cefotetan prophylaxis on surgical site infection in elective colorectal surgery.
Surg Infect. 2008;9(2):131-137.
37. Waisbren E, Rosen H, Bader AM, Lipsitz SR, Rogers SO Jr, Eriksson E. Percent body fat and prediction of surgical site infection.
J Am Coll Surg. 2010;210(4):381-389.
38. Edmiston CE, Krepel C, Kelly H, et al. Perioperative antibiotic
prophylaxis in the gastric bypass patient: do we achieve therapeutic levels? Surgery. 2004;136(4):738-747.
39. Koopman E, Nix DE, Erstad BL, et al. End-of-procedure cefazolin concentrations after 1105 administration for prevention of
surgical-site infection. Am J Health-Syst Pharm. 2007;64:19271934.
40. Forse RA, Karam B, MacLean LD, Christou NV. Antibiotic prophylaxis for surgery in morbidly obese patients. Surgery. 1989;
106:750-756.
41. Pai MP, Bearden DT. Antimicrobial dosing considerations in
obese adult patients: insights from the society of infectious diseases pharmacists. Pharmacotherapy. 2007;27(8):1081-1091.
42. Bunn F, Cunningham ME, Handscomb K. Prophylactic antibiotics to prevent surgical site infection after breast cancer surgery.
Cochrane Database Syst Rev. 2006;2:CD005360.

5/22/2012 6:18:07 PM

898

Surgery: Evidence-Based Practice

43. Sanchez-Manuel FJ, Lozano-Garca J, Seco-Gil JL. Antibiotic


prophylaxis for hernia repair. Cochrane Database Syst Rev.
2007;18(3):CD003769.
44. Bowater RJ, Stirling SA, Lilford RJ. Is antibiotic prophylaxis in surgery a generally effective intervention? Testing a generic hypothesis over a set of meta-analyses. Ann Surg. 2009;249(4):551-556.
45. Den Hoed PT, Boelhouwer RU, Veen HF, et al. Infections and
bacteriological data after laparoscopic and open gallbladder surgery. J Hosp Infect. 1998;39:27-37.
46. Lippert H, Gastinger J. Antimicrobial prophylaxis in laparoscopic
and conventional cholecystectomy. Chemotherapy. 1998;44:355363.
47. Zhou H, Zhang J, Wang Q, Hu Z. Meta-analysis: antibiotic prophylaxis in elective laparoscopic cholecystectomy. Aliment Pharmacol Ther. 2009;29:1086-1095.
48. Lewis RT. Oral versus systemic antibiotic prophylaxis in elective colon surgery: a randomized study and meta-analysis send a
message from the 1990s. Can J Surg. 2002;45:173-180.
49. Nelson RL, Glenny AM, Song F. Antimicrobial prophylaxis for
colorectal surgery. Cochrane Database Syst Rev. 2009;1:CD001181.
DOI: 10.1002/14651858.CD001181.
50. Guenaga KK, MatosD,Wille-Jrgensen P. Mechanical bowel preparation for elective colorectal surgery. Cochrane Database Syst
Rev. 2009;1:CD001544.
51. Slim K, Vicaut E, Launay-Savary MV, Contant C, Chipponi J.
Updated systematic review and meta-analysis of randomized
clinical trials on the role of mechanical bowel preparation before
colorectal surgery. Ann Surg. 2009;249(2):203-209.
52. Zhu QD, Zhang QY, Zeng QQ, et al. Efficacy of mechanical bowel
preparation with polyethylene glycol in prevention of postoperative complications in elective colorectal surgery: a meta-analysis.
Int J Colorectal Dis. 2010;25:267-275.
53. Nelson RL, Glenny AM, Song F. Antimicrobial prophylaxis for
colorectal surgery. Cochrane Database Syst Rev. 2009;1:CD001181.
DOI: 10.1002/14651858.CD001181.
54. Englesbe MJ, Brooks L, Kubus J, et al. A statewide assessment of
surgical site infection following colectomy: the role of oral antibiotics. Ann Surg. 2010;252(3):514-519.
55. Hidron AI, Edwards JR, Patel J, Horan TC, Sievert DM, Pollock
DA, Fridkin SK; National Healthcare Safety Network Team; Participating National Healthcare Safety Network Facilities. NHSN
annual update: antimicrobial-resistant pathogens associated
with healthcare-associated infections: annual summary of data

PMPH_CH113.indd 898

56.

57.

58.

59.

60.

61.

62.

63.

64.

65.

reported to the National Healthcare Safety Network at the Centers for Disease Control and Prevention, 2006-2007. Infect Control Hosp Epidemiol. 2008;29(11):996-1011. Erratum in: Infect
Control Hosp Epidemiol. 2009;30(1):107.
Butterly A, Schmidt U, Wiener-Kronish J. Methicillin-resistant
Staphylococcus aureus colonization, its relationship to nosocomial infection, and efficacy of control methods. Anesthesiology.
2010;113:1453-1459.
Davis KA, Stewart JJ, Crouch HK, Florez CE, Hospenthal DR:
Methicillin-resistant Staphylococcus aureus (MRSA) nares colonization at hospital admission and its effect on subsequent MRSA
infection. Clin Infect Dis. 2004;39:776-782.
Mazuski JE. Vancomycin-resistant Enterococcus: risk factors,
surveillance, infections, and treatment. Surg Inf. 2008;9(6):
567-571.
Joels CS, Matthews BD, Sigmon LB, et al. Clinical characteristics and outcomes of surgical patients with vancomycin-resistant
enterococcal infections. Am Surg. 2003;69:514-519.
Kaye KS, Engemann JJ, Mozaffari E, Carmeli Y. Reference group
choice and antibiotic resistance outcomes. Emerg Infect Dis.
2004;10:1125-1128.
Engemann JJ, Carmeli Y, Cosgrove SE, et al. Adverse clinical and
economic outcomes attributable to methicillin resistance among
patients with Staphylococcus aureus surgical site infection. Clin
Infect Dis. 2003;36(5):592-598.
Finkelstein R, Rabino G, Mashiah T, et al. Vancomycin versus
cefazolin prophylaxis for cardiac surgery in the setting of a high
prevalence of methicillin-resistant staphylococcal infections. J
Thorac Cardiovasc Surg. 2002;123(2):326-332.
Tacconelli E, Cataldo MA, Albanese A, et al. Vancomycin versus cefazolin prophylaxis for cerebrospinal shunt placement in
a hospital with a high prevalence of meticillin-resistant Staphylococcus aureus. J Hosp Infect. 2008;69(4):337-344. Epub July 7,
2008.
Bratzler DW, Houck PM, for the Surgical Infection Prevention
Guidelines Writers Workgroup. Antimicrobial prophylaxis for
surgery: an advisory statement from the National Surgical Infection Prevention Project. Clin Infect Dis. 2004;38:1706-1715. Am J
Surg. 2005;189(4):395-404.
Cranny G, Elliott R, Weatherly H, et al. A systematic review and
economic model of switching from non-glycopeptide to glycopeptide antibiotic prophylaxis for surgery. Health Technol Assess.
2008;12(1):iii-iv, xi-xii, 1-147.

5/22/2012 6:18:07 PM

Commentary on Surgical Site


Infection (SSI) and Prophylactic
Antibiotics
Timothy C. Fabian

This is an arena in which I cut my academic teeth and am pleased


the editors asked me to revisit this important clinical domain.
The authors have addressed 10 salient questions related to issues
concerning the development and prevention of surgical site infection (SSI). As opposed to most areas of clinical practice, they have
demonstrated in this evidence-based review that there is strong
evidence to support best practice. In fact, 6 of the 10 questions
have sufficient I and II levels of evidence to support Grade A recommendations! This commentary will briefly examine each of
those questions.

with mixed infections with gram-negatives being introduced in


the clean-contaminated arena. They iterate that enterococci and
fungal infections are common in the immunosuppressed transplant population.
3. What is the optimal timing of the initial dose of IV antibiotics for SSI prevention? Grade A recommendation
This question harkens back to the early days of basic and clinical
research addressing prophylactic/preventive antibiotic administration. Altemeier and Burke performed elegantly simple laboratory studies demonstrating the importance of having adequate
antibiotic levels in tissues at the time of bacterial contamination
in order to ameliorate infections.1,2 Subsequently, Stone and Polk
produced some of the earliest clinical prospective, randomized trials ever done in surgery, which confirmed those basic laboratory
experiments.3,4 Very little has been added to the principles of antibiotic prophylaxis since those seminal investigations. However,
the importance of antibiotic half-life should be stressed. Antibiotics should generally be administered at least every third half-life
in order to maintain satisfactory tissue levels. Clinical translation:
avoid short half-life agents for prophylaxis, and redose during
long operations.

1. What are risk factors for SSI? Grade A recommendation


The risk factors for surgical site infection have been developed over
the past half century, and comprise some of the most well-established
data related to SSI. Wound classification (clean, clean-contaminated, and contaminated), American Society of Anesthesiologist
score, and length of operative procedure are those factors related
to both type of operative procedure and overall patient fitness.
Patient, environmental, and treatment factors are classical areas
that the authors define with evidence as predisposing to infection.
Comorbidities and immunosuppressive states have likewise long
been appreciated as contributors to septic morbidity. Important
areas that I believe are not as widely appreciated by surgeons as
contributors to SSI, as they should be, include preparation of the
surgical site and perioperative management. Evidence concerning inadequate disinfection of instruments, skin preparation, hair
removal, perioperative transfusion, and hypothermia underscore
that relevance. Although there has been a lot of data generated
regarding contributions of hyperglycemia to morbidity in surgical patients, I believe there is more work to be done. I have some
concern regarding too tight glucose control leading to another set
of problems. Advancing technology for continuous glucose monitoring and feedback control loops are overdue.

4. What is the optimal duration of IV antibiotics for SSI prevention? Grade A recommendation
The historical studies noted above came to the same basic conclusion as the authors in this chapter: no benefit is added to prolonging prophylaxis beyond 24 hours. However, it is justifiably noted
that there really is not enough data to demonstrate that there is
no advantage to 24-hour courses compared with single dose in
complex clean-contaminated procedures (I would add trauma),
whereas there is sufficient data to demonstrate no advantage
beyond 24 hours. They also importantly highlight that singledose aerobic and anaerobic gram-negative prophylaxis is appropriate for uncomplicated appendicitis. The authors also succinctly
address and define the Surgical Care Improvement Project (SCIP)
criteria that are being applied for surgical grade cards and reimbursement patterns (pay for performance).

2. What are the most common causative pathogens in SSI? N/A


recommendation
No surprises here. The authors note that gram-positive organisms, primarily staphylococci, are the predominant offenders

899

PMPH_CH113.indd 899

5/22/2012 6:18:07 PM

900

Surgery: Evidence-Based Practice

5. What dosing modifications should be used for obese patients?


Grade B recommendation
Given the national epidemic of obesity, it is unfortunate that there
is insufficient data for a Grade A recommendationfertile ground
for research. Although sufficient data are available to show that
morbidly obese patients are at increased SSI risk, the pathophysiology is unclear. That lack of clarity makes it difficult to provide
conclusive strategies for infection prevention. The authors recommend cephalosporins, due to their low cost and safety profi le, at
double dosage for patients >80 kg and at triple dosage for those
>120 kga recommendation that appears appropriate. Future
research addressing pharmacokinetics, pharmacodynamics, and
antibiotic classes is clearly justified.
6. Are prophylactic antibiotics indicated for SSI prevention in
all general surgery procedures, including clean breast and hernia surgery? Grade A recommendation
Yes! This is one of the most important and controversial questions that the authors have addressed. Traditionally, clean procedures have not been recommended for antibiotic prophylaxis,
as the conventional wisdom was that most infections arose from
colonizing bacteria from the contaminated portions of cleancontaminated procedures. Current research including randomized controlled trials and meta-analyses clearly demonstrate a
reduction in SSI with single-dose prophylaxis for breast surgery
and herniorrhaphy with or without adjunctive mesh. This has significant impact due to the tremendous number of these elective
procedures. It should be noted that there are no data to support
prophylaxis in other elective general surgical procedures.
7. Are prophylactic antibiotics indicated for SSI prevention in
low-risk elective cholecystectomy? Grade B recommendation
Existing data suggest that antibiotic prophylaxis is of no benefit
for uncomplicated cholecystectomy. Although in the setting of
complicated cholecystectomy (cholecystitis, pancreatitis, diabetes,
immunosuppression, laparoscopic conversion, and intraoperative
gallbladder rupture), prophylactic antibiotics appear efficacious.
As we are now in the era of laparoscopic cholecystectomy, when
bile spillage and conversion are not easily predicted, the authors
have chosen a pragmatic approach to answer the question with a
Grade B recommendation: a single dose of antibiotic given within
60 minutes of incision is recommended.

not used. That leads us into the next question. But dont get your
hopes up regarding a definitive answer.
9. Is an MBP required for elective colorectal surgery? Does MBP
reduce SSI in colorectal surgery? Grade A recommendation
Those pesky meta-analyses! They include nonhomogenous study
populations, without defining important study parameters: MBP
without oral antibiotics included, no mention of whether oral antibiotics are included, inappropriate parenteral antibiotic utilization,
or colon versus rectal resection. The authors recommendations
are the least satisfying of all their recommendationharkening
back to King Solomon: MBP is not required, but if it is used oral
antibiotics are also necessary. I understand the authors dilemma.
The research trail is difficult to follow in an evidence-based fashion, especially in the flux of the healthcare delivery system. Bottom line: If MBP is used, oral antibiotics combined with systemic
antibiotics will reduce SSI.
10. In patients colonized or infected with MRSA or VRE,
should IV prophylactic antibiotics be used to cover these pathogens? Grade C recommendation
The authors note both the rising incidence of methicillin-resistant Staphylococcus aureus (MRSA) and vancomycin-resistant
Enterococcus (VRE) infections, and that colonization with those
organisms are associated with infection risks. What is unknown
is whether prophylaxis in individuals colonized reduces the risk,
although conventional wisdom might suggest that to be the case. It
is also unknown whether institutions with high MRSA infection/
colonization rates should routinely use vancomycin prophylaxis.
That is a double-edged sword due to the potential for increasing
VRE in the face of routine vancomycin usage. There are ongoing
prospective studies of MRSA colonization patterns. Perhaps data
will be produced that will provide some guidance for antibiotic prophylaxis. In the meantime, vancomycin should probably be used
for major elective procedures for patients with known colonization
or in institutions with high incidence of MRSA infection rates.
In summary, the authors provide a well-executed evidencebased analysis of many common issues related to SSI and prevention of such. Their recommendations are justified by the currently
available evidence that provide us with practical guidelines for
patient management. I thoroughly enjoyed this chapter and learnt
a lot from it.

8. Are oral antibiotics necessary for SSI prevention in colorectal surgery? Grade A recommendation

REFERENCES

Questions related to oral and parenteral antibiotic prophylaxis


for elective colorectal surgery have been a source of obfuscation
for many of us, especially in recent years. Life has become more
complex since the landmark study of oral prophylaxis by Nichols and Condon seemed to resolve the issue.5 Most operations are
performed as same day procedures and there is a lack of control of
mechanical bowel preparation (MBP) and patient compliance with
such. Of course, that is also confounded by the fact that the value
of mechanical preparation is questioned (Question 9). I thank the
authors for pointing me in the right direction. Unfortunately, they
had to take a King Solomon approach due to the annoying issue
of mechanical preparation: Oral antibiotics should be used with
MBP and parenteral antibioticsbut not if mechanical prep is

1. Altemeier WA, Culbertson WR, Vetto M. Prophylactic antibiotic


therapy. Arch Surg. 1955;71:2.
2. Burke JF. The effective period of preventive antibiotic action in
experimental incisions and dermal lesions. Surgery. 1961;50:161.
3. Stone HH, Haney BB, Kolb LA, Geheber CE, Hooper CA. Prophylactic and preventive antibiotic therapy: timing, duration and
economics. Ann Surg. 1979;89(6):691-698.
4. Polk HC Jr., Lopez-Mayor JF. Postoperative wound infection: a
prospective study of determinant factors and prevention. Surgery.
1969;66:97.
5. Clarke JS, Condon RE, Bartlett JG, Gorbach SL, Nichols RL, Ochi
S. Preoperative oral antibiotics reduce septic complications of
colon operations: results of prospective, random, double-blind
clinical study. Ann Surg. 177;186(3):251-259.

PMPH_CH113.indd 900

5/22/2012 6:18:07 PM

CHAPTER
CHAPTER114
1

Management of Acute
Myocardial Infarction and
Cardiogenic Shock
Antonio Hernandez

INTRODUCTION

ratio in order to meet metabolic demands. Unfortunately, at a


resting state the heart extracts approximately 50% to 75% of oxygen, and thus the only way to increase the delivery of oxygen to
the myocardial tissue is by increasing blood flow. The basis of this
physiology makes it likely to develop demand/supply mismatch
and consequent ischemia. If the area of ischemia is not reperfused
quickly, the ischemic tissue progresses to myocardial necrosis.
Etiologies for MI vary, but all essentially have the same final
pathwayincreased demand for oxygen with inadequate supply.
Coronary vasospasm is less frequent, and in the trauma and/or
aortic dissection patient, coronary artery injury may result in
ischemia. Nevertheless, rupture of a plaque and an embolic event
is the most common etiology for MI.

Cardiovascular disease remains to be the leading cause of death


in the United States. According to the American Heart Association, cardiovascular disease (CVD) mortality accounted for one
of every three deaths in the United States in 2007, with one of
every six deaths from coronary heart disease.1 An estimated
785,000 Americans will have a new coronary attack, and approximately 470,00 will have a recurrent attack. Approximately every
25 seconds, an American will experience a coronary event and each
minute someone will die from one.1 Also, from 1997 to 2007, the
death rate due to CVD declined from 27.8% to 14.2%,1 which is
quite an improvement but remains high. The focus in this chapter
will be related to the incidence of myocardial infarction (MI) and
consequent cardiogenic shock in perioperative patients. Although
there is good evidence for the use of thrombolytic therapy in STelevation MI, on the basis of the 2007 focused update of the American College of Cardiology/American Heart Association (ACC/
AHA),2 thrombolytic therapy is often contraindicated in the postoperative population. Hence, therapies suggested by the ACC/AHA
for ST-elevation and non-ST-elevation MI3 and how they apply
tothe postoperative population are reviewed in the chapter. The
evidence related to MI with respect to the etiology and pathophysiology, diagnosis, and management are also discussed. At the conclusion of the chapter, the current evidence-based strategies for the
management of cardiogenic shock are reviewed.

DIAGNOSIS
In evaluating a patient for myocardial ischemia or infarction,
the clinician must have a high index of clinical suspicion, a low
threshold for interrogating the disease process further, and
remain vigilant for the confirmation or exclusion of the disease
process while monitoring for signs and symptoms of acute disease progression. In the nonintubated or nonanesthetized patient,
clinical presentation is very helpful. However, in the perioperative
patient population that has experienced surgery, or remains under
the influence of anesthetic and analgesic agents or remains intubated on a mechanical ventilator, the value of signs and symptoms
do not contribute as strongly in alerting the physician about ongoing ischemic or myocardial injury. As a result, the perioperative
physician must rely strongly on monitors and biomarkers when
evaluating a patient for a myocardial event. As such monitors of
choice include ECG, echocardiography, and to a less extent pulmonary artery catheter. Biomarkers include creatine phosphokinase (CPK), creatine phosphokinase myocardial band (CKMB),
and troponin levels.

PATHOPHYSIOLOGY AND ETIOLOGY


Unlike the rest of the organ systems in the body, the hearts oxygen
extraction ratio is significantly higher. The brain, which is metabolically expensive, only extracts 25% of the oxygen at a resting state. In
conditions of impaired perfusion, the brain has the luxury of maintaining cerebral metabolism by increasing the oxygen extraction
901

PMPH_CH114.indd 901

5/22/2012 6:18:38 PM

902

Surgery: Evidence-Based Practice

The typical presentation of an ECG for an acute MI include


ST-segment elevation in two consecutive or contiguous leads
that represent the same coronary artery territory, and not how they
appear in sequence on the ECG. ST elevation should be 1 mm
or 0.1 mV. Other ECG findings include inversion of T waves and
finally the development of a Q wave. The area that may be elusive on an ECG includes the inferior lateral wall. The presentation
here would include increased voltage over the R waves, peaked T
waves, and ST depression on leads V1-V2. Also, the development of
a new left bundle branch block should be approached as an acute
MI and managed as such, until biomarker data exclude the likelihood of an acute event.
The biomarkers listed above follow a pattern of progression
that often makes it challenging to assess for reinfarction. Upon
the onset of an MI, CPK and CKMB serum levels begin to rise at
4 to 8 hours, peak at approximately 18 hours, and return to baseline after 2 to 4 days. Troponin levels are more specific, and it is the
better biomarker to follow. Troponin serum levels rise at 6 hours,
and may remain elevated for a few days, particularly if the patient
has coexisting renal insufficiency. Specific levels of biomarkers
should be referenced with the specific institutional standards as
they may vary.
Finally, the role of echocardiography is an excellent complement to the methods described. Echocardiography provides information about specific segments of each wall. Previously we used
a 16-segment model for interpretation of systolic function,4 but
we have recently adopted a 17-segment model for the left ventricle
(LV), septum, and true apex. The transthoracic approach is noninvasive and essentially harmless to the patient. In the hands of
a skilled echocardiographer, hemodynamic parameters can be
estimated to include cardiac output, pulmonary artery systolic
pressure, and mean arterial pressure, and valvular structural
abnormalities can be estimated to include endocarditis; it also
provides information about cardiomyopathy patterns and pericardial integrity and estimation of pericardial volume. Although
the pulmonary artery catheter was considered the gold standard
in previous years, invasive monitoring is becoming less utilized.
Use of a pulmonary artery catheter may complement the techniques described above.

MANAGEMENT
The following section involves the discussion of strategies for the
management of an acute MI. Owing to recent contribution from
the ACC/AHA, the management of ST-elevation and non-STelevation MI independently are specifically addressed. Finally, the
strategies on cardiogenic shock as a consequence of an acute MI
are discussed.
Generally, the goals for both LV and right ventricle (RV)
acute MI are to reduce the oxygen consumption and increase the
delivery of oxygen to the myocardium. In the following two sections, the management for ST-elevation MI and non-ST-elevation
MI independently as per the ACC/AHA guidelines are discussed.
Overall, the goal is to establish revascularization.

Thrombolytic Therapy
There is good evidence for the use of thrombolytic therapy for
revascularization during an acute MI.5 However, in postoperative
patients, the use of thrombolytic therapy is often contraindicated

PMPH_CH114.indd 902

due to recent surgery. Nevertheless, a list of contraindications are


given below:

Active internal bleeding.


Intracranial neoplasm, aneurysm, or A-V malformation.
Neurosurgery or cerebral vascular accident within 6 weeks.
Trauma or major surgery within 2 weeks.
Aortic dissection.

1. What is the optimal time from door to percutaneous coronary intervention that reduces mortality?

ST-Elevation MI
The following strategies for management of an acute MI will be
based on current guidelines forwarded by the ACC/AHA for STelevation MI.2 The current recommendation is to establish reperfusion as quickly as possible. For patients experiencing a coronary
event, the health system goal should be to have the patient receive
an intervention within 90 minutes, from arrival to balloon
time.6 For our particular patient population, access to a percutaneous coronary intervention (PCI) laboratory should be quite
easy. It has been demonstrated that outcomes are better when
patients are cared for in centers of high-volume PCI experience.7
In a study by Nallamothu et al.7 in which they evaluated 37,233
patients from 463 hospitals, the investigators identified that transfer to a center with PCI specialization and a door to balloon time
less than 90 minutes (median time 99.6 vs. 118.3 minutes; P < .001)
resulted in improved survival (relative risk, 0.78; P = .001). More
recently, the CARESS-in-AMI trial8 and the Transfer-AMI study 9
provided further evidence that rapid triage to a center with angioplasty capability results in improved survival, even in patients
who have received thrombolytic therapy. In the CARESS-in-AMI
study, 600 patients with ST-elevation MI were randomized to
immediate transfer for PCI or to standard treatment with transfer
for rescue PCI if needed. The standard-of-care treatment included
half-dose reteplase, abciximab, heparin, and ASA within 12 hours
of symptom onset. The primary outcome which consisted of composite all-cause mortality, reinfarction, and refractory myocardial
ischemia within 30 days occurred significantly less often in the
immediate PCI group than in the standard-of-care/rescue PCI
group (4.4% vs. 10.7%; P = .004). There were no significant differences in the rate of bleeding or stroke between the groups. There
was a shorter median time from fibrinolytic therapy to transfer to
a PCI-capable center in the immediate versus standard-of-care/
rescue PCI group (110 vs. 180 minutes; P = .0001). The TransferAMI study, further substantiated these findings in their 1059patient study. Patients were randomized to a pharmacoinvasive
strategy (immediate transfer for PCI within 6 hours of fibrinolytic
therapy) or to standard-of-care after fibrinolytic therapy (which
included rescue PCI for ongoing chest pain and <50% resolution
of ST elevation at 60 to 90 minutes or hemodynamic instability).
The primary endpoint (30-day composite occurrence of death,
reinfarction, recurrent ischemia, now or worsening heart failure,
and cardiogenic shock) of 11.0% occurred in the pharmacoinvasive group compared to 17.2% in the standard-of-care group
(P = .004). Time to thrombolytic administration was approximately 2 hours in both the groups. However, median time from
administration of thrombolytic therapy to catheterization was
2.8 hours in the pharmacoinvasive group versus 32.5 hours in the
standard-of-care group. Both of these studies support the theory
that the earlier the PCI is done, the better the outcomes.

5/22/2012 6:18:38 PM

Management of Acute MI and Cardiogenic Shock

Answer: Ninety minutes is the optimal time from door to PCI.


Patients who received an intervention within this timeframe experienced a reduction in mortality (Grade A recommendation).
2. What -blocker is recommended for management of an
acute MI, and is an intravenous dose superior?

-Blocker Therapy
According to the updated guidelines by the ACC/AHA, oral
-blocker therapy should be initiated in the first 24 hours for
patients who do not have the following: (1) signs of heart failure,
(2) evidence of a low output state, (3) increased risk for cardiogenic shock, or (4) other relative contraindications to -blockade
(heart block, asthma, or reactive airway disease).2 In previous
studies, intravenous (IV) -blocker therapy had not shown to be
superior to the oral route of administration with the exception
of IV atenolol.10 A post hoc analysis of atenolol use identified no
significant change in mortality.11 Current Class I Level A recommendations include the initiation of oral -blocker (metoprolol)
therapy unless contraindicated within 24 hours of the acute event,
and is beneficial for secondary prevention and related complications.12 The oral dose can be titrated to achieve rate control, and
vigilance must be maintained to monitor for plausible complication from -blocker therapy.
Answer: The current ACC/AHA recommendations suggest
that an oral dose of -blocker therapy (metoprolol) is the optimal
treatment. On review of IV -blocker therapy, mortality is not
affected when compared to oral -blocker therapy that improved
mortality. In the absence of contraindications, metoprolol via oral
route of administration is superior over IV -blocker therapy.
Early initiation of low-dose -blocker therapy and careful titration should be the goal until rate control has been achieved (Grade
A recommendation).
3. Is a baby aspirin (81 mg) adequate for management of an
acute MI?

Antiplatelet Therapy
Aspirin 162 to 325 mg should be initiated on all patients suspected
of experiencing an acute MI unless contraindicated. The use of
aspirin alone reduces the incidence of reinfarction and mortality
by 23% without any other adjuncts.13
Answer: A minimum of 162 mg of aspirin should be administered within 10 minutes of recognizing that the patient is experiencing an acute MI (Grade A recommendation).
4. Is clopidogrel indicated in the management of an acute MI?
The efficacy of thienopyridines in the management of ST-elevation
MI, clopidogrel primarily, has been tested in two large trials since
the 2004 ACC/AHA guidelines publication. The COMMIT-CCS-2
that included 45,852 patients who received 75 mg of clopidogrel
daily in addition to a daily dose of 162 mg of aspirin. This trial
achieved an endpoint of all-cause mortality reduction from 8.1% in
the placebo group to 7.5% in the clopidogrel group (P = .03), and the
rate of cerebral and major noncerebral bleeding was 0.55% in the
placebo group and 0.58% in the clopidogrel group (P = .59).14 The
other trial was the CLARITY-TIMI 28 that included clopidogrel
added to thrombolytic therapy. This study has not been elaborated
in this chapter as it is not relevant to the postsurgical population,

PMPH_CH114.indd 903

903

but it has to be commented that there was an improvement of the


endpoint, which was an occluded infarct artery on angiography or
death or recurrent MI before angiography. This was reduced from
21.7% in the placebo group versus 15.0% in the clopidogrel group.15
Suffice to say that clopidogrel is considered a Class I Level A recommendation for adjuvant therapy for ST-elevation MI. Nevertheless,
in the postsurgical and trauma patient population, thienopyridines
should be used with caution.
Answer: Adding clopidogrel to aspirin does improve outcome
in non-ST-elevation MI and should be considered. However, caution should be taken when used in postoperative patients (Grade
A recommendation).
Glycoprotein IIb/IIIa receptor antagonists are used in conjunction with PCI and have no role in independent use as an adjuvant, without the involvement of a PCI specialist.

Anticoagulants
Administration of unfractionated heparin is often administered
on a weight-based protocol to include a bolus of 60 units (U)/
kilogram (kg) up to a maximum of 4000 U and an initial infusion rate of 12 U/kg/h with a goal to keep the partial thromboplastin time between 50 and 70 seconds. Unfractionated
heparin currently holds a Class IIa Level B recommendation.
There is increasing evidence that low-molecular weight heparin
(LMWH) is at least as efficacious as unfractionated heparin, but
according to the recent 2007 update by the ACC/AHA, LMWH
is listed as a Class IIa Level c recommendation. Both of these
are beneficial in patients without thrombolytic therapy, which is
representative of our patient population.

Nitrates
Although not addressed by current guidelines, the use of nitrates
continues to be the standard of practice. In particular, nitroglycerin (NTG) transdermal, sublingual, or via infusion therapy is
frequently initiated to aid in improving angina and perfusion to
the injured myocardium until direct revascularization is implemented. It should be noted that there is a lack of evidence to demonstrate an improvement in mortality from nitrates. Recently, the
GISSI-3 trial compared angiotensin converting enzyme inhibitor
(ACE-I) versus transdermal NTG versus ACE-I with transdermal
NTG versus placebo. All patients received aspirin, IV and oral
-blocker therapy, and thrombolytic therapy. The result demonstrated a benefit from the use of lisinopril, regardless of whether
transdermal NTG was added.16 When used IV, the dose of nitroglycerin is either 0.25 to 0.5 mcg/kg/min or 10 mcg/min and
titrated to effect as long as the patient is hemodynamically appropriate. Remember to use appropriate tubing to avoid chelating of
nitroglycerin before entering the patient.

Angiotensin Converting Enzyme Inhibitors


5. What ACE-I is indicated for the management of an acute MI,
and is an IV dose superior?
6. Does the addition of an angiotensin receptor blocker improve
the benefit of an ACE-I?
Several trials have demonstrated the benefit of initiating ACE-I
therapy as soon as the patient tolerates its use. The SAVE trial17
demonstrated an improvement in mortality just over 20%, and the

5/22/2012 6:18:38 PM

904

Surgery: Evidence-Based Practice

HOPE trial18 improved survival related to cardiac events as well as


a reduction in stroke. ACE-I should be initiated within 24 hours
if tolerated, and like -blockers, the oral route of administration
results in improved outcomes. The recommendation is to initiate
a low dose and titrate the dose as tolerated. For those patients who
do not tolerate ACE-I due to the adverse reactions, similar benefits have been noted with angiotensin receptor blockers (ARBs).
The VALIANT trial assessed the effect of captopril, valsartan, and
the combination of both.19 It was noted that both captopril and
valsartan were as effective, but when used together the risks of an
adverse effect outweighed the benefit to the patient.
Answer: The current ACC/AHA recommendations suggest
that an oral dose of ACE-I therapy is the optimal treatment. On
review of IV ACE-I therapy, mortality is not affected. However, oral
ACE-I therapy did improved mortality. The following is related to
oral ACE-I therapy (Grade A recommendation for Left ventricular
ejection fraction (LVEF) <40%. For low-risk patients >40% LVEF, it
is Grade B recommendation).
Answer: Addition of an ARB does not improve the outcome
of the patient, and was noted to introduce more adverse effects.
ARBs are as effective as ACE-I and can be used when ACE-I is
not tolerated by the patient due to adverse reactions (Grade A
recommendation).

Non-ST-Elevation MI
The key for non-ST-elevation MI is to establish reperfusion by
thrombolytic therapy if PCI is not available.

-Blocker Therapy
Like with ST-elevation MI, the use of oral -blockers are more
advantageous than IV -blockers. The recommendation is Class
I Level B to initiate within 24 hours of the acute coronary event,
as long at the patient does not have contraindications as listed in
the ST-elevation section above.20 Initiate a low-dose therapy and
titrate to achieve rate control.

Antiplatelet Therapy
As before, the use of aspirin is invaluable and should be initiated
within 10 minutes of identifying signs and symptoms of an acute
MI, unless contraindicated. The patient should continue to receive
this therapy as it not only reduces mortality in this group by nearly
50% but also reduces reinfarction.
The addition of clopidogrel to this group of 12,562 patients in
the CURE trial demonstrated a benefit in mortality, myocardial
infarction, and stroke with only 1% risk of major nonlife threatening bleeds (P = .001).21 The current guidelines recommend either/
or clopidogrel 300 mg load or a GP IIb/IIIa load if an early intervention strategy is anticipated, and this is Class I Level A recommendation.20 Again, clopidogrel should be used with caution in
the postoperative population.

Anticoagulants
Unfractionated heparin as well as LMWH remains a Class I
Level A recommendation. In this patient population, based on
the ESSENCE trial that included 22,000 patients, the researchers noted a statistically significant reduction in the combined

PMPH_CH114.indd 904

endpoint of death or nonfatal MI at 30 days for enoxaparin versus unfractionated heparin in the overall trial populations (10.1%
vs. 11.0%; OR, 0.91; 95% CI, 0.830.99; number needed to treat,
107).22 In the TIMI 11B trial, LMWH demonstrated to be superior
to unfractionated heparin without increased risk of bleeding. 23
The dose for LMWH is 1 mg/kg every 12 hours subcutaneous. The
unfractionated heparin dose is the same as described in the STelevation section above.

Nitrates
Like with ST-elevation MI, there is no evidence that nitrates will
improve outcome, but they are helpful in managing the patients
symptoms of angina. Initiate at the same doses described above,
and titrate to effect as long as the patient tolerates its use. Avoid
use in patients with a systolic blood pressure below 90 mm Hg.

Cardiogenic Shock
Cardiogenic shock is one of the complications from an acute MI.
Management strategies vary, and there is no evidence to clearly
guide our choice in agents with improved outcome in a large,
multicenter trial. However, both Dobutamine and Milrinone have
demonstrated improved cardiac index with their use in LV failure, but there is no conclusive evidence of improved outcome in
the setting of acute MI. There is increasing evidence for the use
of sildenafil for RV failure, but its role in RV failure in the setting
of acute MI is limited. There is limited evidence that supports the
efficacy of sildenafil as a good agent to reduce pulmonary vascular
resistance, while decreasing Left ventricular end diastolic pressure
(LVEDP) and improving cardiac index.24 RV failure as a result of
pulmonary hypertension will respond well to sildenafil and actually
improves the quality of life after 12 weeks25 and after 6 months.26
Since nitrates are the standard of therapy in an acute MI, sildenafil
should likely be avoided until more evidence is available.
Intra-Aortic Balloon counterpulsation (IABCP) in acute MI
has been used for nearly 30 years. Unfortunately, there are limited data to determine if its use impacts mortality, even though a
study was conducted in which data were collected prospectively
including 250 medical centers worldwide and 5495 patients with
acute MI and IABCP.27 Nevertheless, it is a plausible strategy to
augment cardiac index but primarily alleviate the LV from added
work during an ischemic event.

CONCLUSION
Diagnosis of an MI in the postoperative patient requires that the
clinician has a high index of suspicion, as often this patient population is sedated and intubated or under the influence of analgesic therapy. Thus reliance on monitors and biomarkers is key, and
rapid implementation of a plan to establish reperfusion and antiischemic therapy is essential. Overall, the oral route of administration for both -blocker therapy and ACE-I therapy is more
efficacious, even when low-dose therapy is initiated and titrated
carefully. Antiplatelet therapy is of upmost importance regardless
of whether the patient will receive thrombolytic therapy. Aspirin
(162325 mg) should be initiated and continued indefinitely unless
contraindicated. Anticoagulation with unfractionated heparin
has been the standard for some time and presently receives the

5/22/2012 6:18:38 PM

Management of Acute MI and Cardiogenic Shock

most evidence for its use, but emerging data support superiority
of LMWH over unfractionated heparin. Regardless of this, anticoagulation is an adjuvant to antiplatelet therapy in establishing
reperfusion and reducing the risk of restenosis in the acute phase.
Finally, implementation of a plan that is easily reproducible and

905

communicated is key to successful delivery of evidence-based


care. Development of protocol- or algorithm-driven therapy is
the key to eliminating deviation from evidence-based practice
in a setting of a variety of practitioners from different training
backgrounds.

Clinical Question Summary


Question

Answer

1 What is the optimal time


from door to PCI that
reduces mortality?

Ninety minutes is the optimal time from door to PCI. Patients


who received an intervention within this time frame
experienced a reduction in mortality.

2, 6

2 What -blocker is
recommended for
management of an
acute MI, and is an
intravenous dose
superior?

The current ACC/AHA recommendations suggest that an


oral dose of -blocker therapy (metoprolol) is the optimal
treatment. On review of IV -blocker therapy, mortality is
not affected when compared to oral -blocker therapy, which
improved mortality.

2, 9, 10

3 What ACE-I is indicated


for the management of
an acute MI, and is an
IV dose superior?

The current ACC/AHA recommendations suggest that an oral


dose of ACE-I therapy is the optimal treatment. On review
of IV ACE-I therapy, mortality is not affected when compared
to oral ACE-I therapy, which improved mortality. For LVEF
<40%, it is a Class I Level A recommendation. For low-risk
patients >40% LVEF, it is IIa Level B recommendation.

A for LVEF <40%


B for LVEF >40%

4 Does the addition of an


angiotensin receptor
blocker improve the
benefit of an ACE-I?

Addition of an ARB does not improve the outcome of the


patient, and was noted to introduce more adverse effects.
ARBs are as effective as ACE-I and can be used when ACE-I is
not tolerated by the patient due to adverse reactions.

2, 17

5 Is a baby aspirin (81


mg) adequate for
management of an
acute MI?

A minimum of 162 mg of aspirin should be administered within


10 minutes of recognizing that the patient is experiencing an
acute MI.

6 Is clopidogrel indicated
in the management of
an acute MI?

Adding clopidogrel to aspirin does improve outcome in nonST-elevation MI and should be considered. However, caution
should be taken when used in postoperative patients.

2, 19

REFERENCES
1. American Heart Association. 2001 Heart and stroke statistical
update. Dallas, Texas: American Heart Association, 2000. Available
at http://www.americanheart.org/statistics/index.html. Accessed
February 2001.
2. JACC. 2007 Focused update of the ACC/AHA 2004 guidelines
for management of patients with ST-elevation myocardial infarction. 2008;51(2):210-247.
3. Annals of Emergency Medicine. 2007 Update to the ACC/AHA
Guidelines for the Management of Patients With Unstable Angina
and Non-ST-Segment Elevation Myocardial Infarction: Implications for Emergency Department Practice. 51(5):591-606.
4. Shanewise JS, Cheung AT, Aronson S, et al. ASE/SCA guidelines for performing a comprehensive intraoperative multiplane
transesophageal echocardiography examination: recommendations of the American Society of Echocardiography Council
for Intraoperative Echocardiography and the Society of Cardiovascular Anesthesiologists Task Force for Certification in
Perioperative Transesophageal Echocardiography. Anesthesia
Analgesia. 1999; 89(4):870-884.

PMPH_CH114.indd 905

Grade

References

5. The Gusto Investigators. An international randomized trial comparing four thrombolytic strategies for acute myocardial infarction. NEJM. 329(10):673-682.
6. Khot UN, Johnson ML, et al. Emergency department physician activation of the catheterization laboratory and immediate
transfer to an immediately available catheterization laboratory
reduce door-to-balloon time in st-elevation myocardial infarction. Circulation. 2007;116(3):67-76.
7. Nallamothu BK, Wang Y, Magid DJ, et al. Relation between hospital specialization with primary percutaneous coronary intervention and clinical outcomes in ST-segment elevation myocardial
infarction: National Registry of Myocardial Infarction-4 analysis. Circulation. 2006;113:222-229.
8. Cantor WJ, Fitchett D, Borgundvaag B, et al. Routine early
angioplasty after fibrinolysis for acute myocardial infarction. N
Engl J Med. 2009;360:2705-2718.
9. Di Mario C, Dudek D, Piscione F, et al. Immediate angioplasty
versus standard therapy with rescue angioplasty after thrombolysis in the Combined Abciximab REteplase Stent Study in Acute
Myocardial Infarction (CARESS-in-AMI): an open, prospective,
randomized, multicentre trial. Lancet. 2008;371:559-568.

5/22/2012 6:18:38 PM

906

Surgery: Evidence-Based Practice

10. Pfisterer M, Cox JL, Granger CB, et al. Atenolol use and clinical outcomes after thrombolysis for acute myocardial infarction:
the (ateplase) for occluded coronary arteries. JACC. 1998;32:
634-640.
11. Antman EM, Anbe DT, Armstrong PW, et al. ACC/AHA guidelines for the management of patients with ST-elevation myocardial infarction: a report of the American College of Cardiology/
American Heart Association Task Force on Practice Guidelines
(Committee to Revise the 1999 Guidelines for the Management of patients with acute myocardial infarction). JACC. 2004;
44:e1-211.
12. Lopez-Sendon J, Swedberg K, McMurray J, et al. Expert consensus document on beta-adrenergic receptor blockers. Eur Heart J.
2004;25:1341-1362.
13. ISIS-2 Collaborative Group. Randomized trial of intravenous
streptokinase, oral aspirin, both, or neither among 17,187
cases of suspected acute myocardial infarction: ISIS-2. Lancet.
1988;2:349-360.
14. Chen ZM, Jiang LX, Chen YP, et al. Addition of Clopidogrel t
aspirin in 45,852 patients with acute myocardial infarction: randomized placebo-controlled trial. Lancet. 2005;366;1607-1621.
15. Scirica BM, Sabatine MS, Morrow DA, et al. The role of clopidogrel in early and sustained arterial patency after fibrinolysis for ST-segment elevation myocardial infarction: the ECG
CLARITY-TIMI 28 study. JACC. 2006;48:37-42.
16. GISSI-3 Investigators. Causes of death in patients with acute
myocardial infarction treated with angiotensin-converting
enzyme inhibitors: findings from the Gruppo Italiano per lo Studio della Sopravvivenza nellInfarto (GISSI)3 trial. Am Heart J.
2008;155:388-394.
17. Pfeffer MA, Braunwald E, Moye LA, et al. Effect of captopril on
mortality and morbidity in patients with left ventricular dysfunction after f infarction. Results of the survival and ventricular enlargement trial. The SAVE Investigators. N Engl J Med.
1992;327:669-677.
18. Yusuf S, Sleight P, Pogue J, et al. Effects of an angiotensinconverting-enzyme inhibitor, ramipril, on cardiovascular events
in high-risk patients. The Heart Outcomes Prevention Evaluation Study Investigators. N Engl J Med. 2000;342:145-153.
19. Pfeffer MA, McMurray JJ, Velazquez EJ, et al. Valsartan, captopril, or both in myocardial infarction complicated by heart

PMPH_CH114.indd 906

20.

21.

22.

23.

24.

25.
26.

27.

failure, left ventricular dysfunction, or both. N Engl J Med. 2003;


349:1893-1906.
Anderson JL, Adams CD, Antman EM, et al. ACC/AHA 2007
guidelines for the management of patients with unstable angina/
nonST-elevation myocardial infarction: a report of the American College of Cardiology/American Heart Association Task
Force on Practice Guidelines (Writing Committee to Revise the
2002 Guidelines for the Management of Patients With Unstable
Angina/NonST-Elevation Myocardial Infarction): developed
in collaboration with the American College of Emergency Physicians, American College of Physicians, Society for Academic
Emergency Medicine, Society for Cardiovascular Angiography
and Interventions, and Society of Thoracic Surgeons. J Am Coll
Cardiol. 2007;50:e1-157.
Yusuf S, Zhao F, Mehta SR, Chrolavicius S, Tognoni G, Fox KK.
Effects of clopidogrel in addition to aspirin in patients with
acute coronary syndromes without ST-segment elevation. N Engl
J Med. 2001;345:494-502.
Petersen JL, Califf RM, et al. Efficacy and bleeding complications
among patients randomized to enoxaparin or unfractionated
heparin for antithrombin therapy in nonST-segment elevation acute coronary syndromes. a systematic overview. JAMA.
2004;292:89-96.
Antman EM, McCabe CH, Braunwald E, TIMI 11B investigators. Enoxaparin prevents death and cardiac ischemic events in
unstable angina/nonQ-wave myocardial infarction results of
the Thrombolysis In Myocardial Infarction (TIMI) 11B Trial.
Circulation. 1999;100:1593-1601.
Michelakis E, Tymchak W, Archer S, et al. Oral sildenafi l is an
effective and specific pulmonary vasodilator in patients with
pulmonary arterial hypertension. Comparison with inhaled
nitric oxide. Circulation. 2002;105:2398-2403.
SUPER Study Group. Sildenafi l Citrate Therapy for Arterial Pulmonary Hypertension. N Engl Med. 2005;353:2148-2157.
Pepke-Zaba J, Gilbert C, Collings L, Brown MCJ. Sildenafi l
improves health-related quality of life in patients with pulmonary arterial hypertension. Chest. 2008;133:183-189.
Stone GW, Ohman EM, Ferguson JJ III, et al. Contemporary
utilization and outcomes of intra-aortic balloon counterpulsation in acute myocardial infarction. JACC. 2003;41(11):
1940-1945.

5/22/2012 6:18:38 PM

Commentary on
Management of Acute
Myocardial Infarction and
Cardiogenic Shock
Kenneth Waxman

The chapter Management of Acute Myocardial Infarction and


Cardiogenic Shock by Dr Hernandez provides an excellent general review of the current management of myocardial infarction.
However, important questions remain about the specific management of those patients who suffer myocardial infarctions in the
perioperative setting.
Evidence regarding the best methods to prevent perioperative
myocardial infarction remains controversial. Although several
small randomized trials demonstrated that prophylactic -blocker
therapy reduced the incidence of myocardial infarctions, subsequent larger prospective trials failed to reproduce these benefits.1-2
In fact, one large trial (Perioperative Ischaemic Evaluation [POISE])
reported both increased mortality and increased risk of ischemic
strokes in patients treated with metoprolol, despite a reduction in
nonfatal perioperative myocardial infarctions. Although it seems
logical that avoidance of intraoperative and postoperative tachycardia, hypotension, and decreased coronary perfusion should
prevent myocardial infarctions, the best methodology to achieve
these goals has not been demonstrated. Furthermore, the issues of
increased platelet adhesion and increased thombogenesis in the
perioperative period are important potential risk factors for acute
coronary syndromes, but evidence-based approaches to decrease
these risks have also not been established.
Excellent studies have been reported regarding the diagnosis
of perioperative myocardial infarction. Most patients with perioperative myocardial infarction do not have or cannot report chest
pain. Similarly, ST-segment elevation is most often absent. However, although symptoms of chest pain and ECG changes may be
nonspecific in the perioperative period, it should be emphasized
that troponin levels are both diagnostic and prognostic.3-4
The chapter accurately describes the current recommendations for therapy of myocardial infarction in the emergency
department setting, the goals of which are to reestablish myocardial perfusion as quickly as possible. However, the risk of bleeding
in the perioperative period makes these recommendations difficult
to follow for surgical patients suffering myocardial infarctions.
Although it would seem that postoperative patients are closely
monitored in hospitals, and are thus in ideal locations to receive
early and aggressive therapy for their myocardial ischemia, such
therapy is often withheld because of concern for bleeding risks.
However, there is a paucity of evidence on which to base factual

risk to benefit decisions. For example, there are few, if any, data to
determine what the bleeding risk is for each specific therapy for
each type of operation. Neither is it clear what the prognosis of a
particular perioperative myocardial infarction will be if aggressive therapy is withheld, though hints of poor prognosis may be
gleaned from extensive ECG changes, poor myocardial contractility, and high troponin levels. Although caution often dictates that
aggressive therapy be withheld, such caution might not always
lead to optimal medical decisions.
The benefits of early angiography and coronary dilatation
and stenting are proven for acute myocardial infarctions. Yet, this
therapy is often withheld from postoperative patients because of
concerns regarding the need for antiplatelet therapy to maintain
stent patency. However, the absolute risk of bleeding from antiplatelet therapy has not been established in postoperative patients.
In all likelihood, such risk varies with the type of operation and
potential difficulty in controlling bleeding if it were to occur. Similarly, fibrinolytic therapy is of proven benefit, but this therapy too
is usually withheld in the postoperative period. Again, the absolute risks of postoperative fibrinolytic therapy are not established,
and most likely vary by operative procedure.
In summary, whereas significant progress has been made in
the management of acute myocardial infarction in general, surgical patients often do not benefit from this progress, because
of concerns for bleeding risks from aggressive therapy. This is a
major issue, because the incidence of perioperative myocardial
infarction is considerable, and is likely to further increase in the
future as our operative population ages.5 Yet data are scarce. In
fact, there are no prospective randomized studies on the treatment
of perioperative myocardial infarction. It is clear that additional
research is needed to define optimal approaches to minimize the
risk of occurrence of perioperative myocardial infarctions. In
addition, further studies are needed to determine the risks and
benefits of aggressive interventional therapies for myocardial
infarction in the postoperative period.

REFERENCES
1. POISE Study Group, Devereaux PJ, Yang H, et al. Effects of
extended-release metoprolor succinate in patients undergoing
907

PMPH_CH114.indd 907

5/22/2012 6:18:38 PM

908

Surgery: Evidence-Based Practice

non-cardiac surgery (POISE trial): a randomized controlled trial.


Lancet. 2008:371(9627):1839-1847.
2. Kaafarani H, Atluir PV, Thornby J, et al. -blockade in noncardiac surgery: outcome at all levels of cardiac risk. Arch Surg. 2008;
143(10);940-944.
3. Kim LJ, Martinez EA, Faraday N, et al. Cardiac troponin I predicts
short-term mortality in vascular surgery patients. Circulation.
2002;106:2366-2373.

PMPH_CH114.indd 908

4. Landdesberg G, Shatz V, Akopnik I, et al. Association of cardiac


troponin, CK-MB, and postoperative myocardial ischemia with
long-term survival after major vascular surgery. J Am Coll Cardiol.
2003;42:1547-1551.
5. Devereaux PJ, Goldman L, Cook DJ, et al. Perioperative cardiac events
in patients undergoing noncardiac surgery: a review of the magnitude of the problem, the pathophysiology of the events and methods
to estimate and communicate risk. CMAJ. 2005;173;627-632.

5/22/2012 6:18:38 PM

CHAPTER 115

Abnormal Surgical and


Postoperative Bleeding
Andrew L. Tang, Nicholas Tarmey, Joseph Dubose, and Peter Rhee

INTRODUCTION

Routine screening tests of coagulation, such as prothrombin


time (PT) and activated partial thromboplastin time (aPTT), do
not rule out VWD due to their lack of sensitivity. Prior to the 1970s,
VWD could only be diagnosed on the basis of low levels of plasma
factor VIII and a prolonged bleeding time. More specific assays for
VWD have been developed, however, due to their individual limited sensitivity, specificity, and reproducibility. VWD disease is
generally diagnosed with a combination of four assays that measure the bleeding time, the levels of factor VIII, VWF antigen, and
the ristocetin cofactor activity.5 Types I and II VWD can be treated
with the antidiuretic hormone analogue desmopressin via its production stimulation of both VWF and factor VIII. Type III VWD
and refractive or severe cases can be treated with VWF/factor VIII
specific concentrates or the administration of cryoprecipitate.
Hemophilia A and B are sex-linked recessive disorders resulting in factor VIII and IX deficiencies. Hemophilia A is the second
most common factor deficiency after VWD with a prevalence of
1 in 5000 male births. The disease is stratified as mild (factor level
>5%), moderate (1% to 5%), and severe (<1%). Effective hemostasis
requires a minimum factor VIII level of 30%, and spontaneous
bleeding can occur with factor levels less than 5%.6
In comparison, hemophilia B has a prevalence of 1 in 30,000
male births. It is clinically indistinguishable from hemophilia A.
Laboratory screening tests may reveal a prolonged aPTT, although
a greater proportion of patients have mild or moderate disease.7
Other inherited coagulation disorders include fibrinogen,
prothrombin, factor V, VII, X, XI, XIII deficiencies, although their
prevalence is extremely rare (1:500,0001 million).6
Certain acquired diseases or medications adversely affect the
coagulation cascade. Advanced liver diseases result in the impaired
synthesis and degradation of both plasma coagulation as well as
anticoagulation factors. In addition, hypersplenism causes thrombocytopenia from platelet sequestration and increased platelet
destruction. Uremia is known to cause a reversible platelet dysfunction, although the exact mechanism has not been elucidated.
Malnutrition and hemodialysis may cause vitamin C deficiency
resulting in vascular wall collagen malformation and loss of capillary integrity. Malnutrition may also cause vitamin K deficiency

Abnormal perioperative bleeding presents a daunting challenge to


the surgeon. Systematic scientific review has attempted to clarify
the incidence and causes of coagulopathy; challenged some of the
common practices such as routine coagulopathy screening; and
also substantiated the usage of potentially life-saving new medications such as recombinant factor VIIa and prothrombin complex concentrate. This chapter utilizes the most robust scientific
studies available to address questions regarding the epidemiology,
diagnosis, and some of the novel medications available for the
treatment of abnormal bleeding.

INCIDENCE/DIAGNOSIS
1. What is the incidence and causes of abnormal surgical and
postoperative bleeding?
The overall incidence of abnormal surgical and postoperative
bleeding in the general population is between 0.032% and 8%.1,2
Coagulopathies can be separated into the global categories of congenital and acquired conditions. Von Willebrand disease (VWD)
is the most commonly encountered congenital coagulopathy with
a prevalence of 1% to 2% according to screening studies.3,4 However, estimates based on referral for symptoms of bleeding suggest that the prevalence of clinically significant VWD is 30 to 100
per million.3,4 The disease is inherited in an autosomal dominant
pattern and results in either a quantitative (Type I) or qualitative
(Type II) deficiency of von Willebrand factor (VWF), a multimeric
protein required for platelet adhesion and plasma factor VIII
stabilization. Type I VWD is found in 60% to 80% of cases, and
although the level of VWF is normal in Type II VWD, the protein
is qualitatively defective and results in inadequate hemostasis in
20% to 30% of cases.5 Type III VWD is autosomal recessive and
results in nearly undetectable levels of VWF and low levels of factor VIII. Although Type III VWD only comprises 1% of patients
afflicted with the disease, with an incidence of 1 in 1 million people, its symptoms are usually more frequent and severe.5
909

PMPH_CH115.indd 909

5/22/2012 6:19:09 PM

910

Surgery: Evidence-Based Practice

with resultant deficiencies in factors II, VII, IX, X, proteins C


and S. Hypothermia below 34C has been found to correlate with
significant platelet enzymatic and protein dysfunction, and has
been identified as an independent predictor of mortality after cavitary explorations in trauma.8,9
Other commonly encountered conditions that exacerbate coagulopathy include acidosis, sepsis, and medications such as aspirin,
warfarin, or clopidogrel. Consumptive and dilutional coagulopathies are particularly encountered in hemorrhagic shock resuscitation with fluids deprived of factors necessary for coagulation.
Considering the various causes of coagulopathy, the overall
estimated incidence of abnormal perioperative bleeding is 0.032%
to 8%. The lack of a standardized definition for abnormal bleeding
among the various studies, however, makes a systematic review of the
data difficult to synthesize. Although PT and aPTT were originally
developed to aid in the diagnosis of inherited bleeding disorders
such as hemophilias, they are now routinely used for preoperative
screening. In a prospective multicenter observational trial of 3242
consecutive general surgery patients, Houry et al. examined the correlation between preoperative coagulation tests (prothrombin time,
activated partial thromboplastin time, platelet count, and bleeding
time) and the rate of perioperative modifications such as postponing operations, ordering additional coagulation tests, and the rate of
postoperative bleeding. The bleeding rate for patients with normal
and abnormal coagulation tests were 0.032 and 0.045, respectively
(Level 1b).2,10 In a prospective multicenter observational trial of 1479
pediatric tonsillectomies, Gabriel et al. found abnormal bleeding as
determined by the operating surgeon to be 7% intraoperatively and
3% postoperatively. The bleeding rate was 0.1 and 0.17, respectively,
for patients with normal and abnormal coagulation tests (Level 1b).1,2
Burk et al. prospectively examined 1603 pediatric tonsillectomies and
found 2% with abnormal screening studies (complete blood count,
PT, PTT, bleeding time), of which 1% had preoperative change in
management. The overall rate of bleeding severe enough to warrant
prolonged hospitalization or readmission was 2.3% (Level 1b).11
Answer: Multiple congenital and acquired conditions contribute to abnormal operative and postoperative bleeding (Grade
A recommendation). The incidence of abnormal surgical and
postoperative bleeding in elective surgery is between 0.032% and
8%. However, the lack of a standardized definition for abnormal
bleeding makes the systematic review of the various studies difficult to perform (Grade A recommendation).
2. What is the appropriate preoperative coagulation workup
for patients undergoing elective surgery?
The normal range of PT and aPTT is calculated by the mean +/ 2
standard deviations of measurements in healthy patients without
bleeding dyscrasias. By definition, 2.5% of measurements taken
in normal individuals will show prolonged clotting time.2 On the
contrary, certain potentially clinically significant but mild forms
of VWD, hemophilia A or B, and even more rare disorders such
as factor XIII and alpha-2-antiplasmin deficiency may result in
normal PT and PTT. Platelet qualitative disorders related to medications (ASA, clopidogrel), uremia, and hypothermia are also
not apparent based on platelet counts. Platelet dysfunction may
result in an abnormal bleeding test; however, it is a test no longer
readily accessible in most hospitals (see Table 115.1). The British
Committee for Standards in Hematology published guidelines on
the preoperative testing of PT and aPTT based on a systematic
review of the literature. A total of 12,000 patients were included

PMPH_CH115.indd 910

in 9 observational studies (3 prospective) encompassing adult and


pediatric ENT and adult general surgery patients. An abnormal
coagulation test was found to have poor positive predictive value
(0.030.22) and low likelihood ratio (0.945.1) for operative and
postoperative bleeding. In addition, perioperative bleeding rates
were similar in patients with and without abnormal coagulation
tests (Level 1a evidence).2
The cornerstone of surgical risk assessment is a detailed history and physical examination. Beyond the obvious history of
bleeding diathesis from dental and previous surgical procedures,
more insidious clues to potential coagulation disorders include
heavy menstrual bleeding, recurrent epistaxis, spontaneous
bruising, chronic iron deficiency anemia, and chronic renal or
liver disease. Physical examination may reveal petechiae, ecchymosis, mucus membrane bleeding, stigmata of malnutrition, or
renal or liver disease.
In a series of more than 1000 patients, Robbins and Rose
examined the correlation between history and physical examination to aPTT abnormalities. They found that 82% of abnormal
tests were in patients with clinical indications of increased bleeding risks.12 Eisenberg et al. found 13 patients (2.7%) with abnormal screening PT and aPTT among a retrospective review of 480
surgical, obstetrical, and gynecological patients without indications of bleeding disorders by history and physical examination.
Only 1 of the 13 patients had postoperative hemorrhage (Level
2b).13 Suchman and Mushlin stratified 12,338 patients undergoing invasive diagnostic or therapeutic procedures into high- and
low-risk groups based on the presence of known coagulopathy,
potential factor deficiency (e.g., liver disease, malabsorption, or
malnutrition), trauma, or hemorrhage. The probability of postoperative hemorrhage was 0.22% in the low-risk group and 1.7% in
the high-risk group (Level 2b).14
If the history and physical examination suggest predisposing conditions for bleeding, then coagulation profi le consisting of
PT, aPTT, and platelet counts, along with any additional testing
should be obtained based on the suspected diagnosis.
Answer: Laboratory markers including PT, aPTT, and platelet count have poor correlation with the actual bleeding risk during elective surgery (Grade A recommendation). Surgical bleeding
risk assessment should be based on a detailed history and physical
examination with particular focus on previous bleeding diathesis
and risk factor for coagulopathy. Selective coagulation testing
should be based on the degree of clinical suspicion for coagulopathy and the nature of the procedure (Grade B recommendation).
3. Does perioperative prophylactic subcutaneous heparin or
low-molecular weight heparin increase intraoperative or postoperative bleeding risk?
The optimal timing of heparin prophylaxis represents a balance
between efficacy in preventing venous thromboembolism (VTE)
and the risks of increasing perioperative bleeding. Most evidence
for the optimal timing of heparin prophylaxis is from studies of
low-molecular weight heparin (LMWH) in major orthopedic surgery. The only large randomized controlled trial (RCT) comparing times of administration for the same drug was reported by
Hull et al. in 2000, who randomized 1472 elective hip arthroplasty
patients into three groups.15 The first group began dalteparin less
than 2 hours preoperatively, the second group began dalteparin
4 to 6 hours after surgery, and the third (control) group began
oral warfarin on the evening after surgery. Only preoperative

5/22/2012 6:19:10 PM

Abnormal Surgical and Postoperative Bleeding

911

Table 115.1 Common Coagulopathic Conditions

Acquired

Congenital

Abnormal Tests

Therapy

VWD

aPTT*
Factor VIII level
Bleeding time
VWF antigen
Ristocetin cofactor activity

Desmopressin
Cryoprecipitate
FFP (if cryoprecipitate unavailable)
VWF/factor VIII concentrate

Hemophilia A

aPTT
Factor VIII level

Desmopressin
Factor VIII concentrate
Cryoprecipitate

Hemophilia B

aPTT
Factor VII level

Factor IX concentrate
Cryoprecipitate

Liver failure

Thrombocytopenia
aPTT
PT

Platelet transfusion
Vitamin K
FFP

Uremia

Bleeding time

Desmopressin
Hemodialysis
Platelet transfusion

Coumadin

PT

Vitamin K, and
PCC (first line), or
FFP

Enoxaparin

Anti-factor Xa level

Protamine sulfate**

Heparin

aPTT

Protamine sulfate

Clopidogrel

Bleeding time
Platelet aggregometry

Platelet transfusion

ASA

Bleeding time

Platelet transfusion

*Only when Factor VIII levels are sufficiently reduced.


**Anti-factor Xa activity is only partially neutralized by protamine sulfate.
PT, prothrombin time; aPTT, activated partial thromboplastin time; FFP, fresh frozen plasma; PCC, prothrombin complex concentrate.

dalteparin was associated with a significantly higher risk of bleeding versus the control group, and there was no benefit in terms of
VTE reduction for preoperative versus postoperative dalteparin.
In 2002, Strebel et al. reported a meta-analysis of 14 RCTs
of LMWH prophylaxis in elective hip surgery.16 They found no
significant difference in rates of deep venous thrombosis (DVT)
or major bleeding for patients given LMWH >12 hours preoperatively versus >12 hours postoperatively. Patients beginning
LMWH within 12 hours of the beginning or end of surgery had a
lower risk of DVT, but this benefit was offset by increased rates of
major bleeding.
For nonorthopedic surgery, evidence for the optimal timing of heparin prophylaxis is limited to observational studies. In
2008, Hansen et al. retrospectively studied heparin prophylaxis
for hysterectomy in 9949 women and found a significantly lower
rate of bleeding when heparin was started postoperatively versus
preoperatively.17 Rates of VTE were similar between groups.
Guidelines issued in 2008 by the American College of Chest
Physicians made recommendations on the timing of heparin prophylaxis only for orthopedic surgery.18 For major orthopedic
surgery they recommended LMWH either preoperatively or
postoperatively. For elective hip arthroplasty they recommended
either; a full dose of LMWH 12 hours before surgery; a half dose
of LMWH 4 to 6 hours after surgery; or a full dose of LMWH 12
to 24 hours after surgery.

PMPH_CH115.indd 911

Answer: Heparin thromboprophylaxis given in close proximity to surgery increases bleeding risk but also protects from VTE.
For elective hip surgery, consideration should be given to early
postoperative, rather than preoperative administration, in order
to limit the risk of bleeding. (Grade B recommendation, Level 2b
evidence).
4. Do certain over-the-counter nutritional supplements affect
the coagulation profi le?
The use of complementary alternative medicines (CAMs) including various herbs and dietary supplements has increased by 450%
in the past decade in the United States.19 In a survey of 2186 patients
undergoing elective surgeries, Adusumilli et al. found that 57% of
respondents admitted to using herbal medicine at some point in
their life, and one in six respondents continued using herbal medicine during the month of surgery.20 CAMs are classified as dietary
supplements under the Dietary Supplements Health and Education Act of 1994. The law exempts herbal medications from the
same rigorous Food and Drug Administration approval process as
exists for conventional medications. In addition, there is no mechanism for postmarketing surveillance and no centralized mandatory reporting agency for adverse events. Consequently, product
quality and label fidelity may vary among various brands, and the
true incidence and nature of adverse events are unknown.21

5/22/2012 6:19:10 PM

912

Surgery: Evidence-Based Practice

Ginkgo Biloba, Ginseng, Garlic, chondroitin, St. Johns wort,


and Saw Palmetto are among the more commonly encountered
herbal medications in the general population. All have been implicated in abnormal operative bleeding. However, a large volume of
the incriminating literature is limited to case reports and small
case series that are confounded by multiple variables and missing
information. Most importantly, the anecdotal reports provide no
scientific causative link between the use of CAM and abnormal
operative bleeding.
Answer: Although a surgical consideration, there is inconclusive evidence linking the use of CAM to increased surgical
bleeding risk (Grade C recommendation).

MANAGEMENT
5. Is factor VII a useful adjunct in treating abnormal operative
and postoperative bleeding?
Recombinant activated factor VII (rFVIIa) acts by inducing
thrombin generation and contributing to the formation of a stabilized fibrin clot. Recently, it has been studied for its potential to
prevent and treat coagulopathic bleeding from trauma or major
surgery. Although initial studies of rFVIIa in trauma were promising, a large phase III trial was terminated early following an
interim data analysis on the grounds of futility for demonstrating
a significant effect on the primary endpoint of reduction in mortality.22 The study did, however, confirm the hemostatic effect of
rFVIIa with a significant reduction in blood transfusion requirements following traumatic hemorrhage.
In elective surgery, most studies of rFVIIa have been small and
focussed on prophylaxis for perioperative bleeding. In 2003 Friederich et al. reported an RCT of 36 patients randomized to receive
prophylactic rFVIIa or placebo during retropubic prostatectomy.23
Perioperative bleeding was significantly reduced but the study was
too small to assess properly the effect on mortality or serious adverse
events (SAEs). Further studies, including four RCTs of elective liver
surgery, failed to demonstrate a significant reduction in mortality
or transfusion requirements with prophylactic rFVIIa.
One RCT has studied rFVIIa for the treatment of serious
postoperative bleeding after cardiothoracic surgery. In a placebocontrolled RCT of 172 patients, Gill et al. found that rFVIIa in
addition to standard therapy was associated with a significant
reduction in rates of transfusion and reoperation.24 There was,
however, a trend toward increased critical SAEs in the rFVIIa
group and the authors felt unable to conclude that rFVIIa was
both safe and effective.
Concerns have been raised recently over the potential for
rFVIIa to promote thromboembolism. A meta-analysis, published
in 2010, examined the safety data from 35 studies with a total of
4468 patients who received rFVIIa.25 There was a small but significant increase in arterial thromboembolic adverse events (TAEs)
with rFVIIa versus placebo (5.5% vs. 3.2%, P = .003). These risks
increased with age and patients over 75 years suffered a threefold
increase in arterial TAEs (10.8% vs. 4.1%, P = .02).
Answer: rFVIIa should be considered as an adjunct for the
treatment of refractory coagulopathic bleeding (Grade A recommendation, Level 1 evidence). There is currently insufficient
evidence to recommend the prophylactic use of rFVIIa during
major surgery and the risk of arterial thromboembolism should
be weighed in any decision on the use of rFVIIa.

PMPH_CH115.indd 912

6. What is the role of prothrombin complex concentrates for


treatment of the bleeding patient?
Prothrombin complex concentrates (PCCs) are clotting products
manufactured from human plasma that contain clotting factors II,
IX, X and, usually, factor VII. PCCs are mainly used for urgent warfarin reversal, where they may be preferred over fresh frozen plasma
(FFP) due to their low volume and speed of administration.
Evidence for PCC in warfarin reversal remains limited to
small or observational studies. In 2004, Lubetsky et al. reported
a prospective, nonrandomized study of PCC for urgent warfarin reversal in 20 patients.26 Ten minutes after dosing, the mean
International Normalized Ratio (INR) for all patients was reduced
from 6.1 to 1.5 and the hemostatic response was graded as good
in 85% of subjects. No TAEs or other complications were reported.
In 1997, Makris et al. retrospectively compared PCC with FFP
for urgent warfarin reversal.27 Fifteen minutes after dosing, the
INR was corrected in 28 of 29 patients who were given PCC, but
remained above 1.5 in all 12 of the patients who were given FFP.
In 2008 Leissinger et al. reviewed evidence for the safety and
efficacy of PCC for urgent warfarin reversal.28 Fourteen small
studies were found with a total of 469 patients given PCC, 7 of
whom experienced TAEs. The authors concluded that PCC was
more effective than FFP and did not appear to increase the risk
of TAEs. Consensus guidelines published by the American College of Chest Physicians in 2008 recommend PCC to immediately reverse the INR in patients with life-threatening bleeding
or intracranial hemorrhage.29
There is considerably less evidence to support the use of PCC
in patients not anticoagulated with vitamin K antagonists. In
2008 Bruce and Nokes reported a case series of 24 patients treated
with PCC for coagulopathic bleeding, 16 of which were not anticoagulated with warfarin.30 PCC was associated with improved
hemostasis and laboratory tests of coagulation, without apparent complications. Similar findings were reported by Joseph et al.
at the 69th annual meeting of the American Association for the
Surgery of Trauma. In a retrospective cohort study of 45 coagulopathic trauma patients, PCC was equally effective at reducing
INR, regardless of whether the coagulopathy was due to warfarin
(n = 25) or a trauma-related coagulopathy (n = 20). Despite these
promising results, there have not yet been any prospective, controlled trials for this use of PCC.
Answer: PCC (with vitamin K) should be considered as firstline therapy for emergency reversal of warfarin anticoagulation
(Grade B recommendation, Level 3 evidence). For patients not
anticoagulated with vitamin K antagonists, there is currently
insufficient evidence to recommend the routine use of PCC.
7. What is the role of tranexamic acid in treating
coagulopathy?
Tranexamic acid (TXA) is a serine protease inhibitor that binds
plasminogen to inhibit fibrinolysis. Most evidence for the use of
TXA is from studies of prophylactic use in cardiac surgery, where
it is given to reduce perioperative bleeding. A meta-analysis published in 2011 by the Cochrane Collaboration included a total of 65
RCTs comparing TXA with placebo, 34 of which involved cardiac
surgery.31 Pooled results supported the hemostatic effect of TXA,
with significant reductions in perioperative blood loss (mean 300
mL; 95%CI: 130471 mL) and transfusion requirements (RR 0.68;
95%CI: 0.570.81). There was no significant reduction in rates of

5/22/2012 6:19:10 PM

Abnormal Surgical and Postoperative Bleeding

mortality or reoperation, although these results were limited by


underpowering.
There were similar findings in 27 RCTs of prophylactic TXA
for orthopedic surgery. TXA reduced perioperative bleeding by
446 mL (95%CI: 338555 mL), and transfusion requirements by
51% (RR 0.49; 95%CI: 0.390.62). Again, there was no overall
effect on mortality. In RCTs of other types of surgery, one study
of vascular surgery showed a significant reduction in transfusion
requirements, whereas two studies of liver surgery and one study
of gynecological surgery showed no benefit from TXA. Despite a
theoretical risk of increasing TAEs with TXA, pooled safety data
from all RCTs were reassuring, with no increase in mortality or
any type of serious adverse events.
There has been new interest in TXA for uncontrolled hemorrhage following the publication in 2010 of a large RCT of TXA for
traumatic bleeding.32 Over 20,000 trauma patients with, or at risk
of, significant bleeding were randomized to receive TXA or placebo with a primary outcome measure of 28-day mortality. TXA
significantly reduced mortality (RR 0.91; 95%CI: 0.850.97) and
death due to bleeding, without increasing the incidence of vascular occlusive events.
Answer: TXA is an effective and apparently safe hemostatic
agent in cardiac and orthopedic surgery and may also reduce
mortality from traumatic bleeding. Consideration of TXA is recommended for reducing blood loss from cardiac and major orthopedic surgery (Grade A recommendation, Level 1a evidence) and
for the management of coagulopathic bleeding (Grade B recommendation, extrapolation from Level 1b study).
8. How should fresh frozen plasma be used in the treatment of
operative and postoperative bleeding?
Fresh frozen plasma (FFP) is produced from the separation of
whole blood and may be used to treat congenital or acquired coagulopathies. Recently, there has been particular interest in attempting to determine the optimum ratio of FFP to packed red blood
cells (PRBC) during massive transfusion, defined as the transfusion of 10 or more units of PRBC in 24 hours. Cryoprecipitate, a
pooled product produced from FFP that contains higher levels of
fibrinogen and factor VIII, has been used for the treatment of specific factor deficiencies but has not been studied rigorously outside
of this setting.
Evidence for FFP:PRBC ratios is mostly from retrospective
studies of trauma patients. In 2007, Borgman et al. reported a
retrospective study of 246 massively transfused military trauma
patients in Iraq.33 Higher ratios of FFP:PRBC were independently
associated with survival and the best results were seen with an
FFP:PRBC ratio of 1:1.4 or higher.
These findings were confirmed by two further retrospective
studies in massively transfused civilian trauma patients. Holcomb
et al.s 2008 study included 466 patients and found the lowest mortality rates with FFP:PRBC ratios of 1:2 or higher.34 Teixeira et al.s
2009 study included 383 patients and found the lowest mortality rates with FFP:PRBC ratios of 1:3.35 Interestingly, in the latter
study there appeared to be no additional survival benefit when
ratios were higher than 1:3.
These results have not been replicated in the absence of massive transfusion. In 2010, Inaba et al. reported a retrospective study
of 284 case-matched nonmassively transfused trauma patients.36
The use of FFP was not associated with a significant difference in
mortality but was associated with significant increases in ARDS,

PMPH_CH115.indd 913

913

organ dysfunction, and sepsis. The authors concluded that the


optimal trigger for the use of FFP remains uncertain.
Answer: During massive transfusion, FFP should be transfused at a ratio of FFP:PRBC approaching that of whole blood (1:3
FFP:PRBC or greater) (Grade B recommendation, Level 3 evidence).
9. When should platelets be administered perioperatively or
postoperatively to bleeding patients?
During massive transfusion, platelets may be consumed, diluted,
or impaired by acidosis or hypothermia. Platelets have conventionally been given late during transfusion based on laboratory
evidence of thrombocytopenia, as supported by 2006 guidelines
from the American Society of Anesthesiologists.37 Recent evidence, however, suggests giving platelets earlier, and in greater
quantities than this.
Most evidence for platelet transfusion in bleeding patients is
from observational studies of trauma resuscitation. In 2008 Holcomb et al. reported a retrospective study of 466 massively transfused trauma patients, comparing mortality between groups based
on the ratio of transfused units of FFP and platelets to PRBC.34
Mortality was significantly reduced when the ratio of FFP:PRBC
was 1:2 or when the ratio of platelets:PRBC was 1:2. The best
results were seen when patients received high ratios of both FFP
and platelets to PRBC.
A similar study was reported in 2009 by Perkins et al., who
retrospectively examined rates of platelet transfusion in 464 military trauma casualties.38 They also found that higher ratios of
platelets:PRBC were associated with reduced mortality and that
increased platelet transfusion was independently associated with
survival. They recognized, however, the limitations of this type of
research, including the potential for hidden confounders.
In nontrauma research, Johansson et al. in 2007 investigated the effect of implementing a new transfusion protocol
for ruptured abdominal aortic aneurysm that included early
transfusion of platelets.39 Compared with historical controls,
patients reached intensive care unit (ICU) with significantly
higher platelet counts, required less postoperative transfusion,
and had improved rates of survival. Although more FFP was also
given under the new protocol, multivariate analysis showed that
higher platelet counts on admission to ICU were independently
associated with survival.
These findings were confirmed by a similar study, in trauma
patients, reported by Gunter et al. in 2008.40 Following the introduction of a new massive transfusion protocol that encouraged
early use of platelets and FFP, the authors found that higher ratios
of both FFP:PRBC and platelets:PRBC were independently associated with improved survival.
Answer: For patients with uncontrolled hemorrhage, early,
empirical use of platelets is recommended as part of a massive
transfusion protocol with a ratio of platelets:PRBC approaching
that of whole blood (Grade B recommendation, Level 3 evidence).

CONCLUSION
Perioperative bleeding is one of the most common and potentially
serious complications of surgery. A multitude of conditions, congenital and acquired, contribute to coagulopathy. Unfortunately,
our routine coagulation panel consisting of PT, aPTT, and platelet

5/22/2012 6:19:10 PM

914

Surgery: Evidence-Based Practice

count do not adequately predict the potential for perioperative


bleeding complications. Instead, a detailed history and physical examination aimed at identifying bleeding risks may alarm
the treating surgeon to conduct further selective coagulopathy
workup. As the usage of CAM increases among the general population, multiple reports have appeared in the literature raising
the potential connection between their use and increased surgical
bleeding risk. However, these reports are not convincingly substantiated by rigorous scientific evaluation.
Recombinant factor VIIa has been rigorously studied and
has the scientific support to be considered a powerful adjunct in
the treatment of coagulopathic patients with refractory bleeding.
PCC, a product manufactured from human plasma containing
the vitamin K dependent clotting factors II, VII, IX, and X, should
be used as first-line therapy in the emergent reversal of Coumadin. Its potential use in patients not anticoagulated by vitamin

K antagonists, although substantiated by small case series, needs


further scientific investigation.
Thromboprophylaxis for at-risk surgical patients raises the
balance between increased bleeding risk and delay in prophylaxis. The optimal timing to initiate thromboprophylaxis relative
to surgery has not been identified except for major orthopedic
procedures. TXA has been successfully used in cardiac surgery to
reverse the coagulopathy induced by cardiopulmonary bypass. Its
potential use outside of this setting, although promising, requires
further investigation.
A paradigm shift has swept through transfusion medicine
after data showing the mortality benefit of transfusion of PRBC,
FFP, and platelets mimicking that of whole blood emerged from
the Gulf War. There is convincing evidence to support a transfusion ratio of 1:1-2 for PRBC and FFP; however, the optimal platelet
ratio relative to PRBC is still under investigation.

Clinical Question Summary


Question

Answer

1 What is the incidence and


causes of abnormal surgical
and postoperative bleeding?

Although there is no standardized definition


of abnormal operative and postoperative
bleeding, various prospective observational
studies estimate the incidence to be 1%
to 8%.
The causes of abnormal surgical and
postoperative bleeding are various
congenital defects of the coagulation
cascade and acquired diseases that alter the
coagulation cascade.

1b

1, 2, 10, 11

1a

3-9

Detailed history and physical examination


with particular focus on previous bleeding
diathesis and risk factors for coagulopathy.
Further laboratory testing should be
based on clinical suspicion of underlying
pathologies.
PT, aPTT, and platelet count have poor
correlation with the actual bleeding risk
during elective surgery.

1b

1, 2, 10-14

1a

1, 2, 10-14

3 Does perioperative
prophylactic subcutaneous
heparin or low-molecular
weight heparin increase
intraoperative or
postoperative bleeding risk?

Heparin thromboprophylaxis given in close


proximity to surgery increases bleeding risk
but also protects from VTE.
For elective hip surgery, consideration should
be given to early postoperative, rather than
preoperative administration, in order to
limit the risk of bleeding.

16

15

4 Do certain over-the-counter
nutritional supplements
affect the coagulation profile?

There is inconclusive evidence linking CAMs to


increased surgical bleeding risk.

19-21

5 Is factor VIIa a useful adjunct in


treating abnormal operative
and postoperative bleeding?

rFVIIa should be considered as an adjunct for


the treatment of refractory coagulopathic
bleeding.

22, 24

6 What is the role of


prothrombin complex
concentrates in the treatment
of the bleeding patient?

PCC (with vitamin K) should be considered as


first-line therapy for emergency reversal of
warfarin anticoagulation.

26, 28

2 What is the appropriate


preoperative coagulation
workup for patients
undergoing elective surgery?

Level of Grade of
References
Evidence Recommendation

(Continued)

PMPH_CH115.indd 914

5/22/2012 6:19:10 PM

Abnormal Surgical and Postoperative Bleeding

915

(Continued)
Question

Answer

7 What is the role of


tranexamic acid in treating
coagulopathy?

Consideration of TXA is recommended for


reducing blood loss from cardiac and major
orthopedic surgery.
Consideration of TXA is recommended for
the management of coagulopathic bleeding.

31

B
(extrapolated)

32

8 How should fresh frozen


plasma be used in the
treatment of operative and
postoperative bleeding?

During massive transfusion, FFP should


be transfused at a ratio of FFP:PRBC
approaching that of whole blood
(1:3 FFP:PRBC or greater).

33-35

9 When should platelets be


administered perioperatively
or postoperatively to
bleeding patients?

For patients with uncontrolled hemorrhage,


early, empirical use of platelets is
recommended as part of a massive
transfusion protocol with a ratio of
platelets:PRBC approaching that of whole
blood.

34, 38

REFERENCES
1. Gabriel P, Mazoit X, Ecoffey C. Relationship between clinical
history, coagulation tests, and perioperative bleeding during
tonsillectomies in pediatrics. J Clin Anesth. 2000;12(4):288-291.
2. Chee YL, Crawford JC, Watson HG, Greaves M. Guidelines on
the assessment of bleeding risk prior to surgery or invasive procedures. British Committee for Standards in Haematology. Br J
Haematol. 2008;140(5):496-504.
3. Rodeghiero F, Castaman G, Dini E. Epidemiological investigation of the prevalence of von Willebrands disease. Blood.
1987;69(2):454-459.
4. Werner EJ, Broxson EH, Tucker EL, Giroux DS, Shults J, Abshire
TC. Prevalence of von Willebrand disease in children: a multiethnic study. J Pediatr. 1993;123(6):893-898.
5. Mannucci PM. Treatment of von Willebrands Disease. N Engl J
Med. 2004;351(7):683-694.
6. Mannucci PM, Tuddenham EG. The hemophiliasfrom royal
genes to gene therapy. N Engl J Med. 2001;344(23):1773-1779.
7. Larson PJ, High KA. Biology of inherited coagulopathies: factor
IX. Hematol Oncol Clin North Am. 1992;6(5):999-1009.
8 Watts DD, Trask A, Soeken K, Perdue P, Dols S, Kaufmann C.
Hypothermic coagulopathy in trauma: effect of varying levels
of hypothermia on enzyme speed, platelet function, and fibrinolytic activity. J Trauma. 1998;44(5):846-854.
9. Inaba K, Teixeira PG, Rhee P, et al. Mortality impact of hypothermia after cavitary explorations in trauma. World J Surg.
2009;33(4):864-869.
10. Houry S, Georgeac C, Hay JM, Fingerhut A, Boudet MJ. A prospective multicenter evaluation of preoperative hemostatic
screening tests. The French Associations for Surgical Research.
Am J Surg. 1995;170(1):19-23.
11. Burk CD, Miller L, Handler SD, Cohen AR. Preoperative history
and coagulation screening in children undergoing tonsillectomy.
Pediatrics. 1992;89(4 Pt 2):691-695.
12. Robbins JA, Rose SD. Partial thromboplastin time as a screening
test. Ann Intern Med. 1979;90(5):796-797.
13. Eisenberg JM, Clarke JR, Sussman SA. Prothrombin and partial
thromboplastin times as preoperative screening tests. Arch Surg.
1982;117(1):48-51.

PMPH_CH115.indd 915

Level of Grade of
References
Evidence Recommendation

14. Suchman AL, Mushlin AI. How well does the activated partial
thromboplastin time predict postoperative hemorrhage? JAMA.
1986;256(6):750-753.
15. Hull RD, Pineo GF, Francis C, et al. Low-molecular-weight heparin prophylaxis using dalteparin in close proximity to surgery vs
warfarin in hip arthroplasty patients: a double-blind, randomized comparison. The North American Fragmin Trial Investigators. Arch Intern Med. 2000;160(14):2199-2207.
16. Strebel N, Prins M, Agnelli G, Buller HR. Preoperative or postoperative start of prophylaxis for venous thromboembolism
with low-molecular-weight heparin in elective hip surgery? Arch
Intern Med. 2002;162(13):1451-1456.
17. Hansen CT, Kehlet H, Moller C, Morch L, Utzon J, Ottesen B.
Timing of heparin prophylaxis and bleeding complications in
hysterectomy a nationwide prospective cohort study of 9,949 Danish women. Acta Obstet Gynecol Scand. 2008;87(10):1039-1047.
18. Geerts WH, Bergqvist D, Pineo GF, et al. Prevention of venous
thromboembolism: American College of Chest Physicians
Evidence-Based Clinical Practice Guidelines (8th Edition).
Chest. 2008;133(6 Suppl):381S-453S.
19. Eisenberg DM, Davis RB, Ettner SL, et al. Trends in alternative medicine use in the United States, 1990-1997: results of a
follow-up national survey. JAMA. 1998;280(18):1569-1575.
20. Adusumilli PS, Ben-Porat L, Pereira M, Roesler D, Leitman IM.
The prevalence and predictors of herbal medicine use in surgical
patients. J Am Coll Surg. 2004;198(4):583-590.
21. Cui J, Garle M, Eneroth P, Bjorkhem I. What do commercial ginseng preparations contain? Lancet. 1994;344(8915):134.
22. Hauser CJ, Boffard K, Dutton R, et al. Results of the CONTROL
trial: efficacy and safety of recombinant activated Factor VII in
the management of refractory traumatic hemorrhage. J Trauma.
2010;69(3):489-500.
23. Friederich PW, Henny CP, Messelink EJ, et al. Effect of recombinant activated factor VII on perioperative blood loss in patients
undergoing retropubic prostatectomy: a double-blind placebocontrolled randomised trial. Lancet. 2003;361(9353):201-205.
24. Gill R, Herbertson M, Vuylsteke A, et al. Safety and efficacy
of recombinant activated factor VII: a randomized placebocontrolled trial in the setting of bleeding after cardiac surgery.
Circulation. 2009;120(1):21-27.

5/22/2012 6:19:10 PM

916

Surgery: Evidence-Based Practice

25. Levi M, Levy JH, Andersen HF, Truloff D. Safety of recombinant


activated factor VII in randomized clinical trials. N Engl J Med.
2010;363(19):1791-1800.
26. Lubetsky A, Hoff man R, Zimlichman R, et al. Efficacy and safety
of a prothrombin complex concentrate (Octaplex) for rapid reversal of oral anticoagulation. Thromb Res. 2004;113(6):371-378.
27. Makris M, Greaves M, Phillips WS, Kitchen S, Rosendaal FR,
Preston EF. Emergency oral anticoagulant reversal: the relative
efficacy of infusions of fresh frozen plasma and clotting factor
concentrate on correction of the coagulopathy. Thromb Haemost. 1997;77(3):477-480.
28. Leissinger CA, Blatt PM, Hoots WK, Ewenstein B. Role of prothrombin complex concentrates in reversing warfarin anticoagulation: a review of the literature. Am J Hematol. 2008;83(2):
137-143.
29. Ansell J, Hirsh J, Hylek E, Jacobson A, Crowther M, Palareti G.
Pharmacology and management of the vitamin K antagonists:
American College of Chest Physicians Evidence-Based Clinical
Practice Guidelines (8th Edition). Chest. 2008;133(6 Suppl):160S198S.
30. Bruce D, Nokes TJ. Prothrombin complex concentrate (Beriplex
P/N) in severe bleeding: experience in a large tertiary hospital.
Crit Care. 2008;12(4):R105.
31. Henry DA, Carless PA, Moxey AJ, et al. Anti-fibrinolytic use for
minimising perioperative allogeneic blood transfusion. Cochrane
Database Syst Rev. 2011;1:CD001886.
32. Shakur H, Roberts I, Bautista R, et al. Effects of tranexamic acid
on death, vascular occlusive events, and blood transfusion in
trauma patients with significant haemorrhage (CRASH-2): a randomised, placebo-controlled trial. Lancet. 2010;376(9734):23-32.

PMPH_CH115.indd 916

33. Borgman MA, Spinella PC, Perkins JG, et al. The ratio of blood
products transfused affects mortality in patients receiving massive transfusions at a combat support hospital. J Trauma-Injury
Infect Critical Care. 2007;63(4):805-813.
34. Holcomb JB, Wade CE, Michalek JE, et al. Increased plasma and
platelet to red blood cell ratios improves outcome in 466 massively transfused civilian trauma patients. Annals of Surgery.
2008;248(3):447-458.
35. Teixeira PG, Inaba K, Shulman I, et al. Impact of plasma transfusion in massively transfused trauma patients. J Trauma.
2009;66(3):693-697.
36. Inaba K, Branco BC, Rhee P, et al. Impact of plasma transfusion
in trauma patients who do not require massive transfusion. J Am
Coll Surg. 2010;210(6):957-965.
37. Practice guidelines for perioperative blood transfusion and adjuvant therapies: an updated report by the American Society of
Anesthesiologists Task Force on Perioperative Blood Transfusion
and Adjuvant Therapies. Anesthesiology. 2006;105(1):198-208.
38. Perkins JG, Cap AP, Spinella PC, et al. An evaluation of the
impact of apheresis platelets used in the setting of massively
transfused trauma patients. J Trauma. 2009;66(4 Suppl):S77-84;
discussion S84-75.
39. Johansson PI, Stensballe J, Rosenberg I, Hilslov TL, Jorgensen L,
Secher NH. Proactive administration of platelets and plasma for
patients with a ruptured abdominal aortic aneurysm: evaluating a
change in transfusion practice. Transfusion. 2007;47(4):593-598.
40. Gunter OL, Jr., Au BK, Isbell JM, Mowery NT, Young PP, Cotton
BA. Optimizing outcomes in damage control resuscitation: identifying blood product ratios associated with improved survival.
J Trauma. 2008;65(3):527-534.

5/22/2012 6:19:10 PM

CHAPTER 116

Management of Sedation
and Delirium in the ICU
Robert Chen and Sangeeta Mehta

defined is the term acute brain failure.5 As it is well appreciated


that acute congestive heart failure may arise following a variety of
physiologic insults, through a number of mechanisms and require
multimodal (and often multisystem) treatment, delirium should
be viewed similarly.
The Society of Critical Care Medicine (SCCM) published
guidelines for analgesia, sedation, and treatment of delirium in
2002.6 In summary, they recommended that sedation be offered
after appropriate analgesia. A sedation scale should be used and
sedation goals reevaluated regularly. Delirium was to be screened
regularly and treated with haloperidol. Sleep promotion was felt
to be helpful in preventing or treating delirium. In a survey of
Canadian intensivists conducted months after the publication of
these guidelines, a huge variety in the prescribing practices was
seen.7 Half of the respondents did not use a sedation scale and over
95% did not use formalized delirium screening tools. This result is
similar to another survey that described the sizable disconnect
between knowledge and practice8 with only 16% of respondents
using a delirium assessment tool. These results would be analogous to drug therapy for hypertension titrated against myocardial
infarction or stroke rather than blood pressure measurements.
This review offers an evidence-based approach to sedation
and delirium in the ICU (see Figure 116.1).
Clearly, the intensivist must provide comfort to patients in
the ICU. Drug therapy is almost universally given. Analgesia
must be the first priority in the surgical ICU. Poor pain control
unleashes a neuron-hormonal cascade that can lead to cardiovascular complications such as dysrhythmias, congestive heart
failure, myocardial ischemia, and infarction. A patients unwillingness to participate in physiotherapy and deep breathing exercises secondary to pain can cause respiratory complications.
Unfortunately, it is not uncommon for postoperative ICU patients
to be receiving sedative agents alone without accompanying analgesia. With the notable exceptions of some anticonvulsants (gabapentin, pregabalin)9-11 and central acting -2 agonists (clonidine
and dexmedetomidine),12,13 no other sedating medications have
analgesia effects. Avoiding opiates has been proposed as a strategy
to reduce delirium, particularly in the elderly. Local or regional

INTRODUCTION
The intensive care unit (ICU) is a stressful environment for patients
and those who care for them. Similar to the emergency room, it
functions 24 hours a day. For the patients, the days and nights are
fi lled with bright illumination, noises of medical devices and their
alarms, and pain. Like commercial casinos, the ICU robs those
within of time cues: there seem to be no chronologic landmarks
of sleeping or eating. Activity seems continuous, the stimulation
from sound, sight, and touch never stops, nor does it seem predictable. ICUs caring for trauma patients are additionally challenging
in that admissions may be polarized toward evenings and weekends, further disrupting a patients usual daily rhythm. Medical
and surgical ICU patients may have pain from their underlying
disease, or due to the presence of endotracheal tubes and lines,
or procedures such as suctioning. Postoperatively, patients have
pain related to their incision, drains, or other implanted equipment. Patients may have been taking psychotropic drugs, recreational or prescribed, prior to admission. Withdrawal from these
medications can lead to mental and physical stress. Necessary
medications may have important central nervous system (CNS)
side effects, such as the sedation of antiseizure medications, or the
dysphoria of systemic steroids. Sedatives, paralytics,1 or medical
conditions can reduce the patients ability to communicate and
advocate. The patient may also manifest expected CNS changes
associated with brain injury or disease. The awake and alert patient
often expresses fear, panic, and helplessness in such a surreal environment.2 Patients have described confusional states, delusions,
and hallucinations.3
Clinically, these stressors can lead to the subjective diagnosis
of agitation or unpleasant psychomotor arousal. Delirium in the
ICU is part of the same spectrum that was called ICU-itis, ICU
syndrome, or ICU psychosis in the past. The Diagnostic and
Statistical Manual, 4th edition text revision (DSM 4 TR) defines
delirium as being characterized by a disturbance of consciousness and a change in cognition that develop over a short period
of time.4 Delirium is differentiated from other steadily dementing illness by its acute onset and provocation. Even more elegantly
917

PMPH_CH116.indd 917

5/22/2012 6:19:40 PM

918

Surgery: Evidence-Based Practice

Admit to
ICU

Screen for delirium risk

PATIENT:
Comfortable, Calm, Safe?

Continued
assessment

NO
YES

Document with Pain Scale


Multimodal analgesia

YES

Document with sedation scale.


Pro/Dex for 48h, then switch to
protocol driven benzodiazepine
Sedation interruption daily

Patient in
pain?
NO
Patient
agitated?
NO
YES
Patient
delirious

Document with delirium scale


Address environmental factors,
haloperidol

NO
YES
NO

Withdrawing
from ETOH?

Symptom guided algorithm with


benzodiazepines

Pro = propofol, Dex = dexmedetomidine

Figure 116.1 Algorithm for ICU admission and patient care.

anesthetic techniques have in some series reduced postoperative


and ICU delirium.14,15 Recent reviews of the literature however
have demonstrated less favorable conclusions.16,17 Given its lengthy
history and wealth of experience, opiates are still the mainstay of
an analgesic regime for surgical or trauma pain.
1. What is the impact of oversedation and how can sedation
needs be assessed and managed?
Two relatively separate bodies of literature have examined daily
interruption of sedation (sedation vacation or drug holiday) and
protocol driven sedation using sedation scales.
De Jonghe et al. published in 2000 a comprehensive review
of sedation scales in the ICU.18 His exhaustive literature search
revealed 25 different methods starting with Ramsey in 1974.
Ramseys study was a prospective observational study using alphadolone infusions to achieve sedation in the ICU.19 The outcome
was measured by a six-point scale created empirically where 1 represented agitation and 6 oversedation. The scale has since been
validated for reliability and validity. Only three other scales met
the criteria for reliability and validity: the Comfort scale,20 Sedation Agitation Scale (SAS),21 and Motor Activity Assessment Scale
(MAAS).22
The Comfort scale was a multidimensional examination in
the pediatric ICU setting that naturally did not include the ability

PMPH_CH116.indd 918

to obey commands. The SAS, in some ways like the Ramsey, began
empirically in order to study haloperidol infusions in critical care.
The MAAS was designed and validated in a surgical ICU setting.
This rapid to administer seven-point scale (0 to 6) included elements of patient safety when agitated.
Since De Jonghes review, several more scales have been published meeting criteria for reliability and validity including the
Richmond Agitation Sedation Scale (RASS),23 and De Jonghes own
groups Adaptation to the Intensive Care Environment (ATICE).24
ATICE is a multidimensional scale that assesses the domains of
consciousness and tolerance to the ICU environment with subscales within each domain. The awakeness and calmness scales
(2 of 5 in the ATICE) are represented in Table 116.1 as a single
dimension for ease of comparison to the previously described
sedation scores. The table demonstrates that any of the aforementioned scales could be easily used to evaluate the clinical spectrum
of sedation requirement. Ramseys scale is polarized toward the
description of sedation while more recent scales have further subdivided ranges of agitation.
Dr Jean-Louis Vincent published his famous FASTHUG
paper in 2005. He wondered in terms of analgesia and sedation
assessment if the assessment scales were so simple that one may
wonder if they are necessary.25 Dr Papadimos group applied
the original FASTHUG idea and in addition added a twice daily
assessment of sedation in a surgical ICU. They were able to halve
the rate of ventilator-associated pneumonia (VAP) against historical controls.26 These results were particularly significant as
the severity of illness index that was calculated was higher in the
intervention group. It is interesting to note that despite a higher
index of severity of illness, there was a trend toward a shorter
length of stay (LOS) for those patients who were FASTHUGed
with sedation assessments.
By instituting a sedation guideline, Adam et al., in a mixed
medical and surgical ICU, were able to halve the cost of their sedative medications versus historical controls. Unfortunately, LOS
calculations were only done on half of the sample due to a change
in patient bed utilization for cardiac surgery patients before and
after the guideline implementation. Interestingly, the LOS was
similar before and after the guideline implementation.27 In using
his own ATICE scale as part of a sedative algorithm in a prospective cohort trial, De Jonghes group was able to demonstrate faster
arousal from sedation, and thus shorter ventilated times in a medical ICU. Statistically, the reduction was in the order of 5 days.28
Dr Marshalls institution in Boston instituted a sedation guideline
that followed those published by the SCCM.29 An audit the following year demonstrated surprisingly poor compliance. By charging
a clinical pharmacist with the daily role of assuring adherence to
the already established protocol, the ventilated days and ICU LOS
were almost halved. Despite Dr Vincents lack of enthusiasm for
sedation scales, even if only included in a protocol, studies would
suggest that they are very necessary if for nothing else than to
assure proper assessment of the patient.
Answer: Oversedation leads to prolonged mechanical ventilation and its associated complications such as VAP, increased
ICU LOS, and increased financial costs. (Grade A recommendation, Level 1b evidence).
Dr Kresss paper in 2000 was very important in introducing
the concept of a daily sedation interruption.30 In a prospective, randomized fashion, patients admitted to a medical ICU were chosen
to have infusions of midazolam or propofol interrupted daily or
receive routine care. Clearly, blinding would be challenging. Both

5/22/2012 6:19:40 PM

PMPH_CH116.indd 919

1
Responsive only
to noxious
stimuli (opens
eyes, raises
eyebrows, or
turns head)
4
No response
to voice,
responds
to physical
stimuli

1
Facial movement
to strong
stimuli

0
Unresponsive

5
Unarousable

0
No eye
opening

Motor Activity
Assessment
Scale
(MAAS)

Richmond
Agitation
Sedation
Scale

Adaptation to
Intensive
Care
Environment

Awake 0-5
Calm 0-3

2
Very sedated,
arouses but
does not
communicate
or follow
commands

1
Unarousable

Sedation
Agitation
Scale (SAS)

2
Eyes open to
strong stimuli

5
Sluggish response
to noise or
glabellar tap

6
No response
to noise or
glabellar tap

Ramsay

4
Brisk response
to noise or
glabellar tap

Table 116.1 Comparison of Sedation Scales

3
Calm,
cooperative

0
Alert, calm

2
Responsive to name
or touch

3
Moderate sedation

4
Eyes open to voice

Calm

1
Eyes open
spontaneously

3
Eyes open to light
stimuli

1
Awakens to voice
for more than 10s

2
Awakens to voice
for less than 10s

4
Calm and
cooperative

2
Cooperative,
oriented,
tranquil

3
Sedated, awakens to
verbal stimuli or
gentle shaking

3
Responds to oral
commands only

+1
Anxious

4
Restless,
cooperative
(picking
at sheets,
tubes)

2
Agitated but
responds to
verbal order

+2
Frequent nonpurposeful
movements;
ventilator
dyssynchony

5
Agitated
(does not
consistently
follow
commands)

5
Agitated,
anxious, or
physically
agitated;
calms to
verbal
instructions

1
Anxious,
agitated,
restless

1
Agitated, does
not respond to
verbal order

+3
Pulls or removes
tubes

6
Very agitated,
requiring
restraint, verbal
reminding;
biting the
endotracheal
tube

0
Life threatening
agitation

+4
Combative,
dangerous

6
Dangerously
agitated

7
Dangerous
agitation;
pulling at the
endotrachea
tube, catheters;
climbing,
striking

Management of Sedation and Delirium in the ICU

919

5/22/2012 6:19:41 PM

920

Surgery: Evidence-Based Practice

groups received morphine analgesia. The duration of mechanical


ventilation and LOS in the ICU was a third lower in the intervention group. Two groups have since demonstrated a 50% reduction in VAP when they reinterpreted FASTHUG to include daily
sedation interruption.31,32 Although, by definition, the measured
reductions are from multifactorial manipulation, it is plausible
that daily interruption of sedation was important.
Daily sedative interruption may not be ideal for all patients.
De Wit and colleagues randomized patients to protocolized sedation/analgesia or daily sedation interruption where the sedation regimen was directed ad hoc by the ICU team. The study
was stopped early by the Data Safety Monitoring Board, after 74
patients had been enrolled, due to a higher mortality rate in the
daily interruption group. Nevertheless, the duration of mechanical ventilation in the groups differed significantly (6.7 days in the
daily interruption vs. 3.9 in the sedation protocol).33 This group
hypothesized that the worse clinical outcomes in the intervention
group were related to a high proportion of enrolled patients having drug- and alcohol-use disorders, therefore subjecting them to
the risk of withdrawal.
Beyond the advantages alluded to titrated sedation or daily
sedation interruption, an awake patient offers the neurointensivist valuable additional clinical signs and symptoms. Skoglund and
colleagues published a feasibility trial of daily neurologic wake
up test in 21 patients with CNS diseases such as traumatic brain
injury and subarachnoid haemorrhages. They concluded that
patients can be awoken safely with the caveat that there will be
associated increases in intracranial pressure that in some required
immediate drainage of cerebrospinal fluid.34 Once again the intensivist most consider the risk to benefit of daily sedation interruption in these patients.
Critical care resuscitation is guided by measurable values
in almost every domain: cerebral perfusion pressure, alveolararterial gradient, cardiac index, serum lactate, blood urea nitrogen, and so on. The clinical evidence continues to suggest that most
patients are treated for agitation with medication using informal
criteria. There are many variables to examine in sedation research.
Clearly, following a protocol with a defined sedative goal will prevent oversedation. Whether stopping appropriate sedation can
confer additional benefit to the patient is clearly still in question.
Answer: The best approach for sedation/analgesia management should be to ask the patient. If unable, sedation requirements
should be re-examined frequently using a sedation scale. The use
of a sedation protocol or daily interruption of sedation should be
considered (Grade A recommendation, Level 1b evidence).
Propofol, the polysubstituted benzene intravenous (IV) anesthetic, was first introduced as an anesthetic induction agent in the
early 1980s.35-37 Its pharmacological properties of relatively quick
redistribution and metabolism made it an ideal agent for IV sedation in the critical care setting. Studies comparing propofol to
other medications consistently show the superiority of propofol
in ICU for its titratability.38,39 Unfortunately, the propofol infusion syndrome or PRIS, which was first reported in the pediatric
literature40 followed by reports in the adult population, make it
unsuitable for prolonged use. Patients who had prolonged highdose sedation succumbed to hemodynamic failure with progressive acidosis, rhabdomyolysis, hyperlipidemia, and liver changes
on various imaging modalities. The syndrome and its possible
pathophysiology has been well reviewed in the anesthetic and
critical care literature.41-43 The American College of Critical Care

PMPH_CH116.indd 920

Medicine in their 2002 clinical practice guidelines44 advocates


caution in the use of propofol particularly in the setting of raised
Intra-cranial pressure (ICP). In addition, as a Class B recommendation, the guidelines suggest lipid monitoring and inclusion of
propofols lipid carrier as parenteral nutrition content. As the body
of case literature has increased, North American healthcare agencies have issued warnings against prolonged infusion. The North
American manufacturer has contraindicated propofol as an ICU
sedative in patients under 18 years of age.45
Dexmedetomidine has several advantages as an ICU sedative.
As opposed to the gamma amino buteric acid (GABA) agonists
(benzodiazepines, barbiturates, propofol), it is a centrally acting
-2 agonist. It is analgetic at the spinal level while simultaneously
providing sedation through central downregulation of norepinephrine release.46 Given its utility as a co-analgetic, it has been
studied more often in the postoperative setting.47,48 In the ICU, its
sedative quality is reputed at least as good as propofol yet allows
for a qualitative ease in arousal.49 Owing to its relatively recent
introduction and limited long-term experience, its indication is
for sedation duration of less than 24 hours.50 There are, however,
studies in the ICU setting where infusions up to a weeks duration
have not demonstrated negative effects.51,52
Answer: Propofol or dexmedetomidine should be reserved
for ICU sedation where the duration is thought to be no more
than 24 to 48 hours, Propofol should be given at an infusion rate
of no more that 4 to 5 mg/kg/h (Grade C recommendation, Level
4 evidence).
2. What is the role of target-controlled infusions for sedation
in the ICU?
Similar to the tasks of an anesthetist, the intensivist is often
asked to simultaneously teach53 and perform complex diagnostic
procedures,54 while leading patient care. The associated task saturation55 is often rectified by off-loading tasks to a period where
the clinician has spare capacity.56 Management of sedation, as a
longer interval goal, may be one of the critical care elements that is
commonly off-loaded as evidenced by the relative failure to follow
sedation guidelines or protocols.
In the acute care setting, anesthetists have the unusual ability
to continually assess the pharmacologic concentration of inhaled
anesthetic gases at the effect site. The exhaled concentration of
anesthetic gases reasonably approximates brain anesthetic concentration at steady state. Whereas IV drugs are typically dosed
with estimation of the patients volume of distribution and elimination, the effector site concentration can only be assumed to be
therapeutic based on clinical effect. In the delivery of anesthetic
gases, the administration is thus tied to a pharmacologic model
that can be adjusted concurrently for each patient.
Pharmacologic modeling for IV medications are well established. However, without easy methods to measure effector site or
even plasma concentration, medications are simply dosed to effect.
The integration of a pharmacokinetic drug model that includes
changes due to age and BMI57 into a computer controlled pump
allows the clinician to dose medication based on sound pharmacological principles and also choose a dose within a therapeutic
concentration windowin the same way that having sedation
managed by a pharmacist by titrating to endpoints such as sedation level and target-controlled infusion (TCI) uses a computer
system to do so.

5/22/2012 6:19:41 PM

Management of Sedation and Delirium in the ICU

TCI and it synonyms (computer controlled infusion pump,


computer assisted continuous infusion, or computer assisted
titrated IV anesthesia) sought to make IV drugs as easy to use as
anesthetic vaporizers.58
When administering IV anesthetics, TCI has been shown to
be at least equivalent if not superior to human controlled infusions when evaluated for stable depth of anesthesia and recovery
characteristics with propofol,59 remifentanyl,60 and muscle relaxants.61 Cochranes systematic review of the topic in 2008 concluded
as well that TCI propofol was not inferior to human controlled
anesthesia or sedation.62 The next generation of TCI links feedback through a clinical goal to the dosing regime thus closing
the control loop. Dr Puri et al. published a study examining a
closed-loop TCI propofol system titrated to therapeutic bispectral index values in elective general surgery patients63 and later in
cardiac surgery patients.64 In both cases, in order to achieve the
desired level of anesthesia, the TCI system performed better than
the anesthetist. Presently, closed-loop TCI is only available as a
research modality.
Few TCI studies have been done in critical care. Nevertheless,
TCI sedation in the ICU has been shown to be feasible.65 Rather
than interpreting TCI as a poor candidate for critical care sedation based on nonsuperiority in systemic reviews, one could read
into the data that TCI would be equivalent to issuing each ICU
patient an anesthetist or pharmacist with anticipated benefits of
improved sedation outcome.29
Answer: TCI has been proven noninferior to human titration
in general anesthesia applications. TCI may be a valuable sedation
tool in ICU (Grade B recommendation, Level of evidence 2b).
3. How is delirium identified in critically ill patients?
The incidence of delirium in general hospital population is
reported to be from one in ten patients to more than 50% in the
elderly population. Delirium in critical care units has been suggested in up to 80% of patients.66 The true incidence in the ICU
is impossible to quantify as patients receiving necessary sedation
makes the assessment difficult. Although there are postulated
biologic mechanisms, there is no consensus for a final common
pathway. Like ARDS the risk factors are numerous and common
to almost all patients in the ICU.67 They include premorbid patient
factors such as advanced age, dementia, CNS and psychiatric disease, medical comorbidities (renal, hepatic diseases, etc.), drug or
alcohol dependence, and vision or hearing impairment. Concurrent medical factors such as trauma or other severe illness, sleep
deprivation, polypharmacy (including anesthesia care), and psychotropic medication (including opiates) are often unavoidable.
As delirium can appear as agitation from insufficient sedation, the intensivist must be aware that prescribed benzodiazepine
may be causal particularly in the elderly.68 Similar to the complications associated with postoperative pain, delirium can increase the
patients metabolic demands leading to cardiac decompensation,
particularly in patients with cardiac disease and limited physiologic
reserve. Failure to participate in respiratory physiotherapy due to
delirium could lead to pulmonary complications such as atelectasis,
failure of secretion clearance, and subsequent infection. Delirium
has been shown to be an independent risk factor for prolonged ICU
stay.69 The diagnosis increased ICU and hospital costs by 39% and
31%, respectively, in one prospective study.70 The effects of delirium in the ICU may persist well after discharge as a risk factor for

PMPH_CH116.indd 921

921

longer-term cognitive dysfunction.71 As the syndrome is believed to


be preventable by some authors, it has been used as a quality of care
marker.72 Inouye estimates that as a potentially preventable adverse
event, delirium costs $4 to $7 billion to the American healthcare
system.73 Assessing and treating delirium is now slowly being recognized as an important part of ICU care.
As with all psychiatric diseases, the diagnosis of delirium is
based on clinical criteria. The American Psychiatric Association
issues a cautionary statement: The proper use of these criteria
requires specialized clinical training that provides both a body
of knowledge and clinical skills. Said differently, the DSM 4 TR
should not be used by nonpsychiatrists.
With the help of a psychiatrist and a geriatrician following the
DSM 4 definition of delirium, Ely et al. were able to demonstrate
that nurses and physicians were able to use a screening test with
high sensitivity and specificity in the ICU.74 The Confusion Assessment Method (CAM) test was developed and tested in nonintubated patients previously. The following four features of delirium
were examined: acute onset, inattention, disorganized thinking,
and altered level of consciousness. The inattention test, the Attention Screening Examinations (ACE), was modified to allow nonverbal answers. Patients nodded or squeezed the examiners hands
to answer questions. In 1996, nursing research led to the creation
and validation of the NEECHAM confusion scale.75 Without formally addressing acuity, the NEECHAM index also examined inattention, level of consciousness, and disorganization but included a
subjective examination of the patients motoric behavior. Vital signs
including oxygen saturation and urinary continence along with
vital sign stability were also included in the NEECHAM scale. The
NEECHAM scale returns a range of possibilities including normal,
at risk, early, or mild delirium and moderate to severe delirium.
When compared side by side76 in nonintubated patients, the
two screening tools resulted in identical abilities to identify the
severely delirious patient. As expected there were no mild or at
risk patients identified by the CAM ICU. The CAM ICU rating has
been shown to be reliable in trauma patients in a study that documented the implementation and subsequent compliance with
the scale.77 The postimplementation follow-up was particularly
telling. Almost half of the nurses felt the scale did not enhance
patient care since the physicians did not modify treatment based
on the results.
Answer: The rate of delirium is high and underdiagnosed.
Identify delirium with a standardized exam or scale (Grade A recommendation, Level 1b evidence).
4. How should delirium be treated in the ICU?
Treatment of delirium is based initially at reducing or modifying any of the existing risk factors (Table 116.2). The APA in
their guidelines in 1999,78 which was updated in 2004,79 describe
addressing the somatic risk factorsorienting the patients after
providing hearing and visual aids, and re-establishing a normal
sleep-wake cycle. Inouye had studied prospectively, in a nonrandomized fashion, the usual care of geriatric patients and care in
a specialized unit where particular efforts were made to reorient patients, reduce visual and hearing impairment, re-establish
a normal sleep-wake cycle, and aggressively treat dehydration.80
The intervention patients had a relative reduction of approximately 30% of the development of delirium, the number of days
with delirium, and the total number of episodes. The guidelines

5/22/2012 6:19:41 PM

922

Surgery: Evidence-Based Practice

Table 116.2 Partial List of Risk Factors for Delirium


Predisposing pPatient factors:
Advanced age
Dementia
CNS and psychiatric disease
Medical comorbidities: renal, hepatic, etc. diseases
Drug or alcohol dependence
Vision or hearing impairment
Medical factors:
Trauma
Severe illness
Sleep deprivation
Polypharmacy
Psychotropic Rx (including opiates)

suggest that if delirium cannot be treated nonpharmacologically


then haloperidol is indicated. One retrospective chart analysis
found the administration of haloperidol in critical care patients to
be associated with reduced mortality.81 A postulated mechanism
was the early treatment of delirium, which reduced the associated morbidity and mortality. In a prospective, randomized and
blinded study, Kalisvaarts group gave hip surgery patients low
dose haloperidol or placebo.82 Although no change in the incidence of delirium was observed, the duration of delirium was
halved, possibly contributing to significant reduction in LOS.
Answer: Treat delirium with environmental changes (risk
factor reduction or elimination) then haloperidol (Grade A recommendation, Level 1b evidence).
In their 2007 report, the WHO suggested that just under half
of adults globally consume alcohol, amounting to approximately
2 billion adults worldwide. In addition, since in many developing
and developed countries, alcohol is produced by individuals on
a home or craft basis, the above statistic may be an important
underestimate.83 A patients decompensation secondary to alcohol withdrawal may be the first presentation of alcohol-related
diseasethus an opportunity for the intensivist as a public health
advocate.
Studies have identified alcohol as an important factor in motor
vehicle accidents, in nearly half of blunt head and spine injuries,84,85
and more than half of trauma patients. Alcohol in the trauma
patient is associated with delirium and longer ICU and hospital
stays.86 High consumption of alcohol in surgical patients is associated with higher rates of ICU admission as well as mortality.87

Benzodiazepine has been the standard of care for alcohol


withdrawal. Kahn et al.s retrospective chart review, comparing benzodiazepine use in the emergency room in two Canadian
urban hospitals for alcohol withdrawal patients, noted that one
hospital administered a three times higher dose of benzodiazepine. Patients who received the higher doses had significantly
fewer seizures without a longer LOS.88 A more formal although
still retrospective study by Gold et al. examined the total drug
doses and effects before and after the implementation of guidelines
for escalating doses of benzodiazepines.89 Despite receiving twice
the amount of benzodiazepines than historical controls, patients
treated with the new protocol were less likely to be intubated, and
thus less likely to suffer the complications of intubation and ventilation. In summary, individualized symptom-based management
with liberal benzodiazepine use is more effective than the previously common practice of pretreating withdrawal in a prophylactic fashion. Prospective randomized control trials have supported
the same conclusions.90,91 Symptom-driven therapy reduces the
possibility of overmedicating the at-risk-for-withdrawal patient
that can occur with regularly scheduled prophylactic therapy. The
most commonly studied tool for the diagnosis and monitoring
of withdrawal is the well-validated Clinical Institute Withdrawal
Assessment for Alcohol (revised) or CIWA-Ar.92 It is a multidimensional scale that rates ten objective and subjective symptoms
of withdrawal. With a total possible score of 67, many centers
continue to dose benzodiazepines hourly until the score falls under
10 to 15 for patients at risk of withdrawal seizures.
When the topic was reviewed by the Cochrane group, benzodiazepines, not surprisingly, were found to be superior to placebo.
Using the narrow assessment of seizure control in withdrawal,
benzodiazepines were not necessarily superior to other treatments such as anticonvulsants.93 However, since benzodiazepines
provide more than seizure control, they are more effective than
anticonvulsants in treating alcohol withdrawal.
Answer: Treat delirium tremens (alcohol withdrawal) with
benzodiazepines in a symptom-driven approach (Grade B recommendation, Level 2b evidence).

CONCLUSION
Appropriate sedation and treatment of delirium in the ICU has the
potential to decrease ICU-related complications. Many clinicians
still sedate patients using informal measures, which may lead to
oversedation. In the same way that resuscitation of sepsis, myocardial infarction, and stroke have benefited from evidence-based
protocols, sedation in the ICU should follow the same model. As
clinicians adhere poorly to sedation protocols, research examining the use of target-controlled infusions in critical care may prove
fruitful. Many clinicians ignore delirium as a preventable illness.

Clinical Question Summary


Question

Answer

Grade of
References
Recommendation

1 What are the


consequences of
oversedation in the
ICU?

Oversedation leads to prolonged mechanical ventilation and


its associated complications such as ventilator-associated
pneumonia (VAP), increased ICU length of stay (LOS), and
increased financial costs.

Grade A, Level of
evidence 1b

26-29, 31,
32

(Continued)

PMPH_CH116.indd 922

5/22/2012 6:19:41 PM

Management of Sedation and Delirium in the ICU

923

(Continued)
Question

Answer

Grade of
References
Recommendation

2 How can sedation


needs be assessed?

Best approachask the patient! If unable, sedation requirements


should be re-examined frequently using a sedation scale. The
use of a sedation protocol or daily interruption of sedation
should be considered.

Grade A, Level of
evidence 1b

18-24, 33

3 How can sedation


needs be managed?

Propofol or dexmedetomidine should be reserved for ICU


sedation where the duration is thought to be no more than
2448 h, Propofol should be given at an infusion rate of no
more that 45 mg/kg/h.

Grade C, Level of
evidence 4

45, 50

4 What is the role of


target-controlled
infusions for sedation
in the ICU?

TCI has been proven noninferior to human titration in general


anesthesia applications. Use in ICU should be considered.

Grade B, Level of
evidence 2b

62

5 How is delirium
identified in critically
ill patients?

The rate of delirium is high and under diagnosed. Identify delirium


with a standardized examination or scale.

Grade A, Level of
evidence 1b

66, 74-77

6 How should delirium


be treated in the
ICU?

Treat delirium with environmental changes (risk factor reduction


or elimination) then haloperidol.
Treat delirium tremens (alcohol withdrawal) with benzodiazepines
in a symptom-driven approach.

Grade A, Level of
evidence 1b
Grade B, Level of
evidence 2b

78-81

REFERENCES
1. Ballard N, Robley L, Barrett D, et al . Patients recollections of
therapeutic paralysis in the intensive care unit. Am J Crit Care.
2006;15(1):86-94.
2. Magarey JM, McCutcheon HH. Fishing with the dead
recall of memories from the ICU. Intensive Crit Care Nurs.
2005;21(6):344-354.
3. Granberg A, Engberg IB, Lundberg D. Acute confusion and
unreal experiences in intensive care patients in relation to the ICU
syndrome. Part II. Intensive Crit Care Nurs. 1999;15(1):19-33.
4. Diagnostic and Statistical Manual of Mental Disorders. 4th ed.,
text revision. Washington: American Psychiatric Association; 2000.
5. Lipowsky, Delirium, ZJ. Acute Brain Failure in Man. Springfield,
IL: Charles C. Thomas; 1980.
6. Jacobi J, Fraser GL, Coursin DB, et al. Task Force of the American College of Critical Care Medicine (ACCM) of the Society of
Critical Care Medicine (SCCM), American Society of HealthSystem Pharmacists (ASHP), American College of Chest Physicians. Clinical practice guidelines for the sustained use of
sedatives and analgesics in the critically ill adult. Crit Care Med.
2002;30(1):119-141. No abstract available. Erratum in: Crit Care
Med. 2002;30(3):726.
7. Mehta S, Burry L, Fischer S, et al.; Canadian Critical Care Trials Group. Canadian survey of the use of sedatives, analgesics,
and neuromuscular blocking agents in critically ill patients. Crit
Care Med. 2006;34(2):374-380.
8. Ely EW, Stephens RK, Jackson JC, et al. Current opinions regarding the importance, diagnosis, and management of delirium in
the intensive care unit: a survey of 912 healthcare professionals.
Crit Care Med. 2004;32(1):106-112.
9. Gilron I. Gabapentin and pregabalin for chronic neuropathic
and early postsurgical pain:current evidence and future directions. Curr Opin Anaesthesiol. 2007;20(5):456-472.

PMPH_CH116.indd 923

88-91

10. Tiippana EM, Hamunen K, Kontinen VK, Kalso E. Do surgical patients benefit from perioperative gabapentin/pregabalin?
A systematic review of efficacy and safety. Anesth Analg. 2007;
104(6):1545-1556.
11. Peng PW, Wijeysundera DN, Li CC. Use of gabapentin for
perioperative pain controla meta-analysis. Pain Res Manag.
2007;12(2):85-92.
12. Smith H, Elliott J. Alpha(2) receptors and agonists in pain management. Curr Opin Anaesthesiol. 2001;14(5):513-518.
13. Martin E, Ramsay G, Mantz J, et al. The role of the alpha2adrenoceptor agonist dexmedetomidine in postsurgical sedation
in the intensive care unit. J Intensive Care Med. 2003;18(1):29-41.
14. Bracco D, Noiseux N, Dubois MJ, et al.; PeriCARG Group, Montral. Epidural anesthesia improves outcome and resource use in
cardiac surgery: a single-center study of a 1293-patient cohort.
Heart Surg Forum. 2007;10(6):E449-458.
15. Papaioannou A, Fraidakis O, Michaloudis D, et al. The impact of
the type of anaesthesia on cognitive status and delirium during
the first postoperative days in elderly patients. Eur J Anaesthesiol.
2005;22(7):492-499.
16. Bryson GL, Wyand A. Evidence-based clinical update: general
anesthesia and the risk of delirium and postoperative cognitive
dysfunction. Can J Anaesth. 2006;53(7):669-677.
17. Wu CL, Hsu W, Richman JM, et al. Postoperative cognitive
function as an outcome of regional anesthesia and analgesia. Reg
Anesth Pain Med. 2004;29(3):257-268.
18. De Jonghe B, Cook D, Appere-De-Vecchi C, et al. Using and
understanding sedation scoring systems: a systematic review.
Intensive Care Med. 2000;26(3):275-285.
19. Ramsay MA, Savege TM, Simpson BR, et al. Controlled sedation
with alphaxalone-alphadolone. Br Med J. 1974;2(5920):656-659.
20. Ambuel B, Hamlett KW, Marx CM, et al. Assessing distress in
pediatric intensive care environments: the COMFORT scale. J
Pediatr Psychol. 1992;17(1):95-109.

5/22/2012 6:19:41 PM

924

Surgery: Evidence-Based Practice

21. Riker RR, Fraser GL, Cox PM. Continuous infusion of haloperidol controls agitation in critically ill patients. Crit Care Med.
1994;22(3):433-440.
22. Devlin JW, Boleski G, Mlynarek M, et al. Motor Activity Assessment Scale: a valid and reliable sedation scale for use with
mechanically ventilated patients in an adult surgical intensive
care unit. Crit Care Med. 1999;27(7):1271-1275.
23. Sessler CN, Gosnell MS, Grap MJ, et al. The Richmond AgitationSedation Scale: validity and reliability in adult intensive
care unit patients. Am J Respir Crit Care Med. 2002;166(10):
1338-1344.
24. De Jonghe B, Cook D, Griffith L, et al. Adaptation to the Intensive Care Environment (ATICE): development and validation
of a new sedation assessment instrument. Crit Care Med. 2003;
31(9):2344-2354.
25. Vincent JL. Give you patient a fast hug (at least) once a day. Crit
Care Med. 2005;33(6):1225-1229.
26. Papadimos TJ, Hensley SJ, Duggan JM, et al. Implementation of
the FASTHUG concept decreases the incidence of ventilatorassociated pneumonia in a surgical intensive care unit. Patient
Saf Surg. 2008;2:3.
27. Adam C, Rosser D, Manji M. Impact of introducing a sedation
management guideline in intensive care. Anaesthesia. 2006;
61(3):260-263.
28. De Jonghe B, Bastuji-Garin S, Fangio P, et al. Sedation algorithm
in critically ill patients without acute brain injury. Crit Care Med.
2005;33(1):120-127.
29. Marshall J, Finn CA, Theodore AC. Impact of a clinical pharmacist-enforced intensive care unit sedation protocol on duration of mechanical ventilation and hospital stay. Crit Care Med.
2008;36(2):427-433.
30. Kress JP, Pohlman AS, OConnor MF, et al. Daily interruption of
sedative infusions in critically ill patients undergoing mechanical ventilation. N Engl J Med. 2000;342(20):1471-1477.
31. Resar R, Pronovost P, Haraden C, et al. Using a bundle approach
to improve ventilator care processes and reduce ventilatorassociated pneumonia. Jt Comm J Qual Patient Saf. 2005;31(5):
243-248.
32. Youngquist P, Carroll M, Farber M, et al. Implementing a ventilator bundle in a community hospital. Jt Comm J Qual Patient
Saf. 2007;33(4):219-225.
33. de Wit M, Gennings C, Jenvey WI, et al. Randomized trial comparing daily interruption of sedation and nursing implemented
sedation algorithm in medical intensive care unit patients. Crit
Care. 2008;12(3):R70.
34. Skoglund K, Enblad P, Marklund N. Effects of the neurological
wake-up test on intracranial pressure and cerebral perfusion
pressure in brain-injured patients Neurocrit Care. 2009;11(2):
135-142.
35. Dundee JW. New i.v. anaesthetics. Br J Anaesth. 1979;51(7):
641-648.
36. Rogers KM, Dewar KM, McCubbin TD, et al. Preliminary experience with ICI 35 868 as an i.v. induction agent: comparison
with althesin. Br J Anaesth. 1980;52(8):807-810.
37. Rutter DV, Morgan M, Lumley J, et al. ICI 35868 (Diprivan): a
new intravenous induction agent. A comparison with methohexitone. Anaesthesia. 1980;35(12):1188-1192.
38. Girard TD, Kress JP, Fuchs BD, et al. Efficacy and safety of a
paired sedation and ventilator weaning protocol for mechanically ventilated patients in intensive care (Awakening and
Breathing Controlled trial): a randomised controlled trial. Lancet. 2008;371(9607):126-134.

PMPH_CH116.indd 924

39. Fong JJ, Kanji S, Dasta JF, et al. Propofol associated with a shorter
duration of mechanical ventilation than scheduled intermittent
lorazepam: a database analysis using Project IMPACT. Ann
Pharmacother. 2007;41(12):1986-1991.
40. Parke TJ, Stevens JE, Rice AS, et al. Metabolic acidosis and fatal
myocardial failure after propofol infusion in children: five case
reports. BMJ. 1992;305(6854):613-616.
41. Wysowski DK, Pollock ML. Reports of death with use of propofol (Diprivan) for nonprocedural (long-term) sedation and literature review. Anesthesiology. 2006;105(5):1047-1051.
42. Kam PC, Cardone D. Propofol infusion syndrome. Anaesthesia.
2007;62(7):690-701.
43. Vasile B, Rasulo F, Candiani A, et al. The pathophysiology of
propofol infusion syndrome: a simple name for a complex syndrome. Intensive Care Med. 2003;29(9):1417-1425.
44. Jacobi J, Fraser GL, Coursin DB, et al.; Task Force of the American
College of Critical Care Medicine (ACCM) of the Society of Critical Care Medicine (SCCM), American Society of Health-System
Pharmacists (ASHP), American College of Chest Physicians. Clinical practice guidelines for the sustained use of sedatives and analgesics in the critically ill adult. Crit Care Med. 2002;30(1):119-141.
45. Diprivan 1% (Propofol injection 10mg/mL) drug monograph.
AstraZenica Canada, Mississauga, 2004.
46. Coursin DB, Coursin DB, Maccioli GA. Dexmedetomidine. Curr
Opin Crit Care. 2001;7(4):221-226.
47. Vandermeulen E. Systemic analgesia and co-analgesia. Acta
Anaesthesiol Belg. 2006;57(2):113-120.
48. Weinbroum AA, Ben-Abraham R. Dextromethorphan and dexmedetomidine: new agents for the control of perioperative pain.
Eur J Surg. 2001;167(8):563-569.
49. Venn RM, Grounds RM. Comparison between dexmedetomidine and propofol for sedation in the intensive care unit: patient
and clinician perceptions. Br J Anaesth. 2001;87(5):684-690.
50. Precedex (dexmedetomidine) drug monograph. Hospira, Lake
Forrest IL, 2009.
51. Venn M, Newman J, Grounds M. A phase II study to evaluate the
efficacy of dexmedetomidine for sedation in the medical intensive care unit. Intensive Care Med. 2003;29(2):201-207.
52. Shehabi Y, Ruettimann U, Adamson H, et al. Dexmedetomidine
infusion for more than 24 hours in critically ill patients: sedative and cardiovascular effects. Intensive Care Med. 2004;30(12):
2188-2196.
53. Weinger MB, Reddy SB, Slagle JM. Multiple measures of anesthesia workload during teaching and nonteaching cases. Anesth
Anag. 2004;98:1419-1425.
54. Beaulieu Y. Bedside echocardiography in the assessment of the
critically ill. Crit Care Med. 2007;35(5 Suppl):S235-249.
55. Weinger MB, Herndon OW, Zornow MH, et al. An objective
methodology for task analysis and workload assessment in anesthesia providers. Anesthesiology. 1994;80(1):77-92.
56. Weinger MB, Herndon OW, Gaba DM. The effect of electronic
record keeping and transesophageal echocardiography on task
distribution, workload, and vigilance during cardiac anesthesia.
Anesthesiology. 1997;87(1):144-155.
57. Sadean MR, Glass PS. Pharmacokinetic-pharmacodynamic
modeling in anesthesia, intensive care and pain medicine. Curr
Opin Anaesthesiol. 2009;22(4):463-468.
58. Egan TD. Target-controlled drug delivery: progress toward an
intravenous vaporizer and automated anesthetic administration. Anesthesiology. 2003;99(5):1214-1219.
59. Chen G, Buell O, Gruenewald M, et al. A comparison between
target-controlled and manually controlled propofol infusions in

5/22/2012 6:19:41 PM

Management of Sedation and Delirium in the ICU

60.

61.

62.

63.

64.

65.

66.

67.

68.

69.

70.

71.

72.

73.
74.

75.

76.

PMPH_CH116.indd 925

patients undergoing routine surgical procedures. Eur J Anaesthesiol. 2009;26(11):928-935.


Moerman AT, Herregods LL, De Vos MM, et al. Manual versus
target-controlled infusion remifentanil administration in spontaneously breathing patients. Anesth Analg. 2009;108(3):828-834.
Ebeling BJ, Mller W, Tonner P, et al. Adaptive feedback-controlled
infusion versus repetitive injections of vecuronium in patients
during isoflurane anesthesia. Clin Anesth. 1991;3(3):181-185.
Leslie K, Clavisi O, Hargrove J. Target-controlled infusion
versus manually-controlled infusion of propofol for general
anaesthesia or sedation in adults. Cochrane Database Syst Rev.
2008;3:CD006059.
Puri GD, Kumar B, Aveek J. Closed-loop anaesthesia delivery
system (CLADS) using bispectral index: a performance assessment study. Anaesth Intensive Care. 2007;35(3):357-362.
Agarwal J, Puri GD, Mathew PJ. Comparison of closed loop vs.
manual administration of propofol using the Bispectral index in
cardiac surgery. Acta Anaesthesiol Scand. 2009;53(3):390-397.
McMurray TJ, Johnston JR, Milligan KR, et al. Propofol sedation
using Diprifusor target-controlled infusion in adult intensive
care unit patients. Anaesthesia. 2004;59(7):636-641.
Milbrandt EB, Deppen S, Harrison PL, et al. Costs associated
with delirium in mechanically ventilated patients. Crit Care
Med. 2004;32(4):955-962.
Dasgupta M, Dumbrell AC. Preoperative risk assessment for
delirium after noncardiac surgery: a systematic review. J Am
Geriatr Soc. 2006;54(10):1578-1589.
Pandharipande P, Shintani A, Peterson J, et al. Lorazepam is an
independent risk factor for transitioning to delirium in intensive
care unit patients. Anesthesiology. 2006;104(1):21-26.
Thomason JW, Shintani A, Peterson JF, et al. Intensive care unit
delirium is an independent predictor of longer hospital stay: a
prospective analysis of 261 non-ventilated patients. Crit Care.
2005;9(4):R375-381.
Milbrandt EB, Deppen S, Harrison PL, et al. Costs associated
with delirium in mechanically ventilated patients. Crit Care
Med. 2004;32(4):955-962.
Jackson JC, Gordon SM, Hart RP, et al. The association between
delirium and cognitive decline: a review of the empirical literature. Neuropsychol Rev. 2004;14(2):87-98.
Arora VM, Johnson M, Olson J, et al. Using assessing care of
vulnerable elders quality indicators to measure quality of hospital care for vulnerable elders. J Am Geriatr Soc. 2007;55(11):
1705-1711.
Inouye SK. Delirium in older persons. N Engl J Med. 2006ar
16;354(11):1157-65.
Ely EW, Inouye SK, Bernard GR, et al. Delirium in mechanically ventilated patients: validity and reliability of the confusion assessment method for the intensive care unit (CAM-ICU).
JAMA. 2001;286(21):2703-2710.
Neelon VJ, Champagne MT, Carlson JR, et al. The NEECHAM
Confusion Scale: construction, validation, and clinical testing.
Nurs Res. 1996;45(6):324-330.
Van Rompaey B, Schuurmans MJ, Shortridge-Baggett LM, et al.
A comparison of the CAM-ICU and the NEECHAM Confusion

77.

78.
79.

80.

81.

82.

83.

84.
85.
86.

87.

88.

89.

90.

91.

92.

93.

925

Scale in intensive care delirium assessment: an observational


study in non-intubated patients. Crit Care. 2008;12(1):R16.
Soja SL, Pandharipande PP, Fleming SB, et al. Implementation,
reliability testing, and compliance monitoring of the Confusion Assessment Method for the Intensive Care Unit in trauma
patients. Intensive Care Med. 2008 Feb 23.
Anonymous. Practice guideline for the treatment of patients
with delirium. Am J Psych. 1999;156(5 Suppl):1-20.
Cook IA. Guideline Watch: Practice Guideline for the Treatment
of Patients with Delirium. Arlington VA: American Psychiatric
Association, 2004.
Inouye SK, Bogardus ST Jr, Charpentier PA, et al. A multicomponent intervention to prevent delirium in hospitalized older
patients. N Engl J Med. 1999;340(9):669-676.
Milbrandt EB, Kersten A, Kong L, et al. Haloperidol use is associated with lower hospital mortality in mechanically ventilated
patients. Crit Care Med. 2005;33(1):226-229.
Kalisvaart KJ, de Jonghe JF, Bogaards MJ, et al. Haloperidol
prophylaxis for elderly hip-surgery patients at risk for delirium: a randomized placebo-controlled study. J Am Geriatr Soc.
2005;53(10):1658-1666.
WHO Expert Committee on Problems Related to Alcohol Consumption. Second Report. Geneva. World Health Organization.
2007 (WHO Technical Report Series 944).
Golan JD, Marcoux J, Golan E, et al. Traumatic brain injury in
intoxicated patients. J Trauma. 2007;63(2):365-369.
Kluger Y, Jarosz D, Paul DB, et al. Diving injuries: a preventable
catastrophe. J Trauma. 1994;36(3):349-351.
Blondell RD, Powell GE, Dodds HN, et al. Admission characteristics of trauma patients in whom delirium develops. Am J Surg.
2004;187(3):332-337.
Delgado-Rodrguez M, Gmez-Ortega A, Mariscal-Ortiz M,
et al. Alcohol drinking as a predictor of intensive care and hospital mortality in general surgery: a prospective study. Addiction.
2003;98(5):611-616.
Kahan M, Borgundvaag B, Borsoi D, et al. Treatment variability
and outcome differences in the emergency department management of alcohol withdrawal. CJEM. 2005;7(2):87-92.
Gold JA, Rimal B, Nolan A, et al. A strategy of escalating doses of
benzodiazepines and Phenobarbital administration reduces the
need for mechanical ventilation in delirium tremens. Crit Care
Med. 2007;35(3):724-730.
Saitz R, Mayo-Smith MF, Roberts MS, et al. Individualized treatment for alcohol withdrawal. A randomized double-blind controlled trial. JAMA. 1994;272(7):519-523.
Daeppen JB, Gache P, Landry U, et al. Symptom-triggered vs
fi xed-schedule doses of benzodiazepine for alcohol withdrawal:
a randomized treatment trial. Arch Intern Med. 2002;162(10):
1117-1121.
Sullivan JT, Sykora K, Schneiderman J, et al. Assessment of alcohol withdrawal: the revised clinical institute withdrawal assessment for alcohol scale (CIWA-Ar). Br J Addict. 1989;84(11):
1353-1357.
Ntais C, Pakos E, Kyzas P, et al. Benzodiazepines for alcohol
withdrawal. Cochrane Database Syst Rev. 2005;3:1-97.

5/22/2012 6:19:41 PM

CHAPTER 117
V
Postoperative Respiratory
Failure
Mollie M. James and Gregory J. Beilman

smoking cessation reduces the risk of postoperative pulmonary


complications.3 However, smoking cessation for 1- to 8-weeks
duration prior to surgery is associated with even higher risks than
continuing to smoke through the perioperative period.4
Other patient-related risk factors include age over 60, ASA
Class II or higher, and patients who are dependent on assistance
for activities of daily living. Patients with weight loss, or other
conditions suggesting hypoalbuminemia should be investigated.
An albumin level less than 35 g/L is associated with respiratory
failure.3,4 Asthma does not contribute to risk, with an incidence of
bronchospasm less than 2%. Morbid obesity is not a clinical risk
factor; however, obstructive sleep apnea as a comorbid condition
can increase the risk of postoperative hypoxemia.3,4
Many types of surgery increase the risk of respiratory failure, including surgery of the upper aerodigestive tract abdominal,
thoracic, neurosurgery, and vascular procedures. The incidence of
respiratory failure begins to increase after 3 hours of anesthesia.
Having a general anesthesia or a procedure under emergent conditions is also associated with a higher incidence of postoperative
respiratory failure.3
Preoperative evaluation is generally limited to a physical
examination. If there is any suspicion of acute infection, this
should be worked up appropriately. However, the routine use of
chest radiographs is discouraged since it very rarely changes the
operative plan. Pulmonary function tests are not helpful in identifying patients who will have postoperative problems.3
The most effective ways to reduce the risk of postoperative
complications is to encourage patients to cough and deep breathe.
Some patients who are unable to do this voluntarily may benefit
from continuous positive airway pressure (CPAP). Pain control
should be adequate to allow for deep breathing. The use of epidural catheters in the postoperative period has been associated with
improved pulmonary function. This difference has not reached
statistical significance.3
Venous thromboembolism (VTE) prophylaxis is a focus of
national surgery quality improvement efforts. The incidence of
VTE in general surgery patients is 15% to 40%, and the risk of fatal

General surgeons are increasingly encountering patients with


complex medical problems requiring surgical intervention. As
this trend continues, surgeons can expect that some patients will
develop postoperative respiratory failure. Surgeons must maintain
familiarity with the techniques to stabilize patients with evidence
of respiratory failure and strategies to minimize mechanical ventilation. Whether the surgeon is primarily managing the patients
pulmonary status or working with intensivists, a thorough understanding of the treatment for respiratory failure improves overall
quality of care.
The incidence of respiratory failure in hospitalized patients
is about 15% with a mortality risk of 25% to 40% for patients
requiring mechanical ventilation. Respiratory failure affects the
elderly disproportionately, with higher incidence and mortality
for patients over the age of 65.1
1. Can you assess the risk for respiratory failure after an elective surgery?
In many practices, a significant amount of effort is spent on the preoperative cardiac risk assessment with less attention on a patients
respiratory status. Pulmonary complications are more common
in the postoperative setting, and the need for mechanical ventilation carries a mortality risk.2 Surgeons should be attentive to the
risk factors for postoperative pulmonary complications during
preoperative evaluation. Some patient-related risk factors can be
optimized to reduce risk. Procedure-related risk is generally not
modifiable, but alterations in the procedure to decrease perioperative pulmonary compromise may improve patient outcomes.
When evaluating preoperative risk, there is no patient that
is absolutely precluded from a surgical procedure (outside of the
chest) due to the severity of underlying lung disease. Chronic
obstructive pulmonary disease (COPD) and smoking both
increase risk of postoperative respiratory failure. Patients with
COPD can be optimized by adding bronchodilators to improve
airflow. Underlying bronchitis or pneumonia should be treated
preoperatively. If surgery can be delayed for more than 8 weeks,

926

PMPH_CH117.indd 926

5/22/2012 8:01:35 PM

Postoperative Respiratory Failure

pulmonary embolism (PE) 2% to 3%.5 Patients should undergo a


risk assessment on admission. Patients who are considered moderate or higher risk, over age 40, having a major procedure, or have
additional risk factors should be given prophylaxis with subcutaneous low-dose unfractionated heparin (LDUH) or low-molecular
weight heparin (LMWH). Prophylaxis has been shown to reduce
the risk of VTE with a minimal increase in bleeding risk.6

927

reduction in minute ventilation. Air-trapping can reduce the tidal


volume and cause accumulation of CO2. Narcotic pain medications, splinting, reduction in mental status, and neuromuscular
disorders can reduce respiratory drive and minute ventilation,
leading to hypercapneic respiratory failure.
3. What are the treatments for each type of respiratory failure?

Atelectasis
Recommendations
1. Provide a thorough physical examination, and restrict
pulmonary function tests and imaging studies for patients
with shortness of breath of unknown etiology (Grade C).
2. Assess the risk and modify the procedure to minimize the risk
of pulmonary complications (Grade C).
3. Provide smoking cessation counseling if the surgery can be
delayed for >8 weeks (Grade B).
4. Coughing and deep breathing after surgery can minimize
pulmonary complications (Grade B).
2. What are the types and causes of respiratory failure that can
occur postoperatively?
Patients experience respiratory problems for three main reasons: airway obstruction, inadequate oxygenation, or inadequate
ventilation.
Airway obstruction is the least common of the three, but
poses an emergency that requires rapid intervention. One cause
of airway obstruction is airway edema from either intraoperative positioning or fluid overload. Angioedema is uncommon, but
should be considered if there is airway compromise in the face of
starting commonly associated medications (i.e., ACE inhibitors).
Airway edema is usually identified by a lack of cuff leak at the
bedside prior to extubation. In this instance, the patient should
remain intubated until the edema improves. The patient is positioned at 90o and intravenous steroids are commonly given to
reduce airway swelling. Patients who do not develop a cuff leak
within a 7- to 10-day treatment period may require a tracheotomy
to maintain a definitive airway.
A second type of airway obstruction is caused by postoperative bleeding in the neck. A hematoma can displace the trachea,
causing airway obstruction and making endotracheal intubation difficult or impossible. The first key to management in these
patients is opening the incision at the bedside to evacuate the
hematoma. Once this is done, supplemental oxygen can be supplied or the patient can be intubated. The patient should then be
transferred to the operating room for exploration and definitive
treatment.
Hypoxemic respiratory failure is an inability for oxygen to
diff use into the pulmonary circulation. This can be clinically
silent or the patient can become dyspneic with low oxygen saturations. Hypoxemic respiratory failure is generally due to a barrier
to oxygen diff usion into the bloodstream: interstitial lung disease,
infectious processes, inflammatory processes, aspiration pneumonitis, or PE. Increased oxygen consumption and high-minute
ventilation can also be a cause of respiratory failure.
Hypercapneic respiratory failure is the accumulation of carbon dioxide due to the inability to ventilate this by-product of
metabolism. Hypercapneic respiratory failure is generally due to a

PMPH_CH117.indd 927

Atelectasis is a significant problem in the postoperative setting.


Microcollapse of the alveoli can lead to hypoxemia, reduced compliance, and increased pulmonary vascular resistance. Approximately
90% of patients can have atelectasis, and the effect is seen as early as
5 minutes after induction of general anesthesia.7,8 Ventilating
patients with 100% fraction of inspired oxygen (FiO2) makes all the
alveolar gas available to be absorbed into the circulation, leading to
alveolar collapse. At lower fraction of inhaled oxygen, nitrogen gas
occupies a significant volume of the alveoli and keeps the alveoli
open as exhalation begins. Reducing FiO2 to the minimum necessary
to maintain oxygenation is one method to reduce atelectasis. The
addition of a sigh breath followed by 10-cm H2O PEEP extended
the time to recurrence of atelectasis from 5 to 40 minutes. 7

Hospital-Associated Pneumonia/VentilatorAssociated Pneumonia (HAP/VAP)


Patients with new onset hypoxemia with either a fever (temperature >101.5oF), new sputum production, productive cough, or
infi ltrate on chest radiograph should undergo evaluation for HAP.
HAP is defined as pneumonia diagnosed more than 48 hours
after admission or in association with a healthcare facility. (HCAP,
healthcare-associated pneumonia) Postoperative nosocomial pneumonia progresses to severe hypoxemia and the need for mechanical ventilation in 30% to 50% of patients.9 The Infectious Disease
Society of America recommends lower respiratory tract culture
(bronchoalveolar lavage or mini-BAL) upon recognition of illness
and before starting broad-spectrum antibiotics.
VAP is described as a new infiltrate, with fever, production
of sputum, and worsening oxygenation, with onset more than
48 hours after initiating mechanical ventilation. The estimated
incidence is 30% to 50%, with most diagnoses made in the first
3 days of ICU admission. Selection of an antimicrobial agent for
initial treatment of VAP should cover for resistant organisms if
the patient has significant risk factors. Koulenti et al. evaluated
the diagnosis and treatment of nosocomial pneumonia in 1089
ICU patients, and found that 69.5% had confirmatory isolates. Of
the isolates, 16% were methicillin-resistant Staphylococcus aureus
(MRSA), 23.1% were Pseudomonas aeruginosa, and 19.1% were
Acinetobacter baumannii.10
Several considerations must be given when treating HAP or
VAP. First, antibiotics should be started as soon as infection is suspected, especially in the face of hypotension. A review in Critical
Care Medicine, 2006, correlated the rate of death with delay in
antibacterial administration. Patients receiving appropriate antibiotics within 1 hour of diagnosis had a survival rate of 79.9%.
For each hour that medications were delayed, there was a decrease
in survival of 7.6%. In both univariate and multivariate analyses,
time to initiation of effective antimicrobial therapy was the strongest predictor of outcome.

5/22/2012 8:01:35 PM

928

Surgery: Evidence-Based Practice

Second, the initial antibacterial agent must have appropriate


coverage.11 Kumar et al. demonstrated a five-fold increase in mortality when inappropriate antimicrobials were initially selected
(10.3 vs. 52%, odds ratio 9.45, P < .0001). In a multivariate analysis,
inappropriate initial antimicrobial agent was most highly associated with death (odds ratio 8.99; 95% CI: 6.612.23).12
Prevention of VAP is a key component of the bundled vent
orders. The head of the bed should be elevated to 30o to 45o to
minimize reflux and aspiration. Oral cares consisting of twice
daily mouth washing with chlorhexidine gluconate should be
done to minimize oral contamination. Endotracheal tubes with
a subglottic suctioning prevent build-up of secretions along the
endotracheal tube and have shown some efficacy in reducing
VAP.13
The de-escalation and duration of antimicrobials is an important component of VAP treatment. A study including 138 surgical
ICU patients diagnosed with nosocomial pneumonia was evaluated by Eachempati et al. The group found no significant increase
in recurrent pneumonia (27.3% vs. 35.1%) or mortality (33.8% vs.
42.1%) in patients with de-escalation of their antimicrobial therapy
after initial appropriate therapy.14 Consideration should also be given
to discontinuing antimicrobial therapy after 72 hours if no infection is confirmed with cultures. A study by Aarts et al. evaluated 143
patients suspected of having a nosocomial pneumonia. Antimicrobials were continued in 59% of patients without positive cultures and
the mortality in this group was 32.1%, vs. 7.7% (OR 5.7, P = 0.005) in
the group in which antimicrobials were discontinued.15
In patients with a confirmed pneumonia, the duration of
therapy affects the recurrence rates of pneumonia. A head-to-head
comparison of 401 patients randomized to an 8-day or 15-day
course of antimicrobials demonstrated similar mortality rates
and incidence of recurrence in both groups. Patients who were in
the 8-day treatment group and had a recurrence were less likely to
have a resistant bacteria isolated (42.1% vs. 62%, P = .04). A 15-day
course is recommended for patients with isolate of nonfermenting
gram-negative bacilli due to higher recurrence rates in the group
treated with the shorter course (40.6% vs. 25.4%).16

Recommendations
1. Consider HAP/VAP for patients with fever, and 2/3 of the
following: productive cough or sputum generation, new or
worsening hypoxemia, and infi ltrates on chest radiograph
(Grade A).
2. Emprically treat with broad-spectrum antimicrobial agents
after lower respiratory tract sampling (MRSA and Pseudomonas
coverage if patient is at risk for exposure) (Grade A).
3. De-escalate or discontinue therapy as cultures are finalized
(Grade A).
4. Treat for 8 days, except for lactose-fermenting gram-negative
rods, which should be treated for 15 days (Grade A).
5. De-escalate antimicrobial therapy to narrow coverage once
cultures and sensitivities are finalized (Grade A).
6. Continuous aspiration of subglottic secretions can reduce the
risk of VAP (Grade A).
7. The head of the bed should be elevated to 30 to 45o to prevent
aspiration (Grade A).
8. Enteral feeding is the preferred form of nutrition in mechanically
ventilated patients (Grade A).

PMPH_CH117.indd 928

Acute Lung Injury/Adult Respiratory


Distress Syndrome
Acute lung injury (ALI) is a spectrum of severe lung disease that
accounts for more than 75,000 deaths annually. ALI is an interstitial process that limits oxygen diff usion from the alveoli into the
capillaries. It is defined as a ratio of the arterial partial pressure of
oxygen (PaO2) to the FiO2 between 200 and 300, suggesting moderate hypoxemia.17 Transfusion-related acute lung injury (TRALI)
is a variant of ALI, manifesting as severe hypoxemia after administration of blood products. It is self-limited and the patients need
supportive care for 48 to 72 hours.
Adult respiratory distress syndrome (ARDS) is defined as a
ratio of the arterial PaO2 to FiO2 less than 200. Additional criteria
needed to diagnose ARDS include new onset of bilateral infiltrates
on chest radiograph, normal left atrial pressures, and a predisposing
condition (sepsis, blood transfusion, pulmonary contusion or pneumonia, surgery, and many more processes that stimulate an inflammatory process in the body). The mortality of ARDS is around 40%
and has not changed significantly in the last 20 years.17

Pulmonary Embolism
Pulmonary embolism (PE) is a clot that travels into the pulmonary circulation causing high pulmonary artery pressures and
a ventilation-perfusion mismatch by shunting blood away from
oxygenated areas of the lung. Patients presenting with shock and
a pulmonary embolus have three- to seven-fold higher mortality.5
Most efforts should be spent on providing timely and appropriate
VTE prophylaxis.
Risk factors for PE include hospital admission, surgery,
obesity, malignancy, immobility, trauma, and hypercoagulable
states. If a patient has sudden onset of hypoxemia, chest pain,
or arrhythmia in the postoperative setting, PE must be considered in the differential diagnosis. Evaluation may include chest
radiograph, arterial blood gas, or venous duplex ultrasound of
the lower extremities. Although a CT angiogram is currently the
diagnostic gold standard, some patients will be too unstable or
have renal failure that would preclude use of intravenous contrast.
An echocardiogram is indicated for patients with suspected PE
who are too unstable for CT angiogram, or for patients with large
fi lling defects.5 If the patient has new onset right ventricular dilation, one can begin treating for PE until the diagnosis can be made
using a V:Q scan or CT angiogram.18
The treatment for PE includes immediate anticoagulation with
a heparin drip or low molecular weight heparin. Patients should
be ambulated early to prevent clot propagation.19 For patients with
hemodynamic instability and rapid clinical decompensation,
thrombolytics are indicated. An inferior vena cava fi lter should be
considered in patients with a contraindication to anticoagulation,
failure of medical anticoagulation, and hemodynamic instability
when thrombolytics are contraindicated.

Pulmonary Hypertension
Pulmonary hypertension is defined as elevation in the mean
pulmonary artery pressure above 25 mm Hg with normal left
heart function. These patients should avoid hypoxemia or acidosis, which induce pulmonary vasospasm and lead to clinical

5/22/2012 8:01:35 PM

Postoperative Respiratory Failure

decompensation.20 Acute changes in pulmonary pressures can


lead to right heart failure and cardiogenic shock due to lack of
flow through the lungs and into the left heart. Treatment priorities
include providing adequate preload as the patients cardiac output
will be dependent on adequate fi lling pressures. Next, selective
vasodilators, such as epoprostenol, can be given to reduce pulmonary vascular resistance.21 Inotropic agents, such as dobutamine
or milrinone, can be added to augment ventricular function and
increase pulmonary perfusion and cardiac output. Afterload
reduction and treatment of underlying conditions are a key component of treating acute pulmonary hypertension.

Neurogenic Disorders
Patients with neuromuscular disorders require close monitoring
and early support of ventilation for respiratory failure. Conditions that should be considered include neuropathy from diabetes
or critical illness, Guillain-Barre syndrome, myasthenia gravis,
and spinal cord injury. Respiratory failure can present as airway
obstruction from mental status change, secretions, or aspiration.
Hypoxemia is usually from interstitial processes. Patients with
peripheral neuropathy may initially compensate low tidal volume
with high respiratory rate. Patients with central neuropathy can
not compensate and are at risk for CO2 narcosis.
4. What types of respiratory support can by provided to augment pulmonary function?
Providing the appropriate supportive measures is the first priority in managing patients with respiratory failure. Patients should
be clinically stabilized before a diagnostic evaluation is initiated.
Supplemental oxygen can be administered to patients who are
awake with isolated hypoxemia. This can be administered through
a variety of cannulas and masks; each 1 L of flow increases the
FiO2 by 3%. The device used to supplement oxygen should be tailored to the patients need and tolerance of the device.
Noninvasive positive-pressure ventilation (NIPPV) is being
used with increasing frequency. Basically, a mask is applied to
the face so a ventilator can deliver an inspiratory pressure above
positive end expiratory pressure (PEEP) to augment the patients
ventilation and oxygenation. NIPPV is generally accepted for
two indications: congestive heart failure with acute volume overload and COPD exacerbation. Patients with respiratory failure of
unknown etiology should not be treated with NIPPV, except as
a means to preoxygenate before intubation. If NIPPV is utilized,
there should be specific, short-term endpoints. Failure to meet the
treatment goals should prompt endotracheal intubation. NIPPV is
contraindicated in patients with mental status changes, vomiting,
upper abdominal surgery, or profound hypoxemia. If a patient is
dependent on maximum NIPPV support for oxygenation, they
can rapidly desaturate during intubation. Extubation failure
should not be treated with NIPPV. Esteban et al. demonstrated
an increased mortality rate, delay to intubation, and no reduction
in reintubation for patients managed with NIPPV compared with
immediate reintubation.22
Mechanical ventilation via an endotracheal tube is the mainstay of therapy for patients with respiratory failure. Patients should
be intubated for severe or unexplained hypoxemia, hypercapnea,
tachypnea, sepsis or septic shock, and mental status change with
inability to protect the airway. Patients who are hemodynamically

PMPH_CH117.indd 929

929

unstable should have a secured airway so the patient can be stabilized. Standard modes of ventilation include pressure-control and
volume-control modes, which assist the patient with each inspiratory trigger.

Tidal Volume
The standard tidal volume setting was traditionally in the 10 to
12 mL/kg of ideal body weight. Over the last decade, the strategy has changed significantly, based on animal studies suggesting
ventilator-induced lung injury. The ARDSnet study was a multicenter trial that randomized patients with ARDS to receive standard ventilation (10 mL/kg I) or a low tidal volume (6 mL/kg)
based on ideal body weight. The study was stopped early due to the
mortality benefit seen with the low tidal volume group, 31% versus
39.8% (P = .007). Part of the benefit was felt to be due to lower peak
airway pressures 25 versus 33 cm H2O, with a reduction in barotrauma.23 Patients treated with low VT ventilation had a greater mean
standard deviation (SD) of the number of days free of mechanical ventilation (12 11 vs. 10 11 days, respectively; P = .007)
and a greater number of days free of non-pulmonary organ failure
(15 11 vs. 12 11 days, respectively; P = .006).23
Reducing the tidal volume reduces the minute ventilation as
well. In order to maintain adequate minute ventilation, the respiratory rate is increased. As the minute ventilation drops, CO2
can begin to accumulate. Permissive hypercapnea, allowing the
CO2 to climb in order to achieve low tidal volume ventilation, has
become a widely accepted practice. The goal is generally to keep
the pH >7.15 and add a sodium bicarbonate drip if needed to help
offset the respiratory acidosis.23
If a patient fails standard ventilation modes there are a number of salvage therapies that have been investigated. Highfrequency oscillatory ventilation is an ideal mode of ventilation
for ARDS patients as it is the natural culmination of low tidal volume ventilation. This mode of ventilation rapidly delivers small
tidal volumes that are typically 1 to 5 mL/kg,24 possibly improving
gas exchange and reducing ventilator-induced lung injury. However, both randomized controlled trials (RCTs) that have been
conducted to date to evaluate the efficacy of high-frequency oscillatory ventilation in the treatment of ARDS have failed to demonstrate an improvement in mortality.25,26 Similarly, to date most of
the other promising interventions found to reduce lung injury and
improve outcome in animal studies (e.g., prone position, surfactant supplementation, nitric oxide, lung recruitment maneuvers)
have not been found to significantly improve patient survival or
outcome on adult intensive care patients with ALI/ARDS.27-34 In
randomized trial evaluating the use of prone positioning, Guerin
et al. found no reduction in mortality, but improved oxygenation
and a lower incidence of VAP.29
A recent meta-analysis on the use of steroids in ALI/ARDS
(nine randomized trials were selected using variable dose and
duration of steroids: four studies on preventative use of steroids
and five studies with steroid administration after onset of ARDS)
found some evidence that suggested that giving corticosteroids
to prevent the development of ARDS was actually associated
with the subsequent development of ARDS, and was also associated with a weakly increased risk of death in patients who subsequently developed ARDS.35 The authors also found that giving
corticosteroids after the onset of ARDS was associated with a

5/22/2012 8:01:35 PM

930

Surgery: Evidence-Based Practice

trend toward reduced mortality. Steroid therapy was also associated with substantially more ventilator-free days compared with
controls (mean difference 4.05 days, 95% credible interval, 0.22
8.71), no evidence was found of an association between odds of
mortality and time to treatment in hours, and corticosteroids
were not associated with increase in risk of infection; however, a
trend was found toward increased risk of infection with increasing steroid dose.
The role of extracorporeal membrane oxygenation (ECMO)
has not yet been validated for patients with ARDS. Only two RCTs
have been reported in the literature and both trials failed to demonstrate benefit on outcome or survival.36-38 ECMO has become
an option for patients with a reversible etiology of respiratory failure, as a bridge to definitive treatment or recovery. A review paper
from the University of Michigan demonstrated a 52% survival
when ECMO was utilized for ARDS.39 Similar results have been
demonstrated in small series.40,41

Recommendations
1. Low VT ventilation (68 mL/kg of predicted body weight)
should be the initial target in all patients with ALI/ARDS as
it is the only method of mechanical ventilation demonstrating
improved survival (Grade A).
2. Consider salvage therapies in patients with a reversible cause of
cardiorespiratory failure (Grade B).
5. How can the need for respiratory support be minimized?
Once the patient improves, the clinician must evaluate the patients
ability to be separated from mechanical support. A study by Esteban et al. compared four methods of weaning: intermittent mandatory ventilation (IMV), pressure support ventilation (PSV), and
spontaneous breathing trial (SBT) through a t-tube. They randomized 130 patients who failed an SBT to IMV, PSV, SBT twice daily,
or SBT once daily. The time to extubation was 4 days for IMV and
3 days for the other modes. In comparing once daily SBT to IMV,
the relative rate of extubation was 2.83 (P < .006) and compared
with PCV was 2.05 (P < .04). The timing of extubation was similar
whether the SBT was done once daily or more often.42
Daily screening for readiness of SBT is one approach to
reduce the use of mechanical ventilation. Ely et al. randomized
300 patients to standard therapy or daily screening for SBT.
After a successful SBT, each patient was further evaluated for
extubation potential. Th is protocolized intervention reduced

median ventilator days from 6 to 4.5 (P = .003) without increasing


complications, such as self-extubation.43
In order to facilitate weaning trials, the level of sedation
needs to allow active participation. Kress and colleagues compared 128 patients randomized to daily sedation holidays or
standard therapy. They found by holding the sedation every day
and allowing the patients to wake up, they reduced mechanical
ventilation days from 7.3 to 4.9 days (P = .004).44 Additional benefits included reduced ICU length of stay from 9.9 to 6.4 days
(P = .02) and fewer studies ordered to evaluate mental status
changes (6 vs. 16).44
More alert patients allow for more interaction, and early physical therapy (PT) and occupational therapy (OT) has been shown
to reduce the time of MV. Schweickert et al. evaluated the effect of
integrating PT/OT within 48 hours of MVthis simple intervention led to significantly more patients returning home (59% vs. 35%,
P = .02). Patients had a shorter duration of delirium (2 vs. 4 days,
P = .02), and more ventilator-free days (23.5 vs. 21.1 days, P = .05).
Only 4% of patients were found to be intolerant of the therapy sessions, mostly due to ventilator asynchrony.45

Recommendations
1. Daily SBTs should be done to reduce the time a patient is
supported by mechanical ventilation (Grade A).
2. Hold sedation once daily until the patient is awake, then resume
as needed at a lower level to reduce the number of ventilator
days (Grade A).
3. Initiate PT/OT within 48 hours of mechanical ventilation to
reduce the number of ventilator days (Grade A).

CONCLUSION
Surgeons can take an active role in managing patients with postoperative respiratory failure. Understanding the types of respiratory failure, need for ventilatory support, and causes of respiratory
failure will improve patient care and family communication during this stressful time. Appropriate use of diagnostic studies will
more effectively outline the etiology of respiratory failure and lead
to early and effective treatment. Adopting modern principles of
vent weaning, sedation, and physical therapy will allow surgeons
to become good stewards of high-acuity resources and have the
optimal outcomes for patients.

Clinical Question Summary


Question

Recommendation

1 Can you assess


the risk for
respiratory failure
after an elective
surgery?

Provide a thorough physical examination, and restrict


pulmonary function tests and imaging studies for
patients with shortness of breath of unknown
etiology.
Assess the risk and modify the procedure to minimize
the risk of pulmonary complications.

Level of Grade of
References
Evidence Recommendation
III

3, 4

(Continued)

PMPH_CH117.indd 930

5/22/2012 8:01:36 PM

Postoperative Respiratory Failure

931

(Continued)
Question

Recommendation

2 Can you prevent


respiratory failure
after an elective
general surgery?

Provide smoking cessation counseling if the surgery can


be delayed for >8 weeks.
Coughing and deep breathing after surgery can minimize
pulmonary complications.

II

3, 4

Chemical prophylaxis is indicated for patients at


moderate or high risk for VTE and acceptable
bleeding risk.

Consider HAP/VAP for patients with fever, and 2/3


of the following: productive cough or sputum
generation, new or worsening hypoxemia and
infiltrates on chest radiograph.
Empirically treat with broad-spectrum antimicrobial
agents after lower respiratory tract sampling
(MRSA and Pseudomonas coverage if pt is at risk for
exposure)
De-escalate or discontinue therapy as cultures are
finalized.
Treat for 8 days, except for lactose-fermenting gramnegative rods, which should be treated for 15 days.

II

10-13, 46

De-escalate antimicrobial therapy to narrow coverage


once cultures and sensitivities are finalized.

II

14

The duration of therapy should be 8-day course of


antimicrobials for nonresistant bacteria; 15 days for
resistant strains.

16

Continuous aspiration of subglottic secretions can


reduce the risk of VAP
Elevate the head of bed to 3045.
Enteral feeding is the preferred form of nutritional
support in mechanically ventilated patients.

13

The head of the bed should be elevated to 3045o to


prevent aspiration.

13

Enteral feeding is the preferred form of nutrition in


mechanically ventilated patients.

13

Low V T ventilation (68 mL/kg of predicted body


weight) should be the initial target in all patients with
ALI/ARDS as it is the only method of mechanical
ventilation demonstrating improved survival.

Ib

23

Consider salvage therapies in patients with a reversible


cause of cardiorespiratory failure.

II

24-34, 40, 41

Daily SBTs should be done to reduce the time a patient


is supported by mechanical ventilation.

Ia

43

Hold sedation once daily until the patient is awake, then


resume as needed at a lower level to reduce the
number of ventilator days.

44

Initiate PT/OT within 48 hours of mechanical ventilation


to reduce the number of ventilator days.

Ib

45

3 What is the
treatment for
HAP/VAP?

4 What is the
treatment/
prevention for
HAP/VAP?

5 What is the
treatment for
ARDS?

6 How can the need


for respiratory
support be
minimized?

PMPH_CH117.indd 931

Level of Grade of
References
Evidence Recommendation

5/22/2012 8:01:36 PM

932

Surgery: Evidence-Based Practice

REFERENCES
1. Behrendt CE. Acute respiratory failure in the United States: incidence and 31-day survival. Chest. 2000;118:1100-1105.
2. Lawrence VA, Dhanda R, Hilsenbeck SG, Page CP. Risk of pulmonary complications after elective abdominal surgery. Chest.
1996;110:744-750.
3. Qaseem A, Snow V, Fitterman N, et al. Risk assessment for and
strategies to reduce perioperative pulmonary complications
for patients undergoing noncardiothoracic surgery: a guideline from the American College of Physicians. Ann Intern Med.
2006;144:575-580.
4. Smetana GW. Preoperative pulmonary assessment of the older
adult. Clin Geriatr Med. 2003;19:35-55.
5. Wood KE. Major pulmonary embolism: review of a pathophysiologic approach to the golden hour of hemodynamically significant pulmonary embolism. Chest. 2002;121:877-905.
6. Proceedings of the Seventh ACCP Conference on antithrombotic
and thrombolytic therapy: evidence-based guidelines. Chest.
2004;126:172S-696S.
7. Neumann P, Rothen HU, Berglund JE, Valtysson J, Magnusson
A, Hedenstierna G. Positive end-expiratory pressure prevents
atelectasis during general anaesthesia even in the presence of a
high inspired oxygen concentration. Acta Anaesthesiol Scand.
1999;43:295-301.
8. Duggan M, Kavanagh BP. Pulmonary atelectasis: a pathogenic
perioperative entity. Anesthesiology. 2005;102:838-854.
9. Brown PP, Kugelmass AD, Cohen DJ, et al. The frequency and
cost of complications associated with coronary artery bypass
graft ing surgery: results from the United States medicare program. Ann Thorac Surg. 2008;85:1980-1986.
10. Koulenti D, Lisboa T, Brun-Buisson C, et al. Spectrum of practice
in the diagnosis of nosocomial pneumonia in patients requiring
mechanical ventilation in European intensive care units. Crit
Care Med. 2009;37:2360-2368.
11. Kumar A, Roberts D, Wood KE, et al. Duration of hypotension
before initiation of effective antimicrobial therapy is the critical
determinant of survival in human septic shock. Crit Care Med.
2006;34:1589-1596.
12. Kumar A, Ellis P, Arabi Y, et al. Initiation of inappropriate antimicrobial therapy results in a fivefold reduction of survival in
human septic shock. Chest. 2009;136:1237-1248.
13. Guidelines for the management of adults with hospital-acquired,
ventilator-associated, and healthcare-associated pneumonia. Am
J Respir Crit Care Med. 2005;171:388-416.
14. Eachempati SR, Hydo LJ, Shou J, Barie PS. Does de-escalation of
antibiotic therapy for ventilator-associated pneumonia affect the
likelihood of recurrent pneumonia or mortality in critically ill
surgical patients? J Trauma. 2009;66:1343-1348.
15. Aarts MA, Brun-Buisson C, Cook DJ, et al. Antibiotic management of suspected nosocomial ICU-acquired infection: does
prolonged empiric therapy improve outcome? Intensive Care
Med. 2007;33:1369-1378.
16. Chastre J, Wolff M, Fagon JY, et al. Comparison of 8 vs 15 days of
antibiotic therapy for ventilator-associated pneumonia in adults:
a randomized trial. JAMA. 2003;290:2588-2598.
17. Rubenfeld GD, Caldwell E, Peabody E, et al. Incidence and outcomes of acute lung injury. N Engl J Med. 2005;353:1685-1693.
18. Stein PD, Woodard PK, Weg JG, et al. Diagnostic pathways in
acute pulmonary embolism: recommendations of the PIOPED II
Investigators. Radiology. 2007;242:15-21.

PMPH_CH117.indd 932

19. Partsch H, Blattler W. Compression and walking versus bed rest


in the treatment of proximal deep venous thrombosis with low
molecular weight heparin. J Vasc Surg. 2000;32:861-869.
20. Rubin LJ. Diagnosis and management of pulmonary arterial
hypertension: ACCP evidence-based clinical practice guidelines.
Chest. 2004;126:4S-6S.
21. McLaughlin VV, Genthner DE, Panella MM, Rich S. Reduction
in pulmonary vascular resistance with long-term epoprostenol
(prostacyclin) therapy in primary pulmonary hypertension. N
Engl J Med. 1998;338:273-277.
22. Esteban A, Frutos-Vivar F, Ferguson ND, et al. Noninvasive positive-pressure ventilation for respiratory failure after extubation.
N Engl J Med. 2004;350:2452-2460.
23. Ventilation with lower tidal volumes as compared with traditional tidal volumes for acute lung injury and the acute respiratory distress syndrome. The Acute Respiratory Distress
Syndrome Network. N Engl J Med. 2000;342:1301-1308.
24. Ferguson ND, Stewart TE. New therapies for adults with acute
lung injury. High-frequency oscillatory ventilation. Crit Care
Clin. 2002;18:91-106.
25. Derdak S, Mehta S, Stewart TE, et al. High-frequency oscillatory
ventilation for acute respiratory distress syndrome in adults:
a randomized, controlled trial. Am J Respir Crit Care Med.
2002;166:801-808.
26. Bollen CW, van Well GT, Sherry T, et al. High frequency oscillatory ventilation compared with conventional mechanical
ventilation in adult respiratory distress syndrome: a randomized controlled trial [ISCRCTN24242669]. Crit Care. 2005;9:
R430-439.
27. Spragg RG, Lewis JF, Walmrath HD, et al. Effect of recombinant
surfactant protein C-based surfactant on the acute respiratory
distress syndrome. N Engl J Med. 2004;351:884-892.
28. Gattinoni L, Tognoni G, Pesenti A, et al. Effect of prone positioning on the survival of patients with acute respiratory failure. N
Engl J Med. 2001;345:568-573.
29. Guerin C, Gaillard S, Lemasson S, et al. Effects of systematic
prone positioning in hypoxemic acute respiratory failure: a randomized controlled trial. JAMA. 2004;292:2379-2387.
30. Kallet RH. Evidence-based management of acute lung injury
and acute respiratory distress syndrome. Respir Care. 2004;49:
793-809.
31. Adhikari NK, Burns KE, Friedrich JO, Granton JT, Cook DJ,
Meade MO. Effect of nitric oxide on oxygenation and mortality
in acute lung injury: systematic review and meta-analysis. BMJ.
2007;334:779.
32. Sokol J, Jacobs SE, Bohn D. Inhaled nitric oxide for acute hypoxic
respiratory failure in children and adults: a meta-analysis. Anesth
Analg. 2003;97:989-998.
33. Mancebo J, Fernandez R, Blanch L, et al. A multicenter trial of
prolonged prone ventilation in severe acute respiratory distress
syndrome. Am J Respir Crit Care Med. 2006;173:1233-1239.
34. Adhikari N, Burns KE, Meade MO. Pharmacologic therapies for
adults with acute lung injury and acute respiratory distress syndrome. Cochrane Database Syst Rev. 2004:CD004477.
35. Peter JV, John P, Graham PL, Moran JL, George IA, Bersten A.
Corticosteroids in the prevention and treatment of acute respiratory distress syndrome (ARDS) in adults: meta-analysis. BMJ.
2008;336:1006-1009.
36. Zapol WM, Snider MT, Hill JD, et al. Extracorporeal membrane
oxygenation in severe acute respiratory failure. A randomized
prospective study. JAMA. 1979;242:2193-2196.

5/22/2012 8:01:36 PM

Postoperative Respiratory Failure

37. Morris AH, Wallace CJ, Menlove RL, et al. Randomized clinical
trial of pressure-controlled inverse ratio ventilation and extracorporeal CO2 removal for adult respiratory distress syndrome.
Am J Respir Crit Care Med. 1994;149:295-305.
38. Chalwin RP, Moran JL, Graham PL. The role of extracorporeal
membrane oxygenation for treatment of the adult respiratory
distress syndrome: review and quantitative analysis. Anaesth
Intensive Care. 2008;36:152-161.
39. Hemmila MR, Rowe SA, Boules TN, et al. Extracorporeal life
support for severe acute respiratory distress syndrome in adults.
Ann Surg. 2004;240:595-605; discussion 605-597.
40. Frenckner B, Palmer P, Linden V. Extracorporeal respiratory
support and minimally invasive ventilation in severe ards. Minerva Anestesiol. 2002;68:381-386.
41. Peek GJ, Clemens F, Elbourne D, et al. Cesar: Conventional ventilatory support vs extracorporeal membrane oxygenation for severe
adult respiratory failure. BMC Health Serv Res. 2006;6:163.

PMPH_CH117.indd 933

933

42. Esteban A, Frutos F, Tobin MJ, et al. A comparison of four methods of weaning patients from mechanical ventilation. Spanish Lung Failure Collaborative Group. N Engl J Med. 1995;332:
345-350.
43. Ely EW, Baker AM, Dunagan DP, et al. Effect on the duration of
mechanical ventilation of identifying patients capable of breathing spontaneously. N Engl J Med. 1996;335:1864-1869.
44. Kress JP, Pohlman AS, OConnor MF, Hall JB. Daily interruption of sedative infusions in critically ill patients undergoing
mechanical ventilation. N Engl J Med. 2000;342:1471-1477.
45. Schweickert WD, Pohlman MC, Pohlman AS, et al. Early physical and occupational therapy in mechanically ventilated, critically ill patients: a randomised controlled trial. Lancet. 2009;373:
1874-1882.
46. Kollef MH, Ward S. The influence of mini-bal cultures on
patient outcomes: implications for the antibiotic management of
ventilator-associated pneumonia. Chest. 1998;113:412-420.

5/22/2012 8:01:36 PM

Commentary on
Postoperative Respiratory Failure
Suresh Agarwal

Dr. Beilmans thorough and comprehensive examination of postoperative respiratory failure is a complete examination of many
topics related to a common problem that is likely to be experienced by all surgeons. He has systematically examined topics from
etiology, diagnosis, and treatment of a variety of different types of
respiratory failure that a post-operative patient may experience.
One of the most interesting and most frequently discussed topics
was touched upon as well: Acute lung injury (ALI) and the acute
respiratory distress syndrome (ARDS).
ALI and ARDS represent a spectrum of clinical syndromes
of rapid respiratory system deterioration that are associated with
both pulmonary and systemic illness. Moreover, these syndromes
are associated with 3040% mortality with our current standard
of care and are responsible for approximately 75,000 deaths in
the United States annually. In fact, the age adjusted incidence of
acute lung injury has been shown to be as high as 86.2 per 100,000
person years with mortality increasing with age.1 Its impact upon
patient function and recovery is profound, lasts far beyond time
spent in the intensive care unit, and involves both pulmonary and
extra-pulmonary systems.2 Multiple studies have been conducted
to examine methodology to reduce mortality and improve outcome in ARDS and ALI; however, there have been few positive
outcomes resulting in management of this difficult disease process. Furthermore, sparse research of the impact of modern ventilator modalities upon patients with ARDS/ALI exists.
Current evidence-based care of ALI consists of a strategy
of mechanical ventilation utilizing low lung volumes (ARDSNet
ventilation) intended to limit further stretch-induced lung injury
exacerbated by the ventilator.3 Specifically, the ARDSNet group
demonstrated that ventilating patients with tidal volumes of
6 ml/kg of predicted body weight compared with traditional volumes of 12 ml/kg of predicted body weight resulted in an absolute
mortality reduction of 8.8%. However, when further examined,
it was noted that although there was a decrease in mortality in
patients who were assigned to low volume ventilation group,
there was no difference in time on the ventilator in survivors of
both groups.4 This strategy has been shown to be associated with
increased lung injury in a subset of patients, and still is associated
with ~30% mortality rates.5
Nuckton et al. found that increased pulmonary dead space
fraction is an independent, but quantifiable marker for death.6
Attempts at decreasing the amount of non-aerated lung and

increasing functional residual capacity by varying the amount


of positive end expiratory pressure (PEEP) have also been made,
but with limited success. The ARDSNet clinical trial group placed
patients with ARDS on a tidal volume of 6 ml/kg and varied the
PEEP between 8.3 cm of water (low PEEP) and 13.2 cm of water
(high PEEP) and noted that the outcomes in both groups were
similar.7 Gattinoni et al. further examined the potential for lung
recruitment by varying pressures in breath holding maneuvers
from pressures of 5 to 45 cm of water and found that patients vary
greatly in their potential for aeration and that this potential correlates with their responsiveness to PEEP.8
Pharmacologic and physiologic intervention, similarly, have
met with little success in changing outcome. A single center study
demonstrated that steroid therapy may be beneficial in treating
patients with ARDS,9 however, this has not been corroborated
by larger center trials. Despite initially improving oxygenation,
inhaled nitric oxide has failed to demonstrate an improvement
in outcome in multiple trials.10 Similarly, exogenous surfactant administration was introduced with a great deal of positive
speculation, however, it has not shown to be of benefit in clinical
trials.11,12 Prone positioning, likewise, seems to improve oxygenation, but not survival.13
As mentioned, low-tidal volume ventilation remains the
mainstay of management of patients with ALI. In addition, evidence exists demonstrating that low volume ventilation is not
without its faults, having been shown to increase atelectasis,
weaning time, and the incidence of pneumonia.14 Since the initial
ARDSNet articles were published, multiple modes of ventilation
have emerged and gained popularity. Despite this, none has been
compared with the standard of care in managing this difficult
group of patients.
This entity along with hospital-acquired pneumonia, ventilator-associated pneumonia, pulmonary embolism, and pneumo/
hemo-thorax remain problems that may be able to be prevented,
but to this day remain a large burden to patients and the health
care system. As Dr. Beilman has clearly advocated and supported
with recent evidence, many of these issues can be prevented with
vigilance, proper pre-, peri-, and post-operative planning, and
adequate prophylaxis. In fact, many of these conditions have
become never events as classified by the Institute for Healthcare
Improvement and in the future, will not be covered by governmental and private payers.

934

PMPH_CH117.indd 934

5/22/2012 8:01:36 PM

Postoperative Respiratory Failure

REFERENCES
1. Rubenfeld GD, Caldwell E, Peabody E, et al. Incidence and Outcomes of Acute Lung Injury. N Engl J Med 2005;353:1685-1693.
2. Herridge MS, Cheung AM, Tansey CM, et al. One-Year Outcomes in Survivors of the Acute Respiratory Distress Syndrome.
N Engl J Med 2003;348:683-693.
3. Amato MBP, Barbas CSV, Medeiros DM, et al. Effect of a Protective-Ventilation Strategy on Mortality in the Acute Respiratory
Distress Syndrome. N Engl J Med 1998;338:347-354.
4. The Acute Respiratory Distress Syndrome Network. Ventilation
with Lower Tidal Volumes as Compared with Traditional Tidal
Volumes for Acute Lung Injury and the Acute Respiratory Distress Syndrome. N Engl J Med 2000;342:1301-1308.
5. Terragni PP, Rosboch G, Tealdi A, et al. Tidal Hyperinflation
during Low Tidal Volume Ventilation in Acute Respiratory Distress Syndrome. Am J Respir Crit Care 2007;175:160-166.
6. Nuckton TJ, Alonso JA, Kallet RH, et al. Pulmonary Dead-Space
Ventilation as a Risk Factor for Death in the Acute Respiratory
Distress Syndrome. N Engl J Med 2002;346:1281-1286.
7. The Acute Respiratory Distress Syndrome Network. Higher versus
Lower Positive End Espiratory Pressures in Patients with Acute
Respiratory Distress Syndrome. N Engl J Med 2004:351(4):327-336.

PMPH_CH117.indd 935

935

8. Gattinoni L, Caironi P, Cressoni M, et al. Lung Recruitment in


Patients with the Acute Respiratory Distress Syndrome. N Engl J
Med 2006,354:17,1775-1786.
9. Meduri GU, Headley AS, Golden E, et al. Effect of Prolonged
Methylprednisolone Therapy in Unresolving Acute Respiratory
Distress Syndrome. A randomized controlled trial. JAMA 1998;
280:59-65.
10. Sokol J, Jacobs SE, Bohn D. Inhaled Nitric Oxide for Acut/e
Hypoxic Respiratory Failure in Children and Adults. Anesth
Analg 2003;97:989-998.
11. Spragg RG, Lewis JF, Walmrath, et al. Effect of Recombinant
Surfactant Protein C-Based Surfactant on the Acute Respiratory
Distress Syndrome. N Engl J Med 2004;351:884-892.
12. Anzueto A, Baughman RP, Guntupalli KK, et al. Aerosolized
Surfactant in Adults with Sepsis-Induced Acute Respiratory Distress Syndrome. N Engl J Med 1996;334:1417-1421.
13. Fattinoni L, Tognoni G, Pesenti A, et al. Effect of Prone Positioning on the Survival of Patients with Acute Respiratory Failure. N
Engl J Med 2001;345:568-573.
14. Terragni PP, Rosboch G, Tealdi A, et al. Tidal Hyperinflation during Low Tidal Volume Ventilation in Acute Respiratory Distress Syndrome. Am J Respir Crit Care 2007;175:
160-166.

5/22/2012 8:01:36 PM

CHAPTER 118

Postoperative Hepatic Failure


Joseph Love and Rafael Diaz-Flores

Assessment of preoperative liver function is clearly an important


part of evaluating patients for hepatic surgery. Several studies
have been done to find those patients who will be able to withstand resection and more importantly, survive.
In recent years, surgeons tested the limits of anatomic boundaries and increased tumor volumes in an attempt to expand the
indications for major liver resection. It must be remembered that
surgical resection remains the mainstay of treatment for cure in
patients with hepatic malignancies. It is from this that preoperative
techniques such as portal vein embolization and aggressive chemotherapy have been utilized. Finding the patients who will have
adequate functional reserve postoperatively with the superimposed
demands of regeneration remains elusive. Hepatobiliary surgeons
have used preoperative selection criteria largely based on clinical
classification systems and the surgeons experience. Despite these
tools, liver failure remains a significant contributor to perioperative morbidity and mortality. Overall, liver failure leading to death
ranges from 2% to 65% following hepatic resection for hepatocellular carcinoma. The mortality rate due to liver failure in this group of
patients may not be too surprising when one considers the increased
rate of cirrhosis as the etiology for their tumors. What is surprising
are the mortality rates after resection for colorectal cancer, which
have a wide range of up to 50% of death from liver failure.1
Preexisting liver disease also contributes to complications
and poor outcomes in trauma and general surgery patients. The
difficulty remains of being able to identify which patients need
special attention or alternative treatments.
This chapter will attempt to review several of the tests available to assess hepatic function and new treatment options, along
with causes that may be avoided in the perioperative period.

in a retrospective review of 53 patients with cirrhosis undergoing


intraabdominal surgery and found that both a low hemoglobin
level and renal failure with dialysis were strong negative predictors. In addition, high model for end-stage liver disease (MELD)
score and/or high Child-Turcotte-Pugh (CTP), an open intraabdominal procedure, a low albumin level, and African-American
race were significant negative predictors. High total bilirubin
level, International Normalized Ratio (INR), creatinine, age, Alanine Transaminase (ALT), presence of ascites, male sex, and presence of hepatic encephalopathy were not found to be individually
or significantly associated to outcome.2
In most of the studies, control groups are lacking. Differences in the surgical diagnoses and timing of operations add to the
variability observed in many of them. Del Olmo et al. designed a
case-control study looking at 135 patients with cirrhosis undergoing nonhepatic procedures and 86 matched controls. By univariate
analysis, they found that the need for intraoperative transfusions,
prothrombin time, and CTP score were significantly associated with
postoperative liver failure. Duration of surgery, prothrombin time,
CTP score, and the presence of complications (related to cirrhosis
or not) were all significantly associated to mortality individually.3
Answer: Given the many different clinical scenarios, such as
diversity of patient factors and the inherent variability of liver failure, predicting postoperative liver failure from isolated liver function tests or other biochemical abnormalities is difficult. There is
enough evidence to conclude that abnormal function tests, anemia,
concomitant kidney failure with need for dialysis, and a low albumin
levelwhen present alone or in combinationincrease the risk of
postoperative failure in some patients (Grade C recommendation).
2. Are there predictive factors in patients without known
liver disease who are at risk of developing hepatic failure after
hepatic resection?

1. Do isolated liver function tests or other biochemical values,


or preoperative patient characteristics have clinical relevance
in the postoperative setting?

For patients undergoing major liver resection, Oussoultzoglou et al.


performed a retrospective descriptive cohort study to review outcomes of 67 consecutive patients undergoing resection of at least
four contiguous liver segments. In their univariate analysis of preoperative characteristics they found that alanine aminotransferase

In patients with cirrhosis, isolated elevated liver function tests and


other clinical and demographic factors have inconsistently been
found to be associated with poor postoperative outcome. Befeler
et al. performed a univariate analysis of baseline characteristics
936

PMPH_CH118.indd 936

5/22/2012 6:21:09 PM

Postoperative Hepatic Failure

blood levels greater than 40 U/L, prothrombin ratio of less than


70%, preoperative indocyanine green retention rate at 15 minutes
(ICGR-15) of greater than 15%, preoperative biliary drainage, and
performance of extrahepatic duct resection all significantly predicted the occurrence of in-hospital death. The risk was also directly
related to the sum of risk factors present. Occurrence of a sepsis-like
syndrome in patients with postoperative liver failure was the main
complication leading to death in patients undergoing a major hepatectomy with an extrahepatic bile duct resection.4
Perhaps the largest population-based study to date comparing postoperative mortality in cirrhotics versus noncirrhotics was
a population-based data outcomes retrospective review of the
Nationwide Inpatient Sample from 1998 to 2005 using multivariate
logistic regression analysis to adjust for sociodemographic and clinical covariates. Nguyen et al. looked at patients undergoing colorectal surgery. The study included 499541 patients with no cirrhosis
who were compared with 2909 patients with cirrhosis and no portal
hypertension (PHTN) and 1133 patients with cirrhosis and PHTN.
They found the mortality to be 29% in patients with cirrhosis and
PHTN and 14% in patients without PHTN. The risk was lower in
elective operations than in emergent indications, with an odds ratio
of 0.61 (95%CI: 0.590.62). Interestingly, these investigators found
no difference in mortality in colorectal procedures completed in a
liver transplant center. This could be explained by a selection bias
where sicker patients with worse baseline prognosis may be preferentially admitted to liver transplant centers.5
Answer: Preoperative liver dysfunction as evidenced by an
elevated ALT level or prolonged prothrombin time, preoperative
ICGR-15 > 15%, and performance of extrahepatic duct resection are all predictive factors of posthepatectomy liver failure in
patients with preexisting liver disease. The presence of cirrhosis
itself is also a risk factor, which is potentiated by concomitant portal vein thrombosis (Grade B recommendation).
3. What is the best predictor of postoperative hepatic failure
(MELD, Child-Pugh, or strict liver function tests)?
Both the MELD and CTP scores have been reported to be correlated with postoperative morbidity and mortality in patients
with end-stage liver disease (ESLD). The existence of jaundice,
prolonged prothrombin time, ascites, encephalopathy, hypoalbuminemia, portal hypertension, renal insufficiency, hyponatremia,
infection, and anemia have been proposed as risk factors.6 These
observations have been made based on retrospective observational
studies. No multicenter prospective studies have been performed
to date to refine the risk assessment and stratification of patients
with ESLD using these clinical indicators.
A retrospective review of 587 patients undergoing elective
hepatic resection in cirrhotic and noncirrhotic patients, using
the NSQIP database, compared MELD and CTP scores, Charlson
Index of Comorbidity, and the American Society of Anesthesiologists physical status classification (ASA) as predictors of morbidity and mortality. CTP score and ASA classification were found
to be very reliable predictors whereas MELD score correlated
poorly with short- and long-term outcomes. This may be related
to the fact that the MELD score was originally created to stratify
patients awaiting liver transplantation or undergoing Transjugular intrahepatic portosystemic shunt (TIPS) procedures, not those
who are otherwise healthy and not cirrhotic.7
In a retrospective review of 1347 patients with gastric cancer,
of which 25 had liver cirrhosis (15 with Childs A and 10 with Childs

PMPH_CH118.indd 937

937

B cirrhosis), the risk of postoperative intractable ascites was found


to be high, particularly in Grade B patients. There was no difference in mortality.8 In another small-scale retrospective review of
1189 patients undergoing infrarenal abdominal aortic aneurysm
repair, of which 24 had cirrhosis (22 Childs A and 2 Childs B), with
a median MELD score of 8, the CTP B was associated with a significantly higher operative time and intraoperative blood transfusion
requirements. Both CTP B and a MELD score equal to or greater
than 10 were associated with reduced midterm survival rates. None
of the deaths was from postoperative liver failure, but from complications of cirrhosis, such as upper gastrointestinal bleeding.9
Befeler et al. retrospectively compared the MELD and CTP
scores in 53 patients with cirrhosis undergoing nonhepatic
intraabdominal surgery. They found a MELD score over 14 to be a
better predictor of poor outcome than a CTP class C classification.
Although CTP B and C identified all patients with poor outcome,
it had low positive predictive value and positive likelihood ratio,
limiting it as a clinical tool. A MELD score of 9 or greater had a
similar performance to CTP B or C, whereas a MELD score of 14
or higher had improved sensitivity and negative predictive value
for poor outcome.10
All of these studies are limited not only by their retrospective
nature, but also by the fact that patients with a high CTP or MELD
score frequently are not operated upon given their reduced life
expectancy and the more frequent presence of severe comorbidities. The large population-based retrospective study by Nguyen
et al. is also limited by the absence of available data to further classify patients into Childs A, B, or C or to give them a MELD score.
It is clear from these and other studies in the literature that cirrhosis alone is a risk factor for postoperative liver failure, complications, and mortality, and that the degree of liver failure, whether
measured by CTP or MELD score, correlates with mortality.
Answer: MELD and CTP scores as well as ASA classifications
are useful adjuncts in predicting postoperative hepatic failure.
In the presence of cirrhosis, MELD score seems to be the strongest indicator, whereas a high ASA score may be a better option
for patients without preoperative liver dysfunction (Grade B
recommendation).
4. Are there preferred preoperative tests, such as indocyanine green elimination, that help predict postoperative liver
failure?
Several options exist for quantifying liver function such as the
above stated scores CTP and MELD. However, these classifications
provide only a rough estimate of liver function. Following resection, there will invariably be some liver dysfunction related to the
surgical procedure, use of certain anesthetics, and the metabolic
stress placed on the remnant liver. An important determinant of
outcome depends on the functional volume of the remnant liver.
To overcome some of the limitations associated with the various scoring systems, imaging and biochemical studies have been
designed to assist in determining the liver function and volume
after resection. There is no single test available that can provide all
the necessary information to determine functional liver reserve.
Several studies using computed tomography (CT)-based volumetric analysis have been done looking at relative residual liver
volume (%RLV) as a source of postoperative liver failure. A volume of less than 25% has been associated with liver dysfunction in
patients with normal livers.11,12 Schindl et al. in a more recent study
looking at 104 patients without chronic liver disease undergoing

5/22/2012 6:21:09 PM

938

Surgery: Evidence-Based Practice

liver resection for metastatic colorectal cancer reported a correlation between %RLV and postoperative liver dysfunction. Their
unique study used a simple formula subtracting the tumor volume from the total liver volume giving a difference known as the
total functional liver volume (TFLV). The volume of the proposed
preoperative resected liver was then subtracted from the TFLV
leaving the residual volume. This was then expressed as a percent
of the TFLV. Analysis of their data demonstrated that a %RLV of
26.6% was a significant risk factor for hepatic dysfunction.13
With the increasing power of personal computers, this modality may be used even in austere or remote locations. A recent paper
was published comparing traditional CT-volumetric analysis by
radiologists with free-source soft ware interpreted by nonradiologists on patients scheduled for elective right hepatectomy. A total
of 15 patients were included. There were no major differences in
total liver volumes, resection volumes, or tumor volumes between
groups suggesting volumetry can be accurately determined on a
PC by a nonradiologist.14
Of the biochemical tests used, the indocyanine green (ICG)
retention is the most common.15 The ICG and ICG retention rate
at 15 minutes are thought to reflect the quantitative liver function before hepatectomy. However, it remains imperfect because it
depends on hepatic blood flow and the functional capacity of the
liver both of which can be variable even in the same patient at different time of the day. There is general agreement on the retention
values that support major hepatic resection. An ICG 15 value of
15% or more is considered impaired. Thus, Child-Pugh A patients
with a value of more than 15% ICG 15 have limited functional
reserve and may be considered bad risk for resection. In the setting of resection for hepatocellular carcinoma, ICG was useful for
evaluating the remnant liver function before, during, and after
liver resection. The ICG clearance was a significant predictor of
liver failure.16 ICG 15 correlates well with CTP scores; however, no
study has been able to show superiority in assessing hepatic functional reserve or predicting liver failure better than Child-Pugh.
Another modality that has been evaluated is the technetium99m-diethylenetriaminepentaacetic acid galactosyl human serum
albumin (99mTc-GSA), which is an agent that is closely related to
hepatocellular function. The exact technique to perform the test
is beyond the scope of this chapter; however, several papers have
used the maximal removal rate (GSA-Rmax) as an index to evaluate the functional reserve capacity of the liver postresection. Kwon
et al. found that a GSA-Rmax value greater than 0.15 mg/min in
the residual liver was necessary to avoid postoperative hyperbilirubinemia hepatic failure.17
A recent study done by Wakamatsu evaluated 99mTc-GSA as
a preoperative test to predict residual liver function.18 This was the
first such study using this modality to predict recovery after partial hepatectomy. Using single photon emission computed tomography (SPECT), they were able to map the expected residual liver.
GSA in the residual liver (GSA-RL) was then calculated using
analysis soft ware. The team then followed postoperative albumin
and cholinesterase as markers for recovery of liver function. There
was a correlation between the GSA-RL and the return of liver
function in the postpartial hepatectomy patient. There were several limitations such as selection bias and the failure to define the
baseline liver function of the study patients. This modality may be
limited due to the use of SPECT and CT scan separately. Each test
may not be available universally, prohibiting widespread use.
de Graaf et al. utilized 99mTc-mebrofenin hepatobiliary scintigraphy (HBS) to predict liver failure after major liver resection.19

PMPH_CH118.indd 938

99mTc-mebrofenin is useful given its ability to be taken up by the


liver and secreted in the bile similar to bilirubin. Earlier studies
had suggested that HBS with 99Tc-mebrofenin was able to predict
future liver failure in patients undergoing minor and major liver
resections. An uptake of less than 2.5%/min/m2 was a significant
factor for the development of liver dysfunction.20 In the previously
mentioned study, the investigators were interested in finding out
if this would be similar in high-risk patients. When compared
with traditional volumetric analysis, HBS was able to reliably predict postoperative liver failure. Data analysis revealed a similar
uptake of 2.69/min/m2 as a cutoff when liver failure became more
frequent. These findings have led to the use of HBS in conjunction with CT-volumetric at their institution. One benefit of this
adjunctive preoperative study is its ease and possible availability
in any institution with nuclear medicine capabilities.
Answer: Accurate measurement of liver function is essential
in determining resectability in patients requiring liver resection.
With the ever-expanding indications for hepatectomy, it would
be ideal to have a test to predict which patients may have a significant problem with postoperative liver failure. The above studies suffer appreciably from the lack of a proper definition of what
constitutes postoperative liver failure. As a result, it is hard to
compare the modalities. Presently, there is no study that has been
shown to be superior to Child-Pugh in predicting postoperative
liver failure and death. Further research is warranted (Grade B
recommendation).
5. What is the best marker for adequate resuscitation in surgery/trauma patients with cirrhosis or liver failure?
Liver failure is associated with altered regulation of total body-water
volume and its distribution in the body. This imbalance is primarily responsible for intraabdominal fluid accumulations known as
ascites. The most likely theory on ascites today supports the view
of portal hypertension leading to a release of mediators causing
arteriolar vasodilation. This reduces the central circulating volume
activating a neurohormonal response causing the kidney to absorb
sodium and water. Soluble mediators are also released from the dysfunctional endothelium leading to vasodilation in the splanchnic
arteriolar bed and an associated drop in arteriolar pressure. This
drop activates the renin-angiotensin-aldosterone system leading to
sodium and water retention.21 The drop in pressure leads to activation of the sympathetic nervous system and release of arginine vasopressin, promoting a compensatory vasoconstriction that can cause
a decrease in the glomerular filtration rate and in the excretion of
water and solutes by the kidney.22 Cirrhosis also leads to an increase
in cardiac output secondary to increased preload and reduced afterload due to decreased systemic vascular resistance and increased
heart rate. Patients with cirrhosis also tend to be resistant to vasopressor effects such as norepinephrine and vasopressin.23 Despite
increases in cardiac output, cirrhosis is associated with impaired
cardiac function leading systolic dysfunction under conditions of
cardiac stress. Fluid loading in cirrhotic patients may be necessary
to improve hemodynamics.24 However, fluids may simply expand
the extravascular space and worsen organ function and ascities. It is
from this and many other reasons that fluid resuscitation is difficult
in cirrhotic patients.
Several studies have evaluated goal-based therapies for guiding fluid management in patients with advanced cirrhosis and
renal failure. Recently a group of patients with advanced cirrhosis and functional renal failure were investigated using a protocol

5/22/2012 6:21:09 PM

Postoperative Hepatic Failure

utilizing 400 cc of 20% albumin for volume expansion. They


found a significant increase in central blood volume as reflected
in an improvement in cardiac index. Central venous pressure was
unaffected by the infusion.25 A similar study evaluated cirrhotic
patients with renal failure undergoing paracentesis. The protocol
involved giving patients a bolus of 200 cc of 20% albumin followed
by paracentesis. Patients were then given 8 g of albumin for every
1 L of ascites removed. A pulmonary catheter was used to monitor cardiac output. This goal-directed therapy with an albumin
infusion led to improved renal function. Central venous pressure
was around 14 cm H20 in all patients during the initial bolus. The
cardiac index was noted to increase by approximately 15% during
the infusion suggesting that even though the CVP appeared normal, the patients still responded to the fluid.26 The improvement
in renal function is likely multifactoral and both studies suffer
from no control groups. However, both demonstrated the central
pressures shortcoming especially in cirrhotic patients where the
intraabdominal pressure makes interpretation difficult.
Answer: While clearly more investigation needs to be done,
as data on the specific hemodynamic responses to fluid expansion
in cirrhosis remain sparse, goal-directed therapy with volumetric
indicators might be useful (Grade C recommendation).
6. Does renal replacement therapy improve outcomes in postoperative liver failure?
The use of renal replacement therapy provides the opportunity
of support to those patients who develop hepatorenal syndrome,
particularly in those awaiting transplant or in those with acute,
potentially reversible conditions such as alcoholic hepatitis. There
are few studies that have evaluated renal replacement therapy in
the setting of hepatorenal syndrome, and the optimal method is
neither clear nor is it clear whether the method improves prognosis
or outcome. A recent prospective observational study evaluated the
predictive factors of overall and 30-day survival in patients with
Child-Pugh Class C cirrhosis and hepatorenal syndrome. Elevated
Child-Pugh, higher Acute Physiology and Chronic Health Evaluation II scores, elevated INR, and lower albumin were found to
be predictors of survival, independent of the need for mechanical
ventilation. The use of renal replacement therapy was not predictive of improved survival in these patients.27 As of yet, there is no

939

data from studies comparing renal replacement therapy to vasoconstrictor or albumin with vasoconstrictor administration.28
Answer: Given the paucity of data, it seems reasonable to
start patients who develop hepatorenal syndrome with colloid and
vasoconstrictors as discussed above and reserve renal replacement
therapy for those patients who are not responsive or develop other
accepted indications for renal replacement therapy (e.g., overt
uremia, hyperkalemia, and metabolic acidosis).
7. How effective are liver replacement therapies?
Several systems exist including the Molecular Adsorbent Recirculating System (MARS), the Fractionated Plasma Separation,
Adsorption and Dialysis system, single-pass albumin dialysis, and
single-pass albumin extended dialysis. The description of each
system can be found elsewhere.
MARS is the most clinically investigated of the artificial
hepatic support systems. In one study, 13 patients with hepatorenal syndrome who were considered high risk for TIPS or other
therapy due to cirrhosis were randomized to either MARS treatment (8 patients) or treatment with hemodiafi ltration (5 patients)
alone. Patients treated with MARS showed significant improvement in bilirubin, serum creatinine levels, prothrombin time, and
control of serum sodium. No significant changes were found in
the hemodiafi ltration group.29 The benefit was also noted in a nonrandomized study of seven patients with cirrhosis and acute renal
failure. Treatment with MARS resulted in increased urine output,
decreases in bilirubin, urea, and INR. Three of the patients survived to hospital discharge.30
There are few reports describing the use of the other mentioned modalities. A recent case report described improvements
in creatinine, INR, and transaminase levels with the use of singlepass albumin dialysis in a 24-year-old with hepatitis A. As of yet,
no studies have been done using this modality for hepatorenal
syndrome.
Answer: The potential improvement of hepatic replacement
or support therapy remains to be fully examined. As noted, the
use of hepatic support systems can be associated with improved
relative laboratory values; however, more research is needed to
determine the impact and benefit of these therapies (Grade C
recommendation).

Clinical Question Summary


Question

Answer

Grade of
References
Recommendation

1 Do isolated elevated liver


function values have
clinical relevance in the
postoperative setting?

There is enough evidence to conclude that abnormal function


tests, anemia, concomitant kidney failure with need for
dialysis, and a low albumin level, when present alone or in
combination, increase the risk of postoperative failure in
some patients.

1-3

2 Are there predictive


factors in patients without
known liver disease who
are at risk of developing
hepatic failure in the
postoperative setting?

Preoperative liver dysfunction as evidenced by an elevated


ALT level or prolonged prothrombin time, preoperative
ICGR-15 > 15%, and performance of extrahepatic duct
resection are all predictive factors of posthepatectomy
liver failure in patients with preexisting liver disease. The
presence of cirrhosis itself is also a risk factor, which is
potentiated by concomitant portal vein thrombosis.

4-5

(Continued)

PMPH_CH118.indd 939

5/22/2012 6:21:09 PM

940

Surgery: Evidence-Based Practice

(Continued)
Question

Answer

3 What is the best predictor


of postoperative hepatic
failure (MELD, ChildPugh, or strict liver
function tests)?

MELD and Child-Pugh scores as well as ASA classifications are


useful adjuncts in predicting postoperative hepatic failure.
In the presence of cirrhosis, MELD score seems to be the
strongest indicator, while a high ASA score may be a better
option for patients without preoperative liver dysfunction.

4 Are there preferred


preoperative tests such
as indocyanine green
elimination that help
predict postoperative
liver failure?

The studies suffer from the lack of a proper definition of what


constitutes postoperative liver failure. Presently there is no
study that has been shown to be superior to Child-Pugh
in predicting postoperative liver failure and death. Further
research is needed.

11-20

5 What is the best marker


for adequate resuscitation
in surgery/trauma patients
with cirrhosis or liver
failure?

Goal-directed therapy with volumetric indicators might be


useful.

21-26

6 Does renal replacement


therapy improve
outcomes in
postoperative liver failure?

Given the paucity of data, it seems reasonable to start


patients who develop hepatorenal syndrome with colloid
and vasoconstrictors as discussed above and reserve
renal replacement therapy for those patients who are not
responsive or develop other accepted indications for renal
replacement therapy.

27-28

7 How effective are liver


replacement therapies?

The use of hepatic support systems can be associated with


improved relative laboratory values; however, more
research is needed to determine the impact and benefit of
these therapies.

29-30

REFERENCES
1. Schneider P. Preoperative assessment of liver function. Surg Clin
N Am. 2004;84:355-373.
2. Befeler A, Palmer D, Hoff man M, Longo W, Solomon H, Di Bisceglie. The safety of intra-abdominal surgery in patients with
cirrhosis. Arch Surg. 2005;140:650-654.
3. Del Olmo J, Flor-Lorente B, Flor-Civera B, et al. Risk factors
for nonhepatic surgery in patients with cirrhosis. World J Surg.
2003;27:647-652.
4. Oussoultzoglou E, Jaeck D, Addeo P, et al. Prediction of mortality rate after major hepatectomy in patients without cirrhosis.
Arch Surg. 2010;145:1075-1081.
5. Nguyen G, Correia A, Thuluvath P. The impact of cirrhosis
and portal hypertension on mortality following colorectal surgery: a nationwide, population-based study. Dis Colon Rectum.
2009;52(8):1367-1374.
6. Remzi F, Kirat H. Gastrointestinal surgery in patients with liver
failure. J Dig Dis. 2011;12:33-35.
7. Schroeder R, Marroquin C, Phillps Bute B, et al. Predictive indices of morbidity and mortality after liver resection. Ann Surg.
2006;243:373-379.
8. Ikeda Y, Tatsuo K, Kosugi S, et al. Gastric cancer surgery for
patients with liver cirrhosis. WJGS. 2009;1(1):49-55.
9. Marroco-Trishitta M, Kahlberg A, Astore D, Tshiombo G, Mascia D,
Chiesa R. Outcome in cirrhotic patients after elective surgical repair
of infrarenal aortic aneurysm. J Vasc Surg. 2011;53(4):906-911.

PMPH_CH118.indd 940

Grade of
References
Recommendation
6-10

10. Befeler A, Palmer D, Hoff man M, Longo W, Solomon H, Di Bisceglie. The safety of intra-abdominal surgery in patients with
cirrhosis. Arch Surg. 2005;140:650-654.
11. Shoup M, Gonen M, DAngelica M, et al. Volumetric analysis
predicts hepatic dysfunction in patients undergoing major liver
resection. J Gastrointest Surg. 2003;127:2477-2483.
12. Vauthey J, Chaoui A, Do K, et al. Standardized measurement of
the future liver remnant prior to extending liver resection: methodology and clinical associations. Surgery. 2000;127:512-519.
13. Schindl M, Redhead D, Fearon K, Garden O, Wigmore S. The
value of residual liver volume as a predictor of hepatic dysfunction and infection after major liver resection. Gut. 2005;54:
289-296.
14. Dello S, Stoot J, van Stiphout R, et al. Prospective volumetric assessment of the liver on a personal computer by nonradiologists prior to partial hepatectomy. World J Surg. 2011:35:
386-392.
15. Schneider P. Preoperative assessment of liver function. Surg Clin
N Am. 2004;84:355-373.
16. Ohwada S, Kawate S, Hamada K, et al. Perioperative real-time
monitoring of indocyanine green clearance by pulse spectrophotometry predicts remnant liver functional reserve in resection of
hepatocellular carcinoma. Br J Surg. 2006;93:339-346.
17. Kwon A, Matsui Y, Kaibori M, Ha-Kawa K, Preoperative regional
maximal removal rate of technetium-99m-galactosyl human
serum albumin (GSA-Rmax) is useful for judging the safety of
hepatic resection. Surgery. 2006;140:379-386.

5/22/2012 6:21:09 PM

Postoperative Hepatic Failure

18. Wakamatsu H, Nagamachi S, Kiyohara S, et al. Predictive value


of Tc-99m galactosyl human serum albumin liver SPECT on the
assessment of functional recovery after partial hepatectomy:
a comparison with CT volumetry. Ann Nuci Med. 2010;24:
729-734.
19. de Graaf W, van Lienden K, Dinant S, et al. Assessment of future
liver function using hepatobiliary scintigraphy in patients undergoing major liver resection. J Gastrointest Surg. 2010;14:369-378.
20. Dinant S, de Graaf W, Verwer B, et al. Risk assessment of posthepatectomy liver failure using hepatobiliary scintigraphy and CT
volumetry. J Nucl Med. 2007;48:685-692.
21. Polli F, Gattinoni L. Balancing volume resuscitation and ascites
management in cirrhosis. Curr Opin Anaesthesiol. 2010;23:
151-158.
22. Gines P, Schrier R. Renal failure in cirrhosis. N Engl J Med.
2009;361:1279-1290.
23. Polli F, Gattinoni L. Balancing volume resuscitation and ascites
management in cirrhosis. Curr Opin Anaesthesiol. 2010;23:
151-158.
24. Grose R, Nolan J, Dillon J, et al. Exercise-induced left ventricular
dysfunction in alcoholic and non-alcoholic cirrhosis. J Hepatol.
1995;22:326-332.

PMPH_CH118.indd 941

941

25. Umgelter A, Wagner K, Reindl W, Nurtsch N, Huber W, Schmid


R. Haemodynamic effects of plasma expansion with hyperoncotic albumin in cirrhotic patients with renal failure: a prospective interventional study. BMC Gastroenterol. 2008;8:39.
26. Umgelter A, Reindl W, Wagner K , et al. Effects of plasma expansion with albumin and paracentesis on haemodynamics and kidney function in critically ill cirrhotic patients with tense ascities
and hepatorenal syndrome: a prospective uncontrolled trial. Crit
Care. 2008;12:R4.
27. Witzke O, Baumann M, Patschan D, et al. Which patients benefit
from hemodialysis therapy in hepatorenal syndrome? J Gastroenterol Hepatol. 2004;19:1369-1373.
28. Gines P, Schrier R. Renal failure in cirrhosis. N Engl J of Med.
2009;13:1279-1290.
29. Mitzner S, Stange J, Klammt S, et al. Improvements of hepatorenal syndrome with extracorporeal albumin dialysis MARS:
results of a prospective, randomized, controlled clinical trial.
Liver Transpl. 2000;6:277-286.
30. McIntyre C, Fluck R, Freeman J, Lambie S. Characterization of
treatment dose delivered by albumin dialysis in the treatment
of acute renal failure associated with severe hepatic dysfunction.
Clin Nephrol. 2002;58:376-383.

5/22/2012 6:21:09 PM

CHAPTER 119

Acute Renal Dysfunction


Matthew ORourke & Jonathan Barasch

INTRODUCTION

RISK FACTORS

Acute renal dysfunction is a commonly encountered clinical entity


with profound implications. In the United States alone, one million patients each year are diagnosed with acute renal dysfunction,
and the incidence is rising.1-4 This is a rapidly progressive disease
that independently predicts morbidity and mortality. Patients
with acute renal dysfunction often require admission to the intensive care unit, dialysis initiation, and prolonged hospitalization.
They encounter significant risk of in-hospital death and long-term
development of chronic kidney disease.5-7 Recent revelations that
even small changes in kidney function are associated with significant morbidity and mortality underscore the significance of this
condition.8
Until recently, the term acute renal dysfunction and numerous others like it were employed to describe various stages of
kidney impairment. In 2004, the Acute Dialysis Quality Initiative Group published a consensus defi nition known as the risk,
injury, failure, loss, end-stage renal disease (RIFLE) classification system of acute kidney injury (AKI). Th is scheme identified three grades of AKI severity (risk, injury, and failure) based
on relative changes in serum creatinine and/or absolute changes
in urine output, and two AKI outcome classes (loss and endstage) determined by the required duration of renal replacement therapy.9 From this description, it is clear that the term
AKI encompasses a spectrum of renal dysfunction: AKI is
neither acute tubular necrosis nor renal failure, but this term
also includes less severe alterations in kidney function, including the rapidly reversible physiologic changes typical of prerenal
azotemia. RIFLE criteria have been validated as an independent
predictor of in-hospital mortality, with elevated risk of death
in all grades and increasing across grades.10-12 These criteria
have established the uniformity required for research on the
numerous unknowns that hinder the prevention and treatment
of AKI.

1. Who is at risk for AKI?


Although a combination of vascular, tubular, and inflammatory
factors underlie most cases of AKI, several cohort studies have
attempted to define risk factors for the development of AKI. Among
critically ill patients, sepsis is the most common condition associated with AKI. Bagshaw et al. found that AKI occurred in 42% of
septic patients admitted to intensive care units.13 In a prospective
cohort study, Uchino et al. studied 30,000 patients admitted to 54
intensive care units and found that sepsis contributed to 47% of
cases of AKI.14 This study also highlighted another group at particular risk for AKI, as 30% of critically ill patients who progressed
to AKI had some degree of baseline renal dysfunction. This finding is supported by data from Hsu et al. who reported that chronic
kidney disease is a risk factor for the onset of dialysis-requiring
AKI during hospitalization.15 Chronic kidney disease limits the
kidneys capacity for intrinsic repair, and this may contribute to
elevated rates of AKI demonstrated in patients with baseline renal
dysfunction. Diminished capacity for renal repair appears to place
older patients at risk for AKI as well.16 Retrospective reviews of
public and private health delivery systems data in the United States
reveal consistently elevated rates of AKI in older patients.2,4
Hypovolemia is a risk factor for renal ischemia and thus AKI.
Although trials are not necessary to reveal the benefit of fluid
repletion in hypovolemic shock, randomized clinical trials have
not demonstrated a difference between crystalloid and colloid
solutions in this setting.17 Reduced effective intravascular volume
also predisposes to AKI. This risk factor emerges through studies
of AKI in the setting of severe burns, liver failure, and cardiac
failure.18-20
Trauma patients are at risk for AKI, particularly those with
rhabdomyolysis. In a review of 436 consecutive admissions to a
level 1 trauma center, Gomes et al. found that 50% satisfied RIFLE

942

PMPH_CH119.indd 942

5/22/2012 6:21:41 PM

Acute Renal Dysfunction

criteria, with higher severity of trauma score more likely to result


in AKI.21 Small studies have revealed that the incidence of AKI in
rhabdomyolysis ranges from 20% to 50%.22-24
Both patient and surgical characteristics can herald perioperative AKI. Preexisting conditions such as diabetes, chronic
kidney diseases, cardiac disease, and advanced age can affect
rates of perioperative AKI.25,26 Surgical procedures that produce
higher rates of AKI include cardiac surgery requiring cardiopulmonary bypass, major intraabdominal surgery, vascular surgery
requiring aortic manipulation and/or cross-clamping, and organ
transplantation.
Answer: AKI occurs in numerous clinical settings and therefore has a variety of risk factors. Conditions that confer significant
risk of AKI include sepsis, baseline renal dysfunction, advanced
age, major surgery, and trauma, especially in the setting of burns
and rhabdomyolysis (Grade B recommendation).

DIAGNOSIS
2. What is the optimal diagnostic test for AKI?
Despite its prevalence, the timely recognition of AKI remains difficult due to inadequacies of serum creatinine (sCr), the current
diagnostic standard. RIFLE criteria require the identification
of a change in sCr from baseline. Th is creates several problems:
(1) baseline sCr may be unknown; (2) significant time may elapse
after an injury until sCr reaches a diagnostic threshold; (3) the
level of sCr may not accurately reflect the degree of renal injury,
since the kinetics of sCr are influenced by age, gender, muscle
mass, nutritional status, hemodynamics, fluid status, medications, and underlying Chronic Kidney Disease (CKD); and
(4) in fact, among hospitalized patients, transient azotemia is
a frequent cause of AKI.27-30 These limitations have generated
intense interest in the identification of biomarkers that allow for
the early diagnosis of AKI and the differentiation of AKI from
other processes, which also elevate sCr. 31,32 A variety of renal
insults have been found to elevate several urinary biomarkers, including urinary neutrophil gelatinase-associated lipocalin (uNGAL), kidney injury molecule 1 (uKIM-1), interleukin
18 (uIL-18), liver fatty acid binding protein (uL-FABP), cystatinC (uCysC) and the fractional excretion of sodium (FENa). 33-36
However, the investigation of these biomarkers to diagnose
AKI has been limited to only small, single-center studies, usually involving specific types of AKI in homogenous populations of
patients. Larger studies in heterogeneous populations are lacking, and it is unclear which urinary biomarker performs best in
prospectively diagnosing AKI at patient presentation.
Answer: While standardized creatinine-based definitions of
AKI now exist, novel biomarkers hold promise for expedient and
accurate diagnosis (Grade B recommendation).

MANAGEMENT
3. Does the time of renal replacement therapy initiation, modality, or intensity impact mortality?
Renal replacement therapy (RRT) is the definitive treatment for
complications of AKI (extracellular fluid volume overload and
solute imbalance) that are intractable to medical management.

PMPH_CH119.indd 943

943

Nevertheless, the current literature offers incomplete guidance


as to the optimal timing, method, and intensity of such therapy.
While case-control and retrospective studies suggested that
early dialysis reduces mortality, two randomized clinical trials
produced conflicting results. In 2002, a trial of 106 patients initiated early dialysis if urine output was less than 30 mL/h after
6 hours but did not find a difference with regard to mortality or
recovery of renal function in survivors.37 A 2004 trial found a large
reduction in mortality with early dialysis, defined by postoperative urine output (Relative Risk (RR), 0.17; 95%CI, 0.050.61), but
weak methods and a small sample size (N = 28) temper this conclusion.38 The lone observational study in this area found that the risk
of death in critically ill AKI patients was significantly decreased
by initiating RRT before levels of blood urea nitrogen were greater
than 76 mg/dL (adjusted hazard ratio, 0.54; CI, 0.340.86).39 On
the basis of this evidence, a 2008 systematic review concluded that
available literature does not permit a definitive statement as to the
optimal timing of acute RRT initiation.40
Debate persists regarding the preferred mode of dialysis as
well. Available methods include intermittent hemodialysis (IHD)
and continuous renal replacement therapy (CRRT), and significant
differences exist between these techniques. Intermittent hemodialysis is conventionally performed three times a week at 4 hours per
session via venovenous access and requires hemodynamic stability. Using venovenous or arteriovenous access and reduced blood
flow rates, CRRT is performed continuously and offers gradual solute and fluid clearance. Despite the theoretical advantage of more
physiologic restoration of solute and fluid balance, CRRT has not
been found to offer a survival benefit as compared to intermittent
hemodialysis. Although methodological concerns plague this literature, no recent randomized controlled trial has demonstrated
a mortality advantage to CRRT.41-45 Similarly, numerous metaanalyses have concluded that CRRT does not appear to confer a
survival advantage over intermittent hemodialysis.40,46-48
Six randomized trials have analyzed whether CRRT dose
impacts mortality. Two early studies produced preliminary evidence to suggest that increased CRRT intensity decreases mortality. Ronco et al. found that patients who received doses of
45 mL/kg/h or 35 mL/kg/h demonstrated reduced mortality compared with patients who received doses of 20 mL/kg/h (RR, 0.72;
95% CI, 0.540.94 and RR, 0.73; 95% CI, 0.560.96, respectively).49
No mortality difference was observed between patients receiving
45 mL/kg/h and 35 mL/kg/h. In 2006, Saudan et al. seemed to
replicate these findings by reporting a 20% reduction in all-cause
mortality at 90 days with the addition of dialysate at a dose of
18 mL/kg/h to continuous venovenous hemofi ltration at an ultrafi ltration rate of approximately 25 mL/kg/h, albeit it may be difficult to interpret the additive value of two distinct modalities. 50 In
spite of these studies, the preponderance of subsequent evidence
has come to suggest that CRRT dose does not affect mortality. In
2002, Bouman et al. found no decrease in mortality among 106
patients who sustained higher hemofi ltration volumes compared
with lower volumes (48 mL/kg/h vs. 20 mL/kg/h).37 Similarly, Tolwani et al. found no difference in mortality between 200 patients
randomized to receive CRRT dose of 35 mL/kg/h or 20 mL/kg/h.51
In a multicenter, 1100-patient randomized trial, Palevsky and
colleagues did not note a mortality difference between patients
receiving CRRT dose of 35 mL/kg/h versus 20 mL/kg/h.52 Bellomo
and colleagues conducted a 1500-patient trial and found no mortality difference at 90 days between patients who received a dose
of 40 mL/kg/h and those who received 25 mL/kg/h.53 In addition,

5/22/2012 6:21:41 PM

944

Surgery: Evidence-Based Practice

none of these studies reported a difference in rates of recovery of


renal function following more intense CRRT treatment. Given
this body of evidence, most authors recommend seeking to
achieve flow rates of 35 mL/kg/h during CRRT for AKI.
It should be noted that there has been recent interest in pursuing CRRT doses that approximate physiologic glomerular filtration rates. In 2008, Chung et al. applied CRRT initially at doses
as high as 120 mL/kg/h to 29 patients with AKI in the setting of
severe burns.54 Compared to historical controls, these patients
demonstrated reduced 28-day mortality. Larger randomized trials are underway to further investigate this intriguing concept.
Early studies also produced evidence which suggested that
more frequent intermittent hemodialysis might reduce mortality
among critically ill patients with AKI. Schiffl et al. studied 160
patients and found reduced mortality among patients treated with
daily IHD as compared to an alternating day schedule.55 Another
group found reduced mortality among 34 patients randomized
to receive either IHD to maintain blood urea nitrogen levels less
than 60 mg/dL and serum creatinine levels less than 5 mg/dL as
compared to conventional IHD schedule.56 In spite of these early
reports, a more recent randomized trial with 1100 patients failed
to demonstrate a mortality benefit to daily IHD.52
Answer: Neither time of RRT initiation nor modality nor
intensity seem to impact mortality or renal recovery (Grade B
recommendation).
4. What factors impact RRT selection?
Dialysis membranes are characterized by permeability to water,
flux, and by the degree to which the complement system is activated when the membrane is exposed to blood, an aspect known
as compatibility. Trials have analyzed whether these factors impact
survival and renal recovery in critically ill patients with AKI. A systematic review of the literature analyzed four trials that compared
high- and low-flux dialysis membranes in critically ill patients with
AKI. No difference was detected in either mortality (RR, 0.91; 95%
CI, 0.741.11) or chronic dialysis dependence among survivors
(RR, 1.02; 95% CI, 0.751.39).40 Given this and because high-flux
membranes aid hemofiltration, most authors recommend highflux membranes when using CRRT.
This same review analyzed seven trials that studied biocompatible and bioincompatible membranes. In these 719 patients,
a nonsignificant trend toward increased mortality was apparent among patients treated with bioincompatible membranes
(RR, 1.11; 95% CI, 0.941.31) without a difference in dialysis
dependence among survivors.40 The mortality risk became more
evident when membranes composed of unsubstituted cellulose, a
type of bioincompatible membrane, were employed.

Answer: Dialysis membrane characteristics do not appear to


impact mortality or preserve renal function. Unsubstituted cellulose membranes should be avoided (Grade B recommendation).
5. What are potential pharmacologic treatments of AKI?
As AKI is characterized by a spectrum of volume responsiveness,
initial therapeutic efforts may include a judicious trial of fluid repletion in the appropriate clinical setting. A recent systematic review
demonstrated that no difference exists between repletion with either
crystalloid or colloid solution. Diuretics are often used in an attempt
to improve urine output in AKI. Numerous studies have found that
diuretics do not decrease mortality or improve renal outcomes in
established renal failure.57-62 Although appropriate for short-term
volume control, diuretics should not delay dialysis initiation.
Various vasoactive substances have been trialed in AKI. Low
or renal dose dopamine has been proposed to preferentially reduce
renal vasoconstriction, and thus has been advocated as a technique
to ameliorate renal dysfunction. Several recent meta-analyses have
not documented reduced mortality or improved renal function
with low-dose dopamine and have explicitly argued against its
use.63,64 Fenoldapam is a pure dopamine A-1 receptor agonist that
increases blood flow to the renal cortex and outer medulla. Two
meta-analyses, one in critically ill patients following cardiac surgery and one in critically ill patients with or at risk for AKI, found
that fenoldapam reduced the need for renal replacement therapy,
decreased mortality, and reduced length of stay.65,66 Heterogeneity
among analyzed studies severely limits this conclusion; large randomized clinical trials are needed before widespread implementation is justified. Atrial natriuretic peptide (ANP) is produced by
cardiac myocytes and increases glomerular filtration rate through
afferent arteriolar vasodilation and efferent arteriolar vasoconstriction. ANP has been mainly studied in the setting of postcardiac
surgery AKI. Although systematic reviews found a reduced need
for RRT in AKI patients, other meta-analyses have concluded that
the lack of high-quality studies limits this conclusion.67,68
Another area of active research concerns the use of growth
factors in the treatment of AKI. Noting that expression in animal
models of insulin-like growth factor (IGF) decreased during ischemia and increased coincident with renal recovery, Hammerman
found that IGF administration ameliorated AKI in animal models.69 Translation of this concept from animals to humans has, however, proven difficult. Similarly, while large doses of erythropoietin
improved AKI in animal models, a randomized, placebo-controlled
trial of erythropoietin revealed that prophylactic administration
did not decrease rates of AKI in the intensive care setting.70
Answer: Medical therapies to limit or reverse AKI have thus
far eluded researchers (Grade B recommendation).

Clinical Question Summary


Question

Answer

Grade of
Recommendation

References

1 Who is at risk for AKI?

Patients with sepsis, baseline renal dysfunction, older


age, trauma, major surgery, or hypovolemia.

13-26

2 What is the optimal diagnostic


test for AKI?

Serial serum creatinine and urine output


measurements currently define acute kidney injury.

27-36
(Continued)

PMPH_CH119.indd 944

5/22/2012 6:21:42 PM

Acute Renal Dysfunction

945

(Continued)
Question

Answer

3 Does the time of renal replacement


therapy initiation, modality, or
dose impact mortality?

The effectiveness of early initiation, modality, and


dialysis intensity is unclear.

37-56

4 What factors impact RRT


selection?

Membrane characteristics do not appear to alter


mortality.

39

5 What are potential pharmacologic


treatments of AKI?

Medical therapies to limit or reverse AKI have thus far


eluded researchers.

57-70

REFERENCES
1. Chertow GM, Burdick E, Honour M, Bonventre JV, Bates DW.
Acute kidney injury, mortality, length of stay, and costs in hospitalized patients. J Am Soc Nephrol. 2005;16(11):3365-3370.
2. Hsu CY, McCulloch CE, Fan D, Ordonez JD, Chertow GM, Go
AS. Community-based incidence of acute renal failure. Kidney
Int. 2007;72(2):208-212.
3. Waikar SS, Curhan GC, Wald R, McCarthy EP, Chertow GM.
Declining mortality in patients with acute renal failure, 1988 to
2002. J Am Soc Nephrol. 2006;17(4):1143-1150.
4. Xue JL, Daniels F, Star RA, et al. Incidence and mortality of acute
renal failure in Medicare beneficiaries, 1992 to 2001. J Am Soc
Nephrol. 2006;17(4):1135-1142.
5. Venkatachalam MA, Griffin KA, Lan R, Geng H, Saikumar P,
Bidani AK. Acute kidney injury: a springboard for progression
in chronic kidney disease. Am J Physiol Renal Physiol. 2010 May;
298(5):F1078F1094.
6. Nash K, Hafeez A, Hou S. Hospital-acquired renal insufficiency.
Am J Kidney Dis. 2002;39(5):930-936.
7. Liano F, Pascual J. Epidemiology of acute renal failure: a prospective, multicenter, community-based study. Madrid Acute
Renal Failure Study Group. Kidney Int. 1996;50(3):811-818.
8. Lassnigg A, Schmidlin D, Mouhieddine M, et al. Minimal changes
of serum creatinine predict prognosis in patients after cardiothoracic surgery: a prospective cohort study. J Am Soc Nephrol.
2004;15(6):1597-1605.
9. Hoste EA, Clermont G, Kersten A, et al. RIFLE criteria for acute
kidney injury are associated with hospital mortality in critically
ill patients: a cohort analysis. Crit Care. 2006;10(3):R73.
10. Abosaif NY, Tolba YA, Heap M, Russell J, El Nahas AM. The outcome of acute renal failure in the intensive care unit according
to RIFLE: model application, sensitivity, and predictability. Am J
Kidney Dis. 2005;46(6):1038-1048.
11. Uchino S, Bellomo R, Goldsmith D, Bates S, Ronco C. An assessment of the RIFLE criteria for acute renal failure in hospitalized
patients. Crit Care Med. 2006;34(7):1913-1917.
12. Ostermann M, Chang RW. Acute kidney injury in the intensive
care unit according to RIFLE. Crit Care Med. 2007;35(8):1837-1843;
quiz 1852.
13. Bagshaw SM, George C, Bellomo R, Committee ADM. Early acute
kidney injury and sepsis: a multicentre evaluation. Crit Care.
2008;12(2):R47.
14. Uchino S, Kellum JA, Bellomo R, et al. Acute renal failure in critically ill patients: a multinational, multicenter study. JAMA. 2005;
294(7):813-818.
15. Hsu CY, Ordonez JD, Chertow GM, Fan D, McCulloch CE, Go
AS. The risk of acute renal failure in patients with chronic kidney
disease. Kidney Int. 2008;74(1):101-107.

PMPH_CH119.indd 945

Grade of
Recommendation

References

16. Rosner MH. The pathogenesis of susceptibility to acute kidney


injury in the elderly. Curr Aging Sci. 2009;2(2):158-164.
17. Finfer S, Bellomo R, Boyce N, et al. A comparison of albumin
and saline for fluid resuscitation in the intensive care unit. N
Engl J Med. 2004;350(22):2247-2256.
18. Brusselaers N, Monstrey S, Colpaert K, Decruyenaere J, Blot SI,
Hoste EA. Outcome of acute kidney injury in severe burns: a
systematic review and meta-analysis. Intensive Care Med. 2010;
36(6):915-925.
19. Gines P, Guevara M, Arroyo V, Rodes J. Hepatorenal syndrome.
Lancet. 2003;362(9398):1819-1827.
20. Ronco C, McCullough P, Anker SD, et al. Cardio-renal syndromes: report from the consensus conference of the acute dialysis quality initiative. Eur Heart J. 2010;31(6):703-711.
21. Gomes E, Antunes R, Dias C, Araujo R, Costa-Pereira A. Acute
kidney injury in severe trauma assessed by RIFLE criteria: a common feature without implications on mortality? Scand J Trauma
Resusc Emerg Med. 2010;18:1.
22. Holt SG, Moore KP. Pathogenesis and treatment of renal dysfunction in rhabdomyolysis. Intensive Care Med. 2001;27(5):
803-811.
23. Melli G, Chaudhry V, Cornblath DR. Rhabdomyolysis: an evaluation of 475 hospitalized patients. Medicine (Baltimore). 2005;
84(6):377-385.
24. Ward MM. Factors predictive of acute renal failure in rhabdomyolysis. Arch Intern Med. 1988;148(7):1553-1557.
25. Mehta RH, Grab JD, OBrien SM, et al. Bedside tool for predicting the risk of postoperative dialysis in patients undergoing cardiac surgery. Circulation. 2006;114(21):2208-2216; quiz
2208.
26. Thakar CV, Arrigain S, Worley S, Yared JP, Paganini EP. A clinical score to predict acute renal failure after cardiac surgery. J Am
Soc Nephrol. 2005;16(1):162-168.
27. Waikar SS, Bonventre JV. Creatinine kinetics and the definition
of acute kidney injury. J Am Soc Nephrol. 2009;20(3):672-679.
28. Perrone RD, Madias NE, Levey AS. Serum creatinine as an index
of renal function: new insights into old concepts. Clin Chem. 1992;
38(10):1933-1953.
29. Star RA. Treatment of acute renal failure. Kidney Int. 1998;
54(6):1817-1831.
30. Uchino S, Bellomo R, Bagshaw SM, Goldsmith D. Transient
azotaemia is associated with a high risk of death in hospitalized
patients. Nephrol Dial Transplant. 2010;25(6):1833-1839.
31. Szczech LA. The development of urinary biomarkers for kidney
disease is the search for our renal troponin. J Am Soc Nephrol.
2009;20(8):1656-1657.
32. Honore PM, Joannes-Boyau O, Boer W. The early biomarker of
acute kidney injury: in search of the Holy Grail. Intensive Care
Med. 2007;33(11):1866-1868.

5/22/2012 6:21:42 PM

946

Surgery: Evidence-Based Practice

33. Coca SG, Yalavarthy R, Concato J, Parikh CR. Biomarkers for


the diagnosis and risk stratification of acute kidney injury: a systematic review. Kidney Int. 2008;73(9):1008-1016.
34. Sise ME, Barasch J, Devarajan P, Nickolas TL. Elevated urine
neutrophil gelatinase-associated lipocalin can diagnose acute
kidney injury in patients with chronic kidney diseases. Kidney
Int. 2009;75(1):115-116; author reply 116.
35. Portilla D, Dent C, Sugaya T, et al. Liver fatty acid-binding protein as a biomarker of acute kidney injury after cardiac surgery.
Kidney Int. 2008;73(4):465-472.
36. Miller TR, Anderson RJ, Linas SL, et al. Urinary diagnostic indices in acute renal failure: a prospective study. Ann Intern Med.
1978;89(1):47-50.
37. Bouman CS, Oudemans-Van Straaten HM, Tijssen JG, Zandstra
DF, Kesecioglu J. Effects of early high-volume continuous venovenous hemofi ltration on survival and recovery of renal function
in intensive care patients with acute renal failure: a prospective,
randomized trial. Crit Care Med. 2002;30(10):2205-2211.
38. Sugahara S, Suzuki H. Early start on continuous hemodialysis
therapy improves survival rate in patients with acute renal failure following coronary bypass surgery. Hemodial Int. 2004;8(4):
320-325.
39. Liu KD, Himmelfarb J, Paganini E, et al. Timing of initiation of
dialysis in critically ill patients with acute kidney injury. Clin J
Am Soc Nephrol. 2006;1(5):915-919.
40. Pannu N, Klarenbach S, Wiebe N, Manns B, Tonelli M, Alberta
Kidney Disease N. Renal replacement therapy in patients with
acute renal failure: a systematic review. JAMA. 2008;299(7):
793-805.
41. Mehta RL, McDonald B, Gabbai FB, et al. A randomized clinical trial of continuous versus intermittent dialysis for acute renal
failure. Kidney Int. 2001;60(3):1154-1163.
42. Uehlinger DE, Jakob SM, Ferrari P, et al. Comparison of continuous and intermittent renal replacement therapy for acute renal
failure. Nephrol Dial Transplant. 2005;20(8):1630-1637.
43. Augustine JJ, Sandy D, Seifert TH, Paganini EP. A randomized
controlled trial comparing intermittent with continuous dialysis
in patients with ARF. Am J Kidney Dis. 2004;44(6):1000-1007.
44. Vinsonneau C, Camus C, Combes A, et al. Continuous venovenous haemodiafi ltration versus intermittent haemodialysis
for acute renal failure in patients with multiple-organ dysfunction syndrome: a multicentre randomised trial. Lancet. 2006;
368(9533):379-385.
45. Cho KC, Himmelfarb J, Paganini E, et al. Survival by dialysis
modality in critically ill patients with acute kidney injury. J Am
Soc Nephrol. 2006;17(11):3132-3138.
46. Kellum J, Palevsky PM. Renal support in acute kidney injury.
Lancet. 2006;368(9533):344-345.
47. Bagshaw SM, Berthiaume LR, Delaney A, Bellomo R. Continuous versus intermittent renal replacement therapy for critically
ill patients with acute kidney injury: a meta-analysis. Crit Care
Med. 2008;36(2):610-617.
48. Rabindranath K, Adams J, Macleod AM, Muirhead N. Intermittent versus continuous renal replacement therapy for acute renal
failure in adults. Cochrane Database Syst Rev. 2007;3:CD003773.
49. Ronco C, Bellomo R, Homel P, et al. Effects of different doses
in continuous veno-venous haemofi ltration on outcomes of
acute renal failure: a prospective randomised trial. Lancet. 2000;
356(9223):26-30.
50. Saudan P, Niederberger M, De Seigneux S, et al. Adding a dialysis
dose to continuous hemofi ltration increases survival in patients
with acute renal failure. Kidney Int. 2006;70(7):1312-1317.

PMPH_CH119.indd 946

51. Tolwani AJ, Campbell RC, Stofan BS, Lai KR, Oster RA, Wille
KM. Standard versus high-dose CVVHDF for ICU-related acute
renal failure. J Am Soc Nephrol. 2008;19(6):1233-1238.
52. Network VNARFT, Palevsky PM, Zhang JH, et al. Intensity of
renal support in critically ill patients with acute kidney injury.
N Engl J Med. 2008;359(1):7-20.
53. Investigators RRTS, Bellomo R, Cass A, et al. Intensity of continuous renal-replacement therapy in critically ill patients. N Engl J
Med. 2009;361(17):1627-1638.
54. Chung KK, Lundy JB, Matson JR, et al. Continuous venovenous
hemofi ltration in severely burned patients with acute kidney
injury: a cohort study. Crit Care. 2009;13(3):R62.
55. Schiffl H, Lang SM, Fischer R. Daily hemodialysis and the outcome of acute renal failure. N Engl J Med. 2002;346(5):305-310.
56. Gillum DM, Dixon BS, Yanover MJ, et al. The role of intensive dialysis in acute renal failure. Clin Nephrol. 1986;25(5):
249-255.
57. Hager B, Betschart M, Krapf R. Effect of postoperative intravenous loop diuretic on renal function after major surgery. Schweiz
Med Wochenschr. 1996;126(16):666-673.
58. van der Voort PH, Boerma EC, Koopmans M, et al. Furosemide
does not improve renal recovery after hemofi ltration for acute
renal failure in critically ill patients: a double blind randomized
controlled trial. Crit Care Med. 2009;37(2):533-538.
59. Ho KM, Sheridan DJ. Meta-analysis of frusemide to prevent or
treat acute renal failure. BMJ. 2006;333(7565):420.
60. Mehta RL, Pascual MT, Soroko S, Chertow GM, Group PS.
Diuretics, mortality, and nonrecovery of renal function in acute
renal failure. JAMA. 2002;288(20):2547-2553.
61. Bagshaw SM, Delaney A, Haase M, Ghali WA, Bellomo R. Loop
diuretics in the management of acute renal failure: a systematic
review and meta-analysis. Crit Care Resusc. 2007;9(1):60-68.
62. Sampath S, Moran JL, Graham PL, Rockliff S, Bersten AD,
Abrams KR. The efficacy of loop diuretics in acute renal failure:
assessment using Bayesian evidence synthesis techniques. Crit
Care Med. 2007;35(11):2516-2524.
63. Holmes CL, Walley KR. Bad medicine: low-dose dopamine in
the ICU. Chest. 2003;123(4):1266-1275.
64. Friedrich JO, Adhikari N, Herridge MS, Beyene J. Meta-analysis:
low-dose dopamine increases urine output but does not prevent renal dysfunction or death. Ann Intern Med. 2005;142(7):
510-524.
65. Landoni G, Biondi-Zoccai GG, Marino G, et al. Fenoldopam
reduces the need for renal replacement therapy and in-hospital
death in cardiovascular surgery: a meta-analysis. J Cardiothorac
Vasc Anesth. 2008;22(1):27-33.
66. Landoni G, Biondi-Zoccai GG, Tumlin JA, et al. Beneficial impact
of fenoldopam in critically ill patients with or at risk for acute
renal failure: a meta-analysis of randomized clinical trials. Am J
Kidney Dis. 2007;49(1):56-68.
67. Nigwekar SU, Navaneethan SD, Parikh CR, Hix JK. Atrial natriuretic peptide for preventing and treating acute kidney injury.
Cochrane Database Syst Rev. 2009;4:CD006028.
68. Nigwekar SU, Navaneethan SD, Parikh CR, Hix JK. Atrial
natriuretic peptide for management of acute kidney injury: a
systematic review and meta-analysis. Clin J Am Soc Nephrol.
2009;4(2):261-272.
69. Hammerman MR. Growth factors and apoptosis in acute renal
injury. Curr Opin Nephrol Hypertens. 1998;7(4):419-424.
70. Endre ZH, Walker RJ, Pickering JW, et al. Early intervention
with erythropoietin does not affect the outcome of acute kidney
injury. Kidney Int. 2010;77(11):1020-1030.

5/22/2012 6:21:42 PM

Commentary on
Acute Renal Dysfunction
Rao R. Ivatury

Drs ORourke and Barasch have nicely summarized the current


evidence and provided thoughtful answers to commonly posed
questions regarding the difficult problem of acute kidney injury
(AKI). Our understanding of AKI injury is only being refined
recently, albeit with multinational studies, meta-analyses, and
consensus definitions. Despite the admonition of evidence-based
schemes to disregard consensus opinions, in the absence of higher
levels of evidence, such consensus definitions and concepts, especially when enunciated and adjudicated by international expert
committees, can lend clarity to existing thought on the subject.
Such an event occurred in May 2007 and addressed Prevention
and Management of Acute Renal Failure in the ICU Patient with
participation from five distinguished societies: ATS/ERS/ESICM/
SCCM/SRLF. Five important questions were addressed and a jury
summarized the available evidence in a publication in May 2010.1
The reader is encouraged to study this report carefully along with
the excellent chapter by Drs ORourke and Barasch.
The panel recommended the use of acute kidney insufficiency (AKI) as the most appropriate description of kidney injury
and recognized the following as risk factors: age, sepsis, infusion
of contrast medium, diabetes, rhabdomyolysis, preexisting renal
disease, hypovolemia, shock, and high-risk surgical situations,
such as major intraabdominal surgery, vascular surgery requiring
aortic manipulation and/or cross clamping, cardiac surgery, and
organ transplantation. The panelists confirmed that AKI significantly contributes to the morbidity and mortality of critically ill
patients.
In terms of diagnosis of AKI, it is important to bear in mind
the wide spectrum of disease, which is reflected by two definitions. The first is the RIFLE classification (R for Risk: a 1.5-fold
increase in serum creatinine or oliguria more than 6 hours, I for
Injury: a 2-fold increase in serum creatinine or oliguria for 12
hours, and F for Failure: a 3-fold increase in creatinine or a level
of 4 mg/dL with an acute increase of at least 0.5 mg/dL or anuria
for 12 hours). The outcome variables are denoted by loss (L) and
end-stage kidney disease (E). The second definition, AKIN (acute
kidney injury network), was proposed by three nephrology societies and the European Society of Intensive Care Medicine. The
three stages of increasing severity are defined by worsening creatinine or decreased urine output. It is evident that, despite their
limitations, the time-honored criteria of serum creatinine and
urine output constitute the diagnostic basis of AKI. As the authors
of the current chapter correctly concluded, this is a Grade B

recommendation. Newer biomarkers are being studied but are not


ready for widespread clinical use.2
As in other areas of AKI, no Level I evidence or Grade A
recommendations exist regarding renal replacement therapy
(RRT). RRT aims at averting the immediate life-threatening
consequences of severe renal dysfunction. The two forms of RRT,
continuous and intermittent, have their own perceived advantages in terms of efficacy, ease of administration, and cost.3 Each
has its supporters and neither enjoys defi nitive evidence in its
favor. Numerous studies, including a few small randomized
studies, produced confl icting results. More recently, two large
multicenter, randomized controlled trials, the Veterans Affairs/
National Institutes of Health Acute Renal Failure Trial Network
(ATN) study and the Randomized Evaluation of Normal versus
Augmented Level Replacement Therapy (RENAL) trial, have now
examined the use of RRT in the ICU. The ATN study was conducted in ICUs throughout the United States and the RENAL
study was conducted in ICUs in Australia and New Zealand. No
significant differences in mortality rates were observed between
high-intensity and low-intensity treatment in prespecified subgroups (patients with sepsis and patients requiring vasopressors)
in either study. No definitive evidence was found to recommend
escalation of continuous RRT beyond conventional doses of 25
mL/kg/h. Many questions still remained regarding the timing of
therapy, the role of intermittent dialysis in the ICU, and the effect
of therapy choice on renal recovery. What about early versus late
initiation of RRT in critically ill patients? In a very recent report,
Karvellas et al.4 analyzed existing literature in a meta-analysis.
They concluded that the evidence in favor of early therapy having survival benefit was not supported by high-quality evidence.
Similarly, the evidence for supporting high-dose RRT, synthetic
dialysis membranes (as opposed to cellulose-based membranes)
was found to be inconclusive.
If RRT is not the definitive answer for AKI, is there any role
for other therapies? Once again, the evidence is less than convincing.5 Loop diuretics plus fluids seem to increase the risk of
developing acute renal failure compared with fluids alone, both in
high-risk and critically ill patients. In addition, they do not seem
to improve renal function or mortality compared with placebo in
people with AKI. In fact, they may add to morbidity by increasing
the risk of ototoxicity and volume depletion. Similarly, there was
no supporting evidence in favor of continuous infusion of loop
diuretics with or without intravenous albumin supplementation.
947

PMPH_CH119.indd 947

5/22/2012 6:21:42 PM

948

Surgery: Evidence-Based Practice

Even though two meta-analyses6,7 reported beneficial effects with


fenoldapam, a pure dopamine A-1 receptor agonist that increases
blood flow to the renal cortex and outer medulla, these studies
have several limitations calling into question their results. Neither
natriuretic peptides nor dopamine seem beneficial in either highrisk or critically ill patients.
In spite of this litany of negative comments, a lot of good treatment strategies can be provided to the AKI patient. The reader is
advised to study in detail the following recommendations proposed by the consensus panel1:

Correct hypovolemia.
Use diuretics only to test renal responsiveness after adequate
fluid loading.
Avoid and discontinue all potentially nephrotoxic drugs.
Correct hypotension as quickly as possible with a target MAP
of 65 mm Hg or greater.
Supplement protein intake to counteract the highly catabolic
state by protein administration of up to 2.0 g/kg/day. If possible, determine protein and caloric requirements on an individual basis using metabolic measurements.
Correct metabolic derangements, reduce fluid overload, and
mitigate the harmful effects of these disturbances on other
failing organs. Look for and control an inciting or exacerbating
event (sepsis, intraabdominal hypertension, etc.).

By following these principles and providing meticulous critical


care by a multidisciplinary approach to these patients, the prognosis of AKI can be improved. Further improvements in AKI must

PMPH_CH119.indd 948

await carefully planned and executed randomized trials to determine optimal management.

REFERENCES
1. Brochard L, Abroug F, Brenner M, et al. An official ATS/ERS/
ESICM/SCCM/SRLF statement: prevention and management of
acute renal failure in the ICU patient. Am J Respir Crit Care Med.
2010;181(10):1128-1155.
2. Bentley ML. Acute kidney insufficiency in the critically ill. J Pharm
Practice. 2011;24(1):61-69.
3. Prowle JR, Bellomo R. Continuous renal replacement therapy:
recent advances and future research. Nat Rev Nephrol. 2010;6(9):
521-529. Epub 2010 Jul 20.
4. Karvellas CJ, Farhat MR, Sajjad I, et al. A comparison of early versus
late initiation of renal replacement therapy in critically ill patients
with acute kidney injury: a systematic review and meta-analysis. Crit
Care. 2011;15(1):R72.
5. Kellum JA, Leblanc M, Venkataraman R. Acute renal failure. Clin
Evid (Online). 2008; 2008:2001 Published online 2008 September 3.
6. Landoni G, Biondi-Zoccai GG, Marino G, et al. Fenoldopam
reduces the need for renal replacement therapy and in-hospital
death in cardiovascular surgery: a meta-analysis. J Cardiothorac
Vasc Anesth. 2008;22(1):27-33.
7. Landoni G, Biondi-Zoccai GG, Tumlin JA, et al. Beneficial impact
of fenoldopam in critically ill patients with or at risk for acute
renal failure: a meta-analysis of randomized clinical trials. Am J
Kidney Dis. 2007;49(1):56-68.

5/22/2012 6:21:42 PM

CHAPTER 120

Perioperative Endocrine
Dysfunction
Catherine A. Madorin and Kaare J. Weber

DIABETES

Surgery and general anesthesia induce a stress response that


affects glucose homeostasis. The release of epinephrine, glucagon,
cortisol, growth hormone, and inflammatory cytokines all contribute to elevation of glucose levels after surgery. The type of procedure will influence the amount of this hormone release. General
anesthesia is associated with a higher stress response than epidural anesthesia or sedation. Likewise, extensive surgeries such as
cardiac bypass induce higher levels of insulin resistance. Nil per
os (NPO) status prior to and often after surgery may make the
glycemic response harder to predict.
The goals of perioperative diabetic management include
maintaining fluid and electrolyte balance and avoiding diabetic
ketoacidosis (DKA), hypoglycemia, and significant hyperglycemia.
There is a paucity of evidence-based data on perioperative glucose
targets, and therefore guidelines vary widely.7 Uncontrolled diabetes can lead to volume depletion from osmotic diuresis, as well
as life-threatening conditions such as DKA or nonketotic hyperosmolar state (NKH). Patients with type 1 DM are insulin deficient and more likely to develop ketosis. In general, blood glucose
(BG) levels should be between 80 and 150 mg/dL, even in nondiabetics. The American Diabetes Association endorses fasting
glucose levels of 140 mg/dL for hospitalized patients, and random
BG <180 mg/dL. For ICU or post-MI patients, studies show that
maintaining a BG of 80 to 110 mg/dL may reduce mortality. However, additional analysis of two randomized trials looking at tight
control of BG in diabetic ICU patients suggests that there may be
no mortality benefit for known diabetics.8 An observational study
of 531 patients admitted to surgical and medical ICU suggests that
there is an association between lower BG levels (<200 mg/dL) and
lower mortality.9

1. What are the perioperative concerns for diabetic patients?


Diabetes mellitus (DM) affects over 8% of the US population, and it
is estimated that up to 50% of these patients will undergo surgery. In
addition, the many complications of DM including neuropathy, retinopathy, nephropathy, and vasculopathy will often ultimately require
surgical intervention.1,2 Diabetic patients are also more likely to suffer
from high cardiovascular morbidity and perioperative infection.3,4
Primary concern should be focused on cardiopulmonary
risk assessment and modification. Other comorbidities, such as
hypertension, obesity, and renal disease, must also be considered.
Preoperative evaluation of the diabetic patient should begin with
assessment of glycemic control and complications of diabetes. A
high HbA1c level is indicative of poor glycemic control and levels
>7 may signify higher risk of postoperative infection.5 Likewise,
glucose levels >200 mg/dL have also been associated with higher
incidence of deep wound infections.6 (See Table 120.1.)
Table 120.1 Preoperative Considerations for Diabetic
Patients

Type of DMI vs. II

HbA1c level, frequency of monitoring and glycemic range

Type of procedure: cardiac vs. noncardiac, length and


complexity of case

2. How is glycemic control best optimized in the diabetic


patient undergoing surgery?

Type of anesthesia required: general, epidural, sedation,


local

Perioperative glycemic control differs depending on the type of diabetes. Diabetic patients treated with diet alone often do not require
additional perioperative treatment. Patients on oral hypoglycemic
agents should discontinue treatment on the morning of surgery.

Evaluation of complications such as HTN, CAD, PVD, renal


disease, retinopathy, neuropathy

Presence and frequency of hypoglycemic episodes


Current and former medical treatment of DM

HTN = hypertensive nephropathy; CAD = coronary artery disease; PVD = peripheral


vascular disease.

949

PMPH_CH120.indd 949

5/22/2012 6:22:12 PM

950

Surgery: Evidence-Based Practice

Table 120.2 Complications of Oral Hypoglycemic Agents


Long-Acting sulfonylureas

Hypoglycemia, weight gain

Metformin

Lactic acidosis contraindication: renal failure, hepatic failure

Thiazolidinediones

Fluid retention, peripheral edema, congestive heart failure

DPP-IV inhibitors

Postoperative ileus

Long-acting sulfonylureas in particular increase the risk of hypoglycemia. Metformin is associated with lactic acidosis in the setting
of renal insufficiency. Thiazolidinediones may worsen fluid retention and exacerbate peripheral edema and congestive heart failure.
Newer agents, such as DPP-IV inhibitors, may adversely alter gastrointestinal (GI) motility and prolong postoperative ileus.
Preoperatively, insulin-dependent patients may replace longacting insulin for intermediate-acting insulin for 1 to 2 days prior
to surgery to avoid hypoglycemia. Another strategy is to reduce
the dose of long-acting insulin on the night before surgery. There
is no data to support one method over the other. Intravenous (IV)
dextrose and hydration should be given while the patient is NPO.
Type 1 diabetic patients must be continued on basal insulin even
when NPO to avoid ketoacidosis. This is because basal metabolism
demands one half of daily insulin in the absence of oral intake.10
The type and timing of the procedure also affects the insulin
regimen. For short, ambulatory procedures, patients may hold or
decrease their morning dose of insulin. Ideally, only intermediateacting insulin should be used in order to provide intraoperative
basal insulin and avoid ketosis. Dextrose should be given with
IV fluids to avoid hypoglycemia. Long procedures will usually
require intraoperative IV insulin to provide stable BG levels.10,11
Insulin and glucose may be infused separately or as combined
glucose insulin potassium (GIK) solution. Certain procedures,
such as coronary artery bypass, are associated with much higher
insulin requirements than others.12
Postoperative glycemic control is dependent on the dietary
status of the patient. For patients who remain NPO postoperatively, insulin infusions may be continued or subcutaneous insulin may be given concurrently with dextrose solution. A sliding
scale of short-acting insulin usually is added to provide extra coverage for hyperglycemia. It is important to note that sliding scale
regimens are not intended as the sole treatment of diabetes as they
are purely reactive and may lead to wide fluctuations in glucose
levels. Once adequate oral intake is established, patients may be
restarted on home oral or insulin regimens. Care must be taken
to consider the side-effect profi le of oral agents before deciding to
restart them (see Table 120.2).

THYROID
3. What is the perioperative management of hyperthyroidism?
A patient with known or suspected thyroid disease should be
evaluated with a thyroid function panel. Hyperthyroidism is most
commonly caused by the diff use enlargement of the thyroid gland
in Graves disease, but it may also occur with thyroiditis, toxic
solitary nodule, or toxic multinodular goiter. Secondary causes of
hyperthyroidism include pregnancy and choriocarcinoma. Manifestations of hyperthyroidism include weight loss, heat intolerance, diarrhea, weakness, tachycardia, and cardiac arrythmias.

PMPH_CH120.indd 950

Elective surgery in hyperthyroid patients should be deferred


until the patient is euthyroid. Patients taking antithyroid medication such as propylthiouracil (PTU) or methimazole, and those
on -blockers or digoxin, should take their medication on the day
of surgery.13 An electrolyte panel and electrocardiogram (EKG)
are essential parts of the preoperative evaluation to confirm a
euthyroid state. Response to medical management is measured by
the normalization of free T4 and T3 levels, as thyroid stimulating
hormone (TSH) may takes months to respond, even though the
patient is clinically euthyroid. In addition, patients with enlarging goiters can result in compressive symptoms, including airway
compromise that may complicate intubation for general anesthesia. Additional preoperative imaging including CT scan can help
define the extent of compression and aid in the planning of airway
management. Preoperative imaging is essential for the anesthesiologist in planning the approach to intubation, which may include
fiberoptic intubation or even awake intubation.
Thyroid storm is thyrotoxicosis that occurs with stressinduced exacerbation of the hyperthyroid state. It is characterized by life-threatening hyperpyrexia, tachycardia, stupor, and
coma. Differential diagnosis includes malignant hyperthermia,
pheochromocytoma crisis, and neuroleptic malignant syndrome.
There are no diagnostic tests for thyroid storm, and a high clinical
suspicion is necessary. Treatment of thyroid storm includes correction of the precipitating cause, administration of antithyroid
medications, iodine, and supportive care.14,15 Hyperpyrexia should
be treated with acetaminophen. Aspirin should be avoided, as it
may increase levels of circulating thyroid hormone.16 -blockers
are used to treat tachycardia; thionamides (PTU and methimazole) block new thyroid hormone synthesis; and iodine solution
blocks the release of thyroid hormone. Select patients may benefit
from glucocorticoids to decrease T4 to T3 conversion and augment the stress response. The combined use of PTU, iodine, and
dexamethasone has been shown to normalize levels in T3 within
24 to 48 hours.15 (See Table 120.3.)
4. How should hypothyroid patients be optimized for surgery?
Hypothyroidism is common, affecting approximately 1% of the
population. It is characterized by lethargy, fatigue, anorexia,
headache, depression, and cold intolerance. Chronic autoimmune
thyroiditis (Hashimotos thyroiditis) is the most common cause
of hypothyroidism, but there are many iatrogenic causes of hypothyroidism. Patients with or who have undergone treatment for
pituitary or hypothalamic disorders may be hypothyroid. Many
common drugs may also induce hypothyroidism, including lithium, amiodarone, iron, and cholestyramine. The preferred treatment of hypothyroidism is tetraiodothyronine (T4) replacement
therapy with levothyroxine.
Patients with mild to moderate hypothyroidism may undergo
surgery without significant additional risk.17-19 These patients are
more sensitive, however, to sedatives and analgesics. Stress from

5/22/2012 6:22:13 PM

Perioperative Endocrine Dysfunction

951

Table 120.3 Medical Treatment of Thyroid Storm


IV Fluids

Correct volume depletion and hypernatriemia

Antithyroid medications (PTU, methimazole)

PTU is preferable to methimazole as it inhibits conversion of T4 to T3 in both


peripheral and thyroid tissue

Iodine

Blocks the release of thyroid hormone

Acetaminophen

Lower temperature

Dexamethasone

Augment and support stress response

Beta-blockers

Decrease tachycardia

surgery may also unmask previously subclinical hypothyroidism.


Patients with severe hypothyroidism, manifested by pericardial
effusion or heart failure, who must undergo urgent or emergent surgery should be given intravenous levothyroxine prior to and after
surgery. Severe hypothyroidism is associated with an increased
risk of postoperative adrenal insufficiency. In addition, thyroid
hormone replacement has also been known to precipitate adrenal
crisis. Therefore, it is recommended that glucocorticoids be administered to severely hypothyroid patients undergoing surgery.20
Myxedema coma is a rare postoperative complication of severe
hypothyroidism. It is characterized by hypothermia, bradycardia,
hyponatriemia, heart failure, seizures, and coma. Infection, cold
exposure, sedatives, and analgesics are common precipitating
exposures. The mainstay of therapy is urgent administration of
intravenous levothyroxine. Dehydration is commonly present and
patients usually require aggressive volume resuscitation. Caution
should be used when attempting to treat hypothermia, as it will
induce vasodilatation that may precipitate cardiovascular collapse
in patients with intravascular volume depletion and cardiac insufficiency.21 Adrenal insufficiency is commonly seen in myxedema
coma and all patients should receive intravenous glucocorticoids.
Resolution of symptoms is usually seen within 24 hours.20

PARATHYROID

hypovolemia, and patients should have electrolyte abnormalities


and volume depletion corrected preoperatively. An ECG is sufficient to evaluate for short PR or QT intervals from hypercalcemia.
For patients with pHPT undergoing elective procedures in whom
parathyroidectomy is not feasible, hydration, diuretics, and use
of calicitonin can temper hyperparathyroidism to allow for safe
intervention.
Parathyroidectomy for single gland disease may result in
hypocalcemia due to hungry bone syndrome, with increased bone
uptake of calcium, phosphate, and magnesium. After removal of
continual hyperparathyroid stimulation, the bones may act as a
sink for calcium, precipitating symptomatic hypocalcemia. Symptoms will improve over weeks to months depending on the severity of bone disease. Hypocalcemia due to hypoparathyroidism can
be seen after four gland parathyroid exploration as well as after
total thyroidectomy from injury to the parathyroid glands. Most
patients regain parathyroid function over days to weeks, but it is
only considered permanent after 6 months. Retrospective studies
suggest that the incidence of significant hypocalcemia (calcium
<8.5 mg/dL beyond the third postoperative day) after total parathyroidectomy is between 13% and 20%.22,23
Symptomatic patients may require intravenous calcium gluconate. Most patients can be discharged home and managed as
outpatients once asymptomatic on an oral regimen of calcium carbonate, usually 2 to 4 g of elemental calcium daily. Vitamin D supplementation may be added to increase absorption of calcium.

5. How is calcium balanced pre- and postoperatively in hyperparathyroidism?


The most common cause of hypercalcemia in the outpatient setting
is primary hyperparathyroidism. Primary hyperparathyroidism
(pHPT) usually results from enlargement of a single adenomatous
gland, resulting in hypercalcemia. Up to 20% of patients, however, may have four gland hyperplasia. Patients with chronic renal
insufficiency may develop secondary hyperparathyroidism; however, these patients usually have low to normal calcium levels due
to Vitamin D deficiency.
Calcium plays an integral role in energy metabolism, hormone secretion, enzyme action, neurotransmitter signaling, and
muscle contraction. The hypercalcemia associated with primary
hyperparathyroidism can result in lethargy, confusion, constipation, pancreatitis, nephrolithiasis, and bone loss. Hyperparathyroid patients with moderate hypercalcemia and normal cardiac
and renal function do not require special preoperative planning.
However, severe hyperparathyroidism requires urgent correction prior to elective surgery. Rarely, acute severe hypercalcemia can precipitate hypercalcemic crisis, manifested as oliguria,
somnolence, and coma. Severe hypercalcemia will also result in

PMPH_CH120.indd 951

ADRENAL
6. What is the preoperative workup of an adrenal incidentaloma?
Adrenal masses are common, with autopsy studies suggesting
up to 10% of the population have adrenal masses.24 Technologic
advances and widespread use of imaging techniques has led to
increased identification of these disorders. All adrenal masses
should be evaluated for malignancy and functionality. Masses
greater than 4 cm in diameter are suspicious for malignancy and
should be removed. Most adrenal incidentalomas are nonfunctional, but 10% to 15% are hormone-secreting. A meta-analysis
of 828 articles on adrenal incidentalomas found that 89.7% were
nonfunctional, 6.4% had evidence of subclinical Cushings syndrome, 3.1% were pheochromocytomas, and 0.6% were primary
aldosterone-secreting tumors. Patients with adrenal incidentalomas should receive a thorough history and physical, plasma-free
metanephrines, cortisol levels, and an electrolyte panel to evaluate for functional hormone secretion.

5/22/2012 6:22:13 PM

952

Surgery: Evidence-Based Practice

Glucocorticoid excess (Cushings syndrome) can result from


endogenous oversecretion or exogenous use of steroids. Cushings
syndrome is commonly caused by a pituitary adenoma (6575%),
or by ectopic adrenocorticotrophic hormone (ACTH) secretion by
tumors. Up to 20% of Cushings syndrome may be due to a functional adrenal adenoma, the most common of all functioning adrenal masses. Glucocorticoid secretion by adrenal adenomas is often
subclinical. Cushings syndrome due to primary adrenal disease is
usually ACTH-independent, and may be due to either a unilateral
adenoma or bilateral macronodular hyperplasia. Patients without
overt clinical evidence of Cushings syndrome still have increased
rates of hypertension, diabetes, and atherosclerosis from continuous excess glucocorticoid secretion.25 Symptomatic Cushings
syndrome due to a unilateral adrenal adenoma is an indication
for resection. However, there are no randomized controlled trials
evaluating surgical resection for subclinical Cushings syndrome.
Low-risk surgical patients and those with documented autonomous glucocorticoid secretion are candidates for adrenalectomy.
Patients who undergo adrenalectomy will require perioperative
glucocorticoid supplementation to prevent adrenal insufficiency.
Functional adrenal masses may also lead to mineralocorticoid excess, associated with hypokalemia, hypernatriemia,
hypertension, muscle weakness, and metabolic alkalosis. Primary hyperaldosteronism (Conns syndrome) is usually due to a
unilateral adrenal adenoma, but up to 40% of patients may have
bilateral adrenal hyperplasia. Patients with Cushings syndrome
may also have symptoms of mineralocorticoid excess because glucocorticoids have mineralocorticoid properties as well. Preoperatively, patients should be treated with the aldosterone antagonist
spironolactone. Ideally, elective surgery may be delayed for 1 to 2
weeks of spironolactone treatment to allow for normalization of
total body potassium. For more emergent surgery, patients may
require large amounts of potassium supplementation as total body
potassium deficit may be profound.
7. What patients are at risk for perioperative adrenal insufficiency and how is it best prevented?
The hypothalamic-pituitary-adrenal (HPA) axis is a critical component of the bodys ability to mount a surgical stress response.
Derangement of the HPA axis can produce a life-threatening
adrenal insufficiency, characterized by fever, hypotension, hyperkalemia, and hyponatriemia. Primary adrenal insufficiency (Addisons disease) may be due to autoimmune adrenal destruction,
tuberculosis, hemorrhage, and infection. In addition, patients
undergoing resection of a cortisol-secreting adenoma may
develop adrenal insufficiency due to suppression of the contralateral gland. However, the most common postoperative cause of
adrenal insufficiency is exogenous steroid use. Exogenous steroid
use leads to suppression of hypothalamic corticotrophin-releasing
hormone (CRH) and ACTH with relative atrophy of the adrenal
cortex. Under times of stress, patients may be unable to respond
with adequate release of glucocorticoids and adrenal insufficiency
ensues.
Patients taking exogenous steroids should be considered at
risk for postoperative adrenal insufficiency. Any method of steroid
intake, oral, topical, and inhaled, may precipitate adrenal suppression.26 Patients taking <5 mg prednisone daily do not experience significant HPA axis suppression.27 Patients taking 20 mg
prednisone daily are at risk for perioperative adrenal suppression.

PMPH_CH120.indd 952

Patients taking these doses for 1 month may have HPA suppression for up to 6 to 12 months after cessation of therapy.18,28-31 Etomidate, a commonly used anesthetic agent, may inhibit steroid
synthesis and should be avoided in patients considered at risk for
perioperative adrenal insufficiency.32
Clear guidelines on which patients should receive perioperative stress-dose steroids, and at what dosage, have not been
determined. Primate studies suggest that supraphysiologic steroid
administration intraoperatively can prevent Addisonian crisis
with minimal added risk.33 The decision to administer perioperative glucocorticoids should be based on assessment of clinical
risk, extent of surgery, acuity of surgery, and presence of findings consistent with adrenal insufficiency.20,34 Time permitting, a
short ACTH stimulation test may be obtained to evaluate adrenocortical function and direct glucocorticoid supplementation to
patients with poor adrenal response.35 Patients who receive stressdose steroids are typically continued on a short taper for 24 to
48 hours. Although chronic steroid use has been linked to poor
wound healing and increased rate of infection, acute supratherapeutic perioperative glucocorticoid administration has not been
proven to adversely affect wound healing or infection rates in the
postoperative patient.
8. What is the perioperative management of pheochromocytoma?
Pheochromocytomas are rare catecholamine secreting neuroendocrine tumors. Pheochromocytomas may occur sporadically or
as part of a Multiple Endocrine Neoplasia syndrome (MENIIA
& B). Symptoms include hypertension, palpitations, flushing,
sweating, and headache. Chronic, severe hypertension can lead
to hypertrophic cardiomyopathy, which may be reversible after
removal of the tumor.36 If pheochromocytoma is suspected, the
most sensitive test for diagnosis is plasma-free metanephrines. If
there is biochemical evidence of pheochromocytoma, radiographic
imaging studies (i.e., MRI or nuclear imaging) are obtained for
localization.
Surgery can induce catecholamine crisis in patients with
pheochromocytoma. Rarely, intraoperative catecholamine crisis
is the initial presentation of a pheochromocytoma. Patients are
optimized with a combination of -adrenergic and -adrenergic
blockade for a minimum of 1 to 2 weeks preoperatively. Initiation
of -adrenergic blockade is associated with a reduction in perioperative mortality from 40% to 60% down to 0% to 6%.37-39 Both
nonselective (phenoxybenzamine) and selective 1-adrenergic
agents (prazosin) may be used. Patients with pheochromocytoma
are intravascularly volume depleted from chronic vasoconstriction. Preoperative -blockade for 1 to 2 weeks preoperatively
allows for correction of this volume depletion, often followed
by a decrease in hematocrit. Adequate -blockade parameters
include (1) maintaining blood pressure <160/90 mm Hg, (2) presence of orthostatic hypotension, (3) no ST-T waves changes within
1 week of surgery, and (4) 1 premature ventricular contraction
every 5 minutes.39 For patients with persistent tachycardia or
hypertension, a -blocker may be added. It is important to initiate
-blockade before -blockers; failure to do so may result in unopposed -receptor stimulation by circulating catecholamines.
Intraoperatively, the physician must be prepared to deal
with either hypertensive or hypotensive crisis. Arterial line
placement for continuous blood pressure monitoring as well

5/22/2012 6:22:13 PM

Perioperative Endocrine Dysfunction

as central line placement is advisable, in case administration


of vasoactive agents is necessary. Agents that directly or indirectly increase catecholamine levels, such as ketamine, should
be avoided. Morphine, meperidine, and droperidol have been
associated with hypertension and may be a trigger of pheochromocytoma crisis.40,41 Postoperatively, patients may become

953

profoundly hypotensive and require large amounts of intravenous fluids. Th is is attributed to continued adrenergic blockade for 24 to 48 hours postoperatively. However, this should not
be mistaken for postoperative hemorrhage, which should be
excluded immediately with a low threshold with return to the
operating room.

Clinical Question Summary


Question

Answer

References

1 What are the


perioperative concerns
for diabetic patients?

Primary concern should focus on cardiopulmonary risk assessment and modification.


Consider other comorbidities, such as hypertension, obesity, and renal disease.
Assess glycemic control and complications of diabetes.

1-9

2 How is glycemic control


best optimized in
the diabetic patient
undergoing surgery?

Patients on oral hypoglycemic agents should discontinue treatment on the morning of


surgery. Insulin-dependent patients should replace intermediate-acting insulin with
long-acting insulin for 1 to 2 days before surgery. IV dextrose and hydration should be
given while patient is NPO. Postoperative glycemic control is dependent on dietary
status.

10-12

3 What is the
perioperative
management of
hyperthyroidism?

A thyroid function panel should be used for evaluation. Surgery should be deferred until
patient is euthyroid. Patients taking antithyroid medication should take medication
on day of surgery. Preoperative imaging should be used to plan the approach to
intubation. Maintain high clinical suspicion of thyroid storm.

13-16

4 How should hypothyroid


patients be optimized
for surgery?

Patients with mild to moderate hypothyroidism may undergo surgery without significant
additional risk, though more sensitive to sedatives and analgesics. Patients with severe
hypothyroidism, manifested by pericardial effusion or heart failure, should be given
levothyroxine before and after surgery. Myxedema coma is a rare postoperative
complication of severe hypothyroidism.

17-21

5 How is calcium
balanced pre- and
postoperatively in
hyperparathyroidism?

Most common cause of hypercalcemia in the outpatient setting is primary


hyperparathyroidism. Up to 20% of patients may have 4-gland hyperplasia. Incidence
of significant hypocalcemia after total parathyroidectomy is from 13 to 20%.
Symptomatic patients may require IV calcium gluconate, but most can be managed as
outpatients. Vitamin D supplementation may be added.

22, 23

6 What is the
perioperative
workup of an adrenal
incidentaloma?

Most adrenal incidentalomas are nonfunctional, but 1015% are hormone-secreting.


Patients with adrenal incidentalomas should be evaluated by a thorough history and
physical examination, plasma-free metanephrines, cortisol levels, and electrolyte
panel. Adrenalectomy patients require perioperative glucocorticoid supplementation.
Preoperatively, Conns or Cushings syndrome patients should be given spironolactone
for 1 to 2 weeks to normalize potassium levels. For emergent surgery, patients may
require large amounts of potassium supplementation.

24, 25

7 What patients are at


risk for perioperative
adrenal insufficiency
and how is it best
prevented?

Patients taking exogenous steroids should be considered at risk for postoperative


adrenal insufficiency. Patients taking less than 5mg of prednisone per day do not
experience significant HPA axis suppression. Clear guidelines on which patients
should receive perioperative stress-dose steroids and the dosage have not been
determined. Administration of perioperative glucocorticoids should be based on
clinical risk, extent of surgery, acuity of surgery, and presence of findings consistent
with adrenal insufficiency.

26-35

8 What is the
perioperative
management of
pehochromocytoma?

Suspected pheochromocytoma should be confirmed by plasma-free metanephrines. If


present, radiographic imaging studies should be used for localization. Surgery can
induce catecholamine crisis and patients can be optimized with - and -adrenergic
blockade for 1 to 2 weeks preoperatively. Nonselective and selective 1-adrenergic
agents may both be used. A -blocker may be added for patients with persistent
tachycardia or hypertension. Intraoperatively, the physician must be prepared to
deal with either hyper- or hypotensive crises. Agents that increase catecholamine
levels should be avoided. Postoperatively, patients may become hypotensive and will
require IV fluids. Do not mistake this for postoperative hemorrhage, which should be
excluded immediately.

36-41

PMPH_CH120.indd 953

5/22/2012 6:22:13 PM

954

Surgery: Evidence-Based Practice

REFERENCES
1. Glister BC, Vigersky RA. Perioperative management of type 1
diabetes mellitus. Endocrinol Metab Clin North Am. 2003;32(2):
411-436.
2. Clement S, Braithwaite SS, Magee MF, Michelle F, Ahmann A,
Smith E, et al. Management of diabetes and hyperglycemia in
hospitals. Diabetes Care. 2004;27(2):553-591.
3. Malone DL, Genuit T, Tracy JK, Cannon C, Napolitano LM.
Surgical site infections: reanalysis of risk factors. J Surg Res.
2002;103(1):89-95.
4. Thomas L, Marcantonio SM, Edward R, Mangione CM, Thomas EJ,
Polanczyk CA, et al. Derivation and prospective validation of a
simple index for prediction of cardiac risk of major noncardiac
surgery. Circulation. 1999;100(10):1043-1049.
5. Dronge AS, Perkal MF, Kancir S, Concato J, Aslan M, Rosenthal RA.
Long-term glycemic control and postoperative infectious complications. Arch Surg. 2006;141(4):375-380; discussion 380.
6. Trick WE, Scheckler WE, Tokars JI, Jones KC, Reppen ML, Smith EM,
et al. Modifiable risk factors associated with deep sternal site
infection after coronary artery bypass graft ing. J Thorac Cardiovasc Surg. 2000;119(1):108-114.
7. Moghissi ES, Korytkowski MT, DiDardo M, Einhorn D, Hellman R,
Hirsch IB, et al. American Association of Clinical Endocrinologists and American Diabetes Association consensus statement on inpatient glycemic control. Diabetes Care. 2009;32(6):
1119-1131.
8. Van den Bergh G, Wilmer A, Milants I, Wouters PJ, Bouckaert B,
Bruyninckx F, et al. Intensive insulin therapy in mixed medical/surgical intensive care units: benefit versus harm. Diabetes.
2006;55(11):3151-3159.
9. Finney SJ, Zekveld C, Elia A, Evans TW. Glucose control and
mortality in critically ill patients. JAMA. 2003;290(15):2041-2047.
10. Watts NB, Gebhart SS, Clark RV, Phillips LS. Postoperative
management of diabetes mellitus: steady-state glucose control
with bedside algorithm for insulin adjustment. Diabetes Care.
1987;10(6):722-728.
11. Pezzarossa A, Taddei F, Cimicchi MC, et al. Perioperative management of diabetic subjects. Subcutaneous versus intravenous
insulin administration during glucose-potassium infusion. Diabetes Care. 1988;11(1):52-58.
12. Hoogwerf BJ. Perioperative management of diabetes mellitus: striving for metabolic balance. Cleve Clin J Med. 1992;59(5):447-449.
13. Spell NO, 3rd, Stopping and restarting medications in the perioperative period. Med Clin North Am. 2001;85(5):1117-1128.
14. Abuid J, Larsen PR. Triiodothyronine and thyroxine in hyperthyroidism. Comparison of the acute changes during therapy
with antithyroid agents. J Clin Invest. 1974;54(1):201-208.
15. Croxson MS, Hall TD, Nicoloff JT. Combination drug therapy
for treatment of hyperthyroid Graves disease. J Clin Endocrinol
Metab. 1977;45(4):623-630.
16. Larsen PR. Salicylate-induced increases in free triiodothyronine
in human serum. Evidence of inhibition of triiodothyronine
binding to thyroxine-binding globulin and thyroxine-binding
prealbumin. J Clin Invest. 1972;51(5):1125-1134.
17. Ladenson PW, Levin AA, Ridgway EC, Daniels GH. Complications of surgery in hypothyroid patients. Am J Med. 1984;77(2):
261-266.
18. Schiff RL, Welsh GA. Perioperative evaluation and management
of the patient with endocrine dysfunction. Med Clin North Am.
2003;87(1):175-192.
19. Weinberg AD, Brennan MD, Gorman CA, Marsh HM, OFallon WM.
Outcome of anesthesia and surgery in hypothyroid patients.
Arch Intern Med. 1983;143(5):893-897.

PMPH_CH120.indd 954

20. Kohl BA, Schwartz S. How to manage perioperative endocrine


insufficiency. Anesthesiol Clin. 2010;28(1):139-155.
21. Connery LE, Coursin DB. Assessment and therapy of selected
endocrine disorders. Anesthesiol Clin North Am. 2004;22(1):
93-123.
22. Brasier AR, Nussbaum SR. Hungry bone syndrome: clinical and
biochemical predictors of its occurrence after parathyroid surgery. Am J Med. 1988;84(4):654-660.
23. Jofr R, Lpez Gmez JM, Menrguez J, et al. Parathyroidectomy: whom and when? Kidney Int Suppl. 2003;85:S97-S100.
24. Hedeland H, Ostberg G, Hokfelt B. On the prevalence of
adrenocortical adenomas in an autopsy material in relation
to hypertension and diabetes. Acta Med Scand. 1968;184(3):
211-214.
25. Emral R, Uysal AR, Asik M, Gullu S, Corapcioglu D, Tonyukuk V,
Erdogan G. Prevalence of subclinical Cushings syndrome in 70
patients with adrenal incidentaloma: clinical, biochemical and
surgical outcomes. Endocr J. 2003; 50(4):399-408.
26. Axelrod L. Perioperative management of patients treated with
glucocorticoids. Endocrinol Metab Clin North Am. 2003;32(2):
367-383.
27. Jabbour SA. Steroids and the surgical patient. Med Clin North
Am. 2001;85(5):1311-1317.
28. Nicholson G, Burrin JM,Hall GM. Peri-operative steroid supplementation. Anaesthesia. 1998;53(11):1091-1104.
29. Henzen C, Suter A, Lerch E, Urbinelli R, Schorno XH, Briner VA.
Suppression and recovery of adrenal response after short-term,
high-dose glucocorticoid treatment. Lancet. 2000;355(9203):
542-545.
30. Hopkins RL, Leinung MC. Exogenous Cushings syndrome and
glucocorticoid withdrawal. Endocrinol Metab Clin North Am.
2005;34(2):371-384, ix.
31. Graber AL, Ney RL, Nicholson WE, Island DP, Liddle GW.
Natural history of pituitary-adrenal recovery following longterm suppression with corticosteroids. J Clin Endocrinol Metab.
1965;25:11-16.
32. Wagner RL, White PF, Kan PB, Rosenthal MH, Feldman D. Inhibition of adrenal steroidogenesis by the anesthetic etomidate.
N Engl J Med. 1984;310(22):1415-1421.
33. Udelsman R, Ramp J, Gallucci WT, et al. Adaptation during surgical stress. A reevaluation of the role of glucocorticoids. J Clin
Invest. 1986;77(4):1377-1381.
34. Kohl BA, Schwartz S. Surgery in the patient with endocrine dysfunction. Anesthesiol Clin. 2009;27(4):687-703.
35. Kehlet H, Binder C. Value of an ACTH test in assessing hypothalamic-pituitary-adrenocortical function in glucocorticoidtreated patients. Br Med J. 1973;2(5859):147-149.
36. Liao WB, Liu CF, Chiang CW, Kung CT, Lee CW. Cardiovascular manifestations of pheochromocytoma. Am J Emerg Med.
2000;18(5):622-625.
37. Witteles RM, Kaplan EL, Roizen MF. Safe and cost-effective
preoperative preparation of patients with pheochromocytoma.
Anesth Analg. 2000;91(2):302-304.
38. Lucon AM, Pereira MA, Mendona BB, et al. Pheochromocytoma: study of 50 cases. J Urol. 1997;157(4):1208-1212.
39. Roizen MF, Hunt TK, Beaupre PN, et al. The effect of alphaadrenergic blockade on cardiac performance and tissue oxygen
delivery during excision of pheochromocytoma. Surgery. 1983;
94(6):941-945.
40. Kinney MA, Narr BJ, Warner MA. Perioperative management
of pheochromocytoma. J Cardiothorac Vasc Anesth. 2002;16(3):
359-369.
41. Jovenich JJ. Anesthesia in adrenal surgery. Urol Clin North Am.
1989;16(3):583-587.

5/22/2012 6:22:13 PM

INDEX
Note: Page numbers followed by f and t denote figures and tables, respectively.
Abdominal aortic aneurysm (AAA), 661
endovascular repair
choice of devices, 663
endoleaks management, 663
versus open repair, 662
preservation of collateral flow in, 664
for ruptured aneurysms, 664
medical therapies for, 661
open repair
eligibility for, 663
preservation of collateral flow in, 664
transabdominal versus retroperitoneal, 663664
retroperitoneal approaches for, 663664
risk factors for, 661
rupture and recommended repair, risk of, 662
screening for, 661
transabdominal approaches for, 663664
UK small aneurysm trial, 662
Abdominal ultrasound, for appendicitis, 277
Abdominal wall defects, repair of, 366367
Abdominoperineal resection (APR), for squamous cell
carcinoma, 254
Ablation of liver tumors
operative, 324
percutaneous, 324
radiofrequency, 323324
Abnormal surgical and postoperative bleeding, 909915
coagulation profi le, nutritional supplements effect on,
911912
heparin prophylaxis, timing of, 910911
incidence and causes of, 909910
management of
fresh frozen plasma, 913
platelets, administration of, 913
prothrombin complex concentrates, 912
recombinant activated factor VII, 912
tranexamic acid, 912913
surgical patients, preoperative coagulation workup
for, 910, 911t
Abscess(es)
amebic, 329331
intraabdominal, 174175
pancreatic, 438, 439
perirectal, 295
pyogenic, 329331
splenic, 488489
ACE inhibitors, for airway obstruction, 927
Acetaminophen
for hyperpyrexia, 950
for thyroid storm, 951t

Achalasia. See also Diff use esophageal spasm (DES)


diagnosis
based on clinical presentation of, 37
nonsurgical treatment options for, 38
surgery for, 38
Acid suppression therapy, following repair of esophageal
atresia, 100
Acinar cell carcinoma (ACC). See also Carcinoma
distinguished from pancreatic ductal
adenocarcinoma, 471
pancreatic, 470471
prognosis of, 471
Activated partial thromboplastin time (aPTT), 909, 910
Acute arterial ischemia from embolus
fasciotomies with lower extremity revascularization
for, 725726
thrombolysis for, 722724
Acute cholangitis due to common bile duct stones, biliary
decompression, 388389
Acute cholecystitis, 380384
antibiotics for, 383
diagnosis of
clinical criteria for, 380381
imaging study for, 381
epidemiology of, 380
history of, 380
laparoscopic or open cholecystectomy for, 381382
nonoperative management of, 382383
pathophysiology of, 380381
percutaneous cholecystostomy for, in high-risk surgical
patients, 383
surgical intervention for, timing of, 382
Acute colonic pseudo-obstruction (ACPO), 222
complications of, 224
management of, 224
prevalence of, 224
treatment for, 224225
Acute Dialysis Quality Initiative Group, 942
Acute kidney injury (AKI). See Acute renal dysfunction
Acute kidney injury network (AKIN), 947
Acute lung injury (ALI)
respiratory support for, 929930
treatment for, 928
Acute mesenteric ischemia (AMI)
anatomy of, 756
epidemiology of, 756
evaluation pathway for, 756757
history of, 756
physiology of, 756
risk factors of, 756
treatment for, 757

955

PMPH_INDEX.indd 955

5/22/2012 6:24:24 PM

956

Index

Acute myocardial infarction (MI)


diagnosis of, 901902
etiology of, 901
management of, 902904
non-ST-elevation, 904
pathophysiology of, 901
ST-elevation, 902904
Acute pancreatitis (AP), 421427
endoscopic retrograde cholangiopancreatography for,
421422
gastric feeding versus jejunal feeding for, 423424
prophylatic antibiotics for, 422423
Acute Physiology and Chronic Health Evaluation (APACHE II),
879, 882
Acute renal dysfunction, 942945, 947948
diagnosis of, 943
management of, 943944
pharmacologic treatment for, 944
RIFLE criteria of, 942, 943, 947
risk factors of, 942943
Acute respiratory distress syndrome (ARDS), 846
Adalimumab, for Crohns disease, 216
Adaptation to the Intensive Care Environment (ATICE), 918
Adenocarcinoma. See also Carcinoma
diagnosis and evaluation of, 255
of esophagus, 92
follow-up, 255
gastric. See Gastric adenocarcinoma
pancreatic. See Pancreatic adenocarcinoma
prognosis of, 255
of small bowel, 186
adjuvant therapy for, 189
incidence, histology and location of, 187
prognostic factors for survival for, 189
recurrence, treatment for, 190
treatment of, 188
treatment for, 255
Adenomatosis polyposis syndromes, 260261
Adenosquamous carcinoma, pancreatic, 470
Adjuvant biologic therapy, for breast cancer, 606607
Adjuvant endocrine therapy, for breast cancer, 605606
Adjuvant! Online computer program, 608
Adjuvant radiotherapy, for malignant melanoma, 803
Adjuvant systemic therapy, for malignant melanoma, 803
Adrenalectomy
for adrenal incidentaloma, 537538
laparoscopic
for adrenal tumors, 539
for pheochromocytoma, 539
for metastases to adrenal, 540
Adrenal incidentaloma, 537542, 951953
evaluation of
cost-effective approach to, 539
resection, indications for, 537539
perioperative management, risk factors of, 951
preoperative management of, 951
Adrenocortical carcinoma (ACC), 537542. See also Carcinoma
adjuvant therapy for, 539
operative approach to, 539
persistent/recurrent, treatment for, 540
Adrenocorticotrophic hormone (ACTH), 952

PMPH_INDEX.indd 956

Adult respiratory distress syndrome (ARDS), 921


respiratory support for, 929930
treatment for, 928
Adults
short bowel syndrome, autologous intestinal reconstruction
surgery for, 198199
solitary neck mass in, 814
solitary neck mass in, 815
umbilical hernia in, 526
-2 Agonists, for preoperative risk factors, 863, 861t
Airway obstruction, 927
Albumin, 853
for sepsis, 844
Albuterol, for hyperkalemia, 841842, 853
Allografts, peripheral nerve injury with, 820
Amebic abscess. See also Abscess(es)
clinical features of, 330
diagnosis of, 330331
epidemiologic characteristics of, 329
incidence of, 329
pathiphysiology of, 330
treatment for, 331
-Aminobutyric acid (GABA), 358
Aminoglycoside, for surgical site infection, 894
Amiodarone, for hypothyroidism, 950
Amoxicillin, and pseudomembranous colitis, 240
Ampicillin, for surgical site infection, 892
Anal canal, tumors of, 253256
Anal dilatation with surgical hemorrhoidectomy, for
hemorrhoids, 287288
Anal fissures, 294297
lateral internal sphincterotomy for, 294
nonoperative therapies for, 294
Anal fistulas, setons for, 295
Anal intraepithelial neoplasia (AIN)
diagnosis and evaluation of, 255
follow-up, 255
prognosis of, 255
treatment for, 255
Anal manometry, for fecal incontinence, 300
Anal margin, tumors of, 256257
Anal region
cancers of, 253
tumors
analogy of, 253
histology of, 253
Anal strictures, treatment for, 294295
Anastomosis
complications in mechanical bowel
preparation, 267268
effect on CD outcome and recurrence, 177
primary, 223
for complicated acute diverticulitis, 207208
during surgery of ischemic colitis, 234
for radiation enteritis, 219
rectal, 223
Anastomotic leak
fluid resuscitation volume affecting, 846847
role of stents in, 8687
Anastomotic stenosis, for CBD stones, 412413
Anastrolze, for breast cancer, 592, 593

5/22/2012 6:24:25 PM

Index

Anesthesia
cutaneous, for diabetic peripheral neuropathy, 763
local versus general, 515516
AneuRx, 663
Angioedema, 927
Angiography
for lower gastrointestinal bleeding, 314315
for UGI bleeding, 156157
Angioplasty
for diabetic foot revascularization, 764, 768
percutaneous
for AVF maturation, 743
for renal artery stenosis, 750
Angiotensin converting enzyme inhibitor (ACE-I), for
ST-elevation MI, 903904
Angiotensin receptor blockers (ARBs), for
ST-elevation MI, 904
Anklebrachial pressure index (ABI), 701
Anorectal melanoma
diagnosis and evaluation of, 255
follow-up, 256
prognosis of, 255256
treatment for, 256
Anterior repair with mesh
for lumbar hernias, 527
for spigelian hernias, 527
Antibiotic prophylaxis, for surgical site infection, 892893
Antibiotics
for acute cholecystitis, 383
for appendicitis, 281
preoperative dose of, for infectious complications, 279
-associated colitis. See Pseudomembranous colitis
for bacteremia, 878
for diabetic foot osteomyelitis, 764
for esophageal caustic injury, 71
exposure, and pseudomembranous colitis, 240
for hepatic encephalopathy, 360
for ischemic colitis, 233234
for necrotizing soft tissue infections, 829
for perirectal abscesses, 295
for postsplenectomy sepsis, 501502
prophylactic
for acute pancreatitis, 422423
for pancreatitis, 433
for surgical site infection
intravenous, 892
oral, 894
prophylactic, 893894
for umbilical/epigastric hernias, 527
Anticoagulants
for chronic mesenteric ischemia, 757
for non-ST-elevation MI, 904
for ST-elevation MI, 903
Anticoagulation effect, on graft patency, 738
Antimicrobial therapy
for bacteremia, 879
broad-spectrum, 833
Antiplatelet agents, for chronic mesenteric ischemia, 757
Antiplatelet therapy
for non-ST-elevation MI, 904
for ST-elevation MI, 903

PMPH_INDEX.indd 957

957

Antireflux surgery
for children with EA/TEF, 100101
for gastroesophageal reflux disease, 61
Antithyroid drugs (ATDs), for Graves disease, 559
Aortic dissection, 669673
chronic, 673
clinical presentation of, 669670
diagnosis of, 670
epidemiology of, 669670
management of, 670672
endovascular therapy, 671672
medical therapy, 671
open surgical therapy, 671
outcome of, 672673
Aortofemoral grafting
for aortoiliac reconstruction approach, 694
distal anastomosis, 694
outflow vasculature, 694695
proximal anastomosis, 694
Aortoiliac occlusive disease (AOID), 691695
adjunct medications and follow-up protocol support for, 695
diagnosis of, 692695
endovascular repair, lesions favorable for, 692
epidemiology of, 691692
history of, 691
open reconstruction trump endovascular repair for, 693694
primary stenting versus selective stenting for, 693
technical factors for reconstruction of, 694695
APACHE II scoring, for predicting mortality in ECF, 192
APC gene, 261
Appendectomy, for appendicitis
interval, 280281
laparoscopic, 279280
open, 279280
Appendicitis, 275282
appendectomy
interval, 280281
laparoscopic versus open, 279280
diagnosis of, 276277
imaging modalities for, 277
pain medicine and, 276277
preoperative dose of antibiotics, for infectious
complications, 279
signs and symptoms of, 275276
Aromatase inhibitors (AI), 606
Arterial pseudoaneurysms (PAs), 677681
diagnosis of, 678
infected femoral, management of, 680
prevention of, 679680
risk factors for, 677678
treatment of, 678679
Arterial pulse contour (APCO) analysis, for perioperative cardiac
monitoring, 870, 871
Arteriography, for chronic mesenteric ischemia, 757
Arteriovenous fistulas (AVFs), 677681
diagnosis of, 678
dialysis access creation, anatomic location for, 743
maturation of, 743
postoperative surveillance of, 743
prevention of, 679680
risk factors of, 677678
treatment of, 678679

5/22/2012 6:24:25 PM

958

Index

Arteriovenous grafts (AVGs)


dialysis access creation, anatomic location for, 743
maturation of, 743
postoperative surveillance of, 743
Artificial sphincters, for fecal incontinence, 300
Aspirin
for carotid occlusive disease, 684686
and coagulopathy, 910
plus dipyridamole, 686
effect on graft patency, 737738
for ST-elevation MI, 903
Asymptomatic hernias, repair of, 513514
Atelectasis, 927
Atenolol, for ST-elevation MI, 903
Atherosclerotic renovascular disease, 749752
Atrial fibrillation patients, anticoagulation for embolus
prevention in, 724725
Atrial natriuretic peptide (ANP), 944
Attention Screening Examinations (ACE), 921
Attrition bias, 10. See also Bias
Autoimmune hemolytic anemia (AIHA), 484
Autologous intestinal reconstruction surgery, for short bowel
syndrome, 198199
Autologous stem cell therapy, for thromboangiitis obliterans, 732
Autologous tissue flap reconstruction, 624
Axillary lymph node dissection (ALND), 608
accuracy of, 598599
completion, for in patients with SLN metastasis, 599
for early-stage breast cancer, 588590
for locally advanced breast cancer, 591592
Axillary lymph node sampling, for male breast cancer, 619
Azathioprine, for Crohns disease, 216
Bacillus Calmette-Guerin (BCG), 803
Bacteremia, 876880, 882883
comorbidities of, 877
diagnosis of, 877878
factors associated with mortality, 879
incidence of, 876877
nosocomial, 879
pathogens associated with, 878
predispositions of, 877
treatment for, 878879
Balloon expulsion test, for pelvic floor dysfunction, 301
Balloon tamponade of bleeding esophageal varices, using
SengstakenBlakemore tube, 349
Banding ligation, endoscopic, for acute variceal bleeding, 156
BannayanRileyRuvalcaba syndrome, 261
Bariatric surgery. See also Surgery
centers, accreditation of
and surgery outcomes, relationship between, 139
common procedures, 134
factors to consider prior, 133134
indications for, 132133
postoperative care following, 137138
procedures, comparison of, 137
Basal acid output (BAO) test, for ZollingerEllison
syndrome, 122
Basal cell carcinoma (BCC). See also Carcinoma
diagnosis and evaluation of, 256
follow-up, 256
prognosis of, 256
treatment for, 256

PMPH_INDEX.indd 958

Benign biliary strictures, 394397


differential diagnosis of, 396
predispose to, 394395
surgery for, 396397
Benign breast diseases, surgical management of, 581
Benign esophageal stricture, role of stents in, 8687
Benign papillary lesions, excision of, 582
B. holmesii, 500
Bias, 45
attrition, 10
detection, 1011
performance, 10
publication, 67
selection, 10
Bilateral hernias, optimal approach for, 515
Bile duct tumors, 403407
adjuvant therapies for, 406
diagnosis of, 404
resection of, 405406
staging of, 404405
symptoms of, 403404
unresectable, palliative options for, 406407
Biliary drainage prior to surgical resection, in patient with
jaundice from pancreas cancer, 413
Biliary sphincteroplasty, for Sphincter of Oddi dysfunction,
424425
Biliopancreatic diversion (BPD), 136
Biofeedback
for fecal incontinence, 299300
for pelvic floor dysfunction, 301
Biologic grafts, peripheral nerve injury with, 820
Biopsy(ies)
for esophageal cancer, 92
fine-needle aspiration. See Fine-needle aspiration biopsy
incisional, 808
percutaneous breast, 580
sentinel lymph node, 597601, 619, 801802
for soft tissue sarcomas, 808
stereotactic, 580
ultrasound-guided core needle, 580, 618
Bipolar diathermy hemorrhoids (PPH), 288
Bispectral index (BIS) index, 358
Bleeding peptic ulcer
indication for operation for, 111
initial approach for management of, 110
pharmacotherapy for, 111
role of endoscopy in, 110111
Blinding, 45, 10
double, 5
-Blockers
for abdominal aortic aneurysm management, 661
for aortic dissection, 670
for hyperkalemia, 841, 853
for hyperthyroidism, 950
for non-ST-elevation MI, 904
for pheochromocytoma, 952
for portal hypertension, 346
for preoperative risk factors, 863, 861t, 868
for ST-elevation MI, 903
for tachycardia, 950
for thyroid storm, 951t
for variceal bleeding, 155

5/22/2012 6:24:25 PM

Index

Blunt splenic injury (BSI)


conservatively treated patients, follow-up
observation for, 493
physical activity, resuming, 493494
Body mass index (BMI), 854
Boerhaaves syndrome, 29
Botulinum toxin (Botox)
for achalasiapasm, 38
for anal fissures, 294
chemodenervation, for neurogenic thoracic outlet
syndrome, 713
for diff use esophageal spasm, 38
for esophageal pulsion diverticula, 50
Bowel anastamosis, stapled or handsewn techniques for,
difference between, 169
Bowel bypass surgery, for radiation enteritis, 219
Bowel wall
lesions extrinsic to, 222
lesions intrinsic to, 222
Bowens disease
diagnosis and evaluation of, 256
follow-up, 256
prognosis of, 256
treatment for, 256
Brachial plexus, imaging assessment of, 711
Bran failure, acute, 917
Breast cancer, 587593. See also Cancer
adjuvant biologic therapies in, 606607
adjuvant chemotherapy in, 604605
adjuvant endocrine therapy in, 605606
ALND for. See Axillary lymph node dissection (ALND)
axillary lymph node dissection for, 588590
early-stage
systemic treatment strategies for, 604608
liver resection for, 325
locally advanced. See Locally advanced breast cancer
male. See Male breast cancer
neoadjuvant therapy for, 592593, 607608
nipple- and areola-SSM in, 593
radiation therapy for, 590
screening for, 577580
SLN biopsy for, 597601
stage IV, 593
Breast implant, 624
Breast reconstruction following mastectomy, 622625
age limit on, 624
breast implant/autologous tissue flap for, 624
free microvascular TRAM flap versus pedicle TRAM flap,
624625
neoadjuvant/adjuvant therapies and, 623
reconstruction after radiation therapy, 624
smoking-related complications, 622
surveillance for breast cancer recurrence, 623
technique selection, importance of patient anatomy in,
623624
Breast self-examination (BSE), 577578
Breast surgery, for surgical site infection, 893894. See also
Surgery
Broad-spectrum antimicrobial therapy, for necrotizing fasciitis, 833
Bronchoscopy/esophagoscopy, for EA/TEF, 98
Bronchoscopy, for metabolic squamous cell carcinoma of
unknown primary, 816

PMPH_INDEX.indd 959

959

Budd-Chiari syndrome, 361


Buergers Disease. See Thromboangiitis obliterans (TAO)
Calciphylaxis, 569
Calcitonin, for hypercalcemia, 848
Calcium channel blockers
for anal fissures, 294
for Raynauds syndrome, 730731
Cancer. See also Carcinoma
in anal region, 253
breast. See Breast cancer
esophageal. See Esophageal cancer
colon, 221
colorectal, 245248
diff use gastric, 146
esophageal cancer, 9194
gastric. See Gastric cancers
non-small cell lung, 632
ovarian, 325
pancreatic, 324, 413
rectal. See Rectal cancer
renal, 325
stomach, 324325
Capsaicin, for disbetic foot, 763
Capsule endoscopy, for small bowel tumors, 188
Captopril, for ST-elevation MI, 904
Carbamazepine, for disbetic foot, 763
Carbohydrate antigen 19-9 (CA19-9), 471
Carcinoid of small bowel, 186
adjuvant therapy for, 189
incidence, histology and location of, 187
prognostic factors for survival for, 189
recurrence, treatment for, 190
treatment of, 188
Carcinoma. See also Cancer
acinar cell, 470471
adenocarcinoma. See also Adenocarcinoma
adrenocortical, 537542
basal cell, 256
ductal carcinoma in situ, 600601, 613614
hepatocellular, 323326
hilar cholangiocarcinoma, 413
in situ carcinoma of the breast, 612615
invasive ductal, 588
invasive lobular carcinoma of the breast, 588
lobular carcinoma in situ, 614615
pancreatic adenosquamous, 470
squamous cell. See Squamous cell carcinoma
verrucous, 257
Cardiac disease, acute renal dysfunction and, 943
Cardiogenic shock, 904
Cardiomyotomy
for achalasia, 4142
postsurgical management for, 39
Carotid aneurysmal disease, 688
Carotid endarterectomy, 687688
Carotid occlusive disease
diagnosis of, 685686
indications for, 686
medical treatment for, 686
surgery for, 686687

5/22/2012 6:24:25 PM

960

Index

Carotid stenosis
asymptomatic, stroke risk for, 685
symptomatic, stroke risk for, 684685
Carotid stenting, 687688
Case-control studies, 34
Catecholamines, 545, 546
Catechol-O-methyltransferase, 545, 546
Catheter-directed thrombolysis, for late graft occlusion, 736737
C. canimorsus, 500
Cecal vovulus, operative management of, 224
Cecostomy, for acute colonic pseudo-obstruction, 225
Cefadroxil, for lymphedema, 793
Cefazolin
for pneumothorax, 651
for surgical site infection, 893
Ceftazidime, for necrotizing soft tissue infections, 829
Central line associated bloodstream infection (CLABSI), 884
prevention
behavioral interventions for, 885886
education, role, 885
Central venous catheter (CVC) infections, prevention of,
884887
anti-infective catheters, utilization of, 886887
behavioral interventions for, 885886
catheter colonization, risk factors associated with, 886
chlorhexidine, role of, 885886, 883f
femoral insertion site, use of, 884
Central venous pressure (CVP), 878
monitoring, 869870
Cephalexin, for lymphedema, 793
Cephalosporin
for pneumothorax, 651
and pseudomembranous colitis, 240
for surgical site infection, 893
Certolizumab pegol, for Crohns disease, 216
Charlson Index of Comorbidity, 937
Chemoprevention, for lobular carcinoma in situ, 615
Chemoprophylaxis, for lobular neoplasia, 582
Chemoradiotherapy (CRT)
for adenocarcinoma, 255
for anorectal melanoma, 255
versus chemotherapy, preoperative
for esophageal cancer, 93
for pancreatic adenocarcinoma, 460462
for squamous cell carcinoma, 254255
of anal region, 256
of esophagus, 91
Chemotherapy
adjuvant, for biliary tract tumors, 406
for breast cancer, 604605
for male breast cancer, 619
neoadjuvant, 809
for gastric cancers, 148
for pancreatic adenocarcinoma, 459462
postoperative, 809
for gastric cancers, 149
for recurrent sarcomas, 810
for small bowel cancer, 189
for soft tissue sarcomas, 809
for surgically resectable esophageal tumor outcomes, 7780

PMPH_INDEX.indd 960

Chest wall tumors, 636641


clinical presentation of, 636
diagnosis of, 637
imaging of, 637
peri- and postoperative care, 640
reconstruction methods, 639640
resection
bony thorax reconstruction after, 639
complications of, 640
considerations prior to, 638639
follow-up postresection, 640641
postresection pleural space, management of, 640
technical considerations of, 639
soft-tissue sarcomas, 637
treatment for, 638
types of, 637638
Chest x-ray (CXR)
for esophageal pulsion diverticula, 49
after open/percutaneous tracheostomy, 644
Chewing gum and POI duration, 166
Chimeric monoclonal antibody (cA2), for Crohns disease, 216
Chlorhexidine, for central venous catheter infections, 885886
Cholecystectomy
in at-risk populations, 376377
for common bile duct stones with/without gallstones,
389390
cost-effectiveness of, 376
for gallstones, 376
laparoscopic/open
for acute cholecystitis, 381382
for CBD stones, 411
percutaneous, for acute cholecystitis, 383
prophylactic
in diabetes mellitus, 376
in transplant patients, 376
for surgical site infection, 894, 900
T-tube drainage after, insertion of, for common bile duct
stones, 388
Cholestyramine
for hypothyroidism, 950
for pseudomembranous colitis, 240
Chondroma, 638
Chondrosarcoma, 638
Chronic lymphocytic leukemia (CLL), 482483
Chronic mesenteric ischemia (CMI)
elective therapy for, 757
endovascular or open surgery for, 757
evaluation pathway for, 757
prevention of, 757
risk factors of, 756
Chronic myeloid leukemia, indications for splenectomy in, 481
Chronic obstructive pulmonary disease (COPD), 846, 864, 926
Chronic pancreatitis, 449454
endoscopic intervention for, 451
pain in, etiology of, 450
pathophysiology of, 449450
surgical management of, indications and outcomes in,
451453
Cilostazol, effect on graft patency, 737, 738
Ciprofloxacin, for necrotizing soft tissue infections, 829

5/22/2012 6:24:25 PM

Index

Cirrhosis, 366369, 937


abdominal wall defects, repair of, 366367, 367t
diagnosis of, 937938
fluid management for, 938939
morbidity/mortality risk factors for, 368
scoring system to predict, 368369
patient optimization, preoperative, 367368
predictor of, 937
with symptomatic gallstone disease, laparoscopic
cholecystectomy for, 366
Cisplatin. See also ECF
-based CRT, for esophageal cancer, 94
for biliary tract tumors, 406407
for squamous cell carcinoma, 254
Clindamycin
for lymphedema, 793
and pseudomembranous colitis, 240
Clinical breast examination (CBE), 578
Clinical Institute Withdrawal Assessment for Alcohol (revised)
(CIWA-Ar), 922
Clopidogrel
for carotid occlusive disease, 686
and coagulopathy, 910
effect on graft patency, 737, 738
for non-ST-elevation MI, 904
for ST-elevation MI, 903
Clostridial necrotizing myositis, 834
Clostridium difficile (C. difficile) colitis. See Pseudomembranous
colitis
CMF (cyclophosphamide, methotrexate, and 5-FU), for breast
cancer, 592
Coagulopathy, 909
preoperative management of, 909
Colectomy
with ileostomy, for pseudomembranous colitis,
241
selection criteria and outcomes for, slow-transit
constipation, 301
Collagen, 820
Colon cancer and large bowel obstruction, 221.
See also Cancer
Colonic stenting, for large bowel obstruction, 224225
Colonic tube placement, for acute colonic
pseudo-obstruction, 225
Colonic volvulus
and large bowel obstruction, 221222
operative management of, 223224
Colonoscopy
for familial adenomatous polyposis, 263264
for ischemic colitis, 233
for lower gastrointestinal bleeding, 314315
for Lynch syndrome, 263
for pseudomembranous colitis, 239
for rectovaginal fistulas, 306
Colorectal cancer, 245248. See also Cancer
Colorectal hepatic metastases, treatment for, 335336
Colorectal surgery. See also Surgery
for surgical site infection, 894895
mechanical bowel preparation for, 7
mechanical bowel preparation prior to, 268269

PMPH_INDEX.indd 961

961

Colostomy
for acute colonic pseudo-obstruction, 225
for squamous cell carcinoma, 254
Combined partial gastric resection, for peptic ulcer disease, 112
Comfort scale, 918
Common bile duct (CBD)
benign strictures, 412413
malignant strictures, 413414
Common bile duct exploration (CBDE)
T-tube drainage after choledochotomy during, 388
Common bile duct stones (CBDS), 386390, 410412
with acute cholangitis, biliary decompression for, 388389
detection of, 386387
endoscopic retrograde cholangiopancreatography for,
411412
evaluation of, 387388
gaining access to, following Rouxen-Y gastric bypass, 412
with/without gallstones, cholecystectomy for, 389390
noninvasive imaging modalities for, 411
prediction of, 410411
pregnant women with, 411412
surgery-first strategy for, 387, 388
T-tube drainage after choledochotomy, insertion of, 388
Communication skills, 19
improvement, and surgical care error reduction, 1920
Community-associated methicillin resistant Staphylococcus
aureus (CA-MRSA), 826. See also Methicillin resistant
Staphylococcus aureus
Complementary alternative medicines (CAMs), for abnormal
surgical and postoperative bleeding, 911912
Complete surgical excision (CSE)
for lobular neoplasia, 581
radial scar diagnosis and, 581
Completion angiography, 701
Compression, for varicose veins/vein insufficiency, 770
Computed tomographic angiography (CTA)
for aortic dissection, 670
for arterial pseudoaneurysms, 678
for arteriovenous fistulas, 678
for diabetic foot, 763
pulmonary, 786
Computed tomography (CT)
for appendicitis, 277279
for biliary tract tumors, 404
for blunt splenic injury, 492
contrast-enhanced
for liver abscesses, 331
for pancreatic pseudocysts, 437f, 438, 441
for esophageal cancer, 91
for esophageal perforation, 2829
for esophageal pulsion diverticula, 49
for gallstone ileus, 401
for gastric adenocarcinoma, 147
for hepatic encephalopathy, 358, 359
for hyperthyroidism, 950
for incomplete SBO, 169
for insulinoma, 467
for large bowel obstruction, 222, 223
for liver abscesses, 330, 331
for liver malignancies, 335336

5/22/2012 6:24:25 PM

962

Index

Computed tomography (continued)


for lower gastrointestinal bleeding, 314, 315
for malignant melanoma, 801
for metabolic squamous cell carcinoma of unknown primary, 816
for mucinous cystic neoplasm, 471, 472f
for necrotizing fasciitis, 833
for necrotizing myositis, 834
for necrotizing soft tissue infections, 828
for pancreatic adenocarcinoma, 458, 459
for primary chest wall tumors, 637
for pulmonary embolism, 928
for rectal cancer, 245, 246
for rectovaginal fistulas, 307
for small bowel tumors, 188
for solitary neck mass, 814
for thyroiditis, 561
for VIPoma, 469f
-volumetric analysis, for postoperative hepatic failure, 937938
for ZollingerEllison syndrome, 124
Confusion Assessment Method (CAM) test, 921
Conns syndrome, 952
Conservative splenic surgery, for postsplenectomy sepsis, 503504
Constipation
anorectal physiology/colonic transit time studies for, 301
medical management of, 300301
slow-transit, selection criteria and outcomes for colectomy
in, 301
Continuous renal replacement therapy (CRRT), for acute renal
dysfunction, 943944
Continuous tube feeding vs intermittent tube feeding, 856857
Contrast-enhanced computed tomography (CECT). See also
Computed tomography
for liver abscesses, 331
for pancreatic pseudocysts, 437f, 438, 441
Contrast esophagogram, for esophageal perforation, 28
Contrast lymphography, for lymphedema, 793
Contrast studies, for rectovaginal fistulas, 306
Contrast venography, for venous thoracic outlet syndrome, 712
Coronary artery bypass graft (CABG), for preoperative risk
factors, 863
Corticosteroids
for Crohns disease, 215216
for esophageal caustic injury, 7172
Corticotrophin-releasing hormone (CRH), 952
Cowden syndrome, 261
C-reactive protein (CRP), 276
Crew resource management (CRM), 19
Cricopharyngeal myotomy, for Zenkers diverticulum, 47, 48
Cricothyroidotomy, emergent, conversion to tracheostomy
after, 645
Critical limb ischemia (CLI), 699
Bypass versus Angioplasty in Severe Ischemia of the Leg
(BASIL) trial, 700
infrainguinal revascularization in, 700
Crohns disease (CD), 215219
distinguished from ulcerative colitis, 177
etiology of
environmental factors, 176
genetic factors, 176
immunologic factors, 176179
microbiologic factors, 176

PMPH_INDEX.indd 962

fecal diversion use in, 178


laparoscopic surgery for, 179, 217
medical management for, 215216
outcome and recurrence, anastomosis effect on, 177
small bowel, 174
indications for surgery in, 174
resection margins during surgery, 178179
strictureplasty for, 176177
surgical intervention for, 216217
symptoms of, 215
Crohns Disease Activity Index (CDAI), 215, 216
Cross-sectional studies, 4
CURE trial, 904
Cushings syndrome, 952
Cyclophosphamide for breast cancer, 592. See also CMF
Cystatinc (uCysC), 943
Cytoreductive surgery
for liver malignancies, 337
for ZollingerEllison syndrome, 128
Dalteparin, for abnormal surgical and postoperative bleeding, 910
Danish Hernia Database analysis, 514
Daptomycin, for necrotizing soft tissue infections, 829
Data analysis, 89
Data collection, 89
Deep venous thrombosis (DVT), 775780, 911
complications of, 776778
diagnosis of, 775776
prophylaxis of, 778
screening of, 775776
treatment for, 778779
Defecography, 301
Dehydration, in patients with Myxedema coma, 951
Delirium
for critically ill patients, 921
defined, 917
management in ICU, 921922
Desmoid tumor, 638
Detection bias, 1011. See also Bias
Devascularization procedures, for portal hypertension, 348
Dexamethasone, for thyroid storm, 950, 951t
Dexmedetomidine, 920
Diabetes mellitus (DM), 949950
acute renal dysfunction and, 943
glycemic control
postoperative, 950
preoperative, 949950
oral hypoglycemic agents, complications of, 950t
patients, prophylactic cholecystectomy in, 376
preoperative considerations for, 949t
type 1, 949, 950
Diabetic foot osteomyelitis (DFO)
antibiotics for, 764
diagnosis of, 763764
Diabetic foot, 762765
diagnosis of, 762763
risk factors of, 762
treatment for, 763764
ulcer (DFU), 764
Diabetic ketoacidosis (DKA), 949
Diabetic peripheral neuropathy, 763

5/22/2012 6:24:25 PM

Index

Diarrhea, C. difficile-associated, 243244


Dietary modifications
for constipation, 301
for hepatic encephalopathy, 360361
for intractable ascites, 353
for preventing acute uncomplicated diverticulitis, 209
Dietary Supplements Health and Education Act of 1994, 911
Diff use esophageal spasm (DES). See also Achalasia
diagnosis of, 37
nonsurgical treatment options for, 38
surgery for, 38
Diff use gastric cancer (DGC), 146. See also Cancer
Digital subtraction angiography (DSA), for diabetic foot, 763
Digoxin, for hyperthyroidism, 950
3,4 Dihydroxyphenolglycol (DHPG), 545
Diphenoxylate, for fecal incontinence, 299
Discharge, 17
Distal pancreatectomy (DP), for chronic pancreatitis, 453
Distal selective embolization, 492493
Diuretics
for acute renal dysfunction, 944
for hypocalcemia, 853
Diverticular disease of colon. See Diverticulitis
Diverticulitis, 207211
acute
complicated acute diverticulitis, surgery for, 207208
younger patients after single attack of, 209
laparoscopic versus open colectomy for, 209210
uncomplicated
recurrence prevention, dietary recommendations
for, 209
sigmoid resection after, 208209
Dobutamine, for pulmonary hypertension, 929
Docetaxel, for breast cancer, 592
Dopamine, for acute renal dysfunction, 944
Doppler ultrasound. See also Ultrasound
for blunt splenic injury, 492
for deep venous thrombosis, 776
for small bowel anastomosis, 169
Double blinding, 5. See also Blinding
Doxorubicin, for breast cancer, 592
Doxycycline
for necrotizing soft tissue infections, 829
for surgical site infection, 892
DPP-IV inhibitors, and diabetes, 950t
DRIL procedure, for arterial steal secondary to AVFs and
AVGs, 743
Drotrecogin alpha, for necrotizing myositis, 835
Ductal carcinoma in situ (DCIS). See also Carcinoma
radiation therapy for, 613614
SLN biopsy for, 600601, 614
surgical management of, 612613
Duloxetine, for diabetic foot, 763
Dumping syndrome, after bariatric surgery, 139
Duodenal gastrinoma, 120
Duodenal preserving pancreatic head resection (DPPHR), for
chronic pancreatitis, 452
Duodenotomy, for ZollingerEllison syndrome, 123124
Duplex ultrasound. See also Ultrasound
for arterial pseudoaneurysms, 678
for arteriovenous fistulas, 678

PMPH_INDEX.indd 963

963

for carotid stenosis, 685


for diabetic foot, 763
for renal artery stenosis, 750
Dyslipidemia, after bariatric surgery, 139
Dysphagia, esophageal stents for, 83, 86
Early Breast Cancer Trialists Collaborative Group (EBCTCG),
604, 605
Early gastric cancer (EGC). See also Cancer
criteria for curative endoscopic resection, 147t
treatment of, 147148
ECF (epirubicin, cisplatin, 5-FU)
effect on oxyntic parietal cells, 119f
for esophageal cancer, 94
for gastric cancers, 148
Electrical stimulation, for peripheral nerve injuries, 820821
Electrocardiogram (ECG)
for acute myocardial infarction, 902
for hypercalcemia, 951
for hyperthyroidism, 950
Electrocautery, for anal intraepithelial neoplasia, 255
Electrolyte abnormalities
associated with seizure, 848
in gastric outlet obstruction, 847
Electromyography, for constipation, 301
Embolization, transcatheter, for lower gastrointestinal bleeding,
315316
Endoanal ultrasound, for fecal incontinence, 300
Endobiliary stenting, for chronic pancreatitis, 412, 413
Endocarditis, 883
Endoleaks, management of, 663
Endologix, 663
Endoluminal therapy, for esophageal perforation, 30
Endorectal advancement flap (ERAF), for fistula-in-ano, 296
Endorectal ultrasound (ERUS), for rectal cancer, 245246
Endoscopic biliary decompression, for acute
cholangitis, 388389
Endoscopic dilatation
for achalasia, 41
for diff use esophageal spasm, 41
for esophageal pulsion diverticula, 50
Endoscopic diverticulotomy
flexible, 48
rigid, 47
for Zenkers diverticulum, 46t, 47
Endoscopic mucosal resection (EMR), for early gastric
cancer, 146
Endoscopic retrograde cholangiopancreatography (ERCP)
for acute pancreatitis, 421422
for benign biliary strictures, 396
for biliary tract tumors, 404
for CBD stones, 411
as definitive therapy, 411
postoperative, 411
for pregnant women, 411412
preoperative, 411
success rate of, 411
for hilar cholangiocarcinoma, 413
for common bile duct stones, 387388
Endoscopic submucosal dissection (ESD), for early gastric
cancer, 146

5/22/2012 6:24:25 PM

964

Index

Endoscopic ultrasound (EUS). See also Ultrasound


for CBD stones, 411
for common bile duct stones, 386
for esophageal tumors, 7475, 91, 92
for gastric adenocarcinoma, 146
-guided FNA, for pancreatic adenocarcinoma, 458459
for insulinoma, 467468
for pancreatic pseudocysts, 441
for ZollingerEllison syndrome, 123
Endoscopic variceal band ligation (EBL), for portal
hypertension, 346
Endoscopy
for achalasia, 37, 41
for bleeding peptic ulcers, 110111
for chronic pancreatitis, 450
for esophageal cancer, 91
for esophageal pulsion diverticula, 47
for familial adenomatous polyposis, 264
for gallstone ileus, 401
for gastroesophageal reflux disease, 5960
for lower gastrointestinal bleeding
capsule, 316
push, 316
for small bowel diverticula, 190
for Sphincter of Oddi dysfunction, 425426
timing in acute phase of caustic injury, 71
for UGI bleeding, 156
for weight loss, 137
for Zenkers diverticulum, 46, 47
Endovascular repair (EVAR)
AOID lesions favorable for, 692
choice of devices, 663
management of endoleaks, 663
versus open repair, 662
preservation of collateral flow in, 664
for ruptured aneurysms, 664
Endovascular therapy (EVT)
for acute mesenteric ischemia, 757
for arterial pseudoaneurysms, 679
for arteriovenous fistulas, 679
in end-stage renal disease (ESRD) patients, 702
versus open bypass surgery (OBS), 700, 702
outcome of, 701
patency, 701
End-stage liver disease (ESLD), 937
Endurant, 663
Enteral feeding, 198
Enteral nutrition (EN), 196
for enterocutaneous fistula, 192
for pancreatitis, 431432
preoperative delivery of, 855856
for short bowel syndrome, 197
transition to, 197
Enteroclysis, for small bowel tumors, 187188
Enterocutaneous fistula (ECF), 192
factors predicting mortality in, 192
nonresectional approach for, 194
optimal nutritional strategy in, 192
optimal timing for elective surgical intervention for, 193194
resectional approach for, 194
somatostatin analogs for, 192193

PMPH_INDEX.indd 964

Enterolithotomy, for gallstone ileus, 399


Enzyme-linked-immunosorbent serologic assay (ELISA),
for C. difficile-associated diarrhea, 239
EPCAM (TACSTD1) gene mutations, and inherited colorectal
cancer syndromes, 261262
Epidermal growth factor receptor (EGFR), 254
Epigastric hernia, 525528. See also Hernia
repair of
laparoscopic, 526527
preoperative antibiotics for, 527
using mesh, 526
Epirubicin. See also ECF
effect on oxyntic parietal cells, 119f
for esophageal cancer, 94
for gastric cancers, 148
Errors, during surgical care, 1417
patterns, 17
safety models to reduce, 1718
Erythromycin, for surgical site infection, 894
Erythropoietin, for acute renal dysfunction, 944
Esophageal atresia and tracheoesophageal fistula (EA/TEF)
conduit for creation of neoesophagus, 99100
diagnosis of, 98
environmental/genetic risk factors for, 98
high-output fistula, stabilization of patient with, 99
long-gap, timing for surgical repair of, 99
short-gap, length for primary repair in, 99
Esophageal cancer. See also Cancer
diagnostic and imaging studies prior to and during
treatment of, 9192
modality therapy for, 9394
optimal neoadjuvant regimen for, 94
surgery for, 9394
Esophageal caustic injury, 70
acute phase, endoscopy timing in, 71
antibiotic therapy for, 71
chronic phase, indication for surgery in, 72
corticosteroids for, 7172
severity prediction through early signs and symptoms,
7071
Esophageal function testing, for diagnosis of achalasia and DES,
3738
Esophageal myotomy, for esophageal pulsion
diverticula, 49, 50
Esophageal perforation, 2536
diagnostic studies for, 2829
exclusion/diversion procedure after, 30
intrathoracic, primary repair of, 2930
nonoperative management of, 29
symptoms and physical findings of, 2728
Esophageal pulsion diverticula
diagnostic tests for, 49
intervention indication for, 4950
optimal intervention for, 4749
symptoms associated with, 47
Esophageal stents, 83
for anastomotic leak, 8687
for benign esophageal strictures, 8687
indications for, 8384
optimal insertion technique, 8586
options of, 8485

5/22/2012 6:24:25 PM

Index

Esophageal tumors, 74
clinical staging of, 75
esophagectomy
transthoracic versus transhiatal, 7576
surgically resectable, chemoradiation for, 7677
Esophagectomy
for achalasia, 41
for esophageal perforation, 30
outcomes, and pyloric drainage procedure, 7677
transhiatal, 7576
transthoracic, 7576
Esophagogastroduodenoscopy (EGD), for esophageal perforation, 28
Esophagogram
for achalasia, 37, 41
for esophageal pulsion diverticula, 49
Esophagoscopy, for metabolic squamous cell carcinoma of
unknown primary, 816
ESSENCE trial, 904
Essential thrombocythemia (ET), 482
EU Hernia Trialists Collaboration meta-analysis, 514
Evidence-based medicine, 111
bias in, 45
publication bias, 67
risk of, 1011
combing and synthesizing, 5
data collection and analysis of, 89
defined, 1
example of, 7
external validity of, 5
GRADE criteria for, 23, 3t
hierarchy of, 2
interventions, effects of, 11
meta-analysis of, 6
methods of, 8
narrative reviews of, 56
quality and strength of, assigning, 3
results, 910
analyzing and presenting, 9
statistical precision of, 5
study design of, 34
systematic reviews of, 6
systems for rating, 13
weight of evidence, 5
EVLT, for venous insufficiency, 770
Ewings sarcoma, 638
Excluder, 663
Exemestane, for breast cancer, 592
Exercise therapy
after bariatric surgery, 138
for lymphedema, 793
for peripheral nerve injuries, 820821
Exogenous estrogen intake and male breast cancer, 618
Extensive mobilization with primary closure under tension
for primary repair in short-gap esophageal atresia, 98
External beam irradiation therapy (EBRT)
for metastases to adrenal, 541
for squamous cell carcinoma of the anal region, 256
External hemorrhoids (EHs), management of, 289290
External validity, 5
Extracorporeal membrane oxygenation (ECMO), 930
for necrotizing myositis, 835

PMPH_INDEX.indd 965

965

F-18 fluorodeoxyglucose (FDG)-positron emission tomography,


for diabetic foot, 764
Familial adenomatous polyposis (FAP), 260
attenuated, 260
Familial colorectal cancer type X (FCCTX), 261
Familial hypocalciuric hypercalcemia (FHH), 568
Fasciotomy with lower extremity revascularization, for acute
arterial ischemia from embolus, 725726
FASTHUG, 918
Fasting serum gastrin test, for ZollingerEllison syndrome, 122
Fecal bacteriotherapy, for pseudomembranous colitis, 240
Fecal diversion, for Crohns disease, 178
Fecal incontinence
artificial sphincters for, 300
initial evaluation and management of, 299300
sacral nerve stimulators for, 300
surgical patients, workup for, 300
Feltys syndrome, 484
Femoral insertion site, use in catheter insertion, 884
Femoropopliteal and tibioperoneal diseases, 699704
Femoropopliteal lesions, TASC II classification of, 700t
Fenoldapam, for acute renal dysfunction, 944
Fiber supplementation
for fecal incontinence, 299
for hemorrhoids symptom reduction, 287
for uncomplicated diverticulitis, 209
Fibrin glue
for fistula-in-ano, 296
for rectovaginal fistulas, 309
Fibrous dysplasia, 638
Filariasis, 792
Fine-needle aspiration (FNA) biopsy. See also Biopsy
for benign biliary strictures, 396
for breast cancer, 580581
EUS-guided, 458459
for infected pancreatic collections, 431
for nontoxic goiter, 563
for soft tissue sarcomas, 808
for solitary neck mass, 814, 815
Fissures, anal. See Anal fissures
Fistula
plugs, for fistula-in-ano, 296
and small bowel Crohns disease, 175
Fistula-in-ano
operative approaches to, 295
prediction/prevention, following perirectal abscesses
drainage, 295
sphincter-sparing techniques for, 295
Fistulectomy
for fistula-in-ano, 295, 296
for rectovaginal fistulas, 309
Flap techniques, for anal strictures, 295
FloTrac/Vigileo, for perioperative cardiac monitoring, 870, 871, 872t
Fluid collections, acute, defined, 437438
Fluorescein, for small bowel anastomosis, 170
Fluoroquinolones and pseudomembranous colitis, 240
5-Fluorouracil (5-FU). See also CMF; ECF
-based CRT, for esophageal cancer, 94
for biliary tract tumors, 406
for breast cancer, 592
for pancreatic adenocarcinoma, 460, 461, 471
for squamous cell carcinoma, 254

5/22/2012 6:24:25 PM

966

Index

Fondaparinux, for deep venous thrombosis, 778


Forest plot, 6
Fractional excretion of sodium (FENa), 848, 943
Fractionated plasma separation,
adsorption and dialysis system
for postoperative hepatic failure, 939
Free microvascular TRAM flap, 624625
Free perforation and small bowel Crohns disease, 175
Fresh frozen plasma (FFP), for abnormal surgical and
postoperative bleeding, 913
Fundoplication
for esophageal pulsion diverticula, 50
for gastroesophageal reflux disease, 61
Fungemia, 879
Funnel plot, 7
Fusidic acid, for pseudomembranous colitis, 240
Future liver remnant (FLR), 335
Gabapentin, for diabetic foot, 763
Gadobenate dimeglumine, for liver resection, 323
Gadoxetate disodium, for liver resection, 323
Gallbladder tumors, 403407
adjuvant therapies for, 406
diagnosis of, 404
and gallstones, 377
resection of, 405406
staging of, 404405
symptoms of, 403404
unresectable, palliative options for, 406407
Gallstone ileus, 399401
diagnosis of, 400401
enterolithotomy for, 399
laparoscopic treatment for, 399400, 398t
treatment for, 401
Gallstones, 375377
asymptomatic, history of, 375376
CBDS with/without, cholecystectomy for, 389390
cholecystectomy for, 376
in at-risk populations, 376377
cost-effectiveness of, 376
frequency of, 375
gallbladder cancer and, 377
Gamma amino buteric acid (GABA) agonists, 920
Gardner syndrome, 260261
Gas gangrene, 826827
Gastrectomy
for early gastric cancer, 147148
extent of, 148149
for gastric cancers, 148
Gastric acid suppression and pseudomembranous colitis, 240
Gastric adenocarcinoma, 146. See also Adenocarcinoma
adjuvant therapy, consideration of, 149
diagnostic and staging work-up for, 146147
neoadjuvant therapy for, 148
resection and lymphadenectomy, extent of, 148149
types of, 146
Gastric cancer. See also Cancer
advance, treatment of, 148149
genetic mutations in, 146
prevalence and incidence of, 146

PMPH_INDEX.indd 966

risk factors for, 146


stage IV, surgical management of, 149
Gastric feeding, for acute pancreatitis, 423424
Gastric outlet obstruction (GOO), electrolyte abnormalities in, 847
Gastrinoma, 469470. See also ZollingerEllison syndrome
advanced disease, surgical management of, 128
anatomic localization of, 468t
diagnosis of, 468t
disease states associated with, 470t
duodenal, 120
multiple, 121f, 128
liver metastases from, patients survival of, 121
pancreatic, 121
size and location of, 120
surgical management, outcomes of, 126t127t
symptoms of, 468t
Gastrinoma resection, for MEN-1, 125, 128
Gastroesophageal reflux disease (GERD)
endoscopy for, 5960
fundoplication for, 61
mainstay of therapy for, 60
medical treatment versus surgical treatment, 6061
risk factors for, 59
24-hour esophageal pH monitoring in, 60
Gastroesophageal varices (GEV), bleeding
emergency shunt surgery for, 348349
liver transplantation for, 348349
Gastrointestinal stromal tumors (GIST)
adjuvant therapy for, 189
incidence, histology and location of, 187
prognostic factors for survival for, 190
treatment for, 188
recurrence, 190
Gastrojejunostomy (GJJ), for gastric outlet obstruction, 149
Gemcitabine
for biliary tract tumors, 406
for pancreatic adenocarcinoma, 460462
for pancreatic ductal adenocarcinoma, 471
Gemcitabineoxaliplatin (GEMOX), for biliary tract tumors, 406
Genetic counseling, for inherited colorectal cancer, 264
Giant cell tumor, 638
Glasgow Coma Scale (GCS), 358
Glucagon-like peptide 2 (GLP-2), 198
Glucagonoma, 470
anatomic localization of, 468t
diagnosis of, 468t
symptoms of, 468t
Glucocorticoids
for adrenal incidentaloma, 952
for myxedema coma, 951
for thyroid storm, 950
Glutamine, 197198
Glyceryl trinitrate (GTN)
for anal fissures, 294
for hemorrhoids, 287
Glycoprotein IIb/IIIa receptor antagonists, for ST-elevation MI,
903
Grading of Recommendations Assessment, Development, and
Evaluation (GRADE) Working Group
criteria for evidence-based medicine, 23, 3t

5/22/2012 6:24:25 PM

Index

Graft occlusion, 735738


early
causes of, 735
treatment for, 736
late
causes of, 736
treatment for, 736737
patency
anticoagulation effect on, 738
platelet inhibition effect on, 737738
Graves disease, 950
Graves ophthalmopathy (GO), 559
Growth hormone (GH), for short bowel syndrome, 197198
Gynecological malignancies, in HNPCC patients, 264
Gynecomastia, 617
Hairy cell leukemia (HCL), 482, 483
Hamartomatous polyposis syndromes, 261
Harmonic scalpel hemorrhoidectomy (HSH), for hemorrhoids,
288
Heller cardiomyotomy, surgical approach for, 39
Hellers myotomy, for achalasia, 39
Hemicolectomy, for colon cancer, 223
Hemodialysis, 842
vascular access for
autologous fistula versus prosthetic graft, 742744
complications of, 743744
preoperative planning, 742
Hemolytic anemia, indications for splenectomy in, 483484
Hemophilia A, 909
Hemophilia B, 909
Hemophilus influenza infections, 500
vaccinations for post-SAE patients against, 491
Hemorrhoidectomy
open versus closed, 288
Hemorrhoids, 287290
external hemorrhoids, management of, 289290
internal hemorrhoids, management of, 287289
nonoperative management of, 287
observation with no procedural intervention of, 287
operative strategies for, 288289
postoperative pain, controlling, 289
postoperative urinary retention, low volume resuscitation
for, 289
thrombosed hemorrhoids, management of, 289290
Hemothorax, retained, management of, 653
Heparin
low molecular weight. See Low molecular weight heparin
for pulmonary embolism, 786
unfractionated. See Unfractionated heparin
for venous thromboembolism, 910
Hepatic angiosarcoma (HAS), 807
Hepatic encephalopathy
causes of, 359360
differential diagnosis of, 359
following TIPS, predicting, 361
mortality rate associated with, 361
risk factors of, 357
severity of, grading, 358359
transplantation for, 361362

PMPH_INDEX.indd 967

967

treatment for, 360361


types of, 357358
Hepatic encephalopathy scoring algorithm (HESA), 358
Hepatic infections, 329331
clinical features of, 330
diagnosis of, 330331
epidemiologic characteristics of, 329
incidence of, 329
pathophysiology of, 330
treatment for, 331
Hepatocellular carcinoma (HCC), 323. See also Carcinoma
liver resection for, 325326
staging of patients prior to resection, 334
treatment for, 335
Herceptin Adjuvant trial (HERA) trial, 606
Hereditary diff use gastric cancer (HDGC), 146
Hereditary hemorrhagic telangiectasia (HHT), 261
Hereditary spherocytosis (HS), 484
Hernia
epigastric. See Epigastric hernia
incisional, 530532
and large bowel obstruction, 221
recurrence, Risk factors for, 515
recurrent inguinal, 518521
chronic pain following repair of, incidence of, 520
laparoscopic repair of, 518, 520
nonoperative management of, 519
open of, 518520
pattern of, 519
primary repair of, 518
repair, for surgical site infection, 893894. See also Surgery
ventral, 530532
umbilical. See Umbilical hernia
HES 130/0.4 (Voluven), 844845, 853
H-grafts, low diameter, for portal hypertension, 347
High reliability theory, 18
High-Reliability Organizations (HROs), 1718
Hilar cholangiocarcinoma. See also Carcinoma
biliary decompression in, 413
percutaneous transhepatic cholangiography for, 413
Hirschsprungs disease, 301
HMB-45 markers, 800, 801
HOPE trial, 904
Hospital-associated pneumonia (HAP), 927
House advancement flap, for anal strictures, 295
H. pylori
positive PUD, management of, 110
H2RA therapy
for NSAID-induced gastrointestinal complications, 110
for stress ulcer prophylaxis, 154
Human factors (HFs), 19
Hungry bone syndrome, hypocalcemia, 950
Hyberbaric oxygen (HBO) therapy, for necrotizing myositis, 834835
Hydatid cysts
clinical features of, 330
diagnosis of, 330331
epidemiologic characteristics of, 329
incidence of, 329
pathiphysiology of, 330
treatment for, 331

5/22/2012 6:24:25 PM

968

Index

Hydroxyethyl starch (HES), 843


Hyperbaric oxygen therapy (HBOT)
for diabetic foot osteomyelitis, 764
for necrotizing soft tissue infections, 829830
Hypercalcemia, 951
magnesium abnormalities and, 847848
Hypercapneic respiratory failure, 927
Hypergastrinemia, disease states associated with, 470t
Hyperkalemia, 853
defined, 841
treatment for, 841842
Hypermagnesemia, 845846, 853
Hyperparathyroidism, 568
diagnosis of, 568569
optimal treatments for, 569
parathyroid surgery, complications of, 571572
preoperative imaging in, 570
preoperative management of, 569570
surgery for, 570571
Hyperplastic/mixed polyposis syndromes, 261
Hyperpyrexia, preoperative management of, 950
Hypertension, renovascular. See Renovascular hypertension
Hyperthyroidism, 559565
diagnostic test for, 559
preoperative management of, 950
surgery for, 560561
treatment for, 559560
Hypertonic saline solutions (HTSs), for traumatic injury, 847
Hypocalcemia, 568, 571572
hungry bone syndrome due to, 951
hypocalcemia due to hypoparathyroidism, 951
parathyroidectomy for, 951
Hypomagnesemia, 846, 853
Hyponatremia, 357
magnesium abnormalities and, 849, 853
Hypoparathyroidism, 555556
Hypothermic isolated limb perfusion (ILP), for malignant
melanoma, 803
Hypothyroidism, surgery for, 950951
Hypovolemia, 942
Hypoxemic respiratory failure, 927
I-123 radioactive iodine scan, for hyperthyroidism, 559
Ileorectal anastomosis (IRA)
for constipation, 301
surveillance after, 263
Ilioinguinal neurectomy, 515
Iloprost, for thromboangiitis obliterans, 732733
Imatinib, for sarcoma of small bowel, 189
Immediate presurgical care, 16
Immune thrombocytopenia (ITP), indications for splenectomy
in, 483
Immunoglobulin therapy, for necrotizing myositis, 835
Immunohistochemistry, 814
Immunosuppression and pseudomembranous colitis, 240
In situ carcinoma of the breast, 612615. See also Carcinoma
Incisional biopsy, for soft tissue sarcomas, 808. See also Biopsy
Incisional hernias, 530532. See also Hernia
repair of
biologic versus synthetic mesh, 530
open versus laproscopic, 531

PMPH_INDEX.indd 968

recurrencde rate for, 531


wound infection, rate of, 532
watchful waiting for, 531
Indocyanine green (ICG) retention, 938
Infected pancreatic collections, 430435
diagnosis of, 431
management of, 433434
nomenclature of, 430431
nutrition for, 431433
Infectious Disease Society of America, 927
Inferior mesenteric artery (IMA), 664
reimplantation of, 234
Inferior vena cava (IVC) filters, for pulmonary embolism, 787788
Inflammatory bowel disease (IBD)
Crohns disease
laparoscopic surgery for, 217
medical management for, 215216
surgical intervention for, 216217
symptoms of, 215
ulcerative colitis
laparoscopic surgery for, 218219
medical management for, 218
symptoms of, 215
Infliximab
for Crohns disease, 216
for ulcerative colitis, 218
Infrainguinal arterial disease
classification of, 699
revascularization, 699700
comorbidities, influence of, 703
determination of outcomes of, 700704
selection of, 700
technical success, assessing, 701
Inguinal hernias, 513, 514. See also Hernia
repair (IHR), 367
Inguinodynia/chronic groin pain, 515
Inherited colorectal cancer syndromes, 260265. See also Cancer
evaluation of, 262263
genetic counseling for, 264
molecular mechanisms of, 261262
prevalence of, 260
surgery for, 263
surveillance for, 263264
Inotropes, for necrotizing soft tissue infections, 829
Insulin-like growth factor (IGF), 944
for soft tissue sarcomas, 810
Insulinoma, 467468
anatomic localization of, 468t
diagnosis of, 468t
symptoms of, 468t
INT-0102 trial, 605
Intact parathyroid hormone (iPTH) levels, 568
Intensity-modulated radiation therapy (IMRT), radiation
enteritis due to, 229
Intensive care unit (ICU), 917923
delirium
for critically ill patients, 921
management of, 921922
sedation management in
oversedation, 918, 920
target-controlled infusion, 920921

5/22/2012 6:24:25 PM

Index

Interferon- (IFN-), for malignant melanoma, 803


Intergroup Melanoma Surgical Trial, 802
Interleukin 18 (uIL-18), 943
Intermittent hemodialysis (IHD), for acute renal dysfunction,
943, 944
Intermittent mandatory ventilation (IMV), 930
Intermittent thermodilution (ITD), for perioperative cardiac
monitoring, 870
Intermittent tube feeding vs continuous tube feeding, 856857
Internal iliac artery (IIA) embolization, 664
Intestinal obstruction and small bowel Crohns disease, 174
Intestinal rehabilitation programs (IRPs), for short bowel
syndrome, 199200
Intestinal transplantation (IT), for short bowel
syndrome, 199
Intraabdominal abscesses and small bowel Crohns disease,
174175
Intraabdominal adhesions, 167168
Intra-aortic balloon counterpulsation (IABCP), 904
Intractable ascites, medical management of, 353355
liver transplantation, 354
paracentesis in, 353
peritoneovenous shunting, 354
transjugular intrahepatic portosystemic shunting, 354
Intraductal papillary mucinous neoplasm (IPMN)
pancreatectomy for, 471472
preoperative differentiation of, 471472
resection patients, surveillance for, 472
Intraductal ultrasonography (IDUS), for benign biliary
strictures, 396
Intraluminal lesions, 222
Intraoperative angiography, 701
Intraoperative cholangiography (IOC)
for benign biliary strictures, 394
for bile duct injuries, 395396
for CBD stones, 411
Intraoperative duplex scanning, 701
Intraoperative ultrasound (IOUS), for intrapancreatic
gastrinomas, 123
Intrapopliteal grafts, patency of, 704t
Intravenous (IV) fluids, for thyroid storm, 951t
Intravenous antibiotics, for surgical site infection. See also
Antibiotics
duration of, 892
timing of, 892
Intravenous fluids, for necrotizing soft tissue infections, 829
Intravenous immunoglobulin (IVIG), for necrotizing soft tissue
infections, 829
Intrinsic renal failure, magnesium abnormalities and, 848
Invasive ductal carcinoma, 588. See also Carcinoma
Invasive lobular carcinoma of the breast, 588. See also Carcinoma
Iodine solution, for thyroid storm, 950, 951t
Iron, for hypothyroidism, 950
Irreversible electroporation (IRE), for liver malignancies, 336
Ischemic colitis (IC), 232235
association with thrombotic/thromboembolic
conditions, 233
diagnosis of, 233
prevention of, 233
risk factors for, 232
treatment for, 233234

PMPH_INDEX.indd 969

969

Isolated elevated liver function tests, for postoperative hepatic


failure, 936
Isolated limb infusion (ILI), for malignant melanoma, 803
Jaundice, obstructive. See Obstructive jaundice
Jejunal feeding, for acute pancreatitis, 423424
Jejunoileal bypass (JIB), 134
Juvenile polyposis (JPS), 261
Kanamycin, for surgical site infection, 894
Keel technique, 526
Kidney injury molecule 1 (uKIM-1), 943
Klinefelters syndrome, 617
Laboratory Risk Indicator for Necrotizing Fasciitis (LRINEC)
score, 828
Lactulose, for hepatic encephalopathy, 360
Langerhans cell histiocytosis, 638
Lanreotide, for enterocutaneous fistula, 193
Laparoscopic adjustable gastric banding, 135136
Laparoscopic adrenalectomy, 548
Laparoscopic appendectomy (LA), for appendicitis, 279280
Laparoscopic cholecystectomy (LC), for cirrhotics with
symptomatic gallstone disease, 366
Laparoscopic colectomy, for diverticular disease of colon,
209210
Laparoscopic common bile duct exploration (LCBDE), for CBD
stones, 411
Laparoscopic hernia, 514. See also Hernia
Laparoscopic pancreas surgery, for chronic pancreatitis, 453
Laparoscopic resection
for adrenocortical carcinoma, 539
for inguinal hernias, 518, 520
of lumbar hernias, 527
for metastases to adrenal, 540541
of obturator hernias, 527
of umbilical/epigastric hernias, 526527
Laparoscopic Roux-Y gastric bypass (LRYGB), 134135
Laparoscopic sleeve gastrectomy (LSG), 136
Laparotomy
intraabdominal adhesion formation following
techniques/agents to decrease, 167168
for small bowel diverticula, 190
for ZollingerEllison syndrome, 128
Laproscopic repair, of incisional hernias, recurrence rate of, 531
Large bowel obstruction, 221225
causes of, 222
colonic stenting for, 224225
diagnosis of, 222223
laparoscopic evaluation for, 223224
surgery for, 223
symptoms of, 221222
l-Arginine, for diabetic peripheral neuropathy, 763
Laryngoscopy, 570
Laser ablation, for liver malignancies, 336
Lasix and magnesium abnormalities, 849
Lateral internal sphincterotomy (LIS), for anal fissures, 294
Lateral pancreaticojejunostomy (LPJ), for chronic
pancreatitis, 451
Leadership, 19
Left ventricular end-diastolic area index (LVEDAI), 873

5/22/2012 6:24:25 PM

970

Index

Letrozole, for breast cancer, 592, 593


Levothyroxine
for hypothyroidism, 950, 951
for nontoxic nodular goiter, 563
for thyroiditis, 562
Lichtenstein herniorrhaphy, 514
Lichtenstein repair, of inguinal hernia, 519
LiDCO, for perioperative cardiac monitoring, 870, 871, 872t
Lidocaine, for diabetic foot, 763
Li-Faumeni syndrome, 807
Ligation of transsphincteric fistula tract (LIFT), for
fistula-in-ano, 296
Limb-sparing surgery, for soft tissue sarcomas, 809
Linezolid, for necrotizing soft tissue infections, 829
Lipid disorders, after bariatric surgery, 139
Lipoblastoma, 638
Lipoma, 638
Lithium, for hypothyroidism, 950
Lithotripsy, for gallstone ileus, 401
Liver fatty acid binding protein (uL-FABP), 943
Liver
imaging of, 323
surgery, 323327
laparoscopic, 326
transplantation (LT)
GEV bleeding, 348349
for intractable ascites, 354
for portal hypertension, 348349
Lobectomy, for toxic STNs, 561
Lobular carcinoma in situ (LCIS). See also Carcinoma
chemoprevention for, 615
surgical management of, 614615
Lobular neoplasia (LN), surgical management of, 581582
Local excision (LE)
for adenocarcinoma, 255
for anorectal melanoma, 256
for basal cell carcinoma, 256
for Bowens disease, 256
for rectal cancer, 246
for squamous cell carcinoma, 254
for verrucous carcinoma, 257
Localized pancreatic head resection with longitudinal
pancreaticojejunostomy (LR-LPJ), for chronic pancreatitis,
451452
Locally advanced breast cancer (LABC). See also Cancer
axillary lymph node dissection for, 591592
management of, 590591
postmastectomy radiation therapy, 592
Longitudinal intestinal lengthening and tailoring (LILT)
procedure, 198
Loperamide, for fecal incontinence, 299
Low anterior resection (LAR), for rectal cancer, 247
Low-dose unfractionated heparin (LDUH), for venous
thromboembolism, 927. See also Heparin
Lower gastrointestinal bleeding (LGIB), 314317
capsule and push endoscopy for, 316
diagnosis of
accuracy of, 314315
testing order, 314
intra-arterial vasopressin infusion versus transcatheter
embolization for, 315316

PMPH_INDEX.indd 970

surgical intervention, criteria for, 316


Tc-99 sulfur colloid injection versus Tc-99 TRBCs for, 315
urgent colonoscopy for, 315
Low-molecular weight heparin (LMWH). See also Heparin
for abnormal surgical and postoperative
bleeding, 910, 911
for deep venous thrombosis, 778
for non-ST-elevation MI, 904
for pulmonary embolism, 786788
for ST-elevation MI, 903
for venous thromboembolism, 927
Low-salt diets, for intractable ascites, 353
Lubiprostone, for constipation, 301
Lumbar hernias, repair of, 527
preoperative antibiotics for, 527
Lung cancer staging, 631634
Lymphadenectomy
for early gastric cancer, 148
extended, for esophageal tumors, 7576
extent of, 148149
for gastric cancers, 148149
Lymphangiectasia, 793
Lymphedema, 792795
classification of, 793
diagnosis of, 793
microvascular surgery for, 794795
risk factors of, 792
treatment for, 793794
Lymphoma of small bowel, 186
adjuvant therapy for, 189
incidence, histology and location of, 187
prognostic factors for survival for, 190
recurrence, treatment for, 190
treatment for, 188
Lymphoscintigraphy, for lymphedema, 793
Lynch syndrome criteria, 262t
Magnesium abnormalities
anastomotic leak, fluid resuscitation volume affecting,
846847
effect on respiratory muscle function, 845846
electrolyte abnormalities, 847, 848
and hypercalcemia, 847848
hypertonic saline solutions, role of, 847
and hyponatremia, 849, 853
and intrinsic renal failure, 848
and prerenal azotemia, 848
serum albumin, effect on calcium measurements,
848849, 853
and SIADH, 849, 853
and transfusion-related acute lung injury, 849
Magnetic resonance angiography (MRA)
for aortic dissection, 670
for arterial pseudoaneurysms, 678
for arteriovenous fistulas, 678
for diabetic foot, 763
Magnetic resonance cholangiopancreatography (MRCP)
for benign biliary strictures, 396
for biliary tract tumors, 404
for CBD stones, 411
for common bile duct stones, 386

5/22/2012 6:24:25 PM

Index

Magnetic resonance imaging (MRI)


for acinar cell carcinoma (ACC), 471, 469f
for appendicitis, 277, 278
for biliary tract tumors, 404
for diabetic foot, 764
endoanal, for rectovaginal fistulas, 306
for gastric adenocarcinoma, 147
for hepatic encephalopathy, 358, 359, 362
for insulinoma, 467
for liver abscesses, 331
for liver malignancies, 335336
for liver resection, 323
for metabolic squamous cell carcinoma of unknown
primary, 816
for necrotizing myositis, 834
for necrotizing soft tissue infections, 828
for pancreatic pseudocysts, 441
for primary chest wall tumors, 637
for rectal cancer, 245, 246
for solitary neck mass, 814
T1-weighted, 358
for VIPoma, 469f
for ZollingerEllison syndrome, 124
Male breast cancer, 617620. See also Breast cancer
adjuvant therapy for, 619
clinical presentation of, 618
histopathological types of, 618
prognosis of, 619620
risk factors of, 617618
surgical management of, 618619
Malignancies, 488
and small bowel Crohns disease, 175
Malignant liver tumors, 334338
ablative therapies for, 336337
imaging techniques for, 334335
minimally invasive therapies for, 337338
palliative procedures for, 337
staging of patients prior to resection, 334
treatment algorithms for, 335336
Malignant melanoma
clinical signs of, 800
diagnosis of, 800
excisions for, 801
incidence of, 799
staging of, 800801
sentinel node biopsy for, 801802
complete regional lymph node dissections and,
802803
sunlight, impact of, 799800
treatment for, 803
Malnutrition, 909910
risk/diagnosis of, 854855
Mammalian target of rapamycin (mTOR) signaling, 810
Mammography
for male breast cancer, 618
for screening breast cancer, 578590
Manometry
for achalasia, 37, 38
for esophageal pulsion diverticula, 47, 49
for Zenkers diverticulum, 47, 49
Mastectomy, for male breast cancer, 618619

PMPH_INDEX.indd 971

971

Mechanical bowel preparation (MBP)


for elective colorectal surgery, 7
prior to colorectal surgery, 268269
risk factors of, 268
surgical site infection/anastomotic complications, incidence
of, 267268
Meckels diverticulum, surgical removal of, 190
Medial antebrachial cutaneous nerve conduction studies, 711
Melan-A/MART I markers, 800, 801
Mesenteric to left PV bypass (Meso-Rex bypass), for portal
hypertension, 348
Mesh repair, 515
biologic vs. synthetic, recurrence rate of hernias, 530531
and native tissue repairs, 514
for umbilical/epigastric hernia, 526
Mesocaval shunting, for portal hypertension, 347348
Meta-analysis, 6
Metabolic squamous cell carcinoma of unknown primary
panendoscopy, role of, 815816
PET scanning for, 815
tonsillectomy for, 816
Metal covered esophageal stents, 8485
Metanephrine, 545
Metastases to adrenal, management of, 540541
Metastatic adenocarcinoma, 814. See also Carcinoma
Metastatic cervical lymph node location, for solitary neck
mass, 815
Metastatic disease, treatment for, 810
Metformin and diabetes, 950t
Methicillin resistant Staphylococcus aureus (MRSA)
community-associated, 826
NSTIs caused by, 829
Methicillin-resistant Staphylococcus aureus infection, in surgical
site, 895, 900
Methimazole
for Graves disease, 559
for hyperthyroidism, 950, 951t
and tracheoesophageal anomalies, 98
Methotrexate. See also CMF
for breast cancer, 592
for Crohns disease, 216
Methoxytyramine, 545
Methylmethacrylate, 639
Metoprolol, for ST-elevation MI, 903
Metronidazole
for pseudomembranous colitis, 240, 244
for surgical site infection, 892, 894
Mexilentine, for disbetic foot, 763
Microvascular surgery, for lymphedema, 794795
Microwave coagulation therapy (MCT), for liver malignancies,
336337
MilliganMorgan hemorrhoidectomy (MMH), 288
Milrinone, for pulmonary hypertension, 929
Minimal access percutaneous retroperitoneal necrosectomy
(MAPRN), for infected pancreatic collections, 434
Minimally invasive necrosectomy, for infected pancreatic
collections, 433434
Minimally invasive parathyroidectomy (MIP), 570
Mirizzi syndrome, 396
Misoprostol, for NSAID-induced gastrointestinal
complications, 110

5/22/2012 6:24:25 PM

972

Index

Mitomycin C (MMC)
for biliary tract tumors, 406
for squamous cell carcinoma, 254
Mitotane, for adrenocortical carcinoma, 540
MLH1 gene mutations and inherited colorectal cancer
syndromes, 261
Modality therapy, combined, for esophageal cancer, 92
Model for end-stage liver disease (MELD) score, 936, 937
Molecular adsorbent recirculating system (MARS), for
postoperative hepatic failure, 939
Molecular markers
for nontoxic goiter, 563
of pancreatic adenosquamous carcinoma, 470471
Monocyte chemoattractant protein-1 (MCP-1), 845
Morphine sulfate, for appendicitis, 276277
Motor Activity Assessment Scale (MAAS), 918
MSH2 gene mutations and inherited colorectal cancer
syndromes, 261
MSH6 gene mutations and inherited colorectal cancer
syndromes, 261
Mucinous cystic neoplasm (MCN)
pathology of, 471
preoperative differentiation of, 471472
Multicenter Selective Lymphadenectomy Trial I (MSLT I), 802
Multidetector row helical CT (MDCT), for lower gastrointestinal
bleeding, 315
Multinodular goiter (MNG), treatment for, 560
Multiple duodenal gastrinomas, in MEN-1 with
ZollingerEllison syndrome, 121
Multiple endocrine neoplasia type 1 (MEN-1). See also
ZollingerEllison syndrome (ZES)
multiple duodenal gastrinomas in, 121f
surgical management of, 125128
ZollingerEllison syndrome association with, 120
Multiple Endocrine Neoplasia Type 2 (MEN 2), 544
Multiple-organ dysfunction syndrome (MODS), 879
Multivariate logistic regression analysis, 937
Myelofibrosis with myeloid metaplasia (MMM), 481482
MYH-associated polyposis (MAP), 261
MYH gene mutations and inherited colorectal cancer
syndromes, 261
Myocardian infarction (MI), 862
Myocardium at risk, 867
Myxedema coma, 951
NaHCO3, 842
Narrative reviews, 56
Natalizumab, for Crohns disease, 216
National Surgical Adjuvant Breast and Bowel Project (NSABP)
B-18 trial, 607
Nausea and vomiting, postoperative, 843844
NCCTG (N9831) trial, 606
Neck exploration, for ZollingerEllison syndrome, 125
Neck hematoma, 555
Necrotizing fasciitis
defined, 833
diagnosis of, 833
treatment for, 834
Necrotizing myositis, 834835
diagnosis of, 834
resection in, 834
treatment for, 834835

PMPH_INDEX.indd 972

Necrotizing soft tissue infections (NSTIs), 826830


defined, 833
diagnosis of, 828
incidence of, 826
management of, 829830
risk factors and causative organisms, 826828, 827t
symptoms of, 828t
type I, 826
type II, 826
NEECHAM confusion scale, 921
Neoadjuvant chemotherapy. See also Chemotherapy
for breast cancer, 592593, 607608
and breast reconstruction following mastectomy, 623
for soft tissue sarcomas, 809
for surgically resectable esophageal tumor outcomes, 7880
Neoesophagus, conduit for creation of, 99100
Neomycin
for hepatic encephalopathy, 360
for surgical site infection, 894
Neostigmine, for acute colonic pseudo-obstruction, 224225
Nerve injuries, 556
Nerve transfers, 820
Neurectomy and postoperative groin pain, 515
Neuroendocrine liver metastases (NLM), 336
Neurofibromatosis type 1 (NF1), 544
Neurogenic disorders, 929
Neurotmesis, 818
Neurotrophic factors localization and peripheral nerve injuries,
820821
Nissen fundoplication, for gastroesophageal reflux disease, 61
Nitazoxanide, for pseudomembranous colitis, 240
Nitrates
for non-ST-elevation MI, 904
for Raynauds syndrome, 731
for ST-elevation MI, 903
N. meningitidis, 500
N-methyl-d-aspartate, for disbetic foot, 763
Noncardiac surgery
cardiac complications of, guidelines for reducing, 859863,
860t
clinical predictors, 862t
energy requirements, 861t
pulmonary complications after, guidelines for reducing, 864
Nonhepatic intraabdominal surgery, for cirrhosis, 937. See also
Sugery
Non-Hodgkins lymphoma (NHL), indications for splenectomy
in, 482483
Noninvasive positive-pressure ventilation (NIPPV), 929
Nonleukemic chronic myeloid disorders, indications for
splenectomy in, 481482
Nonmyeloid leukemia, indications for splenectomy in, 482483
Nonoperative management (NOM), for splenic salvage, 490
Nonpolyposis syndromes, 261
Non-small cell lung cancer (NSCLC), 632. See also Cancer
Nontoxic goiter
diagnosis of, 562563
medical therapy for, 563
molecular markers for, 563
thyroidectomy for, 563
Normetanephrine, 545
Nosocomial bacteremia, 879. See also Bacteremia
NSABP (B-31) trial, 606

5/22/2012 6:24:25 PM

Index

NSABP B-18 trial, 608


NSAIDs, for peptic ulcer disease, 110
Nutritional management, after bariatric surgery, 138
Obesity
global, magnitude and impact of, 132
and male breast cancer, 617
Obstructing colon cancer, operative management of, 223
Obstructive defecation syndrome (ODS), 302
Obstructive jaundice, 410415
benign CBD strictures, 412413
from chronic pancreatitis, 412
CBD stones, 410412
noninvasive imaging modalities for, 411
endoscopic retrograde cholangiopancreatography for,
411412
gaining access to, following Rouxen-Y gastric bypass, 412
prediction of, 410411
pregnant women with, 411412
malignant CBD strictures, 413414
pancreatic cancer patients, biliary drainage prior to
surgical resection in, 413
per oral choledochoscopy for, 414
plastic vs. metal stents for, 413
Obturator hernias, repair of, 527
Occupational therapy (OT), for postoperative respiratory
failure, 930
Octreoscan. See Somatostatin receptor scintigraphy (SRS)
Octreotide, 155156
for carcinoid of small bowel, 189
for enterocutaneous fistula, 193
for liver metastases, 190
for short bowel syndrome, 198
for VIPoma, 468469
Ogilvies Syndrome, 223
Omeprazole
for UGI bleeding, 154, 155
for ZollingerEllison syndrome, 125
O-Methylation, 545
Open appendectomy (OA), for appendicitis, 279280
Open bypass surgery (OBS)
versus endovascular therapy (EVT), 700, 702
for end-stage renal disease, 702
outcomes of, grafts affecting, 702703
patency in, 701
Open colectomy, for diverticular disease of colon, 209210
Open reconstruction, for renal artery stenosis, 750
Open repair (OR)
eligibility for, 663
versus endovascular repair, 662
for inguinal hernias, 514
recurrence rate of, 531
for inguinal hernias, 518520
of obturator hernias, 527
preservation of collateral flow in, 664
transabdominal versus retroperitoneal approaches, 663664
Operative ablation, of liver tumors, 324
Operative bypass, for diabetic foot revascularization, 764, 768
Opioids, for disbetic foot, 763
Oral antibiotics, for surgical site infection, 894, 900. See also
Antibiotics
Osteoblastoma, 638

PMPH_INDEX.indd 973

973

Osteochondroma, 638
Osteoid osteoma, 638
Osteomyelitis of diabetic foot, diagnosis of, 763764
Osteosarcom, 638
Ovarian cancer, liver resection for, 325. See also Cancer
Oversedation, 918, 920. See also Sedation management,
in ICU
Oxaliplatingemcitabine, for biliary tract tumors, 406
Oxycodone, for diabetic peripheral neuropathy, 763
Paclitaxel
-based CRT, for esophageal cancer, 94
for breast cancer, 592
Pagets disease
diagnosis and evaluation of, 256
follow-up, 257
prognosis of, 256257
treatment for, 257
Pain
chronic, after inguinal hernia repair, 520
in chronic pancreatitis, etiology of, 450
Pamidronate, for hypercalcemia, 848
Pancreas divisum (PD), surgery for, 426
Pancreatectomy
for intraductal papillary mucinous neoplasm, 471472
with lymphadenectomy, for gastric cancers, 149
Pancreatic abscess, defined, 438, 439
Pancreatic adenocarcinoma, 458463. See also Carcinoma
adjuvant therapy for, 460461
borderline resectable, 459460
locally advanced, 459460
neoadjuvant therapy for, 461462
resectable, 459460
staging of, 458459
Pancreatic adenosquamous carcinoma (PASC), 470. See also
Carcinoma
and acinar cell carcinoma, differentiation between, 471
molecular markers of, 470471
and pancreatic ductal adenocarcinoma, differentiation
between, 470
Pancreatic cancer. See also Cancer
liver resection for, 324
with obstructive jaundice, biliary drainage prior to surgical
resection for, 413
Pancreatic ductal adenocarcinoma (PDA), 470. See also
Carcinoma
Pancreatic endocrine neoplasms, 467474
Pancreatic gastrinomas, 121, 123
Pancreatic necrosis, defined, 438
Pancreatic pseudocysts, 437445
acute, defined, 438439
formation
pancreatitis etiology and, 439
risk of, 439440
incidence of, 439
complicated, 441
imaging modalities for, 441442
nonoperative expectant management of, 440
outcomes, effect of surgical intervention delays on, 444
for pancreatic pseudocysts, 441
therapeutic intervention for, 442444
timing of intervention for, 441

5/22/2012 6:24:25 PM

974

Index

Pancreatic septoplasty, for Sphincter of Oddi dysfunction,


424425
Pancreatic tumors, 488
unusual, 467475
Pancreaticoduodenectomy
for adenocarcinoma of small bowel, 188
for chronic pancreatitis, 452
Pancreatitis
early/delayed feeding for, 432
enteral/parenteral feeding for, 431432
etiology of, 439
Pancreatoblastoma
age of occurence, 473
clinical presentation of, 473
Panendoscopy, for metabolic squamous cell carcinoma of
unknown primary, 815816
Pantoprazole, for ZollingerEllison syndrome, 125
Paracentesis-induced circulatory dysfunction (PICD), 353
Parathyroid. See also Thyroid
adenomas, 571
preoperative management of, 951
Parathyroidectomy, 951
Parenteral nutrition (PN)
-associated liver disease (PNALD), 197
for pancreatitis, 431432
for short bowel syndrome, 197
Partial splenectomy, for postsplenectomy sepsis, 503, 504
Patency
of above-knee femoropopliteal grafts, 703t
of below-knee femoropopliteal grafts, 703t
in endovascular therapy, 701
of intrapopliteal grafts, 6t
in open bypass surgery, 700
Patient safety, in surgical care, 1420
discharge, 17
immediate presurgical care, 16
patient selection, 16
postsurgical care, 17
surgical disease, diagnosis of, 16
surgical intervention, timing of, 16
surgical patient, initial assessment of, 15
surgical procedures, 16
Patient selection, 16
Pedicle TRAM flaps, 624625
Pelvic floor dysfunction (PFD), management of, 301
Penicillin, for lymphedema, 793
Peptic ulcer disease (PUD)
bleeding peptic ulcer. See Bleeding peptic ulcer
H. pylori positive, therapy for, 110
risk factors for, 109110
surgery, changes in, 109
perforated, management of, 112113
risk minimization in patients requiring regular
NSAIDs, 110
Per oral choledochoscopy, for malignant CBD strictures, 414
Percutaneous ablation, of liver tumors, 324
Percutaneous balloon angioplasty, for central venous stenosis,
743
Percutaneous biliary drainage procedures, for biliary tract
tumors, 405, 406
Percutaneous breast biopsies, 580. See also Biopsy(ies)

PMPH_INDEX.indd 974

Percutaneous cholecystostomy, for acute cholecystitis, in


high-risk surgical patients, 383
Percutaneous coronary intervention (PCI), 902903
Percutaneous drainage, for splenic abscess, 488
Percutaneous transhepatic cholangiography (PTC)
for benign biliary strictures, 396
for hilar cholangiocarcinoma, 413
Percutaneous transluminal balloon angioplasty (PTA), for
critical limb ischemia, 702
Percutaneous transluminal coronary angioplasty (PTCA), for
preoperative risk factors, 863
Performance bias, 10. See also Bias
Perineal repair, for rectovaginal fistulas, 307
Perineoproctectomy, for rectovaginal fistulas, 309
Perioperative cardiac monitoring
CVP monitoring, 869870
dynamic indices response to IV volume administration, 871873
pulse contour output devices for, 870871
Perioperative endocrine dysfunction
adrenal incidentaloma, 951953
diabetes mellitus, 949950
parathyroid, 951
thyroid, 950951
Perioperative fluid management
complications associated with colorectal surgery, 269
Perioperative oxygen administration
complications associated with colorectal surgery, 269
Peripheral arterial disease (PAD), 691692, 699
Fontaine and Rutherford classification, 699t
Peripheral arterial embolus, 722726
acute arterial ischemia from
fasciotomies with lower extremity revascularization for,
725726
thrombolysis for, 722724
chronic anticoagulation for, 724725
Peripheral nerve injury repair
indications and options for, 819820
levels of evidence, 822
methods of, 819
safe to mobilize and rehabilitate an extremity after, 821822
therapeutic adjuncts for, 820821
timing of, 818819
Perirectal abscesses. See also Abscess(es)
antibiotics for, 295
drainage of
development of a fistula-in-ano following, prediction/
prevention of, 295
operative strategies for, 295
Peritoneovenous shunts, for intractable ascites, 354
Perityphlitis, 275
PET-CT scanning, for hyperparathyroidism, 570
pH study, for esophageal pulsion diverticula, 49
Phenoxybenzamine, for pheochromocytoma, 952
Pheochromocytoma
diagnosis, 545546
genetic mutations, 544545
imaging technologies, 546547
laparoscopic adrenalectomy for, 539
perioperative management of, 952953
prognosis/malignant potential, 547
treatment for, 548

5/22/2012 6:24:25 PM

Index

Photodynamic therapy, for anal intraepithelial


neoplasia, 255
Physical activity, after bariatric surgery, 138
Physical therapy (PT)
for neurogenic thoracic outlet syndrome, 712713
for postoperative respiratory failure, 930
PiCCO, for perioperative cardiac monitoring,
870871
PINI scoring system, 855
Plasma-free metanephrines, 546
Plasmapheresis, for necrotizing myositis, 835
Plasmin formation, activators and inhibitors of, 722t
Platelet inhibitors, effect on graft patency, 737738
Platelet-derived growth factor receptor, for soft
tissue sarcomas, 810
Platelets administration, for abnormal surgical and
postoperative bleeding, 913
Plugs, for rectovaginal fistulas, 309
PMS2 gene mutations and inherited colorectal cancer
syndromes, 261
Pneumococcal pneumonia, 883
Pneumococus infections
vaccinations for post-SAE patients against, 491
Pneumothorax
thoracostomy tube placement for, 651
treatment for, 650
Polycythemia vera (PV), 482
Polyethylene glycol (PEG), for constipation, 301
Polyglycolic acid (PGA), 820
Polyglycolic acid mesh (PGA-c), 820
Polylactide-caprolactone (PLCL), 820
Polymerase chain reaction (PCR) test, for C. difficile-associated
diarrhea, 239, 243
Polytetrafluoroethylene (PTFE), 820
-covered stents, 361
for incisional hernia, 530, 531
Portal hypertension (PHTN), 345351, 937
anatomy of, 347
classification of, 346, 347
defined, 345
etiology of, 347
incidence of, 345
mortality risk associated with, 345
pathophysiology of, 345346
surgical shunting for, 346347
transjugular intrahepatic portosystemic shunting for,
346347
treatment algorithm for, 346
Portal-systemic encephalopathy (PSE) index, 358
Porto-caval shunt (PCS) surgery, for portal hypertension with
bleeding esophageal varices, 347
Portomesenteric system, anatomy of, 345, 347
Positrom emission tomography (PET)
for esophageal cancer, 91
for hyperparathyroidism, 570
for liver malignancies, 335, 336
for malignant melanoma, 801
for metabolic squamous cell carcinoma of unknown
primary, 815, 816
for primary chest wall tumors, 637
for rectal cancer, 246

PMPH_INDEX.indd 975

975

Positron emission tomography/computed tomography (PET/CT),


for esophageal tumors, 74
Postanesthesia care unit (PACU), 843, 844
Postmastectomy radiation therapy (PMRT), 624
for locally advanced breast cancer, 592
Postoperative chemotherapy, for soft tissue sarcomas, 809.
See also Chemotherapy
Postoperative groin pain and neurectomy, 515
Postoperative hepatic failure, 936940
diagnosis of, 937938
fluid management for, 938939
fractionated plasma separation, adsorption and dialysis
system for, 939
isolated elevated liver function tests for, 936
molecular adsorbent recirculating system for,
939
predictor of, 937
renal replacement therapy for, 939
risk factors of, 936937
single-pass albumin dialysis for, 939
single-pass albumin extended dialysis for, 939
Postoperative ileus (POI), duration of, 165
agents/techniques to improve, 165166
chewing gum and, 166
selective opiate receptor inhibitors and, 166167
Postoperative nausea and vomiting (PONV), risk factors of,
843844, 853
Postoperative respiratory failure, 926931
causes of, 927
after surgery, risk for, 926927
recommendations, 927
respiratory support for
need for, minimizing, 930
types of, 929930
treatment for, 927929
types of, 927
Postsplenectomy sepsis (PSS), 498505
antibiotics for, 501502
clinical presentations and outcomes of, 500501
conservative splenic surgery for, 503504
onset, interval between splenectomy and, 499
pathogens causing, 500
risk factors associated with, 498500
splenic autotransplantation for, 503504
splenic embolization for, 503504
splenosis for, 503504
vaccinations for, 502
Postsurgical care, 17
Postthrombotic syndrome, 777778
Potassium elimination and hyperkalemia, 842
Pouch anal anastomosis, for rectovaginal fistulas, 307
Prazosin, for pheochromocytoma, 952
Pregabalin, for diabetic foot, 763
Preoperative bowel preparation, 267270
mechanical
prior to colorectal surgery, 268269
risk factors of, 268
surgical site infection/anastomotic complications,
incidence of, 267268
perioperative fluid management, 269
perioperative oxygen administration, 270

5/22/2012 6:24:26 PM

976

Index

Prerenal azotemia, magnesium abnormalities and, 848


Pressure support ventilation (PSV), 930
Primary hyperaldosteronism. See Conns syndrome
Primary hyperparathyroidism (pHPT), 568, 569, 951
Primary sclerosing cholangitis (PSC), 395
biliary strictures management from, 412413
Probe to the bone (PTB) sign, 763
Probiotics, for pancreatitis, 432433
Procedure for Prolapse and Hemorrhoids (PPH), 288
Prolene Hernia System (PHS), 527
Prophylactic antibiotics. See also Antibiotics
for pancreatitis, 433
for surgical site infection, 893894, 900
Prophylactic cholecystectomy. See also Cholecystectomy
in diabetes mellitus, 376
in transplant patients, 376
Propofol infusion syndrome (PRIS), 920
Propofol, 920
Propylthiouracil (PTU), for Graves disease, 559
for hyperthyroidism, 950
for tachycardia, 950
for thyroid storm, 951t
Prospective cohort studies, 4
Prostaglandins, for Raynauds syndrome, 731
Prothrombin complex concentrates (PCCs), 912
for abnormal surgical and postoperative bleeding, 912
Prothrombin time (PT), 909, 910
Proton pump inhibitors (PPIs)
plus bismuth, metronidazole, and tetracycline, 110
plus clarithromycin and amoxicillin/metronidazole, 110
for gastric acid suppression, 468469
for gastroesophageal reflux disease, 60
for NSAID-induced gastrointestinal complications, 110
for peptic ulcer disease, 110
and pseudomembranous colitis, 240
for stress ulcer prophylaxis, 154, 155
for ZollingerEllison syndrome, 125
Proximal splenic embolization (PSE), 491493
Pseudoaneurysms (PSAs)
traumatic splenic, matural history of, 493
Pseudoaneurysms, AVF/AVG, treatment for, 743744
Pseudomembranous colitis, 239241
diagnosis of, 239
pathogenesis of, 239
risk factors of, 240
surgical intervention for, 241
treatment for
asymptomatic patients, 240
mild to moderate disease, 240
severe disease, 240
virulent strain, emergence of, 240241
Publication bias, 67. See also Bias
Pudendal nerve terminal motor latency (PNTML), for fecal
incontinence, 300
Pulmonary artery catheter (PAC), 869, 871
for perioperative cardiac monitoring, 869, 871, 872t
Pulmonary complications after noncardiac surgery, guidelines
for reducing, 864
Pulmonary embolism (PE), 785789, 926
diagnosis of, 786
prevention of, 786788

PMPH_INDEX.indd 976

risk factors of, 785786


symptomatic, 777
treatment for, 788, 928929
Pulmonary hypertension, 929
Pulse pressure variation (PPV), 872, 873
Pyloric drainage procedure and outcomes of esophagectomy,
7677
Pylorus preserving pancreaticoduodectomy (PPPD), for chronic
pancreatitis, 452
Pylorus, feeding, 856
Pyogenic abscess. See also Abscess(es)
clinical features of, 330
diagnosis of, 330331
epidemiologic characteristics of, 329
incidence of, 329
pathiphysiology of, 330
treatment for, 331
Radial scar (RS), 581
Radial sclerosing lesion. See Radial scar (RS)
Radiation enteritis, 228230
causes of, 228
dose of radiation, 228229
due to IMRT, 229
mode of action, 228
surgery for, 229
Radiation exposure and male breast cancer, 618
Radiation therapy (RT)
adjuvant/neoadjuvant, in esophageal cancer, 91, 9293
for breast cancer, 590
for ductal carcinoma in situ, 613614
for male breast cancer, 619
for recurrent sarcomas, 810
for squamous cell carcinoma, 254
for surgically resectable esophageal tumor outcomes, 7780
Radical surgery, for early gastric cancer, 147
Radiofrequency ablation (RFA)
for liver malignancies, 336, 337
for liver tumors, 323324
for venous insufficiency, 770
for ZollingerEllison syndrome, 128
Radionuclide scanning
for acute cholecystitis, 381
for lower gastrointestinal bleeding, 314315
Radiotherapy
bowel injury due to, 229
for soft tissue sarcomas, 809
RAI therapy
for Graves disease, 560
for toxic MNGs and STNs, 560
Randomized Evaluation of Normal versus Augmented Level
Replacement Therapy (RENAL) trial, 947
Ransperineal repair, for defecation disorder, 302
Raynauds syndrome, 730732
calcium channel blockers for, 730731
nitrates for, 731
prostaglandins for, 731
sildenafi l for, 732
Recombinant activated factor VII (rFVIIa), for abnormal surgical
and postoperative bleeding, 912
Recombinant human-activated protein C (Xigris), for sepsis, 879

5/22/2012 6:24:26 PM

Index

Rectal cancer, 245248. See also Cancer


diverting ostomy with LAR for, 247
local excision for, 246
minimally invasive surgery for, 246247
preoperative staging method for, 245
total mesorectal excision for, 247248
Rectal elimination and hyperkalemia, 842
Rectocele with defecation disorder, surgery for, 302
Rectovaginal fistula, 305310
abdominal repair of, indications for, 307
clinical presentation of, 305306
diagnosis of, 306307
etiology of, 305t
perineal/transanal repair of, indications for, 307308
risks associated with, 305
secondary to Crohns disease, management of, 308309
surgery for, 309
Recurrent inguinal hernia, 518521. See also Hernia
chronic pain following repair of, incidence of, 520
laparoscopic repair of, 518, 520
nonoperative management of, 519
open of, 518520
pattern of, 519
primary repair of, 518
Recurrent laryngeal nerve (RLN) dysfunction, 570
Recurrent sarcomas, treatment for, 809810
Regional limb perfusion, for malignant
melanoma, 802803
Renal artery stenosis, 749
angioplasty
versus open reconstruction for, 750
versus medical treatment for, 750751
plus stenting for, 751
bilateral, 751f
diagnosis of, 750
natural history of, 749750
Renal cancer, liver resection for, 325. See also Cancer
Renal replacement therapy (RRT)
for acute renal dysfunction, 943, 947
selection factors, 944
continuous, 943944
Renal replacement therapy (RRT), for postoperative hepatic
failure, 939
Renovascular hypertension, pathophysiology of, 749
Retroperitoneal (RP) versus transabdominal (TP) open repair,
663664
Retroperitoneal tumor, 488
Revised Cardiac Risk Index, 862
Revisional bariatric surgery (RBS), 137138
Rhomboid rotational flaps, for anal strictures, 295
Richmond Agitation Sedation Scale (RASS), 918
Rifampicin, for pseudomembranous colitis, 240
Rifaximin
for hepatic encephalopathy, 360
for pseudomembranous colitis, 240
RIFLE (Risk, injury, failure, loss, end-stage renal disease)
criteria, 942, 943, 947
Ringer lactate, 843
Rituximab
for Graves ophthalmopathy, 560
for thrombotic thrombocytopenic purpura, 483

PMPH_INDEX.indd 977

977

Rouxen-Y gastric bypass (RYGBP), 134, 843844


gaining access to CBD following, 412
Roux-Y hepatico-jejunostomy, for benign biliary strictures, 394, 397
Rubber band ligation (RBL), for hemorrhoids, 287
S-100 markers, 800
Sacral nerve stimulator (SNS), for fecal incontinence, 300
Safety models, to reduce errors in surgical care, 1718
Salbutamol, for hyperkalemia, 842
Saline versus albumin fluid evaluation study (SAFE), 844
Sarcoma of small bowel
adjuvant therapy for, 189
incidence, histology and location of, 187
prognostic factors for survival for, 190
recurrence, treatment for, 190
treatment for, 188
SAVE trial, 903
Scalene muscle blocks, 710
Schwannoma, 638
Secondary hyperparathyroidism, 568
management of, 569
Secretin stimulation test, for ZollingerEllison syndrome, 122
Sedation Agitation Scale (SAS), 921
Sedation management, in ICU
oversedation, 918, 920
target-controlled infusion, 920921
Segmental colectomy, for Lynch syndrome, 263
Seizure, electrolyte abnormalities associated with, 848
Selection bias, 10. See also Bias
Selective arterial secret injection (SASI) test, for
ZollingerEllison syndrome, 125
Selective opiate receptor inhibitors and POI duration, 166167
Self-expanding metallic stents (SEMS), for acute large bowel
obstruction, 223
SengstakenBlakemore tube, balloon tamponade of bleeding
esophageal varices using, 349
Sentinel lymph node (SLN) biopsy, 608. See also Biopsy
for axillary lymph node dissection, 591
biopsy. See also Biopsy(ies)
accuracy of, 598599
for ductal carcinoma in situ, 600601, 614
identification of technique for, 597598
prior to preoperative chemotherapy, 599600
for early-stage breast cancer, 588589
for male breast cancer, 619
for malignant melanoma, 801802
complete regional lymph node dissections and, 802803
metastasis, surgical management of, 599
Sentinel lymphadenectomy, for malignant melanoma, 802
Sepsis
albumin infusion for, 844
factors associated with mortality, 879
incidence of, 876877
-like syndrome, 937
postsplenectomy. See Postsplenectomy sepsis (PSS)
Septicemia. See Sepsis
Sequential compression devices (SCDs), 786787
Serial transverse enteroplasty (STEP), for short bowel syndrome,
198199
Serotonin (5-HT3) antagonists, for postoperative nausea and
vomiting, 843

5/22/2012 6:24:26 PM

978

Index

Serous cystic neoplasm (SCN)


malignant transformation of, 472
preoperative differentiation of, 472
Serum albumin
effect on calcium measurements, 848849, 853
for predicting mortality in ECF, 192
Serum calcium and parathyroid hormone test, for
ZollingerEllison syndrome, 122
Serum creatinine (sCr), inadequacies of, acute renal dysfunction
and, 943
Setons, for anal fistulas, 295
cutting, 295
draining, 295
Short bowel syndrome (SBS), 196200
autologous intestinal reconstruction surgery for, 198199
enteral nutrition for, 197
growth hormone for, 197198
hormonal agents, future directions of, 198
intestinal failure risk after massive resection, 196
intestinal rehabilitation programs for, 199200
intestinal transplantation for, 199
octreotide, role of, 198
parenteral nutrition for, 197
risk of, after surgery for radiation enteritis, 229
teduglutide for, 198
Sickle cell disease (SCD), 483
patients with gallstones, 377
Sigmoid colon resection
after uncomplicated diverticulitis, 208209
for younger patients after single attack of acute diverticulitis,
209
Sigmoid volvulus, operative management of, 223224
Sigmoidoscopy
for C. difficile-associated diarrhea, 237
for familial adenomatous polyposis, 263264
Sildenafi l, for Raynauds syndrome, 732
Silent gallstones. See Gallstones
Single photon emission computed tomography (SPECT)
for postoperative hepatic failure, 937
for ZollingerEllison syndrome, 124
Single-pass albumin dialysis
extended dialysis, 939
for postoperative hepatic failure, 939
Situational awareness (SA), 1819
Small bowel anastomosis, 169170
Small bowel diverticular disease, 190
Small bowel obstruction (SBO)
adhesional, 167
incomplete, CT for, 169
partial, ischemia prediction in patients with, 170
water-soluble contrast medium for, 168169
Small bowel surgery, 165. See also Surgery
Small bowel tumors
diagnosis of, 187
incidence, histology and location of, 187
prognostic factors for survival for, 189
recurrence, treatment for, 190
risk factors for, 188
role of adjuvant therapy in, 188
treatment of, 188
types of, 186

PMPH_INDEX.indd 978

Small-nerve-fiber neuropathy, for diabetic foot, 763


Smoking
and outcome of breast reconstructive surgery, 622
and postoperative respiratory failure, 927
Sodium-polystyrene sulfonate, for hyperkalemia, 853
Soft tissue sarcomas (STSs), 638, 807811
clinical presentation for, 808
epidemiology of, 807
etiology of, 807808
chemicals, 807
environmental factors, 807
genetic factors, 807
host factors, 807
pathology of, 808
surveillance of, 809
treatment for, 808810
Solid pseudopapillary tumor (SPT), genetic markers for, 472473
Solitary neck mass, 814816
age factors associated with, 814, 815
cystic, 815
FNA biopsy for, 815
primary tumor site, prediction of, 815
risk factors of, 815
Solitary plasmacystoma, 638
Solitary toxic nodule (STN), treatment for, 560
Somatostatin analogs
for enterocutaneous fistula, 192193
for VIPoma, 468469
Somatostatin receptor scintigraphy (SRS), for ZollingerEllison
syndrome, 124125
Sphincter of Oddi dysfunction (SOD)
biliary sphincteroplasty for, 424425
classification of, 424t
endoscopy for, 425426
pancreatic septoplasty for, 424425
surgery for, 425426
Sphincter-sparing techniques, for fistula-in-ano, 296
Spigelian hernias, repair of, 527
Spironolactone, for ascites, 353
Splenectomy
hematologic indications for, 481485
in chronic myeloid leukemia, 481
for Feltys syndrome, benefits of, 484
in hemolytic anemia, 483484
in immune thrombocytopenia, 483
laparoscopic, in hematologic disease, 484485
in non-Hodgkins lymphoma, 482483
in nonleukemic chronic myeloid disorders, 481482
in nonmyeloid leukemia, 482483
in thrombotic thrombocytopenic purpura, 483
with lymphadenectomy, for gastric cancers, 149
and onset of sepsis, interval between, 499
Splenic abscess, 488489. See also Abscess(es)
Splenic angiographic embolization (SAE)
proximal, 491493
timing of, 491
vaccination against Pneumococus and Hemophilus influenza
infections, 491
Splenic autotransplantation, for postsplenectomy sepsis,
503504
Splenic embolization, for postsplenectomy sepsis, 503504

5/22/2012 6:24:26 PM

Index

Splenic salvage
nonoperative management for, 490494
repeated imaging for, 492
Splenocaval shunting, for portal hypertension, 348
Splenomegaly, 489
Spleno-mesenteric system
anatomy of, 347
etiology of, 347
Splenorenal shunting, for portal hypertension, 348
Splenosis, for postsplenectomy sepsis, 503504
Spontaneous breathing trial (SBT), 930
Sporadic gastrinomas patients, surgical management of
ZES in, 128
Squamous cell carcinoma (SCC). See also Carcinoma
of anal region
diagnosis and evaluation of, 256
follow-up, 256
prognosis of, 256
treatment for, 256
diagnosis and evaluation of, 253254
of esophagus
treatment of, 92
follow-up, 254
prognosis of, 254
treatment for, 254255
Staging laparoscopy (SL), for pancreatic adenocarcinoma, 459
Stapled hemorrhoidopexy (SH), for hemorrhoids, 288289
Stapled transanal rectal resection (STARR) procedure, 302
Statin therapy
for abdominal aortic aneurysm management, 661
for carotid occlusive disease, 686
for chronic mesenteric ischemia, 757
Statistical precision, 5
Stenosis, renal artery. See Renal artery stenosis
Stenting plus angioplasty, for renal artery stenosis, 751
Stereotactic breast biopsies, 580. See also Biopsy(ies)
Steroids, for adrenal incidentaloma, 952
Stewart-Treves syndrome, 807
Stomach cancer, liver resection for, 324325. See also Cancer
Stomas, for large bowel obstruction, 223
Streptococcal toxic shock syndrome, 834
Streptococcus pneumoniae, 500
Strictureplasty, for Crohns disease, 176177
Strictures, anal. See Anal strictures
Stroke risk
for asymptomatic carotid stenosis, 685
for symptomatic carotid stenosis, 684685
Stroke volume variation (SVV), 872, 873
Subtotal colectomy
for acute colonic pseudo-obstruction, 224
for large bowel obstruction, 223
Subtotal parathyroidectomy, for ZollingerEllison syndrome, 125
Succinate dehydrogenase subunit B gene (SDHB), 544
Sulfasalazine, for Crohns disease, 216
Sunlight, impact on malignant melanoma, 799800
Supervision, 19
Suppurative thyroiditis, acute, 561
Surface oxygen tension, for small bowel anastomosis, 169, 170
Surgery
for achalasiapasm, 38
for acute cholecystitis, 382

PMPH_INDEX.indd 979

979

for acute colonic pseudo-obstruction, 225


for acute kidney injury, risk factors associated with, 943
for acute mesenteric ischemia, 757
bariatric. See Bariatric surgery
for benign biliary strictures, 396397
for benign breast diseases, 581
breast, 893894
for chronic pancreatitis, 451453
for cirrhosis, 937
colorectal. See Colorectal surgery
with combined therapy, for adenocarcinoma of
esophagus, 92
for diff use esophageal spasm, 38
for ductal carcinoma in situ, 612613
for hernia, 893894
for hyperthyroidism, 560561
for hypothyroidism, 950951
for infected pancreatic collections, 434
for large bowel obstruction, 223
for liver abscesses, 323327, 331
for lobular carcinoma in situ, 614615
for lobular neoplasia, 581582
for lower gastrointestinal bleeding, 316
for male breast cancer, 618619
for necrotizing soft tissue infections, 829
for pancreas divisum, 426
for pheochromocytoma, 952
for pseudomembranous colitis, 241
postoperative respiratory failure after, risk for, 926927
procedures, for patient safety, 16
for rectocele with defecation disorder, 302
for rectovaginal fistula, 309
SLN metastasis, management of, 599
small bowel, 165
for soft tissue sarcomas, 808809
for Sphincter of Oddi dysfunction, 425426
for surgical site infection, 893894
Surgical Care Improvement Project (SCIP), 892, 893t
Surgical disease, diagnosis of, 16
Surgical Infection Prevention (SIP) Project, 892
Surgical intervention, timing of, 16
Surgical nutrition
intervention of
continuous tube feeds vs intermittent tube feeds,
856857
enteral nutrition, preoperative delivery of, 855856
pylorus, feeding, 856
total parenteral nutrition, delivery of, 856
risk/diagnosis of, 854855
Surgical patient, initial assessment of, 15
Surgical patient, preoperative risk of, 859864, 867868
cardiac complications of noncardiac surgery, guidelines for
reducing, 859863, 860t
clinical predictors, 862t
energy requirements, 861t
factors associated with, 859, 860t
management of, 863864
physical examination, components of, 859t
pulmonary complications after noncardiac surgery,
guidelines for reducing, 864
treatment for, 863t, 868

5/22/2012 6:24:26 PM

980

Index

Surgical shunting (SS)


evaluation for, 347
for portal hypertension, 346347
Surgical site infection (SSI)
causative pathogens in, 891892, 899
colorectal surgery, mechanical bowel preparation role in,
894895
defined, 891
during mechanical bowel preparation, 267268
methicillin-resistant Staphylococcus aureus, 895, 900
national efforts in reducing, 892
obese patients, dose modifications for, 893, 900
prevention, intravenous antibiotics for
antibiotic prophylaxis, 892893
duration of, 892, 899
oral antibiotics, 894, 900
prophylactic antibiotics, 893894, 900
timing of, 892, 899
risk factors of, 891, 899
vancomycin-resistant Enterococcus, 895, 900
Surveillance, Epidemiology, and End-Results (SEER) registry, 608
Syndrome of inappropriate antidiuretic hormone (SIADH)
magnesium abnormalities and, 849, 853
Synthetic grafts, peripheral nerve injury with, 820
Systematic reviews, 6
Systemic inflammatory response syndrome (SIRS), 876, 882
Systolic blood pressure variation (SPV), 872, 873
T1-weighted MRI, for hepatic encephalopathy, 358. See also
Magnetic resonance imaging
Tachycardia, 950
Talent, 663
Tamoxifen, 606
for breast cancer, 592, 593
for male breast cancer, 619
Target-controlled infusion (TCI), of sedation, 920921. See also
Sedation management, in ICU
Teamwork improvement, and surgical care error reduction, 1920
Technetium-99m
-diethylenetriaminepentaacetic acid galactosyl human
serum albumin, for postoperative hepatic failure, 937
-mebrofenin hepatobiliary scintigraphy, for postoperative
hepatic failure, 937
pertechnetate scan, for hyperthyroidism, 559
sulfur colloid injection, for lower gastrointestinal bleeding, 315
TRBCs, or lower gastrointestinal bleeding, 315
Teduglutide, for short bowel syndrome, 198
Teicoplanin, for pseudomembranous colitis, 240
Tension-free repair, 514
Tertiary hyperparathyroidism, 568
management of, 569
Tetraiodothyronine replacement therapy, for
hypothyroidism, 950
-Thalassemia major, 484
Thalassemias, 483484
Thiazide diuretics, for hypercalcemia, 848
Thiazolidinediones and diabetes, 950t
Thienopyridines, for ST-elevation MI, 903
Thoracic outlet syndromes, 707717
arterial, 708709

PMPH_INDEX.indd 980

diagnosis of, 712


treatment outcomes for, 715, 716
etiology of, 707
neurogenic, 707708
diagnosis of, 709710
brachial plexus, imaging assessment of, 711
medial antebrachial cutaneous nerve conduction
studies, 711
response to treatment, clinical predictors of, 710711
scalene muscle blocks, 710
treatment for
botulinum toxin chemodenervation, 713
functional outcomes and QOL following surgery,
715716
operations for recurrent TOS, 714715
physical therapy, 712713
primary operations for, 713714
venous, 708
diagnosis of, 712
treatment of
Thoracostomy tube
management of, 651652
removal of, 652653
Thorax, musculocutaneous flaps in, 639
Three dimensionalconformal radiation therapy (3D-CRT)
for large bowel obstruction, 229
for short bowel syndrome, 229
Thrombectomy, after failure of AVFs and AVGs, 743
Thrombin injection, for arterial pseudoaneurysms, 679
Thromboangiitis obliterans (TAO), 732733
autologous stem cell therapy for, 732
iloprost for, 732733
Thromboelastography (TEG), 843, 845
Thrombolysis
for acute arterial ischemia from embolus, 722724
for acute myocardial infarction, 902
catheter-directed, for late graft occlusion, 736737
for venous thoracic outlet syndrome, 712
Thromboprophylaxis, for deep venous thrombosis, 778
Thrombosis of false lumen, as predictor of aortic mortalities, 672
Thrombotic thrombocytopenic purpura (TTP), indications for
splenectomy in, 483
Thrombotic/thromboembolic conditions, ischemic colitis
associated with, 233
Thyroid, 950951. See also Hyperthyroidism
nodules, 552
clinical evaluation for, 552553
FNA thyroid cytology and, 553
hypoparathyroidism, 555556
identification of, 552
neck hematoma, 555
nerve injuries, 556
nonoperative follow-up for, 553555
operative approaches and operative management, 555
operative morbidity of thyroidectomy, 555, 556557
operative management of, 555
thyroid ultrasounds role to evaluate, 553
parathyroid, 951
preoperative management of, 950
storm, preoperative management of, 950, 951t

5/22/2012 6:24:26 PM

Index

Thyroid stimulating hormone (TSH), 950


Thyroidectomy
for Graves disease, 559560
for Graves ophthalmopathy, 559
for nontoxic goiter, 562563
operative morbidity of, 555, 556557
for thyroiditis, 561562
for toxic MNGs and STNs, 560561
Thyroiditis
acute suppurative, 561
chronic, 561
diagnostic test for, 561
subacute, 561
surgery for, 561562
Thyroid-stimulating hormone (TSH)
for hyperthyroidism, 559
suppression, 559, 561563
Tidal volume, 929930
Tigation, transpleural, of fistula with gastrostomy tube, 98
TIPS, hepatic encephalopathy following, 361
Tissue transfer procedures, for rectovaginal fistulas, 309
Tonsillectomy, for metabolic squamous cell carcinoma of
unknown primary, 816
Total abdominal colectomy
for constipation, 301
for familial adenomatous polyposis, 263
for Lynch syndrome, 263
Total functional liver volume (TFLV), 938
Total mesorectal excision (TME), for rectal cancer, 247248
Total pancreatectomy with islet autotransplantation (TPIAT), for
chronic pancreatitis, 453
Total parenteral nutrition (TPN)
delivery of, 856
for enterocutaneous fistula, 192
Total proctocolectomy, for familial adenomatous polyposis, 263
Total thyroidectomy (TT)
for Graves disease, 560
for Graves ophthalmopathy, 560561
Totally extraperitoneal (TEP) approach, 514
Toupet fundoplication, for gastroesophageal reflux disease, 61
Tracheoesophageal fistula. See Esophageal atresia and
tracheoesophageal fistula (EA/TEF)
Tracheostomy, 642646
conversion after emergent cricothyroidotomy, 645
impact on swallowing and aspiration, 645
open vs. percutaneous, 643644
routine chest x-ray after, 644
patient populations benefit from, 643
percutaneous, bronchoscopic guidance for, 644
timing of, 642643
Tramadol, for disbetic foot, 763
Tranexamic acid (TXA), for abnormal surgical and postoperative
bleeding, 912913
Transabdominal (TP) versus retroperitoneal (RP) open repair,
663664
Transabdominal preperitoneal (TAPP) approach, 514
Transabdominal repair, of lumbar hernias, 527
Transanal excision (TAE), for rectal cancer, 246
Transanal flap repair, for rectovaginal fistulas, 307, 308309
Transanal repair, for defecation disorder, 302

PMPH_INDEX.indd 981

981

Transarterial chemo-embolization (TACE), for ZollingerEllison


syndrome, 128
Transcatheter embolization, for lower gastrointestinal bleeding,
315316
Transdermal nitroglycerin (NTG), for ST-elevation MI, 903
Transesophageal echocardiography (TEE), for aortic dissection,
670
Transfusion-related acute lung injury (TRALI), 928
management of, 849
Transhiatal esophagectomy, for esophageal tumors, 7576
Transient hypocalcemia, 571
Transjugular intrahepatic portosystemic shunt (TIPS)
role in intractable ascites, 354
for portal hypertension, 346347
Transplant patients, prophylactic cholecystectomy in, 376
Transplantation, for hepatic encephalopathy, 361362
Transpulmonary thermodilution (TPTD), for perioperative
cardiac monitoring, 870
Transthoracic echocardiography (TTE), for aortic dissection, 670
Transthoracic esophagectomy, for esophageal tumors, 7576
Transvaginal repair, for defecation disorder, 302
Transverse rectus abdominis myocutaneous (TRAM) flap
reconstructions, 622
free microvascular TRAM flap versus pedicle TRAM flap,
624625
Trastuzumab, 606, 607
Trauma
acute renal dysfunction and, 942943
with liver failure, fluid management for, 938939
patients, risk of postsplenectomy sepsis in, 499
Traumatic brain injury (TBI), hypertonic saline solutions
for, 847
Tricyclic antidepressants, for diabetic peripheral
neuropathy, 763
Tris-hydroxymethyl aminomethane (THAM), role during
resuscitation, 842843
T-tube drainage after choledochotomy, insertion of, for common
bile duct stones, 388
Tumor necrosis factor alpha (TNF-), 845
for Crohns disease, 216
for malignant melanoma, 803
Tumor(s)
of anal canal, 253256
of anal margin, 256257
of anal region, 253257
Turcot syndrome, 260
24-hour esophageal pH monitoring, for gastroesophageal reflux
disease, 60
Type 1 endoleak, 663
Type 2 diabetes mellitus (T2DM), after bariatric surgery, 138
Type 2 endoleaks, 662
Tyrosine-protein kinase Kit, for soft tissue sarcomas, 810
UK National Health Service
evidence-based medicine, hierarchies of, 2
Ulcerative colitis (UC), 215219
differentiating CD from, 177
laparoscopic surgery for, 218219
medical management for, 218
symptoms of, 215

5/22/2012 6:24:26 PM

982

Index

Ultrasound (US)
for acute cholecystitis, 380
anal, for rectovaginal fistulas, 306, 307
for biliary tract tumors, 404
Doppler
for blunt splenic injury, 492
for deep venous thrombosis, 776
for small bowel anastomosis, 169
Duplex
for arterial pseudoaneurysms, 678
for arteriovenous fistulas, 678
for carotid stenosis, 685
for diabetic foot, 763
for renal artery stenosis, 750
elastography, for nontoxic goiter, 563
endoanal, for fecal incontinence, 300
endorectal, for rectal cancer, 245246
endoscopic. See Endoscopic ultrasound
-guided aspiration, for liver abscesses, 330
-guided breast biopsies, 580. See also Biopsy(ies)
-guided core needle biopsy, for male breast cancer, 618
for nontoxic goiter, 563
for thyroiditis, 561
transabdominal, 123
for ZollingerEllison syndrome, 123
Umbilical hernia, 525528. See also Hernia
in adults and children, natural history of, 526
repair (UHR), 366367
laparoscopic, 526527
using mesh, 526
preoperative antibiotics for, 527
Unfractionated heparin (UFH). See also Heparin
for deep venous thrombosis, 778
for non-ST-elevation MI, 904
for pulmonary embolism, 786788
for ST-elevation MI, 903
Union for International Cancer Control (UICC) lung cancer
staging model, 632
Upper endoscopy, for small bowel tumors, 187
Upper gastrointestinal bleeding, 154
rebleeding, surgical techniques associated with lowest rate
of, 111
role of endoscopy in, 156
role of interventional radiology in, 156157
role of medical therapy
in prevention of, 154155
in treatment of, 155
Upper gastrointestinal endoscopy, for gastric adenocarcinoma, 146
Upper gastrointestinal study with a small bowel follow through
(UGISBFT), 187
Uremia, 909
Urinary neutrophil gelatinase-associated lipocalin (uNGAL), 943
US Preventive Services Task Force
evidence-based medicine, hierarchies of, 2
US-PEA, for ultinodular goiter, 560
Vaccinations, for postsplenectomy sepsis, 502
Vacuumassisted wound closure devices, 834
Vaginal deliveries and rectovaginal fistulas, 305
Vagotomy, for peptic ulcer disease, 111, 112
VALIANT trial, 904

PMPH_INDEX.indd 982

Valsartan, for ST-elevation MI, 904


Valve reconstruction, for venous insufficiency, 770771
Vancomycin
for necrotizing soft tissue infections, 829
for pseudomembranous colitis, 240, 244
Vancomycin-resistant Enterococcus infection, in surgical site,
895, 900
Varicose veins, 769771
Vascular closure devices (VCDs), for arterial access
catheterization site management, 679680
Vascular endothelial growth factor receptor, for soft tissue
sarcomas, 810
Vascular endothelial growth factor receptor-3 (VEGFR-3), 792
Vascular syndromes, 730733
Raynauds syndrome, 730732
thromboangiitis obliterans, 732733
Vasoactive intestinal polypeptide-oma (VIPoma), 468469
anatomic localization of, 468t
diagnosis of, 468t
differential, 469t
symptoms of, 468t
Vasopressin infusion, intra-arterial, for LGIB, 315316
Vasopressin-2 receptor antagonists, magnesium abnormalities
and, 849
Vasopressors, for necrotizing soft tissue infections, 829
Vein grafts, peripheral nerve injury with, 820
Venous insufficiency, 769771
compression for, 770
diagnosis of, 770
invasive procedures for, 770
risk factors of, 769770
valve reconstruction for, 770771
Venous thromboembolism (VTE), 911, 926, 927
treatment for, 910
Ventilator-associated pneumonia (VAP), 918
treatment for, 927928
Ventral hernia, 530532. See also Hernia
Verrucous carcinoma. See also Carcinoma
diagnosis and evaluation of, 257
follow-up, 257
prognosis of, 257
treatment for, 257
Vertical banded gastroplasty (VBG), 136
Veterans Affairs cooperative trial, 514
Veterans Affairs trial, 515
Veterans Affairs/National Institutes of Health Acute Renal
Failure Trial Network, 947
Video-assisted retroperitoneal debridement (VARD), for infected
pancreatic collections, 433
Video-assisted thoracoscopy (VATS), for hemothorax, 653
Videofluorographic contrast swallow study, for Zenkers
diverticulum, 47
Visual analogue scale (VAS), 843
Vitamin D supplementation, for hypocalcemia, 951
Vitamin-K antagonists (VKA), for deep venous
thrombosis, 778
Von Willebrand disease (VWD)
type I, 909
type II, 909
type III, 909
von-Hippel Lindau (VHL) disease, 544

5/22/2012 6:24:26 PM

Index

Warfarin
for abnormal surgical and postoperative bleeding, 910
and coagulopathy, 910
effect on graft patency, 737, 738
for prevention of embolus, 724
Watchful waiting
for incisional hernias, 531
repair, of inguinal hernia, 519
Water seal, use after thoracostomy tube placement, 651652
Water-soluble contrast medium, for SBO, 168169
Wedge resection
for adenocarcinoma of small bowel, 188
for carcinoid of small bowel, 188
for lymphoma of small bowel, 188
Weight loss, 854
surgery (WLS). See Bariatric surgery
Whipple pancreaticoduodenectomy, for ZollingerEllison
syndrome, 128
YV sliding flap, for anal strictures, 295
Zenith, 663
Zenkers diverticulum (ZD), 45
diagnosis of, 47
optimal intervention for, 4749
symptoms associated with, 47
treatment options for, 47
Zoledronic acid, for hypercalcemia, 848

983

ZollingerEllison syndrome (ZES)


associated with MEN type 1, 120
characteristics of
size and location, 120
clinical presentation of, 121122
diagnosis of
basal acid output, 122
fasting serum gastrin, 122
laboratory studies, 122
postsecretin challenge, 122
serum calcium and parathyroid hormone, 122
differential diagnosis of, 122
incidence of, 120
localization of, 122
computed tomography, 124
duodenotomy, 123124
magnetic resonance imaging, 124
selective arterial secret injection test, 125
somatostatin receptor scintigraphy, 124125
tumor localization modalities, sensitivities of, 123t
ultrasound, 123
metastatic potential and survival of, 121
pathophysiology of, 119120
patients distribution based on surgical results, 120f
treatment of
medical management, 125
surgical management, 125128

ERRNVPHGLFRVRUJ

PMPH_INDEX.indd 983

5/22/2012 6:24:26 PM

PMPH_INDEX.indd 984

5/22/2012 6:24:26 PM

Das könnte Ihnen auch gefallen